Goldman's Cecil medicine [25th ed] 9781455750177, 1455750174

To be the best doctor you can be, you need the best information. For more than 90 years, what is now calledGoldman-Cecil

471 14 624MB

English Pages 1 recurs en línia (2 vols) [2064] Year 2015,2016

Report DMCA / Copyright

DOWNLOAD PDF FILE

Recommend Papers

Goldman's Cecil medicine [25th ed]
 9781455750177, 1455750174

  • 0 0 0
  • Like this paper and download? You can publish your own PDF file online for free in a few minutes! Sign Up
File loading please wait...
Citation preview

QUICK REFERENCE (QR) VIDEO ACCESS The images below are QR codes. Each code corresponds to a video from the Goldman-Cecil Medicine 25 collection. For fast and easy video access, right from your mobile device, follow these instructions. The videos are also available on Expertconsult.com.

What You Need

• A mobile device, such as a smartphone or tablet, equipped with a camera and Internet access • A QR code reader application (If you do not already have a reader installed on your mobile device, look for free versions in your app store.)

How It Works

• Open the QR code reader application on your mobile device. • Point the device’s camera at the code and scan. • Each code opens an individual video player for instant viewing—no log-on required.

Confusion Assessment Method (CAM) Chapter 28, Video 28-1 – Marcos Mialnez, Jorge G. Ruiz, and Rosanne M. Leipzig

Standard Echocardiographic Views: Four-Chamber Image Plane Chapter 55, Video 55-1D – Catherine M. Otto

Interlaminar Epidural Steroid Injection Chapter 30, Video 30-1 – Ali Turabi

Dilated Cardiomyopathy: Long Axis View Chapter 55, Video 55-2A – Catherine M. Otto

Standard Echocardiographic Views: Long Axis Image Plane Chapter 55, Video 55-1A – Catherine M. Otto

Dilated Cardiomyopathy: Short Axis View Chapter 55, Video 55-2B – Catherine M. Otto

Standard Echocardiographic Views: Short Axis Image Plane Chapter 55, Video 55-1B – Catherine M. Otto

Dilated Cardiomyopathy: Apical Four-Chamber View Chapter 55, Video 55-2C – Catherine M. Otto

Standard Echocardiographic Views: Short Axis Image Plane Chapter 55, Video 55-1C – Catherine M. Otto

Three-Dimensional Echocardiography Chapter 55, Video 55-3 – Catherine M. Otto

4

Quick Reference (QR) Video Access

Stress Echocardiography: Normal Reaction Chapter 55, Video 55-4A – Catherine M. Otto

Perimembranous Ventricular Septal Defect Chapter 69, Video 69-2 – Ariane J. Marelli

Stress Echocardiography: Normal Reaction Chapter 55, Video 55-4B – Catherine M. Otto

Coronary Stent Placement Chapter 74, Video 74-1 – Paul S. Teirstein

Stress Echocardiography: Proximal Stenosis of the Left Anterior Descending Coronary Artery Chapter 55, Video 55-4C – Catherine M. Otto

Guidewire Passage Chapter 74, Video 74-2 – Paul S. Teirstein

Stress Echocardiography: Proximal Stenosis of the Left Anterior Descending Coronary Artery Chapter 55, Video 55-4D – Catherine M. Otto

Delivering the Stent Chapter 74, Video 74-3 – Paul S. Teirstein

Pericardial Effusion: Parasternal Long Axis Chapter 55, Video 55-5A – Catherine M. Otto

Inflating the Stent Chapter 74, Video 74-4 – Paul S. Teirstein

Pericardial Effusion: Parasternal Short Axis Chapter 55, Video 55-5B – Catherine M. Otto

Final Result Chapter 74, Video 74-5 – Paul S. Teirstein

Pericardial Effusion: Apical Four-Chamber Views Chapter 55, Video 55-5C – Catherine M. Otto

Superficial Femoral Artery (SFA) Stent Procedure Chapter 79, Video 79-1 – Christopher J. White

Secundum Atrial Septal Defect Chapter 69, Video 69-1 – Ariane J. Marelli

Orthotopic Bicaval Cardiac Transplantation Chapter 82, Video 82-1 – Y. Joseph Woo

Quick Reference (QR) Video Access

Wheezing Chapter 87, Video 87-1 – Jeffrey M. Drazen

Endoscopic Mucosal Resection Using Saline Lift Polypectomy of a Colon Adenoma Followed by Closure of the Mucosal Defect with Clips Chapter 193, Video 193-3 – Douglas O. Faigel

VATS Wedge Resection Chapter 101, Video 101-1 – Malcolm M. DeCamp

Endoscopic View of Rectal Cancer Chapter 193, Video 193-4 – Douglas O. Faigel

Ventilation of an Ex Vivo Rat Lung Chapter 105, Video 105-1 – Arthur S. Slutsky, George Volgyesi, and Tom Whitehead

Endoscopic Ultrasound Chapter 193, Video 193-5 – Douglas O. Faigel

Renal Artery Stent Chapter 125, Video 125-1 – Renato M. Santos and Thomas D. DuBose, Jr.

Laparoscopic Roux-en-Y Gastric Bypass Chapter 220, Video 220-1 – James M. Swain

Interpretation of a Computed Tomographic Colonography Chapter 133, Video 133-1 – David H. Kim

Pituitary Surgery Chapter 224, Video 224-1 – Ivan Ciric

Donor Liver Transportation–Donor and Recipient Chapter 154, Video 154-1 – Igal Kam, Thomas Bak, and Michael Wachs

Skin Testing Chapter 251, Video 251-1 – Larry Borish

Snare Polypectomy of a Colon Adenoma Chapter 193, Video 193-1 – Douglas O. Faigel

Nasal Endoscopy Chapter 251, Video 251-2 – Larry Borish

Laparascopic-Assisted Double Balloon Enteroscopy with Polypectomy of a Jejunal Adenoma Followed by Surgical Oversew of the Polypectomy Site Chapter 193, Video 193-2 – Douglas O. Faigel

Hip Arthroscopy Osteochondroplasty Chapter 276, Video 276-1 – Bryan T. Kelly

5

6

Quick Reference (QR) Video Access

Cervical Provocation Chapter 400, Video 400-1 – Richard L. Barbano

Left Rolandic Seizure Chapter 403, Video 403-2 – Samuel Wiebe

Spurling Maneuver Chapter 400, Video 400-2 – Richard L. Barbano

Left Temporal Complex Partial Seizure Chapter 403, Video 403-3 – Samuel Wiebe

Cervical Distraction Test Chapter 400, Video 400-3 – Richard L. Barbano

Left Temporal Complex Partial Seizure Postictal Confusion Chapter 403, Video 403-4 – Samuel Wiebe

Straight Leg Raise Chapter 400, Video 400-4 – Richard L. Barbano

Left Temporal Complex Partial Seizure Chapter 403, Video 403-5 – Samuel Wiebe

Contralateral Straight Leg Raise Chapter 400, Video 400-5 – Richard L. Barbano

Supplementary Sensory-Motor Seizure Chapter 403, Video 403-6 – Samuel Wiebe

Seated Straight Leg Raise Chapter 400, Video 400-6 – Richard L. Barbano

Right Posterior Temporal Seizure - Dramatic Frontal Semiology Chapter 403, Video 403-7 – Samuel Wiebe

Discectomy Chapter 400, Video 400-7 – Jason H. Huang

Right Mesial Frontal Seizure Chapter 403, Video 403-8 – Samuel Wiebe

Absence Seizure Chapter 403, Video 403-1 – Samuel Wiebe

Nonconvulsive Status Epilepticus Chapter 403, Video 403-9 – Samuel Wiebe

Quick Reference (QR) Video Access

GTC Seizure Tonic Phase Chapter 403, Video 403-10 – Samuel Wiebe

Minimally Conscious State Chapter 404, Video 404-3 – James L. Bernat and Eelco F. M. Wijdicks

GTC Seizure Clonic Phase Chapter 403, Video 403-11 – Samuel Wiebe

Akinetic Mutism Chapter 404, Video 404-4 – James L. Bernat and Eelco F. M. Wijdicks

Myoclonic Facial Seizure Chapter 403, Video 403-12 – Samuel Wiebe

Early Parkinson’s Disease Chapter 409, Video 409-1 – Anthony E. Lang

Tonic Seizure Lennox Gastaut Chapter 403, Video 403-13 – Samuel Wiebe

Freezing of Gait in Parkinson’s Disease Chapter 409, Video 409-2 – Anthony E. Lang

Atonic Seizure Lennox Gastaut Chapter 403, Video 403-14 – Samuel Wiebe

Gunslinger Gait in Progressive Supranuclear Palsy Chapter 409, Video 409-3 – Anthony E. Lang

Reflex Auditory Seizure Chapter 403, Video 403-15 – Samuel Wiebe

Supranuclear Gaze Palsy in Progressive Supranuclear Palsy Chapter 409, Video 409-4 – Anthony E. Lang

Four Score Chapter 404, Video 404-1 – James L. Bernat and Eelco F. M. Wijdicks

Applause Sign in Progressive Supranuclear Palsy Chapter 409, Video 409-5 – Anthony E. Lang

Persistent Vegetative State Chapter 404, Video 404-2 – James L. Bernat and Eelco F. M. Wijdicks

Apraxia of Eyelid Opening in Progressive Supranuclear Palsy Chapter 409, Video 409-6 – Anthony E. Lang

7

8

Quick Reference (QR) Video Access

Cranial Dystonia in Multiple System Atrophy Chapter 409, Video 409-7 – Anthony E. Lang

Hemiballism Chapter 410, Video 410-3 – Anthony E. Lang

Anterocollis in Multiple System Atrophy Chapter 409, Video 409-8 – Anthony E. Lang

Blepharospasm Chapter 410, Video 410-4 – Anthony E. Lang

Stridor in Multiple System Atrophy Chapter 409, Video 409-9 – Anthony E. Lang

Oromandibular Dystonia Chapter 410, Video 410-5 – Anthony E. Lang

Alien Limb Phenomenon in Corticobasal Syndrome Chapter 409, Video 409-10 – Anthony E. Lang

Cervical Dystonia Chapter 410, Video 410-6 – Anthony E. Lang

Myoclonus in Corticobasal Syndrome Chapter 409, Video 409-11 – Anthony E. Lang

Writer’s Cramp Chapter 410, Video 410-7 – Anthony E. Lang

Levodopa-Induced Dyskinesia in Parkinson’s Disease Chapter 409, Video 409-12 – Anthony E. Lang

Embouchure Dystonia Chapter 410, Video 410-8 – Anthony E. Lang

Essential Tremor Chapter 410, Video 410-1 – Anthony E. Lang

Sensory Trick in Cervical Dystonia Chapter 410, Video 410-9 – Anthony E. Lang

Huntington’s Disease Chapter 410, Video 410-2 – Anthony E. Lang

Generalized Dystonia Chapter 410, Video 410-10 – Anthony E. Lang

Quick Reference (QR) Video Access

Tics Chapter 410, Video 410-11 – Anthony E. Lang

Limb Symptoms and Signs Chapter 419, Video 419-1 – Pamela J. Shaw

Tardive Dyskinesia Chapter 410, Video 410-12 – Anthony E. Lang

Bulbar Symptoms and Signs Chapter 419, Video 419-2 – Pamela J. Shaw

Hemifacial Spasm Chapter 410, Video 410-13 – Anthony E. Lang

Normal Swallowing Chapter 419, Video 419-3 – Pamela J. Shaw

Wernickes Encephalopathy Eye Movements: Before Thiamine Chapter 416, Video 416-1 – Barbara S. Koppel

Charcot-Marie-Tooth Disease Exam and Walk Chapter 420, Video 420-1 – Michael E. Shy

Wernickes Encephalopathy Eye Movements: After Thiamine Chapter 416, Video 416-2 – Barbara S. Koppel

9

This page intentionally left blank

GOLDMAN-CECIL MEDICINE

This page intentionally left blank

GOLDMAN-CECIL MEDICINE 25TH EDITION Volume II

EDITED BY

LEE GOLDMAN, MD

Harold and Margaret Hatch Professor Executive Vice President and Dean of the Faculties of Health Sciences and Medicine Chief Executive, Columbia University Medical Center Columbia University New York, New York

ANDREW I. SCHAFER, MD

Professor of Medicine Director, Richard T. Silver Center for Myeloproliferative Neoplasms Weill Cornell Medical College New York, New York

1600 John F. Kennedy Blvd. Ste. 1800 Philadelphia, PA 19103-2899

GOLDMAN-CECIL MEDICINE, 25TH EDITION

ISBN: 978-1-4557-5017-7 Volume 1 Part Number: 9996096564 Volume 2 Part Number: 9996096629

International Edition (IE):

ISBN: 978-0-323-28800-2 IE Volume 1 Part Number: 9996118347 IE Volume 2 Part Number: 9996118282

Copyright © 2016, 2012, 2008, 2004, 2000, 1996, 1991, 1988, 1982, 1979, 1975, 1971, 1963, 1959, 1955 by Saunders, an imprint of Elsevier Inc. Copyright 1951, 1947, 1943, 1940, 1937, 1933, 1930, 1927 by Saunders, an imprint of Elsevier Inc. Copyright renewed 1991 by Paul Beeson. Copyright renewed 1979 by Russell L. Cecil and Robert F. Loeb. Copyright renewed 1987, 1975, 1971, 1965, 1961, 1958, 1955 by Elsevier Inc. All rights reserved. No part of this publication may be reproduced or transmitted in any form or by any means, electronic or mechanical, including photocopying, recording, or any information storage and retrieval system, without permission in writing from the Publisher. Details on how to seek permission, further information about the Publisher’s permissions policies and our arrangements with organizations such as the Copyright Clearance Center and the Copyright Licensing Agency, can be found at our website: www.elsevier.com/permissions. This book and the individual contributions contained in it are protected under copyright by the Publisher (other than as may be noted herein).

Notices Knowledge and best practice in this field are constantly changing. As new research and experience broaden our understanding, changes in research methods, professional practices, or medical treatment may become necessary. Practitioners and researchers must always rely on their own experience and knowledge in evaluating and using any information, methods, compounds, or experiments described herein. In using such information or methods they should be mindful of their own safety and the safety of others, including parties for whom they have a professional responsibility. With respect to any drug or pharmaceutical products identified, readers are advised to check the most current information provided (i) on procedures featured or (ii) by the manufacturer of each product to be administered, to verify the recommended dose or formula, the method and duration of administration, and contraindications. It is the responsibility of practitioners, relying on their own experience and knowledge of their patients, to make diagnoses, to determine dosages and the best treatment for each individual patient, and to take all appropriate safety precautions. To the fullest extent of the law, neither the Publisher nor the authors, contributors, or editors assume any liability for any injury and/or damage to persons or property as a matter of products liability, negligence or otherwise, or from any use or operation of any methods, products, instructions, or ideas contained in the material herein. Library of Congress Cataloging-in-Publication Data Goldman’s Cecil medicine.   Goldman-Cecil medicine / [edited by] Lee Goldman, Andrew I. Schafer.—25th edition.    p. ; cm.   Cecil medicine   Preceded by Goldman’s Cecil medicine / [edited by] Lee Goldman, Andrew I. Schafer. 24th ed. c2012.   Includes bibliographical references.   ISBN 978-1-4557-5017-7 (hardcover, 2 vol set : alk. paper)—ISBN 978-0-323-28800-2 (international edition : alk. paper)—ISBN 978-9996096563 (volume 1 : alk. paper)—ISBN 9996096564 (volume 1 : alk. paper)—ISBN 978-9996096624 (volume 2 : alk. paper)—ISBN 9996096629 (volume 2 : alk. paper)   I.  Goldman, Lee (Physician), editor.  II.  Schafer, Andrew I., editor.  III.  Title.  IV.  Title: Cecil medicine.   [DNLM:  1.  Medicine.  WB 100]   RC46   616—dc23 2014049904 Executive Content Strategist: Kate Dimock Senior Content Development Manager: Maureen Iannuzzi Publishing Services Manager: Anne Altepeter Senior Project Manager: Cindy Thoms Design Specialist: Paula Catalano Printed in the United States of America Last digit is the print number:  9  8  7  6  5  4  3  2  1

ASSOCIATE EDITORS Mary K. Crow, MD

Joseph P. Routh Professor of Rheumatic Diseases in Medicine Weill Cornell Medical College Physician-in-Chief and Benjamin M. Rosen Chair in Immunology and Inflammation Research Hospital for Special Surgery New York, New York

James H. Doroshow, MD Bethesda, Maryland

Jeffrey M. Drazen, MD

Distinguished Parker B. Francis Professor of Medicine Harvard Medical School Senior Physician Brigham and Women’s Hospital Boston, Massachusetts

Robert C. Griggs, MD

Professor of Neurology, Medicine, Pediatrics, and Pathology and Laboratory Medicine University of Rochester School of Medicine and Dentistry Rochester, New York

Donald W. Landry, MD, PhD

Samuel Bard Professor of Medicine Chair, Department of Medicine Physician-in-Chief Columbia University Medical Center New York, New York

Wendy Levinson, MD Professor of Medicine Chair Emeritus Department of Medicine University of Toronto Toronto, Ontario, Canada

Anil K. Rustgi, MD

T. Grier Miller Professor of Medicine and Genetics Chief of Gastroenterology American Cancer Society Professor University of Pennsylvania Perelman School of Medicine Philadelphia, Pennsylvania

W. Michael Scheld, MD

Bayer-Gerald L. Mandell Professor of Infectious Diseases Professor of Medicine Clinical Professor of Neurosurgery Director, Pfizer Initiative in International Health University of Virginia Health System Charlottesville, Virginia

Allen M. Spiegel, MD

Dean Albert Einstein College of Medicine Bronx, New York

This page intentionally left blank

PREFACE In the 90 years since the first edition of the Cecil Textbook of Medicine was published, almost everything we know about internal medicine has changed. Progress in medical science is now occurring at an ever-accelerating pace, and it is doing so within the framework of transformational changes in clinical practice and the delivery of health care at individual, social, and global levels. This textbook and its associated electronic products incorporate the latest medical knowledge in multiple formats that should appeal to students and seasoned practitioners regardless of how they prefer to access this rapidly changing information. Even as Cecil’s specific information has changed, however, we have remained true to the tradition of a comprehensive textbook of medicine that carefully explains the why (the underlying pathophysiology of disease) and the how (now expected to be evidence-based from randomized controlled trials and meta-analyses). Descriptions of physiology and pathophysiology include the latest genetic advances in a practical format that strives to be useful to the nonexpert. Medicine has entered an era when the acuity of illness and the limited time available to evaluate a patient have diminished the ability of physicians to satisfy their intellectual curiosity. As a result, the acquisition of information, quite easily achieved in this era, is often confused with knowledge. We have attempted to address this dilemma with a textbook that not only informs but also stimulates new questions and gives a glimpse of the future path to new knowledge. Grade A evidence is specifically highlighted in the text and referenced at the end of each chapter. In addition to the information provided in the textbook, the Cecil website supplies expanded content and functionality. In many cases, the full articles referenced in each chapter can be accessed from the Cecil website. The website is also continuously updated to incorporate subsequent Grade A information, other evidence, and new discoveries. The sections for each organ system begin with a chapter that summarizes an approach to patients with key symptoms, signs, or laboratory abnormalities associated with dysfunction of that organ system. As summarized in E-Table 1-1, the text specifically provides clear, concise information regarding how a physician should approach more than 100 common symptoms, signs, and laboratory abnormalities, usually with a flow diagram, a table, or both for easy reference. In this way, Cecil remains a comprehensive text to guide diagnosis and therapy, not only for patients with suspected or known diseases but also for patients who may have undiagnosed abnormalities that require an initial evaluation. Just as each edition brings new authors, it also reminds us of our gratitude to past editors and authors. Previous editors of Cecil include a short but remarkably distinguished group of leaders of American medicine: Russell Cecil, Paul Beeson, Walsh McDermott, James Wyngaarden, Lloyd H. Smith,

Jr., Fred Plum, J. Claude Bennett, and Dennis Ausiello. As we welcome new associate editors—Mary K. Crow, James H. Doroshow, and Allen M. Spiegel—we also express our appreciation to William P. Arend, James O. Armitage, David R. Clemmons, and other associate editors from the previous editions on whose foundation we have built. Our returning associate editors—Jeffrey M. Drazen, Robert C. Griggs, Donald W. Landry, Wendy Levinson, Anil K. Rustgi, and W. Michael Scheld—continue to make critical contributions to the selection of authors and the review and approval of all manuscripts. The editors, however, are fully responsible for the book as well as the integration among chapters. The tradition of Cecil is that all chapters are written by distinguished experts in each field. We are also most grateful for the editorial assistance in New York of Maribel Lim and Silva Sergenian. These individuals and others in our offices have shown extraordinary dedication and equanimity in working with authors and editors to manage the unending flow of manuscripts, figures, and permissions. We also thank Cassondra Andreychik, Ved Bhushan Arya, Cameron Harrison, Karen Krok, Robert J. Mentz, Gaétane Nocturne, Patrice Savard, Senthil Senniappan, Tejpratap Tiwari, and Sangeetha Venkatarajan, who contributed to various chapters, and we mourn the passing of Morton N. Swartz, MD, co-author of the chapter on “Meningitis: Bacterial, Viral, and Other” and Donald E. Low, MD, author of the chapter “Nonpneumococcal Streptococcal Infections, Rheumatic Fever.” At Elsevier, we are most indebted to Kate Dimock and Maureen Iannuzzi, and also thank Maria Holman, Gabriela Benner, Cindy Thoms, Anne Altepeter, Linda McKinley, Paula Catalano, and Kristin Koehler, who have been critical to the planning and production process under the guidance of Mary Gatsch. Many of the clinical photographs were supplied by Charles D. Forbes and William F. Jackson, authors of Color Atlas and Text of Clinical Medicine, Third Edition, published in 2003 by Elsevier Science Ltd. We thank them for graciously permitting us to include their pictures in our book. We have been exposed to remarkable physicians in our lifetimes and would like to acknowledge the mentorship and support of several of those who exemplify this paradigm— Eugene Braunwald, Lloyd H. Smith, Jr., Frank Gardner, and William Castle. Finally, we would like to thank the Goldman family—Jill, Jeff, Abigail, Mira, Samuel, Daniel, Robyn, Tobin, and Dashel—and the Schafer family— Pauline, Eric, Melissa, Nathaniel, Pam, John, Evan, Samantha, Kate, and Sean, for their understanding of the time and focus required to edit a book that attempts to sustain the tradition of our predecessors and to meet the needs of today’s physician. LEE GOLDMAN, MD ANDREW I. SCHAFER, MD

This page intentionally left blank

CONTRIBUTORS Charles S. Abrams, MD Professor of Medicine, Pathology, and Laboratory Medicine, University of Pennsylvania School of Medicine; Director, PENN-Chop Blood Center for Patient Care & Discovery, Hospital of the University of Pennsylvania, Philadelphia, Pennsylvania Thrombocytopenia Frank J. Accurso, MD Professor of Pediatrics, University of Colorado School of Medicine; Attending Physician, Children’s Hospital Colorado, Aurora, Colorado Cystic Fibrosis Ronald S. Adler, MD, PhD Professor of Radiology, New York University School of Medicine; Department of Radiology, NYU Langone Medical Center, New York, New York Imaging Studies in the Rheumatic Diseases Cem Akin, MD, PhD Associate Professor, Harvard Medical School; Attending Physician, Director, Mastocytosis Center, Brigham and Women’s Hospital, Department of Medicine, Division of Rheumatology, Immunology, and Allergy, Boston, Massachusetts Mastocytosis Allen J. Aksamit, Jr., MD Professor of Neurology, Mayo Clinic College of Medicine, Consultant in Neurology, Mayo Clinic, Rochester, Minnesota Acute Viral Encephalitis Qais Al-Awqati, MB ChB Robert F. Loeb Professor of Medicine, Jay I. Meltzer Professor of Nephrology and Hypertension, Professor of Physiology and Cellular Biophysics, Division of Nephrology, Columbia University, College of Physicians and Surgeons, New York, New York Structure and Function of the Kidneys Ban Mishu Allos, MD Associate Professor of Medicine, Division of Infectious Diseases, Associate Professor, Preventive Medicine, Vanderbilt University School of Medicine, Nashville, Tennessee Campylobacter Infections David Altshuler, MD, PhD Professor of Genetics and of Medicine, Harvard Medical School, Massachusetts General Hospital; Professor of Biology (Adjunct), Massachusetts Institute of Technology, Boston and Cambridge, Massachusetts The Inherited Basis of Common Diseases

Larry J. Anderson, MD Professor, Division of Infectious Disease, Department of Pediatrics, Emory University School of Medicine and Children’s Healthcare of Atlanta, Atlanta, Georgia Coronaviruses Aśok C. Antony, MD Chancellor’s Professor of Medicine, Indiana University School of Medicine; Attending Physician, Indiana University Health Affiliated Hospitals and Richard L. Roudebush Veterans Affairs Medical Center, Indianapolis, Indiana Megaloblastic Anemias Gerald B. Appel, MD Professor of Medicine, Division of Nephrology, Department of Medicine, Columbia University College of Physicians and Surgeons, New York, New York Glomerular Disorders and Nephrotic Syndromes Frederick R. Appelbaum, MD Executive Vice President and Deputy Director, Fred Hutchinson Cancer Research Center; President, Seattle Cancer Care Alliance; Professor, Division of Medical Oncology, University of Washington School of Medicine, Seattle Washington The Acute Leukemias Suneel S. Apte, MBBS, DPhil Staff, Cleveland Clinic Lerner College of Medicine at Case Western Reserve University, Cleveland, Ohio Connective Tissue Structure and Function James O. Armitage, MD The Joe Shapiro Professor of Internal Medicine, University of Nebraska Medical Center, Omaha, Nebraska Approach to the Patient with Lymphadenopathy and Splenomegaly, Non-Hodgkin Lymphomas M. Amin Arnaout, MD Professor of Medicine, Departments of Medicine and Developmental and Regenerative Biology, Harvard Medical School; Physician and Chief Emeritus, Division of Nephrology, Massachusetts General Hospital, Boston, Massachusetts Cystic Kidney Diseases Robert M. Arnold, MD Leo H. Criep Professor of Clinical Care, Chief, Section of Palliative Care and Medical Ethics, University of Pittsburgh; Medical Director, UPMC Palliative and Supportive Care Institute, Pittsburgh, Pennsylvania Care of Dying Patients and Their Families

Michael Aminoff, MD, DSc Professor, Department of Neurology, University of California San Francisco, San Francisco, California Approach to the Patient with Neurologic Disease

David Atkins, MD, MPH Director, Health Services Research and Development, Veterans Health Administration, Washington, D.C. The Periodic Health Examination

Jeffrey L. Anderson, MD Professor of Internal Medicine, University of Utah School of Medicine; Vice-Chair for Research, Department of Internal Medicine, Associate Chief of Cardiology and Director of Cardiovascular Research, Intermountain Medical Center, Intermountain Healthcare, Salt Lake City, Utah ST Segment Elevation Acute Myocardial Infarction and Complications of Myocardial Infarction

John P. Atkinson, MD Chief, Division of Rheumatology, Internal Medicine, Washington University School of Medicine in St. Louis, St. Louis, Missouri Complement System in Disease

x

Contributors

Bruce R. Bacon, MD Endowed Chair in Gastroenterology, Professor of Internal Medicine, Co-Director, Saint Louis University Liver Center; Director, Saint Louis University Abdominal Transplant Center, Saint Louis University School of Medicine, St. Louis, Missouri Iron Overload (Hemochromatosis) Larry M. Baddour, MD Professor of Medicine, Chair, Division of Infectious Diseases, Mayo Clinic, Rochester, Minnesota Infective Endocarditis Grover C. Bagby, MD Professor of Medicine and Molecular and Medical Genetics, Knight Cancer Institute at Oregon Health and Science University and Portland VA Medical Center, Portland, Oregon Aplastic Anemia and Related Bone Marrow Failure States Barbara J. Bain, MBBS Professor in Diagnostic Haematology, Imperial College London; Honorary Consultant Haematologist, St. Mary’s Hospital, London, United Kingdom The Peripheral Blood Smear Dean F. Bajorin, MD Attending Physician and Member, Medicine, Memorial Hospital, Memorial Sloan Kettering Cancer Center; Professor of Medicine, Weill Cornell Medical College, New York, New York Tumors of the Kidney, Bladder, Ureters, and Renal Pelvis

Stephen G. Baum, MD Chairman of Medicine, Mount Sinai Beth Israel Hospital; Professor of Medicine and of Microbiology and Immunology, Albert Einstein College of Medicine, New York, New York Mycoplasma Infections Daniel G. Bausch, MD, MPH&TM Associate Professor, Department of Tropical Medicine, Tulane University Health Sciences Center, New Orleans, Louisiana Viral Hemorrhagic Fevers Arnold S. Bayer, MD Professor of Medicine, David Geffen School of Medicine at University of California Los Angeles; LA Biomedical Research Institute; Vice Chair for Academic Affairs, Department of Medicine, Harbor-UCLA Medical Center, Los Angeles, California Infective Endocarditis Hasan Bazari, MD Associate Professor of Medicine, Harvard Medical School, Department of Medicine, Clinical Director, Nephrology, Program Director, Internal Medicine Residency Program, Massachusetts General Hospital, Boston, Massachusetts Approach to the Patient with Renal Disease John H. Beigel, MD National Institute of Allergy and Infectious Diseases, National Institutes of Health, Bethesda, Maryland Antiviral Therapy (Non-HIV)

Robert W. Baloh, MD Professor of Neurology, University of California Los Angeles School of Medicine, Los Angeles, California Neuro-Ophthalmology; Smell and Taste; Hearing and Equilibrium

George A. Beller, MD Professor of Medicine, University of Virginia Health System, Charlottesville, Virginia Noninvasive Cardiac Imaging

Jonathan Barasch, MD, PhD Professor of Medicine and Pathology and Cell Biology, Department of Medicine, Division of Nephrology, Columbia University College of Physicians & Surgeons, New York, New York Structure and Function of the Kidneys

Robert M. Bennett, MD Professor of Medicine, Oregon Health and Science University, Portland, Oregon Fibromyalgia, Chronic Fatigue Syndrome, and Myofascial Pain

Richard L. Barbano, MD, PhD Professor of Neurology, University of Rochester, Rochester, New York Mechanical and Other Lesions of the Spine, Nerve Roots, and Spinal Cord Elizabeth Barrett-Connor, MD Professor of Community and Family Medicine, University of California San Diego, San Diego, California Menopause John R. Bartholomew, MD Section Head, Vascular Medicine, Cardiovascular Medicine, Cleveland Clinic, Professor of Medicine, Cleveland Clinic Lerner College of Medicine of Case Western Reserve University, Cleveland, Ohio Other Peripheral Arterial Diseases Mary Barton, MD, MPP Vice President, Performance Measurement, National Committee for Quality Assurance, Washington, D.C. The Periodic Health Examination Robert C. Basner, MD Professor of Medicine, Columbia University Medical Center; Director, Columbia University Cardiopulmonary Sleep and Ventilatory Disorders Center, Columbia University College of Physicians and Surgeons, New York, New York Obstructive Sleep Apnea

Joseph R. Berger, MD Professor of Neurology, Chief of the Multiple Sclerosis Division, Department of Neurology, Perelman School of Medicine, University of Pennsylvania, Philadelphia, Pennsylvania Cytomegalovirus, Epstein-Barr Virus, and Slow Virus Infections of the Central Nervous System; Neurologic Complications of Human Immunodeficiency Virus Infection; Brain Abscess and Parameningeal Infections Paul D. Berk, MD Professor of Medicine, Department of Medicine, Columbia University College of Physicians and Surgeons, New York, New York Approach to the Patient with Jaundice or Abnormal Liver Tests Nancy Berliner, MD Professor of Medicine, Harvard Medical School; Chief, Division of Hematology, Brigham and Women’s Hospital, Boston, Massachusetts Leukocytosis and Leukopenia James L. Bernat, MD Louis and Ruth Frank Professor of Neuroscience, Professor of Neurology and Medicine, Geisel School of Medicine at Dartmouth, Hanover, New Hampshire; Department of Neurology, Dartmouth-Hitchcock Medical Center, Lebanon, New Hampshire Coma, Vegetative State, and Brain Death Philip J. Bierman, MD Professor, Department of Internal Medicine, University of Nebraska Medical Center, Omaha, Nebraska Approach to the Patient with Lymphadenopathy and Splenomegaly; Non-Hodgkin Lymphomas

Contributors

xi

Michael R. Bishop, MD Professor of Medicine, Director, Hematopoietic Cellular Therapy Program, Section of Hematology and Oncology, Department of Medicine, University of Chicago, Chicago, Illinois Hematopoietic Stem Cell Transplantation

William E. Boden, MD Professor of Medicine, Albany Medical College; Chief of Medicine, Albany Stratton VA Medical Center; Vice-Chairman, Department of Medicine, Albany Medical Center, Albany, New York Angina Pectoris and Stable Ischemic Heart Disease

Bruce R. Bistrian, MD, PhD, MPH Professor of Medicine, Beth Israel Deaconess Medical Center; Professor of Medicine, Harvard Medical School, Boston, Massachusetts Nutritional Assessment

Jean Bolognia, MD Professor of Dermatology, Yale Medical School; Attending Physician, Yale-New Haven Hospital, New Haven, Connecticut Infections, Hyperpigmentation and Hypopigmentation, Regional Dermatology, and Distinctive Lesions in Black Skin

Joseph J. Biundo, MD Clinical Professor of Medicine, Tulane Medical Center, New Orleans, Louisiana Bursitis, Tendinitis, and Other Periarticular Disorders and Sports Medicine Adrian R. Black, PhD Assistant Professor, Director of Tissue Sciences for the Eppley Institute, The Eppley Institute for Research in Cancer and Allied Diseases, University of Nebraska Medical Center, Omaha, Nebraska Cancer Biology and Genetics Charles D. Blanke, MD Professor of Medicine, Oregon Health and Science University, Portland, Oregon Neoplasms of the Small and Large Intestine Joel N. Blankson, MD, PhD Associate Professor, Johns Hopkins University School of Medicine, Baltimore, Maryland Immunopathogenesis of Human Immunodeficiency Virus Infection Martin J. Blaser, MD Muriel and George Singer Professor of Medicine, Professor of Microbiology, Director, Human Microbiome Program, New York University Langone Medical Center, New York, New York Acid Peptic Disease; Human Microbiome William A. Blattner, MD Professor and Associate Director, Institute of Human Virology, School of Medicine, University of Maryland; Professor of Medicine, School of Medicine, University of Maryland; Professor and Head, Division of Cancer Epidemiology, Department of Epidemiology and Public Health, School of Medicine, University of Maryland, Baltimore, Maryland Retroviruses Other Than Human Immunodeficiency Virus Thomas P. Bleck, MD Professor of Neurological Sciences, Neurosurgery, Internal Medicine, and Anesthesiology, Associate Chief Medical Officer (Critical Care), Rush Medical College, Chicago, Illinois Arboviruses Affecting the Central Nervous System Joel A. Block, MD The Willard L. Wood MD Professor and Director, Division of Rheumatology, Rush University Medical Center, Chicago, Illinois Osteoarthritis Henk Blom, MD Laboratory of Clinical Biochemistry and Metabolism, Department of General Pediatrics, Adolescent Medicine and Neonatology, University Medical Centre Freiburg, Head of Laboratory/Clinical Biochemical Geneticist, Freiburg, Germany Homocystinuria and Hyperhomocysteinemia Olaf A. Bodamer, MD Medical Genetics, University of Miami Hospital, Miami, Florida Approach to Inborn Errors of Metabolism

Robert A. Bonomo, MD Chief, Medical Service, Louis Stokes Cleveland VA Medical Center; Professor of Medicine, Pharmacology, Biochemistry, Molecular Biology, and Microbiology, Case Western Reserve University School of Medicine, Cleveland, Ohio Diseases Caused by Acinetobacter and Stenotrophomonas Species Larry Borish, MD Professor of Medicine, Allergy, and Clinical Immunology, University of Virginia Health System, Charlottesville, Virgina Allergic Rhinitis and Chronic Sinusitis Patrick J. Bosque, MD Associate Professor of Neurology, University of Colorado Denver School of Medicine; Neurologist, Denver Health Medical Center, Denver, Colorado Prion Diseases David J. Brenner, PhD, DSc Higgins Professor of Radiation Biophysics, Center for Radiological Research, Columbia University Medical Center, New York, New York Radiation Injury Itzhak Brook, MD, MSc Professor of Pediatrics and Medicine, Georgetown University, Georgetown University Medical Center, Washington, D.C. Diseases Caused by Non–Spore-Forming Anaerobic Bacteria; Actinomycosis Enrico Brunetti, MD Assistant Professor of Infectious Diseases, University of Pavia; Attending Physician, Division of Infectious and Tropical Diseases, IRCCS San Matteo Hospital Foundation; Co-Director, WHO Collaborating Centre for Clinical Management of Cystic Echinococcosis, Pavia, Italy Cestodes David M. Buchner, MD, MPH Shahid and Ann Carlson Khan Professor in Applied Health Sciences, Department of Kinesiology and Community Health, University of Illinois at Urbana-Champaign, Champaign, Illinois Physical Activity Pierre A. Buffet, MD, PhD Research Unit Head, Erythrocyte Parasite Pathogenesis Research Team INSERM–University Paris 6, CIMI–Paris Research Center, University Pierre and Marie Curie; Associate Professor of Parasitology, Faculty of Medicine, University Pierre and Marie Curie, Pitié-Salpêtrière Hospital, Paris, France Leishmaniasis H. Franklin Bunn, MD Professor of Medicine, Harvard Medical School; Physician, Brigham and Women’s Hospital, Boston, Massachusetts Approach to the Anemias David A. Bushinsky, MD John J. Kuiper Distinguished Professor of Medicine, Chief, Nephrology Division, University of Rochester School of Medicine; Associate Chair for Academic Affairs in Medicine, University of Rochester Medical Center, Rochester, New York Nephrolithiasis

xii

Contributors

Vivian P. Bykerk, MD Associate Professor of Medicine, Weill Cornell Medical College; Associate Attending Physician, Hospital for Special Surgery, New York, New York Approach to the Patient with Rheumatic Disease Peter A. Calabresi, MD Professor of Neurology and Director of the Richard T. Johnson Division of Neuroimmunology and Neuroinfectious Diseases, Johns Hopkins University; Director of the Multiple Sclerosis Center, Johns Hopkins Hospital, Baltimore, Maryland Multiple Sclerosis and Demyelinating Conditions of the Central Nervous System David P. Calfee, MD, MS Associate Professor of Medicine and Healthcare Policy and Research, Weill Cornell Medical College; Chief Hospital Epidemiologist, New YorkPresbyterian Hospital/Weill Cornell Medical Center, New York, New York Prevention and Control of Health Care–Associated Infections Douglas Cameron, MD, MBA Professor of Ophthalmology and Visual Neurosciences, University of Minnesota, Minneapolis, Minnesota Diseases of the Visual System Michael Camilleri, MD Atherton and Winifred W. Bean Professor, Professor of Medicine, Pharmacology, and Physiology, College of Medicine, Mayo Clinic, Consultant, Division of Gastroenterology and Hepatology, Mayo Clinic, Rochester, Minnesota Disorders of Gastrointestinal Motility Grant W. Cannon, MD Thomas E. and Rebecca D. Jeremy Presidential Endowed Chair for Arthritis Research, Associate Chief of Staff for Academic Affiliations, George E. Wahlen VA Medical Center, Salt Lake City, Utah Immunosuppressing Drugs Including Corticosteroids Maria Domenica Cappellini, MD Professor of Internal Medicine, University of Milan, Fondazione IRCCS Ca’ Granda Ospedale Maggiore Policlinico, University of Milan, Milan, Italy The Thalassemias Blase A. Carabello, MD Professor of Medicine, Chairman, Department of Cardiology, Mount Sinai Beth Israel Heart Institute, New York, New York Valvular Heart Disease Edgar M. Carvalho, MD Professor of Medicine and Clinical Immunology, Faculdade de Medicina da Bahia, Universidade Federal da Bahia and Escola Bahiana de Medicina e Saúde Pública, Salvador, Bahia, Brazil Schistosomiasis (Bilharziasis) William H. Catherino, MD, PhD Professor and Research Head, Department of Obstetrics and Gynecology, Uniformed Services University of the Health Sciences Division of Reproductive Endocrinology and Infertility; Program in Reproductive and Adult Endocrinology, Eunice Kennedy Shriver National Institute of Child Health and Human Development, National Institutes of Health, Bethesda, Maryland Ovaries and Development; Reproductive Endocrinology and Infertility Jane A. Cauley, DrPH Professor of Epidemiology, University of Pittsburgh Graduate School of Public Health, Vice Chair of the Department of Epidemiology, Pittsburgh, Pennsylvania Epidemiology of Aging: Implications of the Aging of Society

Naga P. Chalasani, MD David W. Crabb Professor and Director, Division of Gastroenterology and Hepatology, Indiana University School of Medicine, Indianapolis, Indiana Alcoholic and Nonalcoholic Steatohepatitis Henry F. Chambers, MD Professor of Medicine, University of California San Francisco School of Medicine; Director, Clinical Research Services, Clinical and Translational Sciences Institute, San Francisco, California Staphylococcal Infections William P. Cheshire, Jr., MD Professor of Neurology, Mayo Clinic, Jacksonville, Florida Autonomic Disorders and Their Management Ilseung Cho, MD, MS Assistant Professor of Medicine, Division of Gastroenterology, Department of Medicine, New York University, New York, New York Human Microbiome Arun Chockalingam, PhD Professor of Epidemiology and Global Health, Director, Office of Global Health Education and Training; Dalla Lana Faculty of Public Health, University of Toronto, Toronto, Ontario, Canada Global Health David C. Christiani, MD Professor of Medicine, Harvard Medical School; Physician, Pulmonary and Critical Care, Massachusetts General Hospital; Elkan Blout Professor of Environmental Genetics, Environmental Health, Harvard School of Public Health, Boston, Massachusetts Physical and Chemical Injuries of the Lung David H. Chu, MD, PhD Director, Contact Dermatitis, Division of Dermatology and Cutaneous Surgery, Scripps Clinic Medical Group, La Jolla, California Structure and Function of the Skin Theodore J. Cieslak, MD Pediatric Infectious Diseases, Clinical Professor of Pediatrics, University of Texas Health Science Center at San Antonio; Department of Pediatrics, Fort Sam Houston, Texas Bioterrorism Carolyn Clancy, MD Interim Under Secretary for Health, Veterans Administration, Washington, D.C. Measuring Health and Health Care David R. Clemmons, MD Kenan Professor of Medicine, University of North Carolina School of Medicine; Attending Physician, Medicine, UNC Hospitals, Chapel Hill, North Carolina Approach to the Patient with Endocrine Disease David Cohen, MD Professor of Medicine, Division of Nephrology; Medical Director, Kidney and Pancreas Transplantation, Columbia University Medical Center, New York, New York Treatment of Irreversible Renal Failure Jeffrey Cohen, MD Chief, Laboratory of Infectious Diseases, National Institute of Allergy and Infectious Diseases, National Institutes of Health, Bethesda, Maryland Varicella-Zoster Virus (Chickenpox, Shingles)

Contributors

xiii

Myron S. Cohen, MD Associate Vice Chancellor for Global Health, Director, UNC Institute for Global Health and Infectious Diseases, Chief, Division of Infectious Diseases, Yeargan-Bate Eminent Professor of Medicine, Microbiology, and Immunology and Epidemiology, Chapel Hill, North Carolina Approach to the Patient with a Sexually Transmitted Infection; Prevention of Human Immunodeficiency Virus Infection

Mary K. Crow, MD Joseph P. Routh Professor of Rheumatic Diseases in Medicine, Weill Cornell Medical College; Physician in Chief and Benjamin M. Rosen Chair in Immunology and Inflammation Research, Hospital for Special Surgery, New York, New York The Innate Immune Systems; Approach to the Patient with Rheumatic Disease; Systemic Lupus Erythematosus

Steven P. Cohen, MD Professor of Anesthesiology and Critical Care Medicine and Physical Medicine and Rehabilitation, Johns Hopkins School of Medicine, Baltimore, Maryland, and Uniformed Services University of the Health Sciences, Bethesda, Maryland; Director, Pain Research, Walter Reed National Military Medical Center, Bethesda, Maryland Pain

John A. Crump, MB ChB, MD, DTM&H McKinlay Professor of Global Health, Centre for International Health, University of Otago, Dunedin, New Zealand Salmonella Infections (Including Enteric Fever)

Steven L. Cohn, MD Professor of Clinical Medicine, University of Miami Miller School of Medicine; Medical Director, UHealth Preoperative Assessment Center; Director, Medical Consultation Service, University of Miami Hospital, Miami, Florida Preoperative Evaluation Robert Colebunders, MD Emeritus Professor, Institute of Tropical Medicine, Antwerp, Belgium Immune Reconstitution Inflammatory Syndrome in HIV/AIDS Joseph M. Connors, MD Clinical Professor, University of British Columbia; Clinical Director, BC Cancer Agency Centre for Lymphoid Cancer, Vancouver, British Columbia, Canada Hodgkin Lymphoma Deborah J. Cook, MD, MSc Professor of Medicine, Clinical Epidemiology and Biostatistics, McMaster University, Hamilton, Ontario, Canada Approach to the Patient in a Critical Care Setting Kenneth H. Cowan, MD, PhD Director, Fred & Pamela Buffett Cancer Center; Director, The Eppley Institute for Research in Cancer and Allied Diseases; Professor of Medicine, University of Nebraska Medical Center, Omaha, Nebraska Cancer Biology and Genetics Joseph Craft, MD Paul B. Beeson Professor of Medicine and Immunobiology, Section Chief, Rheumatology, Program Director, Investigative Medicine, Department of Internal Medicine, Yale University School of Medicine, New Haven, Connecticut The Adaptive Immune Systems Jill Patricia Crandall, MD Professor of Clinical Medicine, Division of Endocrinology and Diabetes Research Center, Albert Einstein College of Medicine, Bronx, New York Diabetes Mellitus Simon L. Croft, BSc, PhD Professor of Parasitology, Faculty of Infectious and Tropical Diseases, London School of Hygiene and Tropical Medicine, London, United Kingdom Leishmaniasis Kristina Crothers, MD Associate Professor, Department of Medicine, Division of Pulmonary and Critical Care, University of Washington School of Medicine, Seattle, Washington Pulmonary Manifestations of Human Immunodeficiency Virus and Acquired Immunodeficiency Syndrome

Mark R. Cullen, MD Professor of Medicine, Department of Medicine, Stanford University School of Medicine, Stanford, California Principles of Occupational and Environmental Medicine Charlotte Cunningham-Rundles, MD, PhD Professor of Medicine and Pediatrics, Icahn School of Medicine at Mount Sinai, New York, New York Primary Immunodeficiency Diseases Inger K. Damon, MD, PhD Director, Division of High Consequence Pathogens and Pathology, Centers for Disease Control and Prevention, Atlanta, Georgia Smallpox, Monkeypox, and Other Poxvirus Infections Troy E. Daniels, DDS, MS Professor Emeritus of Oral Pathology and Pathology, University of California San Francisco, San Francisco, California Diseases of the Mouth and Salivary Glands Nancy E. Davidson, MD Hillman Professor of Oncology, University of Pittsburgh; Director, University of Pittsburgh Cancer Institute and UPMC CancerCenter, Pittsburgh, Pennsylvania Breast Cancer and Benign Breast Disorders Lisa M. DeAngelis, MD Chair, Department of Neurology, Memorial Sloan-Kettering Cancer Center; Professor of Neurology, Weill Cornell Medical College, New York, New York Tumors of the Central Nervous System Malcolm M. DeCamp, MD Fowler McCormick Professor of Surgery, Feinberg School of Medicine, Northwestern University; Chief, Division of Thoracic Surgery, Northwestern Memorial Hospital, Chicago, Illinois Interventional and Surgical Approaches to Lung Disease Carlos del Rio, MD Hubert Professor and Chair and Professor of Medicine, Hubert Department of Global Health, Rollins School of Public Health and Department of Medicine, Emory University School of Medicine, Atlanta, Georgia Prevention of Human Immunodeficiency Virus Infection Patricia A. Deuster, PhD, MPH Professor and Director, Consortium for Health and Military Performance, Department of Military and Emergency Medicine, Uniformed Services University of the Health Sciences, Bethesda, Maryland Rhabdomyolysis Robert B. Diasio, MD William J. and Charles H. Mayo Professor, Molecular Pharmacology and Experimental Therapeutics and Oncology, Mayo Clinic, Rochester, Minnesota Principles of Drug Therapy

xiv

Contributors

David J. Diemert, MD Associate Professor, Department of Microbiology, Immunology, and Tropical Medicine, School of Medicine and Health Sciences, The George Washington University, Washington, D.C. Intestinal Nematode Infections; Tissue Nematode Infections Kathleen B. Digre, MD Professor of Neurology, Ophthalmology, Director, Division of Headache and Neuro-Ophthalmology, University of Utah, Salt Lake City, Utah Headaches and Other Head Pain James H. Doroshow, MD Bethesda, Maryland Approach to the Patient with Cancer; Malignant Tumors of Bone, Sarcomas, and Other Soft Tissue Neoplasms John M. Douglas, Jr., MD Executive Director, Tri-County Health Department, Greenwood Village, Colorado Papillomavirus Jeffrey M. Drazen, MD Distinguished Parker B. Francis Professor of Medicine, Harvard Medical School; Senior Physician, Brigham and Women’s Hospital, Boston, Massachusetts Asthma Stephen C. Dreskin, MD, PhD Professor of Medicine and Immunology, Division of Allergy and Clinical Immunology, Department of Medicine, University of Colorado Denver, School of Medicine, Aurora, Colorado Urticaria and Angioedema W. Lawrence Drew, MD, PhD Professor Emeritus, Laboratory Medicine and Medicine, University of California San Francisco, San Francisco, California Cytomegalovirus George L. Drusano, MD Professor and Director, Institute for Therapeutic Innovation, College of Medicine, University of Florida, Lake Nona, Florida Antibacterial Chemotherapy Thomas D. DuBose, Jr., MD Emeritus Professor of Internal Medicine and Nephrology, Wake Forest School of Medicine, Winston-Salem, North Carolina Vascular Disorders of the Kidney F. Daniel Duffy, MD Professor of Internal Medicine and Steve Landgarten Chair in Medical Leadership, School of Community Medicine, University of Oklahoma College of Medicine, Tulsa, Oklahoma Counseling for Behavior Change Herbert L. DuPont, MD, MACP Mary W. Kelsey Chair and Director, Center for Infectious Diseases, University of Texas School of Public Health; H. Irving Schweppe Chair of Internal Medicine and Vice Chairman, Department of Medicine, Baylor College of Medicine; Chief of Internal Medicine, St. Luke’s Hospital System, Houston, Texas Approach to the Patient with Suspected Enteric Infection Madeleine Duvic, MD Professor and Deputy Chairman, Department of Dermatology, The University of Texas MD Anderson Cancer Center, Houston, Texas Urticaria, Drug Hypersensitivity Rashes, Nodules and Tumors, and Atrophic Diseases

Kathryn M. Edwards, MD Sarah H. Sell and Cornelius Vanderbilt Chair in Pediatrics, Vanderbilt University School of Medicine; Director, Vanderbilt Vaccine Research Program, Monroe Carrell Jr. Children’s Hospital at Vanderbilt, Nashville, Tennessee Parainfluenza Viral Disease N. Lawrence Edwards, MD Professor of Medicine, Vice Chairman, Department of Medicine, University of Florida; Chief, Section of Rheumatology, Medical Service, Malcom Randall Veterans Affairs Medical Center, Gainesville, Florida Crystal Deposition Diseases Lawrence H. Einhorn, MD Distinguished Professor, Department of Medicine, Division of Hematology/Oncology, Livestrong Foundation Professor of Oncology, Indiana University School of Medicine, Indianapolis, Indiana Testicular Cancer Ronald J. Elin, MD, PhD A.J. Miller Professor and Chairman, Department of Pathology and Laboratory Medicine, University of Louisville School of Medicine, Louisville, Kentucky Reference Intervals and Laboratory Values George M. Eliopoulos, MD Professor of Medicine, Harvard Medical School; Physician, Division of Infectious Diseases, Beth Israel Deaconess Medical Center, Boston, Massachusetts Principles of Anti-Infective Therapy Perry Elliott, MD Professor in Inherited Cardiovascular Disease, Institute of Cardiovascular Science, University College London, London, United Kingdom Diseases of the Myocardium and Endocardium Jerrold J. Ellner, MD Professor of Medicine, Boston University School of Medicine; Chief, Section of Infectious Diseases, Boston Medical Center, Boston, Massachusetts Tuberculosis Dirk M. Elston, MD Director, Ackerman Academy of Dermatopathology, New York, New York Arthropods and Leeches Ezekiel J. Emanuel, MD, PhD Vice Provost for Global Initiatives, Diane V.S. Levy and Robert M. Levy University Professor, Chair, Department of Medical Ethics and Health Policy, University of Pennsylvania, Philadelphia, Pennsylvania Bioethics in the Practice of Medicine Joel D. Ernst, MD Director, Division of Infectious Diseases and Immunology, Jeffrey Bergstein Professor of Medicine, Professor of Medicine, Pathology, and Microbiology, New York University School of Medicine; Attending Physician, New York University Langone Medical Center, New York, New York Leprosy (Hansen Disease) Gregory T. Everson, MD Professor of Medicine, Director of Hepatology, University of Colorado School of Medicine, Aurora, Colorado Hepatic Failure and Liver Transplantation Amelia Evoli, MD Associate Professor of Neurology, Catholic University, Agostino Gemelli University Hospital, Rome, Italy Disorders of Neuromuscular Transmission

Contributors Douglas O. Faigel, MD Professor of Medicine, Mayo Clinic, Chair, Division of Gastroenterology and Hepatology, Scottsdale, Arizona Neoplasms of the Small and Large Intestine

Manuel A. Franco, MD, PhD Director of Postgraduate Programs, School of Sciences, Pontificia Universidad Javeriana, Bogota, Colombia Rotaviruses, Noroviruses, and Other Gastrointestinal Viruses

Matthew E. Falagas, MD, MSc, DSc Director, Alfa Institute of Biomedical Sciences, Athens, Greece; Adjunct Associate Professor of Medicine, Tufts University School of Medicine, Boston, Massachusetts; Chief, Department of Medicine and Infectious Diseases, Iaso General Hospital, Iaso Group, Athens, Greece Pseudomonas and Related Gram-Negative Bacillary Infections

David O. Freedman, MD Professor of Medicine and Microbiology, University of Alabama at Birmingham; Director, Gorgas Center for Geographic Medicine, Birmingham, Alabama Approach to the Patient before and after Travel

Gary W. Falk, MD, MS Professor of Medicine, Division of Gastroenterology, University of Pennsylvania Perelman School of Medicine, Philadelphia, Pennsylvania Diseases of the Esophagus Gene Feder, MBBS, MD Professor, Centre for Academic Primary Care, School of Social and Community Medicine, University of Bristol; General Practitioner, Helios Medical Centre, Bristol, United Kingdom Intimate Partner Violence David J. Feller-Kopman, MD Director, Bronchoscopy and Interventional Pulmonology, Associate Professor of Medicine, The Johns Hopkins University, Baltimore, Maryland Interventional and Surgical Approaches to Lung Disease Gary S. Firestein, MD Dean and Associate Vice Chancellor of Translational Medicine, University of California San Diego School of Medicine, La Jolla, California Mechanisms of Inflammation and Tissue Repair Glenn I. Fishman, MD Director, Leon H. Charney Division of Cardiology, Vice-Chair for Research, Department of Medicine, William Goldring Professor of Medicine, New York University School of Medicine, New York, New York Principles of Electrophysiology Lee A. Fleisher, MD Robert D. Dripps Professor and Chair, Anesthesiology and Critical Care, Professor of Medicine, University of Pennsylvania Perelman School of Medicine, Philadelphia, Pennsylvania Overview of Anesthesia Paul W. Flint, MD Professor and Chair, Otolaryngology, Head and Neck Surgery, Oregon Health and Science University, Portland, Oregon Throat Disorders Evan L. Fogel, MD, MSc Professor of Clinical Medicine, Indiana University School of Medicine, Indianapolis, Indiana Diseases of the Gallbladder and Bile Ducts Marsha D. Ford, MD Adjunct Professor of Emergency Medicine, School of Medicine, University of North Carolina-Chapel Hill; Director, Carolinas Poison Center, Carolinas HealthCare System, Charlotte, North Carolina Acute Poisoning Chris E. Forsmark, MD Professor of Medicine, Chief, Division of Gastroenterology, Hepatology, and Nutrition, University of Florida, Gainesville, Florida Pancreatitis Vance G. Fowler, Jr., MD, MHS Professor of Medicine, Duke University Medical Center, Durham, North Carolina Infective Endocarditis

xv

Martyn A. French, MD Professor in Clinical Immunology, School of Pathology and Laboratory Medicine, University of Western Australia, Perth, Australia Immune Reconstitution Inflammatory Syndrome in HIV/AIDS Karen Freund, MD, MPH Professor of Medicine, Associate Director, Tufts Clinical and Translational Science Institute, Tufts University School of Medicine, Tufts Medical Center, Boston, Massachusetts Approach to Women’s Health Cem Gabay, MD Professor of Medicine, Head, Division of Rheumatology, University Hospitals of Geneva, Geneva, Switzerland Biologic Agents Kenneth L. Gage, PhD Chief, Entomology and Ecology Activity, Centers for Disease Control and Prevention, Division of Vector-Borne Diseases, Bacterial Diseases Branch, Fort Collins, Colorado Plague and Other Yersinia Infections John N. Galgiani, MD Professor of Medicine, Valley Fever Center for Excellence, University of Arizona, Tucson, Arizona Coccidioidomycosis Patrick G. Gallagher, MD Professor of Pediatrics, Pathology, and Genetics, Yale University School of Medicine; Attending Physician, Yale–New Haven Hospital, New Haven, Connecticut Hemolytic Anemias: Red Blood Cell Membrane and Metabolic Defects Leonard Ganz, MD Director of Cardiac Electrophysiology, Heritage Valley Health System, Beaver, Pennsylvania Electrocardiography Hasan Garan, MD Director, Cardiac Electrophysiology, Dickinson W. Richards, Jr. Professor of Medicine, Columbia University Medical Center, New York, New York Ventricular Arrhythmias Guadalupe Garcia-Tsao, MD Professor of Medicine, Yale University School of Medicine; Chief, Digestive Diseases, VA Connecticut Healthcare System, West Haven, Connecticut Cirrhosis and Its Sequelae William M. Geisler, MD, MPH Professor of Medicine, University of Alabama at Birmingham, Birmingham, Alabama Diseases Caused by Chlamydiae Tony P. George, MD Division of Brain and Therapeutics, Department of Psychiatry, University of Toronto; Schizophrenia Division, The Centre for Addiction and Mental Health, Toronto, Ontario, Canada Nicotine and Tobacco

xvi

Contributors

Lior Gepstein, MD, PhD Edna and Jonathan Sohnis Professor in Medicine and Physiology, Rappaport Faculty of Medicine and Research Institute, Technion–Israel Institute of Technology, Rambam Health Care Campus, Haifa, Israel Gene and Cell Therapy

Larry B. Goldstein, MD Professor of Neurology, Director, Duke Stroke Center, Neurology, Duke University; Staff Neurologist, Durham VA Medical Center, Durham, North Carolina Approach to Cerebrovascular Diseases; Ischemic Cerebrovascular Disease

Susan I. Gerber, MD Team Lead, Respiratory Viruses/Picornaviruses, Division of Viral Diseases/Epidemiology Branch, National Center for Immunization and Respiratory Diseases, Centers for Disease Control and Prevention, Atlanta, Georgia Coronaviruses

Lawrence T. Goodnough, MD Professor of Pathology and Medicine, Stanford University; Director, Transfusion Service, Stanford University Medical Center, Stanford, California Transfusion Medicine

Dale N. Gerding, MD Professor of Medicine, Loyola University Chicago Stritch School of Medicine, Research Physician, Edward Hines, Jr. VA Hospital, Hines, Illinois Clostridial Infections Morie A. Gertz, MD Consultant, Division of Hematology, Mayo Clinic, Rochester, Minnesota; Roland Seidler, Jr. Professor of the Art of Medicine in Honor of Michael D. Brennan, MD, Professor of Medicine, Mayo Clinic, College of Medicine, Rochester, Minnesota Amyloidosis Gordon D. Ginder, MD Professor, Internal Medicine, Director, Massey Cancer Center, Virginia Commonwealth University, Richmond, Virginia Microcytic and Hypochromic Anemias Jeffrey S. Ginsberg, MD Professor of Medicine, McMaster University, Member of Thrombosis and Atherosclerosis Research Institute, St. Joseph’s Healthcare Hamilton, Hamilton, Ontario, Canada Peripheral Venous Disease Geoffrey S. Ginsburg, MD, PhD Director, Duke Center for Applied Genomics and Precision Medicine; Professor of Medicine, Pathology and Biomedical Engineering, Duke University, Durham, North Carolina Applications of Molecular Technologies to Clinical Medicine Michael Glogauer, DDS, PhD Professor, Faculty of Dentistry, University of Toronto, Toronto, Ontario, Canada Disorders of Phagocyte Function John W. Gnann, Jr., MD Professor of Medicine, Department of Medicine, Division of Infectious Diseases, Medical University of South Carolina, Charleston, South Carolina Mumps Matthew R. Golden, MD, MPH Professor of Medicine, University of Washington, Director, HIV/STD Program, Public Health–Seattle & King County, Seattle, Washington Neisseria Gonorrhoeae Infections Lee Goldman, MD Harold and Margaret Hatch Professor, Executive Vice President and Dean of the Faculties of Health Sciences and Medicine, Chief Executive, Columbia University Medical Center, Columbia University, New York, New York Approach to Medicine, the Patient, and the Medical Profession: Medicine as a Learned and Humane Profession; Approach to the Patient with Possible Cardiovascular Disease Ellie J.C. Goldstein, MD Clinical Professor of Medicine, David Geffen School of Medicine at University of California Los Angeles, Los Angeles, California; Director, R.M. Alden Research Laboratory, Santa Monica, California Diseases Caused by Non–Spore-Forming Anaerobic Bacteria

Eduardo H. Gotuzzo, MD Professor of Medicine, Director, Alexander von Humboldt Tropical Medicine Institute, Universidad Peruana Cayetano Heredia; Chief Physician, Department of Infectious, Tropical, and Dermatologic Diseases, National Hospital Cayetano Heredia, Lima, Peru Cholera and Other Vibrio Infections; Liver, Intestinal, and Lung Fluke Infections Deborah Grady, MD, MPH Professor of Medicine, University of California San Francisco, San Francisco, California Menopause Leslie C. Grammer, MD Professor of Medicine, Northwestern University Feinberg School of Medicine; Attending Physician, Northwestern Memorial Hospital, Chicago, Illinois Drug Allergy F. Anthony Greco, MD Medical Director, Sarah Cannon Cancer Center, Nashville, Tennessee Cancer of Unknown Primary Origin Harry B. Greenberg, MD Professor, Departments of Medicine and Microbiology and Immunology, Stanford University School of Medicine, Stanford, California Rotaviruses, Noroviruses, and Other Gastrointestinal Viruses Steven A. Greenberg, MD Associate Professor of Neurology, Harvard Medical School; Associate Neurologist, Brigham and Women’s Hospital, Boston, Massachusetts Inflammatory Myopathies Robert C. Griggs, MD Professor of Neurology, Medicine, Pediatrics, and Pathology and Laboratory Medicine, University of Rochester School of Medicine and Dentistry, Rochester, New York Approach to the Patient with Neurologic Disease Lev M. Grinberg, MD, PhD Professor, Chief, Department of Pathology, Ural Medical University; Chief Researcher of the Ural Scientific Research Institute of Phthisiopulmonology, Chief Pathologist of Ekaterinburg, Ekaterinburg, Russia Anthrax Daniel Grossman, MD Vice President for Research, Ibis Reproductive Health, Oakland, California; Assistant Clinical Professor, Bixby Center for Global Reproductive Health, Department of Obstetrics, Gynecology and Reproductive Sciences, University of California San Francisco, San Francisco, California Contraception Lisa M. Guay-Woodford, MD Hudson Professor of Pediatrics, The George Washington University; Director, Center for Translational Science, Director, Clinical and Translational Institute at Children’s National, Children’s National Health System, Washington, D.C. Hereditary Nephropathies and Developmental Abnormalities of the Urinary Tract

Contributors Richard L. Guerrant, MD Thomas H. Hunter Professor of International Medicine, Founding Director, Center for Global Health, Division of Infectious Diseases and International Health, University of Virginia School of Medicine, University of Virginia Health Sciences Center, Charlottesville, Virginia Cryptosporidiosis Roy M. Gulick, MD, MPH Gladys and Roland Harrison Professor of Medicine, Medicine/Infectious Diseases, Weill Cornell Medical College; Attending Physician, New York– Presbyterian Hospital, New York, New York Antiretrovial Therapy of HIV/AIDS Klaus D. Hagspiel, MD Professor of Radiology, Medicine, and Pediatrics, Chief, Noninvasive Cardiovascular Imaging, University of Virginia Health System, Charlottesville, Virginia Noninvasive Cardiac Imaging John D. Hainsworth, MD Chief Scientific Officer, Sarah Cannon Research Institute, Nashville, Tennessee Cancer of Unknown Primary Origin Anders Hamsten, MD, PhD Professor of Cardiovascular Diseases, Center for Molecular Medicine and Department of Cardiology, Karolinska University Hospital, Department of Medicine, Karolinska Institute, Stockholm, Sweden Atherosclerosis, Thrombosis, and Vascular Biology Kenneth R. Hande, MD Professor of Medicine and Pharmacology, Vanderbilt/Ingram Cancer Center, Vanderbilt University School of Medicine, Nashville, Tennessee Neuroendocrine Tumors and the Carcinoid Syndrome H. Hunter Handsfield, MD Professor Emeritus of Medicine, University of Washington Center for AIDS and STD, Seattle, Washington Neisseria Gonorrhoeae Infections Göran K. Hansson, MD, PhD Professor of Cardiovascular Research, Center for Molecular Medicine at Karolinska University Hospital, Department of Medicine, Karolinska Institute, Stockholm, Sweden Atherosclerosis, Thrombosis, and Vascular Biology Raymond C. Harris, MD Professor of Medicine, Ann and Roscoe R. Robinson Chair in Nephrology, Chief, Division of Nephrology, Vanderbilt University School of Medicine, Nashville, Tennessee Diabetes and the Kidney Stephen Crane Hauser, MD Associate Professor of Medicine, Internal Medicine, Division of Gastroenterology and Hepatology, Mayo Clinic College of Medicine, Rochester, Minnesota Vascular Diseases of the Gastrointestinal Tract Frederick G. Hayden, MD Stuart S. Richardson Professor of Clinical Virology and Professor of Medicine, University of Virginia School of Medicine; Staff Physician, University of Virginia Health System, Charlottesville, Virginia Influenza Douglas C. Heimburger, MD, MS Professor of Medicine, Associate Director for Education and Training, Vanderbilt University School of Medicine, Vanderbilt Institute for Global Health, Nashville, Tennessee Nutrition’s Interface with Health and Disease

xvii

Erik L. Hewlett, MD Professor of Medicine and of Microbiology, Immunology, and Cancer Biology, University of Virginia School of Medicine, University of Virginia Health System, Charlottesville, Virginia Whooping Cough and Other Bordetella Infections Richard J. Hift, PhD, MMed School of Clinical Medicine, University of KwaZulu-Natal, Durban, South Africa The Porphyrias David R. Hill, MD, DTM&H Professor of Medical Sciences, Director of Global Public Health, Frank H. Netter MD School of Medicine at Quinnipiac University, Hamden, Connecticut Giardiasis Nicholas S. Hill, MD Professor of Medicine, Tufts University School of Medicine; Chief, Division of Pulmonary, Critical Care, and Sleep Medicine, Tufts Medical Center, Boston, Massachusetts Respiratory Monitoring in Critical Care L. David Hillis, MD Professor and Chair, Department of Medicine, University of Texas Health Science Center at San Antonio, San Antonio, Texas Acute Coronary Syndrome: Unstable Angina and Non-ST Elevation Myocardial Infarction Jack Hirsh, CM, MD, DSc Professor Emeritus, McMaster University, Hamilton, Ontario, Canada Antithrombotic Therapy Steven M. Holland, MD Chief, Laboratory of Clinical Infectious Diseases, National Institute of Allergy and Infectious Diseases, National Institutes of Health, Bethesda, Maryland The Nontuberculous Mycobacteria Steven M. Hollenberg, MD Professor of Medicine, Cooper Medical School of Rowan University; Director, Coronary Care Unit, Cooper University Hospital, Camden, New Jersey Cardiogenic Shock Edward W. Hook III, MD Professor and Director, Division of Infectious Diseases, University of Alabama at Birmingham, Birmingham, Alabama Granuloma Inguinale (Donovanosis); Syphilis; Nonsyphilitic Treponematoses David J. Hunter, MBBS, MPH, ScD Vincent L. Gregory Professor of Cancer Prevention, Harvard School of Public Health; Professor of Medicine, Harvard Medical School, Brigham and Women’s Hospital, Boston, Massachusetts The Epidemiology of Cancer Khalid Hussain, MBChB, MD, MSc Developmental Endocrinology Research Group, Clinical and Molecular Genetics Unit, Institute of Child Health, University College London, Department of Paediatric Endocrinology, Great Ormond Street Hospital for Children, London, United Kingdom Hypoglycemia/Pancreatic Islet Cell Disorders Steven E. Hyman, MD Director, Stanley Center for Psychiatric Research, Broad Institute, Distinguished Service Professor of Stem Cell and Regenerative Biology, Harvard University, Cambridge, Massachusetts Biology of Addiction

xviii

Contributors

Michael C. Iannuzzi, MD, MBA Chairman, Department of Internal Medicine, State University of New York Upstate Medical University, Syracuse, New York Sarcoidosis

Richard C. Jordan, DDS, PhD Professor of Oral Pathology, Pathology and Radiation Oncology, University of California San Francisco, San Francisco, California Diseases of the Mouth and Salivary Glands

Robert D. Inman, MD Professor of Medicine and Immunology, University of Toronto; Staff Rheumatologist, University Health Network, Toronto, Ontario, Canada The Spondyloarthropathies

Ralph F. Józefowicz, MD Professor, Neurology and Medicine, University of Rochester, Rochester, New York Approach to the Patient with Neurologic Disease

Sharon K. Inouye, MD, MPH Professor of Medicine, Harvard Medical School; Director, Aging Brain Center, Institute for Aging Research, Hebrew SeniorLife, Boston, Massachusetts Neuropsychiatric Aspects of Aging; Delirium or Acute Mental Status Change in the Older Patient

Stephen G. Kaler, MD Senior Investigator and Head, Section on Translational Neuroscience, Molecular Medicine Program, Eunice Kennedy Shriver National Institute of Child Health and Human Development, Bethesda, Maryland Wilson Disease

Geoffrey K. Isbister, MD, BSc Associate Professor, Clinical Toxicologist, Calvary Mater Newcastle, Callaghan, Senior Research Academic, School of Medicine and Public Health, University of Newcastle, New South Wales, Australia Envenomation Michael G. Ison, MD, MS Associate Professor in Medicine-Infectious Diseases and Surgery-Organ Transplantation, Northwestern University Feinberg School of Medicine, Chicago, Illinois Adenovirus Diseases Elias Jabbour, MD Associate Professor, Department of Leukemia, Division of Medicine, The University of Texas MD Anderson Cancer Center, Houston, Texas The Chronic Leukemias Michael R. Jaff, DO Professor of Medicine, Harvard Medical School, Chair, Institute for Heart, Vascular, and Stroke Care, Massachusetts General Hospital, Boston, Massachusetts Other Peripheral Arterial Diseases Joanna C. Jen, MD, PhD Professor of Neurology, University of California Los Angeles School of Medicine, Los Angeles, California Neuro-Ophthalmology; Smell and Taste; Hearing and Equilibrium Dennis M. Jensen, MD Professor of Medicine, David Geffen School of Medicine at University of California Los Angeles; Staff Physician, Medicine-GI, VA Greater Los Angeles Healthcare System; Key Investigator, Director, Human Studies Core & GI Hemostasis Research Unit, CURE Digestive Diseases Research Center, Los Angeles, California Gastrointestinal Hemorrhage Michael D. Jensen, MD Professor of Medicine, Endocrine Research Unit, Director, Obesity Treatment Research Program, Mayo Clinic, Rochester, Minnesota Obesity Robert T. Jensen, MD Chief, Cell Biology Section, Digestive Disease Branch, National Institute of Diabetes and Digestive and Kidney Diseases, National Institutes of Health, Clinical Center, Bethesda, Maryland Pancreatic Neuroendocrine Tumors Stuart Johnson, MD Professor of Medicine, Loyola University Chicago Stritch School of Medicine; Associate Chief of Staff for Research, Edward Hines, Jr. VA Hospital, Hines, Illinois Clostridial Infections

Moses R. Kamya, MB ChB, MMed, MPH, PhD Chairman, Department of Medicine, Makerere University College of Health Sciences, Kampala, Uganda Malaria Louise W. Kao, MD Associate Professor of Emergency Medicine, Department of Emergency Medicine, Indiana University School of Medicine, Indianapolis, Indiana Chronic Poisoning: Trace Metals and Others Steven A. Kaplan, MD E. Darracott Vaughan, Jr. Professor of Urology, Chief, Institute for Bladder and Prostate Health, Weill Cornell Medical College; Director, Iris Cantor Men’s Health Center, NewYork–Presbyterian Hospital, New York, New York Benign Prostatic Hyperplasia and Prostatitis Daniel L. Kastner, MD, PhD Scientific Director, National Human Genome Research Institute, National Institutes of Health, Bethesda, Maryland The Systemic Autoinflammatory Diseases Sekar Kathiresan, MD Associate Professor in Medicine, Harvard Medical School; Director, Preventive Cardiology, Massachusetts General Hospital, Boston, Massachusetts The Inherited Basis of Common Diseases David A. Katzka, MD Professor of and Consultant in Medicine, Gastroenterology, Mayo Clinic, Rochester, Minnesota Diseases of the Esophagus Debra K. Katzman, MD Professor of Pediatrics, Senior Associate Scientist, The Research Institute, The Hospital for Sick Children and University of Toronto, Toronto, Ontario, Canada Adolescent Medicine Carol A. Kauffman, MD Professor of Internal Medicine, University of Michigan Medical School; Chief, Infectious Diseases Section, Veterans Affairs Ann Arbor Healthcare System, Ann Arbor, Michigan Histoplasmosis; Blastomycosis; Paracoccidioidomycosis; Cryptococcosis; Sporotrichosis; Candidiasis Kenneth Kaushansky, MD Senior Vice President for Health Sciences, Dean, School of Medicine, Stony Brook University, Stony Brook, New York Hematopoiesis and Hematopoietic Growth Factors Keith S. Kaye, MD, MPH Professor of Medicine, Division of Infectious Diseases, Wayne State University School of Medicine, Detroit, Michigan Diseases Caused by Acinetobacter and Stenotrophomonas Species

Contributors Armand Keating, MD Professor of Medicine, Director, Division of Hematology, Epstein Chair in Cell Therapy and Transplantation, Professor, Institute of Biomaterials and Biomedical Engineering, University of Toronto, Toronto, Ontario, Canada Hematopoietic Stem Cell Transplantation Robin K. Kelley, MD Assistant Professor of Medicine, University of California San Francisco, Helen Diller Family Comprehensive Cancer Center, San Francisco, California Liver and Biliary Tract Cancers Morton Kern, MD Chief of Medicine, VA Long Beach Health Care System School of Medicine; Professor of Medicine, Associate Chief, Cardiology, University of California–Irvine, Irvine, California Catheterization and Angiography Gerald T. Keusch, MD Professor of Medicine and International Health and Public Health, Boston University School of Medicine, Boston, Massachusetts Shigellosis Fadlo R. Khuri, MD Professor and Chair, Hematology and Medical Oncology, Deputy Director, Winship Cancer Institute, Emory University, Atlanta, Georgia Lung Cancer and Other Pulmonary Neoplasms David H. Kim, MD Vice Chair of Education, Professor of Radiology, Section of Abdominal Imaging, University of Wisconsin School of Medicine and Public Health, Madison, Wisconsin Diagnostic Imaging Procedures in Gastroenterology

xix

Kevin M. Korenblat, MD Associate Professor of Medicine, Department of Medicine, Washington University School of Medicine, St. Louis, Missouri Approach to the Patient with Jaundice or Abnormal Liver Tests Bruce R. Korf, MD, PhD Wayne H. and Sara Crews Finley Chair in Medical Genetics, Professor and Chair, Department of Genetics, University of Alabama at Birmingham, Birmingham, Alabama Principles of Genetics Neil J. Korman, MD, PhD Professor, Dermatology, Case Western Reserve University School of Medicine, University Hospitals Case Medical Center, Cleveland, Ohio Macular, Papular, Vesiculobullous, and Pustular Diseases Mark G. Kortepeter, MD, MPH Associate Dean for Research, Associate Professor of Preventive Medicine and Medicine, Consultant to the Army Surgeon General for Biodefense; Office of the Dean, Edward Hébert School of Medicine, Uniformed Services University of the Health Sciences, Bethesda, Maryland Bioterrorism Joseph A. Kovacs, MD Senior Investigator and Head, AIDS Section, Critical Care Medicine Department, National Institutes of Health, Bethesda, Maryland Pneumocystis Pneumonia Thomas O. Kovacs, MD Professor of Medicine, Division of Digestive Diseases, David Geffen School of Medicine at University of California Los Angeles, Los Angeles, California Gastrointestinal Hemorrhage

Matthew Kim, MD Instructor of Medicine, Harvard Medical School; Associate Physician, Brigham and Women’s Hospital, Boston, Massachusetts Thyroid

Monica Kraft, MD Professor of Medicine, Duke University School of Medicine; Chief, Division of Pulmonary, Allergy, and Critical Care Medicine, Duke University Medical Center, Durham, North Carolina Approach to the Patient with Respiratory Disease

Louis V. Kirchhoff, MD, MPH Professor, Departments of Internal Medicine (Infectious Diseases) and Epidemiology, University of Iowa Health Care; Staff Physician, Medical Service, Department of Veterans Affairs Medical Center, Iowa City, Iowa Chagas Disease

Christopher M. Kramer, MD Ruth C. Heede Professor of Cardiology, Professor of Radiology, Director, Cardiovascular Imaging Center, University of Virginia Health System, Charlottesville, Virginia Noninvasive Cardiac Imaging

David S. Knopman, MD Professor of Neurology, Mayo Clinic College of Medicine, Rochester, Minnesota Regional Cerebral Dysfunction: Higher Mental Function; Alzheimer Disease and Other Dementias

Donna M. Krasnewich, MD, PhD Program Director, National Institute of General Medical Sciences, National Institutes of Health, Bethesda, Maryland The Lysosomal Storage Diseases

Tamsin A. Knox, MD, MPH Associate Professor of Medicine, Nutrition/Infection Unit, Tufts University School of Medicine, Boston, Massachusetts Gastrointestinal Manifestions of HIV and AIDS D.P. Kontoyiannis, MD, ScD Professor, Department of Infectious Diseases, Infection Control and Employee Health, The University of Texas MD Anderson Cancer Center, Houston, Texas Mucormycosis; Mycetoma Barbara S. Koppel, MD Professor of Clinical Neurology, New York Medical College, Chief of Neurology, Metropolitan Hospital Center, New York City Health and Hospital Corporation, New York, New York Nutritional and Alcohol-Related Neurologic Disorders

Peter J. Krause, MD Senior Research Scientist in Epidemiology, Medicine, and Pediatrics, Yale School of Public Health and Yale School of Medicine, New Haven, Connecticut Babesiosis and Other Protozoan Diseases John F. Kuemmerle, MD Chair, Division of Gastroenterology, Hepatology, and Nutrition, Professor of Medicine, and Physiology and Biophysics, Center for Digestive Health, Virginia Commonwealth University, Richmond, Virginia Inflammatory and Anatomic Diseases of the Intestine, Peritoneum, Mesentery, and Omentum Ernst J. Kuipers, MD, PhD Professor of Medicine, Department of Gastroenterology and Hepatology, Chief Executive Officer, Erasmus MC University Medical Center, Rotterdam, The Netherlands Acid Peptic Disease

xx

Contributors

Paul W. Ladenson, MD Professor of Medicine, Pathology, Oncology, and Radiology and Radiological Sciences, John Eager Howard Professor of Endocrinology and Metabolism, University Distinguished Service Professor, The Johns Hopkins University School of Medicine; Physician and Division Director, The Johns Hopkins Hospital, Baltimore, Maryland Thyroid Daniel Laheru, MD Ian T. MacMillan Professorship in Clinical Pancreatic Research, Medical Oncology, Johns Hopkins University School of Medicine, Baltimore, Maryland Pancreatic Cancer Donald W. Landry, MD, PhD Samuel Bard Professor of Medicine, Chair, Department of Medicine, Physician-in-Chief, NewYork-Presbyterian Hospital/Columbia University Medical Center, New York, New York Approach to the Patient with Renal Disease Anthony E. Lang, MD Director, Division of Neurology, Jack Clark Chair for Research in Parkinson’s Disease, University of Toronto; Director, Morton and Gloria Shulman Movement Disorders Clinic and the Edmond J. Safra Program in Parkinson’s Disease and the Lily Safra Chair in Movement Disorders, Toronto Western Hospital, Toronto, Ontario, Canada Parkinsonism; Other Movement Disorders

Gary R. Lichtenstein, MD Professor of Medicine, Perelman School of Medicine at the University of Pennsylvania, Director, Center for Inflammatory Bowel Disease, University of Pennsylvania, Philadelphia, Pennsylvania Inflammatory Bowel Disease Henry W. Lim, MD Chairman and C.S. Livingood Chair, Department of Dermatology, Henry Ford Hospital; Senior Vice President for Academic Affairs, Henry Ford Health System, Detroit, Michigan Eczemas, Photodermatoses, Papulosquamous (Including Fungal) Diseases, and Figurate Erythemas Aldo A.M. Lima, MD, PhD Professor of Medicine and Pharmacology, School of Medicine, Federal University of Ceará, Fortaleza, Ceará, Brazil Cryptosporidiosis; Amebiasis Geoffrey S.F. Ling, MD, PhD Professor of Neurology, Uniformed Services University of the Health Sciences, Bethesda, Maryland Traumatic Brain Injury and Spinal Cord Injury William C. Little, MD Patrick Lehan Professor of Cardiovascular Medicine, Chair, Department of Medicine, University of Mississippi Medical Center, Jackson, Mississippi Pericardial Diseases

Richard A. Lange, MD, MBA President and Dean, Paul L. Foster School of Medicine, Texas Tech University Health Sciences Center El Paso, El Paso, Texas Acute Coronary Syndrome: Unstable Angina and Non-ST Elevation Myocardial Infarction

Donald M. Lloyd-Jones, MD, ScM Senior Associate Dean, Chair, Department of Preventive Medicine, Eileen M. Foell Professor of Preventive Medicine and Medicine, Northwestern University Feinberg School of Medicine, Chicago, Illinois Epidemiology of Cardiovascular Disease

Frank A. Lederle, MD Core Investigator, Center for Chronic Disease Outcomes Research, Minneapolis VA Medical Center; Professor of Medicine, University of Minnesota School of Medicine, Minneapolis, Minnesota Diseases of the Aorta

Bennett Lorber, MD Thomas M. Durant Professor of Medicine and Professor of Microbiology and Immunology, Temple University School of Medicine, Philadelphia, Pennsylvania Listeriosis

Thomas H. Lee, MD, MSc Senior Physician, Department of Medicine, Brigham and Women’s Hospital; Chief Medical Officer, Press Ganey, Boston, Massachusetts Using Data for Clinical Decisions

Donald E. Low, MD† Nonpneumococcal Streptococcal Infections, Rheumatic Fever

William M. Lee, MD Professor of Internal Medicine, University of Texas Southwestern Medical Center, Dallas, Texas Toxin- and Drug-Induced Liver Disease James E. Leggett, MD Associate Professor, Department of Medicine, Oregon Health and Science University; Infectious Diseases, Department of Medical Education, Providence Portland Medical Center, Portland, Oregon Approach to Fever or Suspected Infection in the Normal Host Stuart Levin, MD Professor of Medicine, Emeritus Chairman, Department of Medicine, Rush University Medical Center, Chicago, Illinois Zoonoses Stephanie M. Levine, MD Professor of Medicine, Division of Pulmonary Diseases and Critical Care Medicine, The University of Texas Health Science Center San Antonio, South Texas Veterans Health Care System, San Antonio, Texas Alveolar Filling Disorders

Daniel R. Lucey, MD, MPH Adjunct Professor, Microbiology and Immunology, Georgetown University Medical Center, Washington, D.C. Anthrax James R. Lupski, MD, PhD Cullen Professor of Molecular and Human Genetics, Professor of Pediatrics, Baylor College of Medicine and Texas Children’s Hospital, Houston, Texas Gene, Genomic, and Chromosomal Disorders Jeffrey M. Lyness, MD Senior Associate Dean for Academic Affairs, Professor of Psychiatry and Neurology, University of Rochester School of Medicine and Dentistry, Rochester, New York Psychiatric Disorders in Medical Practice Bruce W. Lytle, MD Chair, Heart and Vascular Institute, Professor of Surgery, Thoracic and Cardiovascular Surgery, Cleveland Clinic, Cleveland, Ohio Interventional and Surgical Treatment of Coronary Artery Disease



Deceased.

Contributors C. Ronald MacKenzie, MD Assistant Attending Physician, Department of Medicine-Rheumatology, C. Ronald MacKenzie Chair in Ethics and Medicine, Hospital for Special Surgery, Associate Professor of Clinical Medicine and Medical Ethics, Weill Cornell Medical College of Cornell University, New York, New York Surgical Treatment of Joint Disease Harriet L. MacMillan, MD, MSc Professor, Departments of Psychiatry and Behavioural Neurosciences, and Pediatrics, Chedoke Health Chair in Child Psychiatry, Offord Centre for Child Studies, McMaster University, Hamilton, Ontario, Canada Intimate Partner Violence Robert D. Madoff, MD Professor of Surgery, Stanley M. Goldberg, MD, Chair, Colon and Rectal Surgery, University of Minnesota, Minneapolis, Minnesota Diseases of the Rectum and Anus Frank Maldarelli, MD, PhD Head, Clinical Retrovirology Section, HIV Drug Resistance Program, National Cancer Institute, National Institutes of Health, Bethesda, Maryland Biology of Human Immunodeficiency Viruses Atul Malhotra, MD Chief of Pulmonary and Critical Care, Kenneth M. Moser Professor of Medicine, Director of Sleep Medicine, University of California San Diego, La Jolla, California Disorders of Ventilatory Control Mark J. Manary, MD Helene B. Roberson Professor of Pediatrics, Washington University School of Medicine; Attending Physician, St. Louis Children’s Hospital, St. Louis, Missouri; Adjunct Professor, Children’s Nutrition Research Center, Baylor College of Medicine, Houston, Texas; Senior Lecturer in Community Health, University of Malawi College of Medicine, Blantyre, Malawi Protein-Energy Malnutrition Donna Mancini, MD Professor of Medicine, Department of Medicine, Division of Cardiology, Columbia University College of Physicians and Surgeons, Center for Advanced Cardiac Care, Columbia University Medical Center, New York, New York Cardiac Transplantation Lionel A. Mandell, MD Professor of Medicine, Faculty of Health Sciences, McMaster University, Hamilton, Ontario, Canada Streptococcus Pneumoniae Infections Peter Manu, MD Professor of Medicine and Psychiatry, Hofstra North Shore–LIJ School of Medicine at Hofstra University, Hempstead, New York; Adjunct Professor of Clinical Medicine, Psychiatry and Behavioral Sciences, Albert Einstein College of Medicine, Bronx, New York; Director of Medical Services, Zucker Hillside Hospital, Glen Oaks, New York Medical Consultation in Psychiatry Ariane Marelli, MD, MPH Professor of Medicine, McGill University, Director, McGill Adult Unit for Congenital Heart Disease, Associate Director, Academic Affairs and Research, Cardiology, McGill University Health Centre, Montreal, Québec, Canada Congenital Heart Disease in Adults Xavier Mariette, MD, PhD Professor, Rheumatology, Université Paris-Sud, AP-HP, Le Kremlin Bicêtre, France Sjögren Syndrome

xxi

Andrew R. Marks, MD Wu Professor and Chair, Department of Physiology and Cellular Biophysics, Founding Director, Helen and Clyde Wu Center for Molecular Cardiology, Columbia University College of Physicians and Surgeons, New York, New York Cardiac Function and Circulatory Control Kieren A. Marr, MD Professor of Medicine and Oncology, The Johns Hopkins University, Director, Transplant and Oncology Infectious Diseases, Baltimore, Maryland Approach to Fever and Suspected Infection in the Compromised Host Thomas J. Marrie, MD Dean, Faculty of Medicine, Dalhousie University; Professor of Medicine, Capital District Health Authority, Halifax, Nova Scotia, Canada Legionella Infections Paul Martin, MD Professor of Medicine and Chief, Division of Hepatology, Miller School of Medicine, University of Miami, Miami, Florida Approach to the Patient with Liver Disease Joel B. Mason, MD Professor of Medicine and Nutrition, Tufts University; Staff Physician, Divisions of Gastroenterology and Clinical Nutrition, Tufts Medical Center, Boston, Massachusetts Vitamins, Trace Minerals, and Other Micronutrients Henry Masur, MD Chief, Critical Care Medicine Department, Clinical Center, National Institutes of Health, Bethesda, Maryland Infectious and Metabolic Complications of HIV and AIDS Eric L. Matteson, MD, MPH Professor of Medicine, Mayo Clinic College of Medicine, Consultant, Divisions of Rheumatology and Epidemiology, Mayo Clinic, Rochester, Minnesota Infections of Bursae, Joints, and Bones Michael A. Matthay, MD Professor, Departments of Medicine and Anesthesia, University of California San Francisco, San Francisco, California Acute Respiratory Failure Toby A. Maurer, MD Professor of Dermatology, University of California San Francisco; Chief of Dermatology, San Francisco General Hospital, San Francisco, California Skin Manifestations in Patients with Human Immunodeficiency Virus Infection Emeran A. Mayer, MD, PhD Professor of Medicine, Physiology, and Psychiatry, Division of Digestive Diseases, Department of Medicine, University of California Los Angeles, Los Angeles, California Functional Gastrointestinal Disorders: Irritable Bowel Syndrome, Dyspepsia, Chest Pain of Presumed Esophageal Origin, and Heartburn Stephan A. Mayer, MD Director, Institute for Critical Care Medicine, Icahn School of Medicine at Mount Sinai, New York, New York Hemorrhagic Cerebrovascular Disease Stephen A. McClave, MD Professor of Medicine, Director of Clinical Nutrition, University of Louisville School of Medicine, Louisville, Kentucky Enteral Nutrition F. Dennis McCool, MD Professor of Medicine, The Warren Alpert Medical School of Brown University; Medical Director of Sleep Center, Memorial Hospital of Rhode Island, Pawtucket, Rhode Island Diseases of the Diaphragm, Chest Wall, Pleura, and Mediastinum

xxii

Contributors

Charles E. McCulloch, PhD Professor of Biostatistics, Department of Epidemiology and Biostatistics, University of California San Francisco, San Francisco, California Statistical Interpretation of Data

Ernest Moy, MD, MPH Medical Officer, Center for Quality Improvement and Patient Safety Agency for Healthcare Research and Quality, Rockville, Maryland Measuring Health and Health Care

William J. McKenna, MD Professor of Cardiology, Institute of Cardiovascular Science, University College London, London, United Kingdom Diseases of the Myocardium and Endocardium

Atis Muehlenbachs, MD, PhD Infectious Diseases Pathology Branch, Centers for Disease Control and Prevention, Atlanta, Georgia Leptospirosis

Vallerie McLaughlin, MD Kim A. Eagle, MD, Endowed Professor of Cardiovascular Medicine, Director, Pulmonary Hypertension Program, University of Michigan, Ann Arbor, Michigan Pulmonary Hypertension

Andrew H. Murr, MD Professor and Chairman, Roger Boles, MD Endowed Chair in Otolaryngology Education, Department of Otolaryngology-Head and Neck Surgery, University of California San Francisco School of Medicine, San Francisco, California Approach to the Patient with Nose, Sinus, and Ear Disorders

John J.V. McMurray, MB, MD Professor of Cardiology, Institute of Cardiovascular and Medical Sciences, University of Glasgow, Glasgow, Scotland, United Kingdom Heart Failure: Management and Prognosis Kenneth R. McQuaid, MD Professor of Clinical Medicine, Marvin H. Sleisenger Endowed Chair, Vice Chairman, University of California San Francisco; Chief, Medical Services and Gastroenterology, San Francisco VA Medical Center, San Francisco, California Approach to the Patient with Gastrointestinal Disease Marc Michel, MD Professor of Internal Medicine, Head of the Unit of Internal Medicine at Henri Mondor University Hospital, National Referral Center for Adult’s Immune Cytopenias, Creteil, France Autoimmune and Intravascular Hemolytic Anemias Jonathan W. Mink, MD, PhD Frederick A. Horner, MD Endowed Professor in Pediatric Neurology, Professor of Neurology, Neurobiology & Anatomy, Brain & Cognitive Sciences, and Pediatrics, Chief, Division of Child Neurology, Vice Chair, Department of Neurology, University of Rochester, Rochester, New York Congenital, Developmental, and Neurocutaneous Disorders William E. Mitch, MD Gordon A. Cain Chair in Nephrology, Director of Nephrology, Baylor College of Medicine, Houston, Texas Chronic Kidney Disease Mark E. Molitch, MD Martha Leland Sherwin Professor of Endocrinology, Division of Endocrinology, Metabolism, and Molecular Medicine, Northwestern University Feinberg School of Medicine, Chicago, Illinois Neuroendocrinology and the Neuroendocrine System; Anterior Pituitary Bruce A. Molitoris, MD Professor of Medicine, and Cellular and Integrative Physiology Director, Indiana Center for Biological Microscopy, Indiana University, Indianapolis, Indiana Acute Kidney Injury Jose G. Montoya, MD Professor of Medicine, Division of Infectious Disease and Geographic Medicine, Stanford University School of Medicine, Stanford, California; Director, Palo Alto Medical Foundation Toxoplasma Serology Laboratory, National Reference Center for the Study and Diagnosis of Toxoplasmosis, Palo Alto, California Toxoplasmosis Alison Morris, MD, MS Associate Professor of Medicine, Clinical Translational Science, and Immunology, Division of Pulmonary, Allergy, and Critical Care Medicine, University of Pittsburgh School of Medicine, Pittsburgh, Pennsylvania Pulmonary Manifestations of Human Immunodeficiency Virus and Acquired Immunodeficiency Syndrome

Daniel M. Musher, MD Professor of Medicine, Molecular Virology, and Microbiology, Distinguished Service Professor, Baylor College of Medicine, Infectious Disease Section, Michael E. DeBakey Veterans Affairs Medical Center, Houston, Texas Overview of Pneumonia Robert J. Myerburg, MD Professor of Medicine and Physiology, Division of Cardiology, Department of Medicine, American Heart Association Chair in Cardiovascular Research, University of Miami Miller School of Medicine, Miami, Florida Approach to Cardiac Arrest and Life-Threatening Arrhythmias Sandesh C.S. Nagamani, MD Assistant Professor, Department of Molecular and Human Genetics, Director, Clinic for Metabolic and Genetic Disorders of Bone, Baylor College of Medicine and Texas Children’s Hospital, Houston, Texas Gene, Genomic, and Chromosomal Disorders Stanley J. Naides, MD Medical Director and Interim Scientific Director, Immunology, Quest Diagnostics Nichols Institute, San Juan Capistrano, California Arboviruses Causing Fever and Rash Syndromes Yoshifumi Naka, MD, PhD Professor of Surgery, Department of Surgery, Columbia University College of Physicians and Surgeons, New York, New York Cardiac Transplantation Theodore E. Nash, MD Principal Investigator, Clinical Parasitology Section, Laboratory of Parasitic Diseases, National Institute of Allergy and Infectious Diseases, National Institutes of Health, Bethesda, Maryland Giardiasis Avindra Nath, MD Chief, Section of Infections of the Nervous System, National Institute of Neurological Disorders and Stroke, National Institutes of Health, Bethesda, Maryland Cytomegalovirus, Epstein-Barr Virus, and Slow Virus Infections of the Central Nervous System; Neurologic Complications of Human Immunodeficiency Virus Infection; Meningitis: Bacterial, Viral, and Other; Brain Abscess and Parameningeal Infections Eric G. Neilson, MD Vice President for Medical Affairs and Lewis Landsberg Dean, Northwestern University Feinberg School of Medicine, Northwestern Memorial Hospital, Chicago, Illinois Tubulointerstitial Nephritis

Contributors Lawrence S. Neinstein, MD Professor of Pediatrics and Medicine, Keck School of Medicine of USC; Executive Director, Engemann Student Health Center, Division Head of College Health, Assistant Provost, Student Health and Wellness, University of Southern California, Los Angeles, California Adolescent Medicine Lewis S. Nelson, MD Professor of Emergency Medicine, Director, Fellowship in Medical Toxicology, New York University School of Medicine; Attending Physician, New York University Langone Medical Center and Bellevue Hospital Center, New York, New York Acute Poisoning Eric J. Nestler, MD, PhD Nash Family Professor and Chair, Department of Neuroscience, Director, The Friedman Brain Institute, Icahn School of Medicine at Mount Sinai, New York, New York Biology of Addiction Anne B. Newman, MD, MPH Professor of Epidemiology, The University of Pittsburgh Graduate School of Public Health; Chair, Department of Epidemiology, Director, University of Pittsburgh Center for Aging and Population Health, Pittsburgh, Pennsylvania Epidemiology of Aging: Implications of the Aging of Society Thomas B. Newman, MD, MPH Professor, Epidemiology & Biostatistics and Pediatrics, University of California San Francisco, San Francisco, California Statistical Interpretation of Data William L. Nichols, MD Associate Professor, Medicine and Laboratory Medicine, Mayo Clinic College of Medicine; Staff Physician, Special Coagulation Laboratory, Comprehensive Hemophilia Center, and Coagulation Clinic, Mayo Clinic, Rochester, Minnesota Von Willebrand Disease and Hemorrhagic Abnormalities of Platelet and Vascular Function Lindsay E. Nicolle, MD Professor of Internal Medicine and Medical Microbiology, University of Manitoba, Health Sciences Centre, Winnipeg, Manitoba, Canada Approach to the Patient with Urinary Tract Infection Lynnette K. Nieman, MD Senior Investigator, Program on Reproductive and Adult Endocrinology, Eunice Kennedy Shriver National Institute of Child Health and Human Development, Bethesda, Maryland Approach to the Patient with Endocrine Disease; Adrenal Cortex; Polyglandular Disorders Dennis E. Niewoehner, MD Professor of Medicine, University of Minnesota; Staff Physician, Minneapolis Veterans Affairs Health Care System, Minneapolis, Minnesota Chronic Obstructive Pulmonary Disease S. Ragnar Norrby, MD, PhD Director General, Swedish Institute for Infectious Disease Control, Solna, Sweden Approach to the Patient with Urinary Tract Infection Susan O’Brien, MD Professor, Department of Leukemia, Division of Medicine, The University of Texas MD Anderson Cancer Center, Houston, Texas The Chronic Leukemias Christopher M. O’Connor, MD Professor of Medicine and Chief, Division of Cardiology, Director, Duke Heart Center, Durham, North Carolina Heart Failure: Pathophysiology and Diagnosis

xxiii

Francis G. O’Connor, MD, MPH Professor and Chair, Military and Emergency Medicine, Medical Director, Uniformed Services University Consortium for Health and Military Performance, Bethesda, Maryland Disorders Due to Heat and Cold; Rhabdomyolysis Patrick G. O’Connor, MD, MPH Professor and Chief, General Internal Medicine, Yale University School of Medicine, New Haven, Connecticut Alcohol Abuse and Dependence James R. O’Dell, MD Bruce Professor and Vice Chair of Internal Medicine, Chief, Division of Rheumatology, University of Nebraska Medical Center and Omaha VA Nebraska–Western Iowa Health Care System, Omaha, Nebraska Rheumatoid Arthritis Anne E. O’Donnell, MD Professor of Medicine, Chief, Division of Pulmonary, Critical Care, and Sleep Medicine, Georgetown University Medical Center, Washington, D.C. Bronchiectasis, Atelectasis, Cysts, and Localized Lung Disorders Jae K. Oh, MD Professor of Medicine, Director, Echocardiography Core Laboratory and Pericardial Clinic, Division of Cardiovascular Diseases, Co-Director, Integrated Cardiac Imaging, Division of Cardiovascular Diseases, Mayo Clinic, Rochester, Minnesota Pericardial Diseases Jeffrey E. Olgin, MD Gallo-Chatterjee Distinguished Professor of Medicine, Chief, Division of Cardiology, Co-Director, Heart and Vascular Center, University of California San Francisco, San Francisco, California Approach to the Patient with Suspected Arrhythmia Walter A. Orenstein, MD Professor of Medicine, Pediatrics, and Global Health, Emory University School of Medicine, Atlanta, Georgia Immunization Douglas R. Osmon, MD, MPH Professor of Medicine, Mayo Clinic College of Medicine; Consultant, Division of Infectious Diseases, Mayo Clinic, Rochester, Minnesota Infections of Bursae, Joints, and Bones Catherine M. Otto, MD J. Ward Kennedy-Hamilton Endowed Chair in Cardiology, Professor of Medicine, University of Washington School of Medicine; Director, Heart Valve Clinic, University of Washington Medical Center, Seattle, Washington Echocardiography Mark Papania, MD, MPH Medical Epidemiologist, Division of Viral Diseases, Measles, Mumps, Rubella, and Herpes Virus Laboratory Branch, Centers for Disease Control and Prevention, Atlanta, Georgia Measles Peter G. Pappas, MD Professor of Medicine, University of Alabama at Birmingham, Birmingham, Alabama Dematiaceous Fungal Infections Pankaj Jay Pasricha, MD Director, The Johns Hopkins Center for Neurogastroenterology; Professor of Medicine and Neurosciences, The Johns Hopkins School of Medicine; Professor of Innovation Management, Johns Hopkins Carey Business School, Baltimore, Maryland Gastrointestinal Endoscopy

xxiv

Contributors

David L. Paterson, MD Professor of Medicine, University of Queensland Centre for Clinical Research, Royal Brisbane and Women’s Hospital Campus, Brisbane, Queensland, Australia Infections Due to Other Members of the Enterobacteriaceae, Including Management of Multidrug Resistant Strains Carlo Patrono, MD Professor and Chair of Pharmacology, Department of Pharmacology, Catholic University School of Medicine, Rome, Italy Prostaglandin, Aspirin, and Related Compounds Jean-Michel Pawlotsky, MD, PhD Professor of Medicine, The University of Paris-Est; Director, National Reference Center for Viral Hepatitis B, C, and Delta and Department of Virology, Henri Mondor University Hospital; Director, Department of Molecular Virology and Immunology, Institut Mondor de Recherche Biomédicale, Créteil, France Acute Viral Hepatitis; Chronic Viral and Autoimmune Hepatitis Richard D. Pearson, MD Professor of Medicine and Pathology, University of Virginia School of Medicine and University of Virginia Health System, Charlottesville, Virginia Antiparasitic Therapy Trish M. Perl, MD, MSc Professor of Medicine and Pathology, The Johns Hopkins School of Medicine; Professor of Epidemiology, Johns Hopkins Bloomberg School of Public Health; Infectious Diseases Specialist and Senior Epidemiologist, The Johns Hopkins Hospital and Health System, Baltimore, Maryland Enterococcal Infections Adam Perlman, MD, MPH Associate Professor, Department of Medicine, Duke University Medical Center; Executive Director, Duke Integrative Medicine, Duke University Health System, Durham, North Carolina Complementary and Alternative Medicine William A. Petri, Jr., MD, PhD Wade Hampton Frost Professor, Departments of Medicine, Pathology, Microbiology, Immunology, and Cancer Biology, School of Medicine, University of Virginia; Chief, Division of Infectious Diseases and International Health, University of Virginia Hospitals, Charlottesville, Virginia Relapsing Fever and Other Borrelia Infections; African Sleeping Sickness; Amebiasis Marc A. Pfeffer, MD, PhD Dzau Professor of Medicine, Harvard Medical School; Senior Physician, Cardiovascular Division, Brigham and Women’s Hospital, Boston, Massachusetts Heart Failure: Management and Prognosis Perry J. Pickhardt, MD Professor of Radiology and Chief, Gastrointestinal Imaging, Section of Abdominal Imaging, University of Wisconsin School of Medicine and Public Health, Madison, Wisconsin Diagnostic Imaging Procedures in Gastroenterology David S. Pisetsky, MD, PhD Chief of Rheumatology, Medical Research Service, Durham VA Medical Center; Professor of Medicine and Immunology, Department of Medicine, Duke University Medical Center, Durham, North Carolina Laboratory Testing in the Rheumatic Diseases Marshall R. Posner, MD Professor of Medicine, Director of Head and Neck Medical Oncology, Director of the Office of Cancer Clinical Trials, The Tisch Cancer Institute, Icahn School of Medicine at Mount Sinai, New York, New York Head and Neck Cancer

Frank Powell, PhD Professor of Medicine, Chief of Physiology, University of California San Diego, La Jolla, California Disorders of Ventilatory Control Reed E. Pyeritz, MD, PhD William Smilow Professor of Medicine and Genetics and Vice Chair for Academic Affairs, Perelman School of Medicine at the University of Pennsylvania, Philadelphia, Pennsylvania Inherited Diseases of Connective Tissue Thomas C. Quinn, MD, MSc Associate Director for International Research, Head, Section of International HIV/AIDS Research, Division of Intramural Research, National Institute of Allergy and Infectious Diseases, National Institutes of Health; Professor of Medicine, Pathology, International Health, Molecular Microbiology and Immunology, and Epidemiology, The Johns Hopkins Medical Institutions, Baltimore, Maryland Epidemiology and Diagnosis of Human Immunodeficiency Virus Infection and Acquired Immunodeficiency Syndrome Jai Radhakrishnan, MD, MS Professor of Medicine, Division of Nephrology, Department of Medicine, Columbia University Medical Center; Associate Division Chief for Clinical Affairs, Division of Nephrology, New York-Presbyterian Hospital, New York, New York Glomerular Disorders and Nephrotic Syndromes Petros I. Rafailidis, MD, PhD, MSc Senior Researcher, Alfa Institute of Biomedical Sciences, Attending Physician, Department of Medicine and Hematology, Athens Medical Center, Athens Medical Group, Athens, Greece Pseudomonas and Related Gram-Negative Bacillary Infections Ganesh Raghu, MD Adjunct Professor of Medicine and Laboratory Medicine, University of Washington, Director, CENTER for Interstitial Lung Diseases at the University of Washington; Co-Director, Scleroderma Clinic, University of Washington Medical Center, Seattle, Washington Interstitial Lung Disease Margaret Ragni, MD, MPH Professor of Medicine and Clinical Translational Science, Department of Hematology/Oncology, University of Pittsburgh Medical Center; Director, Hemophilia Center of Western Pennsylvania, Pittsburgh, Pennsylvania Hemorrhagic Disorders: Coagulation Factor Deficiencies Srinivasa N. Raja, MD Professor of Anesthesiology and Neurology, Director, Division of Pain Medicine, The Johns Hopkins University School of Medicine, Baltimore, Maryland Pain S. Vincent Rajkumar, MD Professor of Medicine, Division of Hematology, Mayo Clinic, Rochester, Minnesota Plasma Cell Disorders Stuart H. Ralston, MB ChB, MD Professor of Rheumatology, Institute of Genetics and Molecular Medicine, Western General Hospital, The University of Edinburgh, Edinburgh, United Kingdom Paget Disease of Bone Didier Raoult, MD, PhD Professor, Aix Marseille Université, Faculté de Médecine; Chief, Hôpital de la Timone, Fédération de Microbiologie Clinique, Marseille, France Bartonella Infections; Rickettsial Infections

Contributors Robert W. Rebar, MD Professor, Department of Obstetrics and Gynecology, Western Michigan University Homer Stryker MD School of Medicine, Kalamazoo, Michigan Ovaries and Development; Reproductive Endocrinology and Infertility Annette C. Reboli, MD Founding Vice Dean, Professor of Medicine, Cooper Medical School of Rowan University, Cooper University Healthcare, Department of Medicine, Division of Infectious Diseases, Camden, New Jersey Erysipelothrix Infections

xxv

Karen Rosene-Montella, MD Professor and Vice Chair of Medicine, Director of Obstetric Medicine, The Warren Alpert Medical School of Brown University; Senior Vice President, Women’s Services and Clinical Integration, Lifespan Health System, Providence, Rhode Island Common Medical Problems in Pregnancy Philip J. Rosenthal, MD Professor, Department of Medicine, University of California San Francisco, San Francisco, California Malaria

K. Rajender Reddy, MD Professor of Medicine, Professor of Medicine in Surgery, Perelman School of Medicine at the University of Pennsylvania; Director of Hepatology, Director, Viral Hepatitis Center, Hospital of the University of Pennsylvania, Philadelphia, Pennsylvania Bacterial, Parasitic, Fungal, and Granulomatous Liver Diseases

Marc E. Rothenberg, MD, PhD Director, Division of Allergy and Immunology, Director, Cincinnati Center for Eosinophilic Disorders; Professor of Pediatrics, Cincinnati Children’s Hospital Medical Center, University of Cincinnati College of Medicine, Cincinnati, Ohio Eosinophilic Syndromes

Donald A. Redelmeier, MD Professor of Medicine, University of Toronto; Senior Scientist and Staff Physician, Sunnybrook Health Sciences Centre, Toronto, Ontario, Canada Postoperative Care and Complications

James A. Russell, MD Professor of Medicine, University of British Columbia; Associate Director, Intensive Care Unit, St. Paul’s Hospital, Vancouver, British Columbia, Canada Shock Syndromes Related to Sepsis

Susan E. Reef, MD Centers for Disease Control and Prevention, Atlanta, Georgia Rubella (German Measles)

Anil K. Rustgi, MD T. Grier Miller Professor of Medicine and Genetics, Chief of Gastroenterology, American Cancer Society; Professor, Perelman School of Medicine at the University of Pennsylvania, Philadelphia, Pennsylvania Neoplasms of the Esophagus and Stomach

Neil M. Resnick, MD Thomas P. Detre Endowed Chair in Gerontology and Geriatric Medicine, Professor of Medicine and Division Chief, Geriatrics, Associate Director, University of Pittsburgh Institute on Aging, University of Pittsburgh; Chief, Division of Geriatric Medicine and Gerontology, University of Pittsburgh Medical Center, Pittsburgh, Pennsylvania Incontinence David B. Reuben, MD Director, Multicampus Program in Geriatric Medicine and Gerontology; Chief, Division of Geriatrics, Archstone Professor of Medicine, David Geffen School of Medicine at University of California Los Angeles, Los Angeles, California Geriatric Assessment Emanuel P. Rivers, MD, MPH Professor and Vice Chairman of Emergency Medicine, Wayne State University; Senior Staff Attending, Critical Care and Emergency Medicine, Henry Ford Hospital, Detroit, Michigan Approach to the Patient with Shock Joseph G. Rogers, MD Professor of Medicine, Senior Vice Chief for Clinical Affairs, Division of Cardiology, Durham, North Carolina Heart Failure: Pathophysiology and Diagnosis Jean-Marc Rolain, PharmD, PhD Professor, Institut Hospitalo-Universitaire Méditerranée-Infection, Aix-Marseille Université, Marseille, France Bartonella Infections José R. Romero, MD Professor of Pediatrics, University of Arkansas for Medical Sciences, Horace C. Cabe Professor of Infectious Diseases; Director, Section of Pediatric Infectious Diseases, Arkansas Children’s Hospital, Little Rock, Arkansas Enteroviruses

Daniel E. Rusyniak, MD Professor of Emergency Medicine, Adjunct Professor of Neurology and Pharmacology and Toxicology, Department of Emergency Medicine, Indiana University School of Medicine, Indianapolis, Indiana Chronic Poisoning: Trace Metals and Others Robert A. Salata, MD Professor and Executive Vice Chair, Department of Medicine, Chief, Division of Infectious Diseases and HIV Medicine, Case Western Reserve University, University Hospitals Case Medical Center, Cleveland, Ohio Brucellosis Jane E. Salmon, MD Collette Kean Research Chair, Hospital for Special Surgery, Professor of Medicine, Weill Cornell Medical College, New York, New York Mechanisms of Immune-Mediated Tissue Injury Edsel Maurice T. Salvana, MD, DTM&H Associate Professor of Medicine, Section of Infectious Diseases, Department of Medicine, Philippine General Hospital; Director, Institute of Molecular Biology and Biotechnology, National Institutes of Health, University of the Philippines Manila, Manila, Philippines Brucellosis Renato M. Santos, MD Associate Professor, Cardiology, Wake Forest School of Medicine, Winston-Salem, North Carolina Vascular Disorders of the Kidney Michael N. Sawka, PhD Professor, School of Applied Physiology, Georgia Institute of Technology, Atlanta, Georgia Disorders Due to Heat and Cold Paul D. Scanlon, MD Professor of Medicine, Division of Pulmonary and Critical Care Medicine, Mayo Clinic, Rochester, Minnesota Respiratory Function: Mechanisms and Testing

xxvi

Contributors

Carla Scanzello, MD, PhD Assistant Professor of Medicine, Division of Rheumatology, Perelman School of Medicine at the University of Pennsylvania and Translational Musculoskeletal Research Center, Philadelphia Veterans Affairs Medical Center, Philadelphia, Pennsylvania Osteoarthritis Andrew I. Schafer, MD Professor of Medicine, Director, Richard T. Silver Center for Myeloproliferative Neoplasms, Weill Cornell Medical College, New York, New York Approach to Medicine, the Patient, and the Medical Profession: Medicine as a Learned and Humane Profession; Approach to the Patient with Bleeding and Thrombosis; Hemorrhagic Disorders: Disseminated Intravascular Coagulation, Liver Failure, and Vitamin K Deficiency; Thrombotic Disorders: Hypercoagulable States William Schaffner, MD Professor and Chair, Department of Preventive Medicine, Department of Health Policy; Professor of Medicine (Infectious Diseases), Vanderbilt University School of Medicine, Nashville, Tennessee Tularemia and Other Francisella Infections W. Michael Scheld, MD Bayer-Gerald L. Mandell Professor of Infectious Diseases, Professor of Medicine, Clinical Professor of Neurosurgery, Director, Pfizer Initiative in International Health, University of Virginia Health System, Charlottesville, Virginia Introduction to Microbial Disease: Host-Pathogen Interactions Manuel Schiff, MD Professor, Université Paris 7 Denis Diderot, Sorbonne Paris Cité, Head of Metabolic Unit/Reference Center for Inborn Errors of Metabolism, Robert Debré University Hospital, APHP, Paris, France Homocystinuria and Hyperhomocysteinemia Michael L. Schilsky, MD Associate Professor, Medicine and Surgery, Yale University School of Medicine, New Haven, Connecticut Wilson Disease Robert T. Schooley, MD Professor and Head, Division of Infectious Diseases, Executive Vice Chair for Academic Affairs, Department of Medicine, University of California San Diego, La Jolla, California Epstein-Barr Virus Infection David L. Schriger, MD, MPH Professor, Department of Emergency Medicine, University of California Los Angeles, Los Angeles, California Approach to the Patient with Abnormal Vital Signs Steven A. Schroeder, MD Distinguished Professor of Health and Healthcare and of Medicine, University of California San Francisco, San Francisco, California Socioeconomic Issues in Medicine Lynn M. Schuchter, MD Professor of Medicine, University of Pennsylvania; Chief, Hematology/ Oncology Division, Program Leader, Melanoma and Cutaneous Malignancies Program, Abramson Cancer Center, Hospital of the University of Pennsylvania, Philadelphia, Pennsylvania Melanoma and Nonmelanoma Skin Cancers Sam Schulman, MD, PhD Professor, Division of Hematology and Thromboembolism, Director of Clinical Thromboembolism Program, Department of Medicine, McMaster University, Hamilton, Ontario, Canada Antithrombotic Therapy

Lawrence B. Schwartz, MD, PhD Charles and Evelyn Thomas Professor of Medicine, Internal Medicine, Virginia Commonwealth University, Richmond, Virginia Systemic Anaphylaxis, Food Allergy, and Insect Sting Allergy Carlos Seas, MD Associate Professor of Medicine, Vice Director, Alexander von Humboldt Tropical Medicine Institute, Universidad Peruana Cayetano Heredia; Attending Physician, Department of Infectious, Tropical, and Dermatologic Diseases, National Hospital Cayetano Heredia, Lima, Peru Cholera and Other Vibrio Infections Steven A. Seifert, MD Professor of Emergency Medicine, University of New Mexico School of Medicine, Medical Director, New Mexico Poison and Drug Information Center, University of New Mexico Health Sciences Center, Albuquerque, New Mexico Envenomation Julian L. Seifter, MD Associate Professor of Medicine, Harvard Medical School; Senior Physician, Brigham and Women’s Hospital, Boston, Massachusetts Potassium Disorders; Acid-Base Disorders Duygu Selcen, MD Associate Professor of Neurology and Pediatrics, Department of Neurology, Mayo Clinic, Rochester, Minnesota Muscle Diseases Clay F. Semenkovich, MD Herbert S. Gasser Professor and Chief, Division of Endocrinology, Metabolism and Lipid Research, Washington University School of Medicine, St. Louis, Missouri Disorders of Lipid Metabolism Carol E. Semrad, MD Professor of Medicine, The University of Chicago Medicine, GI Section, Chicago, Illinois Approach to the Patient with Diarrhea and Malabsorption Harry Shamoon, MD Professor of Medicine and Associate Dean for Clinical and Translational Research, Albert Einstein College of Medicine; Director, Harold and Muriel Block Institute for Clinical and Translational Research at Einstein and Montefiore, Bronx, New York Diabetes Mellitus James C. Shaw, MD Associate Professor, Department of Medicine, University of Toronto; Head, Division of Dermatology, Department of Medicine, Women’s College Hospital, Toronto, Ontario, Canada Examination of the Skin and an Approach to Diagnosing Skin Diseases Pamela J. Shaw, DBE, MBBS, MD Professor of Neurology, University of Sheffield, Consultant Neurologist, Royal Hallamshire Hospital, Sheffield, United Kingdom Amyotrophic Lateral Sclerosis and Other Motor Neuron Diseases Robert L. Sheridan, MD Associate Professor of Surgery, Burn Service Medical Director, Boston Shriners Hospital for Children, Massachusetts General Hospital, Division of Burns, Harvard Medical School, Boston, Massachusetts Medical Aspects of Injuries and Burns Stuart Sherman, MD Professor of Medicine and Radiology, Director of ERCP, Indiana University School of Medicine, Indianapolis, Indiana Diseases of the Gallbladder and Bile Ducts

Contributors Michael E. Shy, MD Professor of Neurology, Pediatrics, and Physiology, University of Iowa, Iowa City, Iowa Peripheral Neuropathies

Frederick S. Southwick, MD Professor of Medicine, Division of Infectious Diseases, University of Florida and VF Health, Gainesville, Florida Nocardiosis

Ellen Sidransky, MD Chief, Section on Molecular Neurogenetics, Medical Genetics Branch, National Human Genome Research Institute, National Institutes of Health, Bethesda, Maryland The Lysosomal Storage Diseases

Allen M. Spiegel, MD Dean, Albert Einstein College of Medicine, Bronx, New York Principles of Endocrinology; Polyglandular Disorders

Richard M. Siegel, MD, PhD Clinical Director, National Institute of Arthritis, Musculoskeletal, and Skin Diseases, National Institutes of Health, Bethesda, Maryland The Systemic Autoinflammatory Diseases Robert F. Siliciano, MD, PhD Professor, The Johns Hopkins University School of Medicine, Howard Hughes Medical Institute, Baltimore, Maryland Immunopathogenesis of Human Immunodeficiency Virus Infection Michael S. Simberkoff, MD Chief of Staff, VA New York Harbor Healthcare System; Professor of Medicine, NYU School of Medicine, New York, New York Haemophilus and Moraxella Infections David L. Simel, MD, MHS Professor of Medicine, Duke University; Chief, Medical Service, Durham Veterans Affairs Medical Center, Durham, North Carolina Approach to the Patient: History and Physical Examination Kamaljit Singh, MD Associate Professor of Medicine, Attending Physician, Infectious Diseases, Rush University Medical Center, Chicago, Illinois Zoonoses Karl Skorecki, MD Annie Chutick Professor in Medicine, Rappaport Faculty of Medicine and Research Institute, Technion–Israel Institute of Technology; Director, Medical and Research Development, Rambam Health Care Campus, Haifa, Israel Gene and Cell Therapy; Disorders of Sodium and Water Homeostasis Itzchak Slotki, MD Associate Professor of Medicine, Hebrew University, Hadassah Medical School; Director, Division of Adult Nephrology, Shaare Zedek Medical Center, Jerusalem, Israel Disorders of Sodium and Water Homeostasis Arthur S. Slutsky, MD Professor of Medicine, Surgery, and Biomedical Engineering, University of Toronto; Vice President (Research), St. Michael’s Hospital, Keenan Research Centre, Li Ka Shing Knowledge Institute, Toronto, Ontario, Canada Acute Respiratory Failure; Mechanical Ventilation Eric J. Small, MD Professor of Medicine and Urology, Deputy Director and Director of Clinical Sciences, Helen Diller Family Comprehensive Cancer Center; Chief, Division of Hematology and Oncology, University of California San Francisco School of Medicine, San Francisco, California Prostate Cancer Gerald W. Smetana, MD Professor of Medicine, Harvard Medical School; Division of General Medicine and Primary Care, Beth Israel Deaconess Medical Center, Boston, Massachusetts Principles of Medical Consultation

xxvii

Robert F. Spiera, MD Professor of Clinical Medicine, Weill Cornell Medical College; Director, Scleroderma, Vasculitis, and Myositis Center, The Hospital for Special Surgery, New York, New York Polymyalgia Rheumatica and Temporal Arteritis Stanley M. Spinola, MD Professor and Chair, Department of Microbiology and Immunology, Professor of Medicine, Microbiology and Immunology, and Pathology and Laboratory Medicine, Indiana University School of Medicine, Indianapolis, Indiana Chancroid David Spriggs, MD Head, Division of Solid Tumor Oncology, Department of Medicine, Memorial Sloan Kettering Cancer Center; Professor of Medicine, Department of Medicine, Weill Cornell Medical College, New York, New York Gynecologic Cancers Paweł Stankiewicz, MD, PhD Department of Molecular and Human Genetics, Baylor College of Medicine, Houston, Texas Gene, Genomic, and Chromosomal Disorders Paul Stark, MD Professor Emeritus, University of California San Diego; Chief of Cardiothoracic Radiology, VA San Diego Healthcare System, San Diego, California Imaging in Pulmonary Disease David P. Steensma, MD Professor of Medicine, Harvard Medical School, Adult Leukemia Program, Dana-Farber Cancer Institute, Boston, Massachusetts Myelodysplastic Syndrome Martin H. Steinberg, MD Professor of Medicine, Pediatrics, and Pathology and Laboratory Medicine, Boston University School of Medicine; Director, Center of Excellence in Sickle Cell Disease, Boston Medical Center, Boston, Massachusetts Sickle Cell Disease and Other Hemoglobinopathies Theodore S. Steiner, MD Associate Professor, University of British Columbia; Associate Head, Division of Infectious Diseases, Vancouver General Hospital, Vancouver, British Columbia, Canada Escherichia Coli Enteric Infections David S. Stephens, MD Stephen W. Schwarzmann Distinguished Professor of Medicine, Emory University School of Medicine and Woodruff Health Sciences Center, Atlanta, Georgia Neisseria Meningitidis Infections David A. Stevens, MD Professor of Medicine, Stanford University Medical School; President, Principal Investigator, Infectious Diseases Research Laboratory, California Institute for Medical Research, San Jose and Stanford, California Systemic Antifungal Agents

xxviii

Contributors

James K. Stoller, MD, MS Chairman, Education Institute, Jean Wall Bennett Professor of Medicine, Cleveland Clinic Lerner College of Medicine; Staff, Respiratory Institute, Cleveland Clinic, Cleveland, Ohio Respiratory Monitoring in Critical Care John H. Stone, MD, MPH Professor of Medicine, Director, Clinical Rheumatology, Harvard Medical School, Massachusetts General Hospital, Boston, Massachusetts The Systemic Vasculitides Richard M. Stone, MD Professor of Medicine, Harvard Medical School, Clinical Director, Adult Leukemia Program, Dana-Farber Cancer Institute, Boston, Massachusetts Myelodysplastic Syndrome Raymond A. Strikas, MD, MPH Education Team Lead, Immunization Services Division, National Center for Immunization and Respiratory Diseases, Centers for Disease Control and Prevention, Atlanta, Georgia Immunization Edwin P. Su, MD Associate Professor of Clinical Orthopaedics, Orthopaedic Surgery, Weill Cornell University Medical College; Associate Attending Orthopaedic Surgeon, Adult Reconstruction and Joint Replacement, Hospital for Special Surgery, New York, New York Surgical Treatment of Joint Disease Roland W. Sutter, MD, MPH&TM Coordinator, Research, Policy and Product Development, Polio Operations and Research Department, World Health Organization, Geneva, Switzerland Diphtheria and Other Corynebacteria Infections Ronald S. Swerdloff, MD Professor of Medicine, David Geffen School of Medicine at University of California Los Angeles; Chief, Division of Endocrinology, Department of Medicine, Harbor-UCLA Medical Center, Torrance, California The Testis and Male Hypogonadism, Infertility, and Sexual Dysfunction Heidi Swygard, MD, MPH Associate Professor of Medicine, University of North Carolina at Chapel Hill, Chapel Hill, North Carolina Approach to the Patient with a Sexually Transmitted Infection Megan Sykes, MD Michael J. Friedlander Professor of Medicine, Director, Columbia Center for Translational Immunology, Columbia University Medical Center, New York, New York Transplantation Immunology Marian Tanofsky-Kraff, PhD Associate Professor, Department of Medical and Clinical Psychology, Uniformed Services University of Health Sciences, Bethesda, Maryland Eating Disorders Susan M. Tarlo, MBBS Professor of Medicine, Department of Medicine and Dalla Lana School of Public Health, University of Toronto, Respiratory Physician, University Health Network, Toronto Western Hospital and St. Michael’s Hospital, Toronto, Ontario, Canada Occupational Lung Disease Victoria M. Taylor, MD, MPH Professor of Medicine, University of Washington, Fred Hutchinson Cancer Research Center, Seattle, Washington Cultural Context of Medicine

Ayalew Tefferi, MD Professor of Medicine, Department of Hematology, Mayo Clinic, Rochester, Minnesota Polycythemia Vera, Essential Thrombocythemia, and Primary Myelofibrosis Paul S. Teirstein, MD Chief of Cardiology, Department of Medicine, Scripps Clinic, La Jolla, California Interventional and Surgical Treatment of Coronary Artery Disease Sam R. Telford III, ScD Professor, Tufts University Cummings School of Veterinary Medicine, North Grafton, Massachusetts Babesiosis and Other Protozoan Diseases Rajesh V. Thakker, MD May Professor of Medicine, University of Oxford; Radcliffe Department of Clinical Medicine, OCDEM, Churchill Hospital, Headington, Oxford, United Kingdom The Parathyroid Glands, Hypercalcemia, and Hypocalcemia Antonella Tosti, MD Professor of Clinical Dermatology, Department of Dermatology and Cutaneous Surgery, University of Miami, Miami, Florida Diseases of Hair and Nails Indi Trehan, MD, MPH, DTM&H Assistant Professor of Pediatrics, Washington University School of Medicine; Attending Physician, St. Louis Children’s Hospital, BarnesJewish Hospital, St. Louis, Missouri; Visiting Honorary Lecturer in Paediatrics and Child Health, University of Malawi College of Medicine; Consultant Paediatrician, Queen Elizabeth Central Hospital, Blantyre, Malawi Protein-Energy Malnutrition Ronald B. Turner, MD Professor of Pediatrics, University of Virginia School of Medicine, Charlottesville, Virginia The Common Cold Thomas S. Uldrick, MD Staff Clinician, HIV and AIDS Malignancy Branch, National Cancer Institute, Bethesda, Maryland Hematology and Oncology in Patients with Human Immunodeficiency Virus Infection Anthony M. Valeri, MD Professor of Medicine, Columbia University Medical Center; Director, Hemodialysis, Medical Director, Kidney and Pancreas Transplantation, New York-Presbyterian Hospital (CUMC); Director, Hemodialysis, Columbia University Dialysis Center, New York, New York Treatment of Irreversible Renal Failure John Varga, MD John and Nancy Hughes Professor of Medicine, Northwestern University Feinberg School of Medicine, Chicago, Illinois Systemic Sclerosis (Scleroderma) Bradley V. Vaughn, MD Professor of Neurology, Department of Neurology, University of North Carolina, Chapel Hill, North Carolina Disorders of Sleep Alan P. Venook, MD Professor of Medicine, University of California San Francisco, Helen Diller Family Comprehensive Cancer Center, San Francisco, California Liver and Biliary Tract Cancers Joseph G. Verbalis, MD Professor of Medicine, Georgetown University; Chief, Endocrinology and Metabolism, Georgetown University Hospital, Washington, D.C. Posterior Pituitary

Contributors Ronald G. Victor, MD Professor of Medicine, Burns and Allen Chair in Cardiology Research, Director, Hypertension Center, Associate Director, The Heart Institute, Cedars-Sinai Medical Center, Los Angeles, California Arterial Hypertension Angela Vincent, MBBS Professor of Neuroimmunology, University of Oxford; Honorary Consultant in Immunology, Oxford University Hospital Trust, Oxford, United Kingdom Disorders of Neuromuscular Transmission Robert M. Wachter, MD Professor and Associate Chairman, Department of Medicine, University of California San Francisco, San Francisco, California Quality of Care and Patient Safety Edward H. Wagner, MD, MPH Director Emeritus, MacColl Center for Health Care Innovation, Group Health Research Institute, Seattle, Washington Comprehensive Chronic Disease Management Edward E. Walsh, MD Professor of Medicine, University of Rochester School of Medicine and Dentistry; Head, Infectious Diseases, Rochester General Hospital, Rochester, New York Respiratory Syncytial Virus Thomas J. Walsh, MD Director, Transplantation-Oncology Infectious Diseases Program, Chief, Infectious Diseases Translational Research Laboratory, Professor of Medicine, Pediatrics, and Microbiology and Immunology, Weill Cornell Medical Center; Henry Schueler Foundation Scholar, Sharp Family Foundation Scholar in Pediatric Infectious Diseases, Adjunct Professor of Pathology, The Johns Hopkins University School of Medicine; Adjunct Professor of Medicine, The University of Maryland School of Medicine, Baltimore, Maryland Aspergillosis Jeremy D. Walston, MD Raymond and Anna Lublin Professor of Geriatric Medicine and Gerontology, The Johns Hopkins University School of Medicine, Baltimore, Maryland Common Clinical Sequelae of Aging Christina Wang, MD Professor of Medicine, David Geffen School of Medicine at University of California Los Angeles; Associate Director, UCLA Clinical and Translational Research Institute, Harbor-UCLA Medical Center, Torrance, California The Testis and Male Hypogonadism, Infertility, and Sexual Dysfunction Christine Wanke, MD Professor of Medicine and Public Health, Director, Division of Nutrition and Infection, Associate Chair, Department of Public Health, Tufts University School of Medicine, Boston, Massachusetts Gastrointestinal Manifestions of HIV and AIDS Stephen I. Wasserman, MD Professor of Medicine, University of California San Diego, La Jolla, California Approach to the Patient with Allergic or Immunologic Disease Thomas J. Weber, MD Associate Professor, Medicine/Endocrinology, Duke University, Durham, North Carolina Approach to the Patient with Metabolic Bone Disease; Osteoporosis Heiner Wedemeyer, MD Professor, Department of Gastroenterology, Hepatology, and Endocrinology, Hannover Medical School, Hannover, Germany Acute Viral Hepatitis

xxix

Geoffrey A. Weinberg, MD Professor of Pediatrics, University of Rochester School of Medicine and Dentistry; Director, Pediatric HIV Program, Golisano Children’s Hospital at University of Rochester Medical Center, Rochester, New York Parainfluenza Viral Disease David A. Weinstein, MD, MMSc Professor of Pediatric Endocrinology, Director, Glycogen Storage Disease Program, Division of Pediatric Endocrinology, University of Florida College of Medicine, Gainesville, Florida Glycogen Storage Diseases Robert S. Weinstein, MD Professor of Medicine, Department of Medicine, University of Arkansas for Medical Sciences; Staff Endocrinologist, Department of Medicine, Central Arkansas Veterans Health Care System, Little Rock, Arkansas Osteomalacia and Rickets Roger D. Weiss, MD Professor of Psychiatry, Harvard Medical School, Boston, Massachusetts; Chief, Division of Alcohol and Drug Abuse, McLean Hospital, Belmont, Massachusetts Drug Abuse and Dependence Martin Weisse, MD Chair, Pediatrics, Tripler Army Medical Center, Honolulu, Hawaii; Professor, Pediatrics, Uniformed Services University of the Health Sciences, Bethesda, Maryland Measles Jeffrey I. Weitz, MD Professor of Medicine and Biochemistry, McMaster University; Executive Director, Thrombosis and Atherosclerosis Research Institute, Hamilton, Ontario, Canada Pulmonary Embolism Samuel A. Wells, Jr., MD Medical Oncology Branch, National Cancer Institute, National Institutes of Health, Bethesda, Maryland Medullary Thyroid Carcinoma Richard P. Wenzel, MD, MSc Professor and Former Chairman, Internal Medicine, Virginia Commonwealth University, Richmond, Virginia Acute Bronchitis and Tracheitis Victoria P. Werth, MD Professor of Dermatology and Medicine, Hospital of the University of Pennsylvania and Philadelphia Veterans Administration Medical Center; Chief, Dermatology Division, Philadelphia Veterans Administration Medical Center, Philadelphia, Pennsylvania Principles of Therapy of Skin Diseases Sterling G. West, MD, MACP Professor of Medicine, University of Colorado School of Medicine; Associate Division Head for Clinical and Educational Affairs, University of Colorado Division of Rheumatology, Aurora, Colorado Systemic Diseases in Which Arthritis Is a Feature A. Clinton White, Jr., MD Paul R. Stalnaker Distinguished Professor and Director, Infectious Disease Division, Department of Internal Medicine, University of Texas Medical Branch, Galveston, Texas Cestodes Christopher J. White, MD Professor of Medicine, Ochsner Clinical School, University of Queensland School of Medicine; System Chairman of Cardiovascular Diseases, Ochsner Medical Center, New Orleans, Louisiana Atherosclerotic Peripheral Arterial Disease; Electrophysiologic Interventional Procedures and Surgery

xxx

Contributors

Perrin C. White, MD Professor of Pediatrics, The Audry Newman Rapoport Distinguished Chair in Pediatric Endocrinology, University of Texas Southwestern Medical Center, Chief of Endocrinology, Children’s Medical Center Dallas, Dallas, Texas Disorders of Sexual Development Richard J. Whitley, MD Distinguished Professor of Pediatrics, Loeb Eminent Scholar Chair in Pediatrics, Professor of Pediatrics, Microbiology, Medicine, and Neurosurgery, The University of Alabama at Birmingham, Birmingham, Alabama Herpes Simplex Virus Infections Michael P. Whyte, MD Professor of Medicine, Pediatrics, and Genetics, Division of Bone and Mineral Diseases, Washington University School of Medicine; MedicalScientific Director, Center for Metabolic Bone Disease and Molecular Research, Shriners Hospital for Children, St. Louis, Missouri Osteonecrosis, Osteosclerosis/Hyperostosis, and Other Disorders of Bone Samuel Wiebe, MD, MSc Professor of Clinical Neurosciences, University of Calgary; Co-Director, Calgary Epilepsy Program, Alberta Health Services, Foothills Medical Centre, Calgary, Alberta, Canada The Epilepsies Jeanine P. Wiener-Kronish, MD Henry Isaiah Dorr Professor of Research and Teaching in Anaesthesia and Anesthestist-in-Chief, Department of Anesthesia, Critical Care and Pain Medicine, Massachusetts General Hospital/Harvard Medical School, Boston, Massachusetts Overview of Anesthesia Eelco F.M. Wijdicks, MD, PhD Professor of Neurology, Division of Critical Care Neurology, Department of Neurology, Mayo Clinic, Rochester, Minnesota Coma, Vegetative State, and Brain Death David J. Wilber, MD George M. Eisenberg Professor of Medicine, Loyola Stritch School of Medicine; Director, Division of Cardiology, Director, Clinical Electrophysiology, Loyola University Medical Center, Maywood, Illinois Electrophysiologic Interventional Procedures and Surgery Beverly Winikoff, MD, MPH President, Gynuity Health Projects; Professor of Clinical Population and Family Health, Mailman School of Public Health, Columbia University, New York, New York Contraception Gary P. Wormser, MD Professor of Medicine and Chief, Division of Infectious Diseases, Department of Medicine, New York Medical College, Valhalla, New York Lyme Disease

Myron Yanoff, MD Professor and Chair, Ophthalmology, Drexel University College of Medicine, Philadelphia, Pennsylvania Diseases of the Visual System Robert Yarchoan, MD Branch Chief, HIV and AIDS Malignancy Branch, National Cancer Institute, Bethesda, Maryland Hematology and Oncology in Patients with Human Immunodeficiency Virus Infection Neal S. Young, MD Chief, Hematology Branch, NHLBI and Director, Trans-NIH Center for Human Immunology, Autoimmunity, and Inflammation, National Institutes of Health, Bethesda, Maryland Parvovirus William F. Young, Jr., MD, MSc Professor of Medicine, Mayo Clinic College of Medicine; Chair, Division of Endocrinology, Diabetes, Metabolism, and Nutrition, Mayo Clinic, Rochester, Minnesota Adrenal Medulla, Catecholamines, and Pheochromocytoma Alan S.L. Yu, MB, BChir Harry Statland and Solon Summerfield Professor of Medicine, Director, Division of Nephrology and Hypertension and the Kidney Institute, University of Kansas Medical Center, Kansas City, Kansas Disorders of Magnesium and Phosphorus Sherif R. Zaki, MD, PhD Chief, Infectious Diseases Pathology Branch, Centers for Disease Control and Prevention, Atlanta, Georgia Leptospirosis Mark L. Zeidel, MD Herman L. Blumgart Professor of Medicine, Harvard Medical School; Physician-in-Chief and Chairman, Department of Medicine, Beth Israel Deaconess Medical Center, Boston, Massachusetts Obstructive Uropathy Thomas R. Ziegler, MD Professor, Department of Medicine, Division of Endocrinology, Metabolism, and Lipids, Emory University School of Medicine, Atlanta, Georgia Malnutrition, Nutritional Assessment, and Nutritional Support in Adult Hospitalized Patients Peter Zimetbaum, MD Associate Professor of Medicine, Harvard Medical School; Director of Clinical Cardiology, Beth Israel Deaconess Medical Center, Boston, Massachusetts Cardiac Arrhythmias with Supraventricular Origin

CONTENTS VOLUME I

24 Geriatric Assessment

SECTION I: SOCIAL AND ETHICAL ISSUES IN MEDICINE

25 Common Clinical Sequelae of Aging

1 Approach to Medicine, the Patient, and the Medical Profession: Medicine as a Learned and Humane Profession

2 4 9 15

VICTORIA M. TAYLOR

5 Socioeconomic Issues in Medicine

17

STEVEN A. SCHROEDER

6 Global Health

19

ARUN CHOCKALINGAM

24 28 32

THOMAS B. NEWMAN AND CHARLES E. McCULLOCH

10 Using Data for Clinical Decisions

44

ROBERT M. WACHTER

13 Comprehensive Chronic Disease Management

46

EDWARD H. WAGNER

58 61 65 78 82 86

MARK G. KORTEPETER AND THEODORE J. CIESLAK

22 Chronic Poisoning: Trace Metals and Others

92

LOUISE W. KAO AND DANIEL E. RUSYNIAK

ANNE B. NEWMAN AND JANE A. CAULEY

162

GRANT W. CANNON

36 Biologic Agents and Signaling Inhibitors

169

CEM GABAY

37 Prostanoids, Aspirin, and Related Compounds

172

CARLO PATRONO

175 181

ADAM PERLMAN

40 Principles of Genetics

186

BRUCE R. KORF

41 Gene, Genomic, and Chromosomal Disorders

189

SANDESH C.S. NAGAMANI, PAWEŁ STANKIEWICZ, AND JAMES R. LUPSKI

42 The Inherited Basis of Common Diseases 43 Application of Molecular Technologies to Clinical Medicine

196 200

GEOFFREY S. GINSBURG

203

LIOR GEPSTEIN AND KARL SKORECKI

SECTION VII: PRINCIPLES OF IMMUNOLOGY AND INFLAMMATION 45 The Innate Immune System

SECTION IV: AGING AND GERIATRIC MEDICINE 23 Epidemiology of Aging: Implications of an Aging Society

156

ROGER D. WEISS

44 Regenerative Medicine, Cell, and Gene Therapies

DAVID J. BRENNER

21 Bioterrorism

149

SEKAR KATHIRESAN AND DAVID ALTSHULER

MARK R. CULLEN

20 Radiation Injury

145

PATRICK G. O’CONNOR

SECTION VI: GENETICS

RAYMOND A. STRIKAS AND WALTER A. ORENSTEIN

19 Principles of Occupational and Environmental Medicine

143

TONY P. GEORGE

54

DEBRA K. KATZMAN AND LAWRENCE S. NEINSTEIN

18 Immunization

133

ERIC J. NESTLER AND STEVEN E. HYMAN

39 Complementary and Alternative Medicine

DAVID M. BUCHNER

17 Adolescent Medicine

31 Biology of Addiction

52

DAVID ATKINS AND MARY BARTON

16 Physical Activity

30 Pain

SAM SCHULMAN AND JACK HIRSH

F. DANIEL DUFFY

15 The Periodic Health Examination

124

ROBERT B. DIASIO

38 Antithrombotic Therapy

SECTION III: PREVENTIVE AND ENVIRONMENTAL ISSUES 14 Counseling for Behavior Change

29 Principles of Drug Therapy

35 Immunosuppressing Drugs Including Corticosteroids 41

CAROLYN M. CLANCY AND ERNEST MOY

12 Quality of Care and Patient Safety

117

SECTION V: CLINICAL PHARMACOLOGY

34 Drugs of Abuse 37

THOMAS H. LEE

11 Measuring Health and Health Care

114

SHARON K. INOUYE

33 Alcohol Use Disorders

DAVID L. SCHRIGER

9 Statistical Interpretation of Data

28 Delirium or Acute Mental Status Change in the Older Patient

32 Nicotine and Tobacco

DAVID L. SIMEL

8 Approach to the Patient with Abnormal Vital Signs

27 Neuropsychiatric Aspects of Aging

STEVEN P. COHEN AND SRINIVASA N. RAJA

SECTION II: PRINCIPLES OF EVALUATION AND MANAGEMENT 7 Approach to the Patient: History and Physical Examination

110

SHARON K. INOUYE

ROBERT ARNOLD

4 Cultural Context of Medicine

26 Incontinence NEIL M. RESNICK

EZEKIEL J. EMANUEL

3 Care of Dying Patients and Their Families

106

JEREMY D. WALSTON

LEE GOLDMAN AND ANDREW I. SCHAFER

2 Bioethics in the Practice of Medicine

102

DAVID B. REUBEN

216

MARY K. CROW

100

46 The Adaptive Immune System JOSEPH CRAFT

220

xxxii

Contents

47 Mechanisms of Immune-Mediated Tissue Injury

226

JANE E. SALMON

48 Mechanisms of Inflammation and Tissue Repair

230 236 240

248 257 262

ANDREW R. MARKS

267

LEONARD GANZ

55 Echocardiography

274

CATHERINE M. OTTO

56 Noninvasive Cardiac Imaging

282

CHRISTOPHER M. KRAMER, GEORGE A. BELLER, AND KLAUS D. HAGSPIEL

57 Catheterization and Angiography

292 298 305 320 339 344

JEFFREY E. OLGIN

63 Approach to Cardiac Arrest and Life-Threatening Arrhythmias

352 356 367 374 381

RONALD G. VICTOR

68 Pulmonary Hypertension

397

VALLERIE McLAUGHLIN

69 Congenital Heart Disease in Adults

405

ARIANE J. MARELLI

70 Atherosclerosis, Thrombosis, and Vascular Biology

417

GÖRAN K. HANSSON AND ANDERS HAMSTEN

71 Angina Pectoris and Stable Ischemic Heart Disease

420

WILLIAM E. BODEN

72 Acute Coronary Syndrome: Unstable Angina and Non-ST Elevation Myocardial Infarction

432

PAUL S. TEIRSTEIN AND BRUCE W. LYTLE

526

MONICA KRAFT

84 Imaging in Pulmonary Disease

531

PAUL STARK

85 Respiratory Function: Mechanisms and Testing

539

86 Disorders of Ventilatory Control

545

87 Asthma

548

88 Chronic Obstructive Pulmonary Disease

555

89 Cystic Fibrosis

562

90 Bronchiectasis, Atelectasis, Cysts, and Localized Lung Disorders

566

ANNE E. O’DONNELL

571

92 Interstitial Lung Disease

575

93 Occupational Lung Disease

588

SUSAN M. TARLO

94 Physical and Chemical Injuries of the Lung

595

95 Sarcoidosis

603

MICHAEL C. IANNUZZI

96 Acute Bronchitis and Tracheitis

608

RICHARD P. WENZEL

97 Overview of Pneumonia

610

DANIEL M. MUSHER

98 Pulmonary Embolism

620

JEFFREY I. WEITZ

99 Diseases of the Diaphragm, Chest Wall, Pleura, and Mediastinum

627

F. DENNIS McCOOL

100 Obstructive Sleep Apnea

638

101 Interventional and Surgical Approaches to Lung Disease

642

DAVID J. FELLER-KOPMAN AND MALCOLM M. DeCAMP

441

JEFFREY L. ANDERSON

74 Interventional and Surgical Treatment of Coronary Artery Disease

83 Approach to the Patient with Respiratory Disease

ROBERT C. BASNER

RICHARD A. LANGE AND L. DAVID HILLIS

73 ST Segment Elevation Acute Myocardial Infarction and Complications of Myocardial Infarction

SECTION IX: RESPIRATORY DISEASES

DAVID C. CHRISTIANI

DAVID J. WILBER

67 Arterial Hypertension

519

DONNA MANCINI AND YOSHIFUMI NAKA

GANESH RAGHU

HASAN GARAN

66 Electrophysiologic Interventional Procedures and Surgery

82 Cardiac Transplantation

STEPHANIE M. LEVINE

PETER ZIMETBAUM

65 Ventricular Arrhythmias

511

JEFFREY S. GINSBERG

91 Alveolar Filling Disorders

ROBERT J. MYERBURG

64 Cardiac Arrhythmias with Supraventricular Origin

81 Peripheral Venous Disease

FRANK J. ACCURSO

GLENN I. FISHMAN

62 Approach to the Patient with Suspected Arrhythmia

504

MICHAEL R. JAFF AND JOHN R. BARTHOLOMEW

DENNIS E. NIEWOEHNER

WILLIAM J. McKENNA AND PERRY ELLIOTT

61 Principles of Electrophysiology

80 Other Peripheral Arterial Diseases

JEFFREY M. DRAZEN

JOHN J.V. McMURRAY AND MARC A. PFEFFER

60 Diseases of the Myocardium and Endocardium

497

ATUL MALHOTRA AND FRANK POWELL

CHRISTOPHER M. O’CONNOR AND JOSEPH G. ROGERS

59 Heart Failure: Management and Prognosis

492

PAUL D. SCANLON

MORTON KERN

58 Heart Failure: Pathophysiology and Diagnosis

483

CHRISTOPHER J. WHITE

DONALD M. LLOYD-JONES

54 Electrocardiography

78 Diseases of the Aorta 79 Atherosclerotic Peripheral Arterial Disease

LEE GOLDMAN

53 Cardiac Function and Circulatory Control

77 Pericardial Diseases

FRANK A. LEDERLE

SECTION VIII: CARDIOVASCULAR DISEASE

52 Epidemiology of Cardiovascular Disease

474

WILLIAM C. LITTLE AND JAE K. OH

JOHN P. ATKINSON

51 Approach to the Patient with Possible Cardiovascular Disease

76 Infective Endocarditis VANCE G. FOWLER, JR., ARNOLD S. BAYER, AND LARRY M. BADDOUR

MEGAN SYKES

50 Complement System in Disease

461

BLASE A. CARABELLO

GARY S. FIRESTEIN

49 Transplantation Immunology

75 Valvular Heart Disease

SECTION X: CRITICAL CARE MEDICINE 102 Approach to the Patient in a Critical Care Setting

650

DEBORAH J. COOK

456

103 Respiratory Monitoring in Critical Care JAMES K. STOLLER AND NICHOLAS S. HILL

652

Contents 104 Acute Respiratory Failure

655

MICHAEL A. MATTHAY AND ARTHUR S. SLUTSKY

105 Mechanical Ventilation

664 672 681 685 691

MICHAEL N. SAWKA AND FRANCIS G. O’CONNOR

110 Acute Poisoning

696

LEWIS S. NELSON AND MARSHA D. FORD

111 Medical Aspects of Injuries and Burns

711

ROBERT L. SHERIDAN

112 Envenomation

717

GEOFFREY K. ISBISTER AND STEVEN A. SEIFERT

113 Rhabdomyolysis

723

FRANCIS G. O’CONNOR AND PATRICIA A. DEUSTER

728

DONALD W. LANDRY AND HASAN BAZARI

115 Structure and Function of the Kidneys

737

QAIS AL-AWQATI AND JONATHAN BARASCH

116 Disorders of Sodium and Water Homeostasis

741

ITZCHAK SLOTKI AND KARL SKORECKI

117 Potassium Disorders

879

136 Disorders of Gastrointestinal Motility

884

137 Functional Gastrointestinal Disorders: Irritable Bowel Syndrome, Dyspepsia, Chest Pain of Presumed Esophageal Origin, and Heartburn

890

EMERAN A. MAYER

138 Diseases of the Esophagus

896

GARY W. FALK AND DAVID A. KATZKA

139 Acid Peptic Disease

908

ERNST J. KUIPERS AND MARTIN J. BLASER

140 Approach to the Patient with Diarrhea and Malabsorption

918

CAROL E. SEMRAD

141 Inflammatory Bowel Disease

935

GARY R. LICHTENSTEIN

SECTION XI: RENAL AND GENITOURINARY DISEASES 114 Approach to the Patient with Renal Disease

135 Gastrointestinal Hemorrhage

MICHAEL CAMILLERI

JAMES A. RUSSELL

109 Disorders Due to Heat and Cold

872

THOMAS O. KOVACS AND DENNIS M. JENSEN

STEVEN M. HOLLENBERG

108 Shock Syndromes Related to Sepsis

134 Gastrointestinal Endoscopy PANKAJ JAY PASRICHA

EMANUEL P. RIVERS

107 Cardiogenic Shock

866

DAVID H. KIM AND PERRY J. PICKHARDT

ARTHUR S. SLUTSKY

106 Approach to the Patient with Shock

133 Diagnostic Imaging Procedures in Gastroenterology

xxxiii

755

142 Inflammatory and Anatomic Diseases of the Intestine, Peritoneum, Mesentery, and Omentum

943

JOHN F. KUEMMERLE

143 Vascular Diseases of the Gastrointestinal Tract

951

STEPHEN CRANE HAUSER

144 Pancreatitis

959

CHRIS E. FORSMARK

145 Diseases of the Rectum and Anus

967

ROBERT D. MADOFF

JULIAN L. SEIFTER

118 Acid-Base Disorders

762

SECTION XIII: DISEASES OF THE LIVER, GALLBLADDER, AND BILE DUCTS

774

146 Approach to the Patient with Liver Disease

778

147 Approach to the Patient with Jaundice or Abnormal Liver Tests

JULIAN L. SEIFTER

119 Disorders of Magnesium and Phosphorus ALAN S.L. YU

120 Acute Kidney Injury BRUCE A. MOLITORIS

121 Glomerular Disorders and Nephrotic Syndromes

783

GERALD B. APPEL AND JAI RADHAKRISHNAN

122 Tubulointerstitial Nephritis

793

ERIC G. NEILSON

123 Obstructive Uropathy

799

MARK L. ZEIDEL

124 Diabetes and the Kidney

804

RAYMOND C. HARRIS

125 Vascular Disorders of the Kidney

807

THOMAS D. DUBOSE, JR. AND RENATO M. SANTOS

126 Nephrolithiasis

811

DAVID A. BUSHINSKY

127 Cystic Kidney Diseases

816

M. AMIN ARNAOUT

128 Hereditary Nephropathies and Developmental Abnormalities of the Urinary Tract

822

129 Benign Prostatic Hyperplasia and Prostatitis

827

STEVEN A. KAPLAN

130 Chronic Kidney Disease

833 841

149 Chronic Viral and Autoimmune Hepatitis

1000

JEAN-MICHEL PAWLOTSKY

150 Toxin- and Drug-Induced Liver Disease

1006

WILLIAM M. LEE

151 Bacterial, Parasitic, Fungal, and Granulomatous Liver Diseases

1011

K. RAJENDER REDDY

152 Alcoholic and Nonalcoholic Steatohepatitis

1019

NAGA P. CHALASANI

153 Cirrhosis and Its Sequelae

1023

GUADALUPE GARCIA-TSAO

1031

GREGORY T. EVERSON

155 Diseases of the Gallbladder and Bile Ducts

1038

EVAN L. FOGEL AND STUART SHERMAN

SECTION XIV: HEMATOLOGIC DISEASES 156 Hematopoiesis and Hematopoietic Growth Factors 157 The Peripheral Blood Smear 158 Approach to the Anemias

SECTION XII: GASTROINTESTINAL DISEASES KENNETH R. McQUAID

993

JEAN-MICHEL PAWLOTSKY

1050 1052

BARBARA J. BAIN

DAVID COHEN AND ANTHONY MICHAEL VALERI

132 Approach to the Patient with Gastrointestinal Disease

148 Acute Viral Hepatitis

KENNETH KAUSHANSKY

WILLIAM E. MITCH

131 Treatment of Irreversible Renal Failure

983

PAUL D. BERK AND KEVIN M. KORENBLAT

154 Hepatic Failure and Liver Transplantation

LISA M. GUAY-WOODFORD

976

PAUL MARTIN

1059

H. FRANKLIN BUNN

850

159 Microcytic and Hypochromic Anemias GORDON D. GINDER

1068

xxxiv

Contents

160 Autoimmune and Intravascular Hemolytic Anemias

1073

MARC MICHEL

161 Hemolytic Anemias: Red Blood Cell Membrane and Metabolic Defects

1080 1089 1095

MARTIN H. STEINBERG

164 Megaloblastic Anemias

1104

AŚOK C. ANTONY

165 Aplastic Anemia and Related Bone Marrow Failure States

1114

GROVER C. BAGBY

166 Polycythemia Vera, Essential Thrombocythemia, and Primary Myelofibrosis

1138 1142 1151 1154 1159

CHARLES S. ABRAMS

173 Von Willebrand Disease and Hemorrhagic Abnormalities of Platelet and Vascular Function

1167 1172 1181

ANDREW I. SCHAFER

176 Thrombotic Disorders: Hypercoagulable States

1185

ANDREW I. SCHAFER

177 Transfusion Medicine

1191

LAWRENCE T. GOODNOUGH

178 Hematopoietic Stem Cell Transplantation

1198

ARMAND KEATING AND MICHAEL R. BISHOP

1222

DAVID J. HUNTER

181 Cancer Biology and Genetics

1226

ADRIAN R. BLACK AND KENNETH H. COWAN

182 Myelodysplastic Syndromes

1233

DAVID P. STEENSMA AND RICHARD M. STONE

183 The Acute Leukemias

1239

FREDERICK R. APPELBAUM

184 The Chronic Leukemias

1246

SUSAN O’BRIEN AND ELIAS JABBOUR

185 Non-Hodgkin Lymphomas

1257

PHILIP J. BIERMAN AND JAMES O. ARMITAGE

186 Hodgkin Lymphoma

1268

JOSEPH M. CONNORS

187 Plasma Cell Disorders

1273

S. VINCENT RAJKUMAR

188 Amyloidosis MORIE A. GERTZ

1334 1339

ROBIN K. KELLEY AND ALAN P. VENOOK

1345

DEAN F. BAJORIN

1351

NANCY E. DAVIDSON

199 Gynecologic Cancers

1360

DAVID SPRIGGS

200 Testicular Cancer

1365

LAWRENCE H. EINHORN

201 Prostate Cancer

1367

ERIC J. SMALL

202 Malignant Tumors of Bone, Sarcomas, and Other Soft Tissue Neoplasms

1370

JAMES H. DOROSHOW

203 Melanoma and Nonmelanoma Skin Cancers 204 Cancer of Unknown Primary Origin

1373 1378

VOLUME II SECTION XVI: METABOLIC DISEASES 205 Approach to Inborn Errors of Metabolism

1384

OLAF A. BODAMER

206 Disorders of Lipid Metabolism

1389

CLAY F. SEMENKOVICH

207 Glycogen Storage Diseases

1397

DAVID A. WEINSTEIN

208 Lysosomal Storage Diseases 209 Homocystinuria and Hyperhomocysteinemia

1206

JAMES H. DOROSHOW

180 Epidemiology of Cancer

195 Pancreatic Neuroendocrine Tumors

1399

DONNA M. KRASNEWICH AND ELLEN SIDRANSKY

SECTION XV: ONCOLOGY 179 Approach to the Patient with Cancer

1332

DANIEL LAHERU

JOHN D. HAINSWORTH AND F. ANTHONY GRECO

MARGARET V. RAGNI

175 Hemorrhagic Disorders: Disseminated Intravascular Coagulation, Liver Failure, and Vitamin K Deficiency

194 Pancreatic Cancer

LYNN M. SCHUCHTER

WILLIAM L. NICHOLS

174 Hemorrhagic Disorders: Coagulation Factor Deficiencies

1320

CHARLES D. BLANKE AND DOUGLAS O. FAIGEL

198 Breast Cancer and Benign Breast Disorders

ANDREW I. SCHAFER

172 Thrombocytopenia

193 Neoplasms of the Small and Large Intestine

197 Tumors of the Kidney, Bladder, Ureters, and Renal Pelvis

MARC E. ROTHENBERG

171 Approach to the Patient with Bleeding and Thrombosis

1313

ANIL K. RUSTGI

1129

MICHAEL GLOGAUER

170 Eosinophilic Syndromes

1303

FADLO R. KHURI

196 Liver and Biliary Tract Cancers

JAMES O. ARMITAGE AND PHILIP J. BIERMAN

169 Disorders of Phagocyte Function

191 Lung Cancer and Other Pulmonary Neoplasms

1121

NANCY BERLINER

168 Approach to the Patient with Lymphadenopathy and Splenomegaly

1297

MARSHALL R. POSNER

ROBERT T. JENSEN

AYALEW TEFFERI

167 Leukocytosis and Leukopenia

190 Head and Neck Cancer

192 Neoplasms of the Esophagus and Stomach

MARIA DOMENICA CAPPELLINI

163 Sickle Cell Disease and Other Hemoglobinopathies

1287

LISA M. DeANGELIS

PATRICK G. GALLAGHER

162 The Thalassemias

189 Tumors of the Central Nervous System

1403

MANUEL SCHIFF AND HENK BLOM

210 The Porphyrias

1408

RICHARD J. HIFT

211 Wilson Disease

1416

STEPHEN G. KALER AND MICHAEL L. SCHILSKY

212 Iron Overload (Hemochromatosis)

1418

BRUCE R. BACON

SECTION XVII: NUTRITIONAL DISEASES 213 Nutrition’s Interface with Health and Disease

1426

DOUGLAS C. HEIMBURGER

214 Nutritional Assessment

1430

BRUCE R. BISTRIAN

215 Protein-Energy Malnutrition

1434

MARK J. MANARY AND INDI TREHAN

216 Enteral Nutrition

1437

STEPHEN A. McCLAVE

217 Malnutrition, Nutritional Assessment, and Nutritional Support in Adult Hospitalized Patients

1440

THOMAS R. ZIEGLER

1284

218 Vitamins, Trace Minerals, and Other Micronutrients JOEL B. MASON

1445

Contents 219 Eating Disorders

1455

MARIAN TANOFSKY-KRAFF

220 Obesity

1458

1468

DAVID R. CLEMMONS AND LYNNETTE K. NIEMAN

1471

ALLEN M. SPIEGEL

223 Neuroendocrinology and the Neuroendocrine System

1473

MARK E. MOLITCH

224 Anterior Pituitary

1480

MARK E. MOLITCH

225 Posterior Pituitary

1494

JOSEPH G. VERBALIS

226 Thyroid

1500

MATTHEW KIM AND PAUL W. LADENSON

227 Adrenal Cortex

1514

LYNNETTE K. NIEMAN

228 Adrenal Medulla, Catecholamines, and Pheochromocytoma

1521 1527

JILL CRANDALL AND HARRY SHAMOON

230 Hypoglycemia and Pancreatic Islet Cell Disorders

1548

KHALID HUSSAIN

231 Polyglandular Disorders

1555

LYNNETTE K. NIEMAN AND ALLEN M. SPIEGEL

232 Neuroendocrine Tumors and the Carcinoid Syndrome

1557

KENNETH R. HANDE

233 Disorders of Sexual Development

1560

PERRIN C. WHITE

234 The Testis and Male Hypogonadism, Infertility, and Sexual Dysfunction

SECTION XXI: DISEASES OF ALLERGY AND CLINICAL IMMUNOLOGY 249 Approach to the Patient with Allergic or Immunologic Disease 250 Primary Immunodeficiency Diseases 251 Allergic Rhinitis and Chronic Sinusitis 252 Urticaria and Angioedema 253 Systemic Anaphylaxis, Food Allergy, and Insect Sting Allergy 254 Drug Allergy

1568 1579 1584

ROBERT W. REBAR AND WILLIAM H. CATHERINO

1600

KAREN FREUND

1604

BEVERLY WINIKOFF AND DANIEL GROSSMAN

1610

KAREN ROSENE-MONTELLA

1623

DEBORAH GRADY AND ELIZABETH BARRETT-CONNOR

1629

GENE FEDER AND HARRIET L. MacMILLAN

255 Mastocytosis

SECTION XXII: RHEUMATIC DISEASES 256 Approach to the Patient with Rheumatic Disease

1636 1637

THOMAS J. WEBER

244 Osteomalacia and Rickets

1645

ROBERT S. WEINSTEIN

245 The Parathyroid Glands, Hypercalcemia and Hypocalcemia

257 Laboratory Testing in the Rheumatic Diseases

SAMUEL A. WELLS, JR.

1718

DAVID S. PISETSKY

258 Imaging Studies in the Rheumatic Diseases

1723

RONALD S. ADLER

1730

SUNEEL S. APTE

260 Inherited Diseases of Connective Tissue

1733

REED E. PYERITZ

262 Osteoarthritis

1739 1744

263 Bursitis, Tendinitis, and Other Periarticular Disorders and Sports Medicine

1749

JOSEPH J. BIUNDO

264 Rheumatoid Arthritis

1754

JAMES R. O’DELL

265 The Spondyloarthropathies

1762

ROBERT D. INMAN

266 Systemic Lupus Erythematosus

1769

MARY K. CROW

267 Systemic Sclerosis (Scleroderma)

1777 1785

XAVIER MARIETTE

269 Inflammatory Myopathies

1789

STEVEN A. GREENBERG

270 The Systemic Vasculitides

1793

JOHN H. STONE

271 Polymyalgia Rheumatica and Temporal Arteritis

1801

ROBERT F. SPIERA

1649

272 Infections of Bursae, Joints, and Bones

1662

273 Crystal Deposition Diseases

RAJESH V. THAKKER

246 Medullary Thyroid Carcinoma

1712

VIVIAN P. BYKERK AND MARY K. CROW

268 Sjögren Syndrome

THOMAS J. WEBER

243 Osteoporosis

1706

CEM AKIN

JOHN VARGA

SECTION XX: DISEASES OF BONE AND MINERAL METABOLISM 242 Approach to the Patient with Metabolic Bone Disease

1703

JOEL A. BLOCK AND CARLA SCANZELLO

SECTION XIX: WOMEN’S HEALTH

241 Intimate Partner Violence

1698

LESLIE C. GRAMMER

236 Reproductive Endocrinology and Infertility

240 Menopause

1693

STEPHEN C. DRESKIN

RICHARD M. SIEGEL AND DANIEL L. KASTNER

239 Common Medical Problems in Pregnancy

1687

LARRY BORISH

261 The Systemic Autoinflammatory Diseases

238 Contraception

1677

CHARLOTTE CUNNINGHAM-RUNDLES

ROBERT W. REBAR AND WILLIAM H. CATHERINO

237 Approach to Women’s Health

1674

STEPHEN I. WASSERMAN

259 Connective Tissue Structure and Function

RONALD S. SWERDLOFF AND CHRISTINA WANG

235 Ovaries and Pubertal Development

1667

LAWRENCE B. SCHWARTZ

WILLIAM F. YOUNG, JR.

229 Diabetes Mellitus

248 Osteonecrosis, Osteosclerosis/Hyperostosis, and Other Disorders of Bone MICHAEL P. WHYTE

SECTION XVIII: ENDOCRINE DISEASES 222 Principles of Endocrinology

1664

STUART H. RALSTON

MICHAEL D. JENSEN

221 Approach to the Patient with Endocrine Disease

247 Paget Disease of Bone

xxxv

1805

ERIC L. MATTESON AND DOUGLAS R. OSMON N. LAWRENCE EDWARDS

1811

xxxvi

Contents

274 Fibromyalgia, Chronic Fatigue Syndrome, and Myofascial Pain

1817

ROBERT M. BENNETT

275 Systemic Diseases in Which Arthritis Is a Feature

1823

STERLING G. WEST

276 Surgical Treatment of Joint Diseases

1828

C. RONALD MacKENZIE AND EDWIN P. SU

1837 1838

MARTIN J. BLASER AND ILSEUNG CHO

279 Principles of Anti-Infective Therapy

1843

GEORGE M. ELIOPOULOS

280 Approach to Fever or Suspected Infection in the Normal Host

1849

JAMES E. LEGGETT

281 Approach to Fever and Suspected Infection in the Compromised Host

1854 1861

1872

S. RAGNAR NORRBY AND LINDSAY E. NICOLLE

285 Approach to the Patient with a Sexually Transmitted Infection

1918 1920 1923 1924

MICHAEL S. SIMBERKOFF

1984

313 Whooping Cough and Other Bordetella Infections

1990 1993

THOMAS J. MARRIE

1996

JEAN-MARC ROLAIN AND DIDIER RAOULT

2001

EDWARD W. HOOK III

2002

STEPHEN G. BAUM

2007

WILLIAM M. GEISLER

2013

EDWARD W. HOOK III

2020

EDWARD W. HOOK III

2021

GARY P. WORMSER

322 Relapsing Fever and Other Borrelia Infections

2027

WILLIAM A. PETRI, JR.

323 Leptospirosis

2028

ATIS MUEHLENBACHS AND SHERIF R. ZAKI

324 Tuberculosis

2030

JERROLD J. ELLNER

325 The Nontuberculous Mycobacteria

2039

STEVEN M. HOLLAND

326 Leprosy (Hansen Disease)

2042

JOEL D. ERNST

1931

327 Rickettsial Infections

2046

DIDIER RAOULT

1934

328 Zoonoses

2056

STUART LEVIN AND KAMALJIT SINGH

1940

MATTHEW R. GOLDEN AND H. HUNTER HANDSFIELD

300 Haemophilus and Moraxella Infections

1982

KENNETH L. GAGE AND PAUL S. MEAD

321 Lyme Disease 1915

DAVID S. STEPHENS

299 Neisseria Gonorrhoeae Infections

312 Plague and Other Yersinia Infections

320 Nonsyphilitic Treponematoses 1913

ITZHAK BROOK

298 Neisseria Meningitidis Infections

1979

WILLIAM SCHAFFNER

319 Syphilis 1906

DALE N. GERDING AND STUART JOHNSON

297 Diseases Caused by Non–Spore-Forming Anaerobic Bacteria

1975

EDSEL MAURICE T. SALVANA AND ROBERT A. SALATA

318 Diseases Caused by Chlamydiae

ANNETTE C. REBOLI

296 Clostridial Infections

310 Brucellosis

1902

DANIEL R. LUCEY AND LEV M. GRINBERG

295 Erysipelothrix Infections

309 Shigellosis

317 Mycoplasma Infections

BENNETT LORBER

294 Anthrax

1971

JOHN A. CRUMP

1896

ROLAND W. SUTTER

293 Listeriosis

1968

KEITH S. KAYE AND ROBERT A. BONOMO

316 Granuloma Inguinale (Donovanosis)

TRISH M. PERL

292 Diphtheria and Other Corynebacterium Infections

307 Diseases Caused by Acinetobacter and Stenotrophomonas Species

1885

DONALD E. LOW

291 Enterococcal Infections

1962

MATTHEW E. FALAGAS AND PETROS I. RAFAILIDIS

315 Bartonella Infections

LIONEL A. MANDELL

290 Nonpneumococcal Streptococcal Infections and Rheumatic Fever

306 Pseudomonas and Related Gram-Negative Bacillary Infections

1881

HENRY F. CHAMBERS

289 Streptococcus Pneumoniae Infections

1960

DAVID L. PATERSON

314 Legionella Infections

GEORGE L. DRUSANO

288 Staphylococcal Infections

305 Infections Due to Other Members of the Enterobacteriaceae, Including Management of Multidrug-Resistant Strains

1876

DAVID O. FREEDMAN

287 Antibacterial Chemotherapy

1956

ERIK L. HEWLETT

HEIDI SWYGARD AND MYRON S. COHEN

286 Approach to the Patient before and after Travel

1953

BAN MISHU ALLOS

311 Tularemia and Other Francisella Infections 1868

HERBERT L. DUPONT

284 Approach to the Patient with Urinary Tract Infection

303 Campylobacter Infections

GERALD T. KEUSCH

DAVID P. CALFEE

283 Approach to the Patient with Suspected Enteric Infection

1950

EDUARDO GOTUZZO AND CARLOS SEAS

308 Salmonella Infections (Including Enteric Fever)

KIEREN A. MARR

282 Prevention and Control of Health Care–Associated Infections

302 Cholera and Other Vibrio Infections

THEODORE S. STEINER

W. MICHAEL SCHELD

278 The Human Microbiome

1949

STANLEY M. SPINOLA

304 Escherichia Coli Enteric Infections

SECTION XXIII: INFECTIOUS DISEASES 277 Introduction to Microbial Disease: Host-Pathogen Interactions

301 Chancroid

329 Actinomycosis

2060

ITZHAK BROOK

1946

330 Nocardiosis FREDERICK S. SOUTHWICK

2062

Contents 331 Systemic Antifungal Agents

2064

DAVID A. STEVENS

332 Histoplasmosis

2070 2072

366 Coronaviruses

2075

367 Measles

2076

368 Rubella (German Measles)

2078

369 Mumps

2079

370 Cytomegalovirus, Epstein-Barr Virus, and Slow Virus Infections of the Central Nervous System

CAROL A. KAUFFMAN

337 Sporotrichosis CAROL A. KAUFFMAN

338 Candidiasis CAROL A. KAUFFMAN

339 Aspergillosis

2083

THOMAS J. WALSH

340 Mucormycosis

2087

D.P. KONTOYIANNIS

341 Pneumocystis Pneumonia

2091

JOSEPH A. KOVACS

342 Mycetoma

2099

D.P. KONTOYIANNIS

343 Dematiaceous Fungal Infections

2101

PETER G. PAPPAS

344 Antiparasitic Therapy

2103

RICHARD D. PEARSON

345 Malaria

2107

PHILIP J. ROSENTHAL AND MOSES R. KAMYA

346 African Sleeping Sickness

2113

WILLIAM A. PETRI, JR.

347 Chagas Disease

2116

LOUIS V. KIRCHHOFF

348 Leishmaniasis

2120

353 Babesiosis and Other Protozoan Diseases 354 Cestodes

2147 2153

EDGAR M. CARVALHO AND ALDO A. M. LIMA

356 Liver, Intestinal, and Lung Fluke Infections

2155

EDUARDO GOTUZZO

357 Intestinal Nematode Infections

2159

DAVID J. DIEMERT

358 Tissue Nematode Infections

2164

DAVID J. DIEMERT

359 Arthropods and Leeches

2171

DIRK M. ELSTON

360 Antiviral Therapy (Non-HIV)

2177

EDWARD E. WALSH

2214

INGER K. DAMON

373 Papillomavirus

2219

JOHN M. DOUGLAS, JR.

374 Herpes Simplex Virus Infections

2223

RICHARD J. WHITLEY

375 Varicella-Zoster Virus (Chickenpox, Shingles)

2227

JEFFREY COHEN

376 Cytomegalovirus

2229

W. LAWRENCE DREW

377 Epstein-Barr Virus Infection

2232

ROBERT T. SCHOOLEY

378 Retroviruses Other Than Human Immunodeficiency Virus

2235

WILLIAM A. BLATTNER

379 Enteroviruses

2239

JOSÉ R. ROMERO

2244

MANUEL A. FRANCO AND HARRY B. GREENBERG

2247

DANIEL G. BAUSCH

2256

STANLEY J. NAIDES

2262

THOMAS P. BLECK

SECTION XXIV: HIV AND THE ACQUIRED IMMUNODEFICIENCY SYNDROME 384 Epidemiology and Diagnosis of Human Immunodeficiency Virus Infection and Acquired Immunodeficiency Syndrome

2272

THOMAS C. QUINN

385 Immunopathogenesis of Human Immunodeficiency Virus Infection

2278

JOEL N. BLANKSON AND ROBERT F. SILICIANO

386 Biology of Human Immunodeficiency Viruses

2280

FRANK MALDARELLI

387 Prevention of Human Immunodeficiency Virus Infection

2285

388 Antiretroviral Therapy of HIV/AIDS

2287

ROY M. GULICK

2185

RONALD B. TURNER

362 Respiratory Syncytial Virus

2212

CARLOS del RIO AND MYRON S. COHEN

JOHN H. BEIGEL

361 The Common Cold

372 Smallpox, Monkeypox, and Other Poxvirus Infections

383 Arboviruses Affecting the Central Nervous System

2142

2208

NEAL S. YOUNG

2138

A. CLINTON WHITE AND ENRICO BRUNETTI

355 Schistosomiasis (Bilharziasis)

371 Parvovirus

382 Arboviruses Causing Fever and Rash Syndromes

SAM R. TELFORD III AND PETER J. KRAUSE

2206

JOSEPH R. BERGER AND AVINDRA NATH

2135

WILLIAM A. PETRI, JR. AND ALDO A.M. LIMA

2204

JOHN W. GNANN, JR.

381 Viral Hemorrhagic Fevers

THEODORE E. NASH AND DAVID R. HILL

352 Amebiasis

2202

SUSAN E. REEF

2133

ALDO A.M. LIMA AND RICHARD L. GUERRANT

351 Giardiasis

2199

MARTIN WEISSE AND MARK PAPANIA

380 Rotaviruses, Noroviruses, and Other Gastrointestinal Viruses

JOSE G. MONTOYA

350 Cryptosporidiosis

2197

SUSAN I. GERBER AND LARRY J. ANDERSON

2125

SIMON L. CROFT AND PIERRE A. BUFFET

349 Toxoplasmosis

365 Adenovirus Diseases

2074

CAROL A. KAUFFMAN

336 Cryptococcosis

2191

MICHAEL G. ISON

CAROL A. KAUFFMAN

335 Paracoccidioidomycosis

364 Influenza FREDERICK G. HAYDEN

JOHN N. GALGIANI

334 Blastomycosis

2188

GEOFFREY A. WEINBERG AND KATHRYN M. EDWARDS

CAROL A. KAUFFMAN

333 Coccidioidomycosis

363 Parainfluenza Viral Disease

xxxvii

389 Infectious and Metabolic Complications of HIV and AIDS

2292

HENRY MASUR, LETHA M. HEALEY, AND COLLEEN HADIGAN

2187

390 Gastrointestinal Manifestations of HIV and AIDS TAMSIN A. KNOX AND CHRISTINE WANKE

2302

xxxviii

Contents

391 Pulmonary Manifestations of Human Immunodeficiency Virus and the Acquired Immunodeficiency Syndrome

2305

KRISTINA CROTHERS AND ALISON MORRIS

392 Skin Manifestations in Patients with Human Immunodeficiency Virus Infection

2318 2322 2328 2332

2338 2346

JEFFREY M. LYNESS

2356

KATHLEEN B. DIGRE

399 Traumatic Brain Injury and Spinal Cord Injury

2364

GEOFFREY S.F. LING

400 Mechanical and Other Lesions of the Spine, Nerve Roots, and Spinal Cord

2370 2382

DAVID S. KNOPMAN

402 Alzheimer Disease and Other Dementias

2388

DAVID S. KNOPMAN

403 The Epilepsies

2399

SAMUEL WIEBE

404 Coma, Vegetative State, and Brain Death

2409

JAMES L. BERNAT AND EELCO F.M. WIJDICKS

405 Disorders of Sleep

2415

BRADLEY V. VAUGHN

406 Approach to Cerebrovascular Diseases

2424

LARRY B. GOLDSTEIN

407 Ischemic Cerebrovascular Disease

2434 2445

STEPHAN A. MAYER

409 Parkinsonism

2454

ANTHONY E. LANG

410 Other Movement Disorders

2461

ANTHONY E. LANG

411 Multiple Sclerosis and Demyelinating Conditions of the Central Nervous System

2471

PETER A. CALABRESI

412 Meningitis: Bacterial, Viral, and Other

2480

AVINDRA NATH

413 Brain Abscess and Parameningeal Infections

2495

AVINDRA NATH AND JOSEPH BERGER

414 Acute Viral Encephalitis

2500

ALLEN J. AKSAMIT, JR.

415 Prion Diseases

2504

PATRICK J. BOSQUE

416 Nutritional and Alcohol-Related Neurologic Disorders

2506

BARBARA S. KOPPEL

417 Congenital, Developmental, and Neurocutaneous Disorders

WILLIAM P. CHESHIRE, JR.

424 Neuro-Ophthalmology

2573 2579

TROY E. DANIELS AND RICHARD C. JORDAN

426 Approach to the Patient with Nose, Sinus, and Ear Disorders

2585

ANDREW H. MURR

427 Smell and Taste

2592

ROBERT W. BALOH AND JOANNA C. JEN

428 Hearing and Equilibrium

2593

ROBERT W. BALOH AND JOANNA C. JEN

2601

PAUL W. FLINT

SECTION XXVII: MEDICAL CONSULTATION 430 Principles of Medical Consultation

2608

GERALD W. SMETANA

431 Preoperative Evaluation

2611

STEVEN L. COHN

432 Overview of Anesthesia

2617

JEANINE P. WIENER-KRONISH AND LEE A. FLEISHER

433 Postoperative Care and Complications

2621

DONALD A. REDELMEIER

434 Medical Consultation in Psychiatry

2625

PETER MANU

SECTION XXVIII: SKIN DISEASES 435 Structure and Function of the Skin

2632

436 Examination of the Skin and an Approach to Diagnosing Skin Diseases

2637

JAMES C. SHAW

437 Principles of Therapy of Skin Diseases

2657

VICTORIA P. WERTH

438 Eczemas, Photodermatoses, Papulosquamous (Including Fungal) Diseases, and Figurate Erythemas

2662

HENRY W. LIM

439 Macular, Papular, Vesiculobullous, and Pustular Diseases

2671

NEIL J. KORMAN

440 Urticaria, Drug Hypersensitivity Rashes, Nodules and Tumors, and Atrophic Diseases

2683

MADELEINE DUVIC

441 Infections, Hyperpigmentation and Hypopigmentation, Regional Dermatology, and Distinctive Lesions in Black Skin

2695

JEAN BOLOGNIA

442 Diseases of Hair and Nails

2703

ANTONELLA TOSTI

2512

APPENDIX-LABORATORY REFERENCE INTERVALS AND VALUES

2517

Reference Intervals and Laboratory Values

JONATHAN W. MINK

418 Autonomic Disorders and Their Management

2556

DAVID H. CHU

LARRY B. GOLDSTEIN

408 Hemorrhagic Cerebrovascular Disease

423 Diseases of the Visual System

429 Throat Disorders

RICHARD L. BARBANO

401 Regional Cerebral Dysfunction: Higher Mental Functions

2547

SECTION XXVI: EYE, EAR, NOSE, AND THROAT DISEASES

425 Diseases of the Mouth and Salivary Glands

ROBERT C. GRIGGS, RALPH F. JÓZEFOWICZ, AND MICHAEL AMINOFF

398 Headaches and Other Head Pain

422 Disorders of Neuromuscular Transmission

ROBERT W. BALOH AND JOANNA C. JEN

SECTION XXV: NEUROLOGY 397 Psychiatric Disorders in Medical Practice

2537

DUYGU SELCEN

MYRON YANOFF AND J. DOUGLAS CAMERON

ROBERT COLEBUNDERS AND MARTYN A. FRENCH

396 Approach to the Patient with Neurologic Disease

2527

MICHAEL E. SHY

AMELIA EVOLI AND ANGELA VINCENT

JOSEPH R. BERGER AND AVINDRA NATH

395 Immune Reconstitution Inflammatory Syndrome in HIV/AIDS

420 Peripheral Neuropathies 421 Muscle Diseases

THOMAS S. ULDRICK AND ROBERT YARCHOAN

394 Neurologic Complications of Human Immunodeficiency Virus Infection

2522

PAMELA J. SHAW

TOBY MAURER

393 Hematology and Oncology in Patients with Human Immunodeficiency Virus Infection

419 Amyotrophic Lateral Sclerosis and Other Motor Neuron Diseases

RONALD J. ELIN

2712

VIDEO CONTENTS This icon appears throughout the book to indicate chapters with accompanying video available on ExpertConsult.com. For quick viewing, use your smartphone to scan the QR codes in the front of the book. Aging and Geriatric Medicine Confusion Assessment Method (CAM) Video 28-1 – MARCOS MIALNEZ, JORGE G. RUIZ, AND ROSANNE M. LEIPZIG

Clinical Pharmacology Interlaminar Epidural Steroid Injection Video 30-1 – ALI TURABI

Cardiovascular Disease Standard Echocardiographic Views: Long Axis Image Plane Video 55-1A – CATHERINE M. OTTO Standard Echocardiographic Views: Short Axis Image Plane Video 55-1B – CATHERINE M. OTTO Standard Echocardiographic Views: Short Axis Image Plane Video 55-1C – CATHERINE M. OTTO Standard Echocardiographic Views: Four-Chamber Image Plane Video 55-1D – CATHERINE M. OTTO Dilated Cardiomyopathy: Long Axis View Video 55-2A – CATHERINE M. OTTO Dilated Cardiomyopathy: Short Axis View Video 55-2B – CATHERINE M. OTTO Dilated Cardiomyopathy: Apical Four-Chamber View Video 55-2C – CATHERINE M. OTTO Three-Dimensional Echocardiography Video 55-3 – CATHERINE M. OTTO Stress Echocardiography: Normal Reaction Video 55-4A – CATHERINE M. OTTO Stress Echocardiography: Normal Reaction Video 55-4B – CATHERINE M. OTTO Stress Echocardiography: Proximal Stenosis of the Left Anterior Descending Coronary Artery Video 55-4C – CATHERINE M. OTTO Stress Echocardiography: Proximal Stenosis of the Left Anterior Descending Coronary Artery Video 55-4D – CATHERINE M. OTTO Pericardial Effusion: Parasternal Long Axis Video 55-5A – CATHERINE M. OTTO Pericardial Effusion: Parasternal Short Axis Video 55-5B – CATHERINE M. OTTO Pericardial Effusion: Apical Four-Chamber Views Video 55-5C – CATHERINE M. OTTO Secundum Atrial Septal Defect Video 69-1 – ARIANE J. MARELLI Perimembranous Ventricular Septal Defect Video 69-2 – ARIANE J. MARELLI Coronary Stent Placement Video 74-1 – PAUL S. TEIRSTEIN Guidewire Passage Video 74-2 – PAUL S. TEIRSTEIN Delivering the Stent Video 74-3 – PAUL S. TEIRSTEIN Inflating the Stent Video 74-4 – PAUL S. TEIRSTEIN

Final Result Video 74-5 – PAUL S. TEIRSTEIN Superficial Femoral Artery (SFA) Stent Procedure Video 79-1 – CHRISTOPHER J. WHITE Orthotopic Bicaval Cardiac Transplantation Video 82-1 – Y. JOSEPH WOO

Respiratory Diseases Wheezing Video 87-1 – JEFFREY M. DRAZEN Technique for Use of a Metered-Dose Inhaler Video 87-2 – LESLIE HENDELES and the New England Journal of Medicine VATS Wedge Resection Video 101-1 – MALCOLM M. DeCAMP

Critical Care Medicine Ventilation of an Ex Vivo Rat Lung Video 105-1 – ARTHUR S. SLUTSKY, GEORGE VOLGYESI, AND TOM WHITEHEAD

Renal and Genitourinary Diseases Renal Artery Stent Video 125-1 – RENATO M. SANTOS AND THOMAS D. DUBOSE, JR. Interpretation of a Computed Tomographic Colonography Video 133-1 – DAVID H. KIM Donor Liver Transplantation—Donor and Recipient Video 154-1 – IGAL KAM, THOMAS BAK, AND MICHAEL WACHS

Oncology Snare Polypectomy of a Colon Adenoma Video 193-1 – DOUGLAS O. FAIGEL Laparoscopic-Assisted Double Balloon Enteroscopy with Polypectomy of a Jejunal Adenoma Followed by Surgical Oversew of the Polypectomy Site Video 193-2 – DOUGLAS O. FAIGEL Endoscopic Mucosal Resection Using Saline Lift Polypectomy of a Colon Adenoma Followed by Closure of the Mucosal Defect with Clips Video 193-3 – DOUGLAS O. FAIGEL Endoscopic View of Rectal Cancer Video 193-4 – DOUGLAS O. FAIGEL Endoscopic Ultrasound Video 193-5 – DOUGLAS O. FAIGEL

Nutritional Diseases Laparoscopic Roux-en-Y Gastric Bypass Video 220-1 – JAMES M. SWAIN

Endocrine Diseases Pituitary Surgery Video 224-1 – IVAN CIRIC

Diseases of Allergy and Clinical Immunology Skin Testing Video 251-1 – LARRY BORISH Nasal Endoscopy Video 251-2 – LARRY BORISH

xl

Video Contents

Rheumatic Diseases Hip Arthroscopy Osteochondroplasty Video 276-1 – BRYAN T. KELLY

Neurology Cervical Provocation Video 400-1 – RICHARD L. BARBANO Spurling Maneuver Video 400-2 – RICHARD L. BARBANO Cervical Distraction Test Video 400-3 – RICHARD L. BARBANO Straight Leg Raise Video 400-4 – RICHARD L. BARBANO Contralateral Straight Leg Raise Video 400-5 – RICHARD L. BARBANO Seated Straight Leg Raise Video 400-6 – RICHARD L. BARBANO Discectomy Video 400-7 – JASON H. HUANG Absence Seizure Video 403-1 – SAMUEL WIEBE Left Rolandic Seizure Video 403-2 – SAMUEL WIEBE Left Temporal Complex Partial Seizure Video 403-3 – SAMUEL WIEBE Left Temporal Complex Partial Seizure Postictal Confusion Video 403-4 – SAMUEL WIEBE Left Temporal Complex Partial Seizure Video 403-5 – SAMUEL WIEBE Supplementary Sensory-Motor Seizure Video 403-6 – SAMUEL WIEBE Right Posterior Temporal Seizure-Dramatic Frontal Semiology Video 403-7 – SAMUEL WIEBE Right Mesial Frontal Seizure Video 403-8 – SAMUEL WIEBE Nonconvulsive Status Epilepticus Video 403-9 – SAMUEL WIEBE GTC Seizure Tonic Phase Video 403-10 – SAMUEL WIEBE GTC Seizure Clonic Phase Video 403-11 – SAMUEL WIEBE Myoclonic Facial Seizure Video 403-12 – SAMUEL WIEBE Tonic Seizure Lennox Gastaut Video 403-13 – SAMUEL WIEBE Atonic Seizure Lennox Gastaut Video 403-14 – SAMUEL WIEBE Reflex Auditory Seizure Video 403-15 – SAMUEL WIEBE Four Score Video 404-1 – JAMES L. BERNAT AND EELCO F.M. WIJDICKS Persistent Vegetative State Video 404-2 – JAMES L. BERNAT AND EELCO F.M. WIJDICKS Minimally Conscious State Video 404-3 – JAMES L. BERNAT AND EELCO F.M. WIJDICKS Akinetic Mutism Video 404-4 – JAMES L. BERNAT AND EELCO F.M. WIJDICKS Early Parkinson’s Disease Video 409-1 – ANTHONY E. LANG Freezing of Gait in Parkinson’s Disease Video 409-2 – ANTHONY E. LANG

Gunslinger Gait in Progressive Supranuclear Palsy Video 409-3 – ANTHONY E. LANG Supranuclear Gaze Palsy in Progressive Supranuclear Palsy Video 409-4 – ANTHONY E. LANG Applause Sign in Progressive Supranuclear Palsy Video 409-5 – ANTHONY E. LANG Apraxia of Eyelid Opening in Progressive Supranuclear Palsy Video 409-6 – ANTHONY E. LANG Cranial Dystonia in Multiple System Atrophy Video 409-7 – ANTHONY E. LANG Anterocollis in Multiple System Atrophy Video 409-8 – ANTHONY E. LANG Stridor in Multiple System Atrophy Video 409-9 – ANTHONY E. LANG Alien Limb Phenomenon in Corticobasal Syndrome Video 409-10 – ANTHONY E. LANG Myoclonus in Corticobasal Syndrome Video 409-11 – ANTHONY E. LANG Levodopa-Induced Dyskinesia in Parkinson’s Disease Video 409-12 – ANTHONY E. LANG Essential Tremor Video 410-1 – ANTHONY E. LANG Huntington’s Disease Video 410-2 – ANTHONY E. LANG Hemiballism Video 410-3 – ANTHONY E. LANG Blepharospasm Video 410-4 – ANTHONY E. LANG Oromandibular Dystonia Video 410-5 – ANTHONY E. LANG Cervical Dystonia Video 410-6 – ANTHONY E. LANG Writer’s Cramp Video 410-7 – ANTHONY E. LANG Embouchure Dystonia Video 410-8 – ANTHONY E. LANG Sensory Trick in Cervical Dystonia Video 410-9 – ANTHONY E. LANG Generalized Dystonia Video 410-10 – ANTHONY E. LANG Tics Video 410-11 – ANTHONY E. LANG Tardive Dyskinesia Video 410-12 – ANTHONY E. LANG Hemifacial Spasm Video 410-13 – ANTHONY E. LANG Wernicke Encephalopathy Eye Movements: Before Thiamine Video 416-1 – BARBARA S. KOPPEL Wernicke Encephalopathy Eye Movements: After Thiamine Video 416-2 – BARBARA S. KOPPEL Limb Symptoms and Signs Video 419-1 – PAMELA J. SHAW Bulbar Symptoms and Signs Video 419-2 – PAMELA J. SHAW Normal Swallowing Video 419-3 – PAMELA J. SHAW Charcot-Marie-Tooth Disease Exam and Walk Video 420-1 – MICHAEL E. SHY

XVI

METABOLIC DISEASES 205 APPROACH TO INBORN ERRORS OF METABOLISM

206 DISORDERS OF LIPID METABOLISM 207 GLYCOGEN STORAGE DISEASES

208 LYSOSOMAL STORAGE DISEASES

211 WILSON DISEASE

209 HOMOCYSTINURIA AND

212 IRON OVERLOAD

HYPERHOMOCYSTEINEMIA

210 THE PORPHYRIAS

(HEMOCHROMATOSIS)

1384

CHAPTER 205  Approach to Inborn Errors of Metabolism  

205  APPROACH TO INBORN ERRORS OF METABOLISM OLAF A. BODAMER

DEFINITION

The term metabolism (Greek: metabolé, “change”) refers to the network of chemical reactions that sustain the human organism through the digestion, absorption, transport, and utilization of nutrients. Inborn errors of metabolism are genetic disorders that affect these intrinsic metabolic pathways through deficiencies of enzymes, membrane transporter proteins, signaling peptides, or structural proteins. The resulting clinical phenotype follows a spectrum of different organ manifestations that may be progressive, fluctuating, or stationary in nature and may be manifested at any age. Any inborn error of metabolism can principally be manifested during adolescence or adulthood, although severe presentations are typically recognized during infancy and childhood.

HISTORY

Archibald Garrod pioneered the field of inborn errors of metabolism after recognizing alkaptonuria as one of the first metabolic conditions due to homozygosity of mutant alleles in 1902. He had the foresight to recognize the autosomal recessive inheritance of additional inborn errors of metabolism, including cystinuria, pentosuria, and albinism, and to speculate about “chemical individuality” as one of the driving forces of selection and evolution. However, it was not until the early 1950s that the deficiency of homogentisate 1,2-dioxygenase (HGD) was recognized as the underlying cause of alkaptonuria, and it took many more years to identify pathogenic mutations in the HGD gene. The advent of novel analytical techniques led to the molecular and biochemical characterization of known inborn errors of metabolism and the delineation and recognition of new clinical phenotypes, some of which were previously not presumed to be due to inborn errors of metabolism. The completion of the first human genome in 2001 and the following “genomics” revolution laid the foundation for the successive identification of many additional inborn errors of metabolism through next-generation sequencing, bringing the total number of catalogued inborn errors of metabolism to more than 1500 (March 2014). The initiation of population-based newborn screening in 1964 through Robert Guthrie resulted in its recognition as an important public health measure to prevent morbidity and mortality of inborn errors of metabolism. More than 4 million newborn infants are screened annually in the United States for 31 core conditions, including mostly inborn errors of metabolism. As a consequence, approximately 12,500 newborn infants are diagnosed each year through newborn screening. Rarely, mothers with an inborn error of metabolism are diagnosed through newborn screening of their infants subsequent to placental transfer of pathognomonic metabolites.

EPIDEMIOLOGY

Inborn errors of metabolism occur in all populations, although their incidence and prevalence rates may vary considerably because of differences in carrier rates. These variations are readily explained by the presence of founder mutations, for example, in individuals of Ashkenazi Jewish or Amish ancestry, or by an increased rate of parental consanguinity that leads to a relative increase in mutant allele frequency (Table 205-1). Knowledge of the increased carrier frequencies is instrumental for preconception genetic counseling and targeted carrier screening.

PATHOBIOLOGY

The complexity of human metabolism and its spatial relationship with the human proteome, genome, and methylome are poorly understood. Naturally occurring variants in human nucleotide sequences may or may not result in variation of amino acid sequences in peptides and proteins. It is now well established from whole exome and genome sequencing that individuals may carry in excess of 10,000 nucleotide variants; most variants are silent, singlenucleotide polymorphic variants. Up to 4% of variants may be pathogenic in

either recessive or dominant genes. These variants in particular will lead to functional changes in proteins that may render the affected individual susceptible to disease, increase the risk for undesired side effects on treatment with certain drugs, or increase the risk for genetic conditions in future generations. Variation of human peptides and proteins is not merely explained through genomic sequence variation. Post-transcriptional alternative splicing will generate tissue-specific isoforms of proteins that are adapted to their functional needs through post-translational modification and conformational plasticity.

Genetics

Inborn errors of metabolism are monogenic conditions that follow autosomal recessive or dominant, X-linked recessive or dominant, or mitochondrial inheritance patterns. Of note is the existence of genetic or environmental modifiers that contribute to the interindividual and intrafamilial variability of phenotypic expression, although for most inborn errors of metabolism, these modifiers remain elusive. In case of mitochondrial inheritance, heteroplasmy (the random distribution and expression of mitochondrial mutations in different organs) may explain by itself the striking variability of clinical symptoms in mitochondrial conditions. The concept of synergistic heterozygosity (i.e., heterozygosity for pathogenic mutations affecting different enzymes simultaneously within the same pathway) may explain why some individuals with symptoms reminiscent of inborn errors of metabolism are not formally diagnosed.

Pathophysiology

The severity of any given inborn error of metabolism depends on the degree of enzyme deficiency and the complex interaction of the underlying pathogenic mutations, genetic modifiers, and environment. Hypomorphic mutations may not lead to overt disease until adulthood, whereas severe mutations in the same gene may lead to infantile-onset disease associated with significant morbidity and mortality. The underlying pathophysiologic mechanisms may contribute individually or in combination to the disease state (Table 205-2). Complete blockage of a catabolic pathway may result in accumulation of toxic substrates, activation of secondary minor pathways, or a relative shortage of downstream products. As a consequence, different organs may be affected by the same metabolic defect. An example is homocystinuria due to mutations in the gene for cystathionine β-synthase, which causes lens dislocation and intellectual disabilities and increases the risk for cardiovascular disease. Accumulation of homocysteine contributes to the vascular risk, whereas lack of the downstream product cysteine is an important factor in the dislocation of the lens through loosening of the zonular fibers (Table 205-3).

Clinical Phenotype

Inborn errors of metabolism typically affect multiple organs and, in more than 50% of cases, the central and peripheral nervous systems and muscles. One or more organ manifestations may dominate the clinical phenotype, although oligosymptomatic cases may occur. The clinical phenotype represents a continuous clinical spectrum ranging from the severe end, presenting during infancy, to the mild end of the spectrum, presenting during adolescence or adulthood. Some affected individuals may never come to medical attention because of almost complete absence of symptoms or atypical presentation. Recent data from newborn screening programs suggest much higher incidence rates for some inborn errors of metabolism due to the detection of a high rate of mild cases in which disease-related signs or symptoms may never develop. Some clinical signs are pathognomonic for an inborn error of metabolism, whereas others should raise the suspicion for the presence of an inborn error of metabolism (Table 205-4).

Classification

Inborn errors of metabolism can be classified on the basis of the underlying pathomechanism (see Table 205-2), the nature or localization of the protein involved (see Table 205-3), or the clinical phenotype (see Table 205-4). The most logical classification is based on the nature or localization of the affected protein and pathway.

INBORN ERRORS OF METABOLISM

Disorders of Protein Metabolism

These conditions are due to cytosolic or mitochondrial enzyme deficiencies affecting mostly catabolic pathways (Table 205-5). Disorders of protein

1385

CHAPTER 205  Approach to Inborn Errors of Metabolism  

TABLE 205-1  INCIDENCE OF INBORN ERRORS OF METABOLISM DISORDER

GENE

INCIDENCE*

CARRIER RATE

POPULATION

Familial hypercholesterolemia

LDLR

Phenylketonuria

PAH

1 : 4000 100 mg/day triiodothyronine. These deficiency. Maternal iodine to an individual who was formerly hormones circulate largely bound deficiency leads to fetal deficiency, deficient occasionally induces to thyroxine-binding globulin. which produces spontaneous hyperthyroidism. [1.1 mg] They modulate resting energy abortions, stillbirths, expenditure and, in the hypothyroidism, cretinism, and developing human, growth and dwarfism. Permanent cognitive development. deficits may result from iodine deficiency during the first 2 years of life. In the adult, compensatory hypertrophy of the thyroid (goiter) occurs along with varying degrees of hypothyroidism. [150 µg]

Iodine status of a population can be estimated by the prevalence of goiter. Urinary excretion of iodine is an effective laboratory means of assessment. Thyroid-stimulating hormone blood level is an indirect and therefore not entirely specific means of assessment.

Iron

Conveys the capacity to participate Iron deficiency is the most common in redox reactions to a number of micronutrient deficiency in the metalloproteins, such as world. Women of childbearing age hemoglobin, myoglobin, are the group at highest risk cytochrome enzymes, and many because of menstrual blood losses, oxidases and oxygenases. The pregnancy, and lactation. The primary storage form of iron is classic deficiency syndrome is ferritin and, to a lesser degree, hypochromic, microcytic anemia. hemosiderin. Intestinal Glossitis and koilonychia (“spoon” absorption is 15-20% for “heme” nails) are also observed. Easy iron and 1-8% for iron contained fatigability often is an early in vegetables. Absorption of the symptom, before anemia appears. latter form is enhanced by the In children, mild deficiency of ascorbic acid in foodstuffs; by insufficient severity to cause poultry, fish, or beef; and by an anemia is associated with iron-deficient state. It is behavioral disturbances and decreased by phytate and tannins. poor school performance. [postmenopausal F and M: 8 mg; premenopausal F: 18 mg]

Manganese

Toxicity by oral ingestion is unknown in A component of several Manganese deficiency in the human humans. Toxic inhalation causes metalloenzymes. Most has not been conclusively hallucinations, other alterations in manganese is in mitochondria, demonstrated. It is said to cause mentation, and extrapyramidal where it is a component of hypocholesterolemia, weight loss, movement disorders. [11 mg] manganese superoxide dismutase. hair and nail changes, dermatitis, and impaired synthesis of vitamin K–dependent proteins. [F: 1.8 mg; M: 2.3 mg]

Until the deficiency syndrome is better defined, an appropriate measure of status will be difficult to develop.

A probable case of human deficiency Toxicity not well described in humans, although it may interfere with copper is described as being secondary to metabolism at high doses. [2 mg] parenteral administration of sulfite and resulted in hyperoxypurinemia, hypouricemia, and low sulfate excretion. [45 µg]

Laboratory means of assessment are not meaningful until the deficiency syndrome is better described.

Molybdenum A cofactor in several enzymes, most prominently xanthine oxidase and sulfite oxidase

Iron overload typically occurs when Negative iron balance initially leads habitual dietary intake is extremely to depletion of iron stores in the high, intestinal absorption is excessive, bone marrow; a bone marrow repeated parenteral administration biopsy and the concentration of occurs, or a combination of these serum ferritin are accurate factors exists. Excessive iron stores indicators of early depletion. As usually accumulate in the the severity of deficiency reticuloendothelial tissues and cause proceeds, serum iron (SI) little damage (hemosiderosis). If decreases and total iron-binding overload continues, iron eventually capacity (TIBC) increases; an begins to accumulate in tissues such as iron saturation (SI/TIBC) of the hepatic parenchyma, pancreas, 60% suggest iron recessive trait. Excessive intestinal overload, although systemic absorption of iron is seen in inflammation elevates serum homozygotes. [45 mg] ferritin regardless of iron status.

1452

CHAPTER 218  Vitamins, Trace Minerals, and Other Micronutrients  

TABLE 218-3  NUTRITIONAL TRACE ELEMENTS AND THEIR CLINICAL IMPLICATIONS—cont’d BIOCHEMISTRY AND PHYSIOLOGY

TOXICITY [TUL†]

DEFICIENCY [RDA*]

ASSESSMENT OF STATUS

Selenium

Erythrocyte glutathione peroxidase Toxicity is associated with nausea, Most dietary selenium is in the form Deficiency is rare in North America activity and plasma or whole diarrhea, alterations in mental status, of an amino acid complex. Nearly but has been observed in blood selenium concentrations are peripheral neuropathy, and loss of hair complete absorption of such individuals on long-term TPN the most commonly used and nails; such symptoms were forms occurs. Homeostasis is lacking selenium. Such individuals methods of assessment. They are observed in adults who inadvertently largely performed by the kidney, have myalgias or cardiomyopathies. consumed 27-2400 mg. [400 µg] moderately accurate indicators of which regulates urinary excretion Populations in some regions of the status. as a function of selenium status. world, most notably some parts of Selenium is a component of China, have marginal intake of several enzymes, most notably selenium. In these regions Keshan’s glutathione peroxidase and disease, a condition characterized superoxide dismutase. These by cardiomyopathy, is endemic; it enzymes protect against oxidative can be prevented (but not treated) and free radical damage of by selenium supplementation. [55 µg] various cell structures. The antioxidant protection conveyed by selenium apparently operates in conjunction with vitamin E because deficiency of one seems to potentiate damage induced by a deficiency of the other. Selenium also participates in the enzymatic conversion of thyroxine to its more active metabolite, triiodothyronine.

Zinc

No accurate indicators of zinc status Intestinal absorption occurs by a Zinc deficiency has its most profound Acute zinc toxicity can usually be induced by ingestion of >200 mg of exist for routine clinical use. specific process that is enhanced effect on rapidly proliferating zinc in a single day (in adults). It is Plasma, red blood cell, and hair by pregnancy and corticosteroids tissues. Mild deficiency: growth manifested by epigastric pain, nausea, zinc concentrations are often and diminished by coingestion of retardation in children. More severe vomiting, and diarrhea. Hyperpnea, misleading. Acute illness, in phytates, phosphates, iron, deficiency: growth arrest, diaphoresis, and weakness may follow particular, is known to diminish copper, lead, or calcium. teratogenicity, hypogonadism and inhalation of zinc fumes. Copper and plasma zinc levels, in part by Diminished intake of zinc leads infertility, dysgeusia, poor wound zinc compete for intestinal absorption: inducing a shift of zinc out of the to an increased efficiency of healing, diarrhea, dermatitis on the long-term ingestion of >25 mg/day of plasma compartment and into the absorption and decreased fecal extremities and around orifices, zinc may lead to copper deficiency. liver. Functional tests that excretion, providing a means of glossitis, alopecia, corneal Long-term ingestion of >150 mg/day determine dark adaptation, taste zinc homeostasis. Zinc is a clouding, loss of dark adaptation, has been reported to cause gastric acuity, and rate of wound healing component of more than 100 and behavioral changes. Impaired erosions, low high-density lipoprotein lack specificity. enzymes, among which are DNA cellular immunity is observed. cholesterol levels, and impaired polymerase, RNA polymerase, Excessive loss of gastrointestinal cellular immunity. [40 mg] and transfer RNA synthetase. secretions through chronic diarrhea and fistulas may precipitate deficiency. Acrodermatitis enteropathica is a rare, recessively inherited disease in which intestinal absorption of zinc is impaired. [F: 8 mg; M: 11 mg]

*Recommended daily allowance (RDA) established for female (F) and male (M) adults by the U.S. Food and Nutrition Board, 1999-2001. In some instances, insufficient data exist to establish an RDA, in which case the adequate intake (AI) established by the board is listed. † Tolerable upper limit (TUL) established for adults by the U.S. Food and Nutrition Board, 1999-2001.

A, E, and C to meet the metabolic demands incurred by milk production in addition to the aforementioned needs observed in pregnancy. Aside from its general role in supporting the rapid proliferation of placental and fetal tissues, folate plays a specific role in the prevention of particular birth defects. A 20 to 85% reduction in births complicated by neural tube defects (NTDs, i.e., spina bifida and anencephaly) has been realized by providing women with a daily supplement of folic acid in the form of supplements or fortified foods. The optimal dose is not well defined, but 200 to 400 µg/day clearly affords a substantial degree of protection. Populations with a high background rate of NTD births attain the largest reductions in NTDs from supplemental folate. However, because the nascent neural tube closes about day 20 after conception, the additional folate must be provided before this time to be effective.

Infancy

Infancy carries particular vulnerabilities to specific micronutrient inadequacies. Healthy infants in the United States are typically supplemented with vitamin K at birth and with iron and vitamin D during the course of the first year because of their particular susceptibility to deficiencies of these nutrients.

Women of Childbearing Age

The ability to maintain adequate iron status from menarche through menopause is compromised in women by the additional losses incurred by menstruation, pregnancy, and lactation. Therefore, it is not surprising that the population subset that almost invariably displays the highest rate of iron deficiency is women of childbearing age.

Elderly Persons

Specific dietary recommendations for elderly people have been formally incorporated into the recommended dietary allowances (RDA) because aging has an impact on the need for certain micronutrients. Vitamin B12 status declines significantly with aging, in large part because of the high prevalence of atrophic gastritis and its associated impairment in protein-bound vitamin B12 absorption.2 Estimates suggest that 10 to 20% of the elderly population is at risk for clinically significant vitamin B12 deficiency. Consequently, elderly persons should consume some of their vitamin B12 requirement in the crystalline form rather than solely from the naturally occurring protein-bound forms found in food because absorption of the crystalline form is not impaired by atrophic gastritis. Elderly people also require greater quantities of vitamins B6 and D to maintain health compared with younger adults, as reflected in

CHAPTER 218  Vitamins, Trace Minerals, and Other Micronutrients  

the new RDAs (see Table 218-2). For instance, the RDA of vitamin D in persons older than 70 years is now set at 20 µg/day (800 IU), as opposed to adults who are 70 years of age or younger, whose RDA is 15 µg/day.3,4 This increased need appears to result from diminished cutaneous synthesis of vitamin D by senile skin and from decreased sun exposure, which appears to be particularly important in elders residing in institutional facilities. The need for crystalline vitamin B12 and for a quantity of vitamin D that is difficult to achieve without resorting to a supplement suggests that universal use of a daily supplement pill containing these nutrients would benefit elderly people. Widespread use of a multivitamin that contains a broad spectrum of micronutrients is more controversial, in part because of concerns about subtle toxicity. For example, elders with chronic renal failure appear to have a vulnerability to vitamin A toxicity, suggesting that use of supplements containing this vitamin is contraindicated.

PATHOPHYSIOLOGIC AND PHARMACOLOGIC FACTORS

Diseases of the Gastrointestinal Tract

Malabsorption and maldigestion predispose to multiple micronutrient deficiencies. Both fat- and water-soluble micronutrients (except vitamin B12) are absorbed predominantly in the proximal small intestine. Therefore, diffuse mucosal diseases affecting the proximal portion of the gastrointestinal tract are likely to result in deficiencies. Even in the absence of mucosal disease of the proximal small intestine, extensive ileal disease, small bowel bacterial overgrowth, and chronic cholestasis can each interfere with the maintenance of adequate intraluminal conjugated bile acid concentrations and thereby impair absorption of fat-soluble vitamins. Maldigestion is usually the result of chronic pancreatitis. Untreated, it frequently causes malabsorption and deficiencies of fat-soluble vitamins. Vitamin B12 malabsorption can often be demonstrated in this setting, a result of inadequate R-protein digestion, but clinical vitamin B12 deficiency is rarely reported.

Inborn Errors of Metabolism

Myriad rare inborn errors of metabolism have been described for vitamins and minerals that impair an individual’s ability to assimilate, to use, or to retain a particular micronutrient (Chapter 205). Such defects are usually partial and can often be overcome, to a certain extent, by administering doses of the nutrient that are several orders of magnitude greater than usually required. Suspicion for such defects should be entertained if a known defect exists in the family, a deficiency syndrome arises at birth or during infancy, or the deficiency syndrome is present despite adequate dietary intake and the absence of any disease that would impair the ability to assimilate the nutrient.

Medications

Long-term administration of many drugs may adversely affect micronutrient status. The manner in which drug-nutrient interactions occur varies; some of the more common mechanisms are outlined in Table 218-4. Some drugs exert their therapeutic effects by specifically inhibiting the actions of a micronutrient. Examples include coumarin, which inhibits γ-carboxylation

TABLE 218-4  DRUG-MEDIATED EFFECTS ON MICRONUTRIENT STATUS: EXAMPLES DRUG

NUTRIENT

MECHANISM OF INTERACTION

Dextroamphetamine, Potentially all Induces anorexia fenfluramine, levodopa micronutrients Cholestyramine

Vitamin D, folate

Adsorbs nutrient, decreases absorption

Omeprazole

Vitamin B12

Modest bacterial overgrowth, decreases gastric acid, impairs absorption

Sulfasalazine

Folate

Impairs absorption and inhibits folate-dependent enzymes

Isoniazid

Pyridoxine

Impairs utilization of B6

Nonsteroidal antiinflammatory drugs

Iron

Gastrointestinal blood loss

Penicillamine

Zinc

Increases renal excretion

1453

reactions mediated by vitamin K, and methotrexate, which binds tightly to dihydrofolate reductase, thereby inhibiting folate metabolism.

Toxins

Tobacco smoking alters the metabolism of several vitamins, including folate and vitamins C and E. In large surveys, diminished plasma levels of folate and ascorbic acid have been observed in chronic smokers. Smoking is also associated with diminished levels of folate in cells of the oral mucosa, diminished ascorbic acid levels in leukocytes, and decreased concentrations of vitamin E in the alveolar fluid, findings providing evidence that many tissues can be affected by smoking and that the effect does not simply represent a shift of these micronutrients out of the plasma compartment.

ADVANCES IN NUTRITIONAL SCIENCE

New Frontiers in Marginal Deficiency States of Micronutrients Does Optimal Intake of Micronutrients Optimize Health?

Updating the definition of a micronutrient deficiency and establishing recommended daily intakes that are consistent with the most recent evidence have proved difficult for several reasons. In some instances, a novel biochemical or physiologic role for a nutrient has been identified but the question that arises is whether optimization of such functions translates into optimization of health. For example, providing supplemental vitamin E to elderly individuals whose vitamin E status falls within normative standards enhances T-lymphocyte responsiveness; nevertheless, it is unclear whether this translates into diminished infection rates. Another difficult problem pertains to the use of micronutrients in supraphysiologic quantities that exceed all conventional concepts of what is necessary for health. Some micronutrients, when they are taken in large quantities, have effects on physiologic functions that impart apparent health benefits. The ingestion of gram quantities of niacin to reduce low-density lipoprotein (LDL) cholesterol is an example. Such physiologic effects are not observed at more conventional levels of intake and are therefore usually considered pharmacologic effects of the nutrient. Thus, the determination of optimal nutrient intake is highly dependent on which physiologic effect is sought. Furthermore, if only a segment of the population will benefit from supraphysiologic quantities of a nutrient, should dietary guidelines for the remainder of the population be established according to this effect? Determining an adequate level of intake implies the existence of a means of measuring nutrient status. In seeking an appropriate measure of nutrient status, the diversity of function often makes it difficult to decide which measurement is the most germane. Tobacco smoking, for example, diminishes vitamin E levels in alveolar fluid but not in the serum. Thus, the concepts of localized nutrient deficiencies and tissue-specific requirements add an additional level of complexity to the determination of nutrient status.

Redefinition of Nutritional Requirements Folate

An example of the complexities that have arisen in redefining the criteria for vitamin deficiencies and vitamin requirements is the water-soluble vitamin folate. In the past, guidelines regarding its necessary intake were straightforward because they were based solely on the prevention of megaloblastic anemia. Measurement of serum and erythrocyte folate concentrations was the most common means of assessing status, and maintaining these levels within accepted normative ranges provided assurance that folate status was adequate to prevent anemia. However, degrees of deficiency that are insufficient to cause anemia may still disturb normal biochemical and physiologic homeostasis and, in some instances, cause clinical disease. Clinical trials have demonstrated that women taking folic acid supplements at the time of conception have a markedly lower chance of delivering a baby with an NTD compared with women who are not folate supplemented but whose folate status falls within a conventionally accepted range. This observation compelled the U.S. government to mandate the fortification of flour, beginning in 1998. Present recommendations are that women of childbearing age consume 400 µg/day of folic acid in the form of supplements or fortified foods, although the dose-response curve of this effect is ill-defined. Less than optimal intake of folate is also evidenced by an increase in serum homocysteine, an amino acid that is normally metabolized by a folatedependent pathway. Before the federally mandated fortification of flour, the median intake of folate among adults was half of the present RDA, and a substantial minority of Americans had significantly elevated serum homocysteine levels. Elevated homocysteine is associated with the development of

1454

CHAPTER 218  Vitamins, Trace Minerals, and Other Micronutrients  

occlusive vascular disease and accelerated cognitive decline. In randomized clinical trials, however, supplementation with folate, vitamin B12, and vitamin B6 has shown no benefit against cardiovascular disease despite its ability to lower homocysteine levels. A1  Such supplementation also has no clear benefit for cognitive function, except perhaps in patients with low baseline folate levels. A2  A compelling body of observations in both humans and animals has demonstrated that habitually low consumption of folate substantially increases the risk of colorectal cancer5 and perhaps cancers of other organs, such as those of the breast and pancreas. This inverse relationship is observed even when folate status (or dietary intake) falls within the range of conventionally accepted normative values. This relationship has further complicated the determination of what constitutes an optimal intake of folate because the recent epidemiologic data suggest that about 500 µg constitutes the optimal daily intake for suppressing the risk of colon cancer. The issue is further confounded by observations, albeit controversial, suggesting that exceptionally high doses of supplemental folic acid among those who unknowingly harbor precancerous or cancerous lesions may paradoxically enhance the progression of these neoplasms,6 thereby underscoring the potential for harm produced by taking a nutrient outside of its physiologic window. The most recent update of the U.S. RDA for folate raised the value from 200 to 400 µg/day, citing both the prevention of anemia and optimization of serum homocysteine as criteria, and recommended that women capable of becoming pregnant consume an additional 400 µg/day in the form of supplements or fortified food. The issues surrounding the prevention of cardiovascular disease, cancer, and cognitive decline were not incorporated into that 1998 determination because the existing data at the time were inconclusive. However, future revisions of the RDAs may integrate some of this new knowledge. The potential for toxicity, the criterion for which was primarily linked to its ability to mask vitamin B12 deficiency, was dealt with by setting the TUL at 1000 µg/day of folic acid obtained from supplements and fortified foods in addition to that obtained from natural food sources (see Table 218-2). Table 218-5 lists several examples of biochemical functions of vitamins that were not formerly recognized. As the clinical significance of each of these new roles is defined and as quantities of each vitamin needed to optimize such functions are determined, redefinition of the desirable range of vitamin status is likely to occur. Future efforts to refine appropriate dietary goals for each micronutrient will, however, need to take into consideration an important theme that is underscored by the previous discussion: the level of consumption of a particular micronutrient that conveys health benefits to one segment of the population is not necessarily beneficial, or even appropriate, for all segments of society.

Antioxidant and Free Radical Scavenging Vitamins and Provitamins

Vitamins A, C, and E as well as many of the carotenoids are effective antioxidants. In addition, vitamins C and E and some of the carotenoids can scavenge free radicals when these nutrients are taken in adequate quantities. Oxidation and free radical damage have been implicated as important contributors to common degenerative illnesses, such as atherosclerosis, cancer, cataracts, and retinal degeneration. Clinical trials to test the efficacy of antioxidant supplements have generally shown no benefit and in some instances

TABLE 218-5  NEWLY IDENTIFIED ROLES FOR VITAMINS VITAMIN OR PROVITAMIN

CLASSIC ROLE

NEW ROLE

β-Carotene

Pro-vitamin A

Antioxidant, free radical

Niacin

NAD/NADP coenzyme

Reduction of LDL, elevation of HDL cholesterol

Folate

Hematopoietic factor

Diminishes homocysteinemia

Vitamin A

Transduction of visual input in retina

Induction and maintenance of epithelial differentiation, signal in embryogenesis

Vitamin D

Regulator of calcium

Retards epithelial proliferation; promotes differentiation

Vitamin B6

Coenzyme for transamination

Modulation of steroid activity

HDL = high-density lipoprotein; LDL = low-density lipoprotein; NAD = nicotinamide adenine dinucleotide; NADP = nicotinamide adenine dinucleotide phosphate.

harm,7 although growing evidence indicates that health benefits of such supplements can be realized in populations with marginal antioxidant status. Two large-scale clinical intervention trials with β-carotene supplements conducted in the 1990s reported increased rates of lung cancer among the recipients of the carotenoid. Subsequent mechanistic studies indicated that the large doses administered (20 to 30 mg/day) result in asymmetrical cleavage of the carotenoid into unnatural products that antagonize normal signaling pathways in the lung epithelium, whereas lower supplemental doses undergo symmetrical cleavage into two molecules of vitamin A, thereby protecting against neoplastic transformation. LDL oxidized in vivo is atherogenic. Prevention of LDL oxidation, at least in animal models, retards the process of atherogenesis. Supplementation of human subjects with several times the RDA of α-tocopherol, and perhaps some of the other antioxidant micronutrients, is an effective means of preventing LDL oxidation. Human intervention trials with vitamin E or other antioxidant nutrients, however, have generally been unable to demonstrate clinical benefits in the reduction of cardiovascular events. There nevertheless has been a sizable reduction in cardiovascular events observed with vitamin E supplementation among populations of patients who are under exceptional oxidative stress, such as those with chronic renal failure and certain classes of diabetics, suggesting that it is only among select groups of individuals that a clinical benefit may be realized. Epidemiologic studies indicate that occurrence of cancers of the oral cavity, lung, esophagus, and stomach (and perhaps the colorectum) is inversely related to dietary intake of fresh vegetables and fruits. Careful dissection of dietary data suggests that β-carotene and vitamin E content are strongly predictive components of these foodstuffs. High doses of vitamin A and some of its synthetic analogues (e.g., 13-cis-retinoic acid) can effectively reduce the recurrence of head and neck cancers, although hepatic toxicity is sometimes a limiting factor in such cancer preventive therapy. Similarly, these agents, as well as β-carotene or vitamin E, taken in large doses have been shown significantly to promote the regression of oral leukoplakia, a premalignant lesion. Daily supplementation with one to three times the U.S. RDA of β-carotene, selenium, and vitamin E has been shown to reduce the incidence of adenocarcinoma of the stomach in a region of China where the disease as well as marginal vitamin status is particularly prevalent. However, as mentioned earlier, trials conducted in developed Western countries have observed no diminution of lung cancer among smokers with daily supplementation of β-carotene and vitamin E. Epidemiologic associations also suggest an inverse relationship between lens cataract or macular degeneration and the intake of vitamin C, vitamin E, and β-carotene. These common degenerative conditions of the eye are caused, at least in part, by photo-oxidation. Some evidence in animal models indicates that they can be retarded by supraphysiologic supplementation with vitamin C or E. When tested under the conditions of a rigorously conducted multicenter, controlled trial, daily supplementation with a combination of vitamin C, vitamin E, and β-carotene (with or without zinc) had no effects compared with placebo on the likelihood for development of cataracts. However, the combination that included zinc produced an approximately 30% decline in the progression of early macular degeneration to an advanced stage and the likelihood of moderate visual acuity loss. Further investigation is necessary to define the circumstances more clearly under which antioxidant nutrients can be used to prevent or to treat chronic degenerative diseases.

Vitamin B12 and Neuropsychiatric Disease

Plasma vitamin B12 concentrations are considered to be an accurate indication of vitamin B12 status. Values greater than 150 pg/mL were thought, until recently, to exclude vitamin B12 deficiency as a cause of neurologic or psychiatric syndromes.8 Recent observations now indicate that 7 to 10% of individuals who have plasma vitamin B12 values between 150 and 400 pg/mL may develop neuropsychiatric complications of vitamin B12 deficiency in the absence of any indications of megaloblastic anemia. Such individuals can be identified by the demonstration of an elevated level of methylmalonic acid in the blood that decreases to normal levels with parenteral vitamin B12 administration. An elevation in serum methylmalonic acid is both a sensitive and a specific indication of cellular vitamin B12 deficiency. An alternative approach is to administer several parenteral injections of vitamin B12 to an individual who has an otherwise unexplained neuropsychiatric syndrome and whose plasma vitamin B12 level falls in the range of 150 to 400 pg/mL. Awareness of this phenomenon is particularly important because it has become clear that atrophic gastritis, an asymptomatic condition that affects approximately

CHAPTER 219  Eating Disorders  

30% of the elderly population, frequently produces a modest decrease in vitamin B12 status; similarly, long-term use of proton pump inhibitor drugs inhibits absorption and also increases the risk of clinically significant deficiency.9

Is Routine Multivitamin and Multimineral Supplementation Beneficial?

A common query by patients is whether regular use of a multivitamin or multimineral supplement is safe and efficacious in the maintenance of health. Although there is not a unanimous consensus about the “correct” answer to this question, the weight of available evidence indicates that for the general adult North American population, supplementation offers little or no benefit in regard to the prevention of the common chronic degenerative diseases, such as vascular disease, cancer, and dementia. A3-A5  Although this apparent lack of efficacy has been notably contradicted by two clinical trials conducted in Western industrialized countries in which men taking multivitamins realized modest decreases in the incidence of cancer,10 such benefits have not been substantiated by other investigations. Although daily supplementation at the levels found in most multivitamin preparations probably presents no risk of harm, adverse health effects have been observed in several rigorously performed clinical trials in which longterm supplementation with micronutrients at levels that exceed the RDA (or conventional levels of dietary intake) by several-fold was examined. For example, an increased incidence of prostate cancer was observed in the SELECT trial, in which vitamin E was administered at a dose of 400 IU/day, and β-carotene supplementation resulted in an increased incidence of lung cancer among heavy smokers in the ATBC and CARET trials at doses of 20 to 30 mg/day. This is not to say that health benefits cannot be realized from supplementation in select groups of individuals, although some thought needs to be exercised to determine which segments of the population should be targeted and what specific nutrients should be administered. Certainly, health benefits are likely in individuals whose dietary intake is chronically inadequate or in patients whose medical conditions are often complicated by micronutrient deficiencies, such as those on chronic renal dialysis or among individuals with marginally controlled intestinal malabsorption. The elderly frequently cannot achieve recommended intakes of vitamin D and calcium with diet alone, and therefore targeted supplementation with these nutrients is often indicated. Similarly, the high prevalence of atrophic gastritis among the elderly as well as the frequent use of proton pump inhibitor drugs each conspire to impair adequate vitamin B12 status.11 Moreover, in many regions of the world, there continues to be a high prevalence of marginal micronutrient status among the general adult population, and in such areas widespread supplementation may be indicated; the Linxian trial in China, in which supplementation with a mixture of several antioxidant micronutrients led to a sizable decrease in gastric cancer, is one such example.

1455

219  EATING DISORDERS MARIAN TANOFSKY-KRAFF

DEFINITION

Feeding and eating disorders are defined as syndromes “characterized by a persistent disturbance of eating or eating-related behavior that results in the altered consumption or absorption of food and that significantly impairs physical health or psychosocial functioning.” The Diagnostic and Statistical Manual of Mental Disorders, fifth edition (DSM-5) defines anorexia nervosa (AN), bulimia nervosa (BN), and binge eating disorder (BED) as primary diagnoses in adolescents and adults. All other diagnoses are identified as Unspecified Feeding or Eating Disorder and represent presentations that do not meet the criteria for the primary eating disorders but nonetheless cause significant distress and impairment. The severity of each disorder is also specified as mild, moderate, severe, or extreme. Given the recent publication of the DSM-5, most empirical data available to date involve the criteria of the Diagnostic and Statistical Manual of Mental Disorders, fourth edition, text revision (DSM-IV-TR, published in 2000).

ANOREXIA NERVOSA

AN involves a restriction of “energy intake relative to requirements, leading to a significantly low body weight in the context of age, sex, developmental trajectory, and physical health.”1 Individuals with AN experience an intense fear of gaining weight or becoming fat, are overly concerned with weight or shape, and often may not recognize the seriousness of their low body weight. AN has two subtypes: restricting and binge-eating/purging. DSM-5 criteria for AN are listed in Table 219-1.

BULIMIA NERVOSA

A diagnosis of BN requires recurrent episodes of binge eating (i.e., the consumption of an unambiguously large amount of food given the context, accompanied by a sense of loss of control over eating). Episodes of binge eating co-occur with behaviors intended to compensate for energy consumed and to prevent weight gain, such as self-induced vomiting and fasting. Binge eating and compensatory behaviors must occur, on average, at least once a week for 3 months. The self-esteem of individuals with BN is excessively influenced by their body weight and shape. DSM-5 criteria for BN are outlined in Table 219-2.

BINGE EATING DISORDER

Grade A References A1. Clarke R, Halsey J, Lewington S, et al. Effects of lowering homocysteine levels with B vitamins on cardiovascular disease, cancer, and cause-specific mortality: meta-analysis of 8 randomized trials involving 37,485 individuals. Arch Intern Med. 2010;170:1622-1631. A2. Balk EM, Raman G, Tatsioni A, et al. Vitamin B6, B12, and folic acid supplementation and cognitive function: a systematic review of randomized trials. Arch Intern Med. 2007;167:21-30. A3. Fortmann S, Burda B, Senger C, et al. Vitamin and mineral supplements in the primary prevention of cardiovascular disease and cancer: an updated systematic evidence review for the U.S. Preventive Services Task Force. Ann Intern Med. 2013;159:824-834. A4. Grodstein F, O’Brien J, Kang J, et al. Long-term multivitamin supplementation and cognitive function in men: a randomized trial. Ann Intern Med. 2013;159:806-814. A5. Lamas G, Roineau R, Goertz C, et al. Oral high-dose multivitamins and minerals after myocardial infarction. A randomized trial. Ann Intern Med. 2013;159:797.

GENERAL REFERENCES For the General References and other additional features, please visit Expert Consult at https://expertconsult.inkling.com.

BED is characterized by recurrent episodes of binge eating in the absence of regular compensatory behaviors that are present in BN. The binge

TABLE 219-1  DSM-5 DIAGNOSTIC CRITERIA FOR ANOREXIA NERVOSA A. Restriction of energy intake relative to requirements, leading to a significantly low body weight in the context of age, sex, developmental trajectory, and physical health. Significantly low weight is defined as a weight that is less than minimally normal or, for children and adolescents, less than minimally expected. B. Intense fear of gaining weight or becoming fat, or persistent behavior that interferes with weight gain, even though at a significantly low weight. C. Disturbance in the way in which one’s body weight or shape is experienced, undue influence of body weight or shape on self-evaluation, or persistent lack of recognition of the seriousness of the current low body weight. Specify whether: Restricting type: During the last 3 months, the individual has not engaged in recurrent episodes of binge eating or purging behavior (i.e., self-induced vomiting or the misuse of laxatives, diuretics, or enemas). This subtype describes presentations in which weight loss is accomplished primarily through dieting, fasting, and/or excessive exercise. Binge-eating/purging type: During the last 3 months, the individual has engaged in recurrent episodes of binge eating or purging behavior (i.e., self-induced vomiting or the misuse of laxatives, diuretics, or enemas). From Diagnostic and Statistical Manual of Mental Disorders. 5th ed. Washington, DC: American Psychiatric Association; 2013.

CHAPTER 218  Vitamins, Trace Minerals, and Other Micronutrients  

GENERAL REFERENCES 1. Ribas GS, Vargas CR, Wajner M. l-Carnitine supplementation as a potential antioxidant therapy for inherited neurometabolic disorders. Gene. 2014;533:469-476. 2. Johnson M, Hausman D, Davey A, et al. Vitamin B12 deficiency in African American and white octogenarians and centenarians in Georgia. J Nutr Health Aging. 2010;14:339-345. 3. Dietary reference intakes for calcium and vitamin D. www.iom.edu/Reports/2010/Dietary-Reference-Intakes-for-calcium-and-vitamin-D.aspx; Accessed March 23, 2015. 4. Chung M, Lee J, Terasawa T, et al. Vitamin D with or without calcium supplementation for prevention of cancer and fractures: an updated meta-analysis for the U.S. Preventive Services Task Force. Ann Intern Med. 2011;155:827-838. 5. Gibson T, Weinstein S, Pfeiffer R, et al. Pre- and postfortification intake of folate and risk of colorectal cancer in a large prospective cohort study in the U.S. Am J Clin Nutr. 2011;94:1053-1062.

1455.e1

6. Figueiredo JC, Grau MV, Haile RW, et al. Folic acid and risk of prostate cancer: results from a randomized clinical trial. J Natl Cancer Inst. 2009;101:432-435. 7. Klein E, Thompson I Jr, Tangen C, et al. Vitamin E and the risk of prostate cancer: the Selenium and Vitamin E Cancer Prevention Trial (SELECT). JAMA. 2011;306:1549-1556. 8. Hunt A, Harrington D, Robinson S. Vitamin B12 deficiency. BMJ. 2014;349:g5226. 9. Lam JR, Schneider JL, Zhao W, et al. Proton pump inhibitor and histamine 2 receptor antagonist use and vitamin B12 deficiency. JAMA. 2013;310:2435-2442. 10. Gaziano J, Sesso H, Christen W, et al. Multivitamins in the prevention of cancer in men: the Physicians’ Health Study II randomized controlled trial. JAMA. 2012;308:1871-1880. 11. Stabler SP. Clinical practice. Vitamin B12 deficiency. N Engl J Med. 2013;368:149-160.

1456

CHAPTER 219  Eating Disorders  

TABLE 219-2  DSM-5 DIAGNOSTIC CRITERIA FOR BULIMIA NERVOSA A. Recurrent episodes of binge eating. An episode of binge eating is characterized by both of the following: 1. Eating, in a discrete period of time (e.g., within any 2-hour period), an amount of food that is definitely larger than most people would eat during a similar period of time and under similar circumstances. 2. A sense of lack of control over eating during the episodes (e.g., a feeling that one cannot stop eating or control what or how much one is eating). B. Recurrent inappropriate compensatory behavior in order to prevent weight gain, such as self-induced vomiting; misuse of laxatives, diuretics, enemas, or other medications; fasting; or excessive exercise. C. The binge eating and inappropriate compensatory behaviors both occur, on average, at least once a week for 3 months. D. Self-evaluation is unduly influenced by body shape and weight. E. The disturbance does not occur exclusively during episodes of anorexia nervosa. From Diagnostic and Statistical Manual of Mental Disorders. 5th ed. Washington, DC: American Psychiatric Association; 2013.

TABLE 219-3  DSM-5 DIAGNOSTIC CRITERIA FOR BINGE EATING DISORDER A. Recurrent episodes of binge eating. An episode of binge eating is characterized by both of the following: 1. Eating, in a discrete period of time (e.g., within any 2-hour period), an amount of food that is definitely larger than most people would eat during a similar period of time and under similar circumstances. 2. A sense of lack of control over eating during the episodes (e.g., a feeling that one cannot stop eating or control what or how much one is eating). B. The binge eating episodes are associated with three (or more) of the following: 1. Eating much more rapidly than normal. 2. Eating until feeling uncomfortably full. 3. Eating large amounts of food when not feeling physically hungry. 4. Eating alone because of feeling embarrassed by how much one is eating. 5. Feeling disgusted with oneself, depressed, or very guilty afterward. C. Marked distress regarding binge eating is present. D. The binge eating occurs, on average, at least once a week for 3 months. E. The binge eating is not associated with the recurrent use of inappropriate compensatory behavior as in bulimia nervosa and does not occur exclusively during the course of bulimia nervosa or anorexia nervosa. From Diagnostic and Statistical Manual of Mental Disorders. 5th ed. Washington, DC: American Psychiatric Association; 2013.

episodes are distinguished by at least three associated characteristics, such as eating rapidly, eating until feeling uncomfortably full, and feeling disgust and guilt regarding the episodes. Individuals experience marked distress surrounding the binge episodes, and the binge eating episodes must occur, on average, at least once a week for 3 months. DSM-5 criteria for BED are listed in Table 219-3.

EPIDEMIOLOGY

Data suggest a lifetime prevalence of AN of approximately 0.6%, with higher rates among women (0.9%) compared with men (0.3%).2 The lifetime prevalence of BN appears to be about 1%, with higher rates among women (1.5%) than among men (0.5%). The lifetime prevalence of BED is estimated at 3.5% for women and 2.0% for men. Among adolescents, lifetime prevalence estimates of AN, BN, and BED have been reported at 0.3%, 0.9%, and 1.6%, respectively.3 Contrary to the view that eating disorders afflict only nonHispanic white, affluent women, individuals of all races, ethnicities, and cultures are affected by these diagnoses.4

PATHOBIOLOGY

Research regarding the neuropathology of eating disorders is in nascent stages. Data suggest that several brain regions may be involved in and potentially interact in the manifestation of all eating disorders. Individuals with eating disorders appear to have brain function alterations in emotional/ limbic, reward, and cognitive control circuits.5 Fear circuitry networks involving the amygdala, anterior cingulate cortex, hippocampus, insula, striatum, and prefrontal cortex have demonstrated differential activation among individuals with eating disorders (with the majority of research in AN) compared with controls. Specifically, there tends to be a hyper-responsiveness in the limbic circuitry in response to potentially threatening cues, such as food and body weight/shape. There also appear to be alterations in reward function in patients with AN, but the direction is unclear. By contrast, individuals with BN and BED

consistently demonstrate hyper-responsivity in reward and somatosensory regions on exposure to food images. Data also suggest that individuals with eating disorders may have dysregulated frontal cortical cognitive neural networks acting in concert with regional reward systems.6 Individuals with eating disorders have demonstrated impaired cognitive flexibility. Specific to BN, impulsivity and poor inhibitory control have also been reported. Given the brain regions implicated in eating disorders, current studies have focused on the role of dopamine and serotonin in the manifestation of eating disorders. Individuals with AN appear to have impaired dopaminergic signaling, particularly in striatal circuits, that might contribute to altered reward and affect, decision making, and executive control as well as compulsivity and decreased food ingestion. Moreover, emerging clinical research suggests that striatal dopamine abnormalities exist in individuals with BN and BED. Because serotonin (5-hydroxytryptamine) 1A and 2A receptors and the serotonin transporter may play a part in symptoms of eating disorders, such as impulse control and associated mood symptoms, it is likely that interactions between the serotonin and dopaminergic systems contribute to eating disorders.

Risk Factors

Eating disorders develop as the result of multiple biological, psychological, and sociocultural factors. AN, BN, and BED aggregate in families, with estimates from twin studies suggesting that 40 to 60% of vulnerability for eating disorders is genetic. Studies have reported links between eating disorders and polymorphisms in the serotonin transporter gene (SLC6A4), the dopamine D2 receptor (DRD2) gene, the µ1 opioid receptor (OPRM1) gene, the fat mass and obesity-associated (FTO) gene, and the brain-derived neurotrophic factor (BDNF) gene. Although genetic linkage and association studies have implicated several susceptibility loci for AN, BN, and BED, specific genes that consistently lend vulnerability to eating disorders are less conclusive.7 Female sex, pediatric overweight, elevated shape and weight concerns, sexual abuse, trauma, and mood disorders have been identified as risk factors for all eating disorders. Personality-related variables, such as impulsivity and perfectionism, appear to be linked to eating disorders.8 Importantly, internalization to the “thin ideal” (a sociocultural emphasis on shape and weight and a marked preference for a thin body type) with resulting weight and shape concerns has been proposed to contribute to eating disorder development, particularly among adolescents who are under strong influence from their peer and family environments. For example, parental overconcern about eating, shape, and weight as well as weight-related teasing by family members confers risk for eating disorders. Specific to BED, maltreatment, including teasing and bullying, and perceived stress are risk factors for the disorder.9

CLINICAL MANIFESTATIONS

Symptoms and Signs

For AN, physical symptoms and signs may include amenorrhea, constipation, cold intolerance, anemia, and lanugo hair. Reduced bone density is believed to predict the onset of premature osteopenia and osteoporosis. Health problems associated with malnutrition affect cardiovascular, gastrointestinal, reproductive, and endocrine systems. Individuals with AN frequently present with comorbid psychiatric disorders, including mood and anxiety disorders (e.g., social phobia, specific phobia, post-traumatic stress disorder), and high rates of suicidal ideation and behavior. Individuals with BN present with signs and symptoms most commonly associated with purging behavior. These include dental enamel erosion secondary to vomiting, gastrointestinal symptoms, salivary gland hypertrophy, and electrolyte disturbances. Electrolyte abnormalities can have deleterious effects on the renal and cardiovascular systems. BN patients are at risk for cardiometabolic conditions (e.g., diabetes, stroke) as well as chronic pain. Metabolic acidosis can also occur in patients who are abusing laxatives as a result of the loss of bicarbonate from the bowel. Noninflammatory swelling of the salivary glands is a common clinical manifestation of BN. The most common psychiatric comorbidities in BN are major depressive disorder, anxiety disorders, substance use disorders, and disruptive behavioral disorders. Individuals with BED are frequently overweight or obese. However, adults with BED are likely to report the development of diagnoses of metabolic syndrome components (e.g., dyslipidemia, hypertension, type 2 diabetes) after accounting for the contribution of body weight. The presence of BED may affect bariatric surgery outcome, resulting in less weight loss or more weight regain, but this is not a consistent finding. However, the presence of “loss of control” eating after surgery consistently predicts less weight loss or greater weight regain. Compared with obese adults without BED, those with the disorder experience significant impairment in a number of domains of

CHAPTER 219  Eating Disorders  

psychosocial functioning, including a poorer quality of life and more impaired functioning in their home and social lives. Individuals with BED often have higher levels of disability, health problems, and work productivity impairment compared with obese and healthy controls without binge eating. With regard to comorbid psychiatric diagnoses, adults with BED experience Axis I psychiatric disorders at a rate comparable to (or higher than) that of individuals with AN or BN, including major depressive disorder, anxiety disorders, substance use disorders, and disruptive behavioral disorders.

Natural History

AN is typically manifested during adolescence, although the disorder can develop before puberty. BN frequently develops during later adolescence or early adulthood. BED is often manifested in adulthood, but adolescents also present with the disorder. Several retrospective and prospective studies report that binge and “out of control” eating occur as early as middle childhood. Data on the natural course of eating disorders in the clear absence of treatment are limited. Eating disorders tend to exhibit a remitting and relapsing natural course across the lifespan, and there appear to be high rates of diagnostic crossover.10 Treatment outcome data indicate that AN tends to transition to BN or an Unspecified Eating Disorder, and those with BN and BED tend to migrate from one to the other.

DIAGNOSIS

A number of structured, well-validated assessments for the diagnosis of eating disorders exist. These include but are not limited to the Structured Clinical Interview for the DSM and the Eating Disorder Examination. However, eating disorders are typically diagnosed by review of the patient’s history, symptoms, and behaviors in an interview format. Evaluation of comorbid psychiatric problems, most notably mood, anxiety, substance use disorders, and disruptive behavioral disorders, is also required. Information should be gathered on interpersonal relationships, history of sexual and physical abuse, self-harm, and suicidal ideation or behavior. Family involvement is crucial, particularly for pediatric patients. A complete physical examination to assess body composition, vital signs, cardiovascular function, and hematologic and blood chemistry parameters is recommended for all patients.

TREATMENT  Anorexia Nervosa

There is limited evidence on effective treatments for AN. For severely underweight patients, inpatient medical monitoring and supervised nutrition  rehabilitation are required. The optimal setting (inpatient versus outpatient treatment) remains a subject of debate, and the evaluation of treatment costs in AN plays an important role in determining treatment. However, for pediatric patients, family-based psychotherapy, particularly during the early phases of the disorder, has demonstrated effectiveness.11 Maudsley’s family-based therapy involves both joint family sessions and simultaneous but independent patient/family intervention. Antidepressants (e.g., selective serotonin reuptake inhibitors) are associated with high rates of noncompliance, and compelling evidence of beneficial effects has not been found. The use of antipsychotic drugs has been explored, but results regarding their effectiveness remain nondefinitive.

Bulimia Nervosa

Cognitive-behavioral therapy (CBT) has been recognized as the treatment of choice for BN. A1  Interpersonal psychotherapy (IPT) is also effective for the treatment of BN, particularly for those who are nonresponsive to CBT. There is growing support that pharmacotherapy may be helpful for some patients with BN. Antidepressants, especially selective serotonin reuptake inhibitors, are modestly effective for reducing binge eating in BN over the short and long term. Topiramate has consistently been shown to decrease binge eating in BN, but side effects may limit its usefulness. It is unclear whether combination therapy may be required for optimal outcomes.

Binge Eating Disorder

Psychological treatment for BED aims to reduce binge eating, weight and shape concerns, and prevent excess weight gain and/or induce modest weight loss. The psychotherapies most evaluated in clinical trials include CBT, IPT, behavioral weight loss, and CBT guided self-help (CBTgsh) approaches. CBT and IPT are first-line treatments. Given its cost-effectiveness, CBTgsh may be an optimal treatment option when specialist care is not available. A2  With regard to pharmacologic treatment in BED, three medications or classes of medications have been studied in two or more placebo-controlled trials. Selective serotonin reuptake inhibitors, sibutramine, and topiramate all produce reductions in frequency of binge eating relative to placebo in short-term

1457

trials.12 However, sibutramine has been withdrawn from the market, and topiramate is frequently associated with problematic cognitive effects, thus limiting its clinical utility.

PREVENTION

Whereas an increasing number of macro-level environmental public health initiatives have emerged (i.e., anti-dieting media campaigns and sanctions on advertising practices propagating an ideal of extreme thinness), few empirical data exist evaluating their efficacy. However, there are more data on individual, micro-level interventions aimed at reducing proximal eating disorder risk factors as well as current and distal eating pathology. Selected, interactive, multisession programs with adolescent girls may be more effective than universal, didactic, heterogeneous-sampled and single-session programs in reducing risk factors for eating disorder symptoms. For example, a dissonancebased program aimed at reducing eating disorder risk factors in adolescent girls has demonstrated effectiveness.

  PROGNOSIS

Anorexia Nervosa

Remission rates vary widely for AN. Lower remission rates (29%) have been observed, particularly in studies with the shortest follow-up duration. However, most individuals with AN (approximately 76%) treated in outpatient settings will remit within 5 years after the initiation of treatment. Most individuals who do not achieve remission from AN during follow-up periods transition to a diagnosis of BN or an Unspecified Eating Disorder, which likely captures partial syndrome AN. Among psychiatric diagnoses, AN consistently has one of the highest mortality rates due to suicide, nutritional deficits, cardiac complications, and substance abuse.13 The crude cumulative mortality rate is 2.8%, with longer duration of illness before receiving treatment and the need for inpatient treatment as negative prognostic indicators for AN.14 Predictors of relapse include desiring a lower weight at the end of treatment and receiving treatment in a general (versus specialty) clinic.

Bulimia Nervosa

Similar to AN, most individuals with BN (70% or more) who receive treatment fully remit when assessed 5 to 20 years later, with remission rates being much lower (27 to 28%) at 1-year follow-up. If individuals with BN do not achieve remission within 5 years, however, they are likely to exhibit a chronic course of the illness. Mortality rates for BN range between 0 and 2%. Diagnostic crossover from BN to AN is relatively rare; yet, there is frequent diagnostic crossover between BN and BED, which may suggest a possible common psychological and/or biologic maintaining process. Negative prognostic indicators for BN include endorsement of greater psychiatric comorbidity, multiple impulsive behaviors (e.g., self-harm, substance use disorder), and a family history of alcohol abuse. Individuals who receive inpatient treatment or have a low motivation for engaging in treatment are more likely to relapse.

Binge Eating Disorder

A paucity of data exists on the long-term outcomes for BED patients. There are data to suggest that at 1 year after outpatient treatment, upwards of 80% of patients remit. In one clinical trial that examined 4-year outcomes, between 52 and 76% of individuals receiving psychological treatment for BED demonstrated remission from binge eating.15 These preliminary data suggest that the prognostic trajectory may be similar to that of BN. Diagnostic crossover from BED to BN is high, whereas crossover to AN is relatively rare. Although examination of prognostic indicators for BED is in its early stages, patients reporting an undue influence of their body shape or weight on self-evaluation are less likely to have remission from binge eating at 12-month follow-up.16 Rapid remission of binge eating has also been shown to be a positive prognostic indicator for binge remission.17

Grade A References A1. Lock J, Le Grange D, Agras WS, et al. Randomized clinical trial comparing family-based treatment with adolescent-focused individual therapy for adolescents with anorexia nervosa. Arch Gen Psychiatry. 2010;67:1025-1032. A2. Poulsen S, Lunn S, Daniel SI, et al. A randomized controlled trial of psychoanalytic psychotherapy or cognitive-behavioral therapy for bulimia nervosa. Am J Psychiatry. 2014;171:109-116.

GENERAL REFERENCES For the General References and other additional features, please visit Expert Consult at https://expertconsult.inkling.com.

CHAPTER 219  Eating Disorders  

GENERAL REFERENCES 1. Attia E, Becker AE, Bryant-Waugh R, et al. Feeding and eating disorders in DSM-5. Am J Psychiatry. 2013;170:1237-1239. 2. Smink FR, van Hoeken D, Hoek HW. Epidemiology, course, and outcome of eating disorders. Curr Opin Psychiatry. 2013;26:543-548. 3. Swanson SA, Crow SJ, Le Grange D, et al. Prevalence and correlates of eating disorders in adolescents. Results from the national comorbidity survey replication adolescent supplement. Arch Gen Psychiatry. 2011;68:714-723. 4. Pike KM, Hoek HW, Dunne PE. Cultural trends and eating disorders. Curr Opin Psychiatry. 2014;27:436-442. 5. von Hausswolff-Juhlin Y, Brooks SJ, Larsson M. The neurobiology of eating disorders-a clinical perspective. Acta Psychiatr Scand. 2014;[Epub ahead of print]. 6. Frank GK, Kaye WH. Current status of functional imaging in eating disorders. Int J Eat Disord. 2012;45:723-736. 7. Stefano GB, Ptacek R, Kuzelova H, et al. Convergent dysregulation of frontal cortical cognitive and reward systems in eating disorders. Med Sci Monit. 2013;19:353-358. 8. Kessler RC, Berglund PA, Chiu WT, et al. The prevalence and correlates of binge eating disorder in the World Health Organization World Mental Health Surveys. Biol Psychiatry. 2013;73: 904-914.

1457.e1

9. Tanofsky-Kraff M, Bulik CM, Marcus MD, et al. Binge eating disorder: the next generation of research. Int J Eat Disord. 2013;46:193-207. 10. Allen KL, Byrne SM, Oddy WH, et al. DSM-IV-TR and DSM-5 eating disorders in adolescents: prevalence, stability, and psychosocial correlates in a population-based sample of male and female adolescents. J Abnorm Psychol. 2013;122:720-732. 11. Focker M, Knoll S, Hebebrand J. Anorexia nervosa. Eur Child Adolesc Psychiatry. 2013;22 (suppl 1):S29-S35. 12. McElroy SL, Guerdjikova AI, Mori N, et al. Pharmacological management of binge eating disorder: current and emerging treatment options. Ther Clin Risk Manag. 2012;8:219-241. 13. Campbell K, Peebles R. Eating disorders in children and adolescents: state of the art review. Pediatrics. 2014;134:582-592. 14. Keel PK, Brown TA. Update on course and outcome in eating disorders. Int J Eat Disord. 2010;43:195-204. 15. Franko DL, Keshaviah A, Eddy KT, et al. A longitudinal investigation of mortality in anorexia nervosa and bulimia nervosa. Am J Psychiatry. 2013;170:917-925. 16. Hilbert A, Bishop ME, Stein RI, et al. Long-term efficacy of psychological treatments for binge eating disorder. Br J Psychiatry. 2012;200:232-237. 17. Grilo CM, White MA, Gueorguieva R, et al. Predictive significance of the overvaluation of shape/ weight in obese patients with binge eating disorder: findings from a randomized controlled trial with 12-month follow-up. Psychol Med. 2013;43:1335-1344.

1457.e2

CHAPTER 219  Eating Disorders  

REVIEW QUESTIONS 1. A 21-year-old woman presents with scarred knuckles, faded wrist scarring, electrolyte imbalance, and low weight (body mass index [BMI] = 20). The menstrual cycle is reported as abnormal and infrequent. The patient denies current abnormal or restrictive eating patterns and self-injurious and compensatory behaviors. The patient expresses clear discomfort during collection of weight and avoids eye contact with clinical staff. No binge eating is reported. What is the most likely diagnosis for this individual? A. Anorexia nervosa, general diagnosis: patient exhibits low body weight, potential self-harming behaviors, amenorrhea, and clear discomfort with body weight. B. Bulimia nervosa, general diagnosis: low weight, knuckle scarring, and electrolyte imbalance are consistent with bulimia nervosa. C. Anorexia nervosa, purging subtype: low weight, potential self-harming behaviors, and amenorrhea are consistent with anorexia nervosa; knuckle scarring and electrolyte imbalance are consistent with anorexia nervosa purging subtype. D. No current eating disorder diagnosis: patient’s BMI is not significantly low within the context of age and sex to indicate a clinical diagnosis of anorexia nervosa, and binge eating and compensatory behaviors are not present. E. Bulimia nervosa, restricting subtype: presence of menstrual cycle, discomfort with body weight, knuckle scarring, and electrolyte imbalance are consistent with bulimia nervosa; low body weight is consistent with bulimia nervosa restricting subtype. Answer: D  The patient’s body weight is within a normal range, and denial of abnormal or restrictive eating patterns excludes a diagnosis of anorexia nervosa. Although knuckle scarring and electrolyte imbalance are suggestive of self-induced vomiting, a diagnosis of bulimia nervosa cannot be made without clear indication of the regular use of compensatory behaviors. Although amenorrhea is often a symptom, it is currently neither a necessary nor conclusive criterion for the diagnosis of anorexia nervosa. Although the patient cannot be given a conclusive eating diagnosis, further psychological and physical examination is warranted. 2. A 19-year-old female college student presents with current bulimia nervosa. The patient reports feeling very dissatisfied with her body shape and weight and engaging in binge eating while watching reality television. What is the best initial course of treatment action for you to suggest to the patient? A. Instruct the patient to modify television preferences to reduce episodes of binge eating while opening up a dialogue with friends about healthy eating habits. B. Refer the patient to a therapist for cognitive-behavioral therapy. C. Prescribe topiramate to help the patient reduce episodes of binge eating. D. Instruct the patient to keep a journal documenting emotions surrounding binge episodes for personal reflection. E. Refer the patient to a therapist for dialectical behavioral therapy. Answer: B  Cognitive-behavioral therapy remains the “gold standard” for the treatment of bulimia nervosa. Although other interventions, such as interpersonal psychotherapy and dialectical behavioral therapy, have shown efficacy in treatment of bulimia nervosa, cognitive-behavioral therapy shows the greatest efficacy across studies and subjects. Although discussing or expressing emotions regarding food and body shape and weight can be helpful for individuals with bulimia nervosa, constructive guidance is generally necessary for symptom reduction and cessation. Although topiramate has been shown to help individuals reduce binge eating, it has a number of cognitive side effects that may make its use impractical.

3. Which of the following is not a criterion for the diagnosis of binge eating disorder? A. BMI ≥ 30 (body mass index obesity threshold) B. Marked distress surrounding binge episodes C. Binge eating D. Lack of regular compensatory behaviors E. Weekly episodes of binge eating for at least 3 months Answer: A  There is no weight requirement for a diagnosis of binge eating disorder. Although individuals with binge eating disorder are often overweight or obese, BMI is not a required criterion for the diagnosis. 4. Which of the following statements regarding the genetics of disordered eating is true? A. Identical twins are 100% concordant for anorexia nervosa. B. Several genes have been conclusively linked to increased vulnerability for eating disorders. C. Of the neurotransmitters, dopamine and norepinephrine are the most likely candidates for eating disorder maintenance. D. Media generally play a bigger role than genetics in the onset of eating disorders. E. Children of individuals with binge eating disorder are more likely to experience out of control eating episodes than anorexia nervosa. Answer: E  Studies show that disordered eating behaviors, such as binge or loss of control eating, are often highly heritable. Offspring of mothers with binge eating have a higher genetic propensity to develop binge eating than do children of mothers who do not exhibit the behavior. Although monozygous twins share equal genetic predisposition for the development of disordered eating behaviors, no eating disorder shows 100% concordance between monozygous twins. Furthermore, although several gene loci have been indicated in the development and maintenance of eating disorders, evidence does not conclusively point to specific genes as responsible for disordered eating. Although studies have indicated the role of norepinephrine in the onset and maintenance of eating disorders, dopamine and serotonin are currently generally considered to be the most likely neurotransmitters to influence disordered eating behaviors. Findings regarding the effect of media on increasing risk for disordered eating are mixed; genetics, however, are strongly implicated. 5. Which of the following is not a finding regarding the neuropathology of eating disorders? A. Individuals with bulimia nervosa and binge eating disorders are hyperresponsive to food images in reward and somatosensory regions. B. Individuals with bulimia nervosa exhibit decreased impulsivity and enhanced inhibitory control relative to control subjects. C. Individuals with anorexia nervosa exhibit impaired dopaminergic signaling in striatal circuits. D. Individuals with anorexia nervosa exhibit differential activation of fear circuits relative to control subjects. E. Findings indicate that dysregulation in the limbic system may be influential in the development of eating disorders. Answer: B  Data suggest that individuals with bulimia nervosa exhibit increased impulsivity and decreased inhibitory control relative to control subjects.

1458

CHAPTER 220  Obesity  

220  OBESITY MICHAEL D. JENSEN

Obesity is the most common nutritional disorder in the United States and directly or indirectly accounts for a significant portion of health-related expenses. The safest treatment approaches (lifestyle change and behavior modification) are not those commonly employed by physicians and require considerable time to implement. The recently released Guideline for the Management of Overweight and Obesity in Adults provides direction for clinicians for the treatment of obesity.

DEFINITION

The Guideline for the Management of Overweight and Obesity in Adults produced by the National Institutes of Health and the National Heart, Lung, and Blood Institute (NHLBI) and disseminated by the American College of Cardiology (ACC), the American Heart Association (AHA), and The Obesity Society (TOS) provides evidence-informed, scientifically based recommendations on evaluation and management of overweight and obesity. Body mass index (BMI) continues to be the recommended approach to categorize weight relative to height for adults. BMI is calculated as weight (in kilograms) divided by height squared (in meters): BMI =

weight (kg ) height 2 (m 2 )

To calculate BMI with pounds and inches, the formula is modified as follows: BMI =

weight (lb) height 2 (in 2 )

The guideline suggested that no changes are indicated in the weight classifications by BMI, which are summarized in Table 220-1. Individuals who are overweight (BMI of 25.0 to 29.9) may or may not be overfat. Some adults may be overweight because of increased muscle mass, which is a straightforward clinical observation. Although, in general, the risk for development of adiposity-related health problems increases continuously as the BMI exceeds 25, the new guideline continues to recommend the use of waist circumference measurements to discriminate among patients who may require more testing. Overweight and class I obese patients with a waist circumference in the high-risk category deserve a discussion of lifestyle issues as they relate to health and weight loss. Some individuals with a BMI of 27 to 29.9 develop serious metabolic complications that improve with weight loss and are candidates for more aggressive treatment, including pharmacotherapy if it is needed. Asian populations, in particular, are at risk for the typical metabolic complications of obesity at lower BMI and waist circumferences than those for whites, Hispanics, blacks, and Polynesians; the guideline for at-risk BMI in Asian populations is 23 to 24.

TABLE 220-1  CLASSIFICATION OF OVERWEIGHT AND OBESITY BY BODY MASS INDEX (BMI) BMI (kg/m2) 40) is one of the key features that would prompt consideration of a patient for bariatric surgery when medical treatments have failed. Patients with class II obesity (BMI of 35.0 to 39.9) may be considered for bariatric surgery if medical treatments have failed and if severe, life-threatening complications are present. As noted, the new NHLBI/ACC/AHA/TOS guidelines continue to recommend waist circumference as an office assessment tool to help with the treatment decision-making process. The new guidelines suggest that the previous waist circumference cut points of more than 102 cm (40 inches) for men and more than 88 cm (35 inches) for women are indicators of increased metabolic risk. However, the report stated that the relationships between disease risk and waist circumference are continuous and progressive, with no obvious cut points. The recommendation is to measure waist circumference in overweight and class I obesity adults. Those adults with waist circumferences above the cut points deserve further evaluation to detect other cardiovascular disease risk factors. Adults with class II or class III obesity are at sufficiently high risk that waist circumference information does not appear to add valuable information. These definitions of overweight and obesity and of high-risk waist circumference are generally applicable to those of European and African descent, but lower values are recommended for those of Asian descent. The risks of metabolic abnormalities occur at lower BMI and lower waist circumference in these populations.

I

30.0-34.9

Obesity

II

35.0-39.9

Extreme obesity

III

≥40

Jensen MD, Ryan DH, Apovian CM, et al. 2013 AHA/ACC/TOS guideline for the management of overweight and obesity in adults: a report of the American College of Cardiology/American Heart Association Task Force on Practice Guidelines and The Obesity Society. J Am Coll Cardiol. 2014;63:2985-3023.

Although the number of overweight and obese adults in the United States has increased dramatically during the past 30 years, the increase in the prevalence is now slowing or leveling off.1 In 2009-2010, the prevalence of obesity was 35.5% among adult men and 35.8% among adult women, with no significant change compared with 2003-2008. Approximately 60% of U.S. men and 51% of U.S. women are overweight or obese, although a greater percentage of women than men are obese. There are substantial differences in the prevalence of obesity by age, race, and socioeconomic status. The prevalence of obesity in adults tends to rise steadily from the ages of 20 to 60 years, decreasing in later years. It has been estimated that almost 75% of men aged 60 to 69 years in the United States have a BMI of more than 25. The increase in mean BMI with age is not as much of a threat to population health as is a similar increase in the BMI of younger populations. The lowest mortality rates for young adults are for a BMI in the lower part of the normal range (20.0 to 24.9), whereas the BMI associated with the lowest mortality rates is somewhat above 25 kg/m2 for those in the 60s and 70s. Physicians should base their weight recommendations for individual patients on whether adverse health consequences associated with obesity are present. The differences in overweight and obesity among African Americans, Mexican Americans, and European Americans are not subtle. African American women and Mexican Americans of both sexes have the highest rates of overweight and obesity in the United States. In interpreting these data, however, it is important to keep in mind that there is an inverse relationship between socioeconomic status and obesity, especially among women (Chapter 5). Women in lower socioeconomic classes are much more likely than those in higher socioeconomic classes to be obese. This association reduces but does not eliminate the racial differences in the prevalence of obesity. Whether the remaining racial differences in the prevalence of obesity are due to genetic, constitutional, or social factors is not yet known.

PATHOBIOLOGY

Etiology

Genetic and constitutional susceptibility to obesity are heavily influenced by the environment. Evidence from studies of twins adopted into different families indicates that within a given environment, a significant portion of the variation in weight is genetic.2 That said, the remarkable increase in the prevalence of obesity in the United States during the past 3 decades is unlikely to be due to wholesale changes in the genetic makeup of Americans.

CHAPTER 220  Obesity  

Genetic Aspects of Human Obesity

Although obesity susceptibility is a classic polygenic condition, there are also a number of syndromic and monogenic obesity syndromes. The longrecognized genetic defects resulting in obesity include Prader-Willi and Laurence-Moon-Biedl syndromes. More recently, rare monogenic forms of human obesity due to mutations in the leptin gene, the leptin receptor gene, and the melanocortin signaling system genes have been described. These gene mutations are most often associated with increased appetite rather than with reduced energy expenditure. Genome-wide association studies have reported a number of genes associated with higher BMI. Those that appear to predict the greatest amount of variance in BMI include the fat mass and obesity-associated (FTO) gene and the melanocortin-4 receptor (MC4R) gene. Other genes that have been reliably associated with obesity include TMEM18, KCTD15, GNPDA2, SH2B1, MTCH2, and NEGR1. Together, however, the combined effects of all the identified genetic contributions account for less than 1% of the variance in BMI. This emphasizes both the huge environmental effects and the polygenic nature of susceptibility to obesity.

Constitutional Influences on Obesity

A number of environmental factors can result in long-term, epigenetic effects on body weight regulation and the susceptibility to obesity-related health problems. These epigenetic effects are ascribed to processes that include changes in DNA methylation, acetylation, and chromatin remodeling. The effect of the intrauterine environment and the perinatal period on subsequent weight and health is best studied. Undernutrition in the last trimester of pregnancy and in the early postnatal period decreases the risk of adult obesity, although the low birthweight associated with undernutrition (or smoking) in late pregnancy also increases the risk of adulthood hypertension, abnormal glucose tolerance, and cardiovascular disease. In contrast, undernutrition limited to the first two trimesters of pregnancy is associated with an increased probability of adult obesity. The infants of diabetic mothers tend to be fatter than those of nondiabetic mothers, and children of diabetic mothers have a greater prevalence of obesity when they are 5 to 19 years old, independent of whether their mother is obese. Finally, intrauterine exposure to the diabetic environment results in an increased risk of diabetes mellitus and obesity in the offspring. Thus, the issue of the genes versus the environment in regard to obesity and metabolic complications of obesity is blurred in the intrauterine and perinatal time intervals. One of the striking and worrisome aspects of these metabolic effects is not only the long-term effects on the individual’s weight regulation and health but also the suggestion that these traits can be passed on to future generations.

Environmental Contributors to Human Obesity

Dramatic changes in the environment of Western countries have occurred during the past 50 years, including reduced demands for physical activity and alterations in the food supply. These food supply changes appear to have either increased or prevented the expected decrease in energy intake that would be needed to match the reduced energy expenditure from physical activity.3

Food

A number of environmental factors can influence food intake (Table 220-2). Consuming energy-dense foods results in greater energy intake because

TABLE 220-2  ENVIRONMENTAL FACTORS PROMOTING OBESITY DIETARY

ACTIVITY

↑ Energy density of foods

↑ Sedentary behavior

↑ Portion size

↓ Activities of daily living

↑ Variety

↓ Employment physical activity

*

↑ Palatability ↑ Availability ↓ Cost ↑ Caloric beverages (sugar-sweetened beverages) *

Variety of sweets, snacks, and entrees.

1459

adults tend to respond to food volume rather than to the energy content. This factor likely accounts for the association between high-fat diets and excess body weight; many high-fat foods are also energy dense. When humans consume diets that are high in fat but low in energy density, energy intake is not greater than would be expected on the basis of the energy density of the foods. Larger food portion size has also been shown to increase food intake. Given the trend in the United States to serve larger portions of food and beverage, this could contribute to greater obesity risk. Food variety can also affect energy intake. An increased variety of entrees, sweets, snacks, and carbohydrates in the diet is associated with an increase in body fatness and food intake. In contrast, an increase in the variety of vegetables available does not appear to increase energy intake and is not associated with increased body fatness. Other factors that may have broad population effects in the United States include the reduced costs of food, increased availability, and palatability. Finally, there is evidence that consumption of sugar-sweetened beverages, such as soft drinks and fruit juices, is not accompanied by a decrease in food intake to offset the extra energy intake. The implication is that some types of beverages will add to the energy intake during the day and promote weight gain. A number of psychological factors also influence how the properties of food affect energy intake. Individuals vary with respect to their dietary restraint (the tendency to consciously limit food intake to control weight), their feelings of hunger, or their disinhibition (the tendency to overeat opportunistically). It has been proposed that interindividual differences in these factors modify how food variety and portion size affect the eating profile. The social context in which food is consumed and the emotional state of the individual also modulate food intake.

Physical Activity

Physical activity can be divided into three categories: (1) exercise (fitnessand sports-related activities); (2) work-related physical activity; and (3) nonexercise, nonemployment (spontaneous) activity. Tables are widely available that allow one to calculate energy expenditure on the basis of an individual’s weight as well as the type and duration of exercise. Only about 20 to 30% of Americans engage in exercise at the recommended frequency, intensity, or duration that could be expected to have a protective effect on the development of obesity and other health problems, but this does not seem to have changed in recent decades. Recent data suggest that the amount of time spent in sedentary activities (e.g., watching television, using the computer) is an independent predictor of metabolic abnormalities associated with obesity over and above the effects of exercise. Thus, to the extent that reduced physical activity is contributing to the epidemic of obesity, it is likely that it is reduced employment-related and spontaneous physical activity that is changing. Although it is becoming easier for individuals to measure the energy expended in nonexercise activity with step counters and electronic motion detection devices, there are insufficient longitudinal, population-based data to define the extent to which changes in this activity parameter have occurred. Certainly, employment-related physical activity has decreased with the advent of more automated systems in the workplace. One estimate suggests that between 1982 and 1992, energy expenditure at work decreased by about 50 kcal/day. The additional workplace changes since that time have probably reduced employment physical activity further. The other component of nonexercise physical activity, the activities of daily living, has probably been reduced by the plethora of labor-saving conveniences (e.g., drive-through food and banking, escalators, remote controls, e-mail, online shopping) now available. Again, there are few hard data to assess how much of a change has actually occurred, although a reduction in daily walking trips and an increase in daily automobile trips have been documented. There is a large amount of information on how differences in sedentary activity (television watching, video games, and computer use) relate to obesity and obesity complications. The evidence indicates that more time spent in sedentary pursuits is associated with an increased risk of overweight and obesity. The striking aspect to these studies is that the adverse effect of sedentary activities is independent of participation in traditional exercise activities. Understanding the contributions of decreased work-related physical activity, decreases in activity of daily living, and increases in sedentary behavior can help the physician working with the patient to uncover patterns that may relate to weight gain. Physicians who are aware of these environmental factors are in a better position to help their obese patients identify which of these

1460

CHAPTER 220  Obesity  

environmental factors are contributing to the problem and to develop plans for intervention. In this regard, patients who regularly use step counters or other types of activity-monitoring devices will be better able to self-identify and modify their behavior to obtain sufficient physical activity.

Regulation of Body Weight and Energy Balance

The regulation of adult body weight is a well-balanced process. For example, the typical U.S. adult will take in and expend approximately 2000 to 3000 kcal/day. If there were a consistent error of even 1% in overconsumption of food, this would result in the gain of approximately 25 to 30 pounds of fat every 10 years, assuming no change in energy expenditure. It follows that most adults regulate their average energy balance with greater than 1% precision. There appears to be regulation of both energy intake and energy expenditure through conscious and unconscious processes. The excess energy consumed by adults is generally stored as triglycerides in adipocytes. Humans continuously recruit new adipocytes from a large preadipocyte pool to replace dying adipocytes. Although the primary means by which abdominal adipose tissue mass expands is through increased fat cell size (adipocyte hypertrophy), this process can store only a limited amount of fat. Adults who gain leg fat accumulate more rather than larger adipocytes on average, resulting in a net increase in adipocyte number as more new adipocytes are created than needed to replace dying cells. Some adults recruit new adipocytes more readily than others do and thus gain weight more so from adipocyte hyperplasia (increased fat cell number) than from hypertrophy. Those who gain fat with large adipocytes, especially in association with an adipose tissue inflammatory response (greater numbers of classically activated macrophages and other immune cells), are more likely to be insulin resistant and to have signs of low-grade systemic inflammation (increased C-reactive protein, mildly elevated interleukin-6 and tumor necrosis factor). Leptin, a cytokine family protein that is secreted almost exclusively by adipocytes, was the first identified adipose tissue hormone; it has been shown to have potent central nervous system effects on food intake in humans. Leptin also has other hypothalamic-pituitary functions and is proposed to have diverse peripheral physiologic actions. The leptin-deficient animal model of obesity, the ob/ob mouse, is severely obese, hyperphagic, hypometabolic, and sexually immature and has low levels of spontaneous activity. Administration of leptin to this animal corrects all of these defects. A few leptin-deficient humans (due to mutations in the leptin gene) have been identified. These children had very low plasma leptin concentrations, were hyperphagic and severely obese, and responded to exogenous leptin administration with dramatic weight loss, reduced food intake, and accelerated maturation of the pituitary-gonadal axis. Overwhelmingly, however, obese humans are not leptin deficient and in fact have high plasma leptin concentrations unless they are in a major negative energy balance circumstance. Because leptin is secreted as a function of percentage body fat, and because women have more body fat than men for any given BMI, they also have higher plasma leptin concentrations. Thus, screening for leptin deficiency is not warranted except in severe, hyperphagic obesity that begins in early childhood, is accompanied by sexual immaturity, and exists in the absence of other known causes (e.g., Prader-Willi syndrome). Some animal models of genetic obesity (the db/db mouse and fa/fa rat) have defective leptin receptors, making them unresponsive to leptin. Although rare cases of obese humans with defective leptin receptor genes have been reported, again it appears that leptin resistance due to leptin receptor defects (or genetic post-receptor signaling abnormalities) is extremely uncommon. Clinical screening for leptin receptor mutations is not warranted, given that no treatment exists.

Energy Intake

Much of what has been learned about the biologic regulation of food intake has been from the study of animal models. These signals may affect different aspects of eating behavior. They can affect hunger, the compelling need or desire for food; satiation, the state of being satisfactorily full and unable to take on more; or satiety, the sense of no longer being hungry, a complex set of postprandial events that affect the interval to the next meal or the amount consumed at the next meal. Some of the signals that alter eating behavior affect one aspect and others affect multiple aspects. For example, ghrelin, a peptide produced by the stomach, increases hunger but does not appear to affect satiation or satiety. Cholecystokinin causes satiation but has no effect on satiety. Leptin appears to act on multiple pathways; leptin deficiency is associated with increased hunger and reduced satiation and satiety.

TABLE 220-3  SUGGESTED BIOLOGIC MODULATORS OF FOOD INTAKE PROPOSED EFFECT ON FOOD INTAKE

PERIPHERAL SIGNAL Vagal



Cholecystokinin



Apolipoprotein A-IV



Insulin



Peptide YY3-36



Glucagon-like peptide 1



Other glucagon-related peptides



Leptin

+ when leptin ↓↓

Ghrelin

+

Tumor necrosis factor-α



Obestatin



TABLE 220-4  CENTRAL NERVOUS SYSTEM MODULATORS OF ENERGY BALANCE CENTRAL ANABOLIC (↑ INTAKE)

CENTRAL CATABOLIC (↓ INTAKE)

Neuropeptide Y

α-Melanocyte-stimulating hormone

Agouti-related protein

Corticotropin-releasing hormone

Melanin-concentrating hormone

Thyrotropin-releasing hormone

Hypocretins and orexins

Cocaine- and amphetamine-regulated transcript

Galanin

Interleukin-1β

Norepinephrine

Urocortin

Endogenous endocannabinoids (anandamide and 2-arachidonoylglycerol)

Oxytocin Neurotensin Serotonin

Peripheral satiety signals act to inhibit further food intake at some point during meal consumption. Some of the signals reach the brain through the vagus nerve and some through the systemic circulation. Examples of the proposed factors modulating appetite are listed in Table 220-3. The compounds range from gut-derived (ghrelin, cholecystokinin, glucagon-like peptide 1) and pancreas-derived (insulin) hormones to peptides such as apolipoprotein A-IV, which is secreted with chylomicrons. The signals are thought to be triggered both by mechanical stimuli (e.g., the fullness of the stomach) and by the presence of nutrients in the jejunum and ileum. The central nervous system regulation of food intake is becoming better understood. A number of neuropeptides, lipid derivatives, and monoamines have either anabolic (increased food intake with or without decreased energy expenditure) or catabolic (decreased food intake with or without increased energy expenditure) properties. A list of these molecules is provided in Table 220-4. Many of these compounds serve more than one function, such as regulation of hormone secretion (thyrotropin-releasing hormone and corticotropin-releasing hormone), wakefulness (norepinephrine), and behavior-reinforcing systems (endocannabinoids).

Energy Expenditure

There is a wide range of daily energy expenditure in adults, from less than 1400 kcal/day to more than 5000 kcal/day, with larger, more physically active individuals having the greatest energy needs. Typically, daily energy expenditure is divided into resting (or basal) metabolic rate, the thermic effect of food, and physical activity energy expenditure.

Basal Metabolic Rate

The basal metabolic rate (BMR) is the energy expenditure of lying still at rest, awake, in the overnight postabsorptive state. The resting metabolic rate (RMR) is similarly defined but is not necessarily measured before arising from bed. For most sedentary adult Americans, the RMR represents the

CHAPTER 220  Obesity  

major portion of energy expended during the day and may range from less than 1200 to more than 3000 kcal/day. Most (~80%) of the BMR can be explained by the amount of lean tissue an individual has. There are a number of formulas that can be used to estimate BMR. The Harris-Benedict formula (available through numerous online calculators) predicts BMR on the basis of height, weight, age, and sex and is accurate to within 10% in approximately 90% of adults with BMIs of 18.5 to 45 kg/m2. Not all components of lean tissue consume oxygen at the same relative rates. Visceral or splanchnic bed tissues account for about 25% of RMR but a much smaller proportion of body weight. The brain, which is only a small percentage of body weight, accounts for almost 15% of RMR. Likewise, the heart (~7%) and kidneys (~5 to 10%) account for greater portions of resting energy needs than their relative contribution to body mass. In contrast, resting muscle makes up 40 to 50% of lean tissue mass but accounts for only 25% of RMR. This contribution changes dramatically with exercise, however, at which time muscle can account for 80 to 90% of energy expenditure. Adipose tissue is a minor contributor to daily energy expenditure, consuming only approximately 3 kcal/kg of body fat per day. Brown fat is adipose tissue that expresses large amounts of uncoupling protein-1, a protein that allows a mitochondrial membrane proton leak, resulting in heat release as opposed to chemical work from adenosine triphosphate—“uncoupling” of substrate oxidation from chemical or mechanical work. This thermogenic tissue was thought to be present only in human infants but has recently been shown to exist in adults.4 Methods used to detect brown fat largely rely on 18F-fluorodeoxyglucose positron emission tomography scanning of humans exposed to cold. Lean adults are more likely than obese adults to have brown fat, and brown fat is more readily detectable after obese adults lose weight. Whether brown fat plays any meaningful role in thermogenesis is currently a matter of debate. Although most of the RMR can be accounted for by the mass of lean tissue, there are also other, more subtle influences on RMR. Age, sex (women have slightly lower BMR even corrected for fat-free mass), and fat mass affect RMR. Small changes in BMR occur during the menstrual cycle (luteal phase > follicular phase). There is also evidence that heritable or family factors influence BMR, accounting for as much as 10% of the interindividual differences. There are both obligatory and facultative components to RMR. With an energy-restricted diet, significant reductions in BMR relative to the amount of fat-free mass occur. Reductions in the production of triiodothyronine from thyroxine and the sympathetic nervous system drive are thought to contribute to this phenomenon. Likewise, during brief periods of overfeeding, RMR increases slightly above that which would be expected for the amount of lean tissue present. It has been proposed that individuals with BMRs lower than predicted are at increased risk of future weight gain. Published data suggest that the relative risk is small, and clinical effort to identify such patients is not warranted. Measurement of BMR is sometimes helpful in the evaluation of patients who insist that they are unable to lose weight while following diets containing less than 1000 kcal/day. Almost without fail, if BMR is measured with a reliable instrument, it is substantially greater than the reported food intake. This underscores the fact that most adults are unreliable in assessing their own food intake.

The Thermic Effect of Food

An average of 10% of the energy content of food is expended in the process of digestion, absorption, and metabolism of nutrients. There is a significant interindividual variability in this value, however, ranging from a low of about 3% to a high of about 15% of meal calories that are “wasted” in the postprandial interval. The thermic effect of a meal is related to its carbohydrate and protein calorie content; the fat content has little stimulatory effect. Both obligatory (60 to 70%) and facultative (30 to 40%) components of the thermic effect of food have been identified. The obligatory components no doubt reflect the energy costs of digestion, absorption, and storage of nutrients. The two factors thought to play a role in the facultative component of the thermic effect of food are the postprandial insulin response and activation of the sympathetic nervous system. The thermic effect of food is somewhat lower in insulin-resistant and obese humans, but this has not been linked to future obesity.

Physical Activity Energy Expenditure

The energy expenditure of physical activity is a product of the amount of work done and the work efficiency of the individual. Tracking the total

1461

amount of physical activity that humans perform throughout the day is becoming easier with a variety of relatively inexpensive devices. By doing so, it is also possible to calculate the energy expended with published values for estimating the energy costs of work performed. Work units are expressed as metabolic equivalents (METs), a multiple of the RMR. If an individual’s RMR is 1 kcal/minute (1440 kcal/day), a workload of 5 METs would be 5 kcal/minute. Although most sedentary individuals can work for only a limited amount of time at relatively low workloads, highly trained athletes can work at extremely high METs (>16) for extended periods. This is because athletes have both a greater peak work capacity (or maximal amount of calories or oxygen that can be consumed) and a higher lactate threshold. The lactate threshold is closely related to the level at which exercise begins to become so uncomfortable that it cannot be maintained much longer. The biochemical definition of lactate threshold describes the progressive rise in blood lactate concentrations observed during sufficiently high-intensity exercise. The lactate threshold may range from 50 to 90% of an individual’s peak work capacity. Training raises the lactate threshold closer to the maximal workload and thus allows individuals to work at higher rates for longer periods. Thus, highly fit individuals can expend much greater amounts of energy per minute of exercise with less sense of discomfort than can obese, sedentary individuals who typically have low aerobic fitness and low lactate thresholds (sometimes on the order of 4 to 5 METs). The lactate threshold can be even lower in obese patients with type 2 diabetes, such that walking a mere 3 miles per hour can exceed their lactate threshold. Appreciation of the physical limitations of patients, which can usually be overcome with a carefully designed training program, is necessary to provide realistic activity recommendations. Exercise (fitness- and sports-related activities) is commonly considered the main component of physical activity thermogenesis. Because most adults do not exercise at high levels or for a sufficient duration to expend a large amount of energy, focusing solely on “exercise” as the main component of physical activity will miss significant opportunities for improving energy balance. The benefits of and energy expended in nonexercise activity can be far greater than with exercise, given the limited amount of time and effort that most patients can commit to exercise. Nonexercise activity thermogenesis (NEAT) is the calorie expense of performing all activities other than exercise- or employment-related and spontaneous activity. The range of observed NEAT under controlled (metabolic chamber) conditions has been less than 100 to more than 800 kcal/day. The energy expended from a physically demanding job or volitional exercise may or may not be offset by reductions in spontaneous (nonemployment) activity. For example, young adult men and women respond differently to 1 year of extra exercise; men lose weight and women do not, despite the absence of detectable change in food intake. Women must either reduce spontaneous activity in response to exercise or have subtle increases in food intake. It has also been shown that the variations in unconscious increases in NEAT relate strongly to the amount of fat gained in response to overeating. Low levels of NEAT have been reported to predict future weight gain in some populations, and there may be differences between lean and obese persons in the daily amount of NEAT, which could relate to differential tendencies to regulate weight.

Secondary Causes of Obesity Medications

A number of medications cause weight gain in some or most of the patients for whom they are prescribed. Awareness of the medications that have this potential can facilitate weight loss treatment in some patients. Table 220-5 lists a number of medications that are associated with weight gain as well as alternative treatment approaches, if any, for the underlying condition.

Diseases

Less than 1% of obese patients have an underlying disease that can explain the development of their obesity. Endocrinopathies are the most common secondary cause of obesity. These include Cushing’s syndrome (Chapter 227), hypothalamic damage resulting in overeating (most commonly after pituitary surgery), insulinoma (Chapter 230), and hypothyroidism (Chapter 226). A Cushing syndrome–like fat distribution is common; therefore, other physical or laboratory findings are the best clues to whether to test for this condition. These include the classic purple striae, thinning skin, easy bruising, proximal muscle weakness, and electrolyte abnormalities. Correction of Cushing’s syndrome commonly results in substantial loss of excess body fat. Insulinoma is a rare tumor, and only a small portion of patients with

1462

CHAPTER 220  Obesity  

TABLE 220-5  PHARMACOLOGIC INFLUENCES IN WEIGHT GAIN AND ALTERNATIVE THERAPIES DRUGS THAT MAY PROMOTE WEIGHT GAIN

ALTERNATIVE TREATMENTS: WEIGHT NEUTRAL OR WEIGHT LOSS

PSYCHIATRIC AND NEUROLOGIC MEDICATIONS Antipsychotics: olanzapine, clozapine, risperidone, quetiapine, aripiprazole Antidepressants   Tricyclics: imipramine, amitriptyline   Triazolopyridines: trazodone   Serotonin reuptake inhibitors: paroxetine, fluoxetine, citalopram Tetracyclics: mirtazapine Monamine oxidase inhibitors Antiepileptic drugs: gabapentin (higher doses), valproic acid, carbamazepine, divalproex Mood stabilizers: lithium, carbamazepine, lamotrigine, gabapentin (higher doses)

ALTERNATIVE PSYCHIATRIC AND NEUROLOGIC MEDICATIONS Ziprasidone Nortriptyline, bupropion, nefazodone, fluvoxamine, sertraline, duloxetine Topiramate, zonisamide (weight loss), lamotrigine (less weight gain)

STEROID HORMONES Progestational steroids Corticosteroids Hormonal contraceptives

ALTERNATIVES TO STEROID HORMONES Barrier methods, intrauterine device Nonsteroidal anti-inflammatory drugs

ANTIDIABETES AGENTS Insulin (most forms) Sulfonylureas Thiazolidinediones

ALTERNATIVE ANTIDIABETES AGENTS Metformin Acarbose, miglitol Exenatide Dipeptidyl peptidase 4 inhibitors Liraglutide Sodium-glucose co-transporter 2 inhibitors

ANTIHISTAMINES Commonly reported with older agents; also oxatomide, loratadine, and azelastine

ALTERNATIVE TO ANTIHISTAMINES Decongestants, mast cell stabilizers, antagonists of endogenous mediators of inflammation

ANTIHYPERTENSIVE AGENTS α-Adrenergic and β-adrenergic receptor blockers Calcium channel blockers: nisoldipine

ALTERNATIVE ANTIHYPERTENSIVE AGENTS Angiotensin-converting enzyme inhibitors Calcium channel blockers: most other agents Angiotensin receptor blockers Diuretics

HIGHLY ACTIVE ANTIRETROVIRAL THERAPY

insulinoma develop obesity. The weight gain associated with hypothyroidism is largely due to fluid retention and resolves with thyroid hormone replacement. Unfortunately, successful treatment is not available for hyperphagia due to hypothalamic damage. Adult patients with growth hormone deficiency, most commonly after hypophysectomy, may lose excess body fat with growth hormone replacement therapy.

Psychosocial Aspects of Obesity

Sexual, physical, and emotional abuse, especially in women, can result in long-term adverse consequences, including obesity. The effects of the abuse tend to be most profound if it occurs in childhood and adolescence. These women may be severely obese, suffer from chronic depression, and experience a number of psychosomatic symptoms, particularly chronic gas­ trointestinal distress. Identifying these issues before initiation of weight loss programs is important because successful weight loss may actually aggravate the distress experienced by these women. In addition, appropriate referral for psychiatric help may be needed before initiation of treatment for obesity.

PATHOPHYSIOLOGY

Metabolic Complications of Obesity

A central or upper body fat distribution is more predictive than total fat mass of the metabolic complications of obesity. Adipose tissue release of free fatty acids (FFAs) and glycerol into the circulation through lipolysis provides 50 to 100% of daily energy needs. Adipose tissue lipolysis is regulated primarily by insulin (inhibition) and catecholamines (stimulation), although growth hormone, cortisol, and atrial natriuretic peptide also stimulate lipolysis. Upper body obesity is associated with several abnormalities of adipose tissue lipolysis, most remarkably with higher postprandial FFA release and concentrations; this abnormality is particularly evident in type 2 diabetes mellitus. Abnormally high FFA concentrations can contribute to a number of the metabolic complications of obesity.

Insulin Resistance

The term insulin resistance is typically used in referring to the ability of insulin to promote glucose uptake and to inhibit the release of glucose into the circulation. The primary site of insulin-stimulated glucose uptake, oxidation,

and storage is skeletal muscle. The principal site of glucose production is the liver. Insulin resistance initially leads to hyperinsulinemia and may eventually lead to the development of type 2 diabetes mellitus (Chapter 229). The ability of insulin to promote glucose uptake, oxidation, and storage in muscle and to suppress plasma FFA concentrations is reduced in upper body obesity. High plasma FFA concentrations can induce a state of insulin resistance both in the muscle (glucose uptake) and in the liver (glucose release), independent of obesity. Thus, abnormal regulation of adipose tissue FFA export is a significant component of the development of insulin resistance. It is hypothesized that excess FFAs induce muscle insulin resistance by promoting increased synthesis of diacylglycerols and ceramides, both of which can interfere with the normal insulin signaling pathway. Dysregulated production of a number of adipose-derived hormones, also called adipokines, is hypothesized to contribute to insulin resistance and the metabolic complications of obesity. Adiponectin, an adipocyte-derived hormone that improves insulin action, is secreted at reduced rates in obesity and diabetes. Increased production of resistin, interleukin-6, tumor necrosis factor, and retinol-binding protein-4 by adipose tissue has been linked to insulin resistance in animal models. We currently lack the experimental evidence from human studies to know what role adipokines play in the metabolic complications of obesity.

Islet Cell Failure and Type 2 Diabetes Mellitus

Type 2 diabetes usually results from defects in both insulin secretion and insulin action (Chapter 229). Many obese individuals are insulin resistant, yet only a subset will develop diabetes mellitus. It follows that those who develop type 2 diabetes develop pancreatic β-cell decompensation with subsequent hyperglycemia. Animal (rodent) studies have suggested that a process referred to as lipotoxicity is involved in pancreatic β-cell failure. In this model, increased FFAs are proposed to contribute to the insulin secretory abnormalities seen in obesity and ultimately to lead to β-cell failure. There is some evidence that elevated FFAs have adverse effects on islet β-cell function in humans. Another potential contributor to β-cell failure in obesity is the overproduction of islet amyloid polypeptide. This protein is co-secreted with insulin and, because of its tertiary structure, can form toxic amyloid deposits in β cells. Amyloid deposits have been found in the pancreatic islets obtained at autopsy from patients with type 2 diabetes mellitus.

CHAPTER 220  Obesity  

1463

Blood pressure can be increased by a number of mechanisms (Chapter 67). Increased circulating blood volume, abnormal vasoconstriction, decreased vascular relaxation, and increased cardiac output may all contribute to hypertension in obesity. The effect of hyperinsulinemia to increase renal sodium absorption may contribute to hypertension through increased circulating blood volume. Abnormalities of vascular resistance also contribute to the pathophysiologic process of obesity-related hypertension. Under some experimental conditions, elevated FFAs have been found to cause increased vasoconstriction and reduced nitric oxide–mediated vasorelaxation, similar to that seen in the metabolic syndrome. Some obese adults have increased sympathetic nervous system activity, which could contribute to obesityassociated hypertension. Finally, angiotensinogen (also produced by adipocytes) is a precursor of the vasoconstrictor angiotensin II and is proposed to contribute to elevated blood pressure.

Sleep apnea is most likely explained by enlargement of upper airway soft tissue, resulting in collapse of the upper airways with inspiration during sleep. The obstruction leads to apneas, with hypoxemia, hypercarbia, and high catecholamine and endothelin levels. The frequent arousals to restore breathing result in poor sleep quality. Sleep apnea is associated with an increased risk of hypertension, and if sleep apnea is severe, it can lead to right-sided heart failure and sudden death. A history of daytime hypersomnolence, loud snoring, restless sleep, or morning headaches is suggestive of obstructive sleep apnea. Treatment of sleep apnea is important to improve cardiovascular risk, and the failure to recognize and to treat this complication may make weight loss intervention strategies much less successful. Epidemiologic studies have linked short sleep duration and disruptions of circadian rhythm with increased risk of metabolic syndrome and diabetes. Experimentally induced sleep restriction combined with circadian disruption in humans led to decreased RMR and increased postprandial plasma glucose levels due to inadequate insulin secretion.

Dyslipidemia

Cancer

Hypertension

Upper body obesity and type 2 diabetes mellitus are associated with increased triglycerides, decreased high-density lipoprotein (HDL) cholesterol, and a high proportion of small, dense low-density lipoprotein (LDL) particles (Chapter 206). This dyslipidemia contributes to the increased cardiovascular risk observed in the metabolic syndrome. Fasting hypertriglyceridemia is caused by increased hepatic very low density lipoprotein (VLDL) secretion, which may be driven by increased delivery of FFAs to the liver coming from both visceral fat and upper body subcutaneous fat. The reduced HDL cholesterol concentrations and the increased small, dense LDL particle concentrations associated with upper body obesity are likely an indirect consequence of elevated triglyceride-rich VLDL. Increased cholesterol ester transfer protein activity and hepatic lipase activity can theoretically account for the atherogenic shifts in triglycerides and cholesterol between lipoproteins. Genetic influences play a significant role in the expression of these lipid abnormalities. Polymorphisms in the genes for apolipoprotein E, lipoprotein lipase, apolipoprotein B-100, and apolipoprotein A-II are correlated with increased triglycerides and decreased HDL.

Endocrine Manifestations of Obesity

Obesity is associated with abnormalities of the endocrine system, one of the most common being polycystic ovary syndrome. This syndrome (Chapter 236) is characterized by mild hirsutism and irregular menses or amenorrhea with anovulatory cycles. It is most commonly linked with obesity and often improves with weight loss and other treatments that improve insulin resistance. The insulin resistance associated with obesity may trigger the development of polycystic ovary syndrome in susceptible individuals. Whereas mild to moderate androgen overproduction is a feature of upper body obesity in women, obese men may suffer from mild to severe hypothalamic hypogonadism. This androgen deficiency improves with weight loss, and attempts to treat this condition with testosterone replacement offer little clinical benefit. There has been some concern that testosterone treatment of obese men may increase the risk of obstructive sleep apnea and perhaps even cardiovascular events. Although estrogens are not elevated in obese premenopausal women, they remain somewhat above postmenopausal levels in obese postmenopausal women. Serum growth hormone concentrations are commonly low in obese adults, but insulin-like growth factor-I concentrations are often normal, and growth hormone concentrations increase with weight loss. Treatment of these patients with growth hormone has been reported to worsen insulin resistance and glucose intolerance and cannot be justified, considering the costs and poor risk-to-benefit ratio.

Mechanical Complications of Obesity

The excess body weight associated with obesity is thought to be responsible for the increased prevalence of lower extremity degenerative joint disease. Extreme obesity can result in premature degenerative joint disease, and this may be especially difficult to treat surgically, given the greater stress on joint replacements. Severely obese individuals may also have problems with venous stasis, which is occasionally aggravated by right-sided heart failure (see later).

Obstructive Sleep Apnea and Sleep Restriction

Sleep apnea (Chapter 100) is common in severely obese patients, tending to be more prevalent in men and in women with an upper body/visceral obesity.

The risk of breast cancer and endometrial cancer is increased in obese women (Chapter 180). It is thought that this may be due to the increased estrogen levels associated with obesity in the postmenopausal woman. Obese men also have a higher mortality of cancers of the prostate and colon. The reasons for this association are unknown.

Gastrointestinal Disorders

Gastroesophageal reflux disease and gallstones are more prevalent in obese patients. Likewise, fatty liver and nonalcoholic steatohepatitis (Chapter 152) are more common in obesity. Nonalcoholic steatohepatitis can eventually progress to life-threatening hepatic cirrhosis. Weight loss and interventions that improve insulin sensitivity have been shown to improve fatty liver and nonalcoholic steatohepatitis.

DIAGNOSIS

Evaluation of Obesity

In the office practice, obtaining height and weight allows calculation of BMI. For patients with a BMI above 25 and below 35, a second piece of information—the waist circumference—provides an added indicator as to whether the patient is at greater risk for adverse consequences (see earlier). Measurement of blood pressure (which may require a large blood pressure cuff) then provides a third item of health information at almost no cost. The presence or absence of dyslipidemia (HDL cholesterol < 45 mg/dL for women, HDL cholesterol < 35 mg/dL for men, or triglycerides > 150 mg/ dL), hypertension, glucose intolerance and diabetes, and hyperuricemia should be documented. A history suggestive of sleep apnea should prompt a referral for overnight oximetry or a sleep disorder evaluation. A review of the patient’s lifestyle, including an assessment of physical activity level and eating habits, may help provide information about why the patient is obese. A family history of obesity, or long-standing obesity, provides evidence against a secondary cause of obesity. A careful medication history and social history may help the clinician identify precipitating factors that can be modified. By emphasizing the role of modifiable lifestyle factors that predispose to disease risk, as opposed to focusing solely on the patient’s weight, it may be possible to initiate a conversation about weight/disease management in a less threatening manner from the patient’s perspective. Before a patient enters a weight management program, it is helpful to ensure that the patient is interested and ready to make lifestyle changes and has realistic goals and expectations. Patients who expect to lose large amounts of weight in a short time are virtually doomed to disappointment. Medical treatment programs, even if they include pharmacotherapy, struggle to routinely achieve sustained weight loss of more than 10%. Although this amount of weight loss is sufficient to markedly reduce the medical compli­ cations of obesity, disappointment with “only” 10% weight loss may cause patients to abandon a medically successful program. Helping the patient to understand that lifestyle changes resulting in achievable (10%) weight loss is a reasonable, initial goal can be one of the more challenging aspects for a physician. It is sometimes necessary to delay entry into treatment programs if a patient is not ready to make lifestyle changes. In this case, a reasonable strategy is to remind the patient periodically of the potential health benefits of improved activity and eating habits. Once a willingness to make changes is apparent, treatment is more likely to succeed.

1464

CHAPTER 220  Obesity  

TREATMENT  Obesity represents an individual’s response to the environment based on genetics and learned behavior and is best viewed as a chronic disease. Therefore, treatment must be considered a long-term issue, much like diabetes, hypertension, or dyslipidemia. Substantial weight loss can be induced through severe calorie restriction, but without approaches to ensure behavioral changes, body fat is invariably regained. To the extent that environmental factors contribute to a patient’s overweight status, and to the extent that the macroenvironment is unlikely to change, patients must learn how to make permanent lifestyle changes (eating and activity behavior) to hope for permanent weight loss. Behavior modification approaches,5 which can help patients recognize and circumvent environmental cues for sedentary behavior and overeating, can increase the likelihood that patients will accomplish these lifestyle changes. A1 A randomized study has shown that intensive lifestyle intervention (as compared to only support and education) is associated with fewer hospitalizations, fewer medications, and lower health care costs in overweight or obese adults with type 2 diabetes. A2  Reducing energy intake is the most efficient and effective means to lose weight. For example, creating a 500 kcal/day deficit by reduced food intake will theoretically result in the loss of 1 pound of fat per week. Although possible, it is much more difficult to increase energy expenditure by 500 kcal/ week through exercise. Higher levels of physical activity can prevent weight

Patient encounter

Measure weight, height; calculate BMI

gain (or weight regain after weight loss). Some patients are able to change eating and activity habits on their own, given the proper information, whereas others require formal or informal behavior modification interventions (see later) to help make these changes. In some instances, pharmacotherapy or surgery may be needed for treatment of obesity. A flow diagram on how to evaluate and to manage patients with overweight and obesity is presented in Figure 220-1.

Diet

Changes in eating habits must be permanent if weight loss is to be maintained. An experienced registered dietitian can be helpful in the evaluation of a patient’s eating habits and will be able to provide the needed education. The diet history may identify eating behaviors that result in excess energy intake. Although it is important to address specific adverse eating behaviors, patients need to understand some general principles regarding diet. Reducing the energy density of food (most commonly accomplished by reducing dietary fat) can allow patients to feel satiated while consuming fewer calories. The NHLBI/ AHA/ACC/TOS obesity guideline6 recommends that providers prescribe 1200 to 1500 kcal/day for women and 1500 to 1800 kcal/day for men. Alternatively, diets that produce an energy deficit of 500 to 750 kcal/day can be recommended. Because there appears to be no clear superiority of one diet over another with regard to weight loss, A3  it is recommended that providers prescribe one of the evidence-based diets that restricts selected food types (e.g.,

BMI 25- 20 µg/dL.

ACTH stimulation test: ACTH1-24 (cosyntropin), 0.25 mg IM or IV. Cortisol is measured at 0, 30, and 60 min.

Normal response is GH > 4.1 µg/L. Normal response is GH > 3 µg/L.

In most normal individuals, the basal ACTH increases two- to four-fold and reaches a peak (20-100 pg/mL). ACTH responses may be delayed in cases of hypothalamic dysfunction. Cortisol levels usually reach 20-25 µg/dL. A normal response is cortisol > 18 µg/dL. In suspected hypothalamic-pituitary deficiency, a low-dose (1-µg) test may be more sensitive.

Thyroid-stimulating hormone (TSH)

Basal thyroid function tests: free T4, free T3, TSH

Low free thyroid hormone levels in the setting of TSH levels that are not appropriately increased

Luteinizing hormone (LH), follicle-stimulating hormone (FSH)

Basal levels of LH, FSH, testosterone, estrogen

Basal LH and FSH should be increased in postmenopausal women. Low testosterone levels in conjunction with low or low-normal LH and FSH are consistent with gonadotropin deficiency.

*Values are with polyclonal assays. CRH = corticotropin-releasing hormone; GHRH = growth hormone–releasing hormone; IM = intramuscularly; IV = intravenously; T3 = triiodothyronine; T4 = thyroxine.

TABLE 224-3  HORMONAL REPLACEMENT THERAPY IN HYPOPITUITARISM* PITUITARY AXIS

HORMONAL REPLACEMENTS

Growth hormone (GH)

In children, GH (0.25 mg/kg) SC daily. In adults, GH (0.3-1.2 mg) SC daily. Titrate dose to achieve IGF-I levels in middle to upper part of normal range. Women receiving oral estrogens require higher doses.

Prolactin

None

Adrenocorticotropic hormone–cortisol

Hydrocortisone (10-15 mg PO q am; 5-10 mg PO q pm) or prednisone (2.5 mg PO q am; 2.5 mg PO q pm). Dose adjusted on clinical basis. Stress dosing: 50-75 mg hydrocortisone IV q8h

Thyroid-stimulating hormone–thyroid

l-thyroxine (0.075-0.15 mg) PO daily

Gonadotropins–gonads

FSH and LH (or HCG) can be used to induce ovulation in women. HCG alone or with FSH can be used to induce spermatogenesis in men. In men, testosterone enanthate (100-300 mg) IM q1-3wk or testosterone cyclopentylpropionate (100-300 mg) IM q1-3wk. Testosterone transdermal patches can also be used (5 mg daily). Testosterone gel 5-10 g daily. In women, conjugated estrogens (0.625-1.25 mg) PO days 1-25 each month, cycled with medroxyprogesterone acetate (5-10 mg) PO days 15-25 each month. Low-dose contraceptive pills may also be used. Estrogen-containing transdermal patches are also available.

Posterior pituitary

Desmopressin, 0.05-0.2 mL (5-20 µg) intranasally once or twice daily, or tablets (0.1-0.4 mg q8-12h) or 0.5 mL (2 µg) SC

*Replacement therapy is dictated by the types of hormone deficiencies and by the clinical circumstances. In each case, the recommended preparations and doses are representative but need to be adjusted for individual patients. Other hormonal preparations are also available. FSH = follicle-stimulating hormone; GnRH = gonadotropin-releasing hormone; HCG = human chorionic gonadotropin; IGF-I = insulin-like growth factor-I; IM = intramuscularly; LH = luteinizing hormone; PO, orally; SC = subcutaneously.

  PITUITARY TUMORS PATHOBIOLOGY

Pituitary tumors are classified according to the hormones they produce and their size: microadenomas, less than 10 mm in diameter; macroadenomas, more than 10 mm in diameter; and macroadenomas with extrasellar extension. In general, the levels of hormones produced by the tumors parallel the size of the tumors. The prevalence of the different types of pituitary adenomas, based on surgical data, is summarized in Table 224-4. Immunohistochemical studies using antibodies specific for each of the major pituitary hormones have been used to define tumor phenotype. Pituitary adenomas are rarely malignant but can be locally invasive. Most pituitary tumors are monoclonal. This finding does not exclude a role for hormonal stimulation as a predisposing factor for somatic mutations, and the hormonal environment may also affect the rate of tumor growth (e.g., the growth of ACTH-secreting tumors following bilateral adrenalectomy). Supporting the concept that somatic mutations lead to pituitary tumorigenesis, a subset (35 to 40%) of somatotroph adenomas have activating mutations in the gene for the Gsα-subunit, resulting in two different amino acid (Arg201 and Glu227) substitutions.7 Either mutation causes the Gsα-subunit to

stimulate adenylyl cyclase in a constitutive manner. The elevated intracellular cyclic adenosine monophosphate levels lead to increased cell growth as well as GH production. Mutations in other oncogenes, such as ras, Rb, and p53, are uncommon in pituitary tumors. Thus, the nature of the somatic mutations causing most pituitary tumors remains unknown. At least five types of inherited predispositions to pituitary tumors are recognized. Patients with McCune-Albright syndrome (Chapter 231) occasionally develop pituitary adenomas as well as characteristic abnormalities in other tissues, particularly the ovary, bone, and thyroid. Interestingly, the McCune-Albright syndrome is also caused by mutations in the gene for the Gsα-subunit. However, the somatic mutations in McCune-Albright occur early during development, so that multiple tissues are affected. In multiple endocrine neoplasia type 1 (MEN 1) (Chapter 231), the menin gene is mutated, so that the predisposition to pituitary tumors is inherited in an autosomal dominant manner and occurs in conjunction with tumors of the parathyroid and pancreas.8 Familial isolated pituitary adenoma (FIPA) syndrome is autosomal dominant and has low or variable penetrance. Germline mutations have been found in the gene for the aryl hydrocarbon receptor– interacting protein (AIP), which functions as a tumor suppressor. Such mutations have been found in about one third of FIPA families, most commonly

1484

CHAPTER 224  Anterior Pituitary  

TABLE 224-4  PREVALENCE OF DIFFERENT TYPES OF PITUITARY ADENOMAS TYPE OF PITUITARY ADENOMA

DISORDER

HORMONE PRODUCED

PREVALENCE (%)*

Somatotroph

Acromegaly and gigantism

Growth hormone

10-15

Lactotroph (prolactinoma)

Hypogonadism, galactorrhea

Prolactin

25-40

Corticotroph

Cushing’s disease

Adrenocorticotropic hormone

10-15

Gonadotroph

Mass effects, hypopituitarism

Follicle-stimulating hormone and luteinizing hormone

15-20

Thyrotroph

Hyperthyroidism

Thyroid-stimulating hormone

Nonfunctioning/ null cell

Mass effects, hypopituitarism

None

TREATMENT  Surgery

50% reduction in tumor size), but such size reduction is seen in only about two thirds of patients treated with bromocriptine. Improvements in visual field defects can be seen in about 90% of patients with defects when treated with cabergoline. Thus, it is reasonable to use cabergoline as first-line therapy even in patients with visual field defects, so long as visual acuity is not threatened by rapid progression or recent tumor hemorrhage. Many patients treated with cabergoline whose tumors shrink to the point of nonvisualization on MRI and whose PRL levels are normal can maintain normal PRL levels and not experience tumor reexpansion after therapy has been tapered off. In some cases, prolactinomas appear to be resistant to a dopamine agonist. In these cases, switching from bromocriptine to cabergoline may be successful. Larger-than-standard doses (>2 mg/week) of cabergoline may be effective in normalizing PRL levels. The very high doses of cabergoline used in patients with Parkinson’s disease have been associated with cardiac valvular abnormalities; such abnormalities have not been found with conventional doses of cabergoline used in patients with prolactinomas, but monitoring with echocardiography may be prudent in patients taking larger-than-standard doses. Alternatively, transsphenoidal surgery may be used. Although initial remission rates (80 to 90%) for transsphenoidal surgery of microprolactinomas are good, there is long-term recurrence in about 20% of patients. For macroprolactinomas, the initial remission rates with surgery are closer to 30%, with a similar recurrence rate. Radiation therapy, usually stereotactic, is reserved for patients with macroadenomas not responding to either medical or surgical treatment. Dopamine agonist therapy for infertility, or when there is a possibility of pregnancy, deserves special consideration. These medications can induce ovulation in 80 to 90% of patients with hyperprolactinemia. Although neither bromocriptine nor cabergoline has been associated with congenital malformations, they should be stopped once pregnancy has been achieved. A form of barrier contraception is usually recommended until two to three regular menstrual cycles have occurred. Subsequently, pregnancy can be confirmed if a menstrual period is missed, allowing discontinuation of medication with exposure of the fetus to the drug for only 3 to 5 weeks. At present, the safety data for pregnancy outcome are more limited for cabergoline; therefore, some clinicians prefer bromocriptine when fertility is desired. Less than 3% of patients with microadenomas, but 23% of patients with macroadenomas, develop symptoms of tumor enlargement (headaches, visual field defects) during pregnancy (Fig. 224-6). If symptoms develop, MRI and formal visual field testing should be performed. If there is evidence of visual field compromise or tumor growth, dopamine agonist therapy should be restarted to shrink the tumor. PRL levels are not very useful because they are normally increased in pregnancy and an enlarging tumor may not cause PRL production to increase substantially. Because problems of tumor growth occur most  often in patients with macroadenomas, consideration can also be given to the option of transsphenoidal decompression before pregnancy in women with large tumors, so long as fertility can be preserved. If the patient is far advanced in her gestation at the time tumor growth occurs, consideration could also be given to delivering the baby.

  ADRENOCORTICOTROPIC HORMONE

ACTH is a 39–amino acid peptide that is derived from a precursor polypeptide, proopiomelanocortin (POMC; 241 amino acids), which encodes several peptides, including ACTH and β-lipotropin (Chapter 223). The biologically active portion of ACTH resides within the first 18 of its 39 amino acids. However, because a synthetic peptide (cosyntropin) that includes the first 24 amino acids has a longer half-life, it is used clinically to assess adrenocortical function. In cases with neoplastic ectopic production of ACTH, the levels of precursor peptides or their processed products may be elevated.

CHAPTER 224  Anterior Pituitary  

A

B

C

D

1489

FIGURE 224-6.  Magnetic resonance image showing enlargement of prolactinoma during pregnancy. Above, Coronal (A) and sagittal (B) views show intrasellar prolactin-secreting macroadenoma prior to conception. Below, Coronal (C) and sagittal (D) views show enlargement of the prolactinoma at 7 months’ gestation. (From Molitch ME. Medical treatment of prolactinomas. Endocrinol Metab Clin North Am. 1999;28:143.)

The primary effect of ACTH is to stimulate the adrenal gland to produce cortisol. It also stimulates secretion of adrenal androgens and mineralocorticoids, although production of mineralocorticoids is controlled primarily through non–ACTH-dependent mechanisms (i.e., the renin-angiotensin system) (Chapter 227). Consequently, mineralocorticoid function is preserved in ACTH deficiency, in contrast to primary adrenal insufficiency, which is characterized by loss of glucocorticoid and mineralocorticoid function. Long-term stimulation by ACTH causes adrenal hyperplasia and enlargement. On the other hand, ACTH deficiency leads to adrenal atrophy. Hypothalamic corticotropin-releasing hormone (CRH) is the most important stimulator of ACTH secretion. Chronic stimulation by CRH causes corticotroph cell hyperplasia, which can be seen in cases of ectopic CRH production. Cortisol inhibits ACTH secretion, blunts the ACTH response to CRH, and inhibits CRH production. After prolonged glucocorticoid suppression of the hypothalamic-pituitary-adrenal axis, the amount of endogenous CRH secretion appears to be rate limiting and can require several months to recover. Plasma ACTH is secreted in discrete pulses (10 to 80 pg/mL), so random measurements are of little value. Most clinical tests are therefore based on levels of cortisol or its metabolites, which tend to integrate the effects of ACTH. ACTH and cortisol secretion exhibit marked diurnal rhythms, being greatest at night several hours after the initiation of sleep. Cortisol levels are highest in the early morning and reach a nadir in the late afternoon and evening. Patients with Cushing’s disease lose or exhibit a blunted diurnal rhythm of ACTH and cortisol secretion. ACTH secretion can be stimulated by a variety of different forms of stress, including psychological stimuli such as fright, anticipation of athletic competition, or surgery. Depression is associated with activation of the hypothalamic-pituitary-adrenal axis and impaired dexamethasone suppressibility. Hypoglycemia induces ACTH

secretion through a central mechanism. The resulting increase in cortisol secretion represents one of several counter-regulatory mechanisms that increase glucose production. Insulin-induced hypoglycemia provides a mechanism for testing the integrity of the hypothalamic-pituitary-adrenal axis (see Table 224-2). Serious trauma and infection activate an array of cytokines that stimulate CRH and ACTH secretion. Because cortisol levels are often increased substantially in these circumstances, similar adjustments in cortisol replacement doses may be required in seriously ill patients with adrenal insufficiency.

Adrenocorticotropic Hormone Deficiency: Secondary Hypocortisolism Secondary hypocortisolism causes symptoms of glucocorticoid deficiency, including nausea, vomiting, weakness, fatigue, fever, and hypotension. In addition to reduced levels of cortisol, abnormal laboratory test findings can include hyponatremia, hypoglycemia, and eosinophilia. Depending on its cause, the severity of cortisol deficiency in cases of secondary adrenal insufficiency is often not as marked as in primary adrenal insufficiency (Chapter 227). In addition, mineralocorticoid function is preserved in secondary adrenal deficiency. Consequently, the clinical manifestations of volume depletion are less pronounced, and hyperkalemia is not a feature of ACTH deficiency. Because ACTH levels are low, hyperpigmentation is not seen as in primary adrenal insufficiency. In women, reduced adrenal androgens can decrease libido and cause loss of axillary and pubic hair. The most common cause of ACTH deficiency is treatment with exogenous glucocorticoids, which causes suppression of the hypothalamic-pituitaryadrenal axis. Sudden withdrawal of glucocorticoids or an increased requirement induced by the superimposition of severe illness can elicit symptoms of glucocorticoid deficiency. Congenital forms of ACTH deficiency are rare. When present, ACTH deficiency usually occurs in combination with the loss

1490

CHAPTER 224  Anterior Pituitary  

of other pituitary hormones, although acquired, isolated ACTH deficiency does occur, particularly in women with lymphocytic hypophysitis. ACTH reserve is most often evaluated using the insulin tolerance test. Caution should be exercised before inducing hypoglycemia in patients with suspected adrenal insufficiency. Insulin-induced hypoglycemia stimulates central responses to neuroglycopenia (Chapter 230) and mimics some stresses that activate ACTH secretion. ACTH stimulation tests using ACTH1-24 (cosyntropin) can accurately evaluate primary adrenocortical insufficiency but may less accurately assess secondary adrenal insufficiency. A variation of the ACTH stimulation test using the low dose of 1 µg has been found to be useful for diagnosing secondary adrenal insufficiency in some studies. Deficiency of ACTH is treated by replacement with glucocorticoids. Doses need to be individualized and are based largely on clinical criteria in which symptoms of glucocorticoid deficiency are balanced against features of glucocorticoid excess. Typical amounts of hydrocortisone are in the range of 15 to 20 mg/day in divided doses. Such doses are usually doubled in the event of mild to moderate illness. Patients should wear MedicAlert tags and be instructed in the warning signs of cortisol deficiency: nausea, vomiting, abdominal pain, low-grade fever, fatigue, and postural dizziness. Emergency injection kits of hydrocortisone are frequently provided for home use in the event that vomiting precludes taking oral steroids, or for severe sudden stress (e.g., a fracture). Stress doses of steroids should be used during times of illness. Current recommendations call for doses in the range of 50 to 75 mg every 8 hours for severe stress. Mineralocorticoid replacement is not required in patients with ACTH deficiency.

Cushing’s Disease

PATHOBIOLOGY

Cushing’s disease results from a pituitary adenoma that causes excess production of ACTH. It should be distinguished from a variety of other causes of Cushing’s syndrome (glucocorticoid excess), which include adrenal causes (adenomas, carcinomas) of cortisol excess, ectopic production of ACTH and CRH, and physiologic states that result in overproduction of cortisol. Cushing’s disease accounts for 60 to 70% of cases of Cushing’s syndrome. Ten to 15% of pituitary tumors secrete ACTH. Cushing’s disease occurs about eight times more often in women than in men. Most ACTH-producing pituitary neoplasms, like other pituitary tumors, are monoclonal, implying a primary defect in corticotroph cells. In addition, there are rare cases of corticotroph hyperplasia causing Cushing’s syndrome that are secondary to CRH production by either adjacent CRH-producing intrasellar gangliocytomas or ectopic CRH-producing cancers. Most (80 to 90%) of the ACTH-secreting tumors are microadenomas at the time of diagnosis. The clinical features of cortisol excess may allow detection of corticotroph adenomas before they have grown to a larger size. High levels of cortisol may also restrain tumor growth. ACTH-secreting macroadenomas may be locally invasive.

CLINICAL MANIFESTATIONS

The clinical features of Cushing’s disease are caused by the effects of excess glucocorticoids and by the hypersecretion of ACTH and other POMC peptide products. The severity of the features of Cushing’s disease varies greatly and appears to reflect not only the level of free cortisol but also the duration of the disease and perhaps the sensitivity to glucocorticoid action. In florid cases of Cushing’s disease (Fig. 224-7), the constellation of symptoms and physical features is readily recognized. Early in the disease or in mild cases, it can be challenging to distinguish the clinical features of Cushing’s disease from similar traits that are seen in the normal population. Clinical suspicion is of paramount importance. On the other hand, one must be discriminating and not formally evaluate everyone with obesity, hypertension, and glucose intolerance. Of the many features listed in Table 224-6, some are relatively specific for Cushing’s disease. For example, the centripetal distribution of fat with the characteristic “buffalo hump,” “moon facies,” and deposition of fat in the supraclavicular area but not in the extremities is much more specific than generalized obesity. Striae that are wide (>1 cm) and purple reflect steroid-induced thinning of the dermis and can be distinguished from the more common “stretch marks.” Numerous spontaneous ecchymoses also occur because of thinning of the skin and capillary fragility. Proximal muscle weakness represents another manifestation of glucocorticoid excess. Osteopenia and hypokalemia, when present, provide objective evidence consistent with ACTH excess. Hypokalemia results from the effects of ACTH on mineralocorticoid production but also from the ability of high levels of cortisol to saturate 11β-dehydrogenase, an enzyme in the kidney that

A

B

FIGURE 224-7.  (A) This 30 year old woman initially presented with a three year history of increasing facial hair, facial rounding, abdominal obesity, hypertension, diabetes, and oligomenorrhea. She had no muscle weakness or pigmented striae. (B) Following successful transsphenoidal resection of her ACTH-secreting microadenoma, she had a dramatic improvement in her clinical appearance with resolution of her diabetes and hypertension.

TABLE 224-6  CLINICAL FEATURES OF CUSHING’S DISEASE GENERAL Obesity (centripetal distribution) “Moon facies” and mild proptosis Increased supraclavicular fat and “buffalo hump” Hypertension SKIN Hyperpigmentation Facial plethora Hirsutism Violaceous striae and thin skin Capillary fragility and easy bruising Acne Edema MUSCULOSKELETAL Muscle weakness (proximal) Osteoporosis and back pain REPRODUCTIVE Decreased libido Oligomenorrhea and amenorrhea NEUROPSYCHIATRIC Depression Irritability and emotional lability Psychosis METABOLIC Hypokalemia and alkalosis Hypercalciuria and renal stones Glucose intolerance or diabetes mellitus Impaired wound healing Impaired resistance to infection Granulocytosis and lymphopenia TUMOR MASS EFFECTS Headache Visual field loss Hypopituitarism

inactivates cortisol. As a result, cortisol can “spill over” and act on mineralocorticoid receptors in the distal tubule. The hyperpigmentation associated with Cushing’s disease is not as striking as that seen in Addison’s disease or in ectopic ACTH syndrome, but in association with other findings, it should raise the suspicion of Cushing’s disease and help distinguish it from adrenal causes of hypercortisolemia. Hirsutism and acne are caused by increased production of adrenal androgens and are more prominent in patients with Cushing’s disease than in those with adrenal adenomas, in whom glucocorticoids tend to be the predominant product. Oligomenorrhea and amenorrhea probably have several causes, including androgen effects on the reproductive axis and glucocorticoid inhibition of GnRH, which may also account for diminished libido. Hypertension and glucose intolerance are caused by glucocorticoid excess. Immunosuppression, venous thrombo-

1491

CHAPTER 224  Anterior Pituitary  

TABLE 224-7  TESTS USED IN THE DIFFERENTIAL DIAGNOSIS OF CUSHING’S SYNDROME* ETIOLOGY

OVERNIGHT DEXAMETHASONE SUPPRESSION TEST

PLASMA ACTH

CORTICOTROPIN-RELEASING HORMONE STIMULATION OF ACTH

PETROSAL-TO-PERIPHERAL ACTH RATIO

Normal

Suppression

Normal

Normal

Pituitary

No suppression

Normal or high

Normal or increased

>2

Ectopic

No suppression

High or normal

No response

200 pg/ mL) and can be associated with hyperpigmentation. Clinical features of Cushing’s syndrome may be altered by the rapid onset of extreme hyper­ cortisolemia coincident with elements of tumor cachexia. Pronounced weakness, fluid retention, glucose intolerance, hypokalemia, and poor skin integrity are often seen. Ectopic ACTH syndrome is readily recognized in its classic form. However, a subset of tumors, particularly carcinoids (Chapter 232), exhibit dexamethasone suppression that is similar to that seen with pituitary adenomas. When suspected, carcinoids can sometimes be detected by CT or MRI, but many are too small to be seen even with these techniques.

Because of these exceptions to the high-dose dexamethasone test, a variety of procedures have been devised in an attempt to further distinguish ectopic and pituitary dependent sources of ACTH. CRH testing may also prove useful, with pituitary tumors exhibiting an increase in ACTH and tumors making ACTH ectopically having little or no response. In recent years, inferior petrosal sinus sampling has been used to distinguish pituitary and ectopic sources of ACTH when the source of ACTH is not obvious based on the clinical circumstances, biochemical evaluation, and imaging studies. This test requires an experienced radiologist for safe and effective catheterization of the petrosal sinuses. Blood samples are taken simultaneously from the left and right petrosal sinuses and from the periphery before and after CRH stimulation. In the case of ACTH-producing pituitary adenomas, there is a gradient in ACTH levels between the central and peripheral blood specimens. When clinical and biochemical studies suggest the presence of a pituitary adenoma, pituitary imaging should be performed using CT or MRI. Most ACTH-secreting pituitary adenomas are small, and scans are normal in more than half of patients.

TREATMENT  The efficacy of transsphenoidal surgery for Cushing’s disease is greatly aided by making the correct diagnosis preoperatively. In experienced hands, surgical cures of ACTH-producing microadenomas occur in 75 to 90% of patients undergoing a first operation. As in other pituitary tumors, complete remissions with macroadenomas are much less common. In the event of surgical remission or cure, postoperative hypocortisolism is to be expected because of suppression of the hypothalamic-pituitary axis. After coverage for steroid withdrawal in the postoperative period, cortisol replacement should gradually be decreased to allow recovery of the hypothalamic-pituitary-adrenal axis; recovery may take up to 1 year. If transsphenoidal surgery is unsuccessful, reoperation may be indicated and can result in remission in up to 50% of patients18; in this circumstance, consideration should be given to performing a total hypophysectomy at reoperation. If transsphenoidal surgery cannot be performed or has failed, alternative forms of therapy should be used to prevent the long-term consequences of hypercortisolism.19 Pituitary irradiation is often the second line of treatment for Cushing’s disease. It is more efficacious in children and in younger patients, but even in older adults, remissions can be achieved in about 50% within 2 years. To prevent the continued ravages of hypercortisolism during this period, however, concomitant medical therapy (see later) is usually given. Bilateral adrenalectomy represents another alternative for patients with severe hypercortisolism after transsphenoidal surgery. It rapidly and effectively lowers cortisol levels but is associated with relatively high morbidity and mortality rates (as high as 5%) because of the associated metabolic and immune system alterations caused by hypercortisolism. The morbidity has been reduced in recent years by introduction of the laparoscopic approach. After adrenalectomy, patients must be maintained on glucocorticoids and mineralocorticoids and are at risk for the development of Nelson syndrome (see later). Medical therapy for Cushing’s disease has its primary role in preparation for surgery or control of hypercortisolism following unsuccessful surgery. It may also be used during the interval when radiation therapy is taking effect (Fig. 224-8). The antifungal agent ketoconazole is effective in decreasing glucocorticoid biosynthesis and also inhibits ACTH secretion, so it has been the most common medical therapy used; it has hepatotoxicity and is not actually approved for use for Cushing’s syndrome. Other drugs that interfere with cortisol synthesis (e.g., mitotane, etomidate, metyrapone) have been used less commonly. In a few small studies, cabergoline has also been shown to cause a normalization of cortisol levels in about one third of patients with Cushing’s disease, although it has not been approved for this indication. Recently,  two new drugs have been approved for the treatment of Cushing’s disease. Mifepristone is a progesterone receptor blocker that also is a glucocorticoid receptor blocker and is highly effective in improving clinical signs and symptoms of Cushing’s syndrome; because of its mechanism of action, with

1492

CHAPTER 224  Anterior Pituitary  

GONADOTROPINS (FOLLICLE-STIMULATING HORMONE AND LUTEINIZING HORMONE)

CRH Pituitary

Cabergoline Pasireotide

ACTH

Adrenals

Ketoconazole Mitotane Metyrapone Etomidate

Cortisol

Mifepristone GR Tissues

FIGURE 224-8.  Medical therapies for Cushing’s disease. Cabergoline and pasireotide act to decrease ACTH secretion from the corticotroph tumor. Several drugs (ketoconazole, metyrapone, mitotane, etomidate) work at the adrenal level to decrease cortisol synthesis. Mitotane works at the adrenal level by blocking cortisol action at the glucocorticoid receptor. ACTH = adrenocorticotropic hormone; CRH = corticotropin-releasing hormone; GR = glucocorticoid receptor. (Adapted from Petersenn S. Medical management of Cushing’s disease. In: Swearingen B, Biller BMK (eds) Endocrine updates vol. 31: Cushing’s disease. 2011, Springer, New York, pp 167–182, Figure 1. As adapted from Petersenn S, Endocrine Updates, 2011, Springer Science+Business Media B.V.)

The pituitary glycoprotein hormones include FSH, LH, and TSH. Chorionic gonadotropin, which is structurally very similar to LH, is made in the placenta. Each of the glycoprotein hormones has a specific β-subunit that forms a noncovalently bound dimer with the common α-subunit. The αand β-subunits each undergo glycosylation, which is important for correct hormone folding, intracellular transport, and secretion. Glycosylation is also required for biologic activity, presumably because of effects on the tertiary structure of the hormones. The gonadotropins are involved in sexual differentiation, sex steroid production, and gametogenesis. The regulation and physiologic roles of gonadotropins are quite different in males and females. In males, receptors for FSH are located on Sertoli cells and seminiferous tubules, whereas LH receptors are located on Leydig cells in the testis. LH stimulates androgen production by the Leydig cells. FSH is involved primarily in sperm maturation in the seminiferous tubules. Thus, FSH and LH act together to induce spermatogenesis (Chapter 234). In females, ovarian FSH receptors are located on granulosa cells, where they induce enzymes involved in estrogen biosynthesis. LH receptors are located predominantly on thecal cells in the ovary and stimulate the production of ovarian androgens and steroid precursors that are transported to granulosa cells for aromatization to estrogens. The pattern of FSH and LH secretion during the menstrual cycle results in follicular recruitment and maturation (largely FSH mediated), followed by ovulation (largely LH mediated) and steroid production by the corpus luteum (Chapter 235). Gonadotropin secretion is regulated primarily by the hypothalamic decapeptide GnRH. The gonadotroph cell is exquisitely sensitive to the pattern of GnRH stimulation. Continuous, rather than pulsatile, exposure to GnRH causes gonadotroph desensitization and suppression of LH and FSH. Gonadotroph sensitivity to GnRH is modulated by sex steroids and probably other hypothalamic peptides such as neuropeptide Y. Increased GnRH secretion, in combination with a higher density of GnRH receptors and rising estradiol concentrations, accounts in part for the dramatic release of gonadotropins that induces ovulation.

Hypogonadotropic Hypogonadism

CLINICAL MANIFESTATIONS AND DIAGNOSIS

treatment, cortisol and ACTH levels actually rise while the clinical signs improve. The higher levels of cortisol may “spill over” to the mineralocorticoid receptor, causing a blood pressure rise and hypokalemia. Symptoms of glucocorticoid withdrawal and even adrenal insufficiency may occur. Blockade of the progesterone receptor by mifepristone may cause menorrhagia. Pasireotide is a somatostatin receptor analog that has much greater activity on corticotroph adenomas than other somatostatin analogs because it has additional activity at the somatostatin-5 receptor. A large multicenter study has recently shown clinical and biochemical efficacy in patients with Cushing’s disease A3 ; the major adverse effect is hyperglycemia and a worsening of preexisting diabetes mellitus. This worsening of glucose levels is likely due to a reduction in insulin and glucagon-like peptide (GLP)-1 levels.

Nelson Syndrome Nelson syndrome was initially described as the appearance of a pituitary adenoma after bilateral adrenalectomy. In addition to an enlarging pituitary mass, the syndrome is characterized by very high ACTH levels and hyperpigmentation. It is caused by a preexisting ACTH-producing tumor that grows in the absence of feedback inhibition by high levels of glucocorticoids. The incidence of clinically significant Nelson syndrome after adrenalectomy for Cushing’s disease varies from 10 to 50% in different series.20 Patients with Cushing’s disease who have undergone adrenalectomy should be followed with imaging studies and plasma ACTH levels, because tumors that cause Nelson syndrome can be very aggressive. When there is evidence of mass effects or rapid growth, transsphenoidal surgery should be performed. Postoperative irradiation may provide additional benefit, although it appears to be less efficacious than in other ACTH-producing adenomas. In theory, Nelson syndrome could also occur in patients being treated with mifepristone, and such patients should also be monitored with periodic pituitary MRI scans, although short-term follow-up data are reassuring at this point.

Clinical features of hypogonadotropic hypogonadism in women are primarily due to estrogen deficiency and include breast atrophy, vaginal dryness, and diminished libido. Hot flashes are uncommon, in contrast to postmenopausal estrogen deficiency. In premenopausal women, normal menstrual cycles provide evidence for an intact hypothalamic-pituitary-gonadal axis. LH and FSH levels should be increased in postmenopausal women, and normal levels may indicate deficiency. Hypogonadism in men causes decreased libido and sexual function. In men, low testosterone without elevation of LH and FSH is consistent with impaired hypothalamic-pituitary reserve. GnRH stimulation can distinguish hypothalamic and pituitary deficiency but may require multiple injections to prime the pituitary. A congenital form of hypogonadotropic hypogonadism is caused by deficiency of GnRH, which in turn causes deficiencies of LH and FSH. When associated with anosmia (absent sense of smell), the condition is referred to as Kallmann’s syndrome (Chapter 223). Secondary hypogonadotropic hypogonadism is relatively common. In most cases, it is reversible and is caused by weight loss, anorexia nervosa, stress, heavy exercise, or severe illness. Reversible forms of secondary hypogonadotropic hypogonadism are caused by GnRH deficiency and are more common in women than men. A variety of pathologic conditions can cause secondary hypogonadotropic hypogonadism, often in association with deficiencies of other pituitary hormones (see Table 224-1). These include hypothalamic lesions and central nervous system irradiation. Pituitary tumors can suppress gonadotropins because of stalk compression and disruption of pulsatile GnRH input, as well as by direct destruction of normal pituitary tissue. Hyperprolactinemia can suppress GnRH and lead to reduced gonadotropin levels. In contrast to the aforementioned causes of hypogonadotropic hypogonadism that result from GnRH deficiency, primary deficiencies of LH and FSH are uncommon. An acquired form of isolated gonadotropin deficiency is rarely encountered and may have an autoimmune basis. Mutations in the LHβ or FSHβ genes have been described in case reports and cause selective loss of individual gonadotropins. Inactivating mutations in the GnRH

CHAPTER 224  Anterior Pituitary  

In women, the pattern of GnRH pulse frequency varies across the menstrual cycle (Chapter 235). The combination of GnRH stimulation in conjunction with ovarian feedback regulation results in a complex orchestration of positive and negative hormonal signals that converge at the gonadotroph to regulate LH and FSH secretion. The typical 28-day menstrual cycle is divided into follicular and luteal phases that are separated by ovulation on day 14. Unlike chronic exposure to low concentrations of estrogens, which exert negative feedback regulation and inhibit GnRH, the increasing concentration of estrogen before the LH surge exerts positive feedback regulation that results in increased GnRH pulse frequency. Increased GnRH in combination with increased gonadotroph sensitivity to GnRH results in the LH-FSH surge. During the luteal phase, the gonadotropin pulse frequency is reduced. In addition to feedback regulation by steroids, ovarian peptides such as inhibin also play a role in control of the reproductive axis. Inhibin causes selective suppression of FSH without affecting LH secretion. A homodimer of inhibin β-subunits, referred to as activin, has opposite actions and

1492.e1

selectively stimulates FSH, but its predominant physiologic action is to increase ovarian granulosa cell responsivity to FSH. Circulating inhibin provides one of the negative feedback inputs that leads to FSH suppression as the follicle develops. The perimenopause is characterized by a gradual cessation of ovarian function. After several years of menstrual cycles that are sometimes anovulatory or irregular, menses cease, thereby defining the menopause (Chapter 240). At menopause, the decline in estrogen and progesterone causes loss of feedback inhibition and a marked increase in LH and FSH levels. In males, the regulation of the hypothalamic-pituitary-gonadal axis is relatively constant. Testosterone inhibits the hypothalamic-pituitary axis, although its actions are mediated in part by aromatization to estrogens. Most of the inhibition by gonadal steroids occurs at the hypothalamic level. In contrast to menopause in women, there is no analogous abrupt change in hormone levels in men. There is, however, a gradual decline in testosterone levels associated with an increase in LH and FSH with aging.

CHAPTER 224  Anterior Pituitary  

receptor and the LH and FSH receptors causing hypogonadotropic hypogonadism have also been reported.

TREATMENT  In premenopausal women, preparations of estrogen and progestins should be used for hormonal replacement and to allow cyclical growth of the endometrium. Pulsatile GnRH (for GnRH-deficient patients) has been given to induce ovulation and fertility but is not commonly used at present. Gonadotropin injections are more commonly used when fertility is desired. Testosterone can be replaced in men, using intramuscular injections that are given at 2- to 4-week intervals. Doses and the intervals between injections should be adjusted on an individual basis using libido and testosterone levels before the next injection as a guide. Oral preparations of androgens should be avoided because of hepatotoxicity. Transdermal patch and gel preparations are also available and maintain more stable testosterone levels but are more expensive. With gels, care has to be used to prevent exposure of the partner or children. Although induction of spermatogenesis can be achieved using pulsatile GnRH (for GnRH-deficient patients), injections of gonadotropins are more commonly used. Pulsatile GnRH has been used to induce puberty and fertility in both males and females with Kallmann’s syndrome and other forms of GnRH deficiency, but more commonly, injections of gonadotropins are used. Secondary hypogonadotropic hypogonadism is ideally treated by correcting the underlying cause.21 Many women have a discrete threshold for weight or exercise level that will cause loss of menstrual periods. When it is not possible to correct the underlying abnormality, hormonal replacement can be used in women for protection against osteopenia and to cycle the endometrium. Permanent idiopathic hypogonadotropic hypogonadism can also occur in both sexes and will require hormone replacement.

Follicle-Stimulating Hormone– and Luteinizing Hormone–Producing Tumors

PATHOBIOLOGY

The majority (70 to 80%) of pituitary tumors classified previously as nonfunctioning adenomas can be shown to produce low levels of intact glycoprotein hormones or their uncombined α- or β-subunits. Biosynthetic defects in the tumor cells account for relatively inefficient hormone secretion as well as the propensity to produce uncombined subunits. FSH is produced more commonly than LH. Elevated levels of free α-subunits are noted more often than increased free β-subunits.

CLINICAL MANIFESTATIONS

Gonadotropin-producing tumors are somewhat more common in men than women and increase in prevalence with age. FSH- and LH-producing tumors do not usually cause a characteristic hormone excess syndrome. The tumors, typically large macroadenomas, present as clinically nonfunctioning tumors with symptoms and signs related to local mass effects. Visual field loss is found in more than 70% of patients. Many are detected incidentally by CT and MRI performed for unrelated indications. Symptoms of hypogonadism with loss of libido are also common. Men with predominantly FSH-secreting tumors may present with testicular enlargement from hypertrophy of the seminiferous tubules but may also be hypogonadal due to low levels of testosterone. These patients must be distinguished from those with primary hypogonadism due to testicular dysfunction. Tumors that primarily secrete LH are rare but can cause increased testosterone levels. Premenopausal women with gonadotropin-producing tumors may experience menstrual irregularity or secondary hypogonadism. Postmenopausal women often show reduced gonadotropin levels because the mass effects of the gonadotropin-producing tumors cause stalk compression, impairing GnRH stimulation of gonadotropins from normal pituitary cells.

DIAGNOSIS

Because of the absence of a clinical syndrome in most patients, almost all gonadotropin-producing pituitary tumors are diagnosed by postoperative immunohistochemistry, because they had presented with mass effects. There is no particular clinical benefit to distinguish whether a nonfunctioning adenoma is truly a gonadotroph adenoma. Some patients can have moderately elevated PRL levels that are caused by tumor mass effects. It is important to distinguish this group from patients with true prolactinomas. As noted earlier, many women, including those in the postmenopausal group, have

1493

paradoxically low gonadotropin levels. Thus, the absence of elevated gonadotropins does not exclude the diagnosis of a gonadotropin-producing tumor.

TREATMENT  Because the major symptoms of the gonadotropin-producing tumors are due to extrasellar extension and local mass effects, the main aim of treatment is reduction in tumor size. Complete or partial reversal of visual field defects and hypopituitarism can be accomplished by surgery unless these conditions have been of long standing. Transsphenoidal surgery is rarely curative, however, because of the large size of the tumors. Patients with significant residual tumor may benefit from radiation therapy. Because most tumors are slow growing, when no tumor is visible postoperatively by MRI, the patient may be followed with yearly monitoring for tumor recurrence, using visual fields and CT or MRI. If tumor markers such as free α- or β-subunit levels are available, they can also be used to monitor tumor function. When follow-up studies show tumor regrowth, repeat surgery, radiation therapy, or both are indicated. Medical therapy with dopamine agonists and somatostatin analogues has been successful in only a minority of patients.

  THYROID-STIMULATING HORMONE

Like the other glycoprotein hormones, TSH is a heterodimer composed of the common α-subunit and the unique TSH β-subunit. Normal levels of TSH range from 0.4 to 4.0 µU/mL. The detection limit for current TSH assays is less than 0.01 µU/mL, allowing measurement of suppressed TSH levels in patients with hyperthyroidism. TSH controls thyroid hormone (T4 and T3) synthesis and secretion from the thyroid gland. Hypothalamic TRH stimulates TSH synthesis and secretion. Somatostatin and dopamine can inhibit TSH secretion, but their roles in normal physiology have not been clearly elucidated. Thyroid hormones have an inhibitory effect on the production of TRH and TSH and constitute a powerful negative feedback loop acting at both the hypothalamic and pituitary levels. Secretion of TSH is pulsatile, but the amplitude of the pulses is relatively small and does not create the difficulties in measurement of TSH that are encountered with measurements of other pituitary hormones. Because of the integrated nature of the hypothalamic-pituitary-thyroid axis, thyroid function tests are best interpreted when concentrations of TSH, free T4, and free T3 levels are known. Except in conditions of secondary hypothyroidism or TSH-secreting pituitary tumors (see later), TSH levels provide an excellent screening test for thyroid dysfunction.

Central Hypothyroidism Central forms of hypothyroidism are due to loss of either TSH or TRH.22 Three different types of congenital TSH deficiency are caused by genetic mutations. One type involves mutations in the TSHβ gene. A second involves mutations in PIT1, which causes combined deficiencies of GH, PRL, and TSH (see earlier). A third involves a mutation in the gene for TRH. Acquired central forms of hypothyroidism are usually associated with other pituitary hormone deficiencies, and usually there is no goiter because of low TSH levels. Tests for TSH deficiency are best performed by analyzing free T4 levels in combination with TSH. Low free T4 without elevated TSH is consistent with central hypothyroidism. In some patients with hypothalamic disease, the TSH level is partially elevated in the presence of low levels of free T4, but the bioactivity of the TSH is reduced. Central forms of hypothyroidism must be distinguished from the sick-euthyroid condition (Chapter 226). Laboratory tests in the sick-euthyroid syndrome progress through several phases but can include prolonged periods when both TSH and free thyroid hormone levels are low. It can be very difficult in these patients to exclude central hypothyroidism unequivocally. In addition to the clinical setting in which thyroid function tests are measured, the presence of normal thyroid function tests before the illness and the absence of known hypothalamic or pituitary disease make true central hypothyroidism unlikely. Increased levels of reverse T3 are suggestive of sick-euthyroidism, and free T4 and T3 may be in the normal or low-normal range in sick-euthyroid patients. When TSH deficiency is documented, thyroid hormone is replaced using daily doses of l-thyroxine (0.05 to 0.15 mg/day). Because TSH cannot be used as an end point, one monitors the patient clinically and with serum levels of free T4 and T3.

Thyroid-Stimulating Hormone–Secreting Tumors TSH-secreting tumors are rare and account for between 1 and 3% of pituitary tumors. A recent analysis from Sweden showed that the prevalence was only

1494

CHAPTER 225  Posterior Pituitary  

23

2.8 per 1 million. As many as 45% of TSH-producing tumors are plurihormonal. GH and PRL are co-secreted most often, perhaps reflecting the common cellular lineage for thyrotrophs, somatotrophs, and lactotrophs. Long-standing severe hypothyroidism can cause thyrotroph hyperplasia and pituitary enlargement. These hyperplastic masses regress with thyroid hormone replacement therapy, however. Most true TSH-producing tumors are relatively autonomous and respond weakly, if at all, to TRH stimulation or thyroid hormone suppression. TSH-secreting tumors are usually macroadenomas by the time a diagnosis has been made. Consequently, many patients exhibit mass effects of the tumor, as well as hyperthyroidism. The clinical features of TSH-secreting tumors resemble those of Graves’ disease, except that features of autoimmunity (e.g., ophthalmopathy) are absent. Circulating levels of T4 and T3 range widely but can be elevated as much as two- to three-fold. Diffuse goiter is present in most patients with TSH-producing tumors, and the 24-hour uptake of radioiodine is elevated. Because feedback inhibition of TSH is impaired in TSH-producing tumors, TSH levels are inappropriately elevated in the presence of high levels of T4 and T3. TSH levels produced by tumors range from the low-normal range to as high as 500 µU/mL, but most levels are minimally elevated. Most TSHproducing tumors (>80%) secrete excess free α-subunit, and its assessment can be very useful in confirming the diagnosis. Thus, the diagnosis can usually be made by demonstrating that a hyperthyroid patient has a detectable serum TSH level associated with excess secretion of the free α-subunit. The finding of a mass lesion on CT or MRI confirms the diagnosis. Several other causes of inappropriate TSH secretion should be considered, including resistance to thyroid hormone and familial dysalbuminemic hyperthyroxinemia and other disorders that alter serum thyroid hormone binding proteins.

TREATMENT  The goals of therapy are to treat the underlying TSH-secreting tumor and to correct the hyperthyroidism. Transsphenoidal surgery alone is rarely curative because of the large size of most tumors, but it can alleviate mass effects and lower TSH levels. As in other large pituitary tumors, adjunctive irradiation may be required to control tumor growth. Somatostatin analogues have been used as adjunctive medical therapy, and they decrease TSH and α-subunit levels in about 80% of patients with TSH-secreting tumors, but consistent effects on tumor growth have not been demonstrated. Hyperthyroidism caused by TSH-secreting tumors can also be treated using antithyroid drugs or radioiodine.

  CLINICALLY NONFUNCTIONING PITUITARY TUMORS

Most clinically nonfunctioning adenomas can be shown to produce low levels of the free α-subunit, free β-subunits of FSH and LH, and intact FSH and LH when analyzed by immunocytochemistry or messenger RNA expression. A smaller fraction can be shown to produce low levels of other pituitary hormones, particularly ACTH or GH, that escaped detection based on routine endocrine testing.24 Even with detailed analyses of hormone production, a subset (10 to 20%) of nonfunctioning adenomas does not appear to produce any of the known pituitary hormones. The clinical features and management of nonfunctioning tumors are similar to those for gonadotropin-producing tumors. The major signs and symptoms result from tumor mass effects that cause visual field defects, headache and other neurologic symptoms, and hypopituitarism. Transsphenoidal surgery is the primary mode of treatment, with a goal of debulking the tumor to relieve mass effects. Because there are no serum tumor markers, patients must be followed by CT or MRI in conjunction with visual field tests. Some pituitary tumors are discovered as incidental findings on CT or MRI scans that were done for other reasons.25 Such tumors should be screened for hormone oversecretion with measurement of PRL, IGF-I, and a midnight salivary cortisol or an overnight dexamethasone suppression test, but most will be found to be nonfunctioning. If the tumor abuts the optic chiasm, a formal visual field examination should be performed. Over several years, about 10% of incidental microadenomas and 20% of macroadenomas enlarge. Indications for surgery include compression of the optic chiasm, with or without visual field defects and significant tumor enlargement. Hypopituitarism is also a relative indication for surgery (Fig. e224-1). In the absence of these indications for surgery, it is reasonable to follow such patients with MRI scans to look for size change at yearly intervals initially and then at less frequent intervals.

Grade A References A1. Barake M, Klibanski A, Tritos NA. Effects of recombinant human growth hormone therapy on bone mineral density in adults with growth hormone deficiency: a meta-analysis. J Clin Endocrinol Metab. 2014;99:852-860. A2. Gadelha MR, Bronstein MD, Brue T, et al. Pasireotide versus continued treatment with octreotide or lanreotide in patients with inadequately controlled acromegaly (PAOLA): a randomised, phase 3 trial. Lancet Diabetes Endocrinol. 2014;2:875-884. A3. Colao A, Petersenn S, Newell-Price J, et al. A 12-month phase 3 study of pasireotide in Cushing’s disease. N Engl J Med. 2012;366:914-924.

GENERAL REFERENCES For the General References and other additional features, please visit Expert Consult at https://expertconsult.inkling.com.

225  POSTERIOR PITUITARY JOSEPH G. VERBALIS

  ANATOMY AND HORMONE SYNTHESIS

The hormones of the posterior pituitary, vasopressin and oxytocin, are synthesized in specialized neurons in the hypothalamus, the neurohypophysial neurons. These neurons, notable for their large size, are termed magnocellular neurons. In the hypothalamus, the magnocellular neurons are clustered in the paired paraventricular and supraoptic nuclei (Fig. 225-1). Vasopressin and oxytocin are also synthesized in parvicellular (i.e., small cell) neurons of the paraventricular nuclei, and vasopressin (but not oxytocin) is also synthesized in the suprachiasmatic nucleus. Transcription of vasopressin and oxytocin messenger RNA and translation of the vasopressin and oxytocin prohormones occur entirely in the cell bodies of the neurohypophysial neurons. The prohormones provasopressin and prooxytocin are packaged along with processing enzymes into neurosecretory granules that are transported out of the perikaryon of the neurohypophysial neurons via microtubules and down the long axons that form the supraopticohypophysial tract, which terminates in the posterior pituitary. During transport, the processing enzymes cleave provasopressin into vasopressin (9 amino acids), vasopressin-neurophysin (95 amino acids), and vasopressin glycopeptide, or copeptin (39 amino acids). Pro-oxytocin is similarly cleaved to oxytocin (which differs from vasopressin by only two of nine amino acids) and oxytocin-neurophysin. The neurophysins form neurophysin-hormone complexes that stabilize the hormones. Stimulatory (e.g., glutamatergic, cholinergic, and angiotensin) neurotransmitter terminals and inhibitory (e.g., γ-aminobutyric acid and noradrenergic) neurotransmitter terminals control the release of vasopressin through the activity of synaptic contacts on the neurohypophyseal cell bodies. Physiologic release of vasopressin or oxytocin into the general circulation occurs at the level of the posterior pituitary, where, in response to an action potential, intracellular calcium is increased and causes the neurosecretory granules to fuse with the axon membrane, thereby releasing each hormone into the general circulation. Although each of the other prohormone fragments are released into the circulation, vasopressin and oxytocin are the only biologically active components of the prohormones. Factors that stimulate the release of neurohypophysial hormones also stimulate their synthesis. Because synthesis is delayed, maintenance of a large store of hormone in the posterior pituitary is essential to enable the instantaneous release of each hormone that is necessary following acute hemorrhage (vasopressin) or during parturition (oxytocin). In most species, sufficient vasopressin is stored in the posterior pituitary to support maximal antidiuresis for several days and to maintain baseline levels of antidiuresis for weeks.

Vasopressin Vasopressin and Regulation of Osmolality

The primary physiologic action of vasopressin is its function as a waterretaining hormone. The central sensing system (osmostat) for controlling the release of vasopressin is anatomically discrete, located in a small area of the hypothalamus just anterior to the third ventricle (see Fig. 225-1). The

CHAPTER 224  Anterior Pituitary  

1494.e1

Evaluation of pituitary function

Hyperfunctioning

Clinically nonfunctioning

Prolactinoma

Other

Dopamine agonist

Surgery

1 cm

Visual fields R/O pituitary hypofunction

Repeat MRI at 1, 2, 5 yrs

Repeat MRI at 0.5, 1, 2, 5 yrs

No change

Tumor growth Abnormal fields

No further studies (?)

Surgery

E-FIGURE 224-1.  Flow diagram indicating the approach to the patient found to have a pituitary incidentaloma. The first step is to evaluate patients for pituitary hyperfunction and then treat those found to be hyperfunctioning. Of patients with tumors that are clinically nonfunctioning, those with macroadenomas are evaluated further for evidence of chiasmal compression and hypopituitarism. Scans are then repeated at progressively longer intervals to assess for enlargement of the tumors. (Reproduced from Molitch ME. Nonsecreting tumors and pituitary incidentalomas. Endocrinol Metab Clin North Am. 2008;37:151-171.)

1494.e2

CHAPTER 224  Anterior Pituitary  

GENERAL REFERENCES 1. Cohen LE. Genetic disorders of the pituitary. Curr Opin Endocrinol Diabetes Obes. 2012;19:33-39. 2. Mete O, Asa SL. Clinicopathological correlations in pituitary adenomas. Brain Pathol. 2012;22: 443-453. 3. Hess CP, Dillon WP. Imaging the pituitary and parasellar region. Neurosurg Clin North Am. 2012;23:529-542. 4. Caturegli P, Iwama S. From Japan with love: another tessera in the hypophysitis mosaic. J Clin Endocrinol Metab. 2013;98:1865-1868. 5. Burman P, Mattsson AF, Johannsson G, et al. Deaths among adult patients with hypopituitarism: hypocortisolism during acute stress, and de novo malignant brain tumors contribute to an increased mortality. J Clin Endocrinol Metab. 2013;98:1466-1473. 6. Molitch ME, Clemmons DR, Malozowski S, et al. Evaluation and treatment of adult growth hormone deficiency: an Endocrine Society clinical practice guideline. J Clin Endocrinol Metab. 2011;96:1587-1609. 7. Melmed S. Pathogenesis of pituitary tumors. Nat Rev Endocrinol. 2011;7:257-266. 8. Vasilev V, Daly AF, Petrossians P, et al. Familial pituitary tumor syndromes. Endocr Pract. 2011;17(suppl 3):41-46. 9. Beckers A. Means, motives and opportunity: SDH mutations are suspects in pituitary tumors. J Clin Endocrinol Metab. 2013;98:2274-2276. 10. Fraser CL, Biousse V, Newman NJ. Visual outcomes after treatment of pituitary adenomas. Neurosurg Clin North Am. 2012;23:607-620. 11. Melmed S, Casaneuva FF, Hoffman AR, et al. Diagnosis and treatment of hyperprolactinemia. An Endocrine Society clinical practice guideline. J Clin Endocrinol Metab. 2011;96:273-288. 12. Devin JK. Hypopituitarism and central diabetes insipidus. Perioperative diagnosis and management. Neurosurg Clin North Am. 2012;23:679-689.

13. Swearingen B. Update on pituitary surgery. J Clin Endocrinol Metab. 2012;97:1071-1081. 14. Sheehan JP, Xu Z, Lobo MJ. External beam radiation therapy and stereotactic radiosurgery for pituitary adenomas. Neurosurg Clin North Am. 2012;23:571-586. 15. Katznelson L, Laws E, Molitch M, et al. Diagnosis and treatment of acromegaly. An Endocrine Society clinical practice guideline. J Clin Endocrinol Metab. 2014;99:3933-3951. 16. Rogers A, Karavitaki N, Wass JA. Diagnosis and management of prolactinomas and non-functioning pituitary adenomas. BMJ. 2014;349:g5390. 17. Dekkers OM, Horváth-Puhó E, Jørgensen JOL, et al. Multisystem morbidity and mortality in Cushing’s syndrome: a cohort study. J Clin Endocrinol Metab. 2013;98:2277-2284. 18. Bertagna X, Guignat L. Approach to the Cushing’s disease patient with persistent/recurrent hypercortisolism after pituitary surgery. J Clin Endocrinol Metab. 2013;98:1307-1318. 19. Feelders RA, Hofland LJ. Medical treatment of Cushing’s disease. J Clin Endocrinol Metab. 2013;98:425-438. 20. Barber TM, Adams E, Wass JA. Nelson syndrome: definition and management. Handb Clin Neurol. 2014;124:327-337. 21. Silveira LFG, Latronico AC. Approach to the patient with hypogonadotropic hypogonadism. J Clin Endocrinol Metab. 2013;98:1781-1788. 22. Persani L. Clinical Review: central hypothyroidism: pathogenic, diagnostic, and therapeutic challenges. J Clin Endocrinol Metab. 2012;97:3068-3078. 23. Önnestam L, Berinder K, Burman P, et al. National incidence and prevalence of TSH-secreting pituitary adenomas in Sweden. J Clin Endocrinol Metab. 2013;98:626-635. 24. Cooper O, Melmed S. Subclinical hyperfunctioning pituitary adenomas: the silent tumors. Best Pract Clin Endocrinol Metab. 2012;26:447-460. 25. Molitch ME. Management of incidentally found nonfunctional pituitary tumors. Neurosurg Clin North Am. 2012;23:53-554.

CHAPTER 224  Anterior Pituitary  

REVIEW QUESTIONS 1. A 25-year-old woman has had amenorrhea and galactorrhea for 2 years and is found to have hyperprolactinemia. Her PRL level is 513 ng/mL (normal 2.5 to 23.6 ng/mL) and magnetic resonance imaging showed a 1.3-cm macroadenoma. The rest of her evaluation is normal, with the exception of a serum calcium of 11.4 mg/dL. She recalls that one of her cousins also has a pituitary tumor. Which of the following genes should be analyzed for a mutation? A. PROP1 B. Pit1 C. Menin D. Ret proto-oncogene E. Ras Answer: C  Menin is encoded by the gene mutated in multiple endocrine neoplasia (MEN1), characterized by multiple endocrine tumors of the parathyroid glands, pancreatic islets, and anterior pituitary, especially prolactinomas. For incorrect answers, see introductory section on “Anatomy and Embryology.” 2. Which of the following medications used to treat pituitary tumor syndromes acts by blocking a hormone receptor? A. Pasireotide B. Cabergoline C. Ketoconazole D. Mifepristone E. Bromocriptine Answer: D  See Figure 224-8 and its legend.

1494.e3

3. A 67-year-old man is found to have enlarging hands and feet and has been referred by his dentist because of prognathism. To determine whether he has acromegaly, which of the following tests should be carried out? A. Pituitary magnetic resonance imaging B. Overnight 1-mg dexamethasone suppression test C. Insulin-induced hypoglycemia stimulation test D. Measurement of insulin-like growth factor (IGF)-I E. Inferior petrosal sinus sampling with GHRH stimulation Answer: D  See section on “Diagnosis” under “Growth Hormone Excess: Acromegaly and Gigantism.” 4. Magnetic resonance imaging (MRI) on a 73-year-old woman experiencing dizziness reveals an incidental 3-mm lesion in her pituitary that is compatible with a pituitary adenoma. She has otherwise been well. Testing for hormone oversecretion is negative. What should be the next step in management? A. Refer her to an experienced neurosurgeon. B. Repeat the MRI in 1 year to look for size change. C. Refer her to ophthalmology for a visual field examination. D. Begin bromocriptine treatment. E. Refer her for a preoperative cardiac stress test. Answer: B  See section on “Clinically Nonfunctioning Pituitary Tumors” at the end of the chapter and also e-Figure 224-1.

1495

CHAPTER 225  Posterior Pituitary  

Osmoreceptor

PVN

Thirst Osm

Plasma Plasma osmolality AVP (mOsm/kg) (pg/mL) 304

SON

302 Ant. Pit.

300

8

298

7

296

6

294 Thirst

FIGURE 225-1.  Sagittal view of the head, demonstrating the position of the neuro-

hypophysis. The magnocellular neurons are clustered in two paraventricular nuclei (PVN) and two supraoptic nuclei (SON). Only one nucleus of each pair is illustrated. The supraoptic nuclei are lateral to the edge of the optic chiasm, whereas the paraventricular nuclei are central along the wall of the third ventricle. The axons of the four nuclei combine to form the supraopticohypophysial tract as they course through the pituitary stalk to their storage terminals in the posterior pituitary. The osmostat (Osm) is in the hypothalamus anterior to the third ventricle; the thirst center (Thirst) is distributed across different brain areas. Ant. Pit. = anterior pituitary. (From Buonocore CM, Robinson AG. Diagnosis and management of diabetes insipidus during medical emergencies. Endocrinol Metab Clin North Am. 1993;22:411-423.)

osmostat controls the release of vasopressin to cause water retention and also stimulates thirst to cause water repletion. Osmotic regulation of vasopressin release and osmotic regulation of thirst are usually tightly coupled, but they can be dissociated under pathologic conditions. The primary extracellular osmolyte to which the osmoreceptor responds is sodium. Under normal physiologic conditions, glucose and urea cross neuron cell membranes and do not stimulate the release of vasopressin. Although basal osmolality in normal subjects ranges between 280 and 295 mOsm/kg H2O, extracellular fluid osmolality for each individual is maintained within narrow ranges. Increases in plasma osmolality as small as 1 to 2% are sufficient to stimulate vasopressin release. Basal plasma levels of vasopressin are generally 0.5 to 2 pg/mL, which maintains urine osmolality above plasma osmolality and urine volume in the range of 2 to 3 L/day. When vasopressin levels are suppressed below 0.5 pg/mL, maximal urine osmolality decreases to below 100 mOsm/kg H2O, and a free water diuresis (or “aquaresis”) ensues at levels that approach 800 to 1000 mL/hour (18 to 24 L/day). Increases in plasma osmolality cause a linear increase in plasma vasopressin and a corresponding linear increase in urine osmolality. At a plasma osmolality of approximately 295 mOsm/kg H2O, urine osmolality is maximally concentrated to 1000 to 1200 mOsm/kg H2O. Thus, the entire physiologic range of urine osmolality is accomplished by relatively small changes in plasma vasopressin levels of 0 to 5 pg/mL (Fig. 225-2). To maintain fluid balance, water must be not only conserved but also consumed to replace insensible water losses and obligate urine output. Thirst is not stimulated until a somewhat higher plasma osmolality (5 to 10 mOsm/ kg H2O) than the threshold for release of vasopressin. Most humans derive sufficient water from habitual fluid intake and catabolism of food to maintain plasma osmolality below the threshold that activates thirst. Therefore, under normal physiologic conditions, water balance (and hence plasma osmolality) is regulated more by secretion of vasopressin than by thirst. However, with severe degrees of dehydration, thirst is essential to restore body water deficits. Vasopressin acts on the V2 subtype of vasopressin receptors in the collecting duct principal cells of the kidney to cause water retention, or antidiuresis. Vasopressin V2 receptors are G protein–coupled receptors that activate adenylate cyclase, with subsequent increased intracellular cyclic adenosine monophosphate (cAMP) levels upon ligand activation of the receptor. The increased cAMP initiates the movement of aquaporin-2 (AQP2) water channels to the apical (luminal) membrane of the collecting duct cells. These channels allow facilitated rapid transport of water from the collecting duct lumen into the principal cell along osmotic gradients. The water then exits the cell through the basolateral membrane into the kidney medullary circulation through constitutively expressed aquaporin-3 and aquaporin-4 water channels.1 This entire process is termed antidiuresis. In the absence of vasopressin, the AQP2 channels are reinternalized from the apical membrane into subapical vesicles. This prevents active reabsorption of water from

Osmotic threshold

9

292

5

290

4

288

3

286

2

284 282

1

Urine osmolality (mOsm/kg) 1000 800 600 400 200

280

2 4 6 8 10 12 14 16 18 20

278

Urine volume (L/day)

276 274 272 270 FIGURE 225-2.  Idealized schematic of the normal physiologic relationships among plasma osmolality, plasma vasopressin (AVP), urine osmolality, and urine volume. The entire physiologic range of urine osmolality occurs with plasma vasopressin levels from 0 to 5 pg/mL. Increases in plasma osmolality above approximately 290 to 295 mOsm/kg H2O result in increases in plasma vasopressin but no further concentration of the urine, which is limited by the maximal osmolality in the inner medulla. The relation of volume (calculated on the basis of a constant osmolar load) is inversely exponential to the other parameters. Because of this relationship, urine volume does not change substantially until there is nearly absent vasopressin secretion, after which urine volume increases dramatically. (Calculated from formulas presented in Robertson GL, Shelton RL, Athar S. The osmoregulation of vasopressin. Kidney Int. 1976;10:25-37. Figure drawn by J.G. Verbalis, Georgetown University, Washington, DC.)

the collecting duct lumen, resulting in diuresis. In addition to this rapid “shuttling” of the AQP2 channels to regulate water reabsorption on a minute-to-minute basis, vasopressin also acts through the V2 receptors to regulate long-term stores of AQP2—that is, increased vasopressin stimulates AQP2 synthesis, and the absence of vasopressin results in decreased AQP2 synthesis. The hypertonic medullary interstitium is the determinant of the maximal concentration of the urine, which is in equilibrium with the osmolality of the inner medulla of the kidney under conditions of maximal antidiuresis (Chapter 115).

Vasopressin and Pressure and Volume Regulation

High-pressure baroreceptors are located in the aorta and carotid sinus, and low-pressure baroreceptors are located in the right and left atria. Decreases in blood pressure or intravascular volume stimulate vasopressin release, whereas situations that increase blood volume or left atrial pressure (e.g., negative-pressure breathing) decrease the secretion of vasopressin. The release of vasopressin in response to changes in volume or pressure is much less sensitive than the release in response to osmoreceptors; generally a 10 to 15% reduction in blood volume or pressure is needed to stimulate the release of vasopressin. However, once arterial pressure falls below this threshold, the stimulated response is exponential resulting in plasma levels of vasopressin that are markedly greater than those resulting from osmotic stimulation. The pressor effects of vasopressin are mediated through a separate vasopressin receptor subtype, the V1a receptors, located on vascular smooth muscle. The relatively insensitive regulation of vasopressin secretion by changes in volume and pressure and the modest role of vasopressin to regulate blood pressure are consistent with the notion that regulation of sodium homeostasis by the renin-angiotensin-aldosterone system (Chapter 227) is more important for controlling extracellular and blood volume than is the regulation of water homeostasis. However, the pressor effects of vasopressin to increase blood pressure can become prominent when other blood pressure

1496

CHAPTER 225  Posterior Pituitary  

regulatory systems are deficient (e.g., autonomic neuropathy or reninangiotensin-aldosterone system blockade) or in states of pathologic vasodilation (e.g., liver cirrhosis, septic shock).

Vasopressin and Adrenocorticotropic Hormone

Vasopressin stimulates adrenocorticotropic hormone (ACTH) secretion via stimulation of the vasopressin V1b receptor subtypes that are located on anterior pituitary corticotroph cells. Although the major regulator of ACTH secretion is corticotropin-releasing hormone (Chapter 224), vasopressin activates a different signal transduction system in the corticotrophs, so these hormones have synergistic effects on ACTH secretion.

Interaction of Osmotic and Volume Regulation

The vasopressin system has evolved to optimize mammalian water homeostasis. Water is consumed as available in the absence of stimulated thirst, and vasopressin secretion then regulates water excretion to maintain plasma osmolality. Thirst serves as a backup mechanism if dehydration becomes excessive. Because pressure-volume regulation of vasopressin is less sensitive, modest changes in pressure or volume, which are exacerbated by upright posture, do not interfere with the regulation of osmolality. Yet the pressor effect of high vasopressin levels serves to maintain blood pressure if volume depletion or hypotension becomes excessive. Usually, the physiologic regulation of osmolality and pressure-volume are synergistic. Dehydration causes an increase in plasma osmolality and a decrease in blood volume, both of which stimulate the release of vasopressin. Conversely, excess fluid administration causes a decrease in plasma osmolality and an expansion of blood volume, both of which inhibit vasopressin secretion. Other factors can also modulate osmotic release and action of vasopressin. With volume expansion, natriuretic factors such as atrial natriuretic peptide and brain natriuretic peptide are released from atrial myocytes and act at the kidney to induce a natriuresis. Brain natriuretic peptide is also synthesized in the hypothalamus, where it may act to decrease vasopressin secretion. During pregnancy, there is a decrease of plasma osmolality by approximately 10 mOsm/kg H2O as a result of a resetting of the osmostat for vasopressin secretion, and the osmostat for thirst is reset downward in parallel. These effects appear to be mediated by the placental hormone relaxin. Abnormalities in water and electrolyte balance are common in the elderly. This is due in part to age-related changes in body volume (as much as a 50% decrease in total body water occurs in those older than 75 years) and renal function. The elderly also have a decreased sense of thirst. Although there is a normal or even increased ability to secrete vasopressin with age, there is a decreased ability to achieve either maximal urine concentration to retain water or maximal dilution of urine to excrete water. Consequently, the elderly are particularly prone to both hypernatremia or hyponatremia with diseases that affect water balance or from the drugs used to treat various diseases.2

Oxytocin

Prolactin is the main hormone necessary for milk production, but oxytocin is essential for milk secretion. Suckling stimulates tactile receptors, producing an afferent signal to the hypothalamus that causes a synchronized release of oxytocin from the posterior pituitary. Oxytocin binds to oxytocin receptors in the breast and induces contraction of myoepithelial cells around the alveoli and ductules to eject milk. In addition, upregulation of uterine oxytocin receptors dramatically increases uterine smooth muscle contractions in response to oxytocin secretion at the end of pregnancy. The greatest release of oxytocin occurs with, not before, delivery of the infant, probably secondary to stretching of the vaginal wall. Because transgenic mice lacking either oxytocin or oxytocin receptors have normal parturition, oxytocin release may be more important to induce uterine contraction to inhibit blood loss after delivery than to initiate parturition. No pathologic syndromes of either increased or decreased secretion of oxytocin have yet been defined, but experimental studies have implicated oxytocin in maternal and affiliative behavior as well as bone formation.3 However, because of the structural similarity between vasopressin and oxytocin, at high plasma levels oxytocin can activate vasopressin receptors, and vasopressin can activate oxytocin receptors, both of which can have pathologic consequences.

  SYNDROME OF INAPPROPRIATE ANTIDIURETIC HORMONE SECRETION

Excess secretion of vasopressin can be caused by abnormally regulated secretion from the posterior pituitary, or by ectopic synthesis and secretion of vasopressin by tumors. Osmotically inappropriate secretion of vasopressin

causes renal water retention and volume expansion of body fluids, with consequent dilutional hyponatremia. This disorder is called the syndrome of inappropriate antidiuretic hormone secretion (SIADH) and is discussed in Chapter 116.

  DIABETES INSIPIDUS DEFINITION

Diabetes insipidus is the excretion of a large volume of hypotonic insipid (tasteless) urine, usually manifested by polyuria (increased urination) and polydipsia (increased thirst).4 The large urine volume, usually in excess of 50 to 60 mL/kg/day, must be distinguished from an increased frequency of small urine volumes and from large volumes of isotonic or hypertonic urine, both of which have different clinical significance.

PATHOBIOLOGY

Five pathophysiologic mechanisms must be considered in the differential diagnosis of diabetes insipidus. 1. Central diabetes insipidus is caused by the inability of the hypothalamus– posterior pituitary to secrete (and usually to synthesize) vasopressin in response to increased osmolality. No concentration of the dilute glomerular filtrate takes place in the renal collecting duct, and consequently, a large volume of hypotonic (i.e., dilute) urine is excreted. This produces a secondary increase in serum osmolality, with stimulation of thirst and secondary polydipsia. Levels of vasopressin in plasma are unmeasurable or inappropriately low for the plasma osmolality. 2. Nephrogenic diabetes insipidus is caused by the inability of an otherwise normal kidney to respond to vasopressin. As in hypothalamic (central) diabetes insipidus, the dilute glomerular filtrate entering the collecting duct is excreted as a large volume of hypotonic urine. The rise in plasma osmolality that occurs stimulates thirst and produces polydipsia. Unlike central diabetes insipidus, however, measured levels of vasopressin in plasma are high or appropriate for plasma osmolality. 3. Gestational diabetes insipidus is a rare condition produced by elevated levels or activity of placental cystine aminopeptidase (oxytocinase or vasopressinase) during pregnancy. The rapid destruction of vasopressin produces diabetes insipidus with polyuria and secondary stimulation of thirst with polydipsia. Because of the circulating vasopressinase, plasma vasopressin levels usually cannot be measured. 4. Primary polydipsia is a disorder of excess fluid ingestion rather than of vasopressin secretion or activity. Excessive ingested water produces a mild decrease in plasma osmolality that shuts off the secretion of vasopressin. In the absence of vasopressin action on the kidney, urine does not become concentrated, and a large volume of hypotonic urine is excreted. The amount of vasopressin in plasma is unmeasurable or low but is appropriate for the low plasma osmolality. 5. Osmoreceptor dysfunction is a variant of central diabetes insipidus in which the neurohypophysis is intact, but the osmoreceptive cells in the anterior hypothalamus have been damaged (see Fig. 225-1). Because the osmoreceptor cells are necessary for osmotically stimulated vasopressin secretion, the patient manifests polyuria. However, because the osmoreceptor cells also control thirst, these patients do not have polydipsia. As a result, they are characterized by elevated serum sodium levels and plasma osmolalities. For this reason, this disorder has also been called essential hypernatremia and adipsic hypernatremia, in recognition of the profound thirst deficits found in most of the affected patients. Although the pathophysiologic mechanisms for each of these five disorders are distinct, patients in the first four categories usually manifest polyuria and polydipsia, and the serum sodium level is usually normal because an intact thirst mechanism is sufficiently sensitive to maintain water homeostasis in the first three disorders, and the normal kidney has sufficient capacity to excrete the excess water load in the fourth. The fifth category of osmoreceptor dysfunction is the exception, owing to a defective thirst mechanism leading to hypernatremia.

CLINICAL MANIFESTATIONS

Central Diabetes Insipidus

The sudden appearance of hypotonic polyuria5 after transcranial surgery in the area of the hypothalamus or after head trauma with basal skull fracture and hypothalamic damage obviously suggests the diagnosis of central diabetes insipidus.6 In these situations, if the patient is unconscious and unable to recognize thirst, hypernatremia is a common accompaniment. However, even in patients with more insidious progression of a specific disease or in patients

CHAPTER 225  Posterior Pituitary  

with idiopathic central diabetes insipidus, the onset of polyuria is often relatively abrupt and occurs over several days or weeks. Most patients do not notice polyuria until urine volume exceeds 4 L/day, and as illustrated in Figure 225-2, urine volume does not exceed 4 L/day until the ability to concentrate the urine is severely limited and plasma vasopressin is nearly absent. As few as 10 to 15% of the normal number of vasopressinergic neurons in the hypothalamus is sufficient to maintain an asymptomatic urine volume, but the further loss of just a small number of these neurons produces a rapid increase in urine volume and symptomatic polyuria. Urine volume seldom exceeds the amount of dilute fluid delivered to the collecting duct (≈18 to 24 L in humans); in many cases, urine volume is significantly less because patients voluntarily restrict fluid intake, which causes some mild volume contraction and increased proximal tubular reabsorption of fluid. Patients often express a preference for cold liquids, which are more effective in assuaging thirst. Both thirst and increased urine output persist through the night, impairing sleep. Patients with partial central diabetes insipidus have some ability to secrete vasopressin, but this secretion is markedly attenuated at normal levels of plasma osmolality. Therefore, these patients often have symptoms and urine volume similar to those of patients with complete central diabetes insipidus. Because most patients with central diabetes insipidus have sufficient thirst to drink fluid to match urine output, few laboratory abnormalities are present at the time of initial evaluation. The serum sodium level can be in the high-normal range, whereas the blood urea nitrogen level can be low secondary to the large urine volume. Uric acid is relatively high because of the modest volume contraction and lack of action of vasopressin on V1a receptors in the kidney, which stimulate the clearance of uric acid. Uric acid levels greater than 5 mg/dL can distinguish diabetes insipidus from primary polydipsia.

Osmoreceptor Dysfunction

A variant of central diabetes insipidus is the syndrome of osmoreceptor dysfunction. Physiologic maneuvers demonstrate that when patients are euvolemic, an increase in plasma osmolality produces neither secretion of vasopressin nor a sensation of thirst. However, vasopressin is still synthesized by the hypothalamus and stored in the posterior pituitary, because stimulation of baroreceptors by hypovolemia or hypotension results in the prompt secretion of vasopressin; the kidney is responsive because vasopressin release by volume receptor stimulation causes urinary concentration. Because patients lack thirst, they are chronically dehydrated, often with markedly increased serum sodium levels. However, it is the dehydration-induced volume depletion, not the increased osmolality, that eventually stimulates the secretion of vasopressin. The volume of urine output depends on the degree of dehydration-induced secretion of vasopressin. If sufficient fluid replacement is given to return extracellular fluid volume to normal, these patients are unable to regulate vasopressin by osmolality and again become polyuric, thereby manifesting their underlying central diabetes insipidus. Lesions that cause osmoreceptor dysfunction are similar to lesions that can cause central diabetes insipidus, but in contrast to central diabetes insipidus these lesions usually occur more rostrally in the hypothalamus, consistent with the anterior hypothalamic location of the primary osmoreceptor cells (see Fig. 15-2). One lesion that is unique to this disorder is an anterior communicating cerebral artery aneurysm, particularly following resection of the aneurysm. Central diabetes insipidus can be inherited as an autosomal dominant disease that is typically characterized by an asymptomatic infancy and an onset later in childhood. Most genetic defects are either in the signal peptide of the pre-prohormone or in the neurophysin portion of the prohormone.7 Mutations involving the vasopressin sequence itself are few. Most cases are believed to result from disruption of cleavage from the signal peptide or abnormal folding of the neurophysin, which slows trafficking of the mutant prohormone through the endoplasmic reticulum, leading to neuronal cell dysfunction and/or death. Because this is a cumulative process, this explains the later onset of central diabetes insipidus with these types of mutations. Myxedema and adrenal insufficiency both impair the ability to excrete free water by renal mechanisms. The simultaneous occurrence of either of these diseases with central diabetes insipidus (as can occur with a tumor of the hypothalamus or pituitary) can decrease an otherwise large urine output, thereby masking the symptoms of diabetes insipidus. Replacement treatment for the anterior pituitary deficiency, especially glucocorticoids, can then cause a sudden and massive excretion of dilute urine. Similarly, the onset of either hypothyroidism or adrenal insufficiency during the course of diabetes insipidus can decrease the need for vasopressin replacement and in some cases can even cause hyponatremia. Central diabetes insipidus occurs commonly in patients with severe brain ischemia, and is often indicative of

1497

brain death. Treatment of the diabetes insipidus along with any coexistent anterior pituitary hormone deficiencies can be used to preserve donor organs in such cases.

Nephrogenic Diabetes Insipidus

Nephrogenic diabetes insipidus is caused by mutations of the vasopressin V2 receptor or the vasopressin-induced water channel AQP2, or by impairments in the signal transduction system linking V2 receptor activation and AQP2 membrane insertion. Familial nephrogenic diabetes insipidus is a rare disease, most cases of which (>90%) are due to mutations of the V2 receptor. More than 100 different V2 receptor mutations have been described and can be classified into several different general categories based on differences in transport of the mutant receptor to the cell surface and vasopressin binding or stimulation of adenylate cyclase. Because the gene for the V2 receptor is located on the X chromosome, this is an X-linked recessive disease. Symptoms are noted only in affected males, who often present with vomiting, constipation, failure to thrive, fever, and polyuria during the first week of life. Hypernatremia with a hypotonic urine is typically present. The phenotype is similar in the less than 10% of patients with mutations of the AQP2 water channel, but because the AQP2 gene is located on chromosome 12, mutations cause autosomal recessive disease; consequently, consanguinity and a family history of the disease in men and women is common, and this disorder should be suspected when the proband is female.8 Nephrogenic diabetes insipidus can also be acquired during treatment with certain drugs such as demeclocycline (which can be used to treat inappropriate secretion of vasopressin), lithium carbonate (used to treat bipolar disorders), and fluoride (previously used in fluorocarbon anesthetics), and from electrolyte abnormalities such as severe hypokalemia and hypercalcemia. All causes of acquired nephrogenic diabetes insipidus have in common the decreased synthesis and function of AQP2 due to impaired vasopressin signaling from V2 receptor binding and activation. Other diseases of the kidney can produce polyuria and inability to concentrate the urine secondary to altered renal medullary blood flow or to other disorders that inhibit maintenance of the hyperosmolar concentrating gradient in the inner medulla. Renal manifestations of such disorders (e.g., sickle cell disease, sarcoidosis, pyelonephritis, multiple myeloma, analgesic nephropathy) are discussed in Chapter 121.

Gestational Diabetes Insipidus

In pregnancy, there is an increased metabolism of vasopressin due to cystine aminopeptidase (oxytocinase or vasopressinase), an enzyme that degrades oxytocin and prevents premature uterine contractions. Normally, this is compensated for by increased synthesis and secretion of vasopressin. Rarely, women with normal regulation of vasopressin develop diabetes insipidus because of markedly elevated levels of vasopressinase. Some of these patients have accompanying preeclampsia, acute fatty liver, and coagulopathies, but causal relations between diabetes insipidus and these abnormalities have not been identified. In general, diabetes insipidus does not persist after the pregnancy ends and does not recur in subsequent normal pregnancies.9 Polyuria can also manifest in patients who have limited vasopressin reserve (partial central diabetes insipidus) or who respond poorly to vasopressin action (compensated nephrogenic diabetes insipidus). Treatment may be required only during the pregnancy, and the patient often returns to her previous baseline function without the need for therapy when the pregnancy ends. Less commonly, central diabetes insipidus of another cause first becomes symptomatic during pregnancy and then persists afterward, following the usual course of diabetes insipidus.

Primary Polydipsia

Excessive fluid intake also causes hypotonic polyuria and, by definition, polydipsia. This disorder must be differentiated from the various causes of diabetes insipidus. Despite normal pituitary and kidney function, patients with this disorder share many characteristics of both central diabetes insipidus (vasopressin secretion is suppressed as a result of decreased plasma osmolality) and nephrogenic diabetes insipidus (kidney AQP2 expression is decreased as a result of suppressed plasma vasopressin levels). Many different names have been used for this excessive fluid intake, including dipsogenic diabetes insipidus, but primary polydipsia remains the best descriptor to avoid confusing this order with diabetes insipidus as classically defined. Primary polydipsia is sometimes due to a severe mental illness such as schizophrenia, mania, or obsessive-compulsive disorder, in which case it is called psychogenic polydipsia. These patients usually deny true thirst and attribute their polydipsia to bizarre motives, such as a need to cleanse the

CHAPTER 225  Posterior Pituitary  

body of poisons. The incidence in psychiatric hospitals can be as high as 40%, and there is no obvious explanation for the polydipsia. Primary polydipsia can also be caused by an abnormality in the osmoregulatory control of thirst, in which case it is called dipsogenic diabetes insipidus. These patients have no overt psychiatric illness and invariably attribute their polydipsia to a nearly constant thirst. Dipsogenic diabetes insipidus is usually idiopathic, but it can also be secondary to organic structural lesions in the hypothalamus identical to those causing central diabetes insipidus, such as neurosarcoidosis of the hypothalamus, tuberculous meningitis, multiple sclerosis, or trauma. Consequently, all polydipsic patients should be evaluated with magnetic resonance imaging (MRI) of the brain before it is concluded that excessive water intake is due to an idiopathic or psychiatric cause. Primary polydipsia can also be produced by diseases or drugs that cause a dry mouth, or by any peripheral disorder causing marked elevations of renin or angiotensin. Finally, primary polydipsia is sometimes caused by physicians, nurses, lay practitioners, or health writers who recommend a high fluid intake for valid (e.g., recurrent nephrolithiasis) or unsubstantiated health reasons. These patients lack overt signs of mental illness, but they also deny thirst and usually attribute their polydipsia to habits acquired from years of adherence to a drinking regimen. Laboratory studies in these patients are generally normal, although the serum sodium concentration is sometimes at the low end of the normal range, and the level of uric acid is lower than in patients with other forms of diabetes insipidus.

Normal Primary polydipsia Partial central DI

Complete central DI Nephrogenic DI

1000 Urine osmolality (mOsm)

1498

desmopressin

800 600 400 200 0 0

2

4 6 8 10 Hours of water deprivation

12

14

FIGURE 225-3.  Responses to the water deprivation test to differentiate various types of diabetes insipidus (DI) and primary polydipsia (as described by Miller M, Dalakos T, Moses AM, et al. Recognition of partial defects in antidiuretic hormone secretion. Ann Intern Med. 1970;73:721-729). The response to dehydration reaches a plateau, and the subsequent change in urine osmolality in response to administered desmopressin is illustrated. See the discussion in the text.

DIAGNOSIS

Physiologic Diagnosis

Diabetes insipidus should be considered in all patients presenting with significant polyuria, defined as urine output greater than 50 mL/kg/day. Although osmotic diuresis secondary to hyperglycemia, intravenous contrast agents, or renal injury is a more common clinical cause of polyuria, the medical history, an isotonic urine osmolality, and routine clinical laboratory tests generally distinguish these disorders from diabetes insipidus. A diagnosis of diabetes insipidus can be made when urine osmolality is inappropriately low in the presence of an elevated plasma osmolality as a result of increased serum sodium concentration. These criteria are sometimes met at the initial examination, especially in cases of acute diabetes insipidus occurring after trauma or surgery with inadequate fluid replacement. In such patients with hypernatremia and hypotonic urine osmolality with normal renal function, one need only administer a vasopressin agonist to differentiate central diabetes insipidus, in which a renal response with decreased urine volume and increased urine osmolality occurs, from nephrogenic diabetes insipidus, in which a subnormal renal response is seen. Sometimes in the postoperative state, a water diuresis occurs as a result of water retention during the surgical procedure. Vasopressin is normally secreted in response to surgical stress, causing fluid administered intravenously during the procedure to be retained. During recovery, vasopressin levels fall, and a diuresis of the retained fluid occurs. In this case, the serum sodium level is almost always normal; however, if additional fluid is administered to match the urine output, persistent polyuria can be mistaken for diabetes insipidus. In this situation, the physician should decrease the rate of fluid administered and follow the urine output and serum sodium level. If the urine output decreases and the serum sodium level remains normal, no treatment is necessary; if serum sodium rises above the normal range and the urine remains hypotonic, diabetes insipidus is likely, and the response to a vasopressin agonist can ascertain the type (central versus nephrogenic). Most outpatients with diabetes insipidus are not hypernatremic, because the polydipsia produced by a normal thirst response is generally sufficient to maintain water homeostasis. Instead, they present with polyuria, polydipsia, and a normal sodium level. In these patients, further testing is necessary to increase serum osmolality and then measure the plasma vasopressin level or the urinary response to an administered vasopressin agonist. The best described test is the water deprivation test (Fig. 225-3),10 which should be carried out under controlled observation in the hospital or an appropriately equipped outpatient area. The exact timing of the test depends on the patient’s symptoms. If the patient has marked polyuria during the night, it is best to begin the test during the day because the patient may become overly dehydrated overnight. However, if the patient has only two or three episodes of nocturia per night, it is best to begin the test in the evening so that the major part of the dehydration takes place when the patient is asleep. In either case, the patient is weighed at the beginning of the test, and all subsequent fluids are withheld. The volume and osmolality of all excreted urine are measured, and the patient is reweighed after each liter of urine output. When three

consecutive urine samples have an osmolality differing by no more than 10% and the patient has lost at least 2% of body weight, a blood sample is obtained for the measurement of serum osmolality, sodium, and plasma vasopressin. The patient is then given 2 µg of desmopressin intravenously or intramuscularly and observed for an additional 2 hours. Adults with normal vasopressin secretion concentrate their urine to greater than 800 mOsm/kg H2O and have less than a 10% increase in urine osmolality in response to administered desmopressin. Patients with complete central diabetes insipidus have minimal concentration of the urine with dehydration, and a marked increase in urine osmolality (usually >50%) in response to administered desmopressin. Patients with nephrogenic diabetes insipidus usually have no increase in urine concentration in response to administered desmopressin, although in some cases of acquired nephrogenic diabetes insipidus, some increased urinary concentration (but generally < 10%) can occur. Nephrogenic diabetes insipidus is best distinguished from central diabetes insipidus by the measurement of vasopressin in plasma; plasma vasopressin levels are elevated in cases of nephrogenic diabetes insipidus, especially after dehydration. In patients with partial central diabetes insipidus and patients with primary polydipsia, the urine is often somewhat concentrated in response to dehydration, but not to the maximum of a normal person. The chronically reduced level of vasopressin downregulates the synthesis of AQP2 water channels, and the large urine volume, regardless of cause, washes out the medullary osmotic gradient that is the determinant of maximal urine concentration. When desmopressin is administered, patients with partial central diabetes insipidus have a further increase (usually > 10% but < 50%) in urine osmolality, whereas most patients with primary polydipsia have no further increase (i.e., 145 mmol/L). Consequently, some investigators recommend a limited infusion of hypertonic (3%) sodium chloride solution to achieve these elevated levels if they are not achieved by the water deprivation itself. Measurement of the C-terminal fragment of the vasopressin prohormone copeptin may be a better surrogate measure of vasopressin secretion.11 In some difficult cases, the response to treatment with a vasopressin agonist can be a useful aid to diagnosis. If a decrease in polyuria and thirst with maintenance of normal serum sodium concentration occurs, a diagnosis

CHAPTER 225  Posterior Pituitary  

of partial central diabetes insipidus is likely; however, if polydipsia persists and hyponatremia develops, a diagnosis of primary polydipsia is confirmed.

Etiologic Diagnosis

If the water deprivation test confirms that inadequate vasopressin secretion is responsible for the polyuria, the underlying cause must be determined. MRI of the hypothalamic-pituitary area is the most important diagnostic tool in these cases. The three areas of interest are the immediate suprasellar region of the hypothalamus, the pituitary stalk, and the posterior pituitary within the sella turcica (see the earlier discussion of anatomy). Most slow-growing tumors confined to the sella do not cause diabetes insipidus. To cause central diabetes insipidus, tumors in the hypothalamic area immediately above the sella must be either sufficiently large to destroy 80 to 90% of the vasopressin cells or located where the paths of the four nuclear groups converge at the origin of the pituitary stalk, just above the diaphragma sellae. Primary tumors, especially craniopharyngioma and suprasellar germinoma, metastatic tumors, and infiltrative diseases can also cause diabetes insipidus by infiltration of the pituitary stalk, which is then thickened (i.e., >2 mm) on MRI. On T1-weighted MRI, the vasopressin and oxytocin stored in neurosecretory granules in the posterior pituitary are visualized as a bright spot in the sella turcica. Most but not all normal subjects have this bright spot (it is absent more frequently in elderly and dehydrated patients); in most but not all patients with central diabetes insipidus, the bright spot is absent. Thickening of the stalk and absence of the bright spot are therefore especially suggestive of a hypothalamic disease process.12 Tumors that cause central diabetes insipidus are most often benign primary intracranial tumors such as craniopharyngioma, ependymoma (suprasellar germinoma), and pinealoma, which arise in the third ventricle. Primary tumors of the anterior pituitary (Chapter 224) cause diabetes insipidus only when substantial suprasellar extension is present. However, rapidly growing intrasellar lesions, such as metastases from carcinomas of the lung, breast, and melanoma or hemorrhage into pituitary adenomas, can cause diabetes insipidus because there is insufficient time for the vasopressin axons to adapt by releasing vasopressin from the hypothalamus. Metastases to the hypothalamus can also destroy the supraopticohypophysial tract and produce diabetes insipidus. Granulomatous diseases, such as Langerhans cell histiocytosis, sarcoidosis, tuberculosis, and leukemic infiltrates and lymphomas of the hypothalamus, can cause diabetes insipidus by destroying vasopressin cells. In such patients, the diagnosis is usually suspected on the basis of peripheral manifestations of the respective diseases. Lymphocytic infundibuloneurohypophysitis is an autoimmune disease similar to lymphocytic hypophysitis of the anterior pituitary (Chapter 224) in which lymphocytes infiltrate the neurohypophysis to produce diabetes insipidus. The hallmarks of this process are a thickened pituitary stalk and an absence of the pituitary bright spot in a patient with the abrupt onset of polyuria and polydipsia, particularly a postpartum female. The diagnosis was originally demonstrated by pituitary biopsy, but now is more commonly made by regression of the thickened stalk with continued MRI follow-up. When no specific cause is identified, the diagnosis of exclusion is idiopathic diabetes insipidus; but most such cases are probably caused by an autoimmune disease, and other autoimmune diseases, including anterior pituitary hypophysitis,13 are often recognized in affected patients. When central nervous system disease is suspected but not diagnosed by MRI or general physical examination, cerebrospinal fluid obtained by lumbar puncture may be helpful in identifying tumor cells or markers of tumors or inflammatory processes (e.g., elevated angiotensin-converting enzyme levels with neurosarcoidosis, elevated β-HCG levels with germinomas). A family history suggestive of diabetes insipidus should be investigated with genetic testing for inherited mutations in the vasopressin or vasopressin receptor genes depending on the site of the defect.

TREATMENT  Because excess excretion of water is the primary manifestation of diabetes insipidus, water replacement in adequate quantities avoids the metabolic complications of all forms of this disease. However, oral or intravenous administration of the volume of fluid required to replace the often large urinary losses in diabetes insipidus is difficult and inconvenient. The goal of therapy is therefore to reduce the amount of polyuria and polydipsia to a tolerable level while avoiding overtreatment, which can produce water retention and hyponatremia.

1499

Central Diabetes Insipidus

The best therapeutic agent for the treatment of central diabetes insipidus is the vasopressin agonist desmopressin.14 Desmopressin is different from vasopressin in that the amino group of the N-terminal cystine residue has been removed to prolong the duration of action, and d-arginine has been substituted for l-arginine in position 8 to decrease the vasopressor effects. At therapeutic dosages, this agent acts primarily on V2 or antidiuretic receptors, with minimal activity at V1a or pressor receptors. Desmopressin is available as tablets of 0.1 or 0.2 mg for oral administration and in either a spray bottle that delivers a fixed dose of 10 µg in 100 µL or a bottle with a rhinal catheter that can deliver 50 to 200 µL (5 to 20 µg) for intranasal administration. When therapy is initiated, it is generally best to begin with a low dose (e.g., half of a 0.1-mg tablet, 5 µg by the rhinal tube, or a single 100 µL spray of 10 µg) at bedtime to allow the patient to sleep through the night, and then determine the duration of action by quantifying the polyuria the next day. The duration of action of a single dose varies from 6 to 24 hours, but in most patients, a good therapeutic response can be achieved on an every-12-hour schedule for the nasal spray or an 8- or 12-hour schedule for the tablets. Desmopressin is also available for parenteral use in 1-mL vials of 4 µg/mL. Parenteral administration is especially useful postoperatively or when a patient is unable to take the nasal preparation. In hospitalized patients, some physicians add vasopressin directly to a crystalloid solution to infuse doses in the range of 0.25 to 2.7 mIU/kg/hour to cause modest but persistent urinary concentration as a treatment of diabetes insipidus. With any form of desmopressin administration, serum sodium levels should be monitored regularly to prevent the development of hyponatremia.15

Osmoreceptor Dysfunction

Because the diabetes insipidus of patients with osmoreceptor dysfunction is central, they respond to desmopressin as do patients with central diabetes insipidus. However, because of their thirst defect, this is usually not sufficient to maintain normal plasma osmolality. Consequently, they must be given a “prescription” for amounts of fluids to be consumed each 24 hours in order to maintain normal serum sodium levels and plasma osmolalities. This must be individualized to each patient because overconsumption of fluid coupled with desmopressin administration can produce severe hyponatremia. Body weights using an accurate scale is useful as a guide to preventing under- or overhydration, but frequent monitoring of serum sodium levels is usually necessary as well.

Nephrogenic Diabetes Insipidus

Although most patients with nephrogenic diabetes insipidus do not respond to desmopressin, a small number have a partial response to higher doses (e.g., 10 to 20 µg subcutaneously or intranasally). For the majority of patients who have no response to desmopressin, some orally administered pharmacologic agents are also useful in treating nephrogenic diabetes insipidus. Thiazide diuretics cause sodium depletion and volume contraction and decrease urine volume by increasing proximal tubular reabsorption of glomerular filtrate. Prostaglandin synthase inhibitors (e.g., indomethacin) block the action of prostaglandin E to inhibit the action of vasopressin on the kidney. Chlorothiazide, amiloride, and prostaglandin synthase inhibitors are useful to reduce polyuria in nephrogenic diabetes insipidus. However, none of these agents has been approved by the U.S. Food and Drug Administration for the treatment of diabetes insipidus; therefore, the prescribing physician should be aware of potential toxicities and side effects. In cases of drug-induced nephrogenic diabetes insipidus, the most direct therapy is discontinuation of the offending agent, if possible. Symptomatic nephrogenic diabetes insipidus is usually treated with a thiazide diuretic, which is enhanced by coadministration of the potassium-sparing diuretic amiloride. Amiloride can be especially beneficial in cases of nephrogenic diabetes insipidus induced by lithium, because the drug decreases the entrance of lithium into cells in the distal tubule. When diuretics are used to treat nephrogenic diabetes insipidus, special attention should be paid to the possibility that the induced dehydration may increase the concentration of other drugs.

Gestational Diabetes Insipidus

During pregnancy, vasopressinase increases the metabolism of vasopressin but not of desmopressin, so desmopressin is the drug of choice for these patients. The vasopressinase activity subsides by a few weeks after delivery, and patients with the onset of partial diabetes insipidus during pregnancy may become asymptomatic after delivery. An additional advantage of des­ mopressin is that it has little action on the oxytocin receptors of the uterus. During pregnancy, normal plasma osmolality decreases by approximately 10 mOsm/kg H2O because of changes in serum sodium, so pregnant patients with diabetes insipidus require only enough desmopressin to maintain the serum sodium at this lower level.

Correction of Hyperosmolality

Some situations require special attention during therapy. Rarely, if patients with diabetes insipidus are unable to drink or are given a hypertonic solution, severe hypernatremia can develop acutely. Osmotic equilibrium with the

1500

CHAPTER 226  Thyroid  

intracellular water of neurons and glia produces shrinking of the brain. The brain is in a closed vault (i.e., the skull), and when the brain shrinks, traction on the vasculature of the central nervous system can cause the rupture of blood vessels and subarachnoid or intracerebral hemorrhage. If the hypernatremia persists for a longer time, the neurons accommodate by producing organic osmolytes (previously called idiogenic osmoles), which limit the amount of brain shrinkage. Once this adaptation has occurred, a too-rapid lowering of osmolality in the extracellular fluid will produce a shift of water into the brain and cause cerebral edema. In this situation, desmopressin can be administered to produce constant antidiuresis, and the amount of water given can be regulated to decrease osmolality by no more than approximately 12 mEq/L every 24 hours. Postoperatively or after head trauma, diabetes insipidus can be transient (see Prognosis), and the need for long-term maintenance therapy cannot be immediately established.

PROGNOSIS

The prognosis of properly treated diabetes insipidus is excellent. If nephrogenic diabetes insipidus is diagnosed and treated early, intracranial calcification and mental retardation do not occur. When the diabetes insipidus is secondary to a recognized disease process, that disease generally determines the ultimate prognosis. In some specific clinical situations, the course is different and characteristic. The development of diabetes insipidus after surgical or traumatic injury to the neurohypophysis can follow any of several welldefined patterns (Fig. 225-4). In some patients, polyuria develops 1 to 4 days after injury and resolves spontaneously. Less often, the diabetes insipidus is permanent and continues indefinitely. Most interestingly, one can see a “triphasic” response that has been well described after pituitary stalk transection. The first phase of diabetes insipidus is due to axon shock and lack of function of the damaged neurons. This phase lasts several hours to several days and is followed by a second, antidiuretic phase that is due to the uncontrolled release of vasopressin from the disconnected and degenerating posterior pituitary or from the remaining severed neurons. Overly aggressive administration of fluids during this second phase does not suppress the

UV

Transient DI

A

T

UV

Permanent DI

B

T Triphasic Response I

UV

III II

uncontrolled vasopressin release from the damaged neurohypophysis and can lead to hyponatremia. The antidiuresis can last 2 to 14 days, after which diabetes insipidus recurs after depletion of vasopressin from the degenerating posterior pituitary gland (third phase). Transient hyponatremia without preceding or subsequent diabetes insipidus has been reported after transsphenoidal surgery for pituitary microadenomas. Once a deficiency of vasopressin secretion has been present for more than a few weeks, it rarely improves, even if the underlying cause of the neurohypophysial destruction is eliminated. The major exception to this is postoperative diabetes insipidus, in which spontaneous resolution is the rule. Although recovery from diabetes insipidus that persists more than several weeks postoperatively is less common, and is uncommon after 1 year of continued diabetes insipidus, well-documented cases of recovery as long as 10 years after the initiating event have been reported. Potential return of function is a reason to occasionally withhold therapy during long-term treatment. Diabetes insipidus should not be considered idiopathic until at least 4 years of follow-up. During this interval, annual computed tomography or MRI is indicated to search for a tumor or infiltrative process that may not have been detected at the initial examination. GENERAL REFERENCES For the General References and other additional features, please visit Expert Consult at https://expertconsult.inkling.com.

226  THYROID MATTHEW KIM AND PAUL W. LADENSON The adult thyroid gland contains two lobes that wrap along the anterolateral aspects of the trachea, midway between the thyroid cartilage and the suprasternal notch. Each lobe is demarcated into upper, middle, and lower poles. The right and left lobes are connected by an isthmus on the anterior aspect of the trachea just below the cricoid cartilage. The normal terminus of the thyroglossal duct can persist as a pyramidal lobe, which is often palpably enlarged in diffuse thyroid disorders such as autoimmune thyroiditis and Graves disease. With thyroid enlargement, the attachment of the sternothyroid muscle to the trachea limits upward expansion of the lobes, but further lateral, posterior, and downward growth may lead the gland to extend into the superior mediastinum, compressing the trachea and veins at the thoracic outlet. The parathyroid glands usually lie behind the superior and inferior poles of the thyroid lobes. The recurrent laryngeal nerves course upward along the tracheoesophageal groove, from which branches pass behind each thyroid lobe to innervate the larynx. Thyroid tissue is composed of clustered spherical follicles, each containing a single layer of follicular epithelial cells known as thyrocytes that surround a lumen containing colloid. The principal component of colloid is thyroglobulin, a thyrocyte-specific protein. Parafollicular C cells, which are derived from neural crest tissue and produce calcitonin, are widely dispersed between follicles.

  PHYSIOLOGY

Thyroid Hormone Synthesis and Secretion C

T

FIGURE 225-4.  A to C, Diagrammatic summary of the major patterns of postoperative and post-traumatic diabetes insipidus (DI). The abscissa represents time (T) after the initial injury (arrow); the ordinate represents urinary volume (Uv) relative to a hypothetical “normal” urine output of 2 to 3 L/24 hours (solid line). See the discussion in the text. During the triphasic response (C), uncontrolled release of vasopressin from the disconnected or damaged posterior pituitary gland causes an antidiuresis that can lead to water retention and a dilutional hyponatremia. Diabetes insipidus returns as the third phase after the stored hormone in the posterior pituitary has been depleted. (From Verbalis JG, Robinson AG, Moses AM. Postoperative and post-traumatic diabetes insipidus. In: Czernichow AP, Robinson A, eds. Diabetes Insipidus in Man: Frontiers of Hormone Research. Basel: S Karger; 1985:247.)

Dietary iodine in the form of iodide (I−) or iodate (IO3−) is absorbed by the gastrointestinal tract and distributed in the extracellular fluid. Daily iodine intake in the United States equals or exceeds the recommended daily intake of 150 µg because of the widespread use of iodized salt and iodate preservatives in baked goods. Circulating iodide is actively transported into the thyrocyte by the sodium-iodide symporter. Within the thyrocyte, iodide is rapidly oxidized by H2O2 in a reaction catalyzed by thyroid peroxidase. The reactive intermediate formed is covalently bound to tyrosyl residues present in thyroglobulin to generate monoiodotyrosine and diiodotyrosine residues through a process known as organification. Thyroid peroxidase also catalyzes the coupling of the monoiodotyrosine and diiodotyrosine residues to generate thyroxine (T4) and triiodothyronine (T3) residues in thyroglobulin,

CHAPTER 225  Posterior Pituitary  

GENERAL REFERENCES 1. Kortenoeven ML, Fenton RA. Renal aquaporins and water balance disorders. Biochim Biophys Acta. 2014;1840:1533-1549. 2. Cowen LE, Hodak SP, Verbalis JG. Age-associated abnormalities of water homeostasis. Endocrinol Metab Clin North Am. 2013;42:349-370. 3. Colaianni G, Tamma R, Di BA, et al. The oxytocin-bone axis. J Neuroendocrinol. 2014;26:53-57. 4. Leroy C, Karrouz W, Douillard C, et al. Diabetes insipidus. Ann Endocrinol (Paris). 2013;74: 496-507. 5. Jakes AD, Bhandari S. Investigating polyuria. BMJ. 2013;347:f6772. 6. Schreckinger M, Szerlip N, Mittal S. Diabetes insipidus following resection of pituitary tumors. Clin Neurol Neurosurg. 2013;115:121-126. 7. Babey M, Kopp P, Robertson GL. Familial forms of diabetes insipidus: clinical and molecular characteristics. Nat Rev Endocrinol. 2011;7:701-714. 8. Bichet DG. Physiopathology of hereditary polyuric states: a molecular view of renal function. Swiss Med Wkly. 2012;142:w13613. 9. Aleksandrov N, Audibert F, Bedard MJ, et al. Gestational diabetes insipidus: a review of an underdiagnosed condition. J Obstet Gynaecol Can. 2010;32:225-231.

1500.e1

10. Fenske W, Allolio B. Current state and future perspectives in the diagnosis of diabetes insipidus: a clinical review. J Clin Endocrinol Metab. 2012;97:3426-3437. 11. Fenske W, Quinkler M, Lorenz D, et al. Copeptin in the differential diagnosis of the polydipsiapolyuria syndrome–revisiting the direct and indirect water deprivation tests. J Clin Endocrinol Metab. 2011;96:1506-1515. 12. Di IN, Napoli F, Allegri AE, et al. Diabetes insipidus–diagnosis and management. Horm Res Paediatr. 2012;77:69-84. 13. Bando H, Iguchi G, Fukuoka H, et al. The prevalence of IgG4-related hypophysitis in 170 consecutive patients with hypopituitarism and/or central diabetes insipidus and review of the literature. Eur J Endocrinol. 2014;170:161-172. 14. Oiso Y, Robertson GL, Norgaard JP, et al. Clinical review: treatment of neurohypophyseal diabetes insipidus. J Clin Endocrinol Metab. 2013;98:3958-3967. 15. Behan LA, Sherlock M, Moyles P, et al. Abnormal plasma sodium concentrations in patients treated with desmopressin for cranial diabetes insipidus: results of a long term retrospective study. Eur J Endocrinol. 2015;172:243-250.

1500.e2

CHAPTER 225  Posterior Pituitary  

REVIEW QUESTIONS 1. The hormone vasopressin (AVP) that regulates body water balance is synthesized where? A. The anterior pituitary gland B. The posterior pituitary gland C. The supraoptic and paraventricular nuclei of the hypothalamus D. The anterior hypothalamus near the osmoreceptor cells E. The principal collecting duct cells of the kidney Answer: C  The supraoptic and paraventricular nuclei of the hypothalamus. The hormones of the posterior pituitary, vasopressin and oxytocin, are synthesized in specialized neurons in the hypothalamus, the neurohypophysial neurons. These neurons, notable for their large size, are termed magnocellular neurons. In the hypothalamus, magnocellular neurons are clustered in the paired paraventricular and supraoptic nuclei (see Fig. 232-1). The synthesized vasopressin prohormone is transported down the axons of the magnocellular neurons to the posterior pituitary gland, where vasopressin is released in response to specific stimuli. 2. Vasopressin secretion is stimulated at what levels of plasma osmolality? A. Plasma osmolality above the thirst threshold of 295 mOsm/kg H2O B. Increases of plasma osmolality of 4 to 5% C. Increases of plasma osmolality of 2 to 3% D. Increases of plasma osmolality of 1 to 2% E. Decreases in plasma osmolality of 1 to 2% Answer: D  Increases of plasma osmolality of 1 to 2%. Increases in plasma osmolality as small as 1 to 2% stimulate vasopressin release. Basal plasma levels of vasopressin are generally 0.5 to 2 pg/mL, which is sufficient to maintain urine osmolality above plasma osmolality and urine volume in the range of 2 to 3 L/day. When vasopressin levels are suppressed below 0.5 pg/mL, maximal urine osmolality decreases to less than 100 mOsm/kg H2O, and a free water diuresis (or “aquaresis”) ensues at levels approaching 800 to 1000 mL/hour (18 to 24 L/day). Increases in plasma osmolality cause a linear increase in plasma vasopressin and a corresponding linear increase in urine osmolality. At a plasma osmolality of approximately 295 mOsm/kg H2O, urine osmolality is maximally concentrated to 1000 to 1200 mOsm/kg H2O. Thus, the entire physiologic range of urine osmolality is accomplished by relatively small changes in plasma vasopressin of 0 to 5 pg/mL (see Fig. 232-2). 3. Patients with diabetes insipidus and inability to concentrate their urine usually present with what manifestations? A. Polyuria, polydipsia, and hyperosmolality B. Polyuria, polydipsia, and dehydration C. Polyuria, polydipsia, and hypernatremia D. Polyuria, polydipsia, and elevated BUN and creatinine E. Polyuria, polydipsia, and normal serum sodium, osmolality, BUN, and creatinine Answer: E  Polyuria, polydipsia, and normal serum sodium, osmolality, BUN, and creatinine. Most patients with diabetes insipidus are not hypernatremic, because the polydipsia produced by a normal thirst response is generally sufficient to maintain water homeostasis. Instead, they present with polyuria, polydipsia, and a normal sodium level and osmolality. Because their fluid intake is sufficient to maintain homeostasis, they are not dehydrated and do not have an elevated BUN or creatinine. In these patients, further testing is necessary to increase serum osmolality and then measure the plasma vasopressin level or the urinary response to an administered vasopressin agonist.

4. After a water deprivation test, a patient increases their urine osmolality from 350 to 375 mOsm/kg H2O following administration of desmopressin (7% increase). What is the most likely diagnosis? A. Primary polydipsia B. Nephrogenic diabetes insipidus C. Partial hypothalamic diabetes insipidus D. Gestational diabetes insipidus E. Osmoreceptor dysfunction Answer: A  Primary polydipsia. Adults with normal vasopressin secretion can concentrate their urine to greater than 800 mOsm/kg H2O and have less than a 10% increase in urine osmolality in response to administered desmopressin. Patients with complete central diabetes insipidus have minimal concentration of the urine with dehydration, and a marked increase in urine osmolality (usually > 50%) in response to administered desmopressin. Patients with nephrogenic diabetes insipidus usually have no increase in urine concentration in response to administered desmopressin, although in some cases of acquired nephrogenic diabetes insipidus, some increased urinary concentration (generally < 10%) can occur. In patients with partial central diabetes insipidus and patients with primary polydipsia, the urine is often somewhat concentrated in response to dehydration, but not to the maximum of a normal person. The chronically reduced level of vasopressin downregulates the synthesis of aquaporin-2 water channels, and the large urine volume, regardless of cause, washes out the medullary osmotic gradient that determines the maximal urine concentration. When desmopressin is administered, patients with partial central diabetes insipidus have a further increase (usually > 10% but < 50%) in urine osmolality, whereas most patients with primary polydipsia have no further increase (i.e., 10 mIU/L) in conjunction with a free T4 level below the lower limit of the reference range. In subclinical hypothyroidism, the TSH level is only modestly elevated; the free T4 level remains in the low-normal to normal range. Secondary or central hypothyroidism refers to deficient thyroid gland function that is the result of inadequate stimulation by TSH. This is due in turn to production of either insufficient or inactive TSH from a number of congenital or acquired pituitary and hypothalamic disorders (Chapter 224).

EPIDEMIOLOGY

Primary hypothyroidism is common, occurring in 5% of individuals. Mild hypothyroidism is present in as many as 15% of older adults. Hypothyroidism is more common in women. It is more prevalent among whites and Latin Americans. Secondary hypothyroidism is rare, representing less than 1% of cases.

PATHOBIOLOGY

Dietary iodine deficiency is a cause of primary hypothyroidism in regions where this micronutrient deficiency exists and is uncorrected by iodine supplementation. The most common cause of primary hypothyroidism in developed countries is autoimmune (or Hashimoto’s) thyroiditis, a condition in which defective immune tolerance leads to inflammatory destruction of thyroid tissue and impaired gland function.2 The condition is characterized by a lymphocytic infiltrate and fibrosis. Circulating antithyroid

CHAPTER 226  Thyroid  

antibodies directed against thyroid peroxidase and thyroglobulin are markers of the disease, but glandular inflammation is principally the result of altered T-cell-mediated immunity. There is a genetic predisposition to the condition, with linkage studies suggesting a polygenic basis. Patients with autoimmune thyroiditis may have other endocrine and nonendocrine autoimmune disorders. It may be a component of the type 2 polyglandular autoimmune syndrome associated with autoimmune adrenal insufficiency and type 1 diabetes mellitus. It is less commonly a component of the type 1 syndrome, which includes adrenal insufficiency, hypoparathyroidism, and chronic mucocutaneous candidiasis (Chapter 231). Other nonendocrine autoimmune conditions associated with autoimmune thyroiditis include atrophic gastritis, pernicious anemia, systemic sclerosis, Sjögren’s syndrome, celiac disease, and vitiligo. Individuals treated with the immunomodulatory agent interferon-α may develop autoimmune thyroiditis with transient or permanent hypothyroidism. Surgical resection of the thyroid gland predictably leads to hypothyroidism. Radioactive iodine therapy for treatment of hyperthyroidism commonly destroys sufficient thyroid tissue to cause postablative hypothyroidism. External beam radiation therapy for head and neck cancer can also cause thyroid gland failure. Exposure to pharmacologic and radiocontrast agents that contain large amounts of iodine (e.g., amiodarone, radiocontrast dyes, some expectorants, topical disinfectants) can disrupt thyroid hormone production. Lithium inhibits secretion of T4 and T3, leading to hypothyroidism in 10% of treated patients. Other pharmacologic agents reported to cause hypothyroidism include stavudine, thalidomide, lenalidomide, imatinib, sunitinib, sorafenib, motesanib, bexarotene, ipilimumab, and aminoglutethimide. There are a number of other rare causes of primary hypothyroidism (see Table 226-3). Congenital hypothyroidism can be due to agenesis or dysgenesis of the thyroid gland or to mutations in genes encoding the enzymes catalyzing thyroid hormone synthesis. Infiltrative disorders that can disrupt thyroid function include hemochromatosis, amyloidosis, systemic sclerosis, and invasive fibrous thyroiditis (also known as Riedel’s thyroiditis). The thyroid gland inflammation that occurs with subacute thyroiditis and painless (postpartum) thyroiditis causes transient hypothyroidism from which most patients recover. Consumptive hypothyroidism can occur in individuals with hemangiomas expressing the type 3 deiodinase, which converts T4 to biologically inactive reverse T3. Secondary or central hypothyroidism may be caused by a number of disorders that impair normal hypothalamic or pituitary control of the thyroid gland (Chapter 224). Infiltrative disorders affecting the hypothalamus that can interfere with TRH secretion include sarcoidosis, hemochromatosis, and histiocytosis. Masses that impinge on the pituitary stalk can impede TRH delivery through the hypophyseal portal system. Compression of thyrotrophic cells by pituitary adenomas and other masses in the sella turcica can inhibit synthesis and secretion of TSH. Surgery and radiation therapy to treat pituitary adenomas can destroy thyrotrophic cells, leading to secondary hypothyroidism that develops as a component of panhypopituitarism. Other disorders associated with secondary hypothyroidism include lymphocytic hypophysitis, pituitary metastases from primary malignant neoplasms,

apoplexy, infarction caused by hemorrhage at the time of delivery in women (also known as Sheehan syndrome [Chapter 224]), and head trauma.

CLINICAL MANIFESTATIONS

Symptoms and Signs

Symptoms of hypothyroidism include fatigue, lethargy, weight gain despite poor appetite, cold intolerance, hoarseness, constipation, weakness, myalgias, arthralgias, paresthesias, dry skin, and hair loss. Females may develop precocious puberty, menorrhagia, amenorrhea, and galactorrhea. Affected individuals may experience depressed mood with limited initiative and sociability. Cognitive deficits can range from mild lapses in memory to delirium, dementia, seizures, and coma. The nonspecific nature of most of these symptoms makes it difficult to determine which patients presenting with them have hypothyroidism rather than other causes. Furthermore, in most cases, hypothyroidism is insidious in onset, making its recognition difficult. Symptoms that are new, progressive, or present in combination are more likely to be due to hypothyroidism. The physical findings associated with hypothyroidism vary according to the age at onset and disease severity. Children may present with delayed linear growth despite weight gain, precocious or delayed puberty, and pseudohypertrophy of muscle. Adults can present with bradycardia, diastolic hypertension, and mild hypothermia. The skin may be coarse, dry, yellow, and cool to the touch as a result of peripheral vasoconstriction. Diffuse thinning of scalp hair accompanied by thinning of the lateral eyebrows may occur. The nails may become brittle. Examination of the chest may reveal distant heart sounds. The extremities may reveal diffuse nonpitting edema caused by the deposition of glycosaminoglycans. Neurologic examination may reveal slow, dysarthric speech and diffuse slowing of deep tendon reflexes with a marked delay in the terminal relaxation phase. Examination of the neck may reveal a range of findings. Healed cervical incisional scars in this region may indicate a history of surgical resection of thyroid tissue. In autoimmune thyroiditis, the thyroid gland may be normal in size, diffusely enlarged, or atrophic to the degree it may be difficult to palpate at all. It may be soft and smooth with a lobular texture, or firm and irregular with a variegated nodular texture.

Other Routine Test Abnormalities

Routine blood tests may reveal anemia (which is typically macrocytic), hyponatremia, hypoglycemia, and elevated creatine phosphokinase, prolactin, homocysteine, triglyceride, and total and LDL cholesterol levels. Electrocardiography may show sinus bradycardia with low voltage in the limb leads. Chest radiography may show a widened cardiac silhouette, and echocardiography may confirm a pericardial effusion.

DIAGNOSIS

Suspected primary hypothyroidism is confirmed by an elevated TSH level (Fig. 226-1). Established reference ranges for TSH levels typically extend from 0.5 to 4.5 mIU/L. However, the distribution of values within this range is skewed toward the lower half, such that the mean TSH level in adults is

Suspected Hypothyroidism

Check TSH and free T4

TSH

TSH

Normal / TSH

Normal TSH

Free T4

Normal free T4

Free T4

Normal free T4

Mild or subclinical hypothyroidism

• Central hypothyroidism • Nonthyroidal illness • Drug effect

Primary hypothyroidism • Thyroid hormone replacement targeted to maintain a normal TSH level

• Consider thyroid hormone replacement

1503

FIGURE 226-1.  Laboratory assessment of suspected hypothyroidism. TSH = thyroid-stimulating hormone.

Normal

1504

CHAPTER 226  Thyroid  

1.5 mIU/L. Measurement of the free T4 level confirms the diagnosis of primary hypothyroidism and characterizes its severity. A low free T4 level in conjunction with a persistently elevated TSH level represents overt primary hypothyroidism, whereas a low-normal free T4 level with an elevated TSH level is termed mild or subclinical primary hypothyroidism. Other uncommon causes of isolated TSH elevation should be considered in appropriate settings, including recovery from severe systemic illness, renal failure, and adrenal insufficiency. The underlying cause of primary hypothyroidism is usually clinically obvious, and laboratory testing is unnecessary in most cases. When confirmation is required (e.g., to convince a patient the condition is permanent), serum antithyroid antibodies may be assessed. Measurement of thyroid peroxidase antibody is a more sensitive test than thyroglobulin antibody for this purpose. However, 10% of patients with histologically documented autoimmune thyroiditis have no circulating antithyroid antibodies. When clinical findings such as the presence of a sellar mass, previous pituitary surgery or irradiation, or other pituitary axis hormone deficiencies suggest the possibility of secondary hypothyroidism, the TSH level cannot be relied on to provide an accurate index of thyroid function. In these settings, the serum free T4 level must be assessed, and a low or even low-normal free T4 level can confirm the diagnosis. The TSH level in patients with secondary hypothyroidism can be low, normal, or even modestly elevated.

TREATMENT  The goals of thyroid hormone replacement therapy are straightforward: to replace endogenous thyroid hormone production, to avoid iatrogenic thyrotoxicosis, and (rarely) to treat systemic complications of severe hypothyroidism. Levothyroxine sodium (hereafter thyroxine) is the hormonal preparation of choice.3 Thyroxine has a number of favorable pharmacokinetic characteristics. It is well absorbed, and its plasma protein binding gives it a 7-day half-life, permitting daily dosing. Thyroxine is physiologically deiodinated to the more biologically active T3 in peripheral tissues. However, thyroxine has a narrow therapeutic index, and doses differing by as little as 12% can have clinical consequences. Tablets of multiple dose strengths ranging from 25 to 300 µg are available. Regulatory standards ensure pharmaceutical equivalence in terms of mass of thyroxine, but bioavailability may differ by as much as 12% among different preparations. Consequently, adherence to a single thyroxine formulation is advisable. The optimal dose of thyroxine for replacement therapy is related to lean body weight, with most adults requiring a daily dose of approximately 1.8 µg/ kg. The dose requirement for elderly adults is typically lower (e.g., 1 µg/kg/day) because of slower metabolic clearance. Patients with postsurgical or postablative hypothyroidism usually require a higher daily dose than patients with autoimmune thyroiditis, who may have some residual gland function. Patients with coexisting malabsorptive disorders may require higher and variable doses. Certain medications, mineral supplements, and foods can interfere with thyroxine absorption, including ferrous sulfate, calcium carbonate, aluminum hydroxide, sucralfate, cholestyramine, and soy-containing foods (Table 226-4). Thyroxine doses should be separated from these substances by 8 hours or longer. Thyroxine dose requirements may increase as a result of accelerated metabolic clearance in several circumstances. Patients with nephrotic syndrome and other systemic illnesses that lead to rapid clearance of thyroid hormone require higher daily doses. Dose requirements increase by an average of 75% in most pregnant women as a result of placental deiodinative metabolism of thyroxine.4 Simultaneous treatment with phenytoin, phenobarbital, carbamazepine, or rifampin also typically accelerates thyroxine metabolism. Most adults without known or suspected coronary artery disease can be started on a full replacement dose of thyroxine. The initial dose can be calculated on the basis of the patient’s weight and age, rounding down to the nearest available dose strength. For patients with primary hypothyroidism, adequacy of thyroxine therapy can be assessed by TSH measurement 4 to 6 weeks after therapy is started. The target TSH level for most treated individuals should be the lower half of the reference range (i.e., 0.5 to 2.0 mIU/L). Once an adequate dose has been established, the TSH level should be checked annually. In patients with secondary hypothyroidism, the serum free T4 level should be monitored 2 to 4 weeks after the thyroxine dose is started or adjusted, with a target free T4 level in the upper half of the reference range.

Management of Complications

Complications of thyroxine therapy are limited to iatrogenic thyrotoxicosis and, rarely, adverse effects of restoring euthyroidism. Typical symptoms and signs of thyrotoxicosis usually accompany significant degrees of overtreatment. However, even a modestly excessive thyroxine dose can induce bone mineral loss, especially in postmenopausal women, and it can increase the risk of atrial fibrillation in older individuals. In patients with underlying coronary

TABLE 226-4  INTERFERENCE WITH THYROXINE REPLACEMENT THERAPY FACTORS CONTRIBUTING TO UNDERREPLACEMENT Inadequate prescribed dose Limited compliance Decreased absorption due to ingestion of agents that bind thyroxine Ferrous sulfate Calcium carbonate Aluminum hydroxide Sucralfate Cholestyramine Soy protein Increased metabolism of thyroxine Pregnancy Drugs Phenytoin Phenobarbital Carbamazepine Rifampin Diminishing residual thyroid function Changing formulations FACTORS CONTRIBUTING TO OVERREPLACEMENT Excessive prescribed dose Factitious ingestion of additional doses Decreased metabolism of thyroxine due to aging Increasing residual thyroid function Changing formulations

artery disease, the positive chronotropic and inotropic effects of thyroxine may exacerbate myocardial ischemia.5 Consequently, adults with known or suspected ischemic heart disease should be started on a low dose that is titrated upward in small increments once tolerance is ensured (e.g., starting with 25 µg daily, then increasing the dose by 12.5 to 25 µg every 4 to 6 weeks). In some cases, β-blocker therapy may need to be intensified to counter the induction of myocardial ischemia. However, deliberate suboptimal dosing of thyroxine should be avoided. If necessary, coronary revascularization may be required before euthyroidism can be fully restored. Coexisting adrenal insufficiency associated with hypopituitarism or the type 2 polyglandular autoimmune syndrome may be unmasked when cortisol clearance is accelerated by a return to the euthyroid state. Other adverse effects that infrequently occur with thyroxine therapy include transient hair loss, acute sympathomimetic symptoms that resolve with dose reduction and slow advancement, and pseudotumor cerebri in children. A minority of patients with thyroxine-treated hypothyroidism continue to report bothersome symptoms despite biochemical evidence of adequate thyroid hormone replacement. Several randomized clinical trials have shown that combinations of T3 and T4—in the form of desiccated thyroid or synthetic thyroid hormone preparations—are not superior to T4 alone. A1 

Subclinical and Mild Hypothyroidism

Whether individuals diagnosed with subclinical hypothyroidism (i.e., an elevated or high-normal TSH level with a free T4 level within the reference range) benefit from thyroxine therapy remains controversial.6 In practice, many providers opt for a trial of therapy in mildly hypothyroid patients who are symptomatic, have underlying hypercholesterolemia, or have a high likelihood of progressing to overt hypothyroidism. Predictors of progressive thyroid failure include age older than 65 years, TSH level higher than 10 mIU/L, and the presence of circulating thyroid autoantibodies, indicating underlying autoimmune thyroiditis.

Myxedema Coma

Severe hypothyroidism can culminate in myxedema coma, a life-threatening condition characterized by hypothermia, bradycardia, hypotension, altered mental status, and multisystem organ failure. Risk factors include advanced age, poor access to health care, and other underlying major organ system diseases. Most patients have severe and long-standing thyroid hormone deficiency. Treatment should include thyroxine (1.8 µg/kg/day, with or without a 500-µg loading dose). Some experts advocate coadministration of triiodothyronine in divided doses to compensate for impaired conversion of T4 to T3. No controlled trials have been performed to evaluate the relative benefits and risks of these different approaches. Glucocorticoids should be administered in stress doses after a cosyntropin stimulation test has been performed to check for evidence of concomitant adrenal insufficiency (Chapter 227). Care should be taken to avoid exposure to potent sedative or analgesic agents that may exacerbate altered mental status. Hypothermia should be treated with external warming to reduce the risk of circulatory collapse.

CHAPTER 226  Thyroid  

Nonthyroidal Illness

In patients with severe nonthyroidal illness, a characteristic constellation of thyroid function test changes occurs that often appears to be consistent with hypothyroidism (see Fig. 226-1).7 The T3 level usually declines as a result of decreased extrathyroidal T4-to-T3 conversion. With increasingly severe disease, total T4 and free T4 levels also decline. TSH levels are usually low to low-normal. During the course of recovery, the TSH level can rise above the upper limit of the normal range, producing a profile that can be mistaken for primary hypothyroidism. Clinical correlation is essential to assess thyroid function in severely ill patients (e.g., a history of preexisting thyroid or pituitary disease, the presence of a goiter, or features suggesting other elements of hypopituitarism). Because no benefit of thyroid hormone treatment has been shown for these patients, observation with retesting 6 to 8 weeks after recovery is the preferred approach.

  THYROTOXICOSIS DEFINITION AND EPIDEMIOLOGY

Thyrotoxicosis is a systemic syndrome caused by exposure to excessive thyroid hormone (Table 226-5). Its prevalence is 1 in 2000 adults, affecting 1% of all individuals during the course of their lifetime.

PATHOBIOLOGY

Thyrotoxicosis is the result of excessive circulating and tissue effects of thyroid hormone. Strictly speaking, hyperthyroidism refers to those forms of thyrotoxicosis that are caused by excessive production of thyroid hormone by the thyroid gland due to a thyrotropic stimulus or autonomous thyroid tissue function (see Table 226-5). In Graves disease, the most common cause of hyperthyroidism, the thyroid gland is stimulated by autoantibodies that bind to and activate the TSH receptor. Excessive secretion of TSH causes hyperthyroidism in patients with rare TSH-secreting pituitary adenomas (Chapter 224). CG, a glycoprotein with high TSH homology, can cause transient gestational hyperthyroidism during pregnancy, when a choriocarcinoma or a germ cell tumor produces variant forms of HCG that are more active or when mutant TSH receptors bind HCG more avidly, as occurs in familial gestational thyrotoxicosis. Autonomous production of thyroid hormone occurs when thyrocytes function independently of TSH receptor activation. This can occur as a result of growth of a benign functioning thyroid adenoma or growth of multiple autonomously functioning nodules forming a toxic multinodular goiter. In rare cases, it can occur when patients with well-differentiated thyroid cancer present with functioning metastases. In some toxic adenomas, somatic

TABLE 226-5  ETIOLOGIES OF THYROTOXICOSIS HYPERTHYROIDISM Antibody-mediated stimulation of thyroid tissue Graves disease Autonomously functioning thyroid tissue Toxic multinodular goiter Toxic adenoma Iodine exposure Autonomously functioning heterotopic thyroid tissue Struma ovarii Metastatic differentiated thyroid cancer Excessive secretion of TSH TSH-secreting pituitary adenoma NONHYPERTHYROID THYROTOXICOSIS Ingestion of exogenous thyroid hormone Pharmacologic Levothyroxine Liothyronine Combination preparations Nonpharmacologic Dietary supplements Improperly processed meat products Inflammation causing release of endogenous thyroid hormone Subacute thyroiditis Autoimmune thyroiditis TSH = thyroid-stimulating hormone.

1505

mutations in the TSH receptor gene lead to constitutive activation. In patients whose thyroid glands have the potential for autonomous function, exposure to excessive amounts of iodine in the form of amiodarone or iodinated contrast agents can provoke hyperthyroidism. Transient thyrotoxicosis can also be caused by inflammatory conditions that release an excessive amount of thyroid hormone stored in the gland (see the section on thyroiditis). These include subacute thyroiditis, which is believed to be caused by a viral infection; acute or suppurative thyroiditis, caused by bacterial infection; radiation-induced thyroiditis; and pharmacologic thyroiditis (e.g., due to amiodarone). Autoimmunity can also provoke an inflammatory thyroiditis that causes transient thyrotoxicosis. This commonly occurs in the setting of lymphocytic thyroiditis (also known as silent, painless, or postpartum thyroiditis). It rarely occurs in the setting of autoimmune thyroiditis (also known as Hashimoto’s thyroiditis). In rare cases, excess thyroid hormone can be secreted by ectopic thyroid tissue located anywhere from the base of the tongue to the mediastinum, or by heterotopic thyroid tissue that develops as part of an ovarian teratoma (a condition known as struma ovarii). Thyrotoxicosis can also be caused by ingestion of excessive amounts of thyroid hormone. This is most often the result of the prescription of excessive doses of pharmacologic preparations of thyroid hormone, but it can rarely be due to surreptitious or accidental ingestion.

CLINICAL MANIFESTATIONS

Symptoms and Signs

The classic symptoms of thyrotoxicosis include weight loss despite a hearty appetite, heat intolerance, palpitations, tremor, and hyperdefecation (increased frequency of formed bowel movements). Thyrotoxicosis can escape early detection because of its presentation with common nonspecific symptoms such as fatigue, insomnia, anxiety, irritability, weakness, atypical chest pain, or dyspnea on exertion. Delayed recognition may also occur when atypical symptoms such as headache, weight loss, periodic paralysis, or nausea and vomiting dominate the clinical picture. Elderly patients may present with apathetic thyrotoxicosis typified by weight loss and the absence of sympathomimetic symptoms and signs. Signs of thyrotoxicosis include resting tachycardia, systolic hypertension with a widened pulse pressure, warm moist skin with a velvety texture, onycholysis, and a staring gaze with lid lag (noted to be present when a rim of sclera is visible between the upper eyelid and the superior margin of the iris on downward gaze). Cardiac examination may reveal a prominent apical impulse and a systolic flow murmur. Neurologic findings may include a restless, impatient demeanor, pressured speech, proximal muscle weakness, distal hand tremor, and brisk deep-tendon reflexes. Clinical findings often provide clues to the underlying cause.8 In Graves disease, the gland is diffusely enlarged with a smooth or slightly lobulated contour, and may manifest an audible bruit or palpable thrill. Thyroid ophthalmopathy and dermopathy are also unique to Graves disease. In patients with toxic nodular goiter, one or more discrete nodules may be appreciated. In subacute thyroiditis, the gland is modestly enlarged, extremely tender, and firm. A history of recent pregnancy suggests possible painless thyroiditis. Recent exposure to amiodarone, other iodine-containing compounds, interferon-α, or pharmacologic preparations of thyroid hormone may suggest the characteristic forms of thyrotoxicosis associated with these agents.

Graves Disease

DEFINITION

Graves disease is an autoimmune disorder characterized by a variable combination of hyperthyroidism, ophthalmopathy (also known as thyroid eye disease), and dermopathy.

EPIDEMIOLOGY

Graves disease is more common among women, but it also affects men. It can develop at any time during life, but the onset most often occurs between 30 and 60 years of age.

PATHOBIOLOGY

The proximate cause of hyperthyroidism in Graves disease is the production of thyroid-stimulating immunoglobulins that bind to and activate the TSH receptor, promoting thyroid hormone secretion and gland growth. Thyrotropin (TSH) receptor antibodies of the stimulating variety are the hallmark of hyperthyroidism in Graves disease. Other thyroid autoantibodies commonly identified in the setting of Graves disease include thyroid peroxidase

1506

CHAPTER 226  Thyroid  

antibodies, thyroglobulin antibodies, and TSH receptor antibodies. Although the fundamental cause of Graves disease remains unknown, a genetic predisposition is implicated by a higher incidence in monozygotic twins and firstdegree relatives of affected individuals. Environmental factors implicated in triggering the onset of Graves disease include exposure to cigarette smoke, high dietary iodine intake, and perhaps stressful life events and certain antecedent infections.

CLINICAL MANIFESTATIONS

Affected individuals usually present with thyrotoxicosis and a thyroid gland that is diffusely enlarged with a rubbery consistency, smooth contour, definable pyramidal lobe, and audible bruit or palpable thrill due to increased blood flow. When it is clinically evident, thyroid eye disease usually presents within a few months of onset. In rare cases, it may develop long before, long after, or without any biochemical confirmation of hyperthyroidism.

PROGNOSIS

The hyperthyroidism associated with this condition often follows a persistent and progressive course, but one fourth of patients with Graves disease demonstrate spontaneous disease remission.

OPHTHALMOPATHY DEFINITION

Thyroid eye disease is a distinctive disorder characterized by inflammation and swelling of the extraocular muscles and orbital fat, eyelid retraction, periorbital edema, episcleral vascular injection, conjunctival swelling (chemosis), and proptosis (also called exophthalmos).9 Swelling of soft tissues within the confines of the orbits precipitated by fibroblast growth and inflammatory cell infiltrate can cause proptosis, entrapment of extraocular muscles, and compression of the optic nerve.

CLINICAL MANIFESTATIONS

Affected individuals typically complain of a change in eye appearance, ocular irritation, foreign body sensation, dryness, and ironically, excessive tearing. More severe involvement may cause exposure keratitis with corneal ulceration, diplopia, and blurred vision. On examination, patients may have a staring gaze, a rim of sclera visible between the upper eyelid and the superior margin of the iris during downward gaze (lid lag), signs of conjunctival inflammation, periorbital edema, and abnormalities of conjugate gaze, color vision, and visual acuity (Fig. 226-2). The precise degree of proptosis can be

measured with an exophthalmometer. Orbital imaging with CT scanning or ultrasonography can confirm the diagnosis, which must sometimes be distinguished from other causes of bilateral and unilateral proptosis.

TREATMENT  Treatment of mild thyroid eye disease focuses on protecting the cornea from exposure and desiccation with moisturizing drops and ointment, glasses, and sometimes taping the eyelids closed at bedtime. Selenium supplementation may help to relieve some of the symptoms associated with active inflammation in mild to moderate cases. A2  High-dose systemic glucocorticoid therapy can attenuate orbital inflammation in more severe cases. Orbital irradiation may be helpful in controlling inflammatory symptoms in some patients. Persistent corneal exposure, diplopia, altered vision due to optic nerve compression, and cosmetic issues may require surgery to decompress the orbits and readjust the extraocular muscles. Immunosuppressive agents and plasmapheresis have been used in severely affected patients, with anecdotal success.

DERMOPATHY

Infiltrative dermopathy, the least common aspect of Graves disease, is precipitated by the deposition of glycosaminoglycans in the dermis of the skin. Affected individuals usually present with mildly pruritic, orange peel–like thickening of the skin along the anterior aspects of the shins, known as pretibial myxedema. The dorsal aspects of the feet and fingers, the extensor surface of the elbows, and the face are more rarely affected. The diagnosis can be confirmed by skin biopsy. Treatment of early infiltrative dermopathy with topical glucocorticoids under an occlusive wrap may limit its progression. Treatments involving the use of intradermal or systemic glucocorticoids, long-acting somatostatin analogues, and even surgical resection of soft tissue have demonstrated limited success.

Toxic Adenoma A toxic adenoma is a solitary, autonomously functioning thyroid neoplasm that synthesizes and secretes excessive amounts of thyroid hormone independent of TSH stimulation. These neoplasms are almost always benign. Most grow large enough to be palpated by the time they present with thyrotoxicosis. Somatic gene mutations causing constitutive activation of the TSH receptor and the α-subunit of the stimulatory guanine nucleotide binding protein (Gsα) have been identified in a subset of toxic adenomas. Hyperthyroidism caused by a toxic adenoma does not remit spontaneously, except in unusual cases complicated by hemorrhagic infarction of the neoplasm.

Toxic Multinodular Goiter A toxic multinodular goiter is composed of multiple autonomously functioning thyroid nodules that synthesize and secrete excessive amounts of thyroid hormone. In some patients with nontoxic multinodular goiters, hyperthyroidism can be precipitated by exposure to excessive amounts of iodine. Most affected individuals have a goiter with multiple palpable thyroid nodules. Progressive enlargement may go undetected when there is substernal extension of nodular tissue. Toxic multinodular goiters are more common among older individuals.

A

B FIGURE 226-2.  Graves ophthalmopathy. A, A 59-year-old woman with excess proptosis, moderate eyelid edema, and erythema with moderate eyelid retraction affecting all four eyelids. Conjunctival chemosis (edema) and erythema with bilateral edema of the caruncles, with prolapse of the right caruncle, are evident. B, A 40-year-old woman with excess proptosis, minimal bilateral injection, and chemosis with slight erythema of the eyelids. On slit lamp examination, she also had evidence of moderate superior limbic keratoconjunctivitis. (From Bahn RS. Graves’ ophthalmopathy. N Engl J Med. 2010;362:726738. Copyright 2010, Massachusetts Medical Society. All rights reserved.)

TSH-Secreting Pituitary Adenoma TSH-secreting pituitary adenomas represent less than 1% of all functioning pituitary tumors (Chapter 224). Patients may present with typical clinical manifestations of thyrotoxicosis, a diffuse goiter, symptoms and signs precipitated by an expanding sellar mass, syndromes associated with co-secretion of other anterior pituitary hormones (growth hormone, prolactin, or adrenocorticotropic hormone), or symptoms and signs of hypopituitarism. The key to suspecting the condition is usually recognition of an inappropriately nonsuppressed TSH level in a patient with thyrotoxicosis. The diagnosis is confirmed in most cases when laboratory testing reveals an elevated circulating level of the pituitary glycoprotein α-subunit in conjunction with a radiographically definable sellar mass.

DIAGNOSIS

Laboratory Findings

Abnormalities detected in routinely ordered laboratory tests are often the first clues to the presence of thyrotoxicosis. Thyrotoxic patients may have hypercalcemia or hypercalciuria, increased alkaline phosphatase levels, modestly elevated transaminase levels, and low or declining total and LDL

CHAPTER 226  Thyroid  

1507

Suspected Thyrotoxicosis

Check TSH and free T4

TSH

TSH

Normal / TSH

Normal TSH

Free T4

Normal free T4

Free T4

Normal free T4

Thyrotoxicosis

• T3 thyrotoxicosis • Mild or subclinical thyrotoxicosis • Nonthyroidal illness

• TSH-secreting pituitary adenoma • Thyroid hormone resistance syndrome • Familial dysalbuminemic hyperthyroxinemia

Normal

FIGURE 226-3.  Laboratory assessment of suspected thyrotoxicosis. TSH = thyroid-stimulating hormone.

cholesterol levels. When they are measured, ferritin and angiotensinconverting enzyme levels are often increased. Electrocardiography typically reveals resting sinus tachycardia or atrial tachyarrhythmias, particularly atrial fibrillation with a rapid ventricular response. In severe cases, chest radiography may reveal cardiomegaly. In most patients with suspected thyrotoxicosis, the diagnosis can be confirmed by measurement of a TSH level (Fig. 226-3). Sensitive TSH immunoassays with a detection limit of less than 0.02 mIU/L can accurately discriminate between clearly suppressed TSH levels, characteristic of all common forms of thyrotoxicosis, and mildly suppressed levels that fall just beneath the reference range, as may occur in otherwise sick individuals. Only the rare conditions associated with TSH-mediated hyperthyroidism (TSH-secreting pituitary tumors and isolated pituitary resistance to thyroid hormone) lack TSH suppression when testing for thyrotoxicosis. Measurement of serum free T4 and T3 levels confirms the diagnosis of thyrotoxicosis, defines its severity, and occasionally provides a clue to its underlying cause. Overt thyrotoxicosis is characterized by free T4 or T3 levels above the upper limit of the reference range, whereas mild or subclinical thyrotoxicosis is characterized by a suppressed TSH level with free T4 and T3 levels within the normal reference range. When only the free T4 or T3 concentrations are elevated, the terms T4 toxicosis and T3 toxicosis are applied, respectively.

Differential Diagnosis

Once thyrotoxicosis is confirmed, it is important to define its underlying cause to determine the most appropriate course of treatment. The relative degrees of T4 and T3 elevation sometimes can be helpful. Predominantly T3 toxicosis is typical of Graves disease and can also occur with toxic nodular goiter. In contrast, predominantly T4 toxicosis is more typical of subacute or painless thyroiditis. T4 toxicosis is also more common in patients with iodineinduced hyperthyroidism. Other laboratory tests are sometimes helpful in differential diagnosis. Antithyrotropin-receptor antibodies are pathognomonic of Graves disease. Levels of antithyrotropin-receptor antibodies are especially high in thyroid dermopathy and correlate positively with the clinical features and prognosis of Graves ophthalmopathy. An elevated ESR is typically seen in subacute thyroiditis. Imaging studies can be helpful for the differential diagnosis. The fractional thyroidal uptake of radiotracer by the thyroid and its distribution in the gland on scintigraphic scanning often helps establish a definitive diagnosis (Table 226-6). Thyroid ultrasonography can confirm the presence of solitary or multiple thyroid nodules. Chest radiography and CT scanning may help delineate a substernal goiter.

TREATMENT  Selection of the most effective treatment for a specific condition causing thyrotoxicosis requires an understanding of the underlying pathophysiologic process and natural history. A3  For example, toxic multinodular goiter does not

TABLE 226-6  RADIOGRAPHIC EVALUATION OF SUSPECTED THYROTOXICOSIS

ETIOLOGY

FRACTIONAL 24-HOUR RADIOIODINE UPTAKE (%)

THYROID SCAN APPEARANCE

Graves disease

35-95

Diffuse increased homogeneous uptake; visible pyramidal lobe extending from isthmus

Toxic adenoma

20-60

Solitary focus of intense uptake; suppression of uptake in remainder of thyroid

Toxic multinodular goiter

20-60

Patchy heterogeneous foci of increased uptake interspersed with regions of diminished uptake

Subacute thyroiditis

0-2

Minimal to absent uptake

Autoimmune thyroiditis

0-2

Minimal to absent uptake; patchy heterogeneous uptake during recovery

Iodine-induced hyperthyroidism

0-2

Minimal to absent uptake

Exogenous thyroid hormone intoxication

0-2

Minimal to absent uptake

Metastatic differentiated thyroid cancer

0-5

Focal uptake in metastases

TSH-secreting pituitary adenoma

30-80

Diffuse increased homogeneous uptake

TSH = thyroid-stimulating hormone.

remit and requires definitive radioiodine treatment or surgery; subacute thyroiditis subsides spontaneously and requires only temporizing symptomatic therapy.

β-Blockers

β-Blockers help alleviate the sympathomimetic manifestations of thyrotoxicosis, regardless of the underlying cause. Palpitations, tremor, and anxiety can often be promptly controlled. However, other clinical features of thyrotoxicosis, including weight loss, heat intolerance, and fatigue, are not ameliorated by these agents. In thyrotoxic patients with marked sinus tachycardia or atrial fibrillation with a rapid ventricular response rate, β-blockers can be used as rate-controlling agents. Propranolol also partially inhibits extrathyroidal conversion of T4 to T3, which may be of added benefit in patients with severe thyrotoxicosis. Propranolol can be started at a dose of 20 to 40 mg every 8 hours and titrated upward to a maximal daily dose of 240 mg on the basis of symptom control. Sustained-release propranolol or longer-acting β-blockers, such as

1508

CHAPTER 226  Thyroid  

metoprolol and atenolol, can also be used. β-Blockers should be used with caution in thyrotoxic patients with a history of obstructive pulmonary disease, Raynaud’s phenomenon, or heart failure. Esmolol can be used when a shortacting parenteral agent is required for heart rate control in patients with thyrotoxic heart failure. For patients with transient forms of thyrotoxicosis (subacute thyroiditis, autoimmune thyroiditis, or exogenous thyroid hormone intoxication), a β-blocker may be the only treatment required. In patients with more sustained conditions, such as Graves disease or toxic nodular goiter, β-blockers provide prompt initial relief of symptoms while definitive treatment with antithyroid drugs, radioiodine, or surgery is implemented.

Antithyroid Drugs

The thionamides inhibit thyroid hormone biosynthesis by competitively inhibiting iodine organification and iodotyrosine coupling. These agents are used for the treatment of thyrotoxicosis caused by overproduction of thyroid hormones. Because the thionamides block only new thyroid hormone synthesis, glandular stores of preexisting thyroid hormone must be exhausted before they are fully effective. This may require 3 to 8 weeks in patients with Graves disease or toxic multinodular goiter. Although antithyroid drugs can provide long-term control of hyperthyroidism, they are most appropriately used when there is a possibility that the underlying condition will remit, as in Graves disease, or when thyrotoxicosis must be attenuated before radioiodine treatment or surgery. Two thionamide agents are currently available: methimazole and propylthiouracil. Methimazole can be taken as a single daily dose because of its longer half-life and higher effective intrathyroidal concentration. This can bolster patients’ adherence and drug effectiveness. Propylthiouracil also inhibits extrathyroidal conversion of T4 to T3, an effect that may be beneficial in patients with severe complicated thyrotoxicosis. Propylthiouracil is preferred for pregnant hyperthyroid women in the first trimester because methimazole has been rarely associated with the congenital anomalies of choanal atresia and cutis aplasia.10 However, the shorter half-life of propylthiouracil necessitates its administration three or four times daily. Furthermore, risk of severe hepatotoxicity associated with the use of propylthiouracil has prompted the recommendation that methimazole be the first-line antithyroid drug to treat hyperthyroidism in children and adults, including women after the first trimester of pregnancy. A4  For patients with mild to moderate hyperthyroidism, methimazole is usually started at a dose of 10 to 30 mg once daily and increased to as much as 90 mg daily. For patients with more severe hyperthyroidism, thyrotoxicosis complicated by cardiac disease, or concomitant pregnancy, propylthiouracil can be started at a dose of 50 to 200 mg every 6 to 8 hours. Methimazole can be given rectally if necessary. The anticipated duration of treatment depends on the underlying cause. In patients with toxic multinodular goiter, antithyroid drugs are generally used only to restore euthyroidism in anticipation of definitive therapy. An effective dose can be continued for 6 to 24 months in a patient with Graves disease, before it is tapered off to determine whether there has been a remission of the patient’s autoimmune thyroid disease. Patients most likely to respond are those who present with mild clinical and biochemical hyperthyroidism, a small thyroid gland, and no active ophthalmopathy. Patients treated with antithyroid drugs should have thyroid function tests checked every 3 to 12 weeks during dose titration to monitor for iatrogenic hypothyroidism. Common side effects include rash, pruritus, fever, and arthralgias, which affect 5% of thionamide-treated patients. Agranulocytosis and hepatitis are rare but potentially fatal adverse reactions to thionamide medications. Their presentations are relatively sudden in onset and unpredictable. Monitoring of leukocyte counts and liver function test results is not useful as a preventive measure. Patients who are prescribed antithyroid drugs should be cautioned about manifestations of these adverse reactions and should be instructed to discontinue treatment and seek medical attention if they develop a high fever, pharyngitis, jaundice, or abdominal pain.

Radioactive Iodine

The selective uptake and concentration of iodide in thyrocytes permits the use of radioactive iodine to treat hyperthyroidism.11 Once it is concentrated in the gland after oral administration, 131I destroys thyroid tissue and controls hyperthyroidism, usually within 1 to 2 months. The dose of 131I can be calculated on the basis of the fractional uptake of radioiodine, but the outcome of dosimetry is not superior to that achieved with the administration of empirical doses. Patients can be treated on an outpatient basis, with precautions taken to prevent exposure of others. Approximately three quarters of patients are cured with a single dose of radioiodine. The principal side effect of radioactive iodine therapy is postablative hypothyroidism, which develops in most individuals receiving treatment for Graves disease and in a lesser proportion of patients treated for toxic nodular goiter. Lifelong monitoring of thyroid function is required because patients develop postablative hypothyroidism at a rate of 3% per year. Another less common complication is a transient exacerbation of thyrotoxicosis, which occurs in one quarter of patients during the first month after treatment as a result of radiation thyroiditis. Long-term follow-up studies have shown that radioiodine-

treated patients with Graves disease do not have any greater risk of thyroid cancer or other malignant neoplasms. However, hyperthyroid children and adolescents treated with radioactive iodine are more likely to develop benign nodules. Among hyperthyroid women treated with radioiodine, the incidences of infertility, spontaneous abortion, and children with birth defects are not increased. Diagnostic or therapeutic radioactive iodine is contraindicated in women during pregnancy, and treated women should avoid pregnancy until euthyroidism has been confirmed 3 to 6 months after administration of a dose.

Other Drugs

Saturated solution of potassium iodide (SSKI) or Lugol’s solution transiently inhibit the synthesis and release of thyroid hormone from the gland. They may be used to accelerate recovery after radioactive iodine treatment, to prepare patients for thyroidectomy, and to augment other treatments used to control severe thyrotoxicosis (see later). Iodinated radiocontrast agents inhibit the release of thyroid hormone while blocking peripheral conversion of T4 to T3. Lithium carbonate also inhibits the release of thyroid hormone. Rarely, these agents are used in combination with thionamides to treat patients with severe thyrotoxicosis. They may also help provide temporary control of hyperthyroidism when severe allergies preclude the continued use of thionamides. Cholestyramine can be used to bind thyroid hormone in the gut to interrupt enterohepatic circulation in cases of suspected exogenous thyroid hormone intoxication.

Surgery

Surgery has a limited role because of its potential to injure the adjacent recurrent laryngeal nerves and parathyroid glands. Resection of a toxic adenoma by lobectomy is curative and often preserves sufficient normal thyroid tissue for euthyroidism to be maintained. Consequently, it is often recommended in younger individuals. Toxic multinodular goiters causing compressive symptoms or cosmetic disfigurement may be appropriately managed with surgical resection. Although surgery is seldom recommended in the United States for the treatment of hyperthyroid Graves disease, it may be appropriate when other modalities are contraindicated, such as when there has been an adverse reaction to an antithyroid drug in pregnancy, when a thyroid nodule is thought to be malignant, or when hyperparathyroidism also requires surgical intervention.

Specific Treatment Scenarios Pregnancy

Pregnant patients with hyperthyroidism present special challenges. Diagnosis requires a careful assessment of symptoms, especially heat intolerance, palpitations, and vomiting, which also occur during normal pregnancy. The serum total T4 level is elevated because of increased TBG, and the TSH level can be suppressed in the first trimester as a result of HCG-mediated thyroid stimulation. Diagnostic radionuclide imaging studies are contraindicated. After diagnostic confirmation, hyperthyroidism must be treated because it is associated with an increased risk of spontaneous abortion, premature labor, low birth weight, and toxemia. β-Blockers should be used only transiently to control severe symptoms. Propylthiouracil is the preferred thionamide for treatment of Graves disease during the first trimester of pregnancy, because it crosses the placenta less readily than methimazole and because methimazole has been rarely linked to congenital malformations (i.e., choanal atresia and cutis aplasia). However, owing to the risk of very rare but potentially fatal propylthiouracil-related hepatitis, methimazole is preferred after the first trimester. Because Graves disease often remits later in pregnancy, antithyroid drug dose requirements often decline as gestation progresses. Measurement of maternal thyroid-stimulating immunoglobulin levels can help predict the risk of an infant developing neonatal Graves disease.

Subclinical and Mild Hyperthyroidism

Patients with subclinical or mild hyperthyroidism (i.e., a suppressed serum TSH with normal free T4 and T3 levels) may have symptoms that justify treatment. In patients with a serum TSH level suppressed to less than 0.1 mIU/L, bone mineral loss can lead to osteoporosis, particularly in postmenopausal women. Atrial fibrillation occurs more commonly in mildly hyperthyroid patients aged 60 years and older with TSH suppression below normal. It is less clear, however, whether younger asymptomatic patients with modestly suppressed TSH levels (e.g., 0.1 to 0.5 mIU/L) require anything more than periodic monitoring.

Thyrotoxic Crisis

Thyrotoxic crisis, also known as thyroid storm, is a potentially life-threatening syndrome that is usually the end result of severe and sustained thyrotoxicosis. It can affect patients with other medical conditions that render them vulnerable to the cardiovascular, neuropsychiatric, and gastrointestinal effects of exposure to excessive amounts of thyroid hormone. Thyrotoxic crisis typically develops in the setting of inadequately treated Graves disease and may be precipitated by intercurrent illness, surgery, or treatment with radioactive iodine. Affected individuals present with fever, atrial tachyarrhythmias,

CHAPTER 226  Thyroid  

congestive heart failure, nausea and vomiting, diarrhea, and seizures. Mental status changes can include agitation, delirium, psychosis, and coma. Prompt recognition and treatment in a monitored setting are crucial. A multifaceted treatment regimen should incorporate antipyretics, β-blockers, thionamides, iodinated contrast agents, and glucocorticoids, as well as aggressive evaluation and management of underlying medical problems.

1509

PATHOBIOLOGY

This painless inflammation of the thyroid gland can cause transient thyrotoxicosis followed by transient or persistent hypothyroidism. Each of these phases of thyroid dysfunction typically lasts 2 to 8 weeks. This condition is believed to reflect transient autoimmunity.

DIAGNOSIS

  THYROIDITIS

Subacute (de Quervain’s) Thyroiditis

PATHOBIOLOGY

Transient thyrotoxicosis results from the uncontrolled release of thyroid hormone from the inflamed gland. After 2 to 8 weeks, when the supply of stored hormone is exhausted, thyrotoxicosis resolves spontaneously. Hypothyroidism ensues because the gland’s biosynthetic capabilities remain impaired. This is also transient (lasting ≈ 1 month), with subsequent restoration of normal thyroid function in most patients.

CLINICAL MANIFESTATIONS

Subacute thyroiditis is characterized by painful enlargement of the thyroid, systemic inflammatory symptoms, and transient thyrotoxicosis that is often followed by transient hypothyroidism. The histologic pattern shows inflammatory cell infiltrates that are believed to be the result of a viral infection. Many patients with subacute thyroiditis report antecedent upper respiratory infections. Patients usually present with pain localized to the thyroid or radiating to the throat, ears, or jaw. Constitutional symptoms, including fever, chills, sweats, and malaise, are often present. On occasion, these inflammatory features may dominate the presentation. Examination of the thyroid typically reveals an exquisitely tender, modestly enlarged, and woody, hard gland.

DIAGNOSIS

Differential Diagnosis

The differential diagnosis of thyroid pain must be considered in the evaluation of patients presenting with pain and tenderness localized to the lower anterior neck. In addition to subacute thyroiditis, potential causes of thyroid pain include acute (suppurative) thyroiditis, hemorrhage into an existing thyroid nodule, and rapid growth of anaplastic thyroid cancer, diffusely infiltrating thyroid cancer, or thyroid lymphoma.

Laboratory Findings

Laboratory testing in patients with subacute thyroiditis reveals a profile of overt thyrotoxicosis. Elevated T4 levels are usually proportionately higher than T3 levels. Patients typically have an elevated ESR during the acute phase. The fractional uptake of radioiodine is typically less than 2% at 24 hours (see Table 226-6).

TREATMENT  High-dose aspirin or naproxen sodium can be used to treat thyroid pain and systemic inflammatory symptoms. Patients who fail to respond may require glucocorticoid therapy, but it must be tapered over several weeks to prevent a relapse, prolonging the overall course of the illness. Symptoms ascribed to transient thyrotoxicosis may respond to treatment with a β-blocker continued for a limited course of 1 to 3 weeks. Patients who progress to symptomatic hypothyroidism may need short-term thyroxine replacement therapy, but most do not require long-term thyroid hormone replacement.

Lymphocytic (Postpartum, Painless, Silent) Thyroiditis

EPIDEMIOLOGY

Lymphocytic thyroiditis occurs most commonly in postpartum women, affecting as many as 6% of women 2 to 12 months after delivery or termination. Rarely, this condition occurs in non-postpartum women or in men. Predisposing factors include a history of previous episodes of postpartum thyroiditis, type 1 diabetes mellitus, and circulating antithyroid autoantibodies.

The diagnosis of lymphocytic thyroiditis is often overlooked when nonspecific symptoms of thyrotoxicosis (e.g., weight loss, insomnia, anxiety) or hypothyroidism (e.g., fatigue, depression) are misinterpreted as common postpartum complaints. The thyroid gland is nontender and either normal in size or modestly enlarged. Once it is considered, a diagnosis of lymphocytic thyroiditis can be readily confirmed or excluded by laboratory testing, which reveals a suppressed TSH level during phases of thyrotoxicosis and an elevated TSH level during phases of hypothyroidism. This condition must be distinguished from Graves disease, which can also present in the same time frame after delivery. Relative degrees of T4 and T3 elevation can sometimes provide a clue to which condition is present; lymphocytic thyroiditis is typically characterized by predominant increases in T4 levels. Fractional uptake of radioiodine is either absent or very low in the setting of lymphocytic thyroiditis, whereas it is increased in active Graves disease (see Table 226-6).

TREATMENT  Lymphocytic thyroiditis can often be managed with reassurance and observation alone. Symptomatic thyrotoxicosis can be treated with a course of β-blocker therapy. Overt hypothyroidism may require short-term thyroxine replacement.

PROGNOSIS

Most patients with lymphocytic thyroiditis eventually return to a euthyroid state, but 25% develop persistent hypothyroidism due to classic autoimmune thyroiditis.

Acute (Suppurative) Thyroiditis Infection of the thyroid gland is a rare condition that typically presents with severe thyroid pain, fever, and other systemic manifestations of infection. Bacterial infection of thyroid tissue can be the result of direct spread of grampositive or gram-negative pathogens through fistulas communicating with the piriform sinus or the skin. Hematogenous spread of bacterial, mycobacterial, fungal, or parasitic organisms, especially Pneumocystis carinii, can occur in immunocompromised individuals. On examination, affected patients are typically febrile, with asymmetrical swelling of a thyroid that is tender, warm, and fluctuant to firm in consistency beneath erythematous skin. Ultrasonography may reveal an abscess that can be aspirated to identify a pathogen. Patients with suppurative thyroiditis require prompt treatment with appropriate antibiotics. Surgical drainage of abscesses may be required.

Other Forms of Thyroiditis Certain drugs can cause thyroid gland inflammation. Amiodarone can produce a painless thyroiditis associated with thyrotoxicosis. Whenever possible, this should be distinguished from the iodine-induced form of thyrotoxicosis that can also be associated with amiodarone therapy. The former is optimally treated with glucocorticoid therapy, whereas the latter is managed with antithyroid drugs.12 Interferon-α can provoke a painless thyroiditis associated with transient thyrotoxicosis. This must be differentiated from interferon-α–induced Graves disease; the former is managed with β-blockers and the latter with antithyroid drugs. Riedel’s thyroiditis or struma is characterized by fibrotic replacement of the thyroid, with adherence and infiltration of adjacent structures that causes local compressive symptoms. In this idiopathic condition, the thyroid is substantially enlarged, hardened, and fixed. Affected patients may also develop mediastinal and retroperitoneal fibrosis, sclerosing cholangitis, or orbital pseudotumor. Diagnosis requires open biopsy. Surgical excision is difficult or impossible. Glucocorticoid therapy and tamoxifen therapy have been anecdotally reported to be effective.

1510

CHAPTER 226  Thyroid  

  GOITER DEFINITION

Goiters can be classified as diffuse or nodular, nontoxic or toxic (i.e., associated with thyroid hormone overproduction), and benign or malignant. Thyroid enlargement can be the result of thyrocyte proliferation stimulated by circulating factors (e.g., TSH and thyroid-stimulating autoantibodies), infiltration of the gland by inflammatory or malignant cells, or benign or malignant neoplastic changes within the gland itself. In a patient with a goiter, three clinical issues must be considered: enlargement causing local compressive or cosmetic concern, gland hyperfunction or hypofunction, and potential malignancy.

EPIDEMIOLOGY

Dietary iodine deficiency represents the most common cause of goiter worldwide. It is encountered in the United States only among immigrants from iodine-deficient regions. Younger patients present with diffuse or simple goiters that shrink in response to adequate iodine supplementation. In older individuals, iodine-deficient goiters become multinodular and do not decrease in size with iodine repletion. Excessive iodine exposure can provoke thyrotoxicosis in these patients.

PATHOBIOLOGY

Benign multinodular goiter or adenoma can be the result of genetic defects that lead to dyshormonogenesis, including mutations in the thyroglobulin, thyroid peroxidase, dual oxidase, and pendrin genes. Similarly, exposure to goitrogenic substances in foodstuffs, water, or drugs (e.g., lithium carbonate) that inhibit the normal steps in thyroid hormone synthesis can lead to goiter. In most patients, the underlying cause is unknown. Autoimmune thyroiditis typically produces a modest goiter as a result of glandular infiltration with lymphocytes, inflammatory changes in thyrocytes, and fibrosis. The hypothyroid state caused by autoimmune thyroiditis results in increased TSH, which further stimulates thyroid enlargement. Graves disease is also characterized by diffuse thyroid enlargement due to the action of thyroid-stimulating immunoglobulins. Other forms of thyroiditis can present with goitrous enlargement of the thyroid gland, including subacute, lymphocytic, and acute (suppurative) thyroiditis (see earlier sections). Malignant neoplasms that involve the gland diffusely, including thyroid lymphoma and infiltrative papillary, medullary, and anaplastic thyroid cancers, may present as rapidly enlarging goiters (see later sections). Affected patients often experience local pain and symptoms related to tumor expansion.

DIAGNOSIS

Laboratory Findings

A TSH level determines whether there is primary hypothyroidism or thyrotoxicosis. Elevated antithyroid peroxidase antibody titers can confirm suspected autoimmune thyroiditis. In asymptomatic patients with a modest diffuse goiter, no further evaluation may be indicated. Other blood tests (e.g., ESR for subacute thyroiditis or calcitonin for medullary thyroid cancer) can be useful when clinical clues suggest specific diagnoses.

Imaging

Cervical ultrasonography is the best imaging technique to define the character and extent of a goiter limited to the neck. It can help determine whether a goiter is diffuse or nodular, whether the thyroid is impinging on other cervical structures, and whether lymphadenopathy is present. Ultrasonography is also essential for guidance of fine-needle aspiration for cytologic differential diagnosis (see later). When a goiter extends posteriorly or beneath the sternal notch into the thorax, CT or MRI may be required. The administration of iodine-containing radiocontrast dye should generally be avoided in the evaluation of patients with goiters, because the stable iodide load may interfere with subsequent radioiodine imaging or therapy. Thyroid radionuclide uptake studies with 99mTc pertechnetate or 123I can help characterize the functional status of the gland. Radionuclide scanning can help determine the cause of a goiter and whether a superior mediastinal mass is thyroid tissue. Barium swallow radiographs with fixed-diameter markers and pulmonary function testing with flow-volume loops can help determine whether symptoms are directly related to compression of the esophagus or trachea, respectively. Laryngoscopy is useful to evaluate vocal cord function in patients with potential recurrent laryngeal nerve involvement.

TREATMENT  Once thyroid dysfunction and malignant disease have been excluded, asymptomatic patients with goiters can be observed with periodic clinical assessment. Ultrasonography can be relied on as a reproducible technique for monitoring the size of an enlarged thyroid gland. Thyroxine therapy to suppress TSH levels is effective in shrinking goiters in only a minority of patients. Furthermore, chronic thyroid hormone treatment carries the risks of symptomatic thyrotoxicosis, atrial fibrillation, and bone mineral loss. Patients with benign multinodular goiters causing local compressive symptoms or cosmetic concerns can be treated with surgery or radioactive iodine therapy. Surgery is often preferred when a patient has substantial gland enlargement causing compressive complications, especially when there is substernal extension of the goiter or acute obstructive symptoms. When surgery is contraindicated by the patient’s health status, radioactive iodine therapy has been shown to reduce goiter size by an average of 50% over 12 to 24 months.

Clinical Examination

The first step in evaluating a suspected goiter is to confirm whether neck swelling represents enlargement of the thyroid. Redundant skin and subcutaneous fat in the lower anterior neck can be mistaken for an enlarged thyroid. These findings can usually be distinguished from true thyroid enlargement by palpating a normal thyroid beneath the misleading soft tissue and by observing that the fullness does not rise and fall with deglutition. Ultrasonography may help resolve uncertainty. A patient’s history can provide important clues to the underlying cause. A childhood social history may confirm previous iodine deficiency. Symptoms of hypothyroidism may suggest autoimmune thyroiditis, whereas clinical evidence of thyrotoxicosis may suggest Graves disease or toxic multinodular goiter. Clinical findings may lead to recognition of one of the various forms of thyroiditis (e.g., pain in subacute thyroiditis or postpartum status in lymphocytic thyroiditis). Symptoms suggesting the invasion of adjacent structures may raise concerns about malignant disease or Riedel’s thyroiditis. On examination, diffuse enlargement favors one of the forms of thyroiditis, Graves disease, or a diffusely infiltrating malignant neoplasm. Nodular enlargement is more likely to reflect a benign multinodular goiter or malignant neoplasm. The precise size of the gland should be documented. Dysphonia, tracheal deviation, cervical lymphadenopathy, and venous engorgement in the neck should be noted. Subtotal obstruction of the thoracic outlet may be revealed by having the patient touch his or her hands together above the head (Pemberton’s maneuver) while checking for signs of facial plethora and cervical venous distention.

  THYROID NODULES EPIDEMIOLOGY

Thyroid nodules are common, being detected by palpation in 6% of women and 2% of men. Contemporary high-resolution ultrasonography identifies thyroid nodules in as many as 50% of all adults. Although the majority of these represent small, benign adenomatoid nodules or cysts, 5 to 10% of thyroid nodules are malignant. Less commonly, thyroid nodules are clinical problems by virtue of being hyperfunctioning or causing local compressive symptoms or cosmetic dissatisfaction.

DIAGNOSIS

Thyroid nodules can be noted by the patient or their physician in the absence of any other complaints.13 It is also common for thyroid nodules to be detected incidentally on imaging procedures, such as carotid ultrasonography and cervical spine CT or MRI. Symptoms of compression or invasion of adjacent tissues suggest that a nodule may be malignant. These include pain in the lower anterior neck, cough or dyspnea due to tracheal compression, hemoptysis due to tracheal invasion, dysphonia due to recurrent laryngeal nerve encasement, and dysphagia or odynophagia due to esophageal compression. Certain other symptoms and signs lead to the consideration of specific underlying conditions. A toxic adenoma should be suspected in a patient with a thyroid nodule and the classic clinical manifestations of thyrotoxicosis. Hypothyroid symptoms and signs suggest autoimmune thyroiditis

CHAPTER 226  Thyroid  

with asymmetrical thyroid enlargement. Hypercalcitoninemia associated with the metastatic spread of medullary thyroid cancer can cause pruritus, flushing, and diarrhea. The clinical assessment should also include symptoms and signs related to common sites of thyroid cancer metastasis, such as chest pain, dyspnea, bone pain, and neurologic findings. Thyroid nodules rarely can be due to metastasis from other primary malignant neoplasms including kidney, colon, and breast cancers.

History

A special predisposition to thyroid cancer is suggested by a personal history of therapeutic neck irradiation in childhood. Family history can be informative if relatives have had medullary or papillary thyroid cancers, which are familial in 50% and 10% of cases, respectively. The possibility of medullary thyroid cancer should also be considered when there is a personal or family history of clinical problems associated with multiple endocrine neoplasia type 2 (MEN 2) syndromes, including hyperparathyroidism and pheochromocytoma (Chapter 231).

Physical Examination

Physical examination of a thyroid nodule should seek to define its size, consistency, surface texture, mobility, and tenderness. The presence of malignant disease is suggested by fixation and ipsilateral regional adenopathy or vocal cord paresis. Multinodularity of the gland may reflect benign nodular goiter, but it is not sufficiently reassuring to dispense with further diagnostic testing.14 This is particularly true for a so-called dominant nodule that is larger, enlarging faster, or more symptomatic than others present in the thyroid.

Laboratory Findings

Routine laboratory testing includes measurement of TSH levels to identify patients with hyperthyroidism or hypothyroidism. When the TSH level is low or undetectable, the possibility of a benign autonomously functioning toxic adenoma can be pursued with radionuclide thyroid scanning (see Table 226-6). If an elevated TSH level indicates primary hypothyroidism, antithyroid peroxidase antibody titers can confirm whether the patient has autoimmune thyroiditis. Ultrasonography can distinguish asymmetrical enlargement caused by autoimmune thyroiditis from a discrete nodule. Calcitonin levels should be measured in patients with a known or suspected family history of MEN 2 or familial medullary thyroid cancer. Serum thyroglobulin measurement is not helpful in distinguishing benign from malignant thyroid abnormalities.

Imaging

Cervical ultrasonography helps confirm that a mass is within the thyroid, accurately defines its size, classifies it as cystic or solid, and determines whether additional nodules are present. Ultrasonography occasionally reveals other suspicious findings in nodules, such as fine calcifications, irregular nodule borders, and cervical adenopathy. Radionuclide scanning with radioiodine or technetium pertechnetate is helpful only in selected cases. In patients with a thyroid nodule and a suppressed TSH level, scanning can confirm that the nodule is hyperfunctioning or “hot,” in which case biopsy is usually not required.

Invasive Evaluation Fine-Needle Aspiration Biopsy

Fine-needle aspiration biopsy is the most accurate test to exclude or confirm malignant disease in patients with a nodule and a normal TSH level (Fig. 226-4). Most solid nodules and complex cysts larger than 1.0 to 1.5 cm in diameter should be sampled. Although aspiration can be directed by palpation alone when a nodule is readily definable, ultrasonography provides more certain guidance for the sampling of poorly localized lesions, often revealing additional nodules that should be assessed. The cytologic assessment of aspirated material must first confirm that there is adequate material for assessment (e.g., 6 clumps of 10 cells on 2 slides). Biopsies with inadequate specimens, which are more common in cystic lesions, must be repeated. Ultrasonographic guidance and on-site preliminary cytologic assessment can improve the yield of biopsy. In accordance with the Bethesda System for Reporting Thyroid Cytopathology, a sampled nodule can be categorized as benign, atypical, suspicious for a follicular neoplasm, suspicious for malignancy, or malignant (Table 226-7).15 Benign nodules typically yield samples containing clusters of normalappearing follicular epithelial cells with colloid. Pure colloid cysts may have

1511

scant epithelium. This classification is highly accurate, with a false-negative rate of less than 3% in sonographically directed biopsy specimens, and surgical resection is not required. In most cases, conservative observation based on yearly clinical or sonographic reassessment can be recommended. Further enlargement during observation (i.e., >20% increase in two of three dimensions) should prompt a repeat biopsy. Surgical resection should be considered if a cytologically benign nodule continues to grow, causing compressive symptoms or cosmetic disfigurement. Cytologic material classified as malignant typically contains abundant epithelial cells with atypical nuclear features, overlapping, and scant or absent colloid. This is also a highly reliable finding, with 98% of such lesions found to be thyroid cancers on subsequent resection. Consequently, bilateral thyroidectomy is indicated in patients without contraindications to operation. Samples that contain sparser quantities of epithelial cells with similar atypical nuclear features may be classified as suspicious for malignancy. Approximately 75% of nodules in this category represent thyroid cancers. One in five biopsies yields adequate but diagnostically indeterminate cytologic material.16 Specific findings that classify an indeterminate nodule as suspicious for a follicular neoplasm include abundant follicular or Hürthle cells in microfollicles with little or no colloid and minor degrees of nuclear atypia, potentially indicative of papillary cancer. Although the majority of such indeterminate nodules are benign follicular adenomas, 15 to 30% are thyroid carcinomas. Biopsy samples that reveal nuclear or architectural features considered to be abnormal but not clearly suspicious for malignancy or a follicular neoplasm are classified as demonstrating atypia of undetermined significance. Nodules that initially fall into this category have been estimated to harbor malignancy at rates ranging from 5 to 25%. Repeat sampling may provide a more specific diagnosis to guide further management in 75% of cases. Definitive determination of whether a suspicious or atypical nodule represents a focus of malignancy requires surgery targeted to remove either the lobe of the thyroid containing the nodule or the entire gland for surgical pathologic examination. Unilateral thyroid lobectomy has the advantage of a lower incidence of surgical complications and postoperative hypothyroidism when the lesion is benign, but it necessitates a subsequent completion thyroidectomy for most patients who prove to have cancer. Molecular diagnostic testing is available to reduce the number of surgeries performed in patients with cytologically indeterminate nodules, approximately 75% of which prove to be histopathologically benign. There are two general strategies: (1) testing aspirated material for oncogenic mutations associated with thyroid malignancies and (2) gene expression classifier microarrays designed to identify benign nodules. For a typical population of cytologically indeterminate nodule patients with a prevalence of thyroid cancer of 20 to 35%, the negative predictive value of oncogenic testing and gene expression classification have been shown to be approximately 85% and 95%, respectively. For patients with no clinical features of malignancy, particularly middle-aged or older women with multinodular glands in whom the prevalence of malignancy is 5% or less, vigilant observation with serial sonography is an alternative.

  THYROID CANCER

Cancers of the thyroid gland have a spectrum of behavior that ranges from incidentally detected and clinically inconsequential microcarcinomas to aggressive and virtually untreatable anaplastic malignant neoplasms. When thyroid cancer is diagnosed early, treatment is effective for most types. Most thyroid cancers present as thyroid nodules that are either asymptomatic or associated with local cervical symptoms or adenopathy. Less often, thyroid cancers first present with manifestations of metastatic disease, such as a pulmonary mass or bone pain.

Papillary and Follicular (Epithelial) Thyroid Carcinomas Papillary and follicular thyroid cancers arise from follicular epithelium and often retain responsiveness to TSH, produce thyroglobulin, and concentrate iodide. They are distinguished by their histopathologic appearances and characteristic patterns of progression. Hürthle cell carcinoma of the thyroid is composed of thyrocytes with abundant mitochondria-laden cytoplasm, and behaves like a follicular thyroid cancer, although it typically does not have iodine-concentrating ability.

EPIDEMIOLOGY

Approximately 60,000 new cases of thyroid cancer are diagnosed annually in the United States. Thyroid cancer is three times more common in women, in

1512

CHAPTER 226  Thyroid  

Thyroid nodule

Check TSH

TSH

Normal TSH

TSH

Check thyroid scan

Greater than 1.0–1.5 cm in maximal diameter

Toxic adenoma

Fine-needle aspiration biopsy

Check antithyroid antibody titers

Autoimmune thyroiditis Consider ultrasonography to distinguish between unilateral enlargement and a discrete nodule

Radioiodine Surgery

Thyroid hormone replacement

Benign No specific therapy Consider surgery if enlargement leads to compressive symptoms or cosmetic concerns

Papillary thyroid carcinoma Surgery

Malignant Suspicious for malignancy

Medullary thyroid carcinoma

Atypia of undetermined significance or follicular neoplasm Consider surgery for definitive diagnosis Molecular genetic testing Monitor size

Thyroid lymphoma External beam radiation Chemotherapy

Surgery

Nondiagnostic Unsatisfactory Atypia of undetermined significance Ultrasound-guided fine needle aspiration biopsy

Anaplastic thyroid carcinoma External beam radiation Chemotherapy

FIGURE 226-4.  Evaluation of a thyroid nodule. TSH = thyroid-stimulating hormone.

TABLE 226-7  BETHESDA SYSTEM FOR REPORTING CYTOPATHOLOGY CYTOLOGIC DIAGNOSIS Benign

RISK OF MALIGNANCY 0-3%

Atypia of undetermined significance

20-25%

Suspicious for a follicular neoplasm

15-30%

Suspicious for malignancy

60-77%

Malignant

97-99%

Diagnostic categories associated with risk of malignancy. Adapted from Bongiovanny M, Spitale A, Faquin WC, et al. The Bethesda System for Reporting Thyroid Cytopathology: a meta-analysis. Acta Cytol. 2012;56:333-339.

whom its incidence is currently rising faster than that of any other malignancy. There are estimated to be 450,000 U.S. thyroid cancer survivors who require lifelong follow-up for recurrence. Papillary thyroid carcinoma is the most common form of thyroid cancer, representing 90% of cases. The mean age at diagnosis is 45 years, but papillary thyroid carcinoma does occur in children and increases in incidence with age.

PATHOBIOLOGY

Irradiation of the thyroid gland in childhood is a risk factor, as evidenced by the epidemics of thyroid cancer that have followed both external beam radiation therapy for benign childhood conditions (e.g., tonsillitis and acne) and

radioiodine exposure after nuclear incidents. A substantial body of evidence now implicates RET/PTC and BRAF gene mutations that activate the MAP kinase signaling pathway in the pathogenesis and progression of papillary thyroid cancer. Most papillary thyroid carcinomas are slow growing and either remain confined to the gland or metastasize to cervical lymph nodes. Papillary microcarcinomas are a common incidental pathologic finding in 5% of thyroid glands excised for other reasons. However, papillary thyroid carcinomas can be more aggressive, with extension into adjacent tissues, extensive nodal involvement, and distant metastatic spread, most commonly to the lungs. Such aggressive behavior is, in general, more common in older patients. Follicular and Hürthle cell thyroid carcinomas account for 9% of all thyroid cancers. When these tumors show histologic evidence of invading only the tumor capsule, they are termed minimally invasive and generally behave like papillary thyroid carcinomas. However, follicular and Hürthle cell carcinomas with vascular invasion are more likely to be associated with distant metastatic disease, which most commonly involves the lungs and skeleton.

TREATMENT  Treatment of epithelial thyroid cancer entails surgery, often followed by radioiodine ablation of remnant thyroid tissue. A5  Total or near-total thyroidectomy with selective central compartment lymph node resection is usually the appropriate initial surgical procedure. Thyroid surgery can be complicated by hypoparathyroidism or recurrent laryngeal nerve injury, which causes hoarseness if it is unilateral and airway obstruction if it is bilateral. The rationale for bilateral surgery is the frequent presence of bilateral disease in papillary

CHAPTER 226  Thyroid  

thyroid cancer and the lower risk of recurrence after bilateral gland removal. In addition, there is greater accuracy in detecting residual disease after the eradication of all remaining normal thyroid tissue. A prospective cohort study of a national database of 30-day follow-up of patients undergoing thyroidectomy (for cancer or other indications) documented the increased risk of major pulmonary, cardiac, and infectious complications in the elderly. Elderly patients (65 to 79 years old) are twice as likely, and the most elderly (80 years or older) are 5 times as likely as young patients (16 to 64 years old) to have major systematic complications.17

Follow-up

131

Postoperatively, I administration after TSH stimulation can be employed to ablate the small amount of normal thyroid tissue that usually remains after surgery. This tissue, if it is not destroyed, leaves patients with circulating thyroglobulin and iodine-concentrating tissue on whole body scanning, decreasing the accuracy of these tests to identify residual disease. In controlled but nonrandomized trials, radioiodine has been associated with a lower rate of tumor recurrence in patients with advanced disease (stages 3 and 4) at presentation (see later), but there is no demonstrated clinical benefit of adjunctive radioiodine therapy for patients with lower stages of disease. TSH stimulation of residual thyroid tissue, which is essential for effective radioiodine therapy, can be accomplished either by the temporary withdrawal of thyroid hormone therapy to promote endogenous TSH production or by the administration of recombinant thyrotropin, which avoids the morbidity of hypothyroidism. Thyroxine therapy is appropriate for all patients with treated thyroid cancer, regardless of the extent of surgery and whether they received radioiodine ablative therapy. In addition to providing thyroid hormone replacement, thyroxine can be adjusted to suppress the patient’s circulating TSH level to the low or low-normal range to reduce the likelihood of tumor recurrence. In determining the extent to which the TSH level should be suppressed, the patient’s risk of cancer recurrence must be balanced against potential thyrotoxic complications such as bone mineral loss in postmenopausal women and atrial fibrillation in older patients. Long-term monitoring of patients entails periodic clinical assessment, measurement of serum thyroglobulin levels, radioiodine imaging in the early postoperative phase, and occasional use of ultrasonography. Clinically, patients should be assessed for local neck symptoms or recurrent cervical masses, as well as for optimization of thyroid hormone therapy. For patients with treated epithelial thyroid cancers, thyroglobulin is a more specific tumor marker if all remaining normal thyroid tissue has been ablated. For patients with undetectable thyroglobulin levels on TSH-suppressive thyroid hormone therapy, thyroglobulin measurement after recombinant TSH stimulation can sometimes reveal residual disease. Radioiodine scanning after TSH stimulation can be helpful in patients who have previously undergone radioiodine ablation, but once radioiodine imaging is negative, it offers little or no advantage over measurement of stimulated thyroglobulin levels. This is particularly true in recurrent papillary thyroid cancers, which often lose the ability to concentrate iodine. Unfortunately, thyroglobulin testing is impossible in the 20% of patients who have circulating thyroglobulin autoantibodies that interfere with thyroglobulin immunoassays. Because most epithelial thyroid cancer recurrences are in cervical nodes or soft tissues, ultrasonography is useful for postoperative monitoring, particularly in patients who presented with extensive cervical disease or who have persistently detectable serum thyroglobulin. CT scanning of the chest should be employed to detect intrathoracic disease in patients whose findings suggest recurrence outside the neck. In patients with substantial detectable thyroglobulin levels (>10 ng/mL) and negative findings on standard imaging studies, PET scanning can identify sites of residual disease in more than 50% of patients. Localization of recurrent cervical disease is usually an indication for comprehensive compartmental neck dissection. Distant and nonresectable metastases that are iodine avid, which occur more commonly in patients with invasive follicular thyroid cancer, can be treated with repeated doses of 131I. Symptomatic hilar node and bone metastases can be treated palliatively with external beam radiation therapy. Surgery can be employed for isolated metastatic disease sites. Conventional chemotherapy has limited efficacy in the treatment of differentiated thyroid cancer, but newer biologic agents targeting the molecular pathways involved in the pathogenesis of thyroid cancer hold promise.18 For example, the multikinase inhibitor sorafenib was shown in a phase 3 trial to double progression free survival to almost 11 months in patients with metastatic non–iodine avid epithelial thyroid cancers and shrink disease sites in 12% of patients. A6  Another multikinase inhibitor, vandetanib, is also effective against locally advanced or metastatic differentiated thyroid cancer. A7  However, multikinase inhibitors commonly have adverse effects, and because they are only tumoristatic must be used continuously.

PROGNOSIS

The TNM (tumor, node, metastasis) staging system is commonly used to stage epithelial thyroid cancers. In addition to tumor size, extent of node

1513

involvement, and presence of distant metastatic disease, the age of the patient at presentation is an important predictor of outcome. Patients younger than 45 years have a better prognosis than older individuals. The overall age-adjusted 10-year survival rates for patients with papillary and follicular thyroid cancer are 98% and 92%, respectively. However, disease recurrence is relatively common, occurring in approximately one third of patients with papillary thyroid cancer. Consequently, patients with treated thyroid cancer must be monitored for recurrent disease.

Medullary Thyroid Carcinoma Patients with medullary thyroid cancer (Chapter 246) typically present with a thyroid nodule, cervical adenopathy, distant disease, or symptoms of flushing, diarrhea, and pruritus when the circulating calcitonin level is markedly elevated. Features of the other elements of MEN 2a (e.g., hypertension) or MEN 2b (e.g., marfanoid habitus, submucosal neuromas) should be sought.

Anaplastic Thyroid Carcinoma Anaplastic thyroid carcinoma is a rare, histologically undifferentiated, clinically aggressive malignant neoplasm that typically arises in older patients, one fourth of whom present with evidence of a preceding differentiated thyroid cancer. Affected patients present with a rapidly enlarging mass in the anterior or lateral neck associated with pain, tenderness, and compressive symptoms including dysphagia, dysphonia, and stridorous dyspnea. Fineneedle aspiration biopsy of the mass usually yields large, pleomorphic, undifferentiated cells, but open surgical biopsy is sometimes required to confirm the diagnosis. Most cases are unresectable at presentation because of invasion of cervical structures. Surgery is not curative and should aim to secure the patient’s airway. A percutaneous gastrostomy tube is often placed to ensure adequate nutrition in the face of esophageal impingement. Conventional therapy consisting of combined external beam radiation therapy and chemotherapy with doxorubicin with or without cisplatin produces an initial response in 25% of patients. Rare patients with disease limited to the neck may have extended survival, but almost all patients relapse within a few months and succumb to their disease, with median survival ranging from 3 to 7 months. Current research is focused on the use of targeted antiangiogenic agents to treat unresponsive disease.

Thyroid Lymphoma Lymphoma rarely arises in the thyroid gland, typically presenting in older persons as a rapidly enlarging and painful diffuse goiter. Patients often have a preceding history of autoimmune thyroiditis. The diagnosis is further suspected when fine-needle aspiration biopsy yields abundant lymphocytes without other cellular features of autoimmune thyroiditis. Immunohistochemical staining and flow cytometry of sampled material can characterize a monoclonal lymphocyte population. Surgical biopsy is sometimes required to establish the diagnosis. In 50% of cases, lymphoma is primary to the thyroid gland, and it is usually an intermediate-grade non-Hodgkin’s–type lymphoma (Chapter 185). Surgical resection of the thyroid is usually not indicated, but elective tracheostomy may be required if tracheal compression is imminent. Most patients respond to treatment with combined external beam radiation therapy and chemotherapy. Disease-free survival rates vary with the disease stage at diagnosis and the initial response to combination therapy.

Grade A References A1. McDermott MT. Does combination T4 and T3 therapy make sense? Endocr Pract. 2012;18: 750-757. A2. Marcocci C, Kahaly GJ, Krassas GE, et al. Selenium and the course of mild Graves’ orbitopathy. N Engl J Med. 2011;364:1920-1931. A3. Bahn Chair RS, Burch HB, Cooper DS, et al. Hyperthyroidism and other causes of thyrotoxicosis: management guidelines of the American Thyroid Association and American Association of Clinical Endocrinologists. Thyroid. 2011;21:593-646. A4. Stagnaro-Green A, Abalovich M, Alexander E, et al. Guidelines of the American Thyroid Association for the diagnosis and management of thyroid disease during pregnancy and postpartum. Thyroid. 2011;21:1081-1125. A5. Smallridge RC, Ain KB, Asa SL, et al. American Thyroid Association guidelines for management of patients with anaplastic thyroid cancer. Thyroid. 2012;22:1104-1139. A6. Brose MS, Nutting CM, Jarzab B, et al. Sorafenib in radioactive iodine-refractory, locally advanced or metastatic differentiated thyroid cancer: a randomised, double-blind, phase 3 trial. Lancet. 2014; 384:319-328.

1514

CHAPTER 227  Adrenal Cortex  

A7. Leboulleux S, Bastholt L, Krause T, et  al. Vandetanib in locally advanced or metastatic differentiated thyroid cancer: a randomised, double-blind, phase 2 trial. Lancet Oncol. 2012;13: 897-905.

GENERAL REFERENCES For the General References and other additional features, please visit Expert Consult at https://expertconsult.inkling.com.

227  ADRENAL CORTEX LYNNETTE K. NIEMAN The adrenal glands weigh 6 to 8 g in adults (Fig. 227-1). Each contains a cortex, which makes steroid hormones, and a medulla, which produces catecholamines. Diseases of the adrenal medulla are discussed in Chapter 228. In the adrenal cortex, production of the three major classes of steroids occurs in specific zones: the outermost layer, the glomerulosa, produces mineralocorticoids, primarily aldosterone; the middle layer, the fasciculata, produces glucocorticoids, primarily cortisol; the innermost layer, the reticularis, produces adrenal “androgens,” primarily dehydroepiandrosterone (DHEA) and its sulfated conjugate (DHEA-S) (Fig. 227-2). This division reflects the fact that certain critical enzymes are restricted to specific zones, resulting in the ability or inability to synthesize specific end products.

  FUNCTION

The actions and regulation of these steroid classes differ. Mineralocorticoids act through the renal mineralocorticoid receptor to promote the reabsorption of sodium and the secretion of potassium. In addition to this classic action, mineralocorticoids have important action on the vasculature and may exacerbate the metabolic syndrome.1 Aldosterone secretion is stimulated primarily by hyperkalemia and angiotensin II (which itself is stimulated by hypovolemia and excess renin). These agents increase the production of aldosterone synthase to restore homeostasis through this feedback loop. Aldosterone production is stimulated to a much smaller degree by adrenocorticotropic hormone (ACTH). Cortisol and other glucocorticoids act through the glucocorticoid receptor type 2 and its isoforms. The actions of this class of steroids are much broader, including effects on carbohydrate handling, lipid and calcium metabolism, and the immune and nervous systems. Cortisol production is regulated primarily by ACTH, which is secreted in a circadian rhythm in response to corticotropin-releasing hormone (CRH) so that cortisol levels are highest in the morning and fall to a nadir around midnight. Cortisol coordinates ACTH production through negative feedback at the pituitary (ACTH) and hypothalamus (CRH). Vasopressin secretion also plays a role in stimulating ACTH release. DHEA and DHEA-S are the most abundant products of the adrenal gland. They exert their estrogenic and androgenic effects as prohormones, being converted to estrogens and testosterone in the peripheral tissues and activating the androgen and estrogen receptors. There is no known regulator of DHEA synthesis, but its production declines with age.

  DISORDERS OF ADRENAL FUNCTION

Most disorders of the adrenal cortex reflect overproduction or underproduction of the products of a single synthetic zone—cortisol, aldosterone, or testosterone or estrogen (Fig. 227-3). The congenital adrenal hyperplasias are an exception and are manifested with both overproduction and underproduction. Abnormal secretion is suggested by clinical features of each disorder and is reflected in plasma or urine levels of the relevant hormones or by the consequent increases or decreases in feedback systems, which form the basis of the biochemical diagnostic tests.

Glucocorticoid Excess: Cushing Syndrome

CLINICAL MANIFESTATIONS

FIGURE 227-1.  Magnetic resonance images of the abdomen showing the position and relative size of the normal adrenal glands.

Capsule Zona glomerulosa

Zona fasciculata

Zona reticularis Medulla

Cushing syndrome is a symptom complex that reflects excessive tissue exposure to cortisol. Classic features of Cushing syndrome include weight gain, plethora, hypertension, and striae (Table 227-1). Not all patients have all features; the number and severity of features correlate roughly with the duration and severity of hypercortisolism. Because many of the signs and symptoms are nonspecific, the diagnosis may be confused with psychiatric disorders, polycystic ovary syndrome, the metabolic syndrome, simple obesity, fibromyalgia, or acute illness. However, because worsening hyper­ cortisolism may precipitate hypertension, glucose intolerance, infections, psychiatric disturbances, impaired cognition, and hypercoagulability, it is important to identify this treatable disorder to prevent its associated morbidity and mortality.2 Changes in mood and cognition are useful markers of hypercortisolism. These include irritability, crying, and restlessness; depressed mood; decreased libido; insomnia; anxiety; and decreased concentration and impaired memory.

DIAGNOSIS

Clinical Examination

Cushing syndrome screening is most likely to be positive in the presence of signs that are typical of glucocorticoid excess, such as abnormal fat distribution in the supraclavicular and temporal fossae, proximal muscle weakness, wide (>1 cm) purple striae, and new irritability, decreased cognition, and decreased short-term memory. Testing is indicated when clinical features have progressed over time. For example, oligomenorrhea is more suggestive of Cushing syndrome if a woman previously had regular menses. Serial seven subtractions and recall of three cities (or objects) are useful bedside strategies to identify deficits in cognition and memory.

Laboratory Findings FIGURE 227-2.  Histologic section through a normal adult adrenal gland showing

the progression (from outside to inside) of the zona glomerulosa, zona fasciculata, zona reticularis, and medulla.

Exogenous administration of glucocorticoid should be excluded before screening for endogenous Cushing syndrome. In the absence of pseudoCushing states (see later), at least two different screening test results should

CHAPTER 226  Thyroid  

GENERAL REFERENCES 1. Bremner AP, Feddema P, Leedman PJ, et al. Age-related changes in thyroid function: a longitudinal study of a community-based cohort. J Clin Endocrinol Metab. 2012;97:1554-1562. 2. Caturegli P, De Remigis A, Rose NR. Hashimoto thyroiditis: clinical and diagnostic criteria. Autoimmun Rev. 2014;13:391-397. 3. Garber JR, Cobin RH, Gharib H, et al. Clinical practice guidelines for hypothyroidism in adults: cosponsored by the American Association of Clinical Endocrinologists and the American Thyroid Association. Thyroid. 2012;22:1200-1235. 4. Stagnaro-Green A, Pearce E. Thyroid disorders in pregnancy. Nat Rev Endocrinol. 2012;8:650-658. 5. Grais IM, Sowers JR. Thyroid and the heart. Am J Med. 2014;127:691-698. 6. Rugge JB, Bougatsos C, Chou R. Screening and treatment of thyroid dysfunction: an evidence review for the U.S. Preventive Services Task Force. Ann Intern Med. 2015;162:35-45. 7. Pappa TA, Vagenakis AG, Alevizaki M. The nonthyroidal illness syndrome in the non-critically ill patient. Eur J Clin Invest. 2011;41:212-220. 8. Vaidya B, Pearce SH. Diagnosis and management of thyrotoxicosis. BMJ. 2014;349:g5128. 9. Bahn RS. Graves’ ophthalmopathy. N Engl J Med. 2010;362:726-738.

1514.e1

10. Cooper DS, Laurberg P. Hyperthyroidism in pregnancy. Lancet Diabetes Endocrinol. 2013;1: 238-249. 11. Lee SL. Radioactive iodine therapy. Curr Opin Endocrinol Diabetes Obes. 2012;19:420-428. 12. Bogazzi F, Tomisti L, Bartalena L, et al. Amiodarone and the thyroid: a 2012 update. J Endocrinol Invest. 2012;35:340-348. 13. Niedziela M. Thyroid nodules. Best Pract Res Clin Endocrinol Metab. 2014;28:245-277. 14. Brito JP, Yarur AJ, Prokop LJ, et al. Prevalence of thyroid cancer in multinodular goiter versus single nodule: a systematic review and meta-analysis. Thyroid. 2013;23:449-455. 15. Bongiovanni M, Spitale A, Faquin WC, et al. The Bethesda System for Reporting Thyroid Cytopathology: a meta-analysis. Acta Cytol. 2012;56:333-339. 16. Alexander EK, Kennedy GC, Baloch ZW, et al. Preoperative diagnosis of benign thyroid nodules with indeterminate cytology. N Engl J Med. 2012;367:705-715. 17. Grogan RH, Mitmaker EJ, Hwang J, et al. A population-based prospective cohort study of complications after thyroidectomy in the elderly. J Clin Endocrinol Metab. 2012;97:1645-1653. 18. Marotta V, Sciammarella C, Vitale M, et al. The evolving field of kinase inhibitors in thyroid cancer. Crit Rev Oncol Hematol. 2015;93:60-73.

1514.e2

CHAPTER 226  Thyroid  

REVIEW QUESTIONS 1. A 74-year-old man with a history of hypertension and hypercholesterolemia is hospitalized after presenting with a 3-month history of progressive fatigue and dyspnea on exertion. His weight on admission is 164 lbs, his pulse is 44 bpm, and lab tests show CPK 528 U/L, TSH 65 mU/L, free T4 0.2 ng/dL, and T4 2.3 µg/dL. A pharmacologic nuclear stress test reveals findings consistent with diffuse myocardial ischemia. Subsequent coronary angiography reveals diffuse three-vessel disease that is not amenable to percutaneous stenting. A consulting cardiologist has recommended that he undergo coronary artery bypass surgery. What would you recommend? A. Start levothyroxine at a dose of 125 µg daily. B. Administer a 60-µg intravenous dose of levothyroxine daily. C. Check antithyroid peroxidase and antithyroglobulin antibodies. D. Start levothyroxine at a dose of 12.5 µg daily. Answer: D  Start levothyroxine at a dose of 12.5 µg daily. This patient requires treatment with levothyroxine, but it should be started at the lowest possible dose, with provisions to gradually increase it as tolerated in light of his coronary artery disease. A full replacement dose given orally or an adjusted dose given intravenously could exacerbate cardiac ischemia to the point of causing an infarction. Checking antithyroid peroxidase and antithyroglobulin antibodies would not provide any additional information. In the absence of any known history of thyroid surgery or external radiation treatment to the head and neck, it can be presumed that his severe hypothyroidism is due to autoimmune thyroiditis. 2. A 28-year-old woman with a 6-year history of hypothyroidism has been treated with levothyroxine at a dose of 125 µg daily. Lab tests checked 5 months ago right before she stopped taking cyclic oral contraceptives showed TSH 1.3 mU/L. Three weeks ago she checked a home pregnancy test that was positive. She had lab tests checked by her obstetrician that showed hemoglobin 10.2 g/dL, T4 12.5 µg/dL, and T3 185 ng/dL. She is estimated to be at 8 weeks’ gestation and has started taking prenatal vitamins and iron sulfate at a dose of 325 mg twice daily. What should you do? A. Have her continue levothyroxine at a dose of 125 µg daily, with instructions to take it regularly with other medications. B. Increase her dose of levothyroxine to 150 µg daily. C. Decrease her dose of levothyroxine to 112 µg daily. D. Check a thyroid uptake and scan. E. Check a TSH level. Answer: E  Check a TSH level. The only way to determine whether her current dose of levothyroxine is providing an adequate level of replacement is to check a TSH level. Total T4 and T3 levels measured while taking oral contraceptives or during pregnancy may be elevated or high-normal owing to increased production of thyroxine-binding globulin stimulated by increased estrogen levels. As such, they may not correlate with free thyroid hormone levels. Decreasing her dose of levothyroxine would be inappropriate because she may require a moderate to substantial increase in her dose during the course of a pregnancy. Empirically increasing a dose of levothyroxine may cause iatrogenic thyrotoxicosis characterized by symptoms that may be difficult to distinguish from normal physiologic changes of pregnancy. Exposure to radionuclide tracer is contraindicated during pregnancy, and in any event a thyroid uptake and scan would be unnecessary in a patient with known hypothyroidism. Doses of levothyroxine should always be separated from doses of iron sulfate by at least 4 hours to avoid interactions that can block absorption of both agents.

3. A 76-year-old woman presenting with a 1-week history of a cough, fever, and audible stridor is diagnosed with community-acquired pneumonia. A chest x-ray does not show evidence of an infiltrate but does reveal marked rightward tracheal deviation with a visible mediastinal soft tissue mass. A non-contrast chest computed tomography scan reveals multiple bilateral thyroid nodules, with an 8.5-cm left lower-pole nodule extending below the clavicle and sternum, with compression and narrowing of the trachea. Lab tests show TSH less than 0.001 mU/L, free T4 2.8 ng/dL, and T3 245 ng/dL. A thyroid uptake and scan reveals 24-hour uptake of 37%, with tracer accumulation localized to two right-sided thyroid nodules and the substernal left-sided thyroid nodule. What should you do next? A. Check pulmonary function tests with flow-volume loops. B. Prescribe a 12-month course of methimazole 10 mg daily. C. Refer the patient to a thyroid surgeon. D. Administer a 30-mCi dose of I-131. E. Start levothyroxine at a dose of 137 µg daily. Answer: C  Refer the patient to a thyroid surgeon. A multinodular goiter that has extended substernally to the point of causing tracheal compression should be resected by an experienced thyroid surgeon, irrespective of its functional status. The presence of audible stridor and evidence of tracheal narrowing on radiographic images obviates the need for pulmonary function testing with flow-volume loops. Treatment with methimazole might help control hyperthyroidism caused by autonomously functioning thyroid nodules but will not be a permanent solution and would not shrink the dominant nodule to any extent. Treatment with I-131 might help shrink the dominant nodule over time but will require 12 to 24 months to be effective. Treatment with levothyroxine to try to suppress further enlargement of thyroid tissue may be marginally effective at best in euthyroid patients and would be completely ineffective—and potentially dangerous—in an elderly woman presenting with hyperthyroidism. 4. A 33-year-old man is noted to have palpable enlargement of the right side of his thyroid on a routine physical exam. A thyroid ultrasound reveals a solitary 3.1-cm right-sided nodule with smooth borders. Lab tests show TSH 0.1 mU/L and T4 11.5 µg/dL. He reports a history of occasional symptomatic palpitations and weight loss of 5 lbs over the course of 3 months despite an increase in his appetite. He is not taking any medications and has not noted any problems with dysphagia or dysphonia. What should you do next? A. Perform a fine-needle aspiration biopsy of the right-sided nodule. B. Administer a 15-mCi dose of I-131. C. Refer the patient to a thyroid surgeon. D. Start methimazole at a dose of 5 mg daily. E. Perform a 123-I thyroid scan. Answer: E  Perform a 123-I thyroid scan. When a patient presenting with a thyroid nodule who is not taking levothyroxine is noted to have a suppressed TSH level, a radionuclide thyroid scan should be checked to determine if the nodule is an autonomously functioning toxic adenoma. If a nodule is “hot” on the scan, it does not need to be biopsied. If it is “cold” on the scan, with evidence of increased tracer uptake in surrounding tissue consistent with Graves disease, fine-needle aspiration biopsy should be performed to confirm it is benign. Treatment of a toxic adenoma or Graves disease with methimazole or radioactive iodine may be indicated but should only be considered after it has been determined whether a biopsy is necessary. Referral for thyroid surgery would only be indicated if a biopsy of a cold nodule revealed suspicious or malignant cytopathology, or if there were contraindications to treatment of a toxic adenoma with radioactive iodine or methimazole.

CHAPTER 226  Thyroid  

5. A 55-year-old woman is noted to be tachycardic with a resting pulse of 104 during a routine physical exam. Lab tests checked to evaluate this show TSH 0.002 mU/L, with follow-up lab tests showing free T4 2.3 ng/ dL and T3 289 ng/dL. She denies any history of anterior neck discomfort, weight loss, palpitations, anxiety, tremor, heat intolerance, or insomnia. Physical examination reveals tachycardia with a regular rhythm, a slightly enlarged thyroid without any discrete nodularity, and no evidence of proptosis or ocular irritation. She does have a history of osteopenia, with a lumbar spine T-score of −2.3 identified on a DEXA scan checked soon after the onset of menopause. What should you do next? A. Check antithyroid peroxidase and antithyroglobulin antibodies. B. Check a radionuclide thyroid uptake and scan. C. Start methimazole at a dose of 10 mg daily. D. Refer the patient to a thyroid surgeon. E. Check an ESR.

1514.e3

Answer: B  Check a radionuclide thyroid uptake and scan. This patient is presenting with thyrotoxicosis without any referable symptoms or clinical findings suggestive of a specific cause. Checking a thyroid uptake would help distinguish between a high-uptake state caused by hyperthyroidism driven by increased production of thyroid hormone, and a low-uptake state caused by inflammation with leakage of stored thyroid hormone. If a high-uptake state is identified, a scan will help distinguish whether hyperthyroidism is caused by Graves disease, a toxic adenoma, or a toxic multinodular goiter. Elevated antithyroid peroxidase and antithyroglobulin antibodies may identify underlying autoimmune thyroiditis but will not definitively determine the proximate cause of thyrotoxicosis. Treatment with methimazole or referral for thyroid surgery would only be considered after confirmation of a diagnosis. Checking an erythrocyte sedimentation rate (ESR) would not be informative, because subacute thyroiditis would be unlikely in the absence of localized discomfort.

1515

CHAPTER 227  Adrenal Cortex  



CRH

CRH

CRH

CRH

CRH

ACTH

Cortisol ACTH

Cortisol ACTH

+ –

ACTH

Cortisol ACTH

Cortisol

Cortisol

+

A

B

C

D

E

FIGURE 227-3.  Physiology of the adrenal axis in health, Cushing syndrome, and pseudo-Cushing states. A, In healthy individuals, cortisol production is stimulated by the increased hypothalamic release of corticotropin-releasing hormone (CRH), which then travels down the pituitary stalk to stimulate adrenocorticotropic hormone (ACTH) secretion and release by corticotropes. Circulating ACTH stimulates adrenal gland production and secretion of cortisol. Cortisol then functions in a negative feedback mechanism to inhibit both CRH and ACTH. B, In Cushing disease, a pituitary tumor releases excessive amounts of ACTH, which results in increased cortisol secretion by the adrenal glands. C, In ectopic ACTH secretion, a nonpituitary ACTH-secreting tumor releases excessive amounts of ACTH, which results in increased cortisol secretion by the adrenal glands. D, In ACTH-independent adrenal forms of Cushing syndrome, the adrenal tumor autonomously releases excess amounts of cortisol. In all forms of Cushing syndrome, the negative feedback effects of excessive cortisol inhibit endogenous CRH and ACTH secretion, so that circulating ACTH levels reflect the underlying tumor (levels are normal or increased) or independent cortisol production (levels are suppressed). E, In pseudo-Cushing states, central stimulation increases CRH secretion, which in turn increases ACTH and hence cortisol production. In this setting, the negative feedback effects of excessive cortisol inhibit endogenous CRH and ACTH secretion, so that cortisol levels are ultimately constrained, albeit at an increased level.

TABLE 227-1  THE FREQUENCY OF CLINICAL SIGNS AND SYMPTOMS OF CUSHING SYNDROME SIGN OR SYMPTOM Decreased libido in men and women

PERCENTAGE 100

Obesity or weight gain

97

Plethora

94

Round face

88

Menstrual changes

84

Hirsutism

81

Hypertension

74

Ecchymoses

62

Lethargy, depression

62

Striae

56

Weakness

56

Electrocardiographic changes or atherosclerosis

55

Dorsal fat pad

54

Edema

50

Abnormal glucose tolerance

50

Osteopenia or fracture

50

Headache

47

Backache

43

Recurrent infections

25

Abdominal pain

21

Acne

21

Female balding

13

be abnormal to establish the diagnosis. Tests for the differential diagnosis of Cushing syndrome should not be used to make the diagnosis. Figure 227-4 is the Endocrine Society’s recommended algorithm for testing of patients suspected of having Cushing syndrome.3

Urine, Saliva, and Serum Cortisol Measurements

Urine free cortisol (UFC) excretion during 24 hours is a good screening test. Specific, structurally based assay techniques, such as high-performance liquid chromatography and tandem mass spectrometry, are the “gold standard.” The upper-normal limit of these tests is much lower and more specific than that of antibody-based assays, in which other steroids may cross-react. This crossreactivity may be an advantage in screening for hypercortisolism. UFC excretion also may be increased in the so-called pseudo-Cushing states, including psychiatric disorders (depression, anxiety disorder, obsessivecompulsive disorder), chronic pain, severe exercise, alcoholism, uncontrolled

diabetes, and morbid obesity. Here, it is hypothesized that higher brain pathways stimulate CRH release and activation of the entire hypothalamicpituitary-adrenal axis (see Fig. 227-3E). Cortisol negative feedback inhibition on CRH and pituitary ACTH release restrains the resulting hypercortisoluria to less than four-fold greater than normal. Thus, Cushing syndrome cannot be diagnosed with certainty unless values reach this threshold. Conversely, patients with Cushing syndrome may have normal UFC excretion because of mild or intermittent hypercortisolism or altered renal metabolism of cortisol. If UFC is only mildly elevated and clinical features are minimal, it is best to treat any pseudo-Cushing state and to remeasure UFC excretion with the expectation that it will normalize. Alternatively, if UFC values are normal but clinical suspicion is high, repeated measurement might disclose intermittent hypercortisolism. Measurement of plasma cortisol at midnight distinguishes pseudo-Cushing states from Cushing syndrome with 95% diagnostic accuracy; a level greater than 7.5 µg/dL is required for the diagnosis of Cushing syndrome. Measurement of salivary cortisol at bedtime or at midnight works as well, is more convenient, and may be the best screening test in patients with mild or intermittent hypercortisolism.4,5 However, the criteria for its interpretation differ, so each assay must be validated before it is used for this purpose.

Dexamethasone Suppression Tests

The dexamethasone suppression test is a simple screening test that takes advantage of the negative feedback effect of glucocorticoids to reduce ACTH (and hence serum cortisol). Dexamethasone 1 mg is given orally between 11:00 pm and midnight, and plasma cortisol is measured between 8:00 and 9:00 the next morning. The test has an 8% false-negative rate in patients with Cushing disease and a 30% false-positive rate in chronic illness, obesity, psychiatric disorders, and normal individuals. As a result, Cushing syndrome cannot be diagnosed by this test alone unless the result is extremely abnormal. The 2-day, 2-mg dexamethasone suppression test discriminates patients with a pseudo-Cushing state if plasma cortisol end points of less than 1.4 or 2.2 µg/dL are used. Dexamethasone 500 µg is given orally every 6 hours for eight doses, and plasma cortisol is measured 2 hours after the last dose. The test has excellent sensitivity (90 to 100%) and specificity (97 to 100%) for discriminating Cushing syndrome, but it is costly and requires excellent compliance of the patient. The immediate subsequent administration of CRH (1 µg per kilogram of body weight intravenously) and the measurement of cortisol 15 minutes later increased the sensitivity and specificity to 100% in a small study of patients, with values above 1.4 µg/dL indicating Cushing syndrome. Although this combined dexamethasone-CRH test has high diagnostic accuracy, it has the same disadvantages as the 2-day dexamethasone suppression test and the added cost of CRH testing. Because of these drawbacks, these tests are usually reserved for patients with ambiguous or confusing results on other screening tests. CRH is available commercially (Acthrel), with Food and Drug Administration–approved labeling for the differential diagnosis of Cushing syndrome. Its use in the dexamethasone-CRH test is an off-label use.

1516

CHAPTER 227  Adrenal Cortex  

Cushing syndrome suspected (consider endocinologist consultation)

TABLE 227-2  ETIOLOGY OF CUSHING SYNDROME EXOGENOUS Most common cause of Cushing syndrome:    Glucocorticoid or ACTH driven    May be factitious or iatrogenic

Exclude exogenous glucocorticoid exposure Perform one of the following tests

24-h UFC (≥ 2 tests)

Overnight 1-mg DST

Late night salivary cortisol (≥ 2 tests)

Use 48-h, 2-mg DST in certain populations (see text)

ENDOGENOUS ACTH independent—autonomous adrenal activation (20% of all cases)    Adrenal adenoma (40-50%)    Adrenal carcinoma (40-50%)    Primary pigmented nodular adrenal disease    McCune-Albright syndrome     Massive macronodular adrenal disease     Gastric inhibitory polypeptide or food induced ACTH dependent—adrenal activation by excessive ACTH (80% of all cases)    Corticotrope adenoma (80%)    Ectopic ACTH secretion (20%)    Ectopic CRH secretion (rare)

ACTH = adrenocorticotropic hormone; CRH = corticotropin-releasing hormone.

Any abnormal result

Normal (CS unlikely)

Exclude physiologic causes of hypercortisolism Consult endocrinologist Perform 1 or 2 other studies shown above Suggest consider repeating the abnormal study Suggest Dex-CRH or midnight serum cortisol in certain populations

Discrepant (Suggest additional evaluation)

Abnormal

Normal (CS unlikely)

Cushing syndrome

FIGURE 227-4.  Algorithm for testing of patients suspected of having Cushing syndrome (CS). All statements are recommendations except for those prefaced by “suggest”. Diagnostic criteria that suggest Cushing syndrome are urine free cortisol (UFC) greater than the normal range for the assay, serum cortisol greater than 1.8 µg/dL (50 nmol/liter) after 1 mg dexamethasone (1-mg DST), and late-night salivary cortisol greater than 145 ng/dL (4 nmol/liter). Dex-CRH = dexamethasone–corticotropin-releasing hormone test; DST = dexamethasone suppression test. (Reprinted with permission from Nieman LK, Biller BM, Findling JW, et al. The diagnosis of Cushing syndrome: an Endocrine Society clinical practice guideline. J Clin Endocrinol Metab. 2008;93:1526-1540.)

Any dexamethasone test may give false results in patients with abnormal metabolic clearance of the drug. Agents that induce the cytochrome P-450 CYP3A4 enzymes (alcohol, rifampin, phenytoin, phenobarbital) increase dexamethasone clearance, whereas renal or hepatic failure decreases it. Measurement of a dexamethasone level can determine whether its clearance has been altered.

Differential Diagnosis

The causes of endogenous Cushing syndrome can be divided broadly into ACTH-dependent (80%) and ACTH-independent (20%) forms (Table 2272). Hypercortisolism from autonomously functioning adrenal tumors suppresses ACTH, whereas in primary disorders of ACTH excess, the adrenal glands respond to tumor-derived ACTH. Plasma ACTH concentration distinguishes between these causes. ACTH is usually less than 10 pg/mL in primary adrenal disorders but is also suppressed by exogenous steroids, whether they are prescribed intentionally (iatrogenic Cushing syndrome) or taken factitiously. Patients in the latter group often have had multiple surgical procedures and do not reveal that they are self-administering steroids. As a

result, patients must be queried closely about exogenous steroid administration, recognizing that parenteral, inhaled, and topical steroids can all cause glucocorticoid excess. Patients with endogenous Cushing syndrome and low ACTH concentrations should undergo adrenal imaging to identify the site of adrenal abnormality. Nonautonomous adrenal tissue atrophies when ACTH support is subnormal. Because of this, the common ACTH-independent forms of Cushing syndrome—adrenal adenoma and carcinoma—are manifested as a unilateral adrenal mass, with atrophy of the adjacent and contralateral tissue on magnetic resonance imaging or computed tomography. Bilateral forms of primary adrenal disease are rare and may be manifested with small or large adrenal nodules.6 Primary pigmented nodular adrenal disease occurs primarily in children and young adults and is characterized by small to normal-sized adrenal glands containing small (15 pg/mL; 3.3 pmol/L) is consistent with an ACTH-producing tumor.7 Intermediate ACTH concentrations between 5 and 15 pg/mL (1.1 to 3.3 pmol/L) in a two-site sandwich assay are not diagnostic. In these patients, suboptimal cortisol responses to CRH stimulation may identify the minority of cases of ACTH-independent Cushing syndrome with borderline basal ACTH values. In addition, a suppressed plasma DHEA-S value supports the diagnosis of an ACTHindependent disorder. Cushing disease,8 an ACTH-secreting pituitary adenoma, is the most common cause of Cushing syndrome. It is more common in women than in men (6 : 1 ratio), with a mean age at onset in the fourth decade. ACTH also may be secreted ectopically by a variety of neuroendocrine tumors, as shown in Table 227-3. Pituitary magnetic resonance imaging shows a tumor in only about 40 to 50% of patients with Cushing disease, but it is obtained routinely in patients with ACTH-dependent disease to exclude a macroadenoma or abnormal anatomy before petrosal sinus sampling or surgery. A pituitary lesion less than 6 mm is seen in up to 10% of healthy individuals and so does not always indicate Cushing disease. Biochemical tests must be used to distinguish among the ACTH-dependent causes of Cushing syndrome, and they must be performed after a 6- to 8-week period of sustained hypercortisolism sufficient to suppress normal corticotrope function. Inferior petrosal sinus sampling is the best test to distinguish between a pituitary and an ectopic source of excess ACTH; worldwide, the overall

CHAPTER 227  Adrenal Cortex  

TABLE 227-3  THE INCIDENCE AND TYPES OF TUMORS CAUSING THE SYNDROME OF ECTOPIC ACTH SECRETION TUMOR TYPE Carcinoma of lung (small cell or oat cell)

PERCENTAGE 19-50

Carcinoid of bronchus

2-37

Carcinoid of thymus

8-12

Pancreatic tumors, carcinoid and islet cell

4-12

Pheochromocytoma, neuroblastoma, ganglioma, paraganglioma

5-12

Medullary carcinoma of the thyroid

0-5

Miscellaneous*

20) of morning aldosterone to plasma renin activity (Fig. 227-5). One of four tests (usually salt loading) is used to confirm primary hyperaldosteronism by demonstrating a lack of aldosterone suppression.10

Differential Diagnosis

Having made the diagnosis of aldosterone-dependent mineralocorticoid excess, one must differentiate between the two most common adrenal causes—hyperplasia and adenoma—after excluding potential rare causes of hyperaldosteronism. Two rare autosomal dominant forms of familial hyperaldosteronism are type 1, a glucocorticoid-suppressible hyperaldosteronism, and type 2. Familial hyperaldosteronism type 1 is caused by a genetic swap of the promoter for CYP11B1 (11β-hydroxylase) with that of CYP11B2 (aldosterone synthase), forming a chimeric gene in which ACTH stimulates aldosterone synthase. It should be suspected in the setting of familial disease, particularly if there is a history of early-onset cardiovascular events, and is confirmed by gene testing (see http://www.brighamandwomens .org/Departments_and_Services/medicine/services/endocrine/Services/ gra/default.aspx). The genetic abnormality in familial hyperaldosteronism

1518

CHAPTER 227  Adrenal Cortex  

SUSPECTED PRIMARY ALDOSTERONISM (PA) (Hypertension at early age with hypokalemia or difficult to control)

TABLE 227-4  CAUSES OF MINERALOCORTICOID EXCESS PRIMARY HYPERALDOSTERONISM: HIGH ALDOSTERONE, LOW RENIN Aldosterone-producing adenomas (30-50%) Bilateral zona glomerulosa hyperplasia Familial hyperaldosteronism   Type 1: glucocorticoid-remediable hyperaldosteronism—this results from formation of a chimeric gene containing the regulator portion of 11β-hydroxylase (normally regulated by ACTH) and the synthetic region of aldosterone synthase; as a result, ACTH stimulates aldosterone synthase and hence aldosterone production   Type 2: adrenal adenomas or hyperplasia expressed in a familial pattern   Type 3: caused by mutant KCNJ5, often younger and more severe than Type 2 Aldosterone-producing adrenal carcinoma Ectopic aldosterone secretion (rare): kidney, ovary

Morning plasma aldosterone : renin ratio > 20 : 1

No

Yes

PA unlikely

Any of 4 confirmatory tests: • oral Na+ loading or • saline infusion or • fludrocortisone suppression or • captopril challenge

SECONDARY HYPERALDOSTERONISM: HIGH ALDOSTERONE, HIGH RENIN

Positive

Renovascular hypertension and aortic stenosis Diuretic use Renin-secreting tumors Severe cardiac failure APPARENT MINERALOCORTICOID EXCESS: LOW ALDOSTERONE, LOW RENIN Licorice ingestion: licorice (candy or flavored tobacco) containing glycyrrhetinic acid (or similar compounds such as carbenoxolone) inhibits renal 11β-hydroxysteroid dehydrogenase type 2, reducing cortisol conversion to cortisone and enabling cortisol to act as an endogenous mineralocorticoid Severe hypercortisolism: similar in mechanism to licorice ingestion; very high cortisol levels are thought to overwhelm the ability of 11β-hydroxysteroid dehydrogenase type 2 to convert cortisol to cortisone in the kidney; cortisol itself then acts as a potent mineralocorticoid Liddle’s syndrome: mutation of the β or γ subunit of the collecting tubule sodium channel leads to a constitutive increase in sodium reabsorption and potassium excretion 11β-Hydroxylase deficiency form of congenital adrenal hyperplasia: 11-deoxycortisol accumulates because of an inability to convert it to cortisol 17-Hydroxylase deficiency form of congenital adrenal hyperplasia: deoxycorticosterone and corticosterone are increased

type 2 is not known; its clinical presentation is similar to sporadic hyperaldosteronism. Recent studies demonstrate rare germline mutations of a potassium channel in familial hyperalsosteronism, and somatic adrenal mutations in about 40% of patients. For the more common conditions, adrenal computed tomography scans may show nonfunctioning nodules and falsely suggest an adenoma.11 The responses to physiologic maneuvers, such as upright posture, and salt loading with oral or intravenous sodium tend to be preserved in patients with hyperplasia, but there is significant overlap among groups of patients. The best diagnostic test involves the measurement of cortisol and aldosterone in bilateral adrenal venous effluent and a peripheral vein before and during an ACTH infusion. Cortisol is used to evaluate catheter placement in the adrenal veins, as levels from the two sides should be similar. When an adenoma is present, the aldosterone-to-cortisol ratio on one side is usually at least five-fold greater than the other, which may be similar to the periphery, indicating suppression. Bilateral hyperplasia tends to produce similar values on each side.

Negative

Adrenal CT

Surgical treatment possible

Medical treatment

Adrenal venous sampling

Mineralocorticoid receptor antagonist

Unilateral

Bilateral

Laparoscopic adrenalectomy FIGURE 227-5.  Algorithm for the detection, confirmation, subtype testing, and treatment of primary aldosteronism (PA). We recommend the case detection of PA in patient groups with relatively high prevalence of PA. These include patients with moderate, severe, or resistant hypertension; spontaneous or diuretic-induced hypokalemia; hypertension with adrenal incidentaloma; or a family history of early-onset hypertension or cerebrovascular accident at a young age (20 HU) E. Rapid contrast washout with more than 50% washout at 10 minutes Answer: D  See section on diagnosis. Imaging characteristics of an incidentally discovered adrenal mass consistent with pheochromocytoma include increased baseline CT Hounsfield unit density (e.g., >20 HU); marked enhancement with intravenous contrast medium on CT; high signal intensity on T2-weighted MRI; cystic and hemorrhagic changes; bilaterality; and large size (>4 cm). In addition, pheochromocytomas are characterized by slow contrast washout (e.g., 50% from untreated baseline); all others would be considered for moderateintensity treatment (lowering of LDL cholesterol 30 to 1000 mg/dL) is indicated to prevent acute pancreatitis.

ANTIPLATELET THERAPY

Prophylactic aspirin therapy is widely used for prevention of cardiovascular events in high-risk patients (i.e., those with prior myocardial infarction or stroke), with reported risk reductions of about 12% (Chapter 38). Results from clinical trials in patients with diabetes suggest that aspirin may be somewhat less effective for CVD prevention than in patients without diabetes, although this has not been a consistent finding. Current guidelines recommend aspirin therapy for diabetic patients with a prior CVD event (secondary prevention) or with increased CVD risk (10-year risk of >10%). This includes most men older than 50 years or women older than 60 years who also have one or more additional CVD risk factors: smoking, hypertension, albuminuria, dyslipidemia, or family history of CVD. For patients at lower CVD risk, the potential adverse effects from bleeding may outweigh the potential benefits, and routine use is not recommended. The optimal dose (balancing thrombosis prevention with the risk of bleeding) of aspirin has not been established and may differ according to patient characteristics, but 75 to 162 mg/day is commonly recommended. For high-risk patients who are unable to tolerate aspirin, clopidogrel is an effective alternative.

TREATMENT OF ESTABLISHED CARDIOVASCULAR DISEASE IN DIABETES  In general, treatment of clinically established CVD, including acute coronary syndromes and stable angina, is similar in diabetic and nondiabetic patients. There is some evidence that ischemic symptoms may be less intense, atypical, or absent in diabetic patients, leading to higher rates of “silent” myocardial infarction. However, a strategy of screening for ischemic heart disease, by exercise stress testing, in asymptomatic patients did not result in lower event rates or improved outcomes. Therefore, current recommendations are for coronary artery disease screening in patients with symptoms suggestive of ischemia. The role of intravenous insulin (with or without potassium and glucose infusion) in the setting of acute myocardial infarction has been considered in a few studies. In the Diabetes and Insulin-Glucose Infusion in Acute Myocardial Infarction (DIGAMI) study, acute myocardial infarction patients with diabetes were treated with standard therapy or with insulin infusion during the first 48 hours, followed by continued insulin use after hospital discharge. Mortality after 1 year was reduced by 30% in the insulin-treated group. However, the implications of these results have been debated because factors other than insulin treatment differed between the two groups (i.e., sulfonylureas were routinely used in the standard therapy group but withdrawn from the insulin group). These findings subsequently were not confirmed in a follow-up study, and this approach has largely been abandoned.

1548

CHAPTER 230  Hypoglycemia and Pancreatic Islet Cell Disorders  

Several studies have addressed the roles of medical therapy and revascularization in diabetic patients with coronary artery disease. Among them, the Bypass Angioplasty Revascularization Investigation 2 Diabetes (BARI 2D) study demonstrated that a policy of medical management (including aggressive risk factor modification) was as effective as early revascularization in diabetic patients with stable angina. In the Future Revascularization Evaluation in Patients with Diabetes Mellitus: Optimal Management of Multivessel Disease (FREEDOM) trial, diabetic patients with multivessel coronary disease had better outcome (reduced rates of death from any cause or nonfatal myocardial infarction) with coronary bypass surgery compared with percutaneous intervention with drug-eluting stents, although strokes were more frequent in the surgical group.

Grade A References A1. Misso ML, Egberts KJ, Page M, et al. Continuous subcutaneous insulin infusion (CSII) versus multiple insulin injections for type 1 diabetes mellitus. Cochrane Databse Syst Rev. 2010;1: CD005103. A2. Hemmingsen B, Lunc S, Gluud C, et al. Targeting intensive glycaemic control versus targeting conventional glycaemic control for type 2 diabetes mellitus. Cochrane Database Syst Rev. 2013;11:CD008143. A3. Schauer PR, Bhatt DL, Kirwan JP, et al. Bariatric surgery versus intensive medical therapy for diabetes—3 year outcomes. N Engl J Med. 2014;370:2002-2013. A4. Reznik Y, Cohen O, Aronson R, et al. Insulin pump treatment compared with multiple daily injections for treatment of type 2 diabetes (OpT2mise): a randomised open-label controlled trial. Lancet. 2014;384:1265-1272. A5. Finfer S, Liu B, Chittock DR, et al. The NICE-SUGAR Study Investigators. Hypoglycemia and risk of death in critically ill patients. N Engl J Med. 2012;367:1108-1118. A6. Diabetes Prevention Program Research Group. 10-year follow-up of diabetes incidence and weight loss in the Diabetes Prevention Program Outcomes Study. Lancet. 2009;374:1677-1686. A7. Gaede P, Lund-Andersen H, Parving HH, et al. Effect of a multifactorial intervention on mortality in type 2 diabetes. N Engl J Med. 2008;358:580-591. A8. Nguyen Q, Brown D, Marcus D, et al. Ranibizumab for diabetic macular edema: results from 2 phase III randomized trials: RISE and RIDE. Ophthalmology. 2012;119:789-801. A9. Keech AC, Mitchell P, Summanen PA, et al. Effect of fenofibrate on the need for laser treatment for diabetic retinopathy (FIELD study): a randomized controlled trial. Lancet. 2007;370: 1687-1697. A10. Fried L, Emanuele N, Zhang J, et al. Combined angiotensin inhibition for the treatment of diabetic nephropathy. N Engl J Med. 2013;369:1892-1903. A11. Wing RR, Bolin P, Brancati FL, et al. Cardiovascular effects of intensive lifestyle intervention in type 2 diabetes. N Engl J Med. 2013;369:145-154. A12. Zoungas S, Chalmers J, Neal B, et al. Follow-up of blood-pressure lowering and glucose control in type 2 diabetes. N Engl J Med. 2014;371:1392-1406.

GENERAL REFERENCES For the General References and other additional features, please visit Expert Consult at https://expertconsult.inkling.com.

230  HYPOGLYCEMIA AND PANCREATIC ISLET CELL DISORDERS KHALID HUSSAIN

to 3 days after birth, low plasma glucose concentrations are common in normal neonates, which makes it difficult to identify the minority who have a persistent hypoglycemia disorder or a genetic hypoglycemia disorder. Third, the importance of early recognition and treatment of such persistent hypoglycemia disorders in neonates is emphasized by reports that developmental handicap, which might have been avoidable by early recognition and treatment, occurs in 25 to 50% of cases with congenital hyperinsulinism.1 There is no absolute number that defines hypoglycemia in adults and in children. The current adult recommendations define clinical hypoglycemia as a plasma (or serum) glucose concentration low enough to cause symptoms and/or signs, including impairment of brain function. Because the clinical manifestations and symptoms of hypoglycemia are nonspecific, it is therefore not possible to state a single plasma glucose concentration that categorically defines hypoglycemia. The measured plasma or serum glucose concentration may be low owing to an artifact (e.g., when the blood sample is collected in a tube that does not contain an inhibitor of glycolysis and when separation of the plasma or serum from the formed elements is delayed). For these reasons, guidelines in adults emphasize the value of Whipple triad for confirming hypoglycemia: (1) symptoms and/or signs compatible with hypoglycemia, (2) a low measured plasma glucose concentration, and (3) resolution of symptoms and signs when glucose concentrations are raised. Because circulating fuels such as ketone bodies can be used by the brain, lower plasma glucose concentrations can occur in healthy individuals, particularly in women and children, without symptoms or signs during extended fasting. Therefore, for all of these reasons, it is not possible to state a single plasma glucose concentration that categorically defines hypoglycemia. The aim of this chapter is to outline the physiologic and biochemical changes associated with maintenance of a normal blood glucose level, describe the role of the counter-regulatory hormones, review the different hypoglycemia disorders observed in adults and children, and then finally discuss the various management strategies.

PATHOBIOLOGY

Physiologic and Biochemical Changes During Fasting and Feeding Overview

Plasma glucose concentration is tightly controlled by a balance between glucose production and utilization. Glucose is derived from three sources: (1) intestinal absorption that follows digestion of dietary carbohydrates; (2) glycogenolysis, the breakdown of glycogen, which is the polymerized storage form of glucose; and (3) gluconeogenesis, the formation of glucose from precursors including lactate (and pyruvate), amino acids (especially alanine and glutamine), and to a lesser extent, glycerol. Normally, there is tight coordination between rates of endogenous glucose influx into the circulation and glucose efflux out of the circulation into insulin-dependent tissues (skeletal muscle, adipose tissue, and liver). This coordination, despite periods of feeding and fasting, maintains the plasma glucose concentration in a relatively narrow range between 70 and 110 mg/dL (3.8 to 6 mmol/L). Figure 230-1 shows an outline of glucose physiology. Glucose is an obligate metabolic fuel for the brain under physiologic conditions. Unlike other body tissues, the brain cannot oxidize fatty acids, and neither can it synthesize/store glucose for later use. It is dependent on a continuous supply of glucose from the circulation. Given the vital importance of brain function and the above circumstances, it is not surprising that physiologic mechanisms have evolved for the maintenance of plasma glucose concentrations.

Changes During Fasting

DEFINITIONS

Hypoglycemia is one of the most common biochemical abnormalities observed in clinical practice. It is a biochemical finding and not a diagnosis. Hypoglycemic disorders are more common in neonates, infants and children as compared to adults. Inappropriately treated hypoglycemia can have severe consequences, including seizures, permanent brain injury, or death. This is especially the case in neonates with persistent forms of hypoglycemia, who are at high risk of brain injury from delays in diagnosis and effective therapy. Hypoglycemic disorders in neonates, infants, and children differ from adults in important aspects. First, they are most often due to congenital or genetic disorders, such as disorders of insulin secretion, as well as a range of metabolic and endocrine diseases. Second, during a transitional period of 1

During fasting, the basal rate of glucose output by the liver is precisely matched to glucose uptake by various body tissues. They average 2.2 mg/kg/ minute in healthy adults after an overnight fast. In infants, these rates are much higher (≈6 mg/kg/minute) because of their greater brain mass relative to their body weight. The brain is responsible for nearly two thirds of basal glucose utilization. The remaining one third is used by red blood cells, renal medulla, and to some extent muscle and fat. Hepatic glucose production results from a combination of glycogenolysis and gluconeogenesis. Endogenous glucose production is also contributed by gluconeogenesis in the kidneys. Breakdown of stored hepatic glycogen is a readily available source of free glucose. However, in an average adult, this process can only provide less than an 8-hour supply of free glucose. (In infants, this may provide only 4 hours of free glucose.) Considering this

CHAPTER 229  Diabetes Mellitus  

GENERAL REFERENCES 1. Atkinson MA, Eisenbarth GS, Michels AW. Type 1 diabetes. Lancet. 2014;383:69-82. 2. Keenan H, Sun JK, Levine J, et al. Residual insulin production and pancreatic B-cell turnover after 50 years of diabetes: Joslin Medalist Study. Diabetes. 2010;59:2846-2853. 3. Evert A, Boucher J, Cypress M, et al. Nutrition therapy recommendations for the management of adults with diabetes. Diabetes Care. 2014;37(suppl 1):S120-S143. 4. Skyler JS. Primary and secondary prevention of type 1 diabetes. Diabet Med. 2013;30:161-169. 5. Miller RG, Secrest AM, Sharma RK. Improvements in the life expectancy of type 1 diabetes: the Pittsburgh Epidemiology of Diabetes Complications study cohort. Diabetes. 2012;61:2987-2992. 6. Lind M, Svensson AM, Kosiborod M, et al. Glycemic control and excess mortality in type 1 diabetes. N Engl J Med. 2014;371:1972-1982. 7. Gregg EW, Li Y, Wang J, et al. Changes in diabetes-related complications in the United States, 19902010. N Engl J Med. 2014;370:1514-1523. 8. Kahn SE, Cooper ME, Del Prato S. Pathophysiology and treatment of type 2 diabetes: perspectives on the past, present, and future. Lancet. 2014;383:1068-1083. 9. Ng HJ, Gloyn AL. Bridging the gap between genetic associations and molecular mechanisms for type 2 diabetes. Curr Diab Rep. 2013;13:778-785. 10. Hales CN, Barker DJ. Type 2 (non-insulin-dependent) diabetes mellitus: the thrifty phenotype hypothesis. Int J Epidemiol. 2013;42:1215-1222. 11. Bray G, Edelstein S, Crandall JP, Diabetes Prevention Program Research Group, et al. Long-term safety, tolerability, and weight loss associated with metformin in the Diabetes Prevention Program Outcomes Study. Diabetes Care. 2012;35:731-737. 12. Qaseem A, Humphrey LL, Chou R, et al. Use of intensive insulin therapy for the management of glycemic control in hospitalized patients: a clinical practice guideline from the American College of Physicians. Ann Intern Med. 2011;154:260-267.

1548.e1

13. Seaquist ER, Anderson J, Childs B, et al. Hypoglycemia and diabetes: a report of a workgroup of the American Diabetes Association and the Endocrine Society. Diabetes Care. 2013;36: 1384-1395. 14. Nathan DM, Bayless M, Cleary P, et al. Diabetes control and complications trial/epidemiology of diabetes interventions and complications study at 30 years: advances and contributions. Diabetes. 2013;62:3976-3986. 15. Wheeler ML, Dunbar SA, Jaacks LM, et al. Macronutrients, food groups and eating patterns in the management of diabetes: a systematic review of the literature. Diabetes Care. 2012;35:434-445. 16. Griebeler ML, Morey-Vargas OL, Brito JP, et al. Pharmacologic interventions for painful diabetic neuropathy: An umbrella systematic review and comparative effectiveness network meta-analysis. Ann Intern Med. 2014;161:639-649. 17. Kishore P, Kim S, Crandall JP. Glycemic control and cardiovascular disease: what’s a doctor to do? Curr Diab Rep. 2012;12:255-264. 18. James PA, Oparil S, Carter BL, et al. 2014 evidence-based guideline for the management of high blood pressure in adults: report from the panel members appointed to the Eighth Joint National Committee ( JNC 8). JAMA. 2014;311:507-520. 19. Stone NJ, Robinson JG, Lichtenstein AH, et al. 2013 ACC/AHA guideline on the treatment of blood cholesterol to reduce atherosclerotic cardiovascular risk in adults: a report of the American College of Cardiology/American Heart Association Task Force on Practice Guidelines. J Am Coll Cardiol. 2014;63:2889-2934. 20. Preiss D, Seshasai SR, Welsh P. Risk of incident diabetes with intensive-dose compared with moderate-dose statin therapy: a meta-analysis. JAMA. 2011;305:2556-2564.

1548.e2

CHAPTER 229  Diabetes Mellitus  

REVIEW QUESTIONS 1. A 51-year-old man has a history of type 2 diabetes mellitus for 6 years. Past medical history is significant for chronic hepatitis C infection, chronic kidney disease stage 3, and a recent hospitalization for an upper gastrointestinal bleed. He takes a sulfonylurea for blood glucose control and rarely checks his blood glucose level. Fasting plasma glucose concentration in the office is 195 mg/dL, and his HbA1c is 6.8%. What do you conclude about his glucose control? A. His average blood glucose concentration during the past 3 months is approximately 140 mg/dL. B. HbA1c may be falsely high because of chronic kidney disease. C. HbA1c may be falsely low because of liver disease. D. HbA1c levels are increased after acute blood loss. E. HbA1c levels are more reflective of postprandial than of fasting glucose concentration. Answer: C  This patient has several reasons that his HbA1c may not accurately reflect his mean plasma glucose concentration. HbA1c results may be influenced by a number of factors, including conditions that alter red cell survival or cause interference with a specific assay. The HbA1c may be falsely low in this patient because of cirrhosis (increased red cell turnover), recovery from recent acute blood loss (greater percentage of younger erythrocytes with shorter exposure to glucose), or transfusion (dilution of patient’s blood with nondiabetic donor blood). In these instances, measurement of glycated serum proteins (fructosamine) or direct measurement of plasma glucose concentration will more accurately reflect glycemic control. Additional information about HbA1c assay methodology and interpretation of results can be obtained from the National Glycohemoglobin Standardization Program: http://www.ngsp.org. 2. A 38-year-old woman has had type 1 diabetes mellitus since the age of 12 years. She has maintained excellent control (HbA1c 6.0%) with a basal/ bolus injection regimen. She tests her glucose level four or five times a day, and review of her meter download shows many glucose levels in the 30s and 40s. However, the patient is unconcerned because she has no symptoms at these times. On questioning, she admits to recently “spacing out” while driving, which led to a minor traffic accident. Regarding the etiology and treatment of hypoglycemia in this patient: A. She has adapted to low blood glucose concentration and no change in treatment is required. B. She has developed hypoglycemia unawareness and her target HbA1c should be increased. C. Strict avoidance of hypoglycemia is of little benefit in reversing hypoglycemia-associated autonomic failure. D. An excessive counter-regulatory hormone response to hypoglycemia may contribute to her lack of symptoms. E. Treatment with β-blocker should be considered. Answer: B  In patients with long-standing diabetes, the counter-regulatory systems that normally would counteract the decline of glucose to dangerous levels may be impaired. This is especially true for patients with type 1 diabetes, who often have defects in glucagon and epinephrine response during hypoglycemia. This decrease in epinephrine response during hypoglycemia is accompanied by an attenuated autonomic neural response, which results in the clinical syndrome of impaired awareness of hypoglycemia. Without autonomic symptoms, mild hypoglycemia may proceed unnoticed to more advanced and dangerous phases. There is, however, evidence that hypoglycemia-associated autonomic failure can be reversed by strict avoidance of hypoglycemia, which can be facilitated by increasing target glucose levels. Reno CM, Litvin M, Clark AL, Fisher SJ. Defective counterregulation and hypoglycemia unawareness in diabetes: mechanisms and emerging treatments. Endocrinol Metab Clin North Am. 2013;42:15-38.

3. A mother brings her 19-year-old son, who has type 1 diabetes and uses an insulin pump, to see you and to ask for referral to a dietitian. She complains that her son refuses to follow his “diabetic diet” and frequently eats junk food, including fast food (burgers, fries, pizza) and ice cream. She also worries that he sometimes skips meals, saying he is not hungry. His body mass index is 22, and his recent laboratory results show an HbA1c of 7.8%, low-density lipoprotein cholesterol of 95, and normal triglycerides. Which of the following dietary recommendations is most appropriate for this patient? A. An 1800-calorie/day American Diabetes Association diet B. A low-carbohydrate diet C. A low-protein diet D. A low-fat, high-fiber diet E. A flexible “heart healthy” meal plan that limits concentrated sweets and emphasizes fruits and vegetables Answer: E  Dietary recommendations for patients with diabetes have changed substantially over time, from the extremely low-carbohydrate, highfat diets used before the discovery of insulin as a therapy, to “exchange diets,” to more flexible meal plans. For patients with type 1 diabetes, the key element is for the patient to learn to match mealtime insulin doses to the carbohydrate content of the meal. Severely restricted diets (very low carbohydrate or low calorie) are neither required nor advisable, although avoidance of large carbohydrate loads will help minimize post-meal glycemic excursions. For most patients with type 2 diabetes who are typically overweight or obese, moderate carbohydrate intake and reduction in total calories are advised. Current recommendations allow a variety of eating styles and ethnic food preferences, with emphasis on fruits and vegetables, low-fat protein sources, and use of monounsaturated or polyunsaturated fats. Evert A, Boucher J, Cypress M, et al. Nutrition therapy recommendations for the management of adults with diabetes. Diabetes Care. 2013;36: 3821-3842. Lasa A, Miranda J, Bullo M, et al. Comparative effect of two Mediterranean diets versus a low-fat diet on glycaemic control in individuals with type 2 diabetes. Eur J Clin Nutr. 2014;68:767-772. 4. A 54-year-old woman presents to her physician for treatment of hypertension. She had gestational diabetes during her last pregnancy 15 years ago, and there is a family history of type 2 diabetes (mother and older brother). Her body mass index is 36. Fasting glucose concentration is 110 mg/dL, and HbA1c is 6.2%. Which of the following have been shown to reduce the progression to diabetes in high-risk patients? A. Weight loss by reduced calorie diet B. Treatment with metformin C. Treatment with acarbose D. Bariatric surgery E. All of the above Answer: E  This patient has multiple risk factors for the development of type 2 diabetes, including family history, prior history of gestational diabetes, obesity, and hypertension. In addition, her glucose and HbA1c levels are already elevated above normal, in the defined “pre-diabetes” range. All of the treatments listed have been shown to prevent or to delay the onset of diabetes in randomized clinical trials. The most consistent evidence comes from weight loss trials. Both the Finnish Diabetes Prevention Study and the U.S. Diabetes Prevention Program reported 58% reduction in diabetes with a hypocalorie, reduced fat diet combined with moderate-intensity physical activity. Weight loss achieved with bariatric surgery is also highly effective in preventing (or even reversing) diabetes. Medications, including metformin, the α-glucosidase inhibitor acarbose, and troglitazone (a thiazolidinedione), have also been shown to reduce diabetes in high-risk patients, although somewhat less effectively than by lifestyle modification. Lifestyle changes and metformin are reported to be cost-effective interventions, but whether delay or prevention of type 2 diabetes will result in lower rates of cardiovascular disease and diabetes microvascular complications will require longer-term follow-up studies. Schwarz PE, Greaves CJ, Lindstrom J, et al. Nonpharmacologic interventions for the prevention of type 2 diabetes mellitus. Nat Rev Endocrinol. 2012;8:363-373.

CHAPTER 229  Diabetes Mellitus  

5. A 28-year-old woman with type 1 diabetes since the age of 12 years is considering having a child. Currently, her blood glucose is reasonably well controlled, although she admits this was not the case during her teens and early 20s, when her HbA1c was in the 9 to 11% range. She has mild background diabetic retinopathy, normal blood pressure, urine albumin-tocreatinine ratio of 25 mg/g, and normal findings on foot examination. Which of the following is true? A. She should delay pregnancy until she has achieved optimal glucose control (HbA1c ~6.5%). B. She should be treated with an angiotensin-converting enzyme inhibitor to prevent progression of renal disease during pregnancy. C. Progression of her retinopathy during pregnancy is likely to result in vision loss. D. The risk of her child’s developing type 1 diabetes is 25 to 50%. E. She should be advised to avoid pregnancy because of the risk of both maternal and fetal complications.

1548.e3

Answer: A  Although pregnancies in women with type 1 diabetes are generally considered “high risk,” the outlook for patients with minimal complications and good metabolic control is good. Women with advanced renal disease (proteinuria, reduced glomerular filtration rate) or proliferative retinopathy may experience rapid progression during pregnancy because of the influence of hormonal and hemodynamic changes and should be monitored closely by specialists. Use of angiotensin-converting enzyme inhibitors is contraindicated during pregnancy because of the risk of fetal renal damage. The key to successful pregnancy outcomes is achieving optimal glucose control before conception because the developing fetus is most susceptible to the teratogenic effects of hyperglycemia in the first 6 to 8 weeks of pregnancy, before the time that most women are aware of being pregnant. Maintaining strict glucose control during the pregnancy will reduce the risk of fetal complications, such as macrosomia and hyperbilirubinemia. Many patients benefit from use of an insulin pump and continuous glucose monitoring during this time. Although a child of a mother with type 1 diabetes is at increased risk of diabetes compared with the general population, the risk is less than 10%.

CHAPTER 230  Hypoglycemia and Pancreatic Islet Cell Disorders  

Phosphorylase

1549

Glycogen Liver

Glucose Glucokinase

Glycogen synthase

Glucose-6-phosphatase

Glucose-6-phosphate Phosphomutase

Glucose-1-phosphate

Fructose-6-phosphate Phosphofructokinase

Fructose 1,6-bisphosphatase

Fructose 1,6-bisphosphate

Glyceraldehyde 3-phosphate

Phosphoenolpyruvate Lactate dehydrogenase Pyruvate Phosphoenolpyruvate carboxykinase

Lactate

Pyruvate dehydrogenase Acetyl CoA

Citrate Isocitrate

Oxaloacetate Malate

Citric acid cycle

Fumarate

α-ketoglutarate Succinyl-CoA

Mitochondria Succinate

FIGURE 230-1.  Outline of the biochemical pathways involved in glucose physiology.

limited capacity of glycogenolysis, gluconeogenesis is very important in supporting hepatic glycogen stores during an overnight fast. Gluconeogenesis uses a number of key enzymes: pyruvate carboxylase, phosphoenolpyruvate carboxykinase (PEPCK), and fructose-1,6bisphosphatase and its precursors, including lactate, alanine, glutamine, glycerol, and pyruvate. Muscle and adipose tissue, which utilize glucose in the fed state, respond to prolonged fasting by reducing their glucose uptake virtually to zero and satisfying their energy requirements by the β-oxidation of fatty acids. Additionally, through the process of proteolysis, muscle tissue provides amino acids to the liver to serve as gluconeogenic precursors for net glucose formation. Changes in the hormonal milieu during fasting (suppressed insulin and elevated counter-regulatory hormones) stimulate ketogenesis. Ketones become a major source of fuel for the brain when glucose utilization by the brain declines. This results in a decrease in the rate of gluconeogenesis required to maintain the plasma glucose concentration and hence in diminished protein wasting.

Changes During Feeding

After a meal, glucose absorption into the circulation increases glucose concentrations, which stimulates secretion of insulin from the pancreatic β-cells and suppresses secretion of glucagon from the pancreatic α-cells. This change

in the hormonal milieu switches off endogenous hepatic glucose production and accelerates glucose utilization by liver, muscle, and adipose tissue. Glucose concentration then returns gradually to the postabsorptive level, at which endogenous glucose production is equal to the glucose uptake by peripheral tissues.

Counter-Regulatory Hormonal Responses to Hypoglycemia The counter-regulatory hormones play a key role in the maintenance of normal blood glucose concentration.2 If counter-regulation is intact, hypoglycemia (irrespective of the cause) will result in a decrease in insulin secretion and an increase in glucagon, epinephrine, norepinephrine, cortisol, and growth hormone (GH) secretion. Glucagon secretion increases rapidly in response to hypoglycemia, and studies have shown that the glucagon response is the primary essential defense mechanism against acute hypoglycemia. GH and cortisol have numerous effects on glucose metabolism, including increasing the rate of gluconeogenesis and antagonizing the effects of insulin. In adults, the glycemic thresholds for the activation of glucose counterregulatory hormones such as GH and cortisol lie within or just below the physiologic blood glucose concentration and slightly higher than the threshold for symptoms. This suggests that GH and cortisol secretion increase in response to blood glucose concentrations within the normoglycemic range,

1550

CHAPTER 230  Hypoglycemia and Pancreatic Islet Cell Disorders  

Glucose and ketone bodies

• Glycogenolysis • Gluconeogenesis • Ketogenesis

Lactate Alanine

Proteolysis activation Muscle tissue

Liver

Fatty acids and glycerol Lipolysis

Cortisol, growth hormone, glucagon and adrenaline act as counter regulatory hormones and increase glucose output from the liver by increasing the processes of glycogenolysis and gluconeogenesis

Adipose tissue FIGURE 230-2.  The role of the counter-regulatory hormones, glycogenolysis, gluconeogenesis, and lipolysis in glucose physiology.

and these increases are inversely proportional to the nadir in blood glucose. Figure 230-2 outlines the role of the counter-regulatory hormones. Insulin secretion from β-cells of the pancreas in healthy individuals is inhibited as blood glucose concentration falls below 72 mg/dL (4.0 mmol/L). As insulin secretion is reduced, the repressive effect of insulin on pancreatic α-cell function is removed, thereby rapidly increasing glucagon secretion. Glucagon acts on the liver to increase hepatic glycogenolysis and gluconeogenesis. When the blood glucose concentration falls further (≈68 mg/dL [3.8 mmol/L]), epinephrine and norepinephrine are released both from the adrenals and directly into interstitial fluid from nerve terminals, further suppressing insulin secretion, increasing glucagon secretion, and decreasing peripheral glucose utilization in the muscle and increasing lipolysis in the adipose tissues. Additional responses include GH and cortisol secretion, which occur below a blood glucose concentration of around 66 mg/dL (≈3.7 mmol/L) and are initiators of the adaptive response to hypoglycemia (e.g., during prolonged starvation); glucose-raising actions are much slower in onset (several hours). These hormone responses stimulate lipolysis, ketogenesis, and gluconeogenesis. Permissive amounts of cortisol and GH are required for a normal hepatic response to glucagon and epinephrine. In healthy individuals, this system ensures that hypoglycemia is rarely experienced and would only occur during starvation or ultra-endurance sports. Drugs or diseases that inhibit counter-regulatory secretion or action predispose patients to hypoglycemia. Activation of counter-regulation depends on effective detection of falling blood glucose levels. This is achieved by the complex integration of various glucose-sensing systems in both the periphery and central nervous system.3 Fluctuations in peripheral glucose levels are detected by glucose-sensing neurons in the oral cavity, gut, portal/mesenteric vein (PMV), and carotid body. PMV neurons detect changes in blood glucose prior to entry into the liver from the gut. This information is then relayed through the vagus nerve and spinal cord to the hindbrain and then to the hypothalamus. In addition, the hypothalamus, because of its location adjacent to the third ventricle and median eminence, may sample factors from peripheral circulation, including glucose, as well as hormones such as insulin and leptin. Although a complex network of glucose sensors has been described in the central nervous system and peripherally, the brain appears to have the dominant role during hypoglycemia and, specifically, the ventromedial region of the hypothalamus (VMH). VMH neurons contain the same glucose-sensing mechanisms (e.g., glucokinase, ATP-sensitive K+ channels) as found in pancreatic β-cells.

CLINICAL MANIFESTATIONS

Symptoms of Hypoglycemia

The symptoms of hypoglycemia reflect the responses of the brain to a decrease in the blood glucose level; such symptoms may be nonspecific and

vague, especially in the childhood period. Children may not be able to communicate their hypoglycemic symptoms. The symptoms of hypoglycemia may be categorized into two main groups: (1) those that arise as a result of the central nervous system being deprived of glucose (neuroglycopenic) and (2) symptoms arising from the perception of physiologic changes caused by the central nervous system–mediated sympatho-adrenal discharge triggered by hypoglycemia (neurogenic or autonomic).4 The neurogenic symptoms of hypoglycemia are largely the result of sympathetic neural, rather than adrenomedullary, activation. Neuroglycopenic symptoms (e.g., dizziness, confusion, tiredness, difficulty with speaking, headache, inability to concentrate, coma, and seizures) arise from the failure of brain function itself and are caused by deficient supply of glucose to the brain.5 Neurogenic symptoms include both adrenergic responses (catecholamine-mediated symptoms such as palpitations, tremor, and anxiety) and cholinergic responses (acetylcholine-mediated symptoms such as sweating, hunger, paresthesias). Awareness of hypoglycemia chiefly depends on perception of the central and peripheral effects of neurogenic (as opposed to neuroglycopenic) responses to hypoglycemia. In nondiabetic adults during acute insulin-induced hypoglycemia, autonomic symptoms become apparent at a threshold of approximately 60 mg/ dL (3.3 mmol/L), and impairment of brain function manifested by neuroglycopenic symptoms occurs at a threshold of approximately 50 mg/dL (2.8 mmol/L) in arterialized venous blood (venous levels would be ≈ 3 mg/dL [0.16 mmol/L] less). However, in patients with recurrent hypoglycemia, the glycemic thresholds for responses to hypoglycemia are reset at a lower plasma glucose concentration. The glucose thresholds for the activation of neuroglycopenic and autonomic symptoms in children are not as clearly defined as in adults. The symptoms and signs of hypoglycemia are not influenced by the rate of blood glucose decline in nondiabetic individuals.

Clinical Approach to the Patient with Hypoglycemia

A careful clinical history, description of symptoms, physical examination, and a systematic step-by-step approach are the cornerstones of establishing a diagnosis. The symptoms of hypoglycemia may be very nonspecific, hence any symptomatic child or adult must have the blood glucose level measured and documented. The relationship of a hypoglycemic episode to the most recent meal can be important diagnostically. Hypoglycemia occurring after a short fast (2 to 3 hours) may be suggestive of hyperinsulinism or glycogen storage disease. Hypoglycemia occurring after a long fast (12 to 14 hours) may suggest a disorder of gluconeogenesis. Postprandial hypoglycemia may indicate galactosemia, hereditary fructose intolerance, dumping syndrome, insulinoma, insulin autoimmune syndrome, and noninsulinoma pancreatogenous hypoglycemia syndrome. In both children and adults, a clear documentation of the medication history is important.

DIAGNOSIS

After the clinical history has been taken and the examination completed, a diagnostic cascade of appropriate tests is necessary. These may be guided in the context of the most common causes of hypoglycemia as listed in Table 230-1. The current adult recommendations6 state that evaluation and management of hypoglycemia should only be undertaken in patients in whom Whipple’s triad—symptoms, signs, or both consistent with hypoglycemia, a low plasma glucose concentration, and resolution of those symptoms or signs after the plasma glucose concentration is raised—is documented. However, this does not apply to children for the reasons discussed earlier.

Causes of Hypoglycemia

Hypoglycemia is more common in the childhood period than in adults and can be due to a large number of causes. Table 230-1 summarizes the differential diagnosis of hypoglycemia.

Hypoglycemia Due to Excess Production of Hormones

Inappropriate and excess production of certain hormones can lead to hypoglycemia. The two most common conditions associated with excess production of a hormone are hyperinsulinemic hypoglycemia (HH) and non–islet cell tumor hypoglycemia (NICTH) or IGF-2-oma (insulin-like growth factor–secreting tumor). Inappropriate production of insulin can either lead to fasting hypoglycemia or postprandial hypoglycemia.

CHAPTER 230  Hypoglycemia and Pancreatic Islet Cell Disorders  

1551

Hyperinsulinemic Hypoglycemia TABLE 230-1  DIFFERENTIAL DIAGNOSIS OF HYPOGLYCEMIA* HYPERINSULINEMIC HYPOGLYCEMIA (INCLUDING POSTPRANDIAL) Transient: infant of diabetic mother, perinatal asphyxia, Rhesus disease, intrauterine growth retardation, Beckwith-Wiedemann syndrome Congenital: ABCC8, KCNJ11, GCK, GDH, HADH, HNF4A, SLC16A1 Dumping syndrome Insulin receptor mutations and antibodies Insulinoma Noninsulinoma pancreatogenous hypoglycemia (adults) Gastric bypass surgery for morbid obesity Non–islet cell tumor hypoglycemia (NICTH) or IGF-2-oma Insulin autoimmune syndrome Insulin factitious hypoglycemia HORMONAL DEFICIENCY/RESISTANCE Adrenocorticotropic hormone Cortisol Growth hormone Glucagon† Adrenaline†

HH is a heterogeneous group of disorders characterized by unregulated insulin secretion from pancreatic β-cells. In the face of hypoglycemia, patients have inappropriately detectable serum insulin levels, low ketone bodies, and low fatty acids and show a glycemic response to glucagon.7 Congenital Forms of Hyperinsulinemic Hypoglycemia

In patients with congenital forms of HH, mutations in the key genes (ABCC8, KCNJ11, GLUD1, GCK, HADH, SLC16A1, HNF4A, HNF1A, and UCP2) regulating insulin secretion have been identified.8 Children with inactivating mutations in the genes ABCC8 and KCNJ11 present with the most severe forms of congenital HH, typically in the newborn period. Hyperinsulinismhyperammonemia syndrome due to activating mutations in the GLUD1 gene and activating mutations in the GCK gene, leading to HH, have both been described in adults as well as children. Exercise-induced HH due to activating mutations in the SLC16A1 gene has also been recognized in adults. Insulinoma

Carnitine deficiency (primary and secondary) Carnitine palmitoyltransferase deficiency (CPT 1 and 2) Carnitine transporter defects

An insulinoma is the commonest cause of endogenous HH in adults. Insulinomas have the highest incidence in the fifth and sixth decades.9 Insulinomas are insulin-secreting tumors of pancreatic origin, with an incidence of 1 to 4 per million. The majority (90%) of them are benign, solitary, intrapancreatic and less than 2 cm in diameter. Classically, symptoms become evident in the fasting state or following exercise. However, it is now known that insulinoma can also present with postprandial symptoms. Diagnosis is based on findings of abnormal serum levels of insulin and C-peptide (also proinsulin) at the time of fasting hypoglycemia. An insulinoma can occur either in isolation or in association with multiple endocrine neoplasia type 1 (MEN 1), with a lifetime prevalence of 10% among adults carrying mutations in MEN1 (Chapter 231). Around 6% of insulinomas occur in patients with MEN 1, and most insulinomas are benign, but 5 to 10% are malignant.

FATTY ACID OXIDATION

Postprandial Hyperinsulinemic Hypoglycemia

DEFECTS IN HEPATIC GLYCOGEN RELEASE/STORAGE Glycogen storage diseases: glucose-6-phosphatase, amylo-1,6-glucosidase deficiency, liver phosphorylase deficiency, glycogen storage disease type 0 DEFECTS IN GLUCONEOGENESIS Fructose-1,6-bisphosphatase deficiency, phosphoenolpyruvate carboxykinase deficiency, pyruvate carboxylase deficiency CARNITINE METABOLISM

Medium-chain acyl-CoA dehydrogenase (MCAD) deficiency Very long-chain acyl-CoA dehydrogenase (VLCAD) deficiency Short-chain acyl-CoA dehydrogenase (SCAD) deficiency Long/short-chain L-3-hydroxyacyl-CoA (L/SCHAD) deficiency DEFECTS IN KETONE BODY SYNTHESIS/UTILIZATION HMG-CoA synthase deficiency, HMG-CoA lyase deficiency Succinyl-CoA: 3-oxoacid-CoA transferase (SCOT) deficiency METABOLIC CONDITIONS (COMMON ONES) Organic acidemias (propionic, methylmalonic) Maple syrup urine disease, galactosemia, fructosemia, tyrosinemia Hereditary fructose intolerance Mitochondrial respiratory chain complex deficiencies Congenital disorders of glycosylation (CGD) DRUG INDUCED Sulfonylureas Insulin β-Blockers Salicylates Alcohol Quinine Haloperidol Pentamidine Levofloxacin Methadone Disopyramide Indomethacin Cibenzoline Gatifloxacin MISCELLANEOUS CAUSES (MECHANISM[S] NOT CLEAR) Idiopathic ketotic hypoglycemia (diagnosis of exclusion) Infections (sepsis, malaria), congenital heart disease *Boldface indicates more common in adults. † No human case yet reported with glucagon or adrenaline deficiency. HMG = 3-hydroxy-3-methylglutaryl; IGF = insulin-like growth factor.

Postprandial hyperinsulinemic hypoglycemia (PPHH) refers to hypoglycemia within a few hours of meal ingestion, secondary to inappropriate insulin secretion in response to a meal. If PPHH is clinically suspected, then an oral glucose tolerance test (OGTT) or a mixed-meal provocation test is performed. (See later in section “Investigations for Hypoglycemia.”) A physiologic dip in the blood glucose level seen in OGTT might lead to misdiagnosis. However, corresponding biochemical evidence of endogenous HH and symptoms of neuroglycopenia during a hypoglycemic episode would help distinguish between pathologic PPHH and reactive hypoglycemia. A decrease of more than 108 mg/dL (6 mmol/L) between peak and nadir blood glucose during OGTT has been used as a diagnostic criterion for dumping syndrome in adults. dumping syndrome.  Dumping syndrome seen in infants after Nissen fundoplication is a classic example of PPHH. Precipitous emptying of hyperosmolar carbohydrate-containing solutions into the small bowel results in rapid glucose absorption, hyperglycemia, and reactive hypoglycemia. These children also tend to have abnormally exaggerated secretion of glucagon-like peptide-1 (GLP-1), which may contribute to the exaggerated insulin surge and resultant hypoglycemia.10 insulin autoimmune syndrome.  Insulin autoimmune syndrome, or Hirata’s disease, is a rare condition characterized by HH associated with high titers of antibodies to endogenous insulin in the absence of pathologic abnormalities of pancreatic islets and prior exposure to exogenous insulin. The disease is extremely uncommon in Western countries. Insulin autoimmune syndrome affects men and women equally and is seen more frequently in patients older than 40 years. The binding kinetics of endogenous insulin by the antibodies are thought to lead to physiologically inappropriate levels of bioavailable insulin, causing either hyper- or hypoglycemia. In this syndrome, the insulin levels are markedly elevated, usually above 100 mU/L. After a meal or glucose load, these patients often demonstrate initial hyperglycemia, followed by hypoglycemia a few hours later. The hyperglycemia is caused by the anti-insulin antibodies that bind the insulin secreted in response to rising blood glucose levels after a meal. This binding reduces the bioavailability of the secreted insulin to the receptors in the liver and peripheral tissues, resulting in hyperglycemia and further insulin secretion. As blood glucose concentrations begin to decrease and insulin secretion declines, the insulin bound to the antibodies is released, resulting in inappropriately high free insulin concentrations for the blood glucose, causing hypoglycemia.

1552

CHAPTER 230  Hypoglycemia and Pancreatic Islet Cell Disorders  

pphh in patients with insulin-receptor mutations.  PPHH has been described in patients who carried a heterozygote mutation (Arg1174Gln) in the insulin-receptor gene. Hyperinsulinism seems to be associated with decreased degradation rather than increased secretion of insulin, as evidenced by increased fasting levels of serum insulin despite normal levels of serum C-peptide and reduced clearance of exogenous insulin during clamp studies. pphh after gastric bypass surgery.  A consequence of the obesity epidemic is the increasing use of gastric bypass surgery for patients with severe, medically complicated obesity (Chapter 220), which has led to a number of reports of postprandial HH.11 In a review of the Swedish Bariatric Surgery registry, the incidence of hospitalization for hypoglycemia in post–gastric bypass patients was reported as less than 1%. A number of different explanations have been suggested to explain hypoglycemia post gastric bypass surgery. This can either be a manifestation of dumping syndrome or improved insulin sensitivity following weight loss unmasking an underlying hyperinsulinemia syndrome. The hypoglycemia could also be due to an effect on the enteroinsular axis induced by the diversion of nutrients into the small intestine. The principal reason seems to be enhanced postprandial insulin secretion, thought to be due primarily to increased secretion of glucose-dependent insulinotropic polypeptide (GIP) and especially GLP-1. GLP-1 levels are now well documented to be increased two- to five-fold after gastric bypass. The elevations of incretins tend to be seen early, even as early as 2 days after gastric bypass, and levels may decline as substantial weight loss and normalization of insulin sensitivity occurs. In patients with PPHH, elevated levels of GIP and GLP-1 persist for years after surgery. Increased postprandial insulin secretion by incretins is mediated by islet cell hypertrophy and hyperplasia. Both GIP and GLP-1 have been implicated in increasing pancreatic β-cell mass in rodent models. GLP-1 regulates islet growth by inducing the expression of the transcription factor pancreaticduodenum homeobox-1 (PDX-1). Overexpression of IGF-2 and IGF-1 receptor alpha (IGF1Rα) have been found in pancreatic tissue removed from patients with persistent PPHH after gastric bypass surgery as compared to controls. These findings are suggestive of the role of growth factors in islet hyperfunction seen in post–gastric bypass patients. noninsulinoma pancreatogenous hypoglycemia syndrome.  Noninsulinoma pancreatogenous hypoglycemia syndrome (NIPHS) is characterized by postprandial neuroglycopenia in the presence of negative prolonged fasting tests and negative perioperative localization studies for insulinoma.12 However, in some patients the selective arterial calcium stimulation test is positive, with the histology of the resected pancreas showing nesidioblastosis. The underlying genetic basis of NIPHS is not known. These patients are negative for ABCC8/KCNJ11 mutations and show islet hypertrophy histologically (as observed in diffuse congenital HH). Immunohistologic studies of the resected pancreatic tissue have failed to show an increased rate of proliferation of β-cells or abnormal synthesis and/ or processing of either proinsulin or amylin. Neither has there been any evidence of overexpression of pancreatic differentiation factors, PDX-1, and Nkx-6.1, nor the calcium-sensing receptor (CaSR). Insulin Factitious Hypoglycemia

Hypoglycemia can also be induced pharmacologically, either intentionally as a diagnostic tool, accidentally as a complication of the treatment of diabetes mellitus, or as a consequence of poisoning either with insulin itself or with drugs (e.g., sulfonylureas) that stimulate insulin release. Whenever severe hypoglycemia occurs with documented hyperinsulinism, the possibility of Munchausen’s syndrome by proxy should be considered in children. The possibility of malicious administration of insulin or an oral sulfonylurea should always be suspected in cases of sudden onset of hypoglycemia in a previously healthy individual. In the case of insulin administration, the clue in the biochemistry will be a raised insulin level accompanied by normal C-peptide.

Non–Islet Cell Tumor Hypoglycemia, or IGF-2-oma

NICTH, or IGF-2-oma, denotes the syndrome of hypoglycemia produced by or associated with any neoplasm other than an insulinoma. These are usually tumors of mesenchymal and epithelial origin (including hepatomas, fibromas, and fibrosarcomas). The underlying mechanism of hypoglycemia in nearly all patients with this syndrome is overproduction of IGF-2 by the

tumor, which includes mature IGF-2 and incompletely processed forms of IGF-2, referred to collectively as “big” IGF-2.13 The elevated IGF-2–related peptides mimic the fasting hypoglycemia characteristic of patients with insulin-producing islet cell tumors. Rarely, markedly elevated IGF-2 levels produce somatic changes suggestive of acromegaly. Typically, the elevated IGF-2 levels are associated with suppressed plasma levels of insulin, IGF-1, and GH.

Hypoglycemia Due to Hormone Deficiency

Deficiency of glucagon, adrenaline, GH, and cortisol can cause hypoglycemia. Glucagon and adrenaline deficiency is extremely rare, and so far no true human, genetically proven defects in glucagon and adrenaline deficiency have been described. Children and adults can present with hypoglycemia due to deficiency of various hormones. This might be either in isolation (e.g., isolated GH, adrenocorticotropic hormone [ACTH], or cortisol deficiency) or in combination with other hormones, such as in patients with hypopituitarism. The etiology of the hypoglycemia resulting from cortisol and GH deficiency is due to a combination of factors, including reduced gluconeogenic substrate availability (decreased mobilization of fats and proteins) and increased glucose utilization due to increased insulin sensitivity of tissues in the absence of these two hormones. Acquired hypopituitarism may result from tumors (most commonly craniopharyngioma), radiation, infection, hydrocephalus, vascular anomalies, and trauma. Addison disease (AD) results from adrenal cortex hypofunction/ dysfunction, with deficient production of glucocorticoids, mineralocorticoids, and androgens, and with high levels of both ACTH and plasma renin activity (Chapter 227). Autoimmune AD is the most frequent etiologic form in adult patients, accounting for about 80% of cases, followed by posttuberculosis AD in 10 to 15%; the remaining 5% of cases are due to vascular, neoplastic, or rare genetic forms. The markers of autoimmune AD are adrenal cortex (ACA) or 21-hydroxylase autoantibodies (21-OHAbs), and they are present at diagnosis in more than 90% of cases. In autoimmune AD, the adrenal cortex is infiltrated by lymphocytes and plasma cells, and the glands are sclerotic and reduced in volume. Autoimmune AD occurs mainly in middle-aged women, alone or associated with other (clinical, subclinical, or potential) autoimmune diseases, giving rise to various forms of autoimmune polyglandular syndrome. Replacement therapy with gluco- and mineralocorticoids is life-saving for patients with chronic adrenal insufficiency.

Hypoglycemia Due to Defects in Hepatic Glycogen Release/Storage

Glucose-6-phosphatase deficiency (glycogen storage disease [GSD] type I, Von Gierke disease) is the commonest of the glycogen storage diseases causing hypoglycemia (Chapter 207). Deficiency of this enzyme results in the inability to release free glucose from glucose-6-phosphate, with resultant hepatomegaly due to stored glycogen. These children and adults present with recurrent hypoglycemia associated with lactic acidosis, hyperuricemia, and hyperlipidemia.14 The two other glycogen storage diseases causing hypoglycemia are due to deficiencies of the enzymes amylo-1,6-glucosidase (GSD type III) and liver phosphorylase (GSD type VI). The clinical and biochemical features of GSD-III subjects are quite heterogeneous.

Hypoglycemia Due to Defects in Gluconeogenesis

Gluconeogenesis, or the formation of glucose from mainly lactate/pyruvate, glycerol, glutamine, and alanine, plays an essential role in the maintenance of normoglycemia during fasting. Inborn deficiencies are known in each of the four enzymes of the glycolytic-gluconeogenic pathway that ensure a unidirectional flux from pyruvate to glucose: pyruvate carboxylase, phosphoenolpyruvate carboxykinase (PEPCK), fructose-1,6-bisphosphatase, and glucose-6-phosphatase. Gluconeogenesis can essentially be viewed as a reversal of glycolysis but with a few important differences. Patients with defects in gluconeogenesis present with fasting hypoglycemia and lactic acidosis. Pyruvate carboxylase deficiency may lead to a more widespread clinical presentation, with lactic acidosis, severe mental and developmental retardation, and proximal renal tubular acidosis.

Hypoglycemia Due to Disorders of Carnitine Metabolism and Defects of Fatty Acid Oxidation

Serious clinical consequences may occur if fatty acid oxidation (FAO) is impaired, including hypoglycemic seizures, muscle damage, cardiomyopathy, metabolic acidosis, and liver dysfunction. Fatty acids are taken up by

1553

CHAPTER 230  Hypoglycemia and Pancreatic Islet Cell Disorders  

hepatocytes and muscle, where they are subsequently activated to their coenzyme A (CoA) esters. FAO disorders are individually rare, but they are collectively common because of the number of different enzymes affected. When defects occur in fatty acid degradation, excess acylcarnitine intermediates accumulate in the tissues, including heart, liver, and skeletal muscle, which can lead to organ dysfunction. The diversion of acyl-CoA intermediates into β-oxidation results in accumulation of toxic dicarboxylic acids. Acylcarnitines that spill into the blood provide a marker for diagnosis. Primary carnitine deficiency is an autosomal recessive disorder of fatty acid oxidation that can present at different ages with hypoketotic hypoglycemia and cardiomyopathy and/or skeletal myopathy (Chapter 205). This disease is suspected based on reduced levels of carnitine in plasma and confirmed by measurement of carnitine transport in the patient’s fibroblasts. Carnitine transport is markedly reduced (usually < 5% of normal) in fibroblasts from patients with primary carnitine deficiency. Patients with the hepatic isoform of carnitine palmityltransferase (CPT)-1 deficiency present with hypoketotic hypoglycemia in the neonatal period. The commonest disorder of fatty acid β-oxidation is medium-chain acylCoA dehydrogenase (MCAD) deficiency, an autosomal recessive disease presenting in children who are typically asymptomatic except during times of fasting and metabolic stress, usually associated with a viral illness, when they present with fasting nonketotic hypoglycemia; if undiagnosed, 20 to 25% of affected patients will die during the first episode.

Metabolic Diseases

Hypoglycemia can also be due to a number of metabolic conditions (Chapter 205), including galactosemia, fructosemia, tyrosinemia, organic acidemias, maple syrup urine disease, glutaric aciduria type II, and in mitochondrial respiratory chain defects. Hereditary fructose intolerance, caused by catalytic deficiency of aldolase B (fructose-1,6-phosphate aldolase), is a recessively inherited condition in which affected homozygotes develop hypoglycemia and severe abdominal symptoms after taking foods containing fructose and cognate sugars. Continued ingestion of noxious sugars leads to hepatic and renal injury and growth retardation.

Noninsulinoma Islet Cell Tumors

Islet cell tumors present an important challenge to the clinician because of their protean manifestations and potential lethality. These tumors can be clinically silent or active (functioning). Early diagnosis is essential and depends on recognition of the classic and variant clinical syndromes followed by confirmation of elevated peptide levels by radioimmunoassay.15 Glucagonoma, gastrinoma, VIPoma (VIP = vasoactive intestinal peptide), somatostatinoma, and ACTHoma are functioning tumors that may occur in isolation but can also be part of MEN 1 syndrome (Chapter 231) and von Hippel-Lindau disease (Chapter 417). Tumor marker measurement gives useful information for the follow-up and management of patients with noninsulinoma islet cell tumors (neuroendocrine tumors). The currently used tumor markers are neuron-specific enolase (NSE) and chromogranin A (CgA). The clinical accuracy of these biomarkers depends on histotype and disease extent. CgA is thought to be the optimal marker for most neuroendocrine tumors, because it is independent of the biological characteristics of the tumor.

Glucagonoma

Glucagonomas are α-cell tumors that, when they are active, produce a syndrome characterized by necrolytic migratory erythema, diabetes mellitus, weight loss, anemia, glossitis, thromboembolism, neuropsychiatric disturbances, and hyperglucagonemia. Tumor characterization is made by computed tomography (CT) and/or pancreatic endoscopic ultrasonic and indium-labeled octreoscan. The diagnosis is established by documenting the presence of hyperglucagonemia, with diagnostic levels being generally above 500 pg/mL (normal, 1.5 mL

  Morphology   Vitality (live)

>15 million/mL >39 million/ejaculate >40% motile >32% progressively motile >4% normal† >58%

Leukocytes

30-40 mIU/mL) Gonadal dysgenesis With stigmata of Turner syndrome Pure (46,XX or 46,XY) Mixed Ovarian failure with normal ovarian development Genetic disorders Autoimmune disorders Gonadotropin receptor or postreceptor defects (resistant ovary or Savage’s syndrome?) Enzymatic defects (17α-hydroxylase deficiency, galactosemia) Physical causes Irradiation Chemotherapeutic agents Viral agents Idiopathic

Hypogonadotropic or normogonadotropic hypogonadism (LH and FSH 30 mIU/mL) imply ovarian failure and require further evaluation. Incipient ovarian failure should be considered in any woman with basal FSH levels of 15 mIU/mL or higher other than during the midcycle LH surge. Many clinicians believe that chromosomal evaluation is indicated in all individuals with elevated FSH levels before age 40 years, and it is certainly indicated if hypergonadotropic amenorrhea begins before age 30 years. If FSH levels are low or normal, the measurement of total testosterone levels may be helpful whether or not there is any evidence of hirsutism or virilization. Hyperandrogenic women need not be hirsute because some have relative insensitivity of the hair follicles to androgens. Mildly increased levels of testosterone (and perhaps dehydroepiandrosterone sulfate as well) suggest polycystic ovary syndrome (PCOS). However, total circulating androgen levels need not be elevated because of the alterations in metabolic clearance rate and sex hormone–binding globulin that are present in PCOS. Consequently, some clinicians prefer to measure circulating free testosterone levels. Circulating levels of LH and FSH may aid in differentiation of PCOS from hypothalamic-pituitary dysfunction. LH levels are often elevated in PCOS so that the ratio of LH to FSH is increased; however, LH levels may be identical to those observed in normal women in the follicular phase. In contrast, levels of LH and FSH are normal or slightly reduced in hypothalamicpituitary dysfunction. There is some overlap between women with “polycystic ovarian–like” disorders and those with hypothalamic-pituitary dysfunction. Radiographic assessment of the sella turcica is indicated in all amenorrheic women in whom both LH and FSH levels are consistently low (both 200 ng/dL or DS > 7.0 µg/mL

Normal or mildly T DS 5.0-7.0 µg/mL

Evaluate for neoplasm

Evaluate for “adult-onset” CAH

Normal or mildly T and/or DS (normal or LH)

Mildly T (normal or LH)

PCOlike

Normal T (normal or LH)

HCA

Evaluate for “adult-onset” CAH FIGURE 236-3.  Biochemical evaluation of amenorrhea. This schema must be considered an adjunct to the clinical evaluation of the patient. See text for details. CAH = congenital adrenal hyperplasia; DS = dehydroepiandrosterone sulfate; FSH = follicle-stimulating hormone; HCA = hypothalamic chronic anovulation; LH = luteinizing hormone; PCO-like = polycystic ovarian syndrome–like; PRL = prolactin; T = testosterone; TSH = thyroid-stimulating hormone.

mutations, and complex multifactorial polygenic inheritance), physical and environmental causes, and autoimmune disturbances. In addition, there may be families in which menopause begins earlier than the expected age without any further pathologic cause.

Genetic Abnormalities

Several pathologic conditions with dysgenetic gonads involve elevated gonadotropin levels and amenorrhea as well as abnormalities of the X chromosome. The term gonadal dysgenesis refers to individuals with undifferentiated streak gonads without any association with either extragonadal stigmata or sex chromosome aberrations. Because individuals with gonadal dysgenesis have the normal complement of oocytes at 20 weeks of fetal age but virtually none by birth, this disorder is a form of premature ovarian failure.

Turner’s Syndrome

Turner’s syndrome (also see Chapter 233) describes patients with streak gonads composed of fibrous stroma and four cardinal features: a female phenotype; sexual infantilism; short stature; and several physical abnormalities, sometimes including webbed neck, low-set ears, multiple pigmented nevi, double eyelashes, micrognathia, epicanthal folds, shieldlike chest with microthelia, short fourth metacarpals, increased carrying angle of the arms, and certain renal and cardiovascular defects (most commonly coarctation of the aorta and aortic stenosis) (Fig. 236-4).11 The diagnosis can sometimes be made at birth because of unexplained lymphedema of the hands and feet. The syndrome is associated with an abnormality of sex chromosome number, morphology, or both. Most commonly, the second sex chromosome is absent (45,X). Turner’s syndrome is the single most common chromosome disorder in humans, but more than 95% of such fetuses are aborted, and the incidence in newborns is approximately 1 in 3000 to 5000. Chromosome breakage and mosaicism occur as well. In mosaic individuals with a normal 46,XX cell line, sufficient follicles may persist postnatally to initiate pubertal changes and to cause ovulation so that pregnancy is possible. Deletions of the X-chromosomelinked SHOX gene explain many of the dysmorphic skeletal features that are present, including the short stature. It is believed that the number of phenotypic findings may be related to the percentage of cells that are 45,X. There also may be an effect of imprinting with the variation in phenotype partly explained by the parental origin of the one remaining X chromosome.

Pure Gonadal Dysgenesis

Pure gonadal dysgenesis is the term given to phenotypically female individuals with streak gonads who are of normal stature and have none of the physical stigmata associated with Turner’s syndrome. Such individuals have either a 46,XX or 46,XY karyotype. The 46,XX defect may be inherited as an autosomal recessive, with 10% having associated nerve deafness. The 46,XY defect may be inherited as an X-linked recessive, with clitorimegaly occurring in 10 to 15% and gonadal tumors developing in 25% if the gonads are not removed.

Mutations in the X Chromosome Associated with Premature Ovarian Failure

Several regions of the X chromosome are now recognized to contain mutations in genes that may result in premature ovarian failure. Of particular note is the fragile X mental retardation (FMR1) gene. More than 5% of women with 46,XX spontaneous premature ovarian failure have mutations of the FMR1 gene. This risk is increased if there is a family history of premature ovarian failure. A family history of fragile X syndrome, unexplained mental retardation, dementia, developmental delay of a child, or tremor-ataxia syndrome is reason for genetic counseling. Mutations in the FMR1 gene are known to be associated with a neurodegenerative disorder. Women with mutations in the FMR1 gene are at risk for having a child with mental retardation, should they be one of the 6 to 8% of women with premature ovarian failure who conceive spontaneously. For FMR1, a CGG repeat sequence occurs, with up to 60 repeats being normal. Expansion to more than 200 repeats leads to the fragile X syndrome, with the high level of repeats causing hypermethylation of the gene promoter and silencing of the gene. Female carriers of the permutation have an unstable intermediate number of repeats (i.e., 60 to 199) and the predisposition for premature ovarian failure.

Trisomy X

Trisomy X (46,XXX karyotype) is also associated with premature menopause, although many such individuals have normal reproductive lives. Premature menopause can also occur in mosaic individuals with cell lines with excess X chromosomes. When gonadal abnormalities occur in women with excess X chromosomes, they seem to occur after ovarian differentiation so that some ovarian function is possible. Only later in life do such women develop secondary amenorrhea and premature ovarian failure.

CHAPTER 236  Reproductive Endocrinology and Infertility  

1591

(p450arom) enzymes may also lead to ovarian failure. Women with galactosemia also experience ovarian failure early in life, even when a galactoserestricted diet is introduced early in infancy. Mutations of several autosomal genes result in premature ovarian failure. Included in this growing list are mutations involving FSHR (the FSH receptor gene), FOXL2 (a forkhead transcription factor associated with the blepharophimosis-ptosis-epicanthus inversus syndrome), INHA (the inhibin-α gene), E1F2B (a family of genes associated with central nervous system leukodystrophy and ovarian failure), PMM2 (the gene for phos­phomannomutase), GALT (the gene for galactose-1-phosphate uridyltransferase), and AIRE (leading to the autoimmune polyendocrinopathycandidiasis-ectodermal dystrophy syndrome). Myotonic dystrophy (Chapter 421) is caused by an autosomal triple repeat mutation, like the fragile X syndrome, that is similarly associated with premature loss of germ cells from the ovary. The list of mutations associated with early ovarian failure continues to increase as the function of more genes is determined.

Mutations Involving Reproductive Hormones, Their Receptors, and Action

The resistant ovary (Savage’s) syndrome occurs in young amenorrheic women who have elevated peripheral gonadotropin concentrations, normal (although immature) follicles present on ovarian biopsy, 46,XX karyotype with no evidence of mosaicism, fully developed secondary sex characteristics, and ovarian resistance to stimulation with human menopausal or pituitary gonadotropins. At least some of these women have mutations in the FSH receptor. It is probably inappropriate to use the term “resistant ovary syndrome” because it is likely that this is a heterogeneous disorder due to various genetic mutations.

A

Other Causes Physical and Environmental

Irradiation and chemotherapeutic agents used to treat various malignant diseases may also cause premature ovarian failure. Ovulation and cyclic menses return in some of these patients even after prolonged intervals of hypergonadotropic amenorrhea associated with signs and symptoms of profound hypoestrogenism. In general, the younger the individual at the time of treatment, the less likely is she to have permanent ovarian failure after the completion of therapy. Rarely, mumps affects the ovaries and causes ovarian failure.

Autoimmune Disorders

Premature ovarian failure may occur in conjunction with a variety of autoimmune disorders. The most well-known syndrome (autoimmune polyglandular syndrome type 1) involves hypoadrenalism, hypoparathyroidism, and mucocutaneous candidiasis together with ovarian failure (Chapter 231). Testing for adrenal antibodies by indirect immunofluorescence will identify the 4% of women with spontaneous premature ovarian failure who have steroidogenic cell autoimmunity and are at risk for adrenal insufficiency. Thyroiditis is the most commonly associated abnormality. Antibodies to the FSH receptor have been identified in a few cases. These associations make it mandatory to rule out other potentially life-threatening endocrinopathies in young women with hypergonadotropic amenorrhea.

TREATMENT 

B FIGURE 236-4.  Adult with Turner’s syndrome. This woman was seen at age 56 years by the author of the chapter (Dr. Rebar), and was case number 2, an adolescent at that time, in the original publication of Dr. Henry Turner describing the syndrome.

Known Genetic Alterations of Specific Genes

In girls with the rare syndrome of 17α-hydroxylase deficiency involving p450c17 who survive until the expected age of puberty, sexual infantilism and primary amenorrhea occur together with elevated levels of gonadotropins (also see Chapter 233). Defects in the 20,22-lyase (p450scc) or aromatase

Women with hypergonadotropic amenorrhea and ovarian failure should be treated identically whether or not they have signs of hypoestrogenism or desire pregnancy. Counseling and psychological support are indicated in women in whom the diagnosis of premature ovarian failure is made. Ovarian biopsy is not indicated to document the existence of follicles because only a small portion of each ovary can be sampled and because pregnancies have resulted in patients who had biopsy samples devoid of follicles. Estrogen replacement is warranted to prevent the accelerated bone loss known to occur in affected women (Chapter 243). The estrogen should be given sequentially with a progestin to prevent endometrial hyperplasia. Young women with ovarian failure may require twice as much estrogen as postmenopausal women for relief of signs and symptoms of hypoestrogenism. Inexplicably, women with premature ovarian failure may conceive while taking exogenous estrogen, even in the form of oral contraceptive agents, at the same rate as those not taking estrogen, so barrier contraception should be discussed if pregnancy is not desired. Women with hypergonadotropic amenorrhea are rarely able to become pregnant. It is not clear why pregnancy may rarely occur in such women, but

1592

CHAPTER 236  Reproductive Endocrinology and Infertility  

the pregnancy and delivery rate is 6 to 8%. Infertility treatment of young women with hypergonadotropic amenorrhea involves hormone replacement to mimic the normal menstrual cycle and embryo transfer by use of donor oocytes. Whether women with gonadal dysgenesis should be offered pregnancy by use of donor oocytes is now the subject of debate because a markedly increased incidence of aortic rupture during pregnancy secondary to medial necrosis has been documented. Women with Turner’s syndrome contemplating pregnancy should be counseled regarding the risks. The coordination of health care of adult women with Turner’s syndrome often falls to the endocrinologist because many of the complications of the disease are endocrinologic: hypothyroidism, diabetes, hypertension, obesity, osteoporosis, and hypogonadism. However, guidelines have been published about the surveillance of other multisystem conditions for which Turner’s syndrome patients are at risk, including significant psychosocial problems, congenital heart disease, deafness, and gastrointestinal and hepatic disorders.

CHRONIC ANOVULATION

Chronic anovulation, the most frequent form of amenorrhea encountered in women of reproductive age, implies that functional ovarian follicles remain and that cyclic ovulation can be induced with appropriate therapy (Table 236-4). The cause of the anovulation should be determined. The pathophysiologic bases for several forms of anovulation are unknown, but the anovulation can be interrupted transiently by nonspecific induction of ovulation in most affected women. Anovulation can result in either amenorrhea or irregular (generally less frequent) menses.

Hypothalamic Chronic Anovulation

DEFINITION

Hypothalamic chronic anovulation (HCA) represents a heterogeneous group of disorders with similar manifestations. Emotional and physical stress,

TABLE 236-4  CAUSES OF CHRONIC ANOVULATION Chronic anovulation of hypothalamic-pituitary origin Hypothalamic chronic anovulation Psychogenic Exercise associated Associated with diet, weight loss, or malnutrition Anorexia nervosa and bulimia Pseudocyesis Forms of isolated (idiopathic) hypogonadotropic hypogonadism (including Kallmann’s syndrome) Due to hypothalamic-pituitary damage Pituitary and parapituitary tumors Empty sella syndrome Following surgery Following irradiation Following trauma Following infection Following infarction Idiopathic hypopituitarism Hypothalamic-pituitary dysfunction or failure with hyperprolactinemia (multiple causes) Due to systemic diseases Chronic anovulation due to inappropriate feedback (i.e., polycystic ovary syndrome) Excessive extraglandular estrogen production (i.e., obesity) Abnormal buffering involving sex hormone–binding globulin (including liver disease) Functional androgen excess (adrenal or ovarian) Neoplasms producing androgens or estrogens Neoplasms producing chorionic gonadotropin Chronic anovulation due to other endocrine and metabolic disorders Adrenal hyperfunction Cushing’s syndrome Congenital adrenal hyperplasia (female pseudohermaphroditism) Thyroid dysfunction Hyperthyroidism Hypothyroidism Prolactin or growth hormone excess Hypothalamic dysfunction Pituitary dysfunction (microadenomas and macroadenomas) Drug induced Malnutrition

excessive exercise, nutritional deficiencies, weight loss, reduced body fat, and other unrecognized factors may contribute in varying proportions to the anovulation. Women with HCA have normal neuroanatomic findings.

ANOREXIA NERVOSA

Individuals with amenorrhea and significant weight loss should be examined for the possibility of anorexia nervosa (Chapter 219).

ISOLATED HYPOGONADOTROPIC HYPOGONADISM

Affected individuals have absence of spontaneous pubertal development. Most have functional GnRH deficiency, but some have abnormalities of gonadotropin deficiency localized to the pituitary gland. Kallmann’s syndrome is a familial disorder consisting of gonadotropin deficiency, anosmia or hyposmia, and color blindness in men or, more rarely, in women (Chapter 223). Partial or complete agenesis of the olfactory bulb is present on autopsy, accounting for use of the term olfactogenital dysplasia. Isolated gonadotropin deficiency in the absence of anosmia occurs as well. Sexual infantilism with a eunuchoid habitus is the clinical hallmark of this disorder, but moderate breast development may occur. Circulating LH and FSH levels are low but almost always detectable. Mutations in KAL1, FGFR1, FGF8, PROK2, ROKR2, HS6ST1, WDR11, or CHD7 have been identified in a minority of patients with Kallmann’s syndrome. Ovulation induction requires use of exogenous gonadotropins and HCG or pulsatile GnRH. Estrogen replacement therapy is indicated in these women until pregnancy is desired. It may not be possible to distinguish between partial isolated gonadotropin deficiency and functional HCA in all cases.

HYPOPITUITARISM

Hypopituitarism may be obvious on cursory inspection or sufficiently subtle to require endocrine testing (Chapter 224). The clinical presentation depends on the age at onset, the cause, and the nutritional status of the individual. Failure of development of secondary sex characteristics must always raise the question of hypopituitarism. Ovulation can be induced successfully with exogenous gonadotropins when pregnancy is desired and after the hypopituitarism is treated appropriately. Replacement therapy with estrogen is indicated.

HYPERPROLACTINEMIA

Galactorrhea associated with hyperprolactinemia, whatever the cause, almost always occurs together with amenorrhea caused by hypothalamic-pituitary dysfunction or failure. Many conditions can cause excess prolactin secretion (Chapter 224). A prolactinoma must be excluded. Hirsutism may be observed occasionally in association with amenorrhea-galactorrhea and hyperprolactinemia. Elevated levels of the adrenal androgens dehydroepiandrosterone and dehydroepiandrosterone sulfate may be observed and may account for the polycystic-type ovaries present in some hyperprolactinemic women.

FAILURE OF THE HYPOTHALAMIC-PITUITARY UNIT

The hypothalamic-pituitary unit may also fail to function normally in a number of stressful, debilitating, systemic illnesses that interfere with somatic growth and development. Chronic renal failure, liver disease, and diabetes mellitus are the most prominent examples.

DIAGNOSIS

Abrupt cessation of menses in women younger than 30 years who have no anatomic abnormalities of the hypothalamic-pituitary-ovarian axis and no other endocrine disturbances suggests a diagnosis of HCA. Affected individuals tend to be bright, educated, and engaged in intellectual occupations and may well give a history of psychosexual problems and socioenvironmental trauma. HCA is characterized by low to normal levels of gonadotropins and relative hypoestrogenism. Rarely, however, do affected women present with signs and symptoms of estrogen deficiency. It is important to rule out a central lesion as the cause of the hypogonadotropic hypogonadism in women who appear to have HCA.

TREATMENT  Psychological counseling or a change in lifestyle, especially for women engaged in strenuous exercise programs, may be effective in inducing cyclic ovulation and menses in women with functional HCA. Cognitive behavior therapy is effective in a proportion of women with functional HCA. For women

CHAPTER 236  Reproductive Endocrinology and Infertility  

1593

desiring pregnancy, ovulation can also be induced with clomiphene citrate (50 to 100 mg/day for 5 days beginning on the third to fifth day of withdrawal bleeding). Treatment with exogenous gonadotropins to induce follicular maturation followed by HCG to induce follicular rupture may be effective in women who do not ovulate in response to clomiphene. Because women with HCA have low circulating levels of leptin, investigators have given recombinant leptin and documented that ovulation may resume in some affected women. Given the heterogeneous nature of the disorder, it is not surprising that exogenous leptin is not effective in all women. Most physicians advocate the use of exogenous gonadal steroids to prevent osteoporosis. A regimen can consist of daily oral conjugated or esterified estrogens (0.625 to 1.25 mg), ethinyl estradiol (20 µg), or micronized estradiol-17β (1 to 2 mg) or transdermal estradiol-17β (0.05 to 0.10 mg) daily, with oral medroxyprogesterone acetate (5 to 10 mg) added for the first 12 to 14 days of each month. Sexually active women can be given oral contraceptive agents as an alternative. If steroid therapy is administered, patients must be informed that the amenorrhea will probably be present when therapy is discontinued. Other physicians believe that only periodic observation is indicated, with barrier methods of contraception recommended for fertility control. Adequate ingestion of calcium should be ensured regardless of therapy. Contraception is needed for sexually active women with HCA because the functional defect is mild in these disorders and may resolve spontaneously at any time, with ovulation occurring before any episode of menstruation.

Chronic Anovulation Related to Inappropriate Feedback

DEFINITION

PCOS is a heterogeneous disorder in which there is considerable clinical and biochemical variability among affected individuals. (See PCOS as the most common cause of heterosexual pubertal development in Chapter 235.) PCOS is currently considered to exist in women with any two of the following: (1) oligo-ovulation or anovulation, (2) hyperandrogenism, or (3) polycystic ovaries on ultrasound, and in whom other etiologies have been eliminated. PCOS is the classic disorder in which the amenorrhea or oligomenorrhea results from inappropriate feedback of gonadal steroids from the ovaries.

A

PATHOBIOLOGY

Current evidence suggests that the hypothalamic-pituitary unit is intact and that a functional derangement, perhaps involving insulin-like growth factors (IGFs) such as IGF-I within the ovary, results in abnormal gonadotropin secretion. PCOS is characterized by insulin resistance and compensatory hyperinsulinemia. (See association between PCOS, insulin resistance, and obesity in Chapter 220.) The insulin resistance has been found in affected women of many racial and ethnic groups, implying that it is a universal characteristic and that a common defect may be present. There is increasing evidence of specific genetic abnormalities in some women with PCOS.

CLINICAL MANIFESTATIONS

Although patients usually present with amenorrhea, hirsutism, and obesity, affected women may instead complain of irregular and profuse uterine bleeding, may not have hirsutism, and may be of normal weight (Fig. 236-5). Excess androgen from any source or increased extraglandular conversion of androgens to estrogens can lead to the typical findings of PCOS. Included are such diverse disorders as Cushing’s syndrome, mild congenital adrenal hyperplasia, virilizing tumors of adrenal or ovarian origin, hyperthyroidism and hypothyroidism, obesity, and primary PCOS with no other recognizable cause. In the primary syndrome, the irregular menses, mild obesity, and hirsutism begin during puberty and typically become more severe with time, although there is increasing evidence of improvement in the years just before menopause. Obesity alone can lead to a polycystic ovarian–like syndrome, with the degree of obesity required to cause anovulation varying widely. The increase in the prevalence of obesity is leading to an increased prevalence of PCOS. All such patients are well estrogenized regardless of whether they present with primary or secondary amenorrhea or dysfunctional bleeding. LH concentrations tend to be elevated, with relatively low and constant FSH levels, but both may be in the normal range for the follicular phase of the menstrual cycle. Levels of most circulating androgens, especially testosterone, tend to be mildly elevated.

DIAGNOSIS

After exclusion of other etiologies, two of the following three are required for diagnosis of PCOS: (1) hyperandrogenism (clinical or biochemical);

B FIGURE 236-5.  Adult with polycystic ovary syndrome (PCOS). This 28-year-old woman with documented PCOS had elevated luteinizing hormone levels, irregular menses, and hirsutism since puberty. Note the increased hair in the midline extending up to and above the umbilicus. Other findings (which are not necessarily abnormal) are periareolar hair and hypertrichosis of the arms.

(2) oligo-ovulation or anovulation; (3) polycystic ovaries on ultrasound examination or at surgery.12 This definition is confusing to clinicians because it implies that hirsute women with polycystic ovaries on ultrasound examination who ovulate regularly should be considered to have PCOS. Moreover, it is clear that

1594

CHAPTER 236  Reproductive Endocrinology and Infertility  

polycystic ovaries may be identified on ultrasound examination in normal women. In any case, the aim of the diagnostic evaluation is to rule out any causes (such as neoplasms) that require definitive therapy. Hirsutism should be evaluated as detailed in Chapter 442. A particularly severely affected subset of women present with marked obesity, anovulation, mild glucose intolerance with high levels of circulating insulin, acanthosis nigricans, hyperuricemia, severe hirsutism, and elevated circulating androgen levels. These women have hyperthecosis of the ovaries, in which the androgen-producing cells in the stromal, hilar, and thecal regions are increased greatly in number. Hyperthecosis should probably be viewed as a part of the spectrum of disorders constituting PCOS.

TREATMENT  Patients generally require therapy for hirsutism, for induction of ovulation if pregnancy is desired, and for prevention of estrogen-induced endometrial hyperplasia and cancer. No ideal therapy exists; the therapeutic approach must be individualized. The risks for metabolic syndrome, cardiovascular disease, and diabetes mellitus are increased in women with PCOS, at least in part because of the increased androgens and insulin resistance. Moreover, many women have elevated cholesterol levels.

Medical Therapy

In the anovulatory woman not desiring pregnancy who is not hirsute, therapy with intermittent progestin administration (e.g., medroxyprogesterone acetate, 5 to 10 mg orally for 10 to 14 days each month) or oral contraceptives can be provided to reduce the increased risk for endometrial carcinoma that is present in such a woman with unopposed estrogen. All women using intermittent progestin administration should be cautioned about the need for effective contraception if they are sexually active because these agents do not inhibit ovulation when they are administered intermittently. Improvements in insulin sensitivity in women with polycystic ovaries, either through lifestyle changes (i.e., exercise and diet) or through pharmacologic intervention, consistently result in improvements in the reproductive and metabolic abnormalities. Resumption of ovulation may occur in up to 60 to 70% of affected women. The longest and largest published experiences with any agent that improves insulin sensitivity in PCOS is with metformin, a biguanide that functions primarily by suppressing hepatic gluconeogenesis and also improves insulin sensitivity. A6  Its use in PCOS leads to reductions in insulin and androgen levels and resumption of menses in some women. Divided doses of 1500 to 2000 mg/ day have proved effective. Some clinicians advocate giving metformin to all women with polycystic ovaries, whereas others would administer such an agent only to those with documented insulin resistance. Some clinicians also advocate giving metformin first to women who desire pregnancy and then adding an agent to induce ovulation if the metformin proves ineffective. These agents are not approved for use in pregnant women or for the induction of ovulation.

Treatment Considering Pregnancy

Oral contraceptive agents are the first line of therapy for hirsute anovulatory woman not desiring pregnancy and offer protection from endometrial hyperplasia. In women with PCOS desiring pregnancy, clomiphene citrate or letrozole can be used to induce ovulation. A7  A8  Letrozole is not approved for this use by the FDA, but a large multicenter randomized trial has demonstrated its superiority to clomiphene in obese women with PCOS. A9  About 75 to 80% conceive with such therapy. In addition to insulin-sensitizing agents, other possible methods of inducing ovulation include use of exogenous gonadotropins and HCG, and laparoscopic ovarian surgery with multiple punctures of the ovary by diathermy or laser. A large clinical trial documented that clomiphene citrate is more effective than metformin in inducing ovulation and resulting in pregnancy; there was no further improvement when the two agents were used concurrently. A10  ,

Surgical Treatment

Laparoscopic ovarian surgery can achieve unifollicular ovulation or make it easier for medical ovulation induction but increases the risk for development of ovarian adhesions (themselves leading to infertility). It may be successful in a small subset of women with PCOS who are geographically removed from good medical care.

Chronic Anovulation Related to Other Endocrine and Metabolic Disorders Adrenal hyperfunction appears to cause chronic anovulation by inducing a polycystic ovarian-like syndrome secondary to increased adrenal androgen secretion. Both hyperthyroidism and hypothyroidism are associated with a

variety of menstrual disturbances, including dysfunctional uterine bleeding and amenorrhea as a result of alterations in the metabolism of androgens and estrogens. These metabolic changes in turn result in inappropriate steroid feedback and chronic anovulation.

Luteinized Unruptured Follicle Syndrome The luteinized unruptured follicle syndrome refers to the development of a dominant follicle without its subsequent disruption and release of the ovum. The abnormality can be diagnosed by ultrasonography or by the absence of evidence of ovulation when the ovary is viewed at laparoscopy. The disorder occurs infrequently and is not a significant cause of infertility. Menstrual cycles in which no ovum is released are characterized by presumptive evidence of ovulation, including biphasic basal body temperatures, secretory endometrium, normal LH surge, and normal progesterone production in the luteal phase.

Luteal Phase Dysfunction

PATHOBIOLOGY

Progesterone secretion in the luteal phase may be reduced in duration (termed luteal phase insufficiency) or in amount (termed luteal phase inadequacy). More rarely, the endometrium may be unable to respond to secreted progesterone because of the absence of progesterone receptors. These disorders represent causes of infertility (because of inability of fertilized ova to implant) in less than 5% of infertile couples. Abnormalities of the follicular phase, especially in the frequency of gonadotropin pulses, may account for most luteal phase defects. Luteal phase defects may also occur sporadically in normally ovulating women.

DIAGNOSIS

Luteal phase dysfunction may be associated with several clinical entities, including mild or intermittent hyperprolactinemia, strenuous physical exercise, inadequately treated 21-hydroxylase deficiency, and recurrent miscarriage. Luteal dysfunction occurs more commonly at the extremes of reproductive life and in the first menstrual cycles after full-term delivery, abortion, or discontinuation of oral contraceptives. It may also occur during ovulatory cycles induced with clomiphene citrate or exogenous gonadotropins and HCG.

TREATMENT  Treatment of luteal dysfunction is controversial. Any underlying defect should be treated. If subsequent luteal function depends on prior follicular development, modification of follicular development with either clomiphene citrate (25 to 100 mg daily by mouth for 5 days beginning on cycle day 3 to 5) or FSH (75 to 300 IU intramuscularly for 3 to 5 days beginning on cycle day 3 to 5) can be used; HCG (2500 to 5000 IU intramuscularly at 2- to 3-day intervals beginning with the shift in basal body temperature) or progesterone (12.5 mg intramuscularly in oil daily or 25 mg twice a day as rectal or vaginal suppositories) can be used as well. Bromocriptine may correct the abnormality in individuals with hyperprolactinemia. Synthetic progestational agents should not be used to treat luteal phase defects because of their possible association with congenital anomalies. Furthermore, the synthetic progestins produce an abnormal endometrium. None of these agents has been shown to increase the pregnancy rate.

INFERTILITY DEFINITION

The World Health Organization (WHO) has defined infertility as “a disease of the reproductive system defined by the failure to achieve a clinical pregnancy after 12 months or more of regular unprotected sexual intercourse.” Sterility is total inability to reproduce. More than 10% of couples in the United States seek medical assistance for infertility. The requirements for pregnancy to occur are several: • The male must produce adequate numbers of normal, motile spermatozoa. • The male must be capable of ejaculating the sperm through a patent ductal system. • The sperm must be able to traverse an unobstructed female reproductive tract. • The female must ovulate and release an ovum.

CHAPTER 236  Reproductive Endocrinology and Infertility  

TABLE 236-5  CAUSES OF INFERTILITY AND THEIR APPROXIMATE INCIDENCE (WHERE AVAILABLE)* Male factors (40%) Decreased production of spermatozoa Varicocele Testicular failure Endocrine disorders Cryptorchidism Stress, smoking, caffeine, nicotine, recreational drugs Ductal obstruction Epididymal (after infection) Congenital absence of vas deferens Ejaculatory duct (after infection) After vasectomy Inability to deliver sperm into vagina Ejaculatory disturbances Hypospadias Sexual problems (i.e., impotence), medical or psychological Abnormal semen Infection Abnormal volume Abnormal viscosity Immunologic factors Sperm-immobilizing antibodies Sperm-agglutinating antibodies Female factors Fallopian tube disease (20-30%) Pelvic inflammatory disease or puerperal infection Congenital anomalies Endometriosis Secondary to past peritonitis of nongenital origin Amenorrhea and anovulation (15%) Minor ovulatory disturbances (25% vertebral height loss) appear to better predict future fractures, as do nonvertebral fractures. When fractures are suspected, CT and MRI may be used, given that plain radiographs have a lower sensitivity acutely and with stress fractures. MRI also can be used to define a vertebral fracture with persistent swelling and edema based on T2 characteristics, potentially identifying patients who could benefit from vertebroplasty or kyphoplasty (see later). Finally, whole body bone scintigraphy is the most sensitive test for fracture but can be falsely positive because of inflammation, infection, or tumor and usually is positive for 6 to 12 months after a fracture event.

FIGURE 243-5.  Incidental vertebral compression fractures on chest radiograph. Lateral radiograph of the chest of a 74-year-old man studied for cough. No relevant pulmonary abnormality was noted on the frontal radiograph (not shown). Examination of the thoracic spine shows the presence of a mild anterior wedge compression fracture of T9 (thick arrow) and moderate anterior wedge fracture of T6 (thin arrow). Neither fracture was reported on the radiographic report.

DIAGNOSIS

1641

Although a recent fragility fracture is a reasonable basis for diagnosis of osteoporosis, other skeletal conditions should also be entertained, including inherited and acquired osteomalacias and pathologic fracture due to malignancy. These disorders can often be distinguished by history and physical examination, although additional investigations may be required. This distinction is critical because therapies may differ greatly between disorders. Most patients with osteoporosis are diagnosed on the basis of BMD measurement, generally by dual-energy x-ray absorptiometry (DXA). DXA is a low-radiation-based radiologic measurement of the areal bone density (g/ cm2) of the lumbar spine, proximal femur, and distal radius. Osteoporosis can be diagnosed if the BMD of a postmenopausal woman or man older than 50 years is more than 2.5 SD below young average normal (T score ≤ −2.5). A T score between −1.0 and −2.5 is considered low bone density or osteopenia, and a Z score (age-matched BMD) in premenopausal women and men younger than 50 years that is more than 2 SD below that of an a average age-matched individual is considered low bone density for age. BMD is an independent predictor of fracture risk, such that the relative risk for fracture increases by 1.5- to 2-fold for each 1 SD decrease in T score. In addition, fracture risk increases exponentially below a T score of −2.5. Furthermore, BMD of the femoral neck may be used in fracture prediction models such as FRAX to better define an individual’s risk for subsequent fracture (see later). In addition to DXA, other modalities are also used to diagnose osteoporosis, including quantitative CT of the spine (QCT) and wrist and tibia (pQCT), finger DXA, and ultrasound of the calcaneus or wrist. Measurement of BMD by all of these techniques has been shown to globally predict fractures, akin to DXA. QCT and pQCT provide additional information on cortical and trabecular bone compartments but are accompanied by higher radiation exposure and poorer reproducibility compared with DXA. Ultrasound is radiation free and easy to operate but is less sensitive in diagnosing osteoporosis and does not measure change in a reliable fashion in response to age or treatment, making it useful as a screening modality but not for longitudinal care. Although DXA is an effective diagnostic tool, several potential limitations and caveats need to be considered by the clinician. First, DXA cannot distinguish between low bone density and undermineralized bone matrix, the latter of which occurs in osteomalacia (Chapter 244). BMD may also be quite disparate between regions, perhaps in more than one third of individuals. This inconsistency results from a number of factors, including differences in bone composition (predominantly trabecular bone in the spine and cortical bone in the one-third radius) with resultant variations in rates of bone loss due to aging and disease (vertebral bone loss with menopause and glucocorticoid use versus cortical bone loss in hyperparathyroidism). Degenerative changes due to aging, such as facet osteoarthritis and aortic calcification, may artifactually raise spine BMD value. Given these considerations, the lowest skeletal site should be used for diagnosis. Finally, BMD should be measured longitudinally on the same DXA machine if possible, because of intermachine and intermanufacturer differences that may confound the ability to validly measure change over time. Despite these caveats, DXA remains the best method to diagnose and manage osteoporosis by bone density testing. Despite its utility, bone density has been limited historically in optimally predicting fracture risk in individual patients. In addition, BMD does not take into account clinical factors that independently predict fracture. Under this premise, fracture prediction models have been developed that combine BMD and risk factors to better stratify fracture risk. The best known and most widely used of these prediction models is FRAX. FRAX was developed by the World Health Organization in collaboration with national and international osteoporosis foundations as an Internet-based computer algorithm that defines a person’s 10-year risk for hip and major osteoporotic fracture (hip, clinical spine, forearm, and proximal humerus all combined). The model uses country-specific data on clinical risk factors and femoral neck BMD to calculate fracture probability and is available as a web-based tool that can be used by clinicians with their patients to assist in making informed decisions on osteoporosis management (http://www.shef.ac.uk/FRAX/). FRAX can be also be used to define country-specific recommended diagnostic and treatment thresholds. An example of this is the National Osteoporosis Foundation guidance that 10-year risks equal to or exceeding 3 and 20% for hip and major fracture risk, respectively, warrant consideration of pharmacologic treatment, which is based on cost-effective analyses in the United States. Furthermore, the number needed to treat (NNT) can be determined to best

1642

CHAPTER 243  Osteoporosis  

inform patients of their expected risk and benefit of treatment (e.g., bisphosphonate use roughly reduces hip fracture risk by half, or from 10 to 5%, with NNT of 1/0.05, or 20 patients treated to prevent one hip fracture). Despite its utility and ease of use, FRAX does have limitations. These include inability to use patients who are not treatment naïve, underestimation of fracture risk in those with disparately lower BMD in the spine than hip, absence of fall history/fall risk in the model, and the use of fixed clinical risk factors. Modifications have been suggested for the latest guidelines, including adjusting FRAX score up or down based on glucocorticoid dose. In addition, although fracture risk calculators that incorporate fall risk are available (e.g., from the Garvan Institute), they do not include the competing risk of mortality as FRAX does. As such, FRAX should be viewed, not as a perfect, but rather as a complementary tool to BMD in best defining a person’s risk for fracture and candidacy for pharmacologic intervention. Finally, all patients presenting with osteoporosis require an assessment for secondary causes of bone loss, given that 20 to 25% of women and perhaps an even greater portion of men will have identifiable additional etiologies that may contribute to bone loss (see Table 243-1). Most patients will have had routine chemistry, hematology, and thyroid studies as part of their annual examination. The 25(OH)D level should be measured in all patients for multiple reasons, as previously discussed. Additional investigations may also be considered, as directed by the clinical history and physical examination. In addition, a greater degree of BMD deficit (i.e., lower Z score) indicates a need for more extensive testing, given the greater likelihood of secondary causes being present. Bone turnover markers (BTMs) are serum and urinary products of bone formation or resorption that can also be used to assist in management. Available tests include bone-specific alkaline phosphatase, osteocalcin, type I procollagen amino-terminal propeptide, and type I procollagen carboxy-terminal propeptide as formation markers, and serum and urine C- and N-terminal peptides of type I collagen as resorption markers, among others. Their use is predicated on studies showing that high bone turnover increases fracture risk independent of BMD. In addition, fracture risk reduction correlates well with reduction in bone turnover based on clinical trials with anticatabolic agents. Nonetheless, their clinical utility has been tempered to date by several issues. First, there is significant biologic variability due to nonmodifiable (e.g., age, gender, underlying comorbid disease, medications) and modifiable (e.g., time of day, food intake, presence of fracture) factors that limit the ability to detect meaningful change over time in an individual patient. Second, optimal specimen processing is required for valid results and interpretation. Finally, and perhaps in part secondary to these issues and others, evidence to date does not demonstrate significant benefit of BTMs in individual patients in securely predicting bone density increase, fracture risk reduction, or cost effectiveness through patient feedback and improved adherence. Therefore, at present, BTMs should not be used in routine clinical practice, although they could help inform management in more complicated cases of metabolic bone disease.9

PREVENTION AND TREATMENT  Calcium

Adequate intake of calcium is critical to the optimal accumulation and maintenance of bone mineral density. Calcium supplementation has meaningful impact on BMD and fracture risk reduction, although the latter is much less certain. BMD is modestly improved by 1 to 2%, although evidence of a definitive reduction in hip and nonvertebral fracture risk when given without vitamin D is lacking at present, although previous studies have suggested a trend toward vertebral fracture risk reduction. Given the established increase in rate of nephrolithiasis and a possible, albeit unproven, potential increase in nonfatal cardiac events with higher dose calcium supplementation, it would seem prudent to recommend that adults with osteoporosis obtain 1200 to 1500 mg of calcium from a combination of supplements and dietary sources.

Vitamin D

Appropriate circulating levels of 25(OH)D are necessary for optimal intestinal absorption of calcium and skeletal accrual and maintenance. Despite this, a significant proportion of children and adults have vitamin D levels that would be deemed insufficient (i.e., 25[OH]D < 20 ng/mL). Data in adults with osteoporosis confirm a benefit of vitamin D supplementation for fracture risk reduction, although the effect is dependent on the patient population and the amount of supplementation. Doses of 400 to 800 IU of vitamin D combined with 1000 mg of calcium reduce the risk for hip fracture in postmeno-

pausal women and men aged 65 years and older, although the benefit is less certain for community-dwelling individuals than for those in assisted living centers. A1  In addition, it appears that a 25(OH)D level of at least 30 ng/mL is needed to reduce the risk for hip fracture, A2  although there is some controversy over this recommendation based on the results of other meta-analyses with likely different methodologies. In addition, a daily vitamin D intake of at least 800 IU also reduces the risk for falls, likely by improving muscle strength and reducing body sway. A3  It should be noted that the recent U.S. Preventive Services Task Force recommendation against the use of calcium and vitamin D10 was based on the general U.S. population and does not pertain to patients with osteoporosis. Finally, although activated vitamin D analogues such as calcitriol and α-calcidiol have been shown to reduce fracture risk, they are generally not indicated based on unacceptable risk for hypercalcemia. The exception to use of vitamin D analogues is possibly patients with stage 3 and 4 chronic kidney disease, wherein treatment of secondary hyperparathyroidism could provide skeletal benefit.

Exercise and Lifestyle

Physical activity is also a critical element of osteoporosis management, which can be indirectly inferred based on the known profound effects of decreased gravitational force (i.e., immobilization, paraplegia, weightlessness in space) on inducing bone loss. Physical activity likely also confers additional benefits through enhanced muscle strength, improved cardiovascular status, and reduction in fall risk. Meta-analysis confirmed a modest benefit of exercise on lumbar spine (mean difference = 0.85%) and trochanteric BMD (mean difference = 1.03%) in postmenopausal women compared with placebo, although it did not show significant changes in femoral neck or total hip BMD. A4  However, studies to date have not confirmed an improvement in bone strength with exercise in this patient group. Studies concerning middle-aged and older men are much more limited in number and quality, although preliminary evidence suggests that resistance training with or without impactloading activities has the greatest BMD benefit. Importantly, although none of the aforementioned studies have demonstrated a clear antifracture benefit from exercise, there are abundant data that multiple targeted exercise interventions do reduce either the risk for falling (Tai Chi) or both the rate and risk for falling (group and home-based exercise programs), A5  which is most likely an inciting event in older patients incurring an osteoporotic fracture. Finally, modification of aberrant lifestyles is also indicated in patients with osteoporosis, especially tobacco cessation and moderation of caffeine, carbonated beverage, and alcohol intake. Data are lacking, however, on whether these reduce overall fracture risk.

Medications

There is robust evidence that pharmacologic therapy significantly reduces the risk for osteoporotic fracture in a clinically meaningful and cost-effective manner.11 Medications approved for osteoporosis can be classified based on their mechanism of action: anticatabolic (i.e., antiresorptive) and anabolic (i.e., bone building).

Anticatabolic Agents

Anticatabolic medications, or antiresorptive as they were more commonly known, inhibit osteoclast recruitment, function, and/or survival, resulting in reductions in skeletal turnover and bone loss. These agents, depending  on the potency and persistence of bone effect, reduce the number of new activation sites (BMUs) and the bone remodeling space, thereby improving BMD while strengthening the skeletal microstructure and reducing fracture risk.

Bisphosphonates

Bisphosphonates (BPs) are the most widely prescribed and used medications for the treatment of osteoporosis, owing in large part to good tolerability and an ability to dose them infrequently (from once weekly to once yearly, depending on the drug). BPs are chemically engineered analogues of the naturally occurring molecule pyrophosphate in which a carbon is substituted for an oxygen. As a result, BPs have an extremely high affinity for hydroxyapatite crystals within bone. After incorporation into bone, BPs are taken up by osteoclasts and thereafter inhibit cellular attachment, function, and survival. The carbon side-chain molecules largely determine skeletal affinity and potency of BP effect. The first-generation BP etidronate, which is not approved in the United States for treatment of osteoporosis, is the least potent agent of the class. It must also be given in an interrupted fashion for 2 weeks every 3 months owing to the potential to cause focal osteomalacia, and it may cause lower gastrointestinal symptoms (i.e., abdominal pain and diarrhea). Nonetheless, it is has been shown to reduce the risk for vertebral but not nonvertebral nor hip fractures. Three oral bisphosphonates are approved by the U.S. Food and Drug Administration (FDA) and currently available in the United States: alendronate, risedronate, and ibandronate, in order of time since initial FDA approval. All three drugs are also available as generic preparations, although some differences do exist between the brand name and generic drugs in regard to the

CHAPTER 243  Osteoporosis  

TABLE 243-2  STRENGTH OF EVIDENCE FOR THE REDUCTION OF RISK FOR FRACTURE TYPES WITH PHARMACOTHERAPY IN WOMEN WITH POSTMENOPAUSAL OSTEOPOROSIS FRACTURE SKELETAL SITES VERTEBRAL

NONVERTEBRAL

HIP

WRIST

Alendronate

•••

•••

•••



Ibandronate

•••

••



ı

Risedronate

•••

•••

•••



Zoledronate

•••

•••

•••

ı

Denosumab

•••

•••

•••

ı

Teriparatide

•••

••



ı

Raloxifene

•••

ı

ı

ı

Strength of evidence symbol legend: ı = insufficient strength of evidence; • = low strength of evidence; • • = moderate strength of evidence; • • • = high strength of evidence. (Adapted with permission from Levis S, Theodore G. Summary of AHRQ’s comparative effectiveness review of treatment to prevent fractures in men and women with low bone density or osteoporosis: update of the 2007 report. J Manag Care Pharm. 2012;18[4 Suppl B]:S1-S15, discussion S13.)

inactive excipients. The oral BPS may be administered once weekly (alendronate and risedronate) or once monthly (risedronate and ibandronate), fasting in the morning with water only, and the patient must remain fasting in a sitting or standing position for 30 to 60 minutes after the dose. Recently, a delayedrelease formulation of risedronate (Atelvia) was approved that may be taken immediately after breakfast. The most common side effect is precipitation or aggravation of gastroesophageal reflux, although most patients tolerate the drugs without difficulty. In light of this side effect and a potential risk for esophageal irritation and ulceration, these drugs are contraindicated in patients with functional or anatomic disorders of esophageal transit (i.e., esophageal stricture, achalasia). All three drugs significantly reduce the risk for vertebral fractures, although high-strength evidence for hip and nonvertebral fracture risk reduction exists for alendronate and risedronate but not ibandronate (Table 243-2). A6  In addition, studies confirm a persistent BMD and likely antifracture benefit after 5 years of therapy. Parenteral BPs are also approved and available for osteoporosis treatment, although they should be considered second line to oral BPs based on overall risk-benefit assessment in most osteoporotic patients. They may be considered for use in patients with contraindications to oral BPs (e.g., esophageal disease, inability to sit upright and/or fast after dose), documented or expected poor adherence to oral BPs, or failure to respond to oral BPs or other FDAapproved therapies (recurrent fractures, declining BMD). Zoledronic acid, 5 mg once yearly, and ibandronate, 3 mg quarterly, may be given, although highstrength evidence would favor the use of zoledronic acid, given its unequivocal effect on spine, hip, and nonvertebral fracture risk reduction. A6  Intriguingly, zoledronic acid has also been shown to reduce mortality in women and men following a low-trauma hip fracture, A7  although the mechanism of the mortality benefit is unknown. Finally, BMD remains stable and the antifracture effect likely persists for 3 years after three annual doses of zoledronic acid. A8  Both drugs are associated with an approximately 15 to 20% likelihood of a flu-like reaction, typically consisting of fever, arthralgias, and myalgias, usually limited to the first infusion, and generally lasting 24 to 48 hours, although symptoms lasting weeks to months have rarely been reported to the FDA. Both drugs confer a higher risk as well for delayed healing of exposed bone in the oral cavity compared with oral BPs (see later). Rare, considerably more serious adverse effects have been associated with both oral and intravenous BPs. Osteonecrosis of the jaw, which is defined as exposed bone within the oral cavity for more than 8 weeks following an invasive dental procedure (e.g., tooth extraction, dental implant) or spontaneous tooth loss, occurs in roughly 1 in 10,000 to 100,000 patients treated with oral BPs, although it likely occurs in 1 in 1000 to 10,000 in intravenous BP−treated patients with osteoporosis. Current evidence suggests that microbial biofilm formation on an acellular bone surface, perhaps facilitated by BPs and the non-BP drug denosumab (see later), may be operative in the development of this disorder. As such, patients on intravenous BPs should maintain optimal oral hygiene and consider a BP holiday or delay in dose if invasive oral procedures are planned. Atypical femoral fractures have also been recently described in patients on long-term bisphosphonate therapy, generally after 5 years or more of treatment. Patients will typically have prodromal thigh or groin pain, which is referable to a stress fracture of a thickened lateral femoral cortex, inferior to the greater trochanter. These fractures can be bilateral in

1643

nature and may be identified radiographically with plain films, MRI, or CT. These patients are at risk for low-trauma, severe, oblique, “chalk-stick” fractures, which often represent orthopedic repair and healing challenges. Fortunately, the estimated prevalence of atypical femoral fractures is low (~1 in 5,000 to 10,000). Nonetheless, the severe manifestations of osteonecrosis of the jaw and atypical femoral fractures make it prudent for clinicians to consider a BP drug holiday, particularly given strong evidence of continued benefit on discontinuation.

Selective Estrogen Receptor Modulators

Selective estrogen receptor modulators (SERMs) are compounds that bind to the estrogen receptor and thereby influence bone and reproductive biology. As with estrogen (see later), SERMs are anticatabolic agents in bone, acting through a reduction in cytokines (RANKL, tumor necrosis factor-α) that engender osteoclast activation and function. Raloxifene is the only FDA-approved drug for prevention and treatment of osteoporosis in menopausal women, although the breast cancer drug tamoxifen likely has skeletal benefits as well. Both drugs have antiestrogenic effects in the breast and are FDA approved  for the prevention of breast cancer in high-risk patients. Raloxifene does reduce the risk for vertebral fractures by approximately 30 to 50% but does not reduce the risk for hip and nonvertebral fractures. This antifracture profile positions it as an alternative to bisphosphonates in postmenopausal women with osteopenia and a relatively low risk for hip and other nonspine fractures. The most common side effects include hot flushes and leg cramps in about 10 to 15% and about 5% of patients, respectively. SERMs also increase the risk for deep vein thrombosis, with an absolute risk of roughly 1 in 400, akin to that seen with oral estrogen replacement therapy (ERT). Raloxifene has also been associated with an increased risk for fatal stroke in women at higher baseline risk for stroke, likely precluding its general consideration in women older than 65 years.

Estrogen

ERT, either alone or in combination with a progestin in women with an intact uterus, had historically been a frontline agent in the management  of osteoporosis in postmenopausal women (Chapter 240). ERT prevents  bone loss if administered to women at menopause and significantly increases BMD by approximately 3 to 5% in woman who are well into their menopausal years. Although lower doses of estrogen may have skeletal benefits, more standard doses of estrogen (0.625 mg of conjugated equine estrogen and 1.0 mg of ethinyl estradiol) have been proved efficacious. Long-term estrogen therapy reduces the risk for all clinical fractures by about 27%, based  on the available moderate-quality evidence. A9  ERT is also the most efficacious agent available for treatment of vasomotor symptoms. These data not­ withstanding, ERT is associated with an increased risk for stroke (34% increase), A9  and the use of continuous combined hormone replacement therapy (HRT) confers an unacceptable greater global risk than benefit in woman initiating HRT, based on the results of the Woman’s Health Initiative. These results, however, may not be applicable to the younger postmenopausal population, based on differences in cardiovascular risk, although data confirming this are currently lacking. Both ERT and HRT are also associated with a two- to three-fold increase in the risk for venous thromboembolic disease. Therefore, ERT/HRT is recommended only for postmenopausal women at significant risk for fracture for whom other antifracture therapies are unsuitable.

Denosumab

As detailed previously, increased osteoclast activation through the RANKL pathway is a key mechanism through which bone loss occurs in menopause and other osteoporotic conditions. Intuitively, a therapy that targets this process directly would be desirable. Denosumab is a fully human monoclonal antibody to RANKL that is FDA approved for the treatment of osteoporosis in postmenopausal women and in men, as well as for individuals with breast and prostate cancer to reduce bone loss associated with hormonal deprivation therapy. It is administered twice yearly as a subcutaneous injection in the clinic and clearly reduces the risk for spine, hip, and nonvertebral fractures. A10  Denosumab does not undergo hepatic or renal metabolism and thus can potentially be used in patients with more advanced renal dysfunction, unlike BPs. In contrast to BPs, it is reversible, such that robust bone loss ensues once the medication is stopped. Denosumab is well tolerated in clinical studies, although a higher incidence of skin conditions (eczema and erysipelas) and infections, including serious infections that required hospitalization, were observed in drug- versus placebo-treated subjects. Therefore, the drug is likely not suitable for patients on immunosuppressant therapy who are at higher baseline risk for infection.

Anabolic Agents

Although anticatabolic drugs are effective at retarding bone loss and reducing fracture risk, anabolic or “bone-building” drugs would be preferred. Teriparatide (TPTD) is a recombinant human parathyroid hormone analogue that encompasses amino acids 1 to 34 and was approved by the FDA in 2002. Given

1644

CHAPTER 243  Osteoporosis  

as a self-administered once-daily subcutaneous injection, TPTD is truly anabolic based on robust increases in bone density (~10% over 2 years in the lumbar spine) and bone formation as determined by bone biopsies and other sophisticated imaging studies. More important, TPTD significantly reduces the risk for vertebral and nonvertebral fractures by approximately two thirds and one half, respectively. Because bone resorption increases along with bone formation, bone loss generally ensues on cessation of therapy, necessitating the initiation of an anticatabolic bone drug to preserve the increase in BMD facilitated by TPTD. Finally, although it is plausible to consider that a combination of TPTD and an anticatabolic drug is more beneficial than either drug alone, evidence from randomized controlled trials to date has failed to confirm this. Recent studies, however, suggest that the combination of TPTD and denosumab may have a truly synergistic effect on BMD. TPTD is more expensive than other treatments for osteoporosis, although it is generally covered by insurance in patients who have severe osteoporosis (based on BMD and/or fracture risk) and who cannot tolerate or have contraindications to other antifracture agents. The drug is generally well tolerated, with the most common adverse effects being dizziness and leg cramps. TPTD has a black box warning, based on the fact that toxicology studies in rats revealed an increase in risk for osteosarcoma in animals treated with suprapharmacologic doses of the drug, particularly in growing animals. Given this, the drug is contra-indicated for patients who are at a higher baseline risk for osteosarcoma, including patients with Paget disease and previous therapeutic radiotherapy, as well as younger individuals with open epiphyses. Fortunately, there has not been an observed increase in the rate of osteosarcoma in teriparatide treated patients above that expected in the general population to date.

Other Therapies and Treatment Considerations Currently Available and Emerging Therapies

Nasal calcitonin is FDA approved and available at the time of this writing for treatment of postmenopausal osteoporosis, although it is widely considered the weakest antifracture agent based on marginal vertebral fracture benefit. In addition, recent human studies have suggested a possible link to cancer, potentially further limiting its clinical utility and future availability in the United States. Strontium ranelate is approved in Europe for the treatment of osteoporosis and may have a dual proformation-anticatabolic effect on bone. It has been shown to reduce the risk for vertebral and nonvertebral fractures, as well as clinical osteoporotic fractures. A11  It is not available for use in the United States, and alternative forms of strontium salts cannot be assumed to be effective as well. In addition, BMD by DXA cannot be followed in patients on strontium because of artifactual increases in BMD related to the incorporation into bone of the strontium salt. Emerging therapies on the horizon will likely provide additional tools to treat this debilitating disease, including new anticatabolic agents (e.g., cathepsin K inhibitors) and new anabolic therapies (e.g., sclerostin antibody). Odanacatib is an oral, small molecule that reversibly inhibits cathepsin K, which is produced by activated osteoclasts and primarily is responsible for the breakdown of type 1 collagen. Odanacatib also does not significantly suppress bone formation, perhaps “uncoupling” bone turnover in a favorable fashion to potentiate improvements in BMD. Sclerostin is a naturally occurring inhibitor of the Wnt pathway and bone formation, and preliminary clinical studies do confirm a significant anabolic effect and BMD increase with intermittent administration of a monoclonal antibody to sclerostin. A12  Interestingly, unlike TPTD, inhibition of sclerostin does not appear to stimulate bone resorption, potentially affording greater and more persistent gains in BMD. Ongoing and future clinical studies are needed to confirm an antifracture benefit of this compound.

Glucocorticoid-Induced and Male Osteoporosis

As detailed previously, glucocorticoids are a major cause of and the most common etiology of medication-related secondary osteoporosis. Glucocorticoids are prescribed for a number of common inflammatory conditions, often in a chronic, long-term manner. They are potent suppressors of bone formation and at higher doses likely increase bone resorption, principally through central suppression of sex steroid production. This resultant “uncoupling” of bone turnover can result in dramatic declines in BMD within the first 6 months of starting therapy. In addition to bone loss, there is good evidence to support that individuals on glucocorticoids may fracture at a higher level on BMD compared with non-glucocorticoid-treated patients. Fracture rates are increased as well with doses of prednisone as low as 2.5 mg per day, although the increase in risk appears to attenuate with glucocorticoid discontinuation. The treatment approach to glucocorticoid-induced osteoporosis is similar to osteoporosis in general, with the exception that attempts should be made to reduce the steroid dose to as low as the underlying treated disease will permit.12 Calcium and vitamin D are important adjuncts but are insufficient to prevent bone loss or fractures. Although not clearly evidence-based, replacement of deficient sex steroids is a reasonable strategy in younger individuals who are at lower risk for fracture. The BPs alendronate, risedronate, and zoledronic acid are FDA approved for glucocorticoid-induced osteoporosis in

women and men, although the established benefit is based primarily on BMD improvement. A more logical and indeed superior treatment of glucocorticoidinduced osteoporosis is TPTD, which as an anabolic drug more directly addresses the primary mechanism of bone loss in glucocorticoid-induced osteoporosis: osteoblast inhibition. TPTD is FDA approved for treatment of glucocorticoid-induced osteoporosis in women and men and is superior to alendronate in improving BMD and vertebral fracture risk reduction. A13  Although the drug was used for 36 months in this head-to-head trial, treatment is advised for no more than 24 months based on previously mentioned safety considerations. Male osteoporosis historically has been under-recognized and underappreciated by primary care clinicians and patients alike, although the current data support a significantly more prevalent and clinically significant disorder. More than 2 million men in the United States have osteoporosis, and one in four men older than 50 years will suffer a fragility fracture in their remaining lifetime. Roughly 30% of vertebral and hip fractures combined occur in men, and these are the more common fractures in older men. In addition, men have a substantially higher mortality after hip fracture compared with women. As in women, aging, low body weight, and prior fragility fractures are independent predictors of fracture. In some contradistinction to women, however, osteoporosis in men is more commonly multifactorial in etiology, with the most common secondary causes being excess glucocorticoids, hypogonadism, and alcohol overuse. Despite these associations and others (current smoking, history of falls), there is not at present sufficient evidence to warrant use of a specific testing or screening strategy to identify men at higher risk for fracture. The laboratory work-up of male osteoporosis is similar to that for women, with the exception of a morning fasting testosterone level. Idiopathic osteoporosis may also occur, particularly in younger men with no discernable cause. Genetic factors may well be important in these men, with studies suggesting an association with lower production and circulating levels of estrogen. As in women, primary treatment of male osteoporosis is targeted at lifestyle changes, adequate nutrition (calcium and vitamin D), and exercise. Bisphosphonates (oral and intravenous), denosumab, and TPTD are all effective at improving BMD in men. Although more limited in scope, antifracture efficacy is evident for denosumab in men with prostate cancer on androgen deprivation therapy. True antifracture efficacy for the other agents is either less convincing or absent, based on the paucity of randomized controlled trial data, although this should not be construed as a reason not to treat. Testosterone replacement of men with significant biochemical hypogonadism (total T score < 200 ng/dL) does improve bone density, although data on fracture risk reduction are lacking. In older men (>50 years) at a substantial risk for fracture based on history and risk factors, androgen replacement should be considered second line behind the aforementioned other therapies, based on overall risk-benefit and lack-of-fracture data.

Vertebroplasty and Kyphoplasty and Low-Intensity Vibration

Although often clinically silent, vertebral fractures may cause acute and severe back pain. In addition, up to one third of vertebral fractures remain chronically painful, perhaps related to incomplete healing or instability of  the fracture. Over the past decade, vertebroplasty and kyphoplasty have been developed and advanced to reduce the morbidity associated with  acute spine fractures. These invasive procedures introduce, through the spinal pedicles, a cement-like substance (polymethylmethacrylate) to the compressed vertebral body, with (kyphoplasty) or without (vertebroplasty) use of saline-infused balloon tamps that permit a few millimeters of elevation of the vertebral end plates. Initial randomized trials suggested a benefit of vertebroplasty over conservative management in patients with acute vertebral fractures, although a recent meta-analysis of patient-level data from  two randomized controlled trials did not confirm this finding. A14  Additionally, there may be a concern about fracture of adjacent vertebrae following the procedure, reinforcing the need for further, adequately powered and designed clinical trials. Low-intensity vibration is also under active investigation as an anticatabolic and possibly anabolic intervention for osteoporosis. Animal studies using lowintensity vibration appears to show enhanced osteoblast and hindered osteoclast development, thereby “coupling” bone remodeling. Clinical studies suggest a modest but significant BMD benefit in postmenopausal women and other groups (children with cerebral palsy, adults on prolonged bed rest), although further studies are needed to confirm a true clinical and ideally antifracture benefit of this intervention.

PROGNOSIS

It stands to reason that, based on the information and data discussed previously, the burden incurred by individual patients and society as a whole can be significantly lessened through a combination of diagnostic, preventive, and therapeutic interventions. Although there is no true “cure” for osteoporosis, current pharmacotherapies reduce the risk for fracture roughly by half.

CHAPTER 244  Osteomalacia And Rickets  

This reduction is critical because there is robust evidence to suggest an independent increase in mortality after an osteoporotic fracture, including fractures of the spine, humerus, tibia, and pelvis, as well as the proximal femur. Moreover, available data, primarily from randomized controlled trials with bisphosphonates, confirm a statistically significant reduction in death with pharmacologic treatment of osteoporosis, although the mechanism of this effect is not known.13 These data further underscore the importance of identifying and treating patients with osteoporosis.

Grade A References A1. Avenell A, Gillespie WJ, Gillespie LD, et al. Vitamin D and vitamin D analogues for preventing fractures associated with involutional and post-menopausal osteoporosis. Cochrane Database Syst Rev. 2014;14:CD000227. A2. Bischoff-Ferrari HA, Willett WC, Wong JB, et al. Prevention of nonvertebral fractures with oral vitamin D and dose dependency: a meta-analysis of randomized controlled trials. Arch Intern Med. 2009;169:551-561. A3. Bischoff-Ferrari HA, Dawson-Hughes B, Staehelin HB, et al. Fall prevention with supplemental and active forms of vitamin D: a meta-analysis of randomised controlled trials. BMJ. 2009;339:b3692. A4. Howe TE, Shea B, Dawson LJ, et al. Exercise for preventing and treating osteoporosis in postmenopausal women. Cochrane Database Syst Rev. 2011;7:CD000333. A5. Gillespie LD, Robertson MC, Gillespie WJ, et al. Interventions for preventing falls in older people living in the community. Cochrane Database Syst Rev. 2012;9:CD007146. A6. Levis S, Theodore G. Summary of AHRQ’s comparative effectiveness review of treatment to prevent fractures in men and women with low bone density or osteoporosis: update of the 2007 report. J Manag Care Pharm. 2012;18:S1-S15. A7. Lyles KW, Colon-Emeric CS, Magaziner JS, et al. Zoledronic acid and clinical fractures and mortality after hip fracture. N Engl J Med. 2007;357:1799-1809. A8. Black DM, Rein IR, Boonen S, et al. The effect of 3 versus 6 years of zoledronic acid treatment of osteoporosis: a randomized extension to the HORIZON-Pivotal Fracture Trial (PFT). J Bone Miner Res. 2012;27:243-254. A9. Marjoribanks J, Farguhar C, Roberts H, et al. Long term hormone therapy for perimenopausal and postmenopausal women. Cochrane Database Syst Rev. 2012;7:CD004143. A10. Cummings SR, Ensrud K, Delmas PD, et al. Denosumab for prevention of fractures in postmenopausal women with osteoporosis. N Engl J Med. 2009;361:756-765. A11. Kanis JA, Johansson H, Oden A, et al. A meta-analysis of the effect of strontium ranelate on the risk of vertebral and non-vertebral fracture in postmenopausal osteoporosis and the interaction with FRAX(®). Osteoporos Int. 2011;22:2347-2355. A12. McClung MR, Grauer A, Boonen S, et al. Romosozumab in postmenopausal women with low bone mineral density. N Engl J Med. 2014;370:412-420. A13. Saag KG, Zanchetta JR, Devogelaer JP, et al. Effects of teriparatide versus alendronate for treating glucocorticoid-induced osteoporosis: thirty-six-month results of a randomized, double-blind, controlled trial. Arthritis Rheum. 2009;60:3346-3355. A14. Staples MP, Kallmes DF, Comstock BA, et al. Effectiveness of vertebroplasty using individual patient data from two randomised placebo controlled trials: meta-analysis. BMJ. 2011;343:d3952.

1645

TABLE 244-1  CAUSES OF OSTEOMALACIA VITAMIN D DEFICIENCY Dietary deprivation and lack of sunlight exposure VITAMIN D MALABSORPTION Postgastrectomy Gastric bypass for obesity Gluten enteropathy Small bowel disease or resection Pancreatic insufficiency Cholestyramine therapy for cholestatic liver disease Laxative abuse IMPAIRED 1-HYDROXYLATION OF 25-HYDROXYVITAMIN D Vitamin D−dependent rickets type I X-linked hypophosphatemia Autosomal dominant hypophosphatemic rickets/osteomalacia Oncogenic osteomalacia IMPAIRED TARGET-ORGAN RESPONSE TO 1,25-DIHYDROXYVITAMIN D Vitamin D−dependent rickets type II HYPOPHOSPHATEMIA X-linked hypophosphatemia Autosomal dominant hypophosphatemic rickets/osteomalacia Sporadic hypophosphatemia Fibrous dysplasia Oncogenic osteomalacia Antacid-induced osteomalacia Chronic metabolic acidosis Paraproteinemia Saccharated ferric oxide Tenofovir Cadmium INHIBITORS OF MINERALIZATION Etidronate Fluoride Aluminum Iron MISCELLANEOUS Hypophosphatasia Axial osteomalacia Fibrogenesis imperfecta ossium

GENERAL REFERENCES For the General References and other additional features, please visit Expert Consult at https://expertconsult.inkling.com.

244  OSTEOMALACIA AND RICKETS ROBERT S. WEINSTEIN

DEFINITION

Rickets refers to impaired mineralization of the cartilaginous growth plate and abnormal endochondral bone formation and therefore cannot occur in adults after epiphyseal closure.1 Osteomalacia, literally meaning softening of bone, refers to defective or delayed mineralization of the organic matrix of bone, or osteoid, at the interface between calcified bone and osteoid, and may occur at any age. Both rickets and osteomalacia may be present in a growing child, but defective mineralization can cause only osteomalacia in adults; therefore, this chapter will focus on osteomalacia. Despite advances in our understanding of vitamin D metabolism and the increased sensitivity of measurements of serum 25-hydroxyvitamin D, osteomalacia remains a common and frequently overlooked disorder in the world. Optimal therapy requires precise identification of the etiology of the abnormal mineralization, which may present a problem because there are numerous causes (Table 244-1). However, after a correct diagnosis is made, therapy is usually gratifying and often spectacular. Early recognition of osteomalacia depends on familiarity

with the typical clinical manifestations and settings. It is helpful to appreciate that the bone disease almost always manifests in the same manner regardless of the cause of the osteomalacia.

EPIDEMIOLOGY

About 20% of North American women receiving treatment for osteoporosis have 25-hydroxyvitamin D levels below 20 ng/mL (adequate values are greater than 30 ng/mL), and 8% have levels below 15 ng/mL. This indicates that, at the least, impaired bone mineralization could be a confounding factor in their osteoporosis treatment and, at worst, osteomalacia is the correct diagnosis (a defect in mineralization) rather than osteoporosis (a reduced amount of normally mineralized bone). An inadequate response to the bisphosphonate treatment commonly used for postmenopausal osteoporosis is four times more likely when 25-hydroxyvitamin D levels are subnormal than when the levels are above 30 ng/mL.2 Vitamin D deficiency is more common in elderly people, especially in nonaffluent people during the winter, at higher latitudes, and with low sun exposure. Vitamin D deficiency is also commonly found in medical inpatients, institutionalized patients, and postmenopausal women with acute hip fracture.3 The prevalence of osteomalacia due to vitamin D deficiency varies with the referral source. The disorder is far more frequent when patients are referred from geriatricians, gastroenterologists (osteomalacia may be found in up to 30% of patients with gastric surgery or bypass for obesity), nursing homes, or orthopedists concerned about symmetrical or nonhealing fractures. The most common hypophosphatemic osteomalacia is the inherited disease X-linked hypophosphatemia (XLH), but affected adults infrequently present to internists and then only when troubled by severe bone pain or nonunion of fractures.

CHAPTER 243  Osteoporosis  

GENERAL REFERENCES 1. Griffith JF, Genant HK. New advances in imaging osteoporosis and its complications. Endocrine. 2012;42:39-51. 2. Leslie WD, Morin SN. Osteoporosis epidemiology 2013: implications for diagnosis, risk assessment, and treatment. Curr Opin Rheumatol. 2014;26:440-446. 3. van den Bergh JP, van Geel TA, Geusens PP. Osteoporosis, frailty and fracture: implications for case finding and therapy. Nat Rev Rheumatol. 2012;8:163-172. 4. Richards JB, Zheng HF, Spector TD. Genetics of osteoporosis from genome-wide association studies: advances and challenges. Nat Rev Genet. 2012;13:576-588. 5. Nguyen TV, Eisman JA. Genetic profiling and individualized assessment of fracture risk. Nat Rev Endocrinol. 2013;9:153-161. 6. Kawai M, de Paula FJ, Rosen CJ. New insights into osteoporosis: the bone-fat connection. J Intern Med. 2012;272:317-329.

1645.e1

7. Bonewald LF, Kiel DP, Clemens TL, et al. Forum on bone and skeletal muscle interactions: summary of the proceedings of an ASBMR workshop. J Bone Miner Res. 2013;28:1857-1865. 8. Gerdhem P. Osteoporosis and fragility fractures: vertebral fractures. Best Pract Res Clin Rheumatol. 2013;27:43-55. 9. Naylor K, Eastell R. Bone turnover markers: use in osteoporosis. Nat Rev Rheumatol. 2012;8:379-389. 10. Moyer VA. Vitamin D and calcium supplementation to prevent fractures in adults: U.S. Preventive Services Task Force recommendation statement. Ann Intern Med. 2013;158:691-696. 11. Andreopoulou P, Bockman RS. Management of postmenopausal osteoporosis. Annu Rev Med. 2014;66:329-342. 12. Rizzoli R, Biver E. Glucocorticoid-induced osteoporosis: who to treat with what agent? Nat Rev Rheumatol. 2015;11:98-109. 13. Grey A, Bolland MJ. The effect of treatments for osteoporosis on mortality. Osteoporos Int. 2013;24:1-6.

1645.e2

CHAPTER 243  Osteoporosis  

REVIEW QUESTIONS 1. A 60-year-old woman presents to her physician to discuss her recent bone density results and management options. She has treated hypertension and a history of atrial fibrillation, as well as rheumatoid arthritis, for which she takes infliximab. Family history is notable for a hip fracture in her mother after a fall at age 65 years. She drinks socially and does not smoke. Her physical examination is unremarkable, including only 1 inch of height loss from her young adult maximum. Her spine examination reveals normal curvature, no kyphosis, normal rib-pelvis distance of 3 fingerbreadths, and 0 fingerbreadth wall-occiput distance. Laboratory studies are normal, including 25(OH)D level. Duel-energy x-ray absorptiometry (DXA) bone density reveals lumbar spine, femoral neck, and total hip T scores of −2.5, −3.0, and −2.7, respectively. Her FRAX 10-year estimates of hip and major osteoporotic fracture are 5.6 and 29%, respectively. What is the best management recommendation for this woman? A. Calcium and vitamin D supplementation alone B. Hormone replacement therapy C. Raloxifene D. Oral bisphosphonate E. Denosumab Answer: D  This patient has osteoporosis based on T score at the lumbar spine and proximal femur. In addition, she has an absolute fracture risk that supports pharmacologic intervention on a cost-effectiveness basis. Oral bisphosphonates are effective and generally frontline therapy. Active use of immunosuppressive therapy increases her risk for infection, which was seen more frequently in denosumab-treated patients in randomized controlled trials. Her cardiovascular history increases her risk for stroke, which has been observed more frequently in raloxifene- and estrogen-treated patients. Finally, calcium and vitamin D are important adjuncts to her management but should not be considered adequate alone for fracture risk reduction in this woman. 2. A 65-year-old woman presents with 6 months of progressive lower extremity pain and describes difficulty ascending stairs because of pain and weakness. She also brings a recent outside DXA bone density test, which shows total hip T and Z scores of −4.5 and −2.5, respectively. She does not have a history of fragility fractures. Past medical history is notable for hypertension and long-standing irritable bowel syndrome. She takes 600 mg of calcium and 400 IU of vitamin D daily, but no other medications. Family history is negative for osteoporosis or parental hip fracture. She does not smoke or drink alcohol. Physical examination is notable for tenderness to palpation over the mid-tibia bilaterally. She also has a wide-based, nonantalgic gait. What is the next best choice for her skeletal management? A. Double her calcium and vitamin D daily intake. B. Measure serum 25(OH)D level. C. Start an oral bisphosphonate. D. Measure fasting serum C telopeptide level (CTx) E. Start teriparatide therapy. Answer: B  This patient has lower bone density than expected for age based on Z score of −2.0 or less. This suggests a secondary cause for low bone mineral density (BMD) besides menopause. Her clinical presentation is consistent with osteomalacia, which cannot be distinguished from osteoporosis based solely on BMD. In addition, the prevalence of vitamin D deficiency or insufficiency and established hip fracture efficacy warrant its identification and treatment as the next best step in patient management. The recommended increase in calcium and vitamin D would be insufficient to treat vitamin D deficiency−related osteomalacia in this woman, and bisphosphonate and teriparatide therapy would be inappropriate until the vitamin D deficiency is corrected. Finally, bone turnover markers cannot be used independently for diagnosis in patients with metabolic bone disease.

3. An 81-year-old man is admitted after a fall and low-impact fracture of the proximal femur. His history is notable for Parkinson’s disease treated with dopamine agonist therapy. On physical examination, patient has a mild resting tremor while lying in bed. Laboratory investigations do not reveal secondary causes of osteoporosis, including a 25(OH)D level of 30 ng/ mL and testosterone of 300 ng/dL. The patient has normal renal function and calcium. The patient undergoes successful operative repair. After discharge, the patient undergoes 4 weeks of inpatient rehabilitation. What is the most definitive choice for this patient’s metabolic bone management? A. Start calcium and vitamin D supplementation. B. Increase dopamine agonist therapy for Parkinson’s disease. C. Give zoledronic acid, 5 mg intravenously. D. Start testosterone replacement therapy. E. Prescribe ergocalciferol 50,000 IU once weekly. Answer: C  This gentleman has incurred a low trauma femur fracture, which greatly increases his risk for subsequent fractures. Furthermore, the fracture confers a mortality rate up to 30% within 1 year of the event. Zoledronic acid has been proved not only to reduce the risk for subsequent fracture but also to reduce mortality by 27% Calcium and vitamin D are important adjuncts to treatment but not definitive. Given his level of 25(OH)D, additional antifracture and/or fall risk reduction would not be anticipated from pharmacologic vitamin D treatment. There is no evidence that testosterone treatment of either eugonadal or hypogonadal men reduces fracture risk. Finally, optimal management of his Parkinson’s disease, which does not appear undertreated in this patient, might reduce his risk for falls but not his risk for fracture. 4. A 55-year-old white woman presents to her physician with intense midback pain after a fall onto her backside while walking. Her history is notable for a recent diagnosis of polymyalgia rheumatica, for which she has taken prednisone 10 mg daily for the last 6 months. She does have known osteopenia, with lumbar spine, femoral neck, and total hip T scores of −1.5, −1.2, and −0.8, respectively. Physical examination is notable for tenderness to palpation and percussion over the lower thoracic spine. Plain films of the thoracolumbar spine reveal a new moderate (35%) anterior compression fracture at T11. Laboratory work-up, including 25(OH), is within normal limits. Other than continuing the calcium and vitamin D that the patient is currently taking, what is the most appropriate pharmacologic choice for treatment at this time? A. Vitamin D 50,000 IU once weekly B. Alendronate 70 mg once weekly C. Risedronate 150 mg once monthly D. Teriparatide 20 mcg subcutaneously daily E. Denosumab 60 mg subcutaneously every 6 months Answer: D  This woman presents with an acute vertebral fracture following a low trauma event. Her level of BMD is not osteoporotic, underscoring the independent contribution of glucocorticoids to fracture risk such that fractures may occur at a higher (i.e., better) level of BMD. The recent nature of her fracture further underscores the need for initiation of treatment now. There is no evidence that increasing this patient’s vitamin D level will further reduce her risk for fracture. Denosumab will likely reduce her risk for subsequent fracture, although the association with infection and use of concurrent prednisone in this woman preclude its consideration at present. Alendronate and risedronate are approved for the prevention and treatment of glucocorticoid-induced osteoporosis (GIO). Despite this, teriparatide has been proved more effective that bisphosphonates in patients with GIO and is indicated for this woman, assuming she has no contraindications to treatment.

CHAPTER 243  Osteoporosis  

5. A 70-year-old man presents with acute, mid-back pain after lifting a bag of topsoil while gardening 1 week ago. His medical history is notable for known male hypogonadism due to mumps orchitis in his 20s, although he does not take testosterone because of severe benign prostatic hypertrophy. He does take 600 mg of calcium twice daily and 1000 IU of vitamin D once daily. On physical examination, he has tenderness to palpation and percussion over his mid-thoracic spine. Laboratory investigations are unrevealing, including a 25(OH) level of 45 ng/mL. DXA bone density study confirms osteoporosis with lumbar spine T score of −3.0, although femoral neck and total hip T scores are normal. Radiographs of the thoracic spine reveal a new severe (50%) biconcave compression fracture at T8 without apparent widening of the pedicles. In addition to prescribing alendronate, which of the following is best adjunct management for this patient? A. Analgesic therapy and referral to physical therapy for post-fracture consultation B. Prescription for a spinal brace that patient should wear fro 6 months C. Percutaneous vertebroplasty D. Nasal calcitonin use for 1 year E. No additional interventions are required at this time.

1645.e3

Answer: A  This gentleman has osteoporosis secondary to long-standing hypogonadism, exhibited by predominantly great bone density loss and deficit in cancellous bone (i.e., spine). He has incurred by definition a lowtrauma fracture and is in need of a pharmacologic antifracture therapy (i.e., alendronate). Although he has significant pain that must be addressed, there is no evidence that conservative therapy with analgesics is inferior to vertebroplasty in regard to pain management. In addition, his radiographic findings do not suggest fracture instability that might benefit from vertebroplasty. Physical therapy is also helpful both short term as an adjunct to pain management and long term regarding modification of lifestyle and exercise to maximally reduce future fracture risk. A supportive spine brace may be used, although its use should be limited to only 4 to 6 weeks because of the necessary induction of paraspinal muscle weakness associated with long-term use. Similarly, nasal calcitonin may have some analgesic benefit in the acute, postfracture period based on limited data, although there is no evidence of a long-term analgesic benefit with continued use of the drug.

1646

PATHOBIOLOGY

CHAPTER 244  Osteomalacia And Rickets  

A review of normal bone remodeling and the mineralization of osteoid (bone matrix) serves as a background to understand the abnormal mineralization characteristic of osteomalacia. Bone remodeling or turnover is carried out by teams of juxtaposed osteoclasts and osteoblasts, comprising temporary anatomical structures known as basic multicellular units (BMUs). In cortical bone, the BMUs drill tunnels or “cutting cones” through the compact tissue; whereas in spongy, cancellous bone, they gouge across the trabecular surface, forming serpiginous trenches. Bone turnover begins by conversion of a quiescent skeletal surface to a remodeling site, a process referred to as activation. Activation involves proliferation of new blood vessels needed to bring recruited osteoclast progenitors to the remodeling site and retraction of the flat, pavement-like bone-lining cells that cover the quiescent surfaces to expose the mineralized bone surface. The recruited cells become multinucleated osteoclasts, which attach to the newly exposed bone surface with a ring of contractile proteins sealing off a subosteoclastic resorption compartment. Lysosomal enzymes, hydrogen ions, and collagenase are secreted through the microvilli of the ruffled underside border of the osteoclasts, and these chemicals begin to excavate a resorption cavity. The osteoclasts remove both the bone mineral and matrix. It is a misunderstanding to attribute to these cells or to metabolic acidosis the ability to remove only the mineral, leaving behind demineralized osteoid. Demineralized bone in vivo is a misnomer. Demineralized or decalcified bone only occurs when bones are placed in acid (1N HCl) or chelating solutions (ethylenediaminetetraacetate, or EDTA). Osteoclasts are motile cells, capable of resorbing more than just the cavity within which they are identified. After an osteoclast digs a cavity, it may detach from bone and move on to a new resorption site or die by apoptosis and be quickly removed by phagocytes. When the osteoclasts have moved on, osteoblasts assemble to reconstitute the previously resorbed cavity with new bone. In any established BMU, both events are happening at the same time; bone formation begins to occur while bone resorption advances. Between the end of bone resorption and the beginning of bone formation is the reversal phase, when mononuclear phagocytes smooth out the jagged erosion bays. During this phase, the old bone is coated by a thin layer of cement substance, a collagen- and mineral-poor matrix rich in glycosaminoglycans, glycoproteins, and acid phosphatase, to which the new osteoblasts attach. In adults, new osteoblasts assemble only at sites where osteoclasts have recently been eroding bone; a phenomenon referred to as coupling. The arrival of the osteoblasts in the right place at the right time and in sufficient numbers to reconstitute the cavity is referred to as remodeling balance and is likely due to proportional production of osteoblasts and osteoclasts in the bone marrow, release of osteoblast-recruiting substances from the resorbed bone, and chemotaxis by the cement substances. As osteoblasts complete their bone matrix synthesis and move away from the cement line, they gradually flatten. Some osteoblasts become bone-lining cells, and some become osteocytes, but as many as 65% of the osteoblasts that originally assembled at the remodeling site die by apoptosis. It is the balance between cell proliferation and apoptosis that determines the amount of work performed by these cells. Normally, up to 70% of the mineralization of the osteoid deposited by the osteoblasts starts within 4 to 12 days and proceeds at about 1 µm per day; but in osteomalacia, mineral deposition in the osteoid slows or stops completely, while the osteoblasts continue to make osteoid, which then accumulates in excessive amounts. Therefore, normal osteoid width is about 4 to 12 µm, but in osteomalacia, the osteoid width may become dramatically augmented. Depending on the extent of the delay in mineralization, overt osteomalacia may take many years to develop. In normal subjects, further mineralization proceeds slowly over months to years and at the cost of displacement of the water in the hydroxyapatite crystals, resulting in a modest increase in brittleness and the eventual need for another round of remodeling. Even though 1 million BMUs are undergoing remodeling every day, bone mass in a healthy adult is preserved thanks to a remarkably tight balance between the amount of bone resorbed and the amount formed during each cycle of remodeling. By this means, the adult skeleton is almost completely regenerated every 10 years. Mineralization requires the availability of sufficient calcium and phosphorus at the remodeling site, the presence of a normal bone collagen matrix, the absence of inhibitors of mineralization, and an adequate amount of skeletal alkaline phosphatase activity. Defects in these requirements are the cause of most forms of osteomalacia. Deficiency of vitamin D per se has traditionally been incriminated as the cause of the osteomalacia, but today, considerable evidence indicates that the abnormal mineralization associated with vitamin

D deficiency depends more on the deficiency of calcium and phosphorus than the absence of a direct effect of vitamin D on bone cells. The primary function of vitamin D is to provide adequate levels of calcium and phosphorus by increasing their intestinal absorption. Chronic metabolic acidosis has also been identified as a cause of osteomalacia, but evidence suggests that the bone disease associated with chronic metabolic acidosis is primarily due to the associated hypophosphatemia.

CLINICAL MANIFESTATIONS

The clinical presentation of osteomalacia depends on three overlapping manifestations: those due to the underlying disorder, such as gastrointestinal disease or surgery (especially troublesome are gastric resection, stapling or bypass for obesity, and intestinal malabsorption); those due to hypocalcemia or hypophosphatemia; and those directly due to the bone disease. The most common symptoms and signs are bone pain, muscle weakness, and bone tenderness. The bone pain is usually nonspecific and poorly localized. Because of the paucity of findings, the pain is often attributed to rheumatism or neurosis. It may be worse at night and after sudden movements such as turning in bed or the change from sitting to standing. Most often, the pain is in the lower back, pelvis and legs and is worse on weight bearing, resulting in a characteristic flat-footed, springless, waddling gait made worse by proximal muscle weakness. The gait has been referred to as “mother penguin’s walk.” Patients may complain that they can only climb stairs by pulling themselves up with the hand rail or rise from sitting in a chair or on the toilet by using their hands to push off. The decrease in strength is usually far greater than the degree of muscle wasting. Fasciculations are absent, and both reflexes and sensation remain normal. The bulbar, facial, and ocular muscles are always spared. However, muscle weakness is conspicuously absent when the osteomalacia is due to X-linked hypophosphatemia (see Table 244-1). Often, bone tenderness can be elicited by rib cage compression or pressing on the tibiae, wrists, pubic rami, or iliac crests. Hypocalcemia is usually mild to moderate but, rarely, can be severe enough to present with paresthesias, muscle cramps, a positive Chvostek’s sign, or seizures. If the osteomalacia is mistaken for osteoporosis and treatment is started with a bisphosphonate, the patient may experience new-onset paresthesias, muscle cramps, and palpitations. This not uncommon scenario occurs because the antiresorptive treatment interferes with the compensatory secondary hyperparathyroidism and aggravates the hypocalcemia.

DIAGNOSIS

Biochemical changes depend on the stage of the disease and its etiology. In vitamin D deficiency, hypophosphatemia precedes and is more severe than the hypocalcemia because of the secondary or compensatory hyperparathyroidism (Chapter 245) that almost invariably accompanies the disorder by the time that osteomalacia has occurred. In malabsorption, hypomagnesemia may contribute to the hypocalcemia, and hypoalbuminemia may lead to a spurious diagnosis of hypocalcemia. Increased serum alkaline phosphatase activity is classically associated with osteomalacia due to vitamin D deficiency but is not an early or reliable clue because some patients may have normal or borderline levels. The serum 25-hydroxyvitamin D levels are often less than 10 to 15 ng/mL. In contrast, serum 1,25-dihydroxyvitamin D levels are usually elevated because of the concomitant hyperparathyroidism and do not contribute to the diagnosis of osteomalacia except in the rare abnormalities of vitamin D resistance (when 1,25-dihydroxyvitamin D levels may be extraordinarily high) or when 1-hydroxylation is defective (and 1,25dihydroxyvitamin D levels are low). Quite a different pattern occurs with the inherited disease hypophosphatasia: serum 25-hydroxyvitamin D and calcium are normal, phosphorus and vitamin B6 levels are high normal or frankly elevated, and alkaline phosphatase activity is below the normal range.4 Radiographic findings may be absent with early osteomalacia, and only blurred margins of the cancellous bone with thin cortices may be noted. The presence of bilateral, thin (2 to 3 mm), radiolucent bands known as pseudofractures (Fig. 244-1) found perpendicular to the periosteal surface in ribs, pubic and ischial rami, the neck of the femur, and metatarsals and below the glenoid fossa on the outer border of the scapulae are generally considered to be pathognomonic of osteomalacia, but this classical radiographic sign is infrequent today. Rarely, it may be seen in disorders lacking excessive osteoid. These pseudofractures (sometimes called Looser’s zones or Milkman’s fractures) show increased uptake on bone scans (Fig. 244-2) and may lead to an inappropriate search for a primary malignancy. Bone mineral density T scores are often −3 or −4, with the radial diaphyseal density lower than that of the lumbar spine or total proximal femur.

CHAPTER 244  Osteomalacia And Rickets  

1647

FIGURE 244-3. An undecalcified bone biopsy specimen shows the characteristic abundant osteoid and flattened osteoblasts of osteomalacia (normally mineralized bone is blue and osteoid is red).

FIGURE 244-1.  Radiographic evidence of a pseudofracture of the femoral neck is suspicious for osteomalacia (arrow).

FIGURE 244-2. In osteomalacia, focal increased uptake of radionuclide on a bone scan may erroneously suggest metastatic disease (arrows).

Although characteristic clinical, radiographic, and biochemical findings may suggest osteomalacia, the absence of these findings cannot exclude the diagnosis. Quantitative histologic examination of undecalcified bone is, therefore, required to establish the unequivocal presence of osteomalacia (Fig. 244-3). Rigorous kinetic criteria for the histologic recognition of osteomalacia are necessary to preserve the traditional clinical, biochemical, and therapeutic connotations of the term. Therefore, a review of the quantitative bone histologic findings or histomorphometry in osteomalacia is useful.5 The histomorphometric diagnosis of osteomalacia requires the simultaneous presence of three findings: (1) excessive osteoid (osteoid area >10% of the cancellous bone area; normal is 15 µm; normal is 4 to 12 µm), (3) and prolongation of the mineralization lag time (>100 days; normal is 9 to 20 days), as determined by the osteoid width divided by the distance between and linear extent of double tetracycline labels observed in the bone after the patient receives two time-spaced courses of oral tetracycline. Tetracycline is deposited early in the course of hydroxyapatite crystal formation and generates bright stripes at the interface of mineralized bone and osteoid when viewed with fluorescent microscopy. If the two time-spaced courses of tetracycline (1 g/day for 3 days) are separated by a 14-day interval, the rate of mineralization (µm/day) can be calculated by measuring the average distance between the double labels divided by the number of days between the two courses. When the double labels are numerous and widely spaced, mineralization is intact and excess osteoid must

be due to increased bone turnover. A paucity of tetracycline labels that are narrowly spaced indicates that if excessive osteoid is present, it must be due to the delayed or ceased mineralization of osteomalacia (Fig. 244-4). Therefore, it follows that excessive osteoid can occur from two distinct mechanisms. Osteomalacia is the consequence of defective mineralization, while osteoid production continues. However, osteoid will also accumulate with accelerated bone formation if the rate of osteoid deposition exceeds the rate of mineralization, as occurs in states of greatly increased bone turnover, such as hyperparathyroidism (Chapter 245), Paget’s disease (Chapter 247), or thyrotoxicosis (Chapter 226). Even though osteoblasts in osteomalacia are usually sparse and flattened, whereas they are numerous, plump, and cuboidal with high bone turnover, these two groups of disorders can only be reliably distinguished with the use of tetracycline markers. The treatment of increased bone turnover and of defective mineralization is completely different, which is why the three histomorphometric criteria are necessary. Additionally, evaluation of each of the criteria in isolation has limitations. Regarding the first requirement, a small increase in the osteoid area relative to the total bone area may occur in osteoporosis, with a decrease in the amount of mineralized bone. In the second requirement, wide osteoid seams may be seen in some specimens obtained from patients with severe secondary hyperparathyroidism, such as those on maintenance hemodialysis therapy (Chapter 131). In the third requirement, reduced mineral appositional rate and increased mineralization lag time are nonspecific indices of impaired matrix synthesis by osteoblasts, as is often found in patients with involutional osteoporosis. Only when all three requirements are fulfilled is the diagnosis of osteomalacia irrefutable. Several presumed causes of osteomalacia (anticonvulsant drugs, metabolic acidosis without hypophosphatemia, pseudohypoparathyroidism, and chronic renal failure) have not fulfilled all of these requirements and primarily represent secondary hyperparathyroidism. Patients with the nephrotic syndrome lose albumin and vitamin D metabolites in the urine, but evidence indicates that serum ionized calcium and parathyroid hormone levels are normal and metabolic bone disease in adults with the nephrotic syndrome is absent. Muscle weakness and bone pain are significantly more common in patients in whom the rigorous histologic diagnosis of osteomalacia has been proved. However, bone biopsy is not always necessary to be reasonably certain of the diagnosis. When biopsy is necessary, the local pathologist must be familiar with the processing of undecalcified bone specimens and plastic embedding; otherwise, the best solution is to refer the patient to a histomorphometry center for biopsy. This ensures satisfactory communication between the clinician, operator, and pathologist and is the best insurance against the incomplete, broken, fragmented, or accidentally decalcified bone specimens. Such referral may be indispensable in the evaluation of a patient with unusually painful disease or progressive loss of bone mineral density, particularly when the results of the physical examination, radiographs, and biochemical findings are ambiguous. Biopsy may also be indicated in patients with unexplained chronic hypophosphatemia. The best approach is to avoid overlooking the diagnosis of osteomalacia by maintaining a high degree of suspicion in the typical clinical settings.6 This is especially important because osteomalacia can usually be successfully

1648

CHAPTER 244  Osteomalacia And Rickets  

A

B

FIGURE 244-4.  Histomorphometric diagnosis of osteomalacia by fluorescence imaging of double tetracycline labeling. A, Tetracycline double labels are numerous, discrete, and widely spaced, as is typical of intact mineralization. B, The tetracycline labels are mostly single despite the administration of two time-spaced doses of oral tetracycline, indicating that mineralization must be delayed or ceased, as is typical of osteomalacia.

treated. An investigation for osteomalacia is indicated in elderly patients with bone pain and muscle weakness, in patients with gastric surgery and low bone mineral density or bone pain, and in patients with persistent hypophosphatemia. Unexplained elevations of the serum alkaline phosphatase activity are usually due to drugs (e.g., anticonvulsants, anabolic steroids, phenothiazines, or antibiotics) or Paget’s disease of bone (Chapter 247) but rarely may be the only biochemical clue to osteomalacia in a patient with variable skeletal discomfort. Bilateral or slowly healing fractures also warrant an investigation for osteomalacia.

TREATMENT  Understanding of the treatment of osteomalacia is facilitated by dividing the disease into four subgroups. The first subgroup is osteomalacia due to disorders of vitamin D absorption or metabolism; the second is osteomalacia due to chronic hypophosphatemia. Most patients with osteomalacia will be in these first two subgroups. Treatment of osteomalacia caused by these two subgroups is discussed in detail in the next two sections. The third subgroup includes osteomalacia caused by inhibitors of mineralization, such as etidronate (the first oral bisphosphonate, now rarely used in North America);  high doses of fluoride; accumulation of a skeletal burden of aluminum from water used for dialysis or as a contaminate in solutions used for parenteral nutrition (now rarely seen); iron overload as in thalassemia; and cadmium, which induces the proximal tubular lesion of Fanconi’s syndrome and causes osteomalacia due to the resultant hypophosphatemia. The fourth subgroup includes miscellaneous causes of osteomalacia that lack specific therapy but are fortunately quite rare. This last subgroup includes the variable forms of the heritable disorder hypophosphatasia, caused by a deficiency of the tissuenonspecific (liver, bone, kidney) isoenzyme of alkaline phosphatase (although, therapeutic trials have shown that enzyme replacement is effective); axial osteomalacia, a sporadic osteosclerotic disorder primarily affecting middleaged men and presenting with mild to moderate pain in the spine and pelvis (but without fractures), apparently due to the production of an abnormal and poorly mineralized bone matrix by osteoblasts; and fibrogenesis imperfecta ossium, another sporadic disorder presenting with intractable bone pain and fractures, mainly in middle-aged men and women and apparently also due  to production of an abnormal bone matrix lacking the normal collagen birefringence by osteoblasts. In axial osteomalacia and fibrogenesis imperfecta ossium, serum calcium, phosphorus, and vitamin D levels are normal, but serum alkaline phosphates activity may be increased. General measures for this last subgroup include routine nutritional advice and avoidance of further bone loss due to postmenopausal or involutional osteoporosis. High-dose vitamin D therapy in these disorders has caused nephrocalcinosis, nephrolithiasis, and renal insufficiency and must be avoided.

Osteomalacia Due to Vitamin D Disorders

Iron deficiency anemia, hypocalcemia, weight loss, glossitis, or pruritic rash and bone discomfort in a patient with low bone mineral density point to celiac disease even without gastrointestinal symptoms. These signs suggest the need to test for immunoglobulin A antiendomysial and antitissue transglutaminase antibodies. Cholestyramine therapy for cholestatic liver disease may increase malabsorption of vitamin D by binding bile salts. Laxative abuse may cause osteomalacia and severe resistance to vitamin D supplementation, including treatment with calcitriol. Advice on nutrition and sun exposure, discontinuation of offending drugs, adherence to a gluten-free diet, and pancreatic enzyme replacement may cure the mineralization defect in mild cases without the need for additional treatment. Patients with severe disease will usually require vitamin D and calcium supplementation. Replacement doses depend on the serum 25-hydroxyvitamin

TABLE 244-2  VITAMIN D PREPARATIONS FOR TREATMENT OF OSTEOMALACIA VITAMIN D3 (CHOLECALCIFEROL)

CALCITRIOL (1,25[OH]2D3)

Trade names

Calciferol, BIOTECH†

Calcitriol

Dosage form

Caps: 50,000 units = 1.25 mg

Caps: 0.25 and 0.50 µg

Dosage:

0.50-2.0 µg/day

If serum 25(OH)D = 20 to 30 ng/mL

50,000 units once a week × 10 weeks* and once a month thereafter

If serum 25(OH)D = 10 to 20 ng/mL

50,000 units twice a week × 10 weeks* and twice a month thereafter

If serum 25(OH)D = less than 10 ng/mL

50,000 units three times a week × 10 weeks* and three times a month thereafter

Dosage in resistant cases

Up to 50,000 units per day

5-20 µg/day

Time to reach maximum effects

4-10 weeks

3-7 days

Persistence of effects after 6-30 weeks cessation

3-7 days

Cost in U.S.

$130/100 capsules of 0.25 µg $150/100 capsules of 0.5 µg

$40/100 capsules of 50,000 units

*If not >30 ng/mL after 10 weeks, exclude malabsorption, gluten enteropathy, and noncompliance. High-quality cholecalciferol, free from gluten, dairy, egg, fish nuts, soy, or artificial colors can be obtained from †BIOTECH at 1-800-345-1199. Weekly tanning bed treatments may be used if oral vitamin therapy fails or a switch to the more costly calcitriol may be necessary. 25(OH)D = 25-hydroxyvitamin D.

D level, as shown in Table 244-2. Because pharmacologic doses of any vitamin D preparation carry the risk for vitamin D intoxication, increases in the dose must be made carefully. The interval between increments in dosage should  be at least the time required to reach maximal effects plus about 50%. However, experience with the doses given in Table 244-2 indicates that serum 25-hydroxyvitamin D levels rarely reach 80 to 100 ng/mL. Vitamin D intoxication is unlikely even with of levels of 200 to 250 ng/mL. The goal is to raise the serum 25-hydoxyvitamin D level well above 30 ng/mL and restore the elevated parathyroid hormone concentration to normal without hypercalcemia or hypercalciuria. Urinary calcium excretion should be monitored when treatment has normalized the serum calcium level. The urinary calcium-to-  creatinine ratio (mg/mg) should be kept below 0.22. Approximately 1 to 1.5 g/ day of oral elemental calcium is a reasonable initial dose. Frequent small doses (three times a day) are more effective and tolerable than fewer larger ones, and the absorbability of calcium supplements is enhanced with meals. Most patient do well using the calcium preparations used for osteoporosis, such as calcium carbonate (40% calcium as in Os-Cal or the equivalent) or calcium citrate (21% calcium as in Caltrate or the equivalent). Some patients who cannot tolerate calcium carbonate or citrate experience fewer adverse gastrointestinal symptoms with the use of the chocolate or coffee-flavored formulations known as Viactiv (500 mg calcium per tablet). The vitamin D content of these calcium supplements is trivial in the treatment of osteomalacia.

CHAPTER 245  The Parathyroid Glands, Hypercalcemia and Hypocalcemia  

In patients with malabsorption, vitamin D requirements may increase during periods of increased diarrhea, and calcitriol may be easier for these patients to absorb. Its rapid onset of action and disappearance after cessation add to the safety of treatment, albeit at far greater cost. Calcium, phosphorus, potassium, magnesium, multivitamins, and gonadal steroids may also be beneficial in patients with malabsorption. Some patients do not tolerate any form of oral vitamin D, and the parenteral ergocalciferol preparations in North America are ineffective. These patients can be improved, although not restored to normal, by the use of weekly tanning bed treatments to areas of their bodies not normally exposed to the sun, an attempt to minimize solar-induced skin cancer. Calcitriol is the drug of choice in patients with the autosomal recessive disease, vitamin D−dependent rickets type I, in which the 1α-hydoxylase enzyme necessary to convert 25-hydroxyvitamin D to 1,25-dihydroxyvitamin D is deficient. In vitamin D−dependent rickets type II, another rare autosomal recessive disease presenting with alopecia, diminished target sensitivity to 1,25-dihydroxyvitamin D may require extraordinarily high doses of calcitriol.  If oral treatment fails, nocturnal infusions of calcium and phosphorus have been successful, providing additional evidence that the osteomalacia is due to inadequate calcium and phosphorus rather than the defect in vitamin D metabolism. An increase in the serum alkaline phosphatase activity (the healing “flare”) and a small increase in the serum and urine calcium levels are the earliest signs of effective treatment. Thereafter, the serum alkaline phosphatase activity level falls progressively as healing occurs. At the start of therapy, serum calcium levels should be measured at weekly intervals. If hypoalbuminemia is present, serum ionized calcium determinations are more useful. When therapy appears stabilized, biweekly or monthly intervals are usually sufficient for the first 3 or 4 months, but even with long-term therapy, measurements should be at least two to three times a year. In some patients with severe osteomalacia, bone pain and paresthesias may increase and the serum calcium levels decrease during the first few weeks of therapy. This is due to the increased skeletal avidity for mineral during healing and indicates the need for additional calcium supplementation.

Osteomalacia Due to Hypophosphatemia

Therapy of chronic hypophosphatemia is aimed at maintaining normal concentrations of serum phosphorus without inducing secondary hyper­ parathyroidism or nephrocalcinosis. This considerably difficult task requires divided doses of phosphorus supplements (1 to 3 g/day) and calcitriol (1 to 4 µg/day) to increase the absorption of phosphorus and try to prevent the increase in parathyroid hormone (Tables 244-2 and 244-3). If phosphorusinduced secondary hyperparathyroidism develops, the phosphorus supplements are rapidly excreted, and therapy thus is not only futile but also causes the additional bone disease of hyperparathyroidism. Baseline and yearly renal ultrasound examinations are necessary to recognize early nephrocalcinosis or nephrolithiasis. X-linked hypophosphatemia (XLH) is the most common cause of chronic hypophosphatemia, and the presence of a positive family history, pediatric onset, and bowed legs usually substantiates the diagnosis. Treatment with an anti-FG23 antibody can raise serum phosphorus and 1,25 vitamin D levels, A1  although the long-term benefits are not yet known. However, some hypophosphatemic patients have an autosomal dominant family history and present in adulthood with osteomalacia but without lower extremity deformities. Like patients affected with XLH, these patients may present with bone pain, pseudofractures, and high-normal or frankly elevated levels of fibroblast growth factor 23 (FGF23), a phosphaturic protein that interferes with 1-hydroxylation of 25-hydroxyvitamin D.7 Patients with autosomal dominant hypophosphatemic osteomalacia (ADHR) appear to acquire the renal phosphate losses in adolescence or adulthood, whereas other patients with ADHR may lose the

1649

defect as they age. A diagnostic problem arises when a patient with chronic hypophosphatemia presents without a positive family history or bowed legs because this presentation resembles that of patients with oncogenic osteomalacia. This disorder is associated with a variety of small, hard to find, benign mesenchymal tumors that secrete FGF23.8,9 The muscle pain, weakness, fractures, and osteomalacia characteristic of this syndrome are due to hypophosphatemia made worse by inappropriately low levels of 1,25-dihydroxy vitamin D. Oncogenic osteomalacia is also treated with phosphorus supplementation and calcitriol until the offending tumor can be located and resected. Improved tumor localization has been reported with positron emission tomography and computed tomography.9-11 Surgical correction of deformities should be postponed until medical management achieves persistently normal levels of calcium, phosphorus, and alkaline phosphatase activity. An exception to this rule is an acute fracture of the femoral neck. Prompt surgical repair may be essential to avoid osteonecrosis. Oncogenic osteomalacia rarely occurs with malignant tumors that secrete FGF23, but unless surgical resection is complete, the osteomalacia will persist. Recent evidence suggests that tumors in surgically difficult locations may be treated by radiofrequency ablation.12 Antacid-induced hypophosphatemia due to the ingestion of large quantities of phosphate-binding antacids has become rare with the increased availability of proton pump inhibitors but still occurs occasionally.

PREVENTION

Advice about vitamin D supplementation should help to prevent osteomalacia caused by vitamin D deficiency, but this has proved to be difficult because routine supplements may be inadequate and compliance with nutritional supplements is poor. The optimal vitamin D supplementation dosage is not clear, but most bone and mineral problems are avoided by 50,000 units of ergocalciferol given once monthly. Notable exceptions occur in patients with celiac disease, gastric surgery, or bypass for obesity, who often require much larger amounts (see Table 244-2). In patients with osteomalacia due to hypophosphatemia, the need for phosphorus supplementation may be lifelong. Rare exceptions occur in oncogenic osteomalacia if complete surgical removal or destruction of the tumor is accomplished.

PROGNOSIS

The response to appropriate treatment in most forms of osteomalacia is usually excellent. Improvements in bone pain and muscle weakness usually occur within 2 or 3 months, and healing of skeletal lesions occurs within 6 to 18 months. Depending on the quantity of excess osteoid, repeat bone mineral density determinations may show as much as 20% gains at the lumbar spine and total proximal femur. However, bone density at the radial diaphysis may not improve because of the irreversible loss of cortical bone resulting from prolonged secondary hyperparathyroidism. Furthermore, if decreased bone volume is present in addition to excess osteoid, skeletal recovery may be incomplete, and the risk for fractures may remain increased.

Grade A Reference A1. Carpenter TO, Imel EA, Ruppe MD, et al. Randomized trial of the anti-FGF23 antibody KRN23 in X-linked hypophosphatemia. J Clin Invest. 2014;124:1587-1597.

GENERAL REFERENCES

TABLE 244-3  PHOSPHATE PREPARATIONS FOR TREATMENT OF OSTEOMALACIA TABLET MARKINGS PREPARATION AND SHAPE Neutra-Phos

0

Neutra-Phos-K

0

K-Phos Neutral

“Beach 11-25” oblong

K-Phos Original

“Beach 1111” round

SODIUM CONTENT (mEq)

POTASSIUM CONTENT (mEq)

AMOUNT THAT CONTAINS 1 GRAM OF ELEMENTAL PHOSPHORUS

28.5

28.5

4 unit dose caps* 4 unit dose caps*

0

57.0

50.4

4.6

4 tabs

0

33.0

9 tabs

*Each unit dose cap is reconstituted with 75 mL of water, fruit juice, or cola, and this formulation is preferred by children. The unit dose cap contains the powder concentrate and is not to be swallowed undiluted. Adults prefer the K-Phos Neutral tablets.

For the General References and other additional features, please visit Expert Consult at https://expertconsult.inkling.com.

245  THE PARATHYROID GLANDS, HYPERCALCEMIA AND HYPOCALCEMIA RAJESH V. THAKKER

CALCIUM METABOLISM

A healthy adult body has a total of 1 kg of calcium; about 99% of this is present within the crystal structure of bone mineral, and less than 1% is in soluble form in the extracellular and intracellular fluid compartments. In the extracellular fluid compartment (ECF), about half of the total calcium is

CHAPTER 244  Osteomalacia And Rickets  

GENERAL REFERENCES 1. Elder CJ, Bishop NJ. Rickets. Lancet. 2014;383:1665-1676. 2. Peris P, Martinez-Ferrer A, Monegal A, et al. 25 Hydroxyvitamin D serum levels influence adequate response to bisphosphonate treatment in postmenopausal osteoporosis. Bone. 2012;51: 54-58. 3. van Schoor NM, Lips P. Worldwide vitamin D status. Best Pract Res Clin Endocrinol Metab. 2011;25:671-680. 4. Berkseth KE, Tebben PJ, Drake MT. Clinical spectrum of hypophosphatasia diagnosed in adults. Bone. 2013;54:21-27. 5. Kulak CA, Dempster DW. Bone histomorphometry: a concise review for endocrinologists and clinicians. Arq Bras Endoocrinol Metab. 2010;54:87-98. 6. Priemel M, von Domarus C, Klatte TO, et al. Bone mineralization defects and vitamin D deficiency: histomorphometric analysis of iliac crest bone biopsies and circulating 25-hydroxyvitamin D in 675 patients. J Bone Miner Res. 2010;25:305-312. 7. Quarles LD. “Dem bones” are made for more than walking. Nat Med. 2011;17:428-430.

1649.e1

8. Manger B, Schett G. Paraneoplastic syndromes in heumatology. Nat Rev Rheumatol. 2014;10: 663-670. 9. Chong WH, Molinolo AA, Chen CC, et al. Tumor-induced osteomalacia. Endocr Relat Cancer. 2011;18:53-77. 10. Chong WH, Andreopoulou P, Chen CC, et al. Tumor localization and biochemical response to cure in tumor-induced osteomalacia. J Bone Miner Res. 2013;28:1386-1398. 11. Clifton-Bligh RJ, Hofman MS, Duncan E, et al. Improving diagnosis of tumor-induced osteomalacia with gallium-68 DOTATATE PET/CT. J Clin Endocrinol Metab. 2013;98:687-694. 12. Jadhav S, Kasaliwal R, Shetty NS, et al. Radiofrequency ablation, an effective modality of treatment in tumor-induced osteomalacia: a case series of three patients. J Clin Endocrinol Metab. 2014;99: 3049-3054.

1649.e2

CHAPTER 244  Osteomalacia And Rickets  

REVIEW QUESTIONS 1. Defective mineralization in osteomalacia is due to lack of one or more of the following except which one? A. Adequate calcium and phosphorus at the remodeling site B. Adequate amount of skeletal alkaline phosphatase C. Normal pH at the site of calcification D. Presence of a normal bone collagen matrix E. Adequate level of fluoride Answer: E  Fluoride is an inhibitor of mineralization. See Pathobiology. 2. The correct sequence of steps involving bone remodeling is represented by which one of the following? A. Reversal, activation, bone resorption by osteoclasts, osteoblast assembly and new bone formation B. Activation, bone resorption by osteoclasts, reversal phase, osteoblast assembly and new bone formation C. Bone resorption by osteoclasts, activation, osteoblast assembly and new bone formation, reversal D. Activation, osteoblast assembly and new bone formation, bone resorption by osteoclasts, reversal E. None of the above Answer: B  See Pathobiology. 3. All of the following may be signs or symptoms of osteomalacia except which one? A. Nonspecific and poorly localized bone pain B. Bone pain after sudden movements C. Fasciculations and absent reflexes D. Flat-footed waddling gait E. Muscle weakness Answer: C  See Clinical Manifestations.

4. Excessive osteoid due to osteomalacia can be reliably distinguished from that caused by increased bone turnover by which one of the following? A. Serum alkaline phosphatase B. Presence of pseudofractures C. Increased radionuclide on a bone scan D. Phosphate level E. Bone histomorphometry using tetracycline labels Answer: E  See Diagnosis. 5. A 30-year-old woman presents to her primary care physician with fatigue, generalized bone pains, weight loss, and a pruritic rash on her elbows and back. Laboratory studies show: hemoglobin 10 (male, 14-17 g/dL; female, 12-16 g/dL); serum iron 20 (60-160 µg/dL); serum calcium 8 (9-10.5 mg/dL). What is the next appropriate diagnostic test? A. Bone mineral density scan B. Hemoglobin electrophoresis C. Colonoscopy D. Antiendomysial immunoglobulin A antibodies E. Skin biopsy Answer: D  The patient has gluten enteropathy, which may occur even without gastrointestinal symptoms. See Treatment.

1650

CHAPTER 245  The Parathyroid Glands, Hypercalcemia and Hypocalcemia  

Dietary Ca++, VitD

+

Parathyroid

ECF

Ca++

Ca++

+

Intestine

Bone

PTH

+

UV Light Skin

7-dehydrocholesterol

Vit D + Ca++

+ Kidney

1,25(OH)2 VitD 25(OH) VitD Liver

1αOHase

24OHase 1,24,25(OH)3 VitD

FIGURE 245-1. Regulation of extracellular fluid (ECF) calcium (Ca2+) by parathyroid hormone (PTH) action on kidney, bone, and intestine. A decrease in ECF Ca2+ is sensed by the calcium-sensing receptor (see Fig. 245-2), and this leads to an increase in PTH secretion and a reduction in PTH degradation. The increased circulating PTH predominantly acts directly on kidney and bone that possess the PTH receptor (PTHR, Fig. 245-2). The skeletal effects of PTH are to increase (+) osteoclastic bone reabsorption. However, because osteoclasts do not have PTHRs, this action is mediated by the osteoblasts, which do have PTHRs and in response release cytokines and factors in turn that activate osteoclasts. In the kidney, PTH stimulates (+) the 1α-hydroxylase (1αOHase) to increase the conversion of 25-hydroxyvitamin D [25(OH)VitD] to the active metabolite 1,25-dihydroxyvitamin D [1,25(OH)2VitD]. In addition, PTH increases (+) the reabsorption of Ca2+ from the renal distal tubule and inhibits the reabsorption of phosphate from the proximal tubule, thereby leading to hypercalcemia and hypophosphatemia. PTH also inhibits Na+, H+ antiporter activity and bicarbonate reabsorption, thereby causing a mild hyperchloremic acidosis. The elevated 1,25(OH)2VitD acts on the intestine to increase (+) absorption of dietary calcium and phosphate. It is important to note that PTH does not appear to have a direct action on the gut. Thus, in response to hypocalcemia and the increase in PTH secretion, all of these direct and indirect actions of PTH on the kidney, bone, and intestine will help to increase ECF Ca2+, which in turn will act through the calcium-sensing receptor to decrease PTH secretion. (From Thakker RV, Bringhurst FR, Jüppner HH. Regulation of calcium homeostasis and genetic disorders that affect calcium metabolism. In: Jameson JL, De Groot LJ, giudice LC, et al., eds. Endocrinology: Adult & Pediatric. 7th ed. Philadelphia: Saunders; 2016.)

ionized, and the rest is principally bound to albumin or complexed with counter-ions. Ionized calcium in the ECF plays an important role in many physiologic pathways, including muscle contraction, secretion of neurotransmitters and hormones, and coagulation pathways. Ionized serum calcium concentrations range from 4.65 to 5.25 mg/dL (1.16 to 1.31 mmol/L), and the total serum calcium concentration ranges from 8.5 to 10.5 mg/dL (2.12 to 2.62 mmol/L).1 However, the usual 2 : 1 ratio of total to ionized calcium may be disturbed by disorders such as metabolic acidosis, which reduces calcium binding by proteins, or by changes in protein concentration, caused by cirrhosis, dehydration, venous stasis, or multiple myeloma1. In view of this, total serum calcium concentrations are adjusted, or “corrected,” to a reference albumin concentration: the actual total serum calcium value is adjusted by adding or subtracting 0.8 mg/dL (0.016 mmol/L) for every 1 g/ dL (1 g/L) of albumin below or above a reference albumin concentration of 4 g/dL (40 g/L), respectively. The control of body calcium involves a balance between the amounts that are absorbed from the gut, deposited into bone and into cells, and excreted from the kidney (Fig. 245-1).2 This fine balance, involving three organs, is chiefly under the control of parathyroid hormone (PTH), which is synthesized and secreted by the parathyroid glands. Hypocalcemia leads to an increased secretion of PTH, whereas hypercalcemia results in diminished PTH secretion. Regulation of extracellular calcium takes place through complex interactions (Fig. 245-2) at the target organs of the major calciumregulating hormone, PTH, and vitamin D and its active metabolites, 1,25-dihydroxyvitamin D (1,25[OH]2D).

PARATHYROID GLANDS, PARATHYROID HORMONE, PTH GENE, AND PARATHYROID HORMONE ACTIONS

Parathyroid Glands

There are usually four parathyroid glands, which are located in close proximity to the superior and inferior poles of the lobes of the thyroid gland. The superior parathyroids are derived from the endoderm of the embryonic fourth pharyngeal pouches, and the inferior parathyroids are derived with the thymus from the endoderm of the third pharyngeal pouches. Extra parathyroid glands are commonly found in aberrant locations along this migrating

path and also within the thymus and thyroid. Parathyroid cells express a G protein−coupled receptor (GPCR), referred to as the calcium-sensing receptor (CaSR), that detects changes in extracellular calcium and leads to alterations in PTH secretions.3 For example, activation of the CaSR, which is also expressed in renal tubular cells, as a result of elevated extracellular calcium concentrations causes G protein−dependent stimulation of phospholipase C activity through Gαq and Gα11, which leads to accumulation of inositol 1,4,5-trisphosphate and an increase in intracellular calcium concentrations.4 These changes, in turn, lead to reduced circulating PTH concentrations and increased urinary calcium excretion. Disorders of the parathyroid glands may cause hypercalcemia or hypocalcemia, and these can be classified according to whether they arise from an excess of PTH, its deficiency, or insensitivity to its effects (Table 245-1; see Fig. 245-2).

Parathyroid Hormone and PTH Gene

The mature PTH peptide is encoded by the PTH gene and secreted from the parathyroid chief cells as an 84−amino acid peptide; however, when the PTH mRNA is first translated, it is as pre-proPTH peptide. The “pre” sequence consists of a 25−amino acid signal peptide (leader sequence) that is responsible for directing the nascent peptide into the endoplasmic reticulum to be packaged for secretion from the cell. The “pro” sequence is 6 amino acids in length and, although its function is less well defined than that of the “pre” sequence, is also essential for correct PTH processing and secretion. After the 84−amino acid mature PTH peptide is secreted from the parathyroid cell, it is cleared from the circulation with a short half-life of about 2 minutes, by nonsaturable hepatic uptake and renal excretion.

Parathyroid Hormone Actions

PTH shares a receptor with PTH-related peptide (PTHrP); this PTH/ PTHrP receptor (see Fig. 245-2) is a member of a subgroup of the G protein−coupled receptor family.5 PTH/PTHrP receptors are expressed in kidney and bone, where PTH is its predominant agonist, and thus PTH acts directly on kidney and bone cells and indirectly on intestinal cells (see Fig. 245-1) to enhance renal calcium reabsorption, release stored calcium in bones into the ECF, and increase gut calcium absorption, respectively. Expression of the PTH/PTHrP receptor also occurs in the brain, heart, skin, lung, liver, and testis, where it mediates the actions of PTHrP. Mutations

CHAPTER 245  The Parathyroid Glands, Hypercalcemia and Hypocalcemia  

PTH is an 84−amino acid peptide encoded by the PTH gene, which is located on chromosome 11p15 and consists of three exons (transcribed regions) that are separated by two introns. Exon 1 of the PTH gene is 85 base pairs (bp) in length and is untranslated, whereas exons 2 and 3 code for the 115−amino acid pre-proPTH peptide. Exon 2 is 90 bp in length and encodes the initiation (ATG) codon, the prehormone sequence, and part of the prohormone sequence. Exon 3 is 612 bp and encodes the remainder of the prohormone sequence, the mature PTH peptide, and the 3′ untranslated region. The 5′ regulatory sequence of the human PTH gene contains a vitamin D response element 125 bp upstream of the transcription start site, which downregulates PTH messenger RNA (mRNA) transcription in response to vitamin D receptor binding. PTH gene transcription (as well as PTH peptide secretion) is also dependent on the extracellular calcium concentration, although the presence of a specific upstream “calcium response element” has not yet been demonstrated.

1650.e1

CHAPTER 245  The Parathyroid Glands, Hypercalcemia and Hypocalcemia  

Ca2+ CaSR

Loss of function FHH1, NSHPT

Gain of function ADH1

FHH2

ADH2, hypoparathyroidism

1651

Parathyroid cell

PLC

Gi

Gq/11

AC

AP2σ

ATP

PIP2

MELAS, KSS MTPD5

cAMP

DAG + IP3

TBCE

[Ca2+]i PKC Proto-oncogenes and tumor-suppressor genes

Transcription factors (e.g., GATA3, GCM2, AIRE, FAM 111A)

PTHrP

Kenny-Caffey type 1 Sanjad-Sakati Parathyroid tumors (e.g., MEN 1, HPT-JT, CCND1, Rb, CDC73)

PTH

Target cell (e.g., kidney, bone, cartilage)

FHH3

Hypoparathyroidism DiGeorge syndrome, HDR, hypoparathyroidism, APECED, Kenny-Caffey type 2

PTH

PTH/PTHrP receptor

Gs

Gq/11

Blomstrand’s lethal chondrodysplasia

Jansen’s metaphyseal chondrodysplasia

Pseudohypoparathyroidism

McCune-Albright syndrome

AC

PLC PIP2 IP3 + DAG

ATP cAMP

Gq/11

ADH2, hypoparathyroidism FBHH, NSHPT

ADH1

CaSR

FIGURE 245-2. Schematic representation of some of the components involved in calcium homeostasis. Alterations in extracellular calcium are detected by the calcium-sensing receptor (CaSR), which is a 1078−amino acid G protein−coupled receptor. The PTH/PTHrP receptor, which mediates the actions of PTH and PTHrP, is also a G protein−coupled receptor. Thus, Ca2+, PTH, and PTHrP involve G protein−coupled signaling pathways, and interaction with their specific receptors can lead to activation of Gs, Gi, and Gq, respectively. Gs stimulates adenylcyclase (AC), which catalyzes the formation of cyclic adenosine monophosphate (cAMP) from adenosine triphosphate (ATP). Gi inhibits AC activity. cAMP stimulates protein kinase A (PKA), which phosphorylates cell-specific substrates. Activation of Gq stimulates phospholipase C (PLC), which catalyzes the hydrolysis of the phosphoinositide (PIP2) to inositol triphosphate (IP3), which then increases intracellular calcium, and diacylglycerol (DAG), activating protein kinase C (PKC). These proximal signals modulate downstream pathways, which result in specific physiologic effects. Loss of function in several genes, shown with their respective sites of action on the right, has been identified in specific disorders of calcium homeostasis (also see Table 245-1). (From Thakker RV, Bringhurst FR, Jüppner H. Regulation of calcium homeostasis and genetic disorders that affect calcium metabolism. In: Jameson JL, De Groot LJ, giudice LC, et al., eds. Endocrinology: Adult & Pediatric. 7th ed. Philadelphia: Saunders; 2016.)

involving the genes that encode these proteins and receptors in this calciumregulating pathway (see Fig. 245-2) are associated with hypercalcemic and hypocalcemic disorders (see Table 245-1).

Renal Actions

Calcium is absorbed at multiple sites and by different mechanisms, which include passive paracellular or active transcellular transport, along the renal tubule.2 The renal actions of PTH are to (1) stimulate activity of the proximal tubular cell 1α-hydroxylase; (2) increase reabsorption of calcium by the cells of the distal tubule, connecting tubules and the thick ascending loop of Henle (TAL); and (3) inhibit phosphate reabsorption by proximal tubular cells (see Fig. 245-1). PTH increases the formation of biologically active 1,25(OH)2D from its precursor 25-OH-D by stimulating the activity of the renal 1α-hydroxylase and inhibiting the 24-hydroxylase, which metabolizes 1,25(OH)2D to the inactive 24,25(OH)2D form (see Fig. 245-1). PTH regulates calcium reabsorption by distal tubular cells by upregulating expression

of the transient receptor potential vanilloid 5 (TRPV5), thereby promoting calcium entry into the cell, and increasing calbindin-D28K expression to enhance transcellular calcium reabsorption by increased buffering of subapical Ca2+ ions. In the TAL, PTH may increase active transcellular transport of calcium, as well as paracellular calcium transport, by augmenting the transepithelial voltage gradient. Phosphate transport in proximal tubular cells is mediated by the luminal membrane sodium-phosphate cotransporters 2a and 2c (NPT2a and NPT2c), and PTH actions lead to internalization and degradation of NPT2a and NPT2c, thereby resulting in decreased reabsorption of phosphate.

Skeletal Actions

PTH acts directly on osteoblasts and indirectly on osteoclasts to increase their numbers and activity, thereby enhancing bone turnover and release of stored calcium. Thus, PTH increases the size of the osteoblast precursor pool, increases the bone-forming activity of mature osteoblasts, and stimulates

1652

CHAPTER 245  The Parathyroid Glands, Hypercalcemia and Hypocalcemia  

TABLE 245-1  PARATHYROID DISEASES AND THEIR CHROMOSOMAL LOCATIONS METABOLIC

CHROMOSOMAL

ABNORMALITY

DISEASE

INHERITANCE

GENE/GENE PRODUCT

LOCATION

HYPERCALCEMIA Multiple endocrine neoplasia type 1 Multiple endocrine neoplasia type 2 Hereditary hyperparathyroidism and jaw tumors (HPT-JT) Sporadic hyperparathyroidism

Autosomal dominant Autosomal dominant Autosomal dominant

MENIN RET PARAFIBROMIN

11q13 10q11.2 1q31.2

Sporadic

Parathyroid carcinoma

Autosomal dominant or sporadic

PRAD1/CCND1 Retinoblastoma Unknown PARAFIBROMIN

11q13 13q14 1p32-pter 1q31.2

Retinoblastoma

13q14

CaSR Gα11 AP2S1 CaSR

3q 21.1 19p13 19q13 3q21.1

PTHR/PTHrP receptor Elastin, LIMK (and other genes) CYP24A Gsα

3p21.3 7q11.23 20q13.2-q13.3 20q13.3

Autosomal dominant Autosomal recessive X-linked recessive Autosomal dominant Autosomal dominant Autosomal recessive

PTH, GCMB PTH, GCMB SOX3 CaSR Gα11 AIRE-1

11p15* 11p15*, 6p24.2 Xq26–27 3q21.1 19p13 21q22.3

Maternal

Mitochondrial genome

Autosomal dominant Autosomal dominant Autosomal recessive Autosomal dominant

TBX1 GATA3 PTHR/PTHrP receptor TBCE

22q11.2/10p 10p15 3p21.3 1q42.3

Autosomal recessive Autosomal recessive† Autosomal recessive Autosomal dominant† Autosomal dominant Autosomal dominant parentally imprinted Autosomal dominant parentally imprinted

FAMIIIA Unknown Unknown Unknown Unknown? GNAS exons 1-3

11q12.1 ? ? ? ? 20q13.3

GNAS Upstream deletion

20q13.3

Familial benign hypercalcemia (FBH)   FBH1   FBH2   FBH3 Neonatal severe hyperparathyroidism (NSHPT) Jansen’s disease Williams syndrome Infantile hypercalcemia McCune-Albright syndrome

Autosomal dominant Autosomal dominant Autosomal dominant Autosomal recessive or autosomal dominant Autosomal dominant Autosomal dominant Autosomal recessive Mutations during early embryonic development?

HYPOCALCEMIA Isolated hypoparathyroidism Autosomal dominant hypocalcemia type 1 (ADH1) Autosomal dominant hypocalcemia type 2 (ADH2) Hypoparathyroidism associated with polyglandular autoimmune syndrome (APECED) Hypoparathyroidism associated with Kearns-Sayre and MELAS Hypoparathyroidism associated with complex congenital syndromes   DiGeorge syndrome   HDR syndrome   Blomstrand’s lethal chondrodysplasia   Kenney-Caffey syndrome type 1, Sanjad-Sakati syndrome   Kenney-Caffey syndrome type 2   Barakat syndrome   Lymphedema   Nephropathy, nerve deafness   Nerve deafness without renal dysplasia Pseudohypoparathyroidism (type 1a) Pseudohypoparathyroidism (type 1b)

HDR = hypoparathyroidism, deafness, and renal dysplasia; MELAS = mitochondrial encephalopathy, stroke-like episodes, and lactic acidosis; ? = location not known. *Mutations of PTH gene are identified only in some families. † Most likely inheritance.

osteoblasts to release cytokines such as colony-stimulating factor 1 and receptor activator of nuclear factor-κB (NF-κB) ligand (RANKL), which stimulate the formation of new osteoclasts and activate mature osteoclasts. PTH also inhibits osteoblast production of osteoprotegrin (OPG), which is a soluble decoy receptor for RANKL that inhibits osteoclast development. Calcium transport involves TRPV4 and TRPV5 in bone cells; TRPV4 regulates intracellular calcium concentrations in osteoblasts and osteoclasts, whereas TRPV5, expressed in osteoclasts, participates to remove the mineral bone matrix.2 The net result of persistent elevations of PTH is linked to an increase in osteoclast activity more than osteoblast activity, hence liberating the stores of calcium to the ECF (see Fig. 245-1).

Intestinal Actions

Calcium is absorbed throughout the intestine by passive paracellular routes and active transcellular routes, which involve TRPV6 and calbindin D9K. PTH exerts indirect actions on intestinal calcium absorption by increasing the circulating 1,25(OH)2D concentrations (see Fig. 245-1). The increased 1,25(OH)2D concentrations increase TRPV6 expression, which facilitates enhanced calcium entry into the cell from the lumen, and cytosolic calbindin D9K expression, which facilitates transcellular transport of calcium.

HYPERCALCEMIA DEFINITION

Hypercalcemia is defined as a serum calcium concentration greater than 2 standard deviations above the normal mean, and this is usually a total serum calcium above 10.5 mg/dL (2.62 mmol/L) and an ionized serum calcium of above 5.25 mg/dL (1.31 mmol/L). There is no formal grading system for defining the severity of hypercalcemia, but mild, moderate, and severe hypercalcemia is generally considered for total serum calcium concentrations less than 12 mg/dL (3 mmol/L), between 12 and 14 mg/dL (3 to 3.5 mmol/L), and greater than 14 mg/dL (3.50 mmol/L), respectively.

PATHOBIOLOGY

Hypercalcemia may arise through one of three mechanisms: increased bone resorption, increased gastrointestinal absorption of calcium, and decreased renal calcium excretion (see Fig. 245-1). For example, lytic bone metastases cause increased bone resorption; thiazide diuretics lead to a decrease in calcium excretion; and excessive PTH will either directly or indirectly, by increasing 1,25(OH)2D production, stimulate bone resorption and calcium

CHAPTER 245  The Parathyroid Glands, Hypercalcemia and Hypocalcemia  

absorption from the gut and renal tubules.6 The causes of hypercalcemia may be classified according to whether serum PTH concentrations are elevated (i.e., primary or tertiary hyperparathyroidism due to parathyroid tumors) or reduced (i.e., not due to parathyroid tumors but instead to an excessive production of PTHrP by a cancer; a defect in the PTH receptor, for example, the PTH/PTHrP receptor; an excess production of downstream mediators, for example, 1,25(OH)2D; or an altered set point in the calcium-sensing receptor) (Table 245-2; see Fig. 245-2). Primary hyperparathyroidism and malignancy are the most common causes and account for more than 90% of patients with hypercalcemia. Detailed clinical history and examination will usually help to differentiate between these two diagnoses. In primary hyperparathyroidism, the hypercalcemia is often less than 12 mg/dL (3 mmol/L), asymptomatic, and may have been present for months or years. If symptoms, such as nephrolithiasis, are present, then they have usually been present for several months. However, in malignancy, the patients are usually acutely ill, often with neurologic symptoms; the hypercalcemia is more than 12 mg/dL (3 mmol/L); and the cancer (e.g., lung, breast, or myeloma) is often readily apparent. Hypercalcemia from causes other than primary hyperparathyroidism or malignancy may also occur (see Table 245-2), and a careful history (e.g., for vitamin D ingestion, drugs, renal disease) and examination (e.g., for thyrotoxicosis, adrenal disease, granulomatous diseases), together with appropriate investigations (Table 245-3; Fig. 245-3), are essential for establishing the diagnosis.

CLINICAL MANIFESTATIONS AND DIAGNOSIS

The clinical presentation of hypercalcemia varies from a mild, asymptomatic, biochemical abnormality detected during routine screening to a lifethreatening medical emergency. In general, the presence or absence of symptoms correlates with the severity and rapidity of onset of the hypercalcemia.

TABLE 245-2  CAUSES OF HYPERCALCEMIA

1653

Thus, symptoms do not usually develop when serum calcium is below 12 mg/dL (3 mmol/L) and are invariably present when the hypercalcemia exceeds 14 mg/dL (3.5 mmol/L). However, there is a considerable variability, and some patients may be symptomatic with mild hypercalcemia. Although there are many causes of hypercalcemia (see Table 245-2), the signs and symptoms of hypercalcemia are similar, regardless of etiology. Indeed, the clinical manifestations of hypercalcemia involve several organ systems that include the renal, musculoskeletal, gastrointestinal, neurologic, and cardiac systems (Table 245-4), and many of these have been referred to as “moans, groans, pains, and stones.” Investigations should be directed at confirming the presence of hypercalcemia and establishing the cause (Table 245-5; see Table 245-3).

TABLE 245-3  PRELIMINARY INVESTIGATIONS FOR HYPERCALCEMIA BLOOD × 2-3 estimations of serum calcium, phosphate, albumin, urea and electrolytes, creatinine, alkaline phosphatase, liver function tests Parathyroid hormone Complete blood count Electrophoretic protein strip or serum protein electrophoresis 25-OH-D3 (and if indicated, 1,25[OH]2D3) Thyroid function tests Magnesium Parathyroid hormone−related peptide (if malignancy suspected) URINE × 2-3 estimations of 24-hr urinary calcium and creatinine clearance, and clearance ratios Imaging Chest radiograph Radiograph of hands Ultrasound of kidneys

HIGH PARATHYROID HORMONE LEVELS Primary hyperparathyroidism* (adenoma, hyperplasia, or carcinoma): nonfamilial or familial, e.g., MEN 1, MEN 2, HPT-JT, FIHP Tertiary hyperparathyroidism (hyperplasia or adenoma in chronic renal failure) LOW PARATHYROID HORMONE LEVELS Malignancy* Primary • Parathyroid hormone−related peptide (PTHrP): carcinoma of lung, esophagus, renal cell, ovary, and bladder • Excess production of 1,25(OH)2D (lymphoma) Secondary • Lytic bone metastases* (multiple myeloma* and breast carcinoma*) • Other location, ectopic factors (e.g., cytokines) Excess vitamin D Exogenous vitamin D toxicity by parent D compound, 25(OH) vitamin D3, or 1,25(OH)2 vitamin D3 in vitamin preparations, cod liver oil, herbal medicines Endogenous production of 25(OH) vitamin D3—Williams syndrome Endogenous production of 1,25(OH)2 vitamin D3, e.g., granulomatous disorders (sarcoidosis, HIV, TB, histoplasmosis, coccidioidomycosis, leprosy), lymphoma, and infantile hypercalcemia Drugs Thiazide diuretics Lithium Total parenteral nutrition Estrogens/antiestrogens, testosterone Milk-alkali syndrome Vitamin A toxicity Aluminum intoxication (in chronic renal failure) Aminophylline Nonparathyroid endocrine disorders Thyrotoxicosis Pheochromocytoma Acute adrenal insufficiency Vasoactive intestinal polypeptide hormone producing tumor (VIPoma) Immobilization INAPPROPRIATE PARATHYROID HORMONE LEVELS DUE TO ALTERED SET POINT Familial benign hypocalciuric hypercalcemia (FBH or FHH) *Most common causes. FIHP = familial isolated hyperparathyroidism; HIV = human immunodeficiency virus; HPT-JT = hyperparathyroidism with jaw tumors; MEN = multiple endocrine neoplasia; TB = tuberculosis.

TABLE 245-4  CLINICAL FEATURES OF HYPERCALCEMIA Renal Stones (nephrolithiasis) and nephrocalcinosis, polyuria, polydipsia Musculoskeletal Bone pain, osteopenia, fractures, muscular weakness, especially proximal myopathy Gastrointestinal Nausea, vomiting, lack of appetite, constipation, peptic ulcers, and pancreatitis Neurologic Tiredness, lethargy, inability to concentrate, increased sleepiness, depression, confusion, coma Cardiac Bradycardia, first-degree atrioventricular block, arrhythmias, shortened QT interval

TABLE 245-5  SUMMARY OF GUIDELINES FOR PARATHYROID SURGERY IN PRIMARY HYPERPARATHYROIDISM PATIENTS Surgery* recommended if patient meets any one of the following criteria: • Serum calcium >1 mg/dL (0.25 mmol/L) above upper limit of normal • Any complication of primary hyperparathyroidism (e.g., nephrolithiasis† or bone erosions of osteitis fibrosa cystica) • An episode of acute primary hyperparathyroidism with life-threatening hypercalcemia • Significant reduction in creatinine clearance (i.e., 10.5 mg/dL (2.62 mmol/L) or ionized calcium > 5.25 mg/dL (1.31 mmol/L)

Confirm hypercalcemia and undertake clinical assessment for risk factors (e.g., malignancy, chronic renal failure, excessive vitamin D intake, drugs such as thiazides, family history of MEN or FHH)

Undertake appropriate tests as outlined in Table 245-3, and evaluate results

Suppressed PTH

Consider malignancy and outcome of clinical evaluations and serum tests (e.g., EPS or PTHrP), or radiology (e.g., chest radiograph)

25-OH-D elevated

Elevated (or normal) PTH

Normal 25-OH-D

Elevated serum creatinine and PO4 and EGFR 0.01

Normal serum creatinine and PO4, and UCCR 9 mmol/L per 24 hr) was included as a recommendation for parathyroid surgery in asymptomatic primary hyperparathyroidism by the Second International Conference (2002), but not by the Third International Conference (2008). However, some physicians still regard marked hypercalciuria as an indication for parathyroid surgery.

1661.e3

4. Which of the following statements regarding parathyroid tumors in multiple endocrine neoplasia (MEN) syndrome is false? A. In MEN 1, parathyroid tumors occur with equal frequency in men and women. B. In MEN 2b and MEN 3, parathyroid tumors are a rare occurrence. C. In MEN 1, minimally invasive surgery is not a suitable approach because of the high occurrence of multiple parathyroid tumors. D. Patients with MEN 2a have a higher risk for developing parathyroid carcinomas. E. Parathyroid tumors occur in more than 95% of MEN 1 patients. Answer: D  MEN 1 is an autosomal dominant disorder, and this generally affects men and women equally. Parathyroid tumors are found in more than 95% of patients with MEN 1, who usually have multiple parathyroid tumors, and hence minimally invasive surgery is not recommended. Parathyroid tumors rarely occur in patients with MEN 2b or MEN 3, and about 20% of MEN 2a patients will usually have parathyroid hyperplasia and not parathyroid carcinoma. However, patients with hyperparathyroidism with jaw tumors are at high risk for developing parathyroid carcinomas. 5. Which of the following is not a cause of hypocalcemia? A. Vitamin D−resistance disorders B. Secondary hyperparathyroidism C. Adrenal insufficiency D. Autoimmunity E. Acute pancreatitis Answer: C  Acute adrenal insufficiency is associated with hypercalcemia, not hypocalcemia. Hypocalcemia typically occurs in patients with secondary hyperparathyroidism and in those with vitamin D−resistance disorders, and it may be found in patients with acute pancreatitis. In addition, autoimmune destruction of the parathyroids, resulting in hypoparathyroidism, or autoantibodies to the calcium-sensing receptor may be associated with hypocalcemia.

1662

CHAPTER 246  Medullary Thyroid Carcinoma  

246  MEDULLARY THYROID CARCINOMA SAMUEL A. WELLS, JR.

MEDULLARY THYROID CARCINOMA DEFINITION

Medullary thyroid carcinoma (MTC) is an uncommon cancer that arises from the neural crest—derived C cells of the thyroid gland.

EPIDEMIOLOGY

MTC accounts for 5% of thyroid cancers, and there will be approximately 3000 new cases in the United States in 2015. MTC occurs either sporadically (75% of cases) or as part of the multiple endocrine neoplasia (MEN) syndromes, MEN 2A (Online Mendelian Inheritance in Man [OMIM] #171400) or MEN 2B (OMIM #162300), or the related syndrome familial medullary thyroid carcinoma (FMTC) (OMIM #155240).1 The incidence of MEN 2A and FMTC combined is approximately 1 in 100,000 live births, whereas the incidence of MEN 2B is approximately 1 in 2,000,000 live births.

PATHOBIOLOGY

As the lateral thyroid complex closes during embryogenesis, the C cells are incorporated within the middle and upper portions of the thyroid lobes. Because of its anatomic location, MTC is classified as a thyroid tumor; however, considering its origin from the neural crest rather than the thyroid follicular cells, it is a neuroendocrine tumor. Sporadic MTC occurs as a solitary tumor in one thyroid lobe, whereas hereditary MTC develops in both thyroid lobes and is multicentric. In patients with hereditary MTC, the first manifestation of a C-cell disorder is C-cell hyperplasia (CCH) that progresses over time to microinvasive MTC and then to invasive MTC.2 The C-cell mass is much greater in the thyroid glands of men than in women, which accounts for the higher serum calcitonin levels seen in men compared with women. The C cells have diverse biosynthetic activity and secrete calcitonin (CTN) and carcinoembryonic antigen (CEA), which are excellent serum markers for the presence of a C-cell disorder. CTN was once thought to be important in calcium homeostasis; however, its physiologic importance has been called into question. The RET protooncogene encodes a single-pass transmembrane receptor of the tyrosine kinase family of proteins. At several stages of development, it is expressed in cells derived from the branchial arches (parathyroids), the neural crest (brain, parasympathetic and sympathetic ganglia, thyroid C cells, adrenal medulla, and enteric ganglia), and the urogenital system. Activating, germline point mutations in RET are present in virtually all hereditary MTCs, and somatic RET mutations are present in approximately half of sporadic MTCs.3,4 Recently, it was discovered that somatic mutations in HRAS, KRAS, and rarely NRAS are present in sporadic MTCs and are almost always mutually exclusive with the presence of somatic RET mutations.5 Approximately 75 RET mutations have been reported in association with MEN 2A, MEN 2B, and FMTC. The mutations for MEN 2A and FMTC are located in exons 5, 8, 10, 11, 13, 14, 15, and 16. The mutations for MEN 2A are mostly located in the extracellular, cysteine-rich region of exon 10 (including codons 609, 611, 618, and 620) and exon 11 (including codons 630 and 634). Approximately 85% of the mutations associated with MEN 2A involve RET codon 634, about half of which are C634R RET mutations. The RET mutations in MEN 2B cause constitutive activation, which alters substrate specificity, presumably owing to a conformational change in the binding pocket of the kinase. Approximately 95% of mutations causing MEN 2B are in codon M918T, and 5% are in codon A883F. Rare cases of MEN 2B are caused by double somatic RET mutations involving codon V804M and either codon Y806C, S904C, or E805K. In 50% of patients with MEN 2B and 10% of patients with MEN 2A and FMTC, the disease arises de novo. In such founder cases, the de novo mutation almost always derives from the paternal allele.6

CLINICAL MANIFESTATIONS

The peak incidence of sporadic MTC is in the fifth decade of life, and most patients present with a solitary thyroid nodule and lymph node metastases. Clinically, the tumors are more aggressive than papillary thyroid carcinoma

TABLE 246-1  CLINICAL MANIFESTATIONS OF MULTIPLE ENDOCRINE NEOPLASIA 2A, 2B, AND FAMILIAL MEDULLARY THYROID CARCINOMA MULTIPLE ENDOCRINE NEOPLASIA (MEN) 2A Medullary thyroid carcinoma (~100%) Pheochromocytoma (incidence of 50% in families with a RET codon 634 germline mutation but less in families with other RET codon mutations) Hyperparathyroidism (incidence of 30% in families with a RET codon 634 germline mutation but less in families with other RET codon mutations.) VARIANTS OF MEN 2A MEN 2A with cutaneous lichen amyloidosis (almost always associated with a RET codon 634 germline mutation.) MEN 2A with Hirschsprung disease (most common in families with RET germline mutation most commonly involving codon 620) FAMILIAL MEDULLARY THYROID CARCINOMA (FMTC) Since the original description of this syndrome, there has been confusion about the designation FMTC. Most clinicians now consider it a variant of MEN 2A. MEN 2B Medullary thyroid carcinoma (~100%) Pheochromocytoma (50%) Mucosal neuroma, ganglioneuromatosis, marfanoid habitus, colonic abnormalities, characteristic physical appearance (~100%)

and follicular thyroid carcinoma but less aggressive than anaplastic thyroid carcinoma (Chapter 226). The 10-year survival rate is 75%. In the expectation of detecting MTC at any early stage, clinicians in Europe evaluate serum CTN levels in patients with thyroid nodules who have no history of hereditary MTC. The detection rate of MTC is less than 0.5%, however, and clinicians in the United States have not adopted this practice. The clinical manifestations of MEN 2A, MEN 2B, and FMTC are listed in Table 246-1. MEN 2A (80% of cases), MEN 2B (5% of cases), and FMTC (15% of cases) are inherited as autosomal dominant traits with near-complete penetrance and, in the cases of MEN 2A and MEN 2B, variable expressivity. Approximately 50% of patients with MEN 2A (and a codon 634 mutation) develop pheochromocytomas (Chapters 228 and 231), the frequency being much lower in association with mutations in codons 609, 611, 618, and 620.7 Before the availability of biochemical and genetic screening in families with MEN 2A, the most frequent cause of death was pheochromocytoma, not MTC. The deaths occurred most often in patients during childbirth or interventional procedures. Thus, pheochromocytoma must be excluded in patients with a confirmed or presumptive diagnosis of hereditary MTC (Chapter 228). With rare exceptions, the pheochromocytoma should be excised first in patients who also have MTC. Parathyroid hyperplasia occurs in up to 30% of patients with MEN 2A and is usually associated with a RET codon 634 mutation. The disease is frequently asymptomatic, with the only abnormality being an elevated serum calcium concentration.8 Patients with MEN 2A may also develop cutaneous lichen amyloidosis (CLA) or Hirschsprung disease (HD).9 CLA occurs in about 25% of patients and involves the interscapular region of the back, corresponding to dermatomes T2 through T6. Pruritus, the dominant symptom, leads to repetitive scratching and secondary skin changes characterized by the deposition of amyloid. The lesion may be evident in infancy, thus serving as a precursor marker of MEN 2A. Cutaneous lichen amyloidosis is almost always associated with a RET codon 634 mutation. HD, manifested by the absence of intrinsic ganglion cells in the distal gastrointestinal tract, has been reported in 30 or more families with MEN 2A or FMTC and is associated with mutations in RET exon 10 involving codons 609 (15%), 611 (4%), 618 (30%), and 620 (50%). In functional studies, the cell surface expression of RET with these codon mutations is lower than that found with a codon 634 mutation. This suggests a novel mechanism whereby the specified RET mutations have low transforming activity, which is sufficient to trigger the development of MTC and pheochromocytoma, yet is insufficient to stimulate differentiation of intestinal ganglion cells. It is also of interest that 50% of patients with familial HD and 30% of patients with sporadic HD have germline RET mutations. Patients with MEN 2B develop mucosal neuromas, ganglioneuromatosis throughout the aerodigestive tract, hypotonia, skeletal malformations, and medullated corneal nerves. They also develop colonic dysfunction manifested by abdominal pain and occasionally intestinal obstruction. Patients

CHAPTER 246  Medullary Thyroid Carcinoma  

have a characteristic physical appearance, which may not be evident early in life. The failure to diagnose MEN 2B at a young age can be catastrophic because MTC is often evident soon after birth, and regional or distant metastases occur soon thereafter. The MTC associated with MEN 2B is much more aggressive than that occurring with MEN 2A or FMTC. The primary basis for the difference is that MEN 2B mutations are associated with significantly higher basal kinase activity compared with mutations in MEN 2A and FMTC. Patients with FMTC develop only MTC, which, relative to the tumors in patients with MEN 2A and MEN 2B, is slow growing. Many clinicians consider FMTC a variant of MEN 2A.

DIAGNOSIS

The measurement of serum levels of CTN, either in the basal state or following the intravenous administration of the secretagogues calcium, pentagastrin, or both, was initially the primary method of establishing the diagnosis of a C-cell disorder. With the discovery that MEN 2A, MEN 2B, and FMTC are caused by mutations in the RET protooncogene, direct DNA analysis became the method of choice for identifying affected family members who had inherited a mutated RET allele. At present, the determination of CTN is primarily used to detect persistent or recurrent MTC following thyroidectomy or to evaluate response to therapy in patients with regional or metastatic disease. As we have learned more about the variable clinical expression of MEN 2A in families with the identical RET mutation, however, the measurement of basal and stimulated serum CTN levels has assumed importance in timing early thyroidectomy in young family members with RET mutations.10 The two-site, two-step, chemiluminescent, immunometric assay that is highly specific for monomeric CTN is the preferred method for quantitating serum CTN levels. At present, direct DNA analysis of RET has become the preferred method of detecting RET mutations in families with hereditary MTC. The Gene Tests directory currently lists 63 laboratories that perform DNA analysis for RET mutations (http://www.genetests.org). Almost all laboratories use direct sequence analysis to evaluate mutations in exons 10, 11, 13, 14, 15, and 16, and some laboratories include exon 8. If no mutations are found in these exons, the entire coding region of RET can be sequenced. It is important to perform direct DNA analysis for RET mutations in all patients with presumed sporadic MTC because approximately 7% of them will have hereditary MTC. A diagnosis of hereditary MTC in this setting mandates a different treatment strategy for the patient, as well as his family members, who should be offered the opportunity for clinical evaluation and genetic testing.

TREATMENT  The primary treatment for patients with MTC, whether sporadic or hereditary, is total thyroidectomy. Resection of lymph nodes in the central compartment is included in all adults and in children with MEN 2B but is excluded in outwardly normal youngsters with MEN 2A and FMTC who are undergoing early thyroidectomy based on directed DNA analysis. If enlarged cervical lymph nodes are evident on preoperative ultrasound examination or at the time of thyroidectomy, the involved anatomic nodal compartment should also be resected. During the thyroidectomy, great care must be taken to preserve the parathyroid glands, the recurrent laryngeal nerves, and the external branch of the superior laryngeal nerve. Postoperatively, serum calcitonin is normal in only 10% of patients with node-positive disease compared with 60% of patients with node-negative disease. Many patients with regional lymph node metastases have a good prognosis, however, with 5- and 10-year survival rates of 80 and 70%, respectively. Repeat neck operation following initial thyroidectomy is indicated in patients with complications from recurrent tumor compressing or invading vital structures, such as the spinal cord, airway, or esophagus. Also, patients who have intractable diarrhea due to markedly elevated tumor hormone secretions, presumably CTN, may obtain symptom relief by tumor debulking. Patients who develop persistent or recurrent MTC following thyroidectomy, as indicated by elevated serum levels of CTN or CEA, are also candidates for reoperation; however, the benefit of such surgical procedures is open to question because there are no long-term data on quality of life and survival. Rarely, patients with MTC develop Cushing syndrome (Chapter 227) due to the inappropriate secretion of adrenocorticotrophic hormone (ACTH) or corticotropinreleasing hormone. Such patients have advanced disease, and bilateral adrenalectomy may be required if steroidogenesis inhibitors are ineffective. Inappropriate secretion of ACTH is a poor prognostic sign, associated with an average survival of 2 years. The treatment of pheochromocytoma is adrenalectomy, as described in Chapter 228. Hyperparathyroidism (Chapter 245) is managed by either 

1663

subtotal parathyroidectomy or total parathyroidectomy with heterotopic autotransplantation. For patients with locally advanced or metastatic MTC, single-agent or combined chemotherapeutic regimens have been minimally effective, being characterized by low response rates of short duration. External beam radiotherapy is indicated primarily for the treatment of localized metastases, primarily of the central nervous system or bone. With the demonstration that the tyrosine kinase inhibitor imatinib induced remissions in patients with chronic myelogenous leukemia and gastrointestinal stromal tumors, there was hope that similar molecular targeted therapeutics (MTTs) would be developed for other solid tumors, including MTC. In a recent prospective, randomized, placebo-controlled, double-blind, phase III trial, patients treated with the MTT vandetanib had a significantly prolonged progression-free survival compared with placebo. A1  On the basis of this study, the U.S. Food and Drug Administration (FDA) approved vandetanib for the treatment of patients with advanced MTC.11 The FDA also recently approved a second MTT, cabozantinib,12 based on similar results of a phase III trial. A2  Thus, effective systemic therapies are available for patients with advanced MTC, and additional studies of other MTTs have recently been initiated.

PREVENTION

In patients with a hereditary cancer syndrome, the removal of an organ destined to become malignant should be considered in the light of five factors. There should be (1) near-complete penetrance of the mutated gene, (2) a reliable method of detecting family members who have inherited a mutated allele, (3) minimal morbidity associated with removal of the organ at risk, (4) excellent replacement therapy for the function of the removed organ, and (5) a reliable method for determining whether the operative procedure has been curative. Few hereditary malignancies meet all of these criteria; fortunately, MEN 2A, MEN 2B, and FMTC meet each of them. Young members of kindred with hereditary MTC who are found to have a mutated RET allele on genetic screening have the greatest likelihood of being cured by early thyroidectomy. Surgeons in several countries have reported success with this operative procedure, and the question is no longer should it be done but at what age. The Consensus Committee of the 7th International Workshop on MEN, the National Comprehensive Cancer Network, and the American Thyroid Association have all proposed guidelines for the timing of prophylactic thyroidectomy in patients with MEN 2A, MEN 2B, and FMTC13. The recommendations of the three groups are similar, in that children with MEN 2B (or with mutations in codons 918 or 882) should have thyroidectomy at the time of diagnosis, even during the first months of life. Children with MEN 2A and mutations in codons 611, 618, 620, or 634 should have the thyroid removed at or before 5 years of age. In children with mutations in other RET codons, the recommended timing of thyroidectomy is less clear but is generally between 5 and 10 years of age.

PROGNOSIS

Several factors portend an adverse outcome in patients with MTC.14 Poor prognosis is associated with older age, advanced disease at the time of diagnosis of a large primary tumor, lymph node metastases, markedly elevated serum levels of CTN and CEA preoperatively, extrathyroidal invasion of the trachea or soft tissues, and distant metastases. Patients with MEN 2B and patients with MEN 2A who have RET mutations in codon 634 have a poorer prognosis than those with RET mutations in other codons. Also, in patients with sporadic MTC, the presence of a RET M918T mutation, compared with other codon mutations, is associated with a more aggressive tumor and a poor prognosis. Patients apparently cured by thyroidectomy are followed at 6-month intervals with measurement of serum levels of CTN and CEA. The doubling times of serum CTN are especially useful in predicting the course of the disease. CTN doubling times of less than 6 months (compared with those greater than 24 months) are associated with a very poor prognosis.

Grade A References A1. Wells SA Jr, Robinson BG, Gagel RF, et al. Vandetanib in patients with locally advanced or metastatic medullary thyroid cancer: a randomized, double-blind phase III trial. J Clin Oncol. 2012;30:134-141. A2. Elisei R, Schlumberger MJ, Müller SP, et al. Cabozantinib in progressive medullary thyroid cancer. J Clin Oncol. 2013;31:3639-3646.

1664

CHAPTER 247  Paget Disease of Bone  

GENERAL REFERENCES For the General References and other additional features, please visit Expert Consult at https://expertconsult.inkling.com.

247  PAGET DISEASE OF BONE

the rest of the world through emigration. Paget disease is strongly related to age; in the United Kingdom, the incidence is 0.3 to 0.5 per 10,000 person years in those aged 55 to 59 years but doubles in frequency each decade thereafter to reach an incidence of 5.4 per 10,000 person years in women and 7.6 per 10,000 person years in men in those 85 years and older. The prevalence and severity of Paget disease have diminished in most countries over the past 25 years.1 The causes are unclear, but suggested explanations include influx of migrants from low prevalence areas in some populations, improved nutrition, a more sedentary lifestyle with a reduction in skeletal injuries, and reduced exposure to infections.2

PATHOBIOLOGY

STUART H. RALSTON

DEFINITION

Paget disease of bone is a disorder of skeleton characterized by increased and disorganized bone remodeling affecting one or more skeletal sites. Affected bones enlarge, become deformed, and are at increased risk for pathologic fractures

EPIDEMIOLOGY

The population prevalence of Paget disease is about 1% in the United States and 2% in the United Kingdom. It is also common in Western Europe and in people of European descent who have migrated to other parts of the world. Paget disease is rare in Scandinavians, Africans, and Asians. These differences are thought to have a genetic basis and to be caused by founder mutations that occurred in Europeans many centuries ago with subsequent spread to

Susceptibility to Paget disease seems to be genetically determined, but environmental factors also play a key role in regulating onset and severity of the disease. The importance of genetics is emphasized by the fact that between 15 and 40% of patients have a positive family history and that the risk for developing Paget disease in a first-degree relative of a patient is about sevenfold higher than in the general population.3 In many families, the disease is transmitted in an autosomal dominant manner, although penetrance is incomplete. The most important susceptibility gene for classical Paget disease is SQSTM1. Mutations of SQSTM1 are present in up to 40% of patients with a family history and 5 to 10% of people with sporadic disease. The SQSTM1 gene encodes a protein called p62 that is involved in regulating signal transduction downstream of the receptor activator of nuclear factor κB (RANK), which plays a critical role in regulating osteoclastogenesis when activated by RANK ligand (RANKL) (Fig. 247-1). The disease-causing mutations cluster in the ubiquitin-associated domain and have the effect of upregulating nuclear factor κB (NFκB) signaling and stimulating osteoclastogenesis by complex mechanisms that are reviewed in detail elsewhere. Genome-wide association

Stem cell Stromal cell

M-CSF

OCL/M∅ precursor OPG RANK RANKL Ubiquitin

TRAF6 p62

CYLD

OCL precursor VCP DC-STAMP

IκB

NFκB

OPTN Nucleus

Osteoclast

NFκB

Gene expression

FIGURE 247-1.  Regulators of osteoclast dysfunction in Paget disease. Some of the key molecules that have been implicated in the pathogenesis of Paget disease are illustrated. Macrophage colony-stimulating factor (M-CSF) encoded by CSF1 is required for differentiation of stem cells to the osteoclast/macrophage (OCL/MØ) lineage. Osteoclast differentiation and activity are enhanced when RANK (encoded by TNFRSF11A) is activated by RANKL but inhibited by OPG (encoded by TNFRSF11B). Fusion of osteoclast precursors to form mature osteoclasts requires DC-STAMP (encoded by TM7SF4). Within the cell (inset), p62 (encoded by SQSTM1) is required for signal transduction downstream of the RANK receptor and is also involved in regulating autophagy. Both VCP (encoded by VCP) and OPTN (encoded by OPTN) also play a role in regulating NFκB signaling and autophagy.

CHAPTER 246  Medullary Thyroid Carcinoma  

GENERAL REFERENCES 1. Wells SA Jr, Pacini F, Robinson BG, et al. Multiple endocrine neoplasia type 2 and familial medullary thyroid carcinoma: an update. J Clin Endocrinol Metab. 2013;98:3149-3164. 2. Mete O, Asa SL. Precursor lesions of endocrine system neoplasms. Pathology. 2013;45:316-330. 3. Frank-Raue K, Rondot S, Raue F. Molecular genetics and phenomics of RET mutations: impact on prognosis of MTC. Mol Cell Endocrinol. 2010;322:2-7. 4. Agrawal N, Jiao Y, Sausen M, et al. Exomic sequencing of medullary thyroid cancer reveals dominant and mutually exclusive oncogenic mutations in RET and RAS. J Clin Endocrinol Metab. 2013;98:E364-E369. 5. Moura MM, Cavaco BM, Pinto AE, et al. High prevalence of RAS mutations in RET-negative sporadic medullary thyroid carcinomas. J Clin Endocrinol Metab. 2011;96:E863-E868. 6. Choi SK, Yoon SR, Calabrese P, et al. Positive selection for new disease mutations in the human germline: evidence from the heritable cancer syndrome multiple endocrine neoplasia type 2B. PLoS Genet. 2012;8:e1002420. 7. Welander J, Soderkvist P, Gimm O. Genetics and clinical characteristics of hereditary pheochromocytomas and paragangliomas. Endocr Relat Cancer. 2011;18:R253-R276.

1664.e1

8. Scholten A, Schreinemakers JM, Pieterman CR, et al. Evolution of surgical treatment of primary hyperparathyroidism in patients with multiple endocrine neoplasia type 2A. Endocr Pract. 2011;17:7-15. 9. Moore SW, Zaahl M. The Hirschsprung’s-multiple endocrine neoplasia connection. Clinics. 2012;67:63-67. 10. Rowland KJ, Moley JF. Hereditary thyroid cancer syndromes and genetic testing. J Surg Oncol. 2015;111:51-60. 11. Karras S, Anagnostis P, Krassas GE. Vandetanib for the treatment of thyroid cancer: an update. Expert Opin Drug Metab Toxicol. 2014;10:469-481. 12. Nix NM, Braun K. Cabozantinib for the treatment of metastatic medullary thyroid carcinoma. J Adv Pract Oncol. 2014;5:47-50. 13. Tuttle RM, Ball DW, Byrd D, et al. Medullary carcinoma. J Natl Compr Cancer Netw. 2010;8:512-530. 14. Ho AS, Wang L, Palmer FL. Postoperative nomogram for predicting cancer-specific mortality in medullary thyroid cancer. Ann Surg Oncol. 2014; [Epub ahead of print].

1664.e2

CHAPTER 246  Medullary Thyroid Carcinoma  

REVIEW QUESTIONS 1. Sporadic medullary thyroid carcinomas (MTCs) without somatic RET mutations most commonly have which of the following? A. Somatic RAS mutations B. Germline RET mutations C. Somatic BRAF mutations D. No additional genetic mutations E. Gene fusions involving the RET protooncogene Answer: A  About half of patients with sporadic MTC have somatic RET mutations, and about 70% of sporadic MTCs with no somatic RET mutations have HRAS, KRAS, or rarely NRAS mutations. (Moura MM, Cavaco BM, Pinto AE, Leite V. High prevalence of RAS mutations in RET-negative sporadic medullary thyroid carcinomas. J Clin Endocrinol Metab. 2011;96: E863-868.)

4. In patients with MEN 2B where the RET mutation arises de novo,the mutated allele comes from which parent(s)? A. The mother B. The father C. Either parent D. Neither parent Answer: B  In patients with de novo MEN 2B, where neither parent expresses the characteristic phenotype associated with the disease, the mutated RET allele is virtually always inherited from the father. The average age of males who transmit the disease is significantly greater than the average age of all fathers. This appears to be due to a selective advantage acquired by the testis stem cells, which increases the mutation’s frequency in the testis. (Choi SK, Yoon SR, Calabrese P, Arnheim N. Positive selection for new disease mutations in the human germline: evidence from the heritable cancer syndrome multiple endocrine neoplasia type 2B. PLoS Genet. 2012;8:e1002420.)

2. Patients with MEN 2A who develop pheochromocytomas and hyperparathyroidism, in addition to MTC, most commonly have mutations in which codon? A. RET codon 620. B. RET codon 918 C. RET codon 634 E. RET codon 791 E. RET codon 804 Answer: C  There is a clear relationship between genotype and phenotype in patients with MEN 2A. Virtually all patients with this syndrome develop MTC, but the development of pheochromocytoma and hyperparathyroidism is highly variable, occurring much more often in patients with RET 634 codon mutations and much less frequently in patients with other RET codons mutations. (Wells SA Jr, Pacini F, Robinson BG, Santoro M. Multiple endocrine neoplasia type 2 and familial medullary thyroid carcinoma: an update. J Clin Endocrinol Metab. 2013;98:3149-3164.)

5. In patients with hereditary MTC, thyroidectomy should be performed within the first 5 years of life in which cases? A. All patients with a germline RET codon mutation B. Only patients with MEN 2B C. Patients with MEN 2A who have a germline RET codon 634 mutation and patients with MEN 2B D. Patients with a germline RAS mutation E. Patients who only have an elevated serum calcitonin level Answer: C  The published guidelines for managing patients with hereditary MTC all discuss the timing of thyroidectomy in children who have inherited a mutated RET allele. There is uniform agreement that thyroidectomy should be performed before 5 years of age in patients with RET mutations in codon 634, and as soon as the diagnosis is made in patients with MEN 2B, even in the first months of life. (Kloos RT, Eng C, Evans DB, et al. Medullary thyroid cancer: management guidelines of the American Thyroid Association. Thyroid. 2009;19:565-612.)

3. After thyroidectomy, the most reliable indicator of prognosis is which of the following? A. The specific RET or RAS mutation associated with the MTC B. The serum levels of calcitonin (CTN) and carcinoembryonic antigen (CEA) immediately postoperatively C. The rate at which serum CTN and CEA double D. The completeness of the thyroidectomy E. The gender of the patient Answer: C  The presence of an elevated serum CTN or CEA in the immediate postoperative period or soon thereafter is diagnostic of persistent or recurrent MTC. Although this determination is valuable, it is not as significant an indicator of prognosis as the rate at which serum levels of CTN and CEA double. Doubling times of less than 6 months for either CTN or CEA are a very poor prognostic sign. (Meijer JA, le Cessie S, van den Hout WB, et al. Calcitonin and carcinoembryonic antigen doubling times as prognostic factors in medullary thyroid carcinoma: a structured meta-analysis. Clin Endocrinol. 2010;72:534-542.)

1665

CHAPTER 247  Paget Disease of Bone  

TABLE 247-1  GENES THAT PREDISPOSE TO PAGET DISEASE−LIKE SYNDROMES SYNDROME

GENE (PROTEIN)

GENE FUNCTION

Familial expansile osteolysis Early-onset familial Paget disease of bone Expansile skeletal hyperplasia

TNFRSF11A (RANK)

Enhances osteoclast differentiation and bone resorption

Autosomal dominant Insertion mutations exon 1

Onset during adolescence, extensive bone lesions, deafness, tooth loss.

Juvenile Paget disease

TNFRSF11B (OPG)

Inhibits osteoclast differentiation and bone resorption

Autosomal recessive Various loss of function mutations

Onset during childhood, extensive bone lesions, deafness, fractures, deformity, premature cardiovascular disease

Inclusion body myopathy, Paget disease, and frontotemporal dementia

VCP (p97)

Multiple cellular functions, including roles in NFκB signaling and autophagy

Autosomal dominant Loss of function mutations in UBA domain

Onset during 3rd-4th decades, with myopathy, Paget disease, and dementia occurring during 5th-6th decades

studies have identified seven other loci that predispose to Paget disease, which individually increase the risk between 1.4 and 1.7 fold.4,5 These loci have additive effects such that individuals who carry several predisposing alleles have a substantially increased risk for developing Paget disease. Many of these loci lie close to genes that play key roles in osteoclast function, including CSF1, which encodes macrophage colony-stimulating factor (M-CSF); TNFRSF11A, which encodes RANK; TM7SF4, which encodes DC-STAMP; and OPTN, which encodes optineurin (see Fig. 247-1). Several inherited diseases with clinical features overlapping with those of Paget disease have also been described (Table 247-1). These syndromes are also caused by mutations in genes that regulate osteoclast function. Under normal circumstances, bone is renewed and repaired in an orderly and tightly regulated fashion through the process of bone remodeling. The bone remodeling process is highly abnormal in Paget disease. Osteoclasts are increased in number, larger than normal, and hypernucleated. Some contain nuclear inclusion bodies. These were originally thought to be paramyxovirus nucleocapsids, but it has been suggested more recently that they may be aggregates of un-degraded proteins caused by defects in the autophagy pathway. Bone formation is also markedly increased, but the amount of new bone that is formed greatly exceeds that which has been removed by osteoclast activity, leading to enlargement and deformity of affected bones (Fig. 247-2). The bone that is formed is laid down in a disorganized fashion (woven bone) and has impaired mechanical strength. Other features include increased vascularity and marrow fibrosis. The focal nature of Paget disease remains a puzzle. Suggested explanations include the occurrence of somatic mutations in affected bones, which locally increase osteoclast activity, or excessive mechanical loading or skeletal injuries early in life, which by causing microdamage act as a focus for localized increases in bone remodeling.

CLINICAL MANIFESTATIONS

It has been estimated that between 7 and 16% of patients with Paget disease come to medical attention, and the presentation is highly variable.6 Many patients are asymptomatic, and Paget disease is detected as the result of a raised serum alkaline phosphatase (ALP) or an abnormal radiograph in patients who are being investigated for another reason. In those that do present clinically, symptoms that are attributable to Paget disease are observed in about 75% of cases. The most common is pain, which can be due to either increased bone turnover or a complication such as osteoarthritis, spinal stenosis, pseudofractures, or nerve compression syndromes. Deafness may occur in patients with skull involvement, but this is usually conductive rather than due to auditory nerve compression. Osteosarcoma occurs in less than 0.5% of cases but should be suspected in patients who experience a sudden increase in bone pain or swelling of an affected site. Other, rare complications include obstructive hydrocephalus, high-output cardiac failure, and hypercalcemia in patients who are immobilized. The risk for cardiovascular disease is increased in patients with Paget disease compared with age- and gendermatched controls, probably owing to an increased prevalence of vascular calcification. Most patients have no clinical signs but some present with bone deformity (see Fig. 247-2) or warmth of the skin overlying an affected bone.

DIAGNOSIS

The diagnosis can usually be made by radiograph, which shows the typical features of focal osteolysis with coarsening of the trabecular pattern, bone expansion, and cortical thickening (see Fig. 247-2). Occasionally, the disease

INHERITANCE/MUTATION

CLINICAL FEATURES

may be predominantly lytic in nature (see Fig. 247-2). The most sensitive way of defining the extent of Paget disease is a radionuclide bone scan in which tracer uptake is intensely increased at affected sites (see Fig. 247-2). Imaging with magnetic resonance imaging and computed tomography is not usually required unless complications such as spinal stenosis or osteosarcoma are suspected. Laboratory testing should include assessment of renal function, calcium, albumin, alkaline phosphatase (ALP), and 25(OH)D levels; liver function should be assessed to rule out the possibility that elevations in ALP are of hepatic origin. Typically, Paget disease presents with an elevation in ALP with otherwise normal biochemistries, but normal levels of ALP do not exclude the diagnosis. Vitamin D deficiency is a common finding but most likely reflects the fact that Paget disease predominantly affects older people in whom vitamin D deficiency is prevalent. Specialized markers such as bonespecific ALP or procollagen type 1 N-terminal propeptide can be useful in patients with coexisting liver disease but otherwise offer no advantage over total ALP in diagnosis and assessing treatment response. Susceptibility to Paget disease can be assessed in relatives of affected patients by genetic testing for SQSTM1 mutations, although this is not commonly performed in routine clinical practice. The differential diagnosis includes hyperostosis frontalis interna (a benign condition characterized by osteosclerosis of the frontal bones of the skull), fibrous dysplasia, pustulotic arthro-osteitis (which can present with mixed osteosclerotic and osteolytic lesions of the clavicle and ribs),7 and osteosclerotic metastases, particularly from carcinoma of the prostate. Usually, Paget disease can be distinguished from these conditions biochemically and through imaging, but occasionally, biopsy of an affected site may be required.

Treatment  The most common indication for medical treatment of Paget disease is bone pain localized to an affected site.8 Although such pain may be caused by increased metabolic activity, other causes may also be operative, including nerve compression syndromes, pseudofractures, secondary osteoarthritis, and other musculoskeletal conditions. Careful assessment of the patient is therefore necessary to decide on the most appropriate treatment. Bone pain caused by increased metabolic activity is localized to the affected site and is usually accompanied by a raised ALP level. It is common to encounter patients in whom pain occurs in the presence of coexisting osteoarthritis, bone deformity, or other musculoskeletal conditions. In such cases, it can be difficult to be sure about the origin of the pain, and many clinicians give a therapeutic trial of bisphosphonates. If the pain responds, then one can assume it was due to increased metabolic activity; if it does not, further evaluation should be undertaken to identify the cause and treat the patient appropriately. Pseudofractures represent a distinct management problem. These are areas of focal osteolysis that traverse the lateral cortex of weight-bearing bones of the lower limbs. Some remain stable for prolonged periods without causing symptoms; others regress spontaneously; and others progress to pathologic fracture, often in association with a localized increase in pain at the affected site.9

Bisphosphonates

Bisphosphonates are the drugs of first choice for the treatment of pain that is thought to be due to increased metabolic activity. Nowadays, nitrogencontaining bisphosphonates (aminobisphosphonates) are used in preference to older bisphosphonates because of their greater potency (Table 247-2).

1666

CHAPTER 247  Paget Disease of Bone  

B

A

C

D

FIGURE 247-2.  Radiographic and histologic features of Paget disease. A, Radionuclide bone scan image showing intense tracer uptake, typical of Paget disease of bone (PDB) in the right femur. B, Radiograph of an affected left femur showing bone expansion with mixed osteolytic/osteosclerotic areas and loss of normal trabecular pattern. A pseudofracture is visible in the lateral cortex (arrow). C, Histologic features from a hematoxylin and eosin stained section. A large osteoclast is visible (black arrow) close to an area of new bone formation (white arrows). There is extensive marrow fibrosis. Irregular cement lines typical of woven bone are apparent to the right of section. D, Predominantly lytic Paget disease of the left femur. The lytic area involves the intertrochanteric region and extends down the femoral shaft (white arrows).

Placebo controlled trials have shown that most bisphosphonates are effective at improving bone pain in Paget disease. Superiority of aminobisphosphonates over etidronate and tiludronate at suppressing ALP levels has been demonstrated, but with little difference in the response of pain. There are limited data comparing different aminobisphosphonates. In an open-label study comparing pamidronate 180 mg intravenously in unit doses of 30 mg weekly or 60 mg alternate weeks to oral alendronate 40 mg daily given in 3-monthly blocks over a 2-year period, there were no significant differences in the proportion of patients who achieved normal levels of ALP (86 and 91% respectively) or in symptomatic response. A1  Another study that compared a single infusion of 5 mg zoledronic acid with oral risedronate 30 mg daily for 2 months showed significant superiority of zoledronic acid in lowering ALP. Those randomized to zoledronic acid had greater improvement in some domains of health-related quality of life, but the differences between groups were small (1 to 2 points) and below the threshold of 5 points, which is considered clinically significant. A2  Another randomized trial compared the effects of giving repeated course of bisphosphonates (mainly risedronate) with the aim of normalizing ALP (intensive treatment), with therapy primarily aimed at

controlling symptoms (symptomatic therapy) in Paget disease. A3  This showed no difference in response of pain, quality of life, or complications between the groups, indicating that trying to restore ALP to normal confers no clinical advantage in most patients with Paget disease. After initiation of bisphosphonate therapy, levels of ALP start to fall within about 10 days and reach a nadir between 3 and 6 months. Levels of ALP can remain suppressed for many months or years thereafter, particularly with zoledronic acid. Symptoms can improve while ALP levels are still falling, and good clinical responses are often observed in patients whose ALP levels are not restored to normal. Intravenous bisphosphonates can cause transient bone pain, myalgia, headache, nausea, pyrexia, and fatigue within 1 to 3 days of the infusion in about 25% of cases (acute phase response). These symptoms can be ameliorated by acetaminophen given before and for a few days after the infusion, but they almost always subside within 7 days even without treatment. The acute phase response is much less common after second and subsequent infusions. Hypocalcemia may occur, particularly in patients with substantial elevations in bone turnover and vitamin D deficiency. The risk can be 

1667

CHAPTER 248  OSTEONECROSIS, OSTEOSCLEROSIS/HYPEROSTOSIS  

TABLE 247-2  BISPHOSPHONATES USED IN THE TREATMENT OF PAGET DISEASE DRUG

DOSE

COMMON ADVERSE EFFECTS

bisphosphonate therapy might impair fracture union and bone repair, but there is little evidence to suggest that this is a problem in clinical practice.10 Orthopedic surgery may also be required in patients who develop osteosarcoma, but the prognosis is poor even with aggressive operative treatment.

Oral Etidronate*

400 mg/day orally for 3-6 mo

Diarrhea, nausea, abdominal pain

Tiludronate

400 mg/day orally for 3 mo

Diarrhea, nausea, dyspepsia

Risedronate

30 mg/day orally for 2 mo

Dyspepsia, esophagitis

Alendronic acid†

40 mg/day orally for 6 mo

Dyspepsia, esophagitis

Pamidronate

180 mg IV in unit doses of 30 mg weekly or 60 mg alternate weeks

Acute phase response, hypocalcemia

Zoledronic acid

5 mg IV

Acute phase response, hypocalcemia

Intravenous

*Now seldom used. † Not licensed in the United Kingdom or Europe for Paget disease. Etidronate, pamidronate, tiludronate, and risedronate should be avoided if estimated glomerular filtration rate (eGFR) 24 hr

20 yr

20 ng/mL* *Criterion not valid if there is an associated clonal myeloid disorder. From Horny HP, Metcalfe DD, Bennett JM, et al. Mastocytosis. In: Swerdlow SH, Campo E, Harris NL, et al, eds. WHO Classification of Tumours of Haematopoietic and Lymphoid Tissues. Lyon, France: IARC Press; 2008:54-63.

sections (Fig. 255-3A). Such clusters are frequently observed around blood vessels and next to bone trabeculae in bone marrow biopsy sections. Immunohistochemical staining for tryptase is the recommended method for visualization of mast cells. Routine hematoxylin and eosin or metachromatic stains such as toluidine blue are not sufficiently sensitive to demonstrate subtle mast cell infiltrates or abnormal morphologic features of mast cells within the infiltrates in decalcified bone marrow biopsy sections. Mast cell morphology in bone marrow provides important clues to the diagnosis of systemic mastocytosis. Bone marrow mast cells in systemic mastocytosis often display atypical morphology, such as an elongated (spindle) shape, hypogranularity, and an eccentric or lobulated nucleus (Fig. 255-3B). These atypical mast cells are usually observed in close association with bone

1709

CHAPTER 255  Mastocytosis  

marrow spicules in the aspirate smear. Mast cells in mast cell leukemia (MCL) may be very sparsely granulated. Flow cytometric analysis of the mast cells in a bone marrow aspirate, when it is performed appropriately, is a sensitive diagnostic aid. The mean percentage of mast cells in a healthy bone marrow aspirate is approximately 0.02%, and it does not exceed 1% in most patients with mastocytosis. Therefore, to visualize the mast cell population correctly, the total cell numbers analyzed by flow cytometry should be significantly higher than those in other, more routine evaluations (e.g., leukemia phenotyping). The characteristic flow cytometric finding of systemic mastocytosis is the aberrant expression of CD25 or CD2 on CD117+ mast cells. CD25 is more sensitive than CD2 as CD2 may be absent or weakly expressed in some cases of advanced mastocytosis. Aberrant CD25 expression can also be demonstrated by immunohistochemical staining of bone marrow biopsy specimens.6 Serum tryptase level may be elevated in patients with mastocytosis.7 Currently available commercial tryptase immunoassays measure levels of total tryptase, the sum of mature tryptase, and tryptase precursors. Mature tryptase enzyme is a serine protease stored in mast cell granules and is transiently elevated in serum or plasma after mast cell degranulation episodes, such as anaphylaxis. In contrast, tryptase precursor proenzymes (α and β protryptases) are constitutively secreted outside the cell, and their serum levels at baseline correlate with mast cell burden. The median serum tryptase level in a healthy population is approximately 5 ng/mL. A serum tryptase level higher than 20 ng/mL raises suspicion for systemic mastocytosis in the appropriate clinical setting. A normal tryptase level does not rule out a diagnosis of mastocytosis, and increased tryptase levels can be seen in other conditions, such as myelodysplastic syndromes, acute myeloid leukemias, chronic eosinophilic leukemia, and chronic renal insufficiency. Metabolites of histamine, such as N-methylhistamine, and prostaglandin D2 can be elevated in a 24-hour urine specimen but are neither more sensitive nor more specific than the baseline serum tryptase measurement in mastocytosis. Demonstration of a codon 816 KIT mutation (D816V) may be necessary to fulfill the diagnostic criteria in patients lacking the major criterion (see Pathobiology).8 Examination of lesional tissues, such as skin and bone marrow, affords the highest sensitivity. Codon 816 KIT mutations have been detected in a variety of other neoplastic diseases, such as core binding factor acute myeloid leukemias, sinonasal lymphomas, and seminomas, in addition to mastocytosis. A rare histologic variant with clustering of mature round mast cells without CD25 expression termed well-differentiated systemic mastocytosis has been described. These patients generally have a history of childhood-onset mastocytosis without the D816V KIT mutation and therefore may respond to imatinib as opposed to those with typical systemic mastocytosis carrying the D816V mutation (see Treatment).

World Health Organization Disease Categories

Each patient diagnosed with mastocytosis should be assigned a category of disease according to the WHO classification (see Table 255-1). Cutaneous mastocytosis in the absence of bone marrow and internal organ involvement is the most common category in patients with pediatric-onset disease. Systemic mastocytosis is divided into the categories of indolent systemic mastocytosis, systemic mastocytosis with associated clonal hematologic non–mast cell lineage disease (SM-AHNMD), aggressive systemic mastocytosis, and MCL. An algorithm for classification of systemic mastocytosis is presented in Figure 255-4. Indolent systemic mastocytosis is the most common category in adults. Patients in this category usually have a normal life expectancy compared with age-matched general populations, although they experience symptoms related to release of mast cell mediators.9 Indolent systemic mastocytosis follows a persistent course, and progression to a more advanced category is unusual ( 10% or bone marrow aspirate mast cells > 20% No

Yes

Non–mast cell clonal hematologic disorder (e.g., MPD or MDS)

Mast cell leukemia

No

Yes

Evidence of end-organ dysfunction (e.g., ascites, malabsorption)

SM-AHNMD

No

Yes

ISM

ASM

FIGURE 255-4.  An algorithm for classification of systemic mastocytosis. ASM = aggressive systemic mastocytosis; ISM = indolent systemic mastocytosis; MDS = myelodysplastic syndromes; MPD = myeloproliferative disorders (neoplasms); SM-AHNMD = systemic mastocytosis with associated clonal hematologic non–mast cell lineage disease.

circulation or 20% or more mast cells in bone marrow aspirate smears, or both. To diagnose MCL, the mast cell percentage in bone marrow aspirate smears should be assessed in an area of the slide that is sufficiently distant from the spicules. Mast cell sarcoma and extracutaneous mastocytoma are rare diagnoses characterized by malignant and benign solid mast cell collections, respectively. There is a subset of patients with recurrent idiopathic or hymenoptera venom–induced anaphylaxis who have evidence of clonal mast cells carrying the D816V KIT mutation or aberrantly expressing surface CD25, without fully meeting the WHO diagnostic criteria and without displaying urticaria pigmentosa skin lesions. Such patients are provisionally referred to as having a monoclonal mast cell activation syndrome.

TREATMENT  The major goal of treatment for all categories of mastocytosis is symptom control. A reduction in mast cell numbers is considered only in disease categories with a poor prognosis (i.e., SM-AHNMD, aggressive systemic mastocytosis, MCL, and mast cell sarcoma).10,11 Current treatment modalities have not been shown to change the natural course of the disease.12

Medical Therapy

Patients with cutaneous and indolent systemic mastocytosis are treated symptomatically. Pruritus in mastocytosis usually responds to scheduled doses of histamine1-receptor blocker antihistamines, such as fexofenadine or cetirizine. Sedating antihistamines, such as hydroxyzine or diphenhydramine, may be used before bedtime. Photochemotherapy (oral psoralen plus ultraviolet A) or phototherapy may be helpful in patients with refractory pruritus; it results in symptomatic improvement and temporary fading of the pigmented skin lesions in up to 50% of patients. The side effects of phototherapy, including increased risk of skin cancer, should be taken into account when this treatment is considered. Histamine2-receptor blocker antihistamines, such as ranitidine or famotidine, are usually prescribed as a first-line treatment for patients with gastrointestinal complaints, such as heartburn, nausea, and abdominal pain. Proton pump inhibitors may be added in patients whose abdominal symptoms are refractory to histamine2-receptor blockers. Oral cromolyn sodium (adult dose, 200 mg four times daily) has been effective in reducing abdominal pain, diarrhea, nausea, vomiting, and pruritus in various studies, although the beneficial effects are variable among patients. Finally, low to moderate doses of systemic glucocorticoids can be beneficial in unusual cases of aggressive mastocytosis presenting with recalcitrant diarrhea associated with malabsorption or hepatomegaly with ascites.

1710

CHAPTER 255  Mastocytosis  

Cysteinyl leukotrienes, such as LTC4, that are produced after mast cell activation are thought to contribute to symptoms in mastocytosis. Therefore, drugs targeting the synthesis or receptor binding of leukotrienes are usually added to the treatment regimens of patients who derive suboptimal relief of itching and abdominal pain from histamine receptor–blocking therapy. However, there have been no controlled studies evaluating the clinical efficacy of this class of drugs in patients with mastocytosis. Self-administered epinephrine should be considered for all patients even if they do not have any history of hypotensive or anaphylactic episodes resulting in presyncope or syncope from acute mast cell degranulation. These episodes should be treated like systemic anaphylaxis (Chapter 253). Cytoreductive therapy, considered in aggressive disease variants associated with poor prognosis, has yielded disappointing results thus far. Some patients with recurrent life-threatening episodes of mast cell mediator release unresponsive to conventional therapy may also be candidates for cytoreductive therapy after careful consideration of risks and benefits. Approaches to cytoreductive treatment of mastocytosis have included interferon alfa-2b and the nucleoside analogue 2-chlorodeoxyadenosine. Interferon alfa-2b (0.5 to 5 million units, three to five times per week), alone or with prednisone, has been reported to partially improve clinical and laboratory abnormalities in approximately 50% of patients with aggressive systemic mastocytosis, patients with osteoporosis and pathologic fractures, and patients with recalcitrant recurrent anaphylaxis, although complete histopathologic and molecular remissions appear to be rare. Interferon alfa is difficult to tolerate because of its many side effects, including influenza-like symptoms, bone pain, and depression. A regimen of 2-chlorodeoxyadenosine (0.10 to 0.17 mg/kg/day for 5 days, repeated at intervals of 4 to 8 weeks) has been reported to result in partial and transient responses in patients with advanced categories of disease in case reports and small series. MCL usually is treated with polychemotherapy as acute myeloid leukemia (Chapter 183), although a successful treatment regimen has not yet been identified. Imatinib, a tyrosine kinase inhibitor with activity against wild-type KIT, PDGFR, and abl, has been effective in a small number of patients without D816V KIT mutation or with the FIP1L1-PDGFRA fusion gene, who present with chronic eosinophilic leukemia (Chapter 170) with a modest increase in bone marrow mast cells. However, most patients with mastocytosis have the D816V KIT mutation, which confers resistance to imatinib, and therefore are not appropriate candidates for this therapy.13

Ancillary and Other Therapies

Avoidance of the triggers of mast cell degranulation is an important adjunct to the pharmacologic treatment of symptoms. These show remarkable individual variation among patients (see Pathobiology), and the individual medical history can be helpful in identifying such triggers. General anesthesia and surgery impose an additional risk to patients with mastocytosis because several agents that are used perioperatively, such as muscle relaxants, opioid analgesics, and nonsteroidal anti-inflammatory drugs, can induce acute mast

cell degranulation. Prior surgical and anesthesia records should be obtained if available, and an appropriate strategy for the anesthetic management of the patient should be determined, with close communication involving the patient, anesthesiologist, surgeon, and an allergist. Non–mast cell clonal hematologic disorders associated with mastocytosis should be treated according to the standard-of-care guidelines for those disorders, regardless of the presence of mastocytosis. Bone marrow transplantation (Chapter 178) has yielded variable results for the treatment of mast cell disease, and occasional cases resulting in complete remission have been reported. Venom immunotherapy is recommended for those with a history of systemic reactions to hymenoptera who have evidence of IgE-mediated sensitization (by blood or skin allergy testing). Most experts recommend the duration of the therapy to be indefinite as fatalities have been reported after discontinuation of immunotherapy. Because of the high prevalence of osteoporosis and pathologic bone fractures in mastocytosis, bone densitometry should be considered a standard diagnostic procedure in adult patients with mastocytosis. If osteoporosis is detected, it should be treated per standard recommendations (Chapter 243).

PROGNOSIS

The prognosis for mastocytosis varies by the category of disease. At least 50% of patients with pediatric-onset cutaneous mastocytosis have complete resolution of the disease by adolescence, and the great majority of the rest of those patients experience improvement or fading of the skin lesions. Indolent systemic mastocytosis is a persistent disease but has a good prognosis without a decrease in life expectancy, and progression to a more aggressive disease category is rare.14 Factors associated with poorer prognosis have been reported as the absence of urticaria pigmentosa, older age at onset of symptoms, elevated serum lactate dehydrogenase or alkaline phosphatase, thrombocytopenia, anemia, peripheral blood smear abnormalities, and detectability of the D816V KIT mutation in peripheral blood. The prognosis for SM-AHNMD is determined by the prognosis for the associated hematologic disorder. Aggressive systemic mastocytosis and MCL have poor prognoses, with median survival times of less than 3 years and less than 1 year, respectively. GENERAL REFERENCES For the General References and other additional features, please visit Expert Consult at https://expertconsult.inkling.com.

CHAPTER 255  Mastocytosis  

GENERAL REFERENCES 1. Carter MC, Metcalfe DD, Komarow HD. Mastocytosis. Immunol Allergy Clin North Am. 2014;34:181-196. 2. Molderings GJ. The genetic basis of mast cell activation disease—looking through a glass darkly. Crit Rev Oncol Hematol. 2015;93:75-89. 3. Alvarez-Twose I, Bonadonna P, Matito A, et al. Systemic mastocytosis as a risk factor for severe Hymenoptera sting–induced anaphylaxis. J Allergy Clin Immunol. 2013;131:614-615. 4. Akin C. Anaphylaxis and mast cell disease. What is the risk? Curr Allergy Asthma Rep. 2010;10:34-38. 5. van der Veer E, van der Goot W, de Monchy JG, et al. High prevalence of fractures and osteoporosis in patients with indolent systemic mastocytosis. Allergy. 2012;67:431-438. 6. Morgado JM, Sánchez-Muñoz L, Teodósio CG, et al. Immunophenotyping in systemic mastocytosis diagnosis: “CD25 positive” alone is more informative than the “CD25 and/or CD2” WHO criterion. Mod Pathol. 2012;25:516-521. 7. Valent P, Sperr WR, Sotlar K, et al. The serum tryptase test: an emerging robust biomarker in clinical hematology. Expert Rev Hematol. 2014;7:683-690.

1710.e1

8. Valent P, Escribano L, Broesby-Olsen S, et al. Proposed diagnostic algorithm for patients with suspected mastocytosis: a proposal of the European Competence Network on Mastocytosis. Allergy. 2014;69:1267-1274. 9. Rabenhorst A, Christopeit B, Leja S, et al. Serum levels of bone cytokines are increased in indolent systemic mastocytosis associated with osteopenia or osteoporosis. J Allergy Clin Immunol. 2013;132:1234-1237. 10. Siebenhaar F, Akin C, Bindslev-Jensen C, et al. Treatment strategies in mastocytosis. Immunol Allergy Clin North Am. 2014;34:433-447. 11. Pardanani A, Tefferi A. Systemic mastocytosis in adults: a review on prognosis and treatment based on 342 Mayo Clinic patients and current literature. Curr Opin Hematol. 2010;17:125-132. 12. Cardet JC, Akin C, Lee MJ. Mastocytosis: update on pharmacotherapy and future directions. Expert Opin Pharmacother. 2013;14:2033-2045. 13. Ustun C, DeRemer DL, Akin C. Tyrosine kinase inhibitors in the treatment of systemic mastocytosis. Leuk Res. 2011;35:1143-1152. 14. Brockow K. Epidemiology, prognosis, and risk factors in mastocytosis. Immunol Allergy Clin North Am. 2014;34:283-295.

1710.e2

CHAPTER 255  Mastocytosis  

REVIEW QUESTIONS 1. Which of the following markers is aberrantly expressed by mast cells in patients with systemic mastocytosis? A. CD25 B. CD117 (Kit) C. High-affinity IgE receptor D. CD45 Answer: A  CD25 (alpha chain of the interleukin-2 receptor) is aberrantly expressed and is a sensitive diagnostic marker in systemic mastocytosis. All others are expressed by both normal and pathologic mast cells. (Valent P, Horny HP, Escribano L, et al. Diagnostic criteria and classification of mastocytosis: a consensus proposal. Leuk Res. 2001;25:603-625.) 2. A 25-year-old woman has recently been diagnosed with systemic mastocytosis by a bone marrow biopsy. There was no other hematologic disease noted. She experiences pruritus of the skin in hot weather and occasional abdominal cramping but is otherwise asymptomatic. KIT mutational analysis reveals the D816V mutation in bone marrow. Which of the following is the most appropriate treatment? A. Imatinib B. H1 and H2 antihistamines C. Prednisone D. Interferon alfa E. Bone marrow transplantation Answer: B  This patient has indolent systemic mastocytosis and should be treated symptomatically. Cytoreductive therapies such as interferon alfa are indicated in patients with aggressive mastocytosis. The patient is not a candidate for imatinib because of indolent disease and presence of D816V KIT mutation. The patient’s symptoms do not warrant bone marrow transplantation or use of systemic glucocorticoids, which are associated with long-term adverse effects. (Valent P, Akin C, Escribano L, et al. Standards and standardization in mastocytosis: consensus statements on diagnostics, treatment recommendations, and response criteria. Eur J Clin Invest. 2007;37:435-453.) 3. A 35-year-old farmer with a 10-year history of mastocytosis who is otherwise healthy has recently experienced anaphylaxis with brief loss of consciousness after being stung by a honeybee. Allergy skin testing confirmed honeybee allergy. What is the most appropriate next step in management of honeybee allergy in this patient? A. Venom immunotherapy is not recommended because of increased risk of reactions to immunotherapy due to mastocytosis. B. Recommend venom immunotherapy for 5 years. C. Recommend venom immunotherapy indefinitely. D. Confirm skin test results by blood testing before recommending immunotherapy. Answer: C  Patients with mastocytosis are at increased risk for life-threatening reactions to hymenoptera stings. Fatalities have been reported in patients after discontinuation of venom immunotherapy. Therefore, most experts recommend immunotherapy when IgE-mediated sensitization is found by either skin or blood testing in these patients. Whereas the patients are also at increased risk of having a systemic reaction during immunotherapy, the riskbenefit ratio is often considered favorable to initiate immunotherapy. The patients should always continue to carry a self-injectable epinephrine as the protection is lower than for those without mastocytosis. (Niedoszytko M, de Monchy J, van Doormaal JJ, et al. Mastocytosis and insect venom allergy: diagnosis, safety and efficacy of venom immunotherapy. Allergy. 2009;64:12371245; and Alvarez-Twose I, Bonadonna P, Matito A, et al. Systemic mastocytosis as a risk factor for severe Hymenoptera sting–induced anaphylaxis. J Allergy Clin Immunol. 2013;131:614-615.)

4. Which of the following patients is a candidate for therapy with imatinib? A. 30-year-old with adult-onset indolent systemic mastocytosis with D816V KIT mutation who remains symptomatic despite symptomatic therapy B. 30-year-old with adult-onset systemic mastocytosis with occasional symptoms on symptomatic therapy; peripheral blood analysis negative for D816V KIT mutation C. 60-year-old with systemic mastocytosis, splenomegaly, and pancytopenias; peripheral blood and bone marrow positive for D816V KIT mutation D. 40-year-old with childhood-onset cutaneous mastocytosis that persisted into adulthood; patient recently developed progressive splenomegaly and anemia; mutational analysis of bone marrow shows an exon 8 KIT mutation Answer: D  Patients with D816V KIT mutation regardless of the category of mastocytosis are resistant to therapy with imatinib. Patients with cutaneous or indolent systemic mastocytosis with symptoms manageable by symptomatic therapy are not candidates for cytoreductive therapy regardless of the mutational status. Patients with well-differentiated systemic mastocytosis without codon 816 (exon 17) KIT mutations have a high likelihood of response to imatinib. (Akin C, Fumo G, Yavuz AS, et al. A novel form of mastocytosis associated with a transmembrane c-kit mutation and response to imatinib. Blood. 2004;103:3222-3225.) 5. Which of the following mast cell mediator measurements raises the greatest suspicion for mastocytosis? A. Tryptase level of 50 ng/mL in a patient with perioperative anaphylaxis, obtained 1 hour after the event B. Baseline tryptase level of 90 ng/mL in a patient who experiences recurrent anaphylactic episodes of unclear etiology C. Elevated urinary N-methylhistamine in a patient with recurrent unexplained flushing D. Elevated urinary prostaglandin D2 in a patient with chronic diarrhea and osteoporosis Answer: B  Urinary metabolites of mast cell mediators are neither more sensitive nor more specific than measurement of baseline tryptase in diagnosis of systemic mastocytosis. Tryptase levels increase (usually within 4 hours) after an anaphylactic event regardless of whether it is associated with mastocytosis. Some patients with recurrent unexplained anaphylaxis who may have been previously diagnosed with idiopathic anaphylaxis may have mastocytosis as an underlying disorder (especially if the anaphylactic episodes involve hypotension and syncope rather than urticaria or angioedema). The diagnosis should be confirmed by a bone marrow biopsy in the patient in option D. (Akin C, Scott LM, Kocabas CN, et al. Demonstration of an aberrant mastcell population with clonal markers in a subset of patients with “idiopathic” anaphylaxis. Blood. 2007;110:2331-2333; and Akin C, Metcalfe DD. Surrogate markers of disease in mastocytosis. Int Arch Allergy Immunol. 2002;127: 133-136.)

XXII

RHEUMATIC DISEASES 256 APPROACH TO THE PATIENT WITH RHEUMATIC DISEASE

257 LABORATORY TESTING IN THE RHEUMATIC DISEASES

258 IMAGING STUDIES IN THE RHEUMATIC DISEASES

259 CONNECTIVE TISSUE STRUCTURE AND FUNCTION

262 OSTEOARTHRITIS

270 THE SYSTEMIC VASCULITIDES

263 BURSITIS, TENDINITIS, AND OTHER

271 POLYMYALGIA RHEUMATICA AND

PERIARTICULAR DISORDERS AND SPORTS MEDICINE

264 RHEUMATOID ARTHRITIS

AND BONES

273 CRYSTAL DEPOSITION DISEASES

266 SYSTEMIC LUPUS ERYTHEMATOSUS

274 FIBROMYALGIA, CHRONIC FATIGUE

267 SYSTEMIC SCLEROSIS

261 THE SYSTEMIC AUTOINFLAMMATORY

268 SJÖGREN’S SYNDROME

DISEASES

272 INFECTIONS OF BURSAE, JOINTS,

265 THE SPONDYLOARTHROPATHIES

260 INHERITED DISEASES OF CONNECTIVE TISSUE

TEMPORAL ARTERITIS

(SCLERODERMA)

269 INFLAMMATORY MYOPATHIES

SYNDROME, AND MYOFASCIAL PAIN

275 SYSTEMIC DISEASES IN WHICH ARTHRITIS IS A FEATURE

276 SURGICAL TREATMENT OF JOINT DISEASES

1712

CHAPTER 256  Approach to the Patient with Rheumatic Disease  

256  APPROACH TO THE PATIENT WITH RHEUMATIC DISEASE VIVIAN P. BYKERK AND MARY K. CROW

Rheumatic diseases are common and an important cause of reduced quality of life, increased comorbidity, and reduced life expectancy. They incur a significant socioeconomic burden and warrant expertise on the part of all physicians who treat patients. This chapter provides a framework to approach the evaluation of patients who present with signs and symptoms suggesting a rheumatic disease. An algorithmic approach is provided that allows the physician to incorporate presenting features, patient characteristics, anatomic structures, along with diagnostic tests, to facilitate a diagnosis and treatment plan.

DEFINITION AND CATEGORIZATION

Rheumatic diseases are disorders of connective tissue in which general or localized inflammation frequently manifests as pain attributable to peripheral joints, the spine, or muscles. Systemic features such as stiffness, fever, or weight loss and a multitude of extramusculoskeletal features, ranging from skin rashes to renal dysfunction, often accompany rheumatic diseases. In most cases, the basic underlying pathology is understood, although this is less true of pain disorders appearing alone or accompanying a rheumatic disease (Chapter 30). For most rheumatic disorders, the underlying molecular mechanisms through which environmental triggers and genetic susceptibility factors collaborate to result in a particular rheumatic disease in an individual remain to be fully elucidated. These diseases can be broadly considered as those that are primarily degenerative, with inflammation occurring secondarily, and those in which inflammation is the primary mediator of the disease. In the latter, the pathogenesis can be mediated through aberrant immune responses or as a result of metabolic abnormalities.

Histopathology

Rheumatic diseases are often also termed connective tissue diseases, understandably, because connective tissue is the most abundant tissue in the body supporting and connecting other tissues and organs. Loose and dense connective tissues include cellular components and extracellular matrix. Loose connective tissue fills spaces between muscle sheaths, encases blood and lymphatic vessels, and holds fibroblasts that synthesize collagen fibers. It includes reticular fibers that provide the skeleton of muscle cells, nerves, and capillaries. Dense connective tissue supports the body’s soft tissues and includes more collagen fibers and fewer cells. It is found in the dermis, joint capsules, cartilage, bone, and fascia of muscles, and it forms tendons, ligaments, and points of connection where these insert into bone (aponeurosis). Cells included in connective tissue may be wandering, such as mast cells or macrophages, or resident cells, such as fibroblasts, fibrocytes, and reticular cells. Fibroblasts are responsible for synthesizing collagen, elastic reticular fibers, and ground substance of extracellular matrix, including tissue fluids and collagen fibers. Importantly, connective tissue is integrated with cells associated with the body’s defense system: lymphocytes, plasma cells, macrophages, dendritic cells, and eosinophils. The close proximity of connective tissue to blood vessels and cells of the immune system provides the setting for a group of disorders that are mediated by impaired immune system regulation and disruptions of the vascular system.

Classification of Rheumatic Diseases

More than 100 types of rheumatic diseases have been described. Although these can be considered to be based primarily on one of two degenerative or inflammatory overarching processes, one can further subdivide rheumatic diseases as follows (and also outlined in Table 256-1): (1) those associated with degeneration of connective tissues attributable to (a) trauma, (b) structural/mechanical imbalances, or (3) inherent early demise of cellular components; (2) those associated with systemic autoimmunity,1 often linked with measurable autoantibodies that can manifest primarily with (a) synovitis, (b) widespread organ involvement, (c) inflamed blood vessels, or (d) inflammation of muscle; (3) other inflammatory connective tissue diseases

involving more dense tissues, not associated with the formation of autoantibodies and hence termed seronegative rheumatic diseases or spondyloarthropathies; (4) diseases in which inflammation of the vasculature, particularly small, medium, or large arteries, is the predominant feature; (5) autoinflammatory diseases that can be associated with crystal deposition or genetic mutations involving cytokine pathways; and (6) pain syndromes that must often be considered in the context of these diseases, in which some appear to be comorbid and closely linked to the underlying rheumatic disease, such as diffuse pain associated with Sjögren’s syndrome, hypermobility of connective tissue, or those regional pain syndromes that are anatomically linked to mechanical disruption. Patients presenting with generalized pain syndromes require investigation to exclude a connective tissue disease. Increasingly, genotypes have been identified that are associated with diseases that fall into each of these categories, and in some cases specific immunologic pathways have allowed grouping of a set of rheumatic diseases previously considered more distinct. Mimics of rheumatic diseases exist, and clinicians need to be mindful of considering these when evaluating a patient for a rheumatic disease. For instance, arthropathies and syndromes resembling a rheumatic disease can occur in the settings of both infection and malignancy. Autoimmune phenomena are increasingly being recognized in the setting of malignancy.2 Red flags for each need to be considered in the assessment of a patient for possible rheumatic disease. No classification of rheumatic disease can completely explain its genesis. However, considering these in a classification schema can aid in the approach to a patient in whom these disorders are being considered (see Table 256-1).

EPIDEMIOLOGY

Although connective tissue diseases can generally be categorized as noted in Table 256-1, in adults there are six prototypical rheumatic diseases most often assessed and managed by rheumatologists: rheumatoid arthritis (RA), systemic lupus erythematosus (SLE), systemic sclerosis (SSc), spondyloarthropathies (SpA) (primarily ankylosing spondylitis [AS]), Sjögren’s syndrome (SS), and vasculitis. These diseases are ubiquitous throughout the world (Table 256-2) and for the most part have a similar incidence and prevalence throughout the globe. Each is associated with differing immune aberrations and mechanisms of inflammatory damage, although the cause and reasons for chronicity still remain unknown. Autoimmune rheumatic diseases are also among the leading causes of death and morbidity in the industrial world, in part related to associated comorbid diseases, particularly comorbid cardiovascular disease. They contribute to a significant socioeconomic burden. Increasing evidence points to risks for their genesis relating to environmental factors, socioeconomic factors, and exposure to infectious agents, ultraviolet radiation, and pollutants. Smoking, in particular, has been associated with an increased risk for SLE and RA in genetically susceptible individuals in Western cultures. Effects of migration elucidate some of these risks. For instance, Africans who migrate far away from their native environmental and cultural origins appear to have an increased susceptibility to SLE. Also, reports have linked occupational exposures, such as silica dust, mercury, pesticides, solvents, and metals, to an increased risk for SLE and RA. In some cases, geographic clusters of rare autoimmune disease argue for specific genetic determinants. For example, with SSc, higher incidence, prevalence, and mortality rates have been reported in African American populations compared with white populations, and the prevalence has been reported as higher in southern Europe, particularly Italy (prevalence of 7 to 33 per 100,000). Additionally, social and demographic factors may contribute to the epidemiology of rheumatic diseases. For example, the prevalence of SLE is reported as very high in Georgia, United States, whereas the prevalence of AS is rare in malaria endemic regions where HLA-B27 genotypes are rare. Inflammatory arthropathies, including RA and AS, have a higher prevalence in North American Native populations.

CLINICAL MANIFESTATIONS

It is important to understand the potential clinical manifestations and natural history of rheumatic diseases. Primary care and hospital-based health care providers are often the first to evaluate a patient with an evolving rheumatic disease, and they need to be attuned to the presenting features to make a timely diagnosis. In many cases, the presentation could signal a life- or organthreatening condition. Evaluation of constitutional, systemic and joint symptoms should always include rheumatic disease in the differential diagnosis.

1713

CHAPTER 256  Approach to the Patient with Rheumatic Disease  

TABLE 256-1  COMMON RHEUMATIC DISEASES BROADLY CLASSIFIED ACCORDING TO PATHOGENESIS DEGENERATIVE DISEASES OF BONES AND JOINTS

SYSTEMIC AUTOIMMUNE DISEASES

SERONEGATIVE SPONDYLOARTHROPATHIES

VASCULAR RHEUMATIC DISEASES

AUTOINFLAMMATORY DISEASES

PAIN DISORDERS

Osteoarthritis

Rheumatoid arthritis

Ankylosing spondylitis

ANCA- associated vasculitis

Adult-onset Still’s disease

Regional myofascial pain syndromes

DISH

Systemic lupus erythematosus

Psoriatic arthritis

Temporal artery vasculitis

Crystal diseases

Tendonitis/bursitis

Degenerative disc disease

Sjögren’s syndrome

Reactive arthritis

Polymyalgia rheumatica

Pediatric periodic fever syndromes

Adhesive capsulitis

Spinal stenosis

Inflammatory myopathies (polymyositis, dermatomyositis)

Enteropathic arthritis

Behçet’s disease

Osteoporosis

Systemic sclerosis

Reflex sympathetic dystrophy Pain with hypermobility syndromes Fibromyalgia*

*The only pain disorder that has not been associated primarily with inflammation. ANCA = antineutrophil cytoplasmic antibody; DISH = diffuse idiopathic sclerosing hyperostosis (also linked to metabolic factors, including elevated growth hormone).

TABLE 256-2  WORLDWIDE PREVALENCE* AND INCIDENCE OF RHEUMATIC DISEASES ASSOCIATED WITH AUTOIMMUNITY DISEASE

NORTH AMERICA

CENTRAL AMERICA

SOUTH AMERICA

EUROPE

MIDDLE EAST

ASIA

SUB SAHARAN AFRICA

100-500

200-900 (2-7)

200-1500

100-800 (40-90)

Rare - 900

2000

50-60 (5)

N/A

20-70 (2-7)

N/A

20-70 (3)

Rare

20-80 (11)

N/A

N/A

95%)

Rim

Anti-double-stranded DNA

Double-stranded DNA

SLE (50%)

Speckled

Anti-Sm Anti-U1-RNP Anti-Ro (SS-A) Anti-La (SS-B) Anti-Ku Anti-SCL-70

snRNP proteins U1 snRNP proteins Two proteins complexed to small RNAs Y1-Y5 Single protein plus RNA polymerase III transcript DNA binding protein DNA topoisomerase I

SLE (30%) SLE (30%); MCTD (>95%) SLE (30%); Sjögren syndrome (70-80%) SLE (15%); Sjögren syndrome (50-70%) SLE (10%) PSS (40-70%); CREST (10-20%)

Nucleolar

Anti-PM-Scl Anti-Mi-2 Anti-RNA polymerase

Nucleolar protein complex Nuclear protein complex Subunits of RNA polymerase I

PSS (3%); PM (8%) DM (15-20%) PSS (4%)

Dividing cell

Anticentromere Antiproliferating cell nuclear antigen

Centromere/kinetochore protein Auxiliary protein of DNA polymerase δ

CREST (80%); PSS (30%) SLE (3%)

Cytoplasmic

Anti-Jo-1 Anti-PL-7 Anti-PL-12 Anti-SRP Anti-ribosomal P

Histidyl tRNA synthetase Threonyl tRNA synthetase Alanyl tRNA synthetase Signal recognition particle Large ribosomal subunit

ILD in PM/DM (18-25%) PM/DM (3%) PM (4%) SLE (10%) PM/DM (3%)

CREST = calcinosis, Raynaud phenomenon, esophageal dysmotility, sclerodactyly, and telangiectasia; DM = dermatomyositis; ILD = interstitial lung disease; MCTD = mixed connective tissue disease; PM = polymyositis; PSS = progressive systemic sclerosis (diffuse scleroderma); SLE = systemic lupus erythematosus; snRNP = small nuclear ribonucleoprotein; tRNA = transfer RNA.

1722

CHAPTER 257  Laboratory Testing in the Rheumatic Diseases  

6

correlate. Nevertheless, for each assay, the dynamic range for testing is large. With treatment and disease quiescence, anti-DNA antibodies may essentially disappear; with flare, levels may increase dramatically. This property distinguishes anti-DNA antibodies from other ANAs in SLE, levels of which tend to be more consistent over time. As is the case for other ANAs, the appearance of anti-DNA antibodies in the serum may precede other manifestations of SLE, suggesting vigilance if these antibodies are present in patients who have symptoms that suggest a CTD but lack other evidence to establish a firm diagnosis.

Other Antinuclear Antibodies

Anti-Sm and anti-RNP antibodies are related specificities that commonly occur together in the sera of patients with SLE, a phenomenon called linkage. These antibodies bind proteins on subcellular particles called snRNPs (small nuclear ribonucleoproteins) that are composed of a set of proteins and uridine-rich RNAs. Anti-Sm and anti-RNP antibodies differ in protein specificity and in the ability to cause immunoprecipitation of the bound RNA molecules. Anti-Sm antibodies occur only in patients with SLE and represent a serologic marker in disease classification. In contrast, anti-RNP antibodies can appear in the sera of patients with other clinical presentations and, in the absence of anti-Sm, may characterize patients with overlapping CTD features, so-called mixed CTD or MCTD. In SLE, the frequencies of anti-Sm and anti-RNP antibodies vary among racial and ethnic groups, although a clear association with particular clinical manifestations has not been established. Anti-Ro and anti-La antibodies (or anti-SS-A and anti-SS-B), another set of linked ANAs, are directed to protein-RNA complexes that are involved in cellular metabolism of RNA. These antibodies are expressed more widely in patients with CTD and appear in the sera of patients with SLE, RA, and Sjögren’s syndrome, among others. Assessment of these antibodies is important because of their association with the neonatal lupus syndrome, which results from the transplacental passage of antibodies and causes congenital heart block as well as rash in the neonate. Although both Sm/RNP and Ro/ La are complexes of proteins and RNA, these antibodies appear to be expressed by different patient subsets, suggesting distinct mechanisms of induction and clinical associations.7 Although ANAs are directed to ubiquitous antigens, they nevertheless are expressed in disease-specific patterns and may show association with particular organ-specific manifestations. These associations include anti–ribosomal P antibodies with central nervous systemic involvement in SLE, antibodies to DNA topoisomerase 1 (anti-SCL-70) with progressive systemic sclerosis (diffuse scleroderma), antibodies to centromeres with CREST syndrome (calcinosis, Raynaud phenomenon, esophageal dysmotility, sclerodactyly, and telangiectasia), and antibodies to histidyl transfer RNA synthetase (antiJo-1) with interstitial lung disease in scleroderma (Chapter 267). In inflammatory myopathies, the presence of certain autoantibodies may be associated with particular patterns of disease, with antibodies to the enzyme 3-hydroxy3-methylglutaryl-coenzyme A (HMG-CoA) reductase present in a syndrome of necrotizing myositis; the syndrome can occur in patients treated with statins, which can inhibit the enzyme.8 In addition to their association with specific disease manifestations, antibodies to both DNA- and RNA-binding proteins such as Sm and RNP may contribute to overall immune dysregulation in patients with autoimmune disease because of their formation of immune complexes containing DNA or RNA. These complexes can stimulate the production of type 1 interferon by triggering both TLR and non-TLR nucleic acid sensors as well as other cellular receptors (e.g., Fc receptors). Because immunoassays of interferon with patient sera are limited, the presence of interferon is observed more clearly in the pattern of gene expression known as the interferon signature in peripheral blood cells. This signature can be assessed by both microarray assays and measurement of more limited sets of messenger RNA molecules that are induced by interferon. Because antibodies to RNA-binding proteins in particular may promote this pattern, the serologic assay of these ANAs may allow assessment of the likelihood of both nonspecific and specific immunologic disturbances.9

Antibodies to Phospholipids

Originally defined by their effects on in vitro clotting tests, antibodies to phospholipids (APLs) are associated with in vivo thrombosis and have been termed lupus anticoagulants (LACs). Patients with these antibodies display a clinical condition, termed the antiphospholipid antibody syndrome, which is characterized by arterial or venous thrombosis, thrombocytopenia, and firsttrimester spontaneous abortions (Chapter 176). This syndrome may occur by itself or in the context of SLE, where it may contribute to the acceleration

of atherosclerosis, premature stroke, and myocardial infarction. The laboratory evaluation of this condition involves specific assays of antibodies to phospholipids and related proteins as well as functional assays of clotting. Because expression of these antibodies may vary over time, testing must be performed on more than one occasion at least 6 weeks apart. Furthermore, the results of immunochemical and functional assays may not be congruent because they are likely related to the heterogeneity of antibodies. The serology of APLs is complicated because it is related to the nature of the antigenic targets as well as heterogeneity among patients.10,11 These antigens include phospholipids such as cardiolipin. Cardiolipin, however, can bind to the protein β2-glycoprotein 1, which is also a target for antibodies in this condition. Serologic evaluation thus involves assays with a complex of cardiolipin and β2-glycoprotein 1 as well as β2-glcycoprotein in an ELISA format using reagents to measure IgG and IgM. The association of antibodies with thrombosis appears strongest with IgG antibodies; determination of the IgA isotype may also be informative depending on the results of IgG and IgM assays. ELISAs for these antibodies are not yet standardized, making it important to specify assay features related to quantitation such as cutoff values used to define positivity and values that are considered significant. Functional assays for LACs involve tests directed at inhibition of in vitro clotting (e.g., activated partial thromboplastin time, dilute Russell viper venom time), recognizing the discordance between in vivo thrombosis and in vitro anticoagulation. Functional assays to detect lupus anticoagulants involve a mixing step in which patient plasma is mixed with normal plasma to determine the presence of an inhibitor (i.e., an antibody) as opposed to a deficiency state. The mechanisms by which antibodies to phospholipids and related proteins may cause thrombosis in vivo are unknown, although these antibodies may interact with the surface of cells (e.g., endothelium) to promote a prothrombotic state. Assessing the likelihood of the syndrome is best accomplished by considering results of both the immunoassays and functional assays in the context of the individual patient because both factors may promote thrombotic events.

Complement

Assessment of the complement system can provide valuable information on the activity of diseases in which immune complex deposition may promote inflammation and tissue injury (Chapter 50). This system involves a large number of proteins that function in enzyme cascades to generate degradation products that amplify immunologic reactions and promote the destruction or removal of foreign organisms as well as damaged cells. In the setting of SLE and in certain forms of vasculitis and glomerulonephritis, immune complexes activate complement to promote local inflammation. This activation can be measured in terms of the total complement level in the blood by functional assays of hemolytic activity; by measurement of individual complement components such as C3 and C4, whose levels are reduced by cleavage during activation; by measurement of split products of cleaved complement components; and by measurement of complement fragments bound to red blood cells during complement activation. Proteins of the complement system are acute phase reactants and can increase with inflammation, including active disease. Correspondingly, low levels may reflect inherited complement deficiency rather than consumption; genetic deficiency of C1q, for example, is highly associated with SLE.

Antineutrophil Cytoplasmic Antibodies

Antineutrophil cytoplasmic antibodies (ANCAs) are autoantibodies that react to determinants in the neutrophil and occur prominently in patients with certain forms of necrotizing vasculitis or rapidly progressive glomerulonephritis. Reflecting the serology, conditions have been called ANCAassociated vasculitis (AAV). Two main forms of ANCA have been distinguished on the basis of the target antigens and pattern of immunofluorescence staining of fixed neutrophils: PR3-ANCA (C-ANCA), which reacts with proteinase-3 (PR3), and MPO-ANCA (P-ANCA), which reacts with myeloperoxidase (MPO). By immunofluorescence, PR3-ANCA shows staining in the cytoplasm; staining by MPO-ANCA localizes in the perinuclear area. ANCAs to other proteins have also been identified, but these may also occur in conditions other than vasculitis. In the evaluation of severe, multisystem inflammatory disease, ANCA testing is important to evaluate diagnostic possibilities.12 ANCAs occur in association with varying clinical manifestations in patients with AAV and help define patterns of clinical involvement in terms of organ system involvement as well as histopathology (e.g., presence of granulomatous inflammation). PR3-ANCA occurs commonly in patients with granulomatosis with polyangiitis (GPA, formerly called Wegener granulomatosis) as well

CHAPTER 258  Imaging Studies in the Rheumatic Diseases  

eosinophilic granulomatosis with polyangiitis (EGPA, formerly called Churg-Strauss disease); MPO-ANCA marks the course of vasculitis caused by microscopic polyangiitis.13 Although there is overlap between serology and clinical features, PR3-ANCA occurs commonly in patients with upper airway disease, whereas MPO-ANA occurs commonly patients with patients with rapidly progressive renal disease (Chapter 270). In patients with ANCA-associated glomerulonephritis, the kidney lacks evidence of immune deposits, as indicated by the lack of staining for immunoglobulins or complement. Kidney disease of this kind is termed pauciimmune glomerulonephritis. Although ANCA testing is useful in initial diagnosis, its role for assessing disease activity is less certain. Occasionally, in patients who are desperately ill and cannot tolerate a lung or kidney biopsy, the presence of an ANCA can be used as preliminary evidence for diagnosis to allow the initiation of immunosuppressive therapy. ANCA testing is also useful for assessing the likelihood for relapse because patients who express PR3-ANCA appear at risk for recurrent disease.

Cryoglobulins

Cryoglobulins are serum immunoglobulins that precipitate in the cold and promote the pathogenesis of systemic inflammatory disease through tissue deposition. The presence of a cryoglobulin is detected by allowing blood, collected warm, to remain cool at 2° to 4° C for 1 or more days. After centrifugation, the amount of cryoprecipitate is measured and expressed as a cryocrit. In the preanalytical phase, it is important that the blood remain at a temperature of 37° C during all steps from drawing the blood from the patient to separation of the serum fraction after coagulation. Thermos flasks with preheated sand or water and other special devices are available to keep the blood tubes at 37° C during transport. If these steps are not taken, cryoprecipitation at even room temperature may already occur before separation of the serum from the blood cells, possibly resulting in false-negative results.14 Subsequent analysis of the cryoprecipitate by immunochemical assays allows determination of its components. Cryoglobulins can be classified into three main types on the basis of their composition: (1) single, or type I; (2) mixed, type II; and (3) mixed, type III. A type I cryoglobulin consists of only a monoclonal immunoglobulin that precipitates in the cold. A mixed-type cryoglobulin contains RFs bound to polyclonal IgG to form an immune complex. In type II cryoglobulins, the IgM RF is monoclonal, and in type III, the IgM RF is polyclonal. Type I cryoglobulins occur in patients with lymphoproliferative disorders such as Waldenström macroglobulinemia, multiple myeloma, or chronic lymphocytic lymphoma (Chapters 184 and 187). In contrast, patients with mixed cryoglobulins can present with a wide range of signs and symptoms resulting from vasculitis. These manifestations include purpura (a sign of leukocytoclastic vasculitis), weakness, arthritis, and neuropathy, representing a syndrome known as essential mixed cryoglobulinemia. Most patients with this condition have infection with hepatitis C virus, with viral components present in the complexes. These patients have serologic evidence of this infection as well as manifestations attributable to the underlying liver disease. As in the case of other CTDs and systemic inflammatory diseases, the evaluation of patients with essential mixed cryoglobulinemia demands attention to the entire patient and the impact of disease on multiple organs. GENERAL REFERENCES For the General References and other additional features, please visit Expert Consult at https://expertconsult.inkling.com.

258  IMAGING STUDIES IN THE RHEUMATIC DISEASES RONALD S. ADLER Historically, rheumatic disorders have been well characterized by conventional imaging. In as much as these disorders often manifest in characteristic distributions and present with specific alterations in the appendicular or axial skeleton and adjacent soft tissues, radiographic evaluation has been sufficient to both characterize the abnormalities as well as provide a relatively small

1723

number of differential possibilities as to the specific disease. The most wellstudied example is rheumatoid arthritis (RA) in which symmetric involvement of the metacarpophalangeal joints, uniform joint space narrowing, periarticular osteopenia, and juxta-articular erosions along the “bare areas” are pathognomonic. The development of new therapeutic alternatives for the inflammatory arthritides, so-called disease-modifying antirheumatic drugs (DMARDs), and chondroprotective strategies in the case of osteoarthrosis, require methods to diagnose these diseases at an earlier stage, characterize the degree of inflammation, and provide a useful metric to assess therapeutic response.1 Indeed, it has become necessary to assess for possible joint and soft tissue abnormalities before irreversible tissue damage, the latter often being the case when the radiographic findings are abnormal. Fortunately, the requirement to achieve earlier diagnosis has paralleled advances in imaging. Ultrasonography and magnetic resonance imaging (MRI) have largely supplanted conventional radiographic evaluation in the imaging work-up of patients with suspected rheumatologic disorders and negative radiographs.2,3 The term molecular imaging has been applied, particularly in the case of MRI and positron emission tomography (PET), in as much as these modalities reflect local tissue environment or metabolic activity.

  RADIOGRAPHIC EVALUATION

Radiographic evaluation is among the first studies ordered in patients with a suspected rheumatologic disorder. In the current digital era, conventional analog-based radiographs have been largely replaced by computed radiography. Images are usually displayed on workstations with high-resolution monitors within the context of a picture archiving system (PACS). Digital radiographs are of high spatial resolution but relatively poor soft tissue contrast. These images are amenable to a variety of image processing schemes, resulting in enhanced definition of the cortical surfaces and cancellous bone, which may be of value in displaying subtle erosions. It is important to recognize that radiographs are projection images. To detect an abnormality, it may be necessary to view a joint or other structure at a specific angle. For instance, subtle erosions may only be apparent when viewed tangentially as opposed to en face. It is therefore necessary to have specific image protocols in order to optimally display the joint, cortical surface, or soft tissue structure. Most radiographic evaluations contain at least two orthogonal projections. The addition of an oblique view or other specialized projection may be necessary to address a specific clinical question. The nature and distribution of joint space narrowing, presence of osteopenia, new bone formation, soft tissue swelling, soft tissue calcification, chondrocalcinosis, presence and nature of erosions, and assessment for joint malalignment may allow a specific diagnosis and help determine the severity of disease (Fig. 258-1). For instance, the presence of a juxta-articular erosion extending over an adjacent area of slightly hyperdense soft tissue swelling in the setting of normal bone mineralization with maintenance of the adjacent joint space is diagnostic of gout, in contrast to RA noted earlier. The seronegative arthritides, such as psoriatic arthritis, have a characteristic appearance in the small joints of the hand and feet, including a predilection for distal joints, asymmetry, and appositional new bone formation. Table 258-1 summarizes some of the features of several of the more common diseases that may be encountered in clinical practice. Finally, radiographs provide a direct means for needle localization during percutaneous procedures, predominantly joint injections, aspirations, and some biopsies. These are generally performed while imaging in real time (fluoroscopy) using short bursts of low-intensity x-rays enhanced through an image intensifier. Injection of joints under fluoroscopic guidance provides a convenient means to ensure intra-articular deposition of therapeutic agent or for diagnostic aspiration. Intra-articular location is verified by injection of a small amount of a standard iodinated contrast material. Arthrography using fluoroscopic guidance can be used as a primary diagnostic tool, but this application has largely been replaced by intra-articular injection of contrast followed by computed tomography (CT) or MRI. For some procedures, CT may be preferable, depending on the location of the abnormality. The principal disadvantages of fluoroscopy relate to the use of ionizing radiation and poor soft tissue contrast. The latter becomes important with needle placements near neurovascular structures that may be potentially compromised by poor position. CT allows greater control over needle placement at the cost of greater levels of radiation exposure. Ultrasonography has replaced fluoroscopy and CT for a large number of percutaneous procedures. MRI provides another method to perform a variety of procedures without the necessity of ionizing radiation. These options will be discussed in greater detail below.

CHAPTER 257  Laboratory Testing in the Rheumatic Diseases  

GENERAL REFERENCES 1. Ansar W, Ghosh S. C-reactive protein and the biology of disease. Immunol Res. 2013;56:131-142. 2. Van Venrooij WJ, Van Beers JBC, Pruijn GJM. Anti-CCP antibodies: the past, the present and the future. Nat Rev Rheumatol. 2011;7:391-398. 3. Aleteha D, Neogi T, Silman AJ, et al. 2010 Rheumatoid arthritis classification criteria: an American College of Rheumatology/European League against rheumatism collaborative initiative. Arthritis Rheum. 2010;62:2569-2681. 4. Satoh M, Chan EK, Ho LA, et al. Prevalence and sociodemographic correlates of antinuclear antibodies in the United States. Arthritis Rheum. 2012;64:2319-2327. 5. Bizzaro N, Villata D, Giavarina D, et al. Are anti-nucleosome antibodies a better diagnostic marker than anti-dsDNA antibodies for systemic lupus erythematosus? A systematic review and a study of metanalysis. Autoimmun Rev. 2012;12:97-106. 6. Pisetsky DS. Standardization of anti-DNA antibody assays. Immunol Res. 2013;56:420-424. 7. Ching KH, Burbelo PD, Tipton C, et al. Two major autoantibody clusters in systemic lupus erythematosus. PLoS ONE. 2012;7:e32001.

1723.e1

8. Hamann PD, Cooper RG, McHugh NJ, et al. Statin-induced necrotizing myositis—a discrete autoimmune entity within the “statin-induced myopathy spectrum.”. Autoimmun Rev. 2013;12: 1177-1181. 9. Rönnblom L, Eloranta M-L. The interferon signature in autoimmune disease. Rheumatology. 2013;25:248-253. 10. Favaloro EJ. Variability and diagnostic utility of antiphospholipid antibodies including lupus anticoagulants. Int J Lab Hematol. 2013;35:269-274. 11. Lakos G, Favaloro EJ, Harris EN. International consensus guidelines on anticardiolipin and antiβ2-glycoprotein I testing. Arthritis Rheum. 2012;64:1-10. 12. Lionaki S, Blyth ER, Hogan SL. Classification of antineutrophil cytoplasmic autoantibody vasculitides. Arthritis Rheum. 2012;64:3452-3462. 13. Lally L, Spiera R. Current landscape of antineutrophil cytoplasmic antibody-associated vasculitis: classification, diagnosis, and treatment. Rheum Dis Clin North Am. 2015;41:1-19. 14. Damoiseaux J. The diagnosis and classification of the cryoglobulinemic syndrome. Autoimmun Rev. 2014;13:359-362.

1724

CHAPTER 258  Imaging Studies in the Rheumatic Diseases  

A

C

B

FIGURE 258-1.  Three hands with different diagnoses. A, Gout. Radiograph of the left hand showing multiple dense soft tissue nodules (n) with multiple small erosions affecting the ulnar styloid, triquetrum and fifth ray. A large erosion (arrow) at the fifth distal interphalangeal (DIP) joint demonstrates bone formation extending circumferentially about the adjacent tophaceous deposit typical of an overhanging edge. Bone mineralization and joint spaces are preserved. B, Rheumatoid arthritis. There is ulnar deviation of the second through fifth metacarpophalangeal (MCP) joints with uniform joint space loss involving the MCP joints and the carpus. The DIP joints are spared. Periarticular demineralization is present with small erosions along the radiovolar aspect of the second (arrow) MCP joint. C, Osteoarthrosis. Soft tissue swelling affecting the third digit with joint space narrowing and bone production affecting the DIP joints, third and fifth proximal interphalangeal (PIP) joints, basal joint of the thumb, and scaphotrapeziotrapezoid joint. There are subchondral cystic changes at the third PIP joint having an erosive character (arrow). Mineralization is preserved, as are the radiocarpal and MCP joint spaces.

TABLE 258-1  DISTINGUISHING RADIOGRAPHIC FEATURES OF SEVERAL COMMON RHEUMATIC DISEASES CONDITION Rheumatoid arthritis

COMMON SITES Hands: MCP, PIP; wrists: intercarpal, DRUJ, ulnar styloid; feet: fifth MTP, cervical spine (atlantoaxial, apophyseal)

DISTRIBUTION Bilateral, symmetric, polyarticular

Osteoarthritis (primary) Hands (DIP), wrists (basal joint, STT),feet Symmetric, weight-bearing joints (first MTP), hips (superolateral), knees (medial), spine (discs, facet, apophyseal, uncovertebral)

RADIOGRAPHIC FEATURES Periarticular osteopenia, periarticular swelling, subluxations (e.g., ulnar, volar), uniform joint space loss, erosions (bare areas) Normal or increased density, nonuniform joint space loss, subchondral sclerosis, cysts, bone formation (osteophytes) Spine–disc space narrowing, end plate sclerosis and bone formation

Psoriatic arthritis

Hands (DIP, terminal tufts), feet (IP joints), entheses (calcaneus: plantar, posterior), spine, sacroiliac joints

Asymmetric (single ray), polyarticular, Normal or increased density, periosteal bone formation, segmental (intervertebral, apophyseal) soft tissue swelling, ankylosis (SI joints), thick hyperostosis spine (nonmarginal syndesmophytes), juxta- and periarticular erosions

Ankylosing spondylitis

Spine, SI joints, fibrous joints (pubic symphysis), entheses (adductor origin), rhizomelic joints (hips, shoulders)

Symmetric, continuous (may affect entire Normal or increased density, erosions (spine squaring, spine-bamboo spine) shining corner) with superimposed bone formation (ankylosis: SI), thin (marginal) syndesmophytes)

Gout

Feet (first MTP), other damaged joints, elbow, knee, hindfoot

Asymmetric, extensor surfaces (elbow), abnormal joints (e.g., osteoarthritic joints)

Normal joint space, normal or increased density, dense soft tissue nodules (tophi), para-articular and subchondral erosions with bone formation along tophi (overhanging edge)

Calcium pyrophosphate dihydrate crystal deposition disease

Hands (second, third MCP), wrists (radiocarpal), TFC, knees (lateral compartment and patella-femoral, menisci)

Symmetric, fibrocartilaginous joints

Normal or increased density, hypertrophic bone formation, subchondral or periarticular cysts, chondrocalcinosis (hyaline, fibrocartilage), periarticular, peritendinous, periligamentous calcification

Infection

Any joint, pyogenic, TB

Monoarticular (mostly), any joint

Pyogenic (osteopenia; 8-10 days), joint space widened (early), joint space loss (rapid development), soft tissue swelling, erosions (both sides of joint), sequestra, periostitis, TB (joint space and mineralization may be preserved), juxta-articular erosions, spine–disc space loss and end plate erosion

DIP = distal interphalangeal; DRUJ = distal radial ulnar joint; IP = interphalangeal; MCP = metacarpophalangeal; MTP = metatarsophalangeal; PIP = proximal interphalangeal; SI = sacroiliac; STT = scaphotrapezotraphezoid; TB = tuberculosis; TFC = triangular fibrocartilage.

CHAPTER 258  Imaging Studies in the Rheumatic Diseases  

  COMPUTED TOMOGRAPHY

Computed tomography provides a two-dimensional map of tissue attenuation obtained from external x-ray source(s) located on a rotating gantry, whose radiation is detected by a series of detectors opposite the source. The current generation of CT scanners uses multiple detectors (16, 32, 64, and so on), allowing rapid image acquisition that can be displayed in a single plane in real time (CT-fluoroscopy) or as extremely thin section contiguous or overlapping acquisitions in the axial plane. The acquired images can be reconstructed in multiple planes with equivalent (isotropic) resolution elements (voxels) or as a three-dimensional rendering. Some scanners use dual energy sources, taking advantage of differences in the attenuation characteristics of various tissues at different energies. This has received greatest attention in the setting of gout, enabling a definitive diagnosis as well as depicting tophaceous deposits in anatomic locations not conducive to radiographs or ultrasound.4 Computed tomography allows the best assessment of trabecular and cortical bone, providing an excellent means to assess fractures and erosions, the presence of new bone formation (e.g., fracture callus), and degenerative or inflammatory arthritis. Soft tissue mineralization can likewise be well characterized, providing important information as to its etiology. Joints that are difficult to assess on radiographs, including the sacroiliac, temporomandibular, wrist, and sternoclavicular joints, are well seen on CT (Fig. 258-2). Computed tomography generally has poor soft tissue contrast. Nevertheless, it is still very useful in performing a number of guided procedures because of its tomographic nature and rapid image acquisition capability. Improved soft tissue contrast can be obtained with use of iodinated contrast material. A number of soft tissue tumors, inflammatory synovitis, and infectious processes display pathologic enhancement after contrast administration. CT can likewise be used to produce angiographic displays (CTA) when used in combination with contrast, providing exquisite detail of central and peripheral vascular disease, including in patients with suspected vasculitis. These agents are typically administered intravenously following well-defined enhancement characteristics. CTA has become the method of choice in evaluating patients with suspected pulmonary embolism. Likewise, contrast agents may be used to improve intra-articular contrast (CT arthrography), currently the method of choice in assessing internal derangement in the postoperative shoulder, knee, and so on and in patients who are unable to undergo MRI (e.g., those with claustrophobia, aneurysm clips, or cardiac pacemakers). Imaging of cartilage and soft tissue abnormalities usually depends on pathologic imbibition of contrast material, indicative of degeneration or tearing. A limitation of this approach resides in the fact that some abnormalities may remain occult. An example is the inability to detect a bursal-sided rotator cuff tear after shoulder CT arthrography. The radiation dose from CT can be high, especially when using the newer scanners. This is most significant when one is looking to minimize exposure, such as in children, requiring protocols specifically designed for the pediatric

A

1725

population. Intravenous (IV) use of iodinated contrast agents are contraindicated in patients with impaired renal function or history of allergic reaction. Nonionic agents can diminish the associated risks but still should be used with caution.

  ULTRASONOGRAPHY

Ultrasound imaging takes advantage of the near uniform speed of sound and predictable attenuation characteristics of sound propagation in soft tissue. In general, anatomic images derive from specular surfaces whose dimensions exceed the ultrasound wavelength; inherent noise (speckle) within the image derives from small scatterers, smaller than the resolution element of the transducer. Modern ultrasound equipment contains various methods to reduce speckle in the image, resulting in a more anatomic rendition of the soft tissues. Rapid image acquisition and processing enables ultrasonography to be performed in real time (≈30 frames per second). Ultrasonography is also conducive to evaluation of blood flow from which estimates of flow velocity can be obtained through the Doppler equation. Doppler information is typically reported by either continuously estimating velocity at a specific depth (spectral Doppler) or through a color encoded two-dimensional map (color or power Doppler). There is great appeal for using ultrasonography in patients with rheumatic disorders. There is no ionizing radiation, and it is real time, inexpensive, relatively portable, and well tolerated. Historically, however, ultrasonography has played only a limited role in the diagnostic assessment and treatment of patients with suspected musculoskeletal abnormalities, being used to differentiate fluid-filled from solid masses. The detection of a Baker cyst in the knee or the presence of a joint effusion constituted two major applications. There has also been limited application of ultrasonography to perform imageguided aspirations and biopsies. Within the United States, in particular, the development of MRI further limited the musculoskeletal applications of ultrasonography. With the development of linear high-frequency small parts transducers; new imaging capabilities of ultrasound scanners; and the evolution of a new class of compact, portable (laptop) ultrasound units that have excellent image quality, the role of ultrasonography has dramatically changed in recent years.5,6 These new applications have paralleled the development of new classes of DMARDs for which diagnosis of inflammatory synovitis prior to joint destruction is optimal. The current generation of ultrasound scanners enables examination of the small joints of the hands and feet, allowing early detection of synovitis (Fig. 258-3). Typically, a 10-MHz or higher frequency linear transducer is used. The displacement of the joint capsule by hypoechoic (dark) soft tissue that displays vascularity on Doppler imaging or is incompressible with direct pressure by the transducer is characteristic, allowing differentiation of synovitis from an effusion. In addition to the detection of synovitis, ultrasonography has been shown to be more sensitive than conventional radiographs in the detection of erosions. Erosions appear as discrete irregular discontinuities

B

FIGURE 258-2.  Infectious sacroiliitis in a 12-year-old boy with a 2-week history of back and left hip pain. A, Axial computed tomography (CT) image of the pelvis at the level of the sacroiliac joints (SIs) photographed using window settings optimized for bone detail. There is clear asymmetry in the two SI joints, with the left appearing more irregular. The cortical margins of the left sacral ala are less distinct and there is an isolated bone fragment (arrow) surrounded by soft tissue suspicious for a sequestrum. B, CT-guided aspiration of the left SI joint confirmed an infectious origin.

1726

CHAPTER 258  Imaging Studies in the Rheumatic Diseases  

A

B

FIGURE 258-3.  Synovitis on ultrasonography in a female patient with normal hand radiographs. A, Gray-scale ultrasound image obtained along the dorsal aspect of the radioscaphoid joint shows hypoechoic soft tissue (arrows) distending the dorsal recess. The cortical margins of the scaphoid (S) and radius (R) appear as bright reflectors on ultrasonography. B, Power Doppler image depicts the marked vascularity (red color hues) of the soft tissue illustrating the level of disease activity.

in the normally smooth hyperechoic (bright), reflecting cortical surfaces, often seen in continuity with adjacent inflammatory soft tissue. There is some variation in the appearance of synovitis among various arthritides. The distribution, presence, or lack of symmetry and other concomitant findings may be necessary to obtain a specific diagnosis. The level of vascularity on color-flow imaging can reflect active inflammation, correlating with clinical and biochemical parameters. A parametric image encoding either mean Doppler shift (color Doppler) or amplitude (power Doppler) is typically used as a standard Doppler map. Both maps can be used to detect abnormal levels of vascularity. Whereas power Doppler provides an indirect measure of the number of moving scatterers within the region being scanned, color Doppler provides a velocity map, and therefore is more subject to artifact (angle dependence and sampling errors). When combined with color-flow imaging, the activity of the synovitis can be estimated. Ultrasound contrast agents can depict capillary flow, resulting in significantly improved detection sensitivity of synovial inflammation, and are used extensively in Europe. They constitute microbubble agents encased in a lipid or polysaccharide shell that can be instilled as either bolus or constant infusion, with the shell being metabolized in the liver and the gas exhaled in the lungs. These agents have biological half-lives on the order of minutes and are best suited to examining target joints. Contrast agents have received Food and Drug Administration approval for cardiovascular applications and therefore can only be used off label for the assessment of synovitis. Articular and fibrocartilage have characteristic appearances on ultrasonography. Whereas the former appears as a thin hypoechoic band paralleling the articular surface, fibrocartilage appears hyperechoic. Chondrocalcinosis appears as discrete hyperechoic foci with the substance of the cartilage, in which case its presence is suggestive of calcium pyrophosphate deposition disease. Calcification along the margin of the articular cartilage gives rise to the double-line sign seen in gout. Tendons and muscles have characteristic appearances on ultrasonography. The presence of tendinosis, tendon tears, muscle edema or inflammation, atrophy, and tears can be diagnosed. Ultrasonography is very sensitive, although not specific, for the detection of small amounts of calcification or ossification. It is an excellent method to assess for calcific peritendinitis and to provide guidance for treatment. Abnormal fluid distention of synovial lined structures can be assessed and treated under ultrasound guidance. Ultrasonography is an excellent modality to provide image guidance for therapeutic aspiration and injection of small and large joints, tendon sheaths, and cysts (e.g., bursae, ganglion, paralabral cysts, hematomas, abscesses,) (Fig. 258-4). The real-time capability of ultrasonography is useful to demonstrate the presence of subluxations or painful snapping, to document the distribution of injected material, and to assess adhesions. Ultrasonography is considered the method of choice to detect foreign bodies. Nerves also have a characteristic appearance on ultrasonography. In crosssection, a nerve often has a “cluster of grapes” appearance, with nerve fascicles appearing hypoechoic and surrounded by hyperechoic endo- and epineural fat. In long axis, nerves display a characteristic tram-track appearance. Ultrasonography has been shown to be useful in the diagnosis and treatment of carpal tunnel syndrome and cubital tunnel syndrome. It is an excellent modality to assess for the presence of posttraumatic or postsurgical and interdigital neuromas and to provide image guidance for treatment, including therapeutic injections and ablative therapy.

Although ultrasonography is well suited to the evaluation of superficial structures, it is less well suited to deep structures. Frequency and penetration are reciprocally related: the higher the frequency, the better the axial resolution but poorer the degree of penetration. A 15-MHz linear transducer would work well in the hand but not in the hip. Examination of a hip might require a 5-MHz transducer and curved transducer geometry with reduced image quality. Excessive abdominal fat can further limit acoustic penetration and distort the ultrasound beam, limiting image quality. Diagnostic ultrasonography does not penetrate bone, resulting in limited acoustic access to joint structures. In some instances, soft tissue contrast can be poor. An inexperienced scanner may find ligaments and tendinous insertions difficult to differentiate from adjacent fibrofatty structures.

  MAGNETIC RESONANCE IMAGING

The natural abundance of hydrogen in biological systems and an inherent property of hydrogen, called spin, form the basis of conventional MRI. When placed in a strong magnetic field, protons tend to align themselves along the direction of the field. Magnetic field strengths are specified as Tesla and can be variable between clinical scanners. The majority of scanners in clinical usage vary between 1 and 3 Tesla. Application of a radiofrequency (RF) pulse to the system of protons induces the spins to rotate away from the direction of the field, during which time they precess about the direction of the magnetic field at a characteristic frequency, called the Larmor frequency. When the RF pulse is turned off, the spins relax toward their initial state determined by two tissue-dependent relaxation times, T1 and T2, which vary with field strength. T1 (also known as the spin-lattice relaxation) and T2 (or the transverse relaxation time) along with proton density are the principal determinants of signal intensity. The image can emphasize either the T1 or T2 characteristics of the tissue, impacting tissue contrast. Different tissues have varying appearance often based of levels of fat and water content, reflected by their inherent T1 and T2 relaxation times. Tissue morphology is often characterized by their appearance on T1-weighted or proton density images: tendon, muscle, fat, marrow, cortical bone, articular, and fibrocartilage have characteristic appearances. Many pathologic states, alternatively, are characterized by increased mobile water or effective T2 lengthening. Examples include soft tissue edema, inflammatory infiltrates, and neoplasm (Fig. 2585). Images that emphasize T2 contrast are therefore helpful to display most pathologic states. Selective maps of T2 have been used to characterize the state of articular cartilage in early degenerative disease.7 Other cartilage specific properties that relate to water content, glycosaminoglycan (GAG) content, and integrity of collagen architecture can be assessed using T2 and other parametric maps that can be derived from the MR data (Fig. 258-6). The widely used contrast for MRI studies is a neutral, hydrophilic salt of the gadolinium chelate, gadolinium diethylenetriamine-penta-acetic acid (Gd-DTPA). Gadolinium can be injected intravenously or directly into the joint. IV injection (indirect magnetic resonance arthrography) carries the contrast in the vascular system to areas of hyperemia and inflammation (Fig. 258-7). It can be used for assessment of synovial activity in inflammatory joint diseases.8 Gadolinium is taken up in inflamed synovium and is able to demonstrate thickened pannus. The slope of the early time-signal intensity curve provides a measure of tissue perfusion and can quantify inflammatory activity. Contrast material excreted into the synovial fluid provides excellent depiction of intra-articular structures and can be used in lieu of arthrographic

CHAPTER 258  Imaging Studies in the Rheumatic Diseases  

B

A

C

1727

D

FIGURE 258-4.  Ultrasound-guided therapy in the first metatarsophalangeal (MTP) joint of a patient with pain and swelling. A, Longitudinal gray scale image of the dorsal recess of the first MTP joint. Fluid and soft tissue distend the joint capsule (arrows). The metatarsal (m) and proximal phalanx (p) are labeled. Note that a thin hypoechoic (dark) band parallels the surface of the metatarsal head, corresponding to the overlying articular cartilage. B, Increased vascularity (red color hues) demonstrated on power Doppler imaging within the dorsal recess reflects the level of disease activity. C, Transverse gray scale ultrasound image shows a needle (N) within the distended dorsal recess from which several drops of synovial fluid were aspirated followed by therapeutic injection. D, Postinjection transverse ultrasonography depicts low level echoes (small echogenic foci within dorsal recess) and microbubbles (arrows) within the distended joint capsule from injected material. Whereas microbubbles aggregate along the nondependent portion of the distended joint capsule, injected material tends to settle to the deep portion of the recess.

A

B

FIGURE 258-5.  Magnetic resonance image of the right wrist in a female patient with advanced rheumatoid arthritis. A, Proton density coronal image shows loss of normally bright marrow signal within the scaphoid and lunate bones (arrows). The proximal scaphoid is eroded, and the lunate appears deformed and translocated and volarly tilted (not shown), giving rise to its triangular appearance. The distal ulna (u) is poorly visualized because of a large erosion. Intermediate-intensity material (appears dark gray) within the carpus and distal radioulnar joint is difficult to separate from the distal ulna, lunate, and scaphoid. The triquetrum (t), hamate (h), trapezium (tr1) and trapezoid (tr2), capitate (c), and radius (r) are labeled. B, Fluid-sensitive coronal image emphasizing T2 relaxation demonstrates increased signal intensity (bright) within the inflammatory pannus, compatible with increase in mobile water associated with inflammation. Increased signal intensity is evident within the lunate, scaphoid, and distal ulna, including focal areas within the distal row of carpal bones, corresponding to small erosions. Diffuse increased signal within the distal radius likely reflects reactive marrow edema (asterisk).

1728

CHAPTER 258  Imaging Studies in the Rheumatic Diseases  

A

B

C

D

FIGURE 258-6.  Imaging of the soft tissues in a patient with retrocalcaneal pain demonstrating complementary nature of magnetic resonance imaging and ultrasonography. Whereas the proton-density sagittal image (A) emphasizes anatomic detail, the fluid-sensitive image (B) depicts thickening and increased signal intensity within the distal Achilles tendon, reflecting tendinosis, retrocalcaneal bursitis, or a tear of the deep surface of the tendon. Surrounding increased signal intensity (bright areas) within the adjacent soft tissue reflects adjacent soft tissue edema. Long-axis gray scale (C) and power Doppler (D) images of the same patient obtained when the patient presented for ultrasound-guided therapeutic injection. The tendon (arrows) is inhomogeneous. A prominent hypoechoic collection deep to the tendon is compatible with retrocalcaneal bursitis (b). There is prominent increased vascularity on power Doppler imaging at the margin of the bursa and tendon. The calcaneus (calc) is labeled.

direct techniques. In glycosaminoglycan (GAG) -depleted cartilage, there can be delayed uptake of contrast into the cartilage, which would normally be inhibited by the negatively charged GAG molecules. Patients with renal disease who receive IV injection of gadolinium can develop nephrogenic systemic fibrosis (NSF). (Also see the section Nephrogenic Systemic Fibrosis in Chapter 267.) When the kidney cannot sufficiently clear out the gadolinium, it produces fibrosis of many tissues, including the skin, muscle, heart, nerves, and pleura. To date, NSF has been seen only in patients who have been given IV gadolinium with acute or chronic renal insufficiency. The changes in the skin with NSF are usually bilateral and symmetrical, primarily involving the extremities and the trunk. These changes can mimic systemic sclerosis but, unlike that disease, the face is usually spared. If renal function improves, the skin lesions may stabilize or get better, although in some patients, the process progresses, affecting mobility and causing severe pain. Injection of dilute gadolinium into the joint (direct magnetic resonance arthrography) is helpful for outlining structures to determine whether there is morphologic damage. Injection is usually performed either under fluoroscopic or ultrasound guidance. This technique is particularly effective for visualization of small structures such as the labrum of the hip or shoulder if there is no joint effusion. It is also helpful for demonstrating breakdown of soft tissue structures that normally prevent communication between joint compartments such as the rotator cuff, triangular fibrocartilage of the wrist, and ligaments in the various joints. Newer techniques that enable image acquisition in near real time as well as the development of MR-compatible needles now permit a variety of percutaneous procedures to be performed directly under MR guidance.

  SCINTIGRAPHY

Scintigraphy by its nature represents physiologic imaging because it derives from labeling physiologically occurring substances with a gamma-emitting

radionuclide and uses detectors in the form of gamma cameras arranged in a planar or circumferential configuration to determine the distribution of radionuclide within the tissue. Scintigraphy can provide a global assessment of abnormal tracer update or can be performed using a targeted approach (Fig. 258-8). Images often provide high tissue contrast but are of relatively poor spatial resolution. Commonly used agents vary from tagged red blood cells to assess blood flow; agents that reflect bone metabolism (technetium99m methylene diphosphate [Tc-MDP]); agents that reflect glucose metabolism (18-fluorine deoxy-glucose [18-FDG]), in the case of PET ; and agents that concentrate at sites of inflammation, such as autologous white blood cells labeled with 111In (Indium) and 67Ga-citrate (gallium). Clinical applications include detection of a variety of malignancies, osteomyelitis, vascular graft infection, multifocal infectious disease, inflammatory diseases such as RA, vasculitis, inflammatory bowel disease, sarcoidosis, fever of unknown origin, and infection of joint prostheses. Traditional nuclear medicine involves use of single gamma photon emissions as a product of nuclear decay. The information can be displayed using planar imaging through a single (or multiple) pinhole camera or displayed tomographically in a manner similar to CT (single-photon emission computed tomography [SPECT]). Bone scintigraphy uses Tc-MDP as the radioactive tracer. The isotope goes to areas of high bone turnover and vascular flow as well as areas of calcium or bone deposition. Three-phase bone scans are obtained at different intervals after injection, reflecting the early vascular phase, the intermediate blood pool phase, and the late phase. Each phase allows for further characterization of the disease process. Abnormal tracer uptake is seen in areas of inflammation, infection, neoplasm, osteonecrosis, and fracture. The scan is most useful to identify the location of lesions within the skeleton but is nonspecific. Positron emission tomography scans use the appearance of two simul­ taneously produced 511-KEV gamma rays after annihilation of a positron and electron pair to localize the distribution of radionuclide. The

CHAPTER 258  Imaging Studies in the Rheumatic Diseases  

1729

A 0

(min–1)

0.38

0

0.57

0

0.05

100 90 80 70 60 50 40 30 20 10

B ms

A

B

FIGURE 258-7.  Functional magnetic resonance image. A, Parametric image derived from fitting a twocompartment model of soft tissue and synovial enhancement after intravenous administration of contrast material. Three parameters extracted from the time-intensity curves are displayed as parametric images: Ktrans (left) provides a measure of contrast exchange into the extravascular soft tissues; Ve (center) and Vp (right) reflect the relative distribution volumes for the extravascular space and plasma, respectively. The arrows depict a region of increased synovial volume and enhancement at the second MCP joint. Increased values of Ktrans and Ve illustrate increased vascular permeability at the site of inflammation. (Printed with permission of Dr. Luis Beltran.) B, Sagittal T2 map of the knee in which relative T2 relaxation is color encoded, showing regions of higher cartilage T2 values at the femoral condyle and tibial plateau. This reflects alterations in cartilage collagen architecture and water content and possibly early osteoarthritis. (Printed with permission of Dr. Gregory Chang.)

C

D

FIGURE 258-8.  Rectilinear bone scan in a patient with back pain. Anterior (A) and posterior (B) delayed images of the axial and appendicular skeleton demonstrate increased tracer uptake in the region of the sacral ala, left ankle, and right midfoot (arrows). Follow-up radiographs confirmed the presence of bilateral sacral ala fractures. Note that the central pooling of tracer in the expected location of the urinary bladder is normal. Bone scans provide a sensitive but nonspecific method to evaluate the appendicular and axial skeletal. Increased uptake in the feet in this patient was attributed to degenerative change.

1730

CHAPTER 259  Connective Tissue Structure and Function  

near-simultaneous detection of the photons (coincidence counting) provides an estimate of source tracer concentration. Newer PET scanners are often used in combination with either CT or MRI to achieve improved spatial registration, allow accurate estimates of soft tissue attenuation, provide highquality anatomic images, and quantify metabolic activity.9 Combined PET-CT or PET-MRI provides high-resolution images of abnormal metabolic activity and may ultimately provide the most definitive maps of inflammatory activity in patients with rheumatic disease.10 Early results to date have been promising and are expected to provide sensitive evaluation of the response to DMARDs in patients with inflammatory arthritis. GENERAL REFERENCES

In addition to their structural role, connective tissues are involved in storage and activation of growth factors, cytokines, and morphogens. Some connective tissue molecules mediate inflammation when released as intact molecules, or may acquire new properties as proteolytically derived fragments (matrikines).1 Connective tissues such as bone, cartilage, and tendon harbor mesenchymal stem-like cells (MSC) that undergo expansion and differentiation during growth, repair, and regeneration and are therapeutically valuable. Bone contributes to calcium and phosphate homeostasis. A broad perspective on connective tissue is thus crucial for understanding the complex clinical presentation of inherited connective tissue disorders, pathways of tissue repair and regeneration, and pathogenesis of degenerative and autoimmune conditions.

For the General References and other additional features, please visit Expert Consult at https://expertconsult.inkling.com.

259  CONNECTIVE TISSUE STRUCTURE AND FUNCTION SUNEEL S. APTE Connective tissues are those having a primary mechanical or structural role. They are typically skeletal or osteoarticular tissues such as bone, cartilage, tendon, ligament, fascia, intervertebral disc, and joint-associated structures such as synovium and fibrocartilaginous menisci, although adipose tissue is also included (Table 259-1). However, all viscera, glands, and vascular and nervous tissue contain varying amounts of connective tissue elements that maintain proper spatial relationships between their cells, organize them into functional units, and provide an internal fibrous skeleton or external protective capsule. These connective tissue elements primarily comprise extracellular matrix (ECM), which is organized as a basement membrane or an interstitial matrix. Examples include elastic fibers in the aorta and lung that mediate stretch and recoil, glomerular basement membrane that participates in filtration, and the fibrous endoskeleton that connects the heart valves and transmits myocardial contraction. Indeed, heart valves and chordae tendinae have regional structural similarities to cartilage and tendons.

  CONNECTIVE TISSUE CELLS, THEIR EXTRACELLULAR MATRIX AND   TISSUE-SPECIFIC BIOMECHANICS

Connective tissues comprise cells, typically derived from mesoderm, or in the craniofacial region, derived from the neural crest, and the ECM they secrete and assemble around them. Unique morphologic and biosynthetic properties of the differentiated cells specify osteoblasts, osteoclasts, and osteocytes (in bone); chondrocytes (in cartilage); and tenocytes (in tendons), as well as mixed phenotypes such as cells of fibrocartilage (in the intervertebral disc, some tendon and ligament insertions, and joint menisci). Chondrocytes alter their biosynthetic profile as they terminally differentiate into hypertrophic chondrocytes in growth plate cartilage, downregulating aggrecan and collagen II expression and synthesizing collagen X as a specialized product.2 As the growth plate transitions into bone, the hypertrophic cells die, blood vessels invade the cartilage, and the cartilage matrix is gradually replaced by bone. Fibroblasts, which secrete collagens as a major product, are the dominant cells in tendons, fascia, ligaments, and dermis. The abundant ECM of connective tissues is their major defining characteristic, but in other tissues and organs, fibroblasts are relatively quiescent, and the interstitial ECM is less organized and not as abundant. However, under appropriate stimuli, fibroblasts transition to highly contractile, biosynthetically active myofibroblasts, which are associated with hypertrophic scars and fibrosis. During embryonic development, undifferentiated connective tissue cells (mesenchymal cells) produce a hydrated, loose, provisional ECM optimal for migration, branching, and folding of individual cells and cell collectives, such as epithelial sheets. As embryogenesis progresses, provisional ECM is remodeled by matrix-degrading proteinases and replaced by specialized connective tissues whose mechanical properties3 are better suited to weight bearing, locomotion, and increased circulatory stress. This requirement for increased mechanical strength is not met in the severest forms of inherited connective

TABLE 259-1  DIVERSITY OF CONNECTIVE TISSUES STRUCTURE

FUNCTION

CELLS

KEY MATRIX COMPONENTS

Adipose tissue

Energy metabolism Physical protection

Adipocytes, fibroblasts, endothelial cells

Collagen I, collagen III, microfibrils, collagen IV, laminin

Basement membranes

Epithelial support Cell polarity Filtration barrier Cell barrier Transparency (lens)

Epithelial and endothelial cells, myotubes, adipocytes, lens fibers

Collagen IV, laminin, nidogens, perlecan

Bone

Structural support Hematopoiesis Mineral storage

Osteoblasts, osteoclasts, osteocytes

Collagen I, osteocalcin, bone sialoprotein, hydroxyapatite

Cartilage

Bone growth Joint motion Load transmission

Chondrocytes

Collagen II, aggrecan

Dermis

Elasticity and resiliency

Fibroblasts

Collagen I, elastin

Ligament

Connects bone to bone

Fibroblasts

Collagen I, SLRPs

Tendon

Connects muscle to bone

Fibroblasts

Collagen I, SLRPs, fibrillins

Visceral stroma

Internal scaffold and capsule

Fibroblasts

Collagen I, collagen III, versican, fibronectin

Synovium

Produces synovial fluid

Fibroblasts, macrophages

Collagen I, collagen III

Vessel wall

Barrier, elastic recoil

Endothelium, vascular smooth muscle cells

Collagen III, collagen IV, elastin, fibrillins, fibulins, versican

SLRP = small leucine repeat-rich protein.

CHAPTER 258  Imaging Studies in the Rheumatic Diseases  

GENERAL REFERENCES 1. Haavardsholm EA, Lie E, Lillegraven S. Should modern imaging be part of remission criteria in rheumatoid arthritis? Best Pract Res Clin Rheumatol. 2012;26:767-785. 2. Kang T, Horton L, Emery P, et al. Value of ultrasound in rheumatologic diseases. J Korean Med Sci. 2013;28:497-507. 3. Sofka CM. Tracking rheumatic disease through imaging. Rheum Dis Clin North Am. 2013;39: 633-644. 4. Desai MA, Peterson JJ, Garner HW, et al. Clinical utility of dual energy CT for evaluation of tophaceous gout. Radiographics. 2011;31:1365-1375. 5. Mandl P, Kurucz R, Niedermayer D, et al. Contributions of ultrasound beyond clinical data in assessing inflammatory disease activity in rheumatoid arthritis: current insights and future prospects. Rheumatology (Oxford). 2014;53:2136-2142.

1730.e1

6. Rowbotham EL, Wakefield RJ, Granger AJ. The technique and application of ultrasound in the diagnosis and management of inflammatory arthritis. Semin Musculoskel Radiol. 2012;16:360-366. 7. Palmer AJR, Brown CP, McNally EG, et al. Non-invasive imaging of cartilage in early osteoarthritis. Bone Joint J. 2013;95-B:738-746. 8. McQueen FM. MRI in rheumatoid arthritis: a useful tool for the clinician? Postgrad Med J. 2014;90:332-339. 9. Yamashita H, Takahashi H, Kubota K, et al. Utility of fluorodeoxyglucose positron emission tomography/computed tomography for early diagnosis and evaluation of disease activity of relapsing polychondritis: a case series and literature review. Rheumatology (Oxford). 2014;53: 1482-1490. 10. Gotthardt M, Bleeker-Rovers CP, Boerman OC, et al. Imaging 0f inflammation by PET, conventional scintigraphy and other imaging techniques. J Nucl Med Technol. 2013;41:157-169.

1730.e2

CHAPTER 258  Imaging Studies in the Rheumatic Diseases  

REVIEW QUESTIONS 1. With regard to imaging of gout, the following statement(s) is (are) true: A. The presence of periarticular articular erosions and soft tissue nodules are pathognomonic. B. Osteopenia is a hallmark of the disease. C. Dual-energy computed tomography can be of value to assess disease extent. D. It has a characteristic appearance on ultrasound in these patients. E. C and D are correct. Answer: E  Dual-energy CT is well suited to assess the distribution of tophaceous deposits in patients with gout because of differences of the absorption characteristics of sodium urate crystals compared with other types of deposition diseases. On ultrasonography, calcification along the superficial margin of the cartilage is characteristic of gout, giving rise to a double-line sign. 2. A patient presents with radial-sided wrist pain that extends proximally into the forearm. What imaging test(s) would assess possible etiologies? A. Radiographs alone should be adequate B. MRI C. Ultrasonography D. B and C are correct E. A and B are correct Answer: D  The clinical history suggests DeQuervain’s tenosynovitis. Radiographs would allow assessment of the osseus structures but not the adjacent tendons. Although MRI would provide the most complete examination of the radial-sided structures, ultrasonography is well suited to evaluating the soft tissues, including the first dorsal compartment tendons. One also could perform ultrasound-guided therapy at the time of diagnosis. 3. A patient is suspected of having sacroiliitis. What imaging study would be appropriate to initially evaluate the patient? A. Radiographs of the SI joints B. Computed tomography C. Nuclear scintigraphy (bone scan) D. MRI E. Ultrasonography Answer: A  Dedicated radiographic views of the sacroiliac joints should be the first study ordered to assess for possible erosions and may be sufficient to establish the diagnosis. Computed tomography would provide a more sensitive evaluation for subtle erosions as well as the adjacent soft tissues, particularly with the addition of intravenous contrast. Computed tomography provides an ideal method to perform guided therapy or aspiration. A bone scan would provide a sensitive evaluation of the SI joints but findings would be likewise abnormal for trauma, inflammatory, or degenerative etiologies. A bone scan would allow a global assessment of the axial and appendicular skeleton to determine whether other sites are potentially affected. MRI allows the best assessment of the bone, adjacent marrow space, and soft tissues.

4. A patient has shoulder pain and gives a history of prior dislocation. There is an equivocal abnormality on the humeral head on radiographic evaluation. Which additional study should be considered? A. Ultrasonography to rule out a rotator cuff tear B. Noncontrast computed tomography C. Additional specialized radiographs to evaluate the scapula D. Direct MR arthrography E. Nuclear scintigraphy Answer: D  Although ultrasonography could evaluate the rotator cuff, it does not provide adequate assessment of the capsular labral complex. Crosssectional imaging with intra-articular contrast would best accomplish this. MR arthrography would be optimal. Direct arthrography has been the method of choice in assessing the glenoid labrum, surrounding ligaments, and capsule. CT arthrography is of value, particularly in the postoperative shoulder, and provides indirect imaging of internal structures by coating them with contrast material. Noncontrast CT would be very limited in assessing the labroligamentous complex, even in the presence of a joint effusion caused by poor soft tissue contrast. 5. A patient with early rheumatoid arthritis is being considered for placement on a DMARD. All of the imaging studies below could assess the level of disease activity and response to therapy except A. 18FDG scan. B. radiographs of the hands and wrists. C. gray-scale ultrasonography with power Doppler. D. MRI with gadolinium. E. parametric MR imaging of distribution volumes of contrast in the extravascular space. Answer: B  Radiographic findings in rheumatoid arthritis usually occur when there has already been irreversible joint damage. Ideally, therapy would be instituted on a radiographically negative patient. The remaining examinations can provide sensitive evaluation of disease activity before the development of either bone or cartilage erosion.

CHAPTER 259  Connective Tissue Structure and Function  

tissue disorders affecting the vasculature, bone, or cartilage, with neonatal, juvenile, or early adult mortality resulting. Connective tissues continually sense and adapt to their mechanical environment. The anabolic response of many connective tissues to optimal levels of mechanical stress is now recognized as a crucial, remediable determinant of health. This concept is embodied in Wolff ’s law of bone remodeling, which states that bone remodels in response to the mechanical loads imposed on it. The composition of specific connective tissues reflects adaptations in response to their mechanical environment or requirements for other specialized functions. For example, articular cartilage and other hyaline cartilages comprise proteoglycan aggregates that exert a swelling pressure and a network of collagen II fibrils that restrain them, contributing to compressive strength and shock absorption.4 The superficial zone of articular cartilage, in contrast, in enriched in collagenous fibrils that are arranged parallel to the surface to resist shear forces. A mucinous glycoprotein, lubricin, present on the joint surfaces, ensures the low coefficient of friction of synovial joints. In tendons, the major ECM component is collagen I, which has high tensile strength, but in bone, a composite of collagen I and calcium hydroxyapatite provides tensile and compressive strength. The contractile apparatus of skeletal muscle is connected by the dystrophin–glycoprotein complex to the muscle basement membrane and via hierarchical assemblies of interstitial matrix (endomysium, perimysium, and epimysium) to tendons and ultimately to bone, thus efficiently transmitting muscle contraction forces.5 In the intervertebral disc, the nucleus pulposus is rich in aggrecan, which exerts a swelling pressure that is constrained by the surrounding concentric lamellae of fibrillar collagen in the annulus fibrosus. This composite structure absorbs vertical loads on the spine while limiting deformation of the nucleus pulposus. The interfaces between different connective tissues show structured transitions, highlighted by fibrocartilage present at tendon insertion sites and by Sharpey fibers, which seamlessly integrate perimysial collagen of muscle fibers with collagen in bone. In knee menisci, the cells and ECM of the inner third are cartilaginous, with abundant proteoglycans. Whereas the cells of the outer third, which is vascular and connected to ligaments, are fibroblastic, with corresponding predominance of fibrillar collagens, the cells and ECM in the middle third have an intermediate fibrochondrocyte phenotype.

  MAJOR EXTRACELLULAR MATRIX COMPONENTS

Extracellular matrix comprises collagens; the glycosaminoglycan (GAG) hyaluronan (HA); proteoglycans; a variety of glycoproteins; phosphoproteins (especially in bones and teeth); and matricellular proteins such as thrombospondin, tenascin, and periostin, which regulate cellular functions via ECM but do not have a structural role. The distribution of ECM molecules and macromolecules is tightly regulated to achieve specific microenvironments that control proliferation, differentiation, polarity, and migration of cells and provide niches for postnatal stem cells. Collagens, the most abundant proteins of the body, comprise several distinct molecules, each containing at least one triple-helical domain composed of αchains having a repeating Gly-X-Y sequence. Twenty-eight different collagen types are formed from the products of more than 40 genes encoding collagen α chains. The presence of glycine, the smallest amino acid, at every third position, permits triple-helix formation; the amino acid proline, which is frequently modified to form hydroxyproline at the Y position, ensures stability of the triple helix. Fibrillar collagens (e.g., types I-III, V, XI) have long triple-helical (collagenous) regions, so they form rodlike structures. Whereas collagen I, the most widely distributed and abundant, is a heterotrimer comprising two α1(I) chains and an α2(I) chain, collagen II and III are homotrimers of α1(II) and α1(III) chains, respectively. Mutations of either collagen I chain cause the majority of cases of osteogenesis imperfect (Chapter 260) and infrequently can also cause rare subtypes of Ehlers-Danlos syndrome (EDS) (Chapter 260). The triple helices of nonfibrillar collagens are shorter and interspersed with interruptions of the Gly-X-Y sequence or noncollagenous domains that introduce flexible regions. Collagen XIII, XVII, XXIII, and XXV are transmembrane proteins. Fibrillar collagens are synthesized as procollagens having bulky terminal propeptides. The folding of the triple helix occurs intracellularly and is propagated from the C- to the N-terminus to form homo- or heterotrimeric triple helices. The propeptides are subsequently excised by specific proteinases, resulting in rodlike tropocollagen that can be assembled into tightly packed, quarter-staggered fibrils. Failure to remove the N-propeptide of collagen I impairs fibril assembly and leads to a specific type of EDS with severe skin fragility (type VIIc or dermatosparactic type). Whereas collagen I is the major component of bone, dermis, tendons, ligaments, and the sclera of the eye, collagen III is abundant in the skin, lung, and vasculature, explaining

1731

the association of genes encoding these collagens with osteogenesis imperfecta and vascular EDS (also known as type IV). Collagen II, together with minor amounts of collagen IX and collagen XI, predominates in cartilage, ocular vitreous and the nucleus pulposus of the intervertebral disc, and mutations of either of these collagen genes can cause the Stickler syndrome (triad of cartilage, eye, and hearing anomalies). Collagen fibrils can consist of more than one collagen type; for instance, collagen V is found in heterotypic fibrils with collagen I and is required for nucleation of fibrillogenesis to form large collagen I fibrils. Basement membranes are formed from collagen IV, with significant content of laminins, nidogens, and the heparin sulfate proteoglycan perlecan, which is also an important constituent of cartilage. Basement membranes have crucial roles in regulating molecular transport, such as in glomerular filtration, and in establishment and maintenance of epithelial polarity. Collagen VI is widely distributed as beaded microfibrils that form pericellular matrices in fibroblasts and, alongside collagen VII anchoring fibrils, connect basement membranes to interstitial ECM. Hyaluronan, a polymer with repeating disaccharide units, can achieve a molecular weight in the millions of Daltons. It is a key component of cartilage matrix, pericellular matrix of many cell types, and a mediator of inflammation. Proteoglycans are characterized by covalent attachment of glycosaminoglycans, such as chondroitin, heparan, and keratan sulfate, to a core protein. Aggrecan and versican are large chondroitin sulfate proteoglycans that form giant aggregates with hyaluronan in chondrocytes or nucleus pulposus cells and fibroblasts, respectively. On the other hand, some small leucine repeat-rich proteins (SLRPs) such as decorin, biglycan, and lumican are proteoglycans that may have only one or two glycosaminoglycan chains. This class of molecules, which also includes fibromodulin, interacts with collagen fibrils to cross-link them and regulate fibril diameter. They have been shown to bind transforming growth factor β (TGF-β) and, when released from connective tissue, to provide danger signals to the immune system. During connective tissue healing, fibrin and fibronectin provide a transitional ECM permissive for cell migration, differentiation, and other aspects of the repair process. Elastic fibers are formed by coacervation of a soluble precursor named tropoelastin. Elastic fiber assembly is guided by tissue microfibrils that are formed from three large glycoproteins named fibrillins.6 Fibrillin-1 is abundant in the aorta, ocular zonule, and perichondrium of bone and is mutated in Marfan syndrome (Chapter 260), resulting in aneurysms, ectopia lentis, and skeletal overgrowth. Fibrillin-2 mutations cause Beals syndrome, with skeletal overgrowth and limb contractures as major features but typically, not severe cardiac or eye problems. The elastic fiber-microfibril network additionally contains versican, microfibril-associated glycoproteins, latent TGF-β–binding proteins, and fibulins. Cutis laxa, which primarily affects the skin but can also involve vasculature and internal organs, can be caused by mutations affecting elastin and fibulin-5. Most ECM molecules undergo one or more posttranslational modifications. These include enzymatic formation of intra- and interchain disulfide bonds, phosphorylation, and several kinds of glycosylation, including addition of N- and O-linked sugars or glycosaminoglycans such as chondroitin, keratan, or heparan sulfates. Whereas N-linked sugars on some proteins are essential for protein folding and secretion, glycosaminoglycan (GAG) chains provide crucial biophysical properties (such as of aggrecan) and mediate intermolecular interactions. During its biosynthesis, collagen undergoes lysyl and prolyl hydroxylation. Some lysyl and hydroxylysl residues are modified extracellularly by lysyl oxidase and form stable cross-links between adjacent collagen molecules, which strengthens bone, tendons, and skin.7 Gene defects affecting collagen-modifying enzymes or subunits of the molecular complexes they operate in lead to various recessive forms of osteogenesis imperfecta or EDS. l-Ascorbic acid is a cofactor for lysyl hydroxylase and prolyl hydroxylase and stimulates procollagen synthesis. Its nutritional deficiency leads to scurvy by reducing collagen synthesis, triple-helical stability, and tropocollagen cross-linking. Lathyrism (a now rarely seen neurotoxic disease) is caused by excess ingestion of β-aminopropionitrile, which inhibits lysyl oxidase–mediated formation of lysine aldehydes, which are the precursors of the major collagen and elastin cross-links, and leads to connective tissue fragility. Copper is a required cofactor for lysyl oxidase, and its deficiency can also lead to lathyrism.

CELL-MATRIX INTERACTIONS IN CONNECTIVE TISSUE REGULATION

Cellular ECM receptors mediate cell-matrix interactions that ensure force transmission from cells to ECM8; the dynamic reciprocity between cells and

1732

CHAPTER 259  Connective Tissue Structure and Function  

ECM is crucial for environmental sensing and adaptive responses. Receptors provide feedback to the cells regarding the quality, content, or mechanical properties of the matrix (outside-in signaling) to generate appropriate responses, including cell proliferation or migration, or lead to altered ECM synthesis or degradation. Alternatively, cytoplasmic signals may alter cellmatrix interactions (inside-out signaling). Integrins are a large group of heterodimeric receptors with distinct binding preferences that comprise α and β subunits having short cytoplasmic domains. Integrins are crucial for fibronectin assembly, which in turn substantially influences assembly of collagen, fibrillins, and TGF-β activation. Intracellular signals activate integrins to promote high-affinity binding to ECM proteins, and binding of ECM proteins via specific integrin-binding motifs such as Arg-Gly-Asp initiates clustering of cell adhesion complexes and intracellular signaling that, among diverse effects, may elicit the production of inflammatory cytokines. Integrin αvβ6 transmits cellular traction to matrix-bound latent TGF-β, exposing the active growth factor. Because osteoclast attachment to and spreading on bone surfaces is dependent on αvβ3, it is being targeted for treatment of osteoporosis (Chapter 243) using chemical antagonists or blocking antibodies. Syndecans are transmembrane heparan sulfate proteoglycans that work as coreceptors alongside high-affinity ECM receptors such as integrins. Discoidin domain receptors are receptor tyrosine kinases activated by binding to native triple-helical collagen.9 DDR1 and DDR2 bind to fibrillar collagens I-III and collagen V. Collagen IV activates DDR1 but not DDR2, whereas collagen X activates DDR2. With their extended structure, extensive posttranslational modification, and exposure of numerous binding sites, ECM molecules such as collagens, fibrillins, and the classical cell-binding protein fibronectin bind to multiple receptor types. Among several hyaluronanbinding molecules on the cell surface, including hyaluronan synthases, CD44 is a major hyaluronan receptor with a role in assembling the pericellular matrix and in inflammation.

EXTRACELLULAR MATRIX NETWORKS AND CONNECTIVE TISSUE DISORDERS

Extracellular matrix molecules and macromolecules are assembled to form higher order (supramolecular) complexes and networks whose varied composition and geometry further diversify connective tissues. These are exemplified by parallel arrays of extensively cross-linked collagen fibrils conferring high tensile strength to tendons, ligaments, and bone, by the multidirectional or “basket-weave” arrangement of collagen in dermis, both permitting and constraining multiaxial mobility, and by the concentric elastic lamellae and crimped collagen in the aorta that regulate hemodynamics. In the eye, orthogonally oriented collagen fibrils in corneal stroma and the lattice-like structure of collagen VIII in the Descemet membrane allow transparency and are crucial for normal vision. Cartilage and nucleus pulposus ECM comprises large aggregates of hyaluronan and aggrecan, with hundreds of aggrecan molecules attached to each hyaluronan polymer via their N-terminal domain, leaving the C-terminal domain free for interactions with fibrillins, fibulins, and other ECM networks. Each aggrecan molecule has approximately 100 chondroitin sulfate chains and several keratan sulfate chains. Their high fixed-charge density creates an osmotic environment that favors water retention and restricts water flux in cartilage. Through the attachment of hyaluronan to cell-surface receptors, the aggrecan–hyaluronan network is retained by the chondrocyte as a conspicuous pericellular matrix (or glycocalyx). In connective tissue cells other than chondrocytes, pericellular matrices use versican or heparin sulfate proteoglycans instead of aggrecan. The pericellular matrix occupies the crucial interface between cells and their environment and influences cell behavior. It is a provisional ECM in microcosm because it is metabolically more active than further removed matrix, which is more stable. Some interstitial collagen and elastin can be stable for decades. Connective tissue disorders arise from inborn defects affecting cells or ECM, as well as cellular dysfunction caused by repetitive injury, inflammation, or metabolic or aging processes. Because ECM is such a crucial component of connective tissues, most inherited and acquired connective tissue disorders result from mutant ECM molecules, insufficient but essentially normal matrix (e.g., osteoporosis), or excess or inappropriately deposited matrix (e.g., adhesions, fibrosis, and scleroderma). Osteoarthritis (OA) involves all structures that form the joint, including the synovium and subchondral bone, and appears to result from a combination of genetic and nongenetic factors (Chapter 262). OA of the hip and hands has a stronger genetic component than OA of the knee. Because the function of cartilage is to absorb impact and distribute it to bone without aberrant loading of joint

structures, variations in genes that affect joint congruity and alignment or cartilage or meniscal integrity could predispose to OA. For instance, variations in genes encoding cartilage ECM components mutated in chondrodysplasias (Chapter 205), such as collagen II, IX, cartilage oligomeric protein, or matrilins, have been associated with early-onset OA, suggesting a genetic link and a spectrum extending from OA predisposition at the milder end to severe chondrodysplasia. Another genetic link is with the mechanisms that influence the formation of joints, correlating with the association between joint malalignment and OA. Variations in genes associated with the TGF-β superfamily pathway, including GDF5 (encoding a morphogen required for joint development), ASPN (encoding an ECM protein that binds TGFβ), and SMAD3 (encoding a cytoplasm-nucleus signaling intermediary), are associated with OA.10 Factors that affect load distribution across joints, such as ligament and meniscal tears or hip dysplasia, predispose to OA. Cartilage breakdown in arthritis is not caused by wear and tear but by altered cellmatrix interactions that lead to enzymatic digestion. Loss of aggrecan, followed by loss of SLRPs and other molecules from the surface of collagen fibrils, are initial changes that render collagen fibrils susceptible to destruction by collagenases. The loss of articular cartilage collagen II is thought to constitute an irreversible change in joint disease. Whatever the original insult, the response of chondrocytes or synovium can lead to a vicious cycle of joint destruction because the cells may respond by producing excess ECM and ECM-degrading proteases, with release of both intact and fragmented molecules that may further potentiate inflammation. Rodent immune arthritis models have revealed a role for the alternative complement pathway in arthritis, and cartilage proteins and their fragments released by proteolytic breakdown can have complex effects on the complement pathways that lead to both activation and suppression. Cell-matrix interactions are profoundly affected in muscular dystrophies (Chapter 421), revealing a continuum that is essential for force transmission from the cytoskeleton to interstitial matrix. Mutations affecting components of the dystrophin–glycoprotein complex including dystroglycan, a receptor that connects muscle cytoskeleton to laminin in the muscle basement membrane, muscle basement membrane (laminin and collagen IV), and collagen VI filaments in ECM, cause a variety of muscular dystrophies. Specific adhesion complexes such as hemidesmosomes in the epidermis mediate anchorage to basement membranes and underlying interstitial ECM. Autoantibodies against collagen XVII and VII, or laminin 5 mutations, affect major components of these complexes and lead to blistering skin diseases (Chapter 439).

PROTEASES AND CONNECTIVE TISSUE TURNOVER

Reflecting continuous adaptation to their environment, connective tissues have intrinsic mechanisms that ensure ECM renewal (i.e., through coupled synthesis and degradation). ECM molecules are substrates for several proteinase classes, chiefly matrix metalloproteinases (MMPs), astacin MMPs such as BMP1 and tolloids, cathepsins, and A disintegrin-like and metalloproteinase domain with thrombospondin type 1 repeats (ADAMTS).11 Although these proteases are typically considered to be catabolic, some are also essential for the maturation of precursor proteins, such as ADAMTS2, which excises the amino-propeptide of procollagen I, II, and III and is defective in EDS, dermatosparactic type. BMP1, which excises the C-propeptides of procollagen I, II, and III and cleaves lysyl oxidase and several other proteins, is deficient in a type of recessive osteogenesis imperfecta. Although most MMPs are secreted, a class of membrane-type MMPs is cell-surface bound. Most MMPs require proteolytic activation by other MMPs or serine proteinases such as plasmin and furin, and are inhibited by tissue inhibitors of MMPs (TIMPs) and α2-macroglobulin. Whereas MT1-MMP is crucial for collagen I proteolysis in bone, MMP-13 has been implicated as a major collagen II degrading enzyme in arthritis. Cell surface MMPs such as MT1-MMP and A disintegrin-like and MMPs (ADAM) are responsible for ectodomain shedding of ECM receptors, cell-surface cytokines and cytokine receptors, together regulating a variety of inflammatory and oncogenic situations. ADAMTS4 and ADAMTS5, also known as aggrecanase-1 and -2, respectively, are principal aggrecan-degrading proteases implicated in OA and, together with MMP-13, are potential drug targets in this disorder. However, MMPs, ADAMs, and ADAMTSs have structurally similar catalytic domains and zinc and calcium-dependent proteolytic mechanisms, which renders selective inhibition of any single proteinase from these classes challenging. Indeed, a major side effect of MMP inhibitors used in clinical trials for cancer was connective tissue stiffness and inflammation, presumably resulting from reduced physiological turnover of collagen and other

CHAPTER 260  Inherited Diseases of Connective Tissue  

ECM proteins. Among other proteinases, cathepsin K produced by osteoclasts is active at acidic pH, unlike MMPs, and therefore efficiently digests bone collagen in the acidic osteoclast–bone interface. Neutrophil elastase (a serine proteinase), MMP-9, and MMP-12 (metalloelastase) can degrade elastin. Proteolysis of ECM proteins can release bioactive fragments (matrikines) such as endostatin (from collagen XVIII) or endorepellin (from perlecan), which are antiangiogenic; some fibronectin fragments released in OA are proinflammatory. Hydroxyproline antibodies that recognize ECM-fragments with specific cleaved ends (neoepitope antibodies), as well as cross-linked collagen fragments released by proteolytic activity, are useful biomarkers of bone and cartilage turnover.

  AGING OF CONNECTIVE TISSUE

Some visible hallmarks of aging result not only from cellular senescence but also reduced ECM synthesis and increased catabolism, as well as greater connective tissue fragility. This leads, for example, to reduced bone mass or osteoporosis; thinning and loss of elastic properties of dermal ECM, which are visible as wrinkles and sagging skin; reduced volume of intervertebral discs; and increased capillary fragility. Loss of HA and GAGs reduces tissue hydration and of collagen and elastin reduces tensile strength and elasticity, respectively. Products of ECM proteolysis stimulate cells to release free radicals and cytokines that further accelerate ECM breakdown or lead to cell death. Extrinsic factors such as sunburn and ultraviolet irradiation induce cytokines that accelerate this process. In addition, aging collagen can be cross-linked by the Maillard reaction, especially when glucose levels are high, resulting in its modification by advanced glycation end products (AGE), which renders it inflexible, alters the rate of turnover, and affects binding to matrix receptors.12 AGE binding to a cellular receptor has been shown to lead to cellular dysfunction, oxidative stress, and inflammation. In summary, connective tissue provides the framework of the musculoskeletal system and a structural scaffold for internal organs while serving as a reservoir for growth factors, cytokines, and stem-like precursor cells. Inherited connective tissue disorders have the potential to disturb these functions in specific ways. Acquired disorders such as OA and fibrosis reflect perturbation of diverse physiological networks and pathways, with altered cell-matrix interactions at the center. GENERAL REFERENCES For the General References and other additional features, please visit Expert Consult at https://expertconsult.inkling.com.

260  INHERITED DISEASES OF CONNECTIVE TISSUE REED E. PYERITZ

  MUCOPOLYSACCHARIDOSES DEFINITION

Proteoglycans are ubiquitous components of the extracellular matrix (ECM) and the surfaces of cells, and they are among the largest and most complex of human molecules. Proteoglycans consist of a protein core to which are covalently bound glycosaminoglycans (GAGs; formerly called mucopolysaccharides) of several types: dermatan sulfate, heparan sulfate, keratan sulfate, and chondroitin sulfate. These four polymeric molecules are cleaved from their protein core in lysosomes; then they, plus hyaluronan (a GAG lacking a protein core), are catabolized further in lysosomes in a stepwise fashion by more than a dozen enzymes. Genetic defects in any one of these enzymes lead to the accumulation of GAG metabolites in lysosomes, with profound disruption of cellular physiology. The phenotypes resulting from deficiencies of these catabolic enzymes are termed mucopolysaccharidoses (MPSs) and are classified into seven types (Table 260-1). Several additional storage disorders, termed mucolipidoses (MLs), are caused by a genetic

1733

defect in posttranslational modification of lysosomal enzymes and share features with the MPSs.

EPIDEMIOLOGY

All MPS disorders are rare, each with an incidence of one or fewer cases per 100,000 births, and are without ethnic predilection.

PATHOBIOLOGY

With the exception of MPS II (Hunter syndrome), which is X-linked, each of these disorders is autosomal recessive. All MPSs are caused by deficiency of a single lysosomal enzyme responsible for a specific step in GAG metabolism. Catabolism of GAG proceeds normally until the step requiring the defective enzyme, when further normal metabolism halts. Although a minor degree of nonspecific breakdown occurs, resulting in urinary excretion of cleaved GAG that can be useful diagnostically, the accumulation of GAG within lysosomes of cells of mesenchymal origin; endothelium; and, in most cases, neurons causes widespread, progressive cellular dysfunction and clinical effects. Lysosomal enzymes are targeted to lysosomes by posttranslational addition of mannose 6-phosphate. Deficiency of the phosphotransferase that catalyzes the first step in this reaction results in an inability to catabolize any GAG molecules. The catabolic enzymes, which normally would be transported into lysosomes, instead are secreted from the cell and are found in unusually high concentrations in plasma, providing one diagnostic test for MLs.1

Pathology

All pathologic manifestations of MPS and ML disorders worsen with age, and some are present from early developmental stages. Gross anatomic hallmarks are hepatosplenomegaly, marked skeletal alterations (termed dysostosis multiplex)2 that result in short stature and thoracic cage deformity, thickening and narrowing of airways and arteries, and coarsening of facial features. Although mental retardation is a prominent feature of some of these conditions, the brain may show only ventriculomegaly secondary to communicating hydrocephalus. On microscopy, mesenchymal cells show a cytoplasm full of apparently empty vacuoles; these are lysosomes from which GAG has been removed by fixation. Cells cultured from patients show greatly enlarged lysosomes filled with granular material. In the severe form of ML, dense inclusions are present, which gave rise to the common name, I-cell disease.

CLINICAL MANIFESTATIONS

Each of the disorders in Table 260-1 shows a wide spectrum of clinical severity. This wide spectrum has led to a classification that gives the impression of separate disorders within some of the MPS and ML types, but these represent the apparent ends of the continuum. Some of the disorders result in death by adolescence (Hurler syndrome, severe Hunter syndrome, ML II), but others are commonly compatible with survival to adulthood. The latter group of disorders is emphasized here. The milder end of the MPS I spectrum, Scheie syndrome, may not be diagnosed until adulthood; patients present with stiffened joints, corneal clouding and glaucoma, carpal tunnel syndrome, and aortic valvular disease. Stature and intelligence are not affected. The main health risks are valvular involvement, thickening of meninges that can produce a myelopathy, and thickening of the upper airways that can produce obstructive symptoms and sleep apnea. The milder form of MPS II, Hunter syndrome, is distinctive because it is X-linked (affecting males almost exclusively), and the cornea shows little overt clouding. Cervical myelopathy, obstructive airway disease, and cor pulmonale are important concerns. A combined conductive and neurosensory hearing loss is common. Neither MPS IV (Morquio syndrome) nor MPS VI (Maroteaux-Lamy syndrome) affects intelligence. Both syndromes often are associated with severe skeletal changes, which are distinct radiographically but produce similar problems of kyphoscoliosis, pectus carinatum, restrictive lung disease, severe short stature, and joint degeneration. Cervical myelopathy resulting from a thickened dura is common to both disorders and is accentuated by odontoid hypoplasia in MPS IV. Thickening of the aortic and mitral valves may produce severe dysfunction necessitating their replacement. General anesthesia is especially hazardous because of the narrow upper and middle airways and cervical instability. Patients with ML III (pseudo-Hurler polydystrophy) resemble patients with MPS VI but often have mild to moderate mental retardation. Aortic regurgitation is common.

CHAPTER 259  Connective Tissue Structure and Function  

GENERAL REFERENCES 1. Frey H, Schroeder N, Manon-Jensen T, et al. Biological interplay between proteoglycans and their innate immune receptors in inflammation. FEBS J. 2013;280:2165-2179. 2. Fosang AJ, Beier F. Emerging frontiers in cartilage and chondrocyte biology. Best Pract Res Clin Rheumatol. 2011;25:751-766. 3. Nandadasa S, Foulcer S, Apte SS. The multiple complex roles of versican and its proteolytic turnover by ADAMTS proteases during embryogenesis. Matrix Biol. 2014;35:34-41. 4. Heinegard D, Saxne T. The role of the cartilage matrix in osteoarthritis. Nat Rev Rheumatol. 2011;7:50-56. 5. Carmignac V, Durbeej M. Cell-matrix interactions in muscle disease. J Pathol. 2012;226: 200-218. 6. Baldwin AK, Simpson A, Steer R, et al. Elastic fibres in health and disease. Expert Rev Mol Med. 2013;15:e8.

1733.e1

7. Byers PH, Pyott SM. Recessively inherited forms of osteogenesis imperfecta. Annu Rev Genet. 2012;46:475-497. 8. Ross TD, Coon BG, Yun S, et al. Integrins in mechanotransduction. Curr Opin Cell Biol. 2013; 25:613-618. 9. Fu HL, Valiathan RR, Arkwright R, et al. Discoidin domain receptors: unique receptor tyrosine kinases in collagen-mediated signaling. The J Biol Chem. 2013;288:7430-7437. 10. Sandell LJ. Etiology of osteoarthritis: genetics and synovial joint development. Nat Rev Rheumatol. 2012;8:77-89. 11. Gargiulo S, Gamba P, Poli G, et al. Metalloproteinases and metalloproteinase inhibitors in agerelated diseases. Curr Pharm Des. 2014;20:2993-3018. 12. Semba RD, Nicklett EJ, Ferrucci L. Does accumulation of advanced glycation end products contribute to the aging phenotype? J Gerontol A Biol Sci Med Sci. 2010;65:963-975.

1734

CHAPTER 260  Inherited Diseases of Connective Tissue  

TABLE 260-1  MUCOPOLYSACCHARIDOSES AND MUCOLIPIDOSES TYPE

EPONYM OR COMMON NAME

INHERITANCE

OMIM*

MPS IH

Hurler syndrome

DM and short stature; MR; corneal clouding; HS; heart disease; death in childhood

AR

252800

α-l-iduronidase

MPS IS

Scheie syndrome

Coarse facies; stiff joints, corneal clouding; aortic valve disease; normal intelligence and lifespan

AR

252800

α-l-iduronidase

MPS II

Hunter syndrome

Severe form: coarse facies, DM and short stature, HS; MR; no corneal clouding; death by late adolescence Mild form: coarse facies, short stature; normal intelligence; survival to adulthood

XL

309900

Iduronate sulfatase

CLINICAL FEATURES

ENZYMATIC DEFECT

MPS IIIA

Sanfilippo A

Severe MR and hyperactivity; mild somatic changes

AR

252900

Heparan N-sulfatase

MPS IIIB

Sanfilippo B

Same as MPS IIIA

AR

252920

α-N-acetylglucosaminidase

MPS IIIC

Sanfilippo C

Same as MPS IIIA

AR

252930

Acetyl-coenzyme A: α-glucosaminide acetyltransferase

MPS IIID

Sanfilippo D

Same as MPS IIIA

AR

252940

N-acetylglucosamine 6-sulfatase

MPS IVA

Morquio A

Short stature and distinct skeletal dysplasia with odontoid hypoplasia and myelopathy; corneal clouding; normal intelligence; valvular heart disease

AR

253000

Galactose 6-sulfatase

MPS IVB

Morquio B

Same as MPS IVA

AR

253010

β-Galactosidase

MPS VI

Maroteaux-Lamy

DM and short stature; corneal clouding; normal intelligence; aortic stenosis; leukocyte inclusions; hydrocephalus in severe form

AR

253200

N-acetylgalactosamine

MPS VII

Sly syndrome

DM; HS; widely variable, including MR

AR

253220

β-Glucuronidase

MPS IX



Short stature; periarticular soft tissue masses

AR

601492

Hyaluronidase

ML II

I-cell disease

Similar to but more severe than MPS IH but with cellular inclusions; no mucopolysacchariduria

AR

252500

UDP-N-acetylglucosamine: lysosomal enzyme N-acetylglucosaminyl-1- phosphotransferase

ML III

Pseudo-Hurler polydystrophy

Short stature and mild DM; stiff joints, mild MR; survival to adulthood

AR

252500

Same as ML II arthropathy, coarse facies; variable but milder

*Entries in Online Mendelian Inheritance in Man, OMIM. McKusick-Nathans Institute of Genetic Medicine. Baltimore: Johns Hopkins University. http://omim.org. AR = autosomal recessive; DM = dysostosis multiplex; HS = hepatosplenomegaly; MR= mental retardation; XL = X-linked.

DIAGNOSIS

Differential Diagnosis

Diagnosis of these conditions is difficult in young children, before most of the clinical features have progressed, but should be considered in any person with hepatosplenomegaly and coarsening of the facial features. Evaluation requires a pedigree analysis, ophthalmologic examination, skeletal radiographic survey, echocardiography, and analysis of the urine for excretion of GAGs. Often the specific MPS is evident from radiographs, the presence or absence of corneal clouding, and the pattern of mucopolysacchariduria. Enzymatic analysis of leukocytes confirms the diagnosis. Patients with MLs do not show mucopolysacchariduria but have marked elevation of all the GAG catabolic lysosomal enzymes in plasma.

TREATMENT  Ventriculoperitoneal shunting is necessary if intracranial pressure is elevated. Close attention to hearing and visual problems is essential throughout life. Many adults with MPS or ML require surgery for carpal tunnel syndrome. Cardiovascular surgery for valvular or coronary disease may be necessary. All use of anesthesia is high risk because of the narrow airways and, in the case of MPS IV, atlantoaxial instability. For patients who remain ambulatory, selective joint replacement can be beneficial. Because of the morbidity associated with thoracic cage deformity, consideration should be given to stabilizing the spinal deformity before it becomes severe. Replacement of the deficient enzyme via intravenous infusion is being studied for most of the MPS disorders. Laronidase (Aldurazyme) has been approved in the United States for treatment of MPS I. An infusion every 2 weeks for 1 year in adolescent and adult patients resulted in substantial reduction in hepatosplenomegaly and modest improvement in pulmonary function, sleep apnea, and joint mobility. Whether early institution of therapy in young children modulates mental retardation in the Hurler variant of MPS I is uncertain. Galsulfase (Naglazyme) has been approved for the treatment of MPS VI, in which somatic rather than neurologic problems predominate. Bone marrow transplantation has been attempted in many of the MPS disorders, with mixed success. The earlier transplantation occurs, the better the outcome

in terms of somatic problems, but prevention of mental retardation has not occurred. Current recommendations based on consensus in Europe calls for hematopoietic stem cell transplantation for patients with Hurler syndrome before the age of 2.5 years. Enzyme replacement should be started in all patients when diagnosed.3

  MARFAN SYNDROME DEFINITION

Marfan syndrome is an autosomal dominant, pleiotropic disorder caused by defects in the principal component of the extracellular microfibril, the large glycoprotein fibrillin-1. The disease manifestations occur in multiple systems, especially the eye, skeleton, heart, aorta, lung, and integument. Notable features include dislocation of the ocular lens, tall stature with particularly long limbs and digits, deformity of the thoracic cage from pectus carinatum or excavatum with abnormal curvature of the spine, mitral and tricuspid valve prolapse, dilation of the sinuses of Valsalva and predisposition to aortic dissection, spontaneous pneumothorax, abnormal skin stretch marks, hernias, and dural ectasia. If untreated, patients often die before 30 or 40 years of age from aortic dissection or congestive heart failure.

EPIDEMIOLOGY

Marfan syndrome is a common Mendelian disorder, with an estimated incidence of about one per 5000 births. Marfan syndrome is found throughout the world, without ethnic or geographic predilection.

PATHOBIOLOGY

Pathogenesis

Mutations in FBN1, which maps to human chromosome 15q21.1 and encodes fibrillin-1, cause Marfan syndrome and related connective tissue disorders. More than 1000 distinct mutations have been found, and few occur in more than one family. Patients are heterozygous for mutations in FBN1, leading to autosomal dominant inheritance. Extracellular microfibrils are

CHAPTER 260  Inherited Diseases of Connective Tissue  

polymers of many fibrillin-1 molecules and are ubiquitous in the ECM of most tissues. Latent transforming growth factor β (TGF-β) binding protein, which keeps the cytokine inactive, bears striking homology to regions of fibrillin. Abnormalities of either the quality or the quantity of microfibrils disrupt normal signaling by TGF-β, especially during embryonic development and postnatal growth. Studies in mice engineered to harbor human mutations in FBN1 showed that excessive TGF-β signaling causes abnormal lung septation (the precursor to pneumothorax), mitral valve prolapse, muscular hypoplasia, and aortic dilatation. This fundamental shift in understanding of the pathogenesis of Marfan syndrome has suggested novel therapies, such as with small molecules that affect the activity of TGF-β or its downstream signaling. The features of Marfan syndrome are highly variable, even among relatives who share the same mutation in FBN1. This variability persists after accounting for the effects of age. Men tend to be affected more severely, for unclear reasons.

Pathology

The features of Marfan syndrome are age dependent. Some severely affected infants have flagrant features and often die of mitral regurgitation and heart failure despite aggressive management. At the other end of the clinical spectrum, Marfan syndrome merges with several related disorders, and patients may not come to medical attention, let alone receive a definitive diagnosis, until adulthood. None of the gross or microscopic pathologic changes is specific for Marfan syndrome. The medial degeneration of the aortic wall, characterized by disarray and fragmentation of the elastic fibers and increased proteoglycan (often inappropriately termed cystic medial necrosis) also can be seen in other disorders and in older people with hypertension. Aortic dissection (Chapter 78) usually begins just superior to the aortic valve (type A) and often progresses to the bifurcation. Death usually results from retrograde dissection and hemopericardium. About 10% of dissections begin in the descending thoracic aorta (type B).

CLINICAL MANIFESTATIONS

The lens tends to be displaced superiorly, and usually the zonules remain intact. The retina is at increased risk of detachment, especially in patients who are highly myopic. Tubular bones overgrow, accounting for the disproportionate tall stature (dolichostenomelia), long digits (arachnodactyly), and sternal deformity. Ligaments may be lax, causing scoliosis and joint hypermobility. Alternatively, congenital contractures are common, especially of the elbows. The palate typically is highly arched, and the dentition can be crowded and maloccluded. Mitral valve prolapse occurs in about 80% of cases, and the valve leaflets become progressively thickened (myxomatous on histopathology) (Chapter 75). The mitral annulus may dilate and calcify. Aortic root dilation begins in the sinuses of Valsalva and progresses with age, albeit at highly variable rates (Chapter 78).4 Most males with Marfan syndrome have an aortic root dimension above the upper limit of normal for their body surface area by adolescence. Some females show a slower progression and may have a root diameter near the upper limit of normal well into adulthood. The dilation usually does not involve the distal ascending aorta. Spontaneous pneumothorax, resulting from rupture of apical blebs, occurs in about 5% of patients. Stretch marks (striae atrophicae) occur over areas of flexural stress, such as the shoulders, breasts, and lower back. The neural canal in the lumbosacral region is enlarged in most people with Marfan syndrome; this may be visible on plain radiographs, especially if the neuroforamina are widened. Imaging by computed tomography or magnetic resonance imaging is diagnostic and should be used in patients with back pain and radicular symptoms. Dural ectasia progresses with age; large anterior meningoceles in the pelvis are a severe manifestation.5 Simple cysts in the liver and kidneys are common, increase with age, and seldom cause clinical problems. Sleep apnea is of increased frequency in adults.6

DIAGNOSIS

Differential Diagnosis

The conditions that overlap clinically and genetically with Marfan syndrome include familial aortic aneurysm, familial ectopia lentis, mitral valve prolapse, mild aortic dilation, striae, skeleton (MASS) phenotype (which includes many families with mitral valve prolapse syndrome), and Loeys-Dietz syndrome. Most of these conditions are diagnosed clinically, so differentiating among them is arbitrary. A careful family history is essential to this process. Molecular genetic testing has a limited role. However, if the mutation in

1735

FBN1 is known in a family, analysis of DNA can be used effectively for presymptomatic or prenatal diagnosis. Loeys-Dietz syndrome, which is associated with generalized arterial tortuosity and susceptibility to dissection, is caused by mutation in either of two receptors for TGF-β, TGFBR1 and TGFBR2, and molecular analysis is clinically available. A question of Marfan syndrome arises most commonly in tall, lanky adolescents who have several minor skeletal features, nearsightedness, and athletic desires. A detailed ophthalmologic examination with full pupillary dilation and a transthoracic echocardiogram are essential components in the evaluation. If these test results are negative and no one in the family has a history of Marfan syndrome or aortic dissection, the patient probably can be reassured.7

TREATMENT  Life expectancy for those with Marfan syndrome has improved markedly, to the point that many patients can expect survival to advanced years. All patients should be seen at least annually by a physician who manages the overall care. Most patients require annual ophthalmologic and cardiologic consultation and orthopedic consultation as required by specific problems. Lens subluxation often requires surgical correction.8 A number of studies, but only one randomized clinical trial, support the prophylactic use of β-adrenergic blockade from an early age to slow the rate of aortic root dilation and protect against aortic dissection. Based on studies of the Marfan mouse, therapies that interfere with excess signaling through pathways mediated by TGF-β are being studied in human clinical trials. One large European trial suggested a benefit of the angiotensin receptor blocker losartan on aortic root dilatation rate, A1  but a large international trial found no benefit of losartan compared with atenolol. A2  Prophylactic surgical repair of the aortic root has had the greatest beneficial impact. The composite graft, involving a prosthetic valve in a Dacron tube and implantation of the coronary ostia into the graft, was the first approach to produce markedly improved survival in these patients. More recently, replacement of the aneurysm and preservation of the native aortic valve have shown promise and should be considered first.9,10 For adults, aortic root surgery should be strongly considered when the maximal aortic diameter reaches 45 mm, and a family history of aortic dissection should prompt earlier repair (Chapter 78).

  EHLERS-DANLOS SYNDROMES DEFINITION

The Ehlers-Danlos syndromes (EDSs) are clinically variable and genetically heterogeneous. Diagnoses still are based largely on the bedside examination. The unifying themes among these disorders are fragility of tissues, joint hypermobility, and skin hyperextensibility.11

EPIDEMIOLOGY

No accurate data exist, but an incidence of about one in 5000 births is a reasonable estimate of how many individuals qualify for one of the EDS diagnoses. Each type represents something of a clinical spectrum, with the mild end merging with what might be considered normal variation. Just as the diagnostic criteria are arbitrary, so would be any determination of prevalence based on phenotypic criteria. The extent to which normal variation in joint hypermobility, skin elasticity, and tissue fragility represents genetic variation at loci that encode collagen or other ECM genes requires considerable research.

PATHOBIOLOGY

Pathogenesis

Defects in collagen and other proteins in the ECM of various tissues underlie all forms of EDS that have been elucidated so far. The specific mutations occur in a variety of genes, with the effect of altering the structure, synthesis, posttranslational modifications, or stability of the collagens involved. The known molecular defects are listed in Table 260-2.

Pathology

Few findings in the routine pathologic evaluation distinguish among the various types of EDS or even distinguish individual types from normal. Thickness of the dermis is decreased in some forms, especially the vascular type, and the walls of arteries are reduced in thickness in this type. By electron microscopy, the classic, hypermobile, and kyphoscoliotic types have abnormal collagen fibers, especially when viewed in cross section (variable and often increased fiber diameter with an irregular outline). In the vascular type,

1736

CHAPTER 260  Inherited Diseases of Connective Tissue  

TABLE 260-2  EHLERS-DANLOS SYNDROMES TYPE

FORMER NAME

CLINICAL FEATURES*

INHERITANCE

OMIM†

MOLECULAR DEFECT

Classic

EDS I and II

Joint hypermobility; skin hyperextensibility; atrophic scars; smooth, velvety skin; subcutaneous spheroids

AD

130000 130010

Structure of type V collagen caused by mutations in COL5A1 or COL5A2

Hypermobility

EDS III

Joint hypermobility; some skin hyperextensibility, with or without a smooth, velvety texture

AD AR

130020 225320

? Tenascin-X (TNX)

Vascular

EDS IV

Thin skin; easy bruising; pinched nose; acrogeria; rupture of large- and medium-caliber arteries, uterus, and large bowel

AD

130050 (225350) (225360)

Deficient type III collagen (COL3A1)

Kyphoscoliotic

EDS VI

Joint hypermobility; congenital, progressive rupture; scoliosis; scleral fragility with globe rupture; tissue fragility, aortic dilation, MVP

AR

225400

Deficiency of lysyl hydroxylase

Arthrochalasis

EDS VII A

Joint hypermobility, severe, with subluxations, congenital hip dislocation; and skin hyperextensibility; tissue fragility

AD

130060

No cleavage of amino terminus of type I procollagen caused by mutations in COL1A1 or COL1A2

Dermatosparaxis

EDS VII C

Severe skin fragility; decreased skin elasticity, easy bruising; hernias; premature rupture of fetal membranes

AR

225410

No cleavage of amino terminus of type I procollagen caused by deficiency of peptidase

Unclassified types

EDS V EDS VIII EDS X EDS XI EDS IX EDS, progeroid form

Classic features Classic features and periodontal disease Mild classic features, MVP Joint instability Classic features; occipital horns Classic features and premature aging

XL AD ? AD XL AR

305200 130080 225310 147900 309400 130700

? ? ? ? Allelic to Menkes syndrome Deficiency of galactosyltransferase I

*Listed in order of diagnostic importance. † Entries in Online Mendelian Inheritance in Man, OMIM. McKusick-Nathans Institute of Genetic Medicine. Baltimore: Johns Hopkins University. http://omim.org. AD = autosomal dominant; AR = autosomal recessive; EDS = Ehlers-Danlos syndrome; MVP = mitral valve prolapse; XL = X-linked.

some patients have dilated endoplasmic reticulum consistent with aberrant secretion of type III collagen molecules.

CLINICAL MANIFESTATIONS

The major and minor features of each EDS are detailed in Table 260-2. Infants with classic EDS often are born prematurely by 4 to 8 weeks because of rupture of fetal membranes. Diagnosis of the vascular and kyphoscoliotic types is important because of their cardiovascular features. The vascular type, previously termed EDS IV, is characterized by spontaneous rupture of large arteries and hollow organs, especially the colon and uterus, and pneumothorax. Because these events carry considerable morbidity, life expectancy is reduced, on average, by more than half. During pregnancy, women with this form of EDS are especially vulnerable to rupture of major arteries and the uterus. In the kyphoscoliotic type, aortic root dilation and aortic regurgitation can develop. Patients with most forms of EDS are prone to develop mitral valve prolapse, and progression to mitral regurgitation (Chapter 75) occurs more often than in the common form of mitral valve prolapse.

DIAGNOSIS

Differential Diagnosis

By carefully adherence to the clinical features shown in Table 260-2 and judicious use of laboratory tests, the various defined types of EDS can be differentiated. Many specific non-EDS syndromes need to be excluded. The kyphoscoliotic type of EDS in infants shares some features with severe Marfan syndrome. Patients with Larsen syndrome may resemble patients with the arthrochalasis type of EDS. The skin redundancy and loss of elasticity of the dermatosparaxis type of EDS is reminiscent of autosomal dominant cutis laxa, which is not associated with easy bruising or tissue fragility. The most difficult decision is whether any diagnosis of EDS is warranted. Patients who have only joint hypermobility without skin changes should not be labeled with EDS; a diagnosis of familial joint hypermobility might be more appropriate. Familial joint instability involves a predisposition to dislocations of major joints that is rare in most types of EDS except for arthrochalasis.

TREATMENT  Management of most skin and joint problems should be conservative and preventive. Sutures need to be placed with careful attention to approximating the margins and avoiding tension; removable sutures should be left in place

for twice the usual time. Most instances of joint hypermobility and pain in EDS do not require surgical treatment. Benefit often is derived from physical therapy designed to strengthen the muscles that provide support for the loose ligaments. All patients should receive genetic counseling about the mode of inheritance and their risk of having children affected with EDS. The possibility of prenatal diagnosis exists for all of the EDS types with defined molecular or biochemical defects. The vascular type of EDS requires particular surgical care; the ruptured arteries are difficult to repair because of the pronounced vascular fragility. Experienced vascular surgeons are having some success with prophylactic repair of vessels deemed to be at risk of dissection or rupture.12 One clinical trial suggested improved outcomes with prophylactic β-adrenergic blockade. A3  Rupture of the bowel is a surgical emergency. Because the risk of uterine and vascular rupture is especially high during pregnancy in women with the vascular form, affected women should be advised that there is a substantial risk of death related to pregnancy and delivery. Patients should be advised to avoid contact sports and to treat blood pressure elevations aggressively. Arteriography and arterial lines should be avoided if possible. Biochemical and genetic screening holds the potential for reassuring relatives at risk that they do not have a defect in type III collagen. The kyphoscoliotic type of EDS may improve with large doses of vitamin C (1-4 g/day) because ascorbate is a cofactor for the enzyme that is deficient. No other metabolic or genetic therapy is effective in other forms of EDS.

  OSTEOGENESIS IMPERFECTA SYNDROMES DEFINITION

The heterogeneous group of disorders called osteogenesis imperfecta (OI) includes, at one end of the severity spectrum, a type that is lethal prenatally or in the neonatal period and, at the other, such mild features that distinguishing affected individuals from the general population is difficult. The unifying feature is hereditary osteopenia (insufficient bone), with primary defects in the protein matrix in bone and other tissues. The clinical syndromes all involve osteoporosis with liability to fracture (Chapter 243).

EPIDEMIOLOGY

No careful epidemiologic study has been performed, and the milder forms of type I OI merge with the phenotypes of familial osteoporosis, fracture

CHAPTER 260  Inherited Diseases of Connective Tissue  

1737

TABLE 260-3  OSTEOGENESIS IMPERFECTA TYPE

INHERITANCE

OMIM*

I

Fractures variable in number; little deformity; stature normal or nearly so; blue sclerae; hearing loss common but not always present; DI uncommon

CLINICAL FEATURES

AD

166200

Typically, one nonfunctional COL1A1 allele

BASIC DEFECTS

II

Lethal in utero or shortly after birth; many fractures at birth typically involving ribs (may appear “beaded”) and other long bones; little calvaria; pulmonary hypertension

AD

166210

AR

259400

COL1A1 or COL1A2: substitution of glycyl residues; occasionally deletions of a portion of the triple-helical domain Deletion in COL1A2 plus a nonfunctional allele

III

Fractures common, but long bones progressively deform starting in utero; stature markedly reduced; sclerae often blue but become lighter with age; DI and hearing loss common

AD AR (rare)

259420 259440

One single amino acid substitution Two mutations in COL1A1 and/or COL1A2 (rarely)

IV

Fractures common; stature usually reduced; bone deformity common but rarely severe; scleral hue normal to grayish; hearing loss variable; DI common

AD

166220 166240

Point mutations in COL1A1 or COL1A2 Exon skipping mutations in COL1A2

V

Similar to type IV without DI or blue sclerae; fractures develop hyperplastic callus; calcification of the interosseous membrane between the radius and ulna

AD

610967

?

VI

Similar to type IV without DI, blue sclerae or Wormian bones; excess osteoid present in bone

?

610968

?

VII

Similar to types II or III with fractures at birth, blue sclerae, no DI; presence of rhizomelic limb shortening and coxa vara

AR

610682

Mutations in CRTAP

VIII

Similar to types II or III with fractures at birth

AR

610915

Mutations in LEPRE1

IX

Similar to types II or III with fractures at birth

AR

259440

Mutations in PPIB

*Entries in Online Mendelian Inheritance in Man, OMIM. McKusick-Nathans Institute of Genetic Medicine. Baltimore: Johns Hopkins University. http://omim.org. AD = autosomal dominant; AR = autosomal recessive; DI = dentinogenesis imperfecta.

susceptibility, and joint hypermobility found in the general population. A crude estimate of the overall prevalence of OI is one to two per 20,000 births. The neonatal lethal form (type II), which is almost always caused by a new mutation in a parental gamete, has an incidence of about one in 50,000 births.

PATHOBIOLOGY

Pathogenesis

Most patients in whom mutations have been found usually have defects in the two genes that encode the procollagen chains of type I collagen, COL1A1 and COL1A2. Type I collagen is composed of two α1(I) and one α2(I) procollagen chains; the mature fiber requires considerable posttranslational modification, which occurs appropriately only if the three procollagen chains have intertwined to form a triple helix that is perfect and completed at the right speed. A mutation that affects formation of the triple helix, such as substitution of one of the mandatory glycine residues that occurs at every third position, also has adverse effects on the modifications that render the molecule capable of forming effective mature fibers. As a result, a single nucleotide change resulting in a missense mutation can have profound effects on the ECM and produce a severe condition.13 Alternatively, and at first glance paradoxically, a mutation that eliminates an entire allele, or at least production of any product capable of intertwining with normal procollagen chains, has a much milder effect on the ECM and on the severity of OI. Examples of the most common classes of mutations are shown in Table 260-3. Hundreds of mutations have been described. Patients with mutations in COL1A1 or COL1A2 are heterozygous, and thus the most common forms of OI are inherited as autosomal dominant traits. Several autosomal recessive forms of OI occur because of mutations in genes that encode enzymes that process type I collagen into mature fibrils.14

Pathology

Other than the gross pathology associated with the clinical manifestations, the most characteristic pathology is a primary reduction in bone matrix with secondary undermineralization.

CLINICAL MANIFESTATIONS

The major phenotypic features of OI are shown in Table 260-3. Among the most common forms, the most severe type is type II, followed in decreasing order by types III, IV, and I. In type II, infants either are stillborn or die soon after birth of pulmonary failure secondary to the small thorax, which usually is compromised further by myriad rib fractures. A few infants have

survived for at least a few years but require enormous attention to their medical needs. Type III OI may be confused with type II at birth, but survival alone helps make the distinction. Bony deformity is pronounced and not necessarily caused by fractures. Mobility is impaired, and most patients require a wheelchair at an early age. Stature may be severely compromised. Because of progressive vertebral column deformity and rib fractures, restrictive lung disease is a common problem as patients age; many die of pulmonary complications. Basilar impression causing compression of the brain stem and the craniocervical junction can produce central sleep apnea, headache, and upper motor neuron signs. Patients with type IV OI generally have reduced stature, some bony deformity, and abnormal teeth that are opalescent and wear easily (dentinogenesis imperfecta). As in type I OI, the tendency to fracture is highest in childhood and lessens with adolescence. A distinguishing characteristic of type IV OI is a normal scleral hue. Type I OI is probably the most common form and is associated with a bluish or blue-gray scleral hue. People with type I OI who also have dentinogenesis imperfecta tend to have more severe skeletal problems. The risk of fracture diminishes during adulthood but reemerges as a major concern for women after menopause. Hearing impairment in all forms of OI is common and age related, being rare before adolescence. The deficits are of either a mixed or a predominantly conductive form. The recessive forms of OI (types VI-IX) range in severity from type IV to type II and may have distinctive radiologic or histopathologic findings.

DIAGNOSIS

Differential Diagnosis

The range of diagnostic possibilities in a person with multiple fractures largely depends on age. In infancy, the genetic conditions hypophosphatasia, severe osteochondrodysplasias (e.g., achondrogenesis and forms of spondyloepiphyseal dysplasia), and Menkes syndrome need to be excluded when a diagnosis of type II or type III OI is considered. The radiographic features eventually become entirely diagnostic, but often the neonatologist has to arrive at a definitive answer in short order. Analysis of serum alkaline phosphatase and copper can be helpful. In childhood, the most common situation leading to consideration of a mild form of OI is child abuse. In this situation, the pattern of fracture is usually distinct, and bone mineralization should be normal if the child is the object of nonaccidental or repeated accidental trauma. Abnormal scleral hue, dentinogenesis imperfecta, and wormian bones (microfractures along the cranial sutures) all support the diagnosis of

1738

CHAPTER 260  Inherited Diseases of Connective Tissue  

OI. The legal and child-protective systems often request exclusion of OI by analysis of collagen production from cultured skin fibroblasts or analysis of DNA for a mutation. In older children, the disorder idiopathic juvenile osteoporosis should be considered in any patient seen initially with repeated fractures. Many osteochondrodysplasias are associated with short stature, skeletal deformity, and a tendency to fracture. Pyknodysostosis and osteopetrosis are associated with sclerotic bones rather than osteoporotic ones. In adulthood, early-onset osteoporosis may be confused with OI (Chapter 243). Mutations in type I collagen also cause familial osteoporosis, and the skeletal phenotypes merge; patients with true OI may have scleral, hearing, or dental abnormalities and a positive family history. Analysis of the specific enzymes defective in the recessive forms of OI is useful for establishing the diagnosis and enabling reproductive counseling and prenatal diagnosis if desired.

TREATMENT  Management of the skeletal complications largely depends on orthopedic, physical, and occupational therapy approaches. Risedronate (2.5 or 5 mg daily) increases bone mineral density and reduces both first and recurrent fractures in children with OI. A4  The long-term goals are for the patient to maintain function and independence as an individual. These goals can be advanced in some patients by judicious use of intramedullary rods in the long bones of the legs; if mobility and especially ambulation can be maintained, the demineralization associated with inactivity can be avoided. Unaffected parents of a child with OI and all affected individuals should have genetic counseling. For the parents of a child with type II OI, the possibility of germinal mosaicism (which has been well documented in this condition) should not be overlooked. If one parent has a “new” mutation in one of the type I procollagen genes and multiple gonadal cells carry this mutation,  the risk of recurrence in future children is not negligible. If the mutation in the affected child can be defined, the risk of recurrence can be quantified (through molecular analysis of sperm) if the mutation arose in the father.

  PSEUDOXANTHOMA ELASTICUM DEFINITION

Pseudoxanthoma elasticum (PXE) is a heritable disorder of connective tissue with pleiotropic manifestations wherever elastic fibers are found but primarily in the skin, eye, and vasculature.15 Life expectancy is reduced, on average, because of a predisposition to myocardial infarction and gastrointestinal hemorrhage.

EPIDEMIOLOGY

The exact frequency of PXE is unknown, but it is probably underdiagnosed. Rough approximations suggest a prevalence of one in 25,000 to 100,000 births. Males and females are equally affected, although women are more likely to seek medical attention out of concern for the skin changes.

PATHOBIOLOGY

Pathogenesis

In most families, PXE occurs as an autosomal recessive trait, which means, given relatively small sibships, that many patients will have no affected relatives. Apparent autosomal dominant inheritance may reflect expression in occasional heterozygotes. The gene for PXE maps to human chromosome 16 and encodes one of the adenosine triphosphate (ATP)-binding cassette transporters (ABCC6). Because of the prominent histopathologic feature of calcification of elastic tissue, this gene may be important in calcium homeostasis. It is unclear, however, whether calcification is a primary or a secondary phenomenon in PXE.

Pathology

The hallmark of PXE, and an important diagnostic clue, is the histopathologic finding of hyperproliferated elastic fibers in the mid-dermis; these fibers become fragmented, clumped, and calcified. An arteriolar sclerosis develops in the media of muscular arteries and arterioles; the lumen may become progressively and concentrically narrowed. Alternatively, microaneurysms can form. Thickening of the endocardium, especially in the atria, develops in some patients. In the eye, Bruch membrane becomes calcified and fragmented.

CLINICAL MANIFESTATIONS

Because of the pleiotropic nature of PXE, the diagnosis initially may be suspected by any of a variety of clinicians, especially dermatologists, ophthalmologists, cardiologists, and gastroenterologists. The condition gains its name from the dermatologic feature of yellowish papules that appear at areas of flexural stress, especially the neck, groin, and popliteal and cubital fossae; in periumbilical regions; and on the buccal mucosa. The appearance of affected skin has been likened to that of a “plucked chicken.” Over time, affected areas coalesce and become thickened. Changes in the eye begin as a generalized, subtle, mottled pattern in the retina (peau d’orange) and progress to the characteristic angioid streaks. The latter changes are not specific for PXE and can be seen in diabetes mellitus, sickle cell disease, and a variety of other conditions. Streaks represent breaks in Bruch’s membrane, an elastic lamina that lies between the retinal vasculature and the choroid. Spontaneous hemorrhages, especially those involving the macula, lead to progressive visual loss.16 Involvement of arteries of various calibers produces problems because of occlusion and hemorrhage.17 The lifetime risk of serious gastrointestinal hemorrhage from any site, but especially the stomach, is about 10%. Hypertension is relatively common, in part because of involvement of the renal vasculature. Progressive occlusion of peripheral arteries leads to absence of pulses; acral ischemia is rare because of the development of collaterals. The risk for stroke, myocardial infarction, abdominal angina, and intermittent claudication is increased independent of other risk factors. Impaired left ventricular function is common in adults.

DIAGNOSIS

Differential Diagnosis

Whole exome sequencing is an efficient and sensitive way to make the diagnosis.18 An acquired form of PXE has been reported and is also of unclear etiology. This form is difficult to differentiate from a sporadic case in a family because of heterozygosity in the parents, but it tends to affect only the skin. As suggested by the name, the cutaneous features of PXE need to be differentiated from those of true xanthoma, which results from a disorder of lipid metabolism (Chapter 206). The dermatologic manifestations need to be differentiated from those of Miescher elastoma, elastic tissue nevi (BuschkeOllendorff syndrome), and solar elastosis.

TREATMENT  No cure for or means of preventing PXE is known. In many instances, careful attention to the ocular features by a retinal specialist experienced in PXE can delay but not prevent loss of vision. The risk of gastrointestinal hemorrhage suggests that patients should avoid gastric irritants such as aspirin, nonsteroidal anti-inflammatory drugs, and excessive alcohol. Stool should be checked regularly for occult blood, and angiography may be necessary to detect the source of bleeding. All standard risk factors for atherosclerosis should be managed aggressively. Complaints of chest pain should prompt a rigorous investigation for coronary artery disease. Angioplasty has not been reported to be effective, and the coronary lesions tend to be diffuse. Coronary artery bypass graft surgery has been performed, but long-term results have not been reported. It may be theoretically advantageous to use vein grafts rather than the internal mammary artery for bypass. The excessive wrinkling and pseudoxanthoma in exposed areas can be ameliorated by plastic surgery.

FUTURE DIRECTIONS

Each of these disorders poses special considerations in clinical diagnosis, utility of molecular testing, genetic counseling, and management. For the storage disorders, the clinical utility of enzyme replacement therapy is actively being pursued by several pharmaceutical companies. For several of the other conditions, somatic stem cell therapy offers some promise but is years away from routine clinical use. In Marfan syndrome, clinical trials of drugs that modulate activity of TGF-β are underway. Additionally, close medical management for individuals detected as being at heightened risk for cardiovascular, skeletal, and ocular complications will remain a mainstay.

Grade A References A1. Groenink M, den Hartog AW, Franken R, et al. Losartan reduces aortic dilatation rate in adults with Marfan syndrome: a randomized controlled trial. Eur Heart J. 2013;34:3491-3500.

CHAPTER 261  The Systemic Autoinflammatory Diseases  

A2. Lacro RV, Dietz HC, Sleeper LA, et al. Atenolol versus losartan in children and young adults with Marfan’s syndrome. N Engl J Med. 2014;371:2061-2071. A3. Ong KT, Perdu J, De Backer J, et al. Effect of celiprolol on prevention of cardiovascular events in vascular Ehlers-Danlos syndrome: a prospective, randomized, open, blinded-endpoints trial. Lancet. 2010;376:1476-1484. A4. Bishop N, Adami S, Ahmed SF, et al. Risedronate in children with osteogenesis imperfecta: a randomised, double-blind, placebo-controlled trial. Lancet. 2013;382:1424-1432.

GENERAL REFERENCES For the General References and other additional features, please visit Expert Consult at https://expertconsult.inkling.com.

261  THE SYSTEMIC AUTOINFLAMMATORY DISEASES RICHARD M. SIEGEL AND DANIEL L. KASTNER

DEFINITION

The systemic autoinflammatory diseases (Table 261-1) are a group of illnesses characterized by seemingly unprovoked inflammation, without evidence of high-titer autoantibodies or antigen-specific T cells, thus distinguishing them from the more classic autoimmune diseases. The first conditions recognized as autoinflammatory were the hereditary recurrent fevers, a group of mendelian disorders characterized by episodic or fluctuating degrees of fever and localized inflammation. The scope of autoinflammatory disease has been broadened to include other heritable illnesses, including disorders in which purulent or granulomatous inflammation predominates, as well as inherited disorders of the complement system (Chapter 50).1-6 In addition, in numerous autoinflammatory conditions, some of which manifest in childhood and others that occur later in life, there is a complex interaction of genetic susceptibilities and environmental factors. These illnesses include systemic-onset juvenile idiopathic arthritis (Still’s disease), Behçet’s disease, and even the crystalline arthritides. Recent advances in the genetics and pathophysiology of the inherited autoinflammatory diseases suggest that these conditions are inborn errors of innate immunity, the phylogenetically more primitive part of the immune system that uses germline membrane and intracellular receptors expressed in granulocytes and macrophages to mount the body’s first line of defense against pathogens (Chapters 45 and 48).

  HEREDITARY RECURRENT FEVER SYNDROMES

Familial Mediterranean Fever

DEFINITION

Familial Mediterranean fever (FMF) is a recessively inherited illness that typically manifests with 12- to 72-hour episodes of fever and localized serosal, synovial, or cutaneous inflammation. Between attacks, patients usually feel completely well, although biochemical evidence of inflammation may remain, and some patients eventually develop systemic amyloidosis. Before the identification of the causative gene, FMF was defined purely clinically; clinical features remain an important part of the diagnosis, because some patients with typical disease have only one, or sometimes no, demonstrable mutation in MEFV, the only known causative gene.

EPIDEMIOLOGY

FMF is most common in individuals of Jewish, Arab, Armenian, Turkish, and Italian ancestry. The frequency of asymptomatic carriers of a single MEFV mutation in these populations is as high as 1 in 5, a finding that suggests a selective advantage for heterozygotes. With genetic testing, FMF is now frequently recognized in both Ashkenazi (eastern European) and non-Ashkenazi Jewish populations, as well as in Mediterranean populations previously thought not to be at risk. Mutation-positive individuals with typical symptoms have been documented worldwide. FMF usually manifests in childhood, sometimes even in infancy, although approximately 10% of patients

1739

experience their first attack as adults; infrequently, FMF first occurs in persons older than 40 years.

PATHOBIOLOGY

MEFV, the gene for FMF, was identified by positional cloning in 1997. It encodes a 781–amino acid protein denoted pyrin (or marenostrin) that is expressed in granulocytes, monocytes, and dendritic cells, as well as in peritoneal, synovial, and dermal fibroblasts. The N-terminal 92 amino acids of pyrin are the prototype for a motif, the PYRIN domain, that is involved in protein-protein interactions; this domain defines a family of more than 20 human proteins, including pyrin itself, involved in the regulation of cytokine production (particularly the interleukin-1 [IL-1] family), nuclear factor kappa B (NF-κB) activation, and apoptosis. More than 50 FMF-associated mutations in pyrin have been identified, many of which reside in the C-terminal domain encoded by exon 10 of MEFV. An even larger number of variants of unknown significance have been described in individual patients with a spectrum of inflammatory phenotypes.

CLINICAL MANIFESTATIONS

Episodes of FMF are more properly termed recurrent than periodic, and some patients associate attacks with psychological stress or physical exertion. Women of childbearing age sometimes experience their attacks with menses, with remissions during pregnancy. Some patients are unaware of fever during the attacks, but it is almost always observed when sought. Serosal involvement in FMF is usually peritoneal or pleural. Abdominal attacks are the most frequent, and they may vary from mild discomfort to frank peritonitis, with boardlike rigidity, direct and rebound tenderness, and air-fluid levels on upright films of the abdomen. Regardless of the severity of the abdominal attack, constipation is much more common than diarrhea. When a laparotomy or laparoscopy is performed during an attack, a small amount of sterile exudate rich in polymorphonuclear leukocytes is found. Except for serosal inflammation, the appendix is normal. Repeated abdominal attacks may cause peritoneal adhesions, but ascites is rare. Pleurisy, usually unilateral, may accompany abdominal pain, or it may occur independently. Physical findings, if present, may include diminished breath sounds and a pleural friction rub, whereas x-ray films may show a small effusion or atelectasis. With multiple attacks, pleural thickening may develop. Symptomatic nonuremic pericardial involvement in FMF has been reported but is unusual. In adults, the arthritis of FMF typically manifests as monoarticular involvement of the knee, hip, or ankle, and attacks of arthritis may persist for up to 1 week at a time. In children, oligoarticular or polyarticular joint involvement may occur. Large joint effusions are sometimes present, and the synovial fluid may have as many as 100,000 leukocytes/mm3. In approximately 5% of patients who are not treated with prophylactic colchicine, chronic arthritis (usually of the hip or knee) may develop, often necessitating joint replacement surgery. Regardless of colchicine treatment or a particular human leukocyte antigen (HLA-B27) status, some patients with FMF develop sacroiliitis. Arthralgia without frank arthritis is common in FMF. Cutaneous manifestations of FMF tend to be less common than serosal or synovial involvement. The characteristic skin lesion of FMF is erysipeloid erythema, a painful, demarcated erythematous area most often seen on the lower leg, ankle, or dorsum of the foot. This rash may occur independently, or it may accompany an episode of arthritis. Histologically, a mixed perivascular cellular infiltrate is seen. Other acute manifestations of FMF include unilateral scrotal inflammation (the tunica vaginalis is an embryologic remnant of the peritoneal membrane) and myalgia, either with fever or, especially in children, without fever and induced by vigorous exercise. Various forms of vasculitis also have been associated with FMF; Henoch-Schönlein purpura may occur in children with FMF; less frequently, polyarteritis nodosa is seen.

COMPLICATIONS

Before the widespread use of colchicine prophylaxis, systemic AA amyloidosis (Chapter 188) was a frequent complication of FMF, caused by the ectopic deposition of a misfolded fragment of serum amyloid A (SAA), an acute phase reactant, in the gastrointestinal tract, kidneys, spleen, lung, testes, and adrenals. Malabsorption and nephrotic proteinuria leading to renal failure are the most common manifestations of AA amyloidosis. Cardiomyopathy is less common, and neuropathy and arthropathy are rare. Several risk factors for amyloidosis development in FMF have been identified, including late diagnosis of FMF, colchicine noncompliance, male gender, and specific

CHAPTER 260  Inherited Diseases of Connective Tissue  

GENERAL REFERENCES 1. Campos D, Monaga M. Mucopolysaccharidosis type I: current knowledge on it pathophysiological mechanisms. Metab Brain Dis. 2012;27:121-129. 2. Stevenson DA, Steiner RD. Skeletal abnormalities in lysosomal storage diseases. Pediatr Endocrinol Rev. 2013;10(suppl 2):406-416. 3. De Ru MH, Boelens JJ, Das AM, et al. Enzyme replacement therapy and/or hematopoietic stem cell transplantation at diagnosis in patients with mucopolysaccharidosis type I: results of a European consensus procedure. Orphanet J Rare Dis. 2011;6:55. 4. Jondeau G, Detaint D, Tubach F, et al. Aortic event rate in the Marfan population. Circulation. 2012;125:226-232. 5. Sheikhzadeh S, Sondermann C, Rybczynski M, et al. Comprehensive analysis of dural ectasia in 150 patients with a causative FBN1 mutation. Clin Genet. 2014;86:238-245. 6. Rybczynski M, Koschyk D, Karmeier A, et al. Frequency of sleep apnea in adults with the Marfan syndrome. Am J Cardiol. 2010;105:1836-1841. 7. Pyeritz RE. Evaluation of the tall adolescent with some features of Marfan syndrome. Genet Med. 2012;14:171-177. 8. Miraldi Utz V, Coussa RG, Traboulsi EI. Surgical management of lens subluxation in Marfan syndrome. J AAPOS. 2014;18:140-146. 9. Song HK, Preiss LR, Maslen CL, et al. Valve-sparing aortic root replacement in patients with Marfan syndrome enrolled in the National Registry of Genetically Triggered Thoracic Aortic Aneurysms and Cardiovascular Conditions. J Heart Valve Dis. 2014;23:292-298.

1739.e1

10. Svensson LG, Blackstone EH, Alsalihi M, et al. Midterm results of David reimplantation in patients with connective tissue disorders. Ann Thorac Surg. 2013;95:555-562. 11. De Paepe A, Malfait F. The Ehlers-Danlos syndrome, a disorder with many faces. Clin Genet. 2012;82:1-11. 12. Lum YW, Brooke BS, Black JH III. Contemporary management of vascular Ehlers-Danlos syndrome. Curr Opin Cardiol. 2011;26:494-501. 13. Li Q, Jiang Q, Uitto J. Ectopic mineralization disorders of the extracellular matrix of connective tissue: molecular genetics and pathomechanisms of aberrant calcification. Matrix Biol. 2014;33: 23-28. 14. Byers PH, Pyott SM. Recessively inherited forms of osteogenesis imperfect. Annu Rev Genet. 2012;46:475-497. 15. Uitto J, Jiang Q, Varadi A, et al. Pseudoxanthoma elasticum: diagnostic features, classification, and treatment options. Expert Opin Orphan Drugs. 2014;2:567-577. 16. Ebran JM, Milea D, Trelohan A, et al. New insights into the visual prognosis of pseudoxanthoma elasticum. Br J Ophthalmol. 2014;98:142-143. 17. Campens L, Vanakker OM, Trachet B, et al. Characterization of cardiovascular involvement in pseudoxanthoma elasticum families. Arterioscler Thromb Vasc Biol. 2013;33:2646-2652. 18. Hosen MJ, Van Nieuwerburgh F, Steyaert W, et al. Efficiency of exome sequencing for the molecular diagnosis of pseudoxanthoma elasticum. J Invest Dermatol. 2014;[Epub ahead of print].

1739.e2

CHAPTER 260  Inherited Diseases of Connective Tissue  

REVIEW QUESTIONS 1. Osteogenesis imperfect syndromes demonstrate A. intergenic heterogeneity. B. intragenic heterogeneity. C. variable expression. D. pleiotropy. E. all of the above. Answer: E  All four of the choices are correct. Mutations in multiple genes can cause OI (intergenic heterogeneity). With a single locus, many different mutant alleles have been discovered (intragenic heterogeneity). Within a given type of OI, relatives with the same mutation can demonstrate different features of varying severity (variable expression). Multiple organ systems are affected in OI (pleiotropy). 2. Which of the following is not a common feature of Marfan syndrome? A. Pulmonary arteriovenous malformation B. Aortic root dilatation C. Mitral valve prolapse D. Pulmonic artery dilatation E. Aortic dissection Answer: A  All four of the latter choices are common features. Arteriovenous malformations are common in hereditary hemorrhagic telangiectasia but not in Marfan syndrome. 3. Hypertension is a relatively common feature in which syndrome? A. Osteogenesis imperfect B. Hurler syndrome C. Marfan syndrome D. Vascular Ehlers-Danlos syndrome E. Pseudoxanthoma elasticum Answer: E  Because of partial occlusion of the renal arteries, elevated blood pressure can occur in PXE. None of the other syndromes has a predisposition to hypertension.

4. Which of the following is not a currently accepted approach to management of Marfan syndrome? A. Prophylactic surgery of the aortic root B. Exercise restriction C. Annual echocardiography D. Gene therapy E. Chronic β-adrenergic blockade Answer: D  At the present time, no approach to correcting the specific mutation in FBN1 is feasible. All of the other approaches are thought to be beneficial. 5. If treatment is not offered, which of the following conditions is associated with the best prognosis? A. Ehlers-Danlos syndrome, hypermobility type B. Osteogenesis imperfect, type II C. Marfan syndrome D. Hurler syndrome E. Ehlers-Danlos syndrome, vascular type Answer: A  The hypermobility form of Ehlers-Danlos syndrome has little to no added mortality. Osteogenesis imperfect type II is usually lethal in infancy. Marfan syndrome has reduced life expectancy because of aortic dissection. Patients with Hurler syndrome rarely survive to their third decade. Patients with the vascular form of Ehlers-Danlos syndrome are at high risk of death from arterial or bowel rupture.

1740

CHAPTER 261  The Systemic Autoinflammatory Diseases  

TABLE 261-1  SYSTEMIC AUTOINFLAMMATORY DISEASES: A PARTIAL LISTING INHERITED AUTOINFLAMMATORY DISEASES

INHERITANCE/ETIOLOGY

GENES OR RISK FACTORS

OMIM*

HEREDITARY RECURRENT FEVER SYNDROMES Familial Mediterranean fever (FMF) Tumor necrosis factor receptor–associated periodic syndrome (TRAPS) Hyperimmunoglobulinemia D with periodic fever syndrome (HIDS) Familial cold autoinflammatory syndrome (FCAS) Muckle-Wells syndrome (MWS) Neonatal-onset multisystem inflammatory disease (NOMID)/chronic infantile neurologic cutaneous and articular (CINCA) syndrome

Autosomal recessive Autosomal dominant

MEFV† TNFRSF1A†

249100 142680

Autosomal recessive

MVK†

260920

Autosomal dominant Autosomal dominant Sporadic, autosomal dominant

NLRP3 (formerly CIAS1)† NLRP3 (formerly CIAS1)† NLRP3 (formerly CIAS1)†

120100 191900 607115

Autosomal dominant

NOD2/CARD15†

186580



GRANULOMATOUS DISORDERS Granulomatous inflammatory arthritis, dermatitis, and uveitis (Blau’s syndrome) Early-onset sarcoidosis Crohn’s disease

Sporadic, autosomal dominant Complex inheritance

NOD2/CARD15 NOD2/CARD15†

609464 266600

Autosomal dominant

PSTPIP1†

604416

Deficiency of interleukin-1 receptor antagonist (DIRA) Chronic recurrent multifocal osteomyelitis (CRMO)

Autosomal recessive Sporadic, autosomal recessive

612852 259680

Synovitis acne pustulosis hyperostosis osteitis syndrome (SAPHO)

Idiopathic

IL1RN LPIN2,† when associated with congenital dyserythropoietic anemia (Majeed syndrome) —

Autosomal dominant Autosomal dominant, sporadic Complex inheritance

C1NH HF1 (complement factor H) HF1 (complement factor H)

106100 235400 603075

Syndrome of periodic fever with aphthous stomatitis, pharyngitis, and cervical adenopathy (PFAPA) Autoinflammation, lipodystrophy, and dermatosis syndrome (Nakajo-Nishimura syndrome, JMP syndrome, CANDLE syndrome) Systemic-onset juvenile idiopathic arthritis (SOJIA) Adult-onset Still’s disease Schnitzler’s syndrome Behçet’s disease

Idiopathic



Autosomal recessive

PSMB8

256040

Complex inheritance Idiopathic Idiopathic Complex inheritance

604302 — — 109650

Crystalline arthropathies

Complex inheritance

IL-6, MIF polymorphisms — — HLA-B51, polymorphisms in IL10, IL23R, CCR1, STAT4, KLRC4, ERAP1, MEFV, TLR4 SLC2A9/GLUT9, ABCG2

PYOGENIC DISORDERS Syndrome of pyogenic arthritis with pyoderma gangrenosum and acne (PAPA) AUTOINFLAMMATORY DISORDERS OF SKIN AND BONE



COMPLEMENT DISORDERS Hereditary angioedema Hemolytic-uremic syndrome Age-related macular degeneration OTHER AUTOINFLAMMATORY SYNDROMES —



*Online Mendelian Inheritance in Man, an online catalogue of genetic disorders, available at http://www.ncbi.nlm.nih.gov/entrez/query.fcgi?db=OMIM. Accessed September 29, 2014. † An updated list of disease-associated mutations is available online at http://fmf.igh.cnrs.fr/infevers. Accessed September 29, 2014.

genotypes of the MEFV and SAA genes. Amyloidosis in FMF is less common in the United States than in the Middle East. Abdominal fat aspirates are much less sensitive than rectal or renal biopsy in detecting the amyloidosis of FMF. The latter procedure may be preferred, because of the increasing recognition of nonamyloid glomerular disease in FMF. With early diagnosis, aggressive suppression of the acute phase response with colchicine or adjunctive agents may lead to improvement, but for patients with renal failure, early renal transplantation is preferred.

DIAGNOSIS

Based on a simple recessive model of inheritance, two mutations in MEFV, in trans, should be identified to establish the genetic diagnosis of FMF. Nevertheless, the interpretation of genetic testing is complicated by complex alleles consisting of various combinations of mutations in cis, as well as by the observations that as many as one third of patients with clinically typical FMF have only one demonstrable mutation in MEFV, and a few patients with typical disease have no identifiable MEFV mutations. These latter two findings suggest that, under some circumstances, one MEFV mutation may be sufficient for symptoms or that additional genes for FMF exist. For these reasons, clinical data remain an essential part of the diagnosis of FMF, and genetic testing plays an adjunctive role in settings in which clinical experience is limited.7 Clinical criteria emphasize attack duration (12 to 72 hours); recurrence of symptoms (three or more episodes); documented fever (rectal temperature > 38° C); painful manifestations in the abdomen, chest,

joints, or skin; and the absence of other causative factors. The differential diagnosis includes the other hereditary recurrent fever syndromes (Table 261-2), as well as other conditions specific to the clinical setting. For patients with recurrent abdominal pain, considerations include gynecologic disorders, porphyria (Chapter 210; which can be distinguished by hypertension during attacks, dominant inheritance, and urine porphyrins), and hereditary angioedema (Chapter 252; which usually does not cause fever). The syndrome of periodic fever with aphthous stomatitis, pharyngitis, and cervical adenopathy is probably the most common cause of unexplained recurrent fever in children and is also included in the differential diagnosis. In patients presenting primarily with recurrent monoarthritis, joint aspiration for cultures and crystals may aid in excluding bacterial and crystalline arthritis, respectively. Still’s disease in children (systemic-onset juvenile idiopathic arthritis) and adults (adult-onset Still’s disease) is also considered in the differential diagnosis. Adult-onset Still’s disease8 (see Table 261-1) is an uncommon autoinflammatory condition of unknown cause that is not considered to be hereditary. It is characterized by spiking fever, an evanescent salmon-pink maculopapular rash, arthritis, and neutrophilic leukocytosis. It can be clinically distinguished from FMF by the pattern of fever (intermittent quotidian in Still’s disease vs. discrete episodes in FMF), the pattern of arthritis (chronic polyarthritis vs. intermittent monoarthritis), the characteristic skin involvement (evanescent rash vs. erysipeloid erythema), and the presence of lymphadenopathy (more common in Still’s disease).

1741

CHAPTER 261  The Systemic Autoinflammatory Diseases  

TABLE 261-2  CLINICAL FEATURES OF SELECTED HEREDITARY RECURRENT FEVER SYNDROMES CLINICAL FEATURE Typical ethnicity

FMF Arab, Armenian, Italian, Jewish, Turkish

TRAPS Any ethnicity

HIDS

FCAS

Dutch, other North European

European

MWS European

NOMID/CINCA Any

Attack duration

12-72 hr

Days to weeks

3-7 days

12-24 hr

1-2 days

Continuous, with flares

Abdominal attacks

Sterile peritonitis, constipation more often than diarrhea

Severe pain, vomiting, peritonitis

Sterile peritonitis, diarrhea, rarely constipation

Nausea

Abdominal pain

Not common

Pleural attacks

Common

Common

Rare

Not seen

Rare

Rare

Joint/bone involvement

Monoarthritis, rarely protracted arthritis in knee or hip

Arthritis in large joints, arthralgia

Arthralgia, symmetrical polyarthritis

Polyarthralgia

Polyarthralgia, oligoarthritis, clubbing

Epiphyseal overgrowth, contractures, intermittent or chronic arthritis, clubbing

Skin rash

Erysipeloid erythema on lower leg, ankle, foot

Migratory rash, underlying myalgia

Diffuse maculopapular rash, urticaria

Cold-induced urticaria-like rash

Urticaria-like rash

Urticaria-like rash

Lymphatic involvement

Splenomegaly, occasional lymphadenopathy

Splenomegaly, occasional lymphadenopathy

Cervical adenopathy in children

Not seen

Rare

Hepatosplenomegaly, adenopathy

Neurologic involvement

Aseptic meningitis?

Controversial

Headache

Headache

Sensorineural deafness

Sensorineural deafness, chronic aseptic meningitis, intellectual disability, headache

Ophthalmologic involvement

Rare

Conjunctivitis, periorbital edema, rarely uveitis

Uncommon

Conjunctivitis

Conjunctivitis, episcleritis

Uveitis, conjunctivitis, progressive vision loss

Vasculitis

Henoch-Schönlein purpura (HSP), polyarteritis nodosa

HSP, lymphocytic vasculitis

Cutaneous vasculitis common, rarely HSP

Not seen

Not seen

Occasional

Systemic amyloidosis

Risk depends on MEFV and SAA genotypes; more common in Middle East

Occurs in ~10%; risk increased with cysteine mutations

Rare

Rare

Occurs in ~25%

May develop in some, usually in adulthood

FCAS = familial cold autoinflammatory syndrome; FMF = familial Mediterranean fever; HIDS = hyperimmunoglobulinemia D with periodic fever syndrome; MWS = Muckle-Wells syndrome; NOMID/CINCA = neonatal-onset multisystem inflammatory disease/chronic infantile neurologic cutaneous and articular syndrome; TRAPS = tumor necrosis factor receptor–associated periodic syndrome.

TREATMENT  The mainstay of therapy for FMF is daily oral colchicine, which can prevent both acute attacks of FMF and the development of systemic amyloidosis. Colchicine probably works by several mechanisms, including its effects on inhibiting leukocyte adhesion and modulating cytokine production. In adults, the therapeutic dose is 1.2 to 1.8 mg/day, and nearly 90% of patients note significant improvement at this dose. The major side effects are gastrointestinal, and they can usually be minimized by gradually increasing the dosage and avoiding milk products in patients who develop lactose intolerance. Most experts continue to prescribe colchicine to patients during pregnancy, with the recommendation that amniocentesis be performed to exclude trisomy 21, for which there may be a slightly increased risk. Use of colchicine in lactating women is considered safe. Intravenous colchicine should be used with extreme caution, if at all, in FMF, because fatal toxicity has been reported in patients already receiving oral colchicine who are given the drug intravenously. IL-1 inhibitors9 may be effective in patients who are poorly responsive to colchicine or who cannot tolerate therapeutic doses.

Tumor Necrosis Factor Receptor–Associated Periodic Syndrome

DEFINITION

Worldwide, the tumor necrosis factor (TNF) receptor–associated periodic syndrome (TRAPS) is the second most frequently diagnosed hereditary recurrent fever syndrome, behind FMF. TRAPS is defined by recurrent episodes of fever and localized inflammation, in many ways resembling FMF, but differing in key details (noted later) and caused by mutations in the 55-kD receptor for TNF (TNFRSF1A, TNFR1, p55, CD120a). Whereas a positive genetic test is not necessary to diagnose FMF, the diagnosis of TRAPS requires the identification of a TNF receptor mutation. One of the first wellcharacterized families with what was later defined as TRAPS was of Irish ancestry, and the condition was termed familial Hibernian fever to emphasize

the ethnic background and clinical differences from FMF. However, with the discovery of TNF receptor mutations in families of other ancestries, the ethnically neutral TRAPS nomenclature was proposed.

PATHOBIOLOGY

The p55 TNF receptor is composed of four cysteine-rich extracellular domains, a transmembrane region, and an intracellular death domain. To date, nearly all of the more than 90 mutations described are in the extracellular domains and approximately one third are missense substitutions of cysteine residues that abolish highly conserved disulfide bonds. The initial description of TRAPS documented a defect in activation-induced ectodomain cleavage of the p55 receptor in patients with the C52F TNFRSF1A mutation, possibly leading to a defect in homeostasis by impaired downregulation of membrane receptors and diminished shedding of potentially antagonistic soluble receptor molecules. More recent studies indicate a more complex pathogenetic picture, because not all mutant receptors exhibit this shedding defect. Additional mechanisms by which p55 mutations may lead to autoinflammation include impaired leukocyte apoptosis and impaired intracellular receptor trafficking, with possible constitutive activation of mitogen-activated protein (MAP) kinases by intracellular aggregates of mutant receptors.10

DIAGNOSIS

Although genetic testing is necessary for the diagnosis of TRAPS, certain clinical clues can help distinguish TRAPS from FMF. These include ethnicity (FMF is seen predominantly in Mediterranean and Middle Eastern populations, whereas TRAPS has a more widespread distribution), mode of inheritance (autosomal recessive in FMF, dominant in TRAPS), and duration of attacks, which tends to be longer in TRAPS and sometimes approaches continuous symptoms. The rash of FMF is typically erysipeloid erythema on the lower extremity, whereas patients with TRAPS often have a distinctive erythematous rash, often with underlying myalgia, which may migrate on the trunk or centrifugally on the extremities. Ocular involvement, with periorbital edema, conjunctivitis, and occasionally even uveitis, is observed in

1742

CHAPTER 261  The Systemic Autoinflammatory Diseases  

TRAPS but not in FMF. Finally, whereas colchicine is much more effective than corticosteroids in FMF, the opposite is true in TRAPS. Nevertheless, aside from the difference in duration and susceptibility to pharmacologic intervention, the abdominal, pleural, synovial, and even scrotal manifestations of the two diseases are rather similar. The usual age of onset for TRAPS is also in childhood, and systemic AA amyloidosis is seen in approximately 10% of untreated patients with TRAPS. As in FMF, life expectancy in TRAPS is normal in patients whose disease is not complicated by amyloidosis. As noted earlier, the diagnosis of TRAPS is established by the identification of TNFRSF1A mutations in the appropriate clinical setting. One variant, the substitution of glutamine for arginine at residue 92 (R92Q), is present in more than 1% of whites and may be associated with a broader spectrum of symptoms than is typically seen in TRAPS, including early inflammatory arthritis or, in some cases, no symptoms at all. The substitution of lysine for proline at residue 46 (P46L) is common among African American patients with TRAPS and is associated with a receptor shedding defect, but it is also seen among healthy African American controls. These findings establish a “gray zone” for the diagnosis of TRAPS and emphasize the potential role of polymorphisms in the recurrent fever genes in other more common phenotypes.

TREATMENT  The treatment of TRAPS depends on the frequency and severity of attacks. Patients with relatively infrequent, mild episodes may respond to nonsteroidal anti-inflammatory drugs (NSAIDs). Patients with more severe attacks that occur infrequently may be treated with corticosteroids, although increasing doses may be required as the episodes become more frequent and toxicities may become limiting. For patients with severe attacks occurring once a month or more frequently, treatment with etanercept, the soluble p75 TNF receptor:Fc fusion protein, may be warranted. This may be a unique effect of etanercept, because there is anecdotal evidence that monoclonal antibodies against TNF may actually exacerbate TRAPS. Consistent with a model implicating multiple cytokines in the pathogenesis of TRAPS, IL-1 inhibitors have also been found effective in TRAPS.11

  INTERLEUKIN-1–ASSOCIATED AUTOINFLAMMATORY DISEASES

The IL-1–associated autoinflammatory diseases are linked by markedly increased expression or cellular responsiveness to this cytokine, and dramatic resolution of symptoms with IL-1 blockade. Interleukin-1α and Interleukin1β (IL-1α and IL-1β) are structurally related cytokines released from cells triggered by a number of inflammatory stimuli, such as lipopolysaccharide. They mediate inflammatory responses by binding to a common receptor that is present on the surface of a wide variety of cell types and signals to activate inflammatory genes through the nuclear factor kappa B (NF-KB) transcription factor complex. IL-1 is part of a larger family of cytokines including IL-18, IL-33, and IL-36, which bind to related receptors and share the property of not having a characteristic signal peptide that normally targets cytokines to secretory vesicles. Because of this, IL-1 family cytokines may be secreted only by dead or dying cells, functioning as molecular markers of cellular stress, which can trigger beneficial inflammatory responses to infection and injury. IL-1β and IL-18 are unique in that they are not biologically active until cleaved by the protease caspase-1. Caspase-1 is itself activated in cytoplasmic protein complexes containing various sensor proteins such as NLRP3, and the adapter protein ASC. These complexes are referred to as inflammasomes because of their ability to trigger IL-1–mediated inflammation. Autoinflammatory diseases caused by mutations in genes encoding proteins that process or sense IL-1 are described later.

Cryopyrin-Associated Periodic Syndromes: The Cryopyrinopathies Three rare, recurrent febrile disorders usually beginning early in life have been associated with mutations in NLRP3 (formerly CIAS1), the gene encoding a protein variously named cryopyrin, NLRP3, NALP3, PYPAF1, or CATERPILLER 1.1, a key component of the NLRP3 inflammasome that activates caspase-1. These disorders are referred to as cryopyrinopathies or cryopyrinassociated periodic syndromes (CAPS). The least severe clinical phenotype is familial cold autoinflammatory syndrome (FCAS; formerly called familial cold urticaria), which is dominantly inherited and is notable for day-long attacks of chills, fever, headache, diffuse urticarial skin rash, arthralgia, and

conjunctivitis, precipitated by generalized cold exposure. Amyloidosis is rare in FCAS. Of intermediate severity is Muckle-Wells syndrome (MWS), also dominantly inherited, in which 1- to 2-day episodes of chills, fever, urticarial rash, limb pain, and arthritis occur independently of cold exposure. Sensorineural hearing loss is common in MWS, and systemic amyloidosis also may occur. The most severe NLRP3-associated phenotype is neonatal-onset multisystem inflammatory disease (NOMID), known in Europe as chronic infantile neurologic cutaneous and articular (CINCA) syndrome. It is usually sporadic owing to the reduced reproductive fitness of most affected individuals. Fever and constitutional symptoms occur almost daily, often from birth, with generalized urticarial skin rash, a peculiar arthropathy characterized by epiphyseal overgrowth of the long bones, and central nervous system (CNS) involvement that includes chronic aseptic meningitis, uveitis, and cochlear inflammation, which may lead to intellectual disability, blindness, and deafness. In all three cryopyrinopathies, the rash is not true urticaria because there is a neutrophilic rather than a mast cell infiltrate and serum histamine levels are normal. The protein mutated in all three disorders is NLRP3, a critical component of the eponymous NLRP3 inflammasome, which serves as an intracellular scaffold for the processing of IL-1β. The alternative name of this protein, cryopyrin, refers to its aminoterminal PYRIN domain, the basis for a structural and functional relationship to the protein mutated in FMF. Diseaseassociated cryopyrin mutations are thought to decrease the threshold for inflammasome activation, thereby increasing IL-1β production. The discovery that the NLRP3 inflammasome is also necessary for IL-1β production in response to crystalline forms of monosodium urate and calcium pyrophosphate connected the pathophysiology of these rare autoinflammatory diseases to crystal-induced arthritis (Chapter 273), which shares some clinical features, such as episodic, self-limited attacks, with autoinflammatory diseases. Because there are patients with FCAS, MWS, and NOMID/CINCA without demonstrable NLRP3 mutations, these diagnoses remain clinical, although genetic testing serves as a valuable adjunct and has greatly increased the recognition of all three conditions. Deep sequencing has identified somatic NLRP3 mutations in some patients with symptoms consistent with CAPS who are negative for mutations by standard genetic testing. In addition, overlap syndromes that are intermediate between FCAS and MWS and between MWS and NOMID/CINCA have been reported.

TREATMENT  Blockade with anakinra, a recombinant IL-1 receptor antagonist, is effective in controlling fever and acute phase reactants in all three cryopyrinopathies, and longitudinal analysis of a large series of patients at the National Institutes of Health showed that long-term treatment with anakinra markedly decreased CNS inflammation and end-organ damage in NOMID/CINCA, which led to the regulatory approval of anakinra for the treatment of this condition in the United States and Europe. Recent studies have also documented the efficacy of rilonacept, another soluble IL-1 blocker, and canakinumab, a monoclonal antibody against IL-1β, in FCAS and MWS, although these agents may be less effective against NOMID/CINCA because of reduced penetration into the CNS. The efficacy of canakinumab suggests that the major biologic effect of cryopyrin in humans is mediated through IL-1β rather than by IL-1α or other distinct inflammatory pathways.

Deficiency of Interleukin-1 Receptor Antagonist Deficiency of the IL-1 receptor antagonist (DIRA) is characterized by the neonatal onset of a pustular skin rash, multifocal osteomyelitis, periostitis, and, rarely, vasculitis.12 Fever is not a prominent finding, although acute phase reactants are markedly elevated. DIRA is caused by recessively inherited lossof-function mutations in IL1RN, which encodes the IL-1 receptor antagonist (IL-1Ra). Patients usually present within the first 2 weeks of life with skin lesions ranging from discrete crops of pustules to generalized severe pustulosis or ichthyosiform lesions. Histologic examination demonstrates extensive neutrophilic infiltrates in the dermis and epidermis. Typical radiographic findings include multifocal osteolytic lesions, periosteal elevation of the long bones, heterotopic ossification of the proximal femurs, and widening of the anterior rib ends. Bone biopsies demonstrate sterile purulent osteomyelitis, fibrosis, and sclerosis. To date, five different IL1RN mutations have been identified, three of which are truncating point mutations that drastically reduce IL-1Ra messenger RNA and protein levels. The fourth is a 15bp in-frame deletion, and the fifth is a 175-kilobase genomic deletion in

CHAPTER 261  The Systemic Autoinflammatory Diseases  

chromosome 2q that subsumes IL1RN and five other genes in the IL-1 family. In DIRA, the lack of IL-1Ra leads to unopposed IL-1β and IL-1α signaling, whereas in the cryopyrinopathies, NLRP3 mutations lead to inflammasome activation and increased IL-1β production. DIRA patients respond dramatically to anakinra, a recombinant form of the protein they lack.

  OTHER INHERITED SYSTEMIC AUTOINFLAMMATORY DISEASES

Syndrome of Pyogenic Arthritis with Pyoderma Gangrenosum and Acne The syndrome of pyogenic arthritis with pyoderma gangrenosum and acne (PAPA) is a rare, dominantly inherited autoinflammatory disease charac­ terized by intermittent episodes of sterile pyogenic arthritis, pyoderma gangrenosum, and severe cystic acne. It is caused by mutations in prolineserine-threonine phosphatase–interacting protein 1 (PSTPIP1), also known as CD2BP1. PSTPIP1 is a cytoskeletal protein that interacts with certain other proteins involved in the immune response, including CD2; the WiskottAldrich syndrome protein (WASP); a phosphatase denoted PTP-PEST; and pyrin, the FMF protein. PAPA mutations abrogate the binding of PSTPIP1 to PTP-PEST, leading to hyperphosphorylation of PSTPIP1 and increased binding to pyrin. Both in patients and in cell lines, this finding is associated with markedly increased IL-1β production. Early in life, PAPA tends to present with monoarticular or pauciarticular pyogenic arthritis, sometimes induced by trauma. In the absence of treatment, arthritis may progress to severe joint damage and ankylosis. As patients reach puberty, skin manifestations begin to predominate, including disfiguring cystic acne. Pathergy also may develop, and extensive pyoderma gangrenosum may require opiates for pain control. The diagnosis of PAPA syndrome is made by documenting PSTPIP1 mutations in the appropriate clinical setting. High doses of corticosteroids have been used in PAPA, with varying success, and patients with arthritis sometimes require aspiration, intra-articular corticosteroids, or open drainage. Newer investigational approaches for PAPA syndrome focus on the use of targeted cytokine inhibitors. Anecdotal evidence supports the use of anakinra for the arthritis and monoclonal anti-TNF antibodies for the pyoderma gangrenosum of PAPA.

Granulomatous Inflammatory Arthritis, Dermatitis, and Uveitis (Blau’s Syndrome) Blau’s syndrome is a rare, dominantly inherited illness characterized by the following features: early-onset granulomatous synovitis often complicated by cyst formation and camptodactyly (flexion contractures of the fingers and toes); granulomatous anterior and posterior uveitis, sometimes causing retinal detachment, glaucoma, cataracts, and blindness; and an intermittent papular rash with noncaseating granulomas. Lung or other visceral involvement is generally not present. However, visceral involvement of the liver and spleen is observed in early-onset sarcoidosis (Chapter 95), which is phenotypically quite similar to Blau’s syndrome. Both Blau’s syndrome and some cases of early-onset sarcoidosis are caused by mutations in NOD2/CARD15. Distinct variants of NOD2/CARD15 have been associated with susceptibility to Crohn’s disease, which manifests as granulomatous inflammation of the gastrointestinal tract (Chapter 141). The protein encoded by this gene is thought to be an intracellular sensor of bacterial products. Crohn’s disease— associated mutations in the ligand-binding, leucine-rich repeat region of the protein may alter responses to bacterial products in the gastrointestinal tract to cause inflammation, whereas Blau’s syndrome mutations in the nucleotide binding domain may lead to constitutive extraintestinal inflammation. Topical and systemic corticosteroids are currently the mainstay of treatment of Blau’s syndrome. There are case reports of the efficacy of TNF and IL-1 inhibitors in this disease.

Hyperimmunoglobulinemia D with Periodic Fever Syndrome Hyperimmunoglobulinemia D with periodic fever syndrome (HIDS) was first described in 1984 as an FMF-like illness seen in six patients of Dutch ancestry. Besides the difference in ethnicity, a key distinction was the observation of extremely high levels of immunoglobulin D (IgD) in the serum of these patients, thus prompting the HIDS nomenclature. HIDS is now recognized in a broader ethnic distribution, although northern Europeans still predominate. Overall, HIDS is still quite rare. Family studies documented autosomal recessive inheritance. In 1999, patients with HIDS were found to have mutations in MVK, which encodes the mevalonate kinase enzyme

1743

involved in the biosynthesis of cholesterol and nonsterol isoprenes. Enzyme activity in patients is markedly reduced, but not absent. The elevated immunoglobulin D (IgD) levels seen in HIDS appear to be an epiphenomenon and do not correlate with disease severity either among patients or in a given patient over time, although IgD may contribute to the release of proinflammatory cytokines in vitro. Moreover, modest elevations of IgD are seen in several inflammatory conditions, including chronic infections, and can be observed in other hereditary recurrent fever syndromes. Up to 20% of patients (particularly young children) with typical recurrent fevers and MVK mutations have normal serum IgD levels. Current data suggest that isoprenoid deficiency may play a more important pathogenic role in the pathophysiology of HIDS. In vitro studies suggest that isoprenoid deficiency may lead to excessive IL-1β production, and increased body temperature can further decrease mevalonate kinase enzymatic activity, thereby creating a vicious circle in which infection or immunization can precipitate HIDS attacks. One of the well-recognized clinical characteristics of HIDS is the provocation of attacks by immunizations. Other distinguishing clinical features include a very early age of onset (average age, 6 months), a duration of attacks intermediate between FMF and TRAPS (3 to 7 days), prominent cervical lymphadenopathy during attacks, polyarticular joint involvement, a diffuse maculopapular rash, the predominance of diarrhea over constipation with abdominal attacks, and the infrequency of pleuritic attacks or systemic amyloidosis. The diagnosis of HIDS can be established in a patient with recurrent episodes of fever and typical associated findings by documenting either two mutations in MVK or elevated levels of mevalonic acid, the substrate for mevalonate kinase, in the urine during attacks. Approximately 10% of patients with otherwise typical disease have only a single identifiable MVK mutation. The significance of elevated IgD without genetic or biochemical findings remains unknown. NSAIDs or corticosteroids are sometimes useful in the treatment of the arthritic manifestations of HIDS. Colchicine is generally not effective. Numerous agents are investigational in HIDS, including the statins, TNF inhibitors, and IL-1 inhibitors. Patients with HIDS have a normal lifespan, and attacks may become somewhat less frequent in adulthood.

Proteasome-Associated Systemic Inflammatory Diseases Recently, a constellation of diseases have been described linked to recessive loss of function mutations in PSMB8, which encodes the β5i subunit of the proteasome, also known as LMP7. An autosomal recessive syndrome in adults characterized by recurrent fevers, progressive lipodystrophy, joint contractures, and cardiac manifestations was linked to homozygous missense mutations in PSMB8.13 Patients with a pediatric syndrome termed CANDLE (chronic atypical neutrophilic dermatosis with lipodystrophy and elevated temperature) were found to have homozygous missense and nonsense mutations in PSMB8, with some patients having only one known PSMB8 mutation. It is not yet clear whether these syndromes represent identical diseases related to loss of function of β5i. The β5i proteasome subunit is one of the subunits that that are induced in immune cells through immune stimuli such as interferons, altering the proteasome so that it more efficiently processes peptides for antigen presentation to T cells. However, there is no indication of a T cell component to this disease, and studies have shown that the β5i proteasome subunit can be expressed in nonimmune cells such as adipocytes. A striking interferon transcriptional signature, similar to that seen in systemic lupus erythematosus, was observed in circulating blood cells from patients with CANDLE. Defective degradation of proteins in cells lacking β5i may result in buildup of ubiquitinated proteins, which somehow triggers interferon production, or PSMB8 deficiency may enhance interferon signaling by stabilizing components of the interferon signal transduction machinery that are negatively regulated by ubiquitin-proteasome degradation. Whichever the mechanisms, the link to interferon hyperactivity suggests that blocking interferons with antibodies or inhibitors of interferon signal transcution may be effective in the therapy of CANDLE and possibly other PSMB8-associated syndromes.

New Autoinflammatory Syndromes and the Promise of Whole-Exome Sequencing Recent years have seen a dramatic acceleration in the pace of discovery of new mendelian inflammatory diseases as a result of the availability of wholeexome sequencing, which allows unbiased identification of disease-causing mutations in protein coding sequences, although it should be noted that accurate clinical description of these syndromes is as important as the genetic tools for identification of new syndromes. These discoveries have confirmed

1744

CHAPTER 262  Osteoarthritis  

the role of gene products in human inflammation that were identified in other animal model systems and identified new genes and proteins not previously thought to be involved in the regulation of inflammation. Early-onset, apparently sporadic, cases of inflammatory syndromes have often turned out to be due to de novo mutations in a child when screened against parental DNA. For example, inherited gain-of-function mutations in CARD14, encoding an adapter protein in innate immune sensing, cause dominantly inherited familial psoriasis, and a more severe gain-of-function de novo mutation in the same gene caused infantile-onset severe pustular psoriasis. Recessive mutations causing systemic autoinflammatory disease also have been identified by whole-exome sequencing from just a few families. Recent examples of novel diseases discovered through these methods include a syndrome characterized by fevers, early-onset strokes, and vasculopathy or frank vasculitis caused by autosomal recessive mutations in CECR1, encoding adenosine deaminase 2 (ADA2), a serum protein with newly recognized effects on macrophage differentiation and vascular development.14 Recessive mutations in HOIL1 that impair the addition of linear ubiquitin chains to receptor signaling complexes cause a complex syndrome marked by autoinflammation and immunodeficiency and intramuscular glycogen deposition. Gain-of-function mutations in PLCG2, encoding phospholipase Cγ2, an enzyme with essential functions in B-cell receptor and Fc Rceptor signaling, cause a dominantly inherited autoinflammatory syndrome characterized by blistering skin lesions, bronchiolitis, arthralgia, ocular inflammation, and enterocolitis in the absence of autoantibodies. GENERAL REFERENCES For the General References and other additional features, please visit Expert Consult at https://expertconsult.inkling.com.

262  OSTEOARTHRITIS JOEL A. BLOCK AND CARLA SCANZELLO

DEFINITION

Osteoarthritis (OA) is a heterogeneous disease that has many names, including degenerative joint disease and osteoarthrosis. It is a joint disease characterized clinically by pain and functional loss. Although local inflammation of involved joints is common, OA is not associated with a systemic inflammatory process, in contrast to other arthritides. It is the most common form of arthritis and accounts for the overwhelming majority of arthritis cases, and its prevalence is expected to rise dramatically during the next 20 years as global populations age. OA is often neglected either because it is not a fatal disease or because many physicians assume it is a normal part of aging and is not inherently treatable. Yet it results in vast direct medical costs and significant loss of work; it is the leading indication for total joint replacement and is a leading cause of work disability. Formal definitions of OA have evolved as our understanding of pathophysiology has progressed. Whereas it conventionally had been considered primarily a degenerative process of cartilage, it is now clear that OA involves the entire joint. Thus, a modern definition of OA is a painful degenerative process involving progressive deterioration of all joint structures and remodeling of subchondral bone that is not primarily inflammatory. It is important to distinguish true OA from asymptomatic structural degeneration of joints that is virtually universal during normal aging.

EPIDEMIOLOGY

As an age-related disease, OA prevalence has risen substantially with the aging population of the developed world; an estimated 27 million people had physician-diagnosed OA in the United States in 2005, increased from 21 million a decade earlier, and this number is expected to reach 67 million patients with clinically significant OA by 2030. For epidemiologic studies, OA is often defined radiographically by the presence of osteophytes, joint space narrowing, and subchondral sclerosis. However, a substantial number

of individuals have these x-ray changes but remain clinically asymptomatic. Thus, estimates of OA prevalence vary widely depending on whether one is assessing radiographic or symptomatic OA. In either case, the lifetime risk is exceedingly high. There is general concordance among epidemiologic studies across North American, Asian, and European populations that the prevalence of radiographic OA in the knees, hips, and hands is quite low before age 45 and increases dramatically with aging, with most people having x-ray evidence of OA in at least one joint by the seventh decade. Symptomatic knee OA affects between 7% and 17% of those older than 44 years, with rates increasing with age; women have higher prevalence than men, and African Americans have higher prevalence than white Americans. Symptomatic hip OA is less prevalent than knee OA, with overall rates between 6.7% and 9.7% among those over age 44; as with knee OA, prevalence is higher among the elderly, women, and African Americans.1 Symptomatic hand OA affects at least 6.8% of those older than 25, occurring in women more than two-fold more frequently than in men. Hand OA may be less common in African Americans than in white Americans. Risk factors for the development of OA and for specific joint involvement have been extensively studied. Among nonmodifiable risk factors, the strongest is aging. This is true both for radiographic changes of OA and for symptomatic involvement. In addition, female sex is a risk for prevalence and severity of OA, especially after menopause. There is a significant heritable component, particularly for hand OA and hip OA. This component is estimated at 48 to 65% for so-called generalized OA characterized by osteophytes of the distal interphalangeal joints (Heberden nodes) or the proximal interphalangeal joints (Bouchard’s nodes). Modifiable OA risk factors may provide clues for preventive strategies. The most important of these is obesity, which alone confers an approximately three-fold increased risk for incident OA. Occupational and lifestyle activities that involve repeated trauma or excessive loading may be associated with increased risk for OA. These include chronic squatting, bending, and lifting such as by warehouse workers and laborers, who have increased knee involvement, and, classically, pneumatic drill operators who develop OA of the wrist and elbow. Significant trauma, such as major knee or ankle injury, is strongly associated with subsequent development of OA in the injured joint. Aberrant loading of joints is an important risk factor for the development and progression of OA. For example, excessive loading of the knee has been observed to result in significantly increased risk for progression to advanced knee OA. In addition, joint alignment is an important parameter in OA, and malalignment at the knee is among the strongest predictors of OA progression.2

PATHOBIOLOGY

Although degeneration of articular cartilage is a central common pathway in OA, multiple joint and periarticular tissues are compromised and contribute to clinical manifestations. Pathologic changes in synovium, ligaments, supporting musculature, and fibrocartilagenous structures such as the menisci in the knee are common.3 Unlike autoimmune arthritides, OA does not affect extra-articular organs. However, the chronic pain of OA involves both the peripheral and central nervous system (CNS) and OA-related disability degrades the general physical and mental health of the patient. Appreciation of the global effects of OA has important implications for current and future treatment approaches.

Tissues Central to the Osteoarthritis Process Cartilage

The hallmark of osteoarthritis is progressive deterioration of articular cartilage. Normal articular cartilage distributes loads across joint surfaces and allows for almost frictionless joint motion. These functions are furnished by the extracellular matrix, which accounts for more than 90% of the tissue volume and is organized by a network of collagen type II fibers, which provides tensile strength, entrapping aggrecan complexes, a proteoglycan that confers compressive stiffness and resilience (Fig. 262-1A and C). There is one major cell type, the chondrocyte, that synthesizes these matrix components. In mature cartilage, turnover of extracellular matrix molecules, particularly collagen type II, is slow. Cartilage change in OA begins with swelling of the matrix, then progresses through stages of surface roughening, fibrillation, fissuring, and eventually full-thickness erosion. These are accompanied by activation of chondrocytes to increase synthesis of proteolytic enzymes that degrade matrix.4 Matrix metalloproteinase-13 (MMP-13; collagenase-3) plays a central role in collagen type II degradation, while ADAMTS-4 and -5 (a disintegrin and metalloproteinase with thrombospondin motifs-4 and -5) proteases are important

CHAPTER 261  The Systemic Autoinflammatory Diseases  

GENERAL REFERENCES 1. Broderick L, De Nardo D, Franklin BS, et al. The inflammasomes and autoinflammatory syndromes. Annu Rev Pathol. 2015;10:395-424. 2. Henderson C, Goldbach-Mansky R. Monogenic autoinflammatory diseases: new insights into clinical aspects and pathogenesis. Curr Opin Rheumatol. 2010;22:567-578. 3. Jacobs Z, Ciaccio CE. Periodic fever syndromes. Curr Allergy Asthma Rep. 2010;10:393-404. 4. Kastner DL, Aksentijevich I, Goldbach-Mansky R. Autoinflammatory disease reloaded: a clinical perspective. Cell. 2010;140:784-790. 5. Wurster VM, Carlucci JG, Edwards KM. Periodic fever syndromes. Pediatr Ann. 2011;40:48-54. 6. Infevers database. A compendium of mutations associated with known autoinflammatory syndromes with links to relevant publications. http://fmf.igh.cnrs.fr/ISSAID/infevers/. Accessed January 20, 2015. 7. Berkun Y, Eisenstein EM. Diagnostic criteria of familial Mediterranean fever. Autoimmun Rev. 2014;13:388-390. 8. Gerfaud-Valentin M, Jamilloux Y, Iwaz J, et al. Adult-onset Still’s disease. Autoimmun Rev. 2014;13: 708-722.

1744.e1

9. Jesus AA, Goldbach-Mansky R. IL-1 blockade in autoimflammatory syndromes. Annu Rev Med. 2014;65:223-244. 10. Bachetti T, Ceccherini I. Tumor necrosis factor receptor-associated periodic syndrome as a model linking autophagy and inflammation in protein aggregation diseases. J Mol Med (Berl). 2014;92: 583-594. 11. Ter Haar NM, Frenkel J. Treatment of hereditary autoinflammatory diseases. Curr Opin Rheumatol. 2014;26:252-258. 12. Aksentijevich I, Masters SL, Ferguson PJ, et al. An autoinflammatory disease with deficiency of the interleukin-1-receptor antagonist. N Engl J Med. 2009;360:2426-2437. 13. Liu Y, Ramot Y, Torrelo A, et al. Mutations in proteasome subunit beta type 8 cause chronic atypical neutrophilic dermatosis with lipodystrophy and elevated temperature with evidence of genetic and phenotypic heterogeneity. Arthritis Rheum. 2012;64:895-907. 14. Zhou Q, Yang D, Ombrello A, et al. Early-onset stroke and vasculopathy associated with mutations in ADA2. N Engl J Med. 2014;370:907-916.

1744.e2

CHAPTER 261  The Systemic Autoinflammatory Diseases  

REVIEW QUESTIONS 1. Which of the following is not commonly associated with the development of AA amyloidosis? A. Hyper-IgD syndrome (HIDS) B. Familial Mediterranean fever (FMF) C. Tumor necrosis factor receptor–associated periodic syndrome (TRAPS) D. Muckle-Wells syndrome Answer: A  All of the listed periodic fever syndromes are commonly associated with the development of AA amyloidosis with the exception of hyperIgD syndrome, which generally has a benign course. 2. Patients with which monogenic autoinflammatory disease often have a history of flaring after routine immunizations? A. FMF B. HIDS C. TRAPS D. FCAS Answer: B  Of these syndromes, only HIDS has a substantiated association of flares with routine immunizations. Psychological stress is often cited as a trigger for flares in FMF and TRAPS, and cold exposure triggers a systemic inflammatory flare and hivelike skin lesions in FCAS. 3. A 30-year-old woman of Armenian and Jewish ancestry presents with a life-long history of unexplained febrile episodes. These attacks are variable in length, with the shortest lasting a few days and the longest lasting over 1 month. The fevers are accompanied by severe abdominal pain or pleuritic chest pain, periorbital edema, arthralgia, and a painful migratory erythematous rash. Corticosteroids ameliorate her symptoms. During pregnancy 7 years ago she was totally free of fevers, but she developed a severe attack in the postpartum period. She is currently not experiencing an attack, but has an erythrocyte sedimentation rate of 85 (Westergren), C-reactive protein of 100 mg/L, urine protein-to-creatinine ratio of 5.3, and serum creatinine of 2.5. Which of the following is most likely true? A. The patient probably has familial Mediterranean fever, and should undergo genetic testing for MEFV mutations, have a rectal biopsy to rule out amyloidosis, and commence treatment with intravenous colchicine. B. The patient has chronic atypical neutrophilic dermatosis with lipo­ dystropy and elevated temperature (CANDLE syndrome), with the T75M mutation in PSMB8, encoding a component of the immunoproteasome. C. The patient has a mutation at a cysteine residue in the extracellular domain of the p55 TNF receptor and possible amyloidosis. She should undergo a rectal biopsy and should be treated with etanercept or anakinra in an effort to normalize her acute phase reactants. D. The patient has a mutation in the NLRP3 (CIAS1) gene, and could be treated with anakinra, rilonacept, or canakinumab for neonatal-onset multisystem inflammatory disease (NOMID). Answer: C  Despite the demographic information that may point toward FMF, the presence of longer disease flares of longer than 7 days, periorbital edema, painful migratory rash, and remission of symptoms with pregnancy are all characteristic of the TNFR1-associated periodic syndrome (TRAPS). Patients with TRAPS and structure-disrupting mutations in TNFR1 have an elevated risk for amyloidosis. TRAPS does not have a predilection for a specific ethnic group or geographic location.

4. A 3-year-old boy of northern European ancestry presents to the autoinflammatory disease clinic for an initial evaluation. Per his mother’s report, he began experiencing febrile attacks at the age of 3 months that last 4 to 5 days and occur every 1 to 2 months. Associated symptoms include a nonpruritic macular rash and oral ulcers, and on three occasions he has had genital ulcers. He tends to have flares approximately 2 days after immunizations and after routine viral illnesses. There is no family history of febrile illnesses. He is treated with ibuprofen and acetaminophen and has been given a couple of courses of prednisolone without complete resolution of symptoms. He had been well before his visit, but his mother thinks he is starting to have a flare. On physical examination, he has conjunctival injection, cervical lymphadenopathy but no rashes, oral or genital ulcers, or arthritis. Laboratory studies reveal an ESR of 57 mm/ hour, CRP of 55 mg/L, and IgD level at the top of the normal range. Which of the following is most likely true? A. The patient probably has periodic fever with aphthous stomatitis, pharyngitis, and cervical adenitis (PFAPA). He should be given either 1 mg/kg of prednisolone at the onset of symptoms or anakinra 2 mg/ kg SC for 1 to 2 days at the onset of symptoms. B. The patient has Behçet’s disease and should consider colchicine for his oral ulcers. He should undergo HLA typing to assess for the presence of HLA-B51 or other Behçet’s-associated MHC findings. C. The patient has hyperimmunoglobulinemia D syndrome and should initiate therapy with anakinra at the time of flares or weekly etanercept. He should be tested for mutations in the MVK gene. D. The patient has cyclic neutropenia and should be tested for mutations in the ELA-2 gene. He should also have weekly CBC drawn to assess for periodic neutropenia. Answer: C  Despite the oral and genital ulcers that might suggest Behçet’s disease, the history of flares after vaccinations or viral illness, the pattern and age of onset, and the ethnic background more strongly suggest that the patient has hyperimmunoglobulinemia D syndrome. IgD levels can be normal in this disease, and a normal IgD should not rule out this diagnosis, which can be confirmed by finding a heterozygous mutation in the mevalonate kinase gene. 5. A 16-year-old high school student is evaluated in the emergency department for 18 hours of abdominal pain, pleurisy, and fever. At that time, he had diffuse rebound tenderness, temperature of 102.5° F (39.2°degrees C), and WBC of 17,000/mm3. He was admitted to general surgery and after 12 hours of persistent symptoms, he was taken to the operating room for exploratory surgery. A normal-appearing appendix was laparascopically removed. Postoperatively, the fever persisted and he was noted to have a left pleural effusion. Additional history reveals that he has had similar episodes of febrile illness in the past. On physical examination, he has no rash but has moderate abdominal tenderness with guarding and a left pleural friction rub. Laboratory data reveal an ESR of 80 mm/hour and a negative ANA and RF. While waiting for the results of genetic testing, which of the following medications is most appropriate to start at this time? A. Indomethacin, 50 mg orally three times daily B. Prednisone, 40 mg orally daily C. Colchicine, 0.6 mg orally twice daily D. Anakinra, 100 mg subcutaneously daily E. Acetylsalicylic acid, 1000 mg orally daily Answer: C  The duration of fevers, association with pleural and peritoneal inflammation, and lack of other localizing symptoms are most consistent in this case with familial Mediterranean fever. Colchicine is effective in the prophylaxis of future attacks in FMF. Indomethacin and prednisone can be of some value in terminating attacks but should be reserved for refractory cases. Anakinra can be effective in the treatment of refractory FMF, but there is no need to use this agent in patients who respond to colchicine.

CHAPTER 262  Osteoarthritis  

Normal Joint

1745

Osteoarthritic joint

Synovial membrane hyperplasia, inflammation, fibrosis

Synovial membrane

Cartilage

Leukocyte

Osteophyte Collagen fibrils

Subchondral bone

Proteoglycan complexes

A

Osteophyte

Roughening and loss of cartilage matrix

Cartilage

B

Thickened subchondral bone

Normal

Osteoarthritis

Synovial membrane

Synovial membrane

Cartilage Cartilage

Bone

Bone

C

D

FIGURE 262-1.  Pathologic features of osteoarthritic joint tissues. A, Features of a normal adult synovial joint. Healthy adult articular cartilage is characterized by a smooth surface and extracellular matrix (ECM) composed of a collagen type II fibrillar network and large proteoglycan complexes. The ECM is produced and maintained by the cellular components of cartilage, chondrocytes. The subchondral bone consists of a thin cortical layer and underlying trabecular bone. The synovial membrane lines the joint capsule and attaches at the cartilage-bone interface. In the normal state, it consists of a lining layer 1 or 2 cells thick, with underlying vascularized loose connective tissue. B, Typical changes to tissues seen in osteoarthritis (OA). Enzymatic activities (ADAMTS-4,5 and MMP-13 in particular) cleave proteoglycan and collagen components of the ECM, leading to loss of these molecules  from the matrix. As the process advances, the articular cartilage thins and fibrillates and eventually fissures down to the underlying bone are seen. Simultaneously, a remodeling response in the bone is observed. Thickening of the cortical subchondral bone layer occurs, and new bone growth at the margins appears as osteophytes. The synovial membrane changes observed in OA patients include lining layer hyperplasia, inflammation in the form of leukocyte infiltration, and fibrosis which can be seen to varying degrees. Photomicrographs of human joint tissues showing these features are depicted in C (normal tissues) and D (OA tissues). (C and D courtesy of Edward F. DiCarlo, MD. Hospital for Special Surgery, New York, NY).

for loss of aggrecan, but other enzymes participate (see Fig. 262-1B and D). Concomitantly, the activated chondrocytes proliferate to form clonal clusters and produce inflammatory mediators, including interleukin-1α and β, (IL1α and IL-1β), IL-6, tumor necrosis factor-α (TNF-α) and nitric oxide (NO), which accelerate the degradative cycle and stimulate chondrocyte apoptosis. Thus, both cellular and molecular components of cartilage are lost as the process progresses.5

chanical loading. This may result in thinning (attrition) and reduced bone density, leading to subchondral cyst formation in early disease, but progresses to subchondral sclerosis as bone formation outpaces resorption (see Fig. 262-1B and D).6 Remodeling at joint margins and entheses results in osteophytes (bone spurs). An important role for the growth factors transforming growth factor-β (TGF-β) and bone morphogenetic protein-2 (BMP-2) in driving osteophyte formation has been demonstrated in animal models.

Bone

Synovium

The cortical bone underlying articular cartilage (subchondral bone) supports load-bearing and transmits mechanical signals to articular chondrocytes. In OA, there is increased remodeling, likely in response to abnormal biome-

Synovial involvement in OA is more variable than in rheumatoid arthritis, but low-grade synovitis, characterized by infiltration of macrophages and lymphocytes, increased vascularity, synovial lining hyperplasia, and fibrosis,

1746

CHAPTER 262  Osteoarthritis  

Pathogenic factors

Tissue reaction

Altered loading • obesity • joint injury/trauma • malalignment • overuse

Cartilage Chondrocyte activation • proteases • cytokines

Altered matrix • aging • metabolic derangement • genetic defects

Bone Aberrant bone remodeling • growth factors • cytokines

Secondary factors Inflammation • synovitis • adipokines

Weakening of support structures • menisci • ligaments • muscles

Synovium Inflammation Fibrosis Muscle Altered strength Joint tissue destruction & pain

Tendons/ligaments Matrix disruption Nerves Altered afferent function • proprioception • vibratory • nociception

Joint dysfunction

Disability

FIGURE 262-2.  Schematic of the pathophysiology of osteoarthritis. A variety of stimuli can alter biomechanical loading patterns of joint tissues or exert biologic pressure on the extracellular matrix. These result in specific responses in the various joint tissues. Aided by secondary factors, these lead to tissue degeneration, pain, and ultimately joint dysfunction.

is common and occurs in up to 75% of patients (see Fig. 262-1B and D). Although this synovial reaction is likely a secondary response to molecular breakdown products present in the OA joint, it results in release of cytokines (e.g., IL-6, TNF-α), chemokines (IL-8, monocyte chemoattractant protein 1 [MCP-1], C-C motif chemokine 19 [CCL19]), enzymatic mediators of cartilage catabolism, and growth factors involved in bone remodeling, and when present it is associated with more severe symptoms and possibly more rapid progression of cartilage degradation.7

Menisci and Ligaments

Traumatic and athletic injuries to the menisci and ligaments are known risk factors for incident OA. However, meniscal and ligamentous abnormalities are common in patients with OA even without injury and are detectable in 80% and 30% (respectively) of patients undergoing magnetic resonance imaging (MRI). These structures are sensitive to the same inflammatory and enzymatic mediators that promote cartilage deterioration in OA, and their compromise promotes degenerative injuries, accelerated cartilage erosion, joint instability, and mechanical symptoms (i.e., locking or catching).

Muscles

Periarticular muscle weakness and atrophy are characteristic of hip and knee OA. Such weakness is associated with altered gait kinematics and pathologic joint loading, but it remains unclear whether this is a cause or an effect of OA.

Peripheral and Central Nervous System

Somatosensory deficits associated with neuromuscular function, including proprioceptive and widespread vibratory deficits, have been described.8 Also, patients with OA exhibit signs of peripheral and CNS sensitization in the form of hyperalgesia and allodynia. Inflammatory mediators (e.g., MCP-1) and growth factors (e.g., nerve growth factor [NGF]) have been implicated in pain pathways in OA.9 Additional work is necessary to fully understand the mechanisms leading to symptomatic OA, but pain in OA is clearly multifactorial, involving the peripheral and the central nervous systems in addition to joint tissues and inflammation.

Pathogenic Factors (Fig. 262-2) Biomechanics

Aberrant loading of joints mediates evolution of structural joint degeneration and may contribute to initiation of the process. This is evident in the high risk that malalignment of the knee carries for knee OA progression. Weightbearing regions of OA joints bear greater loads during ambulation than normal, and these loading patterns have been shown to be predictive of subsequent progression of lower extremity joint disease longitudinally. Mechanistically, abnormal biomechanical loading promotes pathologic activation of chondrocytes and bone remodeling, and may be precipitated by joint injury, congenital dysplasias, malalignment, joint instability (e.g., in the setting of aging or heritable connective tissue diseases), and abnormal neuromuscular control (e.g., neuropathic diseases), each of which is a risk factor for OA.

Metabolic Factors

Several metabolic conditions, including alkaptonuria, acromegaly, hemochromotosis, and the metabolic syndrome, among others, predispose individuals to early OA, though they have different mechanisms and disease patterns. In the metabolic syndrome, obesity increases loads on weightbearing joints, but its association with OA of non–weight-bearing joints in the hands suggests a nonmechanical etiology as well. A possible mechanism may be related to the overproduction of adipokines, IL-6, and leptin leading to chronic low-grade systemic inflammation that may potentiate molecular deterioration of joint tissues and contribute to joint symptoms.10

Genetic Factors

Rare single-gene defects may lead to early or aggressive OA. Heritable defects in the collagen II gene have occasionally been described in families with chondrodysplasias, as have lubricin defects. However, variations in multiple genes are likely implicated in the majority of OA cases. Genome-wide association studies have identified polymorphisms in genes involved in proliferation, skeletal development (e.g., GDF5), and regulation of body weight associated with risk for radiographic knee and hip OA.11 Additional polymorphisms have been associated with pain in OA (e.g., PACE4 and TRPV1).

CHAPTER 262  Osteoarthritis  

Aging

Aging-related molecular and cellular changes likely contribute to OA pathogenesis.12 Modifications of extracellular matrix components, such as accumulation of advanced-glycation end products, and carboxylation associated with oxidative-stress, occur with advancing age. These can alter protein folding, weaken tissues, and increase susceptibility to proteolytic cleavage. Agerelated changes to chondrocytes and other cells include the “senescenceassociated secretory phenotype” associated with decreased proliferative capacity but increased secretory activity that may promote abnormal chondrocyte responses to injury and aberrant loading. Nonetheless, the link between aging-related changes and symptoms remains poorly understood.

CLINICAL MANIFESTATIONS OF OSTEOARTHRITIS

Symptoms

Pain is the most prominent symptom of OA. Although often limited to affected joints, it can become widespread over time.13 The quality of pain experienced is variable, ranging from “aching” joint pain to less localized periarticular or radiating pain. OA pain is typically worsened by joint use while stiffness is exacerbated by prolonged inactivity. Morning stiffness may occur but is brief, lasting less than 30 minutes. Other symptoms include joint instability, limitation of motion, locking, and a grinding feeling with motion. Symptom severity varies over time, but with advanced disease, pain becomes persistent and can disturb sleep.

Patterns of Joint Involvement

The joint pattern may help distinguish OA from other forms of arthritis (Fig. 262-3).

Lower Extremities

The large weight-bearing joints (knees and hips) are most commonly affected. Knee OA encompasses any of the three compartments. The medial compartment is involved in the majority of patients and may lead to varus

1747

(bow-legged) deformity. Lateral compartment OA may result in valgus (knock-kneed) deformity, and patellofemoral OA typically causes pain exacerbated by descending stairs. Meniscal or ligamentous degeneration often accompanies OA and exacerbates knee instability. Effusions, synovial (Baker’s) cysts, and anserine or prepatellar bursitis may be present and cause additional pain. Hip OA often begins with restricted internal rotation and progresses to limited motion in all directions and limb length discrepancy. Pain is felt in the groin area and may radiate to the anterior thigh and knee and be confused with knee pain. It also must be differentiated from lateral thigh pain, which more likely originates from other structures (e.g., trochanteric bursa, illiotibial band, lumbar spine). In the foot, OA typically involves the first metatarsophalangeal joint, resulting in bunion deformity.

Spine

Spinal OA typically involves the lumbar and cervical regions, affecting the apophyseal (facet) and uncovertebral (joints of Luschka) joints. Low-grade inflammation and bone remodeling can lead to local pain, and nerve root compression by osteophytes causes radicular, radiating pain. Degenerative disc disease often coexists with spinal OA, and together they contribute to spinal stenosis causing muscle weakness, paresthesias, and numbness. In severe cases, cord impingement with myelopathy may result.

Upper Extremities

The small joints of the hands are most commonly affected, specifically the distal and proximal interphalangeal joints (DIPs and PIPs) and first carpometacarpal (CMC) joints. Osteophytosis of DIPs and PIPs leads to bony, palpable Heberden and Bouchard nodes, respectively. This pattern is more common in white women and is termed primary generalized OA. Patients may experience difficulty grasping, opening jars, buttoning clothes, and turning doorknobs. Erosive or inflammatory OA is a less common but distinct subset in which erosions develop in the DIPs and PIPs, and the patient experiences repeated episodes of acute inflammatory symptoms.

Cervical spine

Lumbar spine

Hips

Distal and proximal interphalangeal joints

Knees

First metatarsophalangeal joints (Photo courtesy Augustine Manadan, MD, John Stroger Hospital of Cook County, Chicago, IL)

FIGURE 262-3.  Commonly affected joints in osteoarthritis. Joints in which symptomatic osteoarthritis typically develops are depicted in the whole-body drawing in the center. The distal and proximal interphalangeal joints of the hands characteristically exhibit palpable bony bumps (right lower panel). Synovial effusions and bony outgrowths, or osteophytes, may be visualized in the knee (left upper panel), and radiographic involvement of the knee (left lower panel) or of the hip (right upper panel) shows characteristic features of joint space narrowing, osteophytes (black arrows), and subchondral sclerosis (blue arrows).

1748

CHAPTER 262  Osteoarthritis  

DIAGNOSIS

The diagnosis of OA depends primarily on the clinical presentation; unlike other rheumatic diseases, imaging and laboratory analysis have relatively small roles in diagnosis. Because OA primarily affects tissues of the joint and does not involve systemic inflammation, laboratory tests are more useful for excluding competing diagnoses than for establishing an OA diagnosis.

Physical Examination

When cartilage surfaces become roughened, crepitus (palpable or audible crackling) may be detected by physical examination. As cartilage is further compromised, small fragments can dislodge and, if sufficiently large, may restrict joint motion and cause locking. Osteophytes can be palpated in superficial joints as bony projections and may result in deformities. Their growth is sometimes inflammatory, with erythema, tenderness, and swelling. Heberden and Bouchard nodes of the DIP and PIP joints, respectively, are typical, but squaring of the first CMC joint and palpable knee osteophytes are also common. These deformities may eventually restrict joint range of motion. Classically, severe joint inflammation is not appreciated; however, effusions are common and may cause mild joint warmth.

Laboratory Evaluation Blood Tests

OA is limited to joints; thus, tests of systemic inflammation and those that assess critical organ function are generally normal. These include the sedimentation rate, conventional C-reactive protein, and other acute phase reactants, as well as the blood count and the comprehensive metabolic panel. Nonetheless, OA is prevalent among the elderly and concomitant diseases may confound the interpretation of laboratory testing. Tests for autoantibodies are unremarkable, though low-titer detection of nonspecific rheumatoid factors and antinuclear antibody (ANA) may be seen in the normal aging population.

Synovial Fluid

When sampled, synovial fluid total leukocyte counts are typically less than 1500 to 2000 cells/mm3 with a predominance of lymphocytes rather than neutrophils. Incidental findings of crystals, such as calcium pyrophosphate dihydrate, or of cartilage fragments may sometimes be observed.

Biomarkers

There has been an aggressive search for macromolecules measurable in blood, synovial fluid, or urine that provide prognostic or diagnostic value in OA. A variety of macromolecules, typically breakdown or cleavage products of cartilage or bone matrix, have been identified that have a statistical correlation with OA progression or pain. The OA Initiative is an ongoing longitudinal study of several thousand individuals, spearheaded by the National Institutes of Health, to identify novel body fluid and imaging biomarkers that may provide predictive information about OA onset and progression. But no biomarkers have yet been demonstrated to be useful for clinical evaluation.

Imaging Radiography

Conventional radiography is the mainstay for imaging OA. Characteristic features include narrowed joint space, osteophytes, and subchondral bone sclerosis (see Fig. 262-3). Not all patients have all three features, and there is an imperfect relationship between radiographic appearance and clinical symptoms. Disease progression can be monitored by longitudinal imaging, both by qualitative grading and by quantitative assessment of joint space narrowing.

Magnetic Resonance Imaging

MRI provides the most detailed images of joint structures and, in contrast to radiography, detects subtle defects of articular cartilage.14 In addition, MRI detects subchondral bone marrow lesions that are associated with symptomatic disease. MRI remains predominantly a research tool in OA, where it has been helpful in identifying soft tissue pathologic conditions and measuring early cartilage pathology; clinically, it has little role in routine OA evaluation and management. MRI is often used in OA to exclude other potential sources of pain, such as degenerative menisci, ligamentous tears, and other intraarticular pathology.

Ultrasonography

Although operator dependent, careful ultrasonography reveals outstanding details of joint structure and does not expose the patient to ionizing radiation. It may identify osteophytes undetectable by standard radiography, and articular cartilage lesions can often be observed. Its primary use, though, may be to detect local synovitis that is prevalent in OA; it remains unclear whether it can play a significant role in diagnosis.

Radionuclide Imaging

Whereas radioisotope scanning formerly had utility in OA, it has been largely supplanted by more sensitive and specific modalities for detecting articular pathology.

TREATMENT  Many physicians persist in counseling their symptomatic patients that there is nothing to be done, and that OA is an inevitable sign of “getting old.” Although no interventions have been demonstrated to alter the natural history of structural joint degeneration, symptomatic OA is not inevitable, and many strategies provide relief and maintain function among those with even advanced OA.15 Patient education and support are critical; self-help programs alone have been shown to improve outcomes in OA. Patients should be provided thorough information about the natural course of OA, their role in disease management, and appropriate expectations. In general, the goals of OA therapy are similar to the treatment of any disease—to maintain or restore function, relieve symptoms, and prevent disease progression.

Maintenance of Function

Strategies to retain function and independence in patients with OA include ambulatory assistive devices, such as canes and walkers, which provide stability in addition to reducing loading across arthritic joints. Motorized carts can assist individuals with severe knee or hip OA to retain independence in the community. Physical therapy can help to retain strength and range of motion, and occupational therapy can provide customized assist devices and braces. Whenever possible, counseling on weight loss for overweight or obese patients should be provided as weight loss can improve both pain and function. A1 

Symptom Palliation

A variety of strategies can provide effective pain palliation in OA. These include physical measures, medical therapy, and surgical interventions.16

Physical Measures

Exercise is especially important in OA, both to improve function and to palliate pain.17 Exercises aimed at strengthening muscles surrounding affected joints are of value physiologically and have been consistently demonstrated to provide significant pain relief. Patients should be encouraged to exercise regularly and may benefit from physical therapy for instruction in appropriate strength training and to improve and maintain range of motion. During painful flares and immediately after exertion, application of heat or ice to affected joints can be useful. Pain may be mediated by aberrant biomechanical loading. Therefore, canes and walkers, which significantly reduce loads across the knee during gait, can reduce pain and improve stability. Similarly, unloading knee braces, when tolerated, may provide palliation for knee OA.

Medication

Most patients will need more pain relief than is provided by physical measures. Among the elderly, choice of medication is often influenced by comorbidities. Topical agents may reduce risk for of systemic adverse effects and are appropriate when only a few joints are symptomatic. Topical capsaicin has been approved for knee OA; its use requires frequent application and careful handwashing after contact. Topical nonsteroidal anti-inflammatory drugs (NSAIDs) are available, including salicylic acid and diclofenac. When physical measures and topical agents are insufficient, oral analgesics are used. Many specialty organizations, including the American College of Rheumatology,18 suggest that acetaminophen may be beneficial, especially among patients with contraindications to NSAIDs such as renal dysfunction or cardiac disease. Nonetheless, acetaminophen may not be effective for longterm analgesia and chronic use carries its own potential toxicities, including liver damage and hypertension. Therefore, acetaminophen may be most properly used for short-term flares of OA pain, typically lasting no more than a few weeks. Approved alternatives to acetaminophen include tramadol and opiates, which though demonstrated to relieve OA pain, also substantially increase morbidity among the elderly, especially the risk for traumatic falls. Finally, neuroactive agents are widely used for OA pain. One such medication, duloxetine hydrochloride, a serotonin and norepinephrine reuptake inhibitor (SNRI), received U.S. Food and Drug Administration approval in 2010 for the treatment of musculoskeletal pain, including OA, based on positive results in clinical trials. A2 

1749

CHAPTER 263  BURSITIS, TENDINITIS, AND OTHER PERIARTICULAR DISORDERS  

NSAIDs have been demonstrated to be effective for OA pain and may maintain efficacy for years. A3  These medications remain the mainstay of OA medical therapy, but are not an option for many patients with renal, cardiac, or gastrointestinal conditions. Proton pump inhibitors or misoprostol can provide gastric protection in middle-aged and elderly patients and those at risk for gastrointestinal bleeding. Cyclooxygenase-2 inhibitors also may be used; in the United States, celecoxib is the only representative of this class available. Six weeks of low-dose oral prednisolone (7.5 mg daily) is also effective, and its benefits can persist for at least another six weeks. A4 

Intra-articular Therapy

Intra-articular glucocorticoids may provide short-term relief of OA pain, often lasting months. A5  Their use is limited to three or four times per year in any single joint because of theoretical concerns of toxicity to articular cartilage. A variety of hyaluronan derivatives are available for injection to relieve OA pain, but controversy exists as to whether they are more effective than placebo. Originally developed to supplement viscosity of synovial fluid in an attempt to improve articular lubrication, residence time in the joint is too brief to have this effect.

Surgical Approaches

Joint replacement surgery restores function and relives pain in the majority of patients and is the most important therapeutic advance in OA treatment to date (Chapter 276). It should be reserved for those in whom pain or joint dysfunction significantly limits normal life activities despite optimal medical and physical management. The presence of advanced structural degeneration of the joints alone, without severe symptoms, should not be an indication for arthroplasty. Knees and hips are most frequently replaced, but good results are now obtained in other joints as well. The durability of joint prostheses is limited, so joint replacement surgery should be delayed in younger patients when practical. Aside from total joint replacement, there are a variety of temporizing strategies that may be used in joints that have less severe structural degeneration, including realignment osteotomy in the knee and hemiarthroplasty.

Delay of Disease Progression

There has been extensive effort to identify disease-modifying OA drugs (DMOADS) that would retard disease progression and affect OA morbidity. To date, no true DMOADs have been identified, although investigation continues into agents targeting cartilage and joint tissue metabolism and inflammation. There remains optimism that as the mechanism of joint degeneration becomes more fully elucidated, rational drug discovery may identify effective DMOADs. In addition, tissue engineering approaches and mesenchymal stem cell technology may permit the development of functional joint tissue replacement in the future. Current cartilage replacement techniques are not indicated for OA treatment, but are restricted to patients with isolated chondral defects.

extrusion, and maceration). MRI-identified subchondral bone marrow lesions are associated with pain severity and progressive cartilage loss.

PREVENTION

No current treatments substantially alter OA progression, but some strategies reduce the risk for development of OA and ameliorate symptomatic progression. Obesity may be the most modifiable of the strong OA risk factors. Weight loss in adulthood reduces the risk for incident radiographic and symptomatic OA and reduces pain severity in patients who already have OA. A6  Exercise is an important component of weight strategies and ameliorates pain, but specific types of exercise have not yet shown consistent preventive effects. Strategies to decrease joint injuries in young athletes are critical to reducing post-traumatic OA,19 and proper conditioning has been shown to reduce knee injuries among female soccer players. A7  New insights into the biology and biomechanics of OA in the coming years may be expected to yield novel strategies to prevent the onset and progression of the disease.

Grade A References A1. Bliddal H, Leeds AR, Stigsgaard L, et al. Weight loss as treatment for knee osteoarthritis symptoms in obese patients: 1-year results from a randomised controlled trial. Ann Rheum Dis. 2011;70: 1798-1803. A2. Chappell AS, Ossanna MJ, Liu-Seifert H, et al. Duloxetine, a centrally acting analgesic, in the treatment of patients with osteoarthritis knee pain: a 13-week, randomized, placebo-controlled trial. Pain. 2009;146:253-260. A3. Chou R, McDonagh MS, Nakamoto E, et al. Analgesics for osteoarthritis: an update of the 2006 Comparative Effectiveness Review. Agency for Healthcare Research and Quality (US); 2011. Report No.: 11(12)-EHC076-EF. A4. Abou-Raya A, Abou-Raya S, Khadrawi T, et al. Effect of low-dose oral prednisolone on symptoms and systemic inflammation in older adults with moderate to severe knee osteoarthritis: a randomized placebo-controlled trial. J Rheumatol. 2014;41:53-59. A5. Yavuz U, Sokucu S, Albayrak A, et al. Efficacy comparisons of the intraarticular steroidal agents in the patients with knee osteoarthritis. Rheumatol Int. 2012;32:3391-3396. A6. Messier SP, Mihalko SL, Legault C, et al. Effects of intensive diet and exercise on knee joint loads, inflammation, and clinical outcomes among overweight and obese adults with knee osteoarthritis: the IDEA randomized clinical trial. JAMA. 2013;310:1263-1273. A7. Steffen K, Emery CA, Romiti M, et al. High adherence to a neuromuscular injury prevention programme (FIFA 11+) improves functional balance and reduces injury risk in Canadian youth female football players: a cluster randomised trial. Br J Sports Med. 2013;47:794-802.

GENERAL REFERENCES For the General References and other additional features, please visit Expert Consult at https://expertconsult.inkling.com.

Biomechanically Active Approaches

OA progression is at least partly mediated by aberrant loading of joints, so improving loading patterns should affect structural progression. At present, no approaches to alter loading have yet been shown to substantially affect OA progression, but specialized footwear, gait modifications, and mechanically derived exercise regimens are under active investigation.

Complementary and Alternative Approaches

Similar to what occurs with other chronic pain conditions, the overwhelming majority of patients with OA try complementary approaches (Chapter 39). Among the more popular are glucosamine, chondroitin, and acupuncture. None has been clearly demonstrated to substantially retard joint degeneration and independently funded trials have been negative, but many patients feel pain improvement. In any blinded study of OA pain, a substantial placebo response is typically observed. Many complementary approaches have been systematically studied, and controversy remains regarding the incremental pain relief provided by these modalities over that obtained with placebo. Regardless, many of these approaches can be safely used by individual patients who derive relief from them.

PROGNOSIS

Once structural joint damage is present, it is likely to progress, although at variable rates. Slowly progressive structural disease in the absence of severe symptoms may never require surgical intervention, whereas rapidly progressive symptomatic disease might prompt early intervention. The causes of this variability remain unclear, but several factors may contribute to an individual’s prognosis. Female gender, obesity, and pain severity are associated with both the incidence and progression of radiographic knee OA, and joint malalignment (varus or valgus) is a strong predictor of progression. Symptomatically, MRI or sonographically detected synovial thickening or effusion may predict progressive cartilage defects and pain severity. In the hip, joint shape is predictive of radiographic progression, as is meniscal pathology (tears,

263  BURSITIS, TENDINITIS, AND OTHER PERIARTICULAR DISORDERS AND SPORTS MEDICINE JOSEPH J. BIUNDO

DEFINITION

An array of painful and sometimes disabling musculoskeletal syndromes exist that are not articular in origin but arise from tendons and bursae. These conditions are referred to by various names, in addition to tendinitis and bursitis, including the terms nonarticular rheumatism, soft tissue diseases, regional rheumatic pain syndromes, overuse syndromes, and repetitive use syndromes (Tables 263-1 and 263-2). These entities are often ignored, misdiagnosed as arthritis, or attributed to the aging process; awareness of the existence of these conditions and knowledge of basic musculoskeletal anatomy (Figs. 263-1 and 263-2) are the fundamental requirements for diagnosis. This knowledge is coupled with brief but specific physical diagnosis techniques. The accurate diagnosis and successful treatment of these conditions is gratifying to the clinician because many people can be relieved of their chronic painful syndromes. Various terms regarding tendon injuries are used and may be confusing. The main term used is tendinitis. Tendinosis has been proposed as the correct terminology because there are degenerative changes in the tendon but very few inflammatory cells. In addition, fatty mucoid degeneration and hyaline

CHAPTER 262  Osteoarthritis  

GENERAL REFERENCES 1. Neogi T, Zhang Y. Epidemiology of osteoarthritis. Rheum Dis Clin North Am. 2013;39:1-19. 2. Chapple CM, Nicholson H, Baxter GD, et al. Patient characteristics that predict progression of knee osteoarthritis: a systematic review of prognostic studies. Arthritis Care Res. 2011;63:1115-1125. 3. Loeser RF, Goldring SR, Scanzello CR, et al. Osteoarthritis: a disease of the joint as an organ. Arthritis Rheum. 2012;64:1697-1707. 4. Xia B, Di C, Zhang J, et al. Osteoarthritis pathogenesis: a review of molecular mechanisms. Calcif Tissue Int. 2014;95:495-505. 5. Liu-Bryan R, Terkeltaub R. Emerging regulators of the inflammatory process in osteoarthritis. Nat Rev Rheumatol. 2015;11:35-44. 6. Burr DB, Gallant MA. Bone remodelling in osteoarthritis. Nat Rev Rheumatol. 2012;8:665-673. 7. Scanzello CR, Goldring SR. The role of synovitis in osteoarthritis pathogenesis. Bone. 2012;51: 249-257. 8. Roos EM, Herzog W, Block JA, et al. Muscle weakness, afferent sensory dysfunction and exercise in knee osteoarthritis. Nat Rev Rheumatol. 2011;7:57-63. 9. Malfait AM, Schnitzer TJ. Towards a mechanism-based approach to pain management in osteoarthritis. Nat Rev Rheumatol. 2013;9:654-664. 10. Zhuo Q, Yang W, Chen J, et al. Metabolic syndrome meets osteoarthritis. Nat Rev Rheumatol. 2012;8:729-737.

1749.e1

11. Reynard LN, Loughlin J. The genetics and functional analysis of primary osteoarthritis susceptibility. Expert Rev Mol Med. 2013;15:e2. 12. Loeser RF. Aging processes and the development of osteoarthritis. Curr Opin Rheumatol. 2013; 25:108-113. 13. Hunter DJ, Guermazi A, Roemer F, et al. Structural correlates of pain in joints with osteoarthritis. Osteoarthritis Cartilage. 2013;21:1170-1178. 14. Braun HJ, Gold GE. Diagnosis of osteoarthritis: imaging. [Review]. Bone. 2012;51:278-288. 15. Osteoarthritis: Care and Management in Adults. NICE Clinical Guidelines, No. 177. London: National Clinical Guideline Centre; 2014. 16. McAlindon TE, Bannuru RR, Sullivan MC, et al. OARSI guidelines for the non-surgical management of knee osteoarthritis. Osteoarthritis Cartilage. 2014;22:363-388. 17. Golightly YM, Allen KD, Caine DJ. A comprehensive review of the effectiveness of different exercise programs for patients with osteoarthritis. Phys Sportsmed. 2012;40:52-65. 18. Hochberg MC, Altman RD, April KT, et al. American College of Rheumatology 2012 recommendations for the use of nonpharmacologic and pharmacologic therapies in osteoarthritis of the hand, hip, and knee. Arthritis Care Res (Hoboken). 2012;64:465-474. 19. Bennell K, Hunter DJ, Vicenzino B. Long-term effects of sport: preventing and managing OA in the athlete. Nat Rev Rheumatol. 2012;8:747-752.

1750

CHAPTER 263  BURSITIS, TENDINITIS, AND OTHER PERIARTICULAR DISORDERS  

TABLE 263-1  MUSCULOSKELETAL CONDITIONS BY ETIOLOGY TENDINITIS

TENDON RUPTURE

Rotator cuff

Rotator cuff

Subacromial

BURSITIS

Bicipital

Bicipital

Olecranon

Volar flexor

Quadriceps

Trochanteric

de Quervain

Patellar

Ischial

Patellar

Posterior tibialis

Iliopsoas

Posterior tibialis

Achilles

Pes anserine

Achilles

Prepatellar

Epicondylitis

Retrocalcaneal

TABLE 263-2  TENDINITIS AND BURSITIS CONDITIONS BY REGION

EPIDEMIOLOGY

The incidence of the nonarticular syndromes of bursitis and tendinitis is high. They are more common than both rheumatoid arthritis (RA) and systemic lupus erythematosus (SLE). For example, the incidence of shoulder pain, largely a result of rotator cuff tendinitis and rotator cuff tear, was approximately 20% in a population older than 70 years of age.

SHOULDER Rotator cuff tendinitis Rotator cuff tear Bicipital tendinitis Subacromial bursitis Adhesive capsulitis

DIAGNOSIS

ELBOW

A precise history is needed to identify the conditions present, and more than one syndrome can occur concomitantly. A working knowledge of regional anatomy and an approach that uses a regional differential diagnosis will help in obtaining a specific diagnosis. A complete neuromusculoskeletal examination should be performed, emphasizing careful palpation, passive range of motion (ROM), and active ROM alone or sometimes with resistance. Systemic and infectious causes must be considered. Diagnostic ultrasonography and magnetic resonance imaging (MRI) are sometimes useful in confirming a diagnosis.

Olecranon bursitis Medial epicondylitis Lateral epicondylitis WRIST AND HAND de Quervain’s tenosynovitis Volar flexor tenosynovitis Ganglion HIP Trochanteric bursitis Iliopsoas bursitis Ischial bursitis Coccydynia

TREATMENT 

KNEE

Treatment of tendinitis and bursitis includes use of nonsteroidal antiinflammatory drugs (NSAIDs), relative rest of the injured site, stretching and strengthening exercises, friction massage, use of modalities (heat, ice, and ultrasound), splinting, corticosteroid injections, A1  and surgery. A comprehensive management of these regional syndromes should be undertaken, rather than relying on oral medications alone. The causative aspects should be evaluated, and activity modification should be advised as needed. The goals of therapeutic exercise are to increase flexibility by stretching, increase muscle strength by resistive exercises, and improve muscle endurance by some repetitive regimen. Caution should be exerted in performing corticosteroid injections; the injections should not be placed into the tendon proper, but rather into the peritendinous sheath. The injected solution should be placed beneath the subcutaneous tissue, to avoid skin and subcutaneous fat atrophy, and

Prepatellar bursitis Pes anserine bursitis Popliteal cyst (Baker’s cyst) Patellar tendinitis Patellar/quadriceps tendon tear ANKLE AND FOOT Achilles tendinitis Achilles tendon tear Posterior tibial tendinitis Posterior tibial tendon tear Retrocalcaneal bursitis Plantar fasciitis

Subacromial bursa

features occur in these tendon syndromes. These tendon conditions are described by some as a tendinopathy because use of this term avoids the need to decide whether inflammation is a factor. Also, tendons may rupture or tear, partially or completely. The term tendon insufficiency is used when the tendon is stretched or is partially or even completely torn. The terms tenosynovitis and peritendinitis refer to an inflammatory response of the tenosynovium or peritendon, respectively. Tendon syndromes are basically “overuse” injuries. Tendinitis may occur when the tendon repeatedly bears more load than it can withstand. This may result from excessively high loads across normal tendons or from normal loads across degenerated tendons. In addition to load and repetitiveness, tendon changes resulting from immobility and from aging may play a role, as may the use of certain medications such as fluoroquinolones and corticosteroids.1 Bursae are closed sacs lined by a synovial membrane and serve as a cushion. They are located between tendon and bone, tendon and tendon, or bone and skin and allow smooth gliding between these structures. A bursa, which normally has a small amount of bursal fluid, can become inflamed from trauma or overuse, or become infected, producing a bursitis. When this occurs, some swelling and pain of the bursa may be present.

Acromion

Supraspinatus muscle

Greater tubercle of humerus Deltoid muscle Scapula

A

Glenohumeral joint

B

FIGURE 263-1.  Relationship of subacromial bursa (shown in blue) to supraspinatus muscle and acromion process. A, In the position of adduction of the humerus. To show this bursa more clearly, the synovial membrane of the glenohumeral joint is not shown in blue. B, In the position of abduction of the humerus, the acromion impinges on the subacromial bursa  and the insertion of the supraspinatus tendon. (From Polley  HF, Hunder GG, eds. Rheumatologic Interviewing and Physical Examination of the Joints, 2nd ed. Philadelphia: WB Saunders; 1978:65.)

CHAPTER 263  BURSITIS, TENDINITIS, AND OTHER PERIARTICULAR DISORDERS  

1751

Inguinal ligament

Anterior-superior iliac spine Inguinal ligament

Iliofemoral ligament

Pubofemoral ligament

Femoral artery

Iliopsoas bursa

Greater trochanter

Femur

A

Lesser trochanter

Ischial tuberosity

Trochanteric bursa

Ischiogluteal bursa

B

injections should not be given too frequently, to avoid the possibility of weakening and rupture of the tendon. The accuracy of injections may be improved with concomitant use of diagnostic ultrasonography to assist with determining the correct needle site. Also, fluoroscopically guided injections can be used to increase accuracy.

FIGURE 263-2.  Musculoskeletal anatomy of the hip. A, Anterior aspect of the hip joint and bony structures. B, Relationship of the distended iliopsoas, trochanteric, and ischiogluteal bursae (shown in blue) to the hip joint and adjacent structures. (From Polley HF, Hunder GG, eds. Rheumatologic Interviewing and Physical Examination of the Joints, 2nd ed. Philadelphia: WB Saunders; 1978:183.)

TABLE 263-3  ADDITIONAL SPORTS-RELATED CONDITIONS SHOULDER Acromioclavicular separation Glenoid labial tear (SLAP lesion) Glenohumeral instability with dislocation ELBOW

  SPORTS MEDICINE INJURIES

An overlap exists between commonly occurring conditions of tendinitis and bursitis and those attributed to sports injuries (Table 263-3). For example, lateral epicondylitis, frequently referred to as tennis elbow, occurs more commonly secondary to non–sports-related causes. In contradistinction, iliotibial band syndrome is usually related to sports. Other entities that occur more often in relation to sports include ligamentous knee injuries, patellar tendinitis, ankle sprains, turf toe, and acromioclavicular separations. It is important in both categories to know the anatomy and biomechanics of the condition, so as to better diagnose and treat the problem. Sports-related injuries are helped with the classic RICE treatment, consisting of rest, ice, compression, and elevation. Often, anti-inflammatory and analgesic drugs are used. However, there is less use of corticosteroid injections in athletic injuries than in routine cases of tendinitis and bursitis.

  DISORDERS OF THE SHOULDER REGION

Shoulder pain is one of the most common musculoskeletal complaints in people older than 40 years of age. In younger people, athletic injuries are a frequent source of such pain. Rotator cuff tendinitis, or impingement syndrome, is the most common cause of shoulder pain.2 Tendinitis (and not bursitis) is the primary cause of pain, but secondary involvement of the subacromial bursa occurs in some cases. The condition may be acute or chronic and may or may not be associated with calcific deposits within the tendon. The key finding is pain in the rotator cuff on active abduction, especially between 60 and 120 degrees, and sometimes when lowering the arm. In more severe cases, pain may begin on initial abduction and continue throughout the ROM. Typically, chronic rotator cuff tendinitis manifests as an ache in the shoulder, usually over the lateral deltoid, and occurs with various movements, especially abduction and internal rotation. Other symptoms include difficulty in dressing oneself and night pain because of difficulty in positioning the shoulders. The physical findings include pain and loss of active abduction and internal rotation, less pain on passive motion, tenderness of the area of supraspinatus insertion, and a positive impingement sign (Fig. 263-3, Neer’s sign, which is pain occurring in forced flexion.3 The causes of rotator cuff tendinitis are multifactorial, but relative overuse, especially from overhead activity causing impingement of the rotator cuff, is commonly implicated. Treatment consists of rest and modalities such as hot packs, ultrasound, or cold applications, with specific ROM exercises as soon as tolerated. NSAIDs are often beneficial, but the most frequent treatment is injection of a depot corticosteroid A2  into the subacromial bursa, the floor of which is contiguous with the rotator cuff. Disodium ethylenediaminetetraacetic acid (EDTA) administered by phonophoresis and mesotherapy to patients with calcific tendinitis of the shoulder

Triceps tendinopathy Little League elbow (apophysitis) Distal biceps tendinitis WRIST AND HAND Gamekeeper’s thumb (skier’s thumb) Mallet finger (baseball finger) Extensor carpi ulnaris tendinitis Rupture of flexor digitorum profundus tendon Injury to triangular fibrocartilage HIP Adductor strain (groin pull) Hip pointer Hamstring strain KNEE Anterior cruciate tear Posterior cruciate tear Medial collateral ligament tear/strain Lateral collateral ligament tear/strain Popliteal tendinitis Medial and lateral meniscal tears Patellar tendinitis Iliotibial band syndrome ANKLE AND FOOT Ankle sprain Turf toe Stress fracture

has been found to be effective in pain reduction, improvement in shoulder function, and disappearance of calcifications. A3  Extracorporeal shock wave therapy can also be beneficial for chronic calcific tendinitis. A4  In a rotator cuff tear, an acute tear after trauma is usually easily recognized. The trauma may be superimposed on an already degenerative and possibly even partially torn cuff. In cases of trauma resulting in a ruptured cuff, fracture of the humeral head and dislocation of the joint also should be considered. However, most patients with a tear recall no trauma. In these cases, degeneration of the rotator cuff occurs gradually, resulting ultimately in a complete tear. Rotator cuff tears are classified as small (≤1 cm), medium (1 to 3 cm), large (3 to 5 cm), or massive (>5 cm). Shoulder pain, weakness on abduction, and loss of motion occur in varying degrees, ranging from severe pain and mild weakness to no pain and marked weakness. A positive drop-arm sign with inability to maintain actively 90 degrees of passive shoulder abduction may be present in patients with large or massive tears. Small complete tears

1752

CHAPTER 263  BURSITIS, TENDINITIS, AND OTHER PERIARTICULAR DISORDERS  

FIGURE 263-3.  The impingement sign is elicited by forced forward elevation of the arm. Pain results as the greater tuberosity impinges on the acromion. The examiner’s hand prevents scapular rotation. This maneuver may be positive in other periarticular disorders. (From Neer CS II. Impingement lesions. Clin Orthop. 1983;173:70-77.)

and incomplete tears of the rotator cuff are treated conservatively with rest, physical therapy, and NSAIDs. Although its role has not yet been established by careful studies, a subacromial injection of a corticosteroid may relieve pain. Surgical repair may be indicated in younger patients. Bicipital tendinitis is manifested by pain, most often in the anterior region of the shoulder and occasionally more diffusely. The pain may be acute but is usually chronic and is related to impingement of the biceps tendon by the acromion. Tenosynovitis of the long head of the biceps is present, and the tendon may be frayed and fibrotic. Palpation over the bicipital groove reveals localized tenderness. The patient’s response should be compared with the response to palpation of the opposite side (i.e., tendon with normal tenderness). Pain may be reproduced over the bicipital tendon in some cases by supination of the forearm against resistance (Yergason’s sign), shoulder flexion against resistance (Speed’s test), or extension of the shoulder. Treatment of bicipital tendinitis consists of rest, hot packs, ultrasound, and, as pain subsides, passive and then active ROM exercises. NSAIDs may be helpful, and occasionally a small amount of corticosteroid carefully injected into the tendon sheath may be of benefit. Rupture of the biceps tendon can occur at the superior edge of the bicipital groove, producing a characteristic bulbous enlargement of the lateral half of the muscle belly. Adhesive capsulitis (frozen shoulder) is associated with generalized pain and tenderness and severe loss of active and passive motion in all planes. It is rare before 40 years of age but may occur secondary to any type of shoulder problem. However, not every stiff and painful shoulder is necessarily adhesive capsulitis. Inflammatory arthritis and diabetes can cause adhesive capsulitis. Additional factors such as immobility, low pain threshold, depression, and neglect or improper initial treatment also favor the development of a frozen shoulder. Many cases, however, are idiopathic. The joint capsule adheres to the anatomic neck, and the axillary fold binds to itself, causing restricted motion. The capsule becomes thickened and contracted. Arthrography can help confirm this diagnosis by showing a decrease in volume of the shoulder joint capsule. Oral steroids improve pain and range of motion in the short term, but a frozen shoulder is probably best treated with a comprehensive program involving NSAIDs and corticosteroid injections into the glenohumeral joint and the subacromial bursa. Physical therapy consists of ice packs, ultrasound, transcutaneous electrical nerve stimulation, and gentle ROM exercises, beginning with pendulum exercises and wall climbing with the fingers and progressing to active ROM and strengthening exercises.

  DISORDERS OF THE ELBOW REGION

Olecranon bursitis occurs frequently and involves the subcutaneous olecranon bursa, either secondary to trauma or as an idiopathic condition. The bursa is characteristically swollen and tender on pressure, but pain may be minimal and usually no motion is lost. Aspiration may yield clear or bloodtinged fluid with a low viscosity or grossly hemorrhagic fluid. Inflammatory olecranon bursitis may be caused by gout, RA, or calcium pyrophosphate deposition disease, and infection can also cause a bursitis. Aspiration alone and protection from trauma are usually sufficient to resolve the condition. A small dose of corticosteroid may be injected into the bursa. With septic olecranon bursitis, localized erythema is the major clue. Heat, pain, and a positive culture are also frequently present.

FIGURE 263-4.  Injection of de Quervain tenosynovitis.

Lateral epicondylitis, or tennis elbow, is a common condition in those who overuse their arms.4 Localized tenderness directly over or slightly anterior to the lateral epicondyle is the hallmark of this disorder. Pain may occur during handshakes, while lifting a briefcase, or with other similar activities. Probably less than 10% of patients actually acquire lateral epicondylitis through playing tennis. Job and recreational activities, including gardening and athletics, are the usual causes. Pathologically, the condition consists of degeneration of the common extensor tendon, particularly of the extensor carpi radialis brevis tendon.5 Treatment is aimed at altering activities and preventing overuse of the forearm musculature. Ice packs, heat, and NSAIDs are of some benefit. A forearm brace also can be used. A local corticosteroid injection with a 25-gauge needle over the lateral epicondyle often produces satisfactory initial relief. Isometric strengthening is important as the initial part of a rehabilitation program. Medial epicondylitis, or golfer’s elbow, which mainly involves the flexor carpi radialis, is less common and less disabling than lateral epicondylitis. Local pain and tenderness over the medial epicondyle are present, and resistance to wrist flexion exacerbates the pain.

  DISORDERS OF THE WRIST AND HAND

A ganglion is a cystic swelling that arises from a joint or tendon sheath and occurs most commonly over the dorsum of the wrist. It is synovial lined and contains thick, jelly-like fluid. Ganglia apparently develop secondary to trauma or prolonged wrist extension. Usually, the only symptom is swelling, but occasionally a large ganglion produces discomfort on wrist extension. De Quervain’s tenosynovitis may result from repetitive activity that involves pinching with the thumb while moving the wrist. The symptoms are pain, tenderness, and occasionally swelling over the radial styloid. Pathologic findings include inflammation and narrowing of the tendon sheath around the abductor pollicis longus and extensor pollicis brevis. A positive Finkelstein test result is usually seen; pain increases when the thumb is folded across the palm and the fingers are flexed over the thumb as the examiner passively deviates the wrist toward the ulnar side. However, this test also may be positive in patients with osteoarthritis (OA) of the first carpometacarpal joint and must be differentiated from this common condition. Treatment involves splinting, local corticosteroid injection (Fig. 263-4), and NSAIDs as indicated. Rarely, surgical removal of the inflamed tenosynovium is needed. Volar flexor tenosynovitis consists of inflammation of the tendon sheaths of the flexor digitorum superficialis and flexor digitorum profundus tendons in the palm. It is extremely common but often unrecognized. Pain in the palm is felt on finger flexion, but in some cases the pain radiates to the proximal interphalangeal (PIP) and metacarpophalangeal (MCP) joints on the dorsal side, misleading the examiner. The diagnosis is made by palpation and identification of localized tenderness and swelling of the volar tendon sheaths. The middle and index fingers are most commonly involved, but the ring and little fingers also can be affected. Often a nodule composed of fibrous tissue can be palpated in the palm just proximal to the MCP joint on the volar side. The nodule interferes with the normal tendon gliding and can cause a triggering or locking, which may be intermittent and may produce an uncomfortable sensation. Similar involvement can occur at the flexor tendon of the thumb. The most common cause is overuse trauma of the hands from gripping with increased pull on the flexor tendons. It may be part of inflammatory conditions, such as RA, psoriatic arthritis, or apatite crystal deposition disease. It is seen frequently in conjunction with OA of the hands. Injection

CHAPTER 263  BURSITIS, TENDINITIS, AND OTHER PERIARTICULAR DISORDERS  

of a long-acting steroid into the tendon sheath usually relieves the problem, although surgery on the tendon sheath may be needed in unremitting cases. Gamekeeper’s thumb (skier’s thumb) is caused by trauma to the thumb resulting in instability of the first MCP joint. This instability is due to laxity or rupture of the ulnar collateral ligament. It is treated by immobilization, but surgical repair may be necessary. Avulsion of flexor digitorum profundus (jersey finger) may result from trauma, usually in football, when a player grabs onto a jersey. The distal phalanx, usually the fourth, is hyperextended while the digitorum profundus is contracting maximally. The avulsion of the tendon results in an inability to flex the distal phalanx of that digit. Surgery is required to correct the problem.

  DISORDERS OF THE HIP REGION

Although trochanteric bursitis is common, it frequently goes undiagnosed. It occurs predominantly in middle-aged to elderly people, and somewhat more often in women. The main symptom is aching over the trochanteric area and lateral thigh. Walking, various hip movements, and lying on the involved hip may intensify the pain. Onset may be acute, but more often it is gradual, with symptoms lasting for months. In chronic cases, the patient may fail to locate or describe the pain adequately, or the physician may fail to note the symptoms or interpret them correctly. Occasionally, the pain has a pseudoradiculopathic quality, radiating down the lateral aspect of the thigh. In a few cases, the pain is so severe that the patient cannot walk and complains of diffuse pain of the entire thigh. The best way to diagnose trochanteric bursitis is to palpate over the trochanteric area and elicit point tenderness. In addition to specific pain on deep pressure over the trochanter, other tender points may be noted throughout the lateral aspect of the thigh muscle. Pain may be worse with external rotation and abduction against resistance. Although bursitis has historically been described as the principal problem, the condition may actually arise at the insertions of the gluteus medius and gluteus minimus tendons.6 Local trauma and degeneration play a role in the pathogenesis, leading to tendinosis and/or tendon tears. Conditions that may contribute to trochanteric bursitis, apparently by adding stress to the area, include OA of the lumbar spine or of the hip, leg-length discrepancy, and scoliosis. Treatment consists of local injection of depot corticosteroid using a 22-gauge, 3.5-inch needle to ensure that the bursal area is reached (Fig. 263-5). NSAIDs, weight loss, and strengthening and stretching of the gluteus medius muscle and iliotibial band help in management. Coccydynia is manifested by pain in the coccyx area when pressure is applied to the area. This most notably occurs on sitting. The patient squirms from buttock to buttock to relieve the pressure and consequent pain and often chooses to sit on a cushion. The symptoms may be chronic and severe. The condition may relate to a fall on the coccyx, dropping to a hard chair when sitting, or some related trauma to the coccyx. However, at times no obvious cause can be detected. Women are much more frequently affected, perhaps because the lordosis that often occurs in women exposes the coccyx to more trauma. The diagnosis is confirmed by finding localized tenderness over the coccyx on palpation. A plain x-ray film can be obtained to exclude a fracture or dislocation of the coccyx. Treatment with a local injection of 1 mL of a long-acting corticosteroid and 2 mL of a 2% lidocaine solution is usually very effective. The exact nature of the pathology of coccydynia has not been studied, but it is presumed to be a bone bruise. In iliopsoas bursitis, groin and anterior thigh pain are present and worsen on passive hip hyperextension and sometimes on flexion, especially with

FIGURE 263-5.  Injection of trochanteric bursitis.

1753

resistance. Tenderness is palpable over an involved bursa. The patient may hold the hip in flexion and external rotation to eliminate pain and may limp to prevent hyperextension of the hip. The iliopsoas bursa lies behind the iliopsoas muscle, anterior to the hip joint and lateral to the femoral vessels. It communicates with the hip in 15% of cases. The diagnosis is more apparent if a cystic mass is seen (~30% of cases); however, other causes of cystic swelling in the femoral area must first be excluded. A bursal mass can cause femoral venous obstruction or femoral nerve compression. As with most cases of bursitis, acute or recurrent trauma and inflammatory conditions such as RA may lead to iliopsoas bursitis (also called iliopectineal bursitis). Iliopsoas tendinitis may overlap with the bursitis or occur independently in a similar clinical picture. The diagnosis is confirmed by plain x-ray with injection of a contrast medium into the bursa, or by ultrasonography, computed tomography, or MRI. Iliopsoas bursitis/tendinitis usually responds to conservative treatment including physical therapy and corticosteroid injections. With recurrent involvement, excision of the bursa may be necessary. Ischial or ischiogluteal bursitis is caused by trauma or by prolonged sitting on hard surfaces, as evidenced by the name weaver’s bottom. Pain is often exquisite when sitting or lying down. The hamstring muscles originate from the ischial tuberosity, and the ischiogluteal bursa is superficial to the tuberosity. Because the bursa is superficial to the tuberosity, separating the gluteus maximus from the tuberosity, the pain may radiate down the back of the thigh. Point tenderness over the ischial tuberosity is present. Use of cushions, hamstring stretching, and local injection of a corticosteroid are helpful.

  DISORDERS OF THE KNEE REGION

Anserine bursitis is seen predominantly in overweight, middle-aged to elderly women with large legs and OA of the knees. The symptoms are pain and tenderness over the medial aspect of the knee approximately 2 inches below the joint margin, with the pain worsened by climbing stairs. The pes anserinus (Latin for “goose foot”) is composed of the conjoined tendons of the sartorius, gracilis, and semitendinosus muscles. The bursa extends between the above tendons and the tibial collateral ligament. Tendinitis of these tendons, rather than bursitis, is the predominate cause of the syndrome. The diagnosis is made by eliciting exquisite tenderness over the bursal area. Anserine bursitis is often overlooked because it frequently occurs concomitantly with OA of the knee, which, when present, is the assumed cause of pain; however, in some cases of dual involvement, anserine bursitis is the principal source of pain. The treatment is rest, stretching of the adductor and quadriceps muscles, and a corticosteroid injection into the bursa and tendon insertion site. Prepatellar bursitis manifests as a swelling superficial to the kneecap and results from trauma such as frequent kneeling, leading to the name housemaid’s knee. The prepatellar bursa lies anterior to the lower half of the patella and the upper half of the patellar ligament. The pain is generally slight unless pressure is applied directly over the bursa. The infrapatellar bursa, which lies between the patellar ligament and the tibia, is also subject to trauma and swelling. Chronic prepatellar bursitis can be treated by protecting the knee from the irritating trauma. Patellar tendinitis (jumper’s knee) is seen predominantly in athletes engaging in activities such as repetitive running, jumping, or kicking. Pain and tenderness are present over the patellar tendon. Iliotibial band syndrome manifests by lateral knee pain caused by friction between the iliotibial band and the lateral femoral condyle. It is an overuse injury and is seen in runners, cyclists, and other athletes performing repetitive knee flexion activities. Popliteal cysts,7 also known as Baker’s cysts, are not uncommon, and the clinician should be well aware of the possibility of their dissection or rupture. A cystic swelling behind the knee with mild or no discomfort can be the only initial finding. With further distention of the cyst, however, a greater awareness and discomfort are experienced, particularly on full flexion or extension. The cyst is best seen when the patient is standing and examined from behind. Any knee disease having a synovial effusion can develop into a popliteal cyst. Popliteal cysts are most common secondary to RA, OA, or internal derangements of the knee. There are a few reported cases secondary to gout and Reiter’s syndrome. A syndrome of pseudothrombophlebitis may occur as a result of cyst dissection into the calf or actual rupture of the cyst. Findings include diffuse swelling of the calf, pain, and sometimes erythema and edema of the ankle. An ultrasound or arthrogram of the knee confirms both the cyst and the possible dissection or rupture. A cyst related to an inflammatory arthritis is treated by injection of a depot corticosteroid into the knee joint, and possibly into the cyst itself, which usually resolves the problem. If the cyst results from OA or an internal derangement of the knee, surgical repair of the underlying joint lesion is usually necessary to prevent a recurrence of the cyst.

1754

CHAPTER 264  Rheumatoid Arthritis  

In the knee area, tendon ruptures may occur, and quadriceps tendon rupture is involved approximately 50% of the time; otherwise, patellar tendon rupture occurs. Quadriceps tendon rupture is generally caused by sudden violent contractions of the quadriceps muscle when the knee is flexed. A hemarthrosis of the knee joint may follow. Patients with chronic renal failure, RA, hyperparathyroidism, or gout and patients with SLE taking steroids have been reported to have spontaneous ruptures of the quadriceps tendon. The patient experiences a sudden sharp pain and cannot extend the leg. X-ray studies may show a high-riding patella. The tendon is usually found to be degenerated, and surgical repair is often indicated. Rupture of the patellar tendon has been associated with a specific episode of trauma, repetitive trauma from sporting activities, and systemic diseases. Meniscal tears are common causes of knee “locking” and pain. Physical examination may show pain, with or without clicking, when the hip and knee are bent to 90°. Magnetic resonance imaging is the diagnostic test of choice. Physical therapy is often as effective as surgery. A5  A6  ,

  DISORDERS OF THE ANKLE AND FOOT REGION

Achilles tendinitis usually results from trauma, athletic overactivity, or improperly fitting shoes with a stiff heel counter, but it also can be caused by inflammatory conditions such as ankylosing spondylitis, Reiter’s syndrome, gout, RA, and calcium pyrophosphate dihydrate crystal deposition disease.9 Pain, swelling, and tenderness occur over the Achilles tendon at its attachment and in the area proximal to the attachment. Crepitus on motion and pain on dorsiflexion may be present. Management includes NSAIDs, rest, shoe corrections, heel lift, gentle stretching, and sometimes a splint with slight plantar flexion. Local injection of platelet-rich plasma (PRP) has become an increasingly used treatment for releasing growth factors into degenerative tendons; however, more recent randomized, placebo-controlled trials for treatment of chronic Achilles and other tendinopathies have found PRP injections to be ineffective in improving pain and activity. A7  The Achilles tendon is vulnerable to rupture when involved with tendinitis, and treatment with a corticosteroid injection could increase this possibility. Achilles tendon rupture is well known and occurs with a sudden onset of pain during forced dorsiflexion. An audible snap may be heard, followed by difficulty in walking and standing on toes. Swelling and edema over the area usually develop. Diagnosis can be made with the Thompson test, in which the patient kneels on a chair with the feet extending over the edge and the examiner squeezes the calf and pushes toward the knee. Normally this produces plantar flexion, but in a ruptured tendon, no plantar flexion occurs. Achilles tendon rupture usually occurs during athletic events or with trauma from jumps or falls. The tendon is more prone to tear in people with preexisting Achilles tendon disease and in those taking corticosteroids. Orthopedic consultation should be obtained, and immobilization or surgery may be selected, depending on the situation. For acute, severe ankle sprain, a below-knee cast or Aircast produces a faster recovery than a tubular compression bandage, but there is no difference in outcomes at 9 months. A8  Plantar fasciitis, which is seen primarily in persons between 40 and 60 years of age, is characterized by pain in the plantar area of the heel. The onset may be gradual, or it may occur with trauma or overuse from some activity, such as athletics, prolonged walking, using improper shoes, or striking the heel with some force. Plantar fasciitis may be idiopathic; it also is likely to be present in younger patients with spondyloarthritis (Chapter 265). The pain characteristically occurs in the morning on arising and is most severe for the first few steps. After an initial improvement, the pain may worsen later in the day, especially after prolonged standing or walking. The pain is burning, aching, and occasionally lancinating. Palpation typically reveals tenderness anteromedially on the medial calcaneal tubercle at the origin of the plantar fascia. Treatment includes relative rest with a reduction in stressful activities, NSAIDs, use of heel pad or heel cup orthosis, arch support, and stretching of the heel cord and plantar fascia. A local corticosteroid injection, using a 25-gauge needle, is often of help. In posterior tibial tendinitis, pain and tenderness occur just posterior to the medial malleolus; it can be caused by trauma, excessive pronation, RA, or spondyloarthropathy. Extension and flexion may be normal, but pain is present on resisted inversion or passive eversion. The discomfort is usually worse after athletic activity, and swelling and localized tenderness may be present. Treatment usually includes rest, NSAIDs, and possibly a local injection of corticosteroid. Immobilization with a splint is sometimes needed. Posterior tibialis tendon rupture, which is not commonly recognized, is a cause of progressive flat foot. It can result from trauma, chronic tendon degeneration, or RA. An insidious onset of pain and tenderness may be noted along

the course of the tendon just distal to the medial malleolus, along with swelling medial to the hind foot. The unilateral deformity of hind foot valgus and forefoot abduction is an important finding. The forefoot abduction can best be seen from behind; more toes are seen from this position than would be seen normally. The result of the single heel rise test is positive when the patient is unable to rise onto the ball of the affected foot while the contralateral foot is off the floor. Treatment usually includes rest, NSAIDs, and possibly an orthosis. Surgical repair of the tendon is sometimes indicated. Manifestations of retrocalcaneal bursitis include pain at the back of the heel, tenderness of the area anterior to the Achilles tendon, and pain on dorsiflexion. Local swelling is present, with bulging on the medial and lateral aspects of the tendon. Retrocalcaneal bursitis, also called sub-Achilles bursitis, may coexist with Achilles tendinitis, and distinguishing the two is sometimes difficult. This condition may be secondary to RA, spondylitis, a reactive arthritis, gout, or trauma. Turf toe is an injury of the big toe originally described during play on artificial turf. It results from hyperextension of the first metatarsophalangeal (MTP) joint when a fixed, dorsiflexed foot is forced into the ground. The plantar capsular ligament may be sprained or torn. Stress fracture is also known as march fracture or fatigue fracture because it was first associated with spontaneous fracture after long marches in army recruits. Pain, swelling, tenderness, and occasionally erythema develop over the metatarsal area, usually without any clear history of trauma. On questioning, however, the episode of spontaneous pain related to onset of the fracture can be identified in some cases. The neck of the second metatarsal bone is most frequently involved, but the third metatarsal is also a site of fracture. Aside from prolonged marching, other athletic events with overactivity, including jogging, are common causes. Stress fractures may be seen in patients with RA and in elderly people. The difficulty in diagnosing stress fractures is that the initial x-ray films usually show no abnormalities or, at most, only a faint fracture line. A repeat x-ray examination several weeks later shows healing with callus formation. Bone scans aid the early diagnosis of stress fractures by showing an increase in uptake over the fracture site. Usually these fractures heal spontaneously, and rest and strapping of the foot are helpful. Occasionally, a cast is needed.

Grade A References A1. Coombes BK, Bisset L, Vicenzino B. Efficacy and safety of corticosteroid injections and other injections for management of tendinopathy: a systematic review of randomised controlled trials. Lancet. 2010;376:1751-1767. A2. Rhon DI, Boyles RB, Cleland JA. One-year outcome of subacromial corticosteroid injection compared with manual physical therapy for the management of the unilateral shoulder impingement syndrome: a pragmatic randomized trial. Ann Intern Med. 2014;161:161-169. A3. Cacchio A, De Blasis E, Desiati P, et al. Effectiveness of treatment of calcific tendinitis of the shoulder by disodium EDTA. Arthritis Rheum. 2009;61:84-91. A4. Bannuru RR, Flavin NE, Vaysbrot E, et al. High-energy extracorporeal shock-wave therapy for treating chronic calcific tendinitis of the shoulder: a systematic review. Ann Intern Med. 2014;160: 542-549. A5. Katz JN, Brophy RH, Chaisson CE, et al. Surgery versus physical therapy for a meniscal tear and osteoarthritis. N Engl J Med. 2013;368:1675-1684. A6. Sihvonen R, Paavola M, Malmivaara A, et al. Arthroscopic partial meniscectomy versus sham surgery for a degenerative meniscal tear. N Engl J Med. 2013;369:2515-2524. A7. Moraes VY, Lenza M, Tamaoki MJ, et al. Platelet-rich therapies for musculoskeletal soft tissue injuries. Cochrane Database Syst Rev. 2014;4:CD010071. A8. Lamb SE, Marsh JL, Hutton JL, et al. Mechanical supports for acute, severe ankle sprain: a pragmatic multicentre, randomised controlled trial. Lancet. 2009;373:575-581.

GENERAL REFERENCES For the General References and other additional features, please visit Expert Consult at https://expertconsult.inkling.com.

264  RHEUMATOID ARTHRITIS JAMES R. O’DELL

DEFINITION

Rheumatoid arthritis (RA) is a chronic systemic inflammatory disease of unknown etiology that primarily targets synovial tissues. It is relatively common, with a prevalence of slightly less than 1% in adults all over the

CHAPTER 263  BURSITIS, TENDINITIS, AND OTHER PERIARTICULAR DISORDERS  

GENERAL REFERENCES 1. Hall MM, Finnoff JT, Smith J. Musculoskeletal complications of fluoroquinolones: guidelines and precautions for usage in the athletic population. PM R. 2011;3:132-142. 2. Huegel J, Williams AA, Soslowsky LJ. Rotator cuff biology and biomechanics: a review of normal and pathological conditions. Curr Rheumatol Rep. 2015;17:476. 3. Hermans J, Luime JJ, Meuffels DE, et al. Does this patient with shoulder pain have rotator cuff disease?: The Rational Clinical Examination systematic review. JAMA. 2013;310:837-847. 4. Brummel J, Baker CL 3rd, Hopkins R, et al. Epicondylitis: lateral. Sports Med Arthrosc. 2014;22: e1-e6.

1754.e1

5. Frick MA, Murthy NS. Imaging of the elbow: muscle and tendon injuries. Semin Musculoskelet Radiol. 2010;14:430-437. 6. Klauser AS, Martinoli C, Tagliafico A, et al. Greater trochanteric pain syndrome. Semin Musculoskelet Radiol. 2013;17:43-48. 7. Herman AM, Marzo JM. Popliteal cysts: a current review. Orthopedics. 2014;37:e678-e684. 8. Nguyen JC, De Smet AA, Graf BK, et al. MR imaging-based diagnosis and classification of meniscal tears. Radiographics. 2014;34:981-999. 9. Calleja M, Connell DA. The Achilles tendon. Semin Musculoskelet Radiol. 2010;14:307-322.

1754.e2

CHAPTER 263  BURSITIS, TENDINITIS, AND OTHER PERIARTICULAR DISORDERS  

REVIEW QUESTIONS 1. A 78-year-old woman who has had periodic discomfort in her neck and radiating to both shoulders for several years had a fall, landing on her outstretched right arm. She had pain in the right shoulder and weakness on use. At the urgent care center she was found to have some pain on passive motion and was unable to actively abduct the right shoulder. She had a positive drop arm sign of the right shoulder but not of the left one. All upper extremity deep tendon reflexes were normal. Which of the following is the most likely diagnosis? A. Rotator cuff tendinitis is the main diagnosis. B. X-ray examination of the humerus of the injured side is not indicated in this case because tendon injuries are not visualized on plain x-ray film of the shoulder. C. Bicipital tendinitis is the likely diagnosis. D. A tear of the rotator cuff best fits as the diagnosis. E. Cervical spine disc herniation is the most likely diagnosis based on the history of neck pain and arm weakness. Answer: D  Acute trauma is often the cause of rotator cuff tears, although most tears occur gradually through chronic tendinitis. The positive drop arm sign is typical of a significant rotator cuff tear. Sometimes with trauma, a humeral fracture can occur along with the rotator cuff tear. Pain on abduction and especially on resisted abduction are seen in rotator cuff tendinitis; also, the drop arm sign is typically negative. Bicipital tendinitis usually does not cause shoulder weakness. The clinical picture does not fit a cervical disc herniation, and the reflexes are normal. Biundo, JJ: Regional Rheumatic Pain Syndromes, Primer on the Rheumatic Diseases, Thirteenth Edit. 2006 2. A 51-year-old male insurance agent, whose hobby is wood working, complains of right hand pain of approximately 4 months’ duration. He had been doing more wood working during that period until the hand pain began to limit this activity. On examination there were no Heberden’s nodes of the DIP joints, and the PIP and MCP were not tender or swollen. Moderate-to-severe tenderness on palpation was present over the palm side of the hand at the region of the second and third metacarpal bones. Some swelling was detected at these sites. Crepitus was noted on palpation on flexion of these two digits. Which of the following is the most likely diagnosis? A. Because rheumatoid arthritis is a likely diagnosis, you should obtain RF, ANA, and hand x-ray films. B. Because of his age, he most likely has osteoarthritis of the hand. C. He fits the picture of volar flexor tenosynovitis D. A C7 radiculopathy would best explain pain in this part of his hand. E. Overuse of his hands has caused carpal tunnel syndrome. Answer: C  Volar flexor tenosynovitis is one of the most common musculoskeletal syndromes and is frequently overlooked. It is manifest by pain in the palm of the hand. The volar flexor tendon sheath is tender on palpation and thickened. Triggering or snapping of the digit sometimes occurs, especially when more chronic. Overuse of the hand is the usual cause, but is also seen in association with OA of the hands and diabetes. Stretching of the involved digit and an injection of a small amount of a corticosteroid into the tendon sheath is usually helpful. Biundo, JJ: Regional Rheumatic Pain Syndromes, Primer on the Rheumatic Diseases, Thirteenth Edit. 2006

3. A 48-year-old male professional cellist, whose hobby is gardening, has had a 4-month history of right shoulder pain. Pain occurs on various movements of the shoulder, and there is some limitation of movement. Night pain in bed is also reported. On examination, pain limits active abduction to 90 degrees. Less pain is noted on passive abduction, and 180 degrees of abduction is present. The drop arm sign is negative, but pain occurs on active internal rotation of the shoulder. There is no crepitus. Which of the following is the most likely diagnosis? A. Rotator cuff tendinitis is the most likely diagnosis. B. Rotator cuff tear best fits this clinical picture. C. Rupture of the biceps tendon is the diagnosis because the drop arm sign is negative. D. Adhesive capsulitis (frozen shoulder) is the diagnosis. E. OA of the shoulder (glenohumeral joint) is the likely diagnosis, and a plain x-ray film of the shoulder is needed to confirm the diagnosis. Answer: A  The typical physical examination findings of rotator cuff tendinitis are pain on active abduction, less pain on passive abduction, and more pain on resistive abduction. Loss of motion on active movements can occur when more severe. Also, pain may occur on active internal and on external rotation. In addition to the physical examination, ultrasonography and MRI can detect tendinitis, but not plain x-ray examination. In adhesive capsulitis restriction, the range of motion in all directions is usually seen. The full range on abduction and other motions in this case are not typical of a frozen shoulder. OA of the shoulder would not have had a pain history of only 4 months, and it is a much less frequent entity than rotator cuff tendinitis. Crepitus on range of motion is very common in OA of the glenohumeral joint. Biundo, JJ: Regional Rheumatic Pain Syndromes, Primer on the Rheumatic Diseases, Thirteenth Edit. 2006 4. A 76-year-old woman with a known history of moderate OA of both knees and a partially torn right medial meniscus experienced pain in her right knee after stooping down to clean something off the floor. The next day she had discomfort and fullness behind her right knee and swelling of her lower leg and ankle. She was able to walk but had a limp. She was seen that day by her primary care physician for evaluation. Which of the following is the best course of action? A. Begin treatment with heparin or enoxaparin for a deep vein thrombosis. B. Give oral prednisone for 5 days as a tapering dose for an acute flare of the OA of her knee. C. Obtain a new x-ray film of the knees, with a standing AP and lateral view of the involved leg. D. Because of the acute flare of knee symptoms, give colchicines for a possible acute attack of gout. E. Obtain a Doppler ultrasound of the right leg for a deep vein thrombosis and an ultrasound image of the right knee to identify a Baker’s cyst that has dissected, causing a pseudothrombophlebitis picture. Answer: E  Popliteal cysts, also known as Baker’s cysts, are often asymptomatic. However, at times they may enlarge as a result of increasing pressure from a synovial effusion of the knee, flowing through a one-way valve from the knee to the popliteal cyst. The cyst can at times dissect downward, causing swelling of the leg and simulating thrombophlebitis. Thus, the name given to the syndrome is pseudothrombophlebitis. The swelling of the leg may resolve on its own after several days. Often, a corticosteroid injection into the knee joint may help by decreasing the inflammation and synovial effusion of the knee. OA of the knee should not produce swelling of the lower leg, nor should gout. Treatment with anticoagulants should not be given unless the diagnosis of DVT were confirmed. Biundo, JJ: Regional Rheumatic Pain Syndromes, Primer on the Rheumatic Diseases, Thirteenth Edit. 2006

CHAPTER 263  BURSITIS, TENDINITIS, AND OTHER PERIARTICULAR DISORDERS  

5. A 52-year-old moderately obese man complains of pain in his right foot that has been present for approximately 4 months. The pain has become worse recently, and he has had to stop the walks he was taking for exercise. The pain is described to be more in the rear foot and is particularly bad when he first gets out of bed in the morning. He has no history of gout or other arthritis except for some back pain. On examination there is no joint swelling. He does have pes cavus, and tenderness of the plantar surface of the rear foot. His calf muscles are also tight. What is the most likely diagnosis and/or the best course of evaluation and treatment? A. Order an MRI of the foot or technetium bone scan for possible stress fracture. B. Inject the calcaneus with a steroid, because that is the best initial treatment for heel pain. C. Make a tentative diagnosis of plantar fasciitis and treat with analgesics, stretching of calf and plantar fascia, and use of a heel cushion as the initial treatment. D. Obtain uric acid level and start treatment with colchicine. E. Obtain orthopedic consultation for possible Achilles tendon tear. Answer: C  The complaint of morning foot pain, which is called “first step pain,” is typical of plantar fasciitis. Tenderness often elicited on pressure exerted on the plantar surface of the calcaneus. Trauma to the foot from prolonged walking, tight calf muscles, high arched foot with tight plantar fascia, and obesity are factors related to acquiring plantar fasciitis. A steroid injection of the cacaneal area is often helpful, but is not the first line of treatment. With Achilles tendinitis/tear the pain is in the posterior heel and not the plantar surface. A stress fracture is more likely to occur in the metatarsal area and only very rarely occur in the calcaneus. Biundo, JJ: Regional Rheumatic Pain Syndromes, Primer on the Rheumatic Diseases, Thirteenth Edit. 2006

1754.e3

CHAPTER 264  Rheumatoid Arthritis  

world. RA shortens survival and significantly affects quality of life in many patients. Essentially all patients exhibit some systemic features such as fatigue, low-grade fevers, anemia, and elevations of acute phase reactants (erythrocyte sedimentation rate [ESR] or C-reactive protein [CRP]). This systemic inflammation is believed to be responsible for vascular endothelial damage and a marked increased risk for coronary artery disease and congestive heart failure in patients with RA.1 However, the primary target of RA is the synovium and it is responsible for most of the protean clinical features. Synovial tissues proliferate in an uncontrolled fashion, resulting in excess fluid production, destruction of cartilage, erosion of marginal bone, and stretching and damage of the tendons and ligaments. In the past two decades, the treatment of RA has changed dramatically. Current therapeutic strategies should result in over 50% of patients achieving clinical remissions with treatment with appropriate disease-modifying antirheumatic drugs (DMARD) or combinations of DMARDs.

EPIDEMIOLOGY

RA is present all over the world, with a prevalence of 0.5 to 1% of adults and with some differences in certain population groups. For reasons that are still unclear, the prevalence in women is two or three times greater than that in men. RA can occur at any age, but onset before the age of 45 years in men is uncommon. The relatively few well-done inception cohorts suggest that the yearly incidence of RA is approximately 40 per 100,000 for women and about half that for men. These figures vary significantly based on the age of the cohort. The best available data suggest that the incidence of RA in women increases with age until approximately 60 years of age and then plateaus. The incidence rate is much lower in young men, approximately one third that in women, but increases steadily with age and approaches that of women older than 65 years. Because the incidence of RA increases or is stable with age and RA is a lifelong disease, the prevalence of RA increases with each decade. Recent data suggest that the incidence of RA, particularly rheumatoid factor (RF)-negative RA, may be decreasing. The reasons for this are unclear, but, if elucidated, they could provide valuable insights into the etiology and pathogenesis of RA and might allow the implementation of strategies to prevent clinical disease. RA has a significant genetic component; therefore, it is not surprising that RA is reportedly very unusual in certain populations and more common in others. Most notably, cohorts have been described in rural Nigeria in which no individuals are affected with RA; in contrast, a prevalence of RA of 5% has been found in some studies of Chippewa, Yakima, and Inuit Native American tribes.

PATHOBIOLOGY

Genetics

Genetics play a significant role in determining both the risk for developing RA and the severity of the disease.2 Twin studies reveal a concordance rate for RA that averages 15% for monozygotic twins and approximately 5% for dizygotic twins. These data in monozygotic twins simultaneously reveal both the significance of genetic factors and the fact that they are clearly not the only important factor, or else the concordance rate would approach unity. It has been clearly shown that RA is a multigene disease with important contributions from both human leukocyte antigen (HLA) and non-HLA genes. The association of certain HLA alleles, specifically HLA-DR4, with an increased risk for developing RA and of having more severe disease has long been recognized. This association is explained by a particular amino acid sequence in the third hypervariable region on the DRβ1 chain. HLA-DR molecules are present on the surface of antigen-presenting cells and allow T cells to recognize antigen in the context of DR. Hypervariable regions on the DR molecule are particularly important for antigen recognition. E-Table 264-1 details the amino acid sequence of several DRβ1 chains that are associated with RA and some that are not. The amino acid sequence associated with RA has been called the shared epitope or the at-risk allele. It has been shown by a number of investigators that patients with the shared epitope have more severe RA and more extra-articular manifestations than those who are negative. Furthermore, individuals with two copies of the shared epitope, particularly those with HLA-DR4, have a further increased risk for the development of severe RA. This association with a particular antigen recognition site may ultimately aid understanding of the antigen or antigens that are important for triggering RA. Proteins in which arginine has been converted to citrulline are bound with greater avidity by the shared epitope. The importance of certain DRβ1 alleles in

1755

RA supports the concept that T cells are integrally involved in the pathogenesis. Population-based studies have suggested that only 30 to 50% of the genetic risk for RA is explained by genes located in the HLA region. A functional polymorphism for the gene that encodes intracellular protein tyrosine phosphatase nonreceptor 22 (PTPN22) has been reproducibly associated with RA and a number of other autoimmune diseases, including type 1 diabetes, systemic lupus erythematosus, Graves’ disease, and Hashimoto’s thyroiditis. Genome-wide association studies have identified at least 80 other candidate genes associated with RA, including polymorphisms for signal transducer and activator of transcription (STAT4), tumor necrosis factor receptor– associated factor 1 (TRAF-1), and CD40. To complicate things further, HLA-DRB1 03 is associated with lower titers of anti–citrullinated peptide antigen (ACPA) antibodies but is associated with increased risk for cyclic citrullinated peptide–negative RA. The shared epitope is present in approximately 25 to 35% of the white population, but the chance of developing RA among individuals who carry this allele is only approximately 1 in 25. Therefore, despite identifying the most important genetic risk factor for RA, this test has little or no clinical utility. The role of epigenetics in RA is currently receiving attention and may provide important insights.

Etiology

Clearly, other factors, in addition to genetics, are active in precipitating or triggering RA. RA appears to require the complex interaction of genetic and environmental factors with the immune system and ultimately in the synovial tissues throughout the body (Fig. 264-1).3 Sera collected before the development of clinical RA show that immunologic changes predate clinical manifestations by years. Autoantibodies, particularly ACPA antibodies and rheumatoid factor (RF), are present in the sera of many individuals 5 to 10 years before the clinical onset of disease. By following cohorts of people at high risk for RA, investigators are learning much about these early immunologic changes and ultimately it is hoped about the triggers for the disease. The use of oral contraceptives has been associated with a decrease in the incidence of RA; because the effect appears to be strongest for oral contraceptives that have high estrogen content, it is postulated that estrogen is responsible for this protective effect. Studies that have tried to address the question of postmenopausal estrogen use and its effect on RA have yielded conflicting results. Smoking has long been associated with a significant increase in the risk for developing RA, but more recently it has been shown that this is true only for ACPA-positive patients and is not associated with ACPA-negative disease. Furthermore, smoking appears to be a risk factor for RA only in those patients who are positive for shared epitope. Purported triggers for RA in addition to smoking have included bacteria (Mycobacteria, Streptococcus, Mycoplasma, Escherichia coli, Helicobacter pylori), viruses (rubella, Epstein-Barr virus, parvovirus), and periodontal disease.

Genetic background Some HLA-DRBI alleles Other HLA alleles Non-HLA alleles

RA initiated

Trigger ? Bacterial antigens ? Viral antigens ? Smoking ? Others

Ongoing autoimmune response Synovial proliferation and joint destruction FIGURE 264-1.  Initiation of rheumatoid arthritis (RA). HLA = human leukocyte

antigen.

CHAPTER 264  Rheumatoid Arthritis  

1755.e1

E-TABLE 264-1  HUMAN LEUKOCYTE ANTIGEN ASSOCIATIONS WITH RHEUMATOID ARTHRITIS THIRD HYPERVARIABLE REGION AMINO ACID SEQUENCES

HLA TYPES (ALLELES) AND METHODS OF DETECTION ALLOANTISERA (DR)

MLC (DW)

DNA (DRβ1)

70

71

72

73

74

MOST COMMON ETHNIC GROUPS

Q

K

R

A

A

Whites (Western Europe)

ASSOCIATED WITH RA DR4

Dw4

*0401

DR4

Dw14

*0404

R

Whites (Western Europe)

DR4

Dw15

*0405

R

Japanese, Chinese

DR1

Dw1

*0101

R

Asian Indians, Israelis

DR6 (14)

Dw16

*1402

R

Yakima Native Americans

DR10



*1001

R

R

Spanish, Greeks, Israelis

DR4

Dw10

*0402

D

DR4

Dw13

*0403

DR2

Dw2

*1501

DR3

Dw3

*0301

NOT ASSOCIATED WITH RA E

Whites (Eastern Europe)

R D

E

A

Polynesians Whites

G

R

Whites

A = alanine; D = aspartic acid; E = glutamic acid; HLA = human leukocyte antigen; K = lysine; MLC = mixed leukocyte cultures; Q = glutamine; R = arginine; * = the same amino acid in that position as for DRB1*0401.

1756

CHAPTER 264  Rheumatoid Arthritis  

Pathogenesis

The pathogenesis of RA is complex, and there are almost certainly multiple triggering mechanisms, including but not limited to smoking, infection, molecular mimicry, immune complexes, altered T-cell repertoire, and T-cell reactivity. Furthermore, it is likely that the triggers may be different based on the genetic background. As mentioned previously, smoking is a well-known trigger for some individuals but appears to be a risk factor only in those patients who possess the shared epitope. Rheumatic fever, reactive arthritis (formerly known as Reiter’s syndrome), and, more recently, Lyme arthritis are examples of arthritic syndromes for which infectious triggers have clearly been demonstrated, but these triggering agents are often difficult or impossible to isolate at the time when the arthritic syndromes occur. Many other examples exist in animal models of arthritis, including syndromes induced by mycobacteria and streptococci. Reactive arthritis (Chapter 265) has clearly been shown to occur when any one of a myriad of different but specific infectious triggers is presented to a specific location in the body (the gastrointestinal or genitourinary tract) of individuals with a certain genetic background, in most cases HLA-B27. Additionally, in this syndrome, the age and gender of the individual and hence the maturity of the immune system may be critical in the development of clinical disease, which occurs primarily between the ages of 15 and 40 years in males. Once unraveled, the pathophysiology of RA is likely to be similarly complex. Despite the absence of clear evidence linking any infectious agent to RA, it is widely believed that ultimately an important triggering role will be elucidated for infectious or other environmental agents. Once triggers for RA are identified, strategies for prevention can be addressed, but this information may not help individuals with established disease. Possibly infections involving the innate immune system are causative in an early subclinical phase of the rheumatoid disease process, with the agents being absent once clinical disease develops. The relative roles of the cellular versus the humoral immune system in the initiation and perpetuation of RA are much debated; both appear to be important. Most likely, the mechanisms of initiation of the disease process are different from those that perpetuate the chronic disease. T cells, particularly of the activated TH1 and TH17 types, appear to predominate in synovial tissues. These T cells, presumably activated by some as yet unknown antigen presented by macrophages, B cells, or synoviocytes in the context of HLA-DR, secrete cytokines that drive further synovial proliferation. It is believed by many that, although RA may initially be triggered by exogenous antigens, the process, once initiated, may be perpetuated by autoantigens. Macrophagederived cytokines, particularly interleukin-1 (IL-1) and tumor necrosis factor-α (TNF-α), play central roles in this ongoing inflammatory process. As definitive proof, biologic products directed against these cytokines have shown significant efficacy in the treatment of RA. The humoral immune system also plays a role. RF has long been a serologic marker of RA and is well known to correlate with more severe disease,

including erosions of bone, and with the presence of extra-articular features. The reason that RF is produced in excess and the exact role that it plays remain elusive. RF production may increase complement activation and result in the release of lysosomal enzymes, kinins, and oxygen free radicals. ACPA antibodies exhibit a high specificity (95 to 99%) for RA, although their sensitivity for RA with currently available assays is only approximately 70%. Even though both RF and ACPA antibodies also correlate with more aggressive erosive disease, this link is strongest for ACPA antibodies.

Pathology

The synovial tissues are the primary target of the autoimmune inflammatory process that is RA; the reason for this remains elusive. However, the generalized inflammation of RA also involves the vascular endothelium and results in significant premature atherosclerosis. Once RA is initiated, the synovial tissues throughout the body become the site of a complex interaction of T cells, B cells, macrophages, and synovial cells (Fig. 264-2). The resultant proliferation of the synovial tissues (synovitis) causes the production of excessive amounts of synovial fluid and the infiltration of pannus into adjacent bone and cartilage. Synovitis results in the destruction of cartilage and marginal bone and in the stretching or rupture of the joint capsule or tendons and ligaments. In patients, these effects are manifested by the deformities (Fig. 264-3 and E-Fig. 264-1) and disabilities that make up the clinical picture of RA.

FIGURE 264-3.  Severe advanced rheumatoid arthritis of the hands. There is massive tendon swelling over the dorsal surface of both wrists, severe muscle wasting, ulnar deviation of the metacarpophalangeal joints, and swan-neck deformity of the fingers. (From Forbes CD, Jackson WF. Color Atlas and Text of Clinical Medicine, 3rd ed. London: Mosby; 2003.)

Inhibition of proteoglycan synthesis (IL-1)

Bone Joint capsule

Bone resorption (IL-1)

Cartilage Normal synovium Synoviocyte hyperplasia

Vascular injury Influx of immune cells and plasma Initiation

M

Cytokines C T TNF-α, IL-1, IL-6, IL-8, GM-CSF B P

Prostaglandin Iysosomal enzymes O2 radicals

IgM=RF IgG Phagocytosis Cellular reactivation and of immune proliferation complex by cytokines

Immune Responses Cellular Humoral

Inflammation

Collagenase and PGE2 secreted by synoviocytes (induced by IL-1 and TNF-α) Rheumatoid pannus Neutrophils attracted and activated (GM-CSF, IL-8, TNF-α, TGF-β) Destruction

FIGURE 264-2.  Events involved in the pathogenesis of rheumatoid synovitis (progressing from left to right). B = B lymphocyte; C = complement; GM-CSF = granulocytemacrophage colony-stimulating factor; IgG, IgM = immunoglobulin G, M; IL = interleukin; M = macrophage; P = plasma cell; PGE2 = prostaglandin E2; RF = rheumatoid factor; T = T lymphocyte; TGF-β = transforming growth factor-β; TNF-α = tumor necrosis factor-α.

CHAPTER 264  Rheumatoid Arthritis  

A

1756.e1

B

E-FIGURE 264-1.  Subluxation of the cervical spine in patients with rheumatoid arthritis. A, In a lateral radiograph of the cervical spine, the body of C2 and its odontoid process are outlined by the broken lines and the posterior aspect of the anterior segment of C1 is indicated by a solid line. Normally, a space of only 2 to 3 mm separates C1 from C2. The space between C1 and the odontoid of C2 is markedly increased, indicative of subluxation of C1 and C2. B, Lateral view of a pathologic specimen from a patient who died of C1-C2 subluxation. The horizontal arrow shows the odontoid process that subluxed posteriorly, severely compressing and almost severing the cord. The vertical arrow shows a bone graft that had been put in place posteriorly in an attempt to prevent subluxation. Below the arrow, a nonhealing area is present through the bone graft, and inferior to that a wire fixation suture is still in place.

CHAPTER 264  Rheumatoid Arthritis  

Rheumatoid Arthritis

1757

Osteoarthritis

Median nerve in carpal tunnel Tapping produces paresthesias in the shaded area (Tinel’s sign) FIGURE 264-4.  Distribution of involved joints in the two most common forms of

arthritis: rheumatoid arthritis and osteoarthritis. Shaded circles are shown over the involved joint areas.

FIGURE 264-5.  Carpal tunnel syndrome. Distribution of pain and/or paresthesias (shaded area) when the median nerve is compressed by swelling in the wrist (carpal tunnel).

CLINICAL MANIFESTATIONS

Articular Manifestations

RA can affect any of the synovial (diarthrodial) joints (Fig. 264-4). Most commonly, the disease starts in the metacarpophalangeal (MCP), proximal interphalangeal (PIP), and metatarsophalangeal (MTP) joints, followed by the wrists, knees, elbows, ankles, hips, and shoulders, in roughly that order. Early treatment limits the joints involved. Less commonly, and usually later, RA may involve the temporomandibular, cricoarytenoid, and sternoclavicular joints. RA may involve the upper part of the cervical spine, particularly the C1-C2 articulation, but, unlike the spondyloarthropathies (Chapter 265), it does not involve the rest of the spine. RA patients are at an increased risk for osteoporosis (Chapter 243), and this risk should be considered and dealt with early.

Hands

The hands are a major site of involvement, and a significant portion of the disability that RA causes is because of damage and dysfunction of the hands. Typically disease starts with swelling of the PIPs and MCPs. The distal interphalangeal (DIP) joints are almost never involved; significant involvement of the DIP joints should suggest the possibility of a different diagnosis (i.e., osteoarthritis or psoriatic arthritis). Figure 264-3 illustrates the classic ulnar deviation of the MCP joints and swan-neck deformities (hyperextension of the PIP joints) that are commonly seen in late disease. Boutonnière (or buttonhole) deformities also occur as a result of hyperflexion of the PIP joints. If the clinical disease remains active, hand function deteriorates. Sudden loss of function of individual fingers may occur as a result of tendon rupture, which requires the expertise of a hand surgeon to repair.

Feet

Feet, particularly the MTP joints, are involved early in most patients with RA. Radiographic erosions occur at least as early in the feet as in the hands. Subluxation of the toes is common and leads to the dual problem of breakdown of the skin and ulcers on the top of the toes and malalignment of the MTP heads. Painful ambulation develops owing to loss of the cushioning pads that usually protect the heads of the MTP joints.

Wrists

The wrist joints are involved in most patients with RA; radial deviation is the rule, and patients with severe involvement may progress to volar subluxation. Even early in the course of the disease, synovial proliferation in and around the wrists may compress the median nerve, causing carpal tunnel syndrome (Fig. 264-5). Later, this synovial proliferation may invade tendons and lead to rupture of extensor tendons.

A

B

FIGURE 264-6.  Radiographs of the knees in the two most common forms of arthritis: rheumatoid arthritis and osteoarthritis. A, Severe involvement in rheumatoid arthritis, with almost complete symmetrical loss of joint space in both the medial and the lateral compartments, but with little subchondral sclerosis or osteophyte formation. B, Typical osteoarthritis, with severe, near-total loss of joint space of one compartment and a normal or actually increased joint space of the other compartment. Note also the significant subchondral sclerosis in the involved area, typical of osteoarthritis.

Large Joints

Involvement of knees, ankles, elbows, hips, and shoulders is common. Characteristically, the whole joint surface is involved in a symmetrical fashion. Therefore, RA is symmetrical not only from one side of the body to the other but also within the individual joint. In the case of the knee (Fig. 264-6A), the medial and lateral compartments are both severely narrowed in RA; in contrast, in patients with osteoarthritis (see Fig. 264-6B) typically only one compartment of the knee is involved. Synovial cysts may occur around any of the joints (large or small), and they occasionally manifest as soft, fluctuant masses that present diagnostic challenges. When the knee produces excess synovial fluid, it may accumulate in the popliteal space (popliteal or Baker’s cyst) (E-Fig. 264-2). These cysts can cause problems by pressing on the popliteal nerve, artery, or veins. Baker’s cysts may dissect into the tissues of the calf (usually posteriorly), or they may rupture into the upper calf. Dissection may produce only minor symptoms, such as a feeling of fullness; rupture of the cyst with extravasation of the

CHAPTER 264  Rheumatoid Arthritis  

E-FIGURE 264-2.  Arthrogram with a radiocontrast agent injected into the knee. The dye flows into the popliteal space and through a narrow channel into a large synovial cyst (Baker’s cyst) that has dissected into the soft tissue of the calf.

1757.e1

1758

CHAPTER 264  Rheumatoid Arthritis  

inflammatory content produces significant pain and swelling and may be confused with thrombophlebitis, the so-called pseudothrombophlebitis syndrome. Ultrasonography of the popliteal fossa and calf is useful to establish the correct diagnosis and rule out thrombophlebitis, which may be precipitated by popliteal cysts. Treatment of popliteal or any other cyst should be directed at interrupting the inflammatory process through an intra-articular injection of corticosteroid into the associated joint.

Neck

Although most of the axial skeleton is spared in RA, the cervical spine is commonly involved, particularly the C1-C2 articulation. Bony erosions and ligament damage can occur in this area and may lead to subluxation (see E-Fig. 264-1). Most often, subluxation at C1-C2 is minor and without accompanying symptoms; patients and caregivers need only be cautious and avoid actively forcing the neck into positions of flexion. Occasionally, subluxation at C1-C2 is severe and leads to compromise of the cervical cord with symptoms and in some cases death.

Cardiovascular Involvement

Cardiac involvement directly related to RA is uncommon; however, patients with RA have a significantly increased morbidity and mortality from coronary artery disease and congestive heart failure. A meta-analysis of observational studies has shown that the risk for incident cardiovascular disease is increased by 48% in patients with RA compared with that in the general population. The reasons are not clear, but chronic inflammation appears to be the major cause. Some of the medications used to treat RA and a sedentary lifestyle may be additional risk factors for the development of coronary artery disease. Pericardial effusions are common in RA (50% by echocardiography) but usually are asymptomatic. Rarely, long-standing pericardial disease may result in a fibrinous pericarditis, and patients may present clinically with constrictive pericarditis (Chapter 77). A population-based inception cohort of patients with RA in Olmstead County, Minnesota, has shown an increased incidence of venous thromboembolism compared with subjects without RA.

Other Joints

Wherever synovial tissue exists, RA can cause problems. The temporomandibular, cricoarytenoid, and sternoclavicular joints are examples of other joints that may be involved in RA. The cricoarytenoid joint is responsible for abduction and adduction of the vocal cords. Involvement of this joint may lead to a feeling of fullness in the throat, to hoarseness, and, rarely, when the cords are essentially fused in a closed position, to a syndrome of acute respiratory distress with or without stridor. In this latter situation, emergent tracheotomy may be life-saving.

Extra-Articular Manifestations

Systemic features of RA such as fatigue, weight loss, and low-grade fevers occur frequently. As with all the other extra-articular features, they are more common in those patients who possess RF or ACPA antibodies or both (Table 264-1) and respond to treatment of the RA.

Skin

FIGURE 264-7.  Rheumatoid nodules. Large rheumatoid nodules are seen in a classic location along the extensor surface of the forearm and in the olecranon bursa.

Subcutaneous nodules are seen in approximately one fifth of patients with RA, almost exclusively in those who are RF positive. Patients with nodules who are RF negative should be carefully scrutinized for a different diagnosis, such as chronic tophaceous gout. Nodules may occur almost anywhere (e.g., lungs, heart, eye), but most commonly they occur subcutaneously on extensor surfaces (particularly the forearms) (Fig. 264-7), over joints, or over pressure points. Rheumatoid nodules are firm on examination, usually are not tender, have a characteristic histologic picture, and are thought to be initiated by small vessel vasculitis. A syndrome of increased nodulosis, despite good control of the joint disease, has been described with methotrexate therapy (Fig. 264-8). Small vessel vasculitis,4 manifested as digital infarcts or leukocytoclastic vasculitis, may occur in RA (Fig. 264-9) and should prompt more aggressive DMARD treatment. A vasculitis of small and medium arteries that is indistinguishable from polyarteritis nodosa also can be seen with RA and requires aggressive systemic therapy. Finally, pyoderma gangrenosum occurs with increased frequency in association with RA.

TABLE 264-1  EXTRA-ARTICULAR MANIFESTATIONS OF RHEUMATOID ARTHRITIS Skin

Nodules, fragility, vasculitis, pyoderma gangrenosum

Heart

Pericarditis, premature atherosclerosis, vasculitis, valve disease, and valve ring nodules

Lung

Pleural effusions, interstitial lung disease, bronchiolitis obliterans, rheumatoid nodules, vasculitis

Eye

Keratoconjunctivitis sicca, episcleritis, scleritis, scleromalacia perforans, peripheral ulcerative keratopathy

Neurologic

Entrapment neuropathy, cervical myelopathy, mononeuritis multiplex (vasculitis), peripheral neuropathy

Hematopoietic

Anemia, thrombocytosis, lymphadenopathy, Felty’s syndrome

Kidney

Amyloidosis, vasculitis

Bone

Osteopenia

FIGURE 264-8.  Rheumatoid nodulosis. In this patient, multiple rheumatoid nodules are present over joints. In some cases, nodules may dominate the clinical picture. Rarely, this may be seen as a side effect of methotrexate therapy.

FIGURE 264-9.  Small vessel vasculitis. A and B, Rheumatoid vasculitis with small brown infarcts of palms and fingers in chronic rheumatoid arthritis. (Courtesy Dr. Martin Lidsky, Houston, TX.)

CHAPTER 264  Rheumatoid Arthritis  

1759

TABLE 264-2  CLASSIFICATION CRITERIA FOR RHEUMATOID ARTHRITIS* Morning stiffness (≥1 hr) Swelling (soft tissue) of three or more joints Swelling (soft tissue) of hand joints (PIP, MCP, or wrist) Symmetrical swelling (soft tissue) Subcutaneous nodules Serum rheumatoid factor Erosions and/or periarticular osteopenia in hand or wrist joints seen on radiograph *Criteria 1 through 4 must have been continuously present for 6 wk or longer, and criteria 2 through 5 must be observed by a physician. A classification of rheumatoid arthritis requires that 4 of the 7 criteria be fulfilled. MCP = metacarpophalangeal; PIP = proximal interphalangeal.

FIGURE 264-10.  Rheumatoid nodules in the lung. Chest radiograph demonstrates

discrete rheumatoid nodules in both right and left lower lobes. (Courtesy Dr. Martin Lidsky, Houston, TX.)

Pulmonary Manifestations

Pulmonary manifestations of RA include pleural effusions, rheumatoid nodules, and parenchymal lung disease (Chapters 84 and 92). Pleural effusions occur more commonly in men and are usually small and asymptomatic. Of interest, pleural fluid in RA is characterized by low levels of glucose and low pH and, therefore, may at times be confused with empyema. Rheumatoid nodules may occur in the lung, especially in men (Fig. 264-10); these are usually solid but may calcify, cavitate, or become infected. Rarely, pulmonary nodules rupture and produce a pneumothorax. If patients with RA are exposed to coal or silica dust, diffuse nodular densities may occur (Caplan’s syndrome). Differentiating rheumatoid nodules from lung cancer can be problematic, particularly if the lesion is solitary. Therefore, the presence of pulmonary nodules in a patient with RA should precipitate an aggressive diagnostic evaluation. Diffuse interstitial fibrosis occurs in RA and may progress to a honeycomb appearance on radiography with increasing dyspnea. Rarely, bronchiolitis obliterans can be seen, with or without organizing pneumonia.

Ophthalmologic Manifestations

The most common manifestation of RA in the eye is keratoconjunctivitis sicca (dry eyes) from secondary Sjögren’s syndrome (Chapter 268). Patients may have associated xerostomia (dry mouth), parotid gland swelling, or, occasionally, lymphadenopathy. Scleritis also can occur and may be painful, with progression to thinning of the sclera, with deep pigment showing through on physical examination, and may progress to perforation (scleromalacia perforans). Rarely, tendonitis of the superior oblique muscles can result in double vision (Brown’s syndrome).

Neurologic Manifestations

Peripheral nerve entrapment syndromes, including carpal tunnel syndrome (median nerve at the wrist), and tarsal tunnel syndrome (anterior tibial nerve at the ankle), are common in RA. Vasculitis can lead to a stocking and glove neuropathy or mononeuritis multiplex, both of which may require aggressive therapy. Subluxations at C1-C2 may produce myelopathy (see E-Fig. 264-1). Rheumatoid nodules in the central nervous system have been described but are rare and usually asymptomatic.

Felty’s Syndrome

Felty’s syndrome is the triad of RA, splenomegaly, and neutropenia. This complication is seen in patients with severe, RF/ACPA-positive disease and may be accompanied by hepatomegaly, thrombocytopenia, lymphadenopathy, and fevers. Most patients with Felty’s syndrome do not require special therapy; instead, treatment should be directed toward their severe RA. If severe neutropenia (Chapter 167) exists (10 joints (at least one small joint)

0 1 2 3 5

Serology (0-3) Neither RF or ACPA positive At least one test low positive titre At least one test high positive titre

0 2 3

Duration of Synovitis (0-1) 1 & ≤3 × ULN High +: >3 × ULN

Medium/Large joints Knee, hip, shoulders, elbow and ankle Small joints PIPs, MCPs, wrists and MTPs

E-FIGURE 264-3.  American College of Rheumatology/European League Against Rheumatism (ACR/EULAR) classification criteria for rheumatoid arthritis (RA). The classification criteria are shown as a flow diagram for initial decision making and also in tabular American College of Rheumatology/European League Against Rheumatism (ACR/EULAR) classification criteria for rheumatoid arthritis (RA). Form to calculate. ACPA = anti-citrullinated protein antibody; CRP = C-reactive protein; ESR = erythrocyte sedimentation rate; MCP = metacarpal phalangeal joint; MTP = metatarsal phalangeal joint; PIP = proximal interphalangeal joint; RA = rheumatoid arthritis; RF = rheumatoid factor; ULN = upper limit of normal. (From Aletaha D, Neogi T, Silman AJ, et al. 2010 Rheumatoid arthritis classification criteria. An American College of Rheumatology/European League Against Rheumatism collaborative initiative. Arthritis Rheum. 2010;62:2569-2581.

1760.e1

CHAPTER 264  Rheumatoid Arthritis  

TABLE 264-5  GUIDELINES FOR USE OF GLUCOCORTICOIDS Avoid use of glucocorticoids without DMARDs Prednisone, >10 mg/day, is rarely indicated for articular disease Taper to the lowest effective dose Use as “bridge therapy” until DMARD therapy is effective Remember prophylaxis against osteoporosis DMARD = disease-modifying antirheumatic drug.

a disease of glucocorticoid deficiency (an issue that remains unresolved). As was the case with the first patient treated in 1948, glucocorticoids are dramatically and rapidly effective in patients with RA. Not only are glucocorticoids useful for symptomatic improvement but they also significantly decrease the radiographic progression of RA. A2  However, the toxicities of long-term therapy are extensive and potentially devastating. Therefore, the optimal use of these drugs requires an understanding of several principles (Table 264-5). Glucocorticoids remain among the most potent anti-inflammatory treatments available; for this reason, and because of their rapid onset of action, they are ideally suited to help control the inflammation in RA while the much slower-acting DMARDs are starting to work. Prednisone, the most commonly used glucocorticoid, should rarely be used in doses higher than 10 mg/day to treat the stiffness and articular manifestations of RA. At this dose at the start of methotrexate-based treatment, the addition of prednisone reduces erosive joint damage, disease activity, physical disability, and the use of biologic treatment at 2 years. The dose should be slowly tapered to the lowest effective dose, and the concomitant DMARD therapy should be adjusted to make this possible. Glucocorticoids should rarely, if ever, be used to treat RA without concomitant DMARD therapy. The paradigm is to shut off inflammation rapidly with glucocorticoids and then to taper them as the DMARD is taking effect (“bridge therapy”). In all patients receiving glucocorticoids, strong measures should be taken to prevent osteoporosis. Bisphosphonates have been shown to be particularly effective in this regard but are contraindicated in women of childbearing age. Higher doses of glucocorticoids may be necessary to treat extra-articular manifestations, especially vasculitis and scleritis.

Disease-Modifying Antirheumatic Drugs

DMARDs are a group of medications that have the ability to halt the disease process in the synovium and to modify or change the disabling potential of RA. These drugs have the ability to halt or slow the radiographic progression of RA.

Conventional Disease-Modifying Antirheumatic Drugs

Included in this group of medications are methotrexate, sulfasalazine (Azulfidine), gold, antimalarials (hydroxychloroquine [Plaquenil] and others), leflunomide (Arava), azathioprine (Imuran), minocycline, and the newly approved tofacitinib (Xeljanz). It is critically important that clinicians and patients understand that conventional DMARDs take 2 to 6 months to exert their maximal effect, and all require some monitoring (Table 264-6). Therefore, other measures, such as glucocorticoid therapy, may be needed to control the disease while these medications are starting to work. These DMARDs have been shown to be effective in treating both early and more advanced RA. Until additional research elucidates factors that allow selection of the best initial therapy for each patient, the choice will depend on patient and physician concerns about toxicity and monitoring issues, as well as the activity of disease and presence of comorbid conditions. The critical issue is not which DMARD to start first but rather getting the DMARD therapy started early in the disease process.

Methotrexate

Methotrexate is the preferred initial DMARD of most rheumatologists, in part because patients have a more durable response, and because, with correct monitoring, serious toxicities are rare. A3  Methotrexate is dramatically effective in slowing radiographic progression and is usually given orally in doses ranging from 5 to 30 mg/week as a single dose. This once-per-week administration is worthy of emphasis; prior experience with daily therapy in psoriasis has demonstrated the importance of allowing the liver time to recover between doses. Oral absorption of methotrexate is variable; subcutaneous injections of methotrexate may be effective if oral treatment is not.  Side effects of methotrexate include oral ulcers, nausea, hepatotoxicity,  bone marrow suppression, and pneumonitis. With the exception of pneumonitis, these toxicities respond to dose adjustments. Monitoring of blood counts and liver blood tests (albumin and aspartate aminotransaminase [SGOT] or alanine aminotransferase [SGPT]) should be done every 4 to 8 weeks initially and, when stable, every 3 months thereafter, with adjustments in the dose of methotrexate as needed. Renal function is critical for clearance of methotrexate; previously stable patients may experience severe toxicities if renal function deteriorates. Pneumonitis, although rare, is less predictable and can be fatal, particularly if the methotrexate is not stopped or is restarted. Folic acid, 1 to 4 mg/day, can significantly decrease most methotrexate toxicities without interfering with efficacy. If methotrexate alone does not sufficiently control

1761

TABLE 264-6  CAVEATS FOR MONITORING DISEASEMODIFYING ANTIRHEUMATIC DRUG THERAPIES* MEDICATION

CAVEATS

Prednisone

Use as bridge to effective DMARD therapy. Prophylaxis for osteoporosis? (see Table 264-5)

Hydroxychloroquine

Keep dosage lower than 6.5 mg/kg/day. Yearly eye checkup by ophthalmologist

Sulfasalazine

CBC for neutropenia, initially every month, then every 6 mo

Methotrexate

CBC and SGOT/SGPT every 8-12 wk when dose is stable. Many toxicities respond to folic acid or small dose reduction. If pneumonitis, stop and do not restart. Decreasing renal function may precipitate toxicities. Absolute contraindication in pregnancy

Leflunomide

CBC and SGOT/SGPT every 4-8 wk; long half-life may require cholestyramine washout; absolute contraindication in pregnancy

TNF inhibitors

If fevers or infectious symptoms of any kind, stop until symptoms resolve; aggressively work up and treat possible infections. May precipitate congestive heart failure, demyelinating syndromes, or lupus-like syndromes

*Patients receiving DMARDs, both conventional and biologic, should be monitored by a rheumatologist. CBC = compete blood count; DMARD = disease-modifying antirheumatic drug; SGOT = serum glutamate oxaloacetate transaminase (aspartate aminotransferase); SGPT = serum glutamate pyruvate transaminase (alanine aminotransferase); TNF = tumor necrosis factor.

disease, it is combined with other DMARDs. A4  Methotrexate in combination with virtually any of the other DMARDs (conventional or biologic) has been shown to be more effective than either drug alone.

Leflunomide

Leflunomide, a pyrimidine antagonist, has a very long half-life and is most commonly started at 10 to 20 mg/day orally. Diarrhea is the most common toxicity and responds to dose reduction, and doses of leflunomide of 10 to 20 mg three to five times per week are frequently used. Also, because of the long half-life and teratogenic potential of leflunomide, women wishing to become pregnant who have previously received leflunomide, even if therapy was stopped years ago, should have blood levels drawn. If toxicity occurs or if pregnancy is being considered, leflunomide can be rapidly eliminated from the body by treatment with cholestyramine. Laboratory monitoring for hematologic and hepatic toxicity should be done during treatment with leflunomide, as recommended for methotrexate.

Antimalarial Drugs

The antimalarial drugs hydroxychloroquine (Plaquenil) and chloroquine are frequently used for the treatment of RA. They have the least toxicity of any of the DMARDs and do not require monitoring of blood tests. Yearly monitoring by an ophthalmologist after 5 years of therapy is recommended to detect any signs of retinal toxicity (rare). Hydroxychloroquine is the most commonly used preparation and is given orally at 200 to 400 mg/day. These drugs are frequently used in combination with other DMARDs, particularly methotrexate. Hydroxychloroquine decreases cholesterol levels and has recently been shown to decrease the incidence of diabetes in patients with RA.

Sulfasalazine

Sulfasalazine is an effective treatment when given in doses of 1 to 3 g/day. Monitoring of blood counts, particularly white blood cell counts, in the first 6 months is recommended. Sulfasalazine and hydroxychloroquine are often combined with methotrexate, a regimen referred to as triple therapy.

Minocycline

Minocycline 100 mg twice daily has been shown to be an effective treatment for RA, particularly when used in early, RF-positive disease. Chronic therapy (>2 years) with minocycline may lead to cutaneous hyperpigmentation. Minocycline has been associated with drug-induced lupus.

Gold

Gold, the oldest DMARD, is effective but cumbersome and is currently  rarely used.

Tofacitinib

Tofacitinib (Xeljanz) has been recently approved for the treatment of RA in the United States. It is the first Jak kinase inhibitor to be approved for RA. It is given orally at a dose of 5 mg twice daily, and complete blood count and liver function tests should be monitored. Additional toxicity concerns include infections, including tuberculosis, and malignancies. Tofactinib has been

1762

CHAPTER 265  The Spondyloarthropathies  

shown to be effective as initial DMARD therapy, A5  when combined with methotrexate in patients who have had incomplete responses to methotrexate, and in patients who have failed TNF inhibitors. Tofacitinib has not yet been approved in Europe.

Biologic Disease-Modifying Antirheumatic Drugs

Recent research has continued to elucidate the central role that cytokines, most notably TNF-α, IL-1 and IL-6, play in the pathophysiology of RA (Chapter 36). This led directly to the development and clinical use of biologic agents directed against TNF-α11 (etanercept [Enbrel], infliximab [Remicade], adalimumab [Humira], golimumab [Simponi], and certolizumab [Cimzia]), IL-1 (anakinra [Kineret]), and IL-6 (tocilizumab [Actemra). Additionally, monoclonal antibodies that deplete B cells (anti-CD20, rituximab [Rituxan]) and that block the second signal for T cell activation (abatacept [Orencia]) are effective agents in the treatment of RA. All patients with RA receiving biologic therapies should be monitored by a rheumatologist, and their physicians should be aware of the risk for infections that are often atypical.12 All the biologics, when combined with methotrexate, have been shown to decrease disease activity and slow radiographic progression in patients with RA with active disease despite methotrexate. A6  Early treatment with abatacept plus methotrexate was shown to result in greater sustainable clinical, functional, and radiographic benefits than methotrexate alone, with acceptable safety and tolerability, in early erosive RA. Currently, biologic agents should not be used in combination with each other because all studies to date have shown a significant increase in infections. See Chapter 36 for further details on the use of biologic agents in the treatment of RA.

The Order of Therapy in Rheumatoid Arthritis

Several randomized double-blind trials have elucidated the order of therapy in RA. The Treatment of Early Aggressive Rheumatoid Arthritis (TEAR) trial nicely showed that initial therapy with methotrexate in patients with poorprognosis RA was not inferior at 2 years to initial combinations of either conventional DMARDs or the combination of methotrexate and etanercept. A7  The Rheumatoid Arthritis: Comparison of Active Therapies (RACAT) trial has also shown that in those patients who are not controlled on methotrexate alone, the strategy of initially adding sulfasalazine and hydroxychloroquine to methotrexate (triple therapy) was not inferior to the addition of etanercept to methotrexate. A8  Therefore, because of the huge economic advantages, the typical RA patient should be started on methotrexate monotherapy, and, if not controlled after 3 months on maximum methotrexate, advanced to triple therapy. If the patient does not achieve adequate control after 3 to 6 months on triple therapy, either a TNF inhibitor or abatacept should be added to methotrexate.

Treatment of Underlying Conditions

Optimal care of patients with RA requires recognition of the associated comorbid conditions, including an increased risk for cardiovascular death, osteoporosis, infections (especially pneumonia), and certain cancers.

Cardiovascular Disease

Increasingly, cardiovascular disease is being recognized as the cause of much of the excess mortality in RA. Various factors contribute to this mortality, including sedentary lifestyle and glucocorticoid therapy. However, a strong association between chronic inflammation and cardiovascular disease has been identified, and it is likely that this may be the most significant factor. Therapies that control RA earlier and better can be expected to decrease cardiovascular morbidity and mortality. Both methotrexate and TNF inhibitors have been shown to decrease cardiovascular mortality in RA. Clinicians should consider RA a risk factor for cardiovascular disease and should aggressively address other cardiovascular risk factors (Chapter 52) in their patients with RA.13

found that 50% of patients with RA have had to stop working after 10 years (~10 times the average rate). Patients who are RF or ACPA positive and those who are positive for the shared epitope have a worse prognosis, with more erosions and more extra-articular disease (see Table 264-1). Once deformities are found on examination or erosions on radiography, the damage is largely irreversible. It has been clearly shown that erosions occur in most patients in the first 1 to 2 years and that the rate of radiographic damage can be affected by early therapy. A recent cohort study has shown that the aggressive use of disease-modifying agents and the introduction of biologic agents have been associated with substantial reductions in disability. Therefore, early DMARD therapy is critical. Although limited long-term data are available, the current information strongly suggests that patients have the opportunity to benefit greatly if the newer principles of therapy are practiced.

FUTURE DIRECTIONS

Significant advances in the effective treatment of RA have come from an understanding of the cytokine imbalance that accompanies this disease. Much research is focused on the further development of biologic products to modulate this balance. There remains a critical need for a cytokine thermostat that would allow titration of the desired cytokine balance to control disease without altering critical immune functions. Even with existing therapies, there are many different effective options for patients with RA. The challenge for the clinician is to pick the right option for each patient. Few data are currently available to aid in this choice, and the establishment of parameters, genetic or otherwise, that would allow selection of the best initial option for each patient would be a major breakthrough. Finally, elucidation of the trigger or triggers for RA may allow the development of strategies to prevent the onset of clinical disease.

Grade A References A1. Goekoop-Ruiterman YP, de Vries-Bouwstra JK, Allaart CF, et al. Comparison of treatment strategies in early rheumatoid arthritis: a randomized trial. Ann Intern Med. 2007;146:406-415. A2. Bakker MF, Jacobs JW, Welsing PM, et al. Low-dose prednisone inclusion in a methotrexate-based, tight control strategy for early rheumatoid arthritis: a randomized trial. Ann Intern Med. 2012;156:329-339. A3. Lopez-Olivo MA, Siddhanamatha HR, Shea B, et al. Methotrexate for treating rheumatoid arthritis. Cochrane Database Syst Rev. 2014;6:CD000957. A4. Moreland LW, O’Dell JR, Paulus HE, et al. TEAR Investigators. A randomized comparative effectiveness study of oral triple therapy versus etanercept plus methotrexate in early aggressive rheumatoid arthritis. Arthritis Rheum. 2012;64:2824-2835. A5. Lee EB, Fleischmann R, Hall S, et al. Tofacitinib versus methotrexate in rheumatoid arthritis. N Engl J Med. 2014;370:2377-2386. A6. Nam JL, Ramiro S, Gaujoux-Viala C, et al. Efficacy of biological disease-modifying antirheumatic drugs: a systematic literature review informing the 2013 update of the EULAR recommendations for the management of rheumatoid arthritis. Ann Rheum Dis. 2014;73:516-528. A7. O’Dell JR, Curtis JR, Mikuls TR, et al. Validation of the methotrexate-first strategy in patients with early, poor-prognosis rheumatoid arthritis: results from a two-year randomized, double-blind trial. Arthritis Rheum. 2013;65:1985-1994. A8. O’Dell JR, Mikuls TR, Taylor TH, et al. Therapies for active rheumatoid arthritis after methotrexate failure. N Engl J Med. 2013;369:307-318.

GENERAL REFERENCES For the General References and other additional features, please visit Expert Consult at https://expertconsult.inkling.com.

Other Associated Diseases

Osteoporosis is common in patients with RA, and early treatment results in long-term dividends. Patients with RA are at an increased risk for infections, and some forms of treatment further increase this risk. Patients should be cautioned to seek medical attention early for even minor symptoms suggestive of infection, especially if receiving biologic therapy. All patients with RA should receive a pneumococcal vaccine at appropriate intervals and yearly influenza vaccinations. Finally, patients with RA have an increased risk for lymphoma. Occasionally, B-cell lymphomas are associated with immunosuppression and regress after immunosuppression is discontinued. Patients with RA have significantly decreased risk of developing colon cancer. This is thought to be secondary to chronic inhibition of COX by NSAIDs.

PROGNOSIS

RA is not a benign disease and is not limited to the joints. Once established, RA is a lifelong progressive disease that produces significant morbidity in most patients and premature mortality in many. Long-term studies have

265  THE SPONDYLOARTHROPATHIES ROBERT D. INMAN

  COMMON FEATURES OF SPONDYLOARTHRITIS DEFINITION

Spondyloarthritis (SpA) encompasses a group of clinical syndromes that are linked in terms of disease manifestations and genetic susceptibility. The

CHAPTER 264  Rheumatoid Arthritis  

GENERAL REFERENCES 1. Avina-Zubieta JA, Thomas J, Sadatsafavi M, et al. Risk of incident cardiovascular events in patients with rheumatoid arthritis: a meta-analysis of observational studies. Ann Rheum Dis. 2012;71: 1524-1529. 2. Yarwood A, Huizinga TW, Worthington J. The genetics of rheumatoid arthritis: risk and protection in different stages of the evolution of RA. Rheumatology (Oxford). 2014;[Epub ahead of print]. 3. McInnes IB, Schett G. The pathogenesis of rheumatoid arthritis. N Engl J Med. 2011;365: 2205-2219. 4. Makol A, Matteson EL, Warrington KJ. Rheumatoid vasculitis: an update. Curr Opin Rheumatol. 2015;27:63-70. 5. Mjaavatten MD, Bykerk VP. Early rheumatoid arthritis: the performance of the 2010 ACR/EULAR criteria for diagnosing RA. Best Pract Res Clin Rheumatol. 2013;27:451-466. 6. Whiting PF, Smidt N, Sterne JA, et al. Systematic review: accuracy of anti-citrullinated peptide antibodies for diagnosing rheumatoid arthritis. Ann Intern Med. 2010;152:456-464. 7. Vliet Vlieland TP, van den Ende CH. Nonpharmacological treatment of rheumatoid arthritis. Curr Opin Rheumatol. 2011;23:259-264.

1762.e1

8. Gaujoux-Viala C, Gossec L, Cantagrel A, et al. Recommendations of the French Society for Rheumatology for managing rheumatoid arthritis. Joint Bone Spine. 2014;81:287-297. 9. Gramling A, O’Dell JR. Initial management of rheumatoid arthritis. Rheum Dis Clin North Am. 2012;38:311-325. 10. Singh JA, Furst DE, Bharat A, et al. 2012 Update of the 2008 American College of Rheumatology recommendations for the use of disease-modifying antirheumatic drugs and biologic agents in the treatment of rheumatoid arthritis. Arthritis Care Res. 2012;64:625-639. 11. Krishnan E, Lingala B, Bruce B, et al. Disability in rheumatoid arthritis in the era of biological treatments. Ann Rheum Dis. 2012;71:213-218. 12. Singh JA, Wells GA, Christensen R, et al. Adverse effects of biologics: a network meta-analysis and Cochrane overview. Cochrane Database Syst Rev. 2011;2:CD008794. 13. Martin-Martinez MA, Gonzalez-Juanatey C, Castaneda S, et al. Recommendations for the management of cardiovascular risk in patients with rheumatoid arthritis: scientific evidence and expert opinion. Semin Arthritis Rheum. 2014;44:1-8.

1762.e2

CHAPTER 264  Rheumatoid Arthritis  

REVIEW QUESTIONS 1. A 32-year-old woman presents with 3 weeks of symmetrical swelling and stiffness in her PIPs and MCPs. The most helpful laboratory test result in establishing a diagnosis of RA in this patient would be: A. ESR of 65 mm/hour. B. RF of 120 IU (normal 3 mo that improve with exercise but are not relieved by rest Limitation of motion of the lumbar spine in both sagittal and frontal planes Limitation of chest expansion RADIOLOGIC CRITERIA Sacroiliitis: grade ≥ 2 bilateral or grade 3 or 4 unilateral GRADING Definite AS if the radiologic criterion is associated with at least one clinical variable Probable AS if:   The three clinical criteria are present   The radiologic criterion is present without the clinical criteria AS = ankylosing spondylitis.

patient from sleep is often a clue to inflammatory back pain that may have been misdiagnosed as the pain of degenerative disc disease, the latter being a much more common cause of low back pain in the population at large. The pain typically occurs in the region of the sacroiliac joints, with or without slight radiation to the buttock area. Midthoracic pain and cervical pain, particularly at night, are less common but strongly suggest inflammatory back pain when they occur. Fatigue is also a suggestive symptom and is often a major concern for the typical young male patient who has a high functional target in terms of sports and recreation. If the inflammation is inadequately controlled, there is increasing stiffness that may persist most of the day, as well as progressive loss of mobility and flexibility. Peripheral oligoarthritis is seen in up to 30% of patients with AS. Typically, it is an asymmetrical oligoarthritis with a predilection for the lower extremities. It is important to ask about concurrent or previous tendinitis (e.g., Achilles tendinitis) or heel pain (e.g., plantar fasciitis), because either may reflect an enthesitis that is part of the clinical picture. Involvement of the hip can occur at any point in the course of AS and can follow a course to joint destruction. A hip flexion contracture on this basis may contribute to increasing stoop on standing and walking, which may otherwise be attributed to spinal involvement in the disease. Extra-articular features most commonly involve the eye. Ocular involvement may occur in up to 40% of AS patients, most typically acute anterior uveitis (iritis). The uveitis often manifests as a slight impairment in visual acuity, with accompanying photophobia and eye pain. Typically, it is unilateral and recurrent. IBD and psoriasis occur in approximately 10% of AS cohorts. Less common manifestations include aortic insufficiency, cardiac conduction defects, and pulmonary fibrosis.

DIAGNOSIS

Physical Examination

Physical examination of the spine characteristically indicates restricted movement, which in the early stages may reflect paraspinal muscle spasm in part; late in the course it reflects ankylosis of the zygapophyseal joints and syndesmophyte bridging of the vertebral bodies. Forward flexion is restricted and can be monitored by Schober’s test. This test is used to measure mobility in the lower part of the back: with the patient standing upright, a 10-cm span is marked from the fifth lumbar vertebra upward. On maximal forward flexion, the distance between the marks is remeasured. With normal spinal mobility, the flexed distance should register as 15 cm or an increment of 5 cm. Thoracic involvement is measured in chest expansion, with the chest circumference at maximal inspiration being more than 5 cm greater than the circumference at maximal expiration. Changes in cervical mobility can be measured as the occiput-to-wall distance, with the patient’s heels against the wall as the patient attempts to touch the back of the head to the wall. Restricted spinal mobility early in the course of the disease may best be detected by lateral spinal flexion, measured as the difference in the finger-to-floor distance when standing erect compared with maximal bending to the side. Inflammation in the sacroiliac joint may be reflected by joint line tenderness to direct pressure or by the FABERE test (for Flexion, ABduction, External Rotation, and Extension) or Gaenslen maneuver. In the former, the patient lies supine while the examiner flexes and externally rotates the hip. In the latter, the examiner extends the hip by letting the leg dangle off the side of the examining table. In both cases,

CHAPTER 265  The Spondyloarthropathies  

It is clear that HLA is strongly associated with SpA, yet the prevalence of HLA-B27 varies widely in different racial and ethnic clusters around the world. It is virtually absent in aboriginal populations in Australia, occurs in 1% of the population in Japan, in 7% in northern European countries, and in 50% in some of the native tribes in western Canada. The environmentalgenetic interaction that may account for the expansion or restriction of this gene in human populations is unknown, but some evidence indicates that B27 may confer a more effective host response to some viruses, such as hepatitis C. This variability has a practical impact for the clinician. Because the relative risk conferred by a gene reflects the prevalence of the gene in affected individuals versus its prevalence in the normal population, the relative risk for SpA is higher in a population in which the gene is uncommon (e.g., Japan) than in a population in which B27 is more common (e.g., Scandinavia). In the North American white population, the prevalence of the gene is approximately 7%. Thus, there is a 7% “false-positive” rate if one is attempting to use the gene as a diagnostic marker to decipher the cause of chronic back pain in an unselected population of patients. In contrast, 90% of patients with AS are B27 positive, so there is a 10% “false-negative” rate when using the test diagnostically. The key factor is pretest probability. In a patient with chronic back pain that is clearly inflammatory in character, the addition of B27 positivity combines to strengthen the likelihood of AS accounting for the back problem. The presence of distinctive extra-articular features (e.g., uveitis) further increases this likelihood. Other steps have also been taken to define the mechanism whereby B27 confers disease susceptibility, in addition to that of uniquely presenting an

1765.e1

arthritogenic peptide to T cells. According to the theory of molecular mimicry, an autoimmune response can ensue after an infection if the immune response against the pathogen cross-reacts with host antigens. There is a degree of sequence homology between B27 and several candidate gramnegative enteric bacteria, and there is evidence for cross-reacting monoclonal antibodies, but the significance of such homology for disease pathogenesis remains unresolved. It has also been argued that B27 is distinctive in its propensity to misfold in the endoplasmic reticulum, which may induce a pro-inflammatory cascade called the unfolded protein response. Furthermore, B27 may have a distinct tendency to form heavy-chain homodimers at the cell surface, and the possible consequences of this change for the immune response are under investigation. There has also been investigation into the alteration of primary host-pathogen interactions, such as modulation of pathogen invasion, intracellular replication, and clearance. However, no definitive allele-specific relationships have been demonstrated in these studies. In B27 transgenic rats, the spontaneous development of pathology that is strikingly similar to human SpAs has supported the notion that B27 itself is the critical genetic factor in disease pathogenesis. These animals demonstrate pathology similar to that of Crohn’s disease in the GI tract, spondylitis, peripheral arthritis, uveitis, and psoriasiform skin and nail changes. Of interest, if such animals are raised in a germ-free environment, there is a marked reduction in joint and gut disease, implying a dynamic interrelationship between microbial triggers and background host genes that seems to recapitulate the situation seen clinically.

1766

CHAPTER 265  The Spondyloarthropathies  

stress is placed on the sacroiliac joint and may reproduce the back pain if it derives from this site.

Laboratory Findings

Laboratory tests in the evaluation of inflammatory back pain are relatively nonspecific. The ESR and CRP are typically elevated, but normal levels do not exclude inflammatory back pain, and the degree of elevation is typically less than would be seen in acute RA. Anemia of chronic disease may be observed if the condition is long-standing. HLA-B27 is rarely the definitive factor for diagnosis, and the false-positive and false-negative rates have already been discussed; however, in the setting of characteristic back symptoms, the test has reasonably high sensitivity and specificity.

Imaging

Radiographic assessment is important for confirmation of disease, but early in the course there may be no radiographic changes in the sacroiliac joints. If the clinician has a high index of suspicion in such cases, MRI may improve the sensitivity of the plain radiograph because inflammatory changes on MRI predate radiographic changes.11,12 When ordering x-rays, specific views of the sacroiliac joints can be requested. A routine anteroposterior pelvic radiograph is generally the standard diagnostic x-ray. The classic findings are bilateral changes in the sacroiliac joints (Fig. 265-2). Abnormalities include erosions in the joint line, pseudowidening, subchondral sclerosis, and, finally, ankylosis, reflecting complete bony replacement of the sacroiliac joints. Radiographs of the spine may reveal squaring of the vertebral bodies (loss of the normal anterior concavity of the lumbar vertebra) and “shiny corners” (subchondral sclerosis at the upper edge of the vertebral body), both of which are manifestations of enthesitis. Syndesmophytes, which represent marginal bridging of the vertebrae (Figs. 265-3 and 265-4), eventually develop and make the diagnosis clear. Because ankylosis of the apophyseal joints may occur without syndesmophyte formation, it is important to assess the posterior joints on the lateral lumbosacral spine views, as well as the anterior margin of the vertebrae. Eventually, the changes may result in a “bamboo spine,” so called because the bridging syndesmophytes can mimic the appearance of bamboo. It is now appreciated that osteoporosis (Chapter 243) is a significant feature of AS, probably reflecting both the local chronic inflammation and the abnormal biomechanical loading of the vertebrae as the disease progresses.

chronic pain and deformity may develop over time. In both early and late phases of the disease, there may be a significant impact on work disability and quality of life. In only a minority of patients does the full-blown picture of a bamboo spine eventually develop, but there are few variables that can reliably aid in prognosticating the course. At present, the strongest predictor of new syndesmophyte formation is the presence of syndesmophytes at baseline. In AS patients in whom new, refractory spinal pain develops, an intervertebral fracture should be considered, which can occur after only minimal trauma. Additional late complications may include cauda equina syndrome, osteoporotic compression fractures, spondylodiscitis, and restrictive lung disease.

Reactive Arthritis

DEFINITION

ReA is an aseptic arthritis that occurs subsequent to an extra-articular infection, most typically of the GI or GU tract. In the GI tract, the key pathogens

Differential Diagnosis

The differential diagnosis of AS includes the following: osteitis condensans ilii; diffuse idiopathic skeletal hyperostosis (DISH); the syndrome of synovitis, acne, pustulosis, hyperostosis, and osteomyelitis (SAPHO); and some induced hyperostotic states (vitamin A intoxication, fluorosis). New bone formation occurs in degenerative disc disease, but the bulky horizontal appearance of osteophytes is usually easily distinguished from that of syndesmophytes, and narrowing of the disc space is not a feature of AS. Osteoarthritis of the sacroiliac joint has recently been recognized as having a higher prevalence than previously appreciated. The clinical course and severity of AS are highly variable. Inflammatory back pain and stiffness dominate the picture in the early stages, whereas

FIGURE 265-2.  Bilaterally symmetrical sacroiliitis in ankylosing spondylitis.

FIGURE 265-3.  Left, Lumbar spondylitis in ankylosing spondylitis, with symmetrical marginal bridging syndesmophytes and calcification of the spinal ligament. Right, The bulky, nonmarginal, asymmetrical syndesmophytes of reactive arthritis with lumbar spondylitis.

FIGURE 265-4. A 34-year-old man who has had ankylosing spondylitis for 9 years and neck pain. Radiographs demonstrate narrowing of the C2-C3 apophyseal joints posteriorly and anterior bridging marginal syndesmophytes extending from C2 to C5.

CHAPTER 265  The Spondyloarthropathies  

1767

are Salmonella typhimurium, Yersinia enterocolitica, Shigella flexneri, and Campylobacter jejuni. In the GU tract, Chlamydia trachomatis is the most common offender.

EPIDEMIOLOGY

The true incidence and prevalence of ReA are not well defined. In epidemics involving Salmonella (Chapter 308) or Yersinia (Chapter 312), it is estimated that ReA develops in 2 to 7% of infected individuals but in as many as 20% of B27-positive infected individuals. In such epidemic studies, B27 confers risk not only for the onset of arthritis but also for axial involvement and chronicity. Genetic variants in toll-like receptor 2 (TLR-2) are associated with acute ReA, thus implicating host innate immunity as central in ReA. The variability in the rate of ReA is determined by the heterogeneity of the cohorts reported, which introduces confounding variables of different genetic backgrounds in the population and different species of pathogens. Even in the setting of an epidemic point source outbreak, the inoculum varies widely among the exposed individuals, and the genetic makeup of the population at risk (e.g., the prevalence of B27) may differ greatly among different studies. Case ascertainment and relative risk are even more difficult to determine for post-Chlamydia ReA. Young adults in the United States have a high prevalence of asymptomatic Chlamydia carriage in the GU tract, and establishing a causal link between Chlamydia and synovitis can be difficult. Nevertheless, it is with Chlamydia that ReA has been most intensively studied.

FIGURE 265-5.  Keratoderma blennorrhagicum of the feet in reactive arthritis.

PATHOBIOLOGY

Although immunofluorescence studies have identified bacterial antigens in the joints of patients with ReA after both GI and GU infections, it is primarily in post-Chlamydia ReA that results of polymerase chain reaction studies on synovial tissues have most consistently been positive, suggesting that viable Chlamydia may persist in the joints of such patients, albeit in a metabolically altered state. Typically, the onset of arthritis occurs 1 to 3 weeks after the GI or GU infection, but the temporal details are often difficult to define precisely. Although the definition of aseptic arthritis after an extra-articular infection may include a broader range of pathogens (e.g., Chlamydia pneumoniae), sites of infection (e.g., streptococcal pharyngitis), and types of infections (e.g., Giardia infections of the GI tract), these clinical scenarios have not generally been included in the category of ReA. They lack the other associated clinical features of the SpA group of diseases, and they lack an association with B27.

DIAGNOSIS

The pattern of joint involvement in ReA is one of asymmetrical oligoarthritis with a predilection for the lower extremity, a pattern shared by most SpA syndromes. Enthesitis may present as Achilles tendinitis or plantar fasciitis. Dactylitis, appearing as a sausage digit, may also be seen. Dactylitis is the net result of inflammatory changes affecting the joint capsule, entheses, periarticular structures, and periosteal bone. Sacroiliitis may be seen in the acute phase, but radiographic changes are seen largely in patients with a more chronic course.13 When ReA is accompanied by certain extra-articular features such as urethritis, conjunctivitis, or mucocutaneous lesions, the term Reiter’s syndrome has been applied historically, but it is no longer in common use. The urethritis may manifest as dysuria or discharge, and the rash as circinate balanitis, which appears as vesicles or shallow ulcerations on the glans penis. Painless lingual or oral ulcerations may also be seen. The fact that the cervicitis may be less symptomatic could partially account for the underdiagnosis in women. The classic skin manifestation of ReA is keratoderma blennorrhagicum, a painless papulosquamous eruption on the palms or soles (Fig. 265-5). Occasionally, nail dystrophy with pitting and onycholysis or subungual keratosis can be seen. The conjunctivitis can be bilateral and painful; in contrast, the acute anterior uveitis that can also be seen in this setting tends to be less painful and unilateral. Radiographic changes of ReA can be seen in the involved peripheral joints, with early findings consisting of soft tissue swelling and juxta-articular osteopenia. Areas of periostitis and new bone formation may develop in peripheral joints. When changes in the sacroiliac joints are seen, they are typically asymmetrical (Fig. 265-6), in contrast to the symmetrical pattern seen in AS. In the chronic phase, syndesmophytes may develop, but they are described as bulky, nonmarginal, often asymmetrical formations that differ from the classic syndesmophytes of AS. The frequency with which ReA evolves into bona fide AS has not been determined definitively.

FIGURE 265-6.  Bilaterally asymmetrical sacroiliitis in reactive arthritis. Erosions, pseudowidening, and ileal sclerosis are present.

Differential Diagnosis

The most important differential diagnosis for such reactive arthropathies is septic arthritis. Both Yersinia and Salmonella can cause septic arthritis, so an appropriate culture of synovial fluid should precede the diagnosis of ReA whenever possible. The course of ReA is variable, and few prognostic markers are available for the clinician to predict the course in an individual case. The majority of patients have an initial episode lasting 2 to 3 months, but synovitis may persist for a year or longer. In one 5-year follow-up of a point source cohort of post-Salmonella ReA, 20% of patients had ongoing inflammatory joint disease, and some degree of functional disability was observed in 30% of patients 5 years after the onset of disease.

REACTIVE ARTHRITIS AND HUMAN IMMUNODEFICIENCY VIRUS

An aggressive form of SpA may be seen in patients who are concomitantly infected with HIV.14 There is no increased frequency of ReA in patients with HIV, but HIV may alter the course of these arthropathies, with a tendency for a more aggressive and more refractory joint disease. Aggressive skin and joint disease may be seen in patients in whom PsA develops in the setting of HIV infection. Most North American patients with the HIV-ReA constellation are B27 positive, but studies of comparable patients in Africa have found a sizable B27-negative component in such patients. The arthritis in these patients falls into two clinical patterns: (1) an additive, asymmetrical polyarthritis or (2) an intermittent oligoarthritis that most commonly affects the lower extremities. Enthesitis, fasciitis, conjunctivitis, and urethritis can all be

1768

CHAPTER 265  The Spondyloarthropathies  

seen in such patients. Sacroiliitis can occur, although extensive spinal syndesmophyte formation is not common.

Psoriatic Arthritis

EPIDEMIOLOGY

PsA develops in 5 to 7% of patients with psoriasis. Although most cases arise in patients with established cutaneous disease, some patients (particularly children) have arthritis that antedates the appearance of the skin lesions.15 Although the extent of psoriatic skin disease correlates poorly with the development of arthritis, the risk for PsA increases with a family history of SpA. The age at onset can range from 30 to 55 years, with an equal predilection for PsA in women and men. Psoriatic spondylitis has a slight male preponderance. Two large prospective studies suggest that obesity is a significant risk factor for psoriatic arthritis.16,17

PATHOBIOLOGY

The genetic associations with PsA are complex. Psoriasis itself is associated with several HLA loci; some B alleles have been reported, but the dominant element is HLA-Cw6. HLA-B39 and HLA-B27 have been associated with sacroiliitis and axial involvement. No etiologic agent has been proved in PsA, although some investigators have proposed that the disease process represents ReA in response to cutaneous bacteria. The histopathology of the synovitis of PsA is comparable to that of the other forms of SpA, with the absence of the local production of immunoglobulin and rheumatoid factor differentiating this disease from RA. There is the potential for aggressive osteolysis, fibrous ankylosis, and heterotopic new bone formation to occur in PsA. As mentioned earlier, the coexistence of HIV and PsA seems to set the stage for an aggressive course of joint destruction in some patients.

DIAGNOSIS

PsA has a variable manifestation and disease course, but several clinical patterns have been identified in prospectively monitored cohorts of patients. The clinical subsets are not mutually exclusive, nor are they static over time. The most common form, which affects 30 to 50% of patients, is an asymmetrical oligoarthritis that may involve both large and small joints. Dactylitis, arising as sausage digits, can be seen in fingers and toes and actually represents an enthesitis. In the second subset there is selective targeting of the distal interphalangeal joints, seen in 10 to 15% of patients. These changes are strongly associated with nail dystrophy, of which the features are onycholysis, subungual keratosis, pitting, and oil drop–like staining (Fig. 265-7). The third subset (15 to 30% of patients) has a symmetrical polyarthritis that mimics RA in many ways, except for the absence of rheumatoid nodules and rheumatoid factor. The fourth clinical variant is psoriatic spondylitis, which occurs in 20% of patients; 50% of such patients are B27 positive. Finally, arthritis mutilans (5% of patients) is a destructive, erosive arthritis that affects large and small joints and can be associated with marked deformities and significant disability. Radiographic changes in PsA involve soft tissue swelling (particularly in the case of dactylitis), erosions, and periostitis. Axial involvement may lead to the appearance of asymmetrical sacroiliitis with syndesmophytes that are bulky, asymmetrical, and nonmarginal. The classic “pencil-in-cup” deformity may be seen in patients with distal interphalangeal joint disease or arthritis mutilans. Acro-osteolysis is noted in a minority of patients and reflects an aggressive erosive process.

Differential Diagnosis

The diagnosis of PsA depends on finding the typical skin or nail changes in association with one of the articular variants described previously. The differential diagnosis for the skin lesions can include seborrheic dermatitis, dyshidrotic eczema, fungal infection, keratoderma blennorrhagicum, and palmoplantar pustulosis.

TREATMENT  Patients typically receive aggressive treatment for psoriasis (Chapter 438). The advent of biologic agents has had a major impact on the treatment of PsA. The anti-TNF agents have been studied most extensively, indicating the efficacy of infliximab, etanercept, adalimumab, and golimumab. In a phase III randomized placebo-controlled trial, treatment of psoriatic arthritis with subcutaneous golimumab (50 mg to 100 mg every 4 weeks) inhibited the progression of structural damage and demonstrated continued clinical efficacy and safety through 1 year. A8  In a study of patients with PsA, ustekinumab, a monoclonal antibody against interleukin-12/23, was well tolerated, reduced the extent and severity of psoriasis, and was safe. A9  Brodalumab, a human monoclonal antibody against interleukin-17 receptor A (IL17RA), significantly improved response rates among patients with PsA in a phase 2 randomized, double-blind, placebo-controlled study. A10  The European League Against Rheumatism in 2012 published recommendations for the management of PsA with systemic and local (nontopical) symptomatic and disease-modifying antirheumatic drugs. They suggest nonsteroidal anti-inflammatory drugs to relieve musculoskeletal signs and symptoms; treatment with diseasemodifying drugs such as methotrexate, sulfasalazine, or leflunomide in patients with swollen joints, structural damage in the presence of inflammation, or clinically relevant extraarticular manifestations; and anti-TNF agents in patients with active enthesitis and/or dactylitis and insufficient response to other medications. Patients should be switched to another anti-TNF agent if the first is not successful.

Enteropathic Arthritis

DEFINITION

EA refers to the arthritis associated with Crohn’s disease (CD) or ulcerative colitis (UC) (Chapter 141; Table 265-4).

PATHOBIOLOGY

All extraenteric manifestations, including arthritis, occur more commonly in CD than in UC. Peripheral arthritis occurs in 10 to 20% of CD patients and in 2 to 7% of UC patients. This pattern of arthritis occurs more commonly in patients with other extraenteric features (e.g., erythema nodosum, iritis). It is typically an inflammatory nonerosive polyarthritis, predominantly of large joints. In general, the clinical activity of the peripheral arthritis parallels the

TABLE 265-4  ENTEROPATHIC ARTHRITIS FEATURE

PERIPHERAL ARTHRITIS

SACROILIITIS, SPONDYLITIS

10-20%

2-7%

CROHN’S DISEASE (CD) Frequency in CD HLA-B27 associated

No

Yes

Pattern

Transient, symmetrical

Chronic

Course

Related to activity of CD

Unrelated to activity of CD

Effect of surgery

Remission of arthritis uncommon

No effect

Effect of anti-TNF therapy

Effective

Effective

ULCERATIVE COLITIS (UC)

FIGURE 265-7. Nail pitting in psoriasis. The pits are more discrete and regular compared with pits affecting the nail plate in dermatitis.

Frequency in UC

5-10%

2-7%

HLA-B27 associated

No

Yes

Pattern

Transient

Chronic

Course

More common in pancolitis than proctitis; related to activity of UC

Unrelated

Effect of surgery

Remission of arthritis

No effect

HLA = human leukocyte antigen; TNF = tumor necrosis factor.

1769

CHAPTER 266  Systemic Lupus Erythematosus  

activity of the gut inflammation, and measures that control the GI disease usually control the joint disease as well. The peripheral arthritis of EA is not associated with B27. As mentioned earlier, IBD and AS share genetic susceptibility associated with numerous genes. In contrast, the sacroiliitis or spondylitis of EA follows a pattern in which the joint inflammation waxes and wanes independently of the bowel inflammation. Axial disease occurs in 2 to 7% of both CD and UC patients. HLA-B27 is found in 50% of patients with axial arthritis. The course tends to be chronic, as opposed to the transient course of peripheral arthritis. The association of bowel inflammation and arthritis is supported by ileocolonoscopic studies in which subclinical inflammation of the bowel has been demonstrated in diseases covering the entire spectrum of SpAs. Histologic evaluation demonstrates that changes of acute ileitis are seen in postdysenteric ReA, whereas chronic inflammatory changes are more likely to be seen in patients with AS. As mentioned earlier, the abnormalities in the bowel of B27 transgenic rats have strong similarity to the lesions of CD, and a germfree environment minimizes inflammatory changes in both the gut and the joints. This finding argues that altered bowel permeability, with enhanced bacteremia or antigenemia, may provide the link in both cases.

DIAGNOSIS

It is important to recognize that the musculoskeletal features of EA may precede any GI symptoms or signs. Conversely, the diarrhea preceding the onset of peripheral or axial arthritis in a young patient could just as likely represent a food-borne pathogen (e.g., Salmonella, Yersinia), with secondary ReA as IBD and accompanying EA. In the initial assessment of such a patient, it is important to carry out careful and complete stool cultures. If the GI symptoms persist, diagnostic colonoscopy is often required to resolve the issue.

Undifferentiated Spondyloarthritis Despite careful clinical and radiographic assessment, there are still a substantial number of patients who do not fall into one of the classic diagnostic subsets of SpA outlined previously. These patients are often defined as having USpA with peripheral enthesitis, asymmetrical arthritis or sacroiliitis, or iritis in the absence of identifiable antecedent infection or concurrent IBD or psoriasis. The natural history of USpA has not been well defined, and case heterogeneity and diagnostic dilemmas plague a systematic or multicenter approach to the problem. When the clinical course is examined, a number of patients may finally meet the diagnostic criteria for AS, but many retain a distinct USpA pattern for prolonged periods.18

Grade A References A1. Braun J, van der Horst-Bruinsma F, Huang F, et al. Clinical efficacy and safety of etanercept versus sulfasalazine in ankylosing spondylitis patients: a randomized, double-blind study (ASCEND Trial). Arthritis Rheum. 2011;63:1543-1551. A2. Barber CE, Kim J, Inman RD, et al. Antibiotics for treatment of reactive arthritis: a systematic review and metaanalysis. J Rheumatol. 2013;40:916-928. A3. Carter JD, Espinoza LR, Inman RD, et al. Combination antibiotics as a treatment for chronic Chlamydia-induced reactive arthritis: a double-blind, placebo-controlled, prospective trial. Arthritis Rheum. 2010;62:1298-1307. A4. Inman RD, Davis JC, van der Heijde D, et al. Efficacy and safety of golimumab in patients with ankylosing spondylitis: results of the randomized, double-blind, placebo-controlled GO-RAISE trial. Arthritis Rheum. 2008;58:3402-3412. A5. Sieper J, Lenaerts J, Wollenhaupt J, et al. Efficacy and safety of infliximab plus naproxen versus naproxen alone in patients with early, active axial spondyloarthritis: results from the double-blind, placebo-controlled INFAST study, Part 1. Ann Rheum Dis. 2014;73:101-107. A6. Dougados M, van der Heijde D, Sieper J, et al. Symptomatic efficacy of etanercept and its effects on objective signs of inflammation in early nonradiographic axial spondyloarthritis: a multicenter, randomized, double-blind, placebo-controlled trial. Arthritis Rheum. 2014;66:2091-2102. A7. Song IH, Hermann KG, Haibel H, et al. Consistently good clinical response in patients with early axial spondyloarthritis after 3 years of continuous treatment with etanercept: longterm data of the ESTHER trial. J Rheumatol. 2014;41:2034-2040. A8. Kavanaugh A, van der Heijde D, McInnes IB, et al. Golimumab in psoriatic arthritis. One-year clinical efficacy, radiographic, and safety results from a phase III, randomized, placebo-controlled trial. Arthritis Rheum. 2012;64:2504-2517. A9. Gottlieb A, Menter A, Mendelsohn A, et al. Ustekinumab, a human interleukin 12/23 monoclonal antibody, for psoriatic arthritis: randomised, double-blind, placebo-controlled, crossover trial. Lancet. 2009;373:633-640. A10. Mease PJ, Genovese MC, Greenwald MW, et al. Brodalumab, an anti-IL17RA monoclonal antibody, in psoriatic arthritis. N Engl J Med. 2014;370:2295-2306.

GENERAL REFERENCES For the General References and other additional features, please visit Expert Consult at https://expertconsult.inkling.com.

266  SYSTEMIC LUPUS ERYTHEMATOSUS MARY K. CROW

DEFINITION

Systemic lupus erythematosus (SLE) is a multisystemic autoimmune disease that results from immune system–mediated tissue damage. Manifestations of SLE can involve the skin, joints, kidney, central nervous system (CNS), cardiovascular system, serosal membranes, and the hematologic and immune systems. The disease is highly heterogeneous, with individual patients manifesting variable combinations of clinical features. In most patients with SLE, the disease is characterized by a waxing and waning clinical course, although some demonstrate a pattern of chronic activity. The molecular triggers of the disease are not known, but the pathogenesis is understood to involve the production of autoantibodies exhibiting multiple specificities, with reactivity with nucleic acid–binding proteins being a common feature. Immune complexes, along with immune system cells and soluble mediators, generate inflammation and tissue damage. Therapeutic approaches generally involve immunosuppression.

EPIDEMIOLOGY

A notable feature of SLE is that it occurs much more frequently in females than in males. Like Hashimoto’s thyroiditis and Sjögren’s syndrome, the female-to-male ratio is approximately 8 : 1 to 10 : 1 in adults, and most cases are diagnosed between the ages of 15 and 44 years. In children and women older than 55, the ratio is closer to 2 : 1. The prevalence of SLE in the United States is estimated to be approximately 73 per 100,000, and the incidence of new cases is 5.5 per 100,000 per year. The prevalence, severity, and characteristics of disease differ in different ethnic groups, with SLE being 2.3-fold more frequent in African Americans than in the white population. The severity of disease is also greater in Hispanic individuals than in whites, although data for Hispanic populations are less abundant. Asians may also have a higher prevalence of disease than whites. Recent studies of lupus in minority populations indicate that socioeconomic factors are major contributors to the increased prevalence and severity of disease in African Americans and Hispanic Americans.

PATHOBIOLOGY

Current understanding of lupus pathogenesis incorporates roles for genetic susceptibility based on a threshold model involving multiple genes1; environmental triggers, including microbial infection, sunlight, and certain drugs; and altered immune system function. Recent advances in immunology have focused attention on the mechanisms that account for innate immune system activation.2 At least some of the genetic and environmental contributions to lupus are likely to promote innate immune system activation and subsequent autoimmunity. Others may contribute to inflammation and tissue damage. Induction of cellular stress responses, including oxidative modification of cell proteins, is of current interest as a mechanism that links environmental triggers to altered immune function. Murine models have proved useful in identifying genes that could contribute to lupus susceptibility or define patterns of disease. Production of autoantibodies characteristic of SLE and development of nephritis and accelerated death have been demonstrated in numerous murine strains in which immune system genes have been modified. In most cases, no alterations have been noted in the homologous human genes. The ease of induction of lupus-like disease in murine models suggests that there are numerous possible pathogenic paths that might lead to the clinical manifestations of lupus. It is not known which of these molecular pathways is responsible for human SLE, although mediators of the immune response to viral infection, particularly components of the type I interferon response, are associated with lupus in both murine and human systems and are likely to be important in disease pathogenesis.3

Genetics

An important role for a genetic contribution to lupus susceptibility is suggested by the high concordance of disease in monozygotic twins (14 to 57%).

CHAPTER 265  The Spondyloarthropathies  

GENERAL REFERENCES 1. Braun J, van den Berg R, Baraliakos X, et al. 2010 update of the ASAS/EULAR recommendations for the management of ankylosing spondylitis. Ann Rheum Dis. 2011;70:896-904. 2. Cantini F, Niccoli L, Nannini C, et al. Psoriatic arthritis: a systematic review. Int J Rheum Dis. 2010;13:300-317. 3. Gossec L, Smolen JS, Gaujoux-Viala C, et al. European League Against Rheumatism recommendations for the management of psoriatic arthritis with pharmacological therapies. Ann Rheum Dis. 2012;71:4-12. 4. Passalent LA. Physiotherapy for ankylosing spondylitis: evidence and application. Curr Opin Rheumatol. 2011;23:142-147. 5. Heldmann F, Dybowski F, Saracbasi-Zender E, et al. Update on biologic therapy in the management of axial spondyloarthritis. Curr Rheumatol Rep. 2010;12:325-331. 6. Haroon N, Inman RD, Weisman MH, et al. The impact of TNF-inhibitors on radiographic progression in ankylosing spondylitis. Arthritis Rheum. 2013;65:2645-2654. 7. Eder L, Thavaneswaran A, Chandran V, Gladman DD. Tumour necrosis factor α blockers are more effective than methotrexate in the inhibition of radiographic joint damage progression among patients with psoriatic arthritis. Ann Rheum Dis. 2014;73:1007-1011. 8. International Genetics of Ankylosing Spondylitis Consortium (IGAS). Identification of multiple risk variants for ankylosing spondylitis through high-density genotyping of immune-related loci. Nat Genet. 2013;45:730-738. 9. Haroon N, Tsui F, Uchanska-Ziegler B, et al. Endoplasmic reticulum aminopeptidase 1 (ERAP1) exhibits functionally significant interaction with HLA B27 and relates to subtype specificity in ankylosing spondylitis. Ann Rheum Dis. 2012;7:589-595.

1769.e1

10. Dean LE, Jones GT, MacDonald AG, et al. Global prevalence of ankylosing spondylitis. Rheumatology (Oxford). 2014;53:650-657. 11. Jang JH, Ward MM, Rucker AN, et al. Ankylosing spondylitis: patterns of radiographic involvement—a re-examination of accepted principles in a cohort of 769 patients. Radiology. 2011;258:192-198. 12. Paparo F, Revelli M, Semprini A, et al. Seronegative spondyloarthropathies: what radiologists should know. Radiol Med. 2014;119:156-163. 13. Selmi C, Gershwin ME. Diagnosis and classification of reactive arthritis. Autoimmun Rev. 2014;13:546-549. 14. Tikly M, Njobvu P, McGill P. Spondyloarthritis in sub-Saharan Africa. Curr Rheumatol Rep. 2014;16:421. 15. Ficco HM, Citera G, Cocco JA. Prevalence of psoriatic arthritis in psoriasis patients according to newer classification criteria. Clin Rheumatol. 2014;33:243-246. 16. Li W, Han J, Quereshi AA. Obesity and risk of incident psoriatic arthritis in US women. Ann Rheum Dis. 2012;71:1267-1272. 17. Love TJ, Zhu Y, Zhang Y, et al. Obesity and the risk of psoriatic arthritis: a population-based study. Ann Rheum Dis. 2012;71:1273-1277. 18. Sieper J, van der Heijde D. Review: Nonradiographic axial spondyloarthritis: new definition of an old disease? Arthritis Rheum. 2013;65:543-551.

1769.e2

CHAPTER 265  The Spondyloarthropathies  

REVIEW QUESTIONS 1. A 28-year-old man presents with a 2-year history of low back pain and intermittent arthritis in the lower extremity. Physical examination demonstrates bilateral Achilles tendonitis and tenderness to direct pressure over the sacroiliac joints. Which of the following would not support the diagnosis of spondyloarthritis (SpA)? A. Presence of HLA-B27 B. History of anterior uveitis C. Positive rheumatoid factor D. Elevated CRP E. History of inflammatory bowel disease Answer: C  The history and the other optional features would be entirely consistent with a diagnosis of SpA. HLA-B27 is strongly associated with SpA in general, and its strongest association is with ankylosing spondylitis (AS). Anterior uveitis occurs in approximately 30% of AS cases, and is itself associated with HLA-B27 even when AS is not present. Positive rheumatoid factor is characteristic of rheumatoid arthritis but not of SpA, so it would not be expected to be positive in such patients. Elevated C-reactive protein (CRP) is a nonspecific marker of inflammation and would be commonly seen in patients with active SpA. Inflammatory bowel disease occurs in approximately 10% of AS cases, and its presence strongly suggests that concurrent musculoskeletal inflammation that the underlying arthritis is part of the SpA spectrum. 2. A 32-year-old man presents with pain and swelling of one ankle and one knee. The history reveals that he had just returned from a trip to South America and developed diarrhea that was now subsiding. He denied any eye inflammation or skin lesions. Physical examination confirmed tenderness and swelling of an ankle and a knee. The next appropriate diagnostic test would be which of the following? A. Aspiration of the knee swelling and culture of the synovial fluid B. Initiate a course of broad-spectrum antibiotics. C. X-rays of the lumbar and cervical spine D. Check a blood test for the presence of antinuclear antibodies. E. Initiate an empirical trial of methotrexate. Answer: A  A patient with new-onset swelling in the joints, particularly when one or two joints are affected, should always raise the possibility of septic arthritis. When there has been a recent infection elsewhere in the body, it is possible the inciting infection could spread by a hematogenous route to seed the joint. Although the history is suggestive of reactive arthritis (ReA), it is important to rule out septic arthritis with certainty before entertaining that diagnosis. Culture of synovial fluid is the definitive test to rule out septic arthritis. Empirical use of broad-spectrum antibiotics rarely has a place in the management of new-onset arthritis. The first step is to make a diagnosis. X-rays of the spine are important in the evaluation of inflammatory back pain, but in this case there is only peripheral joint involvement. If the course of the arthritis becomes chronic or back pain develops over time, x-rays at that point would be appropriate. Antinuclear antibodies in the serum are the hallmark of many autoimmune rheumatic diseases—in particular, lupus. But with the clinical presentation in this case, there are no features to implicate lupus or related conditions, and it would not be appropriate to search for antinuclear antibodies. The immediate priority is to rule out septic arthritis. It would be inappropriate to institute methotrexate before that is resolved.

3. A 30-year-old man presents with worsening back pain. The history reveals that this has been present for the past 3 years and is increasing in severity. It is characterized by early-morning stiffness, nocturnal pain, and modest improvement with exercise. He initially noted an improvement with an NSAID, but subsequently that effect was lost, and he has tried two other NSAIDs without effect. Physical examination reveals limitation in forward flexion of the lumbar spine. An AP x-ray of the pelvis reveals bilateral erosive changes in the sacroiliac joints. A lateral x-ray of the lumbar spine reveals squaring of several vertebral bodies. The next step in pharmacologic treatment of this condition is which of the following? A. Methotrexate B. Leflunomide C. Sulfasalazine D. TNF inhibitor biologic agents E. Azathioprine Answer: D  The patient meets the diagnostic criteria for ankylosing spondylitis (AS). He has long-standing back pain that has features of inflammatory back pain and has limitation in spinal mobility. The presence of bilateral erosive sacroiliitis provides imaging support for the diagnosis of AS. He has had an adequate trial of NSAIDs, and his back pain has not responded adequately to these agents. The classical DMARDs used in rheumatoid arthritis— methotrexate, leflunomide, sulfasalazine, and azathioprine—have proved ineffective in controlling the spinal inflammation seen in AS. On the other hand, the TNF inhibitors (infliximab, etanercept, adalimumab, golimumab) have established efficacy in controlling the signs and symptoms of AS. 4. A 35-year-old woman is under your care for ankylosing spondylitis (AS). She is HLA-B27 positive and has no extra-articular features accompanying her back pain. Her symptoms have been controlled with NSAIDs in combination with physiotherapy and exercises. She has an 8-year-old son, and she is inquiring about his current and future health prospects. Which of the following would be advisable in this circumstance? A. The son should be checked for HLA-B27 status. B. The mother should be advised that the likelihood of her son developing AS is less than 10%. C. The son should be advised to avoid sports and physical activities. D. Preventive low-dose NSAIDs should be instituted for the son. E. The son should be advised to avoid foreign travel in order to avoid food-borne pathogens. Answer: B  There is a 50% chance that any particular child of a B27-positive patient will be B27 positive, but there is only a 20% chance of a B27-positive individual with a positive family history developing AS. So the likelihood of a child developing AS when he has a B27-positive parent with AS is maximally 10%. It would not be advisable to test an asymptomatic child for HLAB27, because it has low predictive value (as discussed earlier) and could introduce chronic anxiety about future disease, which is unwarranted. There is no reason to limit the sports and recreation of the child. Maintaining general fitness and muscle strength and posture is sound advice even if there is a greater chance of later arthritis than in the general population. There is no evidence that sports-related injuries trigger AS. There is no evidence that NSAIDs prevent the subsequent development of SpA in individuals who are genetically predisposed to SpA. Chronic NSAID therapy carries its own risks and would not be justified in an asymptomatic individual. There is a theoretical concern that being B27 positive could increase the risk of reactive arthritis after a food-borne pathogen illness. This might lead to advising a patient with prior diagnosis of ReA to minimize exposure to food-borne pathogens, but it would be inappropriate to restrict travel for anyone who has never had an episode of ReA.

CHAPTER 265  The Spondyloarthropathies  

5. A 19-year-old female patient has developed a painful swollen knee over the course of 7 days. There is no history of trauma and no associated skin lesions or ocular complaints. The patient relates a history that she has just returned from a trip abroad with the family and had a transient diarrheal episode while traveling. Further questioning reveals that she has had two prior episodes of diarrhea in the past year, each associated with crampy abdominal pain, but on each occasion the diarrhea was self-limited and she did not seek medical attention. Which of the following would not be an appropriate next step in the management of this patient? A. Referral to a gastroenterologist for ileocolonoscopy B. Culture of a stool sample C. Aspiration of the knee and culture of synovial fluid D. Check hemoglobin and ESR. E. Institute a 2-week course of ciprofloxacin.

1769.e3

Answer: E  The patient has developed arthritis in the setting of a recurrent diarrheal course. The key differential is between reactive arthritis (ReA) and enteropathic arthritis (EA) as part of inflammatory bowel disease (IBD). In either case, it would be inappropriate to institute an empirical course of antibiotics before a diagnosis is made. In neither postdysenteric ReA nor in IBD-associated arthritis is there evidence that antibiotics alter the course of the arthritis. It would be appropriate in this case to determine whether the recurrent diarrhea could be a manifestation of Crohn’s disease or ulcerative colitis, and ileocolonoscopy would be indicated to address that question. Recurrent diarrhea might reflect a infectious agent such as Salmonella. Stool cultures would be an appropriate test to address that. Culture of synovial fluid is the definitive test to rule out septic arthritis. Both Salmonella and Yersinia can cause a septic arthritis, so it is appropriate to exclude an infection in the joint before proceeding to other management decisions. Anemia of chronic disease might raise the suspicion that a chronic process, such as IBD, has been present for some time. Elevation in CRP or ESR is appropriate in the work-up of this patient, although this would not differentiate ReA from IBD-related arthritis. These acute phase reactants can serve as useful surrogate markers to monitor response to treatment.

1770

CHAPTER 266  Systemic Lupus Erythematosus  

Genes that might account for increased lupus susceptibility or severity include those encoding components of the complement pathway, including C1q, C2, and C4A (E-Table 266-1). Impaired production of these early complement components may decrease the clearance of apoptotic cells, thereby augmenting the pool of available autoantigens, or decrease the solubility of immune complexes. Polymorphic variants in components of the toll-like receptor (TLR) pathways that regulate type I interferon production, including interferon regulatory factor 5 (IRF5), have been associated with a diagnosis of SLE and increased plasma interferon activity in some populations. Association of SLE with the major histocompatibility complex (MHC) class II alleles human leukocyte antigen (HLA)-DR2 and HLA-DR3 has been documented in many studies and is most striking in patients expressing particular autoantibody specificities. Polymorphisms in the Fc receptor genes FCGR2A and FCGR3A have been associated with SLE nephritis, possibly based on altered clearance of immune complexes. Variants of the PTPN22 gene, which encodes a phosphatase that regulates T-cell activation, are also associated with SLE. Genome-wide association studies have expanded the list of lupus-associated gene variants to include regulators of innate immune system activation (TNFAIP3, ITGAM, IRAK1) and signaling molecules important in lymphocyte activation (STAT4, BANK1, BLK, and LYN). Rare mutations in genes encoding proteins that regulate nucleic acid integrity and degradation, including TREX1, encoding a DNase; SAMHD1, a triphosphohydrolase; RNASEH2A, B and C; and ADAR, an RNA-specific adenosine deaminase, have been documented in some patients with a lupus-like syndrome called Aicardi-Goutieres syndrome, characterized by skin lesions, CNS disease, autoantibodies, and high levels of interferon. Mutations in these genes have also been documented in rare patients with SLE and have provided new insights into the likely contribution of endogenous nucleic acids to innate immune system activation and lupus pathogenesis.4 The available data suggest a common theme: the genes that have been associated with lupus confer either increased activation or impaired regulation of the innate or adaptive immune responses, with increased type I interferon often observed in association with the risk genotype.

Environmental Triggers

Several classes of potential environmental triggers for lupus have been studied.5 Although the female preponderance of SLE implies a role for hormonal factors in the disease, recent concepts describe a possible contribution of epigenetic modification or dosage effects of the X chromosome as accounting for at least some of the sex skewing. A role for microbial triggers— particularly viral infection—has been postulated, consistent with the constitutional symptoms that often characterize the earliest stage of the disease. Epstein-Barr virus has garnered particular interest among investigators, because the frequency of previous infection in SLE patients is significantly higher than in the general population (99 vs. 94%). Evidence of exposure to other viruses, including cytomegalovirus, is equivalent between SLE patients and healthy control subjects. Ultraviolet light exposure is a welldescribed trigger of lupus flares. Possible mechanisms include DNA damage, induction of cellular stress responses, and induction of apoptosis of skin cells, which result in concentration of nucleic acids and associated proteins in cell membrane blebs and increased processing by antigen-presenting cells. Data also support an association between current tobacco use and anti–doublestranded DNA antibodies and lupus disease activity. Certain drugs, including procainamide and hydralazine, can induce a lupus-like syndrome, but the symptoms usually abate after discontinuing use of the drug. These agents may promote demethylation of DNA, thereby altering gene expression and potentially increasing the availability of immunostimulatory DNA. Sulfa antibiotics have been reported to induce lupus flare in some patients. Administration of recombinant interferon-α to patients with hematologic malignancies or hepatitis C infection has been associated with induction of a lupus-like syndrome. In addition, anti–tumor necrosis factor agents have induced lupus autoantibodies and occasionally clinical lupus in patients with rheumatoid arthritis.

Immunologic Triggers

Genetic and environmental factors that increase the probability of development of SLE are likely to act on the immune system to induce autoimmunity and consequent tissue inflammation and damage. In addition to mechanisms that increase the availability of self-antigens (such as ultraviolet light), altered expression of gene products that mediate or regulate apoptosis, or impaired clearance of apoptotic debris, results in generalized activation of the immune system and contributes to autoimmunity in lupus. In parallel with the events

that account for effective immune responses directed at exogenous microbes, the autoimmunity that occurs in SLE patients is likely to require activation of both innate and adaptive immune responses. The innate immune response is first activated by common molecular patterns expressed on the microbe and results in augmented antigen-presenting cell capacity and successful generation of an antigen-specific adaptive immune response. The characterization of the TLR family of pattern recognition receptors has provided new understanding of the mechanisms through which the innate immune system is activated by exogenous and endogenous stimuli, including nucleic acid– containing immune complexes, and promotes induction of a self-directed adaptive immune response.6

Type I Interferon

Recent studies of gene expression in peripheral blood mononuclear cells of SLE patients using microarray technology have demonstrated that activation of genes regulated by type I interferon is a common feature in patients with active disease and may represent innate immune system activation. Interferon (IFN)-α may be responsible for many of the immunologic alterations that have been observed in SLE and is identified as a potential therapeutic target. Immune complexes containing DNA or RNA are postulated to contribute to the production of type I interferon in SLE. Demethylated CpG-rich DNA or RNA associated with nucleic acid–binding proteins can activate plasmacytoid dendritic cells and other immune system cells through TLRs and thereby result in the production of type I interferon (IFN-α or IFN-β) and other proinflammatory cytokines (E-Fig. 266-1). Diverse effects of type I interferon on immune system function are consistent with the altered immune responses observed in SLE patients, including maturation of dendritic cells, increased immunoglobulin class switching to mature immunoglobulin isotypes (immunoglobulin [Ig]G and IgA), and induction of soluble mediators that increase B-cell differentiation and inflammatory responses, such as B-lymphocyte stimulator (BLyS) and IFN-γ.7 Induction of an immunostimulatory microenvironment by IFN-α may support the development of a humoral immune response directed at self-antigens, particularly intracellular particles that contain nucleic acids and nucleic acid–binding proteins. It is not known why some individuals initiate immune system activation directed at self-antigens and others do not. In addition to its effects on immune system function, type I interferon has been associated with altered endothelial cell function and may contribute to the development of atherosclerotic vascular pathology in patients with lupus.8

Autoantibodies

The most characteristic lupus autoantibodies target intracellular particles containing both nucleic acid and nucleic acid–binding proteins. Understanding the significance of induction of these particular autoantibody specificities may provide clues to the etiology of SLE. A recent analysis of the spectrum of autoantibodies present in the sera of individuals in whom SLE is later diagnosed has suggested that autoantibodies reactive with certain RNAbinding proteins, including the Ro protein, occur early in the preclinical stage of the disease, along with a positive antinuclear antibody (ANA) test. These are often followed by anti-DNA antibodies and, finally, by the development of antibodies specific for the spliceosomal proteins Smith (Sm) and ribonucleoprotein (RNP) at approximately the time of diagnosis (Fig. 266-1). These observations suggest that individuals who demonstrate progression from humoral immunity targeting proteins associated with RNA to antibodies that bind DNA and other specificities are those in whom sufficient autoimmunity develops to manifest clinical symptoms. Approximately one third of SLE patients have autoantibodies reactive with phospholipids or the proteins associated with them, particularly β2-glycoprotein I (β2GPI). These autoantibody specificities can also be present independently of SLE in primary antiphospholipid antibody syndrome (Chapter 174).

Immune Complexes and Complement

Tissue and organ damage in SLE is mediated by the deposition or in situ formation of immune complexes and subsequent complement activation and inflammation. The complement system (Chapter 50), composed of more than 30 proteins that act in concert to protect the host against invading organisms, initiates inflammation and tissue injury. Complement activation promotes chemotaxis of inflammatory cells and generates proteolytic fragments that enhance phagocytosis by neutrophils and monocytes. The classic pathway is activated when antibodies bind to antigen and generate potent effectors. Alternative pathway activation mechanisms differ in that they are initiated by the binding of spontaneously activated complement components

CHAPTER 266  Systemic Lupus Erythematosus  

E-TABLE 266-1  GENES ASSOCIATED WITH SYSTEMIC LUPUS ERYTHEMATOSUS GENE*

PROTEIN

BANK1

B-cell scaffold protein with ankyrin repeats 1

BLK

B-lymphocyte–specific tyrosine kinase

C1QA, B, and C

Complement component C1q

C2

Complement component 2

C4A and C4B

Complement component C4

CRP

C-reactive protein

DRB11501

MHC class II (DR2)

DRB10301

MHC class II (DR3)

FCGR2A

Activating FcγRIIA

FCGR3A

Activating FcγRIIIA

IRF5

Interferon regulatory factor 5

ITGAM

Mac1/complement receptor 3

IRAK1

Interleukin-1 receptor–associated kinase 1

LYN

Lyn tyrosine kinase

PTPN22

Protein tyrosine phosphatase nonreceptor type 22

STAT4

Signal transducer and activator of transcription 4

TNFAIP3

A20

TNFSF4

Ox40 ligand

TREX1

DNase III

*Genes listed have an odds ratio of ≥ 1.3. MHC = major histocompatibility complex.

1770.e1

1770.e2

CHAPTER 266  Systemic Lupus Erythematosus  

Potential endogenous ligands: Exogenous ligands:

dsRNA-containing immune complexes

Fibronectin products

ssRNA-containing immune complexes miRNA

CpG DNA-containing immune complexes

dsRNA

LPS

ssRNA

Demethylated CpG DNA

TLR7/8

TLR9

MyD88

MyD88

TLR4

TLR3

Trif

TRAM

TIRAP

Trif

MyD88

Inflammatory cytokines Type I interferon

Inflammatory cytokines

Inflammatory cytokines Type I interferon

Dendritic cell maturation Ig class switching Induction of BLyS, IL-10, interferon-γ, and chemokines E-FIGURE 266-1.  Model for induction of innate immune system activation in systemic lupus erythematosus. Both exogenous and endogenous stimuli can induce toll-like receptor (TLR) activation and thereby result in new gene transcription. Among potential endogenous ligands are immune complexes containing DNA or RNA or matrix-derived components. TLR ligands trigger the activation of intracellular adaptors, including TIR domain-containing adapter-inducing interferon-β (Trif), Trif-related adaptor molecule (TRAM), TIR domaincontaining adapter protein (TIRAP), or myeloid differentiation primary response protein 88 (MyD88), and induce transcription of type I interferons or inflammatory cytokines. Type I interferons mediate diverse effects on immune system cells, including maturation of dendritic cells, increased immunoglobulin (Ig) class switching, and induction of cytokines that promote autoimmunity and inflammation, including B-lymphocyte stimulator (BLyS), interleukin-10 (IL-10), interferon-γ, and chemokines. LPS = lipopolysaccharide.

1771

CHAPTER 266  Systemic Lupus Erythematosus  

Diagnosis

Patients with positive test

100 80 60 ANA Anti-Ro Anti-La APL Anti-ds DNA Anti-Sm Anti-nRNP

40 20 0 −5

−4

−3

−2

−1 0 1 Time (yr)

2

3

4

5

FIGURE 266-1.  Proportion of patients with positive antibody tests relative to the time of diagnosis or appearance of the first clinical manifestation of systemic lupus erythematosus (SLE). For each autoantibody, the proportion of patients testing positive relative to the time of diagnosis or to the time of appearance of the first clinical criterion was assessed. In analyses of the time from antibody development to the diagnosis of SLE, antinuclear antibodies (ANAs) appeared significantly earlier than anti-Sm antibodies and antinuclear ribonucleoprotein (anti-nRNP) antibodies, but not significantly earlier than anti-Ro, anti-La, antiphospholipid (APL), or anti–double-stranded DNA antibodies (anti-dsDNA). (From Arbuckle MR, McClain MT, Rubertone MV, et al. Development of autoantibodies before the clinical onset of systemic lupus erythematosus. N Engl J Med. 2003;349:16.)

FIGURE 266-2.  Malar rash in a patient with systemic lupus erythematosus. Note that the rash does not cross the nasolabial fold. (From Gladman DD, Urowitz MB. Systemic lupus erythematosus: clinical features. In: Klippel JH, Dieppe PA, eds. Rheumatology. 2nd ed. London: Mosby; 1998.)

TABLE 266-1  CLINICAL MANIFESTATIONS OF SYSTEMIC LUPUS ERYTHEMATOSUS to the surfaces of pathogens or self-tissues. C3a, an anaphylatoxin that binds to receptors on leukocytes and other cells, causes activation and release of inflammatory mediators. C5a is a potent soluble inflammatory, anaphylatoxic, and chemotactic molecule that promotes recruitment and activation of neutrophils and monocytes and mediates endothelial cell activation through its receptor. The release of reactive oxygen and nitrogen intermediates is an additional mechanism that contributes to tissue damage. Tissues targeted by immune system activity in lupus include the skin, where immune complexes and complement are deposited in a linear pattern (as demonstrated in the lupus band test, in which deposited antibodies are identified by a fluorescent tag), the glomeruli, and heart valves. Antibodies reactive with hippocampal neurons in the brain can mediate excitotoxic death. Immune and inflammatory mechanisms responsible for the vasculopathy of lupus are multifactorial and not clearly defined. Microvascular damage is observed in splenic arteries and is characterized by the typical onion-skin pattern of concentric connective tissue deposition. In addition to vascular damage mediated by inflammation, thrombosis, including microthrombi, contributes to ischemia and cell necrosis in the brain and other organs.

CLINICAL MANIFESTATIONS

Symptoms and Signs Constitutional Symptoms

SLE is a disease that involves virtually all components of the immune system and can be accompanied by constitutional symptoms similar to those seen in the setting of microbial infection. Fatigue, headaches, weight loss, and fevers are common, along with generalized arthralgias, myalgias, and lymphadenopathy. The level of activity of lupus typically follows a pattern of flares and remissions, although some patients sustain active disease for prolonged periods. Careful monitoring for the development of major organ system disease is important to ensure timely adjustments in medical therapy.

Cutaneous and Mucous Membranes

The skin and mucous membranes are affected in most lupus patients (Table 266-1). The erythematous facial rash with a butterfly distribution across the malar and nasal prominences and sparing of the nasolabial folds is the classic rash of SLE and is seen in 30 to 60% of patients (Fig. 266-2). The butterfly rash is often triggered by sun exposure, but photosensitivity can also be demonstrated diffusely in other areas of the body. The discoid skin lesions are erythematous plaques with central scarring and may be covered with scale. These lesions are seen in about 25% of patients, involve the scalp or the face and ears, and may be associated with alopecia. Discoid lesions can be present in the absence of systemic

MANIFESTATION

APPROXIMATE FREQUENCY (%)

Cutaneous

88

Arthritis/arthralgias

76

Neuropsychiatric

66

Pleurisy/pericarditis

63

Anemia

57

Raynaud’s phenomenon

44

Vasculitis

43

Atherosclerosis

37

Nephritis

31

Thrombocytopenia

30

Sensorimotor neuropathy

28

Cardiac valvar disease

18

Pulmonary alveolar hemorrhage

12

Pancreatitis

10

Myositis

5

Myocarditis

5

manifestations of SLE (discoid lupus). In addition to the scarring alopecia of discoid lupus, more transient alopecia may be a clinical sign of increased disease activity and is associated with apoptosis of cells in the hair follicle. Inflammation of the deep dermis and subcutaneous fat can result in lupus panniculitis, with firm painful nodules that sometimes adhere to the epidermis, causing irregularities in the superficial skin. Subacute cutaneous lupus erythematosus is seen in sun-exposed areas and can involve erythematous plaques or psoriasiform lesions. It is associated with autoantibodies to the Ro (SSA) RNA-binding protein. Mucosal ulcerations, especially of the buccal mucosa and upper palate, result from mucositis and are typical of SLE. Manifestations of vasculopathy are also common in SLE, including arteriolar spasm or infarcts in the nail folds, a diffuse lacey pattern over the skin described as livedo reticularis, and petechial-purpuric or urticarial lesions on the extremities. Vasculopathy in SLE is often associated with the presence of antiphospholipid antibodies.

Musculoskeletal System

Arthralgias and nonerosive arthritis are among the most common clinical features of SLE and are experienced by more than 85% of patients. The

1772

CHAPTER 266  Systemic Lupus Erythematosus  

proximal interphalangeal and metacarpophalangeal joints of the hand are most commonly symptomatic, along with the knees and wrists. In some patients (≈10%), deformities resulting from damage to periarticular tissue can occur, a condition termed Jaccoud arthropathy. The heavy use of corticosteroids in many lupus patients can be accompanied by the development of osteoporosis, including osteoporotic fractures or osteonecrosis, most commonly of the hips, although the underlying vasculopathy can also contribute to joint damage. Inflammation of the muscles with elevated creatine phosphokinase can occur rarely in SLE, and myopathy may be observed as a consequence of corticosteroid therapy. Fibromyalgia, characterized by painful trigger points at characteristic locations, commonly accompanies SLE and can contribute to fatigue and depression.

Renal System

Kidney involvement in SLE is common, with 74% of patients being affected at some time in the course of disease, and is a poor prognostic indicator. Renal pathology is generally attributed to the deposition of circulating immune complexes or in situ formation of these complexes in glomeruli and results in the activation of complement and subsequent recruitment of inflammatory cells. In addition to glomerular inflammation, necrosis, and scarring, renal pathology is characterized by vascular lesions, including thrombotic microangiopathy and extraglomerular vasculitis. Tubulointerstitial disease, including infiltration of the interstitium with mononuclear cells, tubular atrophy, and interstitial fibrosis, is increasingly recognized as associated with a poor prognosis for persistent nephritis and renal survival. Hypertension may be a consequence of significant renal involvement. Most cases of lupus nephritis present a complex immunopathologic picture, but in general, the pattern of renal disease reflects the site of deposition of immunoglobulins and the quality of the effector mechanisms they induce. Mesangial deposition of immunoglobulin induces mesangial cell proliferation and is associated with microscopic hematuria and mild proteinuria (Fig. 266-3). Subendothelial deposition of immune complexes results in proliferative and exudative inflammation, together with hematuria, mild to moderate proteinuria, and reduced glomerular filtration rate. Subepithelial deposition of immune complexes adjacent to podocytes and along the

glomerular basement membrane can result in membranous nephritis with nephrotic-range proteinuria. In addition, antiphospholipid antibodies may support the development of thrombotic or inflammatory vascular lesions within or external to glomeruli. A World Health Organization classification of lupus nephritis lesions was first published in 1975, with subsequent revisions. These classifications were reviewed and rigorously reexamined in the revised International Society of Nephrology and Renal Pathology Society classification criteria for lupus glomerulonephritis (GN) (Table 121-7 in Chapter 121, and also E-Table 2662). Class I and II GN involves mesangial deposition of immune complexes (class I without and class II with mesangial hypercellularity); class III describes focal GN involving less than 50% of total glomeruli; class IV includes diffuse GN involving 50% or more of glomeruli; class V designates membranous lupus nephritis; and class VI is characterized by advanced sclerotic lesions. Classes III and IV have subdivisions for active and sclerotic lesions, and class IV also has subdivisions for segmental and global involvement. Pathologic diagnosis should include descriptions of tubulointerstitial and vascular disease as well as glomerular involvement. The prognosis of class I and class II disease is usually good, whereas class IV, the most common form of lupus nephritis, has the worst prognosis, particularly when the serum creatinine level is elevated at the time of diagnosis. Class V nephritis occurs in 10 to 20% of patients, and the implication for long-term outcome depends on the degree of proteinuria, with mild proteinuria having a good prognosis and nephrotic syndrome with chronic edema having a more negative prognosis. It should be noted that renal veins can occasionally become involved with thrombosis, which then also contributes to nephrotic syndrome. This complication can be evaluated by renal ultrasound (Chapter 121).

Cardiovascular System

Pericarditis and valve nodules were among the first clinical manifestations described in SLE. It is only recently that the extent of premature atherosclerotic disease has been well documented. Pericarditis (Chapter 77) is the most common cardiac manifestation, but it is sometimes recognized only on imaging studies or at autopsy. It is a component of the generalized serositis that is often a feature of SLE and is associated with local autoantibodies and

A

B

C

D

FIGURE 266-3.  Histopathology of lupus nephritis. A, Lupus nephritis class II. A light micrograph of a glomerulus shows mild mesangial hypercellularity (periodic acid–Schiff). B, Lupus nephritis class III (A). Light micrograph showing a glomerulus with segmental endocapillary hypercellularity, mesangial hypercellularity, capillary wall thickening, and early segmental capillary necrosis (methenamine silver). C, Lupus nephritis class IV-G (A/C). A glomerulus manifests global endocapillary proliferation, leukocyte influx and apoptotic bodies, double contours, crescent formation with tubular transformation, early sclerosis, and disruption of Bowman’s capsule (periodic acid–Schiff). D, Thrombotic microangiopathy in a patient with systemic lupus erythematosus and circulating anticoagulant. A glomerulus shows severe capillary and arteriolar thrombosis, endothelial cell swelling and necrosis, neutrophil influx, and stasis of erythrocytes. No signs of immune deposits were found (methenamine silver). (From Wenning JJ, D’Agati VD, Schwartz MM, et al. The classification of glomerulonephritis in systemic lupus erythematosus revisited. J Am Soc Nephrol. 2004;15:241.)

CHAPTER 266  Systemic Lupus Erythematosus  

1772.e1

E-TABLE 266-2  INTERNATIONAL SOCIETY OF NEPHROLOGY/RENAL PATHOLOGY SOCIETY 2003 CLASSIFICATION OF LUPUS NEPHRITIS Class I

Minimal mesangial lupus nephritis Normal glomeruli by light microscopy, but mesangial immune deposits by immunofluorescence

Class II

Mesangial proliferative lupus nephritis Purely mesangial hypercellularity of any degree or mesangial matrix expansion by light microscopy, with mesangial immune deposits. May be a few isolated subepithelial or subendothelial deposits visible by immunofluorescence or electron microscopy, but not by light microscopy

Class III

Focal lupus nephritis* Active or inactive focal, segmental, or global endocapillary or extracapillary glomerulonephritis involving < 50% of all glomeruli, typically with focal subendothelial immune deposits, with or without mesangial alterations

Class III (A)

Active lesions: focal proliferative lupus nephritis

Class III (A/C)

Active and chronic lesions: focal proliferative and sclerosing lupus nephritis

Class III (C)

Chronic inactive lesions with glomerular scars: focal sclerosing lupus nephritis

Class IV

Diffuse lupus nephritis† Active or inactive diffuse, segmental or global endocapillary or extracapillary glomerulonephritis Involving ≥ 50% of all glomeruli, typically with diffuse subendothelial immune deposits, with or without mesangial alterations. This class is divided into diffuse segmental (IV-S) lupus nephritis when ≥ 50% of the involved glomeruli have segmental lesions, and into diffuse global (IV-G) lupus nephritis when ≥ 50% of the involved glomeruli have global lesions. Segmental is defined as a glomerular lesion that involves less than half of the glomerular tuft. This class includes cases with diffuse wire loop deposits but with little or no glomerular proliferation.

Class IV-S (A)

Active lesions: diffuse segmental proliferative lupus nephritis

Class IV-G (A)

Active lesions: diffuse global proliferative lupus nephritis

Class IV-S (A/C)

Active and chronic lesions: diffuse segmental proliferative and sclerosing lupus nephritis Active and chronic lesions: diffuse global proliferative and sclerosing lupus nephritis

Class IV-S (C)

Chronic inactive lesions with scars: diffuse segmental sclerosing lupus nephritis

Class IV-G (C)

Chronic inactive lesions with scars: diffuse global sclerosing lupus nephritis

Class V

Membranous lupus nephritis Global or segmental subepithelial immune deposits or their morphologic sequelae by light microscopy and by immunofluorescence or electron microscopy, with or without mesangial alterations Class V lupus nephritis may occur in combination with class III or IV, in which case both will be diagnosed Class V lupus nephritis shows advanced sclerosis

Class VI

Advanced sclerosis lupus nephritis ≥90% of glomeruli globally sclerosed without residual activity

*Indicate the proportion of glomeruli with active and with sclerotic lesions. † Indicate the proportion of glomeruli with fibrinoid necrosis and/or cellular crescents. From Weening JJ, D’Agati VD, Schwartz MM, et al. The classification of glomerulonephritis in systemic lupus erythematosus revisited. J Am Soc Nephrol. 2004;15:241.

CHAPTER 266  Systemic Lupus Erythematosus  

immune complexes. Pericarditis is usually manifested as substernal chest pain that is improved by bending forward and can be exacerbated by inspiration or coughing. The symptoms and effusions associated with pericarditis are quite responsive to moderate-dose (20 to 30 mg/day of prednisone) corticosteroid treatment. Structural valve abnormalities in SLE range from sterile nodules (originally described by Libman and Sacks) to nonspecific valve thickening (Chapter 75). The nodules are immobile and usually located on the atrial side of the mitral valve and sometimes on the arterial side of the aortic valve. Right-sided lesions are rare. These structural changes may in some cases result in valvular regurgitation. Although valve nodules are detected in most patients with SLE at autopsy, clinically significant valvular heart disease is much less common (1 to 18%). The verrucous valvular lesions of Libman and Sacks are most likely inflammatory in nature and may be associated with the presence of antiphospholipid antibodies. Premature and accelerated atherosclerosis is prevalent in lupus patients, and preclinical atherosclerotic carotid plaque has been documented in 37% of SLE patients as opposed to 15% of age- and sex-matched controls. Traditional cardiovascular risk factors apply, but the diagnosis of SLE is itself a significant risk factor for premature atherosclerosis (Chapter 70). Among the lupus-related mechanisms that confer additional risk for atherosclerosis, IFN-α and oxidative modification of lipid-associated proteins contribute to the accumulation of vascular damage.9 Mortality from atherosclerosis may be up to 10 times greater in patients with SLE than in age- and sex-matched controls. Although not specific to SLE, Raynaud’s phenomenon (Chapter 80), characterized by episodic vasospasm and occlusion of the digital arteries in response to cold and emotional stress, is a feature in up to 60% of SLE patients and contributes to pain and sometimes necrosis of the distal ends of extremities. The character of the digits classically changes from pallor to cyanosis and then to rubor as vascular perfusion becomes impaired and then reperfusion ensues. In addition, small arteries, arterioles, and capillaries can be affected by vasculitis and fibrinoid necrosis, with clinical manifestations that include periungual telangiectases, abdominal pain, and neuropsychiatric symptoms.

Pulmonary System

Pleuritis is the most frequent manifestation of pulmonary involvement in SLE and occurs in about 30% of patients at some point in their disease course. Pleuritis is characterized by pain on respiration and exudative effusions (Chapter 99). Parenchymal disease is less common but may be based on several distinct mechanisms, including pneumonitis in the absence of documented infection and sometimes involving alveolar hemorrhage (in up to 12% of patients), pulmonary embolism secondary to venous thrombosis, or pulmonary hypertension with increased pulmonary resistance and impaired diffusing capacity.

Neuropsychiatric Involvement

Clinical features of SLE that involve the nervous system include both neurologic and psychiatric manifestations. The central and peripheral nervous systems can be affected by the disease. The American College of Rheumatology has identified 19 neuropsychiatric syndromes that can be associated with SLE, and validation of these neuropsychiatric findings has been substantiated in several independent studies. The most common manifestations that are probably attributable to SLE cerebritis include cognitive dysfunction, present in 17 to 66% of SLE patients; psychosis or mood disorder, the former reported in up to 8% of patients; cerebrovascular disease in 5 to 18% of patients; and seizures, present in 6 to 51% of patients. Headaches are also common. Because none of these CNS manifestations is found exclusively in SLE, it can be difficult to be certain that a neuropsychiatric complaint or symptom should be attributed to SLE. Evaluation of neuropsychiatric lupus depends on a careful clinical history and physical and laboratory examinations and, in some cases, imaging studies and analysis of cerebrospinal fluid to rule out infection. Magnetic resonance imaging is useful for detecting intracranial abnormalities, which are seen in 19 to 70% of patients and include white matter lesions, cerebral infarction, venous sinus thrombosis, and sometimes atrophy. More sophisticated imaging techniques such as magnetic resonance angiography and magnetic resonance spectroscopy can be used to assess cerebral blood flow or neuronal metabolism. Cranial nerve and ocular involvement, most likely based on vasculopathy and focal ischemia, can sometimes affect vision. Ocular examination of the

1773

retina can reveal cotton-wool spots as a result of retinal ischemia or necrosis. Although rare, transverse myelopathy, frequently associated with antiphospholipid antibodies, can have devastating consequences, including paraplegia. Sensorimotor neuropathies, often asymmetrical, are more common (up to 28%) and are based on damage to small nerve fibers with vasculopathy in the small arteries that supply the nerve fibers. As is the case with lupus nephritis, the pathophysiologic mechanisms that account for the neuropsychiatric manifestations of SLE are diverse and complex. Recent data suggest that autoantibodies cross-reactive with neuronal cell surface glutamate receptors and DNA may mediate excitotoxic death of neurons and are proposed to contribute to cognitive dysfunction. Antibodies directed against ribosomal P protein have also been associated with neuropsychiatric lupus, and antiphospholipid antibodies can contribute to a procoagulant state, vascular thrombosis, and cerebral ischemia. Cerebral vasculopathy has been clearly demonstrated by angiographic and pathologic studies. Noninflammatory small vessel vasculopathy is the most common lesion and can be associated with microinfarcts. Inflammatory mediators, including the cytokines interleukin (IL)-6 and IFN-α, and matrix metalloproteinases may also contribute to the neuropsychiatric manifestations of SLE.

Gastrointestinal System

Although uncommon, vasculitis of the gastrointestinal tract or mesentery can result in pain and bowel necrosis. Less common than pleuritis and pericarditis, peritonitis can manifest as peritoneal effusion and abdominal pain. Pancreatitis occurs in less than 10% of patients but may also be due to vascular pathology. Lupoid hepatitis, a syndrome that was named for the presence of positive ANAs in patients with chronic active hepatitis, is a misnomer because elevated transaminases are only rarely seen in lupus patients.

Lymphadenopathy

About one third of SLE patients demonstrate diffuse lymphadenopathy at some time during the course of their disease. The nodes are often nontender, and lymphoma is sometimes considered in the differential diagnosis. Biopsy usually reveals follicular hyperplasia, although some histopathologic findings appear similar to the histiocytic necrotizing lymphadenitis that is a feature of Kikuchi’s disease, a self-limited syndrome characterized by fever and lymphadenopathy. Recent multicenter studies have determined the frequency of malignancies in patients with SLE and have found a significant increase in hematologic malignancies, particularly non-Hodgkin’s lymphoma. Splenomegaly is sometimes seen in SLE, and spleen pathology is characterized by a classic onion-skin histology that appears as concentric circles of collagen matrix surrounding splenic arteries and arterioles.

Hematologic System

In addition to autoantibody specificities that are fairly specific for SLE (antiDNA, anti-Sm), antibodies that target each of the cellular blood elements are also common. Anemia is present in about 50% of patients and is multifactorial. It can be associated with a positive Coombs test or microangiopathic hemolysis (Chapter 160) or reflect chronic disease (normochromic, normocytic) (Chapter 158). Leukopenia, particularly lymphopenia, is observed, with the lymphocyte count decreasing in the setting of increased disease activity. Antibodies that bind to lymphocytes and neutrophils have been described, and an increased tendency for lymphocytes to undergo spontaneous apoptosis may contribute to lymphopenia. Idiopathic thrombocytopenic purpura (Chapter 172) can be an early manifestation of SLE, and thrombocytopenia induced by antiplatelet autoantibodies can sometimes lead to a life-threatening risk for hemorrhage. Autoantibodies to clotting factors can also occur and contribute to impaired clot formation and hemorrhage.

Lupus Pregnancy and Neonatal Lupus

Whether pregnancy increases the likelihood of lupus exacerbation has been debated, with differences on this point presented by different investigators. However, abundant data indicate that patients with SLE have worse fetal outcomes than healthy individuals.10 Gestational hypertension, fetal growth restriction, and fetal distress are increased in patients with SLE and may lead to fetal loss or premature delivery. Preeclampsia can contribute to a poor outcome in both the mother and fetus and can be difficult to distinguish from a lupus flare associated with lupus nephritis. Neonatal lupus is a distinct entity that can occur in infants of mothers with or without a diagnosis of SLE.11 The syndrome is characterized by cutaneous lesions and congenital heart block in the infant and the presence of antibodies

1774

CHAPTER 266  Systemic Lupus Erythematosus  

TABLE 266-2  UPDATE OF THE 1982 REVISED CRITERIA FOR CLASSIFICATION OF SYSTEMIC LUPUS ERYTHEMATOSUS CRITERION*

DEFINITION

1. Malar rash

Fixed erythema, flat or raised, over the malar eminences that tends to spare the nasolabial folds

2. Discoid rash

Erythematous raised patches with adherent keratotic scaling and follicular plugging; atrophic scarring may occur in older lesions

3. Photosensitivity

Rash as a result of unusual reaction to sunlight, by history or physician observation

4. Oral ulcers

Oral or nasopharyngeal ulceration, usually painless, observed by a physician

5. Arthritis

Nonerosive arthritis involving 2 or more peripheral joints and characterized by tenderness, swelling, or effusion

6. Serositis

A. Pleuritis—convincing history of pleuritic pain or rubbing heard by a physician or evidence of pleural effusion or B. Pericarditis—documented by electrocardiography, a rub, or evidence of pericardial effusion

7. Renal disorder

A. Persistent proteinuria > 0.5 g/day or > 3+ if quantitation is not performed or B. Cellular casts—may be red cell, hemoglobin, granular, tubular, or mixed

8. Neurologic disorder

A. Seizures—in the absence of offending drugs or known metabolic derangements (e.g., uremia, ketoacidosis, electrolyte imbalance) or B. Psychosis—in the absence of offending drugs or known metabolic derangements (e.g., uremia, ketoacidosis, electrolyte imbalance)

9. Hematologic disorder

A. Hemolytic anemia—with reticulocytosis or B. Leukopenia— 1.6) suggests pulmonary vascular disease. Evidence of interstitial lung disease can be found in almost all patients with SSc and is clinically significant in up to 50%. Risk factors include male sex, African American race, diffuse skin involvement, severe gastroesophageal reflux, and the presence of topoisomerase-I autoantibodies. The most rapid progression in interstitial lung disease occurs within the first 3 years of the disease. Chest radiography is useful for ruling out infection and other causes of pulmonary involvement but is relatively insensitive for detection of early interstitial lung disease. In contrast, HRCT is highly sensitive (Chapter 84). Prominent HRCT findings include reticular linear interstitial opacities,

CHAPTER 267  Systemic Sclerosis (Scleroderma)  

predominantly in the lower lobe periphery, occurring in isolation or in combination with ground-glass opacification. Additional findings include mediastinal lymphadenopathy and, rarely, honeycombing. The extent of lung disease on initial HRCT correlates with progression and prognosis of interstitial lung disease and may provide useful information regarding the need for initiating therapy. Bronchoalveolar lavage (Chapter 85) may be indicated for ruling out occult infection. Lung biopsy is rarely useful. Pulmonary Arterial Hypertension

Approximately 15% of SSc patients develop PAH, defined as a mean pulmonary arterial pressure of 25 mm Hg or greater, with a pulmonary capillary wedge pressure of 15 mm Hg or less (Chapter 68). In the setting of SSc, PAH can occur as an isolated abnormality (World Health Organization [WHO] group I), or coexist with interstitial lung disease (WHO group III). Although the natural history of SSc-associated PAH is variable, patients generally follow a progressive downhill course, with development of right heart failure and increased mortality. Risk factors for SSc-associated PAH include limited cutaneous disease, older age of disease onset, severe Raynaud phenomenon, large number of cutaneous telangiectasias, and anticentromere, U1-RNP, U3-RNP (fibrillarin), Th/To, B23, and β2-glycoprotein I autoantibodies. The initial symptoms of PAH are exertional dyspnea and reduced exercise capacity, but early-stage disease is often clinically silent. With progression, angina, syncope, and symptoms and signs of right-sided heart failure develop. Physical examination shows tachypnea, a prominent pulmonic S2 heart sound, palpable right ventricular heave, elevated jugular venous pressure, and dependent edema. Pulmonary arterial systolic pressures above 40 mm Hg (determined by Doppler echocardiography) suggest PAH, as does an isolated low Dlco or a FVC/Dlco ratio over 1.6. Right heart catheterization is required for confirming the diagnosis of PAH, assessing its severity, and evaluating ventricular function. The serum levels of N-terminal brain natriuretic peptide (NT-pro-BNP) are elevated in PAH and correlate with severity and survival.

1783

Once scleroderma renal crisis sets in, hospitalization and prompt initiation of short-acting ACE inhibitors is essential. The goal is adequate blood pressure control before the onset of renal failure. Despite appropriate timely intervention, more than half of patients with scleroderma renal crisis require hemodialysis, although some of these ultimately recover sufficient renal function to be able to discontinue hemodialysis. Oliguria or a serum creatinine level higher than 3 mg/dL at presentation predict poor outcome. The “prophylactic” use of ACE inhibitors to prevent scleroderma renal crisis is associated with a worse outcome and is not recommended. Chronic Kidney Disease

Kidney biopsies in patients with SSc commonly show chronic changes including reduplication of elastic fibers, sclerosed glomeruli, tubular atrophy, and interstitial fibrosis. In one study, abnormal renal function or proteinuria was detected in more than one third of patients, none of whom progressed to end-stage renal disease. Rarely, glomerulonephritis associated with lupus serologies or antineutrophil cytoplasmic antibody–positive renal vasculitis occurs.

Cardiac Involvement

Scleroderma renal crisis is an uncommon but life-threatening acute complication of SSc, but chronic and indolent kidney disease also occurs.

Cardiac involvement is frequently detected using sensitive diagnostic tools but is commonly clinically silent. Clinical cardiac involvement is more frequently seen in patients with dcSSc; it generally develops early in the course of the disease and is a poor prognostic factor. The endocardium, myocardium, and pericardium may be affected separately or together. Clinical manifestations include tachyarrhythmias, conduction abnormalities, valvular regurgitation, diastolic heart failure, and pericardial effusion. Systemic and pulmonary arterial hypertension, as well as lung and renal involvement, also affect the heart. Conventional echocardiography has a low sensitivity for detecting SSc heart involvement. Tissue Doppler echocardiography, single-photon emission computed tomography, and especially cardiac magnetic resonance imaging (cMRI) reveal a high prevalence of myocardial abnormalities such as abnormal ventricular relaxation and reversible perfusion defects. An elevated level of serum NT-pro-BNP is a sensitive marker for increased pulmonary artery pressure but may also indicate primary cardiac involvement. Myocarditis can develop in association with muscle inflammation. Pericardial effusion develops in more than 15% of SSc patients but is rarely significant.

Scleroderma Renal Crisis

Musculoskeletal Complications

Kidney Involvement

Scleroderma renal crisis, the most dreaded complication of SSc, develops in 10 to 15% of patients, almost always within 4 years of disease onset.11 Prior to the advent of angiotensin-converting enzyme (ACE)-inhibiting drugs in the 1980s, scleroderma renal crisis was invariably fatal, often within weeks. The pathogenesis involves obliterative vasculopathy and luminal narrowing of the renal arcuate arteries. Progressive reduction in renal blood flow, aggravated by vasospasm, leads to juxtaglomerular hyperplasia and increased renin secretion, with further renal vasoconstriction resulting in a vicious cycle that culminates in malignant hypertension (Chapters 67 and 125). Scleroderma renal crisis is a medical emergency. Although most patients present with abrupt onset of hypertension and progressive renal insufficiency, in some cases the blood pressure remains normal or only modestly elevated. Normotensive renal crisis is associated with a poor outcome. Hypertensive encephalopathy and retinopathy, pericarditis, and arrhythmias may complicate scleroderma renal crisis. Urinalysis shows mild proteinuria, granular casts, and microscopic hematuria. When thrombocytopenia and microangiopathic hemolysis with fragmented red blood cells are detected, the diagnosis of thrombotic thrombocytopenic purpura (Chapter 172) is sometimes entertained. In many patients, oliguric renal failure develops over a period of weeks. Kidney biopsy can be useful for diagnosis and prognosis, but the characteristic lesions of intimal and medial proliferation and luminal narrowing are indistinguishable from the changes of accelerated hypertension. Risk factors for scleroderma renal crisis include early-stage disease, rapidly progressive skin involvement and the presence of tendon friction rubs, African American race, male sex, and autoantibodies to RNA polymerases I and III. In contrast, the presence of anticentromere antibodies is associated with a low risk for scleroderma renal crisis. Pericardial effusion, new-onset anemia, and thrombocytopenia may be harbingers of impending scleroderma renal crisis, and a history of recent corticosteroid use is associated with a more than 10-fold increased risk. Accordingly, SSc patients with early and progressive cutaneous disease should be counseled to determine their blood pressure daily. In these patients, corticosteroids should be used only when absolutely required, and at low doses.

Carpal tunnel syndrome (Chapter 420) occurs frequently and may be a presenting manifestation of SSc. Joint mobility is progressively impaired, especially in the hands. Large joint contractures can be accompanied by audible or palpable tendon friction rubs that are caused by extensive fibrosis and adhesion of the tendon sheaths and fascial planes at the affected joint. The presence of tendon friction rubs often signals aggressive disease. Frank joint inflammation is uncommon in SSc; however, erosive polyarthritis in the hands can occur. Muscle weakness may be a sign of deconditioning, disuse atrophy, and malnutrition. Less commonly, inflammatory myositis indistinguishable from idiopathic polymyositis (Chapter 269) occurs in early disease. A noninflammatory myopathy characterized by atrophy and fibrosis in the absence of elevated muscle enzyme levels may occur in late disease. Bone resorption affects the distal tufts of terminal phalanges (acro-osteolysis), mandibular condyles, ribs, and distal clavicles.

Other Clinical Manifestations

In addition to microangiopathy, involvement of larger blood vessels (>100 µm) is common in SSc. Manifestations of macrovascular disease include occlusion of the digital and ulnar arteries, leading to ischemic ulcerations and even loss of digits or limbs. Epidemiologic studies indicate increased risk of coronary artery disease in patients with SSc. Dry eyes and dry mouth are common in SSc, but in contrast to Sjögren syndrome (Chapter 268), salivary gland biopsy in such cases shows fibrosis rather than focal lymphocytic infiltration. Hypothyroidism due to thyroid fibrosis is common and may be associated with antithyroid autoantibodies. Although the brain and central nervous system are generally spared in SSc, autonomic neuropathy, as well as a primarily sensory neuropathy of the trigeminal nerve due to fibrosis or vasculopathy, can occur. Pregnancy in women with active SSc is associated with an increased rate of adverse fetal outcomes. Furthermore, cardiopulmonary involvement might worsen during pregnancy, and scleroderma renal crisis can occur. Inability to attain or maintain penile erection is due to vascular insufficiency and fibrosis; the problem is frequent and may be the presenting disease manifestation in males with SSc.

1784

CHAPTER 267  Systemic Sclerosis (Scleroderma)  

Systemic Sclerosis and Cancer

Patients with SSc have an increased risk of cancer. In these patients, lung cancer and esophageal adenocarcinoma typically occur in the setting of longstanding interstitial lung disease or GERD, and chronic inflammation and tissue repair may be contributing factors. In contrast, breast, lung, and ovarian carcinoma and lymphoma in SSc tend to occur in close temporal association with the clinical onset of SSc and are often associated with anti-RNA polymerase III antibodies. In these cases, SSc might be a paraneoplastic syndrome that is triggered by the antitumor immune response.

DIAGNOSIS

Skin induration in the fingers or proximally (associated with Raynaud phenomenon) and characteristic visceral organ manifestations are sufficient to establish the diagnosis of SSc. Occasionally, diagnostic full-thickness skin biopsy may be required for ruling out scleroderma mimics such as scleredema, scleromyxedema, or nephrogenic systemic fibrosis (see Table 267-1). Primary Raynaud phenomenon is differentiated from SSc by normalappearing nailfold capillaries and absence of autoantibodies. Diagnosing SSc can be difficult in the early stages of the disease because initial symptoms and findings are often nonspecific and can be mistaken for rheumatoid arthritis, SLE, myositis, or undifferentiated connective tissue disease. Rarely, patients with SSc first present with accelerated hypertension or GI bleeding caused by watermelon stomach as the initial manifestation.

Laboratory Features

Anemia is common and may reflect chronic inflammation, GI bleeding from gastric vascular ectasia, erosive gastritis or chronic esophagitis, or folate and vitamin B12 deficiency due to small bowel bacterial overgrowth and malabsorption. Microangiopathic hemolytic anemia (Chapter 160) caused by mechanical trauma and red blood cell fragmentation is a hallmark of scleroderma renal crisis. In contrast to other connective tissue diseases, the erythrocyte sedimentation rate and C-reactive protein generally show only modest elevation. Monitoring serum levels of prealbumin and vitamin K is useful in patients with small bowel bacterial overgrowth and malabsorption. Antinuclear autoantibodies (ANAs) are present in virtually all patients with SSc and can be detected at, or even prior to, disease onset. Autoantibodies specific for SSc are described in Table 267-3. Anticentromere antibodies are associated with PAH, but cardiac involvement, significant pulmonary fibrosis, or scleroderma renal crisis occurs only rarely in these patients. Topoisomerase-I antibody positivity is associated with reduced survival, whereas anticentromere antibody–positive patients have improved survival compared with those without this antibody. Antibodies to RNA polymerase III (recognized based on its speckled immunofluorescence pattern) are associated with increased risk for scleroderma renal crisis. Antibodies to β2glycoprotein I are not specific but in SSc identify increased risk for critical ischemia.

TREATMENT AND PREVENTION  With the exception of ACE inhibitors for scleroderma renal crisis, no therapy to date has been shown to significantly alter the natural history of SSc. In contrast, organ-based treatments are effective in alleviating symptoms and slowing progression of the cumulative organ damage. A significant reduction in disease-related mortality has occurred during the past 25 years. Treatment must be tailored to each patient’s unique needs. Because of the marked heterogeneity in clinical presentation, a thorough and individualized baseline evaluation is paramount. Optimal management incorporates the following principles: prompt diagnosis, accurate classification and risk stratification, early recognition and assessment of organ-based complications, and monitoring progression, disease activity, and response to therapy. Management of complications should be proactive, with regular screening and initiation of appropriate intervention at the earliest possible opportunity. Given the multisystemic nature of SSc, an integrated team-based management approach, typically at specialized medical centers, is desirable. The team should incorporate appropriate medical specialists. Patients are empowered by learning about potential complications, therapeutic options, and the natural history of their disease.

Disease-Modifying Therapy Immunosuppressive Agents

Immunosuppressive agents that are highly effective in the treatment of other connective tissue diseases have generally shown modest or no benefit in SSc.12 Corticosteroids may alleviate stiffness and aching in early-stage dcSSc

but do not slow the progression of skin or internal organ involvement and are associated with an increased risk for scleroderma renal crisis. Therefore, corticosteroids should be avoided if possible; when absolutely necessary, they should be given at the lowest dose possible and for brief periods only. Cyclophosphamide was shown to reduce the progression of symptomatic interstitial lung disease in early SSc. A1  A2  Compared with placebo, patients treated with oral cyclophosphamide showed stabilization and, rarely, modest improvement in respiratory symptoms, pulmonary function, and abnormalities on chest HRCT after 1 year of treatment, but these benefits were shortlived. The use of cyclophosphamide in SSc needs to be balanced against  its potential for side effects, including bone marrow suppression, opportunistic infections, hemorrhagic cystitis, bladder cancer, and premature ovarian failure. In small clinical trials, methotrexate was associated with a modest improvement in skin involvement. Mycophenolate mofetil treatment was shown to improve skin involvement and stabilize lung disease. There is some support in the literature for the use of immunomodulatory agents and interventions including rituximab, intravenous immunoglobulin, and extracorporeal photopheresis for the treatment of SSc. Recent reports suggest that rituximab might be effective in ameliorating skin and lung involvement. In patients with severe SSc who fail to respond to other treatments (Chapter 178), autologous hematopoietic stem cell transplantation (HSCT) improves long term, event-free survival despite an increased treatment-related mortality in the first year. A3  Because of this potential morbidity and mortality and its substantial cost, HSCT is presently considered an investigational therapy for SSc. ,

Antifibrotic Therapy

Because tissue fibrosis causes progressive and irreversible organ damage, drugs that block or slow the fibrotic process represent a rational approach to therapy. d-Penicillamine has been extensively used as an antifibrotic agent. In retrospective studies, d-penicillamine stabilized and improved skin induration, prevented new internal organ involvement, and improved survival. However, in a randomized controlled clinical trial, there was no difference in the extent of skin involvement between patients treated with standard-dose (750 mg/ day) or very low-dose (125 mg every other day) d-penicillamine. Minocycline, bosentan, recombinant relaxin, interferon-γ, and inhibitors of tumor necrosis factor are putative antifibrotic agents that have failed to show meaningful benefit in SSc clinical trials. Small-molecule inhibitors of protein tyrosine kinases used in malignancies (e.g., imatinib, nilotinib, and dasatinib) block signaling by TGF-β and PDGF and thereby prevent fibrotic responses in vitro and in vivo. These agents are currently in clinical trials for SSc.

Treatment of Organ-Specific Complications Gastrointestinal Complications

Because significant gastroesophageal reflux may occur in the absence of symptoms, all patients with SSc should be treated for this complication. Proton pump inhibitors may need to be given in relatively high doses and for prolonged periods, and patients should be instructed to elevate the head of the bed and eat frequent small meals. Recurrent GI bleeding due to GAVE can be treated with laser or argon plasma photocoagulation. Bacterial overgrowth due to small bowel hypomotility causes bloating and diarrhea and may lead to malabsorption, weight loss, and malnutrition. Treatment with short courses of rotating broad-spectrum antibiotics such as metronidazole, erythromycin, and tetracycline can eradicate bacterial overgrowth, but many patients relapse when antibiotics are stopped. In patients with malnutrition but intact small bowel function, enteral nutrition via a jejunostomy can be effective. In others, total parenteral nutrition may be indicated. Refractory hypomotility of the small bowel may respond to subcutaneous octreotide injections. Anorectal complications may respond to sacral neuromodulation.

Vascular Therapy and Raynaud Phenomenon

The goal of therapy is to reduce the frequency and duration of vasospastic episodes, prevent ischemic complications and enhance their healing, and slow the progression of obliterative vasculopathy. Patients should dress warmly, minimize cold exposure, and avoid drugs that could precipitate or exacerbate vasospastic episodes. Calcium-channel blockers such as nifedipine and diltiazem are used commonly for Raynaud’s phenomenon but show only moderate benefit, and their use is often limited by side effects (palpitations, dependent edema, light-headedness). ACE inhibitors do not reduce the frequency or severity of episodes, but angiotensin II receptor blockers such as losartan are effective and generally well tolerated. Patients with severe Raynaud phenomenon require α1-adrenergic receptor blockers (e.g., prazosin), 5-phosphodiesterase inhibitors (e.g., sildenafil), topical nitroglycerine, or intravenous prostaglandins. Low-dose aspirin and dipyridamole prevent platelet aggregation and may have a role as adjunctive agents but must be used with caution in light of the risk of bleeding from GAVE lesions. The endothelin-1 receptor antagonist bosentan reduces development of new ischemic ulcers. Intravenous prostacyclin infusion, local injections of botulinum toxin, and digital sympathectomy are options for some patients with critical digital ischemia. Patients with ischemic digital ulcerations may require

CHAPTER 268  Sjögren Syndrome  

surgical débridement, especially if necrotic tissue is present. Empirical longterm therapy with statins and antioxidants may slow the progression of vascular damage.

Pulmonary Arterial Hypertension

All patients with SSc should be screened for PAH at initial evaluation, and those at high risk on a yearly basis. Treatment for symptomatic PAH should be started with an endothelin-1 receptor antagonist or a 5-phosphodiesterase inhibitor. Diuretics, oral anticoagulation, and digoxin may be used when appropriate. If hypoxemia is documented, supplemental oxygen should be given. If clinical response is inadequate, 5-phosphodiesterase inhibitors may be used in combination with endothelin-1 receptor antagonists. Prostacyclin analogues can be administered intravenously, by continuous subcutaneous infusion, or by frequent inhalations. Lung transplantation remains an option for selected patients with SSc-associated PAH who fail medical therapy.

Treatment and Prevention of Scleroderma Renal Crisis

Prompt recognition of impending or early scleroderma renal crisis is essential. Because patients with early-stage SSc and progressive skin involvement are at highest risk, they should monitor their blood pressure daily and report significant alterations immediately. Corticosteroids should be used only when absolutely necessary and at the lowest possible doses. When scleroderma renal crisis occurs, patients should be hospitalized and treatment with shortacting ACE inhibitors started immediately to achieve prompt blood pressure normalization. There is no evidence that “prophylactic” use of ACE inhibitors can prevent the development of scleroderma renal crisis or ameliorate its severity. Although up to two thirds of patients who develop renal crisis require dialysis, delayed recovery of renal function can occur. Kidney transplantation is appropriate for patients unable to discontinue dialysis after 2 years. Survival with renal transplantation in SSc is comparable to that in other connective tissue diseases, and recurrence of scleroderma renal crisis is rare.

Skin Care

Skin involvement in early SSc is inflammatory and can be controlled with systemic antihistamines or short-term low-dose corticosteroids. Because of the increased risk for scleroderma renal crisis, blood pressure should be carefully monitored. Cyclophosphamide, methotrexate, d-penicillamine, and mycophenolate have been associated with modest improvement in skin induration in early-stage SSc. Skin dryness can be managed with the use of hydrophilic ointments and emollient bath oils. Fingertip ulcerations should be protected by occlusive dressing to promote healing and prevent infection. Infected skin ulcers are treated with topical or oral antibiotics and may necessitate surgical débridement. No medical therapy has been shown to be effective in preventing soft tissue calcification or in promoting its dissolution, and surgical therapy is only occasionally effective.

PROGNOSIS AND NATURAL HISTORY

Patients with dcSSc have a more rapidly progressive course, greater internal organ involvement, and worse prognosis compared to those with lcSSc. However, the outcome of the disease is difficult to predict. Early inflammatory symptoms of dcSSc such as fatigue, edema, arthralgia, and pruritus commonly subside after 2 to 4 years, and skin thickening reaches a plateau followed by slow regression, which characteristically occurs in an order that is the reverse of initial involvement, with softening on the trunk followed by proximal and finally the distal extremities. Sclerodactyly and finger contractures generally persist. Relapse or recurrence of skin thickening is rare. Visceral organ involvement develops and progresses most rapidly during the initial 2 to 4 years of the disease. New organ involvement rarely occurs once the skin involvement has plateaued. Similarly, scleroderma renal crisis almost invariably occurs within the first 4 years of disease. In patients with lcSSc, Raynaud phenomenon may precede other disease manifestations by years or even decades, and visceral organ complications such as PAH and primary biliary cirrhosis generally occur late in the course of the disease. Age- and gender-adjusted mortality rates in patients with SSc are more than five-fold higher than in the general population. The 10-year survival rate is 55% for patients with dcSSc and 75% for patients with lcSSc. Survival correlates with the extent of skin involvement, which represents a surrogate for visceral organ involvement. The leading causes of death are pulmonary fibrosis, PAH, severe GI involvement, and cardiac disease. Markers of poor prognosis include male sex, African American race, older age of disease onset, low body mass index, extensive skin thickening with truncal involvement, and evidence of significant or progressive visceral organ involvement. Autoantibodies to topoisomerase-I or absence of anticentromere antibodies are markers of poor prognosis. In one study, SSc patients who had extensive skin involvement, vital capacity less than 55% of predicted, significant GI involvement, and clinically evident cardiac involvement or scleroderma renal crisis

1785

had a less than 40% 10-year survival. The severity of PAH is correlated with mortality, and SSc patients with a mean pulmonary arterial pressure of 45 mm Hg or higher had a 33% 3-year survival rate. In scleroderma renal crisis, therapy with ACE inhibitors has had a dramatic effect on survival, increasing from less than 10% at 1 year in the pre-ACE inhibitor era to better than 70% 3-year survival at the present time.

  NEPHROGENIC SYSTEMIC FIBROSIS

Nephrogenic systemic fibrosis (NSF) is a novel complication of renal insufficiency with certain clinical features resembling SSc.13 The condition was initially described in 2000 and is now recognized as an emerging problem in patients with chronic renal failure. It is estimated that 2% of patients on longterm hemodialysis might develop NSF. Originally considered a purely dermatologic scleromyxedema-like condition and termed nephrogenic fibrosing dermatopathy, NSF is now recognized to be associated with visceral organ involvement and is therefore more accurately termed nephrogenic systemic fibrosis. The cutaneous manifestations of NSF share histopathologic and clinical features with other scleroderma-spectrum disorders, notably fasciitis and scleromyxedema. In most patients with NSF, the condition develops while undergoing long-term dialysis. However, no association with a particular route or type of renal replacement therapy has been demonstrated. Furthermore, NSF has also been described in patients who have never received dialysis. Histologic hallmarks include cutaneous fibrosis with mucin deposition and accumulation of spindle-shaped cells, including numerous CD34positive cells, in the lesional skin. The clinical hallmark of NSF is thickening and “woody” tightness of skin over the lower and, less commonly, upper extremities and contractures at large joints. A link between NSF and exposure to gadolinium-containing MRI contrast agents was suggested in 2006, leading to a warning by the U.S. Food and Drug Administration regarding the use of these agents in patients with renal insufficiency. This was followed by a substantial decline in the incidence of NSF. The course of NSF is generally progressive, and the prognosis is poor. Some patients show improvement with adjustment to renal replacement therapy, and others respond to renal transplantation. Anecdotal reports describe treatment with phototherapy, imatinib mesylate, and immunosuppressive agents. However, in most patients with NSF, the induration is resistant to therapy and leads to progressive induration, joint contractures, and reduced mobility. The topic of immunoglobulin (Ig)G4-related disease is discussed in Chapter 275.

Grade A References A1. Tashkin DP, Celli B, Senn S, et al. Cyclophosphamide versus placebo in scleroderma lung disease. N Engl J Med. 2006;354:2655-2666. A2. Hoyles RK, Ellis RW, Wellsbury J, et al. A multicenter, prospective, randomized, double-blind, placebo-controlled trial of corticosteroids and intravenous cyclophosphamide followed by oral azathioprine for the treatment of pulmonary fibrosis in scleroderma. Arthritis Rheum. 2006;54: 3962-3970. A3. van Laar JM, Farge D, Sont JK, et al. Autologous hematopoietic stem cell transplantation vs intravenous pulse cyclophosphamide in diffuse cutaneous systemic sclerosis: a randomized clinical trial. JAMA. 2014;311:2490-2498.

GENERAL REFERENCES For the General References and other additional features, please visit Expert Consult at https://expertconsult.inkling.com.

268  SJÖGREN SYNDROME XAVIER MARIETTE

DEFINITION

Sjögren syndrome (SS) is a systemic autoimmune disease characterized by lymphocytic infiltrates of salivary and tear glands, leading to oral and ocular dryness, and by autoantibody secretion. It can be encountered either alone

CHAPTER 267  Systemic Sclerosis (Scleroderma)  

GENERAL REFERENCES 1. van den Hoogen F, Khanna D, Fransen J, et al. 2013 classification criteria for systemic sclerosis: an American College of Rheumatology/European League against Rheumatism collaborative initiative. Arthritis Rheum. 2013;65:2737-2747. 2. Katsumoto TR, Whitfield ML, Connolly MK. The pathogenesis of systemic sclerosis. Annu Rev Pathol. 2011;6:509-537. 3. Trojanowska M. Cellular and molecular aspects of vascular dysfunction in systemic sclerosis. Nat Rev Rheumatol. 2010;6:453-460. 4. Bhattacharyya S, Wei J, Varga J. Understanding fibrosis in systemic sclerosis: shifting paradigms, emerging opportunities. Nat Rev Rheumatol. 2011;8:42-54. 5. Bussone G, Bérezné A, Pestre V, et al. The scleroderma kidney: progress in risk factors, therapy, and prevention. Curr Rheumatol Rep. 2011;13:37-43. Review emphasizing use of ACE inhibitors and avoidance of corticosteroids. 6. Goundry B, Bell L, Langtree M, et al. Diagnosis and management of Raynaud’s phenomenon. BMJ. 2012;344:e289. Review. 7. van Bon L, Affandi AJ, Broen J, et al. Proteome-wide analysis and CXCL4 as a biomarker in systemic sclerosis. N Engl J Med. 2014;433-443.

1785.e1

8. O’Reilly S. Innate immunity in systemic sclerosis pathogenesis. Clin Sci (Lond). 2014;126: 329-337. 9. Bhattacharyya S, Tamaki Z, Wang W, et al. FibronectinEDA promotes chronic cutaneous fibrosis through Toll-like receptor signaling. Sci Transl Med. 2014;6:232ra50. 10. Akter T, Silver RM, Bogatkevich GS. Recent advances in understanding the pathogenesis of scleroderma-interstitial lung disease. Curr Rheumatol Rep. 2014;16:411. 11. Mouthon L, Bérezné A, Bussone G, et al. Scleroderma renal crisis: a rare but severe complication of systemic sclerosis. Clin Rev Allergy Immunol. 2011;40:84-91. Treatment relies on aggressive control of blood pressure with ACE inhibitors, but dialysis is frequently needed, and 5-year survival is 65%. 12. Denton CP, Ong VH. Targeted therapies for systemic sclerosis. Nat Rev Rheumatol. 2013;9: 451-464. 13. Daftari Besheli L, Aran S, Shaqdan K, et al. Current status of nephrogenic systemic fibrosis. Clin Radiol. 2014;69:661-668.

1785.e2

CHAPTER 267  Systemic Sclerosis (Scleroderma)  

REVIEW QUESTIONS 1. Which of the following does not contribute to the pathogenesis of systemic sclerosis (SSc)? A. Vascular wall remodeling B. Tissue fibrosis C. Vasculitis D. T cells E. Hypoxia Answer: C  Vasculitis does not typically occur in SSc. Vascular remodeling leading to hypoxia, inflammation, and fibrosis are the hallmarks of the disease. 2. Which is a characteristic autoantibody seen in SSc patients? A. Anticentromere B. Anti-CCP C. Antihistone D. Antiphospholipid E. Anti-Smith Answer: A  Anticentromere is a hallmark SSc-associated autoantibody, whereas the other autoantibodies are seen in rheumatoid arthritis (B), lupus (C, E), and the antiphospholipid syndrome (D). 3. Which is a major factor in SSc morbidity? A. Glomerulonephritis B. Episcleritis C. Amyloidosis D. Pulmonary hypertension E. Bowel infarction Answer: D  Pulmonary hypertension develops in up to 15% of SSc patients and is a major cause of morbidity and mortality. The other clinical features are uncommon in SSc and indicate other autoimmune or rheumatic diseases.

4. In patients with SSc-associated lung disease, lung biopsy may show: A. Honeycombing B. Interstitial fibrosis C. Plasma cell accumulation D. Intimal proliferation in the small vessels E. All of the above Answer: E  All of the listed pathologic features can be noted in lung biopsies from patients with SSc-associated interstitial lung disease. 5. Which of the following drugs may contribute to scleroderma renal crisis? A. NSAIDs B. Glucocorticoids C. Mycophenolate D. Rituximab E. Penicillamine Answer: B  The use of glucocorticoids has been shown to increase the risk of new-onset scleroderma renal crisis in patients with SSc.

1786

CHAPTER 268  Sjögren Syndrome  

(primary Sjögren syndrome [pSS]) or in the presence of other systemic autoimmune diseases (secondary Sjögren syndrome [sSS]) like rheumatoid arthritis (RA), systemic lupus erythematosus (SLE), inflammatory myositis, and systemic sclerosis. SS in the setting of RA usually follows RA diagnosis by many years and is mainly manifested by keratoconjunctivitis sicca, with systemic features being rather uncommon. Associated with other systemic autoimmune disease, the presentation of sSS is very close to pSS. Of note, pSS may be also associated with organ-specific systemic autoimmune disease, such as autoimmune thyroiditis and primary biliary cirrhosis.

EPIDEMIOLOGY

Primary SS is a common disease that affects 0.1 to 0.6% of the general adult female population.1 A higher prevalence of the disease has been reported (0.5 to 2%), but this must be considered with caution because the reported prevalence of SS depends on the classification criteria used in the various studies, and the prevalence of sicca symptoms in the general population is high. Conversely, in recent studies with strict criteria, a lower prevalence has been found: 1.02 per 10,000 adults.2 Primary SS has a female preponderance (female-to-male ratio at least 9 : 1). The age peak of the disease occurs after menopause in the mid-50s.

PATHOPHYSIOLOGY

Recent years have witnessed major advances in the pathophysiologic mechanisms of the disease. Several studies have confirmed the role of innate immunity, genetics, and B-cell activation and the relation between abnormalities in them. The presence of an interferon (IFN) signature has been shown both in salivary glands and blood.3 Plasmacytoid dendritic cells, the professional cells secreting type 1 IFN, are present within the glands. Type 2 IFN-dependent genes can be overexpressed in salivary glands. Natural killer (NK) cells, another actor of innate immunity able to secrete type 2 IFN are present in salivary glands of patients and play a role in the disease.4

In line with this IFN signature, multiple viral agents have been incriminated as etiologic factors for either the development or the modulation of SS; these include Epstein-Barr virus, retroviruses, and coxsackieviruses, but in all cases the data remain controversial.5 The genetics of pSS is now better understood with the reports of two genome-wide association studies (GWAS).6 Like in other systemic autoimmune diseases, HLA is the most important region associated with the disease, and especially HLA-DR3-DQ1 in patients with autoantibodies. Interestingly, other genes associated with the disease are involved in the IFN response. These include IFN regulatory factor 5 (IRF-5), a pivotal transcription factor in the type 1 IFN pathway, and signal transducer and activator of transcription 4 (STAT-4), and IL-12A, involved in the type 2 IFN pathway. Other genes found to be associated with the disease are TNIP1, playing a role in control of nuclear factor (NF)-κB activation, and CXCR5, involved in germinal center formation. The presence of ectopic salivary gland germinal centers demonstrates the importance of B-cell activation in pSS. Different cytokines may explain this B-cell activation. Several studies have focused on the role of BAFF (B-cell activating factor of the tumor necrosis factor [TNF] family), a cytokine that promotes B-cell maturation, proliferation, and survival. It has been shown that BAFF is enhanced in sera and in salivary glands from pSS patients. Interestingly, BAFF can be secreted by salivary gland epithelial cells, the target of autoimmunity, after stimulation by the innate immune system (type 1 or type 2 IFN, or viral infections). Thus, this cytokine is likely to be a link between innate immunity and autoimmunity. The current hypothetical scenario for the development of pSS is based on the successive activation of innate and adaptive immune systems (Fig. 2681). Environmental factors such as viral infections or hormonal imbalance may act at the initial stage of the disease by activating epithelial cells. This epithelial cell activation is promoted in patients who carry susceptibility factors in the genes for IFN pathway proteins. These patients experience a greater degree of IFN pathway activation, which leads to BAFF overproduction,

Viral or hormonal components PDC TLR stimulation IRF5* STAT4* * Autoimmune associated Other TF polymorphisms Female prevalence IFNα and other inflammatory genes

B Cell

HLA Class II restrictions T Cell

BAFF Apoptosis

DC Blebs

Lymphoid hyperplasia

Autoantibodies α-M3R α-Ro α-La

on

ati

iltr

Inf

Oligoclonal expansion

Apoptosis

Glandular dysfunction FIGURE 268-1.  Hypothetical scenario for development of primary Sjögren syndrome. An environmental factor (e.g., virus) causes epithelial cell and dendritic cell (DC) activation. Plasmacytoid DCs are also activated by immune complexes, promoting interferon (IFN) pathway activation, which leads to BAFF overproduction and to B- and T-cell activation. B-cell activation leads to autoantibody production within germinal center–like structures. Interleukin-12 secreted by myeloid DCs leads to natural killer cell and T-helper 1 activation, which promotes tissue damage and IFN-γ production. IFN-α and IFN-γ enhance BAFF secretion. Epithelial cells release autoantigens that participate in immune complex formation and perpetuate the vicious cycle of immune system overactivation. BAFF = B-cell activating factor of the tumor necrosis factor family; IRF5 = interferon regulatory factor 5; PDC = plasmacytoid dendritic cell; STAT4 = signal transducer and activator of transcription 4; TF = transcription factors; TLR = toll-like receptors.

CHAPTER 268  Sjögren Syndrome  

B- and T-cell activation, and secretion of autoantibodies, especially in predisposed patients. These autoantibodies constitute immune complexes that participate in the maintenance of IFN-α production. Altogether, these steps promote a vicious cycle of immune system activation leading to tissue damage.

CLINICAL MANIFESTATIONS

Glandular

Decreased salivary secretion results in mouth dryness and increased incidence of oral infections, mucosal friability, and dental caries due to loss of the lubricating, buffering, and antimicrobial capacities of saliva.7 Fungal infections (primarily candidiasis) are also common. Parotid salivary gland or other major salivary gland enlargement can also occur. Persistent enlargement should be carefully followed, however, to exclude bacterial superinfection and, more importantly, the development of lymphoma. Decreased lacrimal flow and impaired lacrimal composition lead to damage of the corneal and conjunctival epithelia, a condition known as keratoconjunctivitis sicca. As a result of keratoconjunctivitis sicca, SS patients might experience foreign-body sensation, grittiness, irritation, photosensitivity, and thick rope-like secretions at the inner canthus, all leading to increased discomfort and possibly visual impairment, with considerable functional disability. Furthermore, ocular complications include corneal ulceration and scarring, bacterial keratitis, and eyelid infections that require continuous ophthalmologic care and treatment.

Systemic

In addition to the sicca features, systemic manifestations occur in approximately 20 to 30% of pSS patients. Of note, it has been increasingly appreciated that the extraglandular manifestations in SS can be divided into two major types according to the underlying pathophysiologic mechanism. Thus, lymphocytic infiltration of the epithelia of organs beyond the exocrine glands (e.g., renal, liver, and bronchial epithelial cells) results in interstitial nephritis, autoimmune cholangitis, and obstructive bronchiolitis, respectively. These clinical features seem to appear early and usually have a benign course. On the other hand, immune complex deposition as a result of the ongoing B-cell hyperreactivity can give rise to the extraepithelial manifestations—palpable purpura, glomerulonephritis, interstitial pneumonitis, and peripheral neuropathy—that are linked to increased morbidity and risk for lymphoma development. The main systemic manifestations are listed in Table 268-1. Peripheral neuropathy may occur through various mechanisms. Vasculitis may be present with cryoglobulinemia, leading to both sensory and motor symptoms. More frequently, pure sensory neuropathy is present, sometimes purely ataxic and sometimes in the form of small-fiber neuropathy. This latter entity is difficult to diagnose because clinical and electromyographic examinations are normal. The diagnosis may be made by skin biopsy showing rarefaction of sensory small fibers.

Sjögren Syndrome and Non-Hodgkin Lymphomas

Chronic polyclonal B-cell activation is commonly present in pSS, which may explain why this autoimmune disease has the strongest association with the development of B-cell lymphoma (relative risk, 15 to 20). More recent studies have estimated this risk at a lower level: 6 in Denmark and Sweden, 7 in Taiwan, and 9 in Norway. Lymphomas complicating pSS have specific features (Chapter 185). They are mostly B-cell non-Hodgkin lymphomas with a predominance of lowgrade, marginal-zone histologic type. Mucosal localization is predominant, notably as mucosa-associated lymphoid tissue (MALT) lymphomas. Interestingly, lymphomas often develop in organs where pSS is active, such as salivary glands. In the setting of SS, chronic autoimmune B-cell activation plays the major role in the lymphomagenesis process, and the identified predictors of lymphoma development in pSS are in line with this phenomenon. The main clinical predictors are permanent swelling of salivary glands, splenomegaly, lymphadenopathy, and palpable purpura. The main biological predictors are positivity of rheumatoid factor (RF), cryoglobulinemia, lymphopenia (especially CD4 lymphopenia), low complement levels, and a monoclonal component in serum or urine. Three novel predictive factors for lymphoma development have been recently described: (1) the presence of ectopic germinal centers associated with the occurrence of lymphoma in pSS patients8; (2) demonstration that BAFF levels are increased in pSS patients with current or previous lymphoma compared with patients without lymphoma9; (3) abnormalities of the gene TNFAIP3 coding for the A20 protein that regulates NF-κB activation, found in up to 77% of MALT lymphomas

1787

TABLE 268-1  EXTRAGLANDULAR MANIFESTATIONS OF PRIMARY SJÖGREN SYNDROME CONSTITUTIONAL SYMPTOMS Fatigue Low-grade fever SKIN AND VASCULAR Small vessel vasculitis Raynaud phenomenon Photosensitivity reactions similar to subacute cutaneous systemic lupus erythematosus Xerosis UPPER AND LOWER AIRWAYS Pyogenic sialoadenitis or parotitis Interstitial pneumonitis or fibrosis Chronic bronchitis Bronchiectasis Bronchiolitis obliterans with organizing pneumonia Chronic obstructive pulmonary disease MUSCULOSKELETAL Polyarthralgia, polyarthritis Myopathy, polymyositis RENAL Type I renal tubular acidosis Tubular interstitial nephritis Glomerulonephritis NEUROLOGIC Peripheral motor sensory neuropathy Pure sensory neuropathy (including pure ataxic neuropathy) Small fiber sensitive neuropathy Multiple sclerosis–like focal lesions Spinal cord dysfunction, including transverse myelitis NEOPLASIA Lymphadenopathy, MALT (mucosa-associated lymphoid tissue) lymphoma

complicating pSS.10 In half of the cases, TNFAIP3 mutations or deletions occur within lymphoma cells; in the other 50%, they involve germline TNFAIP3 mutations with functional consequences.

Laboratory Findings

The most common serologic finding in pSS is hypergammaglobulinemia. The elevated γ-globulins contain several autoantibodies directed against non– organ-specific antigens, such as RF and antinuclear antibody (ANA). Specific ANA, anti-SSA/Ro, and anti-SSB/La antibodies are present in 60 to 80% and 30 to 40% of patients, respectively, and anti-SSB/La is never present without anti-SSA/Ro. Of note, the presence of anti-SSA/Ro, possibly with anti-SSB/ La, may mediate complete heart block of newborns owing to cross-mimicry between specific fetal myocardial antigens and epitopes of the SSA/Ro-SSB/ La complex. Anemia of chronic inflammation and high erythrocyte sedimentation rates (due to hypergammaglobulinemia) are frequently encountered, whereas C-reactive protein levels are usually within normal limits. Cytopenias (most frequently lymphopenia and neutropenia) can also occur. In the setting of interstitial nephritis, the presence of hypokalemic, hyperchloremic acidosis might reveal distal renal tubular acidosis. A monoclonal immunoglobulin can be detected in 10 to 15% of patients with SS, depending on the technique used. Approximately 20% of patients with SS have cryoglobulins in their sera. Complement levels may be decreased, especially C4. This low C4 level may be either genetically determined or secondary to consumption (in immune complexes or cryoglobulinemia).

DIAGNOSIS

Differential Diagnosis

The definition of pSS had suffered for a long time from the absence of accurate and consensus-driven diagnostic criteria. This is important because the patients’ main symptoms (dryness, fatigue, and pain) are frequent in the general population. They can be caused by numerous drugs (Table 268-2), anxiety and/or depression, other comorbidities, or aging (Table 268-3). Sarcoidosis can mimic the clinical picture of SS. However, in sarcoidosis minor salivary gland biopsy reveals noncaseating granulomas, and autoantibodies

1788

CHAPTER 268  Sjögren Syndrome  

TABLE 268-2  DRUGS AND TOXINS THAT MIGHT DECREASE LACRIMAL AND SALIVARY SECRETION

TABLE 268-4  CLASSIFICATION CRITERIA FOR SJÖGREN SYNDROME

STRONG EFFECT

I.  OCULAR SYMPTOMS

Atropine, atropinic antiparkinsonian drugs, anticholinergic antihistaminic drugs Antidepressants: imipraminic (amitriptyline) and inhibitors of monoamine oxidase Neuroleptics Morphine, codeine, tramadol A-type botulinum toxin Class IA antiarrhythmic (disopyramide) Isotretinoin Toxins and psychotropic drugs: tobacco, ecstasy, cannabis, cocaine

MODERATE EFFECT β-Adrenergic blockers α-Adrenergic blockers Calcium channel blockers Benzodiazepines Inhibitors of serotonin reuptake (very slight effect) H1 antihistaminic drugs Diuretics Some antiretroviral drugs

TABLE 268-3  THE DIFFERENT CAUSES OF SICCA SYMPTOMS Drugs, particularly psychotropic drugs (see Table 268-2) Aging, postmenopausal estrogen deficiency Prolonged use of contact lenses Fibromyalgia and chronic fatigue syndrome Anxiodepressive syndromes Head and neck radiotherapy Diabetes (uncontrolled) Severe hyperlipidemia Amyloidosis Sarcoidosis Lymphoma Graft-versus-host disease Some viral infections (HIV, HCV, HTVL-1) IgG4-related sialoadenitis Sjögren syndrome HCV = hepatitis C virus; HIV = human immunodeficiency virus; HTVL-1 = human T-lymphocytic virus-1.

are typically absent. Other SS mimickers include chronic graft-versus-host disease, amyloidosis, infection with viruses such as HIV, human T-lymphocytic virus-I (HTLV-I), and hepatitis C virus (HCV), and IgG4-related disease (Chapter 275). The latter disease is important in the differential diagnosis of SS. It more often involves men with salivary or lacrimal gland enlargement (previously called Mickuliz disease) with previous organ-specific autoimmune disease (like autoimmune pancreatitis) without anti-SSA/SSB antibodies. Sicca symptoms without salivary lymphoid infiltrate and without anti-SSA/SSB antibodies may be part of the fibromyalgia syndrome (Chapter 274), and several acronyms have been proposed for designating these patients: sicca asthenia polyalgia syndrome (SAPS) or dry eyes and mouth syndrome (DEMS).

Diagnostic Criteria

International agreement has established a definition of SS based on the American-European Consensus Group (AECG) criteria, which require the presence of either focal lymphocytic infiltrates in minor salivary glands with a focus score of 1 or more, or anti-SSA/SSB autoantibodies (Table 268-4). A new set of preliminary criteria for SS classification was proposed by an expert consensus panel (American College of Rheumatology [ACR]-Sjögren International Collaborative Clinical Alliance [SICCA]). According to these criteria, classification of an individual as a pSS patient requires the presence of two out of three of the following objective items: (1) a positive serum test for anti-Ro/SSA and/or anti-La/SSB antibodies, or positive rheumatoid factor (RF) and antinuclear antibody (ANA) (titer > 1: 320); (2) presence of keratoconjunctivitis sicca, defined by an ocular staining score over 3; and (3) presence of focal lymphocytic sialoadenitis, defined by a focus score of 1 focus/4 mm2 or above in a labial salivary gland biopsy.11

Assessment of Activity of the Disease

An international expert group recently set up an SS activity score under the umbrella of the European League Against Rheumatism (EULAR). Two indices have been developed: (1) a patient-administered questionnaire to assess subjective features, the EULAR Sjögren Syndrome Patient Reported Index (ESSPRI), based on three different visual analogic scores: dryness,

Positive response to at least one of these three questions: 1. Have you had daily, persistent, troublesome dry eyes for more than 3 months? 2. Do you have a recurrent sensation of sand or gravel in the eyes? 3. Do you use tear substitutes more than three times a day? II.  ORAL SYMPTOMS Positive response to at least one of these three questions: 1. Have you had a daily feeling of dry mouth for more than 3 months? 2. Have you had recurrent or persistently swollen salivary glands as an adult? 3. Do you frequently drink liquids to aid in swallowing dry food? III.  OCULAR SIGNS Objective evidence of ocular involvement, defined as a positive result in at least one of the following two tests: 1. Schirmer test (≤5 mm in 5 min) 2. Rose bengal score (≥4 according to van Bijsterveld scoring system) IV.  HISTOPATHOLOGY Focus score ≥1 in a minor salivary gland biopsy specimen (a focus is defined as an agglomerate of at least 50 mononuclear cells; the focus score is defined by the number of foci in 4 mm2 of glandular tissue) V.  SALIVARY GLAND INVOLVEMENT Objective evidence of salivary gland involvement, defined by a positive result in at least one of the following three diagnostic tests: 1. Salivary scintigraphy 2. Parotid sialography 3. Unstimulated salivary flow (≤1.5 mL in 15 min) VI.  AUTOANTIBODIES Presence in the serum of the following autoantibodies: antibodies to Ro (SSA) or La (SSB) antigens, or both RULES FOR CLASSIFICATION In patients without any potentially associated disease, primary Sjögren syndrome is diagnosed if: Four of six criteria are met, including IV or VI; or Three of four criteria from III, IV, V, and VI are met For secondary Sjögren syndrome, criteria I or II plus any two from criteria III, IV, and V should be met EXCLUSION CRITERIA Preexisting lymphoma, AIDS, sarcoidosis, graft-versus-host disease, past head and neck radiation treatment, use of anticholinergic drugs, and hepatitis C From Vitali C, Bombardieri S, Jonsson R, et al. Classification criteria for Sjögren syndrome: a revised version of the European criteria proposed by the American-European Consensus Group. Ann Rheum Dis. 2002;61:554-558.

fatigue, and limb pain12; and (2) a systemic activity index to assess systemic complications, the EULAR Sjögren Syndrome Disease Activity Index (ESSDAI).13 The latter index comprises 12 domains with 3 or 4 levels of activity for each domain. Determination of the threshold of moderate activity as well as the minimal clinically important improvement is in progress, with the objective to base inclusion criteria and primary end-points of future clinical studies on ESSDAI levels.

TREATMENT  Symptomatic Treatment

A recent systematic review of the literature confirms benefits for muscarinic agonists (pilocarpine hydrochloride and more recently cevimeline hydrochloride) for sicca features (oral dryness and, to a lesser extent, ocular dryness). A1  Topical cyclosporine collyrium (0.05%) also was effective for moderate or severe ocular dryness and inflammation in a randomized controlled trial versus placebo, as were 0.1% clobetasone butyrate eyedrops. A2  Environmental measures (avoidance of hot air heating systems or excessive air conditioning, use of a humidifier, appropriate glasses to protect the eye from evaporating air flow) and “little means” (sugar-free chewing gums, regular water drinking, salivary substitutes) might be useful. Regular dental examinations and oral hygiene are crucial for reducing subsequent oral health issues (i.e., caries and periodontal disease associated with xerostomia). To treat pain, simple analgesics should be used first, particularly acetaminophen/ paracetamol, which does not cause dryness.

CHAPTER 269  Inflammatory Myopathies  

269 

Immunomodulatory Drugs

To date, no immunomodulatory drug has proved efficacious in pSS. Severe organ manifestations of pSS have to be treated in accordance with treatment modalities used in SLE or other connective tissue diseases. Randomized trials have assessed hydroxychloroquine in pSS and failed to demonstrate any clinical efficacy. A3  In spite of these negative results on clinical outcomes, hydroxychloroquine is frequently used in pSS, especially to treat arthralgia with or without synovitis or purpura. Controlled studies are needed to assess the use of methotrexate, leflunomide, mycophenolate sodium, azathioprine, and cyclosporine. Intravenous gamma globulin (IVIG) has been used in the treatment of SS-associated sensorimotor neuropathies or non-ataxic sensory neuropathy without any necrotizing vasculitis.

Biologics

Two randomized controlled trials (RCT) of infliximab and etanercept did not show any efficacy of TNF-blocker agents in pSS on a composite primary outcome including limb pain, fatigue, and dryness visual analogue scales (VAS). A4  B-cell targeting appears to be a promising strategy in pSS. Three randomized controlled trials assessed efficacy of the monoclonal anti-CD20 antibody (rituximab). In the first one, a significant improvement from baseline in a fatigue VAS was observed in the rituximab group but not in the placebo group. A5  In the second one, rituximab demonstrated significant efficacy compared to placebo in improving stimulated salivary flow, the primary end-point, but also oral and ocular dryness, fatigue VAS, and systemic complications. A6  In the third study, the composite primary end-point, using 4 VAS, was achieved at 6 weeks but not at 6 months. A7  Lastly, recent data derived from the French Autoimmune and Rituximab (AIR) registry, including 78 pSS patients with mainly systemic manifestations, suggested the efficacy of rituximab on systemic manifestations in approximately two thirds of the patients.14 Overall, rituximab seems to be useful in cases of persistent parotid swelling or systemic complications, especially in cryoglobulinemia-induced vasculitis. Inhibitors of BAFF, especially the anti-BAFF monoclonal antibody belimumab (which is approved in SLE), has been used in pSS in a first open phase 2 study, with promising results.15

FUTURE DIRECTIONS

SS is a model of autoimmune disease, because it can be primary or associated with other autoimmune diseases; it represents autoimmunity where the risk of lymphoma is most important. SS is the autoimmune disease for which the target tissue of autoimmunity is the most easily available, with the lip biopsy being necessary for diagnosis. Recent progress in pathophysiology has emphasized a number of similarities with SLE that support consideration of SS as a sort of lupus of the mucosa. Even if the pathogenetic mechanisms of the disease remain largely unknown, improved knowledge of the effector mechanisms will allow identification of new targets for future therapy. Moreover, with the recently validated composite activity scores of ESSPRI and ESSDAI, the tools are now available to begin new clinical trials with novel drugs for this disease that will improve the poor quality of life currently associated with it.

Grade A References A1. Ramos-Casals M, Tzioufas AG, Stone JH, et al. Treatment of primary Sjögren syndrome: a systematic review. JAMA. 2010;304:452-460. A2. Aragona P, Spinella R, Rania L, et al. Safety and efficacy of 0.1% clobetasone butyrate eyedrops in the treatment of dry eye in Sjogren syndrome. Eur J Ophthalmol. 2013;23:368-376. A3. Gottenberg JE, Ravaud P, Puechal X, et al. Effects of hydroxychloroquine on symptomatic improvement in primary Sjogren syndrome: the JOQUER randomized clinical trial. JAMA. 2014;312: 249-258. A4. Mariette X, Ravaud P, Steinfeld S, et al. Inefficacy of infliximab in primary Sjogren’s syndrome: results of the randomized, controlled Trial of Remicade in Primary Sjogren’s Syndrome (TRIPSS). Arthritis Rheum. 2004;50:1270-1276. A5. Dass S, Bowman SJ, Vital EM, et al. Reduction of fatigue in Sjögren syndrome with rituximab: results of a randomised, double-blind, placebo-controlled pilot study. Ann Rheum Dis. 2008;67: 1541-1544. A6. Meijer JM, Meiners PM, Vissink A, et al. Effectiveness of rituximab treatment in primary Sjögren syndrome: a randomized, double-blind, placebo-controlled trial. Arthritis Rheum. 2010;62: 960-968. A7. Devauchelle-Pensec V, Mariette X, Jousse-Jolin S, et al. Treatment of primary Sjögren syndrome with rituximab: a randomized trial. Ann Intern Med. 2014;160:233-242.

GENERAL REFERENCES For the General References and other additional features, please visit Expert Consult at https://expertconsult.inkling.com.

1789

INFLAMMATORY MYOPATHIES STEVEN A. GREENBERG

OVERVIEW

The inflammatory myopathies are a heterogeneous group of acquired disorders in which the immune system is thought to play a major pathogenic role. Though some genetic disorders affecting muscle also have significant involvement of the immune system and are treated with immunosuppressive therapy as standard of care (e.g., treatment of Duchenne’s muscular dystrophy with corticosteroids), these genetic disorders are not classified as inflammatory myopathies. The four major subtypes of inflammatory myopathy are: dermatomyositis (DM), polymyositis (PM), immune-mediated necrotizing myopathy (IMNM), and inclusion body myositis (IBM; also called sporadic inclusion body myositis [sIBM]). These disorders have distinct clinical and pathologic features and pathophysiologies (Table 269-1). Whereas DM and PM have been described in the medical literature for over 100 years, IMNM and IBM have only become defined as syndromes distinct from PM within the last few decades.

EPIDEMIOLOGY

The prevalence of DM has been estimated at 1001 to 210 per million. The estimated prevalence of PM is confounded by frequent misdiagnosis of IBM and muscular dystrophies as PM. Traditionally, PM has been considered more prevalent (70 per million2), but comparative studies with attention to IBM have found a prevalence of PM of 35 per million, approximately half the prevalence of IBM of 70 per million. The prevalence of IMNM is unknown. DM peaks in prevalence in childhood (7 to 15 years) and in midlife (30 to 50 years), whereas PM peaks in prevalence in midlife. IBM is rarely diagnosed before the age of 40 and is most common after the age of 50. DM and PM have female predominance; IBM has male predominance. Ethnicity and worldwide distribution influence the development of various inflammatory myopathies.

PATHOBIOLOGY

The pathophysiologies of various forms of inflammatory myopathy are poorly understood. These disorders do share in common injury to muscle by the immune system. Much of the theory of pathophysiology of these disorders comes from microscopic examination of muscle biopsies and the distinct pathologies of these disorders (Fig. 269-1). The muscle pathology of DM involves loss of muscle blood vessels and injury to myofibers at the edges of muscle fascicles (i.e., perifascicular atrophy; see Fig. 269-1). The relationship of these two features to each other is uncertain but has been postulated to be due to a primary injury to muscle capillaries, followed by ischemic injury to myofibers. An alternative view is that a common factor injures both myofibers and capillaries.3 Skin pathology shows features analogous to that of muscle, with an interface dermatitis consisting of injury to the basal layer of keratinocytes. Much evidence points toward DM as mediated by the type 1 interferon cytokine family, consisting mainly of interferon (IFN)-α and IFN-β.4,5 Numerous studies of DM skin and muscle samples show marked upregulation of type 1 IFN-inducible transcripts and proteins uniquely in DM among muscle diseases, and similarly to systemic lupus erythematosus among skin diseases. The presence of autoantibodies in some patients with DM, such as antibodies to the type 1 IFN-inducible protein MDA5, is of uncertain significance but seems likely due to an immune reaction to proteins that are not normally expressed at high levels or exposed to the immune system. The paraneoplastic associations of DM suggest that in such patients, an immune reaction against an underlying malignancy results in bystander injury to muscle and skin. Because PM is a diverse group of disorders, the mechanisms involved are likely to be varied. Pathologically, there is an appearance of invasion of muscle fibers by adaptive immune system cells (T cells) that appears to be antigen driven, so that cytotoxic T cell–mediated autoimmunity directed against an unknown target has been a favored hypothesis. The antigens targeted by this process and the fundamental cause are unknown.

CHAPTER 268  Sjögren Syndrome  

GENERAL REFERENCES 1. Qin B, Wang J, Yang Z, et al. Epidemiology of primary Sjogren’s syndrome: a systematic review and meta-analysis. Ann Rheum Dis. 2014; [Epub ahead of print]. 2. Maldini C, Seror R, Fain O, et al. Epidemiology of primary Sjögren syndrome in a French multiracial/ethnic area. Arthritis Care Res (Hoboken). 2013;66:454-463. 3. Nocturne G, Mariette X. Advances in understanding the pathogenesis of primary Sjögren syndrome. Nat Rev Rheumatol. 2013;9:544-556. 4. Rusakiewicz S, Nocturne G, Lazure T, et al. NCR3/NKp30 contributes to pathogenesis in primary Sjogren’s syndrome. Sci Transl Med. 2013;5:195-196. 5. Kivity S, Arango MT, Ehrenfeld M, et al. Infection and autoimmunity in Sjogren’s syndrome: a clinical study and comprehensive review. J Autoimmun. 2014;51:17-22. 6. Lessard CJ, Li H, Adrianto I, et al. Variants at multiple loci implicated in both innate and adaptive immune responses are associated with Sjögren syndrome. Nat Genet. 2013;45:1284-1292. 7. Mavragani CP, Moutsopoulos HM. Sjogren syndrome. CMAJ. 2014;186:E579-E586. 8. Theander E, Vasaitis L, Baecklund E, et al. Lymphoid organisation in labial salivary gland biopsies is a possible predictor for the development of malignant lymphoma in primary Sjögren syndrome. Ann Rheum Dis. 2011;70:1363-1368. 9. Gottenberg JE, Seror R, Miceli-Richard C, et al. Serum levels of beta2-microglobulin and free light chains of immunoglobulins are associated with systemic disease activity in primary Sjögren syndrome. Data at enrollment in the prospective ASSESS cohort. PLoS ONE. 2013;8:e59868.

1789.e1

10. Nocturne G, Boudaoud S, Miceli-Richard C, et al. Germline and somatic genetic variations of TNFAIP3 in lymphoma complicating primary Sjogren’s syndrome. Blood. 2013;122:4068-4076. 11. Shiboski SC, Shiboski CH, Criswell L, et al. American College of Rheumatology classification criteria for Sjögren syndrome: a data-driven, expert consensus approach in the Sjögren International Collaborative Clinical Alliance cohort. Arthritis Care Res (Hoboken). 2012;64:475-487. 12. Seror R, Ravaud P, Mariette X, et al. EULAR Sjogren’s Syndrome Patient Reported Index (ESSPRI): development of a consensus patient index for primary Sjogren’s syndrome. Ann Rheum Dis. 2011;70:968-972. 13. Seror R, Ravaud P, Bowman SJ, et al. EULAR Sjogren’s syndrome disease activity index: development of a consensus systemic disease activity index for primary Sjogren’s syndrome. Ann Rheum Dis. 2010;69:1103-1109. 14. Gottenberg JE, Cinquetti G, Larroche C, et al. Efficacy of rituximab in systemic manifestations of primary Sjogren’s syndrome: results in 78 patients of the AutoImmune and Rituximab registry. Ann Rheum Dis. 2013;72:1026-1031. 15. Mariette X, Seror R, Quartuccio L, et al. Efficacy and safety of belimumab in primary Sjögren syndrome: results of the BELISS open-label phase II study. Ann Rheum Dis. 2015;74:526-531.

1789.e2

CHAPTER 268  Sjögren Syndrome  

REVIEW QUESTIONS 1. A 59-year-old woman is referred to your office because of dry mouth, grittiness of eyes, and a rash on both legs. Physical exam discloses unilateral parotid enlargement and a purpuric rash of the lower extremities. Past medical history is unremarkable to date. Among the following diagnostic tests, which one is the most useful to establish the diagnosis of Sjögren syndrome? A. Complement levels B. Cryoglobulins C. Serum protein electrophoresis D. Salivary flow E. Anti-Ro/SSA antibodies Answer: E  Anti-Ro/SSA positivity is one of the two required 2002 European/American classification criteria for Sjögren syndrome (SS) and one out of the three 2012 preliminary ACR/SICCA 2012 criteria. Low complement levels, especially C4, and cryoglobulinemia have been designated as adverse prognostic factors for lymphoma development and mortality among SS patients. However, they are not included in the classification criteria. Hypergammaglobulinemia or monoclonal gammopathy are likewise not included in the classification criteria of SS. Salivary flow rate of less than 1.5 mL/15 minutes is an objective criterion of oral dryness, included in the 2002 European/American classification SS criteria but not in the 2012 preliminary ACR/SICCA criteria. It is not as specific as anti-Ro/SSA antibodies. 2. A 55-year-old woman presents with mild pain affecting her hands, as well as ocular and mouth dryness for 10 years. Her examination reveals positive ocular staining for both eyes, revealing keratoconjunctivitis sicca, positive anti-Ro/SSA antibodies, and negative HCV and HIV serology. Additional testing included complete blood count, revealing lymphopenia with normal hemoglobin levels and platelet counts. Mixed monoclonal cryoglobulinemia and low C4 levels were also detected. Which of the following statements is true? A. The patient fulfills both the the 2002 American/European Consensus Group (AECG) criteria and 2012 preliminary ACR/SICCA criteria for primary SS (pSS). B. The patient fulfills only the 2002 AECG criteria for pSS. C. A minor salivary gland biopsy is mandatory to classify the patient as having SS according to 2002 AECG group criteria for pSS. D. A minor salivary gland biopsy is mandatory to classify the patient as having SS according to 2012 preliminary ACR/SICCA criteria for pSS. E. The patient suffers from sicca asthenia polyalgia syndrome. Answer: A  Two out of three 2012 preliminary ACR/SICCA criteria are fulfilled, as well as four out of six 2002 AECG criteria. Since anti-Ro/SSA antibodies are present, no histopathologic confirmation is required for the classification of pSS according to 2002 AECG criteria in the presence of subjective and objective symptoms and signs of salivary and lacrimal gland involvement. Sicca asthenia polyalgia syndrome is a diagnosis of exclusion and refers to the presence of fibromyalgia-like features in association with sicca symptomatology.

3. Which of the following is the best option for improving the patient’s ocular dryness? A. Methotrexate B. Pilocarpine C. Hydroxychloroquine D. Ocular cyclosporine drops E. Infliximab Answer: D  Ocular cyclosporine drops have proved useful in the treatment of dry eye. The efficacy of methotrexate on ocular dryness has never been demonstrated. Ocular dryness is less frequently improved than salivary dryness with secretagogues (i.e., pilocarpine). The efficacy of hydroxychloroquine on dryness has not been demonstrated. No efficacy of infliximab has been demonstrated in pSS. 4. Which of the following features is not considered an adverse predictor for lymphoma development in Sjögren syndrome? A. Presence of splenomegaly B. Presence of purpura C. Salivary flow rate less than 1 mL/15 minutes D. Presence of persistent parotid gland enlargement E. Presence of CD4 lymphopenia Answer: C  All the other items are predictive factors of lymphoma. An association has not been reported between salivary flow rate and risk of lymphoma development. 5. A 55-year-old woman is referred to your department for polysynovitis and palpable purpura. She fulfills the AECG 2002 criteria for Sjögren syndrome. Which of the following treatments would you recommend to treat these systemic manifestations? A. Infliximab B. Intravenous immunoglobulin C. Pilocarpine D. Rituximab E. Cyclophosphamide Answer: D  Three randomized controlled trials (RCTs) and one registry study suggested possible efficacy of rituximab in pSS, at least in some systemic manifestations. Two RCTs did not demonstrate any efficacy of tumor necrosis factor blockers (i.e., infliximab) in pSS. IVIG has been proposed in open studies in the treatment of SS-associated neuropathies without vasculitis, but never has demonstrated any efficacy in other systemic manifestations. The secretagogue pilocarpine has no effect on systemic manifestations. The systemic manifestation presented by the patient is not severe enough to propose cyclophosphamide.

1790

CHAPTER 269  Inflammatory Myopathies  

TABLE 269-1  CLASSIFICATION OF INFLAMMATORY MYOPATHIES DISORDER

AGE RANGE

CLINICAL FEATURES

MUSCLE PATHOLOGY

Dermatomyositis

Juvenile and adult forms

Proximal weakness plus skin

Perimysial and perivascular inflammation, perifascicular atrophy

Polymyositis

Adult (rare in childhood)

Proximal weakness

Endomysial inflammation with invasion of non-necrotic muscle fibers

Immune-mediated necrotizing myopathy

Adult

Proximal weakness

Multifocal necrotic muscle fibers

Inclusion body myositis

Adult > 40 years old

Prominent quadriceps and finger flexor weakness; treatment refractory

Endomysial inflammation with invasion of non-necrotic muscle fibers plus rimmed vacuoles

Overlap syndromes

Adult

Myositis plus defined connective tissue disease

Nonspecific inflammation

Other (granulomatous myositis, eosinophilic myositis)

All ages

Proximal or distal weakness

Specific to type (e.g., granulomas present with granulomatous myositis)

Rimmed vacuoles

Inclusion bodies

A

B Necrotizing myopathy

C Dermatomyositis

D Polymyositis

Perimysium

E

Endomysium

F

FIGURE 269-1.  Pathologies of inflammatory myopathies. A and B, Rimmed vacuoles (arrowheads) of inclusion body myositis (IBM). C, Invasion of non-necrotic muscle fiber in IBM. D, Scattered necrotic and regenerating myofibers in immune-mediated necrotizing myopathy. E, Perivascular and perimysial inflammation (arrows), with perifascicular atrophy (arrowheads), in dermatomyositis. F, Endomysial inflammation in polymyositis. With permission from the Inclusion Body Myositis Foundation, Inc.

CHAPTER 269  Inflammatory Myopathies  

IMNM is also a poorly understood disorder.6 It can also be paraneoplastic, suggesting cross-reactions by the immune system with the underlying malignancy and with muscle antigens. More commonly, IMNM occurs in association with treatment with statin drugs. The identification of autoantibodies against the target of statins, 3-hydroxy-3-methylglutaryl-coenzyme A reductase (HMGCR), in the majority of patients who develop IMNM in association with statin use suggests that the upregulation of HMGCR in muscle is directly toxic to muscle and triggers an immune reaction against it. The pathogenesis of IBM is complex. Two dual pathologies have been noted: degeneration of myofibers and of myonuclei in particular, evident as formation of rimmed vacuoles (see Fig. 269-1A and B), and involvement of the immune system.7 The accumulation of more than 75 different proteins into sarcoplasmic aggregates in a small percentage of IBM myofibers has been reported, and has given rise to a number of molecular toxicity hypotheses in which certain specific protein aggregates are theorized as injurious to myofibers. The immune system involvement in IBM is notable in that whereas most other forms of inflammatory myopathy are generally responsive to immunomodulatory treatments, IBM is refractory to treatment. This is particularly remarkable in that IBM has the greatest evidence of all the inflammatory myopathies of a highly refined antigen-driven adaptive immune system involvement. Pathology shows very chronic and often marked but variable inflammatory infiltrates of T cells, myeloid dendritic cells, and plasma cells in muscle. Studies of the T-cell receptors have strongly suggested that T-cell autoimmunity is driven by one or more specific antigens, though the identity of any of these antigens is unknown. Studies of a B-cell pathway in IBM have led to identification of an autoantibody that is highly specific to IBM among muscle diseases. Circulating autoantibodies against a 43-kD muscle protein were reported in 2011, and the identity of this 43-kD protein as cytoplasmic 5′ nucleotidase 1A (cN1A; NT5C1A) was reported in 2013.8,9 cN1A is a nucleotidase that is most abundant in skeletal muscle and involved in the metabolism of nucleic acids. Serum anti-cN1A autoantibodies are present in 50 to 70% of patients with IBM, depending on which assays and what cutoffs are used, and highly specific to IBM (>90 to 95%) among muscle diseases. The role of blood testing for anti-cN1A autoantibodies in the diagnosis and management of patients with suspected IBM is currently being defined, potentially shortening the time to diagnosis, reducing the misdiagnosis rate, and avoiding more invasive muscle biopsy in some patients.

CLINICAL MANIFESTATIONS AND DIAGNOSIS

A diagnosis of inflammatory myopathy is considered when a patient presents with proximal or distal weakness without sensory symptoms, or in patients with the characteristic skin lesions of DM. Less frequently, asymptomatic elevated creatine kinase (CK) levels lead to a diagnosis of inflammatory myopathy. Most patients with DM, PM, or IMNM present with subacute proximal weakness of the arms and legs progressing over months, though these diseases may present acutely. Patients with IBM present later in life, usually symptomatic from slowly progressive weakness of knee extensors and finger flexors. More specific diagnostic considerations for these disorders are considered individually (Table 269-2). Most patients undergo muscle biopsy, or skin biopsy in the case of suspected DM, as part of the diagnostic evaluation.

Dermatomyositis

Patients with DM typically present with characteristic skin lesions or muscle weakness. Virtually pathognomonic skin features are a heliotrope rash, a violaceous periorbital macular erythema, sometimes with edema, and Gottron’s papules, violaceous papules over dorsal metacarpophalangeal and interphalangeal joints of the hands (Fig. 269-2).10 Periungual telangiectasias and thrombosed capillaries, poikiloderma over photoexposed areas such as the upper back (“shawl sign”), non-scarring alopecia, and subcutaneous calcification are other suggestive signs. Prominent pruritus is also a common feature of DM. Muscle weakness in DM is less specific, occurring in a pattern indistinguishable from many other muscle diseases. Useful laboratory studies for the evaluation of suspected DM include serum CK (though CK can be normal or even below typical laboratory lower limits of normal in patients with highly active disease) and DM-associated autoantibody studies (e.g., anti-Jo-1, anti-Mi2, and anti-MDA5). Occasional patients have abnormal serum aldolase but normal serum CK. Skin biopsy showing a cell-poor interface dermatitis supports the diagnosis of DM. Muscle biopsy showing perimysial and perivascular inflammation also

1791

TABLE 269-2  CLINICAL DIAGNOSTIC CRITERIA FOR INFLAMMATORY MYOPATHIES DISORDER

DIAGNOSIS

Dermatomyositis

1. Diagnostic skin involvement (heliotrope rash, Gottron’s papules) OR diagnostic muscle biopsy finding of perifascicular atrophy OR 2. All of the following: • Suggestive skin involvement • Subacute or chronic proximal or distal weakness • Muscle biopsy showing perimysial or perivascular inflammation without features suggesting another disorder (e.g., endomysial inflammation, rimmed vacuoles) OR skin biopsy showing interface dermatitis along with clinical exclusion of lupus erythematosus

Polymyositis

All of the following: 1. Subacute or chronic proximal weakness 2. Elevated serum creatine kinase (CK) 3. Muscle biopsy showing invasion of endomysial inflammation 4. Response to immunotherapy OR appropriate consideration and exclusion of limb-girdle muscular dystrophies and inclusion body myositis

Immune-mediated necrotizing myopathy

Both of the following: 1. Subacute or chronic proximal weakness 2. Muscle biopsy showing necrotizing myopathy, with scattered necrotic or regenerating myofibers and a lack of inflammation other than macrophage invasion of necrotic muscle fiber

Inclusion body myositis

All of the following: 1. Adult > 40 years old 2. Finger flexion or quadriceps weakness 3. Muscle biopsy showing endomysial inflammation OR the presence of serum anti-cN1A autoantibodies 4. Muscle biopsy showing rimmed vacuoles OR invasion of non-necrotic muscle fibers OR the presence of serum anti-cN1A autoantibodies

cNIA = cytoplasmic 5′ nucleotidase 1A.

supports a diagnosis of DM, whereas the presence of perifascicular atrophy in a muscle biopsy is pathognomonic for DM. Because DM is associated with malignancy, appropriate laboratory and radiologic studies should be performed to search for underlying malignancy in all newly diagnosed patients. The most common DM-associated malignancies tend to reflect the overall age and gender cancer rates within the individual patient’s population (i.e., breast, lung, and colorectal cancer in Western countries; nasopharyngeal cancer in Asian populations). This observation supports the notion of DM as a paraneoplastic process that can develop in virtually any kind of cancer. Research diagnostic criteria for DM have been defined.11 Clinical diagnostic criteria are outlined in Table 269-2. The clinical features of muscle weakness in DM are entirely nonspecific, with no particular pattern indicative of DM rather than other muscle diseases. In practice, certain dermatologic clinical findings (heliotrope rash, Gottron’s papules) or muscle biopsy findings (perifascicular atrophy) are considered nearly pathognomonic for DM.

Polymyositis

The diagnosis of PM is often problematic, with historically many patients with genetically defined limb-girdle muscular dystrophies and IBM being misdiagnosed as PM. The 1975 criteria for PM that are frequently cited allow for a diagnosis of “definite” PM without a muscle biopsy. Other research criteria require muscle biopsy. In clinical practice, the core criteria for the diagnosis of PM are subacute proximal weakness, elevated serum CK, and muscle biopsy showing endomysial inflammation without features suggestive of another diagnosis such as IBM (see Table 269-2). Patients with defined connective tissue disorders such as Sjögren’s syndrome or mixed connective tissue disease have “overlap syndromes,” often also classified as PM. Patients with IMNM have historically been classified as PM but are increasingly classified separately. Patients with IBM are frequently misdiagnosed as PM because of a lack of appreciation of characteristic IBM finger flexor weakness, and because muscle biopsies show endomysial inflammation. The presence of autoantibodies such as anti-Jo-1 argue more for PM than IBM, though these may be seen in DM as well.

1792

CHAPTER 269  Inflammatory Myopathies  

A

B

C

D

FIGURE 269-2.  Clinical findings in dermatomyositis. A, Erythematous to violaceous raised papules overlying the metacarpal and interphalangeal joints, known as Gottron’s papules. These are considered the hallmark finding in dermatomyositis. B, Cuticular overgrowth and periungual capillary changes, which include dilated and tortuous blood vessels with areas of atrophy, telangiectasia, vessel dropout, and bushy loop formation along the fingernail bed. C, Erythema and minimal edema involving the upper eyelids, with occasional telangiectasia, known as the heliotrope rash. D, Subcutaneous calcification erupting through skin (arrowhead), seen clinically and by x-ray.

Immune-Mediated Necrotizing Myopathy

IMNM has increasingly been separated from the PM category. Acute or subacute proximal weakness indistinguishable from that of PM or DM and an elevated CK are nonspecific, but muscle biopsy showing scattered necrotic or regenerating myofibers without inflammation other than macrophages invading these necrotic myofibers is typical of IMNM. The presence of antiHMGCR (3-hydroxy-3-methylglutaryl-coenzyme A reductase) or anti-SRP (signal recognition particle) antibodies both suggest IMNM. IMNM, particularly when associated with anti-SRP antibodies, may be paraneoplastic, and laboratory and radiologic evaluation for malignancy should be considered.

Inclusion Body Myositis

IBM has a clinical presentation distinct from other inflammatory myopathies.12 IBM weakness is always slowly progressive rather than the acute or subacute weakness more typically seen in other forms of inflammatory myopathies. Clinical diagnostic criteria are shown in Table 269-2. IBM has a high misdiagnosis rate, estimated at approximately 50% of patients. Symptoms of IBM rarely are present before the age of 40 years and most commonly occur after the age of 50. The distribution of weakness is usually in finger flexors or quadriceps rather than proximal arms (shoulder abduction) or proximal legs (hip flexion), more typical of PM or DM. IBM is a highly atrophying muscle disease, and loss of bulk in medial and lateral anterior thighs and ventral forearms is characteristic. Patients present with difficulty walking, buckling of knees, or weakness of grip. The diagnosis of IBM can be highly suspected in such patients of appropriate age and findings on examination of quadriceps atrophy and weakness of finger flexors, especially flexor digitorum profundi, responsible for flexion of distal fingertips. Examination of the strength in these distal fingertips, which needs to be done one finger at a time, is often the single most helpful approach to the diagnosis of IBM. Serum CK is either normal or modestly elevated (typically < 5 times the upper limit of normal). A serum autoantibody, anti-cN1A (also called antiNT5C1A), appears highly specific to IBM among muscle diseases and may be of diagnostic value. Most patients undergo muscle biopsy, with characteristic features being the presence of rimmed vacuoles seen on hematoxylin and eosin (H&E) and Gomori trichrome staining, along with endomysial inflammation or invasion of non-necrotic muscle fibers. Immunohistochemical stains detecting p62 or TDP-43 are of additional highly specific diagnostic value.

TREATMENT  Generally, most patients with DM, PM, and IMNM respond to immunomodulatory therapies, whereas patients with IBM are almost universally refractory. A general approach to treatment is shown in Figure 269-3.

Treatment of Dermatomyositis and Polymyositis

Most patients with DM and PM are treated with and respond to corticosteroids.13,14 Dosing is typically prednisone at 1 mg/kg/day orally until significant improvement occurs (typically 1 to 3 months), followed by gradual taper of 10 mg/day/month. Second-line agents include methotrexate, azathioprine, cyclosporine, and intravenous immunoglobulin. Second-line agents are used for two reasons: they may have a better side-effect profile than chronic higher doses of corticosteroids, and they may be necessary for patients whose responses are insufficient to corticosteroids alone. An important decision is whether to start second-line agents concurrently with initial corticosteroid treatment or wait and see how low a dose of corticosteroids offers satisfactory control, and then add agents only if the corticosteroid dose cannot be lowered sufficiently. Thus, in the former approach, prednisone 60 mg/day and methotrexate 7.5 mg PO weekly might be started concurrently, and the methotrexate dose increased weekly to 15 to 20 mg PO weekly. Once improvement  is substantial, the dose of prednisone may be tapered over 3 to 6 months. Stability on methotrexate alone would then be followed by gradual reduction in its dose. For patients with severe initial presentations, the combination of corticosteroids and periodic intravenous immunoglobulin (1 g/kg every 2 weeks) may offer a better chance for more rapid improvement. Approximately seven randomized placebo controlled trials have been reported to date in DM or PM. A1  These studies have almost always used the Bohan and Peter criteria for the diagnosis, which may result in inclusion of patients with limb-girdle muscular dystrophies and IBM misdiagnosed as having PM. The largest trial, rituximab in myositis (RIM), enrolling 200 subjects, A2  used a trial design in which all subjects received active drug, but comparator groups were treated “early” or “late” (8 weeks later), and efficacy was based on time to improvement. This study found no significant differences between these early and late treatment groups’ time to improvement, as defined by a specific definition of improvement (DOI) used in that study.

Treatment of Inclusion Body Myositis

A number of controlled and uncontrolled trials of IBM have been  published. None of these demonstrated efficacies of the therapeutic 

CHAPTER 270  The Systemic Vasculitides  

1793

Suspected Inflammatory Myopathy Typical proximal > distal weakness

Finger flexor and quadriceps weakness

Suspect DM or PM or necrotizing myopathy

Suspect IBM Muscle biopsy

Muscle biopsy (+/– skin biopsy)

Muscle weakness

DM skin only

Confirmed IBM

Not-confirmed IBM +/– treatment

Therapy Topical agents; hydroxychloroquine

Prednisone 1 mg/kg/day x 1-3 months; slow taper or Methylprednisolone 500-1000 mg IV/day × 3-5 days, followed by oral prednisone

No medical treatment

Consider 2nd-line agent initially, or wait to see if relapse after taper

2nd-line agents: Methotrexate 7.5 mg PO weekly, increasing by 2.5 mg weekly to 20 mg PO weekly as needed; with folic acid 1 mg daily except on day of methotrexate or IVIG 0.4 g/kg/day × 5 days each month or 1 g/kg/day every 2 weeks or Azathioprine 1-3 mg/kg/day

Refractory—consider IBM

FIGURE 269-3.  Approach to treatment of suspected inflammatory myopathy. DM = dermatomyositis; IBM = inclusion body myositis; IVIG = intravenous immunoglobulin; PM = polymyositis. With permission from the Inclusion Body Myositis Foundation, Inc.

interventions, which have included prednisone, intravenous immunoglobulin, methotrexate, antithymocyte globulin, oxandrolone, interferon-β, and alemtuzumab.15 Current management of patients with IBM is supportive, involving avoidance of falls and the use of ankle supports and gait assistive devices. Tendon transfer to improve hand function has been used.

270  THE SYSTEMIC VASCULITIDES JOHN H. STONE

PROGNOSIS

Most patients with adult DM, PM, and statin-associated IMNM have a good prognosis but require long-standing immunomodulatory therapy. Many patients with juvenile DM may go into long-standing remission or cure with aggressive initial treatment. Patients with anti-SRP–associated IMNM may have severe and difficult-to-treat disease. Patients with IBM generally have a slowly progressive course, with one series showing a mean time to loss of ambulation of 12 years.

Grade A References A1. Gordon PA, Winer JB, Hoogendijk JE, et  al. Immunosuppressant and immunomodulatory treatment for dermatomyositis and polymyositis. Cochrane Database Syst Rev. 2012;8:CD003643. A2. Oddis CV, Reed AM, Aggarwal R, et al. Rituximab in the treatment of refractory adult and juvenile dermatomyositis and adult polymyositis: a randomized, placebo-phase trial. Arthritis Rheum. 2013;65:314-324.

GENERAL REFERENCES For the General References and other additional features, please visit Expert Consult at https://expertconsult.inkling.com.

DEFINITION

The vasculitides are a heterogeneous group of disorders linked by the common finding of destructive inflammation within blood vessel walls. The most current nomenclature scheme1 identifies at least 27 different forms of primary vasculitis (Table 270-1). The major forms of vasculitis are discussed in this chapter.

CLASSIFICATION

Classification by Vessel Size

The etiology of most forms of vasculitis remains unknown, and major gaps exist in our understanding of the pathophysiologic processes. The most valid basis for classification of the vasculitides is the size of the predominant blood vessels involved. The vasculitides are categorized initially by whether the vessels affected are primarily large, medium, or small (Table 270-2). Large vessels are considered the aorta, its primary branches, and any vessel that is not located within an organ such as a muscle, kidney, nerve, or the skin. Medium-sized vessels, in contrast, consist of the main visceral arteries and their branches. (Thus, the renal artery is considered a large vessel, but its intrarenal branches—the interlobar and arcuate arteries—are medium-sized vessels). Finally, small vessels include smaller intraparenchymal arteries as well as arterioles, capillaries, and veins. Medium-vessel vasculitis and even large-vessel vasculitis can also affect small arteries. However, Large-vessel vasculitis affects large arteries more often than medium or small-vessel vasculitis, medium-vessel vasculitis affects predominantly medium arteries, and small-vessel vasculitis affects predominantly small arteries and other small vessels.

Additional Considerations in Classification

Several considerations other than blood vessel size are relevant to the classification of vasculitis (see Table 270-2). These are (1) age, sex, and ethnic

CHAPTER 269  Inflammatory Myopathies  

GENERAL REFERENCES 1. Rosa J, Garrot LF, Navarta DA, et al. Incidence and prevalence of polymyositis and dermatomyositis in a health management organization in Buenos Aires. J Clin Rheumatol. 2013;19:303-307. 2. Meyer A, Meyer N, Schaeffer M, et al. Incidence and prevalence of inflammatory myopathies: a systematic review. Rheumatology (Oxford). 2015;54:50-63. 3. Greenberg SA. Sustained autoimmune mechanisms in dermatomyositis. J Pathol. 2014;233: 215-216. 4. Suárez-Calvet X, Gallardo E, Nogales-Gadea G, et al. Altered RIG-I/DDX58-mediated innate immunity in dermatomyositis. J Pathol. 2014;233:258-268. 5. Greenberg SA. Dermatomyositis and type 1 interferons. Curr Rheum Rep. 2010;12:198-203. 6. Mohassel P, Mammen AL. Statin-associated autoimmune myopathy and anti-HMGCR autoantibodies. Muscle Nerve. 2013;48:477-483. 7. Greenberg SA. Inclusion body myositis. Curr Opin Rheumatol. 2011;23:574-578.

1793.e1

8. Larman BH, Salajegheh M, Nazareno R, et al. Cytosolic 5′-nucleotidase 1A autoimmunity in sporadic inclusion body myositis. Ann Neurol. 2013;73:408-418. 9. Pluk H, van Hoeve BJ, van Dooren SH, et al. Autoantibodies to cytosolic 5′-nucleotidase 1A in inclusion body myositis. Ann Neurol. 2013;73:397-407. 10. Bailey EE, Fiorentino DF. Amyopathic dermatomyositis: definitions, diagnosis, and management. Curr Rheumatol Rep. 2014;16:465. 11. Hornung T, Wenzel J. Innate immune-response mechanisms in dermatomyositis: an update on pathogenesis, diagnosis and treatment. Drugs. 2014;74:981-998. 12. Dimachkie MM, Barohn RJ. Inclusion body myositis. Neurol Clin. 2014;32:629-646. 13. Lam C, Vleugels RA. Management of cutaneous dermatomyositis. Dermatol Ther. 2012;25: 112-134. 14. Lundberg I, Vencovsky J, Alexanderson H. Therapy of myositis: biological and physical. Curr Opin Rheumatol. 2014;26:704-711. 15. Greenberg SA. Pathogenesis and therapy of inclusion body myositis. Curr Opin Neurol. 2012;25: 630-639.

1793.e2

CHAPTER 269  Inflammatory Myopathies  

REVIEW QUESTIONS 1. A 65-year-old man developed slowly progressive difficulty arising from a chair and experienced buckling of the knees while walking, resulting in several falls. Muscle biopsy showed endomysial inflammation and rimmed vacuoles. The correct diagnosis is: A. Polymyositis B. Dermatomyositis C. Inclusion body myositis D. Immune-mediated necrotizing myopathy E. Overlap syndrome Answer: C  Inclusion body myositis (IBM). IBM is a later-onset, slowly progressive disease presenting with prominent quadriceps or finger flexion weakness. In this case, the knee buckling indicates quadriceps weakness. Endomysial inflammation can be seen in either polymyositis or IBM, but rimmed vacuoles confirm IBM as the correct diagnosis. 2. A 35-year-old woman developed proximal weakness and a purplish papular rash over the dorsum of the hands. A skin biopsy might be expected to show what feature and lead to which diagnosis: A. Lymphocytoclastic vasculitis AND polymyositis B. Interface dermatitis AND dermatomyositis C. Granulomas AND granulomatous myositis D. Necrotic cells AND necrotizing myopathy E. Necrotic cells AND dermatomyositis Answer: B  This patient has Gottron’s papules (see Fig. 296-2A), virtually pathognomonic for dermatomyositis. Skin biopsy in dermatomyositis shows an interface dermatitis, with pathology of the basal layer of keratinocytes lying at the border (interface) between the epidermis and the dermis. 3. A 60-year-old man with proximal weakness and mildly elevated serum creatine kinase (CK) is diagnosed with polymyositis and treated with high-dose prednisone 80 mg/day for several months, before slow taper of prednisone to 30 mg/day by month 6. His serum CK improves, but no improvement in strength is present and he appears mildly weaker. Which statement is most likely true? A. He has refractory polymyositis and needs more aggressive therapy. B. He has developed steroid myopathy in addition to his polymyositis. C. The diagnosis of polymyositis should be reconsidered, and he could undergo a second muscle biopsy or blood diagnostic testing for anticN1A autoantibodies as the next step. D. The diagnosis of polymyositis should be reconsidered; he may have inclusion body myositis or a limb-girdle muscular dystrophy. E. C and D Answer: E  Patients with IBM in particular, and sometimes limb-girdle muscular dystrophy, are often misdiagnosed as having polymyositis. Serum CK, modestly elevated in IBM, may lower with prednisone treatment, but patients rarely have improved strength.

4. Which syndrome can be paraneoplastic and should prompt thorough investigation for an underlying malignancy? A. Dermatomyositis B. Polymyositis C. Inclusion body myositis D. Granulomatous myositis E. All of the above Answer: A  Dermatomyositis. A new diagnosis of dermatomyositis should prompt a thorough investigation for an underlying malignancy, with reported rates estimated at 15 to 23%.12 The types of malignancy found in patients with dermatomyositis generally reflect those found in age-, sex-, and population-matched persons without dermatomyositis. Therefore, breast, lung, and colorectal cancer are the three most common cancers from Western country cohorts, while nasopharyngeal carcinoma is the most common dermatomyositis-associated cancer in Asian studies. This finding further supports the role of dermatomyositis as a paraneoplastic process that can occur with virtually any kind of cancer. (Ungprasert P, Bethina NK, Jones CH. Malignancy and idiopathic inflammatory myopathies. N Am J Med Sci. 2013;5:569-572.) 5. Normal serum CK in a patient with proximal muscle weakness excludes the diagnosis of: A. Dermatomyositis B. Inclusion body myositis C. Muscular dystrophy D. Granulomatous myositis E. None of the above Answer: E  None of the above. Serum CK is a helpful indicator of muscle disease when elevated, but normal CK does not exclude muscle disease. In particular, patients with very active dermatomyositis frequently have normal serum CK.

1794

CHAPTER 270  The Systemic Vasculitides  

background of the patient; (2) tropism for particular organs; (3) presence or absence of granulomatous inflammation; (4) participation of immune complexes in the pathophysiologic process; and (5) detection of characteristic autoantibodies in the patients’ serum, such as antineutrophil cytoplasmic antibodies (ANCAs). Age, sex differences, and ethnic variation are discussed later in the section on Epidemiology. The organ tropisms of these disorders are illustrated by the following examples. Whereas immunoglobulin (Ig)A vasculitis (IgAV, also known as Henoch-Schönlein purpura) typically affects the skin, joints, kidneys, and gastrointestinal (GI) tract, granulomatosis with polyangiitis

TABLE 270-1  NAMES FOR VASCULITIDES ADOPTED BY THE 2012 INTERNATIONAL CHAPEL HILL CONSENSUS CONFERENCE ON THE NOMENCLATURE OF VASCULITIDES.1 LARGE-VESSEL VASCULITIS Takayasu arteritis Giant cell arteritis

(GPA; formerly Wegener granulomatosis) classically involves the upper airways, lungs, and kidneys. In contrast to both IgAV and GPA, Cogan syndrome involves the eyes, the audiovestibular apparatus of the inner ear, and (in 10 to 15% of cases) the large arteries. The presence or absence of granulomatous inflammation is a crucial element of vasculitis diagnosis and classification. Granulomatous inflammation implicates a small number of vasculitides that bear this hallmark, including GPA, giant cell arteritis, Takayasu arteritis, and eosinophilic granulomatosis with polyangiitis (EGPA; Churg-Strauss syndrome). Immune complexes are essential to the pathophysiologic mechanism of some forms of small- and medium-vessel vasculitis. Complexes of IgA1, for example, are found in IgAV. Immune complexes consisting of IgG, IgM, complement components, and the hepatitis C virion characterize most cases of mixed cryoglobulinemia. In contrast, “pauci-immune” types of small- and medium-vessel vasculitis, such as GPA and microscopic polyangiitis, have little immunoglobulin or complement deposition within diseased tissues. Many but not all patients with pauci-immune forms of vasculitis are ANCA positive.

EPIDEMIOLOGY

The epidemiologic features of individual forms of systemic vasculitis vary tremendously by geography (Table 270-3). This may reflect genetic influences, variation in environmental exposures, and other unknown disease risk factors. For example, whereas Behçet syndrome is rare in North Americans, affecting only 1 person in approximately 300,000, this condition is several hundred times more common among inhabitants of countries bordering the ancient Silk Route. Similarly, although Takayasu arteritis is rare in the United States—on the order of 3 new cases per million people per year—this disease is reportedly the most common cause of renal artery stenosis in India, where the incidence may be as high as 200 to 300 per million per year. Age is an important consideration in the epidemiology of vasculitis. Eighty percent of patients with Kawasaki disease are younger than 5 years. In contrast, giant cell arteritis virtually never occurs in patients younger than 50 years, and the mean age of patients with this disease is 72. Age may also have an impact on disease severity and outcome. In IgAV, the overwhelming majority of cases in children (who represent 90% of all cases) have selflimited courses, resolving within several weeks. In adults, however, IgAV has a higher likelihood of chronicity and a poor renal outcome. The distribution of sex varies across many forms of vasculitis. Buerger disease is the only form of vasculitis with a striking male predominance. The greater prevalence of smoking among males in most societies probably explains this predilection. In contrast, Takayasu arteritis has an overwhelming

MEDIUM-VESSEL VASCULITIS Polyarteritis nodosa Kawasaki disease Buerger disease* SMALL-VESSEL VASCULITIS Antineutrophil cytoplasmic antibody (ANCA)-associated vasculitis Microscopic polyangiitis Granulomatosis with polyangiitis (formerly Wegener granulomatosis) Eosinophilic granulomatosis with polyangiitis (Churg-Strauss syndrome) Immune complex small-vessel vasculitis Antiglomerular basement membrane disease Cryoglobulinemic vasculitis Immunoglobulin (Ig)A vasculitis (Henoch-Schönlein purpura) Hypocomplementemic urticarial vasculitis VARIABLE-VESSEL VASCULITIS Behçet syndrome Cogan syndrome SINGLE-ORGAN VASCULITIS Cutaneous leukocytoclastic angiitis Cutaneous arteritis Primary central nervous system vasculitis Isolated aortitis VASCULITIS ASSOCIATED WITH SYSTEMIC DISEASE Lupus vasculitis Rheumatoid vasculitis Sarcoid vasculitis Others (e.g., IgG4-related aortitis)

TABLE 270-2  CONSIDERATIONS IN THE CLASSIFICATION OF SYSTEMIC VASCULITIS

VASCULITIS ASSOCIATED WITH PROBABLE ETIOLOGY Hepatitis C virus–associated cryoglobulinemic vasculitis Hepatitis B virus–associated vasculitis Syphilis-associated aortitis Drug-associated immune complex vasculitis Drug-associated ANCA-associated vasculitis Cancer-associated vasculitis Others *Buerger disease (thromboangiitis obliterans) is not always considered to be a primary form of vasculitis and was not included in this consensus statement on nomenclature.1

Size of predominant blood vessels affected Epidemiologic features: Age Sex Ethnic background Pattern of organ involvement Pathologic features: Granulomatous inflammation Immune complex deposition vs. “pauci-immune” histopathology Presence of ANCA in serum ANCA = antineutrophil cytoplasmic antibody.

TABLE 270-3  EPIDEMIOLOGY OF SELECTED VASCULITIDES DISEASE

UNITED STATES

ELSEWHERE

AGE, SEX, AND ETHNIC PREDISPOSITIONS

Giant cell arteritis

Incidence: 240/million (Olmsted County, MN)

220-270/million (Scandinavian countries)

Age > 50 yr, mean age 72 yr; females 3 : 1; northern European ancestry

Takayasu arteritis

Incidence: 3/million

200-300/million (India)

Age < 40 yr; females 9 : 1; Asian

Behçet syndrome

Prevalence: 3/million

3000/million (Turkey)

Silk Route countries

Polyarteritis nodosa

Incidence: 7/million

7/million (Spain)

Slight male predominance

Kawasaki disease

Incidence: 100/million*

900/million ( Japan)

Children of Asian ancestry

Wegener granulomatosis

Incidence: 4/million

8.5/million (United Kingdom)

Whites ≫ blacks

*Among children younger than 5 years. From Gonzalez-Gay MA, Garica-Porrua C. Epidemiology of the vasculitides. Rheum Clin North Am. 2001;27:729-749.

1795

CHAPTER 270  The Systemic Vasculitides  

tendency to occur in females (a 9 : 1 female-to-male ratio). The pauci-immune forms of vasculitis, such as GPA, EGPA, and microscopic polyangiitis, occur in males and females with approximately equal frequencies, but the phenotypic expression of these conditions may be affected by both age and sex. The strongest link between any single gene and vasculitis is the association of HLA-B51 with Behçet syndrome. In Behçet syndrome, 80% of Asian patients have the HLA-B51 gene. The prevalence of HLA-B51 is significantly higher among patients with Behçet syndrome in Japan than among nondisease control subjects (55% versus < 15%). Among the sporadic cases of Behçet syndrome involving whites in the United States, however, HLA-B51 occurs in fewer than 15% of cases. With the exception of Buerger disease and smoking, no definitive associations have been confirmed between disease and environmental or occupational exposures. Associations have been reported but not confirmed between exposures to silica and some types of pauci-immune vasculitis. Studies of potential associations between exposures of any type and vasculitis, however, are complicated frequently by difficulties in obtaining reliable measurements of the levels of the relevant exposure, the likelihood of recall bias among patients who are diagnosed with vasculitis, and the choice of appropriate control groups.

PATHOBIOLOGY

Table 270-4 illustrates the pathologic characteristics of selected forms of vasculitis. Specific pathologic features are discussed in the subsections on each disease. The type of inflammatory cell infiltrate in vasculitis is independent of the size of blood vessels involved. Mixed cell infiltrates in vasculitis are the rule rather than the exception, and histopathologic patterns of vasculitis may include leukocytoclasis (degranulation and destruction of neutrophils within blood vessel walls), granulomatous findings (with or without giant cells), lymphoplasmacytic infiltrates, varying degrees of eosinophilic infiltration, necrosis, and combinations of all these findings.

PATHOPHYSIOLOGY

Some pathophysiologic mechanisms are common to many different forms of vasculitis, regardless of the size of the predominant blood vessels involved. Immune complex deposition, for example, is present in several types of vasculitis that involve both medium-sized and small blood vessels. In this section, the general concepts related to the pathogenesis of large-vessel vasculitides are discussed separately from those of medium- and small-vessel vasculitides.

Large-Vessel Vasculitides

The pathologic process in large-vessel vasculitis appears to begin in the adventitia. In both Takayasu arteritis and giant cell arteritis, abundant numbers of activated T lymphocytes are found within inflamed arterial walls, centering on the adventitia. In Takayasu arteritis, most of these T cells appear to be of the CD8+ subtype. Current evidence suggests that the cytotoxic functions of these cells, mediated by perforin and granzyme B, contribute to smooth muscle cell damage in this disease. CD4+ T-cell responses in Takayasu arteritis have not been well defined. In giant cell arteritis (Chapter 271), much evidence now suggests an antigen-driven disease, with the site of immunologic recognition events being

the adventitia. CD4+ T cells that secrete interferon (IFN)-γ appear to be recruited to the adventitia by a specific antigen(s), the identity of which remains unknown. Both the T cells that orchestrate the transmural inflammation and the inciting antigens are theorized to reach the adventitia through the vasa vasorum. Subsequently, T-cell signals from the adventitia stimulate macrophages and multinucleated giant cells to elaborate an array of downstream mediators, including metalloproteinases and platelet-derived growth factor. Interleukin (IL)-6, known to be a crucial cytokine in giant cell arteritis and probably Takayasu arteritis as well, is produced by macrophages residing in the blood vessel wall. The results of this inflammatory cascade are granulomatous inflammation, destruction of the internal elastic lamina, arterial wall hyperplasia, smooth muscle cell proliferation, intimal thickening, vascular occlusion, and in some cases, weakening of the vessel wall, leading to dilation and aneurysm formation.

Medium- and Small-Vessel Vasculitides

Several different pathophysiologic mechanisms are operative among the medium- and small-vessel vasculitides. In many cases, the mechanisms outlined in the following sections overlap.

Immune Complex–Mediated Vascular Injury

Immune complex–mediated tissue injury does not produce a single clinical syndrome but rather applies to many forms of vasculitis and overlaps with injuries caused by other immune mechanisms. Numerous variables influence immune complex–mediated injury, including the physical properties of the immune complexes (e.g., their size), the ability of the immune complexes to activate complement, the antigen-to-antibody ratio, and the hemodynamic features of specific vascular beds. Immune complexes participate in the pathophysiologic process of some forms of both medium- and small-vessel vasculitis, including polyarteritis nodosa, cryoglobulinemia, IgAV, cutaneous leukocytoclastic angiitis, and rheumatoid vasculitis.

Role of Antineutrophil Cytoplasmic Antibodies

ANCAs are directed against antigens that reside within the primary granules of neutrophils and monocytes. Two types of ANCA are relevant to vasculitis: (1) those directed against proteinase 3 (PR3), known as PR3-ANCA; and (2) those directed against myeloperoxidase (MPO), termed MPO-ANCA. ANCA interact with cytokines, neutrophils, monocytes, and other elements of the immune system to amplify ongoing inflammation in certain forms of vasculitis. A striking and still unexplained feature of ANCA-associated vasculitis (AAV) is that patients with primary forms of these conditions virtually never have antibodies to both PR3 and MPO. Despite the specificity of these antibodies, however, evidence for a primary role of ANCA in the etiology of human disease is still absent. In GPA, abnormal cytokine regulation interacts with the production of ANCA to fuel the inflammatory response. TH1 cytokines such as IFN-γ, IL-12, and tumor necrosis factor (TNF) appear to play important roles. Under the direction of IL-12, CD4+ T cells from patients with GPA produce elevated levels of TNF, and peripheral blood mononuclear cells secrete increased amounts of IFN-γ. Serum levels of soluble receptors for TNF are elevated in patients with active GPA and normalize with the induction of remission. In vitro priming of activated neutrophils with TNF markedly

TABLE 270-4  PATHOLOGIC CHARACTERISTICS OF SELECTED FORMS OF VASCULITIS TAKAYASU ARTERITIS

POLYARTERITIS NODOSA

GRANULOMATOSIS WITH POLYANGIITIS (WEGENER GRANULOMATOSIS)

CHURG-STRAUSS SYNDROME

HENOCHSCHÖNLEIN PURPURA

CUTANEOUS LEUKOCYTOCLASTIC ANGIITIS

Vessels involved

Elastic (large) or muscle (mediumsized) arteries

Small arteries and veins; Medium-sized and small muscle arteries sometimes mediumsized vessels

Small arteries and veins; sometimes medium-sized vessels

Capillaries, venules, and arterioles

Capillaries, venules, and arterioles

Organ involvement

Aorta, aortic arch and Skin, peripheral nerves, Upper respiratory tract, major branches, gastrointestinal tract, lungs, kidneys, skin, and pulmonary and other viscera eyes arteries

Upper respiratory tract, lungs, heart, peripheral nerves

Skin, joints, gastrointestinal tract, kidneys

Skin, joints

Type of vasculitis and Granulomatous with inflammatory cells some giant cells; fibrosis in chronic stages

Necrotizing, with mixed cellular infiltrate

Necrotizing or Leukocytoclastic, with Leukocytoclastic, with Necrotizing or occasional granulomatous (or granulomatous (or some lymphocytes both); prominent and variable eosinophils both); mixed cellular infiltrate plus occasional eosinophils and eosinophils; IgA eosinophils other mixed deposits in affected infiltrate tissues

1796

CHAPTER 270  The Systemic Vasculitides  

enhances the ability of ANCA to stimulate neutrophil degranulation. Despite the strong rationale for anti-TNF strategies in GPA, however, a randomized trial of etanercept showed no efficacy in the maintenance of disease remissions. B-cell depletion is a more effective approach to the treatment of AAV. The efficacy of this treatment strategy probably relates to the removal of several B-cell functions beyond their evolution into plasma cells and the production of ANCA. These other B-cell functions include cytokine production, antigen presentation, and B cell–T cell crosstalk.

Superantigen Model

The degree of immune activation in Kawasaki disease and the acute but generally self-limited nature of this illness imply a potential role for superantigens. Superantigens are proteins produced by microbial pathogens (e.g., Staphylococcus aureus or Streptococcus species) that are capable of stimulating large populations of T cells in a manner unrestricted by the class II major histocompatibility complex (MHC). Superantigens bind directly to conserved amino acid residues outside the antigen-binding groove on class II MHC molecules, thereby selectively stimulating T cells that express particular β-chain variable gene segments. Through the binding of this MHCsuperantigen complex to its cognate T-cell receptors, as many as 20% of circulating lymphocytes may become activated, leading to a potentially enormous outpouring of cytokines. With regard to the etiology of Kawasaki disease, substantial attention has focused on toxic shock syndrome toxin 1, an exotoxin produced by S. aureus.2 Superantigens have also been postulated to play roles in the susceptibility to disease flares in GPA. Nasal carriage of S. aureus and superantigens associated with these organisms has been linked to a greater likelihood of disease flares in some studies.

Anti–Endothelial Cell Antibodies

Anti–endothelial cell antibodies can induce endothelial cell injury and lysis through either complement-mediated cytotoxicity or antibody-dependent cellular cytotoxicity. Both of these mechanisms have been demonstrated to cause endothelial injury in in vitro assays employing sera from patients with systemic vasculitis. The ability of these antibodies to damage endothelial cells is an appealing argument for their potential role in forms of vasculitis in which the endothelium is the focus of the inflammation (as opposed to the more external vessel wall layers). However, the true relevance of anti– endothelial cell antibodies to human disease and their importance within the larger context of other disease mechanisms remain unclear.

CLINICAL MANIFESTATIONS

Large-Vessel Vasculitides Takayasu Arteritis

Takayasu arteritis (Chapter 78) affects the aorta and its major branches. In contrast to atherosclerosis, which is characterized by focal irregular lesions, the lesions of Takayasu arteritis are long, smooth, tapered stenoses (E-Fig. 270-1). The most commonly involved arteries are the subclavian and innominate arteries. Takayasu arteritis has been termed “pulseless disease” because of its ability to obliterate peripheral pulses (particularly in the upper extremities). Exuberant collateral circulation develops over time in response to the gradual narrowing of major arteries, making the loss of digits or limbs from ischemia extremely rare. The pulmonary circulation is involved in approximately 50% of cases of Takayasu arteritis. Patients with severe narrowing of the aortic arch vessels supplying the head may develop Takayasu retinopathy, a hypotensive retinopathy leading to neovascularization. In contrast, patients with prolonged hypertension associated with renal artery stenosis demonstrate the classic ocular features of hypertension: “copper wiring” and multiple retinal infarctions. This complication is particularly difficult to diagnose and dangerous because vascular narrowings of large arteries to the arms and legs can cause underestimations of the true central aortic pressure. Takayasu arteritis involvement of the ascending aorta may lead to aortic dilation, aortic regurgitation, aneurysm formation, and aortic rupture.

TREATMENT  The cornerstone of treatment of Takayasu arteritis is glucocorticoids. For patients with marked symptoms and signs of an inflammatory phase, prednisone (1 mg/kg/day) is usually effective in controlling the disease. This dose should be tapered within 8 to 12 weeks to less than 20 mg/day and ultimately

to less than 10 mg/day as a maintenance dose. Emerging data support a role for IL-6 inhibition in patients with Takayasu arteritis whose prednisone doses cannot be tapered to reasonable levels. Patients have been treated with tocilizumab 8 mg/kg administered intravenously each month or with a corresponding subcutaneous preparation.

Giant Cell Arteritis

Giant cell arteritis is the other primary form of vasculitis that involves arteries far larger than vasculitides of any other category. This disease is discussed in detail elsewhere (Chapter 271).

Medium-Vessel Vasculitides Polyarteritis Nodosa

Polyarteritis nodosa has a striking predilection for certain organs, particularly the skin, peripheral nerves, GI tract, and kidneys.3 This disease usually begins with nonspecific symptoms such as malaise, fatigue, fever, myalgias, and arthralgias. Overt signs of vasculitis may not occur until weeks or months after onset of the first symptoms. Skin lesions of polyarteritis nodosa include livedo reticularis, subcutaneous nodules, ulcers, and digital gangrene. A majority of patients with polyarteritis nodosa (>80% in some series) have vasculitic neuropathy, typically in the pattern of a mononeuritis multiplex. The classic GI manifestation of polyarteritis nodosa is “intestinal angina,” the occurrence of postprandial abdominal pain. Polyarteritis nodosa can also affect individual GI tract organs such as the gallbladder or appendix, presenting as cholecystitis or appendicitis. The typical renal manifestation of polyarteritis nodosa is vasculitic involvement of the medium-sized intrarenal arteries, leading to renin-mediated hypertension and renal infarctions. Cardiac lesions, which usually remain subclinical, may lead to myocardial infarction or congestive heart failure. Polyarteritis nodosa usually spares the lungs. The diagnosis of polyarteritis nodosa requires either a tissue biopsy or an angiogram that demonstrates microaneurysms (Fig. 270-1). Simultaneous nerve and muscle biopsies (e.g., sural nerve and gastrocnemius muscle) are of high yield if there is a clinical suspicion of vasculitic neuropathy. Symptoms suggestive of a neuropathy can be confirmed by electrodiagnostic studies that demonstrate a sensorimotor axonal neuropathy, often in a mononeuritis multiplex pattern. The pathologic changes in polyarteritis nodosa are limited to the arterial circulation, and the lesions are segmental, favoring the branch points of arteries. In gross pathologic specimens, aneurysmal bulges of the arterial wall may be visible. Histologic sections reveal infiltration and destruction of the blood vessel wall by inflammatory cells, accompanied by fibrinoid necrosis. Granulomatous inflammation is absent.

TREATMENT  Approximately half of patients with polyarteritis nodosa achieve remissions or cures with high doses of glucocorticoids alone. Cyclophosphamide (2 mg/ kg/day, adjusted for renal dysfunction) is indicated for patients whose disease is refractory to glucocorticoids or who have serious involvement of major organs. In recent years, therapeutic regimens involving lamivudine or entecavir and plasma exchange have substantially improved the treatment of hepatitis B virus (HBV)-associated polyarteritis nodosa. Because of increasing use of the HBV vaccine, fewer than 10% of polyarteritis nodosa cases now are associated with HBV infections.

Kawasaki Disease

Kawasaki disease occurs exclusively in young children. Because of its striking mucocutaneous findings and lymphadenopathy, Kawasaki disease is also known as mucocutaneous lymph node syndrome. Features of Kawasaki disease include high fevers, cervical adenopathy, conjunctival congestion, buccal erythema, prominence of the tongue papillae (“strawberry tongue”), a polymorphous truncal rash, erythema of the palms and soles, and desquamation of skin from the fingertips occurring days to weeks into the illness.4 In its acuity and severity, Kawasaki disease resembles toxic shock syndrome and scarlet fever, both of which are mediated by superantigens (see Pathophysiology). In a small number of patients with Kawasaki disease, panvasculitis in the coronary vessels leads to acute cardiac complications. Coronary arteritis leads to narrowing of the vessel lumen by the migration of myointimal cells

CHAPTER 270  The Systemic Vasculitides  

A

1796.e1

C

B E-FIGURE 270-1.  Large-vessel disease in Takayasu arteritis. Long, smooth tapering in the left common femoral artery (A) and the right renal artery (B). Dilation of the ascending aorta (C). Aortic regurgitation necessitated an aortic valve replacement and replacement of the ascending aorta with a Gore-Tex graft.

CHAPTER 270  The Systemic Vasculitides  

1797

A

FIGURE 270-2.  Cutaneous small-vessel vasculitis showing palpable purpuric lesions with necrosis and crusting.

TREATMENT  B FIGURE 270-1.  Vasculitis of medium-sized arteries in polyarteritis nodosa. A, Mesenteric angiogram showing numerous aneurysms in medium-sized arteries. B, Fibrinoid necrosis (arrows) in a jejunal artery from a patient who required surgical resection of necrotic bowel.

from the media through the fragmented internal elastic lamina. Direct complications include aneurysmal dilation and thrombosis of the coronary arteries, leading to myocardial infarction and possibly to death (in 1 to 2% of patients with Kawasaki disease during the acute illness). Late mortality from myocardial infarction may occur from the thrombosis of coronary artery aneurysms formed during the initial inflammatory stage. Such myocardial infarctions have been reported in middle-aged individuals who had febrile illnesses consistent with Kawasaki disease in childhood.

TREATMENT  The recommended therapeutic regimen in Kawasaki disease is the combination of intravenous immune globulin (IVIG; 400 mg/kg/day on 4 consecutive days) and acetylsalicylic acid (100 mg/kg/day, lowered to 3 to 5 mg/kg/day after resolution of the fever). IVIG prevents the formation of coronary aneurysms in most cases. Glucocorticoids are reserved for salvage therapy in patients whose treatment with IVIG and acetylsalicylic acid has failed. A1 

Buerger Disease

Buerger disease, also known as thromboangiitis obliterans (Chapter 80), is not always considered to be a primary form of vasculitis and was not included in the most recent consensus statement on nomenclature. Buerger disease has a remarkably strong yet poorly understood association with cigarette smoking and simply does not occur in the absence of exposure to tobacco. The vessels affected by Buerger disease are the distal medium-sized arteries and veins, particularly vessels at the levels of the ankles and wrists. The disease is characterized by thrombotic obliterations that begin distally and proceed proximally. Buerger disease tends to be segmental in nature, involving 5- to 10-cm lengths of blood vessels. Arterial obliteration leads to the development of collateral vessels with a “corkscrew” appearance on angiography. Vascular occlusion in Buerger disease often leads to the loss of digits and, if smoking persists, to loss of larger amounts of tissue (e.g., hands or feet). Despite the intense involvement of the extremities in Buerger disease, internal organ disease almost never occurs.

Complete abstinence from tobacco is essential to the treatment of Buerger disease. Failure to stop smoking is associated with a dramatic increase in the risk of limb loss by amputation. No other therapeutic interventions, including glucocorticoids and anticoagulation, have dramatic effects on Buerger disease.

Small-Vessel Vasculitides Antineutrophil Cytoplasmic Antibody–Associated Vasculitides Granulomatosis with Polyangiitis

Classic GPA (formerly Wegener granulomatosis) involves the upper respiratory tract, lungs, and kidneys.5 Distinctive features may also occur in the eyes, ears, and other organs. The three pathology hallmarks of GPA are (1) granulomatous inflammation in the upper or lower respiratory tract, (2) necrotizing vasculitis affecting arteries or veins, and (3) segmental glomerulonephritis associated with necrosis and thrombosis of capillary loops, with or without granulomatous lesions. Approximately 90% of patients with GPA have nasal involvement, including crusting, bleeding, and obstruction. Cartilaginous inflammation may lead to nasal septal perforation and collapse of the nasal bridge (“saddle nose” deformity). Erosive sinus disease and subglottic stenosis (narrowing of the trachea just below the vocal cords) are highly characteristic of GPA. Both conductive and sensorineural hearing loss can occur in GPA, though conductive lesions caused by middle ear disease are more common. Orbital masses (“pseudotumors” that develop behind the eye), scleritis, and peripheral ulcerative keratitis are the most dangerous ocular lesions. Episcleritis and conjunctivitis also occur. Anterior uveitis is rare. The clinical manifestations of GPA in the lung range from asymptomatic nodules to fulminant alveolar hemorrhage. The most common radiographic findings are pulmonary infiltrates, nodules, and cavitary lesions. Large-airway disease leading to bronchial narrowing is a challenging diagnosis to make because patients present with few symptoms until advanced disease is present. The clinical presentation of renal disease in GPA is usually rapidly progressive glomerulonephritis: hematuria, red blood cell casts, and proteinuria (usually non-nephrotic). Without appropriate therapy, end-stage renal disease may ensue within weeks. Sixty percent of patients with GPA have musculoskeletal symptoms during their disease course. The presenting complaint is frequently arthralgias or an oligoarthritis that is migratory in nature. Skin lesions in GPA include the full panoply of lesions associated with cutaneous vasculitis, including purpura (Fig. 270-2). Cutaneous nodules over the extensor surfaces of joints, particularly the elbow, may mimic rheumatoid nodules. These lesions are known as cutaneous extravascular necrotizing granulomata or Churg-Strauss lesions. Meningeal inflammation, presenting with headaches, cranial neuropathies, and a clinical picture compatible with chronic meningitis, is perhaps the most

1798

CHAPTER 270  The Systemic Vasculitides  

common central nervous system (CNS) manifestation of GPA. Mononeuritis multiplex may affect the peripheral nervous system. GPA is the prototype of conditions associated with ANCA. Positive results of immunofluorescence tests for ANCA in either the cytoplasmic (C-ANCA) or perinuclear (P-ANCA) pattern should be confirmed by enzyme immunoassays for antibodies to either proteinase 3 (PR3) or myeloperoxidase (MPO). An ANCA-negative assay sample does not exclude GPA, because a substantial minority of patients (between 15 and 40% overall) lack these antibodies. Furthermore, ANCA titers do not correlate reliably with disease activity.

TREATMENT  Manifestations of GPA that constitute immediate threats either to the function of a vital organ or to the patient’s life require treatment urgently. From the late 1960s until 2010, the combination of cyclophosphamide (2 mg/kg orally daily) and high doses of glucocorticoids (prednisone 1 mg/kg orally daily, tapered during 6 to 12 months) was the standard of care for GPA. Intermittent administration of cyclophosphamide by IV infusion is also effective in remission induction. However, a multicenter clinical trial that compared rituximab to cyclophosphamide in patients with either GPA or microscopic polyangiitis demonstrated that rituximab (375 mg/m2 weekly times four) is at least as effective as the conventional regimen. A2  A3  Rituximab appears to be more effective for AAV patients who present with disease flares. An alternative dosing regimen of rituximab, 1 g times two separated by 2 weeks, may also be effective. Limited forms of GPA may respond to the combination of methotrexate (up to 25 mg/week) and glucocorticoids, but rituximab is now often employed in this setting as well. Rituximab (e.g., 500 mg every 6 months) is more effective than azathioprine for maintaining remission in patients who demonstrate a tendency to flare. A4  ,

Microscopic Polyangiitis

Microscopic polyangiitis is characterized by (1) nongranulomatous necrotizing vasculitis with few or no immune deposits, (2) involvement of small (and possibly medium-sized) blood vessels in the arterial or venous circulation, and (3) tropism for the kidneys and lungs. Many cases of small-vessel vasculitis once regarded as polyarteritis nodosa are now classified more properly as microscopic polyangiitis. In contrast to polyarteritis nodosa, an ANCAnegative disorder, 70% of microscopic polyangiitis patients are ANCA positive.6 Thus, microscopic polyangiitis is considered to be a form of AAV. The ANCAs in microscopic polyangiitis are usually directed against myeloperoxidase, leading to a perinuclear pattern of staining on immunofluorescence testing (P-ANCA). Microscopic polyangiitis is not characterized by granulomatous inflammation, and upper respiratory tract symptoms, if present at all, are much milder than those associated with GPA.

TREATMENT  The approach to the treatment of microscopic polyangiitis is similar to the treatment of GPA. The combination of rituximab and glucocorticoids is the treatment regimen of choice for most patients with microscopic polyangiitis.

Eosinophilic Granulomatosis with Polyangiitis

EGPA is an eosinophil-rich form of granulomatous inflammation that involves the respiratory tract and other organs. The disease is associated with necrotizing vasculitis of small to medium-sized vessels. Two hallmarks of EGPA are asthma and eosinophilia. Several phases of EGPA are described: • A prodromal phase characterized by the presence of allergic disease (typically asthma or allergic rhinitis), which may last months to many years • An eosinophilia–tissue infiltration phase in which remarkably high peripheral eosinophilia may occur and tissue infiltration by eosinophils is observed in the lung, GI tract, and other tissues • A vasculitic phase in which systemic necrotizing vasculitis afflicts a wide range of organs, ranging from the heart and lungs to peripheral nerves and skin

TREATMENT  Patients with mild disease may be treated with prednisone. Those with evidence of neurologic, cardiac, renal, or GI involvement should be treated with cyclophosphamide in addition to glucocorticoids. Although clinical remissions are obtained in more than 90% of patients with EGPA, disease recurrences are seen in 25%. In most cases, relapses are heralded by the return of eosinophilia. Approximately 50% of cases of EGPA are associated with

ANCA, usually directed against myeloperoxidase, but the percentage may be higher among untreated patients.

Immune Complex–Mediated Vasculitides

Anti–Glomerular Basement Membrane Disease

Anti–glomerular basement membrane (anti-GBM) disease is vasculitis affecting glomerular capillaries, pulmonary capillaries, or both, accompanied by the deposition of anti–basement membrane autoantibodies within basement membranes. Anti-GBM disease is discussed in detail elsewhere (Chapter 121). Immunoglobulin A Vasculitis/Henoch-Schönlein Purpura

IgA vasculitis (IgAV) is characterized by non-thrombocytopenic purpura, arthritis, abdominal pain, and glomerulonephritis. The histopathologic findings are those of a leukocytoclastic vasculitis with IgA deposition. IgAV can develop at any age, but 80 to 90% of the cases occur in children. Although the cause is unknown, the disease’s seasonal variation and the fact that two thirds of patients with IgAV experience antecedent acute upper respiratory illnesses suggest an infectious trigger. Medications such as antibiotics can also trigger IgAV, and environmental triggers are also likely. The diagnosis of IgAV can be confirmed only by demonstration of IgA deposition within and around blood vessel walls. The classic IgAV patient presents with the acute onset of fever, palpable purpura on the lower extremities and buttocks, abdominal pain, arthritis, and hematuria. The clinician must be alert to the possibility of IgAV even when only parts of the syndrome are present. Most patients with IgAV, especially children, have a self-limited disease that lasts an average of 4 weeks.

TREATMENT  Glucocorticoids ameliorate the GI, joint, and skin symptoms in many cases, but some patients respond surprisingly poorly to conventional doses of glucocorticoids, even in doses on the order of 40 to 60 mg/day. Anecdotal evidence suggests that pulse glucocorticoids (e.g., methylprednisolone 500 to 1000 mg/day times three doses) may abort persistent bouts of IgAV. The efficacy of glucocorticoids in the glomerulonephritis associated with this condition is controversial. Uncontrolled studies suggest that methylprednisolone pulses (1 g/day for three doses), followed by oral prednisone combined with azathioprine or mycophenolate mofetil may be useful in severe glomerulonephritis associated with IgAV.

Hypocomplementemic Urticarial Vasculitis

At least three subtypes of urticarial vasculitis are known: (1) normocomplementemic, a form that is generally idiopathic and benign (which may be viewed as a manifestation of cutaneous leukocytoclastic angiitis); (2) hypocomplementemic, a form that is often associated with a systemic inflammatory disease; and (3) hypocomplementemic urticarial vasculitis syndrome (HUVS), a potentially severe condition usually associated with autoantibodies to the collagen-like region of C1q. Most patients with the hypocomplementemic subtype have an underlying systemic disorder, such as systemic lupus erythematosus (Chapter 266) or Sjögren syndrome (Chapter 268). Many HUVS patients have C1q “precipitins,” IgG autoantibodies to the collagen-like region of C1q that trigger the classical pathway of complement activation. The role of anti-C1q antibodies in disease pathogenesis remains unclear. The lesions of urticarial vasculitis must be distinguished from the far more common chronic idiopathic urticaria (Chapters 252 and 440). Unlike idiopathic urticaria, the lesions of urticarial vasculitis last more than 48 hours, often have a purpuric component (i.e., they do not blanch), and resolve with postinflammatory hyperpigmentation. In urticarial vasculitis, lesions associated with vasculitis are often accompanied by stinging or burning. Urticarial vasculitis affects the capillaries and postcapillary venules, showing leukocytoclastic vasculitis on light microscopy. Direct immunofluorescence studies reveal both immunoglobulin and complement deposition in or around blood vessels of the upper dermis or the dermoepidermal junction.

TREATMENT  Patients with urticarial vasculitis whose serum complement levels remain normal during attacks often have self-limited disease and require little therapy.

CHAPTER 270  The Systemic Vasculitides  

Other cases, especially HUVS, may cause life-threatening involvement of the lungs or other organs and require periods of intensive immunosuppression. Treatment decisions in HUVS must be individualized according to the patient’s clinical status.

Cryoglobulinemia

Cryoglobulins are antibodies that precipitate from serum under conditions of cold and resolubilize on rewarming. Cryoglobulins are classified into types I, II, and III on the basis of whether monoclonality and rheumatoid factor activity (the ability to bind to the Fc portion of IgG) are present.7 Type I cryoglobulins, which are monoclonal but lack rheumatoid factor activity, are associated with certain hematopoietic malignant neoplasms (e.g., multiple myeloma) and often lead to hyperviscosity rather than to vasculitis (Chapter 187). In contrast, type II and type III cryoglobulins may be associated with systemic vasculitis involving small (and often medium-sized) blood vessels. Vasculitis results from the deposition of cryoglobulin-containing immune complexes within blood vessel walls and the activation of complement. Cryoglobulin types II and III are termed mixed cryoglobulins because they consist of complexes of both IgG and IgM antibodies. The IgM components in both type II and type III cryoglobulinemia possess rheumatoid factor activity (i.e., assays for rheumatoid factor are positive, indicating binding of the IgM antibody to the Fc portion of IgG). Whereas the IgM component in type II cryoglobulin is monoclonal, the IgM in type III cryoglobulin is polyclonal. Ninety percent of patients with vasculitis secondary to mixed cryoglobulins are hypocomplementemic, with C4 levels characteristically more depressed than C3. Infection with hepatitis C virus (HCV) accounts for at least 80% of the vasculitis cases associated with mixed cryoglobulins.8

TREATMENT  Rapidly progressive sensorineural hearing loss requires early and requires aggressive therapy with high doses of systemic glucocorticoids. Some otolaryngologists also perform intratympanic injections of glucocorticoids. Cytotoxic agents can be considered for patients with suboptimal responses to glucocorticoids who still have salvageable hearing. Many Cogan syndrome patients become candidates for cochlear implants.

Variable-Vessel Vasculitides

The variable-vessel vasculitides have no predominant type of vessel involved but rather can affect vessels of any size (small, medium, and large) and any type (arteries, veins, and capillaries).

Cogan Syndrome

The combination of inflammatory eye disease and vestibuloauditory dysfunction is the sine qua non of Cogan syndrome.9 In addition to inflammatory disease of the eyes and ears, up to 15% of patients with Cogan syndrome have vasculitis involving medium-sized to large blood vessels. Although the ocular manifestations vary, the classic presentation is the combination of interstitial keratitis and sensorineural hearing loss. Cogan syndrome may appear first in either the eyes or the ears. Although intervals as long as 1 to 2 years have been described between the start of disease in one organ and the appearance of disease in the other, the time between disease manifestations in these organs is usually only a matter of months. Patients usually present with photophobia and blurry vision, sometimes accompanied simultaneously by auditory or vestibular dysfunction. The vascular disease in Cogan syndrome resembles that of Takayasu arteritis.

TREATMENT  The optimal therapy for most cases of cryoglobulinemic vasculitis is successful treatment of the underlying HCV infection. For cryoglobulinemic patients with relatively mild disease (e.g., frequent purpuric lesions, shallow cutaneous ulcers), short courses of prednisone followed by the institution of effective therapy for HCV may be sufficient. For patients with severe cutaneous ulcers, mononeuritis multiplex, glomerulonephritis, or other manifestations of severe disease, glucocorticoids, rituximab, and possibly a short course of plasma exchange may be indicated.

Behçet Syndrome

Behçet syndrome may affect small, medium, and large vessels in either the venous or the arterial circulation.10 The most typical lesions in Behçet

1799

syndrome are mucocutaneous, reflecting the involvement of small blood vessels. The triad of recurrent mouth ulcers, genital ulcers, and eye inflammation is the classic presentation. The criteria for diagnosis of the International Study Group for Behçet Syndrome consist of one required manifestation— recurrent oral ulceration—plus at least two of the following: recurrent genital ulceration, characteristic eye or skin lesions, or a pathergy reaction (see later). However, the spectrum of Behçet syndrome encompasses many manifestations not included in these criteria. Large-vessel complications of Behçet syndrome may include aneurysms in the pulmonary and systemic arterial systems. Venous complications include thromboses of the deep venous system, vena cava, portohepatic vein, and cerebral sinus. Pathergy—the development of pustules at the sites of sterile needle pricks—is a distinctive feature in many patients with Behçet syndrome, particularly those of Turkish origin. The arthritis of Behçet syndrome is a nondeforming, oligoarticular, asymmetrical arthritis of large joints. GI lesions in Behçet syndrome typically consist of ulcerations of the distal ileum or cecum. Crohn disease, which can cause genital ulcers as well as GI tract disease, may be particularly difficult to distinguish from Behçet syndrome.

TREATMENT  Low-dose glucocorticoids are effective for intransigent mucocutaneous disease and may have a better side-effect profile than other medications used for this purpose (e.g., thalidomide). Intermittent courses of glucocorticoids during periods of particular mucocutaneous disease activity may be sufficient for patients with mild disease. Severe disease in any organ system almost always requires high doses of prednisone (e.g., 1 mg/kg/day). Azathioprine (2 mg/kg/day), cyclosporine (3 to 5 mg/kg/day in two divided doses), methotrexate (up to 25 mg/week), and interferon alpha (3 million to 5 million units three times a week) are appropriate therapies for many complications of Behçet’s syndrome. TNF inhibition with infliximab (5 mg/kg IV every 4 to 6 weeks) or adalimumab (40 mg every other week) is the treatment of choice for patients with the most severe forms of uveitis or meningoencephalitis.

Selected Single-Organ Vasculitides

Single-organ vasculitis is defined as vasculitis within the vessels of any type or size of a single organ, in the absence of any features (e.g., ANCA) suggesting one of the systemic forms of vasculitis.

Cutaneous Leukocytoclastic Angiitis

Cutaneous leukocytoclastic angiitis has also been termed hypersensitivity vasculitis. Cutaneous leukocytoclastic angiitis is the preferred name because no hypersensitivity or allergy is evident in many cases. Histories of exposure to new medications or to infections may be elicited. An immune complex deposition is central to the pathophysiologic process. Although it is occasionally associated with synovitis, other signs of systemic involvement are absent. The skin lesions in cutaneous leukocytoclastic angiitis occur in “crops,” coinciding with some period of elapsed time following exposure to the inciting antigen. The usual time between the exposure and the onset of clinically evident vasculitis is 10 to 14 days. The lesions typically occur first in dependent regions, such as on the lower extremities or buttocks. The rash may be asymptomatic but is usually accompanied by burning or tingling sensations.

TREATMENT  Keys to the management of cutaneous leukocytoclastic angiitis include (1) exclusion of any underlying form of vasculitis that may cause subclinical involvement of other organs and (2) removal of any agent (e.g., a medication) that may have triggered the vasculitis. For patients in whom a precipitant can be identified, removal of the offending agent usually leads to resolution of the vasculitis within days to weeks. The type, intensity, and duration of therapy for cutaneous leukocytoclastic angiitis are based on the degree of disease severity. Mild cases may be treated simply with leg elevation, H1 antihistamines, or low-dose prednisone. For persistent disease not associated with cutaneous gangrene, colchicine, hydroxychloroquine, or dapsone may be tried. For severe cases, high doses of glucocorticoids are indicated to suppress inflammation quickly and prevent skin ulceration.

Vasculitis of the Central Nervous System

CNS vasculitis includes two major categories of disease, one of which is not a true vasculitis. These conditions are primary angiitis of the CNS (PACNS)

1800

CHAPTER 270  The Systemic Vasculitides  

TABLE 270-5  PRIMARY ANGIITIS OF THE CENTRAL NERVOUS SYSTEM (PACNS) VERSUS REVERSIBLE CEREBRAL VASOCONSTRICTION SYNDROME (RCVS) PACNS

RCVS

Female-to-male ratio

1 : 1

2-3 : 1

Onset

Subacute (weeks to months)

Sudden (seconds to minutes)

Headache

Insidious, dull

Thunderclap

Typical lumbar puncture findings

Abnormal in 50-80%: lymphocytic pleocytosis; elevated protein

Normal

Typical MRI findings

Multifocal subacute infarctions

Normal Watershed infarcts in minority

Typical angiogram findings

Multifocal stenoses/dilatations Normal in up to 40% of cases. Abnormal angiographic features when present cannot be distinguished from RCVS

Utility of brain biopsy Reasonable sensitivity in appropriately selected patients Important for excluding disease mimickers

Little to no role Helpful if confusing clinical situation confounds differentiation from PACNS or PACNS mimickers

MRI = magnetic resonance imaging.

and reversible cerebral vasoconstriction syndrome (RCVS). The diagnosis and management of these two conditions differ dramatically. The clinical, radiologic, and pathologic characteristics of PACNS and RCVS are shown in Table 270-5. Primary Angiitis of the Central Nervous System

PACNS11 typically develops in a subacute fashion, with the evolution of multifocal strokes, encephalopathy, headache, and other clinical features over months. Headache is often the first symptom. As the condition progresses, most patients develop lethargy, confusion, and memory loss. Some patients develop multifocal strokes, seizures, evidence of increased intracranial pressure, or myelopathy. The results of routine laboratory tests (e.g., erythrocyte sedimentation rate) are often normal in PACNS. Lumbar puncture demonstrates abnormalities of the cerebrospinal fluid in approximately 80% of cases, usually a modest monocytosis and elevated protein. Lumbar punctures should be performed in all patients in whom the diagnosis of PACNS is considered seriously. Although the findings on lumbar puncture in PACNS patients are nonspecific, a normal lumbar puncture argues against PACNS, and the procedure frequently identifies important PACNS mimickers such as infection or malignancy. Magnetic resonance imaging (MRI) is the critical imaging modality in PACNS. Because of the subacute nature of the disorder, MRI studies reveal multifocal CNS infarctions in most cases. Strokes, hemorrhagic lesions, and mass lesions typically occur in more than one vascular territory. A normal brain MRI argues strongly against the diagnosis of PACNS. Angiography is less helpful in the evaluation of patients with PACNS for two main reasons. First, the sizes of blood vessels involved in PACNS are often too small to be resolved adequately, even by conventional angiography. The false-negative rate of angiography in PACNS is on the order of 35%. Second, the “classic” string-of-beads abnormality on angiography, produced by segmental arterial narrowing alternating with dilations, is nonspecific and can be mimicked perfectly by a host of nonvasculitic conditions (the most common of which is RCVS). No angiographic pattern is pathognomonic for PACNS, and there is a significant tendency to overdiagnose “vasculitis” on angiographic grounds alone. A normal brain MRI in the setting of an abnormal angiogram suggests RCVS, not PACNS.

TREATMENT  When employed in appropriately selected patients whose history and radiologic studies suggest PACNS, brain biopsy is associated with reasonable

positive and negative predictive values and frequently identifies important PACNS mimickers. Prednisone and cyclophosphamide are appropriate for treatment of patients who have abnormal findings on brain biopsy. Treatment courses of 6 to 12 months are recommended.

Reversible Cerebral Vasoconstriction Syndrome

Eighty percent of patients with RCVS12 are women. RCVS is probably far more common than PACNS. Overtreatment of patients with RCVS who are misdiagnosed as having PACNS leads to substantial morbidity. A careful history is the most important part of the evaluation. In contrast to the subacute course that typifies PACNS, RCVS usually begins in a more dramatic fashion with a “thunderclap” headache. Compared with PACNS, the neurologic signs are less severe in RCVS (e.g., encephalopathy is less common). RCVS frequently occurs in the setting of precipitants associated with vasospasm, such as in the postpartum setting or following the use of vasoactive agents such as nasal decongestants and recreational drugs. The lumbar puncture is usually normal in RCVS, and brain MRI usually does not show multifocal CNS infarctions, with the exception of watershed infarctions mentioned earlier. The typical angiographic findings in RCVS— vascular narrowing and beading—are generally indistinguishable from those of PACNS and conditions that mimic PACNS. Multifocal vascular narrowing is particularly characteristic of RCVS. The most distinctive angiographic feature of RCVS is that the abnormalities are completely reversible, usually within 4 to 8 weeks. These abnormalities in RCVS are caused by vasospasm rather than true vasculitis. In the evaluation of patients with potential RCVS, a diagnostic strategy that can clinch the diagnosis is a follow-up angiogram 4 to 8 weeks after the first. Angiographic abnormalities due to RCVS will resolve in this interval.

TREATMENT  Several approaches to the treatment of RCVS are reasonable. First, one may opt for watchful waiting. It is not clear that immunosuppression is either necessary or helpful. Moreover, attempts to treat vasospasm with calcium-channel blockers may lead to a vascular steal phenomenon, potentially causing harm. Second, because it is frequently difficult to do nothing for a patient with possibly serious CNS disease, calcium-channel blockers (e.g., nifedipine 30 mg three times daily) may be tried. Third, because of the frequent diagnostic uncertainty at the time of presentation, some clinicians opt to treat empirically with glucocorticoids (prednisone 1 mg/kg/day) for 1 month, followed by a taper over several weeks. Fourth, combinations of calcium-channel blockers and glucocorticoids are also reasonable, but cytotoxic therapy is not indicated in RCVS.

DIAGNOSIS

Differential Diagnosis

The major categories of diseases that can mimic vasculitis are displayed in Table 270-6. Certain features of a patient’s case should raise the diagnostic suspicion for vasculitis. First, most cases of vasculitis do not begin suddenly but rather unfold subacutely during weeks or months. Second, pain is usually a prominent feature of vasculitis, resulting from arthritis or arthralgias, myalgias, headaches, neuropathy, testicular infarction, digital ischemia, sinusitis, otalgia, back pain (caused by aortic inflammation), postprandial abdominal pain (caused by mesenteric vasculitis), or other disease manifestations. Third, signs of inflammation such as fever, rash, weight loss, and elevated acute phase reactants are highly characteristic. Finally, multiorgan system involvement is the rule in vasculitis. Ideally, the diagnosis of vasculitis is established through biopsy of an involved organ. Diagnoses based on angiography alone have many potential pitfalls, as discussed in the sections on PACNS and RCVS. Angiographic findings that are “consistent with vasculitis” must be interpreted in the proper context. A diverse array of other diseases, ranging from atherosclerosis to vasospasm to pheochromocytoma, may mimic the angiographic appearance of vasculitis. Systemic vasculitis can also be mimicked by two or more common medical problems or treatment complications occurring simultaneously in the same patient. Finally, high on the differential diagnosis of any individual form of vasculitis are other forms of vasculitis. For example, digital ischemia and splinter hemorrhages may be secondary to idiopathic polyarteritis nodosa. They may also be caused by polyarteritis nodosa associated with HBV infection, GPA, EGPA, microscopic polyangiitis, cryoglobulinemia, Buerger disease, or some other form of vasculitis. Because the

CHAPTER 271  Polymyalgia Rheumatica and Temporal Arteritis  

TABLE 270-6  MAJOR DISEASE CATEGORIES IN THE DIFFERENTIAL DIAGNOSIS OF VASCULITIDES Other forms of vasculitis Simultaneous occurrence of common medical problems in the same patient Infections Bacterial, viral, mycobacterial, fungal Occlusive processes Hypercoagulable states Livedoid vasculopathy (atrophie blanche) Atheroembolic disease Malignant neoplasms Lymphoma (including lymphomatoid granulomatosis) Castleman disease Amyloidosis Paraproteinemias Connective tissue disorders Systemic lupus erythematosus, mixed connective tissue disease Systemic sclerosis Rheumatoid arthritis Miscellaneous Atrial myxoma Calciphylaxis Fibromuscular dysplasia Neutrophilic dermatoses Pyoderma gangrenosum Sarcoidosis Reversible cerebral vasoconstriction syndrome

appropriate interventions for these conditions vary widely, careful distinction among these potential etiologies is essential.

TREATMENT  The intensity of treatment in patients with vasculitis must be guided by the degree of disease activity. Specifically, the treatment of vasculitis should be predicated not only on abnormal laboratory test results but also on clear evidence of active disease. In addition, the intensity of treatment must be adapted to the type of vasculitis. Whereas giant cell arteritis responds to high doses of glucocorticoids in essentially all cases, for example, GPA nearly always requires an additional agent (rituximab, cyclophosphamide, or methotrexate) for disease control. In contrast, despite the dramatic fashion in which they sometimes present, most cases of IgA vasculitis and cutaneous leukocytoclastic angiitis require no immunosuppressive treatment at all. Conventional therapies such as glucocorticoids, immunomodulating agents, and cytotoxic drugs induce remissions and control vasculitis in most cases. Moreover, in some cases—a variable percentage, depending on the type of vasculitis—the disease is curable. Unfortunately, the treatments of vasculitis have enormous potential for toxicity. Regular monitoring of patients’ bone marrow, renal, and hepatic function is essential to avoid treatmentinduced toxicity. Prophylaxis against opportunistic infections, particularly Pneumocystis pneumonia (Chapter 341), is an important part of many vasculitis treatment regimens. During the tapering of immunosuppressive medications, disease flares are common in many forms of vasculitis. A common error is treating patients with high doses of immunosuppressive agents for too long. The most appropriate use of medications such as cyclophosphamide and glucocorticoids is to induce remission as quickly as possible with early, aggressive treatment regimens, and then to convert patients to safer treatments for the maintenance of remission. Rituximab is replacing cyclophosphamide as the drug of choice for some forms of vasculitis, particularly AAV. Patients with AAV who demonstrate a tendency to flare are often retreated with rituximab (500 mg or 1 g) every 4 to 6 months, at least until lengthy periods of disease control are established. Current treatment approaches to specific vasculitides are described under their “Clinical Manifestations.”

PROGNOSIS

Assuming that the diagnosis is made before the patient has become catastrophically ill, the prognosis in systemic vasculitis is determined largely by the answers to four questions: 1. Was the diagnosis established before the occurrence of major irreversible organ damage? 2. Was aggressive (but appropriately dosed) treatment begun in a timely fashion?

1801

3. Was there careful monitoring during treatment, and were specific steps taken to avoid drug-induced toxicity (e.g., opportunistic infection)? 4. Were the potentially toxic medications that induced remission stopped at an appropriate juncture and replaced with less dangerous medications (or was treatment stopped altogether)? For most forms of vasculitis, the factors that determine long-term drugfree remissions remain poorly understood. The likelihood of achieving sustained remissions after discontinuation of all medications (or cures) varies according to the specific type of vasculitis.

FUTURE DIRECTIONS

Compelling laboratory and naturally occurring animal models of disease, combined with the known associations among HBV, HCV, and vasculitis in humans, suggest that additional links between infection and systemic vasculitis may be established in the future. Important strides have been made in the description of cytokine and chemokine pathways that are operative in vascular inflammation, but relevant anticytokine interventions remain to be defined for clinical therapies. B-cell depletion is emerging rapidly as the treatment of choice for some forms of severe vasculitis. IL-6 inhibition strategies are also likely to play important roles soon in the large-vessel vasculitides. Additional studies are required to define the full spectrum of clinical utility of these and other biologic agents.

Grade A References A1. Newburger JW, Sleeper LA, McCrindle BW, et al. Randomized trial of pulsed corticosteroid therapy for primary treatment of Kawasaki disease. N Engl J Med. 2007;356:663-675. A2. Stone JH, Merkel PA, Spiera R, et al. Rituximab versus cyclophosphamide for remission induction in ANCA-associated vasculitis. N Engl J Med. 2010;363:221-232. A3. Specks U, Merkel PA, Seo P, et al. Efficacy of remission-induction regimens for ANCA-associated vasculitis. N Engl J Med. 2013;369:417-427. A4. Guillevin L, Pagnoux C, Karras A, et al. Rituximab versus azathioprine for maintenance in ANCAassociated vasculitis. N Engl J Med. 2014;371:1771-1780.

GENERAL REFERENCES For the General References and other additional features, please visit Expert Consult at https://expertconsult.inkling.com.

271  POLYMYALGIA RHEUMATICA AND TEMPORAL ARTERITIS ROBERT F. SPIERA

DEFINITION

Polymyalgia rheumatica (PMR) and temporal arteritis, also called giant cell arteritis (GCA), are companion systemic inflammatory disorders of unknown etiology that represent a spectrum from severe proximal aches and pains and constitutional symptoms to an occlusive granulomatous vasculitis of medium and large vessels that can lead to permanent blindness or other organ and tissue damage. These disorders occur primarily in patients older than 50 years, in women more than in men; they are propagated by antigen-driven, cell-mediated (TH1) immune mechanisms that may be associated with specific genetic markers and are highly responsive to corticosteroids.

EPIDEMIOLOGY

In the United States, the average annual incidence of PMR is 52.5 per 100,000 patients aged 50 years and older and increases with age. The prevalence is about 0.5 to 0.7%. Internationally, the frequency varies according to country, with the highest rates occurring in the Scandinavian countries.1 The incidence and prevalence of GCA are approximately one third those of PMR.

PATHOBIOLOGY

The etiology of PMR and GCA is unknown, but both demonstrate familial aggregation and have a genetic association with human leukocyte antigen (HLA)-DR4 and a demonstrated sequence polymorphism encoded within the hypervariable region of the HLA-DRβ1*04 gene.2 Other genetic

CHAPTER 270  The Systemic Vasculitides  

GENERAL REFERENCES 1. Jennette JC, Falk RJ, Bacon PA, et al. 2012 Revised International Chapel Hill Consensus Conference Nomenclature of Vasculitides. Arthritis Rheum. 2013;65:1-11. 2. Salgado-Pabon W, Case-Cook LC, Schlievert PM. Molecular analysis of staphylococcal superantigens. Methods Mol Biol. 2014;1085:169-185. 3. Forbess L, Bannykh S. Polyarteritis nodosa. Rheum Dis Clin North Am. 2015;41:33-46. 4. Sundel RP. Kawasaki disease. Rheum Dis Clin North Am. 2015;41:63-73. 5. Comarmond C, Cacoub P. Granulomatosis with polyangiitis (Wegener): clinical aspects and treatment. Autoimmun Rev. 2014;13:1121-1125. 6. Lyons PA, Rayner TF, Trivedi S, et al. Genetically distinct subsets within ANCA-associated vasculitis. N Engl J Med. 2012;367:214-223.

1801.e1

7. Damoiseaux J. The diagnosis and classification of the cryoglobulinemic syndrome. Autoimmun Rev. 2014;13:359-362. 8. Dammacco F, Sansonno D. Therapy for hepatitis C virus-related cryoglobulinemic vasculitis. N Engl J Med. 2013;369:1035-1045. 9. Kessel A, Vadasz Z, Toubi E. Cogan syndrome-pathogenesis, clinical variants and treatment approaches. Autoimmun Rev. 2014;13:351-354. 10. Hatemi G, Yazici Y, Yazici H. Behcet’s syndrome. Rheum Dis Clin North Am. 2013;39:245-261. 11. Adams HP Jr. Cerebral vasculitis. Handb Clin Neurol. 2014;119:475-494. 12. Ducros A. Reversible cerebral vasoconstriction syndrome. Handb Clin Neurol. 2014;121: 1725-1741.

1801.e2

CHAPTER 270  The Systemic Vasculitides  

REVIEW QUESTIONS 1. Immunosuppressive therapy is known to be futile for which disease associated with vascular inflammation? A. Buerger disease (thromboangiitis obliterans) B. Granulomatosis with polyangiitis (formerly Wegener granulomatosis) C. IgA vasculitis (Henoch-Schönlein purpura) D. Giant cell arteritis E. Mixed cryoglobulinemia Answer: A  The only effective intervention known for Buerger disease is smoking cessation. Neither immunosuppression nor anticoagulation is of any demonstrable benefit in relieving the severe digital ischemia associated with this disease. 2. Which form of systemic vasculitis is most likely to present with an orbital pseudotumor? A. Takayasu arteritis B. Granulomatosis with polyangiitis (formerly Wegener granulomatosis) C. Microscopic polyangiitis D. Polyarteritis nodosa E. Behçet syndrome Answer: B  Two common ocular manifestations of granulomatosis with polyangiitis are scleritis and orbital pseudotumor. The latter can lead to proptosis and vision loss if blood circulation to the eye is compromised by the retrobulbar mass. 3. A 24-year-old man from Lebanon presents with multiple aphthous ulcers, bilateral ocular erythema, tender erythematous nodules over his anterior legs, and a deep venous thrombosis. An ophthalmologic examination reveals severe bilateral uveitis affecting both the anterior and posterior compartments of the eye. In addition to anticoagulation and the institution of glucocorticoid therapy, what is the most appropriate approach to treating his disease at this time? A. Intravenous cyclophosphamide B. Azathioprine C. Intravenous immunoglobulin D. Tumor necrosis factor inhibition E. B-cell depletion Answer: D  Tumor necrosis factor inhibition is highly effective in Behçet disease and is employed in the presence of severe disease manifestations such as posterior uveitis or central nervous system (CNS) disease.

4. A 32-year-old woman presents with the rapid onset of the worst headache of her life 1 day after giving birth to a healthy baby boy. The delivery followed an uneventful pregnancy. The patient has no history of migraine headaches, and her past medical history is unremarkable. The patient’s blood pressure is 140/90 mm Hg, and although her neurologic examination is nonfocal, the severity of the headache prompts a computed tomographic (CT) scan of the brain. This is negative for an intracranial bleed, and the headache resolves after approximately 24 hours, during which time it was profoundly disabling. Approximately 18 hours later, the headache returns, again rising to a crescendo within several minutes of onset. A repeat head CT is negative and a lumbar puncture is also performed, revealing a normal opening pressure, 1 white blood cell/mL (100% lymphocytes), and a cerebrospinal fluid protein of 40 mg/dL (normal 30 to 50 mg/dL). The headache improves following round-the-clock narcotic treatment but then worsens again on the third postpartum day. A magnetic resonance imaging study is negative for hemorrhage, cerebral infarctions, and masses, but a fourvessel cerebral angiogram shows alternating areas of vascular narrowing and dilatation (beading) in multiple vascular distributions. What is the most appropriate intervention for this patient now? A. Tighter blood pressure control, with transfer to the intensive care unit for sodium nitroprusside therapy B. Addition of a baby aspirin to her anticoagulation with low-molecularweight heparin C. Pulse methylprednisolone (1000 mg/day for 3 days) followed by prednisone 60 mg/day and cyclophosphamide 100 mg/day D. Nifedipine 30 mg three times daily E. Proceed to brain biopsy Answer: D  Nifedipine. This patient has a history that is classic for reversible cerebral vasoconstriction syndrome (RCVS), an entity that mimics CNS vasculitis but is not an inflammatory condition. The features suggesting RCVS in this patient are the presence of a “thunderclap” headache, the nonfocal neurological examination, the normal cross-sectional imaging and lumbar puncture, and the diffuse beading on angiography. This beading represents vasospasm, which is the essential pathophysiology of RCVS. Calciumchannel blockers are often used in therapy, but prolonged courses of immunosuppression are not indicated. Brain biopsy can be diagnostic in primary angiitis of the CNS, but would not be indicated in a patient like this who has typical clinical and radiographic features of RCVS. 5. A 4-year-old boy presents with 3 days of a febrile illness with temperatures to nearly 105° Fahrenheit. Physical examination reveal an erythematous “strawberry” tongue, swollen lips, and a pink macular rash on the trunk. There is also tender cervical and axillary lymphadenopathy and diffuse swelling of the distal upper and lower extremities. Which treatment is indicated immediately now to prevent the occurrence of coronary aneurysms? A. Intravenous immunoglobulin B. High-dose aspirin therapy C. Pulse methylprednisolone D. Plasma exchange E. Anticoagulation with heparin Answer: A  This child has Kawasaki disease, and the most feared complication of this disorder is formation of coronary aneurysms, which can lead to myocardial infarction. If administered in a timely manner, intravenous immunoglobulin prevents the occurrence of such aneurysms.

1802

CHAPTER 271  Polymyalgia Rheumatica and Temporal Arteritis  

associations have been suggested, including polymorphisms that may be seen in increased frequency in patients with the disease. Disease in genetically predisposed patients may be triggered by environmental factors such as viruses or endogenous antigens such as elastin, and their inflammatory manifestations are directed by specific patterns of cell-mediated, TH1-associated cytokines. The cytokine production by the mononuclear cells in the involved tissues appears to influence the clinical phenotype. Cytokine profiles characterized in temporal artery biopsy specimens obtained from patients with PMR and GCA differ. GCA tissue contains the T-lymphocyte products interferon-γ and interleukin (IL)-2 and the macrophage products IL-1β, IL-6, and transforming growth factor-β. In PMR vascular tissue, transcripts are found for transforming growth factor-β, IL-1, and IL-2 but not for interferon-γ.3 Patients with GCA who present with fever of unknown origin and who do not have ischemic symptoms, such as visual loss, have low interferon-γ levels. Arteries that express high interferon-γ levels typically have multinucleated giant cells present; these cells remove debris and secrete cytokines that stimulate intimal hyperplasia and lead to angiogenesis. IL-17-producing TH17 cells have also been found in involved vascular tissue and peripheral blood of patients with untreated GCA but disappear rapidly with institution of corticosteroid therapy, whereas TH1 cells are more persistent, speaking to the possibility that more than one antigenic trigger may be involved.4 The adventitia is considered the immunologic center in the pathogenesis of GCA. Macrophages and T lymphocytes enter the vessel wall through the vasa vasorum with the aid of adhesion molecules and come into contact with an inciting antigen. Here, it is likely that clonal proliferation of CD4+ T cells is triggered by the presentation of unknown antigens by antigen-presenting cells. The activated CD4 cells produce interferon-γ that attracts macrophages to the arterial wall. Some of these macrophages fuse at the intima-media to form multinucleated giant cells. These cells produce vascular endothelial growth factor, which triggers neovascularization, both at the intima-media junction and at the level of the vasa vasorum, sprouting from the adventitia to the media. The subsequent immunologic events lead to a characteristic topography of mononuclear cells throughout the vessel wall. Products of the giant cells and macrophages at the intima-media junction include collagenase and nitric oxide, both of which probably contribute to tissue damage. The pathologic impact of cytokines leads not only to the characteristic medial damage but also to significant intimal hyperplasia that eventually, if it is not treated, may cause luminal narrowing and tissue ischemia. In GCA, a transmural (involving all layers of the vessel) inflammatory infiltrate, comprising predominantly mononuclear cells and commonly with giant cells, is found in the superficial temporal arteries as well as in other large and medium-sized arteries. In elderly patients, fragmentation of the internal elastica is characteristic and helps differentiate this vascular lesion from that of atherosclerosis. Often, macrophages containing fragments of elastic tissue are found at the intima-media junction, the histologic center of the inflammatory process. As mentioned earlier, immunochemical techniques demonstrate differing patterns of cells and their proinflammatory and profibrotic products in the adventitia, media, and intima. Intimal proliferation may be prominent and lead to luminal narrowing. Fibrinoid necrosis, a common histologic feature in polyarteritis nodosa, is not seen in GCA. In PMR, mononuclear cell inflammation can be found not only in the proximal joints, such as the shoulders, but also in the surrounding tendons, bursae, and soft tissues consistent with enthesitis. Although muscle pains may be present, no muscle inflammation is found.

CLINICAL MANIFESTATIONS

PMR and GCA are systemic inflammatory disorders that occur primarily in patients older than 50 years, in women more than in men (2 : 1), and in whites. PMR and GCA are particularly uncommon in African Americans. Shared characteristics of the two disorders include significant cytokinedriven constitutional symptoms, such as fever, fatigue, and weight loss, as well as a markedly elevated erythrocyte sedimentation rate (ESR), anemia, and thrombocytosis. The musculoskeletal hallmark of PMR is proximal, severe, and symmetrical morning and even day-long stiffness, soreness, and pain in the shoulder, neck, and pelvic girdles.5 Fifty percent of patients with GCA share this characteristic proximal pain syndrome. Carpal tunnel syndrome and hand and knee synovitis may be seen in patients with PMR, but the overall presentation remains predominantly proximal, as opposed to rheumatoid arthritis, in which distal synovitis dominates. Whereas patients with PMR may appear to have proximal muscle weakness, this is invariably due to pain and not muscle inflammation (Table 271-1). Magnetic resonance imaging (MRI)6 and ultrasound studies7 in patients with PMR have

TABLE 271-1  GIANT CELL ARTERITIS: CLINICAL FEATURES INFLAMMATORY Polymyalgia rheumatica: constitutional symptoms Fever Weight loss Fatigue Laboratory abnormalities Hematologic: anemia, thrombocytosis Elevated sedimentation rate, C-reactive protein ISCHEMIC Ocular Diplopia Amaurosis fugax Fixed vision loss Complete blindness Cranial symptoms Headache Jaw claudication Scalp tenderness Scalp or lingual necrosis (rare) Cerebrovascular accidents Large vessel disease Leg or arm claudication Diminished pulses, blood pressure asymmetry Aortic aneurysms LATE COMPLICATIONS Aortic aneurysms Thoracic aorta Abdominal aorta Corticosteroid complications Osteoporosis Fractures Cataracts

confirmed the presence of inflammation of extra-articular synovial structures, in particular subacromial and subdeltoid bursae in the shoulders. Specific signs and symptoms of GCA are best appreciated in their anatomic and physiologic contexts. GCA preferentially affects certain blood vessels, including the branches of the external carotid artery, the ophthalmic artery and particularly its posterior ciliary branches, and the large arteries that arise from the aortic arch and abdominal aorta. Headache and scalp pain are probably the most frequent symptoms, occurring in 50 to 75% of patients. Headache is often the first manifestation of GCA and is described as boring, severe, and constant, unresponsive to simple pain medications and persisting through the night. Classically, patients complain of persistent and prominent temporal headaches, but occipital pains can also occur. Ear, pinna, or parotid gland pain may occur secondary to involvement of the posterior auricular artery. Jaw claudication and pain due to masseter muscle ischemia on chewing occur in 50% of patients. Lingual and maxillary artery involvement can lead to jaw or tongue pain on chewing or talking. The superficial temporal artery may become tortuous, prominent, nodular, or tender, but these findings are not invariable, and an abnormal temporal artery may be found on biopsy in vessels that appear normal. It is important to note that a dry, nonproductive cough can be a feature of the disease because this often may direct the clinician away from considering GCA and more toward consideration of an infectious or neoplastic respiratory cause of the symptoms. Rarely, mononeuritis multiplex and/or sensorineural hearing loss can occur but should lead the clinician to consider other possible vasculitides such as antineutrophil cytoplasmic antibody (ANCA)-associated vasculitis or polyarteritis nodosa. Fixed or intermittent symptoms related to vasculitic involvement of the ophthalmic arteries and its branches are the most dreaded in this illness and demand immediate therapeutic intervention. These symptoms are related to vascular narrowing due to both active inflammation and endothelial injury– mediated vasospasm. Decreased vision secondary to arteritis is the most common serious consequence of GCA, occurring in 20 to 50% of patients who present to ophthalmologists. It is the presenting symptom in 60% of patients with GCA who develop visual loss. A careful history of most patients who present with “sudden” visual loss reveals that preceding headache, constitutional symptoms, and PMR occurred in approximately 40% of patients. Even the evolution of the visual loss was often staggered, with amaurosis fugax in 10% and a partial field defect progressing to complete blindness over days. If GCA remains untreated, the second eye may become involved within

CHAPTER 271  Polymyalgia Rheumatica and Temporal Arteritis  

1803

Elderly patient, PMR symptoms Exclude alternative explanations: infection, malignancy, connective tissue disease Elevated sedimentation rate/CRP?

Yes

No Consider diagnostic trial corticosteroids: prednisone 15 mg qd

Prednisone 15 mg qd

Dramatic clinical response?

No Alternatives: 1. Consider GCA, and do temporal artery biopsy and/or large vessel imaging

Dramatic clinical response?

Yes

Yes

No

Treat as PMR: taper corticosteroids to lowest dose that adequately controls symptoms

Pursue alternative diagnosis such as: infection, malignancy, connective tissue disorder, hypothyroidism

2. Consider • Malignancy • Infection • Connective tissue disorder • Hypothyroidism FIGURE 271-1.  Diagnostic algorithm for polymyalgia rheumatica (PMR). CRP = C-reactive protein; GCA = giant cell arteritis.

1 to 2 weeks. The posterior ciliary arteries are the most frequently involved; thus, anterior ischemic optic neuropathy is the most common lesion, which can be easily defined by an ophthalmologist. Occlusion of the central retinal artery and its branches is uncommon; thus, exudates, hemorrhages, and frank vasculitis are infrequent. Five percent of patients with GCA may present with diplopia or ptosis, which may precede visual loss. The final visual abnormality can be a composite of many ischemic events occurring together in the optic nerve, the extraocular muscles, the chiasm, and the brain itself. Because GCA primarily involves arteries that contain elastica and the elastic lamina is lost as vessels pierce the dura, intracerebral lesions such as strokes are uncommon but not unheard of. Large artery involvement most commonly presents as arm or leg claudication; rarer manifestations are stroke, subclavian steal syndrome, intestinal infarction, and symptomatic aortic aneurysm. Thus, a subclinical arteritis can exist and demands long-term monitoring. There is an emerging appreciation that some older patients classified as having GCA can present with large vessel disease resembling Takayasu’s arteritis clinically, with a paucity of cranial ischemic symptoms but often the presence of PMR-like symptoms. Conversely, in patients presenting with typical GCA with cranial symptoms and a positive temporal artery biopsy, large vessel disease with aortic wall thickening is markedly more frequent than in matched controls without GCA, even early in the disease course. Steroid-treated PMR and GCA are self-limited illnesses lasting 1 to 2 years in most patients. However, a subgroup of patients with both disorders can have active inflammatory disease as manifested by persistent symptoms and blood test signs of active inflammation for 7 to 10 years. Of note is the fact that thoracic aneurysms with giant cells in the tissue can develop as long as 15 years after the diagnosis, successful treatment, and discontinuation of steroids. Indeed, the incidence of thoracic and aortic aneurysms is markedly higher in patients with prior history of presumably successfully treated GCA than in age-matched control subjects. Conversely, in studies of repaired aortic aneurysms, pathologic findings consistent with GCA have been found in approximately 2 to 4% of specimens from individuals without previously recognized or suspected arteritis. In most studies, survival rates for patients with PMR and GCA are similar to those of unaffected persons of the same age. However, one study did show that survival was decreased in a group of patients with GCA who had permanent visual loss and required more than 10 mg of prednisone per day at 6

months. This probably supports the experience that the morbidity and mortality are caused by steroid-related treatment complications in this high-risk, elderly group of patients possessing many comorbid conditions.

DIAGNOSIS

The diagnoses of PMR and GCA are based on clinical facts, with supporting but not diagnostic aid obtained from laboratory tests and temporal artery biopsy (Fig. 271-1).8 No physician should await an abnormal finding on temporal artery biopsy or demand the presence of an elevated ESR before making the definitive diagnosis of GCA in the setting of a characteristic clinical picture. That said, the laboratory hallmark of PMR and GCA is an elevation in IL-6-stimulated acute phase reactants such as the ESR and C-reactive protein. The ESR is usually in excess of 50 mm/hour and may exceed 100 mm/hour. An ESR in the low 20s or 30s, however, does not exclude a diagnosis of PMR or GCA if other characteristic clinical features are present and especially if the patient is already taking steroids. Normocytic, normochromic anemia and thrombocytosis occur in approximately 50% of patients with both disorders and are excellent guides to the state of inflammation. In both PMR and GCA, the frequency of rheumatoid factor, antinuclear antibody, ANCA, monoclonal proteins, and cryoglobulins is not higher than in age-matched control subjects, and complement is not reduced. Alkaline phosphatase activity may be elevated in one third of patients, primarily those with GCA. Although these tests are not indicated in PMR and GCA, muscle enzymes and electromyography are normal, and muscle biopsy shows type II fiber atrophy but no inflammation.

Superficial Temporal Artery Assessment

Temporal artery tenderness, nodularity, and diminished pulsation are typical findings on physical examination in a patient with GCA. Color duplex ultrasonography has been used as an adjunctive noninvasive diagnostic tool in GCA. A hypoechoic halo around the superficial temporal artery has been reported in 73% of patients with biopsy-proven GCA. The halo, representing edema in the arterial wall, was observed bilaterally in a significant subset of patients and disappeared in a mean of 16 days after the initiation of steroids in one study. The presence of the halo in this study had a sensitivity of 73% and was 100% specific for GCA. Other groups have been unable to replicate this experience, however, finding Doppler ultrasonography to be no more

1804

CHAPTER 271  Polymyalgia Rheumatica and Temporal Arteritis  

sensitive or specific than physical examination in patients thought to have the disease. Findings of stenosis or occlusion of temporal arteries by Doppler ultrasound have also been recognized as being modestly sensitive and specific for the diagnosis of GCA in some studies. Operator dependency remains a challenge to the more widespread use of this modality diagnostically. [18F] Fluorodeoxyglucose–positron emission tomography may be helpful in identifying large vessel inflammation suggestive of GCA, but it is not helpful in assessing the temporal arteries themselves, given their relatively small size and high background uptake in that area. Conventional angiography is rarely used in the diagnosis of GCA. Some studies have suggested that MRI/ magnetic resonance angiography (MRA) may be a helpful noninvasive diagnostic modality. Superficial cranial arteries can be visualized, and mural inflammatory changes and luminal narrowing can be identified. Large vessel involvement can also be assessed. Studies have suggested sensitivities and specificities of MRI/MRA similar to those of biopsy in the diagnosis of GCA. Nevertheless, temporal artery biopsy remains the diagnostic “gold standard” in GCA, and given the relatively easy accessibility of the artery and potentially significant morbidity of therapy in GCA, histologic confirmation is favored in most cases. Although temporal artery biopsy continues to be an important diagnostic test for the presence of GCA, a few caveats must be stated. First, in a patient in whom the clinical diagnosis is likely, treatment with steroids should be instituted immediately without waiting for the biopsy results. Second, because of the skipped nature of the pathologic inflammatory lesions in the vessel wall, as many as 20 to 30% of biopsy specimens may be normal despite an overwhelming diagnostic likelihood of GCA. However, because the biopsy is helpful in confirming the diagnosis of GCA, in which high doses of steroids are used, the following guidelines are given. Patients with pure PMR and no GCA signs or symptoms do not need a biopsy. However, because 10% of these patients may develop such clinical manifestations of GCA within the next year, they should be told to report such symptoms immediately. When GCA is likely, an outpatient biopsy should be performed on the symptomatic side of the head, preferably including inflamed areas with tenderness or nodularity and incorporating 2 to 3 cm of vessel. Multiple sections should be requested because of the segmental nature of the disease process. Some rheumatologists routinely request bilateral biopsies, which may increase the likelihood of obtaining an abnormal finding by up to 5%, whereas others perform a contralateral biopsy if the first specimen is normal. Diagnostic biopsy findings continue to be present for as long as 2 to 4 weeks after the clinical diagnosis is made and steroid treatment instituted.9

Differential Diagnosis

The systemic nature of these disorders and the fact that they occur in elderly people demand careful diagnostic scrutiny to avoid missing a malignant neoplasm or major infection and possibly treating patients inappropriately with high-dose steroids. This is true in PMR because there is no diagnostic test and in GCA because the GCA biopsy finding may be normal in the face of active, vision-threatening vasculitis. Infections that must be considered and ruled out if clinically appropriate include tuberculosis, endocarditis, and hepatitis B and C. Malignant neoplasms such as lymphoma and multiple myeloma may mimic PMR, and an age-appropriate cancer evaluation is always indicated in this age group. Autoimmune disorders such as elderly-onset rheumatoid arthritis and systemic lupus erythematosus, as well as dermatomyositis and other types of vasculitis, must be considered in the differential diagnosis and sorted out by employing clinical information and serologic testing. There is support for the concept that elderly-onset rheumatoid arthritis is the same disorder as PMR with negative rheumatoid factor, a more proximal focus of joint inflammation, and a good response to low-dose prednisone. The distinction may be semantic because neither disorder tends to evolve into an erosive arthritis. A more protracted clinical course, however, is often seen in patients in whom distal synovitis is a prominent feature, and those patients are classified as having elderly-onset rheumatoid arthritis. PMR and GCA should always be thought of in the setting of a fever of unknown origin because symptoms and signs can be occult or the history incomplete.

TREATMENT  Both PMR and GCA are highly responsive to corticosteroids, which are the preferred treatment choice.10 This response is so characteristic that an immediate and dramatic improvement in PMR and GCA symptoms within 1 to 3 days after steroid institution supports the diagnosis. Conversely, a lack of rapid and significant improvement in signs, symptoms, and function within 5 to 7 days

should lead the clinician to suspect the initial impression and consider an alternative diagnosis (e.g., tumor or infection) or the presence of GCA in PMR patients that might require a higher steroid dose. Because the inflammatory burden of the two disorders is different, different doses of steroids are employed at the onset of treatment. Whereas PMR usually responds to 15 mg of prednisone daily, GCA usually requires 40 to 60 mg of prednisone per day in divided doses or higher doses if organ or tissue damage is present or threatened.11 In GCA, if visual symptoms are present as a fixed loss or amaurosis fugax, the patient often should be treated with high-dose intravenous methylprednisolone with doses ranging from 40 mg every 8 hours to 1 g/day for 3 days, followed by high-dose oral steroids in divided doses. Within 2 to 3 days after the institution of steroids, most symptoms of PMR or GCA clear rapidly, and patients describe a miraculous improvement. The steroid dose is then maintained for 2 to 3 weeks, during which the ESR, C-reactive protein, hemoglobin, and platelet counts normalize. Steroid taper is then instituted and guided by the clinical response. In PMR, taper is commonly by 1 mg every 7 to 10 days; in GCA, taper is by 5 to 10 mg every 7 to 10 days. In GCA, the use of alternate-day corticosteroid regimens to minimize steroid side effects is generally not recommended because randomized controlled trials have demonstrated higher rates of treatment failure with alternate-day dosing schedules. It is important that the taper be guided primarily by clinical findings (e.g., PMR stiffness, headache, fatigue) and that the level of ESR elevation be considered within that clinical context. One should never “chase the ESR” because the elderly patient would be subjected inappropriately to a dangerously high cumulative dose of steroids with their attendant side effects. An increased dose of prednisone should be based on a change in symptoms, not solely on an increase in the ESR. One possible exception is in a patient with a history of GCA and prior abrupt vision loss in one eye, in whom any further compromise of vision would be catastrophic. The effective dose demanded for a flare often can be as low as 5 to 10 mg of prednisone, and uncommonly up to 60 mg/day to control symptoms (e.g., visual abnormalities). A persistently elevated ESR (>50 mm/hour) without PMR or GCA symptoms should alert the physician to look for alternative causes, such as infection. Treatment is a careful balancing act between disease control and avoidance of steroid-related toxicity. The overall goal of the patient and the physician is to attain the best disease control with the lowest dose of steroids. In most patients, prednisone can be tapered safely in 1 to 2 years. However, other patients may need to take low doses of steroids for 2 years or more. The higher the initial dose and cumulative dose, the greater the likelihood that the patient will develop a major steroid side effect such as sepsis, osteoporosis, osteonecrosis, diabetes, emotional lability, or myopathy. Appropriate immunizations, osteoporosis regimens (calcium, vitamin D, and bisphosphonates), and metabolic monitoring are mandatory in all patients prescribed chronic steroid therapy. The major feared outcome in GCA is ischemic complications of the disease, most often vision loss or, less frequently, cerebrovascular accident. Vision loss is usually irreversible, and although it is uncommon after the diagnosis is suspected and glucocorticoid therapy is instituted, it can occur early in the course of treatment. Aspirin is known to be protective against ischemic events in patients with atherosclerosis and has anti-inflammatory effects in inflamed blood vessels, including inhibition of interferon-γ. A recently reported cumulative meta-analysis of retrospective studies showed that antiplatelet or anticoagulant therapy has a marginal benefit when used together with corticosteroids in patients with established GCA.12 Although this has not been demonstrated in prospective randomized controlled trials, in most patients, adjunctive therapy with low-dose aspirin should be considered unless there is a strong contraindication to its use. Alternative immunosuppressive agents have been tested in both PMR and GCA patients in an attempt to “spare steroids” and to control the inflammatory state. Studies examining the efficacy of methotrexate in GCA have yielded mixed results, with the largest, most recent study showing no incremental benefit from combined therapy. One individual patient meta-analysis of three randomized placebo controlled trials suggested a modest benefit to methotrexate in GCA in terms of affording a steroid-sparing benefit and reducing likelihood of flares. Given the modest nature of the benefit demonstrated and the potential toxicities of methotrexate in this elderly population, methotrexate is not routinely incorporated as first-line therapy in GCA. In PMR, methotrexate has been shown in one study to afford a benefit in terms of steroid sparing and possibly reducing numbers of flares. The magnitude of the benefit appears modest, and no reduction in corticosteroid-related side effects was demonstrated. At present, methotrexate is not routinely used in the management of either disease, but in individual patients with refractory disease  or excessive corticosteroid morbidities, addition of weekly rheumatoid arthritis−level doses of methotrexate (7.5 to 20 mg/week) or azathioprine (2 mg/kg/day) is employed in selected instances. There have been case series suggesting cyclophosphamide may be of value in patients with refractory disease and/or unacceptable corticosteroid toxicity, but adverse events were common, and it is rare to use this agent in GCA. Randomized controlled trials of TNF inhibitors including infliximab and adalimumab in GCA have failed to demonstrate benefit in terms of preventing

CHAPTER 272  Infections of Bursae, Joints, and Bones  

relapses or affording a steroid-sparing benefit.13 Abatacept, a costimulatory molecule blocker, is presently being evaluated in large vessel vasculitis including GCA, in a large randomized trial, but those results are not yet available. Tocilizumab, an IL-6 inhibitor, has been of major interest in GCA and PMR because this cytokine seems pivotal in these disorders. Tocilizumab reliably reduces C-reactive protein and ESR, which in theory could be independent of a true beneficial effect on the underlying arteritis, but early controlled observations are encouraging.14 This will be a challenge in assessing the true efficacy of Tocilizumab in GCA in the larger controlled clinical trials that are in progress.

FUTURE DIRECTIONS

Better understanding of the disease-causing roles of immunologically active cells and their cytokine products, along with genetics and correlations with clinical subsets, will lead to more focused treatment modalities and the avoidance of the need for long-term treatment with steroids. A recently published cohort study revealed that GCA is associated with increased risks for myocardial infarction, stroke, and peripheral vascular disease,15 suggesting that greater attention should be paid to cardiovascular risk reduction in patients with this disease. GENERAL REFERENCES For the General References and other additional features, please visit Expert Consult at https://expertconsult.inkling.com.

272  INFECTIONS OF BURSAE, JOINTS, AND BONES ERIC L. MATTESON AND DOUGLAS R. OSMON

  INFECTION OF BURSAE

Septic Bursitis

DEFINITION

Bursae are the satellite structures that form to protect tissues from bony prominences. The superficial bursae, including the olecranon, prepatella, infrapatella, and bursae over the first metatarsophalangeal bunions, are more likely to become infected than are the deep bursae, such as the subacromial, trochanteric, and iliopsoas bursae.1

EPIDEMIOLOGY

Olecranon bursitis may occur in as many as 10 in 100,000 persons. The majority of cases occur in men, and antecedent trauma to the skin is frequent.

PATHOBIOLOGY

Septic bursitis of superficial bursae is most commonly due to direct inoculation through the overlying skin; less commonly, it is secondary to overlying cellulitis. Most cases of deep septic bursitis are due to contiguous spread from adjacent infected joints or hematogenous seeding. Predisposing risk factors for septic bursitis include trauma to the skin, as may occur in plumbers, athletes, and patients with chronic obstructive pulmonary disease (COPD) who frequently lean on the elbows; prepatellar or infrapatellar septic bursitis occurs in housecleaners, gardeners, and carpet layers. At least one third of patients with septic bursitis have an underlying comorbid illness such as diabetes mellitus, rheumatoid arthritis, gout, COPD, or alcoholism.

CLINICAL MANIFESTATIONS

In immune-competent patients, septic bursitis often but not always presents with fever and erythema and warmth of the overlying skin; there may be swelling of the bursae. In contrast to those with septic arthritis, patients with septic bursitis of superficial bursae have intact range of motion of the joints, which may be limited only at the extremes of flexion. Pain on motion of the

1805

joint and restriction of joint range of motion are highly suggestive of septic arthritis. Acute phase reactants such as C-reactive protein, the sedimentation rate, and the white blood cell count (WBC) may be elevated

DIAGNOSIS

Radiography should be performed to look for a foreign body and to evaluate the surrounding bones. Aspiration of bursal fluid is helpful in the diagnosis of patients who have pain, erythema, and/or swelling of an affected area. However, given the risk for contaminating the bursa if the aspiration occurs through skin involved with cellulitis, many clinicians choose to aspirate a bursa only if empirical antimicrobial therapy has failed. Ultrasound or computed tomography (CT) guidance greatly enhances the successful aspiration of superficial bursae. Care must be taken not to violate the joint space when aspirating a bursa to avoid inoculating the joint space. The leukocyte count of the bursal fluid is generally lower than that seen in septic arthritis, with a mean of 13,500 cells/mm3. Even in immune-competent hosts, cell counts can range from less than 1500 to greater than 100,000/ mm3. A leukocyte count greater than 2000/mm3 has a sensitivity of 94% and a specificity of 79% for superficial (olecranon or prepatellar) bursitis. Bacterial culture and in vitro susceptibilities must be obtained; if additional fluid is available, a Gram stain may be obtained, although its sensitivity may be as low as 15%. The presence of crystals does not exclude the possibility of septic bursitis. Staphylococcus aureus (Chapter 288) is the most common cause of septic bursitis, present in more than 80% of culture-proven cases, followed by β-hemolytic streptococci. Aerobic gram-negative bacilli, including Escherichia coli, Campylobacter jejuni, and Pseudomonas species, are rare causes of septic bursitis. Chronic bursitis may be associated with systemic infections due to Brucella abortus, atypical mycobacteria, or Mycobacterium tuberculosis, as well as fungi; the presence of these infections should raise the possibility of systemic infection.

Differential Diagnosis

In the immune-competent host, nonseptic bursitis (Chapter 263) may have a somewhat more indolent presentation than septic bursitis. The differential diagnosis includes gout, pseudogout, arthritis, and trauma with hemobursa. An overlying cellulitis may be confused with bursitis. Fever is usually not present in nonseptic bursitis due to mechanical or friction trauma.

TREATMENT  Treatment of septic bursitis is guided by knowledge of the putative underlying organisms, in most cases, S. aureus. Because the Gram stain is positive in less than two thirds of patients and cultures may be delayed, empirical therapy is guided by the clinical presentation. Most patients can be treated as outpatients, but those who are immunocompromised may require hospitalization for intravenous antibiotic therapy. Initial ambulatory treatment in patients without comorbidities may consist of an oral antistaphylococcal penicillin or first-generation cephalosporin. If community-acquired methicillin-resistant S. aureus (MRSA) is suspected, co-trimoxazole or minocycline may be added to one of these agents. In patients who are allergic to penicillin, oral clindamycin or linezolid may be used. Patients who have severe inflammation, are septic, or are immunocompromised may require hospitalization for initiation of treatment with intravenous nafcillin, oxacillin, or cefazolin; if MRSA is suspected, intravenous vancomycin, daptomycin, or linezolid should be used. Guidelines on the treatment of MRSA have been published.2 Vancomycin can also be used in patients who are allergic to penicillin. The duration of antimicrobial therapy is guided by the clinical response and comorbid states. It should be continued until there is no longer bursal inflammation. This may require several weeks of intravenous or oral therapy and multiple aspirations. Failure of the septic bursitis to respond to initial antibiotic therapy mandates a second course of therapy; recurrence thereafter or inability to adequately drain the bursa with needle aspiration is an indication for surgical intervention.

PREVENTION

Because superficial septic bursitis is often associated with occasional or avocational activities involving kneeling or resting on the elbows, using protective padding may be helpful.

PROGNOSIS

The optimal duration of therapy is unknown, but prognosis of superficial bursitis is generally excellent. The presence of comorbid conditions,

CHAPTER 271  Polymyalgia Rheumatica and Temporal Arteritis  

GENERAL REFERENCES 1. Kermani TA, Warrington KJ. Polymyalgia rheumatica. Lancet. 2013;381:63-72. 2. Carmona FD, Gonzalez-Gay MA, Martin J. Genetic component of giant cell arteritis. Rheumatol (Oxf). 2014;53:6-18. 3. Samson M, Audia S, Martin L, et al. Pathogenesis of giant cell arteritis: new insights into the implication of CD161+ T cells. Clin Exp Rheumatol. 2013;31(1 suppl 75):S65-S73. 4. Weyand CM, Younge BR, Goronzy JJ. IFN-γ and IL-17: the two faces of T-cell pathology in giant cell arteritis. Curr Opin Rheumatol. 2011;23:43-49. 5. Mackie SL, Mallen CD. Polymyalgia rheumatica. BMJ. 2013;347:f6937. 6. Klink T, Geiger J, Both M, et al. Giant Cell Arteritis: Diagnostic Accuracy of MR Imaging of Superficial Cranial Arteries in Initial Diagnosis-Results from a Multicenter Trial. Radiology. 2014;273: 844-852. 7. Ball EL, Walsh SR, Tang TY, et al. Role of ultrasonography in the diagnosis of temporal arteritis. Br J Surg. 2010;97:1765-1771. 8. Nesher G. The diagnosis and classification of giant cell arteritis. J Autoimmun. 2014;48-49:73-75.

1805.e1

9. Mackie SL, Pease CT. Diagnosis and management of giant cell arteritis and polymyalgia rheumatica: challenges, controversies and practical tips. Postgrad Med. 2013;89:284-292. 10. Weyand CM, Goronzy JJ. Giant-cell arteritis and polymyalgia rheumatic. N Engl J Med. 2014; 371:50-57. 11. Waldman CW, Waldman SD, Waldman RA. Giant cell arteritis. Med Clin North Am. 2013;97: 329-335. 12. Martinez-Taboada VM, Lopez-Hoyos M, Narvaez J, et al. Effect of antiplatelet/anticoagulant therapy on severe ischemic complications in patients with giant cell arteritis: a cumulative metaanalysis. Autoimmun Rev. 2014;13:788-794. 13. Yates M, Loke YK, Watts RA, et al. Prednisolone combined with adjunctive immunosuppression is not superior to prednisolone alone in terms of efficacy and safety in giant cell arteritis: meta-analysis. Clin Rheumatol. 2014;33:227-236. 14. Loricera J, Blanco R, Castañeda S, et al. Tocilizumab in refractory aortitis: study on 16 patients and literature review. Clin Exp Rheumatol. 2014;32:S79-S89. 15. Tomasson G, Peloquin C, Mohammed A, et al. Risk of cardiovascular disease early and late after a diagnosis of giant-cell arteritis: a cohort study. Ann Intern Med. 2014;160:73-80.

1805.e2

CHAPTER 271  Polymyalgia Rheumatica and Temporal Arteritis  

REVIEW QUESTIONS 1. A 74-year-old gentleman presents with a 7-week history of increasing generalized shoulder and hip pain, morning stiffness, low-grade fevers, and a dry cough. He was found to be anemic with a hemoglobin of 9.7 and had a thrombocytosis with a platelet count of 642 and an elevated sedimentation rate of 114. A search for infection and neoplasia has been unrevealing, including a bone marrow biopsy, which did not suggest a myeloproliferative neoplasm. He denies headaches, visual symptoms, or jaw claudication. The patient has reluctance regarding any further interventional procedures. Which of the following would be the best option to establish his diagnosis? A. A computed tomographic (CT) angiogram with imaging of the aortic arch and its major branches B. A conventional angiogram C. Duplex ultrasound of his temporal arteries and axillae. D. A positron emission tomography (PET) scan E. Bilateral temporal artery biopsies Answer: E  The patient likely has giant cell arteritis. Although PET scanning, duplex ultrasound of temporal arteries, and CT angiography may be helpful in some patients, a temporal artery biopsy remains the gold standard and is the most specific if the biopsy can be established. Duplex sonography has yet to be proved reliably accurate. PET scanning and angiography, although potentially helpful, can provide in false-positive results in patients with severe atherosclerosis and false-negative results in patient without clear evidence of large vessel disease. Polymyalgia rheumatica (PMR) may be a possibility here (and this patient indeed has PMR), but the presence of dry cough and degree of inflammatory response makes concurrent giant cell arteritis (GCA) a pressing concern 2. A 77-year-old woman with a history of diabetes, osteoporosis, and modest alcohol use presents with 3 weeks of headaches and feeling systemically unwell and then loss of vision in her left eye 2 days ago. She is found to be anemic with a markedly elevated sedimentation rate, and a temporal artery biopsy confirms the diagnosis of GCA. She is started on 60 mg of daily prednisone. Which of the following would be an appropriate additional intervention at this point? A. Add Infliximab 5 mg/kg monthly as a steroid-sparing intervention. B. Add methotrexate 10 mg weekly with the hope of titrating that to 15 mg weekly to afford a steroid-sparing benefit. C. Change her corticosteroid regimen to 120 mg on alternate days and plan an alternate day taper from there. D. Add low-dose aspirin daily to her regimen. E. Add tocilizumab 8 mg/kg monthly as a steroid-sparing intervention. Answer: D  Patients with GCA often struggle with comorbidities that raise concerns regarding prolonged exposure to corticosteroids. There is at this time, however, no proven steroid-sparing intervention that can decrease the risk for cranial ischemic complications or even that has been demonstrated to reduce corticosteroid-related adverse events. Low-dose aspirin has been shown in two retrospective studies to be likely protective against cranial ischemic complications, so if there is no contraindication, that would be an appropriate intervention. Methotrexate has been shown in one trial in one individual meta-analysis to have a corticosteroid benefit in terms of cumulative steroid dose and also reduced risk for flares, but in the largest multicenter study addressing its efficacy, it was not shown to be superior to placebo. Moreover, in this patient with comorbidities including diabetes and alcohol use, methotrexate could be morbid, so this would not seem an appropriate intervention at this point. Infliximab has been shown in a randomized, double-blinded, placebo-controlled trial not to be helpful in the treatment of GCA and can increase risk for infection in this patient already on high doses of corticosteroids with underlying diabetes. Alternate-day corticosteroid therapy has been shown to increase the risk for later flare and is not generally employed in GCA, particularly earlier in the course of treatment. Tocilizumab, an interleukin-6 inhibitor, is an agent of great interest in temporal arteritis, but it has not yet been shown to have a corticosteroid-sparing or disease-modifying benefit in this disorder; however, trials are presently in progress addressing this issue.

3. A 72-year-old man presents with a 2-month history of morning stiffness, mild synovitis in his knees, puffiness in his hands in the early morning hours, and severe shoulder and thigh girdle myalgias. He is mildly anemic with hemoglobin of 11.6 and has a markedly elevated sedimentation rate at 82. He is seronegative for rheumatoid factor, citric citrullinated peptide (CCP), and antineutrophil cytoplasmic antibodies. He has no evidence of peripheral joint erosions on plain radiographs. Which of the following would be the most appropriate therapeutic intervention at this point? A. Begin naproxen 500 mg twice daily. B. Begin methotrexate 10 mg weekly with the hope of bringing that dose up to 15 mg weekly in the next few weeks. C. Begin methylprednisolone 12 mg daily. D. Begin hydroxychloroquine 200 mg twice daily. E. Begin methylprednisolone 12 mg daily along with methotrexate 10 mg weekly with a plan to taper the corticosteroids to off over the next 4 weeks. Answer: C  The patient described has PMR but presents with some peripheral synovitis, which can be seen in up to 10 to 15% of patients with PMR. The patient clinically is unlikely to have rheumatoid arthritis in the absence of seropositivity for rheumatoid factor and CCP, has typical shoulder and thigh girdle myalgias and morning stiffness, and does not have evidence of erosive disease radiographically. An initial trial of methylprednisolone 12 mg daily would have both therapeutic and likely diagnostic value because if his response is spectacular, that would strongly argue for PMR and continuing that approach to therapy. Although methotrexate and hydroxychloroquine have been agents of interest in polymyalgia rheumatica, their use has not been well established in double-blinded, placebo-controlled trials. One study suggesting a benefit to methotrexate in PMR did not demonstrate a benefit in terms of reducing numbers of corticosteroid-related adverse events, and at longer term follow-up, the initial benefits seen by the addition of methotrexate did not seem to persist. Naproxen would be unlikely to afford adequate relief to this patient and, given his comorbidities, would likely be as problematic if not more problematic than a trial of corticosteroids. 4. An 80-year-old gentleman with a prior history of giant cell arteritis successfully treated 7 years earlier presents with searing chest pain. His past medical history is otherwise only notable for borderline hypertension. He was successfully treated with a 1-year course of steroids but has been free of corticosteroids for 3 years. He has been feeling systemically well until recently. He had not seen a physician in 2 years. Which of the following explanations for his chest pain is of primary concern? A. Dissecting thoracic aneurysm B. Acute pericarditis C. A myocardial infarction related to his increased risk for atherosclerosis given his prior history of GCA D. Costochondritis related to a flare of PMR E. Rib fracture Answer: A  Patients with a prior history of GCA are at increased risk for thoracic aortic aneurysms, even several years after completing therapy without evidence of residual active disease. The searing quality of his pain and his prior history of treated GCA make this a concern that should not be missed. The quality of his pain did not sound suggestive of a myocardial infarction, and patients with GCA have not been demonstrated to be at increased risk for atherosclerotic heart disease, although the possibility of that association has been of interest. Patients with GCA are not particularly at a higher risk for pericarditis. Although PMR can arise even years after successfully treated GCA (or even years after successfully treated PMR), the quality of his pain does not sound suggestive thereof, and costochondritis would not typically be a feature of PMR. The quality of the pain described did not sound suggestive of a rib fracture, but moreover that would be a diagnosis that could be overlooked at least initially without putting the patient at increased risk for a serious cardiovascular compromise.

CHAPTER 271  Polymyalgia Rheumatica and Temporal Arteritis  

5. A 72-year-old woman had presented with an 8-week history of progressively increasing morning stiffness, shoulder and thigh girdle myalgias and arthralgias, and fatigue. An extensive search for underlying infection and age-appropriate screening for underlying neoplasm were unrevealing. A serum immunofixation did not reveal a monoclonal gammopathy. She was found to be anemic with a hemoglobin of 10.7 and had a sedimentation rate of 108. She denied any headaches, scalp sensitivity, jaw claudication, or visual symptoms. She admitted to low-grade fevers but not greater than 100° F. PMR was suspected, and she was treated with methylprednisolone 12 mg daily in divided doses. Three days later, she called to report no significant improvement in her symptoms. It was suggested that her dose be raised to 16 mg daily in divided doses. She comes in for evaluation still complaining of similar myalgias and arthralgias, and her sedimentation rate remains elevated at 62, although her hemoglobin improved slightly to 10.9. Which of the following would be an appropriate next intervention? A. Add methotrexate as a disease-controlling/steroid-sparing intervention. B. Add infliximab to help achieve disease control. C. Refer her for a bone marrow biopsy to exclude underlying myeloproliferative or lymphoproliferative disease. D. Add hydroxychloroquine 400 mg daily. E. Refer her for a temporal artery biopsy and raise the methylprednisolone to 32 mg daily.

1805.e3

Answer: E  This patient presented with fairly typical PMR symptoms but in addition had low-grade fevers, which can be seen but are not as common as they are in GCA. Moreover, she was somewhat anemic. Her lack of response to 12 mg and then even 16 mg of daily methylprednisolone makes PMR unlikely unless in the context of a more substantial inflammatory disease such as GCA. GCA is found in approximately 10% of patients with “pure” PMR, and conversely, up to 50% of patients with GCA can present with PMR-like symptoms.

1806

CHAPTER 272  Infections of Bursae, Joints, and Bones  

especially those associated with deep bursal infections, including septic arthritis, bacteremia, and osteomyelitis, is associated with more intractable and difficult disease.

  INFECTION OF JOINTS

Septic Arthritis

rheumatoid arthritis, osteoarthritis, and low socioeconomic status, as well as advanced age, skin infection, indwelling intravenous catheters, cancer, and immunosuppressive therapies, including biologic response modifiers used in the management of autoimmune diseases such as rheumatoid arthritis and inflammatory bowel disease.

CLINICAL MANIFESTATIONS

DEFINITION

Septic arthritis refers to infection of a joint by a microorganism. It is associated with increased morbidity and mortality, as well as loss of articular integrity and function. Septic arthritis is usually caused by a bacterial infection.3 Other microorganisms can cause infections with clinical characteristics that differ from those of bacterial infections; these are reviewed separately.

NONGONOCOCCAL SEPTIC ARTHRITIS EPIDEMIOLOGY

The incidence of septic arthritis affecting native joints is about 5 to 8 in 100,000 patient years. Among patients presenting with an acutely swollen and painful joint, the prevalence of bacterial arthritis ranges widely, from less than 10% to as high as 27%, depending on the source population. Nongonococcal septic arthritis is the most common form of septic arthritis and is somewhat more common in men than in women.

PATHOBIOLOGY

More than 90% of cases of septic arthritis are due to staphylococci or streptococci (Table 272-1). Septic arthritis can result from direct inoculation (e.g., accidents, bites, surgery) or by extension from infected bone into an adjacent joint space. Approximately 75% of cases are due to hematogenous spread, particularly in patients with indwelling catheters and immunocompromised patients. Septic arthritis due to needle arthrocentesis (80%) of septic arthritis are monoarticular; the knee is involved in more than 50% of cases. Polyarticular joint sepsis may be seen in immunocompromised patients and those with rheumatoid arthritis or systemic lupus erythematosus. Such patients frequently lack typical signs and symptoms of infection and may not appear to be particularly ill at presentation, but they may develop rapid cardiovascular decompensation. This is particularly true for patients who are taking glucocorticosteroids, other immunosuppressive agents, and biologic response modifiers, including tumor necrosis factor-α (TNF-α) inhibitors. Patients with septic arthritis affecting nondiarthrodial joints, such as the acromioclavicular or sacroiliac joints, may have a history of intravenous drug use or may have intravenous catheters in place to treat other medical conditions. Infection of the symphysis pubis is associated with previous urinary tract surgery, pelvic malignancy, intravenous drug use, or vigorous weight-bearing physical activity, such as long-distance running, in female athletes. A finding of microorganisms in the joint should lead to an appropriate history and physical examination to identify a source of hematogenous infection, such as cellulitis, pneumonia, or urinary tract infection. Staphylococci and β-hemolytic streptococci may enter directly through open wounds, whereas gram-negative infection may be associated with bowel or bladder disease.

DIAGNOSIS

Plain radiography should be performed to evaluate the surrounding bones and joint space and to provide a baseline for comparison after therapy is completed. Imaging modalities such as magnetic resonance imaging (MRI), CT, and plain radiography are useful to determine whether there is associated osteomyelitis and in cases of diagnostic uncertainty. Blood cultures are positive in up to 50% of patients with bacterial septic arthritis and should be obtained in all patients in whom this diagnosis is suspected.

TABLE 272-1  MICROORGANISMS RESPONSIBLE FOR ACUTE SEPTIC ARTHRITIS AND ACUTE AND CHRONIC OSTEOMYELITIS SEPTIC ARTHRITIS MICROORGANISM

OSTEOMYELITIS: ACUTE AND CHRONIC FREQUENCY (%)

Gram Positive

60-90

Staphylococcus aureus Group A, B, C streptococci Coagulase-negative staphylococci Streptococcus pneumoniae Enterococcus sp Corynebacterium sp

50-70 15-30 6-20 1-3 60 yr)

10-20 mg daily 20-40 mg daily (depression), up to 80 mg daily (OCD) 20 mg daily 20-50 mg daily 25-50 mg daily 50-200 mg daily

  Paroxetine   Sertraline SNRIs (serotonin and norepinephrine reuptake inhibitors)

TARGET ADULT DOSE RANGE†

Inhibit presynaptic reuptake of serotonin and norepinephrine

  Duloxetine   Venlafaxine

30-60 mg daily 30-60 mg daily on a twice-daily schedule, maximum of 120 mg/day 37.5 mg bid 150-375 mg/day on bid schedule

  Desvenlafaxine

50 mg daily

TCAs (tricyclic antidepressants)

  Amitriptyline

Inhibit presynaptic reuptake of serotonin and norepinephrine (in varying proportions depending on the specific TCA)

25-75 mg qhs

150-300 mg qhs

25-75 mg daily 150-300 mg daily

  Doxepin   Imipramine

25-75 mg qhs 150-300 mg qhs 25-75 mg daily 150-300 mg daily

  Nortriptyline

25-50 mg qhs

  Selegiline   Tranylcypromine Other   Bupropion

  Mirtazapine   Trazodone

  Vilazodone

10 mg bid 15 mg tid (selective MAO-B inhibitor)

5 mg bid

20-60 mg/day in bid-qid dosing 45-90 mg/day in tid or qid dosing 5 mg bid

10 mg tid

30-60 mg/day in tid dosing

Unknown, although it is a weak inhibitor of presynaptic reuptake of norepinephrine and dopamine Antagonist at α2 and 5-HT2 receptors Inhibits presynaptic reuptake of serotonin; antagonist at 5-HT2 and 5-HT3 receptors Inhibits presynaptic reuptake of serotonin; agonist at 5-HT1A receptors

75-150 mg/ day

15 mg qhs 25-50 mg qhs

10 mg daily

COMMENTS

Few drug-drug interactions Enantiomer of citalopram Long half-life; tends to be activating Tends to be sedating Few drug-drug interactions

Nausea, diarrhea, serotonin syndrome, sinus tachycardia, mild elevation in blood pressure, tremor

XR form allows once-daily dosing Metabolite of venlafaxine Anticholinergic effects, sedation, orthostatic hypotension, tremor, cardiac conduction delays, ventricular arrhythmias

50-150 mg qhs

Inhibit monoamine oxidase, the enzyme that catalyzes oxidative metabolism of monoamine neurotransmitters

  Isocarboxazid   Phenelzine

Anticholinergic effects

50 mg daily, maximum of 100 mg ER daily

  Desipramine

MAOIs (monoamine oxidase inhibitors)

SIDE EFFECTS Nausea, diarrhea, sexual dysfunction, serotonin syndrome Risk of QTc prolongation/ torsade de pointes in at-risk patients

Strongly anticholinergic and sedating; aim for combined amitriptyline/nortriptyline blood level of 120-250 ng/mL Aim for blood level of 115-250 ng/mL Strongly sedating Strongly anticholinergic; aim for combined imipramine/ desipramine blood level of 180-350 ng/mL Aim for blood level of 50-150 ng/mL; least anticholinergic of the TCAs

Need for tyramine-free diet to avoid sympathomimetic (hypertensive) crisis; sedation, anticholinergic effects, tremor, orthostatic hypotension

Tyramine-free diet not required

Take with meals

300-450 mg/day

Activating; risk for seizures reduced by divided dosing and careful dosage titration

Divided dosing required unless using SR or XL forms

30-45 mg qhs; maximum of 45 mg qhs 300-600 mg qhs for depression, 25-100 mg qhs for insomnia

Sedation, hyperphagia

Becomes more stimulating at higher doses Few sexual side effects

40 mg daily

Nausea, diarrhea, sexual side effects

Sedation, priapism

*Patients on any of these medications must be monitored for suicidal thoughts. † Target doses in the elderly may be lower. ER = extended release; 5-HT2 = 5-hydroxytryptamine; OCD = obsessive-compulsive disorder; qhs = at bedtime; SR = sustained release; XR = extended release.

Dosage must be increased slowly

2350

CHAPTER 397  Psychiatric Disorders in Medical Practice  

TABLE 397-6  SYMPTOMS/SIGNS OF AN EPISODE OF MANIA DIAGNOSTIC CRITERIA A distinct period of abnormally; persistently elevated, expansive, or irritable mood; and abnormally and persistently increased goal-directed activity or energy lasting ≥ 1 week and present most of the day, nearly every day, AND 3 or more of the following symptoms/signs (4 or more if the mood abnormality is only irritability): Inflated self-esteem/grandiosity Decreased need for sleep More talkative or pressure to keep talking Subjective experience of racing thoughts or flight of ideas observed on examination Distractibility Increase in goal-directed activity or psychomotor agitation Excessive involvement in activities with a high potential for painful consequences MANIC SYMPTOMS/SIGNS GROUPED CONCEPTUALLY, WITH ADDITIONAL COMMON PHENOMENA

episodes. Even though several second-generation antipsychotic medications have received approval by the U.S. Food and Drug Administration (FDA) for their mood-stabilizing properties, their potential to precipitate metabolic syndrome (and to a lesser extent, tardive dyskinesia) should limit their use as maintenance medications to patients for whom other mood stabilizers are inefficacious or poorly tolerated. For acute episodes of mania, second- or firstgeneration antipsychotics are more rapidly efficacious than mood stabilizers, with doses similar to their use for acute psychosis (see Table 397-12). For acute treatment of depressive episodes, antidepressants may be required, but they may precipitate mania. Therefore, patients should receive therapeutic doses of a mood stabilizer first, and exposure to antidepressant medication should be for the minimum dose and duration required. Electroconvulsive therapy is useful for refractory mania or depression and for patients with relative contraindications to medications, such as pregnant women. Standard psychotherapies for unipolar depression also may be used for bipolar depression. Ongoing psychotherapy may be important to encourage compliance with maintenance treatments and help patients manage psychosocial stressors, thereby minimizing their impact on precipitating mania or depression.

Emotional Euphoria Irritability Labile affect Ideational Grandiosity Somatic/Neurovegetative Increased energy Psychomotor agitation Decreased need for sleep Distractibility Other Goal-directed hyperactivity Pressured speech Impaired judgment Flight of ideas Psychotic symptoms (may include delusions, hallucinations, or derailment of thought processes such as loose associations)—defines the subtype “mania with psychotic features” From Diagnostic and Statistical Manual of Mental Disorders. 5th ed. (DSM-5) Washington, DC: American Psychiatric Association, 2013, with permission.

spending, sexual activity, or gambling); increased energy; decreased need for sleep; pressured speech; and distractibility.6 As with major depression, the diagnosis is based on findings from the history and examination revealing a pattern of recurrent manic episodes (Table 397-6) that are usually interspersed with major depressive episodes and cannot be explained by other medical conditions, medications, or other substances. Although persons with bipolar disorder may become psychotic while in manic or depressed states, a history of psychotic symptoms in the absence of mania or depression indicates a diagnosis other than bipolar disorder. Manic and depressive episodes may also be seen in the course of delirium (Chapter 28) and dementia (Chapter 402), in which case the psychiatric symptoms are accompanied by the neurocognitive impairment that is the hallmark of the latter conditions.

TREATMENT  The mainstay of treatment for bipolar disorder is mood stabilizer medications to reduce the frequency and severity of recurrent manic and depressive episodes.7 Traditional mood stabilizers with substantial evidence base to support their use include lithium (typical dose of 600 to 1500 mg/day or higher given in two or three divided doses as needed to achieve plasma levels of 0.6 to 1.2 mEq/L [up to 1.4 mEq/L in acute mania]), valproic acid (typical dose of 500 to 1500 mg/day or higher as tolerated to achieve plasma levels of 50 to 100 µg/mL), and carbamazepine (typical dose of 400 to 1200 mg/day as tolerated to achieve plasma levels of 4 to 12 µg/mL). The combination of lithium plus valproate is superior to valproate alone for prevention of relapses. A8  A number of other anticonvulsants have been tried but generally with less empirical support for their use, although lamotrigine (starting at 25 mg/day, maximum dose of 200 mg/day, titrated slowly to minimize the risk for Stevens-Johnson syndrome) can be used for prophylaxis against depressive

PROGNOSIS

Although the classically described course of bipolar disorder includes return to baseline functioning between episodes, some patients may experience frequent debilitating episodes (known as “rapid cycling,” defined as four or more episodes per year), and others may experience deterioration in overall functioning over time.

  OTHER MOOD DISORDERS

Although the diagnosis of chronic major depression should be made in patients with long-lasting major depressive episodes, others may have chronic (≥2 years) lower-level depressive symptoms known as persistent depressive disorder (dysthymia), a significant minority of whom will improve with a combination of antidepressant medication and psychotherapy. Other patients may have “less than major depression” of shorter duration, often referred to as “subsyndromal” or “subthreshold” depression. Growing evidence suggests that broad psychosocial interventions (e.g., bibliotherapy, social activation) may prevent progression to full-fledged major depression in such patients. Premenstrual dysphoric disorder manifests as cyclical depressive and anxiety symptoms that resolve in the week post menses and recur in the week before the onset of menses; this is the only mood disorder that may respond to brief cyclical administration of antidepressant medication. Less severe bipolar-related disorders include bipolar II disorder, which is characterized by episodes of hypomania (i.e., low-level manic symptoms without substantial functional impairment and without psychosis) and episodes of major depression. Such patients typically seek care during depressive episodes rather than during hypomania, but antidepressant medication may worsen the manic symptoms. It is therefore imperative to ask about a history of manic or hypomanic symptoms in the evaluation of all patients with depression. Cyclothymic disorder, which includes episodes of hypomania and low-level depressive episodes, may be difficult to distinguish from the mood instability seen in “cluster B” personality disorders (see later).

  ANXIETY DISORDERS DEFINITION

The anxiety disorders (Table 397-7) are a group of conditions whose hallmark is idiopathic anxiety, typically accompanied by psychological (i.e., thought content) and somatic symptoms. Anxiety is a common accompanying symptom in many other psychiatric disorders, but the primary anxiety disorders lack the neurocognitive deficits, depressive or manic symptoms, or psychosis seen in the other disorders. Trauma-related and obsessive-compulsive disorders are now classified separately from the anxiety disorders.

EPIDEMIOLOGY

Anxiety disorders are a worldwide problem.8 Panic disorder has a 12-month prevalence of 2 to 3%. Generalized anxiety disorder has a 12-month prevalence of approximately 3%, and the phobias collectively have a prevalence of 10 to 15% in the adult population. Clear data on incidence rates are not available. Most primary anxiety disorders have an age at first onset in adolescence through the mid-30s, with generalized anxiety disorder toward the older end of that range. Most anxiety symptoms with new onset in later life are due to mood or neurocognitive disorders or are secondary to medical illnesses or

CHAPTER 397  Psychiatric Disorders in Medical Practice  

TABLE 397-7  TYPES OF ANXIETY DISORDERS ANXIETY DISORDER

MAJOR CLINICAL CHARACTERISTICS

Panic disorder

Recurrent unexpected panic attacks, typically with anticipatory anxiety and avoidance behavior

Generalized anxiety disorder

Excessive anxiety and worry, not meeting the criteria for other anxiety disorders, lasting ≥ 6 months

Phobias:   Agoraphobia

  Social phobia (social anxiety disorder)

  Specific phobia

Anxiety about or avoidance of places or situations from which escape might be difficult or embarrassing or in which help might not be available in the event of panic symptoms Anxiety provoked by exposure to social situations, typically with ensuing avoidance behavior; may be generalized (i.e., in response to many interpersonal situations) or specific in response to a particular social situation (e.g., using a public restroom, public speaking) Anxiety provoked by exposure to a specific feared object or (nonsocial) situation, typically with ensuing avoidance behavior

Author summary based on categories and criteria from American Psychiatric Association. Diagnostic and Statistical Manual of Mental Disorders. 5th ed. (DSM-5) Washington, DC: American Psychiatric Association; 2013.

drugs; true late-onset primary anxiety disorders are often triggered by traumatic or other stressful life events.

PATHOBIOLOGY

Anxiety may be understood in part as inappropriate triggering of the stress response system, commonly referred to as the “fight-or-flight” response. However, it is important to recognize that the responses involve a wide range of cognitive, motor, neuroendocrine, and autonomic systems and thus are not limited to manifestations of sympathetic nervous system activity. The central nucleus of the amygdala is believed to play a crucial role in coordinating the anxiety response. The amygdala receives excitatory glutamatergic input from several cortical areas and from the thalamus, thereby allowing it to respond to a wide variety of stimuli, including sensory input from the external world, as well as stressors that are processed and recognized by cortical association areas. The amygdala in turn projects to the many brain regions that subserve the clinical manifestations of the anxiety response, in part through its direct projections to the important centers of monoaminergic systems: dopaminergic neurons of the ventral tegmental area in the midbrain, noradrenergic neurons in the locus caeruleus, and serotonergic neurons in the raphe nuclei. From a cognitive psychology perspective, the pathogenesis of many anxiety disorders, particularly panic, may be understood as catastrophic misinterpretations of normal somatic sensations. A vulnerable individual may become aware of a normal or minimally abnormal body sensation, which is interpreted as something concerning, thereby leading to sympathetic and other autonomic arousal, which in turn leads to further somatic sensations (e.g., tachycardia, sweating) in what becomes a vicious cycle of thoughts and somatic symptoms.

CLINICAL MANIFESTATIONS

Most individuals experience one or more somatic symptoms (Table 397-8) that accompany psychic anxiety, regardless of whether the anxiety is normal or part of a pathologic condition. Such somatic symptoms may be referable to virtually every body organ system. Many anxiety disorders include acute, discrete periods of symptoms known as panic attacks. In a panic attack, the patient experiences an abrupt surge in anxiety, fear-related thoughts, and somatic symptoms in the space of a few minutes (“crescendo onset”). The acute symptoms resolve quickly, typically within an hour or less.

Panic Disorder

Panic disorder consists of recurrent panic attacks. Although some panic attacks may be precipitated by situations known to be stressful, at least some attacks must be unexpected (“out of the blue”). Patients also exhibit anticipatory anxiety in which they experience ongoing psychic distress by worrying about their next panic attack or the attack’s effects (e.g., humiliation if the attack were to happen in public view). In addition, patients manifest

2351

TABLE 397-8  COMMON SOMATIC MANIFESTATIONS OF ANXIETY CARDIORESPIRATORY Palpitations Chest pain Dyspnea or sensation of being smothered GASTROINTESTINAL Sensation of choking Dyspepsia Nausea Diarrhea Abdominal bloating or pain GENITOURINARY Urinary frequency or urgency NEUROLOGIC/AUTONOMIC Diaphoresis Warm flushes or chills Dizziness or presyncope Paresthesias Tremor Headache

avoidance behavior by staying away from known triggers or from situations in which having a panic attack might be dangerous (e.g., driving) or particularly distressing (e.g., in public spaces). For many patients, the anticipatory anxiety and avoidance behavior may be more disabling than the panic attacks themselves. Avoidance behavior may overlap with agoraphobia, which is defined as a distressing and disabling fear of places or situations from which escape might be difficult or embarrassing or from which help might not be available in the event of panic-like symptoms. Common agoraphobic foci include being outside one’s home alone, being on bridges or in tunnels, traveling by vehicle, or being in crowds or lines. A third or more of patients with panic disorder have comorbid agoraphobia, whereas others have agoraphobia alone or comorbid with other conditions.

Generalized Anxiety Disorder

This more heterogeneous condition is defined by the presence of clinically significant anxiety and associated somatic symptoms for 6 or more months. Generalized anxiety disorder is often “trumped” in the diagnostic hierarchy by other conditions that produce anxiety.

Phobias

The phobias are a group of conditions defined by the consistent ability of a specific environmental stimulus to elicit a pathologic anxiety response. Exposure to such a stimulus nearly always produces this response, so the patient avoids the stimulus whenever possible or endures the stimulus with considerable distress. In addition to agoraphobia, the other main types of phobias are social phobia (social anxiety disorder) and specific phobias (see Table 397-7).

DIAGNOSIS

Diagnosis of anxiety disorders must rest on consideration of both syndromic and etiologic perspectives.9 From a syndromic perspective, a careful history and mental status examination are required to determine the pattern of anxiety and associated symptoms and to determine whether the phenomenology fits the pattern for any of the anxiety disorders as described earlier. The history and mental status examination must also assess for the presence of any other psychiatric disorder that might truly be comorbid with the anxiety disorder but might also supersede the anxiety disorder in the diagnostic hierarchy. For example, generalized anxiety may be seen as part of neurocognitive disorders (delirium or dementia), depressive or bipolar disorders, and psychotic disorders. From an etiologic perspective, it is important to determine whether the anxiety disorder is primary (idiopathic) or secondary to a systemic or neurologic condition (see Table 397-1), drug intoxication, or withdrawal state. The evaluation should include laboratory tests (e.g., toxic drug screen) as guided by the differential diagnosis generated from the clinical evaluation.

2352

CHAPTER 397  Psychiatric Disorders in Medical Practice  

TABLE 397-9  SELECTED ANTIANXIETY AND HYPNOTIC DRUGS* DRUG

TRADE NAME

INITIAL DOSE

TARGET DOSE RANGE†

SIDE EFFECTS

Benzodiazepines   Lorazepam   Diazepam   Triazolam   Chlordiazepoxide   Temazepam   Alprazolam   Clorazepate   Flurazepam   Oxazepam   Clonazepam   Zaleplon   Zolpidem   Eszopiclone

Sedation, ataxia, risk for falls

Ativan Valium Halcion Librium Restoril Xanax Tranxene Dalmane Serax Klonopin Sonata Ambien Lunesta

0.5 mg bid-qid 2-5 mg bid-tid 0.125 mg qhs 5 mg bid-tid 7.5 mg qhs 0.25 mg tid-qid 7.5-15 mg bid-tid 15-30 mg qhs 10-15 mg tid-qid 0.5 mg bid-tid 5-10 mg qhs 5-10 mg qhs 1-2 mg qhs

2-6 mg/day, tid-qid dosing 10-40 mg/day, bid-tid dosing 0.125-0.25 mg qhs 10-40 mg/day, bid-tid dosing 7.5-30 mg qhs 2-8 mg/day, tid-qid dosing 15-60 mg/day, bid-tid dosing 15-30 mg qhs 10-30 mg tid-qid 0.5-5 mg bid-tid 5-20 mg qhs 5-10 mg qhs 1-3 mg qhs

β-Blockers   Propranolol

Inderal

20 mg bid

Individualize, 40-120 mg/day Bradycardia, hypotension, potential for mental slowing

Rebound insomnia Possibly greater addictive potential Daytime somnolence

COMMENTS Potential for abuse/dependence Reliable IM absorption Long half-life of drug and active metabolites Used as hypnotic Long half-life of drug and active metabolites Used as hypnotic Used as hypnotic Long duration of action “Nonbenzodiazepine” hypnotic “Nonbenzodiazepine” hypnotic “Nonbenzodiazepine” hypnotic Only helps with sympathetically mediated somatic symptoms of anxiety

*Antidepressants (see Table 397-5) are often first-line agents of choice for primary anxiety disorders. † Target doses in the elderly may be lower. qhs = at bedtime.

TREATMENT  Empirical evidence from controlled trials demonstrates the efficacy of cognitive-behavioral psychotherapies for most of the anxiety disorders. A9  Such therapies, which use the principles of learning theory to extinguish unhelpful behavior and positively reinforce more functional behavior, help the patient learn to identify and correct the dysfunctional patterns of thinking (“automatic thoughts”) that underlie or trigger the cognitive-physiologic cascade of pathologic anxiety responses. Cognitive behavioral therapy may be used as sole therapy, particularly for specific phobias, or in combination with pharmacotherapy. Frequently, cognitive behavioral therapy may be administered as part of family therapy (e.g., to help family members avoid behavior that inadvertently reinforces the patient’s symptoms) or in group therapy settings. Although anxiolytic drugs such as the benzodiazepines (Table 397-9) will usually relieve acute anxiety symptoms, concerns about their long-term efficacy and side effects (e.g., risk for abuse, risk for neurocognitive impairment or falls) make antidepressant medications the more attractive pharmacologic agents for most anxiety disorders (see Table 397-5). Most antidepressants, with the probable exception of bupropion, are helpful for panic disorder, generalized anxiety disorder, and social phobia.

PROGNOSIS

In general, most persons with ongoing anxiety disorders tend to have a chronic course of waxing and waning symptoms. Maintenance therapies should often be used for patients with more chronic anxiety disorders, although evidence to support long-term therapies is not as robust as for mood and psychotic disorders.

Obsessive-Compulsive Disorder

The fifth edition of the Diagnostic and Statistical Manual of Mental Disorders (DSM-5) has created a new category of “Obsessive-Compulsive and Related Disorders” in recognition that obsessive-compulsive disorder (OCD) has a distinct pathogenesis from other anxiety disorders. OCD is likely related to other conditions such as body dysmorphic disorder, hoarding disorder, trichotillomania (hair-pulling), and excoriation (skin-picking) disorder. Patients with OCD have recurrent obsessions or compulsions (Table 39710), and most patients have both.10 OCD should not be confused with obsessive-compulsive personality traits or disorder, described later under “Personality Disorders.” Obsessions, not to be confused with obsessing (ruminating) on a topic, are recurrent, persistent, and typically distressing thoughts that at some point during the course of the disorder are experienced as intrusive and unwanted. The latter quality may be described in language such as “I don’t know where this thought comes from” or “I don’t know why I have this thought, I would never actually do such a thing!” Compulsions are repetitive behaviors or mental acts the individual feels

TABLE 397-10  COMMON TYPES OF OBSESSIONS AND COMPULSIONS IN OBSESSIVE-COMPULSIVE DISORDER OBSESSIONS Aggressive (fears of harming self or others, of blurting out obscenities, or of other unwanted aggressive acts; unwanted violent or horrific images) Contamination (concerns about dirt, germs, body waste or secretions, environmental contaminants, or animals/insects) Sexual (concerns about unwanted sexual images or impulses) Hoarding/saving Religious (scrupulosity) (excessive concerns about sacrilege, blasphemy, right/wrong, morality) Need for symmetry/exactness Somatic (excessive concern about illness, body part, or appearance) COMPULSIONS Cleaning/washing (excessive or ritualized handwashing, showering, or other grooming) Checking (checking locks, stove, appliances; checking body in relation to somatic obsessions; checking that did not or will not harm self or others) Repeating rituals (rereading or rewriting; routine activities such as going through a door or arising from a chair) Counting Ordering/arranging Hoarding/saving Adapted from Goodman WK, Price LH, Rasmussen SA, et al. The Yale-Brown Obsessive Compulsive Scale. I. Development, use, and reliability. Arch Gen Psychiatry. 1989;46:1006-1011.

driven to perform in response to an obsession or according to rigid rules. For example, compulsive handwashing may relate to obsessional thoughts about germs or contamination. Patients with OCD typically attempt to ignore, suppress, or neutralize their obsessions, but doing so causes great psychic distress. OCD patients may spend many hours per day related to their obsessions and compulsions. The 12-month prevalence of OCD is approximately 1%, with onset typically in childhood, adolescence, or young adulthood. Remission rates are low in adults, with most persons experiencing a chronic waxing and waning course. Pathogenesis probably involves altered functioning of the striatofrontal systems, as well as a prominent role for central serotonergic systems. Obsessions and compulsions may represent inappropriate triggering of neural “scripts” involving thoughts and behaviors that have been analogized to the scripts involved in animal grooming and other complex but stereotypical behaviors. The only efficacious antidepressants in OCD are those with strong activity on serotonergic systems, such as the selective serotonin reuptake inhibitors and the tricyclic compound clomipramine. Cognitive-behavioral therapies

CHAPTER 397  Psychiatric Disorders in Medical Practice  

also have well-demonstrated efficacy. Deep brain stimulation (DBS)11 targeting the ventral capsule/ventral striatum is FDA approved (as a humanitarian device exemption) for severe treatment-refractory OCD. Although early studies are relatively encouraging, the role of DBS in clinical practice remains to be defined.

Acute Stress Disorder and Post-traumatic Stress Disorder

Acute stress disorder and post-traumatic stress disorder (PTSD) are specific manifestations of symptoms referable to an extremely traumatic event. The event by definition must involve exposure to actual or threatened death, serious injury, or sexual violence, as reported directly by the patient or by family members or friends. Patients suffer from repeated or extreme exposure to aversive details of the event. It is important to recognize that acute stress disorder or PTSD does not develop in all individuals exposed to a common traumatic event (e.g., a natural or man-made disaster). Some individuals may instead develop other anxiety disorders, major depression, mania, or psychosis, and diagnosable psychopathology may never develop at all in some or many others. PTSD symptoms by definition persist for more than 1 month after the traumatic event and include the following types of clinical phenomena: (1) intrusion, such as intrusive memories, dreams, flashbacks, or intensely distressing psychological or physiologic responses to reminders of the trauma; (2) avoidance of distressing memories or external reminders of the trauma; (3) negative cognitions and mood, such as amnesia for aspects of the event, negativistic thoughts about oneself in general or blame related to the event, persistent negative emotions, diminished interests or activities, or feelings of detachment; and (4) alterations in arousal and reactivity. Acute stress disorder by definition resolves in less than 1 month, with symptoms of intrusion, avoidance, or arousal as well as negative mood or dissociative symptoms (e.g., “in a daze”). The 12-month prevalence of PTSD in the United States is about 3%, with projected lifetime risk approaching 9%. About half of adults with PTSD have complete recovery within 3 months, but PTSD may persist for many months or years. Both cognitive-behavioral and psychodynamic psychology perspectives are useful in informing psychotherapeutic treatments.12 Antidepressants also have demonstrated efficacy in PTSD.

  PSYCHOTIC DISORDERS

Psychotic symptoms, defined as a loss of reality testing, include delusions (fixed false beliefs), hallucinations (false sensory perceptions), and major derailments in thought processes (e.g., loose associations). Psychotic symptoms may be seen in the course of neurocognitive, secondary, and mood disorders. The psychotic disorders are defined by the presence of psychotic symptoms in the absence of prominent mood disturbance, or of neurocognitive deficits at the level seen in delirium or dementia. In general, the diagnosis and treatment of patients with psychotic disorders should be conducted in mental health specialty settings, but primary care settings are common points of entry to care.

Schizophrenia

DEFINITION AND EPIDEMIOLOGY

Schizophrenia, the prototypical psychotic disorder, necessarily includes symptoms of psychosis (“positive” symptoms) and also often includes “negative symptoms” such as affective flattening, abulia, apathy, and social withdrawal. The level of functioning is impaired in one or more realms (e.g., occupational, interpersonal, or self-care). The lifetime prevalence of schizophrenia is slightly less than 1%, and its chronic debilitating course takes a considerable toll on patients, families, and society. Peak onset is in late adolescence to young adulthood, slightly younger for males than females. The annual incidence is approximately 15 per 100,000, but with marked variability across study samples and populations. When narrowly defined as above, the condition is slightly more common in males than in females.

PATHOBIOLOGY

The pathogenesis of schizophrenia remains unknown. Twin studies show that the disease is multifactorial. Genetic factors account for up to 50% of the risk, and multiple gene loci appear to be involved. Studies of postmortem brains indicate a nongliotic neuropathologic process with subtle disruptions of cortical cytoarchitecture. It is likely that psychosocial factors and neurodevelopment interact with a nonlocalizable brain “lesion” that is either present at birth or acquired early in life. The dopaminergic mesocortical and mesolimbic pathways are important in the production of psychotic symptoms.

2353

TABLE 397-11  SYMPTOMS AND SIGNS OF MAJOR PSYCHOTIC DISORDERS SCHIZOPHRENIA Delusions Hallucinations Disorganized speech (i.e., thought process derailments) Grossly disorganized or catatonic behavior Negative symptoms: affective flattening, alogia, avolition Major impairment in social or occupational functioning Duration of at least 6 months SCHIZOAFFECTIVE DISORDER During the course of illness, at least one episode of schizophrenia-like psychotic symptoms PLUS a mood syndrome (either major depression or mania) AND During the course of illness, at least 2 weeks of schizophrenia-like psychotic symptoms in the absence of a mood syndrome DELUSIONAL DISORDER One or more delusions for at least 1 month, most often nonbizarre (i.e., potentially plausible, such as delusions of being followed, poisoned, infected, loved at a distance, deceived by a spouse or lover, or having a disease) Not meeting full criteria for an acute episode of schizophrenia Functioning not markedly impaired other than as related to the impact of the delusion(s) and its ramifications Based on criteria from American Psychiatric Association. Diagnostic and Statistical Manual of Mental Disorders. 5th ed. (DSM-5) Washington, DC: American Psychiatric Association; 2013.

DIAGNOSIS

The diagnosis of schizophrenia is based on the presence of delusions, hallucinations, and disorganized speech and behavior, often accompanied by apathy and social withdrawal and resulting in major impairment in functioning for at least 6 months (Table 397-11). In patients with single schizophrenialike psychotic episodes of briefer duration, with subsequent return to asymptomatic baseline functioning, brief psychotic disorder ( 15 days per month). In many patients headaches remain episodic, but about 25% progress to chronic

2360

CHAPTER 398  Headaches and Other Head Pain  

headache. Of the patients with chronic tension-type headache, about a quarter to a third continue as chronic, half can improve to episodic, and in about a quarter medication overuse headache can develop. Episodic tensiontype headaches can last minutes, hours, or days.

DIAGNOSIS

Headaches that can be misdiagnosed as tension-type headache include migraine, hemicrania continua, new daily persistent headache, and headaches caused by brain tumors, elevated or low intracranial pressure, or giant cell arteritis. A careful history is the best way to distinguish other types of headaches.

TREATMENT  Episodic tension-type headaches are generally treated successfully7 with acetaminophen (650 to 1000 mg) or NSAIDs (aspirin, 900 to 1000 mg; naproxen, 250 to 500 mg; ibuprofen, 200 to 800 mg; or ketoprofen, 12.5 to 75 mg). However, analgesic use for more than 3 days per week can worsen headaches and lead to medication-induced headache.

PREVENTION

Chronic tension-type headaches may benefit from prophylactic treatment with amitriptyline (starting with 10 mg at bedtime and increased slowly up to 100 mg until the patient improves or intolerable side effects develop), nortriptyline (25 to 100 mg each evening), doxepin (25 to 75 mg/day), maprotiline (10 to 25/mg/day), or fluoxetine (10 to 20 mg/day). Tricyclics are generally more efficacious than serotonin reuptake inhibitors. A16  Muscle relaxants, physical therapy, localized botulinum toxin injection, and acupuncture can be useful. A17 

PROGNOSIS

Tension-type headache has a variable prognosis. Adolescents with tensiontype headache and two or more psychiatric factors (e.g., depression and anxiety) have a worse prognosis.

  CLUSTER HEADACHE AND OTHER TRIGEMINAL AUTONOMIC CEPHALALGIAS DEFINITION

Trigeminal autonomic cephalalgias, including cluster headaches, are unilateral headaches associated with ipsilateral autonomic features. Other trigeminal autonomic cephalalgias include paroxysmal hemicrania, which is characterized by bouts of headache that persist for 5 to 30 minutes, is generally unilateral, and usually occurs in women; they typically respond to indomethacin. Hemicrania continua, another indomethacin-responsive headache seen in both men and women, is characterized by continuous unilateral pain

and mild associated autonomic features; it frequently coexists with a form of chronic daily headache. Short unilateral neuralgiform headache with conjunctival injection and tearing is a rare trigeminal autonomic cephalalgia that occurs in men; individual headaches persist for only a short time (seconds to 2 minutes).

EPIDEMIOLOGY

Cluster headache occurs in 56 to 401 per 100,000 persons and is more frequent in men (3 : 1 to 7 : 1). Attacks usually begin between 20 and 30 years of age. Paroxysmal hemicrania occurs in 56 to 381 per 100,000 persons; it affects women more often (2 : 1) and can begin at any age but usually commences at 34 to 41 years. Short unilateral neuralgiform headache with conjunctival injection and tearing is rare, with a slight male preponderance (2 : 1).

PATHOBIOLOGY

Cluster headache may have a genetic predisposition. Imaging studies such as positron emission tomography and functional MRI show inferior posterior hypothalamic activation at the onset of cluster headache and other trigeminal autonomic cephalalgias. In addition, the trigeminovascular complex and the cranial autonomic system are activated. The pathophysiology of hemicrania continua is unknown, and there is debate whether it is associated with hypothalamic involvement or whether it resembles migraine.

CLINICAL MANIFESTATIONS

Cluster headache is almost always unilateral, rarely bilateral, and has characteristic ipsilateral autonomic features, commonly including lacrimation and conjunctival injection and occasionally nasal congestion, rhinorrhea, ptosis, miosis, flushing, and eyelid edema (Table 398-5). The location of the pain is usually behind or above the eye or in the temple but can include the forehead, cheek, teeth, or jaw. The pain reaches its maximum intensity in about 9 minutes and tends to end abruptly. Attacks occur one to eight times a day and are usually described as “boring” or “stabbing” excruciating pain that persists for 15 minutes to 2 hours. Migraine symptoms may coexist, including unilateral photophobia, phonophobia, and rarely, an aura. Unlike migraine patients, who usually try to rest, patients with cluster headaches pace and are unable to sit or lie down. Cluster headaches, often precipitated by alcohol, histamine, or nitroglycerin, have a daily periodicity and may also have a seasonal periodicity. For example, episodic cluster headache may occur annually or every 2 years, often in the same season each time. Chronic cluster headache occurs without a remission. Paroxysmal hemicrania is pain of short duration, usually 2 to 30 minutes, and occurs unilaterally around the eye, temple, or maxillary region, sometimes precipitated by head movements. Autonomic features similar to cluster headache can occur. The usual attack rate is up to 40 episodes each day. Bouts of pain may be episodic, separated by a remission, but most patients have daily chronic paroxysmal hemicrania without a remission. Short unilateral neuralgiform headache with conjunctival injection and tearing attacks are unilateral and consistently on the same side. Although the

TABLE 398-5  DISTINGUISHING CHARACTERISTICS OF THE TRIGEMINAL AUTONOMIC CEPHALALGIAS

CHARACTERISTIC Sex—F:M

PAROXYSMAL HEMICRANIA

CLUSTER 1 : 3-7

HEMICRANIA CONTINUA

2 : 1

2 : 1 +

Unilateral

+

+

Attack frequency

1-8/day

1-40/day

Attack duration

15-80 min

2-30 min

SHORT UNILATERAL NEURALGIFORM HEADACHE WITH CONJUNCTIVAL INJECTION AND TEARING 1 : 2 + 3-200/day

Continuous with episodic exacerbations

5-240 sec

Autonomic features

+

+

+ with exacerbations

+

Indomethacin effect



+++

+++



Acute treatment at onset

Oxygen, sumatriptan SC, DHE nasal spray; sumatriptan or zolmitriptan nasal spray (A-level evidence)

None

None

None

Preventive medications

Verapamil, lithium, corticosteroids, anticonvulsants (A level)

Indomethacin (A level)

Indomethacin (A level)

Lamotrigine, topiramate, gabapentin (B level)

DHE = dihydroergotamine; SC = subcutaneous.

CHAPTER 398  Headaches and Other Head Pain  

pain is excruciating, the attack is brief, usually seconds; most patients are free of pain between attacks, although a dull ache can be present. Associated autonomic features include ipsilateral conjunctival injection and tearing.

DIAGNOSIS

The diagnostic criteria for cluster headache include severe unilateral orbital, supraorbital, or temporal pain persisting for 15 to 180 minutes with at least one of the following: ipsilateral conjunctival injection or lacrimation, nasal congestion or rhinorrhea, eyelid edema, forehead and facial sweating, miosis with or without ptosis, and restlessness or agitation. Attacks occur between once and as often as eight times each day. There is no other cause of the disorder. Paroxysmal hemicrania is defined by unilateral pain persisting for 2 to 30 minutes, about 5 times each day, with one or more autonomic features such as conjunctival injection, nasal congestion, eyelid edema, forehead and facial sweating, and miosis or ptosis (or both). Complete prevention may be achieved with indomethacin. Hemicrania continua is a unilateral headache that occurs daily and continuously without pain-free periods; its intensity is moderate, with exacerbations of severe pain. During the exacerbations, at least one ipsilateral autonomic feature is present: conjunctival redness, lacrimation, nasal congestion, ptosis, or miosis. It responds to indomethacin. Short unilateral neuralgiform headache with conjunctival injection and tearing is diagnosed by unilateral orbital, supraorbital, temporal stabbing pain persisting for 5 to 240 seconds at a frequency of 3 to 200 per day. It is associated with conjunctival injection and tearing. An imaging procedure such as MRI is indicated for all patients at the onset of cluster headaches or other trigeminal autonomic cephalalgias, because they can be the result of infection (Chapters 412 to 414), vascular malformation (Chapter 408), or neoplasm, especially a pituitary tumor (Chapter 189). Other possibilities in the differential diagnosis include migraine, hypnic headache (rare short-lasting headaches exclusively during sleep in the elderly), and trigeminal neuralgia.

TREATMENT  Because the course of the headache is brief, oral medications take too long to work to be effective.8 The use of 100% oxygen at 7 to 10 L/min for 15 to 30 minutes benefits some patients. A18  Sumatriptan or zolmitriptan nasal spray or sumatriptan subcutaneously (4 to 6 mg) can be helpful. A19  Dihydroergotamine can be helpful when given nasally, intramuscularly, or even intravenously. Refractory cases may respond to occipital nerve stimulation. Chronic paroxysmal hemicranias and hemicrania continua are characterized by a response to indomethacin, 25 to 50 mg three times daily. Short unilateral neuralgiform headache with conjunctival injection and tearing attacks is so brief that there are no medications to treat it acutely.

PREVENTION

Preventive medications should be started at the beginning of a cluster bout. Verapamil, 240 to 480 mg, is the drug of choice. Lithium (300 mg twice daily) is an alternative. Corticosteroids (e.g., prednisone, 40 mg/day, or dexamethasone, 4 mg twice daily for 2 weeks) act rapidly as a bridge to prevent cluster headache while other preventive medications are started. Valproic acid (500 to 1500 mg/day in divided doses), topiramate (50 to 100 mg/day), melatonin (4 mg at bedtime), and gabapentin (300 mg three times daily) are sometimes beneficial. Surgical approaches, including suboccipital steroid injections, A20  occipital nerve stimulators, sphenopalatine ganglion stimulation, hypothalamic stimulation, and destructive procedures, are sometimes necessary for this disabling headache. Paroxysmal hemicrania and hemicrania continua respond to daily indomethacin (25 to 50 mg three times daily). If the patient cannot tolerate indomethacin, calcium-channel blockers (e.g., verapamil, 240 to 480 mg/ day) or melatonin may be helpful. Preventive treatment of short unilateral neuralgiform headache with conjunctival injection and tearing includes lamotrigine (100 to 400 mg/day), topiramate (50 to 100 mg), gabapentin (300 to 900 mg), or IV lidocaine (starting at 2 mg/minute with cardiac monitoring).9 Short unilateral neuralgiform headache with conjunctival injection and tearing is regarded as a more difficult headache to prevent. Lamotrigine and topiramate may be helpful.

2361

PROGNOSIS

Cluster headache is often a lifelong problem, but remissions may persist for longer periods as the patient ages. The other trigeminal autonomic cephalalgias are probably lifelong; nevertheless, symptomatic treatment combined with preventive medications is helpful.

  CHRONIC DAILY HEADACHE DEFINITION

Though not a specific disorder, chronic daily headache, defined as a headache that is present on more than 15 days per month, is challenging for both patients and physicians. These headaches may be chronic migraine, chronic tension-type headache, new daily persistent headache, or chronic cluster headache, with or without overuse of medications.

EPIDEMIOLOGY

Up to 5% of the population suffers from chronic daily headache, most commonly chronic tension type or chronic migraine. Trigger factors such as a previous infection, mild head injury, or stressful life event are present in 40 to 60% of patients with new daily persistent headache. Risk factors for chronic daily headache include medication overuse, history of migraine headache, frequent headache, depression, female sex, obesity, snoring, stressful life events, and low educational level.

PATHOBIOLOGY

Chronic daily headache is probably related to migraine, with both central and peripheral abnormalities. Once migraine has been prolonged and headache occurs on a daily basis, allodynia, a sense that a usually nonpainful stimulus is becoming painful, often develops. Use of an opiate for more than 8 days per month, especially in men, use of barbiturates for more than 5 days per month, especially in women, or use of triptans for more than 10 to 14 days per month can often lead to chronic migraine headache or at least worsening of headaches.

CLINICAL MANIFESTATIONS

New daily persistent headache is characterized by daily occurrence, onset at specific time, and an unrelenting course. It is generally bilateral, nonpulsating, mild to moderate, and associated with features of migraine, photophobia, phonophobia, or nausea. Severe nausea or vomiting is rare. New daily persistent headache can be disabling and is difficult to treat. Chronic daily headache is often associated with profound psychiatric comorbidity, especially depression and anxiety; such psychiatric comorbidity predicts intractability.

DIAGNOSIS

Diagnosis of chronic daily headache is based on the history. It is important to identify the underlying type of primary chronic daily headache: chronic migraine, chronic tension-type headache, new daily persistent headache, or hemicrania continua. Headaches of less than 4 hours’ duration can also be chronic and daily: cluster headache, paroxysmal hemicrania, hypnic headaches occurring every night (usually in the elderly), and episodic stabbing headache. It is most important to exclude secondary headaches (including post-traumatic headache), headaches associated with vascular disorders (e.g., giant cell arteritis, arteriovenous malformations, carotid and vertebral artery dissections), and headaches associated with nonvascular disorders (e.g., intracranial hypertension, intracranial hypotension, infections). MRI and laboratory studies (e.g., ESR in an elderly individual) are commonly recommended. Lumbar puncture (LP) to assess intracranial pressure may also be indicated in selected patients.

TREATMENT  The most common cause of chronic daily headache is overuse of medications, so patients must be weaned off the overused symptomatic medication. Treatment of underlying depression, anxiety, and pain may also be helpful. Occasionally, hospital admission is necessary to break the headache cycle. Acute migraine-specific treatments (see earlier), especially IV dihydroergotamine (0.5 to 2 mg), are helpful in terminating migrainous attacks.

2362

CHAPTER 398  Headaches and Other Head Pain  

PREVENTION

Medications that are helpful in preventing chronic daily headache include tricyclic antidepressants, selective serotonin reuptake inhibitors if patients are depressed, anticonvulsants, β-blockers, and calcium-channel blockers (see Tables 398-3 and 398-4). For hemicrania continua, indomethacin (25 to 50 mg three times daily) is the preferred treatment.

PROGNOSIS

The prognosis depends on the underlying headache diagnosis. If medication overuse is the cause and the patient is successfully detoxified, about 75% of patients improve when treated with preventive medications. Treatment may fail if the diagnosis is incorrect or because of continued overuse of medications, overuse of caffeine, lack of sleep, dietary or other life triggers, hormonal factors, or psychiatric factors. Explaining medication overuse headache to the patient, in-patient and out-patient detoxification, and multidisciplinary care treatments have been found helpful.

  SECONDARY CAUSES OF HEADACHES

Sinus Headache

Rhinosinusitis (Chapter 426) is characterized by inflammation or infection of the nasal mucosa and sinuses. The sinuses themselves are relatively insensate, but ducts, turbinates, blood vessels, and ostia are the painful structures.10 Headaches attributed to rhinosinusitis are frontal headaches with pain in the face, ears, or teeth. The onset of pain is simultaneous with the rhinosinusitis, and the headache and face pain resolve within 7 days after successful treatment. The diagnosis requires imaging and clinical evidence that support the diagnosis of acute rhinosinusitis. Many acute and most chronic headaches that are initially thought to result from sinus disease are found to be migraine or tension-type headache. The headache should resolve with treatment of acute sinusitis (Chapter 426). If it does not, an underlying primary headache disorder is likely.

Temporal (Giant Cell) Arteritis Temporal arteritis (Chapter 271) is an inflammatory process seen almost exclusively in elderly individuals. Headache, especially pain in the jaw when chewing, is one of the most common features. Its incidence is approximately 12 per 100,000 and increases with age to 51 per 100,000 in individuals older than 80. It affects women more often than men (3 : 1) and is more common in white individuals, especially those of Scandinavian and British descent. It is associated with polymyalgia rheumatica. The headache has no specific feature, but the pain is usually continuous, generalized, and occasionally throbbing. The temples are generally painful, and patients complain of pain when performing certain activities of daily living, such as chewing food or combing their hair. Transient monocular blindness, permanent blindness, and diplopia can occur. Elevation of the ESR and C-reactive protein occurs almost invariably. The diagnosis is made by finding giant cells in a temporal artery biopsy specimen. Immediate treatment with corticosteroids, sometimes before the biopsy result is available, is necessary in doses between 40 and 80 mg daily, with the dose then titrated downward while monitoring the ESR or CRP. Used early enough, corticosteroids (Chapter 271) generally prevent the complications of temporal arteritis, including blindness. The disorder can be long lasting.

Intracranial Hypertension and Pseudotumor Cerebri Intracranial hypertension can be primary and idiopathic or secondary to cerebral venous thrombosis (Chapter 407), a mass in the brain (Chapter 189), hydrocephalus, or other intracranial processes. Pseudotumor cerebri is an all-encompassing term referring to increased intracranial pressure without obvious mass lesions.11 Primary idiopathic intracranial hypertension occurs in obese women of childbearing age. Secondary pseudotumor cerebri causes a similar syndrome but is due to an offending agent such as medications (e.g., tetracycline, minocycline, lithium, vitamin A–related medications, growth hormone), endocrine disorders (e.g., parathyroid dysfunction), and sleep apnea. Idiopathic increased intracranial pressure occurs in 1 to 2 per 100,000 individuals but in 19 to 20 per 100,000 individuals (15 to 55 years of age) who are obese. Women are affected more frequently than men (6-8 : 1). Onset is usually in young adulthood.

The cause of the increased pressure is either poor CSF absorption, as is thought to be the problem in idiopathic intracranial hypertension; venous hypertension, as is seen in venous thrombosis; or a mass that causes an increase in pressure. A genetic component is also likely because there are reports of the condition occurring in families.

CLINICAL MANIFESTATIONS

Idiopathic intracranial hypertension is characterized by headache in more than 90% of individuals, about 90% of whom are obese. The headache may be pulsatile and is frequently felt behind the eyes. Patients often report neck pain, upper back pain, or even radicular pain. The intensity of headache does not correlate with the height of the intracranial pressure. Pulse-synchronous tinnitus is a frequent accompaniment, as are transient visual obscurations and diplopia. On examination, papilledema (Chapter 423, Fig. 423-27) may be found. The remainder of the general and neurologic examination is usually normal in patients with idiopathic intracranial hypertension, but abnormalities on examination may point to a secondary cause, such as underlying venous sinus thrombosis (Chapter 407), ischemic stroke, central nervous system infection (Chapters 412 and 413), or brain tumor (Chapter 189). Although idiopathic intracranial hypertension often persists for years, the condition can be selflimited. In about a third of patients, there are permanent visual sequelae related to the effect of papilledema.

DIAGNOSIS

The diagnosis of intracranial pressure is made by the symptoms and signs such as papilledema (Chapter 423, Fig. 423-27). MRI is necessary to exclude secondary causes of increased intracranial pressure. MR or CT venography is often needed to exclude venous sinus thrombosis (Chapter 407). LP must be performed unless patients have a contraindication such as an intracranial mass lesion, and CSF pressure should be measured. The diagnosis can be made if the pressure is elevated (CSF > 250 mm H2O) and the fluid itself is normal in terms of its protein level, glucose level, and cell count. Visual fields must be examined formally because visual acuity is not affected until late in the course of the disorder.

TREATMENT  Acetazolamide (doses ranging from 500 to 4000 mg daily) combined with a weight loss program is more efficacious for individuals with idiopathic intracranial hypertension and mild to moderate visual loss than is placebo. A21  Any underlying secondary cause should also be treated (e.g., stopping an offending medication, treatment of sleep apnea [Chapter 100]). Weight loss is beneficial in obese subjects. If visual loss progresses, surgical procedures should be considered. Optic nerve sheath fenestration allows CSF to escape through slits or windows in the orbit; sometimes the treatment of one side decreases the optic disc swelling on the other side as well. Complications include visual loss or diplopia, so visual fields must be followed carefully to anticipate and prevent visual loss. Lumbar or ventricular peritoneal diversion procedures also reduce intracranial pressure, but their complications include infection and shunt obstruction.

PROGNOSIS

The prognosis of patients with idiopathic intracranial hypertension is good with treatment, but up to a third of inadequately treated patients can experience permanent defects of visual fields or loss of visual acuity. Individuals are susceptible to recurrence if they suddenly gain weight.

Intracranial Hypotension Intracranial hypotension (or CSF hypovolemia) causes a headache that is characteristically better when the patient is supine and worse when the patient is upright. It can be primary (spontaneous) or secondary to another underlying cause, most commonly a previous LP.12 Intracranial hypotension was once considered rare, but modern imaging techniques suggest an incidence of about 5 per 100,000 per year; it is slightly more common in women than men. The onset is usually at about 40 years of age, but it can occur in children and the elderly. Post-LP headaches occur more commonly but only infrequently persist.

CHAPTER 398  Headaches and Other Head Pain  

PATHOBIOLOGY

The cause of primary intracranial hypotension is thought to be a small leak or tear in the dura, usually in the lumbar region around cystic structures called Tarlov cysts. The cause of intracranial hypotension may not be the tear itself but rather the low CSF volume and low epidural venous pressure that assists in development of the lower pressure and hence the leak. The leaks frequently occur in the thoracic and cervicothoracic junction spine. Previous trauma history is reported in only one third of cases. Genetic and connective tissue disorders (e.g., Ehlers-Danlos syndrome, Marfan syndrome [Chapter 260]) may predispose individuals to have these leaks.

CLINICAL MANIFESTATIONS

Intracranial hypotension is characterized clinically by a positional headache. The location of the pain is variable, and the most constant characteristic is the orthostatic change in the pain. If the leak is untreated for a long time, the headache may lose the orthostatic characteristic. Posterior neck pain can also occur. Changes in hearing, taste, and balance, as well as blurred vision and diplopia, can develop if hindbrain herniation occurs. If very severe hindbrain herniation occurs, changes in consciousness, subdural hygromas, ataxia, a pseudo-frontotemporal dementia can occur.

DIAGNOSIS

The diagnosis of intracranial hypotension is made by MRI showing pachymeningeal enhancement, venous engorgement, dural thickening, pituitary fossa enlargement, and herniation of the hindbrain (Fig. 398-1). Hindbrain herniation appears as a downward descent of the posterior fossa along with loss of the prechiasmatic cistern, flattening of the pons against the clivus, and descent of the cerebellar tonsils, which is often misconstrued as a Chiari I malformation. LP may also show low ( 1 m or 5 stairs)§

100

69

98

50

Headache, vomiting, seizure, intoxication, short-term memory deficit, age > 60 yr, or injury above the clavicles

100

25

GCS score of 15, loss of consciousness, no neurologic deficit, no seizure, no anticoagulation, age ≥ 3 yr

*Validity for identifying patients with traumatic CT findings. † Stiell IG, Wells GA, Vandemheen K, et al. The Canadian CT Head Rule for patients with minor head injury. Lancet. 2001;357:1391-1396. ‡ High-risk patients in whom a CT scan is mandatory. § Medium-risk patients in whom a CT scan is recommended but close clinical observation is an alternative. ¶ Haydel MJ, Preston CA, Mills TJ, et al. Indications for computed tomography in patients with minor head injury. N Engl J Med. 2000;343:100-105. CT = computed tomography; GCS = Glasgow Coma Scale.

TABLE 399-4  AMERICAN ACADEMY OF NEUROLOGY: DIAGNOSIS AND MANAGEMENT OF CONCUSSION CRITICAL STEPS

SUPPORTING INFORMATION

Immediately remove from play or work

Adherence to state concussion laws

First responders should use a validated clinical tool to determine risk of concussion

(See Table 399-1)

Diagnosis and clinical care is made by LHCP

Clinical practice guidelines for treatment of symptoms

Return to play or work only after detailed evaluation and written authorization by LHCP

Graded physical activity that does not exacerbate symptoms Cognitive restructuring through education, reassurance, reattribution of symptoms

Retirement from activity counseling by LHCP

Retirement counseling for patients with history of multiple concussions with subjective neurobehavioral symptoms should begin a discussion with the patient about retirement

LHCP = Licensed health care provider, an individual who has acquired knowledge and skills relevant to evaluation and management of sports concussions and is practicing within the scope of his or her training and experience. Adapted from Giza CC, Kutcher JS, Ashwal S, et al. Summary of evidence-based guideline update: evaluation and management of concussion in sports: report of the Guideline Development Subcommittee of the American Academy of Neurology. Neurology. 2013;80:2250-2257.

eyes”) and postauricular ecchymosis (“Battle sign”) suggest a basal skull fracture. A clear or blood-tinged watery discharge from the nose or ear may be a cerebrospinal fluid leak. Intracranial bleeding caused by traumatic brain injury includes subdural hematoma, epidural hematoma, intraparenchymal hemorrhage, contusion, and traumatic subarachnoid hemorrhage (Chapter 408). The most common is subdural hematoma, which is the basis of approximately 50% of admissions for head injury. Epidural hematoma accounts for about 3%. An associated skull fracture, especially at the temporoparietal junction, increases the incidence of epidural hematoma, usually by disruption of the middle meningeal artery.

Imaging

A computed tomography (CT) scan without contrast should be obtained as soon as possible after the initial clinical assessment. The need for neuroimaging is best determined by using the Glasgow Coma Scale score and a validated clinical prediction instrument such as the Canadian CT Head Rule (Table 399-3). In any patient suspected of having suffered a head injury, the severity of the concussion should be assessed (Table 399-4). A subdural hematoma (Fig. 399-1) is blood that accumulates above the brain but below the dura; on CT imaging it appears as a crescentic or concave opacity overlying the

FIGURE 399-1.  Subdural hematoma.

brain. An epidural hematoma (Fig. 399-2) is blood that accumulates below the skull but above the dura; it appears as a convex or lenticular opacity on CT imaging. Skull fractures are best diagnosed with the use of CT bone windows.

Traumatic Spinal Cord Injury

A detailed neurologic examination is needed to identify the level of the injury and the severity of any deficits, as well as to document the degree of neurologic dysfunction at the earliest time possible. The level of the injury is the lowest spinal cord segment with intact motor and sensory function. Normal neurologic findings in patients with a clear sensorium obviate the need for imaging studies. However, any complaints of pain over the spine, numbness, tingling, or weakness should raise suspicion of spinal cord injury. In particular, a complaint of “burning hands” suggests traumatic spinal cord injury. The time of injury should be recorded as accurately as possible. The prognosis for neurologic improvement is better if the lesion is incomplete as opposed to complete. During the acute period, serial examinations must be performed frequently. If spinal cord injury is suspected, the patient should be appropriately immobilized, such as with a rigid collar and back board. In patients who are able to cooperate with a neurologic examination, are not intoxicated, and do not have painful distracting injuries (e.g., femoral fracture, which would interfere with the leg motor and sensory examination), normal neurologic findings effectively rule out cervical spine disease.

CHAPTER 399  Traumatic Brain Injury and Spinal Cord Injury  

Imaging

In patients who are alert and stable, the Canadian C-Spine Rule (Fig. 399-3) can be used to reduce unnecessary spinal imaging without any adverse effect on patients’ outcomes. A1  In other patients, the radiologic evaluation should begin with plain radiographs of the bony spine, with further neuroimaging of any abnormalities that are found. Bony vertebrae should be examined

2367

with CT, whereas the spinal cord and intervertebral and paravertebral soft tissue are best studied with MRI. A chest radiograph is usually indicated to provide images of the lower cervical and thoracic vertebrae; the presence of a pleural effusion in the setting of a possible thoracic spine injury suggests a hemothorax.

Ligamentous Injury versus Spinal Cord Injury

If plain radiographs of the cervical spine are normal but the patient still complains of neck pain, a ligamentous injury should be considered. Ligamentous injury can be evaluated by flexion-extension radiographs of the cervical spine. If pain prevents an adequate study, patients should be kept in a rigid cervical collar for 3 to 5 days until the pain and muscle spasm resolve. If studies at that time are normal, the patient will no longer require the collar. Conversely, abnormal results warrant surgical evaluation to determine whether further immobilization or surgical correction is necessary.

TREATMENT  The immediate goals of therapy are to arrest ongoing injury, preserve and if possible restore neurologic function, and avoid secondary medical complications. To achieve this goal, an organized team approach is essential. Despite major research efforts, current clinical treatment is largely confined to supportive measures: maintaining perfusion pressure, minimizing intracompartment hypertension (e.g., increased intracranial pressure [ICP]), and indirectly treating edema.

Traumatic Brain Injury Initial Management

It is crucial that prehospital providers optimize perfusion and oxygenation; the duration and severity of hypoxia and hypotension in this critical early period have dramatic consequences on clinical outcome. Treatment begins with immediate attention to airway and cardiopulmonary function, early 

FIGURE 399-2.  Epidural hematoma.

Are there any high-risk factors present that require radiography? Age ≥ 65 years OR Dangerous mechanism* OR Paresthesias in extremities

Yes

C-spine imaging

No

C-spine imaging

No Are there any low-risk factors present that allow safe assessment of range of motion? Simple rear-end MVC† OR Sitting position in ED OR Ambulatory at any time OR Delayed onset of neck pain‡ OR Absence of midline C-spine tenderness

*Dangerous mechanism: • Fall from elevation ≥ 3 ft/5 stairs • Axial load to head (e.g., diving) • MVC high speed (>100 km/hr), rollover, ejection • Motorized recreational vehicles • Bicycle struck or collision

Yes Able to actively rotate neck? 45° Left and right Able No imaging needed

Canadian C-Spine Rule should not be used if: • Nontrauma case • GCS < 15 • Unstable vital signs • Age < 16 years • Acute paralysis • Known vertebral disease • Previous C-spine surgery

Unable

C-spine imaging

† Simple rear-end MVC excludes: • Pushed onto incoming traffic • Hit by bus/large truck • Rollover • Hit by high-speed vehicle ‡ Delayed: • i.e., not immediate onset of neck pain

FIGURE 399-3.  Canadian C-Spine Rule. For alert (Glasgow Coma Scale ≥ 15) and stable trauma patients in whom cervical spine injury is a concern. ED = emergency department; GCS = Glasgow Coma Scale (see Table 399-2); MVC = motor vehicle collision. (Modified from Stiell IG, Clement CM, McKnight RD, et al. Comparative validation of the Canadian C-Spine Rule and the NEXUS low-risk criteria in alert and stable patients. N Engl J Med. 2003;349:2510-2518; and Stiell IG, Wells GA, Vandemheen KL, et al. The Canadian C-Spine Rule for radiography in alert and stable trauma patients. JAMA. 2001;286:1841-1848).

2368

CHAPTER 399  Traumatic Brain Injury and Spinal Cord Injury  

identification of the potential for traumatic brain injury, and minimization of secondary insults such as hypoxia and ischemia.4 Individuals with a suspected concussion immediately should be removed from play or work5 and by law should not return until a detailed evaluation with written authorization can be made by an appropriate and experienced physician expert.6 Patients with mild or moderate traumatic brain injury often have returned to normal or are rapidly recovering by the time they reach advanced medical care. The critical element is the duration of altered mental status, amnesia, or loss of consciousness (see Table 399-4). Longer periods of abnormal sensorium are associated with higher grades of concussion, and higher grades of concussion necessitate longer periods of convalescence.

Severe Traumatic Brain Injury

Patients with Glasgow Coma Scale scores of 8 or less are considered to have severe traumatic brain injury. With this level of impaired consciousness, even with an intact gag reflex, patients are unable to protect their airway adequately. Intubation should be performed with either an endotracheal or a nasotracheal tube, depending on clinical circumstances. The patient should be in a rigid neck collar with the head elevated 30 degrees. The neck collar is used not only to protect the cervical spine until appropriate imaging can be performed but also to keep the head midline to avoid compromising venous drainage. Certain lesions require prompt surgical intervention, whereas others do not. Penetrating wounds, intracerebral hemorrhage with a mass effect (including subdural and epidural blood), and bone injury (e.g., displaced fracture and vertebral subluxation) require emergency surgical evaluation for intervention. However, focal hypoxic-anoxic, diffuse axonal, and diffuse microvascular injuries do not warrant surgical intervention; treatment remains primarily with the critical care clinician. Skull fractures and intracranial hemorrhages require neurosurgical evaluation. In general, if a fracture is displaced more than the thickness of the skull, it needs to be elevated. If a surgical lesion is not identified, the patient should be admitted to an ICU. When intracranial hypertension is suspected, a 30-mL intravenous (IV) dose of 23% hypertonic saline through a central venous catheter A2  may be better than mannitol (at a dose of 0.5 to 1 g/kg IV) to reduce it. IV steroids are of no benefit acutely and increase mortality at 2 weeks after the injury. A3  Continuous infusion of 3% hypertonic saline through a central venous catheter may be started at a rate of 75 to 100 mL/hour, with the goal of a serum sodium level of 150 to 155 mM/L to maintain ICP below 20 mm Hg. Hyperventilation also may be tried but has a potential to exacerbate ischemia; if used, the goal should be hyperventilation to a Pco2 of 34 to 36 mm Hg. Induced hypothermia for traumatic brain injury remains controversial, but high-quality randomized trials show no benefit and perhaps even deleterious effects. A4  For closed head injury, a transfusion threshold of 7 g/dL is preferable to a threshold of 10 g/dL. A5  In addition to ICP control, cerebral perfusion must be maintained. The goal is to maintain cerebral perfusion pressure, which is the difference between mean arterial pressure and ICP, higher than 60 mm Hg. Volume resuscitation is the first therapeutic intervention, with the aim of achieving euvolemia or only slight hypervolemia to a central venous pressure (CVP) goal of 4 to 6 mm Hg. For fluid resuscitation, saline is preferred over albumin. A6  If a cerebral perfusion pressure above 60 mm Hg cannot be achieved with IV fluids alone, vasoactive pharmacologic agents such as norepinephrine (beginning at 2 µg/ minute by continuous IV infusion) and phenylephrine (100 µg/minute) may be required. Invasive hemodynamic monitoring with an arterial pressure line and CVP catheter may be needed. Regular neurologic examinations and appropriate brain imaging are useful to guide ongoing therapy. In one randomized trial, ICP monitoring to maintain a pressure of 20 mm Hg or less was no better than care based on imaging and clinical examination. A7  If the patient is still at a Glasgow Coma Scale score of 8 or less, however, U.S. guidelines recommend that an ICP-monitoring device be used. An intraventricular catheter provides the most reliable data. It is also a treatment option because it allows drainage of cerebrospinal fluid. However, a subdural bolt and fiberoptic catheter are less invasive alternatives.

Pharmacologic Coma and Surgical Decompression

If ICP remains poorly controlled after the aforementioned efforts, pharmacologic coma or surgical decompression is considered. The postulated effect of pharmacologic coma on ICP is through reduction of cerebral metabolism. If the decision to use pharmacologic coma is made, pentobarbital can be administered at a loading dose of 5 mg/kg IV, followed by an infusion of 1 to 3 mg/kg/hour. Another option is propofol (loading dose of 2 mg/kg IV, followed by an infusion of up to 5 mg/kg/hour). Continuous electroencephalographic monitoring is helpful because the target response is burst suppression. Barbiturates and propofol are myocardial depressants, so aggressive cardiovascular management is often necessary to achieve the desired cerebral perfusion pressure. Recalcitrant elevated ICP despite these interventions is an ominous sign. In such cases, bifrontotemporoparietal craniectomy can reduce ICP and the length of ICU stay but has not been shown to improve outcomes. A8  A9  ,

Complications

If the patient is agitated, an evaluation should be made to determine whether the patient is in pain or poorly tolerating mechanical ventilation. If

pain is a concern, a narcotic analgesic such as fentanyl (50 to 100 µg IV) or morphine (1 to 2 mg IV) should be administered. Because these agents are easily reversed by naloxone, periodic reassessment of neurologic status can be performed. If agitation alone is the issue, haloperidol (0.5 to 2 mg IV), a nonsedating agent that still maintains the ability to perform a neurologic examination, should be considered. The Po2 level should be maintained at approximately 100 mm Hg. Phenytoin (loading dose of 1000 mg IV, followed by a maintenance dose of 300 mg/ day IV) reduces seizures during the first week after traumatic brain injury, but its later usefulness is less clear. Fever greatly increases cerebral metabolism; antipyretic interventions such as acetaminophen and cooling blankets should be used as needed. Gastric stress ulcers may be prevented with H2 antagonists such as ranitidine (50 mg IV three times daily) or proton pump inhibitors such as omeprazole (20 mg/day orally [PO]). Low-dose heparin (5000 units subcutaneously [SC] twice daily) or a low-molecular-weight heparin such as enoxaparin (40 mg/day SC) and pneumatic stockings should be instituted to avoid deep vein thrombosis. A nasogastric or orogastric tube should be placed for nutrition. Feeding should be initiated as soon as practical, usually on the second day after injury. Because cerebral edema is a concern, hyperosmotic feeding should be instituted. If ileus is present, total parenteral nutrition (TPN; Chapter 217) should be given. After the first 6 to 12 hours, effort should be made to reduce hyperventilation. Otherwise, the metabolic compensation to chronic hyperventilation negates the ameliorative effects of the respiratory alkalosis. Regular neurologic examinations and monitoring of ICP and cerebral perfusion pressure are useful to guide ongoing therapy. Generally, the peak period of cerebral edema is from 48 to 96 hours after traumatic brain injury. Thereafter, cerebral edema spontaneously resolves, often associated with clinical improvement.

Recovery

Recovering patients may experience “postconcussive syndrome,” which is primarily manifested as headache. Other symptoms may include difficulty concentrating, changes in appetite, sleep abnormalities, and irritability. In general, postconcussive syndrome lasts a few weeks after injury, but it can persist beyond a year or more. Amantadine (100 mg twice daily and increasing up to 200 mg twice daily) can accelerate early recovery after severe traumatic brain injury but may not improve the ultimate outcome. A10  Therapies are based on the patient’s symptoms.7 For headache, nonsteroidal anti-inflammatory agents (e.g., ibuprofen, 400 to 600 mg PO), migraine drugs (e.g., sumatriptan, 25 to 50 mg PO), and biofeedback may be considered. For cognitive dysfunction, neuropsychological testing may be helpful in determining appropriate intervention. Amantadine (100 mg twice daily) is effective for reducing irritability and aggression in post-head trauma patients with normal renal function. A11 

Traumatic Spinal Cord Injury Initial Management

Emergency management of traumatic injury to the spinal cord begins with the basics of airway, breathing, and circulation.8 A secure airway is essential. For patients suffering from high cervical lesions, spontaneous ventilation will be lost. Cervical lesions below C5 may also be associated with impaired ventilatory capability. If there is any concern that the airway or ventilatory effort is compromised, emergency intubation is required. In a patient in whom the cervical spine has not been imaged, the preferred method is nasotracheal intubation under fiberoptic guidance. Other approaches are nasotracheal (blind) or orotracheal intubation, provided in-line traction is applied. Other immediate concerns are bleeding and circulation. Hypotension may be due to either neurogenic shock or hypovolemia. For neurogenic shock, vasopressive pharmacologic agents such as phenylephrine (beginning as a continuous IV infusion at 100 µg/minute with titration to clinical effect) may be needed. If tachycardia is present, hypovolemia is more likely, so fluid resuscitation would be more appropriate.

Targeted Therapy

Methylprednisolone is no longer advocated for the treatment of acute spinal cord injury. The decision for surgical intervention should be based on the stability of the anterior, middle, and posterior vertebral columns. The anterior column consists of the anterior half of the vertebral body and the vertebral disc. The middle column is the posterior half of the body and the disc. The posterior column is composed of the arch, facets, and ligaments. In general, if two of the three columns are damaged, surgical stabilization is needed. If immediate surgery is not indicated, the patient should be admitted to the ICU for further management.

Acute and Subacute Management

Patients with severe spinal cord injuries require close cardiovascular and ventilatory care, supportive care for bladder and bowel function, approaches to avoid pressure ulcers (Chapter 25), and general measures similar to those used for patients with traumatic brain injury.

2369

CHAPTER 399  Traumatic Brain Injury and Spinal Cord Injury  

Neurogenic Shock and Dysautonomia

After traumatic spinal cord injury, patients are at risk for neurogenic shock and dysautonomia. Lesions of the cervical and thoracic spine disrupt the descending sympathetic pathways to the intermediolateral cell column of the thoracolumbar spinal cord, thereby leading to peripheral vasodilation and hypotension. If the lesion is at T3 or above, sympathetic tone to the heart is compromised. In this setting, hypotension is accompanied by bradycardia, thus producing the neurogenic shock triad of bradycardia, hypotension, and peripheral vasodilation. Initial therapy for dysautonomia should be fluid administration to restore an adequate circulating volume with a target CVP of 4 to 6 mm Hg. A hematocrit of 30 is optimal for perfusion of the central nervous system, so blood can be used if the patient is anemic. If blood is not required, either colloid (e.g., albumin solutions) or crystalloid (e.g., normal saline) may be used. If there is a suspicion of cardiac or pulmonary disease, a pulmonary artery catheter may be needed briefly to assess fluid status and the relationship between pulmonary pressure and CVP. Once adequate circulating volume has been achieved, hypotension should be managed with vasopressive agents such as phenylephrine (see earlier), norepinephrine (see earlier), or dopamine (beginning at 1 µg/kg/minute by continuous IV infusion) (Chapter 106), with the goal of a mean arterial pressure of 85 mm Hg or greater. Symptomatic bradycardia can be treated with atropine (1 mg IV).

Nutrition

Until enteral feeding can begin, parenteral nutrition should be used. Ideally, TPN should be started. However, if TPN is not possible, peripheral parenteral nutrition should be used until TPN (Chapter 217) can begin. Energy expenditures of 19 kcal/kg/day for high cervical injuries to 35.8 kcal/kg/day for injuries at T10 and below have been reported. A caloric level of 80% of the HarrisBenedict prediction should be used for quadriplegic patients. The full HarrisBenedict predicted amount should be used in patients with thoracic spine injuries and below. Indirect calorimetry should be used to determine the caloric needs of each patient so as to optimize nutritional support.

Other Therapy

In a randomized trial, pregabalin, 150 to 600 mg/day, was effective in reducing central neuropathic pain after spinal cord injury. A13  Patients with traumatic spinal cord injury have a propensity for the development of decubitus ulcers and pressure sores (Chapter 25). Mechanical kinetic beds, regular log rolling (every 2 hours), and padded orthotics are all useful in minimizing this complication. Orthotics, physical therapy, and occupational therapy (for cervical cord injury) are also important to minimize contractures and begin the rehabilitation process.

Ventilatory Compromise

An injury at C5 or higher results in diaphragmatic denervation and requires complete ventilatory assistance.9 Proper management requires endotracheal or nasotracheal intubation and mechanical ventilation, with an appropriate tidal volume (6 to 10 mL/kg), an Fio2 to achieve a Po2 between 80 and 100 mm Hg, and a rate to give a Pco2 of 40 mm Hg. Positive end-expiratory pressure should also be given to minimize atelectasis (Chapter 90). If the patient does not show signs of ventilatory recovery within 2 weeks of intubation, a tracheostomy should be considered. Lesions below C5 may also be associated with inadequate spontaneous ventilation. Midcervical lesions may be associated with intact but compromised diaphragm function. If suspected, a “sniff” test under fluoroscopy can be performed to determine whether both hemidiaphragms are functioning properly. If not, intubation/tracheostomy with volume-controlled ventilation may be needed. If intact, pressure support ventilation may be sufficient (Chapter 105) to achieve an appropriate tidal volume. Cervical lesions at C6 and below spare the phrenic nerves but may disrupt innervation of the intercostal muscles. The primary finding is decreased cough and an inability to increase ventilation when needed, thereby leading to  atelectasis and pneumonia; assisted elimination of tracheal secretions is essential.

Thromboembolic Disease

Thromboembolic disease (Chapters 81 and 98) is a leading cause of morbidity and mortality after traumatic spinal cord injury. Prolonged immobility of the lower extremities leads to deep venous thrombosis in up to 70% of spinal cord–injured patients. Patients should receive prophylaxis with low-molecularweight heparin (e.g., enoxaparin 30 mg twice daily SC) within 72 hours of injury. Anticoagulation can be held on the day of surgery but should be resumed 24 hours after surgery. A less effective alternative is intermittent compression devices (e.g., pneumatic stockings) with low-dose unfractionated heparin. An inferior vena cava filter may be placed if anticoagulation therapy is contraindicated.

PROGNOSIS

Traumatic Brain Injury

The most useful prognostic indicator after traumatic brain injury is the neurologic examination at initial evaluation. For patients with severe traumatic brain injury, the initial Glasgow Coma Scale score is the most reliable prognostic indicator. The lower the initial Glasgow Coma Scale score, the less likely a patient will have meaningful neurologic or functional recovery. After traumatic brain injury, 40% of patients with a score of 8 have a good recovery versus only 7% when the score is 3. Furthermore, only 27% of patients with a score of 3 survive versus 88% of patients with a score of 8. Patients in whom the Glasgow Coma Scale score remains the same or worsens over a period of 6 hours do worse clinically than those whose score improves. Further prognostic stratification at 24 hours can be based on pupillary responses, motor responses, and age (Chapter 404). Substantial increases of CSF α-synuclein may indicate widespread neurodegeneration and reflect secondary neuropathologic events after severe traumatic brain injury.10 A subsequent head injury before full recovery from even a mild traumatic brain injury may occasionally result in “second impact syndrome,” which can worsen the clinical outcome. When seen (mostly in children and adolescents), coma develops rapidly after the second injury, often within minutes. There is decreased autoregulation, diffuse cerebral edema, and intracranial hypertension. Second impact syndrome is associated with high mortality.

Traumatic Spinal Cord Injury

For traumatic spinal cord injury, the completeness of the injury is the most useful predictor (Table 399-5). A grade “A” or complete motor and sensory deficit below the lesion has a poor prognosis. If such a lesion persists for 24 hours, there is little likelihood of meaningful recovery. On the other hand, even severe partial injuries have a higher probability of recovery.

Visceral Function

The abdominal wall musculature is innervated by T7 to T12. The stomach, small bowel, liver, pancreas, and proximal two thirds of the colon receive innervation from T5 to L2. Spinal cord injury at these levels or above  may impair visceral function. For ileus, a nasogastric tube should be placed to decompress the stomach. Parental nutrition should be started as soon  as possible. Enteral feeding should be delayed until gastrointestinal  motility returns, usually within 2 to 3 weeks. In comparison with conservative bowel management, transanal irrigation improves constipation, fecal  incontinence, and symptom-related quality of life in patients with spinal cord-injuries. A12  Stress-induced peptic ulcer disease occurs in nearly a third of patients without prophylaxis. H2-receptor antagonists such as ranitidine (50 mg IV three times daily) or a proton pump inhibitor such as omeprazole (20 mg/day PO) reduce the incidence of ulcers. Bladder tone may be lost because of spinal shock. A Foley catheter should be placed for a minimum of 5 to 7 days to drain the bladder and evaluate volume and renal status. After spinal shock has resolved, autonomic dysreflexia may occur as a result of bladder distention. Clinical signs such as sweating, skin flushing, and hypertension may be present. Clinical examination with palpation and percussion will reveal a distended bladder, which can be treated by bladder training or intermittent catheterization.

TABLE 399-5  AMERICAN SPINAL INJURY ASSOCIATION IMPAIRMENT SCALE INJURY TYPE

DEFINITION

LIKELIHOOD OF RECOVERY*

A

Complete

No motor or sensory function below the lesion

15.5% (cervical) and 7% (thoracic)

B

Incomplete

Sensory but no motor function

47%

C

Incomplete

Some motor strength ( 3

84%

E

None

Sensory and motor function normal

100%

GRADE

*Data from Coleman WP, Geisler FH. Injury severity as primary predictor of outcome in acute spinal cord injury: retrospective results from a large multicenter clinical trial. Spine J. 2004;4:373-378.

2370

CHAPTER 400  SPINE, NERVE ROOTS, AND SPINAL CORD  

Base of skull

Grade A References A1. Stiell IG, Clement CM, Grimshaw J, et al. Implementation of the Canadian C-Spine Rule: prospective 12 centre cluster randomised trial. BMJ. 2009;339:b4146. A2. Wakai A, McCabe A, Roberts I, et al. Mannitol for acute traumatic brain injury. Cochrane Database Syst Rev. 2013;8:CD001049. A3. Edwards P, Arango M, Balica L, et al. Final results of MRC CRASH, a randomised placebocontrolled trial of intravenous corticosteroid in adults with head injury–outcomes at 6 months. Lancet. 2005;365:1957-1959. A4. Georgiou AP, Manara AR. Role of therapeutic hypothermia in improving outcome after traumatic brain injury: a systematic review. Br J Anaesth. 2013;110:357-367. A5. Robertson CS, Hannay HJ, Yamal JM, et al. Effect of erythropoietin and transfusion threshold on neurological recovery after traumatic brain injury: a randomized clinical trial. JAMA. 2014;312: 36-47. A6. Myburgh J, Cooper DJ, Finfer S, et al. Saline or albumin for fluid resuscitation in patients with traumatic brain injury. N Engl J Med. 2007;357:874-884. A7. Chesnut RM, Temkin N, Carney N, et al. A trial of intracranial-pressure monitoring in traumatic brain injury. N Engl J Med. 2012;367:2471-2481. A8. Timmons SD, Ullman JS, Eisenberg HM. Craniectomy in diffuse traumatic brain injury. N Engl J Med. 2011;365:373. A9. Cooper DJ, Rosenfeld JV, Murray L, et al. Decompressive craniectomy in diffuse traumatic brain injury. N Engl J Med. 2011;364:1493-1502. A10. Giacino JT, Whyte J, Bagiella E, et al. Placebo-controlled trial of amantadine for severe traumatic brain injury. N Engl J Med. 2012;366:819-826. A11. Hammond FM, Bickett AK, Norton JH, et al. Effectiveness of amantadine hydrochloride in the reduction of chronic traumatic brain injury irritability and aggression. J Head Trauma Rehabil. 2014;29:391-399. A12. Christensen P, Bazzocchi G, Coggrave M, et al. A randomized, controlled trial of transanal irrigation versus conservative bowel management in spinal cord-injured patients. Gastroenterology. 2006;131:738-747. A13. Cardenas DD, Nieshoff EC, Suda K, et al. A randomized trial of pregabalin in patients with neuropathic pain due to spinal cord injury. Neurology. 2013;80:533-539.

GENERAL REFERENCES

C1 C2 C3 C4 C4 C5 C5 C6 C6 C7 C7 C8 C1 C2 C3

Cervical enlargement

T1

T1

T2

T2 T3

T3 T4 T6 T7

Thoracic nerves Lumbar nerves

T7

T8

T11

T11

T12

Condus medullaris (termination of spinal cord)

T12

L1

L1 L2

For the General References and other additional features, please visit Expert Consult at https://expertconsult.inkling.com.

L3

Internal terminal filum (pial part)

L4

400  MECHANICAL AND OTHER LESIONS OF THE SPINE, NERVE ROOTS, AND SPINAL CORD RICHARD L. BARBANO

Sacral and coccygeal nerves

T8 T9 T10 T10

T9

Lumbar enlargement

C8 spinal nerve exits below C7 vertebra (there are 8 cervical nerves but only 7 cervial vertebrae) Cervical nerves

T4 T5 T6

T5

C1 spinal nerve exits above C1 vertebra

L2 L3

Cauda equina

L4

L5

L5 Termination of dural sac S2 External terminal filum (dural part)

S1

Sacrum S3 S4 S5 Coccygeal nerve Coccyx

FIGURE 400-1.  Anatomy of the spinal cord.

DEFINITION

Disorders of the spine, nerve roots, and spinal cord are frequent reasons for a patient to visit a physician. Many of these disorders either initially or eventually involve more than one element of the vertebra–spinal cord–nerve root unit, so there is much overlap in the pathobiology and clinical manifestations of these diseases. The spine consists of 30 vertebrae: 7 cervical (C), 12 thoracic (T), 5 lumbar (L), 5 sacral (S), and the coccyx (Fig. 400-1). The ring shape of the bony vertebrae forms a protective circle around the spinal cord while leaving ample room to allow the cord to move within this canal during flexion and extension of the spine. The vertebral bodies help bear the compressive weight of the body and provide the surface area to support the intervertebral discs, which act to cushion the axial force along the spine. The overlapping facet joints and multiple sets of longitudinal ligaments give the spine stability during its many ranges of motion. The posteriorly placed foramina allow the exit of spinal nerves. The spinal cord consists of 31 spinal segments, with one more cervical cord segment (8) than vertebrae; each gives rise to a bilateral pair of spinal nerves. Spinal nerves C1 to C7 exit the canal above their corresponding vertebral body, the C8 nerve exits below the C7 vertebra, and subsequent inferior nerves also exit below the numbered vertebrae. The spinal segments of the cord itself, however, lie progressively superior to the vertebrae, so that the end of the spinal cord, the conus medullaris, in adults is approximately adjacent to the L1 vertebra. The more caudal spinal nerves travel as the cauda equina in the subarachnoid space within the spinal canal before exiting their

respective foramina. The spinal cord does not have a uniform diameter; the cervical and lumbar segments are wider compared with the thoracic and lower sacral areas because the increased motor and sensory neurons supplying the arms and legs enlarge the cord. Spinal nerves are formed by the joining of the anterior and posterior spinal roots, which directly exit and enter the spinal cord. The anterior root derives from axons of the anterior horn cells and lateral columns, and it serves motor and autonomic efferent pathways; the posterior root mostly derives from the axons from the dorsal root ganglion and carries afferent sensory signals (Fig. 400-2). The sensory root is twice the thickness of the motor root and lies in a more anterior and inferior location as it crosses the foramen.

CLINICAL MANIFESTATIONS

Disorders of the spinal nerve root produce signs and symptoms referable to the corresponding dermatome or myotome. By far the most frequent complaint is localized neck or back pain, but compromise of the nerve roots or spinal cord will cause symptoms such as abnormal or painful sensations (paresthesias or dysesthesias), loss of sensation, weakness, and autonomic dysfunction (most commonly bladder or bowel incontinence). When it affects a myotome (the group of muscles served by motor neurons of a spinal cord segment; Fig. 400-3), the motor deficit associated with a spinal root disorder is of the lower motor neuron type. Typical findings are weakness, hypotonia, depressed or absent reflexes, and, if the syndrome has

CHAPTER 399  Traumatic Brain Injury and Spinal Cord Injury  

GENERAL REFERENCES 1. Selvarajah S, Hammond ER, Haider AH, et al. The burden of acute traumatic spinal cord injury among adults in the United States: an update. J Neurotrauma. 2014;31:228-238. 2. Scholten AC, Haagsma JA, Panneman MJ, et al. Traumatic brain injury in the Netherlands: incidence, costs and disability-adjusted life years. PLoS ONE. 2014;9:e110905. 3. Stern RA, Daneshvar DH, Baugh CM, et al. Clinical presentation of chronic traumatic encephalopathy. Neurology. 2013;81:1122-1129. 4. Hodgkinson S, Pollit V, Sharpin C, et al. Early management of head injury: summary of updated NICE guidance. BMJ. 2014;348:g104. 5. Putukian M, Raftery M, Guskiewicz K, et al. Onfield assessment of concussion in the adult athlete. Br J Sports Med. 2013;47:285-288.

2370.e1

6. Giza CC, Kutcher JS, Ashwal S, et al. Summary of evidence-based guideline update: evaluation and management of concussion in sports: report of the Guideline Development Subcommittee of the American Academy of Neurology. Neurology. 2013;80:2250-2257. 7. Marshall S, Bayley M, McCullagh S, et al. Clinical practice guidelines for mild traumatic brain injury and persistent symptoms. Can Fam Physician. 2012;58:257-267. 8. Joint Committee of the AANS and CNS. Guidelines for the management of acute cervical spine and spinal cord injuries. Neurosurgery. 2013;72(suppl 2):1-259. 9. Tester NJ, Fuller DD, Fromm JS, et al. Long-term facilitation of ventilation in humans with chronic spinal cord injury. Am J Respir Crit Care Med. 2014;189:57-65. 10. Mondello S, Buki A, Italiano D, et al. alpha-Synuclein in CSF of patients with severe traumatic brain injury. Neurology. 2013;80:1662-1668.

CHAPTER 400  SPINE, NERVE ROOTS, AND SPINAL CORD  

Aorta Lung Pleura Body of vertebra Dura mater Spinal sensory (dorsal root) ganglion Spinal nerve

2371

persisted for at least several weeks, atrophy with or without fasciculations. Sensation at the root level is diminished or absent for all modalities, but sensation below the affected root level is intact. Conversely, disorders of the spinal cord produce a “level” below which sensation is abnormal and motor deficits are of the upper motor neuron type, with weakness without atrophy (unless complicated by disuse), hypertonia, and increased reflexes. At the level of a spinal cord lesion, the motor deficits can be of the lower motor neuron type as the anterior horn cell bodies or exiting fibers are affected; below this level, an upper motor neuron syndrome will predominate. With strokes (Chapter 406) and other central nervous system (CNS) disorders, the full upper motor neuron syndrome may not be present in the acute phase of cord injury and can take time to appear.

DIAGNOSIS

The clinical history can help localize the patient’s symptoms, especially complaints of pain and sensory alterations that may exist in the absence of objective sensory loss on probing with light touch, pinprick, and vibration stimuli. The neurologic examination should include evaluation of the sensory, motor (Table 400-1), and reflex (Table 400-2) functions. Careful side-to-side comparisons can help assess subtle deficits. For example, elderly patients often have decreased or absent ankle jerks, so contralateral comparison is necessary. However, all muscles receive innervation from more than one root/ spinal cord level, and all roots send fibers to multiple muscles. The clinical implication of this anatomic pattern is that individual muscles are rarely profoundly weak, and patients rarely report isolated muscle weakness in single root involvement syndromes. Likewise, overlap of the sensory dermatomes (see Fig. 400-3) explains why sharp demarcations in the sensory examination rarely occur.

Lateral horn of gray matter of spinal cord Dorsal root

FIGURE 400-2.  Anatomy of the spinal cord: section through a thoracic vertebra.

C2 C3 C4 C5 C6

C2 C3

C4

C5

T1 T2 T3 T4 T5 T6 T7 T8 T9 T10

C6

T12

T11 T12 L1 S2,3

C6

C6 C8

T1 T2 T3 T4 T5 T6 T7 T8 T9 T10 T11

C8

L1 L2 L3 L4 L5

S1 S3 S2

C8

S4

C7

L2

C7

S5 L5

L3

L1 L4

L2 L3

L5

L4 S1

S1

L5

L4

L4

Levels of principal dermatomes C5 Clavicles C5,6,7 Lateral parts of upper limbs C8, T1 Medial sides of upper limbs C6 Thumb C6,7,8 Hand C8 Ring and little fingers T4 Level of nipples

T10 T12 L1,2,3,4 L4,5 S1 L4 S1,2, L5 S1 S2,3,4

Level of umbilicus Inguinal or groin regions Anterior and inner surfaces of lower limbs Foot Medial side of great toe Posterior and outer surfaces of lower limbs Lateral margin of foot and little toe Perineum

FIGURE 400-3.  Schematic demarcation of levels of principal dermatomes shown as distinct segments. There is actually considerable overlap between any two adjacent

dermatomes.

2372

CHAPTER 400  SPINE, NERVE ROOTS, AND SPINAL CORD  

TABLE 400-1  ESSENTIAL MUSCLE TESTING ROOT

MUSCLES

ACTION/TESTING

ARM C5

Deltoid Infraspinatus

Abducts arm Externally rotates arm with elbow flexed

C6

Brachioradialis

Flexes elbow (along with biceps [C5-6])

C7

Triceps Extensor digitorum

Extends elbow Extends fingers

C8

Flexor digitorum Flexor pollicis longus

Flexes fingers (both superficialis and profundus) Flexes distal phalanx of thumb

T1

Interossei

Spread fingers

LEG L1-2

Iliacus

Flexes hip

L2-3

Adductor magnus

Adducts thigh (as part of adductor group)

L3-4

Quadriceps

Extends knee

L4

Tibialis anterior

Dorsiflexes foot

L5

Extensor hallucis longus Extensor digitorum longus

Great toe extension Toe extension

S1

Hamstrings Flexor hallucis longus

Flex knee Flexes great toe (along with S2)

TABLE 400-2  ESSENTIAL REFLEX EXAMINATION REFLEX

EFFECT

ROOT/IMPLICATION

Jaw jerk

Jaw closes with tap on slightly opened jaw

Cranial nerve V; implies lesion above cervical cord

Biceps

Tap tendon: elbow flexion

C5-6; musculocutaneous nerve

Brachioradialis

Tap tendon over distal radius with elbow flexed and mid pronation: elbow flexion

C5-6; radial nerve

Triceps

Tap tendon: elbow extension

C6 < C7

Finger flexion

Tap partially flexed fingertips: finger flexion

C6-T1; when hyperactive, may imply lesion above midcervical spinal cord

Patella

Tap patella: quadriceps contraction (knee extension)

L2-4

Achilles

Tap Achilles tendon: foot plantar flexion

S1-2

Babinski

Scratch sole: great toe flexion

Great toe extension implies lesion of cord (or brain) above L4

Anal wink

Scratch perineum: external anal sphincter contraction

Absence of contraction implies S2-4 lesion

  DISORDERS OF THE SPINE

Neck and Back Pain

DEFINITION

Most neck and back pain is mechanical, that is, theoretically emanating from the spine’s structural elements, which are the vertebrae, discs, ligaments, tendons, and muscles. The location of pain is either axial, which means it is located along the spine itself, or referred. The term perceived pain is sometimes used when the pain from a spinal lesion is felt elsewhere by the patient, whereas referred pain is often used to describe pain that is experienced by the patient in the spinal area but caused by nonspinal structures. If the pain follows a dermatomal (nerve root) distribution, it is referred to as radicular pain, which is likely to involve the nerve root.

EPIDEMIOLOGY

Neck and back pain is a frequent reason for visits to a primary care physician. Low back pain is more common than neck pain, but both are common. The thoracic spine, possibly because of rib attachments and limited range of

motion, is an uncommon location for back pain. An exception to this general rule is the condition of diffuse idiopathic skeletal hyperostosis (DISH; Chapter 273), which is a noninflammatory age-related condition of unknown etiology, characterized by ossification of paravertebral ligaments and peripheral entheses. It is more common in men, with a prevalence of 30% in men by age 65. Pain in the thoracic spine region occurs in up to 80% of patients and is accompanied by notable decreased range of motion. In the general population, the incidence of self-reported neck pain is 213 per 1000; the 12-month prevalence of any pain is typically between 30 and 50%, and pain severe enough to limit activity is between 1.7 and 11.5%. The prevalence is higher among women. Risk factors for neck pain include inherited factors, poor psychological health, and tobacco use; the presence of disc degeneration is not a significant factor. More than 70% of people will experience low back pain significant enough to inhibit their participation in daily activities at some time in their life. The highest prevalence is in the 45- to 64-year age group. There is less of a gender difference than with neck pain, although tobacco use is an associated risk factor. Physical work–related factors (e.g., heavy lifting, prolonged sitting, repetitive twisting) increase risk; prospective studies show that psychosocial issues such as work monotony and job dissatisfaction also are major predisposing factors.

PATHOBIOLOGY

Although the popular impression is that the disc is the source of most spine pain, it is estimated that disc disease such as protrusion accounts for only 5% of all low back problems. Degenerative changes are a much more common cause of both acute and chronic spine pain. There is a genetic predisposition to intervertebral disc degeneration, with heritability estimates in the range of 34 to 61%. Degenerative changes can result in spondylosis, a condition that includes degenerative disc disease with bulging and occasionally herniation. The condition is often accompanied by the formation of osteophytes, ligamentous hypertrophy, and sometimes facet fracture and vertebral subluxation. Spondylosis, which is a consequence of age-related disc disease, is exemplified by the fact that almost everybody has at least anterior osteophytes by the age of 40 years, and it is not necessarily painful. Spondylosis probably starts with age-related disc desiccation and loss of elasticity of the annulus fibrosus. Tension of the longitudinal ligaments results in the formation of hypertrophic osteophytes. Compromise of microvascular supply may also contribute. Eventually, the facet joints can ride over one another, thereby leading to instability, formation of more osteophytes, and inflammation of the synovial joints. If there is fracture of the pars interarticularis, the term spondylolysis is used. Further instability between the intervertebral segments leads to spondylolisthesis, in which one vertebral body shifts sagittally in relation to its adjacent vertebra. Spondylolisthesis is graded by the amount of shift as measured with flexion and extension lateral spine films. DISH is characterized by calcification and ossification of the anterior longitudinal spinal ligament and less frequently the posterior longitudinal ligament. The latter can compromise spinal cord roots as well as the spinal cord itself, especially in the cervical region.1 Whiplash, an acute flexion-extension injury of the cervical spine, is common after motor vehicle accidents and other situations of rapid deceleration. Although specific acute and chronic manifestations are controversial, the acute syndrome is generally accepted to be a result of mechanical irritation of pain-sensitive structures in the cervical spine, with or without nerve root injury. More severe trauma can cause fracture and vertebral instability, both of which require rapid surgical evaluation.

CLINICAL MANIFESTATIONS

Acute neck pain and low back pain are commonly limited to the axial region, although radicular signs and symptoms can occur in the presence of nerve root irritation. The most common radicular pain occurs in the distribution of a dermatome. Other radicular signs and symptoms can include dysesthesias or sensory loss in the affected dermatome, decreased strength in muscles of the affected myotome, and decreased reflex. Cranial nerve findings, diffuse weakness throughout a limb or in more than one limb, hemisensory symptoms, autonomic symptoms, and increased reflexes are not manifestations of spine disease and should prompt more extensive evaluation for other conditions that affect the brain or brain stem. Bowel and bladder symptoms should prompt urgent evaluation of a cauda equina or myelopathy syndrome (see later). Acute spine problems can also cause referred or perceived pain at sites other than their anatomic source. For example, mechanical low back pain may

CHAPTER 400  SPINE, NERVE ROOTS, AND SPINAL CORD  

include aching in the buttocks or thigh, most often posteriorly and occasionally the hip region but rarely below the knee. More commonly, however, the term referred pain denotes the situation in which other structures, usually internal organs, refer pain to the spine or back. Areas of referred pain usually share the same embryologic origin and, during development, the same sensory pathways. Differentiation of referred pain from localized back pain depends on the history and examination. Mechanical pain is often exacerbated by movement such as twisting, bending, extension, or flexion, whereas referred pain tends to be independent of such activities. Chronic spine disorders lead to chronic back pain directly and as a secondary complication. For example, chronic degenerative arthropathy can lead to degenerative lumbar scoliosis with secondary involvement of neural structures. Back pain and radiculopathy are the most prominent symptoms, present in upward of 80% of patients, but symptoms of neurogenic claudication also develop in about 50% of patients.

DIAGNOSIS

History and Clinical Examination

The history and examination are essential for the initial evaluation and triage of patients with neck and back pain. Patients with so-called red flags (Table 400-3) merit special attention, as does any patient who awakens from sleep because of pain or has pain that is constant and unchanged by position, is unremitting and progressive, or is accompanied by any systemic signs or symptoms. As part of the history in the setting of acute neck trauma, well-established screening protocols such as the Canadian C-Spine Rule and the NEXUS Low-Risk Criteria (Fig. 400-4) are validated ways to detect cervical spine fracture and direct appropriate radiographic evaluation. In such a setting, a computed tomographic (CT) scan of the cervical spine would be the imaging test of choice. On clinical examination, inspection should assess evidence of trauma, muscle wasting, fasciculations, erythema, rashes, and scars. Palpation is directed to areas of point tenderness during evaluation for more diffusely tender regions, muscle spasm, and masses. If light percussion of the spinous process evokes significant pain, a focal process, such as fracture, malignant neoplasm (Chapter 189), or infection (Chapter 413), should be considered because such a finding is unusual in typical mechanical spine pain. Finally, the active and passive range of motion for flexion, extension, rotation, and tilt should be noted. Many provocative tests have been described for the evaluation of neck and back pain, but few have undergone formal evaluation of their diagnostic accuracy. For neck pain, contralateral rotation of the neck with extension of the arm and fingers (Video 400-1) suggests cervical root involvement, particularly in combination with other provocative tests, such as the Spurling maneuver, in which the patient’s head is rotated 45 degrees to the contralateral side, with the neck in slight extension to minimize the foraminal opening. Downward pressure on the top of the head by the examiner will reproduce arm dysesthesias (Video 400-2). Provocative tests also can diminish symptoms. For example, in the cervical distraction test, the examiner’s hands are placed under the jaw and occiput; gentle upward pulling of the head will temporarily reduce or alleviate the symptoms (Video 400-3). For low back pain, the straight leg raise (Video 400-4) has sensitivity of 0.85 to 0.91 but a specificity of only 0.26 to 0.52 for the diagnosis of sciatica due to a herniated disc. The crossed straight leg raise test (Video 400-5) has a lower sensitivity of 0.23 to 0.34 but a much higher specificity of 0.86 to

TABLE 400-3  “RED FLAGS” IN THE EVALUATION OF SPINE PAIN Recent significant trauma or minor trauma at age > 50 years Unexplained weight loss Unexplained fever Immunosuppression History of cancer History of prior local surgery Systemic disorder, bone or arthritic disorder Intravenous drug use Prolonged use of corticosteroids or osteoporosis Age > 70 years Focal neurologic deficit with progressive symptoms Duration > 6 weeks Thoracic spine pain Modified from Davis PC, Wippold FJ, Brunberg JA, et al. ACR Appropriateness Criteria on low back pain. J Am Coll Radiol. 2009;6:401-407.

2373

0.90. The seated straight leg raise (Video 400-6) can be used for confirmation of root irritation as the spine-leg angle is increased to 90 degrees. A negative result of the seated straight leg raise in the setting of a positive result of the straight leg raise suggests the possibility of a nonorganic component, although a mechanical alteration of the root exit zone in this position should also be considered.

Ancillary Testing

For neck pain (Fig. 400-5), plain radiography and CT scanning, which are the mainstays of cervical spine imaging, allow adequate view of the bony structures. Magnetic resonance imaging (MRI) has largely replaced myelography, which is still used occasionally to provide information about the spinal cord and nerve roots. However, MR abnormalities are common and can have a high false-positive rate; for example, 12 to 17% of patients younger than 30 years and 86 to 89% of patients aged 60 have disc degeneration as evidenced by loss of signal intensity, disc protrusion, narrowing of the disc space, or foraminal stenosis. Cervical discography also has a high falsepositive rate and cannot be recommended as a diagnostic test in the assessment of neck pain. Uncomplicated acute low back pain, with or without radiculopathy, is generally self-limited, and imaging studies are unnecessary unless any of the red flags (see Table 400-3) are present. A1  The American College of Physicians recommends MRI only in patients who have major or progressive neurologic deficits, in whom a serious underlying condition is expected, or in whom surgery or epidural steroids are being considered.2 For trauma, osteoporosis, or patients older than 70 years, plain radiography may suffice if the results are normal and no other abnormalities are present. Otherwise, with few exceptions, MRI is the test of choice given its superiority in evaluating soft tissue structures and its lack of radiation exposure. Care must be taken, however, to ensure correlation with the clinical syndrome, because 28% of asymptomatic volunteers with a mean age of 42 have herniated discs, 52% have bulging discs, and 14% have annular tears. The percentage of imaging abnormalities increases even more in asymptomatic volunteers older than 60 years; 57% have significantly abnormal scans, with 36% showing herniated discs and close to 98% showing disc degeneration. Abnormalities of the Modic end plate, anterolisthesis, and disc extrusion are more strongly associated with







• •

The NEXUS Low Risk Criteria (NLC) Algorithm for screening of neck injuries No posterior midline cervical spine tenderness—Midline posterior bony cervical-spine tenderness is present if the patient reports pain on palpation of the posterior midline neck from the nuchal ridge to the prominence of the first thoracic vertebrae, or if the patient evinces pain with direct palpation of any cervical spinous process. No evidence of intoxication—Patients should be considered intoxicated if they have either of the following: a recent history provided by the patient, or an observer of intoxication or intoxicating ingestion, or evidence of intoxication on physical examination such as an odor of alcohol, slurred speech, ataxia, dysmetria, or other cerebellar findings, or any behavior consistent with intoxication. Patients may also be considered to be intoxicated if tests of bodily secretions are positive for alcohol or drugs that affect level of alertness. A normal level of alertness—An altered level of alertness can include the following: a Glasgow Coma Scale score of 14 or less; disorientation to person, place, time, or events; an inability to remember three objects at five minutes; a delayed or inappropriate response to external stimuli; or other findings. No focal neurological deficit—A focal neurological deficit is any focal neurological finding on motor or sensory examination. No painful distracting injuries—No precise definition of painful distracting injury is possible. This category includes any condition thought by the clinician to be producing pain sufficient to distract the patient from a second (neck) injury. Such injuries may include, but are not limited to, any long-bone fracture; a visceral injury requiring surgical consultation; a large laceration, degloving injury, or crush injury; large burns; or any other injury causing acute functional impairment. Physicians may also classify any injury as distracting if it is thought to have the potential to impair the patient’s ability to appreciate other injuries.

FIGURE 400-4.  The NEXUS Low-Risk Criteria (NLC) algorithm for screening of neck injuries. (Reproduced from Hoffman JR, Mower WR, Wolfson AB, et al. Validity of a set of clinical criteria to rule out injury to the cervical spine in patients with blunt trauma. National Emergency X-Radiography Utilization Study Group. N Engl J Med. 2000;343: 94-99.)

CHAPTER 400  SPINE, NERVE ROOTS, AND SPINAL CORD  

VIDEO 400-1.  Cervical Rotation as provocative maneuver to elicit radiculopathic

symptoms

VIDEO 400-2.  The Spurling Maneuver to elicit radiculopathic symptoms

VIDEO 400-3.  Cervical distraction maneuver:relief of radiculopathic symptoms

2373.e1

VIDEO 400-4.  Straight Leg Raise: provocative maneuver for lumbar radiculopathy

VIDEO 400-5.  Crossed Straight Leg Raise: provocative maneuver for lumbar

radiculopathy

VIDEO 400-6.  Seated Straight Leg Raise: Provocative maneuver for lumbar

radiculopathy

2374

CHAPTER 400  SPINE, NERVE ROOTS, AND SPINAL CORD  

New consult for neck pain

History: Any red flag symptoms?

Self-assessment of interference with daily activities

Physical exam: Neurological and other

Triage: What kind of neck pain?

Grade I

Grade II

Grade III

Grade IV

No investigations Reassurance Self-care

Assess factors to help decrease interference Discuss options for short-term relief

Monitor if deficits stable and minor Consider MRI and referral if deficits are major or progress Needle EMG might assist

Investigations according to the suspected condition

Options for short-term relief Likely helpful for neck pain after a traffic collision: exercise training and mobilization Likely helpful for neck pain with no trauma: exercise training, mobilization, manipulation, acupuncture, analgesics, low-level laser FIGURE 400-5.  Approach to new-onset neck pain. EMG = electromyography; MRI = magnetic resonance imaging. (Modified from Guzman J, Haldeman S, Carroll LJ, et al. Clinical practice implications of the Bone and Joint Decade 2000-2010 Task Force on Neck Pain and Its Associated Disorders. From concepts and findings to recommendations. Spine. 2008;33:S199-S213.)

TABLE 400-4  MECHANICAL NECK PAIN NECK STRAIN

HERNIATED NUCLEUS PULPOSUS

Age (yr)

20-40

30-50

OSTEOARTHRITIS >50

MYELOPATHY >60

WHIPLASH 30-40

Pain location

Neck

Arm

Neck

Arm/leg

Neck

Onset

Acute

Acute

Insidious

Insidious

Acute

Flexion

+

+





+

Extension



+/−

+

+

+

Plain radiography





+

+



+ = present; − = absent. From Borenstein DG, Wiesel SW, Boden SD. Neck Pain: Medical Diagnosis and Comprehensive Management. Philadelphia: WB Saunders; 1996.

low back pain than is disc degeneration without end plate changes. Situations in which alternative imaging should be considered include spondylosis and stress fracture, for which bone scintigraphy with single-photon emission CT (SPECT) is more sensitive than MRI. CT can also be useful when MRI is contraindicated or to evaluate scoliosis, bone graft integrity, surgical fusion, and instrumentation. For back and neck pain that persists for 6 weeks, electrodiagnostic testing can demonstrate compromise of spinal root function but is not usually helpful in axial spine pain without neurologic symptoms.

Differential Diagnosis

Mechanical or idiopathic pain explains up to 97% of cases of neck pain (Table 400-4) and low back pain (Table 400-5); the remaining 3% is nonmechanical in origin and includes referred pain and other conditions. Acute mechanical neck pain is most often caused by a neck strain, a herniated nucleus pulposus, or whiplash; for pain of insidious onset, osteoarthritis and myelopathy are the leading causes. For back pain, muscle strain and a herniated nucleus pulposus are acute causes; insidious causes include osteoarthritis, spinal stenosis, spondylolisthesis, and scoliosis. Queries regarding the red flags will identify serious and nonmechanical causes of neck and back pain (Fig. 400-6). Abdominal and pelvic structures can refer pain to the low back (referred pain). Abdominal aortic aneurysms (Chapter 78) can present with a mid- to low back ache that may radiate to the hips or anterior thighs. Cholecystitis

(Chapter 155) can cause pain in the midthoracic area; pancreatic disease (Chapter 144) can cause pain in the L1 region; and diverticulitis (Chapter 142) in the left lower quadrant can cause diffuse low back pain. Genitourinary disorders (Chapter 123) can cause colicky referred pain to the flanks and costovertebral angle. Bladder disorders (Chapter 123) may occasionally refer pain to the sacral area, as can prostate problems (Chapter 129). Pelvic disorders in women that can cause referred low back pain include endometriosis (Chapter 236), ectopic pregnancy, and pelvic inflammatory disease (Chapters 299 and 318). Most of these disorders have additional signs and symptoms to aid in the diagnosis. Myocardial ischemia (Chapters 71 to 73) can be associated with anterior neck pain, although less commonly than with left arm or jaw pain. Arterial dissections (Chapter 78) are more commonly associated with neck pain; for example, up to 20% of patients with carotid dissections complain of anterolateral pain, and about 80% of patients with vertebral dissections have posterior or occipital pain. Patients with arterial dissections frequently but not necessarily have signs and symptoms of stroke (Chapter 407). Disorders of the esophagus (Chapter 138) and mass lesions of the throat (Chapters 190 and 429) can also present as neck pain. Acute spine pain can precede the rash in herpes zoster (Chapter 375) or can be seen in the vaso-occlusive crisis of sickle cell anemia (Chapter 163). Infections of the disc (Chapter 413) cause sharp back pain worsened by

2375

CHAPTER 400  SPINE, NERVE ROOTS, AND SPINAL CORD  

TABLE 400-5  MECHANICAL LOW BACK PAIN MUSCLE STRAIN

HERNIATED NUCLEUS PULPOSUS

SPINAL STENOSIS

OSTEOARTHRITIS

SPONDYLOLISTHESIS

SCOLIOSIS

Age (yr)

20-40

30-50

>50

>60

20

30

Pain pattern location

Back (unilateral)

Back (unilateral)

Back (unilateral)

Leg (bilateral)

Back

Back

Onset

Acute

Acute (prior episodes)

Insidious

Insidious

Insidious

Insidious

Standing













Sitting













Bending













Straight leg



+



+ (stress)





Plain radiography





+

+

+

+

From Borenstein DG, Wiesel SW, Boden SD. Low Back Pain: Medical Diagnosis and Comprehensive Management. 2nd ed. Philadelphia: WB Saunders; 1995.

movement. Arachnoiditis (Chapter 412), an inflammatory process of the arachnoid space, can cause diffuse chronic back pain, often after the introduction of foreign substances or manipulation of the intrathecal space. Finally, 20 to 50% of patients with depression (Chapter 397) will complain of back pain that often is diffuse and described in emotionally laden terms. Complaints of low back pain are also common in malingering patients.

TREATMENT  Treatment options vary according to the severity of pain, presence of radicular signs or symptoms, and any underlying disease. Acute nontraumatic neck pain is common and usually benign. Beneficial treatments include nonsteroidal anti-inflammatory drugs (NSAIDs, e.g., ibuprofen 600-800 mg three times daily for 2 weeks), exercise, and physical therapy. A2  The low risk of myocardial infarction or upper gastrointestinal bleed from NSAIDs must be weighed against pain relief and the relative benignity of the condition. Chiropractic manipulation is of benefit but has the rare complication of posterior circulation stroke from arterial dissection, especially in patients younger than 45 years. Low-level laser therapy A3  and acupuncture A4  are also of short-term benefit. Cervical collars and traction are not of established benefit. Even in the absence of radicular pain, surgical intervention for neck pain is indicated in cases of vertebral instability, such as caused by fracture or dislocation. Surgery, including fusion, may be needed for stabilization after surgery for lesions such as tumors, infections, or hemorrhages. For neck pain that is accompanied by signs or symptoms of radiculopathy, surgery should be considered but is not usually the initial therapy (see later). If a cervical spine lesion is causing spinal cord compression, emergent evaluation for possible surgery is indicated (see later). For chronic neck pain, yoga therapy yields significant pain relief with the possible added benefits of improving quality of life and psychological well-being. A5  Because acute low back pain is generally benign, invasive therapy should be avoided in the first 3 months. Conservative treatment includes NSAIDs (e.g., ibuprofen 600-800 mg three times daily as needed) and controlled physical activity; strict bed rest for longer than 2 days is no better than restricted physical activity. A6  Acetaminophen is no better than placebo. A7  Other options include local heat and massage. Spinal manipulation, exercise therapy, massage, and cognitive-behavioral therapy are moderately effective. A8-A10  True acupuncture appears no better than sham acupuncture. A11  For chronic nonradicular low back pain, exercise and cognitive-behavioral therapy are recommended; yoga should also be considered. A12  Prolotherapy, in which a mild irritant is injected into a tendon or ligament to increase blood flow and promote healing, and facet joint injections can be helpful, but  intradiscal steroid injections and percutaneous intradiscal radiofrequency thermocoagulation are not effective. A13  Transcutaneous electrical neurostimulation (TENS) does not appear to provide any functional improvement in patients with chronic lumbar pain. A14  Oral analgesic medications are frequently offered and include NSAIDs; duloxetine (30 to 60 mg daily) may be a reasonable alternative. In one randomized trial, tanezumab (10 to 20 mg intravenously, repeated 8 weeks later), a humanized monoclonal antibody that specifically inhibits nerve growth factor, reduced pain better than naproxen or placebo. A15  When persistent nonradicular low back pain is accompanied by associated degenerative spine changes, surgical fusion is of benefit but not superior to interdisciplinary rehabilitation. A16  If degenerative changes lead to lumbar scoliosis with or without neurologic signs, however, decompressive surgery with or without fusion appears effective for at least 5 years for the majority of patients. Outcomes after instrumented lumbar spinal fusion are improved if rehabilitation is delayed for 12 weeks after the operation. In more focal disor-

ders, surgery to remove disc material pressure from pain-sensitive structures can be considered; techniques include open, laser, and microdiskectomy approaches, with little evidence to favor one option over the other.

PROGNOSIS

Between 50 and 85% of patients who have neck pain that persists for more than 1 day report recurrence of symptoms in 1- and 5-year follow-up. Slightly more than 50% of patients recover within 3 months, and those who remain symptomatic generally have relatively little pain and disability.3 Patients younger than 45 years have less recurrence, and patients aged 45 to 59 have the highest risk. Prior neck injury or pain, coexistent low back pain, and selfperceived poor general health are risk factors for symptoms persisting past 3 months or recurrence. Mechanical spine pain, even with radicular symptoms, resolves without specific intervention within 30 days in many patients and within 3 months in 90% of patients. Recurrence is frequent, however, especially in patients with spondylosis, because the underlying process persists and further degeneration of the spinal elements can be expected. Long-term disability is more common with obesity, low education level, tobacco use, high levels of pain at the onset, tendency to somatization, job dissatisfaction, lack of availability of light-duty employment, and need to perform significant lifting at work. The strongest factors affecting outcome are psychological, especially worrying, fear avoidance, anger, and frustration.4 Genetic variability, such as in polymorphisms of catechol O-methyltransferase, also may play a role in the development of chronic pain.

Spinal Stenosis

DEFINITION

Spinal stenosis, which is a narrowing of the spinal canal, results in compression of neural structures in the cervical and lumbar regions, where the diameter of the spinal cord is largest. Signs and symptoms of spinal stenosis are referable to these levels. In the lumbar region, L4-5 is the most common level of stenosis, followed by L3-4 and L5-S1.

EPIDEMIOLOGY

The annual incidence of spinal stenosis in the United States is about 1 to 2 per l00,000 for the cervical region and 5 per 100,000 for the lumbar region.5 Spinal stenosis often coexists in the cervical and lumbar regions, and the incidence is higher in patients with more complex degenerative anatomy.

PATHOBIOLOGY

Primary spinal stenosis is due to a congenital narrowing of the spinal canal. Causes of secondary stenosis include chronic degenerative conditions such as spondylosis and thickening of the ligamenta flava and longitudinale posterius neoplasia, osteomyelitis, and rheumatoid arthritis. In patients receiving corticosteroids long term, epidural lipomatosis is a cause predominantly at the thoracic level. The underlying cause of symptoms may be multifactorial, with direct nerve pressure, duration of the pressure, capillary restriction, venous congestion, and reliance on the anastomosis of cerebrospinal fluid for metabolic homeostasis all potentially playing a role. In spinal stenosis, myelopathy results from compression of the veins that drain the canal, thereby leading to capillary stasis and edema, which result in

2376

CHAPTER 400  SPINE, NERVE ROOTS, AND SPINAL CORD  

Differential diagnosis of organic causes of low back pain*

Mechanical low back or leg pain (97%)

Nonmechanical spinal conditions (about 1%)

Visceral disease (2%)

Lumbar strain, sprain (70%) Degenerative disc and/or facet disease (10%) Herniated disc (4%) Spinal stenosis (3%) Osteoporotic compression fracture (4%) Spondylolisthesis (2%) Traumatic fracture (75 µV) for > 20% of an epoch. Sleep spindles may persist. N3 persists until transition to N2, R, or an arousal.

No eye movements seen

Variable amplitude, typically lower than N2 and can be as low as R

No memory, least responsive to arousing stimuli, less response to elevated CO2 and low oxygen, but monotonous breathing pattern

Stage R (REM sleep)

Low-amplitude mixed-frequency EEG. Sawtooth waves. R persists until transition to N1, transition to N2, between K complexes without eye movements, or an arousal.

Rapid eye movements

Low muscle tone

Similar response to stimuli as light sleep, irregular breathing pattern, least response to elevated CO2 and low oxygen

*Sleep staging requirements. Boldfaced items are requirements for staging. Italicized items are non-required associated findings that may be present in that stage. EEG = electroencephalogram; EMG = electromyogram; EOG = electro-oculogram. Adapted from the American Academy of Sleep Medicine. The AASM Manual for the Scoring of Sleep and Associated Events. 2nd ed. version 2.1. Westchester, IL: American Academy of Sleep Medicine; 2014.

CHAPTER 404  Coma, Vegetative State, and Brain Death  

GENERAL REFERENCES 1. Edlow JA, Rabinstein A, Traub SJ, et al. Diagnosis of reversible causes of coma. Lancet. 2014;384: 2064-2076. 2. Luce JM. Chronic disorders of consciousness following coma: Part one: medical issues. Chest. 2013;144:1381-1387. 3. Fugate JE, Moore SA, Knopman DS, et al. Cognitive outcomes of patients undergoing therapeutic hypothermia after cardiac arrest. Neurology. 2013;81:40-45. 4. Shprecher D, Mehta L. The syndrome of delayed post-hypoxic leukoencephalopathy. Neurorehabilitation. 2010;26:65-72. 5. Giacino JT, Fins JJ, Laureys S, et al. Disorders of consciousness after acquired brain injury: the state of the science. Nat Rev Neurol. 2014;10:99-114.

2415.e1

6. Stender J, Gosseries O, Bruno MA, et al. Diagnostic precision of PET imaging and functional MRI in disorders of consciousness: a clinical validation study. Lancet. 2014;384:514-522. 7. Estraneo A, Moretta P, Loreto V, et al. Predictors of recovery of responsiveness in prolonged anoxic vegetative state. Neurology. 2013;80:464-470. 8. Luce JM. Chronic disorders of consciousness following coma: Part two: ethical, legal, and social issues. Chest. 2013;144:1388-1393. 9. Bernat JL. Controversies in defining and determining death in critical care. Nat Rev Neurol. 2013;9:164-173. 10. Wijdicks EFM, Varelas PN, Gronset GS, et al. Practice parameter update: determining brain death in adults: report of the Quality Standards Subcommittee of the American Academy of Neurology. Neurology. 2010;74:1911-1918.

2415.e2

CHAPTER 404  Coma, Vegetative State, and Brain Death  

REVIEW QUESTIONS 1. What clinical sign differentiates between a minimally conscious state and a vegetative state? A: Blinking to threat B: Marked dysautonomia C: Abnormal brain stem reflexes D: Myoclonus status E: Tracking an examiner’s hand Answer: E  The only sign that, when found consistently, indicates awareness is the ability to track an object. All other signs can be seen in both conditions. 2. A 70-year-old woman was found snoring and unresponsive by her husband. On arrival to the emergency department, she had marked anisocoria, ocular bobbing, extensor posturing, and irregular breathing with pooling secretions requiring intubation. The vital signs were normal, and she was afebrile. Her computed tomographic (CT) scan was normal. Routine laboratory tests including glucose and blood gases are unrevealing. What is a likely diagnosis? A. Embolus to the basilar artery B. Intoxication/poisoning C. Status epilepticus D. Meningoencephalitis E. Anoxic-ischemic encephalopathy Answer: A  Ocular bobbing, anisocoria, and extensor posturing points to a brain stem lesion. The computed tomographic (CT) scan can be normal in the early hours after presentation. An embolus to the basilar artery may be seen as a hyperdense CT lesion, but a CT angiogram is a better study. Intoxication is unlikely due to localizing findings. Anoxic-ischemic encephalopathy usually spares the brain stem. Meningoencephalitis would be highly unlikely in a nonfebrile, acutely comatose patient with new brain stem findings.

3. What observation precludes brain death examination? A. Core temperature of 36° C B. Systolic blood pressure of 100 mm Hg C. Preserved tendon reflexes and Babinski signs D. Leg withdrawal after noxious stimulus E. Extensor posturing Answer: E  Extensor posturing indicates preserved brain stem function. Generally, blood pressures above 100 mm Hg and core temperatures above 35° C do not influence the neurologic examination. Tendon reflexes, Babinski signs, and leg withdrawal (triple flexion) response can be generated at a spinal level without cortical input and can be preserved in brain dead individuals. 4. A locked-in syndrome is characterized by all the following signs except: A. Absent horizontal eye movements B. Present vertical eye movements C. Blinking on command D. Deafness E. Quadriplegia Answer: D  Locked-in syndrome damages the ventral portion of the pons, sparing structures above and below it. In most patients with a locked-in syndrome, consciousness is spared because the ascending reticular formation, which produces wakefulness, is located in the dorsal portion of the pons. 5. A patient presents to the emergency department in coma (intact brain stem findings and localization only to noxious stimuli; moaning). Vital signs are normal. The CT scan is normal. Arterial blood gases show a metabolic acidosis. Lactate and glucose levels are normal, as are measures of renal function. What additional laboratory test is needed? A. Serum anion and osmolar gap B. Serum ammonia level C. Serum thyroid level D. Serum cortisone level E. Blood cultures Answer: A  Acute lactate metabolic acidosis points to ingestion of salicylates or ethylene glycol. The other disorders generally do not cause a metabolic acidosis.

CHAPTER 405  Disorders of Sleep  

E-FIGURE 405-1.  The two components that produce wakefulness are the reticular activating system (red), which permits sensory input to be relayed by the thalamus, and the monoaminergic nuclei and basal forebrain cholinergic system, which promote responsiveness of the cerebral hemispheres. (Adapted from Saper CB, Scammell TE, Lu J. Hypothalamic regulation of sleep and circadian rhythms. Nature. 2005;437: 1257-1263.)

2415.e3

E-FIGURE 405-2.  Activation of the ventral lateral preoptic nucleus subsequently inhibits (purple arrows) the reticular activating system (red) and the monoaminergic nuclei (green), thereby producing non–rapid eye movement sleep and blocking sensory input to the hemispheres. (Adapted from Saper CB, Scammell TE, Lu J. Hypothalamic regulation of sleep and circadian rhythms. Nature. 2005;437:1257-1263.)

Wake Stage R Stage N1 Stage N2 Stage N3 Time through the night sleep period E-FIGURE 405-4.  Hypnogram showing sleep stages across a sleep period.

+ Norepinephrine AcetylCholine and – Serotonin

E-FIGURE 405-3.  Rapid eye movement sleep (REM) sleep is activated by activation of the subcaeruleus nucleus, which uses acetylcholine (red arrows), and further deactivation (purple arrows) of norepinephrine- and serotonin-producing neurons (green). The reciprocal relationship of REM on (cholinergic neurons) and REM off (norepinephrine and serotonin neurons) is depicted in the lower corner. (Adapted from Saper CB, Scammell TE, Lu J. Hypothalamic regulation of sleep and circadian rhythms. Nature. 2005;437:1257-1263.)

2416

CHAPTER 405  Disorders of Sleep  

2

explain sleep-wake state. Other issues such as psychological status also play a role. In the two-driver model, the homeostatic drive is the accumulation of substances that promote sleepiness while the person is awake. These substances are metabolized during sleep. Mental and physical activities increase this drive by producing neuronal byproducts (e.g., adenosine), whereas caffeine blunts this drive by blocking adenosine. In contrast, the circadian rhythm drive promotes wakefulness and, through its predictable cycle, prepares the body for anticipated activities. The circadian rhythm is a naturally occurring rhythm that is slightly longer than 24 hours but is readjusted each day to maintain alignment with the natural day-night cycle. The circadian rhythm is primarily adjusted by bright light and to a lesser extent by other factors such as exercise, food, and social interactions. The hormone melatonin, which is released in response to darkness, can also influence the phase of the circadian rhythm. Throughout the 24-hour period, the homeostatic and circadian drives maintain balance between the sleep and wake states. When the circadian rhythm is stronger than the homeostatic drive, the person is awake, and when the homeostatic drive is stronger than the circadian rhythm, the person is sleepy (Fig. 405-1). This theoretical model helps explain aspects of sleep-wake regulation, such as the periods of post-lunch sleepiness or evening wakefulness.

(Fig. 405-2) can provide a broad overview of sleep symptoms, including bedtime, wake time, activities, medications, and other substances that could influence sleep. Objective testing of sleep includes actigraphy, polysomnography, multiple sleep latency testing, and maintenance of wakefulness testing. Actigraphy monitors movement, typically of a nondominant extremity, over 7 to 28 days (E-Fig. 405-5). When combined with a sleep diary, actigraphy estimates total sleep time and assesses the sleep-wake schedule. Polysomnography (Chapter 100, Fig. 100-1) assesses both sleep stage and associated physiology. Sleep stage is determined by EEG, electro-oculogram, and submental electromyogram activity. Measures assessing physiology include respiratory function (flow, effort, and gas exchange), limb muscle activity, electrocardiogram, and sometimes esophageal pH or core body temperature. Polysomnography is most useful for sleep disruption such as sleep apnea (Chapter 100), excessive movements, parasomnias, or for unexplained excessive sleepiness3 (Table 405-2). More limited overnight recordings may focus on strictly respiratory

Amount of drive

Circadian drive (wakefulness) Homeostatic drive (sleepiness)

CLINICAL MANIFESTATIONS

Most patients who seek medical help for sleep issues present with one of three complaints: (1) excessive sleepiness, (2) difficulty attaining or sustaining sleep, (3) or unusual events associated with sleep. Excessive sleepiness may be confused with fatigue or lack of energy. Common symptoms include morning headaches, lapses of attention, or diffuse muscle aches. Difficulty with sleep at night may be a clue to daytime issues, and nocturnal events may be a clue to brain issues.

6:00 AM Noon 6:00 PM MN 6:00 AM

DIAGNOSIS

FIGURE 405-1.  Idealized graph depicting the two-process model. The circadian driver promotes wakefulness (orange), and the homeostatic drive promotes sleep (blue). The dynamic interaction of the two drives is shown.

Both subjective information and objective tests are used to investigate sleep complaints. Questionnaires such as the Pittsburgh Sleep Quality Index

TWO WEEK SLEEP DIARY INSTRUCTIONS: 1. Write the date, day of the week, and type of day: Work, School, Day Off, or Vacation. 2. Put the Ietter “C” in the box when you have coffee, cola or tea. Put “M" when you take any medicine. Put ”A” when you drink alcohol. Put “E” when you exercise. 3. Put a Iine (l) to show when you go to bed. Shade in the box that shows when you think you fell asleep. 4. Shade in all the boxes that show when you are asleep at night or when you take a nap during the day. 5. Leave boxes unshaded to show when you wake up at night and when you are awake during the day.

sample

Mon

Work

E

A

I

11AM

10

9

8

7

6AM

5

4

3

2

1AM

Midnight

11PM

10

9

8

7

6PM

5

4

3

2

Day of Type of Day the Work, School week Off, Vacation

1PM

Today’s Date

Noon

SAMPLE ENTRY BELOW: On a Monday when I worked, l jogged on my lunch break at 1 PM, had a glass of wine with dinner at 6 PM, fell asleep watching TV from 7 to 8 PM, went to bed at 10:30 PM, fell asleep around Midnight, woke up and couldn’t got back to sleep at about 4 AM, went back to sleep from 5 to 7 AM, and had coffee and medicine at 7:00 in the morning.

C M

week 1 week 2

Used with permission from the American Academy of Sleep Medicine, Darien, Illinois. FIGURE 405-2.  Example of a sleep diary. Patients record their daily schedule, work, and medications.

CHAPTER 405  Disorders of Sleep  

12:00 PM

8:00 PM

12:00 AM

6:00 AM

12:00 PM

12:00 AM

6:00 AM

12:00 PM

2416.e1

DAY 1 12/11/2000

DAY 2 12/13/2000

DAY 3 12/13/2000

DAY 4 12/14/2000

DAY 5 12/15/2000

DAY 6 12/16/2000

DAY 6 12/17/2000 12:00 PM

6:00 PM

E-FIGURE 405-5.  Actigraphy report. The shaded areas are scored as rest time; in this instance, the bedtime is 11 PM to 12 midnight, and wake time about 6 AM. One nap is observed on day 3 in the middle of the afternoon.

CHAPTER 405  Disorders of Sleep  

TABLE 405-2  INDICATIONS FOR POLYSOMNOGRAPHY POLYSOMNOGRAPHY IS ROUTINELY INDICATED FOR: Diagnosis of sleep-related breathing disorders (SRBD), including suspected obstructive sleep apnea (OSA) in patients with coronary heart disease, history of stroke or transient ischemic attacks, or significant tachyarrhythmias or bradyarrhythmias Positive airway pressure (PAP) titration in patients with sleep-related breathing disorders A preoperative clinical evaluation to evaluate for the presence of OSA before upper airway surgery for snoring or OSA Patients with heart failure if they have nocturnal symptoms suggestive of sleeprelated breathing disorders (disturbed sleep, nocturnal dyspnea, snoring) or if they remain symptomatic despite optimal medical management Patients with neuromuscular disorders and sleep-related symptoms Patients suspected of periodic limb movement disorder Polysomnography and multiple sleep latency test on the ensuing day for patients suspected of having narcolepsy Follow-up polysomnography: After titration of oral appliance treatment in patients with moderate to severe OSA Following surgical treatment of patients with moderate to severe OSA After surgical or dental treatment of patients with SRBDs whose symptoms return Substantial weight gain or loss in patients on PAP for SRBD Insufficient clinical response to PAP therapy Assessment of oral appliance after final fitting (guideline) Evaluation of patients with sleep behaviors suggestive of unusual or atypical parasomnias or in which specific motor patterns are in question POLYSOMNOGRAPHY IS OPTIONAL FOR: Evaluation of sleep behaviors suggestive of potentially injurious parasomnias POLYSOMNOGRAPHY IS NOT ROUTINELY INDICATED FOR: Patients whose symptoms resolve with continuous positive airway pressure (CPAP) treatment Diagnosis of chronic lung disease Diagnosis of typical, uncomplicated, and noninjurious parasomnias when the diagnosis is clearly delineated Patients with a seizure disorder who have no specific complaints consistent with a sleep disorder Diagnosis or treatment of restless legs syndrome, except where diagnostic uncertainty exists Establishing the diagnosis of depression Diagnosis of circadian rhythm sleep disorders

measurements. Two tests quantify the ability to fall asleep and stay awake: the multiple sleep latency test and maintenance of wakefulness test. The multiple sleep latency test quantifies objective sleepiness based upon the time to onset of sleep across five daytime naps. The multiple sleep latency test is useful for narcolepsy, but there is overlap between normal individuals and patients with sleep disruption. The maintenance of wakefulness test quantifies the propensity to stay awake across four 40-minute epochs, and it can provide objective evidence of the daytime efficacy of stimulant therapy.

  HYPERSOMNIA

Sleepiness is normal just prior to a typical sleep period or after prolonged wakefulness. In 5 to 20% of adults, sleep is excessive because it occurs in inappropriate settings. When mild, sleepiness may have a minor effect on quality of life. When severe, however, sleepiness intrudes on activities such as driving, conversation, or eating, and it may cause lapses of attention or diminished cognitive abilities, such as missing an exit on the highway. The perception of sleepiness is reduced with prolonged sleep deprivation, so that chronically sleep-deprived individuals become accustomed to their impairment and fail to recognize their degree of sleepiness.

DIAGNOSIS

Clinicians should question hypersomnic patients for clues about sleep debt, dyssomnia, brain issues, or medical or psychiatric causes (Fig. 405-3). Patients should be queried regarding their schedule during the week and weekends. Information regarding sleep habits and environment may disclose important factors contributing to the sleepiness. Patients with sleep apnea (Chapter 100), narcolepsy, excessive periodic limb movements, circadian rhythm disorders, and parasomnias may have excessive daytime sleepiness as their main complaint. A history of snoring, observed apnea, morning headaches, cataplexy, sleep paralysis, hypnogogic hallucinations, or altered sleep schedule suggests contributions of a specific sleep disorder. Excessive sleepiness can also result from many medical disorders and medications. Patients with heart (Chapter 58), kidney (Chapter 131), or liver failure (Chapter 153), rheumatologic disease, or endocrinologic disorders such as hypothyroidism (Chapter 226) and diabetes (Chapter 229) may note sleepiness and fatigue. Neurologic disorders such as stroke (Chapters 407 and 408), tumor (Chapter 189), demyelinating disease (Chapter 411), and head trauma (Chapter 399) can cause excessive sleepiness. Sleepiness can be quantified subjectively by questionnaires or by physiologic measures such as a multiple sleep latency test. The Epworth Sleepiness

Excessive daytime sleepiness

Deficit in total sleep time

Voluntary restriction Work schedule Poor time management

Deficit in sleep quality Internal issues Sleep apnea Hypoventilation PLMD GERD Pain Heart failure Urinary frequency

External issues Noise Temperature Light Comfort Movement Smell?

2417

Intrinsic sleep mechanism disorder

Circadian rhythm disturbance

Fatigue

Narcolepsy w/ low orexin Narcolepsy w/o low orexin Idiopathic hypersomnia Periodic hypersomnia Neurological disorders

Advanced sleep phase Delayed sleep phase Irregular sleep type Free running Shift work Jet lag

Psychiatric and other medical issues

Secondary to: Medications Herbs and supplements Recreational drugs Other medical disorders FIGURE 405-3.  Differential diagnosis of excessive daytime sleepiness. GERD = gastroesophageal reflux disease; PLMD = periodic limb movement disorder.

2418

CHAPTER 405  Disorders of Sleep  

TABLE 405-3  EPWORTH SLEEPINESS SCALE How likely are you to doze off or fall asleep in the following situations, in contrast to just feeling tired? This refers to your usual way of life in recent time. Even if you have not done some of these things recently, try to work out how they would have affected you. Use the following scale to choose the most appropriate number for each situation. 0 = would never doze 1 = slight chance of dozing 2 = moderate chance of dozing 3 = high chance of dozing SITUATION

CHANCE OF DOZING

Sitting and reading Watching TV Sitting and inactive in a public place (theater or meeting) As a passenger in a car for an hour without a break Lying down to rest in the afternoon when circumstances permit Sitting and talking to someone Sitting quietly after lunch (without alcohol) In a car, while stopped for a few minutes in traffic Total

__________________ __________________ __________________ __________________ __________________ __________________ __________________ __________________ __________________

Adapted from Johns MW. A new method for measuring daytime sleepiness: the Epworth Sleepiness Scale. Sleep. 1991;14:540-545.

Scale quantifies sleepiness by asking the subject to rate on a scale of 0 to 3 (0, no chance; 3, high likelihood) the chance of dozing in eight situations (Table 405-3). A score of 7 is considered average, whereas a score of 10 or more is consistent with subjective sleepiness. This score has a modest correlation with physiologic measures of sleepiness but a better correlation with the respiratory disturbance index in patients with obstructive sleep apnea (Chapter 100). Daytime studies, the multiple sleep latency test, and the maintenance of wakefulness test, may be used to assess sleepiness or wakefulness across a series of trial “naps.” The multiple sleep latency test is validated as a test for narcolepsy, whereas the maintenance of wakefulness test gives a snapshot of the patient’s ability to stay awake.

Other Hypersomnias Idiopathic hypersomnia is a disorder in which hypersomnia cannot be explained by another disorder, is characterized by unrelenting hypersomnia, and is only minimally improved with therapy. Patients find that their symptoms persist despite long sleeping periods. These patients have average sleep latencies of less than 8 minutes, typically do not display REM sleep, but may have stage N3 on their multiple sleep latency test studies. Fluctuating symptoms of hypersomnia can also occur in other disorders such as in KleineLevin syndrome (a unique syndrome of periodic hypersomnia, hyperphagia, and hypersexuality) and in perimenstrual hypersomnia.

TREATMENT  Treatment of sleepiness should focus on correcting the underlying cause of sleepiness.4 Stimulants such as modafinil (200 to 400 mg) A1  should be used only in individuals who are impaired by the symptoms and in whom other therapies have failed to correct the hypersomnia. Some patients with KleineLevin syndrome respond to lithium (Chapter 397).

Narcolepsy

DEFINITION

Narcolepsy includes a tetrad of excessive sleepiness, cataplexy, sleep paralysis, and hypnogogic hallucinations. In the past, narcolepsy was divided into patients with cataplexy (type 1) and patients without it (type 2), but the subtypes of narcolepsy are also defined based upon the presence or absence of the neurotransmitter hypocretin-1.

EPIDEMIOLOGY

Narcolepsy with cataplexy (narcolepsy type 1) affects1 in 2000 to 6000 individuals; 40 to 80% have the complete tetrad, and approximately 50% complain of sleep disruption. Over 90% of individuals in the United States with cataplexy have the HLA-DQB1*0602 gene,5 and a similar percentage have

low CSF hypocretin-1 levels. Narcolepsy without cataplexy (narcolepsy type 2) occurs in about 2 per 1000 individuals; approximately 40% have the HLADQB1*0602 gene, and fewer exhibit low CSF hypocretin-1 levels. Despite the connection to a gene, the risk to first-degree relatives is only 1 to 2%, or about a 10- to 50-fold increased risk compared with the general population.

PATHOBIOLOGY

Narcolepsy with cataplexy (narcolepsy type 1) reflects the loss of hypocretinproducing neurons in the lateral hypothalamus. This neurotransmitter is important for stabilizing the sleep-wake state and for motor control. Thus the manifestations of the disease are related to frequent stage shifts and intrusion of REM sleep atonia into wakefulness. Why these neurons are lost is not known, but immune mechanisms are postulated.6

CLINICAL MANIFESTATIONS

The tetrad of excessive sleepiness, cataplexy, hypnogogic hallucinations, and sleep paralysis are the major clinical manifestations. Cataplexy is abrupt loss of muscle tone triggered by strong emotional stimuli such as laughter, surprise, or anger. Patients are aware of their surroundings but lose muscle control, first in the face and neck, followed by the arms and then the trunk and legs. Hypnogogic (sleep-onset) and hypnopompic (sleep-offset) hallucinations are vivid and often frightening visual or auditory events. Sleep paralysis is an inability to move or speak, typically during the transition out of sleep when individuals have complete or partial awareness of their surroundings. Patients may describe a strong feeling of impending doom, being chased, or having to escape imminent danger. Patients with narcolepsy are often considered perpetually sleepy, but most have normal sleep duration over a 24-hour period. However, their sleep is fragmented, with sleep intruding into daily activities and interrupted at night with wakefulness. Sleep paralysis and hypnogogic hallucinations can occur in normal individuals, especially after sleep deprivation, but cataplexy is virtually pathognomonic for narcolepsy.

DIAGNOSIS

The diagnosis of narcolepsy type 1 and type 2 is based upon a mean sleep latency of less than 8 minutes and the presence of REM sleep on at least two of the five naps of a multiple sleep latency test. The multiple sleep latency test is predicated on the documentation of at least 6 hours of sleep prior to the study. The previous night’s polysomnography must also not show other sleep pathologies. A low cerebrospinal fluid hypocretin level in the setting of excessive sleepiness can also confirm of the diagnosis of narcolepsy type 1, but this finding is not seen in type 2.

TREATMENT  Treatment of narcolepsy focuses on improving symptoms of excessive sleepiness, cataplexy, and REM sleep intrusion into wakefulness (Table 405-4). Sleepiness requires a three-pronged approach of improving the quality and quantity of nighttime sleep, scheduling naps, and prescribing stimulants. Nighttime sleep may be improved with sodium oxybate (20 to 40 mg/kg in divided nighttime doses), which improves daytime alertness and reduces cataplexy. A2  Stimulants such as modafinil (100 to 600 mg/day), armodafinil (50 to 250 mg/day), methylphenidate (5 to 60 mg/day), and dextroamphetamine (5 to 60 mg/day) improve daytime function but do not return the patient to a normal level.7 Patients should not use stimulants in the evening and night hours. Selective serotonin reuptake inhibitors (SSRIs) (Chapter 397, Table 397-5) and combined serotonin-norepinephrine reuptake inhibitors (SNRIs) (Chapter 397, Table 397-5) also reduce cataplexy as well as sleep paralysis and hallucinations.

PROGNOSIS

Narcolepsy is lifelong disorder. Patients who present in adolescence or young adulthood may progress to more severe symptoms, but the disorder does not affect longevity.

  SLEEP-RELATED BREATHING DISORDERS

Sleep Apnea

Sleep apnea (Chapter 100) is defined by repetitive breathing pauses that may interrupt sleep hundreds of times per night. The patient’s bed partner may note that the patient’s breathing has stopped or that the individual “holds their breath,” but most patients are unaware of the events. In the sleep laboratory, apneas are defined by cessation of breathing for more than 10

2419

CHAPTER 405  Disorders of Sleep  

TABLE 405-4  THERAPIES FOR NARCOLEPSY MODALITY

STARTING DOSE

HIGHEST DOSE

DOSE AT

10-15 minutes

15 minutes

Just prior to time needing to be awake

25 mg

300 mg

am

Modafinil

100-200 mg

600 mg

am and noon

Armodafinil

50-150 mg

250 mg

am

Methylphenidate

5-10 mg

120 mg

am and noon

Methylphenidate ER

10-20 mg

120 mg

am

Dextroamphetamine

5-10 mg

60 mg

am and noon

Combination dextroamphetamine/amphetamine

5-10 mg

60 mg

am and noon

225 mg

900 mg

Bedtime and 4 hr into sleep

Scheduled naps OVER-THE-COUNTER STIMULANT Caffeine STIMULANTS

IMPROVE NIGHTTIME SLEEP, DAYTIME ALERTNESS, AND CATAPLEXY Sodium oxybate THERAPIES FOR CATAPLEXY (NOT FDA APPROVED) Fluoxetine

10-20 mg

40 mg

am

Venlafaxine

75 mg

225 mg

am at lower dose or divided

Protriptyline

5 mg

40 mg

am at lower dose or divided

seconds and are usually associated with oxygen desaturation and arousal occurring more frequently than five events per hour of sleep. Sleep apnea is classified as two major forms: obstructive and central. Obstructive apnea is defined as the loss of flow due to obstruction, typically in the upper airway, whereas central apnea is the absence of airflow due to the absence of effort. Obstructive apnea is the most common form of sleep apnea. Approximately 50% of patients with sleep apnea have daytime sleepiness, but other symptoms such as insomnia and parasomnia events may be clues to underlying obstructive sleep apnea. Standardized questionnaires (Table 405-5) can help select patients for definitive polysomnography (Chapter 100, Fig. 100-1). Treatment typically involves the use of continuous positive airway pressure (CPAP), an oral appliance, or surgery. Neither supplemental oxygen A3  nor medication A4  provide substantial benefit. Central apnea is the absence of ventilation without an effort to breath (E-Fig. 405-6) (Chapter 86). These patients have respiratory pauses that are associated with oxygen desaturation and arousals. Central apneas can be caused by cardiac disease, narcotics, or neurologic abnormalities that result in dysregulation of respiration. Cheyne-Stokes breathing, which may have features of both central and obstructive apnea, often occurs only during sleep. The classical Cheyne-Stokes pattern of crescendo-decrescendo breathing with central apnea can be seen in individuals with heart failure, neurologic lesions, and metabolic or toxic encephalopathies. Central apneas can be diagnosed by overnight polysomnography. The addition of a CO2 monitor can distinguish between apnea related to normal or low CO2 versus high CO2 and help direct treatment. Low CO2 levels in the presence of apnea may suggest a high CO2 apnea threshold that may respond to increasing CO2 levels, whereas apnea in the setting of an elevated CO2 level would suggest failure of the respiratory control mechanism, as seen in patients who are taking narcotics. Therapy depends upon the etiology but can include reduction or elimination of respiratory suppressants (narcotics), CPAP, bilevel PAP, nocturnal ventilation, and respiratory stimulants.

HYPOVENTILATION

Hypoventilation, as defined by elevated CO2 levels (Chapter 86), may occur solely during sleep. Patients may note daytime sleepiness, fatigue, morning headache, or unrefreshing sleep. Although the prevalence is unknown, hypoventilation is common in individuals with central obesity, neuromuscular disease, pulmonary disease, and narcotic use. Although typically worse in REM sleep, the elevation of CO2 and commonly coexisting drop in oxygen saturation is more prolonged than the pattern seen with sleep apnea. Hypoventilation syndrome is treated with positive airway pressure or noninvasive ventilation.

Insomnia

DEFINITION

Insomnia is the complaint of difficulty initiating or maintaining sleep, or of unrefreshing sleep that results in daytime symptoms of excessive fatigue or

TABLE 405-5  STOP-BANG QUESTIONNAIRE FOR OBSTRUCTIVE SLEEP APNEA Snoring Do you snore loudly? (louder than talking or loud enough to be heard through closed doors)

Yes

No

Yes

No

Yes

No

Yes

No

Yes

No

Yes

No

Yes

No

Yes

No

Tired Do you often feel tired, fatigued, or sleepy during the daytime? Observed Has anyone observed you stop breathing during your sleep? Blood pressure Do you have or are you being treated for high blood pressure? Body Mass Index (BMI) BMI more than 35 kg/m2? Age Age older than 50 yr? Neck circumference Neck circumference greater than 40 cm? Gender Gender male? Elevated risk of OSA: answering yes to three or more items Low risk of OSA: answering yes to less than three items Adapted from Chung F, Yegneswaran B, Liao P, et al. STOP questionnaire: a tool to screen patients for obstructive sleep apnea. Anesthesiology. 2008;108:812-821.

impairment of performance.8 Daytime sequelae differentiate individuals with a limited need for sleep from individuals with insomnia. Chronic insomnia is defined by symptoms that persist more than 1 month.

EPIDEMIOLOGY

Most individuals have occasional nights with difficulty falling asleep or maintaining sleep, often provoked by psychological challenges or sudden changes in their environment. Approximately 35% of individuals complain of intermittent difficulty with sleep, and approximately 10% have chronic insomnia. Women, older individuals, and patients with psychiatric or chronic medical illness are predisposed to develop insomnia. Insomnia is also more common in individuals with lower socioeconomic status and poor education. Patients with behavioral traits such as obsessive-compulsive tendency, frequent rumination, or poor coping strategies are also at greater risk for insomnia. Lack of “good-quality” sleep disrupts life and may lead to other symptoms.

2419.e1

CHAPTER 405  Disorders of Sleep  

E1-M2 E2-M2 F3-M2 F4-M1 C3-M2 O1-M2 [Chin1-Chi EKG [ARM1-AR [LLEG1-LL [RLEG1-RI PSnor CFlow PNasal [Chest] [Abd] SaO2

94

94

94

94

94

94

95

95

95

95

95

95

95

95

95

95

95

95

94

94

94

95

EtCO2

40

40

40

39

38

38

37

37

37

37

37

37

37

37

37

37

37

37

37

37

38

38

Leak

2

2

2

2

2

2

2

1

2

3

2

3

3

3

3

2

2

2

2

2

3

2

E-FIGURE 405-6.  This epoch from a polysomnographic recording shows repetitive central sleep apnea events (blue arrows). The patient has no chest movement, indicating this event is central in nature.

2420

CHAPTER 405  Disorders of Sleep  

PATHOBIOLOGY

Patients with insomnia frequently give historical clues directed toward the mechanisms behind their insomnia. Studies on patients with insomnia show these individuals are in a state of hyperarousal. Increased brain metabolic rates during NREM sleep may provide a neurophysiologic basis for chronic insomnia. Most patients have multiple factors that contribute to the insomnia, including features that predispose them to insomnia, events that precipitated the insomnia, and behaviors that perpetuate the insomnia. Effective treatment requires identifying these contributing factors. Many patients have a coincident psychiatric disorder (Chapter 397) or psychological or medical issues. Patients with depression or anxiety may have insomnia for years prior to the presentation of the affective disorder. Patients with heart (Chapter 58), liver (Chapter 153), or renal (Chapter 131) failure or disturbances of the gastrointestinal or respiratory systems commonly complain of insomnia. Patients with heart failure may note difficulty remaining in bed owing to breathing issues. Restless legs syndrome frequently presents as insomnia. Pain of any origin can interrupt sleep, and patients with limited mobility, such as muscular dystrophy (Chapter 421) or Parkinson disease (Chapter 409), may have pressure points that awaken them. Sleep schedules may be influenced by disease (e.g., patients with dementia [Chapter 402] in whom circadian rhythm abnormalities promote nighttime awakenings).

CLINICAL MANIFESTATIONS

The patient’s symptom complex may give clues to a poor sleep environment, maladaptive behaviors, psychological stress, psychiatric or neurologic disease, primary sleep disorder, or other medical issues. Insomnia may be initiated by events that shift schedules or by a change in medications. Initiating events may play little role in long-term insomnia but give important clues to preventing further recurrence of the insomnia. If insomnia persists, many patients adopt behaviors that perpetuate the insomnia. Maladaptive habits that may occur during the day or night include heavy daytime caffeine or alcohol use, watching television or playing video games while in bed, or eating or exercising during the usual sleep period. A subgroup of patients may develop sleep phobias or have anxiety about the oncoming sleep period. This expectation of poor sleep promotes apprehension about sleep and may perpetuate counterproductive sleep rituals. These maladaptive behaviors become the predominant feature of the subtype of psychophysiologic insomnia (Table 405-6). Some patients exaggerate their symptoms, whereas other patients may not perceive that they are asleep. Individuals with paradoxical insomnia have normal physiologic sleep but do not recognize that they have been

asleep. Other patients may have the unrealistic expectation that sleep should not be interrupted by any arousals or that they must sleep a set number of hours. Rarely, idiopathic insomnia starts in childhood and continues as a lifelong difficulty of sleep. These patients may have defective sleep mechanisms. Noting the timing of the insomnia during the sleep period may also be helpful. Difficulty with the onset of sleep suggests an underlying delayed sleep phase, and insomnia with early morning arousal suggests underlying depression or advanced sleep phase. Documentation of schedule changes (e.g., from jet lag or shift work) can be useful in determining links to circadian rhythm issues.

DIAGNOSIS

The diagnosis of insomnia is based upon the patient’s history that difficulty with sleep results in daytime sequelae (Fig. 405-4). Frequently, more than one subtype occurs in the same patient, and there is little evidence that subtypes direct therapy.

TABLE 405-6  CLASSIFICATION OF ADULT INSOMNIA INSOMNIA Subtypes: Psychophysiologic insomnia—maladaptive behaviors conditioned in response to associating the bed environment or thoughts of bedtime with heightened arousal; patients typically sleep better in a different environment, such as away on vacation. Idiopathic insomnia—insomnia beginning in infancy or childhood, with a persistent unremitting course and no improvement with change in environment Paradoxical insomnia (sleep state misperception)—insomnia characterized by a marked mismatch between the patient’s description of sleep duration and objective polysomnographic findings INSOMNIA ASSOCIATED WITH Adjustment insomnia—associated with an acute or active psychosocial stressor Inadequate sleep hygiene—associated with lifestyle habits that impair the ability to sleep Insomnia comorbid with a psychiatric disorder—associated with an active psychiatric disorder such as anxiety or depression Insomnia comorbid with a medical condition—associated with a condition such as renal failure, hepatic failure, chronic pain, nocturnal cough or dyspnea, or hot flashes Insomnia caused by a drug or substance—secondary to consumption or discontinuation of medications, drugs of abuse, alcohol, or caffeine

Insomnia

Disturbance of sleep continuity Sleep Disturbances Sleep apnea Hypoventilation RLS GERD Pain Heart failure Other medical disorders

External issues Noise Temperature Light Bedding Movement

Circadian rhythm disturbance

Advanced sleep phase Delayed sleep phase Irregular sleep type Free running Shift work Jet lag

Intrinsic sleep mechanism disorder

Insomnia Subtypes: Psychophysiologic Idiopathic Paradoxical

Psychiatric disorder

Depression Anxiety Mania Bipolar disorder PTSD Adjustment disorder Schizophrenia

Psychological issues

Sleep phobia Stress Conflict Change

Secondary to: Medications Herbs and supplements Recreational drugs Other substances FIGURE 405-4.  Differential diagnosis of insomnia. GERD = gastroesophageal reflux disease; PTSD = post-traumatic stress disorder; RLS = restless leg syndrome.

2421

CHAPTER 405  Disorders of Sleep  

The history should include a review of the patient’s 24-hour schedule, meals, caffeine, tobacco and medicine intake, sleep environment, attitudes about sleep, and the sleep experience. In addition, a thorough history from the bed partner may disclose features the patient is unaware of, such as snoring, limb movements, and sleep habits. Patients should be asked to keep a diary of their daily events for at least 3 weeks. This diary will often show specific patterns or clues the patient is unaware of. Actigraphy can also help determine the patient’s sleep-wake schedule. Polysomnography should be considered only if the patient has symptoms of sleep apnea or has failed multiple therapeutic trials.

TREATMENT  Insomnia is generally treated as a single disorder even though it has been divided into subtypes (see Table 405-6), but effective treatment requires identifying factors that contribute to insomnia. Treatment is multipronged and includes improving behaviors that promote sleep, addressing the perpetuating factors, and deciding if hypnotic medication is appropriate. Every patient with insomnia needs to develop good sleep hygiene habits that should be reviewed with the patient and bed partner. Cognitive behavioral therapy  provides long-term success for insomnia (Table 405-7), but a combination of cognitive behavioral therapy with hypnotics outperforms either alone. The cognitive portion focuses on restructuring beliefs about sleep, whereas behavioral therapies focus on actions that may mitigate maladaptive behaviors and promote better sleep behaviors: progressive relaxation techniques, stimulus control, sleep or time-in-bed restriction, and factors that accentuate homeostatic and circadian drives. Hypnotics are best used for short-term treatment while starting cognitive behavioral therapy (Table 405-8). The benzodiazepine receptor agonists zolpidem (5 to 10 mg [CR form, 6.25 to 12.5 mg]), zaleplon (5 to 10 mg), and longeracting eszopiclone (1 to 3 mg) are the usual initial therapies. Agents with rapid onset and a short half-life are used for difficulties initiating sleep. Agents with a longer half-life or continuous-release agents are used for sleep maintenance. Melatonin A5  or a melatonin receptor agonist (ramelteon 8 mg) improves the initiation and maintenance of sleep. Antidepressant medications (Chapter 397, Table 397-5) are also used for insomnia. Low-dose doxepin (3 and 6 mg) promotes sleep by blocking the effects of central nervous system histamine. In patients with insomnia and an underlying affective disorder, the combination of the short-term use of a hypnotic, such as a benzodiazepine receptor agonist, and long-term antidepressant or anxiolytic is better for both the insomnia and affective disorder than either therapy alone.

PROGNOSIS

Most patients will improve, although some will relapse. Intractable insomnia often heralds an affective disorder (Chapter 397). Except for very rare patients with the prion-induced fatal familial insomnia (Chapter 415), patients with insomnia have only a slightly lower life expectancy, primarily from cardiovascular disease.9

Circadian Rhythm Disorders

DEFINITION

Circadian rhythm disorders cause misalignment of the person’s sleep-wake cycle and the naturally occurring day-night cycle. Pathologic symptoms must be persistent or recurrent, and the patient must incur some social, occupational, or additional impairment. Individuals may note insomnia, excessive daytime sleepiness, or both. Circadian rhythm sleep-wake disorder is typically classified by comparing the patient’s rhythm to the naturally occurring day. Circadian rhythm sleep-wake disorders are subtyped into advanced phase type (early to bed and rise), delayed phase type (late to bed and rise), irregular type (no clear pattern), and free-running type (no entrainment to the environment). Additionally, the inciting situation is included (e.g., jet lag type or shift work type). Another disorder related to circadian rhythm involves the gastrointestinal cycle: night eating syndrome, in which individuals consume over half of their caloric intake after 9 pm.

TABLE 405-7  NONPHARMACOLOGIC THERAPIES FOR INSOMNIA COGNITIVE BEHAVIORAL THERAPY with or without relaxation therapy (Standard) The combination of multiple modalities noted below STIMULUS-CONTROL THERAPY (Standard) Go to bed only when sleepy. Use the bedroom only for sleeping and sex. Go to another room when unable to sleep in 15 to 20 minutes, read or engage in other quiet activities, and return to bed only when sleepy; repeat if necessary. Have a regular wake time regardless of the duration of sleep. Avoid daytime napping. SLEEP-RESTRICTION THERAPY (Guideline) Reduce time in bed to the estimated total sleep time (minimum, 5 hr). Increase time in bed by 15 minutes every week when the patient estimates the sleep efficiency is at least 85% (ratio of time asleep to time in bed). RELAXATION THERAPY (Standard) Physical component: progressive muscle relaxation, autogenic training Mental component: reducing intrusive thoughts through imagery training, meditation, or hypnosis PARADOXICAL INTENTION (Guideline for sleep-onset difficulties) Instruct the patient to remain passively awake in bed and avoid any effort to fall asleep. COGNITIVE THERAPY (Insufficient evidence as a single therapy) Education to alter maladaptive or unrealistic beliefs and attitudes about sleep, such as that a minimum of 8 hours of sleep per night is required for health. SLEEP HYGIENE EDUCATION (Insufficient evidence as a single therapy) Correction of extrinsic factors and behaviors that affect sleep, such as environmental disruption (pets, music, or television ); bedroom temperature; fixation on the bedside clock; use of alcohol, nicotine, or caffeine; lack of exercise or exercise too close to bedtime.

TABLE 405-8  MEDICATIONS FOR INSOMNIA NAME Melatonin

DOSE 0.25-6 mg

TIME OF DOSE

FDA INDICATION

Evening 1-3 hr before bed

No

Grogginess, headache

COMMON SIDE EFFECTS

HALF-LIFE

MECHANISM

30-50 min

Melatonin receptor agonist (dark)

Tryptophan

1-15 g

Evening

No

Drowsiness, headaches, dizziness

1-3 hr

Modulates serotonin

Zolpidem SL

1.75-10 mg

Bedtime

Yes

Sleepiness, amnesia, falls parasomnias

1-2 hr

Benzodiazepine receptor agonist (BZRA)

Zolpidem reg

5-10 mg

Bedtime

Yes

Sleepiness amnesia, falls, parasomnias

1-2 hr

BZRA

Zolpidem CR

6.25-12.5 mg

Bedtime

Yes

Sleepiness, amnesia, falls

1-2 hr but continued release

BZRA

Zaleplon

5-20 mg

Bedtime

Yes

Sleepiness, dizziness, parasomnias

1 hr

BZRA

Eszopiclone

1-3 mg

Bedtime

Yes

Sleepiness, dizziness

4-8 hr

BZRA

Doxepin

3-6 mg

Bedtime

Yes

Drowsiness, dizziness, nausea

17 hr

Histamine receptor antagonist

Mirtazapine

7.5-15 mg

Bedtime

No

Drowsiness, dizziness, weight gain

20 hr

Histamine receptor antagonist

Ramelteon

4-8 mg

Bedtime

Yes

Sleepiness, headache

1-2 hr

Melatonin receptor agonist

FDA = U.S. Food and Drug Administration.

2422

CHAPTER 405  Disorders of Sleep  

EPIDEMIOLOGY

The prevalence of circadian rhythm disorders is not known. Some patterns of sleep are inherent in specific age groups. Advanced sleep phase issues are more common in the elderly, and delayed sleep phase issues are more common in adolescents, but these stereotypes may not indicate the true prevalence of the disorder. Purposeful shifting of the circadian rhythm, such as with shift work or jet lag, is common, but although 28% of the U.S. workforce works nights or rotating shifts, only one third of these individuals have this disorder. A free-running circadian rhythm is more common among blind persons, of whom about 25% have the disorder.

PATHOBIOLOGY

Circadian rhythm sleep-wake disorders may be more prevalent in today’s “24-hour society,” which offers constant stimuli to remain awake. Teens are more vulnerable to the phase delaying effects of light in the evening. The human “master clock” resides in the suprachiasmatic nucleus of the hypothalamus, but peripheral tissues also generate a self-sustained circadian rhythm based upon clock gene expression. About 2 to 10% of genes are expressed with circadian rhythmicity. Abnormalities in genetic clock genes may contribute to circadian rhythm disorders.10 Variations in the Clock, Per2, and Per3 genes appear to influence the morning/evening preference. Advance sleep phase type has been associated with the Per2 S662G mutation and the Ck1d T44A mutation, whereas delayed sleep phase type is associated with the Per3 V647G and Ck1e S408N mutations. The latter mutation is also associated with free-running type. Free-running type in individuals who are blind appears to be related to the loss of the photoreceptive ganglion cell input to the hypothalamus and not into the retina itself.

CLINICAL MANIFESTATIONS

Circadian rhythm disorders cause insomnia and/or excessive sleepiness. Patients may incur sleep deprivation by trying to maintain schedules that are not consistent with their inherent clocks. Some individuals “catch up” on the weekends by sleeping during their preferred times. Once asleep, the patient has sound sleep. Circadian rhythm sleep-wake disorders are associated with an increased risk of accidents and impair quality of life. Delayed phase–type patients typically have trouble falling asleep and may not fall sleep for over 2 hours later than the conventional bedtime (E-Fig. 405-7A). They then have trouble arousing in the morning, preferring late wake times. Advanced phase–type patients fall asleep early in the evening and awaken several hours earlier than the conventional morning awakening (see E-Fig. 405-7B). Patients complain of early morning awakening and the inability to maintain wakefulness during evening activities. Free-running type individuals have a circadian rhythm that continues to run on the 24.3- to 25-hour cycle. In this disorder, also known as non-24-hour sleep-wake rhythm disorder, patients have alternating episodes of insomnia and excessive sleepiness, depending upon the phase of the endogenous sleep-wake cycle. This disorder can be easily confused with a periodic hypersomnia. Irregular-type patients have both excessive sleepiness and insomnia, with deceased functioning during the waking period. Jet lag–type patients are affected by temporary changes in the environment owing to travel across time zones. Most find travel west easier than travel east because of the inherent nature of the clock being longer than 24 hours. Shift work–type patients suffer from circadian misalignment because most shift workers try to resume a typical diurnal pattern on their days off despite nocturnal waking on their work days. This schedule predisposes the worker to poor adaptation.

DIAGNOSIS

The diagnosis of a circadian rhythm disorder is made by history and a 14-day sleep diary or actigraphy recording.11 Normal individuals have a tendency toward “morningness” or “eveningness,” so the diagnosis requires documentation of a negative impact of circadian rhythm on quality of life.

TREATMENT  Most therapy is directed toward aligning the circadian rhythm with the desired sleep-wake schedule, commonly by gradually shifting the sleep-wake schedule and then maintaining the schedule in the correct phase. Shifting the schedule, known as chronotherapy, can be accomplished by allowing a gradual delay or free running of the inherent schedule (promoted with the use of time clues) to move the phase of the circadian rhythm to the desired time.

Time clues may also be used to fix a circadian rhythm into a specific phase, but the circadian rhythm is susceptible to such clues only if they are given at an appropriate time of the endogenous circadian rhythm. For example, bright light given to a normal individual in the evening delays the cycle, whereas bright light in the morning may advance the cycle. A reliable point of reference is the nadir of the temperature cycle, which typically occurs approximately 2 hours prior to the natural wake up time. Typically, bright light, exercise, food, and social interactions delivered prior to the temperature nadir will delay the cycle, whereas these stimuli delivered after the temperature nadir will advance the cycle. Melatonin has an opposite effect and typically advances the cycle if given 2 to 4 hours prior to the onset of sleep and may delay the cycle if used after the temperature nadir. After chronotherapy realigns the circadian rhythm, patients with delayed phase type may benefit from morning bright light or from melatonin in the evening. These individuals are subject to relapses, and the schedule should be strictly maintained. Patients with advanced phase type may be misdiagnosed with depression,12 but they may benefit from evening bright light to delay the onset of sleep. Jet lag can be improved by appropriate use of time clues in reference to the endogenous cycle. Although short-term hypnotics and stimulants are commonly used to help adjustment, these do not realign the circadian rhythm any faster than the usual one time zone per day. For shift-work disordered sleep, modafinil 100 to 200 mg at the start of the shift may help maintain alertness. A6  Patients can wear sunglasses to minimize the phase shifting effects of the morning light. Melatonin once they return home and short-term use of a hypnotic may increase sleep duration. Alternatively, some shift workers may find a modified shift in their sleep schedule more appealing—sleeping 8 am to 4 pm on their work days and 4 am to noon on their days off. With this schedule, the circadian rhythm shifts by only 4 hours, so patients experience fewer total of hours shifting in any period.

Parasomnia

DEFINITION

Parasomnias are undesirable behavioral events or experiential phenomena occurring during entry into, within, or as part of arousal from sleep. These events include abnormal movements, behaviors, emotions, perceptions, dreaming, and activities of the autonomic nervous system. Parasomnias are typically subdivided into disorders of arousal from NREM sleep, REM sleeprelated parasomnias, and other parasomnias. The disorders of arousal include disorders of sleepwalking, sleep terrors, and confusional arousal. REM-related parasomnias include nightmare disorder, REM sleep behavior disorder, and recurrent sleep paralysis. Other parasomnias include sleep-related eating, catathrenia (repetitive nocturnal groaning), or exploding head syndrome.

EPIDEMIOLOGY

Approximately 3% of adults and 15% of children have a sleep-related behavior. Although some parasomnias, such as disorders of arousal from NREM sleep (sleepwalking, sleep terrors, and confusional arousals) are more common in children, others have no age predilection or are more common in older individuals. REM-related parasomnias, such as nightmare disorder, are common among all ages and especially individuals with post-traumatic stress disorder (Chapter 397). REM sleep behavior disorder, which is another REM sleep–related parasomnia, is more common in the elderly, but the exact prevalence is unknown.

CLINICAL MANIFESTATIONS

Individuals and bed partners may complain of frequent movement during sleep. This complaint may be more concerning to the bed partner than the patient. Some individuals will complain of being active sleepers.

DIAGNOSIS

The history is the mainstay of the diagnosis of most parasomnias. Key features include age of onset, time of night of the events, memory for the events, and family history (Table 405-9). Stereotypical behavior, the same behavior with each event, can also help in categorizing the events. Events such as periodic limb movements, rhythmical movement disorder, or epileptic seizures are stereotypical, whereas sleepwalking, sleep or night terrors, and dream enactment have different behavior with each event. Although historical features can help distinguish among these disorders, many patients may require polysomnography to delineate the cause (see Table 405-9).

DISORDERS OF AROUSAL FROM NREM SLEEP

These disorders include a spectrum of behaviors that occur as a partial arousal from deep NREM sleep: sleep walking, sleep terrors, and confusional arousals. Although probably a continuum, the individual symptoms distinguish the

CHAPTER 405  Disorders of Sleep

Wakefulness



6:00 AM Noon 6:00 PM MN 6:00 AM Noon

Wakefulness

A

B

6:00 AM Noon 6:00 PM MN 6:00 AM Noon

E-FIGURE 405-7.  A, An idealized graph of the circadian rhythm (solid orange curve), shows the phase shift delay (dotted curve) caused by bright light given prior to the temperature nadir. B, This idealized graph of the circadian rhythm (solid blue curve), shows the phase shift advance (dotted curve) caused by bright light given after the temperature nadir.

2422.e1

CHAPTER 405  Disorders of Sleep  

2423

TABLE 405-9  KEY FEATURES OF NOCTURNAL EVENTS DISORDER

SYMPTOMS

TIME OF NIGHT

DURATION

FREQUENCY

STEREOTYPICAL

MEMORY

POLYSOMNOGRAPHIC FINDINGS

Sleepwalking

Slow, deliberate, First half of sleep complex behaviors period

Seconds to minutes

Less than one per night to fewer

No

No or partial Arousal from slow wave vague memory sleep

Sleep terrors

Piercing scream, followed by fight or flight response

First half

Seconds to minutes

Less than one per night or fewer

No

No or partial Arousal from slow wave vague memory sleep

Confusional arousals

Variety of unusual behaviors upon sudden awakening

Anytime

Seconds to minutes

Less than one per night or fewer

No

No or partial Arousal from slow wave vague memory sleep

Sleep-related eating

Eating of high-calorie First half or strange foods in a messy manner

Minutes

May occur nightly

No

No or partial Arousal typically from vague memory NREM sleep

Nightmares

Frightening dreams associated with anxiety

Latter half

Seconds to minutes

Variable

No, but may have a common theme

Yes

Events occur in REM sleep

REM sleep behavior disorder

Dream enactment, may be violent

Latter half

Seconds

Multiple times per night

No

Yes

Excessive EMG activity in REM sleep

Rhythmic movement disorder

Rocking, head banging

Near sleep onset but may be throughout the night

Minutes to hours

Multiple times per night

Yes

Yes

Rhythmic movement in transition from waking to sleeping

Catathrenia

Nocturnal prolonged moaning

Intermittent throughout the night

Minutes to hours

Multiple

Yes

No

Prolonged expiratory moans and groans, with slowed respiratory rate

Exploding head syndrome

Loud painless sound of explosion inside the head

Near the onset of sleep

Seconds

Rare, typically infrequent

Yes

Yes

Typically events are close to sleep onset

EMG = electromyogram; NREM = non–rapid eye movement; REM = rapid eye movement.

disorders. Individuals with sleepwalking have ambulation as part of their episodes, whereas sleep terrors are accompanied by a piercing scream or cry and expression of intense fear. Confusional arousals are characterized by disorientation, slow speech, and mentation or inappropriate behavior such as eating, fighting, or sexual intimacy. Pathobiologically, these individuals have NREM sleep simultaneously with the awake state. These events are more common in the first third of the night, are associated with no or little memory for the event, and are not stereotypical. Events are more likely to occur after sleep deprivation, alcohol ingestion, sleeping in strange environments, or coincidental conditions such as sleep apnea that evoke arousals. Patients are neurologically and psychiatrically normal during wakefulness. Polysomnography shows the episodes occur during slow wave sleep, with some features of wakefulness. Therapy includes ensuring safety for those who may injure themselves or others (e.g., placing the bed on the floor, blocking windows, or moving the patient’s bedroom to the ground floor), decreasing factors that may cause arousals, and avoiding inciting factors such as sleep deprivation or alcohol. There are no established medications, but treatment of sleep apnea (Chapter 100) appears to reduce events. Other treatments, such as clonazepam 0.5 to 2 mg and tricyclic antidepressants (Chapter 397, Table 397-5), have been tried with varying success.

REM SLEEP BEHAVIOR DISORDER

In REM sleep behavior disorder, patients lose the characteristic sleep and muscle atonia of REM sleep and act out during their dreams.13 REM sleep behavior disorder can be violent, with patients injuring themselves or bed partners. This elaborate motor activity is often associated with vivid recall of a dream that correlates with the witnessed behavior. Patients can have single or multiple events, commonly in the latter half of the night. REM sleep behavior disorder usually begins in late adulthood, but it can occur in children. This behavior disorder can be induced by medications such as tricyclic antidepressants, monoamine oxidase inhibitors, and serotonin reuptake inhibitors. Chronic REM sleep behavior disorder has been linked to alphasynucleinopathies and the subsequent development of disorders such as Parkinson disease (Chapter 409), multiple system atrophy (Chapter 409), and Lewy body dementia (Chapter 402); over two thirds of adult patients with REM sleep behavior disorder eventually develop a neurodegenerative disease. The diagnosis is based upon the presence of excessive electromyographic

activity during REM sleep and the history of dream enactment. Patients should be evaluated for signs of degenerative disorders (Chapters 402 and 409), strokes (Chapters 407 and 408), posterior fossa tumors (Chapter 189), or demyelinating disease (Chapter 411). Most patients respond well to clonazepam (0.25 to 3 mg) or melatonin (1 to 9 mg).

NIGHTMARES

Nightmares or recurrent disturbing dreams can be a presenting symptom of a sleep disturbance. Nightmares are emotionally intense dreaming associated with fear, anxiety, anger, sadness, or other negative emotions. Individuals awaken from stage R or light NREM sleep to full alertness and usually recall the event immediately. Nightmares are most commonly associated with a psychologically disturbing event, but they also may occur as a result of antihypertensive medications, antidepressants, or dopamine agonists. If related to medication, treatment starts with removal of the provocative substance. Prazosin (5 to 20 mg) and imagery rehearsal may be effective.

OTHER PARASOMNIAS

Individuals with sleep-related eating disorder consume high-calorie, sometimes bizarre, foods during sleep and have no or little memory for the consumption. They have morning anorexia and unexplained weight gain. Catathrenia is a rare disorder characterized by repetitive nocturnal groaning. Bed partners usually express concern because the patient has long expiratory groans that sound mournful. These patients respond to CPAP. Rhythmic movement disorder includes a variety of stereotyped movements, usually involving large muscles, that are sustained into light sleep. Movements may include head banging, body rocking, leg rolling, humming, and chanting. Patients are unaware of the movement or describe the movement as a compulsion prior to sleep. This behavior is difficult to treat but diminishes with age. Exploding head syndrome is an abrupt sensation or perceived loud sound of an explosion near the onset of sleep. It is painless, and the events are not a harbinger of other underlying pathology.

RESTLESS LEGS SYNDROME

Restless legs syndrome (Chapter 420) is characterized by four essential features: discomfort or urge to move, worse with rest, better with movement, and worse in the evening.14 Patients may complain of an unpleasant crawling

2424

CHAPTER 406  Approach to Cerebrovascular Diseases  

or deep unusual sensation in the legs or arms, with improvement after moving the extremities. Patients with restless legs syndrome may relay that the discomfort can be debilitating and cause them to walk or continuously move their legs until the early morning hours. Some patients note that their legs will move or dance on their own, thereby indicating periodic limb movements in wakefulness. About 85 to 90% of restless legs syndrome patients will have periodic limb movements in sleep, but only a minority of patients with periodic limb movements in sleep will meet the clinical criteria of restless legs syndrome. FDA-approved therapies for restless legs syndrome are dopamine agonists (pramipexole 0.125 to 1.5 mg or ropinirole 0.25 to 3 mg), transdermal rotigotine (1 to 3 patch/24 hours), and gabapentinoid medications (e.g., gabapentin-encarbil 600 to 1800 mg).15 Augmentation, in which a dopamine agonist increases the severity of symptoms, is treated by carefully substituting another agent, such as pregabalin (300 mg daily), A7  for the dopamine agonist. In some patients, restless legs syndrome has linked to low iron in the central nervous system. Some patients improve with oral iron therapy (e.g., 325 mg ferrous sulfate two to three times per day for 3 to 4 months until ferritin levels exceed 50 mg/L and iron saturations exceed 20%), and some require more aggressive intravenous iron therapy. More intractable patients may require chronic narcotics (Chapter 30, Table 30-4). A8 

Grade A References A1. Philip P, Chaufton C, Taillard J, et al. Modafinil improves real driving performance in patients with hypersomnia: a randomized double-blind placebo-controlled crossover clinical trial. Sleep. 2014;37:483-487. A2. Alshaikh MK, Tricco AC, Tashkandi M, et al. Sodium oxybate for narcolepsy with cataplexy: systematic review and meta-analysis. J Clin Sleep Med. 2012;8:451-458. A3. Gottlieb DJ, Punjabi NM, Mehra R, et al. CPAP versus oxygen in obstructive sleep apnea. N Engl J Med. 2014;370:2276-2285. A4. Mason M, Welsh EJ, Smith I. Drug therapy for obstructive sleep apnoea in adults. Cochrane Database Syst Rev. 2013;5:CD003002. A5. Ferracioli-Oda E, Qawasmi A, Bloch MH. Meta-analysis: melatonin for the treatment of primary sleep disorders. PLoS ONE. 2013;8:e63773. A6. Liira J, Verbeek JH, Costa G, et al. Pharmacological interventions for sleepiness and sleep disturbances caused by shift work. Cochrane Database Syst Rev. 2014;8:CD009776. A7. Allen RP, Chen C, Garcia-Borreguero D, et al. Comparison of pregabalin with pramipexole for restless legs syndrome. N Engl J Med. 2014;370:621-631. A8. Trenkwalder C, Benes H, Grote L, et al. Prolonged release oxycodone-naloxone for treatment of severe restless legs syndrome after failure of previous treatment: a double-blind, randomised, placebo-controlled trial with an open-label extension. Lancet Neurol. 2013;12:1141-1150.

GENERAL REFERENCES For the General References and other additional features, please visit Expert Consult at https://expertconsult.inkling.com.

406  APPROACH TO CEREBROVASCULAR DISEASES LARRY B. GOLDSTEIN

EPIDEMIOLOGY

Nearly 800,000 Americans have a stroke each year, and about 75% are first strokes. Measured in terms of disease-attributed healthy years of life lost, cerebrovascular disease ranks second in the United States and third worldwide.1 Stroke, which is a generic term for cerebrovascular disease, has fallen from the third to the fourth leading cause of death in the United States (behind heart disease, cancer, and lung and respiratory diseases), primarily because of a dramatic reduction in stroke-related mortality. From 2000 to 2010, the annual stroke death rate in the United States fell by about 36%, with the actual number of stroke-related deaths falling by about 23%.2 Stroke is the underlying cause of death of about 130,000 Americans each year, corresponding to approximately 1 in 19 deaths in the country. About 60% of stroke deaths occur in women, but the rates are actually highest in African American men. It is estimated that someone in the United States has a stroke about once every 40 seconds. The overall prevalence of stroke is estimated at 2.8%, with 6.8 million American adults having had a stroke.2 Even though the incidence of stroke has been declining substantially, largely because of better prevention, the declining case-fatality rate has kept the population prevalence reasonably stable. The risk of stroke generally increases with age, and it doubles for every decade after the age of 55 years. In addition, blacks, people with lower levels of education, individuals who reside in the southeastern portion of the country (the “Stroke Belt”), and individuals with a first-degree relative who had a stroke before the age of 65 years have a higher risk of stroke and of stroke-related mortality. Poor diet, lack of exercise (Chapter 16), cigarette smoking (Chapter 32), exposure to environmental tobacco smoke, obesity (Chapter 220), and excess alcohol consumption (Chapter 33) are lifestyle factors that greatly increase the risk of stroke. Of the medical conditions that increase the risk of stroke, hypertension (Chapter 67) has the highest population-attributable risk. Other stroke risk factors include atrial fibrillation (Chapter 64), diabetes (Chapter 229), dyslipidemia (Chapter 206), inflammatory states, elevated homocysteine levels, high lipoprotein (a), carotid artery stenosis, patent foramen ovale (Chapter 69), other congenital heart defects, and sleep apnea (Chapter 100).3 Coagulation disorders (Chapter 176), oral contraceptive agents (Chapter 238), and migraine headache with aura (Chapter 398) also may contribute to the risk. Mendelian diseases associated with stroke include sickle cell disease (Chapter 163); mitochondrial encephalopathy, lactic acidosis, and stroke (MELAS); cerebral autosomal dominant arteriopathy with subcortical infarcts and leukoencephalopathy (CADASIL; Chapter 402); Fabry disease (Chapters 208 and 275); and Marfan syndrome (Chapter 260). In addition, autosomal dominant polycystic kidney disease (Chapter 127) is associated with intracranial aneurysms and fibromuscular dysplasia. Ehlers-Danlos type IV (Chapter 260) is also associated with intracranial aneurysms as well as cervical arterial dissection. Several genetic polymorphisms also have been associated with stroke (e.g., variants on chromosome 9p21 and 4q25), although these genetic markers are not yet clinically relevant.

PATHOBIOLOGY

Anatomy

An understanding of vascular anatomy and its normal variants as well as their relationships to functional neuroanatomy can provide important clues for identifying the cause of cerebrovascular symptoms and signs in individual patients and can also help guide treatment.

Aortic Arch

DEFINITION

The term cerebrovascular disease refers to a group of conditions in which injury to the brain or spinal cord occurs from a vascular cause. The onset is generally abrupt, but it also can be insidious. Clinical manifestations depend on the location and extent of damage to neural structures. Although risk factors and treatments may overlap, cerebrovascular diseases are pathophysiologically divided into those in which an insufficiency in the blood supply causes ischemic injury and those in which bleeding, either into the parenchyma (intracerebral or much more rarely intraspinal hemorrhage) or into the space between the pial and arachnoid coverings over the brain or spinal cord (subarachnoid hemorrhage), causes direct neural injury, leads to secondary ischemic injury, or acts as a space-occupying lesion. Cerebrovascular disease is often both preventable and treatable.

Paired carotid and vertebral arteries normally supply the brain (Fig. 406-1). The right common carotid artery arises from the brachiocephalic trunk (innominate artery), which then gives rise to the right subclavian artery. The right vertebral artery generally arises from the proximal portion of the right subclavian artery. The left common carotid artery usually arises directly from the aortic arch; but in some individuals, it may arise from the proximal portion of the brachiocephalic trunk (“bovine” anatomy). The left subclavian artery originates from the aortic arch distal to the left common carotid artery and also supplies the left vertebral artery.

Internal Carotid Arteries

The common carotid arteries bifurcate into the internal carotid artery and external carotid artery in the neck, generally at the level of the thyroid cartilage. The bifurcation may less commonly occur above the lower level of the

CHAPTER 405  Disorders of Sleep  

GENERAL REFERENCES 1. Yang G, Lai CS, Cichon J, et al. Sleep promotes branch-specific formation of dendritic spines after learning. Science. 2014;344:1173-1178. 2. Striz M, O’Hara BF. Clock genes and sleep homeostasis: a fundamental link within the two-process model? Sleep. 2013;36:301-302. 3. Qaseem A, Dallas P, Owens DK, et al. Diagnosis of obstructive sleep apnea in adults: a clinical practice guideline from the American College of Physicians. Ann Intern Med. 2014;161:210-220. 4. Mignot EJ. A practical guide to the therapy of narcolepsy and hypersomnia syndromes. Neurother. 2012;9:739-752. 5. Tafti M, Hor H, Dauvilliers Y, et al. DQB1 locus alone explains most of the risk and protection in narcolepsy with cataplexy in Europe. Sleep. 2014;37:19-25. 6. De la Herran-Arita AK, Garcia-Garcia F. Narcolepsy as an immune-mediated disease. Sleep Disord. 2014;2014:792687. 7. Thorpy MJ, Dauvilliers Y. Clinical and practical considerations in the pharmacologic management of narcolepsy. Sleep Med. 2015;16:9-18.

2424.e1

8. Buysse DJ. Insomnia. JAMA. 2013;309:706-716. 9. Li Y, Zhang X, Winkelman JW, et al. Association between insomnia symptoms and mortality: a prospective study of U.S. men. Circulation. 2014;129:737-746. 10. Ebisawa T. Analysis of the molecular pathophysiology of sleep disorders relevant to a disturbed biological clock. Mol Genet Genomics. 2013;288:185-193. 11. Nesbitt AD, Dijk DJ. Out of synch with society: an update on delayed sleep phase disorder. Curr Opin Pulm Med. 2014;20:581-587. 12. Campos Costa I, Nogueira Carvalho H, Fernandes L. Aging, circadian rhythms and depressive disorders: a review. Am J Neurodegener Dis. 2013;2:228-246. 13. St Louis EK. Key sleep neurologic disorders: narcolepsy, restless legs syndrome/Willis-Ekbom disease, and REM sleep behavior disorder. Neurol Clin Pract. 2014;4:16-25. 14. Leschziner G, Gringras P. Restless legs syndrome. BMJ. 2012;344:e3056. 15. Garcia-Borreguero D, Kohnen R, Silber MH, et al. The long-term treatment of restless legs syndrome/Willis-Ekbom disease: evidence-based guidelines and clinical consensus best practice guidance: a report from the International Restless Legs Syndrome Study Group. Sleep Med. 2013;14: 675-684.

2424.e2

CHAPTER 405  Disorders of Sleep  

REVIEW QUESTIONS 1. Which of the following symptoms are clues to an underlying sleep disorder? A. Brief lapses in memory B. Excessive daytime sleepiness C. Insomnia D. Morning headache E. All of the above Answer: E  Sleep disruption can cause all of these symptoms, and all are common among individuals with disturbed nighttime sleep. An individual might be attuned to specific aspects of their lives and perceive the disruption of sleep in different ways. Thus, the clinician can use these symptoms as an opportunity to further question aspects of sleep. 2. Hypersomnia can be caused by which of the following? A. Sleep deprivation B. Sleep apnea C. Medication D. Liver failure E. All of the above Answer: E  The symptom of hypersomnia, or excessive ability to fall asleep, may be created by a wide range of issues. The clinician can think of sleepiness in general categories of sleep deprivation, sleep disruption (external [toxin] or internal [brain issue]) disturbing the mechanism for wake or sleep, a disturbance of circadian timing, or misperception of fatigue. 3. The diagnosis of insomnia requires which of the following? A. Actigraphy B. History of difficulty of sleeping at night C. History of difficulty with sleep at night and daytime sequelae D. History of somnogenic substance use and daytime sequelae E. Polysomnography Answer: C  Insomnia is a clinical diagnosis best made by a detailed history that demonstrates difficulty with sleep and a resulting symptom of daytime impairment. No other laboratory data are needed to make the diagnosis, but other information may be helpful in determining the best therapy in selected cases. Most patients with insomnia do not need polysomnography, although actigraphy can be helpful in determining the patient’s bedtime and wake schedule.

4. The diagnosis of a circadian rhythm disorder is best made by: A. Sleep diary B. Polysomnography C. Multiple sleep latency test D. Maintenance of wakefulness test E. None of the above Answer: A  The diagnosis of a circadian rhythm disorder is dependent upon the complaint of insomnia or excessive sleepiness that is persistent for more than 3 months and can be confirmed by a sleep log or actigraphy for at least 14 days. Unfortunately, polysomnography and the daytime studies do not help in making this diagnosis. 5. Rapid eye movement (REM) sleep behavior disorder, characterized by excessive muscle activity in REM sleep and dream enactment, is most likely a harbinger of: A. Alzheimer disease B. Conversion disorder C. Heart failure D. Parkinson disease E. Renal failure Answer: D  REM sleep behavior disorder has been linked to degenerative disorders known as synucleinopathies (Lewy body dementia, Parkinson disease, and multiple system atrophy). The symptoms of dream enactment may present more than 30 years prior to other symptoms of these degenerative disorders, and up to 80% of individuals with REM sleep behavior disorder develop one of the three. REM sleep behavior disorder also has been associated with other disorders, such as narcolepsy, and with brain stem strokes, lesions, and tumors. However, it is not associated with the other disorders listed as answers.

CHAPTER 406  Approach to Cerebrovascular Diseases  

2425

Carotid siphon Basilar artery Right external carotid artery Right internal carotid artery Left external carotid artery

Left internal carotid artery

Right common carotid artery

Left common carotid artery Right vertebral artery Left vertebral artery

Right subclavian artery Brachiocephalic artery

Left subclavian artery

Descending aorta

Ascending aorta

FIGURE 406-1.  Magnetic resonance angiogram of normally configured aortic arch.

mandible or lower in the neck. The internal carotid artery enters the skull through the foramen lacerum and travels through the petrous bone adjacent to the inner ear. It then enters the cavernous sinus, ascends in an S shape (carotid siphon), penetrates the dura, and finally divides into the anterior cerebral artery and middle cerebral artery (Fig. 406-2). The ophthalmic artery can originate from the internal carotid artery in the carotid siphon, but it more commonly arises from the supraclinoid internal carotid artery, followed by the posterior communicating and anterior choroidal arteries.

External Carotid Arteries

In contrast to the internal carotid arteries, the external carotid arteries have extracranial branches. The superficial temporal arteries (palpable anterior to the ears) and facial arteries can anastomose with the intracranial circulation through branches of the ophthalmic artery and can be clinically important in the setting of a proximal internal carotid artery occlusion.

Vertebral Arteries

Although the vertebral arteries generally arise from the subclavian arteries, they can also originate from the aortic arch or thyrocervical trunk. They most commonly enter the C6 transverse process but may also enter at the C4, C5, or C7 levels. They exit the transverse processes at C1, turn posteriorly behind the atlantoaxial joint, and then pass through the dura at the foramen magnum.

Intracranially, they typically join at the pontomedullary junction to form the single basilar artery, although the vertebral artery can end in the posterior inferior cerebellar artery in some individuals (Fig. 406-3). The portion of the vertebral artery between its origin and its entry into the transverse process is referred to as the V1 segment. The V2 segment refers to the portion of the artery traveling through the transverse foramina; the V3 segment, the portion between where the artery exits the transverse foramina and penetrates the dura; and the V4 segment, the intracranial portion of the artery. One vertebral artery may be hypoplastic (E-Fig. 406-1). Clues are that the ipsilateral transverse foramina are generally smaller on the side of the hypoplastic artery and that the proximal portion of the basilar artery can be displaced ipsilateral to the hypoplastic artery. The V3 segment is particularly vulnerable to mechanical injury that can lead to dissection. The vertebral arteries have medial branches that unite to form the anterior spinal artery and lateral branches that supply the dorsolateral medulla and inferior portion of the cerebellum, which also supplies the vestibular nuclei (Fig. 406-4). Other medial branches of the vertebral artery supply the medullary pyramid, inferior olivary nucleus, medial lemniscus, and hypoglossal nerve fibers. Longer circumferential branches from the vertebral arteries and posterior cerebral arteries supply the spinothalamic tracts and sympathetic fibers as they traverse the medulla, the sensory nuclei, and the descending tracts from cranial nerve V as well as emerging fibers from the vagus and glossopharyngeal nerves.

CHAPTER 406  Approach to Cerebrovascular Diseases  

2425.e1

Anterior communicating artery (ACom) Anterior cerebral artery (A1 segment) Internal carotid artery

Posterior communicating artery (PCom)

Posterior cerebral artery (P1 segment) Basilar artery Vertebral artery

E-FIGURE 406-1.  Magnetic resonance angiogram of the circle of Willis. The approximate positions of the posterior communicating arteries are shown with dotted lines.

2426

CHAPTER 406  Approach to Cerebrovascular Diseases  

Anterior cerebral artery (ACA) Middle cerebral artery-bifurcation

Middle cerebral artery-M2 segment Anterior communicating artery (ACom)

Middle cerebral artery-M1 segment

Carotid siphon Internal carotid artery (ICA)

FIGURE 406-2.  Magnetic resonance angiogram of the intracranial portion of the internal carotid artery and its main branches.

Basilar Artery

The basilar artery has small penetrating branches supplying the dorsal portions of the pons and midbrain (see Figs. 406-3 and 406-4). The anterior inferior cerebellar arteries originate from the mid–basilar artery. They supply portions of the cerebellar hemispheres in addition to the lateral pons; cranial nerves V, VII, and VIII; and pontine portions of the spinothalamic tracts and sympathetic fibers. The two superior cerebellar arteries arise from the distal basilar artery at the level of the midbrain proximal to the common origin of the two posterior cerebral arteries. The oculomotor nerve exits the midbrain between the superior cerebellar artery and posterior cerebral artery. The superior cerebellar arteries give branches supplying the dorsal midbrain, including the colliculi and the superior portions of the cerebellar hemispheres and vermis. The long circumferential vessels also supply the dorsolateral brain stem. In addition to the anterior inferior cerebellar artery and superior cerebellar artery, the basilar artery has paramedian vessels supplying the middle portion of the basis pontis and midline pontine structures, including the corticospinal tracts, medial longitudinal fasciculus, and pontine reticular nuclei. At the midbrain level, paramedian branches of the basilar artery supply the cerebral peduncles, cranial nerve III nuclei and fibers, and medial portions of the red nucleus and medial lemniscus. Short circumferential branches supply the ventrolateral pons and midbrain.

Circle of Willis

The anastomosis at the base of the brain is termed the circle of Willis (see E-Fig. 406-1). The two anterior cerebral arteries are connected by the anterior communicating artery. The posterior communicating arteries connect the supraclinoid internal carotid arteries with the proximal posterior cerebral arteries. In persons with an intact circle of Willis, the entire intracranial cir-

culation can be supplied by a single patent internal carotid artery or vertebral artery. The majority of individuals, however, have an incomplete circle of Willis (see Fig. 406-2). One common variant is for the portion of the anterior cerebral artery between the internal carotid artery and the anterior communicating artery (A1 segment) to be hypoplastic or absent. In this case, both anterior cerebral arteries can be supplied from a single internal carotid artery. Another common variant is for the portion of the posterior cerebral artery between its normal origin from the basilar artery and the posterior communicating artery (P1 segment) to be absent or hypoplastic (termed a “fetal” posterior cerebral artery). In these individuals, the distal posterior cerebral artery territory is supplied by the carotid rather than by the vertebrobasilar arteries.

Anterior Cerebral Arteries

The anterior cerebral arteries travel anteriorly and then turn posteriorly with leptomeningeal branches supplying the medial portions of the frontal and parietal lobes (Figs. 406-5 to 406-7; see also Fig. 406-2). In about half of people, the anterior cerebral artery divides into pericallosal and callosal marginal branches. Terminal portions of the latter artery supply the medial cortex between the parietal and occipital lobes. Damage to this area can be confused with “watershed” hypoperfusion injury. A series of small lenticulostriate arteries originate from the A1 and A2 (between the anterior communicating artery and corpus callosum) segments of the anterior cerebral artery. The recurrent artery of Heubner is a large, important medial striate artery that provides blood supply to the anterior and inferior portions of the anterior limb of the internal capsule, anterior and inferior portions of the caudate nucleus, anterior globus pallidus, putamen, hypothalamus, olfactory bulbs and tracts, and uncinate fasciculus. It can be inadvertently damaged during surgical clipping of an anterior communicating artery aneurysm.

CHAPTER 406  Approach to Cerebrovascular Diseases  

2427

Posterior cerebral artery (PCA) Superior cerebellar artery (SCA)

Basilar artery

Anterior inferior cerebellar artery (AICA)

Posterior inferior cerebellar artery (PICA) Left vertebral artery

Right vertebral artery

FIGURE 406-3.  Magnetic resonance angiogram of the intracranial portion of the vertebrobasilar system.

Anterior Choroidal Artery

The anterior choroidal artery (medial striate artery) commonly arises from the supraclinoid internal carotid artery distal to the posterior communicating artery. It travels posteriorly over the medial optic tract and enters the brain at the choroidal fissure. It gives branches to the optic tract, anterior hippocampus, amygdala, tail of the caudate nucleus, geniculate body, and inferior portion of the posterior limb of the internal capsule (see Fig. 406-7). Ischemic lesions in this area can be confused with lesions arising from the middle cerebral artery.

Middle Cerebral Artery

The middle cerebral artery supplies the bulk of the frontal, parietal, and lateral portions of the temporal lobes (Figs. 406-8 and 406-9; see also Figs. 406-6 and 406-7). The M1 segment refers to the portion of the middle cerebral artery between its origin from the supraclinoid internal carotid artery and its distal branches (see Fig. 406-2). The middle cerebral artery bifurcates in the sylvian fissure in 20 to 30% of individuals and trifurcates in about 70% of individuals. The superior division supplies the frontal and parietal lobes, and the inferior division supplies the lateral portion of the temporal lobe. The M1 segment gives rise to some medial and all of the lateral lenticulostriate arteries. These arteries supply the head and body of the caudate nucleus, the putamen, and the globus pallidus as well as the anterior limb,

genu, and superior portions of the posterior limb of the internal capsule (see Fig. 406-7).

Posterior Cerebral Artery

The distal portion of the posterior cerebral artery divides into an anterior and a posterior division (see Fig. 406-3). The anterior division supplies the inferior and medial portions of the temporal lobe into the middle cranial fossa with distal branches anastomosing with those of the middle cerebral artery (see E-Fig. 406-1). The posterior division supplies the occipital lobe, including the calcarine cortex, with terminal branches anastomosing with those of the middle cerebral artery and anterior cerebral artery. The proximal portions of both the posterior cerebral artery and the posterior communicating artery give off small penetrating arteries to the thalamus (thalamoperforators). In some individuals, a single common artery arising from the P1 segment (artery of Percheron) can supply both thalami. Unless the posterior cerebral artery has a fetal-type origin from the internal carotid artery, thalamic strokes are generally related to the vertebrobasilar circulation. Two posterior choroidal arteries arise separately from the posterior cerebral artery and supply the choroid plexus, posterior thalamus, fornix, and midbrain tectum. Posterior cerebral artery perforators also supply the medial portions of the cerebral peduncles, substantia nigra, red nuclei, hippocampus, and posterior hypothalamus.

2428

CHAPTER 406  Approach to Cerebrovascular Diseases  

Vestibular nuclei

Fasciculus solitarius

Medial longitudinal Reticular fasciculus nuclei

Nucleus gracilis and cuneatus Descending sympathetic tract

X

V (chief sensory)

IV ventricle

V (motor) IV ventricle

XII Spinothalamic tract

Restiform body Spinal tract and nucleus V Spinothalmic tract Nucleus ambiguus (motor IX, X)

Medial longitudinal fasciculus

Motor and sensory V nerve

Pontine nucleus

Pontocerebellar fibers Corticospinal tract

Inferior olive Medial lemniscus Posterior inferior cerebellar artery

Pyramidal tract

Areas of medulla supplied by: Posterior inferior cerebellar artery Lateral branches of vertebral artery

Lateral branches of basilar artery Basilar artery

Vertebral artery

A

Medial lemniscus

Vertebral artery

B

Areas of pons supplied by: Lateral branches of basilar artery

Paramedian branches of basilar artery

Paramedian branches of basilar artery

FIGURE 406-4.  Brain stem blood supply. A, Cross section of the medulla oblongata at the level of the hypoglossal nuclei (cranial nerve XII). Short branches of the vertebral and anterior spinal arteries supply the medulla. Longer circumferential branches, including the posterior inferior cerebellar artery, supply the lateral portions of the medulla. B, Cross section of the midpons region. The medial portion receives blood supply from short, perforating basilar artery branches. More laterally, the blood supply comes from lateral basilar artery branches. (From Zivin JA. Approach to cerebrovascular diseases. In: Goldman L, Schafer AI. Goldman’s Cecil Medicine. 24th ed. Philadelphia: Elsevier Saunders; 2012.)

Frontal lobe

Parietal lobe

Corpus callosum

Midbrain Occipital lobe Pons

Medulla

FIGURE 406-5.  Parasagittal T2-weighted magnetic resonance image showing midline structures.

Cerebellum

CHAPTER 406  Approach to Cerebrovascular Diseases  

Broca area

2429

Motor Sensory Trunk Arm Hand

D

Face Tongue

C

HC P A TC

B

Central speech Auditory area

A

Wernicke area

A

B FLV

Suppl. motor area Motor area Sensory

HC AIC P GP PIC T OR PLV

C Visual cortex

B Anterior cerebral artery

Middle cerebral artery

Posterior cerebral artery

FIGURE 406-6.  Surface cerebral arterial anatomy. Lateral (A) and medial (B) views of the cerebral hemisphere show the surface distributions of the anterior, middle, and posterior cerebral arteries. (From Zivin JA. Approach to cerebrovascular diseases. In: Goldman L, Schafer AI. Goldman’s Cecil Medicine. 24th ed. Philadelphia: Elsevier Saunders; 2012.)

D Anterior cerebral artery

Thalamogeniculate arteries

Recurrent artery of Heubner

Anterior choroidal artery

Lenticulostriate arteries

Posterior cerebral artery

Middle cerebral artery

FIGURE 406-7.  Arterial supply of the deep brain structures. A, Sagittal view of the brain showing the computed tomographic (CT) planes through which views B, C, and D were taken. B, CT plane through the head of the caudate nucleus (HC), putamen (P), amygdala (A), tail of the caudate nucleus (TC), hypothalamus, temporal lobe, midbrain, and cerebellum. C, CT plane through the frontal horn of the lateral ventricle (FLV), head of the caudate nucleus (HC), anterior and posterior limbs of the internal capsule (AIC, PIC), putamen (P), globus pallidus (GP), thalamus (T), optic radiations (OR), and posterior horn of the lateral ventricle (PLV). D, CT plane through the centrum semiovale. (Modified from De Armond S, Fusco MM, Dewey MM. Structure of the Human Brain, a Photographic Atlas. 3rd ed. New York: Oxford University Press; 1989, with permission.)

Venous System

The venous drainage of the brain is divided into superficial and deep systems (Figs. 406-10 and 406-11). Deep structures drain into the inferior sagittal sinus and vein of Galen that join to form the straight sinus, which runs along the tentorium to join the superior sagittal sinus at the torculum. The cerebral veins drain into the sagittal sinus. The two transverse sinuses extend laterally from the torculum into the sigmoid sinus, which then forms the jugular vein. Oftentimes, one hypoplastic transverse sinus can cause confusion if a sinus thrombosis is suspected. In these cases, the jugular notch in the occipital bone and jugular foramen may be smaller on the side of the hypoplastic transverse sinus. Each cavernous sinus surrounds the ipsilateral internal carotid artery. Fibers from cranial nerve VI run within the cavernous sinus inferior to the carotid artery, with fibers from cranial nerves III, IV, V1, and V2 running in its lateral wall. The two cavernous sinuses connect to each other and drain into the petrosal sinus and then the sagittal sinus.

Physiology Cerebral Blood Flow

The brain, which is among the body’s most metabolically active tissues, receives about 14% of resting cardiac output. Normal resting metabolism of brain tissue requires 140 µmol of oxygen and 24 µmol of glucose per 100 g of tissue per minute. Although total blood flow to the brain remains constant in normal conditions, regional flow changes with mental activity, often manifested by changes in synaptic activity, and provides the basis for functional

magnetic resonance imaging or positron emission tomography imaging studies. Approximately 80% of glucose is used to generate energy, with the remainder metabolized to lactate or used for synthetic activities. Little glucose is stored in the brain, and the brain’s high metabolic demand makes it particularly vulnerable to reductions in oxygen and blood supply. Cerebral blood flow at rest averages 50 to 100 mL per 100 g of brain tissue per minute. If blood flow falls below this level, normal neuronal function is suppressed (i.e., neurons become electrically quiescent). If the deficit persists, irrevocable neural injury can result. Cerebral blood flow is regulated though a variety of mechanisms in addition to mental activity. Constant, overall cerebral blood flow is maintained through autoregulation. This autoregulatory relationship is reflected in the equation cerebral blood flow = cerebrovascular resistance/mean arterial pressure. If the mean arterial pressure is decreased, there is a compensatory decrease in cerebrovascular resistance (through dilation of cerebral arterioles) to maintain cerebral blood flow constant. If the mean arterial pressure is increased, there is a compensatory increase in cerebrovascular resistance (through constriction of cerebral arterioles). There are, however, limits to cerebral autoregulation. At mean arterial pressures greater than about 150 mm Hg, cerebral arterioles are maximally constricted, and cerebral blood flow rises. At mean arterial pressures below about 50 mm Hg, cerebral arterioles are maximally dilated, and cerebral blood flow falls. In the setting of

2430

CHAPTER 406  Approach to Cerebrovascular Diseases  

Parietal lobe

Frontal lobe

Leg Motor Broca area

Arm

Sensory

Face

Wernicke area Visual cortex

Occipital lobe

Temporal lobe

FIGURE 406-8.  Sagittal, lateral T1-weighted magnetic resonance image showing cortical motor, sensory, visual, and language areas.

Leg Arm Hand Face

FIGURE 406-9.  Coronal T1-weighted magnetic resonance image showing cortical areas for the leg, arm, hand, and face.

CHAPTER 406  Approach to Cerebrovascular Diseases  

Superior sagittal sinus

2431

Cortical veins

Straight sinus

Internal cerebral veins Basal vein of Rosenthal

Vein of Galen

Torculum Transverse sinus

Inferior petrosal sinus Internal jugular vein

Sigmoid sinus

FIGURE 406-10.  Parasagittal magnetic resonance venogram showing venous structures.

chronic hypertension, the autoregulatory relationship between cerebrovascular resistance and mean arterial pressure is shifted to higher critical mean arterial pressures (i.e., cerebral blood flow falls at a higher mean arterial pressure). Metabolic factors can also affect cerebral blood flow. Hypercapnia causes cerebral vasodilation, and hypocapnia causes cerebral vasoconstriction that is mediated by changes in the pH of the brain’s extracellular fluid. Cerebral blood flow declines by approximately 2% for every 1 mm Hg decline in Pco2. In patients who have increased intracranial pressure and threatened herniation, a short period of hyperventilation (target arterial Pco2 of 30 to 35 mm Hg) can be used as a temporary measure until more definitive treatment can be instituted. The response is only transient because of compensation by the choroid plexus, and a rebound increase in Paco2 can lead to a rise in intracranial pressure when hyperventilation is discontinued.

Blood-Brain Barrier

The triggering of a neuronal action potential depends on the relative concentrations of Na+, K+, and Ca2+, and it is also modulated by Mg2+ and a variety of neurotransmitters. The blood-brain barrier is critical for maintaining the environment necessary for normal neuronal function.4 The blood-brain barrier consists anatomically of the capillary endothelial cells, a basement membrane with pericytes, and astrocytic perivascular footplates. The brain’s vascular endothelial cells, which are the principal component of the bloodbrain barrier, are joined by tight junctions and generally lack the transport channels found elsewhere in the body. As a result, the blood-brain barrier prevents hydrophilic polar and large molecules in the blood from entering

the brain. By comparison, oxygen and carbon dioxide rapidly cross the bloodbrain barrier. Nutrients, toxins, and drugs can cross the blood-brain barrier by simple diffusion, by transport through carrier molecules based on concentration gradients (facilitated transport), or by energy-dependent mechanisms (active transport). Glucose is the brain’s sole source of energy. Glucose transport into the brain is through non–energy-dependent facilitated transport (glucose transporter isotype 1, Glut1). In the setting of ischemia, endothelial cell function can be compromised, and the blood-brain barrier can fail.

The Neurovascular Unit

The concept of the neurovascular unit has become important for understanding the complex relationships between anatomic structures and the integrity of brain function. The term reflects the physiologic interrelatedness of the brain’s various components, including endothelial cells, vascular smooth muscle, adventitial cells, glia, and neurons. The concept reflects the observation that local pH as well as neural activity can affect local cerebral blood flow. In addition to linking neural activity with blood flow and maintaining the blood-brain barrier, the neurovascular unit can secrete a variety of immunologic and neurotrophic factors that further affect both normal function and the brain’s response to injury.

  CEREBRAL ISCHEMIA

Because of its high metabolic demands, brain function is completely dependent on its supply of blood and oxygen. Clinical symptoms ensue when global or regional blood supply falls below the critical 50 mL per 100 g per minute. Permanent neural injury does not occur if the supply of blood and oxygen is

2432

CHAPTER 406  Approach to Cerebrovascular Diseases  

Cerebral veins Sagittal sinus

Torculum Sigmoid sinus

Transverse sinus Internal jugular vein

FIGURE 406-11.  Anteroposterior magnetic resonance venogram showing venous structures.

quickly restored, such as with a faint (Chapter 62) in the setting of a global reduction in the supply of blood or oxygen or a transient ischemic attack (Chapter 407) with brief, local reductions in cerebral blood flow. Certain groups of neurons may be particularly vulnerable to hypoxic-ischemic injury (i.e., regions of the hippocampus, cerebellar Purkinje cells, and neocortical layers III, V, and possibly VI). Hypoxic-ischemic injury can be global, diffuse, or focal.

Out-of-hospital cardiac arrest carries a poor prognosis if effective CPR is not rapidly instituted. A period of therapeutic hypothermia may improve neurologic outcome after resuscitated cardiac arrest if it can be instituted rapidly (Chapter 63). A1  If the cerebral cortex is irreversibly damaged but the relatively resistant brain stem control of respiration and cardiovascular regulation is preserved, the patient can enter a persistent vegetative state (Chapter 404).

Global Ischemic Injury

Diffuse Hypoxic Injury

Global ischemic injury occurs in the setting of complete cardiovascular collapse, such as with ventricular fibrillation, electromechanical dissociation, and asystole (Chapter 63). Some neurons are particularly vulnerable to ischemic injury and will be selectively damaged, whereas neurons only millimeters away may be spared.5 In the setting of hypotension, areas of brain between the territories of major arteries (i.e., between the anterior cerebral artery and middle cerebral artery in the frontal cortex and adjacent subcortical white matter), between the middle cerebral artery and posterior cerebral artery (in the parieto-occipital cortex and adjacent subcortical white matter), and between penetrating arteries from distal branches of the middle cerebral artery and lenticulostriate arteries (deep hemispheric white matter, centrum semiovale) are especially vulnerable and are termed watershed areas. The duration of anoxia, the duration of cardiopulmonary resuscitation (CPR), and the cause of cardiac arrest are related to poor outcome after CPR (Chapters 63 and 404), but none of these factors accurately discriminate between poor and favorable outcomes. Prognosis also cannot be based on the circumstances of CPR or on elevated body temperature alone. Myoclonus or status epilepticus within the first day after cardiac arrest implies a poor prognosis, as does the absence of pupillary or corneal reflexes or extensor motor responses 3 days after cardiac arrest in patients who remain comatose. Bilateral absence of cortical somatosensory evoked responses within 1 to 3 days also portends a poor prognosis.

Diffuse hypoxia can alter cognition, cause confusion, impair consciousness, and lead to coma, which can be irreversible. Causes include travel to high altitudes, severe anemia, and pulmonary disease. Symptoms are generally present when the Pao2 abruptly falls to less than 40 mm Hg. Increases in cerebral blood flow can partially compensate for slow declines in Pao2, which may still cause symptoms with further or rapid reductions.

Focal Ischemic Injury Focal ischemic injury is caused by occlusion of a cervical or intracranial artery that supplies the brain. Although this injury can occur from many causes (including infection, inflammation, metabolic disorders, trauma, and hematologic disorders), the majority of strokes are related to thrombotic or embolic occlusion (Fig. 406-12). If flow is not restored within minutes, a core area of irreversible brain injury is commonly produced. A surrounding area of variable size, depending on the artery involved and the integrity of collaterals in which blood flow is reduced, will suffer injury that is not irreversible. The brain in this area, termed the penumbra, is electrically quiescent and contributes to the resulting neurologic deficit. Because the pH of the extracellular fluid in the penumbral zone is low, vessels are maximally dilated and the cerebral autoregulatory response is inoperative. Because cerebrovascular resistance in the penumbral zone is fixed, any decline in mean arterial pressure can further reduce its cerebral blood flow, thereby extending the volume

CHAPTER 406  Approach to Cerebrovascular Diseases  

Atherothrombotic/ embolic

44

Cardioembolic

21

Small vessel disease

19

Hypertensive Amyloid angiopathy Other

Ischemic 84%

5 3

Intracerebral 10%

2

Aneurysmal

3

AVM-related

1

Other

2

Hemorrhagic 16% Subarachnoid 6%

FIGURE 406-12.  Classification of cerebrovascular disease by cause. AVM = arteriovenous malformation. (From Zivin JA. Approach to cerebrovascular diseases. In: Goldman L, Schafer AI. Goldman’s Cecil Medicine. 24th ed. Philadelphia: Elsevier Saunders; 2012.)

of infarcted brain tissue. A variety of neuroimaging techniques can help distinguish penumbral from infarcted brain tissue (i.e., magnetic resonance diffusion-perfusion mismatch, computed tomographic perfusion imaging) but have not been standardized and have not yet proved useful for clinical decisions regarding the use of reperfusion therapy. Many putative neuroprotective strategies aimed at preserving ischemic brain tissue until it can be reperfused through collateral flow have failed in clinical trials.

PATHOLOGY

Permanent occlusion of a cerebral artery results in necrosis of its supplied neurons, glia, and endothelial cells (pan-necrosis). In gross appearance, the area of infarcted brain may be pale or hemorrhagic if secondary bleeding occurred. Over time, the lesion becomes cavitary (encephalomalacia). On microscopic examination, ischemic neurons initially appear small and angular. The cytoplasm becomes homogeneously eosinophilic, and the nucleus becomes dark and pyknotic. As endothelial cells die, associated areas of petechial hemorrhage may appear. An initial inflammatory reaction may lead to microvascular occlusions, such that flow to ischemic tissue may not be restored even if a proximal thrombus is removed (no-reflow phenomenon). Leukocytes that infiltrate ischemic tissue can also release interleukins and cytokines, which can contribute to cytotoxic injury. Blood macrophages begin to reach the infarcted tissue, and neovascularization peaks after about 2 weeks. Macrophage-mediated removal of cellular debris peaks at about 3 to 4 weeks after the infarct. Astrocytes then form a glial scar around the area of infarction.

PATHOPHYSIOLOGY

Because the brain has no reserve energy supply, energy-dependent neuronal and glial processes stop soon after acute deprivation of blood and oxygen. Calcium ions enter depolarized neurons and glia, where they activate second messengers, including lipases and proteases, thereby releasing free fatty acids and generating free radicals that degrade cellular organelles and membranes. Depolarized neurons also release high levels of excitatory neurotransmitters, such as glutamate into synapses, which leads to further neuronal depolarization and calcium entry. Once this cascade has been initiated, neurons may still degenerate over time by apoptosis (programmed cell death) even if blood flow is restored. Although promising in the laboratory, all attempts to block the ischemic cascade pharmacologically have failed in clinical trials to date.

  CEREBRAL HEMORRHAGE

Subarachnoid hemorrhage, which is bleeding between the pial and arachnoid coverings over the brain, is most commonly related to a ruptured aneurysm

2433

(Chapter 408).6 Cerebral aneurysms may occur spontaneously or be acquired as a result of infection or trauma. They are more common in first-degree relatives of patients who have a cerebral aneurysm and with certain conditions, such as autosomal dominant polycystic kidney disease (Chapter 127) and type IV Ehlers-Danlos syndrome (Chapter 260). Noninfectious aneurysms are typically situated at branch points of major cerebral arteries: anterior cerebral artery–anterior communicating artery, internal carotid artery– posterior communicating artery, middle cerebral artery bifurcation, basilar artery tip. Initial brain injury can be caused by an acute increase in intracranial pressure, with delayed ischemic injury related to the development of vasospasm after 7 to 10 days. Interference with the absorption of cerebrospinal fluid though the arachnoid granulations can lead to communicating hydrocephalus. Clot within the third or fourth ventricle or cerebral aqueducts can cause obstructive hydrocephalus. The most common causes of intracerebral parenchymal brain hemorrhages are hypertension (Chapter 67) and cerebral amyloid angiopathy. Myriad other potential vascular and nonvascular causes, including vascular malformations, vasculitis (Chapter 270), venous sinus thrombosis, and coagulopathies (Chapters 172, 173, and 174), are less common. Some tumors (e.g., melanoma [Chapter 203] and renal cell carcinoma [Chapter 197]) can be manifested as an intracerebral hemorrhage. Hypertension-related intracerebral hemorrhage occurs in typical areas of the brain (i.e., basal ganglia, thalamus, basis pontis, and cerebellum). In contrast, intracerebral hemorrhage related to cerebral amyloid angiopathy is typically lobar and located closer to the cortical surface. Without sequential neuroimaging studies showing an initial area of ischemic injury, lobar hemorrhages may be difficult to distinguish from a hemorrhagic infarction. Susceptibility-weighted brain magnetic resonance imaging sequences may reveal prior microhemorrhages at the graywhite junction in patients with cerebral amyloid angiopathy.

  CEREBRAL EDEMA

When neurons and glia are injured by ischemia, energy metabolism fails and the cells can no longer maintain normal ion gradients between the intracellular and extracellular compartments. The result is cytotoxic edema, in which cells swell soon after the injury. Neurons, glia, and endothelial cells can be affected. Vasogenic edema, which may occur as a result of disruption of the blood-brain barrier due to injury to the endothelium, allows large molecules to pass through the blood-brain barrier and to gain access to the brain. Edema generally peaks between 48 and 72 hours after the onset of ischemic injury. In patients with ischemic stroke, the development of cytotoxic edema can lead to an increase in intracranial pressure and, when severe, herniation. In selected patients, craniotomy can be considered to relieve the pressure until the edema subsides. Neurons, glia, and endothelial cells are also damaged in the setting of intracerebral hemorrhage. The hemorrhage itself is a space-occupying lesion that can also be associated with both cytotoxic and vasogenic edema. Mass effect from cerebellar hemorrhages can compress the fourth ventricle (thereby leading to obstructive hydrocephalus), compress the brain stem (thereby compromising the reticular activating system and impairing consciousness), or cause herniation. Surgical evacuation of intracerebral hemorrhage is not of proven value (Chapter 408), but emergent evacuation of cerebellar hemorrhages can be life-saving and leave surviving patients with little or no long-term functional impairment.

Grade A Reference A1. Arrich J, Holzer M, Havel C, et al. Hypothermia for neuroprotection in adults after cardiopulmonary resuscitation. Cochrane Database Syst Rev. 2012;9:CD004128.

GENERAL REFERENCES For the General References and other additional features, please visit Expert Consult at https://expertconsult.inkling.com.

CHAPTER 406  Approach to Cerebrovascular Diseases  

GENERAL REFERENCES 1. Feigin VL, Forouzanfar MH, Krishnamurthi R, et al. Global and regional burden of stroke during 1990-2010: findings from the Global Burden of Disease Study 2010. Lancet. 2014;383:245-254. 2. Go AS, Mozaffarian D, Roger VL, et al. Heart disease and stroke statistics—2014 update: a report from the American Heart Association. Circulation. 2014;129:e28-e292. 3. Goldstein LB, Bushnell CD, Adams RJ, et al. Guidelines for the primary prevention of stroke: a guideline for healthcare professionals from the American Heart Association/American Stroke Association. Stroke. 2011;42:517-584.

2433.e1

4. Engelhardt B, Liebner S. Novel insights into the development and maintenance of the blood-brain barrier. Cell Tissue Res. 2014;355:687-699. 5. Baron JC, Yamauchi H, Fujioka M, et al. Selective neuronal loss in ischemic stroke and cerebrovascular disease. J Cereb Blood Flow Metab. 2014;34:2-18. 6. Chalouhi N, Hoh BL, Hasan D. Review of cerebral aneurysm formation, growth, and rupture. Stroke. 2013;44:3613-3622.

2433.e2

CHAPTER 406  Approach to Cerebrovascular Diseases  

REVIEW QUESTIONS 1. Which of the following conditions is associated with the highest population-attributable risk of stroke? A. Cigarette smoking B. Diabetes C. Hypertension D. Poor diet E. Sedentary lifestyle Answer: C  Each of the listed conditions and lifestyle factors is associated with an increase in the risk of stroke. Of these, hypertension has the highest population-attributable risk for stroke. 2. The left common carotid artery most commonly arises from which of the following structures? A. Right subclavian artery B. Innominate artery C. Aortic arch D. Left subclavian artery E. Thyrocervical trunk Answer: C  The left common carotid artery most commonly arises directly from the aortic arch. In some individuals, it may arise from the proximal portion of the brachiocephalic trunk (“bovine” anatomy).

3. Which of the following arteries is commonly the first intradural branch of the internal carotid artery? A. Posterior communicating artery B. Ophthalmic artery C. Anterior choroidal artery D. Anterior cerebral artery E. Recurrent cerebral artery (Heubner) Answer: B  The ophthalmic artery may rarely arise from the carotid siphon, but it is generally the first branch after the internal carotid artery pierces the dura, followed by the posterior communicating artery and the anterior choroidal artery. The recurrent cerebral artery is generally a branch of the anterior cerebral artery. 4. Which of the following equations describes the normal cerebral autoregulatory relationship (CBF, cerebral blood flow; CVR, cerebral vascular resistance; MAP, mean arterial pressure)? A. CVR = MAP − CBF B. MAP = CBF − CVR C. CBF = MAP/CVR D. MAP = CVR − CBF E. CBF = CVR/MAP Answer: E  If MAP is decreased, there is a compensatory decrease in CVR (through dilation of cerebral arterioles) to maintain CBF constant. If MAP is increased, there is a compensatory increase in CVR (through constriction of cerebral arterioles). In the setting of ischemia, cerebral arterioles are maximally dilated; decreased MAP leads to a reduction in CBF to ischemic brain.

2434

CHAPTER 407  Ischemic Cerebrovascular Disease  

407  ISCHEMIC CEREBROVASCULAR DISEASE LARRY B. GOLDSTEIN

DEFINITION

Ischemic cerebrovascular disease is caused by an impairment of blood supply to the brain. The injury may be focal (related to occlusion of a single artery), multifocal (related to occlusion of several arteries), or diffuse. Although certain clinical features (e.g., severe hypertension, headache, impaired consciousness) may suggest brain hemorrhage (Chapter 408) rather than ischemia, it is not possible to differentiate the two sets of conditions without a brain imaging study. In the absence of an inflammatory disease such as vasculitis or other rare conditions, simultaneous involvement of more than one vascular distribution suggests a proximal source of embolism (i.e., a cardiogenic or a proximal arterial source). Involvement of a single vascular territory may be due to either local steno-occlusive disease (e.g., atherosclerosis) or a proximal source of embolism. Involvement in the distribution of a single penetrating artery suggests small-vessel type intracranial disease, but ischemic strokes in this distribution may also be caused by proximal arterial steno-occlusive disease or embolism. The definition of ischemic stroke is brain, spinal cord, or retinal cell death attributable to ischemia with neuropathologic, neuroimaging, or clinical evidence of permanent injury.1 Overall, approximately 85% of strokes are related to ischemic disease, with 44% attributable to atherosclerosis (Chapter 70), 21% to cardiogenic embolism, and 20% to small-vessel disease. Transient ischemic attack (TIA) is defined as a brief episode of neurologic dysfunction resulting from focal cerebral ischemia with no evidence of corresponding tissue injury. Symptoms are similar to those of ischemic stroke. Previously characterized as a transient deficit with symptoms persisting for less than 24 hours, evidence of corresponding tissue injury can be seen on brain magnetic resonance imaging (MRI) in 30 to 40% of patients who otherwise fulfill the clinical definition of TIA.

EPIDEMIOLOGY

Stroke (ischemic and hemorrhagic) is the second leading cause of death worldwide and the fourth leading cause of death in the United States.2 It is also one of the leading causes of adult disability. In addition to age, race or ethnicity, and family history, a variety of lifestyle factors and medical conditions increase the risk of stroke (Chapter 406; Table 407-1). Of these, hypertension is the single most important (Chapter 67; Table 407-1), and the risk of stroke increases with increasing blood pressure with no threshold effect. Diabetes (Chapter 229) is associated with an approximate doubling of the risk of stroke (Table 407-1). Atrial fibrillation (Chapter 64; Table 407-1) is

TABLE 407-1  COMMON STROKE RISK FACTORS POPULATIONATTRIBUTABLE RISK

RISK REDUCTION WITH TREATMENT

Cigarette smoking

12-14%

50% within 1 year of quitting

Physical inactivity

30%

?

7%

?

FACTOR LIFESTYLE

Excess alcohol consumption MEDICAL Hypertension

>90%

32%

Diabetes

5-27%



Atrial fibrillation

2-24%

64%

Carotid stenosis

2-7%

50%

Sickle cell disease



91% with transfusion therapy in children

Data from Goldstein LB, Bushnell CD, Adams RJ, et al. Guidelines for the primary prevention of stroke: a guideline for healthcare professionals from the American Heart Association/American Stroke Association. Stroke. 2011;42:517-584.

associated with up to 25% of ischemic strokes, with the absolute risk varying by concomitant risk factors. Extracranial carotid artery stenosis is found in up to 5 to 10% of individuals older than 65 years and is associated with about 10% of all ischemic strokes. Untreated asymptomatic carotid stenosis carries only about a 1 to 2% annual risk of stroke, and the risk may now be much lower, perhaps as low as 0.5% annually, with standard medical therapy. Stroke is also a complication of sickle cell disease (Chapter 163), with risk dramatically reduced with transfusion therapy in high-risk children. Unlike with coronary heart disease, the overall association between high cholesterol concentration and the risk of stroke is less certain. Ischemic stroke risk is associated with higher levels of total cholesterol, whereas the risk of hemorrhagic stroke is increased with lower cholesterol levels. Other factors associated with the risk of stroke include migraine headaches with aura (Chapter 398), particularly in women who smoke and are receiving oral contraceptives; elevated homocysteine level; high lipoprotein (a) level; postmenopausal hormone replacement therapy (Chapter 240); coagulation disorders (Chapter 176); systemic infection (Chapter 76); renal impairment (Chapter 130); low vitamin D levels (Chapters 218 and 244); and a variety of environmental factors, including high levels of air pollution.

PATHOBIOLOGY

For patients who have a TIA and who are by definition at increased risk of having an ischemic stroke during the next few days or weeks or who have an ischemic stroke, distinguishing among the major pathophysiologic causes (i.e., atherothrombotic, cardioembolic, small vessel) is critical to guide secondary prevention. Atherothrombosis due to atherosclerosis (Chapter 70) is the most common cause of a TIA or stroke that is related to steno-occlusive disease in a single artery.3 The ischemia may be caused when progressive stenosis at the site of an atherosclerotic plaque leads to hemodynamic compromise affecting distal brain tissue. Sometimes bleeding into the plaque can lead to abrupt arterial occlusion, and sometimes a thrombus that has formed on an ulcerated plaque may embolize and occlude a distal artery. Occlusion of a cerebral artery, however, does not necessarily lead to ischemic brain injury. Blood may still reach the supplied territory through collaterals, either through the circle of Willis or from extracranial-intracranial anastomoses (see E-Fig. 406-1). Arterial dissection, previously thought to occur only rarely and usually to result in major stroke, is now recognized more frequently on the basis of noninvasive vascular imaging such as MR angiography or computed tomography (CT) angiography. Other arteriopathies, such as fibromuscular dysplasia (Chapters 67, 80, and 125), may also lead to single, large-vessel distribution, ischemic stroke. Atherosclerosis of the ascending aorta or aortic arch can lead to the formation of thrombus, which can then embolize to a cerebral artery. Atrial fibrillation is the single most common cause of cardioembolic stroke, with annual risks of 3 to 5% if it is not treated with anticoagulation but declining to about one fourth of that risk with anticoagulation (Chapter 64). The use of extended cardiac rhythm monitoring (i.e., 30-day event-triggered loop monitoring; Chapter 62) reveals occult atrial fibrillation in up to 25% of patients with an otherwise cryptogenic stroke. Other cardiac causes of cerebral embolism include clots or vegetations in patients with valvular heart disease (Chapter 75), such as mechanical prosthetic heart valves (Chapter 75), infectious endocarditis (Chapter 76), and nonbacterial endocarditis (Chapter 76); and mural thrombi in patients with a cardiomyopathy (Chapter 60) or myocardial infarction (MI), particularly anteroseptal MI (Chapter 73). Paradoxical embolism of a venous clot across a congenital heart defect, such as a patent foramen ovale or an atrial septal defect (Chapter 69), is another potential cause of embolic stroke.4 Small-vessel intracranial disease may result in ischemic stroke in the distribution of a single penetrating vessel. These strokes commonly affect deep structures (e.g., centrum semiovale, basal ganglia, thalamus, internal capsule, pons) and occur more frequently in patients with hypertension and diabetes. Classically, small-vessel strokes are caused by lipohyalinosis, which is a thickening of the vessel wall resulting in a diminished luminal area, but they also can be caused by atherothrombotic embolism. Symptoms of ischemic stroke may worsen during the first hours or days through various mechanisms. For example, decreases in systemic blood pressure may decrease cerebral blood flow to marginally perfused, ischemic brain. In the setting of atherothrombotic disease, a partially occluded artery may progress to complete occlusion. Recurrent embolism may occur from a proximal arterial or cardiac source. Cerebral edema may develop during the first few days after an ischemic stroke, and the resulting mass effect can lead to

CHAPTER 407  Ischemic Cerebrovascular Disease  

TABLE 407-2  CLINICAL MANIFESTATIONS OF ISCHEMIC CEREBROVASCULAR DISEASE OCCLUDED ARTERY

TYPICAL MAJOR CLINICAL MANIFESTATIONS*

Internal carotid artery

Ipsilateral visual loss Ipsilateral middle cerebral artery syndrome

Anterior choroidal artery

Contralateral hemiparesis Contralateral sensory impairment Contralateral visual field defect

Anterior cerebral artery

Contralateral leg > arm paresis Contralateral leg > arm sensory deficit

Middle cerebral artery

Contralateral hemiparesis affecting face and arm > leg Contralateral sensory deficit affecting face and arm > leg Contralateral visual field defect Aphasia (dominant hemisphere) Contralateral hemispatial neglect (nondominant or dominant hemisphere)

Posterior cerebral artery

Contralateral homonymous hemianopia (or homonymous superior or inferior quadrantanopia) Contralateral sensory deficits (thalamic involvement)

Basilar artery tip

Bilateral central visual loss Confusion

Basilar artery

Ipsilateral cranial nerve deficit Contralateral hemiparesis Contralateral sensory impairment affecting arm and/or leg Coordination deficit

Vertebral artery, posterior inferior cerebellar artery

Ipsilateral sensory impairment over the face Dysphagia Ipsilateral Horner syndrome Ataxia

Superior cerebellar artery

Gait ataxia Ipsilateral limb ataxia Variable contralateral limb weakness

*Note: not all may be present.

clinical deterioration (Chapter 406). Secondary bleeding can occur in an area that was primarily the site of an ischemic injury when reperfusion, either through collateral vessels or as the result of a therapeutic intervention, restores blood flow into vessels in which the endothelium was damaged by the original ischemic insult.

CLINICAL MANIFESTATIONS

Neurologic deficits that occur in the setting of ischemic stroke depend on the involved vascular territory (Table 407-2) and underlying cause. Embolic stroke is generally characterized by the presence of a maximal deficit at onset, whereas the onset may be more gradual or stuttering in the setting of an atherothrombotic stroke. The distinction, however, is not of great use for diagnosis in individual patients. Transient symptoms in the same distribution can be caused by TIA if there is no permanent tissue injury.

Internal Carotid Artery

The bifurcation of the common carotid artery into the internal and external carotid arteries in the neck is a common site of atherosclerotic disease (see Fig. 406-1). With occlusion of the internal carotid artery, patients who have an incomplete circle of Willis can suffer profound contralateral loss of motor and sensory function affecting the face, arm, and leg. In patients with an intact anterior communicating artery that can supply the ipsilateral anterior cerebral artery (see E-Fig. 406-1), the leg may be relatively spared, and an internal carotid artery occlusion may be clinically indistinguishable from a middle cerebral artery occlusion. If the anterior communicating artery is absent on the side opposite an internal carotid artery occlusion, the ipsilateral leg may also be affected, and the presentation may be confused with a cardioembolic cause because both hemispheres are involved. Occlusion of the ipsilateral ophthalmic artery can lead to blindness in that eye. Transient symptoms of retinal ischemia, classically described by patients as a “shade coming down over my vision,” indicate amaurosis fugax. Other common symptoms include a darkening or blurring of vision in the affected eye. Transient hypoperfusion ipsilateral to a high-grade internal carotid artery stenosis can cause limbshaking TIAs that can be confused with seizures. Systemic hypotension in the setting of a high-grade carotid stenosis can lead to ischemic injury in

2435

watershed zones between the major intracranial arteries and in the border zone between the distal territories of cortical and lenticulostriate penetrating vessels.

Anterior Choroidal Artery

The anterior choroidal artery generally arises from the supraclinoid portion of the internal carotid artery (see Fig. 406-7). Causes of occlusion of the anterior choroidal artery are similar to those of occlusion of the small intracranial arteries. Symptoms can include contralateral motor and sensory deficits and contralateral visual field deficits, the latter of which can occur in isolation.

Cerebral Arteries

About 2% of strokes are related to isolated occlusion of the anterior cerebral artery (see Figs. 406-6 and 406-7). Occlusion of the A1 segment in patients in whom the contralateral A1 segment is hypoplastic or absent can lead to bilateral leg involvement, abulia, and urinary incontinence because of infarction of both frontal lobes. The middle cerebral artery is the most common artery involved in occlusions related to cardiogenic embolism. It supplies the lateral portions of the frontal, parietal, and temporal lobes as well as the basal ganglia and the anterior limb and genu of the internal capsule. Middle cerebral artery occlusions are characterized by involvement of the contralateral face and arm to a greater extent than of the leg (see Figs. 406-6, 406-8, and 406-9), often accompanied by a contralateral hemispatial neglect. When the dominant hemisphere is involved, the patient may have an aphasia. With frontal lobe involvement, patients often have an ipsilateral, conjugate deviation of the eyes, which can be forced past the midline with vigorous encouragement, oculocephalic maneuvers, or caloric stimulation. Branch middle cerebral artery occlusions can result in partial syndromes. For example, a branch middle cerebral artery occlusion with intact collaterals can cause a global aphasia without an accompanying motor deficit. Anterior branch, dominant hemisphere middle cerebral artery occlusions can cause an expressive, cortical-type motor (Broca) aphasia with sparing of comprehension. Occlusion of the angular branch of the middle cerebral artery can cause receptive, cortical-type (Wernicke) aphasia. Borderzone infarcts can result in transcortical aphasias, characterized by relatively preserved repetitions. Both posterior cerebral arteries arise from the basilar artery in about 75% of people (see E-Fig. 406-1). In the other 25%, one or both P1 segments are hypoplastic or absent, with the posterior cerebral arteries arising from the ipsilateral internal carotid artery (so-called fetal circulation). Without vascular imaging, it is not possible to determine if a posterior cerebral artery distribution infarct (see Figs. 406-6 to 406-8) is related to carotid or vertebrobasilar circulation disease. The posterior cerebral artery and posterior communicating arteries supply the thalamus. Thalamic infarctions can result in contralateral hemianesthesia and ataxia. Contralateral hemiballismus can result if the subthalamic nucleus is damaged. Infarction of the ipsilateral occipital lobe causes a contralateral homonymous hemianopia that can be partial, depending on the extent of injury. The visual field deficit tends to become more congruous in the two eyes as the area of injury becomes more posterior (i.e., the closer to the occipital pole).

Vertebral and Basilar Arteries

Occlusion of the basilar artery (see Figs. 406-3 and 406-4B) can lead to “locked-in syndrome” (Chapter 404) in which the patient is awake and alert, because the periaqueductal gray can receive a separate blood supply, but unable to move or to communicate except for vertical eye movements, because of sparing of the collicular nuclei in the midbrain. The tip of the basilar artery is a common location for embolic occlusion. Symptoms can include visual field defects due to unilateral or bilateral occipital injury and confusional states due to thalamic involvement. Occlusions of penetrating and circumferential branches of the basilar artery and vertebral artery can produce a variety of symptoms (see Table 407-2), depending on the portion of the artery involved, several of which constitute eponymic midbrain (E-Fig. 407-1), pontine (E-Fig. 407-2), or medullary (E-Fig. 407-3) syndromes. Occlusion of the superior cerebellar artery can cause truncal ataxia because of infarction of the cerebellar vermis, with or without ataxia of the ipsilateral limbs, which can be caused by infarction of the ipsilateral cerebellar hemisphere.

Small Vessels

Occlusion of a small penetrating intracranial vessel can result in one of the classic lacunar syndromes (Table 407-3). These syndromes are not otherwise

CHAPTER 407  Ischemic Cerebrovascular Disease  

Claude Syndrome Paramedian infarction • Ipsilateral 3rd nerve • Contralateral ataxia

Benedikt Syndrome Medial infarction • Ipsilateral 3rd nerve • Contralateral ataxia • Contralateral hemiparesis

Weber Syndrome Basal infarction • Ipsilateral 3rd nerve • Contralateral hemiparesis

2435.e1

Raymond Syndrome Mediolateral infarction • Ipsilateral internuclear ophthalmoplegia • Contralateral hemiparesis • Contralateral UMN 7th nerve • Contralateral light touch, proprioception • +/– contralateral pain/temperature (with ipsilateral Horner)

Foville Syndrome

Mediolateral infarction • Ipsilateral LMN 7th nerve • Ipsilateral conjugate gaze palsy • Contralateral hemiparesis

Millard-Gubler Syndrome Basal infarction • Ipsilateral LMN 7th nerve • Ipsilateral 6th nerve • Contralateral hemiparesis • +/– contralateral pain/temperature

E-FIGURE 407-1.  Midbrain stroke syndromes.

Locked-in Syndrome

Bilateral basis pontis infarction • Bilateral LMN 7th nerve • Bilateral 6th nerve • Quadriplegia • +/– bilateral light touch/proprioception

E-FIGURE 407-2.  Pontomedullary and pontine stroke syndromes. LMN = lower motor neuron; UMN = upper motor neuron.

2435.e2

CHAPTER 407  Ischemic Cerebrovascular Disease  

Dejerine Syndrome

Medial infarction • Ipsilateral XII nerve • Contralateral hemiparesis • Contralateral light touch, proprioception

Jackson Syndrome Lateral median infarction • Ipsilateral IX, X nerve • Ipsilateral XII nerve • Variable contralateral hemiparesis

Avellis Syndrome Lateral infarction • Ipsilateral IX, X nerve • Contralateral pain, temperature (ipsilateral Horner) • +/– contralateral hemiparesis

Wallenberg Syndrome Dorsolateral infarction • Ipsilateral facial anesthesia • Contralateral pain, temperature • Ipsilateral Horner • Ipsilateral IX, X • Ipsilateral ataxia (olivocerebellar tract)

E-FIGURE 407-3.  Medullary stroke syndromes.

2436

CHAPTER 407  Ischemic Cerebrovascular Disease  

localizing and can occur with occlusions of small penetrating vessels in either the anterior or vertebrobasilar circulations. Lacunar syndromes are not pathognomonic of small-vessel intracranial disease and can be caused by a variety of other conditions, including emboli from a more proximal arterial or cardioembolic source or brain hemorrhage (Chapter 408).

likelihood of a brain hemorrhage (Chapter 408). In a patient with transient vertigo associated with left arm movement, a reduced blood pressure in that arm suggests subclavian steal syndrome. Detection of an anterior cervical bruit contralateral to symptoms and signs indicative of a middle cerebral artery distribution infarct increase the likelihood of symptomatic carotid stenosis. An irregularly irregular heart rhythm with or without a cardiac murmur may indicate atrial fibrillation and a cardioembolic etiology. Finding a cholesterol embolus on funduscopic examination can be consistent with a proximal source of atheroembolism. Funduscopy can also show evidence of a small-vessel disease related to diabetes or hypertension (see Figs. 423-24 and 423-26). A general neurologic examination (Chapter 396) including evaluations of cognition, language, spatial neglect, cranial nerves, motor function, sensation, coordination, gait, and reflexes is important both for documenting strokerelated deficits and for providing information critical for determining the area of the brain affected by the stroke and the severity of the injury. The use of a standardized graded neurologic impairment assessment provides a tool for measuring the severity of the stroke, determining the risks and benefits of treatment interventions, assessing prognosis, and observing patients objectively over time. The National Institutes of Health Stroke Scale (Table 407-4), which is the most commonly used approach, is both reliable and well validated. The individual items are summed to provide a total score.

DIAGNOSIS

The diagnosis of ischemic stroke depends on acquiring an accurate history, eliciting key findings on general and neurologic examinations, and obtaining supporting data from selected laboratory studies (Fig. 407-1). An initial anatomic and pathophysiologic differential diagnosis is usually established on the basis of the patient’s history. Findings on physical and neurologic examinations can support or refute initial conclusions based on the history and can further refine the differential diagnosis.

History

The abrupt onset of a focal neurologic deficit in the distribution of a specific vascular territory is the hallmark of acute ischemic stroke. The differential and most likely diagnosis can often be determined on the basis of history alone. For example, a patient with a history of atrial fibrillation who abruptly develops word-finding difficulties associated with a right hemiparesis and sensory impairment most likely had a cardiogenic embolus to the left middle cerebral artery. A patient with the acute onset of diplopia, vertigo, and a hemiparesis most likely has a lesion in the brain stem. Goals of the immediate history include determining the exact time when symptoms began or the last time the patient was known to be well, concomitant medical illnesses, risk factors, medications, allergies, and other potential causes for symptoms that might mimic acute ischemic stroke. Because a stroke may affect a patient’s ability to communicate, the history may require input from a witness. Additional details of the patient’s past medical, family, and social history may need to be deferred in the emergent setting, but these issues can be explored if the information is important for acute treatment decisions.

Initial Laboratory Tests

Laboratory testing can help exclude conditions that may mimic, complicate, or lead to an acute ischemic stroke (Table 407-5).5 Tests that should be obtained in all patients with suspected ischemic stroke include a complete blood count and platelet count, prothrombin time/international normalized ratio (INR), activated partial thromboplastin time, blood glucose level, serum electrolytes, tests of renal function, troponin level, and oxygen saturation. An electrocardiogram should be obtained urgently, and the patient should be sent for a CT brain scan or MRI as soon as he or she is stable enough. Additional tests are indicated in selected patients. For example, women of childbearing age should have a pregnancy test. A toxicology screen and blood alcohol levels should be obtained if drug or alcohol abuse is suspected. In patients who may be receiving a direct thrombin inhibitor or a factor Xa inhibitor, a thrombin time or ecarin clot time may be helpful in determining whether the patient is anticoagulated. An elevated erythrocyte sedimentation rate may point to an inflammatory cause or systemic infection. The complete blood count can provide information about both the potential cause of the stroke and possible therapeutic interventions. An elevated white blood cell count may indicate an infectious cause of stroke, such as infective endocarditis (Chapter 76). Systemic infection may also cause a recrudescence of prior stroke symptoms in a patient who had previously

Physical Examination

Severely elevated blood pressures in the setting of neurologic deficits referable to the basal ganglia, thalamus, pons, or cerebellum increase the

TABLE 407-3  LACUNAR SYNDROMES Pure motor stroke Pure sensory stroke Ataxic hemiparesis Clumsy hand–dysarthria

Acute neurologic deficit

Hemorrhagic Stroke

Ischemic Stroke No

No

Yes

Yes

Is the patient a candidate for IV-tPA? No

Yes

IV-tPA Protocol

Is the patient a candidate for endovascular Rx? Initiate Secondary Prevention Prevent Complications Recovery

No

Yes

Endovascular Protocol

General Measures FIGURE 407-1.  Approach to ischemic stroke. IV-tPA = intravenous tissue plasminogen activator; Rx = therapy. (From Goldstein LB. Modern medical management of acute ischemic stroke. Methodist DeBakey Cardiovasc J. 2014;10:99-104.)

CHAPTER 407  Ischemic Cerebrovascular Disease  

2437

TABLE 407-4  NATIONAL INSTITUTES OF HEALTH STROKE SCALE Administer stroke scale items in the order listed. Record performance in each category after each subscale exam. Do not go back and change scores. Follow directions provided for each exam technique. Scores should reflect what the patient does, not what the clinician thinks the patient can do. The clinician should record answers while administering the exam and work quickly. Except where indicated, the patient should not be coached (i.e., repeated requests to patient to make a special effort). INSTRUCTIONS

SCALE DEFINITION

SCORE

1a. Level of Consciousness: The investigator must choose a response if a full evaluation is prevented by such obstacles as an endotracheal tube, language barrier, orotracheal trauma/bandages. A 3 is scored only if the patient makes no movement (other than reflexive posturing) in response to noxious stimulation.

0 = Alert; keenly responsive. 1 = Not alert; but arousable by minor stimulation to obey, answer, or respond. 2 = Not alert; requires repeated stimulation to attend, or is obtunded and requires strong or painful stimulation to make movements (not stereotyped). 3 = Responds only with reflex motor or autonomic effects or totally unresponsive, flaccid, and areflexic.

______

1b. LOC Questions: The patient is asked the month and his/her age. The answer must be correct—there is no partial credit for being close. Aphasic and stuporous patients who do not comprehend the questions will score 2. Patients unable to speak because of endotracheal intubation, orotracheal trauma, severe dysarthria from any cause, language barrier, or any other problem not secondary to aphasia are given a 1. It is important that only the initial answer be graded and that the examiner not “help” the patient with verbal or nonverbal cues.

0 = Answers both questions correctly. 1 = Answers one question correctly. 2 = Answers neither question correctly.

______

1c. LOC Commands: The patient is asked to open and close the eyes and then to grip and release the nonparetic hand. Substitute another one-step command if the hands cannot be used. Credit is given if an unequivocal attempt is made but not completed due to weakness. If the patient does not respond to command, the task should be demonstrated to him or her (pantomime), and the result scored (i.e., follows none, one, or two commands). Patients with trauma, amputation, or other physical impediments should be given suitable one-step commands. Only the first attempt is scored.

0 = Performs both tasks correctly. 1 = Performs one task correctly. 2 = Performs neither task correctly.

______

2. Best Gaze: Only horizontal eye movements will be tested. Voluntary or reflexive (oculocephalic) eye movements will be scored, but caloric testing is not done. If the patient has a conjugate deviation of the eyes that can be overcome by voluntary or reflexive activity, the score will be 1. If a patient has an isolated peripheral nerve paresis (CN III, IV, or VI), score a 1. Gaze is testable in all aphasic patients. Patients with ocular trauma, bandages, preexisting blindness, or other disorder of visual acuity or fields should be tested with reflexive movements, and a choice made by the investigator. Establishing eye contact and then moving about the patient from side to side will occasionally clarify the presence of a partial gaze palsy.

0 = Normal. 1 = Partial gaze palsy; gaze is abnormal in one or both eyes, but forced deviation or total gaze paresis is not present. 2 = Forced deviation, or total gaze paresis not overcome by the oculocephalic maneuver.

______

3. Visual: Visual fields (upper and lower quadrants) are tested by confrontation, using finger counting or visual threat, as appropriate. Patients may be encouraged, but if they look at the side of the moving fingers appropriately, this can be scored as normal. If there is unilateral blindness or enucleation, visual fields in the remaining eye are scored. Score 1 only if a clear-cut asymmetry, including quadrantanopia, is found. If patient is blind from any cause, score 3. Double simultaneous stimulation is performed at this point. If there is extinction, patient receives a 1, and the results are used to respond to item 11.

0 = No visual loss. 1 = Partial hemianopia. 2 = Complete hemianopia. 3 = Bilateral hemianopia (blind including cortical blindness).

______

4. Facial Palsy: Ask—or use pantomime to encourage—the patient to show teeth or raise eyebrows and close eyes. Score symmetry of grimace in response to noxious stimuli in the poorly responsive or noncomprehending patient. If facial trauma/bandages, orotracheal tube, tape, or other physical barriers obscure the face, these should be removed to the extent possible.

0 = Normal symmetrical movements. 1 = Minor paralysis (flattened nasolabial fold, asymmetry on smiling). 2 = Partial paralysis (total or near-total paralysis of lower face). 3 = Complete paralysis of one or both sides (absence of facial movement in the upper and lower face).

______

5. Motor Arm: The limb is placed in the appropriate position: extend the arms (palms down) 90 degrees (if sitting) or 45 degrees (if supine). Drift is scored if the arm falls before 10 seconds. The aphasic patient is encouraged using urgency in the voice and pantomime, but not noxious stimulation. Each limb is tested in turn, beginning with the nonparetic arm. Only in the case of amputation or joint fusion at the shoulder, the examiner should record the score as untestable (UN) and clearly write the explanation for this choice.

0 = No drift; limb holds 90 (or 45) degrees for full 10 seconds. 1 = Drift; limb holds 90 (or 45) degrees, but drifts down before full 10 seconds; does not hit bed or other support. 2 = Some effort against gravity; limb cannot get to or maintain (if cued) 90 (or 45) degrees, drifts down to bed, but has some effort against gravity. 3 = No effort against gravity; limb falls. 4 = No movement. UN = Amputation or joint fusion, explain: _____________________ 5a. Left Arm 5b. Right Arm

______

6. Motor Leg: The limb is placed in the appropriate position: hold the leg at 30 degrees (always tested supine). Drift is scored if the leg falls before 5 seconds. The aphasic patient is encouraged using urgency in the voice and pantomime, but not noxious stimulation. Each limb is tested in turn, beginning with the nonparetic leg. Only in the case of amputation or joint fusion at the hip, the examiner should record the score as untestable (UN) and clearly write the explanation for this choice.

0 = No drift; leg holds 30-degree position for full 5 seconds. 1 = Drift; leg falls by the end of the 5-second period but does not hit bed. 2 = Some effort against gravity; leg falls to bed by 5 seconds, but has some effort against gravity. 3 = No effort against gravity; leg falls to bed immediately. 4 = No movement. UN = Amputation or joint fusion, explain: ________________ 6a. Left Leg 6b. Right Leg

______

2438

CHAPTER 407  Ischemic Cerebrovascular Disease  

TABLE 407-4  NATIONAL INSTITUTES OF HEALTH STROKE SCALE—cont’d INSTRUCTIONS

SCALE DEFINITION

SCORE

7. Limb Ataxia: This item is aimed at finding evidence of a unilateral cerebellar lesion. Test with eyes open. In case of visual defect, ensure testing is done in intact visual field. The finger-nose-finger and heel-shin tests are performed on both sides, and ataxia is scored only if present out of proportion to weakness. Ataxia is absent in the patient who cannot understand or is paralyzed. Only in the case of amputation or joint fusion, the examiner should record the score as untestable (UN) and clearly write the explanation for this choice. In case of blindness, test by having the patient touch nose from extended arm position.

0 = Absent. 1 = Present in one limb. 2 = Present in two limbs. UN = Amputation or joint fusion, explain: ________________

______

8. Sensory: Sensation or grimace to pinprick when tested, or withdrawal from noxious stimulus in the obtunded or aphasic patient. Only sensory loss attributed to stroke is scored as abnormal, and the examiner should test as many body areas (arms [not hands], legs, trunk, face) as needed to accurately check for hemisensory loss. A score of 2, “severe or total sensory loss,” should only be given when a severe or total loss of sensation can be clearly demonstrated. Stuporous and aphasic patients will, therefore, probably score 1 or 0. The patient with brain stem stroke who has bilateral loss of sensation is scored 2. If the patient does not respond and is quadriplegic, score 2. Patients in a coma (item 1a = 3) are automatically given a 2 on this item.

0 = Normal; no sensory loss. 1 = Mild-to-moderate sensory loss; patient feels pinprick is less sharp or is dull on the affected side; or there is a loss of superficial pain with pinprick, but patient is aware of being touched. 2 = Severe to total sensory loss; patient is not aware of being touched in the face, arm, and leg.

______

9. Best Language: A great deal of information about comprehension will be obtained during the preceding sections of the examination. For this scale item, the patient is asked to describe what is happening in the attached picture, to name the items on the attached naming sheet, and to read from the attached list of sentences. Comprehension is judged from responses here, as well as to all of the commands in the preceding general neurological exam. If visual loss interferes with the tests, ask the patient to identify objects placed in the hand, repeat, and produce speech. The intubated patient should be asked to write. The patient in a coma (item 1a = 3) will automatically score 3 on this item. The examiner must choose a score for the patient with stupor or limited cooperation, but a score of 3 should be used only if the patient is mute and follows no one-step commands.

0 = No aphasia; normal. 1 = Mild-to-moderate aphasia; some obvious loss of fluency or facility of comprehension, without significant limitation on ideas expressed or form of expression. Reduction of speech and/or comprehension, however, makes conversation about provided materials difficult or impossible. For example, in conversation about provided materials, examiner can identify picture or naming card content from patient’s response. 2 = Severe aphasia; all communication is through fragmentary expression; great need for inference, questioning, and guessing by the listener. Range of information that can be exchanged is limited; listener carries burden of communication. Examiner cannot identify materials provided from patient response. 3 = Mute, global aphasia; no usable speech or auditory comprehension.

______

10. Dysarthria: If patient is thought to be normal, an adequate sample of speech must be obtained by asking patient to read or repeat words from the attached list. If the patient has severe aphasia, the clarity of articulation of spontaneous speech can be rated. Only if the patient is intubated or has other physical barriers to producing speech, the examiner should record the score as untestable (UN) and clearly write an explanation for this choice. Do not tell the patient why he or she is being tested.

0 = Normal. 1 = Mild-to-moderate dysarthria; patient slurs at least some words and, at worst, can be understood with some difficulty. 2 = Severe dysarthria; patient’s speech is so slurred as to be unintelligible in the absence of or out of proportion to any dysphasia, or is mute/ anarthric. UN = Intubated or other physical barrier, explain:________________ _____________

______

11. Extinction and Inattention (formerly Neglect): Sufficient information to identify neglect may be obtained during the prior testing. If the patient has a severe visual loss preventing visual double simultaneous stimulation, and the cutaneous stimuli are normal, the score is normal. If the patient has aphasia but does appear to attend to both sides, the score is normal. The presence of visual spatial neglect or anosognosia may also be taken as evidence of abnormality. Since the abnormality is scored only if present, the item is never untestable.

0 = No abnormality. 1 = Visual, tactile, auditory, spatial, or personal inattention or extinction to bilateral simultaneous stimulation in one of the sensory modalities. 2 = Profound hemi-inattention or extinction to more than one modality; does not recognize own hand or orients to only one side of space.

______

From http://www.ninds.nih.gov/doctors/nih_stroke_scale.pdf. Accessed February 26, 2015.

recovered or in whom a stroke had not been previously recognized. Polycythemia (Chapter 166) can cause hyperviscosity that leads to occlusion of small intracranial vessels. Thrombocytopenia, either primary or secondary, can lead to platelet thrombi. The prothrombin time/INR and activated thromboplastin time provide indices that may reveal an underlying coagulation disorder, and thrombocytopenia and coagulation disorders may preclude treatment with intravenous recombinant tissue plasminogen activator (rtPA). Both hypoglycemia (Chapter 230) and hyperglycemia (Chapter 229) may cause strokelike symptoms. Impaired renal function (Chapter 130) is a risk factor for ischemic stroke and may increase the risks of using thrombolytic and anticoagulant medications. Abnormalities of other serum electrolytes (e.g., hyponatremia; Chapter 116) can also cause neurologic symptoms. The electrocardiogram may reveal changes suggestive of acute myocardial ischemia as well as atrial fibrillation, the most common cause of embolic stroke. Stroke may also cause a variety of cardiac arrhythmias. Acute MI, especially anteroseptal MI, is associated with a higher risk of cardiogenic embolism, and an acute stroke may also precipitate an MI. A troponin level is usually adequate for this purpose, especially because it remains elevated for

several days after the MI when embolism from a mural thrombus is most likely to occur. Patients with acute stroke should be placed on telemetry monitoring. Urgent echocardiography is used selectively.

Brain Imaging

CT or MRI brain imaging is an essential part of the evaluation of all patients with suspected ischemic stroke. Imaging can locate the area of damage, distinguish a brain hemorrhage from an ischemic stroke, and identify mass lesions such as tumor (Chapter 189), abscess (Chapter 413), or subdural hematoma that can present acutely and mimic a stroke. Brain CT is widely and rapidly available and provides the information necessary for the treatment of most patients with acute stroke. Brain MRI can detect areas of acute ischemic injury not apparent on CT brain imaging (Fig. 407-2), but it cannot be performed in patients with metal implants and devices such as cardiac pacemakers and is a challenge to perform in unstable patients. The changes on brain CT, such as loss of gray-white distinction, loss of the insular ribbon, and blurring of the borders of the basal ganglia, can be subtle. The area of ischemic injury on brain CT scan appears as a relative hypodensity (Fig. 407-3), in contrast to brain hemorrhage, which appears hyperdense

CHAPTER 407  Ischemic Cerebrovascular Disease  

compared with the surrounding brain parenchyma (see Fig. 408-2). CT can also show acute hemorrhage in the subarachnoid space, which can be indicative of aneurysmal rupture (see Fig. 408-1). The dense middle cerebral artery sign or the dot sign, in which an artery in the sylvian fissure may appear dense, can indicate thrombus in these vessels. The findings on CT are often normal in the acute phase of ischemic stroke, and MRI is more sensitive for detecting acute ischemic injury (Fig. 407-4). Because brain CT imaging of posterior fossa structures is often obscured by beam hardening artifact from the petrous bones, MRI is also more sensitive for visualizing the brain stem and cerebellum. MRI signal patterns also can

2439

distinguish acute from subacute and remote ischemic injury, distinguish acute and remote brain hemorrhage, and identify other nonvascular conditions. However, MRI is not required before treatment with intravenous rtPA because CT can reliably exclude parenchymal brain hemorrhage and can detect other common conditions that may mimic a stroke, such as a mass lesion.

Lumbar Puncture

Lumbar puncture is rarely necessary in the evaluation of patients with acute stroke. In occasional patients, meningitis, especially septic meningitis from

TABLE 407-5  IMMEDIATE DIAGNOSTIC STUDIES: EVALUATION OF A PATIENT WITH SUSPECTED ACUTE ISCHEMIC STROKE

4 hours T2-wt.

ALL PATIENTS Noncontrast brain CT or brain MRI Blood glucose Oxygen saturation Serum electrolytes/renal function tests* Complete blood count, including platelet count* Markers of cardiac ischemia* Prothrombin time/INR* Partial thromboplastin time* ECG*

4 hours DWI

SELECTED PATIENTS Thrombin time and/or ecarin clotting time if it is suspected the patient is taking direct thrombin inhibitors or direct factor Xa inhibitors Hepatic function tests Toxicology screen Blood alcohol level Pregnancy test Arterial blood gas tests (if hypoxia is suspected) Chest radiography (if lung disease is suspected) Lumbar puncture (if meningitis is suspected or subarachnoid hemorrhage is suspected but the CT scan is negative for blood) Electroencephalogram (if seizures are suspected) *Although it is desirable to know the results of these tests before giving intravenous recombinant tissue-type plasminogen activator, fibrinolytic therapy should not be delayed while awaiting the results unless (1) there is clinical suspicion of a bleeding abnormality or thrombocytopenia, (2) the patient has received heparin or warfarin, or (3) the patient has received other anticoagulants (direct thrombin inhibitors or direct factor Xa inhibitors). CT = computed tomography; ECG = electrocardiogram; INR = international normalized ratio; MRI = magnetic resonance imaging. From Jauch EC, Saver JL, Adams HP Jr, et al. Guidelines for the early management of patients with acute ischemic stroke: a guideline for healthcare professionals from the American Heart Association/American Stroke Association. Stroke. 2013;44:870-947.

A

30 days T2-wt.

FIGURE 407-2.  Magnetic resonance imaging (MRI) showing possible advantages of diffusion-weighted imaging (DWI) relative to conventional MRI at early times after vascular occlusion. Top, Conventional T2-weighted MRI 4 hours after symptom onset that appears normal. Middle, At the same time, a DWI scan shows abnormalities in the left hemisphere. Bottom, Repeated T2-weighted MRI 1 month later showed an infarction in the same location as the initial DWI scan. (Courtesy Gregory W. Albers, Stanford University, Stanford, Calif.)

B

FIGURE 407-3.  Computed tomographic imaging. A, A computed tomography (CT) scan of a patient with a left hemisphere infarction 6 to 24 hours after the onset of symptoms shows a hypodense area in the basal ganglia region and compression of the frontal horn of the lateral ventricle. B, A CT scan shows the chronic infarction 1 year later; atrophy and loss of tissue volume are visible. (Courtesy Gregory W. Albers, Stanford University, Stanford, Calif.)

2440

CHAPTER 407  Ischemic Cerebrovascular Disease  

Anterior

A

Posterior

Anterior

B

Posterior

Anterior

C

Posterior

FIGURE 407-4.  A, Computed tomographic imaging. B, Magnetic resonance imaging, diffusion sequence. C, Magnetic resonance imaging, apparent diffusion coefficient (ADC) map. Computed tomography shows no evidence of ischemic injury. There is an obvious area of restricted diffusion in the right frontotemporal cortex that is dark on the ADC map, consistent with an area of acute ischemic injury.

cardiogenic embolism in a patient with infective endocarditis, may cause stroke or strokelike symptoms and be an indicator for an urgent lumbar puncture. Brain CT usually demonstrates blood in the subarachnoid space in patients presenting with symptoms and signs of subarachnoid hemorrhage (Chapter 408), such as headache and meningismus. If, however, brain CT fails to visualize a subarachnoid hemorrhage in a patient in whom the clinical suspicion of subarachnoid hemorrhage is high, lumbar puncture should still be obtained.

TABLE 407-6  TIME GOALS FOR EVALUATION AND TREATMENT OF PATIENTS WITH ACUTE ISCHEMIC STROKE TIME AFTER EMERGENCY DEPARTMENT ARRIVAL

Assess ABCs, vital signs Provide oxygen if hypoxemic Obtain intravenous access Obtain laboratory studies CBC, coagulation, electrolytes Check glucose level, treat if indicated Perform screening neurologic assessment Activate stroke team Order “stroke code” brain CT or MRI Obtain 12-lead ECG

25 minutes

Review history Establish time at onset or last known normal Perform neurologic examination NIH Stroke Scale

45 minutes

Review laboratory studies Review brain CT or MRI results Evaluate inclusion and exclusion criteria (see Tables 407-8 and 407-9)

60 minutes

Review risks and benefits Obtain consent Begin infusion

Other Imaging

Carotid duplex ultrasonography, which combines B-mode vascular imaging with measures of blood flow velocity, is commonly used to screen for extracranial carotid artery stenosis but is rarely indicated in the acute setting. Both CT and MR angiography provide noninvasive vascular imaging of the extracranial and intracranial cerebral circulation, and either study can be obtained in conjunction with parenchymal imaging in the acute setting. CT or MR angiography may be helpful in identifying cervical artery dissections in a patient with headache, neck pain, and symptoms and signs consistent with ipsilateral ischemic injury. Venous sinuses should be imaged if a sinus thrombosis is being considered. In patients who are not candidates for intravenous rtPA but who may be considered for acute endovascular therapy, these studies can be used to identify a proximal arterial occlusion. Transcranial Doppler ultrasonography is an alternative for evaluating the proximal cerebral vessels, but it cannot be obtained in some patients because of inadequate sonographic windows. Diagnostic catheter angiography carries about a 0.5 to 1.5% risk of causing a stroke and has largely been supplanted in the acute setting by noninvasive vascular imaging. Catheter angiography is, however, superior to CT or MR angiography for visualizing smaller intracranial vessels and for detecting intracranial vasculopathies such as vasculitis (Chapter 270).

Differential Diagnosis

The hallmark of acute ischemic stroke is the abrupt onset of a focal neurologic deficit, frequently attributable to an area of brain supplied by a specific artery or arteries. In some patients, however, the onset of ischemic stroke may be stuttering, and the stroke symptoms may have been heralded by a prior TIA. The detection of an ipsilateral cervical artery bruit may also support the diagnosis. Embolic stroke typically has its maximal severity at onset, but it can involve multiple vascular territories. The diagnosis of an embolic stroke may be further suggested by finding a cardiac murmur, an irregularly irregular heart rhythm, or signs of emboli in other vascular territories. A variety of other neurologic conditions may also be manifested acutely. Migraine with aura (Chapter 398) can be associated with focal neurologic deficits, including speech impairment, visual changes, vertigo, weakness, numbness, and imbalance. Partial seizures (Chapter 403) may have negative symptoms, including aphasia and paresis, and a patient with a postictal Todd paralysis may appear to have had a stroke. As a further challenge in diagnosis, seizures may occur in patients who are having an acute stroke. In a patient without a previous diagnosis, the first episode of multiple sclerosis (Chapter 411) can mimic a stroke. Mass lesions such as neoplasms (Chapter 189) and abscesses (Chapter 413) are generally associated with a slowly progressive

GOALS

10 minutes

ABCs = airway, breathing, circulation; CBC = complete blood count; CT = computed tomography; ECG = electrocardiogram; MRI = magnetic resonance imaging; NIH = National Institutes of Health.

worsening of neurologic symptoms but may occasionally be manifested acutely. Metabolic disorders such as hypoglycemia (Chapter 230) or hyperglycemia (Chapter 229), toxin exposures (Chapters 22 and 110), and drug intoxications (Chapter 34) can cause focal symptoms similar to stroke. Strokelike symptoms may also be a manifestation of malingering, a conversion disorder, or other psychiatric illness.

TREATMENT  Intravenous rtPA

After initial respiratory and hemodynamic stabilization, the management of patients with acute ischemic stroke is directed at determining expeditiously (Table 407-6) whether treatment with intravenous rtPA is appropriate (Table 407-7; see also Fig. 407-1). Intravenous rtPA, administered within 4.5 hours of the onset of symptoms, A1  does not reduce mortality but results in a higher odds of a better neurologic outcome at 3 months compared with placebo. The benefit of rtPA declines over time within this 4.5-hour treatment window (EFig. 407-4), with the odds ratio for a favorable 3-month outcome declining from 2.55 for treatment within 0 to 90 minutes, to 1.64 for 91 to 180 minutes, to 1.26 for 181 to 270 minutes, and to no statistical benefit for treatment

CHAPTER 407  Ischemic Cerebrovascular Disease  

Modified Rankin score 0-1 Odds ratio estimated by model 95% Cl for estimated odds ratio

4 3 2 1

Composite end point 5 Odds ratio and 95% Cl

Odds ratio and 95% Cl

5

4 3 2 1 0

0 60

A

90 120 150 180 210 240 270 300 330 360 OTT (min)

60

B

Mortality

Parenchymal hemorrhage type 2 Odds ratio and 95% Cl

Odds ratio and 95% Cl

C

90 120 150 180 210 240 270 300 330 360 OTT (min)

20

5 4 3 2 1 0

2440.e1

60

90 120 150 180 210 240 270 300 330 360 OTT (min)

16 12 8 4 0

D

60

90 120 150 180 210 240 270 300 330 360 OTT (min)

E-FIGURE 407-4.  Relation of onset to treatment delay with treatment effect with intravenous rtPA. Relation of stroke onset to start of treatment (OTT) with treatment effect after adjustment for prognostic variables assessed by (A) day 90 modified Rankin score 0-1 versus 2-6 (interaction P = 0269, n = 3530 [excluding EPITHET7 data P = 0116, n = 3431]); (B) global test that incorporates modified Rankin score 0-1 versus 2-6, Barthel Index score 95-100 versus 90 or lower, and NIHSS score 0-1 versus 2 or more (interaction P = 0111, n = 3535 [excluding EPITHET7 data P = 0049, n = 3436]); (C) mortality (interaction P = 0444, n = 3530 [excluding EPITHET7 data P = 0582, n = 3431]); and (D) parenchymal hemorrhage type 2 (interaction P = 4140, n = 3531 [excluding EPITHET7 data P = 4578, n = 3431]). Thus, for parenchymal hemorrhage type 2, the fitted line is not statistically distinguishable from a horizontal line. For each graph, the adjusted odds ratio is shown with the 95% confidence interval (CI). CIs from the models will differ from those shown in the tables because the model uses data from all patients treated within 0 to 360 minutes, whereas the categorized analyses in the tables are based on subsets of patients; the modeled CIs are deemed to be more reliable. (From Lees KR, Bluhmki E, von Kummer R, et al. Time to treatment with intravenous alteplase and outcome in stroke: an updated pooled analysis of ECASS, ATLANTIS, NINDS, and EPITHET trials. Lancet. 2010;375:1695-1703.)

CHAPTER 407  Ischemic Cerebrovascular Disease  

TABLE 407-7  ADMINISTRATION OF rtPA FOR ACUTE ISCHEMIC STROKE Infuse 0.9 mg/kg (maximum dose 90 mg) over 60 minutes, with 10% of the dose given as a bolus over 1 minute. Admit the patient to an intensive care or stroke unit for monitoring. If the patient develops severe headache, acute hypertension, nausea, or vomiting or has a worsening neurological examination, discontinue the infusion (if IV rtPA is being administered) and obtain emergent CT scan. Measure blood pressure and perform neurological assessments every 15 minutes during and after IV rtPA infusion for 2 hours, then every 30 minutes for 6 hours, then hourly until 24 hours after IV rtPA treatment. Increase the frequency of blood pressure measurements if systolic blood pressure is >180 mm Hg or if diastolic blood pressure is >105 mm Hg; administer antihypertensive medications to maintain blood pressure at or below these levels (Table 407-10). Delay placement of nasogastric tubes, indwelling bladder catheters, or intra- arterial pressure catheters if the patient can be safely managed without them. Obtain a follow-up CT or MRI scan at 24 hours after IV rtPA before starting anticoagulants or antiplatelet agents. CT = computed tomography; IV = intravenous; MRI = magnetic resonance imaging; rtPA = recombinant tissue plasminogen activator. From Jauch EC, Saver JL, Adams HP Jr, et al. Guidelines for the early management of patients with acute ischemic stroke: a guideline for healthcare professionals from the American Heart Association/American Stroke Association. Stroke. 2013;44:870-947.

beyond 4.5 hours. Registry studies support a benefit in routine clinical practice similar to that in randomized trials.6 As a result, current guidelines recommend that treatment with intravenous rtPA (Food and Drug Administration approved up to 3 hours after symptom onset) not be given if more than 4.5 hours have elapsed since the onset of symptoms. Treatment with rtPA is efficacious and safe among patients who are chronically treated with warfarin, provided their INR is 1.7 or lower, and is contraindicated with an INR higher than 1.7. A2  Treatment increases the risk of intracranial hemorrhage, but the overall benefit includes these adverse events, which do not significantly increase in frequency during the 4.5-hour treatment window. Some patients have absolute exclusion criteria against treatment with intravenous rtPA (Table 407-8), with additional relative contraindications for treatment between 3 and 4.5 hours (Table 407-9). In patients without contraindications, treatment should begin as soon as possible in either treatment window.

Endovascular Therapy

Although strokes caused by large proximal occlusions tend to benefit less from treatment with intravenous rtPA compared with more distal or smallvessel obstructions, no randomized trials show additional benefit of infusing rtPA directly into the thrombus or of other endovascular treatments compared with intravenous rtPA alone, even though several devices are Food and Drug Administration approved to remove clots from brain blood vessels. As a result, current guidelines recommend treatment with intravenous rtPA even if intraarterial treatments are available. Endovascular therapy can be considered in selected patients who cannot be treated with intravenous rtPA, such as patients who had a recent surgical procedure and who present up to 6 hours after a middle cerebral artery occlusion and perhaps longer after basilar artery occlusion.

Other Treatments

Regardless of whether the patient received intravenous rtPA or endovascular therapy, care in a comprehensive specialized stroke unit that incorporates rehabilitation is associated with better patient outcomes. Urgent anticoagulation to prevent recurrent stroke, to prevent worsening, or to improve functional outcome of patients with acute ischemic stroke is not recommended. Aspirin should not be started within 24 hours of treatment with intravenous rtPA, but aspirin should be started at 325 mg daily within 24 to 48 hours after the onset of stroke. A3  Hemicraniectomy can increase survival in patients with extensive middle cerebral artery strokes, but most survivors will require assistance with their body needs. A4  Antihypertensive medications to reduce blood pressure acutely by 10 to 25% in the first 24 hours with a goal of blood pressure to below 140/90 mm Hg by 1 week does not improve outcomes compared with discontinuation of all antihypertensive medications. A5  Current guidelines recommend that antihypertensive medications not be given unless the blood pressure rises to more than 220/120 mm Hg or higher in the absence of other indications. An exception is that blood pressure can be lowered in patients who are otherwise candidates for intravenous rtPA with a goal of maintaining blood pressures below 180/105 mm Hg after treatment (Fig. 407-5). Several potential complications of acute stroke can often be avoided. Patients with stroke in any vascular distribution are at risk of aspiration

2441

TABLE 407-8  INCLUSION AND EXCLUSION CHARACTERISTICS OF PATIENTS WITH ISCHEMIC STROKE WHO COULD BE TREATED WITH IV rtPA WITHIN 3 HOURS FROM SYMPTOM ONSET INCLUSION CRITERIA Diagnosis of ischemic stroke causing measurable neurological deficit Onset of symptoms 185 mm Hg or diastolic >110 mm Hg) Active internal bleeding Acute bleeding diathesis, including but not limited to Platelet count 1.7 or PT >15 seconds Current use of direct thrombin inhibitors or direct factor Xa inhibitors with elevated sensitive laboratory tests (such as aPTT, INR, platelet count, and ECT; TT; or appropriate factor Xa activity assays) Blood glucose concentration 1 3 cerebral hemisphere) RELATIVE EXCLUSION CRITERIA Recent experience suggests that under some circumstances—with careful consideration and weighting of risk to benefit—patients may receive fibrinolytic therapy despite 1 or more relative contraindications. Consider risk to benefit of IV rtPA administration carefully if any of these relative contraindications are present: Only minor or rapidly improving stroke symptoms (clearing spontaneously) Pregnancy Seizure at onset with postictal residual neurological impairments Major surgery or serious trauma within previous 14 days Recent gastrointestinal or urinary tract hemorrhage (within previous 21 days) Recent acute myocardial infarction (within previous 3 months) NOTES: • The checklist includes some FDA-approved indications and contraindications for administration of IV rtPA for acute ischemic stroke. Recent guideline revisions have modified the original FDA-approved indications. A physician with expertise in acute stroke care may modify this list. • Onset time is defined as either the witnessed onset of symptoms or the time last known normal if symptom onset was not witnessed. • In patients without recent use of oral anticoagulants or heparin, treatment with IV rtPA can be initiated before availability of coagulation test results but should be discontinued if INR is >1.7 or PT is abnormally elevated by local laboratory standards. • In patients without history of thrombocytopenia, treatment with IV rtPA can be initiated before availability of platelet count but should be discontinued if platelet count is 185/110 mm Hg: Pre-treatment

Options Include: • Labetalol (10-20 mg IV over 1-2 minutes, may repeat ×1) • Nicardipine (5 mg/h IV, titrate up by 2.5 mg/h every 5-15 minutes, maximum 15 mg/h; when desired BP reached, adjust to maintain proper BP limits)

If BP is not maintained at or below 185/110 mm Hg, do not administer rtPA

Ongoing treatment during and after acute reperfusion therapy to maintain BP ≤ 180/105 mm Hg: • Monitor BP every 15 minutes for 2 hours from start of rtPA therapy, then every 30 minutes for 6 hours, and then every hour for 16 hours

If systolic BP >180-230 mm Hg or diastolic BP >105-120 mm Hg: • Labetalol (10 mg IV followed by continuous IV infusion 2-8 mg/min) or nicardipine (5 mg/h IV, titrate up to desired effect by 2.5 mg/h every 5-15 minutes, maximum 15 mg/h)

If BP not controlled or diastolic BP >140 mm Hg, consider IV sodium nitroprusside Abbreviations: BP, blood pressure; IV, intravenously; and rtPA, recombinant tissue plasminogen activator. Adapted from Jauch EC, Saver JL, Adams HP, Jr., et al. Guidelines for the early management of patients with acute ischemic stroke: a guideline for healthcare professionals from the American Heart Association/ American Stroke Association. Stroke. 2013;44:870–947. FIGURE 407-5.  Potential Approaches to Arterial Hypertension in Acute Ischemic Stroke Patients Who are Candidates for Acute Reperfusion Therapy.

TABLE 407-9 RELATIVE CONTRAINDICATIONS TO IV rtPA IN PATIENTS WITHIN 3 TO 4.5 HOURS AFTER ONSET OF SYMPTOMS OF ACUTE ISCHEMIC STROKE National Institutes of Health Stroke Scale >25 (see Table 407-4) Age > 80 years old Taking an oral anticoagulant regardless of INR History of diabetes and a prior ischemic stroke INR, international normalized ratio; IV, intravenous; and rtPA, recombinant tissue plasminogen activator. Adapted from Jauch EC, Saver JL, Adams HP, Jr., et al. Guidelines for the early management of patients with acute ischemic stroke: a guideline for healthcare professionals from the American Heart Association/American Stroke Association. Stroke. 2013;44:870-947.

  UNUSUAL CAUSES OF STROKE

Ischemic strokes may be caused by a variety of rarer conditions. Specific treatments, many of which are not supported by extensive clinical trial data, vary accordingly (Table 407-10).

Cerebral Venous Thrombosis Thrombosis of a cerebral venous sinus can cause headache, focal strokelike manifestations, seizures, altered mental status, and papilledema.7 With superior sagittal sinus obstruction (see Fig. 406-10), patients can develop bilateral leg weakness and sensory changes. Obstruction of a transverse sinus or one of the major veins over the cerebral convexity (see Fig. 406-11) can also produce symptoms, depending on the area of the brain that is injured. Cerebral venous sinus thrombosis is an uncommon condition that is usually seen in patients with coagulopathies, disseminated cancer, or a prior inner ear infection. It can also occur in the peripartum period. Venous obstruction can mimic an ischemic arterial stroke, but symptoms and signs are often more diffuse and resemble encephalitis (Chapter 414) or meningitis (Chapter 412). The diagnosis can be suspected on routine CT or MRI and confirmed by CT or MR venography (Fig. 407-6). Initial acute treatment options include either body weight–adjusted subcutaneous low-molecular-weight heparin (see Table 38-2) or dose-adjusted intravenous heparin (see Table 81-4), even if patients have some degree of hemorrhage,8 and one small randomized trial found that treatment with low-molecular-weight heparin was associated with lower mortality. A7  Oral warfarin anticoagulation should be

started and continued for at least 3 months, with an INR target of 2.0 to 3.0. Longer periods of anticoagulation may be considered, depending on the cause of the sinus thrombosis.

Cervical Artery Dissection A cervical artery dissection or cerebral artery dissection, each of which is caused by the formation and subsequent longitudinal extension of an intramural hematoma, can narrow or obstruct the arterial lumen. These dissections can be spontaneous, or they can be associated with major neck injury, relatively minor trauma (such as a chiropractic neck manipulation or neck hyperextension), or otherwise innocuous activities, such as coughing, sneezing, or lifting. Patients may have underlying fibromuscular dysplasia (Chapters 67 and 80); inherited conditions, such as Marfan syndrome (Chapter 260), Ehlers-Danlos syndrome, or tuberous sclerosis (Chapter 417); an elevated blood homocysteine level; or no identified underlying cause. The diagnosis can be challenging but should be considered especially in an otherwise healthy young patient who has neck or facial pain in conjunction with a stroke. MR angiography may show a hyperintense mass adjacent to a flow void, and MR angiography or catheter angiography can show a tapered lumen leading to an obstruction or even a double lumen. Treatment may include thrombolysis, anticoagulation, or endovascular or surgical repair, depending on individual circumstances, but no randomized trials are available to guide such decisions. Vasculitis (Chapters 266, 270, and 271) can cause focal or multifocal cerebral ischemia due to local inflammation, stenosis, and even necrosis of extracranial or intracranial blood vessels.9 Patients can have preexisting or concurrent headaches, cognitive changes, and seizures. Because vasculitis often involves multiple arteries, multiple foci of ischemic injury on neuroimaging studies may mimic multiple emboli. Cerebral angiography classically shows multiple areas of beadlike segmental narrowing, but findings may be normal. Similar findings may occur with other causes of intracranial vasculopathy, and the angiographic appearance is not specific. The diagnosis may require leptomeningeal/cortical biopsy, which may be negative because the inflammatory process can be multifocal rather than diffuse. Examples of vasculitides that can cause strokelike symptoms include primary central nervous system vasculitis, systemic lupus erythematosus (Chapter 266), rheumatoid vasculitis (Chapter 264), Behçet disease (Chapter 270), Takayasu arteritis (Chapters 78 and 270), temporal arteritis (Chapter 271), fibromuscular dysplasia (Chapters 67 and 80), granulomatosis with angiitis (Chapter 270), sarcoidosis (Chapter 95), meningovascular syphilis (Chapter 319), and lymphomatoid angioendotheliomatosis.

CHAPTER 407  Ischemic Cerebrovascular Disease  

2443

TABLE 407-10  USUAL CAUSES OF ISCHEMIC STROKE CAUSE

SETTING

NOTES

Vasculitis (Chapter 270)

Patients commonly but not always have a prior known vasculitic condition

Vasculitis can be manifested with headache, cognitive impairment, multiple areas of infarction, or hemorrhage. Vasculitis affecting the cerebral vasculature can occur in the setting of systemic vasculitis or be confined to the central nervous system (primary vasculitis of the central nervous system). Diagnosis is supported by evidence of an inflammatory response in the spinal fluid, meningeal enhancement on MRI, and a typical pattern of focal stenoses on cerebral angiography. Brain/meningeal biopsy is often necessary to exclude other causes.

Steroids, immunosuppression

POSSIBLE TREATMENT

Sickle cell disease (Chapter 163)

Patients with known sickle cell disease; persons of African, Indian, and Mediterranean descent

Sickle cell disease can cause stroke by occlusion of small brain vessels or by intimal fibrosis leading to large-vessel occlusion. Patients with sickle cell disease can be monitored with transcranial Doppler ultrasound.

Transfusions to reduce hemoglobin S to 200 repeats) in the FMR1 gene.5 Fragile X syndrome may present as only moder­ ate to severe intellectual disability, but it is often associated with a prominent forehead, large ears, prominent jaw, and macro-orchidism. Postpubertal males often have poor impulse control, perseveration, and poor eye contact. Up to 25% of affected males have autism. Heterozygous females may be asymptomatic or may have a syndrome similar to what is seen in males, depending on repeat size and random X-inactivation. Other disorders associated with FMR1 include the fragile X ataxia syn­ drome, which is characterized by the late onset, usually after age 50 years, of progressive cerebellar ataxia and intention tremor in individuals who have an FMR1 premutation (60 to 200 repeats). It occurs equally in males and females. Diagnosis of FMR1 disorders is by molecular genetic testing. Cyto­ genetic testing for fragile sites is no longer recommended because it is less sensitive and more expensive than molecular testing. Treatment is symptom­ atic and supportive. Genetic counseling is recommended for affected indi­ viduals and their families.

Rett Syndrome Rett syndrome is a neurodevelopmental disorder that occurs classically in females with mutations in the MECP2 gene. MECP2 mutations are generally lethal in male embryos, but Rett syndrome has been reported in males with XXY karyotype or with somatic mosaicism. MECP2 is thought to mediate transcriptional silencing of methylated DNA. Most mutations are probably de novo or may reflect germline mosaicism; 99% of cases represent a single

occurrence within a family. Affected girls are usually normal at birth and have apparently normal development for the first 6 to 18 months of life. Brain growth decelerates, and development stagnates, followed by rapid regression of language and motor skills. A classic feature of Rett syndrome is the loss of purposeful hand use and the development of repetitive stereotyped hand movements that usually have the appearance of wringing or clapping. Other features present to variable degree are bruxism, episodic apnea and hyper­ pnea, seizures, gait disorders, and tremor.6 Non-neurologic features include growth failure and wasting, bowel dysmotility, scoliosis, osteopenia, and vasomotor changes in the limbs. Diagnosis is by clinical criteria followed by molecular genetic testing. Treatment is symptomatic.

Autism Autism or autism spectrum disorder is characterized by impaired social com­ munication and interactions as well as restricted and repetitive behaviors. Autism is associated with many different causes and is often idiopathic.7 Fragile X syndrome and tuberous sclerosis are two important entities in which an autistic phenotype can occur and in which autism may be the most prominent feature. Symptoms typically present before 3 years of age and persist into adult­ hood. Autism is a spectrum ranging from severe, with impairment in all domains, to mild with normal intellect and language but with impaired social interactions and repetitive behaviors or restricted interests. Autism has many causes but in most cases is idiopathic. Epilepsy is common in autism. Diag­ nosis is based on careful diagnostic interview and examination (Table 417-1). When epilepsy is present, treatment with antiepileptic medications is indi­ cated. Behavioral therapy can help individuals learn rules for social interaction and can improve communication. It can also help with problematic behavior. Educational support is important. Medications such as atypical antipsychot­ ics, selective serotonin reuptake inhibitors, and anxiolytics (Chapter 397) can help with aggressive behavior, repetitive behaviors, and anxiety.

  NEUROCUTANEOUS DISORDERS

Neurocutaneous disorders are congenital syndromes characterized by dys­ plastic and neoplastic lesions primarily involving the nervous system and skin. The more than 40 described syndromes include neurofibromatosis, tuberous sclerosis, Sturge-Weber syndrome, and von Hippel-Lindau disease.

Neurofibromatosis Neurofibromatosis encompasses a spectrum of syndromes with distinctive neural and cutaneous lesions. The two major forms of neurofibromatosis are genetically and clinically distinct.

NEUROFIBROMATOSIS TYPE 1

Neurofibromatosis type 1, which is the classic disorder described by von Recklinghausen, is an autosomal dominant condition with an incidence of

TABLE 417-1  DIAGNOSTIC CRITERIA FOR AUTISM SPECTRUM DISORDER 1. Deficits in Social Communication/Interaction (must have all three criteria): a. Problems reciprocating social or emotional interaction, including difficulty establishing or maintaining back-and-forth conversations and interactions, inability to initiate an interaction, and problems with shared attention or sharing of emotions and interests with others b. Severe problems maintaining relationships—ranges from lack of interest in other people to difficulties in pretend play and engaging in age-appropriate social activities, and problems adjusting to different social expectations c. Nonverbal communication problems such as abnormal eye contact, posture, facial expressions, tone of voice and gestures, as well as an inability to understand these 2. Restricted and Repetitive Behavior (at least 2 criteria must be met): a. Stereotyped or repetitive speech, motor movements or use of objects b. Excessive adherence to routines, ritualized patterns of verbal or nonverbal behavior, or excessive resistance to change c. Highly restricted interests that are abnormal in intensity or focus d. Hyper- or hyporeactivity to sensory input or unusual interest in sensory aspects of the environment Symptoms must be present in early childhood but may not become fully manifest until social demands exceed capacities. Symptoms need to be functionally impairing and not better described by another DSM-5 diagnosis. From American Psychiatric Association. Diagnostic and Statistical Manual of Mental Disorders. 5th ed. Arlington, VA: American Psychiatric Publishing, 2013.

CHAPTER 417  Congenital, Developmental, and Neurocutaneous Disorders  

1 per 2500 to 3000 births.8 Although it is an autosomal dominant disease, approximately 50% of cases are due to new mutations. Most mutations in NF1 occur in the parental germline. The NF1 gene, which is located on chromosome 17q11.2, codes a protein called neurofibromin, which is thought to function as a tumor suppressor by acting as a negative regulator of the Ras signaling pathway. Neurofibromatosis type 1 is characterized by multiple café au lait spots, axillary and inguinal freckling, multiple discrete cutaneous neurofibromas (Fig. 417-2), and Lisch nodules (Table 417-2). Subcutaneous neurofibromas may be painful or disfiguring. Learning dis­ abilities are present in at least 50% of individuals. Other manifestations include plexiform neurofibromas, optic nerve and other central nervous system (CNS) gliomas, malignant peripheral nerve sheath tumors, tibial dysplasia, and vasculopathy. Management of patients depends on the specific manifestations and often requires multidisciplinary collaboration. Most patients with neurofibromato­ sis type 1 do not require treatment, but all require surveillance (Table 417-3). Subcutaneous, intraspinal, and intracranial tumors can be treated surgically. Optic nerve gliomas may be treated with chemotherapy; both cisplatin and temozolomide have shown some benefit. Radiation is not recommended. Genetic counseling should be provided to all patients and their families.

NEUROFIBROMATOSIS TYPE 2

Neurofibromatosis type 2, which is often referred to as central neurofibroma­ tosis, is an autosomal dominant condition with an incidence of approximately

2515

1 in 25,000 individuals.9 The NF2 gene is located on chromosome 22q12.2. Its gene product merlin is a cytoskeletal protein thought to act as a membranestabilizing protein. The specific function of merlin is unknown. Neurofibro­ matosis type 2 is characterized by bilateral vestibular schwannomas, which usually present with symptoms of tinnitus, hearing loss, and imbalance. The age at onset is usually in young adulthood, but some individuals may develop posterior subcapsular lens opacities or mononeuropathy in childhood. Almost all affected individuals develop bilateral vestibular schwannomas by age 30 (Table 417-4). Affected individuals may also develop schwannomas of other cranial and peripheral nerves, meningiomas, and (rarely) ependymo­ mas or astrocytomas. Posterior subcapsular lens opacities are the most common ocular abnormality. Management is dependent on the specific manifestations and complica­ tions. In individuals who either have tested positive for known NF2 muta­ tions or have a family history of neurofibromatosis type 2 and whose genetic status cannot be determined with genetic testing, annual brain MRI is recom­ mended starting between ages 10 and 12 years and continuing until at least age 40 years. Hearing evaluations may be useful in detecting changes in audi­ tory nerve function before changes can be visualized by MRI. Routine com­ plete eye examinations should be part of the care of all individuals. Bevacizumab, a vascular endothelial growth factor inhibitor (5 mg/kg intravenously every 2 weeks), can improve hearing in some patients with neurofibromatosis type 2 and vestibular schwannomas. A1  Surgical treatment of schwannomas and meningiomas may be indicated to preserve function or to relieve compression of adjacent structures, especially in patients with intra­ medullary spinal tumors. Genetic counseling should be provided to affected individuals and their families.

Tuberous Sclerosis

FIGURE 417-2.  Multiple neurofibromas covering the back of a patient with neurofibromatosis type 1.

TABLE 417-2  DIAGNOSTIC CRITERIA FOR NEUROFIBROMATOSIS TYPE 1 Two or more of the following clinical features signify the presence of neurofibromatosis type 1: Six or more café au lait macules (>0.5 cm at largest diameter in prepubertal individuals or >1.5 cm in individuals past puberty) Axillary freckling or freckling in inguinal regions Two or more neurofibromas of any type or ≥ 1 plexiform neurofibroma Two or more Lisch nodules (iris hamartomas) A distinctive osseous lesion A first-degree relative with neurofibromatosis type 1 diagnosed by using the above-listed criteria

TABLE 417-3  RECOMMENDED SURVEILLANCE IN PATIENTS WITH NEUROFIBROMATOSIS TYPE 1 Annual physical examination by a physician who is familiar with the individual and with the disease Annual ophthalmologic examination in early childhood, less frequent examination in older children and adults Regular developmental assessment by screening questionnaire (in childhood) Regular blood pressure monitoring Other studies only as indicated on the basis of clinically apparent signs or symptoms Monitoring of those who have abnormalities of the central nervous system, skeletal system, or cardiovascular system by an appropriate specialist

Tuberous sclerosis complex is characterized by abnormalities of the brain, kidney, and heart.10 Tuberous sclerosis may occur as an autosomal dominant syndrome or result from spontaneous mutation. Two tuberous sclerosis genes have been identified. TSC1 (chromosome 9q34) codes for a protein called hamartin, a protein that interacts with the product of the TSC2 gene to inhibit the mammalian target of rapamycin (mTOR). TSC2 (chromosome 16p13) codes for tuberin, which interacts with hamartin. TSC2 mutations account for about 60% of individuals with clinical tuberous sclerosis.11 Specific findings vary across individuals, and severity ranges from minimal to severe. Skin lesions are seen in almost 100% of affected individuals, but CNS lesions are the leading cause of morbidity and mortality. Epilepsy is seen in as many as 80% of patients with CNS lesions. Intellectual impairment and developmental delay are common, and up to 40% of patients have an autism spectrum disorder. Giant cell astrocytoma is the leading cause of death. Up to 80% of children with tuberous sclerosis have an identifiable renal lesion (Chapter 197) by 10.5 years of age, and renal disease is the second leading cause of early death in individuals with tuberous sclerosis. Cardiac rhabdo­ myomas, which can occur in up to 50% of patients, are usually present at birth and typically regress over time. Diagnosis of tuberous sclerosis (Table 417-5) is usually clinical and confirmed by identification of calcified or uncalcified hamartomas on imaging studies (Fig. 417-3). Treatment is directed at complications of the disease, particularly epilepsy (Chapter 403). Neurosurgical intervention may sometimes be indicated for epilepsy and for symptomatic treatment of complications such as hydro­ cephalus, which results from midline giant cell tumors. In a randomized

TABLE 417-4  DIAGNOSTIC CRITERIA FOR NEUROFIBROMATOSIS TYPE 2 Presence of one or more of the following makes the diagnosis of neurofibromatosis type 2: • Bilateral vestibular schwannomas • A first-degree relative with neurofibromatosis type 2, and Unilateral vestibular schwannoma, or Any two of: meningioma, schwannoma, glioma, neurofibroma, posterior subcapsular lenticular opacities* • Unilateral vestibular schwannoma, and Any two of: meningioma, schwannoma, glioma, neurofibroma, posterior subcapsular lenticular opacities* • Multiple meningiomas, and Unilateral vestibular schwannoma, or Any two of: schwannoma, glioma, neurofibroma, cataract* *“Any two of ” refers to two individual tumors or cataracts.

2516

CHAPTER 417  Congenital, Developmental, and Neurocutaneous Disorders  

FIGURE 417-3.  Subependymal nodules and multiple cortical tubers in a patient with tuberous sclerosis.

TABLE 417-5  DIAGNOSTIC CRITERIA FOR TUBEROUS SCLEROSIS COMPLEX Definite—Two major features or one major feature plus two minor features Probable—One major feature plus one minor feature Possible—One major feature or two or more minor features MAJOR FEATURES Facial angiofibromas or forehead plaque Nontraumatic ungual or periungual fibromas More than three hypomelanotic macules (ash leaf spots) Shagreen patch (connective tissue nevus) Multiple retinal nodular hamartomas Cortical tuber Subependymal nodule Subependymal giant cell astrocytoma Cardiac rhabdomyoma, single or multiple Lymphangiomyomatosis Renal angiomyolipoma MINOR FEATURES Multiple dental enamel pits Hamartomatous rectal polyps Bone cysts Cerebral white matter radial migration lines Gingival fibromas Nonrenal hamartoma Retinal achromic patch “Confetti” skin lesions Multiple renal cysts

controlled trial, treatment with everolimus (10 mg/day) reduced the size of the angiomyolipomas in 42% percent of participants receiving active drug as compared with 0% of participants receiving placebo. A2  In another doubleblind placebo controlled trial, everolimus titrated to a concentration of 5 to 15 ng/mL was effective in reducing the size subependymal giant cell astro­ cytomas by at least 50% in 35% of participants receiving active drug as com­ pared with 0% of participants receiving placebo. A3  At similar doses, everolimus also can change fractional anisotropy and radial diffusivity, suggesting that the genetic defect of tuberous sclerosis complex in the brain may be modified pharmacologically. Serial brain MRI and renal ultrasound screening may be indicated in some patients because benign tumors of these organs may enlarge rapidly. Genetic counseling is an important part of management.

Sturge-Weber Syndrome Sturge-Weber syndrome is a sporadic disorder characterized by facial vascular nevi, epilepsy, cognitive impairment, and sometimes hemiparesis, hemianop­ sia, or glaucoma. It is most commonly due to somatic mutation in GNAQ (chromosome 9q21).12 The characteristic CNS feature of this disorder is

FIGURE 417-4.  Sturge-Weber syndrome. This patient has a classic diffuse capillary hemangioma in the distribution of the ophthalmic, nasociliary, and maxillary branches of the trigeminal nerve. The lesion extends backward over the anterior two thirds of the crown of the head. (From Forbes CD, Jackson WD. Color Atlas and Text of Clinical Medicine. 2nd ed. London: Mosby; 1996.)

capillary angiomatosis of the pia mater. Cerebral cortical calcifications are generally seen in a pericapillary distribution and are progressive. Most patients with Sturge-Weber syndrome have epilepsy. The diagnosis is usually based on the presence of a facial nevus (Fig. 417-4), which is manifested as a typical port-wine stain, and confirmatory imaging on a contrast brain MRI showing leptomeningeal enhancement. Regular ophthalmologic examination is warranted because of the risk for glaucoma. Treatment is usually aimed at the epilepsy, which can be medically intractable. In patients with intractable epilepsy and infantile-onset hemiple­ gia, hemispherectomy can improve the seizures and the neurodevelopmental outcome.

Von Hippel-Lindau Disease Von Hippel-Lindau disease (i.e., CNS angiomatosis) is an autosomal domi­ nant disorder caused by a defective tumor suppressor gene (VHL) at chro­ mosome 3p25-p26.13 It is characterized by retinal angiomas, brain (usually cerebellar) and spinal cord hemangioblastomas, renal cell carcinomas, endo­ lymphatic sac tumors, pheochromocytomas, papillary cystadenomas of the epididymis, angiomas of the liver and kidney, and cysts of the pancreas, kidney, liver, and epididymis. Both sexes are affected equally. Symptoms typically begin during the third or fourth decade. Retinal inflammation with exudate, hemorrhage, and retinal detachment from the retinal angiomas typically precedes the cerebellar complaints, but the order is not constant. The ocular findings are nonspecific, and the retinal detach­ ment may mask the underlying lesion. Headache, vertigo, and vomiting result from cerebellar tumors. Cerebellar signs such as ataxia, dysdiadochokinesis, and dysmetria are common. Rare patients present with symptoms of spinal cord or visceral lesions, or may have hearing loss from tumors of the endo­ lymphatic sac. Clinical diagnosis is established if the patient has more than one CNS hemangioblastoma, one hemangioblastoma with a visceral manifestation of the disease, or one manifestation of the disease and a known family history. Molecular genetic testing detects mutations in the VHL gene in nearly 100% of affected individuals. For patients with von Hippel-Lindau disease and for those with a diseasecausing VHL mutation, surveillance is recommended with annual ophthal­ mologic examination, annual blood pressure monitoring, measurement of urinary catecholamine metabolites beginning at age 5 years in families with pheochromocytoma, and annual abdominal ultrasound examination begin­ ning at age 16 years, with evaluation of suspicious lesions by computed tomography or MRI. Treatment is symptomatic. Retinal detachments and tumors are treated by laser therapy. Large brain tumors (Chapter 189), renal cell carcinomas (Chapter 197), pheochromocytomas (Chapter 228), epi­ didymal tumors (Chapter 200), and endolymphatic sac tumors are treated surgically; smaller CNS tumors may be treated by gamma knife.

CHAPTER 418  Autonomic Disorders and Their Management  

Grade A References A1. Plotkin SR, Stemmer-Rachamimov AO, Barker FG 2nd, et al. Hearing improvement after bevaci­ zumab in patients with neurofibromatosis type 2. N Engl J Med. 2009;361:358-367. A2. Bissler JJ, Kingswood JC, Radzikowska E, et al. Everolimus for angiomyolipoma associated with tuberous sclerosis complex or sporadic lymphangioleiomyomatosis (EXIST-2): a multicentre, ran­ domised, double-blind, placebo-controlled trial. Lancet. 2013;381:817-824. A3. Franz DN, Belousova E, Sparagana S, et al. Efficacy and safety of everolimus for subependymal giant cell astrocytomas associated with tuberous sclerosis complex (EXIST-1): a multicentre, ran­ domised, placebo-controlled phase 3 trial. Lancet. 2013;381:125-132.

GENERAL REFERENCES For the General References and other additional features, please visit Expert Consult at https://expertconsult.inkling.com.

418  AUTONOMIC DISORDERS AND THEIR MANAGEMENT WILLIAM P. CHESHIRE, JR. The peripheral autonomic nervous system and the central integration of autonomic reflexes maintain homeostasis and modulate the complex physiologic responses at the interface between the internal milieu and the external world. Autonomic activity, which generally occurs below the level of conscious control, regulates cardiovascular, thermal, metabolic, gastrointestinal (GI), urinary, and reproductive functions and coordinates the adaptive response to stress. Many diseases can involve the autonomic nervous system, which in turn can involve all organ systems. Examples include brain lesions that affect any part of the central autonomic network, disorders that damage peripheral nerve function, and systemic illnesses that impair autonomic responses.

EPIDEMIOLOGY

The most frequently disabling manifestation of autonomic failure is orthostatic hypotension, which increases in prevalence with aging and is associated with a two-fold increased risk for falls, fractures, syncope, transient ischemic attacks, and decreased functional capacity in elderly individuals. The prevalence of orthostatic hypotension is 5 to 20% among all elderly persons, but it rises to 30% in persons older than 75 years and to greater than 50% in frail individuals who live in nursing homes. Neurally mediated syncope (vasodepressor, vasovagal), as well as various situational syncopes that occur in response to emotional distress, micturition, defecation, coughing, carotid sinus stimulation, and other factors, accounts for 1 to 3% of all emergency room visits. The lifetime prevalence of syncope is about 20%. Another common autonomic syndrome is postprandial hypotension, which occurs in 20 to 60% of elderly individuals and is associated with an increased risk for mortality. Diabetes mellitus (Chapter 229) is the most common cause of autonomic neuropathy in the developed world. Within 10 to 15 years of the onset of diabetes, laboratory evidence of autonomic neuropathy can be detected in about 30% of patients, and symptomatic autonomic failure with orthostatic hypotension can be detected in about 5% of patients.1 Other autonomic symptoms in diabetics include constipation in 40 to 60% of patients, gastroparesis in 20 to 40%, bladder dysfunction in 30 to 80%, erectile impotence in more than 30% of males, and occasionally intermittent diarrhea. Hyperhidrosis involving the palms and soles represents the most common form of essential hyperhidrosis and affects about 1% of the population. Although excessive sweating is usually symptomatic, anhidrosis may go unnoticed unless it interferes with the thermoregulatory response to heat stress. Impaired thermoregulation can increase mortality during heat waves or in times of heat stress (Chapter 109).

PATHOBIOLOGY

The peripheral autonomic nervous system comprises three main divisions: (1) the sympathetic outflow from the thoracolumbar segments of the spinal cord, (2) the parasympathetic outflow from cranial nerves III, VII, IX, and X

2517

and the sacral spinal segments, and (3) the enteric ganglionated plexuses intrinsic to the wall of the gut. Disorders of the autonomic nervous system may occur suddenly or evolve gradually. They may affect specific or multiple autonomic pathways, depending on their pathogenesis and localization. Orthostatic hypotension, a hallmark of autonomic disorders, results from sympathetic vasomotor denervation, which renders a standing patient unable to constrict the splanchnic and other peripheral vascular beds in response to the pooling of blood volume (300 to 800 mL) owing to gravity. Physiologic effects of sympathetic activation include pupillary dilation, increased heart rate and contractility, increased peripheral vascular resistance, bronchodilation, increased glandular secretions, decreased GI motility, increased sweating, decreased function of reproductive organs, and mobilization of energy substrates. The effects of parasympathetic activation include pupillary constriction, lacrimal and salivary secretion, decreased heart rate and contractility, bronchoconstriction, increased GI motility, and contraction of the detrusor muscle of the bladder. Sympathetic and parasympathetic responses, though generally antagonistic, are not always equally counterbalanced among organ systems. Sympathetic preganglionic neurons, which use acetylcholine as their primary neurotransmitter, originate in the segmentally organized intermediolateral column of the spinal cord and exit via the ventral roots to pass through the white rami communicans and reach the paravertebral sympathetic chain ganglia, which innervate all organs and tissues except those of the abdomen and pelvis. The superior cervical ganglion, for example, innervates cranial structures, and the stellate ganglion innervates the upper limb. Sympathetic preganglionic axons also form the splanchnic nerves, which innervate the celiac, superior mesenteric, and hypogastric ganglia, as well as the adrenal medulla. With the exception of neurons that innervate the sweat glands, which are cholinergic, all other sympathetic postganglionic neurons are adrenergic neurons with norepinephrine as their primary transmitter. Preganglionic innervation of parasympathetic neurons is also cholinergic; however, in contrast to their sympathetic counterparts, the major postganglionic parasympathetic neurotransmitter is acetylcholine. Parasympathetic preganglionic fibers originate in the Edinger-Westphal, salivatory, and vagal dorsal motor nuclei in the brain stem. The ciliary, sphenopalatine, otic, submandibular, sublingual, and pelvic ganglia send postganglionic parasympathetic fibers to their target organs. Cranial nerves IX and X, which constitute the afferent limbs of the baroreceptor reflex, relay beat-to-beat information about systemic arterial pressure to the nucleus of the solitary tract. The GI tract contains neural plexuses, the most prominent of which are the myenteric (Auerbach) plexus found between the two layers of the muscularis externa and the submucosal (Meissner) plexus. Disorders of the enteric nervous system mainly affect GI motility or sphincter control rather than absorptive or secretory functions.

  BRAIN DISORDERS

Important disease targets involved in central autonomic regulation include the interrelated neuronal cell groups of the hypothalamus, as well as the ventrolateral medulla, nucleus of the solitary tract, parabrachial nucleus, periaqueductal gray matter, amygdala, and insular and prefrontal cortices. Together, these relay systems and integrative centers compose the central autonomic network, which when impaired, fails to activate or modulate sympathetic and parasympathetic tone and neurohumoral responses. A number of neurodegenerative disorders disrupt the central pathways of autonomic regulation. The most severe form is multiple system atrophy, a sporadic adult-onset disease in which severe autonomic failure accompanies and may precede parkinsonism (Shy-Drager syndrome; Chapter 409) or cerebellar ataxia (Chapter 410). In multiple system atrophy, degeneration of the striatum, pigmented nuclei, pontine nuclei, inferior olives, cerebellar Purkinje cells, and dorsal vagal and vestibular nuclei is associated with oligodendroglial cytoplasmic inclusions composed of α-synuclein filamentous aggregates. Autonomic dysfunction also occurs in dementia with Lewy bodies (Chapter 402) and to a lesser degree in Parkinson disease,2 both of which also involve abnormal neuronal accumulation of α-synuclein. Essential to the integration of behavioral, autonomic, and neuroendocrine responses is the tightly packed interwoven group of cells that constitute the hypothalamus. Lesions of the anterior hypothalamus may alter thirst perception and sodium regulation. Dysfunction of the magnocellular arginine vasopressin neurons of the supraoptic and supraventricular nuclei can be manifested as decreased secretion of antidiuretic hormone, thereby resulting in diabetes insipidus (Chapter 225) with hypovolemia, or as inappropriately increased secretion of antidiuretic hormone, thereby resulting in hyponatremia. Medial preoptic–anterior hypothalamic dysfunction associated with

CHAPTER 417  Congenital, Developmental, and Neurocutaneous Disorders  

GENERAL REFERENCES 1. Jamuar SS, Lam AN, Kircher M, et al. Somatic mutations in cerebral cortical malformations. N Engl J Med. 2014;371:733-743. 2. Colver A, Fairhurst C, Pharoah PO. Cerebral palsy. Lancet. 2014;383:1240-1249. 3. Doherty D, Millen KJ, Barkovich AJ. Midbrain and hindbrain malformations: advances in clinical diagnosis, imaging, and genetics. Lancet Neurol. 2013;12:381-393. 4. Klekamp J. Surgical treatment of Chiari I malformation—analysis of intraoperative findings, complications, and outcome for 371 foramen magnum decompressions. Neurosurgery. 2012;71: 365-380. 5. Visootsak J, Hipp H, Clark H, et al. Climbing the branches of a family tree: diagnosis of fragile X syndrome. J Pediatr. 2014;164:1292-1295. 6. Neul JL, Kaufmann WE, Glaze DG, et al. Rett syndrome: revised diagnostic criteria and nomencla­ ture. Ann Neurol. 2010;68:944-950.

2517.e1

7. Lai MC, Lombardo MV, Baron-Cohen S. Autism. Lancet. 2014;383:896-910. 8. Gutmann DH, Parada LF, Silva AJ, et al. Neurofibromatosis type 1: modeling CNS dysfunction. J Neurosci. 2012;32:14087-14093. 9. Ferner RE. The neurofibromatoses. Pract Neurol. 2010;10:82-93. 10. Kothare SV, Singh K, Hochman T, et al. Genotype/phenotype in tuberous sclerosis complex: asso­ ciations with clinical and radiologic manifestations. Epilepsia. 2014;55:1025-1029. 11. Kwiatkowski DJ, Manning BD. Molecular basis of giant cells in tuberous sclerosis complex. N Engl J Med. 2014;371:778-780. 12. Shirley MD, Tang H, Gallione CJ, et al. Sturge-Weber syndrome and port-wine stains caused by somatic mutation in GNAQ. N Engl J Med. 2013;368:1971-1979. 13. Richard S, Gardie B, Couve S, et al. Von Hippel-Lindau: how a rare disease illuminates cancer biology. Semin Cancer Biol. 2013;23:26-37.

2517.e2

CHAPTER 417  Congenital, Developmental, and Neurocutaneous Disorders  

REVIEW QUESTIONS 1. An 18-year-old man with more than 10 café au lait spots, axillary freckling, and a family history of neurofibromatosis type 1 has recently come into your practice for primary care. He is otherwise normal. Which of the fol­ lowing should be done as part of routine surveillance for complications of neurofibromatosis type 1? A. Annual brain MRI scan B. Annual physical examination with blood pressure C. Annual electrocardiogram D. Annual orthopedic evaluation E. Annual ophthalmologic examination Answer: B  Current guidelines recommend annual physical examination and blood pressure check in adults with neurofibromatosis type 1. Other evalua­ tions should be based on presence of symptoms or history of nervous system, skeletal or cardiovascular abnormalities. This patient has only cutaneous manifestations.

2. A 21-year-old with tuberous sclerosis has been diagnosed recently with a subependymal giant cell astrocytoma. Which of the following treatments should be considered to reduce the growth of this tumor? A. Everolimus B. Bevacizumab C. Rituximab D. Cyclophosphamide E. Ventriculoperitoneal shunting Answer: A  Everolimus has been shown in a controlled phase 3 randomized trial to reduce the size of subependymal giant cell astrocytomas by more than 50% in 35% of study participants. Bevacizumab has been used in patients with neurofibromatosis type 2 and vestibular schwannomas to improve hearing. The other treatments have not been shown to reduce the growth of subependymal giant cell astrocytomas.

2518

CHAPTER 418  Autonomic Disorders and Their Management  

dysgenesis of the corpus callosum (Shapiro syndrome) is characterized by episodic hyperhidrosis and hypothermia. Lesions of the posterior hypothalamus can result from hypothermia, Wernicke-Korsakoff syndrome (Chapter 416), acute traumatic brain injury (Chapter 399), multiple sclerosis (Chapter 411), mesodiencephalic hematoma, and toluene toxicity. Catastrophic neurologic conditions such as subarachnoid hemorrhage (Chapter 408), head trauma (Chapter 399), status epilepticus (Chapter 403), and acute hydrocephalus with increased intracranial pressure can profoundly stimulate sympathetic responses with cardiovascular consequence. Release of the hypothalamus as a result of cortical inhibition is the presumed mechanism. These paroxysmal sympathetic storms (diencephalic syndrome) are characterized by episodic sympathetic hyperactivity with hypertension, tachycardia, hyperventilation, pupillary dilation, flushing, and diaphoresis. Communicating hydrocephalus, structural lesions affecting the medial frontal cortex, and degenerative conditions affecting the frontobasal ganglia can cause urinary incontinence with uninhibited bladder contractions. Autonomic responses are closely linked to emotional states. Portions of the insular and anterior cingulate cortices mediate the autonomic responses to emotional stress. Because cardioregulatory function is represented within the insular cortex, insular strokes have been associated with destabilization of sympathoregulatory balance and occasional adverse cardiac events. Correlations of electroencephalography and electrocardiography have shown that seizures arising from the mesial temporal lobe may induce ictal tachycardia or, more rarely, bradycardia or asystole. Lesions of the brain stem may also be manifested as autonomic dysfunction. Damage to the medulla oblongata may give rise to hypertension, orthostatic hypotension, or syncope. Medullary ischemia or compression can cause acute neurogenic hypertension (Cushing response). Lateral medullary infarction (Wallenberg syndrome) typically produces ipsilateral Horner syndrome and occasionally more extensive dysautonomia, including bradycardia, acute hypertension, supine hypotension, or central hypoventilation.

TABLE 418-1  SOME CAUSES OF PERIPHERAL AUTONOMIC NEUROPATHY Metabolic

Diabetes mellitus Alcohol Acute intermittent porphyria Uremia

Autoimmune

Autoimmune autonomic ganglionopathy Guillain-Barré syndrome Morvan syndrome Lambert-Eaton myasthenic syndrome Chronic inflammatory demyelinating polyradiculoneuropathy Sjögren syndrome Systemic lupus erythematosus Mixed connective tissue diseases

Paraproteinemic

Amyloidosis

Nutritional

Cyanocobalamin deficiency Thiamine deficiency Gluten-sensitive neuropathy

Toxic

Heavy metals Organic solvents Organophosphates Vacor Acrylamide

Drug induced

Cisplatin Vincristine Amiodarone Metronidazole Perhexiline Paclitaxel

Infectious

Human immunodeficiency virus Leprosy Chagas disease Botulism Diphtheria Lyme disease

Genetic

Hereditary sensory and autonomic neuropathies   Types I and II   Type III (familial dysautonomia)   Type IV (congenital insensitivity to pain)   Type V Fabry disease

Idiopathic

Adie syndrome Ross syndrome Acute cholinergic neuropathy Chronic idiopathic anhidrosis Amyotrophic lateral sclerosis

  SPINAL CORD DISORDERS

Lesions of the spinal cord (Chapter 400), whether compressive, demyelinating, vascular, or neoplastic, commonly result in an overactive bladder (Chapter 26) with symptoms of frequency, urgency, and sometimes incontinence. Lesions that involve the sacral cord segments or cauda equina result in an underactive bladder with incomplete emptying, overflow incontinence, sphincter atonia, and sexual dysfunction. After spinal cord injuries above the level of splanchnic sympathetic outflow at T5, sprouting of afferent fibers in the thoracolumbar dorsal horns and necrosis of the descending white matter connections to sympathetic preganglionic neurons result in autonomic dysreflexia. In these patients, strong peripheral sensory stimuli such as bladder or bowel distention can induce a reversible state of sympathetic hyperresponsiveness that may present with hypertension, diaphoresis, flushing, or headache.

  PERIPHERAL GANGLIONOPATHIES AND NEUROPATHIES

Autonomic dysfunction can arise at the level of the autonomic ganglia or peripheral nerves (Table 418-1). Peripheral autonomic nerves are generally small in caliber and unmyelinated or thinly myelinated. Peripheral neuropathies that selectively involve small nerve fibers can cause various combinations of sensory, sympathetic, or parasympathetic signs and symptoms. Among the peripheral dysautonomias is the syndrome of pure autonomic failure, which is defined as the insidious onset of severe generalized autonomic failure as the sole clinical feature in the absence of any signs of extrapyramidal, cerebellar, or sensory or motor peripheral nerve dysfunction. Pathologic studies have found Lewy body accumulation in autonomic ganglia, peripheral autonomic nerves, the substantia nigra, and the locus ceruleus. Diabetic neuropathy involves autonomic nerves early in its course, and about 20% of patients advance to a clinically consequential cardiovascular autonomic neuropathy, which is a marker for and increased risk of cardiovascular mortality and morbidity.3 The incidence increases with the duration of diabetes and advancing age. In diabetic patients, microvascular ischemia causes progressive peripheral nerve damage, although an autoimmune mechanism may also play a role in a subset of patients. Glycemic burden appears to represent a continuum of risk for autonomic neuropathy, because some patients with impaired glucose regulation or newly diagnosed diabetes already have evidence of small fiber neuropathy. Other syndromes of neurologic autoimmunity (Table 418-2) include Guillain-Barré syndrome, in which antiganglioside antibodies mediate an

TABLE 418-2  AUTOIMMUNE AUTONOMIC NEUROPATHIES CLINICAL SYNDROME

ASSOCIATED AUTOANTIBODY

Autoimmune autonomic ganglionopathy

Anti-GAChR

Guillain-Barré syndrome

Anti-GM1, anti-GM3

Paraneoplastic autonomic neuropathy

ANNA-1 (anti-Hu), PCA-2, CRMP-5

Lambert-Eaton myasthenic syndrome

Anti-VGCC

Morvan syndrome

Anti-VGKC

Sjögren syndrome

SSA (anti-Ro), SSB (anti-La)

Intestinal dysmotility syndromes

Anti-GnRH

ANNA-1 = antineuronal nuclear antibody; anti-GAChR = nicotinic ganglionic acetylcholine receptor antibody; anti-GM1, anti-GM3 = antiganglioside antibody; anti-GnRH = gonadotropinreleasing hormone antibody; anti-VGCC = P/Q-type voltage-gated calcium-channel antibody; anti-VGKC = voltage-gated potassium-channel antibody; CRMP-5 = collapsin response mediator protein 5; PCA-2 = Purkinje cell cytoplasmic antibody type 2.

acute inflammatory demyelinating polyradiculoneuropathy that may be associated with tachycardia, blood pressure lability, and pupillomotor, sudomotor, and vasomotor disturbances (Chapter 420). The syndrome of acute pandysautonomia typically develops dramatically over a period of days to weeks as combined sympathetic and parasympathetic failure with GI dysmotility and, in contrast to Guillain-Barré syndrome, sparing of the somatic nerves. An antecedent (presumably viral) infection is reported in about 50%

CHAPTER 418  Autonomic Disorders and Their Management  

of cases. The finding of antibodies against the nicotinic acetylcholine receptor in the autonomic ganglia in many of these patients has established autoimmune autonomic ganglionopathy as a definable disease entity.4 Ganglionic acetylcholine receptor antibodies occur in some patients with lung cancer or thymoma. Low levels of these antibodies have also been found in a subset of patients with isolated GI dysmotility. Autoimmune neuromyotonia is characterized by peripheral nerve hyperexcitability, insomnia, fluctuating delirium, and prominent dysautonomia with hyperhidrosis and orthostatic intolerance. Most patients have antibodies to voltage-gated potassium channels (Chapter 422). Paraneoplastic autonomic neuropathies, which can predate the diagnosis of malignancy, are a rare epiphenomenon of malignancy, most frequently small cell lung carcinoma (Chapter 191), and can also occur in association with ovarian carcinoma (Chapter 199), breast carcinoma (Chapter 198), thymoma, lymphoma (Chapter 185), and other cancers. The most commonly encountered paraneoplastic antibody is antineuronal nuclear antibody type 1 (ANNA-1 or anti-Hu), which binds to a 35- to 40-kD family of neuronal nuclear RNA-binding proteins, including those in autonomic and enteric ganglia. Antibodies against collapsing response mediator proteins (CRMP-5 or anti-CV2) have also been associated with paraneoplastic autonomic neuropathy. Dysautonomia occurs in approximately 10 to 30% of patients with ANNA-1 and in 30% of patients with CRMP-5 seropositivity. Small cell lung cancer (Chapter 191) has been found in more than 80% of patients seropositive for ANNA-1 or CRMP-5. Hereditary sensory and autonomic neuropathy (HSAN) type I, which is due to mutations of the gene for serine palmitoyltransferase, is inherited in an autosomal dominant pattern and is characterized by distal anhidrosis with loss of nociceptive and thermal perception. Mutations in HSN2 have been linked to HSAN type II, an autosomal recessive neuropathy that causes distal anhidrosis and sensory loss. Sural nerve biopsy specimens disclose virtual absence of myelinated fibers and decreased numbers of unmyelinated fibers. Autonomic involvement is most prominent in HSAN type III, commonly known as familial dysautonomia or Riley-Day syndrome. This autosomal recessive disorder, which affects 1 in 3600 live births in parents of Ashkenazi Jewish descent, has been linked to mutations in the I-κB kinase–associated protein gene (IKBKAP). Pathologic studies disclose severely depleted sympathetic preganglionic and postganglionic neuronal populations with preservation of parasympathetic neurons. Affected children cry without tears, feed poorly, lack lingual fungiform papillae, have depressed patellar reflexes, and are subject to orthostatic hypotension and autonomic storms owing to impaired baroreflex afferent neurons. HSAN type IV, or congenital insensitivity to pain with anhidrosis, is an autosomal recessive disorder that is associated with mental retardation and repeated episodes of fever. HSAN type IV results from mutations in the gene for neurotrophic tyrosine kinase receptor type 1 (NTRK1), which is important for inducing neurite outgrowth in embryonic sensory and sympathetic neurons. Peripheral nerve biopsy discloses virtual absence of unmyelinated fibers. HSAN type V is an autosomal recessive disorder characterized by loss of pain and thermal sensation. It is caused by mutations in the nerve growth factor beta subunit (NGFB) gene. The age-related decline in baroreflex sensitivity, adrenergic responses to sympathetic activation, parasympathetic control of heart rate, esophageal and GI motility, and efficient thermoregulation predisposes the elderly to orthostatic, postprandial, and drug-induced hypotension. Pheochromocytomas (Chapter 228) cause paroxysmal autonomic symptoms, including blood pressure lability, and carcinoid syndrome (Chapter 232) causes flushing.

CLINICAL MANIFESTATIONS

Several distinct patterns of autonomic dysfunction help in identifying underlying causes.

Generalized Autonomic Failure

Early symptoms of adrenergic failure typically include lightheadedness on arising in the morning or following a warm shower, physical exercise, or a large meal. Other common symptoms include male erectile dysfunction, decreased sweating, dry mouth, constipation, and bladder dysfunction. Severe orthostatic hypotension without pulse acceleration (Chapter 62), which is the hallmark of severe generalized autonomic failure, occurs in at least 50% of patients and may be accompanied by supine and nocturnal hypertension, in which the normal diurnal decrease in blood pressure during sleep is reversed.

2519

Acute Autonomic Syndromes

Acute or subacute manifestations of autonomic dysfunction may result from rapidly developing disease or from decompensation of chronic autonomic disease. An abrupt onset of new focal autonomic signs, particularly when accompanied by headache or motor or sensory deficits, should prompt a detailed neurologic assessment for an acute cerebral or spinal syndrome, which may be caused by vascular, traumatic, inflammatory, neoplastic, or infectious diseases.

Chronic Autonomic Syndromes

The clinical spectrum of chronic autonomic neuropathies includes distal small fiber neuropathies with a stocking-and-glove distribution of anhidrosis, often combined with loss of pain and temperature sensibility. Other symptoms include orthostatic hypotension and impaired exercise tolerance, with either an increased resting heart rate due to a parasympathetic cardiovagal neuropathy or a fixed heart rate that does not increase adequately in response to physiologic demands as a result of the involvement of sympathetic fibers. Some patients with distal sudomotor neuropathy will complain of spontaneous or gustatory proximal hyperhidrosis with episodic sweating involving the face, head, and upper part of the trunk. Gastroparesis, which is a common feature of autonomic neuropathies, is characterized by delayed gastric emptying, which may be manifested as early satiety, nausea, anorexia, bloating, and sometimes pain and weight loss. Intestinal dysmotility may cause severe constipation (Chapter 136). In advanced peripheral neuropathies (Chapter 420), in ganglionopathies in which involvement is not length dependent, and in central degenerative disorders, the loss of sweating may extend to proximal body regions or globally. Widespread anhidrosis may result in impaired thermoregulatory sweating and the potential for hyperthermia in conditions of heat stress (Chapter 109).

Paroxysmal Dysautonomias

Paroxysmal and episodic autonomic symptoms include postprandial hypotension, which is a reduction in systolic blood pressure of at least 20 mm Hg within 2 hours of the start of a meal, typically one high in carbohydrate content. Postprandial hypotension may occur in elderly individuals with or without orthostatic hypotension. Dysfunction of the afferent limb of the baroreflex system leads to volatile blood pressure in patients with acute inflammatory demyelinating poly­ neuropathy and syndromes of arterial baroreflex failure. Additional causes of aberrant activation of autonomic reflexes include epilepsy, diencephalic syndrome, subarachnoid hemorrhage, acute head trauma, pheochromocytoma, intoxication, drug withdrawal, neurally mediated syncope, and panic disorder.

Selective Autonomic Syndromes

Regional autonomic disorders are characterized by focal or system-selective autonomic dysfunction. An example is harlequin syndrome, in which heat stress, exercise, or sudden emotion in a patient with hemifacial cutaneous sympathetic denervation evokes a dramatic facial division in which the denervated half remains pale and dry and the intact half flushes red. Oculosympathetic paresis (Horner syndrome) may also be present. Harlequin syndrome may occur in patients with Holmes-Adie syndrome, which consists of tonic pupils with asymmetrical or absent tendon reflexes, and has been described in patients with Ross syndrome, a partial dysautonomia consisting of the clinical triad of unilateral or bilateral tonic pupils, tendon hyporeflexia, and segmental body anhidrosis.

Other Specific Syndromes

Baroreflex failure is commonly the result of damage to the carotid sinus baroreceptors or the glossopharyngeal nerves. Similarly, interruption of vagal input from the aortic arch baroreceptors to the nucleus of the solitary tract can impair baroreflex responses. Baroreflex failure also occurs in patients who have undergone surgery or irradiation of the neck. These patients will exhibit severe and labile hypertension with concomitant tachycardia, palpitations, headache, diaphoresis, and emotional lability.5 Takotsubo cardiomyopathy (Chapter 60) is a stress-induced syndrome of typically reversible left ventricular myocardial dysfunction brought on by a catecholamine surge. Serotonin syndrome develops within hours or days of the addition of a new serotonergic agent to a drug regimen that already enhances serotonergic neurotransmission, on overdose with a selective serotonin reuptake inhibitor, or from abuse of psychostimulants such as amphetamine, methamphetamine,

2520

CHAPTER 418  Autonomic Disorders and Their Management  

and 3,4-methylenedioxymetamphetamine (MDMA or “ecstasy”). Manifestations include agitation, hypervigilance, confusion, hyperthermia, increased sweating, fluctuating blood pressure, hyperreflexia, and myoclonus. Neuroleptic malignant syndrome (Chapter 432) is a potentially lifethreatening hypermetabolic condition that develops within days to weeks in 0.2% of patients who receive drugs that block dopamine 2 receptors. The clinical findings consist of hyperthermia, profuse sweating, muscle rigidity, bradykinesia, and delirium. If the offending medication is not withheld, the syndrome may progress to tachycardia, tachypnea, labile blood pressure, myoclonus, obtundation, and catatonia. Among the infectious neuropathies, tetanus infection (Chapter 296) causes sympathetic overactivity in a third of patients because of the exotoxin tetanospasmin, which is taken up by peripheral nerve terminals and transported across synaptic junctions to reach the central nervous system. There it binds to gangliosides at presynaptic junctions to disinhibit preganglionic neurons and damages autonomic brain stem nuclei. Sympathetic hyperactivity results in labile or persistent hypertension or hypotension, tachyarrhythmias, peripheral vasoconstriction, fever, and profuse sweating. Diphtheritic neuropathy (Chapter 292) causes bulbar weakness and may be associated with cardiovagal impairment but not usually with orthostatic hypotension. The acute cholinergic neuropathy of botulism (Chapter 296) occurs along with bulbar and generalized neuromuscular paralysis 12 to 36 hours after the ingestion of food contaminated with the gram-positive anaerobic bacterium Clostridium botulinum. Botulinum toxin binds with high affinity to presynaptic receptors of cholinergic nerve terminals and inhibits the release of acetylcholine, thereby blocking neuromuscular and cholinergic autonomic transmission. Autonomic manifestations include anhidrosis, dry eyes, dry mouth, paralytic ileus, gastric dilation, urinary retention, and sometimes orthostatic hypotension with fluctuating blood pressure and vasomotor tone. Human immunodeficiency virus infection commonly causes autonomic disturbances, particularly in its advanced stages. Manifestations can include orthostatic hypotension, tachycardia, urinary dysfunction, impotency, diarrhea, and cardiac conduction defects. Perivascular mononuclear inflammatory infiltrates and neuronal degeneration in biopsy specimens of sympathetic ganglia suggest an autoimmune pathogenesis. Chagas disease (Chapter 347) causes a predominantly parasympathetic neuropathy characterized by megaesophagus, megaduodenum, and megacolon, as well as sympathetic cardiovascular failure with cardiomegaly and conduction defects. The autonomic neuropathy has an autoimmune basis and develops years to decades following primary infection with Trypanosoma cruzi. Leprosy (Chapter 326), one of the most common causes of neuropathy worldwide, frequently causes peripheral autonomic neuropathy as a result of an immune reaction against Mycobacterium leprae. Focal anhidrosis occurs in areas of hypopigmented and hypoesthetic skin. Cardiac denervation and orthostatic hypotension have been described. Toxins known to cause autonomic neuropathy include the rodenticide Vacor, as well as thallium, arsenic, mercury, acrylamide, and organic solvents such as carbon disulfide and hexacarbon. Organophosphate poisoning (Chapter 110) induces miosis and copious secretions. Ergot poisoning from rye contaminated with the fungus Claviceps purpurea results in intense vasoconstriction, paresthesia, seizures, and diarrhea. Poisoning with muscarine, which is present in certain poisonous mushrooms (Chapter 110), results in increased salivation, sweating, and lacrimation followed by nausea, abdominal pain, and diarrhea. Medicinal drugs that may induce an autonomic peripheral neuropathy include cisplatin, vincristine, amiodarone, metronidazole, perhexiline maleate, and paclitaxel. Many drugs are capable of increasing or decreasing sweating (Table 418-3). Nutritional deficiencies that lead to autonomic dysfunction include alcoholic neuropathy, which is a dying-back neuropathy identical to that of beriberi that is caused by thiamine deficiency (Chapter 416). Distal parts of the vagus nerve are affected early, and orthostatic hypotension may occur in more advanced stages. Subacute combined degeneration from vitamin B12 deficiency (Chapter 218) results in axonal degeneration and is occasionally manifested as orthostatic hypotension. Autonomic neuropathy has been described in some cases of celiac disease (Chapter 140). Amyloidosis (Chapter 188) results from the focal deposition of insoluble fibrillary proteins arranged in β-pleated sheet configurations within the extracellular space of various tissues, which may include the vasculature of peripheral autonomic nerves and sympathetic ganglia. Amyloid neuropathy is typically manifested as a painful distal small fiber sensory and severe autonomic neuropathy. autonomic dysfunction frequently occurs in primary AL

TABLE 418-3  SOME COMMONLY PRESCRIBED DRUGS THAT AFFECT SWEATING Drugs that increase sweating Opioids Serotonin reuptake inhibitors Anticholinesterases Cholinergic agonists Drugs that decrease sweating M3 anticholinergics Carbonic anhydrase inhibitors Tricyclic antidepressants Neuroleptics Antihistamines Central α-adrenergic agonists Botulinum toxin

(amyloid light-chain), immunoglobulin light chain–associated disease, and hereditary amyloidosis, but only rarely in reactive or AA (amyloid A) amyloidosis.

Functional Dysautonomias

A functional autonomic disorder is a medical condition that impairs normal autonomic function in some way but in the absence of a known structural neurologic deficit. Examples include neurally mediated syncope (Chapter 62), irritable bowel syndrome (Chapter 137), and some forms of orthostatic intolerance and pain, the molecular basis of which await discovery. Syndromic features can help to disambiguate functional dysautonomias from psychosomatic disorders (Chapter 397), which also may manifest autonomic symptoms just as they may manifest sensory or motor symptoms.

DIAGNOSIS

Clinical evaluation of autonomic dysfunction begins with a careful history. It is important to distinguish chronic and stable conditions from progressive and episodic phenomena and to recognize the circumstances that provoke or modify symptoms. Orthostatic hypotension, for example, is typically worse in the morning and may be aggravated by dehydration, deconditioning, prolonged standing, physical exertion, heat, carbohydrate ingestion, or menstruation (Table 418-4).6

Bedside Evaluation

The skin examination should assess turgor, pallor, flushing, and acral cyanosis, as well as any asymmetry of sweating, which may be more palpable than visible. Signs of pupillary asymmetry, ptosis, mucosal dryness, distal sensory or reflex changes, bradykinesia, or rigidity should be noted. Blood pressure and heart rate should be measured with the patient supine and again after standing for 1 to 3 minutes and correlated with symptoms. Orthostatic hypotension is defined as a reduction in systolic blood pressure of at least 20 mm Hg or a reduction in diastolic blood pressure of at least 10 mm Hg, with or without symptoms, within 1 to 3 minutes of assuming an erect posture.7 Neurogenic orthostatic hypotension is typically sustained with continued standing. Measurements taken immediately on standing can be misleading because some healthy young persons without orthostatic hypotension will exhibit transient hypotension within 30 seconds of standing but then recover. Except in patients treated with β-blockers, orthostatic hypotension without reflex tachycardia is evidence of generalized adrenergic failure. If reflex tachycardia occurs, dehydration or excessive venous pooling should be considered. Some patients with orthostatic intolerance experience an abnormal increase in heart rate rather than a drop in blood pressure on standing. Postural tachycardia syndrome8 is defined as an increase in heart rate by more than 30 beats per minute in adults (40 beats per minute in adolescents) and to consistently greater than 120 beats per minute when standing.

Laboratory Evaluation

Appropriate laboratory testing depends on the type and distribution of autonomic dysfunction. Investigations may include a complete blood cell count, fasting glucose, electrolytes, morning cortisol, thyroid function testing, vitamin B12 level, and when indicated, autoimmune markers. Creatine kinase should be checked in patients with hyperthermia. In a patient with an autonomic neuropathy, seropositivity for any of the characterized paraneoplastic autoantibodies should prompt a careful search for an underlying malignancy, even if the results of routine imaging studies

2521

CHAPTER 418  Autonomic Disorders and Their Management  

TABLE 418-4  GRADING OF ORTHOSTATIC INTOLERANCE SYMPTOM FREQUENCY

ACTIVITIES OF DAILY LIVING IN THE UPRIGHT POSTURE

STANDING TIME   (ON MOST OCCASIONS)

ORTHOSTATIC BLOOD PRESSURE

Grade I

Infrequent orthostatic symptoms developing only under conditions of increased stress*

Unrestricted

>15 min

May or may not be abnormal

Grade II

Intermittent orthostatic symptoms occurring at least weekly

Some limitation

>5 min

Some changes in cardiovascular indices, e.g., oscillations, or decrease in pulse pressure by > 50%

Grade III

Frequent orthostatic symptoms occurring on most occasions

Marked limitation

>1 min

Orthostatic hypotension is present > 50% of the time, recorded on different days

Grade IV

Orthostatic symptoms are consistently present

Incapacitated and unable to stand without presyncope or syncope developing

1 : 10,000) of anti-MAG IgM antibodies are associated with neuropathy, and intraneural or systemic

2532

CHAPTER 420  Peripheral Neuropathies  

injection of anti-MAG IgM M proteins causes complement-mediated demyelination of nerves in animals.

DIAGNOSIS

absent in the early stage of the disease but can become prominent later as a result of axonal degeneration. Fasciculations, cramps, and myokymia occur in patients with multifocal motor neuropathy and those with amyotrophic lateral sclerosis (Chapter 419), thereby making distinction between the two disorders difficult. Marked asymmetry in the degree of clinical findings and electrophysiologic abnormalities between contiguous nerves is suggestive of multifocal motor neuropathy rather than amyotrophic lateral sclerosis. Cranial nerve involvement or respiratory failure as a result of unilateral or bilateral phrenic nerve palsy rarely occurs in multifocal motor neuropathy. The presence of sensory loss suggests Lewis-Sumner syndrome.

TREATMENT

The diagnosis is established by the presence of multifocal, persistent partial conduction blocks on motor but not sensory nerve conduction studies. Lewis-Sumner syndrome has sensory loss as well as weakness, with conduction block in both sensory and motor nerves.

CLINICAL MANIFESTATIONS

Most patients with anti-MAG neuropathies are initially seen in their sixth to seventh decade of life with dysesthesias and paresthesias in their legs and unsteadiness while walking because of loss of proprioception. Physical examination shows a length-dependent large fiber sensory neuropathy. Weakness may develop later. Nerve conduction velocities are slow (about 25 m/second) with pronounced delays in distal motor latencies, thus prompting the designation distal acquired demyelinating symmetrical neuropathy to distinguish the disorder from CIDP.

Treatment of neuropathies associated with monoclonal gammopathy is similar to that for CIDP (see earlier). However, patients with anti-MAG-related neuropathies do not respond as well to treatment as do patients with CIDP. Anecdotal data support the benefit of rituximab (375 mg/m2 weekly for 4 weeks) in some patients.

PROGNOSIS

Progression of the neuropathy of monoclonal gammopathy disables about 25% of patients after 10 years and 50% after 15 years. The course of patients with osteosclerotic myeloma and neuropathy depends on the response to treatment of the myeloma. In patients whose myeloma responds to treatment, more than 50% have improvement in neuropathy. In other patients with sensorimotor neuropathies associated with plasma cell dyscrasias, the course is variable, and the M protein may not be related to their neuropathy.

Multifocal Motor Neuropathy and Lewis-Sumner Syndrome

DEFINITION

Multifocal motor neuropathy is characterized by progressive, distal more than proximal, asymmetrical limb weakness, mostly affecting the upper limbs with minimal or no sensory impairment.

EPIDEMIOLOGY

The prevalence of multifocal motor neuropathy is estimated at 2 per 100,000. Men are more frequently affected than women (2.6 : 1). Initial symptoms develop in 80% between the ages of 20 and 50 years, with a mean age at onset of 40 years. Lewis-Sumner syndrome occurs less frequently than multifocal motor neuropathy.

PATHOBIOLOGY

Multifocal motor neuropathy is considered to be an autoimmune neuropathy based on its clinical improvement with immunologically based therapies and because of a frequent association with antiglycolipid antibodies. Patients with MMN often have serum antibodies that react with ganglioside GM1, and these titers decrease during effective treatment. GM1 is highly represented in neural membranes at the nodes of Ranvier, compact myelin, and the motor end plate at the neuromuscular junction. A blocking effect on mouse distal motor nerve conduction has been induced in vitro by sera from multifocal motor neuropathy patients with and without high anti-GM1 antibody titers. These data support the presence of serum factors responsible for conduction block in the sera of patients with multifocal motor neuropathy, although these factors are not invariably related to anti-GM1 antibodies. Lewis-Sumner syndrome, however, is not associated with antiganglioside antibodies.

CLINICAL MANIFESTATIONS

The usual pattern is progressive, distal, asymmetrical arm weakness, often in the distribution of a single nerve. In a minority of patients, weakness may start proximally or in the legs. The disease will frequently affect other nerves, occasionally with a crossed distribution (i.e., one arm and the contralateral leg). Asymmetry and predominance of arm weakness may become less evident as the disease progresses. Localized muscle atrophy may be mild or

DIAGNOSIS

TREATMENT IVIG (2 g/kg) is the initial treatment for multifocal motor neuropathy, and almost 80% of patients respond within a week. However, improvement is typically brief (3 to 6 weeks), so repeated treatments are required indefinitely.11 Clinical improvement is often accompanied by a reduction or resolution of the motor conduction block in some nerves, but it does not consistently correlate with a reduction in antiganglioside antibody titers. Patients may eventually become refractory to IVIG, and another agent may be needed, such as rituximab (e.g., 375 mg/m2 weekly for 4 weeks) or azathioprine (2 to 3 mg/kg/day). Plasma exchange and corticosteroids are generally ineffective and have been associated with worsening neuropathy in some patients. Lewis-Sumner syndrome, which is a multifocal variant of CIDP, responds to the same treatments as CIDP.

  PARANEOPLASTIC NEUROPATHIES DEFINITION

Paraneoplastic neuropathies (Chapter 179) are a “remote effect of cancer” not caused by metastatic invasion of neural tissue; radiation therapy or chemotherapy; metabolic, vascular, or hormonal disturbances; or opportunistic infections. It is hypothesized that they are the result of host immune responses to a tumor antigen or antigens that are also present in neural tissues.

EPIDEMIOLOGY

Paraneoplastic syndromes occur in less than 1% of patients with cancer; peripheral neuropathy is only one of the paraneoplastic syndromes. Although more than 25% of patients with cancer have evident neuropathy on neurologic examination, the relationship to malignancy is unclear in most. Paraneoplastic neuropathy may develop before, during, or after the tumor is diagnosed. In certain tumors, neuropathies are distinctive and should prompt a thorough investigation for cancer. Small cell carcinoma of the lung (Chapter 191) is by far the most common underlying neoplasm, followed by carcinoma of the stomach, breast, colon, rectum, ovary, and prostate.

PATHOBIOLOGY

Subacute sensory neuropathy, the most characteristic paraneoplastic neuropathy, results from an immune-mediated ganglionitis that destroys sensory neurons in the dorsal root ganglia. Mononuclear inflammatory infiltrates composed of CD4+ and prominent CD8+ T cells, along with plasma cells, are found in the stroma surrounding the dorsal root ganglion neurons. Other findings include atrophy of the dorsal roots; loss of sensory neurons, which appear to be replaced by a proliferation of satellite cells (Nageotte nodule); axonal degeneration; and secondary degeneration of the dorsal column of the spinal cord. Inflammatory infiltrates can also be found in peripheral nerves or muscle. Sural nerve biopsies typically reveal only loss of myelinated nerve fibers and are not useful for diagnosis.

CLINICAL MANIFESTATIONS

Subacute sensory neuropathy is characterized by subacute, progressive impairment of all sensory modalities and is associated with severe sensory ataxia and areflexia.12 Subacute sensory neuropathy may precede the diagnosis of tumor by months or even years. At onset, patients may have shooting

CHAPTER 420  Peripheral Neuropathies  

pain and burning sensations. Other symptoms include numbness, tingling, and a progressive sensory loss that may be asymmetrical. Symptoms usually progress rapidly to involve all four limbs, the trunk, and face. Findings may then stabilize, although by this time the patient is often totally disabled. Occasional patients have an indolent course. Neurologic examination reveals loss of deep tendon reflexes and involvement of all modalities of sensation; large fiber modalities such as vibration and joint position sense are most severely affected. The loss of position sense may lead to severe sensory ataxia with pseudoathetoid movements of the hands and an inability to walk despite normal strength. Cranial nerve involvement includes sensorineural deafness, loss of taste, and facial numbness. The asymmetrical pattern of symptoms sometimes suggests a radiculopathy or plexopathy. A paraneoplastic encephalomyelitis characterized by patchy, multifocal neuronal loss in regions of the cerebral hemispheres, the limbic system, the cerebellum, the brain stem, the spinal cord, and autonomic ganglia often develops in patients with subacute sensory neuropathy. Autonomic symptoms include impotence, dry mouth, and constipation.

DIAGNOSIS

The diagnosis is based on recognizing the typical neuropathy in the setting of malignancy.13 The results of routine laboratory studies are generally normal. The diagnosis is supported by finding serum polyclonal IgG anti-Hu antibodies, also called antineuronal antibodies type 1, or by indirect immunofluorescence or immunohistochemistry and confirmed by Western blot analysis. Subacute painful, asymmetrical neuropathy or neuronopathy in an elderly patient should prompt a search for carcinoma of the lung because small cell lung cancer (Chapter 191) accounts for more than 80% of the associated tumors. Subacute sensory neuropathy has also been reported in patients with adenocarcinoma of the lung, breast, ovary, stomach, colon, rectum, and prostate, as well as Hodgkin and non-Hodgkin lymphoma. In patients with no evidence of cancer, detection of anti-Hu antibodies should prompt a computed tomography study of the chest with special attention to the mediastinal lymph nodes. The use of whole body positron emission tomography with fluorodeoxyglucose has been advocated for early diagnosis in patients with anti-Hu antibodies or clinical suspicion of subacute sensory neuropathy because it may reveal neoplastic adenopathy months before computed tomography or magnetic resonance imaging.

TREATMENT Subacute sensory neuropathy responds poorly to plasma exchange, IVIG, or immunosuppressant medications, even when such treatment is started early in the course of the disease. Successful treatment of the tumor rarely induces remission of subacute sensory neuropathy but may stabilize symptoms.

Other Neuropathies Possibly Associated with Cancer

SENSORIMOTOR NEUROPATHY

Sensorimotor neuropathy occurs in approximately 25% of patients with all types of tumors. The neuropathy can have an acute or subacute onset, with a

2533

progressive or relapsing-remitting course. Because no antineuronal antibody has been specifically associated with these neuropathies, their paraneoplastic nature is not established. Severe or relapsing neuropathies often precede the diagnosis of cancer, but the search for malignancy is generally limited to a chest radiograph, stool samples for blood, and routine blood tests. There are no specific treatments for these neuropathies, and their progression does not necessarily correlate with that of the malignancy.

PARANEOPLASTIC VASCULITIS OF NERVES

A nonsystemic vasculitic neuropathy, which may also involve muscle, occurs with various types of tumor, including small cell lung cancer, lymphoma, and carcinoma of the kidney, stomach, and prostate. Neurologic symptoms may develop either before or after the tumor is diagnosed. The neuropathy is subacute and progressive and usually affects older men. Like many paraneoplastic disorders, these neuropathies often respond poorly to treatment.

  VASCULITIC NEUROPATHIES DEFINITION

Vasculitic neuropathies (Table 420-4) typically present as painful acute or semiacute axonal mononeuritis multiplex. There is acute motor and sensory loss in multiple nerve territories. The number of nerves involved may be extensive enough to make the distinction between a multifocal and diffuse neuropathy difficult. Occasionally, vasculitic neuropathy can present as sensory neuropathy, trigeminal neuropathy, compressive neuropathy, or autonomic neuropathy. Neuropathy can occur in systemic vasculitis associated with other organ systems, as well as in nonsystemic vasculitis affecting just nerve and muscle.

EPIDEMIOLOGY

Systemic vasculitic neuropathy is more common than nonsystemic vasculitic neuropathy. Peak ages at onset of both are the fifth to eighth decades, but vasculitis can occur at any age. Neuropathy, particularly mononeuritis multiplex, is common in several forms of systemic vasculitis. Rheumatoid arthritis (Chapter 264) evolves into systemic rheumatoid vasculitis in 5 to 15% of patients, and vasculitic neuropathy will develop in about 50% of these patients. More than 50% of patients with Churg-Strauss syndrome (Chapter 270), 40 to 50% with granulomatosis with polyangiitis (Chapter 270), 35 to 75% with polyarteritis nodosa (Chapter 270), and a majority of patients with mixed cryoglobulinemia (Chapter 187) have neuropathy. Patients with Sjögren syndrome (Chapter 268) are often initially found to have sensory neuropathies. Neuropathies are uncommon in systemic lupus erythematosus.

PATHOBIOLOGY

In patients with mononeuritis multiplex, axonal degeneration develops as a result of nerve ischemia caused by the vasculitic process. Immune-mediated inflammation and necrosis of blood vessel walls occlude the vessel’s lumen, thereby resulting in ischemic damage. Small arteries or arterioles (50 to 300 µm) are most commonly affected, particularly those that occur in watershed areas between the distribution of the major nutrient arteries of proximal nerves. True nerve infarcts are rare.

TABLE 420-4  SYSTEMIC VASCULITIS AND NEUROPATHY TYPE

SEROLOGY FEATURES

ASSOCIATED FEATURES

USUAL NEUROPATHY TYPE

NEUROPATHY PREVALENCE

Rheumatoid arthritis

RF 80–90%

Arthralgias, arthritis frequent; multiple organs

Mononeuritis multiplex and sensorimotor neuropathy

50% of patients with vasculitis

Churg-Strauss

c-ANCA G). It presents with a proximal myopathy associated with slowly progressive ataxia, epilepsy, peripheral neuropathy, and myoclonus. Leber hereditary optic neuropathy (Chapter 424) predominantly affects young adult men, more than 95% of whom have mtDNA point mutations in m.3460G>A, m.11778G>A, or m.14484T>C. Patients develop subacute bilateral visual failure in both eyes within 2 to 3 months. Chronic progressive external ophthalmoplegia with ptosis and gradual limitation of eye movements is seen in up to 20% of mitochondrial disorders. About 95% of patients have sporadic mtDNA point mutations or deletions, but the disease can be inherited as either an autosomal dominant or recessive trait. Mutations in POLG gene, which encodes the mitochondrial polymerase γ, are the most common causes of autosomal dominant or recessive progressive external ophthalmoplegia. Kearns-Sayre syndrome is characterized by the triad of external ophthalmoplegia, retinitis pigmentosa, and onset before the age of 20 years plus at least one of the following: heart block, cerebellar ataxia, or cerebrospinal fluid protein greater than 100 mg/dL. Kearns-Sayre syndrome is usually sporadic and caused by a single deletion of mtDNA. Mitochondrial DNA depletion syndromes can present in neonatal period or infancy with subacute necrotizing encephalomyopathy (Leigh syndrome), hepatorenal failure, cardiomyopathy, and severe lactic acidosis. Children with Pearson syndrome, which is caused by accumulation of mtDNA deletions, typically present with pancytopenia, sideroblastic anemia, and exocrine pancreatic failure. Primary coenzyme Q10 (ubiquinone) deficiency is a rare autosomal recessive disorder that can present with encephalopathy, lipid storage myopathy, myoglobinuria, seizures, and cerebellar ataxia, or as an isolated nephrotic syndrome or an isolated myopathy.

DIAGNOSIS, TREATMENT, AND PROGNOSIS

The investigation of suspected mitochondrial disorders involves a systematic screen for multisystem complications, especially diabetes and cardiomyopathy; muscle biopsy to look for ragged red fibers, cytochrome c oxidase deficiency or biochemical evidence of respiratory chain dysfunction; search for mitochondrial deletion or depletion in muscle; and molecular genetic tests. Some primary mtDNA defects are not detectable in blood, so skeletal muscle is often required for the biochemical and genetic tests. For example, diagnosis of primary coenzyme Q10 (CoQ10) deficiency is made by measuring CoQ10 in muscle but not in blood. Patients with primary CoQ10 deficiency can respond dramatically to CoQ10 supplementation (30 mg/kg/day in children and up to 2400 mg/day in adults in three divided doses). Vitamins and cofactors, including thiamine, riboflavin, and CoQ10, have shown varying degrees of benefit in different mitochondrial diseases. Management is largely supportive with monitoring and treatment of complications. Prognosis varies depending on the phenotype, ranging from the relatively normal life expectancy with chronic external ophthalmoplegia to a relatively rapid demise with Leigh syndrome.

OTHER METABOLIC AND TOXIC MYOPATHIES

Myopathy can complicate many metabolic disorders, including hypothyroidism (Chapter 226), Addison disease (Chapter 227), hyperaldosteronism

TABLE 421-10  TOXIC MYOPATHIES INFLAMMATORY Cimetidine d-Penicillamine Procainamide l-Tryptophan l-Dopa NONINFLAMMATORY NECROTIZING OR VACUOLAR Statins Chloroquine Colchicine Emetine ε-Aminocaproic acid Labetalol Cyclosporine Tacrolimus Isoretinoic acid (vitamin A analogue) Vincristine Alcohol RHABDOMYOLYSIS AND MYOGLOBINURIA Statins Alcohol Heroin Amphetamine Toluene Cocaine ε-Aminocaproic acid Pentazocine Phencyclidine MALIGNANT HYPERTHERMIA Halothane Ethylene Diethyl ether Methoxyflurane Ethyl chloride Trichloroethylene Gallamine Succinylcholine MITOCHONDRIAL Zidovudine MYOTONIA 2,4-d-Chlorophenoxylacetic acid Anthracene-9-carboxylic acid Cholesterol-lowering agents Chloroquine Cyclosporine MYOSIN LOSS Nondepolarizing neuromuscular blocking agents* Intravenous glucocorticosteroids* *In the setting of critical illness. Adapted with revisions from Goldman L, Ausiello DA, eds. Cecil Textbook of Medicine, 23rd ed. Philadelphia: Elsevier; 2008.

(Chapter 227), hyperparathyroidism (Chapter 245), vitamin D deficiency (Chapter 244), and liver and renal failure (Chapters 130 and 153). The myopathy is often subtle, the CK level and EMG are often normal, and the muscle biopsy may be nonspecifically abnormal. Many drugs cause myopathy (Table 421-10) with proximal muscle weakness, muscle pain, and exercise intolerance. The CK and EMG can be normal, and muscle biopsy findings may be nonspecific. The diagnosis may depend on the resolution of symptoms after the toxic agent is removed. Perhaps the most commonly incriminated medications are statins, which can cause muscle pain, an increased CK level, and rarely, myoglobinuria.

  INFLAMMATORY MUSCLE DISEASES

Inflammatory myopathies are a heterogeneous group of acquired muscle diseases (Table 421-11) that usually present with muscle weakness and exercise intolerance, with or without pain. Most patients have an elevated CK level and an abnormal EMG. Muscle biopsy shows an inflammatory infiltrate. However, the inflammatory process can be patchy and missed on the EMG

2547

CHAPTER 422  Disorders of Neuromuscular Transmission  

TABLE 421-11  CLASSIFICATION OF INFLAMMATORY MYOPATHIES IDIOPATHIC Polymyositis Dermatomyositis Inclusion body myositis Overlap syndromes with other connective tissue disease (scleroderma, systemic lupus erythematosus, mixed connective tissue disease, Sjögren syndrome, rheumatoid arthritis, polyarteritis nodosa) Sarcoidosis and other granulomatous myositis Behçet disease Inflammatory myopathies and eosinophilia   Eosinophilic polymyositis   Diffuse fasciitis with eosinophilia Focal myositis Myositis ossificans INFECTIOUS Bacterial: Staphylococcus aureus, streptococci, Escherichia coli, Yersinia sp., Legionella sp., gas gangrene (Clostridium welchii), leprous myositis, Lyme disease (Borrelia burgdorferi) Viral: acute myositis after influenza or other viral infections (adenovirus, coxsackievirus, echovirus, parainfluenza virus, Epstein-Barr virus, arbovirus, cytomegalovirus), retrovirus-related myopathies (HIV, HTLV-1), hepatitis B and C Parasitic: trichinosis (Trichinella spiralis), toxoplasmosis (Toxoplasma gondii), cysticercosis, sarcosporidiosis, trypanosomiasis (Taenia solium) Fungal: Candida sp., Cryptococcus sp., sporotrichosis, actinomycosis, histoplasmosis HIV = human immunodeficiency virus; HTML-1 = human T-lymphotrophic virus 1. From Goldman L, Ausiello DA, eds. Cecil Textbook of Medicine. 23rd ed. Philadelphia: Elsevier; 2008.

or muscle biopsy, especially if the specimen is small or if a clinically unaffected muscle is biopsied. Similarly, a short period of corticosteroid therapy can mask the findings. MRI guidance can help to identify high-yield locations for muscle biopsy. Systemic diseases associated with an inflammatory myopathy include polymyositis, dermatomyositis, inclusion body myositis (Chapter 269), systemic lupus erythematosus (Chapter 266), mixed connective tissue disease (Chapter 77), Sjögren syndrome (Chapter 268), rheumatoid arthritis (Chapter 264), and sarcoidosis (Chapter 95). Systemic viral illnesses and other infectious microorganisms (see Table 421-11) frequently cause muscle pain and an elevated CK, which rarely are major clinical problems.

Sarcopenia and Muscle Wasting Muscle wasting is a common problem in elderly people (Chapter 25), partly related to hormonal changes and largely related to underuse. Critically ill patients rapidly lose muscle owing to inactivity and reduced protein synthesis,19,20 with some potential to slow this process with protein-calorie nutrition (Chapter 111). Sarcopenia is also a prominent feature of many cancers, endstage heart failure (Chapter 58) and renal failure (Chapter 131), and eating disorders (Chapter 219).

Grade A References A1. Manzur AY, Kuntzer T, Pike M, et al. Glucocorticoid corticosteroids for Duchenne muscular dystrophy. Cochrane Database Syst Rev. 2008;1:CD003725. A2. Logigian EL, Martens WB, Moxley RT IV, et al. Mexiletine is an effective antimyotonia treatment in myotonic dystrophy type 1. Neurology. 2010;74:1441-1448. A3. Puymirat J, Bouchard JP, Mathieu J. Efficacy and tolerability of a 20-mg dose of methylphenidate for the treatment of daytime sleepiness in adult patients with myotonic dystrophy type 1: a 2-center, randomized, double-blind, placebo-controlled, 3-week crossover trial. Clin Ther. 2012;34: 1103-1111. A4. Russo V, Rago A, Politano L, et al. The effect of atrial preference pacing on paroxysmal atrial fibrillation incidence in myotonic dystrophy type 1 patients: a prospective, randomized, single-bind cross-over study. Europace. 2012;14:486-489. A5. Statland JM, Bundy BN, Wang Y, et al. Mexiletine for symptoms and signs of myotonia in nondystrophic myotonia: a randomized controlled trial. JAMA. 2012;308:1357-1365. A6. Sansone V, Meola G, Links TP, et al. Treatment for periodic paralysis. Cochrane Database Syst Rev. 2008;1:CD005045. A7. Matthews E, Portaro S, Ke Q, et al. Acetazolamide efficacy in hypokalemic periodic paralysis and the predictive role of genotype. Neurology. 2011;77:1960-1964. A8. Andersen ST, Haller RG, Vissing J. Effect of oral sucrose shortly before exercise on work capacity in McArdle disease. Arch Neurol. 2008;65:786-789. A9. van der Ploeg AT, Clemens PR, Corzo D, et al. A randomized study of alglucosidase alfa in late-onset Pompe’s disease. N Engl J Med. 2010;362:1396-1406.

GENERAL REFERENCES For the General References and other additional features, please visit Expert Consult at https://expertconsult.inkling.com.

422  DISORDERS OF NEUROMUSCULAR TRANSMISSION AMELIA EVOLI AND ANGELA VINCENT

DEFINITION

Neuromuscular transmission depends on the release of acetylcholine from synaptic vesicles that are stored in the terminal boutons of the motor nerve axon (Fig. 422-1). Invasion of the motor nerve terminal by the action potential opens voltage-gated calcium channels, resulting in the Ca2+-dependent release of the vesicular contents into the synaptic space. Acetylcholine binds to the acetylcholine-gated ion channels (acetylcholine receptors [AChRs]) on the postsynaptic membrane, thereby leading to the opening of these channels and a local depolarization, the end- plate potential. If the end-plate potential exceeds the critical firing threshold, voltage-gated sodium channels (sited at the bottom of the postsynaptic folds) open to generate the muscle action potential that propagates along the muscle fiber and activates muscle contraction. The action of acetylcholine is terminated by its dissociation from the AChRs, which close spontaneously after 1 to 4 milliseconds; hydrolysis of acetylcholine by acetylcholinesterase; and acetylcholine diffusion from the synaptic cleft. Meanwhile, in the motor nerve terminal, the voltage-gated calcium channels close spontaneously, and the resting membrane potential is restored through the transient opening of voltage-gated potassium channels. The extent to which the amplitude of the end-plate potential exceeds the threshold for activation of the voltage-gated sodium channels is called the safety factor. In healthy individuals, the amplitude decreases during repeated activity but does not fall below this threshold; thus, neuromuscular transmission is not compromised. However, if there is an abnormally low end-plate potential amplitude, failure of neuromuscular transmission may occur. Causes include defects in the release of acetylcholine, the postsynaptic response to acetylcholine, or the number or sensitivity of the voltage-gated sodium channels. Morphologic changes to the presynaptic or postsynaptic components or to the basal lamina between them may also influence the efficacy of transmission. Although myasthenia gravis and some neurotoxic envenomations (Chapter 112) are the most common disorders of neuromuscular transmission, a number of conditions have been implicated (Table 422-1).

  AUTOIMMUNE DISEASES

Myasthenia Gravis

EPIDEMIOLOGY

Myasthenia is the most common disorder of neuromuscular transmission, with a prevalence of about 15 per 100,000 in Western countries.1 All races can be affected, and it can occur at any age from year 1 onward. There is a small peak in the incidence rate in women in the third decade and a larger peak, the majority males, at later ages. The annual incidence rises to about 5 per 100,000 after age 70 years. It is especially important to differentiate myasthenia gravis from other causes of limb or bulbar muscle weakness in elderly people. Myasthenia gravis itself is heterogeneous and can be divided into different subtypes; the relative frequency of these different forms is not known, but relatively mild childhood forms are frequent in Asian countries. Neonatal myasthenia gravis, due to the placental transfer of maternal antibodies to the AChR or to muscle-specific kinase (MuSK), affects up to one in eight babies born to mothers with myasthenia gravis. Autoimmune myasthenia gravis must be distinguished from congenital myasthenic syndromes, which are caused by gene mutations.

PATHOBIOLOGY

Pathophysiology

Myasthenia gravis is the result of a defect in neuromuscular transmission. The postsynaptic response to acetylcholine, the end-plate potential, is reduced so

CHAPTER 421  Muscle Diseases  

GENERAL REFERENCES 1. Mercuri E, Muntoni F. Muscular dystrophies. Lancet. 2013;381:845-860. 2. Goemans N, Buyse G. Current treatment and management of dystrophinopathies. Curr Treat Options Neurol. 2014;16:287. 3. Jarmin S, Kymalainen H, Popplewell L, et al. New developments in the use of gene therapy to treat Duchenne muscular dystrophy. Expert Opin Biol Ther. 2014;14:209-230. 4. Statland JM, Tawil R. Facioscapulohumeral muscular dystrophy: molecular pathological advances and future directions. Curr Opin Neurol. 2011;24:423-428. 5. Lemmers RJ, Tawil R, Petek LM, et al. Digenic inheritance of an SMCHD1 mutation and an FSHDpermissive D4Z4 allele causes facioscapulohumeral muscular dystrophy type 2. Nat Genet. 2012;44:1370-1374. 6. Hilbert JE, Ashizawa T, Day JW, et al. Diagnostic odyssey of patients with myotonic dystrophy. J Neurol. 2013;260:2497-2504. 7. Laberge L, Gagnon C, Dauvilliers Y. Daytime sleepiness and myotonic dystrophy. Curr Neurol Neurosci Rep. 2013;13:340. 8. Wicklund MP. The muscular dystrophies. Continuum (Minneap Minn). 2013;19:1535-1570. 9. Narayanaswami P, Weiss M, Selcen D, et al. Evidence-based guideline summary: diagnosis and treatment of limb-girdle and distal dystrophies: report of the guideline development subcommittee of the American Academy of Neurology and the practice issues review panel of the American Association of Neuromuscular & Electrodiagnostic Medicine. Neurology. 2014;83:1453-1463.

2547.e1

10. Selcen D. Myofibrillar myopathies. Neuromuscul Disord. 2011;21:161-171. 11. Puckelwartz M, McNally EM. Emery-Dreifuss muscular dystrophy. Handb Clin Neurol. 2011; 101:155-166. 12. Mercuri E, Muntoni F. The ever-expanding spectrum of congenital muscular dystrophies. Ann Neurol. 2012;72:9-17. 13. Bönnemann CG, Wang CH, Quijano-Roy S, et al. Diagnostic approach to the congenital muscular dystrophies. Neuromuscul Disord. 2014;24:289-311. 14. Colombo I, Scoto M, Manzur AY, et al. Congenital myopathies: natural history of a large pediatric cohort. Neurology. 2015;84:28-35. 15. Wang CH, Dowling JJ, North K, et al. Consensus statement on standard of care for congenital myopathies. J Child Neurol. 2012;27:363-382. 16. Horga A, Raja Rayan DL, Matthews E, et al. Prevalence study of genetically defined skeletal muscle channelopathies in England. Neurology. 2013;80:1472-1475. 17. Schapira AH. Mitochondrial diseases. Lancet. 2012;379:1825-1834. 18. Archer SL. Mitochondrial dynamics: mitochondrial fission and fusion in human diseases. N Engl J Med. 2013;369:2236-2251. 19. Puthucheary ZA, Rawal J, McPhail M, et al. Acute skeletal muscle wasting in critical illness. JAMA. 2013;310:1591-1600. 20. Fan E, Cheek F, Chlan L, et al. An official American Thoracic Society Clinical Practice guideline: the diagnosis of intensive care unit-acquired weakness in adults. Am J Respir Crit Care Med. 2014;190:1437-1446.

2548

CHAPTER 422  Disorders of Neuromuscular Transmission  

Fetal AChR γ

α

α δ

The agrin-Lrp4-MuSK-Dok7 pathway

Adult AChR ε

α

β

α β

δ

ACh/α-BuTx binding site

Voltage-gated sodium channel

Main targets for genetic defects

Known targets for autoantibodies Acetylcholine receptor (AChR) Muscle-specific kinase (MuSK) Lipoprotein-related receptor protein 4 (Lrp4) Voltage-gated calcium channel (VGCC) Voltage-gated potassium channel (VGKC)

Acetylcholine receptor (AChR) Receptor aggregating protein at the synapse (Rapsyn) Choline acetyltransferase (CHAT) Acetylcholinesterase (AChE) Dok-7

FIGURE 422-1.  Diagrammatic representation of the neuromuscular junction, indicating the ion channels, receptors, enzymes, and associated proteins that are the most frequent targets for autoimmune diseases (left) or mutations in genetic diseases (right). The acetylcholine receptor exists in fetal and adult isoforms as illustrated at top left. The replacement of the fetal form by the adult form takes place toward the end of gestation in humans. The agrin-Lrp4-MuSK-DOK7 pathway associated with AChRs on the postsynaptic membrane is illustrated at top right. α-BuTx, α-bungarotoxin, the snake toxin that binds with high specificity to the two Ach binding sites on the AChRs.

TABLE 422-1  DISORDERS OF NEUROMUSCULAR TRANSMISSION DISEASE AUTOIMMUNE Myasthenia gravis

TARGET

Transient neonatal myasthenia

AChRs MuSK Lrp4 AChRs, MuSK

Arthrogryposis

Fetal AChR

Lambert-Eaton myasthenic syndrome Acquired neuromyotonia

VGCCs VGKCs

GENETIC Acetylcholine receptor deficiency Acetylcholine receptor deficiency

AChR

AChR kinetic abnormalities

AChR

Choline acetyltransferase deficiency Acetylcholine esterase deficiency

Choline acetyltransferase AChE

Arthrogryposis, multiple pterygium, Escobar’s syndrome

Can occur with rapsyn, δ- or γ-subunit AChR mutations

NEUROTOXIC Botulism

Presynaptic ACh release

Envenomation following bites from snakes, spiders, scorpions, etc. Drugs and insecticides

Varied sites of action Varied sites of action

PATHOBIOLOGY Antibodies to AChR in 85% reduce AChR numbers and EPP amplitude Antibodies to MuSK in 5-10%; mechanism not clear Antibodies to Lrp4 in a proportion of patients; mechanism not clear Maternal antibodies cause transient disease in neonate; not seen commonly if mother receiving treatment Maternal antibodies that inhibit fetal AChR function resulting in paralyses in the fetus in utero, leading to joint contractures and rarely arthrogryposis Antibodies to VGCC in 90% reduce VGCC numbers and ACh release and EPP amplitude Antibodies to VGKC-complex in 40% lead to increased and spontaneous ACh release Recessive mutations in AChR-subunit genes cause reduced AChR expression Recessive mutations in RAPSYN cause reduced anchoring of AChR on the postsynaptic membrane, or in DOK-7 cause a synaptopathy Dominant or recessive mutations in AChR-subunit genes cause kinetic defects—“slow” and “fast” channel syndromes Recessive mutations in the gene for choline acetyltransferase (CHAT) cause reduced ACh release Recessive mutations in the collagen tail (COLQ) that anchors AChE at the neuromuscular junction cause absence of AChE Fetal akinesia

Botulinum toxin gains entry into the presynaptic motor nerve and cleaves proteins involved in ACh release mechanism Neurotoxins specific for VGCCs, VGKCs, AChE, AChRs, voltage-gated sodium channels, and other targets are frequent in many animal venoms and generally inhibit function Muscle relaxants and other drugs Many antibiotics and quinine-related drugs can alter neuromuscular transmission at high dose Organophosphates block AChE and have complicated acute and chronic actions

AChE = acetylcholinesterase; AChR = acetylcholine receptor; EPP = end-plate potential; MuSK = muscle-specific kinase; Lrp4 = low-density lipoprotein-related receptor protein 4; VGCC = voltage-gated calcium channel; VGKC-complex = voltage-gated potassium channel and associated proteins.

that the threshold for activation of the muscle action potential is not reached. At a severely affected end plate, this deficiency can occur at the initiation of contraction, but it is most common during repetitive activity when the endplate potential naturally declines, despite a compensatory rise in the release of acetylcholine. This phenomenon, occurring across many end plates within a muscle, is responsible for the decrement in the amplitude of the compound

muscle action potential on repetitive nerve stimulation, a finding that is diagnostic of a disorder of neuromuscular transmission. In myasthenia gravis, the reduced end-plate potentials result from loss of functional AChRs on the postsynaptic membrane and also from simplification of the postsynaptic folds, which contain the voltage-gated sodium channels. In most patients, these changes are caused by antibodies against the

CHAPTER 422  Disorders of Neuromuscular Transmission  

Direct block of function preventing ACh binding and ion channel opening

A

Cross-linking of AChRs by divalent antibodies leading to increased internalization and degradation

Complement-mediated lysis of the postsynaptic membrane leading to morphologic damage and loss of AChRs

B

C

2549

FIGURE 422-2.  Mechanisms of loss of the acetylcholine receptor (AChR) at the neuromuscular junction. Antibodies can act (A) by directly blocking ACh binding or ion channel function; (B) by cross-linking the AChRs in the membrane, thereby leading to increased internalization and degradation; or (C) by complement-dependent lysis of the AChR-containing postsynaptic membrane. In myasthenia gravis, complement-dependent lysis is likely to be the most important mechanism overall. Interestingly, there is no evidence of complementdependent mechanisms in either Lambert-Eaton myasthenic syndrome or acquired neuromyotonia, in which cross-linking of the respective ion channels with increased internalization seems to be the main mechanism.

AChRs. The pathophysiology in patients with antibodies to other postsynaptic proteins, such as MuSK and the low-density lipoprotein-related receptor protein 4 (Lrp4), involves different mechanisms that are not yet fully understood. Like most synapses, the neuromuscular junction is highly regulated. If the nerve is cut, leading to loss of neuromuscular transmission, the muscle responds by upregulating the expression of AChRs that revert to a fetal phenotype (see Fig. 422-1). Alternatively, if the activity of the postsynaptic muscle decreases, the motor nerve attempts to compensate. Consequently, the synthesis of AChRs in the muscle fiber and the release of acetylcholine from the motor nerve are increased in myasthenia gravis.

Pathogenesis

Myasthenia gravis is an antibody-mediated disease that is associated with other autoimmune disorders, especially thyroid disease (Chapter 226). Younger AChR antibody-positive patients have an increased prevalence of the human leukocyte antigen (HLA)-B8 and -DR3 haplotypes that are also frequently associated with autoimmunity. AChR antibodies are immunoglobulin G (IgG), have high affinity, are highly specific for the native human AChR, and act by three main mechanisms (Fig. 422-2). First, a few antibodies directly inhibit the binding of acetylcholine to the AChR, thereby causing a pharmacologic-like blockade of function.2 Second, because of their divalence, antibodies can bind simultaneously to two adjacent AChRs, through the α-subunits that are present in duplicate in each receptor, to form AChRantibody complexes that are internalized and degraded by the muscle fiber, thereby leading to loss of AChRs. Third, most of the antibodies are IgG1 subclass, which binds and activates complement. The result is activation of the membrane attack complex with destruction of the postsynaptic membrane and morphologic damage. All these effects are strictly limited to the neuromuscular junction; the remainder of the muscle fiber is essentially normal. Specific antibody production requires helper T cells that can recognize AChR epitopes. The thymus gland, which is often abnormal in myasthenia, is thought to play a role in the immune response against AChR.3 In patients with early-onset disease, the thymus is often the site of follicular hyperplasia, with T- and B-cell lymphocytic infiltrates in an expanded medulla. These infiltrates, which are very similar to the germinal centers found in lymph nodes, contain B cells that express surface immunoglobulin specific for AChRs and plasma cells that synthesize AChR antibodies. In the thymic medulla, muscle-like “myoid” cells have AChRs on their surface in both normal and myasthenic individuals; these cells may be an early target of complement and antibodies, thereby providing the antigenic stimulus responsible for chronic germinal center formation and AChR antibody production. In late-onset myasthenia gravis and in patients with MuSK antibodies, the thymus is mostly normal for age. However, some patients without conventional AChR or MuSK antibodies have typical thymic hyperplasia and antibodies that bind to tightly clustered AChR on transfected cells. Thymomas, which are epithelial cell tumors, occur in 10 to 15% of myasthenic patients and nearly always are associated with AChR antibodies. Thymomas associated with myasthenia gravis correspond mainly to the World Health Organization types B1 and B2 and are characterized by active thymopoiesis (i.e., the capacity to promote T-cell maturation and export). Thymoma epithelial cells express muscle antigens and AChR subunits, and they are

thought to be responsible for defective negative selection, with the export to the periphery of autoreactive T lymphocytes. Rarely, myasthenia gravis arises after removal of a thymoma. About 40% of AChR antibody-negative patients have antibodies to MuSK, a muscle tyrosine kinase that is specifically expressed at the neuromuscular junction and plays a crucial role in the formation and maintenance of the postsynaptic membrane.4 MuSK is activated by nerve-secreted agrin, through its coreceptor Lrp4; MuSK phosphorylation and dimerization induce an intracellular signaling cascade that leads to AChR clustering. In animal models, MuSK antibodies cause loss of AChR and reduced postsynaptic folds, as well as a lack of the normal compensatory presynaptic increase in the release of acetylcholine. However, MuSK antibodies are predominantly IgG4 and do not activate complement, so it is not yet clear how these antibodies cause the neuromuscular pathophysiology. A small number of AChR and MuSK antibody-negative patients have serum antibodies to Lrp4.5 Lrp4 antibodies interfere with Lrp4-agrin binding and reduce AChR expression in vitro, and their pathogenicity has been recently demonstrated in an animal model.

CLINICAL MANIFESTATIONS

Myasthenia gravis presents clinically with painless muscle weakness that increases with muscle use and improves after rest. In many patients, the weakness starts in the eye muscles, where it results in double vision and ptosis (drooping eyelids). In others, it may first affect bulbar muscles or limb muscles (Fig. 422-3). Virtually any skeletal muscle may be involved as the illness progresses. Typically, the weakness varies in distribution and severity from day to day or from week to week, and it is often worse in the evening. It may first appear following an infection. Established weakness can increase with anxiety, with infection, or with the menstrual period. Ptosis, which is often asymmetrical, and diplopia initially can be transient and first noticed while driving, for example. Severity can range from mild unilateral ptosis or minimal diplopia to profound bilateral ptosis combined with almost complete ophthalmoplegia. Bulbar symptoms include weakness of facial muscles with difficulties in closing eyes and a “snarling” smile, difficulty in chewing, nasal or slurred speech that can noticeably deteriorate as speech continues, impaired swallowing sometimes associated with nasal regurgitation of fluids, reduced tongue movements, and head droop related to neck weakness. Limb muscle involvement is common, and proximal muscles are usually more involved than distal. Weakness of the legs can lead to collapse when walking and can be misinterpreted as a functional (psychogenic) disorder. Weakness of elbow extension and of finger abduction may be prominent. By contrast, ankle dorsiflexion is rarely affected except in severe disease. Respiratory dysfunction is less common but can be life-threatening, especially if associated with dysphagia. Selective involvement of the diaphragm can cause severe breathlessness in the supine posture. Wasting is uncommon but can affect the facial muscles and tongue, for example, in long-standing disease. Tendon reflexes are typically brisk. Bladder disturbances are rare, and sensory symptoms do not occur.

Subtypes of Myasthenia Gravis

Several subgroups can be distinguished on the basis of clinical and pathologic criteria and can help to inform treatment.

2550

CHAPTER 422  Disorders of Neuromuscular Transmission  

Ocular Myasthenia Gravis

Ocular myasthenia gravis is confined to extraocular muscles; if it remains localized for at least 2 years, subsequent generalization is unlikely. AChR antibody levels are generally low and are undetectable in about 50% of patients. This subgroup rarely is associated with a thymoma. The neuromuscular junction of ocular muscles shows structural and physiologic differences from limb muscles. Ocular weakness is often the presenting symptom not only in myasthenia gravis but also in neurotoxin poisoning, for example, botulism (Chapter 296). Thus, physiologic factors or accessibility of the neuromuscular junctions of ocular muscles to circulating factors may make them particularly vulnerable to antibodies in myasthenia gravis.

Generalized Myasthenia Gravis with Acetylcholine Receptor Antibodies

Among patients with generalized disease and AChR antibodies, there are three clinical subgroups. Early-onset myasthenia gravis is more frequent in females and associates strongly with HLA-A1, -B8, and -DR3. The thymus is generally hyperplastic. AChR antibody titers are usually high and decline to varying degrees after successful treatments, including thymectomy. Late-onset myasthenia gravis is becoming increasingly common with the aging of the population and, when associated with bulbar weakness, may be mistaken for amyotrophic lateral sclerosis (Chapter 419) or brain stem cerebrovascular disease. Among older patients, males are more frequently affected. Thymoma-associated myasthenia gravis is an important distinction because thymectomy or other specific tumor therapy is required. Most patients with thymomas and myasthenia gravis present between the ages of 30 and 60 years.

Myasthenia Gravis with Muscle-Specific Kinase Antibodies

About 15% of myasthenic patients with generalized symptoms do not have detectable AChR antibodies. Up to 40% of these patients have antibodies to MuSK.6 MuSK antibodies are absent or very infrequent in patients with AChR antibodies, patients with persistent ocular symptoms, and patients with thymoma. Compared with typical myasthenia gravis, MuSK antibodypositive disease is characterized by high prevalence in younger females; predominant bulbar, neck, and respiratory muscle weakness; and an increased rate of facial and tongue muscle atrophy.

Myasthenia Gravis with Neither Acetylcholine Receptor nor Muscle-Specific Kinase Antibodies

Some patients with thymus hyperplasia and good response to treatment, including thymectomy, may have antibodies that bind only to clustered AChRs on AChR-expressing cells. A variable proportion of AChR and MuSK antibody-negative patients have Lrp4 antibodies. These antibodies have not been found associated with thymoma.

DIAGNOSIS

FIGURE 422-3.  Marked ocular and facial muscle weakness in a young female with myasthenia gravis.

Diagnosis is based on the clinical features, serologic testing for specific antibodies, electromyography (EMG), and, if doubt still remains or specialized facilities are not available, the clinical response to anticholinesterase medication (Table 422-2).7 Mediastinal imaging is needed to exclude a thymoma especially in patients with AChR antibodies.

TABLE 422-2  DIAGNOSTIC EVALUATION (EXCLUDES NEUROMYOTONIA) AChR MG

MuSK MG

Lrp4-MG SN-MG

NEONATAL MG

BoTx

MM

Onset birth, recovery of muscle strength within 2 mo







+



LEMS

AChR γ-subunit mutations, variable severity

CMS





Onset birth plus arthrogryposis







+



Rapsyn or AChR δ-subunit mutations





Onset at 10%

+

+/−

+/−

+

+

+

+/−



EMG jitter increased

+

+ Especially face muscles

+

+

+

+

+

+/−

Post-tetanic potentiation









+



+



AChE inhibitor response Thymoma

+ +/−

Poor or deterioration

+

+

Often weak

Except SCS, COLQ, or DOK7 mutations











+/−







AChE = acetylcholinesterase; AChR = acetylcholine receptor; BoTx = botulism; ChAT = choline acetyltransferase; CMS = congenital myasthenic syndromes; Dok-7 = downstream of kinase 7; LEMS = Lambert-Eaton myasthenic syndrome; Lrp4 = low-density lipoprotein-related receptor protein 4; MG = myasthenia gravis; MM = mitochondrial myopathy; MuSK = muscle-specific kinase; SCS = slow-channel syndrome; SN = seronegative for AChR and MuSK antibodies.

CHAPTER 422  Disorders of Neuromuscular Transmission  

If AChR antibodies are absent, especially in patients with generalized symptoms, testing for MuSK antibodies is recommended. Both AChR and MuSK antibodies are very specific, and their detection in symptomatic patients confirms the diagnosis. It is not yet clear whether testing for the rarer Lrp4 or clustered AChR antibodies, which is available in specialist centers, will prove helpful in patients with inconclusive clinical and EMG findings. The electrophysiologic abnormality is an abnormally large decrement (>10%) in the amplitude of the compound muscle action potential on lowrate (3-Hz) repetitive nerve stimulation or increased jitter on single-fiber EMG. In patients with MuSK antibodies, EMG abnormalities may be detectable only in facial muscles. These EMG changes are not specific for myasthenia gravis but can occur in any disorder that interferes with neuromuscular transmission. Intravenous administration of edrophonium (Tensilon), a short-acting cholinesterase inhibitor, transiently improves myasthenic weakness but requires an appropriate medical setting, including resuscitative facilities and the availability of atropine, because of the risk for adverse events and severe cholinergic reactions, including syncope. A test dose of 2 mg is given intravenously, followed 30 seconds later by 6 to 8 mg if no adverse event has occurred. The equivalent doses in children are a 20-µg/kg test dose followed by 60 to 80 µg/kg. Some patients improve sufficiently with the test dose, so it is not necessary to give the full dose. An alternative pharmacologic test in adults is a single dose of subcutaneous or intramuscular neostigmine (1 to 2.5 mg) or of oral pyridostigmine (60 mg).

Differential Diagnosis

Congenital myasthenic syndromes (see later) should be considered in patients who have clinical and EMG evidence of myasthenia but are seronegative on antibody assays. Lambert-Eaton myasthenic syndrome almost always begins with difficulty in walking; ocular symptoms are rare, and specific laboratory tests are available (see later). The ocular muscle involvement that characterizes Miller-Fisher syndrome is more rapid in onset than is usual in myasthenia gravis and is associated with GQ1b antibodies (Chapter 420). Mitochondrial myopathy may show signs that are similar to those of myasthenia gravis (e.g., asymmetrical ptosis and limitation of eye movements), and there may be increased jitter on single-fiber EMG, but this condition and oculopharyngeal dystrophy can be distinguished from myasthenia gravis by the nonfluctuating weakness and by muscle biopsy (Chapter 421). In neurasthenia and chronic fatigue syndrome (Chapter 274), the laboratory tests for myasthenia gravis are negative.

TREATMENT  Most patients with AChR antibodies respond to oral pyridostigmine, 30 to 60 mg four or five times daily; in patients with mild disease, this dose may adequately control symptoms.8 Doses in excess of 90 mg are likely to cause gastrointestinal side effects, abdominal cramps and diarrhea, which can be controlled with oral propantheline bromide, 15 mg, or loperamide, 2 mg. Patients with MuSK antibodies generally have an unsatisfactory response.9 In some of these patients, pyridostigmine, even at low doses, can increase weakness and cause nicotinic side effects (muscle cramps and diffuse fasciculations).

Neonatal Myasthenia Gravis

Pyridostigmine, 3 to 5 mg, can be given every 4 hours to about an hour before a feeding. Close monitoring and respiratory support in a special unit may be required.

Ocular Myasthenia

Diplopia can sometimes be helped by the use of prisms. Ocular symptoms that respond incompletely to pyridostigmine are generally improved by lowdose prednisone therapy (e.g., 5 mg every other day), increasing by 5 mg at weekly intervals either until symptoms are completely controlled or until a ceiling dose (e.g., 1 mg/kg) is reached.10 When remission is established, the dose can be slowly reduced (e.g., by 5 mg at 2-weekly intervals) until symptoms recur and then adjusted upward to define the effective minimal dose. Full withdrawal of prednisone is usually followed by a symptomatic relapse. Thymectomy is not considered beneficial for nonthymomatous ocular myasthenia gravis. In patients who fail to respond adequately to prednisone or who are intolerant of the medication, the addition of azathioprine (2 to 2.5 mg/kg body weight) or ocular muscle surgery is an option. However, the diagnosis should be questioned in patients who show no improvement with high-dose prednisone treatment.

2551

Thymoma

Thymoma is usually an indication for surgery, but removal of the tumor seldom improves muscle weakness. If the tumor is locally invasive, postoperative radiotherapy is indicated. If tumor spread is more extensive, chemotherapy with cisplatin-containing regimens can be considered. Thymoma-associated myasthenia gravis is generally severe, and most patients need long-term treatment with steroids and immunosuppressants (see below).11

Generalized Nonthymomatous Myasthenia Gravis

When generalized symptoms are inadequately controlled by pyridostigmine, thymectomy is often recommended even for patients without a thymoma, especially patients younger than age 50 years. On the basis of uncontrolled studies, thymectomy in early-onset patients with AChR antibodies appears to be associated with an increased rate of remission, and a randomized trial evaluating the efficacy of thymectomy for nonthymomatous myasthenia gravis is in progress. By comparison, thymectomy is not beneficial in patients who have MuSK antibodies and in whom the thymus is generally devoid of hyperplastic changes. As a general rule, thymectomy, even in the presence of a thymoma, should never be an emergency treatment but rather should be postponed until a stable control of myasthenic symptoms is achieved. Immunosuppressive therapy with prednisone is usually administered in the initial phases of treatment owing to its short-latency effect. A1  Most patients respond to alternate-day prednisone, started at a low dose (e.g., 10 mg every other day) and increasing by 5 to 10 mg per dose to 1.0 to 1.5 mg/kg. Because starting prednisone can temporarily exacerbate the disease, patients are usually best managed in the hospital, especially if they have bulbar or respiratory muscle involvement. When remission is established, the dose can be reduced by 5 to 10 mg every 2 weeks (or more slowly) to the effective minimal dose. Prophylactic treatment for osteoporosis (Chapter 243) and careful follow-up for other side effects is mandatory in all patients. For long-term treatment, other immunosuppressive medication is required in patients who do not respond satisfactorily to prednisone or who need high maintenance doses. Because these agents have a long latency of effect, they are generally combined with prednisone (see earlier) during initial treatment and then used as monotherapy if steroids can be withdrawn or are contraindicated. Azathioprine (2.5 mg/kg/day) is the preferred treatment; compared with prednisone alone, combination treatment is better tolerated and associated with fewer relapses. A2  Cyclosporine (3 to 5 mg/kg daily) is effective as monotherapy or combined with steroids A3  and is frequently used as the second-choice immunosuppressant. Although the efficacy of mycophenolate mofetil in association with prednisone is questioned, this agent at the standard dose of 2000 mg/day is used in patients who are unresponsive to or intolerant of azathioprine. Tacrolimus is considered a third-line immunosuppressant. A4  Methotrexate (5 to 15 mg weekly) has similar efficacy and safety as azathioprine as a steroid-sparing agent. A5  When remission has been achieved, doses of these agents can be reduced slowly and cautiously; full withdrawal is likely to be followed by relapse. Immunoglobulin infusion and plasmapheresis are equally efficacious A6  for providing short-term improvement, typically persisting 4 to 6 weeks, and can be used in preparation for thymectomy, to cover the initiation of prednisone therapy, or to control disease exacerbations. An immunoglobulin infusion of 1 g/kg given on day 1 only is as effective as 1 g/kg given on day 1 and again on day 2. Because of the short-lived benefits of these therapies, they must be accompanied by additional immunosuppressive therapy (see earlier). Inhibition of the production of acetylcholinesterase using a short antisense oligonucleotide was both effective and safe in a phase Ib study on AChR antibody-positive patients (a phase II study is underway). High-dose cyclophosphamide A7  and two monoclonal antibodies, rituximab (which markedly reduces circulating B cells) and eculizumab (which binds C5 to preventing complement activation), A8  have been used successfully in patients with refractory disease.

PROGNOSIS

The increasing use of immunosuppressive therapies, coupled with advances in critical care, has greatly improved the prognosis of myasthenia gravis. Patients with myasthenic crisis are at high risk for recurrences,12 but many patients achieve optimal control of symptoms with a normal life expectancy. The prognosis is not as good, however, in patients with invasive thymoma, who have a 5-year survival rate of about 80%, or with invasive thymic carcinomas, who have a 5-year survival rate of only about 40%.

Lambert-Eaton Myasthenic Syndrome

DEFINITION AND EPIDEMIOLOGY

The Lambert-Eaton myasthenic syndrome, which is a rare disorder that affects all races, can occur in paraneoplastic and nonparaneoplastic forms. The incidence of the paraneoplastic form is higher, but its shorter survival

2552

CHAPTER 422  Disorders of Neuromuscular Transmission  

results in a similar prevalence of the two types. The associated tumor is usually a small cell lung cancer (about 2% of patients with small cell lung cancer develop Lambert-Eaton myasthenic syndrome), and more rarely a lymphoma. The nonparaneoplastic form associates with HLA-A1, -B8, and -DR3, as in early-onset myasthenia gravis.

PATHOBIOLOGY

Lambert-Eaton myasthenic syndrome is an antibody-mediated presynaptic disorder characterized by a reduced number of acetylcholine quanta (vesicles) released by each nerve impulse. End-plate potentials recorded from intercostal muscle biopsies are consequently much reduced in amplitude. During high-frequency repetitive nerve stimulation, the end-plate potential amplitude increases, probably because build-up of calcium in the motor nerve terminal leads to increased release of acetylcholine. Freeze-fracture electron microscopic studies of motor nerve terminals show that the “active zone” particles, which correspond to voltage-gated calcium channels, are reduced in number and disorganized. The antibodies in Lambert-Eaton myasthenic syndrome bind to the presynaptic nerve terminal at the sites of acetylcholine release and appear to act principally by cross-linking the voltage-gated calcium channels, thereby leading to their clustering and internalization. The antibodies also interfere with transmitter release from postganglionic parasympathetic and sympathetic neurons in injected mice, providing an explanation for the autonomic dysfunction observed in many patients.

CLINICAL MANIFESTATIONS

Almost all patients present with difficulty in walking, which exhibits a rolling characteristic.13 Weakness in ocular, bulbar, and respiratory muscles is less common than in myasthenia gravis. Weakness predominantly affects proximal muscles, which may show augmentation of strength during the first few seconds of a maximal contraction. Reflexes are absent or depressed but can increase after 10 seconds of maximal contraction of the muscle (post-tetanic potentiation). Autonomic symptoms such as dry mouth, constipation, and erectile dysfunction are present in most patients. Cerebellar ataxia may be present in association with small cell lung cancer. Patients with nonparaneoplastic Lambert-Eaton myasthenic syndrome may have other autoimmune diseases, notably vitiligo.

DIAGNOSIS

Diagnosis is based on the clinical features, on a positive serum voltage-gated calcium-channel antibody test, and on the characteristic EMG findings (see Table 422-2). Antibodies specific for the α1A (P/Q) subtype of voltagegated calcium channels are found in 90% of patients, both with and without small cell lung cancer. Patients may not respond convincingly to intravenous edrophonium. On EMG, the amplitude of the resting compound muscle action potential is reduced. It decreases further during low-rate repetitive nerve stimulation but increases by more than 100% immediately after 10 seconds of voluntary contraction of the muscle or during high-frequency (40-Hz) nerve stimulation. Single-fiber EMG is less specific because an increased jitter does not distinguish between myasthenia gravis and LambertEaton myasthenic syndrome. On diagnosis, an extensive search for malignancy is necessary. All patients should undergo thoracic computed tomography scanning and fluorodeoxyglucose positron emission tomography (FDG-PET). If tumor screening is negative, it should be repeated periodically every 3 to 6 months for at least 2 years after the onset of neurologic symptoms.

Differential Diagnosis

Botulinum poisoning (Chapter 296) causes blockade of presynaptic transmitter release at the neuromuscular junction as well as EMG changes similar to those in the Lambert-Eaton myasthenic syndrome. Botulism is detected by finding the toxin in serum or the Clostridium botulinum bacteria in the wound or feces. Myopathies (Chapters 269 and 421) can mimic LambertEaton myasthenic syndrome clinically, but autonomic changes do not occur, EMG findings are different, and muscle biopsy is abnormal.

antibody.14 Specific tumor treatment often leads to improvement of the neurologic disorder. Most patients respond to symptomatic treatment  with 3,4-diaminopyridine (10 to 20 mg four times daily). A9  However, 3,4-diaminopyridine has not yet been approved by the U.S. Food and Drug Administration, but a phosphate version of the drug has been licensed in Europe. Immunosuppressive treatment with prednisone, azathioprine, or cyclosporine may be required in patients with severe weakness, using doses similar to those prescribed for myasthenia gravis. Rituximab has been used in few patients with severe weakness.

PROGNOSIS

Prognosis mainly depends on that of the associated malignancy. Patients with paraneoplastic Lambert-Eaton myasthenic syndrome tend to have a progressive disease and a less satisfactory response to treatment.

  ACQUIRED NEUROMYOTONIA DEFINITION AND EPIDEMIOLOGY

Neuromyotonia, or Isaacs’ syndrome, is a rare disorder primarily characterized by myokymia (spontaneous undulating muscle contractions) that can be intermittent or continuous and may be present during sleep or general anesthesia. It results from the hyperexcitability of motor nerves. A milder variant, the cramp-fasciculation syndrome, is more common.

PATHOBIOLOGY

Neuromyotonia may be associated with other autoimmune diseases or other autoantibodies, and cerebrospinal fluid analysis may show oligoclonal bands. In about 15% of patients, it is paraneoplastic, usually associated with thymoma and more rarely with lung cancer. Occasionally, neuromyotonia follows infection or allergic reactions, and it may improve spontaneously within weeks to months in these cases. In neuromyotonia, peripheral nerve hyperexcitability is caused by dysfunction of voltage-gated potassium channel (Kv1), whose activation within milliseconds of nerve depolarization limits the depolarizing afterpotential and prevents the generation of repetitive discharges. In autoimmune neuromyotonia, pathogenic antibodies may induce loss of voltage-gated potassium channels and are often directed against contactin-associated protein-2, which is required for clustering Kv1 channels at the juxtaparanodal regions.

CLINICAL MANIFESTATIONS

The clinical presentation is variable but can include muscle stiffness, cramps, myokymia, fasciculations, pseudomyotonia (e.g., failure to relax after fist clenching), and weakness. Increased sweating is common. In the crampfasciculation syndrome, symptoms are milder and mostly induced by exertion. Some patients have sensory symptoms, including neuropathic-type pain, or less severe paresthesias, dysesthesia, and numbness, and a few have autonomic and central nervous system features of an encephalopathy, with insomnia, hallucinations, delusions, and mood change (Morvan’s syndrome).

DIAGNOSIS

EMG shows spontaneous motor unit discharges: distinctive doublet, triplet, or multiplet bursts with high intraburst frequency (40 to 300 per second), longer continuous bursts, and postactivation contraction. The abnormal muscle activity may be generated at different sites throughout the length of the nerve but is usually distal. Many patients have serum antibodies to the voltage-gated potassium channel-complex (Kv1 channel and associated proteins), predominantly to contactin-associated protein-2.15 The differential diagnosis includes neuromyotonia caused by acquired and inherited neuropathies and neuromyotonia—usually associated with episodic ataxia— caused by voltage-gated potassium-channel gene mutations (Kv1.1).

TREATMENT AND PROGNOSIS  TREATMENT  Plasmapheresis leads to clinical improvement within a few days in acutely ill patients, and most patients respond to immunosuppressive drugs or  intravenous immunoglobulin therapy. Intravenous immunoglobulin therapy  (1 g/kg for 2 days) improves strength, with an associated decline in specific

Neuromyotonia can be improved by anticonvulsant drugs, such as carbamazepine (up to 800 to 1000 mg daily), phenytoin (up to 300 mg daily), or lamotrigine (up to 100 mg daily), that depress sodium channel function and reduce the hyperexcitability of nerves. Plasmapheresis and intravenous immunoglobulins, using the same regimen as for myasthenia gravis, may be followed by short-term improvement. Immunosuppressive medications (as for

CHAPTER 422  Disorders of Neuromuscular Transmission  

myasthenia gravis) are effective in some patients. Neuromyotonia is often a monophasic disease that can be successfully managed with symptomatic and immunodulating treatment. When it is associated with myasthenia gravis, the administration of pyridostigmine can increase symptoms of motor nerve hyperexcitability. Prognosis is less favorable in cases with central nervous system involvement, which may be associated with invasive thymomas.

  GENETIC MYASTHENIC SYNDROMES Congenital myasthenic syndromes (see Table 422-1) are inherited disorders that result from mutations in genes encoding key proteins at the neuromuscular junction. In the United Kingdom, their prevalence is at least 6 per 1 million population.

PATHOBIOLOGY

Congenital myasthenic syndromes are classified by the site of the mutated protein: presynaptic, synaptic, or postsynaptic. Postsynaptic disorders are more frequent and most commonly involve the AChR ε-subunit gene, in which single nucleotide missense substitutions or frameshift mutations result in complete loss of function of the AChR ε-subunit. Because this subunit replaces the AChR γ-subunit around the time of birth, infants are normal in development but show weakness during late pregnancy and in the neonatal period. Survival depends on the continued expression of the γ-subunit. By contrast, null mutations in non-epsilon subunits are probably lethal. AChR deficiency can also result from defects in the gene for rapsyn, a cytoplasmic protein required for the clustering of the AChRs at the neuromuscular junction. Single-nucleotide changes in genes for any of the AChR subunits can affect affinity for acetylcholine and gating efficiency, thereby leading to kinetic defects. In the fast-channel syndrome (recessive), AChR openings are abnormally brief, whereas the opposite occurs in the slow-channel syndrome (dominant), in which the channel opens for prolonged periods, thereby resulting in subsynaptic accumulation of cations and degenerative changes with loss of AChR. Mutations in the COLQ gene, which gives rise to the collagen tail that anchors acetylcholinesterase in the synaptic cleft, are less common. The absence of acetylcholinesterase is responsible for reduced quantal release and for continuous exposure of the postsynaptic membrane to acetylcholine, thereby leading to cation overload and junctional fold degeneration. Mutations in choline acetyltransferase, the enzyme responsible for the synthesis of acetylcholine, do not always lead to dysfunction at rest; during repetitive activity, however, the amount of acetylcholine in each packet decreases, with consequent failure of neuromuscular transmission. Mutations in DOK7 cause a “synaptopathy” with small, simplified neuromuscular junctions. Dok-7 binds MuSK, and the mutations are thought to impair the signaling that maintains the synaptic structure. Other gene mutations are less common (see Table 422-1).

CLINICAL MANIFESTATIONS

Clinical manifestations may vary from death in utero in severe cases to mild symptoms that present in adulthood.16 Although most cases present in infancy with ptosis, hypotonia, and difficulties with feeding and breathing, the slightly different patterns of muscle weakness provide clues that point to which gene is involved. Arthrogryposis multiplex congenita, indicative of fetal akinesia, occurs with rapsyn mutations. Life-threatening episodic apnea can occur with mutations in choline acetyltransferase or rapsyn or in fastchannel syndromes. Severe ophthalmoplegia occurs in end-plate acetylcholinesterase deficiency, AChR deficiency due to AChR subunit mutations, and fast-channel syndromes, but is rarely seen in the other genetic syndromes. Motor symptoms with DOK7 mutations usually appear at about 2 years of age after the child first learns to walk and are characterized by a limbgirdle weakness associated with ptosis and by facial and bulbar muscle involvement.

2553

DIAGNOSIS

A congenital myasthenic syndrome should be considered when symptoms are evident at birth or during early infancy and other relatives are affected. However, a negative family history does not exclude the diagnosis, and the onset can be later in the slow-channel syndrome, rapsyn, and DOK7 mutations. Impaired neuromuscular transmission can be detected by a decremental response on repetitive nerve stimulation and increased jitter on single-fiber EMG. In the slow-channel and acetylcholinesterase deficiency syndromes, the prolonged end-plate potential outlasts the refractory period of the muscle fiber, and a single nerve stimulus can be followed by a repetitive compound muscle action potential (double response) (see Table 422-2). Genetic analysis is essential to confirm the diagnosis and help in treatment, prognosis, and counseling, although the faulty gene has not been identified in many families. The principal differential diagnoses are spinal muscular atrophy, infant botulism, hereditary neuropathies, and congenital myopathies or muscular dystrophies. Onset in early childhood, adolescence, or adulthood may lead to the incorrect diagnosis of seronegative myasthenia gravis.

TREATMENT AND PROGNOSIS  Many of the congenital myasthenic syndromes respond to acetylcholinesterase inhibitors, as used for myasthenia gravis, and to 3,4-diaminopyridine (1 mg/kg/day in four divided doses). Patients with the slow-channel syndrome respond to quinidine (at doses corresponding to serum levels of 1 to 2.5 mg/L) or to fluoxetine (60 to 100 mg/day in adults though some patients may respond to doses as low as 20 mg), but the use of fluoxetine in children or adolescents requires psychiatric supervision. For syndromes in which the neuromuscular junction is destabilized or there are degenerative changes, such as for Dok-7 or end-plate acetylcholinesterase deficiency, treatment with ephedrine (45 to 100 mg/day in adults, 3 mg/kg/day in children) or salbutamol (0.5 to 4 mg, three times a day) can be remarkably effective. The beneficial effects of this treatment are not seen immediately but build up over a period of 6 months or more. Although congenital disorders can be fatal during infancy, usually because of apneic episodes during infections, most tend to be nonprogressive or even may improve during adolescence or adult life. The exceptions are the slowchannel syndrome and acetylcholinesterase deficiency, which, owing to the excess AChR activations, can be associated with end-plate progressive degenerative changes, although this risk is largely mitigated with treatment.

Grade A References A1. Schneider-Gold C, Gajdos P, Toyka KV, et al. Corticosteroids for myasthenia gravis. Cochrane Database Syst Rev. 2005;2:CD002828. A2. Palace J, Newsom-Davis J, Lecky B. A randomized double-blind trial of prednisolone alone or with azathioprine in myasthenia gravis. Myasthenia Gravis Study Group. Neurology. 1998;50: 1778-1783. A3. Hart IK, Sathasivam S, Sharshar T. Immunosuppressive agents for myasthenia gravis. Cochrane Database Syst Rev. 2007;4:CD005224. A4. Nagane Y, Utsugisawa K, Obara D, et al. Efficacy of low-dose FK506 in the treatment of myasthenia gravis: a randomized pilot study. Eur Neurol. 2005;53:146-150. A5. Heckmann JM, Rawoot A, Bateman K, et al. A single-blinded trial of methotrexate versus azathioprine as steroid-sparing agents in generalized myasthenia gravis. BMC Neurol. 2011;11:97. A6. Gajdos P, Chevret S, Toyka KV. Intravenous immunoglobulin for myasthenia gravis. Cochrane Database Syst Rev. 2012;12:CD002277. A7. De Feo LG, Schottlender J, Martelli NA, et al. Use of intravenous pulsed cyclophosphamide in severe, generalized myasthenia gravis. Muscle Nerve. 2002;26:31-36. A8. Howard JF Jr, Barohn RJ, Cutter GR, et al. A randomized, double-blind, placebo-controlled phase II study of eculizumab in patients with refractory generalized myasthenia gravis. Muscle Nerve. 2013;48:76-84. A9. Keogh M, Sedehizadeh S, Maddison P. Treatment for Lambert-Eaton myasthenic syndrome. Cochrane Database Syst Rev. 2011;2:CD003279.

GENERAL REFERENCES For the General References and other additional features, please visit Expert Consult at https://expertconsult.inkling.com.

CHAPTER 422  Disorders of Neuromuscular Transmission  

GENERAL REFERENCES 1. Andersen JB, Heldal AT, Engeland A, et al. Myasthenia gravis epidemiology in a national cohort; combining multiple disease registries. Acta Neurol Scand Suppl. 2014;26-31. 2. Berrih-Aknin S, Le Panse R. Myasthenia gravis: a comprehensive review of immune dysregulation and etiological mechanisms. J Autoimmun. 2014;52:90-100. 3. Marx A, Pfister F, Schalke B, et al. The different roles of the thymus in the pathogenesis of the various myasthenia gravis subtypes. Autoimmun Rev. 2013;12:875-884. 4. Koneczny I, Cossins J, Vincent A. The role of muscle-specific tyrosine kinase (MuSK) and mystery of MuSK myasthenia gravis. J Anat. 2014;224:29-35. 5. Zisimopoulou P, Evangelakou P, Tzartos J, et al. A comprehensive analysis of the epidemiology and clinical characteristics of anti-LRP4 in myasthenia gravis. J Autoimmun. 2014;52:139-145. 6. El-Salem K, Yassin A, Al-Hayk K, et al. Treatment of MuSK-associated myasthenia gravis. Curr Treat Options Neurol. 2014;16:283. 7. Berrih-Aknin S, Frenkian-Cuvelier M, Eymard B. Diagnostic and clinical classification of autoimmune myasthenia gravis. J Autoimmun. 2014;48-49:143-148. 8. Skeie GO, Apostolski S, Evoli A, et al. Guidelines for treatment of autoimmune neuromuscular transmission disorders. Eur J Neurol. 2010;17:893-902.

2553.e1

9. Reddel SW, Morsch M, Phillips WD. Clinical and scientific aspects of muscle-specific tyrosine kinase-related myasthenia gravis. Curr Opin Neurol. 2014;27:558-565. 10. Kerty E, Elsais A, Argov Z, et al. EFNS/ENS guidelines for the treatment of ocular myasthenia. Eur J Neurol. 2014;21:687-693. 11. Ettinger DS, Riely GJ, Akerley W, et al. Thymomas and thymic carcinomas: clinical practice guidelines in oncology. J Natl Compr Canc Netw. 2013;11:562-576. 12. Kalita J, Kohat AK, Misra UK. Predictors of outcome of myasthenic crisis. Neurol Sci. 2014;35:1109-1114. 13. Titulaer MJ, Lang B, Verschuuren JJ. Lambert-Eaton myasthenic syndrome: from clinical characteristics to therapeutic strategies. Lancet Neurol. 2011;10:1098-1107. 14. Evoli A, Liguori R, Romani A, et al. Italian recommendations for Lambert-Eaton myasthenic syndrome (LEMS) management. Neurol Sci. 2014;35:515-520. 15. Irani SR, Alexander S, Waters P, et al. Antibodies to Kv1 potassium channel-complex proteins leucine-rich, glioma inactivated 1 protein and contactin-associated protein-2 in limbic encephalitis, Morvan’s syndrome and acquired neuromyotonia. Brain. 2010;133:2734-2748. 16. Rodríguez Cruz PM, Palace J, Beeson D. Inherited disorders of the neuromuscular junction: an update. J Neurol. 2014;261:2234-2243.

2553.e2

CHAPTER 422  Disorders of Neuromuscular Transmission  

REVIEW QUESTIONS 1. A 54-year-old man has been recently diagnosed with anti-AChR-positive myasthenia gravis. The disease has had an acute onset and rapid progression, leading in few weeks to severe generalized weakness with marked bulbar symptoms, including dysphagia, dysarthria, neck weakness, and impaired respiratory function. A thymoma is detected on thoracic computed tomography scan. Symptoms are not satisfactorily relieved by fulldose pyridostigmine treatment. Which of the following should be considered as first therapeutic option in this case? A. Immediate thymectomy B. Plasmapheresis (or immunoglobulin infusion) and prednisone C. Plasmapheresis D. Azathioprine E. Rituximab Answer: B  Thymectomy is indicated and should be performed as soon as possible because of the presence of a thymoma. The patient has severe and poorly controlled symptoms. In this situation, surgery should be deferred until the myasthenia gravis improves. Plasmapheresis (or immunoglobulin infusion) provides a rapid onset of improvement, which is, however, shortlived unless immunosuppressive treatment is added. Prednisone is the firstchoice immunosuppressant because of its short-latency effect; the association of plasmapheresis (or immunoglobulin infusion) can enhance the therapeutic effect and minimize the temporary exacerbation that can occur at the start of prednisone administration. Because of its long-latency effect, azathioprine is not the first option in this case. Rituximab is reserved for refractory disease.

2. A 70-year-old woman presents with asymmetrical ptosis, which is more pronounced on her right eye, and intermittent diplopia. Her symptoms fluctuate daily, and they improve with rest but worsen in the evening. Her symptoms have been present for more than 1 year, with periods of spontaneous remission. On the whole, the symptoms do not interfere with her daily activities. On clinical examination, mild fatigability of neck extensors and arm proximal muscles is also evident. Myasthenia gravis is diagnosed by detecting anti-AChR antibodies, increased jitter on SF-EMG, and a positive response to neostigmine. Thoracic computed tomography scan is negative. The patient also has diabetes controlled by diet. Which of the following is the correct recommendation as first treatment in this patient? A. Pyridostigmine B. Prednisone and azathioprine C. Thymectomy D. Plasmapheresis E. Fluoxetine Answer: A  This patient has mild generalized myasthenia gravis with predominantly ocular symptoms. The disease has a slow course and is not disabling. In this context, treatment with pyridostigmine can be sufficient to control patient’s symptoms and should be tried first. Immunosuppressive therapy (prednisone and azathioprine) and plasmapheresis should be considered if the disease worsens. Thymectomy is not indicated because the patient has late-onset nonthymomatous disease. Fluoxetine blocks AChR channels; it is contraindicated in myasthenia gravis, and its use is limited to the treatment of the slow-channel myasthenic syndrome.

This page intentionally left blank

XXVI

EYE, EAR, NOSE, AND THROAT DISEASES 423 DISEASES OF THE VISUAL SYSTEM 424 NEURO-OPHTHALMOLOGY 425 DISEASES OF THE MOUTH AND SALIVARY GLANDS

426 APPROACH TO THE PATIENT WITH

NOSE, SINUS, AND EAR DISORDERS

427 SMELL AND TASTE

428 HEARING AND EQUILIBRIUM 429 THROAT DISORDERS

2556

CHAPTER 423  Diseases of the Visual System  

423  DISEASES OF THE VISUAL SYSTEM MYRON YANOFF AND J. DOUGLAS CAMERON

The eye is a compact, complicated structure (Fig. 423-1) that is remarkably stable throughout life. Once the growth of the eye is complete, at approximately age 3 years, the structure of the eye changes very little for the next 60 to 80 years. The eyelids physically protect the eye. The pathway of light through the eye, termed the visual axis, is transparent and contains no opaque structures such as blood vessels. Light passes through the tear film, cornea, intraocular aqueous, crystalline lens, vitreous, and retina, all of which, except for the crystalline lens, remain essentially transparent throughout life. Delicate intraocular structures are protected by a tough collagenous “eye wall” composed of the cornea and sclera. The optic nerve, which is composed of axons from the retinal ganglion cells, is supported by dura, arachnoid, and pia mater, which are contiguous with the brain. The optic nerve is long enough to allow free excursions of the eye through a 100-degree arc under the influence of six coordinated and critically placed rectus muscles. All these functional components are housed in a bony cavity, the orbit, which protects the eye from external injury. The eyelid skin, only loosely connected to underlying structures, is among the thinnest of the body. The eyelid is unique because it contains the highest density of sebaceous glands in the body. These meibomian glands produce a sebaceous (lipid) material that is the principal evaporation retardant for the tear film. Malorientation of the eyelid margin or malorientation of the cilia (trichiasis) may cause extensive scarring of the anterior surface of the cornea, even to the point of blindness. The eyelid is opened by contraction of the levator muscle. The tendon of the levator tends to degenerate over time to produce mechanical ptosis. The soft tissue of the eyelid is separated from the soft tissue of the orbit by the orbital septum, a major collagenous barrier that protects intraorbital soft tissue from extension of preseptal eyelid inflammation. Extension of inflammation from preseptal cellulitis or ethmoiditis may cause septic optic neuropathy or cavernous sinus thrombosis. The elastic tissue supporting the skin of the anterior eyelid is reduced over time, thereby causing dermatochalasis (“baggy eyelids”). Redundant tissue may be sufficient in quantity to restrict the visual field, particularly superiorly. The conjunctiva is a mucous membrane covered by stratified, nonkeratinizing squamous epithelium containing goblet cells. The epithelium is supported by delicate fibrovascular tissue that contains lymphatic channels. Squamous carcinoma or malignant melanoma originating in the conjunctiva may extend through these channels to regional lymph nodes or beyond. The conjunctival epithelium contains melanocytes. Immune processing cells are present in the epithelium (Langerhans cells) and in the stroma as collections of non-nodal B and T lymphocytes. Non-nodal primary lymphomas, which tend to have an indolent course in this location, may arise from this tissue. The aqueous portion of tears is formed constantly by accessory lacrimal glands in the conjunctiva and eyelid soft tissue as well as by reflex action from the lacrimal gland. Symptoms of itching and burning, as well as periodic disturbance of vision, may result from inadequacies of the tear film layer. Tears drain through puncta at the nasal eyelid margin through the nasolacrimal duct to exits in the nasal cavity inferior to the inferior turbinate. The epithelium of the nasolacrimal duct also contains melanocytes and is supported by a resting lymphocyte population. Neoplasms including lymphoma, concretions (dacryoliths), and tissue injury from trauma may occlude the puncta in adults. The cornea is avascular and lined both anteriorly and posteriorly by surface cells. Lack of an adequate tear film (dry eye syndrome) may seriously alter the ability of the cornea to transmit light, thereby affecting visual acuity. The posterior cellular lining of the cornea is a single layer of highly modified corneal endothelial cells that maintain tissue dehydration. Lack of effective pumping by the endothelial cells will allow excess hydration of the corneal stroma, that is, corneal edema. The corneal stroma is particularly sensitive to proteolysis from collagenases found with certain inflammatory conditions, such as herpes simplex keratitis. The cumulative effect of multiple episodes may be corneal thinning and possible perforation of the cornea.

Intraocular pressure is measured by applanation tomography. The amount of pressure necessary to flatten the central cornea is proportional to the intraocular pressure. The anterior chamber is bounded by the posterior surface of the cornea, the anterior surface of the iris, and the anterior surface of the crystalline lens within the pupillary space. Aqueous material normally flows from the posterior chamber into the anterior chamber through the pupil and exits into the general circulation through the trabecular meshwork. Most causes of pathologically increased intraocular pressure and optic nerve damage (i.e. glaucoma) are due to abnormalities of filtration through the trabecular meshwork. The posterior chamber is bounded by the posterior surface of the iris, the ciliary body circumferentially, and the anterior surface of the vitreous. The crystalline lens is located entirely in the posterior chamber. The anterior segment is composed of the cornea and the anterior and posterior chambers. Most of the anterior segment is derived from the skin and neural crest tissue. The posterior segment is the remainder of the eye. Most of the posterior segment structures are derived from the central nervous system and neural crest tissue. When first formed, the crystalline lens is a totally cellular structure bounded by a true basement membrane. Throughout life, the new cells that are continuously added from the outer layer epithelial cells compress the central cells, thereby resulting in cell degeneration in the central core (nucleus). The lens doubles in volume from birth to age 70 years at the cost of both pliability (presbyopia) and clarity (cataract). The lens is suspended in the posterior chamber by fibers (zonules) attached to the ciliary body. The ciliary body is the posterior extent of the iris. Its surface cells produce aqueous, and its muscles function in accommodation. The vitreous is composed primarily of water and type II collagen. The vitreous makes up the majority of the volume and weight of the eye. It functions as a biochemical sink as well as to maintain neural retinal attachment. With time, the vitreous shrinks and separates from the retina (posterior vitreous detachment). Condensed and displaced vitreous casts a shadow on the retina, which is perceived by the patient as “floaters.” The retina is the site of photochemical conversion of light to electrical energy. Ganglion cells and their axons in the internal retina aggregate at the optic disc to form the optic nerve. Only the inner half of the retina is supplied by intraretinal vessels that are seen by ophthalmoscopy. The outer half of the retina is supplied by large-caliber capillary vessels in the choroid (the choriocapillaris). Only a 500-µm area of the posterior retina, the central macula (about 3 to 5% of the total retina), has the ability to resolve images to 20/20. The remainder of the retina has much less sensitive image resolution. Extensive biochemical support and control of stray light are performed by the retinal pigment epithelium located between the choriocapillaris and the photoreceptor outer segments. The blood-retinal barrier, which protects the biochemical integrity of the retina, is composed of anatomic attachments between neighboring retinal pigment epithelial cells, as well as attachments between vascular endothelial cells of the retinal circulation. The retina is held in place by physiologic forces that may be compromised by holes in the retina (rhegmatogenous retinal detachment) or by fluid accumulating in the subretinal space (serous retinal detachment). The optic nerve is composed of approximately 1 million axons from retinal ganglion cells. Axons are separated into bundles by pial septa, which are in turn enclosed in an arachnoid layer. The dura is contiguous with the posterior sclera and the periosteum of the optic canal. Delicate vessels extending from the dura across the arachnoid to the pial septa supply the optic nerve. The central retinal artery is present in the axial layer of the optic nerve near the eye but does not supply blood to the optic nerve itself. The optic nerve axons travel through a collagenous sieve in the plane of the posterior sclera, the lamina cribrosa. The choroid is that portion of the uveal tract external to the retina. This layer is composed of various calibers of blood vessels that ultimately supply blood for the choriocapillaris. There are no lymphatic channels in the choroid. The sclera is composed of dense, relatively disorganized collagen. It is opaque because of the nonhomogeneous structure of the collagen and the degree of hydration relative to the cornea. There are multiple scleral ostia for arteries, veins, and nerves, both posteriorly and anteriorly. The orbit is composed of bones of the facial skeleton. Sutures between major bones exist in the superior nasal and superior temporal quadrants. Multiple vessels and nerves extend through the thin ethmoid bone from nasal sinus tissue medially. The orbital floor is poorly supported over the maxillary sinus and may rupture with increased intraorbital pressure. The nasolacrimal duct travels through a portion of the lacrimal bone. Portions of the sphenoid

2557

CHAPTER 423  Diseases of the Visual System  

Pupil Iris

Cornea Anterior chamber Posterior chamber Canal of Schlemm

Lens Conjunctiva

Ciliary body Zonule

Medial rectus muscle

Lateral rectus muscle

Retina Choroid

Vitreous body

Sclera Macula Optic disc Optic nerve

20 ft

40 ft

10 ft

FIGURE 423-2.  Snellen visual acuity. The most common test for visual acuity measures the eye’s ability to resolve linear images at a test distance of 20 feet, approximating infinity (parallel rays of light). A 20/20 E subtends 5 minutes of arc at a distance of 20 feet, with each segment of the E subtending 1 minute of arc. The larger letters (e.g., 20/30, 20/40) are determined by the distance at which they subtend an angle of 5 minutes. Thus, an E that subtends 5 minutes at 40 feet, if viewed clearly at 20 feet, indicates 20/40 visual acuity.

Central artery and vein

FIGURE 423-1.  Anatomy of the eye.

bone protect the optic nerve. Major cranial nerves travel through the adjacent superior orbital fissure, also a portion of the sphenoid bone. There is no normal lymphoid tissue in the orbit outside of the lacrimal gland. The rectus muscle may be enlarged by inflammation in thyroid eye disease, but the tendinous insertion into the sclera is not inflamed early in the course of the disease.

CLINICAL MANIFESTATIONS AND DIAGNOSIS

Patients may present with complaints of diminished vision, eye pain, red eyes, or pain around the eye. The causes may be primarily ophthalmic (e.g., cataract) or systemic (e.g., diabetic retinopathy). A comprehensive ophthalmologic examination also should evaluate for possible asymptomatic local (e.g., choroidal melanoma) or systemic (e.g., hypertensive retinopathy) abnormalities in patients with normal acuity and no subjective complaints.

Functional Evaluation

The most objective and common measure of ocular function is line letter acuity, with normal vision (Table 423-1) defined as the ability to see at 20 feet what a normal person sees at 20 feet (Fig. 423-2). Less than 20/20 vision can be caused by an abnormality anywhere from the tear film to the visual cortex of the occipital lobe (see Fig. 423-1). However, normal vision also includes other functions, such as the perception of color, motion, contrast, brightness, field, and depth, for which there is greater variation among individuals and no universally adopted, standardized scales. Normal visual acuity is potentially achievable in essentially all individuals, either naturally or with visual correction. Correction of vision is based on the refraction of light (Fig. 423-3). The diopter (D) is the unit of measurement of the ability of an optical system to refract (bend) light. The normal human eye has the refractive capacity of approximately 60 D. If the eye is too short, light will be focused behind the eye (hyperopia). If the eye is too long, light will be focused in the vitreous in front the retina (myopia). Normally, a person can voluntarily control the crystalline lens, alternating between near and distant tasks. At approximately age 45 years, the lens becomes stiff, and the eye becomes set for distance (presbyopia). Refraction is the method of determining the amount of optical correction (strength of glasses) needed to establish 20/20 (6/6) vision. Examination of pupillary response assesses whether neural function is intact (see Figs. 424-2 and 424-4). Confrontational visual field testing (see Fig. 424-1 in Chapter 424) should be performed in each eye to detect gross quadrantic defects. Extraocular motility should be assessed to exclude nerve or muscle abnormalities (see Table 424-4). Color vision testing plates are a sensitive indicator of optic nerve function. Diagnostic testing during a routine eye examination also includes external examination of the lids and adnexa, applanation tonometry to determine intraocular pressure, biomicroscopy (slit lamp examination) of the anterior

A

B

FIGURE 423-3.  Myopia/hyperopia. A, In the myopic eye, parallel rays of light are focused anterior to the retina. A divergent lens can be used to compensate for the mismatch between refracting power and axial length. B, The hyperopic eye requires the additional power of a convergent lens to bring images into focus on the retina.

TABLE 423-1  VISUAL ACUITIES REQUIRED FOR COMMON DAILY TASKS 20/20

Physiologic vision

20/30-20/100

Driver’s license, varies by state

20/50

Newspaper print

20/70

Large-print Reader’s Digest

20/100

Write a check

20/200

Legally blind

20/400

Paper currency

segment, and ophthalmoscopic examination of the ocular fundus. Other office tests when indicated include exophthalmometry (measurement of proptosis), visual field and electrophysiologic testing, and vascular imaging (fluorescein angiography, mainly in diabetic patients; ocular computed tomography [OCT] to investigate retinal macular disease); and corneal topography. The conjunctiva, cornea, lens, and anterior chamber are evaluated using a slit lamp. The slit lamp is composed of a binocular microscope with variable magnification (40× and 80×) in conjunction with adjustable light sources. An increased concentration of protein can be detected in the anterior chamber because of the Tyndall (flare) effect, indicating vascular incompetence associated with either inflammation or ischemia. Even individual inflammatory cells can be resolved with the slit lamp. A cobalt blue filter can be used to detect fluorescein dye that accumulates in regions of abnormal epithelium (dendrite of herpes simplex keratitis or a corneal abrasion). The slit beam is used to examine the crystalline lens to determine the depth of the anterior chamber and the degree of opacification from a cataract. By using a green filter with the 90-D lens, the retinal vessels and retinal vascular

2558

CHAPTER 423  Diseases of the Visual System  

abnormalities such as microaneurysm can be seen at relatively high magnification.

  COMMON CLINICAL CONDITIONS

Chronic Abnormal Vision

MYOPIA

Nearsightedness (myopia; see Fig. 423-3) is usually discovered during childhood when children cannot perform distant tasks during school (reading the blackboard) or during school screening. Myopia usually progresses until the eye is fully developed, typically by age 20 to 25 years. Rapidly progressing myopia during childhood or at any time after age 25 years requires evaluation for juvenile glaucoma, diabetes mellitus (reversible metabolic changes in the crystalline lens), trauma (development of a cataract), or use of corticosteroids (development of a cataract). Myopia is usually fully correctable with glasses. Laser in situ keratomileusis (LASIK) is one of the refractive surgical procedures that may be used in adults to correct myopia and other refractive errors, with 95% of patients achieving visual acuity of 20/40 or better. A1  Complications of LASIK include glare symptoms, dry eye, and undercorrection or overcorrection. Rare but serious complications include epithelial ingrowth, diffuse keratitis, and flap dislocation. In patients who undergo subsequent cataract surgery, special attention is needed to calculate the parameters for the intraocular lens. Pathologic myopia is a heritable condition causing progressive weakening of the posterior sclera and resulting increases in the axial length of the eye. The posterior radius of curvature of the eye increases (posterior staphyloma). The refractive error is usually above –8 D and may be as high as –20 D in severe cases. Abnormal physical forces in pathologic myopia may lead to retinal hole formation, retinal detachment, or intraocular hemorrhage. The primary abnormality leading to loss of vision is choroidal neovascularization. Associated systemic conditions include trisomy 21, Cornelia de Lange’s syndrome, Stickler’s syndrome, and Marfan syndrome. Clinical surveillance must be maintained to watch for treatable complications such as retinal detachment. Pathologic myopia is usually treated with spectacles or contact lenses. Refractive procedures are less successful in pathologic myopia because of the severity of the refractive errors and the presence of posterior segment abnormalities. Surgical and laser procedures may be required to treat retinal and choroidal lesions (e.g., subretinal neovascularization).

HYPEROPIA

In hyperopia (farsightedness; see Fig. 423-3), in contrast to myopia, the eye tends to have a shorter than average axial length. Compensatory mechanisms of the crystalline lens may functionally correct small degrees of hyperopia until age 40 years, when the crystalline lens loses its pliability. The 40-year-old hyperopic changes from latent to manifest hyperopia simultaneously. The initial pair of glasses may have to correct for both distance and near tasks (bifocals). Refractive surgical procedures can correct up to 5 D of hyperopia.

ASTIGMATISM

Astigmatism usually is the result of irregularities in the radius of curvature of the cornea. Astigmatism is not a pathologic state, but rather a variation in anatomy; most people have some degree of astigmatism. Trauma to the cornea can cause alteration of structure, leading to irregular astigmatism. Symptoms are predominantly blurry vision and difficulty seeing fine detail. Regular astigmatism can be corrected with spectacle lenses or with rigid contact lenses, whereas irregular astigmatism requires rigid contact lenses. Some forms of astigmatism can also be corrected with laser ablation of the cornea.1

KERATOCONUS

Keratoconus is an acquired irregularity of corneal curvature; its cause is controversial but probably at least partly genetic. Onset generally is during adolescence, and the process often evolves over 5 years or so. The prevalence in the United States appears to be about 55 cases per 100,000 people but is probably greater when subclinical cases diagnosed by computerized videokeratography are considered. Patients with keratoconus typically have astigmatism, which may be severe. The condition is usually bilateral but not symmetrical. Mild degrees of keratoconus can be corrected with glasses or contact lenses. Corneal collagen cross-linking can provide a sustained improvement in vision. A2  For severe cases, corneal transplantation (graft) often is very successful.2

STRABISMUS

The control of simultaneous orientation of the two eyes to ensure that the visual axes of both eyes are aligned is not complete until several years following birth. Misalignments of the two eyes (strabismus) may be the result of abnormalities in the central nuclei of the brain, malfunction of one or several peripheral nerves, or intrinsic abnormalities of the rectus or oblique muscles (see Fig. 424-6). If the eyes are not simultaneously stimulated with the images of the same degree of clarity or complexity, only one eye will develop normally. The image processing ability of the blurred eye will not develop (amblyopia). In most cases, only the central vision is affected. The peripheral vision in both eyes is likely to be equal and normal. Amblyopia also may be caused by a marked difference in refractive error between the two eyes (anisometropic amblyopia) or eyelid ptosis (deprivational amblyopia). Ptosis may be neurogenic or mechanical (e.g., congenital eyelid hemangioma). To avoid amblyopia, it is extremely important to refer a child with strabismus to an ophthalmologist as soon as the strabismus is noted. Treatment options include patching or atropine eye drops, which are equally effective in providing good vision if patients are treated before age 7 years.3 The outcome of treatment is more favorable if amblyopia is detected before age 2 to 3 years but may be successful into the teenage years. Esotropia is deviation of one or both eyes inward. Esotropia may not be clinically evident until the child is 3 to 4 months of age. Delay in facial maturation (underdeveloped nasal bridge) may give the appearance of esotropia, even though the visual axes are correctly aligned. In true strabismus, the light reflex will be in the center of one cornea and decentered in the other. Exotropia is deviation of one or both eyes outward. Exotropia tends to be intermittent and less likely to result in amblyopia. Both esotropia and exotropia may be treated by using appropriate corrections of refractive error with glasses (occasionally bifocals). Occasionally extraocular muscle surgery is necessary to correct alignment. To allow the second eye to develop to its full potential, amblyopia is corrected by occluding the stronger eye with a patch or by pharmacologically occluding (blurring) the better eye with atropine. Success is directly related to compliance.

DIPLOPIA (DOUBLE VISION)

Acute onset of diplopia is an ominous sign suggestive of a third nerve palsy (Chapter 424). Diplopia of any kind is an intolerable symptom, but vertical diplopia is less tolerated than horizontal diplopia.

COLOR VISION CHANGE

Most cases of congenital color blindness go undetected for many years. Acquired color deficiency at any age may be caused by a cataract or optic nerve disease.

CHANGE IN VISION

If only one eye has a change in vision, the problem, such as a cataract or retinal detachment, is most likely in that eye. If both eyes have a change in vision, the problem generally is outside of the eye, such as homonymous hemianopia (Chapter 424). Improvement of near vision in middle age may be a sign of cataract (“second sight”) or hyperglycemia. Transient complete unilateral or bilateral loss of vision may be caused by vascular abnormalities inside or outside of the eye (Table 423-2).

Acute Eye Abnormalities

PAIN

The most severe eye pain (Table 423-3), typically associated with a red eye, is caused by acute angle-closure glaucoma. Sharp, intermittent pain is usually caused by ocular surface abnormalities (e.g., corneal foreign body). Burning pain that clears with blinking generally relates to tear film abnormalities (dry eyes). Deep boring pain most often is associated with an ocular abnormality (e.g., uveitis).

RED EYE

A red or inflamed eye can be caused by conjunctivitis, iritis (anterior uveitis), acute glaucoma, corneal trauma, or infection (Table 423-4). Of these causes, all are typically painful, with the occasional exception of conjunctivitis.

DISTORTED VISION

Distorted vision (metamorphopsia), which is the perception that straight lines are distorted or bowed, results from macular dysfunction. Causes

CHAPTER 423  Diseases of the Visual System  

include fluid under the retina; exudative macular degeneration, which tends to elevate the retina; and an epiretinal membrane, which tends to contract the retina.

NIGHT BLINDNESS

Retinitis pigmentosa, vitamin A deficiency, and systemic medications such as phenothiazines can cause true night blindness, in which patients have difficulty seeing any stars in the sky on a clear night and may be unable to ambulate without assistance in a dark environment. Patients with cataracts may have difficulty driving at night because of excessive glare and visual distortion.

SENSATION OF FLASHING LIGHTS

The sudden onset of flashes in the peripheral visual field suggests a posterior vitreous detachment with resulting traction of the vitreous on the peripheral retina, sometimes with a resulting retinal tear. The flashes, which may be more pronounced in the dark and with rapid eye movement, may be associated with the sudden onset of floaters, which can indicate debris or blood in the vitreous cavity. Because a tear in the retina can lead to a retinal detachment, urgent consultation with an ophthalmologist is required.

FLOATERS

Floaters, which are caused by small aggregates in the vitreous cavity, result from the normal aging of the vitreous. The acute onset of vitreous floaters may be associated with uveitis or with the sudden onset of bleeding in the vitreous cavity owing to diabetes or sickle cell anemia. Acute floaters, however, particularly if associated with flashing lights, may indicate a posterior vitreous detachment or a retinal tear with an impending retinal detachment. Urgent ophthalmic referral is essential.

PHOTOPHOBIA

Photophobia, particularly if associated with eye pain, redness, and decreased vision, is a symptom of uveitis or traumatic iritis. Photophobia is also typical of acute migraine and meningeal irritation. Prompt ophthalmologic referral is prudent.

HALOS AROUND LIGHTS

FOREIGN BODY SENSATION

BILATERAL

Amaurosis fugax (carotid artery stenosis) Central retinal artery occlusion Occipital lobe infarct Temporal arteritis Nonarteritic anterior ischemic optic neuropathy Hemorrhage   Preretinal (high altitude, Valsalva)   Vitreous   Aqueous (hyphema) Trauma

Flashing light with a migraine (Chapter 398) is described as scintillations or zigzagging lights that march across the visual field for a few minutes or as long as 30 minutes, sometimes associated with transient visual field loss. Headache is not universal.

Patients with cataracts commonly see halos around lights, particularly when driving at night. Episodic decreased vision, redness, and halos around lights may be symptoms of impending angle-closure glaucoma. Halos also can occur as a complication of refractive eye surgery.

TABLE 423-2  DIFFERENTIAL DIAGNOSIS OF SUDDEN VISUAL LOSS UNILATERAL

2559

A foreign body sensation is commonly caused by dry eyes. An entropion (Fig. 423-4) or misdirected lashes (trichiasis) also can cause a foreign body sensation. Most corneal abrasions cause severe pain, but minor corneal abrasions may be associated with a foreign body sensation rather than the severe pain that usually accompanies a more severe abrasion. An arc welder burn causes a punctate corneal keratopathy, and foreign body sensation may be a prominent symptom. A true conjunctival or corneal foreign body also may be present.

Eclampsia Vertebrobasilar infarct Trauma

EXCESSIVE TEARING

Impairment of tear drainage can occur with an ectropion or any obstructions of the nasolacrimal drainage system. An entropion or abnormal lashes rubbing on the cornea (trichiasis) stimulate tear production.

EYELID TWITCHING

TABLE 423-3  CAUSES OF EYE PAIN Blepharitis Blocked tear duct Chalazion Conjunctivitis Corneal abrasion Dry eyes Ectropion Entropion Foreign object

Any irritation of the conjunctiva or cornea can cause the eyelids to twitch. Occasional twitching of the lids usually is associated with stress or adrenergic stimulation. Benign essential blepharospasm is severe spasm of the lids leading to functional impairment. Multiple sclerosis (Chapter 411) also can cause lid spasm.

Glaucoma Hordeolum (stye) Iritis Keratoconus Optic neuritis Scleritis Trauma Uveitis

CONJUNCTIVITIS

Any ocular inflammation, including corneal ulcers, angle-closure glaucoma, endophthalmitis, and uveitis, can be associated with secondary

TABLE 423-4  DIFFERENTIAL DIAGNOSIS OF COMMON CAUSES OF INFLAMED EYE* FEATURE

ACUTE CONJUNCTIVITIS

Incidence

Extremely common

ACUTE IRITIS† Common

ACUTE GLAUCOMA‡ Uncommon

CORNEAL TRAUMA OR INFECTION Common

Discharge

Moderate to copious

None

None

Watery or purulent

Vision

No effect on vision

Slightly blurred

Markedly blurred

Usually blurred

Pain

None

Moderate

Severe

Moderate to severe

Conjunctival injection

Diffuse: more toward fornices

Mainly circumcorneal

Mainly circumcorneal

Mainly circumcorneal

Cornea

Clear

Usually clear

Steamy

Change in clarity related to cause

Pupil size

Normal

Small

Moderately dilated and fixed

Normal or small

Pupillary light response

Normal

Poor

None

Normal

Intraocular pressure

Normal

Normal

Elevated

Normal

Smear

Causative organisms

No organisms

No organisms

Organisms found only in corneal ulcers related to infection

*Other less common causes of red eyes include endophthalmitis, foreign body, episcleritis, and scleritis. † Acute anterior uveitis. ‡ Angle-closure glaucoma.

2560

CHAPTER 423  Diseases of the Visual System  

FIGURE 423-4.  Involutional entropion. (From Palay DA, Krachmer JH. Primary Care Ophthalmology, 2nd ed. Philadelphia: Elsevier Mosby; 2005.)

FIGURE 423-5.  Ptosis of the right upper lid. (From Palay DA, Krachmer JH. Primary Care Ophthalmology, 2nd ed. Philadelphia: Elsevier Mosby; 2005.)

conjunctivitis. Conjunctivitis usually involves the entire conjunctiva, is associated with a discharge, and usually is not associated with pain (see Table 423-4).

PTOSIS (DROOPY EYELID)

Ptosis (Fig. 423-5) can be caused by a third nerve palsy, which usually is associated with diplopia and a reduction in elevation, depression, and medial movement of the pupil. Horner’s syndrome (see Fig. 424-5) is associated with a small pupil. With myasthenia gravis (Chapter 422), other typical features of muscle weakness are usually present or can be elicited. Some patients have mild mechanical ptosis, especially after eye surgery.

PROPTOSIS (EXOPHTHALMOS)

Proptosis, or a prominent globe, can be a manifestation of thyroid abnormalities, especially Graves’ disease (Chapter 226), in which proptosis is subacute and bilateral but sometimes asymmetrical (Fig. 423-6). An orbital pseudotumor can cause acute, usually unilateral proptosis, with severe pain, particularly with eye movement, and often with decreased vision. An optic nerve tumor causes chronic, unilateral proptosis associated with a slow onset of visual field loss. Acute cellulitis can be associated with unilateral proptosis, severe redness, and moderate to severe pain, commonly with sinusitis and an elevated white blood cell count.

SMALL PUPIL

A unilateral small pupil is best detected in dark conditions. Causes include Horner’s syndrome, associated with ptosis on the same side; the bilaterally small, poorly reacting pupils of tertiary syphilis (Argyll Robertson pupils), which accommodate with normal constriction to a near object; miotic drops (e.g., pilocarpine); traumatic iritis; uveitis; and recent eye surgery.

LARGE PUPIL

Any α-adrenergic or anticholinergic agent placed into the eye can cause a large pupil. With eye trauma, the iris sphincter muscle can be damaged, and an abnormally large pupil can result. Tears in the iris sphincter can sometimes be appreciated on slit lamp examination. Third nerve palsy may cause a dilated pupil associated with ptosis and decreased elevation, depression, and medial eye movement. Adie’s pupil (see Fig. 424-3) is an idiopathic, unilateral large pupil that is hypersensitive to weak cholinergic stimulation. Recent eye surgery, uveitis, closed-angle glaucoma, and traumatic iritis can cause a large pupil.

LEUKOKORIA

Leukokoria (white pupil) in a child is often a sign of retinoblastoma. However, any condition that changes transmission of ambient light to reflection of ambient light through the pupil may cause this sign. Some of the more common nonretinoblastoma conditions presenting with leukokoria (Table 423-5) include cataracts, retinal detachment, persistent hyperplastic primary vitreous (a developmental anomaly of the vitreous resulting in intraocular fibrosis and retinal detachment), Coats’ disease (a developmental vascular malformation of the retina leading to retinal detachment), and ocular toxocariasis (a parasitic intraocular nematode infection, which leads to intraocular scarring and retinal detachment).

FIGURE 423-6.  Graves’ ophthalmopathy with characteristic exophthalmos and eyelid retraction.

TABLE 423-5  DIFFERENTIAL DIAGNOSIS OF LEUKOKORIA Retinoblastoma Cataract Persistent hyperplastic primary vitreous Retinopathy of prematurity (retrolental fibroplasia) Coats’ disease (retinal telangiectasia) Retinal detachment Toxocariasis Familial exudative vitreoretinopathy (FEVR)

Eyelid Abnormalities

ECTROPION AND ENTROPION

An ectropion is an out-turning of the lower lid (Fig. 423-7), typically with the inner part of the lower end visible between the eye globe and the lid. Causes include aging, scarring, a mass on the lower lid, and seventh nerve palsy. Common symptoms include burning, itching, tearing, and the sense of a foreign body. Treatment is symptomatic, unless the underlying cause can be surgically corrected. An entropion, which is an in-turning of the lower lid (see Fig. 423-4), is usually age-related and associated with irritation, burning, and a foreign body sensation. If it leads to trichiasis, in which the eyelashes rub or abrade the cornea, the lashes can be removed with forceps or surgery.

CHAPTER 423  Diseases of the Visual System  

2561

1

FIGURE 423-7.  Involutional ectropion. (From Palay DA, Krachmer JH. Primary Care Ophthalmology, 2nd ed. Elsevier Mosby, Philadelphia, 2005.)

FIGURE 423-9.  A lower lid stye (1). (From Palay DA, Krachmer JH. Primary Care Ophthalmology, 2nd ed. Philadelphia: Elsevier Mosby; 2005.)

FIGURE 423-8.  Bilateral chalazion in the upper eyelids.

CHALAZION

A chalazion (Fig. 423-8) is a localized lipogranulomatous inflammation that results when the eyelid reacts to the contents of a ruptured sebaceous (meibomian) gland. The retained, lipid-rich sebaceous material acts as a foreign material that stimulates a lipogranulomatous foreign body inflammatory reaction. A painless or slightly tender, poorly demarcated, nonmobile nodule forms under the eyelid skin. Most lesions resolve over days to weeks with warm compresses or without specific treatment. Occasionally, an ophthalmologist may inject steroids to reduce inflammation or debulk the foreign material by incision and drainage through the tarsal conjunctiva. Some individuals may have recurrent chalazion.

HORDEOLUM (STYE)

A hordeolum (stye) (Fig. 423-9) is an extremely painful abscess in a hair or eyelash follicle or in a sebaceous gland. Styes are usually self-limited infections that respond to warm compresses and topical antibiotics (e.g., bacitracin or erythromycin ointment or moxifloxacin or gatifloxacin drops). An ophthalmologist may perform incision and drainage if symptoms do not improve within 48 hours.

BLEPHARITIS

Blepharitis (Fig. 423-10), which is a nonspecific inflammation of the eyelid skin, is common, particularly in men. The condition is usually bilateral and symmetrical. Rosacea (Chapter 439) is the most common associated cutaneous condition, and Staphylococcus aureus is the most common infectious agent. If untreated, blepharitis becomes chronic and may lead to corneal and conjunctival inflammation (blepharoconjunctivitis). Ophthalmic antibiotic ointment (e.g., bacitracin or erythromycin) is more efficacious than eye drops, but systemic antibiotics (e.g., minocycline, 50 to 100 mg, or doxycycline, 100 mg, once daily; tetracycline, 250 mg twice daily; or erythromycin, 250 mg three times daily) are recommended if there is any evidence of inflammation of the cornea or conjunctiva.

FIGURE 423-10.  Staphylococcal blepharitis. The lid margins are very red and under high magnification demonstrate tiny ulcerations. (From Palay DA, Krachmer JH. Primary Care Ophthalmology, 2nd ed. Philadelphia: Elsevier Mosby; 2005.)

In seborrheic blepharitis, exfoliated keratinous debris accumulates along the eyelid margin, particularly at the follicles of the eyelashes, and irritates the conjunctiva. Treatment of this chronic condition is directed at mechanically removing the keratinous debris by scrubbing the eyelid and eyelashes daily with a mild detergent (“baby shampoo”) in warm water applied with a soft cloth.

BENIGN EYELID NEOPLASMS

Skin tags, also known as squamous papillomas, are the most common benign skin lesions. Other skin lesions include seborrheic keratitis, actinic keratitis, inverted follicular keratitis, and benign lesions of the eccrine and apocrine systems. Most of these benign lesions are cured by simple excision.

SEBACEOUS CARCINOMA

Sebaceous carcinoma originates from sebaceous glands either in the tarsal plate (meibomian gland) or associated with eyelashes (glands of Zeis) and is capable of producing widespread metastasis resulting in death.4 Muir-Torre syndrome is a syndrome of sebaceous tumors associated with visceral malignancy. Except for chronic, unilateral blepharitis, owing to the peculiar manner of spread of this tumor in the plane of the skin epithelium (pagetoid spread) without causing the formation of nodules, few symptoms occur early in the course of the disease. The tumor may progress to involve the tarsal conjunctiva, the bulbar conjunctiva, and even the corneal epithelium. A characteristic sign is regional loss of eyelashes. When the mass thickens, it may have the appearance of a chalazion, and a history of multiple chalazia in the same region of the eyelid is suggestive of sebaceous carcinoma. Treatment is surgical removal. Surgical margins are difficult to estimate because of the intraepithelial extension of the tumor. Topical mitomycin C has been suggested as treatment for pagetoid invasion of the conjunctiva. In

2562

CHAPTER 423  Diseases of the Visual System  

advanced cases, removal of the eyelids, eye, and orbital contents (exenteration) may be necessary.

BASAL CELL CARCINOMA

Basal cell carcinoma (Fig. 423-11), which originates from the basal cell layer of the epithelium, is a common cutaneous malignancy (Chapter 203). The lesion, which usually is asymptomatic, is often a well-demarcated, elevated nodule that may have a central region of ulceration and fine cutaneous vascular channels (telangiectasias). A common benign cutaneous lesion, sometimes confused clinically with basal cell carcinoma, is seborrheic keratosis (Chapter 440), which tends to be soft and appear hyperpigmented; the most common site is the lower eyelid, especially in the nasal quadrant. Basal cell carcinoma, particularly near the medial canthus, may extend posteriorly into the soft tissues of the orbit. Imaging before surgical excision for medial canthal lesions may be necessary to determine the true extent of the tumor. Basal cell carcinoma is treated by surgical excision, using Mohs’ technique with intraoperative histologic evaluation to determine adequate margins of excision, if possible.5

FIGURE 423-11.  A typical nodular basal cell carcinoma. (From Palay DA, Krachmer JH. Primary Care Ophthalmology, 2nd ed. Philadelphia: Elsevier Mosby; 2005.)

Metastasis is extremely rare. With early detection and adequate excision of the local lesion, the prognosis is excellent.

EYELID SQUAMOUS CELL CARCINOMA

Eyelid squamous carcinoma, which is much less common than basal cell carcinoma, arises from the surface squamous epithelium. Ultraviolet light exposure is the major risk factor. In contrast to basal cell carcinoma, squamous cell carcinoma can metastasize, most often to regional lymph nodes. Treatment is surgical excision. Except in rare circumstances, such as in immunosuppressed patients or patients with xeroderma pigmentosa, the prognosis is excellent.

Ocular Surface Abnormalities

DRY EYES

Even minor disturbances in the tear film can cause itching, burning, pulling, and transient changes in vision. Dry eyes that cause conjunctival hyperemia without purulent discharge particularly disturb some patients. Paradoxically, decreased tearing can result in irritation and secondary increased (reflux) tearing. Most daily tear production is not by the lacrimal gland but by small collections of lacrimal glands, mucus-producing glands, and sebaceous glands located throughout the conjunctiva, eyelid, and anterior orbital soft tissue. Over time, particularly in women, production of tear film diminishes. Because tear film production is lower during sleep, patients often note symptoms on awakening followed by slow resolution over minutes or hours. Wind and low humidity environments, such as in commercial airliners, can exacerbate symptoms. The reduction in aqueous components of tears is often associated with a compensatory increase in mucus production, which tends to blur vision until the patient blinks or uses supplemental tears. These symptoms are particularly prominent in persons who have rheumatoid arthritis (Chapter 264), Sjögren’s syndrome (Chapter 268), Stevens-Johnson syndrome (Chapter 440), and ocular cicatricial pemphigoid (Fig. 423-12). Treatment is not definitive and is rarely satisfactory. No medication increases the production of tears. Low-viscosity artificial tears (e.g., poly­ ethylene glycol 400 0.4%), which do not tend to blur vision but have a short duration of action, are best used during visually important tasks.

Symptoms of Ocular Irritation Consider non-tear film related problems

No

Fluorescein tear break-up time 3 Yes Sjögren’s syndrome

Meibomian gland pathologic signs 1. Orifice metaplasia 2. Acinar atrophy 3. Reduced expressible meibum Yes Meibomian gland disease

No Non-Sjögren’s syndrome

FIGURE 423-12.  Diagnostic algorithm for ocular irritation. (Modified from Pflugfelder SC, Tseng SC, Sanabria O, et al. Evaluation of subjective assessments and objective diagnostic tests for diagnosing tear-film disorders known to cause ocular irritation. Cornea. 1998;17:38-56.)

CHAPTER 423  Diseases of the Visual System  

High-viscosity tears (e.g., carboxymethylcellulose sodium) have a longer duration of action but tend to blur vision; they are best used at bedtime to maintain lubrication of the ocular surface during sleep. When artificial tears do not control symptoms, occlusion of the nasolacrimal duct with synthetic plugs or permanent surgical occlusion tends to retain the tears that are produced. Anti-inflammatory drugs (e.g., cyclosporine 0.05% drops, every 12 hours indefinitely) can preserve glandular tissue that may be affected by local inflammation. For patients with systemic disease associated with dry eyes, effective treatment of the systemic disease sometimes improves the eye abnormalities.

PINGUECULA AND PTERYGIUM

A pinguecula (Fig. 423-13) consists of a limbal (at junction of cornea and sclera) and bulbar conjunctival degenerative process caused by ultraviolet light damage to the subepithelial tissue. It is very common and rarely causes symptoms. If the supportive tissue degeneration extends into the cornea, it becomes a pterygium (Fig. 423-14), which may cause visual changes and require surgical excision. About 2 to 10% with a pterygium have a coexisting squamous carcinoma, which often is clinically unsuspected and diagnosed only by histopathologic examination.6

RECURRENT EROSION

Recurrent erosion of the cornea usually is a delayed reaction to minor traumatic corneal abrasion. The abrasion heals abnormally, resulting in a weakness of the epithelial attachment to underlying tissue. Weeks to months to years later, the patient is awakened in the middle of the night with extreme ocular pain on opening the eyelids. The epithelium has become “stuck” to the overlying upper lid and is mechanically abraded. The condition is treated with hyperosmotic drops and ointment. There is a tendency to recurrence.

2563

ACCIDENTAL TRAUMA

With ocular trauma, many tissues of the eye can be easily disrupted, and the effects of trauma may not be manifest for months or even years after the episode of trauma. If the traumatic episode disrupts the eye wall (cornea and sclera), surgical repair is necessary, usually urgently. If the eye wall is intact, surgical treatment is often not necessary, at least initially.

CORNEAL ABRASION

Shearing trauma or hypoxia associated with overwearing of contact lenses may cause corneal abrasion, one of the most common forms of ocular injury. Symptoms are often intense and intolerable. Healing (i.e., re-epithelialization) of the cornea occurs within 24 to 48 hours. Rust from metallic fragments is toxic to the epithelium and should be removed. Fungal keratitis may complicate injuries from fingernails or vegetable matter, such as tree branches. Treatment usually consists of a topical antibiotic (e.g., erythromycin ointment, four times daily for 10 days) to prevent bacterial keratitis. Subsequent scarring usually does not occur unless deeper structures, such as Bowman’s membrane, are affected. Topical anesthetics never should be prescribed to control pain because they increase the risk for microbial keratitis and scarring and may delay healing.

MAJOR OCULAR TRAUMA

Hyphema (Fig. 423-15) is hemorrhage into the anterior chamber caused by blunt trauma. If the patient is in an intensive care unit and supine, the blood will distribute uniformly over the iris to cause the appearance of increased pigmentation of the iris (heterochromia iridis). If the patient has been sitting, the blood may settle by gravity to form an aqueous-blood interface with the blood in the dependent portion of the anterior chamber. Hyphema, which is a sign of serious ocular damage, may lead to secondary glaucoma and blood staining of the cornea. It requires prompt evaluation by an ophthalmologist. The most common site of rupture of a globe is at the limbus (junction of cornea and sclera), where a pigmented mass may be noted. The mass may be either a blood clot or an anteriorly displaced uveal tract (usually iris). Any manipulation of the globe may force the remaining intraocular tissue through the wound and may make the injury irreparable. Surgical repair is usually indicated. Cataract and retinal detachment are not common except in severe accidental trauma. A unilateral cataract or unilateral glaucoma may occur decades after the injury, even when an injury is too minor to be recalled. Traumatic cataract and traumatic glaucoma are treated in the same manner as other forms of these conditions.

  INFLAMMATORY EYE DISORDERS

Uveitis FIGURE 423-13.  Pinguecula. These lesions are found at the 3-o’clock and 9-o’clock

positions and are extremely common, especially in older patients. (From Palay DA, Krachmer JH. Primary Care Ophthalmology, 2nd ed. Philadelphia: Elsevier Mosby; 2005.)

FIGURE 423-14.  Pterygium. These lesions are found in the horizontal meridian, most common nasally. (From Palay DA, Krachmer JH. Primary Care Ophthalmology, 2nd ed. Philadelphia: Elsevier Mosby; 2005.)

Inflammation of any part or parts of the uveal tract (iris, ciliary body, and choroid) may be called anterior or posterior uveitis, iritis, iridocyclitis, or choroiditis. Symptoms include a red eye (see Table 423-4), decreased vision, and photophobia. The inflammation is chronic, and a cause is rarely found. However, uveitis accompanies many autoimmune diseases, often without correlation with the activity of the systemic inflammation. Anterior

FIGURE 423-15.  Hyphema following cataract surgery. (Courtesy of Dr. Myron Yanoff.)

2564

CHAPTER 423  Diseases of the Visual System  

uveitis or conjunctivitis is nearly universal in patients with reactive arthritis (Chapter 265). About 25% of patients with ankylosing spondylitis (Chapter 265) develop acute, recurrent anterior uveitis. Two to 12% of patients with inflammatory bowel disease (Chapter 141) develop anterior uveitis, which is also common with psoriatic arthritis but not with psoriasis alone (Chapters 265 and 438). Treatment with topical corticosteroids (e.g., prednisolone acetate 1%, one drop in the affected eye or eyes every 1 to 6 hours while awake) is usually sufficient to control the ocular disease.

Endophthalmitis Endophthalmitis is extensive inflammation within the eye from any cause. Most cases of endophthalmitis involve a breach in the eye wall (cornea and sclera), associated with either accidental trauma (incidence of approximately 5%) or surgical procedures (incidence of approximately 39° C or 102° F) with purulent nasal discharge or facial pain for at least 3 to 4 consecutive days; or the onset of worsening symptoms more than 5 days after the onset of an apparent viral upper respiratory tract infection. In patients who meet one or more of those three criteria, the Infectious Diseases Society of America1 recommends empirical antibiotic therapy, preferably with amoxicillin-clavulanate (875 mg/125 mg orally twice daily, increasing to 2000 mg/125 mg orally twice daily in patients with fever greater than 39° C or 102° F, immunocompromise, or recent antibiotic use). In patients who are allergic to penicillin, the best alternatives are doxycycline (100 mg orally twice daily) or a fluoroquinolone (e.g., levofloxacin 500 mg orally daily or moxifloxacin 400 mg orally daily); by comparison, macrolides, trimethoprim-sulfamethoxazole, and second- and third-generation oral cephalosporins are not recommended because of high levels of resistance. The usual course of therapy is 5 to 7 days, regardless of the medication chosen. Intranasal saline irrigations, using either physiologic or hypertonic saline, may be a useful adjunct in patients with acute bacterial rhinosinusitis, A4  but neither topical decongestants nor antihistamines are useful. If patients worsen despite 72 hours of treatment or do not improve after 5 to 7 days, further evaluation should include CT to localize the infection, and cultures— either by direct sinus aspiration or endoscopically guided cultures of the middle meatus; other cultures are unreliable. Chronic sinusitis is a term that encompasses multiple pathophysiologic mechanisms and implies a prolonged course of sinus symptoms that have been refractory to symptomatic treatment over a period of at least 3 months. Chronic sinusitis presents with nasal congestion, nasal drainage, facial pressure, and sometimes anosmia. Unlike acute sinusitis, patients with chronic sinusitis do not typically have fever or severe headache. Corticosteroids, either in a topical spray (e.g., triamcinolone acetonide, two 55-µg sprays to each side of the nose every day; mometasone furoate, two 50-µg sprays to each naris every day; fluticasone propionate, two 50-µg sprays to each naris every day; or budesonide, two 32-µg sprays to each naris every day) for 6 weeks or delivered in an oral tapering dose (prednisone 40 mg per day for 5 days, followed by 30 mg per day for 5 days, followed by 20 mg per day for 5 days, followed by 10 mg per day for 5 days; or methylprednisolone 4 mg tablets beginning with 24 mg the first day and tapering by 4 mg each subsequent day for 6 days) are the mainstay of treating the symptoms of chronic rhinosinusitis. Endoscopically obtained cultures of the middle meatus can help define which patients may improve with culture-guided antibiotic treatment. Antifungal agents, including itraconazole in an oral or aerosolized form and amphotericin B in an aerosolized form, do not appear beneficial in the treatment of typical chronic sinusitis. A5 

Allergic Rhinitis

Allergic rhinitis (Chapter 251) responds to various antihistamines, such as diphenhydramine hydrochloride (25 to 50 mg every 4 to 6 hours), loratadine (5 mg twice a day or 10 mg a day), cetirizine hydrochloride (10 mg a day), fexofenadine hydrochloride (60 mg twice a day or 120 mg/day), and topical nasal steroids, including triamcinolone acetonide (two sprays [55 µg] to each side of the nose every day), mometasone furoate (two sprays [50 µg] to each naris every day), fluticasone propionate (two sprays [50 µg] to each naris every day), and budesonide (two sprays [32 µg] to each naris every day). Oral steroids such as prednisone and methylprednisolone in various doses are sometimes useful as well. Allergic desensitization is sometimes recommended when a discrete allergen elicits a strong reaction in a patient. Allergic desensitization, through injections or the sublingual route, may specifically be beneficial for some inflammatory disorders, such as allergic rhinitis. Topical nasal steroids may be of benefit when acute rhinosinusitis is confirmed by either radiography or nasal endoscopy. A6  However, topical intranasal steroids are also not of proven benefit for treating sinusitis diagnosed by primary care physicians on purely clinical grounds A7  or for treating the common cold. A8  Topical nasal steroids are commonly used with good clinical effect in patients with chronic rhinosinusitis.

Surgical Therapy

Patients who have severe symptoms, who fail to respond to therapy, or who have unusual, or resistant, or recurrent infections should be sent to a specialist for further evaluation and treatment (Table 426-2). Surgery is recommended in patients with benign neoplasms, mucoceles, juvenile nasopharyngeal angiofibroma, and some types of malignancies. Surgery can correct septal deviations and anatomically related nasal obstruction. Surgery on the inferior turbinates may be beneficial for refractory rhinitis. Functional endoscopic surgery, which is designed to preserve mucociliary function and is performed with endoscopes through the nostril without skin incisions, can be useful for recurrent acute sinusitis and chronic rhinosinusitis. A9 

,

Nasal Polyps During the evaluation of symptoms of rhinitis or sinusitis, the physical examination may reveal nasal polyps. Nasal polyps often present with symptoms of

CHAPTER 426  Approach to the Patient with Nose, Sinus, and Ear Disorders  

2589

TABLE 426-2  WHEN TO REFER A PATIENT WITH PRESUMED BACTERIAL RHINOSINUSITIS TO AN ENT SPECIALIST • Temperature >39°C (>102° F); orbital edema; severe headache, visual disturbance, altered mental status, meningeal signs • Failure to respond to more than two courses of antimicrobial therapy • Nosocomial infection, anatomic abnormalities • Immunocompromise or multiple comorbidities • Unusual or resistant pathogens • Fungal sinusitis or granulomatous disease • Recurrent episodes suggesting chronic sinusitis Adapted from Chow AW, Benninger MS, Brook I, et al. IDSA clinical practice guideline for acute bacterial rhinosinusitis in children and adults. Clin Infect Dis. 2012;54:e72-e112.

nasal blockage and anosmia along with typical symptoms of rhinitis. When polyps are present, the nasal congestion is often unrelenting. Sometimes, patients with prolonged symptoms will present with a visible mass in their nostril. Rarely, facial asymmetry or orbital involvement will be the presenting sign of long-ignored nasal polyps. Patient with nasal polyps may be more likely to complain of facial or ear pain than patients with rhinitis without polyps.2 Nasal polyps typically begin near the ethmoid sinuses in the middle meatus and extend into the nose, where they block the nasal airway and/or the sinuses. Nasal polyps may be caused by chronic inflammation and also often occur as part of a rare metabolic disorder of arachidonic acid metabolism triggered by exogenous aspirin intake—known as aspirin-exacerbated respiratory disease. Also known as Samter’s triad, patients with this syndrome have asthma that is exacerbated by aspirin ingestion, a skin rash precipitated by aspirin, and often have difficult-to-control chronic nasal polyposis. This constellation of symptoms is thought to be caused by inflammation elicited by leukotrienes, which are upregulated by the prostaglandin blockade caused by aspirin and sometimes by other nonsteroidal anti-inflammatory drugs. Human papillomavirus (Chapter 373) may cause an inverted papilloma, which presents as a polyp causing unilateral nasal obstruction. This initially benign neoplasm responds to surgical excision but can transform to frank malignancy. Polyps are also seen in patients with cystic fibrosis, especially patients with the delta F508 mutation (Chapter 89). They are also seen in allergic fungal sinusitis, which is manifested by an elevated IgE level, positive fungal cultures (usually for aspergillosis), Charcot-Leyden crystals on histopathology, characteristic densities on CT, and nasal polyposis that is often, but not always, unilateral. Antral choanal polyps may extend into the nasal cavity or nasopharynx and cause obstruction. Nasal polyps will be visible in a careful examination (see Fig. 426-3), and their extent can be shown on a CT scan (Fig. 426-5). Unilateral nasal polyposis is suggestive of antral choanal polyps, malignancy, inverted papilloma, or allergic fungal sinusitis; early biopsy is recommended. Benign inflammatory nasal polyps frequently respond to oral steroids, either in a tapered burst dose or, in rare cases, in small amounts of titrated daily oral steroids such as prednisone (40 mg per day for 5 days, followed by 30 mg per day for 5 days, followed by 20 mg per day for 5 days, followed by 10 mg per day for 5 days) or methylprednisolone (beginning with 24 mg the first day and tapering by 4 mg each subsequent day for 6 days). A10  Topical steroids are also efficacious for treating nasal polyps. A11  Surgery for benign nasal polyposis can improve symptomatic control and reduce the need for oral steroids. Surgery is always recommended for inverted papillomas, antral choanal polyps, and mucoceles, and surgery is likely to be helpful if acute sinusitis has caused central nervous system complications such as brain abscess (Chapter 413), meningitis (Chapter 412), epidural abscess, subdural abscess, or orbital abscess. Occasionally, surgery will be required when an untreated and aggressively growing polyp causes orbital or skull base erosion. Allergic fungal sinusitis is often treated with a combination of surgery, corticosteroids, and sometimes immunotherapy.

Epistaxis For a patient with epistaxis, it is first critical to determine the severity of the blood loss. Persistent bleeding may result from warfarin, antiplatelet agents, or any underlying platelet (Chapters 172 and 173) or clotting deficiency (Chapter 174). Physical examination should focus on inspection of the anterior septum, which is the most frequent point of origin for epistaxis. Frequently, dilated blood vessels on the caudal septum can be seen with anterior

FIGURE 426-5.  Computed tomography showing bilateral nasal polyposis of a

chronic nature.

FIGURE 426-6.  Dilated nasal vessels and crusting typical of a patient with

epistaxis.

rhinoscopy (Fig. 426-6). The combination of unilateral otitis media, epistaxis, nasal congestion, and a neck mass would be concerning for nasopharyngeal carcinoma. Rare tumors that can arise with bleeding include juvenile nasopharyngeal angiofibromas in male patients. Epistaxis can be treated by local pressure, packing (using nasal sponges, balloons, or by 1 2-inch by 72-inch gauze impregnated with petroleum jelly), humidification, and hydration. Hospitalization and transfusion are rarely required. Offending medications should be reduced in dose or discontinued temporarily if possible. Topical vasoconstrictive medication such as oxymetazoline spray, two sprays on each side of the nose every 12 hours for 3 days, can help prevent persistent epistaxis. Occasionally, lasers or other types of cautery are used to improve the problem. At times, surgical arterial clipping or interventional neuroradiologic arterial occlusion can address a specific bleeding area.

  EAR PAIN DEFINITION

Ear pain (Table 426-3) is discomfort perceived by a patient in the area of the temporal bone. Although the discomfort can often be localized by the patient, at times the cause of the discomfort may in fact be distant from the site where the pain is felt. This referred pain can be due to problems in the oral cavity, oropharynx, hypopharynx, or larynx.

2590

CHAPTER 426  Approach to the Patient with Nose, Sinus, and Ear Disorders  

Pars flaccida

TABLE 426-3  CAUSES OF OTALGIA MIDDLE EAR

UPPER AERODIGESTIVE TRACT

Likely

Otitis externa Herpes zoster oticus Chondritis Foreign body

Acute otitis media Acute eardrum perforation Barotrauma Chronic otitis media with impending complication

Tonsillitis Tonsil abscess Deep neck abscess Tumor (especially base of the tongue, tonsil, hypopharynx, larynx, nasopharynx)

Unlikely

Malignant otitis externa Tumor

Tumor

CAUSES OF OTALGIA

EXTERNAL EAR

Short process of malleus Malleus long process Light reflex Pars tensa

FIGURE 426-7.  A normal tympanic membrane. (From Dhillon RS, East CA, eds. Ear, Nose and Throat and Head and Neck Surgery, 2nd ed. Edinburgh: Churchill Livingstone; 1994:2.)

PATHOBIOLOGY

The ear is well supplied with sensory nerves and is positioned on the side of the skull. The ear is divided into the outer ear, or pinna, and the ear canal; the middle ear, which encompasses the tympanic membrane and ossicles (Fig. 426-7); and the inner ear, which consists of the cochlea and the vestibular canals, including the utricle and saccule. In general, otalgia is due to problems in the outer or middle ear. The trigeminal nerve innervates the anterior-superior quadrant of the pinna, whereas the C2 and C3 cervical cutaneous nerves innervate the rest of the majority of the outer ear. However, there are contributions by the 9th and 10th nerves in the ear canal and even a small patch of sensory innervation by the 7th nerve in the posterior superior ear canal. It is the overlap in distribution of the 9th and 10th cranial nerves that establishes the anatomic basis for referred otalgia in diseases of the oral cavity, oropharynx, and larynx. Therefore, ear pain may be due to inflammatory conditions of the skin of the outer ear, the ear canal, or the middle ear, or it may be due to disease processes unrelated to the ear itself.

CLINICAL MANIFESTATIONS

Patients with ear pain often have complaints referable directly to the ear itself. In cases of otitis externa, frankly obvious erythema and swelling of the skin of the ear canal may be present. Even minute physical manipulation of the ear may be excruciating. In chondritis of the pinna, which may be related to rheumatologic disorders, infection, or trauma, the entire pinna may be swollen and painful. Hearing loss accompanying otalgia may indicate middle ear disease, especially otitis media. Patients sometimes complain of pain in the ear after air travel or driving from a mountainous region. Quick changes in pressure, such as encountered in scuba diving, may indicate barotrauma (Chapter 94), in which the eustachian tube is unable to compensate rapidly enough for the changes in pressure that are encountered. Pain may also be a post-traumatic symptom from relatively minor percussion injury, more severe head trauma, or percussion injury related to a blast. Pain related to noise exposure may also indicate damage to the middle ear or even the inner ear. Deep-seated boring pain over the temporal area accompanied by retroorbital pain can be due to petrous apex disease, including petrous apicitis.

DIAGNOSIS

History

A patient with ear pain should be asked to reveal the location of the discomfort, the duration of the symptoms, and any activities related to onset of the condition. As an example, recent swimming would make otitis externa (“swimmer’s ear”) more likely, whereas a recent upper respiratory infection with hearing loss would suggest otitis media. Questions should address possible hearing loss, vertigo, otorrhea, hoarseness, voice change, dysphagia, odynophagia, dyspnea, hemoptysis, hematemesis, and weight loss. A social history with specific concentration on tobacco and alcohol use should be obtained. A possible family history of upper aerodigestive tract and nasopharyngeal carcinoma should be sought. A past surgical history can reveal distant ear or throat surgery.

Physical Examination

A complete head and neck examination, including general assessment for trauma and a basic eye examination, is required. The outer ear and pinna should be examined first. The ear canal should first be palpated and then

FIGURE 426-8.  Otoscopic appearance in otitis media with effusion. The handle and short process of the malleus are brought into relief by retraction of the eardrum. There is a slightly yellow appearance of the eardrum related to the middle ear effusion. (From Dhillon RS, East CA, eds. Ear, Nose and Throat and Head and Neck Surgery, 2nd ed. Edinburgh: Churchill Livingstone; 1994:7.)

inspected. An otoscope with a pneumatic bulb attachment is critical to establish the presence or absence of a middle ear effusion. Inspection of the tympanic membrane should be accomplished with notations made about patency and perforation, translucency of the eardrum, position and definition of the malleus, and the eardrum’s mobility with the ear canal sealed and a puff of air delivered by the pneumatic bulb. Abnormalities may be caused by infection (Fig. 426-8) or barotrauma (Fig. 426-9). Examination with a 512-Hz tuning fork should be performed to determine lateralization of the sound (Weber test) and whether air conduction is superior to bone conduction (Rinne test). Facial nerve function should be assessed (Chapter 396) by determining whether the patient can raise the eyebrows, close the eyes, wrinkle the nose, and purse the lips. The presence or absence of nystagmus should be recorded. Inspection of the nose, oral cavity, oropharynx, and neck should be accompanied by cranial nerve examination (Chapter 396). Palpation of the tongue and tonsils is especially important if the ear pain is intense and persistent. A careful neck examination should be performed to look for masses. Oral cavity infections (Chapter 425), such as a peritonsillar abscess or severe tonsillitis, may arise as ear pain, and the physical examination should reveal trismus, erythema, mass effect, and other common signs of pharyngitis.

Laboratory

An audiogram can assess hearing loss (Chapter 428). A tympanogram measures compliance of the middle ear system and is an accurate method for diagnosis of otitis media. Cultures are rarely performed because they require tympanocentesis, and cultures of the external ear can reveal a vast variety of organisms that are often treated empirically with antibiotics. If a fever and middle ear effusion are present and neck stiffness is found on physical examination, lumbar puncture may rarely be recommended.

CHAPTER 426  Approach to the Patient with Nose, Sinus, and Ear Disorders  

2591

FIGURE 426-9.  Blood in the middle ear (hemotympanum). Causes include otitic barotrauma, secretory otitis media, and a high jugular bulb. (From Dhillon RS, East CA, eds. Ear, Nose and Throat and Head and Neck Surgery, 2nd ed. Edinburgh: Churchill Livingstone; 1994:26.)

Imaging

In general, imaging is indicated if complications of acute or chronic otitis media are suspected or to look for occult causes of otalgia in the upper aerodigestive tract. If a patient is suspected of having meningitis, epidural or subdural abscess, brain abscess, or sagittal sinus thrombosis, imaging is mandatory. Imaging is also useful for operative planning in patients with chronic otitis media or (rarely) to evaluate for the presence of tumors in the middle or external ear.

FIGURE 426-10.  Otitis externa. Otitis externa in a patient’s left ear with the tympanic membrane in the distance. Exudate and erythema are present. The ear canal is quite painful, and a wick may be necessary to maintain patency of the external auditory canal.

rashes. A12  A13  Interestingly, the natural history of acute otitis media is acute perforation of the eardrum, which often results in otorrhea and relief of pain. Most middle ear effusions clear spontaneously within 3 months regardless of whether they are treated. Most perforations of an eardrum caused by trauma heal without surgical intervention, but if an eardrum perforation persists for more than about 3 months, surgical closure and the use of tympanoplasty with or without mastoidectomy can be contemplated. Chronic draining perforations, especially if located in the posterior-superior quadrant of the tympanic membrane, may portend the presence of cholesteatoma and may require tympanomastoid surgery. In patients in whom herpes zoster is suspected, acyclovir can be started at 800 mg by mouth five times per day for 7 days, with or without prednisone (Chapter 375). Intracranial complications of otitis media often need to be addressed surgically. ,

Differential Diagnosis

Otitis externa, which is an infection of the skin of the ear canal, is often due to manipulating the ear after swimming or trying to scratch an ear canal that itches because of skin irritation. Patients exhibit erythema of the canal skin and extreme pain on manipulation of the ear canal. In the presence of concomitant cranial neuropathies, especially in diabetic or otherwise immunocompromised patients, malignant otitis externa with osteomyelitis should be suspected. Inspection of the tympanic membrane may reveal fluid consistent with otitis media; the tuning fork examination should support the presence of conductive hearing loss. Vesicles on the conchal portion of the pinna, especially when accompanied by facial nerve paralysis, strongly suggest herpes zoster oticus with Ramsay Hunt syndrome (Chapter 375). Perforation of the eardrum suggests either acute or chronic otitis media, traumatic perforation, or possibly cholesteatoma (Chapter 428) if the perforation is in the posterior-superior quadrant. Chronic draining otorrhea of long standing with a deep boring pain and perforation of the tympanic membrane suggests a complication of otitis media. If findings on ear and cranial nerve examination are negative but the patient’s complaints of otalgia are persistent, special effort needs to be made to visualize the upper aerodigestive tract, including the nasopharynx, oral cavity, oropharynx, larynx, and hypopharynx, to be sure that infection or tumor is not present in these hard-to-examine areas. MRI can be very useful in these cases.

TREATMENT  Otitis externa is often treated with office suctioning of debris under a microscope and the application of antibiotic drops (ciprofloxacin, tobramycin, neomycin, polymyxin B), with or without hydrocortisone in various combinations.3 Frequently, a small wick or sponge is placed in the ear canal to help maintain patency of the canal and allow facile application of the medications (Fig. 42610). For otitis media, oral antibiotic treatment is directed at eradicating Haemophilus influenzae, Moraxella catarrhalis, Streptococcus pneumoniae, and Staphylococcus aureus with amoxicillin or erythromycin as for sinusitis. The benefit is notable for children 2 years or younger with bilateral otitis media and for older children with otitis plus otorrhea, whereas other patients can be observed without antibiotics. In general, antibiotics provide somewhat better short-term outcomes but at the expense of significantly more diarrhea and

Grade A References A1. Kenealy T, Arroll B. Antibiotics for the common cold and acute purulent rhinitis. Cochrane Database Syst Rev. 2013;6:CD000247. A2. Lemiengre MB, van Driel ML, Merenstein D, et al. Antibiotics for clinically diagnosed acute rhinosinusitis in adults. Cochrane Database Syst Rev. 2012;10:CD006089. A3. Garbutt JM, Banister C, Spitznagel E, et al. Amoxicillin for acute rhinosinusitis: a randomized controlled trial. JAMA. 2012;307:685-692. A4. Wei CC, Adappa ND, Cohen NA. Use of topical nasal therapies in the management of chronic rhinosinusitis. Laryngoscope. 2013;123:2347-2359. A5. Orlandi RR, Smith TL, Marple BF, et al. Update on evidence-based reviews with recommendations in adult chronic rhinosinusitis. Int Forum Allergy Rhinol. 2014;4(suppl 1):S1-S15. A6. Zalmanovici TA, Yaphe J. Intranasal steroids for acute sinusitis. Cochrane Database Syst Rev. 2009;4:CD005149. A7. Williamson IG, Rumsby K, Benge S, et al. Antibiotics and topical nasal steroid for treatment of acute maxillary sinusitis: a randomized controlled trial. JAMA. 2007;298:2487-2496. A8. Hayward G, Thompson MJ, Perera R, et al. Corticosteroids for the common cold. Cochrane Database Syst Rev. 2012;8:CD008116. A9. Smith TL, Kern R, Palmer JN, et al. Medical therapy vs surgery for chronic rhinosinusitis: a prospective, multi-institutional study with 1-year follow-up. Int Forum Allergy Rhinol. 2013;3:4-9. A10. Poetker DM, Jakubowski LA, Lal D, et al. Oral corticosteroids in the management of adult chronic rhinosinusitis with and without nasal polyps: an evidence-based review with recommendations. Int Forum Allergy Rhinol. 2013;3:104-120. A11. Rudmik L, Schlosser RJ, Smith TL, et al. Impact of topical nasal steroid therapy on symptoms of nasal polyposis: a meta-analysis. Laryngoscope. 2012;122:1431-1437. A12. Tähtinen PA, Laine MK, Huovinen P, et al. A placebo-controlled trial of antimicrobial treatment for acute otitis media. N Engl J Med. 2011;364:116-126. A13. Venekamp RP, Sanders S, Glasziou PP, et al. Antibiotics for acute otitis media in children. Cochrane Database Syst Rev. 2013;1:CD000219.

2592

CHAPTER 427  Smell and Taste  

GENERAL REFERENCES For the General References and other additional features, please visit Expert Consult at https://expertconsult.inkling.com.

427  SMELL AND TASTE

nasal mucosa. Specialized bipolar sensory cells in this region thrust short receptor hairs into the overlying mucosa to detect aromatic molecules as they dissolve. Like taste buds, the specialized receptor portion of the bipolar neuron undergoes continuous renewal, with turnover occurring approximately every 30 days. Thin axons of the bipolar neurons course through small holes in the cribriform plate of the ethmoid bone to form connections in the overlying olfactory bulb on the ventral surface of the frontal lobe. From there, second- and third-order neurons project directly and indirectly to the prepiriform cortex and parts of the amygdaloid complex of both sides of the brain, which represents the primary olfactory cortex.

PATHOBIOLOGY

ROBERT W. BALOH AND JOANNA C. JEN

Pathology

Millions of people suffer from disorders of taste and smell, but these disorders are often neglected because they are not fatal and, unlike abnormalities of vision and hearing, are not considered serious handicaps. Chemosensory disorders, however, often reduce the enjoyment and quality of life and are important to patients who suffer from them.

DEFINITION

The sensory receptor for taste, the taste bud, is made up of 50 to 150 cells arranged to form a pear-shaped organ. The lifespan of these cells is 10 to 14 days, and they are constantly being renewed from dividing epithelial cells surrounding the bud. Taste buds are located on the tongue, soft palate, pharynx, larynx, epiglottis, uvula, and upper third of the esophagus. The taste buds located on the anterior two thirds of the tongue and on the palate are innervated by the chorda tympani branch of the seventh cranial nerve. The ninth cranial nerve innervates the posterior third of the tongue. The ninth and tenth nerves innervate taste buds in the pharynx and larynx. Afferent signals from the taste buds project to the nucleus of the solitary tract in the medulla and then through a series of relays to the thalamus and postcentral somatosensory cerebral cortex (primary ipsilateral). Free nerve endings of the fifth cranial nerve are found on the tongue and in the oral cavity, and lesions involving these pathways can also alter taste perception. Olfactory receptors lie in a roughly dime-sized area of specialized pigmented epithelium that arches along the superior aspect of each side of the

Facial nerve (VII)

Disorders of taste interfere with digestion because taste stimulants alter salivary and pancreatic flow, gastric contractions, and intestinal motility. Smell also contributes to the anticipation and ingestion of food because much of what is tasted is derived from olfactory stimulation during ingestion and chewing. An inability to detect noxious tastes and odors can result in food or gas poisoning, particularly in elderly subjects. In the extreme, chemosensory disorders can lead to overwhelming stress, anorexia, and depression. Genes that encode chemoreceptor proteins belong to the G protein–coupled receptor superfamily, which accounts for up to 1% of mammalian genomes.1 Sequence diversity in these genes encodes unique structural motifs that bind to different ligands signaling different odors and tastes. Distinct and dedicated taste receptor cells express unique receptors to detect each of the five basic tastes: sweet (sensed by the heterodimers T1R1 and T1R3), umami (detected by the heterodimers T1R2 and T1R3), bitter (sensed by an estimated 30 T2Rs), sour (sensed by PKD2L1, with membrane-tethered carbonic anhydrase IV sensing carbonation), and salty (epithelial sodium channel). The taste receptor cells transform and transmit information to primary afferents through multiple cranial nerves (VII, IX, and X) that project to the solitary tract nucleus in the brain stem, with relay in the thalamus, and then onward to the primary cortex (Fig. 427-1).

Pathophysiology

Disorders of taste and smell can be divided into local, systemic, and neurologic categories (Table 427-1). The taste buds and the specialized receptor portion of the bipolar olfactory cells are constantly being renewed, and the

To brain

Trigeminal ganglion Brain stem

Chorda tympani nerve Solitary tract nucleus

Glossopharyngeal nerve (IX) Vagus nerve (X)

Superior laryngeal nerve

FIGURE 427-1.  Anatomy of peripheral taste pathways. Taste information is transmitted from the mouth and the pharynx through multiple cranial nerves that project to the solitary tract nucleus in the brain stem, with relay in the thalamus before reaching the cortex. (Copyright 1999-2000 David Klemm. Reproduced from Bromley SM. Smell and taste disorders: a primary care approach. Am Fam Physician. 2000;61:427-436, 438.)

CHAPTER 426  Approach to the Patient with Nose, Sinus, and Ear Disorders  

GENERAL REFERENCES 1. Chow AW, Benninger MS, Brook I, et al. IDSA clinical practice guideline for acute bacterial rhinosinusitis in children and adults. Clin Infect Dis. 2012;54:1041-1045. 2. Dietz de Loos DA, Hopkins C, Fokkens WJ. Symptoms in chronic rhinosinusitis with and without nasal polyps. Laryngoscope. 2013;123:57-63. 3. Rosenfeld RM, Schwartz SR, Cannon CR, et al. Clinical practice guideline: acute otitis externa. Otolaryngol Head Neck Surg. 2014;150(1 suppl):S1-S24.

2592.e1

2592.e2

CHAPTER 426  Approach to the Patient with Nose, Sinus, and Ear Disorders  

REVIEW QUESTIONS 1. A patient without other systemic disease or immunocompromise presents with otitis externa confined to the external auditory canal. Which of the following interventions would be recommended? A. The physician should prescribe an oral quinolone antibiotic. B. The physician should order a computed tomographic scan with contrast of the temporal bone. C. The physician should prescribe topical antibiotics. D. The physician should obtain a neurosurgical consult. E. The physician should recommend oral steroid medication. Answer: C  Otitis externa or “swimmer’s ear” usually responds to topical antibiotic treatment. Oral or systemic antibiotics are specifically not recommended unless the patient is immunocompromised or the infection is spreading to the pinna cartilage of the external ear outside of the external auditory canal. 2. A patient presents to your office with recurrent epistaxis. A crust on the anterior septum is identified on the side of greatest bleeding. There is no active bleeding during the visit. The best course of action is which of the following choices? A. The patient should be admitted to the hospital for interventional embolization of the internal maxillary artery. B. The patient should be electively scheduled for a computed tomographic angiogram. C. The patient should be given a prescription for a β-blocker. D. The patient should be queried about aspirin use and be asked to take an “aspirin holiday.” E. A bone marrow biopsy should be obtained. Answer: D  Patients with epistaxis are often on antiplatelet drugs for a variety of reasons. On intake history, use of nonsteroidal anti-inflammatory drugs should be specifically sought because curtailing antiplatelet medication will often eliminate troublesome epistaxis. 3. An adult patient presents with left ear pain but no hearing loss, beginning 4 weeks ago and partially controlled with narcotic pain medication. On physical examination, there is no evidence of otitis externa or otitis media. Which of the following choices is the best next step? A. A smoking history should be obtained, a neck examination should be performed, and the patient should be referred for upper airway endoscopic examination. B. An empirical trial of oral antibiotics should be prescribed for presumptive otitis media. C. An empirical trial of topical antibiotics should be prescribed for presumptive otitis externa. D. A magnetic resonance image of the brain and temporal bone, with contrast, should be ordered. E. The patient should be sent for a hearing test. Answer: A  Unexplained unilateral otalgia may be a referred pain from an upper aerodigestive tract lesion. Physical examination of the oral cavity, oropharynx, larynx, and hypopharynx should be performed. Tumors of the upper aerodigestive tract often present with an easily palpable, metastatic lymph node in the neck. Smoking, alcohol use, and human papillomavirus infection are risk factors for head and neck cancer.

4. A patient presents with nasal polyps visible on anterior rhinoscopy and also notes a history of asthma exacerbated by nonsteroidal antiinflammatory drug use. What is the best treatment recommendation? A. The patient should be scheduled for nasal surgery. B. The patient should be scheduled for computed tomographic imaging of the sinuses. C. The patient should be given a prescription for ciprofloxacin. D. The patient should be scheduled for a sweat chloride test. E. The patient should be given an oral prednisone taper. Answer: E  This patient likely has aspirin exacerbated respiratory disease, also called Samter’s triad or “aspirin allergy.” Oral corticosteroids can shrink nasal polyps and help control symptoms in patients with this disorder. 5. A patient presents to your office with a 4-day history of an upper respiratory infection that has prevented attendance at school. The physician should take which of the following steps? A. The history suggests chronic infection, and the patient should be given oral antibiotics. B. The history suggests acute infection, and the patient should be given oral antibiotics. C. The history is consistent with a viral upper respiratory infection, and symptomatic treatment should be recommended. D. The patient should be given a prescription for topical antifungal medication. E. None of the above Answer: C  Several meta-analyses have shown that oral antibiotics or topical antibiotics provide no meaningful improvement in sinusitis during the first 7 to 10 days of treatment. Supportive care is the best option; it should include decongestants, hydration, fever control with antipyretics, and patience in allowing the symptoms to resolve.

2593

CHAPTER 428  Hearing and Equilibrium  

TABLE 427-1  COMMON CAUSES OF LOSS OF TASTE AND SMELL TASTE

SMELL

Local

Radiation therapy, oral infections, dentures, dental procedures

Allergic rhinitis, sinusitis, nasal polyposis, upper respiratory infection

Systemic

Cancer, renal failure, hepatic failure, nutritional deficiency (vitamin B3, zinc), Cushing’s syndrome, hypothyroidism, diabetes mellitus, infection (viral), drugs (antirheumatic and antiproliferative, e.g., corticosteroids, cisplatin, carboplatin, cyclophosphamide, doxorubicin, and methotrexate)

Renal failure, hepatic failure, nutritional deficiency (vitamin B12), Cushing’s syndrome, hypothyroidism, diabetes mellitus, infection (viral hepatitis, influenza), drugs (nasal sprays, antihistamines, decongestants, antibiotics, and antirheumatic and antiproliferative drugs that affect taste)

Bell’s palsy, familial dysautonomia, multiple sclerosis

Head trauma, multiple sclerosis, Parkinson’s disease, Alzheimer’s disease, frontal tumor

Neurologic

process of renewal can be affected by nutritional, metabolic, and hormonal states as well as by therapeutic radiation, drugs, and age. For example, with interruption of mitosis by antiproliferative agents, return of normal taste function takes a minimum of 10 days, whereas return to normal olfactory function takes more than 30 days. Diuretics can block apical ion channels on a taste bud, and antifungal drugs inhibit cytochrome P-450–dependent enzymes at the level of the receptors. Numerous local conditions, such as colds and allergies, chronic sinusitis, and nasal polyposis, can influence the sense of smell by restricting airway patency. Accidental blows to the head can shear the fine axons of the bipolar olfactory neurons and result in loss of smell. Lesions of the fifth, seventh (chorda tympani), and ninth nerves can lead to disordered taste sensation. Olfactory and gustatory disturbances can serve as important diagnostic signs for focal neurologic lesions (e.g., frontal lobe tumors). Hallucinations of smell and taste occur in persons with epileptogenic lesions affecting the mesial temporal lobe and insular region, respectively. Finally, olfactory disturbances and hallucinations occur with a number of psychiatric illnesses (particularly depressive illness and schizophrenia).

CLINICAL MANIFESTATIONS

The most frequently encountered causes of loss of smell are local obstructive disease, viral infections, head injuries (Chapter 399), and normal aging (Chapter 25).2,3 Patients can lose their sense of smell not only from chronic allergies and sinusitis (Chapter 426) but also from the nasal sprays and drops that they use to treat these conditions. The most common causes of loss of the sense of taste are viral infections and drug ingestion, particularly antirheumatic and antiproliferative drugs (see Table 427-1). Many of the systemic disorders listed in Table 427-1 probably produce their effect by decreasing the rate of turnover of sensory receptors on the tongue and olfactory epithelia. Disturbances of smell and taste in malnourished patients may be due to specific deficiencies in vitamins and minerals, such as zinc. Viral illnesses, such as influenza (Chapter 364), viral hepatitis (Chapter 148), and allergic rhinitis (Chapter 251), are the most common causes of loss of both taste and smell.4 Multifocal neurologic disorders such as multiple sclerosis (Chapter 411) and traumatic head injuries (Chapter 399) can affect the central olfactory and gustatory pathways at multiple levels; as a result, abnormalities in taste and smell are common in such patients. Loss of smell is increasingly being recognized in the early stages of many neurodegenerative disorders, including Parkinson’s disease, Alzheimer’s dementia, motor neuron disease, and Huntington’s disease.5 An irritative lesion from a neoplastic, inflammatory, or demyelinating process may lead to a persistent disturbance rather than to a loss of taste.

DIAGNOSIS

Olfaction can be tested grossly at the bedside with a few easily recognized odors, such as coffee, chocolate, and the roselike aroma of the compound phenylethyl alcohol. Nasal irritants should be avoided. Each nostril is tested separately to determine whether the problem is unilateral or bilateral. Gustatory sensation is typically tested with weak solutions of sugar, salt, and acetic

acid or vinegar. The patient must keep the tongue protruded and respond to questions by nodding the head or by pointing to names of the tastes written on cards. The anterior two thirds and posterior third of the tongue should be tested separately.

TREATMENT  Treatment of olfactory dysfunction secondary to nasal disease is aimed at opening the air passageways while preserving the olfactory epithelium (Chapter 426). Intranasal steroids for rhinosinusitis (Chapter 426), antibiotics as needed for sinusitis, and therapies for seasonal allergies (Chapter 251) are useful in selected cases. Drugs known to affect taste or smell (see Table 427-1) should be discontinued for a trial. Vitamin and mineral therapies are of unproven benefit.

GENERAL REFERENCES For the General References and other additional features, please visit Expert Consult at https://expertconsult.inkling.com.

428  HEARING AND EQUILIBRIUM ROBERT W. BALOH AND JOANNA C. JEN

DEFINITION

The neural pathways subserving hearing and those most important for equilibrium and spatial orientation are anatomically proximate in much of their course from their end organs in the inner ear to their termination in the superior portion of the temporal lobe. Because of the close anatomic linkage, disorders that affect hearing often affect equilibrium, and vice versa. For this reason, they are considered together here.

PATHOBIOLOGY

Despite their anatomic propinquity, however, substantial pathophysiologic differences make clinical examination of the two systems different. The auditory system is physiologically relatively isolated, so that its function and dysfunction can be tested independently of other neural systems. The vestibular system, in contrast, has many close physiologic links with other neural systems (particularly the visual-oculomotor, somatosensory, and autonomic systems) and can be difficult to test in isolation of these other systems.

DIAGNOSIS

Abnormalities of the auditory system lead to only a few well-defined and unique symptoms (i.e., hearing loss or tinnitus). Abnormalities of the vestibular system can cause symptoms that mimic disorders of other neural structures. Such symptoms include dizziness, visual distortion (oscillopsia), imbalance, nausea, vomiting, and even syncope.

  DISORDERS OF THE AUDITORY SYSTEM DEFINITION

Anatomy and Physiology of Hearing

In normal hearing, sound waves are transmitted from the tympanic membrane through the three ossicles of the air-filled middle ear (air conduction) to the oval window and the basilar membrane of the fluid-sealed cochlea. The ossicles increase the gain from the tympanum to oval window about 18-fold, compensating for the loss that sound waves moving from air to fluid would otherwise suffer. In the absence of this system, sound may reach the cochlea by vibration of the temporal bone (bone conduction) but with much less efficiency (approximately 60-dB loss). Hair cells, tonotopically organized along the cochlear basilar membrane, detect the vibratory movement of that membrane and transduce vibration into nerve impulses. The nerve impulses are relayed by nerve cells that synapse at the base of hair cells and have their

CHAPTER 427  Smell and Taste  

GENERAL REFERENCES 1. Bachmanov AA, Bosak NP, Lin C, et al. Genetics of taste receptors. Curr Pharm Des. 2014;20:2669-2683. 2. Schubert CR, Cruickshanks KJ, Klein BE, et al. Olfactory impairment in older adults: five-year incidence and risk factors. Laryngoscope. 2011;121:873-878.

2593.e1

3. Doty RL, Kamath V. The influences of age on olfaction. Front Psychol. 2014;5:20. 4. Henkin RI, Levy LM, Fordyce A. Taste and smell function in chronic disease: a review of clinical and biochemical evaluations of taste and smell dysfunction in over 5000 patients at The Taste and Smell Clinic in Washington, DC. Am J Otolaryngol. 2013;34:477-489. 5. Doty RL. Olfaction in Parkinson’s disease and related disorders. Neurobiol Dis. 2012;46:527-552.

2593.e2

CHAPTER 427  Smell and Taste  

REVIEW QUESTIONS 1. A 53-year-old woman being treated for metastatic breast cancer complains of loss of taste. Which of the following is least likely to explain the taste loss? A. Radiation therapy B. Poor nutrition C. Chemotherapy D. Age E. Hormonal changes Answer: D  Although age can lead to minor deficits in taste because of decreased turnover in taste bud cells, age would not be likely to explain taste loss in this relatively young woman. A, B, C, and E can cause severe loss of taste, particularly in this clinical circumstance. 2. A 39-year-old woman presents with subacute onset of loss of smell. What is the most likely cause? A. Head trauma B. Viral infection C. Vitamin deficiency D. Multiple sclerosis E. Sinusitis Answer: B  Viral infections are the most common cause of loss of smell. The other conditions also can cause loss of smell but are less likely.

3. A 42-year-old man presents with subacute onset of hemifacial paralysis and persistent taste of sweetness. What clinical feature is not consistent with Bell’s palsy (VII cranial nerve palsy)? A. Hemifacial paralysis B. Loss of taste in the anterior two thirds of the tongue C. Persistent taste of sweetness D. Intact taste in the posterior third of the tongue E. Drooling or dry mouth with difficulty in eating Answer: C  A major branch of the mastoid segment of cranial nerve VII is the chorda tympani, which contains afferent taste fibers from the anterior two thirds of the tongue and is the anatomic basis for the loss of taste in the anterior two thirds of the tongue in idiopathic cranial nerve VII palsy. In contrast with loss of taste, a persistent sense of sweetness is a worrisome symptom for an irritative lesion, such as a neoplastic, inflammatory, or demyelinating lesion involving cranial nerve VII. 4. What is the molecular basis that allows us to taste carbonation in sodas, sparkling wines, and other carbonated drinks? A. G protein–coupled glutamate receptor for umami B. Carbonic anhydrase IV expressed on the surface of sour-sensing cells C. TAS1R heterodimers for sweetness D. TAS2R proteins for bitter taste E. Epithelial sodium channel receptor for salty taste Answer: B  Mountaineers who take acetazolamide (a carbonic anhydrase inhibitor) to prevent altitude sickness have long reported that carbonated beverages lacked pleasurable taste or tingle sensation. Patients treated with acetazolamide and other medications with carbonic anhydrase–inhibiting activities also experience taste disturbances.

2594

CHAPTER 428  Hearing and Equilibrium  

bodies in the spiral ganglion to the cochlear nucleus of the ipsilateral pontine tegmentum. The spiral cochlea mechanically analyzes the frequency content of sound. For high-frequency tones, only sensory cells in the basilar region are activated, whereas for low-frequency tones, all or nearly all sensory cells are activated. Therefore, with lesions of the cochlea and its afferent nerve, the hearing levels for different frequencies are usually unequal, typically resulting in better hearing sensitivity for low-frequency than for high-frequency tones. Within the brain stem, auditory signals ascend from the ventral and dorsal cochlear nuclei to reach the superior olivary nuclei of both sides. Thus, nervous system lesions central to the cochlear nucleus do not cause monaural hearing loss, and conversely, unilateral central lesions do not cause deafness. From these structures, the pathway projects by way of the lateral lemnisci to the inferior colliculi. Each inferior colliculus transmits to the other and to its ipsilateral medial geniculate body, which in turn sends the final projection to the transverse auditory gyrus lying in the superior portion of the ipsilateral temporal lobe. The normal ear can detect sound frequencies ranging between 20 and 20,000 Hz; the upper range drops off fairly rapidly with advancing age. The ear is most sensitive between 500 and 4000 Hz, which roughly corresponds to the frequency range most important for understanding speech. The hearing level in this range has several practical implications in terms of the degree of handicap and the potential for useful correction with amplification. A 30- to 40-dB hearing level in the speech range would impair normal conversation, whereas an 80-dB hearing level would make everyday auditory communication almost impossible (the social definition of deafness).

EPIDEMIOLOGY

About 5% of the world population suffers from disabling hearing loss (defined by the World Health Organization as greater than 40 dB in the better hearing ear in adults and greater than 30 dB in the better hearing ear in children). The prevalence of disabling hearing loss is twice as high in poorer countries compared with richer countries. The prevalence increases with every age decade,1 and it is higher in men than in women across all age decades. Hearing loss is independently associated with accelerated cognitive decline and incident cognitive impairment in community-dwelling older adults.

PATHOBIOLOGY

Localization of Lesions within the Auditory Pathways

Conductive hearing loss results from lesions involving the external or middle ear. It is typically characterized by an approximately equal loss of hearing at all frequencies and by well-preserved speech discrimination once the threshold for hearing is exceeded. Patients with conductive hearing loss can hear speech in a noisy background better than in a quiet background because they can understand loud speech as well as anyone. Sensorineural hearing loss results from lesions of the cochlea or auditory division of the eighth cranial nerve, or both. With sensorineural hearing loss, the hearing levels for different frequencies are usually unequal, typically resulting in better hearing for low- than for high-frequency tones. Patients with sensorineural hearing loss often have difficulty in hearing speech that is mixed with background noise and may be annoyed by loud speech. Three important manifestations of sensorineural lesions are diplacusis, recruitment, and tone decay. Diplacusis and recruitment are common with cochlear lesions; tone decay usually accompanies eighth nerve involvement. Central hearing disorders result from lesions of the central auditory pathways. As a rule, patients with central lesions do not have impaired hearing for pure tones, and they can understand speech as long as it is clearly spoken in a quiet environment. If the listener’s task is made more difficult with the introduction of background noise or competing messages, performance deteriorates more markedly in patients with central lesions than in normal subjects.

DIAGNOSIS

Evaluation Bedside Test

A quick test for hearing loss in the speech range is to observe the response to spoken commands at different intensities (whisper, conversation, shouting). Tuning fork tests permit a rough assessment of the hearing level for pure tones of known frequency. The clinician can use his or her own hearing level as a reference standard. In the Rinne test, nerve conduction is compared with bone conduction by holding a tuning fork (preferably 512 Hz) against the mastoid process until the sound can no longer be heard. It is then placed 1 inch from the ear and, in normal subjects, can be heard about twice as long

by air as by bone. If bone conduction is better than air conduction, the hearing loss is conductive, but care must be taken to ensure that the bone conduction is not heard in the normal ear. In the Weber test, the tuning fork is placed on the patient’s forehead or upper teeth. Normally, this sound is referred to the center of the head. If it is referred to the side of unilateral hearing loss, the hearing loss is conductive; if it is referred away from the side of unilateral hearing loss, the loss is sensorineural.

Audiometry

Pure tone testing is the cornerstone of most auditory examinations. Pure tones at selected frequencies are presented through either earphones (air conduction) or a vibrator pressed against the mastoid portion of the temporal bone (bone conduction), and the minimal level that the subject can hear (threshold) is determined for each frequency. Two speech tests are routinely used. The speech reception threshold is the intensity at which the patient can correctly repeat 50% of the words presented. The speech reception threshold is a test of hearing sensitivity for speech and should reflect the hearing level for pure tones in the speech range. The speech discrimination test is a measure of the patient’s ability to understand speech when it is presented at a level that is easily heard. In patients with eighth nerve lesions, speech discrimination scores can be severely reduced, even when pure tone thresholds are normal or nearly normal; by comparison, in patients with cochlear lesions, discrimination tends to be proportional to the magnitude of hearing loss. Brain stem auditory evoked responses can be recorded from scalp electrodes at 0 to 10 msec (early), 10 to 50 msec (middle), and 50 to 500 msec (late) following a click (a high-frequency stimulus). The early potentials reflect electrical activity at the cochlea, eighth cranial nerve, and brain stem; the later potentials reflect cortical activity. Computer averaging of the responses to 1000 to 2000 clicks separates the evoked potential from background noise. Early evoked responses may be used to estimate the magnitude of hearing loss and to differentiate among cochlea, eighth nerve, and brain stem lesions.

Differential Diagnosis Conductive Hearing Loss

The history, examination, and audiometry usually provide the key differential features for identifying common causes of hearing loss (Fig. 428-1). The most common cause of conductive hearing loss is impacted cerumen in the external canal. This benign condition is usually first noticed after bathing or swimming when a droplet of water closes the remaining tiny passageway. The most common serious cause of conductive hearing loss is inflammation of the middle ear, otitis media, either infective (suppurative; see Fig. 426-8) or noninfective (serous). Fluid accumulates in the middle ear, impairing the conduction of airborne sound to the cochlea. Because the air cavity of the middle ear is in direct connection with the mastoid air cells, infection can spread through the mastoid bone and, occasionally, into the intracranial cavity. Chronic otitis media with perforation of the tympanic membrane can result in an invasion of the middle ear and other pneumatized areas of the temporal bone by keratinizing squamous epithelium (cholesteatoma). Cholesteatomas can produce erosion of the ossicles and bony labyrinth, resulting in a mixed conductive and sensorineural hearing loss. Barotrauma to the middle ear arises with otalgia and hearing loss and can be associated with serous effusion or hematotympanum (see Fig. 426-9). Otosclerosis commonly produces progressive conductive hearing loss by immobilizing the stapes with new bone growth in front of and below the oval window. The hearing loss is typically conductive, although in some persons the cochlea may be invaded by foci of otosclerotic bone, producing an additional sensorineural hearing loss. Otosclerosis usually stabilizes when the hearing level reaches 50 to 60 dB and rarely progresses to deafness. Other common causes of conductive hearing loss include trauma, congenital malformations of the external and middle ear, and glomus body tumors.

Sensorineural Hearing Loss Hereditary Deafness

Genetically determined deafness, usually from hair cell aplasia or deterioration, may be present at birth or may develop in adulthood. The diagnosis of hereditary deafness rests on the finding of a positive family history. Mutations in connexin 26, a key component of gap junctions in the inner ear, account for most cases of recessively inherited deafness. Intrauterine factors resulting in congenital hearing loss include infection (especially rubella); toxic, metabolic, and endocrine disorders; and anoxia associated with Rh incompatibility and difficult deliveries.

2595

CHAPTER 428  Hearing and Equilibrium  

Hearing Loss Abnormal History

Ear examination

Impacted cerumen Otitis media Cholesteatoma Drum perforation Trauma

Neuroimaging

Normal Drugs Noise exposure Hereditary factors Intrauterine factors Trauma

Acoustic neuroma Other C-P angle tumor Multiple sclerosis Brain stem infarct

Abnormal Pure tone audiogram

Sensorineural hearing loss

BAER Normal

Conductive hearing loss Impedance audiometry

Abnormal

Fistula test

Otosclerosis Ossicular chain dysfunction Serous otitis Eustachian tube dysfunction

Normal

Perilymph fistula

Labyrinthitis Labyrinthine infarct Meniere disease Presbycusis Noise induced

FIGURE 428-1.  Evaluation of hearing loss. BAER = brain stem auditory evoked response; C-P = cerebellopontine.

Cochlear Damage

Acute unilateral deafness usually has a cochlear basis. Bacterial or viral infections of the labyrinth, head trauma with fracture or hemorrhage into the cochlea, or vascular occlusion of a terminal branch of the anterior inferior cerebellar artery can extensively damage the cochlea and the vestibular labyrinth. An isolated sudden unilateral sensorineural hearing loss is presumed to reflect a viral infection of the cochlea and auditory nerve terminals.2 Highdose steroids followed by a rapid taper are recommended (see Treatment). Sudden unilateral hearing loss often associated with vertigo and tinnitus can result from a perilymphatic fistula. Such fistulas may be congenital or may follow stapes surgery or head trauma. Drugs

Drugs cause acute and subacute bilateral hearing impairment. Salicylates, furosemide, and ethacrynic acid have the potential to produce transient deafness when they are taken in high doses. More toxic to the cochlea are aminoglycoside antibiotics (gentamicin, tobramycin, amikacin, kanamycin, streptomycin, and neomycin). These agents can destroy cochlear hair cells in direct relation to their serum concentrations. Some antineoplastic chemotherapeutic agents, particularly cisplatin, cause severe ototoxicity. Meniere Disease

Subacute relapsing cochlear deafness occurs with Meniere disease, a condition associated with fluctuating hearing loss and tinnitus, recurrent episodes of abrupt and often severe vertigo, and a sensation of fullness or pressure in the ear. Recurrent endolymphatic hypertension (hydrops) is believed to cause the episodes. On pathologic examination, the endolymphatic sac is dilated, and the hair cells become atrophic. The resulting deafness is subtle and reversible in the early stages but subsequently becomes permanent and is characterized by diplacusis and loudness recruitment. The disorder is usually unilateral, but in about 20 to 40% of patients, bilateral involvement occurs. Presbycusis

The gradual, progressive, bilateral hearing loss commonly associated with advancing age is called presbycusis.3 Presbycusis is not a distinct disease entity but rather represents multiple effects of aging on the auditory system. It may include conductive and central dysfunction, although the most consistent effect of aging is on the sensory cells and neurons of the cochlea. The typical audiogram of presbycusis is a symmetrical high-frequency hearing loss gradually sloping downward with increasing frequency. The most consistent pathologic finding associated with presbycusis is degeneration of sensory cells and nerve fibers at the base of the cochlea. Noise

The recurrent trauma of noise-induced hearing loss affects approximately the same region at the base of the cochlea and is also common, particularly

among those with exposure to loud explosive or industrial noises. Loud, blaring, modern music has become a recent offender. The loss almost always begins at 4000 Hz and does not affect speech discrimination until late in the disease process. With only brief exposure to loud noise (hours to days), there may be only a temporary threshold shift, but with continued exposure, permanent injury begins. The duration and intensity of exposure determine the degree of permanent injury. Acoustic Neuroma

Progressive unilateral hearing loss, which arises insidiously, initially in the high frequencies, and worsens by almost imperceptible degrees, is characteristic of benign neoplasms of the cerebellopontine angle, most commonly acoustic neuromas. In about 10% of cases, the hearing loss can be acute, apparently due to either hemorrhage into the tumor or compression of the labyrinthine vasculature. Magnetic resonance imaging (MRI) with contrast enhancement reliably identifies small acoustic neuromas.

Central Hearing Loss

Central hearing loss is unilateral only if it results from damage to the pontine cochlear nuclei on one side of the brain stem from conditions such as ischemic infarction of the lateral brain stem (e.g., occlusion of the anterior inferior cerebellar artery [Chapter 407]), a plaque of multiple sclerosis (Chapter 411), or, rarely, invasion or compression of the lateral pons by a neoplasm or hematoma (Chapters 189 and 399). Bilateral degeneration of the cochlear nuclei accompanies some of the rare recessive inherited disorders of childhood. As noted, clinically important unilateral hearing loss never results from neurologic disease arising rostral to the cochlear nucleus. Although bilateral hearing loss could, in theory, result from bilateral destruction of central hearing pathways, in practice this is rare because involvement of neighboring structures in the brain stem or hemisphere would usually produce overwhelming neurologic disability.

TREATMENT  If an underlying disorder has not yet destroyed the auditory system and can be ameliorated medically or surgically, hearing may be improved or preserved. Most patients with otosclerosis respond to stapedectomy. Closure of a perilymph fistula may improve hearing. Antibiotic and decongestive treatment of otitis media (Chapter 426) should prevent permanent hearing loss. A brief course of high-dose steroids is commonly used for patients with idiopathic sudden unilateral sensorineural deafness, but the evidence to support this approach is limited. A1  Intratympanic corticosteroid treatment (four doses of 40 mg/mL of methylprednisolone during 2 weeks) is not inferior to oral treatment (60 mg/day of oral prednisone followed by a 5-day taper) for idiopathic sudden sensorineural hearing loss, A2  and combination oral and intratympanic therapy may be better than either alone. A3  A low-salt diet and diuretics are effective in selected cases of Meniere disease. Folic acid

2596

CHAPTER 428  Hearing and Equilibrium  

supplementation appears to reduce the rate of hearing loss in the elderly. A4  Hearing aids amplify sound, usually with the goal of making speech intelligible. Patients with conductive hearing loss require simple amplification, but those with sensorineural hearing loss often need frequency-selective amplification to make hearing aids useful. Cochlear implants can help patients with profound hearing loss if they have some intact auditory nerve fibers.4 Intense postoperative speech recognition training is required.

Tinnitus

DIAGNOSIS

The evaluation of common causes of tinnitus (Fig. 428-2) begins with a careful history to identify common offending drugs.5

Objective Tinnitus

With objective tinnitus, the patient hears a sound arising external to the auditory system, a sound that can usually be heard by the examiner with a stethoscope. Objective tinnitus usually has benign causes, such as noise from temporomandibular joints, opening of eustachian tubes, or repetitive muscle contractions. Sometimes, in a quiet room, the patient can hear the pulsatile flow in the carotid artery or a continuous hum of normal venous outflow through the jugular vein. The latter can be obliterated by compression of the jugular vein or extreme lateral rotation of the neck. Pathologic objective tinnitus occurs when patients hear turbulent flow in vascular anomalies or tumors (e.g., glomus jugulare tumor). Objective tinnitus may also be an early sign of increased intracranial pressure. Such tinnitus, which probably arises from turbulent flow through compressed venous structures at the base of the brain, is usually overshadowed by other neurologic abnormalities.

Subjective Tinnitus

Subjective tinnitus can arise from sites anywhere in the auditory system. The sounds most frequently reported are metallic ringing, buzzing, blowing, roaring, or, less often, bizarre clanging, popping, or nonrhythmic beating. Tinnitus heard as a faint, moderately high pitched, metallic ring can be observed by almost anyone who concentrates attention on auditory events

Audible sounds

Tinnitus

History

Common offending drugs: Quinidine Salicylates Indomethacin Carbamazepine Propranolol Levodopa Aminophylline Caffeine

Ear examination

in a quiet room. Sustained louder tinnitus accompanied by audiometric evidence of deafness occurs in association with both conductive and sensorineural hearing loss. Tinnitus observed with otosclerosis tends to have a roaring or hissing quality, and that associated with Meniere disease often produces sounds that vary widely in intensity with time and quality, sometimes including roaring or clanging. Tinnitus with auditory nerve lesions tends to be higher pitched and ringing in quality. Audiometric and brain stem evoked response testing can help distinguish between lesions involving the conducting apparatus, the cochlea, and the auditory nerve. Tinnitus without observable deafness appears sporadically and for variable lengths of time in many persons without other evidence of an ongoing pathologic process.

TREATMENT  Most patients with tinnitus can be helped by a careful evaluation to exclude serious underlying conditions and by subsequent reassurance when appropriate. Often, exacerbating factors such as chronic anxiety and depression can be treated. In patients with hearing loss and tinnitus, a hearing aid may improve tinnitus because the amplification of ambient sound may effectively mask the tinnitus. This mechanism probably explains the frequent observation that removal of cerumen from the external auditory canal to improve ambient hearing also improves tinnitus. Also, when cerumen is attached to the tympanic membrane, tinnitus may result from local mechanical effects on the conductive system. For patients who find their tinnitus most obtrusive when trying to sleep, recorded masking sounds (e.g., white noise, rainfall, mountain stream) can be helpful. A careful drug history should be taken (see Fig. 428-2), and a drug-free trial period should be considered when possible. No medications are approved for the treatment of tinnitus in the United States or Europe. Benzodiazepines (e.g., diazepam, 2 to 5 mg every 8 hours) or tricyclic amines (e.g., amitriptyline, 25 to 75 mg at bedtime) may provide temporary symptomatic relief of tinnitus, but cognitive-behavioral therapy is a more effective long-term approach that can significantly decrease tinnitus and improve health-related quality of life. A5  In patients with concomitant profound bilateral sensorineural hearing loss, cochlear implants can improve hearing and often decrease tinnitus.

Synchronous with respiration

Patent eustachian tube

Synchronous with pulse

Aneurysm Vascular malformation Vascular tumor Venous hum

Continuous

Venous hum Acoustic emissions

Conductive hearing loss

Impacted cerumen Chronic otitis Otosclerosis

No audible sounds Neurologic examination

Normal

Audiogram

Sensorineural hearing loss

Abnormal (neural) BAER Normal (cochlear)

Brain stem signs Multiple sclerosis Tumor Ischemic infarction

Normal

FIGURE 428-2.  Evaluation of tinnitus. BAER = brain stem auditory evoked response; C-P = cerebellopontine.

Acoustic neuroma Other C-P angle tumors Vascular compression

Idiopathic tinnitus

Noise damage Ototoxic drugs Labyrinthitis Meniere disease Perilymph fistula Presbycusis

CHAPTER 428  Hearing and Equilibrium  

  EQUILIBRIUM–VESTIBULAR SYSTEM

DIAGNOSIS

Evaluation History

PATHOBIOLOGY

Anatomy and Physiology of the Vestibular System

The paired vestibular end organs lie within the temporal bones next to the cochlea. Each organ consists of three semicircular canals that detect angular acceleration and two otolith structures, the utricle and saccule, that detect linear acceleration (including gravitational). Like the cochlea, these organs possess hair cells that act as force transducers, converting the forces associated with head acceleration into afferent nerve impulses. The hair cells of the three semicircular canals, each of which is oriented at right angles to the others, are located in the crista, where their cilia are embedded in a gelatinous mass called the cupula. Movement of the head causes the endolymph to flow either toward or away from the cupula, bending the cilia and, depending on the direction of endolymphatic movements, either exciting or inhibiting the afferent nerves at the base of the hair cells. Because the afferent nerves are tonically active, the baseline activity can be increased or decreased, depending on the direction of hair cell bending. Furthermore, the two sets of semicircular canals are approximately mirror images of each other, so that rotational movement of the head that excites one canal inhibits the analogous canal on the opposite side. The hair cells of the utricle and saccule are located in an area called the macule. The macule of the utricle lies approximately in the plane of the horizontal canal, and the macule of the saccule is approximately in the plane of the anterior canal. The hair cell cilia are embedded in a membrane that contains calcium carbonate crystals or otoliths; the density of otoliths is considerably greater than that of the endolymph. Linear accelerations of the head combine with the linear acceleration of gravity to distort the otolith membrane, thereby bending the cilia of the hair cells and modulating the activity of the afferent nerve terminals at the base of the hair cells. The afferent vestibular nerves have their cell bodies in Scarpa’s ganglion. The nerve fibers travel in the vestibular portion of the eighth cranial nerve contiguous to the acoustic portion. Fibers from different receptor organs terminate in different vestibular nuclei at the pontomedullary junction. There are also direct connections with many portions of the cerebellum, the greatest representation being in the flocculonodular lobe, the so-called vestibular cerebellum. Efferent fibers from the brain stem travel through the vestibular nerve to reach hair cells of the semicircular canals and macules, where they modulate afferent activities. From the vestibular nuclei, secondorder neurons make important connections to the vestibular nuclei of the other side, to the cerebellum, to motor neurons of the spinal cord, to autonomic nuclei in the brain stem, and, most important for the examining clinician, to the nuclei of the oculomotor system. Fibers from the vestibular nuclei also ascend through the brain stem and thalamus to reach the cerebral cortex bilaterally.

Focal neurologic signs

Vertigo History

Most vestibular problems presented to the physician are episodic, and often there are neither symptoms nor signs when the physician examines the patient. The history, therefore, can become paramount for identifying vestibular dysfunction. The history should attempt to distinguish vertigo (the illusion of movement in space) from lightheadedness (presyncope), ataxia (disequilibrium of the body without true movement in space), and psychogenic symptoms (the feeling of dissociation or, sometimes, disequilibrium). About 12% of patients with vertigo have a central cause, and about 88% have a problem with the peripheral vestibular apparatus.6 In general, peripheral vertigo is more severe, is more likely to be associated with hearing loss and tinnitus, and often leads to nausea and vomiting. Nystagmus associated with peripheral vertigo is usually inhibited by visual fixation. Central vertigo is generally less severe than peripheral vertigo and is often associated with other signs of central nervous system disease. The nystagmus of central vertigo is not inhibited by visual fixation and frequently is prominent when vertigo is mild or absent.

Common Causes of Vertigo Physiologic Vertigo

Physiologic vertigo includes common disorders that occur in healthy people, such as motion sickness, space sickness, and height vertigo (Fig. 428-3). In these conditions, vertigo (defined as an illusion of movement) is minimal while autonomic symptoms predominate. With height vertigo, patients may experience acute anxiety and panic reaction. Individuals with motion sickness and space sickness typically develop perspiration, nausea, vomiting, increased salivation, yawning, and generalized malaise. Gastric motility is reduced and digestion impaired. Even the sight or smell of food is distressing. Hyperventilation is a common sign, and the resulting hypocapnia leads to changes in blood volume, with pooling in the lower parts of the body predisposing to postural hypotension and syncope. An unusual variant of motion-induced dizziness occurs when the subject returns to stationary conditions after prolonged exposure to motion (mal de débarquement syndrome). Typically, affected patients report that they feel the persistent rocking sensation of a boat long after returning to solid ground. Rarely, the syndrome can last for months to years after exposure to motion and can even be incapacitating. The cause is unknown. Physiologic vertigo can often be suppressed by supplying sensory cues that help to match the signals originating from different sensory systems. Thus, motion sickness, which is caused by a mismatch of visual and vestibular signals, is exacerbated by sitting in a closed space or reading (giving the visual system the miscue that the environment is stationary). It may be improved by looking out at the horizon. Height vertigo, caused by a mismatch between

Neuroimaging

Multiple sclerosis Infarction Tumor Degenerative disease

Peripheral vestibulopathy Physiologic vertigo Ototoxicity

Examinations Normal

Hearing loss Drugs Trauma Hereditary factors Migraine

2597

Audiometry Normal

Positional nystagmus in plane of canal

Benign positional vertigo

Sensorineural loss

ENG

BAER

Normal

Abnormal

Neuroimaging Positive head-thrust test

Vestibular neuritis Ototoxicity

Acoustic neuroma Other C-P angle tumor

FIGURE 428-3.  Evaluation of vertigo. BAER = brain stem auditory evoked response; C-P = cerebellopontine; ENG = electronystagmography.

ENG

Labyrinthitis Meniere disease Fistula Labyrinthine concussion

2598

CHAPTER 428  Hearing and Equilibrium  

sensation of normal body sway and lack of its visual detection, can often be relieved either by sitting or by visually fixating a nearby stationary object.

Benign Paroxysmal Positional Vertigo (Canalithiasis)

Benign paroxysmal positional vertigo is by far the most common cause of vertigo.7 Patients with this condition develop brief episodes of vertigo (less than 1 minute) with position change, typically when turning over in bed, getting in and out of bed, bending over and straightening up, or extending the neck to look up (so-called top-shelf vertigo). Benign paroxysmal positional vertigo results when otolith debris inadvertently enters one of the semicircular canals. It can occur after head trauma or inner ear infection but most commonly occurs spontaneously in older people. The diagnosis rests on finding characteristic positional nystagmus in the plane of the affected canal (see later). It is important to recognize this syndrome because, in most patients, it can be cured by simple bedside maneuvers (Fig. 428-4). If the history or findings are atypical, the condition must be distinguished from other causes of positional vertigo that may occur with tumors or infarcts of the posterior fossa.

Acute Peripheral Vestibulopathy (Vestibular Neuritis)

One of the most common clinical neurologic syndromes at any age is the acute onset of vertigo, nausea, and vomiting lasting for several days and not

associated with auditory or neurologic symptoms.8 A viral origin is suspected, but attempts to isolate an agent have been unsuccessful, except for occasional findings of a herpes zoster infection. Pathologic studies showing atrophy of one or more vestibular nerve trunks, with or without atrophy of their associated sense organs, are evidence of a vestibular nerve site and, probably, viral cause for many patients with this syndrome. Many patients report an upper respiratory tract illness 1 to 2 weeks before the onset of vertigo. This syndrome occasionally occurs in epidemics (epidemic vertigo), may affect several members of the same family, and more often erupts in the spring and early summer. Most affected patients gradually improve during 1 to 2 weeks, but residual dizziness and imbalance can persist for months.

Meniere Disease

Meniere disease (see earlier) accounts for about 10% of all patients with vertigo.9 The diagnosis is based on documenting episodic severe attacks accompanied by fluctuating hearing levels on audiometric testing beginning in the low frequencies.

Migraine

Vertigo is a common symptom with migraine (Chapter 398). It can occur with headaches or in separate isolated episodes, and it can predate the onset of headache. So-called benign paroxysmal vertigo of childhood is often the

1

2

3

5

A

4

1

2

3

B FIGURE 428-4. Modified Epley’s (A) and Semont’s (B) maneuvers for benign positional vertigo affecting the right posterior semicircular canal. The procedure is reversed to treat the left posterior semicircular canal. The entire sequence should be repeated until no nystagmus is elicited. (From Fife TD, Iverson DJ, Lempert T, et al. Practice parameter: therapies for benign paroxysmal positional vertigo [an evidence-based review]: report of the Quality Standards Subcommittee of the American Academy of Neurology. Neurology. 2008;70:2067-2074.)

CHAPTER 428  Hearing and Equilibrium  

first symptom of migraine. The mechanism of vertigo with migraine is not clear, but both peripheral and central types of nystagmus can occur with attacks. A few develop typical features of Meniere disease.

Post-traumatic Vertigo

Vertigo, hearing loss, and tinnitus often follow a blow to the head (Chapter 399) that does not result in temporal bone fracture, termed labyrinthine concussion. Although they are protected by a bone capsule, the delicate labyrinthine membranes are susceptible to blunt trauma. Blows to the occipital or mastoid region are particularly likely to produce labyrinthine damage. Transverse fractures of the temporal bone typically pass through the vestibule of the inner ear, tearing the membranous labyrinth and lacerating the vestibular and cochlear nerves. Complete loss of vestibular and cochlear function is the usual sequela, and the facial nerve is interrupted in approximately 50% of cases. Examination of the ear often reveals hemotympanum (see Fig. 426-9), but bleeding from the ear seldom occurs because the tympanic membrane usually remains intact. As noted earlier, benign paroxysmal positional vertigo is also a common sequela of head trauma. Fistulas of the oval and round windows can result from impact noise, deep-water diving, severe physical exertion, or blunt head injury without skull fracture. The mechanism of the rupture is a sudden negative or positive pressure change in the middle ear or a sudden increase in cerebrospinal fluid pressure transmitted to the inner ear through the cochlear aqueduct and internal auditory canal. Clinically, the rupture leads to the sudden onset of vertigo or hearing loss, or both. Surgical exploration of the middle ear is warranted when there is a clear relationship between the onset of vertigo or hearing loss, or both, and the onset of severe exertion, barometric change, head injury, or impact noise.

Postconcussion Syndrome

The so-called postconcussion syndrome refers to a vague dizziness (rarely vertigo) associated with anxiety, difficulty in concentrating, headache, and photophobia induced by a head injury resulting in concussion (Chapter 399). On occasion, similar but less pronounced symptoms are associated with mild head injury judged to be trivial at the time. The cause is unknown, but animal studies indicate that small multifocal brain lesions (petechiae) commonly occur after concussive brain injury.

Other Peripheral Causes of Vertigo

Vertigo can be associated with chronic bacterial otomastoiditis, either from direct invasion of the inner ear by the bacteria or by erosion of the labyrinth by a cholesteatoma. Radiographic studies of the temporal bone readily identify these disorders. Autoimmune inner ear disease typically arises with episodic vertigo and fluctuating hearing levels similar to Meniere disease, but it is more fulminant with early bilateral involvement. It can occur in isolation or with other systemic features of autoimmune disease. About two thirds of patients have antibodies directed against heat shock protein 70. The aminoglycosides streptomycin and gentamicin are remarkably selective for vestibular ototoxicity. The patient may suffer acute vertigo if the toxic effect is asymmetrical. More often, there is a progressive symmetrical loss of vestibular function leading to imbalance but not vertigo. Unfortunately, many patients being treated with ototoxic drugs are initially bedridden and unaware of the vestibular impairment until they recover from their acute illness and try to walk. They then discover that they are unsteady on their feet and that the environment tends to jiggle in front of their eyes (oscillopsia). The diagnosis can be made at the bedside with a head-thrust test (bilateral corrective saccades; see later). Caloric and rotational testing can document the degree of vestibular loss. The best treatment is prevention. If the drug is discontinued early during the course of symptoms, the disorder may stabilize or improve.

Vascular Insufficiency

Vertebrobasilar insufficiency is a common cause of vertigo in older people. Whether the vertigo originates from ischemia of the labyrinth, brain stem, or both structures is not always clear because the blood supplies to the labyrinth, eighth cranial nerve, and vestibular nuclei originate from the same source, the basilar vertebral circulation (Chapter 406). Vertigo with vertebrobasilar insufficiency is abrupt in onset, usually lasting several minutes, and is frequently associated with nausea and vomiting. Associated symptoms resulting from ischemia in the remaining territory supplied by the posterior circulation include visual illusions and hallucinations, drop attacks and weakness, visceral sensations, visual field defects, diplopia, and headache. These symptoms occur in episodes either in combination with the vertigo or alone. Vertigo may be an isolated initial symptom of vertebrobasilar ischemia, but repeated

2599

episodes of vertigo without other symptoms should suggest another diagnosis. Vertebrobasilar insufficiency is usually caused by atherosclerosis of the subclavian, vertebral, and basilar arteries. On occasion, episodes of vertebrobasilar insufficiency are precipitated by postural hypotension, Stokes-Adams attacks, or mechanical compression from cervical spondylosis. MRI of the brain is usually normal because the vascular insufficiency is transient and function returns to normal between episodes. Magnetic resonance angiography can identify occlusive vascular disease most commonly involving the vertebral-basilar junction. Vertigo is a common symptom with infarction of the lateral brain stem or cerebellum (Chapter 407), or both. The diagnosis is usually clear, based on the characteristic acute history and pattern of associated symptoms and neurologic findings. On occasion, cerebellar infarction or hemorrhage arises with severe vertigo, vomiting, and ataxia without associated brain stem symptoms and signs that might suggest the erroneous diagnosis of an acute peripheral vestibular disorder. The key differential is the finding of clear cerebellar signs (extremity and gait ataxia) and of direction-changing, gaze-evoked nystagmus. Such patients must be watched carefully for several days because they may develop progressive brain stem dysfunction due to compression by a swollen cerebellum.

Cerebellopontine Angle Tumors

Most tumors growing in the cerebellopontine angle (e.g., acoustic neuroma, meningioma, epidermal cyst) grow slowly, allowing the vestibular system to accommodate so that they produce only a vague sensation of disequilibrium rather than acute vertigo (Chapter 189). On occasion, however, episodic vertigo or positional vertigo heralds the presence of a cerebellopontine angle tumor. In virtually all patients, retrocochlear hearing loss is present, best identified by audiometric testing. MRI with contrast enhancement is the most sensitive diagnostic study for identifying a cerebellopontine angle tumor.

Other Central Causes of Vertigo

Acute vertigo may be the first symptom of multiple sclerosis (Chapter 411), although only a small percentage of young patients with acute vertigo eventually develop multiple sclerosis. Vertigo in multiple sclerosis is usually transient and often associated with other neurologic signs of brain stem disease, in particular, internuclear ophthalmoplegia or cerebellar dysfunction. Vertigo may also be a symptom of parainfectious encephalomyelitis or, rarely, parainfectious cranial polyneuritis. In this instance, the accompanying neurologic signs establish the diagnosis. The Ramsay Hunt syndrome (geniculate ganglion herpes) is characterized by vertigo and hearing loss associated with facial paralysis and, sometimes, pain in the ear. The typical lesions of herpes zoster (Chapter 375), which may follow the appearance of neurologic signs, are found in the external auditory canal and over the palate in some patients. Rarely is herpes zoster responsible for vertigo in the absence of the fullblown syndrome. Granulomatous meningitis (Chapter 412) or leptomeningeal metastasis and cerebral or systemic vasculitis (Chapter 270) may involve the eighth nerve, producing vertigo as an early symptom. In these disorders, cerebrospinal fluid analysis usually suggests the diagnosis (Chapter 396). Patients suffering from temporal lobe epilepsy (Chapter 403) occasionally experience vertigo as the aura. Vertigo in the absence of other neurologic signs or symptoms is never caused by epilepsy or other diseases of the cerebral hemispheres.

Bedside Tests Hyperventilation

If the history is not clear, bedside provocative tests to mimic the symptom may assist in making a pathophysiologic diagnosis.10 Hyperventilation, which lowers the arterial partial pressure of carbon dioxide (Paco2) and decreases cerebral blood flow, causes a lightheaded sensation associated with syncope. Patients with compressive lesions of the vestibular nerve, such as with an acoustic neuroma or cholesteatoma, or with demyelination of the vestibular nerve root entry zone may develop vertigo and nystagmus after hyperventilation. Presumably, metabolic changes associated with hyperventilation trigger the partially damaged nerve to fire inappropriately.

Vestibulospinal Function

Bedside tests of vestibulospinal function are often insensitive because most patients can use vision and proprioceptive signals to compensate for any vestibular loss. Patients with acute unilateral peripheral vestibular lesions may past-point or fall toward the side of the lesion, but within a few days, balance returns to normal. Patients with bilateral peripheral vestibular loss have more

2600

CHAPTER 428  Hearing and Equilibrium  

TABLE 428-1  DESCRIPTION, MECHANISM, AND FOCUS OF DIAGNOSTIC WORK-UP FOR COMMON TYPES OF DIZZINESS TYPE OF DIZZINESS

DESCRIPTION

MECHANISM Imbalance in tonic vestibular activity

FOCUS OF DIAGNOSTIC EVALUATION

Vertigo

Spinning (environment moves), tilt, drunkenness

Near-faint

Lightheaded, swimming

Decreased blood flow to entire brain

Cardiovascular system

Psychophysiologic

Dissociated from body, spinning inside (environment still)

Impaired central integration of sensory signals

Psychiatric assessment

Disequilibrium

Off balance, unsteady on feet

Loss of vestibulospinal, proprioceptive, cerebellar, or motor function

Neurologic assessment

difficulty compensating and usually show some imbalance on the Romberg and tandem walking tests (Chapter 396), particularly with eyes closed.

Doll’s-Eye and Head-Thrust Tests

The vestibulo-ocular reflex can be tested at the bedside with the doll’s-eye and head-thrust tests. In an alert human, rotating the head back and forth in the horizontal plane induces compensatory horizontal eye movements that are dependent on both the visual and vestibular systems. The doll’s-eye test is a test of vestibular function in a comatose patient (Chapter 404) because such patients cannot generate pursuit or corrective fast components. In this setting, conjugate compensatory eye movements indicate normally functioning vestibulo-ocular pathways. Because the vestibulo-ocular reflex has a much higher frequency range than the smooth pursuit system, a qualitative bedside test of vestibular function can be made with the head-thrust test. It is performed by grasping the patient’s head and applying brief, small-amplitude, high-acceleration head thrusts first to one side and then the other. The patient fixates on the examiner’s nose and the examiner watches for corrective saccades, which are a sign of an inappropriate compensatory slow phase.

Caloric Test

The caloric test induces endolymphatic flow in the horizontal semicircular canal and horizontal nystagmus by creating a temperature gradient from one side of the canal to the other. With a cold caloric stimulus, the column of endolymph nearest the middle ear falls because of its increased density. This causes the cupula to deviate away from the utricle (ampullofugal flow) and produces horizontal nystagmus with the fast phase directed away from the stimulated ear. A warm stimulus produces the opposite effect, causing ampullopetal endolymph flow and nystagmus directed toward the stimulated ear (a mnemonic is COWS, meaning cold opposite, warm same). Because of its ready availability, ice water (approximately 0° C) can be used for bedside caloric testing. To bring the horizontal canal into the vertical plane, the patient lies in the supine position with head tilted 30 degrees forward. Infusion of 1 to 3 mL of ice water induces a burst of nystagmus usually lasting about a minute. Greater than a 20% asymmetry in nystagmus duration suggests a lesion on the side of the decreased response. The ice water caloric test is a useful way to test the integrity of the oculomotor pathways in a comatose patient. In this case, ice water induces only a slow tonic deviation toward the side of stimulation.

Positional Tests

Examination for pathologic vestibular nystagmus should include a search for spontaneous and positional nystagmus (see Table 424-5). Because vestibular nystagmus secondary to peripheral vestibular lesions is inhibited with fixation, the yield is increased by impairing fixation with +30 lenses (Frenzel glasses) or infrared video recordings. Two types of positional testing are typically performed: moving the patient from the sitting to head-hanging-right and head-hanging-left positions (Dix-Hallpike test) and turning the head to the right and left while the patient lies supine. Induced positional nystagmus may be paroxysmal or persistent, and it may be in the same direction in all positions or change directions in different positions. The most common cause of positional nystagmus is otolith debris in the semicircular canals, either free floating (paroxysmal) or attached to the cupula (persistent). This type of nystagmus always occurs in the plane of the affected canal—vertical torsional for the vertical canals and horizontal torsional for the horizontal canal. By contrast, central positional nystagmus is often pure vertical or horizontal and cannot be explained by stimulating a single semicircular canal.

Nystagmography

Nystagmography tests oculomotor control by inducing and recording eye movements. A standard test battery includes (1) tests of visual ocular control

Auditory and vestibular systems

(saccades, smooth pursuit, and optokinetic nystagmus), (2) a careful search for pathologic nystagmus with fixation and with eyes open in darkness, and (3) the measurement of induced vestibular nystagmus (caloric and rotational). Nystagmography can be helpful in identifying a vestibular lesion and localizing it within the peripheral and central pathways.

Evaluating the “Dizzy” Patient

The history is key because it determines the type of dizziness (vertigo, near-faint, psychophysiologic disequilibrium), associated symptoms (neurologic, audiologic, cardiac, psychiatric), precipitating factors (position change, trauma, stress, drug ingestion), and predisposing illness (systemic viral infection, cardiac disease, cerebrovascular disease). The history provides direction for both the examination and the diagnostic evaluation (Table 428-1). When focal neurologic signs are found, neuroimaging usually leads to a specific diagnosis. When vertigo is present without focal neurologic symptoms or signs, head-thrust and positional testing are key to localizing the lesion to the labyrinth or eighth nerve. Audiometry and nystagmography are useful if the cause of vertigo is not clear after the history and examination. Patients with psychophysiologic dizziness should be identified early so that needless tests are not obtained. A detailed cardiac evaluation (including loop monitoring) often identifies the cause of episodic near-fainting (Chapters 51 and 62).

TREATMENT  Treatment of vertigo can be divided into three general categories: specific, symptomatic, and rehabilitative. When possible, treatment should be directed at the underlying disorder (Table 428-2). Specific therapies include particle repositioning maneuvers (the Epley and Semont maneuvers; see Fig. 428-4) for benign paroxysmal positional vertigo. A6-A8  For vestibular neuritis, steroids (e.g., methylprednisolone, 1 mg/kg/day for 5 days, then tapered during the next 15 days) are effective, at least for the short term, A9  but antiviral agents are not. For Meniere disease, a low-salt diet and diuretics (e.g., 25 mg hydrochlorothiazide and 50 mg triamterene daily) are effective. Intratympanic gentamicin can significantly reduce vertigo in patients with unilateral Meniere disease who do not respond to medical treatment. A10  Endolymphatic duct blockage is a potential option for medically refractory Meniere disease. A11  In many cases, however, symptomatic treatment either is combined with specific therapy or is the only treatment available. Many different classes of drugs have been found to have antivertiginous properties, and in most instances, the exact mechanism of action is uncertain. All these agents produce potentially unpleasant side effects, and the decision concerning which drug or combination to use is based on their known complications and on the severity and duration of the vertigo. An episode of prolonged, severe vertigo is one of the most distressing symptoms that a patient can experience. Affected patients prefer to lie still with eyes closed in a quiet, dark room. Antivertiginous drugs with sedation, such as promethazine HCl (25 mg) or diazepam (5 mg), may be helpful. Prochlorperazine suppositories (25 mg) may stop vomiting. In more chronic vertiginous disorders, when the patient is trying to carry on normal activity, less sedating antivertiginous medications, such as meclizine (25 mg) or transdermal scopolamine (0.5 mg every 3 days), may provide relief. Chronic use of these drugs should be avoided. Vestibular rehabilitation exercises are designed to help the patient compensate for permanent loss of vestibular function. A12  As the acute stage of nausea and vomiting subsides, the patient should attempt to focus the eyes and to move and hold them in the direction that provokes the most dizziness. A useful exercise involves staring at a visual target while oscillating the head from side to side or up and down, slow at first and then fast. The patient should try to stand and walk, at first in contact with a wall or with an assistant, and make slow supported turns. As improvement occurs, head movements should be added while standing and walking.

2601

CHAPTER 429  Throat Disorders  

persistent throat pain with or without trismus, difficulty in swallowing, difficulty in breathing, hemoptysis, and ear pain with normal ear examination findings. Diagnostic testing options, which include fiberoptic examination, imaging, pulmonary function studies, and laboratory testing, are directed by history, symptoms, and physical findings.

TABLE 428-2  TREATMENT OF COMMON VERTIGO SYNDROMES SYNDROME Benign positional vertigo   Posterior canal variant   Horizontal canal variant Vestibular neuritis Meniere disease   Medical

  Surgical

TREATMENT Epley’s maneuver (see Fig. 428-4) Barbecue roll toward normal side (side with less nystagmus), sleep with normal ear down Methylprednisolone, 100 mg × 3 days, gradual taper during 22 days (must start within 3 days of onset) Low salt (1-2 g salt/day) and either hydrochlorothiazide 25-50 mg/day or hydrochlorothiazide 25 mg/day plus triamterene 50 mg/day Intratympanic gentamicin, vestibular nerve section

Grade A References A1. Wei BP, Stathopoulos D, O’Leary S. Steroids for idiopathic sudden sensorineural hearing loss. Cochrane Database Syst Rev. 2013;7:CD003998. A2. Rauch SD, Halpin CF, Antonelli PJ, et al. Oral vs intratympanic corticosteroid therapy for idiopathic sudden sensorineural hearing loss: a randomized trial. JAMA. 2011;305:2071-2079. A3. Gundogan O, Pinar E, Imre A, et al. Therapeutic efficacy of the combination of intratympanic methylprednisolone and oral steroid for idiopathic sudden deafness. Otolaryngol Head Neck Surg. 2013;149:753-758. A4. Durga J, Verhoef P, Anteunis LJ, et al. Effects of folic acid supplementation on hearing in older adults: a randomized, controlled trial. Lancet. 2007;369:208-216. A5. Cima RF, Maes IH, Joore MA, et al. Specialised treatment based on cognitive behaviour therapy versus usual care for tinnitus: a randomised controlled trial. Lancet. 2012;379:1951-1959. A6. Hunt WT, Zimmermann EF, Hilton MP. Modifications of the Epley (canalith repositioning) manoeuvre for posterior canal benign paroxysmal positional vertigo (BPPV). Cochrane Database Syst Rev. 2012;4:CD008675. A7. Kim JS, Oh SY, Lee SH, et al. Randomized clinical trial for apogeotropic horizontal canal benign paroxysmal positional vertigo. Neurology. 2012;78:159-166. A8. Kim JS, Oh SY, Lee SH, et al. Randomized clinical trial for geotropic horizontal canal benign paroxysmal positional vertigo. Neurology. 2012;79:700-707. A9. Fishman JM, Burgess C, Waddell A. Corticosteroids for the treatment of idiopathic acute vestibular dysfunction (vestibular neuritis). Cochrane Database Syst Rev. 2011;5:CD008607. A10. Pullens B, van Benthem PP. Intratympanic gentamicin for Meniere’s disease or syndrome. Cochrane Database Syst Rev. 2011;3:CD008234. A11. Saliba I, Gabra N, Alzahrani M, et al. Endolymphatic Duct Blockage: A Randomized Controlled Trial of a Novel Surgical Technique for Ménière’s Disease Treatment. Otolaryngol Head Neck Surg. 2015;152:122-129. A12. Hillier SL, McDonnell M. Vestibular rehabilitation for unilateral peripheral vestibular dysfunction. Cochrane Database Syst Rev. 2011;2:CD005397.

  ANATOMY OF THE UPPER AERODIGESTIVE TRACT

The pharynx is divided into three anatomic regions (Fig. 429-1). The nasopharynx is the region above the soft palate and uvula. Its anatomic components include the adenoids, the openings of the eustachian tubes, Rosenmüller’s fossa at the junction of the posterior and lateral walls, and the posterior aspect of the inferior turbinates of the nasal cavity. Diseases of the nasopharynx typically produce few symptoms until the process is well advanced and causes nasal obstruction (Chapter 426), epistaxis (Chapter 426), ear pain (Chapter 426), headache (Chapter 398), or cranial nerve abnormalities due to extension to the skull base. The oropharynx begins at the level of the soft palate and extends inferiorly to the tip of the epiglottis. This region includes the faucial tonsils, the base of the tongue, the lingual tonsils, the soft palate, the uvula, and part of the posterior pharyngeal wall. The hypopharynx, which extends from the tip of the epiglottis to the upper esophagus (the cricopharyngeus muscle) below, includes the larynx (epiglottis, arytenoids, glottis or true vocal cords), the piriform sinuses (pharyngeal folds lateral to the larynx), and the posterior pharyngeal wall. The tip of the epiglottis can be visualized by an experienced examiner with use of a laryngeal mirror or sometimes even on a routine oral examination with just a flashlight and tongue blade. The nasopharynx and hypopharynx are best visualized with a flexible fiberoptic nasopharyngoscope.

  INFECTIOUS DISEASES OF THE UPPER AERODIGESTIVE SYSTEM

Infectious disorders of the upper aerodigestive tract typically are manifested as sore throat (pharyngitis), changes in voice (laryngitis), or both. The clinical evaluation must differentiate among bacterial (usually streptococcus [Chapter 290]), viral, and other infections and systemic causes (Table 4291). Clinical differentiation of sore throat is critical to primary care and emergency management of the airway.

Pharyngitis Bacterial infection accounts for approximately 5 to 10% of pharyngitis in adults compared with 30 to 40% in children. Unfortunately, as many as two thirds of adults with a sore throat are prescribed antibiotics.

GENERAL REFERENCES For the General References and other additional features, please visit Expert Consult at https://expertconsult.inkling.com. Nasopharynx

429  THROAT DISORDERS PAUL W. FLINT Nearly every systemic and infectious disease results in head and neck manifestations, with the majority affecting the upper aerodigestive tract. Diseases of the upper aerodigestive system include infection (acute and chronic; viral, bacterial, and fungal), systemic disease, and neoplasm (Chapter 190), some of which require urgent care or referral to an otolaryngologist. Abnormalities of swallowing, respiratory function, voice, and speech are influenced by the anatomic site involved, the host’s immune status and inflammatory response, the severity of the disease process, and the presence or absence of neurologic involvement. In a patient with hoarseness, current clinical practice guidelines recommend visualization of the larynx for symptoms that persist for 3 months or longer; however, warning signs of a potentially serious or emergent throat condition warrant referral regardless of duration. These conditions include

Uvula Oropharynx

Tongue base

Epiglottis

Hypopharynx

Vocal cords Esophagus Trachea

FIGURE 429-1.  The pharynx (throat) is typically divided into three distinct anatomic regions (nasopharynx, oropharynx, and hypopharynx). (Courtesy Thomas A. Tami, MD.)

CHAPTER 428  Hearing and Equilibrium  

GENERAL REFERENCES 1. Nash SD, Cruickshanks KJ, Klein R, et al. The prevalence of hearing impairment and associated risk factors: the Beaver Dam Offspring Study. Arch Otolaryngol Head Neck Surg. 2011;137:432-439. 2. Schreiber BE, Agrup C, Haskard DO, et al. Sudden sensorineural hearing loss. Lancet. 2010; 375:1203-1211. 3. Pacala JT, Yueh B. Hearing deficits in the older patient: “I didn’t notice anything”. JAMA. 2012; 307:1185-1194. 4. Gaylor JM, Raman G, Chung M, et al. Cochlear implantation in adults: a systematic review and meta-analysis. JAMA Otolaryngol Head Neck Surg. 2013;139:265-272.

2601.e1

5. Baguley D, McFerran D, Hall D. Tinnitus. Lancet. 2013;382:1600-1607. 6. Ozono Y, Kitahara T, Fukushima M, et al. Differential diagnosis of vertigo and dizziness in the emergency department. Acta Otolaryngol. 2014;134:140-145. 7. Kim JS, Zee DS. Clinical practice. Benign paroxysmal positional vertigo. N Engl J Med. 2014;370: 1138-1147. 8. Jeong SH, Kim HJ, Kim JS. Vestibular neuritis. Semin Neurol. 2013;33:185-194. 9. Harcourt J, Barraclough K, Bronstein AM. Meniere’s disease. BMJ. 2014;349:g6544. 10. Huh YE, Kim JS. Bedside evaluation of dizzy patients. J Clin Neurol. 2013;9:203-213.

2601.e2

CHAPTER 428  Hearing and Equilibrium  

REVIEW QUESTIONS 1. Which of the following would not be typical of a conductive hearing loss? A. Marked improvement with amplification B. Ability to hear speech in a noisy background C. Relatively maintained speech recognition D. Loud speech is annoying E. Bone greater than air conduction Answer: D  Patients with conductive hearing loss prefer loud speech, which they can understand as well as normal people can. A, B, C, and E are typical for conductive hearing loss.

4. Which of the following would be an unusual precipitant for benign paroxysmal positional vertigo? A. Turning in bed B. Yoga class C. Driving D. Reaching for something on a high shelf E. Working under an automobile Answer: C  Benign paroxysmal positional vertigo is typically triggered by movement in the vertical plane (plane of the posterior semicircular canal). It would be unusual to make such a movement while driving. The other maneuvers are common precipitating circumstances.

2. Which of the following diagnoses would be least likely to be manifested with a unilateral hearing loss? A. Acoustic neuroma B. Meniere disease C. Otosclerosis D. Brain stem glioma E. Otitis media Answer: D  Brain stem lesions rarely cause unilateral hearing loss unless they involve the root entry zone of the cochlear nerve. Otosclerosis is usually bilateral but often begins unilaterally. The other conditions are typically unilateral.

5. Which of the following would most likely be associated with a positive head-thrust test result? A. Meniere disease B. Lateral medullary infarction C. Cerebellar infarction D. Gentamicin ototoxicity E. Otitis media Answer: D  Gentamicin is remarkably selective for the vestibular system, and the head-thrust test is useful for identifying toxicity at the bedside. The headthrust test result is rarely positive with Meniere disease and would be negative with lateral medullary infarction, cerebellar infarction, and otitis media.

3. Which of the following is least likely to be manifested with vertigo? A. Acoustic neuroma B. Meniere disease C. Lateral medullary infarction D. Migraine E. Vestibular neuritis Answer: A  Acoustic neuroma typically is manifested with unilateral hearing loss or tinnitus. It compresses the vestibular nerve slowly, so central compensation can occur. It rarely is manifested with vertigo. B, C, D, and E commonly are manifested with vertigo.

2602

CHAPTER 429  Throat Disorders  

TABLE 429-1  CLINICAL DIFFERENTIATION OF COMMON CONDITIONS ARISING AS SORE THROAT FEATURE

VIRAL PHARYNGITIS

BACTERIAL TONSILLITIS

PERITONSILLAR ABSCESS

EPIGLOTTITIS

Tonsillar enlargement

Usual

Rare

None

None

Tonsillar exudates

Occasional (infectious mononucleosis)

Usual

Often

None

Tonsillar asymmetry

None

None

Usual

None

Trismus (inability to open jaw)

None

None

Usual

None

Cervical adenopathy

Occasional

Usual (tender)

Usual (tender)

None

Tender larynx

Rare

None

None

Usual

From Tami TA. Throat disorders. In: Goldman L, Schafer AI, eds. Goldman’s Cecil Medicine. 24th ed. Philadelphia: Elsevier Saunders; 2012.

STREPTOCOCCAL INFECTIONS

Group A beta-hemolytic Streptococcus pyogenes (Chapter 290) is the most common cause of bacterial pharyngitis in adults, although it accounts for only 10% of all pharyngitis in adults. Infection is manifested with the rapid onset of sore throat, often accompanied by pain with swallowing, fever, chills, malaise, headache, mild neck stiffness, and anorexia. Hypertrophic tonsils with exudates, foul breath, and tender cervical adenopathy are hallmark findings.1 Some patients have palatal petechiae or a scarlatiniform rash. Rhinorrhea, hoarseness, cough, conjunctivitis, diarrhea, and ulcerative oral lesions are less common. Untreated group A beta-hemolytic S. pyogenes pharyngitis usually resolves within 3 to 7 days. The administration of antibiotics within 24 to 48 hours reduces pain by approximately 1 day, A1  whereas both immediate and delayed antibiotics reduce the risk of suppurative complications.2 Antibiotics also reduce the contagious period from 2 weeks to 24 hours after administration. For prevention of rheumatic fever (Chapter 290), antibiotic therapy must be started within 10 days after the onset of symptoms. The risk of acute poststreptococcal glomerulonephritis (Chapter 121), however, is not affected by antibiotics. To minimize the potential side effects and costs of unnecessary antibiotics, antibiotic therapy should be based on the presence of fever, tender anterior cervical adenopathy, tonsillar swelling or exudates, age, and the absence of cough (Table 429-2). If three or four of these criteria are present, the likelihood of group A beta-hemolytic streptococcal infection is 40 to 60%, whereas fewer criteria are associated with progressively lower probabilities. A rapid antigen test may be obtained because of the residual risk of false-positive or false-negative diagnoses, but its incremental diagnostic value is low, except in borderline cases.3 If the rapid antigen test result is negative but the clinical suspicion remains high, a throat culture specimen should be obtained for confirmation. Antibiotic options (Chapter 290) include penicillin (penicillin VK, 250 mg three times a day or 500 mg twice a day for 5 to 10 days), which is usually chosen to treat acute bacterial pharyngitis, A2  although cefuroxime axetil (250 mg twice a day for 5 to 10 days) is even more effective for primary treatment and can be effective for persistent infection. In patients with proven recurrent infections, clindamycin (300 mg orally three times a day for 10 days) or amoxicillin–clavulanic acid (875 mg orally twice a day, or 500 mg three times a day for 10 days) is recommended. For patients who are allergic to penicillin, azithromycin (500 mg/day for 3 days or a single dose of 2 g) A3  is another alternative. Evidence also suggests that a single dose of oral or intramuscular corticosteroids given at the start of treatment will reduce the pain of severe pharyngitis, especially in children. A4  In patients with recurrent symptomatic episodes despite appropriate antimicrobial therapy, tonsillectomy can decrease future throat infections compared with continued observation. A5  Non–group A beta-hemolytic streptococcal infections (Chapter 290), including groups B, C, and G, can cause acute pharyngitis with a clinical picture that mirrors that of group A beta-hemolytic streptococcal pharyngitis. Glomerulonephritis is a known sequela, whereas rheumatic fever is not. Penicillin or clindamycin, as prescribed for group A beta-hemolytic streptococcus, provides adequate coverage.

NONSTREPTOCOCCAL BACTERIAL PHARYNGITIS

A variety of bacteria other than streptococci can infect the throat. Staphylococcus aureus (Chapter 288) infections, whether caused by methicillin-resistant (MRSA) or methicillin-sensitive (MSSA) strains, usually are manifested with chronic hoarseness. Laryngoscopy typically reveals thickened erythematous vocal folds with edema, whitish debris, and crusting, which may resemble

TABLE 429-2  GUIDELINES FOR THE MANAGEMENT OF PHARYNGITIS* CENTOR SCORE†

PERCENTAGE POSITIVE FOR STREPTOCOCCUS INFECTION

ACP/CDC GUIDELINES

0

7%

Do not test, do not treat

1

12%

Do not test, do not treat

2

21%

Treat if rapid test result positive

3

38%

Option 1: treat if rapid test result positive Option 2: treat empirically

4

57%

Treat empirically

*As recommended by the American College of Physicians (ACP) and the Centers for Disease Control and Prevention (CDC). See Arch Intern Med. 2012;172:847-852. † Calculated as follows: 1 point each for temperature >38° C, absence of cough, presence of swollen and tender anterior cervical nodes, tonsillar swelling or exudate, or age 3 to 14 years; and −1 point for age ≥45 years.

FIGURE 429-2.  Fiberoptic laryngoscopy demonstrating chronic laryngitis secondary to methicillin-sensitive S. aureus infection. Diagnosis is based on culture and biopsy.

leukoplakia (Fig. 429-2). Trimethoprim-sulfamethoxazole (160 mg/800 mg twice daily for 2 to 4 weeks) has been shown to be effective,4 although treatment should be guided by antibiotic sensitivities. Bordetella pertussis (Chapter 313) infections have become more common in adults because of their gradual loss of immune protection after vaccination. Adults typically present with cough, often but not always accompanied by

CHAPTER 429  Throat Disorders  

nonspecific upper respiratory symptoms, fever, and leukocytosis. A B. pertussis serum immunoglobulin G titer higher than 27 IU/mL is highly predictive of recent infection.5 Erythromycin (500 mg four times a day for 14 days) or azithromycin (500 mg single dose orally on day 1, then 250 mg daily on days 2 through 5) is effective treatment. Neisseria gonorrhoeae (Chapter 299) can cause sexually transmitted gingivitis, stomatitis, glossitis, and pharyngitis, especially in men who have sex with men. Treatment is the same as for urogenital disease and should include treatment for chlamydia. Options include ceftriaxone, 250 mg intramuscularly in a single dose; azithromycin, 1 g orally in a single dose; and doxycycline, 100 mg orally twice daily for 7 days. Treponema pallidum (Chapter 319) can cause oral and oropharyngeal ulcerations that involve the lips, tongue, and tonsil. Treatment is the same as that recommended for systemic disease. Chlamydia (Chapter 318), which is commonly associated with pneumonia and bronchitis, also can cause pharyngitis and hoarseness, sometimes as its presenting symptoms. Mycoplasma pneumoniae (Chapter 317), which frequently accounts for 15 to 20% of cases of community-acquired pneumonias, also can cause sore throat, nasal congestion, and coryza. Treatment of chlamydia and mycoplasma infections with tetracyclines, macrolides, and quinolones is the same as for the pneumonias they cause (Chapters 318 and 317). Francisella tularensis (Chapter 311) is a gram-negative bacillus that causes tularemia. Oropharyngeal involvement is associated with fever, pharyngeal erythema, exudative tonsillitis, and tender lymphadenopathy. A false-positive monospot test result and atypical lymphocytosis can mimic infectious mononucleosis. The organism is sensitive to macrolides, fluoroquinolones, and tetracyclines. Corynebacterium diphtheriae infection (Chapter 292) involves the mucosal surfaces of the upper respiratory tract, where it causes a patchy gray-black pseudomembrane in the nasopharynx, oropharynx, larynx, and trachea. About 75% of patients complain of throat pain. Airway obstruction and severe dysphagia are life-threatening sequelae. Antitoxin is given in combination with penicillin, erythromycin, tetracycline, clindamycin, or rifampin. Arcanobacterium haemolyticum (Chapter 292) is a gram-positive bacillus that can cause pneumonia, meningitis, osteomyelitis, brain abscess, and peritonsillar abscess in both normal and immunocompromised patients. Uncomplicated pharyngitis may be treated with erythromycin (500 mg four times a day for 10 days). Complicated infections require intravenous dosing with vancomycin, clindamycin, or cephalexin, with or without gentamicin (see Table 287-4).

2603

infection that has spread to the fascial planes of the neck and chest. These infections require urgent evaluation, usually with a computed tomography scan with contrast enhancement. Aggressive management includes incision and drainage as well as broad-spectrum intravenous antibiotics that cover aerobic and anaerobic bacteria (e.g., clindamycin, 600 mg intravenously every 8 hours; ampicillin-sulbactam, 3 g every 6 hours; or penicillin G, 2 million units every 4 hours plus metronidazole 500 mg every 6 hours).7 Severe infections of the pharynx can cause septic thrombophlebitis of the internal jugular vein (Lemierre’s syndrome), an uncommon but serious complication. Patients typically present with persistent fever, difficulty in swallowing, neck pain, and a neck mass due to an underlying peritonsillar, retropharyngeal, or parapharyngeal abscess.8 Diagnosis is best established with a contrast-enhanced computed tomography scan of the neck. This potentially life-threatening condition is almost always associated with anaerobic infection, especially with Fusobacterium necrophorum or A. haemolyticum (Chapter 297). Infection can extend into the intrathoracic vasculature, and patients can develop bacteremia and septic pulmonary emboli. Treatment should be directed toward anaerobic coverage (e.g., clindamycin, 600 mg every 8 hours, or metronidazole, 500 mg every 6 hours). Anticoagulation with heparin is controversial and generally reserved for persistent septic emboli. Surgical intervention includes drainage of the abscess. Ligation or excision of the jugular vein may be indicated for persistent septic emboli unresponsive to medical management. Mortality rates can be as high as 5%.

VIRAL INFECTIONS

Only about 1% of patients with acute bacterial pharyngitis develop serious suppurative complications.6 The best predictors are severe tonsillar inflammation and severe earache, but most complications occur in patients with neither of these findings. Peritonsillar abscesses usually can be diagnosed on physical examination (Fig. 429-3) and managed by an otolaryngologist in an outpatient setting by surgical or needle drainage. Oral antibiotics, such as amoxicillin (500 mg orally twice a day for 10 days) plus metronidazole (500 mg orally three times a day for 10 days), clindamycin (300 to 600 mg three times a day for 14 days), or amoxicillin-clavulanate (875 mg orally twice a day for 10 days), are recommended. By comparison, patients with deep neck abscesses often have swelling of the external neck, trismus, torticollis, and even a compromised airway due to

In adults, the common cold (Chapter 361) causes 30 to 60% of cases of pharyngitis, with rhinovirus (Chapter 361) accounting for the majority of cases, followed by coronavirus (Chapter 366) and parainfluenza (Chapter 363) (see Table 429-2). U.S. adults experience an average of 2.5 episodes per year of noninfluenza upper respiratory infections, each with a 7.4-day average duration of symptoms. For the entire U.S. population, these 500 million episodes cost an estimated $40 billion annually, in part because of associated systemic symptoms such as fever, cough, and sinusitis and in part because of associated exacerbations of allergies, asthma, and chronic obstructive pulmonary disease. The viral infection that is most likely to be confused with a bacterial infection is mononucleosis. Mononucleosis is caused by Epstein-Barr virus (Chapter 377), which has a seroprevalence of 67% in U.S. children and adolescents aged 6 to 19 years.9 After an incubation period of 3 to 7 weeks, patients present with initial malaise, fever, and chills followed by sore throat, fever, and anorexia. Some patients have associated abdominal discomfort due to splenomegaly or hepatomegaly, headache, stiff neck, and rash. On physical examination, patients have erythematous pharyngitis with exudative tonsillar hypertrophy (Fig. 429-4), prominent lingual tonsils, and adenoid hypertrophy (Waldeyer’s ring). Aphthous-type ulcerations and petechiae may be seen, especially at the junction of the hard and soft palate. Impressive cervical adenopathy is typical, and 50% of patients have splenomegaly. Lymphoid hyperplasia can cause some degree of upper airway obstruction in about 5% of patients. A blood count will show lymphocytosis, usually with more 10% atypical lymphocytes. The heterophil antibody test result is often positive, and Epstein-Barr virus–specific antibody tests are diagnostic. β-Lactam

FIGURE 429-3.  Left peritonsillar abscess identified by bulging anterior pillar and soft palate with midline shift. (Courtesy Thomas A. Tami, MD.)

FIGURE 429-4.  Mononucleosis with symmetrical exudative tonsillitis. (Courtesy Thomas A. Tami, MD.)

PERITONSILLAR ABSCESS AND DEEP SPACE INFECTIONS

2604

CHAPTER 429  Throat Disorders  

antibiotics, which may mistakenly be prescribed, will cause a maculopapular rash in 95% of patients. Laryngeal obstruction may require hospitalization and intravenous corticosteroids (e.g., dexamethasone, 8 to 10 mg intravenously three times a day). Influenza virus infection can include nonexudative pharyngitis, but the predominant symptoms are tracheobronchial, usually accompanied by fever, headache, rhinorrhea, cough, and myalgia, without lymphadenopathy.10 Adenovirus (Chapter 365) can cause pharyngitis associated with fever, nonproductive cough, nasal congestion, myalgia, headache, nausea, vomiting, and diarrhea, especially in outbreaks, such as among military recruits, or in immunocompromised patients. Primary herpes simplex virus (Chapter 374) infection is characterized by pharyngitis with or without gingival stomatitis. Symptoms include sore throat, fever, and malaise; physical findings include erythema and hypertrophy of the tonsils with exudates, often with enlarged, tender cervical nodes. It may be difficult to distinguish clinically from group A beta-hemolytic streptococcal pharyngitis unless patients have herpes-like lesions of the oral cavity or oropharynx. Human immunodeficiency virus infection (Chapter 384) can be manifested as an acute retroviral syndrome that mimics infectious mononucleosis and resolves in 1 to 2 weeks. The diagnosis must be considered in febrile patients with known risk factors. Once infection is established, oral and oropharyngeal infectious ulcerations may be due to herpes simplex virus, cytomegalovirus, syphilis, cryptococcus, histoplasmosis, or mycobacteria. Large, painful, noninfectious aphthous ulcers also can involve the tonsillar fossa, floor of mouth, hypopharynx, and epiglottis (Chapter 425).

FUNGAL INFECTIONS

By far, the most common fungal infection of the oropharynx and larynx is candidiasis. Candida (Chapter 338) is a normal commensal organism of the oral cavity and oropharynx, but it can be an opportunistic infectious agent in immunocompromised patients, patients who have received prior head and neck irradiation, patients with xerostomia or diabetes, and immunocompetent patients who have been treated with antibiotics or with systemic or inhaled steroids. Candida infection is manifested with sore throat, burning of the mouth and tongue, dysgeusia, dysphagia, and hoarseness. White pseudomembranes characteristic of thrush (Fig. 429-5) may involve the oral cavity, oropharynx, hypopharynx, larynx, and esophagus. Treatment includes oral hygiene, probiotics, and topical antifungals (Chapter 338). Fluconazole (200 mg orally once a day for 14 to 21 days) is indicated for laryngeal and esophageal involvement and for recurrent disease. Itraconazole (200 mg orally once a day for 14 to 21 days) should be considered for nonresponders. Other fungal infections may affect the oropharynx and larynx in isolation or as part of a systemic infection. Blastomycosis (Chapter 334) involves the larynx in less than 5% of cases, in which it produces pseudoepitheliomatous

hyperplasia and clinically resembles squamous cell carcinoma. Histoplasmosis (Chapter 332), which is endemic to the Ohio and Mississippi river valleys, can involve the oral cavity, oropharynx, and larynx in immunocompromised patients. Cryptococcosis (Chapter 336) can involve the larynx and occurs most often in an immunocompromised host, in whom pseudoepitheliomatous hyperplasia may occur. Paracoccidioidomycosis (Chapter 335) is the leading cause of fungal laryngitis in South America, especially among farmers. It causes pseudoepitheliomatous hyperplasia that can be misdiagnosed initially as squamous cell carcinoma. Coccidioidomycosis (Chapter 333), which is endemic to southwestern United States, can involve the larynx, in which it results in hoarseness and throat pain and can progress to airway obstruction. For each of these fungal diseases, treatment is similar to what is recommended for the disseminated disease (Chapter 331) that usually accompanies pharyngeal and laryngeal disease.

MYCOBACTERIAL INFECTIONS

The risk that tuberculosis (Chapter 324) infection will involve the oropharynx and larynx is low, with only about 1 to 1.5% of infected tuberculosis patients having involvement of the tonsils or larynx. Tonsillar involvement, which most often occurs in the presence of systemic disease, is manifested as sore throat with exudative tonsillitis and cervical adenopathy. Other diseases that can have a similar clinical presentation include lymphoma (Chapter 185), squamous cell carcinoma (Chapter 190), and sarcoidosis (Chapter 95). Tuberculosis in the larynx is most likely to involve the vocal folds and supraglottis (false vocal folds). It is manifested with hoarseness and odynophagia, and its lesions mimic squamous cell carcinoma. Approximately 50% of cases occur in the presence of active disseminated disease, about one third occur with inactive disease, and the other 15% occur as primary laryngeal disease.11 In contrast, Mycobacterium leprae infection (Chapter 326) is manifested with laryngeal disease in one third of patients with systemic disease. The clinical picture is indistinguishable from that of laryngeal tuberculosis. Atypical mycobacteria rarely involve the larynx, and infection most often is manifested as cervical adenopathy.

CHRONIC TONSILLITIS

Patients may develop deep tonsillar crypts that accumulate debris, such as food or sloughed mucosa, thereby providing an ideal environment for the growth of bacteria, especially anaerobes. Such patients commonly complain of whitish or yellow pieces of semisolid debris on or emanating from their tonsils. These tonsilliths often have a foul taste and odor, and they can cause halitosis. Some patients have chronic sore throats because of the persistent infection. Treatment includes frequent gargling of a hydrogen peroxide mouthwash and occasionally expressing this debris from the tonsil manually. Long-term amoxicillin (500 mg three times a day for 21 days) or clindamycin (300 mg orally three times a day for 21 days) may be effective; however, the presence of Actinomyces, a commensal organism of the oral cavity and oropharynx, is indicative of chronic infection that requires tonsillectomy because even long-term antibiotics are unlikely to be effective.12

Epiglottitis Epiglottitis (supraglottitis) is an uncommon problem in adults and has become even less common in children because of routine Haemophilus influenzae vaccination in children (Chapters 18 and 300). In adults, Streptococcus pneumoniae (Chapter 289) is now the most common organism,13 and adults present with a severe sore throat, odynophagia, fever, and “hot potato” voice. Airway obstruction occurs less frequently than in children, although it should be considered a possibility. Palpation or movement of the larynx causes significant pain. On transnasal fiberoptic laryngoscopy, the larynx typically reveals swelling, erythema, and occasionally exudates of the epiglottis and other supraglottic structures. Patients with a confirmed diagnosis of epiglottitis require intravenous antibiotics (e.g., cefotaxime, 2 g every 6 hours, or ceftriaxone, 1 to 2 g/day, intravenously), and they should be observed in an intensive care unit setting until symptoms improve because of the risk of rapidly progressive airway obstruction.

  NONINFECTIOUS PHARYNGITIS

Laryngopharyngeal Reflux FIGURE 429-5.  Fiberoptic laryngoscopy demonstrating characteristic erythema

and white pseudomembrane secondary to Candida infection.

Patients with gastroesophageal reflux disease (Chapter 138) can develop laryngopharyngeal reflux with intermittent hoarseness, nighttime or chronic cough, postnasal drip, “globus” sensation, reactive airway disease, halitosis,

2605

CHAPTER 429  Throat Disorders  

and brackish or acid taste in the back of the mouth and throat. Findings on laryngoscopy, although nonspecific, may include posterior laryngitis, with swollen and erythematous arytenoid cartilages, thickening of the vocal folds, interarytenoid edema, and thickening of the mucosa. In severe cases, spasm or thickening of the cricopharyngeus muscle, also known as the upper esophageal sphincter (Chapter 138), can cause dysphagia as a result of poor pharyngeal emptying or even spillage of secretions into the larynx with aspiration.14 Treatment should address dietary change, behavioral modification, elevation of the head of the bed at night, and a therapeutic trial of a proton pump inhibitor for up to 3 months (Chapter 138). Empirical treatment without laryngoscopy is reasonable in patients with classic symptoms, but patients who do not respond within 3 months or patients with warning signs (e.g., ear pain, trismus, or odynophagia) require laryngoscopy to exclude more serious causes of hoarseness. In patients with persistent symptoms and a positive pH probe or with evidence of Barrett’s esophagitis, an antireflux procedure (Chapter 138) should be strongly considered.15

SYSTEMIC DISEASES OF THE THROAT AND LARYNGITIS

About 80% of patients with pemphigus (Chapter 439) will have symptoms affecting the nasal cavity, oral cavity, and oropharynx, and half of these patients will have laryngeal involvement.16 Shallow ulcerations with fibrinous material and surrounding erythema are characteristic. Bullous lesions are less likely to be observed because the epithelial layer sloughs during swallowing. Laryngeal involvement may result in stenosis with airway obstruction due to scarring. Upper aerodigestive tract involvement occurs in 35% of patients with pemphigoid (Chapter 439), and 50% of these patients will have laryngeal involvement. Treatment in both disorders consists of high-dose steroids (prednisone, 75 to 100 mg orally per day until remission) during the attack phase and then tapered to a maintenance dose (25 to 50 mg orally every other day). Other immunosuppressive medications, such as azathioprine, cyclophosphamide, or cyclosporine (Chapter 35), may be required in the maintenance phase. Perilesional or intralesional triamcinolone acetonide injections are recommended during the maintenance phase for new lesions.17 Granulomatosis with polyangiitis (Chapter 270) includes laryngeal involvement in 20% of patients, with a predilection for the subglottis (E-Fig. 429-1). Presenting symptoms may include hoarseness, cough, dyspnea, wheezing, and stridor. Flow-volume loops are useful and demonstrate flattening of both inspiratory and expiratory loops characteristic of a fixed extrathoracic obstruction. Patients presenting with airway obstruction require surgical intervention. Active disease with granulation tissue requiring airway management is treated with endoscopic dilation and steroid injection. Systemic immunosuppression is not effective for treatment of laryngeal involvement, so open resection may be required as the disease becomes chronic with the deposition of fibrous tissue.18 Relapsing polychondritis involves cartilage in the ear, nose, and upper and lower airway as well as in the articular joints and costal cartilage. About 50% of patients will develop dyspnea, cough, hoarseness, stridor, or wheezing due to the destruction of cartilage and the resulting loss of structural support of the airway (E-Fig. 429-2).19 Airway obstruction may necessitate stenting or tracheostomy. About 25 to 30% of patients with rheumatoid arthritis (Chapter 264) develop hoarseness, globus symptoms, and difficulty in swallowing. Hoarseness may result from acute inflammation or chronic nodular formation. Bilateral arytenoid joint involvement may impair vocal fold motion and cause airway obstruction with stridor. Surgical intervention may be necessary to open the airway. The role of steroids (systemic or injectable) for airway stenosis in rheumatoid arthritis has not been established. About 1 to 5% of all patients with sarcoidosis (Chapter 95) develop laryngeal involvement, usually manifested as hoarseness. As the disease progresses, however, it can cause a conical stenosis due to thickening of the soft tissue (E-Fig. 429-3). Laryngeal paralysis can be caused by a mass effect or by adenopathy with peripheral nerve compression along the course of the vagus or recurrent laryngeal nerve. Vocal fold involvement responds to intralesional steroid injection, but endoscopic laser excision is recommended in patients with airway symptoms. Amyloidosis (Chapter 188) may deposit anywhere in the upper aerodigestive tract. Hoarseness and cough are the most common symptoms in patients with laryngeal involvement (E-Fig. 429-4), but pharyngeal involvement may be associated with pain. Airway obstruction is rare. Surgical debulking with or without external beam radiation can relieve symptoms and even resolve the lesions.

NEUROLOGIC DISORDERS AFFECTING THE THROAT

Neurologic disorders of the oropharynx, hypopharynx, and larynx may be due to focal diseases or be a local manifestation of generalized neurologic disease. Head and neck manifestations of neurologic and neuromuscular disorders are classified as hyperfunctional and hypofunctional disorders. Hyperfunctional disorders include muscle tension dysphonia, dystonia (Chapter 410), essential tremor (Chapter 410), myoclonus, and stuttering. Hypofunctional neurologic disorders include Parkinson’s disease (Chapter 409), multiple sclerosis (Chapter 411), neuromuscular disorders (Chapters 419 and 422), postpolio syndrome (Chapter 379), myopathies (Chapter 421), medullary disorders, and laryngeal paralysis. Given the variety of disorders with head and neck manifestations, it is critical to identify and to classify the physical findings and associated symptoms. For example, a patient who complains about the sound of the voice may in fact have a normal voice but actually have severe dysarthria and hypernasality secondary to amyotrophic lateral sclerosis. The site of the lesion associated with the neurologic disorder will result in characteristic physical findings and facilitate establishment of a correct diagnosis (Table 429-3).

Hyperfunctional Neurologic Disorders

In dystonia (Chapter 410), spasmodic dysphonia fluctuates moment to moment and day to day. Adductor spasms of the vocal folds produce strained, strangled vocal quality with pitch breaks. Abductor spasms produce breathy hypophonic word breaks. Nonspeech sounds (laughter) and singing voice may be normal. This condition responds well to botulinum toxin injections.20 Muscle tension dysphonia, which may be difficult to distinguish from dystonia, can occur secondary to underlying weakness (paresis, aging) with compensatory hyperfunction. Speech has a rough strained quality, perhaps with pitch breaks, and does not fluctuate moment to moment. It generally responds to voice therapy. Vocal tremor is seen in 30% of patients with essential tremor (Chapter 410) and may be associated with spastic dystonia. Patients have a tremulous, quavering vocal quality, with or without head or hand tremor. Because of involvement of pharyngeal and laryngeal muscles, botulinum toxin may not be effective. Myoclonus (Chapter 410) causes rhythmic contraction of the palate, pharynx, or larynx at a rate of one or two per second. Patients may have audible clicking from the eustachian tube or larynx. Voice may or may not be affected. Palate and vocal folds may be treated with botulinum toxin. Pseudobulbar palsy (Chapters 404 and 419) causes spasticity and hyperreflexia of the bulbar muscles (pharynx, palate, lips tongue, and larynx).

TABLE 429-3  CORRELATING SITE OF LESION WITH PHYSICAL FINDINGS IN NEUROLOGICAL DISORDERS AFFECTING THE THROAT SITE OF LESION

SIGNS

Cortex

Aphasia Aphonia Dysarthria Dysphonia Stridor

Extrapyramidal system

Vocal strain and pitch breaks Tremor Hypophonia/tachyphemia Spasmodic movements Focal, regional, or generalized dystonia

Cerebellum

Ataxia Dysmetria Tremor Incoordination

Brain stem

Flaccid paralysis Associated dense sensory deficit

Peripheral

Focal paresis or paralysis Other cranial nerve ± Palate involvement defines location

Modified from Blitzer A, Alexander RE, Grant NN. Neurologic disorders of the larynx. In: Flint PW, Haughey BH, Lund VJ, et al, eds. Cummings: Otolaryngology–Head and Neck Surgery. 6th ed. Philadelphia: Mosby Elsevier; 2015.

CHAPTER 429  Throat Disorders  

2605.e1

A

E-FIGURE 429-1.  Fiberoptic examination of a patient with granulomatosis with angiitis demonstrating laryngeal involvement at the level of the vocal folds and subglottis.

B

C E-FIGURE 429-2.  Tracheoscopy in a patient with relapsing polychondritis with tracheal involvement. A, On inspiration, negative intrathoracic pressure maintains a patent airway. B, During expiration, positive intrathoracic pressure collapses the airway because of loss of tracheal ring support. C, Computed tomography scan demonstrating thickening of the tracheal wall.

2605.e2

CHAPTER 429  Throat Disorders  

E-FIGURE 429-3.  Supraglottic stenosis secondary to sarcoid demonstrating granulomatous destruction of epiglottis and aryepiglottic folds.

A

B E-FIGURE 429-4.  Amyloid deposition in the hypopharynx (A) and supraglottis and

glottis (B).

2606

CHAPTER 429  Throat Disorders  

Patients develop dysarthria, hypernasality, and a harsh strident strained vocal quality, which is more spastic than spasmodic.

Hypofunctional Neurologic Disorders

Parkinson’s disease affects speech and swallowing in more than 80% of patients. Patients have dysarthria, prosody of speech, hypophonia, tachyphemia, monotonous pitch, and absence of vocal tremor. Laryngeal examination shows bilateral vocal fold bowing with incomplete glottic closure. In advanced disease, vocal fold motion becomes hypokinetic. Pooling of secretions occurs as swallowing dysfunction progresses. Patients benefit from early intervention by speech-language therapists to address both voice and swallowing symptoms. In progressive supranuclear palsy, bulbar symptoms progress more rapidly, with pronounced speech and swallowing difficulty (Chapter 409). In multiple system atrophy (Chapter 409), progressive airway obstruction from bilateral vocal fold motion impairment may necessitate tracheostomy. In myasthenia gravis (Chapter 422), hypernasal speech, palatal weakness, and hypophonia may be accompanied by difficulties with swallowing and respiration. Some patients with amyotrophic lateral sclerosis (Chapter 419) present with bulbar symptoms that result from buccallabial-lingual weakness, which produces speech and swallowing dysfunction, before the definitive diagnosis is made. In multiple sclerosis (Chapter 411), dysphonia and dysarthria are common.

LUMP IN THE THROAT

The sensation of a lump in the throat,21 called globus pharyngeus, should prompt a careful history and physical examination by an otolaryngologist to exclude a serious diagnosis. When no underlying anatomic disease is found, possible causes include gastroesophageal reflux and spasm of the cricopharyngeus. Cognitive-behavioral therapy may be helpful.

HOARSENESS

Hoarseness, also called dysphonia, is characterized by altered voice quality, pitch, loudness, or vocal effort. Relevant history includes smoking status, occupation, and recent procedures involving the neck or affecting the recurrent laryngeal nerve. Symptomatic treatment should be considered in patients who have evidence suggesting a recent bacterial infectious process or gastroesophageal reflux without any historical features, such as ear pain, dysphagia, or odynophagia, or physical findings such as adenopathy or oral lesions suggestive of tumor. Additionally, there is no evidence to support the use of oral corticosteroids in patients with hoarseness. Hoarseness does not resolve within 3 months or the history or physical examination suggests a serious cause, prompt expert laryngoscopy is indicated.22 The symptom of hoarseness invariably points to the larynx as the site of disease. Benign lesions are most common, including vocal nodules (screamer’s nodules), vocal cord cysts, vocal cord granulomas (usually resulting from intubation trauma or laryngeal hyperfunction), and vocal cord papillomas. Malignant neoplasms must be suspected (Chapter 190), especially in patients with a strong smoking history. Of the malignant tumors that can occur in the hypopharynx and larynx, squamous cell carcinoma is the most common and is usually associated with tobacco and ethanol use. However, the incidence of human papillomavirus–

related oropharyngeal squamous cell carcinoma is increasing and should be considered in the nonsmoking population. Squamous cell carcinoma (Chapter 190) can occur on essentially any mucosal surface in the head and neck. Symptoms can range from mild sore throat to hoarseness, severe dysphagia, and odynophagia. Pain is often referred to the jaw or ear. Associated cervical lymph node enlargement is also common in advanced disease. Successful management depends on early detection by a careful examination of the entire upper aerodigestive tract, biopsy with histopathologic examination, and aggressive treatment based on the clinical stage and site of the lesion.

LARYNGEAL PARALYSIS

Laryngeal paralysis most often is manifested as a unilateral paralysis as a result of a mediastinal tumor; surgical trauma during thyroid, carotid, or anterior cervical spine surgery; blunt or penetrating trauma; aortic aneurysm; progressive neurologic disease; or viral or idiopathic causes. The severity of impairment can be determined from subjective criteria based on the patient’s symptoms, such as breathiness, aspiration, and exertional intolerance. Unilateral vocal fold paralysis with a favorable prognosis occurs after blunt trauma, endotracheal intubation, idiopathic vocal fold paralysis, and paralysis associated with viral pathogens (Ramsay Hunt syndrome). In this setting, the severity of aspiration, dysphonia, and electromyographic findings can be used to determine the choice of procedure and timing of intervention. Temporary medialization with collagen injection is warranted in patients with unilateral paralysis and a good prognosis. Patients who have poor prognosis for recovery include those with injury after complete section of the nerve during a surgical resection of tumor, invasion of cranial nerves by a tumor, paralysis associated with thoracic aneurysm, or paralysis due to progressive neurologic disorders. In patients with a low likelihood of recovery, permanent medialization of the paralyzed vocal fold is warranted. Bilateral vocal fold motion impairment, which is less common, has the same causes. Its management is most often directed toward improving the airway because the predominant symptom is airway obstruction.

Grade A References A1. Spinks A, Glasziou PP, Del Mar CB. Antibiotics for sore throat. Cochrane Database Syst Rev. 2013;11:CD000023. A2. van Driel ML, De Sutter AI, Keber N, et al. Different antibiotic treatments for group A streptococcal pharyngitis. Cochrane Database Syst Rev. 2013;4:CD004406. A3. Jorgensen DM. Single-dose extended-release oral azithromycin vs. 3-day azithromycin for the treatment of group A beta-haemolytic streptococcal pharyngitis/tonsillitis in adults and adolescents: a double-blind, double-dummy study. Clin Microbiol Infect. 2009;15:1103-1110. A4. Hayward G, Thompson MJ, Perera R, et al. Corticosteroids as stand alone or add-on treatment for sore throat. Cochrane Database Syst Rev. 2012;10:CD008268. A5. Alho OP, Koivunen P, Penna T, et al. Tonsillectomy versus watchful waiting in recurrent streptococcal pharyngitis in adults: randomised controlled trial. BMJ. 2007;334:939.

GENERAL REFERENCES For the General References and other additional features, please visit Expert Consult at https://expertconsult.inkling.com.

CHAPTER 429  Throat Disorders  

GENERAL REFERENCES 1. Shulman ST, Bisno AL, Clegg HW, et al. Clinical practice guideline for the diagnosis and management of group A streptococcal pharyngitis: 2012 update by the Infectious Diseases Society of America. Clin Infect Dis. 2012;55:e86-e102. 2. Little P, Stuart B, Hobbs FD, et al. Antibiotic prescription strategies for acute sore throat: a prospective observational cohort study. Lancet Infect Dis. 2014;14:213-219. 3. Little P, Hobbs FD, Moore M, et al. Clinical score and rapid antigen detection test to guide antibiotic use for sore throats: randomised controlled trial of PRISM (primary care streptococcal management). BMJ. 2013;347:f5806. 4. Shah MD, Klein AM. Methicillin-resistant and methicillin-sensitive Staphylococcus aureus laryngitis. Laryngoscope. 2012;122:2497-2502. 5. Bock JM, Burtis CC, Poetker DM, et al. Serum immunoglobulin G analysis to establish a delayed diagnosis of chronic cough due to Bordetella pertussis. Otolaryngol Head Neck Surg. 2012;146: 63-67. 6. Little P, Stuart B, Hobbs FD, et al. Predictors of suppurative complications for acute sore throat in primary care: prospective clinical cohort study. BMJ. 2013;347:f6867. 7. Saluja S, Brietzke SE, Egan KK, et al. A prospective study of 113 deep neck infections managed using a clinical practice guideline. Laryngoscope. 2013;123:3211-3218. 8. Gupta N, Kralovic SM, McGraw D. Lemierre syndrome: not so forgotten! Am J Crit Care. 2014;23:176-179. 9. Dowd JB, Palermo T, Brite J, et al. Seroprevalence of Epstein-Barr virus infection in U.S. children ages 6-19, 2003-2010. PLoS OnE. 2013;8:e64921. 10. Yun HC, Fugate WH, Murray CK, et al. Pandemic influenza virus 2009 H1N1 and adenovirus in a high risk population of young adults: epidemiology, comparison of clinical presentations, and coinfection. PLoS OnE. 2014;9:e85094. 11. Benwill JL, Sarria JC. Laryngeal tuberculosis in the United States of America: a forgotten disease. Scand J Infect Dis. 2014;46:241-249.

2606.e1

12. Kutluhan A, Salviz M, Yalciner G, et al. The role of the actinomyces in obstructive tonsillar hypertrophy and recurrent tonsillitis in pediatric population. Int J Pediatr Otorhinolaryngol. 2011;75: 391-394. 13. Isakson M, Hugosson S. Acute epiglottitis: epidemiology and Streptococcus pneumoniae serotype distribution in adults. J Laryngol Otol. 2011;125:390-393. 14. Patcharatrakul T, Gonlachanvit S. Gastroesophageal reflux symptoms in typical and atypical GERD: roles of gastroesophageal acid refluxes and esophageal motility. J Gastroenterol Hepatol. 2014;29: 284-290. 15. Hoppo T, Komatsu Y, Jobe BA. Antireflux surgery in patients with chronic cough and abnormal proximal exposure as measured by hypopharyngeal multichannel intraluminal impedance. JAMA Surg. 2013;148:608-615. 16. Robati RM, Rahmati-Roodsari M, Dabir-Moghaddam P, et al. Mucosal manifestations of pemphigus vulgaris in ear, nose, and throat; before and after treatment. J Am Acad Dermatol. 2012;67: e249-e252. 17. Fortuna G, Mignogna MD. Clinical guidelines for the use of adjuvant triamcinolone acetonide injections in oro-pharyngeal pemphigus vulgaris: the oral medicine point of view. J Oral Pathol Med. 2011;40:359-360. 18. Taylor SC, Clayburgh DR, Rosenbaum JT, et al. Progression and management of Wegener’s granulomatosis in the head and neck. Laryngoscope. 2012;122:1695-1700. 19. Hong G, Kim H. Clinical characteristics and treatment outcomes of patients with relapsing polychondritis with airway involvement. Clin Rheumatol. 2013;32:1329-1335. 20. Sinclair CF, Gurey LE, Blitzer A. Palatal myoclonus: algorithm for management with botulinum toxin based on clinical disease characteristics. Laryngoscope. 2014;124:1164-1169. 21. Foden N, Ellis M, Shepherd K, et al. A feeling of a lump in the throat. BMJ. 2014;348:f7195. 22. Schwartz SR, Cohen SM, Dailey SH, et al. Clinical practice guideline: hoarseness (dysphonia). Otolaryngol Head Neck Surg. 2009;141:S1-S31.

XXVII

MEDICAL CONSULTATION 430 PRINCIPLES OF MEDICAL CONSULTATION

431 PREOPERATIVE EVALUATION

432 OVERVIEW OF ANESTHESIA 433 POSTOPERATIVE CARE AND COMPLICATIONS

434 MEDICAL CONSULTATION IN PSYCHIATRY

2608

CHAPTER 430  Principles of Medical Consultation  

430  PRINCIPLES OF MEDICAL CONSULTATION GERALD W. SMETANA

APPROACH TO MEDICAL CONSULTATION

A general medical or subspecialty medical physician may receive a request to perform a consultation for a variety of purposes. In some settings, a single consultative encounter will be requested, or the consultant will determine that only one visit, either in the inpatient or in the outpatient setting, is necessary. More commonly, the approach will include one or more follow-up visits to meet the goals of the consultation from the perspectives of the requesting physician, patient, and consultant. The consultant generally assumes one of four roles: cognitive consultant, procedural consultant, comanager with shared care, or comanager with principal care. The consultant as a comanager continues to care for a component of the patient’s needs in an ongoing fashion while being careful to coordinate this comanagement with the continuing role of the requesting physician. Finally, in some situations, it may be most appropriate for the physician who initially requested the consultation no longer to play an active role in the care of the patient but rather to transfer ongoing care exclusively to the consultant. From a practical perspective, consulting medical physicians, whether they are generalists or subspecialists, enter into the consultative mode in a relatively limited number of ways. Surgeons may request a preoperative medical consultation to assess operative risk and obtain recommendations regarding perioperative care (Chapter 431) or, after surgery, to seek help in managing specific postoperative complications or assisting in the patient’s long-term management. Both general medical physicians and medical subspecialists appropriately seek help from a subspecialist with particular knowledge in problems outside their own area of expertise to reduce uncertainty. Sometimes these requests are for specific medical procedures, but requests often seek cognitive guidance as well. Finally, noninternists may seek medical consultation for reasons other than perioperative care. For example, a psychiatrist may request a consultation to help determine whether the somatic symptoms of a particular patient represent important medical conditions (Chapter 434). In the peripartum setting, specific complications of pregnancy may require sophisticated medical consultation (Chapter 239). Each of these settings raises different challenges for the medical consultant. In all settings, however, a number of general principles apply and can improve the effectiveness of consultations. Communication between physicians and other team members is critical to the process of consultation in all settings. The burgeoning patient safety movement has developed best practices for “hand-offs” that can be applied to consultations.

SETTING-SPECIFIC CONSULTATIVE ISSUES

An effective consultant must recognize the setting in which the consultation is requested and possess the required content knowledge. Distressingly, a number of studies have demonstrated that the requesting physician and the consultant often have different views on the reasons that a consultation was requested, and this initial disconnection, if present, will doom any medical consultation.

Preoperative Surgical Consultation

In the preoperative setting, the medical consultant should not “clear” a patient for surgery and must avoid the temptation to do so even if asked. Clearance may incorrectly imply that the procedure has no risk or that the medical consultant will take responsibility for having misled the patient and surgeon. Instead, the medical consultant should help determine the inherent risk associated with the proposed procedure for the particular patient, whether the patient is in the best possible condition for surgery, and whether any generic or patient-specific interventions would reduce the risk (Chapter 431). Effective consultation also requires a specialized knowledge base, whether the consultation is focusing on a particular organ system or on overall perioperative risk (Chapter 431). A poor consultant-patient interaction can have a substantial negative effect on a patient’s confidence in the planned therapy.1

Postoperative Surgical Consultation

Surgeons typically request a postoperative medical consultation when a complication has developed that is beyond their area of expertise (Chapter 433). These problems are commonly urgent, so the goal is expeditious consultation and prompt intervention. Management of these problems does not usually differ from management in nonoperative settings. Another reason for a postoperative consultation is to obtain assistance in post-hospitalization care or to facilitate seamless discharge planning. Consultants should aid in the transition to the outpatient or long-term care setting by taking primary or consultative roles as appropriate.

Medical-Medical Consultations

Cross-consultations between medical subspecialists or between a subspecialist and a generalist, in either direction, are quintessential examples of collaboration. For subspecialty consultations, the key is to provide the requested expertise without overstepping into the domain of the expertise of the requesting physician. A growing example of medical-medical consultation is the situation in which a hospitalist assumes principal responsibility for an inpatient admission and then returns the patient to the primary care physician after discharge from the hospital. In this consultative interaction, close communication is critical because primary responsibility for the patient’s care has shifted from the outpatient medical physician to the inpatient physician. This transfer of responsibility is not dissimilar to that occurring when a patient is submitted to the care of a subspecialist for a procedure such as cardiac catheterization or gastrointestinal endoscopy. Similar issues also arise when a critically ill patient with a condition such as a complicated myocardial infarction (Chapter 73) or shock (Chapters 106, 107, and 108) is managed principally by a critical care medical specialist and is then expected to return to the care of the primary physician after hospital discharge. However, a key difference is that the inpatient hospitalist physician, unlike the consulting subspecialist, will not typically have an ongoing comanagement role. Because of the higher risk for discontinuity, effective communication at the time of hospitalization, whenever key issues arise during hospitalization, and at the time of discharge, is even more important in the inpatient hospitalist model than in the other settings in which subspecialists may take on more of a comanagement role. Effective and comprehensive handoffs at the time of hospital discharge improve continuity and reduce the potential for errors and medicolegal liability.

Rapid Response Teams

A recent quality improvement initiative has been the development of rapid response teams. These teams aim to reduce the “failure to rescue,” which often precedes an unplanned intensive care unit (ICU) transfer or non-ICU cardiac arrest. In this model, a prespecified consultative team urgently sees sick, hospitalized patients when a “trigger” abnormality indicates potential impending serious complications. The activation triggers for rapid response teams have varied somewhat among institutions, but there is substantial agreement on what constitutes an appropriate trigger (Table 430-1). Rapid response teams differ from traditional “code blue teams” in several important aspects, the most important of which is their goal to rescue patients before a crisis situation occurs (Table 430-2). Although implementation of these teams has not consistently reduced hospital mortality, most trials have shown a reduction in unplanned ICU transfers and in hospital length of stay. For example, in a trial of 5391 patients at a single academic medical center, patients randomly assigned to a rapid response team had a shorter length of stay, fewer unplanned ICU transfers, and lower mortality in patients with unplanned ICU transfers. A1  Other studies have shown a greater impact on unplanned ICU transfers and a reduction in hospital cardiac arrest rates.2 In a systematic review, rapid response teams significantly reduced non-ICU cardiac arrests and total in-hospital adult mortality.3 This consultative strategy crosses specialties and may include general internal medicine, hospital medicine, and critical care physicians, as well as respiratory therapists and ICU nurses. When an emergency consultation is generated by a trigger event, the consultant’s relationship is primarily with the patient rather than with the referring physician.

Consultations for Special Populations

When consulting for psychiatrists or in the peripartum period, the consultant requires special expertise to understand the different expressions of signs and symptoms in specific populations, as well as how and when to modify typical

CHAPTER 430  Principles of Medical Consultation  

TABLE 430-1  TRIGGERS: CRITERIA FOR MOBILIZING AN IN-HOSPITAL RAPID RESPONSE SERVICE

TABLE 430-3  REASONS TO CONSIDER A SUBSPECIALTY CONSULTATION

VITAL SIGNS

To provide ongoing comanagement: become a partner in inpatient care • For an acute, unstable problem • For a chronic condition

Heart rate • Heart rate 140 beats/min Blood pressure • Systolic blood pressure 30-40 mm Hg below patient’s usual stable blood pressure • Systolic blood pressure >200 mm Hg for >30 min • Diastolic blood pressure >110 mm Hg with symptoms Respiratory rate • Respiratory rate 35 breaths/min • New onset of marked dyspnea, compromised airway, cyanosis Oxygenation • O2 saturation 5 min (except patients with chronic severe hypoxemia) • Need for 100% supplemental O2 or a non-rebreathing O2 mask Temperature • Body temperature >39° C or associated with acute decompensation NEUROLOGIC STATUS • • • •

Acute change in mental status New focal findings Prolonged or repeated seizures ≥2 Point decline in Glasgow coma scale score

GENERAL STATUS • Uncontrollable bleeding • Decreased urine output to 30 ≤30

7-11 ?

1 day 2 days

2 days 3-4 days

Apixaban

>30 ≤30

8-14 ?

1 day 2 days

2 days 3-4 days

DRUG

GENERAL REFERENCES For the General References and other additional features, please visit Expert Consult at https://expertconsult.inkling.com.

432  OVERVIEW OF ANESTHESIA JEANINE P. WIENER-KRONISH AND LEE A. FLEISHER

RENAL DISORDERS

In the United States, more than 40 million procedures, including outpatient procedures that require an anesthetic, are performed annually. Additionally, many invasive procedures outside of the operating room, such as in the gastrointestinal endoscopy and electrophysiology suites, are performed using deep sedation or general anesthesia. With modern techniques, anesthesia causes or contributes to mortality in about 1 per 20,000 healthy patients. Although the worldwide perioperative mortality attributable to anesthesia has declined by more than 90% in the past several decades,1 the overall inpatient postoperative mortality rate remains about 4%, with large variations even among developed countries.2

NEUROLOGIC AND GERIATRIC PROBLEMS

Important aspects of preoperative risk assessment include the type of surgery to be performed, the patient’s underlying medical condition, and the particular demands for anesthesia (Chapter 431). In addition, a number of other issues are relevant to management and anesthetic evaluation.

*Examples include neurosurgery, spine surgery, cardiac, major abdominal, and vascular surgery.

Chronic kidney disease is an independent risk factor for postoperative cardiovascular events and death.21 Patients with chronic kidney disease typically have other comorbid diseases and may also have fluid and electrolyte abnormalities, anemia, and bleeding diatheses, which should be treated and optimized before surgery. Patients maintained on dialysis should ideally undergo dialysis the day before surgery to optimize their volume status, prevent hyperkalemia, and minimize acute shifts in acid-base balance. The risk for a postoperative stroke in unselected patients after general surgery is less than 0.5%, but patients with a history of stroke, older patients, and those undergoing vascular surgery, especially carotid and cardiac surgery, have higher risk. Patients with symptomatic carotid bruits require further investigation and possible intervention before elective surgery (Chapter 407). Patients who newly receive perioperative β-blockers, particularly high doses of metoprolol, appear to be at increased risk for stroke, and the risk must be balanced against the protective effect of β-blockers for perioperative MI. However, there is no evidence that continuation of chronic β-blockade increases risk for postoperative stroke. There is no evidence to support preoperative intervention in patients with asymptomatic bruits before noncardiac surgery. The general recommendation is to delay elective surgery for at least 4 weeks after a stroke, although some data suggest waiting up to 9 months.22 The elderly are at higher risks for a variety of poor postoperative outcomes. Cognitive impairment (Chapters 27 and 28), frailty (Chapter 24), malnutrition, and prior institutionalization all are associated with a poorer prognosis.23

Grade A References A1. Devereaux PJ, Yang H, Yusuf S, et al. Effects of extended-release metoprolol succinate in patients undergoing non-cardiac surgery (POISE trial): a randomised controlled trial. Lancet. 2008;371:1839-1847. A2. Bangalore S, Wetterslev J, Pranesh S, et al. Perioperative beta blockers in patients having noncardiac surgery: a meta-analysis. Lancet. 2008;372:1962-1976. A3. Nowbar AN, Cole GD, Shun-Shin MJ, et al. International RCT-based guidelines for use of preoperative stress testing and perioperative beta-blockers and statins in non-cardiac surgery. Int J Cardiol. 2014;172:138-143. A4. Devereaux PJ, Sessler DI, Leslie K, et al. Clonidine in patients undergoing noncardiac surgery. N Engl J Med. 2014;370:1504-1513. A5. Devereaux PJ, Mrkobrada M, Sessler DI, et al. Aspirin in patients undergoing noncardiac surgery. N Engl J Med. 2014;370:1494-1503. A6. Sanders RD, Nicholson A, Lewis SR, et al. Perioperative statin therapy for improving outcomes during and after noncardiac vascular surgery. Cochrane Database Syst Rev. 2013;7:CD009971. A7. Yealy DM, Kellum JA, Huang DT, et al. A randomized trial of protocol-based care for early septic shock. N Engl J Med. 2014;370:1683-1693. A8. Pearse RM, Harrison DA, MacDonald N, et al. Effect of a perioperative, cardiac output-guided hemodynamic therapy algorithm on outcomes following major gastrointestinal surgery: a randomized clinical trial and systematic review. JAMA. 2014;311:2181-2190. A9. McFalls EO, Ward HB, Moritz TE, et al. Coronary-artery revascularization before elective major vascular surgery. N Engl J Med. 2004;351:2795-2804. A10. Sandham JD, Hull RD, Brant RF, et al. Canadian Critical Care Clinical Trials Group. A randomized, controlled trial of the use of pulmonary-artery catheters in high-risk surgical patients. N Engl J Med. 2003;348:5-14. A11. Yong SL, Coulthard P, Wrzosek A. Supplemental perioperative steroids for surgical patients with adrenal insufficiency. Cochrane Database Syst Rev. 2012;12:CD005367.

PREOPERATIVE ASSESSMENT

Airway Assessment

Assessment of the airway is always necessary, even if regional anesthesia or monitored anesthesia care (local anesthesia with sedation) is planned, because unexpected complications or compromise of airway reflexes may lead to an emergent need to support ventilation. The laryngeal mask airway device allows many patients to be ventilated easily, but it is important to assess the ability to intubate the patient as well as the ability to ventilate. The prevalence of difficult intubation is about 6% for nonobese patients, and reasons for difficulty include airway pathology (e.g., tumors, previous surgery), reduced mobility of the cervical spine, obstructive sleep apnea, or the anatomic relationship between the larynx and trachea.3 In many such patients, endotracheal intubation can be accomplished by using a fiberoptic bronchoscope to place the endotracheal tube through either the nose or mouth. If this approach is not successful, a surgical airway must be created. Criteria for extubation in postoperative patients are similar to those in other patients who receive mechanical ventilation (Chapter 105). Older patients with more severe, comorbid diseases, especially underlying cardiac or pulmonary disease, are more likely to require postoperative reintubation, which is associated with a nine-fold increase in mortality.4

MEDICATION REACTIONS

Malignant Hyperthermia

Malignant hyperthermia (Chapter 434) is characterized by acute hyperpyrexia developing during or immediately after general anesthesia.5 The channels that regulate the duration and amplitude of calcium efflux from the sarcoplasmic reticulum are the ryanodine receptors, which exist as three isoforms. Gain-of-function mutations affecting RyR1, the receptor expressed primarily in skeletal muscle, are present in 1 per 15,000 to 50,000 people and are associated with enhanced sensitivity to halothane and caffeine and with malignant hyperthermia and central core disease. More than 80 distinct mutations have been detected, and mutation of the adult skeletal muscle sodium channel, SCN4A, may also cause the syndrome. Patients with mutations predisposing to malignant hyperthermia function normally at resting conditions, but exposure to volatile anesthetics, including halothane, isoflurane, enflurane, desflurane, and sevoflurane, or exposure to a depolarizing muscle relaxant, succinylcholine, can precipitate life-threatening muscle contractures, increases in heart rate and body temperature, rhabdomyolysis, myoglobinuria, and metabolic acidosis. The mortality rate is 80% in untreated patients but about 5% with current treatment. Note that succinylcholine causes a release of myoglobin from muscle in small amounts even in normal

CHAPTER 431  Preoperative Evaluation  

GENERAL REFERENCES 1. Apfelbaum JL, Connis RT, Nickinovich DG, et al. Practice advisory for preanesthesia evaluation: an updated report by the American Society of Anesthesiologists Task Force on Preanesthesia Evaluation. Anesthesiology. 2012;116:522-538. 2. Fleisher LA, Fleischmann KE, Auerbach AD, et al. 2014 ACC/AHA Guideline on perioperative cardiovascular evaluation and management of patients undergoing noncardiac surgery: executive summary: a report of the American College of Cardiology/American Heart Association Task Force on Practice Guidelines. Circulation. 2014;130:2215-2245. 3. Kristensen SD, Knuuti J, Saraste A, et al. 2014 ESC/ESA Guidelines on non-cardiac surgery: cardiovascular assessment and management: The Joint Task Force on non-cardiac surgery: cardiovascular assessment and management of the European Society of Cardiology (ESC) and the European Society of Anaesthesiology (ESA). Eur Heart J. 2014;35:2383-2431. 4. Bilimoria KY, Liu Y, Paruch JL, et al. Development and evaluation of the universal ACS NSQIP surgical risk calculator: a decision aid and informed consent tool for patients and surgeons. J Am Coll Surg. 2013;217:833-842. 5. Weber M, Luchner A, Seeberger M, et al. Incremental value of high-sensitive troponin T in addition to the revised cardiac index for peri-operative risk stratification in non-cardiac surgery. Eur Heart J. 2013;34:853-862. 6. Rodseth RN, Biccard BM, Chu R, et al. Postoperative B-type natriuretic peptide for prediction of major cardiac events in patients undergoing noncardiac surgery: systematic review and individual patient meta-analysis. Anesthesiology. 2013;119:270-283. 7. Wijeysundera DN, Duncan D, Nkonde-Price C, et al. Perioperative beta blockade in noncardiac surgery: a systematic review for the 2014 ACC/AHA guideline on perioperative cardiovascular evaluation and management of patients undergoing noncardiac surgery: a report of the American College of Cardiology/American Heart Association Task Force on practice guidelines. J Am Coll Cardiol. 2014;64:2406-2425. 8. Dai N, Xu D, Zhang J, et al. Different beta-blockers and initiation time in patients undergoing noncardiac surgery: a meta-analysis. Am J Med Sci. 2014;347:235-244. 9. Hawn MT, Graham LA, Richman JS, et al. Risk of major adverse cardiac events following noncardiac surgery in patients with coronary stents. JAMA. 2013;310:1462-1472. 10. Darvish-Kazem S, Gandhi M, Marcucci M, et al. Perioperative management of antiplatelet therapy in patients with a coronary stent who need noncardiac surgery: a systematic review of clinical practice guidelines. Chest. 2013;144:1848-1856.

2617.e1

11. Agarwal S, Rajamanickam A, Bajaj NS, et al. Impact of aortic stenosis on postoperative outcomes after noncardiac surgeries. Circ Cardiovasc Qual Outcomes. 2013;6:193-200. 12. Bajaj NS, Agarwal S, Rajamanickam A, et al. Impact of severe mitral regurgitation on postoperative outcomes after noncardiac surgery. Am J Med. 2013;126:529-535. 13. Sabaté S, Mazo V, Canet J. Predicting postoperative pulmonary complications: implications for outcomes and costs. Curr Opin Anaesthesiol. 2014;27:201-209. 14. Practice guidelines for the perioperative management of patients with obstructive sleep apnea: an updated report by the American Society of Anesthesiologists Task Force on Perioperative Management of patients with obstructive sleep apnea. Anesthesiology. 2014;120:268-286. 15. Umpierrez GE, Hellman R, Korytkowski MT, et al. Management of hyperglycemia in hospitalized patients in non-critical care setting: an endocrine society clinical practice guideline. J Clin Endocrinol Metab. 2012;97:16-38. 16. Bhangui P, Laurent A, Amathieu R, et al. Assessment of risk for non-hepatic surgery in cirrhotic patients. J Hepatol. 2012;57:874-884. 17. Musallam KM, Tamim HM, Richards T, et al. Preoperative anaemia and postoperative outcomes in non-cardiac surgery: a retrospective cohort study. Lancet. 2011;378:1396-1407. 18. Carson JL, Grossman BJ, Kleinman S, et al. Red blood cell transfusion: a clinical practice guideline from the AABB*. Ann Intern Med. 2012;157:49-58. 19. Douketis JD, Spyropoulos AC, Spencer FA, et al. Perioperative management of antithrombotic therapy: Antithrombotic Therapy and Prevention of Thrombosis, 9th ed. American College of Chest Physicians Evidence-Based Clinical Practice Guidelines. Chest. 2012;141:e326S-350S. 20. Siegal D, Yudin J, Kaatz S, et al. Periprocedural heparin bridging in patients receiving vitamin K antagonists: systematic review and meta-analysis of bleeding and thromboembolic rates. Circulation. 2012;126:1630-1639. 21. Mooney JF, Chow CK, Hillis GS. Perioperative renal function and surgical outcome. Curr Opin Anaesthesiol. 2014;27:195-200. 22. Jørgensen ME, Torp-Pedersen C, Gislason GH, et al. Time elapsed after ischemic stroke and risk of adverse cardiovascular events and mortality following elective noncardiac surgery. JAMA. 2014;312:269-277. 23. Oresanya LB, Lyons WL, Finlayson E. Preoperative assessment of the older patient: a narrative review. JAMA. 2014;311:2110-2120.

2617.e2

CHAPTER 431  Preoperative Evaluation  

REVIEW QUESTIONS 1. A 70-year-old man is scheduled for cataract surgery. Past medical history includes hypertension, hyperlipidemia, and coronary artery disease with a myocardial infarction 10 years ago. He has been asymptomatic since then but has a sedentary lifestyle. He is a former smoker. Medications include aspirin, metoprolol, and atorvastatin. He denies chest pain or dyspnea. Vital signs and physical examination are within normal limits. His electrocardiogram at the time of his last visit 3 months ago showed normal sinus rhythm with left ventricular hypertrophy and evidence of an old inferior wall myocardial infarction. Which of the following would you recommend preoperatively? A. Basic metabolic panel (electrolytes, renal function, glucose) B. Electrocardiogram C. Hemoglobin D. Stress test E. No further testing Answer: E  Despite having multiple risk factors, the patient is undergoing a low-risk procedure, and no further testing would be helpful. His risk for major cardiac complications is less than 1%, and the procedure is not associated with any significant hemodynamic changes or blood loss. Therefore, even an abnormal result (anemia, chronic kidney disease, hyperglycemia) would be unlikely to change management. (Keay L, Lindsley K, Tielsch J, et al. Routine preoperative medical testing for cataract surgery. Cochrane Database Syst Rev. 2012;3:CD007293.) 2. A 60-year-old man is scheduled for colonoscopy and polypectomy. His medical history is pertinent for hypertension, diabetes mellitus, and atrial fibrillation, and he is taking warfarin, metformin, and amlodipine. Which of the following would you recommend regarding perioperative anticoagulation management? A. Continue warfarin B. Continue warfarin but give fresh-frozen plasma 2 hours before surgery C. Stop warfarin 5 days before surgery D. Stop warfarin 5 days before surgery and “bridge” with low-molecularweight heparin preoperatively E. Stop warfarin 5 days before surgery and “bridge” with low-molecularweight heparin preoperatively and postoperatively Answer: C  In general, warfarin needs to be stopped before a polypectomy. This patient has a CHADS2 score of 2, in which case the risk for thromboembolism is low if temporarily stopping warfarin. Furthermore, if full-dose anticoagulation is started too early after the procedure, there is an increased risk for bleeding. The American College of Chest Physicians guidelines recommend stopping warfarin 5 days before the procedure (to allow the international normalized ratio to drop below 1.5) without the need for bridging therapy in this case. (Douketis JD, Spyropoulos AC, Spencer FA, et al; American College of Chest Physicians. Perioperative management of antithrombotic therapy: Antithrombotic Therapy and Prevention of Thrombosis, 9th ed. American College of Chest Physicians Evidence-Based Clinical Practice Guidelines. Chest. 2012;141[2 Suppl]:e326S-50S. Erratum in: Chest. 2012; 141:1129.)

3. A 60-year-old woman is seen preoperatively before a vaginal hysterectomy. She has hypertension, diabetes, chronic kidney disease, and coronary artery disease but no history of myocardial infarction, recent chest pain, or dyspnea. She walks 1 mile 3 times a week and can climb 2 flights of stairs without symptoms. She states she had a negative stress test 5 years ago. She is taking aspirin, nitrates, metoprolol, lisinopril, furosemide, and insulin. Her physical examination is unremarkable, and her preoperative test results are: glucose 150 mg/dL, blood urea nitrogen 40/creatinine 2.1. Her electrocardiogram shows normal sinus rhythm with no ischemic changes. Which of the following would you recommend preoperatively? A. Brain natriuretic peptide B. Two-dimensional echocardiogram C. Exercise electrocardiogram D. Pharmacologic stress test (nuclear or echo) E. No further testing Answer: E  Although she has a revised cardiac risk index score of 3, she has no active cardiac conditions and has a functional capacity of >4 MET, so no further cardiac testing is necessary. Although an elevated preoperative (or postoperative) brain natriuretic peptide level may be associated with an increased risk for complications, the procedural risk is not high, and there is no evidence that any intervention will improve her outcome. Fleisher LA, Fleischmann KE, Auerbach AD, et al. 2014 ACC/AHA Guideline on perioperative cardiovascular evaluation and management of patients undergoing noncardiac surgery: executive summary: a report of the American College of Cardiology/American Heart Association Task Force on Practice Guidelines. Circulation. 2014;130:2215-2245. 4. A 70-year-old man is scheduled to undergo an elective total hip replacement for severe osteoarthritis, which limits his activity. He has coronary artery disease and had a drug eluting stent (DES) placed in his proximal left anterior descending coronary artery 3 months ago. He is on aspirin and clopidogrel and has had no chest pain or dyspnea since the stent placement. Which of the following would you recommend? A. Proceed to surgery; continue aspirin and clopidogrel B. Proceed to surgery; continue aspirin; stop clopidogrel 5-7 days preoperatively C. Proceed to surgery; continue clopidogrel; stop aspirin 5-7 days before surgery D. Proceed to surgery; stop both aspirin and clopidogrel 5-7 days before E. Postpone surgery for at least 3 months; continue aspirin and clopidogrel Answer: E  Current guidelines recommend continuing uninterrupted dual antiplatelet therapy for at least 6 if not 12 months after placement of a DES to minimize the chance of in-stent thrombosis. Although the newest generation of DES may require shorter durations of antiplatelet therapy, at the current time elective surgery should be postponed for at least 6 and preferably 12 months in order to complete the recommended course of uninterrupted dual antiplatelet therapy. At that time, a decision can be made about whether to continue both drugs or to continue just aspirin, which is continued for life. (Darvish-Kazem S, Gandhi M, Marcucci M, Douketis JD. Perioperative management of antiplatelet therapy in patients with a coronary stent who need non-cardiac surgery: a systematic review of clinical practice guidelines. Chest. 2013;144:1848-1856.)

CHAPTER 431  Preoperative Evaluation  

5. A 65-year-old obese woman is seen preoperatively before a total abdominal hysterectomy and bilateral oophorectomy under general anesthesia scheduled for next week. She has a history of asthma, with no recent exacerbations, and cigarette smoking. She has no cough, shortness of breath, or wheezing. She uses albuterol and ipratropium inhalers as needed. Her chest is clear on examination. Which of the following is most likely to reduce her risk for developing postoperative pulmonary complications? A. 10% weight reduction B. 5-day course of broad-spectrum antibiotics C. Incentive spirometry D. Prophylactic corticosteroids E. Smoking cessation

2617.e3

Answer: C  This patient’s risk factors for postoperative pulmonary complications include age, abdominal surgery, general anesthesia, and cigarette smoking. Her asthma is asymptomatic and should not increase her risk. There is no evidence that antibiotics or steroids are helpful in this case. Obesity may increase pulmonary risk, but she will not lose a significant amount of weight in the next week. In order for smoking cessation to improve pulmonary function and decrease complications, she would have to stop at least 4 to 8 weeks before surgery. Lung expansion maneuvers have been show to reduce pulmonary complications and were the only modality given an A level recommendation by the American College of Physicians guidelines. (Qaseem A, Snow V, Fitterman N, et al; Clinical Efficacy Assessment Subcommittee of the American College of Physicians. Risk assessment for and strategies to reduce perioperative pulmonary complications for patients undergoing noncardiothoracic surgery: a guideline from the American College of Physicians. Ann Intern Med. 2006;144:575-80.)

2618

CHAPTER 432  Overview of Anesthesia  

patients. Patients with malignant hyperthermia do not predictably respond to triggering agents, and some patients with malignant hyperthermia have had milder symptoms of malignant hyperthermia after the administration of nontriggering agents. Malignant hyperthermia now often occurs in muted forms, probably because of the decreased use of succinylcholine by anesthesiologists, the diagnostic awareness of malignant hyperthermia by anesthesiologists, the routine use of carbon dioxide monitors so that increases in end-expiratory carbon dioxide are detected quickly, and the availability of dantrolene. If malignant hyperthermia is suspected by obtaining a family history of adverse events with the administration of anesthesia or when a patient has a reaction suspicious for malignant hyperthermia, a muscle biopsy is usually obtained for in vitro contracture testing, which evaluates the muscle contracture responses to caffeine or halothane. Genetic investigations are also recommended, but malignant hyperthermia cannot be excluded on the basis of genetic testing alone because of the diversity of mutations and genes that can be involved in this syndrome. The Malignant Hyperthermia Association of the United States, www.mhaus.org, is available for information to the public, and all medical personnel can get information 24 hours every day on the malignant hyperthermia hotline, 1-800-MHHYPER or 1-800-644-9737. Dantrolene is the drug of choice to prevent and to reverse the symptoms of malignant hyperthermia. Dantrolene decreases muscle sensitivity to caffeine, reduces the calcium release from the sarcoplasmic reticulum, and produces some muscle weakness. Dantrolene comes in 20-mg bottles and must be dissolved in sterile water; the recommended dose is 2.5 mg/kg given rapidly up to 10 mg/kg. Drug should be given every 5-10 minutes until symptoms subside. Other treatments for malignant hyperthermia include: discontinuing the use of any volatile anesthetics; hyperventilating the patient and administering 100% oxygen; administering bicarbonate for severe acidosis; controlling fevers; and maintaining a temperature below 39°C, without causing hypothermia by using iced fluids, surface cooling, and cooling of body cavities if necessary. Monitoring of temperature and vital signs, urinary output, muscle enzymes, glucose, coagulation studies, acid-base status, and gas exchange is recommended. Two other rare congenital myopathies associated with mutations of the RyR1 include central core disease and multiminicore disease. Patients with central core disease present with infantile hypotonia; a muscle biopsy is needed for definitive diagnosis. Multiminicore disease is a nonprogressive congenital myopathy in which infants present with hypotonia, ophthalmoplegia, and arthrogryposis. These children develop scoliosis and eventually may require chronic ventilation. Avoidance of triggering agents is advised for these syndromes and for patients with other myopathies. Both malignant hyperthermia and central core disease are thought to be inherited as autosomal dominant diseases, but extensive genetic analysis has revealed overlapping phenotypes.

Monoamine Oxidase Inhibitors and Serotonin Toxicity

Anesthesiologists routinely ask if patients are taking a monoamine oxidase (MAO) inhibitor because of their many drug interactions with analgesics in perioperative patients. Also, serotonin toxicity has features similar to malignant hyperthermia and must be distinguished from it. Serotonin toxicity, characterized as a triad of neuromuscular hyperactivity (tremor, clonus, myoclonus, hyperreflexia, and pyramidal rigidity), autonomic hyperactivity (diaphoresis, fever, tachycardia, and tachypnea), and altered mental status (agitation, excitement, and confusion) can be precipitated by the coadministration of MAO inhibitors and selective serotonin reuptake inhibitors (SSRIs). Patients who are taking SSRIs have a higher overall perioperative mortality, a higher 30-day readmission rate, and a higher likelihood of bleeding.6 Rigidity, increasing arterial carbon dioxide levels, and fever above 38.5°C are associated with life-threatening toxicity. Ecstasy, or 3,4-met hylenedioxymethamphetamine (MDMA), combined with MAO inhibitors, including moclobemide, can lead to fatalities because it acts as a serotonin releaser. Tramadol, used for pain relief, and venlafaxine, an antidepressant, act as serotonin releasers and are associated with toxicity when used in patients who are taking MAO inhibitors.

Anaphylaxis in the Perioperative Period

The incidence of life-threatening hypersensitivity reactions during anesthesia is 1 : 4000 to 1 : 25,000. Anaphylaxis is caused by immunoglobulin E (IgE)mediated reactions (Chapter 253), whereas anaphylactoid reactions produce the same clinical picture but are not mediated by IgE. Anaphylaxis during anesthesia can present as cardiovascular collapse, airway obstruction,

flushing, or edema of the skin, singly or in combination, so a careful history of any previous allergic reactions to medications and the nature of the reaction must be obtained by the anesthesiologist and other members of the perioperative team. Neuromuscular blocking agents, such as succinylcholine, and opioid analgesics can cause nonimmunologic release of histamine from mast cells and produce a similar clinical syndrome. Antibiotics, protamine, and blood transfusions (Chapter 177), all given routinely during operations, also can elicit a variety of systemic reactions. About 75% of perioperative hypersensitivity reactions appear to be due to muscle relaxants, especially rocuronium and vecuronium, with a mortality of 3 to 6%. In patients with apparent allergic reactions, skin testing is usually performed, and IgE levels are usually obtained to determine whether the patient had an allergic reaction to a perioperative medication.

Latex Allergies

For sensitized patients (Chapter 253), exposure to even low amounts of latexcontaining particles is sufficient to induce a severe anaphylactic reaction. A latex-free operating environment, in which no latex gloves or latex accessories are used, is key in patients with known allergy. Skin prick tests with latex extracts should be considered in patients at high risk for latex allergy. Early aggressive treatment with epinephrine is critical if severe anaphylaxis occurs.

INTRAOPERATIVE MANAGEMENT

There are three general classes of anesthesia: general, regional, and monitored anesthesia care. The same drugs are often used for general anesthesia and monitored anesthesia care; achievement of the two different conditions requires knowledge of the pharmacokinetics of the drugs (Table 432-1).

General Anesthesia

General anesthesia can be achieved with a balanced drug regimen that induces a loss of consciousness, which can range from a deep sedation requiring only airway support to states requiring full ventilatory support because of weakness and loss of respiratory drive. Both intravenous and inhalational drugs can be used to induce and maintain general anesthesia. In contrast, monitored anesthesia care denotes a state in which patients can still control their airway, do not require ventilatory support, but are sleepy, have less pain, and may be amnestic.

Propofol

Propofol, an alkylphenol, is perhaps the most frequently used intravenous anesthetic for induction of anesthesia and is often used for maintenance of anesthesia during short procedures or to achieve deep sedation during monitored anesthesia care. It is lipid soluble and quickly cleared from the central

TABLE 432-1  COMMON ANESTHETIC APPROACHES FOR VARIOUS TYPES OF SURGERY SURGERY ON INTRA-ABDOMINAL OR INTRATHORACIC ORGANS Examples: cardiac surgery, lung resections, gastric bypass General anesthesia usually administered because mechanical ventilation is often required Drugs include premedication for anxiety with midazolam, general anesthesia with volatile anesthetics (desflurane, sevoflurane, nitrous oxide), neuromuscular blockade, and opioid analgesics* Epidural anesthesia and analgesia also used; examples include ropivacaine, lidocaine, with fentanyl SURGERY ON LIMBS Examples: hip replacement, knee replacement, foot or arm surgery Can perform with epidural or spinal anesthesia, depending on the limb. Examples of medications would include tetracaine, lidocaine, ropivacaine, and fentanyl or morphine. Can perform axillary or scalene block; examples include lidocaine and ropivacaine For postoperative pain control: can perform regional blocks that leave the catheter in place, including femoral nerve block, axillary nerve blocks CATARACT SURGERY—LOCAL ANESTHESIA ON EYE WITH OR WITHOUT SEDATION Examples of drugs used for sedation include midazolam and fentanyl *Includes the use of opioids given intraoperatively with effects that extend into the postanesthesia care unit (PACU) or postoperative period, opioids given in the PACU, or opioids given or intended to be given after discharge from the PACU.

CHAPTER 432  Overview of Anesthesia  

compartment, so it is rapidly eliminated even after long periods of continuous infusion. However, the clearance of propofol is changed by gender (men have lower clearance rates than women), size (children require higher doses), age (elderly patients have decreased clearance rates and experience increased effects with the drug), and narcotics, which decrease its clearance. Because of its predilection for causing apnea, propofol should be administered only by someone with expertise in airway management. Propofol also decreases arterial blood pressure, causes pain with injection, and can precipitate myoclonus. Large quantities of propofol can cause the propofol infusion syndrome, which is associated with cardiomyopathy, metabolic acidosis, skeletal myopathy, hyperkalemia, hepatomegaly, and lipemia. Despite these issues, propofol is frequently used because the recovery from propofol is within minutes, even after it is given as a prolonged continuous infusion, in contrast to the longer duration of drug effects seen after the administration of other intravenous sedatives. Propofol is the preferred agent for healthy outpatients undergoing colonoscopy because it leads to earlier discharge and higher patient satisfaction compared with other agents. A1  When combined with midazolam, sedation is deeper, patient satisfaction is higher, and time to discharge is no longer, A2  even when administered by nonanesthesiologists. A3 

Midazolam

Midazolam, a benzodiazepine that produces muscle relaxation through a central mechanism, is hypnotic, sedative, anxiolytic, amnesic, and anticonvulsant. Its amnesic and anticonvulsant effects are mediated through α1subunit-containing γ-aminobutyric acid A (GABAA) receptors, and the anxiolytic and muscle relaxation are mediated through α2-subunit-containing GABAA receptors. Only 20% receptor occupancy is needed to produce anxiolysis, whereas unconsciousness requires 60%. Long-term administration of benzodiazepines produces tolerance, which appears to decrease receptor binding and function. Benzodiazepines cause dose-related depression of the respiratory system, with a peak effect at 3 minutes and significant depression persisting for 60 to 120 minutes. The rate of administration of the drug affects the onset of depression: the faster the drug is given, the quicker the respiratory depression occurs. Benzodiazepines and opioids appear to produce additive respiratory depression, including apnea. Unlike propofol, benzodiazepines used alone decrease blood pressure only modestly. Other drugs, particularly drugs that affect the cytochrome P-450 3A4 enzyme (including azole antifungals, human immunodeficiency virus [HIV] protease inhibitors, and calcium-channel blockers), affect the clearance of midazolam and prolong its half-life significantly. There are several reports of prolonged amnesia in HIV patients who received midazolam for conscious sedation. Midazolam also has an active metabolite and is often associated with delirium in elderly patients (Chapter 28), perhaps because it impairs both implicit and relational memory.

Opioids

Opioids are classified as naturally occurring (morphine, codeine), semisynthetic (heroin), and synthetic (methadone, fentanyl, remfentanil). They can be administered both intravenously and in the neuraxial space (epidural or spinal). There are four opiate receptors (mu, kappa, delta, and nociceptin receptors), which are G protein−coupled receptors. Chronic exposure to agonists leads to cellular adaptation mechanisms that probably are involved in tolerance, dependence, and withdrawal. Clinically, mu agonists are used almost exclusively; mu agonists include morphine, fentanyl, and meperidine. Opioid analgesics are administered because they relieve pain, but they have other important effects, including respiratory depression, decreased gastric emptying, nausea and vomiting, sedation, constipation, pruritus, dependence, and tolerance, when given repeatedly. When opioids are given with propofol or benzodiazepines, there is a synergistic depressive effect on respiration, hence the rationale for monitoring patients who receive medications for conscious sedation.

Ketamine

Ketamine is unique among the intravenous agents because it has analgesic properties and decreases tolerance to opiates. Ketamine produces doserelated analgesia, which may be profound even when patients can keep their eyes open, breathe spontaneously, and protect their own airway with conserved swallowing and cough reflex. Side effects include increased lacrimation, salivation, and muscle tone. Ketamine increases cerebral blood flow, can increase seizure activity, and can produce undesirable psychological reactions; these side effects are dose related and may be minimized by the concomitant use of benzodiazepines. Ketamine is also a bronchial smooth

2619

muscle relaxant and can prevent experimentally induced bronchospasm. Ketamine is usually associated with an increase in blood pressure, heart rate, and cardiac output. These features make ketamine a useful drug for sedating patients with hemodynamic instability.

Dexmedetomidine

Dexmedetomidine is a highly selective α2-agonist that is associated with less respiratory depression and more cooperative behavior than is propofol. Dexmedetomidine also causes hypnosis, analgesia, sympatholysis, and inhibition of insulin secretion. Dexmedetomidine induces sedation with a respiratory pattern and electroencephalographic changes similar to natural sleep. Even high concentrations of dexmedetomidine are associated with preservation of spontaneous respiration; however, when dexmedetomidine is administered in combination with sympatholytic or cholinergic agents, there is a high risk for extreme bradycardia and sinus arrest. Dexmedetomidine is associated with less amnesia than are benzodiazepines. Although propofol and benzodiazepines commonly have been used in critically ill patients to achieve sedation for procedures or for maintenance of mechanical ventilation, dexmedetomidine appears to have significant advantages over benzodiazepines because it causes less delirium and decreases the time that critical care patients spend on ventilators. A4 

Volatile Anesthetics

Volatile (inhalational) anesthetics include desflurane, sevoflurane, isoflurane, and nitrous oxide, as well as halothane, which now is rarely used in the United States. Inhaled anesthetics are absorbed through the respiratory epithelium and mucous membranes of the respiratory tract, and they are excreted mainly by exhalation. Access to the circulation is almost instantaneous, owing to the large pulmonary surface area. The pharmacologic effects of inhaled anesthetics depend primarily on alveolar ventilation, the ventilation-perfusion ratio, coadministered gases, gas flow, and the physicochemical properties of the anesthetic gas rather than on the quantity of drug administered, the extent and rate of absorption, protein binding, excretion, secretion, or metabolism. Based on the available evidence, no inhalational agent appears to be superior to any other. All inhalational agents, with the exception of nitrous oxide, cause dosedependent cardiovascular depression. Severe hepatotoxicity, which led to the discontinuation of the use of chloroform, carbon tetrachloride, and trichloroethylene anesthetics, is seen as fatal hepatic necrosis with in 1 in 10,000 halothane anesthetics. This problem appears to occur much less frequently with isoflurane and desflurane. Mild halothane hepatoxicity is self-limited and can occur with a single exposure, whereas fulminant halothane hepatitis occurs only after multiple exposures to the drug, has a high mortality rate (50%), and is associated with antibodies to halothane-altered antigens. Nitrous oxide, which is the only nonhalogenated agent still used, is not metabolized in human tissues. It irreversibly oxidizes the cobalt atom of vitamin B12, thereby inhibiting the activity of the cobalamin-dependent enzyme methionine synthase. Individuals with vitamin B12 deficiency or with mutations of methionine synthase may be at risk for neurologic injury from nitrous oxide, which should not be used in patients at risk. Exposure to high concentrations of more than 103 ppm may be associated with an increased incidence of abortions and decreased fertility, so exposure should be avoided in patients and personnel at risk. Nitrous oxide is safe in major non-cardiac surgery. A5  General anesthesia can be achieved only by giving combinations of drugs along with nitrous oxide to achieve the desired effects. Based on the available evidence, no one general anesthetic appears to be superior to any other.

Neuromuscular Blockers

Neuromuscular blockers are used to paralyze muscles to facilitate endotracheal intubation and mechanical ventilation, to decrease shivering during induced hypothermia, or to improve conditions for optimal surgery. Succinylcholine causes prolonged depolarization of the neuromuscular junction, thereby resulting in failure to generate an action potential. Within 9 to 13 minutes after 1 mg/kg of succinylcholine, 90% of muscle strength is restored. The very rapid onset and rapid return of muscle function make succinylcholine a useful drug for difficult intubations. Side effects of succinylcholine include hyperkalemia, myalgia, masseter spasm, sinus bradycardia and nodal rhythms, and increased intraocular pressure. Most of the other neuromuscular drugs used by anesthesiologists are nondepolarizing in that they compete with acetylcholine for the neuromuscular junction and can be reversed by increasing the quantity of acetylcholine.

2620

CHAPTER 432  Overview of Anesthesia  

These drugs are categorized by their chemical makeup: steroidal compounds, benzylisoquinolinium compounds, and other chemical compounds. Clinically, a drug is often chosen for its duration of action. Intermediate agents, which act for 20 to 50 minutes and are used most frequently, include vecuronium, rocuronium, atracurium, and cisatracurium. These drugs have different routes of metabolism, so the choice of agent depends in part on the presence of coexisting disease. Sugammadex, a recently developed direct antagonist to the neuromuscular blocking agents, is not approved for use in the United States at the time of this writing. The chronic administration of neuromuscular blocking agents is associated with prolonged paralysis, particularly in patients given concomitant steroids. Other notable interactions with nondepolarizing agents include that antibiotics can increase neuromuscular blockade; magnesium sulfate potentiates neuromuscular blockade; lithium can potentiate neuromuscular blockade with succinylcholine and with pipecuronium; and antiepileptic drugs cause resistance to nondepolarizing muscle blockade so that larger doses must be administered to achieve paralysis; and patients receiving anticonvulsants have accelerated recovery from neuromuscular blockade.

Regional Anesthesia

Regional anesthesia involves the deposition of local anesthetics near nerves, including the deposition of local anesthetics in the epidural space and into the cerebral spinal fluid (CSF). Local anesthetics, which are aminoesters or aminoamides, affect cardiac function, as well as central nervous system function when administered systemically. The binding of the local anesthetic to the sodium channels in the axoplasm prevents opening of the channels and conduction of nerve impulses. The rates of onset and recovery from nerve blockade are controlled by the diffusion of the local anesthetic into and out of the whole nerve. Examples of regional anesthesia include neuraxial techniques, the deposition of local anesthetics near the brachial plexus to anesthetize the arms (axillary or intrascalene blocks), deposition near the femoral or sciatic nerves to anesthetize the legs, deposition near ulnar or radial nerves for lower arm blocks, deposition near the pudenal nerves for groin procedures, and deposition of local anesthesia in the caudal space for groin surgeries. Dentists employ this technique frequently when they inject local anesthesia near various nerves in the oral cavity. Many surgeries, including carotid surgery and the placement of fistulas for dialysis, can be performed with regional anesthesia. Regional anesthetics may also require supplementation with sedation or general anesthesia. The dangers of regional anesthesia include the injection of local anesthesia into the systemic circulation. Systemic toxicity is manifested as convulsions and respiratory depression, which can require assisted ventilation. Tinnitus, visual and auditory disturbances, and dizziness are signs of milder central nervous toxicity. Cardiac toxicity can be manifested by decreases in heart rate, prolonged conduction times, and negative inotropic effects. Bupivacaine toxicity is associated with ventricular fibrillation. Intralipid 20% at various doses (1.5 ml/kg rapid bolus [∼100mL in average adult] followed by infusion of 0.25 ml/kg/min for 10 minutes) has been reported in case reports and in animal studies to reverse these toxic effects, although the optimal dosing has yet to be determined. Furthermore, the prolonged duration of many of the local anesthetics may require the institution of cardiopulmonary bypass until the drugs are metabolized.

Neuraxial (Spinal and Epidural) Anesthesia and Analgesia

Spinal anesthesia is the instillation of local anesthetics into the CSF. Epidural anesthesia is the instillation of larger volumes of local anesthetics into the epidural space, which is the potential space that exists just before the CSF. Spinal anesthesia is associated with an increased incidence of headache in younger patients, so epidural anesthesia is often used in younger patients. Complications of epidural and spinal anesthesia and analgesia include failed blocks, postdural puncture headaches, and toxicity from the local anesthetics. Another major concern of neuraxial anesthesia is that patients on antiplatelet agents may develop epidural hematomas, although epidural hematoma remains a rare event, occurring in fewer than 1 in 150,000 operations even in the presence of potent antiplatelet agents. Other more rare complications of epidural and spinal anesthetics, in addition to the effects of local anesthesia outlined previously, include intracranial subdural hematoma, transverse myelitis, hypotension, and cardiac arrest. Postoperative epidural analgesia, by which either local anesthesia or local anesthesia and narcotics are instilled into the epidural space for postoperative pain control, is associated with superior pain control, lower doses of

opioids, improved bowel mobility, slightly decreased length of stay in the intensive care unit, and a slight decrease in the requirement for mechanical ventilation. A6 

GENERAL VERSUS REGIONAL ANESTHESIA

The decision regarding what type of anesthesia should be administered often depends on the requirements of the surgery. For example, laparoscopic surgery requires general anesthesia because the insufflations of gases impair the ability to breathe adequately. General anesthesia is also required for surgeries on the airway or thorax because mechanical ventilation is usually needed to sustain adequate respiration. Low tidal volume and low positive end-expiratory pressure are preferred. A7  Procedures that do not allow any movement (e.g., precise procedures in the brain) often require general anesthesia and paralysis. For patients in whom the intraoperative technique could include general anesthesia, regional anesthesia, or a combination of the two, regional anesthesia may minimize pulmonary complications, but the data are conflicting. Side effects of general anesthesia depend on the drugs used to achieve anesthesia, whether neuromuscular blockade is administered, and whether mechanical ventilation is used. Complications of endotracheal intubation include local pain, trauma to the airway, swelling, vocal cord paralysis, increased bronchospasm, and death from improper placement. Volatile anesthetics are associated with postoperative atelectasis (Chapter 90), whereas regional anesthesia helps preserve respiratory dynamics. Postoperative cognitive dysfunction (Chapter 28) does not seem to depend on the type of anesthesia administered.

NAUSEA AND VOMITING

Postoperative nausea and vomiting are more likely with volatile anesthetics but also are common when perioperative opioids are administered. Prophylactic ondansetron, dexamethasone, and droperidol each reduce postoperative nausea and vomiting, independently, by about 26%, with the main predictor for efficacy being the patient’s risk for nausea and vomiting. A8  It should be noted that droperidol has received a “black box” warning from the U.S. Food and Drug Administration, so it is not used very often in the United States. Total intravenous anesthesia with propofol reduces postoperative nausea and vomiting by only about 20%, often because narcotics are still administered. The use of spinal or epidural anesthesia may decrease the incidence of nausea and vomiting. In addition to general anesthesia, risk factors for postoperative nausea and vomiting include female gender, a prior history of nausea and vomiting, a history of motion sickness, nonsmoking, and intended administration of opioids for postoperative analgesia. If three or more risk factors are present, patients generally are recommended to receive at least two prophylactic pharmacologic antiemetic agents of different classes (e.g., selected among ondansetron or another 5-HT3 antagonist, droperidol, dexamethasone, scopolamine, or phenothiazides) preoperatively for the prevention of nausea and vomiting.

Grade A References A1. Wang D, Chen C, Chen J, et al. The use of propofol as a sedative agent in gastrointestinal endoscopy: a meta-analysis. PLoS ONE. 2013;8:e53311. A2. Wang D, Wang S, Chen J, et al. Propofol combined with traditional sedative agents versus propofolalone sedation for gastrointestinal endoscopy: a meta-analysis. Scand J Gastroenterol. 2013;48: 101-110. A3. Molina-Infante J, Dueñas-Sadornil C, Mateos-Rodríguez JM, et al. Nonanesthesiologistadministered propofol versus midazolam and propofol, titrated to moderate sedation, for colonoscopy: a randomized controlled trial. Dig Dis Sci. 2012;57:2385-2393. A4. Riker RR, Shehabi Y, Bokesch PM, et al. Dexmedetomidine vs midazolam for sedation of critically ill patients. JAMA. 2009;301:489-499. A5. Myles PS, Leslie K, Chan MT, et al. The safety of addition of nitrous oxide to general anaesthesia in at-risk patients having major non-cardiac surgery (ENIGMA-II): a randomised, single-blind trial. Lancet. 2014;384:1446-1454. A6. Popping DM, Elia N, Van Aken HK, et al. Impact of epidural analgesia on mortality and morbidity after surgery: systematic review and meta-analysis of randomized controlled trials. Ann Surg. 2013;259:1056-1067. A7. Hemmes SN, Gama de Abreu M, Pelosi P, et al. High versus low positive end-expiratory pressure during general anaesthesia for open abdominal surgery (PROVHILO trial): a multicentre randomised controlled trial. Lancet. 2014;384:495-503. A8. Apfel CC, Korttila K, Abdalla M, et al. A factorial trial of six interventions for the prevention of postoperative nausea and vomiting. N Engl J Med. 2004;350:2441-2451.

GENERAL REFERENCES For the General References and other additional features, please visit Expert Consult at https://expertconsult.inkling.com.

CHAPTER 432  Overview of Anesthesia  

GENERAL REFERENCES 1. Bainbridge D, Martin J, Arango M, et al. Perioperative and anaesthetic-related mortality in developed and developing countries: a systematic review and meta-analysis. Lancet. 2012;380:1075-1081. 2. Pearse RM, Moreno RP, Bauer P, et al. Mortality after surgery in Europe: a 7 day cohort study. Lancet. 2012;380:1059-1065. 3. De Jong A, Molinari N, Terzi N, et al. Early identification of patients at risk for difficult intubation in the intensive care unit: development and validation of the MACOCHA score in a multicenter cohort study. Am J Respir Crit Care Med. 2013;187:832-839.

2620.e1

4. Ramachandran SK, Nafiu OO, Ghaferi A, et al. Independent predictors and outcomes of unanticipated early postoperative tracheal intubation after nonemergent, noncardiac surgery. Anesthesiology. 2011;115:44-53. 5. Riazi S, Larach MG, Hu C, et al. Malignant hyperthermia in Canada: characteristics of index anesthetics in 129 malignant hyperthermia susceptible probands. Anesth Analg. 2014;118:381-387. 6. Auerbach AD, Vittinghoff E, Maselli J, et al. Perioperative use of selective serotonin reuptake inhibitors and risks for adverse outcomes of surgery. JAMA Intern Med. 2013;173:1075-1081.

2620.e2

CHAPTER 432  Overview of Anesthesia  

REVIEW QUESTIONS 1. A 75-year-old healthy woman asks your opinion if she should undergo an elective back operation. She is concerned about her perioperative risks for mortality. You tell her that her that perioperative mortality is mostly determined by which of the following? A. Patient’s underlying medical problems B. The surgical procedure C. The anesthetic D. The volume of procedures done at a hospital Answer: A. 2. A 55-year-old woman comes to you for a preoperative assessment. She has a long history of refractory depression. You inquire whether she is on monoamine oxidase inhibitors, so your list of medications should include which of the following? A. Selegiline B. Moclobemide C. Pargyline D. Isoniazid Answer: All are correct.  See Gillman PK. Monoamine oxidase inhibitors, opioid analgesics and serotonin toxicity. Br J Anaesth. 2005;95:434-441.

3. You are treating a 35-year-old man who is on chronic opioids for pain. He comes into the hospital with a broken limb, and his pain is not adequately treated despite increasing doses of opioid analgesics. You suggest ketamine because it will do all of the following except which one? A. Decrease the need for opioids B. Provide analgesia C. Improve sedation D. Decrease respiration Answer is D.  Ketamine preserves respiration.

CHAPTER 433  Postoperative Care and Complications  

433  POSTOPERATIVE CARE AND COMPLICATIONS DONALD A. REDELMEIER

POSTOPERATIVE CARE

Overview

2621

because a single problem arises at an awkward moment, such as a patient in whom acute dyspnea develops when another patient is having a seizure. Still other mistakes relate to the fallibility of human memory and attention, such as when a normal blood glucose value in the morning leads clinicians to presume that the level is still normal at night. These errors can result in substantial harm, failures of clinicians to learn from past mistakes, and unprofessional reactions related to embarrassment. None of these patterns are unique to postoperative care, yet the fast and unfamiliar terrain of surgical settings can make even simple mistakes difficult to avoid.

Checking Orders

Postoperative medical complications are common, potentially fatal, and variable across different settings. Large national studies show about a two-fold difference in risk for mortality between high- and low-ranked hospitals. However, analyses disagree about how much these differences reflect a greater incidence of each complication (failure of prevention around the time of surgery), a heightened lethality of each complication (failure to rescue in the aftermath of surgery), or the differences in the severity of disease or surgical skill. Regardless of the explanation, the purpose of medical consultation is to relieve human suffering by the prevention, detection, and correction of postoperative complications. The main constraint is that the consultant often has limited ongoing direct contact with the patient before or after the perioperative interval.

The first method for reducing errors after surgery is to check the postoperative orders already written for the patient. Such double-checking is a tedious task, and clinicians often direct insufficient attention to this review in the faulty belief that most of the work is already done. Ironically, checking orders written by another clinician requires more than customary attention because of the challenges of following someone else’s legibility, sequencing, and preferences. The set of orders may need to be read twice: once for errors of commission (e.g., a calcium-channel blocker ordered at the wrong dose) and once for errors of omission (e.g., a β-blocker inadvertently not reordered after surgery). A classic mistake on postoperative orders is failure to follow through on interventions initiated immediately before surgery (e.g., delirium tremens prophylaxis). A particularly vexing issue is the need for repeated rechecking on subsequent days (e.g., new orders for sedative drugs).

Effective Teamwork

Recommended Prophylaxis

Focusing on Recovery

Future Prevention

The medical consultant (Chapter 430) in the postoperative setting must have both a knowledge of medicine and an appreciation of the team psychology that can improve the outcomes of patients.1 In contrast to other settings, the internist is not the team leader, often does not maintain an ongoing relationship with the patient, and does not have the authority of the most responsible physician. Moreover, patients may be dispersed across diverse surgical services, each with its own orientation and culture. The challenges of coordination and communication are enormous, particularly given the multiple other health care professionals involved in complex surgical cases. Considerable tact is often needed to avoid antagonizing the surgeon, disrupting the team’s dynamics, or inducing a cascade of cumbersome inopportune testing. The development and use of safety checklists can be an effective way for teamwork to improve outcomes.2 Facilitating the patient’s recovery from surgery differs conceptually from managing patients with acute exacerbations of chronic disease. In the postoperative setting, many therapies need to be stopped at some point because the patient has recovered, such as discontinuing a urinary catheter because the patient can now void spontaneously or discontinuing a major tranquilizer because the patient is now oriented and coherent. Discontinuation of many other interventions requires substantial judgment, such as the decision when to discontinue intravenous access, supplemental oxygen, and intermittent laxatives. Much depends on experience and reconsideration of an individual patient’s situation on a regular basis.

Reading Anesthesia Records

A focused review of the anesthesia record is essential because the consultant is rarely present during the operation. Perhaps the most basic information to identify is the date of surgery because the time elapsed helps in interpreting the patient’s current state of recovery. Sometimes the date is not immediately evident if more than one surgery has been performed, a planned operation was canceled, or misquotations have arisen. Data about the duration of surgery, type of anesthesia (e.g., regional, spinal, or general [Chapter 432]), and major intraoperative events help establish reasonable expectations about the future course as well as the possibility of specific complications (e.g., epidural hematoma after spinal anesthesia). Sharing some of the basic data with the patient is often helpful because many individuals either benefit from repetition or are not otherwise informed.

Patterns of Mistakes

Medical errors (Chapter 12) that arise in postoperative care often seem mundane in retrospect yet can be lethal if undetected. Some patterns of mistakes have the feature of “double trouble,” such as when a patient has both a potassium level of 2.0 mEq/L and an international normalized ratio of 2.0 but care focuses on only one of these abnormalities. Other mistakes occur

Some complications are sufficiently frequent and serious that routine prophylaxis is merited in the postoperative setting. For example, systemic anticoagulation is indicated for most patients at risk for postoperative deep venous thrombosis (Chapters 38 and 81). The 2012 American College of Chest Physicians evidence-based clinical practice guidelines provide specific recommendations for both nonorthopedic A1  and orthopedic A2  surgery patients. Gastric acid suppression (Chapter 139) is justified for patients at high risk for postoperative gastric bleeding. Parenteral antibiotics are indicated for patients undergoing prosthetic joint replacement. In contrast, antibiotic prophylaxis is indicated only for selected patients who are at high risk for endocarditis (Chapter 76). The optimal method for gauging whether a patient is at high risk for each complication is contentious and thereby leads to variation in practice patterns across different settings. A postoperative consultant who maintains communication, facilitates the patient’s recovery, and avoids postoperative mistakes also has the chance to initiate medical interventions for general medical care. Such opportunities for prevention might include influenza vaccination, colon cancer screening, and cholesterol reduction. The main advantage of such comprehensive care is that it conforms to the ideal of providing all services possible to the individual. The main disadvantage of such comprehensive care is the potential for creating unintended chaos, confusion, or misquotation (Chapter 430). Such unintended consequences distract the surgical team from the primary goal and also carry some risk for side effects at a time when the patient is trying to recover from surgery. Many effective postoperative consultants will defer such opportunities for prevention to the physicians who assume longterm responsibility for the patient’s care.

COMPLICATIONS

Symptoms Chest Pain

Chest pain is a common problem after surgery and has an extensive differential diagnosis (Chapter 51). In the postoperative setting, the immediate consideration is an acute ischemic myocardial event. The diagnosis of a perioperative myocardial infarction (MI) differs somewhat from communityacquired MI (Table 433-1). Interpretation of a patient’s symptoms, examination findings, and electrocardiogram is often problematic because of changes related to surgery and anesthesia. Instead, diagnosis is heavily dependent on biomarkers, such as an elevated troponin level, especially because many postoperative MIs are painless. Among patients undergoing noncardiac surgery, the peak postoperative troponin level during the first 3 days after surgery is significantly associated with 30-day mortality, even if patients do not have any other evidence of an acute MI.3 Management priorities include supplemental oxygen, heart rate control, and correction of severe anemia. Thrombolysis is often contraindicated, but percutaneous coronary intervention may

2622

CHAPTER 433  Postoperative Care and Complications  

be considered. In the absence of data from randomized trials, other therapies, such as aspirin, clopidogrel, nitrates, statins, and angiotensin-converting enzyme inhibitors, should be used on a case-by-case basis (Chapter 73).

Dyspnea

Shortness of breath (Chapter 83) after surgery has an extensive differential diagnosis (Table 433-2). The three key considerations are fluid overload/ heart failure (Chapter 58), pulmonary embolism (Chapter 98), and air space disease (a continuum encompassing atelectasis [Chapter 90], bronchitis [Chapter 96], aspiration [Chapter 94], mucous plugging, and pneumonia).

TABLE 433-1  CRITERIA FOR DIAGNOSIS OF POSTOPERATIVE MYOCARDIAL INFARCTION The diagnosis of perioperative MI requires any one of the following criteria. Criterion 1: A typical rise in the troponin level or a typical fall in an elevated troponin level detected at its peak after surgery in a patient without a documented alternative explanation for an elevated troponin level (e.g., pulmonary embolism) or a rapid rise and fall in CK-MB only if troponin measurement is unavailable.* This criterion requires that one of the following criteria be met: Ischemic signs or symptoms (e.g., chest, arm, or jaw discomfort; shortness of breath; pulmonary edema) Development of pathologic Q waves on an ECG Changes on an ECG indicative of ischemia Coronary artery intervention New or presumed new cardiac wall motion abnormality on echocardiography or new or presumed new fixed defect on radionuclide imaging Criterion 2: Pathologic findings of acute or healing MI

Distinguishing among these considerations requires focusing on the speed of onset, timing relative to surgery, vital signs, findings on oximetry, and physical examination findings (Chapter 83). Fluid overload is most commonly seen soon after the cessation of positive-pressure ventilation or vasodilating analgesia. It is also common 3 to 5 days postoperatively when fluid that had been “third spaced” is mobilized into the intravascular compartment. Interventions that are safe in most situations include administration of oxygen and withholding of sedation. The use of continuous positive airway pressure can reduce the rate of reintubation in hypoxemic postoperative patients. A3  Other interventions that will be helpful or harmful, depending on the specific situation, include diuretics, opioids, elaborate medical imaging, and vigorous physiotherapy.

Anorexia

Loss of appetite (Chapter 132) after surgery has an extensive differential diagnosis that can be narrowed substantially if the patient was eating properly before surgery. The immediate priority is to search for and correct underlying contributors. Oral, enteral, or parenteral support is not the priority initially, although such support may become necessary. Drug toxicity is a particularly common, easily detected when considered, and a rapidly reversible contributor to postoperative anorexia. Anatomic abnormalities are usually evident by medical imaging studies. Other common metabolic contributors include abnormalities in electrolytes, calcium, phosphorus, and magnesium. Acalculous cholecystitis (Chapter 155) is an important postoperative complication that must be considered in a patient with right upper quadrant tenderness.

Vomiting

Criterion 3: Development of new pathologic Q waves on an ECG if troponin levels were not obtained or were obtained at times that could have missed the clinical event *Because CK-MB is both less sensitive and less specific than troponin levels in the perioperative setting than in other settings, it should be used for diagnostic purposes only when troponin levels are not obtainable. CK-MB = creatine kinase MB isoenzyme; ECG = electrocardiogram; MI = myocardial infarction. Reproduced with permission from Devereaux PJ, Goldman L, Yusuf S, et al. Surveillance and prevention of major perioperative ischemic cardiac events in patients undergoing noncardiac surgery: a review. CMAJ. 2005;173:779-788.

Vomiting is the extreme form of nausea in the postoperative setting, and the two symptoms share the same differential diagnosis. In most patients, vomiting is unexpected and merits immediate attention. Initial management is to ensure that the patient’s airway is protected, to discontinue oral medications (and find parenteral substitutes if necessary), and to consider insertion of a nasogastric tube. In patients after gastrointestinal surgery, the priority considerations include the possibility of an anastomotic leak, peritoneal abscess, and other anatomic abnormality. In patients after operations on more remote parts of the body, the priorities are emetogenic medications (such

TABLE 433-2  DISTINGUISHING AMONG COMMON CAUSES OF ACUTE POSTOPERATIVE DYSPNEA PULMONARY AIR SPACE

FLUID OVERLOAD/HEART FAILURE

PULMONARY THROMBOEMBOLISM

CHARACTERISTICS OF TIMING Days since surgery

1-7 days

0-5 days

5-28 days

Speed of onset

1-3 days

1-24 hours

1-5 minutes

PREVIOUS HISTORY Previous lung disease

++ ++

Previous heart failure

++

Previous venous thrombosis ABNORMAL VITAL SIGNS Temperature Heart rate

+

+

+

Blood pressure

+

+

++

Respiratory rate

+

++

+

Oximetry

++

+

+

+

+

+

++

PHYSICAL EXAMINATION Jugular venous distention Pulmonary rales

++

S3 gallop RESPONSE TO TREATMENT Oxygen

+

+

+

Anticholinergic bronchodilators

+

+

+

Withdrawal of sedatives

+

+

+

Aggressive physiotherapy

++

Diuretics/afterload reduction

++

CHAPTER 433  Postoperative Care and Complications  

as postoperative chemotherapy), gastroparesis associated with autonomic neuropathy, and fecal impaction. Multiple antinausea medications are available for symptomatic relief (e.g., prochlorperazine, ondansetron, dexamethasone, droperidol) and act in an additive manner when they are used in combination (e.g., prochlorperazine 5 mg intramuscularly plus ondansetron 4 mg intramuscularly). A4  If no reversible contributor is identified, the default diagnosis is prolonged idiopathic ileus, and a therapeutic trial of intravenous neostigmine can be considered (e.g., neostigmine 2.5 mg intravenously during 5 minutes).

Diarrhea

Diarrhea (Chapter 140) is relatively rare after surgery and involves a limited number of possibilities if the patient’s bowel movements were normal before surgery. In such cases, the situation represents an acute-onset diarrhea that is usually secretory in nature. The immediate priority is to exclude toxic megacolon, which is caused by overgrowth of toxigenic Clostridium difficile and is a potential emergency. Clinical evaluation for toxic megacolon requires assessment for tachycardia, hypotension, delirium, and other signs of sepsis rather than waiting on initial stool studies for confirmation of infection with C. difficile. Risk factors for antibiotic-associated diarrhea include advanced age, use of broad-spectrum antibiotics (e.g., third-generation cephalosporins), and unknown host susceptibility factors (e.g., past episodes of pseudomembranous colitis). A definitive diagnosis is frequently never established, and treatment focuses on feeding the patient a lactose-free diet while avoiding intestinal paralytics. Complete resolution is typical, provided adequate fluid and electrolyte levels are maintained.

Weakness

Generalized weakness after surgery is almost inevitable, but focal weakness may reflect nerve damage caused by intraoperative positioning (e.g., damage to the facial nerve after carotid endarterectomy) and rarely indicates a new intracranial event (e.g., intracerebral bleeding [Chapter 408] secondary to anticoagulation). Neurologic deficits are often overlooked during the initial postoperative interval and may become apparent only after the patient has regained strength elsewhere in the body. Conversely, new deficits that are evident early after surgery and resolve rapidly thereafter may reflect an old stroke that was fully compensated during less stressful circumstances. Medical imaging of the brain is worthwhile if no explanation is apparent on initial assessment. Nonfocal weakness commonly responds to physical therapy.

Delirium

Changes in mental status after surgery are common, especially in elderly patients (Chapter 28), and can be remarkably difficult to correct. The immediate priorities are to determine whether the impairment is acute or chronic and to detect easily reversible contributors (such as infection, hypoglycemia, and alkalosis). A complete assessment is often unnecessary if the patient had normal mental status before surgery because many dementia syndromes are thereby excluded (such as vitamin B12 deficiency, tertiary syphilis, and Alzheimer’s disease). Immediate treatment usually focuses on discontinuation of medications such as anticholinergics, narcotics, and tranquilizers. The patient may also benefit from the continuous presence of friends and family members, who can provide frequent orientation and constant attention. A sense of patience is necessary because delirium rarely resolves instantly. The benefit of low-dose neuroleptics (e.g., risperidone, 0.5 mg by mouth twice daily) remains uncertain. Delirium after surgery is associated with a significant decline in cognitive ability during the next year, with a trajectory characterized by an initial decline and prolonged impairment.4

Seizures

The development of uncontrolled seizures (Chapter 403) after surgery is rare. The immediate priorities are to exclude status epilepticus, to uncover any past history of a seizure disorder, and to identify provocative factors. Neurosurgical patients typically undergo a standardized treatment protocol, including steroids and imaging. Other conditions that can cause abnormal movements must be excluded, such as septic rigors, delirium tremens, Parkinson’s disease, major psychopathology, hypothermic shivering, and hypercapnic asterixis. Additional considerations include detection and correction of any underlying metabolic abnormalities, such as hypocalcemia, hypoxemia, hyponatremia, hypophosphatemia, and drug toxicity. Treatment focuses primarily on reversing the underlying precipitating cause and providing nonspecific care with benzodiazepines, phenytoin, and ongoing monitoring.

2623

Signs Hypertension

Hypertension may reflect a variety of disorders and must be treated in a manner that neither overreacts nor underreacts to the situation. Hypertension is particularly common after neurosurgical procedures or carotid endarterectomy. The initial assessment focuses on whether the patient has chronic hypertension based on the past history, current electrocardiogram, or findings on funduscopy. Other potential causes include undertreated pain, agitated delirium, fluid overload, alcohol withdrawal, and inadvertent discontinuation of chronic antihypertensive medications. In uncertain cases, systemic analgesia is often helpful, along with nitrates (e.g., nitroglycerin, 0.4 mg/hour transdermally) and β-blockers (e.g., metoprolol, 5 mg intravenously). The major complication of treatment is the potential for overcorrection and inadvertent hypotension; such errors are particularly common in patients with no evidence of past hypertension.

Hypotension

Hypotension (Chapter 8) after surgery is generally an emergency, and the immediate concern is internal bleeding, especially after intra-abdominal operations or when anticoagulation is used to prevent venous thrombosis. The initial stages of hypotension are frequently unrecognized because of biologic stress responses by patients, psychological denial by clinicians, and misattribution to the concurrent use of analgesia. Early hypotension is particularly easy to overlook if the patient has coexisting chronic hypertension and the apparently “normal” blood pressure is dismissed as unremarkable. Treatment usually entails volume supplementation,5 vasopressors as needed (Chapters 107 and 108), serial assessments, and a search for underlying causes. An extensive differential diagnosis sometimes needs to be considered if no anatomic cause related to surgery is evident (Chapter 106). Routine use of pulmonary artery catheters to guide therapy is not helpful, A5  whereas prompt echocardiography can almost always find the cause of severe hemodynamic instability.

Tachycardia

Tachycardia after surgery can be caused by myriad arrhythmias (Chapters 64 and 65) and may contribute to postoperative cardiac ischemia. Distinguishing between newly detected and newly incident tachycardia can sometimes be accomplished by determining whether the patient does or does not complain of palpitations. An initial assessment also requires review of the electrocardiogram to distinguish atrial fibrillation from other disorders. The goal of treatment is to identify and to correct precipitating factors, such as pain, blood loss, hypoxia, electrolyte abnormalities, fluid overload, volume depletion, pulmonary embolism, and drug withdrawal. Most arrhythmias respond to correction of the underlying abnormality. Specific antiarrhythmic treatment, when needed, is generally similar to that used in the nonoperative setting (Chapters 64 and 65). For atrial fibrillation, anticoagulation is sometimes contraindicated; in such situations, cardioversion within 48 hours merits consideration. Postoperative atrial fibrillation portends a 2-fold higher long-term risk of ischemic stroke.6

Fever

Fever (Chapter 280) after surgery is common, frequently perplexing, and often multifactorial. Worrisome possibilities include transfusion reactions (Chapter 177), hospital-acquired pneumonia (Chapter 97), urinary tract infection (Chapter 284), line sepsis (Chapter 282), and wound infection. In many cases, no definitive cause is found, the patient recovers spontaneously, and the default diagnosis is atelectasis. Detailed evaluation, when necessary, requires culture of blood, urine, and the surgical site to identify specific microbiologic organisms. Selection of empirical antibiotics is usually based on local practice patterns and hospital ecology, with the disadvantages of breeding resistant organisms. Hydration, nutrition, and general supportive care are important yet frequently neglected needs of patients with prolonged elevations in body temperature. Selective decontamination of the digestive tract and oropharynx appears to be beneficial but chlorhexidine is not. A6 

Edema

Peripheral edema (Chapter 51), which is often first noticed by nursing staff after surgery, is rarely life-threatening unless it is treated with excessive diuretics. The cause is usually multifactorial and includes increased hydrostatic pressure (including heart failure and gravity from intraoperative positioning), decreased oncotic pressure (related to hypoalbuminemia from decreased

2624

CHAPTER 433  Postoperative Care and Complications  

liver production or increased losses), and capillary leak (potentially caused by medications or tissue reactions). Treatment focuses on correction of underlying abnormalities, maintenance of nutrition, judicious use of diuretics, monitoring of renal function, provision of systemic anticoagulation against deep venous thrombosis, and efforts toward mobilization of the patient. A low-salt diet, afterload reduction, and aldosterone antagonists (e.g., oral spironolactone, 25 mg once daily) may be helpful in patients in whom heart failure (Chapter 59) is the dominant mechanism.

Laboratory Leukocytosis

An elevated white blood cell count (Chapter 167) can have many causes, but the immediate priority is to exclude a life-threatening septic process (Chapters 106 and 108). Direct microscopic examination of the peripheral blood smear (Chapter 157) can be helpful to check for toxic granulations (see Fig. 157-18), Döhle’s bodies (see Fig. 157-19), and a shift toward primitive band cells. Many cases are due to noninfectious causes, including infarcted tissue (skin, heart, intestinal tract), inflammatory conditions (renal insufficiency, diabetic ketoacidosis, lupus erythematosus), and demargination stress reactions (dehydration, systemic corticosteroids, inotropic medications). In the absence of direct evidence, some clinicians may initiate antibiotics empirically, whereas others may elect waiting. Substantial controversy remains about the proper duration of an empirical trial of antibiotics when no cause is discovered and the patient is otherwise recovering.

Anemia

Anemia (Chapter 158) is common and sometimes underappreciated because coexisting volume depletion causes the blood hemoglobin concentration to underestimate the degree of blood loss. Major perioperative hemorrhage is associated with subsequent stroke and MI in patients undergoing noncardiac, non-neurologic surgery.7 The initial priority is to differentiate bleeding at the surgical site from other causes. In many cases, the exact cause is unclear, and substantial uncertainty may arise over the need to initiate gastric acid suppression therapy or to interrupt systemic anticoagulation against venous thrombosis. In a large randomized trial, transfusion at a hemoglobin threshold of 10 g/dL was no better than transfusion for symptoms of anemia or at the physician’s discretion for a hemoglobin level of below 8 g/dL. A7  Guidelines for transfusion therapy depend on the patient’s cardiac reserve as well as on the available blood bank supply at the particular medical center. A reasonable goal is to maintain a hemoglobin level of 7 mg/dL or higher, except in patients with cardiovascular disease, in whom a hemoglobin goal of 10 mg/dL is reasonable. The immediate postoperative interval is not usually the appropriate time to initiate erythropoietin, oral iron replacement, or detailed evaluations for other hematologic abnormalities.

Abnormalities in Platelet Count

Patients often have abnormal platelet counts after surgery yet rarely require further evaluation or treatment. In most cases, the thrombocytopenia is mild, does not require transfusion therapy, resolves in a few weeks, and is not a sign of an ominous disorder (e.g., sepsis or heparin-induced thrombocytopenia). Platelet transfusions are indicated if the decrease in platelet count is extreme, accompanied by evidence of major blood loss, or related to recent surgery on the central nervous system (including the eye). Thrombocytosis is also common about a month after surgery and is occasionally extreme. However, even postoperative thrombocytosis exceeding 1,000,000/mL rarely necessitates treatment, does not predispose patients to unwanted clotting disorders, and typically resolves spontaneously after a few weeks.

Abnormal Sodium Concentration

Both hyponatremia and hypernatremia (Chapter 116) are frequent complications in the postoperative setting. The immediate priorities are to assess the patient’s intravascular volume status and to correct possible volume depletion. The causes of hyponatremia are multifactorial, including excessive use of diuretics, high levels of intrinsic antidiuretic hormone (as a result of factors such as drugs, pain, and mechanical ventilation), and unmeasured osmoles (e.g., intravenous contrast agents). This risk for postoperative hyponatremia may be reduced by the administration of isotonic saline rather than by water restriction. A8  Once hyponatremia develops, vasopressin antagonists (Chapter 116) are effective for both hypervolemic and euvolemic hyponatremia. A9  Hypernatremia is always due to free water deficiency, which may indicate severe cognitive impairment or other factors interfering with the ability to express thirst or to ingest water. Correction is similar to that in the

nonoperative setting. Abnormalities in sodium concentration require careful follow-up, can recur at any point after an operation, yet are rarely the root cause of a patient’s inability to recover from surgery.

Abnormal Serum Potassium Concentration

Hyperkalemia and hypokalemia (Chapter 117) are also frequent postoperative complications. The immediate priority is to assess and to stabilize the patient’s electrocardiographic findings. Hyperkalemia is usually due to cellular shifts, renal failure, and tissue destruction (including hemolysis). Hyperkalemia will generally be corrected with treatments that shift potassium into cells (e.g., intravenous glucose with or without insulin) and enhance total excretion (e.g., gastrointestinal binding agents). Hypokalemia is usually due to inadequate intake, excessive loss, or cellular shifts. Hypokalemia will generally be corrected with replacement therapy and rarely requires aldosterone antagonism. Both abnormalities can usually be treated as in the nonoperative setting (Chapter 117). The prognosis is favorable if the patient’s electrocardiogram shows no major dysrhythmias and if renal function is preserved.

Alkalosis

Systemic alkalosis (Chapter 118) typically requires volume supplementation because the cause is generally intravascular volume depletion. Blood gas determinations may be necessary in some cases to exclude the possibility of concurrent carbon dioxide retention with compensatory metabolic alkalosis. Untreated, alkalosis can result in altered mentation, cardiac arrhythmias, and delayed mobilization. Most patients with postoperative alkalosis do not require carbonic anhydrase inhibitors or intravenous acid. The prognosis is usually favorable, with gradual correction during a period of several days. Rapid correction of alkalosis, unlike rapid correction of hyponatremia, is not known to cause neurologic injury.

Azotemia

The initial assessment of an elevated serum creatinine concentration (Chapter 114) focuses on reviewing previous values (to distinguish acute from chronic renal insufficiency) and identifying contributing factors (such as prerenal volume depletion, intrarenal nephrotoxins, or postrenal urethral obstruction). A trial of intravenous fluids may be useful on both a diagnostic and therapeutic basis. Treatment is the same as in the nonoperative setting. Subsequent monitoring is always necessary, with serum creatinine measurements obtained on a daily basis. Serial measurements of urinary volume and body weight as well as a urine culture are occasionally helpful in selected cases. The prognosis is dependent on the underlying factors and is less favorable after cardiac surgery.

Hyperbilirubinemia

Elevations in serum bilirubin (Chapter 147) are rare after surgery, even though abnormalities in liver enzyme levels occur frequently with general anesthesia. The most benign explanation is Gilbert’s syndrome, but the immediate priority is to assess for possible hepatic failure (especially in patients who have received halothane). As in the nonoperative setting, treatment involves withdrawing potential hepatotoxins, supporting the patient, and allowing time for liver function to recover (Chapter 153). Treatment of hepatic encephalopathy is particularly important because of the constipation and generalized catabolic state that also follow major surgery. Monitoring should include serial measurement of liver function on a daily basis because each component (e.g., bilirubin, albumen, prothrombin time) can be altered by factors unrelated to the liver. The prognosis is unfavorable if the patient’s liver function fails to recover quickly.

Hypoalbuminemia

Reductions in serum albumin are common after surgery and are an ominous prognostic finding. The cause is rarely decreased production if the reduction in the albumin level occurs rapidly. Possible explanations include nephrotic syndrome, capillary leak into extravascular spaces, and occult catabolism in unrecognized sites. Treatment with albumin infusions does not usually normalize the biochemical abnormality and does not seem to improve patient mortality rates significantly. The main priorities are to continue nutritional support, to preserve skin integrity, to minimize the use of systemic diuretics, to correct any contributing factors, and to consider correlated serum protein deficiencies (such as reduced levels of immunoglobulins or antithrombin III). Hypoalbuminemia can also cause indirect harm by the loss of carrier proteins, which thereby predisposes patients to potential drug toxicity. The

2625

CHAPTER 434  Medical Consultation in Psychiatry  

long-term prognosis is favorable for survivors because the serum albumin level will eventually fully return to normal.

Abnormalities in Blood Glucose Concentration

In patients with diabetes mellitus (Chapter 229), serum blood glucose concentrations often become unstable after surgery because of altered dietary intake, decreased physical activity, and the release of counter-regulatory hormones. The priority is to avoid hypoglycemia, severe hyperglycemia, diabetic ketoacidosis, cerebral damage, and repeated events. Intensive control of the blood glucose level increases the risk for severe hypoglycemia and death, so a target glucose level of about 140 to 200 mg/dL is recommended. A10  Rapid reversal of sepsis or focal infection can lead to a precipitous decrease in insulin requirements; in such cases, vigilance is required because unsuspected hypoglycemia may cause permanent damage or be fatal in a patient who may otherwise seem to be sleeping. Patients need to be forewarned that temporary doses of subcutaneous insulin may be required but do not commit the patient to chronic insulin therapy. Monitoring involves serial measurement of blood glucose concentration until the patient is eating in a reliable manner.

Special Situations Multiplicity

Some postoperative complications are difficult to classify because no single dominant problem is apparent by symptoms, signs, or laboratory test results. Instead, patients may have multiple problems that need to be addressed simultaneously. The immediate goal is to set priorities and to avoid the temptation to try to eliminate every possibility on the first day. The corollary is to continue to check progress during the subsequent days needed for a complete diagnosis and successful therapy. Because so many concerns require attention in the postoperative interval, the risk is that clinicians will lose track of a secondary issue and make an error that seems obvious in retrospect.

Redundancy

Postoperative complications sometimes generate multiple consultations with physicians who have overlapping abilities. An example might be a patient with a postoperative fever that prompts consultations from pulmonology, nephrology, dermatology, general medicine, and infectious disease specialists. In theory, gathering a critical mass of medical experts together should increase the likelihood of accurate diagnosis, timely treatment, and foolproof follow-up. In reality, however, coordination and communication are never perfect. Personal rivalries, diffusion of responsibility, and many other psychological factors may impede interactions among consultants. Opportunities for miscommunication may be further accentuated if the patient has an exotic diagnosis that is a special draw on the consultant’s attention (e.g., pheochromocytoma). Arguing in front of the patient, in view of other professionals, or in the medical record can be demoralizing. The priority is to communicate effectively with the surgical team responsible for the patient and to encourage that team to make final decisions.

Ambiguity

Another vexation occurs when an urgent request is not connected to a clear rationale (Chapter 430). Diplomacy is needed to establish whether the motivation reflects medicolegal concerns rather than biologic changes in the patient. Sometimes the stimulus that drives the consultation can best be addressed by providing reassurance and confirmation. Sometimes the stimulus is an obscure preexisting disorder (e.g., moyamoya disease), and the surgical team has neither the experience nor the time to investigate how this unrelated medical condition can influence recovery from surgery. Sometimes the stimulus is an unspoken political wish to transfer the care of a burdensome patient from one physician to another. A consultant should develop an understanding of how to interact with other clinicians under such ambiguous circumstances.

Setting Priorities

Requests for consultation often arrive outside conventional working hours, are usually tinged with a sense of urgency, and sometimes cluster to encompass more than one patient. Developing an effective method for prioritizing patients is a crucial clinical skill. One communication strategy is to provide an objective estimated time of arrival for the initial request from the surgical team. An often helpful treatment strategy is to make some safe suggestions at the time of the initial request so that clear-cut recommendations can be instituted during the interval before the patient is seen and used later to help evaluate the patient’s status and course.

Aftermath

In many postoperative cases, the original reason for consultation may resolve and no major issues remain. The situation now provides an opportunity to review the patient, particularly for the appropriate use of unrelated medications. The consultant may often detect excessive medications that were appropriate early in the hospital course but have ceased to be necessary, thereby justifying discontinuation (e.g., diuretics, antibiotics, bronchodilators). Discontinuing medications that have become superfluous requires initiative and wisdom, and the common mistake is to propagate unnecessary medications in stable patients under the rationale of “don’t mess with success.” The ability to watch the patient for several hours or for a day or two often presents an ideal opportunity for the safe withdrawal of medications. Ironically, discontinuing a treatment sometimes requires more skill, time, and initiative than starting it.

Grade A References A1. Gould MK, Garcia DA, Wren SM, et al. Prevention of VTE in nonorthopedic surgical patients. Antithrombotic Therapy and Prevention of Thrombosis, 9th ed: American College of Chest Physicians Evidence-Based Clinical Practice Guidelines. Chest. 2012;141:e227S-e277S. A2. Falck-Ytter Y, Francis CW, Johanson NA, et al. Prevention of VTE in orthopedic surgery patients. Antithrombotic Therapy and Prevention of Thrombosis, 9th ed: American College of Chest Physicians Evidence-Based Clinical Practice Guidelines. Chest. 2012;141:e278S-325S. A3. Squadrone V, Coha M, Cerutti E, et al. Continuous positive airway pressure for treatment of postoperative hypoxemia: a randomized controlled trial. JAMA. 2005;293:589-595. A4. Apfel CC, Korttila K, Abdalla M, et al. A factorial trial of six interventions for the prevention of postoperative nausea and vomiting. N Engl J Med. 2004;350:2441-2451. A5. Sandham JD, Jull RD, Brant RF, et al. A randomized, controlled trial of the use of pulmonary-artery catheters in high-risk surgical patients. N Engl J Med. 2003;348:5-14. A6. Price R, MacLennan G, Glen J. SuDDICU Collaboration. Selective digestive or oropharyngeal decontamination and topical oropharyngeal chlorhexidine for prevention of death in general intensive care: systematic review and network meta-analysis. BMJ. 2014;348:g2197. A7. Carson JL, Terrin ML, Noveck H, et al. Liberal or restrictive transfusion in high-risk patients after hip surgery. N Engl J Med. 2011;365:2453-2462. A8. Neville KA, Sandeman DJ, Rubinstein A, et al. Prevention of hyponatremia during maintenance intravenous fluid administration: a prospective randomized study of fluid type versus fluid rate. J Pediatr. 2010;156:313-319. A9. Rozen-Svi B, Yahav D, Gheorghiade M, et al. Vasopressin receptor antagonists for the treatment of hyponatremia: systematic review and meta-analysis. Am J Kidney Dis. 2010;56:325-337. A10. Kansagara D, Fu R, Freeman M, et al. Intensive insulin therapy in hospitalized patients: a systematic review. Ann Intern Med. 2011;154:268-282.

GENERAL REFERENCES For the General References and other additional features, please visit Expert Consult at https://expertconsult.inkling.com.

434  MEDICAL CONSULTATION IN PSYCHIATRY PETER MANU

HEALTH STATUS IN PSYCHIATRIC PATIENTS

The physical health of psychiatric patients is poor, with a higher risk for death at an early age than in the mentally sane. Poverty, social neglect, substandard medical care, unhealthy life habits, and complications of psychiatric treatments are major contributors to the increased morbidity and mortality of patients with chronic psychiatric disorders, who have a 20-year average decline in life expectancy. Integrated care models that coordinate general medical care with psychiatric care can help meet the needs of patients with chronic psychiatric problems.1

MEDICAL EVALUATION IN PSYCHIATRIC SETTINGS

Inpatient psychiatric care is provided in the United States in 444 non-federal, self-standing psychiatric hospitals with a total of 101,500 beds and 1373 general hospitals with inpatient psychiatric units. Requirements for medical training in psychiatry vary. In the United States, psychiatric residents have up

CHAPTER 433  Postoperative Care and Complications  

GENERAL REFERENCES 1. de Vries EN, Prins HA, Crolla RM, et al. Effect of a comprehensive surgical safety system on patient outcomes. N Engl J Med. 2010;363:1928-1937. 2. Urbach DR, Govindarajan A, Saskin R, et al. Introduction of surgical safety checklists in Ontario, Canada. N Engl J Med. 2014;370:1029-1038. 3. Botto F, Alonso-Coello P, Chan MT, et al. Myocardial injury after noncardiac surgery: a large, international, prospective cohort study establishing diagnostic criteria, characteristics, predictors, and 30-day outcomes. Anesthesiology. 2014;120:564-578.

2625.e1

4. Saczynski JS, Marcantonio ER, Quach L, et al. Cognitive trajectories after postoperative delirium. N Engl J Med. 2012;367:30-39. 5. Pearse RM, Ackland GL. Perioperative fluid therapy. BMJ. 2012;344:e2865. 6. Gialdini G, Nearing K, Bhave PD, et al. Perioperative atrial fibrillation and the long-term risk of ischemic stroke. JAMA. 2014;312:616-622. 7. Kamel H, Johnston SC, Kirkham JC, et al. Association between major perioperative hemorrhage and stroke or Q-wave myocardial infarction. Circulation. 2012;126:207-212.

2625.e2

CHAPTER 433  Postoperative Care and Complications  

REVIEW QUESTIONS 1. An 83-year-old woman who fell in a nursing home is admitted because she now is unable to walk. The diagnosis is a hip fracture, and the plan is for surgical repair. She is taking multiple chronic medications that might be sources of drug toxicity if surgery leads to changes in liver function, kidney function, oral intake, or drug-drug interactions. Which of the following medications, however, would not merit careful monitoring and dose adjustment during the perioperative interval? A. l-Thyroxine B. Lithium C. Phenytoin D. Cyclosporine E. Metoprolol Answer: A  Changes in drug absorption and elimination lead to large changes in the pharmacodynamics of most medications. As a consequence, routine measurement of lithium levels, phenytoin levels, and cyclosporine levels is indicated in patients receiving such medications. Doses of β-blockers, such a metoprolol, may also need to be adjusted on the basis of the patient’s heart rate and blood pressure. One exception is that thyroid supplements, such as l-thyroxine, do not typically require monitoring and adjustment. 2. A 72-year-old man has been diagnosed with ocular cataracts and is awaiting carotid endarterectomy surgery. His list of medications needs to be reviewed to distinguish drugs that are essential in the postoperative setting from other drugs that are counterproductive (or superfluous) in the postoperative setting. Which of the following chronic medications should not be routinely discontinued in this patient? A. Alendronate B. Calcium carbonate C. Iron sulfate D. Ginkgo biloba E. Aspirin Answer: E  The patient is receiving multiple medications that are often available without a prescription in some countries. Each is relatively safe among otherwise healthy outpatients, can cause problems in patients who are recovering from surgery, and is a long-acting metabolic agent that could be discontinued with no immediate withdrawal syndrome. Calcium carbonate is typically formulated in large pills that might be aspirated in the immediate postoperative setting. Iron sulfate might worsen postoperative gastritis even though it was tolerated on an outpatient basis. Ginkgo biloba can increase the risk of postoperative hepatitis when it is taken in the perioperative setting. Alendronate can lead to hemorrhagic esophagitis arising from impaired swallowing after surgery. Aspirin is the major exception with this particular operation, in which randomized trials suggest a significantly reduced risk of a postoperative stroke after carotid surgery that outweighs the risk of a secondary gastrointestinal tract hemorrhage.

3. A 94-year-old woman presents with postmenopausal painless vaginal bleeding. Her baseline functional status is excellent, and she walks half an hour each day for exercise. The diagnosis is endometrial cancer, and she undergoes a total abdominal hysterectomy. On the first day after surgery, she develops delirium characterized as a hypoactive state with no oral intake, simple one-word verbal responses to questioning, and an inability to follow simple commands. Which of the following would not be a contributing cause to her delirium? A. Carbon dioxide narcosis B. Adverse drug reaction C. Hypothyroidism D. Alcohol withdrawal syndrome E. Surgical site infection Answer: C  The patient has developed an acute postoperative delirium with no clear localizing findings. An adverse drug reaction is a leading consideration because of drug-drug interactions, altered pharmacokinetics, or other mechanisms. Carbon dioxide narcosis is a possibility, particularly if the patient has undiagnosed sleep apnea, respiratory depression with analgesia, and pulmonary atelectasis after abdominal surgery. Alcohol withdrawal with hypoactive delirium tremens is also a potential, as is an occult surgical site infection. Hypothyroidism, however, rarely is a major contributor to acute delirium. Therefore, hypothyroidism is one factor that is unlikely to explain postoperative delirium.

2626

CHAPTER 434  Medical Consultation in Psychiatry  

TABLE 434-1  COMMON REASONS FOR TRANSFERRING PSYCHIATRIC INPATIENTS WITH ACUTE MEDICAL DETERIORATION TO A GENERAL HOSPITAL

TABLE 434-2  MAJOR MEDICAL COMPLICATIONS OF PSYCHOTROPIC DRUGS

Fever

17%

Neurologic deficits, seizures, alteration of consciousness

14%

Fall, head trauma

13%

Abdominal pain, gastrointestinal bleeding

10%

Dyspnea, hypoxia

10%

Chest pain

8%

Urinary retention, azotemia, electrolyte imbalance

7%

Arrhythmia, hypotension, syncope

6%

Edema, cellulitis

5%

All others

10%

From Manu P, Asif M, Khan S, et al. Risk factors for medical deterioration of psychiatric inpatients: opportunities for early recognition and prevention. Compr Psychiatry. 2012;53:968-974.

CARDIOVASCULAR Cardiomyopathy Hypertension Myocarditis Orthostatic hypotension QTc prolongation Venous thromboembolism RESPIRATORY Choking Laryngospasm Respiratory depression GASTROINTESTINAL Bowel obstruction Dysphagia Hepatic impairment Pancreatitis

to 4 months of inpatient medical training. With such limited medical training, psychiatrists cannot be expected to be experts in assessing nonpsychiatric problems. Free-standing psychiatric facilities employ consultants to evaluate medical issues and to manage outpatient-level conditions, but they are not equipped to provide inpatient-level medical or surgical services. Patients who require a higher level of care are transferred to a medical, surgical, or medical/ psychiatric unit of a general hospital. As many as 15% of patients who are admitted for psychiatric care may be transferred to a general hospital for medical conditions that arise or deteriorate during inpatient psychiatric care.2 Febrile illnesses, acute neurologic changes, and falls account for nearly half of the transfers (Table 434-1). Nearly half of the patients admitted for dementia with behavioral disturbance will develop a medical complication during an inpatient psychiatric stay, a rate that is two- to three-fold higher than for other psychiatric patients. Renal insufficiency, anemia, poor nutritional status, and older age are also independent predictors of medical deterioration. Medical deterioration can have major adverse consequences for psychiatric inpatients. First, it may lead to life-threatening complications if the condition is not rapidly diagnosed and treated. Second, it interrupts behavioral interventions and may require the discontinuation of psychotropic drug treatment or electroconvulsive therapy (ECT). Third, it prolongs the length of stay and can add considerable expense to the episode of psychiatric illness, especially when patients are transferred from locked psychiatric units to a general hospital, where they require constant observation by qualified personnel.

EVALUATION OF CHIEF COMPLAINTS

Medical consultation for psychiatric patients creates unique challenges in evaluation of the chief complaint. Many patients with outpatient psychiatric disorders have somatic symptoms, such as fatigue, weakness, dizziness, headache, insomnia, widespread pain, and constipation. In most of these patients, the underlying mental illnesses are mood disorders (unipolar depression and dysthymia), anxiety disorders (panic disorder and generalized anxiety disorder), somatoform disorders, substance use disorders (most often alcohol, opiates, cocaine, and benzodiazepine), and borderline personality disorder. As a group, these patients have many physical complaints and resist a psychological explanation for their symptoms even when the medical evaluation fails to identify objective abnormalities. Standardized screening approaches can help clinicians and patients alike.3 In contrast, patients admitted for inpatient psychiatric treatment often have psychotic disorders, developmental abnormalities, or dementia with behavioral disturbance; they are frequently vague or silent about physical suffering and only rarely voice somatic delusions. Such patients may deny pain even after bowel perforation or myocardial infarction. Medical consultation in psychiatry is not more difficult than in other clinical settings but must be informed by knowledge about the serious complications of psychiatric treatments (Table 434-2). A rigorous diagnostic evaluation is therefore required to avoid the errors of omission created by the weak correlation between complaints and disease. This evaluation should include consideration of physical disorders, drug effects (adverse reaction, toxicity, or withdrawal), cognitive impairment (delirium), sensory impairment (loss of

Clozapine MAO inhibitors, venlafaxine Clozapine Tricyclics, trazodone, antipsychotics Antipsychotics, tricyclics Clozapine, risperidone, phenothiazines Antipsychotics, tricyclics Antipsychotics Benzodiazepines, barbiturates, methadone, antidepressants, atypical antipsychotics Tricyclics, antipsychotics Tricyclics, antipsychotics Carbamazepine, valproic acid, phenothiazines, mirtazapine, nefazodone, quetiapine, olanzapine, clozapine, MAO inhibitors, naltrexone Carbamazepine, valproic acid, clozapine, olanzapine

KIDNEY AND URINARY TRACT Renal insufficiency Lithium, clozapine Urinary retention Antipsychotics, tricyclics ENDOCRINE Hyperprolactinemia Hypothyroidism Inappropriate ADH secretion Metabolic syndrome HEMATOLOGIC Leukocytosis Neutropenia Thrombocytopenia MUSCULOSKELETAL Rhabdomyolysis* SKIN Stevens-Johnson syndrome OTHER Fever* Seizure

First-generation antipsychotics, risperidone Lithium, quetiapine Serotonin reuptake inhibitors, methadone, tricyclics Clozapine, olanzapine, risperidone, quetiapine Lithium Clozapine, olanzapine, risperidone, carbamazepine, valproate, mirtazapine Carbamazepine, valproate Antipsychotics, serotonin reuptake inhibitors, MAO inhibitors Lamotrigine, carbamazepine, barbiturates Antipsychotics, serotonin reuptake inhibitors, MAO inhibitors Bupropion, MAO inhibitors, tricyclics, phenothiazines

*Includes neuroleptic malignant syndrome and serotonin syndrome. ADH = antidiuretic hormone; MAO = monoamine oxidase.

vision, hearing, speech, or postural balance), and situational maladjustment (isolation, overload, or loss of privacy). Attribution of symptoms to the patient’s psychiatric condition should remain a diagnosis of exclusion.

MEDICAL COMPLICATIONS OF PSYCHIATRIC TREATMENTS

Antipsychotic-Induced Metabolic Syndrome

Metabolic syndrome is more prevalent (range, 29 to 63%) in schizophrenic and other psychiatric patients treated with second-generation antipsychotics, especially clozapine but also olanzapine, risperidone, and quetiapine because they induce substantial weight gain. The mechanism of weight gain centers on the drug’s affinity for the histamine1 (H1)–receptor and the neurobiologic mechanisms that regulate appetite and metabolism through the production and activity of serotonin, leptin, and tumor necrosis factor-α. As a result, up to 40% of patients receiving long-term treatment with antipsychotics have prediabetes, and about 10% have diabetes.4 Because the glucose intolerance often seen in patients treated with antipsychotic agents is due to insulin resistance (Chapter 229), it is best treated by aggressive weight reduction (Chapter 213), increasing physical activity (Chapter 16), and metformin (Chapter 229) or a combination of these. For example, in randomized trials, metformin, 850 mg twice daily, significantly reduced the weight gain and reversed the metabolic abnormalities associated with the initiation of second-generation antipsychotic drugs, A1  and a single daily dose of 750 mg may prevent weight gain if it is initiated at the onset of

CHAPTER 434  Medical Consultation in Psychiatry  

olanzapine treatment. A2  Another option is to switch from olanzapine, quetiapine, or risperidone to aripiprazole. A3  In treating hypertension, β-adrenergic blockers should be used cautiously in patients receiving neuroleptics because they increase the potential for orthostatic hypotension, syncope, and falls. Smoking cessation programs combining transdermal or transmucosal nicotine replacement, bupropion, and cognitive-behavioral therapy should be strongly recommended despite the dismally high failure rates that have been reported in patients with chronic schizophrenia (Chapter 32).

2627

shortest QTc on a 12-lead electrocardiogram) longer than 100 milliseconds, and increase in QTc duration of more than 60 milliseconds in comparison to the baseline measurement should prompt re-evaluation of the risks and benefits associated with the drugs in question.

Choking and Laryngeal Dystonia

Antipsychotic-induced myocarditis and cardiomyopathy (Chapter 60) are most common in patients treated with clozapine (0.9%) and fluphenazine (0.4%). In contrast, the risk for these complications is only 0.1% in patients receiving haloperidol, thioridazine, and risperidone. The accepted pathophysiologic explanation for myocarditis is an immunoglobulin E–mediated acute hypersensitivity reaction, similar to the allergic myocarditis produced by penicillins, sulfonamides, and methyldopa. In a small number of patients, a competing hypothesis proposes that clozapine induces hypereosinophilic myocarditis, colitis, hepatitis, pancreatitis, alveolitis, and interstitial nephritis. A direct cardiotoxic effect of drug metabolites cannot be excluded. In patients in whom myocarditis develops, the mortality rate is as high as 50%, with almost half the deaths occurring suddenly and unexpectedly. The average duration of exposure to clozapine before diagnosis or death is 21 days, and the dosage range is 50 to 725 mg/day. Common symptoms are fever (48%), dyspnea (35%), influenza-like illness (30%), chest pain (22%), and fatigue (17%). Laboratory features include left ventricular hypokinesia or reduced ejection fraction (48%) or pericardial effusion (17%) on echocardiography, nonspecific repolarization abnormalities on electrocardiography (35%), peripheral eosinophilia (35%), elevated creatine kinase and troponin levels (22%), and radiographic evidence of heart failure (13%). The diagnosis can be confirmed by endomyocardial biopsy showing fraying of myocytes and perivascular infiltrates with degranulated eosinophils. Among survivors, symptoms resolve or substantially improve after discontinuation of clozapine and treatment with high-dose corticosteroids (e.g., prednisone, 1 mg/kg/day for 4 days, tapered to 0.33 mg/kg/day for the following 4 days). Clozapine-induced dilated cardiomyopathy may be caused by an evolving myocarditis or by chronic injury mediated by free radicals, similar to the myocarditis produced by doxorubicin (Chapter 60). The demographic features are similar to those of myocarditis, but the mean duration of treatment before diagnosis is much longer (9 months vs. 3 weeks), and the mortality rate is lower (22% vs. 51%). Patients have clinical or echocardiographic evidence of left ventricular dysfunction without eosinophilia or enzymatic evidence of myocardial necrosis.

Asphyxia deaths from choking occur at a rate of 0.8% per 1000 psychiatric patients each year, a frequency that is more than 100 times greater than in the general population. In addition, videofluoroscopy demonstrates silent aspiration in 38% of psychiatric patients who survive a choking incident. Half the psychiatric patients with dysphagia have a fast-eating syndrome seen in association with restlessness, poor chewing skills, food pocketing in the cheeks, and attention deficits that characterize psychotic disorders and mental retardation. Bradykinetic dysphagia, which is seen in 25% of psychiatric patients with choking episodes, is due to the antidopaminergic and anticholinergic effects of psychotropic medications. This condition, which features reduced lingual range of motion, increased oral transit time, decreased pharyngeal peristalsis, and delayed initiation of the swallowing reflex, is seen in patients with neurologic features of drug-induced Parkinsonism (Chapter 409). Dyskinetic dysphagia (7% of choking cases), which generally occurs in patients maintained with long-term antipsychotic medication, is part of the clinical spectrum of tardive dyskinesia (Chapter 410). The examination reveals involuntary contractions of the tongue and perioral musculature, clumsiness of voluntary movements of the tongue, and discontinuous bolus propulsion in the oral stage. In the remaining patients, the dysphagia is due to cerebrovascular disease (11%) or to pharyngeal or esophageal disease (7%). Laryngeal dystonia, which is a life-threatening complication of antipsychotic drug therapy, primarily with haloperidol and phenothiazines, is produced by acute spasmodic contraction of the adductor laryngeal muscles. Symptoms include respiratory distress, dysphonia, and stridor. Neurolepticinduced bronchospasm may precede the onset of stridor. Patients typically indicate extreme subjective distress by clutching their anterior cervical area. Most patients also have other dystonias involving the head and neck, including torticollis, retrocollis, trismus, tongue protrusion, and deviation of the eyes (up, down, or sideward). In general, the symptoms and signs develop in the first week after starting or rapidly increasing the dose of neuroleptic medications. A reduction in the dose of anticholinergic or antiparkinsonian medication used to prevent or to treat extrapyramidal symptoms can also precipitate laryngeal dystonia. The condition is more common in young men and must be distinguished from epiglottitis, allergic or anaphylactic laryngeal edema or laryngospasm, mechanical obstruction, and psychogenic stridor. Intravenous administration of diphenhydramine (initial dose, 25 mg; may repeat after 5 minutes if symptoms persist) is the treatment of choice, and endotracheal intubation is seldom required.

Prolonged QTc Interval and Sudden Death

Drug-Induced Neutropenia and Agranulocytosis

Antipsychotic-Induced Myocarditis and Cardiomyopathy

Significant prolongation of the QTc interval (Chapter 65) leading to ventricular tachyarrhythmias and sudden cardiac death (Chapter 63) can occur after antipsychotic treatment with the usual doses of thioridazine, haloperidol, and sertindole. Abnormal myocardial repolarization has been observed during treatment with most antipsychotic medications and after intentional or accidental overdoses of tricyclic antidepressants, lithium, and methadone (Chapter 110). All antipsychotics affect the cardiac potassium channel by blocking the rapidly activating component of the rectifier potassium current (Chapter 61). This effect translates into a dose-dependent increase in the duration of phase 3 of the action potential. The drug concentration that produces 50% inhibition of rapid potassium outflow varies for each drug (e.g., 1 nmol/L for haloperidol and 6 nmol/L for olanzapine). Compared with nonusers of antipsychotic drugs, the risk for sudden death is twice as high for current users of conventional (first-generation) antipsychotics and is 2.25 times higher for current users of atypical (second-generation) antipsychotics. All patients about to start antipsychotic drugs should be asked about a personal history of syncope and a family history of long QT syndrome or sudden death at a young age. A baseline electrocardiogram and serum electrolyte values should be obtained before starting of antipsychotic drug therapy, tricyclic antidepressants, and methadone. Interval electrocardiograms should be obtained after each increase in medication in older patients, patients with known heart disease, and those starting other drugs known to produce QTc prolongation or hypokalemia (Chapter 65). A QTc interval of 500 milliseconds or longer requires the discontinuation of all drugs that affect membrane repolarization. QTc intervals longer than 450 milliseconds in men and 470 milliseconds in women, QTc dispersion (difference between the longest and

Drug-induced neutropenia with absolute neutrophil counts of less than 1500/µL has been observed during treatment with most second-generation antipsychotics (clozapine, olanzapine, risperidone, and quetiapine) and mood stabilizers (carbamazepine, valproic acid, and lamotrigine) as well as with some antidepressant drugs (tricyclic antidepressants and mirtazapine). Clozapine-induced neutropenia occurs in 4 to 5% of patients within 6 months after treatment is started and progresses to agranulocytosis in 10% or more of neutropenic patients if the drug is continued. In vitro, clozapine toxicity requires peroxide and peroxidase, and the defect in oxidation is related to abnormalities in the NQO2 (quinone oxidoreductase) gene involved in drug detoxification. Treatment with clozapine should be started only if the baseline absolute neutrophil count is higher than 1500/µL. The concomitant use of carbamazepine, angiotensin-converting enzyme inhibitors, sulfonamides, propylthiouracil, and mirtazapine should be avoided because they can produce neutropenia and increase the risk for agranulocytosis. Clozapine should be stopped and the patient evaluated immediately for fever, oral ulcerations, and symptoms or signs of infection. Complete blood counts should be obtained once a week for the first 26 weeks and every other week thereafter, and clozapine should be stopped and all medications reassessed if the absolute neutrophil count drops below 1500/µL. Clozapine-related agranulocytosis has been treated successfully with colony-stimulating factors (either granulocyte or granulocyte-macrophage colony-stimulating factor). Neutropenia has also been associated with olanzapine, risperidone, and quetiapine in patients who have never received clozapine. Treatment with anticonvulsant mood stabilizers, particularly carbamazepine, is associated with a dose-dependent neutropenia and thrombocytopenia in approximately

2628

CHAPTER 434  Medical Consultation in Psychiatry  

TABLE 434-3  DIFFERENTIAL DIAGNOSIS OF NEUROLEPTIC MALIGNANT SYNDROME Infection of the central nervous system Infection in patients with drug-induced Parkinsonism Drug overdose (psychostimulants, antidepressants, lithium, anticholinergics)

TABLE 434-4  CLASSES OF MEDICATIONS THAT PRODUCE SEROTONIN SYNDROME IN PSYCHIATRIC PATIENTS Selective serotonin reuptake inhibitors Monoamine oxidase inhibitors

Alcohol or drug withdrawal (benzodiazepines, barbiturates, antiparkinsonian drugs)

Atypical antipsychotics

Side effects of nonpsychotropic dopamine-depleting drugs (reserpine, metoclopramide, prochlorperazine, promethazine)

Heterocyclic antidepressants Trazodone

Cholinergic rebound

Dual-uptake inhibitors

Serotonin syndrome

Psychostimulants

Thyrotoxicosis

Buspirone

Malignant hyperthermia

Mood stabilizers Analgesics

10% of patients in the first 6 months of treatment and should be monitored with complete blood counts twice each month during this period.

Neuroleptic Malignant Syndrome

Neuroleptic malignant syndrome, which occurs in approximately 0.2% of patients receiving neuroleptics, must be part of the differential diagnosis of fever and rhabdomyolysis (Chapter 113) in a psychiatric patient (Table 4343). The frequency is greater in young men and patients who are malnourished or dehydrated, have Parkinson’s disease, or are treated parenterally with large doses of neuroleptics during short periods. The main diagnostic criteria are elevated temperature (higher than 104° F [40° C] in 40% of patients) and diffuse muscle rigidity (ranging from mild hypertonicity to severe “lead pipe” stiffness). In addition, two or more of the following are required for a definitive diagnosis: (1) autonomic instability (tachycardia, elevated or labile blood pressure, postural hypotension, diaphoresis, sialorrhea, and urinary incontinence), (2) changes in mental status (ranging from confusion to mutism or coma), (3) leukocytosis (up to 20,000/mL), and (4) elevated creatine kinase (up to 100,000 IU/L). Other clinical manifestations include bradykinesia, chorea, dystonias, dysphagia, dysarthria or aphonia, seizures, and tremor. The severity of rhabdomyolysis correlates with the creatine kinase level and with the presence of myoglobinemia, myoglobinuria, metabolic acidosis, and azotemia. The electroencephalogram shows nonspecific slowing in slightly more than half of patients. The time lag from starting of the drug to the onset of neuroleptic malignant syndrome is generally short, with 30% of cases developing within 48 hours and 96% within the first month of treatment. The exception appears to be clozapine-associated neuroleptic malignant syndrome, which has an average time lag of 50 days. Neuroleptic syndrome is sometimes confused with severe catatonia (Chapter 397), but the catatonic signs in neuroleptic malignant syndrome are usually restricted to mutism and akinesia. Furthermore, hyperthermia, rigidity, tremor, and rhabdomyolysis are not present in patients with catatonia. Nonetheless, close medical follow-up of severely catatonic patients is warranted because they are at very high risk (22%) for neuroleptic malignant syndrome. Untreated, neuroleptic malignant syndrome has a mortality rate of 10% as a result of acute renal failure, aspiration pneumonia, adult respiratory distress syndrome, disseminated intravascular coagulation, and cerebellar neuronal degeneration. Most fatalities are avoidable if the diagnosis is made early, the neuroleptic agent is discontinued rapidly, and the patient is immediately transferred to an intensive care setting for supportive and specific therapy. Bromocriptine (2.5 mg three times daily orally or through a nasogastric tube; may increase by 2.5 mg three times daily to a maximal daily dose of 40 mg) or amantadine (100 mg orally or through a nasogastric tube twice daily; may increase to 300 mg/day in divided doses) should be used in moderately severe cases and continued until the muscle rigidity and metabolic abnormalities have significantly improved. The skeletal muscle relaxant dantrolene (starting with a dose of 1 mg/kg intravenously and titrated to a maximal dose of 10 mg/ kg/day divided into three intravenous or oral doses) should be added to bromocriptine or amantadine in patients with fulminant hypermetabolic features and those with persistent muscle rigidity despite treatment with dopamine agonists. Refractory neuroleptic malignant syndrome improves after ECT.

Serotonin Syndrome

Serotonin syndrome (Chapter 432) is an adverse drug reaction primarily produced by excess serotonergic agonism of central nervous system and

Antiemetics Cough suppressants Dietary supplements

peripheral serotonin receptors by selected drugs (Table 434-4).5 In postmarketing surveillance studies of the newer antidepressants, the syndrome has an incidence of 4 cases per 10,000 patient-months in patients who start taking nefazodone, a drug that inhibits neuronal uptake of serotonin and norepinephrine and also acts as a 5-hydroxytryptamine type 2 (5-HT2) receptor antagonist. The syndrome also occurs in 15% of patients with intentional overdose of selective serotonin reuptake inhibitors (SSRIs). The serotonin syndrome is caused by overstimulation of 5-HT1A and possibly also 5-HT2 receptors through excess of serotonin precursors or agonists, increased serotonin release, reduced serotonin uptake, and decreased serotonin metabolism. Severe cases of the syndrome have been more frequently reported in patients treated with monoamine oxidase inhibitors who took over-the-counter dextromethorphan or the illegal methylenedioxymethamphetamine (Ecstasy) or who started treatment with serotonin reuptake inhibitors, meperidine, or atypical antipsychotics such as aripiprazole. Potentially life-threatening, the syndrome is characterized by changes in mental status (ranging from agitation to confusion and coma), autonomic instability (tachycardia, labile or high blood pressure, diaphoresis, and diarrhea), neuromuscular abnormalities (myoclonus, mydriasis, ocular clonus, rigidity, hyperreflexia, tremors, and shivering), and hyperthermia. The symptoms occur within the first 24 hours and sometimes within minutes after the initial use of medication, a change in dose, addition of a new drug, or overdose attempt. Death may occur as a consequence of rhabdomyolysis with renal failure, hyperkalemia, disseminated intravascular coagulation, and acute respiratory distress syndrome. The differential diagnosis includes neuroleptic malignant syndrome, viral or bacterial meningitis or encephalitis, heat stroke (Chapter 109), anticholinergic toxidrome (Chapter 110), and drug (Chapter 34) or alcohol (Chapter 33) withdrawal. General management includes immediate discontinuation of serotonergic drugs, comprehensive supportive therapy, and benzodiazepines for control of agitation and myoclonus. Specific therapy relies on the use of cyproheptadine (an H1-receptor antagonist with antiserotonergic and anticholinergic properties) and chlorpromazine (a 5-HT1A and 5-HT2 receptor antagonist). Cyproheptadine should be started at a dose of 12 mg administered orally or through a nasogastric tube, with additional 2-mg doses given every 2 hours until symptoms improve or the maximal dose of 32 mg has been reached. The usual maintenance dose of cyproheptadine is 8 mg three times daily. Chlorpromazine (50 mg intramuscularly; may repeat three or four times daily and increase gradually to 400 mg/day in divided doses) is indicated in patients with severe symptoms who must be treated parenterally. Rapid improvement has also been observed after single doses of olanzapine (10 mg administered sublingually). Chlorpromazine and olanzapine should be used only after the possibility of neuroleptic malignant syndrome has been excluded.

Antipsychotic-Induced Hyperprolactinemia

Drug-induced hyperprolactinemia is produced by first-generation antipsychotic medications and by risperidone, but it is rare with other atypical antipsychotics such as aripiprazole, olanzapine, and ziprasidone. In patients treated with prolactin-raising antipsychotic medications, hormone levels are above the normal limit in 60% of women and 40% of men. Symptomatic

2629

CHAPTER 434  Medical Consultation in Psychiatry  

hyperprolactinemia (Chapter 224) occurs in about one third of these patients and is generally associated with a 10-fold increase above baseline levels. Excess prolactin leads to dysfunction of target tissues (galactorrhea, oligomenorrhea and amenorrhea, infertility, sexual impairment, and gynecomastia) as well as an increased risk for breast cancer, osteoporosis, and cardiovascular disease. The mechanism of antipsychotic-related hyperprolactinemia is suppression of dopamine inhibition of lactotroph cells in the hypothalamus. Brain imaging is required in symptomatic patients and those with significant elevated prolactin levels to exclude tumors of the pituitary and hypothalamus.

Psychogenic Polydipsia and Drug-Induced Hyponatremia

Hyponatremia, which is present in approximately 6% of patients at the time of admission to inpatient psychiatric care and is more common in older patients and those with hypertension, approximately doubles the risk of medical deterioration.6 The diagnosis is often delayed because traditional manifestations of hyponatremia, such as lethargy, restlessness, weakness, and disorientation, overlap with features of psychiatric disorders. Patients with psychogenic polydipsia typically have serum hypo-osmolality and a maximally dilute urine (urine osmolality 2.0

Stabilized

0-1 factors: Low risk

Optimize condition or transfer to medical facility

2 factors: Moderate risk

Appropriate consultation Optimize BP control

3 or more factors: High risk

FIGURE 434-1.  Risk assessment before electroconvulsive therapy. BP = blood pressure; CAD = coronary artery disease; CHF = congestive heart failure; COPD = chronic obstructive pulmonary disease; DM = diabetes mellitus; HTN = hypertension. (Modified from Frederickson A, Manu P. Risk assessment prior to electroconvulsive therapy. In Manu P, Suarez RE, Barnett BJ, eds. Handbook of Medicine in Psychiatry. Washington, DC: American Psychiatric Publishing; 2006:687-700.)

setting that allows immediate access to an intensive care unit rather than in the ECT suite of a self-standing psychiatric hospital. Essential medications should be administered with a small amount of fluid 6 hours before ECT. Drugs that can increase or decrease the seizure threshold, such as lidocaine, theophylline, phenothiazine, tricyclic antidepressants, and benzodiazepines, must be discontinued before ECT. Optimal pre-ECT risk assessment, careful anesthesia, and post-ECT monitoring result in a serious complication rate of only 0.9% and essentially no fatalities.

Grade A References A1. Chen CH, Huang MC, Kao CF, et al. Effects of adjunctive metformin on metabolic traits in nondiabetic clozapine-treated patients with schizophrenia and the effect of metformin discontinuation on body weight: a 24-week, randomized, double-blind, placebo-controlled study. J Clin Psychiatry. 2013;74:e424-e430. A2. Wu RR, Zhao JP, Guo XF, et al. Metformin addition attenuates olanzapine-induced weight gain in drug-naïve first-episode schizophrenia patients: a double-blind, placebo-controlled study. Am J Psychiatry. 2008;165:352-358. A3. Stroup TS, Byerly MJ, Nasrallah HA, et al. Effects of switching from olanzapine, quetiapine, and risperidone to aripiprazole on 10-year coronary heart disease risk and metabolic syndrome status: results from a randomized controlled trial. Schizophr Res. 2013;146:190-195. A4. Josiassen RC, Goldman M, Jessani M, et al. Double-blind, placebo-controlled, multicenter trial of a vasopressin V2-antagonist in patients with schizophrenia and hyponatremia. Biol Psychiatry. 2008;54:1097-1100.

GENERAL REFERENCES For the General References and other additional features, please visit Expert Consult at https://expertconsult.inkling.com.

CHAPTER 434  Medical Consultation in Psychiatry  

GENERAL REFERENCES 1. Bradford DW, Cunningham NT, Slubicki MN, et al. An evidence synthesis of care models to improve general medical outcomes for individuals with serious mental illness: a systematic review. J Clin Psychiatry. 2013;74:e754-e764. 2. Manu P, Asif M, Khan S, et al. Risk factors for medical deterioration of psychiatric inpatients: opportunities for early recognition and prevention. Compr Psychiatry. 2012;53:968-974. 3. Rayner L, Matcham F, Hutton J, et al. Embedding integrated mental health assessment and management in general hospital settings: feasibility, acceptability and the prevalence of common mental disorder. Gen Hosp Psychiatry. 2014;36:318-324.

2629.e1

4. Manu P, Correll CU, van Winkel R, et al. Prediabetes in patients treated with antipsychotic drugs. J Clin Psychiatry. 2012;73:460-466. 5. Alusik S, Kalatova D, Paluch Z. Serotonin syndrome. Neuro Endocrinol Lett. 2014;35:265-273. 6. Manu P, Ray K, Rein JL, et al. Medical outcome of psychiatric inpatients with admission hyponatremia. Psychiatry Res. 2012;198:24-27. 7. Atsariyasing W, Goldman MB. A systematic review of the ability of urine concentration to distinguish antipsychotic- from psychosis-induced hyponatremia. Psychiatry Res. 2014;217:129-133.

2629.e2

CHAPTER 434  Medical Consultation in Psychiatry  

REVIEW QUESTIONS 1. Medical deterioration during psychiatric admissions may lead to lifethreatening complications, interrupt biobehavioral interventions, and require transfer to medical facilities. Older age and a diagnosis of dementia with behavioral disturbance are nonmodifiable correlates of an increased rate of acute changes in medical conditions in psychiatric patients. Which of the following is an independent risk factor for medical deterioration in psychiatric settings? A. Blood urea nitrogen concentration (higher) B. Congestive heart failure C. Serum potassium concentration (lower) D. Serum sodium concentration (lower) E. Substance use disorder Answer: A  Among 1000 adults consecutively admitted to a free-standing psychiatric hospital, 14% were transferred to a general hospital for acute medical deterioration. The independent predictors of medical deteriorations were higher blood urea nitrogen concentration (odds ratio, 63.2), lower hemoglobin level (odds ratio, 35.3), and lower albumin level (odds ratio, 7.3). (Manu P, Asif M, Khan S, et al. Risk factors for medical deterioration of psychiatric inpatients: opportunities for early recognition and prevention. Compr Psychiatry. 2012;53:968-974.)

2. The 27-year-old with clozapine-treated schizophrenia admitted for suicidal ideation is prescribed venlafaxine 37.5 mg/day, which is rapidly increased to 150 mg/day without adverse drug reactions. An episode of severe agitation is treated with aripiprazole 9.75 mg administered intramuscularly. The following morning, the patient has good behavioral control and is allowed to play basketball on the outdoor court for 45 minutes. He then becomes febrile (39.1° C) and has two episodes of nonbloody diarrhea. The physical examination is significant for mild and diffuse muscle rigidity, diaphoresis, and ankle clonus. The white blood cell count is 10,200 with an absolute neutrophil count of 4800; the creatine kinase level is 380. The electrolyte values and urinalysis are normal. What is the most likely cause of the febrile syndrome? A. Neuroleptic malignant syndrome B. Eosinophilic colitis C. Viral gastroenteritis and neuroleptic-induced Parkinsonism D. Serotonin syndrome E. Heat stroke Answer: D  Serotonin syndrome may develop rapidly after the addition of an antipsychotic (aripiprazole) with serotonergic enhancing properties in patients treated with a dual (serotonin and norepinephrine) reuptake anti­ depressant drug (venlafaxine). Some of its clinical features, (e.g., diarrhea and clonus) are not seen in neuroleptic malignant syndrome. (Perry PJ, Wilborn CA. Serotonin syndrome vs neuroleptic malignant syndrome: a contrast of causes, diagnoses, and management. Ann Clin Psychiatry. 2012;24: 155-162.)

This page intentionally left blank

XXVIII SKIN DISEASES 435 STRUCTURE AND FUNCTION OF THE SKIN

436 EXAMINATION OF THE SKIN AND

AN APPROACH TO DIAGNOSING SKIN DISEASES

437 PRINCIPLES OF THERAPY OF SKIN DISEASES

438 ECZEMAS, PHOTODERMATOSES,

PAPULOSQUAMOUS (INCLUDING FUNGAL) DISEASES, AND FIGURATE ERYTHEMAS

439 MACULAR, PAPULAR,

VESICULOBULLOUS, AND PUSTULAR DISEASES

440 URTICARIA, DRUG HYPERSENSITIVITY RASHES, NODULES AND TUMORS, AND ATROPHIC DISEASES

441 INFECTIONS, HYPERPIGMENTATION

AND HYPOPIGMENTATION, REGIONAL DERMATOLOGY, AND DISTINCTIVE LESIONS IN BLACK SKIN

442 DISEASES OF HAIR AND NAILS

2632

CHAPTER 435  Structure and Function of the Skin  

435  STRUCTURE AND FUNCTION OF THE SKIN DAVID H. CHU

The skin, as the largest organ of the body, is a complex multifunctional entity with many regional specializations that provide its host both protection from, and interaction with, its environment. The skin is not just an impenetrable shield against external injury but rather a dynamic, intricate, integrated arrangement of cells, tissues, and matrix elements that together perform a variety of functions, as follows: • Physical/mechanical protection: The epidermis, which is the selectively permeable layer outermost covering of the skin, helps regulate water loss and provides a physical barrier against external insults. The underlying dermis provides mechanical strength to the skin. • Thermoregulation: Eccrine glands mediate the excretion of water, evaporation of which is critical for thermoregulation. The cutaneous vasculature also helps regulate heat exchange via vasoconstriction and vasodilatation. • Immunologic surveillance: Resident and transient cells of the immune system provide some of the earliest defenses against infectious diseases, toxins, and malignantly transformed cells. • Sensation: Nerve endings and specialized sensory apparatus embedded within the skin provide sensory inputs such as pain and itch. • External appearance: The contours, texture, pigment, and regional variations of the skin are keys to an individual’s external appearance and perception by others. These various functions of the skin are mediated by one or more of its major regions—the epidermis, dermis, and subcutaneous fat (Fig. 435-1). These anatomic subdivisions are interdependent, functional units, each relying on and connected with its surrounding tissue for regulation and modulation of normal structure and function at molecular, cellular, and tissue levels of organization. Whereas the epidermis and its outer stratum corneum constitute a large part of the physical barrier provided by the skin, the structural integrity of skin as a whole is primarily attributable to the dermis and subcutaneous fat. Antimicrobial activities are provided by the innate immune system and antigen-presenting Langerhans cells of the epidermis, by circulating immune cells that migrate from the blood vessels in the dermis, and by antigenpresenting dendritic cells of the dermis. The most superficial cells of the epidermis provide most of the protection from ultraviolet (UV) irradiation (Chapter 20). Sensation emanates from nerves that initially traverse the subcutaneous fat to the dermis and epidermis before ending in specialized receptive organs or free nerve endings. The largest blood vessels of the skin are found in the subcutaneous fat, where they transport nutrients and circulating immune cells. The cutaneous lymphatics, which also course through the dermis and subcutaneous fat, filter debris and regulate tissue hydration. The skin’s pigmentation is regulated by melanocytes, whereas actinic damage and aging are influenced by the epidermis, the dermis, and the subcutaneous fat.

  EPIDERMIS

Epidermal Differentiation

The epidermis, which is the outermost layer of the skin, is composed primarily of keratinocytes that differentiate to form a stratified squamous epithelium.1 The epidermis is a continually renewing structure that gives rise to specialized derivatives called appendages—pilosebaceous units, sweat glands, and nails. The keratinocytes within the epidermis are organized into four layers, named for either their position or structural property (Fig. 435-2). Cells develop from the basal layer and then undergo programmed biochemical and morphologic changes that ultimately result in formation of the outermost stratum corneum, which serves as a hardened shield against the host environment. Melanocytes, Langerhans cells, and Merkel cells form the bulk of cells that reside within different layers of the epidermis. Some regional differences in the epidermis and its appendages are readily apparent (e.g., the thickness of palmoplantar and truncal skin compared with eyelid skin), whereas other differences are evident only at the microscopic or biochemical

level (e.g., differential expression of keratins in dorsal compared with volar skin).

The Basal Layer

The basal layer, which is the germinative layer of the epidermis, contains proliferative cells that give rise to the more differentiated levels of the epidermis. Keratinocyte differentiation (keratinization) is a genetically programmed, regulated, complex series of morphologic and metabolic changes whose end point is a terminally differentiated, cornified envelope that consists of dead keratinocytes (corneocytes) and contains a protein-reinforced plasma membrane with surface-associated lipids. All epithelial cells have intermediate filaments known as keratins. Keratins primarily serve a structural role, and they are expressed in region-specific and function-specific obligate heteropolymer pairs in a pattern that is determined by cell type, tissue type, developmental stage, differentiation stage, and disease condition (E-Table 435-1). The critical role of these molecules is underscored by the numerous manifestations of disease that arise because of mutations in their genes. Basal cells attach to their underlying basal lamina via keratin filaments at hemidesmosomes (E-Fig. 435-1) in the basement membrane zone. They also attach to each other and to other keratinocytes via desmosomes, which are specialized junctions required for epidermal integrity. Keratinocytes also contain melanosomes—vacuoles that contain pigment that is synthesized by melanocytes and transferred to keratinocytes via phagocytosis. This pigment helps protect against UV damage and contributes to macroscopic skin coloring. In the epidermis, the greatest amount of mitotic activity occurs in the basal layer. Cell kinetic studies suggest that different basal keratinocytes have different proliferative potentials, and in vivo and in vitro studies suggest the existence of long-lived epidermal stem cells. Because basal cells can be expanded in tissue culture and used to reconstitute enough epidermis to cover the entire skin surface of burn patients, these long-lived stem cells appear to have extensive proliferative potential.

The Spinous Layer

The spinous layer is characterized by the presence of abundant desmosomes, which are critical for tight epidermal adhesion and resemble intercellular “spines” under light microscopy. The classic vesiculobullous disease pemphigus vulgaris (Chapter 439) results from desmosomal disruption.

The Granular Layer

The granular layer, so named because of the abundant keratohyalin granules contained in the cells that constitute it, represents an intermediate phase of keratinocyte differentiation. This layer is where a number of the structural components will form the cornified cell envelope of the epidermal barrier, as well as a number of proteins that process these components. Keratohyalin granules are composed primarily of profilaggrin, keratin filaments, and loricrin. Profilaggrin is cleaved into filaggrin monomers, which aggregate with keratin to form macrofilaments. Eventually, filaggrin is degraded into molecules that contribute to hydration of the stratum corneum and help filter UV radiation. Loricrin, which is a cysteine-rich protein, is released from keratohyalin granules and then binds to desmosomal structures. Loricrin is subsequently cross-linked to the plasma membrane by tissue transglutaminases to help form the cornified cell envelope.

The Stratum Corneum

The final stage of granular cell differentiation into a corneocyte involves the cell’s own programmed destruction, a process during which almost all cellular contents are destroyed, with the exception of the keratin filaments and filaggrin matrix. The cells eventually extrude their nuclei and become a flattened stratum corneum that provides mechanical protection and a barrier to the loss of water or the entry of soluble substances from the environment. The stratum corneum barrier is formed by a two-compartment system of lipid-depleted, protein-enriched corneocytes surrounded by a continuous extracellular lipid matrix. The extracellular lipid matrix is primarily responsible for the regulation of permeability, desquamation, antimicrobial peptide activity, toxin exclusion, and selective chemical absorption. By comparison, corneocytes provide mechanical reinforcement, hydration, cytokinemediated initiation of inflammation, and protection from UV damage. Mutations in the transglutaminase enzymes and in structural proteins such as filaggrin and loricrin cause the ichthyoses and keratodermas, which result when a poorly formed epidermal barrier, both functionally and structurally,

CHAPTER 435  Structure and Function of the Skin  

2632.e1

E-TABLE 435-1  KERATIN EXPRESSION AND ASSOCIATED DISEASES KERATIN PAIRS

EXPRESSION PATTERN

DISEASE PHENOTYPE

K1/K10

Suprabasal keratinocytes

Bullous congenital ichthyosiform erythroderma; diffuse nonepidermolytic PPK

K1/K9

Palmoplantar suprabasilar keratinocytes

Epidermolytic PPK

K2e/K10

Upper spinous and granular layers

Ichthyosis bullosa of Siemens

K3/K12

Cornea

Meesmann’s corneal dystrophy

K4/K13

Mucosal epithelium

White sponge nevus

K5/K14

Basal keratinocytes

Epidermolysis bullosa simplex

K6a/K16

Outer root sheath (hair), hyperproliferative keratinocytes, palmoplantar keratinocytes

Pachyonychia congenita type I; focal nonepidermolytic PPK

K6b/K17

Nail bed, epidermal appendages

Pachyonychia congenita type II; steatocystoma multiplex

K8/K18

Simple epithelium

Cryptogenic cirrhosis

PPK = palmoplantar keratoderma

Keratin Desmoplakin

PG

PKP

DSC DSG

A

Keratin 5/14

Basal cell Plectin BP230

α6β4 Lamina lucida

Laminin 332

Kindlin CD151

Anchoring filaments

Collagen XVII

Lamina densa Dermis

Collagen VII

Anchoring fibrils

B E-FIGURE 435-1.  Adhesion in the epidermis. Cell-cell adhesion between keratinocytes is achieved via (A) desmosomes, whereas the basal cell keratinocytes attach to the basal lamina via (B) hemidesmosomes. alpha6β4 = alpha6β4 integrin; DSC = desmocollin; DSG = desmoglein; PG = plakoglobin; PKP = plakophilin. Note that kindlin-1 is an adhesion protein that has been localized not to hemidesmosomes but rather to focal contacts, adhesion complexes that facilitate association of the actin cytoskeleton to the basement membrane.

CHAPTER 435  Structure and Function of the Skin  

Sebaceous gland

2633

Capillary loop

Epidermis Proximal nail fold Matrix

Papillary dermis

Nail plate

Bulge cells Reticular dermis Subcutaneous fat

Bone Eccrine gland Hair follicle

Apocrine gland

FIGURE 435-1.  General structure of the skin and epidermal appendages. The epidermis is the outermost keratinized layer of skin, resting on top of the dermis via a basement membrane zone. The epidermal appendages are specialized structures and include the eccrine and apocrine sweat glands, sebaceous glands, hair, and nails. Capillary loops traverse the dermis to its most superficial aspect for diffusion and transport.

Basal layer

protein-encoding genes, such as ectodermal dysplasia–skin fragility syndrome (plakophilin 1). In addition, autoantibody diseases (e.g., bullous pemphigoid and pemphigus vulgaris) and bacterial infections (e.g., bullous impetigo and staphylococcal scalded skin syndrome) can inactivate these proteins, thereby resulting in numerous adhesion disorders that are manifested clinically as skin fragility and blistering conditions. Calcium is an important mediator of cell adhesion, as evidenced by HaileyHailey and Darier’s diseases, in which mutations in genes encoding calcium transporters disrupt calcium distribution, thereby resulting in characteristic epidermal dyscohesion and blistering. Many of the proteins involved in cellcell adhesion also require calcium for their binding or adhesive properties.

Basement membrane

Nonkeratinocytes of the Epidermis

Stratum corneum Granular layer Spinous layer

FIGURE 435-2. The epidermis and its layers of differentiation.

causes hyperkeratosis and scaling, derangements in water permeability and temperature regulation, and increased susceptibility to infections. Patients with filaggrin mutations are more susceptible to developing atopic dermatitis.2 The sequelae of these mutations highlight the importance of the epidermis and stratum corneum for maintaining tissue and organism homeostasis.

Epidermal Adhesion

Keratinocytes are held together by a number of intercellular junctions, including tight junctions, gap junctions, adherens junctions, and desmosomes. Complex protein networks form each of these types of junctions, and genetic mutations can have significant effects on the function of the epidermis (see E-Table 435-1). Tight junctions determine epidermal permeability by regulating the distribution of water-soluble molecules between adjacent cells. The primary structural proteins in these junctions are the claudins, and mutations in claudin 1 can cause a syndromic form of ichthyosis. Gap junctions, formed primarily by connexins, allow small molecules and electric current to pass between adjacent cells. Connexin mutations can cause a number of phenotypes in genetic syndromes, such as the keratitis-ichthyosis deafness syndrome. Adherens junctions are transmembrane structures that associate with the actin cytoskeleton. The extracellular interactions are calcium-dependent reactions between cadherins. These cadherin proteins associate with an intracellular complex including p120ctn, β-catenin, and plakoglobin. Plakoglobin and p-cadherin have been identified to be mutated in two genetic conditions, Naxos disease (plakoglobin) and hypotrichosis with juvenile macular dystrophy (p-cadherin). Desmosomes are cell-cell junctions that provide strength and rigidity to cells (see E-Fig. 435-1). They also participate in skin and tissue differentiation and development. Three protein families make up desmosomes—the armadillo proteins (such as plakoglobin and plakophilins), cadherins (including desmocollins and desmogleins), and plakins (desmoplakin, envoplakin, periplakin, plectin, BPAG1, corneodesmosin, and microtubule actin cross-linking factor). A number of genetic conditions involve mutations within these

Melanocytes synthesize the pigment molecule melanin, which performs a critical role in absorbing and mitigating UV damage to the skin and underlying tissues.3 A number of enzymes convert the amino acid tyrosine into different forms of melanin, which then are sorted into membrane-bound organelles known as melanosomes. These melanosomes are then transported to the tips of melanocytic dendrites and subsequently transferred to adjacent keratinocytes. Melanocytes are neuroendocrine derived cells that are not intrinsic cells of the skin but migrate there early in development. The function of melanocytes has been highlighted by disorders in melanocyte number or function. The classic dermatologic disease, vitiligo (Chapter 441), is caused by the autoimmune depletion of melanocytes, but disorders of pigmentation can be caused by dysfunction at any of the steps of melanogenesis. For example, various forms of oculocutaneous albinism result from defects in melanin synthesis, whereas the subtypes of Griscelli’s syndrome (skin and hair hypopigmentation, ocular albinism, neurologic disorders) result from mutations involved in lysosomal and melanosomal transfer of pigment to keratinocytes. Keratinocyte-melanocyte interactions are critical for melanocyte homeostasis and differentiation, influencing proliferation, dendricity, and melanization. Merkel cells, which are mechanoreceptors, are located in sites of high tactile sensitivity, including skin of the digits, lips, regions of the oral cavity, and the hair follicle. Ultrastructurally, Merkel cells are easily identified by the membrane-bounded, dense-core granules that contain neurotransmitter-like substances and markers of neuroendocrine cells. Merkel cell–derived neoplasms are particularly aggressive and difficult to treat. Langerhans cells are dendritic antigen-processing and antigen-presenting cells in the epidermis, mostly in a suprabasal position. Although they are not unique to the epidermis, they form 2 to 8% of the total epidermal cell population. Langerhans cells, which present antigens to T cells of the epidermis, are implicated in allergic contact dermatitis, cutaneous leishmaniasis, and human immunodeficiency virus (HIV) infection. Langerhans cells are reduced in the epidermis of patients with conditions such as psoriasis, sarcoidosis, and contact dermatitis, and they are functionally impaired by UV radiation, especially UVB.

Nonresident Cells of the Epidermis

Circulating immune cells that pass through the skin play key roles in infection control and immune surveillance. The activation of γ-δ T cells in the

2634

CHAPTER 435  Structure and Function of the Skin  

epidermis is an important step in wound healing. Recent advances in melanoma therapy (Chapter 203) target T-cell costimulatory molecules, thereby underscoring the importance of this subset of immune cells in host responses to malignancy.

Epidermal Appendages

The epidermal appendages, including the hair follicle, nails, sebaceous gland, eccrine gland, and apocrine gland (see Fig. 435-1), are specialized structures that have particular unique functions. Eccrine glands occur throughout the skin, but they are found in the highest concentrations on the palms, soles, and head. Eccrine glands, which are critical for thermoregulation, secrete a watery solution that includes sodium chloride, urea, uric acid, potassium, and immunoglobulins. Secretion is controlled by cholinergic sympathetic nerves. Apocrine glands, which are scent glands, are found in the axilla, external genitalia, areola, and perianal areas. Secretion by these glands is increased by fear, sexual excitement, and other situations that result in heightened tension. The nail unit (Chapter 442) consists of a nail matrix, nail plate, and nail bed. The proximal nail matrix is an organized germinative epithelium that produces the keratinized nail plate. The nail bed, which is a specialized epithelium located under the nail plate, attaches the nail plate to the digit. Sebaceous glands are found throughout the dermis except for the palms and soles. Many of these glands empty their contents through a duct into the lumen of hair follicles. In certain areas of the body, the glands occur in the absence of hairs (glans penis, lips, labia minora, and eyelids). Sebaceous secretions include triglycerides, waxy esters, squalene, cholesterol, and fatty acids. Hair follicles undergo a carefully orchestrated program of cyclical differentiation and proliferation. The hair follicle bulge consists of pluripotent skin stem cells that can give rise to all the layers of the follicle and hair shaft, as well as the epidermis under certain conditions of wound repair. The hair cycle (Chapter 442) consists of the phases of growth (anagen), involution (catagen), resting (telogen), and eventual release (exogen) of the mature hair follicle.

  THE DERMAL-EPIDERMAL JUNCTION

The dermal-epidermal junction is a basement membrane zone that forms the interface between the epidermis and dermis. The major functions of the dermal-epidermal junction are to attach the epidermis and dermis to each other and provide resistance against external shearing forces. This junction serves as a support for the epidermis, determines the polarity of growth, directs the organization of the cytoskeleton in basal cells, provides developmental signals, and serves as a semipermeable barrier. The dermal-epidermal junction can be subdivided into three supramolecular networks: the hemidesmosome-anchoring filament complex, the basement membrane, and the anchoring fibrils. The critical role of this region in maintaining skin structural integrity is exemplified by the large number of mutations or functional inactivations that cause blistering diseases4,5 (E-Table 435-2). For example, basal keratinocyte cleavage within the superficial dermalepidermal junction causes epidermoloysis bullosa simplex. Abnormalities within the lamina lucida and lamina densa regions cause junctional epidermolysis bullosa, and abnormalities within the sublamina densa/anchoring filaments cause the deep blistering of dystrophic epidermolysis bullosa.

  THE DERMIS

The dermis, which constitutes the bulk of the skin and provides its flexibility and mechanical strength, is an integrated system of connective tissue elements that accommodate networks of nerves and blood vessels. The dermis has two major regions: the upper papillary dermis and the deeper reticular dermis. The papillary dermis, which is usually no more than twice the thickness of the epidermis, runs just beneath it and undulates its contours. The reticular dermis, which forms the bulk of the dermal tissue, is composed primarily of collagen and elastin. The subpapillary plexus, a horizontal plane of vessels, marks the boundary between the papillary and reticular dermis. The lowest boundary of the reticular dermis is defined by the transition of its fibrous connective tissue to the adipose connective tissue of the subcutaneous fat.

Connective Tissue Matrix of the Dermis

Collagen, which forms the bulk of the acellular portion of the dermis, provides both tensile strength and elasticity. The periodically banded, interstitial collagens account for most of the collagen in the adult dermis (type I, 80 to 90%; III, 8 to 12%; and V, < 5%). Type IV collagen is confined to the basal

lamina of the dermal-epidermal junction, blood vessels, and epidermal appendages. Type VI collagen is associated with fibrils and interfibrillar spaces. Type VII collagen forms anchoring fibrils at the dermal-epidermal junction. The classic diseases of collagen function are the various subtypes of Ehlers-Danlos syndrome (Chapter 260),6 a heterogeneous collection of disorders characterized by joint hypermobility, skin extensibility, abnormal scarring, and tissue friability. Elastic connective tissue is a complex molecular mesh that includes elastin and fibrillin and that extends from the lamina densa of the dermal-epidermal junction throughout the dermis and into the connective tissue of the subcutaneous fat. Elastic fibers return the skin to its normal configuration after it has been stretched or deformed. Mutations in elastin, which is the elastic fiber matrix component, causes the disease cutis laxa, a condition that is characterized by sagging skin that has little extensibility and hangs in loose folds, especially noticeable on the neck and in the axillae and groin. Mutations in the gene encoding fibrillin, a microfibril component, are implicated in Marfan syndrome (Chapter 260). Elastic fibers are normally located between bundles of collagen fibers, although in certain pathologic conditions, such as BuschkeOllendorff syndrome (dermatofibrosis lenticularis disseminata), both elastic and collagen fibers become assembled within the same bundle, thereby resulting in the formation of characteristic benign connective tissue nevi and osteopoikilosis. Pseudoxanthoma elasticum (Chapter 260), which is characterized by loss of skin elasticity and calcified elastic fibers, results from mutations in ABCC6, a transmembrane transporter in the ATP-binding cassette transporter family, although the mechanisms responsible for the observed elastic fiber defects are not understood. In addition to genetic mutations, solar radiation and aging also damage elastic fibers. The fibrous and cellular elements of the dermis are embedded within a more amorphous matrix that can bind water and regulate the compressibility of the dermis. Various glycoproteins interact with other matrix components via integrin receptors to facilitate cell migration, adhesion, morphogenesis, and differentiation. Fibronectin, which is synthesized by both epithelial and mesenchymal cells, covers collagen bundles and the elastic network. Vitronectin is present on all elastic fibers except for oxytalan. Tenascin is found around the smooth muscle of blood vessels, arrector pili muscles, and appendages such as sweat glands.

Cellular Components of the Dermis

Fibroblasts, macrophages, and mast cells are the regular residents of the dermis. Fibroblasts migrate through the tissue and are responsible for the synthesis and degradation of matrix proteins and a number of soluble factors in the fibrous and nonfibrous connective tissue. Fibroblasts provide a structural extracellular matrix framework and also promote interactions between the epidermis and the dermis. Fibroblasts are also instrumental in wound healing and scarring, increasing their proliferative and synthetic activity during these processes. Monocytes, macrophages, and dermal dendrocytes constitute the mononuclear phagocytic system of cells in the skin. Macrophages are derived from precursors in the bone marrow, differentiate into circulating monocytes, and then migrate into the dermis to differentiate. These cells are phagocytic, antigen-processing and antigen-presenting, microbicidal, tumoricidal, secretory, and hematopoietic. They are also involved in coagulation, atherogenesis, wound healing, and tissue remodeling. The dermal dendrocyte is a phagocytic fixed connective tissue cell in the dermis of normal skin. These cells are particularly abundant in the papillary dermis and upper reticular dermis, where they function as antigen-presenting cells. They are also likely the cell of origin of a number of benign fibrotic proliferative conditions in the skin, such as dermatofibromas and fibroxanthomas. Mast cells, which are the specialized secretory cells responsible for the immediate-type hypersensitivity reaction in skin, are also involved in subacute and chronic inflammatory processes. These processes are mediated by the histamine, heparin, tryptase, chymase, carboxypeptidase, neutrophil chemotactic factor, and eosinophilic chemotactic factor of anaphylaxis that are synthesized in their granules and released in response to various stimuli. Mast cells also can become hyperplastic and hyperproliferative in mastocytosis (Chapter 255).

  SUBCUTANEOUS FAT

The subcutaneous fat insulates the body, serves as a reserve energy supply, cushions and protects the skin, and allows for its mobility over underlying structures. This fat is a key determinant of body contours and therefore plays

CHAPTER 435  Structure and Function of the Skin  

2634.e1

E-TABLE 435-2  HEMIDESMOSOMAL AND BASEMENT MEMBRANE ZONE COMPONENTS IN SKIN DISEASE BMZ REGION

PROTEINS

ACQUIRED DISEASES

HEREDITARY DISEASES

Cytoskeletal proteins

Keratin 5 and 14

EB simplex

Hemidesmosomal plaque protein

BPAG11/BP230 Plectin

Other intracellular adhesion complex proteins

Kindlin-1

Hemidesmosomal transmembrane components

Collagen XVII/BP180 alpha6β4 integrin CD151

Bullous pemphigoid, cicatricial pemphigoid, linear IgA dermatosis, pemphigoid getationis Bullous pemphigoid, cicatricial pemphigoid

JEB-non-Herlitz JEB w/pyloric atresia Pretibial EB, nephritis, deafness, β-thalassemia minor

Anchoring filament proteins

Laminin 332

Cicatricial pemphigoid

Ectodomain of collagen XVII

Linear IgA dermatosis, bullous pemphigoid

JEB-Herlitz JEB-non-Herlitz JEB-non-Herlitz

Anchoring fibril proteins

Collagen VII

EB acquisita

Dystrophic EB

Bullous pemphigoid Bullous pemphigoid, cicatricial pemphigoid

EB simplex Kindler syndrome

BMZ = basement membrane zone; EB = epidermolysis bullosa; JEB = junctional EB.

2635

CHAPTER 435  Structure and Function of the Skin  

an important cosmetic role as well. At the junction of the deep reticular dermis and the underlying fat layer is an abrupt transition from predominantly fibrous dermal connective tissue to primarily adipose tissue. Despite this anatomic contrast, the dermis and the subcutaneous fat are structurally and functionally integrated through networks of nerves and vessels, as well as in the continuity of epidermal appendages such as growing hair follicles and sweat glands. Adipocytes, which form the bulk of the cells in the subcutaneous fat layer, are organized into lobules defined by septa of fibrous connective tissue. Within the septa run the nerves and vascular structures that supply the region. Fat synthesis and storage can occur by enhanced accumulation of lipid within, or proliferation of, existing adipocytes or by the creation of new adipocytes from undifferentiated mesenchyme. The hormone leptin (Chapter 223), secreted by adipocytes, regulates fat homeostasis. Leptin levels are higher in subcutaneous than omental adipose tissue, thereby suggesting a role for leptin in the distribution of adipose tissue. The importance of the subcutaneous tissue is highlighted in conditions in which it is absent or abnormal. In Werner’s syndrome (Chapter 205), the absence of subcutaneous fat over bone lesions results in ulcers and poor wound healing. In scleroderma (Chapter 267), the replacement of subcutaneous fat with dense fibrous connective tissue results in taut and painful skin. In both the hereditary and acquired lipodystrophies (Chapter 392), loss of subcutaneous fat disrupts glucose, triglyceride, and cholesterol regulation and can cause significant cosmetic alteration. Inflammation of the subcutaneous fat, termed panniculitis (Chapters 142 and 266), can be caused by many different tissue derangements and systemic conditions.

  CUTANEOUS VASCULATURE

Blood Vessels

The rich vascular network of the skin is located at boundaries within the dermis and supplies the epidermal appendages. Dermal vessels branch from musculocutaneous arteries that penetrate the subcutaneous fat and enter the deep reticular dermis. At this point, they are organized into a horizontal arteriolar plexus. From this plexus, ascending arterioles extend toward the epidermis. At the junction between the papillary and reticular dermis, terminal arterioles form the subpapillary plexus. Capillary loops then extend from these terminal arterioles of the plexus into the more superficial papillary dermis. At the apex of each capillary loop, the thinnest portion of the vessel allows for diffusion and transport of material out of the capillary. The descending limbs of capillary loops drain into venous channels of the subpapillary plexus. The postcapillary venules of the subpapillary plexus are responsive to histamine and are therefore often the sites of inflammatory cells. In the adult, the cutaneous vasculature normally remains quiescent, in part owing to inhibition of angiogenesis by factors such as thrombospondin. Pathogenic stimuli, such as from tumors or after a wound, sometimes result in secondary angiogenesis. One of the key mediators of such angiogenesis is vascular endothelial growth factor, which often is secreted by tumors or by keratinocytes. Numerous disorders can manifest themselves within the cutaneous vasculature.7 Leukocytoclastic vasculitis (Chapters 270 and 439), also called cutaneous necrotizing venulitis, occurs within the venules in response to a number of potential pathogenic mechanisms, including medication reactions, infections, neoplasms, and systemic inflammatory conditions. Stasis dermatitis (Chapter 436), urticaria (Chapters 252 and 440), polyarteritis nodosa (Chapter 270), thrombosis (Chapter 70), and thrombophlebitis (Chapter 81) all affect different-sized vessels in the skin, some by occlusion of vessels (vasculopathy) and others by inflammation of the vessels (vasculitis).

Lymphatics

The lymph channels of the skin are responsible for resorbing fluid released by vessels and clearing the tissues of cells, proteins, lipids, bacteria, and degraded substances.8 These vessels begin in blind-ending loops in the papillary dermis and then continue to drain into successively larger plexuses deeper in the tissue. Lymphatic flow in the skin is propelled by arterial pulsations, muscle contractions, and movement of the body. Bicuspid-like valves within the lymphatic vessels promote unidirectional flow. Because lymphatic vessels are often collapsed in skin and because lymphatic vessels have thinner walls than blood vessels, they are rarely seen on routine histologic section. Pathologic conditions that involve or highlight the function of lymphatic vessels include lymphedema, lymphangioma circumscriptum (clustered, deep vesicles on the skin resulting from lymphatic dilation and

malformation), and stasis dermatitis. Lymphatics are also important for the progression and spread of cancer. For example, melanoma cells destroy the endothelial cells of the local lymphatics to gain entry to the lymphatic circulation, and tumors themselves can promote lymphangiogenesis as part of their process of metastasis.

  CUTANEOUS NERVES AND RECEPTORS

The nerve networks of the skin contain somatic sensory and sympathetic autonomic fibers. The sensory fibers alone (free nerve endings) or in conjunction with specialized structures (corpuscular receptors) function as receptors of touch, pain, temperature, itch, and mechanical stimuli. The differing density and types of receptors in different body regions account for the variation in sensory acuity at different body sites. Receptors are particularly dense in hairless areas such as the areola, labia, and glans penis. Sympathetic motor fibers run with the sensory nerves in the dermis until they branch to innervate the sweat glands, vascular smooth muscle, the arrector pili muscle of hair follicles, and sebaceous glands. The nerves of skin branch from musculocutaneous nerves that arise segmentally from spinal nerves. The pattern of nerve fibers in skin is similar to the vascular patterns—nerve fibers form a deep plexus, then ascend to a superficial, subpapillary plexus. Free nerve endings, which are the most widespread sensory receptors in skin, are particularly common in the papillary dermis. The penicillate fibers, which are the primary nerve fibers found subepidermally in haired skin, are rapidly adapting receptors that function in the perception of touch, temperature, pain, and itch. Papillary nerve endings are found at the orifice of a follicle and are thought to be particularly receptive to cold sensation. Corpuscular receptors, which contain a capsule and inner core, are composed of both neural and non-neural components. Meissner corpuscles are elongated or ovoid mechanoreceptors that are located in the dermal papillae of digital skin and oriented vertically toward the epidermal surface. Pacinian corpuscles lie in the deep dermis and subcutaneous tissue of skin that covers weight-bearing surfaces of the body; they have a characteristic capsule and lamellar wrappings and serve as rapidly adapting mechanoreceptors that respond to vibrational stimuli.

Pathophysiology of Pruritus

Pruritus (itching) is mediated by unmyelinated C fibers, the same fibers that are responsible for the transmission of pain.9 The receptors for these fibers are probably the free nerve endings in the dermis and epidermis. Stimuli that trigger these nerve fibers result in the transmission of signals along peripheral nerves to the dorsal root ganglion, spinal cord, and finally to the thalamus and other parts of the brain (E-Fig. 435-2). Pruritus, which can be subdivided into clinical categories based on the presumptive mechanism (Table 435-1),10 is associated with a number of systemic as well as skin-specific diseases (Table 435-2). Antihistamines, although commonly prescribed, are ineffective for many causes of pruritus— with the exception of urticaria—because they primarily cause sedation. Depending on the suspected mechanism of pruritus, classes of medication as diverse as the opiate antagonists (e.g., naltrexone), antidepressants (e.g., mirtazapine), anticonvulsants (e.g., gabapentin), and substance P antagonists (e.g., aprepitant) have proved to be useful (Tables 435-3 and 435-4). Although the precise targets of these classes of medications are poorly understood in pruritus, the clinical effectiveness of these agents may offer insight regarding the molecular and cellular pathways involved in these conditions.11 Ultraviolet

TABLE 435-1  PATHOPHYSIOLOGY OF PRURITUS PRURITUS CATEGORY

PRURITUS MEDIATOR

CLINICAL EXAMPLES

Pruritoceptive

Generated in skin, usually inflammatory

Atopic dermatitis, allergic contact dermatitis, lichen planus

Neurogenic (systemic)

Generated in central nervous system but without any neural pathology

Renal failure, liver disease, lymphoproliferative disorders, malignancy

Neuropathic

Neuronal pathology along afferent pathway

Brachioradial pruritus, notalgia paresthetica

Psychogenic

Caused by psychological disorder

Delusions of parasitosis

CHAPTER 435  Structure and Function of the Skin  

Brain

Thalamus

Contralateral spinothalamic tract

Epidermis Dorsal root ganglion

C-fiber

Spinal cord

E-FIGURE 435-2.  Pathways of pruritus. Itch triggered in the skin sends signals via nerve fibers to the dorsal root ganglion, spinal cord, and ascends via the contralateral spinothalamic tract to project to the thalamus. From there, itch is transmitted to several other regions of the brain.

2635.e1

TABLE 435-2  TERMINOLOGY TO DESCRIBE THE MORPHOLOGY OF INDIVIDUAL SKIN LESIONS TERM

DEFINITION

EXAMPLE

PRIMARY SKIN LESIONS: INITIAL PATHOLOGIC CHANGE Macule

Circumscribed change in skin color that is flush with the surrounding skin. Lesion is 1 cm in diameter

Psoriasis

Scale

Desiccated, thin plates of cornified epidermal cells that form flakes on the skin surface

Ichthyosis

Wheal

Circumscribed, flat-topped, firm elevation of skin with a well-demarcated and palpable margin

Urticaria

Vesicle

Circumscribed, elevated lesion containing clear serous or hemorrhagic fluid that is 2 cm in diameter

Bullous pemphigoid

Pustule

A vesicle containing purulent exudate

Folliculitis

Atrophy

A depression from the surface of the skin with underlying loss of epidermal or dermal substance

Lichen sclerosis et atrophicus

Erosion

A depression from the surface of the skin with a loss of all or part of the epidermis Can be a secondary lesion

Burn

A depression from the surface of the skin with a loss of the entire epidermis and at least some of the dermis Can be a secondary lesion

Ecthyma

Ulceration

TABLE 435-3  TOPICAL TREATMENTS FOR PRURITUS MEDICATION

DOSE

COMMENTS

Emollients

Variable

For skin barrier damage, dry skin itch

Corticosteroids

Variable

Useful in pruritus due to inflammatory skin dermatitides. Low-potency agents safest for use in children and on face and in skin folds

Calcineurin inhibitors

Tacrolimus 0.03 and 0.1% ointment Pimecrolimus 1% cream

For use in atopic dermatitis and contact dermatitis. Particularly useful in facial or anogenital pruritus. May cause transient burning and stinging

Doxepin

5% cream

Topical formulation of tricyclic antidepressant; 20-25% risk for sedation owing to systemic absorption

Menthol

1-5% cream

Useful in patients who report cooling as an alleviating factor. Higher concentrations can cause hypersensitivity reactions and burning sensation

Anesthetic agents   Lidocaine/ prilocaine   Capsaicin

2.5-5%

Useful for neuropathic and postburn itch. Risk for methemoglobinemia Particularly useful in neuropathic itch and itch caused by chronic kidney disease. May cause transient burning Useful on face and genitals, for chronic kidney disease, and for neuropathic itch

  Pramoxine

0.025-0.1% cream 1-2.5%

Data from Yosipovitch G, Bernhard JD. Chronic pruritus. N Engl J Med. 2013;368:1625-1634.

Ruptured bulla

TABLE 435-4  SYSTEMIC TREATMENTS FOR PRURITUS MEDICATION Antihistamines

Antidepressants   Tricyclic antidepressants

DOSE

Hydroxyzine 10-50 mg Sedating, but no direct effect on four times daily pruritus, except in urticaria Cetirizine 10 mg/day Fexofenadine 60-180 mg/day Amitriptyline 25-150 mg/day

Excoriation of acne papule

SECONDARY SKIN LESIONS: RESULT FROM EXTERNAL FORCES SUCH AS SCRATCHING, PICKING, INFECTION, OR HEALING OF PRIMARY LESIONS Lichenification

Dry, leathery thickening of the skin with exaggerated skin markings

Chronic eczema

Scar

An elevated or depressed area of fibrosis of the dermis or subcutaneous tissue resulting from an antecedent destructive process

Healing wound

Fissure

A deep linear split in the skin extending through the epidermis

Traumatized eczema

Crust

Dried exudates of serum, blood, sebum, or purulent material on the surface of the skin

Impetigo

Modified from Armstrong CA. Examination of the skin and approach to diagnosing skin diseases. In: Goldman L, Schafer AI, eds. Goldman’s Cecil Medicine. 24th ed. Philadelphia: Saunders; 2012: Table 444-1.

  Noradrenergic and specific serotonergic antidepressants   Selective serotonin reuptake inhibitors Opioids   µ Antagonist

COMMENTS

Mirtazapine 7.5-15 mg PO at bedtime

Neuropathic itch. May cause drowsiness, dizziness, constipation, urinary retention, blurred vision, palpitations, low blood pressure Nocturnal pruritus. May increase appetite and weight

Paroxetine 10-40 mg/ day PO Fluvoxamine 25-150 mg/day PO Sertraline 50-200 mg/ day PO

Psychiatric patients and paraneoplastic pruritus Psychiatric patients and paraneoplastic pruritus Cholestatic pruritus

Naltrexone 25-50 mg/ day PO

Pruritus associated with cholestatic or chronic kidney disease. Can cause nausea, vomiting, and drowsiness Intractable itch. May cause nausea, vomiting, and drowsiness

  κ Agonist and µ antagonist

Butorphanol, 1-4 mg inhaled at bedtime

Anticonvulsants

Gabapentin 100-1200 mg PO three times per day Pregabalin 25-200 mg PO twice per day

Neuropathic itch and pruritus from chronic kidney disease. Can cause drowsiness, weight gain, and leg swelling

Substance P antagonist

Aprepitant 80 mg/day PO

Sézary syndrome

Immunosuppressants Cyclosporin 2.5-5 mg/ kg/day PO Azathioprine 2.5 mg/ kg/day PO

Short-term use for refractory atopic dermatitis. Monitor blood pressure and renal function Refractory atopic dermatitis. Monitor for myelosuppression

Ultraviolet B radiation (broad and narrow band)

Atopic dermatitis, psoriasis, pruritus from chronic kidney disease

Three times per week

Data from Yosipovitch G, Bernhard JD. Chronic pruritus. N Engl J Med. 2013;368:1625-1634. PO = orally.

CHAPTER 436  AN APPROACH TO DIAGNOSING SKIN DISEASES  

phototherapy is also effective for multiple forms of pruritus, including pruritus that results from chronic kidney disease, as well as more inflammatory conditions such as psoriasis and atopic dermatitis; however, the mechanism underlying the resulting improvement is poorly understood. GENERAL REFERENCES For the General References and other additional features, please visit Expert Consult at https://expertconsult.inkling.com.

436  EXAMINATION OF THE SKIN AND AN APPROACH TO DIAGNOSING SKIN DISEASES JAMES C. SHAW

Dermatology encompasses well over a thousand disease entities, many of which expand further into multiple subclassifications, variants, and etiologies. The first goal of a clinician is quickly to recognize the few skin diseases that can rapidly cause severe morbidity or even kill the patient. For the remaining hundreds of non–life-threatening dermatoses, a careful history and physical examination, which is often performed before or simultaneously with the history, can help the clinician make an accurate diagnosis or obtain expert dermatologic referral for doing so.1

  THE SKIN EXAMINATION

The basic requirements for good examination of the skin are lighting and magnification. The best lighting is natural daylight or window light, but bright fluorescent ceiling lights, surgical lamps, or specialized magnifying lights all suffice. For simple magnification, a 4× hand lens is highly effective, but a 10× handheld polarized dermatoscope is even better, especially for diagnosing melanoma2 and basal cell carcinoma.

Color

2637

Another clue to diagnosis is determining the depth of the lesion (Chapter 435) and whether it involves the epidermis, the dermis, the fat layer below the dermis, or more than one layer (Table 436-2). Epidermal pathology includes the disorderly yet benign growth of keratinocytes resulting in flaking (scaling) and a thickened epidermis (psoriasis); the presence of microvesiculation (histologic term: spongiosis) that leads to oozing of serum (contact dermatitis); and hyperplasia of the epidermis in benign lesions (seborrheic keratoses), infections (warts), or malignancies (basal cell carcinomas and squamous cell carcinomas). Dermal pathology often leads to inflammation because of the recruitment of lymphocytes, neutrophils, and histiocytes, as well as infiltration of malignant cells (lymphoma), antibody deposition (lupus erythematosus, autoimmune blistering diseases, drug eruptions), infections, or vascular damage. Erythema, which is caused by vasodilation of vasculature, is the most common dermal finding, even in diseases that are primarily epidermal. Abnormalities are less common in the subcutaneous fat than in the epidermis or dermis, but benign hyperplasia of the fat layer results in benign lipomas. Vasculitis of midsize vessels will often extend into the fat layer in polyarteritis nodosa (Chapter 270), and several panniculitides (e.g., erythema nodosum) also involve the fat.

  HISTORY

The two most important facts to establish immediately from the history are whether the problem is acute (new onset within the last few days or hours) and whether the patient is systemically ill. Key clues to the diagnosis come from whether the lesions are associated with pruritus or pain, and what medications, especially new medications, the patient is taking (Table 436-3). For example, the absence of pruritus makes the diagnosis of allergic contact dermatitis (Chapter 440) highly unlikely. In the setting of unilateral acute severe pain, burning, and itching, herpes zoster (Chapter 439) should always be considered, even in the absence of skin findings.

  DIFFERENTIAL DIAGNOSIS

Morphology

In the ambulatory outpatient who does not have a life-threatening skin problem, lesions are commonly classified as macules, patches, papules, plaques, nodules, tumors, vesicles, bullae, or pustules (Table 436-4). However, plain language can be just as effective in arriving at an accurate diagnosis. For example, a description of “small, raised solid bumps 0.4 cm in diameter” is as clear as “multiple papules.” Furthermore, if dermatologic terms are used incorrectly, clinicians may be misled toward an incorrect diagnosis. Although primary lesions (which develop de novo) are most helpful in leading to a correct diagnosis, secondary features (which are altered by superinfection, manipulation, healing, etc.) are also important. Text continued on p. 2644

Attention to the color of the skin lesions can be key to making a correct diagnosis (Table 436-1). The color of the adjacent normal skin also can influence the appearance of dermatoses. Subtle colors such as violaceous or yellow or even simple erythema can be more difficult to appreciate in darker skin types than in light-colored skin (Fig. 436-1). The lighter an individual’s skin, the higher the risk is for developing sun-induced skin cancer (Chapter 203).

Palpation

Palpation with an ungloved hand is often important in dermatologic diagnosis. Examples include the skin fibrosis and sclerosis of scleroderma (Chapter 267), the induration and firmness of cellulitis (Chapter 290), the roughness of subtle actinic keratoses (Chapter 440), the firmness of a dermatofibroma (Chapter 440), the palpable purpura of leukocytoclastic vasculitis (Chapter 439), and the difference between a lipoma (soft, subcutaneous) and an epidermoid cyst (firmer, intradermal) or between a dermal nevus (somewhat firm) and a solitary neurofibroma (rubbery soft). A glove should be worn for protection when examining patients who may have blood-borne diseases, such as hepatitis B (Chapters 148 and 149) and human immunodeficiency virus (HIV) infections, or patients who may have secondary syphilis (Chapter 319) or herpes simplex (Chapter 374). Gloves are also essential when the clinician wishes to examine mucosae, denuded skin, blood, or exudates. Routine skin infections such as scabies (Chapter 441), human papillomavirus infection (Chapter 373), and superficial fungal infections (Chapter 438) are not highly contagious, but handwashing after contact is important to prevent transmission.

A

B

FIGURE 436-1.  Skin pigmentation. Lichen planus presents differently in darkly pigmented (A) versus lightly pigmented (B) skin. The violaceous hue seen in B is more muted in A, and these lesions appear brown-black in color. Wickham striae (lacy white pattern) are more easily seen in B. (From Bolognia JL, Jorizzo JL, Schaffer JV. Dermatology, 3rd ed. Philadelphia: Saunders; 2012:8.)

CHAPTER 435  Structure and Function of the Skin  

GENERAL REFERENCES 1. Eckhart L, Lippens S, Tschachler E, et al. Cell death by cornification. Biochim Biophys Acta. 2013;1833:3471-3480. 2. Kubo A, Nagao K, Amagai M. Epidermal barrier dysfunction and cutaneous sensitization in atopic diseases. J Clin Invest. 2012;122:440-447. 3. Bonaventure J, Domingues MJ, Larue L. Cellular and molecular mechanisms controlling the migration of melanocytes and melanoma cells. Pigment Cell Melanoma Res. 2013;26:316-325. 4. Baum S, Sakka N, Artsi O, et al. Diagnosis and classification of autoimmune blistering diseases. Autoimmun Rev. 2014;13:482-489. 5. Kershenovich R, Hodak E, Mimouni D. Diagnosis and classification of pemphigus and bullous pemphigoid. Autoimmun Rev. 2014;13:477-481.

2637.e1

6. Malfait F, De Paepe A. The Ehlers-Danlos syndrome. Adv Exp Med Biol. 2014;802:129-143. 7. Wollina U, Unger L, Haroske G, et al. Classification of vascular disorders in the skin and selected data on new evaluation and treatment. Dermatol Ther. 2012;25:287-296. 8. Kesler CT, Liao S, Munn LL, et al. Lymphatic vessels in health and disease. Wiley Interdiscip Rev Syst Biol Med. 2013;5:111-124. 9. Misery L, Brenaut E, Le Garrec R, et al. Neuropathic pruritus. Nat Rev Neurol. 2014;10:408-416. 10. Yosipovitch G, Bernhard JD. Clinical practice: chronic pruritus. N Engl J Med. 2013;368: 1625-1634. 11. Misery L, Brenaut E, Le Garrec R, et al. Neuropathic pruritus. Nat Rev Neurol. 2014;10:408-416.

2637.e2

CHAPTER 435  Structure and Function of the Skin  

REVIEW QUESTIONS 1. The proliferative layer of the epidermis is the ______. A. Basal layer. B. Spinous layer. C. Granular layer. D. Stratum corneum. E. Stratum lucidum Answer: A  The basal layer is the most undifferentiated layer of the epidermis and the layer in which most proliferation occurs. The other, more superficial layers of the epidermis are formed from keratinocytic differentiation. The stratum lucidum is a compact layer between the granular layer and stratum corneum and is present only in palmoplantar skin. 2. Marfan syndrome is caused by a mutation in the gene encoding _____. A. Elastin B. Fibrillin-1 C. Elaunin D. Type I procollagen E. Type V collagen Answer: B  Mutations in fibrillin-1 cause Marfan syndrome. Mutations in elastin result in cutis laxa. Elaunin is an elastic fiber component that does not have a described genodermatosis. Mutations in type I procollagen cause osteogenesis imperfecta, whereas mutations in type V collagen can result in Ehlers-Danlos syndrome.

3. Which of the following does not play a role in the pathogenesis of itch? A. Dorsal root ganglion B. Thalamus C. Pituitary gland D. Spinothalamic tract neurons E. Free nerve endings Answer: C  The pituitary gland is not involved in the itch pathway. The other choices are all found along the pathway for itch perception between the free nerve endings of the skin and the processing centers in the brain. 4. Defects in all of the following proteins can cause blistering diseases except: A. Keratin 5 B. Laminin 332 C. Filaggrin D. Collagen VII E. Plectin Answer: C  Defects in filaggrin can lead to ichthyosis and atopic dermatitis, but not a blistering disease. Defects in keratin 5 and plectin can cause epidermolysis bullosa simplex. Mutations in laminin 332 can cause junctional epidermolysis bullosa. Collagen VII forms anchoring fibrils, defects in which can cause dystrophic epidermolysis bullosa.

2638

CHAPTER 436  AN APPROACH TO DIAGNOSING SKIN DISEASES  

TABLE 436-1  COLOR CLUES IN DIAGNOSING SKIN DISEASE COLOR

CLINICAL EXAMPLE

Erythema: Pink Pink is a common finding when vasodilation (with minimal cellular infiltrate) is the primary dermal pathology.

FIGURE 436-2.  Urticaria (Chapter 440).

Erythema: Pink-Red Although many dermatoses can be pink-red, pityriasis rosea is classic.

FIGURE 436-3.  Pityriasis rosea (Chapter 438).

Erythema: Orange-Red (with Scale) Color in psoriasis ranges from orange-red to purple-red. A related condition, pityriasis rubra pilaris, also is orange-red.

FIGURE 436-4.  Psoriasis (Chapter 438).

CHAPTER 436  AN APPROACH TO DIAGNOSING SKIN DISEASES  

TABLE 436-1  COLOR CLUES IN DIAGNOSING SKIN DISEASE—cont’d COLOR

CLINICAL EXAMPLE

Erythema: Red-Orange-Pink A mixture of erythematous hues is common and makes diagnosis difficult by color alone.

FIGURE 436-5.  Allergic contact dermatitis (Chapter 438).

Erythema: Copper-Red Mixed cellular infiltrates with plasma cells, classically seen in syphilis.

FIGURE 436-6.  Secondary syphilis (Chapter 319).

Erythema: Red-Brown The color of histiocytic infiltrates, seen commonly in granulomatous diseases (e.g., cutaneous tuberculosis, deep fungal infections, atypical mycobacteria, sarcoidosis, granuloma annulare, leprosy, necrobiosis lipoidica) depends on depth of the infiltrates.

FIGURE 436-7.  Sarcoidosis (Chapter 95).

2639

2640

CHAPTER 436  AN APPROACH TO DIAGNOSING SKIN DISEASES  

TABLE 436-1  COLOR CLUES IN DIAGNOSING SKIN DISEASE—cont’d COLOR

CLINICAL EXAMPLE

FIGURE 436-8.  Leprosy (Chapter 326).

FIGURE 436-9.  Necrobiosis lipoidica (note the characteristic slight yellow tinge;

Chapter 440).

Violaceous: Purple Lymphocytes in the dermis impart a violaceous color in lichen planus, lupus erythematosus, lymphoma cutis, and pseudolymphoma.

FIGURE 436-10.  Lichen planus (pruritic purple polygonal papules; Chapter 438).

CHAPTER 436  AN APPROACH TO DIAGNOSING SKIN DISEASES  

2641

TABLE 436-1  COLOR CLUES IN DIAGNOSING SKIN DISEASE—cont’d COLOR

CLINICAL EXAMPLE

FIGURE 436-11.  Lupus erythematosus on sun-exposed surface (the “butterfly” rash when more confluent; Chapter 266).

Violaceous: Purple to Black Blood in the dermis (e.g., ecchymoses, vasculitis, subungual hemorrhage) leads to a dark purple color with blue and black overtones. Unlike blood in vessels, lesions do not blanch with pressure.

FIGURE 436-12.  Purpura. Small vessel vasculitis with small lesions coalescing into larger plaque (Chapter 439).

FIGURE 436-13.  Subungual hemorrhage can mimic melanoma.

2642

CHAPTER 436  AN APPROACH TO DIAGNOSING SKIN DISEASES  

TABLE 436-1  COLOR CLUES IN DIAGNOSING SKIN DISEASE—cont’d COLOR

CLINICAL EXAMPLE

Gray Necrotic keratinocytes (e.g., center of target lesions of erythema multiforme and the border of pyoderma gangrenosum) impart a grayish color. Gray color also is seen with melanin in the dermis (e.g., postinflammatory pigmentary change).

FIGURE 436-14.  Border of pyoderma gangrenosum (Chapter 440); note gray representing keratinocyte necrosis.

FIGURE 436-15.  Postinflammatory gray centrally in subacute cutaneous lupus (Chapter 266) in person with olive skin.

Black Black is a sign of dermal and epidermal necrosis (e.g., polyarteritis nodosa, antiphospholipid antibody syndrome, warfarin necrosis, heparin necrosis, calciphylaxis, ANCA-positive vasculitis) from vascular compromise, infection (e.g., mucormycosis, aspergillosis, anthrax, DIC from meningococcemia), or from melanin (the more superficial the melanin, the darker the color).

FIGURE 436-16.  Ischemic necrosis secondary to vasculopathy from levamisolecontaminated cocaine. (From Bolognia JL, Jorizzo JL, Schaffer JV. Dermatology, 3rd ed. Philadelphia: Saunders; 2012: Table 0.4.)

CHAPTER 436  AN APPROACH TO DIAGNOSING SKIN DISEASES  

2643

TABLE 436-1  COLOR CLUES IN DIAGNOSING SKIN DISEASE—cont’d COLOR

CLINICAL EXAMPLE

FIGURE 436-17.  Melanoma (Chapter 203) demonstrating superficial deposition of melanin in the nail plate.

Brown Brown is a result of pathologic pigments (e.g., hemosiderin; hemochromatosis; drug induced) in the dermis or normal epidermal melanin in individuals of color.

FIGURE 436-18.  Tinea versicolor (Chapter 438); note bran-colored scaling.

FIGURE 436-19.  Stasis dermatitis (Chapter 440); note brown superiorly from

hemosiderin.

2644

CHAPTER 436  AN APPROACH TO DIAGNOSING SKIN DISEASES  

TABLE 436-1  COLOR CLUES IN DIAGNOSING SKIN DISEASE—cont’d COLOR

CLINICAL EXAMPLE

White White can be seen in hyperplastic mucosal epithelium, sclerosis of dermis, scar tissue, loss of pigmentation, or small epidermal cysts.

FIGURE 436-20.  Oral hairy leukoplakia (Chapter 425).

FIGURE 436-21.  Lichen sclerosis (Chapter 440) (From Bolognia JL, Jorizzo JL, Schaffer JV. Dermatology, 3rd ed. Philadelphia: Saunders; 2012: Fig. 44-12). ANCA = antineutrophil cytoplasmic antibodies; DIC = disseminated intravascular coagulation.

Purpura refers to the appearance of skin into which red blood cells have extravasated, thereby producing a bluish to dark purple to black color. Purpura is macular in cases of tiny petechiae and ecchymoses from minor trauma. In small vessel leukocytoclastic vasculitis, purpura becomes palpable, usually as small papules. In deeper vasculopathies, purpura can be nodular, linear, stellate or associated with ulceration because of damage to vascular structures in the dermis or subcutaneous fat. Scaling, which is a manifestation of abnormal production of stratum corneum, is frequently an important clue to the correct diagnosis. It is commonly seen as a primary component of conditions such as psoriasis, tinea, and pityriasis rosea. Secondary characteristics include crusting, fissures, erosion, ulceration, excoriation, atrophy, and lichenification (Table 436-5). Each may be helpful in diagnosing a skin disorder depending on whether the finding represents a normal evolution of the disease, such as ulcer formation in pyoderma gangrenosum, or an extraneous influence, such as lichenification from scratching in a patient with atopic dermatitis.

Distribution

Some dermatologic diseases can be recognized by where they manifest on the body (Table 436-6). Prime examples include herpes zoster (unilateral dermatomal distribution of vesicles), dermatitis herpetiformis (pruritic vesicles over extensor areas of elbows, knees, and sacral skin), and hidradenitis suppurativa (inflammatory acne-like nodules and cysts in the axillae, inguinal, and intergluteal areas). Other sites manifestation include sun-exposed skin for lupus erythematosus and dermatomyositis, facial distribution for acne vulgaris and rosacea, and penile involvement for psoriasis and lichen planus.

Sun-exacerbated diseases spare areas under the nose and chin, inner arms, intertriginous regions, the mid and distal phalanges, and clothed areas. In a patient with bilateral erythema and swelling of the legs with dermatitis, the odds of cellulitis are extremely low, because cellulitis of the extremities is a unilateral disease. A1 

Life-Threatening Emergencies

In the acutely ill patient with a dermatosis, a clinician must determine whether the patient has a potentially rapidly fatal skin disease—typically a systemic infection or severe drug reaction—and if so, what treatment to initiate urgently. The most common rapidly fatal diseases with skin involvement are disseminated herpes zoster infection3 (mortality up to 30%, Chapter 375); disseminated herpes simplex infection (mortality up to 80% in the presence of encephalitis, Chapter 374); toxic epidermal necrolysis4 or its variant Stevens-Johnson syndrome (mortality up to 20% for toxic epidermal necrolysis Chapter 440); meningococcemia (mortality up to 35%, Chapter 298); toxic shock syndrome (mortality up to 30%, Chapters 288 and 290); necrotizing fasciitis (mortality 20 to 40%, Chapters 288 and 290); and disseminated fungal diseases including candidiasis (Chapter 338), histoplasmosis (Chapter 332), cryptococcosis (Chapter 336). In the acutely ill patient, physical findings that can often help make the correct diagnosis include the following: • Multiple small, discrete lesions (10 to several hundred; 100), small (12 µg/mL

12-117 µmol/L >70 µmol/L

Gentamicin

Serum or plasma (EDTA)

Therapeutic:   Peak (severe infection)   Trough (severe infection) Toxic:   Peak   Trough

8-10 µg/mL 10 µg/mL >2 µg/mL

16.7-20.9 µmol/L 21 µmol/L >4 µmol/L

Glutethimide

Serum

Therapeutic: Toxic:

2-6 µg/mL >5 µg/mL

9-28 µmol/L >23 µmol/L

Haloperidol

Serum or plasma (heparin or EDTA)

Therapeutic: Toxic:

5-17 ng/mL >42 ng/mL

13-45 nmol/L >112 nmol/L

Ibuprofen

Serum or plasma (heparin or EDTA)

Therapeutic: Toxic:

10-50 µg/mL >200 µg/mL

49-243 µmol/L >970 µmol/L

Imipramine + desipramine

Serum or plasma (heparin or EDTA); trough (>12 hr after dose)

Therapeutic: Toxic:

150-300 ng/mL >400 ng/mL

536-1071 nmol/L >1428 nmol/L

Isoniazid

Serum or plasma (heparin or EDTA)

Therapeutic: Toxic:

1-7 µg/mL >20 µg/mL

7-51 µmol/L >146 µmol/L

Itraconazole

Serum or plasma

Therapeutic:

>1.5 µg/mL

>2 µmol/L

Kanamycin

Serum or plasma (EDTA)

Therapeutic:   Peak   Trough (severe infection) Toxic:   Peak   Trough

25-35 µg/mL 4-8 µg/mL

52-72 µmol/L 8-16 µmol/L

>35 µg/mL >10 µg/mL

>72 µmol/L >21 µmol/L

Lidocaine

Serum or plasma (heparin or EDTA); >45 min following bolus dose

Therapeutic: Toxic:

1.5-5 µg/mL >6 µg/mL

6-21 µmol/L >26 µmol/L

Lithium

Serum or plasma (heparin or EDTA); >12 hr after last dose

Therapeutic: Toxic:

0.5-1.2 mEq/L >2 mEq/L

0.5-1 nmol/L >2 µmol/L

Lorazepam

Serum or plasma (heparin or EDTA)

Therapeutic:

50-240 ng/mL

156-746 nmol/L

Meperidine

Serum or plasma (heparin or EDTA)

Therapeutic: Toxic:

70-500 ng/mL >1 µg/mL

283-2020 nmol/L >4040 nmol/L

Meprobamate

Serum

Therapeutic: Toxic:

6-12 µg/mL >60 µg/mL

28-55 µmol/L >275 µmol/L

Methadone

Serum or plasma (heparin or EDTA)

Therapeutic: Toxic:

100-400 ng/mL >2000 ng/mL

0.32-1.29 µmol/L >6.46 µmol/L

Methamphetamine

Serum

Therapeutic: Toxic:

0.01-0.05 µg/mL >0.5 µg/mL

0.07-0.34 µmol/L >3.35 µmol/L

Methaqualone

Serum or plasma (heparin or EDTA)

Therapeutic: Toxic:

2-3 µg/mL >10 µg/mL

8-12 µmol/L >40 µmol/L

Methsuximide (N-desmethyl Serum methsuximide)

Therapeutic: Toxic:

10-40 µg/mL >40 µg/mL

53-212 µmol/L >212 µmol/L

Methyldopa

Plasma (EDTA)

Therapeutic: Toxic:

1-5 µg/mL >7 µg/mL

4.7-23.7 µmol/L >33 µmol/L

Methyprylon

Serum

Therapeutic: Toxic:

8-10 µg/mL >50 µg/mL

43-55 µmol/L >273 µmol/L

Morphine

Serum or plasma (heparin or EDTA)

Therapeutic: Toxic:

10-80 ng/mL >200 ng/mL

35-280 nmol/L >700 nmol/L

Netilmicin

Serum or plasma (EDTA)

Therapeutic:   Peak (severe infection)   Trough (severe infection) Toxic:   Peak   Trough

8-10 µg/mL 2 µg/mL

>21 µmol/L >4 µmol/L

2721

APPENDIX TABLE APPENDIX-4 DRUGS: THERAPEUTIC AND TOXIC—cont’d DRUG

SPECIMEN

REFERENCE INTERVAL (CONVENTIONAL UNITS)

REFERENCE INTERVAL (INTERNATIONAL UNITS)

Nortriptyline

Serum or plasma (heparin or EDTA); trough (>12 hr after dose)

Therapeutic: Toxic:

70-170 ng/mL >500 ng/mL

266-646 nmol/L >1900 nmol/L

Oxazepam

Serum or plasma (heparin or EDTA)

Therapeutic:

0.2-1.4 µg/mL

0.70-4.9 µmol/L

Oxycodone

Serum

Therapeutic: Toxic:

10-100 ng/mL >200 ng/mL

32-317 nmol/L >634 nmol/L

Paroxetine

Serum or plasma

Therapeutic:

70-120 ng/mL

213-365 nmol/L

Pentazocine

Serum or plasma (EDTA)

Therapeutic: Toxic:

0.05-0.2 µg/mL >1 µg/mL

0.2-0.7 µmol/L >3.5 µmol/L

Pentobarbital

Serum or plasma (heparin or EDTA); trough

Therapeutic, hypnotic: Therapeutic, coma: Toxic:

1-5 µg/mL 20-50 µg/mL >10 µg/mL

4-22 µmol/L 88-221 µmol/L >44 µmol/L

Phenacetin

Plasma (EDTA)

Therapeutic: Toxic:

1-30 µg/mL 50-250 µg/mL

6-167 µmol/L 279-1395 µmol/L

Phenobarbital

Serum or plasma (heparin or EDTA); trough

Therapeutic: Toxic:   Slowness, ataxia, nystagmus   Coma with reflexes   Coma without reflexes

10-40 µg/mL

43-173 µmol/L

35-80 µg/mL 65-117 µg/mL >100 µg/mL

151-345 µmol/L 280-504 µmol/L >430 µmol/L

Phensuximide + norphensuximide

Serum or plasma (heparin or EDTA)

Therapeutic:

40-60 µg/mL

212-317 µmol/L

Phenylbutazone

Plasma (EDTA)

Therapeutic: Toxic:

50-100 µg/mL >100 µg/mL

162-324 µmol/L >324 µmol/mL

Phenytoin

Serum or plasma (heparin or EDTA); trough

Therapeutic: Toxic:

10-20 µg/mL >20 µg/mL

40-79 µmol/L >79 µmol/L

Primidone + phenobarbital

Serum or plasma (heparin or EDTA) trough

Therapeutic: Toxic:

5-10 µg/mL >15 µg/mL

23-46 µmol/L >69 µmol/L

Procainamide

Serum or plasma (heparin or EDTA); trough

Therapeutic: Toxic:

4-10 µg/mL >12 µg/mL

17-42 µmol/L >51 µmol/L

Propoxyphene

Plasma (EDTA)

Therapeutic: Toxic:

0.1-0.4 µg/mL >0.5 µg/mL

0.3-1.2 µmol/L >1.5 µmol/L

Propranolol

Serum or plasma (heparin or EDTA); trough

Therapeutic:

20-100 ng/mL

77-386 nmol/L

Protriptyline

Serum or plasma (heparin or EDTA); trough (>12 hr after dose)

Therapeutic: Toxic:

70-260 ng/mL >500 ng/mL

266-988 nmol/L >1900 nmol/L

Quinidine

Serum or plasma (heparin or EDTA); trough

Therapeutic: Toxic:

2-5 µg/mL >6 µg/mL

6-15 µmol/L >19 µmol/L

Risperidone + 9-hydroxyrisperidone

Serum or plasma

Therapeutic:

20-60 ng/mL

49-146 nmol/L

Salicylates

Serum or plasma (heparin or EDTA); trough

Therapeutic: Toxic:

150-300 µg/mL >300 µg/mL

1086-2172 µmol/L >2172 µmol/L

Secobarbital

Serum

Therapeutic: Toxic:

1-2 µg/mL >5 µg/mL

4.2-8.4 µmol/L >21.0 µmol/L

Sirolimus

Whole blood

Therapeutic: Toxic:

4-20 ng/mL >20 ng/mL

4-22 nmol/L >22 nmol/L

Sulfonamides as sulfanilamide

Serum or plasma

Therapeutic: Toxic:

5-15 mg/mL >20 mg/mL

29-87 mmol/L >116 mmol/L

Theophylline

Serum or plasma (heparin or EDTA)

Therapeutic: Toxic:

8-20 µg/mL >20 µg/mL

44-111 µmol/L >111 µmol/L

Thiopental

Serum or plasma (heparin or EDTA); trough

Hypnotic: Coma: Anesthesia: Toxic:

1-5 µg/mL 30-100 µg/mL 7-130 µg/mL >10 µg/mL

4.1-20.7 µmol/L 124-413 µmol/L 29-536 µmol/L >41 µmol/L

Thioridazine

Serum or plasma (heparin or EDTA)

Therapeutic: Toxic:

0.2-2.0 µg/mL >10 µg/mL

0.5-5 µmol/L >27 µmol/L

Tobramycin

Serum or plasma (heparin or EDTA)

Therapeutic:   Peak (severe infection)   Trough (severe infection) Toxic:   Peak   Trough

8-10 µg/mL 2 µg/mL

>21 µmol/L >4 µmol/L

2722

APPENDIX

TABLE APPENDIX-4 DRUGS: THERAPEUTIC AND TOXIC—cont’d DRUG

SPECIMEN

REFERENCE INTERVAL (CONVENTIONAL UNITS)

REFERENCE INTERVAL (INTERNATIONAL UNITS)

Tocainide

Serum or plasma (heparin or EDTA)

Therapeutic: Toxic:

6-15 µg/mL >15 µg/mL

31-78 µmol/L >78 µmol/L

Tolbutamide

Serum

Therapeutic: Toxic:

90-240 µg/mL >640 µg/mL

333-888 µmol/L >2368 µmol/L

Valproic acid

Serum or plasma (heparin or EDTA); trough

Therapeutic: Toxic:

50-100 µg/mL >100 µg/mL

347-693 µmol/L >693 µmol/L

Vancomycin

Serum or plasma (heparin or EDTA); trough

Therapeutic:   Peak:   Trough: Toxic: (not well established)

20-40 µg/mL >10 µg/mL >80 µg/mL

14-28 µmol/L >7 µmol/L >55µmol/L

Warfarin

Serum or plasma (heparin or EDTA)

Therapeutic: Toxic:

1-10 µg/mL >10 µg/mL

3-32 µmol/L >32 µmol/L

Zidovudine

Serum or plasma

Therapeutic:

>0.2 µg/mL

>0.8 µmol/L

INDEX Page numbers followed by “f” indicate figures, “t” indicate tables, and “b” indicate boxes.

A

A blood group antigens, 238 A disintegrin-like and MMPs with thrombospondin type 1 repeats (ADAMTS), 1732-1733 AADLs. See Advanced activities of daily living Abacavir, 2288t CSF-to-plasma ratios, 2331t fixed-dose combinations, 2289, 2290t for HIV/AIDS, 2290t side effects of, 2289 Abatacept (Orencia), 171, 1762 Abciximab (ReoPro), 180, 438t-439t, 448 Abdomen: acute emergencies of, 2252t-2253t Abdominal actinomycosis, 2061 Abdominal aortic aneurysm CT findings, 493, 493f epidemiology of, 492, 492f risk factors for, 492 screening for, 55-56 treatment of, 493 Abdominal compartment syndrome, 715 Abdominal examination, 27, 851 in cardiovascular disease, 254 in heart failure, 303 in liver disease, 978 Abdominal imaging in acute pancreatitis, 962 in gallstones, 1040, 1040f in liver disease, 980 Abdominal injury, 714, 714f Abdominal pain, 854-859 acute, 854-858 approach to, 859f clinical manifestations of, 855t-858t physical findings, 854-858 approach to, 852t, 979t chronic, 859 approach to, 860f clinical manifestations of, 855t-857t in chronic pancreatitis, 965 differential diagnosis of, 857f in HIV infection, 2303t in hospital, 858 location of, 854, 857f pathobiology of, 854 Abdominal tuberculosis, 2034-2035 Abetalipoproteinemia, 929 Abiraterone acetate (Zytiga), 1211t-1216t, 1369 Ablation techniques, 295 Abnormal automaticity, 342 Abnormal movements, 2469 Abnormal vision, chronic, 2558 Abortion HgbA1C and, 1620, 1620f infection associated with, 1926 septic, 2252t-2253t Abrasion, corneal, 2563 Abraxane, 1211t-1216t Abscesses Bezold’s, 2489 brain, 2495-2497 brain stem, 2496, 2497f cold, 2035 in Crohn’s disease, 942 deep neck, 2603 eyelid, 2564, 2564f liver amebic, 1011t, 1013, 2140, 2141t bacterial, 1011t pyogenic, 1011-1012, 1011f

Abscesses (Continued) perianal, 969 peritonsillar, 2602t, 2603, 2603f spinal epidural, 2497-2498, 2498f subdural, 2498, 2498f visceral, 790 Absence seizures atypical, 2402 childhood, 2404-2405 clinical manifestations of, 2401t, 2402 drugs for, 2407t EEG findings, 2342, 2343f Absolute measures, 33 Absolute refractoriness, 341 Absolute risk reduction (ARR), 33 Abstainers, 149 Abstinence, 1880 Abuse. See also Substance abuse alcohol use disorders, 149-156 drugs of, 156-162 intimate partner violence, 1629-1633 physical, 1629-1633, 2339 Acamprosate, 155t, 156, 1021 Acanthamoeba, 2142, 2142t Acanthamoeba keratitis, 2566 Acanthocytes, 1059t Acanthophis (death adders), 720 Acanthosis nigricans, 2700, 2701f, 2702t Acarbose, 1537 Access to health care, 15 access to AIDS therapy, 2272 access to dialysis, 842 disparities in, 15-16 organizational barriers and interventions, 16 Accessory pathway tachycardia, 360-361, 364-365 Accidental infection, 2217 Accidental trauma, ocular, 2563 Accidents radiologic, 82-83, 83t, 85 Swiss cheese model of, 45 Accutane (isotretinoin), 2658 Acebutolol, 388t, 430t Acetaminophen dose-response effects, 1007, 1008t for headache, 2357b, 2358, 2360 metabolism of, 1007-1009, 1007.e1f for spinal stenosis, 2377 toxicity, 699t-703t, 703, 706t-710t, 1008-1009 for venomous fish and stingray injuries, 721b Acetazolamide for altitude sickness, 596-597, 597t, 1884-1885 for calcium channelopathies, 2544 for hyperkalemia, 761 for hyperkalemic periodic paralysis, 762 for hypokalemic periodic paralysis, 762 for intracranial hypertension, 2362 for myoclonus, 2466-2467 for perimenstrual seizures, 2408 for seizures, 2407t for sodium channelopathies, 2544 Acetazolamide-responsive myotonia, 2542t Acetest, 1540 Acetowhitening, 2221 Acetyl sulfisoxazole (Gantrisin), 1088t Acetylcholine, 549 Acetylcholine esterase deficiency, 2548t Acetylcholine receptor antibodies, 2550 Acetylcholine receptor deficiency, 2548t, 2549, 2549f

Acetylcholine receptor kinetic abnormalities, 2548t N-Acetylcysteine, 706t-710t, 1009, 1021 Acetylsalicylic acid. See Aspirin N-Acetyltransferase, 189t Achalasia, 903-904 clinical manifestations of, 903 diagnosis of, 903 epidemiology of, 903 pathobiology of, 903 prevalence of, 903 prognosis for, 904 radiographic findings, 903, 903f treatment of, 903b-904b Achilles reflex, 2372t Achilles tendinitis, 1754 Achilles tendon rupture, 1754 Achlorhydria, 1335t, 1337 Achondroplasia, 1672 Acid excretion absent secretion, 1107 net (NAE), 766 Acid maltase deficiency, 2545 Acid peptic disease, 908-918 acute, 914 clinical manifestations of, 912, 912t, 914 complications of, 915-917 definitions, 908 diagnosis of, 912-914 differential diagnosis of, 914 epidemiology of, 909 H. pylori–negative, non-NSAID, 910 idiopathic, 910 pathobiology of, 909-912 physical findings, 912 prevention of, 915-917 prognosis for, 917 treatment of, 914b-915b Acid-base balance, 763 in hypovolemia, 745 renal mechanisms, 763, 763.e1f Acid-base disorders, 762-774 clinical manifestations of, 764-766 compensatory changes in, 765-766, 765t definition of, 762 diagnosis of, 764-766 epidemiology of, 762 identification of, 764, 765t pathobiology of, 762-764 Acidemia, 762, 765f Acidity, 652 Acidosis definition of, 762 distal renal tubular, 770, 770t, 824 hyperchloremic, 762, 762t, 769-771 hyperkalemic renal tubular, 770t hypochlorhydric, 769 lactic, 767 in HIV infection, 2303t mitochondrial encephalomyopathy with lactic acidosis and strokelike episodes (MELAS), 1652t, 1660-1661, 2546 life-threatening, 661 metabolic, 766-771 anion gap, 766-769 of chronic kidney disease, 840 proximal renal tubular, 823t-824t respiratory, 773 in shock, 675 uremic, 767 Acids carbonic, 763 endogenous, 767-769

Acids (Continued) nonvolatile or fixed, 763 production and excretion by kidney, 763 urinary secretion of, 764 volatile, 763 Acinetobacter baumannii infections, 1863t, 1864, 1968 Acinetobacter baumannii-calcoaceticus complex, 1968 Acinetobacter meningitis, 1969 Acinetobacter species, 1968-1970 extremely drug-resistant, 1968, 1970 identification of, 1968.e3t members, 1968.e3t methods of typing, 1968.e3t multidrug-resistant, 1970 Acinetobacter species infections clinical manifestations of, 1968-1969 epidemiology of, 1968 prevention of, 1970 prognosis for, 1970 treatment of, 1969b-1970b Acitretin, 2658, 2666t, 2667 Aclidinium, 560t ACLS. See Advanced cardiac life support Acne preferred medications for women, 1602t pyogenic arthritis with pyoderma gangrenosum and, 1740t, 1743 Acne keloidalis, 2700, 2701f Acne vulgaris, 2680-2681 definition of, 2680 pathobiology of, 2680 regional involvement, 2701f treatment of, 2680b Acoustic neuroma, 1291, 1292f, 2595 Acquired idiopathic aplastic anemia, 1118t, 1121 Acquired immunodeficiency syndrome (AIDS), 1837, 2272-2278 access to therapy for, 2272 AIDS-defining malignancy, 2322-2323, 2323f, 2323t antiretroviral therapy for, 2287-2292, 2294 CMV infection in, 2230 definition of, 2323 demographic impact, 2272 diagnosis of, 2276-2278 diarrhea in, 926 distinguishing characteristics of, 2252t-2253t economic impact, 2272 gastrointestinal manifestations of, 2302-2305 global response, 2272 global statistics, 2272, 2273t historical perspective on, 2272.e1 Hodgkin’s lymphoma in, 1273 immune reconstitution inflammatory syndrome in, 2332-2335 infectious complications of, 2292-2295 clinical manifestations of, 2293-2294 diagnosis of, 2294 empirical management of, 2294 pathobiology of, 2293 treatment of, 2294b likelihood for development of, 2293, 2293f lymphoma in, 1267 malabsorption in, 931-932 metabolic disorders in, 2295-2302 opportunistic infection in, 2291, 2297t-2301t prophylaxis in, 2068 pulmonary manifestations of, 2305-2318 Volume I  pp 1-1382  •  Volume II  pp 1383-2722

I2

Index

Acquired immunodeficiency syndrome (AIDS) (Continued) regional epidemics, 2272-2278, 2272.e1 social impact, 2272 Acquired neuromyotonia, 2548t, 2552 Acrivastine, 2660t Acrocyanosis, 511 Acrodermatitis chronic atrophicans, 2022, 2022f, 2024 Acrodermatitis enteropathica, 1450t-1452t Acrokeratosis neoplastica, 2670 Acromegaly, 1486 arthritic manifestations of, 1825 clinical features of, 1486, 1486f diagnosis of, 1486 medical therapy for, 1486 Acromion process, 1750f Actemra (tocilizumab), 1762 ACTHomas, 1335t, 1339 Actigraphy, 2416-2417, 2416.e1f Actin assembly, 1145-1146, 1145f Actinic dermatitis, chronic, 2664-2665, 2664f Actinic keratoses, 2687-2688, 2687f Actinic porokreatosis, disseminated superficial, 2651f Actinomyces, 2060 Actinomycetoma, 2062. See also Mycetoma Actinomycosis, 2060-2062 abdominal, 2061 antimicrobial susceptibility patterns, 1934t of central nervous system, 2062 cervicofacial, 2061, 2061f clinical manifestations of, 2061-2062 definition of, 2060 diagnosis of, 2062 epidemiology of, 2060 oral ulcers, 2580t pathophysiology of, 2060 pelvic, 2061-2062 prognosis for, 2062 pulmonary, 2061, 2061f thoracic, 2061, 2061f treatment of, 1933-1934, 2062b Actinomycotic granulomas, 2062 Action dystonia, 2463-2464 Action potential cardiac, 263, 263f, 339-341, 340f, 368.e1f premature, 368.e1f Action potential duration adaption, 341 Activated charcoal, 699t-702t, 705 Activated partial thromboplastin time (aPTT), 515, 1155-1156 Activated protein C, recombinant human (rhAPC), 690, 1183 Activated protein C resistance (factor V Leiden), 1186, 1186t Activia, 893t Activities of daily living (ADLs), 102 Activity energy expenditure, 1433 Acupuncture, 182-183, 182t Acute abdominal emergencies, 2252t-2253t Acute abdominal pain, 854-858 approach to, 859f clinical manifestations of, 854-858, 855t-858t diagnosis of, 858 history in, 854-858 imaging, 858 location of, 854, 857f physical findings, 858, 858t in special populations, 858 treatment of, 859b Acute acalculous cholecystitis, 1041 Acute adult T-cell leukemia/lymphoma, 2238 Acute angle-closure glaucoma, 2566, 2567f Acute aortic dissection, 395t Acute aortic regurgitation, 471 Acute Aspergillus sinusitis, 2083 Acute asymmetrical neuropathy clinical manifestations of, 2535 diagnosis of, 2535 pathobiology of, 2535 treatment of, 2536 Acute autonomic syndromes, 2519 Acute bacterial prostatitis causative organisms, 830 classification of, 831t clinical manifestations of, 831 prognosis for, 833 treatment of, 831 Acute blood loss, 1063

Acute bronchitis and tracheitis, 608-609 clinical manifestations of, 608-609 definition of, 608 diagnosis of, 609 epidemiology of, 608 pathobiology of, 608, 608f prognosis for, 609 treatment of, 609b Acute calculous cholecystitis clinical manifestations of, 1039 complications of, 1041 pathobiology of, 1038-1039 treatment of, 1041 Acute Chagas disease, 2118 Acute chest syndrome, 1100, 1103, 1103t Acute cold autoimmune hemolytic anemia, 1076 Acute conjunctivitis, 2559t Acute coronary syndrome, 432-441, 433f clinical manifestations of, 434 complications of, 440 definition of, 432 diagnosis of, 434-436, 434t differential diagnosis of, 435-436 ECG findings, 434-435 epidemiology of, 432-433 integrated approach to, 440 noninvasive testing for, 435 non-ST segment elevation, 432, 441f pathobiology of, 433-434 physical findings, 434 plaque rupture in, 418 primary, 433 prognosis for, 440 risk assessment in, 435, 437t, 453.e1f serum biomarkers, 435 symptoms, 434 treatment of, 436b-440b antianginal therapy, 436-437 recommendations for, 395t selection of initial strategy for, 440t strategies for, 438t-439t triage for, 435, 436f Acute cough, 526-527, 526t Acute decompensated heart failure factors that may precipitate, 299, 299t with pulmonary edema, 318-319 Acute diarrhea, 919-922 clinical manifestations of, 919-921 definition of, 918 diagnosis of, 921, 921f prevention of, 922 treatment of, 921b-922b Acute disseminated encephalomyelitis, 2479 Acute dystonic reactions, 2467-2468 Acute erythema, 2655 Acute eye abnormalities, 2558-2560 Acute fatty liver of pregnancy, 1622 diagnosis of, 1622 prognosis for, 1622 Swansea diagnostic criteria for, 1622, 1622t Acute febrile neutrophilic dermatosis, 1859, 2682 Acute febrile syndrome, 594-595 clinical manifestations of, 594-595 diagnosis of, 594-595 occupational causes of, 594t treatment of, 595b Acute flaccid paralysis, 2241, 2241t clinical manifestations of, 2241 diagnosis of, 2241 prognosis for, 2241b treatment of, 2241b Acute gallstone pancreatitis, 1043-1044, 1043f Acute generalized exanthematous pustulosis, 2681, 2681f, 2685t, 2686 Acute glaucoma, 2559t Acute glomerulonephritis, 788-790 Acute gonococcal urethritis, 1941, 1941f Acute gout, 1814-1815 Acute gouty arthritis, 1813, 1813f Acute hemolytic anemia, 1088, 1088b Acute hemolytic transfusion reactions, 1196 Acute hemorrhagic conjunctivitis, 2241t, 2243 clinical manifestations of, 2243 prognosis for, 2243 treatment of, 2243b Acute hepatitis, 993. See also Acute viral hepatitis

Acute hepatitis A, 995, 995f Acute hepatitis B, 995-997 clinical manifestations of, 996 diagnosis of, 996 epidemiology of, 995 pathobiology of, 995-996 prevention of, 996-997 prognosis for, 997 routes of transmission, 995 serologic profiles, 996, 996t treatment of, 996b Acute hepatitis C, 997-998 clinical manifestations of, 997 diagnosis of, 997-998 epidemiology of, 997 pathobiology of, 997 prevalence of, 997 prevention of, 998 prognosis for, 998 treatment of, 998b Acute hepatitis D or delta, 998-999 clinical manifestations of, 999 diagnosis of, 999 epidemiology of, 998-999 pathobiology of, 999 prevention of, 999 prognosis for, 999 treatment of, 999b Acute hepatitis E, 999 clinical manifestations of, 999 diagnosis of, 999 epidemiology of, 999 pathobiology of, 999 prevention of, 999 procedure, 999 treatment of, 999b Acute human immunodeficiency virus infection, 2291. See also Human immunodeficiency virus infection Acute hypersensitivity pneumonitis, 589t Acute hyponatremia, 752 Acute idiopathic interstitial pneumonia, 575t, 579-580, 583-584 Acute infectious diarrhea, 919t Acute infectious gastroenteritis, 2244 Acute intermittent porphyria, 1408t acute attacks, 1411 classification of, 1408 clinical manifestations of, 1411-1412 diagnosis of, 1413 epidemiology of, 1408 etiology of, 1409 skin disease in, 1413 treatment of, 1414 Acute interstitial nephritis, 779, 795-796 on biopsy, 778f, 796 clinical manifestations of, 795, 795t diagnosis of, 795-796 differential diagnosis of, 796 indications for biopsy, 796t, 783.e2 medications associated with, 779, 779t prognosis for, 796 treatment of, 796b Acute interstitial pancreatitis, 960 Acute interstitial pneumonia, 571t, 572-573, 583 clinical manifestations of, 573, 573f definition of, 572-573 diagnosis of, 573 pathobiology of, 573 treatment of, 573b Acute invasive aspergillosis, 2083t Acute iritis, 2559t Acute ischemic stroke arterial hypertension in, 2441 diagnosis of, 2439, 2440f differential diagnosis of, 2440 evaluation of, 2440t immediate diagnostic studies, 2439t treatment of, 2440-2441, 2441t-2442t, 2440.e1f endovascular, 2441 recommendations for, 395t time goals for, 2440t Acute kidney injury, 732-733, 778-783 associated with radiocontrast agents, 782-783 categories of, 778, 778f clinical manifestations of, 781 definition of, 778, 779t diagnosis of, 733, 781-782 epidemiology of, 778-779

Acute kidney injury (Continued) etiology of, 733 FENa in, 781, 781t imaging, 733 intrarenal, 780-782 intrinsic, 779, 779f ischemic, 780, 780t laboratory testing in, 733 pathobiology of, 779-781 phases of, 780, 780f postrenal, 779, 781-782 prerenal, 779f, 780 prevention of, 782-783 prognosis for, 783 rhabdomyolysis-induced, 725, 726t treatment of, 782b urinalysis findings in, 781, 781t vicious cycle with CKD, 782, 782f zoonotic, 2057, 2059t Acute leukemias, 1239-1246, 1241f cell of origin, 1240 chemotherapy for, 1242-1243 classification of, 1240, 1240t clinical manifestations of, 1242 clonality of, 1240 complications of, 1245-1246 cytogenetics of, 1241-1242 determinants, 1239-1240 diagnosis of, 1242 differential diagnosis of, 1242 epidemiology of, 1239-1240 genetic predisposition to, 1239 immunophenotyping, 1240-1241 molecular biology of, 1241-1242 morphology of, 1240, 1241f pathobiology of, 1240-1242 treatment of, 1242b-1245b preparation for, 1243 regimens for, 1243t Acute limb ischemia clinical categories of, 499t clinical manifestations of, 499 diagnosis of, 500 pathobiology of, 498-499 treatment of, 500-501, 502f Acute liver failure liver transplantation for, 1033, 1034t treatment of, 1033b Acute lower gastrointestinal bleeding, 875-876, 875f Acute lung injury acute respiratory failure in, 661-662 definition of, 661-662 transfusion-related, 602-603, 1197 treatment of, 662b Acute lymphadenitis, 2168 Acute lymphoblastic leukemia, 1239 B-cell, mature, 1243-1244 blood smear features of, 1058, 1058f Burkitt-like, 1243-1244 classification of, 1240t clinical manifestations of, 1242 cytogenetic abnormalities in, 1242 diagnosis of, 1242 immunophenotyping, 1241 incidence of, 1239 molecularly guided therapeutics for, 202.e1t morphology of, 1240, 1241f Ph-positive, 1243-1244, 1246 postremission chemotherapy for, 1243 prognosis after initial chemotherapy, 1244 relapsed, 1244 remission of complete, 1243 induction of, 1243 targeted therapy for, 202, 202t treatment of, 1243 Acute lymphocytic leukemia, 1201 Acute lymphoid leukemia, 1243t Acute malnutrition in children, 1436.e1f clinical manifestations of, 1434 definition of, 1434 global rates, 1434 inpatient management protocol for, 1436f treatment of, 1436, 1436.e1f Acute mental status change, 117-121 Acute mesenteric ischemia, 952f Acute motor axonal neuropathy, 2524t

Index Acute mountain sickness, 596 clinical manifestations of, 596 prophylaxis of, 596-597 treatment of, 597 Acute myeloid leukemia, 1239 classification of, 1240t cytogenetic abnormalities in, 1241 definition of, 1233 diagnosis of, 1242 hematopoietic stem cell transplantation for, 1201 incidence of, 1239 morphology of, 1240, 1241f in older adults, 1245 postremission therapy for, 1244 prognosis after initial chemotherapy, 1244 recurrent, 1244-1245 remission induction, 1244 risk categories, 1244 secondary, 1240-1242 survival in, 1244, 1244f treatment of, 1243t, 1244-1245 Acute myocardial infarction causes of, 442, 442t clinical manifestations of, 369 coronary angioplasty for, 458, 459f diagnosis of, 442, 444 early acute phase, 443 ECG findings, 443-444, 443f-444f echocardiographic findings, 280t-281t evolved acute phase, 443 hemodynamic subsets, 448t physical findings, 443 in presence of bundle branch block, 444 risk assessment, 248, 250f ST segment elevation (STEMI), 441-456 anticoagulant therapy for, 448f antiplatelet therapy for, 448f, 449-450 assessment of, 445-446 CABG for, 460 clinical manifestations of, 442-443 conditions that can mimic, 444t diagnosis of, 443-445 differential diagnosis of, 444 early hospital phase, 448-449 echocardiography in, 445 electrocardiographic monitoring, 453-454 emergency department care for, 445-446 epidemiology of, 441-442 fibrinolytic therapy for, 447 history in, 442-443 hospital care for, 445-446 imaging in, 445 initial medical management of, 448 laboratory findings, 445 later hospital phase, 449 management of, 445-446 management of complications of, 451-453 mechanical complications of, 453 medical therapy for, 451 pathobiology of, 442 patient education, 455-456 physical findings, 443 post-hospital care, 454-456 prehospital care for, 445 prognosis for, 456 rehabilitation after, 455-456 reperfusion therapy for, 446-448, 446f sample admission orders for, 449t secondary prevention of, 454-455 selection of ICDs for, 454f thromboembolic complications of, 453 treatment of, 445b-456b Acute myocarditis, 290f, 327 Acute myopericarditis, 328f Acute neck pain, 2372 Acute necrotizing pancreatitis, 960 Acute nephritic syndrome, 734 Acute opportunistic infection, 2291 Acute oral ulcers, 2579-2581 Acute osteomyelitis, 1806 Acute pain, 133 Acute pancreatitis, 959-960 causes of, 960t clinical manifestations of, 961 complications of, 961t, 962-963 definition of, 959-960

Acute pancreatitis (Continued) diagnosis of, 961-962 epidemiology of, 960 etiology of, 962 general supportive care for, 962 imaging findings, 961-962, 962f laboratory findings, 961 pathobiology of, 960-961 prevention of, 963 prognosis for, 963 severe, 964 severity of, 962 treatment of, 962b-963b Acute pandysautonomia, 2518-2519 Acute pericarditis, 483-486 clinical manifestations of, 483 diagnosis of, 483-485 diagnostic tests for, 485t differential diagnosis of, 426 ECG findings, 484f epidemiology of, 483 pathobiology of, 483 prognosis for, 486 treatment of, 486b Acute peripheral vestibulopathy, 2598 Acute poisoning, 696-711, 704f Acute promyelocytic leukemia, 1243t, 1245 Acute pulmonary histoplasmosis, 2070 Acute Q fever, 2055t Acute radiation syndromes, 83-84 cerebrovascular syndrome, 84 gastrointestinal syndrome, 84 heamtopoietic syndrome, 84 latency period, 84 manifest illness, 84 prodromal stage, 84 Acute renal failure ischemic, 780, 780t in pregnancy, 1614t zoonotic, 2057, 2059t Acute respiratory distress syndrome, 595, 661-662 clinical manifestations of, 662 definition of, 661-662 diagnosis of, 661-662 disorders associated with, 662t epidemiology of, 662 initial management of, 663f mechanical ventilation for, 668-669 adjunctive approaches, 669 strategy for, 670f pathobiology of, 662 pathology of, 662 pathophysiology of, 662 prognosis for, 662 treatment of, 662b ventilatory strategy for, 670f Acute respiratory failure, 655-659 abbreviations, 656t chest radiograhy findings, 659, 659f with chronic lung disease, 657t classification of, 657t clinical evaluation of, 658-659 clinical manifestations of, 657, 658t clinical scenarios, 658 in COPD, 661 definition of, 655 diagnosis of, 657-659 initial approach to, 657-658 objectives, 657 evaluation of, 659 hypercapnic-hypoxemic, 657t life-threatening aspects, 655 mechanical therapy for, 660 pathobiology of, 655-657 pathophysiology of, 656-657 supportie measures for, 660 syndromes, 661-664 treatment of, 659-660, 659b-660b without lung disease, 664 Acute retinal necrosis, AIDS-associated, 2297t-2301t Acute retroviral syndrome, 2318 Acute rheumatic fever, 1908-1909 Acute septic arthritis, 1806, 1806t Acute severe hypertension, 394-395 Acute spine pain, 2374-2375 Acute stress disorder, 2353 Acute surveillance testing, 1868

Acute symptomatic (provoked) seizures, 2399, 2404 causes of, 2399t incidence of, 2399 risk factors for, 2399 treatment of, 2407 Acute telogen effluvium, 2703, 2703b Acute (suppurative) thyroiditis, 1509 Acute tubular necrosis, 733, 780 Acute tubulointerstitial nephritis causes of, 794, 794t definition of, 793 Acute urinary tract obstruction caused by calculi, 803 treatment of, 802-803 Acute venous thrombosis, 1187b-1188b Acute viral encephalitis, 2500-2504 clinical manifestations of, 2501 diagnosis of, 2501-2502 differential diagnosis of, 2501-2502 epidemiology of, 2500-2501 pathobiology of, 2501 prognosis for, 2504 treatment of, 2504b Acute viral hepatitis, 993-1000 clinical manifestations of, 993-994, 994t diagnosis of, 994 evolution to chronic viral hepatitis, 993t general features of, 993-994 laboratory testing for, 994t pathobiology of, 993 prognosis for, 994 treatment of, 994b types of, 999-1000 typical course of, 993f viruses responsible for, 993, 993t Acute-subacute (juvenile) paracoccidioidomycosis, 2075 Acyclovir, 2181-2182 for AIDS-associated opportunistic infections, 2297t-2301t for encephalitis, 2242 for herpes, 2181t, 2226, 2297t-2301t for herpes simplex encephalitis, 2502, 2504 for herpes zoster, 2680 for herpes zoster ophthalmicus, 2565 mechanisms of excretion and thresholds for dose adjustment, 2182t for neonatal herpes, 2226-2227 for otitis externa, 2591 for post-herpetic neuralgia, 2536 for varicella, 2228, 2297t-2301t, 2679-2680 for viral meningitis, 2492 for VZV postexposure prophylaxis, 2229 for VZV vasculopathy, 2229 Acyclovir-resistant herpes simplex virus, 2297t-2301t Acyclovir-resistant varicella zoster virus, 2229 Acyl-CoA oxidase deficiency, 1387t Adalimumab (Humira), 169 for Behçet’s syndrome, 1799 for inflammatory bowel disease, 939t, 940 for psoriasis, 2666t for rheumatoid arthritis, 1762 for ulcerative colitis, 942 ADAM (a disintegrin and metalloproteinase) family, 234 Adamantanes, 2184 ADAMTS13 (a disintegrin-like and metalloprotease with thrombospondin type 1 motif, member 13), 1165, 1168 Adapalene (Differin), 2658 Adaptive immune response, 226-227, 1007 definition of, 226 effector mechanisms, 226-227 Adaptive immune system, 220-226 cellular elements of, 222-226 description of, 216 principles of activation, 220-222 transition to, 219 Adaptive immunity, 230, 686 Addiction biology of, 143-145 definition of, 143 drug-induced neural plasticity relevant to, 145 neural substrate of, 144-145 risk factors for, 143-144

I3

Addictive drugs, 143, 144t, 145 Addison’s disease, 1473 Addison’s syndrome, 1468 Adefovir, 2179 adverse effects of, 2178t for hepatitis B, 1002, 2177t mechanisms of excretion and thresholds for dose adjustment, 2178t Adenocarcinoma of colon and rectum, 1327-1331 esophageal, 1315-1316 gastric, 1316-1319 invasive mucinous, 571t, 574-575, 575b, 574.e1f large bowel clinical manifestations of, 1328, 1328f radiation therapy for, 1330 sites of development of, 1327-1328, 1327f lepidic predominant nonmucinous, 571t, 574-575 in lung, 1304 minimally invasive, 1304 oral ulcers, 2580t pancreatic ductal, 1332 poorly differentiated, 1379t rectal, 1330-1331 small bowel, 1320 of unknown primary origin management of, 1379-1380 recommended evaluation after light microscopic diagnosis, 1379t recommended treatment, 1380t Adenocarcinoma in situ, 1304 Adenolymphoma, 2584 Adenoma ampullary, 1047, 1047.e1f colon saline lift polypectomy of, 1327.e1 snare polypectomy of, 1326, 1326.e1 gastric, 1320 hepatic, 980-981 hepatocellular, 1608 jejunal, 1326-1327, 1322.e1 pituitary, 1476 prevalence of, 1483, 1484t TSH-secreting, 1506-1507 subtypes, 1326 toxic, 1506 tubular, 1326 tubulovillous, 1326 villous, 933, 1326, 1326f Adenomatoid malformation, congenital cystic, 570 Adenomatous hyperplasia, atypical, 1304 Adenomatous polyps, 1325-1326 clinical manifestations of, 1326 definition of, 1325 diagnosis of, 1326 epidemiology of, 1325 pathobiology of, 1325-1326 Adenosine, 363t-364t Adenosine deaminase deficiency, 1678t Adenosine diphosphate (P2Y12) receptor antagonists, 449-450 Adenosine testing, 348, 363 Adenoviral conjunctivitis, 2564-2565 Adenovirus diseases, 2197-2199 clinical manifestations of, 2198 definition of, 2197 diagnosis of, 2199 epidemiology of, 2197-2198 in immunocompromised patients, 2198 pathobiology of, 2198 prevention of, 2199 prognosis for, 2199 treatment of, 2199b Adenoviruses, 2185t, 2197 enteric, 2244 serotypes, 2197, 2197t, 2198.e1f structure of, 2197, 2197.e3f subgroups, 2197, 2197t Adherence, drug, 315 Adherens junctions, 2633 Adhesions epidermal, 2633, 2632.e1f peritoneal, 949, 950f Adhesive capsulitis, 1713t, 1752 Adie’s pupil, 2560, 2576, 2576f Adipocytes, 2635

Volume I  pp 1-1382  •  Volume II  pp 1383-2722

I4

Index

Adipose tissue, 1730t ADLs. See Activities of daily living Admission orders, ICU, 650-651, 651t Adnexal tumors, 2688 Adolescent care health visit, 63 Adolescent medicine, 61-65 Adolescents approach to, 62-63 body fat in, 61 chronic illness and transition, 64-65 eating disorders in, 64 epilepsy syndromes in, 2406, 2406t leading causes of death, 61-62, 62t normal physical growth and development, 61 obesity in, 64 physical examination of, 63 pregnancy in, 63-64 preventive health care for, 63 psychosocial developmental phases, 61 reproductive health care for, 63-64 substance abuse among, 64 suicide rates, 61 transgendered, 1568 transition to adult health care system, 64-65 vital statistics, 61-62 Adrenal androgen deficiency, 1572 Adrenal axis, 1514, 1515f Adrenal cortex, 1514-1521 Adrenal glands function of, 1514 functional disorders of, 1514-1521 normal, 1514, 1514f in shock, 680 Adrenal hyperplasia, congenital, 1519, 1562-1564, 1584 Adrenal insufficiency, 1519-1520 acute diagnosis of, 1520 treatment of, 1520-1521 causes of, 1519-1520, 1519t chronic, 1521 clinical presentation of, 1520, 1520f diagnosis of, 1520 differential diagnosis of, 1520 preoperative evaluation of, 2616 primary causes of, 1519-1520, 1519t clinical presentation of, 1520, 1520f secondary, 1520 causes of, 1519t clinical manifestations of, 1520, 1520f treatment of, 1520b-1521b Adrenal medulla, 1521-1523 Adrenal steroidogenesis, 1561-1562, 1562f Adrenal tissue replacement, 1519 Adrenalectomy, 1517 Adrenarche, 61, 1570 β-Adrenergic agents, 551 Adrenergic failure, 2521 β-Adrenergic receptor agonists, long-acting, 553 α-Adrenergic receptor blockers (α-blockers) for benign prostatic hyperplasia, 830, 830t contraindications to, 389t for hypertension, 388t, 391 side effects of, 389t, 391 therapeutic principles, 391 β-Adrenergic receptor blockers (β-blockers), 312-313 for acute coronary syndrome, 437, 438t-439t for acute STEMI, 450, 455 for anxiety, 2352t at hospital discharge after MI, 454t clinical benefits, 313 clinical use of, 429 contraindications to, 389t doses, 363t-364t for headache prevention, 2358t for heart failure, 309, 312-313, 306.e1t-306.e2t for hypertension, 388t, 391, 394t, 429 long-term, 455 mechanism of action, 312, 391 practical guidance on use of, 311t practical use, 313 side effects of, 363t-364t, 389t, 391 therapeutic principles, 391 for thyrotoxicosis, 1507 toxicity, 699t-702t, 706t-710t

Adrenergic toxidromes, 696t Adrenocorticotropic hormone, 1488-1492 ACTHomas, 1335t, 1339 ectopic secretion syndrome, 1491, 1516 effects of hypothalamic disease on, 1479 inappropriate secretion of, 1220 petrosal sinus sampling, 1487t vasopressin and, 1496 Adrenocorticotropic hormone deficiency, 1479, 1489-1490 Adrenocorticotropic hormone stimulation test, 1483t, 1521 Adrenoleukodystrophy, 1519, 1519t, 2479 treatment of, 1388t, 2479b X-linked, 1387t Adrenomyeloneuropathy, 2479, 2479b Adriamycin (doxorubicin), 1211t-1216t, 1271-1272 Adrucil (5-fluorouracil), 1211t-1216t Adult (chronic) paracoccidioidomycosis, 2075 Adult T-cell leukemia/lymphoma, 1255, 1266, 1266b, 2236-2238, 2237t acute, 2238 chronic, 2238 smoldering, 2238 Adulterants, 158 Advance directives, 6, 104 Advanced activities of daily living (AADLs), 103 Advanced cardiac life support (ACLS), 30 after acute poisoning, 705 after cardiac arrest, 353-354, 354f-355f Advanced glycation end products, 1543 Advanced Trauma Life Support, 30 Adverse drug reactions, 131-132 allergic, 1703t anaphylactic, 1705, 1705t definition of, 1703 diagnosis of, 132 drugs at risk for, 131t in elderly, 116 epidemiology of, 131 etiology of, 131-132 organ-specific reactions, 1704t predictable toxic responses, 131-132 severe cutaneous adverse reactions (SCARs), 1704 type A, 1703 type B, 1703 unpredictable toxic responses, 132 Adverse Event Reporting System (AERS) (FDA), 132 Adverse events, 45 Advisory Committee on Immunization Practices, 66-67 recommendations for DtaP vaccination, 1929 recommendations for HPV vaccines, 2222t recommendations for rotavirus vaccination, 2246 recommendations for tetanus immunization, 1929 Advocacy, 1631 Aerobic activity, 58-59 AeroBid/AeroBid-M (flunisolide), 553t Aeromonas hydrophila infection, 1869t Afatinib (Gilotrif), 171, 1310t Affect, 104 Affinitor (everolimus), 1211t-1216t Affordable Care Act, 9, 17 Afibrinogenemia, 1175t, 1179 clinical manifestations of, 1179 congenital, 1179 treatment of, 1179b Aflibercept (Zaltrap), 170, 1211t-1216t, 1331 Africa, sub-Saharan HIV/AIDS epidemic, 2272-2278, 2272.e1 life expectancy, 100 meningitis belt, 75, 1882, 2480 African Americans, 15 access to health care, 15 causes of death in, 61 chronic kidney disease in, 833-834 cystic fibrosis in, 562 gallstones in, 1038 gonorrhea in, 1940 heart failure in, 298 HIV/AIDS in, 2276 hypertension in, 382, 393 neutropenia in, 1133

African Americans (Continued) normal physical growth and development, 61 normal pulmonary function values, 540 obesity in, 1458 osteoporotic fractures in, 1637-1638 population, 15 risks for atherosclerotic cardiovascular disease, 260f-261f skin disorders seen in, 2700, 2700t suicide rates, 61 tularemia in, 1982 type 2 diabetes mellitus in, 1532 African Bedouins, 1133 African eye worm, 2170, 2170f African sleeping sickness, 2113-2116 clinical manifestations of, 2114 definition of, 2113 diagnosis of, 2114-2115 epidemiology of, 2113-2114 fatality rate, 2060t pathobiology of, 2114 prevention of, 2115-2116 treatment of, 2115b African snakes, 719 African tick bite fever, 2048t, 2050, 2252t-2253t African trypanosomiasis. See Human African trypanosomiasis Afterdepolarizations, 342 delayed, 342, 368.e1f early, 342, 368.e1f Afterload, 266 Agammaglobulinemia autosomal forms, 1680t genetic forms, 1679-1680 X-linked, 1677, 1679, 1680t Agency for Healthcare Research and Quality (AHRQ), 54 Healthcare Cost and Utilization Project, 43.e1t Medical Expenditure Panel Survey (MEPS), 43.e1t National Healthcare Quality and Disparities Reports, 43.e1t National Quality Measures Clearinghouse, 43 Agendia Inc., 203 Age-related changes in antimicrobial therapy effects, 1845 and cancer, 1222 and cardiovascular disease, 258 cellular, 106 in central nervous system, 114, 114t in connective tissue, 1733 molecular, 106 neuropsychiatric, 114, 115t and osteoarthritis, 1747 pharmacodynamic, 130 pharmacokinetic, 129-130 physiologic dysregulation, 106-107, 107.e1f and pulse pressure, 382, 382f and rheumatic disease, 1717 in skin, 2693-2694 Age-related eye disorders, 2566-2568 Age-related macular degeneration, 1740t, 2567-2568 dry, 2568, 2568f genetics of, 194 wet, 2568, 2568f Aggrastat (tirofiban), 180 Aging, 100-102, 106-110 cardiovascular response to, 267 clinical manifestations of, 107, 107f demographics of, 100, 101t effects of, 107-110 epidemiology of, 100-102, 106 healthy, 102 hypogonadism in, 1571, 1571f neuropsychiatric aspects of, 114-117 pathobiology of, 106-107 pharmacology of, 108 photoaging, 2693-2694 physiology in, 1570-1572 symptoms of, 109-110 worldwide, 100 Aging-related bone loss, 1639-1640 Agkistrodon (cottonmouth snakes), 719 Agnosia, object, 2385 Agoraphobia, 2351t Agranulocytosis, 1115, 2627-2628

Agricultural chemical poisoning, 2252t-2253t AIDS. See Acquired immunodeficiency syndrome (AIDS) AIDS Clinical Trials Group (ACTG), 2325 AIDS enteropathy, 931-932 AIDS-related lymphoma, 2322-2323, 2323t clinical manifestations of, 2323 diagnosis of, 2324 epidemiology of, 2323 pathobiology of, 2323 supportive care for, 2324 treatment of, 2324b AIMS65 scoring system, 916t Air embolism, venous, 626 Air trapping, 542 Airborne allergens, 1688 AIRE (autoimmune regulator) gene, 1660 Airway. See also Lung(s); Respiratory system asthmatic See also Asthma preoperative assessment of, 2617 radio frequency ablation of, 554 responsiveness of, 556 Airway obstruction. See also Chronic obstructive pulmonary disease in bronchiectasis, 568 central, 642-643, 643f causes of, 642, 643t clinical manifestations of, 642 diagnosis of, 642 prognosis for, 643 treatment of, 643b congenital pulmonary malformations, 570 diagnosis of, 528, 529f inflammation, 568 mechanical ventilation for, 669-670 reversible with heart failure, 316 treatment of, 316 upper airway abnormalities associated with obstructive sleep apnea, 639 ways to relieve or bypass, 641 Akathisia, 2468 Akinesia, 2455 ALA dehydratase deficiency porphyria, 1408t clinical manifestations of, 1412 epidemiology of, 1408 etiology of, 1410 Alanine aminotransferase, 992, 992f Alaska Natives, 15 access to health care, 15 population, 15 quality of health care for, 15-16 tularemia among, 1982 type 2 diabetes mellitus among, 1532 Albendazole, 2104 for cystic echinococcosis, 2152 for cysticercosis, 2151 for enteric protozoal infections, 2147t for giardiasis, 2137 for hookworm, 2161 for hydatid cysts, 1015 for nematodes, 2160t, 2165t, 2166 for roundworms, 2103 for tapeworms, 2104 for toxocariasis, 2164 for trichuriasis, 2162 Albinism, 1385t, 2698-2699 Albright’s hereditary osteodystrophy, 1661 Albucid (sulfacetamide), 1088t Albumin for ascites, 1030 fluid replacement with, 680t in hypovolemia, 745 plasma levels, 990 for shock, 680t for spontaneous bacterial peritonitis, 1030 Albumin-to-creatinine ratio, 1545 Albuminuria, 838 Albuterol for acute chest syndrome, 1103 for asthma, 554 for COPD, 560t for hyperkalemia, 761 for status asthmaticus, 554 Alclometasone dipropionate, 2658t Alcohol blood levels, 2510 properties of, 144t Alcohol dependence, 155t Alcohol history, 153-154 Alcohol intoxication, 151 signs of, 2510

Index Alcohol treatment programs, 155 Alcohol Use Disorder Identification Test (AUDIT), 153 Alcohol use disorders, 149-156. See also Substance abuse acute effects of, 151 in adolescents, 65t advice for patients, 154t at-risk drinking criteria for, 150t definition of, 149-150 treatment of, 154 binge drinking, 149-150 and cancer, 1224 and cardiovascular disease, 259 chronic effects of, 151-153 clinical manifestations of, 150-153 complications of, 151, 151t CRAFFT screening tool for, 65t criteria for, 150, 150t definition of, 149 diagnosis of, 153-154, 153t discussion with patients about, 153 epidemiology of, 150 future directions, 156 and gastritis, 912 and heart failure, 315 laboratory findings, 154 medical issues with, 152 moderate drinking, 149 neurologic disorders, 2510-2512, 2511f and pancreatitis, 960, 963 pathobiology of, 150 patterns of, 150t physical findings, 154 prevalence of, 150 prevention of, 154b-156b prevention of relapse, 155-156, 155t problem drinking behavioral interventions for, 57 prevalence of, 150 prognosis for, 156 screening for, 153 terminology for, 150t treatment of, 154, 156t in women, 1603 Alcohol withdrawal, 154-155, 155t Alcohol withdrawal syndrome, 151, 151f Alcoholic cardiomyopathy, 330 Alcoholic cirrhosis, 1034 Alcoholic diarrhea, 933 Alcoholic hepatitis clinical manifestations of, 1020 liver transplantation for, 1034 Alcoholic ketoacidosis, 768 clinical manifestations of, 768 diagnosis of, 768 epidemiology of, 768 pathobiology of, 768 prognosis for, 768 treatment of, 768b Alcoholic liver disease, 1019-1021 clinical manifestations of, 1020 definition of, 1019-1020 diagnosis of, 1020-1021 epidemiology of, 1020 pathobiology of, 1020 physical findings, 1020 prognosis for, 1021 risk factors for, 1020, 1020t treatment of, 1021b Alcoholic neuropathy, 2511 Alcoholic steatohepatitis, 1019-1023 Alcoholics Anonymous (AA), 154-155 Alcoholism, 149, 2339 Alcohol-related chronic pancreatitis, 928 Aldesleukin (human recombinant IL-2, Proleukin), 1211t-1216t Aldicarb, 699t-702t Aldosterone antagonists, 313 for acute STEMI, 451 for hypertension, 394t Aldosterone paradox, 756-757 Aldosteronism, primary diagnosis of, 385t, 1517, 1518f mineralocorticoid-induced hypertension due to, 386 treatment of, 1518, 1518f Alefacept (Amevive), 171

Alemtuzumab (Campath-IH) for chronic lymphocytic leukemia, 1256 for induction therapy, 844-845 for multiple sclerosis, 2477 for mycosis fungoides, 2670t in renal transplantation, 844t Alendronate for bone disease, 928 for complex regional pain syndrome, 2522 for osteoporosis, 1643t for Paget’s disease, 1667t for well-being, 2478 Alert fatigue, 49 Alexia, 2385-2386 Alfacalcidol (1α-hydroxycholecalciferol), 1659 Alfuzosin, 830t Alimta (pemetrexed), 1211t-1216t Aliskiren, 388t Alkalemia, 762 Alkali supplements, 824 Alkaline phosphatase, 989, 992-993, 992f Alkalosis definition of, 762 metabolic, 762, 762t, 771-773 postoperative, 2624 respiratory, 774 Alkaptonuria, 1385t, 1827 Alkeran (melphalan), 1211t-1216t Alkhumra hemorrhagic fever, 2249t clinical manifestations of, 2251t epidemiology of, 2250 pathobiology of, 2251t Alkhumra hemorrhagic fever virus, 2249t Alkylating agents, 168 for cancer, 1211t-1216t classification of, 163t for SLE, 1775-1776 Allegra (fexofenadine), 2660t Allelic affinity, 194 Allelic heterogeneity, 191, 194 Allergens, 1698-1699 airborne, 1688 aspirin (acetylsalicylic acid), 1699 food, 1699 insect sting, 1699 latex, 1699 non–IgE-dependent agonists, 1699-1700 NSAIDs, 1699 perennial, 1688t physical stimuli, 1700 seasonal, 1688, 1688t Allergic alveolitis, extrinsic, 2084-2085 Allergic angiitis, 586 Allergic Aspergillus sinusitis, 2084 Allergic bronchopulmonary aspergillosis, 2084 diagnosis of, 2085 radiologic features of, 2085, 2085f treatment of, 2086 Allergic conjunctivitis, 2564, 2564t Allergic contact dermatitis, 2639f, 2656f, 2663-2664 drug-induced, 2687 occupational, 80t Allergic disease, 1674-1676 definition of, 1674 diagnosis of, 1674-1676 epidemiology of, 1674 history in, 1674-1675 laboratory findings, 1675-1676 laboratory tests, 1676 pathobiology of, 1674 physical findings, 1675 symptoms and signs of, 1675t treatment of, 1675t Allergic fungal sinusitis, 1691, 1691t Allergic reactions, 227 to antimicrobial agents, 1845 drugs implicated in, 1703t transfusion reactions, 1196 Allergic rhinitis, 1687-1691 allergens that cause, 1688, 1688t clinical manifestations of, 1689 diagnosis of, 1689-1691 differential diagnosis of, 1689-1691, 1690t epidemiology of, 1687-1688 medical therapy for, 2588 pathobiology of, 1688-1689

Allergic rhinitis (Continued) perennial, 1688, 1688t seasonal, 1688, 1688t Allergy drug, 1703-1705 food, 934, 1675, 1698-1703 insect sting, 1698-1703 latex, 2618 oral, 1699 photoallergy, 2665, 2665t Allergy testing, 1676, 1676t Alloantibody inhibitors, 1176 Allodynia, 133-134, 2228, 2522 Allogeneic bone marrow transplantation, 1280 Allogeneic hematopoietic stem cell transplantation, 1199-1201 for atypical CML, 1252 for chronic myelogenous leukemia, 1248-1250, 1249t for chronic myelomonocytic leukemia, 1252 process, 1199-1200, 1200f Allogeneic stem cell transplantation, 1128 Allograft rejection. See Rejection Alloimmune thrombocytopenia, neonatal, 1167 AlloMap (XDx Expression Diagnostics, Inc.), 203 Allopurinol for gout, 1815 maximal daily dose, 1815 to prevent calcium stones, 815 for tumor lysis syndrome, 1243 Allorecognition direct, 238, 238f indirect, 238, 238f Almotriptan, 2358-2359, 2359t Alopecia, 2703 androgenetic, 2703t, 2704, 2706 areas, 2705f cicatricial, 2703, 2705-2706, 2707f congenital triangular, 2706 diagnosis of, 2703, 2703t diffuse, 2703-2704, 2703t frontal fibrosing, 2703t, 2705, 2705f patchy, 2703-2706, 2703t, 2704f patterned, 2704 scarring, 2705-2706 traction, 2703t, 2706, 2706f triangular, 2706 Alopecia areata, 2703-2706, 2703t, 2704f Alopecia totalis, 2704-2705 Alopecia universalis, 2704-2705 Alosetron (Lotronex) for diarrhea, 935 for irritable bowel syndrome, 893t, 894 for small bowel rapid transit dysmotility, 888 Alpha particles, 82, 83f α-Blockers. See α-Adrenergic receptor blockers Alphanate SD/HT (Grifols), 1170 Alphavirus infection distinguishing characteristics of, 2252t-2253t geographic spread, 2490 Alport’s syndrome, 791, 1167 Alprazolam for anxiety, 2352t for tremor, 2462 Alprostadil, 1579 Alteplase (recombinant tissue-type plasminogen activator, rt-PA), 180, 447t Altitude sickness, 1884-1885 recommended medications for, 597t treatment of, 597 Altruism, 3 Alum, 1939 Aluminum hydroxide for acute kidney injury, 782 for hyperphosphatemia, 778, 778t for renal bone disease, 840 Aluminum poisoning, 96 chelators for, 93t clinical manifestations of, 96 diagnosis of, 96 diagnostic testing for, 93t prognosis for, 96b treatment of, 96b

I5

Aluminum salts, 778t Alveolar damage, diffuse, 533f Alveolar filling disorders, 571-575, 571t, 586 classification of, 575t definition of, 571 general approach to, 571, 571f Alveolar hemorrhage, diffuse, 571t, 573-574 Alveolar hydatid disease, 2153 clinical manifestations of, 2153 diagnosis of, 2153 epidemiology of, 2153 treatment of, 2153b Alveolar radiographic patterns, 533 Alveolar ventilation (Va), 654 Alveolar-arterial oxygen gradient [P(A-a)o2], 652-653, 653t Alveolitis extrinsic allergic, 2084-2085 pulmonary lymphocyte/cryptogenic fibrosing, 2237t Alvesco (ciclesonide), 553t Alzheimer’s disease, 2384, 2390-2394 clinical manifestations of, 2392 definition of, 2390-2391 diagnosis of, 2392 differential diagnosis of, 2392 early-onset, 2391 epidemiology of, 2391 evidence-based treatment of, 2394 genetics of, 2391-2392 mild, 2392 MRI findings, 2391, 2392f pathobiology of, 2391-2392 pathophysiology of, 2391-2392 posterior cortical atrophy caused by, 2386f prevention of, 2393b-2394b prognosis for, 2394 risk factors for, 2391 treatment of, 2393b-2394b Alzheimer’s disease dementia, 2390-2391 brain imaging findings, 2392, 2393f diagnostic criteria for, 2391t PET findings, 2392, 2393f Alzheimer’s type II astrocytosis, 1025 Amanita poisoning, 1007 Amantadine, 2184 for chorea, 2463 dose recommendations, 2196t for influenza, 2183t, 2196-2197 for irritability and agression post-head trauma, 2368 mechanisms of excretion and thresholds for dose adjustment, 2184t for Parkinson’s disease, 2457t-2459t for recovery after severe TBI, 2368 Ambiguity, 2625 Amblyopia, 2558 anisometropic, 2558 Cuban tobacco-alcohol amblyopia, 2510 deprivational, 2558 Ambroxol, 601 Ambulatory monitoring in arrhythmias, 348 blood pressure monitoring, 383-384 24-hour tracings, 384, 384f recommendations for, 383 recommended indications for, 383t ischemic monitoring, 425 Ambulatory peritoneal dialysis, chronic, 949 Amcinonide, 2658t Amebae, free-living, 2142, 2142t Amebiasis, 2138-2142 clinical features of, 1869t, 2138-2140 definition of, 2138 diagnosis of, 2140-2141 diagnostic tests for, 2140-2141, 2141t distinguishing characteristics of, 2252t-2253t drug therapy for, 2141t epidemiology of, 1013, 2138 extraintestinal, 2140, 2141f intestinal drug therapy for, 2141t endoscopic and pathologic features of, 2139-2140, 2140f intraluminal, asymptomatic, 2138 laboratory findings, 1871f pathobiology of, 2138 prevention of, 2142 Volume I  pp 1-1382  •  Volume II  pp 1383-2722

I6

Index

Amebiasis (Continued) prognosis for, 2142 treatment of, 2141b-2142b, 2141t Amebic colitis, 2139-2140, 2141t Amebic diarrhea, 2139 Amebic dysentery, 2139-2140 Amebic encephalitis, 2106 Amebic liver abscesses, 1013, 1014f, 2140 clinical manifestations of, 1013 computed tomography findings, 1013, 1014f diagnosis of, 1013, 1014f differential diagnosis of, 2140 drug therapy for, 2141t epidemiology of, 1013 features of, 1011t pathobiology of, 1013 treatment of, 1013b-1014b Amenorrhea, 1588-1589 biochemical evaluation of, 1589, 1590f causes of, 1589t clinical evaluation of, 1588-1589 definition of, 1588 diagnosis of, 1588-1589 hypergonadotropic, 1589-1591, 1591b-1592b hypothalamic, 1477-1478 laboratory findings, 1589 pathobiology of, 1588 postpill, 1588 treatment of, 1591b-1592b American Academy of Neurology, 2366t American Association of Blood Banks, 1195 American Board of Internal Medicine, 3 American Cancer Society dietary guidelines, 1428t-1429t navigation program, 16 recommendations for screening mammography, 1359, 1359t American College of Cardiology (ACC) Guideline for the Management of Overweight and Obesity in Adults, 1458, 1464-1465 guidelines for diagnostic tests in patients with palpitations, 251t, 1395 guidelines for treatment of cholesterol, 1395 recommendations, 54-55 American College of Chest Physicians (ACCP), 2621 consensus guidelines, 1617 recommendations for antithrombotic therapy, 175 American College of Physicians (ACP) guidelines for management of pharyngitis, 2602t recommendations for colorectal cancer screening, 1328 American College of Rheumatology, 1748 classification criteria for fibromyalgia, 1818 classification criteria for rheumatoid arthritis, 1759-1760, 1760.e1f guidelines on management of gout, 1814-1815 recommendations on lifestyle and diet for, 1815t American Diabetes Association, 1428t-1429t American Fact Finder (U.S. Census Bureau), 43.e1t American Heart Association (AHA) Guideline for the Management of Overweight and Obesity in Adults, 1458, 1464-1465 guidelines for diagnostic tests in patients with palpitations, 251t, 1395 guidelines for treatment of cholesterol, 1395 recommendations, 54-55 American human monocytic ehrlichiosis, 2053, 2053t clinical manifestations of, 2053 diagnosis of, 2053 epidemiology of, 2053 treatment of, 2053b American Joint Committee on Cancer (AJCC) Cancer Staging Manual, 1309, 1309t classification of pancreatic neuroendocrine tumors, 1335 staging of head and neck squamous cell carcinoma, 1301t staging system, 1206

American Joint Committee on Cancer (AJCC) (Continued) TNM staging of bladder cancer, 1349, 1350t TNM staging of breast cancer, 1353t TNM staging of esophageal cancer, 1315t TNM staging of large bowel cancers, 1329 American Medical Association (AMA), 8-9 American Red Cross, 1191 American Society of Clinical Oncology, 1356t, 1858 American Society of Hypertension, 390f American Spinal Injury Association, 2369t American sylvatic typhus, 2048t American Thoracic Society, 616 American trypanosomiasis. See Chagas disease Amevive (alefacept), 171 Amifostine, 85 Amikacin for AIDS-associated opportunistic infections, 2297t-2301t for bacterial meningitis, 2488t dosage adjustments in renal failure, 129t for nocardiosis, 2064 for pneumonia, 619t, 1966 for Pseudomonas infection, 1966-1967 for pyelonephritis, 1875t recommended doses and schedules, 1891t-1892t Amiloride, 748t for hypertension, 388t for Liddle’s syndrome, 824 for magnesium wasting, 775 Amino acid, 1442t ε-Aminocaproic acid for hemophilia, 1176 for subarachnoid hemorrhage, 2448, 2448t Aminoglycosides for bacterial meningitis, 2488t for endocarditis, 480 mechanism of action, 1889t, 1890-1894 mechanism of resistance, 1890t nephrotoxicity, 1888-1889, 1894f recommended doses and schedules, 1891t-1892t resistance to, 1961 toxicities, 1895t Aminophylline, 554 Aminoquinolines, 2110-2111 5-Aminosalicylates (5-ASA) for colitis, 941 for inflammatory bowel disease, 939, 939t for proctitis, 941 Aminosidine sulfate, 2122 Aminotransferase, 989 with bile duct stones, 1042 elevated, asymptomatic, 992, 992f Amiodarone for advanced cardiac life support, 354f for arrhythmias, 354-355 for atrial fibrillation, 365-366 dose-response effects, 1008t doses and side effects, 363t-364t for electrical storm, 373 ocular effects, 2573, 2573t for supraventricular arrhythmia, 326 for ventricular tachycardia, 373-374 Amish, 1384, 1685 Amitiza (lubiprostone), 893t Amitriptyline (Elavil) for chronic pain, 136, 138t for depression, 2349t for distal symmerical polyneuropathy, 2536 for dyspepsia, 895 for erythromelalgia, 508 for headache prevention, 2358t for irritable bowel syndrome, 893t, 894 for painful dysesthesias and paroxysmal dystonic spasms, 2477 for pruritus, 2636t for tinnitus, 2596 Amlodipine for acute STEMI, 455 for aortic dissection, 496 for atheromatous embolization, 507 for headache prevention, 2358t for hyperadrenergic disorders, 2521-2522 for hypertension, 388t for livedo reticularis, 506 for pernio, 510b

Amlodipine (Continued) for Prinzmetal’s angina, 431 properties of, 430t for pulmonary hypertension, 402 for Raynaud’s phenomenon, 509f Ammonia clearance, 990 Ammoniagenesis, 764 Ammonium lactate lotion, 2667 Ammonium magnesium phosphate crystals, 731-732, 732f Amnesia anterograde cause of, 2384 diagnosis of, 2384 transient global, 2384 Amnesic disorders, 2382-2384 clinical manifestations of, 2383-2384 diagnosis of, 2384 pathobiology of, 2383 Amnesic mild cognitive impairment, 2390t Amniotic fluid embolism, 627, 627.e1f epidemiology of, 627 prognosis for, 627 treatment of, 627b Amoxicillin for cystitis, 1875t for enterococcal infection, 1915.e1t for H. pylori eradication, 915t for leptospirosis, 2030 for Lyme disease, 2025t for peritonsillar abscess, 2603 pharmacokinetic parameters for, 126t recommended doses and schedules, 1891t-1892t for S. pneumoniae infection, 1905, 1905t for Salmonella carriers, 1975 for tonsillitis, 2604 Amoxicillin-clavulanic acid for bacterial overgrowth, 928 for cystitis, 1875t for infectious rhinosinusitis, 2588 for Moraxella infection, 1948 for nocardiosis, 2064 for peritonsillar abscess, 2603 for pharyngitis, 2602 for pneumonia, 616t for pyelonephritis, 1875t recommended doses and schedules, 1891t-1892t for S. pneumoniae infection, 1905t for spontaneous bacterial peritonitis, 1030 Amphetamines, 160 clinical manifestations of, 160 for narcolepsy, 2419t pathobiology of, 160 screening for, 699-702, 702t toxicity, 699t-702t Amphotericin B for AIDS-associated opportunistic infections, 2297t-2301t available formulations, 2064-2065 for bacterial endocarditis, 479t for blastomycosis, 2075 for candidemia, 2082 for coccidioidomycosis, 2073 for cryptococcal meningitis, 2077 for hepatosplenic candidiasis, 1013 for histoplasmosis, 2071 lipid-associated formulations, 2064-2065 for AIDS-associated opportunistic infections, 2297t-2301t pharmacology of, 2065 toxicity of, 2065 liposomal for AIDS-associated opportunistic infections, 2297t-2301t for invasive aspergillosis, 2086, 2086t for visceral leishmaniasis, 2122-2123 mechanism of action, 2064 mechanism of resistance to, 2064 for mucormycosis, 2090, 2090f preparations, 2064-2065 spectrum of activity, 2064 for sporotrichosis, 2079 for visceral leishmaniasis, 2122 Amphotericin B cholesteryl sulfate, 2297t-2301t Amphotericin B colloidal dispersion, 2064 Amphotericin B deoxycholate, 2064-2065 acute reactions to, 2065 administration, 2065

Amphotericin B deoxycholate (Continued) for AIDS-associated opportunistic infections, 2297t-2301t chronic toxicity, 2065 for cryptococcal meningitis, 2077 dosage, 2065 formulation, 2064 nephrotoxicity, 2065 pharmacology, 2065 Amphotericin B lipid complex, 2064 for AIDS-associated opportunistic infections, 2297t-2301t for invasive aspergillosis, 2086t Ampicillin for anthrax exposure, 88b for bacterial endocarditis, 479t for bacterial meningitis, 2487, 2488t for endocarditis, 480 for enterococcal infection, 1915.e1t for leptospirosis, 2030 for peritonitis, 949 for prostatitis, 831 recommended doses and schedules, 1891t-1892t resistance to, 1961 for S. pneumoniae infection, 1905 for Salmonella bacteremia, 1974 for septic shock, 689t for shigellosis, 1978 for ulcerative colitis, 942 Ampicillin-sulbactam for Acinetobacter species infections, 1969 for cholangitis, 1043 for deep neck abscesses, 2603 for pneumonia, 616t, 618, 619t for pyogenic liver abscess, 1012 for S. pneumoniae infection, 1905t Amplatzer devices, 287, 289f Amprenavir, 2288t, 2331t Ampullary tumors, 1047, 1047.e1f Amyl nitrite, 706t-710t Amylase, 961 Amyloid angiopathy, 2450-2451 Amyloid neuropathy, 2520 Amyloidosis, 335, 335f-336f, 1284-1287 cardiomyopathy with, 331 clinical manifestations of, 1284-1285, 1285f definition of, 1284 diagnosis of, 335, 1285-1286, 1286f dialysis-associated, 842 epidemiology of, 335 esophageal manifestations, 906 factor deficiency in, 1178 forms of, 1287 pathobiology of, 335 prognosis for, 335b, 1287 renal, 792 screening for, 1285 secondary, 1287 symptoms and signs of, 1284, 1284t syndromes, 1284t treatment of, 335b, 1286, 1286b upper aerodigestive tract involvement, 2605, 2605.e2f Amylophagia, 1069 Amyotrophic lateral sclerosis, 2522-2525, 2539t bulbar symptoms, 2523, 2523.e1 clinical manifestations of, 2523-2524 definition of, 2522 diagnosis of, 2524-2525 diagnostic criteria for, 2524-2525, 2524t disorders that mimic, 2524-2525, 2524t epidemiology of, 2522 incidence of, 2522 laryngeal, 2606 limb symptoms and signs, 2523, 2523.e1 pathobiology of, 2522-2523 pathology of, 2523 prognosis for, 2525 treatment of, 2525b Anabolic agents, 1643-1644 Anabolic hormones, 1571-1572 Anabolic-androgenic steroids, 162 Anaerobic bacteria, 1931-1932, 1931f Anaerobic infections, 1931-1934 antimicrobial susceptibility patterns, 1933, 1934t clinical manifestations of, 1932-1933, 1933t definition of, 1931

Index Anaerobic infections (Continued) diagnosis of, 1933 location of, 1931, 1931t pathobiology of, 1932 specimen acceptability for culture, 1933, 1933t taxonomy, 1931.e3t treatment of, 1933t virulence factors, 1932, 1932t Anaerobiospirillum thomasii, 1931 Anagen effluvium, 2703-2704 Anakinra (Kineret), 169-170, 1762 Anal canal, 971 Anal cancer, 972-973 clinical manifestations of, 972 diagnosis of, 972 in HIV infection, 2322, 2323t, 2326, 2327f incidence of, 972, 2220 prevention of, 973 treatment of, 972b-973b Anal dysplasia, 972 Anal fissures, 969-970, 970f clinical manifestations of, 970 diagnosis of, 970 epidemiology of, 969 pathobiology of, 970 treatment of, 970b Anal fistula, 969 clinical manifestations of, 969 diagnosis of, 969 epidemiology of, 969 pathobiology of, 969 treatment of, 969b Anal intraepithelial neoplasia, 972 Anal warts, 972, 972f clinical manifestations of, 972 diagnosis of, 972 treatment of, 972b Anal wink reflex, 2372t Analgesia critical care, 650 neuraxial (spinal and epidural), 2620 Analgesic nephropathy, 797, 797f Analgesics adjuvant, 136-137, 138t antipyretic, 136-138 contraindications to, 1088t opioid, 137, 139t-140t preferred medications for women, 1602t topical, 2659-2660 Anaphylactic transfusion reactions, 1196 Anaphylactoid transfusion reactions, 1196 Anaphylatoxin receptors cellular responses, 244t distribution of, 244t Anaphylatoxins, 240, 245 Anaphylaxis, 1675 catamenial, 1700 distinguishing features of, 528t food allergy-associated exercisedependent, 1699 food-induced, 1699 perioperative, 2618 systemic, 1698-1703 Anaplasma, 2047t, 2053 Anaplasma phagocytophilum, 2053-2054 Anaplasmosis, 2047t, 2053-2055, 2053t human granulocytic, 2057t vectors, 2172t Anaplastic astrocytoma, 1293 Anaplastic carcinoma, small cell, 1381 Anaplastic large cell lymphoma, 1266 cutaneous, 1266 systemic, 1266, 1266b treatment of, 1266 Anaplastic lymphoma, 1381 Anaplastic thyroid carcinoma, 1513 Anastomotic ulcers, 911-912 Anastrozole (Arimidex), 1211t-1216t, 1358-1359 Ancef (cefazolin), 1832 Ancylostoma braziliensis, 2160 Ancylostoma caninum, 2160 Ancylostoma ceylanicum, 2160 Ancylostoma duodenale, 2103, 2159-2161 Andersen-Tawil syndrome, 2542t, 2544-2545 Andes virus, 2249t Androderm (testosterone), 1574t AndroGel (testosterone), 1574t

Androgen(s) adrenal, 1514 defects of biosynthesis of, 1564 preparations, 1574t Androgen deficiency, 1572 Androgen excess, 1518-1519 definition of, 1518 diagnosis of, 1518-1519 treatment of, 1519b Androgen insensitivity, 1564 Androgen replacement therapy contraindications to, 1574 indications for, 1574, 1574t preparations, 1574t risks vs benefits, 1575t Androgen resistance, 1573 Androgen-deprivation therapy, 1367-1369, 1369t Androgenetic alopecia, 2703t, 2704 Androgen-sensitive end-organ deficiency, 1573 Anemia acute, 1100 aplastic, 1079, 1114-1121, 1114f approach to, 1059-1068 autoimmune hemolytic in chronic lymphocytic leukemia, 1256 blood smear findings, 1062-1063 bone marrow findings, 1063 of chronic disease, 1064, 1064t, 1071-1072 definition of, 1071 diagnosis of, 1071 laboratory findings, 1070t treatment of, 1071b chronic hemolytic, 1056t of chronic inflammation, 1064-1065, 1071-1072 clinical manifestations of, 1071 definition of, 1071 diagnosis of, 1065 epidemiology of, 1071 pathobiology of, 1065, 1065f, 1071 prognosis for, 1072 treatment of, 1065b of chronic kidney disease, 840 of chronic liver disease, 1066 clinical manifestations of, 1061-1062, 1061f definition of, 1059 diagnosis of, 1062-1063 differential diagnosis of, 1056-1057 due to blood loss, 1063 due to decreased red cell production, 1063-1066, 1063t in elderly, 1066 of endocrine hypofunction, 1066 in end-stage renal disease, 842 Fanconi’s, 1134, 1227t in heart failure, 316 hemolytic, 1066-1067, 1080-1088 acute, 1088, 1088b blood smear features of, 1056t-1057t causes of, 1073, 1074t chronic, 1056t chronic nonspherocytic, 1088, 1088b classification of, 1067t congenital nonspherocytic, 1086 diagnosis of, 1067 differential diagnosis of, 1057 extracorpuscular, 1079-1080 intravascular, 1073-1080 laboratory features of, 1067, 1067t HIV-associated, 2327 diagnosis of, 2327 differential diagnosis of, 2327t epidemiology of, 2327 pathobiology of, 2327 treatment of, 2327b hypochromic microcytic, 1068, 1068f, 1070t initial assessment of, 1062, 1062t iron deficiency, 876, 1068-1071 laboratory evaluation of, 1062-1063 macrocytic, 1064 blood smear features of, 1053f, 1056-1057, 1056t differential diagnosis of, 1053f, 1056-1057, 1056t evaluation of, 1109

Anemia (Continued) megaloblastic, 1104-1114, 1110f blood smear features of, 1056-1057, 1056t causes not responding to cobalamin or folate therapy, 1112t peripheral blood findings, 1136, 1136f treatment of, 1112b-1113b microcytic, 1056, 1063-1064, 1068-1073 normocytic, 1064-1066 normocytic normochromic, 1057 pathobiology of, 1060-1061 pernicious, 1107 persistent, 2214, 2214t physical findings, 1062 postoperative, 2624 refractory anemia with ring sideroblasts, 1236-1237 of renal insufficiency, 1065 pathobiology of, 1065 treatment of, 1065b-1066b sickle cell, 1095-1096 clinical manifestations of, 1099 prevalence of, 1096t sideroblastic, 1072-1073, 1073f blood smear features of, 1056t laboratory findings, 1070t stem cell transplantation for, 1202-1203 symptoms of, 1061-1062 thalassemias, 1089-1095 treatment of, 316, 840, 1067-1068 of uremia, 1065-1066, 1066f Anergy, 239-240 Anesthesia, 2617-2620 approaches, 2618, 2618t general, 2618 general versus regional, 2620 hypersensitivity reactions during, 2618 intraoperative management of, 2618-2620 medication reactions, 2617-2618 monoamine oxidase inhibitor interactions, 2618 neuraxial (spinal and epidural), 2620 in orthopedic surgery, 1829 preoperative assessment for, 2617 regional, 2620 in sickle cell anemia, 1103 Anesthesia records, 2621 Anesthetics topical agents, 2659-2660 toxicity, 706t-710t volatile (inhalational), 2619 Anetoderma, 2694, 2694f Aneuploidy, 1565 Aneurysm aortic, 492-494 abdominal, 55-56, 492-493, 492f-493f thoracic, 394t, 492-494 thoracoabdominal, 492 berry, 2447 cerebral, 2447-2448 fusiform, 2448 intracranial, 2447 left ventricular, 453 mycotic, 2448 pseudoaneurysm, 453 pupillary abnormalities due to, 2576 saccular, 2447, 2447f sinus of Valsalva, 413 visual field abnormalities with, 2575 Angelman’s syndrome, 194 Angiectasia, 957, 957f Angiitis allergic, 586 cutaneous leukocytoclastic clinical manifestations of, 1799 management of, 1799 pathologic characteristics of, 1795t primary, of the central nervous system, 1800, 1800t Angina, 249t in aortic stenosis, 463 chronic diagnostic coronary angiography in, 425t evaluation of, 423f chronic stable, 460 definition of, 420 differential diagnosis of, 426 echocardiographic findings, 280t-281t

I7

Angina (Continued) microvascular, 431-432 Prinzmetal’s, 420-421, 431 rest or unstable, 249t treatment of, 316, 429-430 unstable, 432-441 in acute coronary syndrome, 434 CABG for, 460 definition of, 432 recurrent or refractory, 439-440 treatment of, 439-440 variant, 431 vasospastic, 421 Angina pectoris, 420-432 clinical features of, 421-422 definition of, 420 diagnostic tests for, 424t epidemiology of, 420 exercise ECG in, 423-425 exercise test protocol for, 424 grading of, 420 history in, 421 pathobiology of, 420-421 prognosis for, 432 recommendations for myocardial revascularization in, 428t risk stratification, 426-427 treatment of, 427b-431b, 427f, 428t Anginal equivalents, 421, 460 Angiodysplasia, gastrointestinal, 957, 957b, 957f Angioedema, 1675, 1693-1697 ACE inhibitor-associated, 1697 acquired epidemiology of, 1695 pathobiology of, 1697 treatment of, 1697 classification of, 1694t differential diagnosis of, 1694 epidemiology of, 1693 evaluation of, 1696f future directions, 1696-1697 hereditary, 1697-1698, 1740t pathobiology of, 1693 prognosis for, 1696 treatment of, 1696f visceral, 948 Angiofollicular lymph node hyperplasia, 1267 Angiogenesis, 1230 Angiogenesis inhibitors, 170 adverse effects of, 170 indications for, 170 types of, 170 Angiography, 292-298 catheter, diagnostic, 2440 in chronic stable angina, 423, 423f computed tomography in atherosclerotic PAD, 500, 501f coronary, 287, 287f-289f, 297f, 2617t in coronary artery disease, 425 preoperative, 2617t pulmonary, 621-622, 622f strengths and weaknesses, 2345-2346 coronary, 295, 297f in acute coronary syndrome, 435, 438t-439t in chronic angina and stable ischemic heart disease, 425t diagnostic, 425-426, 425t in heart failure, 305 preoperative, 2617t procedures that accompany, 293t digital subtraction in atherosclerotic PAD, 500, 501f in renal artery stenosis, 807-808 endoscopic retrograde cholangiography, 873 magnetic resonance, 735-736, 736f in atherosclerotic PAD, 500, 501f direct, 1728 in renal artery stenosis, 807-808, 808f strengths and weaknesses, 2345t magnetic resonance cholangiography, T1-weighted, 871 magnetic resonance cholangiopancreatography, 870f, 871 in acute pancreatitis, 962 with bile duct stones, 1042, 1043f in biliary disease, 877 Volume I  pp 1-1382  •  Volume II  pp 1383-2722

I8

Index

Angiography (Continued) in primary sclerosing cholangitis, 1045, 1046f noncardiac, 295 pulmonary, 623 strengths and weaknesses, 2345t superior mesenteric, 953f Angioimmunoblastic T-cell lymphoma, 1074 Angiokeratoma, 1401f, 1402 Angiomatosis, bacillary, 1999-2001 Angiomyolipoma, 2515-2516 Angioplasty coronary, 458 for acute MI, 458, 459f for atherosclerotic PAD, 502, 503f percutaneous transluminal, 502, 503f technique, 456, 456f indications for, 446t Angiosarcoma gastrointestinal, 958 pericardial, 339 Angiostrongyliasis, 2166 clinical manifestations of, 2166 definition of, 2166 diagnosis of, 2166 epidemiology of, 2166 treatment of, 2165t, 2166b Angiostrongylus, 2159, 2164 Angiostrongylus cantonensis, 2166 Angiostrongylus costaricensis, 2166 Angiotensin II, 837, 837t Angiotensin-converting enzyme inhibitorassociated angioedema, 1697 Angiotensin-converting enzyme inhibitors, 307-312 for acute STEMI, 450, 454-455 for angina, 428-429 at hospital discharge after MI, 454t clinical benefits, 312 clinical use, 390 contraindications to, 389t for heart failure, 306.e1t-306.e2t for hypertension, 388t, 390, 394t for ischemic heart disease, 428-429 mechanism of action, 309-312, 390 practical guidance on use of, 310t practical use, 312 side effects of, 389t, 390 Angiotensin-receptor blockers, 307-309, 313 for acute STEMI, 450 clinical benefits, 313 clinical use, 390 contraindications to, 389t for heart failure, 306.e1t-306.e2t for hypertension, 388t, 390, 394t mechanism of action, 313, 390 practical guidance on use of, 310t side effects of, 389t, 390 Angular cheilitis, 2583 Anhidrotic ectodermal dysplasia with immunodeficiency, 1687t Anidulafungin, 2069 adverse effects of, 2069 for candidemia, 2082 drug interactions, 2069 for hepatosplenic candidiasis, 1013 indications for, 2069 pharmacology of, 2069 Animal contact, 1972 Animal Efficacy Rule (FDA), 1988 Animal models of human microbiome, 1843 of systemic sclerosis, 1780.e1 Animal products, contaminated, 1972 Anion gap, 766 Anion gap metabolic acidosis, 766-769 causes of, 766-767, 766t prognosis for, 767 Anisakiasis, 2163 Anisakis, 2159, 2163 Anisocoria, physiologic, 2576 Anisocytosis, 1052f Ankle disorders of, 1754 orthopedic procedures on, 1832 Ankle sprain, 1754 Ankle-brachial index (ABI), 498, 498f Ankylosing spondylitis, 631, 1765-1766 classification of, 1713t clinical manifestations of, 631, 1765 diagnosis of, 631, 1765-1766 diagnostic criteria for, 1765t

Ankylosing spondylitis (Continued) differential diagnosis of, 1764t, 1766 epidemiology of, 631, 1765 imaging of, 536f laboratory findings, 1766 lung disease associated with, 585 pathobiology of, 631 physical findings, 1765-1766 radiographic findings, 1724t, 1766, 1766f treatment of, 631b Ann Arbor staging system, 1270, 1271t Annelids, 2177 Anogenital warts, 2221 Anomalous pulmonary venous return, 285f, 415-416 Anomalous venous connections, 415-416 Anomia, 2384-2385 Anomic aphasia, 2385t Anorectal conditions, 967-973 Anorectal disease in HIV infection, 2304 sexually transmitted diseases, 973 Anorectal pain, 2303t Anorexia approach to, 852t hypothalamic, 1479 management of approaches to, 11t in palliative care, 11t postoperative, 2622 Anorexia nervosa, 1455, 1592 definition of, 1455 diagnostic criteria for, 1455t epidemiology of, 1456 family-based therapy for, 1457 gonadal insufficiency due to, 1574 natural history of, 1457 pathobiology of, 1456 prognosis for, 1457 risk factors for, 1456 symptoms and signs of, 1456 treatment of, 1457 Anovulation, chronic, 1592-1594 related to inappropriate feedback, 1593-1594 related to other endocrine and metabolic disorders, 1594 Anserine bursitis, 1753 Antacids, 900t Antalgic gait, 2341t Anterior cerebral artery, 2426, 2429f Anterior cerebral artery occlusion, 2435, 2435t Anterior choroidal artery, 2427, 2429f Anterior choroidal artery occlusion, 2435, 2435t Anterior cord syndrome, 2365, 2378 Anterior horn cell disease, 2539t. See also Amyotrophic lateral sclerosis Anterior pituitary gland, 1480 Anterior resection syndrome, 1330 Anthelmintics, 1088t Anthracycline dilated cardiomyopathy due to, 330 for Hodgkin’s lymphoma, 1272 Anthrax, 87-88, 87t, 1920-1923, 2058t-2059t clinical manifestations of, 1921-1922 cutaneous clinical manifestations of, 1921 epidemiology of, 1920 fatality rate, 2060t prognosis for, 1923 stages of, 1921 definition of, 1920 diagnosis of, 1922-1923 distinguishing characteristics of, 2252t-2253t epidemiology of, 1920 gastrointestinal, 1921-1922 immunizing agents for, 68t-73t inhalational clinical manifestations of, 1921 diagnosis of, 1922 fatality rate, 1923, 2060t prognosis for, 1923 staging system for, 1921, 1921t injection, 1920-1921, 1921f intestinal, 1921-1922 oropharyngeal, 1921-1922 pathobiology of, 1920-1921 prevention of, 1922 prognosis for, 1923

Anthrax (Continued) treatment of, 1922b future directions, 1923 recommendations for, 1922 Anthrax exposure clinical manifestations of, 87-88 diagnosis of, 88 epidemiology of, 87 pathobiology of, 87-88 prevention of, 88 prognosis for, 88 treatment of, 88b Anthrax meningitis, 1922 diagnosis of, 1922 treatment of, 2487 Anthrax meningoencephalitis, 1922 Anthrax vaccine, 68t-73t, 78 adverse reactions to, 78 indications for, 78 Anthropometry, upper arm, 1432 Antiadhesion molecules, 940 Antianginal therapy, 436-437 Antianxiety drugs, 2352t Antiarrhythmic drugs, 363t-364t for acute MI, 455 for acute STEMI, 451 selection of, 366t Antiatherosclerotic therapy, 419 Antibacterial chemotherapy, 1885-1896. See also Antimicrobial therapy Antibacterial soap, 2681 Antibiograms, 1843 Antibiotic therapy. See Antimicrobial therapy Antibiotic-associated diarrheas, 920 Antibodies, 221 acetylcholine receptor, 2550 alloantibody inhibitors, 1176 anti-CCP, 1719-1720 anti-endothelial cell, 1796 anti-glomerular basement membrane, 734-735 antineutrophil cytoplasmic, 1722-1723 ANCA-associated vasculitis, 1713t, 1795-1798, 1801 in glomerular syndromes, 734 perinuclear, 734 in vasculitis, 1795-1796 antinuclear, 734, 1721-1723, 1721t antiphospholipid antibody syndrome, 1180, 1722, 1774 anti-S. cerevisiae, 939 autoantibodies associated with seropositive systemic autoimmune diseases, 1716t associated with SLE, 1770, 1771f, 1775t in systemic sclerosis, 1780-1781, 1780t thyroid, 1501 without disease specificity, 1716t for cancer treatment, 1209-1218 cardiac antibody-mediated rejection, 521t to citrullinated proteins, 1719-1720 digoxin-specific antibody fragment (Fab), 699t-702t, 706t-710t to DNA, 1721-1722 factor VIII alloantibodies, 1176 factor IX alloantibodies, 1176 fluorescent treponemal antibody absorption (FTA-ABS) test, 2017, 2017t against IL-12 and IL-23, 170 insulin, 1554 monoclonal for asthma, 553 for chronic lymphocytic leukemia, 1256 nomenclature, 169, 169t muscle-specific kinase, 2550 to phospholipids, 1722 red cell, 1074 specificities of, 221 for viral hemorrhagic fever, 2255 Antibody defects, 1679-1681, 1680t incidence of, 1679 with normal immunoglobulins, 1680 treatment of, 1681b Antibody deficiency disorders, 1677, 1677t Anti-C5, 246 Anticatabolic agents, 1642-1643 Anticholinergics for asthma, 552 for benign prostatic hyperplasia, 830, 830t

Anticholinergics (Continued) with psychoactive effects, 116, 117t toxicity, 706t-710t toxidromes, 696t Anticipation, 194 Anticoagulation, 366-367. See also specific anticoagulants for acute coronary syndrome, 438t-439t, 439 for acute STEMI, 448, 448f, 450, 454 for angina, 429 antepartum, 1619t contraindications to, 314 duration of, 625 for ischemic heart disease, 429 long-term, 516-517 new agents, 178 oral, 2616, 2617t perioperative, 2613t, 2616, 2616t in pregnancy, 1618t preoperative, 2616, 2617t for pulmonary embolism, 624-625, 624t-625t side effects of, 517 Anticonvulsants for AIDS-associated opportunistic infections, 2297t-2301t for chronic pain, 136-137, 138t for pruritus, 2636t Anti-D immunoglobulin, 1164t Antidepressants, 2347-2348, 2349t cyclic, 699t-702t for irritable bowel syndrome, 894 for menopausal hot flushes, 1628t-1629t for pruritus, 2636t tricyclic for chronic pain, 136, 138t for depression, 2349t screening for, 699-702, 702t Antidiuresis, 1495 Antidiuretic hormone, 738-739 syndrome of inappropriate secretion of, 1220, 1496 causes of, 751-752, 751t diagnosis of, 750 drug-induced, 2626t, 2629 patterns of serum ADH abnormalities in, 751-752, 751f Antidotes, 705, 706t-710t Anti-endothelial cell antibodies, 1796 Antiepileptic drugs, 2407, 2407t characteristics of, 2408t discontinuing, 2408 Anti-factor Xa monitoring, 515 Antifibrotic therapy, 1784 Antifungal agents for aspergillosis, 2086-2087, 2086t azole, 2066-2069 for dematiaceous infections, 2103 echinocandin, 2069 for mucormycosis, 2090-2091, 2090f systemic, 2064-2070 topical, 2659 Antigen detection tests, 2122 Antigen presentation, 221 Antigen processing, 221-222, 221f Antigen-presenting cells, 1688-1689 Antigens, 237-238 B blood group, 238 cryptococcal polysaccharide antigen (CRAG) assay, 2077 cytotoxic T-lymphocyte antigen (CTLA)-4 (CD152), 223 Epstein-Barr nuclear (EBNAs), 2232 hepatitis B surface (HBsAg), 996, 996f histo-blood group (HBGAs), 2245 human leukocyte (HLAs), 236-237 associations with rheumatoid arthritis, 1755 class I, 237, 237.e1f, 236.e1f, 236.e1t class II, 237, 237.e1f, 236.e1t DRB1, 237, 237.e1t human leukocyte (HLAs), 1755.e1t lymphocyte function-associated antigen-1 (LFA-1, CF11a/CD18), 1145 macrophage antigen-1 (Mac-1), 1145 major histocompatibility, 236-237, 237.e1f MHC class I−related chain A, 237-238 minor histocompatibility, 237 prostate-specific (PSA), 1367, 1380 prostate-specific (PSA) testing, 831 recognition of, 220-222

Index Antigens (Continued) superantigens, 1796 in transplantation, 236-238 Antigen-specific receptors structure of, 220-221 types of, 220-221 Anti-glomerular basement membrane antibodies, 734-735 Anti–glomerular basement membrane disease, 790, 791f, 1798 Antihemophilic factor (factor VIII), 1173 Antihemophilic factor (factor VIII) alloantibodies, 1176 Antihemophilic factor (factor VIII) deficiency, 1175t, 1179 Antihemophilic factor (factor VIII) replacement therapy, 1174-1175 Antihemophilic factor B (factor IX), 1173 Antihemophilic factor B (factor IX) alloantibodies, 1176 Antihemophilic factor B (factor IX) deficiency, 1175t Antihemophilic factor B (factor IX) replacement therapy, 1174-1175 Antihistamines, 2660t for chronic sinusitis, 1691-1692 for pruritus, 2636t Antihypertensive drugs, 387-391 alternatives to, 1462t contraindications to, 389t drug interactions, 391 first-line drugs, 387-391 for menopausal hot flushes, 1628t-1629t oral, 387-391, 388t, 394t in pregnancy, 1613 side effects of, 389t Anti-infective therapy, 1843-1848 Anti-inflammatory therapy perioperative, 1833 topical agents, 2658-2659 Antileukotrienes, 552-553 Antimalarials contraindications to, 1088t for rheumatoid arthritis, 1761 topical drugs, 2658-2659 Antimetabolites, 1211t-1216t Antimicrobial stewardship, 1867, 1865.e1t Antimicrobial therapy administration of, 1848 administrative aspects, 1848 age and, 1845 allergic reactions to, 1845 for anaerobic infection, 1933 for bacterial meningitis, 2485, 2488t bactericidal activity, 1844 for bloating, 893t for brain abscess, 2496, 2496t for bronchectasis, 568-569 cerebrospinal fluid permeability, 2486t choice of antibiotics, 1885-1886 combinations, 1846-1848 contraindications to, 1088t for COPD prophylaxis, 560 for cystitis, 1875, 1875t definitive, 1844 for diarrhea, 922 dosage, 1885-1887, 1891t-1892t drug classes, 1889-1896 drug interactions, 1846 duration of, 1896 effects on institutional environment, 1848 empirical, 1843-1844, 1846 for endocarditis prophylaxis, 483t for eye infections, 2565t failure of, 617, 617t, 1896 formularies, 1848 for gonorrhea, 1944-1945, 1945t for H. pylori eradication, 915t hepatic function and, 1845-1846 host factors, 1846 for infectious diarrhea, 1870t, 1872 for inflammatory bowel disease, 940 interchangeability of drugs, 1848 for malaria, 2112 mechanisms of action, 1888, 1889t minimal inhibitory concentration (MIC), 1844, 1886, 1886f monitoring, 1848 for mycetoma, 2101 for osteomyelitis, 1810, 1810t

Antimicrobial therapy (Continued) pediatric dosing regimens, 1845 for pertussis, 1992 pharmacodynamic considerations, 1848 pharmacokinetic changes, 1888-1889 for pharyngitis, 2602, 2602t for pneumonia, 615-617 in-hospital regimens, 616-617 outpatient regimens, 616 in pregnancy, 1845 for pyelonephritis, 1875, 1875t recommended doses and schedules, 1891t-1892t renal function and, 1845-1846 resistance to, 1838 emergence of, 1887-1888 mechanism of, 1888, 1890t suppression of, 1888 route of administration, 1848 for S. pneumoniae infection, 1905, 1905t schedule, 1885-1886 selection of, 1843-1846 for septic arthritis, 1810t for septic shock, 689t, 690 for spondyloarthritis, 1764 susceptibility to of anaerobic bacteria, 1933, 1934t of gonococci, 1944 synergism, 1847f for tetanus, 1930 topical agents, 2659 toxicity, 1845-1846, 1895t, 1896 for UTI prophylaxis, 1876, 1876t and viral hemorrhagic fever, 2254 Antimüllerian hormone, 1561 Antimuscarinic agents, 830 Antineutrophil cytoplasmic antibodies (ANCAs), 1722-1723 ANCA-associated vasculitis, 1713t, 1795-1798, 1801 in glomerular syndromes, 734 perinuclear, 734 in vasculitis, 1795-1796 Antinuclear antibodies, 734, 1721-1723, 1721t Antioxidants, 235, 455, 1454 Antiparasitic agents, 2103-2107 for Chagas disease, 2118 “Drugs for Parasitic Infections” (The Medical Letter on Drugs and Therapeutics), 2103 for infectious diarrhea, 1870t topical, 2659 Antiphospholipid antibody syndrome, 1180, 1722 catastrophic, 1180 diagnostic criteria for, 1180 laboratory criteria for, 1180 in SLE, 1774 treatment of, 1180b-1181b Antiphospholipid syndrome, 1180 catastrophic, 1190 thrombosis in, 1190, 1190b Antiplatelet agents for acute coronary syndrome, 437-439, 438t-439t for acute MI, 449-450 for acute STEMI, 448f, 454 for angina, 429 for atherosclerotic PAD, 501-502 for ischemic heart disease, 429 perioperative management of, 2613t for prevention of CVD in diabetes, 1547 Antipruritic agents, 2659-2660 Antipsychotic medications, 2353, 2354t Antipsychotic-induced cardiomyopathy, 2626t, 2627 Antipsychotic-induced metabolic syndrome, 2626-2627, 2626t Antipsychotic-induced myocarditis, 2626t, 2627 Antipyretic analgesics, 136-138 Antiretroviral therapy access to, 2272 for acute HIV infection, 2291 for acute opportunistic infection, 2291 CSF-to-plasma ratios, 2330, 2331t development of, 2289, 2289f drugs, 2288t fixed-dose combinations, 2289, 2290t

Antiretroviral therapy (Continued) global expansion, 2272 guidelines for, 2287, 2288t for HIV/AIDS, 2287-2292 initial regimens, 2289, 2290t prompt initiation of, 2294 rationale for starting, 2287 what to change to, 2291 what to start, 2287-2289 when to change, 2290 when to start, 2287, 2288t for HIV-infected patient in intensive care, 2317-2318 for Pneumocystis pneumonia, 2097 for Pneumocystis pneumonia in HIV infection, 2314 in pregnancy, 2291 for prevention, 2291-2292 response to, 2280 side effects of, 2291 Strategies for Management of Antiretroviral Therapy (SMART) study, 2302 toxicity, 2291 Anti-Saccharomyces cerevisiae antibodies, 939 Antisocial personality disorder, 2355t Antithrombin deficiency, 1186t Antithrombin III deficiency, 1185 Antithrombins, 450 Antithrombotic therapy, 175-181 for atherosclerotic PAD, 501-502 long-term, for acute STEMI, 455f pharmacologic agents, 175-181 physiologic mechanisms, 1155 recommendations for, 175, 180t Antithymocyte globulin, 1079, 1120 Antithyroid drugs, 1508 Anti-TNF agents, 1764 α1-Antitrypsin, 558 α1-Antitrypsin deficiency, 187t liver biopsy in, 982-983, 983f liver disease in, 982-983 liver transplantation for, 1035 Antituberculous agents, 1891t-1892t Antivenom, 706t-710t, 717, 719 Antiviral therapy, 2177-2185. See also Antiretroviral therapy for cytomegalovirus infection, 2181t dose recommendations, 2196t for hepatitis B, 1002, 1003t, 2177-2179, 2177t for hepatitis C, 2177t, 2179-2181 for herpes, 2181-2183, 2181t for HIV/HBV coinfection, 2179 for HIV/HCV coinfection, 2181 for influenza, 2183-2184, 2183t, 2196, 2196t for poxvirus illnesses, 2219 topical, 2659 for varicella zoster virus infection, 2181t for viral hemorrhagic fever, 2254-2255 Antley-Bixler syndrome, 1564 Anton’s syndrome, 2575 Anxiety chest pain in, 249t management of approaches to, 11t in palliative care, 11t Anxiety disorders, 2347t, 2350-2353 clinical manifestations of, 2351 definition of, 2350 diagnosis of, 2351 epidemiology of, 2350-2351 generalized, 2351, 2351t illness anxiety disorder, 2355t pathobiology of, 2351 prognosis for, 2352-2353 social anxiety disorder, 2351t somatic manifestations of, 2351t treatment of, 1602t, 2352b types of, 2351t in women, 1602t, 1603 Anxiolytic drugs, 160-161 Aorta anatomy of, 492 layers of, 492 Aortic aneurysm, 492-494 abdominal CT findings, 493, 493f epidemiology of, 492, 492f

I9

Aortic aneurysm (Continued) risk factors for, 492 screening for, 55-56 treatment of, 493 ascending thoracic, 492 clinical manifestations of, 493 definition of, 492 diagnosis of, 493 epidemiology of, 492 pathobiology of, 492-493 prevention of, 494 prognosis for, 494 thoracic preferred drugs for, 394t treatment of, 493-494 thoracoabdominal, 492 treatment of, 493-494 Aortic arch anomalies, 415 normal anatomy, 2424, 2425f Aortic coarctation, 412 CMR findings, 292f diagnosis of, 385t exercise recommendations for, 416t Aortic diseases, 492-497 CMR findings, 289-292 Aortic dissection, 249t, 494-497, 496f acute, 395t classification of, 494, 494f diagnosis of, 495 echocardiographic findings, 280t-281t evaluation for, 495f incidence of, 494 pathobiology of, 494 prognosis for, 496-497 stroke in, 2443t TEE findings, 495, 495f treatment of, 395t, 495-496 Aortic hematoma, intramural, 494-497, 496f Aortic regurgitation, 462t, 470-471 acute, 471 asymptomatic, with normal left ventricular function, 471 chronic, 471 clinical manifestations of, 470 definition of, 470 diagnosis of, 470-471 echocardiographic findings, 471, 471f evaluation of, 279, 471 with left ventricular dysfunction, 471 medical therapy for, 471 pathobiology of, 470 physical findings, 470-471 surgical therapy for, 471 Aortic root dilation, 280t-281t Aortic stenosis, 461-465, 462t clinical manifestations of, 463-464 congenital valvular, 411-412, 412b diagnosis of, 464 Doppler echocardiography in, 464, 464f epidemiology of, 461 exercise recommendations for, 416t hemodynamics of, 294, 294f invasive therapy for, 464-465 low flow, low gradient, 464 measurement of, 279 medical therapy for, 464 pathobiology of, 461-463 pathology of, 461-463, 463f preoperative assessment of, 2615 prognosis for, 465 treatment of, 318, 412b, 464b-465b tricuspid, 461 Aortic valve, 264 bicuspid, 461 congenitally abnormal, 461 Aortic valve area calculation of, 279 formula for, 279 Aortic valve replacement, 464 percutaneous, 464-465 transcatheter, 465, 465f Aortic valve stenosis, congenital, 411-412 diagnosis of, 412 treatment of, 412b Aortic valvotomy, balloon, 465 Aortoenteric fistulas, 958 Aortography, 501f Aortopulmonary window, 410-411 Volume I  pp 1-1382  •  Volume II  pp 1383-2722

I10

Index

APACHE (Acute Physiology and Chronic Health Evaluation) II, 963 Apatite-associated arthropathy, 1816 Aphasia, 2384-2385 anomic, 2385t clinical manifestations of, 2384 definition of, 2384 diagnosis of, 2385 global, 2385, 2385t pathobiology of, 2384 primary progressive nonfluent/agrammatic variant of, 2385, 2397 semantic variant of, 2385, 2397, 2398f principal components of, 2384 treatment of, 2385b Aphasic syndromes, 2384-2385, 2385t Aphthae, 2579 Aphthous stomatitis, 2579-2580 clinical features of, 2580t forms of, 2579 periodic fever with aphthous stomatitis, pharyngitis, and cervical adenopathy, 1740t Aphthous ulcers, 2579, 2580f Apixaban, 178 for anticoagulation, 367 for DVT, 517 for PE, 625 pharmacokinetic characteristics and drug interactions, 178t preoperative management of, 2616, 2617t recommendations for thromboembolic prophylaxis for atrial fibrillation, 366t risk reduction with, 179t Aplasia, pure red cell, 1115 Aplasia cutis congenita, 2706 Aplastic anemia, 1114-1121 acquired, 1114-1115, 1118t acquired idiopathic prognosis for, 1121 severe, 1118t autoimmune, 1114-1115 causes of, 1114, 1114t clinical manifestations of, 1117 definition of, 1114, 1114f diagnosis of, 1117-1119 diagnostic considerations, 1119 diagnostic evaluation of, 1117-1118 diagnostic management of, 1118f differential diagnosis of, 1118-1119 drug-induced, 1115, 1115t drugs associated with, 1115, 1115t epidemiology of, 1114 future treatments, 1121 history in, 1117 incidence of, 1114 infections in, 1115 initial laboratory findings, 1117 management of infections, 1121 pathobiology of, 1114-1117 pathology of, 1114 physical findings, 1117 in pregnancy, 1116 prevention f, 1121 prognosis for, 1121 severe, 1120f supportive care for, 1117, 1121 toxins associated with, 1115, 1115t treatment of, 1079, 1119b-1121b future treatments, 1121 recommendations for, 1118t variants that must be treated differently, 1117 Apnea central, 2419, 2419.e1f obstructive, 638-642, 638f, 2419 sleep apnea, 2418-2419 Apocrine glands, 2634 Apolipoproteins, 1389, 1390t Apomorphine, 2457t-2459t Apoptosis, 686, 1230, 686.e2f Apoptosis regulators, 231.e1t Appearance, general, 252, 303 Appendicitis, 945-946, 946f acute differential diagnosis of, 946t scoring system for, 946t signs of, 946 clinical manifestations of, 946 diagnosis of, 946 differential diagnosis of, 946, 946t

Appendicitis (Continued) prognosis for, 946 treatment of, 946 Appetite, 1460, 1460t Apraclonidine, 2566 Apraxia, ideomotor, 2385 Apremilast, 2666t Aprepitant for nausea and vomiting, 865t for pruritus, 2636t Apriso (mesalamine), 939t Aquaporins, 738 Arachnida, 2173-2177 Arachnodactyly, 1735 Arachnoid cysts, 1292 Aranesp (erythropoietin), 1211t-1216t Arava (leflunomide), 1761 Arboviral encephalitis clinical manifestations of, 2263 diagnosis of, 2263-2264 differential diagnosis of, 2263-2264 distinguishing characteristics of, 2252t-2253t epidemiology of, 2262 features of, 2263t pathobiology of, 2262 prevention of, 2264 treatment of, 2264b vectors, 2172t Arboviral meningitis, 2490 Arboviruses, 2256-2262 CNS-affecting, 2262-2269 Arcalyst (rilonacept), 169-170 Arcanobacterium haemolyticum, 1918t, 2603 Arcapta (indacaterol), 553 Arcobacter butzleri, 1955-1956 Arcobacter cryaerophila, 1955-1956 Ardeparin, 177t ARDS Network, 664t Arenal insufficiency, primary, 1519-1520 Arenaviruses, 2247 Argatroban, 178, 1618t Argentine hemorrhagic fever, 91t, 2249t clinical manifestations of, 2250-2251 management of, 2255 prevention of, 91 treatment of, 91b Argentine hemorrhagic fever vaccine, 2255-2256 Arginine vasopressin, 742, 743f Arginine-GHRH test, 1483t Argininemia, 1386t Argininosuccinic aciduria, 1386t Argyll Robertson pupils, 2560, 2576-2577 Argyria, 97, 97f Arimidex (anastrozole), 1211t-1216t Aripiprazole for psychosis, 2354 for Tourette’s syndrome, 2466 Aristolochic acid nephropathy, 797 Arm muscle circumference (AMC), 1432, 1433t Arm surgery, 2618t Armodafinil, 2418, 2419t Arnold-Chiari malformations, 2513 Aromasin (exemestane), 1211t-1216t Aromatase defects, 1591 Aromatase deficiency, 1564 Arousal confusional, 2422-2423, 2423t disorders of, 2422-2423 Arrhythmia surgery, 380-381 Arrhythmias accelerated idioventricular, 451 approach to, 344-352 bradyarrhythmias, 346, 350t-351t, 355-367 clinical manifestations of, 344-346 diagnosis of, 346-350 diagnostic tests, 347-350 driving with, 351t life-threatening, 352-356 pharmacotherapy for, 354-355 preoperative assessment of, 2615 prognosis for, 355-356 re-entrant, 342-343, 343f sinus, 357f supraventricular, 326, 356-367 supraventricular tachyarrhythmias, 359-363, 452 tachyarrhythmias, 31, 346, 350t-351t treatment of, 309

Arrhythmias (Continued) triggered activity, 342 ventricular, 367-374, 451-452 Arrhythmogenesis, 342-343 Arrhythmogenic right ventricular cardiomyopathy, 321t, 331-332 clinical manifestations of, 331 definition of, 331 diagnosis of, 331 diagnostic criteria for, 333t-334t differential diagnosis of, 331 epidemiology of, 331 in first-degree relatives, 334t MRI findings, 331, 332f pathobiology of, 331 pathology of, 331 prognosis for, 332 treatment of, 331b-332b ventricular tachycardia in, 372, 372f ARRY-520, 1281 Arsenic poisoning, 94-95 chelators for, 93t clinical manifestations of, 95 diagnosis of, 95 diagnostic testing for, 93t epidemiology of, 94-95 pathobiology of, 95 prognosis for, 95 toxic levels, 703t treatment of, 95b, 706t-710t Arsenic trioxide (Trisenox) for acute promyelocytic leukemia, 1245 for cancer, 1211t-1216t for leukemia, 1243t Artemether, 2106, 2111t Artemether and lumefantrine (Coartem), 2105-2106, 2111t Artemisinins, 2111-2112 Arterial blood gases in asthma, 551 systemic analysis of, 652-653 Arterial blood volume, effective, 742 Arterial gas embolism, 597-598 Arterial gas embolism syndrome, 597 Arterial hypertension, 381-397 in acute ischemic stroke, 2441 pulmonary. See Pulmonary hypertension Arterial oxygen content (Cao2), 654 Arterial oxygenation, 657, 658f Arterial stimulation venous sampling, 1554-1555 Arteriovenous fistulas, dural, 2453 Arteriovenous malformations in brain clinical manifestations of, 2453 diagnosis of, 2453 epidemiology of, 2453 pathobiology of, 2453 prognosis for, 2453 in colon, 875-876, 875f spinal, 2380 Artery of Adamkiewicz occlusion, 2381 Artery of Percheron, 2427 Artesunate, 2106, 2111t Arthritis acute gouty, 1813, 1813f carcinomatous polyarthritis, 1826 chronic, 1174, 1174f enteropathic, 1768-1769 classification of, 1713t differential diagnosis of, 1764t fungal, 1808 in hemophilia, 1174, 1174f HTLV-associated, 2237t infectious, 1808-1809 inflammatory granulomatous inflammatory arthritis, dermatitis, and uveitis, 1740t, 1743 pathobiology of, 1828, 1828f peripheral, undifferentiated, 1714f joint, 1828 juvenile idiopathic, systemic-onset, 1740t Lyme, 1808, 2024 Mycoplasma, 1808 osteoarthritis, 1744-1749 classification of, 1713t distribution of involved joints, 1757f genetic components, 1732 pathobiology of, 1828f radiographic features of, 1724f, 1724t in women, 1603

Arthritis (Continued) palmar fasciitis and arthritis syndrome, 1826 pancreatitis-arthritis syndrome, 1824 pathobiology of, 1828 peripheral inflammatory, undifferentiated, 1714f polyarthritis, carcinomatous, 1826 psoriatic, 1768 classification of, 1713t differential diagnosis of, 1764t radiographic features of, 1724t pyogenic in bacterial meningitis, 2485 with pyoderma gangrenosum and acne, 1740t, 1743 reactive, 1766-1768 and chlamydia, 2010 classification of, 1713t differential diagnosis of, 1764t poststreptococcal, 1909-1910 radiographic findings, 1766f rheumatoid, 1754-1762 classification of, 1713t differential diagnosis of, 1764t hematopoietic stem cell transplantation for, 1203 inflammation pathways in, 230.e3t lung disease associated with, 584-585 ocular manifestations of, 2570 pleural fluid characteristics, 634t radiographic features of, 1724f, 1724t, 1727f septic, 1806-1808 anaerobic, 1933 antimicrobial therapy for, 1810t S. aureus, 1899 surgical treatment of, 1832-1833 systemic diseases associated with, 1823-1828 tuberculous, 1808-1809 viral, 1808 Arthrodesis, 1829-1830 Arthrogryposis, 2548t Arthropathy apatite (basic calcium phosphate)– associated, 1816 B19, 2213 basic calcium, 1811 calcium pyrophosphate dehydrate (CPPD), 1811 facet, 142t hemochromatotic, 1826, 1826f hypertrophic osteoarthropathy, 1221, 1221f, 1669, 1825, 1825f Jaccoud, 1771-1772 polyarthropathy syndrome, 2214, 2214t pyrophosphate, 1815 Arthroplasty resurfacing, 1831-1832, 1832f total joint, 1830-1832 Arthropod-borne viruses, 2262, 2263t Arthropods, 2171, 2171t disease vectors, 2171, 2172t reactions to bites and strings from, 2171, 2172f toxic effects, 2171, 2172t Arthroscopy, 1829 Arzerra (ofatumumab), 1211t-1216t Asacol (mesalamine), 939t Asbestos bodies, 593, 594f Asbestos disease, benign, 593 Asbestos exposure imaging of, 538f pleural effusions after, 635 Asbestosis, 589t, 593 Asbestos-related diseases, 593 Ascariasis, 2159-2160 clinical manifestations of, 2160 diagnosis of, 2160 epidemiology of, 2159-2160 prevention of, 2160 treatment of, 2160b, 2160t Ascaris lumbricoides, 1015t-1017t, 2103, 2159, 2159f Ascending thoracic aortic aneurysm, 492 Ascites abdominal findings, 978, 978f approach to, 979t cardiac, 1028t in cirrhosis, 1025-1026, 1028, 1028t, 1030 malignant, 949-950, 950f, 1028t, 1219

Index Ascites (Continued) pancreatic, 966, 966f treatment of, 1030 Ascitic fluid, 1028, 1028t Asenapine, 2354 Aseptic meningitis causes of noninfectious, 2492-2493, 2492t nonviral infectious, 2491t, 2492-2493 definition of, 2489 Asherman’s syndrome, 1589 Ashkenazi Jews, 1384, 1399-1400 Ash-leaf spots, 2699 Asia, 2274-2275 Asian Americans, 15 chronic kidney disease in, 833-834 osteoporotic fractures in, 1637-1638 population, 15 tularemia among, 1982 Asian hair, 2706 Asian snakes, 719 Asians access to health care, 15 normal pulmonary function values, 540 quality of health care, 15 suicide rates, 61 Ask-Tell-Ask approach, 10, 14t, 52 L-Asparaginase (Elspar), 1211t-1216t Aspartylglycosaminuria, 1387, 1403 Aspergilloma, pulmonary, 612, 2084, 612.e1f Aspergillosis, 2083-2087 allergic forms, 2083t clinical manifestations of, 2084 diagnosis of, 2085 treatment of, 2086 antifungal therapy for, 2086-2087 chronic, 2083t, 2086 classification of, 2083t clinical manifestations of, 2083-2084 definition of, 2083 diagnosis of, 2084-2085 disseminated, 2083 epidemiology of, 2083 extrapulmonary, 2083 invasive acute, 2083t antifungal therapy for, 2086, 2086t clinical manifestations of, 2083-2084 diagnosis of, 2084-2085 pulmonary, 2083 radiographic features of, 2084, 2085f treatment of, 2086b-2087b locally invasive, 2084 pleural, 2084 prevention of, 2087 prognosis for, 2087 pulmonary chronic, 2084 chronic cavitary, 2084 chronic necrotizing, 2084 invasive, 2083 tracheobronchial, 2083 treatment of, 2068, 2086 Aspergillus, 2083 Aspergillus empyema, 2083, 2086 Aspergillus endocarditis, 477t, 2083-2084 Aspergillus flatus, 2083 Aspergillus flavus, 2083 Aspergillus fumigatus, 2083 Aspergillus niger, 2083 Aspergillus otomycosis, 2084 Aspergillus pericarditis, 2083-2084 Aspergillus sinusitis acute, 2083 allergic, 2084 Aspergillus terreus, 2083 Aspiration definition of, 601 distinguishing features of, 528t Aspiration injury, 601 clinical manifestations of, 601 definition of, 601 diagnosis of, 601 pathobiology of, 601 prevention of, 601 prognosis for, 601 treatment of, 601b Aspiration pneumonia, 618 clinical manifestations of, 618 diagnosis of, 618

Aspiration pneumonia (Continued) epidemiology of, 618 microbiology of, 618 pathobiology of, 618 prognosis for, 618 treatment of, 618b ASPIRE dashboard (VA), 43.e1t Aspirin (acetylsalicylic acid), 172-175, 178-179 for acute coronary syndrome, 437, 438t-439t for acute MI, 448-449, 454t, 455f for angina, 427f, 429 anti-cancer effects of, 174 for antiphospholipid antibody syndrome, 1180 antithrombotic effects of, 174, 174t for atherosclerotic PAD, 501-502 clinical uses, 179 contraindications to, 1088t, 2257 COX-1 inhibition by, 173-174, 174f dosage adjustments in renal failure, 129t for essential thrombocythemia, 1125-1127 for fibromuscular dysplasia, 505 for frostbite, 510-511 and gastroduodenal ulcers, 909-910 for headache, 2357b for Kawasaki disease, 1797, 2672 for livedo reticularis, 506 long-term, 455f low-dose, 174 lowest effective dose, 174, 174t mechanism of action, 178-179 for migraine headache, 2358 after PCI, 458 pharmacokinetic parameters, 126t pharmacology of, 178-179 for polycythemia vera, 1125 for postoperative care with substitute heart valves, 472 preventive, 57, 1329, 1616, 2444 for renal artery stenosis, 808 for tension-type headache, 2360 for thromboangiitis obliterans, 505 for thrombotic complications, 1190 Aspirin hypersensitivity, 1699 Aspirin-exacerbated respiratory disease, 554, 1691, 1691t Aspirin-induced asthma, 554 Assessment. See also Risk assessment geriatric, 102-106 nutritional, 1430-1434 preoperative, 2611-2617 in psychiatric settings, 2625-2626 Assisted mechanical ventilation, 665 Assisted suicide, 7-8 definition of, 7-8 empirical data for, 8 history of, 7 justification for, 7-8 practical considerations, 8 safeguards, 8 Assistive reproductive technologies, 1596 Association measures, 33, 33.e1t Association studies, 190-191 Astemizole (Hismanal), 2660t Asterixis, 979 Asthma, 548-555, 1674-1675 aspirin-induced, 554 blood findings, 551 with C. pneumoniae infection, 2012.e1 characteristic components, 548 clinical manifestations of, 550 definition of, 548 diagnosis of, 550-551 differential diagnosis of, 551 ECG findings, 551 in emergency department care, 554 epidemiology of, 548 genetics of, 548-549 with heart failure, 316 history in, 550 inflammation pathways in, 230.e3t irritant-induced, 590 laboratory findings, 550-551 during lactation, 1619.e1f mediators of, 549 occupational sensitizer-induced, 588-590 suggestive features, 589t

Asthma (Continued) pathobiology of, 548-550 pathology of, 549 physical findings, 550 physiologic changes in, 549-550, 549f preconception interventions for women, 1612t preferred medications for women, 1602t in pregnancy, 554, 1617-1619 course of, 1617-1619 diagnosis of, 1619 differential diagnosis of, 1619 epidemiology of, 1617-1619 management of, 1619.e1f pathobiology of, 1619 treatment of, 1619b-1620b prevalence of, 548 prevention of, 551 prognosis for, 554 pulmonary function findings, 550, 550t, 550.e1f with pulmonary infection, 554 radiographic findings, 551, 551.e1f rescue treatments for, 551-552 sputum findings, 551 thoracic findings, 550 treatment of, 316, 551b-554b algorithm for, 552f control-driven therapy, 554 controller treatments, 552-554 preconception interventions for women, 1612t preferred medications for women, 1602t rescue treatments, 551-552 work-exacerbated, 590 work-related, 80, 80t, 588-590, 591f Astigmatism, 2558 Astrakhan fever, 2048t, 2050 Astrocytoma anaplastic, 1293 low-grade, 1293 spinal cord, 1297, 1297f Astrocytosis, Alzheimer’s type II, 1025 Astrovirus infection, 2244, 2245t Asymmetrical neuropathies, acute, 2535 Asystole, 355 Atarax (hydroxyzine), 2660t Ataxia adult-onset, 2469t cerebellar, 2341t, 2469-2470 alcohol-induced, 2511 hereditary, 2469-2470 fragile X, 2514 Friedreich’s, 325, 2469, 2469b sensory, 2341t spinocerebellar, 2469-2470 Ataxia-telangiectasia, 1227t, 1683t, 1684, 1684b Atazanavir, 2288t CSF-to-plasma ratios, 2331t for HIV/AIDS, 2289, 2290t side effects of, 2291 Atelectasis, 569-570 clinical manifestations of, 569-570, 569f definition of, 569 diagnosis of, 569-570 epidemiology of, 569 pathobiology of, 569 rounded, 569-570, 569f, 589t treatment of, 570b Atelvia (risedronate), 1642-1643 Atenolol for acute coronary syndrome, 438t-439t for acute STEMI, 450 for alcohol withdrawal, 155t for aortic dissection, 496 clinical use of, 430t for hypertension, 388t for ischemic heart disease, 2614 for premature atrial contractions, 359 for premature ventricular contractions, 372-373 for torsades de pointes, 374 for tremor, 2462 for ventricular tachycardia, 373 Atheroembolic disease, of renal artery, 810, 810b Atheromatous embolization, 506-507, 507t

I11

Atherosclerosis, 417-419 with C. pneumoniae infection, 2012.e1 in chronic kidney disease, 840 inflammation pathways in, 230.e3t lesions or plaque activation of, 418-419 formation of, 417-418, 418f growth and death of, 418 rupture, 418, 419f progression of disease, 418 risk factors for, 417 treatment of, 419, 840 Atherosclerotic peripheral arterial disease, 497-504 antiplatelet therapy for, 501-502 antithrombotic therapy for, 501-502 epidemiology of, 498 exercise therapy for, 501 imaging in, 500, 501f lower extremity, 497 pathobiology of, 498-499 pharmacology of, 502 prognosis for, 503 revascularization for, 502-503, 503f risk factor modification for, 501 treatment of, 500b-503b Atherothrombosis, 418, 419f Athetosis, 2463-2464 Athletes, 273-274, 274t Athlete’s heart, 274f Athletic pseudonephritis, 725 Atopic dermatitis, 2648f, 2663, 2663f in HIV infection, 2321 regional involvement, 2701f Atorvastatin for acute coronary syndrome, 438t-439t for acute MI, 451, 454t for atheromatous embolization, 507 for ischemic heart disease, 428 for lipid disorders, 1395-1397, 1396t for nonalcoholic fatty liver disease, 1022 for prevention of stroke, 2444-2445 Atovaquone for AIDS-associated opportunistic infections, 2297t-2301t for babesiosis, 2144 for malaria, 2111t for Pneumocystis pneumonia, 2096, 2096t, 2098, 2297t-2301t for Pneumocystis pneumonia in HIV infection, 2313t-2314t for toxoplasmic retinitis, 2566 for toxoplasmosis, 2129t Atovaquone and proguanil (Malarone) for babesiosis, 2144 for malaria, 1883, 2105, 2111t-2112t, 2112 ATP-binding cassette A1 (ABCA1), 1390t, 1391 ATR-16 syndrome, 1090-1091 Atractaspididae, 717 Atrial fibrillation, 361-363 ablation of, 380 coarse, 362f lone, 362 with preexcitation, 362f recent-onset, 365f recommendations for thromboembolic prophylaxis, 366t treatment of, 316, 365-366, 452 antithrombotic, 180t long-term, 366 preferred drugs for, 394t surgical, 380 vagally induced, 362 Atrial fibrillation–related stroke, 2445 Atrial flutter, 361, 362f long-term management of, 366 treatment of, 365 Atrial myxoma, 2443t Atrial premature beats, 359, 359f clinical manifestations of, 359 diagnosis of, 359 treatment of, 359 Atrial septal defect, 408 classification of, 408 color flow Doppler findings, 408, 408f diagnosis of, 408 ECG findings, 408, 408f exercise recommendations for, 416t secundum, 284f, 408.e1

Volume I  pp 1-1382  •  Volume II  pp 1383-2722

I12

Index

Atrial septal defect (Continued) treatment of, 409b ultrasonography in, 408, 408f Atrial septostomy, 402 Atrial tachycardia, 360, 360f, 360t acute therapy for, 363-366 long-term management of, 366 multifocal, 360f, 363 Atrioventricular block, 357t causes of, 358t complete, 357-358, 358f first-degree, 269, 357, 357f high-degree or advanced, 357-358, 358f management of, 452 Mobitz type I (Wenckebach), 346, 348, 357, 357f, 452 third-degree, 357-358, 358f treatment of, 358-359 Atrioventricular canal defect, 415 Atrioventricular conduction disturbances, 357-358 delayed conduction, 269 two-to-one conduction, 357, 358f Atrioventricular dissociation, 358 Atrioventricular junctional tachycardias, 360t Atrioventricular nodal disease, 350t-351t Atrioventricular nodal re-entrant tachycardia, 360, 360t, 362f, 363 acute therapy for, 363-366 diagnosis of, 363 long-term management of, 366 At-risk drinking criteria for, 150t definition of, 149-150 treatment of, 154 Atrophic lesions, 2693-2694, 2693f, 2693t, 2702t Atrophy, 2651t-2654t central, 2654f fat atrophy, 2653f skin, 2636t with telangiectasia, 2693t Atropine, 706t-710t for bradycardia, 355f, 452 for drug overdose, 699t-702t for heart block, 358-359, 452 for myocardial infarction, 448 for sinus bradycardia, 452 for sinus node dysfunction and AV block, 358-359 Attention span, 2383t Atypical absence, 2402 Aubagio (teriflunomide), 2477 Audiometry, 2594 AUDIT-C instrument, 57 Auditory disorders, 2593-2596 Auditory evoked potentials, 2344, 2345t, 2594 Auditory examination, 2593 Auditory pathway lesions, 2594 Auditory seizures, 2401t Auer rods, 1058, 1058f Auerbach’s (myenteric) plexus, 896-897, 2517 Aura, 2400-2401 Auscultation cardiac, 253-254 of lungs, 526, 526t Australasian snakes, 720 Australian elapids, 718 Autism, 2514, 2514t Autism spectrum disorder, 2514 Autoantibodies associated with seropositive systemic autoimmune diseases, 1716t associated with systemic lupus erythematosus, 1770, 1771f, 1775t in systemic sclerosis, 1780-1781, 1780t thyroid, 1501 without disease specificity, 1716t Autografts, heart valve, 472t Autoimmune acquired hypoparathyroidism, 1660 Autoimmune aplastic anemia, 1114-1115 Autoimmune autonomic ganglionopathy, 2518t Autoimmune autonomic neuropathies, 2518t Autoimmune destruction, 1519 Autoimmune disease and amenorrhea, 1591 and bronchiectasis, 567 classification of, 227t

Autoimmune disease (Continued) eye disorders, 2570-2571 hematopoietic stem cell transplantation for, 1203 in HIV infection, 2335 in immunocompromised patients, 1855t, 1856 immunomodulatory therapy for, 212-213 of neuromuscular transmission, 2547-2552, 2548t pathogenesis of, 219-220 polyglandular syndromes, 1556-1557, 1556t preconception interventions for women, 1612t preferred medications for women, 1602t systemic, 1713t, 1715t-1716t Autoimmune hemolytic anemia, 1073-1076 characteristics of, 1075t in chronic lymphocytic leukemia, 1256 classification of, 1075-1076 clinical manifestations of, 1075 cold or cold-reactive, 1075-1076 acute, 1076 classification of, 1076 management of, 1077f cold transient, 1075t definition of, 1073 diagnosis of, 1075-1076 epidemiology of, 1073-1074 incidence of, 1073-1074 mixed-type, 1075, 1075t pathobiology of, 1074 subtypes, 1075, 1075t supportive care for, 1076 treatment of, 1076b-1078b warm or warm-reactive, 1075-1076 characteristics of, 1075t pathobiology of, 1074 recommended workup, 1076t secondary, 1076, 1076t treatment of, 1077-1078, 1078f Autoimmune hepatitis, 1006 arthritic manifestations of, 1823 clinical manifestations of, 1006 diagnosis of, 1000t, 1006 epidemiology of, 1006 liver transplantation for, 1035 pathobiology of, 1006 prognosis for, 1006 treatment of, 1006b Autoimmune hypocalciuric hypercalcemia, 1657 Autoimmune hypoparathyroidism, polyglandular, 1652t, 1660 Autoimmune inner ear disease, 2599 Autoimmune lymphoproliferative syndrome, 1074, 1686t Autoimmune neuromyotonia, 2519 Autoimmune neutropenia, secondary, 1135 Autoimmune pancreatitis, 960t, 961, 964 Autoimmune pericarditis, 484t, 491 Autoimmune polyendocrinopathy candidiasis ectodermal dystrophy (APECED) syndrome, 1660, 1686t Autoimmune polyglandular syndrome type 1, 1556-1557 clinical manifestations of, 1557 definition of, 1556 pathogenesis of, 1557 prognosis for, 1557 treatment of, 1557b Autoimmune polyglandular syndrome type 2, 1557 clinical manifestations of, 1557 pathogenesis of, 1557 prognosis for, 1557 treatment of, 1557b Autoimmune rheumatic diseases, 1715 Autoimmune testicular failure, 1573 Autoimmune (Hashimoto’s) thyroiditis, 1502-1503 Autoimmune vasculitis, 2655f Autoimmunity, 225-226, 1700 neurologic, 2518-2519, 2518t rheumatic disease associated with, 1713t in systemic sclerosis, 1780-1781 Autoinflammation, lipodystrophy, and dermatosis syndrome, 1740t Autoinflammatory diseases familial cold, 231, 1740t, 230.e3t IL-1–associated, 1742-1743

Autoinflammatory diseases (Continued) inherited, 1740t rheumatic, 1713t diagnostic clues, 1715t patterns of onset, 1715 of skin and bone, 1740t systemic, 1739-1744, 1740t Autoinflammatory syndromes, 1740t, 1743-1744 Autologous bone marrow mononuclear cells, 451 Autologous stem cell transplantation, 1279-1280 Automated external defibrillators, 353, 354t Automaticity, abnormal, 342 Autonomic disorders, 2517-2522 bedside evaluation of, 2520 clinical manifestations of, 2519-2520 diagnosis of, 2520-2521 epidemiology of, 2517 laboratory evaluation of, 2520-2521 pathobiology of, 2517 prognosis for, 2522 treatment of, 2521b-2522b Autonomic failure generalized, 2519 pure, 2518 Autonomic nervous system, peripheral, 2518 Autonomic nervous system dysfunction, 1930 Autonomic neuropathy autoimmune, 2518t paraneoplastic, 2518t, 2519 peripheral, 2518t toxins that cause, 2520 Autonomic syndromes acute, 2519 chronic, 2519 selective, 2519 Autonomous Detection System, 1922 Autonomy, 4 patient, 3 Auto-PEEP, 667, 667f Autosomal dominant muscular dystrophies, 2541t Autosomal dominant polycystic kidney disease, 816-820 clinical features of, 817t, 818-819 cyst formation in, 816, 818f diagnosis of, 819, 819f epidemiology of, 816 experimental therapy for, 820 gene products in, 818 genetics of, 816-817 pathobiology of, 816-818 pathology of, 816f prevention of, 819b-820b prognosis for, 820 treatment of, 819b-820b Autosomal recessive muscular dystrophies, 2541t Autosomal recessive polycystic kidney disease, 820-821 clinical features of, 817t, 820 definition of, 820 diagnosis of, 819f, 820-821 epidemiology of, 816 pathobiology of, 820 pathology of, 816f prevention of, 821b prognosis for, 821 treatment of, 821b Avastin (bevacizumab), 170, 213, 1211t-1216t Avellis’ syndrome, 2435.e2f Avian H1N1, 2056-2057, 2197 Avian H5N1, 2056-2057, 2060t, 2197 Avian H7N9, 2057, 2197 Avobenzone (Parsol 1789), 2661 Avoidant personality disorder, 2355t Avoiding Cardiovascular Events through Combination Therapy in Patients Living with Systolic Hypertension (ACCOMPLISH) Trial, 392, 392.e1f Avonex (interferon β1a), 2477 Awaji-Shima Consensus Recommendations, 2524t Awareness of environment, 2409 of self, 2409 Axial disease, 1717 Axial dystonia, 2463-2464

Axillary lymph node metastasis, 1379-1380 Axiron (testosterone), 1574t Axitinib (Inlyta), 170, 1211t-1216t Ayurveda, 182t 5-Azacitidine (Vidaza) for cancer, 1211t-1216t for myelodysplastic syndrome, 1238 Azalides, 1889t Azatadine, 2660t Azathioprine (Imuran), 165-166, 1761 adverse side effects of, 165-166, 1037t for autoimmune hepatitis, 1006 for Behçet’s syndrome, 1799 for bullous pemphigoid, 2675 for Crohn’s disease, 940 for epidermolysis bullosa acquisita, 2676 for giant cell arteritis, 1804 for glomerulonephritis, 791 for granulomatosis with polyangiitis, 2534 indications for, 165 for inflammatory bowel disease, 939t, 940 for lupus nephritis, 792 mechanism of action, 165 monitoring, 1037t for multifocal motor neuropathy, 2532 for myasthenia gravis, 2551 for myocarditis, 328-329 for ocular myasthenia gravis, 2551 for pemphigus, 2677 for polymyalgia rheumatica, 1804 for pruritus, 2636t in renal transplantation, 844t for sarcoidosis, 607t for systemic lupus erythematosus, 1776 for systemic vasculitis, 2534 Azelaic acid gel, 2680 Azidothymidine (AZT) (zidovudine), 2287, 2288t for adult T-cell leukemia/lymphoma, 2238 CSF-to-plasma ratios, 2331t fixed-dose combinations, 2289, 2290t side effects of, 2289, 2291 Azithromycin, 1894 for AIDS-associated opportunistic infections, 2297t-2301t for babesiosis, 2144 for bacterial meningitis prophylaxis, 2489 for bronchectasis, 569 for bronchitis and tracheitis, 609 for C. pneumoniae infections, 2012 for cat-scratch disease, 2000 for chancroid, 1950 for chlamydia, 2011, 2565 for cholera, 1952 for COPD, 560 for diarrhea, 922 for enteric fever, 1974 for gonorrhea, 1944-1945, 1945t, 2603 for granuloma inguinale, 2002 for legionnaires’ disease, 1996t for leptospirosis, 2030 for meningococcal disease, 1938, 1939t for Mycoplasma pneumonia, 2006 for Mycoplasma prophylaxis, 2006 for nonsyphilitic treponematoses, 2021 for pertussis, 1992 for pharyngitis, 2602 for pneumonia, 616t for prophylaxis in HIV infection, 2295t for prostatitis, 831-832 for pulmonary infections, 565 recommended doses and schedules, 1891t-1892t for S. pneumoniae infection, 1905, 1905t for septic shock, 689t for shigellosis, 1978 for spondyloarthritis, 1764 for syphilis, 2018 for toxoplasmosis, 2129t for trachoma, 2008 for traveler’s diarrhea, 1884 for urethritis, 1878t Azmacort (triamcinolone acetonide), 553t Azole antifungal agents, 2066-2069 Azotemia postoperative, 2624 prerenal, 779, 781 Aztreonam for bacterial meningitis, 2488t mechanism of action, 1889t for P. aeruginosa bacteremia, 1966 for P. aeruginosa pneumonia, 1966-1967

Index Aztreonam (Continued) for pneumonia, 616t, 1966-1967 for pulmonary infections, 565 recommended doses and schedules, 1891t-1892t resistance to, 1890t Azulfidine (sulfasalazine), 939t, 1761 Azurophil granules, 1144, 1144t

B

B blood group antigens, 238 B cells, 223-225 biologic agents targeting, 171-172 adverse effects of, 172 indications for, 172 mode of action, 171-172 development of, 223-224, 224f differentiation of, 224-225, 224f in graft rejection, 236t hypogammaglobulinemia with, 1679-1680 stimulation of, 224 in systemic sclerosis, 1780-1781 T- and B-cell combined defects, 1678-1679, 1678t prognosis for, 1679b treatment of, 1679b B7.1 (CD80), 222 B7.2 (CD86), 222 B19 arthropathy, 2213 B19 parvovirus infection, 2212 chronic pure red cell aplasia secondary to, 2213, 2213f clinical manifestations of, 2212-2214 congenital, 2214, 2214t diagnosis of, 2214, 2214t epidemiology of, 2212 pathobiology of, 2212 persistent, 2213 prevention of, 2214 syndromes of, 2213-2214 treatment of, 2214b Babesia, 2142 Babesia microti, 1015t-1017t, 2143, 2143f-2144f Babesiosis, 2142-2147 clinical manifestations of, 2143 diagnosis of, 2143-2144, 2143f-2144f epidemiology of, 2142-2143 in immunocompromised patients, 2144 pathobiology of, 2143 prevention of, 2144-2145 prognosis for, 2145 treatment of, 2106, 2144b vectors, 2172t Babinski’s sign, 2372t, 2469, 2473 BabyBIG (botulism immune globulin), 1928 Bacillary angiomatosis, 1999-2001, 1999f diagnosis of, 2000t in HIV infection, 2319 treatment of, 2001 Bacillary dysentery, 2252t-2253t Bacille Calmette-Guérin–associated IRIS, 2334, 2334f Bacillus anthracis, 1920 Bacillus anthracis meningitis, 2487 Bacillus cereus infection, 1869t Bacitracin for eye infections, 2565t polymyxin B and bacitracin (Polypore), 2657 Back pain, 142, 2372-2375. See also Low back pain clinical examination in, 2373 clinical manifestations of, 2372-2373 diagnosis of, 2373-2375 epidemiology of, 2372 history in, 2373 pathobiology of, 2372 prognosis for, 2375 treatment of, 2375b Backwash ileitis, 938 Baclofen for complex regional pain syndrome, 2522 for dystonia, 2465 for hereditary spastic paraplegias, 2470 for stretching, 2476 for trigeminal neuralgia, 2363 BACTEC MGIT 960 system, 2036-2037

Bacteremia Acinetobacter, 1968-1969 anaerobic, 1932 antibiotic regimens for, 689t definition of, 685 enterococcal, 476, 1914-1915, 1915.e1t H. influenzae, 1947 P. aeruginosa, 1964, 1966 pneumococcal, 1902 Salmonella, 1973-1974 staphylococcal, 1898, 1898t Bacteria anaerobic, 1931-1932, 1931f definition of, 1931 location of, 1931, 1931t non–spore-forming, 1931-1934 taxonomy, 1931.e3t virulence factors, 1932, 1932t blood smear, 1059t ESKAPE group, 1963 human microbiome, 1838-1843 protective effects of, 1837 Bacterial conjunctivitis, 2564t, 2565, 2565f Bacterial endocarditis differential diagnosis of, 2484 therapy for, 479t Bacterial esophagitis, 907 Bacterial infections. See also specific infections cutaneous, 2695-2697 dominant species, 1885t fever in, 1849, 1849f hepatic, 1012 in HIV infection, 2319-2320 in immunocompromised patients, 1859t of liver, 1011-1012, 1018t preferred medications for women, 1602t primary sites, 1885t susceptibility to, 1844, 1886 transfusion contamination, 1196 Bacterial killing, 1146 laboratory evaluation of, 1146 oxygen-dependent, 1149-1150 Bacterial killing assays, 1146 Bacterial liver abscesses, 1011t Bacterial meningitis, 2480-2495. See also specific bacteria abnormalities in, 2484 antibiotic regimens for, 689t blood cultures, 2484 cell counts, 2483, 2483t chemoprophylaxis of, 2489 clinical manifestations of, 2481-2483, 2482t complications of, 2484-2485 neurologic, 2482-2483 septic, 2485 CSF findings, 2483-2484, 2483t CSF formula, 2342t definition of, 2480 diagnosis of, 2483-2485 differential diagnosis of, 2484 epidemiology of, 2480-2481 focal cerebral signs, 2482-2483 glucose levels, 2483, 2483t gram-stained smear findings, 2483 history in, 2481-2482 laboratory findings, 2483-2484 neurologic findings, 2482-2483 pathobiology of, 2481 pathogenesis of, 2481 pathology of, 2481 physical findings, 2482 prevention of, 2489 prognosis for, 2489 protein levels, 2483, 2483t radiologic studies, 2484 recurrent, 2485 respiratory tract cultures, 2484 special testing procedures for, 2483 supportive care for, 2489 treatment of, 2485b-2489b, 2487t adjunctive corticosteroids, 2488 antimicrobial, 2485, 2488t duration of therapy, 2487-2488 empirical, 2485 surgical, 2489 vaccination against, 2489 zoonotic, 2487 Bacterial otomastoiditis, chronic, 2599

Bacterial overgrowth abnormalities conductive to, 926t clinical manifestations of, 928 diagnosis of, 928 identification of, 887 intestinal, 928 tests for, 925-926, 925t treatment of, 928b Bacterial pericarditis, 485t, 491 Bacterial peritonitis, spontaneous in cirrhosis, 1025-1026, 1028, 1030 treatment of, 1030 Bacterial pharyngitis, nonstreptococcal, 2602-2603 Bacterial pneumonia atypical, 2252t-2253t HIV-associated, 2306-2311 clinical manifestations of, 2310 diagnosis of, 2310 epidemiology of, 2306-2310 prevention of, 2311 radiographic features of, 2309t treatment of, 2310b-2311b imaging of, 533f recurrent, 2306-2310 Bacterial prostatitis acute causative organisms, 830 classification of, 831t clinical manifestations of, 831 prognosis for, 833 treatment of, 831 chronic classification of, 831t clinical manifestations of, 831 treatment of, 831-832 Bacterial rhinosinusitis, 2589t Bacterial sinusitis treatment of, 1896 when to refer to an ENT specialist, 2589t Bacterial tonsillitis, 2602t Bacterial vaginosis, 1877t, 1879-1880 Bactericidal antagonism, 1847-1848, 1847f Bactericidal synergism, 1847, 1847f Bacteriuria, asymptomatic, 1875 Bacteroides, 1931, 1934t Bacteroides fragilis, 1931, 1934t Bactrian (mupirocin), 2657 Bactrim (trimethoprim-sulfamethoxazole), 2147 Baker’s cysts, 1753, 1757-1758, 1757.e1f Balamuthia, 2142 Balamuthia mandrillaris, 2142t Balance: loss of, 2340 Balance activity, 58-59 Balance assessment, geriatric, 104-105 Balance training, 59 Balantidium coli, 2147t Balkan nephropathy, 797 Ballism, 2463, 2463t Balloon aortic valvotomy, 465 Balloon catheters, 456-457, 457f Balloon cells, 2512 Balloon enteroscopy, laparoscopic-assisted, 1326-1327, 1322.e1 Balloon pumps, intra-aortic, 453, 683, 684f Balloon-expandable coronary stents, 456-457, 457f Balsalazide (Colazal), 939t Bambuterol (Bambec, Oxeol), 553 Band cells, 1143-1144 Band heterotopia (double cortex), 2512 Barakat syndrome, 1652t, 1660, 1661.e1 Barbecue roll, 2601t Bardet-Biedl syndrome, 817t, 821 Bariatric surgery, 1466 Barium enema, double-contrast, 1329 Barlow’s type valve, 467.e1f Barmah Forest virus, 2261 Baroreflex failure, 2519 Barotrauma, 597-598, 2590, 2591f clinical manifestations of, 598 otic, 598 pathobiology of, 597 pulmonary, 597-598, 597.e1f Barrett’s esophagus, 900-902 clinical manifestations of, 900-901 diagnosis of, 901 endoscopic findings, 874, 874f, 898, 899f, 901

I13

Barrett’s esophagus (Continued) epidemiology of, 900 in gastroesophageal reflux disease, 898 long-segment, 1316 prognosis for, 902 short-segment, 1316 treatment of, 901b-902b, 901f Barrier contraceptives, 1606-1607, 1880 Barth’s syndrome, 335, 760, 773, 1134 Bartonella, 1996-1997, 1997t Bartonella endocarditis, 477t, 1999 Bartonella infections, 1996-2001 clinical manifestations of, 1998, 1998t clinical syndromes, 1997 cultures of, 2000 diagnosis of, 1999-2000 differential diagnosis of, 2000 epidemiology of, 1997 in HIV infection, 2319 immunofluorescence of, 2000 immunohistochemistry of, 2000 laboratory tests for, 1999, 2000t microscopic findings, 2000 molecular detection methods, 2000 prevention of, 2001 prognosis for, 2001 of red blood cells, 1997-1998, 1998f serologic tests for, 1999-2000 treatment of, 2000b-2001b vasculoproliferative disease, 1999-2001 vectors, 2172t Bartter-Gitelman disorders, 823-824 Bartter’s syndrome, 383, 745, 772, 823t, 824 pathophysiology of, 823-824, 824.e1f treatment of, 824 Bartter’s syndrome type 5, 823-824 Basal cell carcinoma, 1377-1378, 1377f, 2562 clinical manifestations of, 1377, 1377f diagnosis of, 1378 epidemiology of, 1377 nodular, 2562f pathobiology of, 1377 risk factors for, 1377 treatment of, 1378 Basal energy expenditure (BEE), 1433 Basal metabolic rate (BMR), 1460-1461 Base deficit, 675 Basement membrane, 1730t, 1731 Basement membrane zone, 2634 Basic calcium arthropathy, 1811 Basic calcium phosphate, 1811 Basic calcium phosphate crystals, 1816 Basic calcium phosphate deposition disease, 1816, 1816t Basic calcium phosphate–associated arthropathy, 1811, 1816 pathobiology of, 1816 treatment of, 1816b Basic life support after acute poisoning, 705 CAB (compression-airway-breathing) algorithm for, 353 after cardiac arrest, 353 Basic multicellular units (BMUs), 1639f, 1646 Basilar artery, 2426, 2428f Basilar artery occlusion, 2435, 2435t Basilar artery tip occlusion, 2435, 2435t Basiliximab, 844-845, 844t Basophilic stippling, 1054f Basophilopenia, 1138 Basophils, 217, 1688 Bat-borne viruses, 2247 Bath salts, 723 Batson’s venous plexus, 1809 Batteries, pacemaker, 375, 375.e1f Battering, 1629-1630 Battle’s sign, 2365-2366, 2482 Bayes’ theorem, 38 odds ratio form, 38, 38t Baylisascariasis, 2165 Baylisascaris procyonis, 2165 Bazedoxifene, 1626 Bazett’s formula, 269 Bazex syndrome (acrokeratosis neoplastica), 2670 B-cell disorders, 1676-1677 B-cell immunodeficiency, 2243-2244 B-cell lymphoblastic leukemia acute, mature, 1243-1244 classification of, 1240t Volume I  pp 1-1382  •  Volume II  pp 1383-2722

I14

Index

B-cell lymphoma, 1263-1265 clinical characteristics of, 1261t diffuse large B-cell, 1264-1265, 2324 rare types, 1265 B-cell malignancy, 1258, 1259f B-cell receptor defects, 1680t B-cell receptor signaling pathway inhibitors, 1256 B-cell-activating factor, 219, 224 B-cell-specific activating protein, 1269 B-complex vitamins, 931t Beau’s lines, 2707, 2708f, 2708t Becker’s muscular dystrophy, 2542-2543 clinical features of, 2542-2543, 2542t diagnosis of, 2542-2543 dilated cardiomyopathy in, 329 female carriers of, 2543 prognosis for, 2543 treatment of, 2543b Becker’s myotonia congenita, 2544 Beclomethasone (Qnasl), 554, 1692, 1692t Beclomethasone dipropionate (QVAR), 553t Bedaquiline, 2038 Bedbugs, 2176, 2176f-2177f Bedouins, African, 1133 Bedside maneuvers for BPPV, 2598, 2598f Bedside radiography, 531-532 Bedside tests, 2520 of hearing, 2594 of vertigo, 2599-2600 Bee stings, 2176-2177 Beetle-related dermatitis, 2172t, 2176-2177 Behavior alcohol-related complications, 151t, 153 effects of microbiota on, 1842-1843 personal, 2386-2388 Behavior change counseling for, 52-54 prevention against travel-related health problems, 1882t for weight loss, 1465 Behavioral change specialists, 54 Behavioral interventions, 57 for hot flushes, 1625 for incontinence, 113-114 for insomnia, 2421t for prevention of STIs, 1880 for tobacco dependence, 146-147, 147t Behavior-variant frontotemporal dementia, 2396, 2397t Behçet’s disease, 1740t classification of, 1713t esophageal ulcers in, 906 gastrointestinal involvement, 957 small intestinal ulcers in, 948 Behçet’s syndrome clinical manifestations of, 1799 epidemiology of, 1794-1795, 1794t oral ulcers in, 2579, 2580t treatment of, 1799 Bejel, 2020-2021 Belatacept (Nulojix), 171, 844t Belching (eructation), 859 Belgium, 7-8 Belimumab (Benlysta), 171-172, 1721, 1775-1777 Belladonna, 2521 Bell’s palsy, 2537 clinical manifestations of, 2537 diagnosis of, 2537 prognosis for, 2537 treatment of, 2537b Benadryl (diphenhydramine), 2660t, 2684t Benazepril, 388t Bence Jones proteinemia, 1274 Bence Jones proteinuria in glomerular syndromes, 734 idiopathic, 1275-1277 Bendamustine (Treanda), 1211t-1216t for chronic lymphocytic leukemia, 1255 for non-Hodgkin’s lymphoma, 1263, 1263t Bendopnea, 302 Benedikt’s syndrome, 2435.e1f Beneficence, 4, 7-8 Benign blue nevus, 2690f Benign breast lesions, 1359-1360 nonproliferative, 1359 proliferative, 1359 Benign capillary hemangioma, 2690, 2690f Benign cramp-fasciculation syndrome, 2524t Benign familial neutropenia, 1133 Benign migratory glossitis, 2582

Benign paroxysmal positional vertigo, 2598-2599 bedside maneuvers for, 2598, 2598f treatment of, 2601t Benign prostatic hyperplasia, 827-830 clinical manifestations of, 828 definition of, 827 diagnosis of, 828-829 epidemiology of, 827-828 laboratory findings, 829 pathobiology of, 828 prognosis for, 830 treatment of, 829b-830b algorithm for, 829f medications for, 830, 830t surgical, 830 Benign recurrent intrahepatic cholestasis, 987-988 Benign rolandic epilepsy, 2406 Benlysta (belimumab), 171-172 Benznidazole (Rochagan, Radinil), 2118 Benzodiazepine for anxiety, 2352t screening for, 699-702, 702t Benzodiazepine abuse, 160-161, 161b Benzodiazepine overdose, 706t-710t Benztropine, 2457t-2459t Benzyl benzoate, 2173 Bepridil, 430t Beri-beri, 2506 Berinert (C1 inhibitor concentrate), 1697 Bernard-Soulier disease, 1167 Bernard-Soulier syndrome, 1057-1058, 1171 Bernouilli equation, simplified, 276 Berry aneurysms, 2447 Beryllium disease, chronic, 589t, 592-593 Beryllium exposure, potential, 592t Best practices, 44 Beta particles, 82, 83f Beta rays, 83f β-Blockers. See β-Adrenergic receptor blockers Betaine, 1407 Betaine supplements, 1407 Betamethasone, 164t Betamethasone benzoate, 2658t Betamethasone dipropionate (Diprolene, Diprosone), 2658t Betamethasone valerate, 2658t Betaseron (interferon β1b), 2477 Betaxolol for glaucoma, 2566 for hypertension, 388t Bethanechol, 2238-2239, 2477 Bethesda System for Reporting Thyroid Cytopathology, 1511, 1512t Bethesda units, 1176 Betrixaban, 178 Bevacizumab (Avastin), 170, 213, 1232t for cancer, 1211t-1216t, 1310t, 1319, 1330-1331 for hearing, 2515 Bexarotene (Targretin), 2658, 2670t Bexosero, 1939 Bezold’s abscess, 2489 Bias, 36 Bicalutamide (Casodex), 1211t-1216t, 1369 Bicarbonate (HCO3−), 652 in acid-base balance, 763 for DKA and HHS, 1541f, 1542 for metabolic alkalosis, 773 regulation of, 739-740 for renal tubular acidosis, 771 Biceps reflex, 2372t Bichloroacetic acid, 2222t Bicipital tendinitis, 1752 Biclonal gammopathies, 1275 Bifidobacterium, 2060 Bifidobacterium infantis, 893t Bile acid diarrhea, 928 Bile acid malabsorption, 933 Bile acid sequestrants, 1396, 1396t Bile duct diseases, 1042-1048 Bile duct stones, 1042-1044 clinical manifestations of, 1042 complications of, 1043-1044, 1043f CT findings, 1042, 1043f diagnosis of, 1042, 1043f endoscopic removal of, 877, 877f pathobiology of, 1042 prevention of, 1044 treatment of, 1042b-1044b

Bile duct tumors, 1047 Bile salt export pump, 987-988 Bile salt malabsorption, 928 Bile salt wasting, 928-929 Bile salts, 928 Bilharziasis, 2153-2155 Biliary cirrhosis, primary, 1018t, 1046-1047, 1823 Biliary disease, 877 Biliary fistula, 1046 Biliary pancreatitis, 1043f Biliary sphincterotomy, 877, 877f Biliary strictures, benign, 1044-1046, 1044f-1045f Biliary tract, 1340f Biliary tract cancers, 1339-1345 clinical manifestations of, 1342 definition of, 1339 diagnosis of, 1342-1343 epidemiology of, 1340 pathobiology of, 1341-1342 prevention of, 1345 primary, 1340t prognosis for, 1345 staging, 1342-1343 treatment of, 1344 Biliary tract disease, 987-991 Bilirubin disposition of, 983-984 excretion of, 984 hepatic translocation of, 983-984 kinetics of, 985 metabolism of, 983-984, 984f, 987t plasma levels, 984-985 unconjugated concentration (UCB), 985 Bilirubin clearance (CBR), 985, 990 Bilirubin conjugation acquired defects, 988 genetic disorders of, 985-987 impaired, 985 Bilirubin production, 983 Bilirubin production, increased, 985 Bilirubin production rate (BRP), 985 Bilirubin uptake, decreased, 985 Bilophila wadsworthia, 1931 Binge drinking, 149-150, 2512 Binge eating disorder, 1455-1457 definition of, 1455 diagnostic criteria for, 1455, 1456t natural history of, 1457 pathobiology of, 1456 prognosis for, 1457 risk factors for, 1456 symptoms and signs of, 1456-1457 treatment of, 1457 Bioavailability, 124 Biodosimetry, 84-85 Bioethics, 4-9 Biofeedback, 182t Bioidentical hormones, 1626 Biologic agents, 169-172 for cancer, 1211t-1216t, 1330, 1358 disease-modifying antirheumatic drugs, 1762 for inflammatory bowel disease, 939t, 940 perioperative, 1833 for Sjögren’s syndrome, 1789 for systemic lupus erythematosus, 1776-1777 Bioprostheses, 472t, 473f Biopsy endomyocardial, 298 EUS-guided, 877-878 fine-needle aspiration, 1511 gonadal, 1567 in leprosy, 2045 liver, 991 myocardial, 305 renal, 735 sentinel lymph node, 1376 skin, 2656-2657 small bowel, 925t in systemic mastocytosis, 1708 temporal artery, 1804 Bioterrorism, 86-92 agents of concern, 87, 87t, 1928 epidemiologic clues to, 87, 87t history of, 86 prevention of anthrax due to, 1922 response to attack, 91-92 vaccines against, 78

Biotin, 1446t-1450t Biotransformation, 1006-1007 BioWatch, 1922 Bioweapon agents, 1928 Bipolar disorder, 2348-2350 clinical manifestations of, 2347 definition of, 2348 diagnosis of, 2347 epidemiology of, 2348 molecularly guided therapeutics for, 202.e1t pathobiology of, 2348 prognosis for, 2350 treatment of, 2350b Bird fancier’s lung, 576-577 Bird’s beak deformity, 903, 903f Bisacodyl, 889 BISAP score, 963 Bismuth compound, 915t Bismuth encephalopathy, 96, 97b Bismuth poisoning, 96 clinical manifestations of, 96 diagnosis of, 96 diagnostic testing for, 93t Bismuth subsalicylate (Pept-Bismol), 922, 934-935 Bisoprolol clinical use of, 430t for heart disease, 2614 for heart failure, 311t for hypertension, 388t Bisphosphonates, 1642-1643 for Paget’s disease, 1665-1667, 1667t for well-being, 2478 Bite cells, 1087 Bitot spots, 1446t-1450t Bivalirudin (Hirulog), 178 for acute coronary syndrome, 438t-439t, 439 for acute MI, 448, 450 safety and pharmacokinetics in pregnancy, 1618t Black death, 89, 1987f Black hair, 2706 Black piedra, 2102 Black skin, 2700, 2700t Black snakes (Pseudechis), 720 Blacks, 15. See also African Americans access to health care, 15 population, 15 risks for atherosclerotic cardiovascular disease, 260f-261f Bladder abnormalities, 827 Bladder cancer, 1348-1351 clinical manifestations of, 1349 definition of, 1348, 1350t diagnosis of, 1349 epidemiology of, 1348-1349 grading, 1349 metastatic disease, 1351 muscle-invasive tumors, 1350-1351 non–muscle-invasive tumors, 1349-1350 pathobiology of, 1349 prevalence of, 1348 prognosis for, 1351 staging, 1349, 1350t treatment of, 1349b-1351b Bladder diary, 112-113, 112f Blast injury, 712t, 713, 713f Blastocystic hominis, 2147t Blastomyces dermatitidis, 2074, 2074f Blastomycosis, 2074-2075 clinical manifestations of, 2074 definition of, 2074 diagnosis of, 2074 disseminated infection, 2074 epidemiology of, 2074 pathobiology of, 2074 prognosis for, 2075 pulmonary, 2074 treatment of, 2074b-2075b Blatchford scoring system, 916t Blau’s syndrome, 1740t, 1743 Bleaching, 2661 Bleeding. See also Hemorrhage approach to, 1154-1159 characteristic patterns, 1156t dry, 1160 gastrointestinal, 879-884 in HIV infection, 2303t obscure and occult, 882-884 hemophilia-related, 1174

Index Bleeding (Continued) in liver failure, 1183-1184 clinical manifestations of, 1183-1184 diagnosis of, 1184 pathobiology of, 1183 lower gastrointestinal, 879-880, 882 of qualitative platelet disorders, 1171-1172 small intestinal, 882-883, 883t upper gastrointestinal, 879-880 causes of, 879, 880t endoscopy in, 874-875 hospitalization rate, 879-880 management of, 881f mortality rate, 879-880 treatment of, 880b-881b uterine, abnormal, 1587-1588 variceal, 1029-1030 wet, 1160 Bleeding diathesis, 407, 407f Bleeding disorder panels, 1155-1156 Bleeding disorders diagnosis of, 1157f evaluation of, 1155-1158 history in, 1155 laboratory testing, 1155-1158, 1157f physical findings, 1155 platelet, hereditary, 1171 Bleeding time, 1156 Bleeding ulcers, 915-917 endoscopic parameters, 915-917, 916t endoscopic treatment of, 916, 916t Bleomycin (Blenoxane) for cancer, 1211t-1216t, 1366 for Hodgkin’s lymphoma, 1271-1272 intralesional, 2660 Blepharitis, 2561, 2561f, 2564t Blepharospasm, 2463-2464 Blindness. See also Visual loss night blindness, 2559 Blister beetles, 2172t, 2176 Blister cells, 1057 Blistering diseases, 2634, 2634.e1t hypersensitivity reactions, 2677-2678 immunologically mediated, 2675 infectious, 2678-2680 inherited genetic disorders, 2678 metabolic disorders, 2678 Bloating and distention, 861 and gas, 859-861 medications for, 893t, 894 Blomstrand’s disease, 1661.e1 Blomstrand’s lethal chondrodysplasia, 1652t Blood availability for transfusion, 1191 cellular components, irradiated, 1192 hemoglobin composition of, 1102 leukocyte-reduced components, 1192 in sickle cell disease, 1102 whole blood, 1192 Blood alcohol levels, 2510 Blood component therapy, 1183, 1183t Blood flow anemia and, 1060 coronary, 264 Blood glucose. See also Glucose control postoperative abnormalities, 2625 Blood lipid levels, 258 Blood loss acute, 1063 anemia due to, 1063 chronic, 1063 iron deficiency anemia due to, 1069 Blood pressure and cardiovascular disease, 258 and cardiovascular risk, 381, 381f, 387.e1f elevated, 31. See also Hypertension incidental, 396 low, 31. See also Hypotension mean arterial pressure (MAP), 687t measurement of, 26, 498f accurate, 383-384 monitoring, 383-384 normal ranges, 30, 381, 397 office, 381, 381t, 383 pulmonary, 397 regulation of, 1495-1496 staging of, 381, 381t

Blood pressure (Continued) systolic arterial pressure (SAP) in heart failure, 303 normal value, 687t treatment goals, 391-396, 392t variation in behavioral determinants of, 383 genetic determinants of, 383 Blood products HIV transmission through, 2286 for upper GI bleeding, 880 for viral hemorrhagic fevers, 2254 Blood safety, 1195-1198, 1197f emerging infectious diseases of concern to, 1197 Blood smears indications for, 1056t peripheral, 1052-1059 Blood stream infections central line–associated, 1865-1866 burden, costs, and preventability of, 1862t, 1865 due to S. maltophilia, 1970-1971 due to Acinetobacter species, 1968-1969 due to S. maltophilia, 1970-1971 health care–associated, 1862t Blood transfusion. See also Transfusion for anemia of chronic disease, 1071 for autoimmune hemolytic anemia, 1076 risks of, 1197, 1198t for sideroblastic anemia, 1073 for thalassemia, 1094 Blood typing, 1192-1193 Blood volume arterial, effective, 742 regulation of, 1495-1496 Blood-borne pathogens in health care–associated infections, 1865, 1865.e1t transmission of, 1197-1198, 1197f travel-related health problems, 1884 Blood-brain barrier, 2431 Blood-derived stem/progenitor cells, 205 Bloom’s syndrome, 1227t, 1239, 1370 Blue bloaters, 557 Blue lesions, 2581t Blue nevi, 2689, 2690f Blue rubber bleb nevus syndrome, 958 Blue toe syndrome, 506 Blunt injury, 711, 712t Bocavirus, 2185t Boceprevir, 2177t-2178t, 2180 Body fat, 61 Body fluid compartments, 741, 741f Body fluid volume, 742, 742t Body lice, 2175 clinical manifestations of, 2175, 2175f epidemiology of, 2175 treatment of, 2175b Body mass index (BMI), 631, 1431-1432 at-risk, 1458 calculation of, 1458, 1463 formula for, 1458 and type 2 diabetes, 1534 Body plethysmography, 540 Body temperature elevated, 30 low, 30-31 measurement of, 26 normal, 30, 691, 1849 regulation of, 691, 1849, 691.e3f Body water, 738-739 Body water deficit, total, 754 Body weight, 1431. See also Weight gain; Weight loss adjusted, 1441 and cancer, 1223-1224 guidelines for management of, 59 measurement of, 1431 overweight. See also Obesity classification of, 1458t definition of, 544-545 regulation of, 1460 standards for men and women, 1431, 1432t Boerhaave’s syndrome, 907 clinical manifestations of, 907 diagnosis of, 907 treatment of, 907b Bolivian hemorrhagic fever, 2249t

Bolivians, 1038 Bone(s), 1730t dense, 1668t ischemic necrosis of, 1668t normal biology of, 1638, 1638f osteoarthritis in, 1745 peak mass, 1638-1639 remodeling of, 1638-1639, 1639f, 1646 rheumatoid arthritis in, 1758t spotted, 1671 Bone and joint infections clinical evaluation of, 1807f Pseudomonas, 1965 Bone disorders, 1671-1672 autoinflammatory, 1740t in breast cancer, 1358 degenerative disease, 1713t, 1715t marble bone disease, 1670 metabolic approach to, 1636-1637 in end-stage renal disease, 842 Paget’s disease of bone, 1664-1667 renal, 835-836 in sarcoidosis, 606 in sickle cell anemia, 1101, 1103 solitary plasmacytoma of, 1282 treatment of, 928, 1103 Bone fractures, pathologic, 1219 Bone loss aging-related, 1639-1640 clinical effects of, 1640 mechanisms of, 1639-1640 natural, 1639-1640 secondary causes of, 1640, 1640t Bone marrow in anemia, 1063-1064 in chronic myelogenous leukemia, 1247 empty, 1114 fibrosis of, 1127t in megaloblastic anemia, 1111-1112, 1112f myeloid maturation in, 1129, 1130f in neutropenia, 1135 normal, 1114f stimulation of, 1131 Bone marrow compartment, 1143-1144 Bone marrow failure states, 1114-1121 causes of, 1114, 1114t inherited syndromes, 1116-1117 clinical features of, 1116-1117, 1116t prognosis for, 1121 pathology of, 1114 pathophysiology of, 1114 Bone marrow mononuclear cells, autologous, 451 Bone marrow transplantation allogeneic, 1280 for cancer, 1218 for inborn errors of metabolism, 1389 for multiple myeloma, 1280 for thalassemia, 1094-1095 Bone marrow-derived stem/progenitor cells, 205 Bone mineral density (BMD) measurement of, 1641 normal, 1637f Bone mineral density (BMD) testing, 1636 Bone scintigraphy, 1728, 1729f Bone tumors metastatic, 1371 prevention of, 1371 treatment of, 1371b primary, 1370 clinical manifestations of, 1370 definition of, 1370 diagnosis of, 1370 primary malignant, 1370-1371 Bonferroni correction, 37 Booerhaave’s syndrome, 902 Boomslang (Dispholidus typhus), 719 BOOP. See Bronchiolitis obliterans organizing pneumonia of unknown cause Borderline personality disorder, 2355t Bordetella, 1990-1993 Bordetella bronchiseptica, 1991 Bordetella parapertussis, 1991 Bordetella pertussis, 1991-1992, 1992f, 2602-2603 Borrelia, 2027-2028, 2028b Borrelia afzelii, 2021

I15

Borrelia burgdorferi, 2021 Borrelia duttonii, 2027 Borrelia garinii, 2021 Borrelia parkeri, 2027 Borrelia recurrentis, 2027 Borrelia turicatae, 2027 Borrelial lymphocytoma, 2023, 2023f Bortezomib (Velcade) for amyloidosis, 1286 for cancer, 1211t-1216t for multiple myeloma, 1279-1281, 1280t Bosentan for Eisenmenger’s syndrome, 407b for pulmonary hypertension, 402 for Raynaud’s phenomenon, 509f Bosutinib (Bosulif), 171 for cancer, 1232t for chronic myelogenous leukemia, 1248, 1249t, 1250 Botanicals, 181 Bothrops snakes, 719 Botulinum antitoxin, heptavalent (HBAT), 1928 Botulinum toxin, 1927 lethal doses, 1928 for spasticity, 2238-2239 Botulinum toxin poisoning, 78 Botulinum toxois, 78 Botulism, 87t, 90, 1927-1928, 2548t acute cholinergic neuropathy of, 2520 clinical manifestations of, 90, 1928 definition of, 1927 diagnosis of, 90, 1928 epidemiology of, 1927-1928 fatality rates, 1928 food-borne, 1927 iatrogenic, 1928 immunizations for adults, 66t infant, 1927-1928 pathobiology of, 1928 prevention of, 1928 prognosis for, 90b, 1928 treatment of, 90b, 1928b wound, 1928 Botulism immune globulin (BabyBIG), 1928 Boutonneuse fever, 2050, 2252t-2253t Bovine spongiform encephalopathy, 2504 Bowel. See Colon Bowel habits, 894 Bowel irrigation, 705 Bowel obstruction, antipsychotic-induced, 2626t Bowel perforation, 866-867, 866f Bowen’s disease, 970 Boyle’s law, 598 Brachioradialis reflex, 2372t Bradyarrhythmias, 346, 350t-351t, 355-367 Bradycardia, 31, 357t advanced cardiac life support response to, 355f management of, 452 relative, 2250 sinus, 356-358, 357f toxicant-induced, 696-697 treatment of, 355f Bradykinesia, 2455 Brain arterial supply of, 2429f midline structures, 2428f normal, 2383f physiology of, 2429-2431 vascular anatomy of, 2424-2429 vascular malformations in, 2453-2454 venous system of, 2429, 2431f-2432f Brain abscess, 2495-2497, 2496f, 2501 antibiotic therapy for, 2496, 2496t antimicrobial therapy for, 2496 clinical manifestations of, 2495-2496 conditions that predispose to, 2495t diagnosis of, 2496 epidemiology of, 2495 initial features of, 2495t MRI findings, 2496, 2496f pathobiology of, 2495 prognosis for, 2497 treatment of, 2496b Brain death, 2409, 2410t, 2414-2415 clinical manifestations of, 2414 definition of, 2414 diagnosis of, 2414 Volume I  pp 1-1382  •  Volume II  pp 1383-2722

I16

Index

Brain death (Continued) epidemiology of, 2414 pathobiology of, 2414 tests for, 2414t treatment of, 2414b Brain disorders, autonomic, 2517-2518 Brain herniation, 2576 Brain imaging, 2438-2439 computed tomography, 2439, 2439f-2440f magnetic resonance, 2438, 2439f-2440f Brain injury. See also Traumatic brain injury assessment of, 2365t Brain metastasis, 1294, 1295f clinical manifestations of, 1294 definition of, 1294 diagnosis of, 1294, 1295f effects of microbiota on, 1842-1843 epidemiology of, 1294 treatment of, 1294b-1295b Brain natriuretic peptide, 304, 464 Brain stem blood supply to, 2428f infarction of, 2599 malformations of, 2513 Brain stem abscess, 2496, 2497f Brain stem auditory evoked potentials, 2344, 2345t, 2594 Brain stem glioma, 1293 Brain swelling, 2488 Brain tumors chemotherapy for, 1291 classification of, 1288t clinical manifestations of, 1288, 1289t diagnosis of, 1288-1290 diagnostic tests, 1289-1290 differential diagnosis of, 1290 epidemiology of, 1287 general approach to, 1287-1290 generalized symptoms, 1288 imaging of, 1288-1289 intracranial tumors, 1287-1295 lateralizing symptoms, 1288 pathobiology of, 1288 radiation therapy for, 1291 treatment of, 1290b-1291b, 1290t definitive, 1290-1291 symptomatic, 1290 types of, 1291-1295 Brainerd’s diarrhea, 926 Brain-gut axis, 890, 890.e2f BRAK kinase inhibition, 2661 Brazilian hemorrhagic fever, 2249t Brazilian purpuric fever, 1947 Breast(s) development of in female, 61, 62f Tanner stages, 1588, 1588t examination of, 27 Breast cancer, 187-188, 187t, 1351-1359 chemoprevention of, 57, 1358-1359 clinical manifestations of, 1352 diagnosis of, 1352-1354 early-stage adjuvant anti-HER2 therapy for, 1355-1356 adjuvant chemotherapy for, 1356 adjuvant endocrine therapy for, 1355 adjuvant systemic therapy for, 1355-1356, 1355t follow-up, 1356, 1356t local treatment of, 1354-1355 epidemiology of, 1351-1352 hereditary, 1227t in situ carcinoma, 1354 invasive, 1351, 1354-1355 in men, 1358 metastatic, 2649f biologic agents for, 1358 chemotherapy for, 1357-1358 endocrine therapy for, 1356-1357 treatment of, 1356-1358 molecularly guided therapeutics for, 202.e1t nutritional influences, 1426-1429 pathobiology of, 1351-1352 postmenopausal symptoms in survivors of, 1358 predictive markers, 1352-1354 prevention of, 1358-1359 prognostic markers, 1352-1354 radiotherapy for, 1354-1355 risk, 1359-1360

Breast cancer (Continued) risk factors for, 1351, 1352t screening for, 56, 1358-1359, 1359t special circumstances, 1358 stage III, 1356 stage IV biologic agents for, 1358 chemotherapy for, 1357-1358 endocrine therapy for, 1356-1357 treatment of, 1356-1358, 1357f supportive care for, 1358 targeted therapy for, 202, 202t TNM staging, 1352-1354, 1353t treatment of, 1354b-1358b treatment-related tumor flare, 1357 Breast conservation therapy, 1354 Breast examination clinical, 1359, 1359t self-examination, 1359, 1359t Breast lesions, benign, 1359-1360 nonproliferative, 1359 proliferative, 1359 Breast self-examination, 1359, 1359t Breath sounds, bronchial, 526 Breath tests, 925t, 926 Breathing obstructed, 639 sleep-disordered, 302-303 spontaneous trials, 650, 670-671, 671f work of components of, 655, 655f measurement of, 655 Breathing disorders, sleep-related, 2418-2424 Breathing trials, 650 Breslow’s thickness, 1375 Brief Pain Inventory, 10, 12f-13f Brill-Zinser disease, 2048t, 2052 Brimonidine, 2658 Brincidofovir, 2199, 2219 British anti-Lewisite (BAL) (dimercaprol), 706t-710t for arsenic poisoning, 95b for heavy metal poisoning, 93t British HIV Association (BHIVA), 2288t Broca’s aphasia, 2385, 2385t, 2435 Brodalumab, 2666t Brody’s disease, 2542t, 2544-2545 Bromfenac, 1008t Bromocriptine for cardiomyopathy, 331 for infertility, 1596 for neuroleptic malignant syndrome, 2628 for Parkinson’s disease, 2457t-2459t for premenstrual syndrome, 1587 for type 2 diabetes, 1536 Brompheniramine (Dimetapp), 2660t Bronchial breath sounds, 526 Bronchial thermoplsaty, 642 Bronchiectasis, 566-569 acute exacerbations, 569 anatomic causes of, 566-567 antimicrobial therapy for, 568-569 central, 567, 568f clinical manifestations of, 567 CT findings, 567, 567f-568f, 567.e1f-567.e2f in cystic fibrosis, 564, 564f definition of, 566 diagnosis of, 567-568 distinguishing features of, 528t epidemiology of, 566 “finger-in-glove” central bronchiectasis, 567, 568f genetics of, 566 imaging findings, 567-568, 567f, 567.e1f-567.e2f pathobiology of, 566-567 prevalence of, 566 preventing exacerbations, 568 prognosis for, 569 traction, 580, 580f treatment of, 568b-569b, 568t Bronchioalveolar cell carcinoma, 571t, 574-575 Bronchiolitis chronic, 556 clinical features of, 528t, 589t definition of, 608 diffuse panbronchiolitis, 1965 enteroviral, 2241t in military personnel, 589t occupational, 80

Bronchiolitis obliterans classification of, 646 clinical manifestations of, 646 diagnosis of, 646 pathobiology of, 646 prognosis for, 646 risk factors for, 646 treatment of, 646 Bronchiolitis obliterans organizing pneumonia (BOOP) of unknown cause, 583 Bronchiolitis–-associated interstitial lung disease, 582, 582b, 582f Bronchitis acute bronchitis and tracheitis, 608-609 chronic definition of, 555 pathology of, 556 signs and symptoms of, 608f Bronchoalveolar lavage, 544 Bronchodilator response, 550 Bronchodilators, 559, 560t Bronchogenic carcinoma, 1303 Bronchogenic cysts, 570 Bronchogenic lung cancer, 1303-1313 Bronchopulmonary aspergillosis, allergic, 2084 diagnosis of, 2085 radiologic features of, 2085, 2085f Bronchoscopy, 642-643 for central airway obstruction, 642-643 in surgical approaches, 643 Brown recluse spider bites, 2174 Brown recluse spiders, 2174, 2174f Brown snakes (Pseudonaja), 720 Brown-Séquard syndrome, 2365, 2379 Bruce protocol, 424 modified, 250t Brucella, 1979 Brucella endocarditis, 477t Brucella meningitis, 2487 Brucellosis, 1979-1982 acute disease, 1980t clinical manifestations of, 1980 treatment of, 1981t antibody tests, 1981 in children, 1981t chronic disease, 1980t, 1981 classification of, 1980, 1980t clinical manifestations of, 1980-1981 complications of, 1980 culture of, 1981 definition of, 1979 diagnosis of, 1981 epidemiology of, 1979 etiology of, 1979 immunity against, 1980 incidence of, 1979 localized disease, 1980t clinical manifestations of, 1980 treatment of, 1981t pathobiology of, 1979-1980 pathogenesis of, 1979-1980 prevalence of, 1979 prevention of, 1981 prognosis for, 1982 relapsing, 1980-1981, 1980t standard tube agglutination test for, 1981 subacute disease, 1980, 1980t subclinical illness, 1980, 1980t treatment of, 1981b, 1981t Brudzinski’s sign, 2482 Brugada’s syndrome, 341, 373f cellular basis for, 343, 343f diagnosis of, 372 Brugia malayi, 2103-2104, 2159, 2166, 2167t Brugia timori, 2103-2104, 2159, 2166, 2167t Bruns’ nystagmus, 2579 Bruton’s disease, 1824 Buboes, 1987, 1987f Bubonic plague, 1987, 1987f incubation period, 1987 prognosis for, 1988 symptoms and signs of, 1987 Buckley-Job syndrome, 1683t, 1684 Budd-Chiari syndrome, 958, 959f clinical manifestations of, 958 diagnosis of, 958 prognosis for, 958 treatment of, 958b

Budesonide (Pulmicort, Rhinocort), 1692t for allergic rhinitis, 2588 for asthma, 553t for chronic sinusitis, 1692 for COPD, 560t for Crohn’s disease, 940 for diarrhea, 934 for eosinophilic esophagitis, 902 extended-release (Entocort EC, MMX, UCERIS) for colitis, 941 for inflammatory bowel disease, 939t for proctitis, 941 for inflammatory bowel disease, 939t Buerger’s disease, 505, 506f, 957 clinical manifestations of, 1797 epidemiology of, 1794-1795 treatment of, 1797 Bufadienolides, 699t-702t, 706t-710t Buffalo hump, 2291 Buffalopox, 2215, 2217 Building-related illness, 80t, 81 Bulbitis, 908 Bulimia nervosa, 1455 cognitive-behavioral therapy for, 1457 definition of, 1455 diagnostic criteria for, 1455, 1456t natural history of, 1457 pathobiology of, 1456 prognosis for, 1457 risk factors for, 1456 symptoms and signs of, 1456 treatment of, 1457 Bullae, 2636t, 2647t-2651t, 2650f, 2656f definition of, 2675 vesiculobullous diseases, 2675-2680, 2675t widespread, 2654-2655 Bullous dermatosis, 2676 Bullous impetigo, 2652f, 2678-2679, 2679f Bullous pemphigoid, 2656f, 2675, 2675f definition of, 2675 pathobiology of, 2675 treatment of, 2675b Bullrout, 721 Bumetanide, 388t, 748t Bundle branch blocks diagnosis of acute MI in presence of, 444 left, 270-271, 271f-272f, 271t, 329.e1f right, 270-271, 271f-272f, 271t Bunostomum phlebotomum, 2160 Bupivacaine, 2620 Buprenorphine, 139t-140t, 159 Bupropion for depression, 2349t for tobacco dependence, 147t, 148-149 Burkholderia, 564, 1962, 1967 Burkholderia cepacia, 1963, 1967-1968 Burkholderia cepacia complex, 1962-1963, 1967 Burkholderia mallei, 1962-1963, 1967 Burkholderia pseudomallei, 1962-1963, 1967 Burkitt-like acute lymphoblastic leukemia, 1243-1244 Burkitt’s lymphoma, 1265 clinical characteristics of, 1261t EBV-associated, 2234 in HIV infection, 2324 treatment of, 1265b Burning hands, 2366 Burns, 711-717 Acinetobacter infection in, 1969 critical care for, 715-716 diagnosis of, 713b-716b initial surgical care for, 715 interhospital transport, 713 prehospital care, 713 Pseudomonas infection in, 1965, 1967 surgical care for, 716 treatment of, 713b-716b Bursae, 1805-1806 Bursitis, 1750t anserine, 1753 classification of, 1713t definition of, 1749-1750 diagnosis of, 1750 epidemiology of, 1750 iliopectineal, 1753 iliopsoas, 1753 ischial or ischiogluteal, 1753 olecranon, 1752 prepatellar, 1753 septic, 1805-1806

Index Bursitis (Continued) treatment of, 1750b-1751b trochanteric, 1753, 1753f Buschke-Ollendorff syndrome, 1671, 2689 Bushke-Löwenstein tumors, 972 Busulfan, 1127 1,4-Butanediol, 699t-702t Butorphanol, 137, 139t-140t, 2636t Butterfly rash, 2641f γ-Butyrolactone, 699t-702t Bypass surgery coronary artery bypass, 317, 456, 458 for ischemic heart disease, 431 for multivessel coronary artery disease, 431 recommendations for, 428t gastric bypass postprandial hyperinsulinemic hypoglycemia after, 1552 Roux-en-Y, 1466

C

C cells, 1662 CAB (compression-airway-breathing) algorithm, 353 Cabazitaxel, 1369 Cabergoline for infertility, 1596 for Parkinson’s disease, 2457t-2459t Cachexia, 1221 cardiac, 302 management of approaches to, 11t in palliative care, 11t nutritional therapy for, 1433 in skin disease, 2646t Cadmium nephropathy, 798 Cadmium toxicity, 95-96 clinical manifestations of, 96 diagnosis of, 96 diagnostic testing for, 93t epidemiology of, 95 pathobiology of, 95-96 prognosis for, 96 treatment of, 96b Caffeine for cerebrospinal fluid leaks, 2363 for narcolepsy, 2419t CAGE (Cut down, Annoyed, Guilty, and Eye opener) questionnaire, 153 Calabar swelling, 2170 Calciferol. See Vitamin D Calcifications, in heart, 284-285, 284f, 287, 289f Calcified cysticercosis, 2150 Calcineurin inhibitors nephrotoxicity, 846 for pruritus, 2636t Calcipotriol, 2658 Calcitonin, nasal, 1644 Calcitriol for chronic interstitial nephritis, 798 for hypocalcemia, 1659 for osteomalacia, 1648t, 1649 for renal bone disease, 842 Calcium coronary artery scoring, 287, 287f daily requirements for women, 1601-1602 extracellular fluid, 1650, 1650f metabolism of, 740, 740f, 1649-1650, 1651f in sarcoidosis, 606 recommended intake, 814 serum levels, 1649-1650 for vitamin D deficiency, 1103 Calcium acetate for acute kidney injury, 782 for hyperphosphatemia, 778, 778t Calcium carbonate for hyperphosphatemia, 778t for malabsorption, 931t for osteomalacia, 1648 for renal bone disease, 840 Calcium channelopathies, 2544 Calcium channels, 341, 341.e1f Calcium chloride for drug overdose, 699t-702t for hyperkalemia, 761 for hypermagnesemia, 776

Calcium citrate for malabsorption, 931t for osteomalacia, 1648 Calcium deficiency, 330 Calcium gluconate for arrhythmias, 355 for hyperkalemia, 761 for hypocalcemia, 1658 Calcium homeostasis, 1650, 1651f Calcium oxalate crystals, 731-732 Calcium oxalate nephrolithiasis, 814, 815f Calcium phosphate amorphous crystals, 731-732 Calcium pyrophosphate dihydrate arthropathy, 1811 Calcium pyrophosphate dihydrate crystal deposition disease, 1815 clinical manifestations of, 1815 definition of, 1815 diagnosis of, 1815, 1816f epidemiology of, 1815 pathobiology of, 1815 radiographic findings, 1724t, 1815, 1816f treatment of, 1815b-1816b Calcium pyrophosphate dihydrate crystals, 1811 Calcium salt, 706t-710t, 778t Calcium scoring, preoperative, 2617t Calcium stones kidney stones, 811-812, 814, 815f prevention of recurrence, 814-815 Calcium supplements for cobalamin deficiency, 1107 for diarrhea, 928 for hypocalcemia, 1659 for malabsorption, 931, 931t for osteomalacia, 1648 for osteoporosis, 1642 for prevention of colorectal cancer, 1329 for prevention of preeclampsia, 1616 Calcium-channel blockers for acute coronary syndrome, 437 for acute STEMI, 451, 455 for angina and ischemia, 429 contraindications to, 389t doses and side effects of, 363t-364t for hypertension, 387-390, 388t, 394t mechanism of action, 387-389 overdose, 706t-710t properties of, 430t for pulmonary hypertension, 402 side effects of, 389t, 390 therapeutic principles of, 389 toxicity, 699t-702t, 706t-710t Calcium-sensing receptor, 1650 Calculi acute urinary tract obstruction caused by, 803 bile duct stones, 1042-1044 endoscopic removal of, 877, 877f gallstones, 1038-1041 renal. See Nephrolithiasis Calculous cholecystitis, acute clinical manifestations of, 1039 complications of, 1041 pathobiology of, 1038-1039 treatment of, 1041 California encephalitis, 2263t, 2268-2269 California Infant Botulism Treatment and Prevention Program, 1928 California serogroup encephalitis, 2268-2269 clinical manifestations of, 2269 diagnosis of, 2269 epidemiology of, 2269 features of, 2263t pathobiology of, 2262, 2269 prevention of, 2269 prognosis for, 2269 treatment of, 2269b Caloric needs measurement of, 1433-1434 post-resuscitation, 716 Caloric test, 2600 Calymmatobacterium granulomatis, 2001 Cameron’s ulcers, 908, 911 CAM-ICU, 120 Camouflage, 2661 Camptosar (irinotecan), 1211t-1216t Campylobacter, 1931 Campylobacter coli, 1953

Campylobacter concisus, 1955 Campylobacter curvus, 1955-1956 Campylobacter doylei, 1955-1956 Campylobacter fetus, 1955 Campylobacter helveticus, 1955-1956 Campylobacter hyointestinalis, 1955 Campylobacter infections, 1953-1956 clinical features of, 1869t, 1954-1955, 1954t definition of, 1953-1954 diagnosis of, 1955 differential diagnosis of, 1955 distinguishing characteristics of, 2252t-2253t epidemiology of, 1869t, 1954 pathobiology of, 1954 prevention of, 1955 prognosis for, 1956 treatment of, 1955b Campylobacter insulaenigrae, 1955-1956 Campylobacter jejuni, 1869t, 1953-1954 Campylobacter lari, 1955-1956 Campylobacter mucosalis, 1955-1956 Campylobacter peloridis, 1955-1956 Campylobacter rectus, 1955-1956 Campylobacter troglodytis, 1955-1956 Campylobacter upsaliensis, 1955 Campylobacter ureolyticus, 1955-1956 Camurati-Engelmann disease (progressive diaphyseal dysplasia), 1669 Canada, 18 Canadian Cardiovascular Society (CCS) angina grading scale, 420 functional classification of cardiovascular disability, 251t indications for initial observation and rapid evaluation of syncope, 347t Canadian C-Spine Rule, 2367, 2367f, 2373 Canadian CT Head Rule, 2366, 2366t Canakinumab (Ilaris), 169-170 Canalithiasis, 2598 Canasa (mesalamine), 939t Canavan’s disease, 1385t, 2479 Cancer. See also specific sites and types age-standardized rate, 1222, 1224f anal, 972-973 in HIV infection, 2322, 2323t, 2326, 2327f incidence of, 972, 2220 treatment of, 972b-973b approach to, 1206-1222 biliary tract, 1339-1345 bladder, 1348-1351 breast, 187-188, 187t, 1351-1359 chemoprevention of, 57, 1358-1359 hereditary, 1227t metastatic, 2649f molecularly guided therapeutics for, 202.e1t nutritional influences, 1426-1429 screening for, 56, 1358-1359, 1359t targeted therapy for, 202, 202t causes of, 1222-1225 cell therapy for, 212-213 cervical, 1360-1361 causative factors, 1228 in HIV infection, 2322, 2323t, 2326 natural history of, 2223, 2223f prevalence of, 2220 screening for, 56, 1360-1361 colorectal, 1322-1331 colonic microbiota and, 1842 development of, 1226 endoscopy in, 876 genetic changes associated with, 1226 in inflammatory bowel disease, 942-943 molecularly guided therapeutics for, 202.e1t nutritional influences, 1429 screening for, 56, 1328-1329, 1328t targeted therapy for, 202t complications of, 1218-1219 demographic factors, 1222 development of, 1226 diagnosis of, 1206-1207 diagnostic procedures, 1206 diet recommendations for, 1223 dietary factors, 1427t disease trajectories, 10.e1f endocrine manifestations of, 1219-1220

I17

Cancer (Continued) endometrial, 1361-1362 epidemiology of, 1222-1225 esophageal, 1313-1314 risk factors for, 1314t TNM staging system for, 1315t extragonadal germ cell syndrome, 1381 of fallopian tube, 1363-1364 familial susceptibility syndromes, 1226, 1227t follow-up, 1221-1222 future directions, 1225 gallbladder, 1042 genetics of, 1226-1229 genomic instability and, 1226 gynecologic, 1360-1365 hallmarks, 1229, 1229t head and neck, 1297-1303 in heart failure, 316 hepatobiliary, 1339-1345 hereditary risk syndromes, 1226, 1227t in HIV infection, 2322-2327, 2323f epidemiology of, 2322 prevention of, 2322-2323 screening for, 2322-2323 hypopharyngeal, 1298 immune response to, 1230-1231 immunotherapy for, 1218 incidence of, 1222, 1224f inflammation pathways in, 230.e3t invasion, 1230 laryngeal epidemiology of, 1298 subglottic, 1299 supraglottic, 1299 liver, 1339-1345 lung, 1303-1313 occupational, 589t, 595 screening for, 56 multidisciplinary evaluation, 1206-1207 nasopharyngeal, 1298 neuropathies associated with, 2533 nutritional influences, 1426-1429 nutritional support for, 1218-1219 in obesity, 1218, 1463 in older adults, 1218 oral, 1297-1298 organ dysfunction in, 1218 oropharyngeal, 1298-1299 ovarian, 187t, 1227t, 1363-1364 overdiagnosis of, 54, 1225 pain control in, 1219 pancreatic, 1332-1334 in paranasal sinuses, 1298 partial response, 1209 of piriform sinus, 1299 in pregnancy, 1218 prevention of, 1225 dietary guidelines for, 1428t-1429t primary, 1225 secondary, 1225 progressive disease, 1209 prostate, 1367-1370 incidence of, 15, 1367 screening for, 56 psychosocial support for, 1219 with pulmonary embolism, 625 radiation carcinogenesis, 84 radiation therapy for, 1207-1208 rectal MR staging, 871, 872f surgery for, 1330 remote effects of, 2532 of renal pelvis, 1350t, 1351 reproductive factors, 1225 of reproductive system, 1608 of respiratory tract, occupational, 80, 80t risk factors for, 1226 risk syndromes, 1226, 1227t salivary gland, 1298 screening for adherence to guidelines, 15, 15f future issues, 57-58 for older adults, 108-109 skin basal cell, 1377-1378 in HIV infection, 2321 nonmelanoma, 1377, 2321 squamous cell, 1377-1378 stable disease, 1209 Volume I  pp 1-1382  •  Volume II  pp 1383-2722

I18

Index

Cancer (Continued) staging, 1206-1207 supportive care for, 1218-1219 survivorship, 1221-1222 susceptibility to familial syndromes, 1226, 1227t inherited syndromes, 1226, 1227t symptoms of, 1219 systemic sclerosis and, 1784 systemic therapy for, 1208-1218 FDA-approved drugs, 1211t-1216t settings for, 1209, 1209t testicular, 1365-1366 therapeutic index, 1207 therapeutic paradigm, 1207 therapeutic plan, 1207 therapeutic settings, 1209, 1209t thyroid, 1511-1514 treatment of, 316, 1207b-1219b, 1225b adjuvant therapy, 1209t assessment of response to, 1209 classes of therapeutic agents, 1209-1218 combination therapy, 1208-1209 and diarrhea, 920-921 gene therapy, 212-213 genomic data providing guidance for, 132-133 hormonal therapy, 1218 immunomodulatory therapy, 212-213 intention of, 1207 laser therapy, 2661 neoadjuvant therapy, 1209, 1209t organ-sparing therapy, 1209, 1209t phototherapy, 2661 surgical therapy, 1207 systemic therapy, 1208-1218 targeted therapy, 202, 202t, 2661-2662 of unknown primary origin, 1378-1381 biopsy, 1378-1379 definition of, 1378 empiric chemotherapy for, 1381 initial light microscopy of, 1379 management of, 1379-1381 pathologic evaluation of, 1378-1379 recommended evaluation after initial light microscopy, 1379, 1379t recommended treatment, 1380t search for primary site, 1379 site-specific therapy for, 1381 treatment of, 1379b-1381b of ureters, 1350t, 1351 venous thromboembolism in, 1189 viral infection and, 1228 vocal cord, 1299 vulvar, 1364 in women, 1601 Cancer drugs clinical trials, 1208 combination regimens, 1208-1209 drug interactions, 1208 routes of dosing, 1208 Cancer genes, 1226 Cancer Genome Atlas, 202 Cancer pharmacogenomics, 202 Cancer pharmacology, 1208-1209 Cancer phenotype, 1229-1231 Cancer Staging Manual (AJCC), 1309, 1309t Cancer stem cells, 1231 Cancer wasting syndrome, 1221 Candesartan for heart failure, 310t for hypertension, 388t Candid 1, 2255-2256 Candida, 2079 Candida albicans, 2079 Candida glabrata, 2079 Candida guilliermondii, 2079 Candida krusei, 2079 Candida lusitaniae, 2079 Candida meningitis, 2081 Candida parapsilosis, 2079 Candida tropicalis, 2079 Candidatus neoehrlichia mikurensis, 2055 Candidemia, 2080-2082 Candidiasis, 2079-2083 AIDS-associated, 2295, 2297t-2301t chronic disseminated, 2080, 2081f, 2082 clinical manifestations of, 2079-2081 cutaneous, 2080 definition of, 2079 diagnosis of, 2081

Candidiasis (Continued) disseminated, 1858, 1860f, 2080, 2081f, 2082, 2655f epidemiology of, 2079 esophageal, 907, 907f, 2297t-2301t hepatosplenic, 1012-1013, 2080, 2081f in HIV infection, 2295, 2297t-2301t, 2320 invasive, 2080, 2080f focal infections, 2081-2082 treatment of, 2082 mucocutaneous, 2079-2080 AIDS-associated, 2295, 2297t-2301t chronic, 1686-1687, 1687t, 2080 treatment of, 2082 oral, 2582-2583, 2582f oropharyngeal, 2079-2080, 2297t-2301t osteoarticular, 2081 pathobiology of, 2079 prevention of, 2082 prognosis for, 2082 retinal involvement, 2080, 2081f treatment of, 2081b-2082b fluconazole for, 2067 guidelines for, 2081b-2082b vulvovaginal, 1880, 2297t-2301t white pseudomembranes secondary to, 2604, 2604f Candiduria, 2081 CANDLE syndrome, 1740t Canine granulocytic ehrlichiosis, 2054 Canine monocytic ehrlichiosis, 2054-2055 Canker sores, 2579-2580, 2580t Cannabinoids, 138t Capecitabine (Xeloda), 1211t-1216t, 1319, 1331 Capillaria hepatica, 1015t-1017t Capillaria philippinensis, 2160t, 2164 Capillaritis, idiopathic pulmonary, 586 Capillary hemangioma, benign, 2690, 2690f Caplan’s syndrome, 593 Capnocytophaga, 1931 Capnocytophaga canimorsus infection, 2252t-2253t Capnometry, 653-654, 653f Caprelsa (vandetanib), 170 Capsaicin, 2659-2660 for chronic pain, 137, 138t for pruritus, 2636t Capsule endoscopy, 873t Captopril for acute STEMI, 450-451 for cystine stones, 822 for heart failure, 310t for hypertension, 388t for hypertensive urgency, 396 for left ventricular dysfunction, 452 for renal disease, 1103 Caput medusae, 978, 978f Carbamates antidote for, 706t-710t toxicity, 699t-702t Carbamazepine for alcohol withdrawal, 155t for bipolar disorder, 2350 characteristics of, 2408t for chronic pain, 137, 138t dosage adjustments in renal failure, 129t for glossopharyngeal neuralgia, 2364 for myoclonus, 2466-2467 for neuromyotonia, 2552-2553 for painful dysesthesias and paroxysmal dystonic spasms, 2477 pharmacokinetic parameters for, 126t for seizures, 2407t-2408t for sodium channelopathies, 2544 for syncope, 347 for syphilis, 2018 toxic levels, 703t for trigeminal neuralgia, 2363 Carbapenemases, 1961 Carbapenems for anaerobic infection, 1933, 1934t mechanism of action, 1889t recommended doses and schedules, 1891t-1892t resistance to in health care–associated infections, 1864, 1865.e1t mechanism of, 1890t toxicities, 1895t

Carbidopa for myoclonus, 2466-2467 for Parkinson’s disease, 2457t-2459t Carbocisteine, 2525 Carbohydrates complex, 1399 dietary, 926, 926.e1f disorders of metabolism of, 2545 high-carbohydrate/low-fat diet, 1465 high-fat/low-carbohydrate diet, 1465 for hypoglycemia, 1539 Carbon dioxide elimination of, 763 monitoring, 653-654 partial pressure of (Paco2), 652-653, 1619 production of, 654 transcutaneous measurement of, 653-654 Carbon monoxide: diffusing capacity for (Dlco), 542, 558 Carbon monoxide poisoning, 599 clinical manifestations of, 599 definition of, 599 diagnosis of, 599 epidemiology of, 599 pathobiology of, 599 prognosis for, 599 treatment of, 599b, 706t-710t Carbonic acetazolamide, 2567 Carbonic acid, 763 Carbonic anhydrase inhibitor II deficiency, 823t-824t Carboplatin (Paraplatin), 1211t-1216t for esophageal adenocarcinoma, 1316 for head and neck cancer, 1302 for testicular cancer, 1366 Carboxyhemoglobin, 703t Carboxypeptidase-N (CPN-1), 243t Carbozantanib (Cometriq), 1211t-1216t Carbuncles, 2696 Carcinogenesis, radiation, 84 Carcinoid syndrome, 337-338, 1558-1560 cardiac manifestations of, 1559 chemotherapy for, 1559 clinical manifestations of, 1558-1559 diagnosis of, 1559 differential diagnosis of, 1559 distinguishing features of, 528t epidemiology of, 337-338 gastrointestinal symptoms of, 1558 pathobiology of, 337-338 pathophysiology of, 1558 pellagra in, 1558 pNETs that cause, 1335t, 1339 prognosis for, 338b, 1559-1560 symptoms of, 1559 treatment of, 338b, 1559b Carcinoid tumors, 1557-1558 of lung, 1313 neuroendocrine tumors, 1381 in small bowel, 1320 Carcinoma. See also specific sites and types anaplastic thyroid, 1513 basal cell, 1377-1378, 1377f, 2562 bronchioalveolar cell, 571t, 574-575 bronchogenic, 1303 choroid plexus, 1294 clear cell, 1346, 1346t collecting duct, 1346t epithelial (follicular) thyroid, 1512-1513 extra-ovarian serous, 1380 large cell lung, 1304 large cell neuroendocrine, 1313 medullary thyroid, 1662-1664 Merkel cell, 1228, 1301, 2326, 2689 oropharyngeal, 2322, 2323t papillary renal cell, 1346, 1346t papillary thyroid, 1511-1513 parathyroid, 1652t peritoneal papillary serous, 1380 poorly differentiated, 1379 management of, 1381 recommended evaluation after light microscopic diagnosis, 1379t recommended treatment, 1380t renal cell, 1345-1348 salivary gland, 1313 sebaceous, 2561-2562 serous extra-ovarian, 1380 peritoneal papillary, 1380 small cell anaplastic, 1381

Carcinoma (Continued) squamous cell esophageal, 1314-1315 eyelid, 2562 head and neck, 1301t in lung, 1304 oral, 2580t, 2581, 2581f of skin, 1377-1378, 1377f of unknown primary origin, 1379t-1380t, 1380-1381 Carcinoma in situ breast cancer, 1354 ductal, 1354, 1354t lobular, 1354, 1354t Carcinomatous polyarthritis, 1826 Card agglutination test, 2115 Cardenolides, 699t-702t Cardiac action potential, 263, 263f, 339-341, 340f, 368.e1f Cardiac antibody-mediated rejection, 521t Cardiac arrest, 352-356 AED rapid response to, 354t bradycardic or asystolic, 355f clinical manifestations of, 352 diagnosis of, 352 epidemiology of, 352 incidence of, 352 pathobiology of, 352 prognosis for, 355-356 sudden, 352 tachyarrhythmic, 354 treatment of, 353b-355b Cardiac cachexia, 302 Cardiac care: for noncardiac surgery, 2612, 2614f Cardiac catheterization, 292-298 with acute STEMI, 453 complications of, 293-294, 293t contraindications to, 292, 293t hemodynamic data obtained during, 294-295, 294f indications for, 292, 293t for mitral stenosis evaluation, 466 right-sided, in cardiogenic shock, 683 technique for, 292 transseptal, 295-298 Cardiac catheterization laboratory, 295-298, 292.e3f Cardiac cavity, 339 Cardiac cycle, 265, 265f filling phase, 265 pressure-volume loop, 265 Cardiac drugs. See also specific drugs glycosides, 699t-702t perioperative management of, 2613t with psychoactive effects, 116, 117t Cardiac effusion, 486-489 Cardiac electrophysiology, 339-344 basic concepts of, 339 ionic basis, 339-341 molecular basis, 341-342 signals that regulate contraction, 264 Cardiac energetics, 265 Cardiac examination, 303 Cardiac function, 262-267 Cardiac imaging computed tomography, 287-288, 287f echocardiography, 274-282 noninvasive, 282-292 preoperative, 2617t radiography, 282-285, 282f-283f stress MRI, 425 Cardiac impulse disorders of conduction, 342 disorders of formation, 342 normal path, 356 propagation of, 341 Cardiac index (CI), 687t Cardiac inspection, 253 Cardiac Lyme disease, 2023-2024 Cardiac masses CMR imaging of, 289 echocardiographic findings, 280t-281t Cardiac output (CO), 687t Cardiac resynchronization therapy, 317, 378-379, 379f complications of, 379 for heart failure, 306.e1t-306.e2t indications for, 378-379, 379t Cardiac risk assessment, 2612-2615 noninvasive tests, 2612-2614 perioperative, 2613t

Index Cardiac risk assessment (Continued) pharmacologic stress testing, 2612-2614 risks of surgical procedures, 2613t Cardiac risk indices, 2612 Cardiac silhouette, 285 Cardiac surgery, 374-376 anesthetic approaches for, 2618t in heart failure, 317 Cardiac tamponade, 486-489 echocardiography in, 487-488, 488f treatment of, 338b, 488-489, 489f Cardiac testing, noninvasive, 259 Cardiac transplantation, 317, 519-523 annual procedures, 519, 520f comorbid conditions, 522 contraindications to, 519-523, 520t donor criteria for, 520-521 heart transplantation, 317, 519-523, 202.e1t heart-lung transplantation, 403, 645 immunosuppression for, 521 indications for, 519t matching donors and recipients, 521 molecularly guided therapeutics for, 202.e1t organ allocation for, 520-521 prognosis for, 522-523, 522f rejection in, 521 histologic grading of, 521t treatment of, 521 selection criteria for, 519-523 surgical technique for, 521 Cardiac troponins (cTn) elevated, 444-445, 445t measured by a highly sensitive assay (hs-cTn), 304.e1t Cardiac tumors, 280t-281t, 338-339, 338t CardioDx, Inc., 203 Cardioembolic stroke, 2434 Cardiogenic pulmonary edema, 662, 664t Cardiogenic shock, 674, 681-685 circulatory support for, 684 clinical manifestations of, 682, 682t definition of, 678f-679f, 681 diagnosis of, 681t, 682-683, 684f epidemiology of, 681 etiology of, 678f-679f, 681, 681f, 682t hemodynamic criteria for, 681t inotropes for, 683 management of, 452-453 approach to, 684f initial, 683 in special conditions, 684-685 pathobiology of, 682 prognosis for, 685 rapid ultrasound protocol, 675, 675t therapy for, 678f-679f treatment of, 683b-685b vasopressors for, 683 Cardiology, nuclear, 285-287 in coronary artery disease, 425 in heart failure, 305 Cardiomyocytes, 264.e1f Cardiomyopathy alcoholic, 330 antipsychotic-induced, 2626t, 2627 approach to, 321f arrhythmogenic right ventricular, 321t, 331-332, 372, 372f causes of, 322t-323t chronic ischemic, 370f cirrhotic, 1025 classification of, 320, 321f CMR findings, 288-289, 291f definition of, 320 diabetic, 1546 dilated, 321t, 329-331, 329.e1f acquired, 329 on echocardiography, 275f family screening for, 330 key echocardiographic findings, 280t-281t treatment of heart failure due to, 318 genetic, 187t causes of, 322t-323t ventricular tachycardia in, 372 hypertrophic, 320-327, 321t, 325.e1f apical, 325.e2f-325.e3f echocardiographic findings, 280t281t

Cardiomyopathy (Continued) progressive, 325.e4f treatment of heart failure due to, 318 hypertrophic obstructive, 324-325, 324f key echocardiographic findings, 280t-281t laboratory findings, 330t peripartum, 331 restrictive, 321t, 332-334 differential diagnosis of, 490t echocardiographic findings, 280t-281t overlap with dilated cardiomyopathy, 331 takotsubo (stress-induced), 289, 335, 681 unclassified, 335 Cardiopulmonary exercise tests, 544 Cardiopulmonary resuscitation after cardiac arrest, 353 compression-only, 353 Cardiothoracic ratio, 282f, 283 Cardiotomy, 490-491 Cardiotoxicity, 1203 Cardiovascular disease approach to, 248-257 atherosclerotic, 260f-261f in diabetes, 1546-1547 pathobiology of, 1546 prevention of, 1546 treatment of, 1547b-1548b in dialysis patients, 842 epidemiology of, 19, 20t, 257-262 Framingham risk score for, 259, 259.e1t-259.e2t functional classification of, 251t gene therapy for, 212 global burden, 258 global trends, 19, 20t high-penetrance monogenic disorders, 187t hospitalization due to, 258 laboratory studies, 255-257 medical costs related to, 258 methods of assessment for, 251t physical examination for signs of, 252-257 preoperative assessment of, 2615 prevention of, 261 rate of death due to, 258 in rheumatoid arthritis, 1762 risk assessment, 259-261 global, 384 lifetime risk estimation, 259-261 stratification, 384 risk factors for, 258-262, 385t blood pressure, 381, 381f, 387.e1f established, 258-259 novel markers, 259 short-term, 259 in septic shock, 686-687 clinical manifestations of, 687 diagnosis of, 687-688 symptoms of, 251 cardinal, 249t detection of, 248-252 US burden, 258 in women, 1600 Cardiovascular health definition of, 262t new paradigm for, 261 Cardiovascular magnetic resonance imaging, 288-292, 290f-292f contraindications to, 288 indications for, 288 pulse sequences, 288 Cardiovascular syphilis, 2015-2016 Cardiovascular system. See also Heart age-related changes in, 107 alcohol-related complications, 151t, 152 metabolic regulation of, 264-265 responses to stressors, 266-267, 267.e1f in SLE, 1772-1773 Cardioversion, transthoracic, 376-377 Caregiver education. See also Patient education about heart failure, 315, 315t Caribbean, 2275 Carmustine (BiCNU, BCNU), 1211t-1216t Carney’s complex, 1483-1484, 1516, 1556, 1556t L-Carnitine, 706t-710t Carnitine acylcarnitine translocase deficiency, 1385t

Carnitine deficiency primary, 2545 treatment of, 2545 Carnitine metabolism disorders, 1552-1553 Carnitine palmitoyltransferase I deficiency, 2545 Carnitine palmitoyltransferase II deficiency, 2545 Carnitine replacement, 2545 L-Carnitine supplements, 1445 Carnitine uptake deficiency, 1386t Caroli’s disease, 980 β-Carotene, 1454t β-Carotene supplements, 1454-1455 Carotid artery stenosis, asymptomatic, 2444 Carotid duplex ultrasonography, 2440 Carotid pulse, 26, 252-253, 253f Carpal tunnel syndrome, 1757, 1757f, 1759, 2536-2537 Carpentier-Edwards pericardial valve, 472t, 473f Carpentier-Edwards stented porcine valve, 473f Carpet vipers (Echis), 719 Carteolol, 388t Cartilage, 1730t ischemic necrosis of, 1668t in osteoarthritis, 1744-1745 Cartilage-hair hypoplasia, 1683t, 1685, 1685b Carvajal’s syndrome, 331 Carvedilol for acute MI, 450, 454t, 455 clinical use of, 430t for dilated cardiomyopathy, 330 for heart failure, 309, 311t for hypertension, 388t for varices and varical bleeding, 1029 Case management, 48-49 Case-control studies, 42 Casein kinase 2 (CK2β), 818 Casodex (bicalutamide), 1211t-1216t Caspofungin, 2069 adverse effects of, 2069 for AIDS-associated opportunistic infections, 2297t-2301t for candidemia, 2082 drug interactions, 2069 formulations, 2069 for hepatosplenic candidiasis, 1013 indications for, 2069 for invasive aspergillosis, 2086t pharmacology of, 2069 Cassava tuberis (konzo), 2510 Castleman’s disease, 1267 multicentric, 2326 clinical manifestations of, 2326 diagnosis of, 2326 pathobiology of, 2326 prognosis for, 2326 treatment of, 2326b Castleman’s giant lymph node hyperplasia, 538, 539f Castration-resistant prostate cancer, 1367, 1369 Casts granular, 730-731, 732f hyaline, 730-731, 732f urinary, 730 Catamenial anaphylaxis, 1700 Cataplexy, 2418, 2419t Cataract, 2566 congenital, 2571 radiation, 84 sunflower, 1417 Cataract surgery, 2566 anesthetic approaches for, 2618t hyphema after, 2563f Catathrenia, 2423, 2423t CATCH-22, 1684 Catecholamines, 1521-1523 metabolism of, 1522f, 1523 plasma levels, 1523 synthesis of, 1521-1523, 1522f Catecholamine-secreting tumors evaluation of, 1525-1526, 1525f imaging of, 1526 laboratory findings, 1525-1526 medical therapy for, 1526 medications that may cause, 1524-1525, 1525t

I19

Catecholamine-secreting tumors (Continued) signs and symptoms associated with, 1524, 1524t treatment of, 1525f, 1526b Categorical variables, 32-33 Caterpillars, 2172t, 2177 Catfish (Siluriformes), 721 Catfish injuries, 721 Cathepsins, 1732-1733 Catheter ablation, 379-380 of atrial fibrillation, 380 of ventricular arrhythmias, 380 Catheter angiography, diagnostic, 2440 Catheter-associated urinary tract infections, 1866 burden, costs, and preventability of, 1862t guidelines and recommendations for, 1865.e1t Catheterization, cardiac, 292-298 with acute STEMI, 453 for mitral stenosis evaluation, 466 right-sided, in cardiogenic shock, 683 Catheters balloon angioplasty, 456, 457f pulmonary artery, 2615 vascular catheter-associated infections, 1865.e1t Cat’s eye reflex, 2572 Cat-scratch disease, 1019, 1998-1999, 2057t, 2059t clinical manifestations of, 1998-1999 diagnosis of, 2000t neurologic manifestations of, 1998-1999 ocular manifestations of, 1998-1999 pathobiology of, 1998 treatment of, 2000 Cauda equina syndrome, 2377 Caudalization, 534-535 Causality determination of, 36 inferring, 36 Caustic injury, esophageal, 902 Cavernous angioma or hemangioma, 2453 Cavernous venous sinus thrombosis, septic, 2498-2499 Cavitary pulmonary aspergillosis, chronic, 2084 CC-JEV (Encevac), 2266 CCKoma, 1335t, 1339 CD4+ T cells, 236 depletion of, 2279, 2280f gastrointestinal disease and, 2302, 2303t gastrointestinal diseases with CD4+ counts < 200 cells/µL, 2304-2305 gastrointestinal diseases with CD4+ counts > 200 to 500 cells/µL, 2302-2304 in graft rejection, 236t skin conditions that can occur at any CD4+ count, 2319t skin conditions with CD4+ counts < 200 cells/µL, 2318t CD8+ T cells, 236, 236t CD28, 222 CD35 (CR1) (immune adherence or C4b/ C3b receptor), 243t CD40 deficiency, 1680t CD46 (membrane cofactor protein, MCP), 243t CD55 (decay-accelerating factor, DAF), 243t CD59 (protectin), 243t CD80 (B7.1), 222 CD86 (B7.2), 222 CD88 (C5a receptor, C5aR), 245 CD95 (Fas ligand), 223 CD117, 1372-1373 CD178 (Fas ligand), 223 CDC Wonder, 43.e1t Cefaclor, 1891t-1892t Cefadroxil, 1891t-1892t Cefazolin (Ancef) for keratitis, 2565 for osteomyelitis or septic arthritis, 1810t for peritonitis, 842-843 for prevention of endocarditis, 482 for prevention of postoperative infection, 1832 recommended doses and schedules, 1891t-1892t for S. aureus infection, 1901 Cefdinir, 1891t-1892t Volume I  pp 1-1382  •  Volume II  pp 1383-2722

I20

Index

Cefditoren pivoxil, 1891t-1892t Cefepime for meningitis, 2488t for osteomyelitis or septic arthritis, 1810t for P. aeruginosa infection, 1966 for peritonitis, 949 for pneumonia, 616t, 619t for pyelonephritis, 1875t recommended doses and schedules, 1891t-1892t for S. pneumoniae infection, 1905t for septic shock, 689t Cefixime for cystitis, 1875t for gonorrhea, 1896, 1944-1945, 1945t for pyelonephritis, 1875t recommended doses and schedules, 1891t-1892t for S. pneumoniae infection, 1905t Cefotaxime for AIDS-associated opportunistic infections, 2297t-2301t for epiglottitis, 2604 for gonorrhea, 1945t for H. influenzae infections, 1947-1948 for lateral sinus thrombosis, 2499 for leptospirosis, 2030 for meningitis, 2485-2486, 2488t for meningococcal disease, 1937-1938, 1937t for peritonitis, 1030 for pneumonia, 616t for pyelonephritis, 1875t recommended doses and schedules, 1891t-1892t resistance of Enterobacteriaceae to, 1961 for S. pneumoniae infection, 1905, 1905t for septic shock, 689t for spinal epidural abscess, 2497 for Vibrio infections, 1953 Cefotetan, 946, 1934t Cefoxitin for anaerobic infection, 1933, 1934t for appendicitis, 946 for pelvic inflammatory disease, 1945 recommended doses and schedules, 1891t-1892t Cefpodoxime for cystitis, 1875t recommended doses and schedules, 1891t-1892t for S. pneumoniae infection, 1905t Cefprozil, 1891t-1892t Ceftaroline for meningitis, 2488t for MRSA infection, 1901 for pneumonia, 616t recommended doses and schedules, 1891t-1892t Ceftazidime for cellulitis, 2696 for melioidosis, 1967 for meningitis, 2488t for P. aeruginosa infection, 1966 for peritonitis, 842-843 for pneumonia, 619t for pyelonephritis, 1875t recommended doses and schedules, 1891t-1892t Ceftibuten for cystitis, 1875t for pyelonephritis, 1875t recommended doses and schedules, 1891t-1892t Ceftizoxime, 689t, 1891t-1892t Ceftriaxone for AIDS-associated opportunistic infections, 2297t-2301t for chancroid, 1950 for cholangitis, 1043 for E. rhusiopathiae infection, 1924 for endocarditis, 479t, 480 for enteric fever, 1974 for enterococcal infection, 1915.e1t for epiglottitis, 2604 for gallstones, 1041 for gonorrhea, 1896, 1944-1945, 1945t, 2603 for H. influenzae infections, 1947-1948 for lateral sinus thrombosis, 2499 for leptospirosis, 2030 for Lyme disease, 2025t

Ceftriaxone (Continued) for meningitis, 2485-2486, 2488t, 2489 for meningococcal disease, 1937-1938, 1937t, 1939t for nocardiosis, 2064 for osteomyelitis, 1810t for pelvic inflammatory disease, 1945 for peritonitis, 1030 for pneumonia, 616t, 619t for prophylaxis in HIV infection, 2295t for pyelonephritis, 1875t for pyogenic liver abscess, 1012 recommended doses and schedules, 1891t-1892t resistance of Enterobacteriaceae to, 1961 for S. pneumoniae infection, 1905, 1905t for Salmonella bacteremia, 1974 for septic arthritis, 1808, 1810t for septic shock, 689t for spinal epidural abscess, 2497 for syphilis, 2018 for urethritis, 1878t for varices and varical bleeding, 1029 for Whipple’s disease, 932 Cefuroxime, 1891t-1892t, 1905t Cefuroxime axetil for cystitis, 1875t for Lyme disease, 2025t for pyelonephritis, 1875t recommended doses and schedules, 1891t-1892t Celiac artery, 951 Celiac artery compression syndrome, 958 Celiac disease, 929-931 arthritic manifestations of, 1824 clinical manifestations of, 930 definition of, 929-930 diagnosis of, 930, 930f epidemiology of, 929-930 intestinal biopsy findings, 930, 930f pathobiology of, 930 prognosis for, 931 treatment of, 931b Celiac sprue, 1869t Cell proliferation, sustained, 1229-1230 Cell therapy, 203-210 definition of, 204 for diabetes mellitus, 210 for heart disease, 209-210 immunomodulatory, 212-213 for neurodegenerative disorders, 209 regenerative medicine approaches, 205-209, 208f specific disease applications, 209-210 CellCept (mycophenolate), 2660 Cells of origin, 1240 Cellular rejection, 521t. See also Rejection Cellulitis, 1908, 2564, 2696-2697, 2696f A. baymannii, 1969, 1969f antibiotic regimens for, 689t dissecting, 2706 H. influenzae, 1947 preseptal, 2564, 2564f Centers for Disease Control and Prevention (CDC), 2103 Advisory Committee on Immunization Practices, 66-67 CDC Wonder, 43.e1t Cities Readiness Initiative, 1922 DATA2020, 43.e1t definition of health care–associated infections, 1861 Emergency Operations Center, 1928 Guideline on Management of Opportunistic Infections in Adults and Adolescents, 2294-2295 guidelines and recommendations for prevention and management of health care–associated infections, 1865.e1t guidelines for management of pharyngitis, 2602t Guidelines for the Prevention and Treatment of Opportunistic Infections in HIV-Infected Adults and Adolescents, 2294 guidelines for trichomoniasis, 2145-2146 Health Data Interactive, 43.e1t Health Indicators Warehouse, 43.e1t Medical Eligibility Criteria (MEC) for contraceptive use, 1605-1606 microagglutination test (MAT), 2030

Centers for Disease Control and Prevention (CDC) (Continued) PulseNet, 1918 recommendations for prevention and treatment of anthrax, 1922 recommendations for prevention of HIV in medical care settings, 2287 recommendations for treatment of pertussis, 1992 recommended strategies for prevention of STIs, 1880 Centers for Medicare and Medicaid Services (CMS), 44 Conditions of Participation, 1868.e1 Dialysis Facility Compare, 43.e1t Home Health Compare, 43.e1t Hospital Compare, 43.e1t hospital-acquired condition (HAC) program, 1868.e1 Inpatient Quality Reporting Program, 1868.e1 Nursing Home Compare, 43.e1t Partnership for Patients program, 1868.e1 requirements for reporting of gifts to physicians, 9 Centipeda periodontii, 1931 Centipedes, 2172t, 2175 Central America, 719 Central apnea, 2419, 2419.e1f Central Asia, 2275 Central congenital hypoventilation syndrome, 547 clinical manifestations of, 547 definition of, 547 diagnosis of, 547 epidemiology of, 547 pathobiology of, 547 treatment of, 547b Central cord syndrome, 2365, 2378 Central cord vasculature syndrome, 2381 Central core disease, 2618 Central core myopathy, 2544, 2544.e1f Central European encephalitis, 2263t, 2268 Central hearing loss differential diagnosis of, 2595 pathobiology of, 2594 Central line bundle, 1865-1866 Central line–associated blood stream infections, 1865-1866 burden, costs, and preventability of, 1862t, 1865 due to Acinetobacter species, 1968-1969 due to S. maltophilia, 1970-1971 Central nervous system. See also Brain; Spinal cord age-related changes in, 114, 114t demyelinating conditions of, 2471-2480 drug-induced plasticity of, 145 HIV-associated IRIS, 2332 metastatic tumors of, 1294-1295 and neuroendocrine function, 1475 in osteoarthritis, 1746 primary angiitis of, 1800, 1800t reward circuits, 144-145, 144f tumors of, 1287-1297 vasculitis of, 1799-1800 Central nervous system infections actinomycosis, 2062 in AIDS-related lymphoma, 2324 anaerobic, 1932 arboviral, 2262-2269 cryptococcosis, 2076-2077 enteroviral, 2241-2242 nocardiosis, 2063, 2063f Pseudomonas, 1965 slow virus infections, 2209-2212 staphylococcal, 1899 toxoplasmosis, 2128, 2129f zoonoses, 2057, 2058t Central nervous system leukemia, 1243 Central nervous system lymphoma EBV-associated, 2234 primary, 1294, 1294f AIDS-related, 2324-2325 diagnosis of, 2324-2325 treatment of, 2325b Central nervous system stimulants, 160 Central retinal artery occlusion, 2569, 2569f Central retinal vein occlusion, 2569-2570, 2570f Central sensitivity syndromes, 1818 Central sensitization, 133-134, 1817

Central sleep apnea, 545t Central sleep apnea syndrome, 545t Central spontaneous nystagmus, 2579t Central sympatholytics contraindications to, 389t for hypertension, 388t, 391 mechanism of action, 391 side effects of, 389t, 391 therapeutic principles, 391 Central tendency, 32 Central venous pressure (CVP), 687t Centronuclear myopathy, 2544, 2544.e1f Cephalexin for bacterial overgrowth, 928 for bullous impetigo, 2678-2679 for cellulitis, 2696 for folliculitis, 2681 for furuncles, 2696 for impetigo, 2695-2696 recommended doses and schedules, 1891t-1892t for S. aureus infection, 843, 1901 for UTI prophylaxis, 1876t Cephalic tetanus, 1929 Cephalization, 534-535, 535f Cephalosporins mechanism of action, 1889t for meningitis, 2488t recommended doses and schedules, 1891t-1892t resistance to, 1890t, 1961 toxicities, 1895t Cephradine, 1891t-1892t Cerastes (desert vipers), 719 Cerebellar ataxia, 2341t, 2469-2470 alcohol-induced, 2511 hereditary, 2469-2470 Cerebellopontine angle tumors, 2599 Cerebellum alcohol-induced degeneration of, 2511 infarction of, 2599. See also Ischemic stroke malformations of, 2513 Cerebral aneurysm, 2447-2448 Cerebral artery occlusion, 2435 Cerebral autosomal dominant arteriopathy with subcortical infarcts and leukoencephalopathy (CADASIL), 2394, 2443 Cerebral blood flow, 2429-2431 Cerebral cortex disorders of hemispatial neglect, 2385-2386 clinical manifestations of, 2386 definition of, 2385 diagnosis of, 2386 pathobiology of, 2386 disorders of visual function and hemispatial neglect, 2385-2386 double cortex (band heterotopia), 2512 focal dysplasia of, with balloon cells, 2512 malformations of, 2512 organization, 2430f organization disorders, 2513 Cerebral cysticercosis, 2058t Cerebral dysfunction, regional, 2382-2388 clinical manifestations of, 2382 definition of, 2382 diagnosis of, 2382 Cerebral edema, 2433 in bacterial meningitis, 2481 high-altitude, 597 after intracerebral hemorrhage, 2452 after subarachnoid hemorrhage, 2449 Cerebral hemorrhage, 395t, 2433 Cerebral herniation, 2410, 2412t Cerebral infarction. See Stroke Cerebral ischemia, 2431-2433. See also Ischemic stroke focal injury, 2432-2433 global injury, 2432 Cerebral vasoconstriction syndrome, reversible, 1800, 1800t Cerebral venous thrombosis, 2442, 2444f Cerebritis, 2501 Cerebrospinal fluid examination of, 2341, 2341f in multiple sclerosis, 2475 permeability of antibiotics into, 2486t Cerebrospinal fluid fistulas, 2485 Cerebrospinal fluid formulas, 2341, 2342t Cerebrospinal fluid leaks, 2363

Index Cerebrospinal fluid pressure, elevated, 2488 Cerebrovascular disease. See also Stroke approach to, 2424-2433 brain imaging findings, 2395, 2395f classification of, 2433f definition of, 2424 dementia caused by. See Vascular dementia dietary factors, 1427t epidemiology of, 2424 extensive, 2395f hemorrhagic, 2445-2454 ischemic, 2434-2445 pathobiology of, 2424-2431 pathology of, 2433 pathophysiology of, 2433 in sickle cell anemia, 1099-1100 Cerebrovascular malformations, 2453-2454 clinical manifestations of, 2453 diagnosis of, 2453 epidemiology of, 2453 pathobiology of, 2453 prevention of, 2453 prognosis for, 2453 treatment of, 2453b Cerebrovascular syndrome (acute radiation syndrome), 84 Certolizumab pegol (Cimzia), 169 for inflammatory bowel disease, 939t, 940 for rheumatoid arthritis, 1762 Cerubidine (daunorubicin), 1211t-1216t Cervical adenopathy management of, 1380 periodic fever with aphthous stomatitis, pharyngitis, and, 1740t Cervical artery dissection, 2442-2443, 2443t Cervical cancer, 1360-1361 care of survivors, 1361 causative factors, 1228 clinical manifestations of, 1361 diagnosis of, 1361 epidemiology of, 1360 in HIV infection, 2322, 2323t, 2326 natural history of, 2223, 2223f pathobiology of, 1360 prevalence of, 2220 prevention of, 1361 prognosis for, 1361 screening for, 56, 1360-1361 staging, 1361t survival rate, 1360 treatment of, 1361b Cervical dystonia, 2463-2465 Cervical intraepithelial neoplasia, 2219 natural history of, 2223 treatment of, 2222 Cervical lymph nodes, 2041 Cervical mucus, 1585 Cervical neoplasia, 1877t Cervical radiculopathy, 2378 Cervical spine injury to, 2367, 2367f instability of, 1833 pain in. See Neck pain Cervical squamous intraepithelial lesions, 2221 Cervicitis, 1877t, 1880, 2010 Cervicofacial actinomycosis, 2061, 2061f Cervix cancer of. See Cervical cancer cyclic changes in, 1585 Cestodes, 2104, 2147-2153 definition of, 2147-2148 larval infections, 2148t, 2153 Cetirizine (Zyrtec), 2660t for allergic rhinitis, 2588 for pruritus, 2636t for urticaria, 1695, 2684t Cetuximab (Erbitux), 1232t for cancer, 1211t-1216t for colorectal cancer, 1330-1331 for head and neck cancer, 1302 for lung cancer, 1310t Cevimeline (Evoxac), 1302, 2585 c-Fms, 1050 Chagas disease, 2116-2120 acute, 2117-2118 antiparasitic drugs for, 2118 chronic, 2117, 2117f diagnosis of, 2118 evaluation of, 2118, 2119f

Chagas disease (Continued) chronic cardiopathy, 2117 chronic gastrointestinal (megadisease), 2117, 2119 clinical manifestations of, 2117 CMR findings, 289 definition of, 2116 diagnosis of, 2118 epidemiology of, 2116 immunosuppression and transplantation with, 2117-2118 in Latin America, 2116 neuropathy of, 2520 pathobiology of, 2116-2117 prevention of, 2119-2120 prognosis for, 2120 reactivation of, 2117-2118 symptomatic, 2118-2119 treatment of, 2107, 2118b-2119b in United States, 2116 vectors, 2172t, 2176 ventricular tachycardia in, 371 Chagas megacolon, 2117, 2119 Chalazion, 2561, 2561f Chamberlain procedure, 1308 Chance, 36 Chancres, syphilitic, 2014 Chancroid, 1877t, 1878, 1949-1950 clinical manifestations of, 1949, 1949f definition of, 1949 diagnosis of, 1949-1950 epidemiology of, 1949 pathobiology of, 1949 prevalence of, 1949 prevention of, 1950 prognosis for, 1950 treatment of, 1950b Chancroidal ulcers, 1949, 1949f Change levers for, 44-45 stages of, 52, 53f, 53t Change counseling, 52 fitting into medical practice, 52 stage-specific, 52-54 Channelopathies, genetically inherited, 372 Chapare hemorrhagic fever, 2249t Chapare virus, 2247, 2249t Chaperones, 25 Charcoal, activated, 699t-702t, 705 Charcot-Leyden crystals, 551 Charcot-Marie-Tooth disease, 2529 classification of, 2529.e1t clinical manifestations of, 2529 diagnosis of, 2529 differential diagnosis of, 2529 epidemiology of, 2529 genetics of, 192-194, 192f pathobiology of, 2531, 2529.e2f prevalence of, 2529 treatment of, 2529 Chédiak-Higashi syndrome, 1133t, 1134, 1147t-1148t, 1148-1149, 1171, 1683, 1686t, 1682.e1t clinical manifestations of, 1149, 1683 diagnosis of, 1149 pathobiology of, 1148-1149 treatment of, 1149b Cheilitis, angular, 2583 Chelators, 93t Chemical individuality, 1384 Chemical injuries distinguishing characteristics of, 2252t-2253t to lung, 595-603 stem cell toxicity, 1115-1116 Chemical meningitis, 2484 Chemical sensitivity, occupational, 80t, 81 Chemicals and aplastic anemia, 1118 and leukemia, 1240 Chemoattractant, 1145 Chemokine receptors, 218 Chemoprevention, 55t, 57 Chemotaxis, 1145-1146, 1147t-1148t Chemotherapy adjuvant, 1209, 1356 antibacterial, 1885-1896 for brain tumors, 1291 for breast cancer, 1356-1358

Chemotherapy (Continued) for cancer of unknown primary origin, 1381 for carcinoid syndrome, 1559 for chronic lymphocytic leukemia, 1255-1256 combination, 1208-1209, 1209t for head and neck cancer, 1302 for lung cancer, 1309t, 1311 for non-Hodgkin’s lymphoma, 1263t postremission, 1243 side effects of cardiomyopathy, 330 diarrhea, 920-922 drugs for prevention of, 1218 drugs that ameliorate, 1211t-1216t for testicular cancer, 1366 Chemotherapy-induced neutropenia, 1856, 1856t Chernobyl, 82-83, 83t Chest examination, 27 Chest images, 532-538 Chest injury, 713-714 Chest pain, 248, 302. See also Angina pectoris acute, 248, 250f causes of, 249t estimation of risk for acute MI in, 248, 250f evaluation of, 422, 422f functional differential diagnosis of, 896t of presumed esophageal origin, 895-896, 895t prevalence of, 895-896 prognosis for, 896 treatment of, 896b postoperative, 2621-2622 of presumed esophageal origin, 890, 895-896, 895t recurrent, 451 Chest radiography, 531 in asthma, 551, 551.e1f cardiac silhouette, 285 in chronic angina, 425 double density sign, 283, 283f gooseneck sign, 285 heart evaluation, 282-283, 282f in heart failure, 304 images, 532-538 with implanted devices, 285 in interstitial lung disease, 577-578 in ischemic heart disease, 425 pruned-tree sign, 284, 284f reversed figure-of-3 signs, 285 scimitar sign, 285, 285f techniques for, 531-532 water-bottle sign, 284, 284f Chest wall imaging of, 531-538 nonintegrated, 630 weak, 542, 543t Chest wall disorders, 629-631, 629t Cheyne-Stokes breathing, 545-546, 546f, 2411, 2419 clinical manifestations of, 545-546 diagnosis of, 545-546 epidemiology of, 545 pathobiology of, 545 treatment of, 547b Chiari malformations, 2513 Chiari I, 2513, 2513f Chiari II, 2513 types of, 2513 Chickenpox (varicella), 2227-2229, 2654f-2655f, 2679-2680 AIDS-associated, 2297t-2301t definition of, 2227 immunizing agents for, 68t-73t oral ulcers of, 2580-2581, 2580t Chickenpox vaccine, 77 Chiggers, 2052, 2173-2174 Chikungunya, 2260 Chikungunya fever, 2260 clinical manifestations of, 2260 definition of, 2260 diagnosis of, 2260 distinguishing characteristics of, 2252t-2253t epidemiology of, 2260 pathobiology of, 2260

I21

Chikungunya fever (Continued) prognosis for, 2260 treatment of, 2260b Chilblain, 694 Childbirth blood glucose control during, 1621 infection associated with, 1926 Childhood absence epilepsy, 2404-2405 Childhood cryptosporidiosis, 2135 Children. See also Infants; Neonates acute malnutrition in, 1436.e1f bacterial meningitis in, 2488t brucellosis in, 1981t epilepsy syndromes in, 2404-2406, 2406t gonorrhea in, 1943 hair disorders in, 2706 ovarian function in, 1580-1581 pertussis in, 1991-1992 recurrent parotitis in, 2585 reproductive axis development during, 1570-1571 transgendered, 1568 Child-Turcotte-Pugh scoring system, 1026t, 1031 Chileans, 1038 Chimeras, 193 Chimeric antigen receptor–directed T cells, 1256 ChimeriVax-JE, 2266 ChimeriVax-WN02, 2267 Chinese herbal medicine, 17 Chinese herbal nephropathy, 797 Chinese or oriental liver fluke, 2157 Chironex fleckeri stings, 720-721 Chiropractic, 182t Chlamydia, 2007, 2010, 2603 developmental cycle, 2007, 2007f Chlamydia conjunctivitis, 2564t, 2565 Chlamydia diseases, 2007-2013, 2008t costs attributable to, 2009 epidemiology of, 2009 pathobiology of, 2007 treatment of, 1933-1934 Chlamydia pneumoniae, 2007 Chlamydia pneumoniae infections, 2008t, 2011-2012 association with asthma, 2012.e1 association with atherosclerosis, 2012.e1 association with chronic neurologic diseases, 2012.e1 clinical manifestations of, 2012 diagnosis of, 2012 epidemiology of, 2011-2012 in infants, 2010 treatment of, 2012b Chlamydia psittaci, 2007 Chlamydia psittaci infection, 477t, 2008t, 2012-2013 Chlamydia trachomatis, 2007 Chlamydia trachomatis infections, 2008-2011, 2008t diagnosis of, 2011 epidemiologic features of, 1869t genital clinical manifestations of, 2009-2010, 2009f complications during pregnancy, 2010 natural history of, 2011 nucleic acid amplification testing for, 1941 oropharyngeal, 2010 perinatally transmitted infections, 2009-2011 sexually transmitted infections, 2009-2011 treatment of, 2011b Chlamydia trachomatis urethritis, 2009-2010, 2009f Chlorambucil (Leukeran), 168 for cancer, 1211t-1216t for non-Hodgkin’s lymphoma, 1263 Chloramphenicol for anthrax exposure, 88b for enteric fever, 1974 mechanism of action, 1889t for meningitis, 2487, 2488t for meningococcal disease, 1937t for plague, 89b, 1989t recommended doses and schedules, 1891t-1892t resistance to, 1890t Volume I  pp 1-1382  •  Volume II  pp 1383-2722

I22

Index

Chloramphenicol (Continued) toxicities, 1895t for typhus, 2052b Chlordiazepoxide, 155t, 2352t Chloride channelopathies, 2544 2-Chlorodeoxyadenosine (cladribine) for hairy cell leukemia, 1253 for mastocytosis, 1710 for multiple sclerosis, 2477-2478 Chloromas, 1242 Chloroquine, 2106 contraindications to, 1088t for malaria, 1883, 2106, 2110-2111, 2111t-2112t ocular effects of, 2573, 2573t for porphyria cutanea tarda, 1415 for rheumatoid arthritis, 1761 side effects of, 2571 Chlorothiazide, 761 Chlorpheniramine (Chlor-Trimeton), 2186t, 2660t Chlorpromazine ocular effects of, 2573, 2573t for psychosis, 2354 Chlorthalidone, 748t for hypertension, 388t, 391 to prevent calcium stones, 815 Choking, antipsychotic-induced, 2626t, 2627 Cholangiocarcinoma, 1339, 1340f hilar, 1339, 1340f histology of, 1341f imaging of, 1342, 1344f liver transplantation for, 1033.e1t subtypes, 1339 Cholangiography, endoscopic retrograde, 873 Cholangiohepatitis, oriental, 1044-1045 Cholangiopancreatography endoscopic retrograde, 873, 873t magnetic resonance, 1045, 1046f Cholangitis ascending, 1039, 1039f with bile duct stones, 1043 primary sclerosing, 1045-1046 Cholecalciferol (vitamin D3) for hypocalcemia, 1659 for osteomalacia, 1648t for renal bone disease, 840 synthesis of, 740 for well-being, 2478 Cholecystitis acute, 868, 868f acute acalculous, 1041 acute calculous clinical manifestations of, 1039 complications of, 1041 pathobiology of, 1038-1039 treatment of, 1041 Cholecystokinin for gallstone prevention, 1041 pNETs that secrete CCKoma, 1335t, 1339 Choledochal cysts, 1046, 1046t, 1046.e1f Choledocholithiasis, 1042-1044 ascending cholangitis secondary to, 1039, 1039f classification of, 1042 Cholelithiasis. See Gallstones Cholera, 1950-1952 clinical manifestations of, 1951-1952 definition of, 1950 diagnosis of, 1952 epidemiology of, 1950 pancreatic, 1335t, 1337, 1553 pathobiology of, 1951 prevention of, 1952 prognosis for, 1952 transmission of, 1950 travel-related risk, 1881 treatment of, 1951t-1952t, 1952b world distribution of, 1950, 1951f Cholera toxin, 1951 Cholera vaccines, 1883, 1883t, 1952 Cholescintigraphy, 1040 Cholestasis drug-induced, 1009 familial syndromes, 987-988 intrahepatic, of pregnancy, 1622 obstetric, 1622 pure, 1009 Cholestatic hepatitis, 1009 Cholestatic reactions, 1009 Cholesteatoma, 2594

Cholesterol, 1389 total levels, 1393 treatment of guidelines for, 1395 recommendations for, 1395t Cholesterol absorption inhibitors, 1396t Cholesterol crystal embolization, 506 Cholesterol emboli, 2674 Cholesterol transport, reverse, 1392, 1392f Cholesteryl ester transfer protein, 1390t, 1391 Cholestyramine for diarrhea, 922, 926, 928, 933 effects on micronutrient status, 1453t for gastroenteropathy, 934 for lipid disorders, 1396t for primary sclerosing cholangitis, 1045-1046 for small bowel rapid transit dysmotility, 888 Choline, 1445 Choline acetyltransferase deficiency, 2548t Cholinesterase, 703t Cholinomimetic toxidromes, 696t Chondrodysplasia Blomstrand’s lethal, 1652t rhizomelic punctate type I, 1387t Chondrodystrophic myotonia, 2542t Chondrosarcoma, 1371 Chordae tendineae, 264 Chordomas, 1292 Chorea, 2462-2463 differential diagnosis of, 2462t treatment of, 2463 Choriomeningitis, lymphocytic, 2058t clinical manifestations of, 2491 epidemiology of, 2490 Chorioretinitis, severe, 2131, 2132t Choroid plexus carcinoma, 1294 Choroid plexus papilloma, 1294 Choroidal dystrophy, 2571 Christmas factor (factor IX), 1173 Christmas factor (factor IX) alloantibodies, 1176 Christmas factor (factor IX) deficiency, 1175t Christmas factor (factor IX) replacement therapy, 1174-1175 Chromium, 1450t-1452t Chromoblastomycosis, 2101 Chromomycosis, 2101 clinical manifestations of, 2102 pathogens that cause, 2101, 2102t treatment of, 2103 Chromophobe, 1346t Chromosomal aberrations, 191-193 numerical, 191-192 structural, 192-193 Chromosomal disorders, 186 Chromosomal translocations balanced reciprocal, 193 insertion or insertional, 193 inversions, 193 in non-Hodgkin’s lymphoma, 1258, 1260, 1260t robertsonian, 193 Chromosomes, 191-193 marker, 192 metaphase, 191, 191f Chromothripsis, 192-193 Chronic abdominal pain, 855t-857t, 860f alarm features, 859 Chronic abnormal vision, 2558 Chronic actinic dermatitis, 2664-2665, 2664f Chronic active Epstein-Barr virus infection, 2234 Chronic adult T-cell leukemia/lymphoma, 2238 Chronic ambulatory peritoneal dialysis, 949 Chronic angina coronary artery bypass graft for, 460 diagnostic coronary angiography in, 425t Chronic anovulation, 1592-1594 causes of, 1592t hypothalamic, 1592 related to inappropriate feedback, 1593-1594, 1594b related to other endocrine and metabolic disorders, 1594 Chronic aortic regurgitation, 471 Chronic arthritis, 1174, 1174f Chronic aspergillosis, 2083t, 2086 Chronic autonomic syndromes, 2519

Chronic bacterial otomastoiditis, 2599 Chronic bacterial prostatitis classification of, 831t clinical manifestations of, 831 Chronic beryllium disease, 592-593 chest radiography findings, 593, 593f clinical manifestations of, 593 diagnosis of, 593b epidemiology of, 592-593 pathobiology of, 592-593 treatment of, 593b Chronic blood loss, 1063 Chronic bronchiolitis, 556 Chronic bronchitis, 556 Chronic care case management, 48-49 follow-up, 48-49 goals of, 47 matching patient needs and care delivery, 47-49 planned interactions, 48 team care, 48 Chronic care model, 47-49, 48f Chronic cavitary pulmonary aspergillosis, 2084 Chronic Chagas cardiopathy, 2117 Chronic Chagas disease diagnosis of, 2118 evaluation of, 2118, 2119f Chronic cold agglutinin disease, 1076-1077 Chronic cough, 526t, 1991 causes of, 526t definition of, 527 diagnosis of, 527, 527t management of, 527f Chronic daily headache, 2361-2362 clinical manifestations of, 2361 definition of, 2361 diagnosis of, 2361 epidemiology of, 2361 pathobiology of, 2361 prevention of, 2362 prognosis for, 2362 treatment of, 2361b Chronic diarrhea, 922-926 biopsy of, 925 blood tests in, 923 causes of, 922t, 926-933 clinical manifestations of, 922-923 definition of, 918 diagnosis of, 923-926 endoscopic approach to, 876, 925 imaging in, 923 initial approach to, 924f laboratory tests in, 925-926 treatment of, 934b-935b Chronic disease in adolescents, 64-65 anemia of, 1064, 1064t, 1071-1072 definition of, 1071 diagnosis of, 1071 laboratory findings, 1070t treatment of, 1071b with C. pneumoniae, 2012.e1 definition of, 46 infectious agents in, 1838 management of comprehensive, 46-49 delivery system design, 48-49 interventions for, 47 needs of patients with, 47t in older adults, 100, 102f Chronic disseminated candidiasis, 2080, 2081f, 2082 Chronic dyspnea, 530f Chronic eosinophilic pneumonia, 586 Chronic fatigue syndrome, 1817, 1821-1822 clinical manifestations of, 1821 definition of, 1821 diagnosis of, 1821 diagnostic criteria for, 1821, 1821t differential diagnosis of, 1819.e1t epidemiology of, 1821 pathobiology of, 1821, 1821.e1f prognosis for, 1822 self-reported symptoms, 1821t treatment of, 1821b-1822b Chronic febrile illness, 1852-1853, 1852t Chronic fibrosing interstitial pneumonia, 575t, 580-581 Chronic gastrointestinal Chagas disease (megadisease), 2117, 2119

Chronic granulomatous disease, 1147t-1148t, 1149, 1682.e1t clinical manifestations of, 1149, 1683 diagnosis of, 1149 epidemiology of, 1682-1683 genetics of, 1683 pathobiology of, 1149 treatment of, 1149b Chronic heart failure, 299, 299t Chronic hemodynamic monitors, 305 Chronic hemolytic anemia, 1056t Chronic hepatitis clinical manifestations of, 1000 definition of, 1000 diagnosis of, 1000, 1000t prognosis for, 1001b treatment of, 1001b Chronic hepatitis B, 15, 1001-1004 antiviral therapy for, 1002, 1003t, 2177t, 2178 clinical manifestations of, 1001 diagnosis of, 1000t, 1001 epidemiology of, 1001 liver biopsy in, 1001, 1002f pathobiology of, 1001 prevention of, 1001b-1003b prognosis for, 1003-1004 serologic markers, 1001, 1002t treatment of, 1001b-1003b Chronic hepatitis C, 1004-1005 antiviral therapy for, 2177t clinical manifestations of, 1004 diagnosis of, 1000t, 1004 emerging regimens for, 1005 end-stage disease, 1005 epidemiology of, 1004 HFE mutation analysis in, 1423, 1423.e1f historic standard therapy for, 1004 liver biopsy in, 1002f newly available regimens for, 1004-1005 pathobiology of, 1004 prognosis for, 1005 treatment of, 1004b-1005b Chronic hepatitis D, 1005 clinical manifestations of, 1005 diagnosis of, 1000t, 1005 epidemiology of, 1005 prevention of, 1005 treatment of, 1005b Chronic hepatitis E, 1005 clinical manifestations of, 1005 diagnosis of, 1005 treatment of, 1005b Chronic hyperphosphatemia, 777f Chronic hyperplastic eosinophilic sinusitis (CHES), 1691, 1691t Chronic hypersensitivity pneumonitis, 589t Chronic hypertension, 1612t Chronic hyponatremia, 752 Chronic idiopathic demyelinating polyneuropathy, 2524t Chronic idiopathic neutropenia, 1136 Chronic idiopathic neutrophilia, 1131 Chronic idiopathic secretory diarrhea, 934 Chronic idiopathic urticaria, 1694 Chronic infantile neurologic cutaneous and articular (CINCA) syndrome, 1740t-1741t, 1742 Chronic inflammation anemia of, 1064-1065 clinical manifestations of, 1071 definition of, 1071 epidemiology of, 1071 pathobiology of, 1071 prognosis for, 1072 leukocytosis due to, 1130 Chronic inflammatory demyelinating polyradiculoneuropathy, 2530t, 2531 clinical manifestations of, 2531 definition of, 2531 diagnosis of, 2531 differential diagnosis of, 2531 epidemiology of, 2531 pathobiology of, 2531 treatment of, 2531b Chronic interstitial lung disorders, occupational, 80 Chronic interstitial nephritis, 796-799 biopsy findings, 778f causes of, 797-798 clinical manifestations of, 796 diagnosis of, 796-798

Index Chronic interstitial nephritis (Continued) due to obstruction, 798 prognosis for, 798 treatment of, 798b Chronic intestinal pseudo-obstruction, 887, 887f Chronic ischemic cardiomyopathy, 370f Chronic ischemic heart disease, 371 Chronic kidney disease, 736, 833-841 angiotensin II responses in, 837, 837t clinical manifestations of, 837 complications of, 834t, 837, 837t definition of, 833 diagnosis of, 385t, 837-838 epidemiology of, 833-834 hypertension with, 392 imaging, 838 indications for dialysis, 841, 841t kidney dysfunction in, 834, 834t, 838f laboratory tests, 838 management of, 839f metabolic balance considerations, 834 pathobiology of, 834-837 prognosis for, 840 progression of, 836-837, 838f stage 1, 839 stage 2, 839 stage 3, 839-840 stage 4, 839-840 stages of, 736, 736t, 834t, 837-838 steady-state considerations, 834 in systemic sclerosis, 1783 tradeoff hypothesis for, 834-835, 835f treatment of, 838b-840b vicious cycle with AKI, 782, 782f Chronic Kidney Disease Epidemiology Collaboration (CKD-EPI) equations, 728, 730t, 833 Chronic kidney failure, 736 Chronic laryngitis, 2602, 2602f Chronic leukemias, 1246-1257 Chronic limb ischemia, 499-500 Chronic liver disease anemia of, 1066 cryptogenic, 1006 dietary factors, 1427t liver transplantation for, 1034 nonalcoholic fatty liver disease, 1022f in pregnancy, 1623, 1623b prevalence of, 1023 Chronic low back pain, 142t. See also Low back pain Chronic lower limb ischemia clinical classification of, 499t treatment of, 501 Chronic lung disease, 657t Chronic lymphocytic leukemia, 1253-1257, 1253f, 1263 autoimmune manifestations, 1256 blood smear features of, 1059, 1059f chemotherapy for, 1255-1256 clinical manifestations of, 1253-1254 definition of, 1253 diagnosis of, 1254-1255 differential diagnosis of, 1254-1255, 1254t epidemiology of, 1253 genetics of, 1253 infectious manifestations, 1256 medical therapy for, 1255-1256 pathobiology of, 1253 prognosis for, 1256-1257 prognostic factors, 1255 radiation therapy for, 1256 remission in, 1256t staging, 1255, 1255t treatment of, 1255b-1256b Chronic mediastinitis (fibrosing mediastinitis), 637 Chronic meningitis, 2493-2494 causes of, 2492-2493, 2492t definition of, 2493 infectious causes of, 2492-2493, 2492t with neutrophil predominance, 2494, 2494t noninfectious causes of, 2493-2494, 2494t with predominantly neutrophilic pleocytosis, 2494 Chronic mesenteric ischemia, 911, 911f, 955, 955b

Chronic mitral regurgitation asymptomatic, 468 symptomatic, 468 Chronic mountain sickness, 596 clinical manifestations of, 596 treatment of, 597 Chronic mucocutaneous candidiasis, 1686-1687, 1687t, 2080 Chronic myelogenous leukemia, 1246-1252 accelerated phase, 1248, 1251 atypical, 1252, 1252b blastic phase, 1248, 1251 blood smear features of, 1058-1059, 1059f, 1247, 1247f bone marrow findings, 1247 chronic phase, 1247f-1248f, 1248, 1250-1251 clinical course, 1248 clinical manifestations of, 1247 cytogenetics of, 1247 definition of, 1246 diagnosis of, 1247-1248, 1247f diagnostic algorithm for, 1125f differential diagnosis of, 1247-1248 epidemiology of, 1246 evolution to accelerated and blastic phases, 1248 future directions, 1251 hematopoietic stem cell transplantation for, 1202 molecular pathogenesis of, 1246-1247 neutrophilia in, 1131 pathobiology of, 1246-1247 peptic ulcer disease in, 912 Ph-positive, 1248, 1248f prevalence of, 1246 prognosis for, 1252 subtypes, 1247 survival in, 1248, 1248f targeted therapy for, 202, 202t treatment of, 1248b-1251b, 1249t choice of first-line therapy, 1250 complete molecular response to, 1251 major molecular response to, 1251 monitoring response to therapy, 1251 special considerations, 1251 Chronic myeloid leukemia. See Chronic myelogenous leukemia Chronic myelomonocytic leukemia, 1252, 1252b Chronic myocardial infarction, calcified, 284f Chronic necrotizing pulmonary aspergillosis, 2084 Chronic neutrophilic leukemia, 1131 Chronic nonspherocytic hemolytic anemia, 1088, 1088b Chronic obstructive pulmonary disease, 555-562 acute respiratory failure in, 661 clinical manifestations of, 661 diagnosis of, 661 general care for, 661 key principles, 661t mechanical therapy for, 661 precipitating causes, 661 prognosis for, 661 site of care for, 661 treatment of, 661b airflow obstruction in, 558, 558t clinical manifestations of, 557 clinical phenotypes, 557 definition of, 555 diagnosis of, 558-559 differential diagnosis of, 558-559 distinguishing features of, 528t environmental exposures that cause, 555-556 epidemiology of, 555-556 exacerbations, 561-562, 561t expiratory flow limitations in, 557, 557f genetic factors, 556 guidelines recommendations for, 560t history in, 557 in HIV infection, 2315 clinical manifestations of, 2315 epidemiology of, 2315 pathobiology of, 2315 prognosis for, 2315 radiographic features of, 2309t, 2316.e1f treatment of, 2315b

Chronic obstructive pulmonary disease (Continued) occupational, 589t, 590 pathobiology of, 556-557 pathogenesis of, 556-557 pathology of, 556 physical findings, 557 physiologic aspects of, 557 prognosis for, 562 prophylactic antibiotics for, 560 pulmonary function findings, 558 radiographic findings, 558, 558f-559f severe decision guide for, 645t lung transplantation for, 645t lung volume reduction surgery for, 645t susceptible smokers, 555 treatment of, 559b-562b hospital management of exacerbations, 561t, 562 immunizations for, 560 key management principles, 661t medications for, 560t surgical options, 561 Chronic open-angle glaucoma, 2566, 2567f Chronic opioids, 2613t Chronic osteomyelitis, 1806 Chronic pain, 133, 137t Chronic pancreatitis, 927-928, 963-967 alcohol-related, 928 causes of, 964 clinical manifestations of, 927-928, 964 complications of, 966 CT findings, 964, 964f definition of, 963 diagnosis of, 927-928, 964-965 diagnostic approach to, 965 diagnostic tests for, 964t endoscopic ultrasonography findings, 964-965, 965f epidemiology of, 963 ERCP findings, 964-965, 965f genetics of, 963-964 idiopathic, 964 incidence of, 963 pathobiology of, 963-964 prevention of, 966 prognosis for, 966-967 survival rates, 966-967 treatment of, 965b-966b treatment of pain associated with, 965, 966t Chronic (adult) paracoccidioidomycosis, 2075 Chronic paroxysmal hemicranias, 2361 Chronic partial urinary tract obstruction, 803 Chronic pelvic pain syndrome classification of, 831t treatment of, 832-833 Chronic persistent neutrophilic meningitis causes of, 2494t definition of, 2494 Chronic poisoning, trace metal, 92-98 Chronic polyarthritis, 1764t Chronic portal vein thrombosis, 959 Chronic progressive external ophthalmoplegia, 2546 Chronic prostatitis classification of, 831t treatment of, 832-833 Chronic pulmonary aspergillosis, 2084 Chronic pulmonary histoplasmosis, 2070 Chronic pure red cell aplasia, 2213, 2213f Chronic Q fever, 2055t Chronic recurrent multifocal osteomyelitis, 1740t Chronic renal disease causes of, 833t in pregnancy, 1614t symptoms and signs of, 736f Chronic sinusitis, 1690-1693, 2588 heterogeneity of, 1691t immunotherapy for, 1692-1693 prevention of, 1691 treatment of, 1691b-1693b Chronic sleep-disordered breathing, 303 Chronic spontaneous urticaria, 1694 Chronic stable lower limb ischemia, 499 Chronic telogen effluvium, 2703 Chronic thromboembolic hypertension, 403

I23

Chronic thromboembolic pulmonary hypertension, 625 Chronic tonsillitis, 2604 Chronic tubulointerstitial nephritis causes of, 794, 794t definition of, 793 Chronic ulcerations, oral, 2581 Chronic viral hepatitis, 993t, 1000-1006 Chronic wasting disease, 2506 Chronotherapy, 2422 Chrysaora quinquecirrha (sea nettles), 720 Churg-Strauss syndrome, 586 classification of, 1794 differential diagnosis of, 1153 gastrointestinal involvement, 956 neuropathy in, 2533-2534, 2533t pathologic characteristics of, 1795t treatment of, 2534 Chvostek’s sign, 963, 1646 Chylomicronemia syndrome, 1393-1394 clinical manifestations of, 1393 diagnosis of, 1393-1394 pathobiology of, 1393 treatment of, 1394b Chylomicrons, 1389t Chylothorax, 634 Cialis (tadalafil), 429 Cicatricial alopecia, 2703, 2705-2706 Cicatricial pemphigoid, 906 Ciclesonide (Alvesco, Omnaris), 1692t for asthma, 553t for chronic sinusitis, 1692 Cidofovir, 2183, 2219, 2231 for adenovirus disease, 2199 adverse effects of, 2182t for AIDS-associated opportunistic infections, 2297t-2301t for CMV infection, 2209b, 2232t for herpes, 2181t, 2297t-2301t mechanisms of excretion and thresholds for dose adjustment, 2182t for smallpox, 88b for viral encephalitis, 2504 Cigarette smoking, 145 brief interventions for, 147, 147t and cancer, 1222 cardiovascular disease and, 258-259 clinical manifestations of, 146 cognitive-behavioral therapies for, 147, 147t COPD and, 555, 555f education and, 17, 17f epidemiology of, 145-146 heart failure and, 315 hypertension and, 387 leukocytosis and, 1130 motivational interventions for, 147, 147t neutrophilia and, 1130 passive, 258-259 prevalence of, 17, 18f prevention of, 149 prognosis for, 149 psychosocial treatments for, 146b-149b racial differences in, 17 second-hand, 258-259 treatment of, 146b-149b by women, 1603 Ciguatera, 722 Cilastatin sodium, 949 Ciliary body, 2556 Cilostazol, 179 for atheromatous embolization, 507 for atherosclerotic PAD, 502 Cimetidine (Tagamet), 900t, 2660t Cimzia (certolizumab pegol), 169, 940, 1762 Cinacalcet for hyperparathyroidism, 1655-1656 for hyperphosphatemia, 778 for renal bone disease, 840, 842 Cinryze (C1 inhibitor concentrate), 1697 Ciprofloxacin for AIDS-associated opportunistic infections, 2297t-2301t for anthrax exposure, 88 for bacterial overgrowth, 928 for chancroid, 1950 for cholangitis, 1043 for cholera, 1952 for cystitis, 1875t for diarrhea, 922 Volume I  pp 1-1382  •  Volume II  pp 1383-2722

I24

Index

Ciprofloxacin (Continued) for diverticulitis, 947 for enteric fever, 1974 for eye infections, 2565t for gastroparesis and pseudo-obstruction, 888 for inflammatory bowel disease, 940 for lateral sinus thrombosis, 2499 for legionnaires’ disease, 1996t for meningitis, 2487, 2488t, 2489 for meningococcal disease chemoprophylaxis, 1939t for necrotizing fasciitis, 2696-2697 for osteomyelitis, 1810t for otitis externa, 1967 for P. aeruginosa bacteremia, 1966 for perianal fistulas, 941 for peritonitis, 949 for plague, 89b, 1989t for pneumonia, 616t, 619t for prostatitis, 831-832 for Pseudomonas infection, 1967, 2681, 2697 for pyelonephritis, 1875t for pyogenic liver abscess, 1012 recommended doses and schedules, 1891t-1892t for S. aureus infection, 843 for S. maltophilia infections, 1971 for Salmonella carriers, 1975 for Salmonella infection, 1974 for septic arthritis, 1810t for septic shock, 689t for shigellosis, 1978 for spinal epidural abscess, 2497 for tularemia, 90b, 1983 for UTI prophylaxis, 1876t for verruga peruana, 2000 Circadian rhythm disorders, 2421-2422, 2422.e1f clinical manifestations of, 2422 definition of, 2421 diagnosis of, 2422 epidemiology of, 2422 pathobiology of, 2422 treatment of, 2422b Circadian rhythms, hormonal, 1475, 1475.e1f Circle of Willis, 2426, 2425.e1f Circulation enterohepatic, 984 hyperdynamic, 1023-1025 physiology of, 265-266 Circulatory control, 262-267 Circulatory overload, 1196 Circulatory support for cardiogenic shock, 684 mechanical, 317-318 for septic shock, 689-690 Circumcision, 1880 Circumscribed (patchy) hyperpigmentation, 2700 Circumscribed (patchy) hypopigmentation, 2699 Cirrhosis, 1023-1031, 1024f alcoholic, 1034 biliary, primary, 1046-1047, 1823 causes of, 1023, 1024t clinical manifestations of, 1026-1027 compensated, 1026 computed tomography findings, 1027, 1027f diagnosis of, 1027 management of, 1029f treatment of, 1028 complications of, 1023-1028, 1024f-1025f cardiopulmonary, 1025-1026 pulmonary, 1027, 1031 death rate, 1023 decompensated, 1026-1027, 1029f definition of, 1023 diagnosis of, 1027-1028 epidemiology of, 1023 hepatic dietary factors, 1427t in HIV infection, 2303t in Wilson’s disease, 1416, 1417f imaging studies, 1027, 1027f laboratory findings, 1027 Model for End-Stage Liver Disease (MELD) score, 1031 natural history of, 1026, 1026f pathobiology of, 1023-1026

Cirrhosis (Continued) physical findings, 1027 prevalence of, 1023 prevention of, primary, 1031 procedure, 1031 scoring systems, 1026t surgical therapy for, 1031 treatment of, 1028b-1031b Cirrhotic cardiomyopathy, 1025 Cisatracurium for ARDS, 662, 669 for paralysis in acute liver failure, 1033 Cisplatin (Platinol) for ampullary tumors, 1047 for cancer, 1211t-1216t, 1302, 1312, 1319, 1366 for Cushing’s syndrome, 1517 for osteosarcoma, 1370 Citalopram (Lexapro) for depression, 2349t, 2396 for irritable bowel syndrome, 893t for menopausal hot flushes, 1628t-1629t for Tourette’s syndrome, 2466 Cities Readiness Initiative, 1922 Citrovorum factor, 1211t-1216t Citrullinated proteins, 1719-1720 Citrullinemia, 1386t c-Jun amino terminal kinase ( JNK), 234 c-kit receptor, 1050 CL ELISA assay, 1180 Cladribine (2-chlorodeoxyadenosine, Leustatin) for cancer, 1211t-1216t for hairy cell leukemia, 1253 for multiple sclerosis, 2477-2478 Clarithromycin, 1894 for AIDS-associated opportunistic infections, 2297t-2301t for anthrax exposure, 88b for H. pylori eradication, 915t for legionnaires’ disease, 1996t for pertussis, 1992 recommended doses and schedules, 1891t-1892t for S. pneumoniae infection, 1905, 1905t for toxoplasmosis, 2129t Claritin (loratadine), 2660t, 2684t Clark’s level, 1375 Classification and regression trees, 35 Claude’s syndrome, 2435.e1f Claudication definition of, 499 intermittent prevalence of, 498, 498f prognosis for, 503f pseudoclaudication, 500t true, 500t Clear cell carcinoma, 1346, 1346t Clemastine fumarate, 2660t Clindamycin, 1894-1896 for AIDS-associated opportunistic infections, 2297t-2301t for anaerobic infection, 1933, 1934t for anthrax exposure, 88b for babesiosis, 2144 for deep neck abscesses, 2603 for folliculitis, 2681, 2706 for furuncles, 2696 for hidradenitis suppurativa, 2682 for lateral sinus thrombosis, 2499 for Lemierre’s syndrome, 2603 for malaria, 2111t mechanism of action, 1889t for MRSA infection, 1901 for peritonsillar abscess, 2603 for pharyngitis, 2602 for Pneumocystis pneumonia, 2096, 2096t, 2297t-2301t, 2313t for pneumonia, 618 recommended doses and schedules, 1891t-1892t resistance to, 1890t for septic shock, 689t for tonsillitis, 2604 for toxic shock syndrome, 2672 toxicities, 1895t for toxoplasmic retinitis, 2566 for toxoplasmosis, 2129t Clinical breast examination, 1359, 1359t Clinical decision making, 37-41 Clinical decision support systems, 44-45

Clinical examination. See also Physical examination nutritional assessment, 1430-1433 quantitative principles, 24 Clinical history, 1430-1431 Clinical information systems, 49 Clinical Laboratory Improvement Amendments, 203 Clinical laboratory values, 2712t-2717t Clinical pathways, 44-45 Clinical Pharmacogenetics Implementation Consortium (CPIC), 203.e2t Clinical reference intervals, 2712t-2717t Clinical trials in oncology, 1208 power of, 35 selecting subjects for, 32 Clitorimegaly, 1567 Clobazam, 2408 Clobetasol, 2583 Clobetasol propionate (Temovate), 2658t, 2705 Clofarabine (Clofar), 1211t-1216t Clofazimine, 2045-2046, 2536 Clomiphene citrate, 1592-1595 Clomipramine, 2466 Clonazepam for anxiety, 2352t for myoclonus, 2466-2467 for restless legs syndrome, 2468-2469 for seizures, 2407t for sleep disorders, 2422-2423 for Tourette’s syndrome, 2466 for tremor, 2462 for trigeminal neuralgia, 2363 Clonic seizures, 2401t Clonidine for alcohol withdrawal, 155t for complex regional pain syndrome, 2522 for diarrhea, 933, 935 for hyperadrenergic disorders, 2521-2522 for hypertension, 388t for hypertensive urgency, 396 for menopausal hot flushes, 1628t-1629t for small bowel rapid transit dysmotility, 888 for tobacco dependence, 147t for Tourette’s syndrome, 2466 Cloning, positional, 192 Clonorchiasis, 2157 clinical manifestations of, 2157 epidemiology of, 2157 pathobiology of, 2157 treatment of, 2158 Clonorchis sinensis, 1015t-1017t, 2157 Clopidogrel, 179 for acute coronary syndrome, 437, 438t-439t for acute STEMI, 448-450, 454, 454t, 455f for angina and stable ischemic heart diease, 429 for stroke prevention, 458, 2444 Clorazepate, 2352t, 2407t Clostridial infections, 1924-1931 Clostridial myonecrosis, 1926-1927 clinical manifestations of, 1927 definition of, 1926 diagnosis of, 1927 epidemiology of, 1926 pathobiology of, 1926-1927 prevention of, 1927 prognosis for, 1927 treatment of, 1927b Clostridium baratii, 1927 Clostridium botulinum, 1927 Clostridium butyricum, 1927 Clostridium difficile, 1924 toxin A, 1870t toxin B, 1870t Clostridium difficile infection, 1924-1926 clinical manifestations of, 1925 definition of, 1924 diagnosis of, 1925 epidemiologic features of, 1869t epidemiology of, 1924-1925 health care–associated, 1862-1865 burden, costs, and preventability of, 1862t guidelines and recommendations for, 1865.e1t pathobiology of, 1925 prevention of, 1925

Clostridium difficile infection (Continued) prognosis for, 1926 severe, 1925 treatment of, 1925b Clostridium novyi type A, 1924, 1926 Clostridium perfringens, 1924, 1926 Clostridium perfringens type A diarrhea, 1930-1931 Clostridium perfringens type C enteritis, 1930 clinical manifestations of, 1930 diagnosis of, 1930 pathobiology of, 1930 prevention of, 1930 treatment of, 1930b Clostridium septicum, 1926 Clostridium sordellii, 1924, 1926 Clostridium sordellii infection, 1926 associated with childbirth and medical abortion, 1926 associated with injection drug use, 1926 clinical manifestations of, 1926 definition of, 1926 diagnosis of, 1926 epidemiology of, 1926 pathobiology of, 1926 prevention of, 1926 prognosis for, 1926 treatment of, 1926b Clostridium tetani, 1928-1929 Clotrimazole for AIDS-associated opportunistic infections, 2297t-2301t for fungal infections, 2670-2671 for thrush, 2082 Cloxacillin, 1891t-1892t Clozapine, 2354, 2457t-2459t Clubbing, 254-255, 257f, 2707, 2707f Cluster headache, 2360-2361 clinical manifestations of, 2360-2361, 2360t diagnosis of, 2361 diagnostic criteria for, 2361 differential diagnosis of, 2357t epidemiology of, 2360 pathobiology of, 2360 prevention of, 2361 prognosis for, 2361 treatment of, 2361b Cluster randomized trials, 42 Clustered data, 36 Clustered regulatory interspaced short palindromic repeats (CRISPRs), 210-211, 213 Cnidaria, 720 Coagulation, 231 common pathway of, 1154-1155 contact activation (intrinsic) pathway of, 1154-1155 laboratory values, 2717t-2718t procoagulant response in sepsis, 686, 686.e1f reference intervals, 2717t-2718t response to infection, 686 tissue factor (extrinsic) pathway of, 1154-1155 Coagulation abnormalities, 717-718 acquired, 1180 in liver disease, 1183, 1184b, 1184t treatment of, 718, 1184b in viral hemorrhagic fever, 2255 Coagulation cascade, 1154-1155, 1155f-1156f Coagulation deficiency, 1172-1181 Coagulation disorders. See also Bleeding disorders in bacterial meningitis, 2484-2485 in stroke, 2443t venom-induced consumption coagulopathy, 718 Coagulation protein replacement therapy, 1174-1175, 1175t Coagulation tests, 990 abnormal, 1158 Coarctation of the aorta, 412 chest radiography with, 412, 412f CMR findings, 292f diagnosis of, 385t, 412 exercise recommendations for, 416t 3 sign of, 412, 412f treatment of, 412b-413b Coartem (artemether and lumefantrine), 2105-2106, 2111t

Index Coats’ disease, 2560 Cobalamin. See Vitamin B12 Cobalamin-processing (CblC) defect, 1405 clinical features of, 1406, 1406t diagnosis of, 1406 pathobiology of, 1404f, 1405 prognosis for, 1407 treatment of, 1407 Cobalt toxicity, 93t, 96-97 Cobb angle, 629, 630f Cobicistat fixed-dose combinations, 2289, 2290t for HIV/AIDS, 2290t side effects of, 2289 Cobra venom factor, 246 Cobras, 719 Coca-Cola, 160 Cocaine, 160, 1008t Cocaine use cardiomyopathy due to, 330 clinical manifestations of, 160 epidemiology of, 160 pathobiology of, 160 recommended treatment of, 395t screening for, 699-702, 702t Coccidia, 2146, 2146f Coccidian enteritis, 2146-2147 clinical manifestations of, 2146 diagnosis of, 2146 epidemiology of, 2146 pathobiology of, 2146 prevention of, 2147 prognosis for, 2147 treatment of, 2147b Coccidioides, 2072 Coccidioidomycosis, 2072-2073 clinical characteristics of, 2072t clinical manifestations of, 2072-2073 definition of, 2072 diagnosis of, 2073 epidemiology of, 2072 extrapulmonary dissemination, 2073 incidence of, 2072 oral ulcers, 2580t pathobiology of, 2072 prevalence of, 2072 prognosis for, 2073 pulmonary infection, 2072-2073, 2073f treatment of, 2073b Coccidioidomycosis pneumonia, 532.e1f Coccydynia, 1753 Cochlear damage, 2595 Cockcroft-Gault equations, 728, 730t Code blue teams, 2608 Codeine formulations, dosages, and pharmacologic information, 139t-140t for migraine headache, 2358 for restless legs syndrome, 2468-2469 toxicity, 699t-702t Codfish vertebrae, 1824 Coenurosis, 2153 Coenzyme Q10 (ubiquinone) deficiency, primary, 2546 diagnosis of, 2546 prognosis for, 2546 treatment of, 2546 Coenzyme Q10 (ubiquinone) supplements, 2546 Coffee, 387 Cogan’s syndrome, 1799 clinical manifestations of, 1799 treatment of, 1799 Cognitive assessment, geriatric, 104 Cognitive dysfunction executive, 2386-2388 in fibromyalgia, 1818 in heart failure, 302 Cognitive impairment, 2382 definition of, 2389 mild, 2389-2390 Cognitive therapy, 2348t, 2421t Cognitive-behavioral therapy for bulimia nervosa, 1457 for executive cognitive dysfunction, 2388 for insomnia, 2421t for smoking, 147, 147t Coitus interruptus, 1606 Colazal (balsalazide), 939t

Colchicine, 2659 for acute gout, 1814-1815 for acute pericarditis, 486 for epidermolysis bullosa acquisita, 2676 for familial Mediterranean fever, 1148, 1741 for pericardial constriction, 490 for Sweet’s syndrome, 2682 Cold: disorders due to, 691-695 Cold, common. See Common cold Cold abscesses, 2035 Cold agglutinin disease chronic, 1076-1077 epidemiology of, 1073-1074 Cold agglutinin phenomenon, 2006 Cold agglutinin syndrome, 1075t Cold agglutinins, 1075 Cold autoimmune hemolytic anemia, 1075-1076, 1077f Cold autoinflammatory disease, familial, 231 Cold autoinflammatory syndrome, familial, 1740t-1741t, 1742 Cold hemoglobinuria, paroxysmal characteristics of, 1075t classification of, 1075 epidemiology of, 1073-1074 Cold injury, 693-695 clinical manifestations of, 694 definition of, 693 diagnosis of, 694 epidemiology of, 693-694 factors predisposing to, 694t pathobiology of, 694 peripheral, 693 prevention of, 694 prognosis for, 695 treatment of, 694b-695b Cold urticaria, familial, 1131, 1742 Colesevelam for diarrhea, 933 for lipid disorders, 1396t for small bowel rapid transit dysmotility, 888 for type 2 diabetes, 1536 Colestipol, 1396t Colic, 854 Colistimethate sodium (colistin) for Acinetobacter species infections, 1969-1970 for P. aeruginosa bacteremia, 1966 for Pseudomonas infection, 1967 Colitis amebic, 2139-2140, 2141t Crohn’s, 943 cytomegalovirus, 1861, 2181t, 2297t-2301t differential diagnosis of, 937t extensive, 941-942 indeterminate, 935 ischemic, 953f, 955-956 medical management of, 941-942 microscopic, 934 neutropenic, 948, 948.e1f ulcerative, 1768, 1768t Collagen, 1737, 2634 Collagen vascular disease, 1720 in immunocompromised patients, 1855t pleural fluid characteristics, 634t in pregnancy, 1614t Collagen vascular reactions, 2685t Collagenases, 233t Collagenomas, 2688-2689 Collagenous colitis, 934 Collagens, 1731 Collecting duct carcinoma, 1346t Collecting duct natriuretics, 749 Collecting ducts acid secretion in, 739, 740f functional disorders of, 823t, 824 Colles fracture, 1637 Colloids burn resuscitation formula, 714.e1t for shock, 676 solutions containing, 746 Colon. See also Rectum; Small bowel diverticulitis of, 946-947 inflammation of, 945-948 polyps of, 1325-1327 Colon adenoma saline lift polypectomy of, 1327.e1 snare polypectomy of, 1326, 1326.e1

Colon cancer profile, 1380 Colonic diverticula, 946-947, 947.e1f Colonic microbiome and colorectal cancer, 1842 and inflammatory bowel disease, 1842 Colonic motility, 885 Colonic motility disorders, 888-890 Colonization pressure, 1915 Colonography, CT, 869-870, 869f, 876 Colonoscopy, 1329 applications, 873t surveillance, 942-943 surveillance intervals, 1326t virtual, 869-870, 869f, 1329 Color, skin, 2637, 2638t-2644t Color flow Doppler imaging, 276, 1726 Color vision change, 2558 Colorado tick fever, 2256-2257 clinical manifestations of, 2257 definition of, 2256-2257 diagnosis of, 2257 epidemiology of, 2257 pathobiology of, 2257 prognosis for, 2257 treatment of, 2257b Colorado tick fever virus, 2257 Colorectal cancer, 1322-1331 chemoprevention of, 1329 clinical manifestations of, 1327-1328 colonic microbiota and, 1842 combination therapy for, 1330 development of, 1226, 1226f diagnosis of, 1328-1329 dietary prevention of, 1329-1330 endoscopy in, 876 epidemiology of, 1322 general features of, 1323t genetic changes associated with, 1226, 1226f genetic syndromes, 1325 hereditary nonpolyposis, 1324 in inflammatory bowel disease, 942-943 inherited syndromes, 1323t metastatic locally directed treatment of, 1331 treatment of, 1331 molecular basis, 1327, 1327f molecularly guided therapeutics for, 202.e1t nutritional influences, 1429 pathobiology of, 1322-1323, 1327 predisposing conditions, 1322-1323 prognosis for, 1331 radiation therapy for, 1330 screening for, 56, 1328-1329, 1328t staging, 1329-1330, 1329t surveillance of, 1331 systemic therapy for, 1330 targeted therapy for, 202, 202t treatment of, 1329b-1331b Colorectal polyps, 878-879, 878f Colubridae, 717 Coma, 2409-2413, 2410t causes of, 2410, 2410t-2411t clinical manifestations of, 2411 diagnosis of, 2411-2412, 2411t emergency management of, 2412, 2413t epidemiology of, 2410 FOUR Score assessment of, 2411, 2411t likelihood for recovery of awareness, 2413 neurologic examination in, 2339 pathobiology of, 2410-2411 pharmacologic, 2368 prognosis for, 2413 treatment of, 2412b-2413b Comanagement, 2610 Combination therapy, 1208-1209, 1209t Combined hyperlipidemia, familial, 1393 Comet sign, 593 Cometriq (carbozantanib), 1211t-1216t Comfort care, 10 Common cold, 2185-2187, 2241t clinical manifestations of, 2185-2186 definition of, 2185 diagnosis of, 2186 epidemiology of, 2185 pain-related symptoms, 2186 pathobiology of, 2185 pathogens, 2185 prevention of, 2186

I25

Common cold (Continued) prognosis for, 2186 remedies for, 2186 treatment of, 2186b, 2186t viruses associated with, 2185t Common couple violence, 1629-1630 Common variable immune deficiency, 1679, 1680t Commonwealth Foundation, 43.e1t Communication core skills for, 14t model for discussing palliative care topics, 14t nonverbal empathy, 14t verbal empathy, 14t Community disturbance, 1839-1840 Community health centers, 16 Community health workers, 16-17 Community resources, 49 Community-acquired pneumonia, 1902 due to L. feeleii, 1995, 1995f due to L. pneumophila, 1995, 1995f-1996f incidence of, 610.e1f indications for diagnostic testing, 1904, 1904t staphylococcal, 1899 treatment of, 1905 antibiotic regimens for, 689t duration of therapy, 1896 empirical, 616t, 1905 Comparative genomic hybridization, 195 Compartment syndrome, 726, 1174 Compazine, 2573t Compensated cirrhosis, 1026-1027, 1029f Compensatory responses, 672, 765-766, 765t Complement, 231, 1722 activation of, 241-246, 245f alternative pathway, 241-244 classical pathway, 241-242 lectin pathway, 242 pathologic conditions associated with, 241t pathways of, 241, 242f regulators of, 241, 244, 246f tissue injury or degeneration and, 241, 242t in glomerular disorders, 785t in glomerular syndromes, 734 in systemic lupus erythematosus, 1770-1771 Complement C1 inhibitor, 243t Complement C1 inhibitor concentrate (Berinert, Cinryze), 1697 Complement C1 inhibitor deficiency, 1682, 1682b Complement C3 convertase, 244 Complement C3a, 245 Complement C3a receptor, 245 Complement C3d receptor, 2232 Complement C4 binding protein (C4bp), 243t Complement C4b/C3b receptor (CR1, CD35), 243t Complement C5, 243f, 246 Complement C5 convertase, 244 Complement C5a, 245 Complement C5a receptor (C5aR [CD88]), 245 Complement C5b-9, soluble, 245 Complement defects, 1681t Complement deficiency, 1677t Complement disorders, 1681-1682, 1682b, 1740t Complement factor H gene (CFH), 2574 Complement inhibitors, 246 Complement receptors, 217-218, 246, 1146 Complement regulating complement, 243t Complement system in disease, 240-246, 241t features of, 241, 241t function of, 240, 240f in host defense, 240, 241t Complementary and alternative medicine (CAM), 181-184 definition of, 181 for osteoarthritis, 1749 for pain, 142 use of, 182t Complementary health approaches, 181 Volume I  pp 1-1382  •  Volume II  pp 1383-2722

I26

Index

Complex genomic rearrangements, 192-193, 192f Complex regional pain syndrome, 2522 Comprehensive chronic disease management, 46-49 Comprehensive postoperative care, 2621 Compression stockings, 1191, 1884 Compression ultrasonography, 512-513, 513f Compressive focal motor neuropathy, 2524t Compressive mononeuropathy, 2535 Compressive neuropathy, 2536-2537 Compulsions, 2352-2353, 2352t Computed tomography, 85 abdominal, 1040 in acute pancreatitis, 962, 962f brain imaging, 2439, 2439f-2440f Canadian CT Head Rule, 2366, 2366t cardiac, 287-288, 287f in chronic pancreatitis, 964, 964f fused PET/CT scanning in gastroenterology, 872, 872f in head and neck cancer, 1300, 1300f with gallstones, 1040 in gastroenterology, 868-870, 868f in head and neck cancer, 1300 high-resolution, 578-579 in interstitial lung disease, 578-579 with kidney stones, 734, 734f, 812, 813f of lungs, mediastinum, and chest wall, 532 in minor head injury, 2366t quantitative (QCT), 1641 radiation exposure, 82 renal scans, 733 in rheumatic disease, 1725, 1725f single-photon emission computed tomography in epilepsy, 2404 myocardial perfusion imaging, 285-286, 286f, 286.e1f for myocardial viability determination, 286 strengths and weaknesses, 2345t stress-only protocol, 286 strengths and weaknesses, 2345-2346, 2345t Computed tomography angiography in atherosclerotic PAD, 500, 501f coronary, 287, 287f-289f, 297f, 2617t in coronary artery disease, 425 preoperative, 2617t pulmonary, 621-622, 622f strengths and weaknesses, 2345-2346 Computed tomography colography, 1329 Computed tomography colonography, 869-870, 869f, 876 Computed tomography myelography, 2345t Computer navigation, 1832 Computerized ECG readings, automated, 273 Concussion guidelines for diagnosis and management of, 2366t labyrinthine, 2599 Condoms female, 1605t, 1606f, 1607 male, 1604, 1605t, 1606f, 1607, 1884 Conduction delays, 452 Conductive hearing loss differential diagnosis of, 2594 pathobiology of, 2594 Condyloma. See Warts Confidence, 52 Confidence intervals, 35 Confidentiality, 62-63 Conflicts of interest, 8-9 Confocal microscopy, gastrointestinal, 878-879, 878f Confounding, 36 Confusion Assessment Method (CAM), 120 Confusional arousals, 2422-2423, 2423t Congenital abnormalities of aortic valve, 461 coronary anomalies, 442t cystic adenomatoid malformation of the lung, 570 of esophagus, 908 HgbA1C and, 1620, 1620f of pericardium, 491 pulmonary airway malformation, 570 structural abnormalities, 943-944 Congenital adrenal hyperplasia, 1519, 1562-1564, 1563f, 1584

Congenital cataract, 2571 Congenital defects, 1573 neural tube defects, 1430 Congenital disorders, 2512-2514 complex: hypoparathyroidism associated with, 1652t cystic diseases, thoracic, 570-571 embryopathic, 1476 esophageal, 908 of glycosylation, 1385t, 1387 Congenital enteropeptidase (enterokinase) deficiency, 929 Congenital erythropoietic porphyria, 1408t classification of, 1408 clinical manifestations of, 1411 epidemiology of, 1408 prevention of, 1415 skin disease in, 1412-1413 treatment of, 1414-1415 Congenital heart disease, 405-417 acyanotic, 405 assessment of, 406, 406f clinical manifestations of, 406-407 CMR findings, 289-292, 292f complex lesions, 405 cyanotic, 405 definitions, 405 echocardiographic findings, 280t-281t epidemiology of, 405 exercise recommendations for, 416t, 417 exercise testing, 416-417 genetic determinants, 405 incidence of, 405 isolated shunt lesions, 407-408 native lesions, 405 palliative interventions for, 405, 405t physiologic repair of, 405 prevalence of, 405 prevention of, 405 simple lesions, 405, 407-413 surgical shunts for, 405t, 406 treatment of, 407b, 407f ventricular tachycardia in, 372 Congenital Heinz body hemolytic anemia, 1095 Congenital hypopigmentation, 2699 Congenital hypoventilation, central, 547 Congenital infections, 2230 Congenital insensitivity to pain with anhidrosis, 2519 Congenital large hyperlucent lobe, 570 Congenital lobar emphysema, 570 Congenital muscular dystrophy, 2544-2546 Congenital myasthenic syndromes, 2548t, 2553 clinical manifestations of, 2553 diagnosis of, 2553 pathobiology of, 2553 prognosis for, 2553 treatment of, 2553b Congenital myopathy, 2541t, 2544 Congenital neutropenia, severe, 1133-1134, 1133t, 1683, 1682.e1t Congenital neutropenia syndromes, 1133, 1133t Congenital nonspherocytic hemolytic anemia, 1086 Congenital syphilis clinical manifestations of, 2016 diagnosis of, 2018 treatment of, 2019 Congenital toxoplasmosis diagnosis of, 2130 treatment of, 2131, 2132t Congenital triangular alopecia, 2706 Congenital valvular aortic stenosis, 411-412 diagnosis of, 412 treatment of, 412b Congestion, 682, 682t Congestive colopathy, 958 Congestive enteropathy, 958 Congestive gastropathy, 958 Conivaptan, 752-753 Conjugate gaze: disorders of, 2578-2579 Conjugated or mixed hyperbilirubinemia, 986-987 Conjunctiva, 2556, 2572 Conjunctival injection and tearing: short unilateral neuralgiform headache with, 2357t, 2360 Conjunctival suffusion, 2028-2029, 2029f

Conjunctivitis, 2559-2560 acute, 2559t acute hemorrhagic, 2241t, 2243 adenoviral, 2564-2565 allergic, 2564, 2564t antibiotic regimen for, 1945t bacterial, 2564t, 2565, 2565f chlamydial, 2010, 2564t, 2565 H. influenzae, 1947 herpes simplex, 2564t inclusion, 2010 ophthalmic disorders associated with, 2564t viral, 2564t, 2565f Connective tissue(s), 1730-1733 aging of, 1733 elastic, 2634 elements of, 1730 function of, 1730, 1730t matrix, 1730t, 2634 neoplasms of, 1371-1373 regulation of, 1731-1732 structure of, 1730t turnover, 1732-1733 types of, 1730, 1730t Connective tissue cells, 1730-1732, 1730t Connective tissue disease, 1712, 1720, 1732 chronic febrile illness, 1852-1853, 1852t inherited, 1733-1739 interstitial lung disease associated with, 575t, 584-586 mixed, 585, 1778 undifferentiated, 1720 Connective tissue hyperplasias, 2583, 2583t Consciousness, 2409 dimensions of, 2409 disorders of, 2409, 2410t minimally conscious state, 2410t, 2414, 2414t Consortium of Academic Health Centers for Integrative Medicine, 184 Constipation, 888-889 approach to, 852t, 861 diagnosis of, 889 epidemiology of, 888 management of algorithm for, 889f approaches to, 11t in palliative care, 11t medications for, 893t pathobiology of, 888-889 surgical therapy for, 889 treatment of, 889b Constrictive pericarditis, 489, 490f Consultants, 2610, 2611f, 2625 Consultations curbside, 2610 electronic, 2610 guidelines for, 2610t informal, 2610 mandatory, 2610 medical, 2608-2611, 2611f medical-medical, 2608 referral for intimate partner violence, 1631 referral to behavioral change specialists, 54 referral to ENT specialists, 2589t for special populations, 2608-2609 subspecialty, 2609, 2609t surgical postoperative, 2608 preoperative, 2608 Consumption coagulopathy, venom-induced, 718 Contact activation (intrinsic) factor deficiency, 1178 Contact activation (intrinsic) pathway, 1154-1155 Contact dermatitis allergic, 2639f, 2656f, 2663-2664, 2687 drug-induced, 2687 irritant, 2663-2664 Contact tracing, 2255 Continence, 885 functions during, 885, 885f regulation of, 799-800, 799.e3f Continuous positive airway pressure, 665 for acute respiratory failure, 660 for obstructive sleep apnea, 640-641 Contraception, 1604-1610 for adolescents, 64 barrier, 1606-1607 emergency, 1609

Contraception (Continued) hormonal (steroidal), 1607-1609 injectable, 1609 intravaginal ring, 1609 transdermal patch, 1609 Medical Eligibility Criteria (MEC), 1605-1606 natural, 1606 oral. See also Oral contraceptives failure rates, 1605t hypertension associated with, 393-394 perfect use, 1604-1605, 1605t for prevention of STIs, 1881 risks and benefits, 1605-1606 subdermal implants, 1609 types of, 1606-1610 typical use, 1604-1605, 1605t Contraceptives, 1604-1606. See also Oral contraceptives contraindications to, 1605-1606 effectiveness of, 1604 failure rates, 1604-1605, 1605t, 1606f Contractile cells, 263 Contraction, 264, 263.e1f Contraction coupling, 1822, 263.e1f Contrast echocardiography, 277, 278f Contrast fluoroscopy, 867, 867f Contrast venography, 513, 514f Contrast-enhanced magnetic resonance imaging, 870 Control-driven therapy, 554 Controlled ovarian hyperstimulation, 1596 Controller drugs, 552-554 Conus medullaris syndrome, 2379 Convalescent plasma, 2255 Convergence, 2577 Convergence center, 2577 Conversion disorder, 2355t Convertases, 244 Conviction-confidence ruler, 52, 53f Convulsions, benign, 2404 Convulsive seizures, 2402 Cooley’s anemia, 1091, 1824 Coombs test, 1075 Copaxone (glatiramer acetate), 2477 COPD. See Chronic obstructive pulmonary disease Copeptin, 304.e1t Coping (relapse-prevention) therapies, 147 Copper, 1450t-1452t for copper deficiency, 2507t hepatic values, 1418 for malabsorption, 931t for Menkes disease, 2507t Copper deficiency, 2507t, 2510 Copper T380A IUD, 1609-1610 Copperhead snakes, 719 Coproporphyria, hereditary, 1408t classification of, 1408 epidemiology of, 1408 etiology of, 1410 skin disease in, 1412 Coproporphyrinuria, 1414 Copy-number variants, 190-191, 193f, 194 Coral snakes (elapidae), 719 Corifact (CSL Behring), 1179 Corkscrew esophagus, 904, 904f Cornea, 2556 erosion of, recurrent, 2563 infection of, 2559t trauma to, 2559t Corneal abrasion, 2563 Corneal stromal dystrophy, 2571 Cornelia de Lange’s syndrome, 2558 Cornell voltage criterion hypertrophy, 273 Cornstarch, uncooked, 1399 Coronary angiography in acute coronary syndrome, 435, 438t-439t in chronic angina and stable ischemic heart disease, 425t computed tomography, 287, 287f-289f, 295, 297f, 425, 2617t in coronary artery disease, 425 diagnostic, 425-426, 425t in heart failure, 305 preoperative, 2617t procedures that accompany, 293t Coronary angioplasty, 458 for acute MI, 458, 459f for atherosclerotic PAD, 502, 503f

Index Coronary angioplasty (Continued) percutaneous transluminal, 502, 503f technique, 456, 456f Coronary artery anomalies, 295, 297f, 413 anomalous anatomy of, 287, 287f, 297f anomalous origin of, 413 calcium scoring, 287, 287f congenital anomalies of, 442t fistulas, 412 Coronary artery bypass graft surgery, 317, 456, 458 for acute STEMI, 460 for chronic stable angina, 460 for coexisting cardiac disease, 460 comparison with PCI, 460-461, 460f for failed PCI, 460 indications for, 459-460 for ischemic heart disease, 431 for ischemic syndromes, 460 late outcomes, 459 for multivessel disease, 431 for myocardial infarction, 460 perioperative risks, 459 recommendations for, 428t reoperations, 460 for survival, 460 for symptom relief, 460 types of grafts, 458-459, 459f for unstable angina, 460 Coronary artery disease antithrombotic therapy for, 180t detection of, 285-286 diagnostic tests for, 424t echocardiographic findings, 280t-281t infiltrative and degenerative, 442t interventional treatment of, 456-461 molecularly guided therapeutics for, 202.e1t MR imaging findings, 288 nutritional influences on, 1426 predicted risks for, 260f-261f probability of, 421t, 424f prognostic index, 426t risk for blood pressure and, 381, 381f, 387.e1f screening for, 54-55 risk stratification, 426-427 thrombotic, 442t Coronary artery dissection, spontaneous, 442t Coronary artery surgery, 458-459. See also Coronary artery bypass graft surgery Coronary blood flow, 264 Coronary emboli, 442t Coronary intensive care, 448-449 Coronary ostial occlusion, 442t Coronary revascularization, 438t-439t, 440 Coronary stenosis, 295, 296f Coronary stents, 456-457, 457f balloon-expandable, 456-457, 457f delivery, 457.e1 inflation of, 457.e1 placement of, 457.e1 Coronary vasculitis, 442t Coronary vasospasm, 442t Coronary vessels, 296f Coronavirus infections 229E, OC43, HKU1, and NL63, 2201 clinical manifestations of, 2200 diagnosis of, 2200-2201 epidemiology of, 2199-2200 MERS, 2200-2201 clinical manifestations of, 2200, 2200t treatment of, 2201 pathobiology of, 2200 pneumonia, 612, 613f prevention of, 2201 prognosis for, 2201 SARS, 2201 clinical manifestations of, 2200, 2200t treatment of, 2201 treatment of, 2201b Coronaviruses, 2185, 2185t, 2199-2201 Corrigan’s pulse, 471 Cortical blindness, 2386 Cortical collecting duct, 764, 756.e1f Cortical hyperostosis with syndactyly, 1669 Cortical thrombophlebitis, 2481

Corticosteroids, 162-163 adverse reactions to, 164-165 for AIDS-associated opportunistic infections, 2297t-2301t for asthma, 552, 553t for bacterial meningitis, 2488 cardiovascular effects of, 165 classification of, 163t for COPD, 559, 560t for EBV infection, 2233 exogenous, 2616 genomic effects of, 163 indications for, 163-164 for inflammatory bowel disease, 939-940, 939t inhaled, 552, 553t for interstitial lung disease, 579, 579t intranasal, 1692, 1692t issues with, 163 mechanism of action, 162 for membranoproliferative glomerulonephritis, 788 metabolic effects of, 164 molecular action of, 163 nongenomic effects of, 163 ocular effects of, 2573t perioperative, 1833, 2613t for Pneumocystis pneumonia adjunctive, 2096-2097 in HIV infection, 2314 risks, 2096-2097, 2096.e1f preoperative evaluation of, 2616 for prostate cancer, 1369 for pruritus, 2636t for septic shock, 690 for spondyloarthritis, 1764 stress doses, 164 systemic for asthma, 553 for chronic pain, 138t systemic effects of, 163 tapering, 164-165 for urticaria, 2684 for warm autoimmune hemolytic anemia, 1077 Corticotropin-releasing hormone, 1474 Corticotropin-releasing hormone test, 1483t, 1487t Cortisol midnight plasma levels, 1515 midnight salivary levels, 1487t plasma levels, 1515 salivary levels, 1487t, 1515 serum levels, 1515 urine free, 1515 urine levels, 1515 Cortisone, 164t Corus CAD (CardioDx, Inc.), 203 Corynebacterium, 1915-1918, 1918t Corynebacterium amycolatum, 1918t Corynebacterium diphtheriae, 1915-1916, 1918t, 2603 Corynebacterium glucuronolyticum, 1918t Corynebacterium jeikeium, 1918t Corynebacterium kroppenstedtii, 1918t Corynebacterium minutissimum, 1918t Corynebacterium pseudodiphtheriticum, 1918t Corynebacterium pseudotuberculosis, 1918t Corynebacterium riegelli, 1918t Corynebacterium striatum, 1918t Corynebacterium ulcerans, 1916, 1918t Corynebacterium urealyticum, 1918t Cosmetics, 2661 Cost-benefit analysis, 40-41 Cost-containment strategies, 19 Cost-effectiveness analysis, 40-41, 41t Costello syndrome, 1227t Costimulatory blockade, 239 Costs. See also Health care expenditures of AIDS, 2272 of cardiovascular diseases, 258 quality and, 46 safety and, 46 Cotton dust fever, 589t Cotton mill fever, 594t Cottonmouth snakes (Agkistrodon), 719 Cough acute, 526-527, 526t approach to, 526-527 causes of, 526t

Cough (Continued) chronic, 526t, 1991 definition of, 527 diagnosis of, 527, 527t management of, 527f diagnosis of, 527, 527t management of, 527f treatment of, 527b-528b, 2186 whooping cough (pertussis), 1990-1993 Coumadin (warfarin), 1618t Coumarin derivatives, 516 Counseling for behavior change, 52-54 diet, 57 drug, 315t fitting into medical practice, 52 genetic, 9, 1388 medication use, 315 motivational, 52, 53f for smoking, 147, 147t pretravel consultation with developing world travelers, 1881, 1882t for prevention of STIs, 1880 recommendations for general population, 55t strategies used by alcohol treatment programs, 155 Counseling teamwork, 54 Count variables, 32 Couple therapy, 1632 Coupling, 1646 Cowden’s syndrome, 1227t, 2688, 2688f Cowpox, 2218 Cowpox virus, 2215 Cox (proportional hazards) model, 35 Coxibs, 174-175 inhibition by, 173, 173f, 175 in vascular events, 175 Coxiella, 2047t Coxiella burnetii, 477t, 2055 CR1 (CD35) (immune adherence or C4b/ C3b receptor), 243t Crab lice, 2176 clinical manifestations of, 2176 pathobiology of, 2176 treatment of, 2176b CRAFFT screening tool, 65t Cramps, 2338 benign cramp-fasciculation syndrome, 2524t muscle cramps, 2340, 2539 musician’s cramp, 2463-2464 writer’s cramp, 2463-2465 Cranial dystonia, 2465 Cranial neuropathy, 2535 Cranial polyneuritis, parainfectious, 2599 Craniopharyngiomas, 1476, 2575 Crazy paving, 572, 572f, 602, 602f, 572.e1f C-reactive protein, 1719 Creatine deficiency, 1385t Creatine kinase, 725 Creatine metabolism: disorders of characteristic signs of, 1385t pathophysiologic mechanisms of, 1385t Creatinine albumin-to-creatinine ratio, 1545 plasma levels, 728 serum levels, 728, 729f in acute kidney injury, 778, 779t in hypovolemia, 745 normal, 796 urine levels, 728 Creatinine clearance (CCr) calculation of, 385 formula for, 728 Creola bodies, 551 Creon, 966t Crescentic (rapidly progressive) glomerulonephritis, 790-791, 790t, 791b, 791f CREST syndrome, 958 Creutzfeldt-Jakob disease, 2504 diseases that mimic, 2505t epidemiology of, 2504 new-variant, 2058t, 2060t slowly progressive, 2505 sporadic clinical manifestations of, 2505 genetics of, 2505 prognosis for, 2506

I27

Creutzfeldt-Jakob disease (Continued) variant clinical manifestations of, 2505 epidemiology of, 2504 pathology of, 2504-2505 Cricopharyngeal bars, 904 Crigler-Najjar syndrome type 1, 984-986 bilirubin disposition in, 984 principal features of, 987t Crigler-Najjar syndrome type 2, 985-986 bilirubin disposition in, 984 principal features of, 987t Crimean-Congo hemorrhagic fever, 2249t clinical manifestations of, 2251t epidemiology of, 2250 management of, 2254-2255 pathobiology of, 2250, 2251t prevention of, 91 symptoms and signs of, 2250-2251 Crimean-Congo hemorrhagic fever virus, 91t, 2249t CRISPRs (clustered regulatory interspaced short palindromic repeats), 210-211, 213 Critical care approach to, 650-652 care of dying patients and their families, 9-15 for injured or burned, 715-716 long-term outcomes, 650 predictions, 651-652 preferences for, 651-652 prevention of complications, 650-651 rehabilitation, 716 respiratory monitoring in, 652-655 for septic shock, 690, 691f for type 2 diabetes, 1536 Critical care physicians, 650 Critical illness neuropathy, 2534-2535 clinical manifestations of, 2534 definition of, 2534 diagnosis of, 2534-2535 differential diagnosis of, 2534-2535 epidemiology of, 2534 pathobiology of, 2534 prognosis for, 2535 treatment of, 2535b Critical limb ischemia, 503f Crizotinib, 202, 1310t, 1311 CRM197, 1917 Crocodile tears, 2537 Crohn’s colitis, 943 Crohn’s disease, 1740t clinical manifestations of, 936t colonic microbiota and, 1842 complications of, 942 diagnosis of, 938 endoscopic findings, 938, 938f enteropathic arthritis associated with, 1768, 1768t epidemiology of, 935 esophageal involvement in, 906 extraintestinal manifestations of, 937, 937t fistulizing, 969f medical therapy for, 940-941 treatment of, 969 gastroduodenal ulcers in, 911 genetics of, 935-936 immunomodulatory therapy for, 940 incidence of, 935 medical therapy for, 939-941 mild to moderate, 940 moderate to severe, 940 MR enterography findings, 871, 871f pathology of, 936 pathophysiology of, 936 perianal, 942 physical findings, 937 in pregnancy, 943 prevalence of, 935 prognosis for, 943 radiologic imaging findings, 938-939 serologic markers for, 939 surgical therapy for, 942 symptoms of, 936-937 treatment of, 941f Crohn’s disease-associated fistulas, 940-941, 969, 969f Cromolyn sodium, 1709 Cronkhite-Canada syndrome, 1325 Volume I  pp 1-1382  •  Volume II  pp 1383-2722

I28

Index

Crossmatching, 1192-1193 Cross-presentation pathway, 221-222 Crotalidae, polyvalent, 66t Crotalinae, 717 Crotaline antibenom (CroFab), 719 Crotalus (rattlesnakes), 719 Crotamiton (Eurax), 2173, 2659 Croup, 2189 severe, 2190 treatment of, 2190 Crowe’s sign, 2689 Crush injury, 712f, 712t clinical manifestations of, 711-713 management of, 726 Crush injury syndrome, 724 Crustaceans, 2175 Crusts, 2636t, 2651t-2654t Cruzan case, 6 Cryoglobulin(s), 735, 1059t, 1283, 1723 Cryoglobulinemia, 1273, 1283-1284 clinical manifestations of, 1799 essential mixed, 1723 gastrointestinal involvement, 957 mixed, 793, 2533, 2533t premalignant, 1273t skin infarction in, 1283, 1283f treatment of, 1799 type I, 735, 1283 type II, 735, 1283-1284 type III, 735, 1284 Cryoprecipitate, 1058, 1175 Cryopyrin, 1742 Cryopyrin-associated periodic syndromes, 1742, 1742b Cryopyrinopathies, 1742, 1742b Cryotherapy, 2222t Cryptic shock, 672 Cryptococcal meningitis, 2076-2077 AIDS-associated, 2297t-2301t in immune reconstitution inflammatory syndrome, 2334 treatment of, 2067, 2077, 2297t-2301t Cryptococcal polysaccharide antigen (CRAG) assay, 2077 Cryptococcosis, 2076-2078 central nervous system infection, 2076-2077 clinical manifestations of, 2076 definition of, 2076 diagnosis of, 2076-2077 epidemiology of, 2076 extrapulmonary, 2297t-2301t in HIV infection, 2295, 2297t-2301t, 2320 pathobiology of, 2076 prognosis for, 2077 prophylaxis of, 2077 pulmonary, 2076-2077, 2297t-2301t treatment of, 2077b Cryptococcus gattii, 2076 Cryptococcus neoformans, 2076 Cryptogenic fibrosing alveolitis, 2237t Cryptogenic hepatitis, 1000t, 1006 Cryptogenic organizing pneumonia, 583-584 diagnosis of, 583-584, 584f treatment of, 584b Cryptogenic stroke, 2444-2445 Cryptopenia, 1073-1074 Cryptorchidism, 827 Cryptosporidiidae, 2133 Cryptosporidiosis, 2133-2135 childhood, 2135 clinical manifestations of, 2134-2135 definition of, 2133 in developing countries, 2135 diagnosis of, 2135 epidemiology of, 1869t, 2133 in immunocompromised patients, 2135 laboratory findings, 1871f pathobiology of, 2133-2134 prevention of, 2135 treatment of, 2135b Cryptosporidium, 1015t-1017t, 2133, 2134f Cryptosporidium diarrhea, 2057t Crystal deposition diseases, 1713t, 1811-1816 Crystalline arthropathies, 1740t Crystalloid solutions for hypovolemia, 745 for shock, 676 for therapeutic hypothermia, 695 Crystals, urinary, 731-732, 732f

CSL Behring, 1170, 1179 Cuban tobacco-alcohol amblyopia, 2510 Cubital tunnel syndrome, 2536 Cubozoa stings, 720 Cuff-leak volume, 671 Cullen’s sign, 858t, 961 Cultural competence, 16-17 Cultural competency training, 17 Cultural differences, 15 Culture definition of, 15 of safety, 46 Cultures bacterial, 1886, 1933 Bartonella, 2000 blood, 2484 brucellosis, 1981 mucormycosis, 2089-2090 N. gonorrhoeae, 1941 nasopharyngeal, 1992, 1992f Nocardia, 2064 respiratory tract, 2484 rhinitis, 2587 Rickettsia, 2049 sinusitis, 2587 stool in cholera, 1951t in infectious diarrhea, 1869-1870, 1870t, 1871f viral meningitis, 2491 Cupula, 2597 Curbside consultations, 2610 Curling’s ulcers, 911 Curschmann’s spirals, 551 Cushing’s disease, 1490-1491 clinical features of, 1490-1491, 1490f, 1490t diagnosis of, 1491 medical therapy for, 1491-1492, 1492f pathobiology of, 1490 radiation therapy for, 1491 treatment of, 1491b-1492b Cushing’s response, 2518 Cushing’s syndrome, 1490, 1514-1517 clinical features of, 1514, 1515t cortisol levels in, 1515 diagnosis of, 385t, 1514-1517, 1516f, 1611-1612 differential diagnosis of, 1491, 1491t, 1516-1517 etiology of, 1516t fat distribution in, 1461-1462 laboratory findings, 1514-1516 medical therapy for, 1517 physical findings, 1469 physiology of, 1515f in pregnancy, 1611-1612 radiation therapy for, 1517 surgical therapy for, 1517 symptoms of, 1468 treatment of, 1517b Cushing’s ulcers, 911 Cutaneous anaplastic large cell lymphoma, 1266 Cutaneous anthrax clinical manifestations of, 1921 epidemiology of, 1920 fatality rate, 2060t prognosis for, 1923 stages of, 1921 Cutaneous candidiasis, 2080 Cutaneous diffuse large B-cell lymphoma, 1265 Cutaneous diphtheria, 1917 Cutaneous emboli, 2675 Cutaneous HPV infections, 2220 Cutaneous infections, 2695-2698 Cutaneous larva migrans, 2160, 2160f Cutaneous leishmaniasis, 2057t, 2123-2125 clinical features of, 2123, 2124f complex manifestations of, 2123-2125 diagnosis of, 2123 diffuse, 2123 epidemiology of, 2120 parasitology of, 2123 recidivans, 2123-2125 serology for, 2123 treatment of, 2123b-2125b, 2125.e1f Cutaneous leukocytoclastic angiitis clinical manifestations of, 1799 management of, 1799 pathologic characteristics of, 1795t

Cutaneous lymphoproliferative disorders, 1266 Cutaneous mastocytosis, 1706, 1707f classification of, 1709 diagnosis of, 1707-1708 Cutaneous microbiome, 1841-1842 Cutaneous nocardiosis, 2063 Cutaneous phaeohyphomycosis, 2102 Cutaneous sarcoidosis, 2691, 2691f Cutaneous syndromes, 1858-1859, 1859t Cutaneous systemic sclerosis diffuse, 1781 limited, 1781 Cutaneous T-cell lymphoma, 1265 Cutaneous tularemia, 2060t Cutaneous vasculature, 2635 Cutaneous warts, 2221. See also Warts CVD 103-HgR vaccine, 1952 Cyanide antidote kit, 706t-710t Cyanide poisoning, 599 antidote for, 706t-710t clinical manifestations of, 599 diagnosis of, 599 pathobiology of, 599 treatment of, 599b-600b, 706t-710t Cyanocobalamin, 1112 Cyanosis, 254-255, 256f, 405, 652 Cyclic citrullinated peptide, 1719-1720 Cyclic guanosine monophosphate−adenosine monophosphate synthase, 217 Cyclic neutropenia, 1133t, 1134, 1682.e1t Cyclizine, 865t Cyclooxygenase, 235 Cyclooxygenase inhibitors, 178-179, 174.e1f Cyclooxygenase-1 (COX-1), 172-174, 173f-174f Cyclooxygenase-2 (COX-2), 172-173, 173f, 175, 232 Cyclopentolate, 2570 Cyclophosphamide (Cytoxan, Neosar), 168 for acute interstitial nephritis, 796 adverse reactions to, 168 for amyloidosis, 1286 for bullous pemphigoid, 2675 for Burkitt-like ALL, 1243-1244 for cancer, 1211t-1216t cardiomyopathy due to, 330 for chronic lymphocytic leukemia, 1255 dose-response effects, 1008t for epidermolysis bullosa acquisita, 2676 for glomerulonephritis, 791 for Goodpasture’s syndrome, 574 for granulomatosis with polyangiitis, 586, 1798, 2534 for hemophilia, 1177 for Hodgkin’s lymphoma, 1272 indications for, 168 for leukemia, 1243t for lupus nephritis, 792 mechanism of action, 168 for membranous nephropathy, 787 for minimal change disease, 785-786 for mucous membrane pemphigoid, 2676 for multiple myeloma, 1279, 1280t for non-Hodgkin’s lymphoma, 1263, 1263t for pemphigus, 2677 for polyarteritis nodosa, 1796 for sarcoidosis, 607t for systemic lupus erythematosus, 1775-1776 Cyclospora cayetanensis, 2146, 2146f Cyclospora infection, 1869t, 1871f Cyclosporine adverse effects of, 167, 1037t for anaphylaxis prevention, 1702-1703 for aplastic anemia, 1079, 1120 for Behçet’s syndrome, 1799 for chronic lymphocytic leukemia, 1256 dose-response effects, 1008t for dry eye, 2562-2563, 2570 for epidermolysis bullosa acquisita, 2676 for focal segmental glomerulosclerosis, 786 for immunosuppression, 167, 1037-1038 indications for, 167 for inflammatory bowel disease, 939t for lichen planus, 2668t mechanism of action, 167 for membranous nephropathy, 787 for minimal change disease, 785-786 monitoring, 1037t for myasthenia gravis, 2551 for neutropenia, 1137

Cyclosporine (Continued) for pityriasis rubra pilaris, 2667 for pruritus, 2636t for psoriasis, 2666t for renal transplantation, 844t for urticaria, 1695, 2684 Cyclosporine collyrium, 1788 Cymbalta (duloxetine), 893t Cyproheptadine, 2660t for drug overdose, 699t-702t for migraine headache prevention, 2359 for serotonin syndrome, 2628 for urticaria, 1695 Cyst (tissue cestode) infection, 2149-2153 Cystathionine β-synthase (CBS) deficiency, 1404-1405 clinical features of, 1405, 1406t diagnosis of, 1406 incidence of, 1405 pathobiology of, 1404f, 1405 prevalence of, 1405 prognosis for, 1407 treatment of, 1406-1407 Cystatin C, 304.e1t Cysteamine, 823 Cystic diseases adventitial, of lower extremity arteries, 504 congenital, of thorax, 570-571 congenital adenomatoid malformation of the lung, 570 cystic kidney disease, 816-822 Cystic echinococcosis, 2151-2152 chemotherapy for, 2152 clinical manifestations of, 2151 definition of, 2153 diagnosis of, 2151-2152 diagnostic procedures for, 2152 epidemiology of, 2151 follow-up, 2152 imaging of, 2151-2152 serology for, 2152 treatment of, 2152b percutaneous, 2152 surgical, 2152 ultrasound classification of, 2151, 2152f watch and wait with, 2152 Cystic fibrosis, 562-566, 1386t arthritic manifestations of, 1827 clinical features of, 528t, 564-565 definition of, 562 diagnosis of, 565 in adults, 565, 565t newborn, 565 epidemiology of, 562 gastrointestinal disease in, 564 gene therapy for, 212 general care for, 566 genetics of, 563-564 intestine pathobiology, 563 liver disease in, 563, 983 liver transplantation for, 1033.e1t lung disease in, 562-564, 563f-564f male reproductive tract pathobiology, 563 newborn screening and diagnosis, 565 organ involvement, 563-566 pancreas pathobiology, 563 pathobiology of, 562-564 prevalence of, 562 prevention of, 566 prognosis for, 566 sinus pathobiology, 562-563 sweat gland pathobiology, 563 treatment of, 565b-566b Cystic fibrosis transmembrane conductance regulator (CFTR) protein, 562-564, 563t Cystic hydatid disease, 2147-2148, 2151-2152 Cystic hydatidosis, 2151 Cystic kidney disease, 816-822 acquired, 822 clinical features of, 817t, 822 diagnosis of, 822 prevention and treatment of, 822b prognosis for, 822 clinical features of, 817t definition of, 816 epidemiology of, 816 inherited syndromes, 821-822 medullary, 817t, 821 pathology of, 816, 816f, 820.e1f Cystic medial necrosis, 1735

Index Cysticercosis, 2147-2151, 2150f calcified lesions, 2150 cerebral, 2058t clinical manifestations of, 2150 definition of, 2149-2150 diagnosis of, 2150 epidemiology of, 2150 parenchymal, 2150 pathobiology of, 2150 prognosis for, 2151 single lesion, 2150 subarachnoid, 2150 treatment of, 2151b ventricular, 2150 Cystine stones, 812, 816, 822 Cystinosis, 822-823, 823t, 1385t, 1403 Cystinuria, 731-732, 732f, 812, 814, 822, 823t, 1386t Cystitis differential diagnosis of, 1874, 1874t treatment of, 1875, 1875t Cystoisospora belli, 2146, 2146f Cystoscopy, 831 Cysts arachnoid, 1292 Baker’s, 1753, 1757-1758, 1757.e1f benign pericardial, 491-492 bronchogenic, 570 choledochal, 1046, 1046t, 1046.e1f duplication, 908 echinococcal, 1014, 1015f G. lamblia, 2136, 2136f hepatic, 980 hydatid, 1014 mesenteric, 950 multicystic dysplastic kidney, 825 omental, 950 pericardial, benign, 491-492 polycystic ovarian–like disorders, 1589 polycystic ovary syndrome, 1584 popliteal, 1753, 1757-1758, 1757.e1f renal, simple, 816, 816f, 817t thoracic, 570 Cytarabine (Cytosar-U, Tarabine PFS) for cancer, 1211t-1216t for leukemia, 1243-1244, 1243t Cytisine, 147t Cytochrome P450, 189t Cytochrome P450 oxidoreductase deficiency, 1564 Cytogenetic abnormalities, 1277 Cytokine receptors, 218 Cytokine release syndrome, 171, 1256 Cytokines, 231.e1t, 222.e1t anti-inflammatory, 235 in diabetic nephropathy, 804 pro-inflammatory, 232 type 2, 1689 Cytomegalovirus, 2229-2232 definition of, 2229-2230 structure of, 2229-2230, 2230f Cytomegalovirus colitis, 1861 AIDS-associated, 2297t-2301t antiviral therapy for, 2181t Cytomegalovirus esophagitis AIDS-associated, 2297t-2301t antiviral therapy for, 2181t Cytomegalovirus hepatitis diagnosis of, 1037 liver histology of, 1037.e1f Cytomegalovirus infection, 2208-2209, 2230 AIDS-associated, 2230, 2295, 2297t-2301t antiviral therapy for, 2181t clinical manifestations of, 2208-2209, 2230 congenital, 2230 diagnosis of, 2208-2209, 2231, 2231f epidemiology of, 2230 esophageal, 907 HIV-associated, 2309t in immune reconstitution inflammatory syndrome, 2334-2335 immunization against, 66t after lung transplantation, 646 neonatal, 2230 pathobiology of, 2230 prevention of, 1192, 2231 prognosis for, 2232 in transplant recipients, 2230 treatment of, 2209b, 2231b, 2232t, 2297t-2301t

Cytomegalovirus pneumonia, 2230f, 2297t-2301t Cytomegalovirus retinitis, 2230, 2231f, 2565-2566 AIDS-associated, 2297t-2301t antiviral therapy for, 2181t Cytomegalovirus ventriculitis, 2502t Cytopenia, 1236-1237 idiopathic cytopenias of undetermined significance (ICUS), 1233 immune, drug-induced, 1704t Cytoplasmic nucleic acid sensors, 217 Cytotoxic agents, 1078, 1118, 1209-1218 Cytotoxic T-lymphocyte antigen (CTLA)-4 (CD152), 223 Cytotoxic/spitting cobras (Naja nigricolis, Naja mossambica), 719 Cytovene (dihydroxypropoxymethylguanosine, DHPG), 2231 Cytoxan (cyclophosphamide), 1211t-1216t

D

Dabigatran, 178 for anticoagulation, 367 for deep venous thrombosis, 517 drug interactions, 178t pharmacokinetics of, 178t in pregnancy, 1618t preoperative management of, 2616, 2617t for pulmonary embolism, 625 risk reduction with, 179t for thromboembolic prophylaxis, 366t Dabl Educational Web site, 383 Dabrafenib, 1232, 1232t for lung cancer, 1310t, 1312 for melanoma, 1376 Dacarbazine, 1271-1272, 1376 Daclizumab, 2477-2478 Dacogen (decitabine), 1211t-1216t Dacomitinib, 1310t Dactylitis, 1099 Daily headache, chronic, 2361-2362 Dalbavancin, 1889t Dalteparin, 177t for acute coronary syndrome, 439 in pregnancy, 1618t-1619t for pulmonary embolism, 625t for venous thromboembolism, 1619t Damage control surgery, 715 Danaparoid, 177t, 1618t Danazol for angioedema, 1697 for incontinence, 2238-2239 for premenstrual syndrome, 1587 for primary myelofibrosis, 1128 for warm autoimmune hemolytic anemia, 1077-1078 Dandy-Walker malformation, 2513 Danger-associated molecular patterns, 231 Danon’s disease, 1387, 1399 Dantrolene for malignant hyperthermia, 726, 2618 for neuroleptic malignant syndrome, 2628 Dapsone, 2659 for acne vulgaris, 2680 for AIDS-associated opportunistic infections, 2297t-2301t for bullous pemphigoid, 2675 for epidermolysis bullosa acquisita, 2676 for leprosy, 2045-2046 for neuropathy associated with leprosy, 2536 for pemphigus, 2677 for Pneumocystis pneumonia, 2096, 2096t, 2098, 2098t, 2313t-2314t for toxoplasmosis, 2129t Daptomycin, 1896 for endocarditis, 479t, 480 for enterococcal infection, 1915.e1t mechanism of action, 1889t for meningitis, 2488t for osteomyelitis, 1810t recommended doses and schedules, 1891t-1892t resistance to, 1890t for S. aureus infection, 1901 for septic arthritis, 1810t Daranavir, 2289

Darbepoetin alfa for anemia, 840, 1067, 1103-1104 for chronic interstitial nephritis, 798 Darier’s disease, 2633 Darier’s sign, 2684 Darifenacin, 113t, 830t Dark-field examination, 2016-2017 Darunavir, 2288t, 2290t DAS181, 2190 Dasatinib (Sprycel), 171, 1232t for cancer, 1211t-1216t for chronic myelogenous leukemia, 1248, 1249t, 1250 DASH diet, 1465 Data for clinical decisions, 37-41 clustered, 36 hierarchical, 36 missing, 36 secondary, publicly available, 43.e1t DATA2020 (CDC), 43.e1t Database of Chromosomal Imbalance and Phenotype in Humans using Ensembl Resources (DECIPHER), 191 Daunomycin, 1243t Daunorubicin (Cerubidine), 1211t-1216t, 1243t Dawn phenomenon, 1531 Daycare diarrhea, 921 Daytime sleepiness, excessive, 639, 2417, 2417f D-dimer testing, 513-514, 621 de Musset’s sign, 471 de Quervain’s tenosynovitis, 1752, 1752f de Quervain’s (subacute) thyroiditis, 1509 Dead space ventilation (Vd/Vt), 653 Deafness. See also Hearing loss genetic mutations in, 195 hereditary, 2594 hypoparathyroidism, deafness, and renal anomalies syndrome, 1652t, 1660 nerve, 1652t Death adders (Acanthophis), 720 Death and dying brain death, 2414-2415 tests for, 2414t treatment of, 2414b care of dying patients and their families, 9-15 global causes of death, 21t leading causes of death, 100, 101t locations of choice for, 14 sudden death antipsychotic-induced, 2627 prevention of, 326 Death receptor DR4, 223 Death receptor DR5, 223 Decay-accelerating activity, 244 Decay-accelerating factor (DAF) (CD55), 243t Deceased donors, 844 Decision analysis, 40, 40t Decision making with risk stratification, 426t steps in, 40t strategies for, 40-41 threshold approach, 39 using data for clinical decisions, 37-41 Decision support, 49 Decisional balance, 52, 53t Decision-making capacity, 104 Decitabine (Dacogen), 1211t-1216t, 1238 Declarative memory, 2383, 2383t Decolonization therapy, 1867-1868 Decompensated cirrhosis, 1026-1027, 1029f Decompression illness, 597-598 clinical manifestations of, 598 definition of, 597 epidemiology of, 597 prevention of, 598 prognosis for, 598 treatment of, 598b Decompression procedures, 715, 715f, 716.e1f Decompression sickness, 597-598 classification of, 597 epidemiology of, 597 type I, 597-598 type II, 597-598 Deconditioning, 1820

I29

Decongestants, 1692 Decontamination, 705 Deep brain stimulation, thalamic, 2462 Deep breathing, 182t Deep mycoses, 2580t Deep neck abscesses, 2603 Deep space infections, 2603 Deep vein thrombosis, 511-518, 512f with cancer, 625 clinical assessment of, 514 clinical manifestations of, 512, 621 diagnosis of, 512-515, 513f, 621, 623 differential diagnosis of, 514-515, 514t epidemiology of, 512, 620 imaging findings, 512-513, 513f laboratory findings, 513-514 in malignancy, 1189, 1189f pathobiology of, 512, 620-621 postoperative, 1191 prediction rule for, 512, 512t in pregnancy, 515, 1616-1617 diagnosis of, 1616-1617 pathobiology of, 1616 prognosis for, 1617 treatment of, 517 prevention of, 517-518 prognosis for, 626 recurrent, 514-515 risk of recurrence, 514 testing for thrombophilia after, 1187, 1187f treatment of, 515b-517b guidelines for, 515f long-term, 516-517 of upper extremities, 518 Deer fly fever, 1982 Deer tick (Ixodes scapularis), 2021, 2022f DEET, 1884 Defecation, 885, 885f Deferasirox (Exjade), 1094, 1421, 2091 Deferiprone, 1094, 1104 Deferoxamine, 93t, 96, 706t-710t, 1094 Defibrillation, transthoracic, 376-377 Defibrillators. See Implantable cardioverter-defibrillators Deficiency syndromes, 2536-2537, 2536t Degarelix (Firmagon), 1211t-1216t Degenerative diseases of bone and joints, 1713t, 1715t degenerative disc disease classification of, 1713t pain management, 142t rheumatic, 1714-1715 visual field abnormalities due to, 2575 Degos disease, 957 Degranulation abnormalities, 1147t-1148t, 1148-1149 Dehydrated hereditary stomatocytosis, 1084 Dehydration, 887 Dehydroepiandrosterone, 1514, 1561, 1589 Dejerine’s syndrome, 2435.e2f Dejerine-Sottas neuropathy, 2529 Delavirdine, 2288t, 2331t Delayed reactions drug rashes, 2684, 2685f, 2685t transfusion reactions, 1196-1197 Deletion, 239-240 Delirium, 117-121 clinical manifestations of, 119-120 definition of, 119 diagnosis of, 120 diagnostic criteria for, 119t differential diagnosis of, 120 end-of-life care for, 121 epidemiology of, 119 evaluation of, 120t future directions, 121 laboratory findings, 120 management of, 121 approaches to, 11t in palliative care, 11t medical therapy for, 120-121 in older adults, 117-121 pathobiology of, 119 postoperative, 2623 prevention of, 120, 121t prognosis for, 121 treatment of, 120b-121b Delivery system design, 48-49 Volume I  pp 1-1382  •  Volume II  pp 1383-2722

I30

Index

Delta anion gap, 766 Delta check, 1056 Delta HCO3−, 766 Delta (δ) waves, 356, 356.e3f Delta-delta, 766 Delusional disorder, 2353t, 2354 Delusions of parasitosis, 2177 Dematiaceous fungal infections, 2101-2103, 2102t clinical manifestations of, 2102 definition of, 2101 diagnosis of, 2102 epidemiology of, 2101-2102 foreign body–related, 2102 pathobiology of, 2101-2102 treatment of, 2103b Dematiaceous fungi, 2101, 2102t Dementia, 2388-2389 alcohol-related, 2510 Alzheimer’s disease, 2390-2391 brain imaging findings, 2392, 2393f diagnostic criteria for, 2391t clinical examination in, 2388 clinical manifestations of, 2388 definition of, 2388, 2388t diagnosis of, 2388-2389, 2389f dialysis dementia, 96 differential diagnosis of, 2389, 2390f disease trajectories, 10.e1f end-of-life care for, 2389 epidemiology of, 2388 frontotemporal behavior-variant, 2396, 2397t genes that predispose to, 1665t MRI findings, 2397, 2398f HIV dementia clinical manifestations of, 2329 definition of, 2329 diagnosis of, 2329 epidemiology of, 2329 MRI findings, 2331f pathobiology of, 2329 prognosis for, 2331 treatment of, 2331f with Lewy bodies, 2395-2396 clinical manifestations of, 2396 definition of, 2395 diagnosis of, 2396 diagnostic criteria for, 2395t differential diagnosis of, 2396 epidemiology of, 2395 pathobiology of, 2395-2396 prognosis for, 2396 treatment of, 2396b pathobiology of, 2388 prognosis for, 2389 rapidly progressive differential diagnosis of, 2505, 2505t evaluation of, 2505, 2505t semantic, 2397 vascular, 2394 Demographics, 15 Demyelinating conditions, 2471-2480 Dendritic cells, 216 Dengue, 1884, 2257-2258 clinical manifestations of, 2258 definition of, 2257 diagnosis of, 2258 epidemiology of, 2257-2258 pathobiology of, 2258 prognosis for, 2258b treatment of, 2258b vectors, 2172t Dengue fever virus, 2247, 2249t Dengue hemorrhagic fever, 2249t, 2258 clinical manifestations of, 2250, 2251t distinguishing characteristics of, 2252t-2253t management of, 2254-2255 pathobiology of, 2250, 2251t prognosis for, 2258 Dengue shock syndrome, 2258 fluid management of, 2254 prognosis for, 2258 Dengue virus, 2257 Denileukin diftitox, 2670t Denosumab (Prolia) for cancer, 1211t-1216t for osteoporosis, 1643, 1643t for prostate cancer, 1369 Dental caries, 1272t

Dental procedures: endocarditis prophylaxis for antibiotics for, 483t indications for, 482t Dentate (pectinate) line, 967 Dent’s disease, 823t Deoxycoformycin (pentostatin), 1253 Dependence alcohol, 155t opioid, 158-159 physiologic oxygen supply dependency, 672 Dependent personality disorder, 2355t Depo-Provera, 2408 Depot medroxyprogesterone acetate (DMPA), 1609 Depression diagnosis of, 10 in heart failure, 316 major depressive disorder, 2346-2348 screening for, 54, 104 subsyndromal, 2350 subthreshold, 2350 treatment of, 316, 2348, 2348t, 2396 acute, 2347 approaches to, 11t in palliative care, 11t preferred medications for women, 1602t in women, 1602t, 1603 Deprivational amblyopia, 2558 Dermablend, 2661 Dermacentor ticks, 2174, 2256 Dermal necrosis, 2638t-2644t, 2642f Dermal-epidermal junction, 2634 Dermatitis actinic, chronic, 2664-2665, 2664f allergic, occupational, 80t atopic, 2648f, 2663, 2663f in HIV infection, 2321 regional involvement, 2701f beetle-related, 2172t, 2176-2177 contact allergic, 2639f, 2656f, 2663-2664, 2687 drug-induced, 2687 irritant, 2663-2664 factitial, 2653f granulomatous inflammatory arthritis, dermatitis, and uveitis, 1740t, 1743 infective, 2237t irritant, occupational, 80t neurodermatitis, 2653f nummular, 2651f, 2662, 2662f occupational, 81 perioral, 2680-2681, 2681f photodermatitis, 2321 seborrheic, 2663, 2663f dermatoscopic signs of, 2703t in HIV infection, 2321 regional involvement, 2701f stasis, 2701f stinging sponge, 722 Dermatitis artefacta, 2653f Dermatitis herpetiformis, 2676, 2676f Dermatochalasis, 2556 Dermatofibromas, 2688-2689 Dermatofibrosis lenticularis disseminata, 1671 Dermatographism, 1694 Dermatologic paraneoplastic syndromes, 1220-1221 Dermatologic surgery, 2661 Dermatology, 2637 Dermatomes, 2370-2371, 2371f Dermatomyositis classification of, 1713t, 1790t clinical features of, 1791, 1792f diagnosis of, 1791 diagnostic criteria for, 1791, 1791t lung disease associated with, 585 ocular symptoms of, 2568-2569 pathobiology of, 1789 pathology of, 1789, 1790f prevalence of, 1789 prognosis for, 1793 treatment of, 1792 Dermatophytes, 2320 Dermatophytoses, 2670-2671 Dermatoscopy, 2656 Dermatosis acute febrile neutrophilic, 1859, 2682 autoinflammation, lipodystrophy, and dermatosis syndrome, 1740t febrile neutrophilic, 2692

Dermatosis (Continued) linear IgA bullous dermatosis, 2676 regional, 2700-2702, 2701f regional involvement, 2702t Dermis, 1730t, 2634 cellular components of, 2634 connective tissue matrix of, 2634 Dermographism, 1694 Dermopathy, 1506 Dermoscopy, 2656 Desert horned viper, 718 Desert rheumatism, 2072 Desert vipers (Cerastes), 719 Desferrioxamine, 1104 Desipramine (Norpramin) for chronic pain, 138t for depression, 2349t for dyspepsia, 895 for irritable bowel syndrome, 893t Desirudin, 1618t Desmoid tumors, 950 Desmopressin (DDAVP), 1170 for diabetes insipidus, 1499-1500 for hemophilia, 1176 for hypopituitarism, 1483t for intracerebral hemorrhage, 2452t Desmosomes, 2633 Desonide, 2658t Desoximetasone, 2658t Desquamative interstitial pneumonia, 582-583, 583f diagnosis of, 582-583, 583f, 583.e1f treatment of, 583b Desvenlafaxine for depression, 2349t for menopausal hot flushes, 1628t-1629t Detrusor hyperactivity with impaired contractility, 111, 113t Detrusor overactivity, 111, 113t Detrusor underactivity, 111, 113t Developing world childhood cryptosporidiosis in, 2135 pretravel consultation with travelers to, 1881, 1882t vaccines to consider for travelers to, 1881-1882, 1883t Development, 1570-1572 Developmental disorders, neurological, 2514 Deviation, standard (SD), 32 Devic’s disease, 2478 Dexamethasone, 164t for acute mountain sickness prophylaxis, 596-597 for AIDS-associated opportunistic infections, 2297t-2301t for altitude sickness, 597t for amyloidosis, 1286 for androgen excess, 1519 for brain tumors, 1290 for chronic inflammatory demyelinating polyradiculoneuropathy, 2531 for croup, 2190 for elevated CSF pressure, 2488 for enteric fever, 1974 for focal segmental glomerulosclerosis, 786 for 21-hydroxylase deficiency, 1568 for immune thrombocytopenic purpura, 1164 for infertility, 1595 for leukemia, 1243t for lymphoma, 1294 for meningitis, 2488 for meningococcal disease, 1938 for metastatic spinal cord compression, 2382 for multiple myeloma, 1279-1280, 1280t for nausea and vomiting, 865t for prevention of cluster headache, 2361 for prevention of high-altitude pulmonary edema, 596-597 for refractory ITP, 1164t replacement doses, 1520 for septic shock, 689t for spinal tumors, 1296 for urticaria, 2684t Dexamethasone suppression test, 1487t, 1515-1516 Dexlansoprazole, 900t Dexmedetomidine, 2619 Dexrazoxane (Zinecard), 1211t-1216t

Dextrans, 680t Dextroamphetamine effects on micronutrient status, 1453t for narcolepsy, 2418, 2419t Dextrocardia, 416 Dextrose solution for cold injury, 695 for coma, 2412 contraindications to, 2452 heated, 695 for hypovolemia, 745 for painful episodes, 1102-1103 for renal colic, 813 for rhabdomyolysis-induced acute kidney injury, 726t Diabetes insipidus, 1496-1500 central, 1496-1497, 1499 clinical manifestations of, 1496-1498 definition of, 1496 diagnosis of, 1498-1499 etiologic, 1499 physiologic, 1498-1499 differential diagnosis of, 1496 gestational, 1497, 1499 nephrogenic, 823t, 1497, 1499 pathobiology of, 1496 postoperative, 1500, 1500f post-traumatic, 1500, 1500f prognosis for, 1500 treatment of, 1499b-1500b triphasic response, 1500 Diabetes mellitus, 1527-1548 acute metabolic complications of, 1538-1542 arthritic manifestations of, 1824 blisters in, 2678 cardiomyopathy in, 330 cardiovascular disease in, 1546-1547 cell therapy for, 210 chronic vascular complications of, 1542-1548, 1543f classification of, 1527t complications of, 1546 definition of, 1527-1528 diagnostic criteria for, 1527-1528, 1528t dietary factors, 1427t dietary guidelines for, 1428t-1429t gestational, 1534, 1620-1622 definition of, 1620 dietary recommendations for, 1620 fetal and neonatal effects of, 1621-1622 maternal effects of, 1622 prognosis for, 1621-1622 treatment of, 1620b-1621b with heart failure, 316 incidence of, 15 maternal complications of, 1621-1622 maturity-onset, of youth (MODY), 187t pathobiology of, 1529 single-gene mutations associated with, 1533 microvascular complications of, 1542, 1543f, 1544-1546 new-onset, 846 nutritional influences, 1429 with other disorders or syndromes, 1527 pathobiology of, 1528-1529, 1528f postpartum considerations, 1621 pregestational, 1614t blood glucose control for, 1620-1621 diagnosis of, 1620 preoperative evaluation of, 2615-2616 renal disease secondary to, 804-806 in renal transplantation, 846 risk factors for severe hypoglycemia in, 1539, 1539t screening for, 56 treatment of, 316, 1602t type 1, 1529-1532 classification of, 1527t clinical manifestations of, 1530-1531 daily insulin requirements, 1531 definition of, 1527 epidemiology of, 1529-1530 incidence of, 1529-1530, 1530f insulin therapy for, 1531 pathobiology of, 1530 postpartum considerations, 1621 preclinical phase, 1530, 1531f preconception interventions for women, 1612t prevention of, 1532

Index Diabetes mellitus (Continued) prognosis for, 1532 treatment of, 1531b-1532b type 2, 1462, 1532-1538 atypical, 1534 classification of, 1527t clinical manifestations of, 1534 in critically ill, 1536 definition of, 1527 dietary recommendations for, 1534-1535 epidemiology of, 1532 genetics of, 1533-1534 incidence of, 1530f incretin-based therapy for, 1536 inpatient management of, 1536-1537 insulin therapy for, 1536-1537 ketosis-prone, 1534 in non–critically ill, 1536-1537 pathobiology of, 1532-1534 pharmacologic therapy for, 1535-1536, 1537f polygenic, common, 1533-1534 preconception interventions for women, 1612t prevention of, 1537 prognosis for, 1537-1538 recommended glycemic targets, 1534, 1535t risk factors for, 1537 screening for, 1537, 1538t single-gene mutations associated with, 1533 treatment algorithms, 1536 treatment of, 1534b-1537b typical, 1534 in women, 1601 in women, 1602t Diabetes mellitus–related diarrhea, 933 Diabetes Prevention Program, 1537 Diabetic agents, 2613t Diabetic cardiomyopathy, 1546 Diabetic dyslipidemia, 1394 pathobiology of, 1394 treatment of, 1394b Diabetic foot, 1545-1546 Diabetic ketoacidosis, 767-768, 1530, 1539-1540 clinical manifestations of, 767-768, 1540 definition of, 767 diagnosis of, 768, 1540 diagnostic criteria for, 1540t epidemiology of, 767 management of, 1541f pathobiology of, 767, 1539 precipitants, 1539t prognosis for, 768 treatment of, 768b, 1540b-1542b Diabetic nephropathy, 804-806, 1544-1545 clinical manifestations of, 804-805 development of, 1544-1545, 1545f diagnosis of, 806 epidemiology of, 804 genetics of, 804 hemodynamics of, 804 hypertension with, 392 natural history of, 804-805 pathobiology of, 804 prevention of, 806b recommended intervals for screening, 1544t stage I, 804 stage II, 805 stage III, 805 stage IV, 805 stages of, 804, 805f treatment of, 806b, 806f Diabetic neuropathy, 1545, 2518 acute asymmetrical, 2535 distal symmetrical polyneuropathy and autonomic neuropathy, 2535 lumbosacral radiculoplexus, 2535 peripheral neuropathies, 2535 clinical manifestations of, 2535 definition of, 2535 diagnosis of, 2535 epidemiology of, 2535 pathobiology of, 2535 treatment of, 2536b

Diabetic retinopathy, 1532, 1544, 2568, 2568f Diagnostic and Statistical Manual of Mental Disorders, 5th edition (DSM-5), 150, 156, 2352 diagnostic criteria for anorexia nervosa, 1455, 1455t diagnostic criteria for binge eating disorder, 1455, 1456t diagnostic criteria for bulimia nervosa, 1455, 1456t Diagnostic imaging procedures catheter angiography, 2440 in gastroenterology, 866-872 Dialysis, 841-842 access issues, 842 hemodialysis, 841 in septic shock, 690 toxicants removed by, 710t indications for, 841, 841t medical issues, 842-843 peritoneal, 841-842, 949 Dialysis dementia, 96 Dialysis Facility Compare (CMS), 43.e1t 3,4-Diaminopyridine, 2552-2553 Diamond-Blackfan anemia clinical features of, 1116-1117, 1116t diagnosis of, 1117 differential diagnosis of, 1119 genetics of, 1117 treatment of, 1120 Diapedesis, 1145 DIAPERS mnemonic, 110-111 for causes of transient incontinence, 111t Diaphragm (contraceptive), 1605t, 1607 Diaphragm (muscle), 627 Diaphragmatic disorders, 627-629 Diaphragmatic weakness or paralysis, 627-628, 628.e1f causes of, 628, 628t chest radiography findings, 628, 628f clinical manifestations of, 627-628 diagnosis of, 628 epidemiology of, 627 pathobiology of, 627 sniff test for, 628 treatment of, 628b-629b Diaphyseal dysplasia, progressive (CamuratiEngelmann disease), 1669 Diarrhea acute, 918-922 acute infectious, 919t alcoholic, 933 amebic, 2139 antibiotic-associated, 920 approach to, 852t, 861, 918-935 bile acid, 928 Brainerd’s diarrhea, 926 C. perfringens type A, 1930-1931 cancer treatment and, 920-921 causes of, noninfectious, 1872, 1872t chronic, 922-926 causes of, 922t, 926-933 definition of, 918 endoscopic approach to, 876 initial approach to, 924f treatment of, 934b-935b Cryptosporidium, 2057t daycare diarrhea, 921 definition of, 861, 918 diabetes mellitus–related, 933 elusive, 926 epidemiologic features of, 1869t epidemiology of, 918 factitious, 933-934 fasting, large-volume, 1338 fructose, 933 hemorrhagic, 2057t in HIV infection, 2303t, 2304 in immunocompromised patients, 1860 infectious acute, 919t antimicrobial therapy for, 1870t, 1872 epidemiology of, 919t food-borne, 919-920, 919t laboratory testing in, 1869-1870, 1870t, 1871f prolonged, persistent, 926 therapy for, 1870t

Diarrhea (Continued) treatment of, 922, 1870b-1872b water-borne, 919-920, 919t inflammatory, 926, 934 magnesium, 933 medication-related, 920-921 medications for, 893t nosocomial hospital diarrhea, 920 nuisance, 1925 pathobiology of, 918-919 postoperative, 2623 reduction of, 922 rotavirus-induced treatment of, 2246 vaccination against, 2246 runner’s diarrhea, 921 secretory, true, 933-934 sodium anion–induced, 926 sodium phosphate, 933 sorbitol and fructose, 933 stool examination in, 926 sulfate, 933 traveler’s, 920, 922, 1884 causes of, 1868 education about, 1882t prophylaxis of, 1870t treatment of, 893t, 921b-922b watery, 924f, 926, 933 watery diarrhea, hypokalemia, and achlorhydria, 1335t, 1337, 1553 watery diarrhea, hypokalemic, hypochlorhydric acidosis, 769 zoonotic, 2057, 2059t Diastolic dysfunction, 317 Diastolic ventricular function, 279 Diazepam for alcohol withdrawal, 155t for anxiety, 2352t for diarrhea, 922 for dystonia, 2465 for muscle spasms, 1930 for seizures, 2408, 2489 for tachypnea, 31 for tinnitus, 2596 for vertigo, 2600 Diazinon, 699t-702t Diazoxide, 1555 Dichloralphenazone, 2358 Dichlorphenamide, 2544 Dichotomous outcome variables, 33.e1t Dichotomous predictor variables, 33.e1t Diclofenac, 1764, 2660-2661 Dicloxacillin for bullous impetigo, 2678-2679 for erysipelas, 2696-2697 for furuncles, 2696 for impetigo, 2695-2696 recommended doses and schedules, 1891t-1892t for S. aureus infection, 843, 1901 for staphylococcal scalded skin syndrome, 2679 Dicyclomine, 1048 Didanosine, 2288t, 2289, 2331t Dientamoeba fragilis, 2147t Dieppe, Paul, 1811 Diet(s), 182t, 1430 behavioral interventions for, 57 and cancer, 1223 and cardiovascular disease, 259 for colorectal cancer prevention, 1329-1330 for diabetes prevention, 1537 and disease, 1426, 1427t for eosinophilic esophagitis, 902 for gestational diabetes mellitus, 1620 for glycogen storage disease, 1399 for heart failure, 315 high-carbohydrate/low-fat, 1465 high-fat, 925-926 high-fat/low-carbohydrate, 1465 high-potassium, 762 high-protein, 1399 for hypertension, 387t for infectious diarrhea, 1870t for irritable bowel syndrome, 893-894 low-salt, 2601t and lung cancer, 1426 for Meniere’s disease, 2601t recommendations for, 1223, 1464-1465

I31

Diet(s) (Continued) for type 1 diabetes, 1531 for type 2 diabetes, 1534-1535 for weight loss, 1464-1465 Diet counseling, 57 Dietary fat, 926, 926.e1f Dietary guidelines, 1427t-1429t Dietary Guidelines for Americans (USDA/ HHS), 1428t-1429t, 1430 Dietary history, 850 Dietary planning, 1531 Dietary protein breakdown of, 836, 836.e1f estimation of, 836, 836t Dietary requirements in elderly persons, 1452-1453 in infancy, 1452 in women of childbearing age, 1452 Dietary salt, 2521 Dietary Supplement Health and Education Act (DSHEA), 181 Dietary supplements, 181, 182t N,N-Diethyl-3-methylbenzamide (DEET), 2176 Diethylcarbamazine, 2104 for eye worms, 2170 for lymphatic filariasis, 2168 for nematodes, 2165t for roundworms, 2103-2104 Diethylenetriamine pentaacetate (DTPA), 85 Dieulafoy’s lesion, 874, 879, 882, 883f, 957-958 diagnosis of, 913 endoscopic findings, 957b, 957f treatment of, 957b Dieulafoy’s ulcers, 908 Differentiating agents, 1211t-1216t Differin (adapalene), 2658 Diffuse alopecia, 2703-2704, 2703t Diffuse alveolar damage, 539f Diffuse alveolar hemorrhage, 571t, 573-574 clinical manifestations of, 573-574, 574.e1f definition of, 573 diagnosis of, 574, 574f, 574.e1f pathobiology of, 573 prognosis for, 574 treatment of, 574b Diffuse cutaneous leishmaniasis, 2123 Diffuse esophageal spasm, 904 Diffuse hypoxic injury, 2432 Diffuse idiopathic sclerosing hyperostosis, 2372 classification of, 1713t pathobiology of, 2372 treatment of, 2378 Diffuse idiopathic skeletal hyperostosis, 1816 Diffuse large B-cell lymphoma, 1264-1265 advanced disease, 1265 clinical characteristics of, 1261t cutaneous, 1265 in HIV infection, 2324 localized disease, 1264 primary sites, 1266-1267 salvage therapy for, 1265 subtypes, 1265 variants and subtypes, 1264, 1264t Diffuse lung disease, 532-536, 534f, 535t Diffuse panbronchiolitis, 1965 Diffuse toxoplasmic encephalitis, 2127 Diffusing capacity, 542, 558 Diffusion-weighted imaging (DWI), 2439f Diflorasone diacetate (Florone, Psorcon E), 2658t DiGeorge’s syndrome, 192-194, 192f, 1652t, 1660, 1683t, 1684 clinical manifestations of, 1684 definition of, 1684 diagnosis of, 1684 epidemiology of, 1684 genetic determinants of, 405 pathobiology of, 1684 prognosis for, 1684 treatment of, 1684 Digestion, 927-929, 926.e1f Digestive disease, 1101 Digital pelvic examination, 851 Digital rectal examination, 851, 1367 Digital subtraction angiography in atherosclerotic PAD, 500, 501f in renal artery stenosis, 807-808 Volume I  pp 1-1382  •  Volume II  pp 1383-2722

I32

Index

Digitalis glycosides, 306.e1t-306.e2t Digitalis toxicity, 706t-710t Digoxin, 314 for atrial fibrillation, 452 clinical benefits, 314 doses and side effects of, 363t-364t for heart failure, 314 mechanism of action, 314 ocular effects of, 2573t pharmacokinetic parameters for, 126t in renal failure, 129t side effects of, 314 toxicity, 699t-703t Digoxin overdose, 706t-710t Digoxin-specific antibody fragment (Fab), 699t-702t, 706t-710t Dihydroergotamine, 2358-2359, 2361 Dihydropyridines contraindications to, 389t for hypertension, 388t side effects of, 389t, 390 Dihydrotachysterol, 1659 1,25-Dihydroxy cholecalciferol (calcitriol), 1659 Dihydroxyacetone phosphate acyltransferase deficiency, 1387t Dihydroxypropoxymethylguanosine (DHPG, Cytovene), 2231 Dilated cardiomyopathy, 321t, 329-331, 329.e1f acquired, 329 clinical manifestations of, 329 diagnosis of, 329 prognosis for, 330-331 treatment of, 330b causes of, 330-331 definition of, 329 echocardiographic findings, 275f, 280t-281t, 275.e1 epidemiology of, 329 family screening for, 330 genetic, 329 nonischemic, 371 overlap with restrictive cardiomyopathy, 331 pathobiology of, 329 treatment of heart failure due to, 318 ventricular tachycardia in, 371 Diloxanide furoate, 2141t Diltiazem for acute coronary syndrome, 437, 438t-439t for acute STEMI, 455 for anal fissures, 970 for aortic dissection, 495-496 for cardiogenic shock, 685 for hypertension, 388t for hypertrophic cardiomyopathy, 325 for premature atrial contractions, 359 properties of, 430t for pulmonary hypertension, 402 for ventricular tachycardia, 373 Dilutional thrombocytopenia, 1167 Dimenhydrinate, 865t Dimercaprol (BAL), 706t-710t for arsenic poisoning, 95b contraindications to, 1088t for heavy metal poisoning, 93t 2,3-Dimercaptosuccinic acid (DMSA), 92, 93t, 95b Dimetapp (brompheniramine), 2660t Dimethyl fumarate (Tecfidera), 2477 Dimethyl sulfoxide, 2522 Dipentum (olsalazine), 939t Dipeptidyl peptidase 4 (DPP-4) inhibitors, 1536 Diphenhydramine (Benadryl), 2660t for allergic rhinitis, 2588 for anthrax exposure, 88b for gastroparesis and pseudo-obstruction, 888 for nausea and vomiting, 865t for scombroid, 722 for systemic anaphylaxis, 1702 for urticaria, 1695, 2684t Diphenylcyclopropenone, 2705 Diphenylpyraline, 2660t Diphtheria, 1915-1918 clinical manifestations of, 1916-1917 cutaneous, 1917 definition of, 1915-1916 diagnosis of, 1917

Diphtheria (Continued) differential diagnosis of, 1917 epidemiology of, 1916, 1916f immunizations against, 66t, 68t-73t neuropathy associated with, 2536 pathobiology of, 1916 prevention of, 1917 prognosis for, 1917-1918 respiratory, 1916-1917 treatment of, 1917b, 1917t Diphtheria, tetanus, and pertussis (DTaP) vaccine, 1929, 1992 Diphtheria antitoxin, 1917, 2536 Diphtheria toxin CRM197, 1917 Diphtheria toxoid, 1917 Diphyllobothrium, 1107, 2147-2148, 2148t Diplopia, 2551, 2558 Diprolene (betamethasone dipropionate), 2658t Diprosone (betamethasone dipropionate), 2658t Dipylidium caninum, 2148t, 2149, 2149b Dipyridamole, 179 Direct antiglobulin test (DAT), 1075 Direct factor Xa inhibitors, 178 Direct reprogramming, 207-208 Direct thrombin inhibitors, 178 Dirofilaria immitis, 2171 Dirofilaria repens, 2171 Dirofilaria tenuis, 2171 Dirty bombs, 83 “Dirty lungs”, 533-534 Disability, 101 Disaccharidase deficiency, 933 Disc herniations, 2379 Discharge issues, 458 Discharge medication checklist, 454t Discocytes, 1053f Discoid lupus erythematosus, 1771, 2648f dermatoscopic signs of, 2703t hair loss in, 2705-2706, 2706f Disconjugate gaze-evoked nystagmus, 2579 Discounting, 40 Disease(s). See also specific conditions changing patterns of, 20-22 chronic. See Chronic disease complement system in, 240-246 diet and, 1426 gene therapy for, 211-213 genetics of, 186-189, 201 global burden of, 19-20, 21t history of the present illness, 24 inherited basis of, 196-200 monogenic, 191 nutrition and, 1426-1430 obesity and, 1461-1462 pathogenesis of, 216-220 polygenic, 191 stem cell-derived platforms for modeling of, 210 stratification of, 188 trajectories, 10.e1f Disease risk, 196 Disease-modifying antirheumatic drugs, 1723, 1760-1762 biologic, 1762 caveats for monitoring, 1761t conventional, 1761-1762 perioperative, 1833 for spondyloarthritis, 1764 for systemic sclerosis, 1784 Disease-modifying osteoarthritis drugs, 1749 Disequilibrium, 2600t Disodium cromoglycate, 935 Disopyramide, 325, 363t-364t Dispholidus typhus (boomslang), 719 Dissecting cellulitis, 2706 Disseminated aspergillosis, 2083 Disseminated blastomycosis, 2074 Disseminated candidiasis, 1858, 1860f, 2080 chronic, 2080, 2081f, 2082 Disseminated encephalomyelitis, acute, 2479 Disseminated gonococcal infection, 1943-1944 antibiotic regimens for, 1945t clinical manifestations of, 1943, 1944f diagnosis of, 1943-1944 prevention of, 1944 prognosis for, 1944 skin lesions, 1943, 1944f treatment of, 1945 Disseminated histoplasmosis, 2070-2071

Disseminated intravascular coagulation, 1166, 1181-1183 blood component therapy for, 1183, 1183t causes of, 1181t, 1182 clinical manifestations of, 1182, 1182f coagulation tests for, 1182 definition of, 1181 diagnosis of, 1182-1183 differential diagnosis of, 1182-1183 drug therapy for, 1183, 1183t management of, 2254 pathobiology of, 1181-1182, 1182f pathophysiology of, 1181, 1182f treatment of, 1183b, 1183t Disseminated mucormycosis, 2089 Disseminated nontuberculous mycobacterial disease, 2040 Disseminated sporotrichosis, 2078-2079 Disseminated superficial actinic porokreatosis, 2651f Dissociative shock, 674, 678f-679f Distal acquired demyelinating symmetrical neuropathy, 2532 Distal renal tubular acidosis, 770, 824 causes of, 770t features of, 823t-824t Distal sensory polyneuropathy, 2535 Distal symmetrical polyneuropathy clinical manifestations of, 2535 pathobiology of, 2535 treatment of, 2536 Distal tubule disorders, 823-824, 823t Distal tubule natriuretics, 748-749 Distention, 861 Distorted vision, 2558-2559 Distributive shock, 674-675, 675t, 678f-679f Disulfiram, 155, 155t Diuresis postobstructive, 803 solute, 754 Diuretics, 748-749, 748t clinical benefits of, 309 contraindications to, 389t for heart failure, 309 for hypertension, 388t, 390-391, 394t loop, 748t mechanism of action, 309, 748, 748f potassium-sparing, 748t practical use of, 309 side effects of, 389t, 391 therapeutic principles, 390-391 thiazide, 748t Diversity, 16 Diverticula epiphrenic, 904-905 esophageal, 904-905 mid-esophageal, 904 Diverticular disease, intestinal, 1430 Diverticulitis, 946-947, 947f, 947.e1f clinical manifestations of, 947 diagnosis of, 947 treatment of, 947b Diverticulum Meckel, 943-944 Zenker’s, 905f Dix-Hallpike test, 2600 Dizziness (presyncope), 345-346 diagnostic evaluation of, 2600, 2600t with palpitations, 344, 344f psychophysiologic, 2600t DMSA (2,3-dimercaptosuccinic acid), 92, 93t, 95b DNA, 189 antibodies to, 1721-1722 mitochondrial DNA depletion syndromes, 2546 DNA hypomethylating agents, 1238 DNA recombination defects, 1680t DNA repair gene mutations, 1227t DNA-active drugs, 1211t-1216t DNase, 635 Dobrava-Belgrade virus, 2249t Dobutamine, 676t for heart failure with pulmonary edema, 318 for mitral valve regurgitation, 453 for pulmonary embolism, 623-624 for RV infarction, 452 for shock, 683, 689-690 for ventricular septal defect, 453 Docetaxel (Taxotere), 1211t-1216t, 1302, 1311, 1319, 1369

Docosanol, 2181t, 2183 Documentation anesthesia records, 2621 postoperative care orders, 2621 Dofetilide, 363t-364t DOG1, 1372-1373 Döhle-like bodies, 1057-1058 Döhle’s bodies, 1055f, 1058, 2624 Dolasetron, 865t Dolichostenomelia, 1735 Doll’s-eye test, 2600 Domperidone, 887-888 Donabedian’s triad, 44, 44t Donepezil for Alzheimer’s disease, 2394 for mild cognitive impairment, 2390 for Parkinson’s disease, 2457t-2459t Donor hematopoietic stem cells, 1199 Donor organs, 844 Donovanosis (granuloma inguinale), 2001-2002 Dopamine for heart failure with pulmonary edema, 318 for mitral valve regurgitation, 453 for pulmonary embolism, 623-624 for shock, 676t, 680, 683, 690, 2369 for ventricular septal defect, 453 Dopamine agonist therapy, 1488 Dopa-responsive dystonia, 2465 Doppler echocardiography, 275-276 color flow imaging, 276, 1726 continuous-wave, 276, 279f modalities, 275-276 principles of, 275 pulsed, 275-276 tissue, 276 Doppler shift, 275 Doppler ultrasonography, 275-276, 1726, 2345t, 2440 continuous-wave, 276 pulsed, 275-276 transcranial, 2440 Doripenem for P. aeruginosa bacteremia, 1966 for pyelonephritis, 1875t recommended doses and schedules, 1891t-1892t Dorsal midbrain syndrome, 2578-2579 Dorzolamide, 2566 Dose fractionation, 1886-1887 Double balloon enteroscopy, laparoscopicassisted, 1326-1327, 1322.e1 Double cortex (band heterotopia), 2512 Double density sign, 283, 283f Double vision, 2558 Double-hit lymphoma, 1265 Down syndrome, 192, 192f, 1131 Downbeat nystagmus, 2579 Doxazosin for benign prostatic hyperplasia, 830t for hypertension, 388t for urinary incontinence, 2238-2239 for urinary retention, 2477 Doxecalciferol, 842 Doxepin (Sinequan), 2660, 2660t for depression, 2349t for insomnia, 2421t for pruritus, 2636t for tension-type headache prevention, 2360 for urticaria, 1695 Doxorubicin (Adriamycin, Rubex) for cancer, 1211t-1216t for Cushing’s syndrome, 1517 for Hodgkin’s lymphoma, 1271-1272 for Kaposi’s sarcoma, 2325 for leukemia, 1243t liposomal (Doxil), 1211t-1216t for non-Hodgkin’s lymphoma, 1263t for osteosarcoma, 1370 Doxycycline for acne vulgaris, 2680 for acute bronchitis and tracheitis, 609 for anthrax exposure, 88 for bacillary angiomatosis, 2001 for bacterial meningitis, 2487 for Bartonella endocarditis, 2000-2001 for blepharitis, 2561 for brucellosis, 1981, 1981t for cat-scratch disease, 2000 for chlamydia, 2011

Index Doxycycline (Continued) for COPD exacerbations, 562 for cystitis, 1875t for ehrlichiosis, 2053 for furuncles, 2696 for gastroparesis and pseudo-obstruction, 888 for gonorrhea, 1945t, 2603 for granuloma inguinale, 2002 for H. pylori eradication, 915t for infectious rhinosinusitis, 2588 for legionnaires’ disease, 1996t for leptospirosis, 2030 for leptospirosis prophylaxis, 2030 for Lyme disease, 2025t, 2026 for lymphatic filariasis, 2168 for malaria, 1883, 2105, 2111t-2112t for MRSA infection, 1901 for Mycoplasma pneumonia, 2006 for nematodes, 2165t for onchocerciasis, 2170 for peliosis hepatis, 2001 for pelvic inflammatory disease, 1945 for pityriasis lichenoides, 2669 for plague, 89b, 1989t for pneumonia, 616t for prophylaxis in HIV infection, 2295t for psittacosis, 2012 for Q fever, 2056 recommended doses and schedules, 1891t-1892t for Rocky Mountain spotted fever, 2049 for rosacea, 2680 for S. pneumoniae infection, 1905t for scrub typhus, 2052 for syphilis, 2018 for tick bites, 2174 for tick-borne rickettsioses, 2051 for travel-related health problems, 1884 for trench fever, 2000 for tularemia, 90b, 1983 for typhus, 2052b for urethritis, 1878t for Vibrio infections, 1953 Dracunculiasis, 2165t, 2171 Dracunculus medinensis, 2159, 2171 Dravet’s syndrome, 2404 DRESS syndrome (drug reaction or rash with eosinophilia and systemic symptoms), 1153 Dressings, 2657 Dressler’s syndrome, 491, 635 Drinking at-risk criteria for, 150t definition of, 149-150 treatment of, 154 binge, 149-150 heavy, 149-150 moderate, 149 problem behavioral interventions for, 57 prevalence of, 150 Driving with arrhythmias, 351t with heart failure, 315-316 Dronabinol, 865t Dronedarone, 363t-364t, 366 Droopy eyelid (ptosis), 2560, 2560f Droperidol, 865t Drospirenone and ethinyl estradiol (Yaz), 2660 Drowning, 595-596 clinical manifestations of, 595 definition of, 595 diagnosis of, 595 epidemiology of, 595 near-drowning, 595 pathobiology of, 595 prevention of, 595 prognosis for, 596 treatment of, 595b-596b Droxidopa, 2521 Drug(s). See also specific drugs absorption of, 124 decreased, 130 increased, 130-131 of abuse, 156-162 addictive, 143, 144t, 145 administration of, 124

Drug(s) (Continued) adverse reactions to, 131-132 for alcohol dependence, 155t allergic reactions to, 1703t for amebiasis, 2141t antianxiety, 2352t antiarrhythmic, 354-355, 363t-364t, 366t antidepressant, 2347-2348, 2349t antidiabetic, 1537 antiepileptic, 2407-2408, 2407t-2408t antihypertensive, 387-391 antipsychotic, 2353, 2354t antithrombotic, 175-181 and aplastic anemia, 1118 associated toxidromes, 696, 696t associated with acute interstitial nephritis, 779, 779t associated with aplastic anemia, 1115, 1115t associated with ischemic colitis, 955, 956t associated with photoallergy, 2687t associated with photosensitivity, 2687 associated with sun sensitivity, 2687t for benign prostatic hyperplasia, 830, 830t bioavailability of, 124 for bloating, 894 for bowel habit abnormalities, 894 for cancer, 1211t-1216t for chronic neuropathic pain, 137t clearance of, 124-125 dose adjustments by, 128 tests based on, 990 total, 124 decreasing levels, 127 delivery of, 1208 determination of accumulation of, 126, 126f-127f and dietary requirements, 1445t, 1453 discharge medication checklist, 454t disease-modifying antirheumatic, 1723, 1760-1762 disease-modifying osteoarthritis, 1749 for disseminated intravascular coagulation, 1183, 1183t distribution of, 124 altered, 130 volume of, 124 dose-response effects, 1007, 1008t dosing, 128-129, 1208 elimination of, 124-125, 127, 127f decreased, 131 effects of dose increases on, 127, 127f for erectile dsyfunction, 1578-1579 for esophageal disorders, 900t first-pass effects of, 124 and folate deficiency, 1108 future, 142-143 genetic polymorphisms that affect, 189t genetic testing for, 188-191 germline genetic variants and, 202 half-life of, 125 for heart failure with reduced left ventricular ejection fraction, 309-314 for human African trypanosomiasis, 2115t for hyperphosphatemia, 778t hypnotic, 2352t idiosyncratic responses to, 1118 immunosuppressive, 162-169, 1037-1038, 1037t for incontinence, 114 for inflammatory bowel disease, 939t for influenza, 2196 for insomnia, 2421, 2421t for irritable bowel syndrome, 893t, 894 for ischemic heart disease, 428, 429f laboratory values, 2717t-2718t and leukemia, 1240 for lipid disorders, 1396t loading dose, 125-126 formula for, 125 in renal insufficiency, 128 maintenance dose, 126-127 maternal exposure to, 1564 metabolism of, 1007, 1007t decreased, 130 genetic polymorphisms that affect, 189t increased, 130 and micronutrient status, 1453, 1453t mydriatic, 2576

Drug(s) (Continued) for myeloproliferative neoplasms, 1128 for myofascial trigger points, 1822 neurotoxic effects of, 2548t ocular effects of, 2572-2573, 2573t off-label, 149 for older adults, 116-117, 117t for osteoarthritis, 1748-1749 for osteoporosis, 1642-1644, 1643t over-the-counter, 117t for pain, 136-138, 142-143 and pancreatitis, 960-961, 960t for Parkinson’s disease, 2456-2460, 2457t-2459t perioperative, 2611-2612, 2613t for Pneumocystis pneumonia, 2096t preventive of headache, 2357, 2358t preventive of Pneumocystis pneumonia, 2098, 2098t preventive of relapse to alcohol use, 155-156 preventive of toxicity, 1218 principles of, 124-133, 125f psychoactive effects of, 116, 117t psychotropic, 2626, 2626t to reduce angina and ischemia, 429 reference intervals, 2719t-2722t and rheumatic disease, 1717 for septic shock, 690 side effects of, 2302 sideroblastic anemia due to exposure to, 1072 skin diseases due to, 2646t stem cell-derived platforms for, 210 stroke due to, 2443 targeting, 202 testicular hypogonadism due to, 1573 that affect sweating, 2520, 2520t that alter lipid metabolism, 428 that cause dry eye, 2570t that cause hair loss, 2704t that cause hearing loss, 2595 that cause hypercalcemia, 1658 that induce autonomic neuropathy, 2520 that promote weight gain, 1461, 1462t for thyrotoxicosis, 1508 for tobacco dependence, 147-149, 147t topical, 2657-2658 for type 2 diabetes, 1535-1536, 1537f to use with caution in heart failure, 314 for vasomotor symptoms, 1627 and weight gain, 1461, 1462t for weight loss, 1465-1466 for women of reproductive potential, 1602t Drug abuse, 143, 162, 2339. See also Substance abuse in adolescents, 65t CRAFFT screening tool for, 65t in elderly, 129-130 epidemiology of, 157 injection drug use, 1926 pathobiology of, 157 Drug addiction, 143 Drug Addiction Act, 159 Drug adherence, 315 Drug allergy, 1703-1705 clinical manifestations of, 1704 definition of, 1703 diagnosis of, 1704-1705 differential diagnosis of, 1704-1705 epidemiology of, 1703 future directions, 1705 pathobiology of, 1704 prevention of, 1705 prognosis for, 1705 risk factors for, 1703 treatment of, 1705b evidence-based, 1705 guidelines for, 1705f Drug concentrations interpreting, 127-128 therapeutic window, 127 Drug counseling, 315t Drug Database for Porphyria, 1415 Drug eruptions, 2647f fixed, 2687 in immunocompromised patients, 1858-1859

I33

Drug interactions, 130-131 with antimicrobial therapy, 1846 cancer drugs, 1208 diagnosis of, 131 drugs at risk for, 131t pharmacodynamic, 131 pharmacokinetic, 130-131 prevention of, 131 types of, 130 Drug monitoring, 127-128 Drug overdose approach to, 129 distinguishing characteristics of, 2252t-2253t Drug paraphernalia, 158 Drug rashes, 2684-2687 clinical manifestations of, 2684-2685 definition of, 2684 delayed hypersensitivity reactions, 2684, 2685f, 2685t diagnosis of, 2684-2685 EGFR inhibitor–associated, 2686, 2686f with eosinophilia and systemic symptoms, 2685-2686, 2685t pathobiology of, 2684 prognosis for, 2685b treatment of, 2685b Drug reaction with eosinophilia and systemic symptoms (DRESS) syndrome, 1004 Drug reactions, 1010, 1018t, 2617-2618 cholestatic, 1009 differential diagnosis of, 1010 hepatocellular, 1008-1009 hypersensitivity reactions, 2684, 2685f, 2685t idiosyncratic reactions, 1008 immunoallergic, 1009 neutrophilic, 2686 oral ulcers, 2580t prevention of, 1010 types of, 1008-1010, 1008t Drug resistance chromosomally encoded, 1964 emergence of, 1887-1888 in health care–associated infections, 1863, 1863t prevention of, 1847 suppression of, 1888 Drug screens, 702t Drug sensitivity, 2252t-2253t Drug testing, 699-702, 702t Drug toxicity, 702-703, 703t measurement of, 127, 128f predictable responses, 131-132 reduction of, 1846 stem cell, 1116 unpredictable responses, 132 Drug use disorders clinical manifestations of, 157-158 epidemiology of, 157 medical complications related to, 157-158 pathobiology of, 157 treatment of, 158, 158b Drug-induced aplastic anemia, 1115, 1115t Drug-induced fever, 1853, 1853t Drug-induced gingival hyperplasia, 2583, 2584f Drug-induced hypersensitivity, 1859 Drug-induced immune hemolytic anemia, 1078 Drug-induced interstitial lung disease, 576-577, 577t, 586 Drug-induced liver disease, 1006-1010, 1018t clinical manifestations of, 1008 definition of, 1006 diagnosis of, 1000t, 1008-1010 differential diagnosis of, 1010 epidemiology of, 1006 future directions, 1010 genetics of, 1007 idiosyncratic reactions, 1008 mechanisms of, 1007-1008, 1008.e1f pathobiology of, 1006-1008 pathogenesis of, 1007 prevention of, 1010 treatment of, 1010b Drug-Induced Liver Injury Network, 1010 Drug-induced neutropenia, 1135, 1135f Drug-induced neutrophilia, 1130 Volume I  pp 1-1382  •  Volume II  pp 1383-2722

I34

Index

Drug-induced platelet bleeding disorders, 1171 Drug-induced thrombocytopenia, 1161-1162 clinical manifestations of, 1162 diagnosis of, 1162 drugs associated with, 1162, 1162t pathobiology of, 1161-1162 treatment of, 1162b Drug-metabolizing enzymes, 131-132, 132t Drug-resistant hypertension, 394 “Drugs for Parasitic Infections” (The Medical Letter on Drugs and Therapeutics), 2103 Drusen, 2568 Dry eyes, 1759, 2562-2563, 2564t, 2570, 2570t Dry mouth, 1759, 2585 DTaP (diphtheria, tetanus, and pertussis) vaccine, 1929, 1992 Dual specificity phosphatase 1 (DUSP1), 235 Dual-energy x-ray absorptiometry (DXA), 1641 BMD testing, 1636 guidelines for women, 1602 Dubin-Johnson syndrome, 984, 987t Duchenne’s muscular dystrophy, 2541-2542, 2542f clinical manifestations of, 2541 diagnosis of, 2542 dilated cardiomyopathy in, 329 female carriers of, 2543 prognosis for, 2542 therapeutic strategies for, 1388t treatment of, 1388, 2542b Ductal carcinoma in situ, 1354, 1354t Dukoral (Vibrio cholerae/cholera toxin B subunit vaccine), 1959 Duloxetine (Cymbalta) for chronic pain, 138t for depression, 2349t for distal symmerical polyneuropathy, 2536 for irritable bowel syndrome, 893t for low back pain, 2375 Dumping syndrome, 888, 927, 1551 clinical manifestations of, 928 medical therapy for, 888 Duodenal ulcers, 908-909, 909f Duodenitis, 908 Duplex ultrasonography, carotid, 2440 Duplication cysts, 908 Dupuytren’s contracture, 977-978, 1020, 1020f, 1027 Dural arteriovenous fistulas, 2453 Duroziez’s sign, 471 Dust mites, 1691 Dutasteride, 830t Dying patients. See Critical care Dynamic hyperinflation, 667 Dynamic International Prognostic Scoring System (DIPSS)-plus, 1128-1129 Dysautonomia, 2369 Dysbetalipoproteinemia, 1394 Dyschondroplasia, 1672 Dyscognitive seizures, 2401t Dysentery, 2057t amebic, 2139-2140 bacillary, 2252t-2253t Dysesthesias, 2340 Dysfibrinogenemia, 1175t, 1179 clinical manifestations of, 1179 treatment of, 1179b Dysgerminoma suprasellar, 1476 visual abnormalities with, 2575, 2578-2579 Dyshidrosis, 2662-2663, 2662f Dyskeratosis congenita, 1133t, 1134 clinical features of, 1116-1117, 1116t diagnosis of, 1117 differential diagnosis of, 1119 genetics of, 1117 treatment of, 1119 Dyslipidemia diabetic, 1394, 1547 and obesity, 1463 Dysmenorrhea, 1586 definition of, 1586 pathobiology of, 1586 primary, 1586 secondary, 1586 treatment of, 1587 Dysmyelinating diseases, 2471 Dysnatremia, 741

Dysostosis multiplex, 1733 Dyspareunia diagnosis of, 1627 epidemiology of, 1627 treatment of, 1627, 1629 Dyspepsia approach to, 861, 864f definition of, 861 endoscopy in, 874 epigastric pain syndrome, 894, 894t functional, 890, 894-895 clinical manifestations of, 895 diagnosis of, 895 differential diagnosis of, 895 epidemiology of, 894-895 pathobiology of, 895 prognosis for, 895 Rome III diagnostic criteria for, 894-895, 894t treatment of, 895b nonulcer, 908 postprandial distress syndrome, 894, 894t Dysphagia in amyotrophic lateral sclerosis, 2523 antipsychotic-induced, 2626t endoscopy in, 874 in HIV infection, 2303t Dysphagia lusoria, 415, 852t, 908 Dysphonia, 2606 muscle tension, 2605 spasmodic, 2605 Dysplasia anal, 972 in inflammatory bowel disease, 942-943 Dysplastic nevi, 1374, 1374f, 1375t, 1377 Dysplastic nevus syndrome, 2689, 2690f Dyspnea, 248, 301-302 approach to, 528-529 causes of, 529t chronic, 530f classification of, 398 CMR findings, 291f diagnosis of, 529 management of, 11t palliative care for, 11t paroxysmal nocturnal, 302 postoperative, 2622 acute, 2622t differential diagnosis of, 2622, 2622t severe, 302 treatment of, 529b Dysthymia, 2350 Dystonia, 2463-2465 action, 2463-2464 acute dystonic reactions, 2467-2468 adult-onset idiopathic, 2465 axial, 2463-2464 causes of, 2464t cervical, 2463-2465 classification of, 2464, 2464t clinical manifestations of, 2463-2464 cranial, 2465 definition of, 2463 diagnosis of, 2464 dopa-responsive, 2465 laryngeal, 2463-2464, 2605, 2627 manual, 2463-2464 myoclonus, 2465 oromandibular, 2463-2464 pathobiology of, 2463 primary (idiopathic), 2464-2465, 2464t prognosis for, 2464 pure, 2463 spasmodic, 2463-2464 task-specific, 2463-2465 treatment of, 2465b Dystonia-plus, 2464t, 2465 Dystonic gait, 2341t Dystonic reactions, acute, 2467-2468 Dystonic spasms, 2463-2464 Dystonic tremor, 2463-2464 Dystrophic epidermolysis bullosa, 906 Dystrophinopathy, 2541-2543

E

Eagle-Barrett syndrome, 827 Ear(s). See also Hearing in acute poisoning, 697 examination of, 26 Ear disorders, 2585-2592

Ear infections. See also Otitis externa; Otitis media Pseudomonas, 1964 Ear pain, 2589-2592 clinical manifestations of, 2590 definition of, 2589 diagnosis of, 2590-2591 differential diagnosis of, 2591 history in, 2590 imaging in, 2591 laboratory findings, 2590 pathobiology of, 2590 physical findings, 2590 treatment of, 2591b East African sleeping sickness, 2060t, 2114 Eastern Cooperative Oncology Group (Zubrod) Performance Scale, 1207t Eastern coral snake (Micrurus fulvius), 719 Eastern equine encephalitis, 2263t, 2264 clinical manifestations of, 2264 diagnosis of, 2264 epidemiology of, 2264 fatality rate, 2060t features of, 2263t pathobiology of, 2264 prevention of, 2264 prognosis for, 2264 treatment of, 2264b Eastern Europe, 2275 Eating, sleep-related, 2423, 2423t Eating Disorder Examination, 1457 Eating disorders, 1455-1457 among adolescents, 64 clinical manifestations of, 1456-1457 definition of, 1455 diagnosis of, 1457 epidemiology of, 1456 natural history of, 1457 pathobiology of, 1456 prevention of, 1457 prognosis for, 1457 risk factors for, 1456 symptoms and signs of, 1456-1457 treatment of, 1457b Ebola hemorrhagic fever, 2249t clinical manifestations of, 2250, 2251t, 2252f fatality rate, 2060t management of, 2254-2255 pathobiology of, 2250, 2251t prevention of, 2256 prognosis for, 2256 transmission of, 2247 vaccine for, 2255-2256 Ebola virus, 91, 91t, 2056, 2249t Ebola Zaire virus, 2250 Ebstein’s anomaly, 414 chest radiography findings, 285 exercise recommendations for, 416t incidence and prevalence of, 405 right-sided, 414-415 EBV-encoded small RNA (EBER), 1268 Ecallantide (Kalbitor), 1697 Ecchymoses, 2638t-2644t Eccrine glands, 2634 Echinacea, 2186 Echinocandin antifungal agents, 2069 Echinococcosis, 1014, 1015f cystic, 2151-2152 diagnosis of, 1014, 1015f epidemiology of, 1014 pathobiology of, 1014 Echinococcus granulosus, 1015t-1017t, 2147-2148, 2148t, 2151-2152 Echinococcus multilocularis, 1015t-1017t, 2147-2148, 2148t, 2153 Echinococcus oligarthrus, 2153 Echinococcus vogeli, 2153 Echinoderm microtubule-associated protein-like 4 (EML4), 1305-1306 Echinoderms, 721-722 Echinostoma, 2157-2158 Echis (carpet vipers), 719 Echis antivenom, 718 Echocardiographic-guided percutaneous pericardiocentesis, 488-489, 489f Echocardiography, 274-282 in acute MI, 445 approaches, 276-279 in arrhythmia, 350 cardiac measurements, 274-276, 279-280 in cardiogenic shock, 683

Echocardiography (Continued) contrast, 277, 278f in coronary artery disease, 424t, 425 in dilated cardiomyopathy, 275f Doppler, 275-276 examination with, 280-282 in heart failure, 304-305 imaging, 274-275 indications for, 280, 280t-281t in infective endocarditis, 478, 478f integrating with clinical findings, 282 intracardiac, 279 limitations of, 275 normal findings, 280-282 point-of-care, 276 goals for, 277t indications for, 277t preoperative, 2617t principles of, 274 in pulmonary embolism, 623 standard views, 274, 275f, 274.e1 stress, 277, 278f, 425, 277.e1 three-dimensional, 274-275, 276f, 277, 278f, 277.e1 transesophageal, 276, 277t transthoracic, 276, 276t E-cigarettes, 147t Eclampsia, 1166 liver disease in, 1622 severe, 395t Economic issues, 18-19. See also Costs financial assessment, 105, 106t financial conflicts of interest, 8-9 Ectasias, gastrointestinal, 958 Ecthyma, 2695-2696 Ecthyma gangrenosum, 1858, 1859f, 2695-2697 Ectodermal dysplasia, anhidrotic, 1687t Ectodermal dysplasia–skin fragility syndrome, 2633 Ectoparasites, 2698 Ectopic ACTH secretion syndrome, 1491, 1516 Ectopic atrial rhythms, 350t-351t, 359, 359f clinical manifestations of, 359 diagnosis of, 359 treatment of, 359 Ectopic hormone syndromes, 1219-1220, 1220t Ectopic pregnancy, ruptured, 634t Ectropion, 2560, 2561f Eculizumab, 245f, 1079 Eczema, 2662-2664, 2662t, 2702t Eczema vaccinatum, 2217 Edema, 254 angioedema, 1693-1697, 1694t cerebral, 2433 in bacterial meningitis, 2481 high-altitude, 597 after intracerebral hemorrhage, 2452 after subarachnoid hemorrhage, 2449 diagnostic approach to, 256f due to heart failure, 303 lymphedema, 1652t papilledema, 2575 pitting, 256f postoperative, 2623-2624 pulmonary acute, 319f acute decompensated heart failure with, 318-319 cardiogenic, 662, 664t distinguishing features of, 528t high-altitude, 596-597 imaging of, 534f in pregnancy, 1614t scleromyxedema, 2694 subglottic, 2189, 2189f Edematous malnutrition, 1435-1436, 1435f Edetate calcium disodium, 706t-710t Edoxaban, 178, 178t-179t, 625 Edrophonium (Tensilon), 2551 Education about fibromyalgia, 1818 about glucocorticoid replacement therapy, 1521 about heart failure, 315, 315t about irritable bowel syndrome, 893 sleep hygiene education, 2421t and smoking, 17, 17f about STEMI, 455-456 about traveler’s diarrhea, 1882t

Index Educational programs, 47 Edwards’ syndrome, 192 Efavirenz, 2288t CSF-to-plasma ratios, 2331t fixed-dose combinations, 2289, 2290t for HIV and TB co-treatment, 2038 for HIV/AIDS, 2289, 2290t side effects of, 2289, 2291 Effect-cause, 36 Effective arterial blood volume, 742 Effector cells, 1781 Efferocytosis, 418 Effusion lymphoma, primary, 1265 Effusions cardiac, 486-489 malignant, 1219 otitis media with, 2590, 2590f pericardial, 280, 282f, 280.e1 causes of, 486-487, 486t chest radiography in, 284, 284f, 487, 487f management of, 488f without tamponade, 488 peritoneal, 1219 pleural, 632-635 subdural, 2481 subpulmonic, 537, 537f Effusive-constrictive pericarditis, 490 Eflornithine, 2115 for human African trypanosomiasis, 2115t Egophony, 526 Egypt, 997 Egyptian cobras (Naja haje), 719 Ehlers-Danlos syndrome, 1172, 1735-1736, 1736t, 2634 atrophic skin in, 2693-2694, 2693f clinical manifestations of, 1736 definition of, 1735 diagnosis of, 1736 differential diagnosis of, 1736 epidemiology of, 1735 esophageal involvement, 906 gastrointestinal involvement, 958 pathobiology of, 1735-1736 pathogenesis of, 1735 pathology of, 1735-1736 treatment of, 1736b Ehrlichia, 2047t, 2053 Wisconsin, 2026, 2055 Ehrlichia canis, 2053-2055, 2053t Ehrlichia chaffeensis, 2053 Ehrlichia ewingii, 2053-2054, 2053t Ehrlichia sennetsu, 2053 Ehrlichioses, 2047t, 2053-2055, 2053t distinguishing characteristics of, 2252t-2253t geographic distribution of, 2053-2054, 2054f granulocytic canine, 2054 human, 2053-2054, 2054f monocytic, 2057t, 2059t American human, 2053 canine, 2054-2055 human, 2053, 2054f Japanese, 2053t treatment of, 2053 Eicosanoids, 232 Eisenmenger’s complex, 409 definition of, 405 diagnosis of, 409 treatment of, 410 Eisenmenger’s physiology, 405 Eisenmenger’s syndrome clinical manifestations of, 406 definition of, 405 treatment of, 407 Ejaculatory failure, 1576-1577 Ejection, 265 Ejection fraction, 274-275 formula for, 274-275 preserved, 300-301, 300t, 317 reduced heart failure with, 299-300 selection of ICDs for, 454f three-dimensional echocardiography findings, 274-275, 276f El Escorial Criteria, 2524t El Niño–southern oscillation (ENSO), 1950

El Niño–southern oscillation (ENSO) V. cholerae, 1951 Elapidae (coral snakes), 717, 719 Elastase-1, 925t Elastic connective tissue, 2634 Elastic fibers, 1731 Elavil (amitriptyline), 893t Elbow disorders, 1752 bursitis, 1750t golfer’s elbow, 1752 tendinitis, 1750t tennis elbow, 1751-1752 Elbow procedures, 1832 Elderly. See Older adults Electrical alternans, 487, 487f Electrical cells, 263 Electrical conduction abnormalities, 270-271 of heart, 263, 267, 267f, 356.e3f Electrical injury, 711-717, 712t clinical manifestations of, 711 diagnosis of, 713b-716b interhospital transport, 713 prehospital care, 713 rehabilitation for, 716 surgical care for, 716 systemic sequelae, 712f treatment of, 713b-716b Electrical storm, 373 Electrocardiography, 267-274 abnormal, 270-273 in acute coronary syndrome, 434-435 in acute MI, 443-444, 443f-444f in angina pectoris, 423-425 approach to interpretation, 270-274, 270t in arrhythmias, 347-348 in athletes, 273-274, 274t automated computerized readings, 273 axes of electrical activation, 270, 270f, 270.e1f axis deviation, 270-271 in chronic ischemic heart disease, 422, 422f exercise, 423-425 in heart failure, 304 intracardiac, 356.e3f leads, 268-270, 269.e1f monitoring, 453-454 in narrow-complex tachycardia, 348, 348f normal, 267-270, 268f, 270f normal intervals, 268-269, 268f, 268t in pulmonary embolism, 623, 623f resting, 422, 422f in right ventricular infarction, 444 screening, 274 standards, 268 stepwise approach to interpretation, 270t surface, 356.e3f waves, 267-268, 268f in wide-complex tachycardia, 348, 349f Electrochemical forces, 757 Electroconvulsive therapy, 2629, 2629f Electroencephalography, 2341-2342 abnormalities, 2342, 2343f, 2343t in epilepsy, 2404 normal, 2342, 2343f Electrolytes in hypovolemia, 745 in infectious diarrhea, 1870t intestinal transport mechanisms, 918-919, 918.e3f in viral hemorrhagic fevers, 2254 Electromagnetic radiation, 81-82, 81t Electromagnetic spectrum, 86, 86f Electromyography (EMG), 2343-2344, 2540 abnormalities, 2343-2344, 2344t Electronic consultations, 2610 Electronic nose devices, 545 Electronystagmography, 2345 Electrophysiologic studies, 295 in arrhythmias, 350 impulse propagation, 341 interventional procedures, 374-376 normal, 356 principles of, 339-344 Elephantiasis, 2167-2168, 2168f Eletriptan, 2358-2359, 2359t Elimination, enhanced, 705-706 Elimination diets, 902 Elimite (permethrin), 2659

Ellinghausen-McCullough-Johnson-Harris (EMJH) media, 2030 Elliptocytes, 1054f Elliptocytosis, 1054f, 1083-1084 Eloxatin (oxaliplatin), 1211t-1216t Eltrombopag (Promacta), 1119, 1164, 1184 Elvitegravir, 2288t fixed-dose combinations, 2289, 2290t for HIV/AIDS, 2290t side effects of, 2289 Embden-Meyerhof pathway, 1084-1085, 1085f disorders of, 1086 hexose monophosphate shunt, 1085-1086, 1085f Rapoport-Luebering shunt, 1085, 1085f Embolism. See also Thrombosis amniotic fluid, 627, 627.e1f cholesterol, 2674 cutaneous, 2675 fat, 626, 626.e1f postoperative, 1832-1833 stroke in, 2435 symptoms and signs of, 1833 treatment of, 626b pulmonary, 620-627 superior mesenteric, 953-954 Embryopathic disorders, congenital, 1476 Emergency esophageal, 907-908 life-threatening skin disease, 2644-2655 Emergency contraception, 1609 Emergency department care for acute STEMI, 445-446 for asthma, 554 incidental blood pressure elevation in, 396 Emergency letters, 1389 Emergency Plan for AIDS Relief (U.S.), 2272 Emery-Dreifuss muscular dystrophy, 329, 342, 2544 Emollients, 2636t Empathy nonverbal, 14t verbal, 14t Emphysema congenital lobar, 570 CT findings, 558, 559f endoscopic management of, 645 imaging of, 535f panacinar, 556 pathology of, 556 radiographic findings, 558, 558f, 551.e1f Employment history, 24 Empty sella syndrome, 1482 Empty ventricle syndrome, 1702 Empyema, 634 Aspergillus, 2083, 2086 pleural fluid characteristics, 634, 634t subdural, 2498 Emtricitabine, 1884, 2288t CSF-to-plasma ratios, 2331t fixed-dose combinations, 2289, 2290t for HIV/AIDS, 2289, 2290t initial regimens, 2289, 2290t side effects of, 2289 ENaC antagonists (potassium-sparing diuretics), 391 Enalapril for acute STEMI, 450-451 for dilated cardiomyopathy, 330 for heart failure, 310t for hypertension, 388t Enanthems, 2242 Enbrel (etanercept), 169, 1762 Encarbil, 2424 Encephalitis amoebic, 2106 antiviral therapy for, 2181t arboviral, 2263t arthropod-borne viruses associated with, 2262, 2263t causes of, 2501t, 2503-2504 clinical manifestations of, 2242 cytomegalovirus, 2208, 2209f definition of, 2500 diagnosis of, 2242 Eastern equine, 2060t enteroviral, 2241t, 2242 herpes simplex, 2226, 2226f limbic, 2502t

I35

Encephalitis (Continued) listerial rhombencephalitis, 1919 mosquito-borne, 2058t mumps-induced, 2207 neonatal, 2502t prognosis for, 2242b progressive rubella panencephalitis, 2210 rabies, 2058t subacute sclerosing panencephalitis, 2209-2210 toxoplasmic, diffuse, 2127 treatment of, 2242b viral acute, 2500-2504 CSF formula, 2342t vectors, 2172t West Nile fever, 2057, 2057t Encephalomyelitis acute disseminated, 2479 myalgic, 1821 parainfectious, 2599 post-vaccination, 2217 Encephalomyopathy, mitochondrial, 1652t, 1660-1661, 2546 Encephalopathy bismuth, 96, 97b bovine spongiform, 2504 in cirrhosis, 1025 hepatic, 977, 979t in cirrhosis, 1027-1028 diagnosis of, 1028 EEG findings, 2343f physical findings, 979 treatment of, 1030-1031 hypertensive, 395t leukoencephalopathy cerebral autosomal dominant arteriopathy with subcortical infarcts and (CADASIL), 2394 posterior syndrome, 2575 progressive multifocal, 2210-2212, 2502t posterior reversible syndrome, 1861 variable protease, 2505 Wernicke’s, 2506-2508 Encevac (CC-JEV), 2266 Enchondromatosis, 1672 ENCODE, 189-190 End-diastolic pressure, 265 End-diastolic volume, 265 Endemic malaria, 2108, 2112 Endemic syphilis, 2020 Endemic typhus, 2172t End-inspiratory volume, 542f Endobronchial ultrasound, 642, 1307 Endocardial cushion defect, 415 Endocardial diseases, 335-338 Endocarditis, 2080 Aspergillus, 2083-2084 bacterial differential diagnosis of, 2484 therapy for, 479t in bacterial meningitis, 2485 Bartonella, 1999 diagnosis of, 2000t treatment of, 2000-2001 Brucella, 1981t continuing care for, 480-481 culture-negative, 477, 477t echocardiographic findings, 280t-281t empirical treatment of, 480t enterococcal, 1914-1915, 1915.e1t fungal, 479t, 480 glomerulonephritis with, 790 H. influenzae, 1947 indications for surgery, 481t infectious, 2500, 2500b infective, 474-483 Löffler’s fibroplastic endocarditis, 336 nonbacterial thrombotic (marantic), 338 pneumococcal, 476 prophylaxis of, 416 antibiotic, 483t for dental procedures, 482t recommendations for, 482t prosthetic valve, 477, 477t, 481t S. aureus, 1898-1899, 1899f S. maltophilia, 1970-1971 treatment of, 1896, 2082 zoonotic, 480, 2057, 2059t Volume I  pp 1-1382  •  Volume II  pp 1383-2722

I36

Index

Endocrine disease approach to, 1468-1470 arthritis in, 1823t cardiomyopathy with, 330 chronic anovulation related to, 1594 in chronic kidney disease, 835 evaluation of response to treatment, 1470 follow-up, 1469 genetic diseases, 1473, 1473t genetic testing in, 1470 high-penetrance monogenic disorders, 187t laboratory findings, 1469-1470 physical findings, 1468-1469 preoperative evaluation of, 2615-2616 symptoms and signs of, 1468, 1469t Endocrine dysfunction anemia of hypofunction, 1066 in fibromyalgia, 1820 after hematopoietic stem cell transplantation, 1204 Endocrine pancreatic insufficiency, 966 Endocrine therapy, 1218, 1355-1357 Endocrinology age-related changes in, 108 principles of, 1471-1473 reproductive, 1584-1597 Endofibrosis, iliac artery, 504-505 End-of-life care, 9. See also Palliative care in delirium, 121 in dementia, 2389 in heart failure, 319-320 in hepatobiliary cancer, 1344-1345 hospice care, 14 Endometrial cancer, 1361-1362 clinical manifestations of, 1362 diagnosis of, 1362 epidemiology of, 1361 pathobiology of, 1362 prevention of, 1362 prognosis for, 1362 staging, 1362t treatment of, 1362b Endometriosis, 1586 definition of, 1586 treatment of, 1587 Endometrium, 1585, 1585f Endomyocardial biopsy, 298 Endomyocardial fibrosis, tropical, 337 Endophthalmitis, 2081, 2564, 2566 End-organ deficiency, 1573 End-organ resistance, 1572t Endoscopic myotomy, peroral (POEM procedure), 878-879, 904 Endoscopic retrograde cholangiography, 873 Endoscopic retrograde cholangiopancreatography, 873 applications, 873t in benign biliary strictures, 1044, 1044f-1045f in bile duct stones, 1042, 1043f in biliary disease, 877, 877f in chronic pancreatitis, 964-965, 965f in pancreatic disease, 877-878, 878f Endoscopic ultrasonography, 873, 873t with bile duct stones, 1042, 1043f in chronic pancreatitis, 964-965, 965f esophageal, 1308 in gastric cancer, 1318, 1318f in hypoglycemia, 1554 in lung cancer, 1308 Endoscopic ultrasonography–guided biopsy, 877-878, 878f Endoscopy, 872 ancillary organ imaging, 873 capsule, 873t complications of, 873t diagnostic, 898 in emphysema, 645 functional imaging, 878-879, 878.e1f gastrointestinal, 872-879 in GI disorders, 854 importance of, 872 luminal, 872-873, 873t in pancreatitis, 962 pancreatobiliary, 877-878 Endosomes, 221f Endosteal hyperostosis, 1669, 1669b Endothelial cells, sinusoidal, 1023 Endothelial wall adherence, 1145 Endothelin 1, 304.e1t Endothelium, 686

Endovascular infections, 1965 Endovascular therapy, 2441 End-stage renal disease anemia in, 842 definition of, 833 epidemiology of, 783, 804 life expectancy in, 843, 843f metabolic bone disease in, 842 secondary to diabetes, 804 End-systolic volume, 265 End-tidal capnometry, 653, 653f Energy balance CNS modulators of, 1460, 1460t regulation of, 1460 Energy expenditure, 1460-1461 components of, 1433 measurement of, 1433-1434 of physical activity, 1461 Energy intake, 1460 Energy metabolism, 1231 Energy therapy, 182t, 184, 379 Enfuvirtide, 2288t, 2331t English proficiency, limited, 15-16, 15f Engraftment syndrome, 1203 Enhanced external counterpulsation (EECP), 429-430 Enoxaparin, 177t for acute coronary syndrome, 438t-439t, 439 for acute STEMI, 448, 450 for deep venous thrombosis prevention, 2368-2369 in pregnancy, 1618t-1619t for pulmonary embolism, 625, 625t for venous thromboembolism, 1619t ENT specialists, 2589t Entacapone, 2457t-2459t Entactin, 233t Entamoeba histolytica, 1013, 1015t-1017t, 2138, 2139f Entecavir, 2178 adverse effects of, 2178t for hepatitis B, 1001-1002, 2177t mechanisms of excretion and thresholds for dose adjustment, 2178t Enteral nutrition, 1437-1440 administration of, 1442 complications of, 1439-1440 effects of not providing, 1438 indications for, 1442t initiating, 1438-1439 physiologic benefit of, 1437-1438, 1438t protocol for nasogastric feeding, 1439t Enteric fever clinical manifestations of, 1973 treatment of, 1974 Enteric infections approach to, 1868-1872 clinical features of, 1868-1869, 1869t diagnosis of, 1869-1870 E. coli, 1956-1959 epidemiology of, 1868, 1869t etiology of, 1868-1869 laboratory findings, 1869-1870, 1870t treatment of, 1870b-1872b Enteric nervous system, 884 Enteric neuropathic disorders, 886 Enteric protozoa, 2145-2147, 2147t Enteritis C. perfringens type C enteritis, 1930 Campylobacter clinical features of, 1954, 1954t treatment of, 1955 coccidian, 2146-2147, 2147b radiation, 948 Enteritis necroticans, 1930 Enterobacteriaceae, 1960 Enterobacteriaceae infections, 1960-1962 antibiotic resistance, 1960, 1961t carbapenem-resistant, 1864 guidelines and recommendations for, 1865.e1t treatment of, 1962 diagnosis of, 1961 epidemiology of, 1960 ESBL-producing, 1962 health care–associated, 1864, 1865.e1t medically important members, 1960t multidrug-resistant, 1960t pathobiology of, 1960-1961 prevention of, 1962

Enterobacteriaceae infections (Continued) prognosis for, 1962 treatment of, 1960t, 1961b-1962b Enterobiasis, 2162 clinical manifestations of, 2162 diagnosis of, 2162 epidemiology of, 2162 treatment of, 2160t, 2162b Enterobius vermicularis, 2159, 2162 Enterococcal bacteremia, 476, 1915.e1t Enterococcal infections, 1913-1915 clinical manifestations of, 1914 definition of, 1913 diagnosis of, 1914 epidemiology of, 1913-1914 pathobiology of, 1914 prevention of, 1915 prognosis for, 1915 risk factors for, 1913 treatment of, 1914b-1915b, 1915.e1t vancomycin-resistant, 1913-1914 Enterococcus, 1913 Enterococcus avium, 1913 Enterococcus caccae, 1913 Enterococcus casseliflavus, 1913 Enterococcus cecorum, 1913 Enterococcus dispar, 1913 Enterococcus durans, 1913 Enterococcus faecalis, 1913, 1915.e1t Enterococcus faecium, 1913 antibiotic resistance, 1914 epidemiology of, 1913 in health care–associated infections, 1863t, 1864 treatment options for, 1915.e1t vancomycin-resistant, 1863t, 1864 Enterococcus gallinarum, 1913 Enterococcus gilvus, 1913 Enterococcus hirae, 1913 Enterococcus italicus, 1913 Enterococcus malodoratus, 1913 Enterococcus mundtii, 1913 Enterococcus pallens, 1913 Enterococcus pseudoavium, 1913 Enterococcus raffinosus, 1913 Enterococcus sanguinicola, 1913 Enterocolitis neutropenic, 948, 1860-1861 Salmonella, 1973-1974 Enterocytozoon bieneusi, 2147t Enterocytozoon intestinalis, 2147t Enterohemorrhagic Escherichia coli infection, 2252t-2253t Enterohepatic circulation, 984 Enteropathic arthritis, 1768-1769, 1768t classification of, 1713t definition of, 1768 diagnosis of, 1769 differential diagnosis of, 1764t pathobiology of, 1768-1769 Enteropathogenic yersiniae, 1988-1990 Enteropathogenic yersiniae infections clinical manifestations of, 1990 diagnosis of, 1990 epidemiology of, 1988-1989 pathobiology of, 1989-1990 prevention of, 1990 prognosis for, 1990 treatment of, 1990b Enteropathy gluten-sensitive, 1824 NSAID-induced, 947-948, 948f tropical, 1434 Enteropeptidase (enterokinase) deficiency, congenital, 929 Enteroscopy applications, 873t double balloon, laparoscopic-assisted, 1322.e1 Enteroviral meningitis, 2489-2491 Enterovirus infections asymptomatic, 2241t in B-cell immunodeficiency, 2243-2244 central nervous system infections, 2241-2242 clinical manifestations of, 2240, 2241t clinical syndromes, 2241-2244 diagnosis of, 2240 epidemiology of, 2239-2240 pathobiology of, 2240 prevention of, 2244 prognosis for, 2240b

Enterovirus infections (Continued) respiratory tract syndromes, 2243 treatment of, 2240b Enteroviruses, 2185t, 2239-2244 classification of, 2239, 2239t definition of, 2239 serotypes, 2239, 2240t Entocort EC (budesonide), 939t Entropion, 2559-2560, 2560f Entry inhibitors (EIs), 2288t, 2289 Envenomation, 717-722 epidemiology of, 717 marine, 720-722 scorpion sting, 2548t sea snake, 721 snake bite, 717-720, 2548t spider bite, 2548t treatment of, 717b Environment awareness of, 2409 and obesity, 1459-1460, 1459t Environmental assessment, 105, 106t Environmental control, 1691 Environmental disease, 79-80 Environmental exposures hazardous, 81-82, 81t and non-Hodgkin’s lymphoma, 1258 poisonings, 920 and systemic sclerosis, 1778-1779 that cause COPD, 555-556 Environmental health disorders, 80-81, 80t Environmental history, 79-80, 79.e1f Environmental medicine, 78-82 Environmental stress, 231 Enzalutamide (Xtandi), 1211t-1216t, 1369 Enzyme deficiency states, 1202-1203 Enzyme immunoassays (EIAs), 2017, 2017t Enzyme polymorphisms, 131-132, 132t, 1007 Enzyme therapy for cancer, 1211t-1216t for exocrine pancreatic insufficiency, 966, 966t for Fabry’s disease, 1402, 1402t for Gaucher’s disease, 1401, 1402t for inborn errors of metabolism, 1388 Enzyme-linked immunosorbent assay CL ELISA assay, 1180 in pneumonia, 614 β2-GPI ELISA assay, 1180 Enzymes: serum levels, 989-990 Eosin-5-maleimide, 1082, 1083f Eosinopenia, 1138 Eosinophilia causes of, 1152t chronic hyperplastic eosinophilic sinusitis, 1691, 1691t classification of, 1151 definition of, 1132 drug rash with eosinophilia and systemic symptoms, 2685-2686, 2685t non-AR with, 1691 persistent, 1153, 1153f pulmonary, tropical, 2168 Eosinophilic disorders future directions, 1154 organ-restricted disease, 336 prognosis for, 1154 Eosinophilic esophagitis, 902 diagnosis of, 902t endoscopic findings, 902, 902f treatment options for, 902 Eosinophilic fasciitis, 2694 Eosinophilic folliculitis, 2321, 2321f Eosinophilic gastroenteritis, 934 Eosinophilic granuloma, 576 Eosinophilic granulomatosis with polyangiitis clinical manifestations of, 1798 differential diagnosis of, 1153 treatment of, 1798 Eosinophilic meningitis, 2492-2495, 2493t Eosinophilic myositis, 1790t Eosinophilic pneumonia, chronic, 586 Eosinophilic pustular folliculitis, 2681 Eosinophilic sinusitis, hyperplastic, 1691t Eosinophilic syndromes, 1151-1154 clinical manifestations of, 1152-1153 definition of, 1151 diagnosis of, 1153 diagnostic evaluation of, 1153 differential diagnosis of, 1153 epidemiology of, 1151

Index Eosinophilic syndromes (Continued) genetic etiology of, 1151 pathobiology of, 1152 Eosinophils, 217, 1152, 1152f Epalrestat, 2521 Ependymoma, 1294 Ephedrine, 2553 Epicondylitis lateral, 1752 medial, 1752 Epidemic cholera, 1950 Epidemic febrile polyarthritis, 2261 Epidemic HIV/AIDS, 2272-2278, 2272.e1 Epidemic influenza, 2192-2193, 2192t Epidemic typhus, 2048t, 2051-2052, 2059t clinical manifestations of, 2052 diagnosis of, 2052 epidemiology of, 2051 treatment of, 2052b Epidemics, regional, 2272-2278, 2272.e1 Epidermal adhesion, 2633, 2632.e1f Epidermal appendages, 2633f, 2634 Epidermal growth factor receptor, 171 Epidermal growth factor receptor inhibitor–associated rash, 2686, 2686f Epidermal necrosis, 2638t-2644t, 2642f Epidermal tumors, benign, 2687-2689 Epidermal wrinkling, 2693-2694 Epidermis, 2632-2634, 2633f basal layer, 2632 dermal-epidermal junction, 2634 differentiation of, 2632, 2633f granular layer, 2632 nonkeratinocytes of, 2633 nonresident cells of, 2633-2634 spinous layer, 2632 Epidermodysplasia verruciformis, 2220 Epidermolysis bullosa, 2678 dystrophic, 2678 junctional, 2678 oral ulcers, 2580t subtypes, 2678 Epidermolysis bullosa acquisita, 2676 pathobiology of, 2676 treatment of, 2676b Epidermolysis bullosa simplex, 2678, 2678f Epididymitis, 2010 Epidural anesthesia and analgesia, 2620 Epidural hematoma, 2366, 2367f Epidural tumors, 1296, 1296f Epigastric hernias, 945 Epigastric pain syndrome, 894, 894t Epigenetics, 1306 Epiglottis, 528t Epiglottitis, 2604 clinical features of, 2189, 2190f, 2602t H. influenzae, 1947-1948 H. influenzae type b, 1947 Epilepsy, 2399-2409 absence childhood, 2404-2405 EEG findings, 2342, 2343f adolescent-adult syndromes, 2406t benign, with centrotemporal spikes, 2406 benign rolandic, 2406 childhood syndromes, 2404-2406, 2406t classification of, 2406t clinical manifestations of, 2400-2403 constellations, 2404-2406 definition of, 2399 diagnosis of, 2399, 2404 diagnostic investigations in, 2403-2404 differential diagnosis of, 2403 EEG findings, 2404 epidemiology of, 2399-2400 genetic, with febrile seizures plus (GEFS+), 2404 genetic testing, 2404 genetics of, 2400 imaging findings, 2404, 2405f incidence of, 2399 infantile syndromes, 2404, 2406t juvenile myoclonic, 2406 magnetoencephalography findings, 2404 molecularly guided therapeutics for, 202.e1t myoclonic epilepsy with ragged-red fibers (MERRF), 2546 neonatal syndromes, 2404, 2406t pathobiology of, 2399-2400

Epilepsy (Continued) pathogenesis of, 2399-2400 preconception interventions for women, 1612t prevalence of, 2399 prognosis for, 2409 risk factors for, 2399 rolandic, benign, 2406 severe myoclonic, of infancy, 2404 syndromes, 2404 syndromes with favorable course, 2407 temporal lobe, 2400f, 2406 treatment of, 2406b-2408b Epinephrine for anaphylaxis, 1702-1703 for angioedema, 1697 for croup, 2190 premedication for snake antivenom, 717 for pulseless electrical activity, 354f-355f, 355 for septic shock, 689 for shock, 676t, 680 for urticaria, 1695 Epirubicin (Ellence), 1211t-1216t, 1319 Episodic memory, declarative, 2383t Episodic weakness, 2340 Epistaxis, 2589, 2589f Epithelial cell apoptosis, 686 Epithelial cell infections, 1877t, 1878-1879 Epithelial proliferations, 2583-2584, 2583t Epithelial (follicular) thyroid carcinoma, 1512-1513 Epithelial tumors, papillary, 2583-2584, 2584f Epithelial-to-mesenchymal transition, 1230 Eplerenone, 748t for acute STEMI, 451, 454-455 for heart failure, 312t for hypertension, 388t, 391 for mineralocorticoid-induced hypertension, 386 Epley’s maneuver, 2598f, 2601t Epoetin alfa, 1067 Epogen (erythropoietin), 1211t-1216t Epoprostenol, 402 Eprodisate, 792 Eprosartan, 388t Epsilon waves, 372f Epstein-Barr nuclear antigens (EBNAs), 2232 Epstein-Barr virus encephalitis, 2502t Epstein-Barr virus infection, 2209, 2232-2235 in aplastic anemia, 1115 associated neoplasms, 1268, 2234 chronic active, 2234 clinical manifestations of, 2233-2234 complications of, 2233-2234 definition of, 2232 diagnosis of, 2233, 2603-2604 epidemiology of, 2232 in HIV infection, 2319 pathobiology of, 2232 prevalence of, 2603 prevention of, 2233 prognosis for, 2233-2234 in systemic lupus erythematosus, 1770 treatment of, 2233 Epstein-Barr virus pharyngitis, 2252t-2253t Epstein-Barr virus–associated PTLD, 1038, 2234 Eptifibatide (Integrilin), 180, 438t-439t, 448 Epworth Sleepiness Scale, 2417-2418, 2418t Equilibrium, 2593-2601 Erbitux (cetuximab), 1211t-1216t Erectile dysfunction, 1577-1578 definition of, 1577 diagnosis of, 1578 epidemiology of, 1577 medical therapy for, 1578-1579 pathobiology of, 1577-1578 treatment of, 1578b-1579b, 1578f Erectile physiology, 1570, 1570f Ergocalciferol (vitamin D2), 1649, 1659 Ergonomics, 1822 Ergotamine, 2358-2359 Eribulin mesylate (Halaven), 1211t-1216t Erivedge (vismodegib), 1211t-1216t Erlenmeyer flask deformity, 1400, 1401f Erlotinib (Tarceva), 171, 1211t-1216t, 1232t, 1310t

Erosions, 909f, 2636t, 2651t-2654t definition of, 908 recurrent, of cornea, 2563 Erosive gastritis, 908, 909f Errors, 45 in postoperative care, 2621 radiologic accidents, 82-83, 83t, 85 standard, 35-36 Swiss cheese model of, 45 Ertapenem for pneumonia, 619t, 1905t for pyelonephritis, 1875t for pyogenic liver abscess, 1012 recommended doses and schedules, 1891t-1892t Eructation (belching), 859 Erysipelas, 1898, 1907, 2696-2697, 2696f Erysipeloid infection, 1923, 1924f, 2059t Erysipelothrix infections, 1923-1924 Erysipelothrix rhusiopathiae, 1923 Erysipelothrix rhusiopathiae infection clinical manifestations of, 1923-1924, 1924f diagnosis of, 1924 epidemiology of, 1923 pathobiology of, 1923 patterns of, 1923 prevention of, 1924 treatment of, 1924b Erythema acute, 2655 color clues, 2638t-2644t figurative, 2671 necrolytic acral, 2670, 2670f necrolytic migratory, 1337, 1337f reactive, 2683t toxic, 2697 Erythema ab igne, 508-509, 508f Erythema annulare centrifugum, 2671 Erythema chronicum migrans, 2671 Erythema elevatum diutinum, 2692 Erythema gyratum repens, 2671 Erythema induratum, 2693, 2693f Erythema infectiosum, 2212, 2213f, 2672, 2673f Erythema marginatum, 2684f Erythema migrans, 2021, 2022f differential diagnosis of, 2025 multiple lesions, 2023 single lesion, 2023 symptoms and signs of, 2022-2024, 2023t Erythema multiforme, 2647f, 2677-2678, 2677f, 2685t, 2686 clinical manifestations of, 2677 definition of, 2677 diagnosis of, 2678 oral ulcers with, 2580t, 2581 pathobiology of, 2678 prognosis for, 2678 treatment of, 2678b Erythema nodosum, 605, 2646f, 2685t, 2692-2693, 2692f, 2693t Erythematous (atrophic) oral candidiasis, 2582-2583, 2582f Erythrasma, 2697 Erythroblastosis fetalis, 66t Erythrocyte enzymopathies, 1086, 1087f Erythrocyte sedimentation rate, 735, 1501, 1719 Erythrocytes. See Red blood cells Erythrocytosis causes of, 1124, 1126t classification of, 1126t of cyanotic congenital heart disease, 407, 407f Erythromelalgia, 507, 508f, 508t, 1122-1123, 1122f Erythromycin for bacillary angiomatosis, 2001 for bacterial overgrowth, 928 for blepharitis, 2561 for bronchectasis, 569 for bronchitis, 609 for Campylobacter enteritis, 1955 for chlamydia, 2011 for chlamydial conjunctivitis, 2565 for diphtheria, 1917, 2536 before endoscopy, 880 for erythrasma, 2697

I37

Erythromycin (Continued) for eye infections, 2565t for gastroparesis, 888 for legionnaires’ disease, 1996t for Mycoplasma pneumonia, 2006 for peliosis hepatis, 2001 for pertussis, 1992 for pharyngitis, 2602-2603 for pityriasis lichenoides, 2669 recommended doses and schedules, 1891t-1892t for S. pneumoniae infection, 1905t for tracheitis, 609 for urethritis, 1878t Erythroplakia, 1299, 1299f, 2582 Erythropoiesis, 1061 Erythropoiesis-stimulating agents, 1067, 1236-1237 Erythropoietic porphyria, 1408t congenital, 1408t classification of, 1408 clinical manifestations of, 1411 epidemiology of, 1408 prevention of, 1415 skin disease in, 1412-1413 treatment of, 1415 etiology of, 1410 Erythropoietic protoporphyria classification of, 1408 clinical manifestations of, 1411 congenital, 1412 diagnosis of, 1413 prevention of, 1415 skin disease in, 1412-1413, 1413f treatment of, 1415 Erythropoietin (Aranesp, Epogen, Procrit), 1050 for anemia, 1067, 1071 for cancer, 1211t-1216t clinical uses of, 1050-1051 plasma levels, 1061, 1061f recombinant human (rHuEPO), 1065-1066, 1066f regulation of erythropoiesis by, 1061 for septic shock, 690 for sideroblastic anemia, 1073 Escherichia coli, 1956 Escherichia coli infections clinical features of, 1869t diarrheogenic, 1956 diffusely adherent, 1956, 1957t enteric, 1956-1959 clinical manifestations of, 1958 diagnosis of, 1958-1959 distinguishing characteristics of, 2252t-2253t epidemiology of, 1956-1957 pathobiology of, 1957-1958 prevention of, 1959 prognosis for, 1959 treatment of, 1959b types of, 1956, 1957t enteroadherent, 1956, 1957t enteroaggregative, 1956, 1957t enterohemorrhagic, 1956, 1957t enteroinvasive, 1956, 1957t enteropathogenic, 1956, 1957t enterotoxigenic, 1869t, 1956, 1957t epidemiologic features of, 1869t health care–associated, 1863t shigatoxigenic, 1956, 1957t Escherichia coli O104:H4−induced hemolytic-uremic syndrome, 1166 Escherichia coli O157:H7−induced hemolytic-uremic syndrome, 1166 Escitalopram, 1628t-1629t, 2349t Escobar’s syndrome, 2548t Esdepallethrin, 2173 ESKAPE bacteria, 1963 Esmolol for cardiogenic shock, 685 clinical use of, 430t for electrical storm, 373 for hypertensive emergency, 396t for ventricular tachycardia, 374 Esomeprazole, 900t, 915, 917 Esophageal adenocarcinoma, 1315-1316 clinical manifestations of, 1316 diagnosis of, 1316 epidemiology of, 1315 Volume I  pp 1-1382  •  Volume II  pp 1383-2722

I38

Index

Esophageal adenocarcinoma (Continued) etiology of, 1315 laboratory findings, 1316 pathobiology of, 1315-1316 physical findings, 1316 symptoms and signs of, 1316 treatment of, 1316b Esophageal atresia, 908 Esophageal candidiasis, AIDS-associated, 2297t-2301t Esophageal disease(s), 896-908 congenital disorders, 908 drug therapy for, 900t in HIV infection, 2297t-2301t, 2304 motility disorders, 904 symptoms of, 897 Esophageal diverticula, 904-905, 905f Esophageal emergencies, 907-908 Esophageal fistula, 908 Esophageal function testing, 897 Esophageal infections, 906-907 Esophageal injury caustic, 902 medications associated with, 903t Esophageal motor disorders, 903-904 Esophageal myotomy, peroral endoscopic (POEM), 878-879, 904 Esophageal perforation, iatrogenic, 907 Esophageal reflux, 249t Esophageal reflux testing, 898 Esophageal rings and webs, 905-906 Esophageal rupture, 634, 634t Esophageal spasm, diffuse, 904 Esophageal squamous cell carcinoma, 1314-1315 clinical manifestations of, 1314 diagnosis of, 1314-1315 epidemiology of, 1314 laboratory findings, 1314 medical therapy for, 1315 pathobiology of, 1314 physical findings, 1314 prognosis for, 1315 risk factors for, 1314, 1314t surgical therapy for, 1315 symptoms and signs of, 1314 treatment of, 1315b Esophageal ulcers, 906 Esophageal varices bleeding, 879, 879f endoscopy in, 875, 875f Esophagitis, 902, 2080 bacterial, 907 candidal, 907, 907f cytomegalovirus AIDS-associated, 2297t-2301t antiviral therapy for, 2181t eosinophilic, 902 medications associated with, 903t pill-induced, 902 reflux, 874, 874f treatment of, 2069, 2181t Esophagogastroduodenoscopy applications, 873t in infectious diarrhea, 1869-1870, 1870t Esophagus alcohol-related complications, 151t bird’s beak deformity, 903, 903f cancer of, 1313-1314, 1314t-1315t congenital abnormalities of, 908 corkscrew, 904, 904f jackhammer, 904 neoplasms of, 1313-1314 normal anatomy of, 896-897 nutcracker, 904 physiology of, 896-897 skin disorders that involve, 906 structural abnormalities, 904-906 in systemic diseases, 906 Esotropia, 2558 Essential mixed cryoglobulinemia, 1723 Essential tremor, 2462 treatment of, 2462 vocal tremor in, 2605 Estimation, generalized, 36 Estradiol for hot flushes, 1625-1626 for vaginal dryness, 1627-1629 for vasomotor symptoms, 1626t Estradiol-17β, 1593

Estrogen levels, normal, 1623-1624 Estrogen therapy for abnormal uterine bleeding, 1588 for amenorrhea, 1589 dose tapering, 1627 for hot flashes, 1626 for hot flushes, 1625-1626 hypertension associated with, 393-394 for hypopituitarism, 1483t for osteoporosis, 1593, 1643 postmenopausal, 1626-1627 for vaginal dryness, 1626t, 1627-1629 for vasomotor symptoms, 1625-1627, 1626t Estrostep (norethindrone acetate and ethinyl estradiol), 2660 Eszopiclone, 2352t, 2421, 2421t Etanercept (Enbrel), 169, 1762, 2666t Ethacrynic acid, 388t Ethambutol for AIDS-associated opportunistic infections, 2297t-2301t for M. kansasii lung disease, 2042 ocular effects of, 2573, 2573t recommended doses and schedules, 1891t-1892t for TB in HIV infection, 2312t Ethanol metabolism of, 150, 150f toxic levels, 703t Ethanol excess, 1056t Ethical issues bioethics, 4-9 in genetic testing, 188 Ethinyl estradiol for contraception, 1609 drospirenone and ethinyl estradiol (Yaz), 2660 norethindrone acetate and ethinyl estradiol (Estrostep), 2660 norgestimate and ethinyl estradiol (Ortho Tri-Cyclen), 2660 for osteoporosis prevention, 1593 Ethnic neutropenia, 1133 Ethosuximide characteristics of, 2408t for seizures, 2407t-2408t Ethylene glycol, 769 Ethylene glycol intoxication, 699t-703t, 706t-710t, 769 Ethylenediamine tetraacetic acid (EDTA), 93t Etidronate, 1667, 1667t Etonogestrel, 1609 Etoposide (VP-16, VePesid) for cancer, 1211t-1216t, 1312, 1366 for Cushing’s syndrome, 1517 for Hodgkin’s lymphoma, 1271-1272 for non-Hodgkin’s lymphoma, 1263t Etravirine, 2288t EULAR Sjögren Syndrome Disease Activity Index (ESSDAI), 1788 EULAR Sjögren Syndrome Patient Reported Index (ESSPRI), 1788 Eulexin (flutamide), 1211t-1216t Eumycetoma. See Mycetoma Eurax (crotamiton), 2659 Europe aging of, 100 Central European encephalitis, 2263t, 2268 Eastern Europe, 2275 HIV/AIDS epidemic in, 2275 Western Europe, 2275 European AIDS Clinical Society (EACS), 2288t European Americans, 1458 European encephalitis, 2268 European Federation of Internal Medicine, 3 European League Against Rheumatism, 1759-1760, 1768, 1760.e1f European Neuroendocrine Tumor Society (ENET), 1335 European snakes, 719-720 European Society of Cardiology, 347t Eutectic mixture of local anesthetic (EMLA), 2659-2660 Euthanasia, 7-8 definition of, 7-8 empirical data for, 8 history of, 7 justification for, 7-8

Euthanasia (Continued) practical considerations, 8 safeguards, 8 Evacuation disorders clinical clues suggestive of, 889t diagnosis of, 889 Evaluation of Genomic Applications in Practice and Prevention (EGAPP), 203.e2t Evans syndrome, 1073-1074 Event monitors, 349 Everolimus (Affinitor), 1232t adverse effects of, 1037t for cancer, 1211t-1216t, 1348 for carcinoid syndrome, 1559 for immunosuppression, 1037-1038 monitoring, 1037t in renal transplantation, 844t for tuberous sclerosis complex, 2515-2516 Evidence-based medicine for Alzheimer’s disease, 2394 guidelines for management of fever in neutropenia, 1858 for urticaria, 1695 Evoked potentials, 2344-2345 auditory, 2344, 2345t, 2594 in multiple sclerosis, 2475 somatosensory, 2345 visual, 2344-2345, 2344f Evoxac (cevimeline), 1302 Ewing’s sarcoma, 1371 Ex vivo gene therapy, 211 Exanthem subitum (roseola), 2672-2673 Exanthematic typhus, 2051 Exanthematous pustulosis, acute generalized, 2681, 2681f, 2685t, 2686 Exanthems, 2241t, 2242 HIV seroconversion, 2318 macular, 2671-2675, 2671t maculopapular, 2685t papular, 2671-2675, 2671t Excitation-contraction coupling, 263.e1f Excoriations, 2651t-2654t Executive dysfunction, 2386-2388, 2510 clinical manifestations of, 2387 definition of, 2386 diagnosis of, 2387-2388 pathobiology of, 2387 treatment of, 2388b Exemestane (Aromasin), 1211t-1216t, 1358-1359 Exercise, 1461 for atherosclerotic PAD, 501 cardiovascular responses to, 266-267 definition of, 58 for heart disease, 416t, 417 for heart failure, 315, 315t for osteoarthritis, 1748 for osteoporosis, 1642 for pain, 142 Exercise capacity, 305 Exercise electrocardiography in angina pectoris, 423-425 in coronary artery disease, 424t Exercise prescription, 60 Exercise testing in arrhythmia, 350 cardiopulmonary tests, 544 for congenital heart disease, 416-417 for hypoglycemia, 1555 in interstitial lung disease, 579 preoperative, 2612, 2617t protocols for, 250t Exercise training, 306.e1t-306.e2t Exercise-dependent anaphylaxis, 1699 Exertional rhabdomyolysis, 723-724, 724t Exhaustion, heat, 692 Exjade (deferasirox), 1421 Exocrine pancreatic insufficiency, 966, 966t Exons, 189 Exophthalmos, 2560f Exostoses, 1672 Exotic snake bite, 720 Exotropia, 2558 Expansile osteolysis, familial, 1665t Expansile skeletal hyperplasia, 1665t Expedited partner therapy, 1945 Expiratory pressure, maximal (Pemax or MEP), 655 Expiratory reserve volume, 540, 542f Exploding head syndrome, 2423, 2423t Exposure assessment, 79-80

Exposure history, 79 Extavia (interferon β1b), 2477 External carotid artery, 2425 External genitalia abnormal differentiation of, 1563, 1563f ambiguous, 1563, 1563f External iliac artery dysplasia, 505, 505f External ophthalmoplegia, chronic progressive, 2546 External otitis, malignant, 1967 Extinctions, 1840 Extra-axial tumors, 1291-1292 Extracardiac obstructive shock, 674 definition of, 678f-679f etiology of, 678f-679f therapy for, 678f-679f Extracellular fluid, 741-742, 741f Extracellular fluid volume, 737-738 Extracellular matrix, 1730-1731 cellular interactions, 1731-1732 major components, 1731 networks, 1732 repair of, 234, 234f Extracellular pH, 740 Extracellular volume depletion, 772 Extracorporeal photopheresis, 2670t Extracorporeal removal, 706 Extracorporeal shock wave lithotripsy for gallstones, 1040-1041 for kidney stones, 814 Extracorporeal ultrafiltration, 749 Extracorpuscular hemolytic anemia, 1079-1080 Extracranial carotid artery stenosis, 2445 Extracutaneous mastocytomas, 1709 Extradural tumors, 1295-1296 clinical manifestations of, 1295-1296 diagnosis of, 1296 differential diagnosis of, 1296 epidemiology of, 1295-1296 treatment of, 1296b Extragonadal germ cell cancer syndrome, 1381 Extraintestinal amebiasis, 2140, 2141f Extramedullary tumors, intradural, 1296 Extranodal marginal zone MALT lymphoma, 1261t, 1263 Extranodal natural killer/T-cell lymphoma, 1266 Extra-ovarian serous carcinoma, 1380 Extrapulmonary aspergillosis, 2083 Extremities. See also Foot (feet) examination of in heart failure, 303 for signs of cardiovascular disease, 254-255 Extrinsic allergic alveolitis, 2084-2085 Exudates, 632-634 pleural causes of, 635 fluid characteristics of, 633, 633t Eye(s) abnormalities of acute, 2558-2560 vascular, 2569-2570 in acute poisoning, 697 anatomy of, 2556, 2557f dry, 2562-2563, 2564t, 2570 examination of, 26 foreign body sensation in, 2559 tumors of, 2571-2572 Eye disorders age-related, 2566-2568 autoimmune, 2570-2571 dry eyes, 1759, 2562-2563, 2564t, 2570, 2570t genetic diseases that may become symptomatic during adulthood, 2571 infectious, 2564-2566 inflammatory, 2563-2564, 2570-2571 Nairobi eye or rove beetle dermatitis, 2176 pediatric or adolescent diseases that persist into adulthood, 2571 red eye, 2558, 2559t rheumatoid arthritis, 1758t in sickle cell anemia, 1101 structural, 2566-2568 thyroid disease, 1506 vaccinia autoinoculation, 2217, 2217f visual loss due to, 2574-2575

Index Eye infections antimicrobial therapy for, 2565t Candida, 2080, 2081f, 2082 Pseudomonas, 1964 Eye inflammation, 2559t Eye movements examination of, 2577 spontaneous, 2412, 2412t Wernicke’s encephalopathy, 2507.e1 Eye pain, 2558, 2559t Eye worms, 2170, 2170f Eyelid(s), 2556 abnormalities of, 2560-2562 abscess in, 2564, 2564f benign neoplasms of, 2561 hemangioma of, 2571 ptosis of, 2558, 2560, 2560f squamous cell carcinoma of, 2562 Eyelid twitching, 2559 Ezetimibe, 1396, 1396t

F

F cells, 1100f F wave, 2342 F-75 formula, 1436, 1437t F-100 formula, 1436, 1437t FAB L3, 1243-1244 FABERE test (for Flexion, ABduction, External Rotation, and Extension), 1765-1766 Fabry’s disease, 791, 1387, 1399, 1402 arthritic manifestations of, 1827 cardiac involvement in, 325.e5f characteristic signs of, 1385t clinical features of, 324, 1401f, 1402 diagnosis of, 1402 diagnostic features, 1401t enzyme replacement therapy for, 1388, 1402, 1402t management of, 325 ocular manifestations of, 2573 pathobiology of, 1402 stroke in, 2443-2444, 2443t treatment of, 1388t, 1402b X-linked, 2443-2444 Face examination of, 26 skin lesions on, 2702t Facet arthropathy, 142t Facial lipoatrophy, HIV-associated, 2322, 2322f Facial nerve tumors, 2537 Facial paralysis, 2537 Facioscapulohumeral muscular dystrophy, 2543 clinical manifestations of, 2543 diagnosis of, 2543 prognosis for, 2543 treatment of, 2543b Factitial dermatitis, 2653f Factitious diarrhea, 933-934 Factitious disorder, 2355t Factor I (fibrinogen), 243t Factor I (fibrinogen) deficiency, 1175t Factor II (prothrombin) deficiency, 1175t, 1179-1180 Factor V (labile factor) deficiency, 1175t, 1179 acquired, 1179 combined with factor VIII deficiency, 1179 congenital, 1179 Factor V Leiden mutation (activated protein C resistance), 1186, 1186t Factor VII, recombinant activated, 2452, 2452t Factor VII deficiency, 1175t, 1179-1180 Factor VIIa, recombinant, 1175t, 1176, 1177b, 1180 Factor VIII (antihemophilic factor), 1173 Factor VIII (antihemophilic factor) alloantibodies, 1176 Factor VIII (antihemophilic factor) deficiency, 1175t, 1179 Factor VIII (antihemophilic factor) replacement therapy, 1174-1176 Factor IX (antihemophilic factor B, Christmas factor), 1173 Factor IX (antihemophilic factor B, Christmas factor) alloantibodies, 1176

Factor IX (antihemophilic factor B, Christmas factor) deficiency, 1175t Factor IX (antihemophilic factor B, Christmas factor) replacement therapy, 1174-1175 Factor X (Stuart-Prower factor) deficiency, 1175t, 1179-1180 Factor Xa inhibitors, 517 for acute STEMI, 450 comparison with warfarin, 179t direct, 178 pharmacokinetic characteristics and drug interactions, 178t side effects of, 517 Factor XI deficiency, 1175t, 1177-1178 clinical manifestations of, 1178 diagnosis of, 1178 epidemiology of, 1177-1178 pathobiology of, 1178 treatment of, 1178b Factor XII (Hageman factor) deficeincy, 1175t Factor XIII (fibrin stabilizing factor) deficiency, 1175t, 1178 clinical manifestations of, 1178 diagnosis of, 1178 treatment of, 1179b Factor concentrate-transmitted viral infections, 1176 Factor deficiency, 1178 Factor H, 243t Faget’s sign, 2250 Fainting, 2600t Falasha Jews, 1133 Fallopian tube cancer, 1363-1364 Falls, 110 Falls assessment, 104-105, 110, 110t False-positive rates, 33 Famciclovir for AIDS-associated opportunistic infections, 2297t-2301t for herpes, 2181t, 2226 for herpes zoster, 2680 mechanisms of excretion and thresholds for dose adjustment, 2182t for varicella, 2228 for viral meningitis, 2492 Familial adenomatous polyposis, 187t, 1227t, 1323, 1323f clinical manifestations of, 1323 diagnosis of, 1323 general features of, 1323t pathobiology of, 1323 treatment of, 1323b Familial aggregation, 196 Familial amyloid polyneuropathy, 1033.e1t Familial benign hypercalcemia, 1652t, 1657 Familial cancer susceptibility syndromes, 1226, 1227t Familial cholestasis syndromes, 987-988 Familial cold autoinflammatory disease, 231 Familial cold autoinflammatory syndrome, 1740t-1741t, 1742 Familial cold urticaria, 1131, 1742 Familial combined hyperlipidemia, 1393 diagnosis of, 1393 epidemiology of, 1393 pathobiology of, 1393 Familial dysautonomia, 2519 Familial expansile osteolysis, 1665t Familial gastric carcinoma, 1227t Familial hemophagocytic lymphohistiocytosis syndromes, 1686t Familial hypercholesterolemia, 1393 clinical manifestations of, 1393 epidemiology of, 1393 incidence of, 1385t pathobiology of, 1393 treatment of, 1393b Familial hyperkalemic periodic paralysis, 762 Familial hypertriglyceridemia, 1393 Familial hypocalciuric hypercalcemia, 1657 Familial hypokalemic periodic paralysis, 762 Familial hypomagnesemia, 823t Familial hypophosphatemic rickets, 1386t Familial interstitial pneumonia, 575-576 Familial intrahepatic cholestasis, 987-988 Familial intrahepatic cholestasis 1 gene (FIC1), 987-988

Familial isolated pituitary adenoma syndrome, 1483-1484 Familial malignant melanoma, 1227t Familial Mediterranean fever, 1147t-1148t, 1148, 1739-1740, 1740t clinical features of, 1148, 1739, 1741t complications of, 1739-1740 definition of, 1739 diagnosis of, 1148, 1740 differential diagnosis of, 1740 epidemiology of, 1739 pathobiology of, 1148, 1739 treatment of, 1148b, 1741b Familial medullary thyroid carcinoma, 1662 clinical manifestations of, 1662-1663, 1662t prevention of, 1663 Familial myelodysplastic syndromes, 1233 Familial neutropenia, benign, 1133 Familial Paget’s disease of bone, early-onset, 1665t Familial paraganglioma, 1523 Familial pulmonary fibrosis, 575-576 Familial syndromes, 1661.e1. See also specific conditions Familial Wilms tumor, 1227t Families definition of, 10 of dying patients, 9-15 palliative care for, 10 Family education, 315 about heart failure, 315t Family evaluation, 325 Family history, 25 and cardiovascular disease, 259 malignant, 325 in neurologic examination, 2338 and rheumatic disease, 1717 Family planning, 1605, 1605t, 1606f Family screening for dilated cardiomyopathy, 330 for hemochromatosis, 1422 Family-based linkage analysis, 192 Family-based therapy, 1457 Famotidine, 2660t for acid peptic disease, 915 for dyspepsia, 895 for esophageal disorders, 900t for urticaria, 1695 Fampridine, 2478 Fanconi’s anemia, 1133t, 1134, 1167, 1227t, 1239 clinical features of, 1116-1117, 1116t diagnosis of, 1117 differential diagnosis of, 1119 genetics of, 1117 treatment of, 1119 Fanconi’s syndrome acquired, 796-797 diagnosis of, 797 Fansidar (sulfadoxine-pyrimethamine), 2111 Farnesoid X receptor, 1392 Farsightedness, 2558 Fas ligand, 223 Fascicular branch blocks, 270-271, 271f-272f, 271t Fasciculations, 2539 Fasciitis eosinophilic, 2694 palmar, 1826 plantar, 1754 Fasciola, 1015t-1017t, 2158 Fasciola gigantica, 2156 Fasciola hepatica, 2156 Fascioliasis, 2156-2157 clinical manifestations of, 2156-2157 epidemiology of, 2156 pathobiology of, 2156 treatment of, 2158b Fasciolopsis buski, 2157 Faslodex (fulvestrant), 1211t-1216t Fast imaging employing steadystate acquisition (FIESTA), 2150 Fasting, 1548-1549 Fasting diarrhea, large-volume, 1338 Fasting glucose levels, 1528-1529 risk for type 2 diabetes, 1537 targets, 1534, 1535t, 1536-1537

I39

Fasting tests gastroduodenal manometry, 884, 885f for hypoglycemia, 1554 72-hour, 1554, 1554t Fat dietary, 926, 926.e1f fecal fat tests, 925-926, 925t high-carbohydrate/low-fat diet, 1465 high-fat dat, 925-926 high-fat/low-carbohydrate diet, 1465 subcutaneous, 2634-2635 Fat atrophy, 2653f Fat embolism syndrome, 626, 626.e1f postoperative, 1832-1833 stroke in, 2435 symptoms and signs of, 1833 treatment of, 626b Fat malabsorption, 928-929 Fatigue, 302, 2340 alert fatigue, 49 chronic fatigue syndrome, 1817, 1821-1822, 1819.e1t in fibromyalgia, 1818-1819 management of approaches to, 11t in palliative care, 11t Fatigue fracture, 1754 Fat-soluble vitamins, 2509-2510 Fatty acid metabolism: disorders of, 2545 Fatty acid oxidation defects, 1385t hypoglycemia due to, 1552-1553, 1555 pathophysiologic mechanisms of, 1385t Fatty liver, 80, 1009-1010 acute, of pregnancy, 1622 nonalcoholic disease, 1021-1023 clinical manifestations of, 1021-1022 definition of, 1021 diagnosis of, 1022, 1022f epidemiology of, 1021 magnetic resonance findings, 1022f pathobiology of, 1021 prevention of, 1022 prognosis for, 1022-1023 risk factors for, 1020t treatment of, 1022b Favipiravir, 2196t Fc receptors (FcRs), 217-218, 226 for IgE (FcεRs), 226 for IgG (FcγRs), 226, 1146 Febrile illness approach to, 1850-1851 associated with recent travel, 1850, 1850t chronic, 1852-1853, 1852t Febrile neutropenia antibiotic regimens for, 689t P. aeruginosa infections during, 1964 Febrile neutrophilic dermatosis, 1859, 2682, 2692 Febrile nonhemolytic transfusion reactions, 1196 Febrile polyarthritis, epidemic, 2261 Febrile seizures genetic epilepsy with febrile seizures plus (GEFS+), 2404 prognosis after, 2409 Febuxostat, 1815 Fecal elastase-1 test, 925t Fecal fat tests, 925-926, 925t Fecal incontinence, 866, 970-971 clinical manifestations of, 971 diagnosis of, 971 epidemiology of, 970-971 pathobiology of, 970-971 treatment of, 971b, 971f, 2238-2239 Fecal lactoferrin, 1870t Fecal leukocyte test, 1870t FECH gene, 1410 Federally funded community health centers, 16 Fee splitting, 8 Feeding changes during, 1549 thermal effect of, 1433 Feeding disorders, 1455 FEIBA VH, 1176 Felbamate, 2407t Felodipine, 388t, 430t Felty’s syndrome, 1135, 1759 Female breast development, 61, 62f Female genital discharge, 1877t, 1879-1880 Volume I  pp 1-1382  •  Volume II  pp 1383-2722

I40

Index

Femara (letrozole), 1211t-1216t Feminization, testicular, 1564 Fenfluramine, 1453t Fenofibrate, 1396t Fenretinide, 1358-1359 Fentanyl, 139t-140t, 699t-702t, 2368 Fenthion, 699t-702t Ferning, 1585 Ferric carboxymaltose, 316 Ferric hexacyanoferrate (II) (Prussian blue), 85 Ferritin, 1063-1064, 1064f, 1070 Ferrous fumarate, 1070 Ferrous gluconate, 1070 Ferrous sulfate, 1070, 2424, 2468-2469 Fertility, 1595 Fertility awareness-based contraception, 1605t, 1606f Ferumoxytol, 840 Fesoterodine, 113t, 830t Fetal alcohol syndrome, 2512 Fetal circulation, 2435 Fetal hemoglobin: hereditary persistence of classification of, 1089t clinical manifestations of, 1093 molecular basis, 1090 Fetal monitoring, 1621 Fetal testicular regression syndrome, 1565 Fetal tissue: engraftment of, 205-206 Fetor hepaticus, 977 Fetus development of, 1610, 1611f maternal conditions affecting, 1564 Fever, 1849-1854 antipsychotic-induced, 2626t in bacterial infection, 1849, 1849f, 2485 Brazilian purpuric fever, 1947 cancer-related, 1221 in chemotherapy-induced neutropenia, 1856, 1856t in compromised patients, 1854-1861 definition of, 1849 dengue, 1884 drug-induced, 1853, 1853t enteric clinical manifestations of, 1973 treatment of, 1974 familial Mediterranean fever, 1739-1740, 1740t filarial, 2167 with hepatosplenomegaly, 1852 hereditary syndromes, 1739-1742, 1740t-1741t in HIV infection, 2303t hospital-associated, 1850, 1851t in immunocompromised patients, 1856-1858 in inpatients, 1850-1851 with jaundice, 1852, 1852t with lymphadenopathy, 1852, 1852t Mediterranean spotted fever, 2048t in meningitis, 2485 with musculoskeletal complaints, 1851-1852 in neutropenia diagnostic considerations, 1856-1857 evidence-based guidelines for, 1858 management of, 1857-1858, 1857f Oroya fever, 1997-1998 in outpatients, 1850 pathobiology of, 1849-1850, 1849f periodic with aphthous stomatitis, pharyngitis, and cervical adenopathy, 1740t with hyperimmunoglobulinemia D, 1740t-1741t, 1743 pediatric syndromes, 1713t Pontiac fever, 1993 postoperative, 2623 preferred medications for women, 1602t Q fever, 477t, 2055-2056 with rash, 1851, 1851t with rash involving palms and soles, 1851, 1851t recurrent, 1739-1742, 1740t-1741t relapsing, 2027-2028, 2051, 2057t distinguishing characteristics of, 2252t-2253t louse-borne, 2060t rheumatic, acute, 1908-1909 Rocky Mountain spotted fever, 2047-2050 scarlet fever, 1907, 2671

Fever (Continued) spotted fever group rickettsiae, 2252t-2253t tick-transmitted spotted fevers, 2048t travel-associated, 1850, 1850t trench fever, 1997-1998 of unknown origin, 1150 approach to, 1852-1853 causes of, 1852-1853, 1852t viral hemorrhagic, 87t, 90-91, 2247-2256 West Nile fever encephalitis, 2057t in women, 1602t Fexofenadine (Allegra), 2660t for allergic rhinitis, 2588 for pruritus, 2636t for urticaria, 1695, 2684t Fiber, 889 Fibrates, 1396, 1396t Fibrillary glomerulopathy–immunotactoid glomerulopathy, 793 Fibrin stabilizing factor (factor XIII) deficiency, 1175t, 1178 Fibrinogen concentrate, 2254 Fibrinogen (factor I) deficiency, 1175t Fibrinolytic agents, 180 characteristics of, 447t clinical applications, 180 contraindications to, 447t, 624, 624t indications for, 447t for PE, 624, 624t for STEMI, 446t, 447 Fibroblast growth factor receptor gene (FGFR), 1305-1306 Fibroblast growth factor-23, 776 Fibroblasts, 2634 Fibroelastomas, 339 Fibrogenesis imperfecta ossium, 1669 Fibromatosis, mesenteric, 950 Fibromuscular dysplasia, 505, 505f clinical manifestations of, 505, 807, 807f diagnosis of, 505 prognosis for, 809 “string of beads” lesion of, 385-386, 386f, 505, 505f treatment of, 386, 505b, 808 Fibromyalgia, 1817-1821 associated problems, 1818 classification of, 1713t, 1818 clinical manifestations of, 1817-1818 diagnosis of, 1818 diagnostic criteria for, 1819f education about, 1818 epidemiology of, 1817 history of, 1818 initiation of, 1818 maintenance of, 1818 pathobiology of, 1817 prognosis for, 1820-1821 self-reported symptoms, 1817, 1818t tender points, 1818 treatment of, 1818b-1820b, 1820f Fibronectin, 2634 Fibrosing alveolitis, cryptogenic, 2237t Fibrosing dermatopathy, nephrogenic, 2694, 2694f Fibrosing mediastinitis (chronic mediastinitis), 637 Fibrosis effector cells in, 1781 in systemic sclerosis, 1781 Fibrous dysplasia, 1671-1672, 1672b, 1672f FICA Spiritual Assessment Tool, 10, 14t Fick equation, 654, 1060-1061, 1060f Fidaxomicin, 1925 FIESTA (fast imaging employing steadystate acquisition), 2150 Fifth disease, 2212, 2213f, 2214t Fight-or-flight response, 2351 Figurative erythemas, 2671 Filarial fever, 2167 Filarial parasites, 2166, 2167t Filariases, 2166-2171 definition of, 2166-2167 lymphatic, 2165t, 2167-2168, 2168f vectors, 2172t zoonotic, 2171 Filgrastim (Neupogen), 1211t-1216t Financial assessment, 105, 106t Financial conflicts of interest, 8-9 definition of, 8 empirical data, 8-9 gifts to physicians, 9

Financial conflicts of interest (Continued) history of concern about, 8 justification for concern about, 8 practical considerations, 9 regulation of, 9 Finasteride, 830t, 2704 Fine-needle aspiration, diagnostic, 1307-1308 Fine-needle aspiration biopsy, 1511 Finger flexion reflex, 2372t Fingolimod (Gilenya), 2477 Finland, 603, 1529-1530 FIP1L1-PDGFRA fusion gene, 336, 1151, 1153 FIP1L1-PDGFRA-positive disease prognosis for, 1154 treatment of, 336b, 1154 Firazyr (icatibant), 1697 Fire, 598 Fire sponges (Tedania), 722 Firmagon (degarelix), 1211t-1216t Fish, venomous, 721 Fish oils, 1396, 1396t Fish tank granuloma, 2041 Fish tapeworm (Diphyllobothrium latum), 1107, 2148 Fissures, 2636t, 2651t-2654t Fistulas anal, 969 aortoenteric, 958 arteriovenous, dural, 2453 biliary, 1046 cerebrospinal fluid, 2485 Crohn’s disease-associated, 940-941, 969, 969f perianal, 941 perilymphatic, 2595 pulmonary arteriovenous, 411 tracheoesophageal, 908 Fitness immunological, 1837 physical, health-related, 58 5′-NT, 989 5 As, 147 5-Lipoxygenase-activating protein (FLAP), 232 Fixed acid, 763 Fixed airway obstruction, 544 Flaccid paralysis, acute, 2241, 2241t Flagyl (metronidazole), 2137 Flail chest, 630-631, 631b, 630.e1f Flashing lights, 2559 Flatbush diabetes, 1534, 1539 Flatus, 859-860, 860b Flea-borne spotted fever, 2048t, 2051 Fleas, 2176 Flea-transmitted diseases, 2048t, 2051 Flecainide for atrial fibrillation, 365-366 doses and side effects of, 363t-364t for PVCs, 372-373 Fletcher media, 2030 Flexibility activity, 58-59 Flexible sigmoidoscopy applications, 873t in infectious diarrhea, 1869-1870, 1870t Flexor tenosynovitis, volar, 1752-1753 Flies, 2176 Flinders Island spotted fever, 2048t, 2050 Floaters, 2556, 2559 Flock-worker’s lung, 589t Floppy baby syndrome, 1927-1928 Florone (diflorasone diacetate), 2658t Flovent (fluticasone), 553t Flow cytometry, 1146 Flower cells, 2236-2238, 2237f Flow-volume curves, 539-540, 541f Flt3, 1050 Flt3 ligand, 1050 Flu syndrome, 2057t Fluconazole, 2067-2068 adverse effects of, 2067 for AIDS-associated opportunistic infections, 2297t-2301t for bacterial endocarditis, 479t for blastomycosis, 2074-2075 for candidemia, 2082 for candidiasis, 1013, 2067, 2082, 2297t-2301t, 2582 for cryptococcal meningitis, 2067, 2077 for cryptococcosis, 2077, 2297t-2301t drug interactions, 2067

Fluconazole (Continued) formulations, 2067 indications for, 2067-2068 for mycoses, 2068 pharmacology of, 2067 for prophylaxis in AIDS, 2068 for prophylaxis in neutropenia, 2068 for sporotrichosis, 2078-2079 for thrush, 2082, 2604 Flucytosine, 2065-2066 administration of, 2066 for AIDS-associated opportunistic infections, 2297t-2301t for chromomycosis, 2103 for cryptococcal meningitis, 2077 dosage, 2066 formulation of, 2065-2066 mechanism of action, 2066 pharmacology of, 2065-2066 resistance to, 2066 Fludarabine (Fludara) for cancer, 1211t-1216t for chronic lymphocytic leukemia, 1255 for non-Hodgkin’s lymphoma, 1263, 1263t Fludrocortisone for adrenal hypoplasia, 1567 for adrenal insufficiency, 1521 for hypertension in older adults, 392 for orthostatic intolerance, 2521 for septic shock, 690 for syncope, 347 Fluid compartments, 741-742, 741f Fluid overload, postoperative, 2622t Fluid restriction, 814, 1030 Fluid therapy for burns, 714-715, 714.e1t for cholera, 1952 for dengue shock syndrome, 2254 for diabetic ketoacidosis, 1540, 1541f for diarrhea, 921 for dysautonomia, 2369 for infectious diarrhea, 1870t for intracranial hypertension, 2452t for painful episodes, 1102-1103 for resuscitation, 650 for rhabdomyolysis, 726 for shock, 676-677, 680t, 689 strategies for, 677 for viral hemorrhagic fever, 2254 Fluid transport, 918-919 Fluid volume, 737-738, 742, 742t Fluid-attenuated inversion recovery (FLAIR), 2404 Flukes, 2104, 2155-2159 intestinal, 2156t, 2157 diagnosis of, 2158 prevention of, 2159 treatment of, 2158b liver, 2156-2157, 2156t diagnosis of, 2158 prevention of, 2159 treatment of, 2158b pulmonary, 2156t, 2157-2158 diagnosis of, 2158 prevention of, 2159 treatment of, 2158b Flumazenil, 706t-710t, 2412 Flumethasone pivalate, 2658t Flunisolide (AeroBid/AeroBid-M, Nasarel), 553t, 1692, 1692t Fluocinolone acetonide, 2658t Fluocinonide for mucous membrane pemphigoid, 2583, 2676 for oral mucosal lesions of lupus, 2583 for pemphigus, 2677 potency, 2658t Fluorescence imaging, 1647, 1648f Fluorescent treponemal antibody absorption (FTA-ABS) test, 2017, 2017t Fluorine, 1450t-1452t Fluorodeoxyglucose (FDG), 1309 Fluoropyrimidine, 1319 Fluoroquinolones for bacterial meningitis, 2488t for community-acquired pneumonia, 1896 for gonorrhea, 1896 resistance to, 1961 for S. pneumoniae infection, 1905t for UTIs, 1896 Fluoroscopy, 867-868, 867f

Index 5-Fluorouracil (Adrucil) for actinic keratoses, 2687-2688 for cancer, 1211t-1216t, 1302, 1319, 1330-1331 cardiomyopathy due to, 330 ocular effects of, 2573t for rectal adenocarcinoma, 1330-1331 topical, 2660 Fluoxetine (Prozac) for cataplexy, 2419t for congenital myasthenic syndromes, 2553 for depression, 2349t for irritable bowel syndrome, 893t for menopausal hot flushes, 1628t-1629t for premenstrual syndrome, 1587 for prevention of tension-type headache, 2360 Fluoxymesterone, 1128 Fluphenazine for chorea, 2463 for psychosis, 2354 for Tourette’s syndrome, 2466 Flurazepam, 2352t Flutamide (Eulexin), 1211t-1216t, 1369 Fluticasone (Flonase, Flovent, Veramyst), 1692t for allergic rhinitis, 2588 for asthma, 553t, 554 for chronic sinusitis, 1692, 2588 for COPD, 560t for eosinophilic esophagitis, 902 potency, 2658t Fluvastatin, 1396t Fluvoxamine, 2636t Focal cortical dysplasia, 2512 Focal ischemic injury, 2432-2433 Focal motor neuropathy, compressive, 2524t Focal nodular hyperplasia, hepatic, 981, 982f Focal segmental glomerulosclerosis, 786, 786f collapsing variant, 786 tip lesion variant, 786 treatment of, 786b Focal seizures, 2401-2402, 2402f Focal status epilepticus, 2401-2402, 2402f FODMAPs (fermentable oligosaccharides, disaccharides, and monosaccharides and polypols), 860, 891, 893-894 Folate, 1106, 1109t, 1407 absorption of, 1107-1108 cobalamin interactions, 1108 decreased intake of, 1108 for folate deficiency, 1113, 2507t functions of, 1446t-1450t for IBD in pregnancy, 943 increased requirements for, 1108 intracellular metabolism of, 1108 newly identified role, 1454t normal physiology of, 1107-1108 nutritional requirements, 1453-1454 prophylaxis with, 1113 indications for, 1113.e1t in resource-limited settings, 1113 recommended daily allowance for, 1454 serum levels, 1111, 1111t, 1113, 1108.e1t supplementation effects, 1454 transport of, 1107-1108 Folate (vitamin B9), 1407 Folate cycle, 1404-1405, 1404f Folate deficiency, 1385t-1386t, 2507t, 2508-2509 classic syndrome, 1446t-1450t classification of, 1105t clinical manifestations of, 1108 conditions predisposing to, 1105t diagnosis of, 1111t, 1112 drug-induced, 1108 in infancy, 1108 laboratory findings, 1110-1111 masking of, 1113 nutritional causes, 1108 pathogenesis of, 1108 in pregnancy, 1108 treatment of, 1113 Folate metabolism inhibitors, 2111 Folex (methotrexate), 1211t-1216t Folic acid. See also Folate contraindications to, 1113 drug dosage, 1113

Folic acid (Continued) for folate deficiency, 1113 fortification of food with, 1113 for hereditary spherocytosis, 1083 indications for, 1113.e1t for malabsorption, 931t prophylactic, 1113, 1113.e1t recommendations for, 1453 for thalassemias, 1094 for tropical sprue, 931, 1108 Folic acid deficiency, 1385t Folic acid supplements, 1109t Folinic acid for CBS deficiency, 1407 for chemotherapy side effects, 1211t-1216t for congenital toxoplasmosis, 2132t for drug overdose, 699t-702t, 706t-710t for MTHFR deficiency, 1407 for toxoplasmic retinitis, 2566 for toxoplasmosis, 2129t, 2132t Follicle(s) luteinized unruptured follicle syndrome, 1594 selection of, 1585-1586 Follicle-stimulating hormone, 1492-1493 increased levels, 1589 low levels, 1589 mechanism of action, 1585 Follicle-stimulating hormone–producing tumors, 1493 Follicular helper T cells (TFH), 227 Follicular lymphoma, 1263-1264 advanced disease, 1264 clinical characteristics of, 1261t localized disease, 1264 salvage therapy for, 1264 Follicular Lymphoma International Prognostic Index (FLIPI), 1262 Follicular (epithelial) thyroid carcinoma, 1512-1513 Folliculitis, 1898, 2681, 2696 eosinophilic, 2321, 2321f eosinophilic pustular, 2681 hot tub, 2697 Pityrosporum, 2681 Pseudomonas, 2681, 2681f, 2696-2697 Staphylococcal, 2319, 2319f Folliculitis decalvans, 2703t, 2706, 2706f Folliculogenesis, 1585-1586 Fomepizole, 699t-702t, 706t-710t Fomivirsen, 2181t, 2183 Fondaparinux for acute coronary syndrome, 438t-439t, 439 for acute STEMI, 450 for anticoagulation, 516t for deep venous thrombosis, 516 in pregnancy, 1618t for pulmonary embolism, 624, 625t for superficial thrombophlebitis, 518 for venous thromboembolism, 517-518 Fontaine’s clinical classification, 499t Fontan, 416t Food contaminated, 1972 folic acid fortification of, 1113 and obesity, 1459 precautions for, 1884 ready-to-use therapeutic food (RUTF), 1436, 1437t thermic effects of, 1461 Food allergens, 1699 Food allergy, 934, 1675, 1698-1703 Food allergy-associated exercise-dependent anaphylaxis, 1699 Food and Drug Administration (FDA), 181, 1838 Adverse Event Reporting System (AERS), 132 Animal Efficacy Rule, 1988 approved NAATs, 1941 biomarkers, 203.e2t recommendations for domperidone, 887-888 recommendations for metoclopramide, 887 Food faddism, 1109-1110 Food impaction, 876, 876f

Food intake alterations in, 1479 biologic modulators of, 1460, 1460t CNS modulators of, 1460, 1460t Food poisoning, 920, 1897 Food protein, 1107 Food-borne infectious diarrhea, 919-920, 919t Food-induced anaphylaxis, 1699 Foot (feet) Kaposi’s sarcoma involving, 2325, 2325f Madura foot, 2099 orthopedic procedures on, 1832 rheumatoid arthritis in, 1757 rocker bottom, 1824 skin lesions in, 2702t Foot disorders, 1754 Foot surgery, 2618t Foot-and-mouth disease, 2058t Foramen ovale, patent, 409 Forced expiratory flow (FEF), 539 Forced expiratory volume (FEV), 539 in 1 second (FEV1), 555 annual reduction in, 555, 555f and severity of asthmatic attacks, 550, 550t FEV1/FVC ratio, 539 Forced vital capacity (FVC), 539 Foreign bodies, esophageal, 907-908 Foreign body sensation, 2559 Foreign body–related infections, 2102 Foreign non–IgE-dependent agonists, 1699-1700 Forkhead Box Protein 3 (FoxP3), 239-240 Formivirsen (Vitravene), 210 Formoterol (Foradil), 553, 560t Formularies, 1848 Fortesta (testosterone), 1574t Fortified milk-based formulas, 1436 Fosamprenavir, 2288t, 2290t, 2331t Fosaprepitant, 865t Foscarnet, 2183 adverse effects of, 2182t for AIDS-associated opportunistic infections, 2297t-2301t for cytomegalovirus infection, 2209b, 2231, 2232t for herpes, 2181t, 2297t-2301t mechanisms of excretion and thresholds for dose adjustment, 2182t for varicella zoster virus infection, 2229, 2297t-2301t Fosfomycin, 1875t, 1915.e1t Fosinopril, 388t Fosphenytoin for seizures, 2408, 2452t for status epilepticus, 2408 for subarachnoid hemorrhage, 2448t Fossa of Rosenmüller, 2587 Fotolyn (pralatrexate), 1211t-1216t Founder effects, 1083-1084 FOUR Score tool, 2411, 2411t 4CMenB, 1939 Fovea, 2573 Foville’s syndrome, 2435.e1f Fractional excretion of magnesium (FEMg), 775 Fractional excretion of phosphate (FEPO4), 777 Fractional excretion of sodium (FENa), 733, 781, 781t Fractional excretion of urea (FEurea), 781 Fractional flow reserve (FFR), 295, 458 Fractures bone, 1219 Colles, 1637 early fixation of, 715 fatigue fracture, 1754 hip, 1637, 1638f march, 1754 Milkman’s, 1646 osteoporotic, 1640-1641 radiologic findings, 1641, 1641f reduction of, 1643t risk of, 1637, 1638f pseudofractures, 1646, 1647f stress, 1754 transverse, of temporal bone, 2599 vertebral, 142t risk of, 1637, 1638f treatment of, 142t

I41

Fractures (Continued) vertebral compression, 1641, 1641f wrist, 1637, 1638f Fragile histidine triad gene (FHIT), 1306 Fragile X ataxia syndrome, 2514 Fragile X syndrome, 2514 Fragmentation injury, 712t, 713, 713f Frailty, 101-102, 109 clinical care for, 109, 109f disease trajectories, 10.e1f models of, 36 screening for, 109, 109t Frameshift mutations, 194 Framingham equations, 259, 259.e1t-259.e2t Francis, Edward, 1982 Francisella tularensis, 1982, 2603 Frank-Starling law, 266 Frank-Starling mechanism, 266-267 FRAX (WHO), 1641-1642 Free light chains, serum, 1274 Free radical scavenging vitamins, 1454 Fresh frozen plasma (FFP) for angioedeme prophylaxis, 1697 for coagulopathy, 1184 for factor XI deficiency, 1178 psoralen-treated, 1179 for viral hemorrhagic fever, 2254 Fridericia formula, 269 Friedreich’s ataxia, 325, 2469, 2469b Frontal brain trauma, 2387, 2387f Frontoparietal polymicrogyria, bilateral, 2513 Frontotemporal dementia behavior-variant, 2396, 2397t genes that predispose to, 1665t MRI findings, 2397, 2398f Frontotemporal lobar degeneration, 2396-2398 clinical manifestations of, 2397 definition of, 2396 diagnosis of, 2397, 2398f epidemiology of, 2396 MRI findings, 2397, 2398f pathobiology of, 2396-2397 prognosis for, 2398 treatment of, 2398b Frostbite, 510, 511f, 694-695 Frostnip, 694 Frovatriptan, 2358-2359, 2359t Frozen shoulder, 1752 Fructose diarrhea, 933 Fructose intolerance, hereditary, 823t Fucosidosis, 1387 Fukushima, 82-83, 83t Fulvestrant (Faslodex), 1211t-1216t, 1357 Fumagillin, 2147t Functional assessment, 116 in elderly, 102-103, 106t in rheumatic disease, 1717 Functional dysautonomias, 2520 Functional endoscopic imaging, 878-879, 878.e1f Functional impairment, 116 Functional neurologic symptom disorder, 2355t Functional outlet obstruction, 889 Functional residual capacity (FRC), 540, 542f Fungal arthritis, 1808 Fungal endocarditis, 476 noncandidal, 477t treatment of, 479t, 480 Fungal endophthalmitis, 2566 Fungal hyphae, linear, 2656f Fungal infections. See also specific infections blood smear findings, 1059t dematiaceous, 2101-2103, 2102t, 2103b health care–associated, 1865 in HIV infection, 2320 in immunocompromised patients, 1858, 1859t, 1860 of liver, 1012-1013, 1018t predisposition to, 1687t of skin, 2693 skin diseases, 2665-2671 Fungal meningitis, 2342t Fungal pericarditis, 491 Fungal pharyngitis, 2604 Fungal pneumonia, 2309t, 2314 Fungal sinusitis, 1691, 1691t, 2102 Funguria, 1875-1876

Volume I  pp 1-1382  •  Volume II  pp 1383-2722

I42

Index

Fungus balls, 612, 2588, 612.e1f Furazolidone, 915t Furosemide, 748t for amyloidosis, 335 for ascites, 1030 for chronic kidney disease, 838-839 for hypercalcemia, 1654 for hyperkalemia, 761 for hypermagnesemia, 776 for hypertension, 388t Furuncles, 1898, 2696, 2696f Furuncular myiasis, 2176 Fusiform aneurysms, 2448 Fusobacterium, 1931 Fusobacterium necrophorum, 1934t Fusobacterium nucleatum, 1931, 1934t

G

G protein–coupled receptor gain-of-function mutations, 1473, 1473t G protein–coupled receptor loss-of-function mutations, 1473, 1473t Gabapentin characteristics of, 2408t for chronic pain, 136-137, 138t for cough, 527-528 for headache, 2361 for hot flashes, 1627 for hot flushes, 1628t-1629t for painful dysesthesias and paroxysmal dystonic spasms, 2477 for post-herpetic neuralgia, 2680 for pruritus, 2636t for restless legs syndrome, 2424, 2468-2469 for seizures, 2407t-2408t for tremor, 2462 for trigeminal neuralgia, 2363 for zoster, 2229 Gadolinium adverse effects of, 1728 contraindications to, 807-808, 871 magnetic resonance angiography with, 1728 toxicity, 500 Gadolinium diethylenetriamine-penta-acetic acid (Gd-DTPA), 1726-1728 Gaenslen maneuver, 1765-1766 Gait abnormal, 2340 antalgic, 2341t dystonic, 2341t hemiparetic, 2341t hysterical, 2341t parkinsonian, 2341t spastic, 2341t steppage, 2341t unsteadiness of, 2340 waddling, 1636, 2341t Gait disorders, 2341t Galactomannan, 2084 Galactosemia, 1591 Galantamine, 2394 Galectin-3, 304, 304.e1t Gallavardin’s phenomenon, 463-464 Gallbladder adenocarcinoma, 1339, 1345 Gallbladder cancer, 1042, 1042b Gallbladder disease, 249t, 1038-1042 Gallbladder ejection fraction, low, 1041, 1041t Gallbladder tumors, 1041-1042 benign, 1041-1042 malignant, 1042 Gallstone pancreatitis, 960, 960t acute, 1043-1044, 1043f Gallstones, 1038-1041 clinical manifestations of, 1039 complications of, 1038-1039 CT findings, 1040, 1043f diagnosis of, 1039-1040, 1039f-1040f epidemiology of, 1038 formation of, 1038 oral dissolution therapy for, 1040-1041 pathobiology of, 1038-1039 prevalence of, 1038 prevention of, 1041 radiographic findings, 1040, 1040f risk factors for, 1039t in sickle cell anemia, 1101 silent, 1040 symptomatic, 1040-1041

Gallstones (Continued) treatment of, 1040b-1041b ultrasound findings, 1039-1040, 1039f Galsulfase (Naglazyme), 1734 Gamekeeper’s thumb (skier’s thumb), 1753 Gamma globulin, intravenous for Kawasaki disease, 2672 for SLE, 1776 Gamma rays, 82, 83f Gamma-glutamyltransferase, 989 Ganciclovir, 2182-2183 adverse effects of, 2182t for AIDS-associated opportunistic infections, 2297t-2301t for cytomegalovirus infection, 2209b, 2231, 2232t for herpes, 2181t for herpes zoster keratitis, 2565 mechanisms of excretion and thresholds for dose adjustment, 2182t for viral encephalitis, 2504 Ganetespib, 1310t Ganglia, wrist, 1752 Ganglioglioma, 1294 Ganglioneuroma, paraspinal, 538, 539f Ganglionopathies, autonomic peripheral, 2518-2522 Gangrene, gas, 1926-1927 Gantrisin (acetyl sulfisoxazole), 1088t Gap junction channels, 342 Gap junctions, 2633 Gardner syndrome, 1323-1324 Garrod, Archibald, 1384 Gas and bloating, 859-861 Gas exchange, 557 Gas gangrene, 1926-1927 Gastic ulcers, 1317, 1318f Gastric adenocarcinoma, 1316-1319 clinical manifestations of, 1317 conditions predisposing to or associated with, 1317t diagnosis of, 1317-1318, 1318f epidemiology of, 1316 familial, 1227t genetics of, 1317 laboratory findings, 1317 medical therapy for, 1319 pathobiology of, 1317 physical findings, 1317 prognosis for, 1319 risk factors for, 1316 surgical therapy for, 1318 symptoms and signs of, 1317 TNM staging of, 1318t treatment of, 1318b-1319b, 1319 Gastric analysis, 913 Gastric antral vascular ectasia (GAVE), 958 Gastric bypass anesthetic approaches for, 2618t postprandial hyperinsulinemic hypoglycemia after, 1552 Roux-en-Y, 1466 Gastric electrical stimulation, 888 Gastric emptying, 705 accelerated, 888 Gastric lymphoma clinical manifestations of, 1319 diagnosis of, 1319 epidemiology of, 1319 treatment of, 1319b Gastric masses, 1318, 1318f Gastric microbiome, 1842 Gastric motility, 884-885, 885f, 887 Gastric mucosa, heterotopic (inlet patch), 908 Gastric neoplasms, 1316-1320 Gastric outlet obstruction, 917 Gastric residual volume (GRV), 1439 Gastric ulcers, 908, 911f, 2368 Gastrinomas, 1335-1337, 1335t, 1553 Gastritis definition of, 908 erosive, 909f H. pylori, 909, 909f hypertrophic, 908 nodular, 908 prevalence of, 909 Gastroduodenal manometry, 884, 885f Gastroduodenal ulcers causes of, 909, 914 incidence of, 909

Gastroduodenal ulcers (Continued) malignant, 910-911 in systemic inflammatory disorders, 911 Gastroenteritis acute infectious, 2244 eosinophilic, 934 Gastroenteropathy diagnostic imaging procedures, 866-872 protein-losing, 934 Gastroesophageal junction obstruction, malignant, 877, 877f Gastroesophageal reflux disease, 897-902 clinical manifestations of, 898 definition of, 897 diagnosis of, 898 endoscopy in, 874, 874f epidemiology of, 897 Montreal classification of, 898f pathobiology of, 897-898 preferred medications for women, 1602t prognosis for, 900 treatment of, 898b-900b, 1602t in women, 1602t Gastrointestinal anthrax, 1921-1922 Gastrointestinal bleeding, 879-884 approach to, 852t, 863-866 in HIV infection, 2303t hypovolemia in, 744 lower gastrointestinal bleeding, 875-876, 875f, 882 obscure and occult, 882-884 definition and epidemiology of, 882-883, 883t diagnosis of, 883-884 management of, 883f occult, 866, 876 prophylaxis of, 715 small intestinal bleeding, 882-883, 883t upper gastrointestinal bleeding, 879-880 causes of, 879, 880t endoscopy in, 874-875 management of, 881f treatment of, 880b-881b Gastrointestinal complaints, 861-866 Gastrointestinal disease, 850-866, 852t, 2305 abdominal examination in, 851 acute radiation syndrome, 84 alcohol-related, 151t, 152 arthritis in, 1823t blood tests, 851-854 with CD4+ counts < 200 cells/µL, 2304-2305 with CD4+ counts > 200 to 500 cells/µL, 2302-2304 and CD4+ counts in HIV infection, 2302, 2303t chronic Chagas disease (megadisease), 2117 clinical history, 850 in cystic fibrosis, 564-566 and dietary requirements, 1453 family history, 851 functional disorders, 850, 890-896 definitions, 890 pathobiology of, 890 in HIV infection, 2302-2305, 2303t in immunocompromised patients, 1859t, 1860-1861 laboratory findings, 851-854 nonabdominal examination in, 851 in obesity, 1463 past medical history, 850 physical examination in, 851 social history, 850-851 stool examination in, 854 treatment of, 565-566 vascular, 951-959 Gastrointestinal drugs, 117t Gastrointestinal endoscopy, 872-879 complications of, 873, 873t future directions, 878-879 indications for, 873-877 preparation for, 873 procedures and applications, 872-873, 873t techniques, 878-879 Gastrointestinal infections adenoviral, 2198 Pseudomonas in, 1965 treatment of, 2246b Gastrointestinal motility disorders of, 884-890, 2518t in irritable bowel syndrome, 891

Gastrointestinal mucormycosis, 2088-2089 Gastrointestinal stromal tumors gastric, 1319 histopathology of, 1320, 1320f metastatic, 872f pathobiology of, 1320 small bowel, 1320, 1320f treatment of, 1372-1373 Gastrointestinal system age-related changes in, 107 alcohol-related complications, 151t, 152 motor function of, 884 neuroenteric control, 884 normal physiology of, 884-885 in SLE, 1773 Gastrointestinal tuberculosis, 2035 Gastrointestinal viruses, 2244-2247 Gastroparesis, 886-887 classification of, 885, 886t clinical manifestations of, 886 complications of, 887 diagnosis of, 886-887, 887f medical therapy for, 887-888 pathogenesis of, 887 surgical therapy for, 888 treatment of, 887b-888b Gastroschisis, 944 Gastrospirillum hominis, 912 Gastrostomy, percutaneous endoscopic, 877 GATA2 deficiency, 1686-1687, 1687t Gatifloxacin, 2064, 2488t, 2565t Gaucher cells, 1401f Gaucher’s disease, 587, 1387, 1400-1401 clinical manifestations of, 1400-1401, 1401f diagnosis of, 1401 diagnostic features, 1401t incidence of, 1385t pathobiology of, 1400 treatment of, 1401b enzyme replacement therapy, 1401, 1402t nutritional therapy, 1388 strategies for, 1388t type 1, 1399-1400 type 3, 1399-1400 Gaussian distribution, 32 Gaviscon, 900t Gaze conjugate, 2578-2579 ping-pong, 2412 Gaze palsy, 2577 Gaze-evoked nystagmus, 2579 GB virus C (GBV-C), 1176 Gefitinib (Iressa), 171, 1232t, 1310t Gelatinase granules, 1144, 1144t Gelatins, 680t Gemcitabine (Gemzar) for ampullary tumors, 1047 for cancer, 1211t-1216t, 1334 Gemfibrozil, 428, 1396t Gemifloxacin, 616t, 1905t Gemtuzumab ozogamicin, 1244-1245 Gender differences in antimicrobial therapy, 1845 in hypertension, 382 in infective endocarditis, 474 in rheumatic disease, 1717 Gender identity, 1566-1567 Gender role, 1566 Gene(s), 189. See also specific genes candidate, 190-191 common variants, 190 copy-number variants, 190-191, 193f, 194 definition of, 189 disease genes, 194 epistatic effects of, 196 germline variants, 202 mutations. See Mutations premutation, 194 repetitive elements, 191 single-nucleotide variants, 190, 194 structure of, 189, 190f tandem repeats, 191 types of, 189-190 Gene editing, 213 Gene products, 818 Gene therapy, 210-213 classification of, 210-211 cytotoxic effects of, direct, 212 delivery methods, 211 diseases treated by, 211-213

Index Gene therapy (Continued) ex vivo, 211 for hemophilia A, 1177 for hemophilia B, 1177 for herpes, 2226 immunomodulatory, 212-213 in vivo, 211 mechanism of action, 210-211 for Parkinson’s disease, 2460 strategies for, 211 suicide, 212 for β-thalassemia, 1095 GeneClinics, 2529 General anesthesia, 2618 General hospitals, 2626t Generalizability, 32 Generalized anxiety disorder, 2351, 2351t Generalized autonomic failure, 2519 Generalized estimating equations, 36 Generalized seizures, 2401, 2403t, 2407t Genetic association studies, 190-191 Genetic conditions, 186, 1565-1566 of bilirubin conjugation, 985-987 blistering disorders, 2678 clinical characteristics of, 187 management of, 187 myasthenic syndromes, 2548t, 2553 that may become symptomatic during adulthood, 2571 that predispose to colorectal cancer, 1325 treatment of, 186 Genetic counseling, 9, 1388 Genetic defects, 791 Genetic dilated cardiomyopathy, 329 Genetic epilepsy with febrile seizures plus (GEFS+), 2404 Genetic heterogeneity, 194 Genetic mutations. See Mutations Genetic polymorphisms, 189t definition of, 190 of drug-metabolizing enzymes, 131-132, 132t Genetic profiling, 1233 Genetic risk assessment, 188 Genetic testing, 186-187 assaying genetic variation, 195 bioethical issues raised by, 9 clinical impact of, 190-191 complex multimarker genomic tests, 203 ethical issues in, 188 for hypoglycemia, 1555 in muscle disease, 2540-2541 newborn screening, 201-202 for porphyrias, 1413 preconception carrier testing, 201-202 predictive value of, 188 prenatal diagnosis, 201-202 social issues in, 188 Genetic Testing Registry, 203.e2t Genetic variation, 189-191, 191f assaying, 195 common variants, 190 germline variants, 202 Genetics. See also specific diseases advances in, 186 of diabetic nephropathy, 804 of diseases, 186-189, 201 future directions for, 200 of pain, 134 principles of, 186-189 and stroke, 2443-2444 Genital Chlamydia trachomatis infection, 2009-2010, 2009f Genital discharge, female, 1877t, 1879-1880 Genital herpes, 1877t, 1878, 2225 AIDS-associated, 2297t-2301t antiviral therapy for, 2181t prevalence of, 1877 Genital pain, tardive, 2468 Genital ulcers, 1877t, 1878 Genital warts, 1877t, 2221-2222, 2221f, 2222t Genitalia ambiguous, 1563, 1563f management of, 1567 in ovotesticular disorder of sexual development, 1566 prenatal diagnosis and treatment of, 1568 surgery for, 1567

Genitalia (Continued) examination of, 28 external abnormal differentiation of, 1563, 1563f ambiguous, 1563, 1563f development of, 1561, 1561f differentiation of, 1561f internal, 1561f male: examination of, 28 skin lesions in, 2702t Genitourinary infections adenoviral, 2198 staphylococcal, 1900 Genomes, 200-202 and cancer, 1226 complex multimarker genomic tests, 203 expressed, 203 guidance for cancer therapy from, 132-133 human genome, 189-191, 190t, 196-197, 197t individual, 190 microbial, 202 of pathogens, 1838 whole exome sequencing, 186 whole genome sequencing, 186 Genome-wide association studies, 190-191, 200-201 Genomic disorders, 194 Genomic Health Inc., 203 Genomic instability, 1226 Genomics, 1838 Gentamicin for Acinetobacter species infections, 1970 for bronchectasis, 569 for brucellosis, 1981, 1981t for endocarditis, 479t-480t, 480 for enterococcal infection, 1915.e1t for eye infections, 2565, 2565t for granuloma inguinale, 2002 for meningitis, 2488t for osteomyelitis, 1810t for peritonitis, 842-843 for plague, 89b, 1989t for pneumonia, 616t, 619t for prostatitis, 831 for Pseudomonas infection, 1967 for pyelonephritis, 1875t recommended doses and schedules, 1891t-1892t for septic arthritis, 1810t for septic shock, 689t for trench fever, 2000 for tularemia, 1983 GenXpert, 2037 Geographic differences in hypertension prevalence, 382, 382f in osteoporotic fractures, 1637-1638 Geographic tongue, 2582, 2582f Geophagia, 1069 George III, 1411 Geriatric assessment, 102-106 approaches to, 106t cognitive assessment, 104 components of, 103-105 functional status assessment, 102-103, 106t medical assessment, 104-105, 106t nonmedical, 105 strategic approach to, 105-106 Geriatric Depression Scale, 115, 115t Geriatric syndromes, 100 Germ cell tumors extragonadal germ cell cancer syndrome, 1381 noneminomatous definition of, 1366t treatment of, 1366 German measles (rubella), 2204-2206 German Society for Haematology and Medical Oncology, 1858 Germany, 603 Germline genetic variants, 202 Germline mutations, 193 Gerstmann-Straüssler-Scheinker syndrome clinical manifestations of, 2505 genetics of, 2505 prognosis for, 2506 Gestational diabetes insipidus, 1497, 1499

Gestational diabetes mellitus, 1534, 1620-1622 blood glucose control for, 1620-1621 definition of, 1620 diagnosis of, 1620 dietary recommendations for, 1620 epidemiology of, 1620 fetal and neonatal effects of, 1621-1622 maternal effects of, 1622 pathobiology of, 1620 treatment of, 1620b-1621b Gestational history, 1566 Gestational hypertension, 394t Gestational thrombocytopenia, 1166 Geste antagoniste, 2463-2464 Ghon complex, 2032-2033, 2033f Ghost cells, 1057 Ghrelin, 1474.e1 Giant bullae, 644 Giant cell arteritis, 497, 1801 clinical features of, 1796, 1802, 1802t diagnosis of, 1803-1804 differential diagnosis of, 1804 epidemiology of, 1794t, 1801 gastrointestinal, 956 headache of, 2362 ocular symptoms of, 2570 pathobiology of, 1801-1802 pathogenesis of, 1802 signs and symptoms of, 1802-1803 treatment of, 1804b-1805b, 1805 Giant cell granuloma, 2654f Giant cell myocarditis, 328 Giant cell tumors, tenosynovial, diffuse, 1827-1828 Giant neutrophils, 1058 Giant platelets, 1057-1058, 1058f Giardia duodenalis, 2135 Giardia intestinalis, 2135 Giardia lamblia, 2135-2136, 2136f Giardia lamblia cysts, 2136, 2136f Giardiasis, 931 clinical features of, 1869t, 2137, 2137b definition of, 2135-2136 diagnosis of, 2137 distinguishing characteristics of, 2252t-2253t epidemiology of, 1869t, 2136 laboratory findings, 1871f pathobiology of, 2136-2137 prevention of, 2137 Gifts to physicians, 9 Gigantism, 1486 Gilbert’s syndrome, 985-986 bilirubin disposition in, 984 clinical features of, 987t, 2250 Gilenya (fingolimod), 2477 Gilotrif (afatinib), 171 Gingival hyperplasia, 2583, 2584f Gingivitis, 2585 Gingivostomatitis, 2225 Gitelman’s syndrome, 383, 745, 760, 772, 823t, 824 Glanders, 1967-1968, 2058t Glanzmann thrombasthenia, 1171 Glasgow Coma Scale, 2365-2366, 2365t Glatiramer acetate (Copaxone), 2477 Glaucoma, 2566-2567, 2574 acute differential diagnosis of, 2559t distinguishing characteristics of, 2252t-2253t acute angle-closure, 2566, 2567f chronic open-angle, 2566, 2567f pseudoexfoliative, 2566 GlaxoSmithKline, 2246 Gleason score, 1367-1368 Gleevec (imatinib), 171, 1211t-1216t Glimepiride, 126t Glinides, 1535-1536 Glioblastoma, 1293, 1293f Glioma, 1288, 1289f, 1292-1293 brain stem, 1293 clinical manifestations of, 1293, 1293f definition of, 1292 diagnosis of, 1293 epidemiology of, 1292 optic, 1293 pathobiology of, 1292-1293 treatment of, 1293b

I43

Glivec (imatinib mesylate), 171 Global Alliance for Vaccines and Immunization, 1938-1939 Global amnesia, transient, 2384 Global aphasia, 2385, 2385t Global Fund for AIDS, Tuberculosis, and Malaria, 2272 Global health, 19-22 aging, 100, 101t AIDS, 2272 cancer, 1222, 1222f-1223f definition of, 19 disease burden, 19-20 leprosy, 2042-2043 life expectancy, 100 malnutrition, 1434 measles, 2202 Millennium Development Goals, 19, 20t plague, 1984, 1985f rheumatic disease associated with autoimmunity, 1713t rubella, 2204 sickle cell trait, 1096, 1096f Global Initiative for Asthma (GINA), 552f Global ischemic injury, 2432 Global Pertussis Initiative, 1991 Global Registry of Acute Coronary Events (GRACE) risk algorithm, 435, 453, 453.e1f Globi, 2045 Globin, 1095, 1097f Globin disorders, 187t Globin Gene Server website, 1090 Globoid cell leukodystrophy, 2479 Globoid cells, 2479 Globus pharyngeus, 2606 Glomerular basement membrane, 783, 790, 791f Glomerular disorders, 783-784, 792-793 associated with genetic defects, 791 clinical manifestations of, 783 definition of, 783 diagnosis of, 784 epidemiology of, 783 pathobiology of, 783 serum complement levels in, 785t Glomerular filtration rate, 384 estimation of, 728, 730t plasma creatinine and, 728, 729f tradeoff hypothesis for CKD, 834-835, 835f Glomerular syndromes, 733-735 diagnosis of, 734-735 laboratory findings, 734-735 Glomerulonephritis acute, 788-790 anti-GBM, 790, 791f with endocarditis and visceral abscesses, 790 lupus, 1772 membranoproliferative, 787-788 pauci-immune, 1723 post-streptococcal, 789, 790f, 1910, 2252t-2253t rapidly progressive (crescentic), 781, 790-791, 791f classification of, 790t immune complex, 791 Pauci-immune, 791 treatment of, 791b vasculitis-associated, 791 Glomerulus, 783 Glossitis benign migratory, 2582 median rhomboid, 2582 Glossomegaly, 1284, 1285f Glossopharyngeal neuralgia, 2364 Glucagon for drug overdose, 699t-702t, 706t-710t for hyperinsulinemic hypoglycemia, 1555 for hypoglycemia, 1539 plasma levels, 1337 for systemic anaphylaxis, 1702 Glucagon test, 1483t Glucagon-like peptide 1 (GLP-1), 1529 Glucagon-like peptide 1 (GLP-1) agonists, 1536 Glucagonomas, 1335t, 1337, 1553 clinical features of, 1337, 1337f definition of, 1337 Volume I  pp 1-1382  •  Volume II  pp 1383-2722

I44

Index

Glucagonomas (Continued) diagnosis of, 1337 pathobiology of, 1337 prognosis for, 1337 survival rate, 1337 treatment of, 1337b Glucan, 2084 Glucocorticoid deficiency, 1519-1520 Glucocorticoid excess, 1514-1517 Glucocorticoid replacement therapy for adrenal insufficiency, 1521 assessment of, 1521 dosing, 1520-1521 patient education, 1521 for stress, 1521 Glucocorticoid-induced osteoporosis, 1644 Glucocorticoid-remediable hyperaldosteronism, 823t Glucocorticoids guidelines for use, 1761t preparations, 164t for rheumatoid arthritis, 1760-1761 topical, 2658, 2658t Gluconeogenesis disorders, 1552, 1555 Glucose in bacterial meningitis, 2483, 2483t blood levels control of, 1620-1621 dawn phenomenon, 1531 postoperative abnormalities, 2625 for cold injury, 695 fasting levels, 1528-1529 risk for type 2 diabetes, 1537 targets, 1534, 1535t, 1536-1537 HbA1C and, 1528, 1528t for hyperkalemia, 761 for hypoglycemia, 1539 metabolism of, 1548, 1549f physiology of, 1548, 1549f changes during fasting, 1548-1549 changes during feeding, 1549 plasma levels, 1527, 1548 postprandial levels, 1529, 1534, 1535t, 1536-1537 regulation of insulin secretion by, 1529, 1529f sources of, 1548 in urine, 729 Glucose control for acute STEMI, 451 and cardiovascular disease, 259 consensus views, 1546 fetal monitoring, 1621 for gestational diabetes mellitus, 1620-1621 impaired, 1528 during labor and delivery, 1621 maternal monitoring, 1621 pleural fluid, 634 for prevention of CVD in diabetes, 1546 for prevention of stroke, 2444 Glucose self-monitoring, 1531-1532 Glucose tolerance factor, 1450t-1452t Glucose tolerance test, 1486 Glucose-6-phosphate, 1084, 1085f Glucose-6-phosphate dehydrogenase, 1087 Glucose-6-phosphate dehydrogenase deficiency, 1087-1088, 1147t-1148t African type, 2045 agents to be avoided in, 1088, 1088t clinical manifestations of, 1087-1088 diagnosis of, 1088 epidemiology of, 1087 pathobiology of, 1087 peripheral blood smear features of, 1087f severe, 1149-1150 clinical manifestations of, 1150 diagnosis of, 1150 pathobiology of, 1149-1150 treatment of, 1150b Glucosedependent insulinotropic peptide, 1529 Glucose-galactose malabsorption, 933 Glucose-insulin-potassium, 451 α-Glucosidase inhibitors, 1536 GLUT1 deficiency, 1386, 1386t Glutamine, 932 Glutaric aciduria type I, 1387t Glutaric aciduria type II, 2545 Glutathione metabolism disorders of, 1087-1088 pathway, 1085-1086, 1085f

Glutathione reductase deficiency, 1147t-1148t Glutathione synthetase deficiency, 1147t-1148t, 1149 clinical manifestations of, 1149 diagnosis of, 1149 pathobiology of, 1149 treatment of, 1149b Gluten-sensitive enteropathy, 1824 Glybera, 210 Glycogen, 1397 Glycogen metabolism disorders of, 2542t, 2545 synthesis and degration, 1397-1398, 1397f Glycogen storage diseases, 1133t, 1134, 1397-1399 characteristic signs of, 1385t clinical features of, 1398, 1398t complications of, 1399 definition of, 1397 diagnosis of, 1398 epidemiology of, 1397 hypoglycemia due to, 1552, 1555 incidence of, 1385t pathobiology of, 1397-1398 pathophysiology of, 1385t, 1397-1398, 1397f prevention of complications of, 1399 prognosis for, 1399 treatment of, 1388t, 1398b-1399b Glycogenosis type II, 2545 type IV, 2545 Glycoprotein IIb/IIIa inhibitors, 437-439, 438t-439t Glycoprotein IIb/IIIa receptor antagonists, 450 Glycopyrrolate, 2521, 2525 Glycosides, cardiac, 699t-702t Glycosuria, renal, 823t, 1386t Glycosylated hemoglobin, 1528 Glycosylation disorders, congenital, 1385t, 1387 Glycosylphosphatidylinositol deficiency, 1079 Glycylcyclines, 1894 mechanism of action, 1889t recommended doses and schedules, 1891t-1892t resistance to, 1890t Gnathostoma spinigerum, 2166 Gnathostomiasis, 2165t, 2166 Goals, patient, 103 Goiter, 1510 clinical examination of, 1510 definition of, 1510 diagnosis of, 1510 epidemiology of, 1510 imaging, 1510 laboratory findings, 1510 pathobiology of, 1510 toxic multinodular, 1506 treatment of, 1510b Gold, 1761 Golfer’s elbow, 1752 Golimumab (Simponi), 169 for inflammatory bowel disease, 939t, 940 for rheumatoid arthritis, 1762 for ulcerative colitis, 942 Goltz’s syndrome, 1671 Gonadal biopsy, 1567 Gonadal differentiation, normal, 1560, 1561f Gonadal dysgenesis, 1590 mixed, 1565, 1584 pure, 1590 Gonadal insufficiency, 1574 Gonadal steroidogenesis, 1561-1562, 1562f Gonadotropin-producing tumors, 1493b Gonadotropin-releasing hormone, 1473-1474 Gonadotropins, 1478, 1492-1493, 1596 Gonococcal keratitis, 2565 Gonococcal septic arthritis, 1808 clinical manifestations of, 1808 diagnosis of, 1808 epidemiology of, 1808 treatment of, 1808b Gonococcal urethritis, acute, 1941, 1941f Gonococcemia, 2697, 2697f Gonorrhea in children, 1943 clinical manifestations of, 1941 clinical syndromes, 1941-1945

Gonorrhea (Continued) diagnosis of, 1941 disseminated infection, 1943-1944 epidemiology of, 1940-1941 incidence of, 1877, 1940 lower genital tract, in females, 1942, 1945t microscopic findings, 1941 oral ulcers of, 2580t pathobiology of, 1941 pharyngeal infection, 1943, 2603 prevalence of, 1940 prevention of, 1944 rectal infection, 1943 treatment of, 1944b-1945b, 2603 antibiotic regimens for, 1944-1945, 1945t duration of therapy, 1896 follow-up, 1945 urogenital, in males, 1941-1942, 1945t Goodpasture’s syndrome, 573 clinical manifestations of, 573 diagnosis of, 574 pathobiology of, 573 prognosis for, 574 treatment of, 574 Good’s syndrome, 1679, 1680t Goose foot, 1753 Gooseneck sign, 285 Gordon’s syndrome, 760, 823t Gorlin formula, 464 Goserelin (Zoladex), 1211t-1216t, 1369 Gout, 1811-1814 acute differential diagnosis of, 1814 treatment of, 1814-1815 advanced, 1813, 1815 atypical presentations of, 1813-1814 classic, 1813 definition of, 1811 diagnosis of, 1814 epidemiology of, 1811 with heart failure, 316 incidence of, 1811 natural course of, 1811 pathobiology of, 1811-1813 pathogenesis of, 1813 prevalence of, 1811 radiographic findings, 1724f, 1724t recommendations for lifestyle and diet, 1815t recurrent, 1815 saturnine, 1814 treatment of, 316, 1814, 1814b-1815b Gouty arthritis, acute, 1813, 1813f Gouty tophi, 1813, 1814f GPI. See Glycosylphosphatidylinositol β2-GPI ELISA assay, 1180 GPR56 gene, 2513 Graduate Medical Education (GME), 46 Graft rejection. See Rejection Grafts, 458-459, 459f Graft-versus-host disease, 1200-1201 acute, 1200 classic, 1200 classification of, 1200, 1200t persistent, 1200 recurrent, 1200 chronic, 1201 diarrhea in, 932 in immunocompromised patients, 1858-1859 mechanisms of, 238-239 strategies for prevention of, 239, 239.e1t transfusion-associated, 1197 Graft-versus-leukemia effect, 1199 Graft-versus-tumor effects, 239 Gram stain, 1886 Gram-negative bacillary infections, 1962 health care–associated infections, 1863-1864 meningitis, 2487 Gram-negative organisms, 476, 477t Granada virus, 2259 Grand mal seizures, 2402 Granisetron, 865t Granulation, toxic, 1055f Granulocyte colony-stimulating factor, 1050 clinical uses of, 1050-1051 for glycogen storage disease, 1399 for neutropenia, 1137 recombinant, 1051-1052 in regulation of granulopoiesis, 1144

Granulocyte-macrophage colony-stimulating factor, 232, 1144 Granuloma actinomycotic, 2062 eosinophilic, 576 fish tank, 2041 giant cell, 2654f hepatic, 1019 midline lethal, 1301 oral ulcers, 2580t Granuloma inguinale (donovanosis), 2001-2002 clinical manifestations of, 2001, 2001f definition of, 2001 diagnosis of, 2001-2002 differential diagnosis of, 2001-2002 epidemiology of, 2001 treatment of, 2002b Granulomatosis, with polyangiitis, 586 ANCAs in, 1795-1796 clinical manifestations of, 1797-1798 differential diagnosis of, 1153 laryngeal involvement, 2605, 2605.e1f neuropathy in, 2533, 2533t pathology of, 1795t, 1797 treatment of, 1798, 2534 Granulomatous disorders, 1740t chronic disease, 1682.e1t clinical manifestations of, 1683 epidemiology of, 1682-1683 genetics of, 1683 color in, 2638t-2644t cutaneous disease, 2691 hypercalcemia with, 1658 liver disease, 1018-1019, 1018t Granulomatous inflammatory arthritis, dermatitis, and uveitis, 1740t, 1743 Granulomatous meningitis, 2599 Granulomatous myositis, 1790t Granulopoiesis cellular stages, 1143f regulation of, 1144 site of, 1143-1144 steps in, 1143-1144, 1143f Graves disease, 1505-1506 clinical manifestations of, 1506 definition of, 1505 epidemiology of, 1505 pathobiology of, 1505-1506 in pregnancy, 1508 prognosis for, 1506 treatment of, 1508 Graves’ ophthalmopathy, 1469, 1506, 1506f, 2560f Gray matter, 1821, 1821.e1f Gray platelet syndrome, 1057-1058, 1167 Gray-zone lymphoma, 1265 Grey Turner’s sign, 858t, 961 GRFoma, 1335t, 1338 definition of, 1338 diagnosis of, 1338 pathobiology of, 1338 treatment of, 1339b Grifols, 1170 Griscelli’s syndrome, 2633 Griscelli’s syndrome type 2, 1133t, 1134 Groin, hanging, 2169 Groin hernias, 945 Groin lesions, 2702t Group interventions, 1631-1632 Growth, 1069 Growth differentiation factor GDF-1, 304.e1t Growth factors, 231.e1t hematopoietic, 1050, 1236-1237 myeloid, 1858 Growth hormone, 1478, 1485-1486 for growth hormone deficiency, 1485 for hypopituitarism, 1483t for short-bowel syndrome, 932 Growth hormone deficiency, 1485 clinical manifestations of, 1485 diagnosis of, 1485 pathobiology of, 1485 treatment of, 1485b Growth hormone excess, 1486 clinical manifestations of, 1486 diagnosis of, 1486 pathobiology of, 1486 treatment of, 1486b-1487b Growth hormone secretagogue, 1474.e1

Index Growth hormone–releasing factor (GRF), 1338 Growth hormone–releasing hormone, 1474, 1483t Guaiac-based testing, 1328 Guaifenesin, 2588 Guam, 2522 Guanabenz, 388t Guanarito virus, 2247, 2249t Guanfacine, 388t, 2466 Guided imagery, 182t Guideline for the Management of Overweight and Obesity in Adults (NHLBI), 1458 Guideline on Management of Opportunistic Infections in Adults and Adolescents (CDC-NIH-IDSA), 2294-2295 Guidelines for the Prevention and Treatment of Opportunistic Infections in HIV-Infected Adults and Adolescents (CDC), 2294 Guillain-Barré syndrome, 2518-2519, 2529-2531, 2530t associated autoantibody, 2518-2519, 2518t clinical manifestations of, 2530 CSF formula, 2342t definition of, 2529 diagnosis of, 2530 differential diagnosis of, 2530 epidemiology of, 2529 incidence of, 2529 pathobiology of, 2529-2530 prognosis for, 2530-2531 treatment of, 2530b Guinea worm, 2171 Gustatory seizures, 2401t Gut microbiota and liver disease, 1842 and obesity, 1842 and rheumatoid arthritis, 1842 Guthrie, Robert, 1384 Guttate hypopigmentation, 2699 Guttate (“raindrop”) leukoderma, 2699, 2699f Gynecologic cancers, 1360-1365 Gynecologic infection, 1947

H

H reflex, 2342 H1N1 influenza, 2056-2057, 2197 H5N1 influenza, 2056-2057 fatality rate, 2060t prognosis for, 2197 H7N9 influenza, 2057, 2197 Habits, 1717 HACEK gram-negative organisms, 476, 477t Haemophilus, 1946 Haemophilus aphrophilus, 1948 Haemophilus ducreyi, 1949 Haemophilus infections, 1946-1948 Haemophilus influenzae, 1946 Haemophilus influenzae infections ampicillin-resistant, 1948 clinical manifestations of, 1946-1948 epidemiology of, 1946 pathobiology of, 1946 prevention of, 1948 sites of, 1946t treatment of, 1947b-1948b Haemophilus influenzae meningitis, 1947-1948, 2486 Haemophilus influenzae type b (Hib), 1946 Haemophilus influenzae type b meningitis, 1947 Haemophilus influenzae type b vaccine, 78, 1946, 1948, 2489, 67.e1t-67.e3t Haemophilus parainfluenzae, 1948 Haemophilus paraphrophilus, 1948 Hageman factor (factor XII) deficeincy, 1175t Hailey-Hailey disease, 2633, 2652f Hain Genotype MTBDR plus assay, 2037 Hair normal, 2703 racial differences in, 2706 Hair cycle, 2703 Hair disorders, 2703-2710 in children, 2706 dermatoscopic signs of, 2703t racial differences in, 2706

Hair follicles, 2634, 2703 Hair growth, excessive, 2706-2707 Hair jellyfish stings, 720 Hair loss, 2661, 2703 causes of, 2703, 2703t drugs that induce, 2704t patchy, 2704f-2705f Hairy cell leukemia, 1252-1253, 1252f differential diagnosis of, 1252-1253, 1254t treatment of, 1253b Hairy leukoplakia, 2582, 2582f oral, 2234, 2319, 2644f Halaven (eribulin mesylate), 1211t-1216t Halcinonide, 2658t Hallucinogen persisting perception disorder, 161 Hallucinogens, 161 abuse of clinical manifestations of, 161 epidemiology of, 161 pathobiology of, 161 treatment of, 161b Halobetasol propionate (Ultravate), 2658t Halofantrine, 2111 Haloperidol for chorea, 2463 for dystonia, 2465 for psychosis, 2354 for Tourette’s syndrome, 2466 for traumatic brain injury, 2368 Halos, 2559 Hamartoma hypothalamic, 1476-1477 PTEN hamartoma syndrome, 1325 Hamartomatous polyps, 1325 Hamman-Rich syndrome, 572-573, 583 Hancock modified orifice stented valve, 472t, 473f Hand(s) burning hands, 2366 rheumatoid arthritis in, 1756-1757, 1756f skin lesions in, 2702t Hand-foot syndrome, 1099, 1824 Hand-foot-and-mouth disease, 2241t, 2242 clinical manifestations of, 2242 diagnosis of, 2242 prognosis for, 2242-2243 treatment of, 2242b-2243b Hanging groin, 2169 Hansen’s disease (Leprosy), 2042-2046 Hantaan virus, 2249t Hantavirus, 91t, 2247 Hantavirus cardiopulmonary syndrome, 2058t, 2247, 2249t Hantavirus fever with renal failure, 2057 Hantavirus infections, 2250 Hantavirus pneumonia, 2057 Hantavirus pulmonary syndrome, 2057t, 2247 clinical manifestations of, 2251, 2251t fatality rate, 2060t management of, 2254 pathobiology of, 2251t Haploinsufficiency, 2397 Haplotype, 190-191 Hard metal disease, 589t Harlequin syndrome, 2519 Harris-Benedict equation, 1441, 716.e1t Harrison Narcotic Act, 160 Hartnup’s disease, 823t, 1386t Hashimoto’s (autoimmune) thyroiditis, 1502-1503 HAT Sero + K SeT test, 2115 HAT Sero-Strop, 2115 Hawaiians, 833-834 Hay fever, 2587f Hazard ratios, 35 Hazards, 35 proportional hazards (Cox) model, 35 workplace and ambient environment exposures, 81-82, 81t HBZ gene, 2236 Head and neck cancer, 1297-1303 chemotherapy for, 1302 clinical manifestations of, 1299-1300 definition of, 1297 diagnosis of, 1300-1301 epidemiology of, 1297-1298 pathobiology of, 1298-1299 prognosis for, 1303

Head and neck cancer (Continued) radiation therapy for, 1302 squamous cell carcinoma approach to, 1301t staging, 1301t surgical therapy for, 1302 treatment of, 1301b-1303b follow-up, 1302 future directions, 1303 selection of program, 1301 with unknown primary site approach to, 1300-1301 evaluation of, 1300, 1300f Head and neck examination, 26-27 Head and neck infections, anaerobic, 1932 Head down (Trendelenburg) position, 2448t Head injury, 714 assessment of, 2365t Canadian CT Head Rule, 2366, 2366t traumatic brain injury, 2364-2370 Head lice, 2175 diagnosis of, 2175 epidemiology of, 2175 treatment of, 2175b Head pain, 2356-2364 Headache, 2356-2364 chronic daily, 2361-2362 clinical manifestations of, 2357 cluster, 2357t, 2360-2361 definition of, 2356 diagnosis of, 2357 differential diagnosis of, 2357, 2357t epidemiology of, 2356 migraine, 2357t, 2358-2359 neuralgiform, short unilateral, with conjunctival injection and tearing, 2357t, 2360 pathobiology of, 2356 preferred medications for women, 1602t prevention of, 2357, 2358t prognosis for, 2357 secondary, 2357, 2357t, 2362-2364 sinus, 1690-1691, 2362 tension-type, 2357t, 2359-2360 thunderclap, 1800, 1861 treatment of, 1602t, 2357b in women, 1602t Head-thrust test, 2600 Healing touch, 184 Health cardiovascular definition of, 262t new paradigm for, 261 dietary guidelines for, 1428t-1429t global, 19-22 microbiome and, 1841-1843 millennium development goals, 19 of older adults, 100-102 of psychiatric patients, 2625 social status and, 17-18, 18f Health care access to, 15-16 barriers and interventions to, 16-17 cultural competence in, 16-17 measures of, 41-43, 42t, 41.e3f attributes of, 43t examples, 42t limitations of, 42 purposes, 42 selection of, 42-43 sources, 41 uses for, 42, 43t quality, 15-16 quality measures, 41, 42t Health care delivery matching patient needs and, 47-49 regional collaboration, 1868 risk factors for health care–associated infections, 1862-1863 Health care expenditures, 18-19, 46 for cardiovascular diseases, 258 for diarrhea, 918 for functional dyspepsia, 894 global patterns, 21 for health care–associated infections, 1861, 1862t for natural products, 181 Health care facilities, 1868 Health Care Innovations Exchange, 43 Health care organizations, 49

I45

Health care professionals, 16 Health care resources, 41, 42t Health care systems design of, 48-49 navigation of, 16 Health care workers, 2286-2287 Health care–associated infections, 1861-1868 acute surveillance testing for, 1868 antimicrobial resistance in, 1863, 1863t burden of, 1861-1867, 1862t costs of, 1861, 1862t decolonization therapy for, 1867-1868 definition of, 1861 device-associated, 1865-1866, 1865.e1t fatality rates, 1861, 1862t guidelines and recommendations for, 1865.e1t health care delivery–related factors, 1862-1863 health care–related factors, 1862 host-related factors, 1862 incidence of, 1861, 1862t pathogenesis of, 1862-1863 pathogens, 1862-1865 prevention of goals and incentives, 1868.e1 horizontal strategies for, 1867 National Action Plan to Prevent Healthcare-Associated Infections, 1868.e1 strategies for, 1867-1868 vertical strategies for, 1867 viral, 1865 Health care–associated pneumonia, 618-620 antibiotic treatment of, 619t Pseudomonas in, 1964 treatment of, 619b-620b Health Data Interactive (CDC), 43.e1t Health examination, periodic, 54-58 Health Indicators Warehouse (CDC), 43.e1t Health Level 7, 203 Health measures, 41-43, 42t, 41.e3f attributes of, 43t definition of, 41 examples, 42t limitations of, 42 purposes, 42 selection of, 42-43 sources of, 41 uses for, 42, 43t Health Resources and Services Administration (HRSA), 2287 Health System Management Project (HHS), 43.e1t Health visits with adolescents, 63 conclusion of, 63 Healthcare Cost and Utilization Project (AHRQ), 43.e1t Health-enhancing physical activity, 58 Health-related physical fitness, 58 Healthy aging, 102 Healthy People 2020, 15 Hearing, 2593-2601 anatomy of, 2593-2594 normal, 2594 physiology of, 2593-2594 Hearing Handicap Inventory for the Elderly, 104 Hearing loss, 2593 in acute poisoning, 697 central differential diagnosis of, 2595 pathobiology of, 2594 conductive differential diagnosis of, 2594 pathobiology of, 2594 diagnosis of, 2594-2595 differential diagnosis of, 2594-2595 disabling, 2594 drug-induced, 2595 epidemiology of, 2594 evaluation of, 2594, 2595f noise-induced, 2595 sensorineural differential diagnosis of, 2594-2595 occupational, 80, 80t pathobiology of, 2594 treatment of, 2595b-2596b Volume I  pp 1-1382  •  Volume II  pp 1383-2722

I46

Index

Hearing screening, 57 bedside test, 2594 geriatric, 104 Heart. See also under Cardiac anatomy of, 264-265, 264f, 356 athlete’s, 274f boot-shaped, 285, 285f conduction abnormalities, 270-271 conduction system, 263, 267, 267f development of, 262-263, 263.e1f electrophysiology of, 264, 339-344 energetics of, 265 examination of, 303 Frank-Starling law of, 266 function of, 262-267 functional measurement of, 279-280 inspection of, 253 malposition of, 416 mechanics of, 557 normal activation of, 267 physiology of, 265-266 pressure-volume area, 265 regeneration of, 267 remodeling of, 342 rheumatoid arthritis in, 1758t Starling’s law of, 266, 266f structure of, 262-264 ultrastructure of, 264 univentricular, 415 work of, 265-266 Heart block. See also Atrioventricular block functional, 342-343 management of, 452 MR findings, 291f Heart disease asymptomatic, 307, 309 cell therapy for, 209-210 coexisting, 460 congenital, 405-417 echocardiographic findings, 280t-281t MR findings, 289-292, 292f diagnosis of, 369t dietary factors, 1427t dietary guidelines for prevention of, 1428t-1429t impulse conduction disorders, 342 impulse formation disorders, 342 ischemic chronic, 371 prognosis for, 432 risk reduction strategies for, 2614-2615 stable, 420-432 molecularly guided therapeutics for, 202.e1t prognosis for, 286 in sarcoidosis, 606 structural, 373-374 valvular, 461-473 heart failure due to, 317-318 hemodynamics of, 294-295, 294f-295f preoperative assessment of, 2615 in women, 1612t Heart failure, 267, 298-305, 329.e1f acute decompensated factors that may precipitate, 299, 299t with pulmonary edema, 318-319 in aortic stenosis, 463 biomarkers, 304, 304.e1t, 304.e2f causes of, 298-299, 299t, 318t chest radiography findings, 304 chronic, 299, 299t classification of, 298 clinical manifestations of, 301-304, 304f comorbidity, 316 coronary angiography findings, 305 definition of, 298 in diabetes, 1546 diagnosis of, 301f, 301t, 304-305 diastolic, 267, 317 disparities, 298 drugs to use with caution in, 314 due to hypertrophic cardiomyopathy, 318 due to left ventricular systolic dysfunction practical guidance on use of ACEIs and ARBs for, 310t practical guidance on use of mineralocorticoid receptor antagonists for, 312t practical guidance on use of β-blockers for, 311t due to nonischemic dilated cardiomyopathy, 318

Heart failure (Continued) due to valvular heart disease, 317-318 ECG findings, 304 echocardiography findings, 304-305 education about, 315, 315t end-of-life considerations, 319-320 epidemiology of, 298-299 evaluation of, 27, 305-306 future directions, 320 hemodynamic monitoring in, 305 invasive diagnostics for, 305 laboratory findings, 304 management of, 305-306, 308f, 452-453 general approach to, 306f therapies of proven benefit, 306t misdiagnosis of, 305 MR findings, 291f, 305 nuclear cardiology findings, 305 outpatient follow-up, 319 palliative care for, 319-320 pathobiology of, 299-301, 299t pathophysiology of, 299, 300f physical findings, 303 postoperative, 2622t preoperative assessment of, 2615 with preserved ejection fraction, 300-301, 300t, 317 prognosis for, 319-320 with reduced ejection fraction, 299-300, 309-317 risk factors for, 307 stages of, 299f, 306-320, 308f symptomatic, 309-320 organization of care for, 314-315 pharmacologic issues, 314 with reduced systolic function, 306.e1t-306.e2t treatment of, 309b-319b, 306.e1t-306.e2t symptoms of, 301-303 trajectories of disease, 10.e1f treatment of, 307b-309b ventricular tachycardia in, 371-372 Heart murmurs, 254 causes of, 255t systolic bedside identification of, 254, 255t quiet, 27 Heart rate, 266 elevated, 31 normal, 356, 359 normal resting, 268-269, 268t Heart size, 283 Heart sounds, 253-254, 254f first, 253, 254f, 414-415 fourth, 303 sail sound, 414-415 second, 254f third, 253-254, 254f, 303 tumor plop, 339 Heart transplantation, 317, 519-523, 202.e1t Heart valves location of, 283f mechanical, 472t, 473f prosthetic advantages of, 472t choices among, 472, 473f disadvantages of, 472t postoperative care for, 472 Heartburn approach to, 852t endoscopy in, 874 functional, 895-896 clinical manifestations of, 896 definition of, 895 diagnosis of, 896 epidemiology of, 895-896 prevalence of, 896 prognosis for, 896 treatment of, 896b management of, 899f of presumed esophageal origin, 890 Heart-lung transplantation, 403, 645 Heat: disorders due to, 691-695 Heat exhaustion, 692 Heat illness, 692-693 clinical manifestations of, 692-693 definition of, 692 diagnosis of, 692-693 epidemiology of, 692 factors predisposing to, 692t management of, 693b, 693t

Heat illness (Continued) pathobiology of, 692 prevention of, 693 prognosis for, 693 Heat injury, 692 Heatstroke, 692-693 classic, 692-693, 693t definition of, 692-693 distinguishing characteristics of, 2252t-2253t exertional, 692-693, 693t pathophysiology of, 692.e1f Heaviness, 2338 Heavy chain diseases, 1273, 1283 premalignant, 1273t types of, 1283 α-HCD, 1283 γ-HCD, 1283 µ-HCD, 1283 Heavy metal poisoning, 93t Hedgehog signaling pathway inhibition, 2661 HEEADSSS assessment, 63 Height and cancer, 1223-1224 measurement of, 1431 standards for men and women, 1431, 1432t Height vertigo, 2597 Height-for-age Z-score (HAZ), 1435 Heinz bodies, 1057, 1087, 1087f, 1095 Heinz body hemolytic anemia, congenital, 1095 Helicobacter cinaedi, 1955 Helicobacter fenelliae, 1955 Helicobacter heilmannii, 912 Helicobacter pylori eradication of, 914, 915t gastric colonization, 909, 909f testing for, 913 Helicobacter pylori gastritis, 909 Helicobacter pylori–associated ulcers prevention of, 915 treatment of, 914-915, 915t Heller myotomy, 904 HELLP (hemolysis, elevated liver enzymes, and low platelets), 1166 Helmet cells, 1053f Helminthic infections drugs for, 2104-2105 of liver, 1014-1018 treatment of, 2103-2105 Helper T cells activation of, 1688-1689 follicular (TFH), 227 type 1 (TH1), 227 type 2 (TH2), 227 type 17 (TH17), 227 Helping Patients Who Drink Too Much: A Clinician’s Guide (NIAAA), 154 Hemagglutinin, 2192t Hemangioblastoma, 1294 Hemangioendothelioma, gastrointestinal, 958 Hemangiomas, 2690 benign capillary, 2690, 2690f eyelid, 2571 gastrointestinal, 958 hepatic, 980, 982f Hemarthrosis acute, 1173, 1173f treatment of, 1175-1176 Hematochezia, 875-876, 875f severe colonic sources of, 882, 882t management of, 882, 882f Hematocrit, 1060t Hematologic disorders arthritis in, 1823t high-penetrance monogenic disorders, 187t in HIV infection, 2327-2328 preoperative evaluation of, 2616 in systemic lupus erythematosus, 1773 Hematology laboratory values, 2717t-2718t in liver disease, 990 reference intervals, 2717t-2718t Hematoma aortic, intramural, 494-497, 496f epidural, 2366, 2367f subdural, 2366, 2366f alcohol-related, 2510, 2511f Hematopoiesis, 1050, 1051f

Hematopoietic acute radiation syndrome, 84 Hematopoietic cell expansion, 1050 Hematopoietic cells autoimmuned-mediated failure of single hematopoietic lineages, 1115 clinical uses of, 1050-1052 Hematopoietic growth factors, 1050 for cancer, 1219 clinical uses of, 1050-1052 for myelodysplastic syndromes, 1236-1237 Hematopoietic papules, 2690-2691 Hematopoietic stem cell transplantation, 237, 1198-1204 for acute lymphoblastic leukemia, 1244 for acute myeloid leukemia, 1245 allogeneic, 1199-1201 for atypical CML, 1252 for chronic myelogenous leukemia, 1248-1250, 1249t, 1252 for chronic myelomonocytic leukemia, 1252 process, 1199-1200, 1200f autologous, 1201 for cancer, 1218 cardiac toxicity, 1203 complications of, 1203-1204 conditioning for, 1199-1200, 1200f graft rejection in, 1203 in immunocompromised patients, 1855, 1855t indications for, 1201-1203, 1202f liver toxicity in, 1203 matched sibling donor, 1119-1120 matched unrelated donor, 1120 preparation of recipient for, 1199-1200 pulmonary toxicity, 1203 renal toxicity, 1203 sources of donor cells, 1199 syngeneic, 1201 types of, 1199 Hematopoietic stem cells, 1050 Hematopoietic system age-related changes in, 108 alcohol-related complications, 152 Hematopoietic tumors, 2690-2691 Hematuria, 730, 731f-732f Heme production of, 1063 urine dipstick test for, 729 Heme arginate, 1414 Heme biosynthesis, 1072, 1072f, 1408-1409, 1409f, 1409.e1f Heme oxygenase, 983 Hemiballism, 2463 Hemicrania, paroxysmal, 2360 differential diagnosis of, 2357t prevention of, 2361 Hemicrania continua, 2357t, 2360 diagnosis of, 2361 pathobiology of, 2360 prevention of, 2361 treatment of, 2361 Hemidiaphragmatic paralysis, 628.e1f Hemi-ghost cells, 1057 Hemiparetic gait, 2341t Hemiptera, 2176 Hemispatial neglect, 2386 cortical disorders of, 2385-2386 clinical manifestations of, 2386 definition of, 2385 diagnosis of, 2386 pathobiology of, 2386 Hemochromatosis, 187t, 1418-1423 cardiomyopathy with, 331 clinical manifestations of, 1420-1421 diagnosis of, 1421 family screening for, 1422 genetics of, 1419-1420 hereditary, 335, 982, 1418-1419, 1420t evaluation of, 1421, 1422f laboratory findings, 1420-1421 physical findings, 1420t stages of, 1419 symptoms of, 1420t treatment of, 1421 joint involvement in, 1826 liver biopsy in, 982, 982f liver transplantation for, 1035 pathophysiology of, 1419-1420 population screening for, 1422 prevention of, 1422 prognosis for, 1422-1423

Index Hemochromatosis (Continued) testicular hypogonadism due to, 1573 treatment of, 1421b Hemochromatotic arthropathy, 1826, 1826f Hemodialysis, 841 access issues, 842 in septic shock, 690 toxicants removed by, 710t Hemodynamic compromise, 666 Hemodynamics drug dose adjustments for, 129 general management of, 677f in heart disease, 294-295 in heart failure, 305 normal resting values, 266t Hemofiltration, high-efficiency postdilution online, 841 Hemoglobin, 677-680 blood composition, 1102 components deficient in microcytic anemia, 1063, 1063f fetal: hereditary persistence of, 1089t, 1090, 1093 glycosylated, 1528 in hypovolemia, 745 normal values, 1060t, 1089 oxygen binding, 1060 oxygen-binding curves for, 1060f Hemoglobin A1C conditions that may affect measurement or interpretation of, 1528, 1528t and congenital anomalies, 1620, 1620f and glucose levels, 1528, 1528t Hemoglobin C, homozygous, 1055f Hemoglobin C crystals, 1055f Hemoglobin Constant Spring, 1056 Hemoglobin E, 1093 Hemoglobin E thalassemias, 1093 Hemoglobin H disease, 1056, 1090 Hemoglobin Köln, 1104 Hemoglobinopathies, 1104 arthritic manifestations of, 1824 hematopoietic stem cell transplantation for, 1203 pathobiology of, 1095 prevalence of, 1096t unstable, 1095 Hemoglobinuria paroxysmal cold, 1073-1075, 1075t paroxysmal nocturnal, 1073, 1078-1079, 1116, 1119 Hemolysis, intravascular causes of, 1079 treatment of, 1079 Hemolytic anemia, 1066-1067, 1080-1088 acute, 1088, 1088b autoimmune, 1073-1076 in chronic lymphocytic leukemia, 1256 definition of, 1073 blood smear features of, 1057t causes of, 1073, 1074t chronic, 1088 blood smear features of, 1056t treatment of, 1088b classification of, 1067t congenital, 1086 definition of, 1073 differential diagnosis of, 1057 drug-induced, 1076 extracorpuscular, 1079-1080 Heinz body, congenital, 1095 immune, 1078 inherited, 1089-1095 intravascular, 1073-1080 laboratory features of, 1067, 1067t pathobiology of, 1066-1067 Hemolytic transfusion reactions, 1079, 1196 Hemolytic-uremic syndrome, 735, 1166, 1740t clinical manifestations of, 1166 definition of, 1166 pathobiology of, 1166 prevention of, 1166b treatment of, 1166b Hemophagocytic lymphohistiocystosis, 1147t-1148t, 1150 clinical manifestations of, 1150 diagnosis of, 1150 pathobiology of, 1150 treatment of, 1150b

Hemophagocytic syndrome, 1150 Hemophilia acquired, 1177 clinical manifestations of, 1177 diagnosis of, 1177 epidemiology of, 1177 pathobiology of, 1177 prognosis for, 1177 treatment of, 1177b ancillary therapy for, 1176 arthritic manifestations of, 1824 carrier detection, 1176-1177 complications of, 1173-1174, 1173f-1174f diagnosis of, 1174 hereditary, 1173-1177 prenatal diagnosis of, 1176-1177 prognosis for, 1177 Hemophilia A arthritic manifestations of, 1824 clinical manifestations of, 1173-1174 definition of, 1173 epidemiology of, 1173 gene therapy for, 1177 genetics of, 1173 treatment of, 1174b-1176b Hemophilia B arthritic manifestations of, 1824 clinical manifestations of, 1173-1174 definition of, 1173 epidemiology of, 1173 gene therapy for, 1177 genetics of, 1173 treatment of, 1174b-1176b Hemophilia C, 1175t Hemoptysis approach to, 529-531 diagnosis of, 529-531 massive, 530t special evaluations for, 531t treatment of, 531b Hemorrhage. See also Bleeding alveolar, diffuse, 571t, 573-574 cerebral, 395t, 2433 gastrointestinal, 715, 879-884 intracerebral, 2450-2453 with peptic ulcer disease, 915-917 splinter, 255, 257f, 2708-2709 subarachnoid, 2342t, 2445-2450 subungual, 2638t-2644t, 2641f ulcer, 880, 880t variceal in cirrhosis, 1025-1028 liver biopsy in, 980 into vitreous humor, 2574 Hemorrhagic cerebrovascular disease, 2445-2454 Hemorrhagic conjunctivitis, acute, 2241t, 2243 Hemorrhagic diarrhea, 2057t Hemorrhagic disorders. See also specific disorders coagulation deficiencies, 1172-1181 gastrointestinal, 957-958 vascular, 957-958, 1172 Hemorrhagic fever, viral, 87t, 90-91, 2247-2256 Hemorrhagic fever viruses, 2249t bat-borne, 2247 maintenance in nature, 2247-2250 mosquito-borne, 2247 reservoir control, 2256 rodent-borne, 2247 tick-borne, 2250 transmission of to humans, 2247-2250 human-to-human, 2249t vector control, 2256 Hemorrhagic fever with renal syndrome, 2249t clinical manifestations of, 2251, 2251t pathobiology of, 2251t vaccine for, 2255-2256 Hemorrhagic shock, 673-674 categorization of, 715t classification of, 674t etiology of, 678f-679f initial treatment of, 715t Hemorrhagic smallpox, 2216, 2216t Hemorrhagic telangiectasia, hereditary, 1172, 1172f

Hemorrhoidal disease, 967 Hemorrhoids, 967-968 clinical manifestations of, 967-968 diagnosis of, 967-968 epidemiology of, 967 external, thrombosed, 968, 968f grade 4, 967-968, 968f internal grades, 967-968, 968t management of, 968b, 968t pathobiology of, 967 treatment of, 968b Hemosiderosis, idiopathic pulmonary, 573, 586 Hemostasis characteristic patterns of bleeding in, 1156t mechanisms of, 1154-1155 normal, 1154-1155 Hemostatic agents, 1170 Hemothorax, 634-635 Hemotympanum, 2590, 2591f Henderson-Hasselbalch equation, 652, 739, 763 Hendra virus, 2056, 2058t Hendra virus encephalitis, 2057 Henoch-Schönlein purpura, 789 classification of, 1794 clinical manifestations of, 789, 1798 diagnosis of, 789 gastrointestinal involvement, 957 pathologic characteristics of, 1795t prognosis for, 789b treatment of, 789b Henry’s law, 597-598 Heparin, 176-178, 177f Heparin therapy, 177 for acute coronary syndrome, 439 for acute STEMI, 448 for bleeding before acute leukemia therapy, 1243 for deep venous thrombosis prevention, 2368 initial IV therapy, weight-based, 516t low-molecular-weight, 177, 177f, 177t for acute coronary syndrome, 439 for acute STEMI, 448, 450 for anticoagulation, 516t for deep venous thrombosis, 515, 517, 2368 in pregnancy, 517, 1617, 1618t for pulmonary embolism, 624, 625t for venous thromboembolism, 517-518, 1617 perioperative, 2613t in pregnancy, 1619t for pulmonary embolism, 624 unfractionated for acute coronary syndrome, 438t-439t, 439 for acute limb ischemia, 500 for acute STEMI, 450 for deep venous thrombosis, 517 in pregnancy, 517, 1617, 1618t for venous thromboembolism, 1617 for venous thromboembolism, 1619t Heparin-induced thrombocytopenia, 517, 1159, 1162-1163 clinical manifestations of, 1163 diagnosis of, 1163 epidemiology of, 1162 laboratory assays for, 1163, 1163t pathobiology of, 1162-1163, 1162f pathophysiology of, 1162-1163, 1162f treatment of, 1163b Hepatectomy, living-related donor, 1035, 1035f Hepatic arterial disease, 959 Hepatic cirrhosis dietary factors, 1427t in HIV infection, 2303t in Wilson’s disease, 1416, 1417f Hepatic cysts, 980, 981f Hepatic duct strictures, benign, 1044, 1044f Hepatic encephalopathy, 977, 979t in cirrhosis, 1027-1028, 1030-1031 diagnosis of, 1028 EEG findings, 2342 physical findings in, 979 treatment of, 1030-1031 Hepatic enzymes, 989-990

I47

Hepatic masses approach to, 979t, 980-982 evaluation of, 980, 981f rare malignant neoplasms, 1034-1035 solid masses, 980-982, 982f Hepatitis alcoholic clinical manifestations of, 1020 liver transplantation for, 1034 in aplastic anemia, 1115 autoimmune, 1006 arthritic manifestations of, 1823 diagnosis of, 1000t liver transplantation for, 1035 cholestatic, 1009 cryptogenic, 1000t cytomegalovirus diagnosis of, 1037 liver histology of, 1037.e1f lupoid, 1773 non-1 to non-E, 1000 viral acute, 993-1000 chronic, 1000-1006 distinguishing characteristics of, 2252t-2253t in pregnancy, 1623 transmission-transmitted, 1197-1198 Hepatitis A, 994-995 acute, 995, 995f clinical manifestations of, 994 definition of, 994 diagnosis of, 995 epidemiology of, 994 immunization against, 67f-68f, 67.e1t-67.e3t ischemic, 959 pathobiology of, 994 prevention of, 995 prognosis for, 995 serologic course, 995, 995f travel-related risk, 1881 treatment of, 995b Hepatitis A vaccine, 68t-73t, 73-74, 1881-1882, 1883t adverse reactions to, 74 combined hepatitis A and hepatitis B vaccine, 1883t indications for, 68f, 73-74 Hepatitis A virus, 994 Hepatitis B acute, 995-997 anti-HBc IgM, 996, 996f antiviral therapy for, 2177-2179, 2177t chronic, 15, 1001-1004, 2177t, 2178 coinfection with HIV, 2335 antiretroviral therapy for, 2291 antiviral therapy for, 2179 in immune reconstitution inflammatory syndrome, 2335 immunization against, 67f-68f, 67.e1t-67.e3t in pregnancy, 1623 prevention of, in neonates, 1623 screening for, in pregnancy, 1623 Hepatitis B encephalitis, 2058t Hepatitis B surface antigen (HBsAg), 68t-73t, 996, 996f Hepatitis B vaccine, 68t-73t, 74, 996, 1882, 1883t adverse reactions to, 74 combined hepatitis A and hepatitis B vaccine, 1882, 1883t indications for, 68f, 74 Hepatitis B virus, 995 genotypes, 996 marker kinetics, 996, 996f Hepatitis C acute, 997-998 antiviral therapy for, 2177t, 2179-2181 chronic, 1004-1005 antiviral therapy for, 2177t HFE mutation analysis in, 1423.e1f mutation analysis in, 1423 coinfection with HIV, 2335 antiretroviral therapy for, 2291 antiviral therapy for, 2181 end-stage disease, 1005 in glomerular syndromes, 735 HFE mutation analysis in, 1423.e1f

Volume I  pp 1-1382  •  Volume II  pp 1383-2722

I48

Index

Hepatitis C (Continued) in immune reconstitution inflammatory syndrome, 2335 in pregnancy, 1623, 1623b screening for, 56 Hepatitis C virus, 997 anti-HCV antibodies, 998, 998f, 998t markers, 998, 998f, 998t and non-Hodgkin’s lymphoma, 1258 Hepatitis D or delta acute, 998-999 chronic, 1005 Hepatitis D virus, 998 Hepatitis E acute, 999 chronic, 1005 Hepatitis E virus, 999 Hepatitis G virus, 1176 Hepatobiliary cancer, 1339-1345 clinical manifestations of, 1342 definition of, 1339 diagnosis of, 1342 end-of-life care in, 1344-1345 epidemiology of, 1340 global incidence of, 1339-1340, 1341f prevention of, 1345 primary, 1340t prognosis for, 1345 staging, 1342 treatment of, 1343b-1344b Hepatobiliary disease, 987-991 diagnosis of, 989-991 in HIV infection, 2305 Hepatocellular adenoma, benign, 1608 Hepatocellular carcinoma, 981-982, 1339, 1340f causative factors, 1228 clinical manifestations of, 1342 diagnosis of, 1342, 1343f early stages, 1344 epidemiology of, 1339-1340 global incidence of, 1339-1340, 1341f histology of, 1341f in HIV infection, 2322, 2323t imaging of, 1342, 1343f liver transplantation for, 1034 pathobiology of, 1340-1341 prevention of, 1345 prognosis for, 1345 risk factors for, 1341t staging, 1342 surveillance, 870, 870f trans-arterial chemoembolization of, 1344, 1344f treatment of, 1343-1344 Hepatocellular reactions, 1008-1010, 1009t Hepatomegaly abdominal examination in, 978 diagnostic approach to, 979f in HIV infection, 2303t Hepatopulmonary syndrome, 1025, 1027 in cirrhosis, 1028 diagnostic criteria for, 1028 liver transplantation for, 1033.e1t treatment of, 1031 Hepatorenal syndrome, 1025 treatment of, 1030 type 1, 1026 type 2, 1026 Hepatosplenic candidiasis, 1012-1013, 2080, 2081f clinical manifestations of, 1013 diagnosis of, 1013 epidemiology of, 1012-1013 pathobiology of, 1012-1013 prognosis for, 1013 treatment of, 1013b Hepatosplenic T-cell lymphoma, 1266 Hepatosplenomegaly, 1852 Hepatotoxic agents, 1007-1008 Hepatotoxicity, 706t-710t, 1203. See also Liver disease, drug-induced Hepcidin, 1069, 1420 Heptavalent botulinum antitoxin (HBAT), 1928 HER2/neu, 1355-1356 Herbal medicine, 181, 182t Chinese, 17 perioperative, 2613t Herbicides, 81, 81t Herceptin (trastuzumab), 171, 1211t-1216t

Hereditary angioedema, 1697-1698, 1740t acute attacks, 1697 clinical manifestations of, 1697 definition of, 1697 diagnosis of, 1697 epidemiology of, 1697 pathobiology of, 1697 prognosis for, 1697 prophylaxis of, 1697 treatment of, 1697b future directions, 1697 long-term, 1697 Hereditary cancer risk syndromes, 1226, 1227t Hereditary cerebellar ataxias, 2469-2470 Hereditary coproporphyria, 1408t classification of, 1408 diagnosis of, 1413 epidemiology of, 1408 etiology of, 1410 skin disease in, 1412 Hereditary deafness, 2594 Hereditary elliptocytosis, 1054f, 1083-1084 clinical manifestations of, 1084 definition of, 1083 epidemiology of, 1083 pathobiology of, 1083-1084 treatment of, 1084b Hereditary fructose intolerance, 823t Hereditary hemochromatosis, 335, 1418-1419, 1420t evaluation of, 1422f laboratory findings, 1420-1421, 1421t liver disease in, 982 phlebotomy treatment for, 1421, 1422t physical findings, 1420-1421, 1420t population screening for, 1422 stages of, 1419 symptoms of, 1420-1421, 1420t treatment of, 1421 Hereditary hemophilia, 1173-1177 definition of, 1173 epidemiology of, 1173 Hereditary hemorrhagic telangiectasia, 958, 1172, 1172f Hereditary hyperparathyroidism and jaw tumors, 1652t Hereditary hypomagnesemia, 929 Hereditary motor neuropathy, 2524t Hereditary multiple exostoses, 1672 Hereditary nephropathies, 822-824, 823t Hereditary neuropathy with liability to pressure palsies (HNPP), 194, 2529 Hereditary neutrophilia, 1131 Hereditary nonpolyposis colorectal cancer, 1324 clinical manifestations of, 1324 diagnosis of, 1324 general features of, 1323t pathobiology of, 1324 treatment of, 1324b Hereditary optic neuropathy, Leber’s, 2546 Hereditary paraganglioma, 1227t Hereditary persistence of fetal hemoglobin classification of, 1089t clinical manifestations of, 1093 molecular basis, 1090 Hereditary platelet bleeding disorders, 1171b Hereditary platelet hypofunctional disorders, 1171 Hereditary pyropoikilocytosis, 1054f, 1084 diagnosis of, 1084 treatment of, 1084b Hereditary recurrent fever syndromes, 1739-1742, 1740t-1741t Hereditary renal hypouricemia, 1386t Hereditary retinoblastoma, 1227t Hereditary sensory and autonomic neuropathy, 2519 Hereditary sideroblastic anemia, 1072 Hereditary spastic paraplegias, 2470 clinical features of, 2470, 2524t diagnosis of, 2470 prognosis for, 2470 treatment of, 2470b Hereditary spherocytosis, 1053f, 1081-1083, 1386t clinical manifestations of, 1081 definition of, 1081 diagnosis of, 1081-1083 epidemiology of, 1081

Hereditary spherocytosis (Continued) laboratory findings, 1082-1083, 1082f-1083f pathobiology of, 1081 peripheral blood smear features of, 1082, 1082f surgical management of, 1083 treatment of, 1083b Hereditary stomatocytosis dehydrated, 1084 overhydrated, 1084 syndromes, 1084 variants, 1084 Hereditary vascular hemorrhagic disorders, 1172 Heredity: units of, 189 Heredodegenerative conditions, 2574 Heritability, 196 Hermansky-Pudlak syndrome, 587, 1133t, 1134, 1171 Hermaphroditism, true, 1566 Hernias, 945 cerebral, 2410, 2412t epigastric, 945 groin, 945 hiatal, 906 incisional, 945 pelvic, 945 umbilical, 945 Herniated nucleus pulposus low back pain in, 2375t neck pain in, 2374t Heroin toxicity, 699t-702t Herpangina, 2241t, 2242 clinical manifestations of, 2242 diagnosis of, 2242 oral ulcers of, 2580t Herpes antiviral therapy for, 2181-2183, 2181t genital, 1877t, 1878, 2225 AIDS-associated, 2297t-2301t antiviral therapy for, 2181t prevalence of, 1877 treatment of, 2297t-2301t mucocutaneous AIDS-associated, 2297t-2301t antiviral therapy for, 2181t treatment of, 2297t-2301t neonatal, 2181t orolabial AIDS-associated, 2297t-2301t antiviral therapy for, 2181t treatment of, 2297t-2301t Herpes gestationis, 2675, 2676f Herpes labialis, 2225, 2225f Herpes simiae, 2060t Herpes simplex conjunctivitis, 2564t Herpes simplex encephalitis, 1687t, 2226, 2226f, 2502-2504 clinical manifestations of, 2226, 2502, 2503f diagnosis of, 2226, 2502 epidemiology of, 2502 MRI findings, 2502, 2503f pathobiology of, 2502 prognosis for, 2226b tests for, 2502t treatment of, 2226, 2226b, 2502b Herpes simplex keratitis, 2565, 2565f Herpes simplex virus, 2223-2224 latency and reactivation, 2225, 2225f replication of, 2224, 2224f structure of, 2224, 2224f transmission of, 2224, 2225f Herpes simplex virus infection, 2223-2227, 2679 acyclovir-resistant, 2181t, 2297t-2301t antiviral therapy for, 2181t chronic, 2318-2319, 2318f, 2652f clinical manifestations of, 2225, 2679 cutaneous manifestations of, 2225, 2650f, 2652f definition of, 2223-2224 diagnosis of, 2226, 2679 disseminated, 2655f distinguishing characteristics of, 2252t-2253t epidemiologic features of, 1869t, 2224 esophageal, 906-907 gene therapy for, 2226 in HIV infection, 2295, 2297t-2301t, 2318-2319, 2318f

Herpes simplex virus infection (Continued) in immune reconstitution inflammatory syndrome, 2335 in immunocompromised hosts, 2225-2226 in meningitis, 2490 neonatal, 2226-2227 oral ulcers of, 2580, 2580f, 2580t p, 2224-2225 pathobiology of, 2224-2225, 2225f, 2679 prevention of, 2226 recurrent, 2580, 2580f, 2580t treatment of, 2226b, 2679b Herpes simplex virus type 2 meningitis clinical manifestations of, 2491 epidemiology of, 2490 Herpes zoster, 249t, 2680, 2680f antiviral therapy for, 2181t clinical manifestations of, 2680 disseminated, 2655f in HIV infection, 2297t-2301t, 2318 neuropathies associated with, 2536 oral ulcers associated with, 2580-2581 pathobiology of, 2680 treatment of, 2297t-2301t, 2680b vaccination against indications for, 68f recommended adult schedule, 67f-68f, 67.e1t-67.e3t Herpes zoster ophthalmicus, 2565 Herpes zoster oticus, 2537 Herpetic keratitis, 2225 Heterochromia iridis, 2563 Heterophyes, 2157 Heteroplasmy, 195 Heterotopia band (double cortex), 2512 nodular, 2512 Heterotopic gastric mucosa (inlet patch), 908 Heterozygosity, 196-197 Hexosamine pathway flux, increased, 1544 Hexose monophosphate shunt, 1085-1088, 1085f HFE gene, 1418-1419 C282Y homozygotes, 1422, 1423t in liver disease, 1422-1423, 1423.e1f HHS. See Hyperosmolar hyperglycemic syndrome; United States Department of Health and Human Services Hiatal hernia, 906 Hidradenitis suppurativa, 2681-2682, 2682f clinical manifestations of, 2681-2682 diagnosis of, 2681-2682 epidemiology of, 2681 pathobiology of, 2681 regional involvement, 2701f treatment of, 2682b Hierarchical data, 36 High endothelial venules, 225 High-altitude cerebral edema, 597 High-altitude diseases, 596-597, 596t clinical manifestations of, 596 definition of, 596 diagnosis of, 596 epidemiology of, 596 pathobiology of, 596 prevention of, 596-597 prognosis for, 597 High-altitude pulmonary edema prevention of, 596-597 treatment of, 597 High-density lipoprotein, 1389t, 1392, 1392f High-efficiency postdilution online hemofiltration, 841 Higher mental functions, 2382-2388 High-frequency ventilation, 665-666 Highly pathogenic avian influenza viruses, 2056 Highly-active antiretroviral therapy. See also Antiretroviral therapy for Pneumocystis pneumonia, 2097 response to, 2280 High-risk populations, 55t Hilar cholangiocarcinoma, 1339, 1340f Hilar lymphadenopathy, 291f Hip(s), 1751f Hip disorders, 1750t, 1753 Hip fractures, 1637, 1638f Hip replacement anesthetic approaches for, 2618t total, 1830, 1830f Hippocampal sclerosis, 2406 Hippocampus imaging, 2383, 2383f

Index Hippocratic Oath, 4 Hiroshima, 1240 Hirschsprung’s disease, 889-890 clinical manifestations of, 890 definition of, 889 diagnosis of, 890 pathobiology of, 889-890 treatment of, 890b Hirsutism, 1594, 2706-2707 Hirudin, 178 Hirugen, 178 Hirulog (bivalirudin), 178 His, Wilhelm, 1811 Hismanal (astemizole), 2660t Hispanics or Latinos chronic kidney disease in, 833-834 cystic fibrosis in, 562 heart failure in, 298 HIV/AIDS in, 2276 obesity in, 100 osteoporotic fractures in, 1637-1638 population, 15 type 2 diabetes mellitus in, 1532 Histamine, 232-233, 549 Histamine H2-receptor antagonists, 900t Histiocytic necrotizing lymphadenitis, 1267 Histiocytosis familial hemophagocytic lymphohistiocytosis syndromes, 1686t Langerhans cell, 2689-2690 infiltration of hypothalamus, 1477, 1477f pulmonary, 576, 586-587, 587b, 587.e1f multicentric reticulohistiocytosis, 1826 Histiocytosis X, 576, 2689-2690, 2690f Histo-blood group antigens (HBGAs), 2245 Histoplasma capsulatum, 2070 Histoplasmosis, 2058t, 2070-2072 clinical manifestations of, 2070-2071 definition of, 2070 diagnosis of, 2071 disseminated, 2070-2071 distinguishing characteristics of, 2252t-2253t epidemiology of, 2070 in HIV infection, 2295, 2297t-2301t, 2320 oral ulcers, 2580t pathobiology of, 2070 prognosis for, 2071 prophylaxis to prevent first episode in HIV infection, 2294, 2295t pulmonary acute, 2070 chronic, 2070 complications of, 2070 treatment of, 2071 small intestinal, 948 treatment of, 2071b Histrionic personality disorder, 2355t HIV. See Human immunodeficiency virus Hives, 1693-1697, 2683. See also Urticaria Hoarseness, 2606 Hodgkin’s lymphoma advanced-stage rates of progression, 1271t treatment of, 1272 bulky disease, 1270, 1271f classic, in HIV infection, 2322-2326, 2323t, 2327f clinical manifestations of, 1269 CMR findings, 291f, 1268-1273 complications of, 1272-1273 definition of, 1268 diagnosis of, 1269-1270, 1269f differential diagnosis of, 1269 EBV-associated, 1268, 2234 in elderly, 1273 epidemiology of, 1268 follow-up, 1272 genetic factors, 1268-1269 hematopoietic stem cell transplantation for, 1202 in HIV infection, 2324 imaging of, 1270, 1271f incidence of, 1268 laboratory testing in, 1270 limited-stage, 1268-1273 nodular sclerosing, 1269, 1269f pathobiology of, 1268-1269

Hodgkin’s lymphoma (Continued) physical findings, 1270 positron emission tomography in, 1270 in pregnancy, 1272-1273 refractory or relapsed, 1272, 1272t secondary neoplasms, 1272t staging, 1270, 1270f Ann Arbor system, 1270, 1271t evaluation for, 2324 tests required for, 1270t treatment of, 1270b-1273b future directions, 1273 late complications of, 1272 monitoring after, 1272t special problems in, 1272-1273 treatment plan, 1271t types, 1268, 1269t Holiday heart, 362 Holmes-Adie syndrome, 2519 Holocarboxylase synthetase deficiency, 1387t Holotranscobalamin, 1112 Holter monitors, 348-349 Holt-Oram syndrome, 342 diagnosis of, 408 genetic determinants of, 405 Home blood pressure monitoring, 383-384 Home Health Compare (CMS), 43.e1t Homeopathy, 182t Homocysteine, 1403-1404 metabolism of, 1404-1405, 1404f inherited disorders of, 1404-1405, 1404f intracellular, 1188, 1188.e1f remethylation to methionine, 1404-1405 serum levels, 1111, 1111t Homocysteine-lowering therapy, 1109t Homocystinuria, 1386t, 1403-1407 adult-onset clinical presentation, 1405-1406 classic, 1404-1405 clinical features of, 1405-1406, 1406t definition of, 1403-1405 diagnosis of, 1406 genetic defects associated with, 1405t prevention of, 1407 prognosis for, 1407 treatment of, 1406b-1407b available options, 1407 therapeutic goals, 1406-1407 Homografts, 472t Honeycombing, 533, 534f, 580.e1f Hook effects, 1478, 1484 Hookworms, 2160-2161 clinical manifestations of, 2161 diagnosis of, 2161 pathobiology of, 2161 prevention of, 2161 treatment of, 2103, 2160t, 2161b Hordeolum (stye), 2561, 2561f Hormonal (steroidal) contraceptives, 1607-1609 Hormone deficiency, 1552, 1555, 1564-1565 Hormone insensitivity syndromes, 1564-1565 Hormone receptors cell surface, 1472-1473 function of, 1472-1473, 1472f types of, 1472, 1472f Hormone replacement therapy and acute STEMI, 455 for cancer, 1211t-1216t, 1218 guidelines for, 1627 for hypopituitarism, 1482, 1483t for menopausal hot flushes, 1628t-1629t perioperative, 2613t postmenopausal, 57, 1626-1627, 1626t stopping, 1627 thrombosis with, 1190 topical, 2660 Hormones anabolic, 1571-1572 bioidentical, 1626 circadian rhythms, 1475, 1475.e1f counter-regulatory, 1549-1550, 1550f definition of, 1471 in diabetic nephropathy, 804 ectopic, 1219-1220, 1220t endocrine action, 1471 excess production of, 1550 exogenous, 1225 hypophysiotropic, 1473-1475

Hormones (Continued) mechanism of action, 1472-1473 metabolism of, 1472 paracrine action, 1471 regulation of, 1471-1472, 1471f synthesis of categories of, 1471 regulation of, 1471-1472 transport of, 1472 Horner’s syndrome, 2518-2519, 2576-2577, 2577f, 2577t Horseshore kidney, 826f Hospice, 10, 14 Hospital Compare (CMS), 43.e1t Hospital diarrhea, nosocomial, 920 Hospital rapid response service criteria for mobilizing, 2608, 2609t rapid response teams, 2608 Hospital transfusion committees, 1191, 1192f Hospital transfusion services, 1191, 1192f Hospital-acquired condition (HAC) program (CMS), 1868.e1 Hospital-acquired pneumonia, 618-620 staphylococcal, 1899-1900 treatment of, 619b-620b, 619t, 689t Hospital-associated fever, 1850, 1851t Hospitalism, 2 Hospitalization abdominal pain during, 858 for cardiovascular disease, 258 malnutrition during, 1440 for pneumonia, 615 for ST segment elevation acute MI, 449t Hospitals general, 2626t interfacility collaboration, 1868 Host defense innate immune system in, 216-220 role of innate immune system in reactions, 219 Host-pathogen interactions, 1837-1838 Hot flashes, 1626 Hot flushes alternative therapy for, 1625 behavioral therapy for, 1625 differential diagnosis of, 1625, 1625t medical therapy for, 1625-1627 during menopause, 1624, 1624f nonestrogen drugs for, 1627, 1628t-1629t pathobiology of, 1624-1625 treatment of, 1625 Hot potato voice, 1299 Hot tub folliculitis, 2697 Housemaid’s knee, 1753 Howell-Jolly bodies, 1052f, 1059t, 1127f, 1140, 1141f HPV. See Human papillomavirus Human African trypanosomiasis, 2113 acute stages, 2252t-2253t clinical manifestations of, 2114 diagnosis of, 2114-2115 drugs for, 2115t geographic distribution, 2113-2114, 2113f prevention of, 2115-2116 treatment of, 2107, 2115b vectors, 2172t Human chorionic gonadotropin, 1595-1596 Human genome, 189 structure of, 189, 190t variation in characteristics of, 196 inherited, 196-197 Human Genome Diversity, 189-190 Human Genome Project (HGP), 189, 192, 200 Human granulocytic anaplasmosis, 2057t Human granulocytic ehrlichiosis, 2053-2054, 2053t blood smear features of, 2054, 2054f clinical manifestations of, 2054 diagnosis of, 2054, 2054f epidemiology of, 2053-2054 geographic distribution of, 2053-2054, 2054f treatment of, 2054b Human herpesvirus 8 (HHV-8), 1258, 2334 Human immunodeficiency virus (HIV) biology of, 2280-2285 classification of, 2280-2281

I49

Human immunodeficiency virus (HIV) (Continued) effects on pulmonary immunity, 2305-2306, 2306f gene products, 2283 genome function, 2283, 2281.e1f genome replication, 2283, 2281.e1f genome structure and organization, 2281, 2281.e1f life cycle, 2289, 2289f membrane fusion, 2282, 2282f molecular biology of, 2281-2284 nuclear transport and integration, 2283 origin of, 2280-2281 post-transcriptional processing, 2283 replication of, 2282-2285 attachment phase, 2282, 2282f early events, 2282-2283, 2282f effects on immune system, 2279, 2280f late events, 2283-2284, 2284f reverse transcription of, 2282f, 2283 structure of, 2281-2284 transcription of, 2283, 2284f translation of, 2283 translation of mRNA, 2283-2284, 2284f transmission of to health care workers, 2286-2287 in injection drug users, 2286 modes of, 2285-2287 mother-to-child, 2291 risk for, 2285, 2285f sexual, 2285-2286 through blood products and other tissues, 2286 uncoating, 2282-2283, 2282f virion component transport and assembly, 2284, 2284f virion structure, 2281-2282, 2282f Human immunodeficiency virus (HIV)– associated nephropathy, 735, 793 Human immunodeficiency virus (HIV)– associated neurocognitive dysfunction (HAND), 2302 Human immunodeficiency virus (HIV) dementia clinical manifestations of, 2329 definition of, 2329 diagnosis of, 2329 epidemiology of, 2329 histopathology of, 2329, 2330f MRI findings, 2331f pathobiology of, 2329 prognosis for, 2331 treatment of, 2330, 2331f Human immunodeficiency virus (HIV) entry inhibitors (EIs), 2288t, 2289 Human immunodeficiency virus (HIV) infection, 1837 acute, 2291 in adolescents, 64 antiretroviral therapy for, 2280, 2287-2292, 2294 autonomic disturbances in, 2520 cancer in, 2322-2327 clinical effects of, 2280 coinfection with hepatitis B virus, 2335 antiretroviral therapy for, 2291 antiviral therapy for, 2179 coinfection with hepatitis C virus, 2335 antiretroviral therapy for, 2291 antiviral therapy for, 2181 coinfection with L. donovani, 2123 coinfection with L. infantum, 2123 coinfection with visceral leishmaniasis, 2122 concomitant with reactive arthritis, 1767-1768 concomitant with tuberculosis, 2038 cure of, 2292 diagnosis of, 2276-2278 diagnostic testing for, 2278, 2278f distinguishing characteristics of, 2252t-2253t early evasion of intracellular immunity, 2283 facial lipoatrophy in, 2322, 2322f gastrointestinal manifestations of, 2302-2305 global statistics, 2272, 2273f, 2273t global trends, 2272, 2274f Volume I  pp 1-1382  •  Volume II  pp 1383-2722

I50

Index

Human immunodeficiency virus (HIV) infection (Continued) guidelines for perinatal treatment of, 2291 Guidelines for the Prevention and Treatment of Opportunistic Infections in HIV-Infected Adults and Adolescents, 2294 hematologic abnormalities in, 2327-2328 immune reconstitution inflammatory syndrome in, 2332-2335 immunopathogenesis of, 2278-2280 infectious complications of, 2292-2295, 2318-2320 clinical manifestations of, 2293-2294 diagnosis of, 2294 empirical management of, 2294 guidelines for, 2294-2295 pathobiology of, 2293 prevention of, 2295 treatment of, 2294b inflammatory manifestations of, 2321 intensive care for, 2317-2318 laboratory assays for, 2277-2278 and leprosy, 2045 likelihood for development of AIDS in, 2293, 2293f liver transplantation for, 1035 malabsorption in, 931-932 malaria and, 2108 metabolic complications of, 2295-2302, 2322 molecularly guided therapeutics for, 202.e1t natural history of, 2278-2279, 2279f neurologic complications of, 2328-2332, 2328t neuropathies associated with, 2536 oncology in, 2322-2328 opportunistic infections in guidelines for prevention and treatment of, 2294 prophylaxis of first episode of, 2295t oral lesions, 2579, 2580t papular pruritic eruptions of, 2321 pathobiology of, 2305-2306 prevention of, 2285-2287 antiretroviral therapy for, 2291-2292 interventions for infected individuals, 2287 in medical care settings, 2287 opportunities for, 2285, 2285f pre-exposure prophylaxis (PrEP), 2292 recommendations for, 2287 strategies for, 2285-2287 primary, 2278-2279 prognosis for, 2289 pulmonary manifestations of, 2305-2318 diagnostic approach to, 2307f-2308f evaluation of, 2306 risk of, 2306, 2310f rapid testing for, 2277 regional epidemics, 2274-2275, 2272.e1 risk for acquisition of, 2285 risk for transmission of, 2285, 2285f screening for, 56, 2276-2277 and sexually transmitted infections, 1877 skin manifestations of, 2318-2322 specific immune response to, 2279 syphilis interactions, 2016 testicular hypogonadism due to, 1573 treatment of, 2328 and tuberculosis, 2031 in women, 1603-1604 Human immunodeficiency virus (HIV) integrase, 2288-2289 Human immunodeficiency virus (HIV) integrase inhibitors (INSTIs), 2288t Human immunodeficiency virus (HIV) meningitis, 2491 Human immunodeficiency virus (HIV) myelopathy, 2331, 2332t Human immunodeficiency virus (HIV) myopathy, 2332 Human immunodeficiency virus (HIV) peripheral neuropathy, 2331-2332 Human immunodeficiency virus (HIV) protease, 2289 Human immunodeficiency virus (HIV) reverse transcriptase, 2288-2289 Human immunodeficiency virus (HIV) seroconversion exanthem, 2318

Human immunodeficiency virus (HIV) tubulointerstitial nephropathy, 797 Human immunodeficiency virus type 1 (HIV-1) phylogenetics of, 2281, 2281f subtypes, 2281, 2281f Human immunodeficiency virus type 1 (HIV-1) infection. See Human immunodeficiency virus infection Human immunodeficiency virus type 2 (HIV-2), 2281 Human leukocyte antigens (HLAs), 236-237 associations with rheumatoid arthritis, 1755, 1755.e1t class I, 237, 237.e1f, 236.e1f, 236.e1t class II, 237, 237.e1f, 236.e1t DRB1, 237, 237.e1t Human microbiome, 1838-1843 Human Microbiome Project, 202 Human monocytic ehrlichiosis American, 2053, 2053t diagnosis of, 2053 epidemiology of, 2053 geographic distribution of, 2054f treatment of, 2053 vectors, 2172t Human papillomavirus (HPV), 2219 Human papillomavirus (HPV) infection, 1878-1879 in adolescents, 64 anorectal, 972-973 clinical manifestations of, 2220-2221 cutaneous, 2220 diagnosis of, 2221 epidemiology of, 972, 2219-2220 esophageal, 907 genital, 2220 clinical manifestations of, 2220-2221 natural history of, 2223, 2223f in HIV infection, 2319 natural history of, 2223, 2223f oral clinical manifestations of, 2220-2221 prevalence of, 2220 pathobiology of, 972, 2220 prevention of, 2222-2223 primary, 2222 secondary, 2222-2223 screening for, 2222-2223 treatment of, 2221b-2222b Human papillomavirus oropharyngeal cancer (HPVOPC), 1298-1299 Human papillomavirus (HPV) testing, 2221-2223 Human papillomavirus (HPV) vaccines, 74, 2222 adverse reactions to, 74 agents, 68t-73t indications for, 68f recommendations for, 67f-68f, 2222, 2222t, 67.e1t-67.e3t Human parainfluenza virus (hPIV), 2188 Human recombinant IL-2 (aldesleukin), 1211t-1216t Human T-lymphotrophic virus (HTLV), 2235-2236 associated conditions, 2237t, 2239 epidemiology of, 2235-2236 guidelines for prevention of infection, 2239 life cycle, 2236 transmission of, 2235-2236 Human T-lymphotrophic virus (HTLV)– associated diseases, 2237t Human T-lymphotrophic virus (HTLV)– associated myelopathy/tropical spastic paraparesis, 2237t, 2238-2239 clinical manifestations of, 2238 diagnosis of, 2238 epidemiology of, 2238 pathobiology of, 2238 pathogenesis of, 2236 prognosis for, 2239 treatment of, 2238b-2239b Human T-lymphotrophic virus type 1 (HTLV-1), 1258, 2235-2236 associated conditions, 2237t, 2239 epidemiology of, 2235-2236 guidelines for prevention of infection, 2239 life cycle, 2236 transmission of, 2235t

Human T-lymphotrophic virus type 1 (HTLV-1) (Continued) parenteral, 2236 perinatal, 2235 sexual, 2235 Human T-lymphotrophic virus type 1 (HTLV-1)–associated myelopathy/ tropical spastic paraparesis, 2236, 2237t, 2238-2239 Human T-lymphotrophic virus type 2 (HTLV-2), 2235 associated conditions, 2237t, 2238-2239 epidemiology of, 2235 guidelines for prevention of infection, 2239 life cycle, 2236 transmission of, 2235t parenteral, 2236 perinatal, 2235 sexual, 2235 Human T-lymphotrophic virus type 3 (HTLV-3), 2235-2236 Human T-lymphotrophic virus type 4 (HTLV-4), 2235-2236 Humans, 1972 Humate-P (CSL Behring), 1170 Humidifier fever occupational causes, 594t suggestive features, 589t Humira (adalimumab), 169, 940, 1762 Hunger, 912, 1460 Hunter’s syndrome, 1387, 1399, 1734t clinical manifestations of, 1733 pathobiology of, 1733 Hunt-Hess grading scale, 2447 for subarachnoid hemorrhage, 2447t Huntington’s disease, 187t, 2462-2463 clinical manifestations of, 2463 definition of, 2462 diagnosis of, 2463 epidemiology of, 2462 juvenile, 2463 pathobiology of, 2462-2463 prognosis for, 2463 treatment of, 2463b Hurler’s syndrome, 1387, 1734t clinical manifestations of, 1733 treatment of, 1734 Hyaline casts, 730-731, 732f Hyaluronan, 1731 Hycamtin (topotecan), 1211t-1216t Hydatid disease alveolar, 2153 cystic, 1014, 2151-2152 clinical manifestations of, 1014 diagnosis of, 1014 prognosis for, 1015 treatment of, 1015b Hydatidosis, cystic, 2151 Hydralazine combined with isosorbide dinitrate, 314, 306.e1t-306.e2t for heart failure, 314, 306.e1t-306.e2t for hyperadrenergic disorders, 2521-2522 for hypertension, 388t, 1615t in pregnancy, 1615t Hydration for hypercalcemia, 1654 for pancreatitis, 962 for rhabdomyolysis, 726 Hydrocalycosis, 826 Hydrocalyx, 826 Hydrocephalus in bacterial meningitis, 2481 normal-pressure, 2396 after subarachnoid hemorrhage, 2449 Hydrochlorothiazide (HCTZ), 748t for aortic dissection, 496 for hypertension, 388t, 391 for Meniere’s disease, 2600 for vertigo, 2601t Hydrocodone formulations, dosages, and pharmacologic information, 139t-140t for pleurodynia, 2243 toxicity, 699t-702t Hydrocortisone, 164t for adrenal hypoplasia, 1567 for colitis, 941-942 for Crohn’s disease, 942 for hypercalcemia, 1654 for hypopituitarism, 1483t

Hydrocortisone (Continued) for hypothyroidism, 2616 for idiopathic pulmonary fibrosis, 581 potency, 2658t replacement doses, 1520-1521 for septic shock, 690 topical, 2658t for viral hemorrhagic fever, 2255 Hydrocytes, 1084 Hydrofluoric acid toxicity, 706t-710t Hydrogen breath test, 925t, 926 Hydromorphone for erythromelalgia, 508 formulations, dosages, and pharmacologic information, 139t-140t for frostbite, 510-511 for pancreatitis, 962 for renal colic, 813 Hydromyelia, 2514 Hydronephrosis, 799, 802, 803f Hydrophiidae, 717 Hydropneumothorax, tension, 538f Hydrops fetalis, 1090, 2213, 2214t Hydroquinone, 2700 Hydroxocobalamin for cobalamin deficiency, 1407 for cyanide intoxication, 600, 706t-710t 3-Hydroxy-2-phosphonomethoxypropyl cytosine (HPMPC), 2231 γ-Hydroxybutyrate toxicity, 699t-702t Hydroxychloroquine (Plaquenil), 1761 caveats for, 1761t ocular effects of, 2573, 2573t for pemphigus, 2677 for porphyria cutanea tarda, 2678 for Q fever, 2056 for rheumatoid arthritis, 1761 for sarcoidosis, 607t for systemic lupus erythematosus, 1775 1α-Hydroxycholecalciferol (alfacalcidol), 1659 Hydroxyethyl starch, 680t 17α-Hydroxylase/17,20 lyase deficiency, 1562-1563 17β-Hydroxylase deficiency, 1563-1564 21-Hydroxylase deficiency, 1563, 1563f, 1568 3β-Hydroxysteroid dehydrogenase deficiency, 1563 17-Hydroxysteroid dehydrogenase 3 (17-ketosteroid reductase) deficiency, 1564 5-Hydroxytryptophan, 2466-2467 Hydroxyurea for essential thrombocythemia, 1125-1127 for hypereosinophilic syndrome, 336, 1154 for leukostasis before acute leukemia therapy, 1243 for polycythemia vera, 1125-1127 for primary myelofibrosis, 1128 for sickle cell anemia, 1102, 1102t for splenomegaly, 1125 Hydroxyzine (Atarax), 2660t for eosinophilic pustular folliculitis, 2681 for pruritus, 2636t for urticaria, 1695, 2684t Hymenolepis diminuta, 2148 Hymenolepis nana, 2147-2148, 2148t, 2149b Hymenoptera stings, 2176-2177 Hyoscyamine for diarrhea, 933 for diffuse esophageal spasm, 904 for sphincter of Oddi dysfunction, 1048 Hypalgesia, 2340 Hyperadrenergic disorders, 2521-2522 Hyperaldosteronism diagnosis of, 1517, 1518f glucocorticoid-remediable, 823t Hyperalgesia, 133-134, 2340, 2522 Hyperamagnesemia, 706t-710t Hyperammonemia, valproate-induced, 706t-710t Hyperandrogenism, 1519 Hyperbaric oxygen, 599, 706t-710t Hyperbilirubin, unconjugated, 986 Hyperbilirubinemia, 983-987 conjugated or mixed, 986-987 definition of, 983 diagnostic evaluation of, 988f differential diagnosis of, 986t pathobiology of, 983-985

Index Hyperbilirubinemia (Continued) postoperative, 2624 unconjugated, 984 Hypercalcemia, 1652-1653, 1652t autoimmune hypocalciuric, 1657 causes of, 1652-1653, 1653t, 1654f clinical features of, 1653, 1653t definition of, 1652 diagnosis of, 1653, 1654f disorders and syndromes associated with, 1657-1658 drugs that cause, 1658 endocrine causes of, 1657 familial benign, 1652t, 1657 familial hypocalciuric, 1657 infantile, 1652t, 1657 of malignancy, 1219-1220, 1657-1658 in multiple myeloma, 1281 nephrocalcinosis in, 798 paraneoplastic, 1339 pathobiology of, 1652-1653 pNET causing, 1335t treatment of, 1654b Hypercalciuria, 811, 823t Hypercapnia, 548f Hypercapnic diseases, 546t Hypercapnic-hypoxemic acute respiratory failure, 657t Hyperchloremic acidosis, 769-771 associated with hypokalemia, 769 associated with normal or increased potassium level, 769 causes of, 762, 762t of renal origin, 770-771 Hypercholesterolemia, familial, 1385t, 1393 Hypercoagulable states, 1185-1191 evaluation of, 1158-1159 inherited, 1158, 1158t, 1186t in malignant disease, 1188-1189 primary, 1185-1187, 1185t clinical manifestations of, 1186-1187 diagnosis of, 1187 long-term management of, 1188, 1188t treatment of, 1187b-1188b risk of VTE in, 1186, 1186t secondary, 1188-1191 Hyperdynamic circulatory state, 1023-1025 Hyperemesis gravidarum, 1622, 1622b Hypereosinophilia associated with other diseases, 1153b-1154b idiopathic, 1153 reactive, 1153b-1154b Hypereosinophilic syndrome, 335-336, 1132 classification of, 1151 clinical manifestations of, 336 definition of, 1151 diagnosis of, 336 idiopathic, 1153 prognosis for, 1154 treatment of, 336b, 1154 Hyperesthesia, 2340 Hyperexplexia, 2467 Hyperfibrinolysis, 1183 Hyperfiltration, 804 Hypergastrinemic syndromes, 911 Hyperglycemia, 1479 associated with dawn phenomenon, 1531 in diabetes, 804 fetal effects of exposure to, 1621-1622, 1621f neonatal effects of exposure to, 1621-1622, 1621f in septic shock, 690 states, 1539-1542 vascular complications of, 1543, 1543f Hypergonadism, hypothalamic, 1478 Hypergonadotropic amenorrhea, 1589-1591 causes of, 1591 diagnosis of, 1589 differential diagnosis of, 1589 genetic abnormalities that cause, 1590-1591 genetic alterations that cause, 1591 pathobiology of, 1589-1591 treatment of, 1591b-1592b Hyperhidrosis, 2521 Hyperhistaminic syndromes, 912 Hyperhomocysteinemia, 1188, 1403-1407 clinical manifestations of, 1405-1406 definition of, 1403-1405

Hyperhomocysteinemia (Continued) epidemiology of, 1405 pathobiology of, 1405 prevention of, 1407 prognosis for, 1407 treatment of, 1406b-1407b Hyperimmunoglobulin E syndrome, 1147t-1148t, 1148, 1683t, 1684-1685 clinical manifestations of, 1148, 1684-1685 definition of, 1684 diagnosis of, 1148, 1685 pathobiology of, 1148, 1684-1685 prognosis for, 1685 treatment of, 1148b, 1685b Hyperimmunoglobulin M syndrome, 1679-1680 treatment of, 1681 X-linked, 1680t Hyperimmunoglobulinemia D with periodic fever syndrome, 1740t-1741t, 1743 Hyperinflation, dynamic, 667 Hyperinsulinemic hypoglycemia, 1550-1552 congenital forms, 1551 persistent, of infancy, 1529 postprandial, 1551-1552 treatment of, 1555 Hyperkalemia, 758-759, 761 causes of, 761t definition of, 755 diagnosis of, 759-760 ECG manifestations of, 758-759, 759f hyperchloremic metabolic acidosis of renal origin associated with, 770-771 prognosis for, 762 treatment of, 706t-710t, 761-762 Hyperkalemic, hyperchloremic renal tubular acidosis, 770, 770t Hyperkalemic disorders, 760 Hyperkalemic hypertensive syndromes, 760 Hyperkalemic hypotensive or normotensive syndromes, 760 Hyperkalemic periodic paralysis clinical features of, 2542t familial, 762 Hyperkalemic renal tubular acidosis, 770t Hyperkinetic movement disorders, 2461, 2461t Hyperlipidemia, familial combined, 1393 Hyperlipoproteinemia, 1825 Hyperlucent lungs, 570 Hypermagnesemia, 776 clinical manifestations of, 776 mild, 776 prognosis for, 776 treatment of, 776b Hypermobility, 1713t Hypernatremia, 741, 753-754 causes of, 753t clinical manifestations of, 753-754 definition of, 753 diagnosis of, 754 epidemiology of, 753 hypervolemic, 753 hypovolemic, 753 normovolemic, 753 pathobiology of, 753 pathogenesis of, 743t treatment of, 754b Hyperopia, 2557-2558, 2557f, 2566-2567 Hyperosmolality, 1499-1500 Hyperosmolar hyperglycemic syndrome, 1539, 1542 diagnostic criteria for, 1540t hallmarks, 1542 management of, 1541f pathobiology of, 1539 precipitants, 1539t treatment of, 1542b Hyperostosis, 1668-1669 cortical, with syndactyly, 1669 disorders that cause, 1668t endosteal, 1669, 1669b focal, 1671 osteosclerosis with, 1670-1671 skeletal, diffuse idiopathic, 1816 Hyperoxaluria, 811 liver transplantation for, 1035 primary (type I), 1387t, 1033.e1t

Hyperparathyroidism, 1654-1656 in chronic kidney disease, 834-835, 835f definition of, 1654-1655 familial primary, 1656 hereditary, 1652t, 1656 and jaw tumors, 1652t, 1656 neonatal severe, 1652t, 1657 parathyroidectomy for, 1653t, 1655 primary, 1655-1656 clinical manifestations of, 1655 diagnosis of, 1655 epidemiology of, 1655 familial, 1656 neonatal severe, 1652t, 1657 pathobiology of, 1655 secondary, 834-835, 835f sporadic, 1652t treatment of, 1655b-1656b uremic, 1656 clinical manifestations of, 1656b-1657b treatment of, 1656b-1657b Hyperparathyroidism–jaw tumor syndrome, 1652t, 1656 Hyperphosphatemia, 777-778 causes of, 777t chronic, 777f clinical manifestations of, 777 medications for, 778t pathobiology of, 777 prognosis for, 778 treatment of, 778b Hyperpigmentation, 2699-2700 circumscribed (patchy), 2700 diffuse, 2699 linear, 2700 postinflammatory, 2700, 2700f reticulated, 2700 Hyperplastic eosinophilic sinusitis, 1691t Hyperpolarization-activated cyclic nucleotide gated (HCN) channels, 341 Hyperprolactinemia, 1487-1488, 1592 antipsychotic-induced, 2626t, 2628-2629 causes of, 1488 clinical manifestations of, 1488 diagnosis of, 1488 etiology of, 1478, 1479t hypothalamic, 1478 idiopathic, 1479 pathobiology of, 1487-1488 treatment of, 1488b Hypersecretory syndromes, 913-914 Hypersegmented neutrophils, 1057f Hypersensitivity drug-induced, in immunocompromised patients, 1859 visceral, 890 Hypersensitivity pneumonitis, 585, 590-592, 585.e1f acute, 589t chronic, 589t, 585.e1f classification of, 575t clinical features of, 585, 589t, 592 diagnosis of, 585, 592 epidemiology of, 585 occupational causes of, 590-591, 592t pathobiology of, 585, 591-592 treatment of, 585b-586b, 592b Hypersensitivity reactions, 227-230 during anesthesia, 2618 blistering, 2677-2678 delayed-type, 229-230, 2684, 2685f, 2685t drug rashes, 2684, 2685f, 2685t type I, 227-228, 227t, 228f type II, 227t, 228-229, 228f type III, 227t, 229, 229f type IV, 227t, 229-230, 229f types of, 227t Hypersensitivity vasculitis, 957, 1799 Hypersomnia, 2417-2418 diagnosis of, 2417-2418 idiopathic, 2418 treatment of, 2418b Hypersplenism, 1136, 1141-1142 Hypertension, 307 acute severe, 394-395 arterial, 381-397, 2441 behavioral determinants of, 383 cardiovascular risk factors, 385t chronic, 1612t in chronic kidney disease, 392, 835

I51

Hypertension (Continued) chronic thromboembolic, 403, 403.e1f clinical manifestations of, 383 combined systolic and diastolic hypertension, 382 with cyclosporine, 167 definition of, 258, 381, 384t in diabetes, 1546-1547 in diabetic nephropathy, 392 diagnosis of, 383-384 diastolic, 382 dietary factors, 1427t drug-resistant, 394 echocardiographic findings, 280t-281t epidemiology of, 381-382 in estrogen replacement, 393-394 genetic determinants of, 383 gestational, 394t global prevalence of, 21, 22f hyperkalemic hypertensive syndromes, 760 in hypervolemia, 747 hypokalemic hypertensive disorders, 759 initial evaluation for, 383 intracranial, 2362 management of, 2452t reduction of, 2368 in left ventricular hypertrophy, 392 lifestyle recommendations for, 387t management of, 387b-396b add-on drugs, 391 blood pressure goals, 391-396, 392t dietary guidelines for, 1428t-1429t first-line drugs, 387-391 lifestyle recommendations for, 387t preconception interventions for women, 1612t preferred medications for women, 1602t preferred oral antihypertensive drugs, 394t recommendations for, 390f, 392, 392t recommendations for prevention of stroke, 2444 strategies to optimize, 396t treatment guidelines, 393t masked, 384, 384.e1f Mendelian forms, 386, 386.e1f metabolic alkalosis of renal origin with volume expansion and, 772 metabolic syndrome that accompanies, 383 mineralocorticoid-induced, 386 in minority populations, 393 neurogenic causes of, 387 nocturnal, 384, 384.e1f nutritional influences, 1429 obesity-related, 1463 ocular symptoms of, 2569 office-only or white-coat, 384 with oral contraceptives, 393-394 pathobiology of, 382-383 perioperative, 395t portal in cirrhosis, 1023-1025, 1025f clinically significant, 1027 portopulmonary, 1025-1027 postoperative, 2623 in pregnancy, 394, 1611-1613, 1614t-1615t definition of, 1611 diagnosis of, 1611-1612 epidemiology of, 1611 pathobiology of, 1611 prognosis for, 1613, 1614t treatment of, 1613b preoperative assessment of, 2615 prevalence of, 21, 22f prevention of, 387b-396b primary, 382 with prior coronary events or stroke, 392-393 prognosis for, 396 pseudoresistant, 394, 394t pulmonary, 249t, 397-404 chronic thromboembolic, 625 complications of, 406-407 definition of, 1783 differential diagnosis of, 426 echocardiographic findings, 280t-281t HIV-associated, 2309t in sickle cell anemia, 1101, 1103, 1103t Volume I  pp 1-1382  •  Volume II  pp 1383-2722

I52

Index

Hypertension (Continued) in systemic sclerosis, 1783 treatment of, 1103, 1785 pulmonary venous, 403 renal parenchymal, 384-385 renovascular, 385-386 clinical manifestations of, 385 diagnosis of, 385-386 pathobiology of, 385 resistant, 394, 394t screening for, 54-55 secondary or identifiable, 382, 387-397 guide to evaluation of, 385t identification and treatment of, 384 severe, 394 acute, 394-395 in preeclampsia, 1615t treatment of, 1615t systolic combined with diastolic hypertension, 382 isolated, 382 in older adults, 392, 392f toxicant-induced, 697 white-coat, 383-384 in women, 1602t, 1612t Hypertension retinopathy, 2569, 2569f Hypertensive crisis, 395t Hypertensive emergency, 31, 394 conditions that define, 31 intravenous drugs for, 395-396, 396t recommended treatment of, 395t Hypertensive encephalopathy, 395t Hypertensive retinopathy, 394, 395f Hypertensive urgency, 394, 396 Hyperthermia definition of, 695 malignant, 2617-2618 clinical features of, 2542t toxic myopathies, 2546t treatment of, 726 therapeutic, 695 toxicant-induced, 697 treatment of, 30 Hyperthyroidism cardiomyopathy with, 330 mild, 1508 in pregnancy, 1508 subclinial, 1508 treatment of, 1508 Hyperthyroxinemia, euthyroid, 1502t Hypertonic saline for drowning, 596 for elevated ICP in acute liver failure, 1033 for intracranial hypertension, 2368 Hypertrichosis, 2707 Hypertriglyceridemia, familial, 1393 Hypertrophic cardiomyopathy, 320-327, 321t, 325.e1f apical, 325.e2f-325.e3f clinical manifestations of, 324 definition of, 320 diagnosis of, 324-325 diagnostic criteria for, 325, 325t diagnostic testing, 324-325 echocardiographic findings, 324-325, 324f epidemiology of, 320 family evaluation, 325 in first-degree relatives, 325t interventional therapy for, 325-326 management of, 326f medical therapy for, 325 pathobiology of, 320-321 pathology of, 320-321 pathophysiology of, 321-324 prognosis for, 326-327 progressive, 325.e4f symptom management, 325-326 treatment of heart failure due to, 318, 325b-326b ventricular tachycardia in, 372 Hypertrophic gastritis, 908 Hypertrophic lichen planus, 2667 Hypertrophic obstructive cardiomyopathy, 324-325, 324f Hypertrophic osteoarthropathy, 1221, 1221f, 1669, 1825, 1825f Hyperuricemia, 1811-1814 asymptomatic, 1813 causes of genetic, 1811, 1812t nongenetic, 1811, 1811t

Hyperuricemia (Continued) definition of, 1811 diagnosis of, 1814 due to urate overproduction, 1812-1813 due to urate underexcretion, 1812 epidemiology of, 1811 pathobiology of, 1811-1813 renal failure due to, 798 Hyperuricosuria, 811 Hyperventilation for elevated ICP in acute liver failure, 1033 for vertigo evaluation, 2599 Hyperviscosity syndrome, 1282-1283, 1283f, 2443t Hypervolemia, 741, 746-747 clinical manifestations of, 747 definition of, 746-747 diagnosis of, 747 epidemiology of, 746 pathobiology of, 746-747 pathogenesis of, 743t treatment of, 747b-749b Hypervolemic hyponatremia, 750-751 Hypesthesia, 2340 Hyphema, 2563, 2563f Hypnosis, 182t Hypnotic drugs, 2352t Hypoalbuminemia, postoperative, 2624-2625 Hypoalbuminemic malnutrition, 1433 Hypoaldosteronism, 1521 Hypobicarbonatemia, 745-746 Hypocalcemia, 1652t, 1658 acute, 1658 autosomal dominant type 1, 1652t, 1660 autosomal dominant type 2, 1652t, 1660 causes of, 1656t chronic, 1659 clinical manifestations of, 1658 definition of, 1658 diagnosis of, 1658, 1659f mild, 1658 pathobiology of, 1658 severe, 1658 treatment of, 706t-710t, 1658b-1659b Hypocalciuric hypercalcemia autoimmune, 1657 familial, 1657 Hypochromia, 1053f Hypochromic microcytic anemia, 1068, 1068f Hypocitraturia, 811 Hypocomplementemic urticarial vasculitis, 1798-1799 Hypocortisolism, secondary, 1489-1490 Hypodynamic variables abbreviations, 687t normal values, 687t Hypofunctional neurologic disorders, 2606 Hypogammaglobulinemia arthritic manifestations of, 1824 with B cells, 1679-1680 immunizations for adults, 66t Hypoglycemia, 1548-1555 causes of, 1550-1553 clinical approach to, 1550 clinical manifestations of, 1550 counter-regulatory hormonal responses to, 1549-1550 definition of, 1548 in diabetes, 1538-1539 diagnosis of, 1550-1555 differential diagnosis of, 1551t due to defects in gluconeogenesis, 1552, 1555 due to defects in glycogen release/storage, 1552, 1555 due to defects of fatty acid oxidation, 1552-1553, 1555 due to disorders of carnitine metabolism, 1552-1553 due to excess production of hormones, 1550 due to glycogen storage disease, 1552, 1555 due to hormonal deficiencies, 1555 due to ketone body metabolism disorders, 1555 emergency management of, 1555 exercise testing for, 1555 fasting tests for, 1554 genetic testing for, 1555

Hypoglycemia (Continued) hyperinsulinemic, 1550-1552 congenital forms, 1551 persistent , of infancy, 1529 postprandial, 1551-1552 treatment of, 1555 iatrogenic, 1538, 1538t impaired awareness of, 1538 insulin factitious, 1552 investigations for, 1553-1555, 1553t mixed-meal test for, 1554 nocturnal, 1539 noninsulinoma pancreatogenous syndrome, 1552 non–islet cell tumor, 1550, 1552 pathobiology of, 1548-1550 prevention of, 1539b radiologic investigations for, 1554-1555 risk factors for, 1539, 1539t severe, 1539, 1539t sulfonylurea-induced, 706t-710t symptoms of, 1550 treatment of, 1539b, 1555b tumor-associated, 1220 Whipple’s triad, 1548 Hypoglycemia-associated autonomic failure, 1538 Hypogonadism in aging men, 1571, 1571f associated with systemic diseases, 1573 definition of, 1572 after hematopoietic stem cell transplantation, 1203-1204 hypogonadotropic, 1492-1493, 1565, 1574 causes of, 1572t isolated, 1592 hypothalamic, 1478 male, 1572-1574 clinical manifestations of, 1572-1573 definition of, 1572 diagnosis of, 1574 history in, 1572-1573 laboratory findings, 1573 physical findings, 1573 treatment of, 1574b-1575b primary, 1589-1591 testicular, 1573 Hypogonadotropic disorders, acquired, 1574 Hypogonadotropic hypogonadism, 1492-1493, 1565, 1574 causes of, 1572t clinical manifestations of, 1492-1493 congenital, 1574 diagnosis of, 1492-1493 isolated, 1592 secondary, 1492-1493 treatment of, 1493b Hypokalemia causes of, 760t clinical manifestations of, 758 definition of, 755 ECG manifestations of, 758, 758f hyperchloremic metabolic acidosis of nonrenal origin associated with, 769 prognosis for, 762 treatment of, 761-762 watery diarrhea, hypokalemia, and achlorhydria, 1335t, 1337, 1553 watery diarrhea, hypokalemic, hypochlorhydric acidosis, 769 Hypokalemic disorders, 759 Hypokalemic distal renal tubular acidosis, 770t Hypokalemic hypertensive disorders, 759 Hypokalemic hypotensive syndromes, 759-760 Hypokalemic periodic paralysis clinical features of, 2542t familial, 762 Hypokinetic disorders, 2461 Hypomagnesemia familial, with hypercalciuria, 823t hereditary, 929 isolated, 823t severe, 775 Hypomelanosis, circumscribed, 2699 Hypomimia, 2455 Hyponatremia, 741, 749-752 acute, 752 chronic, 752 in cirrhosis, 1026, 1030-1031 clinical manifestations of, 749

Hyponatremia (Continued) definition of, 749, 1026 diagnosis of, 749-752 diagnostic approach to, 750, 750f drug-induced, 2629 epidemiology of, 749 hypertonic, 749 hypervolemic, 750-751 hypotonic, 749 hypovolemic, 749, 751-752 hypovolemic hypotonic, 752 isotonic, 749 normovolemic, 751-753, 752f pathobiology of, 749 pathogenesis of, 743t pseudohyponatremia, 749 treatment of, 752b-753b, 752f, 1030-1031 Hypoparathyroidism, 1659-1661 acquired, 1659-1660 autoimmune acquired, 1660 polyglandular, 1652t, 1660 clinical features of, 1661t complex syndromes, 1652t, 1660-1661 congenital, 1652t definition of, 1659 familial syndromes, 1661.e1 inherited, 1660 isolated, 1652t, 1659-1660 mitochondrial disorders, 1660-1661 pathobiology of, 1659 polyglandular autoimmune, 1652t, 1660 Hypoparathyroidism, deafness, and renal anomalies syndrome, 1652t, 1660 Hypopharyngeal cancer, 1298 Hypopharynx, 2601, 2601f Hypophonia, 2455 Hypophosphatemia, 776-777 cardiomyopathy with, 330 causes of, 776t clinical manifestations of, 776-777 diagnosis of, 777 osteomalacia due to, 1649 pathobiology of, 776 prognosis for, 777 severe, 777 treatment of, 777b Hypophosphatemic rickets, familial, 1386t Hypophysiotropic hormones, 1473-1475 Hypophysitis, lymphocytic, 1482 Hypopigmentation, 2698-2699 acquired, 2699 circumscribed (patchy), 2699 congenital, 2699 guttate, 2699 linear, 2699 postinflammatory, 2699 Hypopituitarism, 1481-1482, 1592 acquired, 1552 causes of, 1481, 1482t diagnosis of, 1482b hormone replacement therapy in, 1483t pathobiology of, 1481-1482 treatment of, 1482b Hypopnea, 31, 638, 652 Hypopyon, neoplastic, 2572 Hyposplenism, 1127f Hypotension, 31 early-onset, 383 hyperkalemic hypotensive or normotensive syndromes, 760 hypokalemic hypotensive syndromes, 759-760 intracranial, 2362-2363, 2363f orthostatic antipsychotic-induced, 2626t detection of, 392, 392f permissive, 714 postoperative, 2623 postprandial, 392, 392f toxicant-induced, 697 treatment of, 2489 Hypothalamic amenorrhea, 1477-1478 Hypothalamic anorexia, 1479 Hypothalamic chronic anovulation, 1592 definition of, 1592 diagnosis of, 1592 treatment of, 1592b-1593b Hypothalamic hamartoma, 1476-1477 Hypothalamic hypergonadism, 1478 Hypothalamic hyperprolactinemia, 1478 Hypothalamic hypogonadism, 1478

Index Hypothalamic obesity, 1479 Hypothalamic-pituitary function, 1569 Hypothalamic-pituitary unit, 1480, 1480f, 1592 Hypothalamic-pituitary-gonadal axis, 1568-1569, 1568f Hypothalamic-pituitary-thyroid axis, 1471, 1471f Hypothalamus diseases of, 1475-1477 effects on other neurometabolic functions, 1479 effects on pituitary function, 1477-1479 etiology of, 1475, 1476t infiltrative disorders, 1477, 1477.e1f irradiation of, 1477 lesions of, 1520 neuroendocrine organization of, 1473, 1474f trauma to, 1477 tumors affecting, 1476-1477 Hypothermia categories of, 693 definition of, 693-694 stages, 694t therapeutic, 451, 695 toxicant-induced, 697 trauma, 695 treatment of, 30-31, 695t Hypothermic syndromes, 695 Hypothesis testing, 36-37 Hypothyroidism, 1502-1504 antipsychotic-induced, 2626t cardiomyopathy with, 330 causes of, 1502t, 1503 central, 1493, 1503 clinical manifestations of, 1503 complications of, 1504 definition of, 1502 diagnosis of, 1503-1504 epidemiology of, 1502 after Hodgkin’s lymphoma treatment, 1272t hypothalamic, 1479 laboratory assessment of, 1503-1504, 1503f mild, 1503-1504 nonthyroidal illness, 1505 pathobiology of, 1502-1503 preoperative, 2616 secondary, 1503 subclinical primary, 1503-1504 symptoms and signs of, 1503 test abnormalities in, 1503 treatment of, 1504b-1505b, 2616 Hypothyroxinemia, euthyroid, 1502t Hypotonic hyponatremia, hypovolemic, 752 Hypouricemia, hereditary renal, 1386t Hypoventilation approach to, 548f assessment of, 659 diagnosis of, 547 nocturnal, 629-630 during sleep, 2419 treatment of, 547b, 629-630, 2419 Hypoventilation syndromes, 545-548 acquired, 547 central congenital hypoventilation syndrome, 547 treatment of, 2419 Hypovolemia, 741-745 absolute, 743-744, 743t extrarenal causes of, 743t, 744 renal causes of, 743t, 744 treatment of, 745-746 clinical manifestations of, 744 definition of, 742-743 diagnosis of, 744-745 differential diagnosis of, 745 epidemiology of, 743-744 in gastrointestinal hemorrhage, 744 laboratory findings, 745 pathobiology of, 744 pathogenesis of, 743t relative, 743, 743t, 746 in sepsis, 744 treatment of, 745b-746b Hypovolemic hypernatremia, 753 Hypovolemic hyponatremia, 751-752 Hypovolemic hypotonic hyponatremia, 752

Hypovolemic shock, 673-674 definition of, 678f-679f etiology of, 678f-679f rapid ultrasound protocol, 675, 675t therapy for, 678f-679f Hypoxemia, 31 and alveolar-arterial oxygen gradient, 653, 653t assessment of, 659 etiologic mechanisms of, 657t life-threatening, 661 Hypoxemic respiratory failure, 600 Hypoxia, 600 diffuse injury, 2432 tissue, 686 Hysterical gait, 2341t

I

Ibandronate, 1643, 1643t Ibrutinib (Imbruvica), 1211t-1216t, 1256 Ibuprofen for acute mountain sickness, 596-597 for acute pericarditis, 486 for chronic prostatitis/chronic pelvic pain syndrome, 831-832 for ciguatera poisoning, 722 for frostbite, 695 for headache, 2357b, 2358, 2358t, 2360, 2368 for low back pain, 2375 for neck pain, 2375 for pericardial constriction, 490 for pleurodynia, 2243 for renal colic, 813 for snake bite, 718 for spinal stenosis, 2377 for sponge stings, 722 for venomous fish and stingray injuries, 721b Ibutilide, 363t-364t IC51 (Ixiaro, Jespect), 2266 Icatibant (Firazyr), 1697 I-cell disease, 1387, 1733, 1734t Iclusig (ponatinib), 171, 1211t-1216t Icterus, scleral, 977-978, 977f Idarubicin (Idamycin), 1211t-1216t, 1243t, 1244 Idelalisib, 1256 Identity, gender, 1566-1567 Ideomotor apraxia, 2385 Idiopathic cytopenias of undetermined significance (ICUS), 1233 Idiopathic interstitial pneumonia, 579-584 Idiopathic pulmonary capillaritis, 586 Idiopathic pulmonary fibrosis, 579-581 CT findings, 580, 580f, 580.e1f diagnosis of, 580-581, 580f, 580.e1f diagnostic criteria for, 581t pathobiology of, 575-576 prognosis for, 581 treatment of, 581b Idiopathic pulmonary hemosiderosis, 573 Idiopathic (immune) thrombocytopenic purpura, 1163-1164 clinical manifestations of, 1163 definition of, 1163 diagnosis of, 1164 distinguishing characteristics of, 2252t-2253t immunizations for adults, 66t initial management of, 1164t refractory, 1164t treatment of, 1164, 1164b, 1164t Idioventricular arrhythmias, accelerated, 451 Ifosfamide (Ifex) for cancer, 1211t-1216t, 1366 cardiomyopathy due to, 330 Igbo-ora virus, 2260 Ilaris (canakinumab), 169-170 Ileal disease, 928-929 Ileal function tests, 925t Ileitis, 937t Ileocecal syndrome, 1860-1861 Iliac artery endofibrosis, 504-505 Iliopectineal bursitis, 1753 Iliopsoas bursitis, 1753 Iliotibial band syndrome, 1753 Illness anxiety disorder, 2355t Iloperidone, 2354

Iloprost, 505, 509f Imaging. See also specific modalities; specific sites abdominal in acute pancreatitis, 962 in gallstones, 1040, 1040f in liver disease, 980 of brain, 2438-2439 of chest, 532-538 of chest wall, 531-538 diagnostic catheter angiography, 2440 in gastroenterology, 866-872 in epilepsy, 2404 functional endoscopic, 878-879, 878.e1f of lungs, 531-538 of mediastinum, 531-539 medical, 85 molecular, 1723 myocardial perfusion imaging, 285-286, 286f neuroimaging, 2345-2346, 2345t noninvasive cardiac, 282-292 and cardiovascular disease, 259 in osteoarthritis, 1748 pancreatobiliary, 873t, 877-878 pharmacologic stress imaging, 286 in pulmonary diseases, 531-539 in rheumatic diseases, 1723-1730 speckle tracking strain imaging, 276, 276.e1f of thyroid gland, 1501-1502 transluminal, 873t of ventricular function, 287 Imatinib (Gleevec or Glivec), 171, 1232t for cancer, 1211t-1216t for chronic myelogenous leukemia, 1248-1250, 1249t for hypereosinophilic syndrome, 336 mechanism of action, 1249, 1249f Imbruvica (ibrutinib), 1211t-1216t Iminoglycinuria, 1386t Imipenem for Acinetobacter infections, 1969 for anthrax exposure, 88b for melioidosis, 1967 for nocardiosis, 2064 for P. aeruginosa infection, 1966 for peritonitis, 949 for pneumonia, 616t, 618, 619t, 1966 for pyogenic liver abscess, 1012 recommended doses and schedules, 1891t-1892t for S. pneumoniae infection, 1905t for septic shock, 689t Imipramine for chronic pain, 138t for depression, 2349t for diffuse esophageal spasm, 904 for dyspepsia, 895 for headache prevention, 2358t for irritable bowel syndrome, 894 for urinary incontinence, 2238-2239 Imiquimod, 2184, 2660-2661 for AIDS-associated opportunistic infections, 2297t-2301t for genital warts, 2222t Immediate recall, 2383 Immigrants, 15, 15f Immobilization, 1191 Immortality, replicative, 1230 Immune adherence or C4b/C3b receptor (CR1, CD35), 243t Immune cells, 686 Immune complex rapidly progressive glomerulonephritis, 791 Immune complexes, 231, 1770-1771 Immune complex–mediated vascular injury, 1795 Immune complex–mediated vasculitis, 1798-1799 Immune cytopenia, drug-induced, 1704t Immune dysregulation and bronchiectasis, 567 syndromes, 1685-1686, 1686t clinical manifestations of, 1685-1686 diagnosis of, 1686 epidemiology of, 1685 pathobiology of, 1685-1686

I53

Immune dysregulation (Continued) prognosis for, 1686 treatment of, 1686b in systemic sclerosis, 1780 Immune dysregulation polyendocrinopathy enteropathy X-linked (IPEX) syndrome, 1074 Immune hemolytic anemia, drug-induced, 1078 Immune neutropenia, 1135-1136 Immune reconstitution inflammatory syndrome, 2329 BCG-associated, 2334, 2334f clinical manifestations of, 2333-2335 definition of, 2332-2333 diagnosis of, 2335 epidemiology of, 2333 examples, 2333t in HIV infection, 2297t-2301t, 2318, 2321, 2332-2335 MAC-associated, 2333-2334, 2334f P. jiroveci–associated, 2334, 2334f pathobiology of, 2333 prevention of, 2335 prognosis for, 2335 treatment of, 2077, 2335b Immune recovery uveitis, AIDS-associated, 2297t-2301t Immune response adaptive, 226-227 in autoimmune disease, 219-220 to cancer, 1230-1231 chronic activation of, 2279, 2280f costimulatory blockade of, 239 HIV-1–specific, 2279 innate, 219-220, 686 resolution of, 219 Immune system adaptive, 220-226 description of, 216 transition to, 219 age-related changes in, 108 evaluation of, 1677-1678, 1678t and HIV-1 replication, 2279, 2280f innate, 216-220 in autoimmune disease, 219-220 cells of, 216-217 description of, 216 effector mediators of, 218-219 localization of cells, 219 role in host defense reaction, 219 role in immune response resolution, 219 role in wound repair, 219 signaling pathways, 218-219 soluble products of, 219 triggers of, 217-218 and non-Hodgkin’s lymphoma, 1258 Immune (idiopathic) thrombocytopenic purpura, 1163-1164 distinguishing characteristics of, 2252t-2253t immunizations for adults, 66t treatment of, 1164, 1164b, 1164t Immune tolerance, 239-240 Immune-mediated myocarditis, 328 Immune-mediated necrotizing myopathy, 1789 classification of, 1790t diagnosis of, 1792 diagnostic criteria for, 1791t pathology of, 1790f, 1791 prognosis for, 1793 Immune-mediated thrombocytopenia, 1256 Immune-mediated tissue injury, 226-230 Immune-related neuropathies, 2529-2532, 2530t Immunity adaptive, 230, 686 autoimmunity, 225-226, 1780-1781 against Brucella, 1980 cellular defects, 1677, 1677t in systemic sclerosis, 1780 innate, 230, 1838 defects that lead to infections, 1686-1687, 1687b diseases of, 1687t intracellular, 2283 Volume I  pp 1-1382  •  Volume II  pp 1383-2722

I54

Index

Immunity (Continued) pulmonary, 2305-2306, 2306f specific, for HIV, 2279 Immunizations, 57, 65-78 active, 65 against rubella, 2206 against tetanus, 1930 adverse reactions to, 66 characteristics of, 65-73 against COPD, 560 general considerations, 66-67 for heart failure management, 315t for immunocompromised patients, 73 against influenza, 2195 passive, 65 for adults, 66t against rubella, 2206 against tetanus, 1930 against pertussis, 1992-1993 in pregnancy, 73 pretravel, 1882t recommendations for, 66-67, 67f, 67.e1t-67.e3t routine, 1881 against seasonal influenza, 2194t for travelers, 1881-1883 Immunoallergic reactions, 1009 Immunobiologics, 73-77 Immunocompromised patients, 1855-1856, 1855t acute toxoplasmosis in, 2129t, 2131 adenovirus diseases in, 2198 autoimmune disease in, 1855t, 1856 babesiosis in, 2144 collagen vascular disease in, 1855t cryptosporidiosis in, 2135 cutaneous syndromes in, 1858-1859, 1859t fever in, 1856-1858 gastrointestinal syndromes in, 1859t, 1860-1861 HSV infections in, 2225-2226 immunizations for, 68f, 73, 67.e1t-67.e3t infections in, 1858, 1859t, 1860-1861 malignant disease in, 1855, 1855t neurologic syndromes in, 1859t, 1861 parainfluenza virus disease in, 2189 respiratory syndromes in, 1859t, 1860 sinopulmonary syndromes in, 1859t, 1860 toxoplasmosis in, 2130-2131 reactivation of, 2132 treatment of, 2129t, 2131 transplant recipients, 1855-1856, 1855t Immunodeficiency acquired. See Acquired immunodeficiency syndrome (AIDS) anhidrotic ectodermal dysplasia with, 1687t antibody defects, 1679-1681 complement disorders, 1681-1682 hematopoietic stem cell transplantation for, 1202-1203 human immunodeficiency virus. See Human immunodeficiency virus (HIV) human T-lymphotrophic virus (HTLV) association, 2237t inherited, 211-212 phagocyte defects, 1682-1683 primary diseases, 1677, 1678t T- and B-cell combined defects, 1678-1679, 1678t in transplant recipients, 1855 visceral leishmaniasis in, 2123 well-defined defects with syndromic features, 1683-1685, 1683t Immunoelectrophoresis serum, 734 urine, 734 Immunoglobulin anti-D, 1164t for B19 parvovirus infection, 2214 botulism (BabyBIG), 1928 for complex regional pain syndrome, 2522 for IgG antibody defects, 1681 intravenous for chronic lymphocytic leukemia, 1256 for epidermolysis bullosa acquisita, 2676 for Guillain-Barré syndrome, 2530 for immune thrombocytopenic purpura, 1164t

Immunoglobulin (Continued) for inflammatory myopathies, 1792 for Lambert-Eaton myasthenic syndrome, 2552 for myasthenia gravis, 2551 for pemphigus, 2677 for warm autoimmune hemolytic anemia, 1077 modification of, 221 normal, 1680 serum, 1273 tetanus (TIG), 1929-1930, 1929t vaccinia (VIG), 2219 Immunoglobulin A, 1273 Immunoglobulin A deficiency, 1679-1680, 1680t Immunoglobulin A nephropathy, 788-789, 788b-789b, 789f Immunoglobulin A vasculitis, 1798 Immunoglobulin A with IgG subclass deficiency, 1680t Immunoglobulin D, 1273 Immunoglobulin E, 1273 activation of, 1688 allergen-specific, 1675-1676 Fc receptors for (FcεRs), 226 hyper-IgE syndrome, 1683t, 1684-1685 total, 1675-1676 Immunoglobulin G, 1273 Immunoglobulin G Fc receptors (FcγRs), 226, 1146 Immunoglobulin G subclass defects, 1680-1681, 1680t Immunoglobulin G4-related disease, 1267, 1826-1827 Immunoglobulin G4-related tubulointerstitial nephritis, 798 Immunoglobulin H-translocated MGUS, 1274 Immunoglobulin light chain amyloidosis, 1273, 1273t Immunoglobulin M, 1273 anti-HBc, 996, 996f hyper-IgM syndromes, 1679-1680 treatment of, 1681 X-linked, 1680t Immunohistochemical stains of unknown primary origin, 1379 Immunologic disease, 1676-1677 diagnosis of, 1676-1677 epidemiology of, 1676 history in, 1676-1677 key points, 1677t laboratory findings, 1677 physical findings, 1677 Immunologic modifiers, 1211t-1216t Immunologic neuropathy, peripheral, 2529-2532, 2530t Immunologic surveillance, 2632 Immunological fitness, 1837 Immunology, transplantation, 236-240 Immunomodulatory therapy cell therapy, 212-213 gene therapy, 212-213 for inflammatory bowel disease, 939t, 940 for myelodysplastic syndrome, 1237-1238 for Sjögren’s syndrome, 1789 topical, 2660-2661 transfusion effects, 1192 for viral hemorrhagic fever, 2255 Immunophilin-binding agents, 163t, 166-168 Immunoreceptor tyrosine-based activation motifs (ITAMs), 1146 Immunosuppressive drugs, 162-169 adverse reactions to, 168 for aplastic anemia, 1120 for cardiac transplantation, 521 with Chagas disease, 2117-2118 classification of, 163t for liver transplantation, 1037-1038, 1037t for lung transplantation, 646 for myelodysplastic syndrome, 1237 net state of, 1856 nonspecific, 239 perioperative, 1833 for pruritus, 2636t for renal transplantation, 844-845, 844t in sepsis, 686, 686.e2f for systemic lupus erythematosus, 1775-1776 for systemic sclerosis, 1784 topical, 2660

Immunosuppressive drugs (Continued) for warm autoimmune hemolytic anemia, 1078 Immunotherapy for cancer, 1218 for chronic sinusitis, 1692-1693 topical, 2660-2661 Imodium (loperamide), 893t Impaired micelle formation, 928 Impaired mixing, 927 Impetigo, 1898, 1907-1908, 2651f, 2695-2696, 2695f bullous, 2652f, 2678-2679, 2679f Impingement sign, 1751, 1752f Impingement syndrome, 1751 Implanon, 1609 Implantable cardioverter-defibrillators, 316-317, 377-378 for acute STEMI, 451 associated infections, 1865.e1t functions of, 378, 378f for heart failure, 306.e1t-306.e2t indications for, 377, 378t programming, 377-378 pulse generator and leads for, 377, 377f selection of, 454f Implantable loop recorders, 349 Implants, 285, 1831 Imprinting, 195 Imuran (azathioprine), 1761 In vitro allergy testing, 1676t In vitro diagnostic multianalyte index assays (IVDMIAs), 203 In vivo challenge tests, 1676 In vivo gene therapy, 211 Inborn errors of metabolism, 1384-1389 biochemical testing, 1387 characteristic signs of, 1385t classification of, 1384 clinical phenotype, 1384 definition of, 1384 diagnostic tests for, 1387-1388, 1388t and dietary requirements, 1453 disorders of membrane transport, 1386, 1386t disorders of protein metabolism, 1384-1386, 1385t-1386t disorders of the urea cycle, 1386 enzyme therapy for, 1388 epidemiology of, 1384 genetics of, 1384 history of, 1384 incidence of, 1385t molecular testing, 1387 next-generation sequencing, 1387-1388 nutritional therapy for, 1388-1389 pathobiology of, 1384 pathophysiologic mechanisms of, 1385t therapeutic strategies for, 1388t transplantation for, 1389 treatment of, 1388b-1389b vitamin therapy for, 1389 Inborn errors of peroxisomes, 1387t Incidence, 33 Incident reporting systems, 46 Incidentaloma, 1469 pituitary, 1494, 1494.e1f Incisional hernias, 945 Inclusion body myopathy, 1665t Inclusion body myositis, 1789 classification of, 1790t clinical features of, 1792, 2524t diagnosis of, 1792 diagnostic criteria for, 1791t epidemiology of, 1789 pathogenesis of, 1791 pathology of, 1790f, 1791 prognosis for, 1793 sporadic, 1789, 2237t treatment of, 1792-1793 Inclusion conjunctivitis, 2010 Incontinence fecal, 866, 970-971, 971f urinary. See Urinary incontinence Incretin-based therapy, 1536 Indacaterol (Arcapta, Onbrez), 553, 560t Indapamide, 388t, 815 Independent Payment Advisory Board, 19 Indian Health Service, 43.e1t Indian tick typhus, 2050 Indinavir, 2288t, 2289, 2331t Indium lung, 589t

Individuality, chemical, 1384 Indomethacin for acute pericarditis, 486 for Bartter’s syndrome, 824 for diarrhea, 933, 935 for headache prevention, 2358t for hemicrania continua, 2361 for paroxysmal hemicrania, 2361 for spondyloarthritis, 1764 Inducible T-cell costimulator deficiency, 1680t Industrial chemical poisoning, 2252t-2253t Inert gas dilution technique, 540-542 Infant botulism, 1927-1928 Infant inclusion conjunctivitis, 2010 Infantile hypercalcemia, 1652t, 1657 Infants. See also Neonates bacterial meningitis in, 2488t chlamydia pneumonia in, 2010 dietary requirements in, 1452 epilepsy syndromes in, 2404, 2406t folate deficiency in, 1108 persistent hyperinsulinemic hypoglycemia of infancy, 1529 pertussis in, 1991-1992 severe myoclonic epilepsy in, 2404 Infarction, 418-419 cerebral. See Stroke myocardial. See Myocardial infarction renal, 809, 809f right ventricular ECG findings, 444 management of, 452 skin, 1283, 1283f Infection, 1849-1854. See also specific infections accidental, 2217 AIDS-defining, 2292 airway, 661 anaerobic, 1931-1934 in aplastic anemia, 1115, 1121 bacterial hepatic, 1011-1012 preferred medications for women, 1602t primary sites, 1885t susceptibility to, 1844, 1886 blistering diseases, 2678-2680 of bone, 1807f of bursae, 1805-1806 and cancer, 1223 in cardiac transplantation, 522 chronic febrile illness, 1852, 1852t clostridial, 1924-1931 coagulation response to, 686 in compromised patients, 1854-1861 corneal, 2559t cutaneous, 2695-2698, 2695t with cyclosporine, 167 definition of, 1837 device-related, 1866 in dialysis patients, 842-843 disseminated, 1858, 1860f early, 686 emerging, 1197, 1197f, 1838 enteric, 1868-1872, 1870b-1872b Enterobacteriaceae, 1960-1962 enterococcal, 1913-1915 erysipeloid, 1923, 1924f esophageal, 906-907 eye, 2565t with fever and rash, 1851, 1851t fungal dematiaceous, 2102t in immunocompromised patients, 1858 of skin, 2693 genetic factors, 1838 health care–associated, 1861-1868 helminthic, 1014-1018 after hematopoietic stem cell transplantation, 1203 host factors, 1845-1846 host-pathogen interactions, 1837-1838 in immunocompromised patients, 1858-1859, 1859t innate immunity defects that lead to, 1686-1687 of joints, 1806-1809, 1807f larval cestode, 2148t of liver, 1011-1018 after liver transplantation, 1037, 1037f after lung transplantation, 646

Index Infection (Continued) management of initial, 1853-1854 principles of therapy, 1843-1848 in multiple myeloma, 1281 nature of, 1844-1845 and neutrophilia, 1130 nosocomial, 1854 and pancreatitis, 960t, 961 parameningeal, 2484, 2495 parasitic distinguishing features of, 528t of liver, 1013-1018, 1015t-1017t pathobiology of, 1849-1850 postoperative, 1832 post-transplantation, 845, 846f prevention of, 1865.e1t prosthetic joint, 1809 protozoal, 1013-1018 pulmonary and bronchiectasis, 566 concurrent with asthma, 554 treatment of, 565 in renal transplantation, 845 reproductive rate (Ro), 1877 respiratory, 556 rhabdomyolysis in, 724 rheumatic, 1724t secondary, 2254 sexually transmitted, 1876-1877, 1877t in sickle cell anemia, 1100 of soft tissues, 1807f steroid-induced, 164 subclinical, 164 surgical site, 1866-1867 tapeworm, 2147, 2148t tissue cestode (cyst), 2149-2153 travel-related, 1881-1885 of upper aerodigestive system, 2601-2604 urinary tract, 1872-1876 vascular catheter-associated, 1865.e1t venous sinus thrombosis secondary to, 2498-2500 viral and cancer, 1228 factor concentrate-transmitted, 1176 Infection stones, 812 Infection-related neutropenia, 1134-1135 Infectious agents, 1838 Infectious arthritis, 1808-1809 Infectious diarrhea acute, 919t antimicrobial therapy for, 1870t, 1872 diagnosis of, 1870t epidemiology of, 919t food-borne, 919-920, 919t laboratory testing in, 1869-1870, 1870t, 1871f prolonged, persistent, 926 treatment of, 922, 1870b-1872b, 1870t water-borne, 919-920, 919t Infectious Diseases Society of America, 1838 evidence-based guidelines for management of fever in neutropenia, 1858 Guideline on Management of Opportunistic Infections in Adults and Adolescents, 2294-2295 guidelines and recommendations for prevention and management of health care–associated infections, 1865.e1t guidelines for outpatient therapy for pneumonia, 616 guidelines for treatment of blastomycosis, 2074-2075 guidelines for treatment of candidiasis, 2081-2082 guidelines for treatment of cryptococcosis, 2077 guidelines for treatment of histoplasmosis, 2071 recommendations for infectious rhinosinusitis, 2588 recommendations for prevention of HIV in medical care settings, 2287 Infectious endocarditis, 2500, 2500b Infectious eye disorders, 2564-2566 Infectious gastroenteritis, acute, 2244 Infectious mononucleosis, 2232 Infectious neuropathy, 2536

Infectious pericarditis, 484t, 491 Infectious rhinitis, 2588 Infectious rhinosinusitis, 2588 Infectious sacroiliitis, 1725, 1725f Infectious sinusitis, 1691t Infective dermatitis, 2237t Infective endocarditis, 474-483 causative organisms, 476-477 clinical manifestations of, 475 complications of, 481 definition of, 474 diagnosis of, 476-478, 478f diagnostic criteria for, 476t echocardiography in, 478, 478f epidemiology of, 474 future directions, 483 history in, 475 incidence of, 474 laboratory findings, 475t, 477-478 microbiology of, 476-477 modified Duke criteria for, 476, 476t pathobiology of, 474-475 physical findings, 475, 475t predisposing conditions, 474, 474t prevalence of, 474 prevention of, 482 prognosis for, 482-483 surgery for, 481-482, 481t treatment of, 478b-482b Inferences from sample means, 35 from samples, 35-36 Inferior mesenteric artery, 951 Inferior vena cava filters, 625 Infertility, 1594-1595 causes of, 1595t definition of, 1575, 1594 diagnosis of, 1575, 1595 dopamine agonist therapy for, 1488 after hematopoietic stem cell transplantation, 1203-1204 after Hodgkin’s lymphoma treatment, 1272t male, 1575 prevalence of, 1575 treatment of, 1595b-1596b tubal factor infertility (TFI), 1942-1943 Infestations delusions of parasitosis, 2177 in HIV infection, 2320 Infiltrates, 532 Infiltrative disorders atrophic skin conditions, 2693t of hypothalamus, 1477, 1477.e1f ventricular tachycardia in, 372 Inflammasome, 231, 1719 Inflammation, 231 of acute gouty arthritis, 1813, 1813f acute phase response, 1719 anti-inflammatory responses in sepsis, 686, 686.e2f blood markers of, 259 cellular response, 231-232 chronic anemia of, 1064-1065, 1071-1072 leukocytosis due to, 1130 of colon, 945-948 deactivation of, 235 disease examples, 230.e3t eye, 2559t initiation of, 230-231 of intestine, 945-948 markers of, 1719 mechanisms of, 230-235 mediators of corticosteroid effects on, 163 soluble, 232-233, 235 neural networks, 233 neurohemoinflammation, 2450 postinflammatory hyperpigmentation, 2700, 2700f postinflammatory hypopigmentation, 2699 postinflammatory pigmentary change, 2638t-2644t, 2642f resolution of, 234-235, 234f second wave, 231-233 in sepsis, 686, 686.e1f signals for induction and repair of, 231.e1t systemic, 1190-1191 in systemic sclerosis, 1780-1781

Inflammatory arthritis granulomatous, 1740t, 1743 pathobiology of, 1828, 1828f peripheral, undifferentiated, 1714f Inflammatory bowel disease, 935-943 clinical features of, 936-937, 1869t colonic microbiota and, 1842 complications of, 942 definition of, 935 diagnosis of, 937-939, 938f diagnostic evaluation in, 938-939 endoscopic evaluation in, 938 epidemiology of, 935 extraintestinal complications of, 937, 937t follow-up, 942-943 genetics of, 935-936 laboratory findings, 938-939 medical therapy for, 939-940, 939t pathobiology of, 935-936 pathophysiology of, 936 in pregnancy, 943 prognosis for, 943 radiology in, 938-939 serologic markers for, 939 surgical therapy for, 942 treatment of, 939b-943b Inflammatory cells, 234-235 Inflammatory diarrhea, 926, 934 Inflammatory disease in HIV infection, 2321 hypothalamic disorders, 1477 intestinal conditions, 947-948 laboratory evaluation in, 1720-1723 neonatal-onset multisystem, 231, 1740t-1741t, 1742 occupational lung disorders, 80, 80t systemic, 1720-1723 ventricular tachycardia in, 372 visual field abnormalities due to, 2575 Inflammatory eye disorders, 2563-2564, 2570-2571 Inflammatory myelopathy, 2381, 2381b Inflammatory myopathy, 1789-1793, 2546-2547 classification of, 1713t, 1790t, 2546-2547, 2547t clinical manifestations of, 1791-1792 diagnosis of, 1791-1792 diagnostic criteria for, 1791t epidemiology of, 1789 overlap syndromes, 1790t, 1791 pathobiology of, 1789-1791, 1790f prognosis for, 1793 toxic myopathies, 2546t treatment of, 1792b-1793b, 1793f types of, 1789 Inflammatory myositis, 2243 autoantibodies associated with, 1716t clinical manifestations of, 2243 diagnosis of, 2243 idiopathic, 1716t Inflammatory neuropathy, peripheral, 2529-2532, 2530t Inflammatory nodules, 2691-2692 Inflammatory papules, 2690-2691 Inflammatory prostatitis, asymptomatic, 831t Inflammatory sinusitis, 1691t Inflammatory skin lesions, 2691-2692 Inflammatory tumors, 2690-2691 Infliximab (Remicade), 169, 2660-2661 for Behçet’s syndrome, 1799 for Crohn’s disease, 940 for inflammatory bowel disease, 939t for psoriasis, 2666t for rheumatoid arthritis, 1762 for sarcoidosis, 607, 607t Influenza, 2191-2197 antigenic variation of, 2191-2192 antiviral therapy for, 2183-2184, 2183t clinical manifestations of, 2194-2195 complications of groups at increased risk for, 2194, 2194t nonrespiratory, 2195 respiratory, 2194-2195 definition of, 2191 diagnosis of, 2195 distinguishing characteristics of, 2252t-2253t epidemic, 2192-2193, 2192t epidemiology of, 2191-2193

I55

Influenza (Continued) genetic variation of, 2191-2192 interpandemic, 2192-2193 medications for, 2196 pandemic, 2193 pathobiology of, 2193-2194 precautions against, 2196 prevention of, 2195-2196, 2294, 2295t prognosis for, 2197 seasonal, 554, 2194t treatment of, 2196b-2197b vaccination against, 68t-73t, 74, 554, 1882, 1883t, 2195-2196 adverse reactions to, 74 currently approved vaccines, 2195, 2195.e1t indications for, 68f, 74 for prophylaxis in HIV infection, 2295t recommendations for, 67, 68f, 2195, 67.e1t-67.e3t target groups for, 2194t zoonotic, 2193 Influenza A virus, 2191 antigenic subtypes associated with pandemics, 2191t proteins, 2191, 2192t Influenza B virus, 2191 Influenza C virus, 2191 Influenza syndrome, 2194 Influenza virus, 2185t, 2191, 2192f Information sharing Ask-Tell-Ask, 10, 14t, 52 disclosure to patients, 5, 5t discussing palliative care topics, 14t for heart failure management, 315t standards for physician or customary, 5 reasonable person or lay-oriented, 5 substituted judgment, 6 Information systems, 49 Information technology, 47 Informed consent, 4-6 definition of, 5 empirical data on, 5 fundamental elements for disclosure to patients, 5, 5t history of, 4-5 justification for, 5 practical considerations, 5-6 requiements for, 5 Infradian rhythms, 1475 Ingenol mebutate gel, 2687-2688 Ingrown toenails, 2709, 2710f Inguinal adenopathy, 1380-1381 Inhalation injury, 716.e1f Inhalational (volatile) anesthetics, 2619 Inhalational anthrax clinical manifestations of, 1921 diagnosis of, 1922 fatality rate, 1923, 2060t prognosis for, 1923 staging system for, 1921, 1921t Inhaled corticosteroids, 552, 553t Inhalers, 552, 552f Inheritance of common diseases, 196-200 Mendelian, 194-195 multifactorial, 186, 187f nonmendelian, 195 patterns of, 194-195 Inheritance factors, 189, 200 Inherited autoinflammatory diseases, 1740t, 1743-1744 Inherited blistering disorders, 2678 Inherited bone marrow failure syndromes, 1116-1117 clinical features of, 1116-1117, 1116t genetics of, 1117 prognosis for, 1121 Inherited cancer susceptibility syndromes, 1226, 1227t Inherited colorectal cancer syndromes, 1323t Inherited connective tissue diseases, 1733-1739 Inherited cystic syndromes, 821-822 Inherited hemolytic anemia, 1089-1095 Inherited hypercoagulable states, 1158, 1158t Inherited hypoparathyroidism, 1660 Inherited liver disease, 982-983 Inherited muscle diseases, 2541 Volume I  pp 1-1382  •  Volume II  pp 1383-2722

I56

Index

Inherited neuropathy definition of, 2529 peripheral neuropathies, 2529 treatment of, 2529b Inherited optic atrophy, 2575 Inhibitor of κB (IκB), 219 Injectable steroid contraceptives constituents and use, 1609 side effects of, 1609 Injection anthrax, 1920-1921, 1921f Injection drug use, 1926 C. novyi infection in, 1926 HIV transmission in, 2286 HTLV transmission in, 2236 infection associated with, 1926 infective endocarditis in, 474 Injections, 1822 Injury, 711 abdominal, 714, 714f acute kidney injury, 732-733 in adolescents, 61 blast, 712t, 713, 713f blunt, 711, 712t burns, 711-717 caustic, to esophagus, 902 chest, 713-714 classes of, 711, 712t clinical manifestations of, 711-713 cold injury, 693-695 critical care for, 715-716 crush, 711-713, 712f, 712t early local response to, 711 early systemic response to, 711 electrical, 711-717, 712t, 713b-716b esophageal caustic, 902 medications associated with, 903t fragmentation, 712t, 713, 713f head and spine, 714 head injury, 2366, 2366t heat injury, 692 inhalation, 716.e1f initial evaluation and resuscitation, 713-715 intentional, 61 interhospital transport, 713 to kidney, acute, 778-783 laryngotracheal, due to tracheal cannulation, 528t late hypermetabolic systemic response to, 711 late local response to, 711 lightning, 716 local tissue, in snake bite, 718 lung, 595-603 management of, 714-715 mechanisms of, 711 neck, 2373, 2373f in older adults, 716.e1t pathobiology of, 711 penetrating, 712f, 712t peripheral nerve, 1832 prehospital care, 713 prevention of behavioral interventions for, 57 radiation injury, 82-86 rehabilitation, 716-717 spinal cord, 2364-2370 sports medicine, 1751, 1751t stingray injuries, 721 thermal, 711, 712f, 712t traumatic, 711 traumatic brain, 2364-2370 Inlet patch (heterotopic gastric mucosa), 908 Inlyta (axitinib), 170, 1211t-1216t Innate immunity, 230 defects that lead to infections, 1686-1687, 1687b diseases of, 1687t Innate lymphoid cells, type 2, 1689 Inner ear disease, autoimmune, 2599 Innohep (tinzaparin), 177t Innominate artery, 2424 Inotropes, 451, 683 Inpatient psychiatric care, 2625-2626 Insect sting allergens, 1699 Insect sting allergy, 1698-1703 Insecticides, 2548t Insomnia, 639, 2419-2421 classification of, 2420t, 2421 clinical manifestations of, 2420 definition of, 2419

Insomnia (Continued) diagnosis of, 2420-2421 differential diagnosis of, 2420f epidemiology of, 2419 pathobiology of, 2420 prognosis for, 2421 treatment of, 2421b medications, 2421, 2421t nonpharmacologic therapies, 2421, 2421t Inspection, cardiac, 253 Inspiratory capacity, 540 Inspiratory hold maneuver, 654f Inspiratory pressure, maximal (Pimax or MIP), 655 Inspiratory reserve volume, 540 Institute of Medicine (IOM) definition of quality of care, 44 Prepositioning Antibiotics for Anthrax, 1922 quality aims, 44t Summit on Integrative Medicine and the Health of the Public, 184 Instrumental activities of daily living (IADLs), 102-103 Insulin antibodies to, 1554 daily requirements for, 1531 for diabetic ketoacidosis, 1540-1541, 1541f for drug overdose, 699t-702t for hyperkalemia, 761 impaired action of, 1533 impaired secretion of, 1532-1533 for intracerebral hemorrhage, 2452t metabolic effects of, 1529, 1529t preparations, 1531t regulation of, 1529, 1529f resistance to, 835, 1462, 1529, 1533 for septic shock, 690 for subarachnoid hemorrhage, 2448t for type 1 diabetes, 1531 for type 2 diabetes, 1536-1537 Insulin autoimmune syndrome, 1551, 1554 Insulin deficiency, 1528, 1528f Insulin factitious hypoglycemia, 1552 Insulin secretagogues, 1535-1536 Insulin sensitizers, 1535 Insulin tolerance test, 1483t Insulin-glucose, 706t-710t Insulin-like growth factor (IGF)-2-oma, 1550, 1552 Insulinoma, 1335t, 1551, 1554-1555 Insulin-receptor mutations, 1552 Integrase strand transfer inhibitors (INSTIs), 2288t, 2289, 2290t Integrative medicine, 182t Integrilin (eptifibatide), 180 Integrin, 1145 Integrin αIIbβ3 (GPIIb-IIIa) receptor antagonists, 180 β2 Integrins, 1145 Integumentary system, 108 Intensive care. See also Critical care admission orders, 650-651, 651t coronary, 448-449 for HIV-infected patient, 2317-2318 long-term outcomes for survivors, 650 Intensivists, 650 Interfacility collaboration, 1868 Interferon, 2178-2179 stimulator of interferon genes (STING), 217 type I induction of antiviral response, 218, 218f sarcoidosis interactions, 607 in systemic lupus erythematosus, 1770 Interferon regulatory factor (IRF), 219 Interferon-2b, pegylated, 2180-2181 Interferon-α (Intron-A, Roferon) for adult T-cell leukemia/lymphoma, 2238 adverse effects of, 2178t for Behçet’s syndrome, 1799 for cancer, 1211t-1216t cardiomyopathy due to, 330 for hepatitis B, 1002 for hepatitis C, 1004 for HTLV-associated myelopathy/tropical spastic paraparesis, 2238-2239 for hypereosinophilic syndrome, 336, 1154 for melanoma, 1376 for metastatic renal cell carcinoma, 1348

Interferon-α (Intron-A, Roferon) (Continued) for metastatic Zollinger-Ellison syndrome, 1337 for mycosis fungoides, 2670t side effects of, 1004 Interferon-α2a for hepatitis B, 1002, 2177t for hepatitis C, 998, 1004, 2177t mechanisms of excretion and thresholds for dose adjustment, 2178t Interferon-α2b for hepatitis B, 1002, 2177t for hepatitis C, 998, 1004, 2177t for hepatitis D, 1005 for mastocytosis, 1710 Interferon-β1a (Avonex, Rebif), 2477 Interferon-β1b (Betaseron, Extavia), 2477 Interferon-γ, 232 Interferon-γ receptor-1 defects, 1147t-1148t, 1150, 1687t clinical manifestations of, 1150 diagnosis of, 1150 pathobiology of, 1150 treatment of, 1150b Interferon-γ receptor-1 deficiency, 1686-1687 Interferon-γ receptor-2 deficiency, 1686-1687, 1687t Interferon-γ release assays (IGRAs), 2036, 2041 Interferon-γ–IL-12 axis defects, 1147t-1148t Interhospital transport, 713 Interleukin-1, 232 Interleukin-1 inhibitors, 169-170 adverse effects of, 170 indications for, 170 types of, 169-170 Interleukin-1 receptor antagonist deficiency, 1740t, 1742-1743 Interleukin-1–associated autoinflammatory diseases, 1742-1743 Interleukin-2, 1050 cardiotoxicity, 330 for metastatic renal cell carcinoma, 1348 Interleukin-2, human recombinant (aldesleukin), 1211t-1216t Interleukin-3, 1144 Interleukin-4, 1050 Interleukin-5, 1050 Interleukin-6, 232 Interleukin-6 inhibitors, 170 Interleukin-7, 1050 Interleukin-7 deficiency, 1678t Interleukin-12 antibody, 170 Interleukin-12 receptor deficiency, 1686-1687, 1687t Interleukin-15, 1050 Interleukin-17, 232 Interleukin-23, 232 Interleukin-23 antibody, 170 Interleukin-23 receptor deficiency, 1686-1687, 1687t Intermediate life support, 353 Intermediate-density lipoprotein, 1389t Intermittent claudication, 498, 498f, 503f Intermittent mandatory ventilation, 665 Internal carotid artery, 2424-2425, 2426f Internal carotid artery occlusion, 2435, 2435t Internal medicine, 2 International Antiviral Society–USA 2012, 2288t International Association for the Study of Lung Cancer (IASLC), 1305-1306, 1308f International Cancer Genome Consortium, 202 International Federation of Gynecology and Obstetrics (FIGO) staging system for cervical cancer, 1361, 1361t staging system for endometrial cancer, 1362t staging system for ovarian cancer, 1364t International HapMap, 189-190 International normalized ratio (INR), elevated, 176t International Prognosis Scoring System (IPSS), 1238 International Prognosis Scoring System revised (IPSS-R), 1238, 1238t-1239t International Prognostic Index, 1262, 1262t International Prostate Symptom Score (IPSS), 828, 828f

International Society for Heart and Lung Transplantation, 403, 521t International Society of Hypertension, 390f International Society of Nephrology, 792, 1772, 1772.e1t International Society of Urologic Pathology, 1349 International travelers pretravel consultation with, 1881, 1882t vaccinations for, 77-78, 1881-1882, 1883t International Union Against Cancer, 1329 International Union of Immunological Societies, 1677 International Workshop in Chronic Lymphocytic Leukemia, 1256t Internuclear ophthalmoplegia, 2473, 2578, 2578f Interpersonal psychotherapy, 2348t Interpreters, 16 Interstitial lung disease, 575-588 associated with connective tissue disease, 575t, 584-586 associated with pulmonary vasculitis, 575t, 586 bronchiolitis–-associated, 582, 582b, 582f chest radiography findings, 577-578, 578t chronic, 80 classification of, 575t clinical manifestations of, 576 definition of, 575 diagnosis of, 576-579, 576f drug-induced, 576-577, 577t, 586 epidemiology of, 575 evaluation of invasive, 579 noninvasive, 577-579 exercise testing in, 579 forms of, 575t, 586-587 high-resolution CT in, 578-579 history in, 576-577 in HIV infection, 2316-2317 clinical manifestations of, 2317 diagnosis of, 2317 epidemiology of, 2316 pathobiology of, 2316-2317 prognosis for, 2317 radiographic features of, 2309t treatment of, 2317b iatrogenic, 577t incidence of, 575 inherited disorders associated with, 587 inherited forms of, 575t laboratory findings, 577 occupational, 80, 80t, 586 pathobiology of, 575-576 physical findings, 577 prevalence of, 575 pulmonary function findings, 579 respiratory bronchiolitis–-associated, 582 in systemic sclerosis, 1782-1783 treatment of, 579, 579b, 579t Interstitial nephritis, 793 acute, 779, 795-796 on biopsy, 778f, 796 clinical manifestations of, 795, 795t indications for biopsy, 796t, 783.e2 medications associated with, 779, 779t treatment of, 796b chronic, 778f, 796-799 Interstitial pancreatitis, acute, 960 Interstitial pneumonia acute, 571t, 572-573, 583, 583f chronic fibrosing, 575t, 580-581 desquamative, 579-580, 582-583, 583f, 583.e1f familial, 575-576 fibrosing, 579-580 idiopathic, 579-584 acute, 575t, 583-584 classification of, 575t radiographic features of, 578-579, 578t subacute, 575t, 583-584 lymphocytic, 584, 584b lymphoid, 584, 584b, 584.e1f nonspecific, 581, 582f diagnosis of, 581 treatment of, 582b rare, 575t smoking-related, 575t, 582-583 Interstitial pneumonitis, nonspecific, 1779 Interventricular septum, 264 Interviews, motivational, 52, 53f, 147, 147t

Index Intestinal abnormalities, 943-944 Intestinal absorption phases of, 926, 926.e1f tests of, 925t Intestinal amebiasis drug therapy for, 2141t endoscopic and pathologic features of, 2139-2140, 2140f Intestinal anthrax, 1921-1922 Intestinal atresia and stenosis, 944 Intestinal bacterial overgrowth, 928 clinical manifestations of, 928 diagnosis of, 928 treatment of, 928b Intestinal disorders, structural acquired, 944-945 congenital abnormalities, 943-944 Intestinal diverticular disease, 1430 Intestinal dysmotility syndromes autoimmune, 2518t gastrointestinal disorders, 884-890 Intestinal flukes, 2156t, 2157 clinical manifestations of, 2157 diagnosis of, 2158 epidemiology of, 2157 pathobiology of, 2157 prevention of, 2159 treatment of, 2158b Intestinal inflammation, 945-948 Intestinal inflammatory conditions, 947-948 Intestinal ischemia, 951-956, 953f clinical manifestations of, 951 diagnosis of, 951-953 epidemiology of, 951 initial diagnostic evaluation of, 953 management of, 952f noninvasive imaging of, 953 pathobiology of, 951 prognosis for, 953 treatment of, 953b Intestinal malrotation, 944 Intestinal nematodiases, 2163-2164 Intestinal obstruction mechanical, 887 treatment of, 565-566 Intestinal protozoa, 2105 Intestinal pseudo-obstruction, 886-887 chronic, 887f classification of, 885, 886t clinical manifestations of, 886 complications of, 887 diagnosis of, 886-887, 887f medical therapy for, 887-888 pathogenesis of, 887, 887f surgical therapy for, 888 treatment of, 887b-888b Intestinal roundworms, 2159-2164 definition of, 2159 treatment of, 2103, 2160t Intestinal secretion, 918, 919t Intestinal tapeworms, 2148-2149 prevention of, 2148b therapy for, 2148t treatment of, 2148b Intestinal transport mechanisms of, 918-919, 918.e3f rapid transit, 933 Intimate partner violence, 1629-1633 advocacy for, 1631 causation of, 1630 clinical manifestations of, 1630-1631 couple therapy of, 1632 definition of, 1629-1630 disclosure of, 1631 epidemiology of, 1630 identification of, 1631, 1632t inquiry about, 1631 natural history of, 1632-1633 prevention of, 1632 prognosis for, 1632-1633 psychological interventions for, 1631-1632 questions to ask, 1631 recommendations, 1632t referral services for, 1631 risk factors for, 1630, 1630t treatment of, 1631b-1632b treatment of abuser in, 1632 Intimate terrorism, 1629-1630

Intoxication. See also Poisoning alcohol, 151 rhabdomyolysis in, 723 signs of, 2510 Intra-abdominal infections anaerobic, 1932 enterococcal, 1914 Intra-abdominal testes, 1567-1568 Intra-aortic balloon pumps, 453, 683, 684f Intra-articular therapy, 1749 Intra-axial tumors, 1292-1294 Intracardiac echocardiography, 279 Intracavitary mycetoma, 612, 612.e1f Intracavitary tumors, 338-339 Intracellular adhesion molecule 1 (ICAM-1), 232 Intracellular fluid, 741-742, 741f Intracerebral hemorrhage, 2450-2453 acute, 2452t clinical manifestations of, 2451 complications of, 2452 CT findings, 2451, 2451f definition of, 2450 diagnosis of, 2451, 2451f epidemiology of, 2450 lobar, 2451 pathobiology of, 2450-2451, 2450f prevention of, 2452 prognosis for, 2452-2453 in putamen, 2451, 2451f secondary, 2450t sites and sources of, 2450, 2450f surgical management of, 2451-2452 treatment of, 2451b-2452b ventricular drainage of, 2452 Intracranial aneurysms, 2447 Intracranial carotid artery stenosis, 2445 Intracranial hypertension, 2362 clinical manifestations of, 2362 diagnosis of, 2362 management of, 2452t prognosis for, 2362 reduction of, 2368, 2412-2413 treatment of, 2362b Intracranial hypotension, 2362-2363, 2363f clinical manifestations of, 2363 diagnosis of, 2363 pathobiology of, 2363 treatment of, 2363b Intracranial tumors, 1287-1295, 1290t Intractability, 917 Intradural extramedullary tumors, 1296 Intrahepatic cholestasis benign recurrent, 987-988 of pregnancy, 1622 progressive familial, 987-988 Intraluminal amebiasis, 2138 Intraluminal digestion, 927-929 Intramedullary tumors, 1296-1297 clinical manifestations of, 1297 diagnosis of, 1297 treatment of, 1297b Intramural aortic hematoma, 494-497, 496f clinical manifestations of, 494-495 treatment of, 495-496 Intramyocardial tumors, 339 Intraocular pressure, 2556 Intrauterine devices, 1604, 1605t, 1606f, 1609-1610 Intravascular devices, 627 Intravascular hemolysis causes of, 1079 treatment of, 1079 Intravascular hemolytic anemia, 1073-1080 Intravascular large B-cell lymphoma, 1265 Intravenous drugs. See also Drug(s) infusion rate, 126 Intravenous fluids. See also Fluid therapy for shock, 676 Intravenous gamma globulin, 1776, 2672 Intravenous immunoglobulin for chronic inflammatory demyelinating polyradiculoneuropathy, 2531 for chronic lymphocytic leukemia, 1256 for epidermolysis bullosa acquisita, 2676 for Guillain-Barré syndrome, 2530 for inflammatory myopathies, 1792 for ITP, 1164t for Kawasaki disease, 1797

Intravenous immunoglobulin (Continued) for Lambert-Eaton myasthenic syndrome, 2552 for multifocal motor neuropathy, 2532 for myasthenia gravis, 2551 for pemphigus, 2677 for toxic epidermal necrolysis, 2678 Intravenous lipid emulsions, 1443 Intravenous pyelography, 812 Intrinsic factor: absent secretion of, 1107 Intrinsic (contact activation) factor deficiency, 1178 Intrinsic factor receptor disorders, 1107 Intron-A (interferon-α), 1211t-1216t Introns, 189 Intubation, 666, 704-705 Intussusception, 945 clinical manifestations of, 945 diagnosis of, 945 prognosis for, 945 treatment of, 945 Invariant chain, 221, 221f Invasion, cancer, 1230 Invasive aspergillosis acute, 2083t antifungal therapy for, 2086, 2086t clinical manifestations of, 2083-2084 diagnosis of, 2084-2085 pulmonary, 2083 radiographic features of, 2084, 2085f treatment of, 2086b-2087b Invasive candidiasis, 2080, 2080f focal infections, 2081-2082 treatment of, 2082 Invasive mucinous adenocarcinoma, 571t, 574-575, 575b, 574.e1f Invasive procedures diagnostic, 305 suggested minimum platelet counts before, 1161t Inverse psoriasis, 2701f Inversions, 193 paracentric, 193 pericentric, 193 Iodine, 1450t-1452t, 1508 Iodoquinol, 2147t Ion channelopathies, 2544-2545 Ion channels, 263, 263f, 339-341, 340f, 341.e1f Ionizing radiation, 82-85 absorbed dose, 82, 83t and cancer, 1225 collective dose, 82, 83t effective dose, 82, 83t equivalent dose, 82, 83t exposure to clinical manifestations of, 83-84 diagnosis of, 84-85 pathobiology of, 83 prevention of, 85 standard quantities and units associated with, 82, 83t natural sources of, 82 Ipilimumab (Yervoy), 1211t-1216t, 1232t, 1376 Ipratropium, 560t, 2186t, 2588 IRAK4 deficiency, 1686-1687, 1687t Irbesartan, 388t Iressa (gefitinib), 171 Irinotecan (Camptosar), 1211t-1216t, 1330-1331 Iris melanoma, 2572, 2572f Iritis, 2559t, 2570 Iron, 1450t-1452t homeostasis of, 1068-1069, 1068f, 1419-1420, 1419f serum levels, 1063-1064, 1064f supplemental for anemia of chronic inflammation, 1071 failure to respond to, 1070 for malabsorption, 931t oral administration, 1070 parenteral administration, 1070 for restless legs syndrome, 2424, 2468-2469 toxic levels, 703t Iron chelation therapy, 1094 for hemochromatosis, 1421 for myelodysplastic syndromes, 1237 for sickle cell disease, 1103-1104 Iron deficiency, 1065

I57

Iron deficiency anemia, 1068-1071, 1068f clinical manifestations of, 1069 diagnosis of, 1069-1070 endoscopy in, 876 epidemiology of, 1068 GI bleeding in, 884 iron-refractory, 1070 laboratory findings, 1070t pathobiology of, 1068-1069 prognosis for, 1071 treatment of, 1070b Iron gluconate, 931t Iron intake, inadequate, 1069 Iron overdose, 706t-710t Iron overload, 1094, 1418-1423 C282Y homozygotes without, 1422, 1423t classification of syndromes, 1419, 1420t definition of, 1418-1419 epidemiology of, 1418-1419 pathobiology of, 1419-1420 secondary, 1420t Iron sulfate, 931t Iron-refractory iron deficiency anemia, 1070 Irrigation, whole bowel, 705 Irritable bowel syndrome, 890-894 clinical manifestations of, 891 definition of, 890-891 diagnosis of, 891-892 diagnostic algorithm for, 892f diagnostic tests, 892 differential diagnosis of, 892 epidemiology of, 891 genetics of, 891 medications for, 893t pathobiology of, 891 patient education about, 893 pharmacologic therapy for, 894 physician-patient relationship, 893 post-infectious, 1869t prognosis for, 894 psychological therapy for, 894 Rome III diagnostic criteria for, 891-892, 891t symptoms of, 891 treatment of, 892-893, 892b-894b Irritant dermatitis contact, 2663-2664 occupational, 80t Irritant exposure, 590 Irritant-induced asthma, 590 Irukandji syndrome, 721 Irukandji-like stings, 721 Isaac’s syndrome, 2545, 2552 Ischemia intestinal, 951-956, 953f limb acute, 498-501, 499t, 502f chronic, 499-500 lower limb chronic, 499t, 501 chronic critical, 499 chronic stable, 499 mesenteric chronic, 911, 911f pathobiology of, 951 secondary, 951, 951t myocardial differential diagnosis of, 484t silent, 432 small intestinal, 951 therapeutic agents to reduce, 429 tissue: prevention of, 715, 715f, 716.e1f Ischemic acute renal failure, 780, 780t Ischemic bowel syndromes, 953-956 Ischemic cardiomyopathy, chronic, 370f Ischemic cerebrovascular disease, 2434-2445 clinical manifestations of, 2435-2436, 2435t definition of, 2434 pathobiology of, 2434-2435 Ischemic colitis, 955-956 clinical manifestations of, 955 diagnosis of, 956 differential diagnosis of, 956 endoscopic findings, 956f epidemiology of, 955 medications and drugs associated with, 955, 956t pathobiology of, 955 prognosis for, 956 treatment of, 956b

Volume I  pp 1-1382  •  Volume II  pp 1383-2722

I58

Index

Ischemic disease, end-stage, 280t-281t Ischemic heart disease, 420 CABG for, 460 causes of, 421t chronic blood tests in, 423t diagnosis of, 371 symptoms of, 421t diagnosis of, 422 diagnostic coronary angiography in, 425t ECG findings, 422, 422f epidemiology of, 420 invasive therapy for, 2614-2615 laboratory findings, 423 medical therapy for, 2614 pathobiology of, 420-421 physical findings, 421 prognosis for, 432 risk reduction strategies for, 2614-2615 risk stratification, 426-427 stable, 420-432 treatment of, 427b-431b Ischemic hepatitis A, 959 Ischemic nephropathy, 807 Ischemic optic neuropathy, 2575 Ischemic stroke acute arterial hypertension in, 2441 diagnosis of, 2439, 2440f differential diagnosis of, 2440 evaluation of, 2440t immediate diagnostic studies in, 2439t treatment of, 395t, 2440-2441, 2440t-2442t, 2440.e1f antithrombotic therapy for, 180t approach to, 2436f definition of, 2434 diagnosis of, 2436-2440 genetic diseases that can cause, 2443 history in, 2436 initial laboratory tests, 2436-2440 pathobiology of, 2434 physical findings, 2436 treatment of, 2441 endovascular therapy, 2441 thrombolytic therapy, 180t unusual causes of, 2442-2445, 2443t Ischial bursitis, 1753 Ischiogluteal bursitis, 1753 Isobutyric aciduria, 1387t Isocarboxazid, 2349t Isometheptene mucate, 2358 Isoniazid (INH) for AIDS-associated opportunistic infections, 2297t-2301t for M. kansasii lung disease, 2042 and micronutrient status, 1453t recommended doses and schedules, 1891t-1892t toxicity, 706t-710t for tuberculosis in HIV infection, 2311, 2312t Isopropyl alcohol, 769 Isoproterenol, 355 Isosorbide dinitrate, 314, 431t, 306.e1t-306.e2t Isosorbide-5-mononitrate, 431t Isospora infection, 1869t Isotretinoin (Accutane), 2658 for acne vulgaris, 2680 ocular effects of, 2573t for rosacea, 2680 Isovaleric aciduria, 1387t Isradipine, 388t, 430t Israeli spotted fever, 2048t, 2050 Istradefylline, 2457t-2459t Ito cells, 1023 Itraconazole, 2067 adverse effects of, 2067 for AIDS-associated opportunistic infections, 2297t-2301t for allergic bronchopulmonary aspergillosis, 2086 for blastomycosis, 2075 for chromomycosis, 2103 for chronic granulomatous disease, 1149 drug interactions, 2067 formulations, 2067 for histoplasmosis, 2071 for invasive aspergillosis, 2086t for mycetoma, 2101 for paracoccidioidomycosis, 2075-2076

Itraconazole (Continued) for Pityrosporum folliculitis, 2681 for prophylaxis in HIV infection, 2295t for sporotrichosis, 2079 for thrush, 2082, 2604 Ivabradine, 313-314 doses and side effects of, 363t-364t for heart failure, 314 Ivermectin, 2104 for intestinal nematodes, 2160t for intestinal strongyloidiasis, 2163 for lice, 2698 for nematodes, 2165t, 2166 for onchocerciasis, 2169-2170 for roundworms, 2103-2104 for scabies, 2173, 2698 for trichuriasis, 2162 Ixabepilone (Ixempra), 1211t-1216t Ixekizumab, 2666t Ixiaro (IC51), 2266 Ixodes scapularis (deer tick), 2021, 2022f, 2055

J

J wave, 268, 268f, 694, 694f J wave syndromes, 343, 343f Jaccoud arthropathy, 1771-1772 Jackhammer esophagus, 904 Jackson’s syndrome, 2435.e2f Jacobsen/Paris-Trousseau syndrome, 1057-1058 Jadelle implant, 1609 JAK3 deficiency, 1678t Jakafi (ruxolitinib), 1211t-1216t Jamestown Canyon encephalitis, 2268-2269 Janeway’s lesions, 255, 257f, 475, 475t Jansen’s disease, 1652t, 1657 Janus kinase ( JAK), 219, 232 Janus kinase inhibitors, 170 Japan, 100, 603 Japanese encephalitis, 2263t, 2265-2266 clinical manifestations of, 2266 diagnosis of, 2266 epidemiology of, 2265-2266 pathobiology of, 2262, 2266 prevention of, 2266 prognosis for, 2266 travel-related risk, 1881 treatment of, 2266b Japanese encephalitis vaccine, 68t-73t, 77, 1882-1883, 1883t, 2266 adverse reactions to, 77 indications for, 77 Japanese monocytic ehrlichiosis, 2053t Japanese spotted fever, 2048t, 2050 Jarisch-Herxheimer reactions, 2019, 2028, 2030 Jaundice, 983-985, 991-993 in cirrhosis, 1025 definition of, 983 differential diagnosis of, 986t, 991, 991t fever and jaundice, 1852, 1852t in HIV infection, 2303t neonatal, 1088, 1088b obstructive, 991t postoperative, 988-991 in pregnancy, 988 zoonotic, 2057, 2059t Jaw: osteonecrosis of, 1643, 1667-1668, 1668f Jaw jerk reflex, 2372t Jaw tumors hereditary hyperparathyroidism and, 1652t hyperparathyroidism–jaw tumor syndrome, 1656 Jaw winking, 2537 JC polyomavirus, 2334 Jehovah’s Witnesses, 1193 Jejunal adenoma, 1326-1327, 1322.e1 Jejunostomy, percutaneous endoscopic, 877 Jellyfish envenomation, 720 clinical manifestations of, 720 epidemiology of, 720 treatment of, 720b-721b Jellyfish groups, 720 Jenvac, 2266 Jersey finger, 1753 Jervell and Lange-Nielsen syndrome, 405 Jeryl-Lynn mumps vaccine, 2208 Jespect (IC51), 2266 Jet lag, 1884, 2422 JMP syndrome, 1740t

Joint arthritis, 1757-1758, 1828 Joint arthroplasty surgical innovations in, 1830-1832 total joint, 1830-1832 Joint Commission, 44, 46, 1868.e1, 43.e1t Joint disease degenerative, 1713t, 1715t rheumatic, 1717 in sarcoidosis, 606 surgical treatment of, 1828-1833 Joint fluid analysis, 1720, 1720f Joint infections, 1806-1809 clinical evaluation of, 1807f Pseudomonas in, 1965 Joint National Committee, 838, 1428t-1429t Josamycin, 2051 Joubert’s syndrome, 2513 Jugular veins, 252 distention of, 252, 252f examination of, 26, 303 Jugular venous pressure, 252 Jugular venous pulse, normal, 252f Jumper’s knee, 1753 Junctional epidermolysis bullosa, 2678 Junctional nevus, 2689, 2689f Junctional premature beats, 359, 359f Junctional tachycardia, 360, 361f, 363-366 Junín virus, 91t, 2247, 2249t Justice, 3-4 Juvenile idiopathic arthritis, 1740t Juvenile myoclonic epilepsy, 2406 Juvenile (acute-subacute) paracoccidioidomycosis, 2075 Juvenile polyposis, 1323t, 1325

K

Kalbitor (ecallantide), 1697 Kallmann’s syndrome, 1492, 1565, 1570-1571 anovulation in, 1592 X-linked form, 1476 Kaposi’s sarcoma, 2690 causative factors, 1228 in feet, 2325, 2325f in HIV infection, 2316, 2320, 2320f, 2322, 2323t, 2325-2326 clinical manifestations of, 2325 diagnosis of, 2325 epidemiology of, 2325 pathobiology of, 2325 prognosis for, 2326 radiographic features of, 2309t, 2316f staging of, 2325 treatment of, 2325b-2326b Kaposi’s sarcoma herpesvirus (KSHV)– associated lymphoma, 2322, 2324 Karnofsky Performance Status scale, 1207t Kawasaki disease, 2672 clinical manifestations of, 1796-1797, 2672 diagnosis of, 2672 epidemiology of, 1794, 1794t, 2672 treatment of, 1797, 2672b Kayser-Fleischer rings, 977-978, 1417, 1417f, 2510 Kearns-Sayre syndrome, 1652t, 1660-1661, 2546 Kehr’s sign, 858t Keloids, 2700, 2701f Kenalog (triamcinolone), 2684t Kennedy’s disease, 2526 clinical features of, 2524t, 2526 epidemiology of, 2526 pathobiology of, 2526 prognosis for, 2526 treatment of, 2526b Kenney-Caffey syndrome, 1652t, 1660-1661 Kenya tick typhus, 2050 Keratinization, 2632, 2632.e1t Keratinocyte necrosis, 2638t-2644t, 2642f Keratitis Acanthamoeba, 2566 gonococcal, 2565 herpes simplex, 2565f herpetic, 2225 pseudomonal, 1967, 2565 Keratoconjunctivitis, 2181t Keratoconjunctivitis sicca, 1759, 2237t, 2570, 2570t Keratoconus, 2558 Keratocytes, 1053f Keratoderma blennorrhagicum, 1767, 1767f

Keratolytics, 2660 Keratoses actinic, 2687-2688, 2687f seborrheic, 2645f, 2688, 2688f Kerley A lines, 532, 534f Kerley B lines, 283f, 284, 532 Kernig’s sign, 2482 Keshan’s disease, 1450t-1452t Ketamine, 144t for anesthesia, 2619 for erythromelalgia, 508 Keto acids, 768 Ketoacidosis alcoholic, 768, 768b diabetic, 767-768, 1539-1540 diagnostic criteria for, 1540t management of, 1541f precipitants, 1539t treatment of, 768b, 1540b-1542b Ketoconazole, 2066-2067 adverse effects of, 2066-2067 applications, 2066 for Cushing’s syndrome, 1517 formulations, 2066 for fungal infections, 2670-2671 for mycetoma, 2101 for paracoccidioidomycosis, 2075-2076 pharmacology of, 2066 Ketolides, 1894-1896 mechanism of action, 1889t recommended doses and schedules, 1891t-1892t resistance to, 1890t Ketone body metabolism disorders, 1555 Ketoprofen, 2358, 2360 Ketorolac for acute pericarditis, 486 for migraine headache, 2359 for renal colic, 813 17-Ketosteroid reductase (17-hydroxysteroid dehydrogenase 3) deficiency, 1564 Ketostix, 1540 Khmer refugees, 1930 Kidney(s). See also under Renal acid production and excretion by, 763 in acid-base balance, 763, 763.e1f anatomy of, 737f development of, 824 dysfunction of in heart failure, 316 in septic shock, 690 signs of, 1030 treatment of, 316 function of, 737-740, 834, 834t and antimicrobial therapy, 1845-1846 measurement of, 728 malrotation of, 825-826 myeloma, 798, 1281 normal ultrasound findings, 733, 733f regulation of body fluid osmolarity by, 738-739 regulation of extracellular fluid volume by, 737-738 regulation of HCO3 by, 739-740 regulation of plasma PO4 by, 740 regulation of sodium by, 737-738 regulation of water content by, 738-739, 739f rheumatoid arthritis in, 1758t structure of, 737-740, 737f in systemic lupus erythematosus, 1772 Kidney donation, 843-844 Kidney stones, 811. See also Nephrolithiasis calcium stones, 811-812, 814-815 clinical manifestations of, 812 CT findings, 812, 813f cystine stones, 812, 816 imaging, 733, 734f incidence of, 811 initial surgical treatment for, 814 medical evaluation of, 814 medical expulsive therapy for, 813-814 obesity and, 811 prevention of recurrence, 814-816 recurrent, 814 struvite stones, 812, 815-816 types of, 811 uric acid stones, 812, 815 Kikuchi’s disease, 1267 Killer inhibitory receptors (KIRs), 217 Kineret (anakinra), 169-170, 1762 Kininogen, high-molecular-weight, 1178

Index Kinins, 233 Kirk-Richardson syndrome, 1661 Kissing ulcers, 908 Kit mutations, 1700 Klatskin’s tumors, 1339, 1340f Klebsiella, 1863t, 1864 Klebsiella granulomatis, 2001 Klebsiella pneumoniae carbapenemase (KPC), 1864, 1961 Klinefelter’s syndrome, 1565 Klippel-Trenaunay syndrome, 958 Klotho, 836 Knee disorders, 1753-1754 bursitis conditions, 1750t housemaid’s knee, 1753 jumper’s knee, 1753 osteoarthritis, 1757, 1757f rheumatoid arthritis, 1757, 1757f tendinitis conditions, 1750t Knee replacement anesthetic approaches for, 2618t partial, 1830 total, 1830, 1831f unicompartmental, 1831, 1831f Knock-knee deformity, 1670-1671 Koate-DVI (Talecris), 1170 Koch-Weeks bacillus, 1947 Koilonychia, 2707 Konzo (cassava tuberis), 2510 Koplik’s spots, 2202, 2202f, 2580-2581, 2672 Korean Americans, 15 Korotkoff sounds, 26 Korsakoff ’s psychosis, 2508 Korsakoff ’s syndrome, 2384, 2508 Kostmann, Rolf, 1133 Kostmann’s syndrome, 1133-1134 Krabbe disease, 2479 Kristalose (lactulose), 893t Krukenberg’s tumor, 1317 Kupffer cells, 1020 Kuru, 2504 Kussmaul breathing, 2411 Kussmaul’s sign, 452 Kwashiorkor, 1434-1435, 1435f diagnosis of, 1436 global rates, 1434 marasmic, 1434 Kyasanur Forest disease, 2249t, 2263t, 2268 clinical manifestations of, 2250-2251, 2251t epidemiology of, 2250 pathobiology of, 2250, 2251t symptoms and signs of, 2250-2251 vaccine for, 2255-2256 Kyasanur Forest disease virus, 91t, 2249t Kyphoplasty, 1644 Kyphoscoliosis, 629-630 clinical manifestations of, 629 diagnosis of, 629 epidemiology of, 629 pathobiology of, 629 prognosis for, 630 treatment of, 629b-630b Kyphosis, 629

L

LA assay, 1180 La Crosse encephalitis, 2263t, 2268-2269 Labetalol, 430t for aortic dissection, 495-496 for autonomic nervous system dysfunction, 1930 for hypertension, 388t, 394, 394t, 1615t for hypertensive emergency, 395-396, 396t for hypertensive urgency, 396 Labile factor (factor V) deficiency, 1175t, 1179 Labor and delivery, 1621 Laboratory testing anomalies, 3.e1t-3.e3t bleeding disorder panels, 1155-1156 cardiac catheterization, 295-298, 292.e3f interpretation of, 38, 39f, 39t multiple testing, 39 ordering, 37-39, 39t performance characteristics, 37-38 preoperative, 2612t and probability, 38, 38f reference intervals, 2712 selection of, 39-40

Laboratory testing (Continued) values, 2712 when to order, 37-39 Labyrinthine concussion, 2599 Lachesis snakes, 719 Lacosamide for myoclonus, 2466-2467 for seizures, 2407t-2408t Lacrimal gland tumors, 2572 Lacrimal secretion, 1787-1788, 1788t β-Lactamase inhibitors, 1933, 1934t β-Lactamases AmpC, 1961 extended-spectrum, 1961-1962 narrow-spectrum, 1961 β-Lactams, 1889-1890 for bacterial meningitis, 2488t mechanism of action, 1889t resistance to, 1890t, 1960, 1961t for S. pneumoniae infection, 1905t for UTIs, 1896 Lactase deficiency, 929 clinical manifestations of, 929 diagnosis of, 929-932 epidemiology of, 929 pathobiology of, 929 Lactate dehydrogenase, 990 Lactated Ringer’s solution burn resuscitation formula, 714.e1t for shock, 676, 680t Lactation, 1619.e1f Lactic acidosis, 767 clinical manifestations of, 767 diagnosis of, 767 epidemiology of, 767 in HIV infection, 2303t mitochondrial encephalomyopathy with lactic acidosis and strokelike episodes (MELAS), 1652t, 1660-1661, 2546 pathobiology of, 767 prognosis for, 767 treatment of, 767b Lactobacillus GG, 922 Lactulose (Kristalose) for acute liver failure, 1033 for hepatic encephalopathy, 1030 for irritable bowel syndrome, 893t Lacunar stroke syndromes, 2436t Laguna Negra virus, 2249t Lambert-Eaton myasthenic syndrome, 2548t, 2551-2552 associated autoantibody, 2518t clinical manifestations of, 2552 definition of, 2551-2552 diagnosis of, 2552 differential diagnosis of, 2552 epidemiology of, 2551-2552 pathobiology of, 2552 prognosis for, 2552 treatment of, 2552b Lambrolizumab, 1376 Lameness, 2338 Lamivudine, 2179, 2288t adverse effects of, 2178t CSF-to-plasma ratios, 2331t fixed-dose combinations, 2289, 2290t for hepatitis B, 996b, 1002, 2177t for HIV/AIDS, 2290t mechanisms of excretion and thresholds for dose adjustment, 2178t side effects of, 2289 Lamotrigine, 2408t for bipolar disorder, 2350 for chronic pain, 137 for neuromyotonia, 2552-2553 for seizures, 2407t-2408t for short unilateral neuralgiform headache with conjunctival injection and tearing, 2361 for trigeminal neuralgia, 2363 Lampit (nifurtimox), 2118 Lance-Adams syndrome, 2466-2467 Langat encephalitis, 2263t Langat virus, 2268 Langerhans cell histiocytosis, 2689-2690 infiltration of hypothalamus, 1477, 1477f pulmonary, 576, 586-587, 587.e1f clinical manifestations of, 587 diagnosis of, 587 treatment of, 587b

Langerhans cells, 2633 Laninamivir, 2196, 2196t Lansoprazole, 565, 900t, 932 Lanthanum carbonate, 778, 778t, 782 Laparoscopic-assisted double balloon enteroscopy, 1326-1327, 1322.e1 Lapatinib (Tykerb), 171, 1211t-1216t, 1232t Laplace equation, 461-463 Laplace’s law, 265 Laquinimod, 2478 Large B-cell lymphoma intravascular, 1265 mediastinal, 1265 Large bowel adenocarcinoma clinical manifestations of, 1328, 1328f radiation therapy for, 1330 sites of development of, 1327-1328, 1327f Large bowel cancer classification of, 1329 radiation therapy for, 1330 Large cell carcinoma lung carcinoma, 1304 neuroendocrine carcinoma, 1313 Large cell lymphoma, cutaneous anaplastic, 1266 Large granular lymphocyte syndrome, 1135, 1759 Large granular lymphocytosis, 1254t, 1255 Large granular lymphoproliferative disorder, 1255 Large intestine. See also Colon neoplasms of, 1322-1331 Large plaque parapsoriasis, 2669, 2669f Large pupils, 2560 Large T-cell lymphoma, anaplastic, 1261t Large-vessel vasculitis, 956 clinical manifestations of, 1796, 1796.e1f names of, 1794t pathophysiology of, 1795 Laronidase (Aldurazyme), 1734 Laryngeal dystonia, 2463-2464, 2627 Laryngeal paralysis, 2606 Laryngeal tumors, 1299 Laryngitis, 2601 chronic, 2602, 2602f systemic diseases, 2605 Laryngopharyngeal reflux, 2604-2606 Laryngospasm, antipsychotic-induced, 2626t Laryngotracheal injury, 528t Larynx: cancer of, 1298, 1301 Laser in situ keratomileusis (LASIK), 2558 Laser therapy, 2661 Lassa fever, 2249t clinical manifestations of, 2250, 2251t, 2252f fatality rate, 2060t management of, 2254 pathobiology of, 2250, 2251t prevention of, 91 prognosis for, 2256 symptoms and signs of, 2250-2251 vaccine for, 2255-2256 Lassa virus, 91t, 2247, 2249t Latanoprost, 2566 Lateral epicondylitis, 1752 Lateral flow analysis, 2077 Lateral sinus thrombosis, 2499, 2499f clinical manifestations of, 2499 diagnosis of, 2499 prognosis for, 2499 treatment of, 2499b Latex allergens, 1699 Latex allergy, 1698, 2618 Latin America, 2116, 2275 Latinos access to health care, 15 population, 15 quality of health care, 15-16 Laughing gas, 2508 Laurence-Moon-Biedl-Bardet syndrome, 1479 Laxatives, 893t LDK378, 1311 for lung cancer, 1310t Lead poisoning, 92 chelators for, 93t chronic renal failure in, 798 clinical features of, 92, 2524t diagnosis of, 92 diagnostic testing for, 93t

I59

Lead poisoning (Continued) epidemiology of, 92 pathobiology of, 92 prognosis for, 92 toxic levels, 703t treatment of, 92b, 706t-710t Lean body mass, 1433 Lean or Six Sigma, 45 Lean tissue: restoration of, 1433 Leber’s hereditary optic neuropathy, 2546, 2575 Lebrikizumab, 553 Lecithin-cholesterol acyltransferase, 1390t, 1391 Lectin receptors, 217 Lederle, 1904 Leeches, 2172t, 2177 Leflunomide (Arava), 168-169, 1761, 1761t Left anterior fascicular block, 270-271, 271f-272f, 271t Left bundle branch block, 270-271, 271f-272f, 271t, 329.e1f Left posterior fascicular block, 270-271, 271f-272f, 271t Left shift, 1130 Left ventricular aneurysm, 453 Left ventricular dysfunction, 27 aortic regurgitation with, 471 heart failure due to practical guidance on use of ACEIs and ARBs for, 310t practical guidance on use of mineralocorticoid receptor antagonists for, 312t practical guidance on use of β-blockers for, 311t management of, 452 mitral regurgitation with, 469 Left ventricular ejection fraction preserved, 317 three-dimensional echocardiography of, 274-275, 276f Left ventricular enlargement, 283, 283f Left ventricular failure, 535f Left ventricular free wall rupture, 453 Left ventricular hypertrophy, 273, 273f concentric, 461-463 hypertension with, 392 Left ventricular noncompaction, 335, 337f Left ventricular outflow tract obstruction, 411-412 Left ventricular stroke work index (LVSWI), 687t Left ventricular thrombus, 280t-281t Left ventricular volume, 274-275, 276f Leg(s) painful, 2469 skin lesions in, 2702t ulcers of, 1101 Legionella, 1993 Legionella feeleii pneumonia, 1995, 1995f Legionella infections (legionellosis), 477t, 1993-1996 clinical manifestations of, 1994-1995 definition of, 1993 diagnosis of, 1995 epidemiology of, 1993-1994 pathobiology of, 1994 prognosis for, 1996 treatment of, 1996b Legionella longbeachae, 1993-1994 Legionella micdadei, 1993 Legionella pneumophila, 1993, 1993f Legionnaires’ disease, 1993, 1995.e1f clinical manifestations of, 1994-1995 community-acquired, 1995, 1995f diagnosis of, 1995, 1995f differential diagnosis of, 1995 epidemiology of, 1993-1994 nosocomial, 1995.e1f pathobiology of, 1994 prognosis for, 1996 radiographic findings, 612, 612f, 1995, 1995f-1996f risk factors for, 1994, 1994t treatment of, 1996b, 1996t Leigh’s syndrome, 2546 Leiomyoma, gastric, 1319 Leiomyosarcoma, gastric, 1319 Leishmania, 1015t-1017t, 2120, 2121f, 2121t Volume I  pp 1-1382  •  Volume II  pp 1383-2722

I60

Index

Leishmania donovani infection, 2123 Leishmania infantum infection, 2123 Leishmania major, 2123, 2125f Leishmania mexicana, 2057 Leishmaniasis, 2120-2125 cutaneous, 2057t, 2123-2125 epidemiology of, 2120 treatment of, 2123b-2125b, 2125.e1f definition of, 2120 dermal, post–Kala-azar, 2122 epidemiology of, 2120 mucosal, 2123 pathobiology of, 2120 prevention of, 2125 prognosis for, 2125 treatment of, 2107 vectors, 2172t visceral, 2120-2122 epidemiology of, 2120 fatality rate, 2060t Leland Stanford Junior University Board of Trustees, 6-7 Lemierre’s syndrome, 2603 Lenalidomide (Revlimid) for cancer, 1211t-1216t for multiple myeloma, 1279-1280, 1280t for myelodysplastic syndrome, 1237-1238 Lennox-Gastaut syndrome, 2404, 2406 Lens, 2556 Lenticulostriate arteries, 2429f Lentigo maligna melanoma, 1374, 1375f Leopard skin, 2169 Lepidic predominant nonmucinous adenocarcinoma, 571t, 574-575, 575b Lepidopterids, 2177 Lepirudin, 178, 1618t Leprosy (Hansen’s disease), 2042-2046, 2640f autonomic neuropathy of, 2520 biopsies, 2045 borderline forms, 2043-2044, 2043f borderline lepromatous, 2043, 2043f borderline tuberculoid, 2043, 2043f clinical examination findings, 2045 clinical manifestations of, 2044-2045 definition of, 2042 diagnosis of, 2045 epidemiology of, 2042-2043 genetics of, 2043 and HIV, 2045 immunology of, 2043 lepromatous, 2043-2044, 2043f-2044f, 2691, 2692f multibacillary, 2043, 2043f, 2046 neuropathy associated with, 2536 pathobiology of, 2043 pathogenesis of nerve damage in, 2043 paucibacillary, 2043, 2043f, 2046 prevalence of, 2042 prevention of, 2046 prognosis for, 2046 reactional states, 2044-2046 skin smears, 2045 Strategic Plan for the Elimination of Leprosy (WHO), 2042 testicular hypogonadism due to, 1573 treatment of, 2045b-2046b agents for, 2045-2046 regimens for, 2046 response to, 2046 tuberculoid, 2043-2044, 2043f-2044f Leptomeningeal metastasis, 1295, 1295f, 2599 Leptospira, 2028, 2029f Leptospirosis, 2028-2030, 2058t-2059t clinical manifestations of, 2028-2029, 2029f definition of, 2028 diagnosis of, 2030 differential diagnosis of, 2030 distinguishing characteristics of, 2252t-2253t epidemiology of, 2028 pathobiology of, 2028 pathologic features of, 2028, 2029f prevention of, 2030 prognosis for, 2030 severe, 2028-2030, 2029f treatment of, 2030b Lesions. See also Skin lesions atrophic, 2693-2694, 2693f, 2693t, 2702t auditory pathway, 2594

Lesions (Continued) blue, 2581t breast, benign, 1359-1360 nonproliferative, 1359 proliferative, 1359 groin, 2702t Janeway’s, 255, 257f, 475, 475t macular, 2581t, 2582-2583 maculopapular, 2581t melanocytic, 2689 mucous retention, 2584 oculomotor nerve, 2576, 2576f optic chiasm, 2575 optic nerve, 2576, 2576f optic tract, 2575 oral. See Oral lesions pretectum, 2576, 2576f in pupillary pathways, 2575 rupial, 2015 scalp, 2702t sclerotic, 2694 skeletal, 1281 squamous intraepithelial, 2219, 2221 anal, 972 cervical, 2221 prevalence of, 2220 syphilis, 2014, 2014f vascular, 958, 2690 warty, 2688 white, 2581-2582, 2581t Let-down reflex, 1487 Letermovir, 2231 Letrozole (Femara), 1211t-1216t Leucine aminopeptidase, 989 Leucine sensitivity testing, 1555 Leucovorin (Wellcovorin) for AIDS-associated opportunistic infections, 2297t-2301t for chemotherapy side effects, 1211t-1216t for colorectal cancer, 1330 for congenital toxoplasmosis, 2132t for Pneumocystis pneumonia prevention, 2098, 2098t Leukemia acute, 1239-1246 acute lymphoblastic. See Acute lymphoblastic leukemia acute lymphocytic, 1201 acute lymphoid, 1243t acute myeloid. See Acute myeloid leukemia acute promyelocytic, 1243t, 1245 adult T-cell leukemia/lymphoma, 1255, 1266, 1266b, 2236-2238, 2237t acute, 2238 chronic, 2238 smoldering, 2238 arthritic manifestations of, 1825-1826 blood smear features of, 1058-1059 central nervous system, 1243 chronic, 1246-1257 chronic lymphocytic. See Chronic lymphocytic leukemia chronic myelogenous. See Chronic myelogenous leukemia chronic myeloid. See Chronic myelogenous leukemia chronic neutrophilic, 1131 distinguishing characteristics of, 2252t-2253t graft-versus-leukemia effect, 1199 hairy cell, 1252-1253, 1252f differential diagnosis of, 1252-1253, 1254t treatment of, 1253b lymphoblastic, 1262 acute. See Acute lymphoblastic leukemia lymphocytic acute, 1201 chronic. See Chronic lymphocytic leukemia myelogenous. See Chronic myelogenous leukemia myeloid. See Acute myeloid leukemia plasma cell, 1281 prolymphocytic, 1254 Leukemoid reactions, 1130, 1130f Leukeran (chlorambucil), 1211t-1216t Leukine (sargramostim), 1211t-1216t Leukocyte adhesion defects, 1146-1148 clinical manifestations of, 1683 pathobiology of, 1683

Leukocyte adhesion deficiency, 1131, 1146-1148, 1682.e1t Leukocyte adhesion deficiency type 1 (LAD-1), 1146-1147, 1147t-1148t clinical manifestations of, 1147 diagnosis of, 1147 pathobiology of, 1146-1147 treatment of, 1147b Leukocyte adhesion deficiency type 2 (LAD-2), 1147-1148, 1147t-1148t Leukocyte adhesion deficiency type 3 (LAD-3), 1147-1148, 1147t-1148t Leukocyte selectin (L-selectin), 1144 Leukocyte-reduced blood components, 1192 Leukocytes corticosteroid effects on, 163 urine dipstick test for, 729 Leukocytoclastic angiitis, cutaneous clinical manifestations of, 1799 management of, 1799 pathologic characteristics of, 1795t Leukocytoclastic vasculitis, 2674, 2674f, 2684, 2686 Leukocytosis, 1129-1138 antipsychotic-induced, 2626t blood smear features of, 1058 due to chronic inflammation, 1130 due to expansion of other cell lines, 1132 in nonhematologic malignancy, 1130 postoperative, 2624 secondary causes of, 1130-1131 spillover, 1131 Leukoderma, guttate (“raindrop”), 2699, 2699f Leukodystrophy, 2479 globoid cell, 2479 metachromatic, 1402-1403, 2479 Leukoencephalopathy cerebral autosomal dominant arteriopathy with subcortical infarcts and (CADASIL), 2394 posterior syndrome, 2575 progressive multifocal, 2210-2212, 2502t Leukoerythroblastic blood film, 1059t Leukoerythroblastic reaction, 1130 Leukokoria, 2560, 2572 Leukonychia, 2708 Leukopenia, 1129-1138 blood smear features of, 1058 due to deficiency of other cell lines, 1138 Leukoplakia, 1299, 1299f, 2581 hairy, 2582, 2582f oral, 2234, 2319, 2644f Leukostasis, 1243 Leukotriene A4 (LTA4), 232 Leukotriene modifiers, 1692 Leukotrienes, 232, 549 Leuprolide (Lupron, Lupron Depot), 1211t-1216t, 1369 Leustatin (cladribine), 1211t-1216t Levetiracetam, 2408t for myoclonus, 2466-2467 for seizures, 1033, 2407t-2408t for trigeminal neuralgia, 2363 Levitra (vardenafil), 429 Levodopa effects on micronutrient status, 1453t for Parkinson’s disease, 2457t-2459t, 2460 for restless legs syndrome, 2468-2469 Levofloxacin for AIDS-associated opportunistic infections, 2297t-2301t for cholangitis, 1043 for cystitis, 1875t for diarrhea, 922, 1884 for diverticulitis, 947 for gallstones, 1041 for H. pylori, 915t for legionnaires’ disease, 1996, 1996t for osteomyelitis, 1810t for peritonitis, 949 for plague, 89b for pneumonia, 616t, 619t for prostatitis, 831-832 for pyelonephritis, 1875t for pyogenic liver abscess, 1012 recommended doses and schedules, 1891t-1892t for rhinosinusitis, 2588 for S. pneumoniae infection, 1905t for septic arthritis, 1810t for septic shock, 689t

Levofloxacin (Continued) for traveler’s diarrhea, 1884 for tuberculosis, 2312t for ulcerative colitis, 942 for urethritis, 1878t Levonorgestrel, 1609-1610 Levorphanol, 139t-140t Levosimendan, 683 Levothyroxine (T4), 2616 Lewis-Summer syndrome, 2532, 2534 Lewy body dementia, 2395-2396 Lexapro (citalopram), 893t Leydig cell agenesis or hypoplasia, 1564-1565 LFB, 1170 Lhermitte’s sign, 2341 Lialda (mesalamine), 939t Libido, decreased, 1576 Lice, 2175-2176, 2698 body lice, 2175, 2175b, 2175f crab lice, 2176, 2176b head lice, 2175, 2175b pubic lice, 1877t treatment of, 2175b-2176b Lichen myxedematosus, 2694 Lichen nitidus, 2668, 2668f Lichen planopilaris, 2703t, 2705, 2705f Lichen planus, 2581-2582, 2581f, 2637f, 2640f, 2645f, 2667, 2668f clinical manifestations of, 2667 color in, 2638t-2644t diagnosis of, 2667 epidemiology of, 2667 esophageal, 906 hypertrophic, 2667 oral, 2580t, 2667 pathobiology of, 2667 therapeutic options for, 2668b, 2668t of tongue, 2667, 2668f Lichen sclerosis, 2644f Lichen simplex chronicus, 2654f Lichenification, 2636t, 2651t-2654t Liddle’s syndrome, 747, 823t, 824 Lidocaine for advanced cardiac life support, 354f for arrhythmias, 355 for chronic pain, 137, 138t for electrical storm, 373 pharmacokinetic parameters, 126t for pruritus, 2636t in renal failure, 129t for short unilateral neuralgiform headache with conjunctival injection and tearing, 2361 for ventricular tachycardia, 373 Life expectancy, 103, 103t in end-stage renal disease, 843, 843f worldwide, 100 Life support advanced cardiac life support (ACLS), 30 after acute poisoning, 705 after cardiac arrest, 353-354, 354f-355f advanced trauma life support, 30 basic life support after acute poisoning, 705 CAB (compression-airway-breathing) algorithm for, 353 after cardiac arrest, 353 intermediate, 353 withdrawal of, 651-652, 651t Lifespan groups, 1600 Lifestyle management for breast cancer prevention, 1359 for diabetes prevention, 1537 for drug use disorders, 158 for heart failure, 315-316 for hypertension, 387 for ischemic heart disease, 428, 429f for lipid disorders, 1395 to lower blood pressure, 387t for osteoporosis, 1642 for type 1 diabetes, 1531 for type 2 diabetes, 1534-1535, 1537 Lifestyle prescription, 60 Li-Fraumeni syndrome, 1227t, 1370 Ligament(s), 1730t Ligamentous abnormalities, 1746 Ligamentous injury, 2367 Light transmission aggregometry, 1156 Light-chain deposition disease, 792-793 Lightheadedness, 248-251 Light-near dissociation, 2575-2576

Index Lightning injury, 716 Lights flashing, 2559 halos around, 2559 Likelihood ratio (LR), 37t, 38 Limb ischemia acute clinical categories of, 499t clinical manifestations of, 499 diagnosis of, 500 pathobiology of, 498-499 treatment of, 500-501, 502f chronic, 499-500 critical, 503f Limb surgery, 2618t Limb-girdle muscular dystrophy, 2543 Limbic encephalitis, 2502t Limbic seizures, 2401t Linaclotide (Linzess), 889, 893t Lincosamides, 1891t-1892t Linear hyperpigmentation, 2700 Linear hypopigmentation, 2699 Linear immunoglobulin A bullous dermatosis, 2676 Linear regression, multiple, 34 Linezolid, 1894-1896 drug interactions, 1846 for enterococcal infection, 1915.e1t for furuncles, 2696 mechanism of action, 1889t for nocardiosis, 2064 for osteomyelitis, 1810t for pneumonia, 616t, 619t recommended doses and schedules, 1891t-1892t resistance to, 1890t for S. aureus infection, 1901 for S. pneumoniae infection, 1905t for septic arthritis, 1810t Linkage, 1722 Linkage disequilibrium, 190-191 Linzess (linaclotide), 893t Lionfish (Scorpaenidae), 721 Lionfish injuries, 721 Liothyronine (T3), 2616 Lipase, 961 Lipid disorders diagnosis of, 1394-1395 laboratory evaluation of, 1395 prevention of, 1396-1397 risk assessment, 1395 secondary causes of, 1394, 1394t treatment of, 1395b-1396b medical therapy, 1395-1396, 1396t preferred medications for women, 1602t Lipid emulsions, 706t-710t, 1443 Lipid profiles, fasting, 454 Lipid transport, 1389-1391, 1390t Lipid-activated nuclear receptors, 1392-1393 Lipid-lowering therapy for acute coronary syndrome, 438t-439t for acute MI, 451, 454, 454t perioperative, 2613t Lipids abnormal, 54-55 blood levels, 258 erythrocyte membrane, 1080 metabolism of disorders of, 1393-1394, 2542t drugs that alter, 428 endogenous, 1391-1392, 1391f exogenous, 1391 lipid-activated nuclear receptors and, 1392-1393 Lipoaccumulation, 2291 Lipoatrophy, 2291 facial, 2322, 2322f Lipodystrophy autoinflammation, lipodystrophy, and dermatosis syndrome, 1740t HIV lipodystrophy syndrome, 2322, 2322f Lipoid hyperplasia, 1562 Lipoid nephrosis, 785 Lipoid pneumonia, 601-602, 602f clinical manifestations of, 602 definition of, 601 diagnosis of, 602, 602f pathobiology of, 601-602 treatment of, 602b

Lipolysis, impaired, 927 Lipomas, 1292 Liponyssoides sanguineus, 2052 Lipoprotein (a), 1389t Lipoprotein lipase, 1390-1391, 1390t Lipoproteins characteristics of, 1389t lipid transport by, 1389 Liposomal amphotericin B for AIDS-associated opportunistic infections, 2297t-2301t for invasive aspergillosis, 2086, 2086t for visceral leishmaniasis, 2122-2123 Liposomal doxorubicin (Doxil), 1211t-1216t Lipotoxicity, 1021 5-Lipoxygenase, 232 Liquid nitrogen, 2222t Lisch nodules, 1523 Lisinopril for acute MI, 450-451, 454-455, 454t for aortic dissection, 496 for heart failure, 310t for hypertension, 388t for left ventricular dysfunction, 452 Lissencephaly, 2512 Listeria meningitis, 1920, 2480, 2487 Listeria monocytogenes, 1918 Listerial rhombencephalitis, 1919 Listeriosis, 1918-1920, 2058t clinical manifestations of, 1919 definition of, 1918 diagnosis of, 1919 differential diagnosis of, 1919 epidemiology of, 1918 incidence of, 1918 laboratory findings, 1919 pathobiology of, 1918 prevention of, 1919 prognosis for, 1920 treatment of, 1919b-1920b Lisuride, 2457t-2459t Lithium for bipolar disorder, 2350 for cluster headache, 2361 pharmacokinetic parameters, 126t toxic ingestion of, 699t-703t, 710t Lithium carbonate, 129t Livedo racemosa, 506, 506t, 507f Livedo reticularis, 506 Liver. See also under Hepatic alcohol-related complications, 151t, 152 anatomy of, 1340f antimicrobial therapy and, 1845-1846 antipsychotic-induced impairment of, 2626t bilirubin disposition by, 983-984 examination of, 27 fatty, 80, 1009-1010 acute, of pregnancy, 1622 nonalcoholic disease, 1020t, 1021-1023, 1022b, 1022f glycogen release/storage in, 1552 glycogen synthesis and degration in, 1397-1398, 1397f infections of, 1011-1018 bacterial, 1012 helminthic, 1014-1018 parasitic, 1013-1018, 1015t-1017t protozoal, 1013-1018 injury mechanisms, 1008, 1008.e1f lesions of, 868f, 869 Liver abscesses amebic, 1013, 2140 drug therapy for, 2141t features of, 1011t bacterial, 1011t pyogenic, 1011-1012, 1011f Liver biopsy, 980, 982f, 991 Liver cancer, 1339-1345 Liver disease abdominal examination in, 978 abdominal imaging in, 980 advanced, 977-978 alcoholic, 1019-1021 approach to, 976-983, 979t assessment of duration of, 977 with biliary tract disease, 987-991 chronic anemia of, 1066 cryptogenic, 1006

Liver disease (Continued) dietary factors, 1427t in pregnancy, 1623, 1623b coagulation abnormalities in, 1183, 1184b, 1184t diagnostic studies, 979-980, 979t, 988f drug dose adjustments in, 129 drug-induced, 1000t, 1006-1010 end-stage, 1005 family history in, 977 fungal, 1012-1013 granulomatous, 1018-1019 gut microbiota and, 1842 hematologic tests in, 990 HFE mutation analysis in, 1423.e1f history in, 976-977, 991 in HIV infection, 2303-2304 imaging studies, 991-992 inherited, 982-983 laboratory findings, 979-980, 991 liver biopsy in, 980 Model for End-Stage Liver Disease (MELD), 1026t, 2616 mucocutaneous findings, 977-978 mutation analysis in, 1422-1423 parenchymal, 991t physical findings, 977-979, 991 platelet abnormalities in, 1183, 1184t polycystic, 1035 in pregnancy, 1622-1623, 1622b, 1622t preoperative evaluation of, 2616 protozoan, 1014 review of other organ systems, 977 risk factors for, 977 in sarcoidosis, 606 smear features of, 1056t tests for specific diseases, 990 toxin-induced, 1006-1010 vascular, 958-959 Liver failure acute liver transplantation for, 1033, 1034t treatment of, 1033b bleeding in, 1183-1184 clinical manifestations of, 1183-1184 diagnosis of, 1184 pathobiology of, 1183 chronic, 1034 liver transplantation for, 1031-1038 Liver fibrosis, 1023 Liver flukes, 2156-2157, 2156t diagnosis of, 2158 prevention of, 2159 treatment of, 2158b Liver function tests, 989 abnormal, 991-993 based on clearance of metabolites and drugs, 990 reflecting hepatic synthetic function, 990 Liver graft dysfunction, 1035, 1036f Liver transplantation, 1031-1038 for acute liver failure, 1033, 1034t candidates not requiring review, 1033.e1t for chronic liver failure, 1034 for cirrhosis, 1031 contraindications to, 1031, 1032t criteria for, 1034 deceased donor, 1033.e1t evaluation for, 1031 for hepatopulmonary syndrome, 1031 immunosuppressive medications for, 1037-1038, 1037t indications for, 1032-1035, 1032t infectious complications of, 1037, 1037f living donor, 1035 management of, 1038 medical assessment for, 1032 Milan criteria for, 1034 Model for End-Stage Liver Disease (MELD) score for, 1031-1033 preoperative evaluation of donors, 870, 870f priority for, 1033 psychosocial assessment for, 1032-1033 recipient outcomes after, 1035-1038, 1036f rejection of, 1036, 1036f risk factors for poor outcome after, 1035 selection for, 1031-1034, 1034t surgical assessment for, 1032 survival rates, 1031, 1032f

I61

Liver tumors, 1342-1343 Liver X receptors, 1392 Living donor liver transplantation, 1035 Living donor lobar transplantation, 645 Living donor renal transplantation, 846 Living kidney donors, 843 Living-related donor hepatectomy, 1035, 1035f Living-related liver donors, 870, 870f Loa, 2159, 2170 Loa loa, 2103-2104, 2166, 2167t, 2170, 2170f Lobar emphysema, congenital, 570 Lobar hemorrhage, 2451 Lobular carcinoma in situ, 1354, 1354t Lobular panniculitis, 2693 Locally invasive aspergillosis, 2084 Locked-in syndrome, 2409, 2410t, 2414, 2435, 2435.e1f Lockjaw (trismus), 1929 Locus heterogeneity, 191, 194 Loeys-Dietz syndrome, 1735 Löffler’s fibroplastic endocarditis, 336 Löfgren’s syndrome, 604-605, 607 Logistic models, 34 Logistic regression, multiple, 34-35 Loiasis, 2170 clinical manifestations of, 2170, 2170f definition of, 2170 diagnosis of, 2170 epidemiology of, 2170 pathobiology of, 2170 prevention of, 2170 treatment of, 2165t, 2170b Long noncoding RNA (lncRNA), 189 Long QT syndrome, 187t, 373f Long QT syndrome type 3 (LQT3), 341 Long terminal repeats, 2236 Longevity, 102 Loop diuretics, 748t contraindications to, 389t for hypertension, 388t side effects of, 389t Loop natriuretics, 748 Loop of Henle disorders, 823-824, 823t Loop recorders, 349 Loose anagen hair syndrome, 2706 Looser’s zones, 1646 Loperamide (Imodium) for diarrhea, 922, 933-935, 2551 for gastroenteropathy, 934 for irritable bowel syndrome, 893t for small bowel rapid transit dysmotility, 888 Lopinavir, 2288t CSF-to-plasma ratios, 2331t fixed-dose combinations, 2289, 2290t for HIV/AIDS, 2290t side effects of, 2289 Loratadine (Claritin), 2660t for allergic rhinitis, 2588 for urticaria, 1695, 2684t Lorazepam for alcohol withdrawal, 155t for amyotrophic lateral sclerosis, 2525 for anxiety, 2352t for muscle spasms, 1930 for nausea and vomiting, 865t psychoactive effects of, 116 for seizures, 2408, 2489 for status epilepticus, 2408 for tachypnea, 31 Lorcaserin, 1466 Lorenzo’s oil, 2479 Losartan for acute MI, 450-451, 454-455, 454t for heart failure, 310t for hypertension, 388t, 392 Loss of consciousness, transient, 345-346, 346t Loteprednol, 2570 Lotronex (alosetron), 893t Louping ill encephalitis, 2263t, 2268 Louse-borne relapsing fever, 2060t Louse-borne typhus, epidemic, 2051-2052 Louse-transmitted disease, 2048t, 2051-2052 Lovastatin, 1396t Low back pain. See also Back pain ancillary testing in, 2373-2374 chronic, 142t clinical examination in, 2373 Volume I  pp 1-1382  •  Volume II  pp 1383-2722

I62

Index

Low back pain (Continued) clinical manifestations of, 2372 differential diagnosis of, 2374-2375 mechanical, 2374, 2375t prolotherapy for, 2375 refered pain, 2374 treatment of, 142t, 2375 Low-copy repeats (LCRs), 189 Low-density lipoprotein receptor, 1390, 1390t Low-density lipoprotein receptor–related protein, 1390, 1390t Low-density lipoproteins characteristics of, 1389t intensive lowering of, 1395 Löwenstein-Jensen culture, 2036-2037 Lower extremity cystic adventitial disease of arteries in, 504 joints affected in osteoarthritis, 1747, 1747f Lower gastrointestinal bleeding, 882 acute, 875-876, 875f definition of, 882 diagnosis of, 882 epidemiology of, 882 incidence of, 879-880 Lower gastrointestinal tract, 1782 Lower genital tract gonorrhea, in females, 1942 antibiotic regimens for, 1945t clinical manifestations of, 1942 diagnosis of, 1942 prognosis for, 1942 Lower limb ischemia, chronic classification of, 499t critical, 499 stable, 499 treatment of, 501 Lower respiratory tract infections, enteroviral, 2241t Lower urinary tract abnormalities of, 827 causes of established incontinence related to, 111 Lowe’s syndrome, 823t Low-intensity vibration, 1644 Low-molecular-weight heparin (LMWH), 177, 177f, 177t for acute coronary syndrome, 439 for acute STEMI, 448, 450 for anticoagulation, 516t for deep venous thrombosis, 515, 517, 2368 in pregnancy, 517, 1617, 1618t for pulmonary embolism, 624, 625t for venous thromboembolism, 517-518, 1617 LPL gene therapy, 1393 Lubiprostone (Amitiza), 889, 893t Lubricants, 2660 Luciferase immunoprecipitation systems (LIPS) assays, 2163 Lugol’s solution, 1508 Lujo hemorrhagic fever, 2249t, 2250-2251, 2251t Lujo virus, 2247, 2249t Lumbar puncture, 2341, 2439-2440 Lumbar radiculopathy, 2378 Lumbosacral radiculopathy, 142t Lumbosacral radiculoplexus neuropathy, diabetic, 2535 Lumefantrine, 2105, 2111, 2111t Luminal endoscopy, 872-873, 873t Lung(s). See also under Pulmonary carbon dioxide elimination, 763 congenital cystic adenomatoid malformation of, 570 examination of, 27, 303 hyperlucent, 570 imaging of, 531-538 mechanics of, 557 in rheumatoid arthritis, 1758t, 1759f shrinking, 585 in systemic lupus erythematosus, 1773 Lung auscultation, 526, 526t Lung cancer, 1303-1313 bronchogenic, 1303-1313 carcinogenesis of, 1306, 1307f clinical manifestations of, 1306-1307 definition of, 1303 diagnosis of, 1307-1309 diagnostic imaging, 1309

Lung cancer (Continued) diagnostic procedures for, 1307-1309 early detection of, 1312-1313 epidemiology of, 1303-1304 epigenetics of, 1306 histologic groups, 1303 in HIV infection, 2315-2316, 2322, 2323t clinical manifestations of, 2316 epidemiology of, 2315 pathobiology of, 2315-2316 prognosis for, 2316 radiographic features of, 2309t treatment of, 2316b incidence of, 1303 metastatic, 644 molecular pathology of, 1304-1306 non-small cell advanced-stage, 1310-1311 early-stage, 1310 epidemiology of, 1303 histologic markers, 1304t locally advanced, 1310-1311 pathology of, 1304 radiation therapy for, 1310 stage I, 1310 stage II, 1310 stage III, 1310 stage IV, 1310 subtypes, 1304 surgery for, 1309-1312 treatment of, 1309b-1312b nutritional influences, 1426 occupational, 589t, 595 oncogenes, 1304-1306, 1305f pathobiology of, 1304-1306 pathology of, 1304 prevention of, 1312-1313 risk factors for, 1303-1304 screening for, 56 small cell epidemiology of, 1303 pathology of, 1304 treatment of, 1312 in special populations, 1312 staging, 1309 supportive care for, 1312 surveillance, 1312 survivorship, 1312 symptoms and signs of, 1306-1307 treatment of by AJCC stage, 1309t chemotherapy, 1311 in HIV infection, 2316b maintenance therapy, 1311 radiation therapy, 1310 salvage therapy, 1311 surgery, 644 systemic therapy, 1310-1312 targeted therapy, 202, 202t, 1310t Lung disease advanced, 644 anatomic distribution of, 535-536, 535t apical, 536f basal, 536f benign, 643-644 chronic, 657t in cystic fibrosis, 564, 564f diffuse, 532-536, 535t diffuse reticular, 534f distribution patterns, 535t with hypertension, 403 interstitial, 575-588, 1782-1783 interventional approaches to, 642-647 malignant imaging of, 537-538 surgery for, 644 obstructive disorders, 542, 543t, 544 occupational, 588-595 peripheral, 535-536, 536t restrictive disorders, 542, 543t surgical approaches to, 642-647 Lung failure, 10.e1f. See also Respiratory failure Lung injury, 600-603 acute acute respiratory failure in, 661-662 transfusion-related, 602-603, 1197 chemical, 595-603 physical, 595-603 thermal, 598 ventilator-induced, 667-668, 668f Lung resection, 2618t

Lung transplantation, 645-647 bilateral, 403, 645 complications of, 646-647 desicision guide for, 645t evaluation of potential transplant recipients, 645-646 heart-lung transplantation, 403, 645 indications for, 645, 645t infections after, 646 living donor lobar transplantation, 645 outcomes, 647, 647f post-transplantation issues, 646-647 prophylaxis after, 646 for pulmonary hypertension, 403 rejection of acute, 646 chronic, 646-647, 646b single-lung, 645 survival estimates, 647, 647f types of procedures, 645 Lung tumors, 1313 Lung volume, 540-542 absolute, 542 end-inspiratory volume, 542f expiratory reserve volume, 540 flow-volume curves, 539-540, 541f imaging of, 535 inspiratory reserve volume, 540 measurement of inert gas dilution technique for, 540-542 nitrogen washout technique for, 542 partitions, 540, 542f residual volume, 540, 542f tidal volume, 540, 542f Lung volume reduction surgery, 644 desicision guide for, 645t exclusion criteria for, 644t inclusion criteria for, 644t mortality, 644, 644f Lupoid hepatitis, 1773 Lupron (leuprolide), 1211t-1216t Lupron Depot (leuprolide), 1211t-1216t Lupus neonatal, 1773-1774 neuropsychiatric, 1773 Lupus anticoagulant, 1180, 1722 Lupus erythematosus, 2641f color in, 2638t-2644t discoid, 2703t oral ulcers, 2580t systemic. See Systemic lupus erythematosus Lupus flares, 1775 Lupus glomerulonephritis, 1772 Lupus nephritis, 792, 1772 classification of, 792, 792t, 1772, 1772.e1t diffuse proliferative, 792, 792f focal proliferative, 792 histopathology of, 1772, 1772f induction therapy for, 1775, 1776f management of, 1775, 1776f membranous, 792 Lupus panniculitis, 2653f, 2693 Lupus pernio, 605, 605f Lupus pleural effusion, 634t Lupus pleuritis, 634 Lupus pregnancy, 1773-1774 Lupus profundus, 2693 Lupus-like reactions, 2685t Lurasidone, 2354 Luteal phase dysfunction, 1594, 1594b Luteal phase inadequacy, 1594 Luteal phase insufficiency, 1594 Luteinized unruptured follicle syndrome, 1594 Luteinizing hormone, 1492-1493 low levels, 1589 mechanism of action, 1585-1586 Luteinizing-hormone–producing tumors, 1493 Lutembacher’s syndrome, 408 Luteogenesis, 1586 Luteoma of pregnancy, 1564 Luxembourg, 7 Lyme arthritis, 1808, 2024 Lyme borreliosis, 2021 Lyme disease, 358-359, 2021-2027, 2058t-2059t cardiac, 2023-2024 chronic, 2026

Lyme disease (Continued) clinical manifestations of, 2022-2024, 2022f coinfections, 2026 definition of, 2021 diagnosis of, 2024-2025, 2024t diagnostic testing for, 2025 differential diagnosis of, 2025 early disseminated infection, 2023-2024 early localized infection, 2023, 2023t early neurologic, 2023 epidemiology of, 2021 extracutaneous, 2025 laboratory testing for, 2024 late, 2024 late neurologic, 2024 neuropathy associated with, 2536 pathobiology of, 2021-2022 post-Lyme disease symptoms and syndrome, 2026 prevention of, 2026 recommended therapy for, 2025t reinfection, 2026 serologic testing for, 2024-2025 treatment of, 2025b-2026b vectors, 2172t Lymph node hyperplasia, angiofollicular, 1267 Lymph node regions, 1270f Lymph nodes anatomy of, 1138 axillary lymph node metastasis, 1379-1380 on chest images, 536 enlarged, 536, 536t evaluation of, 1139 interventional, 1140 methods for, 1140t IASLC map, 1308f regional clinically apparent, 1376 clinically normal, 1376 management of, 1376 sentinel lymph node biopsy, 1376 Lymphadenitis acute, 2168 histiocytic necrotizing, 1267 toxoplasmic, 2127, 2131 tuberculous, 2034 Lymphadenopathy, 1138-1140 approach to, 1140, 1140t causes of, 1139t diagnosis of, 1138-1140, 1139t differential diagnosis of, 1138-1139 fever and, 1852, 1852t imaging in, 1139 localized, 1140t of onchocerciasis, 2169 physiology of, 1138 scalp eschar, neck lymphadenopathy after tick bite (SENLAT), 2048t, 2050-2051 in systemic lupus erythematosus, 1773 toxoplasmosis with, 2130 Lymphangioleiomyomatosis, 587, 587b, 587.e1f Lymphangitic carcinomatosis, 528t Lymphangitis-associated rickettsiosis (LAR), 2048t Lymphatic drainage, impaired, 933 Lymphatic filariasis, 2167-2168, 2168f clinical manifestations of, 2167 definition of, 2167 diagnosis of, 2168 epidemiology of, 2167 pathobiology of, 2167 prevention of, 2168 treatment of, 2165t, 2168b Lymphatic system cutaneous, 2635 examination of, 26-27 Lymphedema, 1358, 1652t Lymphoblastic leukemia, 1262 acute. See Acute lymphoblastic leukemia Lymphoblastic lymphoma, 1262 Lymphocutaneous sporotrichosis, 2078, 2078f Lymphocyte alveolitis, pulmonary, 2237t Lymphocyte function-associated antigen-1 (LFA-1, CF11a/CD18), 1145 Lymphocytes, 225-226 in graft rejection, 236t large granular lymphocyte syndrome, 1759

Index Lymphocytes (Continued) vacuolated, 1059t villous, 1254t Lymphocytic choriomeningitis, 2058t, 2490-2491 Lymphocytic colitis, 934 Lymphocytic hypophysitis, 1482 Lymphocytic interstitial pneumonia, 584, 584b Lymphocytic leukemia acute, 1201 chronic. See Chronic lymphocytic leukemia Lymphocytic meningitis, chronic (persistent), 2492-2493, 2492t Lymphocytic (postpartum, painless, silent) thyroiditis, 1509 Lymphocytoma, borrelial, 2023, 2023f Lymphocytopenia, 1138 Lymphocytosis, 1132, 1133t, 1255 Lymphoepithelioma, 1298 Lymphogranuloma venereum, 1878, 2010-2011 Lymphohematopoiesis, 1050, 1051f Lymphohistiocystosis, hemophagocytic, 1147t-1148t, 1150 Lymphoid interstitial pneumonia, 584, 584b, 584.e1f Lymphoid tissue, 225-226 Lymphoma adult T-cell lymphoma/leukemia, 1255, 1266, 1266b, 2236-2238, 2237t acute, 2238 chronic, 2238 smoldering, 2238 AIDS-related, 1267, 2323 CNS prophylaxis and treatment in, 2324 in resource-limited settings, 2324 anaplastic, 1381 arthritic manifestations of, 1825-1826 B-cell, 1263-1265 diffuse large, 2324 rare types, 1265 blood smear features of, 1058-1059 Burkitt’s, 1265, 2324 causative factors, 1228 central nervous system, 1294, 1294f, 2324-2325 classification of, 1240t clinical clues to, 2524t diffuse large B-cell, 1264-1265 diseases confused with, 1267-1268 distinguishing characteristics of, 2252t-2253t double-hit, 1265 effusion, 1265 extranodal marginal zone, 1263 in eye, 2572 follicular, 1263-1264 gastric, 1319 gray-zone, 1265 Hodgkin’s, 1268-1273 hematopoietic stem cell transplantation for, 1202 in HIV infection, 2322-2325, 2323t Kaposi’s sarcoma herpesvirus (KSHV)– associated, 2324 leukemic phase, 1254t lymphoblastic, 1262 lymphoplasmacytic, 1265 mantle cell, 1264 marginal zone, splenic, 1265 mucosa-associated lymphoid tissue (MALT), 1319 cutaneous, 2691, 2691f extranodal marginal zone, 1263 non-Hodgkin’s, 1257-1268 chemotherapy for, 1263t chromosomal translocations characteristic of, 1258, 1260, 1260t classification of, 1258-1260, 1261t clinical manifestations of, 1260-1262, 1261f, 1261t definition of, 1257 diagnosis of, 1262 in elderly, 1267 epidemiology of, 1257 evaluation of, 1262t

Lymphoma (Continued) factors associated with, 1258t genetic factors, 1257-1258 hematopoietic stem cell transplantation for, 1202 in HIV infection, 2309t, 2316, 2322-2325, 2323t immunophenotypes of, 1258, 1260t incidence of, 1257, 1257f infectious agents, 1258 International Prognostic Index, 1262, 1262t Kaposi’s sarcoma herpesvirus (KSHV)–associated, 2324 pathobiology of, 1257-1260 pathology of, 1258, 1259f precursor B/T-cell lymphomas, 1261t, 1262-1263 predictive factors, 1787 in pregnancy, 1267 prognostic systems, 1262 Sjögren’s syndrome and, 1787 small bowel, 1320, 1321f staging, 1262, 1262t staging evaluation of, 2324 treatment of, 1262b types, 1261t, 1262-1266 peripheral, 2324, 2324b plasmablastic, 1265, 2324 in post-transplantation lymphoproliferative disorders, 1267 primary effusion, 2324 small lymphocytic, 1254-1255, 1263 T-cell, angioimmunoblastic, 1074 testicular, 1267 thyroid, 1513 Lymphoma cutis, 2638t-2644t Lymphomatoid papulosis, 1266 Lymphoplasmacytic lymphoma, 1261t, 1265 Lymphoproliferative disorders cutaneous, 1266 indolent, 1254t large granular, 1255 post-transplantation, 2649f EBV-associated, 1038, 2234 lymphoma in, 1267 Lymphoproliferative syndromes autoimmune, 1074, 1686t X-linked, 2233 Lymphotoxin-α. See Tumor necrosis factor-α Lynch syndrome, 187t, 1227t, 1362 Lysergic acid diethylamide (LSD), 161 Lysinuria, isolated, 1386t Lysinuric protein intolerance, 1386t Lysosomal acid lipase deficiency, 1385t Lysosomal storage diseases, 1387, 1399-1403 classification of, 1400t clinical manifestations of, 1399, 1401t pathophysiologic mechanisms of, 1385t

M

M (monoclonal) protein, 1273-1274, 1274f, 1908, 1909f, 2531 M1 protein, 2192t M2 protein, 2192t Maalox, 900t MabThera (rituximab), 171-172 Macedonia, 1529-1530 Machupo virus, 91t, 2247, 2249t Macitentan, 402 Macrocytes oval, 1053f polychromatic, 1052f Macrocytic anemia, 1064 blood smear features of, 1053f, 1056-1057, 1056t differential diagnosis of, 1053f, 1056-1057, 1056t evaluation of, 1109, 1109f Macrocytosis, 1109 evaluation of, 1109, 1109f features that suggest cause of, 1056t peripheral blood findings, 1136, 1136f Macroglobulinemia, primary (Waldenström’s), 1265, 1276t, 1282 Macroglossia, 1284, 1285f Macrolides, 1894-1896 mechanism of action, 1889t

Macrolides (Continued) for Mycoplasma pneumonia, 2006 recommended doses and schedules, 1891t-1892t resistance to, 1890t for S. pneumoniae infection, 1905t toxicities, 1895t Macro-ovalocytes, 1053f Macrophage activation syndrome, 1147t-1148t, 1150 Macrophage antigen-1 (Mac-1, CD11b/ CD18), 1145 Macrophage-related abnormalities, 1147t-1148t, 1150 Macrophages, 216, 2634 functions of, 1142-1143, 1143t immune roles of, 1143t role in pathology, 219, 220f Macropolycytes, 1058 Macroprolactin, 1487 Macrotrauma, 1822 Macrovesicular steatosis, 1019, 1019t Macular degeneration, 2574 age-related, 1740t, 2567-2568 Macular exanthems, 2671-2675, 2671t Macular (red or blue) lesions, 2581t, 2582-2583 Macules, 2597, 2636t, 2647t-2651t Maculopapular exanthems, 1858-1859 drug rashes, 2685t in immunocompromised patients, 1858-1859 regional involvement, 2702t Maculopapular (red or blue) lesions, 2581t Maddrey’s discriminant function, 1021 Madura foot, 2099, 2100f Madurella, 2099 Maduromycetoma, 2099 Maduromycosis, 2099 Maffucci’s syndrome, 1672 Magnesium, 774 for acute STEMI, 451 fractional excretion of (FEMg), 775 for Gitelman’s syndrome, 824 hypermagnesemia, 776 for malabsorption, 931t metabolism of, 774-775 serum levels, 775 Magnesium ammonium phosphate (struvite) crystals, 731-732, 732f Magnesium ammonium phosphate (struvite) stones, 812 Magnesium carbonate, 778t Magnesium chloride hexahydrate, 931t Magnesium deficiency, 775-776, 2507t causes of, 775t clinical manifestations of, 775 diagnosis of, 775 pathobiology of, 775 prognosis for, 776 treatment of, 775b Magnesium depletion, 330 Magnesium diarrhea, 933 Magnesium gluconate, 931t Magnesium hydroxide, 778t Magnesium salts, 778t Magnesium sulfate for advanced cardiac life support, 354f for arrhythmias, 355 for magnesium deficiency, 775, 2507t for torsades de pointes, 374 Magnesium toxicity, 776 Magnesium wasting, 775 Magnetic resonance angiography, 735-736, 736f in atherosclerotic PAD, 500, 501f direct, 1728 in renal artery stenosis, 807-808, 808f strengths and weaknesses, 2345t Magnetic resonance cholangiography, T1-weighted, 871 Magnetic resonance cholangiopancreatography, 871 in acute pancreatitis, 962 with bile duct stones, 1042, 1043f in biliary disease, 877 in primary sclerosing cholangitis, 1045, 1046f Magnetic resonance enterography, 871, 871f

I63

Magnetic resonance imaging abdominal examination, 870-871, 870f in acute MI, 445 in acute pancreatitis, 962 brain imaging, 2438, 2440f cardiac in heart failure, 305 preoperative, 2617t stress, 425 cardiovascular, 288-292, 290f-292f contrast-enhanced, 870 functional, 1726-1728, 1729f in gastroenterology, 870-871, 870f, 872f of lungs, mediastinum, and chest wall, 532 in osteoarthritis, 1748 of pituitary, 1480, 1480f-1481f in pulmonary embolism, 623 in rheumatic disease, 1726-1728, 1727f-1729f strengths and weaknesses, 2345-2346, 2345t Magnetic resonance spectroscopy, proton, 2345t Magnetic resonance venography, 513 Magnetoencephalography, 2404 Magnocellular neurons, 1494 Major depressive disorder, 2346-2348, 2347t clinical manifestations of, 2347 definition of, 2346 diagnosis of, 2347 diagnostic criteria for, 2348t epidemiology of, 2346-2347 pathobiology of, 2347 prognosis for, 2348 symptoms and signs of episodes, 2347, 2348t treatment of, 2347b-2348b treatment phases, 2347 Major histocompatibility antigens, 236-237, 237.e1f Major histocompatibility complex (MHC) molecules, 221-222, 221f, 237.e1f Major histocompatibility complex class I−related chain A (MICA) antigens, 237-238 Mal de débarquement syndrome, 2597 Malabsorption approach to, 918-935 bile acid, 933 bile salt, 928 causes of, 923t clinical manifestations of, 922-923, 923t, 928 complications of, 928-929 conditions that cause, 929, 932-933 diagnosis of, 926, 927f glucose-galactose, 933 laboratory tests, 925-926 minerals for, 931 mucosal, 929 radiographic findings, 923 tests for, 925-926, 925t vitamins for, 931, 931t Malabsorptive syndromes, 926 Malar rash, 1771, 1771f Malaria, 2107-2113 antigen detection, 2110 blood smears in, 2110 chemoprophylaxis of, 1883-1884, 2112t clinical features of, 2109-2110 complications of, 2110 definition of, 2107-2108 diagnosis of, 2110 diagnostic tests for, 2110 distinguishing characteristics of, 2252t-2253t endemic regions, 2108, 2112 epidemiology of, 2108 genetics of, 2109 global patterns, 20-22 and HIV infection, 2108 host immunity, 2109 masking of folate deficiency with, 1113 P. falciparum severe, 2110 treatment of, 2111t parasite life cycle, 2108-2109 pathobiology of, 2108-2109 prevention of, 2105-2106, 2112 Volume I  pp 1-1382  •  Volume II  pp 1383-2722

I64

Index

Malaria (Continued) measures for travelers to malariaendemic regions, 2112 pretravel, 1882t prognosis for, 2112 serum folate levels in, 1113, 1108.e1t severe, 2110, 2112 transmission of, 2108 in travelers, 2108, 2112 treatment of, 2105-2106, 2110b-2112b, 2111t uncomplicated, 2109-2110 vectors, 2172t Malaria parasites life cycle of, 2108f pathogenic features of, 2109 Malarone (atovaquone and proguanil) for babesiosis, 2144 for malaria, 1883, 2105, 2111t-2112t, 2112 Malathion, 2173, 2175, 2698 Male circumcision, 1880 Male condom, 1604 Male genitalia: examination of, 28 Male infertility, 1575 definition of, 1575 diagnosis of, 1575, 1576f-1577f epidemiology of, 1575 laboratory tests, 1578t pathobiology of, 1575 treatment of, 1575-1576, 1575b-1576b, 1576f-1577f Male osteoporosis, 1644 Male reproductive axis, 1568-1569, 1568f Male senescence, 1571-1572 Male sterilization, 1604 Malformations of brain stem, 2513 of cerebellum, 2513 of cerebral cortex, 2512 Chiari, 2513 of spinal cord, 2513-2514 Malignancy. See also Cancer; specific sites AIDS-defining, 2322-2323, 2323f, 2323t alcohol-related, 152 arthritis in, 1823t in cardiac transplantation, 522 chronic febrile illness, 1852-1853, 1852t with cyclophosphamide, 168 with cyclosporine, 167 after DVT or PE or thrombosis, 1189, 1189f evaluation for, 1189, 1189f gallbladder, 1042 gastric, 1319 after hematopoietic stem cell transplantation, 1203 hepatic, 1018t in HIV infection, 2304-2305 hypercalcemia of, 1219-1220, 1657-1658 hypercoagulable states in, 1188-1189 in immunocompromised patients, 1855, 1855t myeloid, 1121-1122, 1122t non–AIDS-defining, 2322, 2323t, 2326-2327 nonhematologic, 1130 pleural fluid characteristics, 634t rare, 1034-1035 in renal transplantation, 845-846 Malignant ascites, 949-950, 950f, 1219 Malignant bone tumors, 1370-1371 Malignant effusions, 634, 1219 Malignant external otitis, 1967 Malignant hyperthermia, 2617-2618 clinical features of, 2542t toxic myopathies, 2546t treatment of, 726 Malignant Hyperthermia Association of the United States, 2617-2618 Malignant lung disease imaging of, 537-538 surgery for, 644 Malignant melanoma of conjunctiva, 2572 familial, 1227t in HIV infection, 2320 Malignant mesothelioma, 1313 Malignant monoclonal gammopathy, 1273, 1273t Malignant paraganglioma, 1527 Malignant pericarditis, 491 Malignant pheochromocytoma, 1527 Malignant ulcer disease, 910-911

Mallory-Weiss tears, 879, 879f, 907 Malnutrition, 887 acute in children, 1436.e1f clinical manifestations of, 1434 definition of, 1434 diagnostic and treatment algorithm for, 1436.e1f global rates, 1434 inpatient management protocol for, 1436f treatment of, 1436 edematous, 1435-1436, 1435f in hospitalized patients, 1440 hypoalbuminemic, 1433 protein-energy, 1434-1437 causes of, 1441t definition of, 1431 nutritional therapy for, 1433 secondary, 1436 in skin disease, 2646t Malnutrition Universal Screening Tool, 1433 Malrotation, intestinal, 944 Maltese crosses, 1402 MammaPrint (Agendia Inc.), 203, 1233, 1354 Mammography, screening, 1359, 1359t Manganese, 1450t-1452t Manganese toxicity, 93t, 98 Manganism, 98 Mania diagnostic criteria for, 2350t symptoms and signs of, 2350, 2350t treatment of, 2350 Manic disorder, 2347t Mannitol for drowning, 596 for elevated CSF pressure, 2488 for elevated ICP in acute liver failure, 1033 for intracranial hypertension, 2452t for pulmonary infections, 565 for rhabdomyolysis-induced acute kidney injury, 726t for subarachnoid hemorrhage, 2447 Mannosidosis, 1387 Mansonella ozzardi, 2165t, 2166, 2167t, 2171 Mansonella perstans, 2165t, 2166, 2167t, 2170-2171 Mansonella streptocerca, 2165t, 2166, 2167t, 2171 Mantle cell lymphoma, 1261t, 1264 Manual dystonia, 2463-2464 Maple syrup urine disease, 1385-1386, 1385t-1386t, 1388t Maprotiline, 2360 Marasmic kwashiorkor, 1434 Marasmus (wasting), 1434, 1435f global rates, 1434 nutritional therapy for, 1433 Maraviroc, 2288t Marble bone disease, 1670 Marburg hemorrhagic fever, 2249t clinical manifestations of, 2250, 2251t management of, 2254-2255 pathobiology of, 2250, 2251t prevention of, 91, 2256 prognosis for, 2256 vaccine for, 2255-2256 Marburg virus, 91t, 2056, 2247, 2249t March fracture, 1754 Marchiafava-Bignami syndrome, 2511 Marfan syndrome, 186, 187t, 194, 1734-1735 aortic aneurysm in, 493-494 clinical manifestations of, 1735 definition of, 1734 diagnosis of, 1735 differential diagnosis of, 1735 epidemiology of, 1734 myopia in, 2558 pathobiology of, 1734-1735 pathogenesis of, 1734-1735 pathology of, 1735 treatment of, 1735b Marginal ulcers, 911-912 Marginal zone lymphoma, splenic, 1265 Margination, 1129, 1144 increased, 1136 laboratory evaluation of, 1145 Marijuana, 144t, 161 Marijuana use clinical manifestations of, 161 epidemiology of, 161

Marijuana use (Continued) pathobiology of, 161 treatment of, 161b Marine envenomation, 720-722, 721b Marine invertebrates, 722 Marine poisoning, 722 Marizomib, 1281 Marker chromosomes, 192 Maroteaux-Lamy syndrome, 1387, 1733, 1734t Marseilles fever, 2050 Masked facies, 2455 Massage therapy, 182t, 183 Masses hepatic, 979t, 980-982 mediastinal, 636-637 scrotal, 1365f, 1366 with unknown primary site, 1300-1301 Mast cell activation syndrome, 1700 Mast cell sarcoma, 1709 Mast cells, 217, 1688, 2634 Mastalgia, 1359 Mastectomy, prophylactic, 1358 Mastitis, 1359 Mastocytomas, 1707, 1709 Mastocytosis, 1706-1710 clinical manifestations of, 1706-1707 cutaneous classification of, 1709 diagnosis of, 1707-1708 manifestations of, 1707, 1707f definition of, 1706 diagnosis of, 1707-1709, 1708f diagnostic findings, 1708f epidemiology of, 1706 gastrointestinal symptoms of, 1707 genetics of, 1706 hematologic manifestations of, 1707 musculoskeletal symptoms of, 1707 pathobiology of, 1706 pathogenesis of, 1706 prognosis for, 1710 symptoms of, 1706-1707 systemic, 912, 1706-1707 aggressive, 1709 classification of, 1709f diagnosis of, 1708-1709 diagnostic criteria for, 1708-1709, 1708t indolent, 1709 treatment of, 1709b-1710b ancillary therapies, 1710 medical therapy, 1709-1710 WHO categories of disease, 1706, 1709 Matched sibling donor transplantation, 1119-1120 Matched unrelated donor transplantation, 1120 Material Safety Data Sheets, 79 Maternal conditions, 1564 Maternal drug exposure, 1564 Maternal monitoring, 1621 Matrilysins, 233t Matriptase-2, 1070 Matrix damage, 233-234 Matrix metalloproteinases, 233t, 1732-1733 Maturity-onset diabetes of the young, 187t Maximal expiratory pressure (Pemax or MEP), 655 Maximal inspiratory pressure (Pimax or MIP), 655 Maximal mid-expiratory flow rate (MMEFR), 550, 550t Maximal respiratory pressures, 542 Maximum tolerated dose (MTD), 1232 Mayaro fever, 2261, 2261b Mayaro virus, 2261 Mayer-Rokitansky-Küster-Hauser syndrome, 1565-1567 May-Hegglin anomaly, 1057-1058, 1167 Maze procedure, 380 Mazzotti reaction, 2104 Mazzotti test, 2169 McArdle’s disease, 2340, 2545 McBurney’s sign, 858t McCune-Albright syndrome, 1516, 1556, 1556t, 1652t, 1671, 1671f clinical manifestations of, 1671-1672 pathobiology of, 1671 pituitary tumors in, 1483-1484 treatment of, 1672 McDonald diagnostic criteria, 2474t

Mean(s), 32 sample means, 35 standard error of the mean (SEM), 35 Mean arterial pressure (MAP), 687t Mean cell hemoglobin (MCH), 1060t Mean cell hemoglobin concentration (MCHC) in anemia, 1062 normal values, 1060t Mean cell volume (MCV) in anemia, 1062 normal values, 1060t Meaningful Use standards, 46 Measles, 2202-2204 clinical manifestations of, 2202-2203 complications of, 2203 definition of, 2202 diagnosis of, 2203 differential diagnosis of, 2203, 2203t distinguishing characteristics of, 2252t-2253t epidemiology of, 2202 mild, 2203 modified, 2203 morbilliform eruptions of, 2202f, 2672 oral ulcers of, 2580-2581, 2580t pathobiology of, 2202 postexposure prophylaxis of, 2204 prevention of, 2203-2204 prognosis for, 2204 travel-related risk, 1881 treatment of, 2203b vaccination against, 74-75, 2203-2204 for adults, 66t agents for, 68t-73t recommendations for, 2204 Measles, mumps, rubella (MMR) vaccine, 76, 2203-2204, 2206, 2208 adverse reactions to, 75-76 indications for, 68f, 74-75 recommended adult schedule, 67f-68f, 67.e1t-67.e3t Measles, mumps, rubella, varicella (MMRV) vaccine, 2203-2204, 2206, 2229 Measles rash, 2202, 2202f Measles vaccine adverse reactions to, 75 contraindications to, 75 indications for, 74-75 Measures absolute, 33 of health. See Health measures of health care. See Health care, measures of of quality. See Quality measures relative, 33 Mebendazole, 2104 for intestinal nematodes, 2160t for roundworms, 2103 for trichuriasis, 2162 Mecamylamine, 147t Mechanical circulatory support, 317-318 Mechanical heart valves, 472t, 473f Mechanical ventilation, 664-672 for acute poisoning, 704-705 for acute respiratory distress syndrome, 670f for acute respiratory failure, 660 for ARDS, 663f, 668-669 assisted, 665 complications of, 666-668 discontinuation of, 670-672, 671f high-frequency ventilation, 665-666 initiation of, 668, 669t intermittent mandatory ventilation, 665 lung-protection strategies, 668-669 negative-pressure ventilators, 665 neurally adjusted ventilatory assist, 666 noninvasive, 629, 629t, 669 noninvasive positive-pressure ventilation, 666 for obstructive airway disease, 669-670 positive end-expiratory pressure, 665 positive-pressure ventilators, 665 pressure-controlled ventilation, 665 pressure-support ventilation, 665 proportional assist ventilation, 666 for shock, 680, 688-689 for thermal injury, 599 types of ventilators, 665-666 volume-controlled ventilation, 665 weaning strategies, 670 Mechlorethamine, 1272, 2670t

Index Meckel diverticulum, 943-944 clinical manifestations of, 944 diagnosis of, 944 prognosis for, 944b treatment of, 944b Meclizine, 865t, 2600 Medial epicondylitis, 1752 Medial striate artery, 2427 Median, 32 Median arcuate ligament syndrome, 958 Median rhomboid glossitis, 2582 Mediastinal large B-cell lymphoma, 1261t, 1265 Mediastinal masses, 636-637 anterior, 636-637, 637f causes of, 637t clinical manifestations of, 636, 636f diagnosis of, 636-637 pathobiology of, 636 treatment of, 637b Mediastinitis, 637 Mediastinum, 636-637 anterior compartment lesions, 636-637, 637f compartments of, 538, 538t, 636.e1f imaging of, 531-539, 538.e1f pneumomediastinum, 637 Medicaid, 16, 19 Medical assessment, geriatric, 104-105, 106t Medical care settings, 2287 Medical consultants, 2610, 2611f, 2625 Medical consultations, 2608-2611, 2611f approach to, 2608 curbside, 2610 effects on patient outcome, 2611 electronic, 2610 guidelines for, 2610t informal, 2610 mandatory, 2610 principles of, 2608-2611 in psychiatry, 2625-2629 referral for intimate partner violence, 1631 referral to behavioral change specialists, 54 referral to ENT specialists, 2589t setting-specific issues, 2608-2609 special situations, 2610 strategies for, 2609-2610 subspecialty consultations, 2609, 2609t surgical consultations postoperative, 2608 preoperative, 2608 when to refer to specialists, 2657 Medical devices for heart failure, 316-317 implanted, 285 intravascular, 627 related infections, 1865-1866 guidelines and recommendations for, 1865.e1t staphylococcal, 1900 Medical evaluation preoperative, 2611-2617 in psychiatric settings, 2625-2626 Medical Expenditure Panel Survey (MEPS), 43.e1t Medical history, 24-25, 25t adolescent care health visit, 63 alcohol history, 153-154 cardiac risk assessment, 2612 clinical history, 1430-1431 complete, 251-252 exposure history, 79 general risk assessment, 2611 gestational history, 1566 history of the present illness, 24 most important parts, 1168 neurologic examination, 2338 neurologic history, 2338-2339 past medical history, 24 periodic health examination, 54 review of systems, 25, 25t Medical imaging, 85 The Medical Letter on Drugs and Therapeutics, “Drugs for Parasitic Infections”, 2103 Medical profession, 2-4 Medical professionalism, 3 Medical Research Council, 2540t MedicAlert tags, 1490 Medical-medical consultations, 2608

Medicare, 16 “no pay for adverse events” program, 45 Value-based Purchasing program, 46 Medicare Prescription Drug, Improvement, and Modernization Act, 1868.e1 Medication history, 2338 Medication review, 105 Medication use counseling, 315 Medication-related diarrhea, 920-921 Medications. See Drug(s) Medicine approach to, 2, 3.e1t-3.e3t internal, 2 as learned and humane profession, 2-4 Meditation, 182, 182t Mediterranean diet, 1465 Mediterranean fever, familial, 1739-1740, 1740t Mediterranean spotted fever, 2048t, 2050 Medium vessel vasculitis, 956-957 Medium-chain acyl-CoA dehydrogenase (MCAD) deficiency, 2545 Medium-vessel vasculitis clinical manifestations of, 1796-1797, 1797f names for, 1794t pathophysiology of, 1795-1796 MEDLINE, 24.e1t Medroxyprogesterone acetate for abnormal uterine bleeding, 1588 for amenorrhea, 1589 for hot flushes, 1625-1626 for hypopituitarism, 1483t for menopausal hot flushes, 1628t-1629t for polycystic ovary syndrome, 1594 for prevention of osteoporosis, 1593 for vasomotor symptoms, 1626t Medtronic free style stentless valve, 473f Medtronic-Hall tilting disc valve, 472t, 473f Medullary collecting duct, 764 Medullary cystic kidney disease, 817t, 821 Medullary cystic kidney disease gene (MCKD1), 821 Medullary cystic kidney disease gene (MCKD2), 821 Medullary sponge kidney, 817t, 821 Medullary stroke syndromes, 2435, 2435.e2f Medullary thyroid carcinoma, 1513, 1662-1664 clinical manifestations of, 1662-1663 definition of, 1662 diagnosis of, 1663 epidemiology of, 1662 familial, 1662 clinical manifestations of, 1662-1663, 1662t prevention of, 1663 pathobiology of, 1662 prevention of, 1663 prognosis for, 1663 treatment of, 1663b Medulloblastoma, 1294 MedWatch, 132 Mees’ lines, 95, 2708 Mefloquine, 2105-2106, 2111, 2111t-2112t Megacalycosis, 826 Megacolon, Chagas, 2117, 2119 Megaloblastic anemia, 1104-1114, 1110f blood smear features of, 1056-1057, 1056t, 1057f bone marrow findings, 1111-1112, 1112f causes not responding to cobalamin or folate therapy, 1112t clinical manifestations of, 1108 definition of, 1104-1105 diagnosis of, 1109-1112 diagnostic approach to, 1109 epidemiology of, 1105-1106 history in, 1109-1110 laboratory tests, 1110-1112 masked, 1110, 1110t not caused by cobalamin or folate deficiency, 1105t pathobiology of, 1106-1108 peripheral blood findings, 1136, 1136f physical findings, 1109-1110 prognosis for, 1113 treatment of, 1112b-1113b

Megaloblastosis laboratory testing for, 1110 masked, 1110, 1110t peripheral blood smear features of, 1110, 1110f Megaureter, 826-827, 826f Megestrol, 1628t-1629t Meglumine antimoniate, 2122 Meigs’ syndrome, 635 Meissner’s plexus, 896-897, 2517 Melanocytes, 2633 Melanocytic lesions, 2689 Melanoma, 1373-1377 advanced (stage IV) course of, 1376 treatment of, 1376 amelanotic, 2649f Breslow’s thickness of, 1375 Clark’s level of, 1375 clinical features of, 1374-1375, 1375t color in, 2638t-2644t, 2643f diagnosis of, 1375 epidemiology of, 1373 evaluation of, 1375 follow-up, 1376 genetics of, 1374 iris, 2572, 2572f lentigo maligna, 1374, 1375f malignant familial, 1227t in HIV infection, 2320 ocular, 2572 nail, 2710, 2710f nodular, 1374, 1375f pathobiology of, 1373-1374 prevention, 1376-1377 prognostic factors, 1375 regional lymph nodes, 1376 risk factors for, 1373-1374 staging system, 1375 superficial spreading, 1374, 1375f surveillance, 1376 TNM staging of, 1375.e1t treatment of, 1376, 1376b adjuvant therapy, 1376 targeted therapy, 202, 202t Melanoma differentiation−associated protein 5 (MDA5), 217 Melanoma in situ, 1376 Melanonychia, 2709-2710, 2710f Melarsoprol, 2115, 2115t Melasma, 2700, 2700f Melatonin for insomnia, 2421t for prevention of cluster headache, 2361 for REM sleep behavior disorder, 2396 Melioidosis, 1967-1968 Melorheostosis, 1671, 1671b Melphalan (Alkeran) for autologous stem cell transplantation, 1279 for cancer, 1211t-1216t for multiple myeloma, 1280, 1280t Memantine, 2394, 2457t-2459t Membrane attack complex, 244-245 activation of, 243f, 245t regulation of, 245, 245f Membrane cofactor protein (MCP) (CD46), 243t Membrane transport disorders, 1386, 1386t Membranoproliferative glomerulonephritis, 787-788, 788f clinical manifestations of, 788 diagnosis of, 788 prognosis for, 788b treatment of, 788b Membranous glomerulopathy, 787, 787f Membranous nephropathy, 786-787, 787f clinical manifestations of, 787 diagnosis of, 787 prognosis for, 787 treatment of, 787b Memory, 225, 2382, 2383t declarative, 2383, 2383t function of, 2382-2384 immediate, 2383t procedural, 2383t working, 2383t MenAfriVac, 1938-1939 Menarche, 61

I65

Mendel, Gregor, 189 Mendelian disorder, 191 Mendelian inheritance, 194-195 Mendelian Inheritance in Man, 186 Mendel’s first law, 189 Mendel’s second law, 189 Meniere’s disease, 2595, 2598 tinnitus in, 2596 treatment of, 2595-2596, 2600, 2601t Meningioma, 1288, 1289f, 1291, 1291f, 1296 diagnosis of, 1291 epidemiology of, 1291 skull base, 1291, 1291f treatment of, 1291b visual field abnormalities with, 2575 Meningitis, 2241-2242 Acinetobacter, 1969 AIDS-associated, 2297t-2301t anaerobic, 1932 anthrax, 1922 aseptic, 2207, 2241t definition of, 2489 noninfectious causes of, 2492-2493, 2492t nonviral infectious causes of, 2491t, 2492-2493 bacterial, 2480-2495. See also specific bacteria antibiotic regimens for, 689t CSF formula, 2342t Candida, 2081 chemical, 2484 chronic, 2494, 2494t chronic (intermittent), 2494 chronic (persistent), 2493-2494 infectious causes of, 2492-2493, 2492t with neutrophil predominance, 2494, 2494t noninfectious causes of, 2494t chronic (recurrent), 2485, 2492-2493, 2492t chronic persistent neutrophilic, 2494, 2494t clinical manifestations of, 2241-2242 cryptococcal, 2076-2077 AIDS-associated, 2297t-2301t in immune reconstitution inflammatory syndrome, 2334 treatment of, 2067, 2077, 2297t-2301t definition of, 2480 diagnosis of, 2241-2242 enteroviral, 2241 eosinophilic, 2492-2495 causes of, 2493t fungal, 2342t granulomatous, 2599 H. influenzae, 1947-1948, 2486 H. influenzae type b, 1947 in immunocompromised patients, 1861 intermittent (chronic), 2494 Listeria, 1920, 2480, 2487 lymphocytic, chronic (persistent) infectious causes of, 2492-2493, 2492t noninfectious causes of, 2493-2494, 2494t lymphocytic choriomeningitis, 2058t clinical manifestations of, 2491 epidemiology of, 2490 meningococcal, 1936 antibiotic treatment of, 1937b-1938b, 1937t diagnosis of, 1937 travel-related risk, 1881 treatment of, 1937b-1938b neutrophilic, chronic persistent, 2494, 2494t P. aeruginosa, 1967 persistent (chronic), 2493-2494 infectious causes of, 2492-2493, 2492t with neutrophil predominance, 2494, 2494t noninfectious causes of, 2494t prognosis for, 2242b pyogenic, 2480 recurrent (chronic), 2485, 2492-2493, 2492t relapsing, 2502t S. maltophilia, 1971 syphilitic, 2016 treatment of, 2242b Volume I  pp 1-1382  •  Volume II  pp 1383-2722

I66

Index

Meningitis (Continued) tuberculous, 2034 CSF formula, 2342t definition of, 2480 viral, 2241, 2342t, 2489-2492 zoonotic, 2487 Meningitis belt, 75, 1882, 2480 Meningitis Vaccine Program, 1938-1939 Meningococcal conjugate vaccine, 2489 Meningococcal disease. See Neisseria meningitidis infection Meningococcal meningitis, 1936 antibiotic treatment of, 1937b-1938b, 1937t diagnosis of, 1937 travel-related risk, 1881 treatment of, 2486 Meningococcal vaccines, 68t-73t, 75, 1882, 1883t, 1938-1939, 2489 adverse reactions to, 75 indications for, 68f, 75 recommended adult schedule, 67f-68f, 67.e1t-67.e3t Meningococcemia, 1934, 1936 antibiotic treatment of, 1937b-1938b, 1937t cutaneous manifestations of, 2697, 2697f distinguishing characteristics of, 2252t-2253t Meningococcus (Neisseria meningitidis), 1934 Meningoencephalitis definition of, 2500 in immunocompromised patients, 1861 test for, 2502t Meningovascular syphilis, 2016 Meniscal abnormalities, 1746 Menkes disease, 1416, 2507t, 2510 Menopause, 1623-1629 biology of, 1623-1624 definition of, 1623 epidemiology of, 1623 stages of, 1623-1624, 1624f symptoms of, 1624-1629 vaginal symptoms, 1627, 1627b-1629b vasomotor symptoms of, 1624-1625 Men’s health breast cancer, 1358 men who have sex with men (MSM), 1940, 2272, 2276, 2276f normal body temperature, 1849 risks for atherosclerotic cardiovascular disease, 260f-261f sexual function in, 1570 undervirilized males, 1566-1567 urogenital gonorrhea, 1941-1942 Menstrual cycle changes in target organs during, 1585-1586 follicular (preovulatory) phase, 1584 luteal (postovulatory) phase, 1584-1585 luteal phase dysfunction, 1594 normal, 1584-1585, 1585f ovulatory phase, 1584 selection, 1585-1586 Mental function higher, 2382-2388 normal, 2382 Mental status change, 117-121, 118f Mental status examination, 115-116 Menthol, 2636t Meperidine, 137, 139t-140t, 695 Mepolizumab, 553 α-Mercaptopropionylglycine, 822 6-Mercaptopurine, 165-166 adverse reactions to, 165-166 indications for, 165 for inflammatory bowel disease, 939t, 940 mechanism of action, 165 Merck, 1904, 2246 Mercury poisoning, 92-94 chelators for, 93t clinical manifestations of, 94 diagnosis of, 94 diagnostic testing for, 93t epidemiology of, 92-93 pathobiology of, 93-94 prognosis for, 94 toxic levels, 703t treatment of, 94b, 706t-710t Meridians, 182 Merkel cell carcinoma, 1228, 1301, 2326, 2689

Merkel cell polyomavirus, 2689 Merkel cell tumors, 2689, 2689f Merkel cells, 2633 Meropenem for Acinetobacter infections, 1969-1970 for bacteremia, 1966 for melioidosis, 1967 for meningitis, 2486, 2488t for meningococcal disease, 1937t for pneumonia, 616t, 618, 619t, 1905t, 1966 for pyelonephritis, 1875t for pyogenic liver abscess, 1012 recommended doses and schedules, 1891t-1892t for septic shock, 689t for Whipple’s disease, 932 MERS (Middle East respiratory syndrome), 2056-2057 MERS (Middle East respiratory syndrome) coronavirus infection, 2200-2201 clinical manifestations of, 2200, 2200t prevention of, 2201 treatment of, 2201 Mesalamine (Asacol, Apriso, Canasa, Lialda, Pentasa, Rowasa) for Crohn’s disease, 940 for diarrhea, 934 for inflammatory bowel disease, 939t Mesenteric cysts, 950 Mesenteric diseases, 950-951, 950t Mesenteric fibromatosis, 950 Mesenteric ischemia, 951, 953f acute, 952f chronic, 911, 911f, 955, 955b management of, 952f-953f noncolonic, 954 nonocclusive, 954 secondary, 951, 951t Mesenteric panniculitis, 950 Mesenteric venous thrombosis, 954-955 clinical manifestations of, 954-955 diagnosis of, 955 epidemiology of, 954 pathobiology of, 954 risk factors for, 955t treatment of, 955b Mesial temporal lobe epilepsy, 2406 Mesial temporal sclerosis, 2406 Mesna (Mesnex), 1211t-1216t Mesocardia, 416 Mesothelioma, 593, 635, 635f clinical manifestations of, 635 CT findings, 635, 635f diagnosis of, 635 epidemiology of, 635 malignant, 1313 pathobiology of, 635 prognosis for, 635b treatment of, 635b Meta-analysis, 36 Metabolic acidosis, 766-771 anion gap, 766-769 of chronic kidney disease, 840 clinical manifestations of, 766 compensatory changes in, 765-766, 765t epidemiology of, 766 pathobiology of, 766 treatment of, 771, 840 Metabolic alkalosis, 771-773 causes of, 762, 762t chloride-responsive, 773 chloride-unresponsive, 773 clinical manifestations of, 773 compensatory changes in, 765t diagnosis of, 773 epidemiology of, 771-772 of nonrenal origin associated with normal or expanded volume, 772-773 with extracellular volume depletion, 772 pathobiology of, 771-772 of renal origin associated with volume depletion, 772 with volume expansion and hypertension, 772 treatment of, 773b Metabolic bone disease approach to, 1636-1637 diagnosis of, 1636 in end-stage renal disease, 842 history in, 1636

Metabolic bone disease (Continued) laboratory work-up, 1636, 1636t physical findings, 1636 radiologic studies, 1636 Metabolic disease blistering disorders, 2678 cardiomyopathy with, 330 chronic anovulation related to, 1594 in HIV infection, 2295-2303 hypoglycemia due to, 1553 Metabolic equivalents (METs), 58, 1461 Metabolic monitoring, 1536 Metabolic myelopathy, 2381 Metabolic myopathy, 2542t, 2545-2546, 2545b Metabolic neuropathy, 2535, 2536b Metabolic profiling, 203 Metabolic programming, 1533-1534 Metabolic syndrome, 383, 2626-2627, 2626t Metabolism basal metabolic rate, 1460-1461 cardiovascular, 264-265 definition of, 1384 inborn errors of, 1384-1389, 1453 post-resuscitation issues, 716 protein, 1384-1386 resting metabolic rate (RMR), 1460-1461 Metabolite clearance tests, 990 Metabolomics, 1838 Metachromatic leukodystrophy, 1402-1403, 2479 Metagenome, 1838-1839, 1839f-1840f Metagonimus yokogawai, 2157 Metal fume fever occupational causes, 594t suggestive features, 589t Metals diagnostic testing for, 93t exposures to, 81, 81t heavy metal poisoning, 93t trace metal poisoning, chronic, 92-98 Metamorphopsia, 2558-2559 Metamyelocytes, 1143-1144 Metanephrines, 1524-1525, 1525t Metastatic disease, 1230 axillary lymph node, 1379-1380 bladder cancer, 1351 bone disease prevention of, 1371 treatment of, 1371b bone tumors, 1371 brain, 1294 breast cancer, 1356-1358 central nervous system tumors, 1294-1295 colorectal cancer, 1331 diagnosis of, 1378 leptomeningeal, 1295, 1295f, 2599 in lungs, 644 ocular, 2572 pancreatic cancer, 1334 renal cell carcinoma, 1347-1348 single lesions, 1380 skeletal, 1380 spinal, 1296 spinal cord compression due to, 2381-2382 clinical manifestations of, 2381 diagnosis of, 2381-2382 differential diagnosis of, 2382 prognosis for, 2382 treatment of, 2382b in Zollinger-Ellison syndrome, 1337 Metformin contraindications to, 314 for polycystic ovary syndrome, 1594 for type 2 diabetes, 1535 Methadone formulations, dosages, and pharmacologic information, 139t-140t for opioid dependence, 159 toxicity, 699t-702t Methamphetamine, 160 Methamphetamine abuse, 160 Methanol, 769 Methanol ingestion, 699t-703t, 706t-710t, 769 Methemoglobin, 703t Methemoglobinemia, 1060-1061 Methemoglobin-producing agents, 706t-710t Methicillin-resistant Staphylococcus aureus community vs hospital clones, 1897.e1t virulence factors, 1897

Methicillin-resistant Staphylococcus aureus infection epidemiology of, 1897 health care–associated infections, 1863-1864, 1865.e1t prevention of, 1901 skin and soft tissue infections, 1897 treatment of, 1901 Methimazole, 1508 Methionine, oral, 1407 Methotrexate (Folex, Mexate), 168-169 for Behçet’s syndrome, 1799 for brain tumors, 1291 for bullous pemphigoid, 2675 for Burkitt-like ALL, 1243-1244 for cancer, 1211t-1216t caveats, 1761t for central nervous system leukemia prophylaxis, 1243 for central nervous system lymphoma, 1294 dose-response effects, 1008t for giant cell arteritis, 1804 for glomerulonephritis, 791 for granulomatosis with polyangiitis, 1798, 2534 for inflammatory bowel disease, 939t for inflammatory myopathies, 1792 for leukemia, 1243t for myasthenia gravis, 2551 for mycosis fungoides, 2670t for osteosarcoma, 1370 overdose, 706t-710t perioperative, 1833 for pityriasis lichenoides, 2669 for pityriasis rubra pilaris, 2667 for polymyalgia rheumatica, 1804 for psoriasis, 2666t for rheumatoid arthritis, 1761 for sarcoidosis, 607t for spondyloarthritis, 1764 for systemic lupus erythematosus, 1776 for Takayasu’s arteritis, 497 Methylacyl-CoA racemase deficiency, 1387t Methylcellulose, 934-935 Methyldopa for hypertension, 388t, 394, 394t for menopausal hot flushes, 1628t-1629t Methylene blue, 706t-710t Methylene dioxymethamphetamine (MDMA) (ecstasy), 161, 2519-2520 5,10-Methylenetetrahydrofolate reductase deficiency, 1404-1405 adult-onset clinical presentation of, 1406 clinical features of, 1406, 1406t pathobiology of, 1404f, 1405 prognosis for, 1407 treatment of, 1407 Methylmalonic acid, 1111, 1111t Methylmalonic aciduria, 1385t, 1387t Methylnaltrexone, 889 Methylphenidate for myotonic dystrophy, 2543 for narcolepsy, 2418, 2419t for Tourette’s syndrome, 2466 Methylprednisolone, 164t for acute interstitial pneumonia, 573 for acute rejection, 646, 1036 for AIDS-associated opportunistic infections, 2297t-2301t for asthma, 554 for chronic sinusitis, 2588 for colitis, 941-942 for complex regional pain syndrome, 2522 for giant cell arteritis, 1804 for glomerulonephritis, 791 for Goodpasture’s syndrome, 574 for Guillain-Barré syndrome, 2530 for HTLV-associated myelopathy/tropical spastic paraparesis, 2238-2239 for idiopathic pulmonary fibrosis, 581 for IgA vasculitis, 1798 for inflammatory bowel disease, 939t, 940 for inflammatory myelopathy, 2381 for membranous nephropathy, 787 for multiple myeloma, 1280 for multiple sclerosis, 2477 for nasal polyps, 2589 for pemphigus, 2677 for Pneumocystis pneumonia, 2096t in renal transplantation, 844t for sensorineural hearing loss, 2595-2596

Index Methylprednisolone (Continued) for status asthmaticus, 554 for Stevens-Johnson syndrome, 2686 for systemic lupus erythematosus, 1775 for systemic vasculitis, 2534 for transverse myelitis, 2478 for urticaria, 2684t for vertigo, 2601t for vestibular neuritis, 2600 for warm autoimmune hemolytic anemia, 1077, 1078f Methyltestosterone, 1575 Metoclopramide for bacterial overgrowth, 928 for gastroparesis and pseudo-obstruction, 887-888 for migraine headache, 2358-2359 for nausea and vomiting, 865t recommendations for, 887 Metolazone, 388t, 748t Metoprolol, 430t for acute coronary syndrome, 438t-439t for acute MI, 450, 454t, 455 for aortic dissection, 496 for arrhythmias, 355 for atrial fibrillation, 452 for cardiogenic shock, 685 for heart failure, 311t for hypertension, 388t for premature ventricular contractions, 372-373 for torsades de pointes, 374 for ventricular tachycardia, 373-374 Metronidazole (Flagyl), 2105 for amebic colitis, 2141t for amebic liver abscesses, 1013, 2141t for anaerobic infection, 1933-1934, 1934t for bacterial overgrowth, 928 for C. difficile infection, 1925 for deep neck abscesses, 2603 for diarrhea, 922, 926 for diverticulitis, 947 for enteric protozoal infections, 2147t for gallstones, 1041 for gastroparesis and pseudo-obstruction, 888 for giardiasis, 2137 for H. pylori eradication, 915t for hepatic encephalopathy, 1030 for inflammatory bowel disease, 940 for lateral sinus thrombosis, 2499 for Lemierre’s syndrome, 2603 mechanism of action, 1889t for necrotizing fasciitis, 2696-2697 for pelvic inflammatory disease, 1945 for perianal fistulas, 941 for peritonitis, 949 for peritonsillar abscess, 2603 for pouchitis, 942 for pyogenic liver abscess, 1012 recommended doses and schedules, 1891t-1892t for rosacea, 2680 for septic shock, 689t for tetanus, 1930 toxicities, 1895t for trichomoniasis, 2146 for ulcerative colitis, 942 Metyrapone, 1517 Mevalonic aciduria, 1387t Mevinphos, 699t-702t Mexate (methotrexate), 1211t-1216t Mexican Americans gallstones in, 1038 hypertension in, 382, 393 obesity in, 1458 physical activity in, 58 Mexiletine for chloride channelopathies, 2544 for erythromelalgia, 508 for myotonic dystrophy, 2543 for sodium channelopathies, 2544 Meyer’s loop, 2573-2574 Micafungin, 2069 adverse effects of, 2069 for AIDS-associated opportunistic infections, 2297t-2301t for candidemia, 2082 drug interactions, 2069 for esophagitis, 2069

Micafungin (Continued) formulations, 2069 for hepatosplenic candidiasis, 1013 indications for, 2069 for neutropenia prophylaxis, 2069 pharmacology of, 2069 Micelles, 928 “Mickey Mouse” RBCs, 730 Miconazole, 2297t-2301t, 2582 Microagglutination test (MAT), 2030 Microalbuminuria, 729, 838 Microbial disease, 1837-1838 Microbial genomes, 202 Microbiome, human, 1838-1843 anaerobic bacteria, 1931, 1931f, 1931t analysis of, 1843 animal models, 1843 characterization of, 1839-1840 colonic microbiota, 1842 compositional differences in by anatomic site, 1839, 1839f-1840f during life cycle, 1840-1841, 1840f cutaneous, 1841-1842 disease links, 1841-1843 dynamics of, 1841, 1841f effects on brain and behavior, 1842-1843 equilibrium with host cells, 1841, 1841f extinctions, 1840 gastric, 1842 gut microbiota, 1842 inheritance of, 1840-1841, 1841f postnatal influences on, 1841 Microcytes, 1053f Microcytic anemia, 1063-1064, 1068 differential diagnosis of, 1056 hemoglobin components deficient in, 1063, 1063f hypochromic, 1068, 1068f, 1070t Microcytic red cells, 1053f Microdeletion syndromes, 194 Microduplication syndromes, 194 Microfilariae, 2166-2167, 2167t Micrographia, 2455 Micronutrients depletion of, 1441t dietary requirements, 1445 conditions that increase, 1445-1453 factors that determine, 1445t pathophysiologic factors, 1445t, 1453 pharmacologic factors, 1445t, 1453 physiologic factors, 1445, 1445t in pregnancy, 1445-1452 drug-mediated effects on, 1453, 1453t functions of, 1445 marginal deficiency states, 1453 in nutritional science, 1445 optimal intake, 1445, 1453 types of, 1445 Micropenis, 1567 Micro-RNAs (miRNAs), 189, 1228 Microsatellites, 191 Microscopic colitis, 934 Microscopic polyangiitis, 1798 Microscopy, confocal, 878-879, 878f Microspherocytes, 1053f Microsporidia, 2147t Microsurgery, 1378 Microtophi, 1813 Microtrauma, 1822 Microvasculature, renal, 810 Microvesicular steatosis, 1019, 1019t Micruroides euyxanthus, 719 Micrurus, 719 Micturition, 799-800, 799.e3f Midazolam for anesthesia, 2619 for muscle spasms, 1930 for seizures, 1033, 2408 for status epilepticus, 2408 Midbrain stroke syndromes, 2435, 2435.e1f Middle cerebral artery, 2427, 2429f Middle cerebral artery occlusion, 2435, 2435t Middle East respiratory syndrome (MERS), 2056 Middle East respiratory syndrome (MERS) coronavirus infection, 2200-2201 clinical manifestations of, 2200, 2200t prevention of, 2201 treatment of, 2201 Middle molecules, 836

Midline granuloma lethal, 1301 oral ulcers, 2580t Midnight plasma cortisol, 1515 Midnight salivary cortisol, 1487t Midodrine, 347, 1030, 2521 Mid-upper arm circumference (MUAC), 1435, 1436.e1f Migraine headache, 2358-2359 acute, 2359t clinical manifestations of, 2358 definition of, 2358 diagnosis of, 2358 differential diagnosis of, 2357t epidemiology of, 2358 pathobiology of, 2358 prevention of, 2359 prognosis for, 2359 treatment of, 2358b-2359b, 2359t, 2361 vertigo with, 2598-2599 Migratory erythema, necrolytic, 1337, 1337f Milan criteria, 1034, 1344 Mild cognitive impairment, 2389-2390 amnesic, 2390t clinical manifestations of, 2390 definition of, 2389 diagnosis of, 2390 epidemiology of, 2389 pathobiology of, 2390 prognosis for, 2390 treatment of, 2390b Miliary tuberculosis, 2034, 2035f Military history, 25t Military personnel, 589t Military tuberculosis, 1019 Milk-alkali syndrome, 1658 Milkman’s fractures, 1646 Millard-Gubler syndrome, 2435.e1f Millennium Development Goals, 19, 20t Miller-Fisher syndrome, 2529-2530 Millipedes, 2172t, 2175 Milrinone, 676t, 683 Miltefosine, 2107, 2122 Milwaukee shoulder, 1816, 1816b Minamata disease, 94 Mind and body practices, 182-184 Mind-body medicine, 182, 182t Mineral deficiencies, 2507t, 2510 Mineral dust exposures, 81t, 82 Mineral supplements, 57, 181. See also specific minerals for malabsorption, 931, 931t multimineral supplementation, 1455 Mineralocorticoid deficiency, 1521 clinical manifestations of, 1521 mixed mineralocorticoid and glucocorticoid deficiency, 1519-1520 Mineralocorticoid escape, 746 Mineralocorticoid excess, 746, 1517-1518 apparent, 823t, 1517 causes of, 1517, 1518t diagnosis of, 1517 renin-independent, 1517-1518 diagnosis of, 1517 differential diagnosis of, 1517-1518 treatment of, 1518b Mineralocorticoid receptor antagonists clinical benefits, 313 for heart failure, 313, 306.e1t-306.e2t for hypertension, 391 mechanism of action, 313 practical use, 312t, 313 Mineralocorticoid-induced hypertension due to primary aldosteronism, 386 clinical manifestations of, 386 diagnosis of, 386 pathobiology of, 386 treatment of, 386b Mendelian forms, 386, 386.e1f Mini-Cog Test, 115-116, 115t, 2382 Minimal change disease, 785-786, 785f clinical manifestations of, 785 diagnosis of, 785 microscopic findings, 785, 785f prognosis for, 786 treatment of, 785b-786b Minimally conscious state, 2410t, 2414, 2414t Mini-Nutritional Assessment, 1433 Minisatellites, 191

I67

Minocycline for Acinetobacter infections, 1969 for acne vulgaris, 2680 for blepharitis, 2561 for furuncles, 2696 for MRSA infection, 1901 for nocardiosis, 2064 for pneumothorax, 636 recommended doses and schedules, 1891t-1892t for rheumatoid arthritis, 1761 for rosacea, 2680 for S. maltophilia infections, 1971 Minor head injury, 2366t. See also Head injury Minor histocompatibility antigens, 237 Minority populations, 393 Minoxidil, 388t, 2661, 2704 Minute ventilation (Ve), 654 Miosis, toxicant-induced, 697, 697f Mirabegron, 113t MiraLAX (polyethylene glycol), 893t Mirtazapine, 2349t, 2421t, 2636t Misoprostol, 915 Missense mutations, 194 Missing data, 36 Mites, 2171t-2172t, 2173-2174 disease transmitted by, 2048t, 2051-2052 rashes induced by, 2174 Mitochondrial disorders, 1660-1661 Mitochondrial DNA depletion syndromes, 2546 Mitochondrial encephalomyopathy with lactic acidosis and strokelike episodes (MELAS) syndrome, 1652t, 1660-1661, 2546 Mitochondrial myopathies, 2542t, 2545-2546 clinical manifestations of, 2546 diagnosis of, 2546 prognosis for, 2546 toxic, 2546t treatment of, 2546 Mitochondrial oxidative phosphorylation disorders, 2542t, 2545-2546 Mitochondrial trifunctional protein deficiency syndrome (MTPDS), 1660-1661 Mitogen-activated protein (MAP) kinase, 219 Mitoxantrone (Novantrone), 1369, 2477 Mitral regurgitation, 453, 462t, 467f acute, severe, 468 asymptomatic in elderly patients, 469 with left ventricular dysfunction, 469 with normal left ventricular function, 469 chronic asymptomatic, 468 chronic symptomatic, 468 echocardiography in, 468, 468f evaluation of, 279 functional (secondary), 470 hemodynamic findings, 295.e1f with left ventricular dysfunction, 469 with normal left ventricular function, 469 pathophysiology of, 467, 467f primary (organic), 467-468 clinical manifestations of, 467-468 diagnosis of, 468 epidemiology of, 467 medical therapy for, 468 pathobiology of, 467 surgical therapy for, 468-469 secondary (functional), 470 clinical manifestations of, 470 definition of, 470 diagnosis of, 470 epidemiology of, 470 pathobiology of, 470 therapy, 470b surgery for, 469 treatment of, 318 Mitral stenosis, 462t, 465-466, 466f clinical manifestations of, 465-466 diagnosis of, 466 epidemiology of, 465 evaluation of, 279 hemodynamics of, 294, 295f invasive evaluation of, 466 measurement of, 279 Volume I  pp 1-1382  •  Volume II  pp 1383-2722

I68

Index

Mitral stenosis (Continued) mechanical therapy for, 466-467 medical therapy for, 466 pathobiology of, 465 physical findings, 466 prevention of, 466 Mitral valve, 264 Mitral valve disease antithrombotic therapy for, 180t myxomatous degeneration, 467, 467.e1f Mitral valve prolapse, 467, 469-470, 467.e1f definition of, 469 diagnosis of, 469 history in, 469 noninvasive evaluation of, 469 physical findings, 469 prognosis for, 470 treatment of, 469b-470b Mitral valve repair, 283f, 468-469, 468f Mitral valve replacement with preservation of mitral apparatus, 469 prosthetic, 276, 277f without preservation of mitral apparatus, 469 Mitral valve surgery, 468-469 Mixed connective tissue disease, 585, 1778 Mixed cryoglobulinemia essential, 1723 neuropathy in, 2533, 2533t Mixed gonadal dysgenesis, 1565, 1584 Mixed hyperbilirubinemia, 986-987 Mixed mineralocorticoid and glucocorticoid deficiency, 1519-1520 Mixed sclerosing bone dystrophy, 1671 Mixed-meal test for hypoglycemia, 1554 suggested protocol for, 1554t Mixing, impaired, 927 MK-383 (tirofiban), 180 MLN 9708, 1281 MMX (budesonide), 939t, 941 Mobility assessment, geriatric, 104-105 Mobitz type I AV block (Wenckebach block), 346, 348, 357, 357f, 452 Moccasin distribution, 2670 Modafinil for myotonic dystrophy, 2543 for narcolepsy, 2418, 2419t for shift-work disordered sleep, 2422 for sleepiness, 2418 Model for End-Stage Liver Disease (MELD) score, 1026t, 2616 for cirrhosis, 1031 for liver transplantation, 1031-1033 Modification of Diet in Renal Disease (MDRD) Study equations, 728, 730t Modified Bruce protocol, 250t Modified Harris-Benedict equation, 716.e1t Moexipril, 388t Mohs’ surgery, 1378 Moisturizers, 2660 Mold exposures, 81t, 82 Molecular diagnostic testing, 200, 201t, 1387 Molecular imaging, 1723 Molecular patterns danger-associated, 231 pathogen-associated, 230 Molecular profiling, 200 clinical implementation of, 203 tumor profiling, 1379, 1381 Molecular technologies, 200-203, 200f, 203.e2t Molecular therapy for cancer, 202, 202t, 1208, 1211t-1216t, 202.e1t forms of, 213 site-specific, 1381 Moles atypical (dysplastic nevi), 1374, 1374f, 1375t, 1377 common acquired nevi, 1374, 1374f, 1375t Molluscipoxvirus, 2214-2215, 2215t Molluscum, 1877t Molluscum contagiosum, 2673, 2688f clinical manifestations of, 2218 diagnosis of, 2218 in HIV infection, 2319 prognosis for, 2219 treatment of, 2219 Molluscum contagiosum virus, 2215 Molybdenum, 1450t-1452t

Momelotinib (MMB, GS-0387, CYT387), 1128 Mometasone (Nasonex), 1692, 1692t Mometasone furoate, 2588, 2658t Mönckeberg sclerosis, 837 Monge’s disease, 596 Monilethrix, 2706 Monkeypox, 2057, 2059t, 2215 clinical manifestations of, 2216-2217 epidemiology of, 2215 prognosis for, 2219 rash of, 2216-2217, 2217f Monkeypox viruses, 2214 Monoamine oxidase inhibitors, 2349t, 2618 Monobactams, 1891t-1892t Monoclonal antibodies for asthma, 553 for chronic lymphocytic leukemia, 1256 nomenclature, 169, 169t Monoclonal gammopathy, 1275f, 2530t malignant, 1273, 1273t neuropathy associated with, 2531-2532 clinical manifestations of, 2532 definition of, 2531 diagnosis of, 2532 epidemiology of, 2531 pathobiology of, 2531-2532 prognosis for, 2532 treatment of, 2532b premalignant, 1273, 1273t of undetermined significance, 1274-1275, 2531 association with other disesaes, 1275 clinical manifestations of, 1274 definition of, 1274 diagnosis of, 1274-1275 diagnostic criteria for, 1276t epidemiology of, 1274 hyperdiploid, 1274 IgH-translocated, 1274 light chain, 1275-1277, 1276t pathobiology of, 1274 prevention of, 1275b prognosis for, 1275 risk of progression to myeloma or related disorders, 1276t treatment of, 1275b, 1279 Monoclonal (M) protein, 1273-1274, 1274f, 1908, 1909f, 2531 Monocyte colony-stimulating factor, 1050 Monocytes, 216 autologous bone marrow mononuclear cells, 451 functions of, 1142-1143, 1143t Monocytic ehrlichiosis, 2057t, 2059t canine, 2054-2055 human American, 2053, 2053t geographic distribution of, 2054f Japanese, 2053t Monocytopenia, 1138 Monocytosis, 1132, 1132t Monofilament test, 1545 Monogenic disorders, 186, 191 high-penetrance, 186, 187t versions of common disorders, 187, 187t Monomethylhydrazine poisoning, 706t-710t Mononeuritis multiplex, 2533, 2533t diagnosis of, 2534 differential diagnosis of, 2534 pathobiology of, 2533 Mononeuropathy, compressive, 2535 Mononucleosis, 2232, 2603 Monosodium urate monohydrate, 1811 Monosodium urate monohydrate crystals, 1811, 1813 Monosomy X, 192 Monosulfiram, 2173 Montana, 7 Montelukast, 554, 1695, 2684t Montreal Cognitive Assessment, 115-116, 116f Mood assessment, 104 Mood disorders, 302, 2347t, 2350 Mood stabilizers, 2350 Mooren’s ulcer, 2570 Moraxella, 1948-1949 clinical manifestations of, 1948 pathobiology of, 1948 sites of, 1948t treatment of, 1948b Moraxella catarrhalis, 1948, 1948t

Moraxella lacunata, 1948t Morbidity dietary factors, 1427t and nutrition, 1426-1430 in older adults, 100 in women, 1602-1604 Morbidity and Mortality Weekly Report (CDC), 66-67 Morbilliform eruptions, 1858-1859, 2672-2673 in immunocompromised patients, 1858-1859 of measles, 2202, 2202f Morning stiffness, 1759 Morphea, 2694, 2694f Morphine, 137 for acute limb ischemia, 500 for acute MI, 448 for distal symmerical polyneuropathy, 2536 formulations, dosages, and pharmacologic information, 139t-140t for left ventricular dysfunction, 452 for renal colic, 813 for snake bite, 718 toxicity, 699t-702t Morquio’s A, 1734t Morquio’s B, 1734t Morquio’s syndrome, 1387, 1733 Mortality nutrition and, 1426-1430 of older adults, 100 of women, 1600-1602 Morvan’s syndrome, 2518t Mosaicism, 193 Mosquito protection, 1884 Mosquito-borne encephalitis, 2058t Mosquito-borne viruses, 2247 Moths, 2177 Motility, twitching, 1935 Motility disorders, gastrointestinal, 884-890, 2518t Motion sickness, 2597 Motivational counseling, 52, 53f for smoking, 147, 147t Motor axonal neuropathy, acute, 2524t Motor block, 2455 Motor disturbances axial in Parkinson’s disease, 2455 Motor neuron diseases classification of disorders, 2523t definition of, 2522 disorders that mimic, 2524t Motor neuropathy compressive focal, 2524t hereditary, 2524t multifocal, 2530t, 2532 clinical clues, 2524t differential diagnosis of, 2534 peripheral, 2528t Motor responses, 2412, 2412t Motor seizures, 2401t, 2404, 2405f Mounier-Kuhn syndrome, 567 Mountain sickness acute, 596 clinical manifestations of, 596 prophylaxis of, 596-597 treatment of, 597 chronic, 596 clinical manifestations of, 596 treatment of, 597 Mouse mites, 2052 Mouth diseases of, 2579-2585 examination of, 26 Movement(s) abnormal, 2469 spontaneous, 2340 Movement disorders, 2461-2470 clinical manifestations of, 2461 diagnostic approach to, 2461 drug-induced, 2467-2468 rhythmic movement disorder, 2423, 2423t Movement therapy, 182t, 183 Moving toes, 2469 Moxifloxacin for AIDS-associated opportunistic infections, 2297t-2301t for anaerobic infection, 1934t for bacterial meningitis, 2488t for eye infections, 2565t

Moxifloxacin (Continued) for infectious rhinosinusitis, 2588 for legionnaires’ disease, 1996t for nocardiosis, 2064 for pneumonia, 616t, 619t recommended doses and schedules, 1891t-1892t for S. pneumoniae infection, 1905t for septic shock, 689t for tuberculosis, 2312t Moyamoya disease, 2443t, 2625 Moyamoya syndrome, 2443t Mucinous adenocarcinoma, invasive, 571t, 574-575, 575b, 574.e1f Muckle-Wells syndrome, 231, 1740t-1741t, 1742 Mucoceles, 2584 Mucocutaneous candidiasis, 2079-2080 AIDS-associated, 2295, 2297t-2301t chronic, 1686-1687, 1687t, 2080 treatment of, 2082 Mucocutaneous diseases, oral, 2583 Mucocutaneous herpes AIDS-associated, 2297t-2301t treatment of, 2181t, 2297t-2301t Mucolipidoses, 1733, 1734t Mucolipidosis IV, 1385t Mucopolysaccharidoses, 1387, 1403, 1733-1734, 1734t characteristic signs of, 1385t clinical manifestations of, 1733 definition of, 1733 diagnosis of, 1734 differential diagnosis of, 1734 enzyme replacement therapy for, 1388 epidemiology of, 1733 pathobiology of, 1733 pathology of, 1733 pathophysiology of, 1385t treatment of, 1388t, 1734b Mucormycosis, 2087-2091 clinical manifestations of, 2088-2089 culture for, 2089-2090 definition of, 2087 diagnosis of, 2089-2090 disseminated, 2089 epidemiology of, 2087 gastrointestinal, 2088-2089 histopathology of, 2089 management of, 2090, 2090f management of comorbidity in, 2091 pathobiology of, 2087-2088 pulmonary, 2088, 2089f rare clinical presentations, 2089 rhinocerebral, 2088 skin and soft tissue, 2088 treatment of, 2090b-2091b adjunct, 2091 antifungal therapy, 2090-2091, 2090f strategy for, 2090 Mucosa-associated lymphoid tissue (MALT) lymphoma, 1319 cutaneous, 2691, 2691f extranodal marginal zone, 1263 Mucosal diseases oral, 2579-2583 tests for, 925t Mucosal leishmaniasis, 2123 Mucosal malabsorption, 929 Mucosal pigmentation, 1324, 1325f Mucosal telangiectasia, in colon, 875f Mucositis, 2654, 2655f Mucous membrane pemphigoid, 2676-2677 definition of, 2676 esophageal, 906 oral, 2580t, 2583 pathobiology of, 2676 prognosis for, 2676 treatment of, 2676b Mucous membranes, 1771 Mucous retention lesions, 2584 Mucus retention cysts, 2588 Muehrcke’s lines, 2708f Muir-Torre syndrome, 1324, 2688 Müllerian ducts anomalous, 1583-1584 persistent, 1565 Multibacillary leprosy, 2043, 2043f, 2046 Multicentric reticulohistiocytosis, 1826 Multicystic dysplastic kidney, 825 Multidisciplinary teams, 650 Multidrug resistance, 1838

Index Multidrug resistance-associated protein 3 (MDR3), 987-988 Multidrug-resistant organisms, 1862-1864, 1863t, 1865.e1t Multifactorial disorders, 186, 187f, 188-189 Multifocal atrial tachycardias, 360f, 363 Multifocal leukoencephalopathy, progressive, 2210-2212, 2502t Multifocal motor neuropathy, 2530t, 2532 clinical clues, 2524t clinical manifestations of, 2532 definition of, 2532 diagnosis of, 2532 differential diagnosis of, 2534 epidemiology of, 2532 pathobiology of, 2532 prevalence of, 2532 treatment of, 2532b Multifocal osteomyelitis, chronic recurrent, 1740t Multimineral supplements, 1455 Multiminicore disease, 2618 Multinodular goiter, toxic, 1506 Multiple comparisons, 36 Multiple endocrine neoplasia (MEN), 187t, 1523 clinical manifestations of, 1662t medullary thyroid carcinoma in, 1662 Multiple endocrine neoplasia 1, 1227t, 1556, 1556t, 1652t Multiple endocrine neoplasia 2, 1227t, 1556, 1556t, 1652t Multiple endocrine neoplasia 2a clinical manifestations of, 1662, 1662t medullary thyroid carcinoma in, 1662 prevention of, 1663 prognosis for, 1663 Multiple endocrine neoplasia 2b clinical manifestations of, 1662-1663, 1662t medullary thyroid carcinoma in, 1662 prevention of, 1663 prognosis for, 1663 Multiple endocrine neoplasia 4, 1556, 1556t Multiple exostoses, hereditary, 1672 Multiple hypothesis testing, 36-37, 193 Multiple linear regression, 34 Multiple logistic regression, 34-35 Multiple myeloma, 1277-1281, 1278f chronic renal failure of, 798 clinical manifestations of, 1277-1278, 1277t complications of, 1281 management of, 1281 neurologic, 1278 cytogenetic abnormalities in, 1277 definition of, 1277 diagnosis of, 1278-1279 diagnostic criteria for, 1276t epidemiology of, 1277 history in, 1277 laboratory findings, 1278, 1278f organ involvement, 1278 pathobiology of, 1277 physical findings, 1278 prognosis for, 1281 prognostic factors, 1277t radiologic findings, 1278, 1278f relapsed refractory, 1280-1281 smoldering (asymptomatic), 1276t, 1281 staging, 1277t systemic involvement, 1279 treatment of, 1279b-1281b, 1279f future directions, 1281 hematopoietic stem cell transplantation, 1202 initial therapy, 1279-1280 radiation therapy, 1281 regimens, 1280t variant forms, 1281-1282 Multiple organ failure, 716 Multiple pterygium, 2548t Multiple sclerosis, 2471-2478 behavioral symptoms of, 2473 clinical features of, 2472-2474, 2524t cognitive symptoms of, 2473 conditions that can be mistaken for, 2472t CSF findings, 2475 definition of, 2471

Multiple sclerosis (Continued) diagnosis of, 2474-2476 diagnostic criteria for, 2474t differential diagnosis of, 2476 epidemiology of, 2471 evoked potentials in, 2475 incidence of, 2471 laryngeal, 2606 motor symptoms of, 2473 MRI criteria for, 2474-2475, 2474t MRI findings, 2474-2475, 2474f-2475f optical coherence tomography findings, 2475-2476, 2476f opticospinal, 2478-2479 organ dysfunction in, 2473 pathobiology of, 2471 pathogenesis of, 2472 pathology of, 2471-2472 pattern reversal visual evoked potentials in, 2344-2345 in pregnancy, 2473 presenting symptoms of, 2472 prognosis for, 2478 sensory abnormalities in, 2472 systemic symptoms of, 2473 treatment of, 2476b-2478b approved disease-modifying treatments, 2477 hematopoietic stem cell transplantation, 1203 for specific symptoms, 2476-2477 systemic treatments, 2477 types of, 2473-2474 vertigo with, 2599 visual effects of, 2473 Multiple system atrophy, laryngeal, 2606 Multiple testing, 39 Multiplex ligation-dependent probe amplification (MLPA), 195 Multiplex testing, 1305-1306 Multisystem inflammatory disease, neonatal-onset (NOMID), 231, 1740t-1741t, 1742 Multivariable statistics, 34-35 Multivitamin supplements, 1455 Mumps, 2206-2208, 2207f clinical manifestations of, 2207, 2207f definition of, 2206 diagnosis of, 2207 epidemiology of, 2206 pathobiology of, 2206-2207 prevention of, 2208 testicular hypogonadism due to, 1573 treatment of, 2208b Mumps meningitis, 2490-2491 Mumps vaccine, 68t-73t, 75, 2208 adverse reactions to, 75 indications for, 75 Mumps virus, 212, 2206 Munchausen’s disorder, 2355t Mupirocin (Bactrian) for impetigo, 2695-2696 for S. aureus infection prevention, 1901 for wound healing, 2657 Murine typhus, 2048t, 2051 clinical manifestations of, 2051 definition of, 2051 diagnosis of, 2051 epidemiology of, 2051 prognosis for, 2051 treatment of, 2051b Murphy’s sign, 858t, 868, 978 Murray Valley encephalitis, 2263t, 2268 Muscle(s): structure of, 2538, 2538f Muscle biopsy, 2541 Muscle channelopathy, 2537 Muscle cramps, 2340, 2539 Muscle disease, 2537-2547 assessment of, 2540t clinical features of, 2539-2540, 2539t definition of, 2537 diagnosis of, 2540-2541 EMG findings, 2540 epidemiology of, 2537 genetic testing in, 2540-2541 history in, 2539 imaging findings, 2541 inflammatory, 2546-2547 inherited diseases, 2541 pathobiology of, 2538-2539

Muscle disease (Continued) physical examination in, 2539-2540 related disorders, 2542t Muscle pain, 2340, 2539 Muscle relaxants, 137, 138t Muscle spasms, 1930 Muscle stiffness, 2544-2545 Muscle strain, 2375t Muscle strength grading, 2540, 2540t respiratory, 655 Muscle strengthening, 58-59, 1822 Muscle tension dysphonia, 2605 Muscle testing, 2371, 2372t Muscle wasting, 2547 Muscle weakness, 1746 Muscle-specific kinase antibodies, 2550 Muscular atrophy spinal, 2525-2526 spinobulbar, 2526 Muscular dystrophy, 2541-2544, 2541t autosomal dominant, 2541t autosomal recessive, 2541t congenital, 2544-2546 X-linked, 2541t Muscular rings, 905 Musculoskeletal conditions, 1750t chest pain in, 249t fever and, 1851-1852 laboratory evaluation of, 1719-1720 occupational, 80, 80t of trunk, 80, 80t of upper extremity, 80, 80t Musculoskeletal system age-related changes in, 108 examination of, 27-28 in systemic lupus erythematosus, 1771-1772 Mushroom poisoning, 1010 Musician’s cramp, 2463-2464 Mustard atrial baffle repair, 414, 416t Mutations, 192f, 193-195 amorphic, 193 antimorphic, 193 associated with myelodysplastic syndromes, 1234, 1234t compound heterozygous, 193-194 definition of, 193 DNA repair gene, 1227t double heterozygous, 193-194 dynamic, 194 frameshift, 194 G protein–coupled receptor gain-offunction mutations, 1473, 1473t G protein–coupled receptor loss-offunction mutations, 1473, 1473t germline, 193 hemizygous, 193-194 heterozygous, 193-194 homozygous, 193-194 hypomorphic, 193 linked to type 2 diabetes, 1533 in liver disease, 1422-1423 missense, 194 neomorphic, 193 nonsense, 194 nonsynonymous, 193-194 null, 193 oncogene, 1227, 1227t premutations, 194 random, 1226 rates of, 1226 reproductive hormone, 1591 somatic, 193, 1410 synonymous, 193 tumor suppressor, 1227t X chromosome, 1590 MUTYH-associated polyposis, 1324 clinical features of, 1323t, 1324 diagnosis of, 1324 pathobiology of, 1324 treatment of, 1324b Myalgic encephalomyelitis, 1821 Myasthenia gravis, 2547-2551, 2548t clinical manifestations of, 2549-2550, 2550f diagnosis of, 2550-2551 diagnostic evaluation of, 2550t differential diagnosis of, 2551 epidemiology of, 2547

I69

Myasthenia gravis (Continued) generalized with AChR antibodies, 2550 treatment of, 2551 laryngeal, 2606 with muscle-specific kinase antibodies, 2550 with neither AChR nor MuSK antibodies, 2550 neonatal transient, 2548t treatment of, 2551 ocular, 2550-2551, 2577-2578 pathobiology of, 2547-2549 pathogenesis of, 2549 pathophysiology of, 2547-2549 prognosis for, 2551 subtypes, 2549-2550 thymoma-associated, 2551 treatment of, 2551b Myasthenic syndromes congenital, 2548t, 2553 genetic, 2548t, 2553 Myasthenic weakness, 2551 Mycetoma, 2099-2101, 2100f clinical manifestations of, 2099-2100, 2100f definition of, 2099 diagnosis of, 2100-2101 differential diagnosis of, 2100-2101 epidemiology of, 2099 geographic distribution, 2099, 2100f intracavitary, 612, 612.e1f pathogenesis of, 2099 pathogens that cause, 2099, 2100f treatment of, 2101b Mycobacteria atypical, 2697 nontuberculous, 2039-2042, 2040t Mycobacteria growth indicator tube (MGIT) system, 2036-2037 Mycobacterial disease, nontuberculous in cervical lymph nodes, 2041 clinical manifestations of, 2040-2041 cutaneous, 2697 diagnosis of, 2041-2042 disseminated disease, 2040 epidemiology of, 2040 in HIV infection, 2315 in immune reconstitution inflammatory syndrome, 2333-2334 pathobiology of, 2040, 2041f prevention of, 2042 prognosis for, 2042 pulmonary, 2040-2042 radiographic features of, 2041-2042, 2041f skin and soft tissue, 2041 treatment of, 2042b Mycobacterial pharyngitis, 2604 Mycobacterium abscessus, 2040 Mycobacterium africanum, 2030-2031 Mycobacterium avium complex, 2040 Mycobacterium avium complex infection HIV/AIDS-associated disseminated, 2295, 2297t-2301t radiographic features of, 2309t prevention of, 2042 treatment of, 2042 Mycobacterium avium complex–associated IRIS, 2333-2334, 2334f Mycobacterium avium-intracellulare infection, 1869t Mycobacterium bovis, 2030-2031 Mycobacterium chelonae, 2697 Mycobacterium gordonae, 2041-2042 Mycobacterium kansasii, 2040, 2042, 2309t, 2315 Mycobacterium leprae, 2042, 2604 Mycobacterium marinum, 2041, 2697 Mycobacterium massiliense, 2040 Mycobacterium tuberculosis, 2030-2031, 2697 Mycophenolate mofetil, 166 adverse effects of, 166, 168, 1037t for bullous pemphigoid, 2675 for glomerulonephritis, 791 for glomerulosclerosis, 786 indications for, 166 for lichen planus, 2668t for lupus nephritis, 792 mechanism of action, 166 Volume I  pp 1-1382  •  Volume II  pp 1383-2722

I70

Index

Mycophenolate mofetil (Continued) for minimal change disease, 785-786 monitoring, 166, 1037t for myasthenia gravis, 2551 for pemphigus, 2677 for psoriasis, 2666t in renal transplantation, 844t for sarcoidosis, 607t for skin conditions, 2660 for systemic lupus erythematosus, 1776 Mycoplasma, 2002 Mycoplasma arginini pneumonia, 2058t Mycoplasma arthritis, 1808 Mycoplasma fermentans, 2002-2003, 2002t Mycoplasma genitalium, 2002, 2002t Mycoplasma hominis, 1808, 2002, 2002t Mycoplasma infections, 2002-2007 clinical manifestations of, 2004-2005 epidemiology of, 2002-2003 neurologic complications of, 2005 pathobiology of, 2003 prevention of, 2006 sites of, 2002, 2002t treatment of, 1933-1934 Mycoplasma orale, 2002t Mycoplasma pneumonia, 2002t cardiac complications of, 2005 clinical manifestations of, 2003f, 2004 conditions leading to increased susceptibility for, 2005 dermatologic involvement, 2004-2005, 2004f diagnosis of, 2005-2006 epidemiology of, 2002 extrapulmonary involvement, 2004-2005 hematopoietic complications of, 2005 musculoskeletal complications of, 2005 neurologic complications of, 2005 pathobiology of, 2003 prevention of, 2006 renal complications of, 2005 respiratory infection, 2004, 2004f treatment of, 2006b Mycoplasma pneumoniae, 2603 Mycoplasma salivarium, 2002-2003, 2002t Mycoses, 2068 Mycosis fungoides, 1265, 1265b-1266b, 2648f, 2669-2670, 2669f-2670f, 2670t Mycotic aneurysms, 2448 MyD88 deficiency, 1686-1687, 1687t Mydriatic drugs, 2576 Myelin diseases, 2471t-2472t, 2478-2480 Myelitis cytomegalovirus, 2208 transverse, 2381 Myeloblastomas, 1242 Myeloblasts, 1143-1144 Myelocytes, 1143-1144 Myelodysplastic syndromes, 1233-1239 blood smear features, 1056t clinical manifestations, 1235 definition of, 1233 diagnosis of, 1235-1236 diagnostic criteria for, 1233 dysplastic abnormalities in, 1235, 1235f epidemiology of, 1233 familial, 1233 gene mutations associated with, 1234, 1234t general approach to, 1236 incidence of, 1233 International Prognosis Scoring System (IPSS), 1238 pathobiology of, 1233-1235, 1235f prevention of, 1238 prognosis for, 1238 revised International Prognosis Scoring System (IPSS-R), 1238, 1238t-1239t risk factors for, 1233 secondary, 1233 subtypes, 1236, 1236t supportive care for, 1236-1237 therapy-related, 1233 treatment of, 1236b-1238b, 1237-1238 hematopoietic stem cell transplantation, 1201-1202 immunosuppressive therapy, 1237 iron chelation therapy, 1237 suggested algorithm for, 1237f Myelofibrosis, primary, 1121-1129 bone marrow findings, 1124, 1125f clinical manifestations of, 1124

Myelofibrosis, primary (Continued) diagnosis of, 1124, 1125f diagnostic criteria for, 1124t incidence of, 1122 mutations in, 1122, 1123t prognosis for, 1128-1129 radiation therapy for, 1128 risk stratification in, 1127t risk-adapted therapy for, 1127t treatment of, 1128 Myelogenous leukemia, chronic blood smear features of, 1058-1059, 1059f diagnostic algorithm for, 1125f neutrophilia in, 1131 Myelography, 2345t Myeloid growth factors, 1245, 1858 Myeloid leukemia acute, 1201, 1233 chronic, 1202 Myeloid malignancy, 1121-1122, 1122t Myelokathexis, 1133t, 1134 Myeloma, 1370 chronic renal failure of, 798 nonsecretory, 1281 osteosclerotic, 1281-1282 solitary, 1282 Myeloma kidney, 798, 1281 Myelopathy causes of, 2378t, 2379-2382 clinical manifestations of, 2378 definition of, 2378 differential diagnosis of, 2380 in HIV infection, 2331, 2332t HTLV-associated, 2236, 2237t, 2238-2239 inflammatory, 2381 ischemic, 2380 metabolic, 2381 neck pain in, 2374t vascular, 2380-2381 Myeloperoxidase deficiency, 1147t-1148t, 1149, 1149b Myelophthisis, 1130, 1131f Myeloproliferative disorders, 1121-1122 blood smear features of, 1124, 1125f bone marrow features of, 1124, 1125f neutrophilia in, 1131 Myeloproliferative neoplasms, 1121-1122 BCR-ABL1–negative, 1122, 1123t experimental drug therapy for, 1128 hematopoietic stem cell transplantation for, 1202 thrombosis with, 1190, 1190b Myenteric (Auerbach’s) plexus, 896-897, 2517 MYH-associated polyposis, 1227t Myiasis furuncular, 2176 Mylanta II, 900t Myocardial biopsy, 305 Myocardial contractility, 266 Myocardial disease, 320-335, 321t Myocardial imaging CMR, 291f PET, 286 Myocardial infarction, 249t acute causes of, 442, 442t clinical manifestations of, 369 coronary angioplasty for, 458, 459f diagnosis of, 442, 444 early phase, 443 evolved phase, 443 hemodynamic subsets, 448t key echocardiographic findings, 280t-281t pathobiology of, 442, 442t risk assessment, 248, 250f ST segment elevation, 441-456 chronic, calcified, 284f chronic phase, 443-444 complications of, 441-456 definition of, 441 differential diagnosis of, 484t discharge medication checklist for, 454t epidemiology of, 441-442 left circumflex pattern, 444 MR findings, 290f nonfatal, 260f-261f non-ST segment elevation, 432-441 CABG for, 460 definition of, 432 with OCs, 1608

Myocardial infarction (Continued) pathobiology of, 442, 442t post-MI pericarditis, 485t, 491 post-MI ventricular tachycardia, 371, 368.e1f postoperative, 2621-2622, 2622t risk stratification after, 453-454 ST segment elevation, 432, 441-456 true posterior pattern, 444 Myocardial ischemia differential diagnosis of, 484t silent, 432 Myocardial necrosis, 444-445 Myocardial perfusion imaging, 285-286, 286f imaging agents for, 285 SPECT, 286, 286.e1f Myocardial revascularization, 428t, 430-431 Myocardial sarcoidosis, 291f Myocardial tumors, 338-339 Myocarditis, 327 acute, 290f, 327 antipsychotic-induced, 2626t, 2627 borderline, 327 causes of, 327t clinically suspected, 328 definition of, 327 epidemiology of, 327 giant cell, 328 immune-mediated, 328 MR findings, 328, 328f pathobiology of, 327 patterns of, 327 persistent, 327 resolving or resolved, 327 toxoplasmosis, 328 treatment of, 328b-329b viral, 327-329 Myoclonic epilepsy juvenile, 2406 with ragged-red fibers (MERRF), 2546 severe, of infancy, 2404 Myoclonic seizures clinical manifestations of, 2401t, 2402-2403 drugs for, 2407t Myoclonus, 2466 causes of, 2467t classification of, 2467t clinical manifestations of, 2466 definition of, 2466 diagnosis of, 2466 pathobiology of, 2466 tardive, 2468 in throat, 2605 treatment of, 2466b-2467b Myoclonus dystonia, 2465 Myofascial pain, 1817, 1822-1823 regional syndromes, 1713t treatment of, 142t, 1822-1823, 1822b Myofascial trigger points, 1822, 1822f diagnostic features of, 1822, 1822t injections at, 1822 management of, 1822 Myofibrillar myopathy, 2544, 2544f Myoglobin testing, 725 Myoglobinuria, 2546t Myokyomia, 2473 Myonecrosis, clostridial, 1926-1927 Myopathic shock, 678f-679f Myopathy classification of, 2537, 2537t clinical features of, 2539, 2539t congenital, 2541t, 2544 HIV, 2332 inflammatory, 2546-2547, 2547t metabolic, 2542t, 2545-2546, 2545b mitochondrial, 2542t, 2545-2546 toxic, 2546, 2546t Myopericarditis, 491 acute, 328f clinical manifestations of, 2242 diagnosis of, 2242 enteroviral, 2241t, 2242 prognosis for, 2242b treatment of, 2242b Myopia, 2557-2558, 2557f Myosin loss, 2546t Myositis, 2243 inflammatory, 1716t, 2243 tropical, 1898 Myotomes, 2370-2371

Myotomy Heller, 904 peroral endoscopic (POEM procedure), 878-879, 904 Myotonia, 2539, 2546t Myotonia fluctuans, 2542t Myotonia permanens, 2542t Myotonic dystrophy, 2543-2544 clinical features of, 2539, 2540f, 2543 diagnosis of, 2543 treatment of, 2543b Myotoxicity, local, 718 MyPlate (USDA), 1430, 1430f Myxedema, 1502 Myxedema coma, 1504 Myxoma, 338-339 atrial, 2443t clinical manifestations of, 338 definition of, 338 diagnosis of, 339 epidemiology of, 338 treatment of, 339b

N

Nabilone, 865t Nadolol, 430t for hypertension, 388t for torsades de pointes, 374 for tremor, 2462 for varices and varical bleeding, 1029 NADPH enzyme system, 1146 Nadroparin, 177t Naegleria fowleri, 2142, 2142t Nafcillin, 1891t-1892t for endocarditis, 479t-480t, 480 for lateral sinus thrombosis, 2499 for meningitis, 2488t for osteomyelitis, 1810t for septic arthritis, 1810t for spinal epidural abscess, 2497 for Staphylococcus infection, 1901, 2679 Nagasaki, 1240 Na/H exchangers (NHE3), 763-764 Nail(s), 2634 longitudinal grooves and striations in, 2707 normal, 2707 Nail disorders, 2707-2710 Nail melanoma, 2710, 2710f Nail pigmentation, 2709 Nail pitting, 1768, 1768f, 2707, 2708f Nail-patella syndrome, 791 Nairobi eye or rove beetle dermatitis, 2176 Naja haje (Egyptian cobra), 719 Naja mossambica (spitting cobra), 719 Naja nigricolis (spitting cobra), 719 Nakajo-Nishimura syndrome, 1740t Nalbuphine, 137, 139t-140t Nalmefene, 1021 Naloxone, 139t-140t for cold injury, 695 for coma, 2412 for drug overdose, 699t-702t, 706t-710t Naltrexone for alcohol dependence, 155-156, 155t for alcoholic liver disease, 1021 for opioid dependence, 159 for pruritus, 2636t for tobacco dependence, 147t Naphthalene, 1088t Naproxen for chronic prostatitis/chronic pelvic pain syndrome, 831-832 for diarrhea, 922 for gastroenteropathy, 934 for headache, 2357b, 2358, 2358t, 2360 for spondyloarthritis, 1764 Naratriptan, 2358-2359, 2359t Narcissistic personality disorder, 2355t Narcolepsy, 2418 with cataplexy, 2418 clinical manifestations of, 2418 definition of, 2418 diagnosis of, 2418 epidemiology of, 2418 pathobiology of, 2418 prognosis for, 2418 treatment of, 2418b, 2419t Narcotics, 117t NARES (non-AR with eosinophilia syndrome), 1691 Nasacort (triamcinolone), 1692

Index Nasal complaints, 2586-2589 Nasal congestion, 2186 Nasal cromolyn, 1692 Nasal polyps, 2586, 2587f, 2588-2589, 2589f Nasal treatments, 641 Nasarel (flunisolide), 1692 Nasogastric feeding, 1439t Nasonex (mometasone), 1692 Nasopharyngeal carcinoma causative factors, 1228 classification of, 1298 clinical manifestations of, 1299-1300 EBV-associated, 2234 epidemiology of, 1298 Nasopharynx, 2601, 2601f Natalizumab (Tysabri), 171 for inflammatory bowel disease, 939t, 940 for multiple sclerosis, 2477 National Action Plan to Prevent HealthcareAssociated Infections, 1868.e1 National Cancer Institute (NCI) criteria for chronic lymphocytic leukemia, 1256t SEER program, 43.e1t National Center for Complementary and Alternative Medicine (NCCAM), 181-182 National Cholesterol Education Program Adult Treatment Panel III report, 1395, 1428t-1429t National Comprehensive Care Network (NCCN) guidelines for adjuvant systemic therapy, 1355 guidelines for sarcoma management, 1372 recommendations for myeloid growth factor use, 1858 National Guidelines Clearinghouse, 43 National Hansen’s Disease Program, 2046 National Health and Nutrition Examination Survey (NHANES), 32 standards for arm muscle circumference, 1432, 1433t standards for skinfold measurement, 1432, 1433t National Healthcare Quality and Disparities Reports (AHRQ), 43.e1t National Heart, Lung, and Blood Institute (NHLBI), 1458, 1464-1465 National High Blood Pressure Education Program, 1428t-1429t National Human Genome Research Institute, 194, 201 National Institute for Health and Care Excellence (NICE), 121t National Institute on Alcohol Abuse and Alcoholism, 154 National Institutes of Health (NIH) Chronic Prostatitis Symptom Index (NIH-CPSI), 831, 832f Guideline for the Management of Overweight and Obesity in Adults, 1458, 1465 Guideline on Management of Opportunistic Infections in Adults and Adolescents, 2294-2295 Guidelines for the Prevention and Treatment of Opportunistic Infections in HIV-Infected Adults and Adolescents, 2294 Patient Reported Outcomes Measurement Information System, 43 recommendations for prevention of HIV in medical care settings, 2287 Stroke Scale, 2436, 2437t-2438t on total hip replacement, 1830 National Prion Disorders Pathology Service Center, 2506 National Quality Forum (NQF), 43.e1t National Quality Measures Clearinghouse, 43 National Surgical Improvement Program registry, 42 National Vaccine Injury Compensation Program, 66 Native Americans, 15 access to health care, 15 chronic kidney disease in, 833-834 cystic fibrosis among, 562 gallstones in, 1038 population, 15

Native Americans (Continued) quality of health care, 15-16 suicide rates, 61 tularemia in, 1982 type 2 diabetes mellitus in, 1532 Native Hawaiians, 15 Natriuretics, 748, 748t collecting duct, 749 distal tubule, 748-749 loop, 748 proximal tubule, 748 Natural killer cell inhibitory receptors (KIRs), 217, 239, 239.e1f Natural killer cells, 216, 236t, 238, 239.e1f Natural killer/T-cell lymphoma, extranodal, 1266 Natural products, 181-182, 182t Natural selection, 191 Naturopathy, 182t Naughton protocol, 250t Nausea and vomiting, 861 chemotherapy-related, 1219 definition of, 861 diagnosis of, 861 in HIV infection, 2303t management of, 11t palliative care, 11t postoperative, 2620 treatment of, 852t, 861b, 865t Navelbine (vinorelbine), 1211t-1216t Naxos disease, 331 Near-drowning, 595 Near-fainting, 2600t Nearsightedness, 2558, 2566 Nebivolol, 311t, 388t Nebraska, 7 Necator americanus, 2103, 2159-2161 Neck abscesses, 2603 Neck adenopathy, HBV-positive, 1300f Neck examination, 26-27 Neck injury, 2373, 2373f Neck masses, 1300, 1300f Neck pain, 2372-2375 acute, 2372 ancillary testing in, 2373 anterior, 2374 approach to, 2374f clinical manifestations of, 2372-2373 diagnosis of, 2373-2375 differential diagnosis of, 2374-2375 epidemiology of, 2372 history in, 2373 mechanical, 2374, 2374t new-onset, 2374f pathobiology of, 2372 prognosis for, 2375 rheumatoid arthritis, 1758 scalp eschar, neck lymphadenopathy after tick bite (SENLAT), 2048t, 2050-2051 treatment of, 2375b Neck strain, 2374t Necrobiosis lipoidica, 2640f Necrolytic acral erythema, 2670, 2670f Necrolytic migratory erythema, 1337, 1337f Necrosis dermal, 2638t-2644t, 2642f epidermal, 2638t-2644t, 2642f keratinocyte, 2638t-2644t, 2642f Necrotizing clostridial tissue infection, 1926-1927 Necrotizing fasciitis, 1898, 2696-2697 antibiotic regimens for, 689t definition of, 1908 streptococcal, 1908 Necrotizing lymphadenitis, histiocytic, 1267 Necrotizing myopathies, toxic, 2546t Necrotizing pancreatitis, acute, 960 Necrotizing pneumonia, 612 Necrotizing pulmonary aspergillosis, chronic, 2084 Necrotizing sialometaplasia, 2580t Necrotizing vasculitis, 1858, 1859f Neer’s sign, 1751, 1752f Negative feedback regulation, 1471, 1471f Negative predictive value, 37t Negative selection, 237 Negative-pressure ventilators, 665 Neglect, hemispatial, 2385-2386 Neisseria gonorrhoeae, 1808, 1940

Neisseria gonorrhoeae infection, 1940-1945 antimicrobial susceptibility, 1944 clinical manifestations of, 1941 clinical syndromes, 1941-1945 culture of, 1941 cutaneous, 2697, 2697f diagnosis of, 1941 disseminated, 1943-1944, 1944f epidemiology of, 1869t, 1940-1941 incidence of, 1877 nucleic acid amplification testing for, 1941 pathobiology of, 1941 pharyngeal, 1943, 2603 rectal infection, 1943 septic arthritis, 1808 treatment of, 1944b-1945b duration of therapy, 1896 follow-up, 1945 principles of, 1944 regimens, 1944-1945, 1945t Neisseria lactamica, 1935 Neisseria meningitidis, 1934 Neisseria meningitidis infection, 1934-1940 antibiotic treatment of, 1937, 1937t chemoprophylaxis of, 1938, 1939t clinical manifestations of, 1936-1937, 1937f diagnosis of, 1937 epidemiology of, 1934, 1935f incidence of, 1934 pathobiology of, 1934-1936, 1936f prevention of, 1938-1939 prognosis for, 1939-1940 travel-related risk, 1881 treatment of, 1937b-1938b Nelfinavir, 2289, 2331t Nelson’s syndrome, 1491-1492 Nemaline myopathies, 2544 Nemaline rod myopathy, 2544, 2544.e1f Nematodes (roundworms) intestinal infections, 2103, 2159-2164, 2160t systemic infections, 2103-2104 tissue infections, 2164-2171, 2165t Neofibularia, 722 Neomycin, 1030 Neonatal screening, 201-202, 565 Neonatal-onset multisystem inflammatory disease (NOMID), 231, 1740t-1741t, 1742 Neonates alloimmune thrombocytopenia in, 1167 convulsions in, benign, 2404 cytomegalovirus infection in, 2230 encephalitis test for, 2502t epilepsy syndromes in, 2404, 2406t group B Streptococcus disease in, 1911 herpes in, 2181t herpes simplex in, 2226-2227 hyperparathyroidism in, severe, 1652t, 1657 jaundice in, 1088, 1088b lupus in, 1773-1774 myasthenia gravis in, 2548t, 2551 unconjugated hyperbilirubin in, 986 Neoplasms. See also specific sites and types connective tissue, 1371-1373 distinguishing features of, 528t EBV-associated, 2234 of esophagus, 1313-1314 eyelid, benign, 2561 gastric, 1316-1320 gastrointestinal lesions, vascular, 958 of large intestine, 1322-1331 malignant. See also Cancer alcohol-related, 152 myeloproliferative, 1121-1122 polyglandular syndromes, 1556, 1556t pulmonary, 1303-1313 pupillary abnormalities due to, 2576 of small intestine, 1320-1322 subepithelial, 2583t visual field abnormalities due to, 2575 Neoplastic hypopyon, 2572 Neoplastic pericarditis, 484t-485t Neoplastic polyps, 1326 Neorickettsia, 2047t, 2053 Neorickettsia sennetsu, 2053 Neosar (cyclophosphamide), 1211t-1216t

I71

Neo-Synephrine (phenylephrine), 2586 Nephritic syndrome, 733-735, 788-790 Nephritis acute interstitial, 779-780, 779t athletic pseudonephritis, 725 indications for renal biopsy for, 796t lupus, 1772 classification of, 1772, 1772.e1t histopathology of, 1772, 1772f tubulointerstitial, 793-799 Nephrogenic diabetes insipidus, 823t, 1497, 1499 Nephrogenic fibrosing dermatopathy, 1785, 2694, 2694f Nephrogenic sclerosing dermopathy, 500 Nephrogenic systemic fibrosis, 500, 807-808, 1728, 1785 Nephrolithiasis, 811-816 calcium oxalate, 814, 815f clinical manifestations of, 812 diagnosis of, 812 epidemiology of, 811 medical evaluation of, 814 pathobiology of, 811-812 prognosis for, 816 treatment of, 813b-814b initial surgical treatment, 814 medical therapy, 813 24-hr urine collection in, 814, 814t Nephrolithotomy, percutaneous, 814 Nephronophthisis, 821 clinical features of, 817t, 821 definition of, 821 diagnosis of, 821 epidemiology of, 821 prevention of, 821b treatment of, 821b Nephrons, 737, 738f Nephropathy, 1101, 1652t Nephrotic syndrome, 733-735, 784-788 clinical features of, 784-785, 784t definition of, 784, 837 diagnosis of, 785 epidemiology of, 784 idiopathic, 784-788, 784t pathobiology of, 784 secondary, 784, 784t treatment of, 785b Nernst equation, 339-340 Nerve agents, 706t-710t Nerve biopsy, 2528 Nerve blocks, 141 Nerve conduction study, 2342, 2343t Nerve damage, 2043 Nerve deafness, 1652t Nerve root disorders, 2377-2378 ancillary testing in, 2377 clinical manifestations of, 2377 definition of, 2377 diagnosis of, 2377-2378 differential diagnosis of, 2378 pathobiology of, 2377 treatment of, 2378b Nerve sheath tumors, 1296 Nervous system. See also Autonomic nervous system; Central nervous system; Peripheral nervous system alcohol-related complications, 151-152, 151t cutaneous nerves, 2635-2637 Nesiritide, 318, 749 Net acid excretion (NAE), 766 Netherlands euthanasia and physician-assisted suicide in, 7-8 sarcoidosis in, 603 Netilmicin, 1891t-1892t Netscher, Caspar, 1061, 1061f Neupogen (filgrastim), 1211t-1216t Neural crest cell tumors, 2689 Neural tube defects, 1430 Neuralgia, post-herpetic, 2536, 2680 Neuralgiform headache, short unilateral, 2357t, 2360 Neurally adjusted ventilatory assist, 666 Neuraminidase, 2192t Neuraminidase inhibitors, 2183-2184 Neuraxial (spinal and epidural) anesthesia and analgesia, 2620 Neuritis, vestibular, 2600, 2601t Volume I  pp 1-1382  •  Volume II  pp 1383-2722

I72

Index

Neurocardiogenic syncope, 346-347, 350t-351t Neurocognitive disorders, 2347t HIV-associated, 2329-2331 clinical manifestations of, 2329 definition of, 2329 diagnosis of, 2329 epidemiology of, 2329 evaluation of, 2330t Memorial Sloak-Kettering scale for, 2330t neuropsychological battery proposed for, 2330t pathobiology of, 2329 screening test for, 2330t treatment of, 2330b, 2331f Neurocognitive dysfunction, HIV-associated (HAND), 2302 Neurocutaneous disorders, 2514-2517 Neurocysticercosis, 2150, 2150f Neurodegenerative disorders, 209 Neurodermatitis, 2653f Neuroendocrine carcinoma large cell, in lung, 1313 low-grade (carcinoid-type) tumors, 1381 poorly differentiated, 1381 recommended treatment, 1380t of unknown primary origin, 1381 Neuroendocrine disease, 1475-1479 Neuroendocrine regulation, 1473-1475, 1586, 1586f Neuroendocrine system, 1473, 1474f Neuroendocrine tumors, 1557-1558 localization of, 1555 lung tumors, 1313 pancreatic, 1334-1339, 1335t autosomal dominant inherted disorders associated with, 1335 classification of, 1335 definition of, 1334-1335 epidemiology of, 1335 functional, 1335-1339, 1335t nonfunctional, 1335, 1335t, 1339, 1339b pathobiology of, 1335 well-differentiated, 1557-1558 Neuroenteric control, 884 Neurofibromas, 2689, 2689f Neurofibromatosis, 587, 1523, 2514-2515 stroke in, 2443-2444 type 1, 1523, 2514-2515, 2515f cancer susceptibility, 1227t diagnostic criteria for, 2515t inheritance of, 187t interstitial lung disease, 587 medical implications, 187t recommended surveillance for, 2515t type 2, 2515, 2515t cancer susceptibility, 1227t inheritance of, 187t medical implications, 187t Neurogenic hypertension, 387 Neurogenic shock, 2368-2369 Neurohemoinflammation, 2450 Neurohypophysis, 1494, 1495f Neuroimaging, 2345-2346, 2345t Neuroleptic malignant syndrome, 2468, 2520, 2628 diagnostic criteria for, 2628 differential diagnosis of, 2628t Neuroleptics, 2573t Neurologic agents. See also specific agents alternative medications, 1462t perioperative management of, 2613t Neurologic disease acute disorders requiring immediate diagnosis and treatment, 2339 alcohol-related disorders, 2510-2512, 2511f approach to, 2338-2346 autoimmune syndromes, 2518-2519, 2518t autonomic disorders, 2517-2522 chronic, 2012.e1 clinical manifestations of, 2338 congenital disorders, 2512-2514 developmental disorders, 2514 diagnosis of, 2338-2339 diagnostic challenges, 2339 diagnostic procedures, 2341-2346 high-penetrance monogenic disorders, 187t

Neurologic disease (Continued) hyperfunctional disorders, 2605-2606 hypofunctional disorders, 2606 in immunocompromised patients, 1859t, 1861 Lyme disease early, 2023 late, 2024 neurocutaneous disorders, 2514-2517 nutritional disorders, 2506-2512 in sarcoidosis, 606 throat disorders, 2605-2606, 2605t Neurologic examination, 2339, 2371 in coma, 2339, 2411, 2412t general examination, 2339 in peripheral neuropathy, 2528 Neurologic history, 2338-2339 Neurologic symptoms causes of, 2339 common complaints, 2339-2341 preoperative evaluation of, 2617 sensory symptoms, 2340-2341 Neurologic testing, 350, 2528. See also specific tests Neuromodulation, 141-142 Neuromuscular blockers, 2619-2620 Neuromuscular disorders, 629, 629t Neuromuscular irritability, 1658, 1658t Neuromuscular junction, 2547, 2548f Neuromuscular junction disease acetylcholine receptor loss, 2549, 2549f clinical features of, 2539t Neuromuscular transmission, 2547 disorders of, 2547-2553, 2548t Neuromyelitis optica, 2478-2479, 2575 Neuromyotonia, 2545 acquired, 2548t, 2552 autoimmune, 2519 clinical manifestations of, 2552 definition of, 2552 diagnosis of, 2552 epidemiology of, 2552 pathobiology of, 2552 prognosis for, 2552-2553 treatment of, 2552b-2553b Neuronal ceroid-lipofuscinoses, 1403 Neuronal migration disorders, 2512 Neuronal proliferation disorders, 2512 Neuro-ophthalmology, 2573-2579 Neuropathic pain, 133 characterization of, 134-135, 135t chronic, 137t definition of, 133 pharmacotherapy for, 137t prevalence of, 134t Neuropathy associated with cancer, 2533 associated with monoclonal gammopathy, 2531-2532, 2532b diabetic lumbosacral radiculoplexus, 2535 Neurophysiology, 2528 Neuropsychiatric disorders associated with SLE, 1773 in older adults, 114-115 diagnosis of, 115-116 treatment of, 116b-117b vitamin B12 and, 1454-1455 Neuropsychiatric lupus, 1773 Neuroregulin 1, 304.e1t Neurosarcoidosis, 606, 606t Neurosyphilis, 2016, 2018 Neurotoxic cobras (Naja haje), 719 Neurotoxic disorders clostridial infections, 1927-1930 of neuromuscular transmission, 2548t Neurovascular unit, 2431 Neutral endopeptidase, 2192t Neutrons, 82, 83f Neutropenia, 1133-1138 autoimmune, secondary, 1135 benign familial, 1133 causes of primary, 1133-1134 secondary, 1134-1136 chemotherapy-induced, 1856, 1856t chronic idiopathic, 1136 clinical manifestations of, 1136 congenital syndromes, 1133-1134, 1133t, 1683, 1682.e1t constitutional, 1133 cyclic, 1133t, 1134, 1682.e1t diagnosis of, 1136

Neutropenia (Continued) diagnostic approach to, 1136, 1137f differential diagnosis of, 1133, 1133t drug-induced, 1135, 1135f, 2626t, 2627-2628 due to bone marrow injury, 1135 due to increased margination and hypersplenism, 1136 due to nutritional deficiency, 1136 ethnic, 1133 febrile diagnostic considerations, 1856-1857 evidence-based guidelines for, 1858 management of, 1857-1858, 1857f P. aeruginosa infections during, 1964 immune, 1135-1136 infection-related, 1134-1135 prophylaxis in, 2068-2069 treatment of, 1136b-1138b Neutropenic colitis, 948, 948.e1f Neutropenic enterocolitis, 1860-1861 Neutrophil actin dysfunction, 1147t-1148t Neutrophil elastase (ELA2, ELANE), 1133-1134 Neutrophil extracellular traps, 1146 Neutrophil gelatinase-associated lipocalin, 304.e1t Neutrophilia, 1129-1132 chronic idiopathic, 1131 clinical manifestations of, 1131-1132 diagnosis of, 1131-1132 diagnostic approach to, 1131-1132, 1132f differential diagnosis of, 1129, 1129t drug-induced, 1130 hereditary, 1131 post-splenectomy, 1131 primary causes of, 1131 secondary causes of, 1130-1131 turnout in elections, 1130-1131 Neutrophilic chemotaxis defects, 1147t-1148t, 1148 Neutrophilic degranulation disorders, 1147t-1148t, 1148-1149 Neutrophilic dermatosis, febrile, 1859, 2682, 2692 Neutrophilic drug reactions, 2686 Neutrophilic granules, 1144, 1144t Neutrophilic leukemia, chronic, 1131 Neutrophilic meningitis, chronic persistent, 2494 Neutrophilic pleocytosis: chronic meningitis with, 2494, 2494t Neutrophils, 216-217, 1143-1146 dysfunction of, 1677t functions of, 1142-1143, 1143t giant, 1058 hypersegmented, 1057f, 1136f life cycle of, 1143, 1143f normal dynamics, 1129, 1130f production of, 1050, 1051f pseudo-Pelger, 1057 Nevirapine, 2288t, 2289, 2331t Nevus (nevi) benign blue nevus, 2690f blue, 2689 common acquired (moles), 1374, 1374f, 1375t dysplastic, 1374, 1374f, 2689, 2690f clinical features of, 1375t surveillance of, 1377 junctional, 2689, 2689f Nevus anemicus, 2699 Nevus depigmentosus, 2699 New Delhi metallo-β-lactamase (NDM), 1961 New Jersey Supreme Court, 6 New World viruses, 2247 New York Heart Association, 251t Newborns. See Neonates Newcastle disease virus, 212 Nexavar (sorafenib), 170, 1211t-1216t Nexplanon, 1609 Next-generation sequencing, 195, 201, 1387-1388 NEXUS Low-Risk Criteria, 2373, 2373f Nezelof ’s syndrome, 2213 Niacin (vitamin B3) dose-response effects, 1008t functions of, 1446t-1450t, 1454t for lipid disorders, 1396t Niacin (vitamin B3) deficiency, 2507t

Nicardipine for hypertension, 388t, 395-396, 396t properties of, 430t Niclosamide, 2148, 2148t Nicotinamide, 2507t Nicotine, 144t, 145-149 Nicotine gum, 147-148, 147t Nicotine inhaler, 147t, 148 Nicotine lozenges, 147t Nicotine nasal spray, 147t, 148 Nicotine patch, 147t, 148 Nicotine replacement therapy, 147-148, 147t Nicotine vaccines, 147t Nicotinic acid for lipid disorders, 1396, 1396t for niacin deficiency, 2507t Niemann-Pick disease, 587, 1387, 1403, 2569 Niemann-Pick disease A, 1385t Nifedipine for altitude sickness, 597t for atheromatous embolization, 507 for diffuse esophageal spasm, 904 for high-altitude pulmonary edema prevention, 596-597 for hyperadrenergic disorders, 2521-2522 for hypertension, 388t, 394, 394t, 1615t for kidney stones, 813-814 for livedo reticularis, 506 for pernio, 510b properties of, 430t for pulmonary hypertension, 402 for Raynaud’s phenomenon, 509f for reversible cerebral vasoconstriction syndrome, 1800 for sphincter of Oddi dysfunction, 1048 Nifurtimox (Lampit), 2118 Night blindness, 2509, 2559 Night sweats, 1221 Nightmares, 2423, 2423t Nilotinib (Tasigna), 171, 1232t for cancer, 1211t-1216t for chronic myelogenous leukemia, 1248, 1249t, 1250 Nilutamide, 1369 Nimodipine, 2448t, 2462 Nipah virus, 2056-2057, 2058t Nisoldipine, 388t, 430t Nitazoxanide, 2105 for cryptosporidiosis, 2135 for H. pylori eradication, 915t for rotaviral diarrhea, 2246 for tapeworms, 2148t, 2149 Nitrates, 431t for acute heart failure, 318 for acute STEMI, 450 to reduce angina and ischemia, 429 Nitrendipine, 430t Nitric oxide, 549-551 Nitroblue tetrazolium test, 1146 Nitrofurantoin, 1891t-1892t for cystitis, 1875t for enterococcal infection, 1915.e1t for UTI prophylaxis, 1876t Nitrogen, reactive, 233 Nitrogen washout technique, 542 Nitroglycerin, 431t for acute coronary syndrome, 436-437, 438t-439t for acute MI, 450, 454t, 455 for anal fissures, 970 for esophageal spasm, 904 for hyperadrenergic disorders, 2521-2522 for hypertension, 394, 396t Nitroimidazole tinidazole, 1013 Nitroprusside for aortic dissection, 495-496 for hypertensive emergency, 396t for mitral valve regurgitation, 453 for ventricular septal defect, 453 Nitrous oxide, 2619 Nivolumab, 1376 Nizatidine, 900t, 2660t Nocardia, 2062 culture of, 2064 that cause mycetoma, 2099 Nocardiosis, 2062-2064 central nervous system infection, 2063, 2063f clinical manifestations of, 2063 cutaneous infection, 2063 definition of, 2062 diagnosis of, 2063-2064

Index Nocardiosis (Continued) epidemiology of, 2062-2063 etiology of, 2062 histopathology of, 2064 pathobiology of, 2063 prognosis for, 2064 pulmonary, 2063, 2063f radiographic findings, 2063-2064, 2063f treatment of, 2064b Nociceptive pain, 133-135, 134t-135t Nocturnal dyspnea, paroxysmal, 302 Nocturnal events, 2423t Nocturnal hemoglobinuria, paroxysmal, 1073, 1078-1079, 1116, 1119 Nocturnal hypertension, 384, 384.e1f Nocturnal hypoglycemia, 1539 Nocturnal hypoventilation, 629-630 Nocturnal positive-pressure ventilation, 629, 629t Nodal marginal zone B-cell lymphoma, 1261t, 1265 Nod-like receptor (NLRs), 231 Nodular gastritis, 908 Nodular heterotopia, 2512 Nodular melanoma, 1374 Nodular regenerative hyperplasia, hepatic, 981 Nodular vasculitis, 2693 Nodules, 2636t, 2647t-2651t benign, 2687-2693 inflammatory, 2691-2692 Lisch, 1523 rheumatoid, 1758-1759, 1758f-1759f of skin, 2687, 2687t Noise-induced hearing loss, 2595 Nolvadex (tamoxifen), 1211t-1216t Nonexercise activity thermogenesis (NEAT), 1461 Nonfluency, 2384 Noninsulinoma pancreatogenous hypoglycemia syndrome, 1552 Noninvasive positive-pressure ventilation (NPPV), 629-630, 666 Nonmaleficence, 4 Non-nucleoside reverse transcriptase inhibitors (NNRTIs), 2288t, 2289 CSF-to-plasma ratios, 2331t initial regimens, 2289, 2290t side effects and toxicity of, 2291 Nonselective reuptake inhibitors, 893t, 894 Nonsense mediated decay, 194 Nonsense mutations, 194 Nonsteroidal anti-inflammatory drugs (NSAIDs) assessment of use, 913 complications of, 175, 174.e1f COX inhibition by, 173-174, 173f-174f drug interactions, 175 and gastroduodenal ulcers, 909-910 and micronutrient status, 1453t and peptic ulcers, 915 for rheumatoid arthritis, 1760 sensitivity to, 1699 and small intestinal ulcers, 947-948, 948f for spondyloarthritis, 1764 topical, 2658-2659 traditional, 174-175 in vascular events, 175 Noonan’s syndrome, 405, 1227t Norelgestromin, 1609 Norepinephrine for pulmonary embolism, 623-624 for shock, 676t, 680, 683, 689-690, 2369 for traumatic brain injury, 2368 for vasospasm, 2449 Norethindrone acetate and ethinyl estradiol (Estrostep), 2660 Norfloxacin for cystitis, 1875t for peritonitis, 1030 for urinary tract infections, 1876t for varices and varical bleeding, 1029 Norgestimate and ethinyl estradiol (Ortho Tri-Cyclen), 2660 Normal or Gaussian distribution, 32 Normotension, 381 Norovirus infection clinical features of, 2245-2246, 2245t diagnosis of, 2246 diagnostic criteria for, 2245-2246

Norovirus infection (Continued) epidemiology of, 2244-2245, 2245t pathobiology of, 2245 prevention of, 2246 treatment of, 2246b Noroviruses, 2244 Norpramin (desipramine), 893t North America, 100 North American blastomycosis, 2074 North American snakes, 719 Nortriptyline for depression, 2349t for distal symmerical polyneuropathy, 2536 for erythromelalgia, 508 for headache prevention, 2358t, 2360 for irritable bowel syndrome, 894 for pain, 138t for painful dysesthesias and paroxysmal dystonic spasms, 2477 for tobacco dependence, 147t for zoster, 2229 Nose. See also under Nasal in acute poisoning, 697 examination of, 26 inflammation of. See Rhinitis inspection of, 2586, 2587f Nose disorders, 2585-2592 Nosocomial hospital diarrhea, 920 Nosocomial infection evaluation of, 1854 initial management of, 1854 risk factors for, 1433 Notechis (tiger snakes), 720 Novantrone (mitoxantrone), 2477 NS1 protein, 2192t NS2 protein, 2192t Nuclear cardiology, 285-287 in coronary artery disease, 425 in heart failure, 305 Nuclear factor κ light-chain enhancer of activated B cells (NF-κB), 219 Nuclear factor κB (NF-κB) essential modulator (NEMO), 1686-1687 Nuclear medicine, 872 Nucleic acid amplification tests (NAATs), 1941, 2037 Nucleoprotein, 2192t Nucleoside reverse transcriptase inhibitors (NRTIs), 2288t, 2289 CSF-to-plasma ratios, 2331t initial regimens, 2289 side effects of, 2291 Nucleosome, 1721 Nucleotide metabolism disorders, 1086 Nucleotide-binding oligomerization domain (NOD)-like receptors, 217 Nuisance diarrhea, 1925 Null hypothesis, 35 Nulojix (belatacept), 171 Number needed to treat (NNT), 33 Numbness, 2338 Numerical variables, 32 Nummular dermatitis, 2651f, 2662, 2662f NURSE mnemonic, 14t Nursing Home Compare (CMS), 43.e1t Nursing precautions, 2255 Nutcracker esophagus, 904 Nutric Score, 1438, 1438.e1t Nutrient deficieny, 1441t, 2510 Nutrition, 1426-1430 conditions that impair delivery to systemic circulation, 932-933 diseases influenced by, 1426-1430, 1427t and morbidity, 1426-1430 and mortality, 1426-1430 in spinal cord injury, 2369 Nutrition Risk Score (NRS), 1438, 1438.e1t Nutrition science, 1426 advances in, 1453-1455 micronutrients in, 1445 Nutritional assessment, 1430-1434, 1440-1442 clinical, 1430-1433 composite screening tools, 1433 comprehensive, 1440.e3t geriatric, 105 goals of, 1430 physical examination, 1431-1432 scoring systems, 1438.e1t

Nutritional deficiency in HIV infection, 2302-2303 neurologic disorders, 2510 neutropenia due to, 1136 signs and symptoms of, 1431, 1432t that leads to autonomic dysfunction, 2520 Nutritional optic atrophy, 2575 Nutritional requirements, 1453-1455 Nutritional Risk Index, 1433 Nutritional support, 1431, 1442-1444 for cancer, 1218-1219 enteral, 1437-1440 administration of, 1442 indications for, 1442t factors that determine, 1431 for heart failure, 315 for inborn errors of metabolism, 1388-1389 indications for, 1442t oral, 1442t parenteral administration of, 1442-1444 complications of, 1444, 1444t contraindications to, 1443 indications for, 1442t, 1443 iron repletion therapy, 1070 solutions, 1443, 1443t with renal replacement therapy, 842 Nystagmography, 2600 Nystagmus, 2579 Bruns’, 2579 downbeat, 2579 gaze-evoked, 2579 rebound, 2579 spontaneous, 2579, 2579t Nystatin, 2297t-2301t, 2582

O

Obesity, 631, 1429-1430, 1458-1466 in adolescents, 64 cancer and, 1218 cardiomyopathy and, 330 cardiovascular disease and, 259 causes of, 1461-1462 class II, 1458 classification of, 1458t constitutional effects on, 1459 definition of, 544-545, 1431, 1458 diagnosis of, 1463 endocrine manifestations of, 1463 environmental factors, 1459-1460, 1459t epidemiology of, 1458 etiology of, 1458 evaluation of, 1463, 1464f extreme, 1458 genetic aspects of, 1459 gut microbiota and, 1842 hypothalamic, 1479 kidney stones and, 811 mechanical complications of, 1463 medical complications of, 1462-1463 in older adults, 100, 105 pathobiology of, 631, 1458-1462 pathophysiology of, 1462-1463 prevalence of, 1458 prevention of, 1466 psychosocial aspects, 1462 pulmonary function in, 544-545 secondary causes of, 1461-1462 skin disease and, 2646t treatment of, 1464b-1466b, 1464f medical therapy, 1466 pharmacotherapy, 1465-1466, 1465t type 2 diabetes and, 1534 in women, 1602-1603 The Obesity Society (TOS), 1458, 1464-1465 Obinituzumab, 1263 Object agnosia, 2385 Observation, 35, 303 Observational studies, 42 Obsessive-compulsive disorder, 2352-2353, 2352t, 2355t Obstetric cholestasis, 1622 Obstetric infections anaerobic, 1932 H. influenzae, 1947 Obstructed breathing, 639

I73

Obstruction in Crohn’s disease, 942 pancreatic, 960 Obstructive airway disease, 669-670 Obstructive apnea, 638, 2419 Obstructive hypopnea, 638, 638.e1f Obstructive jaundice, 991t Obstructive lung disorders, 542, 543t, 544. See also Chronic obstructive pulmonary disease Obstructive nephropathy, 799 Obstructive shock definition of, 678f-679f etiology of, 678f-679f extracardiac, 674 rapid ultrasound protocol, 675, 675t therapy for, 678f-679f Obstructive sleep apnea, 638-642, 638f clinical manifestations of, 639 definition of, 638 diagnosis of, 385t, 639 epidemiology of, 638 general measures for, 641 genetics of, 638 in obesity, 1463 pathobiology of, 638-639 pathogenesis of, 638-639 polysomnographic findings, 638f, 639, 640f prognosis for, 642 severe, 640f STOP-Bang Questionnaire for, 2419, 2419t symptoms and signs of, 639 treatment of, 639b-642b mechanical therapy, 640-641 medical therapy, 641 nasal, 641 oxygen therapy, 641 surgical procedures, 642 ways to relieve or bypass obstruction, 641 upper airway abnormalities associated with, 639 Obstructive uropathy, 799-803 clinical manifestations of, 800-801 CT findings, 802, 803f definition of, 799 diagnosis of, 801-802 diagnostic tests, 801t, 802, 802f-803f differential diagnosis of, 802 epidemiology of, 799 laboratory findings, 801 laboratory tests for, 802 lower urinary tract symptoms of, 803 pathobiology of, 799-800 pathology of, 800 pathophysiology of, 800 physical findings, 801 prognosis for, 803 symptoms of, 801 treatment of, 802b-803b ultrasound findings, 802, 802f Obturator sign, 858t Occipital cortex, 2575 Occupational asthma sensitizer-induced, 588-590 suggestive features, 589t Occupational exposures and cancer, 1225 and non-Hodgkin’s lymphoma, 1258 and systemic sclerosis, 1778-1779 Occupational health disorders, 80-81, 80t chronic obstructive pulmonary disease, 590 lung cancer, 589t, 595 lung disease, 588-595 epidemiology of, 588 interstitial, 586 principles of, 79-80 respiratory examples, 589t rheumatic disease, 1717 Occupational history, 24-25, 79-80, 2338, 79.e1f Occupational medicine, 78-82 Ochronosis, 1827 Ockelbo disease, 2262 Ocrelizumab, 171-172 Ocriplasmin, 2567 Octapharma, 1170 Volume I  pp 1-1382  •  Volume II  pp 1383-2722

I74

Index

Octaplas (Octapharma), 1179 Octreotide (Sandostatin) for bacterial overgrowth, 928 for cancer, 1211t-1216t for diarrhea, 922, 932, 935 for drug overdose, 706t-710t dumping syndromes, 888 for glucagonoma, 1337 for hepatorenal syndrome, 1030 for hyperinsulinemic hypoglycemia, 1555 for kidney disease, 820 for metastatic Zollinger-Ellison syndrome, 1337 for short-bowel syndrome, 932 for small bowel rapid transit dysmotility, 888 for varical bleeding, 1029 for VIPoma, 1338b Ocular bobbing, 2579 Ocular dipping, 2412 Ocular dysmetria, 2579 Ocular flutter, 2579 Ocular function: evaluation of, 2557-2558 Ocular infections, adenoviral, 2198 Ocular irritation, 2562, 2562f Ocular metastasis, 2572 Ocular misalignment, 2577-2578 Ocular myasthenia gravis, 2550-2551 Ocular myoclonus, 2579 Ocular onchocerciasis, 2169 Ocular oscillations, 2579 Ocular surface abnormalities, 2562-2563 Ocular symptoms, 2568-2569 Ocular toxoplasmosis, 2127, 2128f, 2130, 2132 Ocular trauma accidental, 2563 major, 2563 Oculoglandular disease, 1983 Oculomotor control, 2577-2579 Oculomotor control abnormalities, 2577-2579 Oculomotor nerve lesions, 2576, 2576f Oculomotor pathways anatomy of, 2577 internuclear, 2577 lesions in, 2577 nuclear, 2577 supranuclear, 2577, 2577t Oculopharyngeal muscular dystrophy, 2544 Odanacatib, 1644 Odds definition of, 37t formula for, 37 post-test, 38 pretest (prior), 37t and probability, 37 Odds ratio (OR), 33-34 Odynophagia, 852t, 2303t Oesophagostomum bifurcum, 2164 Ofatumumab (Arzerra), 171-172, 1211t-1216t, 1256 Off-label medications, 149 Ofloxacin for bacterial meningitis prophylaxis, 2489 for chlamydia, 2011 for eye infections, 2565t for otitis externa, 1967 for prostatitis, 831-832 for urethritis, 1878t Ogilvie’s syndrome, 877 Ohio, 7 Olanzapine, 2354 Old Order Amish, 1685 Old World vipers, 717 Old World viruses, 2247 Older adults activity limitations in, 101f acute mental status change in, 117-121 acute myeloid leukemia in, 1245 adrenal androgen deficiency in, 1572 adverse drug events in, 116 anemia in, 1066 blood pressure treatment goals, 391, 392t cancer in, 1218 causes of death in, 100, 101t chronic conditions in, 100, 102f delirium in, 117-121 dietary requirements, 1452-1453 disability in, 101 drug use in, 129-130 frail, 109

Older adults (Continued) gait disorder of the elderly, 2341t global population, 100, 101t guidelines for, 60 health of, 100-102 Hodgkin’s lymphoma in, 1273 injured, 716, 716.e1t lung cancer in, 1312 mental status change in, 117, 118f mitral regurgitation in, asymptomatic, 469 morbidity in, 100 mortality of, 100 neuropsychiatric disorders in, 114-115 non-Hodgkin’s lymphoma in, 1267 obesity in, 100, 105 osteosarcoma in, 1370 pain management in, 142 parainfluenza virus disease in, 2189 patient goals, 103 pharmacotherapy for guidelines for, 117t principles of, 116-117 physiologic considerations, 716.e1t preventive health care for, 102 preventive services for, 105 prognosis for, 103 psychoactive effects of drugs in, 116 psychosocial considerations, 716.e1t subclinical disease in, 100 systolic hypertension in, 392, 392f testosterone deficiency in, 1571-1572 thyroxine for, 1504 vulnerability in, 109 weight loss in, 105 Olecranon bursitis, 1752 Olesoxime, 2526 Olfaction testing, 2593 Olfactogenital dysplasia, 1592 Olfactory dysfunction, 2593b Olfactory receptors, 2592 Olfactory seizures, 2401t Oligodendroglioma, 1293-1294 Ollier’s disease, 1672 Olmesartan, 388t Olsalazine (Dipentum), 939t Omacetaxine, 1249t, 1250 Omalizumab for asthma, 553 dosing calculators, 553 for urticaria, 1695, 2684 Omega-3 fatty acids, 314 for heart failure, 314, 306.e1t-306.e2t for lipid disorders, 1396 Omega-3-acid ethyl esters, 1396t Omental cysts, 950 Omental diseases, 950-951 Omeprazole for acid peptic disease, 915 for acute pericarditis, 486 for dyspepsia, 895 for esophageal disorders, 900t for functional chest pain or heartburn, 896 for H. pylori–associated ulcers, 914 and micronutrient status, 1453t for short-bowel syndrome, 932 for stress ulcer prevention, 2368-2369 for Zollinger-Ellison syndrome, 1336 Omnaris (ciclesonide), 1692 Omphalocele, 944 Omsk hemorrhagic fever, 2249t, 2263t, 2268 clinical manifestations of, 2250-2251, 2251t epidemiology of, 2250 management of, 2254-2255 pathobiology of, 2250, 2251t symptoms and signs of, 2250-2251 vaccine for, 2255-2256 Omsk hemorrhagic fever virus, 91t, 2249t OMV, 1939 OnabotulinumtoxinA (Botox), 830 Onbrez (indacaterol), 553 Onchocerca volvulus, 2159, 2166, 2167t Onchocerciasis, 2169-2170 clinical manifestations of, 2169 definition of, 2169 diagnosis of, 2169 epidemiology of, 2169 ocular, 2169 pathobiology of, 2169 prevention of, 2170 subcutaneous, 2169

Onchocerciasis (Continued) treatment of, 2104, 2165t, 2169b-2170b vectors, 2172t Onchodermatitis, 2169 Oncocytoma, 1346t Oncogene mutations, 1227, 1227t Oncogenes, 1227-1228 effects of, 1228-1229 in lung cancer, 1304-1306, 1305f miRNAs as, 1228 proto-oncogenes, 1227-1228 sites of action, 1229 Oncogenesis, insertional, 207 Oncogenic viruses, 1240 Oncology clinical trials in, 1208 in HIV infection, 2322-2328 Oncospheres, 2149-2150 Oncotype DX (Genomic Health Inc.), 203, 1233, 1354 Ondansetron for nausea and vomiting, 865t for renal colic, 813 for rotaviral diarrhea, 2246 for small bowel rapid transit dysmotility, 888 Ondine’s curse, 547 1000 Genomes project, 189-190 One-hand, two-feet syndrome, 2670 Ongoing Telmisartan Alone and in Combination with Ramipril Global Endpoint Trial (ONTARGET), 392, 392.e2f Online Mendelian Inheritance in Man (OMIM), 190, 196 Onycholysis, 2709, 2709f Onychomadesis, 2707, 2708t Onychomycosis, 2709 subungual, 2709, 2709f superficial, 2709, 2709f O’nyong-nyong fever, 2260-2261 clinical manifestations of, 2260 definition of, 2260 diagnosis of, 2260-2261 distinguishing characteristics of, 2252t-2253t epidemiology of, 2260 pathobiology of, 2260 prognosis for, 2261 treatment of, 2261b O’nyong-nyong fever virus, 2260 Oogenesis, 1579, 1579f Oophorectomy, prophylactic, 1358 Opacities, pulmonary, 532 large, 532 small, 532-535 Open-angle glaucoma, chronic, 2566, 2567f Operative risk assessment, 2611 Ophthalmic disorders associated with conjunctivitis, 2564t sympathetic ophthalmia, 2571 Ophthalmologic examination, 252 Ophthalmopathy, 1506, 1506f clinical manifestations of, 1506 definition of, 1506 treatment of, 1506b Ophthalmoplegia chronic progressive external, 2546 internuclear, 2473, 2578 Opioids, 158-159 abuse of, 158 clinical manifestations of, 158-159 epidemiology of, 158 factors associated with, 140f medical complications of, 159 pathobiology of, 158 treatment of, 159b-160b analgesics, 137 for anesthesia, 2619 dependence on epidemiology of, 158 longer-term treatment of, 159-160 physical, 159 endogenous peptides, 1475 formulations, dosages, and pharmacologic information, 139t-140t perioperative, 2613t properties of, 144t for pruritus, 2636t screening for, 699-702, 702t toxicity, 699t-702t, 706t-710t

Opioids (Continued) toxidromes, 696t withdrawal from, 159 Opisthorchiasis, 2157 clinical manifestations of, 2157 diagnosis of, 2158 epidemiology of, 2157 pathobiology of, 2157 Opisthorchis, 1015t-1017t, 2158 Opisthorchis felineus, 2157 Opisthorchis viverrini, 2157 Opium tincture, 932 Opportunistic infections, AIDS-associated acute, 2291 antiretroviral therapy for, 2291 Guideline on Management of Opportunistic Infections in Adults and Adolescents, 2294-2295 Guidelines for the Prevention and Treatment of Opportunistic Infections in HIV-Infected Adults and Adolescents, 2294 prophylaxis to prevent first episode of, 2295t treatment of, 2297t-2301t Opsoclonus, 2579 Optic atrophy inherited, 2575 nutritional, 2575 toxic, 2575 Optic chiasm lesions, 2575 Optic disc abnormalities, 26 Optic glioma, 1293 Optic nerve, 2556 Optic nerve disease, 2574-2575 Optic nerve lesions, 2576, 2576f Optic neuritis, 2478, 2574-2575 clinical manifestations of, 2478 diagnosis of, 2478 differential diagnosis of, 2575t treatment of, 2478b Optic neuropathy hereditary, Leber’s, 2546, 2575 ischemic, 2575 nonarteritic, 2570 Optic tract lesions, 2575 Optical coherence tomography, 2475-2476, 2476f OPTN gene, 1664-1665 Optochin, 1902 Oral allergy syndrome, 1699 Oral anticoagulants, 2616, 2617t Oral antihypertensive drugs, preferred, 394t Oral candidiasis, 2582-2583, 2582f Oral cavity. See also Mouth tumors in. See also Oral lesions clinical manifestations of, 1299 epidemiology of, 1297-1298 soft tissue tumors, 2583-2584, 2583t Oral contraceptives, 1604, 1607-1609, 2660 complications of, 1608 contraindications to, 1608 dose-response effects, 1008t failure rates, 1605t, 1606f hypertension associated with, 393-394 management of therapy, 1608-1609 for perimenstrual seizures, 2408 recommendations for perioperative management of, 2613t risk factors for, 1608 side effects of, 1607-1608 thrombosis with, 1190 Oral cromolyn sodium, 1709 Oral dissolution therapy, 1040-1041 Oral glucose suppression test, 1487t Oral hairy leukoplakia, 2234, 2319, 2644f Oral HPV infection, 2220 Oral hydration, 2521 Oral lesions, 2303t, 2304 associated with HIV infection, 2579, 2580t high-risk early lesions, 1299, 1299f red or blue lesions, 2581t white lesions, 2581-2582, 2581t Oral lichen planus, 2667 Oral mucocutaneous diseases, 2583 Oral mucosal diseases, 2579-2583 acute, 2579-2581 chronic, 2581 ulcers, 2580t viral, 2580-2581 Oral nutrition support, 1442t Oral pain, tardive, 2468

Index Oral pigmentations, 2583, 2583t Oral rehydration solutions for cholera, 1951t-1952t, 1952 for diarrhea, 921, 1870-1872, 1870t for norovirus and rotavirus disease, 2246 for orthostatic hypotension and syncope, 2521 recommendations for, 1952 Oral soft tissue tumors, 2583-2584, 2583t Oral squamous cell carcinoma, 2581, 2581f Oral temperature measurement of, 26 normal ranges, 1849 Oral-facial-digital syndrome, 817t, 822 OraQuick in-home HIV test, 2277 Orbit, 2556-2557 Orbital masses, 1797 Orbital pseudotumor, 2570 Orbital restrictive disease, 2577-2578 Orbital tumors, 2572 Orchitis mumps, 2207 testicular hypogonadism due to, 1573 Order sets, 44-45 Ordinal variables, 32 Oregon: physician-assisted suicide in, 7-8 Orencia (abatacept), 171, 1762 Organ transplantation. See also Transplantation; specific procedures in immunocompromised patients, 1855, 1855t for inborn errors of metabolism, 1389 Organic acidopathies, 1385t Organic acidurias, 1386, 1387t Organic dust toxic syndrome occupational causes, 594t suggestive features, 589t Organic solvent exposures, 81, 81t Organizational changes, 47 Organohalide exposures, 81, 81t Organophosphorus compounds antidote for, 706t-710t toxicity, 699t-702t, 706t-710t Organ-restricted eosinophilic disease, 336 Orgasm, impaired, 1576-1577 Oriental cholangiohepatitis, 1044-1045 Orientia, 2047t Orientia tsutsugamushi, 2052 Orientia tsutsugamushi infection, 2047-2052 Orlistat, 1465 ORMDL3 gene, 548-549 Ornithine transcarbamylase deficiency, 1386t Orolabial herpes, 2181t, 2297t-2301t Oromandibular dystonia, 2463-2464 Oropharyngeal anthrax, 1921-1922 Oropharyngeal cancer, 1298-1299, 2322, 2323t Oropharyngeal dysfunction, 903 Oropharyngeal infection C. trachomatis, 2010 candidiasis, 2079-2080, 2297t-2301t Oropharyngeal tularemia, 1983 Oropharynx, 2601, 2601f Orotic aciduria, 1385t Oroya fever, 1997-1998 clinical manifestations of, 1998 diagnosis of, 2000, 2000t pathobiology of, 1997-1998 prognosis for, 2001 treatment of, 2000 Ortho Evra, 1609 Ortho Tri-Cyclen (norgestimate and ethinyl estradiol), 2660 Orthopedic device-associated infections, 1900 Orthopedic surgery anesthesia in, 1829 indications for, 1828 preoperative considerations, 1828-1829 procedures, 1829-1832 Orthopnea, 302 Orthopoxvirus, 2214-2215, 2215t Orthopoxvirus infection clinical manifestations of, 2216-2218 diagnosis of, 2218 prevention of, 2218-2219 prognosis for, 2219 treatment of, 2218-2219 Orthostatic hypotension antipsychotic-induced, 2626t detection of, 392, 392f

Orthostatic intolerance, 2521, 2521t Ortner’s syndrome, 465-466 Osborn ( J) wave, 694, 694f Oscillopsia, 2473, 2599 Oseltamivir, 2183-2184 for acute bronchitis and tracheitis, 609 dose recommendations, 2196t for influenza, 2183t, 2196 mechanisms of excretion and thresholds for dose adjustment, 2184t for pneumonia, 616t Oseltamivir resistance, 2196-2197 Osler’s nodes, 255, 475, 475f, 475t Osler-Weber-Rendu disease, 1172 gastrointestinal involvement, 958 telangiectasia in, 405, 2694-2695 Osmolality arginine vasopressin and, 742, 743f calculated, 766-767 plasma, 749 regulation of, 1494-1495, 1495f Osmolar gap definition of, 766-767 increased, 766t Osmolarity, body fluid, 738-739 Osmoreceptor dysfunction, 1497, 1499 Osmotic demyelination syndrome, 752 Osmotic fragility, 1082, 1083f Osmotic regulation, 1496 Ospemifene, 1629 Osteoarthritis, 1744-1749 age-related changes and, 1747 biomarkers, 1748 biomechanics of, 1746 blood features, 1748 bone change in, 1745 cartilage change in, 1744-1745 classification of, 1713t clinical manifestations of, 1747 definition of, 1744 delay of progression of, 1749 diagnosis of, 1748 distribution of, 1757f epidemiology of, 1744 genetic components, 1732 genetic factors, 1746 imaging findings, 1748 joints affected, 1747, 1747f in knees, 1757, 1757f laboratory findings, 1748 ligamentous abnormalities in, 1746 low back pain in, 2375t lower extremity, 1747, 1747f meniscal abnormalities in, 1746 metabolic factors, 1746 muscle weakness in, 1746 neck pain in, 2374t pathobiology of, 1744-1747, 1828, 1828f pathogenic factors, 1746-1747 pathologic features of, 1744, 1745f pathophysiology of, 1745-1746, 1746f physical findings, 1748 prevention of, 1749 primary generalized, 1747 prognosis for, 1749 pseudo-osteoarthritis, 1815 radiographic features of, 1724f, 1724t spinal, 1747 symptom palliation, 1748-1749 symptoms of, 1747 synovial fluid findings, 1748 synovial involvement in, 1745-1746 tissues central to, 1744-1746 treatment of, 1748b-1749b biomechanically active approaches, 1749 disease-modifying osteoarthritis drugs for, 1749 intra-articular therapy, 1749 surgical approaches, 1749 upper extremity, 1747, 1747f in women, 1603 Osteoarthropathy, hypertrophic, 1221, 1221f, 1669, 1825, 1825f Osteoarticular candidiasis, 2081 Osteoarticular sporotrichosis, 2078 Osteochondrosis, 1667-1668 Osteoclasts, 1638, 1638f, 1664-1665, 1664f Osteocytes, 1638 Osteodystrophy, renal, 835

Osteogenesis imperfecta syndromes, 1736-1738, 1737t clinical manifestations of, 1737 definition of, 1736 diagnosis of, 1737-1738 differential diagnosis of, 1737-1738 epidemiology of, 1736-1737 pathobiology of, 1737 pathogenesis of, 1737 pathology of, 1737 treatment of, 1738b Osteoid-osteocyte cells, 1638 Osteolysis, familial expansile, 1665t Osteomalacia, 1446t-1450t, 1645-1649 causes of, 1645, 1645t clinical manifestations of, 1646 definition of, 1645 diagnosis of, 1646-1648, 1647f-1648f due to hypophosphatemia, 1649 due to vitamin D disorders, 1648-1649 epidemiology of, 1645 pathobiology of, 1646 prevention of, 1649 prognosis for, 1649 radiographic findings, 1646, 1647f treatment of, 1648b-1649b vitamin D preparations for, 1648t Osteomyelitis, 1809-1810 Acinetobacter, 1969 acute, 1806 anaerobic, 1933 chronic, 1806, 1967 chronic recurrent multifocal, 1740t clinical manifestations of, 1810 definition of, 1809 diagnosis of, 1810 epidemiology of, 1809 microorganisms responsible for, 1806 pathobiology of, 1809-1810 prevention of, 1810 prognosis for, 1810 Pseudomonas, 1967 S. aureus, 1899, 1899f treatment of, 1810b antimicrobial therapy, 1810, 1810t duration of therapy, 1896 vertebral, 2035 Osteonecrosis, 1667-1668 causes of, 1668t clinical manifestations of, 1668 definition of, 1667 diagnosis of, 1668 of jaw, 1643, 1667-1668, 1668f oral ulcers, 2580t pathobiology of, 1667-1668 pathogenesis of, 1667-1668 in sickle cell anemia, 1101, 1103 steroid-induced, 165 treatment of, 1103, 1668b Osteopathia striata, 1671 Osteopenia definition of, 1637 laboratory work-up, 1636t Osteopetrosis, 1670, 1670b, 1670f Osteopoikilosis, 1671 Osteoporosis, 1637-1645, 1637f classification of, 1713t clinical manifestations of, 1640-1641 definition of, 1637 diagnosis of, 1641-1642 epidemiology of, 1637-1638 glucocorticoid-induced, 1644 laboratory work-up, 1636, 1636t male, 1644 nutritional influences, 1429 pathobiology of, 1638-1640 prevention of, 1593 prognosis for, 1644-1645 screening for, 56 steroid-induced, 164, 165f treatment of considerations, 1644 currently available therapies, 1644 emerging therapies, 1644 in women, 1601-1602 Osteoporotic fractures, 1640-1641 radiologic findings, 1641, 1641f reduction of, 1643t risk of, 1637, 1638f Osteoprotogerin, 1638, 1638f

I75

Osteosarcoma, 1370, 1370b Osteosclerosis, 1668-1669 definition of, 1668 disorders that cause, 1668t focal, 1671 hepatitis C–associated, 1671 with hyperostosis, 1670-1671 Osteosclerotic myeloma, 1281-1282 Osteotomy, 1829 Otalgia, 2590t Otic barotrauma, 598 Otitis externa, 1967, 2591, 2591f Otitis media differential diagnosis of, 2594 with effusion, 2590, 2590f group A streptococci (GAS), 1907 H. influenzae, 1947-1948 H. influenzae type B, 1947 Otomastoiditis, bacterial, 2599 Otomycosis, Aspergillus, 2084 Otosclerosis, 2594 Outcome variables, 33 continuous, 34 dichotomous, 33-34, 33.e1t OVAI (Vermillion, Inc.), 203 Oval macrocytes, 1053f Ovarian cancer, 187t, 1363-1364 clinical manifestations of, 1363 diagnosis of, 1363 epidemiology of, 1363 hereditary, 1227t management of survivors, 1364 pathobiology of, 1363 prevention of, 1363-1364 prognosis for, 1364 screening for, 1363-1364 staging, 1364t treatment of, 1363b treatment plans, 1364f Ovarian failure premature, 1590 presumptive, 1589-1591 Ovarian hyperstimulation, controlled, 1596 Ovarian hyperstimulation syndrome, 1596 Ovarian insufficiency, primary, 1589-1591 Ovary (ovaries), 1579-1584 during aging, 1579, 1580f anatomy of, 1580, 1580f controlled hyperstimulation, 1596 cyclic changes in, 1585 definition of, 1579 development of, 1579, 1580f embryology of, 1579 function of, 1580-1581 neuroendocrine regulation of, 1586, 1586f organogenesis of, 1579, 1579f physiology of, 1579-1581 polycystic ovarian–like disorders, 1589 polycystic ovary syndrome, 1584 Overactive bladder, 111 Overdiagnosis, 37, 54 Overhydrated hereditary stomatocytosis, 1084 Overnight dexamethasone suppression test, 1487t Over-the-counter drugs, 117t Overuse injuries, 1750 Overuse syndromes, 1749 Overweight. See also Obesity classification of, 1458t definition of, 544-545 OVID search system, 24.e1t Ovotesticular disorder of sexual development, 1566 Ovulation, 1586 Oxacillin for bacterial meningitis, 2488t for osteomyelitis or septic arthritis, 1810t recommended doses and schedules, 1891t-1892t for staphylococcal scalded skin syndrome, 2679 Oxalate, urinary, 811-812 Oxalate crystals, 732f Oxaliplatin (Eloxatin), 1211t-1216t, 1330-1331 Oxamniquine, 2155 Oxantel pamoate, 2160t, 2162 Oxazepam, 155t, 2352t Volume I  pp 1-1382  •  Volume II  pp 1383-2722

I76

Index

Oxcarbazepine, 2408t for chronic pain, 137 for seizures, 2407t-2408t for trigeminal neuralgia, 2363 Oxeol (bambuterol), 553 Oxidative phosphorylation, mitochondrial, 2545-2546 Oximetry in COPD, 558 pulse oximetry, 653 5-Oxoprolinuria, 769 Oxybutynin for benign prostatic hyperplasia, 830t for bladder urgency, 2477 for urinary incontinence, 113t, 2238-2239 Oxycodone formulations, dosages, and pharmacologic information, 139t-140t for renal colic, 813 for restless legs syndrome, 2468-2469 toxicity, 699t-702t for venomous fish and stingray injuries, 721b Oxygen alveolar-arterial oxygen gradient [P(A-a) o2], 652-653, 653t arterial content (Cao2), 654 hemoglobin binding, 1060 myocardial requirements, 442t partial pressure of (Pao2), 652 mechanisms that lead to reduction of, 656 normal values, 652 in pregnancy, 1619 physiologic oxygen supply dependency, 672 reactive, 233 venous content (Cvo2), 654 Oxygen consumption (Vo2), 687t Oxygen consumption index (Vo2I), 687t Oxygen delivery (Do2) normal value, 687t systemic, 654 Oxygen delivery index (Do2I), 687t Oxygen extraction ratio, 687t Oxygen saturation, decreased, 31 Oxygen supply, physiologic, 672 Oxygen therapy for acute respiratory failure, 660-661 blow-by, 2190 for carbon monoxide poisoning, 599 for chronic obstructive pulmonary disease, 559-560, 661 for COPD exacerbations, 562 for croup, 2190 for decreased oxygen saturation, 31 for hypoventilation, 547 for myocardial infarction, 448 for obstructive sleep apnea, 641 Oxygen toxicity, 600 clinical manifestations of, 600 diagnosis of, 600 pathobiology of, 600 prognosis for, 600 treatment of, 600b Oxygen transport anemia and, 1060-1061, 1060f Fick equation for, 1060f systemic, 654 Oxygenation arterial, 657, 658f mechanical therapy to improve, 660 for viral hemorrhagic fever, 2254 Oxygen-binding curves, 1060f Oxyhemoglobin association-dissociation curve, 655, 656f Oxymetazoline, 2186t, 2586 Oxytocin, 1496 Oxyuranus (taipans), 720

P

P values, 35-36 P wave, 267-269, 268t p53, 1229, 1229f Pacemakers, 374-376 batteries for, 375, 375.e1f complications of, 376 mode-switching, 376, 376f pacing modes, 376 permanent, 375-376 indications for, 375-376, 375t lead placement, 375, 375f site of implantation, 375.e1f

Pacemakers (Continued) rate-responsive, 376, 376f temporary, 374-375 wandering, 359, 359f Pachydermoperiostosis, 1669, 1669b Pacific Islanders, 15, 2274-2275 chronic kidney disease in, 833-834 population, 15 quality of health care, 15 tularemia in, 1982 type 2 diabetes mellitus in, 1532 Pacing modes, 376 Paclitaxel (Taxol) for cancer, 1211t-1216t, 1358, 1366 for Kaposi’s sarcoma, 2325 Paclitaxel protein-bound particles (Abraxane), 1211t-1216t Paget’s disease of bone, 1664-1667 arthritic manifestations of, 1825 clinical manifestations of, 1665, 1666f definition of, 1664 diagnosis of, 1665 drug treatment of, 1667 early-onset familial, 1665t epidemiology of, 1664 genes that predispose to, 1664-1665, 1665t histologic features of, 1665, 1666f incidence of, 1664 juvenile, 1665t monitoring, 1667 pathobiology of, 1664-1665 pathogenesis of, 1664-1665, 1664f prognosis for, 1667 radiographic features of, 1665, 1666f treatment of, 1667t monitoring, 1667 nonpharmacologic, 1667 surgical, 1667 Paget’s disease–like syndromes, 1664-1665, 1665t Pagophagia, 1069 Pain, 133-143 abdominal, 854-859 acute, 854-858, 855t-858t, 859f approach to, 852t, 979t chronic, 855t-857t, 859, 860f in chronic pancreatitis, 965 differential diagnosis of, 857f in HIV infection, 2303t location of, 854, 857f acute, 133 anorectal, 2303t back pain, 142, 2372-2375 biopsychosocial model of, 135-136, 136f chest pain, 248 causes of, 249t functional, of presumed esophageal origin, 895-896 chronic, 133, 141, 141t chronic neuropathic, 137t classification of states, 133, 134t definition of, 133 diagnosis of, 134-135 diagnostic tests, 135 dysfunctional, 133 ear pain, 2589-2592 epigastric pain syndrome, 894, 894t eye pain, 2558, 2559t in fibromyalgia, 1818-1819 genetics of, 134 genital, tardive, 2468 headaches and other head pain, 2356-2364 history of, 134-135 in HIV infection, 2303t with hypermobility, 1713t low back pain, 142t, 2375 mechanisms of, 133-134 medications for, 893t mixed, 133, 134t modulation of, 133 muscle, 2340, 2539 myofascial, 1817, 1822-1823 regional syndromes, 1713t treatment of, 142t neck, 2372-2375 neuroanatomy of, 1817, 1817f neuropathic, 133-135, 135t nociceptive, 133-135, 134t-135t oral, tardive, 2468 pathobiology of, 133-134 pathologic, 133 patterns of onset, 1715-1716

Pain (Continued) perceived, 2372 peripheral, 134t physical examination of, 135 prevalence of, 134t radicular, 2372 referred, 2372-2373 regional, complex, 2522 rheumatic, 1713t, 1717, 1749 in skin disease, 2646t somatic, 133, 134t spine acute, 2374-2375 red flags, 2373, 2373t tardive, 2468 thyroid, 1509 visceral, 133, 134t, 890 Pain management, 11t, 135b-142b in cancer, 1219 combination therapy, 137-138, 141f complementary and alternative medical (CAM) therapy, 142 in fibromyalgia, 1818-1819 interventional therapy, 141-142 in older adults, 142 in osteoarthritis, 1748 palliative care, 11t pharmacologic therapy, 136-138 future drugs, 142-143 preferred medications for women, 1602t preventive, 142-143 psychological treatment, 138-141 in renal colic, 813 in sickle cell disease, 1102-1103 Pain syndromes diagnostic clues, 1715t patterns of onset, 1715-1716 Painful episodes, 1099, 1102-1103 Painful hunger, 912 Painful legs and moving toes, 2469 Painless (lymphocytic) thyroiditis, 1509 PAIR (percutaneous aspiration, injection of chemicals, and reaspiration), 2104, 2152 Palifermin, 85 Paliperidone, 2354 Palivizumab, 2188 Palliative care, 9-10, 651-652 domains of, 10-15 for heart failure, 319-320 model for discussing topics, 14t questions to address, 10 surgical shunts for congenital heart lesions, 405t, 406 Palmar erythema, 977-978, 978f Palmar fasciitis and arthritis syndrome, 1826 Palmar rash, 1851, 1851t Palo Alto Medical Foundation Toxoplasma Serology Laboratory (PAMF-TSL), 2128-2130 Palonosetron, 865t Palpable purpura, 2674-2675, 2674f Palpation cardiac, 253 of skin, 2637 Palpitations, 248-251, 344-345 arrhythmic causes of, 350t-351t definition of, 344 evaluation of, 344f guidelines for diagnostic tests in patients with, 251t Palsy Bell’s, 2537 gaze, 2577 pseudobulbar, 2605-2606 sixth nerve, 2577-2578 supranuclear, progressive, 2606 third nerve, 2577-2578 trochlear nerve, 2577-2578 Pamaquine, 1088t Pamidronate, 1281, 1654, 1667t Panbronchiolitis, diffuse, 1965 Pancreas alcohol-related complications, 151t, 152 obstruction of, 960, 960t transplantation of, 1532 Pancreatic ascites, 966, 966f Pancreatic cancer, 1332-1334 Pancreatic cholera, 1335t, 1337, 1553 Pancreatic disease nonmalignant, 878 pleural fluid characteristics, 634t

Pancreatic ductal adenocarcinoma, 1332 adjuvant therapy for, 1333, 1333t advanced, 1334 biomarkers, 1333 clinical manifestations of, 1332-1333 diagnosis of, 1333 differential diagnosis of, 1333 epidemiology of, 1332 genetic progression of, 1332, 1333f imaging of, 1333 incidence of, 1332 laboratory findings, 1333 metastatic disease, 1334, 1334t pathobiology of, 1332 precursor lesions, 1332 prognosis for, 1334 resectable, 1333-1334 screening for, 1333 supportive care for, 1334 survival rate, 1334 treatment of, 1333b-1334b Pancreatic function tests, 925t Pancreatic insufficiency, 565 Pancreatic intraductal papillary mucinous neoplasms, 871f Pancreatic islet cell disorders, 1548-1555 Pancreatic islet cell failure, 1462 Pancreatic islet cell transplantation, 1532 Pancreatic islet cell tumors, 1334-1335, 1553 Pancreatic necrosis, 962, 962f Pancreatic neoplasms, 877-878, 878f Pancreatic neuroendocrine tumors, 1334-1339 Pancreaticobiliary duct junction, anomalous, 1046, 1046.e1f Pancreatitis, 959-967 acute, 959-963 acute interstitial, 960 acute necrotizing, 960 antipsychotic-induced, 2626t autoimmune, 960t, 961, 964 chronic, 927-928, 963-967 gallstone, 960, 960t acute, 1043-1044, 1043f genetics of, 960t, 961 metabolic factors, 960-961, 960t pleural effusions in, 634 recurrent, 964 tropical, 964 Pancreatitis-arthritis syndrome, 1824 Pancreatobiliary endoscopy, 877-878 Pancreatobiliary imaging, 873t, 877-878 Pancreatography, 873, 873t Pancreaze, 966t Pancytopenia, 1117 Pandemic influenza, 2193 Pandemics, 2191t Pandysautonomia, acute, 2518-2519 Panencephalitis rubella, progressive, 2210 subacute sclerosing, 2209-2210 Panic disorder, 2351, 2351t Panitumumab (Vectibix), 171, 1211t-1216t, 1232t, 1331 Panniculitis, 2691-2693 lobular, 2693 lupus, 2693 mesenteric, 950 Panniculitis-like T-cell lymphoma, subcutaneous, 1266 Pansinusitis, bilateral acute, 2587, 2588f Pantoprazole, 880-881, 900t Pantothenic acid (vitamin B5), 1446t-1450t, 2507t Pantothenic acid (vitamin B5) deficiency, 2507t Papaverine for ischemic colitis, 956b for mesenteric ischemia, 953f, 954 for superior mesenteric embolism, 954 Papillary epithelial tumors, 2583-2584, 2584f Papillary muscles, 264 Papillary necrosis, 736 Papillary renal cell carcinoma, 1346, 1346t Papillary serous carcinoma, peritoneal, 1380 Papillary thyroid carcinoma, 1511-1513 Papilledema, 2569, 2569f, 2575, 2575t Papillitis, 2574-2575 Papillomas, squamous, 2561 Papillomavirus, 2219-2223 Pappataci, 2259 Pappenheimer bodies, 1054f

Index Papua New Guinea, 1930, 1949 Papular eruptions, 2321, 2673 Papular exanthems, 2671-2675, 2671t Papular mucinosis, 2694 Papules, 2636t, 2647t-2651t benign, 2687-2693 hematopoietic, 2690-2691 inflammatory, 2690-2691 widespread, 2644-2654, 2655f Papulosquamous diseases, 2665-2671, 2665t, 2702t Paracellular pathway, 737 Paracoccidioides brasiliensis, 2075 Paracoccidioidomycosis, 2075-2076 acute-subacute (juvenile), 2075 chronic (adult), 2075 clinical manifestations of, 2075 definition of, 2075 diagnosis of, 2075 epidemiology of, 2075 pathobiology of, 2075 prognosis for, 2076 treatment of, 2075b-2076b Paradoxical intention, 2421t Paraganglioma, 386, 1523-1526 familial, 1523 genetic testing for, 1523-1524 genetics of, 1523 hereditary, 1227t malignant, 1527 Paragonimiasis, 2057t diagnosis of, 2158 treatment of, 2158 Paragonimus, 2157-2158 Paragonimus westermani, 2157 Parainfectious cranial polyneuritis, 2599 Parainfectious encephalomyelitis, 2599 Parainfluenza virus, 2185t, 2188 human (hPIV), 2188 Parainfluenza virus disease, 2188-2191 clinical manifestations of, 2188-2189 definition of, 2188 diagnosis of, 2189-2190 in elderly, 2189 epidemiology of, 2188 in immunocompromised patients, 2189 prevention of, 2190-2191 primary infection, 2188-2189 prognosis for, 2191 reinfection, 2189 treatment of, 2190b Paralysis acute flaccid, 2241, 2241t ascending, 2530 diaphragmatic, 627-628, 628.e1f familial hyperkalemic periodic paralysis, 762 familial hypokalemic periodic paralysis, 762 laryngeal, 2606 periodic forms of, 2544-2545 hyperkalemic, 2542t hypokalemic, 2542t Parameningeal infections, 2484, 2495 Paramyotonia congenita, 2542t Paranasal sinus cancer, 1298 Paranasal sinus tumors, 1299, 1302 Paraneoplasia, 1220 Paraneoplastic neuropathy, 2532-2533 autonomic, 2518t, 2519 clinical manifestations of, 2532-2533 definition of, 2532 diagnosis of, 2533 epidemiology of, 2532 pathobiology of, 2532 peripheral, 2532-2533 treatment of, 2533b Paraneoplastic syndromes, 1220-1221 clinical clues to, 2524t dermatologic, 1220-1221 diagnosis of, 1220t evaluation of, 1220t rheumatologic, 1221 Paraneoplastic vasculitis of nerves, 2533 Paranoid personality disorder, 2355t Paraparesis, spastic, 2236, 2237t, 2238-2239 Paraplatin (carboplatin), 1211t-1216t Paraplegia, spastic, 2469-2470 differential diagnosis of, 2470t hereditary, 2470, 2524t

Parapoxvirus infection, 2214-2215, 2215t clinical manifestations of, 2218 diagnosis of, 2218 prognosis for, 2219 treatment of, 2219 Parapsoriasis, 2669, 2669f Parasites ectoparasites, 2698 filarial, 2167t Parasitic infections antiparasitic therapy against, 2103-2107 blood smear features of, 1059t cutaneous infestations, 2698 diarrheas, 1869-1870, 1870t distinguishing features of, 528t of liver, 1013-1018, 1015t-1018t Parasitosis: delusions of, 2177 Parasomnia, 639, 2422-2424 clinical manifestations of, 2422 definition of, 2422 diagnosis of, 2422 epidemiology of, 2422 Parathyroid carcinoma, 1652t Parathyroid diseases, 1652t, 1825 Parathyroid glands, 1650 Parathyroid hormone, 1650 effects of, 1650-1652 regulation of extracellular fluid calcium by, 1650, 1650f Parathyroid hormone gene (PTH), 1650 Parathyroidectomy for hyperparathyroidism, 1653t, 1655 minimally invasive, 1655 Paratyphoid fever, travel-related risk, 1881 Parenchymal cysticercosis, 2150 Parenchymal liver disease differential diagnosis of, 991, 991t features of, 991t Parenchymal lung disorders, occupational, 80, 80t Parenteral nutrition administration of, 1442-1444 complications of, 1444, 1444t contraindications to, 1443 indications for, 1442t, 1443 solutions, 1443, 1443t Paresis, general, 2016 Paricalcitol, 842 Parinaud’s syndrome, 1292, 2578-2579 Parkinsonian gait, 2341t Parkinsonism, 2454-2461 cardinal signs of, 2454 causes of, 2454, 2460-2461 differential diagnosis of, 2454t features of, 2455 Parkinsonism minus, 2460-2461 Parkinson’s disease, 2454-2460 clinical manifestations of, 2454-2455 complications of, 2455 diagnosis of, 2455, 2455t diagnostic evaluation of, 2455, 2455t differential diagnosis of, 2456t environmental hypothesis for, 2454 epidemiology of, 2454 late-stage, 2455, 2455t motor symptoms of, 2454-2455 pathobiology of, 2454 pathology of, 2454 prognosis for, 2460 symptoms of, 2455 alleviating, 2460 treatment of, 2456b-2460b drugs for, 2456-2460, 2457t-2459t future directions, 2460 gene therapy, 2460 medical, 2456-2460 surgical, 2460 Paromomycin for amebic liver abscesses, 2141t for enteric protozoal infections, 2147t for giardiasis in pregnancy, 2137 for visceral leishmaniasis, 2122 Paronychia, 2709 Parotid enlargement, asymptomatic, 2585 Parotitis, 2207, 2207f recurrent, of childhood, 2585 Paroxetine (Paxil CR) for depression, 2349t for hot flashes, 1627 for hot flushes, 1628t-1629t

Paroxetine (Paxil CR) (Continued) for irritable bowel syndrome, 893t for pruritus, 2636t for syncope, 347 for Tourette’s syndrome, 2466 Paroxysmal cold hemoglobinuria, 1073-1075, 1075t Paroxysmal dysautonomias, 2519 Paroxysmal hemicrania, 2357t, 2360-2361 chronic, 2361 prevention of, 2361 treatment of, 2361 Paroxysmal nocturnal dyspnea, 302 Paroxysmal nocturnal hemoglobinuria, 1073, 1078-1079, 1116 clinical manifestations of, 1079 clones, 1116 definition of, 1078 diagnosis of, 1079 differential diagnosis of, 1119 epidemiology of, 1078-1079 genetics of, 1079 pathobiology of, 1079 thrombosis in, 1190, 1190b treatment of, 1079b Parsol 1789 (avobenzone), 2661 Partial pressure of carbon dioxide (Paco2), 652-653, 1619 Partial pressure of oxygen (Pao2), 652 normal values, 652 in pregnancy, 1619 reduction of, 656 Partial thromboplastin time, 990 Particulate matter exposures, 81t, 82 Partitioning, recursive, 35 Partner therapy expedited, 1945 patient-delivered (PDPT), 1945 for prevention of STIs, 1880 Parvoviridae, 2212 Parvovirus infections, 2212-2214 Past medical history, 24 Patau’s syndrome, 192 Patches, 2636t, 2647t-2651t Patchy alopecia, 2703-2706, 2703t, 2704f Patchy (circumscribed) hyperpigmentation, 2700 Patchy (circumscribed) hypopigmentation, 2699 Patella reflex, 2372t Patellar tendinitis (jumper’s knee), 1753 Patent ductus arteriosus, 410 anatomy of, 410, 410f diagnosis of, 410 exercise recommendations for, 416t incidence and prevalence of, 405 treatment of, 410b Patent foramen ovale, 409, 409b PATH, 1938-1939 Pathergy, 1799 Pathogen-associated molecular patterns (PAMPs), 225-226, 230 Pathogens blood, 1197-1198, 1197f health care–associated infections, 1862-1865 host-pathogen interactions, 1837-1838 Patient autonomy, 3 Patient communication Ask-Tell-Ask, 10, 14t, 52 core skills for, 14t disclosure to patients, 5, 5t discussion of diagnosis with, 153 discussion of heart failure management with, 315t discussion of palliative care with, 14t inquiry and disclosure of intimate partner violence, 1631 summarizing findings, 28 Patient education about fibromyalgia, 1818 about glucocorticoid replacement therapy, 1521 about heart failure, 315, 315t about irritable bowel syndrome, 893 about myocardial infarction, 455-456 sleep hygiene education, 2421t and smoking, 17, 17f about traveler’s diarrhea, 1882t Patient Health Questionnaire, 104, 302, 2347

I77

Patient isolation, 2255 Patient Protection and Affordable Care Act (PPACA), 19 Patient Reported Outcomes Measurement Information System (NIH), 43 Patient safety, 45-46 epidemiology of, 45 improvement of, 45-46 modern approach to, 45 Patient Self-Determination Act, 6 Patient-controlled analgesia, 137 Patient-delivered partner therapy, 1945 Patient-directed interventions, 47 Patterned alopecia, 2704 Paucibacillary leprosy, 2043, 2043f, 2046 Pauci-immune glomerulonephritis, 791, 1723 Pauci-immune vasculitis, 1794 PA-X protein, 2192t Pay for performance (P4P), 45 Pazopanib (Votrient), 170, 1211t-1216t, 1373 PB1-F2 protein, 2192t Peak expiratory flow rate (PEFR), 550, 550t Peak inspiratory pressure (PIP), 667-668 Pearson’s syndrome, 2546 Pectin, 922 Pectus excavatum, 630, 630b Pediatric periodic fever syndromes, 1713t Pediculosis, 2175-2176 Pegvisomant, 1487 Pelagia stings, 720 Pelargonium sidoides, 609 Pelger neutrophils, 1057 Pelger-Huët anomaly, 1055f, 1057, 1131 Peliosis hepatis, 1999-2001, 2000t Pelizaeus-Merzbacher disease, 2479 Pellagra, 1446t-1450t, 1558 Pelvic actinomycosis, 2061-2062 Pelvic examination, 28, 851 Pelvic hernias, 945 Pelvic inflammatory disease, 1877t, 1880, 1942-1943, 2010 clinical manifestations of, 1942 diagnosis of, 1942 prognosis for, 1942-1943 treatment of, 1945 Pelvic pain syndrome, chronic, 831t, 832-833 Pelviureteral abnormalities, 826 Pemetrexed (Alimta), 1211t-1216t Pemphigoid bullous, 2650f, 2656f cicatricial, 906 mucous membrane esophageal, 906 oral, 2583 upper aerodigestive tract, 2605 Pemphigus, 2677 clinical manifestations of, 2677 definition of, 2677 diagnosis of, 2677 pathobiology of, 2677 prognosis for, 2677 of throat and laryngitis, 2605 treatment of, 2677b Pemphigus vulgaris, 2652f, 2677f esophageal, 906 oral, 2580t, 2583 Penbutolol, 388t Penciclovir, 2181t Pencil cells, 1056 Penetrating injury, 712f, 712t Penicillamine, 1453t D-Penicillamine, 822, 1784 Penicillin, 1891t-1892t for anaerobic infection, 1933-1934, 1934t anaphylactic reaction to, 1705, 1705t for diphtheria, 1917, 2536 for drug-induced liver disease, 1010 for endocarditis, 479t-480t, 480 for enterococal infections, 1915.e1t for erysipelas, 2696-2697 for leptospirosis, 2030 mechanism of action, 1889t for meningitis, 2487 for meningococcal disease, 1937 resistance to, 1890t for S. pneumoniae infection, 1905 for syphilis, 2019, 2019t toxicities, 1895t Volume I  pp 1-1382  •  Volume II  pp 1383-2722

I78

Index

Penicillin G, 1891t-1892t for actinomycosis, 2062 for deep neck abscesses, 2603 for E. rhusiopathiae infection, 1924 for meningitis, 2488t for meningococcal disease, 1937t for osteomyelitis, 1810t pharmacokinetic parameters, 126t for prophylaxis in HIV infection, 2295t in renal failure, 129t for S. pneumoniae infection, 1905 for septic arthritis, 1810t for syphilis, 2018-2019 for toxic shock syndrome, 2672 for treponematoses, 2021 Penicillin V, 1891t-1892t Penicillin VK, 2602 Penicillium marneffei, 2320 Penile prostheses, 1579 Penis, 1570, 1570f Pentamidine for AIDS-associated opportunistic infections, 2297t-2301t for human African trypanosomiasis, 2115, 2115t for Pneumocystis pneumonia, 2096, 2096t, 2098, 2098t, 2313t-2314t for visceral leishmaniasis, 2122 Pentasa (mesalamine), 939t Pentasaccharides, 177-178 Pentastomiasis, 2175 Pentavalent antimonials, 2122 Pentazocine, 137, 139t-140t Pentobarbital for elevated ICP in acute liver failure, 1033 for pharmacologic coma, 2368 for seizures, 1033, 2408 Pentose phosphate pathway, 1085-1086, 1085f Pentose phosphate pathway disorders, 1087-1088 Pentostatin (deoxycoformycin), 1253 Pentoxifylline for atheromatous embolization, 507 for HTLV-associated myelopathy/tropical spastic paraparesis, 2238-2239 for NAFLD, 1022 for pernio, 510b Peppering technique, 1822 Pept-Bismol (bismuth subsalicylate), 922 Peptic strictures, 900 barium radiography findings, 898, 899f, 900 clinical manifestations of, 900 diagnosis of, 900 prognosis for, 900 treatment of, 900b Peptic ulcers, 249t, 908 complications of, 915-917 definition of, 908 differential diagnosis of, 910t drugs associated with, 912 H. pylori, 909, 909f in hypergastrinemic syndromes, 911 incidence of, 909 pathobiology of, 909 prognosis for, 917 symptoms and signs of, 912t Peptide hormones, 1583 Peptides, endogenous opioid, 1475 Peptostreptococcus infection, 1933-1934, 1934t Peramivir, 2196, 2196t Perceived pain, 2372 Percentiles, 32 Perception, 133 Percutaneous aortic valve replacement, 464-465 Percutaneous coronary intervention (PCI), 317, 456-458 advantages of, 446t comparison with CABG, 460-461, 460f complications of, 458 disadvantages of, 446t discharge issues, 458 failed, 460 guidewire passage, 457.e1 for ischemic heart disease, 431 mechanisms, 456-457 for multivessel coronary artery disease, 431 for myocardial infarction, 446-447, 446t

Percutaneous coronary intervention (PCI) (Continued) procedural success, 458 recommendations for, 428t restenosis after, 458 selection of patients for, 458 technical considerations, 456-457 thrombosis after, 458 Percutaneous endoscopic gastrostomy, 877 Percutaneous endoscopic jejunostomy, 877 Percutaneous nephrolithotomy, 814 Percutaneous pericardiocentesis, echocardiographic-guided, 488-489, 489f Percutaneous transluminal angioplasty, 502, 503f Perennial allergic rhinitis, 1688, 1688t Perennial rhinitis, 1674 Perforated ulcers, 917, 917f Performance status, 1206, 1207t Perfusion, asymmetrical, 534-535 Perfusion imaging, myocardial, 285-286, 286f Perfusion scintigraphy, 424t Pergolide, 2457t-2459t Perianal abscess, 969, 969b Perianal disease, 942 Perianal warts, 2221 Periarticular disorders, 1749-1754 Pericardial abnormalities, congenital, 491 Pericardial constriction, 489-490 clinical manifestations of, 489 diagnosis of, 489-490 differential diagnosis of, 490, 490t epidemiology of, 489 pathobiology of, 489 treatment of, 490b Pericardial cysts, benign, 491-492 Pericardial diseases, 483-492 CMR findings, 289, 291f echocardiographic findings, 280t-281t key echocardiographic findings, 280t-281t specific forms, 490-492 Pericardial effusion, 280, 282f, 280.e1 causes of, 486-487, 486t chest radiography in, 284, 284f, 487, 487f malignant, 1219 management of, 488f without tamponade, 488 Pericardial tumors, 338 clinical manifestations of, 338 diagnosis of, 338 prognosis for, 338 treatment of, 338b Pericardiocentesis, 488-489, 489f Pericarditis, 249t acute, 426, 483-486 Aspergillus, 2083-2084 autoimmune, 484t, 491 bacterial, 485t, 491 causes of, 484t-485t clinical presentation of, 485t constrictive, 489, 489f differential diagnosis of, 484t effusive-constrictive, 490 fungal, 491 H. influenzae type b, 1947 infectious, 484t, 491 initial management of, 486f malignant, 491 myopericarditis, 491 neoplastic, 484t-485t post-MI, 485t, 491 postradiation, 491 S. aureus, 1899 tuberculous, 485t, 491, 2034 uremic, 485t, 491 viral, 485t Perilymphatic fistula, 2595 Perinatally transmitted infections, 2009-2011 Perindopril, 388t Periodic fever with aphthous stomatitis, pharyngitis, and cervical adenopathy, 1740t Periodic health examination, 54-58 Periodic paralysis forms of, 2544-2545 hyperkalemic, 2542t hypokalemic, 2542t secondary causes of, 2545t Periodontal disease, 2585 Perioral dermatitis, 2680-2681, 2681f

Peripheral arterial disease, 504-511 antithrombotic therapy for, 180t atherosclerotic, 497-504 initial laboratory findings, 500t lower extremity, 498 mortality rates, 503 prognosis for, 503, 503f Peripheral autonomic nervous system, 2517 Peripheral autonomic neuropathy, 2518-2522, 2518t Peripheral blood reactions, leukemoid, 1130, 1130f Peripheral blood smear, 1052-1059 in erythrocyte enzymopathies, 1086, 1087f in hereditary spherocytosis, 1082, 1082f in megaloblastosis, 1110, 1110f with multiple abnormalities, 1052f normal, 1052f spherocytes on, 1082, 1082t Peripheral cold injury, 693 Peripheral edema, postoperative, 2623-2624 Peripheral ganglionopathies, autonomic, 2518-2522 Peripheral inflammatory arthritis, undifferentiated, 1714f Peripheral lung disease, 535-536, 536t Peripheral lymphoma, 2324, 2324b Peripheral nerve dysfunction, 2455t Peripheral nervous system autonomic, 2517 in orthopedic surgery, 1832 in osteoarthritis, 1746 Peripheral neuropathy, 2518, 2527-2537 autonomic, 2518-2522, 2518t clinical features of, 2527, 2539t compressive, 2536-2537 deficiency syndromes, 2536-2537, 2536t definition of, 2527 diagnosis of, 2527-2529 HIV, 2331-2332 immunologic, 2529-2532, 2530t infectious, 2536 inflammatory, 2529-2532, 2530t inherited, 2529 laboratory findings, 2528-2529 motor, 2528t neurologic examination in, 2528 neurologic testing in, 2528 pathobiology of, 2527 sensory, 2528t systematic approach to, 2527-2528, 2527f toxic neuropathies, 2536-2537, 2536t Peripheral pain, 134t Peripheral spontaneous nystagmus, 2579, 2579t Peripheral taste pathways, 2592-2593, 2592f Peripheral T-cell lymphoma, 1265-1266 unspecified, 1266 definition of, 1266 treatment of, 1266b Peripheral T-cell tolerance, 225-226 Peripheral venous disease, 511-519. See also Deep vein thrombosis Peripheral vestibulopathy, acute, 2598 Perisinusoidal cells, 1023 Peritendinitis, 1749-1750 Peritoneal adhesions, 949, 950f Peritoneal carcinomatosis, 949-950, 1380 Peritoneal dialysis, 841-842, 949 Peritoneal disorders, 949-950 Peritoneal effusions, 1219 Peritoneal papillary serous carcinoma, 1380 Peritonitis, 949-950, 949f bacterial, spontaneous in cirrhosis, 1025-1026, 1028, 1030 treatment of, 1030 Candida, 2081 from chronic ambulatory peritoneal dialysis, 949 clinical manifestations of, 949 diagnosis of, 949 prognosis for, 949b treatment of, 949b Peritonsillar abscess, 2602t, 2603, 2603f Perjeta (pertuzumab), 1211t-1216t Permethrin (Elimite), 2176, 2659 for lice, 2175, 2698 for scabies, 2173, 2698 Pernicious anemia, 1107 Pernio, 510, 510f lupus pernio, 605, 605f prevention of, 510b

Peroral endoscopic myotomy (POEM procedure), 878-879, 904 Peroxisomal disorders, 1387 Peroxisome proliferator-activated receptors, 1392 Peroxisomes: inborn errors of, 1387t Perphenazine, 2354 Persistent anemia, 2214, 2214t Persistent hyperinsulinemic hypoglycemia of infancy, 1529 Persistent müllerian duct syndrome, 1565 Personal behavior, 2386-2388. See also Behavior Personal protective equipment, 2255 Personality disorders, 2347t, 2355, 2355t clinical manifestations of, 2355 diagnosis of, 2355 treatment of, 2355b Personality states, 2355 Personality traits, 2355 Personalized medicine, 210, 1233 Pertussis, 1990-1993, 2602-2603 catarrhal phase, 1991 chemoprophylaxis of, 1992 clinical manifestations of, 1991-1992 complications of, 1991-1992 convalescent phase, 1991 definition of, 1990-1992 diagnosis of, 1992 epidemiology of, 1991 immunization against, 1992-1993 incubation period, 1991 paroxysmal phase, 1991 pathobiology of, 1991 prevention of, 1992-1993 prognosis for, 1993 treatment of, 1992b, 2602-2603 antimicrobial agents for, 1992 recommendations for, 1992 supportive therapy, 1992 Pertussis vaccine, 68t-73t, 75, 1992 adverse reactions to, 75 indications for, 75 Pertuzumab (Perjeta), 1211t-1216t Pertzye, 966t Pesaro classification of, 1094 Pesticides, 81, 81t Pesudomonadaceae, 1962 Petechiae, 475, 475f, 1160, 1160f Petit mal seizures, 2402 Petrolatum (Vaseline), 2657 Pets, 1972 Peutz-Jeghers syndrome, 1227t, 1324 clinical manifestations of, 1324, 1325f diagnosis of, 1324 general features of, 1323t pathobiology of, 1324 treatment of, 1324b Peyronie’s disease, 1579 pH extracellular, 740 Henderson-Hasselbalch equation for, 652 plasma, 739-740 urine, 728-729 Phaeohyphomycosis, 2101 clinical manifestations of, 2102 cutaneous, 2102 diagnosis of, 2102 subcutaneous, 2102 superficial, 2102 systemic, 2102 Phagocyte defects, 1682-1683, 1682.e1t Phagocytes, 1146, 1150-1151 Phagocytic defects, 1146 Phagocytic disorders, 1142-1151, 1147t-1148t clinical manifestations of, 1146, 1146t diagnosis of, 1150-1151, 1151f symptoms suggestive of, 1146t Phagocytosis, 1146 Phagosomes, 1146 Phalen’s sign, 2537 Pharmaceutical Research and Manufacturers of America, 9 Pharmacodynamics age-related changes, 130 in cancer therapy, 1208 Pharmacogenetics, 1208 Pharmacogenomics, 201t, 202 Pharmacokinetics age-related changes, 129-130 boosting, 2289

Index Pharmacokinetics (Continued) in cancer therapy, 1208 clinical applications of, 125-127 parameters for common drugs, 126t principles of, 124-125, 125f Pharmacologic coma, 2368 Pharmacologic stress imaging, 286 Pharmacologic stress testing, 2612-2614 Pharmacologic therapy. See Drug(s) Pharmacology of aging, 108 of cancer, 1208-1209 PharmGKB, 203.e2t Pharyngeal infection, 1943 Pharyngitis, 2601-2604 bacterial, nonstreptococcal, 2602-2603 antimicrobial therapy for, 2602 guidelines for management of, 2602t Epstein-Barr virus, 2252t-2253t, 2603-2604 fungal, 2604 mycobacterial, 2604 noninfectious, 2604-2606 periodic fever with aphthous stomatitis, pharyngitis, and cervical adenopathy, 1740t streptococcal, 1906-1907, 1907f, 1907t, 2252t-2253t, 2602 viral, 2602t, 2603-2604 Pharynx, 2601, 2601f Phencyclidine (PCP), 144t, 162, 699-702, 702t Phenelzine, 2349t, 2359 Phenergan (promethazine), 2660t Phenobarbital characteristics of, 2408t pharmacokinetic parameters, 126t for primary sclerosing cholangitis, 1045-1046 for seizures, 2407t-2408t, 2408 toxic ingestion of, 703t, 710t Phenotype cancer, 1229-1231 clinical, 1384 Phenoxybenzamine, 388t, 1526 Phenoxypenicillin, 2062 Phentermine, 1465-1466 Phentolamine, 396t Phenylephrine, 676t, 680 for cardiogenic shock, 685 for nasal inspection, 2586 for neurogenic shock, 2368 for severe traumatic brain injury, 2368 for vasospasm, 2449 Phenylketonuria, 1385, 1385t-1386t nutritional therapy for, 1388 therapeutic strategies for, 1388t Phenytoin characteristics of, 2408t hepatotoxicity, 1009 for neuromyotonia, 2552-2553 pharmacokinetic parameters, 126t in renal failure, 129t for seizures, 1033, 2407t-2408t, 2408, 2452t for sodium channelopathies, 2544 for status epilepticus, 2408 for subarachnoid hemorrhage, 2448, 2448t toxic levels, 703t for traumatic brain injury, 2368 for trigeminal neuralgia, 2363 Pheochromocytoma, 386, 1523-1526, 1524f cardiomyopathy with, 330 complications of, 1526 diagnosis of, 385t differential diagnosis of, 1524-1525 genetic testing for, 1523-1524 genetics of, 1523 malignant, 1527 in pregnancy, 1527 surgical therapy for, 1526 Pheochromocytoma crisis, 395t Philadelphia chromosome, 1246, 1246f Philadelphia chromosome-positive acute lymphoblastic leukemia, 1243-1244, 1246 Philadelphia chromosome-positive chronic myelogenous leukemia, 1240, 1248f Phlebotomus fever, 2259 clinical manifestations of, 2259 definition of, 2259

Phlebotomus fever (Continued) diagnosis of, 2259 epidemiology of, 2259 treatment of, 2259b Phlebotomus fever virus, 2259 Phlebotomy for hemochromatosis, 1421, 1422t maintenance, 1421 for polycythemia vera, 1125-1127 for porphyria cutanea tarda, 1414-1415 therapeutic, 1421 Phlogystic crystals, 1811 Phobias, 2351, 2351t Phocanema, 2163 Phosphate fractional excretion of (FEPO4), 777 metabolism of, 740, 740f regulation of, 740 Phosphate binders, 840 Phosphate restriction, 840, 842 Phosphate salts, 777 Phosphate supplements for diabetic ketoacidosis, 1542 for osteomalacia, 1649, 1649t Phosphatidylinositol 3-kinase (PI-3K)/Akt pathway, 1228, 1228f Phosphatidylinositol glycan (PIG)-A, 1079, 1116 Phosphodiesterase inhibitors, 179 Phosphodiesterase-5 inhibitors for benign prostatic hyperplasia, 830, 830t for erectile dsyfunction, 1578-1579 Phospholipase A2, cytosolic (cPLA2), 232 Phospholipids, 1722 Phosphonoformic acid, 2231 Phosphorus, 776 for hypophosphatemia, 777 metabolism of, 776 Phosphorus supplements, 1649 Phosphorylase deficiency, 2545 Photoaging, 2693-2694 Photoallergy, 2665, 2665t, 2687t Photodermatitis, 2321 Photodermatoses, 2664-2665, 2664t Photophobia, 2559 Photosensitivity, 2687 drugs associated with, 2687, 2687t immediate, 1413 in porphyrias, 1410 Phototherapy, 2661 Phototoxicity, 2665, 2665t PHOX2B gene, 547 Physalia stings, 720-721 Physical abuse, 1629-1633, 2339 Physical activity, 58-60 behavioral interventions for, 57 categories of, 1459 definition of, 58 energy expenditure of, 1461 guidelines for adults, 59, 60t guidelines for older adults, 60 guidelines for weight management, 59 health risks, 59 health-enhancing, 58 levels of, 58 limitations among older adults, 101f moderate-intensity, 58, 59t and obesity, 1459-1460 preventive health benefits in adults, 58 recommendations for clinical settings, 60 coordinating between medical care and community, 60 for hypertension, 387t tailoring, 60 risk management, 60 scale, 251t therapeutic health benefits of, 59 types of, 58 vigorous-intensity, 58, 59t volume of, 58 for weight loss, 1465 Physical examination, 24-28 of adolescent, 63 breast self-examination, 1359, 1359t cardiac examination, 303 cardiac risk assessment, 2612 chest examination, 27 clinical breast examination, 1359, 1359t essential concepts, 1717-1718

Physical examination (Continued) future directions, 28 general risk assessment, 2611 neurologic examination, 2339 nutritional assessment, 1430-1433 periodic health examination, 54-58 quantitative principles, 24 of respiratory system, 652 rush (shock) examination protocol, 675, 675t for signs of cardiovascular disease, 252-257 skin examination, 2637 summarizing findings for patients, 28 Physical fitness, health-related, 58 Physical inactivity and cancer, 1224 and cardiovascular disease, 259 Physician altruism, 3 Physician Orders for Life-Sustaining Treatment (POLST), 104 Physician Payment Sunshine Act, 9 Physician-assisted suicide. See Assisted suicide Physician-patient relationship, 4-6, 893 Physicians financial conflicts of interest, 8-9 gifts to, 9 primary interests of, 8, 8t Physiologic dysregulation, 106-107, 107.e1f Physiologic oxygen supply dependency, 672 Physiology in aging, 1570-1572 of circulatory control, 265-266 in development, 1570-1572 of heart, 265-266 Physostigmine, 706t-710t Phytonadione (vitamin K) deficiency, 2507t Pica, 1069 Picaridin, 2176 Pick bodies, 2397 Pick’s disease, 2397 Piebaldism, 2699 PIGA (phosphatidylinositol glycan-A gene), 1116 Pigbel, 1930 Pigeon breeder’s lung, 576-577 Pigmentation nail, 2709 oral mucosa, 2583, 2583t skin, 2637, 2637f disorders of, 2698, 2698t, 2702t postinflammatory change, 2638t-2644t, 2642f Pigmented granular casts, 730-731 Pill-induced esophagitis, 902 Pilocarpine, 1302, 2576, 2576f Pimecrolimus, 2636t Pimozide, 2463, 2466 Pindolol, 347, 388t, 430t Pineal region tumors, 1292 Ping-pong gaze, 2412 Pinguecula, 2563, 2563f Pink puffers, 557 Pinta, 2020 Pinworms, 2162, 2162b Piperacillin-tazobactam, 1891t-1892t for cholangitis, 1043 for diverticulitis, 947 for gallstones, 1041 for necrotizing fasciitis, 2696-2697 for P. aeruginosa, 1966 for peritonitis, 949 for pneumonia, 616t, 618, 619t, 1905t, 1966 for pyelonephritis, 1875t for pyogenic liver abscess, 1012 for S. pneumoniae infection, 1905t for septic shock, 689t Pirfenidone, 581 Piribedil, 2457t-2459t Piriform sinus cancer, 1299 Pirornaviridae, 2489 Pit vipers, 717, 719 Pit-1, 1480 Pitavastatin, 1396t Pitting, 1140 Pitting edema, 256f Pittsburgh Sleep Quality Index, 2416

I79

Pituitary adenoma, 1292, 1476 prevalence of, 1483, 1484t TSH-secreting, 1506-1507 visual field abnormalities with, 2575 Pituitary apoplexy, 1292, 1482 Pituitary axis, 1480-1481 Pituitary function tests, 1486, 1487t Pituitary gland anterior, 1480-1494 cell types in, 1480 in hypothalamic disease, 1477-1479 lesions of, 1520 on MRI, 1480, 1480f neuroendocrine organization of, 1473, 1474f normal, 1480f posterior, 1494-1500 radiology of, 1480, 1480f Pituitary incidentaloma, 1494, 1494.e1f Pituitary insufficiency tests, 1482, 1483t Pituitary microadenoma, 1480, 1481f Pituitary surgery, 1484, 1484.e1 Pituitary tumors, 1483-1484 clinical manifestations of, 1484 clinically nonfunctioning, 1494 gonadal insufficiency due to, 1574 medical therapy for, 1484-1485 pathobiology of, 1483-1484 radiation therapy for, 1484 treatment of, 1484b-1485b Pityriasis lichenoides, 2668-2669 Pityriasis lichenoides chronica (PLC), 2668-2669 Pityriasis lichenoides et varioliformis acuta (PLEVA), 2668-2669 Pityriasis rosea, 2638t-2646t, 2667, 2667f clinical manifestations of, 2667 diagnosis of, 2667 epidemiology of, 2667 pathobiology of, 2667 treatment of, 2667b Pityriasis rubra pilaris, 2638t-2644t, 2666, 2667f clinical manifestations of, 2666 diagnosis of, 2666 epidemiology of, 2666 treatment of, 2667b Pityrosporum folliculitis, 2681 Pivmecillinam, 1875t PKD1 gene, 817 PKD2 gene, 817 Plague, 1984-1990, 2058t bioterrorism with, 87t, 89-90 bubonic incubation period, 1987 prognosis for, 1988 symptoms and signs of, 1987 clinical manifestations of, 89 definition of, 1984 diagnosis of, 89, 1987-1988 distribution of, 1984, 1985f-1986f epidemiology of, 89, 1984-1985 fatality rate, 2060t pathobiology of, 89, 1985-1987 pneumonic, 1987, 1987f clinical manifestations of, 1987 distinguishing characteristics of, 2252t-2253t incubation period, 1987 recommendations for treatment, 1989t prevention of, 89-90, 1988 prognosis for, 90, 1988 septicemic, 1984f, 1986-1987, 1987f transmission of, 1984, 1986f treatment of, 89b, 1988b vectors, 2172t Plague septicemia, 1986-1987, 1987f Plakophilin 1, 2633 “Plan, do, study, act” (PDSA), 45 Plantar fasciitis, 1754 Plantar warts, 2221, 2221f Plaque, 2471-2472 atherosclerotic activation of, 418-419 formation of, 418f in HIV infection, 2303t large plaque parapsoriasis, 2669, 2669f pleural, 537, 538f skin lesions, 2636t, 2647t-2651t white lesions, 2581-2582, 2581t

Volume I  pp 1-1382  •  Volume II  pp 1383-2722

I80

Index

Plaque rupture, 418, 419f, 433, 434t Plaquenil (hydroxychloroquine), 1761, 2573t Plasma, convalescent, 2255 Plasma cell disorders, 1273-1284, 1273t, 1276t Plasma cell leukemia, 1281 Plasma exchange for Guillain-Barré syndrome, 2530 for inflammatory myelopathy, 2381 for thrombotic thrombocytopenic purpura, 1165-1166 Plasma membrane transporter protein defects, 1386, 1386t Plasma osmolality, 749 normal values, 1499 in pregnancy, 1499 regulation of, 1494-1495, 1495f Plasma pH, 739-740 Plasma phosphate, 740 Plasma potassium, 740 Plasma transfusion, 1195 Plasma urea, 745 Plasmablastic lymphoma, 1265, 2324 Plasmacytoma extramedullary, 1282 solitary of bone, 1282 diagnostic criteria for, 1276t Plasmapheresis for Goodpasture’s syndrome, 574 for systemic lupus erythematosus, 1776 for thrombotic thrombocytopenic purpura, 1165-1166 Plasminogen activator tissue (t-PA) for frostbite, 510-511 for pleural effusion, 635 for pulmonary embolism, 624t recombinant (rt-PA, alteplase), 180, 447t Plasmodium, 1015t-1017t Plasmodium falciparum, 2108-2109, 2108f Plasmodium falciparum malaria severe, 2110 treatment of, 2111t Plasmodium ovale, 2108-2109, 2108f Plasmodium vivax, 2107-2109, 2108f Plateau pressure (Pplateau or Pplat), 654, 654f, 664t, 667-668 Platelet count in cirrhosis, 1027 postoperative, 2624 Platelet dense (δ) granule storage pool deficiency, 1171 Platelet disorders acquired, 1171-1172, 1172b drug-induced, 1171 hereditary, 1171 hypofunctional, hereditary, 1171 qualitative, 1171-1172 storage pool disorders, 1171 Platelet function analyzer (PFA), 1156 Platelet inhibitors, 178, 179f Platelet plugs, 433-434 Platelet sequestration, 1159-1160 Platelet transfusion, 1160-1161, 1193-1195 for aplastic anemia, 1121 for bleeding before acute leukemia therapy, 1243 clinical trials, 1193t complications of, 1161 for disseminated intravascular coagulation, 2254 platelet dose, 1195 refractory patients, 1161 threshold for, 1195, 1195f Platelet α-granule deficiency, 1171 Platelet-active drugs, 178-180 Platelet-endothelial cell adhesion molecule-1 (PECAM-1, CD31), 1145 Plateletpheresis, 1161 Platelets abnormalities of, 1183, 1184t aggregation of, 433-434 assessment of, 1057-1058 decreased production of, 1160 giant, 1057-1058, 1058f increased destruction of, 1159 normal-sized, 1057f Platinol (cisplatin), 1211t-1216t Pleocytosis, neutrophilic, 2494 Pleura, 631-636

Pleural aspergillosis, 2084 Pleural disease, 536-538 Pleural effusion, 632-635 chest radiography findings, 632, 633f clinical manifestations of, 632 conditions that cause, 632, 632t CT findings, 632, 633f diagnosis of, 632-635, 633f epidemiology of, 632 loculated, 537, 537f lupus, 634t malignant, 634 pancreatic, 966 parapneumonic, 633-634 pathobiology of, 632 prognosis for, 635b subpulmonic, 537, 537f in systemic inflammatory disorders, 634 treatment of, 635b tuberculous, 634, 634t ultrasound findings, 632-633, 633f Pleural exudates, 635 Pleural fluid disease characteristics, 633, 634t mechanisms that promote accumulation of, 632, 632t normal turnover of, 632 Pleural friction rub, 526 Pleural plaques, 537, 538f Pleural thickening, diffuse, 537 Pleural tuberculosis, 2034 Pleurisy, 249t, 634, 634t Pleuritis, 634, 634t, 1773 Pleurodynia, 2241t, 2243 clinical manifestations of, 2243 diagnosis of, 2243 prognosis for, 2243 treatment of, 2243b Pleuropulmonary infections, anaerobic, 1932 Plummer-Vinson syndrome, 1069 Pluripotency, 206f Pluripotent stem cells, induced, 204, 207f Pneumococcal bacteremia, 1902 Pneumococcal conjugate vaccine (PCV13), 68t-73t, 75-76, 617-618, 1905 adverse reactions to, 76 indications for, 76 for prophylaxis in HIV infection, 2295t recommendations for, 618, 67.e1t-67.e3t Pneumococcal endocarditis, 476 Pneumococcal meningitis, 2480, 2485-2486 Pneumococcal pneumonia definition of, 1902 incidence of, 1902 microbiologic diagnosis of, 613, 613f predisposing factors, 1903, 1903t Pneumococcal polysaccharide vaccine (PPSV, PPSV23), 67, 68t-73t, 75, 617-618, 1904-1905 adverse reactions to, 76 indications for, 68f, 76 for prophylaxis in HIV infection, 2295t recommendations for, 67f-68f, 618, 1904-1905, 67.e1t-67.e3t Pneumococcal vaccines, 68t-73t, 75-76, 554 adverse reactions to, 76 indications for, 76 Pneumoconioses, 588, 589t, 593-594 Pneumocystis, 2091, 2097 Pneumocystis jirovecii, 2091 Pneumocystis jirovecii–associated IRIS, 2334, 2334f Pneumocystis pneumonia, 2091-2099 antiretroviral therapy for, 2097 chest radiography in, 2093, 2094f clinical manifestations of, 2093 corticosteroid therapy for adjunctive, 2096-2097 risks, 2096-2097, 2096.e1f CT findings, 2093, 2094f definition of, 2091 diagnosis of, 2093 diagnostic evaluation of, 2093, 2095f epidemiology of, 2091-2092 histopathology of, 2092, 2092f HIV-associated, 2312-2314 antiretroviral therapy for, 2314 clinical manifestations of, 2312-2313, 2313f diagnosis of, 2313 epidemiology of, 2312 pathobiology of, 2312.e1

Pneumocystis pneumonia (Continued) prevention of, 2314, 2314t prognosis for, 2314 radiographic features of, 2309t, 2310f treatment failure, 2314 treatment of, 2295, 2297t-2301t, 2313b-2314b, 2313t immunofluorescent assays for, 2093, 2095f pathobiology of, 2092-2093 prevention of, 2097-2099, 2098t prognosis for, 2099 resistance to treatment, 2097 treatment failure, 2097 treatment of, 2093b-2097b, 2096t Pneumomediastinum, 637, 551.e1f Pneumonia, 610-620 Acinetobacter, 1968 antibiotic therapy for, 615-617 duration of treatment, 617 failure to respond to, 617, 617t ICU regimens, 616t, 617 in-hospital regimens, 616-617, 616t outpatient regimens, 616, 616t aspiration, 618 atypical, 2002 bacterial atypical, 2252t-2253t HIV-associated, 2306-2311 bronchiolitis obliterans organizing pneumonia of unknown cause (BOOP), 583 causes of, 614-615, 614t chlamydia, 2010 chronic eosinophilic, 586 clinical manifestations of, 611 community-acquired, 1902 antibiotic regimens for, 689t empirical treatment of, 616t, 1896 incidence of, 610.e1f indications for diagnostic testing, 1904, 1904t Legionella pneumophila, 1995, 1995f-1996f staphylococcal, 1899 complications of infectious, 617 noninfectious, 617 cryptogenic organizing, 583-584 cytomegalovirus, 2230f, 2297t-2301t definition of, 610 diagnosis of, 611-615, 613f differential diagnosis of, 614-615 enteroviral, 2241t eosinophilic, chronic, 586 epidemiology of, 610 failure to respond to antibiotic therapy, 617, 617t fungal, 2309t, 2314 group A streptococci (GAS), 1907 H. influenzae, 1947 health care–associated, 618-620 burden, costs, and preventability of, 1862t Pseudomonas, 1964 hospital course, 617 hospital-acquired, 618-620 antibiotic regimens for, 689t staphylococcal, 1899-1900 imaging findings, 532, 533f, 532.e1f interstitial acute, 571t, 572-573 chronic fibrosing, 575t, 580-581 desquamative, 583.e1f idiopathic, 575t, 579-584 lymphoid, 584.e1f rare, 575t smoking-related, 575t laboratory findings, 612-613 lipoid, 601-602 Mycoplasma arginini, 2058t necrotizing, 612 noninfectious considerations, 615 overview, 610-620 patchy, 612, 613f pathobiology of, 610-611, 1903, 1903f pathology of, 611 physical findings, 611 with pleurisy, 249t pneumococcal definition of, 1902 incidence of, 1902 microbiologic diagnosis of, 613, 613f

Pneumonia (Continued) predisposing factors, 1903, 1903t radiographic findings, 612, 612f Pneumocystis, 2091-2099 predisposing factors, 610, 610t prevention of, 617-618 prognosis for, 618 Pseudomonas, 1964, 1966-1967 radiographic findings, 611-612, 612f respiratory syncytial virus, 2188 Rhodococcus, 2058t SMART-COP scoring system, 616t staphylococcal hospital-acquired, 1899-1900 treatment of, 615b-617b, 1966-1967 ventilator-associated, 618-620, 1866 burden, costs, and preventability of, 1862t guidelines and recommendations for, 1865.e1t Pseudomonas in, 1964 staphylococcal, 1899-1900 treatment of, 1896 viral, 2314-2315 Pneumonia Outcomes Research Trial (PORT) Severity Index, 615t Pneumonic plague, 1987, 1987f clinical manifestations of, 1987 distinguishing characteristics of, 2252t-2253t incubation period, 1987 treatment of, 1989t Pneumonic tularemia, 1983, 2060t Pneumonitis hypersensitivity, 585, 590-592, 585.e1f acute, 589t chronic, 589t, 585.e1f interstitial, nonspecific, 1779 Pneumothorax, 538, 635-636 clinical manifestations of, 636 diagnosis of, 636, 636f epidemiology of, 636 pathobiology of, 636 right-sided, 636f spontaneous, 249t, 643-644 surgery for, 643-644 tension, 538f, 636 treatment of, 636b Pneumovax (Merck), 1904 Pneumovax 23, 617-618 Pnu-Imune (Lederle), 617-618, 1904 Podophyllotoxin (podofilox), 2222, 2222t POEM procedure (peroral endoscopic myotomy), 878-879, 904 POEMS (polyneuropathy, organomegaly, endocrinopathy, M protein, and skin changes) syndrome, 1276t, 1281-1282, 2699, 2699t Poikilocytes, teardrop, 1054f Poikilocytosis, 1052f Poikilothermy, 1479 Point-of-care echocardiography, 276, 277t Poison centers, 719 Poisoning acute, 696-711 alcohol, 151 arsenic, 94-95 cadmium, 95-96, 798 carbon monoxide, 599 cardiopulmonary effects of, 698-699 cyanide and other gases, 599 dermatologic signs of, 699 diagnosis of, 696-703, 697f diagnostic syndromes, 703 diagnostic tests, 699-703 drug overdose, 127, 128f, 129 environmental, 920 epidemiology of, 696 food, 920 gastrointestinal effects of, 699 heavy metal, 93t history in, 696 imaging findings, 703 initial stabilization, 704-705 lead toxicity, 92, 798 management of, 704b-706b, 704f marine, 722 mercury, 92-94 neurologic signs of, 697-698, 698t oxygen, 600 physical findings, 696-699 prognosis for, 711

Index Poisoning (Continued) shellfish, 722 tetrodotoxin (puffer fish), 722 trace metal, 92-98 treatment of, 704b-706b Poisons, 717 Poliomyelitis clinical manifestations of, 2241 postpolio syndrome, 2524t spinal, 2241 travel-related risk, 1881 Poliovirus vaccine, 68t-73t, 76, 1883, 1883t adverse reactions to, 76 indications for, 76 Poly(ADP-ribose) polymerase inhibitors, 1358 Polyangiitis granulomatosis with, 586 ANCAs in, 1795-1796 clinical manifestations of, 1798 eosinophilic, 1798 laryngeal involvement, 2605, 2605.e1f neuropathy in, 2533, 2533t pathologic characteristics of, 1795t treatment of, 1798 microscopic clinical manifestations of, 1798 treatment of, 1798 Polyarteritis nodosa, 2692-2693 clinical manifestations of, 1796, 1797f epidemiology of, 1794t gastrointestinal, 956 neuropathy in, 2533, 2533t pathologic characteristics of, 1795t treatment of, 1796 Polyarthritis carcinomatous, 1826 chronic, 1764t epidemic febrile, 2261 zoonotic, 2057, 2059t Polyarthropathy syndrome, 2214, 2214t Polychondritis, relapsing, 1827, 2605, 2605.e1f Polychromasia, 1052f, 1067 Polychromatic cells, 1052f Polychromatic macrocytes, 1052f Polycystic kidney and hepatic disease 1 gene (PKHD1), 820 Polycystic kidney disease adult, 187t autosomal dominant, 816-820 clinical features of, 817t, 818-819 cyst formation in, 816, 818f diagnosis of, 819, 819f epidemiology of, 816 pathology of, 816f prevention of, 819b-820b treatment of, 819b-820b autosomal recessive, 820-821 clinical features of, 817t, 820 diagnosis of, 819f, 820-821 epidemiology of, 816 pathology of, 816f prevention of, 821b treatment of, 821b definition of, 816 signaling pathways in, 820.e1f therapeutic interventions for, 820.e1f Polycystic liver disease, 1035 Polycystic ovarian–like disorders, 1589 Polycystic ovarian–like syndrome, 1593 Polycystic ovary syndrome, 1584 clinical features of, 1593, 1593f definition of, 1593 diagnosis of, 1593-1594 pathobiology of, 1593 treatment of, 1594 preferred medications, 1602t surgical, 1594 Polycystin-1 (PC1), 818, 818.e1f Polycystin-2 (PC2), 817-818, 818.e1f Polycythemia vera, 1121-1129 clinical features of, 1123, 1123t definition of, 1121-1122 diagnosis of, 1124 diagnostic algorithm for, 1125f diagnostic criteria for, 1124t epidemiology of, 1122 incidence of, 1122 laboratory features of, 1123t

Polycythemia vera (Continued) mutations in, 1122, 1123t pathobiology of, 1122 prognosis for, 1128 risk stratification in, 1127t, 1128 risk-adapted therapy for, 1127t treatment of, 1125-1128 Polydipsia primary, 1497-1498 psychogenic, 2629 Polyethylene glycol FDG-PET, 1309 for intestinal obstruction, 565-566 for whole bowel irrigation, 705 Polyethylene glycol (MiraLAX), 889, 893t Polygenic diseases, 191 Polyglandular disorders, 1555-1557 autoimmune hypoparathyroidism, 1652t, 1660 autoimmune syndromes, 1556-1557, 1556t, 1660 clinical significance of, 1555-1556 definition of, 1555-1556 neoplastic syndromes, 1556, 1556t Polyhexamethyl biguanide, 2566 Polymer fume fever, 589t, 594t Polymerase chain reaction, 614, 2483 Polymerase proteins, 2192t Polymicrobial infections, 1846 Polymicrogyria, 2513 Polymorphic light eruptions, 2664, 2664f Polymorphisms definition of, 190 enzyme, 131-132, 132t, 1007 genetic, 189t, 190 single nucleotide (SNPs), 190, 196 Polymyalgia rheumatica, 1801-1805 classification of, 1713t clinical manifestations of, 1802-1803 definition of, 1801 diagnosis of, 1803-1804, 1803f differential diagnosis of, 1804 epidemiology of, 1801 etiology of, 1801-1802 incidence of, 1801 pathobiology of, 1801-1802 treatment of, 1804b-1805b, 1805 Polymyositis, 2237t classification of, 1713t, 1790t diagnosis of, 1791 diagnostic criteria for, 1791, 1791t lung disease associated with, 585 pathobiology of, 1789 pathology of, 1790f prevalence of, 1789 prognosis for, 1793 treatment of, 1792 Polymyxin B, 1967, 1969-1970 Polymyxin B and bacitracin (Polypore), 2657 Polyneuropathy, distal symmetrical, 2535 Polyneuropathy, organomegaly, endocrinopathy, M protein, and skin changes (POEMS) syndrome, 1281-1282 cutaneous findings in, 2699, 2699t diagnostic criteria for, 1276t Polyol pathway flux, increased, 1543 Polypectomy of colon adenoma, 1326, 1326.e1 endoscopic, 876, 876f of jejunal adenoma, 1326-1327, 1322.e1 saline lift, 1327.e1 snare, 1326, 1326.e1 Polypharmacy, 137-138 Polypore (polymyxin B and bacitracin), 2657 Polyposis juvenile, 1323t, 1325 MUTYH-associated, 1323t, 1324 MYH-associated, 1227t Polyposis syndromes, 1323-1325 Polyps adenomatous, 1325-1326 of colon, 1325-1327 colorectal, 878-879, 878f definition of, 1325-1327 hamartomatous, 1325 nasal, 2586, 2587f, 2588-2589, 2589f neoplastic, 1326 nonadenomatous, 1325 non-neoplastic, 1325

Polyps (Continued) prognosis for, 1326-1327 serrated, 1326-1327 treatment of, 1326b Polyradiculomyelitis, cytomegalovirus, 2208-2209 Polysomnography, 638f, 639, 640f, 2416-2417, 2417t Polyunsaturated fatty acids, omega-3, 314, 306.e1t-306.e2t Polyuria, 741, 754-755 definition of, 754 in diabetes insipidus, 1496-1497 reasons for, 755t treatment of, 755b Pompe’s disease, 1387, 2545 characteristic signs of, 1385t management of, 325 enzyme replacement therapy, 1388 therapeutic strategies for, 1388t Ponatinib (Iclusig), 171, 1232t for cancer, 1211t-1216t for chronic myelogenous leukemia, 1248, 1249t, 1250 Pontiac fever, 1993-1995 Pontine hemorrhage, 2451 Pontine stroke syndromes, 2435, 2435.e1f Popcorn lung, 589t Popliteal artery entrapment syndrome, 504, 504b Popliteal cysts, 1753, 1757-1758, 1757.e1f Popliteal vein thrombosis, 513f Population global, 100, 101t inferring values from samples, 35-36 Population studies, 188 Porcelain gallbladder, 1042 Porokreatosis, actinic, 2651f Porphyria, 1408-1416, 1408t, 2665 acute, 1408, 1408t clinical manifestations of, 1411-1412 differential diagnosis of, 1413-1414 etiology of, 1409-1410 homozygous, 1412 treatment of, 1414 acute attacks, 1411-1412, 1414 biochemical findings, 1413 classification of, 1408 clinical features of, 1411-1413, 2524t definition of, 1408 diagnosis of, 1413-1414 differential diagnosis of, 1413-1414 epidemiology of, 1408 etiology of, 1409-1410 genetic testing for, 1413 induction of, 1411 investigational therapies for, 1415 nonacute, 1408, 1408t, 1410 pathobiology of, 1408-1411 pathogenesis of, 1410-1411 prevention of, 1415-1416 primary, 1415 secondary, 1415-1416 prognosis for, 1416 pseudoporphyria, 2678 recurrent attacks, 1414 somatic mutations that cause, 1410 treatment of, 1414b-1415b Porphyria cutanea tarda, 1408t, 2665, 2665f, 2678, 2678f classification of, 1408 diagnosis of, 1413 etiology of, 1410 familial, 1410 HFE mutation analysis in, 1423 prevention of, 1415-1416 skin disease in, 1412 treatment of, 1414-1415 Porphyromonas, 1931, 1934t Portal hypertension, 1023-1025, 1025f, 1027 Portal pressure, 1027 Portal vein thrombosis, 959 chronic, 959 clinical manifestations of, 959 diagnosis of, 959 after liver transplantation, 1036-1037 prognosis for, 959 treatment of, 959b on ultrasound, 868, 868f Portopulmonary hypertension, 1025-1028

I81

Portopulmonary syndrome, 1033.e1t Posaconazole, 2068-2069 adverse effects of, 2068 for AIDS-associated opportunistic infections, 2297t-2301t for candidiasis, 2297t-2301t for chromomycosis, 2103 drug interactions, 2068 formulations, 2068 indications for, 2068-2069 for invasive aspergillosis, 2086t for mycoses, 2068 for thrush, 2082 Positional cloning, 192 Positional tests, 2600 Positioning head down (Trendelenburg) position, 2448t prone, 662, 669 sleep, 641 Positive airway pressure adherence, 641 autotitrating, 640-641 bilevel, 640 continuous, 665 for acute respiratory failure, 660 for obstructive sleep apnea, 640-641 equipment for, 640, 641f for obstructive sleep apnea, 640-641 outcomes, 641 patient interfaces, 640-641, 641f prescribing, 640 Positive end-expiratory pressure, 665 for acute lung injury and ARDS, 662 for acute respiratory distress syndrome, 663f, 669 for acute respiratory failure, 660 auto-PEEP, 667, 667f Positive predictive value, 37t Positive selection, 237 Positive-pressure ventilation, 665 nocturnal, 629, 629t noninvasive, 629-630, 666 Positive-pressure ventilators, 665 Positron emission tomography in cancer with unknown primary site, 1379 in coronary artery disease, 424t in epilepsy, 2404 fused PET/CT in gastroenterology, 872, 872f in head and neck cancer, 1300, 1300f in Hodgkin’s lymphoma, 1270 of lungs, mediastinum, and chest wall, 532 of myocardial viability, 286 in neurologic disease, 2345t in rheumatic disease, 1728-1730 Postcardiotomy syndrome, 490-491 Postconcussion syndrome, 2368, 2599 Posterior cerebral artery, 2427, 2428f-2429f Posterior cerebral artery occlusion, 2435, 2435t Posterior cord syndrome, 2379 Posterior inferior cerebellar artery occlusion, 2435t Posterior leukoencephalopathy syndrome, 2575 Posterior pituitary gland, 1494-1500 anatomy of, 1494-1496 hormone synthesis in, 1494-1496 Posterior reversible encephalopathy syndrome, 1861 Posterior tibial tendinitis, 1754 Posterior tibial tendon rupture, 1754 Posterior urethral valves, 826f, 827 Post-herpetic neuralgia, 2536, 2680 Post–Kala-azar dermal leishmaniasis, 2122 Postnasal drip syndrome, 528t Postoperative care, 2621, 2625 aftermath, 2625 comprehensive, 2621 focus on recovery, 2621 patterns of mistakes in, 2621 recommended prophylaxis in, 2621 setting priorities for, 2625 special situations, 2625 teamwork for, 2621 Postoperative care orders, 2621 Postoperative complications, 2621-2625 multiplicity, 2625 redundancy, 2625 Volume I  pp 1-1382  •  Volume II  pp 1383-2722

I82

Index

Postoperative nausea and vomiting, 2620 Postoperative thrombosis, 1191 Postpartum (lymphocytic) thyroiditis, 1509 Postpolio syndrome, 2524t Postpoliomyelitis muscle atrophy, 2241 Postprandial distress syndrome, 894, 894t Postprandial glucose levels, 1534, 1535t, 1536-1537 Postprandial hyperinsulinemic hypoglycemia, 1551-1552 Postprandial hypotension, 392, 392f Post-streptococcal glomerulonephritis, 790b, 790f, 2252t-2253t Post-test odds, 38 Post-test probability, 37t Post-thrombotic syndrome, 518, 518b Post-transplantation lymphoproliferative disorder, 2649f EBV-associated, 1038, 2234 lymphoma in, 1267 Post-traumatic stress disorder, 2353 Posture abnormal, 2340 and myofascial pain, 1822 in Parkinson’s disease, 2455 Post-vaccination encephalomyelitis, 2217 Potassium deficiency of. See Hypokalemia for diabetic ketoacidosis, 1541-1542 excess of. See Hyperkalemia excretion of, 757, 760t-761t high-potassium diet, 762 for hypokalemia, 761-762 in hypovolemia, 745 importance of, 756 increased levels, 769 postoperative abnormalities, 2624 regulation of, 740, 756 renal handling, 756 total amount, 756 Potassium balance, 756-757, 757t Potassium channel blockers, 306.e1t-306.e2t Potassium channels, 341-342, 757, 341.e1f Potassium chloride, 355 Potassium citrate to prevent calcium stones, 815 to prevent cystine stones, 816 to prevent uric acid stones, 815 Potassium deficiency, 2507t Potassium disorders, 755-762 clinical manifestations of, 758-759 definition of, 755 epidemiology of, 755 external, 758 internal, 757-758 pathobiology of, 756-758 postoperative, 2624 prognosis for, 762 treatment of, 761b-762b Potassium hydroxide, 2219 Potassium hydroxide examination, 2656, 2656f Potassium iodide (KI) for radiologic accidents or terrorism, 85 saturated solution (SSKI), 1508, 2078-2079 for sporotrichosis, 2078-2079 for Sweet’s syndrome, 2682 Potassium phosphate salts, 777 Potassium-sparing diuretics (ENaC antagonists), 748t contraindications to, 389t for hypertension, 388t, 391 side effects of, 389t Potocki-Shaffer syndrome, 194 Pouchitis, 942 Powassan virus encephalitis, 2263t, 2268 Power, study, 35 Power Doppler imaging, 1726 Poxvirus infections, 2214-2219 clinical manifestations of, 2216-2218 definition of, 2214 diagnosis of, 2218 epidemiology of, 2214-2215 pathobiology of, 2215-2216 prevention of, 2218b-2219b treatment of, 2218b-2219b Poxviruses, 2214, 2215t PP interval, 268 PPACA. See Patient Protection and Affordable Care Act PPoma, nonfunctioning, 1335t

PR interval, 267-269, 268t Practice guidelines, 44-45 Prader-Willi syndrome, 194, 1479, 1565 Pralatrexate (Fotolyn), 1211t-1216t Pralidoxime chloride, 699t-702t, 706t-710t Pramipexole, 2424, 2457t-2459t, 2468-2469 Pramlintide, 1536 Pramoxine, 2636t Prasugrel, 179 for acute coronary syndrome, 437, 438t-439t for acute MI, 448, 450, 454, 454t, 455f, 458 Pravastatin, 1396t Prayer, intercessory, 184 Prayer sign, 1824 Praziquantel, 2104-2105 for cysticercosis, 2151 for flukes, 2104, 2158 for schistosomiasis, 1018, 2104, 2155, 2155.e1 for tapeworms, 2104, 2148-2149, 2148t Prazosin, 388t Precocious puberty, 1478, 1571 Preconception carrier testing, 201-202 Predictive value negative, 37t positive, 37t Predictor variables, 33 continuous, 34 dichotomous, 33-34, 33.e1t Prednisolone, 164t for acute mountain sickness prophylaxis, 596-597 for alcoholic liver disease, 1021 for Bell’s palsy, 2537 for chronic inflammatory demyelinating polyradiculoneuropathy, 2531 for colitis, 941-942 for COPD exacerbations, 561t for herpes zoster ophthalmicus, 2565 for iritis, 2570 for Kawasaki disease, 2672 for myocarditis, 328-329 for sympathetic ophthalmia, 2571 for warm autoimmune hemolytic anemia, 1077 Prednisone, 164t for acute interstitial nephritis, 796 for acute interstitial pneumonia, 573 for acute kidney injury, 782 for acute pericarditis, 486 for AIDS-associated opportunistic infections, 2297t-2301t for autoimmune hepatitis, 1006 caveats, 1761t for chronic inflammatory demyelinating polyradiculoneuropathy, 2531 for chronic lymphocytic leukemia, 1256 for chronic obstructive pulmonary disease, 559, 562 for chronic pain, 138t for chronic sinusitis, 2588 for cluster headache, 2361 for colitis, 941 for complex regional pain syndrome, 2522 for congenital toxoplasmosis, 2132t for Crohn’s disease, 940 for Diamond-Blackfan anemia, 1120 for diarrhea, 934-935 for Duchenne’s muscular dystrophy, 2542 for EBV infection, 2233 for endocrine dysfunction after HSCT, 1204 for eosinophilic esophagitis, 902 for giant cell arteritis, 497, 1804 for glomerulosclerosis, 786 for Goodpasture’s syndrome, 574 for granulomatosis with polyangiitis, 586, 1798 for hemophilia, 1177 for Hodgkin’s lymphoma, 1271-1272 for HTLV-associated myelopathy/tropical spastic paraparesis, 2238-2239 for hypereosinophilic syndrome, 336 for hypersensitivity pneumonitis, 592 for hypopituitarism, 1483t for IgA nephropathy, 788-789 for immune reconstitution inflammatory syndrome, 2335 for immune thrombocytopenic purpura, 1164, 1164t for inflammatory bowel disease, 939t, 940

Prednisone (Continued) for inflammatory myopathies, 1792 for leprosy reactional states, 2046 for lichen planus, 2668t for lupus nephritis, 792 for membranous nephropathy, 787 for minimal change disease, 785-786 for multiple myeloma, 1280, 1280t for myasthenia gravis, 2551 for myelofibrosis, 1128 for nasal polyps, 2589 for non-Hodgkin’s lymphoma, 1263t for pemphigoid, 2605, 2675-2676 for pemphigus, 2605 for Pneumocystis pneumonia, 2096t for polymyalgia rheumatica, 1804 for prostate cancer, 1369 for pulmonary infections, 565 for radiation lung injury, 601 in renal transplantation, 844t replacement doses, 1520-1521 for respiratory bronchiolitis–-associated interstitial lung disease, 582 for reversible cerebral vasoconstriction syndrome, 1800 for sarcoidosis, 607, 607t for sensorineural hearing loss, 2595-2596 side effects of, 1037t, 1644 for Sweet’s syndrome, 2682 for systemic anaphylaxis, 1702 for systemic lupus erythematosus, 1775 for systemic vasculitis, 2534 for Takayasu’s arteritis, 497, 1796 for toxoplasmic retinitis, 2566 for urticaria, 1695, 2684t for warm autoimmune hemolytic anemia, 1077, 1078f for zoster, 2228-2229 Preeclampsia, 1166, 1613-1616, 1614t definition of, 1613 diagnosis of, 1613-1615 epidemiology of, 1613 liver disease in, 1622 pathobiology of, 1613 pathogenesis of, 1615f prevention of, 1616 prognosis for, 1616 treatment of, 395t, 1615b-1616b, 1615t Pregabalin for distal symmerical polyneuropathy, 2536 for erythromelalgia, 508 for pain, 136-137, 138t, 2369, 2477 for pruritus, 2636t for restless legs syndrome, 2424 for zoster, 2229 Pregestational diabetes mellitus, 1614t Pregnancy acute fatty liver of, 1622 adolescent, 63-64 antimicrobial therapy in, 1845 antiretroviral therapy in, 2291 aplastic anemia in, 1116 asthma in, 554, 1617-1619, 1619.e1f basic principles of, 1610-1611 cancer in, 1218 chronic liver disease in, 1623, 1623b complications of, 1614t deep vein thrombosis in, 515, 517, 1616-1617 diagnosis of rubella in, 2205 dietary requirements in, 1445-1452 ectopic, ruptured, 634t folate deficiency in, 1108 hepatitis B in, 1623 hepatitis C in, 1623, 1623b high-risk, 1100 Hodgkin’s lymphoma in, 1272-1273 hypertension in, 394, 1611-1613 definition of, 1611 diagnosis of, 1611-1612 epidemiology of, 1611 pathobiology of, 1611 prognosis for, 1614t treatment of, 1613b hyperthyroidism in, 1508 immunizations in, 73 incidental thrombocytopenia of, 1166 inflammatory bowel disease in, 943 intrahepatic cholestasis of, 1622 jaundice in, 988 liver disease in, 1622-1623, 1622b, 1622t lupus, 1773-1774

Pregnancy (Continued) luteoma of, 1564 medical problems in, 1610-1623 multiple sclerosis in, 2473 non-Hodgkin’s lymphoma in, 1267 normal changes in, 1610, 1612t pheochromocytoma in, 1527 pulmonary embolism in, 625, 1616-1617 recommended radiation exposures, 1610, 1612t screening for HBV in, 1623 seizures in, 2408 sickle cell anemia in, 1100 syphilis in, 2018-2019 thrombocytopenia during, 1166-1167 thrombosis in, 1190 toxoplasmosis in diagnosis of, 2130 treatment of, 2131, 2132t venous thromboembolism (VTE), 1619t viral hemorrhagic fever in, 2255 viral hepatitis in, 1623 Prehypertension definition of, 381 dietary guidelines for, 1428t-1429t epidemiology of, 381 lifestyle recommendations for, 387t preferred oral antihypertensive drugs for, 394t Prekallikrein deficiency, 1178 Preload, 266 Premature action potential, 368.e1f Premature atrial contractions, 359 Premature ovarian failure, 1590 Premature termination codon, 194 Premature ventricular contractions, 372-373 Prematurity: retinopathy of, 2571 Premenstrual syndrome, 1587 definition of, 1587 diagnosis of, 1587 symptoms of, 1587, 1587t treatment of, 1587b medical therapy, 1587 surgical therapy, 1587 Premenstrual tension, 1587 Premutations, 194 Prenatal diagnosis, 201-202 of congenital toxoplasmosis, 2130 of hemophilia, 1176-1177 Preo-adrenomedullin, mid-regional (MR-proADM), 304.e1t Preoperative evaluation, 2611-2617 airway assessment, 2617 for anesthesia, 2617 cardiac risk assessment, 2612-2615 endocrine, 2615-2616 for noncardiac surgery, 2612-2614, 2617t pulmonary function tests, 2615 pulmonary risk assessment, 2615 recommendations for laboratory testing, 2612t tests, 2611 Prepatellar bursitis, 1753 Prepositioning Antibiotics for Anthrax (IOM), 1922 Presbycusis, 2595 Presbyopia, 2557 Preseptal cellulitis, 2564, 2564f Pressure bandaging, 718 Pressure sores, 108, 108f Pressure-controlled ventilation, 665 Pressure-support ventilation, 665 Pressure-volume diagram, 265 Pressure-volume loop, 265, 266f Pressure-volume relationships, 265 Presyncope (dizziness), 345-346 Pretectum lesions, 2576, 2576f Pretest (prior) odds, 37t Pretest (prior) probability, 37t Prevalence, 33 Preventive health care. See also Immunizations; Screening; specific conditions for adolescents, 63 behavioral interventions, 57 chemoprevention, 57 dietary guidelines for, 1428t-1429t of drug interactions, 131 endocarditis prophylaxis, 416 with dental procedures, 482t recommendations for, 482t future, 2621

Index Preventive health care (Continued) of health care–associated infections, 1867-1868 of injury, 57 for older adults, 102, 105 of pain, 142-143 of radiation exposure to, 85 recommended for postoperative care, 2621 Prevnar7, 617-618 Prevnar13, 617-618 Prevotella, 1931, 1934t Priapism, 1101, 1103 Prilocaine, 2636t Primaquine, 2106 for AIDS-associated opportunistic infections, 2297t-2301t contraindications to, 1088t for malaria, 2111, 2111t-2112t for Pneumocystis pneumonia, 2096, 2096t, 2313t Primary biliary cirrhosis, 1018t, 1046-1047 arthritic manifestations of, 1823 clinical manifestations of, 1047 diagnosis of, 1047 epidemiology of, 1046-1047 liver transplantation for, 1034 pathobiology of, 1047 prognosis for, 1047 treatment of, 1047b Primary care physicians, 2610 Primary lateral sclerosis, 2524t Primary sclerosing cholangitis, 1045-1046 clinical manifestations of, 1045 definition of, 1045 diagnosis of, 1045, 1046f epidemiology of, 1045 liver transplantation for, 1034-1035 pathobiology of, 1045 prevalence of, 1045 prognosis for, 1046 treatment of, 1045b-1046b Primary survey, 713 Primidone, 2407t, 2462 Prinzmetal’s angina, 420-421, 431 Prion diseases, 2504-2506 clinical manifestations of, 2505 definition of, 2504 diagnosis of, 2505-2506 epidemiology of, 2504 genetics of, 2505 pathobiology of, 2504-2505 pathology of, 2504-2505 prevention of, 2506 prognosis for, 2506 treatment of, 2506b Prion protein (PrP), 2504 Prion protein gene (PRNP), 2505 Prior (pretest) odds, 37t Prior (pretest) probability, 37t Priorities: setting, 2625 Prisms, 2551 Pritelivir, 2226 Probability, 37 definition of, 37t formula for, 38 odds and, 37 post-test (posterior), 37t pretest (prior), 37t test results and, 38, 38f Probenecid, 1945, 1975 Probiotics, 893t Problem drinking, 57 Problem-solving therapy, 2348t Procainamide, 126t, 363t-364t for advanced cardiac life support, 354f for arrhythmias, 355 in renal failure, 129t Procarbazine, 1271-1272 Prochlorperazine for migraine headache, 2358-2359 for nausea and vomiting, 865t for vertigo, 2600 Procrit (erythropoietin), 1211t-1216t Proctitis, 941 Professional interpreters, 16 Professionalism, 3-4, 3t Progenitor cells, hematopoietic, 1050 Progesterone for amenorrhea, 1589 for hot flushes, 1625-1626

Progesterone (Continued) for luteal phase dysfunction, 1594 for vasomotor symptoms, 1626t Progestin for abnormal uterine bleeding, 1588 for amenorrhea, 1589 for contraception, 1604 injectable, 1604 side effects and risks of, 1626 synthetic, 1607 for vasomotor symptoms, 1625, 1626t Prograf (tacrolimus), 2660 Progressive aphasia, primary nonfluent/agrammatic variant of, 2385, 2397 semantic variant of, 2385, 2397, 2398f Progressive diaphyseal dysplasia (CamuratiEngelmann disease), 1669, 1669b Progressive external ophthalmoplegia, chronic, 2546 Progressive familial intrahepatic cholestasis, 987-988 Progressive multifocal leukoencephalopathy, 2210-2212 clinical manifestations of, 2210 definition of, 2210 diagnosis of, 2210-2211, 2211f epidemiology of, 2210 imaging in, 2211, 2211f pathology of, 2210, 2211f prognosis for, 2212 test for, 2502t treatment of, 2212b Progressive muscular atrophy, 2524t Progressive relaxation, 182t Progressive rubella panencephalitis, 2210 Progressive supranuclear palsy, 2606 Proguanil atovaquone and proguanil (Malarone) for babesiosis, 2144 for malaria, 1883, 2105, 2111t-2112t, 2112 for malaria, 2105 Prokine (sargramostim), 1211t-1216t Prolactin, 1478-1479, 1487-1488 Prolactin deficiency, 1487 Prolactin inhibitory factor, 1474-1475 Prolactinoma, 1488, 1489f Prolactin-releasing factor, 1475 Prolaris, 1233 Proleukin (aldesleukin), 1211t-1216t Prolia (denosumab), 1211t-1216t Prolotherapy, 2375 Prolymphocytic leukemia, 1254, 1254t Promacta (eltrombopag), 1164 Promethazine (Phenergan), 2660t for diarrhea, 922 for gastroparesis and pseudo-obstruction, 888 for migraine headache, 2359 for nausea and vomiting, 865t for vertigo, 2600 Promyelocytes, 1143-1144 Prone positioning, 662, 669 Propafenone, 363t-364t, 365-366, 372-373 Propantheline, 2521, 2551 Propensity scores, 36 Propionibacterium, 2060 Propionibacterium acnes, 1934t Propionic aciduria, 1385t, 1387t Propofol for anesthesia, 2618-2619 for pharmacologic coma, 2368 for sedation, 1033 for seizures, 1033, 2408 Proportional assist ventilation, 666 Proportional hazards (Cox) model, 35 Proportions, 32-33 Propoxyphene, 139t-140t, 2468-2469 Propranolol, 430t for alcohol withdrawal, 155t for anxiety, 2352t for aortic dissection, 495-496 for arrhythmias, 355 for hypertension, 388t for hypertrophic cardiomyopathy, 325 for hypokalemic periodic paralysis, 762 for thyrotoxicosis, 1507-1508 for tremor, 2462 for varices and varical bleeding, 1029

Proprotein convertase subtilisin/kexin type 9 gene (PCSK9), 194-195, 261 Proprotein convertase subtilisin-like/kexin type 9, 1390, 1390t Propylene glycol, 769 Propylthiouracil, 1508 Prosopagnosia, 2385 Prostacyclin (prostaglandin I2), 232, 510-511 Prostaglandin D2, 172, 173f Prostaglandin E1, 1579 Prostaglandin E2, 172, 173f, 232 Prostaglandin F2α, 172, 173f Prostaglandin H synthase, 172-173 inhibition of, 173-175 Prostaglandin H synthase 1, 172 Prostaglandin H synthase 2, 172 Prostaglandin I2 (prostacyclin), 172, 173f, 232 Prostaglandins, 172, 173f, 232 Prostanoids, 172-175 action of, 172, 173f anti-inflammatory, 235 biosynthesis of, 172, 173f Prostate cancer, 1367-1370 advanced disease, 1369 androgen-deprivation–resistant, 1367 castration-resistant, 1367, 1369 clinical manifestations of, 1367 definition of, 1367 diagnosis of, 1367-1368 epidemiology of, 1367 Gleason score, 1367-1368 high-risk disease, 1368 incidence of, 15, 1367 localized, 1368 low- to intermediate-risk disease, 1368 palliative care, 1369 pathobiology of, 1367 prevention of, 1368 prognosis for, 1369 recurrent disease, 1368-1369 screening for, 56 staging, 1368 treatment of, 1368b-1369b approach to, 1369t future directions, 1369 principles of therapy, 1368 Prostate gland, 827, 828f Prostate-specific antigen (PSA), 1367, 1380 Prostate-specific antigen (PSA) testing, 831 Prostatic obstruction, 316 Prostatitis, 830-833, 831b-833b acute bacterial (type I), 831t causative organisms, 830 clinical manifestations of, 831 prognosis for, 833 treatment of, 831 asymptomatic inflammatory (type IV), 831t chronic (type III) classification of, 831t clinical manifestations of, 831 treatment of, 832-833 chronic bacterial (type II), 831t clinical manifestations of, 831 treatment of, 831-832 classification of, 830, 831t clinical manifestations of, 831 diagnosis of, 831 epidemiology of, 830 laboratory evaluation of, 831 pathobiology of, 830 prognosis for, 833 Prostheses, penile, 1579 Prosthetic heart valves advantages of, 472t choices among, 472, 473f disadvantages of, 472t echocardiographic findings, 280t-281t mechanical valves, 473f postoperative care for, 472 Prosthetic joint infection, 1809 classification of, 1809 prevention of, 1809 Prosthetic mitral valve replacement, 276, 277f Prosthetic valve endocarditis, 477 causes of, 477, 477t indications for surgery, 481t Protamine sulfate, 2452t

I83

Protease inhibitors, 235, 2288t, 2289 CSF-to-plasma ratios, 2331t for HIV/AIDS, 2290t side effects of, 2291 Proteases, 231, 233-234, 1732-1733, 231.e1t Proteasome-associated systemic inflammatory diseases, 1743 Protectin (CD59), 243t Protein(s) ascitic fluid levels, 1028, 1028t in bacterial meningitis, 2483, 2483t citrullinated, 1719-1720 dietary breakdown of, 836, 836.e1f estimation of, 836, 836t intestinal digestion and absorption of, 926, 926.e1f disorders of metabolism of, 1384-1386, 1385t-1386t food, 1107 in lipid transport, 1389, 1390t membrane, 1080f, 1081 recommended dose, 1441-1442 requirements for, 1442t serum, 1432-1433 urine dipstick test for, 729 Protein 4.1, 1083 Protein C deficiency, 1185, 1186f, 1186t prevalence of, 1185 type I, 1185 type II, 1185 Protein kinase C, 1543-1544 Protein restriction, 814 Protein S (vitronectin), 243t Protein S deficiency, 1185-1186, 1186t frequency of, 1185 type I, 1185-1186 type II, 1185-1186 Protein-energy malnutrition, 1434-1437 causes of, 1441t clinical manifestations of, 1434-1435 definition of, 1431, 1434 diagnosis of, 1431, 1435-1436 epidemiology of, 1434 nutritional therapy for, 1433 pathobiology of, 1434 prevention of, 1437 prognosis for, 1437 treatment of, 1436b-1437b Protein/leucine sensitivity testing, 1555 Protein-losing gastroenteropathy, 934, 934b Proteinuria, 784 Proteoglycans, 1731 Proteomics, 203, 1838 Prothrombin complex concentrate, activated, 1177b Prothrombin deficiency, 1175t, 1179-1180 Prothrombin gene mutation (prothrombin G29210A), 1186, 1186t Prothrombin time (PT), 990, 1155-1156 prolonged, 1156-1158, 1158f Proton magnetic resonance spectroscopy, 2345t Proton pump inhibitors for acid peptic disease, 915 for Barrett’s esophagus, 901-902 for esophageal disorders, 900t for H. pylori–associated ulcers, 914, 915t for upper GI bleeding, 880-881 Proto-oncogenes, 1227-1228 Protoporphyria erythropoietic classification of, 1408 clinical manifestations of, 1411 congenital, 1412 diagnosis of, 1413 prevention of, 1415 skin disease in, 1412-1413, 1413f treatment of, 1415 X-linked, 1408, 1408t, 1410, 1413 Protoporphyrin IX, 1063 Protoscolices, 2147-2148 Protozoal infections enteric, 2145-2147, 2147t intestinal, 2105 of liver, 1013-1018 treatment of, 2105-2106 vaginal, 2105 Protriptyline, 2419t Provider-directed interventions, 47 Volume I  pp 1-1382  •  Volume II  pp 1383-2722

I84

Index

Provitamins, 1454 Proxies, 6 Proximal renal tubular acidosis, 823t-824t Proximal tubular cells, 737, 739f Proximal tubules, 763-764 functional disorders of, 822-823, 823t ion transport in, 739, 739f natriuretics of, 748 Prozac (fluoxetine), 893t Prucalopride, 889 Prune-belly syndrome, 827 Pruned-tree sign, 284, 284f Prurigo nodularis, 2321 Pruritic eruptions, papular, 2321 Pruritus (itch) pathophysiology of, 2635-2637, 2635t, 2635.e1f in skin disease, 2646t systemic treatments for, 2636t topical treatments for, 2636t Pruritus ani, 970 aquagenic, 1123 clinical manifestations of, 970 diagnosis of, 970 epidemiology of, 970 incidence of, 970 treatment of, 970b Prussian blue (ferric hexacyanoferrate [II]) for radiologic accidents or terrorism, 85 for thallium poisoning, 93t Pseudechis (black snakes), 720 Pseudoaldosteronism, 824 Pseudoaneurysm, 453 Pseudoathetosis, 2528 Pseudobulbar palsy, 2605-2606 Pseudoclaudication, 500t Pseudo-Cushing states, 1515f Pseudocysts, pancreatic, 966, 966f Pseudoephedrine, 2186t, 2588 Pseudoexfoliative glaucoma, 2566 Pseudofractures, 1646, 1647f Pseudogout, 1811, 1815 Pseudohemoptysis, 529 Pseudo-Hurler polydystrophy, 1733, 1734t Pseudohypoaldosteronism type 1, 770, 823t Pseudohypoaldosteronism type 2, 760, 823t Pseudohyponatremia, 749 Pseudohypoparathyroidism, 1661 clinical features of, 1661t type 1a, 1652t type 1b, 1652t variants, 1661, 1661t Pseudolymphoma, 2638t-2644t Pseudomonal keratitis, 2565 Pseudomonas, 1962 cutaneous infections, 2697 folliculitis, 2681, 2681f, 2696-2697 Pseudomonas aeruginosa, 1962-1963 Pseudomonas aeruginosa infection, 1962-1968 bacterial factors, 1963-1964 in bone and joints, 1965 in central nervous system, 1965 chronic respiratory tract, 1965 clinical manifestations of, 1964-1966 diagnosis of, 1966 in ear, 1964 endovascular, 1965 epidemiology of, 1963 in eye, 1964 gastrointestinal, 1965 health care–associated, 1863t, 1864, 1964 multidrug-resistant, 1963 pathobiology of, 1963-1964 pathogenesis of, 1963 pathology of, 1964 pathophysiology of, 1963 prevention of, 1967-1968 prognosis for, 1968 related gram-negative bacillary infections, 1962 in skin and soft tissue, 1965 treatment of, 1966b-1967b uncommon, 1965 in urinary tract, 1965 virulence factors, 1963.e1t Pseudomonas aeruginosa meningitis, 1967 Pseudomonas aeruginosa pneumonia, 1964, 1966-1967 Pseudomonas alcaligenes, 1962 Pseudomonas CDC group 1, 1962 Pseudomonas cepacia, 1967 Pseudomonas chlororaphis, 1962

Pseudomonas delafieldii, 1962 Pseudomonas fluorescens, 1962, 1965-1966 Pseudomonas kingii, 1962 Pseudomonas luteola, 1962 Pseudomonas maltophilia, 1962 Pseudomonas mendocina, 1962 Pseudomonas oryzihabitans, 1962 Pseudomonas pertucinogena, 1962 Pseudomonas pseudoalcaligenes, 1962 Pseudomonas putida, 1962, 1965-1966 Pseudomonas stutzeri, 1962, 1965-1966 Pseudonaja (brown snakes), 720 Pseudonephritis, athletic, 725 Pseudo-obstruction, intestinal, 885, 886t Pseudo-osteoarthritis, 1815 Pseudoparathyroidism, 1661t Pseudo-Pelger neutrophils, 1057 Pseudoporphyria, 2678 Pseudoprecocious puberty, 1478 Pseudopseudohypoparathyroidism, 1661, 1661t Pseudoterranova, 2163 Pseudothrombocytopenia, 1156 Pseudotumor, orbital, 1797, 2570 Pseudotumor cerebri, 2362 Pseudoxanthoma elasticum, 1738-1739, 2634 clinical manifestations of, 1738 definition of, 1738 diagnosis of, 1738 differential diagnosis of, 1738 epidemiology of, 1738 gastrointestinal involvement, 958 ocular symptoms of, 2569 pathobiology of, 1738 pathogenesis of, 1738 pathology of, 1738 treatment of, 1738b type I, 958 Psittacosis, 2012-2013, 2058t clinical manifestations of, 2012 diagnosis of, 2012 epidemiology of, 2012-2013 prevention of, 2013 treatment of, 2012b Psoas sign, 858t Psoralen with ultraviolet A (PUVA), 2661, 2670t Psorcon E (diflorasone diacetate), 2658t Psoriasis, 2638f, 2648f, 2665-2666, 2666f clinical features of, 2665-2666, 2703t color in, 2638t-2644t diagnosis of, 2666 epidemiology of, 2665 in HIV infection, 2321 inflammation pathways in, 230.e3t inverse, 2701f pathobiology of, 2665 pustular, 2681 regional involvement, 2701f scalp, 2703t sequential therapy for, 2666, 2666t treatment of, 2658, 2666b Psoriatic arthritis, 1768 classification of, 1713t diagnosis of, 1768 differential diagnosis of, 1764t, 1768 epidemiology of, 1768 nail pitting in, 1768, 1768f pathobiology of, 1768 radiographic features of, 1724t treatment of, 1768b Psoroptes mites, 2173 Psychiatric care alternative medications, 1462t evaluation of chief complaints, 2626 general considerations in deciding to refer to, 2356t inpatient settings, 2625-2626 medical consultation in, 2625-2629 reasons for transfer to general hospital, 2626t Psychiatric disorders, 2346-2356, 2347t alcohol-related complications, 151t, 153 causes of, 2346t comorbid conditions, 2346 complications of, 2626, 2626t health status in, 2625 medical complications of, 2626-2629 medical evaluation in, 2625-2626 mood disorders, 2346 in skin disease, 2646t treatment of, 2346

Psychiatric evaluation, 28, 2355-2356 Psychiatry history, 2338 Psychodynamic psychotherapy, 2348t Psychogenic polydipsia, 2629 Psychological symptoms, 2355t in fibromyalgia, 1818, 1820 management of approaches to, 11t palliative care, 10 Psychophysiologic dizziness, 2600t Psychosocial assessment, 1032-1033 Psychosocial issues alcohol-related complications, 151t, 153 in geriatrics, 716.e1t in heart failure management, 315t normal development, 61 in treatment for cigarette smoking, 146b-149b Psychosocial support for cancer, 1219 for sexual development disorders, 1568 Psychostimulants, 144t Psychotherapy cognitive, 2348t interpersonal, 2348t for intimate partner violence, 1631-1632 for irritable bowel syndrome, 894 for pain, 138-141 psychodynamic, 2348t Psychotic disorders, 2347t, 2353-2354, 2353t Psychotropic drugs complications of, 2626, 2626t perioperative, 2613t Psyllium, 934-935 PTEN hamartoma syndrome, 1325 Pterygium, 2548t, 2563, 2563f Ptosis, 2549, 2558, 2560, 2560f Pubarche, premature, 1581 Pubertal maturation, 1588, 1588t Puberty, 1570 asynchronous, 1581 in boys aberrations, 1570-1571 precocious, 1571 reproductive axis development during, 1570-1571 stages of, 1570, 1570t timing of, 1570, 1570f development of aberrations, 1570-1571, 1581-1584, 1582t asynchronous, 1581, 1582t, 1584 delayed (or interrupted), 1581, 1582t, 1583-1584 in girls, 1579-1584 heterosexual, 1582t, 1584 precocious, 1581, 1582t in girls, 1581 aberrations, 1581-1584, 1582t asynchronous, 1581, 1584 delayed (or interrupted), 1581, 1582t, 1583-1584 development of, 1579-1584 hormonal changes during, 1581 ovarian function in, 1580-1581 physical changes during, 1580-1581 precocious, 1581, 1582t timing of, 1581, 1581f heterosexual, 1581 precocious, 1478, 1581-1583 in boys, 1571 diagnosis of, 1581-1583 diagnostic tests for, 1583 differential diagnosis of, 1581-1583 in girls, 1581 incomplete isosexual, 1581 treatment of, 1583b pseudoprecocious, 1478 weight gain during, 61 Pubic hair development, 61, 62f, 1588t Pubic lice, 1877t PUBMED, 24.e1t Puerto Ricans, 587 Puff adder, 719 Puffer fish (tetrodotoxin) poisoning, 722 Pulmicort (budesonide), 553t Pulmonary actinomycosis, 2061, 2061f Pulmonary air space, postoperative, 2622t Pulmonary airway malformation, congenital, 570

Pulmonary alveolar proteinosis, 571-572, 571t clinical manifestations of, 572 CT findings, 572, 572f, 572.e1f diagnosis of, 572, 572f, 572.e1f epidemiology of, 571-572 pathobiology of, 572 radiography findings, 572, 572f treatment of, 572b Pulmonary angiography, 621-623, 622f Pulmonary arterial hypertension. See Pulmonary hypertension Pulmonary arterial hypotension, 284f Pulmonary arteriovenous fistulas, 411 Pulmonary artery catheters, 2615 Pulmonary artery obliterative vasculopathy, 1779-1780, 1780f Pulmonary artery occlusion pressure (PAOP), 687t Pulmonary artery pressure (PAP) calculation of, 279-280, 399, 401f normal value, 687t Pulmonary artery wedge pressure (PAWP), 687t Pulmonary aspergilloma, 612, 2084, 2084f, 612.e1f Pulmonary aspergillosis chronic, 2084 chronic cavitary, 2084 chronic necrotizing, 2084 invasive, 2083 Pulmonary atresia, 285f Pulmonary barotrauma, 597-598 Pulmonary blastomycosis, 2074 Pulmonary blood pressure, 397 Pulmonary capillaritis, idiopathic, 586 Pulmonary congestion, 301-302, 302f Pulmonary cryptococcosis, 2076 diffuse, AIDS-associated, 2297t-2301t treatment of, 2077 Pulmonary disease associated renal diseases, 790t basal, 536f chronic obstructive, 555-562 gene therapy for, 212 high-penetrance monogenic disorders, 187t in HIV infection, 2305-2318 CD4+ cell count ranges, 2309t clinical findings, 2306 diagnostic approach to, 2307f-2308f diagnostic clues to, 2309t diffuse cryptococcosis, 2297t-2301t evaluation of, 2306 pathobiology of, 2310 pathogenesis of, 2310 radiographic findings, 2306, 2309t imaging in, 531-539 nontuberculous mycobacterial, 2040-2042 Pulmonary edema, 283, 283f acute, 319f acute decompensated heart failure with, 318-319 cardiogenic, 662, 664t clinical features of, 528t, 664t flash, 385-386 high-altitude prevention of, 596-597 treatment of, 597 imaging of, 532, 534f, 532.e1f interstitial, 283f photographic negatives of, 586 in pregnancy, 1614t recommended treatment of, 395t Pulmonary embolism, 249t, 620-627 in cancer, 625 chronic, 284f clinical features of, 528t, 621 CT angiography findings, 621-622, 622f definition of, 620 diagnosis of, 621-623, 622f strategies for, 623 tests for, 621-623 differential diagnosis of, 484t ECG findings, 623, 623f epidemiology of, 620 evaluation for malignancy after, 1189, 1189f in hypercoagulable states, 1187b-1188b MRI findings, 623 nonthrombotic, 626-627 pathobiology of, 620-621

I85

Index Pulmonary embolism (Continued) patient subgroups, 625 pleural fluid characteristics, 634t postoperative, 1191 in pregnancy, 625, 1616-1617 prevention of, 625-626 prognosis for, 626 risk stratification, 624t severe, 623-625 testing for thrombophilia after, 1187, 1187f thrombotic, 620-626, 624f treatment of, 623b-625b, 1187b-1188b anticoagulation therapy, 624-625, 625t fibrinolytic therapy, 624, 624t reperfusion therapy, 624 ventilation-perfusion (V/Q) lung scan findings, 622-623, 622f Wells’ clinical prediction rule for, 621, 621t Pulmonary emphysema, 284f Pulmonary eosinophilia, tropical, 2168 Pulmonary examination, 303 Pulmonary fibrosis familial, 575-576 idiopathic, 579-581, 580.e1f pathobiology of, 575-576 treatment of, 581b Pulmonary flukes, 2156t, 2157-2158 clinical manifestations of, 2158 diagnosis of, 2158 epidemiology of, 2157 pathobiology of, 2157-2158 prevention of, 2159 treatment of, 2158b Pulmonary function tests, 539, 540t abnormalities, 542, 543t future directions, 545 interpretation of, 542-544, 543f in interstitial lung disease, 579 in obesity, 544-545 preoperative, 2615 provocative testing, 544 reference equations, 540 tests of ventilation, 540 Pulmonary hemosiderosis, idiopathic, 573, 586 Pulmonary histiocytosis X, 576 Pulmonary histoplasmosis acute, 2070 chronic, 2070 complications of, 2070 treatment of, 2071 Pulmonary (arterial) hypertension, 249t, 397-404 causes of, 403-404 chronic thromboembolic, 403, 625, 398.e1f clinical classification of, 397t clinical manifestations of, 398-399 complications of, 406-407 definition of, 397, 1783 development of, 398.e1f diagnosis of, 399-402, 400f differential diagnosis of, 426 ECG findings, 399, 399f echocardiographic findings, 280t-281t, 399, 401f epidemiology of, 397-398 history in, 398-399 in HIV infection, 2317 clinical manifestations of, 2317 diagnosis of, 2317 epidemiology of, 2317 pathobiology of, 2317 prognosis for, 2317 radiographic features of, 2309t treatment of, 2317b longitudinal evaluation of, 404t pathobiology of, 398 pathophysiology of, 398, 398.e1f physical findings, 399 prognosis for, 404, 404t radiographic findings, 399, 400f secondary to structural disease, 406-407 in sickle cell anemia, 1101, 1103, 1103t symptoms of, 398 in systemic sclerosis, 1783 treatment of, 402b-404b, 1103, 1785 assessing response to, 404 background therapy, 402 evidence-based, 403f

Pulmonary (arterial) hypertension (Continued) general measures, 402 invasive therapy, 402-403 recommendations for, 403 targeted therapy, 402 vasodilator therapy, 402 Pulmonary immunity, 2305-2306, 2306f Pulmonary infections in asthma, 554 in bronchiectasis, 566, 567.e2f in cystic fibrosis, 565 HIV-associated, 2306-2315, 2310f staphylococcal, 1899-1900 treatment of, 565 Pulmonary Langerhans cell histiocytosis, 576, 586-587, 587.e1f Pulmonary lymphocyte alveolitis, 2237t Pulmonary mucormycosis, 2088, 2089f Pulmonary neoplasms, 1303-1313 Pulmonary nocardiosis, 2063, 2063f Pulmonary nodules definition of, 1306 imaging of, 536 solitary diagnostic imaging of, 1309 evaluation of, 1306f surgery for, 644 Pulmonary opacities, 532 alveolar patterns, 533 bronchial patterns, 533-534 large, 532, 532t linear patterns, 532 micronodular, 532 multifocal, 536, 536f nodular patterns, 532 reticular patterns, 532-533 small, 532-535, 532t, 532.e1f vascular patterns, 534-535 Pulmonary pressures, 279-280 Pulmonary rehabilitation, 560-561 Pulmonary risk assessment, 2615 Pulmonary sequestration, 570, 570f Pulmonary sporotrichosis, 2078-2079 Pulmonary stenosis, 416t Pulmonary stents, perioperative, 2613t Pulmonary thromboembolism, postoperative, 2622t Pulmonary toxicity, 1203 Pulmonary valve, 264 Pulmonary valve stenosis, 411 diagnosis of, 411 treatment of, 411b Pulmonary vascular resistance (PVR), 687t Pulmonary vascular resistance index (PVRI), 687t Pulmonary vasculature, 283f, 284, 285f, 556 Pulmonary vasculitis, 575t, 586 Pulmonary venous hypertension, 403 Pulmonary venous return, anomalous, 285f, 415-416 Pulmonic stenosis, 472-473 Pulse abnormalities of, 253t carotid, 26, 252-253, 253f Corrigan’s, 471 jugular venous, 252f low, 31 Quincke’s, 471 Pulse oximetry, 653, 697 Pulse pressure aging and, 382, 382f narrow, 303 Pulse pressure variation, 676 Pulsed Doppler, 275-276 Pulseless disease, 497 Pulseless electrical activity, 355, 355f Pulseless ventricular tachycardia, 354f PulseNet, 1918 Pulsus alternans, 303 Punique virus, 2259 Pupil(s) examination of, 2575-2576 large, 2560 small, 2560 white, 2560 Pupillary abnormalities causes of, 2576-2577 differentiation of, 2576, 2576f Pupillary control, 2575-2577

Pupillary pathways anatomy of, 2575 lesions in, 2575 Pure autonomic failure, 2518 Pure red cell aplasia, 1115 Pure tone testing, 2594 Purified protein derivative (PPD), 2035, 2041 Purine pathway inhibitors, 163t, 165-166 Purine synthesis inhibitors, 1776 Purple toe syndrome, 506 Purpura, 2641f, 2644, 2655, 2673 nonpalpable, 2673-2674 dermal causes of, 2673 systemic causes of, 2674 palpable, 2674-2675, 2674f periorbital, 1284, 1285f post-transfusion, 1167 Purpura fulminans, 2674, 2674f Purpuric eruptions, 2673-2675, 2673t Purpuric fever, Brazilian, 1947 Purulent conjunctivitis, 1947 Pustular eruptions, 2680-2681 Pustular folliculitis, eosinophilic, 2681 Pustular psoriasis, 2681 Pustules, 2636t, 2647t-2651t, 2650f, 2702t Pustulosis, exanthematous, 2681, 2681f, 2685t, 2686 Putamen hemorrahage, 2451, 2451f Puumala virus, 2249t PUVA (psoralen with ultraviolet A), 2661, 2670t Pyelonephritis differential diagnosis of, 1874, 1874t distinguishing characteristics of, 2252t-2253t treatment of, 1874-1875, 1875t Pyknodysostosis, 1670-1671, 1671b Pyoderma gangrenosum, 1740t, 1743, 2638t-2644t, 2642f Pyogenic arthritis in bacterial meningitis, 2485 with pyoderma gangrenosum and acne, 1740t, 1743 Pyogenic disorders, 1740t Pyogenic liver abscess, 1011-1012, 1011f clinical manifestations of, 1012 computed tomography findings, 1011f, 1012 definition of, 1011 diagnosis of, 1012 epidemiology of, 1011 pathobiology of, 1011-1012 prognosis for, 1012 treatment of, 1012b Pyogenic meningitis, 2480 Pyogenic skin ulcer, 2057t Pyomyositis, 1898 Pyramidal tract signs, 2455t Pyrantel pamoate, 2103, 2160t Pyrazinamide, 1891t-1892t, 2297t-2301t, 2312t Pyridostigmine for gastroparesis, 888 for myasthenia gravis, 2551 for orthostatic intolerance, 2521 Pyridoxine (vitamin B6), 1109t for acute poisonings, 706t-710t deficiency of, 1073, 2507t, 2509, 2509b functions of, 1454t for homocystinuria, 1407 toxicity of, 2509 Pyrilamine (Triaminic), 2660t Pyrimethamine for AIDS-associated opportunistic infections, 2297t-2301t for malaria, 2111 for Pneumocystis pneumonia prevention, 2098, 2098t for toxoplasmic retinitis, 2566 for toxoplasmosis, 2129t, 2132t Pyrimidine 5′-nucleotidase deficiency, 1086, 1087f Pyrimidine pathway inhibitors, 163t Pyrophosphate arthropathy, 1815 Pyropoikilocytosis, hereditary, 1054f, 1084 Pyruvate kinase deficiency, 1086 clinical manifestations of, 1086 diagnosis of, 1086 pathobiology of, 1086

Pyruvate kinase deficiency (Continued) peripheral blood smear features of, 1086, 1087f treatment of, 1086b

Q

Q, 2662t Q fever, 477t, 2055-2056, 2058t acute, 2055t chronic, 2055t clinical manifestations of, 2055-2056 definition of, 2055 diagnosis of, 2056 distinguishing characteristics of, 2252t-2253t epidemiology of, 2055 prevention of, 2056 symptoms and signs of, 2055, 2055t treatment of, 2056b Qi (chi), 182 Qi gong, 182t, 184 Qnasl (beclomethasone), 1692 QRS axis, normal, 268t QRS complex, 267-269, 268t QRS voltage, low, 273, 273t QT interval, 269 corrected (QTc), 269 antipsychotic-induced, 2626t Fridericia formula for, 269 normal, 268t prolonged, 2627 long QT syndrome, 187t, 373f long QT syndrome type 3 (LQT3), 341 Quality Check ( Joint Commission), 43.e1t Quality improvement, 45 Quality measures, 44-45, 44t Quality of care, 15-16, 44-46 definition of, 15, 44 disparities in, 15-16 Quality Positioning System (NQF), 43.e1t Quality-adjusted life years, 40 Quality-related problems, 44 QuantiFERON-TB Gold test, 2036 Quantitative sensory testing, 2528 Queensland tick typhus, 2048t, 2050 Quetiapine for dementia with Lewy bodies, 2396 for frontotemporal lobar degeneration symptoms, 2398 for Parkinson’s disease, 2457t-2459t for psychosis, 2354 Quinapril, 388t Quincke’s pulse, 471 Quinidine, 363t-364t for congenital myasthenic syndromes, 2553 for malaria, 2106, 2111, 2111t for ventricular tachycardia, 374 Quinine for babesiosis, 2144 for malaria, 2106, 2111, 2111t Quinlan, 6 Quinolones, 1889t, 1891t-1892t, 1894 for meningococcal disease chemoprophylaxis, 1938 resistance to, 1890t toxicities, 1895t for traveler’s diarrhea, 1884 Quinupristin, 1890t Quinupristin-dalfopristin, 1889t, 1891t1892t, 1896, 1901 QVAR (Beclomethasone dipropionate), 553t

R

Rabbit fever, 1982 Rabbit-derived polyclonal antilymphocyte serum, 844-845 Rabbitpox, 2219 Rabeprazole, 900t Rabies, 2502-2503 clinical manifestations of, 2503 diagnosis of, 2503 epidemiology of, 2502-2503 fatality rate, 2060t immunizations against, 66t, 68t-73t pathobiology of, 2502-2503 travel-related risk, 1881 treatment of, 2503b Rabies encephalitis, 2058t Volume I  pp 1-1382  •  Volume II  pp 1383-2722

I86

Index

Rabies vaccine, 76, 1882, 1883t adverse reactions to, 76 inactivated, 68t-73t indications for, 76 Raccoon eyes, 2365-2366 Racecadotril, 922, 2246 Racial differences in cardiovascular disease, 258 in hair and hair disorders, 2706 in hypertension, 382 in smoking, 17 Radiated cellular blood components, 1192 Radiation of hypothalamus, 1477 ionizing, 82-85, 1225 radiofrequency, 86 Radiation carcinogenesis, 84 Radiation cataractogenesis, 84 Radiation enteritis, 934, 948 Radiation exposure, 82, 1610, 1612t and aplastic anemia, 1118 electromagnetic, 81-82, 81t and leukemia, 1240 major sources of, 82, 83f medical, 82 non-ionizing radiation, 85-86 during pregnancy, 1610, 1612t stem cell toxicity, 1115 stochastic effects of, 84 testicular hypogonadism due to, 1573 ultraviolet, 85-86, 1225 Radiation injury, 82-86 clinical manifestations of, 83 diagnosis of, 84 epidemiology of, 82-83 lung injury, 600-601 clinical manifestations of, 600 definition of, 600 diagnosis of, 600 epidemiology of, 600 pathobiology of, 600 prognosis for, 601 treatment of, 601b neutropenia due to, 1135 pericarditis, 491 treatment of, 85b whole-body syndromes, 85 Radiation myelopathy, 2524t Radiation therapy, 85 adjuvant, 1354-1355 for brain tumors, 1291, 1295 for cancer, 1207-1208, 1302, 1310, 1312, 1330, 1350-1351, 1354-1355, 1368 for chronic lymphocytic leukemia, 1256 for CNS leukemia prophylaxis, 1243 for Cushing’s disease, 1491 for Cushing’s syndrome, 1517 for esophageal adenocarcinoma, 1316 late effects of, organ-specific, 84, 84t for lung cancer, 1309t, 1310 for metastatic Zollinger-Ellison syndrome, 1337 for multiple myeloma, 1281 for myelofibrosis, 1128 for pituitary tumors, 1484 for sarcomas, 1373 sequelae of, 85 stereotactic body, 1309t, 1310 Radiation-induced diarrhea, 922 Radicular pain, 2372 Radiculopathy cervical, 2378 definition of, 2377 lumbar, 2378 truncal, 2535 Radiculoplexus neuropathy, diabetic lumbosacral, 2535 Radinil (benznidazole), 2118 Radioactive iodine (131I), 1508 Radiocontrast agents, 782-783 Radiofrequency ablation of airway smooth muscle, 554 of atrial fibrillation, 380 catheter-based, 379 of supraventricular tachycardias, 380 tissue effects of, 379-380 of ventricular arrhythmias, 380 Radiofrequency radiation, 86 Radiography bedside, 531-532 chest. See Chest radiography

Radiography (Continued) in chronic obstructive pulmonary disease, 558, 558f in gallstones, 1040, 1040f in gastroenterology, 866-867, 866f of heart, 282-285, 282f-283f in osteoarthritis, 1748 Radiologic accidents, 82-83, 83t, 85 Radiologic terrorism, 83, 85 Radionuclide imaging in acute MI, 445 in gastroenterology, 872 in osteoarthritis, 1748 preoperative, 2617t of thyroid, 1502 Radium 223, 1369 Rales, 526 Raloxifene, 1358-1359, 1643t Raltegravir, 2288t for HIV/AIDS, 2289, 2290t for travel-related health problems, 1884 Ramelteon, 2421, 2421t Ramipril for acute MI, 450-451, 454-455, 454t for heart failure, 310t for hypertension, 388t Ramsay Hunt syndrome, 2591, 2599, 2606 Random sampling, simple, 32 Randomized Intervention for Children with Vesicoureteral Reflux, 827 Range of motion, 1718 Ranitidine (Zantac), 2660t for acid peptic disease, 915 for anaphylaxis, 1702 for esophageal disorders, 900t for pancreatic insufficiency, 565 for stress ulcers, 2368-2369 for urticaria, 1695, 2684t RANK (receptor activator of nuclear factor κB), 1277 RANKL (receptor activator of nuclear factor κB ligand), 1277 Ranolazine, 429 Rapid eye movement (REM) sleep, 2415, 2415t, 2415.e3f Rapid plasma reagin (RPR) test, 2017, 2017t Rapid response teams, 2608, 2609t Rapoport-Luebering shunt, 1085, 1085f Rasagiline, 2457t-2459t Rasburicase, 1243 Rash butterfly, 2641f drug rashes, 2684-2687 fever and, 1851, 1851t fifth disease, 2212, 2213f involving palms and soles, 1851, 1851t malar, 1771, 1771f measles rash, 2202, 2202f mite-induced, 2174 of monkeypox, 2216-2217, 2217f of rubella, 2205, 2205f zoonoses that cause, 2057, 2059t Ras/Raf/Erk pathway, 1228, 1228f Rastelli procedure, 414 Rates, 33 Rathke’s cleft cyst, 1476 Rathke’s pouch, 1480 Ratios likelihood, 37t statistical, 33 Rattlesnakes (Crotalus, Sistrurus), 719 Raxibacumab, 88b Raymond’s syndrome, 2435.e1f Raynaud’s phenomenon, 505, 509 differential diagnosis of, 1784 features suggestive of, 509t in Mycoplasma pneumonia, 2005 secondary, 509t in systemic lupus erythematosus, 1773 in systemic sclerosis, 1780-1782 treatment of, 509b, 510t, 1784-1785 underlying conditions associated with, 509t unilateral, 509f vascular therapy for, 1784-1785 Reactive airways dysfunction syndrome, 590 Reactive arthritis, 1766-1768 and chlamydia, 2010 definition of, 1766-1767 diagnosis of, 1767 differential diagnosis of, 1767 epidemiology of, 1767

Reactive arthritis (Continued) with HIV infection, 1767-1768 pathobiology of, 1767 poststreptococcal, 1909-1910 radiographic findings, 1766f-1767f, 1767 skin manifestations of, 1767, 1767f Reactive hyperplasias, 2583, 2583t Reactive nitrogen intermediates (RNIs), 233 Reactive oxygen intermediates (ROIs), 233 Ready-to-use therapeutic food (RUTF), 1436, 1437t Rebif (interferon β1a), 2477 Rebound nystagmus, 2579 Rebound tenderness, 858 Recall, immediate, 2383 Receiver operating characteristic (ROC) curve, 39, 40f Receptor activator of nuclear factor κB (RANK), 1277 Receptor activator of nuclear factor-κB ligand (RANKL), 1638, 1638f Receptor editing, 224 Receptors cutaneous, 2635-2637 in lipid transport, 1389, 1390t recognition, 217-218 Recidivans cutaneous leishmaniasis, 2123-2125 RECIST (Response Evaluation Criteria in Solid Tumors), 1209 Recognition receptors, 217-218 Recombinant tissue-type plasminogen activator (rt-PA, alteplase), 180, 447t for acute ischemic stroke, 2440-2441, 2441t-2442t, 2440.e1f Record keeping anesthesia records, 2621 postoperative care orders, 2621 Recreation, 1717 Rectal cancer adenocarcinoma, 1330-1331 MR staging, 871, 872f surgery for, 1330 Rectal examination, 28, 851 Rectal infection, 1943 Rectal prolapse, 971, 971f clinical manifestations of, 971 diagnosis of, 971 epidemiology of, 971 pathobiology of, 971 treatment of, 972b Rectal temperature measurement of, 26 normal ranges, 1849 Rectum, 967 Recurrent artery of Heubner, 2429f Recursive partitioning, 35 Red blood cell disorders, 2327 Red blood cell membrane, 1080-1084, 1080f Red blood cell membrane defects, 1080-1088 Red blood cell membrane lipids, 1080 Red blood cell membrane proteins, 1080f, 1081 integral, 1081 peripheral, 1081 Red blood cell transfusion, 1191-1193 for anemia, 1068, 1073, 1076, 1121 for aplastic anemia, 1121 clinical practice guidelines for, 1193, 1194t-1195t clinical trials, 1193t immunomodulatory effects of, 1192 for septic shock, 690 for sickle cell disease, 1103-1104 for upper GI bleeding, 880 Red blood cells anemia due to decreased production of, 1063-1066, 1063t antibody-mediated destruction of, 1074 Bartonella infection of, 1997-1998, 1998f components, 1192-1193 distribution width, 1060t dysmorphic, 730, 731f energy metabolism pathways, 1084, 1085f fragmentation of, 1053f isomorphic, 730, 731f metabolic defects, 1080-1088 metabolism of disorders of, 1086-1088 pathways, 1084, 1085f principal functions of, 1080 microcytic, 1053f

Red blood cells (Continued) normal values, 1060t normocytic, 1052f permeability defects, 1084 PNH II, 1079 PNH III, 1079 in urine, 730 Red cell agglutinates, 1055f, 1056 Red cell agglutination, 1055f Red cell antibodies, 1074 Red cell aplasia, chronic pure, 2213, 2213f Red cell fragments, 1053f, 1058 Red eye, 2558, 2559t Red (macular or maculopapular) lesions, 2581t, 2582-2583 Red man syndrome, 1699 5α-Reductase deficiency, 1564 5α-Reductase inhibitors, 830, 830t Redundancy, 2625 Re-entry, 342-343, 343f Refeeding syndrome, 1440, 1444 Reference equations, 540 Reference intervals, 2712 Reference Sequence (RefSeq), 190 Referral to behavioral change specialists, 54 to ENT specialists, 2589t for intimate partner violence, 1631 Referred pain, 2372-2373 Reflex seizures, 2406 Reflex sympathetic dystrophy, 1713t Reflexes baroreflex failure, 2519 examination of, 2371, 2372t H, 2342 let-down, 1487 vestibulo-ocular, 2412, 2412t, 2577 white or cat’s eye, 2572 Reflux esophageal, 249t, 898 gastroesophageal reflux disease, 897-902 endoscopy in, 874, 874f Montreal classification of, 898f preferred medications for women, 1602t treatment of, 898b-900b laryngopharyngeal, 2604-2606 vesicoureteral, 827 Reflux esophagitis, 874, 874f Refractoriness absolute, 341 relative, 341 Refsum’s disease, 1387t Regenerative medicine, 204-209, 208f Regional anesthesia, 2620 Regional collaboration, 1868 Regional rheumatic pain syndromes, 1749 Regional sympathetic dysfunction, 2522 Registries, 42 Regorafenib, 1331 Regression classification and regression trees, 35 multiple linear, 34 multiple logistic, 34-35 Regulators of complement activation, 241, 244, 246f Regulatory T cells, 223 Regurgitation, 852t, 899f Rehabilitation, 716 after acute MI, 455-456 for critically ill, 716 for injury, 716-717 pulmonary, 560-561 after stroke, 2444 Reifenstein’s syndrome, 1564 Reiki, 182t, 184 Reintegration, 716 Reiter’s syndrome, 1754 Rejection acute in liver transplantation, 1036, 1036f in renal transplantation, 845 treatment of, 845 cardiac antibody-mediated, 521t in cardiac transplantation, 521 cellular, 238, 521t chronic in renal transplantation, 845 treatment of, 845 effector mechanisms of, 238 in hematopoietic stem cell transplantation, 1203 hyperacute, 845

Index Rejection (Continued) in liver transplantation, 1036, 1036f lymphocytes involved in, 236t mechanisms of, 238-239 in renal transplantation, 845 strategies to prevent, 239-240 treatment of, 521, 845 Relapse-prevention (coping skills) therapies, 147 Relapsing fever, 2027-2028, 2051, 2057t clinical manifestations of, 2027 definition of, 2027 diagnosis of, 2027, 2027f distinguishing characteristics of, 2252t-2253t epidemiology of, 2027 louse-borne, 2060t pathobiology of, 2027 prevention of, 2028 prognosis for, 2028 treatment of, 2028b Relapsing meningitis, 2502t Relapsing polychondritis, 1827, 2605, 2605.e1f Relative measures, 33 Relative refractoriness, 341 Relative risk (RR), 33 Relative risk reduction (RRR), 33 Relaxation, isovolumic, 265 Relaxation response, 183 Relaxation therapy, 182t, 183-184, 2421t REM (rapid eye movement) sleep, 2415, 2415t, 2415.e3f REM sleep behavior disorder, 2396, 2423, 2423t Remicade (infliximab), 169, 1762, 2660-2661 Remodeling balance, 1646 Renal abnormalities hypoparathyroidism, deafness, and renal anomalies syndrome, 1652t, 1660 in sickle cell disease, 1101, 1101t structural, 825-827 Renal agenesis, 824-825 Renal allograft dysfunction causes of, 846, 847t evaluation of, 846, 847t Renal arteriography, 736 Renal arteriole disorders, 810 Renal artery disease, atheroembolic, 810, 810b Renal artery occlusion clinical manifestations of, 809 diagnosis of, 809 epidemiology of, 809 pathobiology of, 809 prognosis for, 809-810 thromboembolic, 809-810 treatment of, 809b Renal artery stenosis, 807-809 atherosclerotic, 385-386, 386f bilateral, 808f clinical manifestations of, 807-808 definition of, 807 diagnosis of, 385-386, 807-808 epidemiology of, 807 MRA findings, 807-808, 808f pathobiology of, 807 prognosis for, 808-809 treatment of, 808b Renal artery stents, 808, 808.e1 Renal artery thrombosis, 809-810, 809b Renal biopsy, 735 indications for, 796t for nephritis, 796, 796t Renal bone disease, 835-836, 840 Renal cell carcinoma, 1345-1348 associated syndromes, 1346t classification of, 1345, 1345t clinical manifestations of, 1346 definition of, 1345 diagnosis of, 1347 epidemiology of, 1345-1346 genetics of, 1346t histologic subtypes, 1346t localized disease, 1347 metastatic disease, 1347-1348 pathobiology of, 1346 prognosis for, 1348, 1348f symptoms and signs of, 1347t

Renal cell carcinoma (Continued) TNM staging of, 1347t treatment of, 1347b-1348b Renal colic evaluation of, 812, 813f medical therapy for, 813 Renal crisis, scleroderma, 1783, 1785 Renal cysts formation of, 816, 818f simple cysts, 816, 816f, 817t Renal dialysis, 690 Renal disease, 728-736, 729f acute. See Acute kidney injury associated pulmonary diseases, 790t chronic kidney disease, 833-841 diagnosis of, 385t hypertension with, 392 indications for dialysis, 841, 841t preconception interventions for women, 1612t in pregnancy, 1614t in systemic sclerosis, 1783 vicious cycle with AKI, 782, 782f with cyclosporine, 167 cystic, 816-822 diagnosis of, 728-732 drug dose adjustments in, 128-129 end-stage anemia in, 842 definition of, 833 epidemiology of, 783, 804 life expectancy in, 843, 843f metabolic bone disease in, 842 secondary to diabetes, 804 high-penetrance monogenic disorders, 187t monitoring progression of, 819 pathobiology of, 728 polycystic kidney disease adult, 187t autosomal dominant, 816-820, 816f, 817t, 818f-819f, 819b-820b autosomal recessive, 816, 816f, 817t, 819f, 820-821, 821b definition of, 816 signaling pathways in, 820.e1f therapeutic interventions for, 820.e1f preoperative evaluation of, 2617 prerenal injury, 733 in sarcoidosis, 606 secondary to diabetes, 804-806 in sickle cell anemia, 1103 treatment of, 1103 vascular disorders, 735-736, 807-811 Renal dysplasia, 825 Renal ectopia, 825-826 Renal failure, 736 acute. See Acute kidney injury chronic. See Chronic kidney disease drug dosage adjustments in, 128, 129t irreversible, 841-847 in multiple myeloma, 1278 Renal glycosuria, 823t, 1386t Renal hypoplasia, 825 Renal hypouricemia, hereditary, 1386t Renal impairment moderate, 624 severe, 624 Renal infarction, 809, 809f Renal insufficiency anemia of, 1065 antipsychotic-induced, 2626t drug therapy in considerations for, 128-129 loading dose, 128 in multiple myeloma, 1278, 1281 Renal microvasculature, 810 Renal osteodystrophy, 835 Renal parenchymal hypertension, 384-385 Renal parenchymal malformations, 824-825 Renal Pathology Society, 1772, 1772.e1t Renal pelvis cancer, 1350t, 1351 Renal replacement therapy, 806, 841 indications for, 841, 841t medical issues, 842-843 prevalence of, 841 Renal sodium retention, 746, 746t primary, 746-747 secondary, 747 Renal toxins, 779

Renal transplantation, 843-847 allocation of, 844 candidates for, 844 complications of, 845-846 deceased donor, 843.e1f demographics, 843 epidemiology of, 843, 843.e1f immunosuppression for, 844-845, 844t living donor, 846, 843.e1f management after transplantation, 846-847 prognosis for, 846-847 rejection of, 845 acute, 845 chronic, 845 hyperacute, 845 success rates, 843 survival rates, 843, 843f waiting time for, 843, 843.e1f Renal tuberculosis, 2035 Renal tubular acidosis causes of, 770t clinical manifestations of, 770-771 diagnosis of, 770-771, 771f distal, 770, 823t-824t, 824 hyperkalemic, hyperchloremic, 770 inherited, 824t prognosis for, 771 proximal, 823t-824t treatment of, 771b type 1 (hypokalemic distal), 769-770, 770t, 1386t type 2 (proximal), 769-770, 770t, 1386t type 4 (hyperkalemic), 770t Renal tubular toxins, 779t Renal urate underexcretion, 1812 Renal vein thrombosis, 810-811, 810b-811b Renin inhibitors for hypertension, 388t, 390 mechanism of action, 390 side effects of, 390 Renin-angiotensin system inhibitors, 390 Renin-angiotensin-aldosterone system inhibitors, 450-451 Renova (all-trans-retinoic acid), 2658 Renovascular disease, 385t Renovascular hypertension, 385-386 ReoPro (abciximab), 180 Reovirus, 212 Reperfusion therapy for acute MI, 446-448, 446f, 446t for cardiogenic shock, 683-684 for pulmonary embolism, 624 Repetitive stimulation study, 2343 Repetitive use syndromes, 1749 Replens, 1627 Repolarization abnormalities, 273, 273t Reproductive endocrinology, 1584-1597 Reproductive health and cancer, 1225 and iron deficiency anemia, 1069 issues with cyclophosphamide, 168 issues with cyclosporine, 167 Reproductive health care for adolescents, 63-64 for women, 1604 Reproductive hormones, 1591 Reproductive rate (Ro), 1877 Reproductive tract cancer of, 1608 development of, 1561, 1561f, 1570-1571 male, 1568-1569, 1568f Reproductive years: abnormalities of, 1586-1591 Reprogramming, direct, 207-208 Reptilian stare, 2455 Research clinical trials, 35, 1208 selecting subjects for studies, 32 Reserpine for chorea, 2463 for hypertension, 388t Residual volume, 540, 542f Resilience, 1839-1840 Resource-limited settings clinical practice in, 1113-1114 prophylaxis in, 1113 Resources community, 49 health care resource measures, 41, 42t for molecular technologies, 203.e2t

I87

Respirations assessment of, 26 definition of, 655 Respiratory acidosis, 773 clinical manifestations of, 773 compensatory changes in, 765t diagnosis of, 773 treatment of, 774b Respiratory alkalosis, 774 clinical manifestations of, 774 compensatory changes in, 765t, 766 diagnosis of, 774 epidemiology of, 774 pathobiology of, 774 treatment of, 774b Respiratory bronchiolitis–-associated interstitial lung disease, 582, 582b, 582f Respiratory burst, 1146 Respiratory chain complexes, 1387 Respiratory chain enzyme deficiency, 1385t characteristic signs of, 1385t pathophysiologic mechanisms of, 1385t Respiratory compliance measurement of, 654 static (Crs), 654 Respiratory diphtheria, 1916-1917 Respiratory disease, 526-531 antipsychotic-induced depression, 2626t aspirin-exacerbated, 554, 1691, 1691t enteroviral syndromes, 2243 in immunocompromised patients, 1859t, 1860 occupational, 80, 80t, 588-595, 589t in sarcoidosis, 604-605, 604f-605f Respiratory disturbance index, 639 Respiratory drive, 654 Respiratory dysfunction, 655 Respiratory failure acute, 655-659 definition of, 655 hypoxemic, 600 Respiratory function, 539-545 mechanisms of, 539 testing, 539 Respiratory monitoring, 652-655 Respiratory muscle strength, 655 Respiratory rate decreased, 31 incrased, 31 normal, 652 Respiratory syncytial virus, 2185t, 2187 Respiratory syncytial virus infection, 2187-2188 clinical manifestations of, 2187 definition of, 2187 diagnosis of, 2187 epidemiology of, 2187 pathobiology of, 2187 prevention of, 2188 prognosis for, 2188 treatment of, 2187b-2188b Respiratory system age-related changes in, 107 physical examination of, 652 physiologic abnormalities of, 659 Respiratory tract infection, 556 adenoviral, 2198 group A streptococci (GAS), 1906-1907 health care–associated infections, 1865 M. pneumoniae, 2004, 2004f Pseudomonas, 1965 S. maltophilia, 1970 treatment of, 661 Respiratory tract zoonoses, 2057, 2058t Resting membrane potential, 339-340 Resting metabolic rate (RMR), 1460-1461 Resting tremor, 2454-2455 Restless legs syndrome, 2423-2424, 2468 clinical manifestations of, 2468 diagnosis of, 2468 epidemiology of, 2468 pathobiology of, 2468 treatment of, 2468b-2469b Restrictive cardiomyopathy, 321t, 332-334 causes of, 334t clinical manifestations of, 333 definition of, 332 diagnosis of, 333-334 differential diagnosis of, 490t epidemiology of, 332 Volume I  pp 1-1382  •  Volume II  pp 1383-2722

I88

Index

Restrictive cardiomyopathy (Continued) idiopathic, 334f overlap with dilated cardiomyopathy, 331 pathobiology of, 332-333 prognosis for, 334 treatment of, 334b Restrictive lung disorders, 542, 543t, 544 Resuscitation burn resuscitation formula, 714.e1t fluid, 650, 714-715 initial, 713-715 post-resuscitation care, 355, 716 Retapamulin, 2695-2696 Reteplase (r-PA), 180, 447t Reticular activating system, 2415.e3f Reticulated hyperpigmentation, 2700 Reticulocyte count, 1060t, 1062 normal values, 1060t Reticulohistiocytosis, multicentric, 1826 Retina, 2556 Retin-A (all-trans-retinoic acid), 2658 Retinal degeneration, pigmentary, 2574 Retinal detachment, 2567, 2574 Retinal tears, 2574 Retinitis cytomegalovirus, 2230, 2231f, 2565-2566 AIDS-associated, 2297t-2301t antiviral therapy for, 2181t toxoplasmic, 2566 Retinitis pigmentosa, 195, 2559, 2571, 2571f Retinoblastoma, 1227t, 2571-2572 Retinoblastoma protein, 1229, 1229f, 1229t Retinochoroiditis, toxoplasmic, 2127, 2128f Retinoic acid, 2700 all-trans-Retinoic acid (tretinoin, retin-A, Renova), 2658 for acute promyelocytic leukemia, 1245 for cancer, 1211t-1216t for leukemia, 1243t Retinoic acid−inducible gene I (RIG-I), 217 Retinoids, 2658 Retinol (vitamin A), 1446t-1450t, 1454t, 2573t Retinol (vitamin A) deficiency, 2507t, 2509 Retinol (vitamin A) supplements for diarrhea, 926 for malabsorption, 931t for retinitis pigmentosa, 2571 for vitamin A deficiency, 2507t Retinopathy of prematurity, 2571 Retrobulbar neuritis, 2574-2575 Retrolental fibroplasia, 2571 Retropharyngeal abscess, 528t Retroviruses, 2235-2239 Rett’s syndrome, 2514 Reverse cholesterol transport, 1392, 1392f Reversed figure-of-3 signs, 285 Review of systems, 25, 25t Revlimid (lenalidomide), 1211t-1216t Reward circuits, 144-145, 144f Rhabdomyolysis, 723-726, 2539 acquired, 723-724, 723t antipsychotic-induced, 2626t classification of, 724 clinical manifestations of, 725 definition of, 723 diagnosis of, 725 differential diagnosis of, 725 drug-induced, 723-724 epidemiology of, 723 etiology of, 723-724, 723t exertional, 723-724, 724t in infections, 724 inherited, 723-724, 723t in intoxications, 723 laboratory tests in, 725 pathobiology of, 723-724 pathophysiology of, 723-724, 723.e1f prevention of, 725 prognosis for, 726 toxin-induced, 724, 2546t traumatic, 724 treatment of, 725b-726b, 726 Rhabdomyolysis-induced acute kidney injury, 725, 726t rhDNase, 565 Rheumatic diseases, 1712-1718 associated with rheumatoid factor, 1720t autoimmune patterns of onset, 1715 worldwide prevalence and incidence of, 1713t

Rheumatic diseases (Continued) categorization of, 1712 classification of, 1712, 1713t, 1718 clinical manifestations of, 1712-1716 cutaneous manifestations of, 1713-1714 definition of, 1712 degenerative, 1714-1715 diagnosis of, 1715t, 1717-1718 diagnostic evaluation of, 1716-1718 differential diagnosis of, 1716-1718 epidemiology of, 1712 functional assessment in, 1717 histopathology of, 1712 imaging, 1718, 1723-1730 joint involvement, 1713, 1717 laboratory testing, 1718-1723 musculoskeletal findings, 1718 nonspecific clinical presentations, 1713 pain assessment in, 1717 pattern of onset, 1714-1716 physical examination in, 1717-1718 prognostic factors, 1717 radiographic findings, 1723, 1724f, 1724t seronegative, 1712 symptoms and signs of, 1717 treatment of, 1718b vascular, 1713t, 1715t Rheumatic fever, acute, 1908-1909 Rheumatic pain syndromes, regional, 1749 Rheumatic valvular heart disease, 461 Rheumatism desert, 2072 nonarticular, 1749 Rheumatoid arthritis, 1754-1762 articular manifestations of, 1757-1758, 1757f associated diseases, 1762 cardiovascular involvement, 1758, 1758t, 1762 classification of, 1713t, 1759-1760, 1759t, 1760.e1f clinical course, 1759 clinical manifestations of, 1757-1759 definition of, 1754-1755 diagnosis of, 1759-1760 differential diagnosis of, 1760, 1760t, 1764t distribution of, 1757, 1757f, 1759 epidemiology of, 1755 etiology of, 1755, 1755f extra-articular manifestations of, 1758-1759, 1758t in feet, 1757 genetics of, 1755 gut microbiota and, 1842 in hands, 1756-1757, 1756f hematopoietic stem cell transplantation for, 1203 HLA associations, 1755, 1755.e1t incidence of, 1755 inflammation pathways in, 230.e3t initiation of, 1755, 1755f in knees, 1757, 1757f laboratory findings, 1760 in large joints, 1757-1758 in neck, 1758 neurologic manifestations of, 1759, 1756.e1f neuropathy in, 2533, 2533t ocular manifestations of, 2570 ophthalmologic manifestations of, 1758t, 1759 pathobiology of, 1755-1756 pathogenesis of, 1756, 1756f pathology of, 1756, 1756f pleural fluid characteristics, 634t prevalence of, 1755 prognosis for, 1762 pulmonary manifestations of, 584-585, 1758t, 1759, 1759f radiographic features of, 1724f, 1724t, 1727f in skin, 1758, 1758f treatment of, 1760b-1762b future directions, 1762 general measures, 1760 keys to optimize outcome, 1760t medical therapy, 1760-1762 order of therapy, 1762 treatment of underlying conditions, 1762 in wrists, 1757

Rheumatoid factor, 1719 in glomerular syndromes, 734 nonrheumatic conditions associated with, 1720t in pleural fluid, 634 rheumatic diseases associated with, 1720t Rheumatoid nodules, 1758, 1758f Rheumatoid nodulosis, 1758, 1758f Rheumatoid synovitis, 1756, 1756f Rheumatologic agents. See also specific agents perioperative, 2613t Rheumatologic paraneoplastic syndromes, 1221 Rhinitis, 2586-2588 acute purulent, 2588 allergic, 1687-1691, 2588 approach to, 1690f clinical manifestations of, 2586 cultures, 2587 definition of, 2586 diagnosis of, 2586-2588 diagnostic tests, 2587 differential diagnosis of, 1690t, 2587-2588 epidemiology of, 2586 history in, 2586 imaging in, 2587 infectious, 2588 laboratory findings, 2587 pathobiology of, 2586 perennial, 1674 physical findings, 2586-2587 seasonal, 1674 surgical therapy for, 2588 treatment of, 2588b Rhinitis medicamentosa, 1690t Rhinocerebral mucormycosis, 2088 Rhinocort (budesonide), 1692 Rhinopyma, 2680, 2680f Rhinorrhea, 2185-2186 Rhinoscopy, 2586, 2587f Rhinosinusitis acute purulent, 2588 bacterial, 2589t group A streptococci (GAS), 1907 infectious, 2588 laboratory findings, 2587 when to refer to an ENT specialist, 2589t Rhinoviruses, 2185, 2185t Rhizomelic chondrodysplasia punctate type I, 1387t Rhodococcus pneumonia, 2058t Rhombencephalitis, listerial, 1919 Rhomboid glossitis, median, 2582 Rhonchus, 526 Rhythm disturbances, 451-452 Rhythmic movement disorder, 2423, 2423t, 2461t Riastap (CSL Behring), 1179 Ribavirin, 2178t, 2180-2181, 2184 adverse effects of, 1004, 2178t for hepatitis C, 998, 1004, 2177t for hepatitis E, 1005 for pretransplantation chronic liver failure treatment, 1034 for respiratory syncytial virus infection, 2187-2188 for viral hemorrhagic fever, 2255t Riboflavin (vitamin B2), 1446t-1450t, 2545 Ribonucleoproteins, small nuclear (snRNPs), 1722 Rib-pelvis distance, 1636, 1636f Rickets, 1446t-1450t, 1645-1649 definition of, 1645 familial hypophosphatemic, 1386t Rickettsia, 2047, 2047t culture of, 2049 Rickettsia akari, 2052 Rickettsia felis, 2051, 2057 Rickettsia parkeri, 2057 Rickettsia prowazekii, 2046-2047 Rickettsia sennetsu, 2053 Rickettsial infections, 2046-2056, 2048t definition of, 2046-2047 diseases, 2047-2052 spotted fever, 2059t Rickettsiales, 2046-2047, 2047t Rickettsialpox, 2048t, 2052, 2059t clinical manifestations of, 2052 diagnosis of, 2052 epidemiology of, 2052 treatment of, 2052b vectors, 2172t

Rickettsioses, 2046-2052 definition of, 2047 lymphangitis-associated rickettsiosis (LAR), 2048t tick-borne, 2047-2051, 2050f Riedel’s thyroiditis, 1503, 1509 Rieger’s syndrome, 1476 Rifabutin for AIDS-associated opportunistic infections, 2297t-2301t for H. pylori, 915t for HIV and TB co-treatment, 2038, 2312t Rifampicin, 1975, 2536 Rifampin, 1891t-1892t for Acinetobacter, 1970 for AIDS-associated opportunistic infections, 2297t-2301t for anthrax exposure, 88b for bacillary angiomatosis, 2001 for bacterial meningitis, 2487, 2488t, 2489 for brucellosis, 1981, 1981t drug interactions, 1846 for endocarditis, 479t-480t, 480 for H. influenzae infection prevention, 1948 for legionnaires’ disease, 1996t for leprosy, 2045-2046 for M. kansasii lung disease, 2042 mechanism of action, 1889t for meningococcal disease chemoprophylaxis, 1938, 1939t for primary sclerosing cholangitis, 1045-1046 resistance to, 1890t for S. aureus infection prevention, 1901 for spinal epidural abscess, 2497 for tuberculosis, 2312t Rifapentine, 2297t-2301t Rifaximin for acute liver failure, 1033 for bacterial overgrowth, 928 for bloating and distention, 861 for diarrhea, 922 for hepatic encephalopathy, 1030 for irritable bowel syndrome, 893t, 894 Rift Valley fever, 2249t, 2259 clinical manifestations of, 2250-2251, 2251t, 2259 definition of, 2259 diagnosis of, 2259 epidemiology of, 2259 management of, 2254-2255 pathobiology of, 2251t, 2259 prognosis for, 2256, 2260 treatment of, 2260b vaccine for, 2255-2256 Rift Valley fever virus, 91, 91t, 2247, 2249t Right atrial pressure, 279-280 Right bundle branch block, 270-271, 271f-272f, 271t Right coronary artery, anomalous, 287, 287f Right coronary artery stenosis, 296f Right ventricular cardiomyopathy, 321t, 331-332 Right ventricular enlargement, 283, 284f Right ventricular hypertrophy, 273 Right ventricular infarction, 444, 452 Right ventricular outflow tract obstruction, 411-412 Rigidity, 2455 RIG-I-like receptors (RLRs), 217 Riley-Day syndrome, 2519 Rilonacept (Arcalyst), 169-170, 1148 Rilpivirine, 2288t, 2289, 2290t Riluzole, 2463, 2525 Rimantadine, 2183t-2184t, 2184, 2196-2197, 2196t Ringer’s solution, lactated, 680t, 714.e1t Rings and webs esophageal, 905-906 muscular, 905 Rinne test, 2594 Riociguat, 402 Rippling muscle disease, 2542t, 2545 Risedronate (Atelvia) for osteoporosis, 1642-1643, 1643t for Paget’s disease, 1667, 1667t Risk absolute risk reduction (ARR), 33 relative risk (RR), 33 relative risk reduction (RRR), 33

Index Risk assessment cardiac, 2612-2615 noninvasive tests, 2612-2614 perioperative, 2613t pharmacologic stress testing, 2612-2614 risks of surgical procedures, 2613t cardiac risk indices, 2612 components of, 54 before electroconvulsive therapy, 2629, 2629f genetic, 188 operative, 2611 periodic health examination, 54 preoperative, 2611-2612 pulmonary, 2615 Risk difference, 33 Risk estimates, 200 Risk factors, 24-25, 54-55 Risk ratio (RR), 33-34 Risk stratification, 426-427 for decision making, 426t after myocardial infarction, 453-454 Risperidone, 2354, 2466 Rituximab (Rituxan or MabThera), 171-172, 2288t for acute allograft rejection in liver transplantation, 1036 for ANCA-associated vasculitis, 1801 for cancer, 1211t-1216t for chronic lymphocytic leukemia, 1256 CSF-to-plasma ratios, 2331t fixed-dose combinations, 2289, 2290t for follicular lymphoma, 1264 for Goodpasture’s syndrome, 574 for granulomatosis with polyangiitis, 1798 for hemophilia, 1177 for HIV/AIDS, 2289, 2290t for immune thrombocytopenic purpura, 1164, 1164t for membranous nephropathy, 787 for minimal change disease, 785-786 for multicentric Castleman’s disease, 2326 for multifocal motor neuropathy, 2532 for multiple sclerosis, 2477 for non-Hodgkin’s lymphoma, 1263, 1263t for pemphigus, 2677 for rapidly progressive glomerulonephritis, 791 for refractory ITP, 1164t for rheumatoid arthritis, 1762 side effects of, 2289 for systemic lupus erythematosus, 1776-1777 for Waldenström’s macroglobulinemia, 1282 for warm autoimmune hemolytic anemia, 1077-1078 Rivaroxaban, 178, 179t for anticoagulation, 367 drug interactions, 178t in pregnancy, 1618t preoperative, 2616, 2617t for pulmonary embolism, 624-625, 625t for thromboembolic prophylaxis, 366t for venous thromboembolism, 517-518 Rivastigmine, 2394, 2457t-2459t River blindness, 2169 Rizatriptan, 2358-2359, 2359t RNA, 189 long noncoding RNA (lncRNA), 189 messenger RNA (mRNA), 2283-2284 micro-RNA (miRNA), 189, 1228 small, EBV-encoded, 1268 small nucleolar RNA (snoRNA), 189 RNA interference (RNAi), 213 Robertsonian translocations, 193 Robust standard errors, 36 Rochagan (benznidazole), 2118 Rocio encephalitis, 2268 Rock of Gibraltar sign, 537, 537f Rockall scoring system, 880, 881t, 916t Rocker bottom feet, 1824 Rocky Mountain spotted fever, 2047-2050, 2048t, 2057-2058, 2059t, 2174 clinical manifestations of, 2048-2049 diagnosis of, 2049 distinguishing characteristics of, 2252t-2253t epidemiology of, 2047-2048 fatality rate, 2060t

Rocky Mountain spotted fever (Continued) prevalence of, 2047, 2049f prevention of, 2049 prognosis for, 2049-2050 treatment of, 2049b vectors, 2172t Rodent-borne viruses, 2247 Roferon (interferon-α), 1211t-1216t Roflumilast, 559, 560t Rogorafenib (Stivarga), 1211t-1216t Rolandic epilepsy, benign, 2406 Romana sign, 2176 Romano-Ward syndrome, 405 Romhilt-Estes criteria, 273 Romidepsin (Istodax), 1211t-1216t, 2670t Romiplostim, 1164 Ronson criteria, 963 Ropinirole, 2424, 2457t-2459t, 2468-2469 Rosacea, 2680 Rosai-Dorfman disease, 1267 Rosenmüller: fossa of, 2587 Roseola (exanthem subitum), 2672-2673 Ross procedure, 472 Ross River fever, 2261 clinical manifestations of, 2261 diagnosis of, 2261 epidemiology of, 2261 pathobiology of, 2261 treatment of, 2262b Ross River virus, 2261 Ross syndrome, 2519 Rosuvastatin for acute kidney injury prevention, 782-783 for atheromatous embolization, 507 for lipid disorders, 1395-1397, 1396t for stroke prevention, 2444-2445 Rotarix (RV1, GlaxoSmithKline), 2246 Rotashield (Wyeth-Lederle), 2246 RotaTeq (RV5, Merck), 2246 Rotator cuff tears, 1751-1752 Rotator cuff tendinitis, 1751 Rotavirus, 2244 Rotavirus disease clinical features of, 2245t clinical manifestations of, 2246 diagnosis of, 2246 epidemiologic features of, 2245t epidemiology of, 2245 pathobiology of, 2245 prevention of, 2246 treatment of, 2246b vaccination against, 2246 Rotavirus vaccines, 2246 Rothman-Thompson syndrome, 1370 Roth’s spots, 2569, 2569f Rotigotine, 2424, 2457t-2459t, 2468-2469 Rotor’s syndrome, 987t Rouleaux, 1055f Rounded atelectasis, 589t Roundworms (nematodes) intestinal, 2103, 2159-2164 systemic, 2103-2104 Roussel-Uclaf Causality Assessment Method (RUCAM), 1009t, 1010 Roux-en-Y gastric bypass, 1466 Rove beetle dermatitis, 2176 Rove beetles, 2176 Rowasa (mesalamine), 939t RR interval, 268 Rubella (German measles), 2204-2206, 2672 clinical manifestations of, 2205 congenital, 2205 diagnosis of, 2205-2206 pathobiology of, 2205 definition of, 2204 diagnosis of, 2205-2206 distinguishing characteristics of, 2252t-2253t epidemiology of, 2204-2205 immunization against active, 2206 passive, 2206 pathobiology of, 2205 postnatally acquired clinical manifestations of, 2205, 2205f diagnosis of, 2205 in pregnant women, 2205 prevention of, 2206

Rubella (German measles) (Continued) prognosis for, 2206 treatment of, 2206b Rubella panencephalitis, progressive, 2210 Rubella vaccine, 68t-73t, 76 adverse reactions to, 76 indications for, 76 live virus, 68t-73t Rubella virus, 2204 Rubella-containing vaccine (RCV), 2204-2205 Rubeola, 2580-2581, 2580t Rubex (doxorubicin), 1211t-1216t Runner’s diarrhea, 921 Rupial lesions, 2015 Rush (shock) examination protocol, 675, 675t Russell’s viper, 718 Russian spring-summer encephalitis, 2263t, 2268 Rutherford’s clinical classification, 499t Ruxolitinib ( Jakafi), 1128, 1211t-1216t RV1 (GlaxoSmithKline), 2246 RV5 (Merck), 2246 rWN/DEN4Δ30, 2267 Ryanodine receptor (RYR1) gene, 724

S

SA14-14-2, 2266 Sabiá virus, 2247, 2249t Saccades, 2577 Saccular aneurysms, 2447, 2447f Saddle nose deformity, 1797 Safe sex, 2285 Safety blood, 1195-1198 and cost, 46 culture of, 46 patient, 45-46 Safety factor, 341 Sagittal sinus thrombosis, septic, 2500 Salbutamol, 2553 Salgo v. Leland Stanford Junior University Board of Trustees, 6-7 Salicylate intoxication, 699t-703t, 710t, 768 clinical manifestations of, 768 diagnosis of, 768 epidemiology of, 768 pathobiology of, 768 prognosis for, 768 Salicylazosulfapyridine (Salazopyrin), 1088t Saline lift polypectomy, 1327.e1 Saline solution for acute kidney injury, 726t contraindications to, 2452 half-isotonic, 745 for hypercalcemia, 1654 for hypermagnesemia, 776 hypertonic, 1033 for intracranial hypertension, 1033, 2368, 2452t isotonic, 745 for renal colic, 813 for shock, 680t Salivary duct obstruction, 2583t Salivary gland cancer, 1298, 1313 Salivary gland diseases, 2579-2585 primary, 2584 secondary, 2584-2585 treatment of, 2585b Salivary gland enlargement bilateral, 2584-2585 causes of, 2584, 2584t Salivary gland tumors, 1302-1303, 2584 Salivary secretion, decreased associated with systemic diseases, 2584-2585 causes of, 2584, 2585t drugs and toxins that cause, 1787-1788, 1788t without gland enlargement, 2585 Salla disease, 1399-1400 Salmeterol (Serevent) for altitude sickness, 597t for asthma, 553 for chronic obstructive pulmonary disease, 560t for high-altitude pulmonary edema prevention, 596-597 Salmonella, 1971

I89

Salmonella enterica subspecies enterica, 1971 Salmonella infections, 1971-1975 AIDS-associated, 2295, 2297t-2301t antimicrobial resistance of, 1972 asymptomatic carriers, 1973 clinical features of, 1869t, 1973 diagnosis of, 1973-1974 distinguishing characteristics of, 2252t-2253t epidemiology of, 1869t, 1971-1972 etiology of, 1972 laboratory findings, 1871f pathobiology of, 1972-1973 pathophysiology of, 1972-1973 prevention of, 1975 prognosis for, 1975 treatment of, 1974b-1975b, 2295, 2297t-2301t treatment of carriers, 1974-1975 Salt, dietary, 2521 Salt restriction for ascites, 1030 for chronic kidney disease, 838-839 Salt substitutes, 314 Salt-wasting syndromes, 383 Sample means, 35 Sampling, 32 actual sample, 32 describing samples, 32-35 exclusion criteria for, 32 inclusion criteria for, 32 inferring population values from samples, 35-36 intended sample, 32 random, simple, 32 selecting subjects for studies, 32 Samter’s triad, 1691, 1691t Sandfly fever, 2259 Sandfly fever virus, 2259 Sandostatin (octreotide), 1211t-1216t Sanfilippo’s syndrome, 1387, 1734t Sanjad-Sakati syndrome, 1652t, 1661 Sapovirus, 2244 Sappinia, 2142, 2142t Saquinavir, 2288t, 2331t Sarcocystis, 2147t Sarcoidosis, 334, 603-608, 2639f, 2645f bone and joint involvement, 606 calcium metabolism in, 606 cardiac disease, 291f, 606 cardiomyopathy with, 331 central nervous system involvement, 1477 chest CT findings, 604, 604f-605f chest radiography findings, 604, 604f chest radiography staging, 604, 604t clinical manifestations of, 604-606 cutaneous, 2691, 2691f definition of, 603 diagnosis of, 606-607 diagnostic criteria for, 607 early-onset, 1740t epidemiology of, 603 eye disease, 605-606, 2571 hepatic, 606, 1018, 1018t, 1019b in HIV infection, 2335 immunopathogenesis of, 603-604, 603.e3f incidence of, 603 joint manifestations of, 1826, 1827f laryngeal involvement, 2605, 2605.e2f neurologic disease, 606 pathobiology of, 603-604 prognosis for, 607 renal involvement, 606 respiratory system disease, 604-605, 604f-605f skin disease, 605, 605f small intestinal ulcers in, 948 splenic, 606 treatment of, 607b, 607t type 1 interferon therapy interactions, 607 Sarcomas, 1371-1373 clinical manifestations of, 1372 definition of, 1371 diagnosis of, 1372 epidemiology of, 1371-1372 Ewing’s, 1371 Kaposi’s, 2325-2326, 2690 lung, 1313 mast cell, 1709 osteosarcoma, 1370, 1370b

Volume I  pp 1-1382  •  Volume II  pp 1383-2722

I90

Index

Sarcomas (Continued) pathobiology of, 1372 prognosis for, 1373 treatment of, 1372b-1373b uterine, 1362-1363 Sarcomeres, 263.e1f Sarcopenia, 2547 Sarcoptes mites, 2173 Sarcoptes scabiei variant hominis, 2173 Sargramostim (Leukine, Prokine), 1211t-1216t Sarin exposure, 706t-710t SARS (severe acute respiratory syndrome), 2056-2057, 2057t SARS (severe acute respiratory syndrome) coronavirus infection, 2199-2201 clinical manifestations of, 2200, 2200t prevention of, 2201 treatment of, 2201 Satellite cells, 209 Satiation, 1460 Satiety, 1460 Saturated solution of potassium iodide (SSKI), 1508, 2078-2079 Saturday night palsy, 2512 Saturnine gout, 1814 Savage’s syndrome, 1591 Saw-scaled viper, 718 Saxagliptin, 314 Scabies, 1877t, 2173, 2173f, 2657f, 2698, 2698f clinical manifestations of, 2173 definition of, 2173 diagnosis of, 2173 in HIV infection, 2320 identification of, 2656 pathobiology of, 2173 treatment of, 2173b Scalded skin syndrome, staphylococcal, 1897, 1900, 1900f Scales, 2636t, 2644, 2647t-2651t, 2656, 2660 Scalp disorders, 2703t Scalp eschar, neck lymphadenopathy after tick bite (SENLAT), 2048t, 2050-2051 Scalp lesions, 2702t Scalp pain, 2703 Scalp psoriasis, 2703t Scandinavians, 1399-1400, 1529-1530 Scarlatiniform eruptions, 2671-2672, 2671t Scarlet fever, 1907, 2671 Scarpa’s ganglion, 2597 Scarring alopecia, 2705-2706 Scars, 2636t, 2693t SCARs (severe cutaneous adverse reactions), 1704 Scavenger receptors, 217, 1390, 1390t Schatzki’s ring, 905, 905f Scheie’s syndrome, 1387, 1733, 1734t Schilling test, 925t, 1112 Schistocytes, 1053f Schistosoma, 1015t-1017t, 1018f, 2153-2154, 2153f Schistosomiasis, 1015-1018, 2153-2155 acute, 2154 chronic, 2154 clinical manifestations of, 1015, 2154 definition of, 2153 diagnosis of, 1018, 1018f, 2154-2155, 2155t epidemiology of, 1015, 2153-2154 etiology of, 2153-2154 pathobiology of, 1015, 2154 prognosis for, 1018b treatment of, 1018b, 2155b Schizencephaly, 2513 Schizoaffective disorder, 2347, 2353t, 2354 Schizoid personality disorder, 2355t Schizophrenia, 2353 definition of, 2353 diagnosis of, 2353 epidemiology of, 2353 pathobiology of, 2353 prognosis for, 2353 symptoms and signs of, 2353t treatment of, 2353b Schizotypal personality disorder, 2355t Schnitzler’s syndrome, 1740t Schober’s test, 1765-1766 Schumacher, Ralph, 1811 Schwann cells, 2529, 2529.e2f Schwannoma, 2689, 2689f Schwartz-Jampel syndrome, 2542t

Sciatica, 2377 Scimitar sign, 285, 285f Scimitar syndrome, 415-416 Scintigraphy, 1728-1730, 1729f Sclera, 2556 Scleral icterus, 977-978, 977f Scleritis, 2570 Scleroderma, 1777-1785 esophageal manifestations, 906 gastrointestinal involvement, 958 hematopoietic stem cell transplantation for, 1203 imaging of, 536f localized, 1778 Scleroderma renal crisis, 1783, 1785 Scleroderma-like skin induration, 1777-1778, 1778t Scleromyxedema, 2694 Sclerosing cholangitis, primary, 1045-1046 Sclerosing panencephalitis, subacute, 2209-2210 Sclerosis amyotrophic lateral sclerosis, 2522-2525, 2539t atherosclerosis, 417-419 with C. pneumoniae infection, 2012.e1 in chronic kidney disease, 840 inflammation pathways in, 230.e3t treatment of, 419, 840 atrophic skin conditions, 2693t cutaneous systemic diffuse, 1781 limited, 1781 focal segmental glomerulosclerosis, 786, 786b, 786f hippocampal, 2406 lichen, 2644f mesial temporal, 2406 Mönckeberg, 837 multiple sclerosis, 2471-2478, 2476b-2478b osteosclerosis, 1668-1669 otosclerosis, 2594 primary lateral sclerosis, 2524t systemic, 1777-1785 tuberous, 587, 817t, 822 tuberous sclerosis complex, 2515-2516, 2516f, 2516t Sclerosteosis, 1669 Sclerostin, 1644 Sclerotic lesions, 2694 Scoliosis, 629, 2375t Scombroid, 722 Scopolamine, 865t, 2565, 2600 Scorpion fish, 721 Scorpion stings, 66t, 2548t Scorpions, 2172t, 2175 Scrapie, 2506 Screening, 37. See also Laboratory testing for alcohol use, 153 for cancer adherence to guidelines, 15, 15f future issues, 57-58 for older adults, 108-109 drug tests, 699-702, 702t for early disease or asymptomatic risk factors, 54-57 electrocardiography, 274 family, 330, 1422 interventions not recommended for routine use, 55t newborn, 201-202 nutritional assessment, 1433 recommendations for abdominal aortic aneurysm screening, 55-56 recommendations for adolescents, 63 recommendations for colorectal cancer screening, 1328 recommendations for general population, 55t recommendations for high-risk populations, 55t recommendations for mammography, 1359, 1359t test requirements, 55t Scrotal masses, 1365f, 1366 Scrub typhus, 2048t, 2052 clinical manifestations of, 2052 diagnosis of, 2052 epidemiology of, 2052 treatment of, 2052b vectors, 2172t

Scurvy, 1385t Sea nettles (Chrysaora quinquecirrha), 720 Sea snake envenomation, 721 Seasonal allergens, 1688, 1688t Seasonal influenza vaccination, 554, 2194t, 2195 Seasonal rhinitis, 1674, 1688, 1688t Sebaceous carcinoma, 2561-2562 Sebaceous gland hyperplasia, 2680, 2680f Sebaceous glands, 2634 Seborrheic dermatitis, 2663, 2663f dermatoscopic signs of, 2703t in HIV infection, 2321 regional involvement, 2701f Seborrheic keratosis, 2645f, 2688f Second messengers, 1472-1473 Second sight, 2558, 2566 Secondary data, 43.e1t Secondary survey, 713-714 Second-wind phenomenon, 2545 Secretin stimulation test, 925t Secretory diarrhea chronic idiopathic, 934 true, 933-934 Secretory stimulators, 893t Secretory vesicles, 1144, 1144t Sedation, critical care, 650 Sedative-hypnotics, 160-161 with psychoactive effects, 117t toxidromes, 696t Sedentary behavior, 59 SEER program (NCI), 43.e1t Segmental duplications (SDs), 189 Seizures. See also Epilepsy absence atypical, 2402 childhood, 2404-2405 clinical manifestations of, 2401t, 2402 drugs for, 2407t EEG findings, 2342, 2343f acute symptomatic (provoked), 2399, 2404 causes of, 2399t incidence of, 2399 risk factors for, 2399 treatment of, 2407 antipsychotic-induced, 2626t atonic, 2401t, 2403 constellations, 2404-2406 definition of, 2399 diagnosis of, 2403-2406 differential diagnosis of, 2403, 2403t febrile genetic epilepsy with febrile seizures plus (GEFS+), 2404 prognosis after, 2409 focal clinical manifestations of, 2400-2402, 2401t definition of, 2399 drugs for, 2407t EEG findings, 2401, 2402f generalized classification of, 2401t clinical features of, 2401-2403, 2401t, 2403t definition of, 2399 drugs for, 2407t generalized tonic-clonic, 2402, 2403t with less specific age relationship, 2406 myoclonic clinical manifestations of, 2401t, 2402-2403 drugs for, 2407t postoperative, 2623 in pregnancy, 2408 prognosis for, 2409 prophylaxis of, 2452t provoked, 2399 single, 2404 prognosis after, 2409 treatment of, 2406-2407 after subarachnoid hemorrhage, 2449 syndromes, 2404-2406 tonic, 2401t, 2403 tonic-clonic, 2401t, 2402, 2403t treatment of, 2406b-2408b, 2407t considerations for women, 2408 preferred medications for women, 1602t surgical, 2407-2408 in women preferred medications for, 1602t treatment considerations, 2408

Selection balancing, 191-192 natural, 191 Selective autonomic syndromes, 2519 Selective estrogen receptor modulators (SERMs) for breast cancer, 1357-1359 for osteoporosis, 1643 perioperative, 2613t Selective norepinephrine-reuptake inhhibitors (SNRIs), 2349t Selective serotonin-reuptake inhibitors (SSRIs) for depression, 2349t for irritable bowel syndrome, 893t, 894 for pruritus, 2636t toxicity, 699t-702t Selegiline, 2349t, 2457t-2459t Selenium, 1450t-1452t, 2670-2671 Self-awareness, 2409 Self-help groups, 155 Self-management support, 48 Self-poisoning, 696 Semantic dementia, 2397 Semantic memory, declarative, 2383t Semen analysis, 1575, 1578t Seminoma, 1366 Semont’s maneuver, 2598f Semuloparin, 517-518 Senescence, male, 1571-1572 Senile purpura, 108, 108f SENLAT (scalp eschar, neck lymphadenopathy after tick bite), 2048t, 2050-2051 Senning or Mustard atrial baffle repair, 414, 416t Sensation, 2632 Sensitivity, test, 37, 37t Sensitivity analysis, 40 Sensitization, central, 133-134, 1817 Sensitizer-induced occupational asthma, 588-590 causes of, 590t clinical manifestations of, 588-589 diagnosis of, 589 epidemiology of, 588 pathobiology of, 588 prevention of, 590b prognosis for, 590 treatment of, 590b Sensorimotor neuropathy, 2533, 2533t Sensorineural hearing loss differential diagnosis of, 2594-2595 occupational, 80, 80t pathobiology of, 2594 Sensory abnormalities, 2340-2341, 2472 Sensory ataxia, 2341t Sensory examination, 2371 Sensory neuropathy peripheral neuropathies, 2528t, 2533t subacute, 2533 Sensory polyneuropathy, distal, 2535 Sensory testing, quantitative, 2528 Sentinel events, 46 Sentinel lymph node biopsy, 1376 Seoul virus, 2249t Sepsis, 1163 anti-inflammatory responses in, 686, 686.e2f definition of, 685 hypovolemia in, 744 immunosuppressive responses in, 686, 686.e2f inflammatory response in, 686, 686.e1f proapoptotic responses in, 686, 686.e2f procoagulant response in, 686, 686.e1f severe, 685 shock syndromes related to, 685-691 zoonotic, 2057, 2059t Septic abortion, 2252t-2253t Septic arthritis, 1806-1808 acute, 1806, 1806t anaerobic, 1933 antimicrobial therapy for, 1810t definition of, 1806 diagnosis of, 1807 differential diagnosis of, 1807 gonococcal, 1808 H. influenzae type b, 1947 nongonococcal, 1806-1808 clinical manifestations of, 1806 diagnosis of, 1806-1807

Index Septic arthritis (Continued) epidemiology of, 1806 microorganisms responsible for, 1806, 1806t pathobiology of, 1806 prevention of, 1808 prognosis for, 1808 S. aureus, 1899 treatment of, 1807b-1808b Septic bursitis, 1805-1806 clinical manifestations of, 1805 definition of, 1805 diagnosis of, 1805 differential diagnosis of, 1805 epidemiology of, 1805 pathobiology of, 1805 prevention of, 1805 prognosis for, 1805 treatment of, 1805b Septic cavernous venous sinus thrombosis, 2498-2499 clinical manifestations of, 2498 definition of, 2498 diagnosis of, 2499 epidemiology of, 2498 pathobiology of, 2498 prognosis for, 2499 treatment of, 2499b Septic sagittal sinus thrombosis, 2500 clinical manifestations of, 2500 diagnosis of, 2500 prognosis for, 2500 treatment of, 2500b Septic shock, 674, 685-691, 716 antibiotic regimens for, 689t antibiotics for, 690 cardiovascular dysfunction in, 686-687 corticosteroids for, 690 critical care support and management in, 690, 691f definition of, 685 differential diagnosis of, 688 drugs for, 690 epidemiology of, 685 evaluation of, 688f intensive insulin therapy for, 690 late, 686 pathobiology of, 685-687 prevention of, 688 prognosis for, 691 tissue hypoxia in, 686 treatment of, 676, 688b-690b, 688f, 690, 2255 circulatory therapy, 689-690 respiratory therapy, 688-689 Septicemia, 2252t-2253t meningococcal, 1936 staphylococcal, 1182, 1182f Septicemic plague, 1984f, 1986-1987, 1987f Septo-optic dysplasia, 1565 Sequencing, 200-201 clinical, 201 next-generation, 195, 201 somatic, 202 whole exome, 186 whole genome, 186 Serelaxin, 318 Serine protease inhibitors (SERPINs), 235, 241-242 Serine/threonine kinase gene STK11/LKB1, 1306 Serology, 2712t-2717t Seronegative rheumatic diseases, 1712 Seronegative spondyloarthropathies, 1713t, 1715t Seropositive systemic autoimmune diseases, 1716t Serotonin degradation of, 1558, 1558f synthesis of, 1558, 1558f toxicity of, 2618 Serotonin norepinephrine-reuptake inhhibitors (SNRIs), 136, 138t Serotonin syndrome, 1846, 2519-2520 in psychiatric patients, 2628, 2628t Serotonin-reuptake inhibitors, 699t-702t Serous carcinoma, 1380 Serrated polyps, 1326-1327

Sertraline for depression, 2349t, 2396 for menopausal hot flushes, 1628t-1629t for pruritus, 2636t Serum cardiac biomarkers, 444-445 Serum enzymes, 989-990 Serum free light chains, 1274 Serum immunoelectrophoresis, 734 Serum immunoglobulins, 1273 Serum proteins, 1432-1433 Serum-ascites albumin gradient, 1028, 1028t Service models, 188 Sevelamer for acute kidney injury, 782 for hyperphosphatemia, 778, 778t for renal bone disease, 840 Severe acute respiratory syndrome (SARS), 2056-2057, 2057t Severe acute respiratory syndrome (SARS) coronavirus infection, 2199-2201 clinical manifestations of, 2200, 2200t prevention of, 2201 treatment of, 2201 Severe combined immunodeficiency (SCID), 1677 gene therapy for, 211-212 X-linked, 211-212, 1678t Severe cutaneous adverse reactions (SCARs), 1704 Severe myoclonic epilepsy of infancy (Dravet’s syndrome), 2404 Sex assignment, 1567 Sex chromosomes, 1565 Sex partners, 1945 Sex steroids, 1010 Sexual activity and cancer, 1222 and cardiovascular disease, 258 with heart failure, 315 and travel-related health problems, 1884 Sexual development disorders, 1560-1568, 1560t definition of, 1560-1562 diagnosis of, 1566-1567 initial medical management of, 1567 management of, 1566-1568 ovotesticular, 1566 ovotesticular disorder, 1566 physical examination of, 1566 psychosocial support for, 1568 surgical management of, 1567-1568 treatment of, 1567b-1568b Sexual differentiation, 1560-1561, 1561f abnormal, 1563, 1563f defects of, 1562-1564 Sexual dysfunction, 1576-1579, 1596-1597, 1597b Sexual function, 1596 definition of, 1596 in men, 1570 Sexual history, 24 Sexual maturity ratings (SMRs), 61, 62f Sexual orientation, 1566 Sexually transmitted infections, 1876-1877, 1877t. See also specific infections in adolescents, 64 anorectal, 973 chlamydial, 2009-2011 clinical manifestations of, 2009-2011 diagnosis of, 2011 epidemiology of, 2009 prevention of, 2011 treatment of, 2011b definition of, 1876 diagnosis of, 1877 epidemiology of, 1876-1877 etiology of, 1876 and HIV infection, 1877 pathobiology of, 1877 prevention of, 1880-1881 postexposure, 1881 preexposure interventions, 1880-1881 recommended strategies for, 1880 screening for, 56-57 treatment of, 1877 comprehensive strategy for, 1881 syndromic strategies for, 1877 Sézary’s syndrome, 1255, 2661, 2690-2691 Shapiro’s syndrome, 2517-2518 Sheehan syndrome, 1487

Shellfish poisoning, 722 Shift-work disordered sleep, 2422 Shiga-like toxins, 1956 Shigella, 1975 Shigellosis, 1975-1979 clinical features of, 1869t, 1977, 1977t complications of, 1977t definition of, 1975 diagnosis of, 1977-1978 distinguishing characteristics of, 2252t-2253t epidemiology of, 1869t, 1975-1976 incidence of, 1975-1976, 1976f pathobiology of, 1976-1977 prevention of, 1978-1979 prognosis for, 1979 treatment of, 1978b Shingles (zoster), 2227-2229 AIDS-associated, 2297t-2301t clinical manifestations of, 2227-2228, 2228f definition of, 2227 herpes zoster ophthalmicus, 2565 Shock, 672-681 in bacterial meningitis, 2484 cardiogenic, 674, 675t, 681-685 definition of, 678f-679f, 681 diagnosis of, 681t, 684f etiology of, 678f-679f, 681, 681f, 682t management of, 452-453, 683-685, 684f therapy for, 678f-679f clinical manifestations of, 673-675, 674t cryptic, 672 definition of, 672, 675, 678f-679f dengue shock syndrome, 2254 diagnosis of, 675 dissociative, 674 definition of, 678f-679f etiology of, 678f-679f therapy for, 678f-679f distributive, 674, 675t definition of, 678f-679f etiology of, 678f-679f therapy for, 678f-679f epidemiology of, 672 extracardiac/obstructive definition of, 678f-679f etiology of, 678f-679f therapy for, 678f-679f hemorrhagic, 673-674 categorization of, 715t classification of, 674t etiology of, 678f-679f initial treatment of, 715t hypovolemic, 673-674, 675t definition of, 678f-679f etiology of, 678f-679f therapy for, 678f-679f laboratory findings, 674t mixed states, 674-675 myopathic, 678f-679f neurogenic, 2368-2369 obstructive, 675t definition of, 678f-679f etiology of, 678f-679f extracardiac, 674 therapy for, 678f-679f pathobiology of, 672-673 physical findings, 674t prognosis for, 680 septic, 674, 676, 685-691, 716, 2255 spinal, 2365 treatment of, 673f, 675-676, 675b-680b, 677f-679f fluid therapy, 677, 680t mechanical support, 680 vasopressor therapy, 680 vasodilatory, 672 Shock (rush) protocol, 675, 675t Short anagen syndrome, 2706 Short Portable Mental Status Questionnaire, 115-116, 115t, 120 Short tandem repeats (STRs), 191 Short unilateral neuralgiform headache with conjunctival injection and tearing, 2357t, 2360 diagnosis of, 2361 prevention of, 2361 treatment of, 2361

I91

Short-bowel syndrome malabsorption in, 932 prognosis for, 932 treatment of, 932b Shortness of breath. See Dyspnea Shoulder disorders, 1751-1752 bursitis conditions, 1750t frozen shoulder, 1752 Milwaukee shoulder, 1816, 1816b tendinitis conditions, 1750t Shunts isolated lesions, 407-408 palliative surgical, 405t transjugular intrahepatic portosystemic shunt, 1029, 1030f for varical bleeding, 1029-1030 Shwachman-Diamond syndrome, 1133t, 1134, 1683, 1682.e1t Sialadenosis, 2585 Sialidosis, 1387 Sialorrhea, 2455 Sialosis, 2585 Siberian tick typhus, 2048t, 2050 Sibling donor transplantation, 1119-1120 Sicca symptoms causes of, 1787-1788, 1788t treatment of, 1788 Sick building syndrome, 81 “The Sick Lady” (Netscher), 1061, 1061f Sickle cell anemia, 1095-1096 clinical manifestations of, 1099 features of, 1099, 1099t hydroxyurea treatment of, 1102, 1102t in pregnancy, 1100 prevalence of, 1096t prognosis for, 1104 subphenotypes of, 1099, 1100f Sickle cell disease, 1095-1104 from ages 20 to 40 years, 1099-1101 beyond fifth decade, 1101-1102 blood findings, 1102 cardiovascular complications of, 1101-1102 clinical manifestations of, 1098-1102 complications of, 1102-1103 definition of, 1095 diagnosis of, 1097f, 1102 epidemiology of, 1095 family studies, 1102 hematopoietic stem cell transplantation for, 1202-1203 painful episodes, 1099, 1102-1103 pathobiology of, 1095-1098 pathophysiology of, 1096-1098, 1098f prevalence of, 1096t prevention of, 1102 prognosis for, 1104 pulmonary complications of, 1103t renal abnormalities in, 1101, 1101t stroke in, 2443t treatment of, 1102b-1104b future directions, 1104 general measures for, 1102 Sickle cell trait, 1095-1096, 1096f, 1096t Sickle cells, 1053f Sideroblastic anemia, 1072-1073, 1073f clinical manifestations of, 1072 definition of, 1072 diagnosis of, 1072-1073 epidemiology of, 1072 genetics of, 1072 hereditary, 1072 idiopathic forms, 1072 laboratory findings, 1070t pathobiology of, 1072 prognosis for, 1073 smear features of, 1056t treatment of, 1073b Sigmoid diverticulosis, 947f Sigmoidoscopy, flexible applications, 873t in infectious diarrhea, 1869-1870, 1870t Sign of Leser-Trélat, 2688 Signal aliasing, 275-276 Signal transducer and activator of transcription (STAT), 219, 232 Signal transduction, 1472-1473 Signaling inhibitors, 169-172 Volume I  pp 1-1382  •  Volume II  pp 1383-2722

I92

Index

Signaling lymphocyte activation molecule (SLAM)-associated protein (SAP), 2233 Signet ring abnormality, 567.e2f Significance testing, 35-36 Silastic sweat imprint, 2521 Sildenafil (Viagra), 429 for altitude sickness, 597 for benign prostatic hyperplasia, 830t for diffuse esophageal spasm, 904 for erectile dsyfunction, 1578-1579 for idiopathic pulmonary fibrosis, 581 for pulmonary hypertension, 402 Silent myocardial ischemia, 432 Silent (lymphocytic) thyroiditis, 1509 Silicosis, 593-594, 594f jobs that lead to, 594t patterns of mortality from, 588.e3f suggestive features, 589t Silodosin, 830t Siluriformes (catfish), 721 Silver deposition, 93t, 97, 97f Simeprevir, 1004-1005, 2177t-2178t, 2180 Simethicone, 922 Simponi (golimumab), 169, 940, 1762 Simvastatin, 454t, 1395, 1396t Sin Nombre virus, 2249t Sindbis, 2262, 2262b Sindbis virus, 2262 Sinecatechins, 2222t Sinequan (doxepin), 2660t Single nucleotide polymorphisms (SNPs), 190, 196, 1384 Single-nucleotide variants, 190, 194 Single-photon emission computed tomography, 2345t in epilepsy, 2404 myocardial, 285-286, 286f, 286.e1f stress-only protocol, 286 Sinoatrial block, 357f Sinopulmonary syndromes, 1859t, 1860 Sinus arrhythmia, 357f Sinus bradycardia, 356-358, 357f, 452 Sinus complaints, 2586-2589 Sinus disorders, 2585-2592 Sinus headache, 1690-1691, 2362 Sinus node dysfunction, 350t-351t, 356-358, 357t causes of, 358t clinical manifestations of, 358 treatment of, 358b-359b Sinus of Valsalva aneurysms, 413 Sinus of Valsalva anomalies, 413 Sinus rhythm, 362f Sinus tachycardia, 359, 359f, 363-366 Sinus tumors, 1299 Sinusitis, 2586-2588 Aspergillus, 2083-2084 bacterial, 1896 chronic, 1690-1693, 2587-2588 prevention of, 1691 treatment of, 1691b-1693b chronic hyperplastic eosinophilic, 1691, 1691t clinical manifestations of, 2586 cultures, 2587 definition of, 2586 diagnosis of, 2586-2588 diagnostic tests, 2587 differential diagnosis of, 2587-2588 epidemiology of, 2586 fungal, 1691, 1691t, 2102 H. influenzae, 1947 history in, 2586 hyperplastic eosinophilic, 1691t imaging, 2587, 2588f infectious, 1691t inflammatory, 1691t laboratory findings, 2587 pathobiology of, 2586 physical findings, 2586-2587 S. pneumoniae, 1947 surgical therapy for, 2588 symptoms and signs of, 2586, 2586t treatment of, 2588b Sipuleucel-T, 1231, 1369 Sirolimus, 167-168 adverse effects of, 168, 1037t for immunosuppression, 1037-1038 for influenza, 2197 for lymphangioleiomyomatosis, 587 in renal transplantation, 844t

Sister Mary Joseph’s node, 1317, 1328 Sister of P glycoprotein (SPGP), 987-988 Sistrurus (rattlesnakes), 719 Six Sigma, 45 Sixth nerve palsy, isolated, 2577-2578 Sjögren’s syndrome, 584, 1785-1789, 2237t assessment of activity of, 1788 associated autoantibody, 2518t autoantibodies associated with, 1716t classification criteria for, 1788t clinical features of, 1787, 2524t decreased salivary secretion in, 2584 definition of, 1785-1786 diagnosis of, 1787-1788 diagnostic criteria for, 1788 differential diagnosis of, 1787-1788, 1788t epidemiology of, 1786 EULAR Sjögren Syndrome Disease Activity Index (ESSDAI), 1788 EULAR Sjögren Syndrome Patient Reported Index (ESSPRI), 1788 extraglandular manifestations of, 1787, 1787t glandular manifestations of, 1787 laboratory findings, 1787 lung disease in, 585 neuropathy in, 2533-2534, 2533t and non-Hodgkin’s lymphoma, 1787 pathophysiology of, 1786-1787, 1786f primary, 1785-1786, 1786f salivary gland enlargement in, 2584 secondary, 1785-1786 systemic manifestations of, 1787 treatment of, 1788b-1789b future directions, 1789 symptomatic, 1788 Skeletal hyperplasia, expansile, 1665t Skeletal lesions, 1281 Skeletal metastases, 1380 Skeletal muscle, 2538 Skier’s thumb (gamekeeper’s thumb), 1753 Skin, 2632-2637 aging of, 2693-2694 autoinflammatory diseases of, 1740t black, 2700, 2700t functions of, 2632 nodules of, 2687t rheumatoid arthritis in, 1758, 1758f, 1758t structure of, 2632, 2633f in systemic sclerosis, 1781 warfarin-induced necrosis, 176, 1186, 1186f Skin and soft tissue infections, 2695-2698, 2695t Acinetobacter species–associated, 1969, 1969f anaerobic, 1932-1933 clinical evaluation of, 1807f enterococcal, 1914 fungal, 2693 group A Streptococcus (GAS), 1907-1908 mucormycosis, 2088 nontuberculous mycobacterial, 2041 Pseudomonas, 1965 S. maltophilia, 1971 viral, 2697 Skin biopsy, 2528, 2656-2657 Skin cancer basal cell, 1377-1378 clinical manifestations of, 1377-1378 diagnosis of, 1378 follow-up, 1378 in HIV infection, 2321 incidence of, 1377 nonmelanoma, 1377, 2321 prevention of, 1378 risk factors for, 1377 squamous cell, 1377-1378 treatment of, 1378b Skin care, 1785 Skin color, 2637 Skin disease in acutely ill, 2644-2655 in African ancestry patients, 2700, 2700t basement membrane zone components, 2634, 2634.e1t clues from history and general health status, 2646t color clues, 2637, 2638t-2644t conditions that can occur at any CD4+ count, 2319t

Skin disease (Continued) conditions with CD4+ counts < 200 cells/µL, 2318t in congenital erythropoietic porphyria, 1412-1413 diagnostic tests for, 2655-2657 differential diagnosis of, 2637-2657 esophageal disorders, 906 hemidesmosomal components of, 2634.e1t history in, 2637 in HIV infection, 2318-2322 life-threatening emergencies, 2644-2655 morphology of, 2637-2644 photosensitive, 1413 in porphyria, 1412-1413, 1412f in porphyria cutanea tarda, 1412 protection against, 1884 regional involvement, 2701f, 2702t representative diagnoses, 2645t-2646t in sarcoidosis, 605, 605f secondary features of, 2651t-2654t terminology for, 2651t-2654t therapy for, 2657-2658 vesiculo-erosive in porphyria, 1412-1413, 1412f treatment of, 1414 when to refer to specialists, 2657 Skin eruptions. See also Exanthems morbilliform, 1858-1859, 2672-2673 in immunocompromised patients, 1858-1859 of measles, 2202, 2202f papular, 2673 papular pruritic, 2321 polymorphic light eruptions, 2664, 2664f purpuric, 2673-2675, 2673t pustular, 2680-2681 scarlatiniform, 2671-2672, 2671t urticaria-like, 2683t Skin examination, 28, 2637 Skin infarction, 1283, 1283f Skin lesions in black skin, 2700, 2700t color of, 2637, 2638t-2644t descriptions of, 2637 distribution of, 2644 inflammatory, 2691-2692 location of, 2654t morphology of, 2636t, 2637-2644, 2647t-2651t primary, 2636t secondary, 2636t, 2644 zoonoses that cause, 2057, 2059t Skin pigmentation, 2637, 2637f Skin smears, 2045 Skin tags, 2561 Skin testing, 1676t Skin tumors, 2636t, 2647t-2651t, 2687-2693, 2687t hematopoietic, 2690-2691 inflammatory, 2690-2691 Skin ulcers, pyogenic, 2057t Skull base meningioma, 1291, 1291f Slater v. Baker and Stapleton, 5 Sleep in fibromyalgia, 1819-1820 models of, 2415-2416, 2416f non–rapid eye movement (NREM), 638-639, 2415, 2415.e3f rapid eye movement (REM) (stage R), 2415, 2415t, 2415.e3f regulation of, 2415-2416, 2416f slow-wave, 638-639 stage N1, 638-639, 2415, 2415t stage N2, 638-639, 2415, 2415t stage N3 or slow-wave sleep, 638-639, 2415, 2415t stages, 2415, 2415t, 2415.e3f Sleep apnea, 2418-2419 central, 545t definition of, 2418-2419 obstructive, 638-642, 638f diagnosis of, 385t, 639 in obesity, 1463 STOP-Bang Questionnaire for, 2419, 2419t Sleep diaries, 2416, 2416f Sleep disorders, 2415-2424 clinical manifestations of, 2416 definition of, 2415 diagnosis of, 2416-2417 in fibromyalgia, 1818

Sleep disorders (Continued) with heart failure, 302-303 menopause-related disturbances, 1629 pathobiology of, 2415-2416 shift-work disordered sleep, 2422 Sleep hygiene education, 2421t Sleep positioning, 641 Sleep restriction, 1463, 2421t Sleep terrors, 2422-2423, 2423t Sleep-disordered breathing, 302-303 Sleepiness, 2416f Epworth Sleepiness Scale, 2417-2418, 2418t excessive daytime sleepiness, 639, 2417, 2417f treatment of, 2418b Sleep-onset difficulties, 2421t Sleep-related breathing disorders, 2418-2424 Sleep-related eating, 2423, 2423t Sleep-wake cycle, 1475 Sleepwalking, 2422-2423, 2423t Slit lamp examination, 2557-2558 Slow virus infections, 2209-2212 Sly’s syndrome, 1387, 1734t Small airways disease, 555 Small bowel. See also Colon disorders of rapid transit through, 888 disorders of slow transit through, 885-887 extrinsic neuropathic disorders, 886 intrinsic neuropathic disorders, 886 neoplasms of, 1320-1322 obstruction of, 949, 950f rapid transit dysmotility of, 888 Small bowel biopsy, 925t Small bowel contrast imaging, 925-926 Small bowel motility, 884-885, 885f, 887 Small bowel motility disorders, 886-887 Small bowel tumors, 1320-1322 clinical manifestations of, 1321 diagnosis of, 1321 epidemiology of, 1320 pathobiology of, 1320-1321 predisposing conditions, 1321 prognosis for, 1322 staging, 1321 surveillance of, 1321 treatment of, 1321b-1322b Small cell anaplastic carcinoma, 1381 Small cell lung cancer epidemiology of, 1303 pathology of, 1304 treatment of, 1312 Small intestinal bleeding, 882-883, 883t Small intestinal immunoproliferative disease, 1265 Small intestinal ischemia, 951 Small intestinal ulcers, 947-948 causes of, 947, 948t drug-induced, 947-948, 948f Small lymphocytic lymphoma, 1254-1255, 1263 clinical characteristics of, 1261t definition of, 1263 Small molecular mediators, 231.e1t Small nucleolar RNA (snoRNA), 189 Smallpox, 87t, 88-89, 89f clinical manifestations of, 88, 2216, 2216f diagnosis of, 88, 2218 distinguishing characteristics of, 2252t-2253t epidemiology of, 88, 2214-2215 flat, 2216, 2216t hemorrhagic, 2216, 2216t immunizing agents for, 68t-73t modified, 2216, 2216t ordinary, 2216, 2216t ordinary confluent disease, 2216, 2216t ordinary discrete disease, 2216, 2216t ordinary semiconfluent disease, 2216, 2216t pathobiology of, 88, 2215-2216 prevention of, 88-89 prognosis for, 89, 2219 treatment of, 88b, 2219 types of, 2216t Smallpox vaccine, 78, 2218-2219 adverse reactions to, 78 indications for, 78 Small-vessel occlusion, 2435-2436 Small-vessel vasculitis, 957, 1758, 1758f clinical manifestations of, 1797-1799, 1797f

Index Small-vessel vasculitis (Continued) names for, 1794t pathophysiology of, 1795-1796 SMART-COP scoring system, 616t Smell, 2592-2593 disorders of, 2592-2593 loss of, 2593, 2593t Smith-Lemli-Opitz syndrome, 1564 Smith-Magenis syndrome, 194 Smoke inhalation, 598-599 Smoking. See Cigarette smoking Smoking cessation for acute MI, 454 for chronic obstructive pulmonary disease, 559 Smoking-related interstitial pneumonia, 575t, 579-580, 582-583 Smoldering adult T-cell leukemia/lymphoma, 2238 Smoldering (asymptomatic) multiple myeloma, 1276t, 1281 Smoldering (indolent or asymptomatic) Waldenström’s macroglobulinemia, 1276t Smooth muscle, airway, 554 Smooth muscle disorders, 886 Smooth pursuit, 2577 Smudge cells, 1253-1254, 1253f Snake antivenom, 717, 719-720 Snake bite envenomation, 717-720, 2548t antidote for, 706t-710t clinical syndromes, 717-718 diagnosis of, 718 distinguishing characteristics of, 2252t-2253t effects of, 718 laboratory investigations, 718 local myotoxicity, 718 local tissue injury, 718 neuromuscular toxicity, 718-719 treatment of, 718b-719b Snake handlers, 720 Snake syndromes, 719-720 Snare polypectomy, 1326, 1326.e1 Snellen eye chart, 104, 2557f Sniff test, 628, 2369 Snowshow hare encephalitis, 2268 Soaks, 2657 Social anxiety disorders, 2351t Social history, 24-25 Social impairment, 156, 157t Social justice, 3 Social phobia, 2351t Social status, 17-18, 18f Social support assessment, 105, 106t Society for Thoracic Surgeons, 42 Socioeconomic issues, 17-19 Sodium extracellular fluid content, 737-738 fractional excretion of (FENa), 733, 781, 781t reabsorption of, 757, 756.e1f serum levels, 745 Sodium anion–induced diarrhea, 926 Sodium balance, 741-742 Sodium balance disorders, 742-747 epidemiology of, 741 pathogenesis of, 742, 743t pathophysiology of, 742 postoperative, 2624 Sodium bicarbonate (NaHCO3) for chronic interstitial nephritis, 798 for drug overdose, 699t-702t, 706t-710t for lactic acidosis, 767 for metabolic acidosis, 840 for post-resuscitation care, 355 for pulseless electrical activity, 355f Sodium channelopathies, 2544 Sodium channels, 341, 341.e1f Sodium chloride supplements, 1567, 1654 Sodium ferric gluconate, 1070 Sodium glucose cotransporter 2 (SGLT2) inhibitors, 1536 Sodium nitrite, 599-600, 706t-710t Sodium oxybate, 2418, 2419t Sodium phosphate diarrhea, 933 Sodium phosphate salts, 777 Sodium polystyrene sulfonate, 761 Sodium replacement, 1521

Sodium restriction for ascites, 1030 for chronic kidney disease, 838-839 to prevent calcium stone recurrence, 814 treatment of, 747-748 Sodium retention, renal primary, 746-747 secondary, 747 Sodium stigobluconate, 2122 Sodium tablets, 2521 Sodium thiosulfate, 599-600, 706t-710t Sodium urate crystals, 731-732 Sodium-channel blockers, 706t-710t Sodium-retaining states, 746, 746t Sofosbuvir, 2178t, 2180 for hepatitis C, 998, 1004-1005, 2177t for pretransplantation chronic liver failure treatment, 1034 Soft tissue diseases, 1749. See also Skin and soft tissue infections Soft tissue tumors, oral, 2583-2584, 2583t Sokolow-Lyon criterion, 273 SOLER mnemonic, 14t Soles. See also Foot (feet) rash involving, 1851, 1851t Solid masses, hepatic, 980-982, 982f Solid organ transplantation. See also Organ transplantation in immunocompromised patients, 1855, 1855t Solid tumors hematopoietic stem cell transplantation for, 1202 mesenteric, 950 omental, 950 Solifenacin, 113t, 830t Soluble C5b-9, 245 Soluble ST2 (sST2), 304, 304.e1t Solu-Medrol, 1702 Solute diuresis, 754 Solute excretion, 754 Solutes, nonabsorbable or poorly absorbable, 933 Somatic mutations, 193, 1410 Somatic pain, 133, 134t Somatic sequencing, 202 Somatic symptom and related disorders, 2347t, 2354, 2354b, 2355t Somatic symptom disorder, 2355t Somatoform disorders, 2354 Somatosensory deficits, 1746 Somatosensory evoked potentials, 2345 Somatosensory seizures, 2401t Somatostatin, 1474 Somatostatin analogues, 1559 Somatostatinoma, 1335t, 1338, 1553 clinical manifestations of, 1338 definition of, 1338 diagnosis of, 1338 pathobiology of, 1338 prognosis for, 1338 treatment of, 1338b Somatostatinoma syndrome, 1338 Somatotropin release inhibiting factor, 1474 Somnolence, daytime, 639 Sorafenib (Nexavar), 170, 1211t-1216t, 1344 Sorbitol and fructose diarrhea, 933 Sore throat, 2601, 2602t Sotalol, 363t-364t, 430t, 2462 Sotatercept (ACE-011), 1095 Sotos’ syndrome, 194 South America, 719 South American hemorrhagic fevers clinical manifestations of, 2250-2251, 2251t pathobiology of, 2250, 2251t South Sudan, 21 Southern medicine, 17 Southern tick-associated rash illness (STARI), 2021, 2025, 2057-2058, 2057t Sowda, 2169 Space sickness, 2597 Spain, 603 Sparganosis, 2153 Spasm dystonic, 2463-2464 esophageal, diffuse, 904 Spasmodic dystonia, 2463-2464 Spastic gait, 2341t

Spastic paraparesis, 2236, 2237t, 2238-2239 Spastic paraplegias, 2469-2470 differential diagnosis of, 2470t hereditary, 2470, 2524t Spasticity, 2238-2239 Specific granule deficiency, 1147t-1148t, 1149 clinical manifestations of, 1149 diagnosis of, 1149 pathobiology of, 1149 treatment of, 1149b Specificity, test, 37, 37t Specificity spillover, 1473 Speckle tracking strain imaging, 276, 276.e1f Spectinomycin, 1891t-1892t Speech discrimination test, 2594 Speech reception threshold, 2594 Speech therapy, 2385 Speed’s test, 1752 Spermatogenesis, 1569-1570 Spermicides, 1605t, 1606f, 1607 Spherocytes, 1082, 1082t Spherocytosis, hereditary, 1053f, 1081-1083, 1386t Spheroschistocytes, 1053f Sphincter of Oddi, 877 Sphincter of Oddi dysfunction, 1047-1048 Sphincterotomy, biliary, 877, 877f Spider bite envenomation, 66t, 706t-710t, 2548t Spiders, toxic, 2172t, 2174-2175 Spina bifida, 827 Spinal anesthesia and analgesia, 2620 Spinal cord anatomy of, 2366f, 2370, 2371f blood supply of, 2380, 2380f functional elements of, 2378, 2379f lesions of, 2370-2382 malformations of, 2513-2514 principal fiber tracts of, 2379f stimulation of, 141 tethered, 2513-2514 Spinal cord astrocytoma, 1297, 1297f Spinal cord compression, 2381-2382 clinical clues to, 2524t epidural, 1296, 1296f metastatic, 2381-2382 Spinal cord disorders, 2370, 2378-2382 ancillary testing, 2379 autonomic disorders, 2518 causes of, 2378 clinical manifestations of, 2370-2371, 2378-2379 definition of, 2378 diagnosis of, 2379-2380 differential diagnosis of, 2379-2380 epidemiology of, 2378 pathobiology of, 2378 treatment of, 2380b Spinal cord hemorrhage, 2381 Spinal cord injury, 2364-2370 epidemiology of, 2364 impairment scale, 2369t versus ligamentous injury, 2367 pathobiology of, 2364-2365 primary injury phase, 2364 secondary injury phase, 2365 traumatic clinical manifestations of, 2365 diagnosis of, 2366-2367 prognosis for, 2369 treatment of, 2368-2369 treatment of, 2367b-2369b Spinal cord syndromes, 2365 Spinal epidural abscess, 2497-2498, 2498f clinical manifestations of, 2497, 2497t definition of, 2497 diagnosis of, 2497, 2498f epidemiology of, 2497 initial characteristics of, 2497t MRI findings, 2497, 2498f pathobiology of, 2497 prognosis for, 2498 treatment of, 2497b Spinal instability, cervical, 1833 Spinal manipulation, 182t, 184 Spinal muscular atrophy, 2525-2526 clinical clues, 2524t clinical manifestations of, 2525-2526

I93

Spinal muscular atrophy (Continued) definition of, 2525 diagnosis of, 2526 epidemiology of, 2525 pathobiology of, 2525 prognosis for, 2526 treatment of, 2526b type I clinical manifestations of, 2525-2526 diagnosis of, 2526 incidence of, 2525 prognosis for, 2526 type II clinical manifestations of, 2525-2526 diagnosis of, 2526 prognosis for, 2526 treatment of, 2526 type III clinical manifestations of, 2525-2526 diagnosis of, 2526 prognosis for, 2526 treatment of, 2526 Spinal nerve roots, 2370 disorders of, 2370-2371 lesions of, 2370-2382 Spinal nerves, 2370 Spinal osteoarthritis, 1747 Spinal shock, 2365 Spinal stenosis, 2375-2377 classification of, 1713t clinical manifestations of, 2376 definition of, 2375 diagnosis of, 2376-2377 epidemiology of, 2375 low back pain in, 2375t pathobiology of, 2375-2376 prognosis for, 2377 treatment of, 142t, 2377b Spinal tumors, 1295-1297, 1296t extradural, 1295-1296 intradural extramedullary, 1296 intramedullary, 1296-1297 Spine anatomy of, 2370 disorders of, 2370, 2372-2377 lesions of, 2370-2382 orthopedic procedures on, 1832 Spine injury, 714 Spine pain acute, 2374-2375 mechanical, 2375 red flags, 2373, 2373t Spinobulbar muscular atrophy, 2526 Spinocerebellar ataxias, 2469-2470 clinical manifestations of, 2470 diagnosis of, 2470 prognosis for, 2470 treatment of, 2470b Spiramycin, 2132t Spiritual distress, 10, 11t Spirochetes, 2700f Spirometra, 2147-2148, 2148t Spirometra mansonoides, 2153 Spirometry, 539-540 Spironolactone, 748t for ascites, 1030 for cardiovascular disease, 842 for heart failure, 312t for hypertension, 388t, 391 for metabolic alkalosis, 773 for periodic paralysis, 2544-2545 Spleen examination of, 27, 1141-1142, 1141t imaging of, 1142, 1142f laboratory evaluation of, 1141-1142 physical examination of, 1141 Splenectomy for anemia, 1068 for hereditary spherocytosis, 1083 for immune thrombocytopenic purpura, 1164, 1164t post-splenectomy neutrophilia, 1131 for warm autoimmune hemolytic anemia, 1078 Splenic arterial disease, 959 Splenic lymphoma with villous lymphocytes, 1254t Splenic marginal zone lymphoma, 1265 Splenic vascular disease, 958-959 Volume I  pp 1-1382  •  Volume II  pp 1383-2722

I94

Index

Splenic vein thrombosis, 959 Splenomegaly, 1140-1142, 1142f approach to, 1142, 1142t causes of, 1141t definition of, 1140-1141 diagnosis of, 1141-1142 pathobiology of, 1141 in sarcoidosis, 606 Splinter hemorrhage, 255, 257f, 2708-2709 Spondyloarthritis, 1762-1769 axial, 1763t clinical features of, 1763 definition of, 1762-1763 diagnosis of, 1763 diagnostic criteria for, 1763t genetic susceptibility to, 1764-1765 pathobiology of, 1763 subsets, 1762-1763, 1763f treatment of, 1763b-1764b antibiotic therapy, 1764 anti-TNF therapy, 1764 general measures, 1763-1764 medical therapy, 1764 undifferentiated, 1762-1763, 1769 Spondyloarthropathies, 1712 clinical subsets, 1765-1769 patterns of onset, 1715 seronegative, 1713t, 1715t Spondylolisthesis, 2375t Spondylosis, 1832 Sponges (contraceptive), 1605t, 1606f Sponges, stinging, 722 Spooning, 2707, 2707f Sporothrix schenckii, 2078 Sporotrichosis, 2078-2079 clinical features of, 2078, 2124f definition of, 2078 diagnosis of, 2078 disseminated, 2078-2079 epidemiology of, 2078 lymphocutaneous, 2078, 2078f osteoarticular, 2078 pathobiology of, 2078 prognosis for, 2079 pulmonary, 2078-2079 treatment of, 2078b-2079b visceral, 2078 Sports medicine injuries, 1751, 1751t Sports-related conditions, 1751, 1751t Spot sign, 2451 Spotted bone(s), 1671 Spotted fever, 2059t Spotted fever Rickettsia, 2059t, 2252t-2253t Sprain, ankle, 1754 Sprue nontropical (celiac), 1108 tropical, 931, 1108 Sprycel (dasatinib), 171, 1211t-1216t Spur cells, 1066 SQSTM1 gene, 1664-1665 Squamous cell carcinoma esophageal, 1314-1315 eyelid, 2562 head and neck, 1301t in lung, 1304 oral, 2580t, 2581, 2581f of skin, 1377-1378, 1377f clinical manifestations of, 1377-1378, 1377f diagnosis of, 1378 pathobiology of, 1377 risk factors for, 1377 treatment of, 1378 of unknown primary origin management of, 1380-1381 recommended evaluation after light microscopic diagnosis, 1379t recommended treatment, 1380t Squamous intraepithelial lesions, 2219, 2221 anal, 972 cervical, 2221 prevalence of, 2220 Squamous papillomas, 2561 Square root sign, 489 Square-wave jerks, 2579 Squaric acid dibutylester, 2705 the squeezes, 597 Src family kinases, 1146 St. Gallen conference, 1355

St. John’s wort, 314 St. Jude valve, 287, 289f, 472t, 473f St. Louis encephalitis, 2263t, 2267-2268 clinical manifestations of, 2267 diagnosis of, 2267-2268 epidemiology of, 2267 etiology of, 2267 features of, 2263t geographic distribution of, 2490 pathobiology of, 2262, 2267 prevention of, 2268 prognosis for, 2268 treatment of, 2268b ST segment, 267-268, 273 ST2, soluble (sST2), 304, 304.e1t Standard deviation (SD), 32, 35 Standard error of the mean (SEM), 35 Standard errors, 35-36 Standard tube agglutination, 1981 Standardized Assessment of Concussion, 2365 Standardized mean difference (SMD), 34 Staphylococcal blepharitis, 2561, 2561f Staphylococcal bullous impetigo, 2652f Staphylococcal folliculitis, 2319, 2319f Staphylococcal scalded skin syndrome, 2679, 2679f Staphylococcemia, 2252t-2253t Staphylococcus, 1896-1902 Staphylococcus aureus, 476-477, 1896-1901 Staphylococcus aureus bacteremia, 1898, 1898t Staphylococcus aureus infections clinical features of, 1869t, 1897-1900 diagnosis of, 1900 epidemiology of, 1897 health care–associated, 1863t, 1864, 1865.e1t in HIV infection, 2319 laryngitis secondary to, 2602, 2602f mechanisms of disease, 1897 methicillin-resistant, 1897, 1897.e1t guidelines and recommendations for, 1865.e1t health care–associated, 1863-1864, 1865.e1t prevention of, 1901 skin and soft tissue, 1897 pathobiology of, 1897 prevention of, 1901 prognosis for, 1901 skin and soft tissue, 1897-1898 susceptibility testing for, 1900 treatment of, 1900b-1901b vancomycin-resistant, 1901 virulence factors, 1897, 1897.e1t Staphylococcus aureus meningitis, 2480-2481, 2486-2487 Staphylococcus aureus pneumonia, 612, 613f Staphylococcus epidermidis, 1901-1902 Staphylococcus lugdunensis, 1902 STARI (Southern tick-associated rash illness), 2021, 2025, 2057-2058, 2057t Starling forces, 741-742 Starling’s law, 266, 266f Starr-Edwards ball cage valve, 472t, 473f Stasis dermatitis, 2643f, 2701f Statin therapy for acute coronary syndrome, 438t-439t, 439 for ischemic heart disease, 428 for lipid disorders, 1395-1397, 1396t recommendations for, 1395t Statistics, 32-37, 32.e1t bivariate, 33-34 common pitfalls, 36-37 correlated data, 36 limitations of, 32 multivariable, 34-35 role of, 32 univariate for categorical variables, 32-33 for numerical variables, 32 Status asthmaticus, 554 Status epilepticus focal, 2401-2402, 2402f treatment of, 2408 Stavudine, 2288t CSF-to-plasma ratios, 2331t side effects of, 2289, 2291

Steakhouse syndrome, 897 Steatohepatitis, 1009-1010 alcoholic, 1019-1023 nonalcoholic, 1019-1023 clinical manifestations of, 1021-1022 HFE mutation analysis in, 1423, 1423.e1f liver biopsy in, 1022f liver transplantation for, 1034 prognosis for, 1022-1023 Steatorrhea, 852t, 922-923 Steatosis macrovesicular, 1019, 1019t microvesicular, 1019, 1019t Stelara (ustekinumab), 170 Stem cell toxicity, direct, 1115-1116 Stem cell transplantation allogeneic, 1128 autologous, 1279-1280 for chronic lymphocytic leukemia, 1256 evaluation of candidates for, 1117-1118 hematopoietic stem cell transplantation, 237, 1198-1204 for acute myeloid leukemia, 1245 for cancer, 1218 in immunocompromised patients, 1855, 1855t matched sibling donor, 1119-1120 matched unrelated donor, 1120 for inborn errors of metabolism, 1389 for myelodysplastic syndrome, 1238 for sickle cell anemia, 1104 Stem cells, 204, 204f adult (postnatal), 204, 205f cancer, 1231 defining properties of, 204 delivery of, 205 embryonic, 204, 206f hematopoietic, 1050 pluripotent ex vivo differentiation of, 206-207 induced, 204, 207f tissue-specific, 205 Stenotrophomonas maltophilia, 1962-1963, 1967-1968, 1970-1971 Stenotrophomonas maltophilia infections clinical manifestations of, 1970-1971 epidemiology of, 1970 mechanisms of resistance, 1971, 1971t prevention of, 1971 prognosis for, 1971 risk factors for, 1970, 1970t treatment of, 1971b Stenotrophomonas maltophilia meningitis, 1971 Stenting for atherosclerotic PAD, 502 choices related to, 458 coronary, 456-457, 457f delivery, 457.e1 inflation, 457.e1 placement, 457.e1 renal artery, 808, 808.e1 Steppage gait, 2341t Stereoidogenesis, 1561.e1f Stereotactic body radiation therapy, 1309t, 1310 Sterility, 1594 Sterilization, 1604, 1605t, 1606f, 1610 Steroid acute regulatory (StAR) protein, 1569, 1569f Steroid contraceptives, 1607-1609 injectable, 1609 intravaginal ring, 1609 transdermal patch, 1609 Steroid hormones, 1583 Steroid suppression test, 1655 Steroidogenesis defects of, 1562-1564 normal, 1561-1562 pathways for, 1561-1562, 1562f Steroidogenic acute regulatory (StAR) protein, 1561 Steroids anabolic-androgenic, 162 for pericardial constriction, 490 Stevens-Johnson syndrome, 2004-2005, 2004f, 2644, 2685t, 2686 antipsychotic-induced, 2626t mucositis in, 2655f pathogenesis of, 2005 Stickler’s syndrome, 2558

Stiffness morning, 1759 muscle, 2544-2545 and rheumatic disease, 1717 Still’s disease, adult-onset, 1713t, 1740, 1740t Stimulant use disorders, 160b Stimulator of interferon genes (STING), 217 Stimulus-control therapy, 2421t Stinging sponge dermatitis, 722 Stingray injuries, 721 Stings, 2176-2177 Stippling, basophilic, 1054f Stivarga (rogorafenib), 1211t-1216t Stokes-Adams attacks, 2599 Stomach. See also under Gastric alcohol-related complications of, 151t benign tumors of, 1319 disorders of rapid transit through, 888 disorders of slow transit through, 885-887 extrinsic neuropathic, 886 intrinsic neuropathic, 886 lymphoma of, 1319 malignant tumors of, 1319 motility disorders of, 886-887 neoplasms of, 1316-1320 in systemic sclerosis, 1782 Stomatitis, aphthous, 2579-2580 clinical features of, 2580t periodic fever with, 1740t Stomatocytes, 1054f, 1084 Stomatocytosis, 1054f hereditary, 1084 dehydrated, 1084 overhydrated, 1084 variants, 1084 Stonefish (Synanceiidae), 721 Stool examination, 854 currant jelly stools, 945 DNA testing, 1328 fat tests, 925-926, 925t Stool weight, 918 STOP-Bang Questionnaire, 2419, 2419t Strabismus, 2558, 2577-2578, 2578f Strachan’s Jamaican neuropathy, 2510 Strain, neck, 2374t Strategic National Stockpile, 1922 Stratum corneum, 2632-2633 Strawberry cervix, 2145 Strawberry tongue, 1796 Streptococcal pharyngitis, 2252t-2253t, 2602 Streptococcus antibody tests, 1910 group A, 1906-1910, 1906t antimicrobial susceptibility, 1910 clinical manifestations of, 1906-1910 epidemiology of, 1906 invasive disease, 1908 otitis media, 1907 pneumonia, 1907 respiratory tract infections, 1906-1907 rhinosinusitis, 1907 group B, 1906, 1906t, 1910-1911 clinical manifestations of, 1911 epidemiology of, 1910-1911 invasive disease, 1911 neonatal disease, 1911 prevention of, 1910-1911 treatment of, 1911b group C, 1911 group G, 1911 nonpneumococcal, 1906-1913 antimicrobial susceptibility, 1910 diagnosis of infection, 1910 treatment of infection, 1910b nonsuppurative sequelae of infections, 1908-1910 nutritionally variant, 477t poststreptococcal glomerulonephritis, 789, 790f, 1910 poststreptococcal reactive arthritis, 1909-1910 viridans group, 476, 1911-1912 zoonotic, 1912 β-hemolytic, 1906, 1906t group A, 1906, 1906t, 1908, 1910 group B, 1906, 1906t, 1910-1911 group C, 1911 group G, 1911 Streptococcus agalactiae, 1906t, 1910-1911 Streptococcus anginosus, 1906, 1906t, 1911-1912 Streptococcus bovis, 1912

Index Streptococcus canis, 1906t, 1912 Streptococcus constellatus, 1906 Streptococcus dysgalactiae subspecies equisimilis, 1906, 1906t, 1911 clinical manifestations of, 1911 epidemiology of, 1911 treatment of, 1911b Streptococcus equi, 1906t Streptococcus iniae, 1912 Streptococcus intermedius, 1906 Streptococcus milleri, 1906, 1911-1912 Streptococcus pneumoniae, 1902 Streptococcus pneumoniae infections, 1902-1906 antibiotic resistance, 1903 clinical manifestations of, 1904 definition of, 1902 diagnosis of, 1904 epidemiology of, 1902-1903 incidence of, 1902 pathobiology of, 1903 prevention of, 1904-1905 prognosis for, 1905-1906 prophylaxis to prevent first episode in HIV infection, 2294, 2295t treatment of, 1905b directed therapy, 1905 empirical, 1905 regimens, 1905 Streptococcus pneumoniae sinusitis, 1947 Streptococcus pyogenes, 1906-1910, 1906t Streptococcus suis, 1912 Streptococcus suis meningitis, 2487 Streptokinase, 180, 447t for pulmonary embolism, 624t Streptomycin for AIDS-associated opportunistic infections, 2297t-2301t for brucellosis, 1981, 1981t for enterococcal infection, 1915.e1t for plague, 89b, 1989t for tularemia, 1983 for verruga peruana, 2000 Stress acute stress disorder, 2353 altered responsiveness, 890 in cancer, 1230 cardiovascular responses to, 266-267, 267.e1f dosing for, 1521 dysregulation of responses, 106-107, 107.e1f environmental, 231 gonadal insufficiency due to, 1574 and neutrophilia, 1130 post-traumatic stress disorder, 2353 Stress cardiac magnetic resonance imaging, 425 Stress echocardiography, 277, 278f, 425, 277. e1 Stress fracture, 1754 Stress incontinence, 111 clinical manifestations of, 112 treatment of stepwise approach, 113t surgery, 114 Stress tests, 248 for chronic stable angina, 423, 423f noninvasive, for acute MI, 453 pharmacologic, 2612-2614 preoperative, 2612, 2617t protocols for, 250t Stress ulcers, 911 Stress-induced (takotsubo) cardiomyopathy, 289, 335, 681, 2519 Stretching, 1822 Striant, 1574t Stridor, 526 Striped catfish (Plotosus lineatus), 721 Stroke acute ischemic arterial hypertension in, 2441 diagnosis of, 2439, 2440f evaluation of, 2440t immediate diagnostic studies in, 2439t management of, 2440-2441 treatment of, 395t, 2440-2441, 2440t-2442t, 2440.e1f atrial fibrillation–related, 2445

Stroke (Continued) blood pressure and risk for, 381, 381f blood pressure reduction after, 392-393 cardioembolic, 2434 cryptogenic, 2444-2445 death rate, 2424 drug-related causes of, 2443 epidemiology of, 2424, 2434 genetic causes of, 2443-2444 heatstroke, 692-693 classic, 692-693, 693t distinguishing characteristics of, 2252t-2253t exertional, 692-693, 693t pathophysiology of, 692.e1f ischemic antithrombotic therapy for, 180t approach to, 2436f definition of, 2434 diagnosis of, 2436-2440 endovascular therapy for, 2441 genetic diseases that can cause, 2443 history in, 2436 initial laboratory tests, 2436-2440 pathobiology of, 2434 physical findings, 2436 thrombolytic therapy for, 180t treatment of, 2441 unusual causes of, 2442-2445, 2443t lacunar syndromes, 2436t management of, 1103 medullary syndromes, 2435, 2435.e2f midbrain syndromes, 2435, 2435.e1f molecularly guided therapeutics for, 202.e1t mortality rates, 2445 with OCs, 1608 ponpmedullary syndromes, 2435.e1f pontine syndromes, 2435, 2435.e1f predicted risks for, 260f-261f prevalence of, 2424 prevention of, 2444b-2445b in asymptomatic carotid stenosis, 2444 primary, 2444 secondary, 2444-2445 prevention of stroke after, 2444-2445 prognosis for, 2445 recovery/rehabilitation after, 2444 recurrence risk, 2445 related to extracranial carotid artery stenosis, 2445 related to intracranial carotid artery stenosis, 2445 risk factors for, 2434t in sickle cell disease, 1103 unusual causes of, 2442-2445, 2443t Stroke Scale (NIH), 2436, 2437t-2438t Stroke volume calculation of, 276 formula for, 276 Stromelysins, 233t Strongyloides fuelleborni, 2163 Strongyloides stercoralis, 1015t-1017t, 2159, 2162 Strongyloidiasis, 2162-2163 clinical manifestations of, 2163 diagnosis of, 2163 diagnostic evaluation of, 1153 epidemiology of, 2162 pathobiology of, 2162-2163 prevention of, 2163 treatment of, 2160t, 2163b Structured Clinical Interview, 1457 Struvite (magnesium ammonium phosphate) crystals, 731-732, 732f Struvite stones, 812, 814-816 Stuart-Prower factor (factor X) deficiency, 1175t Stunting, 1434, 1837-1838 definition of, 1434 global rates, 1434 moderate, 1435-1436 severe, 1435-1436 treatment of, 1436 Stupor, 2410t Sturge-Weber syndrome, 2516, 2516f Stuttering, 2385 Stye (hordeolum), 2561, 2561f Subacromial bursa, 1750f

Subacute sclerosing panencephalitis, 2209-2210 clinical manifestations of, 2210 diagnosis of, 2210 epidemiology of, 2209-2210 pathobiology of, 2209-2210 treatment of, 2210b Subacute sensory neuropathy, 2533 Subarachnoid cysticercosis, 2150 Subarachnoid hemorrhage, 2445-2450 acute, 2448t angiography findings, 2447 causes of, 2446, 2446t, 2448-2449 cerebrospinal fluid formula, 2342t clinical manifestations of, 2446 CT findings, 2446-2447, 2446f diagnosis of, 2446-2447 epidemiology of, 2445-2446 Hunt-Hess grading scale for, 2447, 2447t laboratory testing in, 2447 lumbar puncture findings, 2447 management protocol for, 2448t medical complications of, 2449 MRI findings, 2447 pathobiology of, 2445-2446 prevention of, 2449 prognosis for, 2449-2450 rebleeding, 2447-2448, 2448f treatment of, 2447b-2449b Subclinical disease, 100 Subcutaneous fat, 2634-2635 Subcutaneous onchocerciasis, 2169 Subcutaneous panniculitis-like T-cell lymphoma, 1266 Subcutaneous phaeohyphomycosis, 2102 Subdural abscess, 2498, 2498f Subdural effusions, 2481 Subdural empyema, 2498 clinical features of, 2498 diagnosis of, 2498 prognosis for, 2498 treatment of, 2498b Subdural hematoma, 2366, 2366f alcohol-related, 2510, 2511f Subepithelial neoplasms, 2583t Subglottic edema, 2189, 2189f Subglottic tumors, 1299, 1301t Subjective Global Assessment, 1433 Submersion incidents, 595-596 Sub-Saharan Africa HIV/AIDS epidemic, 2272-2278, 2272.e1 life expectancy, 100 meningitis belt, 75, 1882, 2480 Subspecialty consultations, 2609, 2609t Substance abuse. See also Drug abuse in adolescents, 64 drugs of abuse, 156-162 in women, 1603 Substance Abuse and Mental Health Services Administration, 159 Substance use disorders. See also Drug use disorders clinical manifestations of, 157 definition of, 156 DSM-5 criteria for, 156, 157t epidemiology of, 157 pathobiology of, 157 Substituted judgment standard, 6 Subungual onychomycosis, 2709, 2709f Succimer (DMSA), 706t-710t Succinylcholine, 2619 Sudden cardiac arrest, 352 Sudden death antipsychotic-induced, 2627 prevention of, 326 Sudden visual loss, 2558, 2559t Sugammadex, 2619-2620 Suicidal behavior, 15 Suicidality, 2355 Suicide, 2355 in adolescents, 61 assisted suicide, 7-8 dietary factors, 1427t Suicide attempts, 2355, 2356t Suicide genes, 212 Suicide risk, 2355, 2356t Sulbactam for bacterial endocarditis, 479t for peritonitis, 949

I95

Sulbactam (Continued) for pneumonia, 616t, 619t for ulcerative colitis, 942 Sulfacetamide (Albucid), 1088t Sulfadiazine, 1891t-1892t for AIDS-associated opportunistic infections, 2297t-2301t for congenital toxoplasmosis, 2132t for toxoplasmic retinitis, 2566 for toxoplasmosis, 2129t, 2132t Sulfadoxine-pyrimethamine (Fansidar), 2111 Sulfamethoxazole, 1891t-1892t for AIDS-associated opportunistic infections, 2297t-2301t for coccidian enteritis, 2147 for pertussis, 1992 Sulfasalazine (Azulfidine), 1761 caveats for, 1761t for Crohn’s disease, 940 for inflammatory bowel disease, 939t and micronutrient status, 1453t for rheumatoid arthritis, 1761 for spondyloarthritis, 1764 Sulfate diarrhea, 933 Sulfonamides, 1889t, 1891t-1892t contraindications to, 1088t resistance to, 1890t toxicities, 1895t Sulfones, 1088t Sulfonylurea-induced hypoglycemia, 706t-710t Sulfonylureas, 1535 Sulfur, precipitated, 2173 Sumatriptan for acute mountain sickness prophylaxis, 596-597 for headache, 2358-2359, 2359t, 2361, 2368 Summer cold, 2243 Summer grippe, 2241t Summer penile syndrome, 2173-2174 Sun protection factor (SPF), 2661 Sun sensitivity, 2687t Sunflower cataracts, 1417 Sunitinib (Sutent), 170, 1211t-1216t Sunscreens, 1376-1377, 2661 Superantigens, 1796 Superficial actinic porokreatosis, 2651f Superficial onychomycosis, 2709, 2709f Superficial phaeohyphomycosis, 2102 Superficial thrombophlebitis, 518 Superior cerebellar artery occlusion, 2435, 2435t Superior mesenteric angiography, 953f Superior mesenteric artery, 951 Superior mesenteric embolism, 953-954 clinical manifestations of, 954 conditions associated with, 954t diagnosis of, 954 treatment of, 954b Superior mesenteric thrombosis, 953f, 954 clinical manifestations of, 953f, 954 diagnosis of, 954 treatment of, 954b Superior sulcus sign, 636 Superior vena cava obstruction, 636, 636f Superoxide dismutase, 235 Supplements, 57, 1455 Supportive care, 1218-1219 Suppurative (acute) thyroiditis, 1509 Supraglottic tumors, 1299, 1301t Supraglottitis, 2604 Supranuclear palsy, progressive, 2606 Suprasellar dysgerminoma, 1476 Supraspinatus muscle, 1750f Supraventricular arrhythmias, 326, 356-367 Supraventricular rhythms, 359 Supraventricular tachyarrhythmias, 359-363, 452 Supraventricular tachycardia, 350t-351t, 360t clinical manifestations of, 363 definition of, 359 diagnosis of, 359, 363 differential diagnosis of, 352, 369, 370t long-term management of, 366-367 positive concordance with, 369-370 radiofrequency ablation of, 380 site and mechanism of, 360f treatment of, 363b-367b Suramin, 2115, 2115t Volume I  pp 1-1382  •  Volume II  pp 1383-2722

I96

Index

Surgery for ambiguous genitalia, 1567 anesthetic approaches for, 2618, 2618t for aortic regurgitation, 471 aortic valve replacement, 464 arrhythmia, 380-381 for arthritis, 1832-1833 for bacterial meningitis, 2489 bariatric, 1466, 1535 for benign lung disease, 643-644 for benign prostatic hyperplasia, 830 for brain tumors, 1290 for breast cancer, 1354 for bronchectasis, 569 bronchoscopic approaches, 643 for burns, 716 for cancer, 1207 for carcinoid syndrome, 1559 cardiac, 374-376, 2618t cardiac evaluation and care algorithm for, 2612, 2614f cataract, 2563f, 2566 for cirrhosis, 1031 for colorectal cancer, 1330 for constipation, 889 for COPD, 561 coronary artery, 458-459 coronary artery bypass graft, 456 for Cushing’s syndrome, 1517 for cystic echinococcosis, 2152 damage control, 715 dermatologic, 2661 for esophageal squamous cell carcinoma, 1315 gastric bypass, 1552 for gastric cancer, 1318 for gastroparesis and pseudo-obstruction, 888 for genital warts, 2222t for head and neck cancer, 1302 for heart failure, 317 for hereditary spherocytosis, 1083 for hyperparathyroidism, 1653t, 1655 for incontinence, 114 for infective endocarditis, 481-482, 481t for inflammatory bowel disease, 942 for intracerebral hemorrhage, 2451-2452 intraoperative management, 2618-2620 joint arthroplasty innovations, 1830-1832 for joint disease, 1828-1833 less invasive, 1830-1831 for lung cancer, 644, 1312 for lung disease, 642-647 approaches, 643 open approaches, 643 lung volume reduction, 644 desicision guide for, 645t exclusion criteria for, 644t inclusion criteria for, 644t mortality, 644, 644f for malignant lung disease, 644 minimally invasive, 1830-1831 for mitral regurgitation, 468-469 mitral valve, 468-469 Mohs’ microsurgery, 1378 for mucormycosis, 2091 for non-small cell lung cancer, 1309-1312 for obstructive sleep apnea, 642 operative risk assessment, 2611 orthopedic anesthesia in, 1829 indications for, 1828 preoperative considerations, 1828-1829 procedures, 1829-1832 for osteoarthritis, 1749 for Paget’s disease of bone, 1667 for pain, 141-142 parathyroid, 1653t, 1655 for Parkinson’s disease, 2460 for pectus excavatum, 630 perioperative anaphylaxis, 2618 perioperative medications, 2611-2612 for pheochromocytoma, 1526 pituitary, 1484, 1484.e1 for pituitary tumors, 1484 for polycystic ovary syndrome, 1594 postoperative care, 2621 postoperative complications, 2621-2625 postoperative consultation, 2608 postoperative diabetes insipidus, 1500, 1500f postoperative infection, 1832

Surgery (Continued) postoperative jaundice, 988-991 postoperative nausea and vomiting, 2620 postoperative thrombosis, 1191 for premenstrual syndrome, 1587 preoperative consultation, 2608 preoperative evaluation, 1158, 2611-2617, 2617t pulmonary risk patient-related factors, 2615 procedure-related factors, 2615 pulmonary risk reduction strategies, 2615 for rhinitis, 2588 risks of procedures, 2613t for seizures, 2407-2408 in sickle cell anemia, 1103 for sinusitis, 2588 smaller incision, 1830-1831 suggested minimum platelet counts before, 1161t for tetanus, 1930 for thyroid disorders, 1508 transsphenoidal for Cushing’s disease, 1491 for growth hormone excess, 1486 for trauma, 715-716 video-assisted thoracoscopic, 643 von Willebrand factor replacement for, 1170t for Zollinger-Ellison syndrome, 1337 Surgical assessment, 1032 Surgical consultation, preoperative, 2608 Surgical decompression, 2368 Surgical history, 24 Surgical shunts, 405t, 406 Surgical site infections, 1866-1867 burden, costs, and preventability of, 1862t guidelines and recommendations for, 1865.e1t staphylococcal, 1898 Surrogate historians, 2338 Surveillance, immunologic, 2632 Surveillance testing, acute, 1868 Survey primary, 713 secondary, 713-714 tertiary, 714 Survival motor neuron gene (SMN), 2525 Survivor management, 1364 Susceptibility testing, 1844 dilution method, 1844 disc diffusion method, 1844 gradient diffusion method, 1844 Sutent (sunitinib), 170, 1211t-1216t Sutterella wadsworthensis, 1931 Swallowing, normal, 2523.e1 Swansea criteria, 1622, 1622t Sweat glands, 2634 Sweating: drugs that affect, 2520, 2520t Sweating disorders, 2521 Sweet’s syndrome, 1859, 2673f, 2682, 2686, 2692, 2692f clinical manifestations of, 2682 treatment of, 2682b Swimmer’s ear, 2590 Swimmer’s itch, 2698 Swimming, 1884 Swiss cheese model of accidents, 45 Switzerland, 7-8 Swollen belly syndrome, 2162-2163 Swyer syndrome, 1565 Swyer-James syndrome, 570 Swyer-James-McLeod syndrome, 534-535, 570 Sydenham’s chorea, 2463 Sylvatic typhus, 2052 Symmetrical polyneuropathy, distal, 2536 Sympathetic dysfunction, regional, 2522 Sympathetic ophthalmia, 2571 Synanceiidae (stonefish), 721 Syncope, 248-251, 345-346 in aortic stenosis, 463 arrhythmic causes of, 350t-351t causes of, 345t, 350t-351t clinical features of, 345t, 2403t indications for initial observation and rapid evaluation of, 347t neurocardiogenic, 346-347, 350t-351t with palpitations, 344, 344f prognosis for, 351 treatment of, 350b vasovagal, 345, 345t

Syndactyly, 1669 Syndesmophytes, 1766, 1766f Syndrome of inappropriate secretion of antidiuretic hormone, 1220, 1496 causes of, 751-752, 751t diagnosis of, 750 drug-induced, 2626t, 2629 serum ADH abnormalities in, 751-752, 751f Syndromes, 3. See also specific syndromes by name Synovectomy, 1829 Synovial fluid sampling, 1748 Synovitis rheumatoid, 1756, 1756f ultrasonography findings, 1725-1726, 1726f Synovitis acute pustulosis hyperostosis osteitis syndrome, 1740t Synovium, 1730t, 1745-1746 SYNTAX trial, 460f, 461 Syphilis, 1877t, 1878, 2013-2020 cardiovascular, 2015-2016 clinical manifestations of, 2014-2016, 2014f color in, 2638t-2644t congenital clinical manifestations of, 2016 diagnosis of, 2018 treatment of, 2019 definition of, 2013 diagnosis of, 2016-2018 differential diagnosis of, 2017 early infectious, 2018 endemic, 2020 epidemiology of, 1869t, 2013-2014 follow-up examinations, 2019 in HIV infection, 2016, 2294, 2295t, 2319-2320 incidence of, 1877, 2013 late, 2015-2016, 2015t late benign, 2015 latent, 2015 lesions of, 2014, 2014f meningovascular, 2016 musculoskeletal involvement, 1809 natural course of, 2014 neurosyphilis, 2016 oral ulcers of, 2580t, 2581 pathobiology of, 2014 in pregnancy, 2018-2019 prevention of, 2019, 2294, 2295t primary, 2014 prognosis for, 2019 relapsing, 2015 secondary, 2014-2015, 2319-2320, 2320f, 2582, 2639f, 2668, 2669f serologic tests for, 2017-2018, 2017t tertiary, 2015, 2015t testing for, 1878, 1879f treatment of, 2018b-2019b, 2019 uveal, 2566 Syphilitic meningitis, 2016 Syringohydromyelia, 2514 System for Thalidomide Education and Prescribing Safety (STEPS) program, 2046 Systemic anaphylaxis, 1698-1703 causes of, 1698, 1699t definition of, 1698 diagnosis of, 1701-1702, 1701f differential diagnosis of, 1702 epidemiology of, 1698 pathobiology of, 1698-1700 pathophysiology of, 1700 prevention of, 1702-1703 treatment of, 1702b, 1702f Systemic antifungal agents, 2064-2070 Systemic diseases with arthritis, 1823-1828 autoimmune, 1713t, 1715t-1716t autoinflammatory, 1739-1744, 1740t definition of, 1739 inherited, 1743-1744 with decreased salivary secretion, 2584-2585 drug rash with eosinophilia and systemic symptoms, 2685-2686, 2685t with hypogonadism, 1573 inflammatory with gastroduodenal ulcers, 911 laboratory evaluation of, 1720-1723

Systemic diseases (Continued) with pleural effusions, 634 proteasome-associated, 1743 with thrombosis, 1190-1191 with ocular symptoms, 2568-2569 of throat and laryngitis, 2605 Systemic fibrosis, nephrogenic, 1728 Systemic lupus erythematosus, 1769-1777 autoantibodies associated with, 1716t, 1770, 1771f, 1775t biologic therapy for, 1776-1777 cardiovascular involvement, 1772-1773 classification of, 1713t, 1774, 1774t clinical manifestations of, 1771-1774, 1771t constitutional symptoms of, 1771 cutaneous manifestations of, 1771, 1771f definition of, 1769 diagnosis of, 1774-1775 EBV-associated, 1770 environmental triggers, 1770 epidemiology of, 1769 esophageal involvement, 906 eye manifestations of, 2571 gastrointestinal involvement, 1773 genes associated with, 1769-1770, 1770.e1t hematopoietic stem cell transplantation for, 1203 immunologic triggers, 1770, 1770.e2f incidence of, 1769 inflammation pathways in, 230.e3t kidney involvement in, 1772 laboratory findings, 1774-1775 management of serious organ-system disease in, 1775 neuropathy in, 2533, 2533t neuropsychiatric manifestations of, 1773 oral mucosal lesions of, 2583 pathobiology of, 1769-1771, 1770.e2f pleural fluid characteristics, 634, 634t preconception interventions for women, 1612t prevalence of, 1769 prognosis for, 1777 pulmonary involvement, 585, 1773 renal involvement, 792 symptoms and signs of, 1771-1774 treatment of, 1775b-1777b adjunctive therapies, 1777 ancillary therapies, 1776-1777 conventional medical therapy, 1775 future directions, 1777 Systemic mastocytosis, 912, 1706-1707 aggressive, 1709 biopsy for, 1708 classification of, 1709f diagnosis of, 1708-1709 diagnostic criteria for, 1708-1709, 1708t diagnostic findings, 1708, 1708f indolent, 1709 Systemic phaeohyphomycosis, 2102 Systemic roundworms, 2103-2104 Systemic sclerosis, 1777-1785 animal models, 1780.e1 antifibrotic therapy, 1784 autoantibodies associated with, 1716t and cancer, 1784 cardiac involvement, 1783 characteristic autoantibodies in, 1780-1781, 1780t classification of, 1713t, 1778, 1778t clinical manifestations of, 1781-1784 cutaneous diffuse, 1781 limited, 1781 definition of, 1777-1778 diagnosis of, 1784 environmental exposures and, 1778-1779 epidemiology of, 1778-1779 etiology of, 1778-1779 facial features in, 1781, 1781f gastrointestinal complications of, 1782, 1784 genetic factors, 1779 initial clinical presentation of, 1781 kidney involvement, 1783 laboratory findings, 1784 lung disease associated with, 584, 1782-1783 musculoskeletal complications of, 1783 natural history of, 1785 occupational exposures and, 1778-1779

Index Systemic sclerosis (Continued) organ-specific complications of, 1781-1785 pathobiology of, 1779-1781 pathogenesis of, 1779, 1779f pathology of, 1779-1780, 1780f pathophysiologic triad of, 1780 prevention of, 1784b-1785b prognosis for, 1785 progressive, 584 skin care in, 1785 skin thickening in, 1781, 1781f treatment of, 1784b-1785b vascular complications of, 1780-1781, 1782f Systemic therapy, adjuvant, 1355-1356, 1355t Systemic vascular resistance (SVR), 672, 687t Systemic vascular resistance index (SVRI), 687t Systemic vasculitic neuropathy, 2533 Systemic vasculitis, 735, 1793-1801 classification of, 1793-1794, 1794t clinical manifestations of, 1796-1800 definition of, 1793 diagnosis of, 1800-1801, 2534 differential diagnosis of, 1800-1801 epidemiology of, 1794-1795, 1794t neuropathies in, 2533, 2533t pathobiology of, 1795 pathophysiology of, 1795-1796 prognosis for, 1801, 2534 superantigen model, 1796 treatment of, 1801, 1801b, 2534 Systemic venous pressure, 253t Systemic venous return, 415 Systems review, 25, 25t Systems thinking, 45 Systolic arterial pressure (SAP), 687t Systolic murmurs bedside identification of, 254, 255t quiet, 27 Systolic thrills, 27 Systolic ventricular function, 279

T

T- and B-cell combined defects, 1678-1679, 1678t, 1679b T cells, 222-223 accessory molecules, 222-223 biologic agents targeting, 171 adverse effects of, 171 indications for, 171 types of, 171 CD4+, 236 depletion of, 2279, 2280f gastrointestinal disease and, 2302, 2303t gastrointestinal diseases with CD4+ counts < 200 cells/µL, 2304-2305 gastrointestinal diseases with CD4+ counts > 200 to 500 cells/µL, 2302-2304 in graft rejection, 236t skin conditions that can occur at any CD4+ count, 2319t skin conditions with CD4+ counts < 200 cells/µL, 2318t CD8+, 236, 236t chimeric antigen receptor–directed, 1256 development of, 222 differentiation of, 223 effector functions of, 223 helper activation of, 1688-1689 follicular (TFH), 227 type 1 (TH1), 227 type 2 (TH2), 227 type 17 (TH17), 227 homeostasis of, 223 maturation of, 222, 222f regulatory, 223 stimulation of, 222-223 trafficking of, 238-239 T wave, 267-268, 273 T3 (triiodothyronine), 1500-1501, 2616 T4 (thyroxine), 1500-1501, 1504, 2616 Tabes dorsalis, 2016 Tacaribe complex viruses, 2247 Tachyarrhythmia, 31, 346, 350t-351t supraventricular, 359-363, 452 Tachyarrhythmic cardiac arrest, 354

Tachy-brady syndrome, 350t-351t, 357f Tachycardia, 31 accessory pathway, 360-361, 364-365 atrial, 360, 360f, 360t acute therapy for, 363-366 long-term management of, 366 multifocal, 360f, 363 atrioventricular junctional, 360t atrioventricular re-entrant, 360, 360t, 362f, 363 acute therapy for, 363-366 diagnosis of, 363 long-term management of, 366 differential diagnosis of, 348 junctional, 360, 361f, 363-366 narrow-complex, 348, 348f postoperative, 2623 sinus, 359, 359f, 363-366 supraventricular. See Supraventricular tachycardia toxicant-induced, 696-697 ventricular. See Ventricular tachycardia wide-complex, 348, 349f Tachypnea, 31 Tacrolimus (Prograf), 167 adverse effects of, 167, 1037t for glomerulosclerosis, 786 for immunosuppression, 1037-1038 for minimal change disease, 785-786 for pruritus, 2636t in renal transplantation, 844t topical, 2660 Tactile fremitus, 526 Tadalafil (Cialis), 429 for altitude sickness, 597t for benign prostatic hyperplasia, 830t for erectile dsyfunction, 1578-1579 for high-altitude pulmonary edema prevention, 596-597 for pulmonary hypertension, 402 Taenia, 2148t Taenia asiatica, 2148t, 2149 Taenia crassiceps, 2153 Taenia multiceps, 2148t, 2153 Taenia saginata, 2147, 2148t, 2149, 2149b Taenia serialis, 2153 Taenia solium, 2147-2151, 2148t Taeniasis, 2149, 2149b Tagamet (cimetidine), 2660t Tai Chi, 110, 182t, 183, 1642 Taipan envenomation, 720 Taipans (Oxyuranus), 720 Takayasu’s arteritis, 497, 1796.e1f classification of, 1794 clinical manifestations of, 1796 epidemiology of, 1794-1795, 1794t gastrointestinal, 956 pathophysiology of, 1795, 1795t treatment of, 1796 Takayasu’s retinopathy, 1796 Takotsubo cardiomyopathy, 289, 335, 681, 2519 Talecris, 1170 TALENs (transcription activator−like effector nucleases), 210-211, 213 Tamm-Horsfall proteins, 730-731 Tamoxifen (Nolvadex) for cancer, 1211t-1216t, 1357-1358 ocular effects of, 2573t Tamponade, 486-489 Tamsulosin for benign prostatic hyperplasia, 830t for kidney stones, 813-814 for urinary retention, 2477 Tandem repeats, 191 Tanezumab, 2375 Tanner stages, 61, 1588, 1588t Tapentadol, 137, 139t-140t Tapeworms, 1107, 2147, 2148t intestinal infections, 2104, 2148-2149, 2148b, 2148t Tarabine PFS (cytarabine), 1211t-1216t Tarceva (erlotinib), 171, 1211t-1216t Tardive akathisia, 2468 Tardive dyskinesia, 2468 clinical manifestations of, 2468 epidemiology of, 2468 pathobiology of, 2468 prognosis for, 2468 treatment of, 2468b

Tardive dystonia, 2468 Tardive myoclonus, 2468 Tardive tics, 2468 Tardive tremor, 2468 Target cells, 1053f, 1059t Target population, 32 Targeted small molecules, 1209-1218 Targeted therapy, 1231-1232, 1232t for cancer, 202, 202t, 2661-2662 efficacy, 1232 for lung cancer, 1310t, 1311-1312 for pulmonary hypertension, 402 resistance associated with, 1232 side effects of, 1232 for traumatic spinal cord injury, 2368 Targretin (bexarotene), 2658 Tarsal tunnel syndrome, 1759, 2536 Tasigna (nilotinib), 171, 1211t-1216t Task-specific dystonia, 2463-2465 Taste, 2592-2593 disorders of diagnosis of, 2593 pathobiology of, 2592 pathophysiology of, 2592-2593 treatment of, 2593b loss of, 2593, 2593t peripheral pathways of, 2592-2593, 2592f Taste buds, 2592 Taurine, 925t Taxol (paclitaxel), 1211t-1216t Taxotere (docetaxel), 1211t-1216t, 1319 Taylor Cyanide Antidote Package (Taylor Pharmaceuticals), 599-600 Tay-Sachs disease, 1387, 1399-1400, 1403 incidence of, 1385t ocular symptoms of, 2569 therapeutic strategies for, 1388t Tazarotene (Tazorac), 2658 Tazobactam, 2696-2697 T-cell disorders, 1676-1677, 1677t, 1686t T-cell leukemia/lymphoma, 1255, 1266, 2236-2238, 2237t acute, 2238 aggressive chronic, 2238 angioimmunoblastic, 1074, 1266 causative factors, 1228 chronic, 2238 clinical features of, 1261t, 2236 cutaneous involvement, 2236-2237, 2238f diagnosis of, 2237-2238 enteropathy-type, 1266 epidemiology of, 2236 extranodal, 1266 hepatosplenic, 1266 incidence of, 2236 lymphoblastic, 1240t lymphoma-type, 2238 mature, 1265-1266 morphologic features of, 2236, 2237f pathobiology of, 2236 pathogenesis of, 2236 peripheral, 1265-1266 clinical characteristics of, 1261t unspecified, 1266 prognosis for, 2238 smoldering, 2238 subcutaneous panniculitis-like, 1266 subtypes, 1266 treatment of, 2238b T-cell malignancy, 1258, 1259f T-cell receptor chain defects, 1678t T-cell receptors, 220-221 T-cell tolerance, 239-240 Td (tetanus and diphtheria toxoid) vaccine, 68t-73t, 76-77, 1917 adverse reactions to, 77 indications for, 68f, 76 recommendations for, 66-67, 67f-68f, 1929t, 67.e1t-67.e3t Tdap (tetanus, diphtheria, and pertussis) vaccine, 68t-73t, 76, 1917, 1929, 1929t, 1992 indications for, 68f recommendations for, 66-67, 67f-68f, 1929, 67.e1t-67.e3t Team care, 3, 48 comanagement, 2610 counseling teamwork, 54 intensivist-led multidisciplinary teams, 650 postoperative care, 2621

I97

Teardrop poikilocytes, 1054f Tearing, excessive, 2559 Tecfidera (dimethyl fumarate), 2477 Technology, 1198 assistive reproductive, 1596 implant, 1831 information technology, 47 molecular, 200-203, 200f, 203.e2t Tecovirimat, 2219 Tedania (fire sponges), 722 Teduglutide, 932 Teflon fever, 594t Teicoplanin, 1889t Telangiectasia, 2694-2695 gastrointestinal, 958 hereditary hemorrhagic, 1172, 1172f mucosal, in colon, 875f regional involvement, 2702t scarring or atrophy with, 2693t Telangiectasia macularis eruptiva perstans, 1707 Telaprevir, 2177t-2178t, 2180 Telavancin, 1889t, 1901 Telbivudine, 2178t, 2179 adverse effects of, 2178t for hepatitis B, 1002, 2177t Teletherapy, 1207 Telithromycin, 1891t-1892t, 1894 Telmisartan, 388t Telogen effluvium, 2703 acute, 2703, 2703b chronic, 2703 Telomerase, 191 Telomeres, 191 Temaril (trimeprazine), 2660t Temazepam, 2352t Temosirolimus (Torisel), 1211t-1216t, 1348 Temovate (clobetasol propionate), 2658t Temozolomide (Temodar), 1211t-1216t, 1291, 1293 Temperature regulation, 691, 1849, 691.e3f cutaneous, 2632 hypothalamic disease and, 1479 Temporal arteritis, 1801-1805 definition of, 1801 headache of, 2362 ocular symptoms of, 2570 Temporal artery superficial assessment of, 1803-1804 vasculitis of, 1713t Temporal artery biopsy, 1804 Temporal bone fractures, 2599 Temporal lobe epilepsy, 2400f mesial, with hippocampal sclerosis, 2406 with vertigo, 2599 Temporal lobe glioblastoma, 1293, 1293f Tendinitis, 1750t Achilles, 1754 bicipital, 1752 definition of, 1749-1750 diagnosis of, 1750 epidemiology of, 1750 patellar, 1753 posterior tibial, 1754 rotator cuff, 1751 treatment of, 1750b-1751b Tendinopathy, 1749-1750 Tendinosis, 1749-1750 Tendon, 1730t Tendon insufficiency, 1749-1750 Tendon rupture, 1750t Tendonitis, 1713t Tenecteplase (TNK-tPA), 180, 447t Tenesmus, 2303t Tennis elbow, 1751-1752 Tenofovir, 2178, 2178t, 2288t CSF-to-plasma ratios, 2331t fixed-dose combinations, 2289, 2290t for hepatitis B, 1001-1002, 2177t for HIV/AIDS, 2289, 2290t side effects of, 2289, 2291 for travel-related health problems, 1884 Tenosynovial giant cell tumors, 1827-1828 Tenosynovitis, 1749-1750 de Quervain’s, 1752, 1752f volar flexor, 1752-1753 Tensilon (edrophonium), 2551 Tension hydropneumothorax, 538f Volume I  pp 1-1382  •  Volume II  pp 1383-2722

I98

Index

Tension pneumothorax, 636 Tension-type headache, 2359-2360 clinical manifestations of, 2359-2360 definition of, 2359 diagnosis of, 2360 differential diagnosis of, 2357t epidemiology of, 2359 pathobiology of, 2359 prevention of, 2360 prognosis for, 2360 treatment of, 2360b Teratogens, 1611 Teratology Information Service, 1611 Teratoma, anterior mediastinal, 538, 539f Terazosin, 388t, 830t Terbinafine for chromomycosis, 2103 for fungal infections, 2670 for onychomycosis, 2709 for sporotrichosis, 2078-2079 Teriflunomide (Aubagio), 2477 Teriparatide, 1643-1644, 1643t Terlipressin, 1030 Termination of medical interventions, 6-7 definition of, 6 empirical data for, 6 history of, 6 justification for, 6 pracical considerations, 6-7, 7t Terminology acute respiratory function abbreviations, 656t for alcohol use, 150t for describing symptoms, 2338 for secondary features of skin disease, 2651t-2654t for skin lesions, 2636t, 2637, 2647t-2651t Ternidens diminutus, 2164 Terrorism bioterrorism, 86-92 intimate, 1629-1630 radiologic, 83, 85 Tertiary survey, 714 Test statistic, 35 Testicular cancer, 1365-1366 clinical manifestations of, 1365 diagnosis of, 1365-1366 disseminated or persistent disease, 1366 epidemiology of, 1365 incidence of, 1365 local and regional disease, 1366 pathobiology of, 1365 poor risk disease, 1366, 1366t prevention of, 1366b prognosis for, 1366 sequelae in long-term survivors, 1366 treatment of, 1366b Testicular examination, 1366 Testicular failure autoimmune, 1573 primary, 1572t Testicular feminization syndrome, 1564 Testicular hypogonadism, primary, 1573 Testicular lymphoma, 1267 Testicular regression syndrome, 1565-1567 Testim (testosterone), 1574t Testis (testes) components of, 1569 intra-abdominal, 1567-1568 physiology of, 1568-1572 Testoderm TTS (testosterone), 1574t Testogel (testosterone), 1574t Testosterone, 1569 for ambiguous genitalia or micropenis, 1567 decreased levels, 1571-1572 effects of, 1569, 1569f for erectile dsyfunction, 1578-1579 free, 1571f implants, 1574t, 1575 increased levels, 1589 plasma levels, 1571f preparations, 1574t serum levels, 1574 synthesis of, 1569, 1569f transbuccal mucoadhesive, 1574t, 1575 transdermal, 1574t, 1575 Testosterone cyclopentylpropionate, 1483t Testosterone cypionate, 1574t, 1575 Testosterone deficiency, 1571-1572 Testosterone enanthate, 1483t, 1574t, 1575

Testosterone gel, 1483t, 1575 Testosterone patch, 1487t, 1575 Testosterone replacement for hypopituitarism, 1483t for male hypogonadism, 1575 Testosterone undecenoate, 1574t, 1575 Tests. See Laboratory testing Tetanus, 1928-1930 antimicrobial therapy for, 1930 cephalic, 1929 clinical manifestations of, 1929 definition of, 1928-1929 diagnosis of, 1929 epidemiology of, 1929 immunization against active, 1930 passive, 1930 recommendations for, 66t, 1929 pathobiology of, 1929 prevention of, 1929-1930, 1929t prognosis for, 1930 supportive care for, 1930 surgery for, 1930 treatment of, 1930b Tetanus, diphtheria, and pertussis (Tdap) vaccine, 68t-73t, 76, 1917, 1929, 1929t, 1992 indications for, 68f recommendations for, 66-67, 67f-68f, 1929, 67.e1t-67.e3t Tetanus and diphtheria toxoid (Td) vaccine, 68t-73t, 76-77, 1917 adverse reactions to, 77 indications for, 68f, 76 recommendations for, 66-67, 67f-68f, 1929t, 67.e1t-67.e3t Tetanus immune globulin (TIG), 1929-1930, 1929t Tetany, 2539 Tethered spinal cord, 2513-2514 Tetrabenazine for chorea, 2463 for dystonia, 2465 for tardive dyskinesia, 2468 for Tourette’s syndrome, 2466 Tetracycline, 1891t-1892t, 1894-1896 for acne vulgaris, 2680 for bacterial overgrowth, 928 for blepharitis, 2561 for brucellosis, 1981t for diarrhea, 926 dose-response effects, 1008t for enteric protozoal infections, 2147t for H. pylori eradication, 915t mechanism of action, 1889t for psittacosis, 2012 resistance to, 1890t, 1961 for rosacea, 2680 for S. pneumoniae infection, 1905t for syphilis, 2018 toxicities, 1895t for tropical sprue, 931, 1108 Tetracycline labeling, double, 1647, 1648f Tetrahydrocannabinol, 699-702, 702t Tetralogy of Fallot, 413-414 chest radiography findings, 285, 285f exercise recommendations for, 416t incidence and prevalence of, 405 Tetralogy of Fallot repair, 414, 414f Tetrodotoxin (puffer fish) poisoning, 722 Thalamic deep brain stimulation, 2462 Thalamogeniculate arteries, 2429f Thalamoperforators, 2427 Thalamotomy, 2462 Thalamus: hemorrahage in, 2451 Thalassemia, 1089-1095, 1202-1203 classification of, 1089, 1089t clinical manifestations of, 1090-1093 definition of, 1089 diagnosis of, 1093-1094 epidemiology of, 1089 experimental therapies for, 1094-1095 genetics of, 1089 molecular basis, 1089-1090 pathobiology of, 1089-1090 syndromes, 1089 treatment of, 1094b-1095b α-Thalassemia, 1056 classification of, 1089t clinical manifestations of, 1090-1091 molecular basis, 1089-1090

β-Thalassemia, 1056 classification of, 1089t, 1090-1091, 1091f clinical manifestations of, 1091-1092, 1093f molecular basis, 1089-1090 pathophysiology of, 1091-1092, 1092f treatment of, 1095 δβ-Thalassemia classification of, 1089t clinical manifestations of, 1093, 1093f molecular basis, 1090 Thalassemia intermedia, 1091-1092, 1094 Thalassemia major, 1091-1092, 1094 Thalassemia minor, 1053f, 1091-1092 Thalidomide (Thalomid), 2659 for cancer, 1211t-1216t for leprosy reactional states, 2046 for multiple myeloma, 1279-1280, 1280t for primary myelofibrosis, 1128 System for Thalidomide Education and Prescribing Safety (STEPS) program, 2046 Thallium poisoning, 97, 703t chelators for, 93t diagnostic testing for, 93t Thelarche, 61, 1581 Theophylline, 126t for acute mountain sickness, 597 for asthma, 553 for chronic obstructive pulmonary disease, 559, 560t in renal failure, 129t toxic levels, 703t Therapeutic hyperthermia, 695 Therapeutic hypothermia, 451, 695 Therapeutic index, 127 Therapeutic privilege, 6 Therapeutic touch, 184 Therapeutic window, 127 Thermal disorders, 507-511, 508f Thermal effect of feeding, 1433 Thermal injury, 712f, 712t clinical manifestations of, 711 to lungs, 598 smoke inhalation and, 598-599 to upper airway, 598 clinical manifestations of, 598-599 diagnosis of, 599 pathogenesis of, 598 prognosis for, 599 treatment of, 599b Thermometers, 26 Thermoplasty, bronchial, 642 Thermoregulation cutaneous, 2632 effects of hypothalamic disease on, 1479 Thermoregulation disorders. See Hyperthermia; Hypothermia Thiamine (vitamin B1) for cold injury, 695 for coma, 2412 functions of, 1446t-1450t for vitamin B1 deficiency, 2507t for Wernicke’s encephalopathy, 2507t Thiamine (vitamin B1) deficiency, 2506-2508, 2507t cardiomyopathy with, 330 clinical manifestations of, 2506-2508 diagnosis of, 2506-2508 mild, 1446t-1450t more severe, 1446t-1450t Thiazide diuretics, 748t contraindications to, 389t for hypertension, 388t side effects of, 389t Thiazide-like diuretics, 388t Thiazolidinediones, 1535 Thick ascending limb transport, 823-824, 824.e1f Thienopyridines, 179-180 Thioctic acid, 1010 Thionamides, 1508 Thiopurine methyltransferase, 189t Thioridazine ocular effects of, 2573, 2573t for psychosis, 2354 Thiothixene, 2354 Third nerve palsy, isolated, 2577-2578 Thomsen’s disease, 2542t, 2544 Thoracic actinomycosis, 2061, 2061f Thoracic aortic aneurysm, 394t, 493-494 Thoracic cysts, 570-571

Thoracic malignancy, 2316 Thoracoabdominal aortic aneurysms, 492 Thoracoscopic surgery, video-assisted, 643 3-day fever, 2259 Three-dimensional echocardiography, 274-275, 276f, 277, 278f, 277.e1 Threshold approach, 39 Throat disorders, 2601-2606 in acute poisoning, 697 lump in the throat, 2606 neurologic, 2605-2606, 2605t hyperfunctional, 2605-2606 hypofunctional, 2606 systemic diseases, 2605 Thrombin inhibitors, 178t-179t direct, 178 oral, 517 in pregnancy, 1618t side effects of, 517 Thromboangiitis obliterans, 505, 957, 1797 Thrombocythemia, essential, 1121-1129 clinical manifestations of, 1122 diagnosis of, 1124, 1125f diagnostic criteria for, 1124t incidence of, 1122 mutations in, 1122, 1123t prognosis for, 1128 risk stratification in, 1127t, 1128 risk-adapted therapy for, 1127t treatment of, 1125-1128 Thrombocytopenia, 1159-1167 alloimmune, neonatal, 1167 antipsychotic-induced, 2626t assessment of, 1057-1058 causes of, 1159, 1160t, 1161-1167 chronic, 1159, 1160f congenital, 1167 definition of, 1159 dilutional, 1167 drug-induced, 1161-1162 drugs associated with, 1162, 1162t gestational, 1166 heparin-induced, 517, 1159, 1162-1163, 1162f in HIV infection, 2328 incidental, of pregnancy, 1166 laboratory tests for, 1156 mechanisms of, 1159 mild, 1159 neonatal alloimmune, 1167 pathobiology of, 1159-1160 platelet transfusion for, threshold, 1195, 1195f in pregnancy, 1166-1167 profound, 1162 severe, 1164 transfusion-refractory, 1161 Thrombocytopenia with absent radius syndrome, 1167 Thrombocytopenic purpura idiopathic (immune), 1163-1164 distinguishing characteristics of, 2252t-2253t immunizations for adults, 66t treatment of, 1164, 1164b, 1164t thrombotic, 735, 1165 distinguishing characteristics of, 2252t-2253t treatment of, 1165b-1166b Thrombocytosis assessment of, 1057-1058 causes of, 1124, 1126t diagnostic evaluation of, 1126f Thromboembolic hypertension, chronic, 403, 403.e1f Thromboembolism prophylaxis of, 366t, 715 pulmonary chronic, 625 postoperative, 2622t renal, 809-810 with STEMI, 453 treatment of, 453, 1612t, 2369 venous, 1832 in women, 1612t Thrombolysis in Myocardial Infarction (TIMI) risk algorithm, 435, 453 Thrombolytic therapy, 517 Thrombophilia, 187t acquired, 1616 genetic, 1616

Index Thrombophilia (Continued) in pregnancy, 1616-1617 testing for, 1187, 1187f Thrombophlebitis cortical, 2481 superficial, 518 Thrombopoiesis-stimulating agents, 1237 Thrombopoietin, 1050-1052, 1164 Thrombopoietin receptor agonists, 1164, 1164t Thrombosed external hemorrhoids, 968, 968f Thrombosis, 418-419, 1154-1159, 2674 in acute coronary syndrome, 433-434 in antiphospholipid syndrome, 1190, 1190b blood markers of, 259 causes of, 1159, 1160t cerebral venous, 2442 deep vein, 511-518 with cancer, 625 diagnostic strategies for, 621, 623 in malignancy, 1189, 1189f postoperative, 1191 in pregnancy, 1616-1617 prognosis for, 626 in HIV infection, 2328 lateral sinus, 2499, 2499f mechanisms of, 1154-1155 mesenteric venous, 954-955 with oral contraceptives, 1608 pathogenesis of, 1185, 1185f after percutaneous coronary intervention, 458 physiologic antithrombotic mechanisms, 1155 portal vein, 959, 1036-1037 postoperative, 1191 post-thrombotic syndrome, 518 in pregnancy, 1190 renal artery, 809b renal vein, 810-811 sagittal sinus, septic, 2500 splenic vein, 959 superior mesenteric, 954 threshold for, 620.e3f traveler’s, 1884 venous sinus, 2498-2500 Thrombotic endocarditis, nonbacterial (marantic), 338 Thrombotic microangiopathy, 793 Thrombotic pulmonary embolism, 620-626, 624f Thrombotic storm, 1190 Thrombotic thrombocytopenic purpura, 735, 1165 clinical manifestations of, 1165 definition of, 1165 diagnosis of, 1165 distinguishing characteristics of, 2252t-2253t pathobiology of, 1165 pathophysiology of, 1165, 1165f treatment of, 1165b-1166b Thromboxane action of, 172, 173f production of, 172, 173f Thromboxane A2, 172, 173f Thrush, 2079-2080, 2080f, 2082, 2604, 2604f Thumbprinting, 953, 956 Thunderclap headache, 1800, 2446 Thymoglobulin, 844t Thymoma, 2549, 2551 Thyroid agents, 2613t Thyroid cancer, 1511-1514 anaplastic carcinoma, 1513 epidemiology of, 1511-1512 epithelial (follicular) carcinoma, 1512-1513 follow-up, 1513 medullary thyroid carcinoma, 1513, 1662 minimally invasive, 1512 papillary carcinoma, 1511-1513 pathobiology of, 1512 prognosis for, 1513 treatment of, 1512b-1513b Thyroid disease arthritic manifestations of, 1824-1825 preconception interventions for women, 1612t in pregnancy, 1614t

Thyroid disease (Continued) preoperative evaluation of, 2616 screening for, 56 surgery for, 1508 Thyroid eye disease, 1506 Thyroid function tests, basal, 1483t Thyroid gland, 1500-1514 anatomic imaging of, 1501-1502 examination of, 26 function of abnormal, 316 control of, 1501 imaging of, 1501-1502 laboratory findings, 1501 physical examination of, 1501 physiology of, 1500-1502 Thyroid hormone effects of, 1501-1502 metabolism of, 1501 normal levels, 1501 synthesis and secretion of, 1500-1501 transport of, 1501 Thyroid hormone replacement therapy, 1504 Thyroid lymphoma, 1513 Thyroid nodules, 1510-1511 diagnosis of, 1510-1511 epidemiology of, 1510 evaluation of, 1512f history with, 1511 imaging findings, 1511 invasive evaluation of, 1511 laboratory findings, 1511 molecular diagnostic testing, 1511 physical examination of, 1511 toxic multinodular goiter, 1506 Thyroid pain, 1509 Thyroid storm, 1508-1509 Thyroid uptake, 1502 Thyroiditis, 1509 acute (suppurative), 1509 autoimmune (Hashimoto’s), 1502-1503 forms of, 1509 lymphocytic (postpartum, painless, silent), 1509, 1509b Riedel’s, 1503, 1509 subacute (de Quervain’s), 1509, 1509b differential diagnosis of, 1509 laboratory findings, 1509 treatment of, 1509b Thyroid-stimulating hormone, 1493-1494 hypothalamic disease and, 1479 normal levels, 1501, 1503-1504 Thyroid-stimulating hormone deficiency, 1493 Thyroid-stimulating hormone–secreting pituitary adenoma, 1506-1507 Thyroid-stimulating hormone–secreting tumors, 1493-1494 Thyrotoxic crisis, 1508-1509 Thyrotoxicosis, 1505-1507 clinical features of, 1505, 2524t definition of, 1505 diagnosis of, 1506-1507 differential diagnosis of, 1507 drugs for, 1508 epidemiology of, 1505 etiology of, 1505, 1505t hypercalcemia associated with, 1657 laboratory assessment of, 1506-1507, 1507f pathobiology of, 1505 radiographic evaluation of, 1507 symptoms and signs of, 1505 transient, 1509 treatment of, 1507b-1509b Thyrotropin, 1501 Thyrotropin-releasing hormone, 1473, 1501 Thyroxine (T4), 1500-1501, 1504, 2616 L-Thyroxine, 1483t Thyroxine replacement therapy, 1504, 1504t Thyroxine-binding globulin, 1501 Thyroxine-binding prealbumin, 1501 Tiagabine, 2407t Tic douloureux. See Trigeminal neuralgia Ticagrelor, 180 for acute coronary syndrome, 437, 438t-439t for acute MI, 448, 450, 454, 454t Ticarcillin-clavulanate, 1012, 1891t-1892t, 1971

Tick paralysis, 2172t Tick-borne diseases Colorado tick fever, 2256-2257 emerging, 2057 Lyme disease, 2021-2027 rickettsioses, 2047-2051 geographic distribution of, 2050, 2050f treatment of, 2051b Rocky Mountain spotted fever, 2047-2050 Southern tick-associated rash illness (STARI), 2021, 2025, 2057-2058, 2057t spotted fevers, 2048t vectors, 2172t Tick-borne encephalitis, 2263t, 2268 clinical manifestations of, 2268 diagnosis of, 2268 epidemiology of, 2268 prevention of, 2268 prognosis for, 2268 treatment of, 2268b Tick-borne viruses, 2250 Tickover, 242-243 Ticks, 2174 deer tick (Ixodes scapularis), 2021, 2022f removal of, 2049, 2050f Ticlopidine, 179 Tics, 2465-2466 clinical manifestations of, 2465 diagnosis of, 2465 epidemiology of, 2465 etiologic classification of, 2466t pathobiology of, 2465 tardive, 2468 Tidal volume, 540, 542f, 660 ARDS Network ventilatory management protocol for, 664t Tigecycline, 1891t-1892t, 1894 for Acinetobacter infections, 1969 for anaerobic infection, 1933, 1934t Tiger snakes (Notechis), 720 Tight junctions, 2633 Tilt table testing, 349-350 Tiludronate, 1667, 1667t Timed up-and-go test, 104-105 Time-to-event variables, 32 Timolol, 388t, 430t, 2567 Tincture of opium, 932 Tinea capitis, 2703t Tinea corporis, 2670 Tinea cruris, 2670, 2670f Tinea manum, 2670 Tinea nigra, 2102 Tinea pedis, 2670 Tinea versicolor, 2643f, 2670-2671, 2671f Tinel’s sign, 2537 Tinidazole (Tindamax), 2105 for amebic liver abscesses, 2141t for giardiasis, 2137 for trichomoniasis, 2146 Tinnitus, 2593, 2596 diagnosis of, 2596 evaluation of, 2596f objective, 2596 subjective, 2596 treatment of, 2596b Tinzaparin (Innohep), 177t in pregnancy, 1618t-1619t for pulmonary embolism, 625t for venous thromboembolism, 1619t Tiotropium, 559, 560t Tipranavir, 2288t Tiredness, 2338 Tirofiban (MK-383, Aggrastat), 180, 438t-439t, 448 TIS system (ACTG), 2325 Tissue cestode (cyst) infection, 2149-2153 Tissue compartment, 1145-1146 Tissue Doppler, 276 Tissue engineering, 208-209 Tissue factor (extrinsic) pathway, 1154-1155 Tissue hypoxia, 686 Tissue inhibitors of metalloproteinase (TIMPs), 235 Tissue injury immune-mediated, 226-230 in inflammation, 233-234 local, 718 Tissue ischemia, 715, 715f, 716.e1f

I99

Tissue nematode infections, 2164-2171, 2165t Tissue plasminogen activator (t-PA) for frostbite, 510-511 for pleural effusion, 635 for pulmonary embolism, 624t recombinant (rt-PA), 180, 447t for acute ischemic stroke, 2440-2441, 2441t-2442t, 2440.e1f Tissue repair, 230-235, 234f Tissue-specific stem/progenitor cells, 205 Tizanidine, 2238-2239, 2476 TM7SF4 gene, 1664-1665 TMPRSS6, 1070 TNF receptor−associated death domain (TRADD) proteins, 219 TNF receptor−associated factors (TRAFs), 219 TNFRSF11A gene, 1664-1665, 1665t TNFRSF11B gene, 1665t TNM (tumor, node, metastasis) staging, 1206, 1354t Tobacco, 145-149, 963 Tobacco control, 1602t Tobacco dependence behavioral treatments for, 146-147, 147t pharmacologic treatments for, 147-149, 147t prevention of, 149 Tobacco history, 24 Tobacco use, 145 behavioral interventions for, 57 clinical manifestations of, 146 diagnosis of, 146 diagnostic criteria for, 146 epidemiology of, 145-146 pathobiology of, 146 prognosis for, 149 Tobramycin, 126t, 1891t-1892t for bacterial endocarditis, 479t for bacterial meningitis, 2488t for bronchectasis, 569 for eye infections, 2565, 2565t for P. aeruginosa pneumonia, 1966-1967 for pneumonia, 616t, 619t for Pseudomonas infection, 1967 for pulmonary infections, 565 for pyelonephritis, 1875t in renal failure, 129t for septic shock, 689t Tocilizumab (Actemra), 170 for giant cell arteritis, 1804-1805 for polymyalgia rheumatica, 1804-1805 for rheumatoid arthritis, 1762 Tocopherol (vitamin E), 1446t-1450t, 1455 for Alzheimer’s disease, 2394 for malabsorption, 931t for nonalcoholic fatty liver disease, 1022 for vitamin E deficiency, 2507t, 2509 for yellow nail syndrome, 2708 Tocopherol (vitamin E) deficiency, 2507t, 2509, 2509b Todd’s paralysis, 2482-2483 Toenails, ingrown, 2709, 2710f Toes, moving, 2469 Tofacitinib (Xeljanz), 170, 1761 for colitis, 941 for psoriasis, 2666t for RA, 1761-1762 Tolcapone, 2457t-2459t Tolerance, 128 immunologic, 225-226, 239-240 T-cell, 239-240 Tolerance signatures, 240 Toll-like receptors (TLRs), 217, 230 Tolterodine, 113t, 830t Tolvaptan for acute decompensated heart failure with pulmonary edema, 318 for autosomal dominant polycystic kidney disease, 820 for hyponatremia, 752-753, 1030 Tongue geographic, 2582, 2582f lichen planus of, 2667, 2668f Tongue base tumors, 1299, 1300f Tongue worm, 2175 Tonic seizures, 2401t, 2403 Tonic-clonic seizures, 2401t, 2402, 2403t Tonicity, 741 Volume I  pp 1-1382  •  Volume II  pp 1383-2722

I100

Index

Tonsillitis bacterial, 2602t chronic, 2604 exudative, 2603, 2603f Tonsils: tumors in, 1299 Tophi gouty, 1813, 1814f microtophi, 1813 Topical medications, 2657-2658 Topiramate, 2408t for chronic pain, 138t for headache, 2358t, 2361 for seizures, 2407t-2408t for sweating disorders, 2521 for tremor, 2462 for trigeminal neuralgia, 2363 for weight loss, 1465-1466 Topotecan (Hycamtin), 1211t-1216t, 1312 Torisel (temosirolimus), 1211t-1216t Toronto Database of Genomic Variants, 191 Torsades de pointes, 342, 370f diagnosis of, 372 pathobiology of, 369 treatment of, 374 Torsemide, 388t, 748t Torsion dystonia, primary, 2464t Toscana virus, 2259 Total body water, 1433 Total body water deficit, 754 Total hip replacement, 1830, 1830f Total joint arthroplasty, 1830-1832 Total knee replacement, 1830, 1831f Total lung capacity, 540, 542f Total peripheral resistance (TPR), 266 Totipotency, 206f Toulouse-Lautrec, Henri de, 1670 Tourette’s syndrome, 2465-2466 clinical manifestations of, 2465-2466 diagnosis of, 2465-2466 epidemiology of, 2465 pathobiology of, 2465 prognosis for, 2466 treatment of, 2466b Tourettism, 2468 Toxic adenoma, 1506 Toxic epidermal necrolysis, 2656f, 2678, 2686, 2686f Toxic erythema, 2697 Toxic granulation, 1055f Toxic metals syndromes specific to, 96-98 trace metals, 92-98 Toxic multinodular goiter, 1506 Toxic myopathies, 2546, 2546t Toxic neuropathies, 2536-2537, 2536t Toxic optic atrophy, 2575 Toxic shock syndrome, 2655, 2656f, 2672 clinical manifestations of, 2672 definition of, 2672 pathobiology of, 2672 prognosis for, 2672 staphylococcal, 1897, 1900 streptococcal, 1908 treatment of, 2672b Toxicants, 699 central nervous system effects of, 697-698, 698t extracorporeal removal of, 706 removed by hemodialysis, 710t Toxicity antimicrobial therapy, 1845 drugs for prevention of, 1218 phototoxicity, 2665, 2665t reduction of, 1846 serotonin, 2618 Toxicology laboratory values, 2712t-2717t reference intervals, 2712t-2717t Toxidromes, 696, 696t Toxin reactions, 1010 differential diagnosis of, 1010 idiosyncratic reactions, 1008 types of, 1008t Toxin-induced liver disease, 1006-1010, 1018t clinical manifestations of, 1008 definition of, 1006 diagnosis of, 1008-1010 differential diagnosis of, 1010 epidemiology of, 1006 future directions, 1010 genetics of, 1007

Toxin-induced liver disease (Continued) pathobiology of, 1006-1008 treatment of, 1010b Toxin-mediated diseases, staphylococcal, 1900 Toxins, 717. See also Intoxication; Poisoning associated with aplastic anemia, 1115, 1115t and dietary requirements, 1453 dose-response effects, 1007, 1008t and pancreatitis, 960-961, 960t renal, 779 renal tubular, 779t sideroblastic anemia due to, 1072 testicular hypogonadism due to, 1573 that cause autonomic neuropathy, 2520 uremic, 836 Toxocara, 1015t-1017t, 2164 Toxocara canis, 2164 Toxocara cati, 2164 Toxocariasis, 2164-2165 clinical manifestations of, 2164 definition of, 2164 diagnosis of, 2164 epidemiology of, 2164 pathobiology of, 2164 prevention of, 2165 treatment of, 2164b, 2165t Toxoplasma gondii, 1015t-1017t, 2125-2126, 2126f Toxoplasma gondii encephalitis, 2295, 2297t-2301t Toxoplasmic encephalitis, diffuse, 2127 Toxoplasmic lymphadenitis, 2127, 2131 Toxoplasmic retinitis, 2566 Toxoplasmic retinochoroiditis, 2127, 2128f Toxoplasmosis, 2058t, 2125-2133 acute, in immunocompromised patients, 2129t, 2131 central nervous system, 2128, 2129f clinical manifestations of, 2127-2128, 2128t congenital diagnosis of, 2130 treatment of, 2131, 2132t definition of, 2125-2126 diagnosis of, 2128-2131 epidemiology of, 2126-2127 HIV-associated, 2309t in immunocompromised patients, 2130-2131 reactivation of, 2132 treatment of, 2129t, 2131 laboratory findings, 2128, 2129t, 2130 with lymphadenopathy, 2130 ocular disease, 2130, 2132 pathobiology of, 2127 pathogenesis of, 2127 pathology of, 2127 in pregnancy diagnosis of, 2130 treatment of, 2131, 2132t prevention of, 2131-2132 prognosis for, 2132 treatment of, 2106, 2131b Toxoplasmosis myocarditis, 328 Trace elements, 1445, 1450t-1452t Trace metals, 92-98 Tracheal cannulation, 528t Tracheal plateau, 544 Tracheitis, 608-609 Tracheobronchitis, H. influenzae, 1947-1948 Tracheoesophageal fistula, 908 Trachinidae (weever fish), 721 Trachoma, 2008-2009 clinical manifestations of, 2008 diagnosis of, 2008 epidemiology of, 2008 prevention of, 2008-2009 treatment of, 2008b Traction alopecia, 2703t, 2706, 2706f Traction bronchiectasis, 580, 580f Traditional Chinese medicine, 182t Traditional healers, 182t TRALI. See Transfusion-related acute lung injury Tramadol, 137, 139t-140t, 965 Tramatenib, 1232t, 1376 Trandolapril, 310t, 388t Tranexamic acid, 1176 Transaminases, 989

Trans-arterial chemoembolization, 1344, 1344f Transbronchial needle aspiration, 1307 Transcatheter aortic valve replacement, 465, 465f Transcobolamin, 1406t Transcranial Doppler, 2345t, 2440 Transcription activator−like effector nucleases (TALENs), 210-211, 213 Transcription factor E2F, 1229, 1229f, 1229t Transcriptomics, 1838 Transcutaneous carbon dioxide measurement, 653-654 Transdermal nicotine patch, 147t, 148 Transdiaphragmatic presure, 628 Transesophageal echocardiography, 276, 277t in aortic dissection, 495, 495f in infective endocarditis, 478 Transferrin saturation, 1063-1064, 1064f Transforming growth factor (TGF)-β, 232 Transforming growth factor (TGF)-β receptor, 219 Transforming practice, 49 Transfusion for autoimmune hemolytic anemia, 1076 blood availability for, 1191 for burns and electrical injuries, 714-715 emerging issues, 1198 erythropoietin, 690 evolving technologies, 1198 guidelines for, 2624 HTLV transmission through, 2236 plasma, 1195 platelet, 1160-1161, 1193-1195 for aplastic anemia, 1121 for bleeding before acute leukemia therapy, 1243 clinical trials, 1193t complications of, 1161 for disseminated intravascular coagulation, 2254 platelet dose, 1195 refractory patients, 1161 threshold for, 1195, 1195f for postoperative anemia, 2624 post-transfusion purpura, 1167 red blood cell, 1191-1193 for anemia, 1068, 1073, 1076, 1121 for aplastic anemia, 1121 clinical practice guidelines for, 1193, 1194t-1195t clinical trials, 1193t immunomodulatory effects of, 1192 for septic shock, 690 for sickle cell disease, 1103-1104 for upper GI bleeding, 880 Transfusion committees, 1191, 1192f Transfusion medicine, 1191-1198 development of, 1191-1195 errors in, 1195-1196 Transfusion reactions, 1196-1198 allergic, 1196 delayed, 1196-1197 febrile nonhemolytic, 1196 hemolytic, 1079, 1196 imaging of, 534f Transfusion services, 1191, 1192f Transfusion-associated adverse reactions, 1196t Transfusion-associated graft-versus-host disease, 1197 Transfusion-refractory patients, 1161 Transfusion-related acute lung injury, 602-603, 1197 clinical manifestations of, 602 definition of, 602 diagnosis of, 603 epidemiology of, 602 incidence of, 602 pathobiology of, 602 possible, 603 prognosis for, 603 treatment of, 603b Transgendered individuals, 1568 Transient aplastic crisis, 2213, 2213f, 2214t Transient global amnesia, 2384 Transient ischemic attack definition of, 2434 pathobiology of, 2434 prevention of stroke after, 2444-2445 prognosis for, 2445 Transient neonatal myasthenia, 2548t

Transjugular intrahepatic portosystemic shunt, 1029, 1030f Transluminal imaging, 873t Transmembrane cytoplasmic protein 43 gene (TMEM43), 331 Transmission-transmitted viruses, 1197, 1197f Transplant vasculopathy, 521-522 Transplantation antigens in, 236-238 bone marrow for cancer, 1218 for inborn errors of metabolism, 1389 for bronchectasis, 569 cardiac, 317, 519-523, 202.e1t in Chagas disease, 2117-2118 of ex vivo differentiation of pluripotent stem cells, 206-207 heart-lung transplantation, 403, 645 hematopoietic stem cell, 237, 1198-1204 for acute myeloid leukemia, 1245 for cancer, 1218 in immunocompromised patients, 1855, 1855t in immunocompromised patients, 1855-1856, 1855t for inborn errors of metabolism, 1389 islet cell, 1532 liver, 1031-1038 for cirrhosis, 1031 contraindications to, 1031 for hepatopulmonary syndrome, 1031 lung, 645-647 contraindications to, 645t desicision guide for, 645t indications for, 645t matched sibling donor, 1119-1120 matched unrelated donor, 1120 pancreas, 1532 renal, 843-847 stem cell allogeneic, 1128 for chronic lymphocytic leukemia, 1256 hematopoietic, 237, 1198-1204 for inborn errors of metabolism, 1389 for myelodysplastic syndrome, 1238 for sickle cell anemia, 1104 Transplantation immunology, 236-240 Transportasome, 1812 Transportation, interhospital, 713 Transporter in antigen processing (TAP), 221-222, 221f Transposition of the great arteries complete, 414, 416t congenitally corrected, 414 Transseptal heart catheterization, 295-298 Transsphenoidal surgery for Cushing’s disease, 1491 for Cushing’s syndrome, 1517 for growth hormone excess, 1486 Trans-sulfuration pathway disorders, 1404-1405 Transthoracic cardioversion and defibrillation, 376-377 complications of, 377 indications for, 377 precautions, 377 techniques for, 376-377 Transthoracic echocardiography, 276, 276t, 478 Transthyretin, 1501 Transudates, 632-633 Transverse myelitis, 2381, 2478 clinical manifestations of, 2478 diagnosis of, 2478 prognosis for, 2478 treatment of, 2478b Tranylcypromine, 2349t Trastuzumab (Herceptin), 171, 1232t for cancer, 1211t-1216t, 1319, 1355-1356, 1358 cardiomyopathy due to, 330 Trauma, 442t barotrauma, 597-598, 2590, 2591f brain, frontal, 2387, 2387f corneal, 2559t epidemiology of, 711 to hypothalamus, 1477 initial surgical care for, 715 macrotrauma, 1822 microtrauma, 1822

Index Trauma (Continued) ocular accidental, 2563 major, 2563 and pancreatitis, 960t, 961 post-traumatic diabetes insipidus, 1500, 1500f post-traumatic stress disorder, 2353 rhabdomyolysis due to, 724 surgical care for, 716 testicular hypogonadism due to, 1573 thrombosis with, 1191 visual field abnormalities due to, 2575 Trauma hypothermia, 695 Traumatic brain injury, 2364-2370 clinical manifestations of, 2365 complications of, 2368 diagnosis of, 2365-2366 epidemiology of, 2364 frontal, 2387, 2387f imaging in, 2366 initial management of, 2367-2368 pathobiology of, 2364-2365 primary injury phase, 2364 prognosis for, 2369 recovery, 2368 secondary injury phase, 2365 severe, 2368 treatment of, 2367b-2369b Traumatic spinal cord injury clinical manifestations of, 2365 diagnosis of, 2366-2367 imaging in, 2367 prognosis for, 2369 targeted therapy for, 2368 treatment of, 2368-2369 acute, 2368-2369 initial management, 2368 subacute, 2368-2369 Traumatic ulcers, oral, 2580t Travelers, 1881-1885 fever in, 1850, 1850t heart failure in, 316 international, 77-78 malaria in, 2108, 2112 post-travel care, 1885 pretravel consultation with, 1881, 1882t vaccinations for, 77-78, 1881-1883, 1883t Traveler’s diarrhea, 920, 1884 causes of, 1868 education about, 1882t prophylaxis of, 1870t treatment of, 922 Traveler’s thrombosis, 1884 Travel-related health problems noninfectious, 1884-1885 preventive measures for, 1882t, 1884, 2112 Travel-related vaccines, 77-78, 1881, 1883, 1883t Trazodone, 2349t Treadmill testing, 425 Treanda (bendamustine), 1211t-1216t Trematodes (flukes), 2104, 2155-2159 Tremor, 2461-2462 clinical manifestations of, 2461-2462 differential diagnosis of, 2461t dystonic, 2463-2464 enhanced physiologic, 2462 essential, 2462, 2605 in Parkinson’s disease, 2454-2455 reemergent, 2454-2455 resting, 2454-2455 tardive, 2468 treatment of, 2462b vocal, 2605 Trench fever, 1997-1998, 2051 clinical manifestations of, 1998 diagnosis of, 2000t pathobiology of, 1997-1998 prognosis for, 2001 treatment of, 2000 Trench foot, 694 Trendelenburg (head down) position, 2448t Treponema carateum, 2020 Treponema pallidum, 2013, 2603 Treponema pallidum particle agglutination (TP-PA) test, 2017, 2017t

Treponema pallidum subspecies endemicum, 2020 Treponema pallidum subspecies pertenue, 2020 Treponemal tests, 2017, 2017t Treponematoses, nonsyphilitic, 2021b Treprostinil, 402 Tretinoin (all-trans-retinoic acid), 2658 Triad syndrome, 827 Triamcinolone (Azmacort, Kenalog, Nasacort), 164t, 1692t, 2658t for allergic rhinitis, 2588 for asthma, 553t for chronic sinusitis, 1692, 2588 for hidradenitis suppurativa, 2682 for lichen planus, 2668t for mycosis fungoides, 2670t for oral mucosal lesions of lupus, 2583 for pityriasis lichenoides, 2669 for urticaria, 2684t Triaminic (pyrilamine), 2660t Triamterene, 748t for hypertension, 388t for Meniere’s disease, 2600 for periodic paralysis, 2544-2545 for vertigo, 2601t Triangular alopecia, 2706 Triazolam, 2352t Triceps reflex, 2372t Triceps skinfold thickness, 1432, 1433t Trichiasis, 2559 Trichinella, 2159, 2164-2165 Trichinella nativa, 2165 Trichinella spiralis, 2165, 2165f Trichinellosis, 2165-2166 clinical manifestations of, 2165-2166 definition of, 2165-2166 diagnosis of, 2166 epidemiology of, 2165 pathobiology of, 2165 prevention of, 2166 treatment of, 2165t, 2166b Trichloroacetic acid, 2222t Trichodynia, 2703 Trichomonas vaginalis, 2145-2146, 2145f Trichomoniasis, 1880, 2145-2146 clinical manifestations of, 2145 diagnosis of, 2145-2146, 2145f epidemiology of, 2145 guidelines for, 2145-2146 pathobiology of, 2145 prevention of, 2146 prognosis for, 2146 treatment of, 2146b Trichostrongylus, 2159, 2160t, 2163 Trichotillomania, 2703t, 2705, 2705f Trichuriasis, 2161-2162 clinical manifestations of, 2162 diagnosis of, 2162 epidemiology of, 2162 prevention of, 2162 treatment of, 2160t, 2162b Trichuris trichiura, 2103, 2159, 2161 Triclabendazole, 2158 Tricuspid aortic valve stenosis, 461 Tricuspid atresia, 415 Tricuspid regurgitation, 303, 471-472 definition of, 471-472 diagnosis of, 472 treatment of, 472b Tricyclic antidepressants for depression, 2349t for irritable bowel syndrome, 894 for pain, 136, 138t, 893t for pruritus, 2636t screening for, 699-702, 702t Trifluridine, 2181t, 2297t-2301t Trifunctional enzyme deficiency, 2545 Trigeminal autonomic cephalalgias, 2360-2361 definition of, 2360 diagnosis of, 2361 distinguishing characteristics of, 2360t prognosis for, 2361 Trigeminal neuralgia, 2363 clinical manifestations of, 2363 diagnosis of, 2363 prognosis for, 2363 treatment of, 2363b Triglycerides, 960-961, 1389, 1395

Trihexyphenidyl, 2457t-2459t, 2465 Triiodothyronine (T3), 1500-1501, 2616 Trimeprazine (Temaril), 2660t Trimethobenzamide, 865t Trimethoprim, 1889t, 1895t for AIDS-associated opportunistic infections, 2297t-2301t for coccidian enteritis, 2147 for cystitis, 1875t for pertussis, 1992 for Pneumocystis pneumonia, 2096, 2096t, 2297t-2301t, 2313t resistance to, 1890t, 1961 for UTI prophylaxis, 1876t Trimethoprim-sulfamethoxazole (Bactrim), 1891t-1892t for AIDS-associated opportunistic infections, 2297t-2301t for bacterial meningitis, 2487, 2488t for chronic granulomatous disease, 1149 for coccidian enteritis, 2147 for COPD exacerbations, 562 for cystitis, 1875t for diarrhea, 922 for enteric fever, 1974 for enteric protozoal infections, 2147t for folliculitis decalvans, 2706 for furuncles, 2696 for gastroparesis and pseudo-obstruction, 888 for hyper-IgE syndrome, 1685 for Moraxella infection, 1948 for MRSA infection, 1901 for nocardiosis, 2064 for pertussis, 1992 for Pneumocystis infection, 2314t for Pneumocystis pneumonia, 2096, 2096t, 2098, 2098t, 2297t-2301t, 2313, 2313t for prostatitis, 831 for pyelonephritis, 1875t resistance to, 1961 for S. aureus infection, 843 for S. maltophilia infections, 1971 for Salmonella bacteremia, 1974 for Salmonella carriers, 1975 for shigellosis, 1978 toxicities, 1895t for toxoplasmic lymphadenitis, 2131 for toxoplasmosis, 2129t for ulcerative colitis, 942 for urinary tract infections, 1876t, 1896 for Whipple’s disease, 932 Triple phosphate crystals, 731-732 Triple phosphate stones, 812 Tripod sign, 652, 652.e3f Trisenox (arsenic trioxide), 1211t-1216t Trismus (lockjaw), 1929 Trisomy X, 1590 Trochanteric bursitis, 1753, 1753f Trochlear nerve palsy, isolated, 2577-2578 Tropheryma whippelii (Whipple’s disease), 477t Tropical endomyocardial fibrosis, 337 Tropical enteropathy, 1434 Tropical myositis, 1898 Tropical pancreatitis, 964 Tropical pulmonary eosinophilia, 2168 Tropical spastic paraparesis, 2237t, 2238-2239 Tropical sprue, 931, 1108 Troponins cardiac (cTn) elevated, 444-445, 445t measured by a highly sensitive assay (hs-cTn), 304.e1t in heart failure, 304 Trospium, 113t, 830t Trousseau’s sign, 963 Truncal radiculopathy, 2535 Trunk musculoskeletal disorders of, 80, 80t skin lesions in, 2702t Truvada, 1884 Trypanosoma, 2113 Trypanosoma brucei brucei, 2113 Trypanosoma brucei gambiense, 2113, 2115t Trypanosoma brucei rhodesiense, 2113, 2115t Trypanosoma cruzi, 2116-2117, 2117f Trypanosoma rhodesiense, 2114, 2115f

I101

Trypanosomiasis, 2115, 2172t. See also Human African trypanosomiasis American. See Chagas disease Tryptophan, 2421t Tubal factor infertility, 1942-1943 Tubal ligations, 1605t, 1606f, 1610 Tuberculin purified protein derivative (PPD), 2035, 2041 Tuberculin skin test, 2035-2036, 2041 Tuberculoid leprosy, 2043, 2043f-2044f Tuberculosis, 2030-2039, 2034f abdominal, 2034-2035 active, 2036-2037 clinical manifestations of, 2032-2035 definition of, 2030-2031 diagnosis of, 2035-2037 distinguishing characteristics of, 2252t-2253t drug-resistant management of, 2038t treatment of, 2038 epidemiology of, 2031 extensively drug-resistant, 2031 extrapulmonary, 2033f, 2034-2035 gastrointestinal, 2035 health care–associated, 1865.e1t hepatic, 1019 HIV-associated, 2038, 2311-2312, 2311.e1f clinical manifestations of, 2311 diagnosis of, 2311 epidemiology of, 2311 indications for testing for, 2312t pathobiology of, 2311.e1, 2311.e1f prevention of, 2311, 2312t prognosis for, 2312 radiographic features of, 2309t, 2310f treatment of, 2295, 2297t-2301t, 2311b, 2312t in immune reconstitution inflammatory syndrome, 2333 incidence of, 2031 miliary, 2034, 2035f multidrug-resistant, 2031, 2038 natural history of, 2032, 2032f oral ulcers, 2580t pathobiology of, 2031-2032 pleural, 2034 pleural fluid characteristics, 634t prevention of, 1884, 2038-2039 primary, 2032-2033 prognosis for, 2039 progressive primary, 2033 pulmonary, 2031, 2033-2034, 2033f reactivation, 2033-2034 reinfection, 2034 renal, 2035 risk factors for, 2031t small intestinal, 948 treatment of, 1896, 2037b-2038b uveal, 2566 Tuberculosis vaccines, 2039 Tuberculous arthritis, 1808-1809 Tuberculous lymphadenitis, 2034 Tuberculous meningitis, 2034, 2342t, 2480 Tuberculous pericarditis, 485t, 491, 2034 Tuberculous pleural effusion, 634, 634t Tuberculous pleurisy, 634 Tuberous sclerosis, 587, 817t, 822 Tuberous sclerosis complex, 2515-2516, 2516f, 2516t Tubular adenoma, 1326 Tubular cell casts, 730-731 Tubular flow rate, 757 Tubular necrosis, acute, 780 Tubulointerstitial diseases, 735, 735t Tubulointerstitial nephritis, 793-799 acute, 793-794, 794t on biopsy, 778f, 783.e2 chronic, 793-794, 794t definition of, 793 epidemiology of, 793-794 IgG4-related, 798 pathobiology of, 794-795 pathology of, 795 pathophysiology of, 794-795 Tubulointerstitial nephritis with uveitis syndrome, 797 Tubulointerstitial nephropathy, HIV, 797 Tubulovillous adenoma, 1326, 1047.e1f Volume I  pp 1-1382  •  Volume II  pp 1383-2722

I102

Index

Tularemia, 87t, 90, 1982-1983, 2058t-2059t clinical manifestations of, 90, 1982-1983 cutaneous, 2060t definition of, 1982 diagnosis of, 90, 1983 differential diagnosis of, 1983 distinguishing characteristics of, 2252t-2253t epidemiology of, 1982 glandular disease, 1983 incidence of, 1982 oculoglandular, 1983 oropharyngeal, 1983 pathobiology of, 1982 pneumonic, 1983, 2060t prevention of, 1983 prognosis for, 90b, 1983 treatment of, 90b, 1983b typhoidal, 1983 ulceroglandular, 1983 vectors, 2172t Tumor(s). See also Cancer; specific tumors adnexal, 2688 ampullary, 1047 benign, of stomach, 1319 benign epidermal, 2687-2689 bile duct, 1047 biology of, 1231-1233 of bone, 1370 brain tumors, 1287-1290 Bushke-Löwenstein tumors, 972 cardiac, 280t-281t, 338-339, 338t catecholamine-secreting, 1524, 1524t of central nervous system, 1287-1297 cerebellopontine angle, 2599 epidural, 1296, 1296f extension into cardiac cavities, 339 extra-axial, 1291-1292 extradural, 1295-1296 of eye, 2571-2572 of facial nerve, 2537 of gallbladder, 1041-1042 hematopoietic stem cell transplantation for, 1202 heterogeneity of, 1231 in HIV infection, 2320-2321 hypothalamic, 1476-1477 intra-axial, 1292-1294 intracavitary, 338-339 intracranial, 1287-1295 intramyocardial, 339 of jaw, 1652t, 1656 lacrimal gland tumors, 2572 malignant, of stomach, 1319 Merkel cell, 2689, 2689f mesenteric desmoid, 950 microenvironment, 1230-1231 myocardial, 338 nerve sheath, 1296 neural crest cell, 2689 oral cavity, 1299 oral soft tissue, 2583, 2583t orbital, 2572 orbital pseudotumor, 2570 pancreatic neuroendocrine, 1334-1339 papillary epithelial, 2583-2584, 2584f paranasal sinus, 1302 pericardial, 338 pituitary, 1483-1484, 1574 salivary gland, 1302-1303, 2584 skin tumors, 2636t, 2647t-2651t, 2687-2693, 2687t small bowel, 1320-1322 solid, 1202 spinal, 1295-1297, 1296t tongue base, 1299, 1300f in tonsils, 1299 TSH-secreting, 1493-1494 Warthin’s, 2584 Tumor, node, metastasis (TNM) staging, 1206, 1354t Tumor DNA, 202 Tumor emboli, 627 Tumor flare, treatment-related, 1357 Tumor growth factor inhibitors, 171 adverse effects of, 171 indications for, 171 types of, 171 Tumor lysis syndrome, 1243 Tumor necrosis factor (TNF), 232 Tumor necrosis factor inhibitors, 1761t, 1764

Tumor necrosis factor receptor–associated periodic syndrome, 1740t, 1741-1742 clinical features of, 1741t definition of, 1741 diagnosis of, 1741-1742 pathobiology of, 1741 treatment of, 1742b Tumor necrosis factor receptors, 219 Tumor necrosis factor-α inhibitors, 169 adverse effects of, 169 indications for, 169 for inflammatory bowel disease, 940 types of, 169 Tumor plop, 339 Tumor suppressor genes, 1306 Tumor suppressor mutations, 1227t Tumor suppressors, 1228-1229 Tumor-associated hypoglycemia, 1220 Tuning fork tests, 2594 Turcot syndrome, 1324 Turf toe, 1754 Turner’s syndrome, 192, 824-825, 1565, 1591f amenorrhea in, 1590 gastrointestinal involvement, 958 treatment of, 1592 24-hr urea nitrogen excretion, 836, 836t 24-hr urine collection, 814 25 By 25 slogan, 21 Twitching motility, 1935 Tykerb (lapatinib), 171, 1211t-1216t Tylosis palmaris et plantaris, 1314 Tympanic membrane, normal, 2590, 2590f Typhlitis, 948, 1860-1861 Typhoid fever distinguishing characteristics of, 2252t-2253t prognosis for, 1975 travel-related risk, 1881 Typhoid vaccines, 68t-73t, 77, 1882, 1883t, 1975 adverse reactions to, 77 indications for, 77 Ty21a vaccine, 1975 typhoid Vi Capsular polysaccharide vaccine, 1975 Typhoidal tularemia, 1983 Typhus American sylvatic, 2048t endemic, 2172t epidemic, 2048t, 2051-2052, 2052b, 2059t exanthematic, 2051 murine, 2048t, 2051 Queensland tick, 2048t scrub, 2048t, 2052, 2172t Siberian tick, 2048t sylvatic, 2052 treatment of, 2052b vectors, 2172t Tyrosine kinase inhibitors, 171 adverse effects of, 171 cardiomyopathy due to, 330 for chronic myelogenous leukemia, 1248-1250, 1249t indications for, 171 Tyrosinemia, 1385 type I, 823t, 1386t, 1388t type II, 1386t Tysabri (natalizumab), 171, 2477 Tzanck smear, 2656, 2657f

U

U wave, 267-269 Ubiquinone (coenzyme Q10) deficiency, 2546 UCERIS (budesonide), 939t Uganda, 2260 UGT1A1, 189t Uhthoff ’s phenomenon, 2473 Ulcer(s) anastomotic, 911-912 aphthous, 2579, 2580f atrophic skin conditions with, 2693t bleeding, 915-917 Cameron’s, 908, 911 chancroidal, 1949, 1949f clean base, 916 complications of, 917 Curling’s, 911 Cushing’s, 911 cutaneous, 2636t, 2651t-2654t, 2695 diagnostic paths and tools for, 913t

Ulcer(s) (Continued) Dieulafoy’s, 908 drugs associated with, 912 duodenal, 908-909, 909f esophageal, 906 gastric, 908, 911f gastroduodenal causes of, 909, 914 incidence of, 909 in systemic inflammatory disorders, 911 genital, 1877t, 1878 H. pylori–associated, 914-915 H. pylori–negative, non-NSAID, 910 idiopathic, 915 intractability of, 917 kissing, 908 leg, 1101 malignant, 910-911 marginal, 911-912 microbial organisms associated with, 911-912 Mooren’s, 2570 NSAID-related, 909-910 oral acute, 2579-2581 chronic, 2581 mucosal, 2580t traumatic, 2580t viral, 2580-2581 peptic, 249t, 908 definition of, 908 in hypergastrinemic syndromes, 911 incidence of, 909 key symptoms and signs of, 912t pathobiology of, 909 perforated, 917, 917f postbulbar, 908 recent hemorrhage, 880, 880t small intestinal, 947-948 stress, 911 venous, 518-519 zoonoses that cause, 2057, 2059t Ulcerative colitis clinical manifestations of, 936t colonic biopsy for, 938, 938f complications of, 942 diagnosis of, 937-938 endoscopic findings in, 938, 938f enteropathic arthritis associated with, 1768, 1768t epidemiology of, 935 extraintestinal complications of, 937t incidence of, 935 pathology of, 936 pathophysiology of, 936 in pregnancy, 943 prevalence of, 935 prognosis for, 943 symptoms of, 937 treatment of, 940f immunomodulatory therapy, 940 maintenance therapy, 942 medical therapy, 939-942 surgical therapy, 942 Ulceroglandular tularemia, 1983 Ulipristal acetate, 1609 Ultrasonography abdominal, in acute pancreatitis, 962 carotid duplex, 2440 compression, 512-513, 513f Doppler, 275-276 continuous-wave, 276 pulsed, 275-276 transcranial, 2440 endobronchial, 642 endoscopic, 873, 873t with bile duct stones, 1042, 1043f in chronic pancreatitis, 964-965, 965f esophageal, 1308 in gastric cancer, 1318, 1318f in hypoglycemia, 1554 in lung cancer, 1308 in gastroenterology, 868, 868f of lungs, mediastinum, and chest wall, 532 in osteoarthritis, 1748 rapid, in shock (rush) protocol, 675t renal, normal, 733, 733f in rheumatic disease, 1725-1726, 1726f, 1728f shock (rush) protocol, 675, 675t strengths and weaknesses, 2345t of thyroid gland, 1501-1502

Ultrasonography (Continued) transabdominal, with gallstones, 1039-1040, 1039f venous, 514 Ultrasound-guided therapy, 1726, 1727f Ultravate (halobetasol propionate), 2658t Ultraviolet A radiation, psoralen with (PUVA), 2661, 2670t Ultraviolet B radiation for cancer, 2661 for lichen planus, 2668t for mycosis fungoides, 2670t for pruritus, 2635-2637, 2636t Ultraviolet radiation exposure, 85-86 and cancer, 1225 clinical manifestations of, 86 pathobiology of, 86 prevention of, 86 Ultresa, 966t Umbilical hernias, 945 Uncinaria stenocephala, 2160 Undervirilized males, 1566-1567 Uniform Anatomical Gift Act, 844 Union for International Cancer Control, 1335 United Kingdom, 603, 1610, 1920 United Nations General Assembly on AIDS, 2272 Millennium Development Goals, 19, 20t United Network for Organ Sharing (UNOS), 519 United States cancers in, 1222 cardiovascular disease burden, 258 causes of death in, 100, 101t Chagas disease in, 2116 dietary guidelines, 1427t Emergency Plan for AIDS Relief, 2272 health care expenditures, 18-19, 21, 46, 258 HIV/AIDS in, 2276-2278, 2276f-2277f population, 15 sarcoidosis, 603 Stroke Belt, 2424 therapeutic technologies, 18 United States Adopted Names Council, 169 United States Census Bureau, 43.e1t United States Department of Agriculture (USDA) Dietary Guidelines for Americans, 1428t-1429t, 1430 MyPlate, 1430, 1430f recommendations for prevention of psittacosis, 2013 United States Department of Health and Human Services (HHS) Antiretroviral treatment guidelines, 2287 Dietary Guidelines for Americans, 1428t-1429t, 1430 guidelines for antiretroviral therapy, 2288t Health System Management Project, 43.e1t National Action Plan to Prevent Healthcare-Associated Infections, 1868.e1 United States Department of Veterans Affairs (VA), 43.e1t United States Supreme Court, 6, 8, 19 Universal precautions, 2286 Unknown primary tumors, 1378-1381 approach to, 1300-1301 evaluation of, 1300, 1300f Upper aerodigestive tract, 2601 Upper aerodigestive tract diseases, infectious, 2601-2604 Upper airway abnormalities associated with obstructive sleep apnea, 639 occupational irritation of, 80t, 81 thermal injury to clinical manifestations of, 598-599 diagnosis of, 599 pathogenesis of, 598 prognosis for, 599 treatment of, 599b Upper arm anthropometry, 1432 Upper extremity deep vein thrombosis of, 518 joints affected in osteoarthritis, 1747, 1747f musculoskeletal disorders of, 80, 80t venous thrombosis of, 518

Index Upper gastrointestinal bleeding, 879-880 AIMS65 scoring system for, 916t Blatchford scoring system for, 916t causes of, 879, 880t clinical manifestations of, 879, 879f diagnosis of, 879-880 endoscopy in, 874-875, 880 hospitalization rate, 879-880 mortality rate, 879-880 Rockall scoring system for, 881t, 916t treatment of, 880b-881b, 881f acute management, 880 endoscopic, 880 medical therapy, 880-881 Upshaw-Schulman syndrome, 1165 Urapadil, 395-396, 396t Urate, 1812, 1812f Urate crystals, 731-732, 733f, 1811, 1813 Urea fractional excretion of urea (FEurea), 781 plasma, in hypovolemia, 745 Urea cycle defects, 1385t, 1386 pathophysiologic mechanisms of, 1385t therapeutic strategies for, 1388t Ureaplasma infection, 2002, 2002t Uremia anemia of, 1065-1066, 1066f definition of, 833 hemolytic-uremic syndrome, 1166 Uremic acidosis, 767 Uremic hyperparathyroidism, 1656 clinical manifestations of, 1656b-1657b pathobiology of, 1656 treatment of, 1656b-1657b Uremic pericarditis, 485t, 491 Uremic toxins, 836 Ureter(s) cancer of, 1350t, 1351 ectopic, 826, 826f Ureteral obstruction, 802, 803f Ureteral stones, 813-814. See also Nephrolithiasis Ureteric anomalies, 825-827, 826f Ureteroscopy, 814 Urethral obstruction, 111, 113t Urethritis, 1877-1878, 1877t chlamydial, 2009-2010 diagnosis of, 1877 gonococcal, 1941, 1941f treatment of, 1878, 1878t Urge incontinence, 111 bladder relaxant agents for, 113t clinical manifestations of, 112 surgery for, 114 treatment of, 113-114 Uric acid crystals, 731-732 Uric acid stones, 812, 815 Urinalysis, 728-732 in acute kidney injury, 781, 781t dipstick test, 728-729 immunoelectrophoresis, 734 quantitative culture, 1874, 1874t 24-hr urine collection, 814, 814t Urinary acid secretion, 764 Urinary alkalinization, 705 Urinary buffers, 764 Urinary incontinence, 110-114 causes related to lower urinary tract, 111 causes unrelated to lower urinary tract, 111-112 clinical evaluation of, 112t clinical manifestations of, 112 definition of, 110 diagnosis of, 112-113 epidemiology of, 110 functional, 111-112 mixed, 112 in older adults, 105 pathobiology of, 110-112 prevention of, 114 prognosis for, 114 screening for, in older adults, 105 transient, 110-111, 111t treatment of, 113b-114b, 2238-2239 behavioral therapy, 113-114 pharmacotherapy, 114 stepwise approach, 113t surgical procedures, 114 in women, 1603

Urinary retention antipsychotic-induced, 2626t treatment of, 2238-2239 Urinary tract abnormalities of, 824-827, 826f age-related changes in, 107-108 development of, 824, 825f lower, 111 Urinary tract infections, 1872-1876 Acinetobacter, 1969 antibiotic regimens for, 689t Candida, 2081 catheter-associated, 1866 burden, costs, and preventability of, 1862t guidelines and recommendations for, 1865.e1t clinical features of, 1873, 1874t complicated host factors, 1872, 1872t treatment of, 1875 definition of, 1872 diagnosis of, 1874 differential diagnosis of, 1874, 1874t enterococcal, 1914-1915, 1915.e1t epidemiology of, 1872 etiology of, 1873, 1873t health care–associated, 1862t imaging findings, 1874 laboratory findings, 1874 P. aeruginosa, 1967 pathobiology of, 1872-1873 prevention of, 1876 prevention of recurrence, 1876, 1876t prognosis for, 1876 Pseudomonas, 1965 S. maltophilia, 1971 treatment of, 1874b-1876b duration of therapy, 1896 follow-up, 1876 Urinary tract obstruction acute caused by calculi, 803 treatment of, 802-803 causes of, 799-800, 799t chronic interstitial nephritis due to, 798 chronic partial, 803 clinical manifestations of, 800, 800t laboratory findings, 800t, 801 Urine albumin-to-creatinine ratio, 1545 appearance of, 728, 730t elements in, 732 glucose in, 729 in hypovolemia, 745 macroscopic appearance, 728, 730t osmolality of, 1494-1495, 1495f output of normal maximum, 754 in shock, 675 pH of, 728-729 specific gravity of, 728 Urine free cortisol excretion in Cushing’s syndrome, 1515 in pseudo-Cushing states, 1515 Urine sediment, 730-732 Urogenital gonorrhea, in males, 1941-1942 antibiotic regimens for, 1945t clinical manifestations of, 1941 diagnosis of, 1941, 1941f physical findings, 1941, 1942f prognosis for, 1942 Urogenital sinus, 1563, 1563f Urokinase, 180 Uroporphyrinogen decarboxylase, 1410 Urosepsis, 1875 Urothelial tumors, 1349, 1350t Ursodeoxycholic acid for gallstones, 1040-1041, 1044 for liver disease in pregnancy, 1622 for primary biliary cirrhosis, 1047 for primary sclerosing cholangitis, 1045-1046 Urticaria, 1675, 1693-1697, 1693f, 2638t-2644t, 2683-2684, 2683f causes of, 2683t chronic idiopathic, 1694 chronic spontaneous, 1694 classification of, 1694t clinical manifestations of, 1694, 2683

Urticaria (Continued) definition of, 2683 diagnosis of, 1694-1695, 2683-2684 differential diagnosis of, 1694-1695 epidemiology of, 1693, 2683 evaluation of, 1696f familial cold, 1131, 1742 future directions, 1696-1697 pathobiology of, 1693-1694, 2683 prevention of, 1695-1696 prognosis for, 1696 regional involvement, 2702t treatment of, 1695, 1695b, 1696f, 2684b, 2684t Urticaria pigmentosa, 1707f, 2684 Urticarial vasculitis hypocomplementemic, 1798-1799 treatment of, 1798-1799 types of, 1798 Urticaria-like eruptions, 2683t U.S. Perinatal Treatment Guidelines, 2291 U.S. Preventive Services Task Force (USPSTF), 54 recommendations, 54 recommendations for abdominal aortic aneurysm screening, 55-56 recommendations for elderly persons, 105 recommendations for general population, 55t recommendations for screening and counseling of adolescents, 63 recommendations for screening mammography, 1359, 1359t U.S. Public Health Service, 2019t Ustekinumab (Stelara), 170, 2660-2661 for psoriasis, 2666t Uterine bleeding, abnormal, 1587-1588 clinical evaluation of, 1588 diagnosis of, 1587-1588 differential diagnosis of, 1587-1588 treatment of, 1588b Uterine sarcoma, 1362-1363 Uveitis, 2237t, 2563-2564 AIDS-associated immune recovery, 2297t-2301t granulomatous inflammatory arthritis, dermatitis, and uveitis, 1740t, 1743 tubulointerstitial nephritis with, 797

V

Vaccine Adverse Event Reporting System, 66 Vaccine Injury Table, 66 Vaccines, 78. See also Immunizations bacterial meningitis, 2489 bioterrorism agent, 78 for developing world travelers, 1881-1882 for heart failure management, 315t for influenza, 2195-2196, 2195.e1t for measles, 2203-2204 meningococcal, 1938-1939 pneumococcal, 554 post-vaccination encephalomyelitis, 2217 recommended adult schedule, 68f rotavirus-induced diarrhea, 2246 route of, 65-66 seasonal influenza, 554 timing of, 65-66 travel-related, 77-78, 1881-1882, 1883t for adults, 1881, 1883t sequence of, 1883 varicella, 2229 for viral hemorrhagic fevers, 2255-2256 zoster, 2229 Vaccinia, 2215, 2217 accidental infection, 2217 autoinoculation of eye, 2217, 2217f generalized, 2217 immunizations for adults, 66t progressive, 2217 Vaccinia immune globulin (VIG), 2219 Vaccinia virus, live, 68t-73t Vacor, 2520 Vacuolar myopathies, toxic, 2546t Vacuolated lymphocytes, 1059t Vagina, 1585 Vaginal atresia, 1565-1566 Vaginal dryness diagnosis of, 1627 epidemiology of, 1627

I103

Vaginal dryness (Continued) estrogen preparations for, 1626t pathobiology of, 1627 treatment of, 1627-1629 Vaginal protozoa, 2105 Vaginitis, 1877t Valacyclovir, 2181-2182, 2182t for AIDS-associated opportunistic infections, 2297t-2301t for herpes, 2181t, 2226 for herpes zoster, 2680 for varicella, 2228 for viral meningitis, 2492 Valganciclovir, 2182-2183, 2182t adverse effects of, 2182t for AIDS-associated opportunistic infections, 2297t-2301t for CMV infection, 2209b, 2231, 2232t for herpes, 2181t for viral encephalitis, 2504 Valproate for headache prevention, 2358t for seizures, 2407t-2408t Valproic acid for bipolar disorder, 2350 for headache prevention, 2361 for myoclonus, 2466-2467 toxic ingestion of, 703t, 710t for trigeminal neuralgia, 2363 Valsalva ratio, 2521 Valsartan for acute MI, 450-451, 454-455, 454t for heart failure, 309, 310t for hypertension, 388t Value, 46 Valvular heart disease, 461-473. See also specific diseases echocardiographic findings, 280t-281t heart failure due to, 317-318 hemodynamics of, 294-295, 294f-295f preoperative assessment of, 2615 rheumatic, 461 severe, 462t Valvular regurgitation echocardiographic findings, 280t-281t evaluation of, 279 mitral valve, 453 Valvular stenosis echocardiographic findings, 280t-281t measurement of, 279 van Buchem’s disease, 1669 van Gogh, Vincent, 1411 Vancomycin, 126t, 1891t-1892t, 1896 for anthrax exposure, 88b for bacterial meningitis, 2488t for C. difficile infection, 1925 for cellulitis, 2696 for diarrhea, 922 for endocarditis, 479t-480t for enterococcal infection, 1915.e1t for lateral sinus thrombosis, 2499 mechanism of action, 1889t for necrotizing fasciitis, 2696-2697 for osteomyelitis, 1810t for peritonitis, 842-843 for pneumonia, 616t, 619t in renal failure, 129t resistance to, 1890t for S. aureus infection, 1901 for S. pneumoniae infection, 1905t for septic arthritis, 1810t for septic shock, 689t for spinal epidural abscess, 2497 for staphylococcal scalded skin syndrome, 2679 for toxic shock syndrome, 2672 toxicities, 1895t Vancomycin-resistant enterococci, 1913 epidemiology of, 1913-1914 health care–associated infections, 1863t, 1864 prevention of, 1915 risk factors for, 1913 treatment of, 1914-1915, 1915.e1t Vandetanib (Caprelsa), 170 Vanishing bile duct syndromes, 1046-1047 Vanishing white matter disease, 2479 Vardenafil (Levitra), 429, 830t, 1578-1579 Varenicline, 147t, 149, 2470 Variability, 32 Volume I  pp 1-1382  •  Volume II  pp 1383-2722

I104

Index

Variable number of tandem repeats (VNTRs), 191 Variable protease encephalopathy, 2505 Variables categorical, 32-33 continuous, 32, 34 count, 32 dichotomous, 32-34 nominal, 32 numerical, 32 ordinal, 32 outcome, 33-34 predictor, 34 time-to-event, 32 types of, 32, 32.e1t Variable-vessel vasculitis, 1794t, 1799 Variant angina, 431 Variceal bleeding in cirrhosis, 1025-1030 endoscopy in, 875, 875f esophageal, 879, 879f liver biopsy in, 980 treatment of, 1029-1030 Variceal ligation, endoscopic, 875, 875f Varicella (chickenpox), 2227-2229, 2655f, 2679-2680 AIDS-associated, 2297t-2301t antiviral therapy for, 2181t clinical manifestations of, 2227, 2228f definition of, 2227 immunizations for, 66t, 67f-68f indications for, 68f oral ulcers of, 2580-2581, 2580t postexposure prophylaxis of, 2229 prevention of, 2229 travel-related risk, 1881 treatment of, 2228 Varicella vaccine, 68t-73t, 77, 1883t, 2229 adverse reactions to, 77 indications for, 77 recommended adult immunization schedule, 67.e1t-67.e3t Varicella zoster virus, 2227 Varicella zoster virus infection, 2227-2229, 2655f acyclovir-resistant, 2229 AIDS-associated, 2295, 2297t-2301t antiviral therapy for, 2181t clinical manifestations of, 2227-2228 complications of, 2229 definition of, 2227 diagnosis of, 2228 disseminated, 2655f distinguishing characteristics of, 2252t-2253t epidemiology of, 2227 pathobiology of, 2227 prevention of, 2229 prognosis for, 2229 treatment of, 2228b-2229b Varicella-zoster virus vaccine, 68t-73t, 77 adverse reactions to, 77 indications for, 77 Varices in cirrhosis, 1025, 1027-1030 liver biopsy in, 980 treatment of, 1029-1030 Varicose veins, 518, 518f Variegate porphyria, 1408t classification of, 1408 diagnosis of, 1413 epidemiology of, 1408 etiology of, 1409 genetic testing for, 1413 prevention of, 1415-1416 skin disease in, 1413 treatment of, 1414 vesiculo-erosive skin disease in, 1412-1413, 1412f Variola minor, 2219 Variola virus, 2214 VariZIG, 2229 Vascular anatomy, 2424-2429 Vascular biology, 417-419 Vascular catheter-associated infections, 1865.e1t Vascular cell adhesion molecule-1 (VCAM-1), 232, 1145 Vascular compartment, 1144-1145 Vascular dementia, 2394-2395 clinical manifestations of, 2394-2395 definition of, 2394

Vascular dementia (Continued) diagnosis of, 2395f diagnostic criteria for, 2394t epidemiology of, 2394 pathobiology of, 2394 prevention of, 2395b prognosis for, 2395 risk factors for, 2394 treatment of, 2395b Vascular disease collagen, 1720 gastrointestinal, 951-959 hepatic, 958-959 of hypothalamus, 1477 of kidney, 735-736, 807-811 rheumatic, 1713t, 1715t splenic, 958-959 tubulointerstitial injury from, 797-798 Vascular endothelial growth factor, 170 Vascular hemorrhagic disorders, 1172 acquired, 1172 gastrointestinal, 957-958 hereditary, 1172 Vascular insufficiency, 2599 Vascular lesions, 958, 2690 Vascular malformations in brain, 2453-2454 cardiac, 415-417 of spinal cord clinical manifestations of, 2381 diagnosis of, 2381 types of, 2380 Vascular myelopathy, 2380-2381 clinical manifestations of, 2381 diagnosis of, 2381 prognosis for, 2381b treatment of, 2381b Vascular reactions, 2685t Vascular rings, 415 Vasculitic neuropathies clinical manifestations of, 2534 definition of, 2533 diagnosis of, 2534 differential diagnosis of, 2534 epidemiology of, 2533 pathobiology of, 2533 prognosis for, 2534 systemic, 2533 treatment of, 2534b Vasculitis, 1167, 2674, 2691 ANCA-associated, 1713t, 1795-1796, 1801 autoimmune, 2655f of central nervous system, 1799-1800 color in, 2638t-2644t diagnostic findings, 2534 differential diagnosis of, 1801t forms of, 1715 gastrointestinal, 956-957 immune complex–mediated, 1798-1799 leukocytoclastic, 2674, 2674f, 2684, 2686 necrotizing, 1858, 1859f nodular, 2693 paraneoplastic, 2533 pathologic characteristics of, 1795t single-organ, 1794t, 1799-1800 small-vessel, 1758, 1758f stroke in, 2443t systemic, 735, 1793-1801 urticarial, 1798 variable-vessel, 1794t, 1799 vertigo with, 2599 Vasculitis neuropathies, 2533-2534, 2533t definition of, 2533 epidemiology of, 2533 pathobiology of, 2533-2534 Vasculitis-associated rapidly progressive glomerulonephritis, 791 Vasculopathy, 1780 Vasectomy, 1604, 1605t, 1606f, 1610 Vaseline (petrolatum), 2657 Vasoconstriction, reversible cerebral, 1800, 1800t Vasodilator paroxysms, 1558-1559 Vasodilators contraindications to, 389t for erectile dysfunction, 1579 for hypertension, 388t, 391 mechanism of action, 391 for pulmonary hypertension, 402 side effects of, 389t, 391 therapeutic principles, 391

Vasomotor symptoms, 1624-1625 definition of, 1624 diagnosis of, 1625 epidemiology of, 1624 estrogen preparations for, 1625, 1626t pathobiology of, 1624-1625 prescription drugs for, 1627 treatment of, 1625b-1627b Vasopressin, 676t, 738-739, 1494-1496 for advanced cardiac life support, 354f plasma, 1495, 1495f for septic shock, 690 Vasopressors, 676, 676t, 680, 683 Vasospasm after subarachnoid hemorrhage, 2447-2449, 2448f symptomatic, 2449, 2449t Vasospastic (or Prinzmetal’s) angina, 421 Vasovagal syncope, 345, 345t VCP gene, 1665t VDJ recombination defects, 1678t Vectibix (panitumumab), 171, 1211t-1216t Vecuronium, 1930 Vedolizumab, 939t, 940 Vegetarianism, 1109-1110 Vegetative state, 2410t, 2413 clinical features of, 2413, 2413t diagnosis of, 2413, 2413t epidemiology of, 2413 pathobiology of, 2413 prognosis for, 2413b treatment of, 2413b Veillonella, 1934t Velcade (bortezomib), 1211t-1216t Velocardiofacial (DiGeorge’s) syndrome, 192-194, 192f Vemurafenib (Zelboraf), 132-133, 202, 1232t for cancer, 1211t-1216t for melanoma, 1376 Venereal Disease Research Laboratory (VDRL) test, 2017, 2017t, 2341 Venereal infections, 2581 Venesection, 1414-1415 Venezuelan encephalitis virus, 2262 Venezuelan equine encephalitis, 2263t, 2265 clinical manifestations of, 2265 diagnosis of, 2265 epidemiology of, 2265 features of, 2263t geographic distribution of, 2490 pathobiology of, 2262, 2265 prevention of, 2265 prognosis for, 2265 treatment of, 2265b Venezuelan hemorrhagic fever, 2249t, 2256 Venlafaxine for cataplexy, 2419t for chronic pain, 138t for depression, 2349t for hot flushes, 1628t-1629t Venom, 717 Venom-induced consumption coagulopathy, 718 Venomous fish, 721 Venous air embolism, 626 clinical manifestations of, 626 diagnosis of, 626 epidemiology of, 626 pathobiology of, 626 prevention of, 626b prognosis for, 626 treatment of, 626b Venous oxygen content (Cvo2), 654 Venous pressure abnormalities, 253t Venous return pulmonary, 415-416 systemic, 415 Venous sinus system, 2498, 2499f Venous sinus thrombosis cavernous, septic, 2498-2499 secondary to infection, 2498-2500 Venous thromboembolism, 511 antipsychotic-induced, 2626t in cancer, 1189 clinical manifestations of, 621 definition of, 620 diagnosis of, 621-623 diagnostic tests for, 621-623 epidemiology of, 620 evaluation for malignancy after, 1189, 1189f heparin for, 177

Venous thromboembolism (Continued) in hypercoagulable states, 1186, 1186t, 1187b-1188b with myeloproliferative neoplasms, 1190 in orthopedic surgery, 1832 pathobiology of, 620-621 in pregnancy diagnosis of, 1616-1617 epidemiology of, 1616 prevention of, 1617 prognosis for, 1617 treatment of, 1617, 1619t prevention of, 517-518, 625-626 prognosis for, 626 recurrent, 1158 Venous thrombosis. See also Deep vein thrombosis cerebral, 2442, 2444f in hypercoagulable states, 1187b-1188b molecularly guided therapeutics for, 202.e1t of upper extremities, 518 Venous ulcers, 518-519 Venous ultrasonography, 514 Ventilation alveolar ventilation (Va), 654 assessment of, 652 dead space (Vd/Vt), 653 measurement of, 654 mechanical, 664-672 minute ventilation (Ve), 654 tests of, 540 for viral hemorrhagic fever, 2254 Ventilation-perfusion (V/Q) scanning, 399-401, 622-623, 622f Ventilator-associated pneumonia, 618-620, 1866 burden, costs, and preventability of, 1862t Pseudomonas in, 1964 staphylococcal, 1899-1900 treatment of, 619b-620b antibiotic, 619t duration of therapy, 1896 guidelines and recommendations for, 1865.e1t Ventilator-induced lung injury, 667-668, 668f Ventilatory compromise, 2369 Ventilatory control, 545 Ventilatory control disorders, 545-548 Ventricular arrhythmias, 367-374, 368f ablation of, 380 causes of, 371 clinical manifestations of, 369 definition of, 367 diagnosis of, 369-372 epidemiology of, 367 management of, 451-452 pathobiology of, 368-369 prognosis for, 374 Ventricular assist devices, 306.e1t-306.e2t Ventricular contractility, 266 Ventricular cysticercosis, 2150 Ventricular drainage external, 2452 of intracerebral hemorrhage, 2452 Ventricular empyema, 2481 Ventricular fibrillation, 370f ACLS response to, 354f acute management of, 373 AED rapid response to cardiac arrests caused by, 354t definition of, 367 diagnosis of, 372 late, 451 prognosis for, 374 treatment of, 373-374 Ventricular flutter, 367 Ventricular function diastolic, 279 imaging of, 287 systolic, 279 Ventricular septal defects, 409 diagnosis of, 409 exercise recommendations for, 416t management of, 453 perimembranous, 409.e1 treatment of, 410b Ventricular tachycardia, 350t-351t, 371.e1f acute management of, 373 definition of, 367 diagnosis of, 369t differential diagnosis of, 352, 369, 370t

Index Ventricular tachycardia (Continued) ECG parameters, 369-370, 371t genetically acquired, 374 iatrogenic, 372, 374 idiopathic, 373, 371.e1f monomorphic, 367, 368f, 370f monomorphic epicardial, 370-371, 371f nonsustained, 352, 353f, 372-373 pleomorphic, 367 polymorphic, 367, 368f post-MI, 371, 368.e1f prognosis for, 374 pulseless, 354f surgery for, 380 sustained, 367 treatment of, 373-374 Ventriculitis, 2481 Ventriculography, 295, 297f VePesid (etoposide), 1211t-1216t Veramyst (fluticasone), 1692 Verapamil, 430t for acute coronary syndrome, 438t-439t for acute MI, 455 for headache prevention, 2358t, 2361 for hemicrania continua, 2361 for hypertension, 388t for hypertrophic cardiomyopathy, 325 for paroxysmal hemicrania, 2361 for small bowel rapid transit dysmotility, 888 for ventricular tachycardia, 373 Verbal empathy, 14t Vermillion, Inc., 203 Vermont: physician-assisted suicide in, 7-8 Verner-Morrison syndrome, 1335t, 1337, 1553 Vero cell vaccine, 1883t Verruca vulgaris, 2673, 2673f Verruga peruana, 1999-2001, 2000t Vertebral artery, 2425, 2427f-2428f Vertebral artery occlusion, 2435, 2435t Vertebral compression fractures, 1641, 1641f Vertebral fracture risk of, 1637, 1638f treatment of, 142t Vertebral osteomyelitis, 2035 Vertebrobasilar insufficiency, 2599 Vertebroplasty, 1644 Vertiginous seizures, 2401t Vertigo bedside tests of, 2599-2600 causes of, 2597-2599 central causes of, 2599 definition of, 2597 diagnostic work-up for, 2600t epidemic, 2598 evaluation of, 2597, 2597f peripheral causes of, 2599 physiologic, 2597-2598 post-traumatic, 2599 top-shelf, 2598 treatment of, 2600b, 2601t with vascular insufficiency, 2599 with vertebrobasilar insufficiency, 2599 Very low density lipoprotein, 1389t Vesicles, 2636t, 2647t-2651t, 2654, 2655f-2656f definition of, 2675 regional involvement, 2702t Vesicoureteral reflux, 827 Vesiculobullous diseases, 2675-2680, 2675t, 2702t Vesiculo-erosive skin disease, 1412-1414, 1412f Vestibular neuritis, 2598, 2600, 2601t Vestibular problems diagnosis of, 2597-2600 evaluation of, 2597 history in, 2597 Vestibular system, 2597-2601 abnormalities of, 2593 anatomy of, 2597 examination of, 2593 physiology of, 2597 Vestibulo-ocular reflex, 2412, 2412t, 2577 Vestibulopathy, peripheral, 2598 Vestibulospinal function tests, 2599-2600 Veterans Affairs Health System, 16 Veterans Health Administration, 42

Viactiv, 1648 Viagra (sildenafil), 429 Vibration, low-intensity, 1644 Vibrio cholerae, 1869t, 1950-1953 Vibrio cholerae O1, 1950-1951 Vibrio cholerae/cholera toxin B subunit vaccine (Dukoral), 1959 Vibrio infections, 1869t, 1950-1953, 1953f Vibrio parahaemolyticus, 1952-1953 Vibrio vulnificus, 1918, 1952-1953 Vidarabine, 2181t Vidaza (5-Azacitidine), 1211t-1216t Video-assisted thoracoscopic surgery, 643 Vietnamese Americans, 15, 17 Vif gene product, 2283 Vigor, 101-102 Vilazodone, 2349t Village hygiene, 2256 Villous adenoma, 933, 1326, 1326f Villous lymphocytes, 1254t Vinblastine, 1271-1272, 1366 Vincristine (Navelbine) for cancer, 1211t-1216t for Hodgkin’s lymphoma, 1271-1272 for hypereosinophilic syndrome, 1154 for non-Hodgkin’s lymphoma, 1263t Vinorelbine (Navelbine), 1211t-1216t Viokace, 966t Violence bilateral, 1629-1630 common couple, 1629-1630 intimate partner, 1629-1633 Vipera, 719-720 Vipera ammodytes, 719-720 Vipera aspis, 719-720 Vipera berus, 719-720 Vipera bites, 720 Viperidae, 717, 719 Viperinae, 717 VIPoma, 1335t, 1337-1338, 1553 clinical manifestations of, 1338 diagnosis of, 1338 epidemiology of, 1338 pathobiology of, 1338 prognosis for, 1338 treatment of, 1338b VIPoma syndrome, 1337-1338 Viral arthritis, 1808 Viral conjunctivitis, 2564t, 2565f Viral encephalitis acute, 2500-2504 CSF formula, 2342t incidence of, 2500-2501 tests for, 2502t vectors, 2172t Viral hemorrhagic fever, 87t, 90-91, 2247-2256 antiviral therapy for, 2254-2255 clinical manifestations of, 90, 2250-2251, 2251t contact tracing, 2255 definition of, 2247 diagnosis of, 91, 2251-2253 differential diagnosis of, 2252t-2253t electrolyte management in, 2254 epidemiology of, 90, 2247-2250 fluid management in, 2254 high-risk exposure to, 2256 laboratory testing in, 2253, 2254t nursing precautions, 2255 pathobiology of, 90, 2250, 2251t patient isolation, 2255 postexposure prophylaxis of, 2256 in pregnancy, 2255 prevention of, 91, 2255-2256 prognosis for, 91, 2256 secondary infection in, 2254 treatment of, 91b, 2254b-2255b guidelines for, 2254-2255 ribavirin therapy, 2255t vaccines against, 2255-2256 vector control, 2256 viruses that cause, 91, 91t Viral hepatitis acute, 993-1000 chronic, 1000-1006 clinical manifestations of, 994t distinguishing characteristics of, 2252t-2253t in pregnancy, 1623

Viral infections. See also specific infections and cancer, 1228 factor concentrate-transmitted, 1176 health care–associated, 1865 in HIV infection, 2318-2319 in immunocompromised patients, 1859t, 1860 of liver, 1018t of skin, 2697 Viral meningitis, 2489-2492 agents, 2489, 2490t antibody detection, 2491 clinical manifestations of, 2490-2491 CSF findings, 2491 CSF formula, 2342t culture of, 2491 definition of, 2489 diagnosis of, 2491-2492 differential diagnosis of, 2491-2492 epidemiology of, 2489-2490 pathobiology of, 2490 polymerase chain reaction in, 2491 prevention of, 2492b prognosis for, 2492 treatment of, 2492b Viral myocarditis, 327-329 causes of, 327t, 328 clinical manifestations of, 327 diagnosis of, 327-328 prognosis for, 329 Viral oral ulcers, 2580-2581 Viral pericarditis, 485t Viral pharyngitis, 2602t, 2603-2604 Viral pneumonia, 2314-2315 Virchow’s node, 1317, 1328 Virchow’s triad, 512 Viridans group streptococci, 476, 1911-1912 Virilization undervirilized males, 1566-1567 virilized females, 1563, 1563f, 1566 Virtual colonoscopy, 869-870, 869f, 1329 Virulomics, 1838 Visceral abscesses, 790 Visceral angioedema, 948 Visceral artery aneurysm, 958 Visceral hypersensitivity, 890 Visceral leishmaniasis, 2120-2122 antigen detection tests, 2122 clinical manifestations of, 2120 coinfection with HIV, 2122 complex manifestations of, 2122 diagnosis of, 2122 epidemiology of, 2120 fatality rate, 2060t in immunodeficiency, 2123 parasitology of, 2122 serology for, 2122 treatment of, 2122b-2123b combinations and coadministration, 2122-2123 general principles, 2122 single-agent therapies, 2122-2123 Visceral pain, 133, 134t, 890 Visceral sporotrichosis, 2078 Visceral stroma, 1730t Vision, 2573-2575 abnormal, chronic, 2558 correction of, 2557 distorted, 2558-2559 double, 2558 normal, 2557 Vision screening, 57 geriatric, 104 Vision testing, 2557, 2574 functional, 2557-2558 Vismodegib (Erivedge), 1211t-1216t, 1378 Visual acuity best-corrected, 2574 normal, 2557 requirements for daily tasks, 2557t Visual axis, 2556 Visual evoked potentials, 2344-2345, 2344f Visual field, 2573-2574, 2574f Visual field testing, 2557, 2574 Visual loss in acute poisoning, 697 alcohol-related, 2511 causes of, 2574-2575 cortical disorders, 2385-2386 gene therapy for, 212

I105

Visual loss (Continued) postgeniculate, 2574 sudden, 2558, 2559t transient monocular, 2574t Visual pathways anatomy of, 2573-2574 lesions in localization of, 2574 visual fields that accompany, 2574, 2574f Visual radiations, 2575 Visual seizures, 2401t Visual system afferent, 2574 diseases of, 2556-2573 Vital capacity, 540, 542f Vital signs abnormal, 28-31 approach to, 30-31 in asymptomatic patients, 29-30, 30t differential diagnosis of, 30 multiple, 30 single, 30 in symptomatic patients who do appear ill, 30 in symptomatic patients who do not appear particularly ill, 30 that require rapid evaluation, 29, 30t treatment options for, 30 in asthma, 550 in heart failure, 303 importance of, 29 measurement of, 25-26, 29 normal ranges, 29t, 30 panic ranges, 29t in poisoning, 696-697 predictive value of, 29 role of, 29-31 as symptoms, 29 Vitamin A (retinol), 1446t-1450t, 1454t, 2573t Vitamin A (retinol) deficiency, 2507t, 2509 Vitamin A (retinol) supplements for diarrhea, 926 for malabsorption, 931t for retinitis pigmentosa, 2571 for vitamin A deficiency, 2507t Vitamin B complex, 931t Vitamin B1 (thiamine) for cold injury, 695 for coma, 2412 functions of, 1446t-1450t for vitamin B1 deficiency, 2507t for Wernicke’s encephalopathy, 2507t Vitamin B1 (thiamine) deficiency, 2506-2508, 2507t cardiomyopathy with, 330 clinical manifestations of, 2506-2508 diagnosis of, 2506-2508 mild, 1446t-1450t more severe, 1446t-1450t Vitamin B2 (riboflavin), 1446t-1450t, 2545 Vitamin B3 (niacin) dose-response effects, 1008t functions of, 1446t-1450t, 1454t for lipid disorders, 1396t Vitamin B3 (niacin) deficiency, 2507t Vitamin B5 (pantothenic acid), 1446t-1450t, 2507t Vitamin B5 (pantothenic acid) deficiency, 2507t Vitamin B6 (pyridoxine), 1109t for acute poisonings, 706t-710t deficiency of, 1073, 2507t, 2509, 2509b functions of, 1446t-1450t, 1454t for homocistinuria, 1407 toxicity of, 2509 Vitamin B9. See Folate Vitamin B12 (cobalamin), 1105-1106 absorption of, 1106-1107, 1106f cellular processing of, 1107 folate interactions, 1108 food sources, 1105 functions of, 1446t-1450t inadequate dissociation from food protein, 1107 malabsorption of, 1386t metabolism of, 1404-1405, 1404f and neuropsychiatric disease, 1454-1455 normal physiology of, 1106-1107 Volume I  pp 1-1382  •  Volume II  pp 1383-2722

I106

Index

Vitamin B12 (cobalamin) (Continued) recommended daily allowance, 1105 serum levels, 1110-1111, 1111t transport of, 1106-1107, 1106f Vitamin B12 (cobalamin) deficiency, 929, 1385t, 2507t, 2508 acquired, 1107 cause of, 1112 classification of, 1105t clinical manifestations of, 2508 conditions predisposing to, 1105t diagnosis of, 1111t, 2508 laboratory evaluation of, 1110-1111 masking of folate deficiency with, 1113 nutritional, 1107 pathogenesis of, 1107 physical findings, 1110 prognosis for, 1113, 2508 serum folate levels in, 1113, 1108.e1t subclinical, 1112-1113 treatment of, 1112, 2508b Vitamin B12 (cobalamin) supplements for bile acid diarrhea, 928 for cobalamin deficiency, 1112-1113, 1407, 2508 dosage, 1112-1113 effects of, 1109t for malabsorption, 931t prophylactic, 1113, 1113.e1t for short-bowel syndrome, 932 for tropical sprue, 931 for vitamin B12 deficiency, 2507t Vitamin C, 1446t-1450t Vitamin C supplements, 1736 Vitamin D (calciferol), 1446t-1450t, 1454t Vitamin D (calciferol) deficiency, 1639-1640, 2507t, 2509 osteomalacia due to, 1648-1649 treatment of, 1103 Vitamin D (calciferol) insufficiency, 1639-1640 Vitamin D (calciferol) supplements, 2507t for malabsorption, 931t for osteomalacia, 1648, 1648t for osteoporosis, 1601-1602, 1642 for prevention of colorectal cancer, 1329 for vitamin D deficiency, 1103, 2507t for well-being, 2478 Vitamin D2 (ergocalciferol) supplements for hypocalcemia, 1659 for prevention of osteomalacia, 1649 Vitamin D3 (cholecalciferol), 740 Vitamin D3 (cholecalciferol) supplements for hypocalcemia, 1659 for osteomalacia, 1648t for renal bone disease, 840 for well-being, 2478 Vitamin E (tocopherol), 1446t-1450t Vitamin E (tocopherol) deficiency, 2507t, 2509, 2509b Vitamin E (tocopherol) supplements, 1455, 2507t, 2509 for Alzheimer’s disease, 2394 for malabsorption, 931t for nonalcoholic fatty liver disease, 1022 for yellow nail syndrome, 2708 Vitamin K, 706t-710t, 1446t-1450t Vitamin K analogues, 1088t Vitamin K antagonists, 175-176, 176f, 706t-710t Vitamin K deficiency, 1184, 2507t, 2510 Vitamin K supplements for malabsorption, 931t for viral hemorrhagic fever, 2254 Vitamin K–dependent coagulation deficiency, 1179-1180 Vitamin supplements, 57, 1455 for inborn errors of metabolism, 1389 multivitamin supplementation, 1455 Vitamins, 181, 1445 deficiencies of, 2507t fat-soluble, 1446t-1450t, 2509-2510 free radical scavenging, 1454 functions of, 1446t-1450t for malabsorption, 931, 931t newly identified roles, 1454t water-soluble, 1446t-1450t, 2506-2509, 2508b Vitiligo, 2633, 2647f, 2699, 2699f, 2701f Vitravene (formivirsen), 210 Vitreous, 2556

Vitronectin (protein S), 243t, 2634 VKORC1 gene, 189t Vocal cord cancer, 1299 Vocal cord dysfunction syndrome, 528t Vocal tremor, 2605 Vogt-Koyanagi-Harada syndrome, 2494 Volar flexor tenosynovitis, 1752-1753 Volatile acid, 763 Volatile (inhalational) anesthetics, 2619 Volume depletion, 452 Volume expansion, 772 Volume of distribution (VD), 124 Volume resuscitation, 2368 Volume-controlled ventilation, 665 Volume-limited ventilation, 665 Volvulus, 944 clinical manifestations of, 944 diagnosis of, 944 prognosis for, 945b sigmoid, 944f, 945 treatment of, 945b Vomiting approach to, 864f definition of, 861 in HIV infection, 2303t nausea and vomiting, 861 postoperative, 2620 treatment of, 865t postoperative, 2622-2623 Von Gierke’s disease, 1552 von Hippel-Lindau disease, 187t, 1523, 1556, 1556t, 2516 von Hippel-Lindau syndrome, 817t, 822, 1227t von Recklinghausen’s disease, 587 von Willebrand disease, 1167-1169, 1175t, 1177 acquired, 1167 classification of, 1168, 1168t clinical assessment of, 1168 clinical manifestations of, 1168 definition of, 1167 diagnosis of, 1168-1169 epidemiology of, 1167 laboratory evaluation of, 1168-1169, 1169t pathobiology of, 1167-1168 treatment of, 1169b-1170b type 1, 1175t type 2, 1175t type 3, 1175t von Willebrand disease (VWD) screen, 1156 von Willebrand factor replacement therapy, 1170, 1170t von Willebrand syndrome, acquired, 1170, 1170b-1171b, 1171t Voriconazole, 2068 adverse effects of, 2068 for AIDS-associated opportunistic infections, 2297t-2301t for aspergillosis, 2068, 2086t for candidemia, 2082 for chromomycosis, 2103 drug interactions, 2068 formulations, 2068 indications for, 2068 for mycoses, 2068 for thrush, 2082 Vorinostat (Zolinza), 1211t-1216t, 2670t Votrient (pazopanib), 170, 1211t-1216t VP-16 (etoposide), 1211t-1216t Vpu gene product, 2283 Vulnerability, 109 Vulvar cancer, 1364 Vulvovaginal candidiasis, AIDS-associated, 2297t-2301t Vulvovaginitis, 2080 VX exposure, 706t-710t

W

Waddling gait, 2341t Waist circumference, 1458, 1463 Wakefulness, 2409, 2415, 2415t, 2416f, 2415.e3f Waldenström’s macroglobulinemia, 1265, 1282 clinical manifestations of, 1282 definition of, 1282 diagnosis of, 1282 diagnostic criteria for, 1276t

Waldenström’s macroglobulinemia (Continued) differential diagnosis of, 1254t laboratory findings, 1282 smoldering (indolent or asymptomatic), 1276t treatment of, 1282b Waldeyer’s ring, 2603 Wallenberg’s syndrome, 2518, 2435.e2f Wandering atrial pacemakers, 359, 359f Warfarin (Coumadin), 175, 176f, 179t for acute MI, 455f adverse effects of, 176, 1186, 1186f for anticoagulation, 366-367 for antiphospholipid antibody syndrome, 1180 for deep venous thrombosis, 516-517 dosing, 176 for elevated INR, 176t for heparin-induced thrombocytopenia, 1163 indications for, 176 for livedo reticularis, 506 monitoring, 175-176 perioperative, 2613t in pregnancy, 517, 1618t for renal artery thrombosis, 809 for renal vein thrombosis, 810-811 reversing effects of, 176 side effects of, 517 for thromboembolic prophylaxis, 366t for thrombosis, 1190 for venous thromboembolism (VTE), 517-518 Warfarin Dosing tool, 203 Warfarin-induced skin necrosis, 1186, 1186f Warm or warm-reactive autoimmune hemolytic anemia recommended workup, 1076t secondary, 1076, 1076t treatment of, 1077-1078, 1078f Warm-up phenomenon, 2544 Warren, John Collins, 6 Warthin’s tumor, 2584 Warthin-Starry stain, 913 Warts, 2673, 2673f anal, 972, 972f anogenital, 2221 common, 2220 cutaneous, 2221 genital, 1877t, 2221-2222, 2221f, 2222t perianal, 2221 plantar, 2220-2221, 2221f treatment of, 2222, 2222t Warty lesions, 2688 Washington State: physician-assisted suicide in, 7-8 Wasting, 101-102, 1434 chronic, 2506 global rates, 1434 in HIV infection, 2305 muscle, 2547 Wasting syndrome, 101-102 Watch and wait, 2152 Water contaminated, 1972 intestinal transport of, 918-919, 918.e3f Water balance, 741 Water balance disorders, 749-755 epidemiology of, 741 pathogenesis of, 742, 743t pathophysiology of, 742 Water bolus treatment, 2521 Water content, 738-739, 739f Water deprivation test, 1498, 1498f Water diuresis, 755 Water exposure, 1884 Water precautions, 1884 Water-borne infectious diarrhea, 919-920, 919t Water-bottle sign, 284, 284f Waterhouse-Friderichsen syndrome, 2482 Watermelon stomach, 874, 958 Watery diarrhea, 924f, 926, 933, 933b Watery diarrhea, hypokalemia, and achlorhydria, 1335t, 1337, 1553 Watery diarrhea, hypokalemic, hypochlorhydric acidosis, 769 WC-BS vaccine, 1952 WDHA (watery diarrhea, hypokalemia, and achlorhydria) syndrome, 1335t, 1337, 1553

Weakness, 2338-2340 diaphragmatic, 627-628 disorders accompanied by, 2340, 2340t episodic, 2340 hypotonic, 2378 intermittent, 2340 myasthenic, 2551 in osteoarthritis, 1746 postoperative, 2623 Weapons of mass destruction, 87, 87t Weber test, 2594 Weber’s line, 2261 Weber’s syndrome, 2435.e1f Webs, esophageal, 905-906, 905f Wedge pressure, 401 Weever fish (Trachinidae), 721 Wegener’s granulomatosis, 536, 586 classification of, 1794 clinical manifestations of, 1797 epidemiology of, 1794t gastrointestinal involvement, 956 pathologic characteristics of, 1795t Weight gain drugs that promote, 1461, 1462t in puberty, 61 in skin disease, 2646t Weight loss, 1431 approach to, 852t behavior modification for, 1465 for diabetes prevention, 1537 diet for, 1464-1465 gonadal insufficiency due to, 1574 in HIV infection, 2303t involuntary, 861 approach to, 863f causes of, 862t-863t diagnosis of, 861 medical therapy for, 1466 for obstructive sleep apnea, 641 in older adults, 105 physical activity for, 1465 secondary to gastrointestinal causes, 862t-863t for type 2 diabetes, 1535 Weil’s disease, 2030 Welch Allyn AudioScope, 104 Welder flash, 86 Well-being, 2478 Wellcovorin (leucovorin), 1211t-1216t Wells’ clinical prediction rule, 621, 621t Wells’ syndrome, 2696 Wenckebach block (Mobitz type I AV block), 346, 348, 357, 357f, 452 Werdnig-Hoffman disease, 2525-2526 Werner’s syndrome, 1370, 2635 Wernicke’s aphasia, 2384-2385, 2385t, 2435 Wernicke’s encephalopathy, 2506-2508, 2507t, 2507.e1 West African sleeping sickness, 2114 West Nile encephalitis, 2057, 2057t, 2263t, 2266-2267 diagnosis of, 2266-2267 features of, 2263t geographic distribution of, 2490 incidence of, 2266 pathobiology of, 2262 test for, 2502t West Nile fever, 2266-2267 clinical manifestations of, 2258, 2266 definition of, 2258 diagnosis of, 2259, 2266-2267 distinguishing characteristics of, 2252t-2253t epidemiology of, 2258 pathobiology of, 2258 prognosis for, 2259 treatment of, 2259b vectors, 2172t West Nile fever and encephalitis, 2266 clinical manifestations of, 2263 diagnosis of, 2266-2267 epidemiology of, 2266 incidence of, 2266, 2266.e1f prevention of, 2267 prognosis for, 2267 time course, 2267f treatment of, 2267b vaccines against, 2267 West Nile fever virus, 2258 West Nile meningoencephalitis, 2266-2267

I107

Index Western equine encephalitis, 2263t, 2264-2265 clinical manifestations of, 2264-2265 diagnosis of, 2265 epidemiology of, 2264 features of, 2263t geographic distribution of, 2490 pathobiology of, 2262, 2264 prevention of, 2265 prognosis for, 2265 treatment of, 2265b Western Europe, 2275 West’s syndrome, 2404 Wheals, 2636t, 2683 Wheezes, 526 Wheezing, 528, 528t, 548 Whiplash, 2374t Whipple’s disease, 477t arthritic manifestations of, 1823-1824 endoscopy in, 876 malabsorption in, 932 Whipple’s triad, 1548 Whipworm, 2161 Whisper voice test, 104 White blood cell disorders, 2328 White blood cells in bacterial meningitis, 2483 normal ranges, 1129 in septic arthritis, 1807 in urine, 730 White cell fragments, 1058 White lesions, 2581-2582, 2581t White matter changes, 1821, 1821.e1f White or cat’s eye reflex, 2572 White pupil, 2560 Whole blood, 1192 Whole bowel irrigation, 705 Whole exome sequencing, 186 Whole genome sequencing, 186 Whole medical systems, 182t, 184 Whole pancreas transplantation, 1532 Whole-exome sequencing, 1743-1744 Whooping cough, 1990-1993, 2058t. See also Pertussis Why Not the Best? (Commonwealth Foundation), 43.e1t Wickham striae, 2637f, 2667 Wiggers diagram, 265f Wilate (Octapharma), 1170 Wilfactin (LFB), 1170 Williams’ disease, 1652t Williams’ syndrome, 186, 405, 1657 Williams-Beuren syndrome, 194 Wilms tumor, familial, 1227t Wilson’s disease, 823t, 1416-1418, 1450t-1452t, 2510 clinical manifestations of, 1416-1417, 1417f definition of, 1416 diagnosis of, 1000t, 1417-1418 epidemiology of, 1416 future directions, 1418 heterozygotes, 1418 laboratory findings, 1417 liver disease in, 983, 1416, 1417f ocular symptoms of, 2569 pathobiology of, 1416 prognosis for, 1418 treatment of, 1418b Wisconsin Ehrlichia, 2026, 2055 Wiskott-Aldrich syndrome, 1167, 1683-1684, 1683t blood smear features of, 1057-1058 clinical manifestations of, 1683-1684 definition of, 1683 diagnosis of, 1684 epidemiology of, 1683 pathobiology of, 1683-1684 prognosis for, 1684 treatment of, 1684b X-linked, 1171 Wiskott-Aldrich syndrome protein (WASP), 1133-1134 Withdrawal, 1605t, 1606, 1606f alcohol, 151, 154-155, 155t Withdrawal of life support, 651-652, 651t Withdrawal of treatment, 14t Wohlfart-Kugelberg-Welander disease, 2525-2526 Wolbachia, 2047t, 2055, 2103-2104

Wolbachia pipientis, 2053 Wolff-Parkinson-White syndrome, 269, 347, 350 Women’s health alcohol use, 1603 anxiety disorders, 1603 approach to, 1600-1604 cancer, 1601 cardiovascular disease, 1600 causes of morbidity, 1602-1604 causes of mortality, 1600-1602 daily requirements for calcium intake, 1601-1602 depression, 1603 dietary guidelines for, 1427t dietary requirements for, 1452 disparities, 1600 HIV/AIDS, 1603-1604, 2276 important issues, 1601t lifespan groups, 1600 lower genital tract gonorrhea, 1942 maternal mortality, 1610 menopause, 1623-1629 normal body temperature, 1849 normal changes in pregnancy, 1610, 1612t obesity, 1602-1603 osteoarthritis, 1603 osteoporosis, 1601-1602 preconception interventions, 1612t preferred medications for, 1602t reproductive issues, 1604 risks for atherosclerotic cardiovascular disease, 260f-261f seizures, 2408 smoking, 1603 substance use, 1603 type 2 diabetes, 1601 urinary incontinence, 1603 Women’s Health Initiative (WHI), 33, 1359 effects of postmenopausal hormone therapy for, 1626-1627, 1626t risk for cardiovascular disease, 58, 59t Work challenges, 592 Work of breathing components of, 655, 655f measurement of, 655 Work-exacerbated asthma, 590 clinical manifestations of, 590 definition of, 590 diagnosis of, 590b epidemiology of, 590 treatment of, 590b Working memory, 2383t Workplace exposures, hazardous, 81-82, 81t Work-related asthma, 588-590, 591f World Cancer Research Fund, 1223 World Health Organization (WHO) categories of mastocytosis disease, 1706, 1709 classification of acute leukemias, 1240, 1240t classification of brain tumors, 1288t classification of dyspnea, 398 classification of Hodgkin’s lymphoma subtypes, 1269, 1269t classification of kidney cancers, 1345, 1345t classification of lupus nephritis lesions, 1772 classification of mastocytosis, 1706, 1706t classification of myeloid malignancies, 1121-1122, 1122t classification of nasopharyngeal cancers, 1298 classification of non-Hodgkin’s lymphoma, 1258-1259, 1261t classification of pancreatic neuroendocrine tumors, 1335 classification of respiratory diphtheria, 1916-1917 definition of acute myeloid leukemia, 1233 definition of adverse drug reactions, 1703 definition of chronic disease, 46 definition of disabling hearing loss, 2594 definition of drug allergy, 1703 definition of infertility, 1594 definition of myelodysplastic syndromes, 1233

World Health Organization (WHO) (Continued) definition of osteoporosis, 1637 definition of peripheral T-cell lymphoma, unspecified, 1266 definition of pertussis (whooping cough), 1992 diagnostic criteria for polycythemia vera, essential thrombocythemia, and primary myelofibrosis, 1124t diagnostic criteria for systemic mastocytosis, 1708-1709, 1708t FRAX, 1641-1642 grading of bladder cancer, 1349 guidelines and recommendations for prevention and management of health care–associated infections, 1865.e1t guidelines for antiretroviral therapy, 2287, 2288t guidelines for fluid management of dengue shock syndrome, 2254 guidelines for treatment of leptospirosis, 2030 Informal Working Group on Echinococcosis standardized ultrasound classification of cystic echinococcosis, 2151, 2152f Meningitis Vaccine Program, 1938-1939 recommendations for intimate partner violence, 1632t recommendations for measles vaccination, 2204 recommendations for oral rehydration solutions, 1952 recommendations for preventive therapy for TB, 2036 recommendations for rotavirus vaccination, 2246 standardized growth charts, 1435 Strategic Plan for the Elimination of Leprosy, 2042 subtypes of myelodysplastic syndrome, 1236, 1236t 25 By 25 slogan, 21 types of smallpox, 2216t Wound botulism, 1928 Wound infections Acinetobacter species, 1969 nosocomial, 1854 P. aeruginosa, 1967 Wound management, 1929-1930, 1929t Wound repair, 219 Wounds, early excision, 715 Wrinkling, 2693-2694 Wrist: orthopedic procedures on, 1832 Wrist disorders, 1752-1753 bursitis conditions, 1750t rheumatoid arthritis, 1757 tendinitis conditions, 1750t Wrist fractures, 1637, 1638f Writer’s cramp, 2463-2465 Wuchereria bancrofti, 2103-2104, 2159, 2166, 2167t, 2168f Wyeth-Lederle, 2246

X

X chromosome mutations, 1590 Xanthomas, 255, 257f Xanthomonas cepacia, 1967 Xanthomonas maltophilia, 1962 XDx Expression Diagnostics, Inc., 203 Xeljanz (tofacitinib), 170, 1761-1762 Xeloda (capecitabine), 1211t-1216t Xerocytes, 1084 Xerocytosis, 1084 Xeroderma pigmentosum, 1227t Xerostomia (dry mouth), 1759, 2585 X-linked adrenoleukodystrophy, 1387t X-linked agammaglobulinemia, 1677, 1680t X-linked chronic granulomatous disease, 211 X-linked muscular dystrophies, 2541t X-linked protoporphyria, 1408, 1408t, 1410, 1413 X-linked severe combined immunodeficiency, 1678t Xpert MTB-RIF, 2037 X-rays, 82, 83f Xtandi (enzalutamide), 1211t-1216t

XTC (Ecstasy) intoxication, 395t XX male syndrome, 1565 XY female syndromes, 1565 46,XY patients, 1567-1568

Y

YAG liser iridectomy, 2567 Yatapoxvirus, 2215, 2215t clinical manifestations of, 2218 diagnosis of, 2218 prognosis for, 2219 Yaws, 2020-2021 Yaz (drospirenone and ethinyl estradiol), 2660 Yellow fever, 2249t clinical manifestations of, 2250-2251, 2251t management of, 2254-2255 pathobiology of, 2250, 2251t sylvatic, 2060t travel-related risk, 1881 vectors, 2172t Yellow fever vaccine, 68t-73t, 77-78, 1882, 1883t, 2255-2256 adverse reactions to, 77-78 indications for, 77 Yellow fever virus, 91t, 2247, 2249t Yellow nail syndrome, 2708, 2708f Yemenite Jews, 1133 Yergason’s sign, 1752 Yersinia, 1984-1990 Yersinia enterocolitica, 1869t, 1984, 1988-1989 Yersinia pestis, 1984, 1984f Yersinia pseudotuberculosis, 1984, 1988-1989 Yersiniosis, 1988-1990 Yervoy (ipilimumab), 1211t-1216t Yoga, 182t, 183 Yohimbine, 2521 Young children, 1991-1992. See also Children

Z

Zafirlukast, 554, 1695 Zalcitabine, 2288t, 2289, 2331t Zaleplon, 2352t, 2421, 2421t Zaltrap (aflibercept), 170, 1211t-1216t Zanamivir, 2183-2184, 2184t, 2196t for acute bronchitis and tracheitis, 609 for influenza, 2183t, 2196-2197 Zantac (ranitidine), 2660t Zelboraf (vemurafenib), 1211t-1216t Zellweger’s spectrum, 1387t Zenker’s diverticulum, 904, 905f Zenpep, 966t Zidovudine (azidothymidine, AZT), 2287, 2288t for adult T-cell leukemia/lymphoma, 2238 CSF-to-plasma ratios, 2331t fixed-dose combinations, 2289, 2290t side effects of, 2289, 2291 Zika virus, 2258 Zileuton, 552-553 Zinc, 93t, 1450t-1452t, 2186 for cholera, 1952 for diarrhea, 922, 926, 2246 for malabsorption, 931t Zinc deficiency, 1450t-1452t Zinc overload, 97-98 Zinecard (dexrazoxane), 1211t-1216t Zinga virus, 2259 Ziprasidone, 2354 Zoladex (goserelin), 1211t-1216t Zoledronic acid (Zometa) for bone pain, 1281 for cancer, 1211t-1216t for hypercalcemia, 1654 for osteoporosis, 1643, 1643t for Paget’s disease, 1667t for prostate cancer, 1369 Zolinza (vorinostat), 1211t-1216t Zollinger-Ellison syndrome, 772, 911, 1335-1337, 1335t, 1553 clinical manifestations of, 1336 definition of, 1335-1336 diagnosis of, 913-914, 1336 differential diagnosis of, 1336 endoscopy in, 1336 Volume I  pp 1-1382  •  Volume II  pp 1383-2722

I108

Index

Zollinger-Ellison syndrome (Continued) epidemiology of, 1335-1336 imaging in, 1336 medical therapy for, 1336 metastatic disease, 1337 pathobiology of, 1336 prognosis for, 1337 surgical therapy for, 1337 treatment of, 1336b-1337b Zolmitriptan, 2358-2359, 2359t, 2361 Zolpidem, 2352t, 2421, 2421t Zometa (zoledronic acid), 1211t-1216t

Zonisamide, 2407t-2408t Zoonoses, 2056-2060 central nervous system, 2057, 2058t, 2262 clinical manifestations of, 2057 definition of, 2056 diagnosis of, 2057-2059 differential diagnosis of, 2059 emerging infections, 2057, 2057t epidemiology of, 2056-2057 fatality rates, 2060, 2060t of liver, 1019

Zoonoses (Continued) prevention of, 2059-2060 prognosis for, 2060 respiratory tract, 2057, 2058t Zoonotic endocarditis, 480 Zoonotic filarial infections, 2171 Zoonotic meningitis, 2487 Zoonotic streptococci, 1912-1913 Zoster (shingles), 2227-2229 AIDS-associated, 2297t-2301t clinical manifestations of, 2227-2228, 2228f

Zoster (shingles) (Continued) definition of, 2227 dermatomal, 2227-2228, 2228f herpes zoster ophthalmicus, 2565 prevention of, 2229 treatment of, 2228-2229 Zoster vaccine, 2229 Zuckerkandl: organ of, 1521 Zydis selegiline, 2457t-2459t Zygomycosis, 2087 Zyrtec (cetirizine), 2660t, 2684t Zytiga (abiraterone acetate), 1211t-1216t

GUIDE TO THE APPROACH TO COMMON SYMPTOMS, SIGNS, AND LABORATORY ABNORMALITIES CHAPTER

SPECIFIC TABLES OR FIGURES

SYMPTOMS Constitutional Fever Fatigue Poor appetite Weight loss Obesity Snoring, sleep disturbances

280 274 132 132, 219 220 100, 405

Tables 280-1 to 280-8 E-Table 274-1 Table 132-1 Figure 132-4; Tables 132-4, 219-1, 219-2 Figure 220-1 Table 405-6

Head, Eyes, Ears, Nose, Throat Headache Visual loss, transient Ear pain Hearing loss Ringing in ears (tinnitus) Vertigo Nasal congestion, rhinitis, or sneezing Loss of smell or taste Dry mouth Sore throat Hoarseness

398 423, 424 426 428 428 428 251, 426 427 425 429 429

Tables 398-1, 398-2 Tables 423-2, 424-1 Table 426-3 Figure 428-1 Figure 428-2 Figure 428-3 Figure 251-1; Table 251-2 Table 427-1 Table 425-7 Figure 429-2; Table 429-1

Cardiopulmonary Chest pain Bronchitis Shortness of breath Palpitations Dizziness Syncope Cardiac arrest Cough Hemoptysis

51, 137 96 51, 83 51, 62 51, 62, 428 62 63 83 83

Tables 51-2, 137-5, 137-6 Figure 83-3 Figure 62-1; Tables 51-4, 62-5 Figure 62-1; Table 428-1 Figure 62-1; Tables 62-1, 62-2, 62-4 Figures 63-2, 63-3 Figure 83-1; Tables 83-2, 83-3 Tables 83-6, 83-7

132 132, 138 135, 153 132, 137, 138, 139

Figure 132-5; Table 132-5 Table 132-1 Figure 135-3; Table 135-1 Figures 132-6, 138-2; Tables 137-3, 137-4, 139-1

132, 142 132, 137 137, 140 135 136, 137 145 145

Figures 132-1, 132-2; Tables 132-2, 132-3, 142-1 Figure 132-3; Tables 132-2, 137-1 Figures 137-1, 140-1 to 140-4 Figures 135-3, 135-4, 135-6; Table 135-4 Figures 136-3, 136-5, 137-1; Table 136-2 Figure 145-5

284, 285 284 26 123 126 285 236 236 240 234 234 200 285

Tables 284-3, 284-5, 285-2 Table 284-3 Tables 26-1 to 26-3 Tables 123-1 to 123-3 Figure 126-1

Gastrointestinal Nausea and vomiting Dysphagia, odynophagia Hematemesis Heartburn/dyspepsia Abdominal pain   Acute   Chronic Diarrhea Melena, blood in stool Constipation Fecal incontinence Anal pain Genitourinary Dysuria Frequency Incontinence Urinary obstruction Renal colic Vaginal discharge Menstrual irregularities Female infertility Hot flushes Erectile dysfunction Male infertility Scrotal mass Genital ulcers or warts

Figure 236-3; Tables 236-3, 236-4 Table 236-5 Table 240-1 Figure 234-10 Figures 234-8, 234-9; Table 234-7 Figure 200-1 Table 285-1



GUIDE TO THE APPROACH TO COMMON SYMPTOMS, SIGNS, AND LABORATORY ABNORMALITIES—cont’d CHAPTER

SPECIFIC TABLES OR FIGURES

Musculoskeletal Neck or back pain

400

Figures 400-4, 400-5, 400-6; Tables 400-3 to 400-5

Painful joints

256

Figure 256-1; Tables 256-1, 256-3

Swollen feet, ankles, or legs   Bilateral   Unilateral

51 81

Figure 51-8 Figure 81-2; Table 81-2

Claudication

79

Table 79-3

Acute limb ischemia

79

Figure 79-5; Table 79-1

Extremities

Neurologic Weakness

396, 420, 421, 422

Tables 396-1, 420-2, 421-2, 421-4

Sensory loss

396, 420

Figure 420-1; Tables 420-1, 420-3 to 420-5

Memory loss

402

Figures 402-1, 402-2; Tables 402-1 to 402-6

Abnormal gait

396

Table 396-2

Seizures

403

Tables 403-1 to 403-6

Integumentary Abnormal bleeding

171

Table 171-1

Rash

436, 441

Figure 436-1; Tables 436-1 to 436-6, 441-5

Hives

252, 440

Figure 252-2; Tables 252-1, 440-1, 440-2

Abnormal pigmentation

441

Table 441-2

Alopecia and hirsutism

442

Tables 442-1, 442-3

Nail disorders

442

Table 442-4

SIGNS Vital Signs Fever

280, 281

Figure 281-1; Tables 280-1 to 280-8, 281-2

Hypothermia

8, 109

Table 109-4

Tachycardia/bradycardia

8, 62, 64, 65

Figures 62-2, 62-3; Tables 64-4, 65-2

Hypertension

67

Table 67-5

Hypotension/shock

8, 106

Figures 106-3, 108-1; Tables 106-1, 107-1, 107-2

Altered respiration

8, 86, 104

Tables 86-1, 86-2, 104-2

Eye pain

423

Table 423-3

Red eye

423

Tables 423-4, 423-6

Dilated pupil

424

Figure 424-4

Nystagmus

424

Table 424-5

Papilledema

424

Table 424-2

Strabismus

424

Figure 424-6

Jaundice

147

Figure 147-2; Tables 147-1 to 147-3

Otitis

426

Table 426-3

Head, Eyes, Ears, Nose, Throat

Sinusitis

251, 426

Tables 251-3, 426-1, 426-2

Oral ulcers and discolorations

425

Tables 425-1 to 425-4

Salivary gland enlargement

425

Table 425-6

Neck mass

190

Figure 190-3

Lymphadenopathy

168

Tables 168-1 to 168-6

Thyroid nodule

226

Figure 226-4

Thyromegaly/goiter

226

Figures 226-1, 226-3

Neck

Breast Breast mass

198

Lungs Wheezes

83

Table 83-4

Heart murmur or extra sounds

51

Figure 51-6; Tables 51-7, 51-8

Jugular venous distention

51

Table 51-6

Carotid pulse abnormalities

51

Figure 51-5

Cardiac

GUIDE TO THE APPROACH TO COMMON SYMPTOMS, SIGNS, AND LABORATORY ABNORMALITIES—cont’d CHAPTER

SPECIFIC TABLES OR FIGURES

Abdomen Hepatomegaly

146

Figure 146-5

Splenomegaly

168

Tables 168-7, 168-9

Acute abdomen

142, 143

Figure 143-1; Table 142-1

Abdominal swelling/ascites

142, 153

Table 153-3

Rectal bleeding/positive stool

135, 193

Figures 135-3, 135-4, 135-6; Table 135-4

Hemorrhoids

145

Table 145-1

Arthritis

256

Figure 256-1

Edema

51

Figure 51-8

Cyanosis

51

Clubbing

51

Figure 51-10

Delirium

28

Figure 28-1; Tables 28-1, 28-2

Psychiatric disturbances

397

Tables 397-1 to 397-4, 397-6 to 397-8, 397-10, 397-11, 397-13, 397-14

Coma

404

Tables 404-1 to 404-4

Stroke

407, 408

Figure 407-1; Tables 407-2, 407-3, 407-5, 407-6, 408-5, 408-6

Movement disorders

409, 410

Tables 409-4, 410-1 to 410-8

Neuropathy

420

Figure 420-1; Tables 420-1 to 420-5, E-Table 420-1

Suspicious mole

203

Table 203-1

Nail diseases

442

Table 442-4

Anemia

158

Tables 158-2 to 158-6

Polycythemia

166

Figure 166-2; Table 166-4

Leukocytosis

167

Figure 167-4; Table 167-1

Lymphocytosis

167

Table 167-3

Monocytosis

167

Table 167-2

Eosinophilia

170

Figure 170-2; Table 170-1

Neutropenia   With fever

167 281

Figure 167-7; Tables 167-4 and 167-5 Figure 281-1

Thrombocytosis

166

Figure 166-6; Table 166-6

Thrombocytopenia

172

Figure 172-1; Tables 172-1, 172-3

Prolonged PT or PTT

171

Figure 171-3

Urinalysis

114, 120

Tables 114-2, 120-6

Abnormal liver enzymes

147

Figures 147-2 to 147-4

Elevated BUN/creatinine   Acute   Chronic

120 130

Figure 120-1; Tables 120-1 to 120-5 Table 130-1

Hyperglycemia

229

Tables 229-1, 229-2

Hypoglycemia

230

Tables 230-1, 230-2

Electrolyte abnormalities

116, 117

Figure 116-4; Tables 116-6, 116-7, 117-2, 117-3

Acid-base disturbances

118

Figures 118-1, 118-2; Tables 118-1 to 118-6

Hypercalcemia

245

Figure 245-3; Tables 245-2 to 245-4

Hypocalcemia

245

Figure 245-4; Table 245-6

Hypo- and hyperphosphatemia

119

Tables 119-2, 119-3

Magnesium deficiency

119

Table 119-1

Elevated Pco2

86

Figure 86-2

Solitary pulmonary nodule

191

Figure 191-2

Pleural effusion

99

Tables 99-4 to 99-6

ECG abnormalities

54

Tables 54-2 to 54-5

Musculoskeletal/Extremities

Neurologic

Skin and Nails

COMMON LABORATORY ABNORMALITIES Hematology/Urinalysis

Chemistries

Chest Radiograph/ECG

BUN = blood urea nitrogen; ECG = electrocardiogram; PT = prothrombin time; PTT = partial thromboplastin time.